Sei sulla pagina 1di 561

MANUAL DE TRABAJO DEL CURSO ENARM CMN SIGLO XXI

Pharmed Solutions Institute


Dr. Angel Mauricio Soriano Perez


MANUAL DE TRABAJO DEL CURSO ENARM CMN SIGLO XXI
CURSO ENARM CMN SIGLO XXI TEL: 36246001 Pharmed Solutions Institute PGINA 2



INTRODUCCION


Este manual de trabajo del curso para aprobar el ENARM es el resultado de ms de 13 aos de estudio de los contenidos del ENARM
desde el ao 2000 a la fecha, la modalidad, contenido y caractersticas pedaggicas han cambiado de forma importante desde entonces
hasta ahora, con la versin moderna del ENARM podemos mencionar que es la mejor.

Los cursos de preparacin para el ENARM que se imparte en diversas instituciones pblicas y privadas han tenido que adaptarse a estos
cambios, sin embargo ni todos los cursos ni los diferentes libros dedicados a la preparacin de los mdicos aspirantes ha tomado con
toda seriedad y responsabilidad modificando y adaptndose a los cambios cruciales para as ofrecer una verdadera alternativa al
mdico general que desea prepararse para aprobar el ENARM.

Nuestro curso, los medios que son empleados y la metodologa han sido ampliamente analizados para ser un curso integral que ofrezca
un medio apropiado para incrementar las probabilidades de ser un seleccionado nacional.

Agradecemos a todos los mdicos especialista que participaron en la realizacin del presente manual de trabajo as como a los
participantes de los diversos cursos.



Direccin General.
Dr. ngel Mauricio Soriano Prez


Ponente Titular Medicina Interna.
Dra. Luz Elena Castro Vargas .

Ponente Titular Pediatra.
Dra. Gabriela Arenas Ornelas.

Ponente Titular Gineco-Obstetricia.
Dra. Rosario Len Gayosso.

Ponente Titular Urgencias.
Dr. Juan Manuel Lara Hernndez.

Ponente Titular Cirugia.
Dr. ngel Santino Figueroa.

Ponente Titular Ingls.
Dr. Juan Carlos Daz Torre.














No hay hombre ms digno de estimacin que el mdico que, habiendo estudiado la naturaleza desde su juventud, conoce las
propiedades del cuerpo humano, las enfermedades que le ataca, los remedios que pueden beneficiarle y que ejerce su arte.

Voltaire





MANUAL DE TRABAJO DEL CURSO ENARM CMN SIGLO XXI
CURSO ENARM CMN SIGLO XXI TEL: 36246001 Pharmed Solutions Institute PGINA 3



















INSTRUCCIONES


Este manual de trabajo esta formada por las areas de medicina interna, urgencias, ginecologa y obstetricia, pediatra y ciruga asi como
una seccin de compresin de ingles.

El manual de trabajo presenta un resumen minimo por cada patologa previo a los casos clnico, las patologas fueron seleccionadas
mediante un anlisis de salida de los sustentantes del ENARM cada ao.

Comprende 450 casos clnicas distribuidas por especialidad proporcionalmente y semejante al ENARM, 405 en espaol y 45 casos
clnicos en ingles.

Cada caso clnico tiene de 1, 2 3 preguntas con 4 posibles respuestas, todas las preguntas se realizaron buscando la mejor respuesta
donde comprenden las siguientes areas de exploracin: diagnostico, tratamiento, patogenia, complicaciones, salud publica, ciencias
bsicas y rehabilitacin.

El orden de los temas se realizar como esta en el ndice, posterior a la sesin clnica, se resolvera los casos clnicos del tema, en
conjunto al profesor. Posteriormente resolver en el auditorio o en su domicilio, las 6 opciones ms de exploracin denominado
explicaciones y preguntas complementarias.

1.- Sesin clnica.
2.- Casos clnicos.
3.- Explicaciones y preguntas complementarias.
4.- Videosesiones para despejar dudas.
5.- Simuladores mensuales.
6.- Elaboracin de casos clnicos por los participantes.






















MANUAL DE TRABAJO DEL CURSO ENARM CMN SIGLO XXI
CURSO ENARM CMN SIGLO XXI TEL: 36246001 Pharmed Solutions Institute PGINA 4



MEDICINA INTERNA
1) HIPERTENSION ARTERIAL PRIMARIA Y SECUNDARIA
2) INSUFICIENCIA CARDIACA AGUDA Y CRONICA
3) CARDIOPATIAS CONGENITICAS Y VALVULOPATIAS
4) MIOCARDITIS, ENDOCARDITIS Y PERICARDITIS
5) ASMA, BRONQUITIS, NEUMONIAS, NEUMOPATIAS.
6) HIPERTENSION PULMONAR Y COR PULMONAR, EPOC
7) CARCER PULMONAR Y BRONCOPULMONAR.
8) TRASTORNOS HIPOTALAMICOS E HIPOFISIARIOS.
9) TRASTORNOS TIROIDEOS Y PARATIROIDEOS.
10) TRASTORNOS SUPRARRENALES Y GONADALES.
11) SOBREPESO, OBESIDAD Y DISLIPIDEMIAS.
12) DIABETES MELLITUS Y COMPLICACIONES.
13) PANCREATITIS AGUDA Y CRONICA.
14) ESTEATOSIS, HIGADO GRASO, CIRROCIS HEPATICA Y HEPATOCARCINOMA
15) LEUCEMIAS, LINFOMAS, LINFOMA NO HODGKIN, LINFOMA HODGKIN
16) FIEBRE REUMATICA, ARTRITIS REUMATOIDE, OSTEARTROSIS, OSTEOCONDRITIS Y GOTA.
17) LUPUS ERITEMATOSO SISTEMICO, VASCULITIS, ARTERITIS DE TAKAYASU, SX DE SJOGREN
18) TUBERCULOSIS, PALUDISMO Y DENGUE, ENFERMEDAD DE CHAGAS, LEPRA,
19) TRIPANOSOMIASIS, LEISHMANIASIS, ESQUISTOSOMIASIS, FILARIASIS, ONCOCERCOSIS.
20) RICKETTSIAS, LEGIONELLA, PSEUDOMONA AERUGINOSA, LISTERIA MONOCYTOGENES, CRIPTOCOCOCIS
21) LEPTOSPIROSIS, ACTINOMICOSIS, NORCARDIA. CMV Y VEB.
22) STAFILOCOCCUS AUREUS, EPIDERMIDIS, ESTREPTOCOCCUS PNEUMONIAE Y HAEMOPHILUS.
23) BORTEDELLA PERTUSIS, MORAXELLA CATARRHALIS, BRUCELLA, FRANCISELLA TULARENSIS, RABIA
24) VIH Y SIDA, ETS, NEISSERIA GONORROEAE, SIFILIS, HERPES, CHLAMYDIA.
25) CANCER E INFECCIONES DEL SISTEMA NERVIOSO CENTRAL.
26) CEFALEA, EPILEPSIAS Y ENCEFALOPATIAS
27) DEMENCIA VASCULAR, ALZHEIMER Y PICK, HUNTINGTON Y PARKINSON
28) ESCLEROSIS MULTIPLE, ESCLEROSIS LATERAL AMNIOTROFICA Y MIASTENIA GRAVIS.
29) DEPRESION, ANSIEDAD, ESQUIZOFRENIA Y TRASTORNOS DE LA ALIMENTACION.
URGENCIAS
1) MUERTE SUBITA, RCP.
2) ESTADO CHOQUE
3) URGENCIAS, EMERGENCIA HIPERTENSIVA. HIPERTENSION MALIGNA.
4) INSUFICIENCIA CARDIACA AGUDA Y CRONICA AGUDIZADA.
5) TAPONAMIENTO CARDIACO, DISECCION DE LA AORTA, ANEURISMA AORTICO.
6) ANGINA ESTABLE, ANGINA INESTABLE, ANGINA DE PRNIZTMETAL.
7) SICA I, SICA II
8) TRASTORNOS DEL RITMO (ARRITMIAS CARDIACAS)
9) FALLA ORGANICA MULTIPLE, COAGULACION INTRAVASCULAR DISEMINADA.
10) ANAFILAXIA Y ALERGIAS.
11) EPOC AGUDIZADO, TROMBOSIS VENOSA PROFUNDA, TROMBOEMBOLIA PULMONAR.
12) SINDROME DE INSUFICIENCIA RESPIRATORIA AGUDA, EDEMA AGUDO PULMONAR.
13) NEUMOTORAX, NEUMOMEDIASTINO, HEMOTORAX, CONTUSION CARDIACA.
14) TRAUMA TORACICO CERRADO Y ABIERTO.
15) TRAUMA ABDOMINAL ABIERTO Y CERRADO.
16) INSUFICIENCIA RENAL AGUDA, GLOMERULOPATIAS AGUDAS.
17) SINDROME NEFRITICO Y NEFROTICO.
18) TRASTORNOS ACIDO-BASE AGUDOS.
19) TRASTORNOS ELECTROLITICOS AGUDOS.
20) HIPOGLUCEMIA, HIPERGLUCEMIA, ESTADO HIPEROSMOLAR, CETOACIDOSIS DIABETICA.
21) TRAUMATISMO CRANEOENCEFLICO, ISQUEMIA CEREBRAL TRANSITORIA, ACCIDENTE VASCULAR CEREBRAL.
22) CRISIS CONVULSIVAS, ESTATUS EPILEPTICO, COMA Y MUERTE CEREBRAL.
23) TRASTORNO BIPOLAR, DELIRIUM, PSICOSIS, SUICIDIO.

GINECOLOGIA Y OBSTETRICIA
1) PLANIFICACION FAMILIAR, ANTICONCEPCION, INFERTILIDAD, ESTERILIDAD.
2) AMENORREA PRIMARIA Y SECUNDARIA, ENDOMETRIOSIS.
3) CERVICOVAGINITIS, ENFERMEDAD PELVICA INFLAMATORIA, ENFERMEDADES DE TRANSMISION SEXUAL.
4) INFECCION POR VIRUS DEL PAPILOMA HUMANO. DISPLASIAS, CANCER CERVICOUTERINO Y OVARICO.
5) SANGRADO UTERINO ANORMAL Y DISFUNCIONAL. SINDROME DE OVARIO POLIQUISTICO.
6) MASTOPATIA FIBROQUISTICA, CANCER DE MAMA, MENOPAUSIA Y CLIMATERIO
MANUAL DE TRABAJO DEL CURSO ENARM CMN SIGLO XXI
CURSO ENARM CMN SIGLO XXI TEL: 36246001 Pharmed Solutions Institute PGINA 5



OBSTETRICIA
1) CONTROL PRENATAL Y DEL EMBARAZO.
2) EMBARAZO ECTOPICO. AMENAZA DE ABORTO, ABORTO, INCOMPETENCIA CERVICAL.
3) HIPEREMESIS GRAVIDICA, ENFERMEDAD TROFOBLASTICA.
4) HIPERTENSION ARTERIAL Y ENFERMEDAD HIPERTENSIVA DEL EMBARAZO.
5) PREECLAMPSIA Y ECLAMPSIA.
6) DIABETES MELLITUS, INTOLERANCIA A LA GLUCOSA, DIABETES GESTACIONAL.
7) POLIHIDRAMNIOS, OLIGOHIDRAMNIOS, PLACENTA PREVIA, DESPRENDIMIENTO DE PLACENTA.
8) ANEMIAS, IVU, PARTO PRETERMINO, PARTO NORMAL Y CESAREA.
9) COMPLICACIONES DEL PUERPERIO, HEMORRAGIA OBSTETRICA, PLACENTA ACCRETA.
10) SINDROME DE TENSION PREMENSTRUAL, PSICOSIS POSPARTO Y DEPRESION POSTPARTO.

PEDIATRIA
1) SX DE ADAPTACION PULMONAR, SX DE DIFICULTAD RESPIRATORIA, TAQUIPNEA TRANSITORIA DEL RN
2) ASPIRACION DE MECONIO, ENFERMEDAD POR MEMBRANA HIALINA, NEUMONIA NEONATAL.
3) SEPSIS NEONATAL, ENTEROCOLITIS NECOTRIZANTE, MENINGITIS NEONATAL.
4) HIPOGLUCEMIA, ICTERICIA, KERNICTERUS.
5) ATRESIA ESOFAGICA, FISTULA TRAQUEOESOFAGICA, HERNIA HIATAL, ESTENOSIS PILORICA.
6) CARDIOPATIAS CONGENITAS Y ADQUIRIDAS
7) KWASHIORKOR, MARASMO, AVITAMINOSIS,
8) OBESIDAD, VACUNACION, CONTROL DEL NIO SANO.
9) DERMATITIS, HIPERQUERATOSIS HIPEDERMOLITICA.
10) IMPETIGO ERISIPELA, PIODERMA, TB CUTANEA, MAL DEL PINTO.
11) HERPES SIMPLE, ZOSTER, VERRUGAS VIRALES, MOLUSCO CONTAGIOSO
12) FOLICULITIS, PEDICULOSIS, ESCABIASIS.
13) TIAS, PITIRIASIS VERSICOLOR, CANDIDIASIS, MICETOMA.
14) URTICARIA, PRURIGOS, ERITEMA PALMAR, ERITEMA NODOSO.
15) DERMATITIS DE CONTACTO, DERMATITIS DEL PAAL.
16) PSORIASIS, DERMATITIS SEBORREICA, ACNE
17) PURPURA DE SCHNLEIN-HENOCH, ANEMIAS,
18) LEUCEMIAS, RETINOBLASTOMA, TUMORES DEL SNC.
19) PAROTIDITIS, HEPATITIS, INFLUENZA, MENINGITIS
20) OTITIS, SINUSITIS, FARINGITIS, AMIGDALITIS
21) EPIGLOTITIS, LARINGOTRAQUEITIS, BRONQUITIS.
22) BRONQUIOLITIS, BRONCONEUMONIA, NEUMONIAS
23) ALERGIAS, ASMA, FIBROSIS QUISTICA.
24) URETRITIS, CISTITIS, PIELONEFRITIS.
25) GLOMERULONEFRITIS, SINDROME NEFRITICO, SINDROME NEFROTICO.
26) GASTROENTERITIS, PARASITOSIS.
27) DESHIDRATACION, LIQUIDOS Y ELECTROLITOS
28) QUEMADURAS, ACCIDENTES
29) PICADURAS, MORDEDURAS
30) INTOXICACIONES, ENVENENAMIENTOS
31) TRAUMATISMO CRANEOENCEFALICO, CEFALEA Y EPILEPSIAS
32) ENURESIS, ENCOPRESIS, TDAH, TRASTORNOS DEL DESARROLLO.
CIRUGIA
1) ESOFAGITIS, ACALASIA, REFLUJO ESOFAGICO Y ERGE.
2) DISPLASIAS DE ESOFAGO, ESOFAGO DE BARRETT, CANCER DE ESOFAGO.
3) VARICES ESOFAGICAS, RUPTURA ESOFAGICA Y HERNIA HIATAL.
4) GASTRITIS, ULCERA GASTRICA Y ULCERA DUODENAL, CANCER GASTRICO.
5) COLANGITIS, COLECISTITIS, COLEDOCOLITIASIS
6) CANCER DE PANCREAS, COLANGIOCARCINOMA,
7) ESTEATOSIS Y CIRROCIS HEPATICA, CARCINOMA HEPATOCELULAR.
8) PANCREATITIS AGUDA Y CRONICA.
9) ISQUEMIA MESENTERICA, INFARTO MESENTERICO, DIVERTICULITIS.
10) APENDICITIS, POLIPOS, COLITIS, CROHN, CUCI.
11) HEMORROIDES, FISTULA ANAL, CANCER DE COLON
12) HERNIAS DIAFRAGMATICAS, HIATAL, DE PARED E INGUINAL,
13) ORQUIEPIDIDIMITIS, TORSION TESTICULAR, CANCER TESTICULAR.
14) HIPERTROFIA PROSTATICA BENIGNA, CANCER DE PROSTATA
15) TCE, TRAUMA FACIAL Y TRAUMA RAQUIMEDULAR.
16) TRAUMA TORACICO, TRAUMA ABDOMINAL Y PELVICO
17) PIE DIABETICO Y COMPLICACIONES QUIRURGICAS DE LA DM.
MANUAL DE TRABAJO DEL CURSO ENARM CMN SIGLO XXI
CURSO ENARM CMN SIGLO XXI TEL: 36246001 Pharmed Solutions Institute PGINA 6

18) RETINOPATIA DIABETICA E HIPERTENSIVA.
19) URGENCIAS OFTALMOLOGICAS, PERFORACION, GLAUCOMA. (CIRUGIA)
20) TRAUMA OTICO RUPTURA, PERFORACION, TRAUMA FACIAL.


MANUAL DE TRABAJO DEL CURSO ENARM CMN SIGLO XXI
CURSO ENARM CMN SIGLO XXI TEL: 36246001 Pharmed Solutions Institute PGINA 7

MEDICINA INTERNA
1) HIPERTENSION ARTERIAL PRIMARIA Y SECUNDARIA
2) INSUFICIENCIA CARDIACA AGUDA Y CRONICA
3) CARDIOPATIAS CONGENITICAS Y VALVULOPATIAS
4) MIOCARDITIS, ENDOCARDITIS Y PERICARDITIS
5) ASMA, BRONQUITIS, NEUMONIAS, NEUMOPATIAS.
6) HIPERTENSION PULMONAR Y COR PULMONAR, EPOC.
7) CARCER PULMONAR Y BRONCOPULMONAR.
8) TRASTORNOS HIPOTALAMICOS E HIPOFISIARIOS.
9) TRASTORNOS TIROIDEOS Y PARATIROIDEOS.
10) TRASTORNOS SUPRARRENALES Y GONADALES.
11) SOBREPESO, OBESIDAD Y DISLIPIDEMIAS.
12) DIABETES MELLITUS Y COMPLICACIONES.
13) PANCREATITIS AGUDA Y CRONICA.
14) ESTEATOSIS, HIGADO GRASO, CIRROCIS HEPATICA Y HEPATOCARCINOMA
15) LEUCEMIAS, LINFOMAS, LINFOMA NO HODGKIN, LINFOMA HODGKIN
16) FIEBRE REUMATICA, ARTRITIS REUMATOIDE, OSTEARTROSIS, OSTEOCONDRITIS Y GOTA.
17) LUPUS ERITEMATOSO SISTEMICO, VASCULITIS, ARTERITIS DE TAKAYASU, SX DE SJOGREN
18) TUBERCULOSIS, PALUDISMO Y DENGUE, ENFERMEDAD DE CHAGAS, LEPRA,
19) TRIPANOSOMIASIS, LEISHMANIASIS, ESQUISTOSOMIASIS, FILARIASIS, ONCOCERCOSIS.
20) RICKETTSIAS, LEGIONELLA, PSEUDOMONA AERUGINOSA, LISTERIA MONOCYTOGENES, CRIPTOCOCOCIS,
21) LEPTOSPIROSIS, ACTINOMICOCIS, NORCARDIA. CMV Y VEB.
22) STAFILOCOCCUS AUREUS, EPIDERMIDIS Y ESTREPTOCOCCUS PNEUMONIAE. HAEMOPHILUS.
23) BORTEDELLA PERTUSIS, MORAXELLA CATARRHALIS, BRUCELLA, FRANCISELLA TULARENSIS, RABIA,
24) VIH Y SIDA, ETS, NEISERIA GONORRAE, SIFILIS, HERPES, CHLAMYDIA.
25) CANCER E INFECCIONES DEL SISTEMA NERVIOSO CENTRAL.
26) CEFALEA, EPILEPSIAS Y ENCEFALOPATIAS
27) DEMENCIA VASCULAR, ALZHEIMER Y PICK, HUNTINGTON Y PARKINSON
28) ESCLEROSIS MULTIPLE, ESCLEROSIS LATERAL AMNIOTROFICA Y MIASTENIA GRAVIS.
29) DEPRESION, ANSIEDAD, ESQUIZOFRENIA Y TRASTORNOS DE LA ALIMENTACION.


MANUAL DE TRABAJO DEL CURSO ENARM CMN SIGLO XXI
CURSO ENARM CMN SIGLO XXI TEL: 36246001 Pharmed Solutions Institute PGINA 8

HIPERTENSIN ARTERIAL PRIMARIA Y SECUNDARIA (TEMA)
DEFINICION: Es el incremento sostenido de la tensin arterial diastlica > 90 mmHg, La OMS la define como un trastorno cuya
patogenia se desconoce en muchos casos, que en ltima instancia causa aumento de la presin diastlica y sistlica, as como
alteraciones del lecho vascular y alteraciones funcionales de los tejidos afectados. EPIDEMIOLOGIA: Prevalencia mundial: 1 billn de
individuos, 7.1 millones de muertes al ao, OMS: 62 % de la enfermedad cerebrovascular y 49 % de la enfermedad isqumica cardiaca.
Prevalencia incrementa conforme avanza la edad, 50% de personas entre 60 y 69 aos y 3/4 de los mayores de 70 aos estn
afectados. La prevalencia de hipertensin sistlica se incrementa con la edad. A partir de los 50 aos forma ms comn. PAD factor de
riesgo CV ms potente hasta los 50 aos. A partir de entonces, la PAS >140mmHg es de importancia mayor como factor de riesgo de
(ECV). En los ensayos clnicos, la terapia antihipertensiva se ha asociado con reducciones en incidencias de ictus de un 35-40 %, de
infarto de miocardio de un 20-25 %, y de insuficiencia cardiaca en ms de un 50 %. Por cada incremento de 20 mmHg en PAS 10 en
PAD, hay el doble de mortalidad por enfermedad isqumica e ictus. Estudio Framingham: PA de 130 a 139/85 a 89 mmHg: 2 veces de
incremento RR de enfermedad cardiovascular comparados con los niveles por debajo de 120/80 mmHg. El riesgo de ECV comienza a
partir de 115/75 mmHg se dobla con cada incremento de 20/10 mmHg. ETIOLOGIA: HTA primaria es ms frecuente, la secundaria
causada por: estenosis de arteria renal, insuficiencia renal, enfermedad de las glndulas suprarrenales, feocromocitomas, hiperplasia
cortical, sndrome de cushing, hiperaldosteronismo, otras causas son la preeclampsia, coartacin de la aorta, enfermeades cerabrales.
Factores de Riesgo Mayores, Hipertensin, Edad (>55 en varones, 65 en mujeres), Diabetes mellitus, LDL elevado, Colest total alto
bajo HDL, TFG < 60 ml/min, Historia familiar de ECV prematura (H <55 M < 65), Microalbuminuria, Obesidad (IMC > 30 Kg/m2)
Inactividad Fsica, Tabaco. FISIOPATOGENIA: Patologia multifactorial producto de la interaccin de factores prevenibles y no
prevenibles, adems de hiperactividad del sistema nervioso simpatico, alteracin de catecolaminas y quimio y barorreceptores.
Alteraciones del sistema renina-angiotensina-aldosterona, altaracion de resistencias vascular, del volumen intravascular, constricion
arteriolar, e hipopotasemia y defecto natriuretico por capacidad disminuida de reducir la carga de sodio e hdrica. Aumento de sodio y
calcio intracelular, alteracin del transporte Na/K. CUADRO CLINICO: Buscar intensionadamente, medida apropiada de PA, Verificacin
en el brazo contralateral, Examen del fondo de ojo, IMC, Auscultacin carotdea, abdominal y soplos femorales, Palpacin de la
glndula tiroidea, Examen cardiaco y pulmonar completo, Exploracin abdominal: riones, deteccin de masas y pulsaciones articas
anormales, palpacin de pulsos y edemas en extremidades inferiores, Valoracin neurolgica, buscar sntomas de rgano blanco,
cefalea, acufenos, fosfenos, apresion torcica, palpitaciones, fatiga, adinamia, cansancio, edema, alteraciones visuales, alteraciones
sensitivas. DIAGNOSTICO: Se recomienda realizar toma de TA a pacientes mayores de 35 aos con o sin sntomas sugestivos de HTA. La
clasificacin de la presin arterial segn la OMS: Normal menor de 140/90 mmHg. Limtrofe: 140-160 y 90-95 respectivamente, HTA
(leve) 140-180 y 90-105; HTA moderada y severa > 180 y > 105 mmHg, o la JNC7: optima <120/80, Normal <130/85, Normal alta
<139/89, HTA estadio 150/99, HTA estadio 2 <179/109, HTA estadio 3 >180/110. TRATAMIENTO: PAS y PAD < 140/90 mmHg. Descenso
en complicaciones cardiovasculares. Con DM o IR el objetivo es de PA es < 130/80 mmHg. IMC 25, NaCl < 6 grs, ejercicio, no alcohol,
tabaco, iniciar hidroclorotiazida genera natriuresis, depleciona el espacio intravascular, inhiben reabsorcin de Na+ y Cl- en el tbulo
distal. IECAs. Bloquean de forma competitiva la enzima que convierte angiotensina I en angiotensina II. Disminucin importante de
angiotensina II y aldosterona, con estmulo secundario de la sntesis de renina. Antagonistas de los receptores AT1: Ejercen un bloqueo
competitivo unindose al receptor AT1 y evitando las acciones derivadas del estmulo de ste. Simpaticoliticos: alfametildopa estimula
los receptores a2adrenrgicos en el SNC, disminuyendo el tono simptico perifrico. Alfabloqueantes: prazocin acta sobre receptores
a-1 postsinpticos. Metoprolol: Reduce gasto cardaco a travs de la frecuencia y del inotropismo cardaco, bloqueo de la liberacin de
renina hasta 60% a nivel renal, calcioantagonistas: verapamilo, diltiazem y nifedipino. PRONOSTICO: Las causas de muerte entre los
hipertensos se distribuye de la siguiente manera: 50% fallecern por enfermedad cardiovascular, principalmente por cardiopata
isqumica. 33% por enfermedad cerebrovascular. 10 a 15% por insuficiencia renal. 2 a 7% por otras causas. COMPLICACIONES: Dao
rgano diana: hipertrofia ventricular izquierda, angina, infarto de miocardio, revascularizacin coronaria, insuficiencia cardiaca, ictus
ataque isqumico transitorio, demencia, enfermedad renal crnica, enfermedad arterial perifrica, retinopata. REHABILITACION:
Disminuir la remodelacin vascular (engrosamiento de la intima, media y adventicia) y la repercusin visceral (remodelado miocrdico,
nefropata hipertensiva, retinopata hipertensiva).

CASO CLINICO
Se trata de femenino de 52 aos de edad la cual acude a consulta por presentar desde hace 6 meses cefalea global de intensidad leve a
moderada, que se incrementa con los esfuerzos y bajo estrs laboral principalmente, agrega que ha tenido episodios de mareo no
relacionado a el movimiento, asi como ruidos e intolerancia a la luz, las ultimas semanas ha notado que le aprietan los zapatos, y se
cansa con facilidad al subir las escaleras en su trabajo. Cuenta con antecedentes familiares de importancia padre finado por IAM sbito
a la edad de 40 aos, madre actualmente viva con diabetes mellitus tipo 2, dislipidemia e hipertensin arterial bajo tratamiento. APP
tabaquismo positivo desde la edad de 20 aos a razn de 5 cigarrillos diarios, alcoholismo social, vida sedentaria y obesidad grado I.
AGO desarrollo eclampsia hace 30 aos y tomo anticonceptivos por 15 aos, A la EF IMC 32, TA 145/100mmHg. FC 93 lpm, FR 23,
Paciente con palidez de tegumentos, manchas algodonosas y hemorragias en llama en fondo de ojo, discreta ingurgitacin yugular al
decbito dorsal, ruidos respiratorios disminuidos, ruidos cardiacos normales, edema maleolar ++, laboratorios Hb 12, Hto 49, glucosa
116, creatinina 1.6, acido urico 9, colesterol 256, triglicridos 380, EGO: proteinuria +, bacterias ++, Estudios de gabinete: tele de torax
con discreta hipertrofia ventricular izquierda, ECG con QRS amplios de V4, V5 y V6.

PREGUNTA
Cul de los siguientes criterios es el ms til para para establecer el diagnostico?

RESPUESTA
a.- Cifras tensionales altas en repetidas ocaciones.
b.- Sospecha de alteraciones de rgano blanco.
c.- Factores de riesgo presentes.
MANUAL DE TRABAJO DEL CURSO ENARM CMN SIGLO XXI
CURSO ENARM CMN SIGLO XXI TEL: 36246001 Pharmed Solutions Institute PGINA 9

d.- Alteraciones de estudios paraclinicos.

PREGUNTA
En que categoria se encuentra actualmente el paciente?

RESPUESTA
a.- Normal superior.
b.- Etapa 2.
c.- Etapa 1.
d.- Etapa 3.

PREGUNTA
Cul es la patologa diferencial ms probable que puede sospecharse con los datos clnicos y laboratorios actuales?

RESPUESTA
a.- Patologa renal.
b.- Uso de estrgenos.
c.- Hipertensin vascular renal.
d.- Hiperaldosteronismo primario.

PREGUNTA
En relacin al estado del paciente, cul de los auxiliares diagnosticos esta indicado para continuar con el manejo integral del
padecimiento?

RESPUESTA
a.- Realizar ecocardiograma.
b.- Realizar urografa excretora.
c.- Repetir BH, QS y EGO.
d.- Solicitar electrolitos sericos.

PREGUNTA
Considerando los signos y sntomas que presenta el paciente, que intervencin no farmacolgica proporciona un mayor impacto para
la reduccin de cifras tensionales?

RESPUESTA
a.- Dieta hiposodica.
b.- Dieta hipocalrica.
c.- Dieta hipoproteica.
d.- Dieta normokalemica.

PREGUNTA
Cul de los siguientes farmacos presenta mayor utilidad para el paciente?

RESPUESTA
a.- IECAs.
b.- Calcio antagonistas.
c.- Diureticos.
d.- ARA II.

PREGUNTA
Cul de los siguientes esquemas farmacolgicos presenta mayor utilidad para el paciente?

RESPUESTA
a.- Metoprolol, hidroclorotiazida y nifedipina.
b.- Captopril, hidroclorotiazida y metoprolol.
c.- Lisonapril, amlodipino nifedipino.
d.- Losartan, hidroclorotiazida.

PREGUNTA
Cules son las condiciones no es causa de envio a segundo nivel a los pacientes con hipertensin arterial de difcil control?

RESPUESTA
a.- Falta de recursos para continuar con el manejo integral del paciente.
b.- Datos sugestivos de dao a rgano blanco.
c.- Requerir ms de 3 farmacos.
d.- Presencia de crisis hipertensivas.
MANUAL DE TRABAJO DEL CURSO ENARM CMN SIGLO XXI
CURSO ENARM CMN SIGLO XXI TEL: 36246001 Pharmed Solutions Institute PGINA 10

PREGUNTA
Cul rgano blanco presenta mayor evidencia de dao en el paciente actual?

RESPUESTA
a.- Retina.
b.- Rion.
c.- Corazn.
d.- Arterias.

PREGUNTA
Cules son los cambios bioqumicos ms probables por los datos clnicos en el caso descrito?

RESPUESTA
a.- Aumento de angiotensina II.
b.- Aumento de endotelinas.
c.- Aumento de noradrenalina.
d.- Disminucin de tromboxano A2

PREGUNTA
Cules de los cambios funcionales no es dependiente del sistema cardiovascular?

RESPUESTA
a.- Hiperactividad simpatica.
b.- Aumento del volumen sanguineo.
c.- Disminucion FNA.
d.- Expancin del volumen extracelular.

PREGUNTA
Cul de los elementos, es menos adecuado para garantizar el adecuado control, vigilancia y seguimiento en este paciente?

RESPUESTA
a.- Toma de presin arterial al menos 2 veces al mes.
b.- Envio oportuno a segundo nivel al diagnostico inicial.
c.- Evaluacin oftalmolgica para manejo de retinopata.
d.- Solicitud de ecocardiograma.

PREGUNTA
Cul de los resultados de los ndices o clasificaciones siguientes es ms firme para la referencia del paciente antes descrito a segundo
nivel de atencin?

RESPUESTA
a.- Clasificacin de keith, Wagener y Barker.
b.- Indice de KDOQI.
c.- ndice de Sokolow-Lyon.
d.- ATP III.

PREGUNTA
Cules son las complicaciones mas frecuentes en el paciente con hipertensin arterial?

RESPUESTA
a.- Infarto agudo al miocardio.
b.- Accidente vascular cerebral.
c.- Insuficiencia cardiaca congestiva.
d.- Insuficiencia renal crnica.

PREGUNTA
Cul de los siguientes cambios histolgicos renales no se presentan en la hipertensin arterial?

RESPUESTA
a.- Inadecuado desarrollo del sistema capilar.
b.- Esclerosis glomerular.
c.- Necrosis fibrinoide glomerular.
d.- Fibrosis intersticial.

PREGUNTA
Cules son los cambios anatomicos menos frecuentes que se presentan de la retina en la hipertensin arterial?
MANUAL DE TRABAJO DEL CURSO ENARM CMN SIGLO XXI
CURSO ENARM CMN SIGLO XXI TEL: 36246001 Pharmed Solutions Institute PGINA 11

RESPUESTA
a.- Papiledema del nervio ptico.
b.- Atrofia.
c.- Hemorragias.
d.- Ceguera.

PREGUNTA
Cul es el efecto farmacodinamico ms til para la nefroproteccion en el tratamiento de la hipertensin arterial?

RESPUESTA
a.- Diurtico con ahorro de K.
b.- Inhibidores de la enzima de conversin de la AGT
c.- Antagonistas de receptores de AGT II.
d.- Bloqueadores de canales de calcio.

PREGUNTA
Cul es la estrategia ms til para la rehabilitacin de las alteraciones cardiovasculares?

RESPUESTA.
a.- Control clnico y de los factores de riesgo.
b.- Entrenamiento fsico programado.
c.- Tratamiento psicolgico.
d.- Estudio de familiares de primer grado.

PREGUNTA
Cul es la estrategia ms til para la rehabilitacin de las alteraciones nefrolgicas del caso actual?

RESPUESTA
a.- Mantener los niveles de tensin arterial estricta.
b.- Monitorizacin de KDOQI semestral.
c.- Reduccin de peso, lpidos y tabaco.
d.- Disminuir la actividad del sistema nervioso simpatico.

PREGUNTA
Cul es la estrategia ms til para rehabilitacion de las alteraciones oftamologicas del caso actual?

RESPUESTA
a.- Mantener los niveles de tensin arterial estricta.
b.- Monitorizacin de KDOQI semestral.
c.- Reduccin de peso, lpidos y tabaco.
d.- Disminuir la actividad del sistema nervioso simpatico.

PREGUNTA
Cul es la accin ms importante para prevenir los factores de riesgo de hipertensin arterial en el primer nivel de atencin?

RESPUESTA
a.- Mantener IMC menor a 25/Kg/m2.
b.- Dieta hiposodica, hipocalrica y normokalemica.
c.- Evitar tabaquismo, alcoholismo y otras toxicomanias.
d.- Estilo de vida no sedentario, ejercicio aerobico y manejo del estrs.

PREGUNTA
Cules son las acciones para limitar los factores de riesgo de hipertensin arterial en el primer nivel de atencin para este caso
especfico?

RESPUESTA
a.- Establecer la meta de tensin arterial 130/90 mmHg.
b.- Uso de IECA para nefroproteccin.
c.- Uso de hipoglucemiante, bezafibrato y pravastatina.
d.- Establecer meta de 10 % de peso por reducir?

CASO CLINICO
Se trata de masculino de 78 aos de edad el cual se acude a consulta por dolor articular en rodilla izquierda, durante la exploracin
usted identifica que su cifras tensionales son las siguientes, TA 80/170mmHg, repite la toma de presin y se mantiene semejante, el
paciente niega sintomatologa asociado a las cifra tensionales. El paciente cuenta con antecedente de un episodio de isquemia cerebral
transitoria hace 6 meses.
MANUAL DE TRABAJO DEL CURSO ENARM CMN SIGLO XXI
CURSO ENARM CMN SIGLO XXI TEL: 36246001 Pharmed Solutions Institute PGINA 12

PREGUNTA
Cual es conducta a seguir para el manejo de este caso?

RESPUESTA
a.- Reduccin de peso y sal.
b.- Hidralacina 25 mg cada 24 hrs.
c.- Captopril e hidralacina.
d.- Enalapril y lisonipril.

PREGUNTA
Cual es el objetivo sobre el ndice de masa corporal para el tratamiento del paciente?

RESPUESTA
a.- Mantener el ndice corporal entre 20 y 25 Kg / m2.
b.- Mantener la tensin arterial sistlica al menos en 140 mmHg.
c.- Mantener la tensin arterial diastlica menor a 90 mmHg?.
d.- Disminuir el riesgo de IAM o EVC.

PREGUNTA
Cual es el tratamiento de eleccin farmacolgico considerando el antecedente del paciente?

RESPUESTA
a.- Enalapril 10 a 20 mg.
b.- Losartan 50 mg
c.- Hidroclorotiacida 12.5 a 25mg.
d.- Amlodipino.

PREGUNTA
El paciente se encuentra estable actualmente con cifras tensionales de 120/80 mmHg, cual de las siguientes medidas de referencia
debe realizar para envio a tercer nivel?

RESPUESTA
a.- Realizar QS, BH, EGO.
b.- Laboratorios de rutina ms Rx de torax y electrocardiograma.
c.- Buscar primeramente dao a rgano blanco.
d.- No es necesario ya que fue un hallazgo reciente.

PREGUNTA
El paciente fue evaluado por cardilogo y volvi para continuo de control, dentro de las medidas enviadas por el mismo fue monitorizar
la tensin arterial supina, sentado y de pie. Cual es la razn de esta medida.

RESPUESTA
a.- Es una medida que garantiza las cifras.
b.- Buscar ortostatismo asintomtico.
c.- Verificar el efecto del tratamiento.
d.- No tiene un efecto demostrable.

CASO CLINICO
Acude a consulta paciente femenino de 51 aos de edad originaria de Campeche, acude a su control mensual, cuenta con antecedentes
de Hipertensin arterial y diabete mellitus desde hace 10 y 5 aos respectivamente, sus signos vitales fueron TA 155/95 mmHg, FC 58
lpm, FR 27 rpm, laboratorios glucosa en ayunas de 210, hemoglobina glucosilada 6, urea 17, creatinin 1,1 mg/dl, refiere haber
presentado mareo ocacional, cefalea global acompaado de sensacin de sueo, frio con mareo ocacional y disnea de leve a
moderados esfuerzos, se observa edema de miembros inferiores. Su tratamiento es captopril, metoprolol y glibenclamida.

PRENGUNTA
Se realiza ECG, considerando las manifestaciones que se han presentado, que conducta teraputica considera ms apropiada realizar
de forma mediata?

a.- Incrementar los farmacos hasta meta de <120/80 mmHg.
b.- Retirar metoprolol continuar con captopril y glibenclamida.
c.- Mantener glibenclamida, retirar metoprolol e iniciar losartan.
d.- Agregar hidroclorotiazida al manejo actual.

PREGUNTA
Cuales son objetivos mas importantes con el nuevo manejo debido a los antecedentes?

MANUAL DE TRABAJO DEL CURSO ENARM CMN SIGLO XXI
CURSO ENARM CMN SIGLO XXI TEL: 36246001 Pharmed Solutions Institute PGINA 13

RESPUESTA
a.- Disminuir el riesgo de IAM.
b.- Disminuir el riesgo a EVC.
c.- Disminuir el bloque b-adrenergico por riesgo a hipoglucemia.
d.- Disminuir el riesgo de progresin de nefropata.

PREGUNTA
Dos aos despus el paciente continua en tratamiento, actualmente ha surgido edema de miembro inferiores, fatiga de medianos
esfuerzos, adems de dos eventos de isquemia cerebral transitoria menor a 2 horas, cual es la conducta a seguir?

RESPUESTA
a.- Indica estudios para identificar alguna complicacin a rgano blanco.
b.- Modifica la dosis para mantener un mejor manejo.
c.- Indica mayores medidas generales de control de la tensin arterial y diabetes mellitus.
d.- Considera suficientes criterios para enviar a segundo nivel.

CASO CLINICO
Se trata de masculino de 22 aos de edad el cual es ingresado al servicio de urgencia debido a la presencia de cefalea intensa,
palpitacin, sensacin de muerte inminente, diaforesis, refiere que ya le haba ocurrido esto anteriormente pero no tan intenso, su
tensin arterial fue de 140/100 mmHg. FC 115, FR 32, se observa agitado con dolor precordial resto de exploracin sin datos por
agregar, laboratorios de rutina sin datos patolgicos.

PREGUNTA
Considerando la presencia del cuadro clnico cual es la conducta a seguir mas adecuada.

RESPUESTA
a.- Evaluar metanefrinas /24 hrs.
b.- Medicina nuclar con MIBG.
c.- Prueba de supresin con clonidina.
d.- Prueba de supresin con glucagon.

PREGUNTA
Posterior a la realizacin del estudio anterior, se observo una imagen isointensa y continuaron los sntomas, cual es la conducta aseguir.

RESPUESTA
a.- Realizar bsqueda en cuello, torax y abdomen.
b.- Identificar CAS/CARS
c.- Supresion con Fluhidrocortisona.
d.- Realizar urografa escretora.

INSUFICIENCIA CARDIACA AGUDA Y CRONICA (TEMA)
DEFINICION: La Insuficiencia Cardaca (IC) puede ser definida como un sndrome clnico complejo que proviene de anormalidades
cardacas estructurales y/o funcionales, adquiridas o hereditarias, que conllevan al empeoramiento de la capacidad de llenado y
eyeccin ventricular. EPIDEMIOLOGIA: Es la tercera causa ms frecuente de ingresos hospitalarios y la primera causa de ingresos por
enfermedad cardiovascular. 15 millones de pacientes con IC. Disfuncin ventricular asintomtica 4% Prevalencia de la IC: 2-3% y
aumenta drsticamente a los 75 aos de edad. 10-20% en el grupo de pacientes de 70-80, La IC-FEC es ms comn en pacientes de
edad avanzada, mujeres, hipertensos y diabticos. La IC es la causa del 5% de los ingresos hospitalarios urgentes, 10% de las camas
hospitalarias, 2% de gastos sanitarios nacionales. ETIOLOGIA: Dao o perdida del musculo cardiaco, isquemia miocrdica, hipertensin
arterial sistmica, miocardiopatas, valvulopatas y arritmias. En muchos de los casos, sin embargo, la etiologa permanece sin
conocerse. La enfermedad coronaria causa la IC en cerca del 70-80% de los pacientes. Enfermedad valvular es origen del 10% de los
casos. Miocardiopatas 10%. FISIOPATOGENIA: Los mecanismos neurohumorales pueden causar efectos indeseables como:
vasoconstriccin prolongada, aumento de poscarga, hipertrofia ventricular, dilatacin ventricular, excesiva retencin de Na y H2O,
Arritmias. PNA: respuesta a dilatacin auricular, hormona contrarreguladora, vasodilatacin, incremento en excrecin de Na y H2O,
niveles reflejan severidad de la IC. Activacion del SN simpatico: vasoconstriccin, incremento en poscarga, incremento en el consumo
miocrdico de o2, hipertrofia ventricular, activa SRAA: incremento en precarga (incremento en trabajo ventricular). CUADRO CLINICO:
Sndrome clnico en el que los pacientes presentan las siguientes caractersticas: Sntomas tpicos de IC (disnea en reposo o con el
ejercicio, fatiga, edema) + signos tpicos de IC (taquicardia, taquipnea, estertores pulmonares y edema perifrico, hepatomegalia) +
Evidencia de una anomalia estructural o funcional del corazn en reposo (cardiomegalia, 3er ruidos, soplos cardiacos, anomalias en ekg,
elevacin de PNA). Criterios de framingham: mayores (disnea paroxistica nocturna u ortopnea, ingurgitacion yugular, estertores,
cardiomegalia, edema agudo de pulmon, galope por tercer tono, PCV > 16mmHg, tiempo de circulacion >25sg, reflujo hepatoyugular.
Criterios menores, edema maleolar, tos nocturna, disnea de esfuerzo, hepatomegalia, derrame pleural, reduccion de la capacidad vital,
taquicardia >120 lpm. DIAGNOSTICO: La ACC/AHA, Estadio A: con alto riesgo de IC. Anomala estructural o funcional no identificada; sin
signos ni sntomas, Estadio B: enfermedad cardiaca estructural desarrollada claramente en relacion con la IC pero sin signos ni
sintomas. Estadio C: IC sintomatica asociada a enfermedad estructural subyacente. Estadio D: enfermedad cardiaca estructural
avanzada y sintomas acusados de IC en reposo a pesar de tratamiento medico maximo. La NYHA: Clase I sin limitaciones de la actividad
fisica, el ejercicio fisico normal no causa fatiga, palpitaciones o disnea, Clase II ligera limitacion de la actividad fisica sin sintomas en
MANUAL DE TRABAJO DEL CURSO ENARM CMN SIGLO XXI
CURSO ENARM CMN SIGLO XXI TEL: 36246001 Pharmed Solutions Institute PGINA 14

reposo; la actividad fisica normal causa fatiga, palpitaciones o disnea. Clase III Acusada limitacion de la actividad fisica, sin sintomas en
reposo, cualquier actividad fisica provoca la aparicion de los sintomas, Clase IV incapacidad de realizar actividad fisica, los sintomas de
la IC estan presentes en reposo y aumentan con cualquier actividad fisica. DIAGNOSTICO DIFERENCIAL: Estados que semejan la
sintomatologa de IC: anemia, tiortoxicosis, embarazo, septicemia, insuficiencia heptica, cortocircuitos A-V, enfermedad de paget,
beriberi. TRATAMIENTO: Autocontrol, adherencia al tratamiento: solo 20-60% cumple Tx farmacolgico y no farmacolgico,
reconocimiento de sntomas, control del peso, dieta: restriccin de Na, restriccin de lquidos. No alcohol: inotrpico negativo,
hipertensa, arritmias. IECA a todos los pacientes con IC sintomtica y una FEVI 40%. En pacientes hospitalizados, se iniciar el
tratamiento con IECA antes del alta. Se indicar el uso de un BB en todos los pacientes con IC sintomtica y una FEVI 40%. ARA II con
una FEVI 35% e IC grave y sintomtica. Sin hiperpotasemia disfuncin renal significativa. Hidralazina + isosorbide en sintomticos
con FEVI 40%, (Tx en intolerancia a IECA y ARA). Digoxina en IC y FA (controla la FC, en ritmo sinusal con IC sintomtica y FEVI 40.
Diurtico recomendado en IC y signos o sntomas clnicos de congestin. Alivio de los sntomas y signos de congestin venosa pulmonar
y sistmica. PRONOSTICO: Del nmero total de pacientes, el 50% fallece a los 4 aos y el 40% de los pacientes ingresados por IC fallece
o reingresa durante el primer ao. COMPLICACIONES: Insuficiencia cardiaca conjestiva, arritmias, muerte sbita, edema agudo
pulmonar, etc. REHABILITACION: Cambio de estilo de vida, evitar la evolucacion de la enfermedad, reducir riesgos, ejercicio
programado.

CASO CLINICO
Se trata de paciente femenino de 73 aos de edad la cual cuenta con 20 aos de hipertensin arterial y diabetes mellitus, con
tratamiento mdico pero mal apego farmacolgico y no farmacolgico, actualmente se encuentra con el siguiente esquema
hidroclorotiazida 100mg/dia, furosemida 20 mg/dia, enalapril 60 mg/dia, amlodipino 10 mg/da, acido acetilsalicilico 100 mg/dia,
glipizida 20 mg dividido en dos dosis, actualmente acude por mareo ocasional con cada de su altura en dos ocasiones, se ha
desorientado dos veces desconociendo a la personas. A la exploracin fsica se observa paciente orientada, adecuado estado de alerta,
con leve desihratacion con facies pletricas, se observa ingurgitacin yugular de 5 cm, campos pulmonares con estertores de
predominio en las bases, se escucha desdoblamiendo del segundo ruido cardiaco, leve hepatomegalia, miembros inferiores con edema
+++, los cuales se encuentran calientes. Signos vitales: TA 105/80mmHg, FC 109 lpm, FR 31rpm. Se realiza ECG que se muestra y tele de
torax. Se calculo una FE >45%,

PREGUNTA
Cules son las manifestaciones clnicas ms especificas para establecer el diagnostico de la enfermedad?

RESPUESTA
a.- Ortopnea, disnea y tos noctura.
b.- Edema, fatiga y congestion torcica.
c.- Electrocardiograma y radiografia de torax.
d.- Ingurgitacion yugular, hepatomegalia y fraccin de eyeccin.

PREGUNTA
Que de Framingham para el dignosticos de IC tiene este paciente?

RESPUESTA
a.- 4 mayores y 2 menores.
b.- 4 mayores y 3 menores.
c.- 3 mayores y 2 menores.
d.- 3 mayores y 3 menores.

PREGUNTA
Cul es el factor de riesgo que puede conducir a un estado descompensado en este caso?

RESPUESTA
a.- Incumplimiento del tratamiento.
b.- SAOS.
c.- Arritmia, anemia hemorragia.
d.- Antiarritmico de clasa Ia y Ic.

PREGUNTA
Cules son las manifestaciones clnicas mas tiles para realizar un diagnostico diferencial de la IC sistlica vs IC diastlica?

RESPUESTA
a.- No son distinguibles.
b.- Disnea y edema.
c.- Taquicardia y taquipnea.
d.- Congestin heptica.

PREGUNTA
Cules son los auxiliares diagnosticos mas tiles que no han sido realizados en el paciente para identificar las complicaciones de la
enfermedad?
MANUAL DE TRABAJO DEL CURSO ENARM CMN SIGLO XXI
CURSO ENARM CMN SIGLO XXI TEL: 36246001 Pharmed Solutions Institute PGINA 15

RESPUESTA
a.- Puebas de funcin heptica.
b.- Electrolitos sericos.
c.- Pruebas de estrs.
d.- Ecocardiograma.

PREGUNTA
Cul es la medida farmacolgica que intervendr predominantemente sobre la respuesta neurohumoral en la retencin de sodio y
liquidos?

RESPUESTA
a.- Sistema renina-angiotensina-aldosterona.
b.- Sistema arginina-vasopresina.
c.- Liberacin de pptido natriuretico auricular.
d.- Inhibe el sistema de transporte Na + Cl -.

PREGUNTA
Cual de los siguientes factores no es determinante del consumo de oxigeno miocardico?

RESPUESTA
a.- Estrs parietal.
b.- Contractilidad.
c.- Frecuencia cardiaca.
d.- Fraccin de eyeccin.

PREGUNTA
Antes de definir el tratamiento farmacolgico, cual de las causas reversibles menos probable se encuentra en el caso?

RESPUESTA
a.- Depresin miocrdica inducida por alcohol.
b.- Antiinflamatorio no esteroideo.
c.- Estado de poscarga elevado.
d.- Uso de bloqueador de canales de calcio.

PREGUNTA
Cual de las siguientes medidas no es la mas recomendable para el actual caso clnico?

RESPUESTA
a.- Dieta con 5 a 6 g. de sal al da.
b.- Dieta sin sodio absoluta.
d.- Programa de gradual de ejercicio.
c.- Dieta normokalemica.

PREGUNTA
Cosiderando el estado actual de velocidad de filtrado glomerular del paciente, cual de los siguientes diureticos proporcionan un
equilibrio adecuado de natriuresis y caliuresis?

RESPUESTA
a.- Espironolactona.
b.- Hidroclorotiazida.
c.- Furosemida.
d.- Clortalidona.

PREGUNTA
Considerando el esquema farmacolgico actual del paciente, que medida farmacolgica es la mas adecuada para promover un menor
ndice de morbilidad y mortalidad?

RESPUESTA
a.- Incrementar la dosis de enalapril.
b.- Aadir losartan.
c.- Iniciar esquema con digoxina.
d.- Iniciar sildenafil.

PREGUNTA
Cul de los cambios neurohumorales que se presentan en la insuficiencia cardiaca es menos probable en este caso?

MANUAL DE TRABAJO DEL CURSO ENARM CMN SIGLO XXI
CURSO ENARM CMN SIGLO XXI TEL: 36246001 Pharmed Solutions Institute PGINA 16

RESPUESTA
a.- Aumento de la contractibilidad miocrdica.
b.- Aumento de la frecuencia cardiaca.
c.- Aumento del tono venoso.
d.- Disminucin del volumen sanguneo central efectivo.

PREGUNTA
Cul de los cambios funcionales que se presentan en el paciente con insuficiencia cardiaca es menos probable en este caso?

RESPUESTA
a.- Volumen sistlico reducido.
b.- Poscarga excesiva.
c.- Depresin de la contractibilidad.
d.- Disminucin del volumen telediastolico.

CASO CLINICO
Se trata de paciente masculino de 63 aos de edad la cual acude a consulta de revisin, en primer nivel de atencion, cuenta con
antecedentes de dislipidemia, hipertensin arterial y diabetes mellitus, actualmente tratada con atorvastatina, hidroclorotiazida,
losartan y glibenclamida, a dosis minima toxica, hace 2 aos presento IAM, el cual se trato oportunamente, acude debido a la presencia
de edema de miembros inferiores, fatiga, disnea de medianos esfuerzos, ortopnea y tos, a la exploracin fsica se aprecia reflujo
hepatoyugular, soplo carotideo, estertores finos bibasales, FC 91 lpm, FR 26, TA 150/80 mmHg.

PREGUNTA
Se solicita estudios de laboratorio y gabinete, los cuales estn pendientes de resultado, considerando los criterios de framingham para
el diagnostico de la insuficiencia cardiaca, cuantos mayores y menores respectivamente presenta?

RESPUESTA
a.- 2 mayores y 3 menores.
b.- 2 mayores y 2 menores.
c.- 3 mayores y 3 menores.
d.- 3 mayores y 2 menores.

PREGUNTA
Cual es la clase funcional y el ICT que en este momento presenta el paciente?

RESPUESTA
a.- Clase funcional I e ICT 0.62.
b.- Clase funcional III e ICT 0.59.
c.- Clase funcional IV e ICT 0.68.
d.- Clase funcional II e ICT 0.70

CASO CLINICO
Masculino de 43 aos de edad el cual reingresa a urgencias debido a dificultad respiratoria, disnea de minimos esfuerzos, tos
incluyendo en la noche, adems de dolor toraccico cuando se presenta la disnea, este dolor disminuye muy lentamente con el reposo,
el paciente cuenta con el antecedente de IAM egresando 15 dias antes de este evento. Se observa reflujo hepatoyugular con
hepatomagalia, se ausculta tercer ruido, Se observo es siguiente ECG. Sus contantes vitales son: IMC 31, FC 119 lpm, FR 31, TA 140/70
mmHg,

PREGUNTA
Cuales son los criterios de framingham presenta el paciente en este momento?

RESPUESTA
a.- 3 menores y 4 mayores.
b.- 4 menores y 4 mayores.
c.- 3 menores y 3 mayores.
d.- 4 menores y 2 mayores.

PREGUNTA
Cual es condicin que se presenta tomando la clasificacin de killip y Kimball.

RESPUESTA
a.- Clase funcional I.
b.- Clase funcional II.
c.- Clase funcional III.
d.- Clase funcional IV.

MANUAL DE TRABAJO DEL CURSO ENARM CMN SIGLO XXI
CURSO ENARM CMN SIGLO XXI TEL: 36246001 Pharmed Solutions Institute PGINA 17

PREGUNTA
Considerando el estado actual del paciente cual es el pronostico?
a.- Tasa de mortalidad del 6%.
b.- Tasa de mortalidad del 17%.
c.- Tasa de mortalidad del 38%.
d.- Tasa de mortalidad del 81%.

CASO CLINICO
Ingresa paciente de 39 aos de edad el cual ingresa a urgencias 4 horas antes, por presencia sbita de dolor retrosternal acompaado
de presin toracicca sensacin de mucho peso sobre su pecho que se presento en la madrugada, nausea, dificultad para respirar,
refiere que la noche anterior comio abundante y consumio alcohol hasta la embriaguez, al ingreso se observa confuso, diafortico con
palidez y cianosis distal, ingurgitacin yugular, se asculta estertores cripitantes bibasales, tercer ruido, TA 90/60 mmHg, FC 132 lpm, FR
36 rpm, Temp. 36.3 grados. Gasto urinario de < 20 ml/h, se observaron los siguientes estudios.

PREGUNTA
Cual es la clase funcional de Kllip que presenta el caso?

RESPUESTA
a.- Clase funcional Killip I.
b.- Clase funcional Killip II.
c.- Clase funcional Killip III.
d.- Clase funcional Killip IV.

PREGUNTA
Cual es el pronostico para la vida en este caso considerando la clase funcional?

a.- Tasa de mortalidad del 6%.
b.- Tasa de mortalidad del 17%.
c.- Tasa de mortalidad del 38%.
d.- Tasa de mortalidad del 81%

PREGUNTA
Se calculo los siguientes resultados (LC) de 2 L/m2/min. Considerando los elementos clnicos y el (LC), en la curva de starling cual es el
diagnostico?

RESPUESTA
a.- Insuficiencia cardiaca.
b.- Edema agudo pulmonar.
c.- Choque cardiogenico.
d.- Choque distributivo.

PREGUNTA
Se realiz ecocardiograma transesofagico de urgencias, donde se reporta FE < 30%, cual es % de mortalidad a 6 meses del paciente?

RESPUESTA
a.- del 15 al 20 %
b.- del 20 al 25 %
c.- del 25 al 30 %
d.- 30 al 35 %

CASO CLINICO
Se trata de masculino de 62 aos de edad diabtico e hipertensin controlado con glibenclamida y enalapril a dosis minimas toxicas
respectivamente, fue fumador hasta los 45 aos. Hace 5 aos presento infarto al miocardio tratado con trombolitico con buena
respuesta, acude a consulta debido cansacio, fatiga y dolor toracicco opresivo que sede con el reposo, ocasionalmente el dolor
toraccico es ardoroso o urente, adems de palpitaciones ocacionales, disnea de medianos esfuerzos, ortopnea as como fatiga, a la EF
presenta ingurgitacin yugular, frote carotideo, area cardiopulmonar con estertores crepitantes finos, palidez de tegumento, cianosis
distal, miembros con edema de miembros inferiores, TA 140/70 mmHg, FC 93 lpm, FR 26 rpm, IMC 29.5.

PREGUNTA
Se realizarn los siguientes estudios de rutina hemoglobina glucosilada 8, creatinina 0.5, acido urico 7.0 mg/dl, colesterol HDL 27 mg/dl,
triglicridos 210 mg/dl, considerando el estado actual de caso clnico, cual es la conducta a seguir?

RESPUESTA
a.- Solicitar BNP NT-proBNF.
b.- Solicitar ecocardiograma transesofagico.
c.- Solicitar ECG en reposo y holter.
MANUAL DE TRABAJO DEL CURSO ENARM CMN SIGLO XXI
CURSO ENARM CMN SIGLO XXI TEL: 36246001 Pharmed Solutions Institute PGINA 18

d.- Solicitar prueba de estrs.

PREGUNTA
Cual es la conducta teraputica a seguir para disminuir el riesgo cardiometabolico?

RESPUESTA
a.- Iniciar hidroclorotiazida
b.- Iniciar verapamilo.
c.- Iniciar betabloqueador.
d.- Iniciar atorvastatina

PREGUNTA
Se realizo ecocardiograma que reporta una FE > 45 %, el electrocardiograma reporta extrasstoles ventriculares, cual es la conducta
farmacolgica mas adecuada para disminuir el remodelado ventricular y mejorar la actividad elctrica cardiaca?

RESPUESTA
a.- Iniciar betabloqueador.
b.- Iniciar verapamilo.
c.- Iniciar losartan.
d.- Iniciar diltiazem.

PREGUNTA
El paciente se mantiene en buenas condiciones sin embargo 12 meses despus acude a cita de control, se realizo electrocardiograma de
control, donde se apresia bloqueo de primer grado, el paciente agrega que la monitorizacin ambulatoria de la presin fue adecuada,
sin embargo la opresin toracicca se ha presentado de forma expontanea y con mayor duracin, asi como la disnea se ha presentado
mas frecuente, cual es el frmaco mas adecuado?

RESPUESTA
a.- Isosorbide 10 mg.
b.- Verapamilo 80 mg.
c.- Diltiazem 60 mg.
d.- Nitroglicerina 18 mg.

CASO CLINICO
Se trata de paciente femenino de 67 aos de edad, que acude a consulta por cansancio, fatiga, dificultad para respirar, dolor toraccico
que disminuye con reposo, disnea de moderados esfuerzos, edema de miembros inferiores hasta 2/3 de la pierna, a la exploracin fsica
se observo palidez de piel y mucosas, llenado capilar lento, estertores crepitantes leves, presencia de galope y trill. La paciente es
hipertensa desde hace 20 aos con moderado apego a tratamiento con captopril 50 mg/dia, hidroclorotiazida 25 mg/12 hrs. Ha
presentado 2 crisis hipertensivas, asi como un evento de insuficiencia mesentrica, sus laboratorios EGO protenuria, glucosuria, uratos,
densidad urinaria disminuida, colesterol 289 mg/dl, triglicridos 720 mg/dl, creatinina 6.2 mg/dl, BUM 29, kalemia de 5.3, los signos
vitales fueron TA 160/105 mmHg, FC 96, FR 28, IMC 32.

PREGUNTA
Se realizo un ecocardiograma donde se observa una FE de < 40 %, durante su ingreso la paciente presento volumen urinario de 20
ml/hr. Considerando las caractersticas actuales cual es la clase funcional que presenta el caso?

RESPUESTA
a.- Clase funcional I.
b.- Clase funcional II.
c.- Clase funcional III.
d.- Clase funcional IV.

PREGUNTA
Considerando las caractersticas actuales cual es el K/DOQI que presenta el paciente, cual es el mtodo mas adecuado para determinar
el estadio de la IRC?

RESPUESTA
a.- Depuracion de creatinina de 24 hrs.
b.- Biopsia renal.
c.- Ultrasonografia renal.
d.- Urografia escretora.

PREGUNTA
Cual es la conducta farmacolgica mas adecuada considerando el K/DOQI y la clase funcional?

RESPUESTA
MANUAL DE TRABAJO DEL CURSO ENARM CMN SIGLO XXI
CURSO ENARM CMN SIGLO XXI TEL: 36246001 Pharmed Solutions Institute PGINA 19

a.- Verapamilo.
b.- Diltiazem.
c.- Telmisartan.
d.- Isosorbide.

PREGUNTA
Cual es la conducta inmediata a seguir para mejorar la precarga disminuyendo los niveles de potasio?

RESPUESTA
a.- Furosemide.
b.- Dialisis peritoneal.
c.- Hemodialisis.
d.- Soluciones metabolizadas.

PREGUNTA
Cual de las siguientes patologias es la principal causa de IRC?

RESPUESTA
a.- Diabetes mellitus.
b.- Hipertensin arterial.
c.- Glomerulonefritis.
d.- Enfermedad renal poliqustica.

PREGUNTA
Cuales factores de riesgo aceleran el deterioro del funcionamiento renal en el caso actual?

RESPUESTA
a.- Dislipidemia.
b.- Edad avanzada.
c.- Diabetes mellitus.
d.- Anemia.

CASO CLINICO
Se trata de masculino de 59 aos de edad, ingresa a urgencias debido a dificultad para respirar, cansancio, fatiga, tos productiva desde
hace 15 dias intensificandoce la noche previa al ingreso actual, cuenta con antecedentes de EPOC diagnosticado hace 10 aos, posterior
a tabaquismo (una cajetilla diaria), con tratamiento de bromuro de iprapropio, salbutamol y betametasona, adems fue diagnosticado
como hipertenso hace 2 aos, inicio con cambios del estilo de vida y dieta con restriccin de sodio, hace un ao se considero incluir
captopril que fue cambiado 3 meses despus por incremento de tos, desde entoces toma losartan, hidralazina. A la EF se observa facies
pletrica con hiperemia conjuntival, mucosas orales cianticas moderadamente, se ausculta un soplo carotideo derecho, ingurgitacin
yugular grado II, estertores subcrepitantes bibasales, con resonancia tipanica a la percusin, los ruidos cardiacos levemente
disminuidos, rtmico, abdomen con hepatomegalia 5 cm debajo del borde costal.

PREGUNTA
Considerando el estado actual del paciente cual es la conducta mas apropiada a seguir, para establecer la funcin cardiopulmonar?

RESPUESTA
a.- Espirometria.
b.- Ecocardiograma.
c.- Valor de natriuretico.
d.- Radiografia de torax.

PREGUNTA
Luego de realizar las pruebas anteriores se observo una FE menor a 40 %, la radiografia de torax mostro un indice cardiotoraccico
patolgico, exudados algodonosos bilaterales, trama brocovascular incrementada, gases con PCO 49, PO2 85, potasio 6.3, Factor
natriuretico disminuido, a la expxloracion fsica se auscultan estertores crepitantes, sibilancias moderadas, a la percusin
hiperresonancia, se alcanza apresiar galope, cual es la conducta mas apropiada a seguir para disminuir la precarga, considerando todos
los factores?

RESPUESTA
a.- Retirar diurtico ahorrador de potasio por diurtico de asa.
b.- Iniciar con calcioantagonista dihidropiridinico de accin rpida.
c.- Incrementar la capacitancia vascular perifrica con nitritos.
d.- Restriccion hdrica, dieta libre de sodio, diurtico de asa.



MANUAL DE TRABAJO DEL CURSO ENARM CMN SIGLO XXI
CURSO ENARM CMN SIGLO XXI TEL: 36246001 Pharmed Solutions Institute PGINA 20

CASO CLINICO
Paciente varn, de 41 aos de edad, diagnosticado previamente de SAF y con antecedentes personales de infarto lacunar
protuberancial izquierdo haca 4 aos, hipertensin arterial e hipercolesterolemia. Acudi a urgencias por dolor retrosternal al esfuerzo
rpidamente progresivo, con algn episodio en reposo, durante las dos semanas previas. El ECG demostraba amputacin de los
vectores iniciales en las precordiales derechas y supradesnivel del segmento ST, de hasta 2 mm, en las derivaciones V1 a V4. En la
auscultacin cardaca se detectaba un soplo holosistlico rudo de intensidad II-III/VI en el foco artico y diastlico II/IV en el borde
esternal izquierdo bajo. CPK mxima de 71 U/l, troponina T de 0,17 ng/ml, 58.000 plaquetas, TTPA 44 s y en el estudio inmunolgico, un
anticoagulante lpico positivo dbil y unos anticuerpos anticardiolipina (AAC) IgG moderadamente elevados (68 unidades fosfolipdicas
GPL). En la radiografa de trax se observaba cardiomegalia global con campos pulmonares limpios. El ecocardiograma transtorcico
objetiv una vlvula artica engrosada, con apertura estentica y gradiente sistlico mximo instantneo estimado de 59 mmHg y
regurgitacin.

PREGUNTA
Cual es la conducta a seguir mas adecuada para identificar el compromiso fisiolgico coronario?

RESPUESTA
a.- Ecocardiograma de estrs con dobutamina.
b.- Prueba de esfuerzo convencional
c.- Coronariografia percutenea.
d.- IRM-f con FDG cardiaca.

PREGUNTA
El paciente presento durante las siguientes semanas se present ortopnea, disnea de medianos esfuerzo, disminucin de la uresis, cabe
destacar de hipokalemia, la gasometra reporto pH 7.21, PCO 41, Na 121, Cl 115, HCO3 42, se diagnostica acidosis respiratoria
secundaria, considerando la patologia de base, cual es la causa menos probable,

RESPUESTA
a.- Depresion respiratoria central secundaria.
b.- Sindrome de Pickwickian.
c.- Edema pulmonar crnico.
d.- Compensacin respiratoria por alcalosis metabolica.

PREGUNTA
24 horas despus de estancia intrahospitalaria, el paciente presenta esputo salmonelado, disminucin del gasto urinario < 25 ml/hora,
confusin con agitacin psicomotriz, diafortico, cianosis central, ingurgitacin yugular III, estertores crepitantes, TA 70/50 mmHg, FC
139 lpm, FR 50 rpm. Cul es la conducta a seguir para mantener la fraccin de eyeccin adecuada sin incrementar la precarga?

RESPUESTA
a.- Dobutamina 20 g/kg/min.
b.- Dopamina > 20 g/kg/min.
c.- Epinefrina > 10 g/min.
d.- Digoxina 10 ug/Kg/min.

PREGUNTA
Considerando la clasificacin funcional clsica de la insuficiencia cardaca (NYHA), cul es la clase del paciente previa a la desicin
anterior?

RESPUESTA
a.- Grado I
b.- Grado IIA
c.- Grado IIIA
d.- Grado IVA

PREGUNTA
Considerando los lineamientos del American College of Cardiology y el American Heart Association, en que estadio se encuentra?

RESPUESTA
a.- Estadio A
b.- Estadio B
c.- Estadio C
d.- Estadio D

CASO CLINICO
Se ingresa a urgencias a paciente femenino de 48 aos de edad, originaria de Veracruz, empleada de agrcola, casada desde los 17 aos,
gesta 5, para 3, aborto 1, cesarea 1, histerectoma a los 38 por miomatosis uterina, no fuma ni toma bebidas alcoholicas,
aparentemente sana, no toma medicamentos, anteriormente presento caries dentales pero fueron tratadas hace 7 meses sin embargo
MANUAL DE TRABAJO DEL CURSO ENARM CMN SIGLO XXI
CURSO ENARM CMN SIGLO XXI TEL: 36246001 Pharmed Solutions Institute PGINA 21

hasta hace 3 meses que comenz padecimiento actual caracterizado por fatiga, disnea de medianos esfuerzos, sensacin de
ahogamiento de predominio nocturno, es ingresada debido a que presenta tos con esputo fino color rosa, disnea de leves esfuerzo, a la
exploracin fsica se observa paciente con estado grave, edematizada con cianosis central y perifrica, ingurgitacin yugular, soplo
mediodiastolico III/IV el cual predomina en posicin erecta, edema de miembros plvicos con lesiones isqumicas en dedos 1 y 2 de pie
derecho, tratado y contaminado con remedios locales, no se apercibe olor ni pus, los signos vitales son los siguientes TA 125/85 mmHg,
FC 102, FR 26, Temperatura 37.5 C. La paciente recibe diagnostico de insuficiencia cardiaca congestiva.

PREGUNTA
Cul es la causa ms probable para la insuficiencia cardiaca congestiva de este caso.

RESPUESTA
a.- Endocarditis bacteriana con insuficiencia valvular.
b.- Estenosis de la valvula mitral.
c.- Mixioma auricular.
d.- Melanoma metastasico.

CASO CLINICO
Se estabiliza a la paciente y se realizan estudios de gabinete donde se identifica una masa tumoral intracavitaria de 6 centmetros,
dentro de los estudios se realizaron TAC, IRM y cateterismo, 4 horas despus la paciente presenta dificultad respiratoria intensa,
presencia de tos abundante con esputo asalmonelado, por lo que se intuba para soporte ventilatorio, se observa cianosis perifrica con
prdida de pulsos distales de miembros plvicos, ausencia de ruidos abdominales seguido de parada cardiaca en dos ocasiones,
declarando su muerte 2 horas ms tarde.

PREGUNTA
Cul es la fue la intervencin ms probable que culmino con la embolia masiva en esta paciente.

RESPUESTA
a.- No se anticoagulo profilcticamente.
b.- El cateterismo cardiaco innecesario.
c.- El desprendimiento del mixioma.
d.- El paciente ya tena signos de embolia.

CASO CLINICO
Hombre de 20 aos de edad sin antecedentes mrbidos conocidos. Present cuadro de 3 das de evolucin caracterizado por
odinofagia, tos seca, dolor precordial y dorsal asociado a disnea de reposo. Consult en centro de atencin donde se le solicita
radiografa de trax, destacando gran cardiomegalia y signos de congestin pulmonar. Se contaba con examen fsico y radiografa de
trax de 3 aos atrs sin ninguna alteracin.

PREGUNTA
Fue derivado a centro de atencin terciaria al cuarto da de evolucin del cuadro. Hemodinamia siempre estable. Se realizo
electrocardiograma (ECG). Cual es la conducta farmacolgica inicial mas apropiada para el caso?

RESPUESTA
a.- Amiodarona de 100 a 400 mg cada 24 horas.
b.- Diltiazem de 90 mg a 120 mg cada 12 horas.
c.- Verapamilo de 240 a 360 mg cada 24 horas.
d.- Esmolol de 50 a 100 g/kg/min.

PREGUNTA
Se realizo un electrocardiograma previo a la decisin farmacolgica, considerando el ECG el cual se observa anormal, cual de las
siguientes alteraciones del ritmo es ms probable encontrar considerando la evolucion?

RESPUESTA
a.- Taquicardia ventricular intermitente.
b.- Taquicardia auricular intermitente rapida
c.- Taquicardia sinusal.
d.- Taquicardia auricular.

PREGUNTA
Ingres hipotenso (84/56 mmHg), taquicrdico (116 latidos por minuto, irregular), afebril, frecuencia respiratoria de 18 por minuto y
saturando 92% con oxgeno ambiental, bien perfundido. La faringe se encontraba algo enrojecida, yugulares ingurgitadas. Sin
congestin pulmonar y examen cardaco demostr tercer ruido y ritmo de galope. Extremidades sin edema. De los exmenes de
laboratorio destacaba discreta leucocitosis, PCR baja, enzimas cardacas y troponinas normales, cual es la conducta mas adecuada a
seguir?


MANUAL DE TRABAJO DEL CURSO ENARM CMN SIGLO XXI
CURSO ENARM CMN SIGLO XXI TEL: 36246001 Pharmed Solutions Institute PGINA 22

RESPUESTA
a.- Prueba de esfuerzo con dobutamina.
b.- Ecocardiograma.
c.- Biopsia endocardica.
d.- Holter ambulatorio.

PREGUNTA
Entre otros estudios se realiz ecocardiograma, considerando el cuadro clnico actual cual de las siguientes aseveraciones es menos
probable esperar?

RESPUESTA
a.- Ventrculo izquierdo (VI) severamente dilatado.
b.- Disfuncin sistlica global severa. FE > 40 %.
c.- Hipokinesia difusa, disfuncin diastlica tipo III de IV.
d.- Disfuncin sistlica ventricular derecha e (HTP).

CASO CLINICO
Una paciente de 41 aos se encontraba en espera para ablacin de va accesoria (VA) en el contexto de un sndrome de Wolff-
Parkinson-White (WPW). La paciente, estando todava asintomtica, haba sido diagnosticada de WPW 9 aos antes en una revisin
rutinaria. Tres aos antes del actual ingreso empez a referir episodios cortos (< 5 min) de palpitaciones de inicio y fin sbitos, que
nunca fueron registradas por electrocardiograma (ECG). En los ltimos 9 meses empez a sufrir clnica compatible con insuficiencia
cardiaca en forma de disnea de esfuerzo clase funcional II de la New York Heart Association. La exploracin fsica era normal, pero en la
ergometra solo alcanz 6 MET.

PREGUNTA
Cuales son los datos que se registraron en la imagen 1A antes de la ablacin?

RESPUESTA
a.- Ritmo sinusal con BRD.
b.- Ritmo sinusal con preexitacion con BRI.
c.- Ritmo sinusal con preexcitacin por una VA anteroseptal derecha con (BRI).
d.- Ritmo sinusal con preexitacion por una VA anterioseptal derecha con BRD.

PREGUNTA
Considerando la evolucin clnica y el ECG 1B, el paciente paso de una FE del 48 % al 63 %, cual es la tasa de mortalidad en este
momento para el pacientes?

RESPUESTA
a.- Tasa de mortalidad del 6%.
b.- Tasa de mortalidad del 17%.
c.- Tasa de mortalidad del 38%.
d.- Tasa de mortalidad del 81%.

CARDIOPATIAS CONGENITICAS (ADULTO)
DEFINICION: La incidencia de las formas moderadas a severas de las cardiopatas congnitas es de 6 por 1000 nacidos vivos, los
pacientes sin manejo farmacolgico y/o quirrgico temprano no llegan a la vida adulta, ya que con el tratamiento adecuado el
pronostico es de 85% para llegar a la vida adulta, la mayor parte de los adultos con cardiopatas congnitas no requieren tratamiento
llegando a la vida adulta. Los defectos mas frecuentes son 1.- defecto septal ventricular estrecho, 2.- defecto artrial septal secundario,
3.- estenosis pulmonar leve a moderada, 4.- valva aortica bicuspidea, 5.- prolapso de valvula mitral. DIAGNOSTICO: Los adultos con
cardiopatas congnitas presentan tres formas bsicas: con una historia de ciruga previa paliativa o reparativa durante la infancia, 2.-
con conocimiento de la cardiopata congnita sin intervencin, 3.- presentacin en la vida adulta sin antecedentes de la infancia del
padecimiento. La excepcin se presenta en los pacientes con persistencia de conducto arterioso leve. Cianosis en el adulto debe buscar
la causa: Eritrocitosis, deficiencia de hierro, sangrado, o cardiovascular. Cuando se presentan las siguientes alteraciones: Alteracion de
la funcin ventricular (es la mas frecuente que presente sntomas), arritmias (las atriales es la mas frecuente, en las ventriculares son
secundarias a fibrosis, dilatacin ventricular, y son causas de muerte sbita). Enfermedad por conduccin, (intriseco o posoperatorio de
enfermedad de noso SA con disfuncin) se puede observar fraccin baja de eyeccin, disincronia, puede causar bloqueos. DEFECTO
ATRIAL SEPTAL: 75 % es defecto secundario a ostium encontrado en el adulto, se observa murmullo y alteraciones inespecficas de RX y
ECG, presencia de segundo ruido, sobrecarga ventricular derecha crnica, hipertensin pulmonar, arritmias atriales, raramente
embolizacion paradoxica). Tratamiento: Especifico por alteracin. PREVENCION: Endocarditis (es mas frecuente cuando hay
antecedentes de cardiopata congnita), Embarazo (cuando hay antecedentes de lesiones estenoticas severas pulmonar, hipertensin
pulmonar, shunts derecha a izquierda, regurgitacin, vigilancia estrecha por riesgo de endocarditis durante el embarazo y sobrecarga a
las 20 a 22 SDG indicado ecocardiografia), Anticoncepcion (Los Estrogenos incrementan mas los riesgos para tromboembolico, usar
contracepcin con progestgenos); Ejercicio (identificar taquicardia inducida, evaluar la competencia cronotropica, se contraindica el
ejercicio intenso isomtrico). DEFECTO SEPTAL VENTRAL: Existen 4 tipos, la perimembranosa es la mas comn, la muscular es comn
en la infancia con cierre expontaneo, puede presentar murmullo holosistolico inversamente relacionado al tamao del defecto, pueden
desarrollar el complejo Eisenmenger al no tratarse. TRATAMIENTO: al desarrollar hipertensin pulmonar puede estar indicado
MANUAL DE TRABAJO DEL CURSO ENARM CMN SIGLO XXI
CURSO ENARM CMN SIGLO XXI TEL: 36246001 Pharmed Solutions Institute PGINA 23

sindanafil o anlogos de prostanglandinas. DEFECTO SEPTAL ATRIOVENTRICULAR: Se presenta lesiones en los septos atriales y
ventriculares clasificados en parcial o complejo, se acompaa comnmente de subestenosis aortica, puede coexistir regurgitacin
mitral. El ECG puede mostrar desviacin a la izquierda del eje, y bloqueo incompleto, lesin frecuente en sndrome de down, requiere
reparacin antes de los 6 meses de edad para prevenir la hipertensin pulmonar y consecuentemente el complejo Eisenmenger.
TETRALOGIA DE FALLOT: Es la mas comn de las cardiopatas congnitas cianogenas representa del 7 al 10 %, difcilmente paciente sin
antecedentes solo en forma minimas, pueden presentar alteraciones de las intervenciones realizadas durante la vida adulta. Las
arritmias son las manifestaciones mas frecuentes en la vida adulta, junto a la disfuncin ventricular derecha, Tratamiento con
marcapaso y defibrilador interno. ESTENOSIS PULMONAR AISLADA: Presentan estenosis infundivular subpulmonar pudiendo sobrevivir
a la vida adulta, presente en sx de noonan, Tratamiento con valvuloplastia con baln. OBSTRUCCION DEL TRACTO DE SALIDA DEL
VENTRICULO IZQUIERDO: Obstruccion valvular bicuspidea, subvalvular o supravalvular, puede presentarse afecciones en multiples
niveles. COARTACION DE LA AORTA: Presente en el adulto con menor posibilidad de reparacin, se presenta predominantemente
distal al inicio de la subclavia izquierda, es silenciosa hasta que se presenta un evento sbito como hipertensivo. TRANSPOSICION DE
GRANDES ARTERIAS: La sobrevida a la vida adulta es casi imposible, se realiza un swich de vasos en la infancia, 50 % de los casos
desarrollan trasntornos sinusales, arritmias, regurgitacin valvular, falla ventricular derecha.

CASO CLINICO
Posteriormente a la correccin de la tetraloga de Fallot realizada con 19 aos de edad, con ampliacin del tracto de salida del
ventrculo derecho, valvotoma pulmonar y cierre de la comunicacin interventricular, deliberadamente quedaron como defectos
residuales, la estenosis pulmonar al nivel valvular por un pequeo anillo pulmonar y la comunicacin interatrial. Pero incluso as,
permaneci con una buena evolucin por 28 aos cuando surgi la primera complicacin por embolia cerebral paradjica con un
discreto accidente cerebrovascular, sin consecuencias sensoriales o motoras. Adems, hace un ao se suscit una complicacin
arrtmica por fibrilacin atrial con frecuencia cardaca elevada, que fue revertida con amiodarona y choque elctrico. Recientemente,
una nueva fibrilacin atrial, pero ahora con una frecuencia ventricular baja (entre 50 a 60 lpm), y insaturacin arterial (70%),
posteriormente a la deambulacin en pocos minutos, motiv la investigacin diagnstica y una conducta ms precisa. Al examen fsico,
el paciente se present en un buen estado general, eupneico, enrojecido, con pulsos normales, pesando 114 kg, altura de 170 cm, PA
de 110/80 mmHg y FC de 60 lpm en ritmo de fibrilacin atrial. La aorta se palpaba discretamente en la frcula. En el precordio, no haba
impulsos o deformidades y el ictus cordis no se palpaba. Los ruidos eran hipofonticos y se auscultaba un soplo sistlico, +/++ de
intensidad, de eyeccin, rudo, en el 1, 2 y 3 espacios intercostales izquierdos en el borde esternal y en la frcula. El hgado no se
palpaba.

PREGUNTA
Cual de las siguientes condiciones es la mas probable causa del estado actual del paciente?

RESPUESTA
a.- Estenosis pulmonar valvular moderada e hipertrofia del ventrculo derecho
b.- Shunt de sangre de la derecha hacia la izquierda por comunicacin interatrial
c.- Defectos residuales posteriores a la correccin de la tetraloga de Fallot
d.- Insaturacin arterial grave a pequeos esfuerzos.

CASO CLINICO
Mujer de 69 aos, intervenida a los 48 aos por CIA tipo ostium primum con disnea de medianos esfuerzos. Se cerr quirrgicamente el
defecto con parche de tefln y se corrigi, mediante sutura, la hendidura de la valva anterior mitral. Precis implante de marcapasos
VVI por bloqueo auriculoventricular completo. Permaneci asintomtica hasta 22 aos despus, cuando present dolor torcico
inespecfico y una prueba de esfuerzo interrumpida por la disnea. Ecocardiogrficamente presentaba un ventrculo izquierdo
hipertrfico con funcin conservada e hipertensin pulmonar severa. La vlvula mitral estaba fibrosada, con insuficiencia moderada e
insuficiencia artica ligera. Presentaba un gradiente sistlico fijo en el tracto de salida del ventrculo izquierdo (TSVI) de 84 mmHg,
ocasionado por una membrana, que pareca partir de la valva anterior mitral. En el cateterismo cardaco se diagnostic una estenosis
severa del tronco de la coronaria izquierda.

PREGUNTA
La ESA tras ciruga correctora de CIA de tipo ostium primum es infrecuente, considerando las datos previos referidos cual de las
siguientes afirmaciones es menos probable?

RESPUESTA
a.- Afecta al 2-15% de los casos
b.- Suele diagnosticarse 6-8 aos tras correccin del defecto en la infancia.
c.- Falta de seguimiento postoperatorio en adultos por evolucin ms lenta.
d.- Su incidencia es de 60-70% y permanece silente durante pocos aos.

CASO CLINICO
Una mujer de 43 aos de edad. Su historial mdico incluye la reparacin de una transicin atrioventricular (AV), comunicacin
interventricular (D-AV) a la edad de 14 aos, que consiste en un ostium primum AVSD y musculoso comunicacin interventricular
restrictiva (VSD), asociado con un velo anterior mitral hendida. Despus de su ciruga, ella permaneci asintomtica hasta que tena 34
aos, cuando experiment la fatiga progresiva. Una ecocardiografa transtorcica mostr shunt residual, tricspide moderada e
insuficiencia artica, insuficiencia mitral moderada y estenosis subartica con un tracto de salida del ventrculo izquierdo fijo (TSVI)
MANUAL DE TRABAJO DEL CURSO ENARM CMN SIGLO XXI
CURSO ENARM CMN SIGLO XXI TEL: 36246001 Pharmed Solutions Institute PGINA 24

gradiente pico de 52 mm Hg. La estenosis subartica se agrav en los ltimos aos, alcanzando un mximo gradiente sistlico de 100
mm Hg. Permaneci activa, pero experiment episodios diaforticas ms frecuentes con actividad leve.

PREGUNTA
Cual de las siguientes alteraciones favorece ms la condicin actual de ESA?

RESPUESTA
a.- Crestas subartica fibromuscular.
b.- Anormales vlvula AV izquierda.
c.- Cuerdas tendinosas de los archivos adjuntos.
d.- Insercin anmala de los msculos papilares del ventrculo izquierdo y generalizada hipoplasia del LVOT.

CASO CLINICO
Masculino de 20 aos con soplo cardaco, cianosis y cansancio observados desde el nacimiento, con ntida acentuacin en los ltimos
aos. Sin control mdico, hematocrito de 83% y hemoglobina de 28 g. Hemafresis repetidas disminuyeron el hematocrito para 73%.
Ninguna medicacin especfica haba sido administrada desde el nacimiento. Examen fsico: Regular estado general, eupneico, cianosis
acentuada, pulsos normales. Peso: 42,6 Kg; Altura: 160 cm; PA: 105/65 mmHg; FC: 82 lpm; FR: 20 rpm. Saturacin O2: 75%. La aorta era
discretamente palpada en la frcula. En el precordio haba pulsos sistlicos discretos, frmito sistlico ntido en todo el borde esternal
izquierdo. Ictus cordis no era palpado. Los ruidos cardacos eran muy hiperfonticos y se auscultaba soplo sistlico acentuado en todo
el borde esternal izquierdo, con ntida irradiacin hacia el rea artica. El hgado no era palpado y los pulmones eran limpios.

PREGUNTA
Considerando el cuadro clnico, antecedetnes y frecuencia de las cardiopatas congenitcas, cual es la condicin mas probable del caso?

RESPUESTA
a.- Estenosis pulmonar.
b.- Doble va de salida de ventrculo derecho.
c.- Doble va de entrada de ventrculo nico.
d.- Estenosis pumonar aislada.

CASO CLINICO
Varn de 21 aos con atresia pulmonar y comunicacin interventricular intervenido de anastomosis aortopulmonar de Waterston-
Cooley (WC) en el primer mes de vida. A los 2 aos se le realiz fstula de Blalock-Taussig (BT) izquierdo modificado y a los 8, ampliacin
de arterias pulmonares con parche de Dacron, implantndose adems un conducto no valvulado entre el ventrculo derecho y las
arterias pulmonares. A los 12 aos se cerr la comunicacin interventricular mediante un parche en teja y se reconstruy el conducto
con tcnica de Danielson. Dos aos ms tarde, el cateterismo cardaco demostr estenosis severa entre el tronco pulmonar y ambas
ramas, con presiones sistmicas en el ventrculo derecho. Se realiz angioplastia con baln de ambas ramas, que fue ineficaz. Nueve
aos despus el paciente se encontraba en grado funcional III de la NYHA y la ecocardiografa puso de manifiesto presin
suprasistmica en el ventrculo derecho, comunicacin interventricular residual pequea con cortocircuito bidireccional e insuficiencia
tricuspdea grado III/IV.

PREGUNTA
Cual de las siguientes causas es menos frecuente en la estenosis e hipoplasias localizadas de las arterias pulmonares secundarias a
ciruga paliativa?

RESPUESTA
a.- Constriccion progresiva.
b.- Ineficacia de angioplastia con baln.
c.- Distorsin de tejido fibroso en su interior.
d.- Crecimiento de tejido fibroso en su interior.

CASO CLINICO
Mujer de 28 aos con atresia pulmonar y comunicacin interventricular intervenida quirrgicamente a los 4 aos de edad,
practicndosele fstula de BT izquierdo clsica. A los 6 aos se le realiz anastomosis de WC por estenosis de la fstula. A los 14 aos
presentaba cianosis progresiva severa y disnea de esfuerzo. El cateterismo cardaco demostr estenosis severa de la fstula de BT e
hipoplasia de las arterias pulmonares, descartndose para ciruga correctora. A los 26 aos, la paciente presentaba cianosis severa,
acropaquias y disnea de mnimos esfuerzos, decidindose la realizacin de una nueva fstula paliativa, que no pudo completarse por la
aparicin de sangrado muy abundante durante la diseccin quirrgica. Mediante angiografa se demostr obstruccin completa de la
fstula de BT en su tercio distal y estenosis severa de la anastomosis de WC.

PREGUNTA
El tratamiento quirrgico de esta complicacin es tcnicamente difcil y la dilatacin con baln habitualmente ineficaz por que razo?

RESPUESTA
a.- Retroceso elstico de la arteria.
b.- Fibrosis persistente suprapulmonar.
MANUAL DE TRABAJO DEL CURSO ENARM CMN SIGLO XXI
CURSO ENARM CMN SIGLO XXI TEL: 36246001 Pharmed Solutions Institute PGINA 25

c.- Falta de angenten inmunomoduladores.
d.- Persistencia de las resistencias intracavitarias.

CASO CLINICO
Varn de 19 aos diagnosticado de tetraloga de Fallot severa a los 3 aos de edad, practicndosele fstula de BT izquierdo seis meses
despus. A los 5 aos se rechaz la ciruga correctora por presentar hipoplasia severa de tronco y arterias pulmonares. Desde entonces
presentaba cianosis progresiva severa, disnea de esfuerzo, acuclillamiento y sncopes. A los 9 aos se realiz ampliacin con parche del
tracto de salida del ventrculo derecho y del origen de ambas ramas pulmonares, as como cierre de la comunicacin interventricular
con parche fenestrado, quedando con presin sistmica en el ventrculo derecho, gradiente transpulmonar severo y cortocircuito
bidireccional ventricular.

PREGUNTA
Cual de las siguientes aseveraciones es correcta, al manejo de stens en esta patologia?.

RESPUESTA
a.- La dilatacin mediante stent no es un medoto seguro ni eficaz.
b.- La combinacin de stent y valvuloplastia con baln tiene mayor eficacia y eficiencia.
c.- La dilatacin mediante stent es un mtodo seguro y eficaz de tratamiento.
d.- En las estenosis de las arterias pulmonares en adultos con cardiopatas congnitas operadas en la infancia no es til los Stens.

VALVULOPATIAS (TEMA)
CIENCIAS BASICAS: Estenosis valvular o dificultad para que la vlvula pueda abrirse. Insuficiencia valvular o imposibilidad de que la
vlvula al cerrarse ocluya completamente el orificio valvular, reflujo anormal de sangre. Focos de auscultacin: foco mitral o apexiano
(5to. espacio intercostal izq. con lnea medio clavicular); foco artico (2do. espacio intercostal con paraesternal der.); foco tricuspdeo
(porcin inferior del esternn); foco pulmonar (2do. espacio intercostal paraesternal der.). Pueden generarse a consecuencia de fiebre
reumtica (infeccin de origen estreptoccico ms mecanismo inmunolgico afectando endocardio valvular), endocarditis bacteriana
(da lugar a formacin de vegetaciones en las vlvulas mutilndolas), malformaciones congnitas, degeneracin mixomatosa (prolapso
de los velos valvulares en insuficiencia), disfuncin y/o rotura de las cuerdas tendinosas (IAM). ESTENOSIS MITRAL: Valvulopata ms
frecuente. Su principal causa es la fiebre reumtica. Ms a mujeres. PATOGENIA: Disminucin del rea del orificio valvular mitral
(normal 5 cm), con lo cual hay dificultad para el relleno del VI, durante la distole, esto va generando hipertrofia de la pared auricular,
porque se requiere mayor presin para mantener el flujo hacia el VI, que puede desencadenar una FA, puede haber estasis de sangre
en la aurcula, generar un trombo y este embolismo sistmico. rea inferior a 2cm, genera sntomas al ejercicio, rea <1cm genera
elevacin de la presin auricular, aumento de presin venosa y capilar (hipertensin pulmonar;) por fallo retrogrado, congestin
pulmonar que genera disnea de reposo, mas EAP. DIAGNOSTICO: Cuadro clnico: Disnea de esfuerzo (sntoma principal), ortopnea,
disnea paroxstica nocturna, edema agudo del pulmn (EAP), episodios de infeccin respiratoria de repeticin, astenia por disminucin
de gasto cardaco, datos de IC der. (Ingurgitacin yugular, hepatomegalia, ascitis) en fases avanzadas, fascies mitral por rubicundez de
las regiones malares, pulso parvo y latido de punta dbil (por cada de vol. sistlico), estertores pulmonares hmedos (si hay EAP).
Auscultacin: Refuerzo del 1er ruido, cierre brusco y enrgico de la vlvula mitral, soplo diastlico con sonoridad de retumbo (se
acenta despus del ejercicio), y reforzamiento presistlico inmediatamente antes del 1er ruido. ECG: Onda p mitral, si no existe FA. Rx:
datos de congestin venocapilar pulmonar, con dilatacin de ramas pulmonares y AI. y ecocardiograma, forma no invasiva mas til para
dx. TRATAMIENTO: Dirigido a aliviar sntomas de congestin pulmonar con diurticos, la FA (beta-bloqueadores, bloqueadores de
canales de calcio) y anticoagulacin. La ciruga es lo definitivo; comisurotoma o cambio valvular. COMPLICACIONES: EAP, hemoptisis,
arritmias (FA); embolismo sistmico (fractura de trombo intraauricular, causando EVC, isquemia mesentrica y de extremidades
inferiores), afona (por compresin de la AI sobre el nervio recurrente). INSUFICIENCIA MITRAL: Causa ms comn cardiopata
reumtica. Pacientes jvenes. PATOGENIA: La vlvula permite reflujo anormal de sangre desde VI a la AI, durante la sstole ventricular,
esto genera sobrecarga en aurcula izq. que lleva a congestin y edema pulmonar. Durante la distole hay sobrecarga de volumen en VI,
generndose una dilatacin compensatoria que genera IC izq. DIAGNSTICO: Cuadro clnico: astenia, fatigabilidad, disnea, ortopnea,
disnea paroxstica nocturna y/o EAP, ICC en fases avanzadas, latido de punta desplazado a la izq. y abajo, amplio poco sostenido, el
ventrculo grande regurgita, puede palparse trill sistlico apical. Auscultacin: 1er ruido apagado (por no cierre completo), soplo
holosistlico (regurgitante, en chorro de vapor), que se propaga hacia la axila, pulso arterial normoamplio y celer. ECG: onda p mitral y
crecimiento de VI con R altas. Rx de trax: dilatacin del VI y AI, con desplazamiento del bronquio principal izq. TRATAMIENTO: Reducir
la pre y poscarga, para disminuir sntomas por congestin pulmonar; el edema pulmonar con diurticos y nitratos; reduccin de la
poscarga con IECAS. Qx definitiva, plastia o cambio valvular. ESTENOSIS ARTICA: Principal causa degenerativa. Adultos 40-60 aos.
Hombres, foma mas comn es la congnita (valva uni o bivalva). PATOGENIA: Dificultad para vaciar el VI, lo que genera aumento de la
poscarga e hipertrofia concntrica del VI (nos puede llevar a isquemia, arritmias y muerte sbita), que conduce a disminucin de la
distensibilidad ventricular, aumentndose la presin telediastolica del VI, mas dilatacin del VI, lleva a insuficiencia mitral, que lleva a IC
retrgrada. DIAGNOSTICO: Cuadro clnico: asintomtica por mucho tiempo. Triada clsica: angina de pecho (30-40%), insuficiencia
cardiaca (20%) y sincope (10-15%, tras esfuerzo intenso), astenia progresiva primer sntoma, disnea de esfuerzo, de reposo,
palpitaciones, visin borrosa. Auscultacin: Latido de punta desplazado hacia la izq. y abajo por hipertrofia de VI, pulso parvo y tardo,
TAS disminuida con escasa modificacin de la TAD. chasquido clic de eyeccin despus del primer ruido, soplo mesositolico de
morfologa romboidal, por disminucin del flujo artico de la vlvula estenosada, cuando la presin intraortica alcanza un determinado
nivel. ECG: signos de crecimiento ventricular izq. con grandes ondas R en V5 y V6 y S profunda en V1 y V2, ondas T negativas por la
sobrecarga de presin y trastornos de la conduccin. Rx de trax: silueta cardiaca normal, porque la hipertrofia concntrica no la
modifica, en fases finales hay cardiomegalia. Ecodopler: nos permite valorar el rea valvular (rea valvular en cm2 >1.5 leve; 1.0-1.5
moderada; < 1.0 severa)y el gradiente transvalvular. TRATAMIENTO: De eleccin es quirrgico (prtesis mecnica). INSUFICIENCIA
ARTICA: Causas fiebre reumtica, sfilis y endocarditis bacteriana. PATOGENIA: Los velos valvulares articos, no tienen un cierre
MANUAL DE TRABAJO DEL CURSO ENARM CMN SIGLO XXI
CURSO ENARM CMN SIGLO XXI TEL: 36246001 Pharmed Solutions Institute PGINA 26

hermtico, lo que genera reflujo diastlico esto aumenta la precarga y genera hipertrofia del VI, que puede llevar a IC, por otro lado el
aumento en el consumo de O2 y la disminucin de la presin diastlica, lleva a isquemia, frecuente en el Sx. de Marfan, diseccin
artica entre otras. DIAGNOSTICO: Cuadro clnico: manifestaciones de IC o isquemia hasta 4 o 5 dcada de la vida, disnea en reposo,
ortopnea, disnea paroxstica nocturna, angina, palpitaciones, pulso capilar (signo de Quincke)c se presiona sobre la ua se observa,
como el borde de la zona plida avanza y retrocede, durante la sstole y distole. Danza carotidea (signo de Corrigan), latido carotideo
visible y muy amplio, ampliacin de la TA diferencial por elevacin de la TA sistlica y disminucin de la diastlica, que incluso puede
ser 0. Auscultacin: 1er y 2do ruidos apagados, soplo pandiastlico decreciente (por la progresiva cada de la presin en la raz de la
aorta durante la regurgitacin), de tono alto, suave y aspirativo, irradindose a lo largo del borde esternal. ECG: Signos de crecimiento
del VI, sobrecarga diastlica (ondas T picudas y supradesnivel del segmento ST. Rx. cardiomegalia con signos de crecimiento del VI y
dilatacin de la raz aortica, ecodopler permite medir grado de ingurgitacin. TRATAMIENTO: Enfocado a corregir sntomas de IC,
oxigeno, diurticos, inotrpicos, IECAS. Qx definitivo, se recomienda en pacientes con dimetros ventriculares >50mm. ESTENOSIS
TRICSPIDEA: Condiciona sobrecarga de presin en sentido retrgrado sobre la aurcula y congestin venosa con clnica de IC der.
Auscultacin en foco tricspide soplo diastlico, con refuerzo presistlico y chasquido de apertura de la tricspide. INSUFICIENCIA
TRICSPIDEA: Clnica de IC der. por regurgitacin sistlica, hacia la aurcula. Auscultacin: soplo sistlico, similar al de la insuficiencia
mitral, pero se ausculta mejor en el foco tricspide., se incrementa durante la inspiracin, por aumentar esta condicin el retorno
venoso. ESTENOSIS PULMONAR: IC der., sobrecarga de presin del ventrculo der.. Auscultacin: Foco pulmonar muestra un clic de
eyeccin y un soplo sistlico de morfologa romboidal. INSUFICIENCIA PULMONAR: IC der., regurgitacin y sobrecarga del VD.
Auscultacin: soplo diastlico decreciente en foco pulmonar, equiparable en su gnesis al de la insuficiencia aortica. CLAVES: La
profilaxis con antibiticos es recomendada, si hay procedimientos invasivos, por alto riesgo de endocarditis infecciosa.

CASO CLINICO
Paciente de 66 aos de edad, con obesidad, hipertensin arterial y dislipemia con diagnstico de estenosis artica grave y cardiopata
coronaria al realizarse ecocardiografas transtorcica y transesofgica; se concluy estenosis artica grave (vlvula artica con
gradientes mximo de 68, medio de 46 y rea de 0,95 cm2) con funcin sistlica conservada e hipertrofia concntrica del ventrculo
izquierdo (septum interventricular de 15 mm y pared posterior de 14 mm). El cateterismo cardiaco demostr enfermedad coronaria
(lesin oclusiva del 80% en el tercio proximal de la descendente anterior y del 75% en el tercio medio de la circunfleja).

PREGUNTA
Considerando los factores del paciente cual es la causa mas probable?

RESPUESTA
a.- Calcificacion valvular.
b.- Degenerativa.
c.- Cardiopatia reumtica.
d.- Fibrosis autoinmune.

PREGUNTA
Cual es el pronstico del paciente conociendo la mortalidad anual de esta patologia?

RESPUESTA
a.- 3 %.
b.- 6 %.
c.- 9 %.
d.- 12 %.

PREGUNTA
Cual de las siguientes etiologias es menos frecuente en este caso?

RESPUESTA
a.- Sindrome de marfan.
b.- Hipercolesterolemia.
c.- Cardiopatia reumtica.
d.- Lupus eritematoso.

CASO CLINICO
Un varn de 87 aos con estenosis artica sintomtica fue remitido para implante de vlvula artica percutnea. El ecocardiograma
preoperatorio mostraba estenosis valvular artica severa (rea calculada, 0,6 cm2), fraccin de eyeccin ventricular izquierda normal y
anatoma cardiaca adecuada para el implante percutneo. La coronariografa previa no mostr lesiones significativas. El paciente fue
rechazado para ciruga por su elevada comorbilidad (EuroSCORE, 23%).

PREGUNTA
Considerando la clasificacin del padecimiento este paciente esta considerado como?

RESPUESTA
a.- Leve.
b.- Moderada.
MANUAL DE TRABAJO DEL CURSO ENARM CMN SIGLO XXI
CURSO ENARM CMN SIGLO XXI TEL: 36246001 Pharmed Solutions Institute PGINA 27

c.- Grave.
d.- Critica.

PREGUNTA
Cual de los siguientes sntomas no esta relacionado con el incremento de la presin venocapilar pulmonar?

RESPUESTA
a.- Disnea en reposo.
b.- Ortopnea
c.- Disnea paroxstica nocturna.
d.- Opresion toracicca.

CASO CLINICO
Paciente de femenino de 65 aos de edad, se detect hipertensin arterial (160/100 mmHg), fibrilacin auricular de inicio
indeterminado. Se inici tratamiento con atenolol 50 mg/da, enalapril 20 mg/da. Desde el ltimo ao disnea progresiva con esfuerzos
menores de los habituales y palpitaciones frecuentes espontneos. En la auscultacin cardaca se apreci arritmia completa por
fibrilacin auricular, un soplo holosistlico 4/6 de alta frecuencia en el foco mitral irradiado a la axila y otro sistlico, de menor
intensidad, en el borde esternal izquierdo bajo, que aumentaba con la inspiracin. En la radiografa de trax se apreci una
cardiomegalia global grado III/IV con campos pulmonares normales. El ECG demostr una fibrilacin auricular controlada, con signos de
crecimiento ventricular izquierdo y alteraciones secundarias de la repolarizacin.

PREGUNTA
Cul es el manejo ms adecuado para el manejo de la insuficiencia cardiaca.

RESPUESTA
a.- Amiodarona.
b.- Hidroclorotiazida.
c.- Digoxina.
d.- Nifedipina.

PREGUNTA
Cul es el manejo ms adecuado para el manejo de los tratornos del ritmo.

RESPUESTA
a.- Amiodarona.
b.- Hidroclorotiazida.
c.- Digoxina.
d.- Nifedipina.

PREGUNTA
Cual es la etiologa mas probable considerando los factores de riesgo que presenta la paciente?

RESPUESTA
a.- Fiebre reumtica.
b.- Sindrome de lutenbacher.
c.- Enfermedad de fabry.
d.- Enfermedad de whipple.

PREGUNTA
Considerando la condicin de la paciente cual de las siguientes causas es la ms frecuente de muerte?

RESPUESTA
a.- Embolia sistmica.
b.- Insuficiencia cardiaca.
c.- Embolia pulmonar.
d.- Infeccin

CASO CLINICO
Mujer de 83 aos, con prtesis mitral mecnica. Fibrilacin auricular crnica. Ecocardiograma transesofgico: rea valvular de 0,5cm2,
hipertensin pulmonar de 75mmHg, fraccin de eyeccin del 65%, anillo artico de 18 mm y prtesis mitral normofuncionante. La
paciente evolucion sin complicaciones. En la revisin de 1 y 3 meses se encuentra en clase funcional I.
PREGUNTA
Cual es el agente etiolgico mas frecuente de esta patologa.

RESPUESTA
a.- Estreptococcos
MANUAL DE TRABAJO DEL CURSO ENARM CMN SIGLO XXI
CURSO ENARM CMN SIGLO XXI TEL: 36246001 Pharmed Solutions Institute PGINA 28

b.- Estafilococcos.
c.- Treponema.
d.- Cocobacilos

CASO CLINICO
Mujer de 74 aos de edad hipertensa, dislipmica, diabtica tipo 2, con nefropata diabtica e insuficiencia renal. Fue sometida a una
comisurotoma mitral, sustitucin valvular mitral con prtesis. Sufre fibrilacin auricular crnica. Deterioro de clase funcional, tras
descartar disfuncin de la prtesis mitral, se la remiti para valoracin de TAVI. Rechazada para ciruga (EuroSCORE, 25%).
Coronariografa: sin estenosis angiogrficas. En el ecocardiograma transesofgico se observ lo siguiente: presin pulmonar sistlica de
80mmHg, anillo artico de 22mm y rea valvular de 0,6cm2. Distancia mitroartica, 7 mm.

PREGUNTA
Cual es el factor de mayor influencia en el pronstico a mediano plazo.

RESPUESTA
a.- Diabetes mellitus.
b.- Dislipidemia.
c.- Hipertensin
d.- Cardiopatia coronaria.

CASO CLINICO
Paciente masculino, 20 aos, previamente sano, presenta episodio de disnea durante el sueo, se quejaba de cansancio a los esfuerzos
medios con evolucin de alrededor de 15 das, siendo caracterizado como clase funcional II de la NYHA (New York Heart Association). Al
examen fsico, presentaba presin arterial 110/70 mmHg, ritmo cardaco regular en dos tiempos, soplo sistlico en foco mitral. El ECG
evidenciaba bloqueo de rama derecha.

PREGUNTA
Cual es la terapeutica mas apropiada para el trastorno del ritmo.

RESPUESTA
a.- Marcapasos.
b.- Amiodarona.
c.- Amlodipido.
d.- Digoxina.

CASO CLINICO
Posteriormente a la correccin de la tetraloga de Fallot realizada a los 19 aos de edad de la paciente, con ampliacin del tracto de
salida del ventrculo derecho, valvotoma pulmonar y cierre de la comunicacin interventricular, quedaron como defectos residuales,
estenosis pulmonar al nivel valvular por un pequeo anillo pulmonar y comunicacin interatrial. Permaneci con una buena evolucin
por 28 aos cuando presenta embolia cerebral por fibrilacin atrial.

PREGUNTA
Considerando el cuadro clnico cual de los siguientes frmacos es el mas apropiado para mantener controlada la agregacin
plaquetaria.

RESPUESTA
a.- Acetilsalicilico.
b.- Clopidrogel.
c.- Acecumarol.
d.- Enoxoparina.

CASO CLINICO
Mujer de 59 aos con historia de implantacin de marcapasos definitivo endocavitario por bloqueo auriculoventricular completo.
Present trombosis de vena cava, que origin un sndrome de vena cava superior y requiri de la colocacin de un stent
autoexpandible. Present estenosis de la vena subclavia izquierda y tromboembolia pulmonar. Es egresado luego de 30 dias de estancia
hospitalaria.
PREGUNTA
Cual de los siguientes factores etiolgicos es mas frecuente en esta patologa.

RESPUESTA
a.- Insuficiencia venosa profunda.
b.- Arterio esclerosis de grandes vasos.
c.- Insuficiencia tricuspidea.
d.- Anticoagulacion insuficiente.


MANUAL DE TRABAJO DEL CURSO ENARM CMN SIGLO XXI
CURSO ENARM CMN SIGLO XXI TEL: 36246001 Pharmed Solutions Institute PGINA 29

PERICARDITIS
DEFINICION: El pericardio consiste en una capa visceral y una lamina parietal que entre si forma un espacio potencial, la cavidad
pericardica contiene 50 ml de plasma infiltrado, sus fnciones limitan la distencin cardiaca, facilita el acople e interacion cardica,
mantienen el volmen de presin cardiaca, la ergometra del ventrculo izquierdo, adems de que lubrica, minimiza la friccion, regula la
inercia, las fuerzas hidrosttica y gravitacionales del coraccion y es una barrera mecnica contra la infecccion, presenta acciones
inmunolgicas, vasomotoras, fibrinoliticas, modula la estructura del miocito, su funcin, asi como su expresin gnica, es un vehiculo
para frmacos y terapia gnica. No es escencial para la vida, sin embargo puede comprometerlo al presentar una patologia.
PERICARDITIS AGUDA: Es un estado agudo firbinoso o sudativo por pericarditis, es un Sx caracterizado por dolor en pecho, friccion
pericardica, cambios ECG, manifestando grado, severidad y localizacin, puede modificar el ST o alteraciones de la repolarizacion como
en el IAM, aun cuando estos son difusos, el RX puede ser normal o relevar la silueta cardiaca por exudado aumentado, sugiriendo
miocarditis o taponade por ejemplo, los marcadores imflamatorios puede incrementarse, en caso de isoenzimas cardiacas es mas
frecuente en epicarditis, en epicarditis idiopticas pueden elevarse semejando IAM, la hospitalizacin de estos casos son para
diferenciar los casos, responden frecuentemente a AINES, aadiendo profilaxis con H2 bloqueadores, la adicion de colchicina es til en
casos de recurrencias, el dolor permanece 1 a 2 dias, el frote y el ECG giualmente, en caso viral se prolonga a mas de 4 dias o en caso de
drogadiccin el dolor es mas severo, requiririendo corticoides sistmicos. DERRAME PERICARDICO: Es el acumulo de transudado,
exudado o hemosudado en el saco pericaidico frecuente en las complicaciones de enfermedades del pericardio, en pericarditis aguda,
el hidropericardio se presenta en pericarditis crnica frecuente en retenion de agua y sodio, o procesos inflamatorios crnicos, la
presencias hemtica o sero hemtica es mas frecuente en infecciones o inflamaciones, en pericarditis quilosa lesiones u obstruccion
del conducto torcico, por coresterol, hipotiroidismo, artritis reumtica, o tuberculososis. El ECG es el procedimiento de eleccin, la
TAC y el IRM son utiles para identificar la inoculacin tpica o atpica, la etiologa del derrame pericardico deber ser identificado por
citologa, anlisis inmunolgico o por biopsia guiada. La pericardiocentesis es diagnostica y teraputica, ya que evita la falla del corazn
hemodinamicamente hablando, por pulso paradjico, disfuncin atrial y ventricular diastlica, generando pulso paradoxico,
disminusion de la presin arterial, hipotensin arterial, la fusin cardiaca decrese hasta verse negativa a negativa, hasta llegar al
prolapso y falla cardiaca por agotacion de los mecanismos compensatorios. TAPONADE CARDIACO: Definido como agudo o crnico, se
observa como una alteracin dinmica continua, con presiones de 10 a 20 mmHg como mximas, en los casos leves es asintomtica, en
la moderada el disconfort y la disnea predominan, las ondas yugulares predominan, se presenta bajo voltaje en el ECG, durante la
inspiracin disminuye, se debe realizar pericardocentesis cuando los sntomas los amerite, a los 50 ml ya iniciaran, sin embargo se
recomienda cutltivo, citologa. PERICARDITIS CONSTRICTIVA: Es una condicin calcificante que limita la funcin diastlica de los
ventrculos, idioptico principalmente, trauma cardiaco, quirrgico o tuberculoso otras infecciones, neoplasias, radioterapia, falla renal
y enfermedades del tejido conectivo como menor causa. La condicin crnica congestivo a semeja enfermedad miocrdica y
enfermedad heptica crnica. El paciente refiere fatiga, disnea, aumento de peso, disconfort abdominal y nausea. Puede llegar a ascitis,
hepatoesplenomegalia, edema confundiendoce con cirrosis, signo de Kussmaul, Knock, tercer ruido, pulso paradjico. QRS de bajo
voltaje, cambios en P y T, fluter atrial es comn, la TAC e IRM define entre pericarditis restrictiva y cardiomiopata contrictiva, pueden
necesitar pericardiectomia con una mortalidad del 30 al 40 %,

CASO CLINICO
Varn de 48 aos, fumador. Haba participado recientemente en la recogida de restos de aves enfermas sacrificadas en granjas.
Consult en urgencias por dolor retroesternal que aumentaba con la inspiracin profunda y la tos empeoraba con el decbito. En la
exploracin fsica destacaba el hallazgo de febrcula (37,5 C) y roce pericrdico. En el electrocardiograma y en la radiografa de trax no
se observaron en aquel momento hallazgos patolgicos. Con el diagnstico de pericarditis aguda el paciente fue remitido a su domicilio
bajo tratamiento con cido acetilsaliclico a dosis antiinflamatorias. Tras una mejora inicial, consult diez das ms tarde por fiebre de
39 C, tos con escasa expectoracin mucosa y dolor torcico de caractersticas pericardticas, de tres das de evolucin.

PREGUNTA
Cual de las siguientes manifestaciones es menos probable encontrar?

RESPUESTA
a.- Roce pericrdico.
b.- Alteracin difusa de la repolarizacin con aplanamiento de las ondas T.
c.- Radiografa de trax con cardiomegalia e infiltrado pulmonar en la lngula.
d.- Determinacin seriada de creatincinasa y de su fraccin MB elevadas.

CASO CLINICO
Mujer de 60 aos con DM diagnosticada 3 aos, HTA de 10 aos de evolucin. Un ao antes tena una creatinina de 0,8 mg/dl; FGE
(MDRD) >60 ml/min/1,73 m; sistemtico y sedimento de orina sin alteraciones. Refiere en las 6 semanas antes del ingreso astenia
progresiva, con posterior aparicin de fiebre y tos seca. No presentaba disnea, dolor torcico ni alteracin en la diuresis. En la
exploracin tena una presin arterial (PA) de 190/80 mmHg, estaba eupneica, con presin venosa yugular elevada, auscultacin
cardaca rtmica con roce pericrdico, normoventilacin en ambos hemotrax y ausencia de edemas. En la analtica, presentaba una
hemoglobina de 7,7 mg/dl, urea de 217 mg/dl, creatinina 4,5 mg/dl, potasio 3,6 mEq/l, saturacin basal de O2 del 98%; en orina, tena
proteinuria de +++ y 60 hemates/campo. En el ECG no haba alteraciones. En la radiografa de trax se observ una gran cardiomegalia,
se solicit una ecocardiografa de urgencia, en la que se evidenci un derrame pericrdico moderado-grave.

PREGUNTA
Cual es la conducta mas apropiada a seguir?


MANUAL DE TRABAJO DEL CURSO ENARM CMN SIGLO XXI
CURSO ENARM CMN SIGLO XXI TEL: 36246001 Pharmed Solutions Institute PGINA 30

RESPUESTA
a.- Pericardiocentesis.
b.- Pericardiectomia.
c.- Diuretico, esteroide.
d.- Esteroide, ciclofosfamida.

CASO CLINICO
Mujer de 69 aos de edad que sufre un trauma torcico cerrado por compresin antero posterior al quedar atrapada por las puertas de
un autobs y 30 das despus comienza a presentar falta de aire a los esfuerzos, aumentando progresivamente hasta desencadenarse a
los pequeos esfuerzos. Ingresa en el hospital con diagnstico de cardiopata isqumica. Se realiza ecocardiograma y se comprueba
gran coleccin lquida pericrdica.

PREGUNTA
Cual de las siguientes manifestaciones es mas frecuente encontrar para establecer el diagnostico actual?

RESPUESTA
a.- Triada de Beck.
b.- Signo de Kussmaul.
c.- Disminucin del voltaje de QRS.
d.- Elevacin de CPK-MB y Troponinas.

PREGUNTA
Considerando la fisiopatogenia del presente caso, que tipo de choque es mas frecuente que presente?

RESPUESTA
a.- Choque Distributivo.
b.- Choque Obstructivo.
c.- Choque Cardiogenito.
d.- Choque hipovolemico.

CASO CLINICO
Varn de 55 aos con recambio valvular mitral (1998) y sndrome pospericardiotoma, que curs con fiebre y cansancio a mnimos
esfuerzos. Posteriormente qued asintomtico. En abril de 2004 present derrame pericrdico severo y taponamiento cardiaco.
Requiri pericardiocentesis. Tras sta, qued inicialmente asintomtico. Reingres en julio de 2004 y refieri que tras el al ta clnica se
sinti bien, pero que en pocas semanas comenz a sentir cansancio a grandes y luego a moderados esfuerzos. En el momento de su
reingreso la presin arterial (PA) era 110/70 mmHg, sin pulso paradjico, pero presentaba ingurgitacin yugular. La auscultacin
cardiaca era rtmica, con 70 lat/min y ruidos protsicos normales. Presentaba una discreta hepatomegalia y edemas con fvea en los
miembros inferiores.

PREGUNTA
Cual de las siguientes afirmaciones no es correcta para el PEC?

RESPUESTA
a.- Es una infrecuente forma de sndrome pericrdico.
b.- Donde la constriccin cardiaca ocurre en presencia de derrame pericrdico significativo.
c.- Los pacientes presentan taponamiento cardiaco, sin sintomticos pospericardiocentesis.
d.- sta se resuelve slo tras pericardiectoma.

CASO CLINICO
Mujer de 32 aos con disnea a los pequeos esfuerzos, edema de miembros inferiores y astenia hace tres meses, se le prescribi
digoxina y furosemida, con una ligera mejora. Rechazaba comorbilidades cardiovasculares, pero con una historia de artralgia
inespecfica en la adolescencia y tres abortos entre 20 y 30 aos de edad. El paciente estaba demacrado con auscultacin pulmonar
normal y las extremidades bien perfundidas. En la evaluacin cardaca, se hallaba con FC = 98 lpm, PA = 100 / 70 mmHg con un ritmo
cardaco regular y un ruido diastlico mitral de +++/4+, definido como un ruido protodiastlico ("knock" pericrdico).
PREGUNTA
Cul es la conducta a seguir para establecer el diagnostico.

RESPUESTA
a.- ECG.
b.- TAC
c.- Rx de torax.
d.- IRM.

PREGUNTA
Cual de las siguientes manifestaciones no es propio de la pericarditis?
MANUAL DE TRABAJO DEL CURSO ENARM CMN SIGLO XXI
CURSO ENARM CMN SIGLO XXI TEL: 36246001 Pharmed Solutions Institute PGINA 31


RESPUESTA
a.- Localizacin retrosternal sbito.
b.- Naturaleza pleurtica.
c.- Se exacerba con la inspiracin.
d.- Disminuye al inclinarse.

PREGUNTA
Cuales no son cambios que se presentan en ECG del paciente?

RESPUESTA
a.- Elevacin del segmento ST de forma cncava.
b.- Hay desarrollo de ondas Q.
c.- El voltaje de la onda R se mantiene.
d.- Depresin del segmento PR.

PREGUNTA
Considerando los antecedentes del caso clnico cual es la etiologa mas frecuente?

RESPUESTA
a.- Idiopatico.
b.- Traumatico.
c.- Bacteriano.
d.- Viral.

PREGUNTA
Cual de no es una secuela de la patologia del paciente?

RESPUESTA
a.- Taponade cardiaco.
b.- Pericarditis recurrente.
c.- Prericarditis restricitva.
d.- Pericarditis constrictiva.

CARDIOMIOPATIA Y MIOCARDITIS (TEMA)
Es un espectro de patologas con diversos mecanismo patognicos, con un final semejante a un sndrome congestivo de falla cardiaca,
el dao o padecimiento del miocardio puede ser primario o secundario, las causas de miocardiopatias mas frecuentes son: miocarditis
infecciosa viral (coxsaxkievirus, echovirus, HIC, Epstein barr, influenza, CMV. Bacteriano (Corynebacterium diphtheriae, streptococcus
pyogenes, staphylococcus aureus, haemophilus pneumoniae, salmonella spp, neisseria gonorrhoeae, leptospirosis, lyme, syphilis,
brucelosis; Fungico (Candida spp, arpergillus spp, histoplasmosis, blastomicosis, cryptoccosis, coccidioidomycosis; Parasitario
(Toxoplasmosis, schistosomiasis, trchinosis). Dilatada (desconocida); Infiltrativa (amiloidosis, sarcoidosis, hemocromatosis).
MIOCARDITIS: Las causas mas frecuentes son procesos infecciosos virales del 1 al 9 % son coxsackie virus B, las manifestaciones son
variadas, puede pasar asintomticas hasta un profundo choque cardiogenico, el antecedente de afeccion viral de 7 a 10 dias y ataque al
estado generalizado el 60 % de los casos, 35% presentan dolor torcico, otra presentacin son los bloqueos cardiacos completos o
taquicardia ventricular. Las manifestaciones clnicas incluyen fiebre, quicardia, signos de insuficiencia cardiaca, disminucin de ruidos
cardiacos, galope, murmullo por regurgitacin mitral, frote pericardico, laboratorios son leucocitosis, eosinofilia, incremento de IgM,
IgG, CPK en aproximadamente 10 %, el ECG taquicardia, alteracin de ST y cambios de la onda T, prolongacin QT, el ECOCG puede
manifestar alteraciones de la funcin ventricular, se clasifica en fulminante, agudo, crnico activo, crnico persistente. Sobrevida a 5
aos de 50 %. El tratamiento es de soporte, diurticos, ECAs, bloqueadores antagonistas de aldosterona, la digoxina puede
incrementar la expresin inflamatoria, AINEs no son utiles, son utiles; esteroides con azatioprine o esteroide con ciclosporina.
ENFERMEDAD DE CHAGAS La tripanosomiasis o enfermedad de chagas es mas comn en america central y del sur, es causada por
tripanosoma cruzi, la lesin cardiaca es mediada por inumunocomplejos. Se disemina via hematogena a varios rganos y sistemas con
una intensa reaccin inflamatoria, fiebre, sudoracin, mialgias y miocarditis. El 5 % son fatales, del 20 al 30 % permanecen
asintomticos en fase latente, los casos crnicos presentan fibrosis de miofibrillas causando cardiomegalia, falla cardiaca, bloqueos y
arritmias. El tratamiento es sintomtico, marcapasos y agente antiparasitarios- ENFERMEDAD DE LYME: Es causada por la infeccin con
la espiroqueta borrelia burgdorferi introducido por picadura, los sntomas iniciales del padecimiento es bloqueo cardiaco completo,
puede verse disfuncin ventricular izquierda, la miopsia muestra datos de miocarditis activa. MIOCARDITIS REUMATICA: Se puede
observar durante la fiebre reumtica aguda seguido de faringitis streptococcica del grupo A, El diagnostico clnico se hace con los
criterios de JONES. Mayores; carditis, poliartritis, corea, eritema marginatm, ndulos subcutneos, y evidencia de infeccin previa. Los
criterios Menores son fiebre artralgias, fiebre reumtica, elevada sedimentacin eritrocitaria, protena C-reactiva y prolongacin del
intervalo PR. El diagnostico es con dos mayores y un mayor y dos menores, del 5 al 10 % desarrollan insuficiencia cardiaca. Se presenta
carditis, lesion valvular aortica predominantemente. ECG prolongacin de PR y cambios inespecficos del ST-T. Tx aspirina, corticoides y
1.2 millones U de penicilina G benzatina. MIOCARDITIS NO INFECCIOSAS: La miocarditis hipersensitivas y son causadas por reaccin a
medicamentos, caracterizada por eosinofilia, infiltracin al miocardio de eosinofilos, clulas gigantes multinucleadas, y leucocitos, los
frmacos mas frecuentes son: metildopa, penicilina, tetraciclina y antituberculosos, pueden causar emadamente raras que generan
miocariditis, progresiva sin respuesta a tratamiento, mas frecuente en adulto jove, asociado a enfermedades autoinmunes 20 %, se
MANUAL DE TRABAJO DEL CURSO ENARM CMN SIGLO XXI
CURSO ENARM CMN SIGLO XXI TEL: 36246001 Pharmed Solutions Institute PGINA 32

observa histiocitos y linfocitos y eosinofilia, es fatal solo ttx transplante. Cardiomiopata periparto, 1 en 3,000 partos, multiples factores,
obesidad, gestacion multiple, preeclapmsia, hipertensin crnica, px con 30 aos aproximadamente. La falla cardiaca es variable inicia
en el tercer trimestre, ECG con hipertrofia ventricular izquierda, pronostico reservado. Miocardiopatias por enfermedades
neuromusculares, distrofias neuromusculares hereditarias, asociadas a cardiomiopatas Becker, Duchenne, Steinert, distrofia miotonica,
Friedreich, Barth. CARDIOMIOPATIAS POR ENFERMEDADES ENDOCRINAS: El exceso o disminucin de hormonas tiroideas, en la
tirotoxicosis se compromete eventualmente la funcin ventricular izquierda con dilatacin pudiendo llegar a falla cardiaca, El
feocromocitoma es otra causa con hipertensin, sudoracin, palpitaciones e hipotensin ortostatica, puede presentarse miocarditis
inducida por catecolaminas, puede llegar a fallar cardiaca o arritmias ventriculares malignas. La acromegalia presenta cardiomiopata en
el 10 al 20 % de los casos por el exceso de hormona de crecimiento que genera hipertrofia de miocitos que se fibrosan, con disfuncin
diastlica y sistlica finalmente, tambin degeneracin del nodo AV con bloqueo completo. MIOCARDIOPATIAS POR TOXINAS: El abuso
crnico de ALCOHOL es un riesgo mayor para desarrollar cardiomiopata congestiva, 45 % de todas las dilatadas, puede generar
insuficiencia cardica, hipertensin y arritmias, el dao es directo, acetilaldehido y metabolitos, asi como deficiencia nutrimentales,
estimulacin simpatica y cofactores toxicos. La edad promedio de 30 a 55 aos, con historia de 10 aos de consumo intenso, la
fibrilacin atrial es la forma mas frecuente de preentacion luego la muerte sbita. COCAINA produce isquemia miocrdica, infarto,
espasmo coronario, arritmias cardiacas, muerte sbita, miocarditis y cardiomiopata dilatada. Mal pronostico, tratamiento sintomtico
los betabloqueadores puede disminuir espasmos. QUIMIOTERAPICOS: doxirrubicina, ciclofosfamida, generan citotoxicidad, la radiacin
de mediastino, el trastuzumab es un anticuerpo monoclonal se puede presentar miotoxicidad. La CLOZAPINA antipsicotico de uso
crnico. Fenotiacinas, cloroquina, litio, cobalto, hidrocarbonos, interferon alfa IL-2. CARDIOMIOPATIAS ASOCIADAS CON DEFICIENCIAS
NUTRIMENTALES: Deficiencia de TIAMINA, resulta en beri beri con sntomas caracteristicos de falla cardiaca, pronostico fatal sin
tratamiento, Deficiencia de VITAMINA D, la deficiencia o el exceso aumenta el riesgo, falta de absorsion de SELENIO, la disminucin de
L-CARNITINA. CARDIOMIOPATIAS TAKO-TSUBO: La miocardiopata de Takotsubo es un sndrome recientemente descrito atribuido al
exceso de catecolaminas, probablemente relacionado con una hiperactividad simptica inducida por un factor estresante de cualquier
tipo. Son numerosas las enfermedades neurolgicas que se han descrito relacionadas con este sndrome: hemorragia subaracnoidea,
ictus, esclerosis mltiple, sndrome de Guillain-Barr, crisis miastnicas y crisis epilpticas. Del mismo modo, son numerosos los casos
de sndrome de Takotsubo asociados a enfermedades psiquitricas, dependencia a opioides, alcoholismo, trastorno manaco-depresivo,
trastorno depresivo, esquizofrenia. Tambin se han relacionado diversos factores emocionales (malas noticias, fiestas sorpresa,
discusiones, divorcio, muertes inesperadas...) y fsicos (ejercicio, neumotrax, hipoglucemia, ataque de asma, cirugas...). El sndrome
de Takotsubo, descrito por primera vez en Japn en 1991, se caracteriza por acinesia o discinesia de la porcin apical y media del
ventrculo izquierdo que no corresponde con un nico territorio vascular en ausencia de enfermedad coronaria, demostrada mediante
cateterismo cardaco. El electrocardiograma puede mostrar elevacin del segmento ST o inversin de la onda T. Como criterios de
exclusin, entre otros, se encuentran antecedentes recientes de hemorragia intracraneal, feocromocitoma, miocarditis, miocardiopata
hipertrfica, hipertiroidismo, pancreatitis y envenenamiento. Como desencadenantes se han relacionado factores de estrs tanto de
tipo emocional como fsico. CARDIOMIOPATIA DILATADA: Es una causa frecuente de insuficiencia cardaca y es el diagnstico ms
frecuente en pacientes sometidos a trasplante cardaco. Desde el punto de vista clnico, la MD se caracteriza por dilatacin y disfuncin
contrctil del ventrculo izquierdo o de ambos ventrculos. La dilatacin ventricular es generalmente severa y se acompaa siempre de
hipertrofia. La MD puede ser idioptica, gentica/familiar, viral y/o inmune, alcohlica/txica, o asociada a otras cardiopatas en las
cuales el grado de disfuncin miocrdica no se explicara por una determinada sobrecarga hemodinmica o severidad de dao
isqumico. Probablemente, el sndrome clnico de la MD representa un final comn al que se llega a travs de mltiples mecanismos
citotxicos, metablicos, inmunolgicos, infecciosos y familiares. Anatoma patolgica: Los estudios post mortem demuestran
habitualmente dilatacin de las cuatro cmaras cardacas, en especial los ventrculos, que se acompaa a veces de aumento de grosor
de la pared. Las vlvulas cardacas son intrnsecamente normales y es frecuente la presencia de trombos intracavitarios. Es tpica la
presencia de miocitos hipertrficos y muertos, sustituidos por fibrosis con variable afectacin del sistema de conduccin. Los
componentes de la matriz extracelular estn aumentados de forma no selectiva. Aunque la ausencia de clulas inflamatorias se utiliza
como criterio para diferenciar la MD de la miocarditis, en algunos casos de MD se detectan clulas T inflamatorias y clulas endoteliales
activadas, sugiriendo la posible contribucin de un proceso inflamatorio crnico en la patogenia de esta enfermedad. La etiologa y los
mecanismos patognicos son desconocidos en alrededor de la mitad de los casos de MD. Para explicar el dao miocrdico crnico y
progresivo se han propuesto tres principales etiopatogenias: a) infeccin viral crnica del miocardio que produce dao celular; b)
alteracin de los mecanismos inmunes que conduce probablemente a una enfermedad autoinmune, y c) factores genticos que seran
directa o indirectamente responsables de la enfermedad. DIAGNSTICO: El estudio del paciente con MD debe enfocarse no slo al
establecimiento del diagnstico sindrmico, sino hacia la identificacin, por los mtodos de diagnstico habituales, de posibles causas
tratables o reversibles de la enfermedad. La historia clnica debe incluir preguntas relativas al posible consumo de alcohol y cocana,
medicamentos, hbitos nutricionales, estancias en zonas endmicas para infecciones, relacin con animales, embarazos recientes,
transfusiones sanguneas, historia familiar de MD, somnolencia diurna y exposicin profesional a txicos. Asimismo, la idea de que la
MD idioptica es con frecuencia un problema gentico hereditario debe ser tenida en cuenta en la prctica clnica, estudiando
sistemticamente a los familiares de primer grado del paciente. En la mayora de los pacientes la MD se manifiesta clnicamente entre
los 20 y 60 aos de edad, aunque la enfermedad puede afectar tambin a nios y ancianos. Los sntomas ms frecuentes son los de
insuficiencia cardaca (disnea de esfuerzo progresiva, ortopnea, disnea paroxstica nocturna y edemas perifricos). Otras formas de
presentacin son la deteccin accidental de cardiomegalia asintomtica y los sntomas relacionados con arritmias, alteraciones de
conduccin, complicaciones tromboemblicas o muerte sbita. La exploracin fsica suele revelar diferentes grados de cardiomegalia y
signos de insuficiencia cardaca. La presencia de un galope presistlico (cuarto ruido) puede preceder a la aparicin de insuficiencia
cardaca. El ritmo de galope ventricular (tercer ruido) es la regla en los casos con descompensacin de la IC. Es frecuente la presencia de
soplos sistlicos de insuficiencia mitral o, menos frecuentemente, tricuspdea. Las dos causas ms frecuentes de muerte en pacientes
con MD e insuficiencia cardaca son la muerte sbita y el fallo de bomba progresivo. MIOCARDIOPATA HIPERTRFICA: La
miocardiopata hipertrfica (MH) es una enfermedad con una importante heterogeneidad en cuanto a su base gentica,
manifestaciones clnicas y pronstico 93-96. Se caracteriza, fundamentalmente, por la presencia de una hipertrofia ventricular de causa
desconocida (generalmente de predominio septal) y por una excelente funcin sistlica 93-96. Los gradientes intraventriculares
MANUAL DE TRABAJO DEL CURSO ENARM CMN SIGLO XXI
CURSO ENARM CMN SIGLO XXI TEL: 36246001 Pharmed Solutions Institute PGINA 33

dinmicos constituyen un rasgo primordial de esta entidad, pero sabemos que slo se detectan en una parte de los pacientes y buena
parte de las manifestaciones son consecuencia de las alteraciones diastlicas. El diagnstico clnico de los pacientes con MH sigue
basndose en la demostracin de una hipertrofia ventricular en ausencia de factores cardacos o sistmicos que la justifiquen. Los
sntomas ms frecuentes -disnea, angina, palpitaciones-, se manifiestan en ms de la mitad de los pacientes, pero son comunes en
otras patologas cardiovasculares. Sin embargo, algunos datos clnicos como, por ejemplo, disnea en presencia de una buena funcin
sistlica o angina con coronarias angiogrficamente normales pueden orientar hacia el diagnstico. Mucho ms especfica, sin
embargo, sera la aparicin de sncope o presncope en individuos jvenes previamente asintomticos. Un episodio de muerte sbita
recuperada en un nio, joven, o adulto, sin duda obliga a descartar esta patologa. Tras la anamnesis, la exploracin fsica dirigida
puede ser diagnstica cuando se detecta una semiologa caracterstica de gradiente intraventricular dinmico, pero en caso contrario,
es poco reveladora. Con frecuencia, un electrocardiograma manifiestamente patolgico (ondas Q, hipertrofia ventricular, patrones de
preexcitacin o severas alteraciones de la repolarizacin), muchas veces en un individuo asintomtico, es la primera clave diagnstica.

CASO CLINICO
Masculino de 38 aos de edad, sin antecedentes personales ni familiares de inters, sin hbitos txicos, ni tratamiento habitual. Refera
un cuadro catarral de 7 das de evolucin tratado con amoxicilina-clavulnico. Un da antes del ingreso present disnea progresiva hasta
la ortopnea, presentaba una tensin arterial indetectable y una expectoracin espumosa, abundante y hemoptica, junto a signos de
insuficiencia respiratoria severa (taquipnea, cianosis, crepitantes pulmonares generalizados y desaturacin del 70%). El ECG mostraba
una taquicardia sinusal a 180 lpm, con QRS ancho, disociacin auriculoventricular y morfologa de bloqueo de rama izquierda, criterios
de taquicardia ventricular (TV) y, la radiografa torcica, unos infiltrados algodonosos bilaterales. Analticamente, salvo poliglobulia,
acidosis metablica e intensa hipoxemia, no presentaba otros hallazgos de inters incluyendo los enzimas cardiacos.

PREGUNTA
Cual es la forma mas frecuente que debute esta patologia MCG?

RESPUESTA
a.- Taquicardia ventricular.
b.- Insuficiencia cardiaca.
c.- Infarto al miocardio.
d.- Bloqueo auriculoventricular completo el 5%.

CASO CLINICO
Paciente mujer de 18 aos de edad, natural y procedente de Lima. Alergia a AINES. Acudi por presentar en forma brusca, malestar
general, nauseas, vmitos y deposiciones lquidas sin moco, ni sangre (4 horas de inicio). En el examen de ingreso se constat que sus
funciones vitales estaban dentro de lmites normales. Se le diagnostic gastroenterocolitis aguda y deshidratacin leve-moderada; en
ese sentido, y de acuerdo con los diagnsticos planteados, en el manejo inicial se administr hidratacin endovenosa con cloruro de
sodio al 9%, pargeverina clorhidrato 10mg y dimenhidrinato 50mg. Una hora despus de la admisin, la paciente desarroll un cuadro
de hipotensin (presin arterial: 70/50 mmHg) y palidez marcada, recibi tratamiento de soporte, pero ante el deterioro clnico
paulatino la paciente fue conducida a la unidad de cuidados intensivos.

PPREGUNTA
Cual es la imprension diagnostica inicial del caso?

RESPUESTA
a.- Estado de choque sptico.
b.- Estado de choque anafilctico.
c.- Estado de choque cardiogenico.
d.- Estado de choque hipovolemico.

CASO CLINICO
Un varn de 42 aos fue hospitalizado por fiebre y sntomas de dolor torcico. Antecedente de (VIH), diagnosticado 15 aos antes del
ingreso. El recuento de clulas T CD4+ al ingreso era de 874 cl./l y la carga viral era indetectable (< 50 copias/ml). En tratamiento con
tenofovir + didanosina + atazanavir + ritonavir, con una carga viral indetectable en los ltimos 6 aos. El cuadro se inici con fiebre,
diarrea, mal estado general y mialgia de 1 semana de duracin. En los 2 das previos al ingreso sufri un dolor torcico punzante intenso
y progresivo, en reposo, con irradiacin a la espalda, que se agravaba con el decbito y la inspiracin profunda. La intensidad del dolor
torcico disminuy tras la administracin de analgsicos intravenosos, pero no con nitroglicerina sublingual. En la exploracin fsica, las
constantes vitales eran normales, excepto la temperatura corporal, que era de 38 C. No presentaba signos de congestin sistmica o
pulmonar ni haba signos de bajo gasto ni dificultad respiratoria. Los ruidos cardiacos eran normales. Los datos de laboratorio revelaron
leucocitosis con linfocitosis, protena C reactiva en 19 mg/ml y un pico de troponina I cardiaca que alcanz 30 ng/ ml. La radiografa de
trax fue anodina. El ECG mostr un ritmo sinusal normal, con una elevacin del segmento ST de 1 mm en las derivaciones DI, aVL y V4-
V6.
PREGUNTA
Cual es el agente mas frecuente de esta patologia?

RESPUESTA
a.- Parvovirus B19.
MANUAL DE TRABAJO DEL CURSO ENARM CMN SIGLO XXI
CURSO ENARM CMN SIGLO XXI TEL: 36246001 Pharmed Solutions Institute PGINA 34

b.- H1N1.
c.- VIH.
d.- Cocxackie B6.

CASO CLINICO
Mujer de 43 aos con historia de epilepsia focal del lbulo temporal izquierdo criptognica farmacorresistente, trastorno lmite de la
personalidad, etilismo crnico, infeccin por el virus de la hepatitis C. Fue ingresada en Urgencias por presentar una crisis focal motora
secundariamente generalizada. Posteriormente, durante las primeras 24 horas de observacin, sufri cuatro crisis tonicoclnicas
generalizadas ms, la frecuencia cardaca durante la crisis descendi hasta 35 latidos por minuto. Mostr tambin disnea y se objetiv
en la auscultacin cardiopulmonar crepitantes bibasales, bradicardia extrema.

PREGUNTA
Considerando las manifestaciones clnicas cual de las siguientes condiciones es la mas probable que presenta el paciente?

RESPUESTA
a.- Choque Distributivo.
b.- Choque Cardiogenico.
c.- Choque Hipovolemico.
d.- Choque Oculto.

CASO CLINICO
Mujer de 50 aos, procedente de zona rural. Present un cuadro clnico de 15 das de evolucin consistente en fiebre, deposiciones
diarricas y, posteriormente, disnea, el cual la oblig a consultar en varias ocasiones. La sintomatologa, especialmente la disnea,
empeor, por lo cual consult nuevamente, encontrndose en el examen fsico signos claros de insuficiencia cardiaca. En la radiografa
de trax se 7evidenci cardiomegalia y derrame pleural, y el ecocardiograma report taponamiento cardiaco.

PREGUNTA
Cual es su conducta a seguir mas adecuada?

RESPUESTA
a.- Pericardiocentesis.
b.- Pericardiotomia.
c.- Diuretico, corticosteroide, b-bloqueador.
d.- Antiparasitario, pericardiocentesis.

CASO CLINICO
Hombre de 26 aos, procedente de zona rural. Consult por cuadro de cinco das de fiebre, escalofros, dolores musculares, cefalea,
artralgias, astenia y adinamia. Se realiz estudio para hemoparsitos, encontrndose ocasionales tripomastigotes. El ecocardiograma
mostr leve derrame pericrdico y leve dilatacin ventricular izquierda. Adems, se encontr franca leucopenia (3.200 leucocitos/l) y
plaquetopenia (69.000 plaquetas/l).

PREGUNTA
Cual es la conducta farmacolgica especifica para el caso?.

RESPUESTA
a.- Metronidazol.
b.- Albendazol.
c.- Benzonidazol.
d.- Prazicuantel.

CASO CLINICO
Varn de 36 aos, labrador, sin antecedentes de inters salvo picaduras frecuentes de garrapatas. Ingresa en nuestro hospital por
episodio sincopal y bloqueo AV avanzado. En las horas siguientes al ingreso se contina apreciando un bloqueo AV completo, con
pausas de hasta 6 segundos de duracin, implantndose un marcapasos provisional. Un mes previo al ingreso refiere un eritema en
regin gltea (cuyas caractersticas no se pudieron precisar), fatiga y mareos. Dos semanas ms tarde, comienza con artralgias en codos
y rodillas, astenia y disnea. La exploracin fsica fue anodina salvo por febrcula vespertina. La radiografa de trax y la analtica bsica
fueron normales.

PREGUNTA
Considerando el agente causal mas probable del caso por sus antecedentes, cual es la pauta antibitica mas adecuada?

RESPUESTA
a.- Ceftriaxona.
b.- Doxiciclina.
c.- Ciprofloxacina.
d.- Gentamicina.
MANUAL DE TRABAJO DEL CURSO ENARM CMN SIGLO XXI
CURSO ENARM CMN SIGLO XXI TEL: 36246001 Pharmed Solutions Institute PGINA 35

CASO CLINICO
Una mujer de 41 aos con antecedente de HTA ligera no tratada acude a urgencias de su hospital presentando dolor torcico de
caractersticas anginosas y elevacin del segmento ST en cara lateral. Desde 3 aos antes sufra un trastorno de ansiedad que haba
requerido ingreso en una ocasin. En las ltimas semanas la paciente sufra un gran estrs laboral. Presin arterial de 110/70 mmHg;
tras nitroglicerina sublingual, la clnica cedi y se normaliz el ECG. A las 12 h vuelve a tener dolor con elevacin del segmento ST,
cateterismo cardiaco normal, pero la ventriculografa muestra una disfuncin medioventricular severa, con hipercontractilidad de los
segmentos basales y apicales. Se inici tratamiento con BB e IECA, manteniendo la anticoagulacin y la aspirina. La paciente
permaneci asintomtica, con creatincinasa normal y troponina I de 1,4. A los 5 das del ingreso se realiz una RM cardiaca que todava
mostr una hipocinesia medioventricular, sin realce tardo tras la administracin de gadolinio. Al ingreso en nuestro hospital se haban
solicitado catecolaminas en sangre y en orina para una mejor caracterizacin del cuadro. stas mostraron una elevacin supranormal
(noradrenalina y adrenalina en sangre > 5.000 y 190 pg/ml; noradrenalina y metanefrina en orina de 24 h, 582 y 5.386 g/24 h).

PREGUNTA
Cual es la conducta mas apropiada a seguir?

RESPUESTA
a.- Resonancia magnetica abdominal.
b.- Tomografia axial computada.
c.- USG abdominal.
d.- Tomografia helicoidal.

CASO CLINICO
Mujer de 30 aos sin antecedentes de cardiopata que ingresa en nuestro centro debido a un cuadro de insuficiencia cardiaca
congestiva un mes y medio despus de un parto sin complicaciones. Se realiz un ecocardiograma en el que se objetiv dilataci n
moderada del ventrculo izquierdo (dimetro telediastlico, 64 mm; dimetro telesistlico, 46 mm) junto con hipocinesia generalizada y
fraccin de eyeccin del ventrculo izquierdo del 32%; la aurcula izquierda estaba ligeramente dilatada y las cavidades derechas, en el
lmite superior de la normalidad; se apreciaron regurgitaciones mitral y tricuspdea moderadas-severas sobre vlvulas estructuralmente
normales, con estimacin de presin arterial pulmonar sistlica de 46 mmHg. Tras la estabilizacin, se realiz una cardiorresonancia
magntica, en la que se confirm que el ventrculo izquierdo estaba moderadamente dilatado, con fraccin de eyeccin del 37%, as
como hipocinesia generalizada del ventrculo derecho con funcin sistlica general severamente deprimida (fraccin de eyeccin, 25%);
en el estudio de retencin miocrdica tarda de contraste con gadolinio, se observ un depsito mesocrdico lineal a nivel septal
extenso. En una coronariografa no se objetivaron lesiones coronarias. Con el diagnstico de miocardiopata periparto.

PREGUNTA
Cuales son los siguiente criterios no es til para establecer el diagnostico de cardiomiopata periparto?

RESPUESTA
a.- Desarrollo de insuficiencia cardiaca en el ltimo mes de embarazo.
b.- Desarrollo en los 2 meses siguientes al parto.
c.- Ausencia de causa identificable para el desarrollo de insuficiencia cardiaca.
D.- Ausencia de enfermedad cardiaca primaria en el ltimo mes de embarazo.

ENDOCARDITIS
La endocarditis infecciosa es una enfermedad causada por un agente microbiano que afecta la capa endotelial de estructuras
intracardiacas que invaliablemente es fatal sin tratamiento. La infeccin ms frecuente reside en una o mas vlvulas que envuelve el
endocardio mural, miocardio y pericardio. Los dispositivos intracardiacos o endovascular es una fuente de infeccin. La mortalidad es
del 25 % dentro de los 6 meses. EPIDEMIOLOGIA: La causa inicial de EI era una complicacin de fiebre reumtica o de origen dental,
actualmente la degeneracin de vlvulas cardiacas o dispositivos se han incrementado. Los pacientes con DM, VIH, IRC esta en mayor
riesgo de EI con riesgo nosocomial. PATOGENESIS: La EI es una bacteremia persistente y continua endocardica o endovascular. El
agente patgeno lesiona estructuras cardiacas y presenta capacidad de adhesin, y evitar la actividad inmunolgica del husped,
generandoce vegetaciones en vlvulas. PATOGENIA: 80 % son estreptococos y estafilococos, en el mundo el Staphylococcus aureus es
el ms comn. ENDOCARDITIS DE VALVULAS NATIVAS: El estreptococo viridans o el estreptococo alfa-hemolitico es el agente mas
comn adquirido en la comunidad. Es un agente nativo en la orofaringe que fcilmente entra a la circulacin via dao dental o gingival.
El estreptococo viridan (S. bovis, S. mutans, S. mitor) nativos en sistema gastrointestinal o debido a patologia del sistema GI. El grupo D
de estrptococos en particular el enterococos spp. Causan hasta el 18 % de los casos de EI. E. fecalis se presenta hasta el 80 %, su ingreso
a circulacin es por manipulacin con sonda Foley, colonoscopia, etc. El estreptococo del grupo A, es una causa rara el S. pneumonie
causa 10 % de EI es agudo, fulminante, asociado a dao severo valvular con extensin perivalvular, complicaciones embolicas,
pericarditis, meningitis con mortalidad del 25 al 50 %. El grupo B (S. agalactiae) se presenta en neonatos, parturientas, en pie diabtico,
carcinoma, falla heptica, alcoholismo, uso de drogas inyectables. Estafilococo aureus causa del 80 a 90 % de IE, es la causa aguda mas
frecuente, la nasofaringe es el sitio mas comn de conolizacion, los factores de riesgo son dilisis, diabetes, quemaduras, VIH, usuarios
MANUAL DE TRABAJO DEL CURSO ENARM CMN SIGLO XXI
CURSO ENARM CMN SIGLO XXI TEL: 36246001 Pharmed Solutions Institute PGINA 36

de drogas, en condiciones dermatolgicas crnicas. La IE fulminante es causada por El S. aureus con falla carrdiaca, afeccion
perivalvular, trastornos de conduccin, infeccin metastasica con mortalidad del 25 al 30 %, ocacionalmente es debido a estafilococo
coagulaza positivo nativo de la piel. El S. epidermidis es una causa importante en dispositivos ol prtesis valvulares. La IE debido a
bacilos gran negativos, es infrecuente, en patologia heptica o prtesis cardiaca, es difcil cultivarlo, Coxiella burneti agente causante de
la fiebre Q, la Bartonella es otra causa. La IE la causa fungica presenta alta mortalidad 20 % en pacientes inmunocomprometidos, con
dispositivos, candida y aspergelius son los mas comunes, generan vegetaciones tratamiento con anfotericina. Puede presentarse por
causas autoinmune. En caso de IE se presenta entre los 2 meses y el primer ao del tratamiento quirrgico, las causas nosocomiales son
estafilococo coagulasa negativo (s. epidemidis).

CASO CLINICO
Mujer de 46 aos, afectada de nefropata lpica IV, en programa de diaslisis peritoneal. Desde entonces mantiene marcadores
positivos. Tuvo un brote cutneo-articular, por lo que reciba con micofenolato sdico a dosis de 180 mg y prednisona a dosis de 5 mg
diarios. Ingresa por disnea y malestar general progresivo, de 15 das de evolucin. Dolor torcico en el hemitrax izquierdo que
aumenta con la inspiracin profunda y que mejora relativamente en anteversin. No refiere sndrome febril, ni otra clnica
acompaante. EF soplo diastlico en el foco artico, irradiado a las cartidas, con roce pericrdico importante, sin signos de fallo
cardaco. El resto de la exploracin fsica fue anodina. En la analtica destacan: leucocitos * 21,3 K/l (4,4-11,3), cayados 3%, neutrfilos
* 92,0% (50-70), linfocitos * 3,0% (25-40), protena C reactiva (PCR) * 17,73 mg/dl (0,1-0,5), procalcitonina * 4,84 ng/ml (<0,5).
Anticuerpos antinucleares (IFI) 320-640 u arb (0-80), anticuerpos anti-ADN (EIA) 1,3/ml (<10 U/ml), anticuerpos anti-ADN (IFI) <80 u arb
(0-80), anticuerpos cardiolipina (IgG) 3,5 U GPL/ml, IgM 2,5 U GPL/ml (negativos), C3 118 mg/dl (79-152), C4 24,4 mg/dl (16-38), urea
127, creatinina 8,46 mg/dl, hemoglobina (Hb) 9,6 g/dl, hematocrito 29%. Tiempo de cefalina 27,9/30 segundos (29-31 s),
anticoagulante lpico positivo.

PREGUNTA
Cual es la conducta antibitica empirica mas adecuada?

RESPUESTA
a.- Vancomicina, ceftriaxona y dicloxacilina.
b.- Amilacina, doxiciclina y ceftaxidima.
c.- Vancomicina, ceftazidima y gentamicina
d.- Gentamicina, vancomicina y imipenem.

CASO CLINICO
Varn de 38 aos de ocupacin pintor, con signos de disnea, ortopnea, edemas en miembros inferiores, adems de registros febriles
aislados de 2 meses de evolucin. Relat tambin la presencia de un soplo cardaco desde haca 3 aos sin estudio, enfermedad
periodontal, caries dentales y otros antecedentes sin relevancia para el caso. Negaba contacto con animales de cra o consumo de
productos sin pasteurizacin. Dos semanas antes de la consulta present tos con expectoracin purulenta, que cedi
espontneamente. Dos horas antes de la consulta present dolor precordial, opresivo, intensidad 8/10, con irradiacin a brazo
izquierdo. El ecocardiograma transtorcico, la presencia de dilatacin de la aurcula izquierda, dos imgenes ecodensas sobre cara
ventricular de la vlvula artica bicspide, compatibles con vegetaciones de 28,4 mm x 9 mm y de 24 mm x 6 mm cada una,
insuficiencia valvular artica grave, ventrculo izquierdo dilatado, hiperdinmico, con signos de sobrecarga de volumen, funcin sistlica
conservada y leve derrame pericrdico, todos signos compatibles con endocarditis infecciosa.

PREGUNTA
Cual de los criterios es el indicado para endocarditis bacteriana?

RESPUESTA
a.- Criterios de Jones.
b.- Criterios de duke.
c.- Criterios de Baltazar.
d.- Criterios de Hermosillo.

PREGUNTA
Se aislo Brucella canis en los cultivos, cual es la conducta teraputica mas adecuada?

RESPUESTA
a.- Doxiciclina, rifampicina y trimetoprim + sulfametoxazol.
b.- Vancomicina, rifampicina y doxiciclina.
c.- Ceftriaxona, doxiciclina y trimetoprim + sulfametoxazol.
d.- Imipenem, doxiciclina y rifampicina.

CASO CLINICO
Varn de 29 aos de edad con cefalea, confusin y fiebre de hasta 40,3 C, de 3 das de evolucin. Es fumador de tabaco, cannabis,
herona y cocana, con hepatopata crnica por virus de la hepatitis C. A la EF resaltaban somnolencia y desorientacin. Laboratorios
con elevacin de las transaminasas y leucocitosis con neutrofilia. Puncin lumbar mostr un lquido cefalorraqudeo inflamatorio
agudo. El diagnstico inicial fue de encefalitis herptica, posteriomente se aislo Staphylococcus aureus en dos hemocultivos, el
MANUAL DE TRABAJO DEL CURSO ENARM CMN SIGLO XXI
CURSO ENARM CMN SIGLO XXI TEL: 36246001 Pharmed Solutions Institute PGINA 37

ecocardiograma transesofgico evidenci una endocarditis mitral: haba una vegetacin mvil de 3 x 3 mm de dimetro, anclada en la
base de la superficie auricular del velo mitral posterior, que estaba perforado y ocasionaba regurgitacin mitral ligera.

PREGUNTA
Cul es el factor ms probable que cause la endocarditis en este paciente.

RESPUESTA
a.- La bacteria aislada.
b.- La adiccin a inyectables.
c.- La hepatitis C.
d.- La vegetacin mitral.

CASO CLINICO
Mujer de 45 aos de edad, fumadora activa, HTA mal controlada, crioglobulinemia mixta en tratamiento con corticoides e insuficiencia
renal crnica por nefroangiosclerosis en hemodilisis. La paciente ingresa ante la sospecha de meningitis, tras presentar un cuadro de
fiebre de 5 das de evolucin y disminucin del nivel de conciencia. En la exploracin fsica con GSC 8 (M4, O2, V2), rigidez de nuca,
soplo sistlico piante en foco mitral; hipoventilacin basal derecha y signos de infeccin. En la radiografa de trax se observa una
imagen de condensacin alveolar bilateral. TAC craneal en la que se objetiva una lesin hipodensa occipital izquierda, sin dilatacin del
sistema ventricular. Se realiza puncin lumbar cuyos resultados son compatibles con meningitis bacteriana aguda, al mismo tiempo se
obtienen hemocultivos y broncoaspirado. Se inici tratamiento antibitico de manera emprica. Posteriormente, se aisl en todas las
muestras cultivadas S. aureus metilicn sensible (SAMS).

PREGUNTA
Cul es la causa ms probable que genera los cambios en la TAC.

RESPUESTA
a.- Proceso infeccioso.
b.- Hipertension arterial.
c.- Incremento de la actividad plaquetaria.
d.- Uso crnico de corticoides.

CASO CLINICO
Se trata de un paciente de sexo masculino, de 51 aos que ingres a urgencias por 10 das de evolucin de fiebre de 40C, asociada a
diaforesis y deposiciones lquidas sin moco ni sangre. Colecistectoma por laparoscopia 41 das antes, que requiri hospitalizacin por
31 das para el tratamiento de una infeccin del sitio operatorio con compromiso de organo/espacio, tratada con meropenem. No se
haba realizado monitorizacin hemodinmica invasiva y no tena antecedentes de uso de drogas ilcitas intravenosas. En el examen
fsico se encontraba alerta, orientado, con FC de 72 latidos por minuto, FR de 16 respiraciones por minuto, TA de 95/57 mm Hg,
temperatura de 38,5C y deshidratacin grado I. Se auscult un soplo holosistlico en el foco artico, grado II/VI. En el abdomen haba
dolor a la palpacin en epigastrio, sin signos de irritacin peritoneal. En los exmenes de laboratorio con anemia normoctica,
normocrmica, elevacin de la protena C reactiva (PCR) y leve aumento de las transaminasas, y fosfatasa alcalina y amilasa en rango
normal.

PREGUNTA
Cual de siguientes factores de riesgo asociados para la patologia actual es menos frecuente?

RESPUESTA
a.- Cardiopata congnita cianosante.
b.- Comunicacin interventricular.
c.- Conducto arterioso permeable.
d.- Presencia de vlvulas protsicas.

PREGUNTA
Entre los siguientes signos, cual es el menos frecuente en la patologia actual?

a.- Embolismo.
b.- Esplegnomegalia.
c.- Aneurisma micoticos.
d.- Lesiones en retina.

ASMA
CIENCIAS BASICAS: Definicin: Obstruccin reversible al flujo areo producida por inflamacin bronquial, as como por respuesta a
diversos estmulos externos o mediadores endgenos. La mayora de los asmticos tienen enfermedad durante la infancia. El asma
ocupacional puede resultar de gran variedad de productos qumicos, incluyendo disocianato de tolueno y anhdrido trimeltico, y esto
puede ser de aparicin en el adulto. Diferentes disparadores pueden empeorar la sintomatologa como alrgenos inhalados, en especial
a individuos sensibles, infecciones virales, bloqueadores B- adrenrgicos pueden empeorar los sntomas de asma y por lo general se
deben evitarse, el ejercicio entre otros como la contaminacin, exposiciones ocupacionales, cambios bruscos de temperatura, ERGE y
MANUAL DE TRABAJO DEL CURSO ENARM CMN SIGLO XXI
CURSO ENARM CMN SIGLO XXI TEL: 36246001 Pharmed Solutions Institute PGINA 38

estrs. SALUD PUBLICA: Prevalencia en Mxico 10% de la poblacin; 80% son nios menores de 8 aos, del total de casos; 12% tienen
entre 9-17 aos y 8% adultos. La mayora de los asmticos son atpicos y a menudo tienen asma y rinitis alrgica. La mayora de los
asmticos tienen enfermedad durante la infancia. PATOGENIA: En el asma la respuesta del rbol bronquial consiste en disminucin del
calibre de su luz originada por 3 fenmenos; 1) constriccin de la musculatura bronquial (broncoconstriccin). 2) edema de la mucosa e
infiltracin celular (inflamacin). 3) incremento de la secrecin bronquial (moco). Los mecanismos anatomopatologicos ms
representativos son hipertrofia de musculo liso bronquial, transformacin de fibroblastos a miofibroblastos y depsito de colgena
subepitelial (fenmeno de remodelacin), todo esto lleva a largo plazo a disminucin de la funcin pulmonar. DIAGNOSTICO: Cuadro
clnico: Sntomas respiratorios comunes son sibilancias, disnea y tos en accesos, opresin torcica, con predominio nocturno y/o
matutino. Exploracin fsica: Depende gravedad podemos observar taquipnea, uso de msculos accesorios, cianosis, voz entrecortada,
diafortico, taqucardico, rudeza respiratoria. La evidencia de rinitis alrgica, sinusitis, reaccin en la piel podran asesorarnos. Pruebas
de funcin pulmonar: La prueba ideal es la espirometra (medimos resistencia al flujo areo), nos reportara un patrn obstructivo con
reduccin de FEV1 (volumen espiratorio forzado en el primer segundo) y CVF (capacidad vital funcional), si hay sntomas de asma y la
espirometra es normal se pueden realizar pruebas de metacolina, histamina o provocacin con ejercicio. La tasa de flujo espiratorio
mximo (PEF) puede ser utilizado, para seguir el control del asma objetivamente en casa. Rx. de trax: Usualmente normal, en
exacerbaciones agudas podemos identificar neumotrax, sobredistencin por atrapamiento areo, horizontalizacin de arcos costales,
abatimiento de hemidiafragmas, infiltrados pulmonares eosinofilicos. CLASIFICACION Y TRATAMIENTO: Asma extrnseca, pacientes en
o que puede demostrarse una reaccin Ag-Ac como desencadenante del proceso, mediadad por IgE, generalmente atpica u
ocupacional. Asma intrnseca no se encuentra Ag especfico, comienza en vida adulta, se puede asociar a plipos nasales, sinusitis,
idiosincrasia, AS o AINES.


Los efectos secundarios de los B2 agonistas son temblores musculares y palpitaciones. Los agentes antiinflamatorios principalmente
usados son los esteroides inhalados (inhiben la infiltracin celular y disminuyen el edema, efecto de 3 a 6 hrs). Son la primera lnea en
pacientes que no se encuentran en crisis, son los mas efectivos en el control del tx. del asma, el efecto local ms importante es la
candidosis oral, y ronquera los ms usados, budesonina, fluticasona y beclometasona, reducen sntomas durante el ejercicio, y
sntomas nocturnos. Los antileucotrienos (montelukast, zafirlukast), beneficiosos solo en algunos puntos. La adrenalina produce
disminucin de la resistencia muscular sobre el rbol bronquial. Otros brocodilatadores incluyen los anticolinrgicos (que se dan
principalmente en EPOC) y la teofilina es un inhibidor de la fosfodiesterasa que aumenta el adenosin monofosfato ciclico en los
neutrfilos, que tienen efecto brocodilatador y antiinflamatorio, pero debido a su toxicidad asociada, no se usa comunmente.
CRISIS ASMATICAS: Caracterizado por aumento del trabajo respiratorio, tos, opresin torcica y sibilancias; usar salbutamol o
terbutalina, micronebulizados, durante 1 h continua para romper el broncoespasmo. Esteroides endovenosos, metilprednisolona o
prednisona para romper cascada de inflamacin.

CASO CLINICO
Mujer de 40 aos, fumadora con una exposicin tabquica de 20 paquetes-ao. Trabajaba en una empresa metalrgica desde haca 3
aos. Desde haca un ao presentaba clnica de disnea, tos y sibilancias de predominio nocturno, que mejoraba los fines de semana y
perodos vacacionales. Requiri un ingreso por un episodio de broncospasmo y al reincorporarse a su lugar habitual de trabajo se
reiniciaron dichos sntomas. Con ttulos de IgE srica total de 59 U/ml. El estudio de la funcin pulmonar fue normal, mostrando una
capacidad vital forzada de 4,17 l (114%), volumen espiratorio forzado en el primer segundo FEV1% del 79% y prueba broncodilatadora
negativa. Se efectu una prueba de metacolina, que fue negativa, con PC20 superior a 16 mg/ml (concentracin de metacolina que
caus un descenso del FEV1 basal del 20%).

PREGUNTA
Considerando el estado actual que grado de asma presenta el caso?

RESPUESTA
a.- Intermitente.
b.- Leve persistente.

GRADO CARACTERISTICAS TX. RESCATE TX. MANTENIMIENTO
INTERMITENTE Sntomas menos de 1 vez por semana
Exacerbaciones de corta duracin
Sntomas nocturnos no ms de 2 veces al mes
FEV1 o PEF >80% del valor predicho
Variabilidad en el PEF o FEV <20%
Los agonistas 2 adrenrgicos, relajan el
musculo liso (salbutamol) de corta accin,
cuantas veces sea necesario
Baja dosis de antiinflamatorios
PERSISTENTE
LEVE
Sntomas ms de 1 vez por semana pero menos de 1 vez al da
Exacerbaciones que pueden afectar la actividad y el sueo
Sntomas nocturnos ms de 2 veces por mes
FEV1 o PEF >80% del valor predicho
Variabilidad en el PEF o FEV1 <20 a 30%
Los agonistas 2 de corta accin, cuantas
veces sea necesario
Agonista B, de larga accin (salmeterol,
formoterol) + antiinflamatorios en baja dosis
MODERADO
PERSISTENTE
Sntomas diarios
Exacerbaciones que afectan la actividad y el sueo
Sntomas nocturnos ms de 1 vez a la semana
Uso diario de inhaladores con b agonistas de accin corta
FEV1 o PEF 60 a 80% del valor predicho
Variabilidad en el PEF o FEV1 >30%
Los agonistas B de corta accin, cuantas
veces sea necesario
Esteroide inhalado dosis baja (u otro
antiinflamatorio a dosis alta) +
broncodilatador de larga accin = reducen
las exacerbaciones y proporciona excelente
tx. a largo plazo
INTENSA
PERSISTENTE
Sntomas diarios
Exacerbaciones frecuentes
Sntomas frecuentes de asma nocturna
FEV1 o PEF <60% valor predicho
Variabilidad en el PEF o FEV1 >30%
Los agonistas B de corta accin, cuantas
veces sea necesario. El uso excesivo
indica un control inadecuado
Esteroide inhalado dosis media (u otro
antiinflamatorio a dosis alta),
broncodilatador de accin larga
MANUAL DE TRABAJO DEL CURSO ENARM CMN SIGLO XXI
CURSO ENARM CMN SIGLO XXI TEL: 36246001 Pharmed Solutions Institute PGINA 39

c.- Moderado persistente.
d.- Intenso persistente.

PREGUNTA
Considerando la gravedad del cuadro, cual es el tratamiento mas adecuado para este caso?

RESPUESTA
a.- Esteroide inhalado dosis media (u otro antiinflamatorio a dosis alta), broncodilatador de accin larga.
b.- Esteroide inhalado dosis baja (u otro antiinflamatorio a dosis alta) + broncodilatador de larga accin.
c.- Agonista B, de larga accin (salmeterol, formoterol) + antiinflamatorios en baja dosis.
d.- Baja dosis de antiinflamatorios.

CASO CLINICO
Paciente de 63 aos de edad. Diabetes mellitus tipo 2 diagnosticada hace un mes, en tratamiento. Exfumador. Sobrepeso (IMC:27).
Asma bronquial de diez aos de evolucin, en tratamiento combinado con un anticolinrgico de larga accin (bromuro de tiotropio), un
antagonista leucotrinico (Montelukast), corticoides orales e inhalados. Hernia hiatal. Estenosis artica leve degenerativa. Hiperplasia
benigna de prstata. Acude al Servicio de Urgencias por clnica de quince das de evolucin con fiebre de predominio vespertino,
malestar general, sudoracin profusa, astenia, prdida de peso, vmitos, dolor epigstrico y somnolencia creciente adems disuria,
polaquiuria, tenesmo o urgencia miccional. Destaca un episodio de odontalgia diez das antes en relacin con absceso peridental,
resuelto con antibioterapia oral (amoxicilina-clavulnico). EF: TA de 89/57 mm Hg, temperatura 39,3C. El paciente se mostraba
consciente, orientado globalmente, con deshidratacin cutneo-mucosa y palidez cutnea, aunque con mucosas normocoloreadas.
Destaca un pequeo hematoma en la cara interna del brazo izquierdo. Auscultacin cardaca era rtmica, FC (120 1pm), FR (24 rpm),
hipoventilando en ambas bases pulmonares con hepatoesplenomegalia. Ligeros edemas en tercio distal de ambas piernas.

PREGUNTA
Considerando el estado actual que grado de asma presenta el caso?

RESPUESTA
a.- Intermitente.
b.- Leve persistente.
c.- Moderado persistente.
d.- Intenso persistente.

PREGUNTA
Considerando la gravedad del cuadro, cual es el tratamiento mas adecuado para este caso?

RESPUESTA
a.- Esteroide inhalado dosis media (u otro antiinflamatorio a dosis alta), broncodilatador de accin larga.
b.- Esteroide inhalado dosis baja (u otro antiinflamatorio a dosis alta) + broncodilatador de larga accin.
c.- Agonista B, de larga accin (salmeterol, formoterol) + antiinflamatorios en baja dosis.
d.- Baja dosis de antiinflamatorios.

PREGUNTA
En caso de requerir tratamiento de rescate, cual es la opcin mas adecuada para el caso?

RESPUESTA
a.- Los agonistas B de corta accin, cuantas veces sea necesario.
b.- Los agonistas B de corta accin, cuantas veces sea necesario. El uso excesivo indica un control inadecuado.
c.- Los agonistas 2 de corta accin, y de larga accin cuantas veces sea necesario.
d.- Los agonistas 2 adrenrgicos, relajan el musculo liso (salbutamol) de corta accin, cuantas veces sea necesario.

CASO CLINICO
Varn de 40 aos, diagnosticado de asma bronquial desde nio. Desde hace un ao hace frecuentes visitas al servicio de urgencias del
hospital por disnea. Ha sido tratado con esteroides inhalados y orales, adrenrgicos beta-2 de corta y larga duracin, y antibiticos. En
su historia actual cabe destacar que desde unos das antes de su ingreso en el hospital presentaba tos, disnea y fiebre. A la EF, disneico,
trax hiperinsuflado y tiraje con disminucin de ruidos respiratorios y presencia de lesiones cutneas eritematosas y vesiculosas en pie
izquierdo, hombros y trax. Las radiografas de trax presentaron una condensacin en el lbulo superior derecho en dos focos y otra
en el lbulo superior izquierdo menos densa. Leucocitos 15.000/l (eosinfilos 31%, segmentados 44%). Segn la espirometra, sus
valores de flujos espiratorios mximos en el primer segundo oscilaron entre 800 y 2.650 ml, es decir, entre el 23 y el 71% del valor
predicho. El servicio de dermatologa inform una consulta sobre las lesiones cutneas como probablemente secundarias a alergia
inespecfica. Se trat con esteroides orales a dosis medias decrecientes, mejorando de su clnica respiratoria y desapareciendo las
lesiones cutneas y las alteraciones radiolgicas, elaborndose el diagnstico de probable SCS.

PREGUNTA
Considerando el estado actual que grado de asma presenta el caso?

MANUAL DE TRABAJO DEL CURSO ENARM CMN SIGLO XXI
CURSO ENARM CMN SIGLO XXI TEL: 36246001 Pharmed Solutions Institute PGINA 40

RESPUESTA
a.- Intermitente.
b.- Leve persistente.
c.- Moderado persistente.
d.- Intenso persistente.

PREGUNTA
Considerando la gravedad del cuadro, cual es el tratamiento mas adecuado para este caso?

RESPUESTA
a.- Esteroide inhalado dosis media (u otro antiinflamatorio a dosis alta), broncodilatador de accin larga.
b.- Esteroide inhalado dosis baja (u otro antiinflamatorio a dosis alta) + broncodilatador de larga accin.
c.- Agonista B, de larga accin (salmeterol, formoterol) + antiinflamatorios en baja dosis.
d.- Baja dosis de antiinflamatorios.

PREGUNTA
En caso de requerir tratamiento de rescate, cual es la opcin mas adecuada para el caso?

RESPUESTA
a.- Los agonistas B de corta accin, cuantas veces sea necesario.
b.- Los agonistas B de corta accin, cuantas veces sea necesario. El uso excesivo indica un control inadecuado.
c.- Los agonistas 2 de corta accin, y de larga accin cuantas veces sea necesario.
d.- Los agonistas 2 adrenrgicos, relajan el musculo liso (salbutamol) de corta accin, cuantas veces sea necesario.

PREGUNTA
El paciente presenta datos sugerentes del sndrome de Churg-Strauss (SCS), cual de las siguientes aseveraciones no es correcta para el
diagnostico de SCS?

RESPUESTA
a.- Sndrome hipereosinoflico de origen desconocido.
b.- Caracterizado por una vasculitis necrosante con granulomas extravasculares.
c.- Infiltrados eosinfilos y de polimorfonucleares en la pared de vasos de mediano y pequeo calibre.
d.- Es muy poco frecuente, afecta sobre todo a menores de 30 aos.

PREGUNTA
Clsicamente el SCS se presenta en 3 fases clnicas, cual no es una de ellas?

RESPUES
a.- Prodrmica (afeccin de vas areas altas y presencia de asma).
b.- Eosinoflica (eosinofilia perifrica e infiltrados pulmonares).
c.- Vascultica (afeccin sistmica).
d.- Necrotica (afeccion necrtica sistmica terminal).

PREGUNTA
Cual de los criterios para el diagnstico establecidos por el American College of Rheumatology (ACR) para SCS no son ciertos?:

RESPUESTA
a.- Asma.
b.- Eosinofilia perifrica mayor del 10%,
c.- Radiculopatias y Polineuritis.
d.- Infiltrados pulmonares transitorios.

NEUMONIA ADQUIRIDA EN LA COMUNIDAD (NAC)
CIENCIAS BASICAS: Definicin: Proceso infeccioso agudo del parnquima pulmonar adquirido fuera del entorno hospitalario, se
relaciona con sntomas y signos pleuropulmonares que acompaan casi siempre a infiltrados recientes observados en una
radiografa de trax. Factores de riesgo: Edad > 65 aos, EPOC, tabaquismo, alcoholismo, Enf. cardiovascular, DM, hepatopata e Insuf.
Renal, aspiracin, exposicin ambiental a aves, IVRS previas, Enf. Estructural pulmonar, Obstruccin de la va area. Etiologa:
Streptococcus pneumoniae 50-65% (>65 aos, alcoholicos, EPOC); Haemophilus influenzae 8-10% (EPOC, fumadores); Staphylococcus
aureus 3-5% (abseso pulmonar), otros Klebsiella, Escherichia, Moraxella catharralis, Mycoplasma pneumoniae, legionella pneumophila,
pesudomona auregunosa, anaerobios (aspiracin, obstruccin de va area).
SALUD PUBLICA: Es una causa de frecuente de morbi-mortalidad en la poblacin general. Incidencia de 2-10 casos/1000
habitantes/ao. 20-35% requieren ingreso hospitalario. Mortalidad del 5% de hospitalizados y de un 35% ingresados a la UCI. La
mortalidad oscila entre un mximo del 61% para las NAC debidas a Pseudomonas y un 35% para las producidas por enterobacterias,
Staphylococcus aureus y las de etiologa mixta. Las tasas de mortalidad por NAC la sitan en el quinto lugar como causa ms frecuente
de muerte en los pases industrializados, tras las enfermedades cardiovasculares, neoplsicas y cerebrovasculares y la enfermedad
pulmonar obstructiva crnica (EPOC). 60-70% de todos los casos de NAC son originados por Streptococcus pneumoniae. En los ltimos
MANUAL DE TRABAJO DEL CURSO ENARM CMN SIGLO XXI
CURSO ENARM CMN SIGLO XXI TEL: 36246001 Pharmed Solutions Institute PGINA 41

aos se ha informado la aparicin de Staphylococcus aureus resistente a la meticilina como causa de infecciones extrahospitalarias
graves. PATOGENIA: Vas de entrada: Aspiracin de organismos que colonizan la orofaringe (mecanismo ms comn, sobre todo en
ancianos, alcoholicos, residentes de silos, DM; mas propensos a infecciones por gram -, provenientes del estmago, comida
contaminada), inhalacin de aerosoles infecciosos, diseminacin hematgena (por el embolismo sptico a partir de focos de infeccin
distantes, adictos a drogas parenterales, endocarditis bacteriana, colonizacin de catteres intravenosos, S. aureus. patgeno ms
frecuente), inoculacin directa (es muy rara, el microorganismo traspasa las barreras). Hay factores que inhiben la actividad mucocoiliar
como: El humo del tabaco, el aire fro, frmacos (anestsicos), xido de azufre, xido de nitrgeno, fibrosis qustica. DIAGNOSTICO:
Cuadro clnico: Tos con o sin esputo, escalofros, fatiga, disnea y dolor torcico pleurtico, hemoptisis, mialgias, en caso de legionella;
puede existir sintomatologa gastrointestinal. Exploracin fsica: taquipnea, matidez a la percusin estertores y/o crepitancias, frmitos
y egofona. Rx de trax; neumona tpica (causada usualmente por Streptococcus pneumoniae, mas en nios y ancianos): consolidacin
lobar y en neumona atpica (influenzae, mycoplasma, legionella, no detectables en tincin de gram -, de ah su nombre, ms en
jvenes) infiltrados difusos: de no observar nada repetir Rx a las 24-48 hrs. Los estudios especiales dirigidos a un agente como
hemocultivo, , cultivos de esputo, tincin gram, Ag urinario para legionellla y Streptococcus solo cuando: neumona extensa que ingresa
a UCI, falta de respuesta al tx., infiltrados cavitarios, leucopenia, alcoholismo severo.
ESCALAS DE GRAVEDAD: tiles para no sobreestimar la intensidad de los cuadros neumnicos y menor tasa de hospitalizacin.

ESCALA DE RIESGO Y MORTALIDAD DE PNEUMONIA SEVERITY INDEX
Puntaje total Riesgo Clase Mortalidad Lugar de Tx.
S/factores de riesgo Bajo I 0.1 Domicilio
<70 Bajo II 0.6 Domicilio
71-90 Bajo III 2.8 Intrahospitalario (24h)
91-130 Moderado IV 8.2 Intrahospitalario
>130 Severo V 29.2 UCI

CRITERIOS DE ADMISION A UCI; CRITERIOS DE NEUMONIA GRAVE
Criterios menores
FR >30rpm o necesidad de apoyo ventilatorio
Indice de PaO2/FiO2 <250
Infiltrados multilobulares
Confusin o desorientacin
Uremia (BUN>20 mg/dl)
Leucopenia <4,000
Trombocitopenia < 100,000
Hipotermia <35
Hipotensin
Hipoglucemia en no diabticos
Alcoholismo
Hiponatremia
Acidosis metablica
Cirrosis
Asplenia
Criterios mayores
Ventilacin mecnica invasiva
Choque sptico que requiere vasopresores


TRATAMIENTO: En la mayora de los casos el tx. es emprico ,pero debe ser activo frente a los patgenos ms frecuentemente
implicados (S. pneumoniae). La duracin del tratamiento minimo 5 das despus de que el paciente permanezca por mas de 48-72 hrs
asintomticos. 7-10 dias en caso de Streptococcus; 10-14 para Mycoplasma. Grupo I; pacientes sin historia de, enfermedad
cardiopulmonar y sin factores de riesgo; 1ra eleccin macrolidos, 2da eleccin doxicilina. Grupo II; pacientes con enfermedad
cardiopulmonar y con factores de riesgo; 1ra eleccin fluroquinolonas (moxifloxacino, levofloxacino) 2da. Eleccin B-lactamico
(cefotaxima, ceftriaxona, ampi/sulbactam). Grupo III; pacientes no admitidos a UCI que tienen lo siguiente: A: con enf.cardiopulmonar y
otros factores modificables y B: sin enf.cardiopulmonar y sin otros factores modificables; 1ra. Eleccin fluroquinolona, 2da. eleccin b-
lactamico+ macrolido. Grado IV: pacientes que ameritan ingreso a UCI. A: sin riesgo para pseudomona 1ra eleccin cefotaxima,
ceftriaxona+ azitromicna o fluoroquinolona. B: con riesgo para pseudomona; 1ra. Eleccin b- lactamico, antineumococo y
antipseudomona (piperacilina/tazobactam, cefepime, imipenem)+ ciprofloxacino o levofloxacino. 2da eleccin piperacilina/tazobactam,
cefepime + aminoglucosidos y azitromicina . Antibiotico recomendado para neumona atpica (legionella, mycoplasma) es claritromicna.
PREVENCION: La primera medida es abandonar el habito tabquico. Vacuna de neumococo se recomienda en >65 aos, o en sujetos de
2-4 aos con factores de riesgo; revacunacin cada 5 aos. Vacuna inactiva contra influenza; >65 aos, 6-49 aos con factores de
riesgo.; revacunacin anual.

CASO CLINICO
Hombre de 68 aos de edad, con historia de consumo de licor cada fin de semana hasta la embriaguez, cuatro das de tos con
expectoracin purulenta, disnea de moderados esfuerzos, fiebre de 38,5C, dolor tipo pleurtico en el hemitrax derecho y
automedicacin con una dosis de dexametasona de 4 mg intramuscular dos das antes de la admisin. Ingres febril, taquicrdico,
taquipneico, hipo-xmico y en la radiografa de trax presentaba signos de derrame pleural y consolidacin en la base derecha. La
evaluacin de laboratorio muestra leucocitosis de 14.500 por mm3, neutrofilia de 96%, hiperglucemia de 638 mg/dl, hiperazoemia
(nitrgeno ureico en sangre de 27,45 mg/dl). El paciente fue hospitalizado con diagnstico de neumona grave extrahospitalaria IIIA,
segn las guas nacionales de manejo de esta enfermedad.

PREGUNTA
Cul es el agente etiolgico ms probable en este paciente?

RESPUESTA
a.- Mycoplasma pneumoniae
b.- Legionella pneumoniae
c.- Pseudomona
Pneumonia Severity Index
Caractersticas del paciente Puntaje
Caractersticas demogrficas
Sexo masculino
Sexo femenino
Asilo

Edad en aos
Edad en aos 10
+10
Comorbilidades
Enf. Neoplsica
Enf. Heptica
Enf. Cardiaca
Enf. cerebrovascular
Insuficiencia renal

+30
+20
+10
+10
+10
Exploracin fsica
Alteracin del estado mental
FR > 30rpm
P.sistlica <90mmHg
Temp. <35C o >40C
FC >125 lpm

+20
+20
+20
+15
+10
Estudios de laboratorio y Rx
pH arterial <7.35
BUN > 10.7 mmol/l
Na < 130 mEq/l
Glucosa >250 mg/dl
Hto. <30%
Derrame pleural

+30
+20
+20
+10
+10
+10
Cualquier
mayor o 3
menores
MANUAL DE TRABAJO DEL CURSO ENARM CMN SIGLO XXI
CURSO ENARM CMN SIGLO XXI TEL: 36246001 Pharmed Solutions Institute PGINA 42

d.- Strepococcus pneumoniae

PREGUNTA
Qu criterios de neumona grave tiene este paciente, para ser tratado intrahospitalariamente?

RESPUESTA
a.- leucocitosis, derrame pleural, tx con dexametasona
b.- Hiperazoemia, hiperglucemia, taquipnea
c.- Hiperazoemia, leucocitosis, consolidacin en base derecha
d.- Hiperglucemia, leucocitosis, tx. con dexametasona

CASO CLINICO
Hombre de 54 aos de edad, con historia de consumo de licor cada fin de semana hasta la embriaguez, cuatro das de tos con
expectoracin purulenta, disnea de moderados esfuerzos, fiebre de 38,5C, dolor tipo pleurtico en el hemitrax derecho y
automedicacin con una dosis de dexametasona de 4 mg intramuscular dos das antes de la admisin. Ingres febril, taquicrdico,
taquipneico, hipoxmico y en la radiografa de trax presentaba signos de derrame pleural y consolidacin en la base derecha.
Leucocitosis de 14.500 por mm3, neutrofilia de 96%, hiperglucemia de 638 mg/dl, hiperazoemia (nitrgeno ureico en sangre de 27,45
mg/dl). El paciente progres rpidamente a falla respiratoria hipoxmica, con choque sptico y, posteriormente, disfuncin orgnica
mltiple.

PREGUNTA
Considerando el cuadro clnico, a que grupo pertenece para considerar tratamiento antibitico?

RESPUESTA
a.- Grupo I
b.- Grupo II.
c.- Grupo III
d.- Grupo IV.

PREGUNTA
Considerando el Pneumonia Severity Index, que puntaje presenta el caso?

RESPUESTA
a.- <70
b.- 71-90
c.- 91-130
d.- >130

PREGUNTA
Considerando la escala de riesgo y mortalidad de pneumonia severity index que riesgo de motralidad presenta el caso?
RESPUESTA
a.- 0.6
b.- 2.8
c.- 8.2
d.- 29.2

NEUMONIA NOSOCOMIAL:
CIENCIAS BASICAS: Se desarrolla despus de una estancia intrahospitalaria de 48hrs, algunos requerirn intubarse y pueden desarrollar
neumona asociada al ventilador. SALUD PUBLICA: La neumona es la 2da. Infeccin nosocomial ms frecuente, con una mortalidad
asociada de 27-50%. Mas en nios, adultos >65aos o con comorbilidades graves asociadas, o sometidas a ciruga toracoabdominal.
Debidas generalmente a la aspiracin de secreciones orofarngeas o de tracto respiratorio superior, contaminacin de equipo de
terapia respiratoria. Las infecciones generalmente son polimicrobianas, con alto contenido de bacilos gram- ; pseudomona aeruginosa
(ms frecuente) escherichia coli, klebsiella pneumoniae y Acinetobacter (55-85%), originan las neumonas nosocomiales, los gram + ,
son responsables del 20-30%; rara vez por hongos y virus (inmunodeprimidos). Recientemente ha emergido infecciones por
Staphylococcus aureus, particularmente resistente a meticilina, ms frecuente en DM, y pacientes de UCI. DIAGNSTICO: La sospecha
dx. surge cuando el paciente presenta infiltrados radiogrficos nuevos y/o progresivos, en conjunto con dos o ms hallazgos clnicos
sugerentes de infeccin (fiebre de recin aparicin >38C, leucocitosis o leucopenia, esputo purulento). Rx de trax; desde infiltrados,
consolidacin, derrames, cavitaciones. Para dx. etolgico: cultivos, hemocultivos, identificacin de legionella requiere
inmunofluorescencia. Si hay derrame pleural de gran tamao y/o datos de sepsis, realizar toracocentesis dx. para descartar empiema o
derrame paraneumonico. TRATAMIENTO: Inicio emprico; 1. Esquema de espectro limitado (menos de 5 das hospitalizado y sin
factores de riesgo para bacterias multiresistente) Ceftriaxona, levofloxacino, ampi/sulbactam, ertapenem. 2. Esquema de espectro
amplio (>5 dias hospitalizado, factores de riesgo para multiresistentes). Esquema 1: a.- cefalosporina o carbapenem antiseudomonica
(cefepime, ceftazidima) (imipenem, meropenem) o un b lactamico + un inhibidor de b- lactamasas (piperacilina/tazobactam). b.-
fluroquinolona antiseudomonica (cipro o levofloxacino). Esquema 2: a.- aminoglucocido. b.- linezolid o vancomicina. La mejora clnica
se observa dentro de 48-72 hrs, posteriores al inicio del antibitico, por esta razn la terapia antibiotica, no debera de ser modificada
MANUAL DE TRABAJO DEL CURSO ENARM CMN SIGLO XXI
CURSO ENARM CMN SIGLO XXI TEL: 36246001 Pharmed Solutions Institute PGINA 43

hasta despus de 72 hrs. L a duracin deber individualizarse, para la neumona por bacterias gram - usualmente es de 14-21 das.
PREVENCION: Control estricto de infecciones, desinfeccin de manos con soluciones etlicas, vigilancia microbiolgica, monitoreo y
retiro temprano de dispositivos invasivos y evitar prcticas errneas en prescripcin de antibiticos, mtodos de ventilacin no
invasivos.

CASOS CLINICOS
Varn de 34 aos, trabajador en la hostelera, fumador y bebedor importante (con dependencia alcohlica). Fue trado a urgencias por
un traumatismo craneoenceflico con hematoma subdural. El paciente ingres en la unidad de cuidados intensivos (UCI) intubado y
conectado a ventilacin mecnica, y recibi tratamiento antibitico emprico con amoxicilina-cido clavulnico. La cifra de leucocitos en
sangre era de 12,05 103/l. En el primer da del ingreso se realiz un broncoaspirado (BAS). En el cultivo creci una flora mixta
respiratoria. A los 8 das del ingreso present fiebre y secreciones purulentas espesas, y en la radiografa de trax se objetiv una
neumona en el lbulo inferior izquierdo complicada con atelectasias bibasales. La concentracin de leucocitos en sangre era en ese
momento de 4,10 103/l.

PREGUNTA
Cual es el agente etiolgico mas probable para este caso?

RESPUESTA
a.- Pseudomona aeruginosa.
b.- Escherichia coli.
c.- klebsiella pneumoniae.
d.- Acinetobacter.

PREGUNTA
Cual es el tratamiento de primera intensin en este caso?

RESPUESTA
a.- Ceftriaxona, levofloxacino.
b.- Cefepime, ceftazidima.
c.- Imipenem, meropenem.
d.- Piperacilina/tazobactam

NEUMOCONIOSIS
CIENCIAS BASICAS: Definicin: Enfermedad producida por acumulacin de polvos minerales secundaria a la inhalacin crnica de los
mismos. Los materiales mas implicados son: asbesto, slice, carbn, berilio, hierro, el dao tisular que provocan depende de; tamao,
tiempo e intensidad de exposicin, estado inmunolgico, relacin inhalacin/ aclaramiento ciliar pulmonar. ASBESTOSIS: Hay 3 tipos de
asbesto: amosita, asbesto azul (cocidolite) y asbesto blanco (crisolito, el 90%del consumo de asbesto es bajo esta forma, es mas soluble
y tiende a fragmentarse). Las formas en las que se inhalan las fibras de asbesto es en anfibole y serpentinas. PATOGENIA: Exposicin
directa, cuando se trabaja con materiales o en lugares donde hay asbestos. Exposicin indirecta, cuando se vive o trabajo cercano a un
lugar de concentracin de asbesto. Lo podemos encontrar en materiales para pulir piedra preciosa, afiladores de piedra, resistencias de
algunos electrodomsticos y algunos plsticos. La enfermedad se puede presentar como: placas pleurales benignas 40%, enf., maligna
20% (mesotelioma maligno), asbestosis como tl 40%. Una vez inhalada las partculas, llegan a los espacios alveolares y son fagocitados
por lo macrfagos, si son menor a 3 m, si son mayores son fagocitadas incompletamente ( cuerpos de asbesto)e, ya fagocitadas son
trasportadas a los ganglios y despus a las pleuras, donde se depositan para formar las placas pleurales, los cuerpos de asbesto se
acumulan de manera progresiva en espacios areos e intersticio, las clulas afectadas liberan citocinas, fibronectina y colgena,
causando as migracin de otras clulas de defensa y favoreciendo proliferacin de fibroblastos, provocando zonas de regeneracin
parenquimatosa, provocando la asbestosis, es decir la fibrosis pulmonar afecta mas zonas inferiores y regiones subpleurales.
DIAGNSTICO: Relacin clara entre exposicin de asbesto y fibrosis pulmonar. El consumo de tabaco aumenta el riesgo de asbestosis.
Cuadro clnico: lo primero es disnea, la cual se va haciendo progresiva, despus tos seca pertinaz, pueden auscultarse estertores
crepitantes basales, a veces hipocratismo. Rx.: opacidades irregulares que al principio de la enfermedad son de localizacin perifrica y
basal, hiperclaridad alrededor de silueta cardiaca. TAC: se puede ver fibrosis, como vidrio despulido, hasta lesiones fibroticas con
engrosamiento de los septos interlobares, engrosamiento de pleura viscerales. El patrn funcional observado en pacientes con
asbestosis es restrictivo. Se puede realizar lavado bronquioalveolar. SILICOSIS: Provocada por inhalacin de cristales de slice, en la
mayora en sus formas de cuarzo, las actividades de riesgo son la construccin, la fundicin, demolicin o reparacin de estructuras de
concreto, taladrar piedras. PATOGENIA: Hay silicosis aguda, donde se observa proteinosis y la crnica la mas frecuente, donde se
observan ndulos con centro fibrotico, rodeado de parnquima con cmulos de slice, que ha sido fagocitado por macrfagos. En la
forma complicada llamada fibrosis masiva progresiva, se observan ndulos confluentes, que se localizan principalmente en lbulos
superiores, que se relacionan con una pobre calidad de vida. La silicosis se relaciona con una mayor susceptibilidad a Tb, artritis
reumatoide, esclerodermia, lupus y enf., renal progresiva. DIAGNSTICO: Cuadro clnico: sntoma principal disnea, la cual es progresiva,
raros otros sntomas. Rx: pequeas opacidades redondeadas, con distribucin casi siempre bilateral, de localizacin en lbulos
superiores, puede haber crecimiento de adenopatas hiliares, las cuales pueden calcificarse en una forma caracterstica conocida como
"cascara de huevo". Funcionalmente la silicosis se comporta con limitacin al flujo areo y con reduccin de la capacidad de difusin.
ANTRACOSIS: Provocada por inhalacin crnica de carbn masiva, la forma mas comn es el antracite. Existen 2 formas principales: la
mcula o ndulo antracotico y fibrosis pulmonar masiva. PATOGENIA: En la antracosis hay mayor susceptibilidad a presentar artritis
reumatoide, y cuando se presentan ambas condiciones, se conoce como Sx., de Caplan, en el cual debe haber un ndulo reumatoideo,
que en algunas condiciones puede cavitarse debido a la necrosis central o bien calcificarse. DIAGNSTICO: La macula o mancha de
MANUAL DE TRABAJO DEL CURSO ENARM CMN SIGLO XXI
CURSO ENARM CMN SIGLO XXI TEL: 36246001 Pharmed Solutions Institute PGINA 44

carbn suele ser una lesin de 5cm que puede observarse principalmente en regiones apicales, en ocasiones rodeadas de parnquima
que muestra enfisema centroacinar. Existen tantas neumoconiosis como diversidad de polvos minerales y son una causa importante de
morbimortalidad a nivel mundial.

CASOS CLINICOS
70 aos, minero de minas carbn jubilado, fumador de 30 cigarrillos/da hasta hace 10 aos, bebedor de l de vino al da y 2 cervezas.
HTA en tratamiento con diurticos y dieta hiposalina. Hipercolesterolemia en tratamiento con dieta. Desde hace 25 aos presenta tos y
expectoracin crnica de predominio matutino. Desde hace 15 aos se asocia disnea de esfuerzo progresiva con aumento de la disnea
en relacin con las infecciones respiratorias. Presenta con frecuencia expectoracin de color negro. Paciente que acude al servicio de
urgencias por un cuadro de aumento de su tos habitual, aumento de su expectoracin habitual, siendo actualmente mucopurulenta y
aumento de la disnea que se hizo de reposo. Desde hace una semana nota que se le hinchan los tobillos. Paciente consciente,
orientado. Ciantico. Taquipneico (30 respiraciones por minuto). Hepatomegalia blanda y no dolorosa de 2 cm. Ingurgitacin yugular
+++. Edemas maleolares ++++. Tonos cardiacos rtmicos a 130 por minuto. Roncus y sibilantes diseminados por ambos campos
pulmonares.TA: 160/100. Hb 16, Hto 49, leucocitos 13500 con aumento de neutrfilos en la formula leucocitaria. P02 52 PC02 40, PH
7.35. RX de torax: Signos de atrapamientoretroesternal y retrocardiaco, engrosamientos peribronquiales. 3 imgenes de aumento de
densidad nodulares de 3-7 cmde dimetro en Lbulo superior derecho y 2 en Lbulo superior izquierdo.

PREGUNTA
Se realizaron pruebas funcionales respiratorias con los siguientes resultados FVC 60%, FEV1 30%, cual es la FEV1-FVC?

RESPUESTA
a.- 40
b.- 50
c.- 60
d.- 30

HIPERTENSION PULMONAR PRIMARIA (HAP)
CIENCIAS BASICAS: Definicin: Entidad que afecta a la circulacin
pulmonar a nivel de las pequeas arterias y arteriolar, caracterizada por
una proliferacin fibromuscular y remodelacin vascular endotelial, que
da lugar a estenosis de la luz vascular. Hemodinmicamente se define
como una presin arterial pulmonar media > 25mmHg en reposo o de 30
mmHg durante el ejercicio, con la adicin de una presin capilar
pulmonar < 15mmHg, con RVP elevada. SALUD PUBLICA: Incidencia
anual es de 1-2 casos por 1000 000, con edad media de diagnstico de
36 aos (4ta-5ta dcadas de la vida). La HAP primaria (idioptica) 30-
40%; predomina en la mujer. Los casos de HAP familiar representan el
10%. La HAP asociada a fenmeno de Eisenmenger (cortocircuito der-
izq), representa 30-35%. La hipertensin es la causa ms comn de cor pulmonale. PATOGENIA: Se han demostrado 3 factores
trascendentales que incrementan las resistencias vasculares pulmonares: a) vasoconstriccin sostenida, b) proliferacin vascular
pulmonar, c) trombosis in situ, condicionando as una arteriolopata obstructiva, dificultando flujo sanguneo, lo que eleva la presin en
las arterias pulmonares, generando demasiada tensin en el VD, hipertrofindolo y llevndolo a insuficiencia. La disfuncin endotelial
juega un rol pivote y trascendental, sobre todo por el imbalance entre las sustancias vasodilatadoras (xido ntrico, prostaciclina, PG- 1-
2, pptido intestinal vasoactivo) y vasoconstrictoras (Tx A-2, endotelina-1, serotonina, factor de crecimiento derivado de cel.
endoteliales). En HAP familiar, se encuentran alteraciones en los receptores que codifican para los factores transformadores de
crecimiento tipo beta ((BMPR-II), que regulan la angiognesis y apoptosis celular. DIAGNSTICO: Cuadro clnico: El sntoma ms
frecuente y ms temprano es la disnea progresiva 60%, acompaada de fatiga, presncope o sncope, edema de miembros inferiores,
cianosis %20, angina, as como dolor precordial secundario a isquemia del VD. Puede haber ingurgitacin yugular, por contractilidad
aumentada de la de la AD, por falla ventricular derecha crnica. Auscultacin: segundo ruido, en su componente pulmonar desdoblado
o acentuado, datos de regurgitacin tricspidea, as como la presencia de galope sobre el precordio correspondiente al VD. El
diagnostico de certezas solo puede establecerse, segn la definicin mediante el cateterismo derecho. ECG, tele de trax,
ecocardiografa transtoracica (valorar afeccin valvular o miocrdica como pb. causa, as como cortocircuitos izq-der). Pruebas de
funcin respiratoria para determinar neumopatas. El gamagrama pulmonar ventilatorio-perfusorio es estudio clave para detectar HAP
tromboembolica crnica, confirmacin dx. con angiografa pulmonar. TRATAMIENTO: Si est limitada la actividad fsica usar diurticos,
para edema perifrico, O2 suplementario si esta reducida la PO2 y anticoagulacin crnica con warfarina (objetivo INR= 2.0-3.0). Como
los vasoconstrictores juegan un papel importante, el tx. con vasodilatadores se basa en tales procesos, pero no se puede predecir quien
responder sin resistencia a los vasodilatadores orales como a los antagonistas de canales de calcio, ya que estos mejoran la sobrevida.
Mediante una prueba se puede predecir que pacientes tendrn respuesta sostenida a dichos agentes, se define una prueba positiva,
cuando al administrar vasodilatadores endovenosos de accin corta como adenosina o epoprostenol, existe una disminucin de
10mmHg o mayor en la presin arterial pulmonar media o como un descenso en ms de 25% de las resistencias vasculares pulmonares
sin presentar variacin en la presin arterial sistmica, solo responden 7-10% a dicho reto, habr que monitorizar hipotensin o
empeoramiento de la insf. Cardiaca der. Otros frmacos son los prostanoides, son sustancias tanto vasodilatadoras como
antiproliferativas, usados por varias vas, epoprostenol endovenoso, iloprost inhalado, treprostinil subcutneo, el mecanismo de accin
radica en estimular la produccin de AMP cclico como segundo mensajero induciendo vasodilatacin. En pacientes con hipertensin
arterial pulmonar existe un dficit relativo de xido ntrico, por lo que los inhibidores de fosfodiesterasas tipo 5, al prolongar la vida
media de los segundos mensajeros GMPc, prolongan el efecto vasodilatador y antiproliferativo del xido ntrico, como es el caso del
CAUSAS DE HIPERTENSION PULMONAR
Hipertensin arterial pulmonar: Hipertensin pulmonar primaria (idiopatica
o familiar). Enfermedades vasculares de la colgena (esclerodermia, artritis,
lupus). Cortocircuitos congnitos sistmico-pulmonares (CIV, CIA, PCA).
Hipertensin portal. Infeccin por VIH. Frmacos anorexigenicos
(fenfluramina, fentermina). Otros: Enf. De depsito de glucgeno, de
Gaucher, hemoglobinopatas,
Hipertensin venosa pulmonar: EPOC. Enfermedad pulmonar intersticial
difusa. Apnea del sueo. Enfermedad con hipotensin alveolar crnica
Tromboembolia pulmonar: Embolismo pulmonar agudo. Embolismo
pulmonar crnico.

MANUAL DE TRABAJO DEL CURSO ENARM CMN SIGLO XXI
CURSO ENARM CMN SIGLO XXI TEL: 36246001 Pharmed Solutions Institute PGINA 45

sildenafil, el cual es de 1ra lnea en el tx. por va oral de la hipertensin arterial pulmonar. Si persiste la Insf. Cardiaca derecha, debe
considerarse el trasplante pulmonar bilateral. PRONSTICO: Deletreo y muy pobre. Sobrevida de 2.5 aos a partir del dx. sin
tratamiento. Supervivencia a 5 aos de 75-85% en pacientes con tx. Adecuado y adyuvante con combinacin o sinergismo de frmacos
con diferentes mecanismos de accin. Las variables ms importantes para establecer pronstico son presin media de la arteria
pulmonar, presin de AD y GC.

CASO CLINICO
Paciente varn de 58 aos de edad, ex fumador (con una dosis acumulada de 20 paquetes-ao) y ex enolismo moderado, y con
antecedentes personales de hipertensin arterial sistmica, linfangiectasias duodenales, epistaxis de repeticin y cirrosis heptica
enlica ChildA (MELD13) con un nico episodio de hemorragia digestiva alta por sangrado de varices esofgicas. Tiene 2 hijas, una de
las cuales refera episodios de epistaxis de repeticin. Ingres por clnica de infeccin respiratoria e insuficiencia respiratoria aguda con
una saturacin de O2 de 77% (FIO20,21). En la radiografa de trax se observ un infiltrado en la base derecha que se interpret como
una probable neumona adquirida en la comunidad y se inici tratamiento antibitico con fluoroquinolonas (levofloxacino).

PREGUNTA
Ante la persistencia de hipoxemia severa refractaria cual es la conducta mas adecuada?

RESPUESTA
a.- Cambiar esquema antibitico.
b.- Realizar TAC.
c.- Realizar ecocardiograma doppler.
d.- Realizar gamagrafia.

CASO CLINICO
Mujer de 40 aos, con xeroftalmia de 5 aos de evolucin a la que se haban aadido en los ltimos 6 meses xerostoma y artralgias, sin
otras manifestaciones extraglandulares. Las pruebas complementarias nos permitieron hacer el diagnstico de SS: ANA positivo, anti-
SSa positivo, anti-SSb positivo, test de Shirmer anormal, datos de inflamacin crnica en biopsia de glndula salival menor y
gammagrafa de glndulas salivales compatible. Se realiz ecocardiograma de rutina donde apareci por primera vez datos de HTP
ligera (32 mm Hg) que se confirm con cateterismo derecho. La paciente present disnea de esfuerzo. En ecocardiografa se estim una
HTP de 46 mm Hg. Mediante TC torcica se descart la existencia de fibrosis pulmonar y eventos tromboemblicos. Actualmente
persisten artralgias que ceden con analgsicos habituales y su disnea est estable.

PREGUNTA
Cual de las siguientes afirmaciones es la incorrecta, respecto a la HTP del caso?

RESPUESTA
a.- En asociacin con enfermedades del tejido conectivo CREST.
b.- En asociacin con LES
c.- Enfermedad mixta del tejido conectivo.
d.- La asociacin con SS es comn.

CASO CLINICO
Un hombre de 33 aos fue admitido, con disnea reciente y progresiva, edema y prdida de peso de aproximadamente 10 kg. Dos
hermanas a los 11 y 14 aos de edad. Al EF, pareca "dbil" (subnutrido) (peso 60 Kg; altura 1,82 m), presentaba hepatomegalia leve y
edema de miembros inferiores. Un sonido alto de cierre de la vlvula pulmonar era odo en el precordio. El examen de los pulmones
revel ronquidos leves y estertores finos en ambos lados, que desaparecieron completamente despus de la administracin de
diurticos. Los datos de la cateterizacin cardaca derecha fueron compatibles con hipertensin arterial pulmonar (HAP). xido ntrico
inhalado y sildenafila fueron eficaces en la reduccin de la resistencia vascular pulmonar. Una acentuada elevacin en la presin de
enmentoencuamiento pulmonar fue registrada durante la inhalacin del xido ntrico.

PREGUNTA
Cual es la conducta adecuada para establecer un diagnostico definitivo?

RESPUESTA
a.- Gamagrama pulmonar.
b.- Ecodoppler pulmonar.
c.- Biopsia pulmonar abierta.
d.- Tomografia axial computada.

CASO CLINICO
Mujer de 20 aos, de profesin camarera, que consult por cuadro de disnea, edema en el miembro superior derecho y sncope. Tena
antecedentes de ser fumadora activa de 15 cigarrillos/da y padecer de rinoconjuntivitis y asma bronquial. La paciente comenz un mes
previo con disnea a moderados esfuerzos. Cuando consult en urgencias, presentaba disnea de reposo, debilidad, astenia y refera
haber tenido un sncope. A la exploracin presentaba PA de 120/80mmHg, FC de 110 lat/min y SatO2 del 95% sin O2 suplementario. Se
apreciaba aumento del permetro del miembro superior derecho, doloroso a la palpacin y sin signos de empastamiento. La analtica
MANUAL DE TRABAJO DEL CURSO ENARM CMN SIGLO XXI
CURSO ENARM CMN SIGLO XXI TEL: 36246001 Pharmed Solutions Institute PGINA 46

presentaba perfiles bioqumico, heptico, renal, hemograma y coagulacin dentro de la normalidad, excepto los dmeros D (2.500
ng/ml).

PREGUNTA
Cual es la conducta a seguir mas adecuada, para confirmar el diagnostico para el caso actual?

RESPUESTA
a.- Realizar rx de torax.
b.- Realizar TAC de torax.
c.- Realizar Angio-TAC de torax.
d.- Realizar IRM de torax.

COR PULMONALE
Enfermedad cardaca pulmonar, y se define como una dilatacin y/o hipertrofia del ventrculo derecho a consecuencia de problemas en
la vasculatura o el parnquima pulmonar y puede conducir a insuficiencia cardaca derecha. El trmino se usa para describir cambios en
la estructura y funcin del ventrculo derecho como resultado de un desorden respiratorio que produzca hipertensin pulmonar. La
hipertrofia del ventrculo derecho es el cambio principal en el cor pulmonale crnico y dilatacin del ventrculo en los casos agudos,
ambos consecuencia de un incremento de presin en el ventrculo derecho del corazn. Sin tratamiento, el cor pulmonale puede causar
insuficiencia cardaca derecha y muerte. ETIOLOGA: Para que una alteracin del corazn derecho sea clasificado como cor pulmonale,
el origen de la anomala debe encontrarse bien en el sistema de circulacin pulmonar o bien en el parnquima pulmonar,
producindose en cualquier caso una hipertensin arterial pulmonar (HTAP). Las causas principales de HTAP son: enfermedades que
producen la oclusin de la red vascular pulmonar: tromboembolismo pulmonar recurrente, hipertensin pulmonar primaria,
enfemedad veno-oclusiva, enfermedad del colgeno vascular o enfermedades pulmonares inducidas por drogas; enfermedades que
producen vasoconstriccin pulmonar hipxica crnica, como ocurre en la bronquitis crnica, la enfermedad pulmonar obstructiva
crnica (EPOC), la fibrosis qustica, la hipoventilacin crnica (que tiene lugar en la obesidad, las enfermedades neuromusculares como
la distrofia muscular de Duchenne o la disfuncin de la pared torcica), o en las personas que viven en altitud. Enfermedades que
producen alteraciones del parnquima pulmonar, como ocurre en la bronquitis crnica, la EPOC, la bronquiectasia, la fibrosis qustica,
las neumoconiosis, la sarcoidosis o la fibrosis pulmonar idioptica. El compromiso del ventrculo derecho causado por un defecto
sistmico, diferente al sistema respiratorio no es considerado cor pulmonale, debe ser de origen pulmonar subyacente. La prevalencia
del cor pulmonale es difcil de definir, porque no todos los casos de enfermedad pulmonar desarrollan cor pulmonale, la capacidad de
diagnosticar la HTAP mediante el examen fsico de rutina es baja y los tests de laboratorio son relativamente poco sensibles. Sin
embargo, avances recientes en imaginera con ecografa Doppler y biomarcadores hace ms fcil el diagnstico del cor pulmonale.
Segn la agudeza y la severidad del estmulo que genera la disfuncin cardaca, se puede distinguir: Cor pulmonale agudo: ocurre tras
un estmulo repentino y severo, que produce dilatacin y fallo del ventrculo derecho, pero no hipertrofia; puede ser producido por:
Embolismo pulmonar masivo; Maligno: carcinomatosis miliar, linfangitis carcinomatosa. Empeoramiento del cor pulmonale crnico. Cor
pulmonale crnico: se produce a causa de un aumento progresivo y lento de la HTAP, que conduce a la hipertrofia y dilatacin del
ventrculo derecho; puede generarse a causa de: Enfermedad obstructiva crnica (EPOC), como bronquitis crnica y enfisema, en la que
la alteracin de la estructura alveolar y la consecuente insuficiencia respiratoria representa el inicio del cor pulmonale. Prdida de
tejido pulmonar por razones traumticas o quirrgicas. Pneumocistitis terminal. Escoliosis, especialmente con desviacin severa de la
columna lo cual interfiere con el desarrollo normal de los pulmones. Dicha restriccin respiratoria causa hipoxemia, hipercapnia con
subsecuente vasoconstriccin pulmonar e hipertrofia del ventrculo derecho. Sarcoidosis por fibrosis del parenquima pulmonar en una
pequea porcin de los pacientes (aprx. 5%). FISIOPATOLOGA: Como se indica en el apartado "Etiologa", existen muchas patologas
diferentes que pueden producir cor pulmonale, pero todas ellas tienen en comn la generacin de hipertensin arterial pulmonar
(HTAP), suficiente para generar la dilatacin del ventrculo derecho, con o sin hipertrofia asociada. Anatmicamente, el ventrculo
derecho tiene una pared delgada, con una compliancia elevada, por lo que est mejor preparado para adaptarse a variaciones de
volumen que a variaciones de presin. Por ello, cuando se produce un aumento de la presin pulmonar (HTAP) y un aumento de la
resistencia vascular pulmonar, el ventrculo derecho no puede desarrollar la fuerza suficiente como para superar dicho aumento, por lo
cual se dilata (en los casos agudos) o se hipertrofia (en los casos crnicos), pudiendo llegar a fallar. La severidad de la HTAP y el fallo del
ventrculo derecho estn influidos por mltiples factores que pueden producirse de manera intermitente, como hipoxia secundaria a
alteraciones en el intercambio gaseoso, hipercapnia y acidosis, adems de cambios en la sobrecarga de volumen del ventrculo derecho
que se producen con el ejercicio, con el aumento del ritmo cardaco, la policitemia o la retencin de sales debido a una disminucin del
gasto cardaco. MANIFESTACIONES CLNICAS: Sntomas: Generalmente estn asociados al problema pulmonar de fondo. El sntoma
ms comn es disnea (dificultad para respirar), debido a que hay un aumento del trabajo necesario para respirar, a consecuencia de los
cambios en la elasticidad del pulmn (como ocurre en la fibrosis) o a modificaciones en la mecnica respiratoria (como sucede en la
EPOC), y ambos pueden verse agravados por un incremento de la hipoxemia. La hipoxemia pulmonar puede generarse por una
disminucin de la permeabilidad de los capilares, por desequilibrios ventilacin-perfusin o por la presencia de un shunt cardaco o
pulmonar. La presencia de ortopnea o disnea paroxstica nocturna son raramente sntomas aislados de fallo cardaco derecho. Pero si
se presentan, indican que hay un aumento del trabajo respiratorio en el decubito. Tambin puede presentarse tos o sncope en
pacientes con cor pulmonale con HTAP severa, debido a la incapacidad del corazn derecho de bombear sangre a travs de los
pulmones hacia el corazn izquierdo. Tambin puede presentarse dolor abdominal y ascitis, as como edema de las extremidades
inferiores. Signos: Muchos de los signos que se encuentran en los pacientes con cor pulmonale tambin se encuentran en pacientes con
una fraccin de eyeccin reducida: taquipnea, presin venosa yugular elevada, hepatomegalia y edema en las extremidades inferiores.
Adems, el ventrculo derecho puede palparse a lo largo del borde izquierdo del esternn o en el epigastrio. En el cor pulmonale, la
aparicin de cianosis es un evento tardo, secundario a una reduccin del gasto cardaco con vasoconstriccin sistmica y desequilibrio
ventilacin-perfusin en el pulmn. EDEMA DE PULMN AGUDO: La causa ms comn del edema pulmonar agudo es el aumento de la
presin capilar (desequilibrio de las fuerzas de Starling) en la microvasculatura pulmonar, causando fuga plasmtica hacia el espacio
MANUAL DE TRABAJO DEL CURSO ENARM CMN SIGLO XXI
CURSO ENARM CMN SIGLO XXI TEL: 36246001 Pharmed Solutions Institute PGINA 47

intersticial. Cualquier evento que curse con hipertensin venosa pulmonar ocasionar una congestin venosa y un aumentado volumen
de sobrecarga al ventrculo derecho. La incapacidad del ventrculo derecho de expandir para adaptarse a ese incrementado volumen
aumenta an ms el volumen venoso, y por ende retrgrada, la presin capilar. La fuga de plasma por una mayor presin hidrosttica
en el capilar pulmonar que en el espacio intersticial acumula lquido en el parnquima pulmonar. La suma de estas anormalidades en el
pulmn y su circulacin vascular es la complicacin ms importante producida por el edema pulmonar en la instalacin del cor
pulmonale. COMPLICACIONES: El cor pulmonale retrasa el flujo de la sangre desde la circulacin venosa hacia la arterial. En la
hiperemia, la sangre se acumula en el sistema venoso, incluyendo la vena heptica. La congestin prolongada de sangre en la regin
centro-lobulillar del hgado conlleva a hipoxia y cambios grasos de los hepatocitos perifricos, produciendo agrandamiento voluminoso
del hgado, llegando a tener hasta el doble del volumen de sangre. El aspecto ciantico combinado con islotes grasos amarillentos le da
la apariencia al hgado de nuez moscada. Primero el hgado se congestiona, aumenta de tamao, se vuelve ciantico y los sinusoides, en
especial en la zona centrolobulillar, se repletan de sangre. Comienza tambin una infiltracin grasosa que termina por darle el tinte
amarillento final. De persistir la deficiencia de la circulacin pulmonar, se instala en las trabculas del hgado una atrofia que suele ser
ciantica. Con el tiempo se agrega una acumulacin fibrtica (cicatrizacin) que tiende a reducir al hgado de tamao y a endurecerlo.
Un proceso patolgico similar ocurre en los riones y el bazo: congestin sangunea, atrofia ciantica y endurecimiento. En los
pulmones, la hiperemia produce ingurgitacin capilar, edema en el intersticio pulmonar y, como consecuencia, fibrosis entre los
alvolos y la matriz capilar, traducindose en trastornos de la relacin perfusin y ventilacin pulmonar. El pulmn adopta una
consistencia dura y (macroscpicamente) de color "rojo morena". El cor pulmonale puede tambin llevar a insuficiencia cardaca
congestiva (ICC), empeorando la respiracin por razn del edema pulmonar, hinchazn de las piernas debido a edema en la periferia y
hepatomegalia congestiva y dolorosa. La ICC es un indicador negativo en el pronstico del cor pulmonale. DIAGNSTICO: La causa
principal de fallo cardaco derecho no es la disfuncin de la vasculatura o el parnquima pulmonar, sino el fallo cardaco izquierdo. Por
ello es importante evaluar la posible disfuncin del ventrculo izquierdo en un paciente con sntomas de fallo cardiaco derecho. Varios
exmenes son de utilidad para el diagnstico del cor pulmonale: Electrocardiograma (ECG): los pacientes con HTAP severa muestran
una onda P pulmonar, una desviacin del eje derecho e hipertrofia del ventrculo derecho; Ecocardiograma: til para determinar el
grosor del ventrculo derecho y las dimensiones de las cmaras cardacas, as como la anatoma de las vlvulas; Ecografa Doppler, para
verificar la presin arterial pulmonar; Radiografa de trax, que puede mostrar un agrandamiento de la arteria pulmonar principal, los
vasos del hilio pulmonar y la arteria pulmonar descendente derecha; TAC del trax, til en el diagnstico de tromboembolia pulmonar
aguda; adems, es el mtodo ms fiable para diagnosticar enfisema y enfermedad intersticial pulmonar; Pruebas de funcin pulmonar;
Gases sanguneos; Cateterizacin de Swan-Ganz: el cateterismo del ventrculo derecho permite confirmar el diagnstico de HTAP y
excluir la presin elevada del ventrculo izquierdo como causa del fallo del ventrculo derecho. TRATAMIENTO: La eliminacin de la
causa es la intervencin ms importante. En el embolismo pulmonar, se apela a trombolisis (disolucin enzimtica del cogulo
sanguneo), en particular si hay trastornos del ventrculo derecho. En la EPOC, la terapia con oxgeno a larga duracin puede mejorar el
cor pulmonale. Los principios generales de tratamiento del cor pulmonale incluyen la reduccin del trabajo respiratorio, mediante
ventilacin mecnica no invasiva, broncodilatadores y esteroides, adems de tratar las infecciones subyacentes. Una oxigenacin
adecuada (saturacin de oxgeno 90-92%) tambin permite reducir la resistencia vascular pulmonar y reducir la demanda sobre el
ventrculo derecho. Los pacientes anmicos necesitan una transfusin y si el hematocrito excede 65% debe efectuarse una flebotoma
para reducir la HTAP. El edema requiere diurticos (reduciendo el sobreuso del corazn), a veces nitratos para mejorar el flujo
sanguneo, inhibidores de la fosfodiesterasa como sildenafil y tadalafil, y ocasionalmente frmacos ionotropos para mejorar la
contractilidad del corazn. Otros medicamentos ayudan a mejorar la hipertensin pulmonar, tales como la prostaciclina o sus anlogos.
Asimismo se han prescrito anticoagulantes y bloqueadores de los canales de calcio para las etapas iniciales del trastorno.

CASO CLINICO
Mujer de 48 aos, exfumadora de 10 paquetes/ao, diagnosticada de sndrome de CREST e hipertensin pulmonar secundaria en
seguimiento en consulta de Neumologa desde hace seis aos. En la actualidad, se encuentra en grado funcional III de la NYHA y est en
tratamiento con acenocumarol, sildenafilo, treprostinil subcutneo y oxigenoterapia domiciliaria. Desde hace dos meses, presenta
disnea de mnimos esfuerzos (grado funcional IV de la NYHA) y edemas en miembros inferiores, por lo que se aadi furosemida y
espironolactona al esquema teraputico. Hace una semana, comenz con disnea de reposo y aumento de los edemas, por lo que se
decidi ingreso hospitalario. A la exploracin estaba afebril, taquicrdica y taquipneica. A la auscultacin cardiaca presentaba un soplo
sistlico en foco pulmonar con refuerzo del segundo tono y en la auscultacin pulmonar, crepitantes finos bibasales. En la exploracin
del abdomen se palpaba ascitis. En miembros inferiores presentaba edemas con fvea hasta rodillas. Se realiz analtica sin alteraciones
significativas y radiografa de trax con aumento del ndice cardiotorcico e infiltrado intersticial bilateral. En el ecocardiograma se
apreciaba dilatacin severa de cavidades derechas con regurgitacin tricuspdea severa y PAPs de 59mmHg.

PREGUNTA
Cual de las siguientes aseveraciones no es correcta en el manejo de esta patologia?

RESPUESTA
a.- El tratamiento de la hipertensin pulmonar se basa en tres pilares fundamentales: medidas generales, anticoagulacin y tratamiento
vasodilatador.
b.- Entre las medidas generales estn evitar el ejercicio fsico que provoque sntomas, las grandes altitudes, las situaciones de estrs
agudo y el embarazo.
c.- La anticoagulacin hay que iniciarla siempre que no exista contraindicacin por el aumento del riesgo de trombosis derivado del
estado de hipercoagulabilidad de la hipertensin pulmonar.
d.- En todos los casos deber realizar vacunacin antigripal, antineumococica y mantener profilaxias antibitica.



MANUAL DE TRABAJO DEL CURSO ENARM CMN SIGLO XXI
CURSO ENARM CMN SIGLO XXI TEL: 36246001 Pharmed Solutions Institute PGINA 48

CASO CLINICO
Un hombre, albail de 50 aos de edad y agricultor fue admitido por falta de aire y tos, de 4 aos de duracin, y edema bilateral
inferior durante 2 meses. En el EF dificultad respiratoria, una frecuencia respiratoria de 50 rpm, pulso de 120 latidos por minuto,
cianosis, hipocratismo digital, pedal edema con fvea bilateral, a la izquierda paraesternal tirn, p2 voz alta, grado 3 pansistlico
murmullo, mxima abajo a la izquierda borde esternal, aumento de la presin venosa yugular, hepatomegalia, ascitis, as como las
conclusiones del pecho de parches sonidos respiratorios bronquiales y crepitantes.

PREGUNTA
Cual de las siguientes acciones no es adecuada para el caso?

RESPUESTA
a.- Oxgeno intermitente 4 l / min.
b.- Furosemida 80 mg cada 12 horas y espironolactona 50 mg.
c.- Ceftriaxona intravenosa y enoxaparina 40 mg subcutnea.
d.- Prednisolona 80 mg al da.

ENFERMEDAD PULMONAR OBSTRUCTIVA CRONICA (EPOC)
CIENCIAS BASICAS: Definicin: Enfermedad inflamatoria, prevenible y tratable con efectos extrapulmonares significativos que pueden
contribuir a la gravedad de los individuos. Su componente pulmonar se caracteriza por limitacin al flujo areo, que no es totalmente
reversible y es usualmente progresiva. El tabaquismo es el mayor factor de riesgo ambiental para EPOC, otros serian ciertas
exposiciones ocupacionales (minera de carbn, oro, fbricas de algodn, etc), en algunas ciudades la combustin de biomasa con mala
ventilacin usada para cocinar, tambin incrementan el riesgo de EPOC. PATOGENIA: El agente irritante (humo de cigarro, biomasa),
desencadena inflamacin de las vas areas y del parnquima pulmonar, activando macrfagos alveolares, PMN, linfocitos T, con la
consiguiente liberacin de mediadores de inflamacin (leucotrienos, TNF a, IL-8) y estos estimulan la liberacin de proteasas,
destruyendo bronquios y parnquima = disminucin de la elasticidad, atrapamiento areo, y obstruccin al flujo areo. La bronquitis
crnica se caracteriza por hiperplasia de las glndulas submucosas de los bronquios con calibre mayor a 2mm con infiltrado
inflamatorio. El enfisema pulmonar se clasifica en centroacinar cuando se afectan las partes centrales del acino (bronquiolo
respiratorio) y estn preservados los alveolos distales y se observa en los pices; el panacinar los acinos estn afectados uniformemente
desde el bronquiolo respiratorio hasta el alvolo y es ms frecuente en las bases. DIAGNOSTICO: Sntomas cardinales: disnea
(progresiva, persistente, y empeora con el ejercicio), tos crnica (>8sem.), intermitente y present durante todo el da, expectoracin
crnica, sibilancias, sensacin de opresin torcica, prdida de peso, ansiedad. Estos sntomas sern ms perceptibles en los perodos
de agudizacin, que pueden deberse a infecciones respiratorias virales o bacterianas e incrementar la severidad del EPOC. Existe una
escasa respuesta a esteroides. Para dx. es indispensable la espirometra, que demuestre obstruccin, que no es del todo reversible,
posterior a aplicar broncodilatadores. Se concluye obstruccin tras relacin FEV1/CVF menor a 70%, el FEV1 sirve para clasificar enf. de
acuerdo a intensidad. La paO2 se mantiene cerca de lo normal hasta que el FEV1 cae <50% del valor predicho; la hipercapnia y la
hipertensin pulmonar son mas comunes por debajo de 25%. La Rx. de trax: poco sensibles, lo mas frecuente rectificacin de arcos
intercostales, abatimiento de diafragmas, hiperclaridad de campos pulmonares, corazn en gota. Medir oximetra de pulso (normal
95% a nivel del mar y 92% en Ciudad de Mxico). Gasometra para identificar insuficiencia respiratoria.

CLASIFICACION:
CLASIFICACION DE GRAVEDAD FUNCIONAL DE EPOC TRATAMIENTO
I LEVE FEV1 >80% Agonista de corta accin o anticolinrgico de corta
accin
II MODERADO FEV1 50-80% Agonista de corta accin o anticolinrgico de corta
accin
III SEVERO FEV1 30-50% Combinar lo anterior: Agonista de larga accin y/o
anticolinrgico y/ esteroides orales
IV MUY SEVERO FEV1 <30% o <50% con Insf. respiratoria y/o cor pulmonale Lo anterior mas esteroides sistemicos
TRATAMIENTO: Suspender la exposicin al agente nocivo y oxgeno suplementario si hay hipoxemia. Los medicamentos slo mejorara
sntomas, calidad de vida y disminucin en nmero de exacerbaciones. Broncodilatadores hay anticolinrgicos de accin larga
(tiotropio), accin corta (bromuro de ipratropio); agonistas b2 de accin prolongada (salmeterol), de accin corta (albuterol, salbutamol
y terbutalina) los cuales se usan como rescate en todos los cuadros; las xantinas (aminofilina, teofilina), las cuales estimulan el mov.
ciliar, broncodilatan, por sus efectos adversos solo se usan en quien no responde a los anteriores. Los esteroides inhalados (
budesonina, fluticasona, triamcinolona), se usan por periodos prolongados, indicados cuando disminuyen el FEV1 en 12% o mas y en
grado III y IV con ms de una exacerbacin al ao, no recomendados por mltiples complicaciones como osteoporosis, ganancia de
peso, cataratas, DM. Los esteroides sistmicos solo en periodos cortos, durante exacerbaciones. El tx. Quirrgico con reduccin de
volumen (resecando 20-30% de cada pice pulmonar) o trasplante pulmonar. Antibiticos fuertemente recomendados, cuando
incrementa volumen de esputo o cambia el color del mismo. PREVENCION: Suspender agente nocivo, dejar de fumar disminuye el
deterioro de la funcin pulmonar y prolonga la vida. PRONSTICO: Los pacientes con EPOC, tienen menor sobrevida; a 3 aos de dx. la
mortalidad es de 20%, a 6 es de 30% y a 8 aos es de 40%. Los factores que ms influyen en la mortalidad son el FEV1, la edad y el IMC.

CASO CLINICO
Paciente de 80 aos, exfumador desde hace 30 aos con una dosis acumulada de 70 paquetes/ao, diagnosticado de enfermedad
pulmonar obstructiva crnica (EPOC) con una obstruccin grave al flujo areo (volumen espiratorio forzado en el primer segundo
[FEV1] del 40% posbroncodilatador), en tratamiento con oxgeno crnico domiciliario a 2l/min durante las 24h (PaO2 en fase estable de
64mmHg con oxgeno y 46mmHg sin l) desde hace 10 aos. El paciente refiere dolor torcico tipo quemazn que le dificulta la
MANUAL DE TRABAJO DEL CURSO ENARM CMN SIGLO XXI
CURSO ENARM CMN SIGLO XXI TEL: 36246001 Pharmed Solutions Institute PGINA 49

respiracin, el dolor es diario y continuo, aunque mejora al tumbarse y se hace ms intenso a ltima hora de la tarde; tambin refera
sensacin de hormigueo en ambos pies y en el muslo derecho, con anestesia del dorso interno del pie izquierdo.

PREGUNTA
Cual es su conducta a seguir mas adecuada para el caso?

RESPUESTA
a.- AINEs ms antihistamnico.
b.- Antiviral tpico y oral.
c.- Gabapentina y capsaisina.
d.- Complejo B1, B6, B12.

CASO CLINICO
Varn de 57 aos, con antecedentes de ex tabaquismo, ex enolismo, bronquiectasias y EPOC grave de 17 aos de evolucin, con
mltiples ingresos por exacerbaciones. Ingres por aumento de su disnea habitual, febrcula y tos con expectoracin purulenta de 10
das de evolucin. Como tratamiento de base precisaba oxgeno domiciliario, broncodilatadores, esteroides inhalados, teofilina,
antibioterapia ocasional y esteroides orales de forma discontinua, que no reciba desde haca 2 meses. En la exploracin estaba
consciente y orientado, sin cianosis, la frecuencia cardaca era de 100 lat/min y la respiratoria de 25 respiraciones/min. En la
auscultacin pulmonar se apreciaban roncus y sibilancias diseminadas, sin otros datos destacables. En las pruebas complementarias
destacaban: 10.700 leucocitos con desviacin izquierda y velocidad de sedimentacin globular de 77 mm/h; gasometra arterial
(fraccin inspiratoria de oxgeno: 0,24) con pH de 7,35, presin arterial de oxgeno de 85 mmHg, presin arterial de anhdrido carbnico
de 51 mmHg y HCO3 de 28.

PREGUNTA
Cual de los siguientes datos radiolgicos es menos probable observar al momento del ingreso?

RESPUESTA
a.- Insuflacin pulmonar, lesiones residuales en ambos pex pulmonares.
b.- Aumento de la trama broncovascular.
c.- Signos de hipertensin pulmonar de tipo precapilar y discreto patrn alveolar en la lngula.
d.- Infiltrados alveolares parcheados bilaterales que afectaban al lbulo inferior izquierdo.

PREGUNTA
El examen microbiolgico y cultivo procedente del catter telescopado y broncoaspirado mostr bacilos grampositivos ramificados,
dbilmente teidos con la tcnica de Ziehl-Neelsen, que se identificaron como N. asteroides, Cual es tratamiento mas adecuado?

RESPUESTA
a.- Amoxicilina y cido clavulnico.
b.- Cefuroxima y gentamicina,
c.- Vancomicina y trimetoprim-sulfametoxazol.
d.- Cefotaxima e imipenem.

CASO CLINIO
Varn de 78 aos, exfumador desde haca 18 aos de 40 paquetes/ao, diagnosticado de EPOC (estadio IV de la clasificacin GOLD) con
buen control en consultas externas; sndrome de apneas-hipopneas del sueo tratado con presin positiva continua en la va area;
insuficiencia respiratoria crnica con oxigenoterapia crnica domiciliaria, y artritis reumatoide en tratamiento con leflunomida. En los 3
meses previos haba ingresado en 3 ocasiones en el hospital por exacerbacin de la EPOC, con aislamiento de Escherichia coli en
esputo, que se trat segn antibiograma con cefditorn por va oral (400mg/12h) durante 21 das. Ingres por un cuadro de 5 das
consistente en aumento de la disnea habitual hasta hacerse de reposo, expectoracin purulenta y febrcula. La exploracin fsica revel
roncus a la auscultacin pulmonar. En la analtica destacaba un valor de protena C reactiva de 2,48mg/dl. La radiografa de trax (figura
1) mostr signos de hipertensin pulmonar precapilar y aumento de la trama broncovascular bilateral. La exploracin funcional
respiratoria era: volumen espiratorio forzado en el primer segundo (FEV1) de 670ml (28%), capacidad vital forzada (FVC) de 1.200ml
(35%) y FEV1/FVC del 56%, con test broncodilatador negativo.

PREGUNTA
Segn la clasificacin de gravedad funcional del EPOC, en cual se encuentra el caso.

RESPUESTA
a.- Leve
b.- Moderado
c.- Severo
d.- Muy Servero

CASO CLINICO
Acude al servicio de urgencias paciente de 45 aos de edad del sexo masculino, refiere que desde hace 3 semanas ha presentado tos,
con secresiones que inicialmente fueron transparentes y actualmente son verdosas amarillentas, agrega que se siente cansado y no ha
MANUAL DE TRABAJO DEL CURSO ENARM CMN SIGLO XXI
CURSO ENARM CMN SIGLO XXI TEL: 36246001 Pharmed Solutions Institute PGINA 50

podido trabajar adecuadamente, esta tos la ha presentado desde hace mas de 5 aos y ha presentado al menos un cuadro semejante a
este una vez al ao, sus antecedentes patolgicos: originario de puebla en comunidad rural que emigro a la ciudad de mexico a los 15
aos, es trabajador de una maderera desde entonces y tabaquismo positivo desde hace 20 aos a razn de una cajetilla diaria. A la
exploracin fsica se observa leve cianosis peribucal, levemente pletrico, campos pulmonares con estertores crepitantes, sibilancias. El
area cardiaca se apresia disminucin de ruidos cardiacos.

PREGUNTA
Se realizo radiografia de torax, cual de las siguientes caractersticas observadas no se observan en la imagen?

RESPUESTA
a.- Rectificacin de los arcos costales.
b.- Abatimiento del diafragma.
c.- Ensanchamiento de los espacios intercostales.
d.- Hiperclaridad de campos pulmonares.

PREGUNTA
Se realizo espirometria tres meses despus del cuadro actual, encontrandoce FEV1 > 55 % y FEV1/CVF 0.8, tomando encuenta los datos
cual es el grado que presenta el paciente, de acuerdo con la Iniciativa Global para la Enfermedad Pulmonar Obstructiva Cronica?

RESPUESTA
a.- Leve.
b.- Moderado.
c.- Severo.
d.- Muy severo.

PREGUNTA
Considerando el diagnotico y clasificacin actual del paciente cuales son las medidas preventivas y de rehabilitacin mas adecuados
para influir sobre el pronostico del paciente?

RESPUESTA
a.- Vacunacion antineumococo y antiinfluenza.
b.- Rehabilitacion pulmonar.
c.- Reseccin de apices y transplante pulmonar.
d.- Retiro de factores de riesgo.

CANCER DE PULMON
CIENCIAS BASICAS: Factores de riesgo: El principal tabaquismo, mostrando una relacin entre consumo y nmero de cigarrillos
consumidos por da, as como por el inici y tiempo de consumo dandonos el indice tabaquico que calcula asi: N. de cigarrillos fumados
al dia x aos de consumo entre 20. Indice tabaquico >20 paquetes/ ao; se considera con incremento en factor de riesgo para cncer
pulmonar. Otros son asbesto, arsenico, niquel. SALUD PUBLICA: Primer lugar de mortalidad en todos los tipos de cncer en el mundo
occidental. En Mxico la tasa de mortalidad para cncer pulmonar es de 125.2 por cada 100,000 en hombres y 48.8 por cada 100,000
en mujeres. Mas comun en hombres de 50-70 aos. PATOGENIA: El humo de tabaco contiene alrededor de 4800 compuestos; 60 son
cancergenos, ya que reaccionan con el DNA humano, formando complejos con las bases nitrogenadas, as en la replicacin de DNA, se
introducen errores en la copia, dando lugar a mutaciones. El mas estudiado es el benzopireno, que hace una tramsversion de guanina
por tiamina y las nitrosaminas que median reacciones de alquilacin de DNA. Se han detectado mutaciones en el gen p53 (cromosoma
17), el cual es un gen supresor de tumores, que al no funcionar adecuadamente permite la sobrevida de elementos geneticamente
daados, que conducen a la transformacin tumoral. Tambin hay alteracines en el gen ras y myc. CLASIFICACIN: Se dividen en
benignas (2-5%) y malignas (95%). Dentro de los benignos, el hamartoma es el mas frecuente, en general asintomtico, los sintomas
dependern de localizacin, intratraqueales; tos, estridor, disnea incluso hemoptisis. Endobronquiales; obstruccin bronquial,
neumona, neumonitis, hiperinsuflacion. Intraparenquimatosos; gnral. asintomaticos sin importar tamao. El hamartoma es una masa
formafa de cartilago, tej. conectivo, grasa, musculo liso y epitelio respiratorio, en Rx. presentan calcificaciones en palomita de maiz,
caracteristico. Malignas: Se dividen en 2: Cncer broncogenico de celulas pequeas (20%; subtipos: carcinoma de cel. pequeas, mixto
de cel. pequeas y grandes, combinado de cel. pequeas) y de celulas no pequeas (80%; subtipos: adenocarcinoma 60%, escamoso o
epidermoide 30%, carcinoma indiferenciado de cel. grandes <10%). TUMOR DE PANCOAST: Es una forma de presentacin del
carcinoma de pulmn, localizado en el surco pulmonar superior, cerca del vrtice pulmonar, evolucin lenta, tendencia a infiltrar los
linfticos subpleurales y por contiguidad raices de 8vo. cervical, 2do y 3er dorsal, cadena simptica y estructuras seas. Clinica: Dolor
em hombro irradiado a lo largo del brazo, patestesias (por infiltracin del plexo braquial), irradiacin a hombro y codo, debilidad y
atrofia de musc. intrnsecos, Sx. Horner (ptosis, miosos, anhidrosis). Rx. opacidad o engrosamiento unilateral en vrtice pulmonar,
destruccion de 1ra y 2da. costilla. CA. BRONCOGENICO DE CEL. PEQUEAS: Muy agresivo, localizacin central, crecimiento rpido,
supervivencia de 2-4 meses del dx. sin tratamiento. Tiende a la diseminacin. Sensible a quimio y radioterapia.tratamiento CA.
BRONCOGENICO DE CEL. NO PEQUEAS: Son agregados heterogneos. Adenocarcinoma pulmonar: 60%, tiende a ser perifrico, se
subdivide en acinares, slidos, papilares y bronquioloalveolares, invade la pleura hasta en 50%, mas comun en pacientes no fumadores
y en mujeres. Escamoso o epidermoide: 30%, deriva del epitelio superficial de los bronquis, suele ser central y con tendencia a la
exfoliacion, crecimiento lento, buen pronostico. Carcinoma de cel. grandes: el menos frecuente y de peor pronstico, clulas con
citoplasma y nucleo grandes.CUADRO CLNICO: Inespecificos; 5-15% asintomticos. Fatiga, anorexia, perdida d peso. Asociados al
tumor; tos seca (lo mas comn), puede haber expectoracin, hemoptisis, dolor torcico, disnea frecuente. Manifestaciones por
MANUAL DE TRABAJO DEL CURSO ENARM CMN SIGLO XXI
CURSO ENARM CMN SIGLO XXI TEL: 36246001 Pharmed Solutions Institute PGINA 51

compresin; Sx. de Horner, paralisis del hemidiafragma por invasin del N. frenico, tumor de Pancoast, Sx. de vena cava superior (por
obstruccin vascular, ingurgitacin yugular, edema en esclavina), paralisis laringea. Metstasis; mas comun a huesos ( fracturas seas),
hgado, glndulas suprarrenales, ganglios retroperitoneales.DIAGNSTICO: Precoz; mayores de 45 aos, fumadores, citologia de
esputo, radiografa de trax, TAC. Pacientes con sintomas y signos; establecer dx. histolgico, biopsia, broncoscopia. TRATAMIENTO:
Carcinoma de cel. no pequeas; ciruga y radioterapia, potencialmente curativo en estadios precoces. Carcinoma de cel. pequeas, es
quimioterapia asociada a radioterapia. CANCER PULMONAR METASTSICO: El pulmon es un sitio muy frecuente para la implantacin
de metstasis de tumores pulmonares y extrapulmonares, hasta 30-40% de los canceres producirn metstasis a pulmn. Los tumores
que mas afectan son: cncer de mama, de prostata, gastrointestinales, rion. Ya sea por via hematgena, via linftica.

CASO CLINICO
Una mujer de 64 aos fue ingresada en el hospital por una disnea creciente y dolor torcico. La paciente tena antecedentes de
carcinoma pulmonar no microctico y estaba recibiendo tratamiento. Anteriormente haba fumado 4-5 cigarrillos al da durante un
periodo de hasta 10 aos. A la exploracin, se trataba de una mujer con sensacin de enfermedad y emaciacin, que estaba
clnicamente anmica y presentaba signos de un derrame pleural izquierdo masivo en la TC torcica. La frecuencia del pulso era de
85/min, la presin arterial de 120/85, y no haba signos de insuficiencia cardiaca. La radiografa de trax confirm el derrame unilateral
izquierdo. La hemoglobina era de 9,1g/100ml, con un frotis de sangre normocrmico y normoctico. Los niveles sricos de ferritina,
vitamina B12 y folato eran normales. La velocidad de sedimentacin globular fue de 90mm en la primera hora.

PREGUNTA
Considerando el caso clnico cual es el agente causal mas probable?

RESPUESTA
a.- Bordetela perturssi.
b.- Streptococcus epidermidis.
c.- Staphylococcus aureus.
c.- Enteroccocus fecalis.

CASO CLINICO
Paciente de 69 aos, exfumador, diabtico y dislipmico con el diagnstico de adenocarcinoma pulmonar obtenido mediante
fibrobroncoscopia que mostr una tumoracin en el orificio del bronquio segmentario anterior del lbulo superior derecho. La
tomografa por emisin de positrones (PET-TC) informaba de una masa hipercaptante (SUVmx. de 15g/ml) paramediastnica en el LSD
compatible con tumoracin maligna, una adenopata paratraqueal inferior derecha con centro graso sin captacin de FDG y una lesin
paratraqueal superior derecha de 8mm discretamente hipermetablica que se interpret como adenopata. Ante estos hallazgos se
estadific como T3NxM0. Al tratarse de un paciente candidato a tratamiento quirrgico con intencionalidad radical, se consider
imprescindible la obtencin de muestras para descartar la afectacin N2.
PREGUNTA
Cual de las siguientes mets es menos frecuente en este caso?

RESPUESTA
a.- Huesos
b.- Hgado,
c.- Tiroides.
d.- Glndulas suprarrenales.

CASO CLINICO
Varn de 26 aos, sin hbitos txicos ni antecedentes patolgicos de inters, que acudi a urgencias por disnea progresiva de una
semana de evolucin y expectoracin hemoptoica en las ltimas 48 h, acompaadas de malestar general, distermia, dolor
centrotorcico, dolor en pelvis derecha y columna lumbar. Hemoglobina de 9 g/dl, hematcrito del 27% y leucocitosis de 13.500
clulas/ l con neutrofilia. En la bioqumica general se observaban patrn de colestasis e hipoalbuminemia. La gasometra mostraba
insuficiencia respiratoria (pH: 7,45; presin arterial de anhdrido carbnico: 37 mmHg; presin arterial de oxgeno: 54 mmHg; HCO3: 27
mmol/l. En el estudio de coagulacin se objetiv una actividad de protrombina del 50%, actividad de tromboplastina tisular de 17 s y
fibringeno de 300 mg/dl. En la radiografa de trax se observaban un derrame pleural derecho que ocupaba un tercio del hemitrax y
patrn intersticial bilateral, ms evidente en los lbulos superior derecho y medio.

PREGUNTA
Considerando el cuadro clnico cual es la conducta inmediata mas adecuada?

RESPUESTA
a.- Toracocentesis.
b.- Toracostomia diagnostica.
c.- Colocacion de sello.
d.- Drenaje endotoracico.

PREGUNTA
Considerando el cuadro clnico, cual es la clase mas probable?

MANUAL DE TRABAJO DEL CURSO ENARM CMN SIGLO XXI
CURSO ENARM CMN SIGLO XXI TEL: 36246001 Pharmed Solutions Institute PGINA 52

RESPUESTA
a.- Cncer broncogenico de celulas pequeas.
b.- Carcinoma de clulas grandes.
c.- Adenocarcinoma escamoso.
d.- Carcinoma indiferenciado de celulas grandes.

CASO CLINICO
Masculino de 51 aos de edad que inicia desde hace 3 das con tos productiva, ataque al estado generalizado, fiebre, dificultad
respiratoria, en Rx de torax se aprecia datos compatibles con neumona lobar inferior derecha y un ndulo redondo de 1,5 cm lbulo
superior izquierdo, recibe tratamiento presentando mejora egresando en una semana, se realiza TAC ambulatoria, donde se aprecia
ndulo de 1.5 por 1.8 ubicado en la misma posicin, no se aprecian calcificaciones con los bordes levemente festoneados, en
mediastino no se observan adenopatas ni datos patolgicos, no se aprecia exudados pleurales.

PREGUNTA
Cul es el estudio ms apropiado para continuar con el diagnostico en este caso

RESPUESTA
a.- MRI
b.- PET con FDG
c.- Broncoscopia.
d.- TAC de control a 6 meses.

CASO CLINICO
Masculino de 71 aos de edad con dolor en la espalda desde hace dos meses de inicio sbito sin factor precipitante, agrega que se
incrementa el dolor cuando se acuesta y disminuye durante el da y con el movimiento. El paciente es originario del Estado de Mxico,
fue trabajador de la construccin con alcoholismo y tabaquismo positivo por ms de 30 aos, los cuales dejo hace 15 aos, no es
diabtico ni hipertenso, al examen fsico se observa con aspecto general adecuado, alerta y orientado, ndice de masa corporal de 25,
se realiza laboratorios de rutina donde solo se realiza el hallazgo de fosfatasa alcalina elevada y en la radiografa de columna se observa
lesin ltica nica en la vertebra L3.

PREGUNTA
El cuadro clnico nos indica una probable tumoracin maligna, cual es la ms probable.

RESPUESTA
a.- Cncer pancretico.
b.- Cncer gstrico.
c.- Cncer tiroideo.
d.- Cncer pulmonar.

TRASTORNOS HIPOTALAMICOS E HIPOFISIARIOS. El hipotlamo libera factores liberadores de hormonas hipofisiaria, asi como
dopamina que inhibe la prolactina evitando su liberacin, adems de la sntesis de arginina, vasopresina y oxitocina, El sndrome
hipotalmico mas frecuente es la secresion anormarl de GnRH consecuentemente reduccin de gonadotrofinas e hipogonadismo. Las
causas mas comunes, esta el estrs, perdida de peso o ejercicio exesivo, el tratamiento envuelve el manejo de la causa cuando es
posible, la administracion pulsatile GnRH es usualmente afectivo. SINDROME HIPOTALAMICO: Poco frecuente, ocurre en tumores,
posterior a neurociruga por craneofaringeoma, o por infiltracin como en la histiocitosis de clulas de Langerhans. La tpica
presentacin es hiperfagia, con cambios de pero, prdida de sensacin, somnolencia, cambios de conducta. Se debe regular los liquidos
diarios y el balance. Los efectos metabolicos del dao hipotalmico por masa son frecuentemente alteracin del apetito (hiperfagia y
obesidad y anorexia), sed, adipsia, beber compulsivo, Temperatura (hipertermia, hipotermia), somnolencia y coma.
HIPOPITUITARISMO: Es una patologia que se caracteriza por la disminucin parcial o completa de las hormonas hipofisiarias anterior o
posterior o como producto de la interferencia del funcionamiento del hipotlamo sobre la actividad hipofisiaria. Las CAUSAS mas
frecuentes son los tumores hipofisiarios, tumores parahipofisiarios (craniofaringiomas, meningiomas, secundarios pecho, pulmon,
cordomas, gliomas) por radioterapia (hipofisiaria, craneal y nasofarngea) por infarto hipofisiario (sndrome de sheehan), Infiltrado
hipofisiario (sarcoidosis, linfociticas, hemocromatosis, histiocitosis de clulas de langerhans, enfermedad de Erdheim-Chester),
sndrome de silla turca vacia, por causa infecciosa, traumatica, sndrome de Kallman.

CASO CLINICO
Varn de 34 aos de edad, sin antecedentes, inici a los 31 aos con dolor tpico, en regin supraorbitaria derecha, intenso, pulstil, de
30-60 minutos de duracin, asociado a inyeccin conjuntival ipsilateral, congestin nasal y en ocasiones rinorrea. Presentaba 2-3
ataques por da en perodos que duraban de 7-10 das y se diagnostic con Cefalea en Racimos CR episdica. El examen neurolgico no
mostr datos patolgicos y la imagen de resonancia magntica (RM) inicial no mostr anormalidades. Se inici el tratamiento con
verapamilo 240 mg/da y se administraba oxgeno y sumatriptn durante los ataques desde el inicio del cuadro, con respuesta
adecuada. Tres aos despus se aadi disminucin de la agudeza visual (AV) y defecto en el campo visual del ojo derecho. La
evaluacin oftalmolgica report AV 20/100 y escotoma central en el ojo derecho y AV 20/20 sin alteracin del campo visual en el ojo
izquierdo.

PREGUNTA
MANUAL DE TRABAJO DEL CURSO ENARM CMN SIGLO XXI
CURSO ENARM CMN SIGLO XXI TEL: 36246001 Pharmed Solutions Institute PGINA 53

Cual es su conducta a seguir mas adecuada para el caso?

RESPUESTA
a.- Realizar RX con nfasis en silla turca.
b.- Evaluacin de fondo de ojo con dilatacin.
c.- Resonancia magnetica contrastada.
d.- Perfil hormonal hipofisiario.

CASO CLINICO
Mujer de 61 aos con diabetes mellitus tipo 2, hipercolesterolemia, histerectoma ms doble anexectoma y sndrome de apnea
obstructiva del sueo (SAOS) severo. Estaba diagnosticada desde haca 3 aos de miocardiopata dilatada con coronarias
angiogrficamente normales y disfuncin ventricular severa. Segua tratamiento con betabloqueador 25 mg, digoxina en dosis bajas,
ARA 2, espironolactona y estatinas. La exploracin fsica mostraba una presin arterial de 120/60 mmHg y sobrepeso; la auscultacin
pulmonar fue normal y la cardiaca era rtmica, con un soplo sistlico eyectivo I/VI. La analtica mostr una glucosa de 148 mg/dl, y el
resto de la bioqumica, la coagulacin y el hemograma eran normales. El electrocardiograma (ECG) estaba en ritmo sinusal a 40 lat/min
con bloqueo de rama izquierda, y haba cardiomegalia en la radiografa de trax.

PREGUNTA
Cual es su conducta mas adecuada a seguir?

RESPUESTA
a.- Perfil hormonal hipofisiario.
b.- Ecocardiograma transesofagico.
c.- Resonanacia magnetica selar.
d.- Coronariografia.

CASO CLINICO
Mujer de 32 aos, con ciclos menstruales regulares, asintomtica y sin tratamientos crnicos, a la que se le practic una RNM craneal
en febrero de tras un traumatismo craneoenceflico. Se objetiv un adenoma hipofisario de 13 x 10 mm, intraselar, lateralizado hacia la
derecha. El campo visual y la exploracin oftalmolgica eran normales, y las determinaciones de hormonas hipofisarias tambin. Tras
tratamiento con cabergolina (0,5 mg dos veces por semana) durante 3 meses. Ante la necesidad de un seguimiento indefinido,
previsiblemente largo por la edad de la paciente, se acord con ella la exresis quirrgica del macroadenoma no funcionante, que se
efectu de forma total y selectiva, por va transesfenoidal sublabial.

PREGUNTA
Cual es el porcentaje de reduccin de los incidentalomas?

RESPUESTA
a.- 60 %
b.- 30 %
c.- 20 %
d.- 10 %

CASO CLINICO
Varn de 66 aos, con depresin no tratada farmacolgicamente, y con hiperplasia benigna de prstata que precis biopsia en 3
ocasiones por cifras elevadas de antgeno prosttico (PSA). El paciente refera astenia intensa, sofocos, prdida de fuerza muscular y
disfuncin erctil completa en el ltimo ao. La campimetra era normal. El estudio hormonal hipofisario mostr una testosterona total
de 0,19 ng/mL (valor normal (VN).: 2,9-8,0), una testosterona libre de 1,2 pg/mL (VN: 9-27), LH: 1,5 U/L (VN: 7-24), prolactina: 35 ng/mL
(VN: 4,0-18,5), subunidad alfa libre de las hormonas glucoproteicas y resto de determinaciones normales.

PREGUNTA
Cual de los siguientes estudios es el mas indicado para establecer una etiologa?

RESPUESTA
a.- USG de Suprarrenales.
b.- USG de Testiculos.
c.- TAC de Hipofisis.
d.- IRM de Hipotalamo.

CASO CLINICO
Nio de cinco aos con un cuadro de pubertad precoz de origen central, que durante el estudio etiolgico fue diagnosticado de una
lesin ocupante de espacio de localizacin supraselar El paciente fue intervenido y posteriormente ingresado en la unidad de cuidados
intensivos peditricos (UCIP) para el control postoperatorio. Se administr tratamiento antiinflamatorio con dexametasona y hormonal
sustitutivo con hidrocortisona. Transcurridas ocho horas desde la intervencin el paciente present poliuria (diuresis de 1.080ml en 4
horas, 8ml/kg/h) y el control analtico revel: sodio plasmtico (Nap): 140mmol/l; osmolaridad plasmtica (Osmp): 287 mosm/kg, con
sodio urinario (Nau): <3mmol/l y una osmolaridad urinaria (Osmu): 108 mosm/kg.
MANUAL DE TRABAJO DEL CURSO ENARM CMN SIGLO XXI
CURSO ENARM CMN SIGLO XXI TEL: 36246001 Pharmed Solutions Institute PGINA 54


PREGUNTA
Considerando el cuadro clnico, cual de los siguientes diagnosticos presuntivo es el mas frecuente?

RESPUESTA
a.- DI transitoria.
b.- DI permanente.
c.- DI patrn bifsico (DI-SIADH).
d.- DI trifsico (DI-SIADH-DI).

CASO CLINICO
Hombre de 63 aos que refiere un cuadro de dos semanas de evolucin caracterizado por fiebre y deposiciones diarreicas que ceden de
forma espontnea; posteriormente, comienza con parestesias en los dedos de las manos y de los pies, alteraciones de la movilidad e
inestabilidad de la marcha, por lo que acude al servicio de urgencias donde la inestabilidad de la marcha se hace ms manifiesta y le
impide caminar, y comienza a presentar paresias de las cuatro extremidades y de la musculatura facial. Como antecedentes personales
destaca que es hipertenso sin tratamiento ni seguimiento y bebedor de 20 gramos de alcohol diarios. En la exploracin neurolgica
destacan: dipleja facial con habla hipofnica, reflejos osteotendinosos rotulianos y aquleos abolidos e hiporreflexia bicipital y tricipital
sin prdida de fuerza, abolicin de la sensibilidad vibratoria y disminucin bilateral de la sensibilidad tctil y algsica con marcha
atxica. En las pruebas complementarias presenta: leucocitos 10.100 con frmula normal, hemoglobina 16 g/l, creatinina 0,6 mg/dl,
cido rico 2,8 mg/dl, sodio 120 mmol/l, potasio 4,3 mmol/l, cloro 87 mmol/l, bicarbonato 16 mEq/l, colesterol total 232 mg/dl, glucosa
110 mg/dl, y osmolaridad srica 259 mOsm/kg. Sodio urinario 144 mmol/l y osmolaridad en orina 719 mOsm/kg.

PREGUNTA
Cual de los siguientes patologas presentan mas frecuentemente SIADH?

RESPUESTA
a.- Traumatismo crneo enceflico.
b.- Tratamiento quirrgico hipofisiario.
c.- Cncer de pulmn de clulas pequeas.
d.- Sndrome paraneoplsico.

CASO CLINICO
Varn de 61 aos, con hipercolesterolemia e hipertensin arterial de muchos aos de evolucin, controlada con medicacin. Se realizo
operacin de menisco de rodilla, bajo anestesia raqudea con 10 mg de bupivacaina 0,5%. Durante la operacin la presin arterial
sistolica decreci desde 120 a 80 mm Hg, permaneciendo en ese nivel durante tres horas y, el pulso, en 30-45 s/m. En el inmediato
postoperatorio el enfermo sufri una agudizacin grave de la cefalea, junto con un cuadro confusional leve y ebre de 39. La cefalea
era generalizada, opresiva y no relacionada con la posicin ortosttica. Cuarenta y ocho horas ms tarde el paciente presenta prdida
de visin bilateral, agitacin y mayor confusin mental. La determinacin de Na plasmtico fue de 129 mmol/L.

PREGUNTA
Cual es la complicacin mas grave tras la correccin rpida del Na plasmtico?

RESPUESTA
a.- Mielonolisis pontina.
b.- Degeneracin pontino cerebelosa.
c.- Mielonolisis cortico basal.
d.- Hemorragia periacueductal.

CASO CLINICO.
Masculino de 73 aos con una prdida de peso de 5 kg, fatiga, prdida de apetito, desinteres por ambiente, falta de intensionalidad y
tos desde hace 2 meses, con expectoracin, Inform fumar un paquete de cigarrillos por da durante los ltimos 50 aos. EF 162 cm de
altura, 53 kg de peso, temperatura era de 35,2 C, el pulso 74/min y la presin arterial 102/60 mmHg. Se auscultaron sibilancias y
roncus audibles en la zona basal izquierda. Se diagnostico cncer pulmonar, se agrego posteriormente poliuria y polidipsia.

PREGUNTA
Cual es su impresin diagnostica en este momento?

RESPUESTA
a.- Diabetes inspida.
b.- Sindrome de addison.
c.- Crisis hipotiroidea.
d.- Panhipopituitarismo.




MANUAL DE TRABAJO DEL CURSO ENARM CMN SIGLO XXI
CURSO ENARM CMN SIGLO XXI TEL: 36246001 Pharmed Solutions Institute PGINA 55

CASO CLINICO
Se trata de femenino de 18 aos con diagnostico de retrazo psicomotor con deterioro del desarrollo del lenguaje a la EF se clasifica con
Tanner 1 con una altura de 144,8 cm. BH y QS sin datos importantes, se detectaron estradiol y progesterona baja. FSH y LH con minima
respuesta a la estimulacin con GnRH, resto de laboratorio sin datos por agregar, campimetra normal.

PREGUNTA
Cual la conducta a seguir mas adecuada para el caso?

RESPUESTA
a.- RX de crneo.
b.- TAC de crneo.
c.- IRM de crneo.
d.- USG plvico.

CASO CLINICO
Mujer de 35 aos, con antecedentes de asma bronquial y consumidora de fin de semana de cocana inhalada, hbito no referido al
ingreso, que acude al Servicio de Urgencias por un cuadro de cefalea intensa de predominio occipital, con nauseas, vmitos, fotofobia y
fiebre de hasta 38 C. Se le practica una tomografa axial computarizada cerebral, que no muestra hallazgos patolgicos, y una puncin
lumbar, que evidencia lquido cefalorraqudeo (LCR) con pleocitosis de predominio polimorfonuclear e hipoglucorraquia. En la
exploracin neurolgica, no se evidenciaron hallazgos excepto rigidez de nuca con signos menngeos positivos.

PREGUNTA
Cual de la siguientes aseveraciones es incierta, en el padecimiento actual?

REPUESTA
a.- La clnica puede ser indistinguible de la de un tumor hipofisario o una infeccin del sistema nervioso central.
b.- Habitualmente, se presentan con insuficiencia pituitaria, manifestada como hiperprolactinemia panhipopituitarismo.
c.- Las alteraciones visuales estn presentes en el 50-75%, la heminopsia uni o bitemporal es el patrn hallado con ms frecuencia.
d.- La fiebre y la leucocitosis estn presentes hasta en el 46% de los pacientes.

CASO CLINICO
Se trata de femenino de 27 aos de edad la cual acude a consulta refiriendo visin borrosa en ambos ojos que le ha causado algunos
accidentes como tropezar o golpearse debido a que no alcanza a ver los objetos, refiere adems que desde hace 2 aos
aproximadamente ha tenido cefalea que anteriormente se controlaba con la administracin de AINES, en el examen del campo visual
con confrontacin digital se revela reduccin de la visin en la periferia izquierda del ojo izquierdo y en la periferia derecha del ojo
derecho, a la exploracin general observa que la paciente presenta obesidad grado I, acn e hirsutismo, la paciente agrega presentar
alteraciones menstruales.

PREGUNTA
Considerando los hallazgos en los campos visuales, donde es ms probable que se ubique la lesin:

RESPUESTA
a.- Espacio suprasillar.
b.- Nervios pticos.
c.- Tracto posquiasmaticos.
d.- Cuerpos geniculados.

CASO CLINICO
Acude a consulta paciente masculino de 31 aos de edad el cual refiere que desde hace 6 meses presenta cefalea continua que no sede
a AINES indicados por su mdico familiar, agrega que tuvo un accidente automovilstico hace un mes, niega nausea o vomito, a la
exploracin fsica se observa con leve crecimiento de areas acras del cuerpo principalmente en dedos y maxilar inferior, sus estudios de
laboratorio muestran dislipidemia e incremento de glucosa, finalmente refiere disfuncin erctil.

PREGUNTA
Cul es la hormona responsable que ms probable se encuentre alterada.

RESPUESTA
a.- ACTH
b.- PRL
c.- GH
d.- TSH.


MANUAL DE TRABAJO DEL CURSO ENARM CMN SIGLO XXI
CURSO ENARM CMN SIGLO XXI TEL: 36246001 Pharmed Solutions Institute PGINA 56

TRASTORNOS TIROIDEOS
CIENCIAS BASICAS: Tiroides mayor glndula endocrina del cuerpo pesa 20grs. La produccin de tiroxina (T4) y triyodotironina (T3), se
controla a travs de un bucle de retroalimentacin endocrina clsica, se forman con la unin de la tiroglobulina mas iodo, mediante una
peroxidacion. Parte de T3 es secretada por tiroides, pero ms es producida por desyodacin de T4 en los tejidos perifricos. Tanto la T4 y
T3 estn ligadas a protenas transportadoras (globulina de unin a la tiroxina 80% (TBG), transtiretina, y albmina) en la circulacin.
Aumento de los niveles de T4 total y T3 con niveles libres normales se observan en los estados de aumento de protenas transportadoras
(embarazo, los estrgenos, cirrosis, hepatitis y trastornos hereditarios). Por el contrario, la disminucin de los niveles totales de T4 y T3,
con niveles libres normales se observan en la enfermedad sistmica grave, enfermedad heptica crnica, y nefrosis.

HIPOTIROIDISMO (Hipofuncin de glndula tiroides)
H. PRIMARIO
(tiroides)
1.- Dficit de yodo: Principal causa a nivel mundial. 2.- Autoinmunitario: Tiroiditis de Hashimoto: 1ra causa donde no existe dficit
de Iodo. Tiroiditis atrfica. 3.- Iatrogena: Tiroidectoma total o subtotal, radiacin de cuello por tx. de linfoma o cncer, uso de I
131
.
4.- Medicamentos: Amiodarona, litio, antitiroideos, ac. p-aminosalicilico, aminoglutecimida. 5.- Congnito: Disgenesia de tiroides
(80-90%), errores en hormonas tiroideas (10-15%), ectopia o hipoplasia de tiroides. 6.- Trastornos que infiltran tiroides:
Amiloidosis, sarcoidosis, hemocromatosis, esclerodermia.
H. TRANSITORIO 1.- Tiroiditis silenciosa (puerperio). 2.- Tiroiditis subaguda
H. SECUNDARIO
(hipfisis)
1.- Hipopituitarismo: Tumores de hipfisis, Qx, radiacin hipofisiaria, trastorno infiltrativo, traumatismos, Sx. de Shehann (necrosis
hipofisiaria post-parto, por colapso circulatorio e isquemia, secundario a hemorragia obsttrica).. 2.- Dficit o inactividad aislada de
la TSH. 3.- Tratamiento con bexaroteno. 4.- Enf. Hipofisiaria: Tumores, traumatismos, trastornos infiltrativos, idioptico.
CUADRO CLINICO Debilidad, cansancio, intolerancia al frio, retardo mental, poco comunicativo, aumento de peso (almacenan glucosa como lpidos),
disminucin de apetito, bradicardia, piel seca y spera, cada de pelo, extremidades fras, macroglosia (por depsito de
glucosaminoglicanos), menorragia, disfona, estreimiento, disminucin de reabsorcin tubular, derrame pericrdico.
DIAGNOSTICO Si TSH elevada; T4L y T4 normales = H. subclnico o leve Buscar Ac-TPO; positivo-tx. con T4. Negativo-vigilancia anual
Si TSH elevada; T4L y T4 disminuidas = H. clnico Buscar Ac TPO; positivo-autoinmunitario (presentes en 90-95%). Negativo-
iatrogeno, otro. Si TSH N o elevada; T4L Yt4 disminuidas = Orienta hacia causa hipofisiaria (H. secundario). TSH elevada marcador
sensible de hipotiroidismo 1ro.
TRATAMIENTO Primera eleccin levotiroxina (T4). Comienza su efecto hasta 4 sem. despus de su inicio. Dosis de 100-150g/24h VO, si hay
cardiopata 12.5-25g, incrementando cada 6-8 semanas. Objetivo TSH normal; llegar al eutiroidismo.
COMA
MIXEMATOSO
Forma ms severa del hipotiroidismo, urgencia mdica, alto ndice de mortalidad. Caracterstico alteraciones de las funciones
mentales. Hipotensin, hipotermia, bradicardia, convulsiones, hiponatremia, hipoglucemia. Factores de riesgo: sepsis, qx.,
anestesia, IAM, trauma, EVC
Se presenta en 8% mujeres y 2% en hombres. TX.: levotiroxina 400g IV impregnacin; 100g IV c/24h para control. Hidrocortisona
50 mgs
CONGENITO Prevalencia 1 de cada 4000, transitorio o permanente dependiendo si madre tiene AC frente a TSH-R. Cuadro clnico: hipotona,
intolerancia a VO, macroglosia, fontanela posterior amplia, ictericia prolongada. DX.: Tamiz neonatal, para evitar dao neurolgico
permanente. TX.: Administracin de T4, dosis 10.15g/kg/da
AUTOINMUNITARIO Prevalencia en >60 aos. Incidencia 4 por cada 100 mujeres y 1 en cada 1000 hombres. Caracterstica clnica presencia de Bocio,
irregular y firme, frecuente asociado a DM, vitligo, LES, artritis reumatoide

HIPERTIROIDISMO (Hiperfuncin de glndula tiroides)
PRIMARIO 1.- Enf. Graves-Basedow. 2.- Bocio multinodular toxico. 3.- Adenoma toxico (estos 3 los ms frecuentes). 4.- Metstasis de cncer
de tiroides. 5.- Mutacin activada del receptor de TSH. 6.- Estruma ovrico. 7.- Fenmeno de Jod-Basedow (exceso de I)
SECUNDARIO 1.- Tumores que secretan gonadotrofina corionica con TSH baja. 2.- Tirotoxicosis en el embarazo con TSH baja. 3.- Tiroiditis
subaguda. 4.- Tiroiditis silenciosa
TIROTOXICOSIS sin
hipertiroidismo
Estado hipermetabolico donde existe un exceso de hormonas T3 y T4 libres. 1.- Frmacos: amiodarona. 2.- radiacin a cuello por
linfomas o cncer. 3.- Infarto de un adenoma. 4.- Tirotoxicosis facticia (ingesta excesiva de T4)
CUADRO CLINICO Hiperactividad, nerviosismo, irritabilidad, disforia, palpitaciones, taquicardia sinusal (lo ms frecuente y que puede llevar a
fibrilacin auricular), fatiga, debilidad muscular, prdida de peso, diarrea, intolerancia al calor, piel caliente, transpiracin
exagerada, poliuria, perdida de libido, temblor, miopata proximal, retraccin palpebral mirada de asombro, ginecomastia
DIAGNOSTICO TSH srica disminuida es un marcador sensible de hipertiroidismo causado por la enfermedad de Graves, los ndulos tiroideos
autnomos, tiroiditis, y el tratamiento con levotiroxina exgena. Nivel T4 elevada suficiente para tirotoxicosis.
TRATAMIENTO Antitiroideos (tionamidas): Carbimazol, metimazol 10-20mg c/8-12h inicio; 2.5-10mg para control. Propiltiouracilo (PTU) (se puede
usar en embarazo) 100-200mg c/6-8h inicio; 50-100mgpara control, efectos secundarios rash, urticaria, fiebre, agranulocitosis (ms
importante). Iodo radiactivo; destruccin de glndula tiroides indicado en recidivas, contraindicacin absoluta en embarazo y
lactancia. Tiroidectoma; cuando falla tx. mdico y se rehsan al Iodo, til en jvenes con bocios voluminosos, complicaciones;
hemorragia, edema larngeo, hipoparatiroidismo, lesin de N. larngeo recurrente. Funcin tiroidea debe ser checada cada 3-4 sem.
despus de iniciar tx., con ajustes para mantener T4 en niveles normales. Propanolol 20-40mg VO, para disminuir taquicardia y
sntomas adrenrgicos.
Tormenta tiroidea: PTU 500mg impregnacin VO, rectal o por sonda nasogstrica, para continuar PTU 200-300mg c/6h, propanolol
y dexametasona.
GRAVES-BASEDOW Hasta 60-80% el ms comn. Principalmente mujeres. Edad entre 20-50 aos. Para su desarrollo se necesita la actividad
inmunolgica de las inmunoglobulinas estimulantes de tiroides (TSI). Se puede desarrollar oftalmopatia por la alteracin de l os
msculos extraoculares por infiltracin de linfocitos T activados, que inducen la liberacin de IL-1, IFN y TNF, que a su vez activan
a los fibroblastos y aumentando sntesis de glicosaminoglicanos, produciendo tumefaccin de los msculos. Cuando se comprime
el N. ptico se produce edema papilar, defectos en campos perifricos y ceguera. Puede haber dermopata; que es una placa
indurada violcea o rosada piel de naranja.
CRISIS TIROTOXICA O
TORMENTA TIROIDEA
Es rara, suele ocurrir despus de una enf. Estresante en pacientes no tratados, se caracteriza por taquicardia, fiebre, delirium,
vomito diarrea, deshidratacin, arritmias, coma. Exacerbacin de hipertiroidismo.

AMIODARONA: Hay dos formas principales de tirotoxicosis inducida por amiodarona (AIT). Tipo 1 AIT se asocia con alteracin tiroidea
subyacente (Enf. preclnica de Graves, bocio nodular). La sntesis de hormona tiroidea se convierte en excesiva como resultado de
incremento de la exposicin a yodo. TIA tipo 2 se produce sin anormalidades intrnsecas de tiroides y es el resultado de la tiroiditis
destructiva. El frmaco debe interrumpirse, si es posible, con la administracin de frmacos antitiroideos altas dosis, en el tipo 1
perclorato potsico y los glucocorticoides en el tipo 2.
MANUAL DE TRABAJO DEL CURSO ENARM CMN SIGLO XXI
CURSO ENARM CMN SIGLO XXI TEL: 36246001 Pharmed Solutions Institute PGINA 57

BOCIO MULTINODULAR TOXICO: Bocio voluminoso, por lo general en edad avanzada y se puede presentar con fibrilacin auricular o
palpitaciones, taquicardia, nerviosismo, prdida de peso. La exposicin reciente a yodo, a partir de los medios de contraste o de otras
fuentes, puede precipitar o exacerbar la tirotoxicosis; esto puede evitarse mediante la administracin previa de un frmaco
antitiroideo. Dx: Niveles de TSH bajos, T4 N o mnimamente elevado. Gammagrafa de la tiroides muestra una captacin heterognea
con zonas mltiple de aumento y disminucin de la captacin. TUMORES TIROIDEOS: Benignos=adenomas, malignos =carcinomas.
Incidencia mundial de 0.5-10 casos por 100 000, ms en mujeres, entre 40-50 aos, peor pronstico en extremos de la vida. Factores de
riesgo: radiacin a cabeza y cuello en la infancia, ndulo tiroideo de 2-4 cm, signos de fijacin a estructuras vecinas, invasin a ganglios
linfticos, parlisis de cuerdas vocales, antecedente de neoplasia endocrina mltiple tipo 2 (NEM2). Dentro de los carcinomas hay
foliculares, papilares y anaplasicos de tiroides (raro, muy maligno y fatal). Carcinoma papilar tiroideo subtipo ms frecuente (70-90%) y
menos agresivo, es multicentrico, el antecedente principal es radiacin a cuello en la infancia, generalmente asintomtico en ocasiones
disfagia y voz ronca, sospecharlo en ndulos solitarios palpables.

CASO CLINICO
Acude a la consulta externa paciente femenino de 25 aos de edad, acude debido a que no produce suficiente leche para lactar tiene 8
dias de puerperio, niega otra sintomatologia, se le explica la tcnica para lactacin, sin embargo a la exploracin se observa incremento
simetrico de glandula tiroides, no se palpan ndulos ni zonas dolorosas, no se identificas cambios de voz, la paciente es originaria de
Chiapas.

PREGUNTA
Cual la conducta a seguir mas adecuada para el caso.

RESPUESTA
a.- Realizar valoracin de THS y T4.
b.- Indicar suplementacion de yodo.
c.- Indicar alimentacin rica en yodo, cita 6 meses.
d.- Solicitar perfil de anticuerpos antitiroideos.

PREGUNTA
6 meses despus se envio a endocrinologa debido a que presento incremento de peso, se mantuvo las mismas caractersticas de
volumen y consistencia de la glandula tiroidea, fue realizado un perfil tiroide con los siguientes resultados T4 total baja, TSH normal e
indico 100 mcg/dia, considerando la evolucin, cual de los siguientes diagnostico diferenciales se debe causas no tiroideas.

RESPUESTA
a.- Sindrome de T4 baja.
b.- Bocio multinodular no toxico.
c.- Bocio multinodular toxico.
d.- Nodulo solitario hiperfuncionante.

CASO CLINICO
Paciente diagnosticada de hipotiroidismo subclnico y tratada adecuadamente con tiroxina oral. Unos meses despus desarrolla
exoftalmos bilateral y simultneamente presenta anticuerpos sricos anti-receptor de TSH positivos. Se suspende entonces el
tratamiento con tiroxina, y se comprueba que el hipotiroidismo ha progresado hasta hacerse primario. Se establece el diagnstico de
enfermedad de Graves hipotiroidea.

PREGUNTA
Cual es la complicacin mas frecuente a largo plazo para el paciente debido al tratamiento que se debe instaurar.

RESPUESTA
a.- Osteoporosis.
b.- Arteriosclerosis.
c.- Edema pulmonar.
d.- Dislipidemia.

CASO CLINICO
Acude a consulta de revisin anual paciente femenino de 31 aos, la cual cuenta con el diagnostico de bocio simple, refiere que ha
presentado auscencia de periodo menstrual en 4 ocaciones en el aos, adems de presentar infecciones de vas areas altas en 2
ocaciones, ha presentado cansancio, debilidad generalizada, incremento de peso. A la exploracin fsica se observa bradipsiquia y
bradilalica, tiene un aspecto seco y aspero. Los signos vitales fueron TA 110/70 mmHg.

PREGUNTA
Se ingreso a la paciente debido al malestar generalizado e hipotermia, cual es la conducta inmediata a seguir para establecer un
diagnostico presuntivo.

RESPUESTA
a.- BH, QS, EGO.
b.- Solicita TSH, T4.
MANUAL DE TRABAJO DEL CURSO ENARM CMN SIGLO XXI
CURSO ENARM CMN SIGLO XXI TEL: 36246001 Pharmed Solutions Institute PGINA 58

c.- Solicitar Anticuerpos peroxidasa tiroideos.
d.- Biopsia de tiroides.

PREGUNTA
Se obtuvieron los siguientes resultados hematocrito 32 %, hemoglobina 10.1, Leucocitos 5,900, glucosa 219 central, colesterol 310.
Considerando los resultados cual es la conducta a seguir ms adecuada.

RESPUESTA
a.- Iniciar levotiroxina 150 mcg/ dia.
b.- Solicitar perfil tiroideo con anticuerpos.
c.- Estabilizacion y posterior envio a segundo nivel.
d.- Administracin de prednisona 20 mg y levotiroxina 100 mcg/dia.

PREGUNTA
2 meses despus regresa la paciente para control por el primer nivel de atencin, en su nota de contrarreferencia se encuentra el
diagnostico de tiroiditis de Hashimoto, considerando la evolucin del paciente, cual de las siguientes comorbilidades es la menos
probable.

RESPUESTA
a.- Diabetes mellitus.
b.- Enfermedad de Addison.
c.- Anemia perniciosa.
d.- Infiltracion leucocitaria hipofisiaria.

CASO CLINICO
Se trata de una mujer de 89 aos con Hipertensin Arterial, Cardiopata Hipertensiva, Fibrilacin Auricular, Insuficiencia Mitral y Artica
moderadas, Enfermedad Cerebral de pequeo vaso y Sndrome Hipoventilacin Obesidad y Colecistectoma. Precisa varios ingresos
hospitalarios por cuadros de FA rpida, Ictus lacunar, Insuficiencia Cardiaca e intoxicacin por Digoxina. Se observa deterioro progresivo
de la Funcin Renal. La Ecografa realizada muestra atrofia renal izquierda (dimetro de 76 mm con desestructuracin crtico-medular)
y rin derecho de 118 mm con moderada atrofia cortical. En este momento se diagnostica de Enfermedad Renal Crnica grado 4. Se
detecta hipotiroidismo primario debido a bocio multinodular por tiroiditis autoinmune.

PREGUNTA
Considerando la comorbilidad cual es la patologa que complica el uso de levotiroxina.

RESPUESTA
a.- Fibrilacion auricular.
b.- Hipertensin arterial.
c.- Deterioro de la funcin renal.
d.- Insuficiencia cardiaca.

CASO CLINICO
Paciente de 44 aos de edad con sndrome de Down que acude a urgencias por cuadro de disnea progresiva de 2 meses de evolucin,
hasta hacerse de reposo, y aumento del permetro abdominal. Analticamente se objetiva: TSH, 18.890 U /ml y T4L: < 0,15 ng/dl. El
ecocadiograma muestra derrame pericrdico severo con signos de compromiso hemodinmico y colapso de cavidades derechas
confirmado posteriormente con tomografa computarizada toracoabdominal con contraste. En la UCI se realiza pericardiocentesis y
evacuacin de 2.400 ml de lquido seroso; los estudios bioqumicos, serolgicos, micro-biolgicos y de inmunidad son negativos. Se
comienza tratamiento sustitutivo con levotiroxina con buena evolucin posterior

PREGUNTA
Cual el pronostico mas probable de este caso.

RESPUESTA
a.- Cardiopatia dilatada.
b.- Bloqueo auricular.
c.- Diestres respiratorio agudo.
d.- Hipotiroidismo subclinico.

CASO CLINICO
Nia de 12 aos de edad, que desde hace 4-5 meses inicia con nerviosismo, taquicardia, intolerancia al calor, sudoracin profusa e
impulsividad para comer, sin prdida de peso ni insomnio. Asimismo, refiere abultamiento en la parte anterior del cuello, sin dolor ni
otros signos inflamatorios asociados. Su madre tiene hipotiroidismo en tratamiento con L-tiroxina desde hace 7 aos. En la exploracin
fsica destaca una protrusin ocular bilateral y un bocio grado II, de consistencia elstica, sin dolor ni otros signos inflamatorios. El
estudio de hormonas tiroideas mostr hipertiroidismo con anticuerpos antitiroideos elevados. La ecografa tiroidea objetiv ambos
lbulos tiroideos aumentados de tamao (derecho 18 x 18,8 x 53mm, izquierdo 15,9 x 18 x 51mm), heterogneos, con mltiples zonas
hipoecognicas de bordes imprecisos que confluyen y una marcada hipervascularidad con Doppler.
MANUAL DE TRABAJO DEL CURSO ENARM CMN SIGLO XXI
CURSO ENARM CMN SIGLO XXI TEL: 36246001 Pharmed Solutions Institute PGINA 59


PREGUNTA
Cual es la medida teraputica mas apropiada a largo plazo.

RESPUESTA
a.- Tiroidectomia parcial.
b.- Iodo radioactivo.
c.- Propiltiouracilo.
d.- Metimazol.

CASO CLINICO
Mujer de 66 aos de edad, que ingresa por proteinuria, edema generalizado e hipertensin de reciente comienzo, refractaria a
tratamiento. En la analtica destacaban: urea 78 mg/dl, creatinina 1,1 mg/dl; aclaramiento de creatinina de 48 ml/min; protenas totales
4,6 g/dl; albmina 1,2 g/dl. Perfil proteico compatible con sndrome nefrtico. Colesterol: 484 mg/dl. Triglicridos: 180 mg/dl. LDL: 386
mg/dl. Autoinmunidad y complemento dentro de la normalidad. Inmunocomplejos circulantes negativos. TSH: 10,17 mU/l, T4: 0,78
mg/dl. Anticuerpos antimicrosomales: 84 U/ml. Anticuerpos antitiroglobulina: 4 U/ml. Anticuerpos antirreceptor TSH: 1,6 U/l.
Proteinuria hasta 10 g/24 hrs. Se realiz biopsia renal eco-dirigida con resultado anatomopatolgico de glomerulonefritis membranosa.
Ante el diagnstico de glomerulonefritis membranosa probablemente secundaria a tiroiditis de Hashimoto, se comenz tratamiento
mediante levotiroxina, estatina y doble bloqueo del sistema renina-angiotensina-aldosterona.

PREGUNTA
Cual es la condicin que genera el edema en este caso.

RESPUESTA
a.- Proteinuria.
b.- Presion hidrosttica.
c.- Glomerulonefritis.
d.- Bloque ECA.

CASO CLINICO
Femenino de 28 aos de edad que acude a urgencias debido a que presenta irritabilidad, ideas delirantes de dao, taquicardia,
diaforesis, taquipnea, hipertermia, opresin torcica, vomito y diarrea as como hipotensin arterial, adems presenta dolor abdominal
difuso, peristaltismo presente, la paciente presenta piel caliente, delgada, pelo quebradizo y con cada de este, al llegar a urgencias se
observa en delirium, los familiares acompaantes refieren que la paciente tiene antecedentes de enfermedad endocrinolgica y se
encontraba en espera de reseccin de ndulos tiroides.

PREGUNTA
Usted considera por el cuadro clnico y el antecedente referido que se trata de una crisis tirotoxica, usted debe decidir cul es el
tratamiento ms adecuado de primera instancia:

RESPUESTA
a.- Emplear rodio radioactivo inmediatamente.
b.- Usted indica dexametazona.
c.- Emplea metoprolol y metimazol.
d.- Decide indicar Propiltiouracilo.

GLANDULAS PARATIROIDES
CIENCIAS BASICAS: Las glndulas producen hormona paratiroidea (PTH), la cual controla las concentraciones de calcio y fosforo en la
sangre, incrementa la resorcin sea, aumenta la recaptacin de calcio y disminucin de fosfato, incrementa la absorcin
gastrointestinal de calcio, e induce la conversin renal de la vitamina D hasta su forma activa. HIPERPARATIROIDISMO PRIMARIO
(HPP): Trastorno endocrino ms frecuente (despus de DM), caracterizado por secrecin excesiva de hormona paratiroidea, que
condiciona a hipercalcemia crnica, predomina en mujeres que en hombres (4:1), entre 4ta y 5ta dcada de la vida, origen
desconocido, se ha descrito asociacin con irradiacin ionizante de cuello, las causas incluyen adenoma 85%, hiperplasia primaria
difusa o nodular 10-55%, y por ltimo el carcinoma <1%. El adenoma paratoiroideo: frecuentemente asociado a patrn familiar,
espordico por inversin del cromosoma 11 y mutacin del gen MEN-1, suelen ser masas nicas de color rojizomarrn con capsula
bien definida, principalmente de cel. prIncipales y solo afectan una glndula. Hiperplasia primaria: Suele afectar las 4 glndulas, se
manifiesta por aumento de las clulas prIncipales, los hallazgos clsicos de hiperplasia y la presencia de tejido adiposo es caracterstico:
Carcinoma de paratiroides: infrecuente ms comn en la 6ta. Dcada de la vida, bulto en cuello, grande (12g) dificultad al hablar o
tragar, astenia y es muy comn este asociado a hipercalcemia severa. PATOGENIA: Existen 3 mecanismos: cambio del punto de
inflexin del sensor de calcio, mayor proporcin de cel. Activas reflejando la mayor actividad secretora, y un aumento del nmero total
de cel. Parenquimatosas. DIAGNOSTICO: Presentacin de 2 formas principales; la que cursa con afeccin renal y la que manifiesta
principalmente una participacin sea. Cuadro clnico: cefalea frecuente, depresin, HTA, incapacidad para concentrarse, hipercalcemia
(22%), normocalcemico (8%). sea; dolor, deformidad quistes y susceptibilidad a las fracturas, lo clsico es la ostetis fibrosa qustica.
Renal; nefrolitiasis (38%), nefrocalcinosis, disminucin del filtrado glomerular y diversas anomalas tubulares, acidosis tubular. Otras
manifestaciones derivan de la hipercalcemia crnica; debilidad muscular proximal, fatiga fcil y atrofia, depresin, cambios de la
personalidad y psicosis, estreimiento, pancreatitis, osteoartritis degenerativa y avulsin de tendones. Laboratorio: El HPP, es la causa
MANUAL DE TRABAJO DEL CURSO ENARM CMN SIGLO XXI
CURSO ENARM CMN SIGLO XXI TEL: 36246001 Pharmed Solutions Institute PGINA 60

mas frecuente de elevacin de calcio srico ambulatorio, hay que recordar la asociacin de HPP a tumores, la determinacin de PTH en
plasma es el mtodo dx., definitivo para discriminar un HPP de otras causas de hipercalcemia. Las nicas hipercalcemias de origen no
paratiroideo, donde la PTH esta elevada son los tratamientos con litio o diurticos tiazidicos. El aumento de calcio filtrado a travs del
glomrulo conduce a hipercalciuria y aunque la PTH produce un aumento de la reabsorcin tubular de calcio, el resultado de ambos
factores contradictorios condiciona a la aparicin de calcio en orina, que justifica la mayor frecuencia de litiasis. Rx de torax (masa
mediastinica), densitometra sea, TAC sesibilidad de 65%. TRATAMIENTO: Indicaciones de tx. mdico inicial; calcemia inferior a
11.5mg/ml, ausencia de sntomas en relacin directa con la enfermedad o contraindicacin para ciruga. Aumentar ingesta hdrica,
reducir calcio de la dieta y movilizacin inmediata, suprimir frmacos como diurticos tiazidicos y carbonato de litio. Se pueden usar
calcitonina y bifosfonatos orales (para inhibir los efectos de la PTH), para disminuir la absorcin intestinal de calcio y en nefrolitiasis, es
til la celulosa fosfato sdica. El tx., de HPP es exclusivamente quirrgico, sobre todo en sus fases sintomticas con hipercalcemia o
complicaciones viscerales. Son indicaciones absolutas de ciruga; calcemia >11mg7dl, hipofosfatemia grave, disminucion de la funcin
renal, urolitiasis metablicamente activa, sntomas objetivos de hipercalcemia , hipercalciuria >400mg/dl, calcificacin de tejidos
blandos, evidencia radiogrfica de ostetis fibroquistica, ulcera pptica o pancreatitis recidivante. HIPOPARATIROIDISMO: El
hipoparatiroidismo es un trastorno causado por la hipofuncin de las glndulas paratiroides, caracterizada por una muy baja
concentracin de hormona paratiroidea (PTH), de calcio y un aumento en la concentracin de fsforo sanguneo. La PTH es la hormona
encargada de mantener los niveles normales de calcio en el suero sanguneo, favoreciendo su absorcin en el tracto digestivo por
medio de la activacin de la vitamina D3 en el rin con su subsiguiente migracin hacia en epitelio intestinal y la resorcin sea
aumentando indirectamente la actividad de los osteoclastos en el hueso, aparte del incremento de la captacin por parte de los tbulos
distales de las nefronas que reabsorben este mineral. CAUSAS: Esta patologa puede ser producida por una variada gama de factores,
entre los que encontramos: La extirpacin quirrgica accidental o intencional durante la tiroidectoma, siendo esta la causa ms
frecuente. Dentro de las causas no quirrgicas los procesos autoinmunes son los ms importantes, encontrando dentro de ellos el
sndrome poliglandular autoinmune La Ausencia o disfuncin congnita de la glndula paratiroides la cual se ha relacionado con
microdelecciones dentro del cromosoma 22q11 (sndrome de DiGeorge). Las deficiencias de magnesio. La Hemocromatosis, causante
de un cuadro de disfuncin de los rganos endocrinos debido al acumulo de hierro en el interior de las clulas. Idiopticos. PATOLOGA:
Siendo el calcio uno de los iones ms importantes en el mantenimiento de la homeostasia normal del cuerpo, adems de estar con
implicado en la contraccin del msculo esqueltico estriado, liso y cardiaco, cualquier variacin en sus concentracin desencadena un
alto nmero de desordenes funcionales todo esto producido en el caso del hipoparatiroidismo por la disminucin del potencial de
accin, lo que lleva al aumento de la excitabilidad celular, favoreciendo en el caso del msculo la aparicin de tetania y el SNC de
convulsiones. Adems de esto el calcio est relacionado con los procesos de secrecin glandular favoreciendo la unin de las vesculas
de secrecin a la pared celular promoviendo as su exocitosis, la disminucin de este mineral afecta significativamente este suceso lo
que se ve dilucidado en la reduccin de la sudoracin que presentan los paciente que tienen este proceso patolgico. CLNICA: La
aparicin de sntomas varan entre una persona y la otra, en especial en los nios, sin embargo, los ms frecuentes pueden incluir:
Disnea, Dolor clico agudo, Hormigueos, Contracciones de distintos grupos musculares. Son muy caractersticos las localizadas en
algunos msculos del brazo y mano. Esta posicin incluso puede ser provocada estimulando los msculos del brazo mediante una
compresin con el manguito de un esfigmomanmetro, lo que se conoce como el signo de Trousseau. Tambin aparecen
frecuentemente cataratas, trastornos psquicos, y piel seca y otros trastornos trficos de la piel. Los trastornos mentales ms
frecuentes de la hipocalcemia aguda causada por el hipoparatiroidismo es el delirium y la psicosis. En la hipocalcemia crnica son ms
frecuentes los sntomas de ansiedad, irritabilidad, labilidad emocional, depresin, psicosis, alteracin de la memoria y de la
concentracin, deterioro cognitivo y a veces retraso mental en funcin de la edad de comienzo. El compromiso intelectual aparece en
el 50% de los casos de hipoparatiroidismo. Variantes clnicas: Pseudohipoparatiroidismo: En este cuadro, adems de los datos descritos,
se aaden trastornos constitucionales como talla corta, cara ancha, y metacarpianos y metatarsianos cortos. Pseudo-
pseudohipoparatiroidismo: En el que slo aparecen los trastornos constitucionales descritos, sin alteracin del calcio. Tratamiento: El
tratamiento consistira en la administracin de la hormona paratiroidea (PTH) pero principalmente de la administracin de calcio y
vitamina D en caso de que se trate de la verdadera forma del hipoparatiroidismo la cual presenta bajos niveles de calcio srico.

CASO CLINICO
Mujer de 77 aos que consulta por disnea progresiva de medianos esfuerzos en el ltimo ao, ortopnea e hinchazn de los miembros
inferiores. Entre sus antecedentes no haba factores de riesgo cardiovascular, no tena hbitos txicos ni reciba ninguna medicacin. El
examen fsico revel una presin arterial de 160/100 mmHg, leve cianosis perioral e ingurgitacin yugular hasta ngulo mandibular. No
tena bocio ni adenopatas perifricas. Los tonos cardacos eran rtmicos, sin soplos, y en la auscultacin pulmonar haba crepitantes
bibasales. Tena leves edemas maleolares bilaterales. La exploracin neurolgica era normal, con negatividad de los signos de
Trousseau y Chvostek. En el hemograma destacaba macrocitosis y en la bioqumica, un calcio corregido con la albmina de 4,2 mg/dl,
fsforo 8 mg/dl, magnesio 1,6 mg/dl y la funcin renal normal. La CPK fue de 739 con CPK-Mb normal, GOT 305 y LDH de 1.038.

PREGUNTA
Cual de las siguientes manifestaciones esperara mas frecuente encontrar en el caso?

RESPUESTA
a.- QT estrecho.
b.- QT alargado.
c.- T picudas.
d.- T aplanada.

CASO CLINICO
Varn de 21 aos, con cefalea y vmitos de 24h de evolucin. En la EF slo se encontr un discreto edema y eritema amigdalar.
Leucocitosis con desviacin izquierda (14.700leucocitos/l con 85% neutrfilos). A las 2448h del ingreso, el cuadro clnico empeor
MANUAL DE TRABAJO DEL CURSO ENARM CMN SIGLO XXI
CURSO ENARM CMN SIGLO XXI TEL: 36246001 Pharmed Solutions Institute PGINA 61

con la aparicin de obnubilacin y somnolencia. En la nueva analtica, destacaba una calcemia de 15,8mg/dl. Se administr
fluidoterapia, corticoides y aporte adicional de potasio y magnesio, evolucionando favorablemente, con recuperacin del estado de
consciencia. El electrocardiograma se normaliz y la sintomatologa digestiva desapareci en 24h. PTH 237pg/ml (965), calciuria
290mg/24h (535), magnesio srico 1,63mg/dl (1,692,29), creatinina 0,46mg/dl (0,530,79).

PREGUNTA
Cual es el diagnostico mas probable?

RESPUESTA
a.- Adenoma de paratiroides.
b.- Hiperplasia de paratiroides.
c.- Carcinoma de paratiroides.
d.- Sindrome paraneoplasico.

PREGUNTA
En caso de la presencia de sndrome paraneoplasico, cual es la fuente mas frecuente?

RESPUESTA
a.- Rion
b.- Pulmon
c.- Cabeza
d.- Cuello

CASO CLINICO
Mujer de 38 aos con IRC secundaria a nefropata tubulointersiticial crnica. A su ingreso destacaban niveles sricos de calcio de 12
mg/dL, Fsforo 6,6 mg/dL, FA 2.490 UI/L, PTHi 1.450 pg/ml. Destacaba la presencia de una tumoracin cervical de 3 x 2 x 1 cm situada
en la zona inferior del lbulo derecho de la glndula tiroides con 4 ganglios linfticos palpables. Radiolgicamente se observaba
resorcin subperistica a nivel de las falanges proximales de ambas manos y una estructura granular a nivel del crneo. Se ingres por
un cuadro de astenia, debilidad muscular y dolores seos generalizados.

PREGUNTA
Cual es la conducta a seguir mas adecuada para el caso clnico?

RESPUESTA
a.- Hidroxido de aluminio.
b.- Quelantes clcicos.
c.- Vitamina D.
D. Paratiroidectoma.

TRASTORNOS DE GLANDULAS SUPRARRENALES
CIENCIAS BASICAS: Las suprarrenales estn compuestas de medula y corteza est dividida en glomerular (produce aldosterona-regula
Na y vol. Extracelular por medio de la retencin de H2O, Na y la excrecin renal de K), fascicular (produce cortisol-aumenta niveles de
glucosa, metabolismo de protenas y lpidos, efecto inmunosupresor, antiinflamatorio, acta en SNC) y reticular (produce cortisol y
esteroides andrognicos: dihidroepiandrostendiona y androstendiona). En mdula las clulas cromafines secretan adrenalina y
adrenalina, cuando la medula produce catecolaminas en exceso conduce a feocromocitoma. HIPERFUNCION DE GLNDULAS
SUPRARRENALES: SINDROME DE CUSHING: La causa ms comn es la iatrognica, debido a la administracin de glucocorticoides. El
Cushing endgeno resulta del exceso de produccin de cortisol (y otras hormonas esteroideas). La mayor causa de hiperplasia
suprarrenal bilateral secundaria es la hipersecrecin de Hormona adrenocorticotropica (ACTH) por la pituitaria (Enf. de Cushing) o de
fuentes ectpicas tales como carcinoma de cel. Pequeas de pulmn, carcinoma medular de tiroides o tumores de timo, pncreas,
ovario. Los adenomas y carcinomas de glndulas suprarrenales representan aproximadamente el 25% de los casos de sndrome de
Cushing. CUADRO CLNICO: Ms comn; obesidad central, HTA, osteoporosis, psicosis, acn, amenorrea y DM, pero inespecficas, ms
especficos incluyen hematomas, estras prpura, miopata proximal, la deposicin de grasa en la cara y las zonas interescapular (facies
de luna y joroba de bfalo), y virilizacin, piel delgada y frgil. La hipopotasemia y alcalosis metablica son prominentes, sobre todo con
la produccin ectpica de ACTH. DIAGNSTICO: Se requiere de demostracin del incremento de cortisol y supresin anormal de
cortisol en respuesta a dexametasona. Para el cribado, medir cortisol libre urinario en 24 hrs. TRATAMIENTO: Para adenoma o
carcinoma suprarrenal requiere escisin quirrgica, dar glucocorticoides antes y despus de la operacin, para regular el estrs.
Metstasis y carcinomas irresecables se tratan con mitotano con incrementos graduales de 6g por da dividido c/8-6h. En ocasiones la
citoreduccin de carcinoma de pulmn, o reseccin de otros tumores, podemos remitir el Sx. de Cushing ectpico. Si las fuentes de
ACTH no pueden ser resecadas, hacer adrenalectoma bilateral total administrar ketoconazol (600-1200mg/dl), metirapona (2-3g/d),
mitotano (2-3mg/d), podran aliviar las manifestaciones por exceso de cortisol. HIPERALDOSTERONISMO: Es causada por
hipersecrecin de aldosterona adrenal. PRIMARIO hipersecrecin autnoma de aldosterona (suprarrenal) con supresin de los niveles
de renina, puede ser debido a un adenoma suprarrenal 35% (productor de aldosterona=Sx. de Conn) o hiperplasia suprarrenal bilateral
60%, patogenia: aldosterona produce retencin renal de sodio y prdida de potasio. Esto se traduce en la expansin de contenido de
sodio del cuerpo, lo que lleva a la supresin de la sntesis de renina renal. La accin directa de la aldosterona en la nefrona distal
provoca la retencin de sodio y prdida de hidrgeno y los iones de potasio, lo que resulta en una alcalosis hipocalemia, la aldosterona
tiene efectos fisiopatolgicos en un rango de otros tejidos, causando fibrosis cardiaca, disfuncin vascular endotelial y nefroesclerosis.
MANUAL DE TRABAJO DEL CURSO ENARM CMN SIGLO XXI
CURSO ENARM CMN SIGLO XXI TEL: 36246001 Pharmed Solutions Institute PGINA 62

El SECUNDARIO ocurre secundario a elevacin de los niveles de renina circulante, esto es tpico en falla cardiaca, cirrosis, sndrome
nefrtico, pero podra ser debido a estenosis de la arteria renal o tratamiento diurtico, muy raro reninoma. CUADRO CLINICO:
Primario; tienen dolores de cabeza e hipertensin diastlica. El edema es caractersticamente ausente, a menos que la insuficiencia
cardaca congestiva o la enfermedad renal est presente. Las prdidas de potasio urinario, puede causar debilidad muscular y fatiga,
aunque los niveles de potasio pueden ser normales en aldosteronismo primario suave. Tambin se puede producir alcalosis metablica
e hipernatremia. DIAGNOSTICO: Se sugiere por la hipertensin y es asociado con hipocalemia persistente, en un paciente sin edema,
que no este recibiendo diurticos ahorradores de K. TRATAMIENTO: La ciruga puede ser curativa en adenoma adrenal, pero no efectiva
en hiperplasia suprarrenal, que se maneja con restriccin de Na y espironolactona. El secundario; es tratado con restriccin de sal y
correccin de la causa subyacente. HIPOFUNCIN DE GLNDULAS SUPRARRENALES: La insuficiencia suprarrenal primaria es debida a
falla de la glndula suprarrenal, mientras la insuficiencia suprarrenal secundaria es debida a falla de la produccin o liberacin de ACTH.
ENFERMEDAD DE ADDISON: Ocurre cuando ms >90% del tejido suprarrenal est destruido quirrgicamente, por enf. granulomatosa
(Tb, histoplasmosis, coccidioidomicosis, criptococosis), va autoinmune, metstasis bilateral, hemorragia bilateral, CMV,HIV,
Amiloidosis, sarcoidosis. CUADRO CLINICO: Fatiga, debilidad, anorexia, nausea y vmito, prdida de peso, dolor abdominal,
pigmentacin cutnea y mucosa, deseo de sal, hipotensin y ocasionalmente hipoglicemia, si hay deplecin del fluido extracelular se
acenta la hipotensin. DIAGNOSTICO: La mejor prueba de deteccin es la respuesta del cortisol 60 min despus de 250 g de ACTH IV
o IM. Los niveles de cortisol deben exceder de 18 mg / dl 30 a 60 minutos despus de ACTH. Si la respuesta es anormal, la deficiencia
primaria y secundaria se puede distinguir mediante la medicin de aldosterona en sangre. TRATAMIENTO: Hidrocortisona 20-30mg/d,
dividido 2/3 en la maana y 1/3 en la tarde es el pilar de la sustitucin de glucocorticoides. La sustitucin de mineralocorticoides es
necesaria en la insuficiencia suprarrenal primaria. Durante la crisis suprarrenal se usan dosis altas de hidrocortisona (10mg/h continuas
IV o 100mg bolo IV tres veces al da). HIPOALDOSTERONISMO: Deficiencia aislada de aldosterona. Con produccin normal de cortisol,
se produce con hiporeninismo, como un defecto biosinttico hereditario, despus de la extirpacin de los adenomas secretores de
aldosterona, y durante el tratamiento prolongado con heparina. Hipoaldosteronismo hiporreninmico es ms frecuente en adultos con
insuficiencia renal leve y la diabetes mellitus en relacin con la hiperpotasemia desproporcionada. Fludrocortisona oral, restablece el
equilibrio electroltico si la ingesta de sal es adecuada. En IRC y falla cardiaca est aprobado el uso de furosemide. MASAS
SUPRARRENALES INCIDENTALES (INCIDENTALOMA): Las masas suprarrenales son comn encontrarlas en TAC o RMI. La mayora (70-
80%), son no funcionantes y la probabilidad de un carcinoma suprarrenal es baja (<0.01%). El primer paso es determinar el estado
funcional y medir metanefrinas libres en plasma para deteccin de feocromocitoma. Si hay un tumor maligno conocido, hay 30-50%de
posibilidades que el incidentaloma sea una metstasis. Evaluacin hormonal adicional debe incluir la prueba durante la noche
dexametasona 1mg supresin de todas los puntos, la renina plasmtica, actividad / aldosterona ratio en pacientes hipertensos, DHEAS
en mujeres con signos de exceso de andrgenos y estradiol en hombres con feminizacin. FEOCROMOCITOMA: Son tumores
adrenomedulares que secretan catecolaminas, son raros, forman <0.1% de las causas de hipertensin, sin embargo es muy importante
su diagnstico debido a: el desarrollo de una crisis hipertensiva fatal, la reversin de todas las manifestaciones despus de remover el
tumor quirrgicamente, la falta de eficacia a largo plazo de tratamiento mdico, la apreciable incidencia de malignidad.
Manifestaciones; hipertensin resistente a tx. convencional, sudoracin e intolerancia al calor (80%), palidez o enrojecimiento,
sentimiento de aprensin, pirexia, dolor de cabeza palpitante y constante (65%), parestesias, disturbios visuales, palpitaciones (65%),
dolor de pecho, hipotensin postural. La ciruga es curativa en la mayora de los pacientes. USOS CLINICOS DE GLUCOCORTICOIDES:
Son usados en una variedad de enfermedades tales como asma, artritis reumatoide y psoriasis. Es casi segura la aparicin de
complicaciones (ganancia de peso, HTA, fascies cushinoide, DM, osteoporosis, miopata, incremento de presin intraocular, necrosis
sea isqumica, infecciones e hipercolesterolemia), por ello debe valorarse riesgo-beneficio. Estos efectos secundarios pueden
minimizarse mediante la eleccin cuidadosa de preparaciones de esteroides, alternando das o interrumpiendo terapias; el uso de
esteroides tpicos, inhalados, intranasales o drmicos siempre que sea posible. Altas dosis de estos pueden ser requeridas durante
periodos de estrs.

CASOS CLINICOS
Paciente de 49 aos de edad de sexo masculino con hipertensin severa, cambios de humor, inestabilidad, alternando entre la
agresividad y la depresin. Se observ debilidad muscular progresiva, oscurecimiento de la piel y aumento de la grasa abdominal. Sus
sntomas y pruebas de laboratorio confirmaron un sndrome de Cushing dependiente de ACTH.

PREGUNTA
Cual es la conducta a seguir para determinar el origen?

RESPUESTA
a.- Radiografia de torax.
b.- Tomografia computarizada abdomen.
c.- Resonancia magnetica selar.
d.- USG suprarrenal.

CASO CLINICO
Femenino de 49 aos de edad, hipertensa con debilidad generalizada que implica sobre todo sus piernas. Con potasio srico 1.8mEq/L y
creatina fosfoquinasa 1753 U / L. En ese momento, se haba observado aldosterona (336.73pg/ml ). La TAC abdominal mostr tumores
suprarrenales bilaterales, homogneos, hipodensas, derecha 2,29 cm y 0,92 cm de dimetro. Recibi adrenalectoma laparoscpica
derecha y un tumor de 2x2 cm fue extirpado.

PREGUNTA
Cual de las siguientes alteraciones ECG es mas probable observar en este caso?

MANUAL DE TRABAJO DEL CURSO ENARM CMN SIGLO XXI
CURSO ENARM CMN SIGLO XXI TEL: 36246001 Pharmed Solutions Institute PGINA 63

RESPUESTA
a.- QT corto.
b.- T invertida.
c.- U normal.
d.- P aplanada.

PREGUNTA
Cual de las siguientes alteraciones acido-base es mas probable observar en esta patologia?

RESPUESTA
a.- Alcalosis hipocalemica.
b.- Alcalosis hipocalcemica.
c.- Acidosis hipocalemica.
d.- Acidosis hipocalcemica.

CASO CLINICO
Un hombre de 53 aos de edad con antecedente de cncer de colon tratado y dos masas en glndulas suprarrenales de 1 y 2 cm
respectivamente, el paciente presenta hipertensin sin control adecuado tratado con amlodipino y telmisartan. Ingreso por cefalea leve
persistente, con TA 160/90 mmHg, con incremento de los ROTs, no se observa edema. Se observo potasio serico de 2.6.

PREGUNTA
Cual es el diagnostico mas probable del presente caso?

RESPUESTA
a.- Aldosteronismo secundario
b.- Aldosteronismo primario.
c.- Pseudoaldosteronismo.
d.- Pseudo-pseudoalteronismo

CASO CLINICO
Mujer de 29 aos de edad, con hipertensin no controlada e hipopotasemia espontnea desde hace 4 aos. En la investigacin de las
causas secundarias de la hipertensin, la relacin de actividad de la renina-aldosterona se elev en dos ocasiones separadas,
diagnosticando como hiperaldosteronismo primario.

PREGUNTA
Cual de las siguientes pruebas confirman el diagnostico?

RESPUESTA
a.- Infusion de solucin glucosada.
b.- Infusin de solucin salina.
c.- Infusion de solucin mixta.
d.- Infusion de soluciones hipertnicas.

CASO CLINICO
Una mujer de 68 aos de edad con nivel de calcio srico de (2,95 mM, el valor normal: 2,25 a 2,75 mM) con un bajo nivel de potasio
(1,7 mM, el valor normal: 3.5 a 5.3 mM), un nivel de aldosterona plasmtica elevada (0.244 nm, el valor normal: 0,014 a 0,083 Nm), y
un bajo niveles de renina plasmtica normal (0.021 pM, el valor normal: desde 0,008 hasta 0,3 pM). TC adrenal mostr un ndulo
redondo de 1 cm en la glndula suprarrenal izquierda (1,1 x 1,0 cm), sugestivo de adenoma suprarrenal.

PREGUNTA
Cual es el diagnostico mas probable de este caso?

RESPUESTA
a.- Hiperaldosteronismo primario.
b.- Hiperaldosteronismo secundario.
c.- Pseudoaldosteronismo.
d.- Hiperaldosteronismo terciario.

PREGUNTA
La paciente se neg a un procedimiento quirrgico, cual es una medida teraputica mas adecuada?

RESPUESTA
a.- Indicar amlodipino.
b.- Indicar espironolactona.
c.- Indicar ECA.
d.- Indicar furosemida.
MANUAL DE TRABAJO DEL CURSO ENARM CMN SIGLO XXI
CURSO ENARM CMN SIGLO XXI TEL: 36246001 Pharmed Solutions Institute PGINA 64

CASO CLINICO
Paciente de sexo masculino, de 63 aos de edad, con antecedentes de tabaquismo activo (40 paquetes/ao), hipertensin arterial
crnica y diabetes mellitus tipo 2 en tratamiento con hipoglicemiantes orales, cardiopata coronaria y enfermedad arterial oclusiva
crnica de extremidades inferiores. Consult por cuadro de un ao de evolucin de astenia, adinamia, baja de peso, hiperpigmentacin
de piel y mucosas, episodios de lipotimia e hipoglicemia sintomtica a pesar de disminucin de la terapia hipoglicemiante y
antihipertensiva.

PREGUNTA
Cul es la conducta a seguir para establecer el diagnostico.

RESPUESTA
a.- Cuantificacion de cortisol.
b.- Cuantificacion de ACTH.
c.- Cuantificacion de CRF.
d.- Cuantificacion de DEAS.

CASO CLINICO
Hombre de 53 aos de edad, que consulto por nuseas, vmito y diarrea, en el examen fisico se detecto hipotensin sostenida con
presiones sistlicas en el rango de 60 a 70 mmHg. Por lo cual se inicio hidratacin intravenosa, sin embargo, el paciente persisti con
una presin arterial de 70/42 mmHg. Los exmenes paraclnicos documentaron hipoglucemia, hiponatremia e hiperkalemia, por lo cual
se hace la impresin diagnstica de Enfermedad de Addison.

PREGUNTA
Cual es la conducta a seguir mas apropiada en el caso?

RESPUESTA
a.- Soluciones meatabolicas y corticoides.
b.- Soluciones normotonica y corticoides.
c.- Soluciones fisiolgicas, catecolaminas y glucocorticoides.
d.- Soluciones metabolicas, catecolaminas y glucocorticoides.

CASO CLINICO
Paciente masculino de 62 aos con antecedentes de ser fumador, que dos meses atrs haba comenzado con prdida del apetito y
fiebre por las tardes de 38 C, Mucosas hmedas hipercoloreadas, tonos cardiacos rtmicos. FC 79 x. TA 160/100. Ultrasonido
abdominal imagen compleja ecolcida a predominio de cara anterior y porcin superior del rin derecho de 76 mm. El trax ofrece
datos de inters como la presencia de una radioopacidad no homognea de contornos irregulares, mas acentuad a la base pulmonar
derecha, fina banda radio opaca, que parte desde la regin perifrica hacia el hilio. 5 das despus del ingreso, el paciente comienza a
presentar cifras tensionales que en algunas ocasiones eran muy altas y otras bajas; en esa oportunidad se aprecia la presencia de
lesiones dermatolgicas papulares rojizas, comienza con lesiones purpuro-hemorrgicas y se auscultan crepitantes bibasales. Tensin
arterial (TA) 160/100 Frecuencia cardiaca (FC): 110 latidos/minuto.

PREGUNTA
Cual es el procentaje diagnostico de feocromocitomas de forma directa?

RESPUESTA
a.- 5 %
b.- 10 %
c.- 15 %
d.- 20 %

MANUAL DE TRABAJO DEL CURSO ENARM CMN SIGLO XXI
CURSO ENARM CMN SIGLO XXI TEL: 36246001 Pharmed Solutions Institute PGINA 65

SOBREPESO Y OBESIDAD
CIENCIAS BASICAS: La obesidad, incluyendo al sobrepeso como un estado premrbido, es una enfermedad crnica caracterizada por
el almacenamiento en exceso de tejido adiposo en el organismo, acompaada de alteraciones metablicas, que predisponen a la
presentacin de trastornos que deterioran el estado de salud, asociada en la mayora de los casos a patologa endcrina, cardiovascular
y ortopdica principalmente y relacionada a factores biolgicos, socioculturales y psicolgicos. SALUD PBLICA: Obesidad
padecimiento costoso (25% mayor que una persona con peso normal). Mxico ocupa 1er lugar en sobrepeso y obesidad (30% de la
poblacin), 2do EU, en Mxico 31% de la poblacin infantil sufre de sobrepeso y obesidad.
CLASIFICACIN:
EVALUACIN: En la evaluacin se debe incluir el IMC (ndice de
masa corporal), medicin de la circunferencia de la cintura. Para
IMC= peso en Kg/ talla
2
mts (kg/m
2
), esta proporciona una
medida precisa del contenido de grasa corporal, en adultos sin
importar gnero. El exceso de grasa abdominal, es un factor de
riesgo por si solo, la medicin de la circunferencia abdominal (a
nivel de crestas iliacas, cinta ajustada sin comprimir piel, con una
espiracin normal), es de particular importancia en pacientes categorizados como "normales" o con "sobrepeso", pero no es necesario
en pacientes con IMC >35kg/m
2
. Los hombres que presentan circunferencia abdominal >102 cm y mujeres con >88 cm, tendrn un
mayor riesgo de desarrollar diabetes, dislipidemia, hipertensin y enf. cardiovasculares, enf. de vesicula biliar; estos pacientes deben
ser colocados en una categoras de riesgo superior, que aquella que les corresponde por IMC. La incidencia de cncer endometrial, de
mama, prstata, colorectal, esteatohepatitis, osteoartritis y gota en hombres y mujeres esta incrementado por la obesidad.
PATOGENIA: Puede ser resultado de aumento del aporte calrico o disminucin gasto energtico o la combinacin de ambos. La
susceptibilidad a la obesidad es de naturaleza polignica, 30-50% de la variabilidad de reservas de grasa puede ser determinada
genticamente, este es el principal factor. Las causas secundarias de obesidad incluyen: enf. Hipotalmica, hipotiroidismo, Sx. de
Cushing e hipogonadismo. El aumento de peso tambin es inducido por frmacos, comn en aquellos que usan antidiabticos,
glucocorticoides, agentes psicotrpicos, estabilizadores del humor (litio), antidepresivos, antiepilpticos. Existen factores que al
combinarse con el sobrepeso y la obesidad, colocan al paciente en un altsimo riesgo de morir prematuramente, entre ellos: infartos,
angina de pecho, ciruga de arterias coronarias, ateroesclerosis, DM tipo II (esta coloca al paciente en riesgo absoluto muy alto). Tres o
mas de los siguientes factores de riesgo definitivamente aaden un riesgo absoluto: HTA, tabaquismo, elevacin del colesterol LDL,
glucosa en ayuno alterada, antecedente de enf. cardiovascular prematura y edad (hombres >45 y mujeres >55). TRATAMIENTO: En
>25kg/m
2
se les sugieren cambios en estilo de vida (dieta, comidas pequeas, frecuentes, desayuno, ejercicio. La prdida de peso se
recomienda en pacientes con IMC >30, en quienes tienen IMC 25-29.9 con factores de riesgo o aquellos que se encuentren en la
categora "alto riesgo" con base en la circunferencia abdominal. El primer objetivo es una reduccin de 10% de la masa corporal en un
periodo de 6 meses (a una velocidad de 0.5-1kg por sem), mediante una reduccin calrica de 500-1000kcal/da, posteriormente
enfocarse en mantenimiento de peso con la combinacin de dieta, actividad fsica y cambios en la conducta. Esta fase de
mantenimiento se define como una ganancia de peso no mayor a 3kg durante 2aos y una reduccin sostenida de la circunferencia
abdominal de por lo menos 4 cm. Se usan cambios en el estilo de vida y adicionar farmacoterapia: sibutramina (inhibidor de la
recaptura de norepinefrina y serotonina), produce prdida de peso de 5-9% en 12 meses, aunque produce alteraciones en la
frecuencia cardiaca y presin arterial, COFEPRIS pidi su retiro e 2010; orlistat (inhibidor de la lipasa intestinal= inhibe la absorcin de
grasas), produce prdida de peso de 9-10%, en 12 meses con cambios en el estilo de vida; metformina tiende a la disminucin del peso
corporal. Si logramos la primer meta, nos enfocamos en la tercera fase que es prdida de peso adicional. En pacientes que no es posible
perder peso, la meta es prevenir mayor ganancia de peso. El tratamiento quirrgico: Estar indicado segn la NOM exclusivamente en
los individuos adultos con obesidad severa e ndice de masa corporal >40, o >35 asociado a comorbilidad importante y cuyo origen en
ambos casos no sea puramente de tipo endcrino. Deber existir el antecedente de tratamiento mdico integral reciente, por ms de
18 meses sin xito; salvo ocasiones cuyo riesgo de muerte, justifique el no haber tenido tratamiento previo. Deber ser resultado de la
decisin de un equipo de salud multidisciplinario. Las cirugas para bajar de peso son A) Restrictivas (limita la cantidad de comida que el
estmago puede contener y frenar el vaciamiento gstrico), prdida significativa y sostenida por mas de 5 aos; aqu encontramos, la
colocacin laparoscpica de la banda gstrica ajustable, gastroplastia de banda vertical, , gastrectoma vertical en banda (qx. De
emergencia). B) Cirugas que limitan la ingesta de alimentos y alteran la digestin. La mas famosa es el puente gstrico tipo Y de Roux.
PRONSTICO: Personas con obesidad mrbida viven entre 8-10 aos menos que quienes tienen un peso normal. Por cada 15kg arriba
del peso ideal se incrementa el riesgo de muerte temprana en 30%. Incrementan riesgo de morbilidad respecto a HTA, dislipidemia, DM
tipo II, EVC, apnea del sueo, enf. arterial coronara.

CASO CLINICO
Varn obeso de 59 aos de edad con hemipleja izquierda. A los 54 aos de edad presento disnea intensa e hipertensin arterial
descontrolada. Recibi el diagnstico de hipertensin arterial a los 44 aos de edad. El examen fsico revel peso de 163,8kg, altura de
1,74 m, ndice de masa corprea 54,1 kg/m2, pulso de 84 lpm, presin arterial de 200/110 mmHg. El shock de punta del corazn fue
palpado en el 6 espacio intercostal, hacia fuera de la lnea hemiclavicular izquierda, y la ausculta no revel ruidos accesorios. Haba
soplo sistlico en rea mitral y borde esternal izquierdo. El abdomen estaba voluminoso sin visceromegalias. Haba edema discreto de
miembros inferiores y los pulsos en estos miembros estaban disminuidos.

PREGUNTA
Cual es la conducta a seguir mas apropiadas.

RESPUESTA
a.- Cirugia bariatrica.
b.- Colocacin de banda gstrica.
IMC (Kg/m
2
) CATEGORIA OMS CATEGORIA SEEDO Riesgo de enfermedad
<18.5 Infrapeso Peso insuficiente
18.5-24.9 Normopeso Normopeso
25.26.9 Sobrepeso Sobrepeso grado I
27-29.9 Sobrepeso grado II (preobesidad) Incrementado
30-34.9 Obesidad moderada Obesidad tipo I Alto
35-39.9 Obesidad severa Obesidad tipo II Muy alto
40-49.9 Obesidad morbida Obesidad tipo III (mrbida) Extremadamente alto
>50 Superobesidad Obesidad IV (extrema) Extremadamente alto
MANUAL DE TRABAJO DEL CURSO ENARM CMN SIGLO XXI
CURSO ENARM CMN SIGLO XXI TEL: 36246001 Pharmed Solutions Institute PGINA 66

c.- Orlistat mas sibutramina.
d.- Dieta estricta, orlistad y metformida.

CASO CLINICO
Mujer de 39 aos con hipertensin arterial, en tratamiento con candesartn, y obesidad grado III que comenz a tratar con sibutramina
12 das antes. Desde el inicio de la toma de medicacin refera cifras ms elevadas de presin arterial y palpitaciones. El da del ingreso
present en reposo dolor intenso retroesternal irradiado a la extremidad superior izquierda y sudoracin de unos 15 min de duracin,
que cedi tras nitroglicerina sublingual, y lleg a urgencias asintomtica. Los mximos sricos de creatincinasa y troponina T fueron 388
UI/l (normal hasta 140) y 0,23 ng/ml (normal hasta 0,035), respectivamente, con curva enzimtica tpica de infarto agudo de miocardio.
El electrocardiograma realizado sin dolor torcico fue normal durante todo el ingreso. El ecocardiograma no mostr alteraciones de la
contractilidad. La coronariografa mostr coronarias normales

PREGUNTA
Se programar para ciruga bariatrica ms liposuccin, considerando la comorbilidad, cual es la complicacin aguda mas frecuente.

RESPUESTA
a.- Embolia grasa.
b.- Embolia pulmonar.
c.- Insuficiencia cardiaca.
d.- Sindrome de absorcin.

CASO CLINICO
Se trata de una mujer de 50 aos de edad con obesidad mrbida (peso 105kg, talla 155cm e IMC de 43) y con antecedentes de
trombosis venosa profunda de repeticin. Ingres en Dermatologa por lesiones cutneas ampollosas generalizadas con biopsia cutnea
compatible con eritema polimorfo que se atribuy al tratamiento anticoagulante. Tras valoracin por el servicio de alergologa, se
aconsej su retirada y su no reintroduccin. Se indic tratamiento con heparina de bajo peso molecular, desarrollando importantes
hematomas dolorosos en el sitio de la administracin, motivo por el que se suspendi.

PREGUNTA
Cual es la medida teraputica que presenta menos complicaciones por las caractersticas del caso?.

RESPUESTA
a.- Cirugia bariatrica.
b.- Colocacin de banda gstrica.
c.- Dieta y ejercicio.
d.- Orlistat.

CASO CLINICO
Paciente varn de 60 aos, con sndrome de Alport, microhematuria y proteinuria de 4gr/da, creatinina de 2 mg/dl, urea 123mg/dl,
perdida moderada de la audicin, hipertensin arterial tratada con enalapril 20 mg, obesidad con IMC=36,26 (P 120Kg, h 1,81mts),
glucemias en ayunas alteradas (menor a 126mg/dl)Hb glicosilada normal, dislipidemia e hiperuricemia en tratamiento. Se indica dieta
bajas en protenas, hipocalrica y se agrega al tratamiento losartan en dosis de 75 mg/ da. Se realiza ciruga de by pass gstrico
presentado en su evolucin disminucin de 35 kg en total llegando a IMC de 25,6. Control nefrolgico: creatinina 1,56mg/dl,
proteinuria 0,3 g/dia, Urea 65mg/dl, normotension con enalapril 5 mg/d, se mantiene dosis mnima de hipolipemiantes, normouricemia
y normoglucemias.

PREGUNTA
Cual es la complicacin ms importante que presenta a largo plazo el paciente?.

RESPUESTA
a.- Insuficiencia renal.
b.- Hipotiriodismo.
c.- Sindrome de mala absorcin
d.- Anemia perniciosa.

MANUAL DE TRABAJO DEL CURSO ENARM CMN SIGLO XXI
CURSO ENARM CMN SIGLO XXI TEL: 36246001 Pharmed Solutions Institute PGINA 67

DISLIPIDEMIAS
CIENCIAS BASICAS: Definicin: son un grupo de trastornos caracterizados por la presencia de concentraciones anmalas de lpidos (LDL,
HDL, triglicridos) en sangre. Son importantes por su contribucin en la gnesis de ateroesclerosis, por lo que se relacionan
directamente con la enf. cardiovascular. SALUD PUBLICA: Prevalencia en adultos mexicanos es de 30% para hipercolesterolemia, 40%
hipertrigliceridemia y 50% para hipoalfalipoproteinemia, siendo la combinacin de esta ltima mas hipertrigliceridemia, lo ms
frecuente en el paciente diabtico. CLASIFICACION: Primarias: Secundarias a causa de obesidad, diabetes mellitus, consumo de alcohol,
anticonceptivos orales, glucocorticoides, falla renal, dao heptico e hipotiroidismo, o empeorar las subyacentes.
CARACTERISTICAS DE LAS PRINCIPALES DISLIPIDEMIAS PRIMARIAS
HIPERCOLESTEROLEMIA AISLADA HIPERTRIGLICERIDEMIA AISLADA COMBINADA
Hipercolesterolemia
familiar
LDL colesterol elevados; colesterol
total de 275-500mg/dl
Autosmica dominante
Puede ser a consecuencia de mutacin
para el receptor LDL
Xantomas tendinosos en edad adulta y
xantelasmas, enfermedad vascular
Complicaciones: cardiopata isqumica
Hipertrigliceridemia
familiar
VLDL elevadas, TG 250-750, leve incremento
de colesterol <250mg/dl
Autosmica dominante. Obesidad,
hiperglicemia e hiperinsulinemia son
caracteristicos
Asintomtico, xantomas eruptivos, puede
estar relacionado con incremento del riesgo
de enfermedad vascular. Complicaciones:
Pancreatitis, sx. metablico
Disbetalipoproteinemia
TG 250-500; VLDL elvado
Alteraciones en la Apo E
Autosmico recesivo, puede
haber Xantomas tuberosos o
palmares
Complicaciones: cardiopata
isqumica y sx. metablico
Hipercolesterolemia
polignica
LDL elevada, colesterol total
<350mg/dl, defectos genticos y
factores a,mbientales, dieta, edad,
ejercicio.
Asintomtica, desarrollan enfermedad
vascular, no xantomas
Deficiencia de
lipoprotein lipasa
familiar
TG = >750
Asintomtica recesiva, que a su vez
deteriora el metabolismo de los
quilomicrones, la acumulacin de estos uede
ser asociada a pancreatitis, dolor abdominal,
hapatoesplenomegalia

DIAGNSTICO: Cuadro clnico: Mas asintomticas, datos que pueden orientar hacia el origen, xantomas tuberosos (ndulos, en codos y
rodillas; disbetalipoproteinemia, hipercolesterolemia familiar), xantomas tendinosos (en tendn calcneo y extensores de manos; LDL
>300mg/dl), xantomas eruptivos (borde eritematoso y centro blanquecino, confluentes; TG >1000mg/dl), el xantoma estra palmaris o
depsito de lpidos en pliegues de las manos (disbetalipoproteinemia), el xantelasma es una ppula o placa amarillenta en prpados de
forma bilateral. Escrutinios a partir de 20 aos y repetirse cada 5 aos, dx., se basa en lpidos sricos, con ayuno de 8-10hrs.
NIVELES NORMALES DE LIPIDOS
LDL <100 optimo; 100-129 casi
optimo; 130-159 limitrofe alto;
160-189 alto; >190 muy alto
Colesterol total <200 normal; 200-
239 limitrofe alto; >240 alto
HDL <40 bajo
(hipoalfalipoproteinemia); 40-59
normal; >60 alto
TG <150 normal; 150-199 limitrofe
alto; 200-499 alto; >500 muy alto
TRATAMIENTO: Determinar nmero de factores de riesgo cardiovascular (hombre >45, mujer >55 aos, historia en familiares de primer
grado de cardiopata isqumica a edad temprana hombre <55, mujer <65 aos, tabaquismo, HTA, HDL <40). El orden en que se deben
perseguir las metas es: primero colesterol LDL, luego triglicridos y al ltimo colesterol HDL. Es importante sealar que cuando hay TG
>500mg/dl, la meta principal son estos, debido al riesgo de desarrollar pancreatitis. El tx., no farmacolgico l( dieta, ejercicio, reduccin
de peso), logra disminuir hasta 10% los niveles de colesterol, cuando no se logra control iniciar tx. Farmacolgico; en general las
estatinas son el frmaco de eleccin para la hipercolesterolemia, mientras que los fibratos para hipertrigliceridemia.
Hipoalfalipoproteinemia usar niacina y fibratos. hipercolesterolemia aislada, estatinas, colestiramina, colestipol, niacina. Estatinas;
Inhibidores de la hidroximetil glutaril coenzima A reductasa (prava, atorva, rosuva, sinvastatinas), tienen efecto hipolipemiante,
antiinflamatorio, mejoran funcin endotelial y disminuye la agregacin plaquetaria, sus efectos adversos mas importantes, miositis,
rabdomiolisis (cuando se usan con fibratos se prefiere gemfibrozil, para disminuir este riesgo), hepatotoxicidad, elevacin de CPK por
arriba de 10 veces el limite superior normal es una indicacin de suspensin de estatinas. Colestiramina y colestipol; quelantes de ac.
biliar, disminuye LDL y aumenta HDL, no usar en hipertrigliceridemia. Fibratos; reducen sntesis de VLDL y LPL, por el hgado
(gemfibrozil, fenofibrato, clofibrato), efectos adversos nausea, exantema, miopatia. Niacina; disminuye sntesis heptica de lipoproteina
con Apo B es el mejor frmaco para elevar HDL, efectos adversos bochorno, nusea, prediabetes, hiperuricemia.

CASO CLINICO
Varon de 78 aos de edad con IRC secundaria a hipertension arterial y diabetes mellitus (creatinina serica basal 1,5 mg/dl), exfumador
con hipercolesterolemia. Se diagnostico de cardiopatia isquemica con enfermedad grave de un vaso, llevandose a cabo una angioplastia
y stent. Un mes despus de este procedimiento, el paciente acudio a Urgencias por hematuria franca, detectando una elevacion de la
creatinina serica a 4,5 mg/dl, motivo por el que ingreso. En la exploracion fisica se detecto una arritmia en la auscultacion cardiaca y
una cianosis en primer, tercer y quinto dedo del pie derecho. El EGO mostraba sangre +++ y la cuantificacion de proteinas de 24 horas
fue de 1,2 g/dia egresa con colesterol 350, triglicridos 780, HDL 28.

PREGUNTA
Cul es la conducta teraputica a seguir al egreso ms adecuada.

RESPUESTA
a.- Bezafibrato y atorvastatina.
b.- Pravastatina, bezafibrato, metformida.
c.- Dieta, ejercicio, bezafibrato y pravastatina.
d.- Dieta, rehabilitacin, bezafibrato y pravastatina.

PREGUNTA
El paciente regresa a urgencias 4 semanas despus, somnoliento, urmico, con CPK de 1870, hematura macroscpica, mialgias
generalizadas, considerando el cuadro, cual es su diagnostico actual?

RESPUESTA
MANUAL DE TRABAJO DEL CURSO ENARM CMN SIGLO XXI
CURSO ENARM CMN SIGLO XXI TEL: 36246001 Pharmed Solutions Institute PGINA 68

a.- Insuficiencia Renal Aguda.
b.- Sindrome Hepatorrenal.
c.- Rabdomiolisis.
d.- Glomerulonefritis.

CASO CLINICO
Paciente de 69 aos de edad con antecedentes de HTA de carcter moderada desde hace 8 aos aproximadamente, con enalapril 10
mg e hidroclorotiazida 12,5 mg una vez por da. Consulta por cefalea a predominio frontal, la cual se manifiesta desde hace varios das
acompaado de sensacin de inestabilidad, mareo que no sabe referir exactamente pero si manifiesta sentirse muy extrao, con
alteraciones de la sensacin de la cara como hormigueo y falta de fuerza. Refiere presentar en una ocasin perdida de la visin de ojo
izquierdo durante menos de 1 minuto.

PREGUNTA
Cual es su conducta a seguir en este momento para el caso actual?

RESPUESTA
a.- IRM crneo contrastada.
b.- Perfil lipidico.
c.- Analitica de electrolitos sericos.
d.- Iniciar antilipemiantes y oxigenadores.

CASO CLINICO
Femenino de 58 aos, antecedentes de diabetes tipo II, con diagnstico de dislipidemia. No presenta enfermedad vascular previa. En
tratamiento con metformina 850 mg/da, AAS 100 mg/da y atorvastatina 10 mg/da, benzafibrato 200mg/dia, acude por dolor
muscular en miembros pelvicos. Padre y to diabtico, hermano con dislipemia. EF: PA: 135/90, IMC: 29, Permetro cintura: 100 cm, Glu:
198, Col: 410, Tg: 560, HDL: 28, HbAic: 8.7, Io: 143/4.3, Cr: 2.1, CPK: 320, Microalb: 30.

PREGUNTA
Cual de los siguientes elementos son ms importantes para considerar rabdomiolisis?

RESPUESTA
a.- Estatinas y fibratos.
b.- Dolor muscular.
c.- Elevacion de CPK.
d.- Valores de creatinina.

PREGUNTA
Cual es el objetivo principal en las dislipidemias?

RESPUESTA
a.- LDL
b.- Trigliceridos
c.- HDL
d.- Glucosa

PREGUTA
Considerando la respuesta anterior cual es la complicacin mas importante para establecer el objetivo?

RESPUESTA
a.- Pancreatitis.
b.- Estado hiperosmolar.
c.- Infarto al miocardio.
d.- Enfermedad vascular.

CASO CLINICO
Pte de 51 aos, diabtico tipo II con glibenclamida 5mg/d + metformina 1700 mg/d y enalapril 20 mg/d. FRP: Sedentarismo y
Tabaquismo (15 cig/d), Antecedentes: Madre y hermano hipertensos, EF: PA: 150/90 mmhg, Peso: 56 kg, Talla: 1.75 mts, Perimtro
Cint.: 103 cm, Rx Tx: Cardiomegalia c/aorta elongada y calcificacin botn artico. Hto: 44, Urea 31. Cr: 0.8. Glu: 118, HbA1c: 7.4; Col:
207, LDL: 433, TG: 408, HDL: 22; Ac.Urico: 7.8. GOT: 52; GPT: 38; Microalb: 40 mg/24hs.

PREGUNTA
Cual es el objetivo principal en la dislipidemia del caso?

RESPUESTA
a.- LDL
b.- Trigliceridos
MANUAL DE TRABAJO DEL CURSO ENARM CMN SIGLO XXI
CURSO ENARM CMN SIGLO XXI TEL: 36246001 Pharmed Solutions Institute PGINA 69

c.- HDL
d.- Glucosa

PREGUNTA
Cual es medicamento de eleccin para incrementar HDL en el presente caso?

RESPUESTA
a.- Niacina.
b.- Nicotinamina.
c.- Riboflavina.
d.- Hidroxicobalamina.

DIABETES MELLITUS TIPO II
CIENCIAS BASICAS: Definicin: Enfermedad sistmica cronico-degenerativa, de caracter heterogneo con grados variables de
predisposicin hereditaria y con participacin de diversos factores ambientales y que se caracteriza por hiperglucemia crnica debido a
la deficiencia en la produccion o accion de la insulina lo que afecta al metabolismo intermedio de los hidratos de carbonola, protenas y
grasas. (def. NOM). Factores de riesgo: Edad, obesidad si esta es central genera mas resistencia a la insulina, insulina(la acantosis
nigricans en pliegues es un dato de inicio de resistencia a la insulina), sexo (M60%; H40%),sedentarismo, dieta hipercalorica. SALUD
PUBLICA: 90-95% DM tipo II, 5-10% DM tipo I. Mortalidad 70 de cada 100,000. 7 de cada 10 diabticos, muere antes de cumplir la edad
promedio. Es la 7ma causa de muerte, principal causa de insuficiencia renal, causa de ceguera en personas de 20-74 aos. PATOGENIA:
Convinacion entre resistencia a la union a la insulina y una inadecuada respuesta secretora, que iniciamente las celulas b del pncreas
incrementa. Crculo vicioso de hiperglucemia (pospandrial) hiperinsulinismo regulacion a la baja de receptores de insulina
(perifricos). DIAGNSTICO: Clinico; poliuria, polidipsia, polifagia, perdida de peso, visin borrosa, susceptibilidad algunas infecciones.
Criterios caso confirmado de diabetes; 1. hemoglobina glicosilada >6.5%. 2. Glucemia plasmatica en ayuno (8hrs) >126mg/dl (normal
60-100mg/dl). 3. Glucemia plasmatica casual >200 mg/dl, en un paiente con sintomas clasicos de hiperglucemia (normal 100-200
mg/dl). 4. Glucemia >200mg/dl a las 2 hrs despus de una carga oral de 75mg de glucosa (PTOG). Prediabetes: Persona que tiene
antecedente padre, madre o ambos con estado metablico entre lo normal y la diabetes glucosa anormal en ayuno 100-125 mg/dl,
PTOG; >140 y <200 mg/dl. (normal <140mg/dl). COMPLICACIONES: En descompensacin; estado hiperosmolar, cetoacidosis,
hipoglucemia. A largo plazo; nefropata = IRC, retinopatia=amaurosis, neuropata perifrica = ulceras, malformacion de Charcot,
neuropata autonmica = vejiga neurogenica, disautonomia cardiovascular, disfuncin sexual, diarrea, estreimiento. TRATAMIENTO:
Nutricional; disminucion de carbohidratos, incremento en fibra, disminucion de lipidos. Aumentar la actividad fsica al menos 150
min./semana como caminar o trotar. Perder 7% de peso corporal (2-4 meses)1. Modificaciones al estilo de vida + metformina (usar
pioglitazona por intolerancia), especial en pacietes con IMC > 35, >60 aos o con diabetes gestacional previa. Pacientes sin sobrepeso
se puede iniciar con sulfonilureas, no control. 2. metformina + sulfonilurea (1ra eleccion). Metformina + tiazolidendionas (2 eleccin) no
control. Metformina + sulfonilurea + insulina o metformina + tiazolidendionas + sulfonilureas. (tiazolidendionas contraindicadas en
pacientes con insuficiencia cardiaca o juntas con insulina). sintomas marcados o glucemias o hb glicosilafas muy elevafas iniciar con
insulina con o sin agentes adicionales. PREVENCION: En diabeticos control glicemico con hb glicosilada 2 por aos, si hay descontrol
hacer cada 3 meses, es lo que tiene de memoria, valores <7% reduce retinopatia, nefropatia. Pacientes asintomticos adultos a
cualquier edad, con sobrepeso u obesidad (IMC>25), con uno o mas factores de riesgo, hacer prueba cada 3 meses si salen normales.
Asintomtico con glicosa en ayuno >100 y <126; repetir glucosa en ayuno, se repite resultado = prediabetes, sale >126 hacer PTGO.
Prediabetes; seguimiento anual. Paciente con cifras normales sin factores de riesgo cada 3 aos. Deteccion de albuminuria una vez por
ao y al momento del dx., toda cifra superiora 300mg/dl = albuminuria clinica o macroalbuminuria. VAloracion por oftalmologia una vez
por ao. PRONSTICO: Las variables que nos pueden hacer predecir el peor pronostico estan relacionadas con el grado de
sedentarismo, coexistencia de IC, descenso del aclaramiento de creatinina (nefropatia principal), y de hemoglobina. REHABILITACION:
MIcro y macroalbuminuria, iniciar trtatamiento con IECAS o ARA II, para nefroproteccion ( sin olvidar medir niveles de K+). Reducir
ingesta de proteinas de 0.8-1 g/kg/da. CLAVES: Biguanidas (Metformina) 1ra. Eleccion en obesos, complicacion mas grave acidosis
lctica. Sulfonilireas (glibenclamida) se eliminan por rin si hay IRC, se acumula; con una depuracin renal <50, ya no dar
hipoglucemiantes por riesgo de hipoglucemia. Insulina glargina, dura 24hrs, se mantienemantiene basal todo el dia.mantiene Insulina
NPH, dura hasta 12hrs, dosis 0.15 UI/kg, 2/3 matutino y 1/3 vespertino. Efecto somogy; hiperglucemia reactiva matutina, secundaria a
una hipoglucemia nocturna.Insulina lispro ultrarapida, ideal para tx. de picos de insulina.

CASO CLINICO
Mujer de 78 aos, independiente en actividades de la vida diaria, con escolaridad completa, tiene antecedentes de hipertensin
arterial, hipotiroidismo, hipoacusia y diabetes mellitus tipo 2 (DM 2) manejada con glibenclamida 10 mg y metaformina 1.700 mg al da,
con lo cual lograba glicemias capilares adecuadas. Fue llevada por familiares al Servicio de Urgencia, para evaluacin por compromiso
cualitativo de conciencia, no fluctuante, de ms de 24 hrs de evolucin, sin signos de focalizacin y con recuperacin espontnea
completa. Su familia seal la presencia cada vez ms frecuente de episodios similares en los ltimos 2 meses, indicando adems,
alteraciones de la memoria reciente, de la conducta y alteraciones autolimitadas de la marcha, sin trastorno del control de esfnteres,
con repercusin funcional por prdida parcial de su autovalencia.

PREGUNTA
Cul es el factor ms probable que genere la hipoglucemia en este paciente.

RESPUESTA
a.- La glucosuria.
b.- El hipoglucemiante.
MANUAL DE TRABAJO DEL CURSO ENARM CMN SIGLO XXI
CURSO ENARM CMN SIGLO XXI TEL: 36246001 Pharmed Solutions Institute PGINA 70

c.- El antipertensivo.
d.- Depuracin renal

CASO CLINICO
Mujer de 70 aos de edad, hipertensa, diabtica de reciente diagnostico durante un examen de rutina y cursando con bacteriuria
asintomatica, procur atencin mdica por tos seca, disnea, inapetencia y desnimo. La paciente saba que era portadora de
hipertensin arterial desde la edad de 37 aos, de diabetes mellitus desde los 56 aos y de hipertrigliceridemia. A los 56 aos de edad,
se quej de palpitaciones taquicrdicas con duracin de 20 minutos, sin sncope. Haca uso de 75 mg de captopril, 50 mg de
clortalidona, 600 mg de quinidina y 0,25 mg de digoxina. El examen fsico revel peso de 54 kg, altura de 1,49 m, presin arterial de
170/110 mmHg. En el examen fsico, fue identificado soplo sistlico +/4+ en rea mitral. Lo restante del examen fsico no revel
alteraciones. El ECG mostr ritmo sinusal, sobrecarga ventricular izquierda, infradesnivel de ST en V5 y V6 y presencia de onda U.

PREGUNTA
Cual de los siguientes frmacos tiene mayor evidencia de efecto cardioprotector en esta condicin del paciente?

RESPUESTA
a.- IECAs
b.- ARA II.
c.- Digoxina.
d.- Clortalidona.

PREGUNTA
Considerando las condiciones del caso clnico cual es la indicacin mas adecuada para envio a olftalmologia?

RESPUESTA
a.- Disminucion de la funcin renal.
b.- Incremento de creatinina.
c.- Aumento de microalbuminauria.
d.- Presencia de fosfenos.

CASO CLINICO
Se trata de masculino de 46 aos de edad el cual acude a consulta debido a que ha presentado un proceso infecciosos urinario ya
tratado, refiere mejora completa posterior a una semana de tratamiento antibitico, a la exploracin identifica un permetro
abdominal de 102 cms debido a ello realiza una glucosa perifrica donde se obtiene 130 mg/dl.

PREGUNTA
Cual es la conducta a seguir para sustentar el diagnostico.

RESPUESTA
a.- Curva de tolerancia a la glucosa.
b.- Glucosa en ayuno.
c.- Determinar hemoglobina glucosilada.
d.- Glucosa pospandrial.

CASO CLINICO
Varn de 47 aos con DM diagnosticada hace 9 aos, en tratamiento actual con esquema insulnico intensivo (una dosis nocturna de
insulina glargina e insulina lispro previo a las comidas). Entre sus antecedentes destacaban gran tabaquismo, dislipidemia mixta en
tratamiento irregular con ciprofibrato 100 mg/da y microalbuminuria de 50 mg/12 h, sin hipertensin arterial en tratamiento con
enalapril 5 mg/da. Consult en un centro de salud por sensacin febril no cuantificada, calofros, odinofagia, nuseas y vmitos
alimentarios de 2 das de evolucin. Se constat deshidratacin moderada, temperatura axilar de 37,8C y glicemia capilar mayor de
400 mg/dl. Fue internado, recibiendo solucin fisiolgica e insulina en esquema de minidosis. Entre los exmenes de laboratorio
obtenidos al ingreso destacaban glicemia 549 mg/dl, creatinina 1,67 mg/dl, pH 6,91, BE -27, HC03 4,6 (en sangre arterial), cetonemia
+++, natremia 141,7 mEq/L, kalemia 5,92 mEq/L, cloremia 98 mEq/L, amilasemia 48 U/L, hematocrito 40,5%, leucocitos 9100 mm3, VHS
38 mm/h y urocultivo negativo.

PREGUNTA
Que cambios electrocardiogrficos es probable esperar.

RESPUESTA
a.- QRS corto.
b.- P-R prolongado.
c.- T picudas.
d.- Ondas U.



MANUAL DE TRABAJO DEL CURSO ENARM CMN SIGLO XXI
CURSO ENARM CMN SIGLO XXI TEL: 36246001 Pharmed Solutions Institute PGINA 71

PREGUNTA
Se envio urocultivo y hemocultivo donde se observo crecimiento de e. coli cual es el manejo antimicrobiano mas adecuado
considerando el KDOQI actual?

RESPUESTA
a.- Ceftriaxona y vancomicina.
b.- Ampicilina y amikacina.
c.- Imipenem y gentamicina.
d.- Imipenem, vacomicina y gatifloxacina.

CASO CLINICO
Hombre de 62 aos con antecedentes de hipertensin arterial, infarto agudo de miocardio y DM tipo 2. Ingresado por pielonefritis con
IR aguda (MDRD 21 ml/min), interpretndose en el contexto de la propia infeccin, agravada por el consumo de antiinflamatorios no
esteroideos (AINE) y persistiendo al ser dado de alta. Un mes despus presenta lo siguiente (MDRD 13,83 ml/min), proteinuria de 5
g/24 h y microhematuria persistente.

PREGUNTA
Cual es su conducta diagnostica mas til en este momento?

RESPUESTA
a.- Urocultivo.
b.- Citologia para malignidad.
c.- Fondo de ojo.
d.- Ecografia abdominal.

PREGUNTA
Considerando el MDRD actual en que grado de IR se encuentra?

RESPUESTA
a.- IR grado 4.
b.- IR grado 3.
c.- IR grado 2.
d.- IR grado 1.

CASO CLINICO
Varn de 58 aos, con hipertensin arterial en tratamiento farmacolgico con regular control y diabetes mellitus tipo 2 en tratamiento
con insulina con buen control metablico. Con (IRC) y microalbuminuria. Ingresa la tensin arterial era 139/87 mmHg, peso de 81 kg,
talla de 180 cm, urea de 61 mg/dl, creatinina 1,7 mg/dl, aclaramiento de creatinina 44,37 ml/min, microalbuminuria 11,75 ug/min y
hemoglobina de 5,5; albmina 4,2 g/dl, protenas totales 6,7 g/dL, colesterol total: 150 mg/dl, triglicridos: 168 mg/dL.

PREGUNTA
En que estadio de IRC se encuentra?

RESPUESTA
a.- Estadio 1.
b.- Estadio 2.
c.- Estadio 3.
d.- Estadio 4.

CASO CLINICO
Varn de 69 aos, con historia de HTA, cardiopata isqumica, hernia hiatal, gastritis crnica y diabetes mellitus tipo II, con pobre
control metablico habitual, tratada con hipoglicemiantes orales, alcoholismo crnico, acude por debilidad muscular proximal en las
cuatro extremidades (dificultad para realizar tareas como abducir los brazos, subir escaleras, levantarse desde una silla baja, caminar
distancias moderadas, etc.), de inicio insidioso, simtrico y lentamente progresivo, acompaada de marcada inestabilidad para la
marcha, prdida de la musculatura distal en las extremidades y saltos musculares, as como una acentuacin de las parestesias
habituales (pinchazos, adormecimientos, etc.) fundamentalmente en horarios nocturnos. Paralelamente, se hallaba aquejada por
disminucin bilateral y rpidamente progresiva de la agudeza visual, astenia, poliuria, polidipsia y disminucin de peso. Paciente vigil,
bien orientado, con deterioro cognitivo, con lenguaje conservado; marcha ligeramente atxica; cuadriparesia flccida simtrica de
predominio proximal, arreflexia osteotendinosa generalizada, hipotrofia muscular distal ms acentuada en las manos y antebrazos,
acompaada de fasciculaciones; signo de Romberg positivo.

PREGUNTA
Cual de los siguientes trastornos es mas probable que presenta actualmente el paciente?

RESPUESTA
a.- Demencia por alcoholismo.
MANUAL DE TRABAJO DEL CURSO ENARM CMN SIGLO XXI
CURSO ENARM CMN SIGLO XXI TEL: 36246001 Pharmed Solutions Institute PGINA 72

b.- Degeneracion cerebelosa.
c.- Polineuropatia diabtica.
d.- Degeneracion pontoolivocerebeloso.
MANUAL DE TRABAJO DEL CURSO ENARM CMN SIGLO XXI
CURSO ENARM CMN SIGLO XXI TEL: 36246001 Pharmed Solutions Institute PGINA 73

PANCREATITIS AGUDA Y CRONICA
CIENCIAS BASICAS:
SALUD PUBLICA: En Europa incidencia de 5.4 casos por 100,000 hab, en EU es de 7.9 por 100,000 hab. La mayora se recupera pero 1-
5% desarrolla un episodio de pancreatitis aguda, y 20% fallece a pesar de tx. Pancreatitis crnica ms frecuente entre 50 y70 aos, con
una prevalencia de 0.04-5%.
PANCREATITIS AGUDA: El espectro patolgico de la pancreatitis
aguda vara desde pancreatitis intersticial, que suele ser un
trastorno leve y autolimitada, a la pancreatitis necrotizante, en el
que el grado de necrosis pancretica se correlaciona con la
severidad del ataque y sus manifestaciones sistmicas.
PATOGENIA: 3 fases: primera; activacin de enzimas pancreticas
(tripsinogeno, quimiotripsinogeno, fosfolipasas y proteasas), lo
cual produce dao tisular, aumento de permeabilidad de
conductos pancreticos, que permiten salida de enzimas al
intersticio generando autodigestin tisular. La tripsina una vez
activada por la enterocinasa y la catepsina B, produce dao directo, activa otras enzimas y estimula produccin de citosinas, las cuales
daan a distancia. Segunda; qumioatraccin, activacin y secuestro de neutrfilos (puede llevar a necrosis y hemorragia). Tercera;
enzimas proteolticas y citosinas en rganos distantes, que se manifiestan como sx. de respuesta inflamatoria sistmica. En litiasis
vesicular, es debida al reflujo de material biliar dentro del conducto pancretico o a la obstruccin del flujo de salida, que ocasiona
incremento de la presin intraductal y cambio en la permeabilidad. DIAGNSTICO: Cuadro clnico; paciente ansioso e inquieto, dolor
abdominal sbito en epigastrio, regin periumbilical (se irradia a espalda y trax, dolor ms intenso en decbito supino, mejora en
posicin de gatillo, sensibilidad abdominal y rigidez), fiebre, nusea, vmito, distensin abdominal, ataque al edo. general. Taquicardia,
taquipnea, hipotensin, disminucin de ruidos intestinales, ndulos eritematosos en piel debido a necrosis grasa subcutnea, signo de
Cullen (hematoma periumbilical), signo de Turner (hematoma en flancos), ictericia en casos secundarios a coledocolitiasis. Laboratorio:
amilasa elevada mas de 3 veces lo normal (mas en casos secundarias a litiasis vesicular), lipasa elevada es la de mayor sensibilidad y
especificidad, leucocitosis, hto >44%, hiperglucemia, hipocalcemia (25%), hiperbilirrubinemia, hipertrigliceridemia (15-20%) AST y DHL
elevadas, nivel de protena C >150mg/dl, habla de necrosis y enfermedad grave. Estudios de imagen, asa centinela es un hallazgo
radiolgico sugestivo de pancreatitis aguda (no especifico), ultrasonido; para visualizar pncreas, gas intestinal, litos, pesudoquistes,
edema o crecimiento de pncreas. TAC gold standard para dx., podemos ver zonas de necrosis, la cual tiene una elevada tasa de
infeccin hasta de 50% y mortalidad asociada hasta de 30%, podemos utilizar criterios de Balthazar. Determinacin de gravedad: Con
criterios de Ranson. TRATAMIENTO: La piedra angular es el ayuno (sonda nasoenteral o parenteral), iniciar VO de 3-5 das de iniciado el
tx., soluciones IV abundantes (mas cristaloides), analgsicos, O2, correccin de alteraciones hidroelectrolticas. CEPRE en pacientes con
pancreatitis secundaria a litos biliares y colangitis. Antibiticos se recomienda en casos con evidencia de necrosis pancretica mayor de
30%, y alta sospecha de infeccin. El frmaco recomendado es imipenem 500mg tres veces al da por 7 das. Ciruga indicada en casos
de necrosis infectada. COMPLICACIONES: necrosis pancretica (40-60%), pseudoquistes (15%), abscesos pancreticos, ascitis
pancretica, insuficiencia renal y/o respiratoria, sangrado gastrointestinal, choque sptico.
PANCREATITIS CRNICA: Dao permanente en la estructura, como consecuencia de un proceso inflamatoria crnica, superpuesto
sobre un pncreas previamente lesionado, que lleva a fibrosis y perdida de la funcin. Se puede dividir en calcificante obstructiva (ms
comn, relacionada con alcohol, hereditaria, hiperlipidemia) y obstructiva (tumores, estenosis o el pncreas divisum). PATOGENIA: Lo
mas comn el consumo crnico de alcohol, en nios la principal causa fibrosis qustica. Menos relacionada con enf. autoinmunitaria,
hipertrigliceridemia, pancreatitis tropical, obstruccin del conducto e idiopticas.
DIAGNSTICO: Cuadro clnico: Dolor sntoma cardinal. Triada presencia de
calcificaciones pancreticas, diabetes mellitus y esteatorrea. Mala digestin y dolor
abdominal muy variable, algo caracterstico es la relacin que guarda con los
alimentos, por lo que el paciente disminuye los mismos y pierde peso. Para el
desarrollo de esteatorrea es necesaria la prdida de 90% o ms de tejido exocrino, lo
cual disminuye la produccin de amilasa, lipasa y proteasas con la consecuente mala
digestin de los nutrientes. No existe un incremento en los niveles de enzimas
pancreticas, las bilirrubinas en suero y la fosfatasa alcalina pueden estar elevadas.
TAC, calcificaciones
pancreticas en 30-60%.
CEPRE, provee
informacin acerca de los
conductos pancreticos.
COMPLICACIONES: Sntomas por Malabsorcin de vitamina B12 en 40%, de
alcohlicos y fibrosis qustica, tolerancia a la glucosa alterado, retinopata
no diabtica debido a deficiencia de vitamina A y zinc, necrosis grasa
subcutnea y dolor seo. Se incrementa el riego de cncer de pncreas.
TRATAMIENTO: Los ataques intermitentes se tratan como la pancreatitis
aguda. Alcohol, comidas grandes y grasosas deben ser evitadas, narcticos
en dolor severo, mantener una adecuada hidratacin y deberan de ser
hospitalizados, si hay sntomas leves de manera ambulatoria. Ciruga podra
controlar el dolor si hay una estenosis ductal. Pancreatectomia subtotal
podra controlar el dolor pero el costo es insuficiencia exocrina y diabetes.


CAUSAS DE PANCREATITIS AGUDA
Causas comunes: Clculos biliares (microlitiasis) 30-60%, Alcohol 15-30%,
Hipertrigliceridemia, CEPRE, trauma, posoperatorio, drogas (aziatropina,
ciclosporina, tacrolimus, 6-mercaptopurina, sulfonamidas, estrgenos, tetraciclinas,
ac. valproico, furosemida), disfuncin del esfnter de Oddi.
Causas menos comunes: Causas vasculares y vasculitis, desordenes del tejido
conectivo, purpura trombocitopenica trombotica, cncer de pncreas,
huipercalcemia, divertculo periampular, pncreas divisum, pancreatitis herditaria,
fibrosis qustica, falla renal.
Causas raras: Infecciones (parotiditis, caxaquievirus, CMV, echovirus), autoinmune



MANUAL DE TRABAJO DEL CURSO ENARM CMN SIGLO XXI
CURSO ENARM CMN SIGLO XXI TEL: 36246001 Pharmed Solutions Institute PGINA 74

CASO CLINICO
Mujer de 34 aos sin antecedentes; ingres en el hospital por pancreatitis aguda biliar leve. La evolucin inicial fue favorable, con
comienzo de tolerancia oral al tercer da. Al quinto da present dolor abdominal espigstrico y en el hipocondrio izquierdo de inicio
brusco, asociado a palidez, sudoracin, taquicardia a 120 lpm y PA de 105/52 mmHg. A la exploracin existan signos de irritacin
peritoneal. Analticamente destacaba hemoglobina (Hb) 8,4g/dl, 4 puntos menos que al ingreso. En la TC abdominal se objetiv la
existencia edema pancretico y un gran hematoma subcapsular esplnico de 16 x 13cm, con lquido libre peritoneal denso de alta
cuanta. No se consigui la estabilizacin hemodinmica con reposicin de volemia y se decidi realizar una laparotoma exploradora
urgente. Tras acceder a la cavidad abdominal a travs de laparotoma media, se puso de manifiesto hemoperitoneo masivo (2.500ml),
un gran hematoma periesplnico y un bazo decapsulado en su cara anterior. Se realiz una esplenectoma y una colecistectoma.

PREGUNTA
Cual de los siguientes factores predictivos tiene mayor relevancia para el pronostico inmediato.

RESPUESTA
a.- Hemoglobina.
b.- Leucocitos.
c.- Glucemia.
d.- TAC.

CASO CLINICO
Paciente de 74 aos con antecedentes de hipertensin arterial, diabetes mellitus no insulinodependiente y enfermedad de Parkinson.
Acudi a las urgencias hospitalarias por dolor abdominal epigstrico de 3 das de evolucin, mareos y sncope. A la exploracin
presentaba una PA de 80/40 mmHg y una frecuencia cardaca de 102 lpm. El abdomen presentaba dolor meso-epigstrico y sensacin
de masa. Analtica: amilasemia de 854 U/l, 13.500 leucocitos/l y Hb de 8g/dl. En la TC se confirm pancreatitis aguda en la cabeza
pancretica, con desestructuracin del parnquima y signos sugestivos de necrosis a ese nivel, complicada con un hematoma
retroperitoneal de 20 x 7 cm, con signos de sangrado arterial activo que situaba en una rama cercana al tronco celiaco.

PREGUNTA
Cual es la complicacin crnica mas frecuente.

RESPUESTA
a.- Insuficiencia pancretica.
b.- Anemia crnica.
c.- Diabetes mellitus.
d.- Pancreatitis crnica.

CASO CLINICO
Mujer de 78 aos de edad, diabtica e hipertensa con dolor abdominal de 24 horas de evolucin. Con inestabilidad hemodinmica,
insuficiencia respiratoria grave e irritacin peritoneal y equimosis periumblical. Se realiza TAC abdominal compatible con pancreatitis
enfisematosa, preservndose solo la cola pancretica y observando en su lugar aire extraluminal que rodea los vasos regionales, as
como de forma difusa intraabdominal. Mltiples infartos hepticos. Se realiza ciruga urgente objetivndose lquido libre peritoneal
serohemorrgico con crepitacin de tejidos blandos y esteatonecrosis difusa. Se reinterviene donde se observa pncreas
desestructurado con aspecto necrohemorrgico. La paciente presenta mala evolucin y fallece a las 48 horas del ingreso.

PREGUNTA
Cual es la complicacin aguda mas frecuentes con descenlace fatal.

RESPUESTA
a.- Sepsis abdominal.
b.- Falla organica multiple.
c.- Coagulacion intravascular diseminada.
d.- Tromboembolia pulmonar.

CASO CLINICO
Masculino de 89 aos de edad, refiere dolor abdominal con nuseas y vmitos de 2 das de evolucin. En la exploracin destaca un
abdomen doloroso de forma generalizada con predominio en hemiabdomen derecho. En la analtica se detectan 20.500 leucocitos
(78% neutrfilos, 17 bandas), hematocrito 42%, actividad de protrombina 80%, fibringeno 600 mg/dl, bilirrubina 3,3 mg/dl, amilasa
229 U/l, lipasa 310 U/l, GPT 141 mg/dl; sedimento de orina y radiografa de trax sin alteraciones significativas. Se mantiene en
observacin progresando el cuadro clnico requiriendo ciruga y estancia en terapia intensiva, se resuelve el cuadro agudo y es egresado
un mes despus de su ingreso.

PREGUNTA
Cual es la sintomatologa crnica ms probable del caso.

RESPUESTA
a.- Esteatorrea.
MANUAL DE TRABAJO DEL CURSO ENARM CMN SIGLO XXI
CURSO ENARM CMN SIGLO XXI TEL: 36246001 Pharmed Solutions Institute PGINA 75

b.- Mala absorcin.
c.- Anemia perniciosa.
d.- Hiperglucemia.

ESTEATOSIS HEPATICA (EH):
CIENCIAS BASICAS: Definicin: es la acumulacin de lpidos histolgicamente visible (grasa en hgado), en el citoplasma de los
hepatocitos. Es la alteracin ms frecuente a nivel heptico. Definicin de
esteatosis no alcohlica: entidad anatomoclnica caracterizada por esteatosis
heptica, diferentes grados de inflamacin y fibrosis, hallazgos semejantes a
hepatopata alcohlica, ausencia de ingesta significativa de alcohol. Se puede
clasificar de acuerdo a su etiologa en esteatosis alcohlica y esteatosis no
alcohlica (Etiologia: sndrome metablico, obesidad, diabetes tipo II e
hiperlipidemia, rpida prdida de peso en los obesos, nutricin parenteral
total, sndrome de intestino corto, gastroplastia, hipolipoproteinemias,
tirosinemi, enf. de Wilson, tratamiento con drogas como amiodarona,
estrgenos, tetraciclinas, coticoides, tamoxifeno, nifedipino, lipodistrofia, hgado graso del embarazo). SALUD PUBLICA: La esteatosis
heptica no alcohlica es un padecimiento que afecta al 20-30% de la poblacin general en varios pases; la prevalencia aumenta en los
sujetos con obesidad mrbida 75-92%, mientras que en la poblacin peditrica es de aproximadamente 13-14%. La forma progresiva de
la esteatosis heptica no alcohlica, la esteatohepatitis, se estima que aproximadamente en 3-5% de los casos puede progresar a
cirrosis. En Mxico la prevalencia de esteatosis heptica no se conoce con precisin; sin embargo, si podemos extrapolar datos de
obesidad obtenidos de la encuesta nacional de salud y nutricin (ENSANUT) de 2006; en ella se informa que el 30% de la poblacin
rene criterios de obesidad y de ellos, ms de las dos terceras partes pueden tener esteatosis en el hgado; por otra parte, la
prevalencia promedio de DMT 2 es de 7%, y el 92% de estos pacientes tienen sndrome metablico y resistencia a la insulina.
CLASIFICACION: Segn el porcentaje de hepatocitos afectados, se clasifica en: 1) Leve; menos de 25% de los hepatocitos afectados, 2)
Moderada; de 25-50%, 3) Severa; ms de 50%. PATOGENIA: La EH, resulta de un desequilibrio entre la sntesis hepatocitica de
triglicridos a partir de cidos grasos y de la secrecin desde el hepatocito bajo forma de lipoprotenas. La retencin de triglicridos en
los hepatocitos es requisito indispensable para el desarrollo de esteatosis heptica. El metabolismo de los triglicridos en el tejido
adiposo da lugar a la liberacin de cidos grasos libres a la circulacin y stos son captados por los hepatocitos. El acumulo de lpidos en
el hgado resulta de una prdida del balance entre la captacin, la sntesis, la salida y la oxidacin de los cidos grasos libres. Estudios
recientes demuestran que el flujo de cidos grasos libres provenientes del tejido adiposo y que llegan al hgado representa la mayor
fuente de la grasa intraheptica, 62% 82% de los triacilgliceroles del hgado y que la contribucin de lipognesis de novo que en
sujetos normales es menor del 5%, en pacientes con esteatohepatitis aumenta hasta ser del 26%. La insulina, inhibe el metabolismo de
los triglicridos en el tejido adiposo, aumenta la sntesis intraheptica de cidos grasos libres y de triglicridos e inhibe la beta
oxidacin de los cidos grasos libres en los hepatocitos. Los estados de hiperinsulinismo como la DMT2 y la obesidad disminuyen la
sensibilidad tisular a la insulina, esta resistencia a la accin de la insulina es el factor ms reproducible en el desarrollo de esteatosis
heptica. Muchos casos son atribuidos al alcohol, su presencia se asocia con un espectro de enfermedades tales como: ingesta de
drogas, txicos, obesidad, dislipidemias, diabetes, caquexia y nutricin parenteral. En la esteatosis heptica al igual que en otros
padecimientos metablicos se altera la liberacin de adipocinas. Estas protenas sintetizadas especficamente en el adipocito,
comparten funciones que regulan energa y procesos de inmunidad. Cuando se altera el lenguaje y comunicacin entre el hgado y el
tejido adiposo da como resultado una serie de anormalidades metablicas e inflamatorias. ESTEATOHEPATITIS: La esteatohepatitis es
una forma ms agresiva de esteatosis heptica en la cul la infiltracin grasa del hgado en un porcentaje variable de pacientes (que
puede llegar hasta el 2030%), se acompaa de intensa actividad necro-inflamatoria y puede progresar a fibrosis y cirrosis heptica y
eventualmente a insuficiencia heptica y hepatocarcinoma. De los factores que participan en la progresin de hgado graso a
esteatohepatitis se reconoce que tanto el sndrome metablico (Resistencia insulnica / hiperinsulinismo / DM tipo II,Obesidad,
Dislipemia (TG >180 o HDL-col <40), HTA)como la resistencia a la insulina son factores predictivos independientes en esa progresin.
DIAGNOSTICO: Clnico: puede haber dolor en hipocondrio derecho y presentar una hepatomegalia lisa, firme, a veces dolorosa.
Laboratorio: nada especifico, puede haber aumento en las transaminasas y GGT pueden dar sospecha. Imagenologa: La ecografa es el
mtodo mas accesible para el diagnstico de EH ( se observa, aumento de la ecogenicidad heptica hgado brillante, acompaado de
una atenuacin del haz sonoro en los sectores profundos del hgado). En funcin de los hallazgos clasificamos a los pacientes en tres
grados: Grado 0: No presenta esteatosis heptica. Grado I: Presenta esteatosis heptica, pudiendo distinguir la grasa periportal. Grado
II: Presenta esteatosis heptica, con borramiento de la grasa periportal. Supone un mayor grado de acmulo graso. TAC Y RM,
diagnstico certero, pero altos costos y disponibilidad. Laparoscpico: observacin del tpico color amarillento de la superficie heptica.
El examen histolgico mediante puncin bipsica heptica, permite el diagnostico de EH, constituyndose en el patrn de oro en el
diagnstico de esta enfermedad. TRATAMIENTO: En el tratamiento propiamente de la esteatosis y esteatohepatitis hay un grupo de
acciones que se aceptan universalmente y que podemos englobarlas como cambios en el estilo de vida, para ello tenemos que
considerar entre otras cosas el consumo de caloras y el tipo de alimentos, la actividad fsica que ese paciente desarrolla, su situacin
socioeconmica, etc. Por experiencia, los mdicos sabemos bien lo difcil que es lograr cambios en el estilo de vida de los pacientes. A la
fecha, no hay un tratamiento farmacolgico que haya demostrado ser de utilidad precisa, algunas modalidades descritas incluyen:
medicamentos utilizados para bajar de peso, medicamentos que intentan bloquear la resistencia a la insulina, agentes hipolipemiantes,
antioxidantes, citoprotectores, adems de la ciruga baritrica. PRONOSTICO: La esteatosis heptica simple parece tener un pronstico
relativamente benigno mientras que aquellos con datos de esteatohepatitis o fibrosis en la biopsia del hgado tienen un peor
pronstico.

CASO CLINICO
Se trata de paciente masculino de 41 aos de edad con antecedentes de obesidad grado II, dislipidemia, bajo tratamiento con
bezafibrato, pravastatina y orlistat adems refiere ingesta crnica de alcohol, fue diagnosticado con esteatosis hace 6 meses y hernia
ETIOPATOGENIA

ESTRS OXIDATIVO
PEROXIDACION LIPIDICA


ESTIMULOS TNF-alfa CELULAS ANTIGENOS
QUIMIOTACTICOS IL-6, IL-8 ESTRELLADAS PROTEICOS
INFLAMACION FIBROSIS H. MALLORY



MANUAL DE TRABAJO DEL CURSO ENARM CMN SIGLO XXI
CURSO ENARM CMN SIGLO XXI TEL: 36246001 Pharmed Solutions Institute PGINA 76

hiatal con mal apego a tratamiento, acude a consulta debido a dolor en cuadrante superior derecho, tos con leve expectoracin, con
disminucin de ruidos respiratorios en bases de predominio derecho, se observa leve tinte ictrico central. Signos vitales TA 130/100
mmHg, FR 27, FC 98, Temperatura 37.6 grados. Se realiza tele de torax donde se observa elevacin del diafragma de predominio
derecho.

PREGUNTA
Cul es la conducta teraputica a seguir ms adecuada.

RESPUESTA
a.- Iniciar antibitico de amplio espectro.
b.- Realizar USG de via biliar.
c.- Indica bloqueadores de H.
d.- Realiza panendoscopia.

CIRROSIS HEPATICA:
CIENCIAS BASICAS: Cirrosis es una definicin histopatolgica, y tienen una variedad de causas, existe lesin hepatocelular, fibrosis
heptica, formacin de ndulos de regeneracin, dando como resultado decremento en la funcin heptica. Recordar: Factores de
coagulacin dependientes de la vitamina K son: II, VII, IX y X. SALUD PBLICA: Causas de cirrosis: alcoholismo (60-70%),
esteatohepatitis no alcohlica (10-15%), hepatitis viral crnica, VHB,VHC (10%), hepatitis autoinmune, cirrosis biliar, cirrosis biliar
primaria (5%), colangitis esclerosante, colangiopata autoinmune, cirrosis cardiaca (Sx. Budd-Chiari= ICD y pericarditis constrictiva),
hemocromatosis, enfermedad de Wilson, deficiencia de antitripsina
1
, fibrosis qustica. PATOGENIA: El principal evento en la gnesis
es el aumento en la produccin y depsito de tejido de colgeno (I,III, el colgeno tipo I es ms raro,). Al ser estimuladas las clulas
hepticas estrelladas expresan ms RNA mensajero para sintetizar colgeno, esto produce la fibrosis heptica, y est en combinacin
con distorsin vascular genera hipertensin portal, la cirrosis se considera irreversible en estadios avanzados; es probable que la
diferencia entre la fibrosis reversible e irreversible sea el tipo de colgeno depositado (tipo I no es degradable, tipo III, s lo es).
Clasificacin histopatolgica: micronodular, tpica de cirrosis alcohlica, se caracteriza por un tamao uniforme de ndulos, <3mm de
dimetro. La variedad macronodular >3mm, se presenta secundaria a hepatitis viral o autoinmune
DIAGNOSTICO: Cuadro clnico: Puede estar ausente, anorexia, fatiga,
trastornos del sueo, nausea, vmito, diarrea, dolor vago en cuadrante
superior der. (70% hgado palpable y firme), debilidad, fiebre, ictericia,
amenorrea, impotencia, infertilidad, angiomas en araas, eritema
palmar, hipertrofia de glndulas paratiroides y lagrimales, contractura
de Dupuytren (fibrosis de la fascia palmar), lneas blancas en la uas,
ginecomastia, atrofia testicular, hepatoesplenomegalia, ascitis,
sangrado gastrointestinal, encefalopata heptica. Laboratorio: Anemia
(microcitica, debida a prdida de sangre, macrocitica, debida a
deficiencia de folato; hemoltica llamada Sx. de Zieve), pancitopenia
(hiperesplenismo), PT prolongado, hiponatremia, alcalosis hipocalemica,
intolerancia a la glucosa, hipoalbuminemia. hiperbilirrubinemia,
aumento de AST, ALT, FA y globulinas, el Us abdominal muestra
hepatomegalia dependiente de lbulo izq. O caudado, descarta ascitis y
hepatocarcinoma, permite evaluar, vena porta, esplnica y heptica.
Diagnstico definitivo a menudo depende de la biopsia de hgado
(percutnea, transyugular o abierta). El Child-Pugh es usado para predecir la severidad de la cirrosis y el riesgo de complicaciones.
COMPLICACIONES: 1.-Hipertensin portal: Se define como el aumento sostenido de la presin hidrosttica en el sistema venoso portal,
que genera un gradiente de presin entre la vena porta y la vena cava inferior >5mmHg, lo cual favorece el desarrollo de circulacin
colateral y derivacin del flujo portal hacia la circulacin sistmica, es causada por aumento en la resistencia intrahepatica e
incremento en el flujo sanguneo esplcnico secundario a vasodilatacin, sus 3 principales complicaciones son: varices gastroesofgicas
con hemorragia, ascitis e hiperesplenismo. La vena porta est formada por vena gstrica isq., esplnica y mesentrica superior. 2.-
Varices gastroesofgicas con hemorragia: Son vasos colaterales generalmente con un gradiente de presin >12mmHg, para la
prevencin primaria del sangrados se utilizan bloqueadores beta no selectivos (propanolol). 3.-Ascitis: acumulacin de lquido en
cavidad peritoneal, se considera la complicacin ms frecuente de la cirrosis, se asocia a una mortalidad de 50% a los 2 aos, dx., por
exploracin fsica y USG abdominal, se debe realizar paracentesis para determinar la albumina, protenas, glucosa, LDH, conteo celular,
tincin de Gramm y cultivo. El tx., consiste en restriccin de la ingesta de Na, diurticos (espironolactona, furosemida). 4.-Peritonitis
bacteriana espontanea (PBE): ocurre en 10-20% de pacientes con cirrosis y ascitis, factores de riesgo, protenas totales menores a 1g/dl,
antecedente de PBE y sangrado gastrointestinal, se manifiesta con fiebre dolor abdominal y encefalopata, dx., con conteo celular de
liq. de ascitis debe tener >250 neutrfilos/mm
3
en el cultivo lo mas frecuente es E. coli, Klensiella (70%), Enterococos y Streptococcus
(30%). 5.- Encefalopata heptica. 6.- Otros com: sndrome hepatorrenal tipo I,II , sndrome hepatopulmonar, hipertensin
portopulmonar, malnutricin, coagulopata, fibrinlisis, trombocitopenia, osteopenia, osteoporosis,, anemia, hemolisis
TRATAMIENTO: La cirrosis heptica compensada no requiere de tratamiento, especifico, solo dieta restringida en protenas (1-1.2
g/Kg), evitar consumo de alcohol y endoscopia al momento del dx., y peridica, cuando no hay varices esofgicas o son muy pequeas,
realizar cada 2 aos; con varices grado II y III iniciar bloqueadores , y cuando hay sangrado endoscopia cada 6-12 meses. El nico
tratamiento que modifica claramente el pronostico es el trasplante heptico. Trasplante de hgado: Indicaciones: Nios; atresia biliar,
hepatitis neonatal, fibrosis heptica congnita, enfermedad de Angilles, enfermedad de Bylers, desordenes inherentes del
metabolismo, enfermedad de Wilsons, enfermedades de depsito lisosomal, Crigler-Najar tipo I, hipercolesterolemia familiar. Adultos;
cirrosis biliar primaria y secundaria, colangitis esclerosante primaria, hepatitis autoinmune, enfermedad de Carolis, cirrosis
MANUAL DE TRABAJO DEL CURSO ENARM CMN SIGLO XXI
CURSO ENARM CMN SIGLO XXI TEL: 36246001 Pharmed Solutions Institute PGINA 77

criptogenica, trombosis venosa heptica, hepatitis crnica con cirrosis, hepatitis fulminante, cirrosis alcohlica, malignidad
hepatocelular primaria, esteatohepatitis no alcohlica Contraindicaciones: Infeccin extrahepatobiliar no controlada, sepsis activa sin
tratar, anomalas congnitas incorregibles que limitan la vida, abuso de sustancias o de alcohol, enfermedad cardiopulmonar avanzada,
malignidad extrahepatobiliar, colangiocarcinoma, SIDA. CIRROSIS ALCOHOLICA: Se debe documentar abuso de alcohol en la historia
clnica, asintomtica, clnica >10 aos de abuso de alchol. En este tipo de cirrosis la atrofia testicular debida a alteraciones hormonales o
efecto toxico del alcohol. Pueden presentar anemia hemoltica por efecto de la hipercolesterolemia, en la membrana eritrocitica, la cual
genera acantocitos. Es caracterstica la elevacin de AST sobre ALT, lo produce relacin AST/ALT >2. La biopsia heptica reporta
necrosis, cuerpos de Mllory e infiltracin por neutrfilos. ESTEATOHEPATITIS NO ALCOHOLICA: Es la causa mas frecuente de cirrosis
criptognica. Enfermedad metablica adquirida que se origina por el depsito de triglicridos en los hepatocitos, asociada a inflamacin
y fibrosis, ciertos factores predisponen como, sexo femenino, obesidad, DM y dislipidemia. Los pacientes suelen presentar dolor en
hipocondrio derecho y hepatomegalia, Laboratorio: hipertransaminemia con predominio de ALT, diagnostico mediante biopsia
heptica. CIRROSIS BILIAR PRIMARIA (CBP): Enfermedad progresiva, con inflamacin y destruccin de conductos biliares
intrahepticos, lo cual produce colestasis crnica y cirrosis; predomina en mujeres (95%), entre 30-65 aos de edad, probablemente
debida a un trastorno autoinmune adems se asocia con frecuencia a enfermedades autoinmunes. Asintomticos, manifestacin inicial
con frecuencia es prurito, predominio nocturno y se asocia a piel seca, se puede ver ictericia, hiperpigmentacin, xantelasma, y
xantomas, hepatoesplenomegalia, malabsorcin intestinal. Laboratorio: anemia normocitica normocromica, eosinofilia, elevacin de
FA, GGT, AST, ALT, bilirrubinas normales al principio, despus elevadas, hipergamaglobulinemia, anticuerpos antimitocondriales (AMA),
presentes hasta en 95%, tienen sensibilidad de 95% y especificidad de 98%. Anticuerpos antinucleares (ANA) en 70% de los casos.
Tratamiento alivi0 del prurito (colestiramina, colestipol, fenobarbital), correcion de las consecuencias de malabsorcin intestinal. El tx.,
especifico de la CBP se basa en administracin de esteroides y ac. ursodexosicolico (13-15mg/Kg/24h), y en el trasplante heptico.
COLANGITIS ESCLEROSANTE PRIMARIA (CEP): Inflamacin y fibrosis de los conductos biliares intra y extrahepticos, los cuales se
estenosan y obliteran, ocasionando cirrosis heptica. La CEP suele asociarse a colitis ulcerativa crnica inespecfica (40-80%), LES y
arttris reumatoide. Predomina en hombres. Presentan astenia progresiva, prurito e ictericia. El hallazgo caracterstico en la biopsia es la
fibrosis concntrica periductal (en cascara de cebolla). No hay tratamiento especfico, se debe considerar el trasplante heptico, ya que
la supevivencia despus del dx., es de 12 aos.

CASO CLINICO
Hombre de 65 aos, con antecedentes de cirrosis heptica por hepatopata crnica VHC (Child-Pugh 5). Habia sufrido un episodio de
hemorragia digestiva alta por sangrado de varices esofgicas, resuelto con ligadura. Mientras pasea y de forma aguda presenta dolor
intenso a nivel hipogstrico. A la llegada del equipo de asistencia mdica extrahospitalario el paciente ya est inconsciente (GCS 7), con
signos de mala perfusin perifrica, intensa palidez y situacin de shock. No hay evidencia de hemorragia digestiva alta. Se procede a
intubacin y reposicin de volemia.

PREGUNTA
Cual de los siguiente sitios de sangrado en la cirrosis heptica es la menos frecuente y muy grave?

RESPUESTA
a.- Hemo-retroperitoneo.
b.- Hemo-peritoneo.
c.- Varices esofagogastricas.
d.- Varices hemorroidales.

PREGUNTA
Considerando la respuesta anterior cuales signos y sntomas son indicativos del sitio de sangrado?

RESPUESTA
a.- Dolor abdominal intenso de inicio brusco y distencin sbita.
b.- Distension abdominal sbita y signos de choque.
c.- Manifestaciones de hipovolemia y dolor abdominal.
d.- Signos de choque y dolor abdominal intenso.

CASO CLINICO
Mujer de 70 aos con cirrosis heptica por el virus de la hepatitis C, en estadio B de Child, con trombosis portal, episodios de
hemorragia digestiva alta, descompensacin hidrpica y un ltimo ingreso por ascitis e hidrotrax secundario, que se resolvi con
tratamiento diurtico. Acudi a urgencias por un cuadro de ascitis, edemas y aumento de la disnea de 15 das de evolucin. En la
auscultacin pulmonar tena disminucin del murmullo vesicular en los dos tercios inferiores del hemitrax izquierdo. En el hemograma
destacaba nicamente la trombopenia (76.000 clulas/l) y en la bioqumica, la elevacin de la creatinina (1,9 mg/dl), bilirrubina (3,1
U/l), gammaglutamiltranspeptidasa (71 U/l) y fosfatasa alcalina (268 U/l), as como el descenso de la albmina (2,3 U/l). En la
radiografa de trax se objetivaba un derrame pleural que ocupaba los dos tercios inferiores del hemitrax izquierdo y sin focos de
compensacion.

PREGUNTA
Considerando el cuadro clnico, cual de las siguientes complicaciones es la mas probable que presenta este caso?



MANUAL DE TRABAJO DEL CURSO ENARM CMN SIGLO XXI
CURSO ENARM CMN SIGLO XXI TEL: 36246001 Pharmed Solutions Institute PGINA 78

RESPUESTA
a.- Peritonitis.
b.- Neumonia.
c.- Sindrome hepato-renal.
d.- Derrame pericardico.

CASO CLINICO
Paciente de sexo femenino de 61 aos de edad, con antecedentes de diabetes mellitus tipo 2, hipertensin arterial y cirrosis heptica
Child B de etiologa no precisada, Consult por cuadro de una semana de evolucin caracterizado por fiebre de predominio nocturno,
cuantificada hasta en 39C y precedida de un da de diarrea no disentrica y autolimitada. Se evalu en Servicio de Urgencia, se
realizaron exmenes dentro de los cuales destacan: hemograma sin leucocitosis ni desviacin a izquierda, pruebas de coagulacin,
electrolitos plasmticos y funcin renal normales. A los tres das presenta espectoracion la cual se cultiva con resultado de L.
monocytogenes.

PREGUNTA
Cual es el tratamiento de primera eleccin ya que el paciente es alrgico a la penicilinas, adems de la patologia de base?

RESPUESTA
a.- Cotrimoxazol.
b.- Eritromicina.
c.- Cloranfenicol.
d.- Tetraciclinas.

CASO CLINICO
Paciente de 38 aos ex-adicto a drogas por va parenteral sufre de forma espontnea sin traumatismo previo dolor abdominal de
localizacin dorsal con irradiacin a ambos hipocondrios, hipotensin arterial y anemizacin importante (5g/dl de hemoglobina), se
realiza TC abdominal ante la sospecha de rotura de anerurisma artico, y se evidencia la existencia de rotura de tumoracin heptica
(hepatocarcinoma) en segmento VIII sobre un hgado cirrtico y hemoperitoneo masivo. El paciente es intervenido quirrgicamente de
forma inmediata realizndose alcoholizacin de la lesin heptica y sutura hemosttica.

PREGUNTA
Cul es el agente etiolgico ms probable.

RESPUESTA
a.- Virus tipo A
b.- Virus tipo B
c.- Virus tipo C
d.- Virus tipo D

CASO CLINICO
Hombre de 65 aos con DHC Child A y HCC nico de 2 cm en lbulo heptico derecho. Sin ascitis al momento del diagnstico pero con
signos de hipertensin portal (HTP) en la resonancia magntica (RM) y adecuada permeabilidad de la vena porta. Fue definido etapa 1
segn la clasificacin de Okuda.

PREGUNTA
Cul es la conducta a seguir mas adecuada.

RESPUESTA
a.- Radioterapia.
b.- Quimioterapia.
c.- Quimioembolizacion.
d.- Transplante heptico.

PREGUNTA
Respecto a las METs en hgado, cual de las siguientes observaciones son menos probable?

RESPUESTA
a.- MTs heptica desde Ca. Mamario.
b.- MTs Heptica desde CA. de Vescula.
c.- Lesin solitaria de MT.
d.- Gran metstasis hiperecoicas creando efecto de masa.

CASO CLINICO
Masculino de 74 aos de edad, refiere cambios evacuatorios hace 2 meses, liquidas, amarillentas, cuatro por das, sin moco ni sangre, a
la semana se anexa al cuadro fiebre recurrente nocturna, no cuantificada precedida de escalofros la cual atena con medio fsicos. A
los 15 das se suma alcuadro dolor abdominal de fuerte intensidad tipo clico, Antecedente de esplenectoma comoresolucin a anemia
MANUAL DE TRABAJO DEL CURSO ENARM CMN SIGLO XXI
CURSO ENARM CMN SIGLO XXI TEL: 36246001 Pharmed Solutions Institute PGINA 79

hemoltica a los 46 aos. Padre fallece a los 73 aos por CA. Gstrico. Hermanos: 12 Vivos, 3 fallecen por CA. Pulmn, CA. seo. y CA.
Gstrico. Hijo fallecido por CA. Testicular. Alcohol desde los 22 aos hasta los 46 aos los fines de semana tipo cerveza llegando a la
embriaguez. Tabquico: Desde los 14 aos hasta los 46 aos 10 paquetes por ao. Refiere disminucin de 20 Kg aproximadamente en 2
meses. Cuello: Adenopata cervical de 2 cm aproximadamente. Pulmn: Crepitantes bilaterales difusos. Abdomen: Hepatometria
12/17/18 cm, bordes lisos irregulares, Hemograma: WBC 9.7, HGB 10.3, PLT 386. NEUT: 73%, LINF: 21,7%. BILIRRUBINA TOTAL 3,50
mg/dl. BILIRRUBINA DIRECTA: 2,12 mg/dl. FOSFATASA ALCALINA 200 UL. UROANALISIS: Bacterias moderadas, leucocitos 4-8 x cpo.
Trazas protenas, Pig. Biliares +++, Bilirrubina +++. TGO: 198,3 U/L. TGP: 30,7 U/L. ALBUMINA 2,5; PROTEINA C REACTIVA 36.

PREGUNTA
Considerando el cuadro, la evolucin y antecedentes, cual es la condicin secundaria que presenta el paciente al padecimiento de
base?

RESPUESTA
a.- Cirrosis
b.- Hepatitis Viral
c.- Adenoma heptico
d.- Hiperplasia adenomatosa

PREGUNTA
Considerando la presencia de los adenomas hepticos, cual de las siguientes afirmaciones cual es menos frecuente?

RESPUESTA
a.- Un adenoma hepatocelular es un tumor frecuente y nocanceroso del hgado.
b.- Los adenomas hepatocelulares inciden principalmenteen mujeres en edad frtil.
c.- En general, no presenta sntomas, de modo que muchos de los casos no se llegan a detectar.
d.- En casos moderados, una denoma puede hacerse canceroso.

PREGUNTA
Relacionado al carcinoma hepatocelular, cual de las siguientes afirmaciones es mas frecuente?

RESPUESTA
a.- Carcinoma hepatocelular en paciente con cirrosis.
b.- Carcinoma hepatocelular multifocal en paciente sin cirrosis.
c.- Un paciente con Sindrome de Bud-Chiari cronico con una lesion nodular sospechosa.
d.- Mismo paciente con el uso de contraste, ayuda a delimitar mejor la lesion.

CASO CLINICO
Mujer de 26 aos de edad. Acudi por un cuadro de una semana de evolucin de dolor en hipocondrio derecho que se acompaa de
astenia, nuseas, coluria, acolia, y febrcula. En la exploracin fsica slo destacaba la presencia de ictericia cutaneomucosa y un
reborde heptico doloroso a la palpacin a 4 cm de la arcada costal.

PREGUNTA
Cul es factor etiolgico ms probable.

RESPUESTA
a.- Viral.
b.- Alcohol.
c.- Oncologico.
d.- Autoinmune.

CASO CLINICO
Mujer de 77 aos diabtica, hipertensa y con hepatopata crnica por el virus de la hepatitis C que consult en Urgencias por cefalea de
dos das de evolucin con posterior aparicin de confusin, desorientacin, agitacion y fiebre de 39,2C. Tos productiva, con campos
pulmonares con ruidos crepitantes, FR 12, FC 92, TA 140/100 mmHg.

PREGUNTA
Cul es la conducta a seguir mas adecuada.

RESPUESTA
a.- Evaluar enzimas hepticas.
b.- Evaluar funcionamiento renal.
c.- Realizar medidas para encefalopata.
d.- Iniciar antibiticos de amplio espectro.



MANUAL DE TRABAJO DEL CURSO ENARM CMN SIGLO XXI
CURSO ENARM CMN SIGLO XXI TEL: 36246001 Pharmed Solutions Institute PGINA 80

CASO CLINICO
Paciente de 40 aos de edad, sexo masculino, con antecedente de depresin en tratamiento con psicoterapia. Consult con cuadro de
un mes de compromiso del estado general. Una semana antes de consultar present dolor en hipocondrio derecho, intermitente, que
no ceda con la ingesta de analgsicos. Tres das antes de consultar present fiebre no cuantificada. FC de 110 lx', FR de 29 rpm, temp
de 38,5C, murmullo disminuido en la base pulmonar derecha y dolor a la palpacin en hipocondrio derecho. Leucocitos 12.000
clulas/mm3, con neutrfilos de 78% y baciliformes de 1%; hemoglobina 12,6 g/dl; velocidad de eritrosedimentacin 85 mm/h;
protena C reactiva (PCR) 239 mg/dl; bilirrubina total 0,34 mg/ di; bilirrubina directa 0,11 mg/dl; fosfatasa alcalina 151 U/L; tiempo de
protrombina 71,9%; amilasa 44 U/L; lipasa 116 U/L. Se realiz ecotomografa abdominal que mostr una lesin de mrgenes irregulares
predominantemente hipoecognica, con zonas hiperecoicas en su interior, de 14 cm x 7 cm, ubicada en el lbulo heptico derecho,
antgeno carcinoembrionario (ACE), 0,6 ng/ml; alfa feto protena (AFP), 0,87 ng/ml, CA-19,9, 0,15 U/ml; anticuerpos IgM para hepatitis
A, negativos; antge-no de superficie de virus de hepatitis B, negativo; anticuerpos anti-hepatitis C, negativos; anticuerpos IgG e IgE para
hidatidosis, negativos; anticuerpos IgG para amebiasis, negativos.

PREGUNTA
Considerando el cuadro clnico y los estudios de laboratorio y gabinete, cual es su impresin diagnostica inicial?

RESPUESTA
a.- Absceso Hepatico.
b.- Quiste Hepatico.
c.- Sarcoma Hepatico.
d.- Hepatocarcinoma.

PREGUNTA
El paciente continuo con fiebre motivo por lo que se instalo tratamiento antibitico, cual es la conducta diagnostica mas adecuada en
este momento?

RESPUESTA
a.- Realizar Biopsia Percutanea.
b.- Realizar Laparatomia Exploratoria.
c.- Realizar Gamagrafia Hepatica.
d.- Realizar Arteriografia Hepatica.

PREGUNTA
Considerando la respuesta previa se determino un HCTS, cual de las siguientes aseveraciones es mas probable para su desarrollo?

RESPUESTA
a.- La patognesis de la transformacin sarcomatoide del carcinoma heptico no ha sido clarificada. Sin embargo, la evidencia sugiere
que las clulas fusadas del hepatocarcinoma representan una diferenciacin sarcomatosa de las clulas epiteliales ms que una
combinacin de sarcoma y hepatocarcinoma.
b.- El mecanismo de degeneracin, necrosis y regeneracin de las clulas de carcinoma debidas a frmacos antineoplsicos o
quimioembolizacin transarterial, han sido postulados como posibles inductores en otros casos.
c.- Los trabajos previos muestran combinaciones con componentes condrosarcomatosos, rabdo-miosarcomatosos y con presencia de
clulas gigantes tipo osteoclasto.
d.-Los hepatocarcinomas en general, se desarrollan en hgados previamente daados con cirrosis post virales y de tipo nutricio-
alcohlico. En el caso de los hepatocarcinomas con componente sarcomatoide.

HEPATITIS:
CIENCIAS BASICAS: Definicin: Infeccin sistmica que afecta principalmente el hgado (inflamacin). Causada por los virus
hepatotroficos (A,B,C,D,E) que es la causa ms comn y por otros virus (VEB, CMV, coxackievirus, etc), alcohol, drogas, hipotensin e
isquemia y enfermedades de la va biliar. Hepatitis aguda: Enfermedad inflamatoria de hgado, de menos de 6 meses de evolucin, por
lo general produce anorexia, ictericia, coluria, y alteraciones en pruebas de funcin heptica. Hepatitis crnica: inflamacin persistente
de hgado, de ms de 6 meses de evolucin, puede desarrollarse de manera favorable o condicionar cirrosis heptica. Hepatitis
fulminante: necrosis masiva del hgado, se manifiesta por encefalopata heptica aguda, coagulopata, insuficiencia renal y coma,
aparece en una enfermedad de hgado con menos de 8 semanas de evolucin. SALUD PUBLICA: 85% de los pacientes con hepatitis A,
tienen recuperacin clnica y bioqumica aprox. a los 3 meses. Cerca de 5% de la poblacin mundial est infectada con VHB, en EU, es
responsable de 5-10% de los casos de hepatopata crnica y cirrosis. El riesgo de transmisin por puncin con aguja es VHB se transmite
en 30% de las exposiciones, VHC en 3% y VIH en 0.3%. La prevalencia mundial es de 0.5-2%, en Mxico es de 1.4% La evolucin natural
de la infeccin por VHC es la hepatitis crnica activa (hasta 74%). El riesgo de presentar carcinoma hepatocelular en pacientes con
cirrosis por VHC es de 1-4% por ao. DIAGNOSTICO: Cuadro clnico: Manifestaciones comunes: Malestar, nausea, vmito, diarrea,
fatiga, febrcula seguida de orina oscura, ictericia y hepatomegalia dolorosa puede ser subclnica y detectarse por niveles elevados de
AST y ALT. Manifestaciones poco comunes: vasculitis, artritis, neuritis ptica, anemia aplasica, aplasia de serie roja, y mielitis transversa.
HEPATITIS VIRALES AGUDAS Y CRONICAS: HEPATITIS A: El VHA, es picornavirus RNA no cubierto. Principal va de transmisin fecal-oral (
alimentos contaminados, guarderas), poblacin de riesgo son nios y adolescente, la infeccin no suele ser grave (autolimitada),
seguido del proceso se producen anticuerpos contra el virus de la hepatitis A, lo cual confiere inmunidad. Incubacin promedio de 30
das, la hepatitis se produce por dao secundario de la respuesta del sistema inmune del husped mediada por dao citotxico por
linfocitos T CD8 y clulas NK. Puede presentarse desde asintomtica hasta insuficiencia heptica aguda. DIAGNOSTICO: IgM anti-VHA
MANUAL DE TRABAJO DEL CURSO ENARM CMN SIGLO XXI
CURSO ENARM CMN SIGLO XXI TEL: 36246001 Pharmed Solutions Institute PGINA 81

en una muestra de suero de convalecientes aguda o temprana, en la exposicin pasada se caracteriza por anticuerpos anti-IgG positivos
con anti-IgM negativos (las cuales se negativizan a los 6 meses), elevacin de ALT, AST, bilirrubinas y FA. TRATAMIENTO: Medidas de
soporte, cuando hay complicaciones como falla heptica fulminante, valorar terapia intensiva y trasplante heptico. PREVENCION:
Inmunizacin pasiva posexposicin, inmunoglobulina humana 0.2 ml/Kg IM, proteccin por 6 meses, indicado en pacientes que
planeen viajar a zona endmica, o contacto ntimo con alguien infectado con VHA. Inmunizacin activa: Vacuna contra VHA (1ml IM y
refuerzo a los 6 y 12, 0.5ml para nios) indicado en pacientes con enf. heptica crnica, varones homosexuales, drogadictos, personal
de salud que trabaje con VHA. HEPATITIS B: El VHB, es hepadnavirus DNA, est compuesto por la polimerasa con actividad de
transcriptasa reversa, una proteincinasa rodeada por el antgeno del ncleo (HBcAg) y una envoltura que contienen el antgeno de
superficie glucoproteico (HBsAg). El HVB puede causar accin citopatica directa y a travs de respuesta inmune. Factores de riesgo son;
contacto sexual, uso de drogas intravenosas (20%), transfusiones, contaminacin con suero o leche materna, principal va perinatal.
Periodo de incubacin de 40-150 das, puede presentarse desde asintomtica hasta hepatitis fulminante (<1%) Hepatitis aguda:
malestar general, nausea, vomito, anorexia, dolor en hipocondrio der., ictericia que dura de 1-3 meses (70% anictericos). Hepatitis
crnica: Ag de superficie positivo por mas de 6 meses, concentraciones sricas de DNA >10 copias/ml, aumento de aminotransferasas
persistentes, biopsia heptica que muestre inflamacin. DIAGNOSTICO: HBsAg en suero infeccin aguda o crnica (si persiste por mas
de 6 meses). IgM anti-HBc (indica infeccin aguda o reciente). La prueba ms sensible para la deteccin de HVB DNA en suero (actividad
viral replicativa en suero). Paciente con AntiHBs es una persona sana vacunada. Las transaminasas pueden aumentar hasta 1000 o
2000 y si permanecen por ms de 6 meses se vuelve crnica. TRATAMIENTO: En aguda es de sostn, en la crnica, suprimir la
replicacin del virus con el fin de evitar progresin a cirrosis y cncer (realizar tamizaje de alfafetoproteina y ultrasonido heptico cada
6 meses). El interfern alfa-2b, 5-10 millones de U, diarias, 3 veces a la semana SC, por 16 sem. Actualmente se usa alfa-2b pegilado, se
aplica una vez por semana a una dosis de 180mg SC por 48 sem. La lamivudina (3TC), inhibe la transcriptasa reversa, disminuyendo la
carga viral, otros adefovir y entecavir. PREVENCION: Vacunacin administrar HBsAg, para estimular la produccin de antiHBs, en
individuos no infectados, efectividad >95%; vacunacin universal en todos los recin nacidos, as como en trabajadores de la salud,
paciente s en hemodilisis, familiares y parejas sexuales de pacientes con HBsAg. Vacunacin via IM a los 0, 1, 6 meses. HEPATITIS C: El
VHC, es RNA monocatenario. Periodo de incubacin de 2 meses. Factores de riesgo, uso de drogas IV, transfusin sangunea antes de
1990, la transmisin vertical es infrecuente, est mas relacionada con una coinfeccin con VIH-1, en la madre. La transmisin sexual
tiende a ser menos frecuente que otros virus (VIH), por la menor carga viral en lquidos y tejidos genitales. Las clulas blanco del VHC
son los hepatocitos y lis linfocitos B. La sintomatologa aparece aprox. De 6-8 sem de la exposicin, puede pasar asintomtico, ictrica,
malestar general, nausea. La infeccin crnica se caracteriza por presentar periodos prolongados sin manifestaciones, la aguda
progresara a crnica, con diversos grados de hepatitis, fibrosis y cirrosis; entre los factores que promueven la progresin clnica se
observan el consumo de alcohol, coinfeccin con VIH-1 o VHB, gnero masculino, y edad mayor al momento de infeccin. Otras
manifestaciones estn asociadas a enf. autoinmuines (tiroideas, DM). DIAGNOSTICO: La prueba de rutina es la serolgica mediante
ensayo inmunoenzimtico, que detecta anticuerpos a partir de 4-10 sem posteriores a la infeccin. El inmunoblot se usa como prueba
confirmatoria. Los estudios moleculares se basa en deteccin del RNA viral mediante PCR, pueden ser cuantitativos (limite menor de
deteccin 100 copias/ml) o cualitativos, la carga viral nos ayuda con la evolucin y respuesta a tx. Los anticuerpos anti-VHC se pueden
detectar en >97% de las personas a los 6 meses posteriores a la exposicin. La biopsia es el estndar de oro para determinar la actividad
de VHC, es el nico predictor pronstico de la progresin. TRATAMIENTO: Indicado en: carga viral positiva, biopsia heptica con fibrosis
> grado 1 o enfermedad heptica compensada (CHILD A o B). Contraindicaciones: cirrosis descompensada, embarazo, enf. psiquitrica,
consumo activo de drogas, enf. pulmonar o cardiaca grave, DM descontrolada, enf. autoinmunes. Frmacos: interfern alfa en
monoterapia o combinado con ribavirina; la combinacin confiere 55% de respuesta viral sostenida. HEPATITIS D: El VHD, es RNA
monocatenario, requiere la presencia del VHB para su supervivencia y replicacin, la forma ms comn de contagio es por compartir
agujas en personas que usan drogas IV, tambin transmisin sexual y perinatal. La coinfeccin aguda es la infeccin por exposicin
simultnea a VHB y VHD. La superinfeccin es la exposicin del VHD, en un individuo ya infectado con VHB. El diagnstico con IgM e IgG
antiVHD o por la deteccin de RNA de VHD en suero. HEPATITIS AIUTOINMUNE: Sospechar ante una hepatitis que presente aumento
de globulinas plasmticas y autoanticuerpos especficos; es una enfermedad necroinflamatoria crnica del hgado de causa
desconocida, parece haber predisposicin gentica (HLA-B8, DR3 y DR5). La hepatitis autoinmune se divide en 3 de acuerdo con los
anticuerpos en suero: Tipo I: Clsica o lupoide, ms comn en mujeres jvenes, 70% menores de 40 aos, los anticuerpos que se
distinguen son los antinucleares (ANA), antimusculo liso (AML), antiactina y antineutrofilos (pANCA) e hipergammaglobulinemia. Tipo II:
Ms comn en Europa, en especial en nios, aqu encontramos annticuerpos anticromosomas de hgado y rin tipo 1 (anti-LKM1),
asociada a enf. inmunolgicas, tiende a progresar rpido a cirrosis. Tipo III: 90% mujeres entre 20-40 aos, se observa anticuerpo anti-
SLA/LP, amplia relacin con tiroiditis autoinmune. La hepatitis autoinmune es de progresin lenta, como hepatitis aguda adems,
mltiples telangiectasias, estras, cutneas, acn, hirsutismo, hepatomegalia, artralgias y amenorrea. Para tx se usan glucocorticoides
de sntesis (prednisona, metilprednisolona), tambin aziatropina en biterapia con cortoicoesteroides (xito de 80-90%). HEPATITIS
ALCOHOLICA: Se presenta con niveles de aminotransferasas <300-500 con AST: ALT >2.1 y GGT elevada, y deficiencia concomitante de
vitamina B12, los gravemente afectados presentan al inicio fiebre, hepatomegalia, leucocitosis, ictericia y coagulopatas, asi como
manifestaciones de hipertensin portal, es comn que desarrollen cirrosis. El pronstico se ve en particular afectado por la presencia
de elevadas concentraciones de bilirrubina (>12mg/dl), ascitis e insuficiencia renal. Tx.: indicado en pacientes con encefalopata,
administracin de prednisona, pentoxifilina por un mes disminuye mortalidad e incidencia de sx. hepatorrenal. TOXICIDAD POR
PARACETAMOL: Se presenta con dosis mayores de 10 gr, puede presentarse en pacientes con desnutricin o alcoholismo con 2-6 gr,
parece proveer un reservorio de grupos sulfidrilo y esto obliga a los metabolitos txicos o estimulacin de la sntesis de glutatin
heptico si la falla heptica se presenta en las primeras 4 hrs, el tx.: consiste en la administracin de carbn activado o colestiramina y
lavado gstrico. La N-acetilcisteina debe ser iniciado dentro de las primeras 8 hrs de la ingestin, pero puede ser efectivo incluso en las
24-36hrs despus de la sobredosis, se prefiere va oral con una dosis carga de 140mg/Kg y despus 70mg/Kg cada 4 hrs por 17 dosis.
Otros medicamentos que producen hepatitis son: amiodarona. Azoles, isoniacida, metildopa, fenitoina, rifampicina, sulfas. Hay que
observar al paciente de 72-96hrs para evaluar dao heptico.


MANUAL DE TRABAJO DEL CURSO ENARM CMN SIGLO XXI
CURSO ENARM CMN SIGLO XXI TEL: 36246001 Pharmed Solutions Institute PGINA 82

CASO CLINICO
Varn de 33 aos no fumador con el nico antecedente personal de hipercolesterolemia a tratamiento con dieta. Realiza ejercicio fsico
de forma regular y su trabajo es sedentario. Refiere malestar general acompaado de cefalea, astenia y fiebre vespertina de hasta 39,2
C, acolia y coluria. En la EF no se observa ictericia, se auscultan niveles hidroaereos aumentados, Refiere haber comido ostras vivas
una semana antes, las PFH se encuentran en parmetros elevados, mas un patrn colestasico, se realiza serologa resultado positivo a
virus a hepatitis A (IgM positiva e IgG negativa).

PREGUNTA
Cual es la complicacin mas frecuente observar en esta patologia?

RESPUESTA
a.- Hepatitis recidivante.
b.- Hepatitis colestsica.
c.- Hepatitis aguda fulminante.
d.- Sndrome post-heptico.

PREGUNTA
Considerando la fisiopatogenia del caso, en cuanto tiempo se puede eliminar el virus por las heces?

RESPUESTA
a.- Una semana antes de la clnica.
b.- Durante la presentacin de la clnica.
c.- Una semana despus de la clnica.
d.- dos semanas despus de la clnica.

PREGUNTA
Habitualmente la acolia y la coluria se presenta cuantos das antes de la ictericia?

RESPUESTA
a.- 1 a 5 dias.
b.- 6 a 10 dias.
c.- 11 a 15 dias.
d.- 16 a 20 dias.

CASO CLINICO
Hombre de 27 aos sin antecedentes mrbidos, consult por compromiso del estado general y fiebre de 5 das, con posterior aparicin
de ictericia. Se hospitaliz con bilirrubina total de 13 mg/dl, aminotransferasa oxalactica de 1.977 mU/ml y pirvica de 5.975 mU/ml,
fosfatasas alcalinas de 157 U/L, y tiempo de protrombina 15%. Del panel viral result positivo un HBsAg. Evolucion con compromiso de
conciencia hasta Glasgow 8, con tomografa computada (TC) de cerebro normal. Fue recibido en la Unidad de Cuidados Intensivos con
Glasgow 5 sospechndose broncoaspiracin. Se conect a ventilacin mecnica, se iniciaron antibiticos y N-acetil cistena. La TC de
trax revel neumona, y la de cerebro edema cerebral leve. La ecotomografa abdominal muestr hgado normal y ascitis leve. Dentro
del estudio etiolgico presenta serologa para virus hepatitis C y A y estudio de autoinmunidad (anticuerpos y recuento de
inmunoglobulinas) negativos. Se repiti el estudio del VHB destacando aparicin de anticuerpos contra el HBsAg (antiHBs) y
negativizacin del HBsAg, con anticore positivos, IgM y total. Se realiz ELISA para VIH, que result positivo, y recuento de CD4 de 634
clulas/ul. La carga viral (CV) del VHB fue de 140 copias/ml, y la del VIH 16.900 copias RNA/ml. 12 dias despus fallece el paciente.

PREGUNTA
Cual de las siguientes observaciones sobre el presente caso clnico explica mas adecuadamente la complicacion del paciente?

RESPUESTA
a.- Falla Hepatica Fulminante por manejo inoportunamente diagnosticada y tratada.
d.- La falla hepatica por VHB con HIV ocurre en aproximadamente 20 %.
c.- En la falla heptica, mientras mayor es la respuesta del hospedero mayor es su probabilidad.
d.- En estados de inmunodepresin como es el tener VIH avanzado, el desarrollo de Falla Hepatica sea infrecuente.

PREGUNTA
Considerando la evolucin del caso, cual es el periodo necesario para considerar falla heptica fulminante.

RESPUESTA
a.- Dentro de las 72 horas.
b.- Dentro de la primera semana.
c.- Dentro de las primeras 8 semanas.
d.- Dentro de las primeras 12 semanas.

PREGUNTA
De las siguientes observaciones relacionadas a esta patologia cual es la menos probable?
MANUAL DE TRABAJO DEL CURSO ENARM CMN SIGLO XXI
CURSO ENARM CMN SIGLO XXI TEL: 36246001 Pharmed Solutions Institute PGINA 83

RESPUESTA
a.- Mexico se encuentra dentro de los pases de baja seroprevalencia.
b.- Se estima que hay almenos de 110,000 portadores crnicos.
c.- El VIH tiene una mayor virulencia que el VHB.
d.- La infeccin crnica por el VHB despus de una exposicin aguda es del 5 %.

PREGUNTA
La infeccin crnica por el VHB, tiene cuatro fases, de acuerdo a la presencia o ausencia del HBeAg, carga viral, nivel de ALT y hallazgos
histolgicos, cual de las siguiente no es correcta?

RESPUESTA
a.- Inmunotolerancia.
b.- Portador activo.
c.- Inmunoeliminacion.
d.- Hepatitis crnica HBeAg negativa.

CASO CLINICO
Masculino de 70 aos con hepatitis crnica por el VHC genotipo 1 de cronologa incierta, infeccin pasada por el VHB y bebedor de
unos 60 gramos de alcohol al dia. Se observo elevacin de GGT y FA, en la ecografa/TAC abdominal convencional en fase venosa se
puso de manifiesto una tumoracin de 7 cm de caractersticas solidas, en lbulo heptico izquierdo, sobre el hgado de morfologa
normal, la biopsia revelo que se trataba de un CHC, sin presencia clnica ni paraclinica de cirrosis previa y el valor de la alfafetoproteina
era de 6.9, se realizo tratamiento QX.

PREGUNTA
Considerando el caso clnico cual es la tasa de sobrevida al ao del presente caso?

RESPUESTA
a.- del 15 %.
b.- del 45 %.
c.- del 75 %
d.- del 95%.

PREGUNTA
Relacionado a la patologia descrita en el caso previo, cual de las siguientes aseveraciones no es cierta?

RESPUESTA
a.- La hepatitis C es una hepatopata crnica que cursa silenciosa y tiene una prevalencia de 1 a 1.9 en Mxico.
b.- La muerte por infeccin crnica de VHC estn esencialmente relacionadas con la descompensacin heptica, enfermedad heptica
terminal y el carcinoma hepatocelular.
c.- Las pruebas serolgicas actuales permiten distinguir entre infeccin aguda, crnica o resuelta, por lo cual se puede llevar un buen
monitoreo.
d.- La infeccin por el VHC se ha situado como la principal causa de hepatopata crnica, cerca del 27 % de los casos de cirrosis y el 25 %
de los casos de hepatocarcinomas.

PREGUNTA
Cual de las siguientes aseveraciones no es correcta en el diagnostico de infeccin por VHC?

RESPUESTA
a.- Durante la fase aguda de la infeccin la dereccion de acido nucleico puede generar inadecua interpretacin de resultados.
b.- Con una prueba anti-VHC positiva se debe realizar prueba PCR-RNA-VHC.
c.- Se debe solicitar prueba de RNA del VHC en los pacientes que se esta considerando la administracin de tratamiento antiviral.
d.- Se debe solicitar una prueba anti-VHC en pacientes con VIH para determinar comorbilidad.

CASO CLINICO
Mujer de 62 aos que refiere, desde hace 13 aos, mltiples episodios de visin borrosa en ambos ojos con recuperacin posterior
completa de la agudeza visual tratada con esteoides, TA 100/70mmHg, Glucosa 84 mg/dl, con antecedente de una hermana con el
mismo cuadro, presin intraocular normal, inicia cuadro actual con dispepsia, vomito, prurito, hiporexia, astenia, GOT 110, GPT 90, GGT
110, hipergammaglobulinemia (IgG 1620), ANA 1/320, AMA negativos, Antigeno de superficie VHB, Anticuerpo anti HBs, Anticuerpo
anti-HBc y Anticuerpo anti-VHC negativos, USG heptico normal.

PREGUNTA
Considerando las manifestaciones clnicas asi como los estudios de laboratorio y gabinete, cual es la conducta mas adecuada?

RESPUESTA
a.- Repetir estudios serolgicos ms sensibles y especficos.
b.- Realizar biopsia heptica percutnea.
MANUAL DE TRABAJO DEL CURSO ENARM CMN SIGLO XXI
CURSO ENARM CMN SIGLO XXI TEL: 36246001 Pharmed Solutions Institute PGINA 84

c.- Mantener en observacion y repertir estudios en 6 meses.
d.- Manejo conservador y corticoides.

PREGUNTA
Considerando la respuesta anterior, los antecedentes familiares y personales cual es el diagnostico mas probable?

RESPUESTA
a.- Hepatitis B.
b.- Hepatitis A.
c.- Hepatitis autoinmune.
d.- Hepatitis Ideopatica.

CASO CLINICO
Masculino de 64 aos con historia de multiples ingresos por alteraciones del estado de conciencia debido a insuficiencia heptica y
atrofia cortical, acude 14 dias despus su ultimo internamiento por alteracin del estado de conciencia, por fiebre, desorientacin y
dolor abdominal, urea 103, creatinina 2,6. LDH 699, CPK 864, GOT 57, GPT 46. La gasometra revela una acidosis respiratoria. En la
radiografia se observa torax enfisematoso e importante cardiomegalia sin signos neumnicos ni deramme pleural.

PREGUNTA
Se diagnostico como neumona, considerando el caso, cual de los siguientes agentes es mas frecuente?

RESPUESTA
a.- Streptococcus neumonae.
b.- Stafilococcus aureus.
c.- Listeria monocitogenes.
d.- Campylobacter jejuni.

CASO CLINICO
Masculino de 54 aos con DM, artrosis lumbar y hepatopata alcoholica, acude por dolor abdominal generalizado, fiebre, perdida de 20
kg en 6 meses. Leucocitosis a expensas de neutrofilos, acompaado de trobocitosis, glucosa 263, urea 185, creatinina 1,2. LDH 204 y
PCR 358. La radiografia de torax no muestra imgenes de condensacin.

PREGUNTA
Se diagnostico como neumona, considerando el caso, cual de los siguientes agentes es menos frecuente?

RESPUESTA
a.- Streptococcus neumonae.
b.- Stafilococcus aureus.
c.- Listeria monocitogenes.
d.- Campylobacter fetus.

CASO CLINICO
Masculino de 18 aos de edad que consulta de rutina, los laboratorios de rutina reportaron elevacin de GOT de 56 y el GPT de 78 las
cuales fueron incrementando lentamente desde hace 16 meses. GGT estaban discretamente aumentada y el resto de parmetros
hepticos eran normales. No haba serologa positiva a virus de la hepatitis B y C. Los autoanticuerpos para descartar hepatopata
autoinmune eran negativos. No hay antecedentes de de hemotransfusion ni quirrgicos. Hace ejercicio y como nico antecedente
refiere tomar finasterida 1 mg diario para evitar la alopecia desde hace 2 aos.

PREGUNTA
Cual es la conducta mas apropiada a seguir?

RESPUESTA
a.- Retirar finasterida.
b.- Realizar USG heptico.
c.- Realizar biopsia heptica.
d.- Repetir serologa.

CASO CLINICO
Varn de 60 aos con antecedentes de hipertensin arterial, adenoma de prstata e hiperuricemia ocasional. Haba sido diagnosticado
de miocardiopata dilatada idioptica haca 5 aos y segua tratamiento con digoxina, ibopamina, quinapril, espironolactona,
hidralacina, dicumarina, furosemida y nitratos transdrmicos, con lo que se mantena en CF-II. Presentaba adems fibrilacin auricular
paroxstica por lo que espordicamente haba recibido AMD por va oral (200 mg/da). No tena antecedentes de enolismo. Nunca se
detectaron alteraciones de la funcin heptica. Ingres por un cuadro de taquicardia ventricular sostenida a 190 por minuto con
morfologa de bloqueo de rama derecha. La arritmia curs con regular tolerancia clnica y sin constatarse en ningn momento signos de
compromiso hemodinmico (hipotensin, oliguria, etc.). Se haba restablecido el ritmo sinusal por medio de un bolo de 100 mg de
lidocana y se haba instaurado tratamiento con AMD intravenosa a dosis de 1.200 mg/da durante los 7 das previos. A su llegada a la
MANUAL DE TRABAJO DEL CURSO ENARM CMN SIGLO XXI
CURSO ENARM CMN SIGLO XXI TEL: 36246001 Pharmed Solutions Institute PGINA 85

unidad coronaria el paciente presentaba estabilidad hemodinmica con presin arterial de 120/80 mmHg y frecuencia cardaca
irregular a 90 por minuto con pulsos normales. Se apreciaban a la auscultacin cardaca un tercer sonidos y un dbil soplo holosistlico
en foco mitral. No haba edemas, ingurgitacin yugular o alteraciones en los ruidos respiratorios. Destacaba especialmente una ictericia
franca en piel y conjuntivas, y una evidente depresin del nivel de conciencia con flapping-tremor. No se apreciaron hepatomegalia,
ascitis ni ningn estigma de hepatopata crnica. En el electrocardiograma se observaba fibrilacin auricular a 90 por minuto con
bloqueo de rama izquierda. La radiografa de trax presentaba cardiomegalia y signos de hipertensin venocapilar pulmonar. En la
analtica al ingreso se observaba como nico dato destacable una alanina-aminotransferasa (ALT) de 52,4 kat/l (normal: 0,46-0,77).

PREGUNTA
Cual es la conducta mas apropiada a seguir?

RESPUESTA
a.- Retirar amiodarona.
b.- Realizar USG heptico.
c.- Realizar biopsia heptica.
d.- Repetir serologa

LINFOMAS
CIENCIAS BASICAS: Todas las neoplasias linfoides son malignas y derivan de una clula que ha sufrido mutacin maligna son
monoclonales, tambin alteran la inmunidad. 80-85%, tiene su origen en clulas B, los derivados de NK o histiociticas son raros. Se
clasifican en Linfoma de Hodgking (EH) y linfomas no Hodgkin (LNH). SALUD PUBLICA: LH: Constituye el 10% de todos los linfomas. Ms
frecuente de 25 aos y >60 aos. LNH constituyen el 3% de todas las neoplasias y son ms frecuentes que Los LH (60% de los linfomas
en adultos, edad promedio 64 aos) El subtipo histolgico ms comn es el difuso B de cel. Grande (LDGBC)s con incidencia en
individuos infectados por el VIH de ms de 100 veces la incidencia en la poblacin en geral (25%), seguido del linfoma folicular.
LINFOMA DE HODGKIN (LH): Origen en cel. B activados del centro germinal. El de celularidad mixta es el subtipo histolgico ms
frecuente en LH extranodales, mientras que el subtipo esclerosis nodular es predominante en los de afectacin nodal. PATOGENIA: Se
sospecha del VEB como agente de transformacin maligna, pacientes con mononucleosis infecciosa, tienen mayor riesgo, se reconoce
el genoma de VEB en 70%, tambin comn en casos de inmunodeficiencia. Caractersticas; extensin ordenada, localizado, rara vez
afecta ganglios mesentricos, rara extensin extraganglionar, muy comn dolor en ganglios linfticos, despus del consumo de alcohol
(<10%), en la variante esclerosis nodular DIAGNSTICO: Tpico escenario paciente joven, masculino, linfadenopatas perifricas no
dolorosas (70%), especialmente en regin cervical (60-80%) o masa mediastinal. Cuadro clnico: Sntomas B: prdida de peso
inexplicable de ms de 10%, en los ltimos 6 meses; fiebre inexplicable >38C, de duracin mayor a 2 semanas; sudoracin nocturna
profusa. Prurito en 10-15% de los casos, no es sntoma B. Fiebre Pel- Ebstein; fiebre cclica que dura de 3-5 das, seguida de un periodo
equivalente sin fiebre. Biopsia excisional ganglionar cel de Reed Sternberg 2 o ms ncleos con nuclolo prominente.
Inmunohistoquimica: positivos para CD15 y 30, nos ayuda a diferenciar entre LNH. PCR en ganglio detecta VEB en 60-80%. Tac de trax,
abdomen y pelvis, PET. ESTADIFICACION: La clasificacin mas usada es la Ann Arbor- Costwold.

ESTADIO I Afecta una sola regin de ganglios linfticos
Afecta a un solo rgano extralinftico
I
IE
ESTADIO II Afecta 2 o mas reas ganglionares en el mismo lado del diafragma
Afecta a un solo rgano extralinftico y sus ganglios regionales al
mismo lado del diafragma
II
IIE
ESTADIO III Afecta reas ganglionares a ambos lados del diafragma
Puede acompaarse de afeccin esplnica
rganos extralinftico localizados

IIIS
IIIE
ESTADIO IV Afecta de forma difusa a uno o ms rganos extralinftico (hgado,
pulmn, medula sea) con o sin involucro ganglionar

Agregar A cuando no se presentan sntomas y B si la persona refiere sntomas B
TRATAMIENTO: Radioterapia en estadios tempranos sin factores desfavorables. Quimioterapia; tratamiento de eleccin para casos
localizados (IA y IIA), 2 ciclos de quimioterapia con ABVD (adriamicina, vinplastina, bleomicina, y decarbazina)+ radioterapia en caso de
urgencias avanzadas ( sx. de vena cava superior, obstruccin de va area). Estadios III y IV quimio combinada. PRONOSTICO: Factores
de mal pronstico: Hb <11mg/dl, albmina <4g/dl, masculino, edad >45 aos, adenopata mediastinal, estadio IV, leucocitosis >15,000,
linfocitosis <500, histologa del tipo celularidad mixta, presencia de sntomas B. Despus de 3 aos sin enfermedad sobrevida >53%.
Tasa de curacin 65%, para pacientes con trastornos avanzados
LINFOMA NO HODGKING (LNH): Tumores slidos malignos de los tej. Linfoides, suelen provenir de cel. B (80%) Diseminacin menos
previsible, generalmente diseminada al realizar el dx., suele afectar ganglios mesentricos, frecuentemente afecta tej. extraganglionar.
Entre los factores de riesgo estn ; inmunodefoiciencias (LNH sistmico puede afectar el miocardio, particularmente en pacientes
inmunocomprometidos), tratamientos con radioterapia o quimioterapia, enfermedades autoinmunes y enfermedades previas. Los
linfomas no hodgkiniano son la causa ms frecuente de linfoma de cabeza y cuello. Se clasifican en bajo grado (transformacin en
agresivos, no curables, edad avanzada, sobrevida larga a 8 aos) y agresivos: potencialmente curables, sobrevida corta si no remiten;
cuadro clnico, sntomas B crecientes. Dentro de los de bajo grado; linfoma folicular, sus clulas neoplsicas se parecen a cel. B
normales, adenopatas indoloras generalizadas, incurables. LNH de bajo grado Linfoma de la zona marginal (MALToma= tumores de los
tej. linfoides asociados a mucosas) se origina, en bazo, ganglios y tej. extraganglionares, afectados por procesos inflamatorios
autoinmunitarios. LNH agresivo en general, debilidad, fatiga, prdida de peso, 50% prdida de peso, hepato y esplenomegalia,
hiperviscosidad (que genera mareos, sordera, cefalea, formacin de complejos y crioglobulinemia) linfoma difuso de cel. B grandes;
afecta una sola regin ganglionar o extraganglionar, primera manifestacin tubo digestivo, piel, hueso cerebro. LNH agresivo Linfoma
de Burkit; tumor de cel. B relativamente maduras, translocacin del gen c- myc en cromosoma 8, en tej. extraganglionar, el LB africano
MANUAL DE TRABAJO DEL CURSO ENARM CMN SIGLO XXI
CURSO ENARM CMN SIGLO XXI TEL: 36246001 Pharmed Solutions Institute PGINA 86

endmico; masa en mandbula, vsceras abdominales (riones, ovarios), agresivo pero responde a quimioterapia fcil. DIAGNOSTICO:
Sospecha clnica, biopsia de ndulo o rgano involucrado. Estos se diagnostican por resultados de laboratorios anormales; citopenias,
hipercalciemia, hiperuricemia, proteinemia o LDH elevada) TRATAMIENTO: LNH; linfoma folicular R-CHOP (rituximab, ciclofosfamida,
hidrixidaunomicina, vincristina y prednisona). LNH agresivos; R-CHOP ms de 50% de curacin. En todos los linfomas en casos
refractarios trasplante de medula sea. PRONSTICO:. LNH agresivos; tasa de remisin de 60-80% , para estadios II al IV. PREVENCION:
Hombre almacenar espermas, mujeres no embarazarse en 2 aos posteriores.

CASO CLINICO
Una paciente de 27 aos, del sexo femenino, se present en la Sala de Emergencias con palpitaciones de inicio sbito y disconfort
torcico. La presin arterial (PA) era de 68 x 40 mmHg y la frecuencia cardaca (FC) era de 184 lpm. Al examen fsico, la paciente
presentaba aumento de volumen de la mama izquierda y masa abdominal difusa. El electrocardiograma (ECG) de 12 derivaciones
realizado en la admisin revel taquicardia de complejo ensanchado con estndar de bloqueo de ramo izquierdo y concordancia
negativa en los electrodos precordiales. Con base en la presentacin inicial, el diagnstico de taquicardia ventricular monomrfica fue
establecido y la paciente fue sometida a cardioversin elctrica para el ritmo sinusal normal. El ECG fue repetido y mostr ritmo sinusal
normal, sin alteraciones del segmento ST (Figura 1B). La paciente recibi una infusin intravenosa de amiodarona y entonces fue
admitida en la Unidad de Terapia Intensiva (UTI). La investigacin fue negativa para isquemia miocrdica o embolia pulmonar. Una
ecocardiografa transtorcica (ETT) revel funciones ventriculares izquierda y derecha normales, derrame pericrdico posterior difuso y
engrosamiento del miocardio basal-septal, que no estaba presente en examen anterior realizado un ao antes para investigacin de
disnea durante actividad fsica.

PREGUNTA
Cul de las siguientes condiciones patolgicas previas es ms probable para el desarrollo del LNH (LDGCB)?

RESPUESTA
a.- Carcinoma metastasico.
b.- Carcinoma pulmonar.
c.- Antecedente de VIH.
d.- Lupus eritematoso sistmico discoide.

PREGUNTA
En pacientes con LNH, cual es la presentacin mas comn de compromiso cardaco?

RESPUESTA
a.-Dolor torcico
B.-Insuficiencia cardaca congestiva
C.-Derrame pericrdico
D.-Bloqueo atrioventricular

PREGUNTA
Cul es el manejo inicial de la taquicardia ventricular en este caso?

RESPUESTA
a.- Cardioversin elctrica
b.- Verapamilo
c.- Amiodarona
d.- Desfibrilacin

CASO CLINICO
Varn de 39 aos con antecedentes de amigdalectoma en infancia, sin hbitos txicos referidos y abuela materna con leucemia
linftica crnica. El paciente es visto en consulta por insuficiencia respiratoria nasal y ronquido asociado a fatiga diurna, ms acentuada
en el ltimo ao, sin otra clnica asociada. Es trabajador a turnos y conductor profesional por lo que se solicita interconsulta en la
Unidad de Trastornos Respiratorios del Sueo, con diagnstico de trastorno de sueo leve. En la exploracin se visualiza, por rinoscopia
anterior, una desviacin septal derecha con hipertrofia turbinal compensatoria. En nasofibroscopia, se identifica una masa en cavum,
compatible con hipertrofia adenoidea. La rinomanometra demuestra mayor resistencia area en fosa nasal derecha, sin gran mejora
tras vasoconstrictor, siendo el resto de la exploracin ORL normal. Dado el agravamiento de la clnica en el ltimo ao y el tamao de la
masa en cavum, se realiza biopsia de la misma, informada como hiperplasia folicular linfoide, y RM, objetivndose una masa de 2,3cm
de dimetro compatible con hipertrofia de tejido adenoideo. Teniendo en cuenta la clnica referida y la exploracin realizada, el
paciente es intervenido de septoplastia con turbinectoma y adenoidectoma. El informe AP es de linfoma de Hodgkin (LH) clsico rico
en linfocitos, sin detectar presencia de virus Epstein Barr (VEB).

PREGUNTA
Cul es la manifestacin clsica de Linfoma Hodgking?

RESPESTA
a.- Paciente joven, masculino, linfadenopatas perifricas.
b.- Adulto mayor, masculino, afeccin extraganglionar
MANUAL DE TRABAJO DEL CURSO ENARM CMN SIGLO XXI
CURSO ENARM CMN SIGLO XXI TEL: 36246001 Pharmed Solutions Institute PGINA 87

c.- Paciente joven, femenino, linfadenopatas perifricas
d.- Paciente joven, masculino, afeccin extraganglionar

PREGUNTA
Qu utilidad tiene hacer estudio inmunohistoquimico a este paciente?

RESPUESTA
a.- Decidir tratamiento
b.- Diferenciar LNH bajo grado de un LNH agresivo
c.- Establecer un diagnstico diferencial con el linfoma no hodgkiniano
d.- Saber si es un tumor en cabeza y cuello

PREGUNTA
El paciente descrito presenta un LH, sin sntomas B, localizado en adenoides, sin afectacin de otras regiones ni adenopatas asociadas.
En qu estadio de la clasificacin de Ann Arbor- Costwold se encuentra?

a.- Estadio IV
b.- Estadio III E-A
c.- Estadio IE-A
d.- Estadio IE-B

CASO CLINICO
Una paciente de 27 aos, del sexo femenino, se present en la Sala de Emergencias con palpitaciones de inicio sbito y disconfort
torcico. La presin arterial (PA) era de 68 / 40 mmHg y la frecuencia cardaca (FC) era de 184 lpm. Al examen fsico, la paciente
presentaba aumento de volumen de la mama izquierda y masa abdominal difusa. El ECG revel taquicardia de complejo ensanchado
con estndar de bloqueo de ramo izquierdo, fue sometida a cardioversin elctrica para el ritmo sinusal normal. El ECG fue repetido y
mostr ritmo sinusal normal, sin alteraciones del segmento ST. La paciente recibi amiodarona. Una ecocardiografa transtorcica
revel derrame pericrdico posterior difuso y engrosamiento del miocardio basal-septal. VHI positiva y SIDA con CD4 de 39
clulas/mm3. El diagnstico de linfoma difuso de grandes clulas B (LDGCB), un linfoma no Hodgkin fue el diagnostico definitivo.

PREGUNTA
Cul es el medio diagnostico mas adecuado.

RESPUESTA
a.- Biopsia.
b.- IRM-f
c.- Frotis perifrico.
d.- BH.

CASO CLINICO
Se trata de masculino de 21 aos de edad el cual acude a consulta externa por cuadro farngeo, se observa odinofagia, rinorrea hialina
con descarga retrofaringea, amgdalas hiperemicas e hipertrficas, adenopata cervical y axilar, al interrogatorio que presenta
frecuentemente cuadros infecciosos, pero en la actualidad refiere que desde hace 5 meses ha presentado fiebre intermitente, ataque al
estado generalizado, fatiga, adinamia, hiporexia, con prdida de peso del 10 % aproximadamente, adems de prurito generalizado.

PREGUNTA
Cul es la condicin ms probable en este caso.

RESPUESTA
a.- Considerando la edad es ms frecuente la mononucleosis infecciosa.
b.- Lo ms probable es Leucemia Linfoblastica Aguda.
c.- Lo ms frecuente es Leucemia Linfocitica Aguda.
d.- Considerando el cuadro Linfoma de Hodgkin.

CASO CLINICO
Varn de 69 aos de edad con antecedentes personales de LMA M1 (mieloblstica sin diferenciacin), en remisin completa (RC) desde
hace 11 meses tras tratamiento quimioterpico segn protocolo PETHEMA-LAM 99. Ingresa por cuadro respiratorio catarral, con
importante congestin nasal, sin respuesta a antibitico. Presenta, a su vez, ppulas eritematosas en espalda, adenopatas
supraclaviculares y submaxilares. Presenta ademas ojo rojo de dos das de evolucin, sin otra sintomatologa ocular. En la exploracin
presentaba una agudeza visual con correccin de 20/20 en OD y 18/20 en OI. Los reflejos, motilidad y tonometra oculares se
encontraban dentro de la normalidad. En la biomicroscopa de polo anterior se observ en OD una lesin carnosa sobreelevada de
color salmn en el tarso superior e inferior. En el OI se encontr una lesin similar en conjuntiva tarsal inferior. En el fondo de ojo no se
apreciaron alteraciones.

PREGUNTA
Cual es la conducta mas apropiada a seguir?
MANUAL DE TRABAJO DEL CURSO ENARM CMN SIGLO XXI
CURSO ENARM CMN SIGLO XXI TEL: 36246001 Pharmed Solutions Institute PGINA 88

RESPUESTA
a.- Biopsia de lesin.
b.- Busquedad de marcadores.
c.- Administracion de esteroides tpicos.
d.- Punsion medular.

PREGUNTA
Sobre la patologia anterior descrita, cual de las siguientes afirmaciones es menos frecuente?

RESPUESTA
a.- La afectacin ocular por la leucemia es bien conocida,
b.- Los ms frecuente es la hemorragia retiniana.
c.- Es debida a la leucocitosis.
d.- La retina es, a su vez, la estructura ocular ms frecuentemente afectada de forma directa por clulas leucmicas, hasta en un 30%.

CASO CLINICO
Paciente de sexo femenino de 32 aos de edad que consulta por presentar equimosis, hematomas generalizados, gingivorragia e
hipertrofia gingival, astenia, adinamia y sndrome febril. Tras un laboratorio, se asume a la paciente como Leucemia Aguda. Se realiza
puncin-aspiracin de mdula sea, cuyo anlisis citogentico es compatible con una Leucemia Mieloide Aguda de subtipo M3. Se inicia
el tratamiento quimioterpico con Mitoxantrona y Tretinoina, farmacoprofilaxis con Dexametasona y Omeprazol. A los 20 das de
iniciado el dicho tratamiento, la paciente desarrolla Sndrome de Diferenciacin.

PREGUNTA
Cual de los siguientes mecanismos fisiopatolgicos no intervienen en el sndrome de diferenciacin presente en el caso?

RESPUESTA
a.- Respuesta inflamatoria sistmica.
b.- Dao endotelial con sndrome de fuga capilar.
c.- Obstruccin de la microcirculacin que produce infiltracin tisular.
d.- Disminucin de la presin oncotica sobre la hidrosttica.

CASO CLINICO
Varn de 14 aos, acude por esplenomegalia. Historia pre-hospitalaria de palidez de piel y mucosas, de moderada intensidad, de un
mes de evolucin, fiebre de 38C sin escalofros ni sudoracin, dolor en el pecho y vmito de contenido alimentario con estras de
sangre, 24hs antes del ingreso. Al examen fsico presenta petequias dispersas en ambas piernas y ppulas en miembro inferior derecho.
El laboratorio informa Leucocitos: 25.200/uL con 68% de blastos, Hemoglobina: 6,1g/dL, Hematocrito: 17%, Plaquetas: 31.000/uL.

PREGUNTA
Cual es la conducta diagnostica mas adecuada?

RESPUESTA
a.- Aspirado de medula osea.
b.- Gamagrama oseo.
c.- IRM.
d.- Marcadore tumorales.

FIEBRE REUMATICA (FRA):
CIENCIAS BASICAS: Enfermedad inflamatoria ocurre como secuela alejada de una infeccin por estreptococo beta hemoltico grupo A,
habitualmente faringoamigdalitis o escarlatina. Su nombre proviene del dao articular, pero los daos ms importantes se producen en
corazn. SALUD PBLICA: La FRA aparece de 0.5-5% de pacientes que hacen faringoamigdalitis estreptoccica. Aparece especialmente
entre 5-15 aos. Mortalidad <1% por carditis grave. Aparicin favorecida por factores climticos (hmedo, frio) y socioeconmico
(hacinamiento, pobreza). PATOGENIA: Existe una relacin entre faringoamigdalitis y estreptococo grupo A y la fiebre reumtica que se
sustentan en: 1. Relacin epidemiolgica sustentada entre estas infecciones y la aparicin de casos de FRA. 2. Comprobacin
inmunolgica de infeccin estreptoccica (antiestreptolisinas en suero). 3. Episodios primarios y secundarios pueden prevenirse
mediante antibitico precoz para la infeccin. Debido a que las toxinas del estreptococo producen inmunidad cruzada.
DIAGNOSTICO: Clnico; Cuadro febril insidioso, con aparicin de artritis (70-80%), es una poliartritis migratoria de grandes
articulaciones. Carditis (40-60%), generalmente dentro de las primeras semanas del episodio, si no hay carditis (infiltrado inflamatorio
que afecta miocardio, endocardio y pericardio) en los primeros meses el pronstico es muy bueno; puede manifestarse por soplos de
insuficiencia mitral (75-80%) o aortica (30%), frote pericardico, galope, cardiomegalia, congestin pulmonar, a veces son poso
MANUAL DE TRABAJO DEL CURSO ENARM CMN SIGLO XXI
CURSO ENARM CMN SIGLO XXI TEL: 36246001 Pharmed Solutions Institute PGINA 89

sintomticas. Corea de Syndenham (10-20%) que son movimientos desordenados, descoordinados, inesperados e involuntarios.
Eritema marginado y ndulos subcutneos de Meynet (<2-3%). Puede haber compromiso del estado gnarl., artralgias. Laboratorio:
Antiestreptolisinas (ASO) elevadas (desde los primeros das y duran hasta 6-8 semanas). VSG Y Protena C reactiva elevadas. Se puede
hacer cultivo farngeo para estreptococo. Histolgico: Los cuerpos de Aschoff son una confluencia de monocitos y macrfagos rodeados
por fibrosis se consideran tpicos de FRA. En ECG puede haber bloque AV simple, fenmeno no diagnostico de carditis. La ecografa
cardiaca es til para el diagnostico y el tratamiento de la carditis (valoracin de dao de vlvulas, y de insuficiencia cardiaca). CRITERIOS
DIAGNSOTICOS de Jones: Criterios mayores; poliartritis, carditis, corea, eritema marginado, ndulos subcutneos. Criterios menores;
fiebre, artralgias, FRA previa, VSG y PCR elevadas, prolongacin de PR. El diagnostico se hace con 2 criterios mayores 1 mayor y 2
menores, junto con alguna evidencia de infeccin estreptoccica, tal como elevacin de las ASO o antecedente de escarlatina, cultivo
farngeo positivo o prueba rpida de antgeno estreptoccico positivo. TRATAMIENTO: Se realiza en 2 partes del episodio actual y
prevencin de la recurrencia. Reposo en caso, de artritis o carditis hasta recuperacin completa. El medicamento de primera eleccin
para la artritis es la aspirina, segunda eleccin naproxen. En la carditis se pueden utilizar corticoides. Prevencin primaria: Tratamiento
adecuado a las faringoamigdalitis estreptoccicas (su diagnostico se basa marcadamente en cuadros febriles y exudativos), con
penicilina benzatinica 1,200,000U dosis nica o eritromicina VO por 10dias (en caso de alergia a penicilinas). Prevencin secundaria:
Uso prolongado de penicilina benzatinica 1,200,000U cada 4 semanas o sulfadiazina 1gr/dia (en caso de alergia a penicilinas). La
duracin de profilaxis depende de la gravedad y edad del paciente. El periodo mnimo recomendado en paciente con antecedente de
FRA, sin carditis es al menos de 5 aos, pero directrices reciente refieren 10 aos o hasta 21 aos de edad. PRONOSTICO: Los episodios
de FRA tienden a remitir espontneamente en 75%, antes de 6 semanas y un 90% antes de 12 semanas. El pronstico a largo plazo
depender de la gravedad de la carditis, aparicin de nuevos brotes y magnitud de dao valvular.

CASO CLINICO
Femenino de 30 aos, antecedente IAM, estenosis mitral con remplazo biologico, tabaquismo, 4 gestas, con dificultad para hablar y
movimientos involuntarios en dedos, manos, boca y lengua haca 18 meses, disnea de moderado esfuerzo. Actualmente con fraccin de
eyeccin de 44% y prtesis biolgica mitral sin alteraciones estructurales con reflujo mnimo. Inici cuadro de cefalea acompaada de
parestesia en miembro superior izquierdo y movimientos involuntarios en manos, boca, lengua y dedos. Enseguida, present
convulsiones tnico-clnicas generalizadas y episodios de alucinacin visual, delirium e intensa labilidad emocional. Al examen fsico, se
encontraba en regular estado general, taquicrdica (frecuencia cardaca=130 latidos por minuto), presin arterial de 120x70 mmHg,
presencia de ruidos rtmicos con hiperfonesis de B1 y frote pericrdico audible en mesosstole y proto y teledistole y estertores finos
crepitantes en ambas bases pulmonares.

PREGUNTA
Que clase funcional de la NYHA se encuentra el paciente?

RESPUESTA
a.- Clase I
b.- Clase II
c.- Clase III
d.- Clase IV

PREGUNTA
Cual es la complicacin neurolgica que se presento en caso?

RESPUESTA
a.- Acidente cerebro vascular.
b.- Corea de huntintong.
c.- EVC y corea.
d.- Corea de Syndeham.

PREGUNTA
Cual es la causa de muerte aguda ms frecuente en estos casos?

RESPUESTA
a.- IAM.
b.- Carditis.
c.- Insuficiencia cardiaca.
d.- Edema agudo pulmonar.

PREGUNTA
Cual de las siguientes medidas farmacolgicas es la mas apropiada para el manejo de sntomas neurolgicos?

RESPUESTA
a.- Fenitoina.
b.- Haloperidol.
c.- Biperideno.
d.- Clonacepam.

MANUAL DE TRABAJO DEL CURSO ENARM CMN SIGLO XXI
CURSO ENARM CMN SIGLO XXI TEL: 36246001 Pharmed Solutions Institute PGINA 90

PREGUNTA
Cual es la causa de la carditis en la fiebre reumtica?

PREGUNTA
a.- Proteina M Streptococcica + N-Metil glucosamina con reaccin cruzada con la miosina.
b.- Proteina M Streptococcica + N-Acetil-D glucosamina con reaccin cruzada con la miosina.
c.- Proteina P Streptococcica + N-Acetil-D glucosamina con reaccin cruzada con la miosina.
d.- Proteina P Streptococcica + N-Metil glucosamina con reaccin cruzada con la miosina.

PREGUNTA
Considerando los Criterios de Jones cual es la respuesta correcta en este caso?

RESPUESTA
a.- 2 Criterios Mayores y 2 menores
b.- 1 Criterio Mayor y 2 menores.
c.- 2 Criterios Mayores y 1 menor.
d.- 3 Criterios Mayores y 2 menores.

CASO CLINICO
Femenino de 18 aos que acude a urgencias por poliartralgias en rodillas y codos de 72 h de evolucin, asociadas a fiebre. Antecedente
de escarlatina hace 3 semanas, tratada con penicilina V 7 das y 3 episodios anuales de faringoamigdalitis estreptoccica en los ltimos
2 aos. A la exploracin, presenta impotencia funcional de rodilla derecha, dificultad para la extensin, posicin en flexo, edema y signo
de la tecla positivo. Dolor a la palpacin en rodilla contralateral y ambos codos, con menor inflamacin e impotencia funcional. Soplo
sistlico II/VI vibratorio multifocal. Con leucocitos 19.910/l, neutrfilos 12.040/l; PCR 28,7mg/l y VSG 55mm.

PREGUNTA
Cul es la conducta a seguir en esta paciente.

RESPUESTA
a.- Succinato de eritromicina, va oral, por diez das.
b.- Amoxicilina, va oral, por diez das 1500 mg una vez al da por diez das.
c.- Cefalexina, 500 mg cada ocho horas, va oral durante diez das.
d.- Clindamicina va hrs VO 600 mg al da c/6 dosis durante 10 das.

CASO CLINICO
Mujer de 31 aos, portadora de cardiopata reumtica, hasta entonces asintomtica, refiri disnea de esfuerzo progresiva y
palpitaciones aun usando regularmente penicilina benzatina de tres en tres semanas. A los 13 aos present artritis migratoria de
grandes articulaciones tras estreptococcia de la orofaringe, y a los 19 aos, en el 14 da de puerperio, se quej de recurrencia de la
artritis, detectndose soplo cardiaco. El examen fsico revel taquipnea, con discreto edema de miembros inferiores. Se constataron los
siguientes datos: PA = 130/70 mmHg; frecuencia cardiaca de 120 rpm; ritmo cardaco regular a tres tiempos con intenso soplo
holosistlico en rea mitral (AM) y soplo sistlico rudo en el rea tricspide (AT); y hepatomegalia dolorosa discreta. Ecocardiograma
doppler fue compatible con insuficiencia mitral importante e insuficiencia tricspide moderada. Hemograma detect anemia
hipocrmica (hemoglobina = 11,1 g/dl), leucocitosis discreta (13.000/mm3) y hemosedimentacin acelerada (37 mm/1 hora). Los
niveles de antiestreptolisina O (AEO) fueron normales.

PREGUNTA
Cul es el tratamiento farmacologa para limitar el dao.

RESPUESTA
a.- Salicilatos dosis 30-60 mg/kg/da.
b.- Paracetamol 20 mg/Kg/dia.
c.- Naproxeno 750 mg/dia.
d.- Prednisona 2mg/kg/dia.













MANUAL DE TRABAJO DEL CURSO ENARM CMN SIGLO XXI
CURSO ENARM CMN SIGLO XXI TEL: 36246001 Pharmed Solutions Institute PGINA 91

ARTRITIS REUMATOIDE (AR):
CIENCIAS BASICAS: AR es la mas comn de las artritis inflamatorias. Es una enfermedad multisistmica crnica, de etiologa
desconocida, caracterizada por inflamacin persistente (sinovitis), y consecuentemente lesin y destruccin de las estructuras
periarticulares y articulares. Participan factores genticos y ambientales los cuales juegan un rol en el desarrollo de la enfermedad.
SALUD PUBLICA: Se presenta 0.5-1% de la poblacin, en relacin mujer:hombre 3:1. Asociada a HLA-DRA-4. Ms frecuente en cuarta y
quinta dcada de la vida. PATOGENIA: En la AR los T CD4 son los principales para desencadenar engrosamiento sinovial e invasin de
articulaciones y cartlagos. En AR el rgano blanco es la membrana
sinovial.
DIAGNOSTICO: Clnico; Poliartritis crnica simtrica, rigidez matutina
(>1h), inicia de forma insidiosa, afecta principalmente manos
(metacarpofalalgicas 90%; carpo 80%), rodilla (60%), pies
(metarasofalangicas 50%), articulacin atlo-axoidea (40%; se manifiesta
como cervicalga). Sntomas generales: astenia, adinamia, hiporexia,
afectacin musculo-esqueltica variable. Exploracin fsica: desviacin
cubital, ndulos de Bouchar (interfalangicas proximales), sinovitis,
dedos en cuello de cisne o botonero, hallux valgus (miembros
inferiores), hipotrofia en msculos interseos en manos (por vasculitis),
disminucin de fuerza prensil, derrame bilateral de rodillas de
predominio izq. (complicacin frecuente de AR, el quiste de Barker en
hueco poplteo asociado a ruptura e invasin muscular).
Manifestaciones extraarticulares: piel; ndulos reumatoides (25-50%).
Hematolgicos; anemia normocitica normocrmica (35-30%), trombocitosis (>50,000), trombocitopenia, linfadenopata. Heptico;
transaminemia. Pulmn (30%), dolor pleurtico (25%), ndulos pulmonares, arteritis pulmonar, enfermedad pulmonar intersticial. Ojos;
queratoconjuntivitis Sicca (35%), ojo seco, epiescleritis, uvetis, queratitis ulcerativa. Corazn; pericarditis, enf. Ateroesclertica
avanzada. Neurolgico; neuropata perifrica, sndrome del tnel del carpo. Renal; nefropata Laboratorio: Factor reumatoide
(FR=presente hasta 65%): Auto-anticuerpos dirigidos contra la porcin de Fc de la IgG tiene sensibilidad de 70%, especificidad 80%;
tambin presente en otros procesos infecciosos crnicos e inmunitarios (Tb, endocarditis, neoplasias, lupus esclerodermia). Nos indica
artritis seronegativa o seropositiva esta ltima se relaciona con forma ms agresiva de la enfermedad. Autoanticuerpos anti-pptido
cclico citrulinado (Anti-Pcc =presente hasta 95%), sensibilidad de 70% y especificidad 90-95%, al igual que FR se relaciona con formas
ms agresivas de la enfermedad. VSG PCR elevadas. Anti-DNA negativo (especifico para (LES). Radiolgico (PA de mano): Inflamacin de
articulaciones metacarpofalngicas, desviacin cubital, pulgar en Z, dedos en cuello de cisne, erosiones seas y osteopenia
yuxtaarticular, luxaciones metacarpofalngicas proximales. CRITERIOS DIAGNOSTICOS: 1. Rigidez matutina al menos de 1 hora de la
mejora mxima. 2. Artritis de 3 o ms articulaciones. 3. Artritis de las manos. 4. Artritis simtrica. 5. Ndulos reumatoideos (sobre
prominencias seas, superficies extensoras o regiones yuxtaarticulares). 6. Factor reumatoide positivo. 7. Cambios radiolgicos. El
diagnostico se hace con 4 de 7 criterios. TRATAMIENTO: AR activa leve: hidroxicloroquina (HCQ) o sulfazalazina (SSZ). AR moderada a
severa: Considerar dosis bajas de prednisona 5-10mgs e iniciar metotrexate (MTX) 10mg, se puede incrementar hasta 25mg/sem. Si hay
una respuesta inadecuada se puede agregar otro FARME (SSZ, HCQ, d-penicilamina)los cuales previenen el dao erosivo , terapia
bilgica (abatacep, etarnecep), anti TNF. Inmunosupresores (aziatropina, leflunomide, ciclosporina). Ciruga puede ser considerada en
una deformidad funcional severa. La prednisona al igual que los AINEs se puede dar como medidas de control, pero no modifican el
curso de la enfermedad. El metotrexate es la piedra angular del tratamiento de la AR (por ser un inductor de la remisin). SINDROMES
ESPECIFICOS EN PACIENTES CON AR: Sndrome de Sjogren secundario= AR + Sicca (xerostoma-xeroftalmia). Sndrome de Felty= AR +
Esplenomegalia + leucopenia (neutropenia). Sndrome de Caplan= AR + Ndulos reumatoideos pulmonares + Neumoconiosis.
Amiloidosis en AR= AR de larga evolucin y grave + proteinuria o insuficiencia renal.

CASO CLINICO
Se trata de un varn de 63 aos con antecedentes de AR de ms de 15 aos de evolucin bajo tratamiento. Entre otros antecedentes
padece hipertensin arterial, dislipidemia y cardiopata isqumica. Ingresa por sndrome nefrtico observndose a la exploracin fsica:
100,7 kg de peso, regular estado general, eupneico en reposo, saturacin del 96%; hbito cushingoide; tensin arterial de 130/70mmHg
y temperatura de 37C; auscultacin cardiopulmonar normal; abdomen globuloso, blando y depresible; edemas con fvea hasta la raz
de los muslos, abdomen y manos; artritis bilateral de las articulaciones metacarpofalngicas y limitacin a la flexo-extensin en ambos
carpos; deformidad de ambos tobillos y pies, ms el derecho. Entre las exploraciones complementarias destaca anemia normoctica
normocrmica; velocidad de sedimentacin globular (VSG) de 130 mm/1. hora; hiperfibrinogenemia reactiva; urea de 112 mg/dl;
creatinina de 3,8 mg/dl; aclaramiento de creatinina de 45,4 ml/min; cido rico de 8,2 mg/dl, protenas totales de 4,5 g/dl; protena C
reactiva de 10,9 mg/dl; proteinograma con aumento de fracciones alfa 1, alfa 2 y betaglobulinas y disminucin de las gammaglobulinas;
albmina de 1,5 g/dl y en el sistemtico de orina se detecta proteinuria, microhematuria y abundantes cilindros granulosos-hialinos y
ms de 11 g de protenas en orina de 24 h.

PREGUNTA
Considerando el manejo habitual del padecimiento de base AR, cual de las siguientes observaciones es mas probable que se encuentre
relacionado con el sndrome nefrotico?

RESPUESTA
a.- Uso crnico de AINES.
b.- Uso de inmunomoduladores.
c.- Presencia de depsitos inmunologicos.
d.- Presencia de amiloidosis secundaria a AR.
ANTIGENO
DESCONOCIDO +
PREDISPOSICION
GENETICA
Respuesta inmunitaria-reaccin
inflamatoria con activacin de
cel. Plasmticas y linfocitos T
Infiltracin sinovial por linfocitos
T CD4> T CD8 y monocitos
Formacin de tejido de
granulacin por activacin de
fibroblastos (pannus) con
hiperplasia de clulas mviles
Manifestaciones generales
por citocinas secretadas
por los macrfagos
Destruccin articular y
sea por citocinas
formadas en el pannus.
MANUAL DE TRABAJO DEL CURSO ENARM CMN SIGLO XXI
CURSO ENARM CMN SIGLO XXI TEL: 36246001 Pharmed Solutions Institute PGINA 92

PREGUNTA
Considerando la pregunta anterior, cual de los siguientes estudios es de mayor utilidad en este caso?

RESPUESTA
a.- Tomografia computada abdominal.
b.- Biopsia renal.
c.- Radiografia de torax.
d.- Ecocardiografia.

PREGUNTA
Cual de las siguientes observaciones referentes al diagnostico de AR es de mayor utilidad?

RESPUESTA
a.- Los anticuerpos anti-CCP tiene un cociente de probabilidad para el diagnostico de AR superior al de Factor Reumatoide y es
pronostico de la enfermedad.
b.- La evaluacin radiogrfica por mtodo de Sharp/van der Heijde permite evaluar dao estructural para (erosion y pinzamiento
articular).
c.- Inflamacin articular con anticuerpos anti-CCP y factor reumatoide elevadoes indicativo de enfermedad agresiva con mal pronostico.
d.- Se deber realizar IRM precoz para detectar erosiones, sinovitis o derrame articular.

CASO CLINICO
Mujer de 68 aos de edad con antecedentes de AR de 20 aos de evolucin, en tratamiento con AINE, glucocorticoides a dosis bajas (5
mg al da de prednisona) y metotrexate; prtesis bilateral de caderas y rodilla derecha; hipertensin arterial, sndrome mielodisplsico
en tratamiento con eritropoyetina e insuficiencia renal crnica leve. Ingresa por cuadro de diarreas de 8 semanas de evolucin, prdida
de peso de 20 kg y edema en los miembros inferiores. En la exploracin fsica se aprecia: 45 kg de peso, regular estado general, tensin
arterial de 110/60 mmHg y una temperatura de 36C; auscultacin cardiopulmonar normal; abdomen blando y depresible; edemas con
fvea hasta raz de muslos y en manos. Artritis de metacarpofalngicas bilaterales y limitacin a la flexo-extensin en ambos carpos. En
la analtica destaca anemia normoctica normocrmica, VSG de 110 mm/1. h, urea de 46 mg/dl, creatinina de 1,3 mg/dl, sistemtico de
orina normal, con protenas en orina de 24 h negativa, proteinograma con aumento de fracciones alfa 1, alfa 2, beta y gammaglobulinas
y albmina disminuida.

PREGUNTA
Considerando el cuadro clnico, cual de los siguientes estudios es el mas adecuado para una aproximacin dignostica?

RESPUESTA
a.- Tomografia abdominal.
b.- Endoscopia abdominal.
c.- Panendoscopia.
d.- Biopsia por colonoscopia.

PREGUNTA
Considerando el manejo habitual del padecimiento de base AR, cual de las siguientes observaciones es mas probable que se encuentre
relacionado con el sndrome diarreico?

RESPUESTA
a.- Uso crnico de AINES.
b.- Uso de inmunomoduladores.
c.- Presencia de depsitos inmunologicos.
d.- Amiloidosis secundaria a AR.

CASO CLINICO
Paciente varn de 60 aos con artritis reumatoide de 6 aos de evolucin en tratamiento con sulfasalazina 1.500mg/da y deflazacort
6mg/da, con buen control clnico y analtico de su enfermedad. Se trata de un paciente fumador activo de 20 cigarrillos/da y con
cardiopata isqumica crnica. En una revisin habitual refiere la aparicin de una tumoracin dolorosa en el hueco poplteo derecho,
cuyo dolor se incrementa con la bipedestacin prolongada. En la exploracin fsica se objetiv una tumoracin en el hueco poplteo.

PREGUNTA
Considerando las complicaciones mas frecuentes en la AR cual de las siguientes es la mas probable?

RESPUESTA
a.- Quiste de Baker.
b.- Aneurisma popitleo.
c.- Lipoma popitleo.
d.- Varice popitlea.


MANUAL DE TRABAJO DEL CURSO ENARM CMN SIGLO XXI
CURSO ENARM CMN SIGLO XXI TEL: 36246001 Pharmed Solutions Institute PGINA 93

PREGUNTA
Cual de las siguientes aseveraciones relacionadas al diagnostico de AR no es adecuado?

RESPUESTA
a.- BH, QS, Transaminasas, VSG, EGO, actividad, funcionalidad articular y dao estructural.
b.- Compromiso en 3 articulaciones metacarpofalangicas o metatarsofalangicas y rigidez matinal de ms de 30 minutos.
c.- La elevacin de la VSG y la PCR tiene estrecha relacin con la actividad inflamatoria de la AR.
d.- La presencia de Factor Reumatoide confiere el diagnostico de artritis reumatoide.

PREGUNTA
Relacionado a al tratamiento cual de las conducta siguientes es la mas apropiada en el presente caso?

RESPUESTA
a.- Los AINES no debe reemplazar a los FARME.
b.- Considerando el uso de corticoides debe usarse calcio y vitamida D.
c.- El MTX es el frmaco de eleccin para AR en riesgo de enfermedad persistente.
d.- En caso de falla con MTX se recomienda LEF SSZ anti-TNF.

OSTEOARTRITIS (OA):
CIENCIAS BASICAS: Enfermedad degenerativa crnica y forma ms comn de enfermedad de las articulaciones, caracterizada por falla
comn y progresiva de todas las estructuras de la articulacin, cartlago, revestimiento, ligamentos y hueso. Acompaadas por el
aumento de espesor y esclerosis de la placa sea subcondral. SALUD PUBLICA: Mas de 70% de personas >70 aos, tienen evidencia
radiogrfica de OA. Es considerada la causa de dolor e incapacidad ms frecuente en ancianos. La incidencia de osteoartrosis de rodilla
se ha calculado en 240 casos por 100, 000 habitantes y es una causa de morbilidad y alto costo para la comunidad.. Su prevalencia es
igual para ambos gneros entre los 45 y 55 aos, pero despus de los 55 aos es ms frecuente en mujeres. Enfermedad articular mas
frecuente en mujeres. PATOGENIA: Deterioro de la articulacin debido con frecuencia a tensin mecnica, alteraciones biomecnicas y
celulares que producen un desequilibrio entre la degradacin y la sntesis de los condrocitos, la matriz extracelular y del hueso
subcondral, lo que hace que el hueso que est bajo el cartlago falle, tiende afectar las articulaciones que se usan con frecuencia
(manos, columna, cadera, rodillas). Los 2 principales componentes del cartlago son: colgeno tipo 2 (provee fuerza tensil) y un
proteoglucano. El OA es caracterizado por deplecin gradual del proteoglucano, desplegando de la matriz el colgeno y perdiendo
colgeno tipo 2, lo que nos lleva a incremento de la vulnerabilidad.
Dentro de la fisiopatologa ocurren cambios morfolgicos:
fragmentacin del cartlago, formacin de fisuras y ulceracin de la
superficie articular, eburneacin, la posterior proliferacin celular en
acmulos. Cambios bioqumicos: mayor cantidad de agua, sintesisi de
fibras de colgeno pequeas y frgiles, menor concentracin de
proteoglucanos y sulfato de queratn con aumento de la
concentracin de sulfato de condritin que desencadena la produccin
de proteoglucanos de menor tamao y de menor capacidad para
agregarse al cido hialurnico, que predispone a fragilidad. Cambios
biomecnicos: perdida de la rigidez y elasticidad del cartlago, Factores
de riesgo: edad avanzada, tener familiares con OA, obesidad, lesiones
en articulacin o uso repetitivo, defectos congnitos, densidad sea,
ocupacin, enfermedades concomitantes.

DIAGNOSTICO: Clnico; Dolor (empeora con la actividad, mejora con el reposo), rigidez matutina breve (<30min), hinchazn
prominente, crujidos, rechinado de toda la articulacin, llevando a un menor funcionamiento de la misma, puede haber aumento de
temperatura local y de volumen articular, si hay derrame, deformidad (angulacion en varo o valgo), subluxacin en casos avanzados,
ndulos de Heberden (interfalangicas distales), Bouchar (interfalangicas proximales). Laboratorio: Generalmente todo normal. VSG y
PCR normales. Si se hace estudio de liquido sinovial, ligeramente turbio, no contienen cristales y con leucocitos <2,000/microlitro y
<25% de neutrofilos. Radiologa: Disminucin de espacio articular, esclerosis subcondral, osteofitos marginales y quistes subcondrales.
COMPLICACIONES: Derrame articular, herniacin capsular, bloqueo articular, tendinitis, estenosis espinal espondilolistesis.
TRATAMIENTO: Ningn tratamiento especifico. Objetivo reducir dolor y mejorar funcionamiento: 1. Medidas fsicas: prdida de peso y
ejercicio con lo que se disminuye dolor, mejora fuerza muscular y disminuye rigidez de la articulacin, tiles usar ortesis o bastones.
Evitar sobrecarga y trauma de articulaciones. Calor o frio local. Reposo articular 2. Medico: Tpico; capsaicina, lidocana, diclofenaco.
Va oral AINES, paracetamol (primera eleccin), se puede considerar el tramadol (200-300mgs dividido en 3 a 4 dosis) si el AINE no es
efectivo. Los corticoides orales no estn indicados en OA. En 2010 se aprob duloxetina para dolor musculo-esqueltico crnico (largo
plazo). La diacerina es una quinona, que posee propiedades antioxidantes, inhibiendo a IL-1 y metaloproteinasas, pero aun no es
contundente su eficacia y seguridad. El Ac. Hialurnico intraarticular; mejora parcial y se usa para retraso de reemplazo de rodilla. Los
glucocorticoides intraarticulares son recomendados en los brotes agudos inflamatorios en las distintas guas de OA, de rodilla, no debe
administrase una inyeccin cada 3 o 4 meses. Ciruga: Artroscopa, artroplastia u osteotoma por angulacin (disminuye el dolor, en
pacientes no candidatos a artroplastia), en casos muy severos. Glucosamina/condroitina, aunque ampliamente vendidos, no estn
aprobados por FDA, su eficacia no esta establecida.



CLASIFICACION DE LA OSTEOARTRITIS
Primaria: localizada; Cadera
Rodilla
Apfisis espinosas y cuerpos vertebrales
(cervical,lumbar)
Mano (interfalangicas distales)
Pie (primer dedo)
Generalizada: Manos
Manos y rodillas
Secundaria: Displasica: condrodisplasias, displasias epifisiarias,
Desplazamiento congnito articular
Postraumtica: aguda, repetitiva, posoperatoria
Insuficiencia estructural: osteonecrosis, osteocondritis
Posinflamatoria: artrits reumatoide y sptica
Endocrina y metabolica: acromegalia, hemocromatosis
Tejido conectivo: Hipermovilidad (Ehlers- Danlos)
Formas generalizadas: Hiperparatiroidismo, acromegalia
MANUAL DE TRABAJO DEL CURSO ENARM CMN SIGLO XXI
CURSO ENARM CMN SIGLO XXI TEL: 36246001 Pharmed Solutions Institute PGINA 94

CASO CLINICO
Varn de 66 aos con antecedentes personales de EPOC severo con hiperreactividad bronquial y bullas enfisematosas en ambos
vrtices pulmonares, en tratamiento con oxigeno domiciliario (15h al da) junto a broncodilatadores, diurticos y corticoides
(prednisona 10mg) a das alternos. El paciente haba tenido varios ingresos hospitalarios por reagudizaciones de EPOC. Refirie dolor en
la regin lumbar central y en ambas caderas con la deambulacin. Se realizaron exploraciones complementarias que incluyeron una
radiografa de pelvis y un TAC lumbosacro que mostraron espndilolisis con espondilolistesis de L5 y artrosis de cadera
respectivamente, procesos a los que se atribuyeron los sntomas. En la radiografa de pelvis inicial se observa estrechamiento del
espacio articular y esclerosis subcondral. Se encuentra en silla de ruedas por la incapacidad para deambular. Leucocitos 9,5103/mm3
(neutrfilos 48.4%), Hb 12mg/dl, Hto 36, plaquetas 355103/mm3, VSG 32mm/h, glucosa 86mg/dl, urea 44mg/dl, creatinina 0,88mg/dl,
colesterol 190mg/dl, triglicridos 116mg/dl, fosforo 4,2mg/dl, calcio total 9,5mg/dl, actividad de protrombina 106%, INR 0,97, ratio
tromboplastina 1,10, protena C 1,22, protena S 0,73, antitrombina III 0,99U/l. Los anticuerpos antinucleares, anticuerpos
antifosfolpido y el factor reumatoide fueron negativos, as como la serologa para virus de hepatitis B, C y VIH.

PREGUNTA
Considerando la entidad nosolgica descrita, asi como los reportes de laboratorio, cual de los siguientes diagnosticos diferenciales es el
mas frecuente?

RESPUESTA
a.- Infeccioso.
b.- Metablico.
c.- Inflamatorio crnico.
d.- Neuropatas.

CASO CLINICO
Mujer de 74 aos de edad que acude por aumento de dolor en la rodilla izquierda. Le administraron paracetamol ms ibuprofeno,
refiere incapaz para limpiar la casa debido a que el dolor empeore. Ella usa un masaje tpico al acostarse y ha intentado usar unas
compresas fras, con beneficios limitados. Es hipertensa bajo tratamiento, En el examen, su presin arterial es de 140/ 92 mm Hg, pulso
es de 80 latidos por minuto. Ella es de 162 cm de alto y pesa 80 kg (ndice de masa corporal de 30,5 kg/m2). Ella califica su dolor como 8
de 10 cuando est activo, y 5 de 10 en reposo. Camina con un bastn y cojea. Sus resultados del examen rodilla izquierda no se han
modificado. Previo anlisis de rayos- x mostraron cambios degenerativos severos. Resultados de creatinina de 90 mmol / L
Aclaramiento de creatinina estimado ( CrCl ) es entre 40 y 60 ml / min.

PREGUNTA
Cual de los siguientes criterior del presente caso, lo hace candidato a artroplastia?

RESPUESTA
a.- Artrosis bicorpartamental.
b.- Artrosis patelofemora aislada.
c.- Artrosis media o lateral.
d.- Necrosis avascular.

PREGUNTA
Considerando el estudio de liquido sinovial para el caso, que resultados es menos probable observar?

RESPUESTA
a.- Celular completa.
b.- Tincion Gram.
c.- Cultivo de liquido.
d.- Cristales de oxalato.

OSTEOCONDRITIS:
CIENCIAS BASICAS: Definicin clnica: Conjunto de trastornos seos de origen idioptico, caracterizados por trastornos en la osificacin
endocondrial que afecta tanto la condrogenesis como la ostogenesis (se presenta en centros seo en pleno crecimiento). Definicin
radiolgica: Trastornos seos idiopticos en relacin a su etiologa que comparten las siguientes caractersticas; 1. Predileccin por el
esqueleto inmaduro. 2. Afeccin de epfisis, apfisis o huesos epifisoides. 3. Cuadro radiolgico caracterizado por fragmentacin,
colapso, esclerosis y posteriormente reosificacin y restitucin de contorno seo. Se da en nios clnicamente sanos y en periodo de
crecimiento rpido (3-15aos) y expuestos a traumatismos; sus localizaciones ms frecuentes son las articulaciones que soportan carga.
PATOGENIA: Idioptica. Son procesos cclicos que pasan por diversas etapas en su evolucin y que pueden dejar secuelas por
deformaciones producidas en el estadio de fragilidad de la osteonecrosis. Se describen 4 etapas de la necrosis: 1. La zona vecina a la
necrosis sufre un proceso de descalcificacin por hiperemia reactiva. La zona necrtica, en cambio no se descalcifica por falta de vasos
que drenen sus componentes minerales. Resulta as una mayor densidad del fragmento necrtico respecto al sano. 2. Ms tarde, la
zona necrtica es invadida gradualmente por tejido conjuntivo y vasos sanguneos, que van a retirar el tejido necrtico por reabsorcin.
3. Esas zonas aparecen rarefactas en medio de la necrosis, fenmenos de sustitucin progresiva. 4. Cuando el proceso de reabsorcin y
reconstitucin est avanzado, todo el fragmento esta descalcificado para ir luego a la recalcificacin y restauracin sea.
DIAGNOSTICO: Clnico; dolor al apoyo, hipersensibilidad a la presin a nivel del hueso afectado. El inicio de la sintomatologa puede ser
agudo o gradual y puede o no existir anamnesis de traumatismo. Radiolgico: No siempre guarda relacin con la intensidad del cuadro
MANUAL DE TRABAJO DEL CURSO ENARM CMN SIGLO XXI
CURSO ENARM CMN SIGLO XXI TEL: 36246001 Pharmed Solutions Institute PGINA 95

clnico. Hay descalcificacin precoz con necrosis subsiguiente y finalmente los signos de reparacin. En las fases precoces se observan
zonas de densidad disminuida, que aumentan en intensidad y extensin; las epfisis se fisuran, se ensanchan presentando un contorno
borroso con aspecto deshilachado. Pueden observarse zonas de hueso necrtico denso. LEGG CALVE PERTHES: Necrosis avascular
idioptica de la cabeza femoral en nios. La teora ms aceptada de su origen es por insuficiente aporte sanguneo al ncleo epifisiario
de la cabeza femoral, debido a oclusin vascular sea de una rama importante de la circunfleja o de una rama terminal. Se presenta
entre 3-10 aos de edad, mas en sexo masculino, con mayor frecuencia unilateral. Tiene una evolucin radiolgica caracterstica: A)
Estadio incipiente o de sinovitis: que dura de 1-3 semanas. B) Estadio necrtico, asptico o avascular: dura varios meses a 1 ao. C)
Estadio regenerativo o de fragmentacin: dura de 1-3 aos. D) Estadio residual, que corresponde a la etapa de crecimiento y de
reparacin definitiva. El objetivo del tratamiento es la contencin de la cabeza femoral dentro del acetbulo, para que ella se rehaga en
forma concntrica. OSGOOD-SCHLATTER: Osteocondritis de la tuberosidad de la tibia, apfisis tibial anterior. Tiene como factor
desencadenante un traumatismo. Ocurre entre los 17 y 20 aos, ms frecuente en masculinos y en deportistas sometidos a
microtraumatismos repetidos. Hay dolor a la presin, hinchazn en tuberosidad anterior de anterior de la tibia en extensin. KOHLER:
Osteocondrosis del escafoides tarsiano: Ms frecuente en hombres (5:1), incidencia entre 5 y 10 aos. Dolor dorsal en mesopie de
meses a aos. RX: aumento perchado de densidad con fragmentacin, en estadios avanzados; aplanamiento, fragmentacin, colapso y
esclerosis. THIEMANN: Necrosis asptica de las epfisis basales de las falanges. Igual en ambos sexos. Caracterizada por hinchazn
progresiva y no dolorosa a nivel de la articulacin interfalangica (proximal de manos y pies). Rx: irregularidad de epfisis de las falanges
que aparecen esclerticas y fragmentadas, ocasionalmente el espacio articular se ve disminuido, base de la falange ensanchada.
PANNER: Osteocondrosis del epicondilo lateral de hmero (capitelum). Rx: fisuracin, aumento de densidad, disminucin del tamao
del capitelum, fragmentacin, resorcin. KIEMBOCK: Osteocondritis del semilunar. Es rara por debajo de los 15 aos, hay antecedente
de un traumatismo, se da mas del lado derecho y en los trabajadores manuales, las personas jvenes acusan dolor progresivo en
mueca con rigidez de la misma e hipersensibilidad local a nivel de semilunar. FREIBERG: Osteocondrosis de la cabeza de los
metatarsianos. Ms frecuente en mujeres (5:1), tpico en segunda dcada de la vida, 10% bilateral. Es rara la afectacin de ms de un
dedo por pie. Presencia en orden decreciente dedo 2>3>4, muy raro en 1 y 5. SCHEUERMANN: Cifosis torcica rgida por acuamiento
de los cuerpos vertebrales que ocurre en la infancia tarda. Rx: Cifosis torcica >45, irregularidad de platillos vertebrales,
estrechamiento del espacio intervertebral.

CASO CLINICO
Femenino de 69 aos de edad con cuadro clnico de 10 meses de evolucin de inicio sbito; caracterizado por dolor tipo punzada en
rodilla izquierda que aparece luego de mantenerla en reposo prolongado, sin predominio de horario, asociado a crepitacin, rigidez e
inflamacin de la rodilla; intensidad 5/10 en la escala visual anloga, la cual cede a medicacin con acetaminofn. Fue valorada por
medicina general quien indico 4 ampollas de betametasona; sulfato de glucosamina, diclofenaco, colestiramina; con mejora de
sntomas. La radiografa ntero-posterior de rodilla izquierda muestra una lesin radio lcida subcondral, rodeada por un halo de
esclerosis. Al examen fsico, presenta crepitacin bilateral de rodilla de predominio izquierdo con inflamacin moderada de rodilla
izquierda. Sinovitis moderada en rodilla izquierda.

PREGUNTA
Considerando el cuadro clinico y las imgenes cual es su impresin dianostica?

RESPUESTA
a.- Osteocondritis Aseptica.
b.- Osteocondritis Disecante de Rodilla.
c.- Osteonecrosis.
d.- Osteocondritis osificante.

PREGUNTA
Considerando la respuesta previa, cual la localizacion mas frecuente?

RESPUESTA
a.- Lateral del cndilo femoral medial.
b.- Rotula.
c.- Cndilo femoral externo.
d.- Meniscos discoideos.

MANUAL DE TRABAJO DEL CURSO ENARM CMN SIGLO XXI
CURSO ENARM CMN SIGLO XXI TEL: 36246001 Pharmed Solutions Institute PGINA 96


GOTA:
CIENCIAS BASICAS: La gota se caracteriza por hiperuricemia y la presencia de episodios inflamatorios habitualmente monoarticulares y
recurrentes con lapsos asintomticos, que son intensos y autolimitados. La inflamacin est directamente relacionada con la presencia
de cristales de urato monosdico (UMS) intraarticular de forma constante. Hiperuricemia: Nivel plasmtico de acido rico >7.0mg/dl en
hombres o ms de 6.0mg/dl en mujeres, surge de causas en la disminucin de su eliminacin, de aumento de la produccin o de
ambos. El acido rico es el producto final de la degradacin de los nucletidos de purina, su produccin est relacionada con la
concentracin intracelular de 5-fosforibosil-1-pirofosfato (PRPP), es el mayor determinante de la velocidad de biosntesis del acido
rico. El acido rico es excretado principalmente por el rin a travs de mecanismos de filtracin glomerular, secrecin tubular y
reabsorcin.
SALUD PBLICA: Gota causa comn de artritis en hombres mayores de 40 aos y mujeres posmenopusicas. La prevalencia de
hiperuricemia en poblacin adulta es de alrededor de 15%.
PATOGENIA: La enfermedad comienza con la formacin y la
subsecuente acumulacin de acido rico secundario a un
aumento en su produccin y/o disminucin en su eliminacin
(fase asintomtica). Posteriormente factores
desencadenantes como alcohol, diurticos, obesidad,
trauma, pH acido, ambientales y otros ocasionan
acumulacin de cristales de urato monosdico en las
articulaciones (artritis), riones (nefropata) y vas urinarias.
El depsito de cristales produce liberacin de citocinas y
activacin, as como reclutamiento de clulas inflamatorias
(PMN) y no inflamatorias (endotelio y sinoviocitos) y
quimiotaxis. No se conoce a detalle el proceso completo de
la formacin de cristales pero algunos de los factores son: el
acido rico tiene menor solubilidad a menor temperatura, por lo cual pueden formarse cristales con mayor facilidad en lugares distales
como dedos de los pies y de las orejas. La solubilidad de los UMS es menor cuando el pH es cido, microtraumas repetidos, alteracin
de la matriz extracelular. DIAGNSOTICO: Clnico; Artritis aguda, la ms frecuente y temprana manifestacin de gota, inicialmente afecta
una articulacin (suele localizarse en la primera metatarsofalangica = podagra), pero puede ser poliarticular en episodios tardos. La
gota aguda inicia por la noche con dolor severo, hinchazn, calor. El ataque generalmente desaparece espontneamente en 3-10 das.
Aunque algunos pacientes suelen tener solo un ataque, otros episodios recurrentes, con intervalos de variacin sin sntomas. La gota
aguda puede ser precipitada por exceso de purinas en la dieta, trauma ciruga, ingestin excesiva de alcohol, terapia hipouricemica y
enfermedades serias como infarto al miocardio. Artritis crnica; una proporcin de pacientes con gota pueden tener una sinovitis
unilateral crnica, esta suele ser rara. Puede tambin presentarse con tofos (agregados de UMS, rodeados por clulas de reaccin
inflamatoria, ndulos, subcutneos, de aspecto amarillento y duros) periarticulares. Tofos extraarticulares: Algunos estn en la cpsula
del olecranon, hlix y antihlix del pabelln auricular, superficie cubital o antebrazo, tendn de Aquiles, generalmente se desarrollan
entre 5-10 aos despus del inicio de ataques de gota. Tenosinovitis. Nefropata por uratos: por depsito de UMS en el intersticio y
pirmides, puede causar IRC. Nefropata aguda por acido rico: causa reversible de falla renal aguda debida a precipitacin de los
uratos en los tbulos, los pacientes que reciben tratamiento citotxico para neoplasia estn en riesgo. Nefrolitiasis por acido rico:
responsable de 10% de las litiasis renales. CRITERIOS PARA LA CLASIFICACION de artritis gotosa aguda: A) La presencia de cristales de
UMS caractersticos en el liquido sinovial (gold estndar para dx) o bien B) Un tofo cuyo contenido de cristales de UMS este
comprobado por medios qumicos o microscopia de luz polarizada o bien C) La presencia de 6 de los siguientes 12 fenmenos: 1. Mas
de un ataque de artritis aguda. 2. Desarrollo mximo de brote inflamatorio en el curso del da. 3. Ataque monoarticular. 4. Franco
enrojecimiento del area articular. 5. Localizacin primera articulacin metatarsofalngica. 6. Ataque unilateral que afecte la primera
articulacin metatarsofalngica. 7. Ataque unilateral en una articulacin del tarso. 8. Tofo: sospecha o confirmacin. 9. Hiperuricemia.
10. Infamacin asimtrica en estudio radiogrfico. 11. Quistes subcutneos sin erosiones seas. 12. Cultivo negativo del liquido articular
obtenido durante un ataque agudo. Los ataques de gota agudos se relacionan con cambios en el nivel plasmtico de AU (tanto aumento
como disminucin). La identificacin de UMS en liquido sinovial es patognomnico, se ven al microscopio como agujas y
birrefringentes. Radiolgico: En ataque agudos pueden mostrar tumefaccin de tejidos blandos. Los tofos son radiotransparentes,
aunque en ocasiones pueden tener calcificaciones en su interior, los cambios caractersticos son edema difuso, prominencias nodulares,
erosiones intraarticulares y extraarticulares con imagen de techo colgante y calcificaciones. TRATAMIENTO: Dentro de la medidas
generales dieta baja en alimentos ricos en purinas como: carnes, vsceras, piel, alimentos ahumados, rostizados, los dos factores ms
importantes que favorecen la hiperuricemia son: ingesta de alcohol y ayuno prolongado. Artritis gotosa aguda: Analgesia, AINES
(indometacina, diclofenaco, naproxen) primera eleccin. Colchicina efectiva dentro de las primeras 24hrs del ataque, contraindicada
cuando hay insuficiencia renal o citopenias. Dosis de 1-6 mg/da. Glucocorticoides intraarticulares; se debe descartar primero artritis
sptica. Glucocorticoides sistmicos; uso breve, considerado en pacientes con ataque de gota poliarticular. Agentes hipouricemicos:
Indicados para iniciar a bajar el acido rico, incluyendo recurrencias de artritis gotosa aguda, artritis poliarticular, gota tofosa, litiasis
renal. No deben iniciarse en el ataque agudo, pueden precipitar una llamarada y se debe considerar su uso concomitante con
colchicina. Alopurinol: disminuye la sntesis de acido rico por inhibicin de la xantino-oxidasa, reducir dosis en insuficiencia renal.
Dosis de 100-300mgs, pueden darse hasta 800mgs. Uricosricos (probenecid, sulfinpirazona): Incrementan la excrecin de acido rico,
por inhibicin de la reabsorcin tubular (inefectivo en insuficiencia renal), no debera usarse en >60 aos de edad. Iniciarse una vez que
haya pasado el ataque agudo.

CASO CLINICO
Varn de 71 aos con antecedentes personales de hipertensin arterial sistmica en tratamiento con enalapril, 20 mg diarios, e
hiperuricemia, con episodios de monoartritis en el pie izquierdo, Ingresa por dolor y tumefaccin en el segundo dedo de la mano
CAUSAS DE HIPERURICEMIA
Por disminucin en la
eliminacin
Hiperuricemia primaria (idioptica)
Hiperuricemia secundaria:
Funcin renal disminuida
Inhibicin de la secrecin de uratos (cetoacidosis, acidosis
lctica).
Por aumento en su
produccin
Hiperuricemia primaria:
Hiperactividad de la fosforibosil fosfato sintetasa
Deficiencia de hipoxantina-guanina fosforibosiltransferasa
Hiperuricemia secundaria:
Ingesta excesiva de purinas
Recambio de nucletidos elevado: psoriasis
Incremento de la degradacin de ATP (ejercicio)
Mecanismo
combinado
Deficiencia de glucosa-6-fosfatasa
Hipoperfusin tisular
Alcoholismo

MANUAL DE TRABAJO DEL CURSO ENARM CMN SIGLO XXI
CURSO ENARM CMN SIGLO XXI TEL: 36246001 Pharmed Solutions Institute PGINA 97

derecha y en la cuarta articulacin interfalngica proximal de la mano izquierda, sin fiebre ni otros datos clnicos. La exploracin fsica
mostr ndulos de Heberden en ambas manos, con signos de artritis en la segunda interfalngica distal de la mano derecha, con gran
tumefaccin distal y aumento de partes blandas, con un tofo acompaante, as como en la cuarta interfalngica proximal de la mano
izquierda. En las pruebas de laboratorio destac una uricemia de 7,20 mg/dl, con hemograma, perfil bioqumico y reactantes de fase
aguda dentro de la normalidad (VSG: 14 mm/h, PCR: 0,49 mg/dl). En las radiografas de las manos se apreciaron cambios degenerativos
en las articulaciones metacarpofalngicas, interfalngicas y trapeciometacarpianas. La interfalngica distal del segundo dedo present
aumento de partes blandas, con disminucin del espacio articular y erosiones en sacabocados. La ecocardiografa y los hemocultivos
seriados fueron negativos.

PREGUNTA
Cual de los siguientes resultados del aspirado de liquido sinovial en este caso es mas frecuente encontrar?

RESPUESTA
a.- Cristales de urato monosdico.
b.- Cristales de pirofosfato clcico.
c.- Ambos tipos de cristales.
d.- Cristales de pirofosfato.

LUPUS ERITEMATOSO SISTEMICO (LES)
CIENCIAS BASICAS: Prototipo de enf. autoinmune, multiorganica, multisistmica, se asocia con la produccin de autoanticuerpos contra
componentes del ncleo, mediada por inmunocomplejos (hipersensibilidad III). SALUD PUBLICA: Ms en mujeres de 20-40aos.
Relacin H:M es de 9:1. Lupus eritematoso sistmico 70%, lupus cutneo l5%, Sx de sobreexposicion/Enf. mixta de tejido conectivo
10%, lupus eritematoso inducido por drogas 5%. PATOGENIA: Enfermedad de etiologa desconocida. Participan para el desarrollo
factores genticos como: HLA-DR2, DR3, DR4 (75%), deficiencia de complemento C1, C2, C4 (a mayor deficiencia mayor riesgo para
nefritis lupica), mayor incidencia en familiares de 1er grado. Factores
ambientales: Rayos UVB (apoptosis de queratinocitos), medicamentos
(hidralacina, procainamida, fenitoina, isoniazida), infecciones (VEB),
mimetismo molecular. Factores hormonales: estrgenos son permisivos
de autoinmunidad, puerperio y anticonceptivos exacerban l LES. Factores
inmunolgicos: falla en el control del sist. Inmunolgico, produccin
excesiva de autoanticuerpos. Se producen autoanticuerpos, se forman
complejos inmunes, disminuye la apoptosis de cel. T autoreactivas, hay
actividad aumentada de linfocitos T cooperadores (CD4) y disminucin de
linfocitos T supresores (CD8); todo esto junto con los factores nos lleva a LES. DIAGNSTICO: Cuadro clnico: Hay perodos de remisin
y exacerbacin, sntomas inespecficos en 95%; fiebre, astenia, adinamia. Manifestaciones musculo-esquelticas son las ms frecuentes
(90-95%); artralgias (70-100%), artritis no erosiva (60%), pulgar en z, mialgias, hiperlaxitud, osteonecrosis avascular (dolor en cadera y/o
hombro). Mucocutaneas frecuentes (80-90%); lo mas comn presencia de lupus discoide (15-30%: cara, hombro, cuello, retroauricular),
rash en alas de mariposa (50%), eritema malar tiene la caracterstica de inestabilizarse e incrementarse con la exposicin al sol,
fotosensibilidad (70%), ulceras orales (40%), vasculitis cutnea y livedo reticularis. Hematolgicas frecuentes (80-90%); anemia (70%),
leuco, linfo y trombocitopenia, linfadenopata y esplenomegalia. Neurolgicas (50-60%); neuralgia, meningitis asptica, disfuncin
cognitiva (50%), sx. neural orgnico (35%), convulsiones y psicosis (20%), neuropata perifrica, EVC, cefalea. Cardiopulmonares;
pleuritis (50%), derrame pleural (30%), neumonitis lupica ( en 10%; de las complicaciones ms graves, mortalidad 60-70%), pericarditis
(30%), endocarditis de Libman-Sack (insf. artica y mitral: es asptica). Renales frecuentes (50%); principal factor que condiciona la
sobrevida del paciente (Cr >1.4, hipotensin y sx. nefrtico), glomerulonefritis membranosa y mesangial, nefritis lupica factor de mal
pronstico y es la principal causa de muerte en lupus hasta 50%. Laboratorio: Perfil metablico (tienen aterogenesis acelerada y lpidos
alterados), EGO (proteinuria), reactantes de fase aguda (PCR y VSG), anticuerpos antinucleares (ANA; considerados gold standar para
LEG, especificidad media), C3 y C4, anti-DNA ds ms especfico para lupus ( especificidad excelente), anti- histona ( relacionado con
lupus inducido por drogas), anti sm ( especificidad excelente), anti-Ro (SSA, se relaciona con lupus neonatal, pedir en embarazadas),
anti-La (SSB, si sale positivo protege contra dao renal). Anti-Ro y anti-La; son especficos pero son ms sensibles para Sx. de Sjogren.

CRITERIOS DIAGNOSTICOS:
Cutneos
1. Rash malar: eritema malar fijo, plano o palpable
2. Rash discoide: parches eritematosos palpables, con ampliacin queratosica y taponamiento folicular, pueden existir cicatrices atroficas.
3. Fotosensibilidad: reaccin inusual a la luz solar, observado por medico
4. Ulceras orales: o nasofaringeas, usualmente indoloras, observadas por medico
Sistmicos
1. Artritis: no erosiva que involucre 2 o ms articulaciones perifricas, caracterizadas por hipersensibilidad, inflamacin o derrame.
2. Serositis: pleuritis o pericarditis
3. Trastorno renal: proteinuria persistente (>0.5 g/d o ms de +++), o cilindros celulares de cualquier tipo.
4. Trastorno neurolgico: convulsiones o psicosis en ausencia de otras causas.
Laboratorio:
1. Trastorno hematolgico: Anemia hemoltica o leucopenia (>4000 en 2 ocasiones), linfopenia (>1500 en 2 ocasiones) o trombocitopenia (>100,000 en
ausencia de drogas causantes)
2. Trastorno inmunolgico: Anti-DNA o anti- sm, o anticuerpos antifosfolipido
3. Anticuerpos antinucleares: en ausencia de drogas conocidas, que puedan ser asociadas a lupus inducido por drogas.
SE DICE QUE UNA PERSONA TIENE LEG, SI PRESENTA 4 DE LOS 11 CRITERIOS DE FORMA SIMULTNEA O SERIADA EN EL CURSO DE LA ENFERMEDAD,
DURANTE O EN CUALQUIER PERIODO DE LA MISMA.
Clasificacin de nefritis lupica segn la OMS
TIPO I Nefropata lupica mesangial mnima
TIPO II Nefropata lupica mesangial
TIPO III Nefropata lupica focal
TIPO IV Nefropata lupica difusa
TIPO V Nefropata lupica membranosa
TIPO VI Nefropata lupica esclerosante (terminal)

MANUAL DE TRABAJO DEL CURSO ENARM CMN SIGLO XXI
CURSO ENARM CMN SIGLO XXI TEL: 36246001 Pharmed Solutions Institute PGINA 98


FARMACOS QUE INDUCEN LUPUS: Constituye el 10%; Hidralacina, procainamida, isoniacida, clorpromacina, d-penicilamina, quinidina,
interferon alfa, fenitoina, anti-TNF,predominan manifestaciones constitucionales y pleuropericarditis, no afeccin renal, ni neurolgica,
90% remite a los 3 meses de suspender el frmaco. Asociado a HLA-DR4, anti- DNA negativos, ANA positivos, anti-histona positivo e
hipocomplementemia. LUPUS NEONATAL: Se produce en menos de 5% de los RN de madres portadoras de autoanticuerpos anti-Ro y
anti-La, hasta en 25% de madre sanas con anti- Ro circulante, lesiones cutneas en ateas fotoexpuestas a partir de los 2 meses de vida,
bloqueo A-V completo (complicacin ms grave del lupus neonatal) y alteraciones hematolgicas. TRATAMIENTO: Basado en el tipo y
la severidad. AINES (Ibuprofeno 400-800mg c/6-8hrs)Corticoesteroides (prednisona 1mg/Kg, en caso de dao orgnico agudo;
metilprednisolona 1g/d x3dias ), hidroxicloroquina (antimalaricos) y aspirina (estos autorizados por la FDA). No exponerse al sol, dietas
sin alfalfa. Manifestaciones leves: AINES, esteroides a dosis bajas (>20mg/kg/da), hidroxicloroquina (400mg/da. efecto secundario
toxicidad retiniana). Lesiones cutneas, fotoproteccin (FPS>100), corticoides tpicos. Manifestaciones graves (neurolupus,
glomerulonefritis, neumonitis, trombocitopenia, anemia hemoltica) pulsos de metilprednisolona, prednisona, inmunosupresores como
aziatropina, metotrexate (son ahorradores de corticoides). 1ra eleccin en nefritis lupica ciclofosfamida (10_15mg/kg/da). Aziatropina;
nefritis lupica y dao hematolgico. Metotrexatearticulares (piedra angular en artritis reumatoide), manifestaciones articulares 7.5-
15mg/sem. Si no responden a la terapia inicial lo ms usado en terapia biolgica es rituximab (anti-CD-20).). Anticoagulacin puede
estar indicada en complicaciones tromboticas.

CASO CLINICO
Mujer de 17 aos que padeca de LES. En el momento del diagnstico debut con una nefritis lpica manifestada por un sndrome
nefrtico, con protenas en orina de 24 horas de 3,3 g, colesterolemia de 280 mg% y albuminemia de 1,8 g/dL, por lo cual recibi
tratamiento con metil-prednisolona succinato en bolos durante tres das (tres dosis), y se mantuvo con prednisona 1 mg/kg de
peso/da, observndose mejora del cuadro. Igualmente la paciente present una cefalea intensa que calmaba parcialmente con
analgsicos y sin signos de irritacin menngea, que fue interpretada como un sntoma asociado al lupus. Continu el tratamiento con
prednisona hasta 15 das despus de su egreso, pero disminuy a una dosis de 35 mg/da. Un mes despus, la paciente acudi
nuevamente para control. Los exmenes demostraron deterioro de la funcin renal, asociado a un sndrome nefrtico impuro, con
hipertensin arterial (160/110). Se aument la dosis de prednisona a 1 mg/kg de peso/da y se agreg captopril 50 mg cada 8 horas. La
paciente refera persistencia de la cefalea frontoparietal, opresiva, de fuerte intensidad, que ya no mejoraba con analgsicos y que era
exacerbada por los movimientos de la cabeza, sin otros signos y sntomas asociados. A los pocos das del ingreso present fiebre de 40
C, nuseas, vmitos y fotofobia.

PREGUNTA
Cual es la conducta a seguir mas adecuada?

RESPUESTA
a.- Agregar diurtico, neuroproteccion y antibitico.
b.- Realizar punsion lumbar para citoquimico, citolgico y cultivo.
c.- Realizar IRM de crneo.
d.- Aumentar corticoide y FARME.

PREGUNTA
Entre 30 y 50% de la morbilidad y mortalidad de los pacientes con lupus eritematoso sistmico (LES) son atribuidas a las infecciones,
cual de las siguientes es la menos frecuente?

RESPUESTA
a.- SNC.
b.- Piel.
c.- Pulmones.
d.- Sistema genitourinario.

CASO CLINICO
Mujer de 67 aos de edad diagnosticada de hipertensin arterial. Ingres por sensacin nauseosa, infeccin urinaria y anemia. Anlisis:
Ht 27,8%; Hb 9,5 g/dl. VSG 63 mm. Urea 182 mg/dl; creatinina 4,7 mg/dl; calcio 9 mg/dl; fsforo 4,2 mg/dl, protenas totales 9 g/dl.
Aclaramiento de creatinina (frmula de Cockroft-Gault): 17,73 ml/min. Inmunoprotenas y complemento normal. Cadenas Kappa-s 774
mg/dl, cadenas Lambda 392 mg/dl. En el proteinograma se observ un pico de amplia base en regin Gamma, con aumento de IgG
(193%) y cadenas ligeras Kappa (191%) y Lambda (180%). ndice K/L= 1,97. Cadenas ligeras en orina: cadenas Kappa 13,7 mg/dl (0-0,7);
cadenas Lambda 6,880 (0-0,39). Se realiz aspirado y biopsia de mdula sea, que result normal. THS: 4,85 UI/ml, T4 libre 1,03 ng/fl;
Ac anti-TPO 22,5 UI/ml; Ac anti-Tiroglobulina 115,3 UI/ml. PTH: 110 pg/ml. Orina (tira reactiva): protenas 25 mg/l; sedimento:
abundantes leucocitos. Marcadores tumorales: normales. Serologa viral: negativa. Autoanticuerpos: ANA +, anti-ADN positivos.
PREGUNTA
Considerando las caractersticas del caso clnico, cual es el abordaje teraputico mas adecuado?

RESPUESTA
a.- Metotrexate.
b.- Ciclosporina.
c.- Prednisona.
d.- Aziatropina.
MANUAL DE TRABAJO DEL CURSO ENARM CMN SIGLO XXI
CURSO ENARM CMN SIGLO XXI TEL: 36246001 Pharmed Solutions Institute PGINA 99

PREGUNTA
El lupus eritematoso sistmico (LES) es una enfermedad inflamatoria con afectacin sistmica, cual es el porcentaje de pacientes que
presentan signos de nefropata?.

RESPUESTA
a.- Al menos 60%.
b.- Al menos 50 %.
c.- Al menos 40 %.
d.- Al menos 30%

CASO CLINICO
Mujer de 47 aos de edad con Nefropata lpica. AHP a los 17 aos presenta Prpura Trombopnica que lleva a una esplenectoma, a
los 19 aos de edad ingresa tras presentar un brote de artralgias, fiebre, edemas y lesiones eritematosas en cara, siendo diagnosticado
de LES. En ese momento destaca la presencia de sndrome nefrtico con alteraciones en el sedimento, una creatinina srica de 1.9
mg/dl y en el estudio cardiolgico se objetiva prolapso de la vlvula mitral. Se practica biopsia renal. El cilindro contiene 12 glomrulos,
grandes, lobulados, con proliferacin mesangial difusa de distribucin irregular, ncleos en carirorrexis, trombos intracapilares
ocasionales y paredes capilares engrosadas en asa de alambre. Las pequeas arterias, no tienen alteraciones. La tcnica de
inmunofluorescencia directa demuestra la presencia de depsitos subendoteliales y mesangiales de IgG, C3, C1q, IgM e IgA.
Diagnstico: Nefritis lpica Clase IV. Se decide tratamiento con corticoides y azatriopina, normalizndose la funcin renal y
negativizndose la actividad inmunolgica, persistiendo proteinuria de 2 gr. /24 h. A los 21 aos se detectan cifras elevadas de tensin
arterial. En su primer embarazo, presenta aborto espontneo tras 2 meses de amenorrea. A los 22 aos se retira la inmunosupresin en
el curso de su segundo embarazo que llega a trmino, presentando hipertensin arterial en el octavo mes. Inmediatamente tras el
parto en mayo de presenta empeoramiento de su situacin con hipertensin arterial y proteinuria de 3 gr. /24 h, reinicindose el
tratamiento con esteroides y azatriopina, quedando la paciente asintomtica, sin proteinuria, con funcin renal normal, estudio
inmunolgico negativo pero persistiendo la hipertensin arterial. A los 25 aos de edad comienza con crisis epilpticas detectndose en
TAC cerebral lesiones isqumicas mltiples y un mes despus, segundo brote lpico con fiebre, afectacin articular, cutnea,
reaparicin de proteinuria con funcin renal normal y actividad inmunolgica. A los 36 aos de edad, es intervenida de aneurisma
fusiforme de la arteria cartida derecha. Un ao despus presenta un accidente cerebro vascular con hemiplejia izquierda y
recuperacin total, se observaron dilataciones en cartida y aorta abdominal en estudios posteriores.

PREGUNTA
Considerando la comorbilidad del LES cual de las siguientes entidades clnicas es mas probable considerando la evolucin del caso?

RESPUESTA
a.- Sindrome infeccioso.
b.- Arteriosclerosis.
c.- Sindrome antifosfolipidico.
d.- Necrosis avascular sea.

CASO CLINICO
Un varn de 70 aos, con antecedentes de diabetes mellitus, ingres por febre de 38,5C de una semana de evolucin. A la exploracin
destacaba presin arterial 106/75 mmHg, frecuencia cardaca de 95 lpm y edema importante hasta ambas rodillas. En el tobillo derecho
y en el taln izquierdo presentaba lceras cutneas con un exudado purulento del cual se tomaron muestras. En la bioqumica, al
ingreso, destacaba leucocitosis 19.800 cl/mm3, AST 68 U/L, creatinina 260 mol/L, urea 16,4 mmol/L; el resto fue normal. El ECG era
anodino. La radiografa de trax evidenci una cardiomegalia moderada. El Eco-Doppler cardaco mostr un derrame pericrdico difuso
de aproximadamente 1 cm, con imgenes compatibles con depsito de fibrina y signos sugestivos de compromiso hemodinmico. En
las 12 horas siguientes present taquicardia y oliguria progresivas, con hipotensin arterial mal tolerada.

PREGUNTA
Cul es la conducta a seguir ms adecuada.

RESPUESTA
a.- Pericardiocentesis.
b.- Pericardientomia urgente.
c.- Manejo conservador.
d.- Correccin del estado hemodinamico.

CASO CLINICO
Paciente con 50 aos de edad, con LES haca cuatro aos. Durante su evolucin present como intercurrencia un sndrome
antifosfolipdico y nefritis lpica que requiri tratamiento con anticoagulantes orales, azatioprina 100 mg/da, hidroxicloroquina 200
mg/da y prednisona 20 mg/da en dosis decreciente. Es ingresado a urgencias por familiares debido a que presenta desorientacin,
dificultad para movilizar el brazo y pierna del mismo lado y cegura sbita.

PREGUNTA
Cul es la complicacin mas probable en este caso?

MANUAL DE TRABAJO DEL CURSO ENARM CMN SIGLO XXI
CURSO ENARM CMN SIGLO XXI TEL: 36246001 Pharmed Solutions Institute PGINA 100

RESPUESTA
a.- Vasculitis cerebral.
b.- Isquemia cerebral transitoria.
c.- Embolia cerebral.
d.- Neurolupus.

CASO CLINICO
Se trata de paciente femenino de 32 aos de edad la cual se encuentra diagnosticada con lupus eritematoso sistmico con 4 aos de
evolucin con tratamiento irregular, ingresa a admisin continua debido a que desde hace 48 horas inicia sbitamente con estado
confusional agudo previamente con letargia, a la exploracin fsica se observa ictrica, su estado no permite responder preguntas, sus
signos vitales se encuentra taquicardia, hipotensin, palidez y disnea, no se auscultan alteraciones cardiorespiratorios, sin embargo en
abdomen se detecta esplegnomegalia, se realizan exmenes de rutina con hemoglobina de 6 g/dl, leucos de 6,200 y plaquetas de
300,000, bilirrubina 4,5 g/dl con reticulocitos de 19 % as como microesferocitosis, resto sin datos por agregar:

PREGUNTA
Cual es la complicacin actual que presenta esta paciente que es potencialmente mortal aguda?

RESPUESTA
a.- Sindrome hemoltico-uremico.
b.- Purpura trombocitopenica trombotica.
c.- Anemia hemoltica autoinmunitaria.
d.- Anemia Hemolitica microangiopatica.

VASCULITIS:
CIENCIAS BASICAS: Grupo heterogneo de enfermedades raras, que pueden ocurrir de forma independiente o como complicaciones de
una enfermedad previamente establecida. Involucran la vasculatura de cualquier rgano, la presentacin depende del calibre del vaso
afectado.
CLASIFICACION:
ARTERITIS DE CELULAS GIGANTES (ACR): La ms comn de las vasculitis
sistmicas. Tambin llamada enf. de Horton, afecta a las ramas craneales
del arco artico, especialmente a la arteria temporal. Ms comn en
mayores de 50 aos. Ms frecuente en mujeres en relacin 2:1. CLINICO:
Fiebre de bajo grado, fatiga, prdida de peso, mialgias, anorexia, cefalea
(60%). Afeccin de la arteria oftlmica (20%), neuritis ptica, diplopa,
amaurosis fugaz y ceguera (dar tx. mdico precoz urgente; esteroides en
dosis altas). Afeccin de la A. facial: claudicacin mandibular. CRITERIOS
DIAGNOSTICOS: 1. Edad >50 aos 2. Cefalea de reciente inicio 3.
Hipersensibilidad de la arteria temporal (piel cabelluda) o disminucin de la pulsacin 4. VSG >50mm/h 5. Biopsia granulomas y
vasculitis. Con 3 de 5 criterios se hace el diagnostico. DIAGNOSTICO: VSG (98%), PCR elevadas, anemia y alteracin de pruebas de la
funcin heptica (70%), biopsia de la arteria afectada (afeccin en parches, no es continua); infiltrado mononuclear, con presencia de
granulomas y clulas gigantes. TRATAMIENTO: Prednisona 1mg/kg/da, durante las primeras semanas con descenso gradual. ARTERITIS
DE TAKAYASU (AT): Llamada enf. sin pulso o sndrome del arco artico. Ms comn en mujeres en edad reproductiva. Proceso
inflamatorio crnico granulomatoso, que afecta la aorta y sus ramas, provocando generalmente sntomas isqumicos. CLINICA: Se
divide en fases: fase I; periodo inflamatorio con fiebre, artralgias y prdida de peso, trill artico o subclavio. Fase II; hipersensibilidad y
dolor en vasos sanguneos, disminucin asimtrica en pulsos en extremidades, claudicacin, hipertensin renovascular, sincope
neurognico. Fase III; periodo fibrotico. La artera ms afectada hasta 93% es la subclavia. CRITERIOS DIAGNOSTICOS: 1. Edad <40aos
de edad 2. Claudicacin de extremidades 3. Disminucin del pulso de la arteria braquial 4. Diferencia de presin arterial sistlica
>100mmHg en los brazos 5. Trill sobre arterias subclavia o aorta. 6. Arteriografa anormal. Con 3 de 6 criterios se hace el diagnostico.
CLASIFICACION: de acuerdo al sitio afectado: I: Ramas del arco artico. IIa Aorta ascendente, arco artico y sus ramas. IIb Aorta
descendente, arco artico y sus ramas , A. torcica descendente. III. A. torcica descendente, A abdominal y/o arterias renales. IV. A.
abdominal y/o A. renales. V. Combinacin de los tipos IIb y IV. DIAGNOSTICO: VSG Y PCR elevadas. Arteriografa=Gold estndar;
oclusin, estenosis, irregularidad y aneurisma. TRATAMIENTO: Esteroides, MTX, Aziatropina y antiplaquetarios, en enfermedad
refractaria micofenolato y ciclofosfamida. PANARTERITIS NODOSA CLASICA (PAN): Inflamacin necrotizante de arteria de pequeo y
mediano calibre (respeta capilares y vnulas). Disminucin de la luz vascular. Se asocia a VHB 30% y a VHC 5% CLINICA: Insuficiencia
renal, HTA (70%), purpura papable (50%), mononeuritis mltiple (50%), dolor abdominal (40%), ICC, IAM, pericarditis (30%), raras
afecciones pulmonares. CRITERIOS DIAGNOSTICOS: 1. Prdida de peso >4Kg 2. Livedo reticularis 3. Dolor e hipersensibilidad testicular
(orquitis) 4. Mialgias, debilidad, dolor de extremidades (50-60%). 5. Mononeuropata o polineuropata 6. Presin diastlica >90mmHg 7.
BUN elevado (>40mg/dl) o Cr mayor de 1.5 mg/dl 8. Virus de hepatitis B 9. Arteriografa anormal (aneurisma, oclusin, estenosis de
arterias viscerales) 10. Biopsia; vasculitis de mediano y pequeos vasos. Con 3 de 10 criterios se hace el diagnostico. DIAGNOSTICO:
Angiografa, biopsia; infiltrado de PMN y necrosis fibrinoide (caracterstico ausencia de granulomas y eosinofilos).TRATAMIENTO:
Glucocorticoides, ciclofosfamida, antiviral si est asociada a VHB O VHC. GRANULOMATOSIS DE WENEGER (GW): Enfermedad
inflamatoria granulomatosa necrotizante con vasculitis sistmica. Predomina en adultos jvenes. Predomina en capilares y vnulas.
CLINICO: Vas areas superiores (95%); Sinusitis, otitis media y destruccin de tabique nasal. Pulmn (90%); Infiltrados pulmonares
cavitados bilaterales y no migratorios. Ndulos hemorrgicos y hemoptisis. Glomerulonefritis (75%); Focal y segmentaria rpidamente
progresiva. Ocular (50%); epiescleritis, uvetis. Neuropatas perifricas o craneales. CRITERIOS DIAGNOSTICOS: 1. Inflamacin oral o
nasal; ulceras orales, descarga nasal purulenta o hemorrgica 2. Radiografa de trax con presencia de ndulos, infiltrados fijos o
VASOS DOMINANTES VASCULITIS
Grandes vasos Arteritis de clulas gigantes (ACR)
Arteritis de Takayasu
Medianos vasos Poliarteritis nodosa (PAN)
Enf. De Kawasaki (AT)
Pequeos vasos y
arterias medianas
ASOCIADAS A ANCAS
Granulomatosis de Weneger (GW)
Sx. de Churg-Strauss (SCS)
Poliangitis microscpica (PA)
Pequeos vasos
(leucocitoclstica)
Purpura de Henoch-Scholein (PHS)
Arteritis leucocitoclstica cutnea

MANUAL DE TRABAJO DEL CURSO ENARM CMN SIGLO XXI
CURSO ENARM CMN SIGLO XXI TEL: 36246001 Pharmed Solutions Institute PGINA 101

cavitaciones 3. Hematuria microscpica o restos de eritrocitos 4. Biopsia con inflamacin granulomatosa. Con 2 de 4 criterios se hace el
diagnostico. DIAGNOSTICO: c-ANCA (Anticuerpos antiproteinasa-3) positivos 90%. TAC o Rx de trax; ndulos, infiltrados, cavitaciones.
Elevacin de BUN, Cr, proteinuria, sedimento urinario activo. Biopsia; inflamacin granulomatosa necrotizante de arteriolas, capilares y
vnulas. TRATAMIENTO: Ciclofosfamida + prednisona, MTX o aziatropina. TMP/SFX puede prevenir las recadas de infecciones de vas
areas superiores. POLIANGITIS MICROSCOPICA (PA): Vasculitis necrotizante de pequeos vasos, puede haber o no granulomas.
CLINICO: Sntomas constitucionales; prdida de peso, fiebre (50%), fatiga, mialgias. Renal; hematuria, glomerulonefritis rpidamente
progresiva (90%). Pulmonar; tos y/o hemoptisis, hemorragia pulmonar (30%). Neurolgico; mononeuritis multiplex (40%).
DIAGNOSTICO: p-ANCA (anti-mieloperoxidasa) positivos 70%. Biopsia; inflacin pauci-inmune necrotizante de arteriolas capilares y
vnulas, con o sin granulomas o infiltrados eosinofilicos. TRATAMIENTO: Ciclofosfamida + aziatropina, MTX o aziatropina (ahorradores
de esteroides). SINDROME DE CHURG-STRAUSS (SCS): Inflamacin granulomatosa eosinofilica que involucra pulmn. A cualquier edad
ms entre 30-40 aos. CLINICO: Asma de difcil control (95%), enfermedad eosinofilica infiltrativa, vasculitis de pequeos vasos con
granulomas. Neuropata; mononeuritis multiplex (60-70%), parestesias en guante o calcetn, arteritis coronaria y miocarditis.
Glomerulonefritis. CRITERIOS DIAGNOSTICO: 1. Asma 2. Eosinofilia (>10%) 3. Mono o polineuropata 4. Infiltrados pulmonares
migratorios transitorios 5. Anormalidades paranasales 6. Eosinofilia extravascular en la biopsia. Con 4 de 6 criterios se hace el
diagnostico. DIAGNOSTICO: Dato cardinal para diagnostico: eosinofilia perifrica 80%. ANCA (c-ANCA o p-ANCA) positivo en 50%.
Biopsia: microgranulomas, necrosis fibrinoide y trombosis de pquelas arterias y venas con infiltrados eosinofilicos. Rx. de trax: puede
haber infiltrados pulmonares. TRATAMIENTO: Ciclofosfamida + prednisona, MTX o aziatropina. PURPURA DE HENOCH-SCHONLEIN
(PHS): La ms comn de las vasculitis (leucocitoclstica) en nios (<16aos). Suele aparecer despus de una infeccin de tracto
respiratorio o ingesta de frmacos, es mediada por IgA. Producida por depsito de inmunocomplejos. CLINICO: Purpura palpable en
superficies extensoras y glteos, poliartralgias, dolor abdominal tipo colico (edema de pared, invaginacin intestinal poco frecuente).
Nefritis; hematuria microscpica y proteinuria. Fiebre. CRITERIOS DIAGNOSTICOS: 1. Purpura palpable 2. Edad menor de 20 aos al
momento de la aparicin 3. Angina intestinal (dolor abdominal) 4. Biopsia; granulocitos en la pared de arteria o venas. Con 2 de 4
criterios se hace el diagnostico. DIAGNOSTICO: Conteo plaquetario, elevacin de IgA srica. Biopsia de piel: vasculitis leucocitoclstica
con IgA y depsitos de C3 en la pared de los vasos. Biopsia renal; deposito mesangial de IgA. TRATAMIENTO: De sostn. Esteroides
(1mg/kg) y frmacos modificadores de enfermedad severa. CRIOGLOBULINEMIA: Vasculitis rara con depsitos inmunes de
crioglobulinas (protenas que se precipitan con el fro), afecta piel y glomrulo, se asocia con infeccin por VHC (90%). CLINICA:
Discinecia, confusin, cefalea, y EVC isqumico, causado por hiperviscocidad en tipo I. Estasis vascular; livedo reticularis, necrosis
digital, en tipo I. Triada de manifestaciones en tipo II Y III; purpura, artralgias, mialgias. CLASIFICACION: Tipo I: Constituida por una sola
inmunogloblina IgM. Macroglobulinemia de Waldestron o mieloma. Tipo II: IgM monoclonal tipo K, que presenta actividad tipo factor
reumatoide e IgG policlonal. Sx. proliferativos. Tipo III: Constituida por 2 inmunoglobulinas (IgM-IgG) de carcter policlonal: Forman
complejos Ag:Ac circulantes: procesos que cursan con inmunocomplejos, infecciones crnicas, cirrosis. DIAGNOSTICO: Serologa viral,
hipocomplementemia, crioglobulinas. TRATAMIENTO: Terapia antiviral, esteroides, ciclofosfamida, plasmaferesis. SINDROME DE
BEHCET: Vasculitis multisistemica, caracterizada por presencia de ulceras genitales y orales recurrentes dolorosas. Se asocia con HLA-
B51. CLINICO: Ulceras orales dolorosas (>3 episodios anuales), de base necrtica amarillenta, se curan en 1 o 2 sem sin dejar cicatriz.
Ulceras genitales (80%), si dejan cicatriz. Foliculitis (80%), eritema nodoso (50%), erupciones acneiformes. Ocular; manifestacin ms
grave, uvetis posterior que puede condicionar prdida de visin (20%, tx. Agresivo con ciclosporina o anti-TNF). Puede haber meningitis
asptica, meningoencefalitis. DIAGNOSTICO: Fenmeno de patergia (patognomnico), se inocula sol. Inyectable lesin de >2cm es
positiva. Biopsias de ulceras. Fondoscopia. CRITERIOS DIAGNOSTICOS: 1. Ulceras aftosas orales recurrentes 2. Ulceras genitales
recurrentes 3. Lesiones oftlmicas: uvetis (con hipopin), escleritis, vasculitis retiniana, neuritis ptica. 4. Lesiones cutneas; ppulas
foliculitis, eritema nodoso 5. Patergia positiva. El diagnostico se hace con el N.1 y 2 ms del resto. TRATAMIENTO: Aziatropina, para
ulceras (colchicina, dapsona, talidomida), corticoides, para uvetis la ciclosporina. ENFERMEDAD DE KAWASAKI: Sindrome
linfomucocutneo. Ms comn en menores de 5 aos (80%), ligero predominio en sexo masculino. Etiologa desconocida. CLINICA:
Cuadro exantemtico febril, que no responde a antibiticos y congestin conjuntival o no. Alteraciones labiales; eritema, fisuras,
sangrado con formacin de costras y lengua en frambuesa. Adenopatas cervicales dolorosas y exantema en tronco y extremidades.
Formacin de aneurismas (25%), vasculitis coronarias (forma ms mortal), carditis (80%). DIAGNOSTICO: Clnico, factores reactantes de
fase aguda, angiografa coronaria, no se asocia a ningn autoanticuerpo. CRITERIOS DIAGNOSTICOS: Fiebre persistente por al menos 5
dias, mas 2 de los siguientes: 1. Cambios en extremidades inferiores 2. Exantema polimorfo 3. Inyeccin conjuntival bilateral 4. Cambios
en labios y cavidad oral 5. Linfadenopata cervical. TRATAMIENTO: Gammaglobulina IV (2g/Kg dosis nica o 400 mg/kg, durante 4 dias.
ASA de 100-1500mg/da. Controversial uso de esteroides

CASO CLINICO
Mujer de 60 aos, con antecedentes de hipertensin arterial, que bruscamente present un cuadro de diarrea matutina, acompaada
durante 30 minutos de cefalea frontal invalidante, que cedi con analgesia convencional y repiti varias maanas sucesivas. Quince das
despus, durante tres horas y coincidiendo con la cefalea, present una hipoestesia derecha global, acompaada de parestesias y
debilidad en la extremidad inferior derecha. TAC craneal y resonancia magntica nuclear (RMN) vertebromedular que no mostraron
enfermedad, ecocardiogrfico fue normal. A las 72 horas, present otro episodio de debilidad, tambin de inicio brusco, con una
duracin de 10 horas y que afect al hemicuerpo izquierdo. Una nueva TAC craneal tampoco mostr enfermedad. El estudio del lquido
cefalorraqudeo incluyendo bioqumica, inmunoglobulinas, anticuerpos antineuronales y bandas oligoclonales result normal, salvo una
discreta linfocitosis, as como negativos fueron los cultivos, las serologas y diversas PCR como virus herpes, BK y JC. Se descart la
enfermedad de Whipple.

PREGUNTA
Cul es el estudio ms adecuado para establecer el diagnostico de vasculitis?

RESPUESTA
a.- Angio-IRM.
MANUAL DE TRABAJO DEL CURSO ENARM CMN SIGLO XXI
CURSO ENARM CMN SIGLO XXI TEL: 36246001 Pharmed Solutions Institute PGINA 102

b.- Angiografia.
c.- Biopsia cerebral.
d.- ANA y ANCA.

CASO CLINICO
Paciente de sexo femenino, de 21 aos de edad, diagnosticada de AT. Consult por dolor articular y claudicacin del miembro superior
izquierdo de 4 meses de evolucin. Al examen fsico presentaba ausencia de pulso radial izquierdo y signos de insuficiencia artica. Se
realiz una angiografa digital (AD) que mostr hallazgos tpicos de AT. Por falta de respuesta al tratamiento con prednisona, se indic
una angioplastia de arteria cartida primitiva izquierda y tratamiento con cido acetilsaliclico (AAS) y metotrexato, logrando mejora
clnica. Pero iniciando con diarrea y dolor abdominal. Se solicit angiografa por resonancia magntica abdominal que no evidenci
alteraciones; la inmunoglobulina A (IgA) anti-endomisio result positiva y biopsia de duodeno.

PREGUNTA
Que resultados histopatolgicos espera encontrar.

RESPUESTA
a.- Atrofia difusa de vellosidades.
b.- Infiltrado inflamatorio en la lamina propia.
c.- Datos de isquemia generalizado.
d.- Infiltrado inflamatorio crnico.

CASO CLINICO
Paciente femenina de 22 aos de edad con historia de 2 meses de evolucin con sinusitis, otitis media bilateral complicada con
mastoiditis, conjuntivitis bilateral, artralgia en hombros, codos, rodillas y tobillos, tos con expectoracin sanguinolenta, debilidad
generalizada y cansancio fcil, asociado a fiebre no cuantificada sin predominio de horario y a prdida de peso aproximadamente de
10% de su peso habitual. Hace ms o menos tres semanas presenta cuadro de odinofagia, lumbalgia, disuria, coluria y refiere un
episodio de epistaxis aislada. SV: Temperatura: 38,2 C, FC: 110 latidos/minuto, FR: 22 ciclos/minuto. Tensin Arterial: 120/90 mmHg.
Radiografa de trax: se identifica infiltrado de tipo intersticial de predominio derecho parahiliar, en menor grado, este tipo de
infiltrado se visualiza hacia la base pulmonar izquierda. No hay signos de lesiones nodulares ni cavitaciones. TAC: infiltrados nodulares
en ambos campos pulmonares, cavitacin en lbulo superior derecho, tenue patrn de vidrio esmerilado posterobasal derecho y
crecimientos ganglionares precarinales, parahiliares y subcarinales.

PREGUNTA
Considerando el diagnostico del caso, cual de las siguientes aseveraciones sobre su tratamiento no es adecuada?

RESPUESTA
a.- Pulsos de ciclofosfamida 1 g intravenoso (I.V.) mensuales por 6 meses y luego pulsos de ciclofosfamida 1 g I.V. cada 3 meses por un
ao.
b.- Pulsos de Metilprednisolona 1 g I.V. cada da por 7 das, luego se administra 1 mg/kg/da por 6 semanas y posterior reduccin
gradual.
c.- La combinacin de ciclofosfamida y metilprednisolona se utiliza para inducir la remisin del cuadro.
d.- Leuprolide 11,5 mg I.M. cada mensual, para evitar amenorrea como efecto secundario de la ciclofosfamida.

PREGUNTA
Segn los criterios diagnsticos de GW segn el American College of Rheumatology (ACR), son: Sedimento urinario anormal (cilindros
hemticos o ms de 5 eritrocitos por campo de alto poder). Radiografa de trax con hallazgos anormales (ndulos, cavidades o
infiltrados). Ulceras orales o secrecin nasal; y biopsia con inflamacin granulomatosa. Cuantos criterios se requieren para confirmar el
diagnostico?

RESPUESTA
a.- 1.
b.- 2.
c.- 3.
d.- 4

CASO CLINICO
Paciente femenina, de 44 aos de edad, con antecedentes patolgicos personales de asma bronquial de inicio tardo (35 aos), quien
llevaba tratamiento esteroideo regular en los perodos de crisis, y que comenz 4 aos despus con entumecimiento y debilidad en
ambos miembros inferiores, por lo que se interpret como un sndrome polineuroptico. A los 3 meses comenz con ligera mejora del
cuadro, fue ingresada en varias ocasiones con la misma impresin diagnstica pero agrego gran debilidad en ambos miembros
inferiores. Al interrogatorio se constat: asma bronquial de inicio tardo precedida de rinitis, fenmeno de Raynaud, diarreas
frecuentes, con antecedentes de gastritis. Al examen fsico: cuadriparesia con predominio en miembros inferiores, con cambios trficos
dstales, dedos en martillo con aumento del arco plantar, disminucin del tono muscular, as como abolicin de los reflejos
osteotendinosos. Eosinofilia marcada (valores de 4). El estudio de rayos X de trax mostr infiltrado inflamatorio. El estudio de
conduccin nerviosa (ECN) evidenci la presencia de un sndrome polineuroptico. Se comenz tratamiento con corticosteroides,
metilprednisolona y se continu con prednisona y vitaminoterapia, con lo cual la paciente mostr una mejora significativa. El ciclo de
MANUAL DE TRABAJO DEL CURSO ENARM CMN SIGLO XXI
CURSO ENARM CMN SIGLO XXI TEL: 36246001 Pharmed Solutions Institute PGINA 103

tratamiento se repiti cada 2 meses. Al cabo de 6 meses la paciente se encuentra deambulando sin apoyo y con discreta afeccin del
ECN.

PREGUNTA
Se diagnostico SCS, considerando la patologia cual de las siguientes manifestaciones no es frecuente observar?

RESPUESTA
a.- Vasculitis de los pequeos vasos.
b.- Granulomas vasculares y/o extravasculares.
c.- Eosinofilia perifrica que ocurre en los pacientes asmticos sin historia de rinitis alrgica o sinusitis.

CASO CLINICO
Paciente femenina, de 45 aos de edad, con antecedentes de asma bronquial que comenz a los 27 aos. Por un cuadro clnico, 3
meses antes, caracterizado por lesiones en piel, diagnosticado una vasculitis. Comenz con tratamiento esteroideo con lo que mejor
su cuadro de forma relativa. Posteriormente la paciente comenz a presentar entumecimiento en el miembro superior derecho y
posteriormente en la extremidad inferior contralateral, con ligera impotencia funcional. Despus padeci de diarreas, edemas en la
cara y en miembros inferiores y cefalea de caracterstica vascular. Se encontr hipoestesia distal, superficial y profunda, cambios
trficos, con disminucin de reflejos osteotendinosos, adems se encontr el fenmeno de Raynaud, hipereosinofilia y aumento del
valor de creatinina (210 ml/L). El ecocardiograma mostr evidencias de pericarditis de pequea cuanta y el rayos X de trax evidenci
infiltrado inflamatorio. Se encontr evidencia de mononeuropata multiplex. La TAC de crneo evidenci la presencia de 3 pequeas
lesiones hipodensas con caractersticas vasculares. Se comenz tratamiento con methilprednisolona, no teniendo una respuesta
adecuada, solo con leve mejora sintomtica. La paciente se mantiene actualmente bajo tratamiento inmunosupresor, azatioprina
100/diarios.

PREGUNTA
Los criterios para la clasificacin del SCS 6 criterios: el asma, la eosinofilia mayor que 10 % en recuento de leucocitos diferenciales, la
mononeuropata o polineuropata, infiltrados pulmonares, anormalidad del seno paranasal y la biopsia de vasos sanguneos con
eosinofilia intra/extravascular. Cuantos criterios son necesarios para establecer el diagnostico?

RESPUESTA
a.- 2.
b.- 3.
c.- 4.
d.- 5.

CASO CLINICO
Paciente varn de 62 aos que acude a consulta externa para una exploracin rutinaria. En la exploracin oftalmolgica la agudeza
visual (AV) fue de 1 en ambos ojos (AO). Con pupilas isocricas y normorreactivas, sin defectos aferentes, tensin por aplanacin de 16
mmHg en AO. En el estudio biomicroscpico anterior destacaba una esclerosis cristaliniana moderada. En la retina del ojo izquierdo (OI)
haba dos manchas algodonosas asintomticas, una de 250 micras y otra puntiforme a lo largo de la arcada temporal inferior. Un
estudio fsico completo no detect patologa. Los estudios de laboratorio fueron normales salvo una protena C reactiva de 4,4 mg/dl,
una velocidad de sedimentacin glomerular (VSG) de 56 mm/1. hora.

PREGUNTA
Cual es la conducta a seguir mas adecuada para establecer el diagnostico?

RESPUESTA
a.- Biopsia de la arteria temporal.
b.- Biopsia de la arteria cartida.
c.- Biopsia de la occipital.
d.- Biopsia de la oftlmica.

CASO CLINICO
Mujer de 83 aos que acude a consulta por disminucin bilateral de la agudeza visual. Entre sus antecedentes destacaba ciruga
bilateral de cataratas hace 5 aos. La AV era de 0,4 en el ojo derecho (OD) y 0,3 en OI. La tensin por aplanacin fue de 18 mmHg en OD
y de 16 mmHg en OI, con pupilas isocricas y normorreactivas. En la exploracin fundoscpica destacaba una esclerosis coroidea y una
mancha algodonosa de 20 micras de dimetro en el OD. La exploracin fsica demostr una dificultad importantsima en la movilidad
con claudicacin de las extremidades inferiores, astenia, un leve soplo carotdeo, cefalea en casco durante 8 meses y unas arterias
temporales endurecidas. Durante el estudio (en 4 das) la paciente desarroll una neuritis ptica anterior en el OD y con velocidad de
sedimentacin glomerular de 94 mm/1. hora, protena C de 5,85 mg/dl, 400 Mil/mm3 plaquetas, anemia hipoproliferativa leve y
elevacin de la fosfatasa alcalina (134 UI/L).

PREGUNTA
Considerando las sintomatologia oftalmolgica cuales la causa mas frecuente?

RESPUESTA
MANUAL DE TRABAJO DEL CURSO ENARM CMN SIGLO XXI
CURSO ENARM CMN SIGLO XXI TEL: 36246001 Pharmed Solutions Institute PGINA 104

a.- Diabetes mellitus.
b.- Hipertensin arterial.
c.- Colagenopatas.
d.- Patologa tumoral.

CASO CLINICO
Mujer de 27 aos que consult por un cuadro de un mes de evolucin de dolor en brazo izquierdo acompaado de parestesias y
paresias homoloterales que aumentaban con la actividad y cedan con el reposo. Antecedentes: 12 aos antes: Fiebre Reumtica
tratada con penicilina benzatnica durante 7 aos, 4 aos antes: Embarazo ectpico en trompa de Falopio con ooforectoma izquierda 3
aos antes: Episodio de dolor lumbar de comienzo sbito durante una actividad fsica tratado con corticoides que abandon por cuenta
propia 2 aos antes: Diagnstico de anemia, tratada con hierro intramuscular durante un ao 1 ao antes: Disminucin de la agudeza
visual y fotofobia durante la actividad fsica que persiste hasta la actualidad. Episodio de taquicardia y disnea de comienzo sbito. Se
diagnostic taquicardia sinusal y comenz tratamiento con atenolol 8 meses antes: Cefalea de inicio sbito, intensa, holocraneana
acompaada de hipertensin arterial en tratamiento con enalapril. Examen Fsico: Signos vitales: PA 140/60 mmHg en brazo derecho,
inaudible en brazo izquierdo; FC 96 lpm; FR 18 cpm; T 36.1 C. Cabeza y Cuello: pulsos carotdeos asimtricos con disminucin franca
del lado izquierdo, frmito carotdeo derecho, soplo carotdeo bilateral. Cardiovascular: soplo sistlico 5/6 en focos artico y pulmonar
que irradia a cuello y aumenta con la inspiracin. Pulsos radial, femoral, tibial posterior y pedio asimtricos, con disminucin en
hemicuerpo izquierdo. Abdomen: hgado palpable a 1cm del reborde costal, Miembros: disminucin de la fuerza en hemicuerpo
izquierdo. Tono, trofismo, temperatura conservada. Hematocrito 36%, Hemoglobina 12 g/dL, Glbulos blancos 9300/mm3, Plaquetas
284000/mm3, Glicemia 112 mg/dL, Urea 37 mg/dL, Creatinina 0.6 mg/dL, Na 134 mEq/L, K 3.3 mEq/L, plasma no ictrico, ASAT 17 UI/L,
ALAT 37 UI/L, Protenas totales 8.8 g/dL, Albmina 4.7 g/dL, Tiempo de protrombina 13.3 segundos, KPTT 33 segundos, Velocidad de
eritrosedimentacin 70 mm/1 hora.

PREGUNTA
Ante la presencia de un cuadro clnico con manifestaciones sistmicas, anemia, velocidad de eritrosedimentacin (VES) elevada y
alteracin de la aorta y sus ramas, debe pensarse en aortitis, de las siguientes patologas cual es la menos frecuente?

RESPUESTA
a.- Espondolitis anquilosante.
b.- Arteritis de clulas gigantes.
c.- Enfermedad de Behct.
d.- Sindrome de Marfan.

PREGUNTA
Considerando su diagnostico, cual es la indicacin mas frecuente para revascularizar?

RESPUESTA
a.- Hipertensin arterial renovascular.
b.- Claudicacin severa de los miembros.
c.- Isquemia mesentrica
d.- Infarto agudo de miocardio

CASO CLINICO
Un varn de 22 aos, consumidor ocasional de cocana y fumador de 20 cigarrillos/da, fue remitido a nuestro centro por dolor torcico.
La exploracin fsica y la analtica de control fueron normales. El electrocardiograma mostr bloqueo de rama derecha con ondas q en
V1, V2 y V3. La radiografa de trax mostr una imagen calcificada, redondeada superpuesta a la silueta cardiaca. En el ecocardiograma
se objetiv un ventrculo izquierdo dilatado y adelgazado, con hipocinesia general ms marcada en el septo y la cara anterior, con
disfuncin sistlica severa (fraccin de eyeccin del 30%). Se realiz una coronariografa que mostr oclusin completa de la arteria
descendente anterior distal a un aneurisma gigante calcificado y oclusin completa de la coronaria derecha.

PREGUNTA
Cual es el porcentaje de afeccion a las arterias coronarias en esta patologia?

RESPUESTA
a.- 25 %.
b.- 35 %.
c.- 45 %
d.- 55 %.

PREGUNTA
El diagnstico clsico de la EK se basa en la presencia de los siguientes sntomas, cual no es frecuente?

RESPUESTA
a.- Fiebre.
b.- Alteraciones cutneas en las extremidades.
c.- Exantema polimrfico.
MANUAL DE TRABAJO DEL CURSO ENARM CMN SIGLO XXI
CURSO ENARM CMN SIGLO XXI TEL: 36246001 Pharmed Solutions Institute PGINA 105

d.- Conjuntivitis bilateral exudativa.

CASO CLINICO
Femenino de 14 aos de edad, acude a consulta por presentar sangramiento en las encas desde hace varios meses, refiere la madre
que el estomatlogo le indic el cepillado correcto y buchadas de manzanilla 3 veces al da. Examen fsico: Se observa presencia de
caries, encas edematosas, al sondeo bolsas de 4mm en el sector anteroinferior (brecha edente de 3.1-3.2), perdidos por movilidad, no
restituidos por prtesis, sarro supragingival y manchas de origen medicamentoso. Se aprecia pigmentacin melnica en la enca
adherida, hay xerostomia, lengua lisa y brillante e higiene deficiente.

PREGUNTA
Cual es la conducta a seguir en relacin al caso?

RESPUESTA
a.- Yodopovidona oral.
b.- Pilocarpina oral.
c.- Clorhexidina oral.
d.- Prednisona oral.

CASO CLINICO
Mujer hipertensa de 74 aos. Unas semanas antes comienza con astenia, fatigabilidad y disnea progresiva, dolor torcico de
caractersticas inespecficas, hipotensin sintomtica y sncope; no refera fiebre. En el momento del ingreso presentaba una presin
arterial sistlica (PAS) de 90 mmHg, un electrocardiograma con taquicardia sinusal, bloqueo de rama derecha ms hemibloqueo
anterior izquierdo, con PR normal y cardiomegalia-congestin en la radiografa de trax. Analtica bsica: creatinina 1,4 mg/dl, Na 121
mEq/l, K 5,6 mEq/l, aclaramiento de creatinina 95 ml/min, microalbuminuria 9,4 mg/dl, troponina I 2,4 ng/ml, cLDL 78 mg/dl, GOT-GTP
239 U/l, GGT 136 U/l, FA 154 U/l, LDH 904 U/l. Leucocitos 5,9 3 103/ l; hemoglobina 11 g/dl; plaquetas 233 3 103/ l; acti vidad de
protrombina 87%, VSG 133 mm en la primera hora y PCR 13,2 mg/dl. Hemocultivos y serologas varias negativas. La ecocardiografa en
el momento del ingreso muestra un ventrculo izquierdo no dilatado, sin alteraciones segmentarias de la contractilidad y disfuncin
sistlica severa, biventricular, con fraccin de eyeccin del ventrculo izquierdo (FEVI) del 15%, sin otros hallazgos.

PREGUNTA
Considerando los hallazgos clnicos mas relevantes cual es el frmaco de eleccin.

RESPUESTA
a.- Corticosteroides.
b.- Azatioprina
c.- Metotrexato
d.- Ciclofosfamida.

TUBERCULOSIS:
CIENCIAS BASICAS: Es una enfermedad infecciosa que suele afectar a los pulmones y es causada por el complejo Mycobacterium
tuberculosis (M. tuberculosis, M. bovis, M. africanum). La infeccin por M. tuberculosis suele ser asintomtica en personas sanas, dado
que su sistema inmunitario acta formando una barrera alrededor de la bacteria. SALUD PBLICA: Considerada una emergencia
mundial por la OMS, es prioridad casos complicados en los extremos de la edad, multifrmacorresistencia y vnculo con el sida y la
diabetes, e inmunocomprometidos (neoplasias, quimioterapias, trasplantes). OMS informa que 1/3 de la poblacin mundial est
infectada por Mycobacterium tuberculosis. Cada ao: >10 millones de casos nuevos y 3.5 millones de defunciones por TB. En frica y
este de Europa: incidencia es de >300 casos/100,000 hab. por ao. La TB que responde al tratamiento estndar tiene una tasa de
curacin >95%, en multiresistencia es de 50-80%. Menos de 30% de los inmunocompetentes logran la curacin y >del 50% muere
dentro de los primeros 5 aos. En multiresistencia la tasa de mortalidad es de 90%. PATOGENIA: La transmisin de bacilos tuberculosos
se produce bsicamente por va area (tambin ingestin de leche de vaca infectada, contacto con personas enfermeas baciliferas o
animales bovinos enfermos). Las personas infectantes eliminan bacilos a partir de aerosoles (tos, expectoracin) y la infecciosidad
depende del nmero de bacilos eliminados y la susceptibilidad del husped. Las partculas aerosolizadas que contienen bacilos, son
suficientemente pequeas para eludir la 1 barrera defensiva (aparato muco-ciliar), para alcanzar los alveolos pulmonares, donde
comienza la multiplicacin de los bacilos, son fagocitados por lo macrfagos alveolares, se liberan multiples citocinas proinflamatorias,
TNF (ocasiona fiebre y caquexia), IL-1 y IL-6, adems liberan proteasas, urocinasa, implicadas en la destruccin tisular. Secundariamente
los bacilos son transportados por los propios macrfagos a los ganglios regionales donde se produce la respuesta inmunitaria mediada
fundamentalmente por los linfocitos T (inmunidad celular). Perodo de incubacin que oscila entre 6 a 8 semanas. Puede ocurrir que
antes del desarrollo de la respuesta inmunitaria celular se produzca una diseminacin va linfo-hematgena que d lugar a siembra de
bacilos en diversos tejidos: zonas apicales de pulmn, vrtebras, epfisis de huesos largos etc. que condicionen la evolucin ulterior a
enfermedad progresiva tras perodos largos de latencia. En la mayora de los casos de infeccin tuberculosa, hay una destruccin
rpida de bacilos y no se produce enfermedad, el nico indicio residual es la positividad de la PPD. El granuloma se forma por la
interaccin del M. tuberculosis, con la respuesta inmune del husped y liberacin tisular de proteasas; al inicio se hallan neutrofilos,
mas tarde necrosis caseosa (tpica de granulomas producidos por micobacterias) y finalmente calcificaciones. El bacilo no siempre es
eliminado y permanece inactivo, causando una infeccin latente. INFECCION PRIMARIA: Afecta pulmn, se adquiere al inhalar bacilo,
se desarrolla despus de una exposicin inicial. Se necesitan de 50-200 microorganismos para producir la infeccin. De las personas
expuestas al bacilo 30% se infecta y 5% desarrolla enfermedad. Los bacilos inhalados causan alveolitis, adenopatas y linfagitis, lo que se
conoce como complejo primario de Ghon. Clnica; hemoptisis, dolor pleurtico, disnea, fiebre, diaforesis nocturna y prdida de peso.
MANUAL DE TRABAJO DEL CURSO ENARM CMN SIGLO XXI
CURSO ENARM CMN SIGLO XXI TEL: 36246001 Pharmed Solutions Institute PGINA 106

Complicacin: TB endobronquial. REACTIVACION: Es progresiva se desarrolla de un foco previo de TB, el cual puede producir TB
pulmonar o extrapulmonar. Ms comn en inmunocomprometidos (IRC, DM, esteroides, desnutricin, deficiencia de vitamina D.
Clnica; tos, prdida de peso, fatiga, fiebre y diaforesis nocturna. Complicaciones agudas; hemoptisis y neumotrax. Cavitaciones en
(20-45%), manifestacin radiolgica ms comn consolidacin Los aspergillomas se desarrollan dentro de la cavitacin (20%). Hasta en
5% de pacientes reactivacin la manifestacin principal es el tuberculoma (centro: material caseoso y periferia histiocitos epiteliales y
cel. Gigantes multinucleadas). DEFINICIONES (Estndares para la atencin de la TB en Mxico): Caso de tuberculosis: Persona en quien
se establece el diagnostico de tuberculosis pulmonar o extrapulmonar y se clasifica por bacteriologa o estudio histopatolgico en caso
confirmado o no confirmado. Caso de tuberculosis confirmado: Toda persona con cuadro clnico compatible con tuberculosis pulmonar
o extrapulmonar que cumpla adems cualesquiera de los siguientes criterios: 1. Aislamiento de Mycobacterium tuberculosis por
cultivo. 2. Resultado positivo en la baciloscopia. 3. Deteccion de genes de micobacterias por mtodos de biologa molecular (PCR o
ampliacin de RNA). Caso de tuberculosis no confirmado: Toda persona con cuadro clnico compatible con tuberculosis pulmonar o
extrapulmonar sin confirmacin por baciloscopa, cultivo o estudios de biologa molecular pero presenta uno o mas criterios como: 1.
Cuantificacin de adenosin desaminasa (ADA): los niveles de ADA parab TB peritoneal y pleural, estn >70U/ml; para SNC >7U/l. 2.
Antecedente epidemiolgico de convivencia con un caso bacilifero. 3. Reactor a la PPD, con o sin antecedente de BCG. 4. Respuesta
favorable al tratamiento antituberculoso. 5. Estudio histopatolgico compatible con TB. Y alguno de los siguientes estudios de gabinete:
A. TB pulmonar; las radiografas de trax PA y lateral pueden mostrar imagen de sndrome del lbulo medio, derrame pleural,
ensanchamiento mediastinal o patrn miliar. B. TB ganglionar; ultrasonido con imgenes de material calcificado y liquido. C. TB del SNC;
la TAC de crneo puede mostrar datos de aracnoiditis e hidrocefalia; la radiografa de crneo, datos de hipertensin endocraneana. D.
TB osea o enfermedad de Pott: las rx., de columna vertebral AP y lateral muestran destruccin de las vertebras dorsolumbares y
rotoxifoescoliosis. E. TB geniturinaria: urografa excretora muestra imgenes tortuosas debido a rigidez de urteres. F. TB abdominal;
USG o TAC muestran ascitis e imgenes compatibles con tabicaciones. La laparoscopia muestra lesiones granulomatosas ascitis y
fibrina. DIAGNOSTICO: Bsqueda entre contactos de un caso de TB y en grupos o poblaciones de alto riesgo. La baciloscopa se debe
realizar en A) En cualquier muestra clnica excepto orina. B) En todos los casos probables, entre los contactos de un caso, en grupos o
poblaciones de alto riesgo, pacientes dados de alta que acudan con tos productiva, en 3 muestras sucesivas de esputo. C)
Independientemente de la edad, en en quienes clnica y radiolgicamente, se sospecha TB, si la primera serie de 3 hubiera sido
negativa. D) En el control del tratamiento antituberculoso, con una muestra cada mes, la ultima al terminar el tratamiento E) En caso de
sospecha de TB extrapulmonar, F) Sospecha de TB en nios. G) Pacientes en tx., estrictamente supervisado, en quienes al cuarto mes,
persiste baciloscopia positiva. H) Para confirmar fracaso de tratamiento. Prueba de la tuberculina (PPD), con derivado de protena
puirificado, tiene un papellimitado en el diagnostico de TB activa. Reactor al PPD, persona que a las 72hrs presenta induracin
intradrmica de 10mm o mas en el sitio de la
aplicacin. En menores de 5 aos con o sin BCG,
recin nacidos, nios desnutridos y personas
inmunodeprimidas, la induracin de 5mm o mas.
Clnico; TB pulmonar activa son tos, a veces con
esputo que puede ser sanguinolento, dolor
torcico, debilidad, prdida de peso, fiebre y
sudoracin nocturna. RADIOLOGICO:
Linfadenopata; alteracin ms comn nios (90-
95%), adultos (40%), ms frecuentes unilaterales,
paratraqueal derecho e hiliar y su frecuencia
disminuye con edad. La combinacin de ganglios
hiliares calcificados y focos de Ghon, se conoce
como Complejo de Ranke (sugiere TB previa,
histoplasmosis). En infeccin primaria: se puede
observar focos de consolidacin generalmente en
lbulo medio o inferior, derrame pleural (60%),
ms unilateral, se presenta 3-7 meses despus de
la exposicin inicial. En reactivacin: afeccin del
segmento apical y posterior de lbulos superiores
(85%), la cavitacin secundaria a necrosis caseosa,
se observa como nivel hidroaereo; se pueden ver
atelectasias lobares. TAC: Los ganglios muestran
hipodensidad central (necrosis caseosa) y
reforzamiento perifrica, que representa el anillo
vascular de tejido granulomatosos inflamatorio que
indica enfermedad activa. En pacientes de alto riesgo, se pueden formar cavitaciones, diseminacin hematogena y tuberculosis miliar.
TUBERCULOSIS MILIAR: Se da por una infeccin diseminada por va hematogena. Afecta de 1-7%, frecuente en ancianos, lactantes e
inmunocomprometidos. Suele manifestase con fiebre, con frecuencia existen granulomas pulmonares, afeccion del SNC, en raros casos
sx. de dificultad respiratoria aguda con mortalidad de 90%. En casos crnicos caquexia, lesiones drmicas maculares o papulares y
tuberculos coroides (2-3mm), estos ltimos patognomnicos de tuberculosis miliar. RX: Mltiples imgenes nodulares <5mm
distribuidos en ambos pulmones difusas bilaterales, engrosamiento de septos interlobulillares. TB Y VIH: La inmunodepresin favorece
manifestaciones atpicas y extrapulmonares o inusuales de TB, al iniciar el tratamiento antirretroviral hay mayor riesgo de desarrollar
TB. Antes de indicar tratamiento, se debe realizar prueba de tuberculina. Las reactivaciones en estos grupos son 20 veces mayor. La TB
aumenta la replicacin viral y la progresin de la enfermedad VIH. La TB se presenta independiente de la cuenta de CD4. La TB miliar y
del SNC son ms frecuentes con cuentas de CD4 <200 cel/mm3.comun. Todos los pacientes VIH positivos, sin datos clnicos de TB,
deben recibir quimioprofilaxis con isoniacida.
MANUAL DE TRABAJO DEL CURSO ENARM CMN SIGLO XXI
CURSO ENARM CMN SIGLO XXI TEL: 36246001 Pharmed Solutions Institute PGINA 107

TRATAMIENTO: Debe de ser estrictamente
supervisado. Quimioprofilaxis, primario acortado y
retratamiento. Se considera TB multiresistente,
cuando no es susceptible a isoniacida, ni rifampicina
administradas simultneamente. Quimioprofilaxis: Por
6 meses a los contactos menores a 5 aos, con o sin
antecedente de BCG. Por 6 meses a contactos de 5-14
aos de edad, no vacunados BCG, en quienes se haya
descartado TB. Por 12 meses a contactos de 15 aos o
ms con inmunosupresin (previo descarte de TB
pulmonar o extrapulmonar). El frmaco a usar es la
isoniacida a dosis de 10mg/kg, sin exceder de 300mgs, en una toma diaria VO, estrictamente supervisada. El tratamiento acortado
estrictamente supervisado (TAES), se instituye a todo caso nuevo y al que lo reanuda posterior al primer abandono. TAES por 25
semanas, hasta completar 105 dosis. En 2 etapas: fase intensiva; 60 dosis, de lunes a sbado con HRZE (ver cuadro 1, medicamentos 1ra
lnea). Fase de sosten; 45 dosis ( ver cuadro 2, intermitente 3 veces a la semana con HR), con frmacos en combinacin fija y etambutol
separado
PREVENCION: Vacunacin; BCG, de bacilos mycobacterium bovis vivos atenuados (bacilo de Calmette y Guerin) inmunidad activa
contra la TB, dosis 0.1ml (contiene como mnimo 200 000UFC), intradrmica, brazo der., se aplica a todos los recin nacidos y hasta los
14 aos de edad cuando se considera necesario, disminuye incidencia de TB en SNC. Contraindicada, en recin nacidos con <2Kg,
personas inmunodeprimidas (excepto en infeccin por VIH en estado asintomtico), fiebre >38.5
NOTAS: La TB ganglionar es la forma ms de TB extrapulmonar. La TB vertebral (torcica) o Enf. de Pott es la manifestacin ms
frecuente de TB sea (50%), mas en varones.

CASO CLINIC
Se trata de paciente masculino de 46 aos de edad, originario de la regin selvtica de Chiapas, campesino, analfabeta el cual acude a
consulta debido a que se encuentra fatigado, lo cual le impide trabajar, agrega perdida de peso y fiebre frecuente, adems refiere que
tienes tos continua con esputo ocacionalmente con trazas de sangre, a la exploracin fsica, no se observa cicatriz de BCG, refiere
consumo de mezcal diario y tabaquismo 3 a 5 cigarrillos diarios, el ndice de masa corporal actual es de 20.5, signos vitales dentro de
parmetros normales, a la auscultacin torcica se apresian ruidos respiratorios anforicos posteriores de predominio superior derecho
con estertores aislados en la misma regin, no se observa cianosis ni hipocratismo, resto sin datos por agregar.

PREGUNTA
Cual es la conducta a seguir mas adecuada.

RESPUESTA
a.- Internamiento en aislamiento ambiental hasta 3 expectoraciones negativas.
b.- Internamiento sin aislamiento.
c.- Realizar biopsia e interconsulta a oncologa.
d.- Realizar prueba PPD.

TUBERCULOSIS
Se ingresa a paciente masculino de 45 aos de edad el cual cuenta con antecedente de ser portador de VIH positivo con cifras de CD4
de 85, actualmente suspendi el tratamiento antirretroviral, el motivo del ingreso es debido a dolor en la regin lumbar intenso el cual
mostro datos de ostemielitis, el paciente cuenta con el antecedente de presencia de tos productiva con esputo amarillento y
sanguinolento, perdida de peso importante, por lo que previamente se realizao baciloscopia siendo positiva para tuberculosis, apoyada
por los parmetro radiogrficos.

PREGUNTA
Cual es el tratamiento inicial mas adecuado.

RESPUESTA
a.- Isoniazida, rifampicina, etambutol y pirazinamida.
b.- Isoniazida, rifampicina, etambutol y pirazinamida mas terapia antirretroviral.
c.- Isoniazida, rifampicina, etambutol, pirazinamida y estreptomicina.
d.- Isoniazida, rifampicina y etambutol.

CASO CLINICO
Paciente de 56 aos, sexo femenino, con antecedente de dermatomiositis, en tratamiento con hidroxicloroquina 200 mg, metotrexato
10 mg y prednisona 30 mg diarios. Fue hospitalizada por cuadro de 10 das de evolucin, con fiebre, compromiso del estado general y
prdida de peso, encontrndose crepitaciones bilaterales al examen pulmonar. La TAC de trax mostr un infiltrado nodular bilateral,
extenso, con cavitaciones biapicales. La baciloscopia obtenida por lavado bronquio-alveolar (LBA) result positiva y la reaccin de
polimerasa en cadena (RPC) positiva para Mycobacterium tuberculosis. Adems lceras en los pulgares aparecidas cuatro semanas
previas al ingreso. Al examen fsico se observ una lcera de bordes bien delimitados con fondo sucio y bordes necrticos, no dolorosa,
ubicada en el pulpejo del dedo pulgar izquierdo. S. aureus y Candidaparapsilosis. La tincin de Ziehl Neelsen en tejido fue positiva.

PREGUNTA
CUADRO 2. Tratamiento primario acortado
FASE INTENSIVA Diario, de lunes a sbado, hasta completar 60 dosis o 2 meses.
Administracin en 1 toma
FARMACOS:
Rifampicina (R)
Isoniacida (H)
Pirazinamida (Z)
Etambutol (E)
Dosis:
600mgs
30mgs
1,500-2000 mg
1,200mgs
FASE DE SOSTEN Intermitente, 3 veces por semana, lunes, mircoles y viernes.
Hasta completar 45 dosis o 4 meses. Administrar en una toma
Frmacos:
Isoniacida (H)
Rifampicina (R)
Dosis:
800mgs
600mgs

MANUAL DE TRABAJO DEL CURSO ENARM CMN SIGLO XXI
CURSO ENARM CMN SIGLO XXI TEL: 36246001 Pharmed Solutions Institute PGINA 108

Cual es el origen mas probable que cause las ulceras en pulgares.

RESPUESTA
a.- Farmacologica.
b.- Bacteriana.
c.- Autoinmune.
d.- Mixta.

CASO CLINICO
Se trata de paciente femenino de 70 aos de edad la cual acude a consulta debido a que presenta desde hace varios meses, tos no
productiva, fatiga, prdida de peso, la paciente cuenta con antecedente de tuberculosis hace mas de 20 aos la cual fue tratada, se
logra obtener esputo para cultivo asi como sangre para cultivo, los reportes de laboratorio informan presencia de bacilos
acidorresistentes que indican complejo Mycobacterium avium, la radiografia de torax reporta bronquiectasias y ndulos pequeos
esparcidos a lo largo del parnquima pulmonar. La paciente no cuenta con otros antecedentes de importancia para el cuadro clnico.

PREGUNTA
Cul es el esquema de tratamiento ms adecuado.

RESPUESTA
a.- Claritromicina y etambutol.
b.- Claritromicina y rifampicina.
c.- Levofloxacino y rifampicina.
d.- Prizinamida, isonizida, rifampicina y etambutol.

PALUDISMO
CIENCIAS BSICAS: Enfermedad parasitaria febril aguda causada por protozoarios del genero plasmodium que se trasmite a los
humanos por la picadura de las hembras infectadas del los mosquitos del genero anopheles. Las especies del plasmodium causantes
son ovale, malaria, vivax y falciparum; estas dos ltimas la de mayor distribucin en el mundo, el ms mortal P. falciparum. Los
mosquitos se cran en agua dulce de poca profundidad y tienen preferencia por los humanos sobre los animales, su transmisin
depende de condiciones climticas, como el rgimen de lluvias, la temperatura y humedad. Enfermedad endmica en poblaciones con
clima tropical. SALUD PUBLICA: OMS; Aparecen aproximadamente de 300-500 millones de casos nuevos cada ao, de los cuales mas
de un milln resultan fatales. El 90% de las muertes atribuibles al paludismo ocurre en frica en nios menores de 5 aos. En Mxico el
principal agente etimolgico es P. vivax, algunos casos aislados por falciparum, en estados de la frontera sur. La transmisin en Mxico
se concentra en costas del Pacifico, Chiapas, Oaxaca, Chihuahua, Baja California y Yucatn. La tasa de mortalidad por paludismo se ha
reducido >25% desde el ao 2000 a nivel mundial. VIH/SIDA + paludismo=altos ndices de morbimortalidad en zonas de alta
prevalencia. PATOGENIA: Inicia con el vector (hembra anopheles) infectada, transfusiones, uso compartido de agujas hipodrmicas
contaminadas. El ciclo de vida de plasmodium se divide en asexual (2 etapas) que se realiza en el humano y sexual en el mosquito. Ciclo
asexual, etapa heptica: tras la picadura del mosquito este inocula el esporozoito existente en su saliva en la sangre del husped, a
travs del torrente sanguneo llegan a los hepatocitos, aqu se multiplican posteriormente se rompe el hepatocito, apareciendo un
nuevo estadio el merozoito, vuelven al torrente sanguneo donde penetran al eritrocito. Inicia etapa eritrocitica: comienza a
alimentarse de la hemoglobina, apareciendo entonces el trofozoito, nuevamente por esquizogenesis se multiplica en el interior de los
eritrocitos, a los cuales rompe liberando nuevos merozoitos y as continua el ciclo, pero algunos se convierten en gametocitos
masculinos y femeninos. Ciclo sexual: se da en la hembra anopheles, la cual adquiere el plasmodium mediante la picadura al humano
infectado con gametocitos en sangre. Estos gametocitos forman un cigoto en el intestino medio del insecto, el cigoto madura hasta
formarse un ovocineto, que migra a la glndula salival del insecto. Al destruirse los eritrocitos hay liberacin de sustancias del parasito
y de hemates a la circulacin sangunea. Los eritrocitos parasitados su membrana se vuelve rgida y se forman protuberancias que
favorecen su adherencia al endotelio vascular en capilares de rganos vitales, fenmeno conocido como "citoadherencia", sobre todo
en cerebro, la adherencia produce enlentecimiento del flujo sanguneo, aumento local de citocinas, glicolisis anaerobia y acidosis
lctica. DIAGNOSTICO: Clnica: Los sntomas aparecen a los 7 das (10-15 das), de la picadura del mosquito, lo primero es fiebre,
acompaada de cefalea, escalofros, debilidad, insomnio, artralgias, mialgias y vomito. Toda persona sospechosa de paludismo con
cuadro escalofros (dura de 15-30 min, pulso dbil y rpido, piel algo ciantica), fiebre (inicio brusco puede llegar hasta 41.5 grados,
puede delirios, convulsiones, taquicardia, hipotensin) y sudoracin, que resida o provenga de reas endmicas se le debern realizar
examen microscpico de gota gruesa de sangre y a los contactos de los casos. La enfermedad tiende hacia la cronicidad, en donde
existen periodos de recadas y latencia por aumento de la parasitemia. Todos los nios de zonas endmicas con enfermedad grave
suelen presentar anemia grave, esplenomegalia, sufrimiento respiratorio relacionado con la acidosis metablica o paludismo cerebral.
En adulto es comn afeccin multiorganica. Laboratorio: Realizar frotis de sangre, finas y de gota gruesa, identificar parsitos y
cuantificar los, el porcentaje de eritrocitos con parsitos dar la medida de la gravedad del la enfermedad. Tcnicas
inmunocromatograficas, detectan el Ag del parasito en sangre. Se puede usar PCR para detencin de estos. Pacientes que sufren
complicaciones o enfermedad severa sobre todo por P. falciparum, pueden presentar hiperparasitemia >100 000 parsitos/microl de
sangre con un >5-10% de eritrocitos parasitados, hemoglobina debajo de 7g/dl, ictericia. COMPLICACIONES: Malaria cerebral,
insuficiencia renal, fiebre biliosa hemoglobinurica, anemia severa, edema pulmonar, dao heptico, hemorragia, coagulopatia.
TRATAMIENTO: Actualmente se dispone de la combinacin de artesunato (4mg/kg) y mefloquina en una sola tableta para falciparum,
no complicado. Para P. vivax, el tratamiento de eleccin para cura radical es primaquina (0.25-0.5 mg/kg), es esquizonticida mas
cloroquina (25mg/kg), son tabletas de 150mgs, por 3 das. La OMS considera que el frmaco de eleccin para P. vivax es cloroquina, las
combinaciones de artesunato, se recomiendan en casos de P. vivax resistente a cloroquina. La quinina, es un ezquizonticida hemtico
muy eficaz, gametocida eficaz contra P. vivax y P. malarie, inactiva frente a P. falciparum. Artemisina por via parenteral accin rpida
MANUAL DE TRABAJO DEL CURSO ENARM CMN SIGLO XXI
CURSO ENARM CMN SIGLO XXI TEL: 36246001 Pharmed Solutions Institute PGINA 109

empleado para P. falciparum resistente a cloroquina, sus derivados son artesunato y artemer ambos en presentacin via oral y
parenteral. La ONU recomienda no emplear artemisina en monoterpia, porque acelera la resistencia, debe tratar e agregarse siempre
algn otro antipaldico. Si no se trata en las primeras 24 hrs el paludismo por P. falciparum, puede agravarse llevando a la muerte. En
caso de P. vivax y ovale, pueden producirse recidivas clnicas semanas o meses despus de la infeccin inicial, aunque el paciente haya
abandonado la zona paldica. Estos nuevos episodios se deben a formas hepticas "durmientes" del parasito (inexistentes en
falciparum, malariae) y para lograra curacin completa es obligatorio tratamiento dirigido a esas formas hepticas. PREVENCIN: La
lucha antivectorial es el medio principal de reducir la transmisin del paludismo. Algunas medidas son: mosquitos tratados con
insecticidas (peritroides), fumigacin de interiores con insecticidas, vigilancia continua y eliminacin del mismo.

CASO CLINICO
Masculino de 35 aos de edad residente del estado de mexico, inicia padecimiento 9 dias posteriores a su regreso de la ribera del rio
Niger en Mal donde permaneci 30 dias, cursando con un cuadro clnico caracterizado por fiebre de 40 C, de predominio vespertino y
nocturno terciada que se controlaba con paracetamol 500 mg cada 8 hrs. Al cuadro clnico se agrego estreimiento, disfagia, tos no
productiva, cefalea 8/10 holocraneana y perdida ponderal de 4 kg en una semana. EF. Temp.39.2 C FC 127, FR 24, TA 122/72 mmHg,
somnoliento, orientado en tres esferas, mucosas secas, ruidos cardiacos aumentados en frecuencia sin agregados, campos pulmonares
bien ventilados, abdomen distendido, doloroso a la palpacin media y profunda en ambos hipocondrios, timpnico a la percusin.

PREGUNTA
Cual es la conducta a seguir en el caso?

RESPUESTA
a.- Artesunato/mefloquina.
b.- Cloroquina.
c.- Cloroquina/mefloquina.
d.- Mefloquina.

CASO CLINICO
Ingresa paciente masculino de 48 aos de edad, ingeniero que acaba de regresar de convencin en medio oriente, presenta fiebre,
diaforesis, ataque al estado generalizado, fatiga y adinamia, as como obnubilacin, a la exploracin se observa hepato y
esplegnomegalia leve a moderada, es ingresado para realizar estudios encontrando paludismo por plasmodium falciparum, parasitemia
de 7 %, hematocrito de 21 %, bilirrubina de 7.8 mg/100 ml, creatinina 2.6 mg/100 ml. Se ingresa posteriormente a cuidados intensivos,
se realiza asistencia ventilatoria y se indica neuroproteccion debido a la gravedad del cuadro.

PREGUNTA
Cual es el mejor esquema de tratamiento para este caso:

RESPUESTA
a.- Quinina intravenosa.
b.- Quinidina Intravenosa.
c.- Artesunato Intravenoso.
d.- Cloroquina.

DENGUE:
CIENCIAS BASICAS: Enfermedad viral de carcter endemico-epidemico. Producido por un arbovirus (flaviviridae). Transmitido por Aedes
aegypti (vector), compuesto de ARN. El cual tiene 4serotipos (DE 1,2,3,4), el 2 es el serio tipo ms peligroso. Cada serio tipo proporciona
una inmunidad especfica para toda la vida, as como inmunidad cruzada a corto plazo. Dos clases de dengue el clsico y el hemorrgico.
SALUD PBLICA: Mxico ocupa el segundo lugar en casos y defunciones de Amrica, despus de Brasil, mayormente entre 10-19 Aos,
ligeramente ms en sexo femenino.OMS; Entre 50-100 millones por ao en el mundo, 500mil hospitalizados. 50mil muertes en ms de
100 pases y 2 mil millones de personas en riesgo. Mayor incidencia en pocas de lluvia. En Mxico hay condiciones que propician la
presencia de mosquitos, temperatura lluvias constantes, condiciones inadecuadas de saneamiento e higiene. PATOGENIA: EL virus
ingresa al organismo por la picadura del artrpodo se replica en los macrfagos y monocitos, lo que produce supresin de la medula
sea. El periodo de incubacin es de 5-7 das. Se desarrolla una respuesta inmune que acaba en el periodo de virema e induce
produccin duradera. Existen 3 etapas: 1. Febril; se da entre el 3-6to das, es variable, presencia de virus en sangre (virema). En esta
etapa no es posible reconocer si el paciente va a evolucionar a la curacin espontnea o si es el comienzo de un dengue grave. Al
disminuir la fiebre, el dolor abdominal se hace intenso y mantenido, se puede observar derrame plural o ascitis. 2. Crtico; momento de
mayor frecuencia de instalacin de choque. Hay extravasacin de plasma (vasodilatacin), su manifestacin ms grave el choque,
grandes hemorragias digestivas, alteraciones hepticas y quiz de otros rganos, ascitis o derrame pleural derecho o bilateral,
aumento de hematocrito (la mxima elevacin de este coincide con el choque) y disminucin de plaquetas. 3. Recuperacin: evidente
mejora del paciente a veces hay sobrecarga de liquido, as como coinfeccion bacteriana. Esta etapa es la de mayor riesgo de aparicin
de complicaciones, hay que vigilar y controlar al paciente 48hrs posteriores al cese de la fiebre, los signos de alarma son: dolor
abdominal intenso y continuo, vmitos frecuentes, somnolencia o irritabilidad, derrame seroso (en peritoneo, pleura o pericardio),
sangrado de mucosa, hepatomegalia (>2cm), aumento rpido de hematocrito y disminucin rpida de plaquetas. DIAGNOSTICO:
Dengue clsico: Inicio repentino, fiebre (de menos de 7 das) bradicardia, pulso lento, ex antena mculas punto forme, rinitis, tos, ardor
farngeo, adenopatas, fatiga, cambios en el sentido del gusto, anorexia, cefaleas, mialgias, artralgias, dolor seo y retroorbitario,
congestin conjuntival, edema palpebral, petequias, equimosis, miocarditis. Dengue hemorrgico: Mas grave, incremento en
permeabilidad vascular. El choque depende de mltiples factores como la presencia de anticuerpos no neutralizantes. Dengue grave;
MANUAL DE TRABAJO DEL CURSO ENARM CMN SIGLO XXI
CURSO ENARM CMN SIGLO XXI TEL: 36246001 Pharmed Solutions Institute PGINA 110

shock hipovolemico por fuga de plasma, diestres respiratorio por acumulacin de lquidos, sangrado grave, dao orgnico importante.
BH; leucopenia y trombocitopenia, a veces transaminasas elevadas. ELISA, para determinacin de anticuerpo IgM e IgG antidengue, se
puede tomar prueba pareada una en fase aguda y otra en convalecencia. PCR o ELISA se usan para deteccin de antgenos virales.
Anticuerpos IgM especficos para virus de dengue: Si es negativo se descarta el diagnostico de dengue; si es positivo es un caso
probable de dengue, se requiere una segunda muestra para confirmar mediante prueba de neutralizacin; negativo=se descarta caso
de dengue, constante=caso de dengue anterior, negativo=se confirma dengue. CASO PROBABLE DE FIEBRE HEMORRGICA POR
DENGUE: Toda persona que adems de un cuadro de probable fiebre por dengue desarrolle fiebre persistente y uno o ms de los
siguientes: A) Datos de fuga de plasma (ascitis, derrame pleural, edema, hipoalbuminemia). B) Datos de fragilidad capilar (petequias,
equimosis, hematomas). C) Hemorragias (gingivorragia, hematemesis, metrorragia). D) Tombocitopenia <100 000 plaquetas/ml3 o
hemoconcentracin. CLASIFICACIN: Del dengue hemorrgico: Grado I; fiebre y sntomas constitucionales no especficos, prueba del
torniquete positiva (nica manifestacin hemorrgica). Grado II; grado I ms petequias, epistaxis, hematemesis, melena. Grado III;
insuficiencia circulatoria (taquicardia, hipotensin, disminucin de la presin diferencial de pulso). Grado IV; choque, pulso y tensin
arterial no detectable. TRATAMIENTO: No especifico, solo de sostn. El dengue sin signos de alarma, ni comorbilidades, manejo
ambulatorio (reposo, aislamiento de mosquitos, lquidos, paracetamol, no aspirina, AINES, ni corticoides, evaluar mejora). Dengue con
comorbilidades, estricto ambulatorio o internacin en sala general (hidratacin va oral o IV con cristaloides a 2-3 ml/kg, mas medidas
anteriores, buscar signos de alarma hasta 48hrs de cese de fiebre, hemograma diario). Dengue con signos de alarma, sin criterios para
dengue grave, (obtener hematocrito antes de expandir al paciente, fisiolgica o Ringer 10ml/kg/hr, en una hora, repetir hto despus de
cada carga y evaluar signos de alarma, mejora reducir goteo, si empeora, aumenta hto y caen plaquetas, manejarlo como dengue
grave). Pacientes con signos de alarma y criterios para dengue grave (tomar hto, iniciar SF o Ringer lactato a 20ml/kg en 15-30 min,
evaluar si hay mejora disminuir a 10ml/kg, si mejora ir disminuyendo lquidos, de no mejorar continuar con coloides, no mejora
considerar uso de drogas vasoactivas, si paciente mejora y hto baja, indica sangrado y requiere transfundir, tx. De hemorragia segn
criterio) Soluciones intravenosas (para reponer prdidas producidas por extravasacin de plasma, a veces se agrava por perdidas del
exterior; sudoracin, vomito, diarrea) y requerimientos transfucionales. PREVENCIN: No se dispone de vacunas, ni antivirales
especficos. La nica forma para controlarlo, es mediante el control del mosquito del dengue y evitar su picadura. Ms de 50% de
criaderos de mosquitos se sita, en tiraderos de llantas y panteones.

CASO CLINICO
Femenino de 18 aos de edad originaria de Tepic, Nayarit camarera, ingreso por hipotensin e hipotermia, refiere que 48 hrs previas al
ingreso inicio con cuadro de cansancio, fiabre, dolor osteomuscular generalizado, dolor retro-ocular y mal estado general, Hg 14.6 Htc
44.7, plaquetas de 51,000/mm y leucocitos 15,200/mm se enviaron muestras para serologa, dos horas despus la Hb 13.7, Htc 43%,
plaquetas de 31,000 a las 5 horas la tensin arterial era de 80/50, FC 100, FR 24, FC 100, temperatura de 35 C.

RESPUESTA
a.- Cristaloides y aminas.
b.- Solucion coloides, aminas vasoactivas.
c.- Aminas vasoactivas, hemoderivados.
d.- Cristaloides, aminas y hemoderivados.

CASO CLINICO
Masculino de 38 aos de edad el cual inicio 5 dias previo a su ingreso con debilidad, hiporexia, fiebre, nausea y vomito, el paciente
presentaba dolor torcico, TA 40/30 mmHg, FC 120, FC 24, Temp 35. La piel marmrea, mucosas secas, Rx de toras con datos de
derrame pleural, torax hipoventilados, dolor epigstrico, Hb 12.8, Htc 51 %, plaquetas 79,000, leucocitos 10,500.

PREGUNTA
Que criterios de gravedad de dengue grave presenta?

RESPUESTA
a.- 1.
b.- 2.
c.- 3.
d.- 4.

CASO CLINICO
Se trata de una mujer de raza negra, de 21 aos de edad, que present un cuadro febril agudo de 39-40 C, seguido en los das
sucesivos de una erupcin cutnea pruriginosa generalizada. Tras buscar asistencia en un centro peditrico de su regin, se diagnostic
clnica y serolgicamente de dengue clsico y se trat sintomticamente con paracetamol. Una semana despus, una vez que los
sntomas de la fase aguda se encontraban en una fase casi completa de regresin, comenz a sentir dificultad para articular el lenguaje
y para deambular, disfagia, debilidad muscular progresiva en los cuatro miembros y disminucin del nivel de conciencia.

PREGUNTA
Cual es la conducta diagnostica mas apropiada para identificar la causa de las complicaciones.

RESPUESTA
a.- IRM.
b.- TAC.
c.- Pruebas de funcionamiento heptico.
MANUAL DE TRABAJO DEL CURSO ENARM CMN SIGLO XXI
CURSO ENARM CMN SIGLO XXI TEL: 36246001 Pharmed Solutions Institute PGINA 111

d.- Biometia hemtica.

CASO CLINICO
Se trata de una mujer de 50 aos fumadora ocasional. Entre sus antecedentes personales de inters, destacaba la realizacin de un
viaje turstico de nueve das de duracin a la Rivera Maya, quince das antes del inicio de los sntomas. Durante el viaje la paciente,
sufri numerosas picaduras de mosquitos. En las 48 horas previas a la consulta, la paciente se autotrat con cido acetilsaliclico (AAS)
como antipirtico y como tratamiento sintomtico de la cefalea. En la exploracin fsica la paciente estaba consciente, orientada y
febricular (37,6C), TA=110/70. Exista rubor facial e inyeccin conjuntival bilateral as como la presencia de un exantema hemorrgico,
simtrico, mculo-papular en tronco y extremidades, especialmente en las inferiores, donde era confluente, y que no se modificaba a la
vitropresin

PREGUNTA
Cual es la conducta mas adecuada para establecer el diagnostico.

RESPUESTA
a.- Demostracin o prueba de NS1+.
b.- Elevacin cuatro veces o ms de ttulos de IgG o IgM contra algn antgeno de virus de dengue en muestras pareadas de suero.
c.- Presencia de antgeno circulante en sangre del virus de dengue determinada mediante ELISA, o de antgeno en hgado por
inmunohistoqumica (inmunoperoxidasa) o inmunofluorescencia.
d.- Deteccin de secuencias genmicas de virus de dengue mediante prueba de PCR.

ENFERMEDAD DE CHAGAS:
CIENCIAS BSICAS: Chagas o trypanosomosis americana, es una infeccin sistmica causada por el protozoario flagelado Trypanozoma
cruzi. Su distribucin en las zonas ms marginadas del continente americano y su curso crnico, degenerativo y muerte repentina,
hacen de esta enfermedad una gran carga para la economa y la salud. Es una zoonosis en la que participan un gran nmero de
reservoros vertebrados (perros, ratas, gatos, zarigueyas) y transmisores triatominos (chinche). SALUD PUBLICA: Elevada prevalencia e
incurabilidad. Se estima que en la regin de las Amricas, se presenta en 21 pases, afecta a unos 7-8 millones de personas, y se
encuentra en riesgo de adquirir la infeccin aprox. 7-8 millones de personas, con 56,000 nuevos casos anuales y 12,000 muertes por
ao. En Mxico es endmica en el estado de Veracruz. PATOGENIA: La infeccin se transmite principalmente por la "chinche besucona"
otros modos de transmisin son; transfucional, congnito, trasplante de rganos y oral (alimentos contaminados con heces del
artrpodo). La chinche durante la picadura defeca en la piel del hospedero, despus de ingerir los parsitos (en su forma de
tripomastigoto til para dx.) en la sangre de un ser humano, estos se transforman en tripomastigotes metaciclicos que son la forma
infectante, estos son excretados en las heces del parasito, que al ponerse en contacto con la conjuntiva o laceracin en la piel, con el
rascado se facilita la entrada del patgeno al torrente circulatorio. Las primeras clulas invadidas por los tripomastigotes son los
histiocitos, donde se transforman en amastigotos (intracelulares, replicativo) y desarrollan varios ciclos de fisin binaria. Casi cualquier
clula puede ser invadida, a excepcin de las neuronas. La infeccin persiste en el cuerpo humano de por vida. DIAGNOSTICO: Espectro
clnico en 3 fases: Fase aguda (70% se encuentran en esta fase); la incubacion de 14 dias y dura de 2-4 meses, asintomtica, se
caracteriza por alta parasitema e invasin tisular multiparenquimatosa, puede presentarse el llamado "chagoma de inoculacin",
ndulo subcutneo con adenitis regional en el sitio de la picadura, cuando el sitio de entrada es la conjuntiva se conoce como signo de
Romaa (edema bipalpebral unilateral, conjuntivitis y linfadenitis preauricular), esta fase se manifiesta con fiebre, linfadenopatas,
hepatoesplenomegalia y mal estado general, se pueden presentar complicaciones como miocarditis aguda, meningoencefalitis, 5% de
nios fallece en esta etapa. Fase indeterminada; duracin variable sin parasitemia detectable, se han reportado anomalas anatmicas y
funcionales, serologa positiva. Fase crnica; hay compromiso visceral irreversible: cardiomiopata chagasica (mocardiopata dilatada,
arritmias severas, bloqueos), o de tubo digestivo, con la mayor frecuencia en intestino o esfago. Las manifestaciones incluyen disnea
de grandes a pequeos esfuerzos, palpitaciones, edema en miembros inferiores, dolor torcico, cuadros sincopales. La destruccin de
clulas ganglionares de la submucosa de tracto digestivo, dan lugar aperistalsia, retencin de residuos y dilatacin de rganos
(megaesofago, megacolon), puede haber disfagia, pirosis, dispepsia. Laboratorio: en etapa aguda bsqueda de Trypanozoma cruzi en
sangre, por examen directo y tincin de extendidos de sangre. En etapa crnica, la parasitemia es transitoria, por ello diagnostico
mediante el hallazgo de anticuerpos circulantes de T. cruzi, con hemaglutinacin directa, ELISA, inmundo fluorescencia, adems
contemplar microscopia de gota gruesa o extendida, PCR, xenodiagnostico indirecto y hemocultivo. Estudios para funcin cardiaca;
ECG, ecocardiografia, Holter, cateterismo, biopsia. TRATAMIENTO: Benzonidazol; 5-7.5 mg/kg/da en fase aguda, casos congnitos y
pacientes en etapa crnica. Nirfurtimox; 8-10mg/kg/dia 60 a 90 dosis, en fase aguda y crnica. Dichos medicamentos, sin embargo no
logran erradicar el parasito, ni previenen la progsion de la enfermedad una vez que esta se ha establecido. En la mocardiopata, se
utilizan digitalicos, antiarritmicos, diurticos, marcapasos, trasplante cardiaco. PREVENCIN: La mejor solucin a largo plazo, consiste
en la eliminacin de los transmisores mediante el mejoramiento de viviendas, cuya construccin a base de adobe permite la aparicin
de grietas que albergan a los insectos, empleo de insecticidas (piretroides), medidas de educacin en salud. Tamizare obligatorio en
bancos de sangre y a mujeres embarazadas en regiones endmicas.

CASO CLINICO
Masculino de 42 aos de edad, agricultor. Motivo de consulta por presentar hace mas menos 15dias dolor de barriga con sensacin
de plenitud abdominal, estreimiento, dolor a la deglusion, edema de miembros plvicos, casi todos los dias faltade aire se siente
cansado con dolor en el pecho que aparece a pequeos esfuerzos aunque no es de mucha intensidad y desaparece con el reposo. Por lo
cual se decide su ingreso paramejor estudio. Refiere que hace mucho tiempo lo pico un insecto y tuvo inflamacin en un ojo pero no
sabe con exactitud que insecto fue ni la fecha. Condiciones de la vivienda: pesima. Casa con piso de tierra agua de pozo y letrina a
distancia. EF: disnea, cardiovascular, dolor precordial simple ligero a moderado que disminuye con el reposo, edema en miembros
inferiores, astenia, dolor en toda la regin abdominal, colicos, disfagia, constipacin, disuria, cefalea, anorexia.
MANUAL DE TRABAJO DEL CURSO ENARM CMN SIGLO XXI
CURSO ENARM CMN SIGLO XXI TEL: 36246001 Pharmed Solutions Institute PGINA 112


PREGUNTA
Cual es el pronostico del caso?

RESPUESTA
a.- Malo para la funcin y malo para la vida
b.- Malo para la funcin y bueno para la vida.
c.- Bueno para la funcin y bueno para la vida.
d.- Bueno para la funcin y malo para la vida.

LEPRA (MYCOBACTERIUM LEPRAE):
CIENCIAS BASICAS: Enfermedad granulomatosa multisistmica crnica causada por Mycobacterium leprae parasito intracelular
obligado, no mvil, no espolurado, que afecta hgado, piel, mucosas de las vas respiratorias altas, testculos (disfuncin sexual, atrofia),
ojos y nervios perifricos. SALUD PBLICA: Actualmente existen zonas de alta incidencia en frica, Asia y Amrica Latina. En Mxico 21
estados presentan incidencia, los de mayor son Sinaloa, Michoacn, Nuevo Len, Guerrero y Nayarit. La razn hombre: mujer 2:1, ms
comn en >25 aos. PATOGENIA: El modo de transmisin e invasin sigue siendo controvertido. Es de notar que pacientes con lepra-
lepromatosa la mayor diseminacin del bacilo al ambiente se realiza a travs de la descarga nasal, el estornudo y la tos; por ello se
plantea que la principal ruta de infeccin es la via area (microgotas), la diseminacin a travs de la piel es menos importante.
Incubacin de 2-4 aos o ms. Para adquirir la infeccin se requiere la convivencia estrecha y prolongada con un enfermo bacilifero y
con inmunidad celular disminuida identificada. DIAGNOSTICO: Clnica; En piel maculas, ppulas, ndulos, placas o infiltraciones,
afectando especialmente macrfagos. La hipopigmentacin o eritema de la piel con dficit sensorial (hiperestesia, hipoestesia o
anestesia), es uno de los signos clnicos ms importantes en el diagnostico, se observan en la palma de la mano o planta del pie, y los
origina una infeccin de la bacteria en las fibras nerviosas (engrosamiento de troncos nerviosos perifricos), alteraciones motoras. El
dao al sistema nervioso es una caracterstica, a pesar de no tener habilidad locomotora, puede moverse en el endotelio a travs del
tejido conectivo y alcanzar las clulas de Schwan. Las pruebas inmunolgicas son muy limitadas, no existe un examen confirmatorio,
baciloscopia positiva con resultado completo, deteccin de anticuerpos anti-PGL-I, donde se presentan grandes cantidades de IgM.
ELISA, pero tienen sensibilida y especificidad baja. El diagnostico histopatolgico es obligatorio para el pronstico y de esta manera
favorecer el tratamiento, se pueden obsrevar infiltrados en la dermis, hipodermis y rganos internos tales como clulas de Virchow,
que son macrfagos con muchos bacilos y gotas de lpidos en su citoplasma, con apariencia espumosa. En secreciones nasales y
cutneas el hallazgo del bacilo aislado en forma de globia puede ser detectado cpon coloracin de Zieehl Neelsen. CLASIFICACION: Para
fines de control sanitario los casos se clasifican en: Multibacilares (MB); los lepromatosos, dimorfos y con infiltracin difusa .
Paucibacilares (PB); los tuberculoides e indeterminados. Clasificacin clnica:
Caractersticas 5 LEPROMATOSA
(progresa a necrosis y ulceracin)
3 BODERLINE
LEPROMATOSA
4 BODERLINE
BODERLINE (dimorfa)
2 BODERLINE
TUBERCULOIDE
1 TUBERCULOIDE
Tipo de lesin Maculas, ppulas, ndulos,
infiltracin difusa
Maculas, ppulas,
ndulos, infiltracin
Placas y lesiones en forma
de cpula y sacabocados
Placas de infiltrados Placas de infiltrados
Numero de
lesiones
Numerosas Muchas Muchas nica, usualmente con
lesiones satlites o >5
lesiones
<5 lesiones
Distribucin de las
lesiones
Simtricas (afecta ms labios,
encas, dorso de lengua, paladar
duro)
Con tendencia a la
simetra
Evidentemente
Asimtricas
No difusas y asimtricas Asimtricas
Definicin de las
lesiones
Imprecisas, difcil de definir la
enfermedad y la afeccin de la piel
Imprecisas delimitadas
por los bordes externos
Imprecisas delimitadas por
los bordes externos mal
definido
Bien definidas, bordes bien
delimitados, eritematosas o
cobrizas
Bien definidas bordes
bien delimitados
Sensibilidad No afectada Disminuida Disminuida Ausente Ausente
Bacilos en las
lesiones en la piel
Muchas globias Muchos Muchos +/- Negativa
TRATAMIENTO: Los esfuerzos globales para controlar la lepra mediante la poliquimioterapia han permitido disminuir significativamente
el nmero de pacientes infectados. Sin embargo la deteccin de casos nuevos y las estrategias de control no han logrado la eficacia
esperada. Casos multibacilares usar dosis mensual supervisada de rifampicina, clofazimina, dapsona y dosis diaria autoadmnistrada de
dapsona y clofazimina. Para casos paucibacilares usar en dosis mensual supervisada (como mnimo 6 dosis mensuales) rifampicina y
dapsona y dosis diaria autoadministrada (mnimo 162 dosis) de dapsona. Usar hasta desaparicin de lesiones.

CASO CLINICO
Masculino de 54 aos de edad, constructor, nacido en Tamaulipas y radicado en Veracruz. Presenta dermatosis diseminada
caracterizada por ndulos de diferentes tamaos desde 3 x 3 hasta 8 x 8 mm, algunos de coloracin de la piel y otros eritematosos en
tronco y extremidades superiores e inferiores de 8 meses de evolucin.

PREGUNTA
Que tipo es?

RESPUESTA
a.- Lepromatosa.
b.- Borderline lepromatosa.
c.- Borderline tuberculoide.
d.- Tuberculoide.



MANUAL DE TRABAJO DEL CURSO ENARM CMN SIGLO XXI
CURSO ENARM CMN SIGLO XXI TEL: 36246001 Pharmed Solutions Institute PGINA 113

CASO CLINICO
Masculino de 59 aos de edad originario de boca de rio, acude por presencia de dermatosis diseminada con ndulos de 5 a 8 mm en
cara, tronco, miembros torcicos y plvicos de un ao de evolucin, con manchas residuales hipercrmicas, con eritema mnimo en
ambas extremidades plvicas. Se detect alopecia en tercio distal de ambas cejas y madarosis bilateral. La investigacin seriada de
bacilos alcohol cido resistentes en linfa y moco nasal fueron positivas. Se agregaban descamacin plantar y onicodistrofia de dedos
gordos de ambos pies. El paciente cursaba con reaccin leprosa tipo II adems de calosfros, artralgias maleolares y epistaxis de
repeticin. La exploracin fsica detect alteraciones de sensibilidad y motoras en dedos de ambas manos e insuficiencia venosa en
ambos miembros plvicos.

PREGUNTA
Que tipo es?

RESPUESTA
a.- Lepromatosa.
b.- Borderline lepromatosa.
c.- Borderline tuberculoide.
d.- Tuberculoide.

TRIPANOSOMIASIS AFRICANA (TRYPANOSOMA BRUCEI):
CIENCIAS BASICAS: Tambin llamada enfermedad del sueo, es una parasitosis transmitida por un vector. ocurre en frica es
transmitida a los seres humanos por la picadura de la mosca tsetse, que ha contrado la infeccin de personas o animales que albergan
los parasitos patgenos para el ser humano. Las personas mas expuestas personas que se dedican a la agricultura, la pesca, la ganadera
o la caza. Trypanosoma brucei gambiense causa 95% de los casos de esta enfermedad y es una infeccin crnica, generalmente
asintomtica, los sntomas aparecen cuando la enfermedad ya esta muy avanzada, en la etapa de afeccin al SNC. Trypanosoma brucei
rhodesiense causa 5% y causa una infeccin aguda, tambin afecta SNC. SALUD PBLICA: en los ltimos 10 aos, mas del 70% de los
casos notificados ocurrieron en la Repblica Democrtica del Congo, en 2010 esta reporto mas de 500 casos nuevos por ao.
PATOGENIA: Transmisin principal por picadura de mosca, tambin atraves de la placenta, con agujas contaminadas. En la primer
etapa, los tripanosomas se multiplican en los tejidos subcutneos, la sangre ya la linfa; se conoce como etapa hemolinfatica y se
caracteriza por episodios de fiebre, cefalea, dolores articulares y prurito. eN la segunda etapa, los parasitos atraviesan la barrera
hematoencefalica e infectan el SNC; es la etapa de afeccin nerviosa, aqu se presentan las manifestaciones mas evidentes como :
cambios en el comportamiento, confusin, trastornos sensoriales y falta de coordinacin. Los trastornos en el ciclo del sueo que le dan
el nombre a la enfermedad, son una caracterstica importante, si no se da tratamiento es mortal. DIAGNOSTICO: Tamizaje de una
posible infeccin (pruebas serologicas y exploracin Clinica), diagnostico de la presencia del parasito, determinacin de la etapa en la
que se encuentra la afeccin (puncin lumbar y estudio del LCR). TRATAMIENTO: Depende de la etapa, cuanto mas pronto se inicie,
mejores las probabilidades de curacin. En la primer etapa: pentamidina y suramina. Tratamiento en la segunda etapa deben ser
medicamentos que atraviesen la barrera hematoencefalica como: melarsoprol, eflornitina, nifurtimox.

CASO CLINICO
Femenino que ingresa por astenia, somnolencia de varios das de evolucin, sin determinar tiempo, acompaada de fiebre de 38.5 C,
las mucosas estn hipocoloreadas, sin otros sntomas y signos acompaantes que ligeras adenopatas cervicales bilaterales, una punta
de bazo y discreta rigidez de nuca. Al da siguiente de su ingreso se constata fiebre de 39 y 40 C, est aptica, astnica, deja de comer y
de hablar pasa a un estado estuporoso y en horas de la madrugada tiene convulsiones tnico-clnicos generalizadas, se observan
temblores que se repetan igual que las convulsiones, a pesar de las medidas antitrmicas y anticonvulsinantes; se muestra
quejumbrosa en oportunidades, posteriormente se comporta muy intranquila y con gran agitacin psicomotora; cae en una fase ms
profunda del coma, no responde sino a estmulos profundos, tiene rigidez de descerebracin. Se constata Kerning positivo y marcada
rigidez nucal.

PREGUNTA
De acuerdo a la etapa cual es el tratamiento de eleccin?

RESPUESTA
a.- Nifurtimox.
d.- Pentamidina.
c.- Suramina.
d.- Prazicuantel.

ONCOCERCOSIS (ONCHOCERCA VOLVULUS):
CIENCIAS BSICAS: Parasitosis del hombre causada por onchocerca volvulus, que afecta piel, ojos llegando a producir ceguera,
transmitida por insectos hematofagos del genero Simulium. SALUD PUBLICA: En regiones subtropicales del frica occidental y en
reducidas reas del Yemen y Amrica Central. En Mxico se encuentran 3 focos endmicos en Oaxaca y norte de Chiapas. PATOGENIA:
La O. volvulus es una filaria, la hembra vivpara, libera embriones mviles y activos (microfilarias, atraviesan los ndulos y llegan a los
tejidos dermicos), en vez de huevos, algunas son ingeridas por una hembra de simulido, la mayora muere y es eliminada por
fagocitosis. Los adultos viven en ndulos fibrosos u oncocercomas subcutneos, aunque algunos se han localizado adheridos al
periostio, oquedades seas. En Mxico, los ndulos se ubican fundamentalmente en la cabeza y tronco, mientras que en frica es ms
comn en cintura plvica. La oncocercosis cutnea resulta de mecanismos originados por el desplazamiento de las microfilarias y de las
secuelas resultantes de reacciones inflamatorias acumuladas. DIAGNOSTICO: Clnica; las primeras manifestaciones son cutneas
MANUAL DE TRABAJO DEL CURSO ENARM CMN SIGLO XXI
CURSO ENARM CMN SIGLO XXI TEL: 36246001 Pharmed Solutions Institute PGINA 114

irritacin, prurito, edema e hipertermia localizadas, la piel se engrosa y hay erupciones papulares y ligeros cambios en la pigmentacin.
El prurito se intensifica y el rascado causa excoriaciones que se infectan secundariamente, zonas de hiperpigmentacion y
despigmentacion puede presentarse liquenificacion (epidermis engrosada, formas nodulares y descamacin). La migracin continua y
prolongada de las microfilarias, junto con la respuesta inflamatoria origina la perdida de elasticidad cutnea y explica la fascies leonina.
Al engrosamiento de la piel mas perdida de elasticidad se debe la paquidermitis. En ojo podemos encontrar las microfilarias en humor
vtreo y acuoso, afectan regiones anterior y posterior de ambos ojos, en cornea se presenta queratitis punteada (corta evolucin)
despus queratitis esclerosante que opacas permanentemente la cornea, la visin perifrica se ve reducida ya la funcin visual limitada.
Tambin se produce uveitis, atrofia del nervio ptico y alteraciones en retina, que causan ceguera irreversible. Epidemiologa: individuo
con residencia permanente o antecedentes de visita en rea endmica. Parsitologico; confirma impresin clinica y se realiza mediante
la observacin de microfilarias que emergen de biopsias cutneas o directamente por la observacin de microfilarias en la cmara
anterior y posterior del ojo con lampara de hendidura. TRATAMIENTO: Para eliminar a los adultos de O. volvulus, la extirpacin de los
ndulos subcutneos palpables, continua siendo el procedimiento de eleccin. Actualmente se utiliza la ivermectina, eficaz contra las
microfilarias, en piel y ojo y el efecto perdura 8 meses, en una sola dosis (0.15mg/kg de peso). En algunos pases se ha utilizado un
esquema mixto, con doxiciclina.

CASO CLINICO
Se trata de una paciente de 24 aos de edad, que acudi por presentar dolor y fotofobia en OD. En la exploracin, se apreci la
existencia de una uvetis anterior, por lo que se comenz un tratamiento con midriticos y antiinflamatorios. A los pocos das acude a
urgencias de Oftalmologa, donde se ve una disminucin del cuadro uvetico anterior, pero, se descubre la existencia de una vitritis con
un foco de coriorretinitis parapapilar inferior, que afecta a la papila, con exudados algodonosos y hemorragias, as como perivasculitis
zonal, observada tambin en la AFG; el fondo de ojo del Ol era normal. Ante este cuadro, la paciente ingresa con la sospecha de
coriorretinitis por citomegalovirus por lo que se inicia tratamiento sistmico con ganciclovir IV. Sin antecedentes patolgicos de inters,
haba recibido tratamiento para una infeccin urinaria haca un mes y tena un antecedente de anexitis, ya curada. En el estudio
analtico inicial apareca una eosinofilia mayor de 1.000/mm3.

PREGUNTA
Cual es la conducta a seguir mas adecuada?.

RESPUESTA
a.- Ascultacion con lmpara de hendidura.
b.- Frotis sanguneo.
c.- Verificar los valores de eosinofilia.
d.- Iniciar ivermectina.

CASO CLINICO
Paciente masculino de 62 aos, natural y procedente de oaxaca, de oficio agricultor, quien consult por presentar tumoraciones
drmicas en miembros superiores e inferiores, prurito generalizado, cefalea, artralgias y disminucin de la agudeza visual, con
presencia de ndulo subcutneo, Bh eosinofilia importante.

PREGUNTA
Cual de los siguientes mtodos diagnosticos es mas adecuado para establecer el diagnostico?

RESPUESTA
a.- Verificar por lmpara de hendidura.
b.- Biopsia del nodulo.
c.- Frotis de sangre.
d.- Eosinofilia persistente.

LEISHMANIASIS (LEISHMANIA ssp):
CIENCIAS BASICAS: Enfermedad tropical, casada por el protozoo leishmania, intracelular obligado del humano (localizado en
macrfagos y clulas dendrticas), produce lesiones a nivel cutneo, mucocutaneo y visceral. Los insectos transmisores son dpteros del
genero Lutzomyia el cual adquiere la infeccin del humano y reservorios como roedores, canidos y primates. Tambin se puede adquirir
por contacto con material de una lesin, trasplante de rganos, transfusin y a travs de placenta. Enfermedad emergente oportunista,
asociada a SIDA. SALUD PBLICA: Se estima que 12 millones de personas se encuentran infectadas, con unos 1-2 millones de casos
nuevos al ao. La falta de sistemas de vigilancia y el subdiagnostico, hacen difcil estimar la real incidencia y el ndice de letalidad. Ms
de 90% de casos de leishmaniasis visceral se presentan en Bangladesh, Sudn, Sudn del Sur, Etiopia y Brasil. En Mxico en Chiapas,
Oaxaca, Tabasco, Veracruz, Puebla y Guerrero. PATOGENIA: Periodo de incubacin de 1-12 semanas. El promastigote metacclico
extracelular (forma infectante), es introducido en la piel del hospedero a travs del piquete de la mosca hembra. Los parsitos son
fagocitados en la piel por macrfagos y clulas de Langerhans y activan el complemento. Aunque muchos promastigotes son destruidos
por los leucocitos PMN, unos pocos se transforman en amatigotes que es intracelular (forma replicativa), se multiplica por divisin
binaria, en los fagolisosomas dentro de fagocitos mononucleares de los hospederos. Las clulas infectadas se rompen y los amastigotes
se diseminan a los tejidos, induciendo a produccin de citocinas proinflamatorias. La leishmaniasis cutnea difusa se presenta como
consecuencia de factores inmunes del hospedero, asociado a ciertas especies del parasito. La forma visceral (kala-azar)es la ms
severa, causada por L. donovani. DIAGNOSTICO: Clnica; Se consideran 2 cuadros clnicos cutneos: la leishmaniosis cutnea localizada
(LCL), generalmente circunscrita al sitio de inoculacin, gracias a la respuesta inmune celular protectora, en Mxico conocida como
ulcera de los chicleros (redondeada de borde elevado y bien definido, indurado, cubierta por una costra amarillenta, cuan do esta se
MANUAL DE TRABAJO DEL CURSO ENARM CMN SIGLO XXI
CURSO ENARM CMN SIGLO XXI TEL: 36246001 Pharmed Solutions Institute PGINA 115

desprende, fondo de tejido de granulacin limpio), con frecuencia afectando el pabelln auricular y la leishmaniasis cutnea
diseminada (LCD), caracterizada por pobre respuesta inmune y diseminacin no controlada en piel, caracterizada por lesiones
nodulares con gran numero de parsitos, diseminadas en toda la piel excepto cuero cabelludo, regin inguinal y axilar, genitales,
plantas y palmas. Fiebre, palidez, anorexia, prdida de peso, tos, vomito, diarrea hepatoesplenomegalia, linfadenopatas, sangrado
gingival, epistaxis, equimosis, anemia; posteriormente taquicardia, ictericia, distencin abdominal, ascitis, edema, sangrados
importantes, hiperpigmentacion, lesiones verrucosas no ulceradas y alopecia. Laboratorio: trombocitopenia, anemia normocitica
normocrmica, leucopenia. Hipoalbuminemia, hipergammaglobulinemia. Biopsia de bazo e hgado (amastigotes). Raspado, biopsia,
improntas y extendido de lesiones y ganglios linfticos. La intradermorreaccin de Montenegro, prueba de hipersensibilidad celular a
antgenos de Leishmania, es un mtodo indirecto. TRATAMIENTO: En casos graves la mortalidad sin tratamiento es de 100%. La
primera lnea esta constituida por los antomoniales pentavalentes: antimoniato de meglumina (glucantime) y estibogluconato de sodio
(pentostam). Otros frmacos sistmicos utilizados son: anfotericina B, paromomicina, sitamaquina.

CASO CLINICO
Mujer de 70 aos que consulta por la aparicin de una ppula en la frente, algo pruriginosa, que sigue creciendo y a la que se van
aadiendo lesiones similares en su periferia, pese al tratamiento emprico con corticoides, sin sospecha diagnstica. La lesin se
considero como placa eritematopapulosa, la biopsia de la lesin demuestra una dermatitis granulomatosa no necrotizante. La reaccin
en cadena de la polimerasa (PCR) diagnostica una leishmaniasis cutnea (LC).

PREGUNTA
Cual es el tratamiento mas adecuado para el caso?

RESPUESTA
a.- Antimoniato de meglumina.
b.- Anfotericina B.
c.- Paromomicina.
d.- Sitamaquina.

ESQUISTOSOMIASIS (SCHISTOSOMA):
CIENCIAS BASICAS: Enfermedad parasitaria crnica causada por duelas sanguneas (trematodos) del genero Schistosoma. Corren el
riesgo de contraer la infeccin las personas que realizan actividades agrcolas, domesticas o recreativas que las exponen a aguas
infestadas. La falta de higiene y las actividades ldicas hacen que los nios sean especialmente vulnerables a la infeccin. SALUD
PBLICA: Es prevalente en las regiones tropicales y subtropicales, en especial en comunidades sin acceso a agua potable. El nmero de
personas tratadas aumento de 12,4 millones en 2006 a 33,5 millones en 2010. Al menos 230 millones de personas necesitan
tratamiento cada ao. PATOGENIA: Las personas se infectan cuando las formas larvarias del parasito, liberadas por caracoles de agua
dulce (son el reservorio, donde maduran los huevos), penetran en la piel durante el contacto con aguas infestadas. En el interior del
organismo, las larvas se convierten en esquistosomas adultos, que viven en los vasos sanguneos, emigran a las venas abdominales
donde se instalaran definitivamente y se reproducirn es aqui donde las hembras ponen sus huevos. Algunos de esos huevos salen del
organismo con las heces o la orina y continan el ciclo vital del parasito. Otros quedan atrapados en los tejidos corporales, donde
causan una reaccin inmunitaria y un dao progresivo de los rganos. Hay dos formas principales: Esquistosomiasis intestinal
(Schistosoma mansoni, japonicum, mekongi) y esquistosomiasis urogenital (Schistosoma haematobium). DIAGNOSTICO: Clnico; Los
sntomas son causados por la reaccin del organismo al huevo y no por el gusano en si mismo. En la forma intestinal, dolor abdominal,
diarrea y sangre en las heces, en casos avanzados, hepatoesplenomegalia, ascitis e hipertensin portal. El signo clsico de la forma
urogenital es la hematuria, puede haber disuria, poliuria, en casos avanzados son frecuentes la fibrosis de la vejiga y los urteres, as
como las lesiones renales. El cncer de vejiga es otra posible complicacin tarda. En las mujeres puede haber lesiones genitales,
hemorragias vaginales, dispareuna y ndulos vulvares. En el hombre trastornos de las vesculas seminales y prstata, en ambos puede
llevar a la infertilidad. En nios puede causar anemia, retraso de crecimiento y problemas de aprendizaje. Laboratorio: Deteccin de
huevos del parasito en muestras de heces (frotis en glicerina con azul de metileno) u orina, en la forma urogenital puede haber
microhematuria. TRATAMIENTO: El prazicuantel es el nico tratamiento disponible contra todas las formas de esquistosomiasis.

CASO CLINICOS
Nio de ocho aos procedente de guerrero, que acude para estudio de hematuria macroscpica al final de la miccin y proteinuria de
ms de un ao de evolucin. Refiere baos anteriores frecuentes en ro. No tiene antecedentes personales ni familiares a destacar. La
exploracin fsica fue normal. Los anlisis de laboratorio muestran un hemograma con anemia microctica, con Hb 10,9 g/dL, Hto 32,8%,
VCM 74,9 fL y eosinofilia con 767 eosinfilos/L; VSG 40 mm/h. La bioqumica muestra urea y creatinina e iones normales. Estudio
inmunolgico con niveles de inmunoglobulinas y complemento dentro de la normalidad. El sistemtico de orina muestra hematuria
microscpica y presencia de cristales de oxalato clcico. La ecografa abdominal fue informada como hidronefrosis grado II de pelvis
renal izquierda. La investigacin de Schistosoma sp. en orina fue positiva visualizndose huevos de S. haematobium. El coprocultivo fue
negativo. Las serologas para Plasmodium falciparum, Entamoeba histolytica, Schistosoma sp., y virus de las hepatitis A, B y C fueron
negativas. El tratamiento se realiz mediante dos dosis de praziquantel a 20 mg/kg/dosis separadas por 12 horas. El paciente no acudi
a posteriores revisiones.

PREGUNTA
Cual de las siguientes manifestaciones es mas caracteristicos en esta patologia?

RESPUESTA
a.- Presencia de huevesillos en heces.
MANUAL DE TRABAJO DEL CURSO ENARM CMN SIGLO XXI
CURSO ENARM CMN SIGLO XXI TEL: 36246001 Pharmed Solutions Institute PGINA 116

b.- Positivo en frotis perifrico.
c.- Hepato y esplenomegalia.
d.- Hematuria y alteracin renal.

FILARIASIS (WUCHERERIA BANCROFTI):
CIENCIAS BASICAS: La filariasis linftica, es causada por la infeccin por nematodos de la familia Filarioidea. Hay tres tipos de estos
gusanos filiformes: Wuchereria bancrofti, que es responsable del 90% de los casos; Brugia malayi, que causa la mayora de los casos
restantes; B. timori, que tambin causa la enfermedad, conocida generalmente como elefantiasis, es una enfermedad tropical
desatendida. La infeccin humana se produce por la transmisin de unos parsitos denominados filarias a travs de los mosquitos.
SALUD PBLICA: En el mundo hay ms de 1300 millones de personas de 72 pases en riesgo de sufrir esta enfermedad. En la actualidad
hay ms de 120 millones de personas infectadas, y unos 40 millones estn desfigurados e incapacitados por la enfermedad.
Aproximadamente un 65% de los infectados viven en la Regin de Asia Sudoriental, el 30% en la Regin de frica, y los dems en otras
zonas tropicales. La filariasis linftica afecta a ms de 25 millones de hombres con enfermedad genital y a ms de 15 millones de
personas con linfedema. Como la prevalencia de la enfermedad y la intensidad de la infeccin estn relacionadas con la pobreza.
PATOGENIA: Cuando un mosquito (cullex, anopheles, aedes), que contiene larvas en estado infectivo pica a una persona deposita los
parsitos en la piel. Desde ah las larvas invaden el organismo, migrando hacia los vasos linfticos donde se desarrollan y transforman
en gusanos adultos en el sistema linftico. Los gusanos adultos se alojan en el sistema linftico y alteran el sistema inmunitario. Estos
gusanos viven entre 6 y 8 aos, y a lo largo de su vida producen millones de pequeas larvas (microfilarias) que circulan en la sangre.
DIAGNOSTICO: La infeccin suele adquirirse en la infancia, pero las manifestaciones dolorosas y muy desfigurantes de la enfermedad
aparecen ms tarde. Mientras que los episodios agudos de la enfermedad causan discapacidad transitoria, la filariasis linftica produce
discapacidad permanente. La filariasis linftica adopta formas asintomticas, agudas y crnicas. La mayora de las infecciones son
asintomticas y no presentan signos externos. A pesar de ello daan el sistema linftico, los riones y el sistema inmunitario. El
linfedema crnico (tumefaccin de los tejidos), o elefantiasis (engrosamiento de la piel), se acompaa a menudo de episodios agudos
de inflamacin local de la piel y de los ganglios y los vasos linfticos. Algunos de esos episodios son causados por la respuesta
inmunitaria del organismo contra el parsito. Sin embargo, la mayora se debe a infecciones bacterianas cutneas porque las defensas
normales se han deteriorado debido al dao linftico. Es frecuente la afectacin de las mamas y de los rganos genitales.
TRATAMIENTO Y PROFILAXIS: El rgimen recomendado es administracin conjunta de dos medicamentos en dosis nicas: albendazol
(400 mg) ms ivermectina (150-200 mcg/kg) en zonas donde la oncocercosis (ceguera de los ros) tambin es endmica, o citrato de
dietilcarbamazina (6 mg/kg) en zonas donde la oncocercosis no es endmica. Estos medicamentos eliminan las microfilarias del
torrente sanguneo y matan a la mayora de los gusanos adultos. Para interrumpir totalmente la transmisin de la infeccin es necesario
que la farmacoterapia colectiva dure entre 4 y 6 aos. Se recomienda que los pacientes con discapacidad crnica (elefantiasis,
linfedema o hidrocele) mantengan una higiene rigurosa y tomen las precauciones necesarias para prevenir las infecciones secundarias y
la agravacin de la enfermedad. Para eleiminacion la OMS recomienda farmacoterapia colectiva.

CASO CLINICO
Mujer de 33 aos originaria de chiapas, ama de casa y agricultora, quien refiere enfermedad insidiosa de 8 aos de evolucin,
caracterizado por la aparicin de lesiones papulares que pustulizan, localizadas a predominio de tronco y extremidades, acompaadas
de prurito y en ocasiones dolor pungitivo. Antecedentes: padeci paludismo y dengue, Al examen fsico, adelgazada, con leve palidez de
piel y mucosas, mltiples mculas hipercrmicas en todo el cuerpo. Se palpan ppulas sin flogosis en axilas, espalda y muslo izquierdo.
A la vitropresin elimina secrecin blanquecina con parsito filiforme de 3 cm. de longitud con cabeza y cola. No adenopatas. No
edemas. No visceromegalia. El resto no contributorio. Su hemograma mostraba Hemoglobina 8.5gr% Leucocitos 6,100 A0, S58, E24, B0,
M6, L12.

PREGUNTA
Respecto al tratamiento cual de los siguientes esquemas es el mas adecuado?

RESPUESTA
a.- Levamisol 150 mg diarios durante 3 meses. Mas Piperacina 400 mg diarios 45 dias.
b.- Albendazol 800 mg diarios por 45 das en dos cursos.
c.- Albendazol (400 mg) ms ivermectina (150-200 mcg/kg).
d.- Citrato de dietilcarbamazina (6 mg/kg).

CRIPTOCOCOSIS (CRYPTOCOCCUS NEOFORMANS):
CIENCIAS BSICAS: Micosis sistmica aguda, subaguda o crnica. Causada por el hongo levaduriforme Cryptococcus neoformans. La
forma pulmonar es generalmente transitoria y leve, la inclusin del sistema nervioso central con meningitis es la forma ms familiar de
la micosis. SALUD PBLICA: Espordico. La meningoencefalitis tiene mortalidad de 100%, en no tratados y 40% en los tratados.
PATOGENIA: La infeccin se adquiere al inhalar el hongo (presente en heces de palomas y sus nidos). El SIDA es el principal factor
predisponente, tambin uso de esteroides, neoplasias; la DM, cirrosis y leucopenia aumentan el riesgo de padecer la enfermedad. Se
transmite en la forma de basidiosporas, presentes en el ambiente por tiempos cortos y son infecciosas para el humano y otros
animales, despus de su diseminacin pueden sintetizar material plisacarido y convertirse en levaduras encapsuladas. Al entrar por
vas respiratorias altas, el hongo coloniza el rbol bronquial, por ello inicia como una enfermedad pulmonar, que se disemina a piel,
huesos, vsceras, abdominales y SNC. DIAGNOSTICO: Clnica; asintomtica, a veces dolor torcico, tos, fiebre, infiltrados intersticiales o
lobeares y derrame pleural; las lesiones pulmonares se caracterizan por inflamacin granulatiza, la diseminacin hematogena permite
que infecte el SNC (sustancia gris cortical y ganglios basales). La meningoencefalitis, los datos clnicos estn relacionados con el
desarrollo de lesiones granulomatosas en las meninges, se presenta con nausea, cefalea, irritabilidad, confusin, demencia, visin
borrosa, fiebre, rigidez de cuello y parlisis de nervios craneales. En el 10% aparecen lesiones cutneas (papilas, lceras, abscesos).
MANUAL DE TRABAJO DEL CURSO ENARM CMN SIGLO XXI
CURSO ENARM CMN SIGLO XXI TEL: 36246001 Pharmed Solutions Institute PGINA 117

Laboratorio: cultivo en Sabouraud, forma colonias de color crema plido. Antgeno manan se detecta en 33% de los casos. Biopsia para
diagnostico de criptococosis pulmonar. LCR vemos levadura encapsulada, hipoglucorraquia, pleocitocis linfocitica y aumento de
protenas. TRATAMIENTO: Anfotericina B a 0.7mg/kg/da y 5-fluorocitocina a 100mg/kg/da durante 6-10 semanas.

CASO CLINICO
Paciente femenina de 35 aos con antecedentes de presentar desde haca dos meses cefalea, irritabilidad, prdida de la memoria,
cambios de personalidad, alteraciones del sensorio, dificultad para caminar por disminucin de la fuerza muscular en el miembro
inferior izquierdo y ataxia de la marcha. EF: presenta alteraciones del sensorio con somnolencia, hemiparesia izquierda ligera a
predominio crural, ataxia de la marcha y rigidez nucal y lesiones vesiculosas herpticas en la regin peribucal. El fondo de ojo mostraba
borramiento de los bordes temporales de ambas papilas. Estando ingresada se queja de vrtigo y presenta varios vmitos. Se le realizan
los siguientes exmenes de laboratorio: Hemoglobina: 122 g/l, Hematocrito: leucocitos: 6.1x109 g/l, neutrfilos: 59, linfocitos: 40,
monocitos: 1; eritrosedimentacin: 120 min; glicemia: 4.8 mmol/l; creatinina: 212 mmol/l, TGP: 11 unid, urocultivo: normal, sin
crecimiento; tomografa axial computarizada (TAC) de crneo simple: sin alteraciones evidentes; Lquido cefalorraqudeo (LCR)
citoqumico claro y transparente, clulas: 11, pandy (-), protenas: 25, glucosa 1.9 mmol/l.

PREGUNTA
Cual es la conducta diagnostica mas adecuada para el presente caso?

RESPUESTA
a.- Serologia e inmunolgica.
b.- Elisa y west-blot.
c.- Resonancia magnetica con contraste.
d.- Cultivo y prueba de tinta china en LCR.

PREGUNTA
El paciente es positivo para VIH, al termino del tratamiento habitual cual de las siguientes alternativas para continuar la profilaxis
secundaria es mas adecua?

RESPUESTA
a.- Fluconazol 200 mg / dia
b.- Ketoconazol 400 mg / dia.
c.- Itraconazol 100 mg / dia
d.- Clortrimazol 300 mg / dia.

CASO CLINICO
Se trata de paciente femenino de 38 aos de edad la cual acude al servicio de urgencias debido a que presenta fiebre, cefalea y dolor en
la regin de la nuca, como antecedentes de importancia la paciente trabaja en un aviario, no es diabtica ni hipertensa, aparentemente
sana antes de este episodio, a la exploracin fsica se observa conciente, irritable, orientada, facies algicas, se realiza TAC de cabeza el
cual no revela datos de importancia, se realiza puncin lumbar reportndose presin de apertura de 20 cmH2O, leucocitos 15 clulas
(90 % monocitos), protenas de 0.6 g/L, glucosa de 50mg/dl y tincin positiva con tinta de la India.

PREGUNTA
Cual es el tratamiento mas adecuado para el caso.

RESPUESTA
a.- Anfotericina.
b.- Fluconazol.
c.- Cexfriaxona.
d.- Vancomicina.

CITOMEGALOVIRUS (CMV):
CIENCIAS BASICAS: CMV es un herpes virus, que hace a las clulas infectadas 2-4 veces el tamao de las clulas circundantes. Como
resultado de la primoinfeccin se establece una infeccin latente o persistente. El virus permanece principalmente en clulas
endoteliales y leucocitos durante toda la vida del individuo a menos que la reactivacin se desencadene por la inmunidad celular
deprimida (por ejemplo, en receptores de trasplante o infectados por el VIH. Enfermedad de inclusin citomeglica (clulas gigantes,
con inclusiones que semejan un ojo de lechuza) SALUD PBLICA: Enfermedad por CMV se encuentra en todo el mundo. En los
Estados Unidos, aprox. 1 % de los recin nacidos estn infectados. La seroprevalencia en pases desarrollados va de 30-70% y en pases
en vas de desarrollo alcanza hasta 90% en la edad adulta. Los homosexuales masculinos tienen mayor seropositividad 94%
PATOGENIA: El virus se puede transmitir por la leche materna, saliva, contacto sexual (virus en semen y secreciones cervicales), va
transplacentaria, transfusin sangunea, transfusin de rganos. La transmisin requiere el contacto repetido o prolongado. La
infeccin se establece en las clulas del hospedero, en la lnea mieloide, que se transforma en el reservorio para infecciones
posteriores. DIAGNOSTICO: Clnico; La infeccin congnita por CMV; se produce en aproximadamente el 5 % de los fetos infectados en
el contexto de la infeccin por CMV materna primaria durante el embarazo. Presentan petequias, hepatoesplenomegalia e ictericia.
Otros hallazgos; microcefalia con o sin calcificaciones cerebrales, retraso del crecimiento intrauterino, prematuridad y coriorretinitis.
Los hallazgos de laboratorio incluyen pruebas anormales de la funcin heptica (PFH), trombocitopenia, hemlisis, y el aumento de los
niveles de protenas del fluido cerebroespinal (CSF). La tasa de mortalidad es del 20-30% de los lactantes con enfermedad grave, los
MANUAL DE TRABAJO DEL CURSO ENARM CMN SIGLO XXI
CURSO ENARM CMN SIGLO XXI TEL: 36246001 Pharmed Solutions Institute PGINA 118

sobrevivientes tienen dificultades intelectuales o auditivas. Infeccin perinatal, se adquiere por la lactancia o contacto con secreciones
maternas infectadas. La mayora de pacientes son asintomticos, pero las infecciones oportunistas neumonitis intersticial y otras
pueden ocurrir, particularmente en los bebs prematuros. Mononucleosis; es el sndrome ms comn del CMV en huspedes
inmunocompetentes. El perodo de incubacin vara de 20 a 60 das. Los sntomas duran 2-6 semanas e incluyen fiebre, fatiga profunda
y malestar general, mialgias, dolor de cabeza, y esplenomegalia, faringitis y adenopatas cervicales son raras. Los hallazgos de
laboratorio incluyen linfocitosis relativa con > 10 % de linfocitos atpicos. El aumento de los niveles sricos de transaminasas y fosfatasa
alcalina as como anormalidades inmunolgicas (por ejemplo, la presencia de crioglobulinas o aglutininas fras) pueden ser evidentes. La
recuperacin es completa, pero astenia postviral puede persistir durante meses. La excrecin de CMV en la orina, las secreciones
genitales, y/o la saliva pueden continuar durante meses o aos. CMV es el patgeno viral ms comn e importante que complica el
trasplante de rganos, la infeccin por CMV es un factor de riesgo de prdida del injerto y muerte. El riesgo de infeccin es mayor 1-4
meses despus del trasplante, pero la retinitis puede ocurrir ms tarde. El rgano trasplantado es un riesgo especial, por ejemplo,
neumonitis por CMV suele seguir un trasplante de pulmn. Alrededor del 15-20% de los receptores de trasplante de mdula sea va a
desarrollar neumona por CMV, 5-13 semanas despus del trasplante, con una tasa de letalidad del 84-88 %. El riesgo de enfermedad
grave se puede reducir por la profilaxis o terapia antiviral preventiva. El CMV es un patgeno importante en pacientes con infeccin por
el VIH cuyo recuento de clulas CD4 + han cado por debajo de 50 a 100 / l. En esta configuracin, el CMV causa retinitis, colitis y
enfermedad diseminada. Pacientes inmunodeprimidos con infeccin por CMV desarrollan fiebre, malestar general, anorexia, fatiga,
sudores nocturnos y artralgias o mialgias. Taquipnea, hipoxia, y tos no productiva. La participacin del tracto gastrointestinal puede ser
localizada o extensa, con lceras que pueden sangrar o perforar. La hepatitis es comn. SNC, con mayor frecuencia afectan a pacientes
infectados por el VIH, retinitis grave, encefalitis, polirradiculomielopata. Retinitis por CMV puede dar lugar a ceguera. Las lesiones
comienzan como pequeas reas blancas de necrosis retiniana granular, con el desarrollo posterior de las hemorragias y edema
retiniano. Infeccin grave se asocia con virema persistente y compromiso multiorgnico. Amplia necrosis suprarrenal se ve a menudo
en la autopsia. El cultivo viral (diagnostico de certeza) y los anticuerpos monoclonales se utilizan para detectar el antgeno temprano
inmediato de CMV. La deteccin de antgenos de CMV en los leucocitos de sangre perifrica o de ADN de CMV en sangre. Los
anticuerpos no pueden ser detectados por un mximo de 4 semanas de la infeccin primaria, y los ttulos pueden permanecer elevados
durante aos. IgM puede ser til en el diagnstico de infeccin aguda. Cuando sea posible, los donantes seronegativos deben ser
utilizados para los receptores de trasplante seronegativos. TRATAMIENTO: Ganciclovir (o valganciclovir, el profrmaco oral de
ganciclovir ) se usa como tratamiento y como profilctico, produce tasas de respuesta del 70-90 % entre los infectados por el VIH,
pacientes con retinitis por CMV o colitis. La terapia de induccin con ganciclovir (5-12mg/kg/da IV 2-7 semanas) o valganciclovir (900
mg/24hrs VO) se da durante 14-21 das. En los receptores de trasplante, se ha demostrado reduccin de la presencia de CMV, cuando
se administra tratamiento profilctico a base de vanganciclovir cuando el paciente es susceptible. Actualmente se puede utilizar el
frmaco durante los primeros 100 das postrasplante. La dosis de induccin es de 900mg c/12 hrs y despus 900mg c/24 hrs. Otros
frmacos activos contra CMV son: El foscarnet es activa contra la infeccin por CMV, pero se reserva para casos de insuficiencia a
ganciclovir o intolerancia a causa de sus efectos txicos, que incluyen la disfuncin renal, hipomagnesemia , hipopotasemia ,
hipocalcemia , y parestesia. Este medicamento debe ser dado a travs de una bomba de infusin, y su administracin debe ser
monitoreado de cerca. Un rgimen de induccin de 60 mg / kg cada 8 horas o 90 mg / kg cada 12 horas durante 2 semanas es seguido
por los regmenes de mantenimiento de 90-120 mg / kg al da. Cidofovir (en casos de retinitis) tiene una intracelular larga vida media.
Los regmenes de induccin de 5 mg / kg por semana durante 2 semanas son seguidos por los regmenes de mantenimiento de 3-5 mg /
kg cada 2 semanas. Cidofovir produce nefrotoxicidad grave por lesin de las clulas del tbulo proximal. El uso de la hidratacin salina y
probenecid reduce este efecto adverso.

CASO CLINICO
Hombre de 53 aos, sometido a trasplante renal hace un mes, que acude a urgencias por cuadro febril con temperatura de 38,5 C de
24 horas de evolucin con escalofros, dolor abdominal leve difuso, astenia, anorexia y disminucin subjetiva del volumen de diuresis. El
cuadro progres con un aumento del dolor abdominal, aparecieron nuevos picos febriles junto con cuadro diarreico, con melena,
asociado a deterioro neurolgico, hepatoesplenomegalia, as como alteracin funcin heptica, anemia y trombocitopenia. Analtica:
GOT/GPT 135/156 U/l; LDH 558 U/l; sodio 130 meq/l, fibringeno 133 mg/dl, hemoglobina 9,2 g/dl, y hematocrito del 26,8% y
plaquetas de 48.000 l con leucocitos normales (5.500 l con frmula normal), aumento muy importante de triglicridos (738 mg/dl),
deterioro progresivo de funcin renal (creatinina entorno a 4-5 mg/dl). Se solicitaron nuevas pruebas en las que destacaron: antgeno
precoz CMV positivo con PCR de CMV mayor a 100.000 copias/ml.

PREGUNTA
Considerando la complicacin, cual es la medida profilctica mas apropiada?

RESPUESTA
a.- Ganciclovir 200 mg por 100 das.
b.- Aciclovir 200 mg por 75 dias.
c.- Cidofovir 500 por 100 dias.
d.- Oseltanmivir 100 mg por 74 dias.

CASO CLINICO
Femenino de 42 aos. Manifiesta 8 meses con herpes recurrente en boca. Ingresa por padecimiento de 2 meses con tos seca en accesos
y disnea rpidamente progresiva. Adems dolor torcico bilateral, fiebre hasta 39C y prdida de peso de 14kg. La recurrencia de la
lesin herptica le ocasionaba disfagia y odinofagia. Al examen fsico presentaba placas blanquecinas en orofaringe; la exploracin del
trax, con disminucin del ruido respiratorio y estertores finos. Al aire ambiente la saturacin de oxgeno era del 86%. El reporte de la
gasometra arterial con oxgeno suplementario al 70% fue: pH 7,30, pCO2 40,5mmHg, pO2 132mmHg, HCO3 19,5mmol/l, exceso de
base 5,8mmol/l, saturacin de oxgeno al 97,9%. ndice de oxigenacin (IO) de 188. Laboratorio: linfopenia de 600clulas/mm3, Hb
MANUAL DE TRABAJO DEL CURSO ENARM CMN SIGLO XXI
CURSO ENARM CMN SIGLO XXI TEL: 36246001 Pharmed Solutions Institute PGINA 119

11,8gr/dl, deshidrogenasa lctica de 971UI/l y albmina 3,3gr/dl. La Rx de trax presentaba opacidades bilaterales en parche con vidrio
deslustrado y neumomediastino, por lo cual, en el diagnstico diferencial se incluy inmunosupresin asociada a VIH y neumona por P.
jirovecii (PJP). El anlisis para VIH por ELISA fue POSITIVO, se confirm por Western Blot. Se le realiz broncoscopia con biopsia
transbronquial y lavado broncoalveolar (LBA). El estudio histopatolgico se reporta en la figura 1. Recibi tratamiento con
Trimetoprim/Sulfametoxasol y Prednisona en dosis de reduccin por 21 das. Se somete a LBA cuyo estudio de patologa confirma la
presencia de hemorragia alveolar reciente y activa (fig. 2B). Adems, por rt-PCR se documenta infeccin por CMV

PREGUNTA
Considerando la complicacin, cual es la medida terapeutica mas apropiada?

RESPUESTA
a.- Ganciclovir.
b.- Aciclovir.
c.- Cidofovir.
d.- Gamaglobulina.

MONONUCLEOSIS INFECCIOSA (VIRUS DE EPSTEIN-BARR):
CIENCIAS BSICAS: Enfermedad causada por el virus de Epstein-Barr (VEB), pertenece a la familia de los gammaherpesvirus (DNA
bicatenario), tambin llamada enfermedad del beso. El VEB se descubri al observar en el microscopio biopsias de linfoma de Burkitt,
un hallazgo fortuito lo asocio a mononucleosis infecciosa. Se trasmite por secreciones de oro faringe y por contacto sexual. SALUD
PUBLICA: Distribucin mundial, con mayor frecuencia en mujeres adolescentes y adultos jvenes. La incidencia a nivel mundial se
aproxima al 100%. La enfermedad recurrente es una fuente de contagio y puede causar diseminacin asintomtica. PATOGENIA: El VEB
infecta las clulas epiteliales de la cavidad oral donde se multiplica en los linfocitos, causa lisis celular, pasa a la saliva y se disemina en
el tejido linftico, es mitogeno para las clulas B y permanece en ellas en forma latente. La inmunidad celular permanente participa en
el control de la infeccin. DIAGNOSTICO: Clnica; en nios generalmente es subclnica, a veces, faringitis, otitis, adenopatas cervicales.
Periodo de incubacin de 10-14 dias, en adultos jvenes fiebre, adenopatas en 2 cadenas ganglionares, exantema maculopapular,
faringitis, fatiga, anorexia, petequias en paladar y frecuentemente hepatoesplenomegalia. Serologa, buscar aglutininas heterofilicas
(titulo >40) o identificacin de cuerpos de inclusin intracelular. En infeccin aguda se observa IgM e IgG contra el antgeno de la casi
de viral (anti-VCA), adems del anticuerpo contra el antgeno temprano (anti-EA). COMPLICACIONES: Puede originar trastornos
neurolgicos como: meningoencefalitis, parlisis de Bell, sndrome de Guillan-Barre, mielitis transversa, neuritis perifrica, anemia
hemoltica, CID. Se le a asociado carcinoma nasofarngeo, linfoma de Burkitt y leucoplasia oral vellosa. TRATAMIENTO: No especifico,
solo sintomtico, reposo para evitar rotura esplndida. Corticoides, si hay obstruccin de va area.

CASO CLINICO
Varn de 19 aos de edad, estudiante, tres semanas antes del ingreso haba comenzado con fiebre, adenopatas laterocervicales
bilaterales, rash cutneo y dolor farngeo. Haba sido seguido de forma ambulatoria, pero se decidi su ingreso ante la intensa
afectacin del estado general. Presentaba leucocitosis (24.500cel/l) con predominio de linfocitos (71%) que en el frotis tenan aspecto
atpico en un alto porcentaje, trombopenia ligera (106.000plaquetas/l), alargamiento discreto de los tiempos de coagulacin (INR 1,7)
y datos de colostasis (bilirrubina directa 3,1mg/dl). Tena un ttulo alto (>1/256) de IgM frente al antgeno de la cpside del VEB. El
estudio serolgico de CMV, VHA, VHB y VIH era negativo. Una TAC de trax y abdomen puso de manifiesto la presencia de
esplenomegalia, hepatomegalia, ascitis y derrame pleural bilateral. Se instaur tratamiento con metilprednisolona (1mg/kg/da) que se
mantuvo durante 30 das. Tras su inicio remiti la fiebre y se produjo una lenta, pero progresiva mejora en un periodo de 35 das. El
paciente fue dado de alta a los 10 das, pero en el seguimiento posterior refera debilidad y dificultad para la concentracin durante tres
meses ms. Los parmetros hepticos se normalizaron a los 6 meses del inicio de los sntomas.

PREGUNTA
La manifestacin ms frecuente de este padecimiento es una mononucleosis severa o fatal, cual de los siguientes cuadros mas
frecuente observar como diagnostico diferencial?

RESPUESTA
a.- Sndrome hemofagoctico asociado a virus.
b.- Trastornos linfoproliferativos.
c.- Disgammaglobulinemias (hipogammaglobulinemia).
d.- Anemia aplstica.

RABIA:
CIENCIAS BSICAS: Zoonosis causada por Rabdovirus, se trasmite al hombre principalmente por la saliva de animales infectados (y en
periodo de transmisin) a partir de una mordedura, rasguo o solucin de continuidad en la piel o mucosas, se presenta como una
encfalomielitis de curso agudo. PATOGENIA: La velocidad con se manifiesta la rabia, depende de la cepa del virus, de la concentracin
de receptores para el virus en las clulas nerviosas del msculo esqueltico, de la magnitud del inoculo, de la internacin en el sitio de
entrada y de la proximidad de la lesin al SNC. Cuando la va de entrada es transcutanea o epidrmica, el virus se queda un tiempo en
el sitio de inoculacin. En ese lapso puede ocurrir una primera replicacin en las clulas nerviosas de las placas musculares ms
cercanas a la herida y con ello aumenta la carga viral, despus el agente infeccioso se desplaza de 8-mm por da, avanzando por
continuidad al infectar nuevas neuronas del SNP. Por endocitosis o fusin de membranas. El camino que sigue el virus de SNP al SNC se
denomina diseminacin centrpeta. Una vez en el SNC la infeccin es irreversible, conduciendo invariablemente a la muerte del
individuo. La multiplicacin del virus en el encfalo inicia en el sistema lmbico y luego al resto del cerebro, produciendo edema,
MANUAL DE TRABAJO DEL CURSO ENARM CMN SIGLO XXI
CURSO ENARM CMN SIGLO XXI TEL: 36246001 Pharmed Solutions Institute PGINA 120

congestin vascular, infiltracin directa de linfocitos e hiperemia de las leptomeninges, despus de haber completado su invasin al
cerebro comienza la diseminacin centrifuga, en la cual el virus regresa a los rganos con alta intervencin nerviosa, cuando llega a
glndulas salivales, se difunde por el nervio trigmino, lo que inicia la eliminacin del virus a travs de la saliva. En perros la eliminacin
de virus por saliva es de 3-10 das antes de que se manifiesten los primeros signos clnicos, esto es importante ya que el animal esta
asintomtico, pero ya es infectivo. CURSO CLNICO: Periodo de incubacin: Va desde la exposicin hasta inicio de signos clnicos,
depende de la cantidad de virus inoculados y sitio anatmico (cabeza 30-48 das, mano 40-59 das, pierna 38-72 das). Las primeras
manifestaciones se presentan de 1-3 meses despus de la agresin. Prodromo y primeros sntomas: Dura de 1-20 das, sntomas
neurolgicos como ansiedad y agitacin que son precedidos por malestar general, parestesias en el sitio de la lesin, fiebre, anorexia,
vomito, dolor de cabeza, letargo y fiebre. Periodo neurolgico agudo: Signos de dao a SNC, se puede presentar como rabia paraltica o
furiosa. La paraltica es ms comn en ganado bovino y roedores, la furiosa en mamferos carnvoros que son reservorios de la
enfermedad. Los signos y sntomas para ambas son: fiebre, rigidez de la nuca, parestesias, fascculacion muscular, convulsiones
generalizadas y focalizadas, hiperventilacin e hipersalivacion, la presencia de hidrofobia que es patognomnico de rabia, dura de 1-5
min y se espacia por periodos de lucidez, es producto de un reflejo exagerado para proteger el tracto respiratorio, ms que como
espasmo de faringe y laringe. En esta fase evoluciona hasta la afeccin del SNC, que incluye hiperactividad, desorientacin, delirio,
alucinaciones, convulsiones, rigidez de nuca y progresan hasta que se presenta parlisis irregular de msculos respiratorios lo que
conduce a estado de coma. Durante el periodo de transicin entre la fase neurolgica y el coma se ostentan lapsos de lucidez y locura,
respiracin rpida e irregular, ya posteriormente parlisis generalizada particularmente en la zonas inervadas por los nervios craneales
y en el msculo cardiaco, se paralizan los msculos respiratorios y sobreviene la muerte. DIAGNOSTICO: Se pueden hacer muestras
postmortem (biopsia 10-20g de cerebro, 1-3 g de hipocampo o 5-10g de medula espinal) en animales y seres humanos y antemortem
(biopsia de cuero cabelludo, impronta de cornea, saliva suero y liquido cefalorraquiedeo), en seres humanos, tomar muestras de 7-14
das de iniciados los signos neurolgicos. La tcnica de rutina para diagnostico de rabia es inmunofluorescencia directa, otras tcnicas
auxiliares son determinacin en LCR, ELISA. TRATAMIENTO: Observar al perro o gato 10 das posteriores a la agresin. La atencin
inmediata de la lesin es; lavar con agua y jabn de forma abundante, desinfectar con agua oxigenada o tintura de yodo, si requiere
sutura aplicar primero inmunoglobulina antirrbica humana y se aproximan los bordes, valorar aplicacin de antibiticos y toxoide
tetnico en heridas contaminadas o punzantes, secar con gasa y cubrir. Si animal agresor no se localiza iniciar tratamiento antirrbico:
A) Exposicin leve; una dosis de vacuna en los das 0, 3, 7, 14, 28, en la regin deltoidea por va IM. B) Exposicin grave; aplicar
inmunoglobulina antirrbica humana lo ms cerca al da de la agresin (20 UI/kg), la mitad alrededor de la herida y el resto IM. Tambin
debemos aplicar la vacuna antirrbica humana, si el animal sigue sano durante 5 dias posteriores se suspende la vacuna, de lo cobtrario
prodeguir. PREVENCIN: El control en las poblaciones y la variacin parenteral y oral de reservorios silvestres y domsticos es ahora el
mejor tipo de prevencin.

CASO CLINICO
Nia de 12 aos ingresa al Hospital General OHorn de la Ciudad de Mrida, referida de la localidad de Tekax, Yucatn, por una historia
de dolor en miembro torcico derecho por aparente traumatismo que no haba cedido al tratamiento mdico, sin ms antecedentes de
importancia. La afeccin continu con limitacin funcional de la mueca y disminucin de la fuerza de dicha extremidad superior, la
cual fue progresiva, hasta acompaarse de dolor torcico con limitacin de la respiracin, fiebre, odinofagia, presentando aerofobia e
hidrofobia y cambios en la conducta que se hicieron mas evidentes, por lo que se sospech de una encefalitis por virus rbico. La nia
contino con deterioro general que hizo necesaria la asistencia ventilatoria, con evolucin trpida y datos clnicos de muerte cerebral
declarada 12 dias despus.

PREGUNTA
Cual es la conducta a seguir?

RESPUESTA
a.- Profilaxia con inmunoglobulina a contactos.
b.- Vacunacion antirrbica canina en cerco sanitario.
c.- Cerco epidemologico con vacunacin antirrbica a contactos.
d- Busqueda de perros sospechos para rabia.

PREGUNTA
Cual de los siguiente vectores no puede transmitir el virus?

RESPUESTA
a.- Canes.
b.- Felinos.
c.- Murcielagos.
d.- Roedores.

CASO CLINICO
Se trata de masculino de 25 aos de edad dedicado al campo, refiere que al estar trabajando fue mordido por perro desconocido, el
cual no pudo capturar, se conoce de casos de rabia en la zona en animales no domesticados, acude a consulta una hora despus del
hecho.

PREGUNTA
Cul es la conducta ms apropiada a seguir en este caso.

MANUAL DE TRABAJO DEL CURSO ENARM CMN SIGLO XXI
CURSO ENARM CMN SIGLO XXI TEL: 36246001 Pharmed Solutions Institute PGINA 121

RESPUESTA
a.- Lavar la herida y vigilancia.
b.- Lavar la herida y administracin de toxoide.
c.- Lavar la herida, administracin de toxoide y aplicar globulina inmunitaria humana.
d.- Lavar la herida, administrar toxoide, aplicar globulina inmunitaria humana y vacuna de clulas diploides humanas.

ENFERMEDAD DEL LEGIONARIO (LEGIONELLA PNEUMOPHILA):
CIENCIAS BSICAS: Llamada as porque afecto a los asistentes de una reunin de la Legin Americana, celebrada en un hotel de
Filadelfia (1976). Legionella bacteria gran negativa, de forma bacilar crece en aguas (entre 20- 50) naturales, lagos, ros, arroyos, lodos.
Supervivencia en aire es corta SALUD PBLICA: Legionella es causa frecuente de neumona comunitaria y nosocomial, con una
mortalidad que oscila entre el 3-50%. CLASIFICACIN: Fiebre de pontiac; El 95%, la forma ms leve y la ms frecuente se parece a una
"gripe", no produce neumona es autolimitada, con buena evolucin. Enfermedad del Legionario; De 1-5%, es la forma ms grave,
provoca neumona y afecta al estado general. La mayora de casos se dan en forma espordica. PATOGENIA: Factores de riesgo: uso
previo de corticoides, e inmunosupresores, neoplasias, trasplante, dilisis, sexo masculino, mayores de 50 aos, tabaquismo, EPOC, va
de contagio parece ser a travs de aerosoles de gotas de agua contaminada que son aspirados o inhalados. Estas gotas preceden de
reservorios de agua infectados por legionella (aparatos de refrigeracin, condensados de agua, duchas, sistemas de canalizacin y
distribucin de agua que sirve de reservorio en hospitales, edificios). Sospechar en un contexto epidemiolgico adecuados datos
clnicos, un habito tabquico importante, la ausencia de supera a antibiticos betalactamicos y la observacin de escasos
microorganismos en un esputo llenos de neutrofilos son caractersticos. Solicitar antgeno de legionella en orina. DIAGNOSTICO:
Clnico: Inicio brusco con fiebre alta de predominio matutino, con afeccin general, tos no productiva, esputos, escalofros, cefalea,
presencia de manifestaciones extrapulmonares (diarreas o confusin mental), hiponatrema y elevacin de la creatincinasa. La
neumona por legionella se incluye dentro de las neumonas de presentacin graves. TRATAMIENTO: De eleccin clsico es la
eritromicina (efectos secundarios; flebitis, alteraciones gastrointestinales, ototoxicidad), por ello se puede utilizar claritromicina (500mg
c/12hrs 10-14 das). Tambin til azitromicina (500mg/da). La experiencia con fluoroquinolonas como levofloxacino mas en caos
graves o neumona intrahospitalaria (500mg/da en los primeros das c/12 hrs e intravenoso) y ofloxacino son positivas.

CASO CLINICO
Varn de 46 aos, fumador de 40 paquetes al ao y ciruga de colesteatoma con parlisis facial perifrica derecha residual. Una semana
antes de la clnica refiere estancia en un hotel del Levante. Acude al mdico de guardia del centro de salud por fiebre de ms de 39 C,
sin otra sintomatologa acompaante y con exploracin fsica normal. Se le pauta tratamiento sintomtico con paracetamol. Al da
siguiente acude a su mdico de cabecera por persistencia de la fiebre, no existiendo variaciones en la exploracin. Se decide continuar
con el tratamiento prescrito el da anterior y observacin domiciliaria. A los cinco das tras persistir la fiebre, y aparecer disnea y tos con
expectoracin hemoptoica, el paciente es remitido al Servicio de Urgencias hospitalario, donde se le practican diversas pruebas
complementarias: hemograma (16.430 leucocitos con 93% neutrfilos), bioqumica (Na 134, PCR 340), GAB (pH 7.46, PO2 60, PCO2 30),
Rx de trax (infiltrado alveolar bilateral con derrame pleural derecho).

PREGUNTA
Considerando el agente infeccioso presuntivo, cual es la conducta diagnostica mas apropiada?

RESPUESTA
a.- Realizar Hemocultivo.
b.- Baciloscopia de Esputo.
c.- Antigeno en Orina.
d.- Inmunoglobulinas.

CASO CLINICO
Se trata de paciente masculino de 57 aos de edad el cual acude al servicio de urgencias debido a tos intensa no productiva y fiebre, el
paciente cuenta con los siguientes antecedentes, originario de guanajuato, empleado de la construccin, alcoholismo y tabaquismo
positivo desde hace 40 aos, actualmente bajo tratamiento por EPOC y cirrosis heptica, se realizan estudio de laboratorio y gabinete
de rutina donde se observa un reporte con disminucin de la respuesta inmunolgica esperada, la radiografia de torax presenta un
patrn caracterstico, sin embargo el cuadro respiratorio bajo se ha intensificado por lo que se traslada a terapia intensiva.

PREGUNTA
Cual es tratamiento antibitico ms apropiado para este caso.

RESPUESTA
a.- Azitromicina.
b.- Levofloxacina.
c.- Trimetoprim/sulfametoxazol.
d.- Penicilina.

RICKETTSIASIS (R. RICKETTSIA; R. CORONII):
CIENCIAS BSICAS: Son bacterias estrictamente intracelulares gramm negativas, que se transmiten a travs de vectores artrpodos
(garrapatas, pulgas y piojos) a diferentes mamferos que actan como reservorios. SALUD PBLICA: Sus caractersticas epidemiolgicas,
su alta infectividad y elevada mortalidad de algunas, pueden hacer que se conviertan en armas biolgicas ideales. CLASIFICACIN:
Existen 2 tipos de vasculitis que en funcin de ellas podemos diferenciar en 2 grupos: grupo de las fiebres manchadas; afectan
MANUAL DE TRABAJO DEL CURSO ENARM CMN SIGLO XXI
CURSO ENARM CMN SIGLO XXI TEL: 36246001 Pharmed Solutions Institute PGINA 122

endotelio, capa intima y media, entre ellas Fiebre manchada de Montaas Rocosas (en Amrica, transmitida por garrapatas causada
por R. rickettsia), Fiebre Botnosa mediterrnea (causada por R. coronii, transmitida por garrapata de perro). El grupo de las fiebres
tficas; afectan endotelio y capa intima entre ellas Tifus o exantema epidmico (causado por R. prowazekii, transmitido por piojos)y
tifus murino (causado por R. typhi, transmitido por pulgas). PATOGENIA: La rickettsia penetra en el organismo, tras la picadura, o
atraves de una solucin de continuidad contaminada con heces de los artrpodos (ingresan a travs del piquete a la hora des rascado),
o bien a travs de la conjuntiva o vas respiratorias, rpidamente alcanza torrente sanguneo, all se une a la clula endotelial y se
producen cambios en la conformacin de la membrana que facilitan la fagocitosis de la bacteria, esta rompe la membrana fagosomica y
escapa libre a citosol aqu se produce la multiplicacin (intracelular)a travs de un mecanismo de expulsin activo, se produce
inoculacin en clulas contiguas al rea infectada, tambin hay diseminacin linftica. Y este ciclo, ocasiona la aparicin de mltiples
focos de vasculitis y un estado procoagulante endovascular erupcin, hemorragias, trombosis y a veces gangrena). La destruccin
celular y liberacin de rickettsias explican la fiebre, las lesiones capilares son la base del colapso vascular y alteracin del estado de
conciencia. El aumento de permeabilidad capilar, produce extravasacion de lquidos, y protenas, dando edema, y agravamiento del
estado de choque. DIAGNOSTICO: Clnico; fase de prdromo; malestar general, cefalea y febrcula. Fase de estado; fiebre elevada
sostenida de 2-3 semanas, cefalea intensa, vrtigos ataque al estado general con gran postracin, piel seca, estado de conciencia
alterado con alternancia de estupor y delirio, tos seca, nausea, vomito. Fase de exantema; mculas eritematosas en axilas y flancos que
se extienden al tronco y finalmente a extremidades. En la eruptiva, es comn que exista oligura, taquicardia desproporcionada a la
temperatura, polipnea. Complicaciones; gangrena por obstruccin. Serologico: IgM e IgG aparecen despus de 1 semana a 10 das, y
duran de 3-4 meses. Existencia de antgenos con prueba de Weil-Flix. En Mxico prueba de Ruiz-Castaeda. TRATAMIENTO: primera
eleccin tetera inclinas, clorhidrato de tetraciclina 40mg/kg dividida c/6 o 8 o 12 hrs VO o parenteral, doxiciclina 2-4mg/kg/da (dosis
mxima 200mg/ dia en dos dosis), mantenerse 3 das despus de remisin trmica. Alternativa cloranfenicol, el tratamiento debe
continuar hasta 24 hrs despus de haber desaparecido la fiebre. El estado de choque, hiponatrema, edema y coagulopatia de
consumo; infusiones de albmina, heparina (la sangre total y plasma contraindicados por CID). Los medicamentos solo detienen el
desarrollo, de las rickettsias sin destruirlas, no son raras las recadas.

CASO CLINICO
Paciente masculino de 8 aos de edad, que inici 5 das previos con presencia de fiebre intensa de hasta 40C de difcil control, dolor
farngeo y tos productiva, astenia y adinamia, llevado con mdico 4 das previos quien recomienda trimetoprim/sulfametoxazol a dosis
de 7mg/kg/da (8 dosis), ambroxol y clorfenamina con paracetamol para tratamiento decuadro respiratorio. El paciente no muestra
mejora clnicamente, e incluso aparecen mltiples zonas de erupcin en lapiel con eritema y petequias, pruriginosas, principalmente
encara y que se han ido extendiendo a tronco y extremidades.Con patrn tipo centrpeto. Al no encontrar mejora es trado a revisin
mdica a la consulta externa de con diagnstico probable de enfermedad de Kawasaki. El cuadro clnico inici posterior a una visita de
establo y en contacto con ganado bovino y ovino, se desconoce s tenan vectores ya que el paciente luce descuidado pues no vive con
los padres. Tiene hbitos de higiene regulares. Bao cada tercer da pero en vacaciones hasta cada semana. No muestra cartilla,
desconoce cules vacunas le faltan. Tuvo varicela a los 4 aos. El paciente se observ hipotrfico, consciente y cooperador, con buena
coloracin de piel y lapresencia de una erupcin maculo papular eritematosa, descamativa, petequial o micromacular en parpados, as
como otras regiones de cara, pecho, dorso, abdomen y extremidades, respetando palmas y plantas de pies. Kerning y
Brudsinskinegativos y sin otros datos de meningismo; cuello simtricocon un ganglio palpable en regin media, mvil no doloroso
menor a 1cm. Hemorragia en flama conjuntival bilateral, sin secrecin conjuntival ni prurito, mucosa nasal conrinorrea hialina, mucosa
oral bien hidratada y faringe hipermica, sin exudados. Trax sin dificultad respiratoria, con estertores gruesos bilaterales transmitidos,
sin problemas a la ventilacin. Precordio rtmico sin soplos y pulsos perifricos normales. Abdomen blando, levemente distendido, con
peristalsis normal, sin hepatomegalia ni esplenomegalia.

PREGUNTA
Cual es el agente etiolgico mas probable?

RESPUESTA
a.- Rickettsia prowazekii
b.- Rickettsia typhi
c.- Rickettsia rickettsii
d.- Rickettsia coronii

CASO CLINICO
Se trata de masculino de 19 aos de edad ingresado al servicio de urgencias presentando fiebre, cefalea intensa, dolos abdominal
difuso pero ligero, pero con mialgias intensas, al examen fsico se observa con 38,5 c, FC 112, TA 120/70 su estado general es adecuado
en crecimiento y desarrollo con diaforesis, se mantiene alerta y orientado, se realizan estudios de laboratorio y gabinete de rutina
donde se observa nicamente plaquetas con 84,000, se realiza puncion lumbar donde se observa auscencia de eritrocitos, protenas y
glucosa normales, solo se observa 5 monocitos. Como antecedente de importancia refiere regresar de un campamento y realizar
escalado de montaas, actualmente continua en el servicio de medicina interna.

PREGUNTA
Considerando los hallazgos asi como los antecedentes usted integra un diagnostico, y debe iniciar tratamiento, cual es el ms
adecuado:

RESPUESTA
a.- Rifampicina.
b.- Doxiciclina.
MANUAL DE TRABAJO DEL CURSO ENARM CMN SIGLO XXI
CURSO ENARM CMN SIGLO XXI TEL: 36246001 Pharmed Solutions Institute PGINA 123

c.- Vancomicina.
d.- Ceftriaxona.

LISTERIOSIS (LISTERIA MONOCYTOGENES):
CIENCIAS BSICAS: Bacteria Gramm positiva anaerobia, facultativa intracelular, puede causar infecciones invasoras muy graves.
Principales reservorios son el suelo, forrajes el agua, los silos y tracto gastrointestinal en aves, peces y mamferos incluyendo el
hombre. Suele considerarsele un patgeno oportunista. SALUD PUBLICA: Distribucin universal, relativamente resistente a la
refrigeracin, la sequedad y el calor extremo. Afecta a inmunodeprimidos, pacientes con enfermedades crnicas debilitantes mayores
de 60 aos, cirrticos, embarazadas en (en la Ciudad de Mxico 1 por 4000) y neonatos (1 por 1500). Contina causando una elevada
morbilidad y una preocupante mortalidad. PATOGENIA: la infeccin se adquiere mediante el consumo de Alimentos contaminados,
carnes, pescado y vegetales, crudos y lcteos no pasteurizados y productos refrigerados, los recin nacidos la adquieren a travs de la
placenta o del canal de parto infectado. Puede sobrevivir en el entorno gstrico, colonizar el intestino y cruzar la barrera
hematoencefalica y maternofetal, con sus protenas de superficie internalinas, se favorece su invasin celular por un mecanismo tipo
zipper, en que la bacteria se hunde progresivamente en la superficie celular, despus estas se lizan y escapan, la protena
polmerizadora de activa, le permite azucares en el citosol celular durante su replicacin intracelular. DIAGNOSTICO: Clnica; La ms
habitual es la listeriosis gastrointestinal; importante puede causar brotes de toxiinfeccion alimentaria, puede ser asintomtica o sufren
deposiciones acuosas, nauseas, vmitos, cefalea, artromialgias y fiebre, sntomas que se limitan a 2 das, salvo que estn
inmunocomprometidos. Listeriosis invasora; enfermedad gestacional y neonatal, frecuentemente fiebre sin foco aparente, la gravedad
de Listeriosis materna radica en el tercer trimestre, ya que puede haber aborto, muerte fetal intrautero, prematuro dad, sepsis, muerte
neonatal. Es fundamental hacer hemocultivos a toda mujer embarazada con fiebre, aunque semiologa parezca banal. Los neonatos
pueden sufrir sepsis, microabscesos y granulomas diseminados (granulomatosis infantoseptica), infeccin respiratoria o meningitis, que
conlleva una mortalidad y morbilidad importantes como la hidrocefalia persistente en el caso de meningitis. Infeccin del SNC; pueden
tener meningitis, meningoencefalitis o cerebritis. Endocarditis es una rara y grave complicacin de la bacteriemia (forma clnica mas
frecuente en inmunodeprimidos, sin foco identificable, ocurre con fiebre, deteriroro rapido y a menudo fulminante) por listeria. El
diagnostico definitivo es aislamiento en sangre, LCR, liquido articular, placenta. TRATAMIENTO: La mayora de los antibiticos
incluyendo penicilinas son bacteriostticos contra la listeria, los aminoglucosidos, glucopeptidos y cotrimoxaxol son bactericidas.
Neonatos y formas graves; cotrimoxaxol 20mg/kg/da, en 4 dosis o gentamicina 5-7mg/kg/da en dosis nica por 3 das. Infeccin del
SNC ampicilina 400mg/kg/ da en 6 dosis, asociada a gentamicina (si no hay dao renal) o ampicilina y cotrimoxaxol (mas eficacia,
menos nefrotoxicidad). En embarazadas ampicilina mejor opcin.

CASO CLINICO
Femenino de 33 aos con fiebre de una semana de evoluacion, de tipo intermitente, mialgias, artralgia y escalofros, acude a consulta a
primer nivel donde no se integra proceso infeccioso, la paciente se encuentra embarazada con 21 SDG por FUM, a la exploracin se
observa con mucosas orales deshidratadas, durante su ingreso presenta nausea, cefalea y diarrea, al tercer dia se observa con
somnolencia con inversin del ciclo vigilia sueo, posteriormente refiere sensacin de salida de liquido transvaginal, al tacto no se
observa elementos ovulo placentarios.

PREGUNTA
Cual es la conducta teraputica mas adecuada sobre el agente casusal del caso?

RESPUESTA
a.- Ampicilina.
b.- Gentamicina.
c.- Cotrimoxaxol.
d.- Ceftriaxona.

PSEUDOMONA AERUGINOSA (PAE):
CIENCIAS BSICAS: Es una bacteria bacilo Gramm negativo, aerobio estricto, dispersa en el ambiente, emergente relevancia como
patgeno oportunista causante de infecciones en pacientes hospitalizados (equipos de ventilacin mecnica, soluciones de limpieza,
instrumental, medicamentos), crticos y/o inmunodeprimidos (quemados, ventilados, fibrosis qustica, EPOC, pos quirrgicos).m La PAE
causa infecciones en la mayora de las partes del cuerpo, se adapta rpidamente al tracto respiratorio y es el lugar ms frecuente de
infeccin por PAE (principal causante de la neumona asociada a la ventilacin mecnica NAR). SALUD PUBLICA: Diseminada
prcticamente en toda la geografa mundial, es cosmopolita. Tasa de mortalidad atribuible a pseudomoma es de 34%. PATOGENIA:
Factores de riesgo; inmunodeprimidos. La fuente de origen puede ser hematogena o como puerta de entrada en un sitio de puncin o a
travs de vas centrales o catteres de dilisis que pueda tener el paciente al parecer la lesin inicial provocada por la P. aeruginosa al
epitelio respiratorio y otras mucosas esta mediada por pili o fimbrias y por un exapolisacorido mucoide conocido como alginato. Existen
receptores de estas adhesinas en las clulas epiteliales. El microorganismo produce diversas enzimas extra celulares como la proteasa
alcalina, elastasa, fosfolipasa, citotoxina y exoenzimas A y S, la alteracin de los tejidos del husped por estos productos bacterianos,
crea las condiciones necesarias para la proliferacin e invasin bacteriana y la consiguiente destruccin del tejido. Infecciones seas
(fracturas, cirugas traumatolgicas) y articulares (osteomielitis vertebral), por bacteriemia, inoculacin directa dentro del hueso y por
diseminacin contigua desde otro sitio de infeccin. Infeccin en SNC (meningitis, infecciones subdurales o extradurales) secundarias a
ciruga y a trauma de crneo o bacteriemias. Infeccin urinaria secundaria a cuerpo extrao (calculo, atenta, sonda) o a estenosis. Una
situacin importante es la infeccin en tejidos quemados. DIAGNOSTICO: El cuadro clnico es el de un paciente sptico, el nico punto
diferente con otros tipos de sepsis por gramm negativos es la presencia de lesiones cutneas llamadas ectima gangrenoso, que se ve
mas frecuente en pacientes neutropenicos. Lesin maculopapular, pequea, roja, dolorosa, mal circunscrita, que comienza de color
rosa, se oscurece hasta volverse prpura y finalmente negra y necrtica. Debe sospecharse neumona nosocomial en un paciente con
MANUAL DE TRABAJO DEL CURSO ENARM CMN SIGLO XXI
CURSO ENARM CMN SIGLO XXI TEL: 36246001 Pharmed Solutions Institute PGINA 124

uno infiltrado radiolgico nuevo o progresivo asociado a datos clnicos sugestivos de infeccin; fiebre, esputo purulento o leucocitosis,
tras diagnostico sindromatico, intentantar confirmacin microbiologa. . La caracterstica ms importante en laboratorio es la
produccin de pigmento llamado piocianina, que le da un color azulado. Obtencin de material respiratorio (BAL, aspirado traqueal,
minibar) TRATAMIENTO: Las pseudomonas son productoras de enzimas MBL (metalo-b-lactamasas) capaces de resistir prcticamente a
todo tipo de antibitico, por ello su actividad es devastadora. Basndose en esquema para neumonas nosocomiales A) inicio reciente y
sin factores de riesgo, cualquier gravedad: ceftriaxona, fluroquinolonas, ampicilina- sulbactam o ertapenem B) inicio tardo o factores
de riesgo, cualquier gravedad: beta-lactamico antipseudomonico (carbenicilina, ticarcilina, mezlocilina, piperacilina) mas
fluoroquinilona o aminoglucosidos mas linezonid o vancomicina. Antibiticos mas utilizados cfepime (1-2 g cada 8 hrs IV), ceftazidime (
2g c/8hrs IV), imipenem (500mgs c/6 o 1g c/8hrs IV), meropenem (1g c/8hrs), gentamicina (7mg/kg/da IV), amikacina (20 mg/kg/da
IV), levofloxacino (750mg/da IV), colistin 100mg c/8hrs, piperacilina/tazobactam (4.5g c/6hrs IV). No hay un esquema especifico lo que
realmente cambia la mortalidad es el tratamiento emprico inicial adecuado.

CASO CLINICO
Un varn de 27 aos, diabtico, fue internado por fiebre, compromiso de conciencia y rigidez de nuca aparecidos tras tres das de
cursar con una infeccin respiratoria aguda. Se le diagnostic una meningoencefalitis asptica (LCR de aspecto claro, con presin de 12
cm H2O; leucocitos: 200/mm3; tincin de Gram directa y cultivo bacteriano: negativos) y recibi ceftriaxona ms vancomicina y
corticoterapia (dexametasona, 10 mg iv cada 6-horas durante 4 das). Inicialmente ingres en una UCI y estuvo en ventilacin mecnica,
siendo desconectado a los 4 das. Desarroll fiebre y tos productiva tras una semana de evolucin. Una Rx de trax detect un infiltrado
pulmonar bilateral. Tres das ms tarde, una segunda Rx revel la presencia de mltiples abscesos pulmonares con niveles hidro-areos.

PREGUNTA
Cual es la conducta diagnostica mas adecuada?

RESPUESTA
a.- TAC de torax.
b.- Toracostomia.
c.- Cultivo de esputo.
d.- Hemocultivo.

PREGUNTA
Se obtiene cultivo positivo para P. aeruginosa, cual es la conducta teraputica asociada mas apropia?

RESPUESTA
a.- Linezolid.
b.- Meropenem.
c.- Amikacina.
d.- Ciprofloxacina.

LEPTOSPIROSIS (LEPTOSPIRA INTERROGANS):
CIENCIAS BSICAS: Se considera la zoonosis ms frecuente, la leptospira interrogans, es la nica espiroqueta que infecta al hombre,
gramm negativo, aerobio y mvil. Frecuente en campesinos, granjeros, taladores, cazadores, trabajadores de cultivo de arroz y
militares, actualmente tambin en nios (natacin cabotaje, ciclismo, caza deportiva, animales domsticos infectados). PATOGENIA:
Las leptospiras en agua o tierra contaminada por orina de animales infectados, penetran a la piel a travs de cortaduras o abrasiones,
membranas, mucosas y conjuntivas, se disemina a travs de la sangre a todos los rganos y sistemas incluyendo el SNC. Los fenmenos
hemorrgicos son secundarios a vasculitis severa con dao endotelial. Los riones presentan nefritis intersticiales, necrosis tubular y
permeabilidad capilar anormal que acompaa a hipovolemia y puede llegar a IRC. La ictericia es secundaria al necrosis centrolobulillar y
proliferacin de clulas de Kupffer, con disfuncin hepatocelular. Miocarditis y uvetis ocurren por invasin tisular. DIAGNOSTICO:
Clnica; leptospirosis anicterica (85-90%), incubacin de 7-12 das, inicia fiebre, cefalea, mialgias calosfros severos y mal estado general,
linfadenopatias hepatoesplenomegalia. Durante este periodo pueden aislaras las leptospiras en sangre o LCR. Entran en perdidos
asintomtico de 1-3 das e inician ahora la llamada fase inmune o desarrollo de anticuerpos IgM especficos, puede reaparecer la fiebre,
cefalea y vmitos y se puede desarrollar meningitis asptica, existe aumento de clulas en el liquido cefalorraquideo, primero
predominando los PMN y despus los mononucleraes, las protenas estn elevadas, glucosa normal. Puede o no haber irritacin
meninges, esta fase dura de 3-40 das, y ya no se pueden aislar las leptospiras de sangre y/LCR, pero si en orina. Leptospirosis ictericia
(sx. de Weill); forma grave y de mayor mortalidad, los pacientes pueden desarrollar ictericia pero no dao renal, lo usual es que haya
ictericia y elevacin de urea y creatinina, as como elevacin de transaminasas, proteinuria, hematuria, cilindruria, puede haber anemia,
trombocitopenia, leucocitosis, hemorragia, Asi como neumonas hemorrgica. Laboratorio observacin de leptospiras es el estndar de
oro para el diagnostico, la bsqueda en orina es la ms confiable. Las pruebas en suero para deteccin de anticuerpos de la fase aguda
(1-2 semanas) y su elevacin en 4 tantos durante la fase convaleciente (2semanas despus), incluyen la prueba de aglutinacin al
microscopio, confirman diagnostico serolgico. Una sola determinacin de ttulos de 1:800 o mayores sugieren fuertemente la
enfermedad. Analizar LCR en pacientes con meningitis. La radiografa de trax puede mostrar cardiomegalia, edema agudo pulmonar
asociado a miocarditis o infiltrados en parches asociados a hemorragia alveolar por capilaritis. TRATAMIENTO: La penicilina G sdica
(20-24 millones U/da c/4-6hrs), doxiciclina (100mg c/12hrs IV) o eritromicina (500mg c/6hrs IV) Son eficaces para detener la infeccin
y el dao orgnico. La doxiciclina se usa para prevencin en dosis semanal de 200mg.



MANUAL DE TRABAJO DEL CURSO ENARM CMN SIGLO XXI
CURSO ENARM CMN SIGLO XXI TEL: 36246001 Pharmed Solutions Institute PGINA 125

CASO CLINICO
Femenino de 29 aos la cual labora como asistente de veterinario, refiere que hace 10 dias inicio con fiebre, escalofros, cefalea,
nauseas, dolor muscular, se automedica con frmacos sintomticos, mejorando su estado general sin embargo regresan los sntomas y
agregandoce ictericia, a la exploracin clnica se presenta 38.9 C temperatura, FC 1010, TA 140/90 mmHg, la saturacin por oximetro 92
%, a la percusin se presenta dolor heptico, se persibe leve crecimiento, sin esplenomegalia, los estudios de laboratorio BUN de 64,
creatinina 3.6, bilirrubina de 64, AST 86, ALT 103, fosfatasa alcalina de 390, leucocitos 11 000, 13% de bandas y 80 %
polimorfonucleares, hematocrito 33% y plaquetas de 145, puncion lumbar revela pleocitosis, TAC de torax muestra inflitrados en flama
difusos compatibles con hemorragia pulmonar.

PREGUNTA
Cual es diagnostico ms probable?

RESPUESTA
a.- Neumonitis intersticial aguda.
b.- Leucemia mieloide aguda.
c.- Fiebre por Streptobacillus moniliformis.
d.- Infeccion por Leptospira interrogans.

ACTINOMICOSIS (ACTINOMYCES ISRAELII):
CIENCIAS BASICAS: Es causada por bacterias anaerbicas o microaerfilos, gramm positivos principalmente del gnero Actinomyces
(por ejemplo, A. israelii), que est caracterizada por formacin de absceso, fibrosis tisular, las regiones ms afectadas son la
cervicofacial, torcica y plvico-abdominal. El germen tiene una baja virulencia y produce enfermedad solo cuando la barrera mucosa
normal se altera, produciendo formacin de abscesos mltiples, fistulizacin o una masa, requiriendo la presencia de otras bacterias,
que destruyan la vascularidad tisular convirtiendo el anaerbica (generalmente infecciones polimicrobianas). Actinomicosis se asocia
con mala higiene dental, absceso dentario, uso de dispositivos anticonceptivos intrauterinos (DIU), y la inmunosupresin. SALUD
PBLICA: Su incidencia est disminuyendo, probablemente como resultado de una mejor higiene dental y un inicio ms temprano del
tratamiento antibitico. PATOGENIA: Los agentes de actinomicosis son miembros de la flora oral normal. La enfermedad se presenta
slo despus de la interrupcin de la barrera de la mucosa. Infeccin local se extiende en forma contigua en una manera lenta y
progresiva, haciendo caso omiso de los planos del tejido. En crecimiento in vivo produce grumos llamados granos o grnulos de azufre.
Las paredes fibrticas de la masa a menudo se describen como "madera. DIAGNOSTICO: La enfermedad oral - cervicofacial: La
infeccin comienza como una inflamacin de los tejidos blandos, abscesos, o masa, a menudo en el ngulo de la mandbula con
extensin contigua al cerebro, espina dorsal cervical, o en el trax. El dolor, la fiebre, y leucocitosis son variables. Enfermedad torcica:
El parnquima pulmonar y / o en el espacio pleural suele participar. Produce dolor en el pecho, fiebre y prdida de peso. Radiografa de
trax muestra una lesin de masa o la neumona. Enfermedad cavitaria o adenopata hiliar pueden ocurrir, y > 50 % de los puntos
tienen engrosamiento pleural, derrame o empiema. Las lesiones cruzan fisuras o pleura y pueden implicar el mediastino, el hueso
contiguo, o la pared torcica. Enfermedad abdominal: El diagnstico es difcil y no se puede hacer hasta meses despus del evento
inicial (por ejemplo, diverticulitis, ciruga intestinal). La enfermedad generalmente se presenta como un absceso, masa o lesin fijado al
tejido subyacente y, a menudo se confunde con el cncer. Fstulas en la pared abdominal, en la regin perianal, u otros rganos pueden
desarrollar e imitar la enfermedad inflamatoria intestinal. Participacin del tracto urogenital puede presentarse como pielonefritis o
absceso perirrenal. Actinomicosis plvica se asocia a menudo con DIU. La presentacin es indolente y puede seguir a la extraccin del
dispositivo se presenta fiebre, prdida de peso, dolor abdominal y sangrado vaginal anormal. Endometritis progresa a masas plvicas o
abscesos tuboovrico. Actinomicosis pueden afectar el tejido msculo-esqueltico, tejido blando, o el SNC y puede difundir va
hematgena, ms comnmente a los pulmones y el hgado. Aspiraciones, biopsias, o la escisin quirrgica puede ser necesaria para
obtener el material para el diagnstico. La identificacin microscpica de los grnulos de azufre en pus o tejidos establece el
diagnstico. Necrosis central de las lesiones con los neutrfilos y los grnulos de azufre es prcticamente de diagnstico de la
enfermedad. Por lo general requieren de 5-7 das, pero puede tardar 2-4 semanas para convertirse en positiva, incluso una sola dosis de
antibitico puede afectar el rendimiento de los cultivos. TRATAMIENTO: Requiere un tratamiento prolongado. Tratamiento IV durante
2-6 semanas (por lo general con la penicilinas- amoxicilina, penicilina G, ampicilina) seguido de tratamiento oral durante 6-12 meses
(por ejemplo, con penicilina o ampicilina) se sugiere para la infeccin grave y enfermedad voluminosa. En particular la enfermedad de la
regin oral y cervicofacial, se puede curar con un tratamiento ms corto. En alrgicos eritromicina o azitromicina. Si el tratamiento se
prolonga ms all del punto de la resolucin de la enfermedad medible (cuantificada mediante TAC o RM), se minimiza la recada.
Agentes alternativos adecuados incluyen las tetraciclinas (por ejemplo, minociclina, 200 mg/da administrado por va IV o PO cada 12
horas) o clindamicina (2,7 g /d administrado por va IV cada 8 horas).

CASO CLINICO
Mujer de 53 aos, con antecedentes de amigdalectoma en la infancia y sin manipulaciones dentales recientes. Consult por sensacin
de cuerpo extrao farngeo derecho de 1 ao de evolucin, con expulsin por boca de material blanquecino al menos en tres ocasiones.
El otorrinolaringlogo extrajo un cmulo de detritus del lecho amigdalar derecho, y apareci en la zona adyacente una pared de
aspecto inflamatorio. El anlisis anatomopatolgico de la muestra inform de masa (1,51,51cm). Durante los 6 meses posteriores de
seguimiento present tres episodios de expulsin con la tos, de masas de aproximadamente 1cm de iguales caractersticas. A la
exploracin fsica destacaba la visualizacin repetida de un cmulo de detritus blanco en una cavidad en el polo superior del lecho
amigdalar derecho, cavidad atribuible a una cicatrizacin anmala de la amigdalectoma previa. La higiene bucal era buena. La analtica
bsica y la radiografa de trax eran normales. Serologa para el virus de la inmunodeficiencia humana, negativa. La tomografa
computarizada cervical identific una prominencia inespecfica a nivel amigdalar derecho, sin objetivar trayectos en partes blandas ni
adenopatas

MANUAL DE TRABAJO DEL CURSO ENARM CMN SIGLO XXI
CURSO ENARM CMN SIGLO XXI TEL: 36246001 Pharmed Solutions Institute PGINA 126

PREGUNTA
Se realiza cultivo y es positivo para actinomyces israelii, cual es la conducta mas adecuada para minimizar recaidas?

RESPUESTA
a.- Amoxicilina.
b.- Clindamicina.
c.- Penicilina.
d.- Tetraciclina.

PREGUNTA
Cual de los siguientes factores de riesgo es mas frecuente para la presencia actinomicosis craneofacial?

a.- Otitis media.
b.- Sinusitis cronica.
c.- Absceso dentario.
d.- Amigdalas hipertrficas.

PREGUNTA
En el caso de la actinomicosis abdominopelviana, cual de los siguientes factores es el mas frecuente?

RESPUESTA
a.- Cuerpo extrao.
b.- Aborto sptico.
c.- EPI.
d.- LUI.

NOCARDIASIS (N. ASTERIODES, N. BRASILIENSIS):
CIENCIAS BASICAS: Nocardias son actinomicetos aerbicos saprfitos comunes en el suelo. Varias especies estn asociadas con la
enfermedad humana. N. asteroides es la especie ms comnmente asociada con la enfermedad invasiva. N. brasiliensis se asocia ms
con lesiones cutneas localizadas. SALUD PBLICA: En los Estados Unidos, 1.100 casos de infeccin por nocardia se producen cada ao,
de los cuales 85 % son pulmonares o sistmicos. El riesgo de enfermedad es mayor de lo habitual entre las personas con deficiencia de
inmunidad mediada por clulas, por ejemplo, que asocia con el linfoma, el trasplante, la terapia con glucocorticoides, o SIDA. Las tasas
de mortalidad son altas entre los pacientes con nocardiosis del cerebro. PATOGENIA: La neumona y la enfermedad diseminada siguen
a la inhalacin de micelios. Nocardiosis causa abscesos con infiltracin de neutrfilos y necrosis. DIAGNOSTICO: Clnico; Enfermedad
pulmonar suele ser subaguda, presentando en das o semanas. Enfermedad extrapulmonar se documenta en > 50 % de los casos, y un
poco de afectacin pulmonar es evidente en el 80 % de los pacientes con enfermedad extrapulmonar. Una tos productiva importante
de pequeas cantidades de esputo espeso purulenta, fiebre, anorexia, prdida de peso y malestar general son comunes, disnea,
hemoptisis y dolor pleurtico son menos comunes. La radiografa de trax; suele mostrar infiltrados nodulares nicos o mltiples de
diferentes tamaos que tienden a cavitacin. Empiema se observ en un tercio de los casos. La infeccin puede diseminarse a los
tejidos adyacentes, como el pericardio o el mediastino. Enfermedad extrapulmonar normalmente se manifiesta como abscesos
subagudos en el cerebro, la piel, los riones, los huesos y / o musculares. Algunos abscesos forman fstulas y pequeas cantidades de
descarga de pus, pero no las de los pulmones o el cerebro. Los abscesos cerebrales son generalmente supratentorial, a menudo
multiloculada, pueden ser nicos o mltiples y tienden a enterrarse en los ventrculos o extender en el espacio subaracnoideo.
Enfermedad despus de la inoculacin transcutnea Celulitis: La celulitis subaguda puede presentar 1-3 semanas despus de una lesin
en la piel (a menudo contaminada con tierra). Sndrome linfocutneo: Una lesin piodrmica se desarrolla en el sitio de inoculacin,
con ulceracin central y secrecin purulenta. Esta lesin suele drenada por ndulos SC a lo largo de los vasos linfticos. Esta forma se
asemeja a la esporotricosis. Actinomicetoma: Son formas nodulares de hinchazn en el sitio de traumatismo local, por lo general en los
pies o las manos. Forma de fstulas y la descarga de drenaje seroso o purulento que puede contener grnulos que consisten en masas
de micelio. Las lesiones, que se extienden poco a poco a lo largo de los planos faciales involucrar a la piel, tejido subcutneo y hueso,
pueden provocar graves deformidades. Infecciones oculares: queratitis por lo general sigue a un trauma ocular. La endoftalmitis puede
ocurrir despus de la ciruga del ojo o en la enfermedad diseminada. Exmenes en los que se identifica a la bacteria; broncoscopia,
cultivo de esputo o pus (en agar enriquecido con extracto de levadura y carbn activado), se pueden ver filamentos gramm positivos,
biopsia de piel, cerebro, pulmn (tambin aspiracin pulmonar percutnea, lavado broncoalveolar). TRATAMIENTO: Las sulfonamidas
son los frmacos de eleccin, Trimetropim (10-20mg/kg) y sulfametoxaxol (50-100mg/kg). Para enfermedad grave, los niveles sricos
de sulfonamidas deben ser monitoreadas y mantenidas a 100-150 mg /mL. Una vez que la enfermedad est controlada, la dosis de
trimetoprim-sulfametoxazol puede ser reducida por 50%. Las pruebas de sensibilidad pueden guiar tratamientos alternativos.En
enfermedad del SNC; minociclina (200-400mg). Sindrome de celulitis linfocutanea; linezolid (1200mg). Actinomicetoma amikacina (10-
15mg/kg), cefotaxima (6g), ceftriaxona (1-2g), imipenem (2g). La terapia para la nocardiosis debe continuar mientras el paciente
permanezca inmunodeprimido. Los abscesos cerebrales que son grandes o no responde a los antibiticos deben ser aspiradas. Las
recadas son comunes. El paciente debe ser seguido por al menos 6 meses despus de la terapia se haya completado.

CASO CLINICO
Mujer de 42 aos, diestra, portadora de poliglobulia diagnosticada 15 aos antes y catalogada como primaria luego de exhaustivos
estudios. Tratada con cido acetil saliclico en las ocasiones que se le realiz flebotomas, tuvo mala tolerancia a las mismas. Un mes
previo fue sometida a un procedimiento de extraccin e implante dentario, presentando 10 a 15 das despus un cuadro de inicio y
progresin insidiosos, en el curso de 15 das, y caracterizado por cefalea, vmitos, fiebre, a lo que se agregaron posteriormente
MANUAL DE TRABAJO DEL CURSO ENARM CMN SIGLO XXI
CURSO ENARM CMN SIGLO XXI TEL: 36246001 Pharmed Solutions Institute PGINA 127

alteraciones de la agudeza visual y campimtricas y en las ltimas cuarenta y ocho horas, depresin de la vigilia. En el examen fsico, la
paciente se presentaba confusa, por momentos excitada, con guio a la amenaza abolido a izquierda, paresia de VI par izquierdo,
parlisis facial central izquierda, paresia leve de miembros izquierdos, con hiporreflexia y signo de Babinski. En la paraclnica de
laboratorio el hematocrito fue de 6,79 %, hemoglobina 20,9 mg/dL, VCM 90,1, HBCM 30,7%, leucocitos 12,800/mm3, PMN 80%,
linfocitos 17%, eosinfilos 1%, basfilos 2%, plaquetas 150/mm3, VES 1 mm 1 hora
PREGUNTA
Cual es la conducta diagnostica mas adecuada a seguir?

RESPUESTA
a.- Hemocultivo.
b.- IRM crneo.
c.- TAC crneo.
d.- Serologia

PREGUNTA
La RM con gadolinio evidencia proceso expansivo cortico-subcortical con anillo grueso irregular de realce con centro necrtico. La
orientacin diagnstica imagenolgica RM orient a un absceso cerebral, se realizo drenaje y cultivo, que resulto positvo a Norcardia
asteroides, cual es la conducta teraputica mas adecuada?

RESPUESTA
a.- Linezolid (1200mg).
b.- Minociclina (200-400mg).
c.- Amikacina (10-15mg/kg)
d.- TMP/SMX (3.200/640mg)

INFECCIONES ESTREPTOCOCICAS:
CIENCIAS BSICAS: Son un genero de los ms importantes como patgeno para el ser humano, son bacterias esfrica, erticos
facultativos, no forman esporas, catalana negativos e inmviles. El Streptococcus pyogenes, es el agente etiolgico mas frecuente de
faringoamigdalitis aguda, pudiendo ocasionar secuelas no supurativas, tales como la fiebre reumtica y la glomerulonefritis post-
estreptoccicas. Otro el Streptococcus pneumoniae. De acuerdo con el tipo de hemolisis de eritrocitos que producen, en beta
(hemolisis completa), alfa (hemolisis parcial) y gamma (sin hemolisis). La primera la presentan prcticamente todos los S. pyogenes y la
segunda los S. pneumoniae y viridans. SALUD PUBLICA: S. pyogenes incidencia mayor en raza blanca, en adolescencia y sexo
masculino. La faringoamigdalitis ms frecuentes en zonas templadas y fras y en invierno, mientras el imptigo y la piodermitis lo son en
los climas tropicales y en meses calurosos. PATOGENIA: La fuente de infeccin son las secreciones de nariz, garganta y piel de
enfermos, la transmisin se realiza persona a persona, mediante secreciones respiratorias al toser o estornudar o por transferencia
manual (piel con traumatismo previo). Incubacin de 2-4 das, la fase aguda constituye el periodo de mayor contagiosidad. Ingresa a
vas respiratorias superiores y se disemina, tiene especial afinidad por el sistema linftico y el lugar de implantacin es el tejido linfoide
de faringe. Se extiende por los linfticos o por continuidad a otras reas cercanas, causando complicaciones tales como adenitis
cervical, absceso periamigdalino o retrofaringeo, sinusitis, otitis. Para el desarrollo de escarlatina, la infeccin farngea es por una cepa
productora de toxina eritrogenica, la cual es antignica y estimula produccin de anticuerpos protectores (por eso solo da un cuadro).
ESTREPTOCOCOS BETA HEMOLITICO DEL GRUPO A o Streptococcus pyogenes: Tiene una estructura celular compleja, su protena M
parece ser su principal factor de virulencia, las cepas ricas en ellas son resistentes a la fagocitosis. Elabora productos extracelulares; la
toxina eritrogenica es la responsable del exantema en la escarlatina, su produccin es inducida por un bacteriofago. La estreptomicina
O y la S ambas txicas para los eritrocitos y otras clulas, incluso miocardio. La elevacin de niveles serios de anti estreptomicina O. Es
un buen indicador de infeccin estreptoccica reciente. Por evidencia seroepidemiologica, se considera que ciertos serotipos,
causantes de faringoamigdalitis, estn ms relacionados con fiebre reumtica, los de piel no causan fiebre reumtica pero ambos son
nefritogenicos (glomerulonefritis). DIAGNOSTICO: Clnica; faringoamigdalitis, forma txica con fiebre alta, nausea y vmitos, comienzo
agudo, odinofagia, cefalea, malestar, faringe hiperemica, exudado blanco-amarillento en parches, adenopatas anterior dolorosa.
Escarlatina; fiebre elevada, vomito, dolor farngeo y cefalea, 12-48 hrs despus aparece exantema y el enantema (lengua=fresa roja,
paladar, amgdalas y faringe) caracterstico. El rash se inicia en el tronco y se generaliza con rapidez, se presenta con una erupcin con
fondo eritematosa y a la vez puntico ene, con seccin tctil de aspereza (papel de lija), respetando la cara. Siempre se presenta la
descamacin en mayor o menor grado. Imptigo; vescula superficial con poco eritema e indolora, progresiva pstula con costra gruesa,
con secrecin meliserica, mas en extremidades y comisuras nasales y bucales. Erisipela; poco frecuente, celulitis caracterizada por una
lesin eritematosa, con engrosamiento de la piel, dolorosa y con aumento de temperatura, borde bien definido. Otras infecciones son;
piodermitis, ectima, celulitis o fscitis necrozante. El diagnostico de faringoamigdalitis se fundamenta en la triada amgdalas con
exudado, odinofagia y adenopatias cervical anterior dolorosa, en un paciente que NO presenta tos, o sntomas nasales. El dx., de
certeza aislamiento de S. pyogenes en cultivo de exudado farngeo, aglutinacin en ltex y coaglutinacion, detectan antgeno. En la
escarlatina el rash, y se puede hacer cultivo y antiestreptolisinas. TRATAMIENTO: Debido a que S. pyogenes sigue siendo sensible a
penicilina, esta es el tx., de eleccin. Nios menores de 4 aos; penicilina procainica 400,000UI c/24 hrs 2-3 das + p. Benzatinica
600,000UI, en una sola dosis, despus de la procainica o penicilina benzatinica 600,000UI dosis nica. Nios mayores de 4 aos y
adultos; penicilina procainica 800, 000UI c/24hrs 2-3 das + P. Benzatinica 1,200,000UI dosis nica despus de la procainica o penicilina
benzatinica 1,200,000UI dosis nica. El medicamento sustituto en alergia a penicilinas es eritromicina oral 30mg/kg/da, en dos o tres
dosis diarias durante 10 das (dosis mxima 2 gramos). En caso de miosotis y fascitis necrozante, se recomienda penicilina, clindamicina
y amikacina , hasta corroborar agente, si es pyogenes continuar solo penicilina y clindamicina.


MANUAL DE TRABAJO DEL CURSO ENARM CMN SIGLO XXI
CURSO ENARM CMN SIGLO XXI TEL: 36246001 Pharmed Solutions Institute PGINA 128

CASO CLINICO
Varn con 43 aos de edad, diabtico sin control, que 24 horas antes present una contusin accidental contra el respaldo de la cama
en la zona periorbitaria derecha, por lo que acudi a consultorio, donde se le realiz una curacin y prescribi tratamiento con anti-
inflamatorios (diclofenaco sdico). En las horas siguientes curs con fiebre, compromiso de conciencia y vmitos. Al examen fsico se
describi un paciente de aspecto txico, con los siguientes signos vitales: frecuencia cardaca: 130/min, frecuencia respiratoria: 49/min,
presin arterial: 77/55 mm Hg, T rectal: 37,8 C, escala de Glasgow neurolgico: 14. Destacaba un gran edema periorbitario bilateral
con eritema y calor en la zona palpebral y en la mejilla derecha. Se hospitaliz con el diagnstico de celulitis preseptal, inicindose
tratamiento con dicloxacilina endovenosa.

PREGUNTA
Cual de los siguientes agentes infecciosos es mas probable aisla en este caso?

RESPUESTA
a.- Estreptococos del grupo A
b.- Estreptococos del grupo B
c.- Estreptococos del grupo C
d.- Estreptococos del grupo D

CASO CLINICO
Varn de 62 aos, sin antecedentes clnicos de inters, que ingres por presentar en las 48-72 h previas un cuadro de fiebre elevada,
sudoracin profusa, escalofros, mialgias y posteriormente cefalea frontooccipital intensa, postracin y hematuria macroscpica. No
presentaba sntomas gastrointestinales, infecciosos, traumatismos ni intervenciones quirrgicas previas. En la exploracin fsica
destacaba rigidez de nuca y afectacin del estado general. No se encontraron signos de afectacin cardiovascular ni digestiva. En los
estudios analticos destacan la presencia de leucocitosis con desviacin izquierda, inicio de patrn analtico de coagulacin intravascular
diseminada (CID) y leucocituria con nitritos positivos en orina. Se realiz puncin lumbar, obtenindose lquido claro a presin normal
cuyo examen mostr: leucocitos 1.200 cl./ml, protenas 8 g/l, glucosa 9 mg/dl, Gram: se identificaron cocos grampositivos en cuyo
cultivo se aisl S. bovis biotipo II

PREGUNTA
Cual de los siguientes grupos agentes infecciosos es mas probable aisla en este caso?

RESPUESTA
a.- Estreptococos del grupo A
b.- Estreptococos del grupo B
c.- Estreptococos del grupo C
d.- Estreptococos del grupo D

CASO CLINICO
Mujer de 59 aos con cuadro de 72 horas de evolucin con astenia, anorexia y deterioro del nivel de conciencia que se acompaaba en
las ltimas 24 horas de disnea. Se observo con insuficiencia respiratoria parcial y una fibrilacin auricular con respuesta ventricular
rpida e inestabilidad hemodinmica secundaria motivo por el que se decidi el traslado a la Unidad de Cuidados Intensivos (UCI).
Ingresa a UCI obnubilada, hipotensa (tensin arterial: 90/60 mmHg), en fibrilacin auricular a 130 lpm, taquipnica a 30 rpm y con
saturacin arterial de oxigeno de 90% segn pulsioximetra respirando con mascarilla al 50% de fraccin inspirada de O2. En la
exploracin fsica destac nicamente la existencia de ingurgitacin yugular bilateral e hipoventilacin en la base pulmonar izquierda. A
nivel analtico llam la atencin la existencia de leucocitosis con 15.300 leucocitos, neutrofilia y una importante desviacin izquierda.
Electrocardiograma con fibrilacin auricular rpida sin alteraciones de la repolarizacin. Radiografa de trax con cardiomegalia y
borramiento de ambos hemidiafragmas. Se canaliz un catter venoso central y un catter PICCO, objetivndose un patrn
hemodinmico compatible con sepsis (gasto cardiaco elevado con resistencias vasculares sistmicas disminuidas) y unos valores de
presin venosa central elevados.

PREGUNTA
Cual de los siguientes grupos agentes infecciosos es mas probable aisla en este caso?

RESPUESTA
a.- Estreptococos del grupo A
b.- Estreptococos del grupo B
c.- Estreptococos del grupo C
d.- Estreptococos no A no B.

CASO CLINICO
Paciente de 35 aos, sexo femenino, no fumadora, fue sometida a tiroidectoma total por neoplasia folicular de tiroides y fue dada de
alta en buenas condiciones generales a las 48 h. Consult 5 dias despues por malestar general, odinofagia, cefalea, sensacin febril,
escalofros, dolor cervical, tos productiva y expectoracin mucopurulenta, siendo admitida a sala de cuidados generales. En el examen
fsico destacaba paciente en regulares condiciones generales, lcida, orientada, FC: 90 lat/min, PA: 106/60 mmHg, FR: 24 resp/min, T:
37,8C. Dolor a la palpacin en la zona de herida operatoria de la regin cervical anterior, faringe sana, examen cardiopulmonar y
abdominal normales. Se plante el diagnstico de infeccin de herida operatoria y se inici tratamiento con ceftriaxona 1 g/da EV. La
MANUAL DE TRABAJO DEL CURSO ENARM CMN SIGLO XXI
CURSO ENARM CMN SIGLO XXI TEL: 36246001 Pharmed Solutions Institute PGINA 129

paciente evolucion febril y con dificultad respiratoria progresiva, requiriendo aporte de oxgeno al 35% para corregir el trastorno del
intercambio gaseoso (Pa02/Fi02: 259). En el hemograma destacaba leucopenia (2.700/mm3) con desviacin a izquierda (13%
baciliformes), PCR: 5,4 mg/dL, GSA: pH: 7,46, Pa02: 54,4 mmHg, Sa02: 89%, PaC02: 27,5 mmHg, B. act: 19,6 mEq/L. La radiografa de
trax mostr opacidades parenqui-matosas en lbulo superior derecho, lbulo medio, lngula y lbulo inferior izquierdo.

PREGUNTA
Cual de los siguientes grupos agentes infecciosos es mas probable aisla en este caso?

RESPUESTA
a.- Estreptococos del grupo A
b.- Estreptococos del grupo B
c.- Estreptococos del grupo C
d.- Estreptococos no A no B.

MANUAL DE TRABAJO DEL CURSO ENARM CMN SIGLO XXI
CURSO ENARM CMN SIGLO XXI TEL: 36246001 Pharmed Solutions Institute PGINA 130

INFECCIONES POR ESTAFILOCOCOS: CIENCIAS BASICAS: Los estafilococos son cocos gram-positivos que forman racimos de uvas en la
tincin de Gramm, son catalasa positivo (a diferencia de los estreptococos), no mviles, aerbicos y anaerobias facultativas.
Staphylococcus aureus, se distingue de otros estafilococos por su produccin de coagulasa, es la especie ms virulentas, causando la
enfermedad a travs de ambos mecanismos no mediadas por toxinas y mediada por toxina. Los estafilococos coagulasa negativos
(ScoN) son menos virulentas que S. aureus, pero son causas importantes y comunes de las infecciones de dispositivos de prtesis. S.
epidermidis con mayor frecuencia causa la enfermedad, este organismo es un componente normal de la piel, orofarngea, y la flora
vaginal. S. saprophyticus es una causa de las infecciones urinarias. Otras dos especies de ScoN, S. lugdunensis y S. schleiferi, son ms
virulentas y causan infecciones graves tales como vlvulas nativas endocarditis y osteomielitis. SALUD PBLICA: S. aureus es
responsable 25 a 35 % de los casos de endocarditis bacteriana. Las
tasas de mortalidad oscilan entre 20 a 40% a pesar de la
disponibilidad de antibiticos eficaces. PATOGENIA: Una vez que se
presenta la infeccin, la invasin local y sistmica ocurre por va
hematgena y liberacin de toxinas. Localmente los organismos
pueden invadir o necrosar el tejido y ocasionar una potente
respuesta inflamatoria (como resultado de abrasiones, quemaduras,
herida, inyecciones o a travs de catteres), mediada
principalmente por PMN (respuesta primaria). La formacin de
abscesos (microcpsula polisacrido antifagoctica, facilita la
evasin de las defensas del husped, til en la formacin de
absceso) es comn, con un centro necrtico que est formado de
pus y una pared de fibrina, que dificulta la penetracin de
antibiticos, y protege de defensas del husped, las bacterias se
adhieren formando una biopelcula similar a la formada por los
ScoN. La infeccin puede diseminarse localmente por la formacin
de trayectos sinuosos y abscesos secundarios. La diseminacin
hematgena puede infectar articulaciones, vlvulas cardiacas y
otros tejidos. Para el desarrollo del choque sptico los monocitos y
macrfagos tienen el papel principal, liberan TNF , IL-1, IL-6, IL-8,
posterior al contacto con el estafilococo, peptidoglocano o acido
lipoteicoico, en respuesta a esto las vas del complemento y
coagulacin se activan, se metaboliza el acido araquidnico y le
libera el factor de activacin de plaquetas; provocndose
hipotensin, fiebre, fuga capilar, CID y disfuncin multiorgnica. Los
estafilococos coagulasa negativos (ScoN), se adhieren a cuerpos
extraos (a travs del acido teicoico en su pared celular y la
produccin de slime que recubre el material extrao) y provocan
infecciones difciles de tratar (flebitis hasta bacteriemia y choque
sptico) sin retirar el material protsico (catteres). La bacteriemia por estafilococo puede complicarse con endocarditis o sndrome
sptico. El estafilococo se adhiere vidamente a la clula endotelial a travs de interacciones adhesina-receptor. Algunos productos de
estafilococos son considerados como superantigenos, que provocan enfermedades graves. La toxina-1 del sndrome de choque toxico,
se une a todas la regiones variables de la clula. DIAGNOSTICO: Infecciones de piel y tejidos blandos; imptigo-infeccin superficial de
piel, buloso (30%) o no buloso (70%), este inicia con una ppula eritematosa, que se presenta en lesiones previas por picadura de
insectos, varicela, eczema, trauma o abrasiones, la ppula progresa a vescula y despus a costra mielisrica, la topografa de las
lesiones es alrededor de nariz y boca raro en extremidades. Foliculitis- pequeas ppulas o pstulas centradas en el folculo piloso y
rodeado de eritema. Fornculos-ndulo profundo y doloroso de color rojizo con formacin de pus, forma pequeos abscesos por la
necrosis del aparato polisebaceo. Paroniquia-infeccin que rodea el borde una ua de mano o pie. Celulitis, matitis, infecciones de
heridas quirrgicas, conjuntivitis purulenta, endoftalmitis. Endocarditis infecciosa-en asociacin con el uso de drogas inyectables,
vlvula protsica, nosocomial, los pacientes tienen fiebre alta, aspecto txico, y dolor torcico pleurtico y producir esputo purulento
que a veces sangriento. La radiografa de trax puede revelar embolia sptica: lesiones pequeas y perifricas. Pericarditis como
consecuencia de una diseminacin contigua de una infeccin respiratoria, es rara. Infecciones en huesos y articulaciones; osteomielitis
derivada de la diseminacin hematgena o propagacin contigua desde un sitio de tejido blando (por ejemplo, lceras diabticas o
vasculares). Osteomielitis hematgena en nios frecuentemente en la metstasis de huesos largos y se presenta con fiebre, dolor de
huesos, y la renuencia a soportar peso. Leucocitosis, aumento de la VSG y hemocultivos positivos son tpicos. Osteomielitis hematgena
en los adultos es a menudo vertebral. La osteomielitis por infecciones de tejidos blandos contiguos se sugiere por la exposicin del
hueso, un trayecto fistuloso drenaje, la falta de cura, el drenaje continuo. S. aureus es tambin causa comn de artritis sptica,
asociada con el trauma o ciruga, o se debida a la diseminacin hematgena, hacer hemocultivo y puncin articular, la evaluacin de
liquido sinovial debe incluir cultivo, tincin de gramm, cuenta celular (75% de PMN), glucosa (2/3 de la srica) y protenas. Infecciones
respiratorias; otitis media, traquetis bacteriana, neumona nosocomial-se produce principalmente en pacientes intubados, en unidades
de cuidados intensivos, tienen un mayor volumen de esputo purulento, fiebre y nuevos infiltrados pulmonares y puede desarrollar
dificultad respiratoria, rara neumona de la comunidad. Infecciones en SNC; meningitis-por extensin de un foco paramenngeo,
durante la bacteriemia se puede producir cerebritis y posteriormente formacin de un absceso cerebral. La incidencia de bacteriemia
durante la siembra metastsica se ha estimado para ser tan alta como 31 %. Los huesos, las articulaciones, los riones y los pulmones
son los ms comnmente infectados. Las infecciones por SARM pueden tener muchas presentaciones inusuales (por ejemplo, la fascitis
necrotizante, neumona necrtica, sepsis, prpura fulminante) que reflejan el aumento de la virulencia de las cepas CA MRSA.
ENFERMEDADES MEDIADAS POR TOXINAS; El sndrome de shock txico (TSS): Se presentaba en mujeres menstruando que utilizaban
tampones de gran absorcin. En la actualidad se conoce que cualquier paciente con infeccin por estafilococo puede presentar TSS, as
DEFINICIONES CLINICAS PARA SINDROME DE CHOQUE TOXICO
Temperatura > 38.9
Exantema macular difuso
Descamacin 1-2 semanas posteriores al inicio de la enfermedad
(+en palmas y plantas)
Hipotensin: Presin sistlica <90mmHg. Cada ortosttica de la
presin diastlica > 15mmHg de acostado a sentado, sincope
ortosttico o mareo.
Afeccin multisistmica: 3 o ms de las siguientes
Gastrointestinal; vomito o diarrea al inicio de la enfermedad
Muscular; mialgia severa o elevacin de CPK, 2 veces arriba de lo
normal
Mucosas; hiperemia en conjuntiva, orofaringe, vagina
Renal; nitrgeno ureico o creatinina por lo menos dos veces arriba
de lo normal o sedimento urinario con piuria (>5 leucos x campo),
en ausencia de infeccin de vas urinarias
Heptica; bilirrubina total, transaminasas 2 veces por arriba de lo
normal
Hematolgica: plaquetas <100,000/mm
3
SNC; desorientacin o alteraciones en la conciencia sin focalizacin,
en ausencia de fiebre o hipotensin
Criterios de laboratorio: Estudios con resultado negativo:
Cultivos de sangre, faringe, LCR (excepto positivo en sangre para S.
aureus)
Ttulos de anticuerpos para leptospira o antiestreptolisinas o
sarampin
Clasificacin de caso:
Probable; 5 de los 6 datos clnicos descritos
Conformado; Los 6 datos clnicos descritos anteriormente,
incluyendo descamacin, a menos que el paciente fallezca antes de
la descamacin

MANUAL DE TRABAJO DEL CURSO ENARM CMN SIGLO XXI
CURSO ENARM CMN SIGLO XXI TEL: 36246001 Pharmed Solutions Institute PGINA 131

en la actualidad <50% de casos reportados se relacionan al uso de tampones y alrededor de 10% se relacionan a diversas infecciones
por estafilococo (empiema, bacteremia, abscesos, infecciones quirrgicas). Ms comn en mujeres jvenes. El sndrome se relaciona
con la enterotoxina F o exotoxina pirognica C o toxina-1 del sndrome de choque toxico, pero las endotoxina A a la E pudieran
contribuir a la presentacin del sndrome; puede manifestarse con signos y sntomas leves hasta una enfermedad fatal rpidamente
progresiva, inicio sbito de exantema difuso (difuso, escalatiniforme, predomina en aras de flexin), fiebre, alta, vomito, diarrea,
cefalea, faringitis, mialgias, alopecia, hipotensin, falla multiorgnica, por la pobre perfusin tisular y dao directo de los mediadores,
falla hematolgica. Los cambios hemodinmicas incluyen disminucin en el tono vasomotor y fuga no hidrosttica del liquido del
espacio intravascular al intersticial. Las complicaciones que pueden llevar a la muerte son choque refractario, falla renal oligrica,
arritmia ventricular, CID y pulmn de choque. La intoxicacin alimentaria: La inoculacin de la toxina de S. aureus, por los
manipuladores de alimentos, coloniza los alimentos. La toxina crece despus en los alimentos que promueven el crecimiento, como la
crema, ensalada de papas, jamn, pollo o carne procesada. La toxina estable al calor no se destruye. Inicio de la enfermedad es rpida y
explosiva, que ocurre dentro de 1-6 h de la ingestin de alimentos contaminados. Los principales signos y sntomas son nuseas y
vmitos, pero pueden ocurrir diarrea, hipotensin y deshidratacin. La fiebre est ausente. Los sntomas se resuelven en 8-10 h. El
sndrome de piel escaldada estafiloccica (SSSS): Tambin conocida como enfermedad de Ritter, caracterstico la piel se desprende
como si se hubiese quemado, afecta ms frecuentemente a los recin nacidos y los nios. La piel es frgil, se puede formar bullas de
pared delgada que contienen lquido, su manejo es control de electrolitos y lavado de manos estricto as como antibiticos. El signo de
Nikolsky la friccin leve sobre la piel produce un desprendimiento de las capas superficiales de la piel. La bula tpica se rompe deja una
rea denudada dejando una superficie hmeda y finalmente ocurrir descamacin. TRATAMIENTO: xito inicial con penicilinas
semisintticas resistentes a las penicilinasas como meticilina, oxacilina, nafcilina, cloxacilina y dicloxacilina. Estos agentes fueron
utilizados ampliamente, hasta la aparicin de cepas resistentes a meticilina (SARM), actualmente se ha descontinuado su uso debido a
que produce nefritis intersticial. La nafcilina y oxacilina se usan en otros pases. En Mxico, la dicloxacilina (250mg a 1 gr en adultos y de
50-400mg/kg/da en nios dividido en 4 dosis), es uno de los frmacos de primera eleccin para el tratamiento de estafilococos. Una
alternativa son las cefalosporinas de primera generacin (cefalozina, cefalotina), segunda generacin (cefuroxima). En infecciones
severas se utiliza la terapia combinada, generalmente con un aminoglucosido para sinergia. Los antibiticos combinados con un
inhibidor de beta-lactamasa (ac. Clavulanico, sulbactam, tazobactam), tambin pueden utilizarse en el tratamiento para infecciones por
estafilococo sensible a meticilina, pero no son la primera eleccin. La vancomicina no debe utilizarse en forma rutinaria, porque es caro,
toxico y requiere una estrecha vigilancia por efectos secundarios, lo ms importante es que favorece la aparicin de cepas resistentes.
A pesar de los reportes de resistencia a meticilina, en infecciones comunitarias por S. aureus, donde se sospeche que la resistencia es
elevada, debern emplearse otras alternativas como clindamicina o trimetropim/sulfametoxaxol. Es indispensable, que se tomen
cultivos antes del inicio de tratamiento, para efectuar modificaciones de acuerdo a los resultados de sensibilidad y la respuesta clnica
del paciente. Cuando el paciente tiene una infeccin severa y sntomas de sepsis, lo fundamental es reconocer la causa y quitar los
factores precipitantes, el apoyo en un servicio de terapia intensiva y el tratamiento emprico con antibiticos de acuerdo a la sospecha
clnica. En el Sndrome de choque toxico est recomendado: dicloxacilina + aminoglucocido, cefalotina + aminoglucocido,
aminoglucocido + glicopeptidos (amikacina-vancomicina), linezolid. En infecciones de SNC, se ha utilizado tanto el tratamiento
sistmico como el intraventricular, la limitacin es la penetracin adecuada del frmaco al SNC, los medicamentos habitualmente
utilizados son dicloxacilina, oxacilina y con vancomicina los niveles no son ptimos. Para tratamiento intraventricular se ha utilizado
gentamicina o amikacina.

CASO CLINICO
Paciente varn de 11 aos de edad, procedente de Chancay, readmitido despus de 19 das de ser sometido a ciruga electiva por un
quiste seo aneurismtico en el fmur izquierdo, realizndole curetaje seo e injerto autlogo de cresta iliaca. Fue dado de alta al
sptimo da. Posterior al alta, present fiebre no cuantificada y malestar general; persistiendo febril los siguientes doce das, motivo de
su readmisin en el servicio de Traumatologa. No tena antecedentes importantes. El nio luca en buen estado general, presentando
fiebre entre 38 y 38.5 C. Al examen se evidenciaba discreto aumento de volumen y dolor a nivel sacro y glteo izquierdo, sin otros
signos de flogosis.Las heridas operatorias habian cicatrizado. El resto del examen fsico era normal. El hemograma presentaba 10,200
leucocitos, sin desviacin izquierda. No tuvo estudio de VSG ni proteina C reactiva. El examen ecogrfico de partes blandas a nivel
sacro-glteo izquierdo revelaba distorsin de las partes blandas en los planos subcutneos y musculares adyacentes a la herida
operatoria, sin evidenciar colecciones. En la tomografa pelvica se evidenci imgenes compatibles con abscesos en partes blandas en
pelvis izquierda. No se procedi a puncin ni drenaje. Cont con solo un cultivo, el cual fue positivo a S. epidermidis

PREGUNTA
Cual es la conducta teraputica para el agente etiolgico aislado?

RESPUESTA
a.- Cefalotina.
b.- Dicloxacilina.
c.- Ciprofloxacina.
d.- Tazobactam.

CASO CLINICO
Femenino de 63 aos, inmunocompetente, que acude por aparicin de un bulto en la cara interna del tercio superior del muslo
derecho. En la TAC se observaron extensas colecciones de contenido lquido, desde la pelvis hasta el compartimento intramuscular de
los msculos psoas-ilaco derecho y obturador derecho. A travs de la regin inguinal derecha, estas se extendan hasta el tercio medio
del muslo derecho; las colecciones de mayor tamao se localizaban en el muslo y se asociaban a reaccin peristica femoral, y llegaron
a medir hasta 11 cm de dimetro mayor. Por cultivo del material de drenaje se diagnostic una infeccin por Staphylococcus aureus
resistente a la meticilina (SARM).
MANUAL DE TRABAJO DEL CURSO ENARM CMN SIGLO XXI
CURSO ENARM CMN SIGLO XXI TEL: 36246001 Pharmed Solutions Institute PGINA 132


PREGUNTA
a.- Rifampicina y minociclina.
b.- Vancomicina y gentamicina.
c.- Ceftriaxiona y amikacina.
d.- Dicloxacilina y vancomicina.

CASO CLINICO
Se trata de un paciente de 79 aos con enfermedad renal crnica estadio 5 secundaria a nefroangioesclerosis y/o nefropata diabtica,
en terapia renal sustitutiva con dilisis peritoneal continua ambulatoria. Como otros antecedentes presenta diabetes mellitus tipo 2 de
larga evolucin, hipertensin arterial y anemia perniciosa. A los 27 meses del inicio de la tcnica, el paciente acude a urgencias con
clnica y recuento de clulas en el lquido de drenaje, compatibles con una peritonitis. Se inicia el protocolo emprico que incluye
vancomicina y ceftazidima, Se enva muestras para cultivo. Se presentan posteriormente 3 episodios de peritonitis con un intervalo de
15 dias minimo entre cada cuadro.

PREGUNTA
Cual es la conducta mas adecuada a seguir?

RESPUESTA
a.- Cambiar de tipo de catter.
b.- Vancomicina intraperitoneal.
c.- Daptomicina IV.
d.- Cambiar a hemodilisis.

CASO CLINICO
Paciente varn de 44 aos, con infeccin por virus de inmunodeficiencia humano (VIH) diagnosticada en marzo de 2005, sin terapia
antirretroviral (linfocitos T CD4/+n 286/mm3, carga viral 89.000 copias/mi). Semanas previas a la consulta realiz viaje a Uruguay y
mantuvo contactos sexuales con ms de una pareja. Una semana antes del ingreso comenz con aumento de volumen labial
diagnosticndose infeccin por virus Herpes simples, indicndose aciclovir 400 mg/12 h vo. Al da siguiente, se agreg ciprofloxacino
500 mg c/12 h por progresin de lesin. Evolucion con fiebre 38C, tos y dolor torcico. Radiografa de trax revel neumona
intersticial bilateral indicndose hospitalizacin, la que el paciente rechaz. Se indic levofloxacino 500 mg/da vo. A las 48 h consult
en por agravamiento de su cuadro clnico y fue hospitalizado. Ingres febril 38C, presin arterial 120/70 y taquicardia de 110x'. Al
examen facial presentaba aumento de volumen flegmonoso del labio superior y en examen pulmonar crpitos bsales bilaterales.
Hemograma con hematocrito 40%, leucocitos 14.300/mm3, baciliformes 4%, VHS 69 y PCR 24,7 mg/dL. Radiografa y tomografa axial
computarizada (TAC) de trax mostraron pleuroneumona bilateral. Puncin pleural dio lquido con pH 7,4, glucosa 5 mg/dL, 26.800
clulas, 61,5% neutrfilos. En cultivos de secrecin de labio superior y lquido pleural se aisl Staphylococcus aureus resistente a B-
lactmicos, ciprofloxacino, eritromicina y resistencia inducible a clindamicina.
PREGUNTA
Cual es la conducta teraputica mas adecuada para el caso?

RESPUESTA
a.- Vancomicina y rifampicina.
b.- Vancomicina y gentamicina.
c.- Ceftriaxiona y amikacina.
d.- Dicloxacilina y vancomicina.

CASO CLINICO
Paciente varn de 23 aos, estando en chiapas sufri picadura de insecto en tobillo derecho. En las horas siguientes, desarroll
aumento de volumen, dolor y eritema en la zona, asociado a fiebre. Evaluado por mdico, se le indic cefadroxilo 500 cada 12 h vo. A
las 24 h present drenaje espontneo de abundante pus de la herida. Cuatro das ms tarde consult por persistencia de lesin
inflamatoria del tobillo. Al examen de ingreso se constat paciente afebril, presin arterial 134/70 mmHg, FC 64x'. En tobillo derecho
haba aumento de volumen eritematoso con lcera central. El hemograma mostr hematocrito 36,8%, leucocitos 6.680 x mm3,
baciliformes 3%, VHS 60 y PCR 6,3 mg/ dL. El cultivo fue positivo para Staphylococcus aureus resistente a B-lactmicos,
sulfametoxazoltrimetropim y ciprofloxacino.

PREGUNTA
Cual es la conducta teraputica a seguir mas adecuada para el caso?

RESPUESTA
a.- Cefazolina y clindamicina intravenosa.
b.- Clindamicina via oral.
c.- Ceftriaxiona y amikacina.
d.- Dicloxacilina y vancomicina.
MANUAL DE TRABAJO DEL CURSO ENARM CMN SIGLO XXI
CURSO ENARM CMN SIGLO XXI TEL: 36246001 Pharmed Solutions Institute PGINA 133

CASO CLINICO
Paciente varn de 39 aos, motociclista. Tres meses previo al ingreso, comenz con furnculos en trax, cuello y extremidades
inferiores las que se trataron con diversos antimicrobianos va oral, con respuesta parcial y aparicin de nuevas lesiones. El estudio
microbiolgico de una lesin del muslo, identific Staphylococcus aureus resistente a cloxacilina y sensible a clindamicina, la que recibi
va oral, sin respuesta. Por progresin de lesin en muslo izquierdo con celulitis extensa. Al examen fsico destacaba lesin pustulosa de
4 cm de dimetro en muslo izquierdo, rodeada de una zona eritematosa extensa que comprometa el tercio inferior del muslo. El
hemograma mostr hematocrito 40%, leucocitos 6.600 x mm3, baciliformes 2%, VHS 32 y PCR 6 mg/dl.

PREGUNTA
Cual es la conducta teraputica a seguir mas adecuada para el caso?
RESPUESTA
a.- Vancomicina y clindamicina intravenosa.
b.- Clindamicina via oral.
c.- Ceftriaxiona y amikacina.
d.- Dicloxacilina y vancomicina.

CASO CLINICO
Se trata de paciente femenino de 31 aos de edad la cual acude a consulta debido a que presenta disnea grave, desorientada,
confundida con tos productiva y fiebre importante, el familiar refiere no tener enfermedades, ser prcticamente sana, soltera y
abogada de profesin, agrega que hace una semana presento un cuadro gripal, caracterizado por fiebre, cefalea, mialgias y malestar
generalizado, sin darle importancia solo toma medidas generales mejorando, dos das despus presenta ataque al estado generalizado,
con dificultad respiratoria, a la exploracin fsica se presenta confusa, diafortica, disneica, con temperatura de 39.6 C, pulso de 130
lpm, TA 90/60 mmHg, FR 39 se ingresa y monitoriza con oximetro que reporta 89 % de saturacin se coloca oxigeno al 100% sin
cambios, la auscultacin pulmonar con sonidos respiratorios anforicos predominante en lbulo inferior izquierdo, la TAC revela necrosis
del lbulo inferior izquierdo. Se realiza intubacin y soporte de lquidos, se sospecha Staphylococcus aureus como agente causal.

PREGUNTA
Cual es tratamiento mas adecuado antes del reporte de los cultivos.

RESPUESTA
a.- Doxiciclina.
b.- Meticilina.
c.- Vancomicina.
d.- Trimetroprim/sulfametoxazol.

CASO CLINICO
Se trata de masculino de 34 aos de edad el cual tiene catter para hemodilisis, actualmente con renopatia terminal, acude a consulta
debido a que presenta fiebre y dolor intenso en la regin baja de la espalda, a la exploracin fsica se observa ansioso, irritable,
diafortico as como ataque al estado generalizado sin embargo signos vitales 120/80, FC 89, FR 21, temperatura 38.7 C, el sitio donde
se encuentra colocado el catter se observa rojo y caliente, no se observan secreciones ni olor, la biometra hemtica reporta 16 700
con 13 % de bandas, se realiza IRM donde se reporta lesiones compatibles con osteomielitis vertebral.

PREGUNTA
Cual es agente etiolgico ms probable.

RESPUESTA
a.- Streptococcos.
b.- Stafilococcos.
c.- Mycoplasmas.
d.- Tuberculosis.

CASO CLINICO
Se trata de masculino de 75 aos de edad el cual cuenta con antecedentes tales como diabetes mellitus de ms de 25 aos de
evolucin, actualmente acude por segunda ocasin a consulta debido a que presenta una ulcera del pie diabtico de 2 meses de
duracin la cual no ha respondido a tratamiento previo con metronidazol y dicloxacilina sin especificar dosis, a la exploracin fsica se
observa lesin con olor ptrido, se trata de una lesin de 2,6 cm con pus, la cual se drena e identifica lesin que llega al hueso con una
profundidad de 2.5 cm, se realiza cultivo y tincin, sin embargo se debe iniciar tratamiento, la tincin reporta flora mixta.

PREGUNTA
Cul es el manejo farmacolgico en espera de los resultados del cultivo.

RESPUESTA
a.- Clindamicina.
b.- Ampicilina/sulbactam.
c.- Metronidazol.
d.- Vancomicina.
MANUAL DE TRABAJO DEL CURSO ENARM CMN SIGLO XXI
CURSO ENARM CMN SIGLO XXI TEL: 36246001 Pharmed Solutions Institute PGINA 134

CASO CLINICO
Femenino de 23 aos de edad, acude a consulta refiriendo fiebre, escalofros asi como dolor de espalda y malestar generalizado.
Cuenta con antecedente de trastorno bipolar y abuso de sustancias, actualmente bajo tratamiento sin embargo con recaidas. A la
exploracin fsica se observa lesiones tipo puncion en pliegue cutneo en antebrazo por probable inyeccin intravenosa de sustancias,
al parecer de drogas sin aceptarlo la paciente, los signos vitales son 38.5 C de temperatura, FC 110 lpm, FR 24, tensin arterial de
110/60 mmHg, Rx de torax con varios infiltrados nodulares perifricos con cavitaciones.

PREGUNTA
Cul es la patologa ms grave que la paciente presenta.

RESPUESTA
a.- Neumonia
b.- Endocarditis.
c.- Septicemia.
d.- CID.

INFLUENZA (HAEMOPHILUS INFLUENZAE):
CIENCIAS BASICAS: Infeccin contagiosa de origen viral, considerada una de las causas ms importantes de infecciones de vas
respiratorias. La relevancia de esta infeccin radica en su frecuente asociacin a complicaciones y muerte en poblaciones susceptibles.
Este grupo de virus es capaz de asociarse a epidemias y pandemias con consecuencias devastadoras. El virus de la influenza puede
afectar la mucosa nasal, faringe, bronquios y a veces hasta los alveolos pulmonares. Los sntomas son parecidos a los del catarro comn
o resfriado, sin embargo son ms severos y su inicio es generalmente abrupto. El Haemophilus influenzae es un pequeo gramm
negativo, ARN, cocobacilo, pleomrfico. La influenza es ocasionada por 3 virus (influenza A,B,C). Las cepas con una cpsula de
polisacrido son ms importante clnicamente, causan la enfermedad invasiva sistmica, principalmente en bebs y nios <6 aos de
edad. Cepas no tipificables de H. influenzae, que estn no encapsulado, provocan la enfermedad mediante invasin local de superficies
mucosas. SALUD PBLICA: La enfermedad generalmente requiere atencin mdica y hospitalizacin, contribuyendo sustancialmente a
prdidas econmicas y exceso en nmero de hospitalizaciones y muerte. Por lo menos 4 pandemias han afectado a la humanidad en el
siglo XIX, y 3 en el XX. El virus de la influenza A se caracteriza por causar enfermedad moderada a grave; tiende a afectar a todos los
grupos etarios y tiene la caracterstica de afectar tanto aves como cerdos. El virus B de la influenza cuadros clnicos menos graves, y
tradicionalmente produce infeccin durante la infancia. La capacidad del virus de la influenza A y B de sufrir cambios antignicos
graduales en sus dos antgenos de superficie: la hemaglutinina y la neuraminidasa, complica la vacunacin contra esta enfermedad.
Afecta a todos los grupos etarios, pero principalmente a <2aos y >65aos. Ocurre principalmente durante los meses de invierno y
principios de la primavera. En Mxico la neumona e influenza representan una de las mayores causas de morbilidad y mortalidad. En
>65 aos de edad, la neumona e influenza se reportan como la sptima causa de mortalidad. PATOGENIA: Los virus de influenza se
diseminan de persona a persona a travs de los actos de toser o estornudar de personas infectadas o enfermas con los virus de
influenza, gotitas en el aire o fmites, el periodo de incubacin es de 1-4 das, la patognesis de la replicacin del virus y su relacin al
desarrollo de manifestaciones clnicas no ha sido completamente descifrada, no obstante la replicacin viral ocurre primariamente en
el epitelio columnar del aparato respiratorio, pero puede ocurrir en cualquier segmento a lo largo del tracto respiratorio. La
regeneracin del epitelio columnar toma de 3-4 semanas, tiempo en el cual las manifestaciones pulmonares pueden persistir.
DIAGNOSTICO: Clnico; Inicio agudo de fiebre, mialgia, cefalea, debilidad extrema, tos no productiva, odinofagia y rinitis, estos sntomas
ocurren en 50-70% de las infecciones. Puede haber dolor retroesternal, fotofobia, dolor abdominal y diarrea, en ancianos puede no
haber fiebre solo delirio y postracin, en nios puede asociarse a otitis media y traqueobronquitis. Podemos encontrar linfadenopata
cervical. Las definiciones clnicas para diagnosticar influenza varan de 63-78% de sensibilidad y de 55-71% de especificidad.
Laboratorio: Aislamiento del virus en muestras de exudado farngeo o nasofarngeo obtenido dentro de los primeros 3 das del inicio de
sntomas. Se considera al cultivo viral (permite tipificar y caracterizarlo antignicamente), como el estndar de oro seguido de
conformacin por inhibicin de la hemaglutinacin. Las tcnicas serolgicas mas recuente usadas son la fijacin de complemento y la
inhibicin por hemaglutinacin. COMPLICACIONES: Neumona primaria por influenza o neumonas secundarias bacterianas (S.
pneumoniae, S. aureus), sinusitis bacteriana, bronquitis, traqueobronquitis y otitis media. Puede ocurrir muerte en 0.5-1 en 1,000
casos. Otras; convulsiones, encefalopata por el virus, o encefalopata asociada a la utilizacin de salicilatos (sndrome de Reye=
encefalopata aguda acompaada de degeneracin grasa visceral), miositis, miocarditis y pericarditis. Puede haber diversas
manifestaciones a nivel de SNC, incluyendo mielitis transversa y Sx. de Guillan Barr. Meningitis que se asocia con una alta morbilidad,
6 % de los pacientes tienen una prdida auditiva neurosensorial, una cuarta parte tienen algunas secuelas importantes, la mortalidad es
de 5 %. Epiglotitis, que ocurre en nios mayores y en ocasiones en adultos, implica la celulitis de la epiglotis y tejidos supraglticos que
comienza con dolor de garganta y progresa rpidamente a la disfagia, salivacin, y obstruccin de va area. TRATAMIENTO: Reposo en
casa, analgsico o antiinflamatorios que no sean de la familia de los salicilatos, no utilizar antibiticos profilcticamente. Los derivados
de amantadino; amantadina (>10 aos 100mgs c/12 hrs >65 100mg c/24 hrs) y rimantadina (>10 aos 100mgs c/12 hrs >65 100mg
c/24 hrs) y los inhibidores de la enzima neuraminidasa; oseltamivir (>13 aos 75mg c/12 hrs) y zanamivir (>10 aos de edad 10mg c/12
hrs), los cuatro han demostrado utilidad clnica reduciendo la duracin de los sntomas. Meningitis en adultos por H. influenzae tipo B:
ceftriaxona (2 g cada 12 h durante 1-2 semanas). Meningitis por H. influenzae tipo B en nios: ceftriaxona (75-100 mg / kg por da,
divididos en dos dosis cada 12 h) y dexametasona (0,6 mg / kg por da en cuatro dosis divididas durante 2 das a la iniciacin del
tratamiento con antibiticos para prevenir prdida de la audicin). La epiglotitis: ceftriaxona (50 mg / kg al da durante 1-2 semanas).
PREVENCION: Vacunacin anual de personas en grupos de alto riesgo, utilizando la vacuna inactivada trivalente por su eficacia y baja
reactogenicidad, los grupos son los siguientes; vacunacin a personal de salud, personal que labora en asilos de ancianos, personas que
viven en contacto intradomiciliario con personas con alto riesgo de desarrollar complicaciones, mujeres con embarazo de alto riesgo
(2do y 3er trimestre), personas mayores de 65 aos, personas entre 50-64 aos (enfermedades cronicodegenerativas), pacientes de
cualquier edad con padecimientos crnicos (principalmente respiratorios y cardiovasculares), vacunacin de nios de 6-59 meses y
MANUAL DE TRABAJO DEL CURSO ENARM CMN SIGLO XXI
CURSO ENARM CMN SIGLO XXI TEL: 36246001 Pharmed Solutions Institute PGINA 135

viajeros (que van a los trpicos o al Hemisferio Sur). La vacuna se administra por va intramuscular, en regin deltoidea (nios regin
anterolateral de muslo). Contraindicada en alergia al huevo o a otros componentes de la vacuna o cuadro febril agudo.

CASO CLINICO
Femenino de 46 aos con antecedentes de asma bronquial tipo III, para lo cual lleva tratamiento con salbutamol (spray). Acudi por
presentar fiebre de 39 0C y trastornos de la conciencia. Cinco das antes haba presentado una crisis asmtica acompaada de un
cuadro febril asociado con sntomas respiratorios altos, que coincidi con un cuadro respiratorio alto en su nieto pequeo atendido por
ella; por lo anterior fue vista en su rea de salud donde le indicaron tratamiento con penicilina. El da antes present vmitos, en
nmero de 12, de tipo bilioso y acompaados de nuseas, con mucha tos y abundante secrecin nasal verdosa y fiebre. En el momento
de ser valorada en el servicio de urgencia llamaba la atencin los trastornos de conciencia que presentaba esta paciente, dados por la
agitacin psicomotora y el estado de estupor. Al examen fsico se constat rigidez nucal y signos de irritacin menngea. Se le realiz
puncin lumbar (PL) de donde se obtuvo un lquido cefalorraqudeo (LCR) de aspecto turbio, as como otros exmenes
complementarios, incluidos hemogramas seriados. El examen citoqumico del LCR mostr los valores siguientes: 435 leucocitos/mm3 a
predominio de polimorfonucleares (PMN), hemates 8/mm3, glucosa en 1,0 mmol/L y protenas totales en 0,95 g /L. En el examen
bacteriolgico del lquido se inform pleomorfismo bacteriano gramnegativo. En el leucograma se constat una cifra de 16 000
leucocitos con predominio de PMN. El cultivo reporto haemophilus influenzae en el LCR.

PREGUNTA
Cual es la conducta mas apropiada a seguir con el diagnostico presuntivo del caso?

RESPUESTA
a.- Rivavirina y amantadina.
b.- Ceftriaxona va endovenosa.
c.- Ceftriaxona y amikacina IV.
d.- Cloranfenicol y ampicilina.

TOS FERINA O COQUELUCHE (BORDETELLA PERTUSSIS):
CIENCIAS BSICAS: Bordetella pertussis causa tos ferina, una infeccin aguda del tracto respiratorio, caracterizada por accesos de tos
parodia ticos acompaado de estridor al final de la inspiracin. B. pertussis es un bacilo aerobio Gramm negativo. SALUD PBLICA: La
tos ferina es altamente contagiosa. En los hogares, las tasas de ataque son el 80% de los contactos no vacunados y 20% entre los
contactos inmunizados. La tos ferina sigue siendo una causa importante de morbilidad y muerte infantil en los pases en desarrollo. En
los Estados Unidos, la incidencia ha aumentado lentamente desde 1976,
particularmente entre los adolescentes y adultos. Tos persistente de duracin> 2 semanas en un adulto puede ser debido a B. pertussis
en 12-30% de los casos. La morbilidad y la mortalidad severa se limitan a lactantes <6 meses de edad. La incidencia en Mxico es de 1-3
por 100,000 habitantes. PATOGENIA: Transmisin por va respiratoria, al estar en contacto con gotas de secreciones, el periodo de
contagiosidad se presenta durante el periodo catarral y la primera semana del inicio de la fase paroxstica. B. pertussis fastidioso se
adhiere a las clulas epiteliales ciliadas (tropismo) de la nasofaringe, donde prolifera y se dirige al epitelio traqueobronquial, es
internalizado por las clulas epiteliales, pero no penetra clulas submucosas, ni invade el torrente sanguneo, se multiplica localmente,
y produce una amplia gama de toxinas y productos biolgicamente activos como toxina pertussis ( la cual acta como exotoxina y
penetra al torrente sanguneo, induce linfocitosis, activacin de islotes pancreticos), hemaglutininas filamentosa (adherencia a clulas
de epitelio), pertactina (adherencia y altamente inmunogenica), BrkA (adherencia y resistencia al complemento), adenilciclasa (inhibe
funcin fagocitica). DIAGNOSTICO: Enfermedad prolongada de tos. Los sntomas suelen ser ms graves en los bebs y nios pequeos.
Tres fases: La fase catarral; posterior a periodo de incubacin de 7-10 das es similar al resfriado comn (rinorrea, estornudos, lagrimeo,
tos leve progresiva) y tiene una duracin de 1-2 semanas y es el periodo de mayor contagiosidad. La fase paroxstica o de estado; los
accesos de tos son paroxsticos, caractersticamente si inspiracin y suelen acompaarse de vomito y cianosis y terminar con un
estridor inspiratorio prolongado que le da el nombre de tos coqueluche. El nmero de accesos le da la gravedad, llegando a 15-20 por
da, sin son frecuentes, puede presentarse sudacin, lasitud y somnolencia y son de predominio nocturno, tiene una duracin de 2-
6semanas. Fase convaleciente: inicia cuando los accesos de tos dejan de ser espasmdicos y dura varias semanas, durante la fase de
accesos pueden exacerbarse si el paciente presenta infeccin en vas areas superiores. En cuadros ms severos predominan datos de
dificultad respiratoria, cianosis y apnea, ms que accesos de tos. Laboratorio: Aislamiento de Bordetella pertussis es 100% especifico, la
posibilidad de positividad declina si la muestra se toma >2 semanas del inicio de la sintomatologa y la muestra para el cultivo se toma
de la nasofaringe posterior. PCR a aumentado su sensibilidad, pero no se usa como mtodo exclusivo y no sustituye al cultivo.
Deteccin de anticuerpos fluorescentes (DAF), til como prueba de deteccin rpida, pero la sensibilidad es baja. Serologa;
demostracin de una elevacin sustancial 4 veces los ttulos de anticuerpos contra diferentes antgenos. COMPLICACIONES: La ms
frecuente y causas de muerte es la neumona bacteriana secundaria, otras son crisis convulsivas y encefalopata TRATAMIENTO: Del 80-
90% de pacientes con tos ferina sin tratamiento depuran espontneamente a B. pertussis de la nasofaringeo en las 3-4 semanas.
Manejo principal ente de soporte. La eritromicina es el antibitico de eleccin, preferentemente en forma de estolato a dosis de
40mg/kg/da por 14 dias. Alternativas claritromicina por 7 das igual efectividad y mejor tolerancia. Azitromicina y TMP/ SFX. En
menores de un mes se da azitromicina (eritro y claritromicina estn contraindicadas). Para la tos se puede manejar benzonatato o
salbutamol en nebulizador. PROFILAXIS POSTEXPOSICION: Contactos asintomticos de un caso ndice que se encuentre dentro de los 21
das del inicio de los accesos de tos, deber recibir profilaxis con antibiticos antes mencionados. PREVENCIN: Vacuna combinada de
toxoide tetnico y difterico (DPT). En Mxico A partir de 1999, poco despus de la introduccin de la vacuna triple viral, se introduce
una vacuna pentavalente (DTwP-hepatitis B-Hib), en tres dosis a los 2,4 y 6 meses de edad y refuerzos con DPT a los 4 aos de edad. A
partir de 2007, se efectu un cambio biolgico a una vacuna pentavalente a base de vacuna acentuar de pertussis (DTaP-IPV-Hib).


MANUAL DE TRABAJO DEL CURSO ENARM CMN SIGLO XXI
CURSO ENARM CMN SIGLO XXI TEL: 36246001 Pharmed Solutions Institute PGINA 136

CASO CLINICO
Una mujer de 64 aos ingresada por una disnea creciente y dolor torcico. La paciente tena antecedentes de carcinoma pulmonar no
microctico y estaba recibiendo tratamiento. Anteriormente haba fumado 4-5 cigarrillos al da durante un periodo de hasta 10 aos. A
la exploracin, se trataba de una mujer con sensacin de enfermedad y emaciacin, que estaba clnicamente anmica y presentaba
signos de un derrame pleural izquierdo masivo en la TC torcica. La frecuencia del pulso era de 85/min, la presin arterial de 120/85, y
no haba signos de insuficiencia cardiaca. La radiografa de trax confirm el derrame unilateral izquierdo. La hemoglobina era de
9,1g/100ml, con un frotis de sangre normocrmico y normoctico. La velocidad de sedimentacin globular fue de 90mm en la primera
hora. Se realiz una aspiracin pleural, en la que se obtuvieron 1.500ml de lquido turbio. Dada la persistencia de las colecciones de
lquido pleural, 2 das despus de la segunda aspiracin se coloc un drenaje de trax. El nivel de protenas fue de 35g/l y la
concentracin de glucosa, de 0,3mmol/l. No se observaron clulas malignas, pero en la PCR se demostr la presencia de B. pertussis. La
paciente no present fiebre ni manifestaciones clnicas de infeccin, y el recuento leucocitario en sangre fue normal. Adems, no hubo
clnica de diarrea y los coprocultivos, urinocultivos y hemocultivos fueron repetidamente negativos.

PREGUNTA
Cual es la conducta antibitica mas apropiada para el caso?

RESPUESTA
a.- Claritromicina.
b.- Azitromicina
c.- TMP/ SFX
d.- Eritromicina.

CASO CLINICO
Mujer de 45 aos, fumadora de 6 cig/da desde los 20 aos, por malestar general, astenia, fatigabilidad y tos seca de predominio
nocturno de seis semanas de evolucin. No refere congestin nasal, rinorrea, descarga nasal posterior, cefalea, pirosis, fiebre,
escalofros, disnea, dolor torcico, expectoracin ni hemoptisis. Antecedentes mrbidos: rinitis alrgica estacional en primavera con
test cutneos positivos a plenes y pastos; refujo gastroesofgico en tratamiento con medidas dietticas y Omeprazol 20 mg/da.
Actividad laboral: enfermera de unidad de hemodilisis de adultos. En el examen fsico, los signos vitales eran normales con una SpO2
de 97% respirando aire ambiente, el examen cardiopulmonar y abdominal eran normales. La paciente fue tratada con medidas
dietticas y posturales antirefujo. Finalmente, la inmunofuorescencia directa de Bordetella pertussis de hisopado nasofarngeo fue
positiva.

PREGUNTA
Cual es la conducta antibitica mas apropiada para el caso?

RESPUESTA
a.- Claritromicina.
b.- Azitromicina
c.- TMP/ SFX
d.- Eritromicina.

MORAXELLA CATARRHALIS:
CIENCIAS BSICAS: Moraxella catarrhalis es un coco gram-negativo, que se asemeja a Neisseria. En el gnero Moraxella podemos
identificar cuatro especies: M. catarrhalis, M. caviae, M. ovis, M. cuniculi. Forma parte de la flora normal de las vas respiratorias
superiores. SALUD PUBLICA: M. catarrhalis coloniza hasta el 50% de los nios sanos (muy relacionados con IRA) y hasta el 3-7% de los
adultos sanos. Las tasas de infeccin pico son a finales de invierno / principios de primavera. Generalmente no se le considera un
agente primario en las infecciones del tracto respiratorio inferior, esto cambia al referirse a individuos mayores de 50 aos, donde es
considerado un patgeno primario de vas respiratorias bajas. Se puede decir que es un agente oportunista que se aprovecha de las
condiciones predisponentes del husped para causar enfermedad y formar parte de los patgenos humanos
emergentes. DIAGNOSTICO: La otitis media y sinusitis: M. catarrhalis es la tercera causa ms frecuente de otitis media en los nios y es
importante aislarlo de los casos de sinusitis aguda y crnica. Conjuntivitis en neonatos. Traqueobronquitis purulenta, bronquitis,
neumona: La mayora son pacientes > 50 aos de edad y con EPOC (a menudo con el cncer de pulmn tambin). Laringitis,
queratinitis, uretritis. Los sntomas son de leves a moderados, la enfermedad invasiva (por ejemplo, enfisema) es poco frecuente.
Laboratorio: Se cultivan muestras de esputo, sangre, secreciones bronquiales, aspirado transtraqueal, lavado broncoalveolar y biopsia
pulmonar. Un mtodo de diferenciacin con la Neisseria, es que la Moraxella Catarrhalis no fermenta los carbohidratos y produce
DNasa. Adems produce Butirato esterasa, que constituye la base de la prueba fluoromtrica rpida para identificarla. En la tincin
aparecen bacilos, cocobacilos o cocos pequeos Gramnegativos. TRATAMIENTO: Este microorganismo presenta una alta resistencia a
beta-lactmicos como la penicilina, ampicilina y amoxicilina. Se puede usar amoxicilina/ac. clavulnico, cefalosporinas de segunda y
tercera generacin, TMP-SMZ (mezcla de una parte de trimetoprim y cinco partes de sulfametoxazol). Esta bacteria presenta
sensibilidad a la quinolonas, eritromicina, tatraciclina, amikacina, imipenem, meropenem, cloranfenicol.

CASO CLINICO
Masculino de 15 aos, fue ingresado con una historia de 3 das de dolor de cabeza y de 2 das de petequias generalizadas, nuseas y
vmitos. Temperatura 36,9 , FC 93 latidos / min y la TA 125/82 mmHg. Estaba alerta y se quej de rigidez en el cuello. No hubo
ninguna secuela motora o sensorial , y el resto de su examen fsico sin complicaciones. Leucocitos perifricos de 30.290 / l (96 %
MANUAL DE TRABAJO DEL CURSO ENARM CMN SIGLO XXI
CURSO ENARM CMN SIGLO XXI TEL: 36246001 Pharmed Solutions Institute PGINA 137

neutrfilos), hemoglobina de 15,4 g / dl, y plaquetas de 151.000 / l . CSF tena un aspecto turbio con un aumento de la concentracin
de protena de 575 mg / dl, leucocitos de 35.500 / l ( 90 % de neutrfilos ) y glucosa de 10 mg / dl.
PREGUNTA
Cual es la conducta antibitica mas apropiada para el caso?

RESPUESTA
a.- Ampicilina y acido clauvulanico.
b.- Ceftazidima y netilmicina.
c.- Quinolonas e imipenem.
d.- Eritromicina y meropenem.

CASO CLINICO
Masculino de 81 aos, con antecedentes de cncer de pncreas y cirrosis heptica por virus de la hepatitis C. Fue ingresado por
inversin del ciclo vigila sueo. A su ingreso, tena ictericia esclertica. Leucocitos 6950 / l (73% neutrfilos), hemoglobina de 12,8 g / dl,
plaquetas de 60.000 / l. LCR estaba claro con protena de 38 mg / dl, glbulos blancos de 9/L, glucosa de 76 mg / dl, nitrgeno ureico /
creatinina 41.8/2.2 mg / dl, amonaco de 287 mg / dl, bilirrubina total de 3,1 mg / dl.

PREGUNTA
Cual es la conducta antibitica mas apropiada para el caso?

RESPUESTA
a.- Ampicilina.
b.- Ceftaxima.
c.- Imipenem.
d.- Eritromicina.

BRUCELOSIS (BRUCELLA MELITENSIS):
CIENCIAS BSICAS: La Brucella es una bacteria gramm negativa, viven en el interior de las clulas del sistema fagocitico mononuclear
(intracelularfacultativo), por ello tiene un curso recidivante, tratamiento difcil y prolongado. Todas las especies de Brucella tienen
como reservorio las ubres o tero gestante de las hembras y los genitales del macho. La B. melitensis (caprinos y ovinos), es el agente
causal ms importante, seguido de B. abortus (bovinos) y B. suis (porcinos) y B. canis (de perros). La brucelosis es una zoonosis
sistmica en el humano puede afectar cualquier rgano o sistema, generalmente adopta un curso crnico, ataque al estado general,
baja letalidad. SALUD PBLICA: OMS, la considera la zoonosis de mayor distribucin en el mundo. Cada ao ocurren medio milln por
lo menos de casos nuevos. En Mxico existe en todo el territorio nacional predominando en un rea triangular con la base en la
frontera norte y el vrtice en el centro. PATOGENIA: La brucelosis se transmite por ingestin, inhalacin o exposicin percutnea o
mucosa, la enfermedad en los seres humanos por lo general se asocia con la exposicin a animales infectados o sus productos, ya sea
en lugares de trabajo (por ejemplo, el trabajo de los mataderos, la agricultura) o el contexto familiar (por ejemplo, el consumo de
alimentos, especialmente productos como leche no
pasteurizada, queso, mantequiilla) contaminados.
Ya en el organismo son fagocitados por leucocitos,
PMN, macrfagos tisulares, donde pueden
sobrevivir y multiplicarse. Una vez rebasada la
barrera linftica, llegan a la circulacin sistmica, y
de esta manera son transportados a los diferentes
rganos del sistema fagocitico mononuclear
(hgado, bazo, medula sea, ganglios). La
persistencia de Brucella dentro de las clulas es
debido a la inhibicin de la fusin fagosoma-
lisosomal, la degranulacion y la activacin del
sistema mieloperoxidasa. La liberacin de las
bacterias de las clulas necrticas puede sobrepasar
la capacidad fagocitica. Las manifestaciones clnicas
estn determinadas en gran parte por la liberacin
de una endotoxina y el grado de hipersensibilidad a los antgenos brucelares. DIAGNOSTICO: Clnico: El perodo de incubacin de 1
semana hasta varios meses es seguido por el desarrollo de la fiebre ondulante, sudores, aumento de la apata, fatiga y anorexia y
sntomas inespecficos como dolor de cabeza, mialgias y escalofros. La brucelosis menudo se presenta con uno de los tres patrones:
una enfermedad febril similar pero menos grave que la fiebre tifoidea, fiebre y monoartritis aguda, tpicamente de la cadera o de la
rodilla, en un nio pequeo (artritis sptica), o la fiebre de larga duracin y baja de la espalda o dolor en la cadera en un hombre mayor
(osteomielitis vertebral). Brucelosis puede causar linfadenopata, hepatoesplenomegalia, epididimoorquitis, compromiso neurolgico y
el absceso focal. El microorganismo se cultiva con xito en el 50-70 % de los casos, pero la cultura de identificacin suele tardar hasta 6
semanas. El diagnostico de certeza es el aislamiento de brucella, en hemocultivo o cultivo de medula sea. Fiebre prolongada
acompaada de ataque articular nos hace sospecharla. Ensayos de aglutinacin para IgM son positivos en la infeccin temprana. Los
ttulos individuales de 1:160 y 1:320-1:640 son diagnsticos en reas no endmicas y endmicas, respectivamente. La brucelosis se
debe distinguir de la tuberculosis, y si esta distincin no es posible, el rgimen debe adaptarse para evitar la monoterapia inadvertida
para la tuberculosis. Brucelosis tiende a causar menos destruccin del hueso y la articulacin de la tuberculosis. TRATAMIENTO: Se
recomienda como primera eleccin estreptomicina a una dosis de 750 mg a 1 g al da (o gentamicina a 5-6 mg / kg al da) durante 14-21
BRUCELOSIS TUBERCULOSIS
SITIO Lumbar y otros Dorsolumbar
LAS VRTEBRAS Mltiple o contiguas Contiguo
DISQUITIS Tarde Temprano
CUERPO Intacto hasta el final Morfologa de perdida temprana
COMPRESION DEL
CANAL
Raro Comn
EPIFISISTIS Anterosuperior Signo de
Poms
General: las regiones de disco
superior e inferior , central,
subperistico
OSTEOFITOS Anterolateral (pico de
loro)
Inusual
DEFORMIDAD En cua poco comun Cua anterior, giba
ABSCESOS
PARAVERTEBRALES
Pequeos, bien
localizados
Prdida Comn y discreta, apfisis
transversa
RECUPERACIN

Esclerosis, todo el
cuerpo
variable

MANUAL DE TRABAJO DEL CURSO ENARM CMN SIGLO XXI
CURSO ENARM CMN SIGLO XXI TEL: 36246001 Pharmed Solutions Institute PGINA 138

das ms doxiciclina en una dosis de 100 mg C/12 hrs durante 6 semanas. Alternativa: TMP/SFX + Rifampicina por 6 semanas.
Enfermedad compleja (por ejemplo, enfermedad neurolgica significativa o endocarditis) requiere al menos 3-6 meses de tratamiento
con mltiples agentes. Alternativa: rifampicina (600-900 mg / d) ms doxiciclina (100 mg bid) durante 6 semanas. El trimetoprim
/sulfametoxazol, se puede dar en vez de doxiciclina - por ejemplo, a los nios o las mujeres embarazadas. La recada se produce en
aproximadamente el 30 % de los casos, por lo general debido a la falta de cumplimiento. El paciente debe controlarse por lo menos 2
aos.

CASO CLINICO
Un paciente varn de 20 aos de edad, que se acababa de alistar en el ejrcito, se present refiriendo tos, expectoracin, dolor
punzante en el hemitrax izquierdo, sudoracin nocturna, anorexia, fiebre ondulante y disnea de casi 10 das de evolucin. Tambin
refiri la prdida de 3kg de peso en el ltimo mes. Presentaba fiebre de 38,4C y su frecuencia respiratoria era de 22/min. La
auscultacin torcica revel estertores inspiratorios basilares bilaterales. Se detect un recuento de leucocitos de 27.930/mm3 y una
velocidad de sedimentacin de 68mm/h. El valor de protena C reactiva era de 141mg/dl. La radiografa de trax mostr un infiltrado
neumnico en la zona media derecha y una opacidad homognea en la zona inferior izquierda. Se document el crecimiento de
Brucella spp. en el hemocultivo.

PREGUNTA
Cual es la conducta antibitica mas apropiada para el caso?

RESPUESTA
a.- Rifampicina
b.- Estreptomicina
c.- Gentamicina
d.- Doxiciclina

CASO CLINICO
Varn de 74 aos haba trabajado, la mayor parte de su vida, en criaderos de aves de zonas rurales del rea Metropolitana. Su cuadro
clnico se haba iniciado aproximadamente tres meses antes con compromiso moderado del estado general, fiebre vespertina y prdida
de peso, que en ese momento cuantificaba en diez kilos. Slo se verific la existencia de fiebre irregular, alrededor de 38C, y
decaimiento. Se inici estudio del sndrome febril prolongado. El hemograma mostraba anemia moderada (hematocrito 34,1%,
hemoglobina 11,3 gr%), leucocitos en rango normal, neutrofilia de 76%) sin desviacin a izquierda y VHS de 36 mm/h. Albuminemia (3,3
gr%), la protena C reactiva moderadamente elevada.

PREGUNTA
Cual es la conducta antibitica mas apropiada para el caso?

RESPUESTA
a.- Rifampicina
b.- Estreptomicina
c.- Gentamicina
d.- Doxiciclina

TULAREMIA (FRANCISELLA TULARENSIS):
CIENCIAS BASICAS: Las infecciones humanas causadas por Francisella tularensis, que es una bacteria facultativa intracelular, se
producen a travs de la interaccin cuando el insecto muerde, o chupa sangre (especialmente las garrapatas y las moscas tabanid),
animales salvajes o domsticas (por ejemplo, conejos salvajes, ardillas), o el medio ambiente. El organismo puede persistir durante
meses en el barro, agua, y los cadveres de animales en descomposicin. Ms de la mitad de los casos en Estados Unidos se producen
en Arkansas, Oklahoma y Missouri. PATOGENIA: La F. tularensis entra en la piel o las membranas mucosas a travs de mordeduras o
rasguos inaparentes o se adquiere por inhalacin o ingestin. Su patogenicicdad se debe a su capacidad para sobrevivir dentro de
macrfagos no estimulados. Los macrfagos fagocitan con facilidad a la F. tularensis pero esta resiste la muerte frente a los radicales
derivados del oxigeno, una respuesta neutrofilica local contraresta la infeccin, esta respuesta evoluciona a una ulcera. DIAGNOSTICO:
El perodo de incubacin es de 2-10 das. La tularemia a menudo comienza con un comienzo agudo de fiebre, escalofros, dolor de
cabeza y mialgias. Uno de varios sndromes puede desarrollar: Tularemia Ulceroglandular / glandular (75-85 % de los casos). El sello es
una induracin, eritema, lceras que no sanan 1-3 semanas duraderos (forma ulceroglandular) que comienza como una ppula
pruriginosa, ulcera, bordes drsticamente marcados y un exudado amarillo, y desarrolla una base de negro. Una lesin primaria de piel
puede no ser evidente en el 5-10% de los casos (forma glandular). Linfadenopata est relacionada con la ubicacin de la picadura de la
garrapata; linfticos inguinales / femoral estn afectadas con mayor frecuencia en los adultos debido a la frecuencia de picaduras en las
piernas. Los ganglios linfticos pueden llegar a ser fluctuante y drene espontneamente. Tularemia oculoglandular: Infeccin de la
conjuntiva, por lo general por el contacto con los dedos contaminados, resultados en conjuntivitis purulenta con adenopata regional y
dolor debilitante. Doloroso linfadenopata preauricular es nico a la tularemia. Orofarngea y gastrointestinal: Adquirida por ingestin,
la infeccin se puede presentar con faringitis y adenopata cervical, ulceraciones intestinales, adenopatas mesentricas, diarrea,
nuseas, vmitos y dolor abdominal. Tularemia pulmonar: La infeccin se adquiere por inhalacin o por va hematgena. El paciente se
presenta con una tos no productiva, disnea, dolor torcico pleurtico, infiltrados irregulares o lobar bilaterales, derrame pleural y
empiema ocasionales en la radiografa de trax. Tularemia tifoidea: Consta de fiebre y signos de sepsis y sin hallazgos focales.
Laboratorio: Tincin policromtica de muestras clnicas (tincin de poca ayuda). Serologa mediante microaglutinacin o la prueba de
aglutinacin en tubo. Un solo ttulo de 1:160 o un aumento de cuatro veces en el ttulo despus de 2-3 semanas se consideran
MANUAL DE TRABAJO DEL CURSO ENARM CMN SIGLO XXI
CURSO ENARM CMN SIGLO XXI TEL: 36246001 Pharmed Solutions Institute PGINA 139

positivos. La cultura es difcil y supone un riesgo importante para el personal de laboratorio. Mtodos de reaccin en cadena de la
polimerasa (PCR) se han utilizado para detectar el ADN de F. tularensis en muestras clnicas. TRATAMIENTO: La gentamicina se
considera el frmaco de eleccin para adultos (5 mg / kg al da C/12hrs) y nios (2,5 mg / kg tres veces al da o 5 mg / kg c/12) con
tularemia. Estreptomicina (1 g cada 12 horas) tambin es eficaz, pero no es tobramicina. Desaparicin de la fiebre por lo general ocurre
dentro de 2 das, pero la curacin de las lesiones de la piel y los ganglios linfticos puede tardar 1-2 semanas. Late supuracin de
ganglios linfticos puede ocurrir, con el tejido necrtico estril. Enfermedad leve a moderada responden rpidamente a tratamiento
pueden ser tratados durante 5-7 das, de lo contrario, el tratamiento se administra durante 7-10 das. Las alternativas incluyen
tetraciclinas o cloranfenicol (tasas de recada de hasta el 20 %). Las fluoroquinolonas han demostrado ser prometedores, pero los
ensayos clnicos estn pendientes.

CASO CLINICO
Se trata de un paciente varn de 36 aos, trabajador en un matadero industrial de aves. Entre sus aficiones destaca la caza de liebres,
conejos y jabales, adems de su hobby como taxidermista (su ltima pieza disecada fue un zorro). No tiene perros ni gatos y niega
haber recibido transfusiones o tener otras enfermedades en curso. El paciente acudi al mdico por presentar fiebre de 40 grados,
sudoracin, quebrantamiento general, mialgias, odinofagia y cefalea. Se interpret el cuadro clnico como un sndrome gripal y se trat
con paracetamol. La fiebre persisti durante los siguientes 3 das, y por ello acudi al hospital. Tras una exploracin fsica y una
radiografa de trax normal, continu con el mismo diagnstico y tratamiento. Aproximadamente una semana despus del comienzo de
los sntomas, present una tumoracin dolorosa en la axila derecha que aument de tamao progresivamente hasta alcanzar los 5 cm
de dimetro. De las pruebas complementarias que all se le realizan destacamos las siguientes: la radiografa de trax es normal, la
analtica presenta neutrofilia con desviacin a la izquierda y la ecografa axilar derecha revela un conglomerado de adenopatas de 5 x 3
cm.

PREGUNTA
Cual es la conducta antibitica mas apropiada para el caso?

RESPUESTA
a.- Rifampicina
b.- Estreptomicina
c.- Gentamicina
d.- Doxiciclina

SINDROME DE INMUNODEDICIENCIA ADQUIRIDA (SIDA); VIRUS DE LA INMUNODEFICIENCIA HUMANA (VIH)
CIENCIAS BASICAS: SIDA; Alteraciones inmunitarias profundas, infecciones bacterianas recurrentes o por grmenes oportunistas y a
aparicin de formas comunes de neoplasias malignas, causada por VIH -1 (Amrica, Europa, Caribe), VIH-2 (oeste de frica). El VIH
pertenece a la familia de retrovirus (poseen enzima transcriptasa reversa) citopaticos, no transformantes denominados lentivirus. Las
regiones codificadores del VIH son los genes: gag=poliproteina precursora, que es escindida para protenas de la capside,
pol=transcripatasa inversa, integrasa y proteasa, env=protenas de la cubierta. SALUD PUBLICA: Los primeros casos de SIDA, fueron
reconocidos en los Angeles en 1981. Segn el CENSIDA, los casos acumulaos suman, hasta junio del 2007, 112,830, de los cuales 2,720,
ocurren en el grupo de los menores de 15 aos. Proporcin hombre: mujer 6:1, en el ao 2003, para el grupo ms afectado (25-39
aos). El riesgo de transmisin vertical del VIH-1 de una madre infectada al producto vara entre 11-50%. La transmisin sexual en
hombres aun es predominantemente homo-bisexual. PATOGENIA: La infeccin por VIH, puede adquirirse por transmisin sexual (80-
90%) exposicin parenteral a sangre o derivados (por agujas en usuarios de drogas, o trabajado a de la salud), y de madres infectadas a
sus productos, durante el perodo perinatal. Rutas menos frecuente a travs de trasplante de rganos, tejidos y semen contaminado. El
riesgo de adquirir infeccin depende del tipo de exposicin (transfusin de un donador seropositivo es de 100%, exposicin nica a
aguja contaminada es de 0.4%). La transmisin del VIH de la madre al nio adems de in tero, puede ocurrir intraparto, por exposicin
del producto a la sangre materna o las secreciones genitales infectadas, esta transmisin se puede evitar mediante estrategias de
profilaxis con antiretrovirales durante el embarazo y parto as como mediante la realizacin de cesrea programa en mujeres infectadas
con VIH-1, con carga viral de >1,000 copias, a la semana 38 de gestacin antes de que se haya desencadenado el TDP y de que se
presente ruptura de membranas. Existe un riego adicional por lactancia materna del 14% en casos de infeccin establecida y 29% de
infeccin primaria. No hay evidencia que el VIH se pueda transmitir entre los miembros de una familia, a travs de contacto estrecho o
por compartir utensilios. El VIH se une a la clula blanco (tiene tropismo por los T CD4), que tiene un receptor de alta afinidad de la
glicoproteina de la envoltura gp120 del virus, lo que permite su unin, una vez unido se produce la fusin de su envoltura externa del
virus a la membrana celular a travs del gp41 con lo que el VIH entra a la clula, pierde su envoltura proteica, libera su ARN y las
protenas codificadas por la regin Pol, como la transcriptasa inversa, en el citoplasma. Esta enzima dirige la construccin de una
cadena de ADN sobre la plantilla de ARN viral, y en segundo paso cataliza la copia de la cadena recin sintetizada para producir un ADN
de cadena doble (proviral). Este ADN proviral se integra en el ADN cromosmico de la clula husped. Despus de su integracin, el
ADN proviral del VIH pasa a un estado productivo, en el cual el ADN proviral se transcribe en ARN viral y ARN mensajero, el cual
codifica la si resist de protenas virales, necesarias para la replicacin viral, mediante el uso de las funciones metablicas de la clula
husped. Los estadios finales comprenden el proceso de ensamblaje o encapsulacin viral y la protrusin en la superficie celular; en
esta etapa el virus adquiere una envoltura lipidica, siendo finalmente liberado para infectar a otras clulas y repetir el ciclo. Durante la
fase asintomtica de VIH (Clnica latente), los niveles circulantes del virus y las cuentas de CD4 permanecen esencialmente constantes
(equilibrio); el grado de infeccin de las clulas es muy semejante al grado de muerte de las clulas infectadas. Con la replicacin viral,
la clula experimenta en pocos das degeneraciones balninformes (tumefaccin celular por alteracin en la permeabilidad de la
membrana), lo que conduce finalmente a la muerte celular. Las clulas infectadas pueden volverse ms susceptibles a la sper
infeccin por otros patgenos, lo cual puede llevar ms rpidamente a la deplecin de clulas T, que son las clulas crticas de la
activacin de la respuesta inmunitaria especifica, tanto celular como humoral. Esta inmunosupresin adquirida, explica la gran
MANUAL DE TRABAJO DEL CURSO ENARM CMN SIGLO XXI
CURSO ENARM CMN SIGLO XXI TEL: 36246001 Pharmed Solutions Institute PGINA 140

susceptibilidad a presentar infecciones por grmenes poco frecuentes, como neoplasias malignas. No solo los linfocitos CD4 se afectan,
tambin monocitos y macrfagos pueden ser infectados por el VIH. La afeccin de los macrfagos alveolares, explica la susceptibilidad
a neumona por Pneumocystis jiroveci. La alteracin funcional de las clulas fagociticas tambin compromete a los neutrfilos PMN, lo
cual explica la susceptibilidad a grmenes piogenos. Tambin hay alteracin en linfocitos B, las cuales consisten en activacin policlonal
de estas clulas, por lo que se presenta hipergammaglobulinemia haciendo susceptible a grmenes encapsulados como Streptococcus
pneumoniae y Haemophilus influenzas.
DIAGNOSTICO: Clnico: La enfermedad de SIDA ocurre cuando el husped infectado, ya con graves alteraciones en sus mecanismos de
defensa, no puede controlar microorganismos oportunistas o neoplasias malignas que rara vez causan enfermedad en el individuo
inmunocompetente. Las manifestaciones especificas ms frecuentes son: infeccin primaria=sndrome retroviral agudo,
dermatolgicamente eritema maculopapular y ulceracin mucocutanea, adems, fiebre, faringitis, linfadenopata, artralgias, mialgias,
anorexia, nusea, prdida de peso, diarrea intermitente. A nivel neurolgico puede haber; Encefalopata; cuando VIH llega a SNC, en la
fase temprana hay disminucin de la capacidad de concentracin, perdida de la memoria y lentitud mental, hipertrofia, ataxia,
temblores, en la fase tarda incrementan todos los sntomas y signos, puede haber convulsiones y psicosis, hay datos de extrema atrofia
cerebral y son comn es los cambios en la sustancia blanca. La incidencia de gastroenteritis es hasta de 77% cursan con diarrea (crnica
o intermitente) en algn momento, la causa ms frecuente es Cryptosporidium. La infeccin por mycobacterium tuberculosis es la
mayor complicacin
oportunista que
afecta a los adultos
con VIH, con cualquier
recuento de CD4 y
puede incrementar el
riesgo de
complicaciones y
mortalidad asociada a
VIH. Muchas de las
infecciones
oportunistas en el
adulto son
secundarias a
reactivacin de
patgenos
oportunistas como:
Mycobacterium
avium, candida,
criptococcus
(meningitis con CD4 <
50 cels/mcl), herpes
simple, toxoplasma,
histoplasma, CMV (coriorretinitis, perdida de visin, infiltrados amarillentos y/o hemorragias). Toxoplasma (causa mas comn de dficit
focal del SNC, cefalea confusin, < 50cels/mcl, TAC lesiones multiples en anillos, tx. Primetamina+ sulfadiazina+ leucovorin por 6 sem),
candida albicans (candidiasis esofagiga definitoria de SIDA, tx. Fluconazol 14-21 dias) adquiridos previamente a la infeccin por VIH. De
acuerdo al INDRE, las infecciones oportunistas ms frecuentes en pacientes con SIDA en Mxico son: 1. Cndidiosis (37.6%) 2.
Neumona por Pneumocystis jiroveci (12.2%). Tuberculosis (11.1%) herpes simple (7.4%), y Herpes zoster (7.2%). Otras manifestaciones
caractersticas son las neoplasias malignas. El Sarcoma de kaposi (neoplasia mas comn en VIH, ndulos, placas, alargamiento de
linfonodos), se observa principalmente en adultos, siendo su presentacin poco frecuente en nios. Dentro de este grupo se incluyen
las neoplasias del SNC, como los linfoms no Hodking del cerebro (200 veces ms comn en pacientes con VIH, origen de cel. B,
sntomas fiebre, diaforesis, perdida de peso). Laboratorio: la demostracin de anticuerpos anti-VIH puede realizarse por varios
mtodos, como ELISA ms utilizado, sensibilidad 95%, especificidad 99%, en las primeras 8-12 semanas puede dar negativo , de ah que
si ELISA sale positiva, se recomienda realizar una segunda prueba ms especifica (prueba confirmatoria) y Western el cual reacciona
con los antgenos virales, esta prueba tiene especificidad del 100%. Se considera como persona infectada por VIH o serio positiva
aquella que presente: a) dos resultados positivos de pruebas de Tamizaje positivos (ELISA, aglutinacin o pruebas rpidas) y pruebas
confirmatorias de Western Blot positiva. b) dos resultados. Pruebas positivas de Tamizaje, con cuadro clnico sugerente de infeccin y c)
alguna prueba
suplementaria positiva (cultivo, Ag p24, PCR). EN infecciones oportunistas, Diagnostico de SIDA: infeccin declaratoria y CD4 <200/ml.
NEUMONA POR Pneumocystis jiroveci: Inicio insidioso, fiebre, tos no productiva, disnea, hipoxemia, linfadenopata generalizada,
diaforesis nocturna. Rx., de trax infiltrados intersticiales bilaterales, neumtorax espontneos, DHL elevada, CD4 <200cel/ml, lavado
bronco alveolar 85-90% sensibilidad, tratamiento de
eleccin TMP/SFX 15mg/kg/d 3-4 dosis por 21 das, se
ha demostrado que agregar esteroides al manejo
incrementa la sobre ida en pacientes con procesos
moderados- severos. Se debe dar profilaxis con
cuentas de CD4 menores de 200/mm
3
, suspenderla si alcanzan >200. CLASIFICACIN:
TRATAMIENTO: Los antiretrovirales (ARV) se debe iniciar en adultos asintomticos con linfocitos CD4 <500-l y en aquellos pacientes
sintomtico, independientemente de la cifra de estos, en sndrome de infeccin aguda, en infeccin crnica (sintomtica, o
asintomtica con CD4 <350/l o decreciendo y VIH RNA >50,000 copoias por ml o incrementando), py profilaxis postexposicion.
Actualmente se dispone de los siguientes AVR: 1. Inhibido res de la transcriptasa reversa anlogos de ncleosidos; zidovudina 200mg
ESTADIOS CLNICOS DE LA INFECCIN POR EL VIH POR LA CDC 1993
ESTADIO A ESTADIO B: Denotan progresin de la
enfermedad, pero que por s solas no
clasifican al paciente como enfermo de
SIDA
ESTADIO C: Define un caso de SIDA por medio de las entidades
clnicas asociadas que se presentan
Infeccin o
Sndrome
retroviral
agudo (SRA)
Infeccin
asintomtica
Linfadenopata
generalizada
persistente
(LGP)
Sintomtico (sin condicin A ni C)
Candidiasis orofarngea o vaginal, por ms
de un mes o que responde pobremente al
tratamiento
Leucoplasia vellosa de la lengua
Displasia cervical severa o carcinoma in
situ
Enfermedad plvica inflamatoria
Herpes zoster en ms de una dermatoma o
por ms de 2 episodios
Purpura trombocitopenica idioptica
Neuropata perifrica
Listeriosis
Angiomatosis bacilar
Sndromes constitucionales (diarrea por
mas de 1 mes, fiebre prolongada de origen
desconocido)

Candidiasis de bronquios, trquea o pulmones
Candidiasis esofgica
Cncer cervical invasivo
Coccidiodomicosis diseminada
Criptococosis crnica intestinal (>1mes)
CMV en cualquier rgano (excepto hgado, bazo o ganglios)
Encefalopata asociada a VIH
Herpes simplex, causando ulceraciones crnicas por ms de un mes
Histoplasmosis diseminada
Isosporosis crnica por ms de un mes
Sarcoma de Kaposi
Linfoma de Burkitt, inmunoblastico o linfoma primario de cerebro
Mycobacterium tuberculosis pulmonar o extrapulmonar
Mycobacterium avium intracellulare
Otras micobacterias diseminadas
Neumonia por P. jiroveci
Leucoencefalopatia multifocal progresiva
Septicemia recurrente por Salmonella
Toxoplasmosis cerebral
Sndrome de emaciacin con prdida de peso de >4.5 kg o ms de
10% del peso usual.
Sistema de clasificacin para la infeccin por VIH/SIDA revisada en 1993. (categora clnica)
Nmero de clulas CD4 A B C
> = 500/mm
3
A1 B1 C1
200-499/mm
3
A2 B2 C2
<200/mm
3
A3 B3 C3
MANUAL DE TRABAJO DEL CURSO ENARM CMN SIGLO XXI
CURSO ENARM CMN SIGLO XXI TEL: 36246001 Pharmed Solutions Institute PGINA 141

c/8hrs o 300 c/12hrs, toxicidad anemia, granulocitopenia, miopatia, didanosina 200 mg c/12hrs, toxicidad pancretitis, neurpatia
perifrica, lamivudina, estavudina, abacavir, emitricitabina 2. Inhibidores no ncleosidos de la transcriptasa reversa: nevirapina (NVP),
toxicidad sx. Steven Johnson, sedacin, hepatitis, efavirenz (EFV). Inhibidores de la proteasa; indavir (IDV) 800mg VO c/8hrs, toxicidad
sabor metlico, nausea, dolor abdominal, ritonavir (RTV), squinavir, amprenavir, lopinavir. Inhibidores de fusin: efuvirtide o T-20. EN
pacientes con SIDA la fibra es un motivo frecuente de consulta y hospitalizacin, determinar si la infeccin no se debe a bacterias, los
agentes frecuentes del proceso infecccioso pueden ser Streptococcus pneumoniae, Haemophilus, Salmonella, Staphilococcus.
Tuberculosis: algunos medicamentos antifimicos interaccionan con los ARV, en este caso retrasar el inicio de los ARV de 2-8 semnas. Se
puede presentar el sndrome de reconstitucin inmune: 8-43% de pacientes que inician terapia antirretroviral con Tb activa; se
manifiesta con fiebre, linfadenopata, empeoramiento de infiltrados pulmonares, derrame pleural.

CASO CLINICO
Se trata de masculino de 31 aos de edad el cual acude a consulta a solicitud de su esposa, ya que ha presentado cambios en su estado
cognitivo y alteraciones en sus actividades de la vida diaria, el paciente cuenta con antecedentes de VIH positivo de diagnostico
reciente, la esposa del paciente ha notado una disminucin significativa de las actividades de su esposo en la casa, ha sido sancionado
en su trabajo, adems se encuentra con afecto embotado, con disminucin de la atencin, no logra concentrarse, pierde el nucleo del
discurso, actualmente su cuenta de CD4 es de 380 con una carga vrica de 78 000/ml, el paciente presenta a la exploracin fsica signos
vitales sin alteraciones, el examen neurolgico no muestra datos de focalizacin, fondo de ojo normal, minimental 22/30, fue enviado a
IRM con atrofia cerebral desproporcionada e inespecfica sin lesiones focales.

PRECUNTA
Considerando el cuadro clnico cual es la conducta a seguir ms apropiada.

RESPUESTA
a.- Terapia antirretrovirica.
b.- PCR para virus JV en LCR.
c.- PCR para micobacteria en LCR.
d.- VDRL para LCR.

CASO CLINICO
Paciente de 30 aos, procedente de la ciudad de pergamino, provincia de Buenos Aires, que es ingresado por presentar fiebre, astenia,
tos productiva, disnea progresiva y sudoracin nocturna. Heterosexual, niega consumo de alcohol y drogas ilcitas, conoce su condicin
de VIH positivo desde hace 7 aos. En enero de 2006 se le diagnostica candidiasis esofgica. Recibi diversos esquemas de terapia
antirretroviral de gran actividad (TARGA) con mala adherencia y consecuentes fracasos teraputicos.

PREGUNTA
Cul es el agente causal ms probable en este caso.

RESPUESTA
a.- Cryptococcus neoformans.
b.- Aspergillus fumigatus.
c.- Mycoplasma kansasii.
d.- Pneumocystis jiroveci

GONORREA (NEISSERIA GONORRHOEAE):
CIENCIAS BASICAS: Enfermedad de transmisin sexual altamente contagiosa y cosmopolita, causada por Neisseria gonprrhoeae, es un
diplococo gramnegativo, aerobio, inmvil, oxidasa y catalasa positivo, que produce inflamacin de los epitelios columnares y
transicionales de la uretra, el recto la faringe y la conjuntiva. Es altamente susceptible a condiciones adversas del ambiente. SALUD
PUBLICA: Gonorrea contina ocupando el primer lugar entre las infecciones notificadas. En Mxico la tasa de morbilidad por 100,000
hab es de 1.17 en 2006. El grupo etario con mayor morbilidad es de los 15-20 aos. PATOGENIA: En la actualidad se tiende a considerar
que el portador subclinico (hombre o mujer) transmite la infeccin a la pareja, que podr o no desarrollar sintomatologa. Factores de
riesgo: trabajadores del sexo, homosexuales, viajeros, trabajadores migratorios, estudiantes, promiscuidad sexual. La gonorrea se
transmite por contacto directo, intimo generalmente sexual (ms comn de un hombre infectado a una mujer sana, que de una mujer
infectada a un hombre sano). Periodo de incubacin de 2-5 das, a los componentes de la superficie del gonococo se les ha hecho
responsables de la integracin con las clulas del hospedero, incluyendo adherencia e invasin a las clulas epiteliales. La vagina no es
infectada gracias al pH acido del moco vaginal que interfiere con la proliferacin del gonococo y a la actividad de un sistema bactericida
mediado por peroxidasa y dependiente de pH. La proctitis gonoccica en la mujer tienen origen en la contaminacin a partir de la via
genital o en el coito rectal, que es un mecanismo fundamental en el hombre; otra posibilidad es la ruptura de un absceso plvico o
prosttico. La diseminacin a partir del sitio mucoso primario se realiza por dos vas: 1. Linftico; Que lleva a las bacterias a la prstata,
epiddimo, glandula de Skene, Bartholin, Cowper, piel de area genital, trompas de Falopio, peritoneo y por contigidad al espacio
perihepatico y 2. Hemtico; que puede acompaarse de artritis, endocarditis, meningitis y dermatitis sptica. DIAGNOSTICO: Clnica;
en el hombre la uretritis aguda, es la presentacin ms frecuente, se inicia con descarga uretral y disuria, secrecin uretral mucoide,
que en pocas horas se torna purulento y blanco amarillento. La complicacin ms frecuente es la epididimitis, en casos poco frecuentes
puede haber linfangitis, absceso periuretral, prostatitis aguda, el paciente tienen un curso a febril. En la mujer el sitio principal de
infeccin es el endocervix, a veces en uretra, glndulas periuretrales y de Bartholin, hasta 90% de las mujeres puede ser asintomtico.
Cuando hay sntomas se relacionan con cervicitis y uretritis, los sntomas incluyen leucorrea, disurias y sangrado intermenstrual.
Teniendo como antecedente la exposicin orogenital puede encontrarse infeccin faringea en 10-20% de las mujeres, en 3-7% de
MANUAL DE TRABAJO DEL CURSO ENARM CMN SIGLO XXI
CURSO ENARM CMN SIGLO XXI TEL: 36246001 Pharmed Solutions Institute PGINA 142

heterosexuales y 10-25% de homosexuales. La conjuntivitis puede ser severa con exudado purulento y avanza rpidamente a ulceracin
cornea. EPI, el riesgo de desarrollarla en una mujer con gonococo es de 10-20% se manifiesta con una combinacin de endometritis,
salpingitis, absceso tubovarico. Las manifestaciones agudas y las secuelas a largo plazo, es una se las principales razones por la cual
debe prevenirse entre ellas histerectomas, embarazos ectopcos, con una frecuencia de 7 veces ms que alguien sin EPI. La infertilidad
por obstruccin tubaria es de 4%, despus de un episodio de EPI, 33% con dos ataques y 60% con tres cuadros. El Sndrome de FItz-
Hugh y Curts se presenta por extensin de la bacteria de las trompas de falopio a la cpsula del hgado. En embarazo genera abortos
espontneos, parto, parto preterimos, RPM, incremento en mortalidad perinatal. La bacteriemia se presenta en 0.5-3%, las
manifestaciones comunes son parte del sndrome artritis-dermatitis, con artralgias migratorias en rodillas, codos y articulaciones
distantes. La dermatitis caracterstica se encuentra en 75% de las pacientes, son papilas y pstulas con un componente hemorrgico, en
nmero de 5-40 y predominan en extremidades. En peditricos; conjuntivitis (oftalma neonatorum), es la ms reconocida y fue causa
de ceguera antes de la profilaxis con nitrato de plata, oh aplicacin tpica con eritromicina y tetraciclina. Laboratorio: tincin de gramm
(diplococos intracelulares) y prueba de estearasa leucocitaria en la descarga uretral si la rincn es positiva y hay ms de 5 leucocitos por
campo dar tratamiento para gonorrea y clamidia. Las muestras para aislamiento de N. gonorreae pueden obtenerse de tracto genital,
orina, ano, orofaringe, conjuntiva, glndulas de Bartholin, trompas de falopio, endometrio, liquido auricular, lesiones en piel. Se puede
usar un prueba directa para la deteccin de gonococo, que es la deteccin de antgeno y cidos nucleicos (PACE). Anticuerpos
fluorescentes (prueba polivalente y anticuerpo monclonal). TRATAMIENTO: El de eleccin en gonorrea aguda no complicada, es la
cefalosporinas, en uretritis, cervicitis y proctitis, dar tratamiento para clamidia ya que se reporta coinfeccion hasta en 50%. Primera
eleccin ceftriaxona 125mg IM una dosis o cefixime 400mg VO una dosis (por la alta resistencia a fluroquinolonas, ya no se
recomiendan), si hay resistencia ciprofloxacina 500mg VO una dosis u ofloxacina 400mgs. Para CLamydia una dosis de azitromicina 1 g
VO. Alternativas ceftizoxima, cefotaxima, cefotetan, probenecid. Infeccin gonococica diseminada (sepsis, artritis, meningitis);
ceftriaxona 1g IM o IV cada 24 hrs, alternativa cefotaxima 1 g IV cada 8 hrs un da o dos y continuar tratamiento por una semana VO
(cefixima 400mg c/12hrs). Prstatitis y epididimitis; ceftriaxona 250mgs IM dosis nica mas doxiciclina 100mg VO c/12 hrs por 10 das.
Profilaxis en conjuntivitis neonatal es con eritromicina oftlmica (0.5%) o tetraciclina oftlmica (1%), uno u otro en una sola aplicacin.

CASO CLINICO
Paciente mujer de 21 aos remitida a nuestro hospital por su mdico de familia con el diagnstico de conjuntivitis aguda purulenta
bilateral, por no responder al tratamiento tpico con la asociacin de gentamicina y dexametasona ni con la asociacin de neomicina,
polimixina y gramicidina. Como antecedentes personales la paciente refiere que es alrgica a las cefalosporinas. En la exploracin
presenta clnica en ambos ojos de edema palpebral severo, abundante secrecin purulenta, y epitelitis con lesin superior yuxtalmbica
en la crnea de ojo derecho (OD) que implica capas profundas del estroma, no observndose sta en el ojo izquierdo (OI). Se remiten
muestras de la secrecin conjuntival al servicio de microbiologa, y se pauta tratamiento emprico con ofloxacino y la asociacin
cloranfenicol y dexametasona tpicos en horas alternas, con limpieza previa de las secreciones con suero fisiolgico, y por va enteral
con ciprofloxacino, betametasona y dexclorfeniramina. En el cultivo se identifica Neisseria Gonorrhoeae, y el antibiograma demuestra
resistencia al ciprofloxacino y tetraciclinas, la paciente es alrgica a las cefalosporinas.

PREGUNTA
Cual es la conducta mas adecuada a seguir?

RESPUESTA
a.- Agregar cloranfenicol.
b.- Agregar vancomicina.
c.- Agregar imipenem.
d.- Agregar doxiciclina.

SIFILIS (TREPONEMA PALLIDUM):
CIENCIAS BASICAS: La sfilis es una Infeccin de Transmisin Sexual (ITS) causada por la espiroqueta, Treponema pallidum. Mide
aproximadamente en promedio 10 micrones de largo. Esta bacteria solo vive en los humanos. Se multiplica por divisin simple y tiene
mucho requerimiento de alimentos especiales para crecer en los medios de cultivos para bacterias. SALUD PBLICA: Segn datos de la
OMS, a nivel mundial existen 12 millones de nuevos casos de sfilis, correspondiendo a una incidencia mundial de la sfilis venrea del
0,4% y la prevalencia del 1%. De estos casos corresponde a Latinoamrica y el Caribe: 3 000 000 de casos por ao. El perodo donde
ms personas se contagian es entre los 20 y los 25 aos de edad. Se reportan anualmente un promedio de 330.000 casos de sfilis en
mujeres embarazadas en Amrica Latina y el Caribe que no reciben un tratamiento adecuado. Ello significa que todos los aos nacen
110.000 nios con Sfilis Congnita. PATOGENIA: La Sfilis es transmitida a travs del contacto sexual, ya sea por va vaginal, anal u oral.
Tambin por el contacto directo con la lesin o herida hmeda de la sfilis, a travs de heridas escoriaciones o fisuras no evidentes.No
dispone de toxinas pero su poder invasivo es extraordinario. Las lesiones de sfilis se les conocen como chancros, aparecen
principalmente en los genitales y no presentan dolor razn por la cual muchas personas no buscan ayuda mdica. Tambin puede
pasarse al besar o a travs del contacto manual u otro contacto personal cercano. Esta enfermedad se busca en toda gestante ya que
las mujeres embarazadas con sfilis pueden transmitir esta infeccin a sus bebs durante el embarazo, antes del parto, a esta forma de
enfermedad se llama Sfilis Congnita. Para la realizacin de transfusiones sanguneas tambin se busca Sfilis en la sangre. Su
transmisin es baja porque solo puede vivir no ms de 24 a 48 horas en la sangre que se conserva en el banco de sangre. En las
relaciones entre hombre y mujer es ms fcil que se contagie el hombre. Una persona que ha tenido Sfilis se puede volver a contagiar
de esta infeccin. La sfilis tiene varias etapas: Incubacin; 10-90 das con una mediana de 3 semanas. Sfilis Primaria; primera etapa,
chancro de inoculacin, es una erosin mucosa superficial, bordes indurados, indolora con secrecin serosa, acompaada de
adenopata regional unilateral, mas frecuentes en genitales externos, crvix, boca, regin perianal y canal anal. Estas lesiones aparecen
en promedio de 2 a 3 semanas despus del contacto con una persona infectada y se han descrito hasta 90 das despus. Las lceras
desaparecen en un perodo de 4 a 6 semanas. La Sfilis Secundaria; se presenta de 2 a 12 semanas despus de la aparicin del chancro,
MANUAL DE TRABAJO DEL CURSO ENARM CMN SIGLO XXI
CURSO ENARM CMN SIGLO XXI TEL: 36246001 Pharmed Solutions Institute PGINA 143

erupcin simtrica maculopapular en todo el cuerpo que incluye palma de manos y planta de pies, en la cavidad oral se ulceran.
Tambin pueden aparecer fiebre, inflamacin de los ganglios, dolor de garganta, dolor de cabeza, prdida de peso, dolores musculares
y fatiga. Los signos y sntomas de la Sfilis Secundaria desaparecern con tratamiento o sin tratamiento, pero la infeccin progresar
hasta la fase latente (enfermedad ms de un ao) y terciaria de la enfermedad, si no se administra ningn tratamiento. La persona
infectada seguir teniendo sfilis aun cuando no tenga ni signos ni sntomas; la infeccin permanece en el organismo. Sfilis Terciaria; en
ella la infeccin se puede dirigir al cerebro (sistema nervioso en general), al corazn, a la piel y a los huesos. Entre los signos y sntomas
se encuentran la dificultad para coordinar los movimientos musculares, parlisis, ceguera gradual, demencia y muchos otros signos
neurolgicos que pueden causar la muerte de la persona infectada. DIAGNSTICO: Para el diagnostico de la Sfilis Primaria es requisito
acudir al mdico quien mediante la observacin y examen del material obtenido de un raspado de la lesin (chancro) en un microscopio
observara a los treponemas causantes de la sfilis. El microscopio que utiliza se llama microscopio de campo oscuro. Tambin es posible
diagnosticar sfilis mediante exmenes de sangre. Existen pruebas llamadas No treponmicas como el RPR (Rapid Plasma Reagin) o
VDRL (Venerial Disease Research Laboratory) que permiten encontrar anticuerpos (defensas) producidas por el cuerpo contra el
Treponema. Otro examen llamado prueba treponmica FTA Abs (Fluorescent Treponemal Antibody Absorption) es una forma de
confirmar los hallazgos previos; es una prueba que indica que el individuo tuvo o no sfilis. Las mujeres embarazadas pueden infectar a
sus bebes con sfilis y esta infeccin podra causar la muerte del bebe. Por esta razn se toma a todas las mujeres gestantes estas
pruebas como parte obligatoria del control pre natal que llevan en los hospitales. Durante la gestacin la madre puede contraer sfilis
pudiendo infectar as a su bebe. De acuerdo al tiempo de infeccin el nio puede nacer con mltiples alteraciones, incluso muerto. Las
madres con diagnstico de sfilis deben de recibir tratamiento inmediatamente. Si tuvo tratamiento irregular (no completo) debe
drsele terapia a ella y al recin nacido. La sfilis del recin nacido recibe el nombre de Sfilis Congnita. Esta infeccin puede ser
temprana o tarda. La temprana, que se observa antes del segundo ao de vida, puede ser fulminante. Puede manifestarse como una
infeccin generalizada, o por lesiones en piel y en reas de mucosas (boca, ano), dolores seos o articulares, anemia, crecimiento del
hgado y del bazo y afectacin del sistema nervioso. La forma tarda, con una persistencia de ms de dos aos, puede originar
deformaciones de huesos y dientes, sordera, neurosfilis (al sistema nervioso) y otras manifestaciones terciarias. Las manifestaciones
clnicas son muy variables. En resumen, el nio puede tener sntomas que lo dejaran con consecuencias graves para su desarrollo
posterior. TRATAMIENTO: En los casos no tratados despus de la fase de latencia, ocurren recaidas en una cuarta parte de los casos
que conducen a nueva fase de secundarismo. La sfilis tiene cura. Si una persona ha tenido sfilis durante menos de un ao, la
enfermedad se curar con una sola inyeccin intramuscular de un antibitico llamado penicilina (2.4 millones de penicilina G
benzatinica en dos dosis separadas, IM profunda). Si una persona ha tenido sfilis por ms de un ao, necesitar de tres dosis de este
antibitico. En nios penicilina G benzatinica 50,000/kg IM una sola dosis. En etapas avanzadas, como en Sfilis Terciaria, 2.4 millones
de penicilina G benzatinica, semanalmente durante 3 semanas, no existen estudios que reporten eficacia de tratamientos alternativos o
en alrgicos a penicilina doxiciclina 100mgs c/12 hrs por 14 dias o tetraciclina 500mgs cuatro veces al da por 14 das. Lamentablemente
el antibitico no revertir las lesiones ya ocasionadas. Las personas que reciben tratamiento contra la sfilis deben abstenerse de tener
todo tipo de relaciones sexuales con parejas nuevas hasta que las lesiones sifilticas hayan desaparecido por completo. La pareja sexual
debe someterse a las de pruebas y recibir tratamiento si es necesario. Seguimiento del tratamiento En todos los pacientes con Sfilis
Primaria y Congnita hay que repetir las pruebas no treponmicas cuantitativas (en los que salen valores como por ejemplo 1/8)
llamadas RPR o VDRL al cabo de uno, tres, seis y 12 meses del tratamiento de la sfilis. Al cabo de 12 meses puede haberse negativizado
el 40-75% de casos de sfilis primaria y el 20-40% de las secundarias. Si a los 12 meses siguen siendo positivas, se hace necesaria una re
evaluacin.

CASO CLINICO
Masculino de 44 aos, alrgico a la penicilina, acude a nuestra consulta con su mujer porque desde hace 2 semanas presenta una lcera
en el glande no dolorosa y adenopatas inguinales. Niega haber tenido relaciones sexuales extramatrimoniales.

PREGUNTA
Cual es la conducta a seguir?

RESPUESTA
a.- Al negar relaciones sexuales extramatrimoniales podemos descartar enfermedades de transmisin sexual.
b.- Se trata de una sfilis primaria, trataremos con una dosis de penicilina G benzatina 2,4 MU i.m.
c.- Entre el diagnstico diferencial de la lcera genital hay las enfermedades de transmisin sexual. Debemos pedir serologas para
sfilis.
d.- Al tratarse de una lesin cutnea derivamos al paciente al dermatlogo de referencia.

PREGUNTA
Si finalmente fuera diagnosticado de sfilis primaria, cual es la conducta prescriptiva mas adecuada?

RESPUSTA
a.- Administraremos una dosis de penicilina benzatina 2,4 MU i.m.
b.- Administraremos penicilina benzatina 2,4 MU i.m. una administracin semanal por 3 semanas.
c.- Doxiclina 100 mg va oral dos veces al da por 14 das.
d.- Doxiciclina 200 mg oral dos veces al da durante 28 das.

CHLAMYDIA TRACHOMATIS:
CIENCIAS BSICAS: Es una infeccin de transmisin sexual bacteriana. La Chlamydia trachomatis, es un parasito intracelular obligado,
tiene al menos 18 serotipos, estos confieren tropismo celular y son especficos de enfermedad: serotipos A, B, Ba y C estn asociados
con tracoma, los del D al K estn asociados con infecciones de transmisin sexual y la perinatal, los tipos L1, L2 L3 son los ms invasivos,
MANUAL DE TRABAJO DEL CURSO ENARM CMN SIGLO XXI
CURSO ENARM CMN SIGLO XXI TEL: 36246001 Pharmed Solutions Institute PGINA 144

se extienden al tejido linftico y producen el sndrome clnico de linfogranuloma venreo y proctocolitis hemorrgica. SALUD PUBLICA:
Causa 30-40% de las uretritis no gonococica. La OMS calcula que existen 89 millones de casos de infeccin por Chlamydia trachomatis
en el mundo. Las secuelas que puede traer esta infeccin son las ms graves y costosas de todas las ETS, exceptuando la infeccin por
VIH. Se ha demostrado que la infeccin por C. trachomatis, puede facilitar la transmisin por VIH. La exposicin a chlamydia, es una
causa importante de infertilidad tubaria. Produce un 13.8% de conjuntivitis neonatal. PATOGENIA: la infeccin se adquiere por
contacto sexual, puede ocurrir colonizacin en el recin nacido de una madre infectada. Las infecciones ms frecuentes son uretritis en
el hombre y cervicitis en la mujer. Estos sitios son la va de entrada del organismo. Tiempo de incubacin de uretritis es de 7-14 das, en
mujeres no est definido. La respuesta inmunolgica parece tener un papel principal en la fisiopatogenia. Una protena de choque de
calor, tiene 50% de homologa con otras protenas de choque de calor humana. Por lo tanto esta respuesta inmune iniciada por la
infeccin puede dar lugar a respuestas dainas. DIAGNOSTICO: Clnico; insidioso, hay secrecin uretral o cervical hialina, disuria, ardor
a la miccin, dispareuna ocasional, cervicitis y bartolinitis. La C. trachomatis no puede invadir el epitelio vaginal en mujeres adultas,
pero podra causar vaginitis en prepberes. Exploracin fsica, crvix con edema, eritema e hipertrofia con descarga mucopurulenta. El
sndrome uretral agudo se define como disuria y polaquiuria, descarga uretral, que es blanca, gris o hialina no purulenta. En
homosexuales es comn proctitis y proctocolitis, los sntomas iniciales son prurito anal y descarga rectal mucosa o mucopurulenta. La
mucosa se ulcera y se presenta un proceso inflamatorio crnico en la pared del intestino, con grnulosas no caseosas y abscesos en las
criptas. Hasta 8% de las mujeres desarrolla salpingitis, aunque se prefiere mas EPI, el espectro de EPI por Chlamydia va desde una
enfermedad grave, con perihepatitis y ascitis a la salpingitis silenciosa teniendo como secuela infertilidad, que se atribuye a
inflamacin, cicatrizacin y oclusin tubaria. Los embarazos ectopcos se presentan 8 veces ms en pacientes que han tenido infeccin
por Chlamydia. LINFOGRANULOMA VENREO: Es endmico en frica, India, Sudamrica y Caribe, tiene 3 estadios: primero pequea
ppula o ulcera herpetiforme, en la mucosa genital o piel adyacente, aparece entre 22-30 das despus de adquirir la infeccin y
cicatriza sin dejar rastro. Segundo, se caracteriza por linfadenopata y sntomas sistmicos (fiebre, cefalea, mialgias),en hombres los
linfonodos inguinales son los ms afectados. Los linfonodos son blandos, con eritema leve, la inflamacin se extiende y forma una
masa, los abscesos dentro de la masa coleasen y forman un bubn que se puede romper espontneamente. Tercero, cronicidad de la
infeccin, con desarrollo de abscesos loculados, fstulas, estenosis asi como elefantiasis genital. La ruptura del bubn alivia los sntomas
y puede drenar durante semanas o meses. TRACOMA: Es una queratoconjuntivitis folicular crnica con neovascularizacion corneal, que
se produce por infecciones repetidas, puede producir ceguera como consecuencia. El tracoma es la principal causa de ceguera
infecciosa en el mundo. NEUMONA: curso insidioso, ms comn en nios de 1-3 meses de edad, caracterizados por accesos de tos
paroxistica, general ente sin fiebre. Laboratorio: La C. trachomatis solo se puede cultivar en clulas McCoy o HeLa, no se visualiza por
tincin gram, por lo que es necesario buscar las tpicas inclusiones intracelulares mediante una tincin de Giemsa. ELISA para identificar
antgenos especficos. En caso de identificar IgG se evaluara el incremento de anticuerpos especficos de la fase aguda y en la fase de
convalecencia o se determinara IgM especfica en la fase aguda. Cuatro tipo de procedimientos confirmatorios estn disponibles: 1.
Evaluacin por microscopia directa de los exudados o raspado del tejido utilizando tincin con anticuerpos fluorescentes. 2.
Aislamiento del organismo en cultivos celulares. 3. Deteccin de antgenos o genes de Chlamydia por medios inmunolgicos o pruebas
de amplificacin de cidos nuclecos. 4. Pruebas serolgicas en busca de anticuerpos contra C. trachomatis. TRATAMIENTO: para
infeccin genitourinaria en adolescentes mayores y adultos, se usa tetraciclina 500mg VO, cuatro veces al da por 7-10 das. Como
alternativa se puede utilizar doxiciclina, sulfonamidas o eritromicina, siendo esta ultima el tratamiento de eleccin en la embarazada.
En linfogranuloma venreo los mismos medicamentos pero no menor a dos semanas. El recomendado es azitromicina 1gr VO dosis
nica o doxiciclina 100mgs VO c/12 hrs por 7 das. Alterativas eritromicina o ciprofloxacina. Para el tracoma tetraciclina oral, mas tpica
o doxiciclina 100mg c/12hrs por 21 das. Para conjuntivitis neonatal 30-50mg/kg/da en 4 dosis durante 14 das, junto con tratamiento
oftlmico tpico con eritromicna o tatraciclina.

CASO CLINICO
Un hombre de 30 aos de edad, con presencia de sangrado con moco por via rectal varias veces al dia por tres semanas. La
sigmoidoscopia reporto mucosa granular con varias lceras a 5-7 cm distales del recto. La biopsia rectal reporto colitis irregular
severamente activa con criptas distorsionadas semejante a la criptitis de la enfermedad de Crohn. El historial clnico adicional revel
que el paciente tuvo relaciones sexuales anales sin proteccin durante el viaje a travs de Europa tres meses antes. Se confirm la
presencia de C. trachomatis.

PREGUNTA
Cual es la conducta teraputica a seguir mas adecuada?

RESPUESTA
a.- Tetraciclina
b.- Doxiciclina
c.- Eritromicina.
d.- Ciprofloxacino.

CASO CLINICO
Masculino de 28 aos de edad, VIH positivo se presenta con sangrado rectal. La anoscopa revel una lcera en el borde anal. Las
biopsias mostraron fragmentos de piel anal con ulceracin, tejido linfoide y tejido de granulacin inflamatorio histolgicamente
sospechoso de una enfermedad inflamatoria del intestino. La sigmoidoscopia mostr una mucosa eritematosa en el recto distal con
pequeas reas de exudados blancos y pliegues apilados. Las biopsias mostraron proctitis linfohistiocitario grave y criptitis. Las pruebas
serolgicas fueron positivos para C. trachomatis .

PREGUNTA
Cual es la conducta teraputica a seguir mas adecuada?
MANUAL DE TRABAJO DEL CURSO ENARM CMN SIGLO XXI
CURSO ENARM CMN SIGLO XXI TEL: 36246001 Pharmed Solutions Institute PGINA 145


RESPUESTA
a.- Tetraciclina
b.- Doxiciclina
c.- Eritromicina.
d.- Ciprofloxacino.

CASO CLINICO
Mujer de 48 aos, nulpara, con antecedente de un embarazo ectpico tubario derecho, que ingresa para una histerectoma electiva
por adenomiosis sintomtica, complicada por una anemia secundaria crnica. Se realiz histerectoma total, con conservacin de
ovarios, por va laparoscpica, sin incidentes. Una vez terminada la ciruga, se realiz la exploracin de la cavidad abdominoplvica,
incluyendo los rganos y paredes, tal como lo hacemos en forma metdica y rutinaria en toda ciruga laparoscpica. Al observar la
superficie heptica, se encontraron adherencias en "cuerda de violn", entre el hgado y la pared abdominal. Durante el postoperatorio,
se le comunic a la paciente el hallazgo y se pregunt a la paciente por antecedente de dolor abdominal en hipocondrio derecho, ante
lo cual relat no tenerlo. Sin embargo, record haber sido tratada con antibiticos, por flujo vaginal persistente 15 aos antes.

PREGUNTA
Cual es el diagnostico diferencial mas probable, considerando los hallazgos del caso?

RESPUESTA
a.- Sndrome de Beckwith Wiedemann.
b.- Sndrome de Fitz-Hugh-Curtis.
c.- Sndrome de Simpson-Golabi-Behmel.
d.- sndrome de Klippel-Trenaunay-Weber

CASO CLINICO
Una mujer de 23 aos present ojo irritado y rojo durante 4 semanas, con sensacin de prdida parcial de la audicin en el odo
derecho durante 5 das, refiriere adems flujo vaginal de color amarillo durante 1 semana. Se diagnostico conjuntivitis folicular del ojo
derecho, cervicitis mucopurulenta, y un derrame en el odo derecho confirmado por timpanometra. Audiometra de tonos puros
mostraron prdida de audicin conductiva en el lado derecho. Se aislo C. trachomatis del ojo derecho, el cuello uterino, y la
nasofaringe, pero no del aspirado del odo medio.

PREGUNTA
Cual es la conducta teraputica a seguir mas adecuada?

RESPUESTA
a.- Tetraciclina
b.- Doxiciclina
c.- Eritromicina.
d.- Ciprofloxacino.

HERPES GENITAL (VIRUS HERPES SIMPLE TIPO 1 Y 2):
CIENCIAS BASICAS: El Herpes Genital es una Infeccin de Transmisin Sexual (ITS) causada por el Virus del Herpes Simple, existen 2: el
tipo 2 (VHS-2) que es el principal causante de esta ITS (aproximadamente 90% de los casos) y el Virus del Herpes Simple tipo 1 (VHS-1)
en aproximadamente el 10% de casos. La palabra herpes deriva del griego herpein que significa arrastrar. Los griegos lo refieren
en sus escritos como dolores que se arrastraban por la piel. SALUD PUBLICA: En Mxico ha existido un incremento en las tasas por
100,000 habitantes actualmente es de 4.07 y 4.61. En EE.UU los casos de herpes genital en adulto varan de 200,000- 30,0000. La
infeccin por VHS-2 genital es ms frecuente en las mujeres (aproximadamente una de cada cuatro mujeres) que entre los hombres
(casi uno de cada cinco). Esto puede deberse a que es ms probable que ocurra la transmisin de hombre a mujer que la transmisin de
mujer a hombre. La prevalencia de infeccin herptica cervical o vulvar en la mujer embarazada es de 1%. El riesgo de infeccin
neonatal asociada a infeccin materna primaria, al momento del parto alcanza hasta 75%. PATOGENIA: El virus requiere transporte en
fluidos corporales (semen, fluidos de tracto genital femenino)o a travs de las lesiones vesiculosas, ingresa a las clulas subdrmicas e
inicia su replicacin en el nucleo, despus las partculas virales son transportadas a travs de ramas nerviosas a los ganglios neuronales
de las races dorsales, aqu es donde permanece de forma latente. DIAGNOSTICO: Clnico; Aproximadamente luego de 2 semanas de
haber adquirido el VHS-2 por contacto sexual, a nivel de la zona genital o regin anal, se presenta una sensacin de calor o quemazn,
escozor y se torna la piel de un color rosado en algunas ocasiones. Posteriormente aparecen las vesculas o ampollas, pstulas que se
ulceran y forman una herida o llaga llamada lcera genital que es dolorosa. Esta lcera se comienza a curar formndose una costra en
su superficie que finalmente desaparece. Acompaando a estas lesiones, la persona con Herpes Genital puede presentar leve sensacin
de fiebre, cefalea ataque al estado general, dispareuna, incontinencia urinaria y linfadenopata inguinal. Desde que aparecen las
lesiones vesiculares hasta que desaparecen pueden pasar de dos a cuatro semanas. En algunas ocasiones se puede presentar otro brote
cercano al primero pero es de menos intensidad y duracin. Esta enfermedad es muy contagiosa, especialmente cuando estn
presentes las ampollas. Tanto el VHS-1 como el VHS-2 pueden encontrarse en las lceras causadas por los virus en los genitales y
pueden ser liberados por las mismas, pero entre brote y brote los virus tambin pueden ser liberados por la piel que no parece afectada
o que no tiene lceras. Esto significa entonces que existen personas contagiando a sus parejas sin saberlo. Una persona solo puede
infectarse con el VHS-2 durante las relaciones sexuales con alguien que tiene la infeccin. En conclusin la transmisin puede darse a
partir de una pareja sexual infectada que no tiene una lcera visible y que no sepa que est infectada. La cervicitis en la mujer puede
MANUAL DE TRABAJO DEL CURSO ENARM CMN SIGLO XXI
CURSO ENARM CMN SIGLO XXI TEL: 36246001 Pharmed Solutions Institute PGINA 146

ser sintomtica, con secrecin vaginal purulenta o asintomtica; los signos clnicos en crvix pueden variar de reas locales friables
eritematosas hasta lesiones ulcerosas extensas en exocrvix o cervicitis necrtica extensa. Entre las complicaciones ms graves de una
infeccin por herpes genital se encuentran la meningitis asptica y la mielitis transversa. Laboratorio: debe ser confirmado, ya que el
diagnostico clnico es no sensible e inespecfico. El diagnostico se establece mediante el aislamiento del virus con cultivo cel ular de una
muestra tomada de las lesiones, observando clulas gigantes multinucleadas con cuerpos de inclusin eosinofilos intranucleares El
Tzank es rpido pero con baja sensibilidad y especificidad, y no distingue entre tipos de VHS. Anticuerpos monoclonales contra
antgenos especficos de virus de herpes. HERPES Y GESTACIN: El Herpes Genital puede causar infecciones potencialmente mortales
en los bebs. Si una mujer tiene el diagnstico de Herpes Genital, se debe programar un parto tipo cesrea. En 85% de los casos se
adquiere en el canal del parto por contacto con las secreciones maternas infectadas. La forma clnica ms frecuente en un 40% es la
infeccin de piel, ojos y boca, esta forma prcticamente no tiene mortalidad, deja dao en el sistema nervioso en 30% de los recin
nacidos no tratados y en 2% de los que reciben antivirales. El paciente puede presentar ictericia, hepatoesplenomegalia, alteraciones
hematolgicas, microftalmia, microcefalia, convulsiones y a veces conjuntivitis, coriorretinitis Los herpes neonatales se presentan como
encefalitis en 35% de los casos y como infeccin diseminada en 25% de los casos; en ambos tipos de manifestacin las complicaciones y
la mortalidad son elevadas. TRATAMIENTO: Hasta el momento no existe cura para esta enfermedad. Se utilizan medicamentos
llamados Antivirales cuya funcin es acortar y prevenir los brotes. El tratamiento ms utilizado es el Aciclovir 200mg VO 5 veces al da
por 7 das (400mg c/8hrs 7-10 das) aunque existen medicamentos ms modernos como el Famciclovir (250mg c/8hrs VO 7-10dias) y el
Valaciclovir (1g c/12hrs VO 7-10 das) que son ms fciles de tomar para los pacientes. Se puede alargar si la formacin de costras es
incompleta despus de 10 das. Para enfermedad diseminada, Herpes Neonatal y pacientes inmunocomprometidos se debe de usar el
Aciclovir endovenoso. Pacientes con cuadros a repeticin de Herpes Genital deben de ser evaluados por un mdico quien le prescribir
tratamiento preventivo llamado profilctico o supresivo (aciclovir 400mgs diarios hasta por 6 aos) para reducir a posibilidad de
transmisin a sus parejas sexuales. Neonatos aciclovir 30mg/kg/dia IV dividido en 3 aplicaciones diarias por 10-21 das.

CASO CLINICO
Un hombre de 34 aos de edad, ingeniero, previamente sano, present en el curso de 24 horas, fiebre, retencin urinaria, debilidad de
las extremidades inferiores y dficits sensoriales irregulares en el trax y abdomen. Como nico antecedente, comunic una tos leve los
ltimos 4-5 das antes de admisin a la clnica. Una evaluacin urolgica descart uropata obstructiva. Al examen neurolgico estaba
alerta, febril (38C), sin dficits en nervios craneales. La fuerza en los miembros superiores era normal, pero se observ una debilidad
moderada en las extremidades inferiores con incapacidad para levantarse desde una posicin sentada. Los reflejos plantares eran
extensores, haba dficit sensorial en la parte posterior del cuello y a nivel del dermatoma D6. El anlisis del LCR mostr 35 glbulos
blancos por mm3, con predominio de clulas mononucleares. La protenorraquia fue 75 mg/dl. El LCR fue positivo para VHH-7. La
resonancia magntica (RNM) cerebral fue normal y la RNM de mdula evidenci lesiones inflamatorias, difusas, extensas en los
segmentos cervical, y dorsal de la mdula.

PREGUNTA
Cual es la conducta teraputica a seguir mas adecuada?

RESPUESTA
a.- Aciclovir
b.- Famciclovir
c.- Valaciclovir
d.- Ganciclovir

CASO CLINICO
Un hombre de 27 aos, publicista, previamente sano, present cuadro de de 5 das de duracin caracterizada por cefalea persistente,
fiebre (38,5C), malestar general y diarrea. En el examen estaba alerta, sin anormalidades en el examen neurolgico; en particular no se
detectaron signos menngeos. El hemograma, velocidad sedimentacin, proteina C reactiva y otros cultivos fueron normales. El LCR
mostr 160 clulas, predominio mononucleares y la protena fue de 75 mg/dl. La deteccion viral por PCR identific VHH-7 en LCR; el
mismo test fue negativo en sangre y tambin negativo al repetirlo al cabo de 1 semana en la muestra inicial de LCR. HIV y HTLV-1
fueron negativos.

PREGUNTA
Cual es la conducta teraputica a seguir mas adecuada?

RESPUESTA
a.- Aciclovir
b.- Famciclovir
c.- Valaciclovir
d.- Ganciclovir

ARTRITIS REUMATOIDE (AR)
CIENCIAS BASICAS: Es las ms comn de las artritis inflamatorias, asociada a HLA-DR4. SALUD PUBLICA: Relacin M:H de 3:1; se
presenta en 0.5-1% de la poblacin. En Mxico 1 milln 700mil personas entre 20 y 50 aos padecen artritis reumatoide. 5-20%
presentan curso autolimitado; otro 5-20% forma clnica mnimamente progresivamente; 60-90% forma de artritis de deterioro
progresivo. PATOGENIA: Antgeno desconocido + predisposicin gentica; genera una respuesta inmunitaria y reaccin inflamatoria con
activacin de cel. Plasmticas y linfocitos T CD4 > T CD8, los cuales infiltran la sinovial, forman tejido de granulacin por activacin de
fibroblastos (pannus), con hiperplasia de clulas mviles; todos estos cambios generan las manifestaciones de la artritis por citocinas
MANUAL DE TRABAJO DEL CURSO ENARM CMN SIGLO XXI
CURSO ENARM CMN SIGLO XXI TEL: 36246001 Pharmed Solutions Institute PGINA 147

secretadas por los macrfagos y destruccin celular y sea por citocinas formadas en el pannus. DIAGNOSTICO: Cuadro clnico:
poliartrits crnica (>6 sem) simtrica, afecta principalmente manos (metacarpofalangicas 90%, carpo 80%), rodillas 60% y pies
(metacarpo e interfalangicas), ms de 4 articulaciones afectadas. Rigidez matutina >1hr y dolor inflamatorio que mejora con la actividad
fsica. Astenia adinamia, hiporexia, cervicalgia. Desviacin cubital de dedos, ndulos de Bouchar (a nivel de interfalangicas proximales),
sinovitis, dedos en cuello de cisne o botonero, hallux valgus (en MsPs), pulgar en Z, ndulos reumatoideos (subcutneos en
superficies de extensin). Tambin hay manifestaciones extraarticulares principalmente a pulmn generando dolor pleurtico y derrame
pleural (25%), hematolgicas: anemia normocitica normocromica (30%), queratoconjuntivitis Sicca (25%) ojo seco, Sx. del tnel del
carpo por atrapamiento. Factor reumatoide (FR) positivo en 65% de los pacientes, son autoanticuerpos dirigidos contra IgG, no es tan
especficos porque puede encontrase en oras enfermedades inflamatorias autoinmunes. La presencia de FR muy elevado, se relaciona
con formas mas agresivas de la enfermedad. Anticuerpos anti-peptido cclico citrulado (Anti-PCC) mas especifico, tiene una
especificidad de 90-95%. VSG y PCR; siempre deben de pedirse, ambos se encuentran elevados. Anti-DNA negativo (especifico para
LES). Cambios Rx: inflamacin de articulaciones metacarpofalangicas, prdida de espacio de articulaciones metacarpo e interfalangicas
y radiocarpiana, erosiones oseas, osteopenia yuxtaarticular, luxacin de metacarpofalangicas, ndulos de Bouchar. Criterios Dx: Rigidez
matutina al menos 1 hora o mas. Artritis de 3 o mas articulaciones. Artritis de las manos. Artritis simetrica. Nodulos reumatoideos. FR
positivo. Cambios radiolgicos; Diagnostico de artritis con 4 de 7 criterios. COMPLICACIONES: Derrame lateral de rodillas de predominio
izq., puede presentarse quiste de Barker en hueco poplteo asociado a ruptura e invasin muscular. TRATAMIENTO: Metotrexate piedra
angular 10 mgs inicial, se puede aumentar hasta 25 mgs/sem.. Prednisona no es de eleccin, se da en dosis bajas y no modifica el curso
de la enfermedad. Los AINES se dan junto con los FARMES (frmacos modificadores de la enfermedad). En artritis activa leve:
Hidroxicloroquina (HCQ) o sulfazalazina (SSZ) ambos tardan 2-3 meses en iniciar su accin. Artritis activa moderada a severa: iniciar con
metotrexate, y considerar prednisona. Si hay una respuesta inadecuada agregar otro FARME (MTX + Abatacep MTX + leflunomide
MTX + SSZ + HCQ MTX + Rituximab). Los biolgicos (anti-TNF) no son actualmente de 1ra. Eleccin. PRONOSTICO: Empeoramiento
progresivo de la salud, ya que daa permanentemente los huesos, cartlagos y articulaciones. Si FR y ANTI-PCC muy elevados indican
mal pronstico y mayor gravedad. PREVENCION: En los 2 primeros aos de inicio de la enfermedad dao severo e irreversible. Un
tratamiento precoz disminuye y previene el dao. Derivacin precoz ante sospecha dx. A reumatologa. Toda paciente requiere contar
con radiografa de manos y pies en AP y oblicuas. La suspensin del habito tabquico puede ayudar a prevenir el desarrollo de AR.
CLAVES: Ndulos de Heberden en interfalangicas distales nos habla de osteoartritis. Raro debut con poliartritis aguda con fiebre y
linfadenopatas, pensar en rotavirus, si hay monoartritis en extremidades inferiores pensar espondiloartropatias seronegativas aun mas
si se acompaa de lumbalga. Sndromes especficos en pacientes con AR: Sx. de Sjogren secundario= AR + Sx. SICCA (xerostoma,
xerostalma). Sx. de Felty= AR+ esplenomegalia+ leucopenia. Sx. de Caplan= AR + ndulos reumatoideos + neumoconiosis. Amiloidosis =
AR de larga evolucin y grave proteinuria e Insf. Renal

















MANUAL DE TRABAJO DEL CURSO ENARM CMN SIGLO XXI
CURSO ENARM CMN SIGLO XXI TEL: 36246001 Pharmed Solutions Institute PGINA 148

NEOPLASIA DEL SISTEMA NERVIOSO CENTRAL
Epidemiologia: la incidencia de los tumores del SNC oscilan entre el 5.70 y el 9.63 en hombres y entre el 4.71 y el 6.95 en mujeres. Sin
embargo, en todas las series los tumores ms frecuentes son los gliomas, y dentro de ellos los glioblastomas. En cuanto a la edad de
presentacin, los tumores del SNC muestran una distribucin bimodal, con un pico en la edad peditrica y un aumento progresivo de la
incidencia en los adultos, hasta alcanzar su mximo entre la sexta y la sptima dcada de la vida. La incidencia combinada de tumores
del SNC en EEUU fue de 6,6 casos nuevos por 100.000 habitantes-ao, en tanto que la mortalidad se estim del 4,7 por 100.000
habitantes/ao. Clasificacin: El grado I incluye tumores de bajo potencial proliferativo y posibilidad de curacin con extirpacin
quirrgica completa. El grado II incluye tumores
con capacidad infiltrativa y capacidad de
recurrencia y progresin a mayor grado de
malignidad, a pesar de presentar un bajo nivel
de actividad proliferativa (por ejemplo, los
gliomas de bajo grado pueden progresar a
gliomas anaplsicos y/o glioblastoma). El grado
III incluye tumores con evidencia histolgica de
malignidad, como atipia nuclear y alta actividad
mittica. El grado IV incluye tumores con
evidencia histolgica de malignidad (atipia
nuclear, alta actividad mittica, necrosis,
proliferacin microvascular) y comportamiento
clnico agresivo con rpida progresin,
recurrencia a pesar de tratamiento intensivo y
muy alta mortalidad. Algunos tumores de grado
IV como los meduloblastomas y tumores de
clulas germinales son rpidamente fatales si
no se tratan, pero se asocian a altas tasas de
respuesta y supervivencia con tratamiento
adecuado. Aunque no son requisitos
indispensables, la capacidad de infiltracin
difusa del tejido sano adyacente y la
propensin a diseminacin leptomenngea (e incluso extraneural) son caractersticas de algunos tumores de grado IV. Diagnstico: Los
tumores del sistema nervioso suelen presentarse con sntomas neurolgicos de instauracin progresiva, aunque tambin es posible la
presentacin aguda (por ejemplo, crisis comiciales o dficit neurolgico agudo secundario a hemorragia intratumoral). As mismo, los
sntomas pueden ser generalizados (por aumento inespecfico de la presin intracraneal) o focales (dependientes de la localizacin
tumoral).La cefalea es el sntoma de presentacin ms frecuente de los tumores cerebrales, aunque como sntoma aislado slo se da
entre un 2 y un 16% de los casos. Las crisis comiciales son el segundo sntoma en frecuencia, pudiendo desarrollarlas hasta un 35% de
pacientes. Son especialmente habituales en gliomas de bajo grado. Pueden tener un inicio focal y reflejar la localizacin del tumor,
aunque en ms de la mitad de los casos se produce una generalizacin secundaria y el inicio focal puede ser muy difcil de identificar en
base a criterios clnicos. Otros sntomas frecuentes de presentacin son la disfuncin cognitiva, las nauseas y vmitos, la disfuncin
endocrina, los sntomas visuales y los sntomas focales dependientes de la localizacin tumoral. Diagnstico por imagen: La tcnica de
neuroimagen de eleccin para el diagnstico y seguimiento de los tumores del sistema nervioso es la resonancia magntica (RM). La
tomografa computarizada (TC) presenta limitaciones importantes en esta patologa, como son su peor resolucin anatmica y la
presencia de artefactos de fosa posterior. A pesar de ello, la TC constituye una tcnica muy til para la evaluacin de complicaciones
hemorrgicas agudas, efecto masa y extensin del edema vasognico asociado al tumor. Es importante recordar que la TC craneal
puede aportar falsos negativos en el diagnstico inicial de un tumor cerebral, en situaciones como tumores pequeos o con poca
densidad celular, masa tumoral isodensa, ausencia de cambios patolgicos significativos en la barrera hematoenceflica, o
estabilizacin de la barrera hematoenceflica por uso previo de corticoides. Existe un conjunto de caractersticas radiolgicas que
pueden orientar hacia el grado de malignidad en la TC, como son: nmero, forma y tamao de las lesiones, localizacin, mrgenes del
tumor, presencia de quistes, calcificaciones o hemorragias, presencia de necrosis intratumoral, edema perilesional, desplazamiento de
estructuras de la lnea media y captacin o no de contraste la lesin. En la TC la imagen de un tumor puede ser hipo, iso o hiperdensa.
Diagnstico anatomopatolgico y tcnicas de biologa molecular: Una vez establecida la sospecha clnica y radiolgica, el diagnstico
de certeza de una neoplasia cerebral requiere el estudio directo del tejido tumoral obtenido mediante biopsia o reseccin quirrgica. El
objetivo es confirmar el diagnstico de un tumor y establecer su naturaleza, tambin dar informacin pronstica e incluso predictiva de
respuesta al tratamiento, como en el caso de la codeleccin 1p 19q en oligodendrogliomas, o la metilacin del promotor de la enzima
reparadora de ADN O-6-metil-guanina-ADN-metil-transferasa (MGMT) en glioblastomas. Tratamiento: La ciruga suele ser el primer
paso en el tratamiento de los tumores primarios del sistema nervioso. En toda ciruga debe plantearse cul es el beneficio esperable,
que deber compararse con el riesgo existente para poder indicar la intervencin. El beneficio esperable puede incluirse en alguno de
estos aspectos: supervivencia, calidad de vida, diagnstico y obtencin de material para investigacin. En determinados tumores
(meningiomas, neurinomas, gliomas grado I de la OMS) la ciruga puede ser curativa por s misma si se extirpa totalmente la lesin. En
gliomas de grados II, III y IV, la evidencia acerca del beneficio de la ciruga sobre la supervivencia es limitada, pero hoy en da se acepta
que los pacientes con una extirpacin amplia tienen mayor supervivencia. El efecto de la ciruga sobre la calidad de vida puede ser
positivo o negativo. El principal perjuicio ocurre si se produce un dficit neurolgico; por ello el objetivo debe ser siempre conseguir la
mxima extirpacin sin provocar dficit. Los ndices de morbilidad actuales se han estimado en torno al 13%, mientras que se considera
que la mortalidad en craneotomas por tumor debe ser inferior al 2%. Una ciruga sin complicaciones produce un efecto beneficioso en
la calidad de vida, al permitir la desaparicin o rpida mejora de los sntomas y la rpida retirada de corticoides. La radioterapia (RT) es
un tratamiento til y eficaz en mltiples neoplasias primarias y secundarias del sistema nervioso central. Se basa en la induccin de
MANUAL DE TRABAJO DEL CURSO ENARM CMN SIGLO XXI
CURSO ENARM CMN SIGLO XXI TEL: 36246001 Pharmed Solutions Institute PGINA 149

dao en el ADN y orgnulos de las clulas malignas, con lo que consigue producir apoptosis y reduccin de la masa tumoral. Sin
embargo, es necesario reducir al mximo el rea de parnquima sano tratado, pues causa tambin desmielinizacin, dao neuronal y
cambios vasculares en el tejido normal. La dosis habitual de los gliomas de alto grado es 60 Gy administrados en fracciones de 1,8-2
Gy/da aplicados al tumor y un rea peritumoral de 2 cm. Los gliomas de bajo grado pueden tratarse con dosis ms bajas. Las nuevas
tcnicas como RT conformada 3D e IMRT consiguen ajustar mejor el rea a tratar y la radiociruga permite tratar de forma muy precisa
volmenes pequeos con mnima afectacin del parnquima circundante. Sin embargo, la radiociruga tiene un papel muy limitado en
el tratamiento de los tumores primarios, siendo su principal indicacin las metstasis cerebrales. Quimioterapia y Agentes Biolgicos:
se basa en tres pilares fundamentales, que son la reseccin quirrgica, la radioterapia y la quimioterapia. El papel de esta ltima ha
cambiado en los ltimos aos, al pasar de su uso limitado en determinados tumores (meduloblastomas, tumores germinales) a formar
parte la temozolomida del tratamiento estndar de primera lnea del glioblastoma, el tumor primario cerebral ms frecuente.
Adicionalmente, cada vez ms estudios muestran que temozolomida es activa en otras neoplasias como los gliomas de bajo grado y los
oligodendrogliomas. Adems de la temozolomida, otros frmacos quimioterpicos usados en el tratamiento de los tumores cerebrales
primarios incluyen, entre otros, las nitrosoureas (BCNU, CCNU), procarbazina, vincristina, derivados del platino, metrotrexate y
citarabina. El descubrimiento de las alteraciones existentes en las vas moleculares que regulan la multiplicacin celular, el crecimiento
celular, la invasividad de tejidos y la angiognesis est permitiendo disear frmacos que bloquean o revierten dichas alteraciones. Este
tipo de frmacos, creados especficamente para inhibir vas moleculares concretas, son los llamados targeted therapies o
tratamientos anti-diana. El primero de ellos que ha llegado a la prctica clnica es el anticuerpo monoclonal anti-VEGF bevacizumab
(Avastin) en gliomas malignos recurrentes, pero existen muchos otros que estn siendo evaluados en ensayos clnicos con resultados
prometedores. Tratamiento sintomtico: Las crisis comiciales pueden aparecer hasta en el 40% de los casos en el momento del debut
de la enfermedad, y hasta en el 60% a lo largo de su evolucin. Todo paciente diagnosticado de un tumor cerebral que ha sufrido
alguna crisis comicial debe recibir tratamiento antiepilptico. Su uso de forma profilctica es sin embargo controvertido, y la
recomendacin actual de la American Academy of Neurology (AAN) es no iniciar tratamiento antiepilptico en pacientes con tumores
cerebrales que no hayan presentado nunca crisis. En caso de ser necesario tratamiento antiepilptico, deben seleccionarse frmacos
con bajo potencial de interacciones (no inductores) y con un buen perfil de efectos secundarios. El edema peritumoral contribuye
significativamente en el deterioro clnico de los pacientes con tumores primarios y metstasis cerebrales. El tratamiento fundamental
del edema peritumoral son los corticoides, siendo el ms utilizado en este contexto la dexametasona, por su buena difusin hacia el
parnquima cerebral y su mnima actividad mineralocorticoide. Su uso debe estar guiado por la clnica del paciente, manteniendo
siempre la menor dosis posible para el control de los sntomas e iniciando su retirada progresiva posteriormente para evitar los efectos
secundarios del tratamiento prolongado con corticoides. La trombosis venosa profunda y el tromboembolismo pulmonar ocurren hasta
en el 30% de los pacientes fuera del periodo periquirrgico. Entre los factores que los favorecen se encuentran la disminucin de la
movilidad de los miembros particos, decbito prolongado, uso de quimioterapia, liberacin de factores procoagulantes por parte del
tumor y el uso de tratamientos antiangiognicos. Se recomienda el uso de medias de compresin elstica o de mecanismos de
compresin mecnica secuencial a todos los pacientes tras reseccin quirrgica. Las heparinas de bajo peso molecular son el
tratamiento del evento tromboemblico, al haberse comprobado que el riesgo de complicaciones hemorrgicas es bajo. La fatiga, las
dificultades cognitivas y la depresin y ansiedad son tambin sntomas frecuentes que pueden tener un gran impacto en la calidad de
vida del paciente. Su origen es a menudo multifactorial. El tratamiento incluye la correccin de factores precipitantes si es posible y el
uso de farmacoterapia especfica (psicoestimulantes, antidepresivos, ansiolticos). Tumores Cerebrales Primarios: Tumores
Neuroepiteliales, Tumores Neuroepiteliales: Neuronales y Neurogliales mixtos, Tumores Neuroepiteliales: Tumores no gliales. Tumores
Menngeos: Tumores de Clulas Meningoteliales, Tumores Menngeos: Tumores Mesenquimales, Tumores Menngeos: Lesiones
Melanocticas Primarias, Tumores de Clulas Germinales, Tumores de la Regin Selar, Linfoma Primario del SNC. Tumores de los
Nervios Craneales y Espinales, Tumores Medulares Primarios.

CASO CLINICO
Caso clnico. Paciente de 19 aos, masculino, consulta por crisis de cefalea intermitente desde hace 5 meses, que aumentaron en
frecuencia e intensidad agregndose vmitos explosivos 3 das previo a la consulta. Al examen neurolgico destac Glasgow 15,
dismetra de extremidad superior derecha y leve paresia facial izquierda. Se le realiz una tomografa computada de encfalo que
mostr tumor de tipo astrocitoma e hidrocefalia. Se realiz reseccin del tumor y la biopsia definitiva confirm el diagnstico
preoperatorio y determin bordes libres de tumor.

PREGUNTA
Cual es la sobrevida a 10 ao del caso clnico?

RESPUESTA
a.- Ms del 90%
b.- Ms del 80 %
c.- Ms del 70%
d.- Ms del 60 %.

CASO CLINICO
Femenino de 60 aos de edad quien tiene el antecedente de tiroidectoma total, cinco meses previos a su ingreso por ndulo tiroideo
reportado por histologa como hiperplasia multinodular multifocal. Su padecimiento tena un ao de evolucin con cefalea
holocraneana persistente, hemiparesia corporal izquierda, as como parestesias mismo lado; presentaba adems, cacosmia y dficit
campimtrico visual ipsilateral. La exploracin neurolgica demostr funciones mentales superiores conservadas. Nervios craneales:
olfacin conservada, fondo de ojo sin papiledema, hemianopsia homnima izquierda. Extremidades: hemiparesia corporal izquierda
4/5, reflejos exaltados y Babinski ipsilateral, marcha partica, no se encontraron alteraciones sensitivas del orden interoceptivo,
propioceptivo o exteroceptivo; sin afeccin menngea o cerebelosa.
MANUAL DE TRABAJO DEL CURSO ENARM CMN SIGLO XXI
CURSO ENARM CMN SIGLO XXI TEL: 36246001 Pharmed Solutions Institute PGINA 150


PREGUNTA
Los glioblastomas multiformes (OMS grado IV) son las neoplasias ms frecuentes y malignas del sistema nervioso central, una variante
histolgica de stos, denominada, de clulas gigantes representa aproximadamente?

RESPUESTA
a.- 1% de todos los tumores cerebrales.
b.- 2% de todos los tumores cerebrales.
c.- 3% de todos los tumores cerebrales.
d.- 4% de todos los tumores cerebrales.

CASO CLINICO
Un hombre de 56 aos de edad que sufre de dolor bilateral de las piernas durante 2 meses, lo que empeora sobre todo por las noches.
No haba antecedentes de la incontinencia urinaria. En la exploracin neurolgica fue normal. Los estudios de neuroimagen como la
resonancia magntica (MRI) de la columna lumbar revelaron una masa extramedular intratecal de 23x23x13 mm, en L1-2 Nivel de disco
intervertebral. La masa era isointensa en imgenes ponderadas en T1 e hiperintensa en las imgenes potenciadas en T2 con
componentes qusticos.

PREGUNTA
Cual es el porcentaje que presentan los oligodendrogliomas de todolo los tumores de la meula espilana de origen primario?

RESPUESTA
a.- 1%
b.- 1.5%
c.- 2%
d.- 2.5%

CASO CLINICO
Femenino de 59 aos con antecedente de dificultad para la marcha de mas de 5 aos de evolucin, acompaada de dolor tipo radicular
de miembros inferiores, el dolor era intenso tanto en la region sacrolumbar como en sus extremidades inferiores, que se
incrementaban con las maniobras de valsava. Desde el inicio de los sntomas requiri de andarea para demabular, limitando la marcha
a pocos metros y presentndose un cuadro de urgencia esfentiriana urinaria y fecal desde hacia 15 meses. Referia dos episodios de
empeoramiento, los cuales fueron tratados con AINES, corticoides e inmunoglobulina sistmica, con minima mejora.

PREGUNTA
Cual es el diagnostico diferencia mas frecuente en este caso?

RESPUESTA
a.- Esclerosis multiple.
b.- Esclerosis lateral amniotrofica.
c.- Mielitis transversa.
d.- Sindrome raquimedular traumatico.

CASO CLINICO
Varn de 17 aos y 10 meses, que acudi a su mdico por cefalea, vmitos y desorientacin. Muri 2 aos despus tras extensin
medular y metstasis sea. TC craneal con contraste i.v. en el que se observa una masa hipercaptante del vermix que invade el 4
ventrculo con pequeos focos qusticos/necrticos. RM axial en secuencia T2 y sagital en T1 con contraste, en el que se observa una
masa muy hiperintensa con edema perilesional en T2, que se tie de manera parcheada, comprime el 4 ventrculo generando intensa
hidrocefalia obstructiva.

PREGUNTA
Cual de las siguientes aseveraciones respuesto a los meduloblastomas es menos probable?.

RESPUESTA
a.- La localizacin del meduloblastoma desmoplsico presenta ms frecuentemente una localizacin fuera de la lnea media (50%).
b.- La hemorragia es un hallazgo de imagen frecuente en los meduloblastomas, pero se presenta aproximadamente en el 10%
c.- Se ha descrito que el nico subtipo que presentaba ligeras diferencias en su apariencia con RM era el desmoplsico, que
tiende a presentar edema marcado (75%).
d.- Se observan signos radiolgicos clsico de hidrocefalia obstructiva.

CASO CLINICO
Nia de 16 aos que acude por cefalea, vmitos, diplopia y sndrome cerebeloso. TC con contraste en el que se objetiva una masa
hiperdensa de 4 cms en el vermix cerebeloso, con realce heterogneo y focos qusticos de necrosis. Masa que comprime el tronco,
hiperintensa en T2 axial e hipointensa en T1 coronal con contraste, de bordes bien definidos, con realce parcheado, y que genera
dilatacin ventricular.

MANUAL DE TRABAJO DEL CURSO ENARM CMN SIGLO XXI
CURSO ENARM CMN SIGLO XXI TEL: 36246001 Pharmed Solutions Institute PGINA 151

PREGUNTA
Cual de las siguientes aseveraciones respuecto a los meduloblastomas es menos probable?.

RESPUESTA
a.- La apariencia clsica en TC es una masa vermiana hiperdensa, bien delimitada, con edema vasognico circundante ligero o
moderado.
b.- Presencia de hidrocefalia y realce homogneo con contraste.
c.- La presencia de calcificaciones y reas necrtico/qusticas son comunes.
d.- La hemorragia es el signo mas frecuente.

PREGUNTA
Cual de las siguientes aseveraciones respuecto a los meduloblastomas es menos probable?.

RESPUESTA
a.- La apariencia tpica en RM es hipointensa en secuencias potenciadas en T1.
b.- La aparencia tpica en RM Isointensa en secuencias potenciadas en T2.
c.- Respecto a la substancia gris cerebral, observndose mayores grados de heterogeneidad en la RM que en TC.
d.- Presentan captacin intensa del gadolinio de forma principalmente uniforme.

CASO CLINICO
Un hombre de 61 aos consulta por un cuadro progresivo de 3 semanas de evolucin de debilidad de extremidades inferiores e
inestabilidad de la marcha. No refiere haber presentado fiebre ni perdida de peso, ni historia previa de incontinencia o disfuncin
vesical o anal, convulsiones o trauma. Sin embargo, tiene antecedentes de abuso de cocana e hipertensin, tratada con lisinopril. Al
examen fsico, presenta signos vitales normales. Cuando se realiza el examen neurolgico, se evidencia debilidad de ambas
extremidades inferiores, hiperreflexia, y debilidad muscular bilateral (puede hacer movimientos en contra de la fuerza de gravedad y
ante una leve resistencia). Presenta marcha atxica. Se continu el estudio con una RMN de la mdula espinal, que, en fase T1, muestra
una lesin a nivel de de T8, extradural y que comprime la mdula. La RMN de cerebro muestra, en fase T1, mltiples lesiones
homogneas. Al realizar una excisin de la lesin medular, el paciente no vuelve a presentar debilidad muscular en las extremidades
inferiores.

PREGUNTA
Los meningiomas son los tumores benignos del sistema central ms comunes, totalizando entre un 15 a 20% de todos los tumores
primarios cerebrales. Cual es la edad mas frecuente de presentacin?

RESPUESTA
a.- 10 a 20 aos.
b.- 20 a 40 aos.
c.- 40 a 60 aos.
d.- 60 a 80 aos.

CASO CLINICO
Se trata de una paciente mujer de 58 aos que consult por una cervicalgia crnica. La paciente present desde dos meses antes de la
consulta episodios de parestesias de miembros superiores que se exacerbaban con las maniobras de valsalva. El examen fsico de la
paciente fue normal. Se le realiz resonancia magntica de columna cervical, donde se observ una gran lesin qustica que se extenda
desde C2 hasta C3. Con el contraste se observ una captacin de un ndulo en la superficie medular que estaba en estrecha relacin
con el quiste intramedular. La paciente fue llevada a ciruga, donde se le realiz un abordaje por va posterior. Se le realiz
laminectoma C2 y C3, y al abrir la dura se encontr un ndulo tumoral vascular en la pared medular. Con tcnicas microquirrgicas se
hizo reseccin de la lesin espinal medular, y al retirar la lesin se produjo drenaje del quiste. En el postoperatorio la paciente present
mejora de la sintomatologa. El estudio de patologa report un hemangioblastoma espinal.

PREGUNTA
Los tumores intramedulares de la columna espinal son raros, cuales su frecuencia dentro de los tumores del sistema nervioso central?

RESPUESTA
a.- 1 a 2 %.
b.- 2 a 8 %.
c.- 8 a 10 %.
d.- 10 a 12 %.

PREGUNTA
Se pueden encontrar a lo largo de toda la mdula espinal, cual de los siguientes es el mas frecuente?

RESPUESTA
a.- Ependimomas.
b.- Astrocitomas
c.- Hemangioblastomas
MANUAL DE TRABAJO DEL CURSO ENARM CMN SIGLO XXI
CURSO ENARM CMN SIGLO XXI TEL: 36246001 Pharmed Solutions Institute PGINA 152

d.- Menigiomas.

CASO CLINICO
Varn de 16 aos con historia de cefalea, episodios de prdida transitoria del conocimiento de dos aos de evolucin asociadas a
prdida bilateral de la visin desde 4 meses. Como antecedente familiar informa hermano mayor fallecido a los 19 aos con
enfermedad metastsica por Osteosarcoma de fmur. El TAC demuestra tumoracin expansiva en ubicacin supraselar, que se refuerza
con el medio de contraste. Le realizan craneotoma para reseccin biopsia de la tumoracin con remisin de muestra a patologa con el
diagnstico de glioma del quiasma.

PREGUNTA
Cual es la localizacin ms predominante de esta patologia?

RESPUESTA
a.- Pineal.
b.- Supraselar.
c.- Tlamo.
d.- Tercer ventrculo.

CASO CLINICO
Varn de 19 aos, con historia de 4 aos de hipoacusia que evoluciona a prdida de la audicin del odo derecho. Dos aos despus se
acompaa de cefalea y prdida de la visin iniciada por el ojo izquierdo que progresa a ceguera total desde 4 meses. Como antecedente
familiar presenta abuela materna fallecida con enfermedad metastsica por carcinoma de cuello uterino. El TAC demostr presencia de
tumoracin en regin pineal y tercer ventrculo con importante hidrocefalia. Le realizaron craneotoma con colocacin de vlvula de
derivacin revertiendo parcialmente la sintomatologa. Dos meses despus recurren las manifestaciones clnicas, el estudio de RM
demuestra tumoracin expansiva e irregular que compromete regiones selar y pineal, piso del tercer ventrculo y ncleos bsales

PREGUNTA
Cual es la tasas de curacin mas aproximada de este padecimiento?

RESPUESTA
a.- 60 %.
b.- 70 %.
c.- 80 %
d.- 90 %

CASO CLINICO
Una mujer de 52 aos consulta por pigmentacin de las uas de un ao de evolucin. No presentaba sntomas sistmicos y era
inmunocompetente. Se le haba diagnosticado hace 12 aos un macro-adenoma pituitario productor de ACTH, que haba sido tratado
con neurociruga endoscpica transnasal. Sin embargo, la ciruga no produjo la cura, por lo que se realiz una adrenalectoma bilateral,
y la paciente reciba mineralo-corticoides y glucocorticoides. Al exmen fsico, algunas de las uas de los dedos y de los pies
presentaban una banda de melanoniquia longitudinal, mientras que otras manifestaban pigmentacin gris negruzca difusa. No se
observaron cambios de color en piel ni mucosas. L os estudios de laboratorio mostraron que los niveles plasmticos de ACTH eran altos,
de 100 pg/mL (normal 10-50 pg/mL). Las imgenes de resonancia magntica (MRI) confirmaron vestigios de adenoma pituitario.

PREGUNTA
Cual de las siguientes afirmaciones relacionadas a la melanoquia es menos probable?

RESPUESTA
a.- Las drogas ms comunes que causan melanoniquia son los agentes quimioterpicos.
b.- Las enfermedades endcrinas que causan incremento de los niveles circulantes de ACTH.
c.- Los melanomas son la principal causa de melanoquia.
d.- Frecuentemente los pinealomas no presentan melanoquia.

CASO CLINICO
Paciente de 35 aos que presenta desde hace 10 aos amenorrea, esterilidad, y galactorrea. A veces alteraciones visuales. Sensibilidad
en las mamas. Disminucin del inters sexual, dolor de cabeza, infertilidad. Se le hizo un TAC y se hall un prolactinoma, al tratarla con
dopamina, (carbegolina) el microadenoma remiti.

PREGUNTA
Cual es el porcentaje de cambio de microadenomas a macroadenomas productores de prolactina?

RESPUESTA
a.- 2 %.
b.- 3 %.
c.- 4 %.
d.- 5 %.
MANUAL DE TRABAJO DEL CURSO ENARM CMN SIGLO XXI
CURSO ENARM CMN SIGLO XXI TEL: 36246001 Pharmed Solutions Institute PGINA 153


CASO CLINICO
Preescolar masculino de 2 aos y 11 meses de edad hospitalizado como desnutrido severo marasmtico. Al momento de ingreso la
madre refera un padecimiento de dos meses de evolucin caracterizado por vmitos post-prandiales persistentes, hiporexia,
disminucin subjetiva y progresiva de peso adems de compromiso del estado general. Es producto de primer embarazo, nacido de
trmino por parto institucional sin complicaciones de una madre de 24 aos sin antecedentes patolgicos de importancia, con llanto
inmediato al nacer y desarrollo psicomotor adecuado hasta el ao de edad, a partir del cual se percibi falta de desarrollo del lenguaje
receptivo y expresivo adems de retraso del desarrollo motor caracterizado por imposibilidad de ponerse de pie y caminar sin apoyo. Al
examen fsico de ingreso se encontraba en mal estado general y nutricional. El indicador peso para la talla era menor a -3 desvos
estndar (DE), exista emaciacin visible y palidez mucocutnea generalizada. Neurolgicamente llamaba la atencin una respuesta
verbal inapropiada, imposibilidad de bipedestacin y aparente afectacin visual manifestada por imposibilidad del paciente de seguir
objetos con la mirada. Su permetro ceflico fue de 48 cm. (adecuado para la edad) y las pupilas eran isocricas reactivas al estmulo
luminoso. El informe histopatolgico conrm el diagnstico de craneofaringioma.

PREGUNTA
Con respecto al diagnostico, cual de las siguientes aseveraciones en menos probable?

RESPUESTA
a.- Tiene un leve predominio en el sexo femenino.
b.- Es un tumor de origen epitelial de la regin sellar que se forma a partir de los restos embrionarios de la bolsa de Rathke.
c.- El tumor puede limitarse a la silla turca, o bien puede extenderse a travs del diafragma sellar y comprimir la va ptica, la
protuberancia o el tercer ventrculo, produciendo hidrocefalia.
d.- Los craneofaringiomas representan entre el 5 y el 10% de todos los tumores intracraneales que se presentan durante la niez.

CASO CLINICO
Mujer de 51 aos que padeca desde haca cuatro meses episodios de repeticin de amaurosis fugax en su ojo izquierdo. Mediante
tomografa axial computerizada se diagnostic una masa orbitaria circunscrita intraconal izquierda sospechosa de hemangioma
cavernoso. Se le practic una orbitotoma lateral izquierda resecndose una masa rojiza encapsulada, sugestiva de hemangioma
cavernoso. Microscpicamente estaba compuesta por reas de clulas fusiformes dispuestas en empalizada (Antoni tipo A),
entremezcladas con reas qusticas donde las clulas estaban rodeadas por una matriz mixoide (Antoni tipo B).

PREGUNTA
Cual es la edad de presentacin mas frecuente de este padecimiento.

RESPUESTA
a.- entre 20 y 30 aos.
b.- entre 20 y 40 aos.
c.- entre 20 y 50 aos.
d.- entre 20 y 60 aos.

INFECCIONES DEL SISTEMA NERVIOSO CENTRAL.
Introduccin: constituyen una emergencia mdica, pues su
alta morbilidad y mortalidad requieren un diagnstico y
tratamiento oportuno. Muchos factores se involucran en la
severidad de las infecciones del SNC. Su ubicacin anatmica
en un espacio seo sellado que no permite una expansin
fcil ante un proceso inflamatorio difuso contribuye a que las
altas posibilidades de dao neurolgico ocurran por efecto
mecnico, como en los sndromes de herniacin. Otros
factores como la competencia inmunolgica de cada
individuo, la penetracin y concentracin de los agentes
antimicrobianos en el sistema nervioso, la edad y las
dificultades diagnsticas contribuyen en la evolucin de los
pacientes con infecciones del sistema nervioso. La edad de
aparicin ms frecuente de meningitis desde la infancia hasta
la segunda y tercera dcada. Por otro lado ha aumentado la proporcin de infecciones nosocomiales. Evaluacin de laboratorio. El
lquido cefalorraquideo (LCR) es el pilar fundamental en el diagnstico de la mayora de las infecciones del SNC. Las caractersticas
iniciales del anlisis citoqumico pueden orientar a un diagnstico especfico aunque se requiere la confirmacin de ste por otros
medios. Sin embargo una buena correlacin clnica inicial es suficiente para tomar una conducta teraputica. En general las
caractersticas tpicas de los diferentes tipos de infeccin menngea de acuerdo con el citoqumico se muestran en la Tabla. Los cultivos
y el Gram del LCR continan siendo los exmenes de eleccin en las meningitis bacterianas. La administracin de antibiticos
intravenosos por dos a tres das antes de la puncin lumbar no altera el conteo de clulas o las concentraciones de protenas y glucosa
en los casos de meningitis bacterianas, pero si reducirn la probabilidad de obtener un cultivo y el Gram del LCR positivos. La
administracin de antibiticos orales antes de la puncin lumbar no altera ninguno de los parmetros en el LCR pero s puede disminuir
el porcentaje de neutrfilos y la probabilidad de resultados positivos en el cultivo o el Gram. El Gram tiene una sensibilidad cercana a
60-90% y una especificidad de 100%. Los cultivos se pueden cultivar en platos de agar con adicin de nutrientes o en caldos. Los
MANUAL DE TRABAJO DEL CURSO ENARM CMN SIGLO XXI
CURSO ENARM CMN SIGLO XXI TEL: 36246001 Pharmed Solutions Institute PGINA 154

cultivos en caldos son ms susceptibles de ser contaminados con Stophylococcus epidermidis mientras que son mas tiles en caso de
infecciones de los equipos de derivacin ventricular. Los hemocultivos identifican el germen causal en 80% de los casos de neumococo,
90% de los de meningococo y 94% en los casos de Haemophylus influenzal. Los casos de meningitis tuberculosa (TBC) o meningitis
micticas son difciles de diagnosticar por cultivo o en el extendido. Son positivos entre 52 a 83% de los casos de meningitis TBC y en
75% de los casos de criptococosis menngea. La sensibilidad y especificidad de los cultivos en estos dos grmenes se aumenta hasta en
87% con cultivos repetidos y con grandes volmenes (hasta 25 ml) de LCR. Las pruebas de ltex para identificacin de antgenos de los
grmenes son rpidas requiriendo entre 10 a 15 minutos para realizarlas y sin un entrenamiento riguroso. Adems tienen la ventaja de
tener pocos falsos positivos. Por ejemplo, para criptococo puede haber falsos positivos ante la presencia de factor reumatoideo. En el
caso de meningitis por Histoplasma capsulatum, las pruebas son muy sensibles pero la especificidad es baja por tener reactividad
cruzada con criptococo, candida y Coccidiodes inmitis. La identificacin de anticuerpos especficos en el LCR es de gran utilidad. En la
neurosfilis un diagnstico definitivo se obtiene con la demostracin de positividad del VDRL en el LCR. Una prueba reactiva en
cualquier ttulo es diagnstica de neurosfilis. Aunque la prueba es muy especfica, la sensibilidad sin embargo vara entre 30-70% y en
muchos casos el diagnstico se basa en la presencia de pleocitosis o protenas elevadas en el LCR en un paciente con VDRL reactiva en
sangre. En los pacientes con VIH las dificultades nacen debido a que puede haber pleocitosis con aumento de protenas en el LCR
debido a la neurosfilis o por la misma infeccin por el VIH. En estos casos tiene mucha utilidad el FTA-ABS (Treponemal antibody-
absortion test) y el test de hemoaglutinacion para Treponema pallidum (MHA-TP) los cuales son muy sensibles para el diagnstico de
neurosfilis. Un resultado no reactivo descarta el diagnstico de neurosfilis en los pacientes con infeccin por VIH. La PCR es la prueba
ms til para el diagnstico de meningitis y encefalitis viral. Adems tiene la ventaja de cuantificar el cido nucleico en las muestras de
LCR con lo cual se puede determinar la progresin de la enfermedad y la respuesta al tratamiento. La prueba se fundamenta en realizar
mediante un sistema trmico cclico una copia y amplificacin de hasta un milln de veces del contenido de ADN del germen presente
en el LCR. En los casos de meningitis TBC los resultados de sensibilidad para la PCR en diferentes estudios han sido variables (entre 54-
100%) y la especificidad entre 89-100%. Sin embargo hay evidencia consistente de que la PCR es ms sensible que el examen
microscpico y el cultivo para TBC. La adenosin deaminasa (ADA) es una enzima que est asociada con enfermedades que producen
una respuesta inmunolgica celular; es de mucha ayuda para el diagnstico de meningitis TBC. Sin embargo, pueden observarse
resultados positivos de esta prueba en linfomas con compromiso menngeo, sarcoidosis, neurobrucelosis y hemorragia subaracnoidea.
Tambin pueden haber falsos negativos. Los diferentes estudios muestran una sensibilidad y especificidad para el diagnstico de 90%
con ttulos mayores de 10 unidades internacionales por litro (UI/L) siendo los ttulos de corte entre 5-10 UI/L. Los resultados de la ADA
pueden elevarse en las dos primeras semanas de tratamiento. Meningitis: Se define meningitis como la presencia de inflamacin
menngea originada por la reaccin inmunolgica del husped ante la presencia de un germen patgeno en el espacio subaracnoideo.
La meningitis es el sndrome infeccioso ms importante del sistema nervioso central. El compromiso parenquimatoso adyacente a las
meninges definir la presencia de meningoencefalitis (encfalo), meningoencefalomielitis (encfalo y mdula),
meningomielorradiculitis (encfalo, mdula y races nerviosas). Los diferentes tipos de meningitis tienen diferente origen y los
patgenos responsables en la mayora de los casos son predecibles, lo cual permite en muchas situaciones clnicas el inicio de una
terapia emprica mientras se obtiene la confirmacin del germen involucrado. La identificacin de grmenes que pueden tener
influencia de tipo endmico es de importancia para las medidas profilcticas y epidemiolgicas en determinadas poblaciones (por ej:
meningococo). Los diferentes tipos de meningitis se pueden definir de acuerdo con el perfil clnico, hallazgos de LCR y la etiologa. Se
definen como meningitis asptica aquellos casos de meningitis en los cuales luego de un estudio completo no se identifica una bacteria
y los cultivos y las pruebas inmunolgicas deben ser negativos. Es de origen viral por lo general. La meningitis sptica es causada por
una bacteria. Tiene con la meningitis asptica un perfil diferente del LCR; su tratamiento y pronstico son completamente distintos a la
anterior. La meningitis crnica se define arbitrariamente como aquella con una evolucin mayor de cuatro semanas luego de haberse
documentado clnicamente que el paciente no se encuentra en una fase de recuperacin de una meningitis aguda. La meningitis
recurrente se refiere a aquellos casos que clnicamente presenta dos o ms episodios de meningitis aguda. El paciente debe haber
evolucionado a la mejora total tanto clnica como en el LCR entre cada uno de los episodios agudos. Meningitis asptica: Por lo general
es de curso benigno y en la mayora de los casos de etiologa viral. Puede haber causas no infecciosas de meningitis asptica. Tiene un
curso clnico bifsico. Primero preceden a su aparicin los sntomas o signos de una afeccin viral respiratoria, gastrointestinal o en la
piel, presentndose posteriormente la fase menngea con sntomas ms especficos como cefalea, fiebre y signos menngeos. Son poco
frecuentes los sntomas de compromiso enceflico como convulsiones o alteracin del estado mental. En algunos pases tiene
predileccin estacional y es ms frecuente en nios. Los enterovirus son los principales agentes causales (entre 55-75%) de los casos de
meningitis asptica y hasta en 95% de los casos cuando se identifica el patgeno. El virus de la parotiditis fue considerado en alguna
ocasin responsable de muchos casos de meningitis asptica pero la incidencia ha declinado desde el uso de la vacuna; sin embargo
una meningitis oculta se presenta en ms de la mitad de los casos de parotiditis y llega a ser sintomtica en solo 30%. Mas an, puede
ocurrir la meningitis sin parotiditis. Los herpes virus raramente causan meningitis; sin embargo el virus del herpes simple (VHS) es el
responsable de 1 a 3% de todos los casos de meningitis asptica. De los dos tipos de herpes simple (VHS 1 oral - VHS 2 genital) el VHS 2
produce meningitis en 11% a 33 % de las personas en el momento de la infeccin genital primaria. El VIH puede causar una meningitis
asptica principalmente en el perodo de la infeccin primaria y durante la seroconversin pero incluso puede producirla muy
temprano en las fases iniciales de la infeccin por VIH. Raramente la tuberculosis se puede comportar como una meningitis asptica
autolimitada. En los casos de endocarditis e infecciones paramenngeas puede presentarse un cuadro de meningitis asptica aunque las
causas sean grmenes no vrales. La PCR es de gran utilidad con una especificidad del 100% y el resultado es disponible en pocas horas.
El manejo es prcticamente de soporte y de alivio de los sntomas. Meningitis sptica (meningitis bacteriana) implican una mayor
morbi-mortalidad y requieren un rpido diagnstico y tratamiento para evitar una evolucin fatal o secuelas irreversibles, por lo tanto
es una emergencia neurolgica. La tasa general de mortalidad es de 25% y de morbilidad hasta 60%. Presentacin clnica: los sntomas
clsicos de presentacin incluyen cefalea, fiebre, escalofros, alteracin de la esfera mental y la presencia de meningismo. Estos
sntomas pueden variar en los neonatos y en los ancianos. En los neonatos predominan los sntomas inespecficos como irritabilidad,
vmito, letargia, dificultad respiratoria y sntomas gastrointestinales, siendo el meningismo poco frecuente. Los ancianos presentan con
mayor frecuencia alteracin en el estado mental asociado a fiebre; la presencia de cefalea y meningismo resulta extica. La meningitis
por meningococo tiene frecuentemente una presentacin dramtica y puede progresar como una enfermedad fulminante muriendo el
MANUAL DE TRABAJO DEL CURSO ENARM CMN SIGLO XXI
CURSO ENARM CMN SIGLO XXI TEL: 36246001 Pharmed Solutions Institute PGINA 155

paciente en pocas horas. Muchas veces los pacientes tienen un rash
cutneo petequial en el tronco y las extremidades inferiores. Este puede,
por contigidad de las lesiones, formar reas extensas de equimosis. La
prpura fulminante es una forma grave de manifestacin de la sepsis por
meningococo; es causada por coagulacin intravascular diseminada y en
muchas ocasiones causa necrosis distal en las extremidades. Los
marcadores de riesgo incrementado de muerte en los pacientes con meningitis por meningococo son: la presencia de ditesis
hemorrgica, signos neurolgicos focales y personas mayores de 60 aos. En general muchos sndromes neurolgicos diferentes al
meningismo pueden ser la forma de presentacin de las meningitis bacterianas. La presencia de signos neurolgicos focales por
compromiso cerebrovascular, hipertensin endocraneana, alteracin en pares craneales, crisis convulsivas y estado confusional son
algunos de ellos.
Tratamiento: el tiempo del inicio del tratamiento influye significativamente en el pronstico. El LCR es el principal soporte diagnstico
en la escogencia de la terapia adecuada, pero en muchas ocasiones la puncin lumbar debe diferirse en aquellos casos de pacientes en
estupor o coma, con signos neurolgicos focales y ante la presencia de crisis convulsivas. En estos casos las neuroimgenes estn
indicadas inicialmente para descartar complicaciones asociadas con la meningitis o hacer un diagnstico diferencial. Una escanografia
de crneo (simple) normal permite una mayor seguridad en la realizacin de la puncin lumbar en estos casos y evitar as
complicaciones. Cabe anotar que ante la sospecha diagnstica debe iniciarse una terapia emprica mientras se realiza la puncin
lumbar. La terapia antibitica no variar significativamente el LCR en las primeras 48 horas. La terapia emprica se escoger de acuerdo
con los grupos de edad, la sensibilidad antibitica de los grmenes
involucrados, su sensibilidad antibitica y el estado inmune de los
pacientes. El Gram puede ser una gua inicial de utilidad pero no
es tan eficaz en todos los casos; las pruebas inmunolgicas son de
gran ayuda para un diagnstico especfico y rpido del germen. La
terapia emprica ms conveniente para iniciar el tratamiento.
Luego, al obtener los resultados del cultivo con las pruebas de
sensibilidad antibitica se decidir el tratamiento definitivo. El
Streptococcus pneumoniae (neumococo) hoy por hoy es el
principal germen patgeno en la meningitis bacteriana en
cualquier grupo de edad. Penicilina cefalosporina con
vancomicina. El Haemophylus influenzae es un coco bacilo
gramnegativo. El nico husped natural de este germen es el
humano y la transmisin persona-persona ocurre por la va
respiratoria. El trauma de crneo reciente, la ciruga neurolgica previa, las sinusitis paranasales, las otitis media y las fstulas del LCR
son los factores de riesgo ms importantes en adultos para padecer una meningitis por H. influenzae. El tratamiento antibitico es
ceftriaxona cefotaxima, por lo tanto son los frmacos de eleccin. La Listeria monocitogenes es un germen beta-hemoltico
facultativamente anaerobio y grampositivo. Los grupos de edad ms frecuentemente involucrados en meningitis por L. monocitogenes
son los neonatos menores de un mes y los adultos mayores de 60 aos. En este grupo la listeria es la causa de 20% de las meningitis.
Los principales factores de riesgo para meningitis por L. monocitogenes son el embarazo, inmunosupresin y edad avanzada. El
tratamiento de eleccin es la ampicilina o la penicilina por un tiempo de cuatro semanas en los pacientes inmunosuprimidos. En los
pacientes alrgicos a las penicilinas el trimetropim-sulfa-metoxazol es la alternativa ms recomendada. El estreptococo del Grupo B es
la causa ms frecuente de sepsis neonatal y es una causa importante de infeccin bacteriana invasiva en los adultos. Es el germen
causal en 70% de los casos de meningitis bacteriana en los neonatos menores de un mes, mientras que en adultos es responsable de
menos de 5% de los casos. El estreptococo del grupo B es altamente susceptible a la penicilina y a la ampicilina siendo las drogas de
primera eleccin. Muchos recomiendan el uso concomitante con gentamicina para el tratamiento de la meningitis neonatal basado en
evidencia de estudios en animales que demuestran sinergismo en la eficacia antibitica. La Neisseria meningitidis (meningococo) es una
bacteria gramnegativa encapsulada que aparece en pares en el extendido. Generalmente coloniza la nasofaringe de manera
asintomtica y la transmisin ocurre de persona a persona mediante secreciones respiratorias. Los individuos esplenectomizados y con
deficiencias de complemento tienen un alto riesgo de infeccin por meningococo. Causa 60% de las meningitis bacterianas entre la
poblacin de 2 a 18 aos de edad y va declinando su frecuencia hasta 5 % en los mayores de 60 aos. Es el responsable en 1/3 de los
casos de meningitis en menores de 2 aos. El frmaco de eleccin es la penicilina o la ampicilina. La ceftriaxona y el cefotaxima tienen
tambin una excelente respuesta y alcanzan buenas concentraciones en el LCR. En Estados Unidos no se han identificados cepas
productoras de beta-lactamasa. Los contactos cercanos en pacientes con meningococemia tienen un riesgo significativamente alto para
desarrollar la infeccin semanas despus de la exposicin. Meningitis tuberculosa: Es la manifestacin ms frecuente de la tuberculosis
en el sistema nervioso. Puede presentarse en muchos casos en forma aislada sin compromiso extramenngeo. Las manifestaciones
clnicas son diferentes en nios, adultos y en los infectados por VIH. Su epidemiologa ha cambiado significativamente aun en pases en
donde la incidencia haba disminuido de manera importante. Este hecho se explica por la epidemia reciente de infeccin por VIH y por
la resistencia que ha desarrollado el bacilo a las terapias convencionales. Manifestaciones clnicas: con fines pronsticos se reconocen
tres estadios de la enfermedad. a) Estadio 1: el paciente est consciente y no presenta signos neurolgicos focales. b) El estadio dos: el
paciente est confuso pero no est en coma, y tiene signos neurolgicos focales como hemiparesia o parlisis de pares craneales. c)
Estadio tres: el paciente se encuentra en coma o en estupor; tiene compromiso mltiple de pares craneales, hemipleja o parapleja. En
los nios es ms frecuente identificar la fase inicial de infeccin a nivel respiratorio, mientras que en el adulto en muchos casos no se
puede identificar y puede pasar mucho tiempo desde la infeccin inicial y la aparicin del compromiso del sistema nervioso. Es
frecuente la presencia de hiponatremia en adultos encontrndose en 45% de los casos y se debe a una secrecin inadecuada de
hormona antidiurtica (SIDHA). En los nios aunque pueden manifestarse tambin la cefalea, fiebre y meningismo, es comn la
presencia de hidrocefalia como primera manifestacin de la infeccin por TBC. Diagnstico: el diagnstico recae principalmente en el
LCR. La pleocitosis a expensas de linfocitos, las protenas altas y la glucorraquia significativamente baja son los hallazgos ms
MANUAL DE TRABAJO DEL CURSO ENARM CMN SIGLO XXI
CURSO ENARM CMN SIGLO XXI TEL: 36246001 Pharmed Solutions Institute PGINA 156

frecuentemente encontrados. En algunos casos hay un predominio polimorfonuclear en las fases iniciales de la infeccin, que tambin
suele observarse cuando se inicia el tratamiento. Este ltimo coincide con el empeoramiento clnico que se observa en algunos
pacientes cuando se inicia la terapia y es muy caracterstico de la meningitis TBC. Por lo general hay un viraje posterior al predominio de
linfocitos pero en algunas ocasiones puede persistir el predominio de polimorfonucleares denominndose meningitis neutroflica
persistente. Se ha encontrado un mayor contenido de protenas en el estadio tres de la enfermedad. Como ya se coment
anteriormente el cultivo y los extendidos para BK tienen una mayor probabilidad de positividad cuando se estudian muestras repetidas
en altos volmenes. La ADA y la PCR recientemente han sido de gran ayuda para un diagnstico ms rpido y en los casos de cultivos
negativos. Las neuroimgenes brindan apoyo en el diagnstico. En la RMC se puede observar el compromiso de las meninges basales
que realzan al inyectar el gadolinio; los sitios ms comprometidos son la fosa interpeduncular, la cisterna ambiens y la regin
quiasmtica. Este realce es ms frecuentemente observado en los pacientes VIH positivos. Las neuroimgenes tambin son de ayuda
para el diagnstico de hidrocefalia, la presencia de vasculitis por la infeccin y para caracterizar los granulomas. Complicaciones: an
con tratamiento y cuidados adecuados las complicaciones se pueden presentar. Algunas de ellas son de tipo cerebrovascular por
compromiso de las arterias intracraneales debido a una panarteritis, producto de la infiltracin de las paredes de los vasos por el
exudado inflamatorio. Lo anterior produce infartos cerebrales. El territorio carotdeo es el ms frecuentemente comprometido. Una
complicacin metablica que puede agravar el cuadro clnico es la hiponatremia por SIDHA; sta debe corregirse rpida y lentamente
para evitar el riesgo de mielinolisis pntica. Se han informado casos de siringomielia muchos aos despus de la infeccin inicial.
Tratamiento: va orientado a eliminar las formas intra y extracelulares del bacilo. Resultan importantes dos factores para evitar la
resistencia: el primero, utilizar mltiples frmacos y el segundo la adherencia al tratamiento. Por lo general el inicio del tratamiento es
emprico, sin obtener una confirmacin por cultivo que puede tardar varias semanas. Lo anterior repercute significativamente en el
pronstico. Se sabe bien que el nmero de organismos presentes en los casos de meningitis TBC es inferior al de las otras formas de la
infeccin; esta ventaja potencial se ve disminuida por el hecho de la penetracin de los frmacos anti-TBC en el SNC. La isoniacida y la
pirazinamida son bactericidas y penetran las meninges inflamadas o no inflamadas alcanzando buenas concentraciones bactericidas.
Las concentraciones que alcanzan en el LCR la estreptomicina intramuscular, la rifampicina y el etambutol apenas sobrepasan las
concentraciones inhibitorias mnimas para la micobacteria; adems no penetran las meninges no inflamadas. Mientras que la
rifampicina es bactericida, el etambutol y la estreptomicina son tuberculostticos. Se aceptan dos tipos de rgimen: uno corto de seis
meses, con cuatro frmacos. En los primeros dos meses: isoniacida 300 mg, rifampicina 600 mg, pirazinamida 1,5 gr al da y
estreptomicina 1g IM (500 mg en ancianos o con peso menor de 50 kg). Luego un perodo de cuatro meses con isoniacida y rifampicina
dos veces por semana o diario. En los nios este segundo perodo debe ser de 10 meses. En los casos de resistencia se debe incluir otro
medicamento como etambutol en vez de estreptomicina y seguir luego con dos drogas (isoniacida y rifampicina) por 9 a 18 meses. La
respuesta a la terapia debe evaluarse por cultivos y por la evolucin clnica de los sntomas. Meningitis por criptococo: En general todas
las meningitis por hongos se consideran que ocurren en pacientes inmunosuprimidos. La va de entrada en la mayora de los casos es a
travs de las vas respiratorias superiores, ingresando en forma de levadura en el alvolo, all puede producir un proceso inflamatorio
que puede ser completamente asintomtico o producir sntomas respiratorios leves y el sistema inmune puede terminar all todo el
proceso; si esto no ocurre as, pasan a la circulacin sistmica. Un defecto en los sistemas celulares permite que los hongos atraviesen
la barrera hematoenceflica. Las infecciones micticas del sistema nervioso pueden coexistir o no con la infeccin en otros rganos
blanco en el organismo. En el caso del criptococo, el rin es el rgano mas frecuentemente comprometido. En el SNC el criptococo
puede comprometer nicamente las meninges o invadir el parenquima y los vasos sanguneos producindose una forma de arteritis
infecciosa. Presentacin clnica: la presentacin clnica de las meningitis por hongos en general es insidiosa, de instauracin subaguda o
crnica; se presenta cefalea, fiebre, escalofros, alteracin del estado general y cambios mentales en un paciente por lo general con
factores de riesgo para adquirir la infeccin (SIDA, cncer, terapia inmunosupresora, trasplante de rganos). Diagnstico: requiere de
sospecha clnica en los pacientes con riesgo y recae principalmente en el aislamiento del criptococo en el LCR. El LCR muestra
generalmente un aumento moderado de la presin; es de aspecto claro y hay pleocitosis que puede alcanzar desde 20 hasta 1.000
clulas por milmetro cbico. El predominio celular es de linfocitos, la glucosa es frecuentemente baja (alrededor de 30 mg/dl) y las
protenas son elevadas entre 50 a 1.000 mg/dl; la tinta china puede mostrar la presencia del criptococo. La prueba debe hacerse con un
control negativo de agua y as estar seguro que la tinta no est infectada por espora. La prueba serolgica de ltex en el LCR es muy
sensible para el diagnstico; se puede cuantificar y es de gran ayuda en el seguimiento. Adems es rpida y de fcil interpretacin.
Tratamiento: el tratamiento de primera lnea para la meningitis por criptococo es la anfotericina B a la dosis de 0,7 mg/kg/da ms 5
flucytocina a la dosis de 25 mg/kg cada seis horas por va oral hasta que los cultivos sean negativos o haya mejora clnica (entre cuatro
a seis semanas). Posteriormente se debe continuar una terapia de mantenimiento con fluconazol por ocho o diez semanas; pero si el
paciente es inmunosuprimido (SIDA, trasplantado, cncer, etc. ) esta terapia de mantenimiento es de por vida.

CASO CLINICO
Paciente de 30 aos, sexo masculino, en tratamiento con prednisona por Lupus Eritematoso Sistmico, que comenz con un cuadro de
aproximadamente dos semanas de evolucin caracterizado por compromiso progresivo del estado general, cefalea holocraneana de
intensidad progresiva, fiebre y diaforesis vespertina por lo que consult. Se le realiz puncin lumbar, obtenindose LCR de aspecto
opalescente, cuyo estudio citoqumico fue compatible con meningitis bacteriana; el cultivo demostr desarrollo de un bacilo Gram (+)
pequea, cuya identificacin microbiolgica confirm Listeria monocytogenes.

PREGUNTA
Cual es la conducta teraputica mas apropiada a seguir.

RESPUESTA
a.- Cefotaxima mas vancomicina.
b.- Ampicilina mas gentamicina.
c.- Trimetroprima mas sulfametoxazol y ampicilina.
d.- Gentamicina mas imipenem.
MANUAL DE TRABAJO DEL CURSO ENARM CMN SIGLO XXI
CURSO ENARM CMN SIGLO XXI TEL: 36246001 Pharmed Solutions Institute PGINA 157

CASO CLINICO
Paciente de 87 aos, sexo masculino, hipertenso, portador de marcapasos definitivo, adems de multiinfarto cerebral y bronquitis
crnica. Ingres con historia de tos productiva y fiebre de aproximadamente 1 semana de evolucin, agregndose posteriormente
desorientacin progresiva y franco compromiso de conciencia que motiv hospitalizacin. Desde su ingreso, muy grave, en sopor, con
signos menngeos esbozados y neumona confirmada tanto clnica como radiolgicamente. Se manej con antibioticoterapia agresiva y
de amplio espectro para neumona adquirida en la comunidad con sospecha de aspiracin; a pesar de lo anterior evoluciona a la falla
orgnica mltiple.


PREGUNTA
Cual es la conducta teraputica mas apropiada a seguir.

RESPUESTA
a.- Cefotaxima mas vancomicina.
b.- Ampicilina mas cefotaxima.
c.- Ampicilina mas levofloxacina.
d.- Gentamicina mas imipenem.

CASO CLINICO
Un varn de 56 aos de edad, con antecedentes de meningitis criptogena y meningitis asptica que presenta hace 10 dias fiebre,
escalofros, rigidez en el cuello y dolor de cabeza. El EF temperatura 36.9 c, FR de 121 lpm. Pareca incmodo, pero sin rigidez de nuca
franca. LCR revelo eritrocitos 34 clulas/mm3, 154 clulas/mm3, leucocitos ( 77 % de neutrfilos , 12 % de linfocitos, histiocitos 10 % ),
la glucosa de 63 mg / dl , la protena de 82 mg / dl. TAC de la cabeza revela estable, leve prominencia difusa del sistema ventricular. El
paciente es alrgico a la penicilina.

PREGUNTA
Cual es la conducta teraputica mas apropiada a seguir.

RESPUESTA
a.- Cefotaxima mas vancomicina.
b.- Meropenen mas aciclovir.
c.- Amikacina mas levofloxacina.
d.- Tratamiento sintomatico.

CASO CLINICO
Varn de 51 aos, ingres por cefalea de 4 semanas y somnolencia. La EF con deterioro neurolgico con 10 de Glasgow. Se realiz una
puncin lumbar (PL): 40clulas/mm3, el 95% mononucleares (MN); glucosa: 4mg/dl, protenas: 0,68mg/l. En la tincin de tinta china se
apreciaron clulas levaduriformes. El ttulo de antgeno criptoccico en plasma era superior a 1/256 y en lquido cefalorraqudeo (LCR),
superior a 1/1.024. No se midi la presin intracraneal. En el cultivo de LCR se aisl C. neoformans, se inici tratamiento con
anfotericina B i.v. a dosis de 4mg/kg/da y 5 flucitosina oral a dosis de 100mg/kg/da. El paciente desarroll sordera bilateral como
complicacin. A los 12 das reingres con afectacin de VII par craneal izquierdo.

PREGUNTA
Cual es la conducta teraputica mas apropiada a seguir.

RESPUESTA
a.- Anfotericina B iv.
b.- Meropenem i.v. a dosis de 1g/8h.
c.- Fluconazol 400mg/da/VO.
d.- Linezolid i.v. a dosis de 600mg/12h.

CASO CLINICO
Varn de 47 aos de edad ex-adicto a drogas por va parenteral y con infeccin por VIH conocida desde haca 6 aos, en tratamiento
con antirretrovirales que cumpla de forma irregular. Ingresa por cefalea de 3 semanas de evolucin de localizacin holocraneal que
aumentaba tras maniobras de Valsalva. Su intensidad ha seguido un perfil progresivo, interfiriendo el sueo y requiriendo dosis
crecientes de analgesia. Ha asociado vmitos en los ltimos das. La familia reconoce enlentecimiento mental y episodios confusionales
recortados durante los cuales realizaba movimientos repetitivos de las manos y de succin. En los 4 das previos a su asistencia en
Urgencias, refera inestabilidad en la marcha, diplopa e hipersomnia diurna. Paciente desorientado parcialmente en el tiempo
(desconoca da del mes y de la semana), aunque orientado en espacio y persona. Bradipsquico. Con rigidez de nuca. Borramiento de
bordes papilares con incipientes hemorragias peripapilares y desaparicin del pulso venoso. Paresia bilateral del VI par. Deficiente
estado nutricional y palidez cutneomucosa. Hemograma: anemia normoctica y normocrmica (hemoglobina 10.5 g/dl). Leucocitos
3200 con linfopenia (15 %). Serie plaquetaria normal. Velocidad de sedimentacin globular 56 mm en la primera hora. Hipernatremia
leve (149 mg/dl), hipertransaminasemia (glutmico-oxalactico transaminasa 78 unidades por litro, glutmico-pirvico transaminasa
120 unidades por litro, gamma-glutamil-transpeptidasa 320 unidades por litro), resto de parmetros normales. Estudio de coagulacin
normal, salvo hiperfibrinogenemia (765). Hipergammaglobulinemia policlonal e hipoalbuminemia discreta.

MANUAL DE TRABAJO DEL CURSO ENARM CMN SIGLO XXI
CURSO ENARM CMN SIGLO XXI TEL: 36246001 Pharmed Solutions Institute PGINA 158

PREGUNTA
Cul de los siguientes diagnsticos le parece el ms probable?

RESPUESTA
a.- Complejo demencia-SIDA (Sndrome de Inmunodeficiencia Adquirida)
d.- Enfermedad de Alzheimer en estadio leve
c.- Pseudodemencia depresiva
d.- Meningitis tuberculosa

CASO CLINICO
Agricultor, del sexo masculino, de 64 aos, admitido en Urgencias con un cuadro de fiebre baja, mialgias, artralgias e hiporexia, haca ya
tres semanas y evolucionando haca ya tres das con tos, disnea y compromiso del estado general. A su llegada, estaba taquipneico,
depletivo, con vmitos, confusin mental y agitacin. Presentaba de rigidez cervical terminal. Estertor crepitante en la base del
hemitrax derecho. Soplo diastlico en foco mitral +2/+6, sin irradiacin y soplo sistlico en foco artico +3/+6, con irradiacin cervical.
Leucograma de 18.700 cels/mm y 6% de bastones. La radiografa de trax mostr opacidad alveolar en la base derecha. El paciente
evolucion con insuficiencia respiratoria aguda, siendo necesario la realizacin de intubacin orotraqueal y ventilacin mecnica. Fue
realizada tomografa computadorizada (TC) de crneo que no arroj alteraciones, y la puncin lumbar con lquido cefalorraqudeo
presentando 78 cels/mm, siendo 87% polimorfonucleares, hipoglicorraquia.

PREGUNTA
Cual es la conducta teraputica mas apropiada a seguir.

RESPUESTA
a.- Ceftriaxone mas vancomicina.
b.- Meropenen mas aciclovir.
c.- Amikacina mas levofloxacina.
d.- Bencilpenicilina benzatinica.

ENFERMEDADES VASCULARES DEL SISTEMA NERVIOSO: Comprenden un conjunto de trastornos de la vasculatura cerebral que
conllevan a una disminucin del flujo sanguneo en el cerebro (flujo sanguneo cerebral o FSC) con la consecuente afectacin, de
manera transitoria o permanente, de la funcin de una regin generalizada del cerebro o de una zona ms pequea o focal, sin que
exista otra causa aparente que el origen vascular. La enfermedad cerebrovascular trae como consecuencia procesos isqumicos o
hemorrgicos, causando o no la subsecuente aparicin de sintomatologa o secuelas neurolgicas. La hipertensin arterial (HTA) es el
principal factor de riesgo de la enfermedad cerebrovascular. Epidemiologa: Actualmente la cifra de muertes por ECV supera los 5
millones anuales, lo que equivale a 1 de cada 10 muertes. Las enfermedades cerebrovasculares ocupan el tercer lugar como causa de
muerte en el mundo occidental, despus de la cardiopata isqumica y el cncer, y la primera causa de invalidez en personas adultas
mayores de 65 aos. La enfermedad cerebrovascular fue tambin la quinta causa principal de prdida de productividad, medido por los
aos de vida ajustados por discapacidad. Ello incluye los aos de prdida de productividad por razn de muerte o distintos grados de
discapacidad. Las mujeres son ms propensas a padecer ECV, sobre todo cuando han perdido el factor protector estrognico. Factores
de Riesgo: El consumo de cigarrillos es el factor de riesgo modificable ms poderoso que contribuye a la enfermedad cerebrovascular,
independiente de otros factores de riesgo. Otros factores de riesgo demostrados son la hipertensin arterial y la diabetes mellitus. Por
su parte, las dislipidemias suelen ser factores de riesgo ms importantes en la enfermedad coronaria que en la cerebrovascular.
Clasificacin: Segn el tiempo de evolucin de la ECV se agrupan en: Accidente isqumico transitorio (AIT). Es cuando los sntomas de la
focalidad neurolgica se recupera en menos de 24 horas sin secuelas, de inicio sbito que por lo general dura menos de 15 minutos.
Dficit isqumico neurolgico reversible (RIND). Es cuando la duracin del dficit persiste por ms de 24 h, pero los sntomas
desaparecen en un plazo de 7 a 21 das, sin secuelas. Accidente cerebrovascular (ACV) establecido: el dficit neurolgico de origen
vascular persiste y no se modifica por ms de tres semanas desde su instauracin y deja secuelas. ACV estable: el dficit neurolgico
persiste sin modificaciones por 24 horas (en los casos de origen carotdeo) o 72 horas (en los casos de origen vertebrobasilares),
pudiendo luego evolucionar hacia un RIND o ACV establecido. ACV en evolucin o progresivo: la focalidad neurolgica aumenta y el
cuadro empeora o aparece nueva clnica en 24 a 48 horas. ACV con tendencia a la mejora o secuelas mnimas: son casos con un curso
regresivo de modo que la recuperacin al cabo de 3 semanas es mayor al 80%. Lo ms frecuente es dividirlos en dos grupos segn el
mecanismo, as suele ser de tipo isqumico o hemorrgico: Isqumico (85% de los casos). Trombtico (Infarto lacunar: oclusin de
vasos cerebrales pequeos ocluyendo la irrigacin sangunea de un volumen pequeo de tejido cerebral. Infarto de un gran vaso
sanguneo. Emblico (Cardioemblico: la embolia proviene del corazn, con frecuencia, de la aurcula cardaca). Arteria-arteria.
Criptognico: la oclusin de un vaso intracraneal sin causa aparente. Hemorrgico: (Intraparenquimatoso, Subdural, Epidural,
Subaracnoideo). Tambin pueden ser clasificados en funcin del rea en el cerebro donde ocurre el mayor dao o por el territorio
vascular afectado y el curso clnico del trastorno. Etiologa: Trombosis en pacientes con arteroesclerosis, la hemorragia cerebral
hipertensiva, el accidente isqumico transitorio, el embolismo y la rotura de aneurismas. Las malformaciones arteriovenosas, la
vasculitis y la tromboflebitis tambin causan con frecuencia ECV. Otras causas menos frecuentes incluyen ciertas alteraciones
hematolgicas como la policitemia y la prpura trombocitopnica, los traumatismos de la arteria cartida, los aneurismas disecantes de
la aorta, la hipotensin arterial sistmica y la jaqueca con dficit neurolgico. Fisiopatologa: El flujo sanguneo cerebral (FSC) es
aproximadamente 15-20% del gasto cardaco total, de 550 - 750 ml de sangre por cada kg de tejido por cada minuto. La disminucin o
interrupcin del flujo sanguneo cerebral produce en el parnquima cerebral dao celular y funcional que empeora con el transcurrir
del tiempo. Penumbra isqumica: En el tejido cerebral donde ocurre la isquemia resultado de la disminucin del FSC, se distinguen dos
zonas: Un ncleo isqumico de isquemia intensa: A los 10 s de isquemia se aprecia prdida de la actividad elctrica neuronal por
alteraciones en los potenciales de membrana notable en el EEG. A los 30 s se observa fallo de la bomba sodio-potasio con alteraciones
MANUAL DE TRABAJO DEL CURSO ENARM CMN SIGLO XXI
CURSO ENARM CMN SIGLO XXI TEL: 36246001 Pharmed Solutions Institute PGINA 159

en el flujo ionico y desequilibrio osmtico con prdida de la funcin neuronal y edema citotxico. Al cabo de 1 min y por el predominio
de la gluclisis anaerbico, aumenta a niveles letales la concentracin de cido lctico y los mediadores de la cascada isqumica.
Despus de 5 min se aprecian cambios irreversibles en los orgnulos intracelulares y muerte neuronal. Rodeando a este ncleo
isqumico evoluciona la llamada penumbra isqumica donde el efecto de la disminucin en el flujo sanguneo cerebral, el cual ha
descendido a niveles crticos alrededor de 15 a 20 mL/100 g/min,7 no ha afectado la viabilidad celular. La extensin del rea de
penumbra depende del mejor o peor funcionamiento de la
circulacin colateral. Vulnerabilidad celular: Las neuronas ms
sensibles a la isquemia son las clulas de la corteza cerebral, del
hipocampo, el cuerpo estriado y las clulas de Purkinje del
cerebelo. De las neuroglas, se afecta primero los
oligodendrocitos, los astrocitos (gliosis reactiva) y por ltimo la
microgla. Las clulas del endotelio vascular son las ltimas en ser
afectadas. Tambin se ha notado una menor concentracin de
clulas progenitoras endoteliales en pacientes con enfermedad
cerebrovascular. Factores de influencia: Ciertos factores
participan en el dao cerebral progresivo, como el calcio, acidosis
lctica, radicales libres, glutamato, el factor de adhesin
plaquetaria y la descripcin gentica del individuo. El dao por
isquemia cerebral se ver mayor o menor dependiendo tambin
de: El estado del flujo sanguneo cerebral regional; El tiempo que
dura la oclusin vascular; El funcionamiento de la criculacin
colateral; El grado de vulnerabilidad celular frente a la isquemia;
La presencia de sustancias vasoactivas como cidos grasos y
radicales libres en la zona afectada; Hiperglicemia; Hipertermia;
Los valores de la tensin arterial; El grado de hipoxia. Cuadro clnico: La presentacin clnica de la ECV se da fundamentalmente de dos
formas: aguda, manifestada por el accidente cerebrovascular y la hemorragia subaracnoidea; y una forma crcnica manifestada por
demencia y epilepsia. Sntomas neurolgicos como debilidad, cambios del lenguaje, visin o cambios en la audicin, trastornos
sensitivos, alteracin del nivel de conciencia, ataxia, u otros cambios en la funcin motora sensorial. La enfermedad mental puede
tambin producir trastornos de la memoria. Diagnstico: Ante la sospecha de enfermedad cerebrovascular, se necesita identificar la
lesin y su ubicacin y obtener informacin sobre el estado estructural del parnquima del cerebro y su condicin hemodinmica como
consecuencia de la lesin. La evaluacin neuropsicolgica de sujetos con dao producido por enfermedad cerebrovascular est
enfocada en conocer las funciones afectadas y depende del tipo de evento. En el infarto cerebral se estudia por imgenes radiolgicas
los aspectos topogrficos de la lesin, especialmente antes de las 24 horas del inicio del trastorno sbito. Los ms utilizados son la
tomografa computarizada, la resonancia magntica y el estudio del flujo sanguneo regional cerebral. Tratamiento: El tratamiento
debe ser individualizado, segn las condiciones de cada paciente y la etapa de la enfermedad cerebrovascular, sopesando los riesgos
frente a los posibles beneficios. En general, hay tres etapas de tratamiento: la prevencin del accidente cerebrovascular; la terapia
provista inmediatamente despus de la persona sufrir un accidente cerebrovascular; y la rehabilitacin del paciente despus de sufrir el
accidente cerebrovascular. Para la prevencin de eventos cerebrovasculares, un estudio demostr que el uso de ramipril era efectivo
en pacientes de alto riesgo con o sin hipertensin arterial o ECV previo. La vitamina E no parece ser efectivo en reducir el riesgo de
accidente cerebrovascular fatal o no fatal. Terapia farmacolgica: El tratamiento mdico est destinado a reducir los riesgos y/o
complicaciones de un accidente cerebrovascular a corto y largo plazo. El uso de antitrombticos se indica tan pronto como se ha
descartado una hemorragia intracraneal. Las enfermedades cerebrovasculares no cardioemblicas, no hemorrgicas son tratadas con
agentes antiplaquetarios, en lugar de la anticoagulacin oral como tratamiento inicial. La aspirina, administrada entre 50-325 mg
diarios, o combinada con dipiridamol de liberacin prolongada, y el clopidogrel (75 mg diarios) son las opciones recomendadas de
primera lnea. La combinacin de aspirina y clopidogrel, que bloquea la activacin de la plaqueta por el difosfato de adenosina, puede
ser superior a la aspirina sola en la reduccin de riesgo de eventos cerebrovasculares isqumicos. La aspirina en combinacin con el
clopidogrel aumenta el riesgo de hemorragia y no se recomienda combinada de forma rutinaria para los pacientes con accidente
isqumico transitorio. Para casos con enfermedad cerebrovascular cardioemblico como la fibrilacin auricular, prtesis de las vlvulas
cardacas o prolapso de la vlvula mitral, se indica la anticoagulacin a largo plazo, principalmente con heparina no freccionada,
obteniendo un INR de 1,5-2,5. La administracin de 325 mg diarios de aspirina se recomienda para aquellos que no pueden tomar
anticoagulantes orales. En casos de infarto agudo de miocardio con trombo ventricular izquierdo puede combinarse la anticoagulacin
oral con aspirina hasta 162 mg diarios. En pacientes con miocardiopata dilatada tambin se indican anticoagulantes orales o algunos
clnicos consideran iniciar la terapia antiplaquetaria. Normalmente no se recomienda aadir agentes antiplaquetarios a la warfarina en
casos con enfermedad reumtica de la vlvula mitral, a menos que el paciente tenga una embolia recurrente a pesar de tener un INR
teraputico. En casos de calcificacin del anillo mitral se suele administrar tratamiento antiplaquetario. Los pacientes con regurgitacin
mitral pueden recibir warfarina o aspirina. Cerca de un 4-28% de los pacientes con hemorragia intracerebral presentan convulsiones, las
cuales pueden ser rpidamente controladas con una benzodiazepina, como lorazepam o diazepam, acompaado de fenitona o
fosfenitona. El uso de una terapia anticonvulsiva de manera profilctica en todos los casos de hemorragia intracerebral es
controvertido, ya que no hay ensayos controlados que han demostrado un beneficio claro. La tromblisis con activador tisular del
plasmingeno se ha definido como el tratamiento de primera lnea del infarto isqumico agudo, pero debe ser administrada durante las
tres horas posteriores al accidente cerebrovascular. Control de la tensin arterial: Aunque no hay estudios controlados que definan los
niveles ptimos de presin arterial en pacientes con ECV, el seguimiento de los niveles de presin arterial es importante. Se cree que la
presin arterial muy elevada puede conducir a nuevas hemorragias y/o la expansin de un hematoma intracraneal. Por otra parte, la
bajada sbita de una presin arterial elevada puede comprometer la perfusin cerebral. Las dos excepciones al manejo conservador de
la hipertensin arterial son posterior al uso de activador del plasmingeno tisular y ante un concomitante infarto de miocardio. El
nicardipino, labetalol, esmolol, y la hidralazina son agentes que pueden ser utilizados cuando sea necesario el control de la presin
MANUAL DE TRABAJO DEL CURSO ENARM CMN SIGLO XXI
CURSO ENARM CMN SIGLO XXI TEL: 36246001 Pharmed Solutions Institute PGINA 160

arterial. No se suele usar el nitroprusiato ya que puede elevar la presin intracraneal. La American Heart Association publica las
siguientes pautas para el tratamiento de la hipertensin arterial: Si la presin arterial sistlica es> 200 mmHg o la presin arterial media
(PAM) es > 150 mmHg, se considera la reduccin agresiva de la presin arterial con la infusin intravenosa continua con valoraciones
frecuentes de la presin arterial (cada 5 min). Si la presin arterial sistlica es > 180 mmHg o PAM es > 130 mmHg y hay evidencia o
sospecha de hipertensin intracraneal (PIC), entonces se considera la vigilancia de la PIC y se reduce la presin arterial con el uso de
medicamentos por va intravenosa de manera intermitente o continua para mantener la presin de perfusin cerebral > 60-80 mmHg.
Si la presin arterial sistlica es > 180 o PAM es > 130 mmHg y no hay pruebas ni sospecha de elevacin de la PIC, se considera una
modesta reduccin de la presin arterial (PAM diana de 110 mmHg o presin arterial deseada de 160/90 mmHg) con controles de la
presin arterial cada 15 minutos. Pronstico: La complicacin ms catastrfica de los diversos tipos de enfermedad cerebrovascular es
la isquemia sbita e irreversible de alguna parte del cerebro, es decir, el accidente cerebrovascular, especialmente frecuente en
ancianos. La severidad vara, desde la recuperacin total de las funciones cerebrales y aquellas que del cerebro dependen, en menos de
24 horas, hasta la discapacidad severa y la muerte. La mortalidad por ECV isqumico en el primer mes oscila entre el 17 y 34%, mientras
la del hemorrgico puede ser dos veces mayor.

CASOS CLINICOS
Mujer de 73 aos sin factores de riesgo vascular que acudi a urgencias por presentar cuadro de inicio sbito de disminucin de la
movilidad en hemicuerpo izquierdo con cada al suelo. A su llegada a urgencias estaba consciente con desviacin conjugada de la
mirada a la derecha y hemipleja completa izquierda, hemihipoestesia y reflejo de Babinsky izquierdo presente. Puntuacin en la escala
NIHSS de 16. TC craneal sin lesiones evidentes. Tras consentimiento informado escrito se aplic tratamiento tromboltico a los 150min
del inicio de los sntomas.

PREGUNTA
Considerando la sintomatologia, cual es la arteria mas probablemente involucrada?

RESPUESTA
a.- Arteria cerebral anterior izquierda.
b.- Arteria cerebral media Izquierda.
c.- Arteria cerebral anterior derecha.
d.- Arteria cerebral media derecha.

CASO CLINICO
Varn de 75 aos, con dislipemia en tratamiento con hipolipemiante como nico factor de riesgo cardiovascular. Trado a urgencias por
cuadro de hemiparesia izquierda, parlisis facial central izquierda, hipoestesia en hemicuerpo izquierdo, disartria y Babinsky izquierdo.
TC crneo sin alteraciones. NIHSS 19. Tras consentimiento informado escrito se administr tratamiento tromboltico con 150min de
evolucin desde el inicio de los sntomas.

PREGUNTA
Considerando la sintomatologia, cual es la arteria mas probablemente involucrada?

RESPUESTA
a.- Arteria cerebral anterior izquierda.
b.- Arteria cerebral media Izquierda.
c.- Arteria cerebral anterior derecha.
d.- Arteria cerebral media derecha.

CASO CLINICO
Varn de 54 aos fumador y con episodio compatible con accidente isqumico transitorio dos aos antes. Trasladado a urgencias por
hemipleja derecha de 5 horas de evolucin, objetivndose plejia completa de la extremidad superior y paresia de la extremidad
inferior, NIHSS 7.

PREGUNTA
Cual es la conducta a seguir mas apropiada al caso?

RESPUESTA
a.- Tratamiento sintomtico.
b.- Tratamiento trobolitico.
c.- Tratamiento antiplaquetario.
d.- Tratamiento anticoagulante.

CASO CLINICO
Mujer de 52 aos fumadora, sin otro factor de riesgo que fue trasladada a urgencias por hemipleja derecha y afasia de instauracin
brusca. Destac a su ingreso hemipleja completa derecha y Babinsky derecho, NIHSS de 18. Se realiz TC craneal sin que se objetivaran
alteraciones. Tras realizar consentimiento informado se aplic trombolisis a los 150min del inicio de los sntomas. A las 24h persista en
la misma situacin neurolgica, realizando TC de control que mostraba infarto isqumico extenso con edema y efecto masa, por lo que
se asoci tratamiento antiedematoso. Pese a ello 48h despus present rpido deterioro neurolgico, con nuevo TC mostrando
MANUAL DE TRABAJO DEL CURSO ENARM CMN SIGLO XXI
CURSO ENARM CMN SIGLO XXI TEL: 36246001 Pharmed Solutions Institute PGINA 161

CRITERIOS DE SCHUMACHER:
Examen neurolgico que evidencie compromiso del sistema nervioso
Evidencia clnica de 2 o ms lesiones del sistema nervioso
Compromiso principalmente de la sustancia blanca
El compromiso del sistema nervioso debe seguir el siguiente patrn: dos o ms
episodios de la enfermedad con una duracin no menor de 24 hrs y usualmente
menos de un mes, o una lenta progresin de la enfermedad por un periodo hasta
de 6 meses.
Inicio de los sntomas en edades comprendidas entre 10-50 aos
Los signos y sntomas no deben ser mejor explicados por otra enfermedad
Esclerosis mltiple definitiva apoyada en clnica
Segn clnica, la esclerosis mltiple definitiva deber reunir una de estas
condiciones:
2 episodios de la enfermedad y evidencia de 2 lesiones separadas
2 episodios de la enfermedad, evidencia clnica de una lesin y paraclnica de otra.
progresin del edema y signos de enclavamiento de tronco, desestimndose ciruga descompresiva. Finalmente la paciente fue incluida
en protocolo de donacin de rganos.

PREGUNTA
Considerando la sintomatologia, cual es la arteria mas probablemente involucrada?

RESPUESTA
a.- Arteria cerebral anterior izquierda.
b.- Arteria cerebral media Izquierda.
c.- Arteria cerebral anterior derecha.
d.- Arteria cerebral media derecha.

ESCLEROSIS MLTIPLE (EM). CIENCIAS BSICAS: La esclerosis mltiple es caracterizada por la inflamacin crnica (reaccin linfocitaria
y mononuclear) y destruccin selectiva de la mielina del SNC, importante con conservacin de los axones, el sistema nervioso perifrico
est a salvo. Hay dao de los oligodendrocitos con proliferacin astrocca. Los hallazgos anteriores junto con la prdida de la mielina
constuyen la placa de desmielinizacin. Aunque pueden verse en la mdula espinal, el tallo cerebral y el nervio ptico, la localizacin
ms frecuente es la periventricular (90%). Estas tambin
pueden encontrarse en la sustancia gris, principalmente en
localizacin subpial. Las placas aisladas raramente miden
ms de 1,5 cm. Etiologa se piensa que es autoinmune,
con la susceptibilidad determinada por factores genticos
y ambientales. SALUD PBLICA: La prevalencia de la
esclerosis mlple es muy variada en diferentes partes del
mundo, y es as como en las latudes norte la enfermedad
es ms frecuente. Por ejemplo, su prevalencia en algunas
zonas de Escocia es de 309 por cada 100.000 habitantes,
mientras que en el Norte de los Estados Unidos la
prevalencia es de 58 por cada 100.000 habitantes y en
Japn, frica y Sudamrica la prevalencia desciende hasta
cuatro personas por cada 100.000 habitantes. La incidencia
de la enfermedad es casi 1,8 veces mayor en mujeres que en hombres. Compromete ms la raza blanca y la edad de aparicin ms
frecuente est entre los 20 y los 40 aos. ASPECTOS CLINICOS: Lo ms comn son los ataques recurrentes de disfuncin neurolgica
focal, por lo general duran semanas o meses. Los pacientes con la forma de recada remisin tienen en promedio un episodio de
disfuncin neurolgica por ao. Cada recada es seguida por una secuela o recuperacin total de su cuadro clnico, las recadas estn
separadas por un periodo asintomtico. Cincuenta por ciento de los pacientes con la forma de recada remisin pueden virar a un
deterioro ms progresivo que se denomina la forma secundariamente progresiva. Aproximadamente 10 a 15 % de los pacientes tienen
una forma primariamente progresiva. En esta forma de la enfermedad hay un deterioro progresivo desde su inicio. Esta es quizs la
clasificacin clnica ms reciente de esclerosis mltiple: Asintomtica (hallazgos incidentales en RMN de cerebro) y la sintomtica que
son: 1. Recada remisin; se caracteriza por ataques recurrentes de disfuncin neurolgica general con o sin recuperacin entre los
ataques, no se observa la progresin de la discapacidad neurolgica. Representa el 85 % de los casos de EM de nueva aparicin. 2.
Secundariamente progresiva; no siempre se presenta inicialmente como recada remisin sino que evoluciona para ser de forma
progresiva. 3. Primariamente progresiva; se caracteriza por una progresin gradual de la discapacidad desde el inicio sin ataques
discretos, el 15% de los casos de EM de nueva aparicin. 4. progresiva con exacerbacin; es una forma rara que comienza con un curso
de primaria progresiva, pero se producen recadas despus superpuestas. La EM es una enfermedad crnica, 15 aos despus del
diagnstico, slo el 20 % de los pacientes tienen ninguna limitacin funcional, de un tercio a la mitad habrn progresado y requerirn
ayuda para la deambulacin. Los sntomas pueden desarrollarse de minutos a horas en 40% de los enfermos, durante varios das en
30% y lentamente durante semanas a meses, en 20% de los pacientes. DIAGNOSTICO: Clnico; El inicio puede ser brusco o insidioso.
Algunos pacientes tienen sntomas que son tan triviales que no pueden buscar atencin mdica durante meses o aos. Algunos
pacientes se presentan inicialmente con deterioro neurolgico lentamente progresivo. Los sntomas a menudo empeoran
transitoriamente con fatiga, el estrs, el ejercicio o el calor. Las manifestaciones de la EM normalmente incluyen debilidad y/o sntomas
sensoriales que implican una extremidad, dificultades visuales, alteraciones de la marcha y la coordinacin, urgencia o frecuencia
urinaria y fatiga anormal. Participacin Motor puede presentarse como una pesada rigidez dbil o torpe extremidad. Hormigueo
localizado, "alfileres y agujas" y sensaciones "muertos" son comunes. La neuritis ptica puede resultar en visin borrosa, especialmente
en el campo visual central, a menudo con dolor retroorbital acentuado por el movimiento del ojo. La participacin del tronco cerebral
puede causar diplopa, nistagmos, vrtigo o dolor facial, entumecimiento, debilidad, hemiespasmo o mioquimia (ondulacin
contracciones musculares). Ataxia, temblor, disartria y siempre son representativas de la enfermedad de las vas cerebelosas. Sntoma
de Lhermitte, una sensacin momentnea elctrica similar al shock provocado por la flexin del cuello, indica enfermedad en la mdula
espinal cervical. CRITERIS DIAGNSTICS: El diagnstico de esclerosis mltiple es de predominio clnico, y existen algunos criterios,
como los de Schumacher, que todava permanecen vigentes. Con el advenimiento de nuevos estudios paraclnicos, como los
potenciales evocados (PE), RMN (revela las reas brillantes multifocales en las secuencias ponderadas en T2 en > 95 % de los pacientes,
a menudo en una ubicacin periventricular) y el estudio de LCR (Hallazgos pleocitosis linfocitaria leve 5-75 clulas (25 %) , bandas
oligoclonales (75-90 %) , elevacin de IgG (80 %) ) TRATAMIENTO: Aunque an no existe un tratamiento definitivo para la EM. En la
actualidad, para los brotes o periodos de exacerbacin de la enfermedad se utilizan la hormona adrenocorticotropa (ACTH) y otros
corticoides. La ACTH puede administrarse siguiendo un protocolo de dosis elevadas de la medicacin que incluye 80 UI diarias
intramusculares o intravenosas por siete das, seguidas de 40 UI intramusculares o intravenosas por cuatro das, finalizado con 20 UI
por tres das. En la actualidad, se utiliza en la mayora de los centros la metilprednisolona, a dosis altas, de 500 mg a 1 g IV diarios por
MANUAL DE TRABAJO DEL CURSO ENARM CMN SIGLO XXI
CURSO ENARM CMN SIGLO XXI TEL: 36246001 Pharmed Solutions Institute PGINA 162

tres a cinco das. Se recomienda administrar esta medicacin en infusin, disolvindola en 100 ml de dextrosa al 5%, para ser
administrada en 30 minutos a 1 hora. Si la sintomatologa del paciente persiste despus de un mes a pesar de este tratamiento, se
recomienda repetir el ciclo de metilprednisolona, seguido por la administracin de 1 mg/kg de peso de prednisona interdiaria,
reducindola en el curso de 8 a 14 semanas. Se han realizado muchos estudios utilizando azatioprina; sin embargo, los resultados no
son muy buenos. Se han utilizado muchos otros tratamientos, como la ciclosporina, la plasmafresis, los ancuerpos monoclonales, el
oxgeno hiperbrico y algunos ms que estn en invesgacin sin resultados sasfactorios. El interfern beta (reduce las tasas de
exacerbacin anual aprox. 30 % y tambin reduce el desarrollo de nuevas lesiones de MRI) ha demostrado ser muy til en el
tratamiento de la esclerosis mltiple especialmente en la forma de recadas y remisin, ha sido recomendado por la Academia
Americana de Neurologa en sus guas de manejo. Se ha demostrado que la medicacin disminuye el nmero de recadas y la severidad
de las mismas. Esta medicacin debe utilizarse de por vida, si no hay disminucin en el nmero de recadas y no se observa beneficio
debe retirarse la medicacin. Terapia sintomtica: la espasticidad puede responder a la terapia fsica , lioresal (20-120 mg/d) , diazepam
(2-40 mg/d) , tizanidina (8-32 mg/d) , dantroleno (25-400 mg/d) y clorhidrato de ciclobenzaprina (10-60 mg/d) . Disestesia puede
responder a la carbamazepina (100-1200 mg/da) , fenitona (300 mg/d) , gabapentina (300-3.600 mg/d), pregabalina (50-300 mg/d) o
amitriptilina (50-200 mg/d) .El tratamiento de los sntomas de la vejiga se basa en la fisiopatologa subyacente investigados con pruebas
urodinmicas: hiperreflexia vesical es tratado con restriccin de lquidos por la noche y la miccin frecuente, y si esto no funciona,
anticolinrgicos como oxibutinina ( 5-15 mg/d) puede ser juzgado; hiporreflexia es tratado con el betanecol colinrgica drogas (10-50
mg c/8hrs) , y disinergia debido a la prdida de la coordinacin entre la pared de la vejiga y los msculos del esfnter se trata con
anticolinrgicos y cateterismo intermitente . La depresin debe ser tratada agresivamente.
PROGRESIVA: Esta enfermedad se observa en pacientes con trastornos linfoproliferavos, como la enfermedad de Hodgin, leucemia
linfocca y linfosarcomas. Tambin puede presentarse en pacientes con sndrome de inmunodeciencia adquirida (SIDA). El JC
papovavirus ha sido encontrado en la mayora de los casos de leucoencefalopaa mulfocal progresiva. La enfermedad cursa en forma
subaguda con cambios de personalidad y demencia. Otros sntomas muy comunes son hemianopsia, disartria y ataxia para la marcha. El
curso de la enfermedad es progresivo e inexorablemente lleva al paciente a la muerte en un periodo de algunos meses. El LCR puede
ser normal, la TAC de cerebro y la RMC ayudan mucho para el diagnstico y muestran el compromiso de la sustancia blanca. Un
diagnstico definitivo de la enfermedad solo puede hacerse por patologa. : Esta entidad la describieron primero
Adams y Mancall (1959). Clnicamente se caracteriza por una paraparesia o cuadriparesia de evolucin rpida, con sntomas
seudobulbares, que se manifiestan por disartria y disfagia, en pacientes con cambios electrolticos importantes. La causa real de la
mielinolisis no es clara, pero se han postulado teoras vasculares y metablicas. El cuadro se observa en alcohlicos y pacientes a
quienes se les corrige rpidamente una hiponatremia. Esta entidad tambin se ha visto asociada a otras patologas, como la
enfermedad de Wilson, la cirrosis, las nefropatas y la enfermedad de Wernicke. Histolgicamente la lesin compromete toda o parte
de la base del puente. Puede afectar el tegmento pnco y ascender hasta el mesencfalo, produciendo desmielinizacin
importante. Para diagnosticar la enfermedad son de gran importancia los potenciales evocados auditivos y la RMC. El tratamiento de
estos pacientes deber

hacerse con solucin salina, administrada con prudencia, y restriccin de lquidos. ENCEFALOMIELITIS
DISEMINADA AGUDA (ADEM): Un fulminante, a menudo devastadora, enfermedad desmielinizante que tiene un curso monofsico y
puede estar asociada con la inmunizacin antecedente o infeccin. Los signos de enfermedad neurolgica diseminada son siempre
presentes (por ejemplo, hemiparesia o tetraparesia, respuesta plantar extensora, perdidos o reflejos tendinosos hiperactivos, prdida
de la sensibilidad, y la participacin de tronco cerebral). Puede presentarse fiebre, dolor de cabeza, meningismo, letargia progresando
al coma y convulsiones. Pleocitosis, generalmente 200 clulas/l, es comn. RM puede revelar extensa realce de la materia blanca en el
cerebro y la mdula espinal. El tratamiento inicial es con glucocorticoides a dosis altas. Los pacientes que no responden pueden
beneficiarse de un curso de la plasmafresis o inmunoglobulinas.

CASO CLINICO
Paciente de 45 aos con un cuadro clnico que comenz hace 2 aos con ligera disminucin de la fuerza muscular, debilidad e
inestabilidad para la marcha, tuvo una evolucin desfavorable y en la actualidad muestra falta de coordinacin, alteraciones visuales,
rigidez muscular y trastornos del habla. Acude a nuestro centro para realizarse Resonancia Magntica por Imgenes de Crneo y
Columna cervical, encontrndose las siguientes alteraciones, en la Resonancia Magntica por Imgenes (RMI) de Crneo (Secuencias
T1, T2 y Flair axial), observamos imgenes ovoides, algunas miden 6 mm de dimetro, perpendiculares al eje mayor de los ventrculos
que se observan isointensas y algunas hipointensas en T1 y que en T2 y Flair se muestran hiperintensas, localizadas nivel del cuerpo
calloso, en la sustancia blanca profunda periventricular y a nivel de la protuberancia y bulbo; por las alteraciones antes descritas se
solicita realizar Resonancia Magntica por Imgenes (RMI) de columna cervical (secuencias T1 y T2 sagital y axial T2), donde
visualizamos a nivel de mdula reas focales hiperintensas con muy ligero efecto de masa y de caractersticas similares a las del
encfalo.

PREGUNTA
Considerando la clasificacin de esta patologia, cual es el grado mas probable del caso?.

RESPUESTA
a.- 1.
b.- 2.
c.- 3.
d.- 4.

CASO CLINICO
Masculino de 68 aos de edad quien, previamente sano, sufre un accidente de trnsito al ser atropellado; producindose politrauma
severo y fractura de base de crneo, fractura expuesta del tercio medio de la tibia derecha y trauma de la rodilla izquierda. Al ingreso
Glasgow 3 se describe pupila izquierda media no reactiva, sospechando un III par craneal compresivo se realiza TC que corrobora la
MANUAL DE TRABAJO DEL CURSO ENARM CMN SIGLO XXI
CURSO ENARM CMN SIGLO XXI TEL: 36246001 Pharmed Solutions Institute PGINA 163

fractura y demuestra una hemorragia subaracnoidea sin hematoma; se instaura tratamiento con fenitona y diurticos; se considera
que presenta un III par craneal perifrico traumtico. En 12 horas Glasgow 7. A los 2 das movilizaba las 4 extremidades y abra los ojos
al llamado, no acataba rdenes; posteriormente su sensorio mejor manteniendo un Glasgow de 14. Durante la estancia el paciente
recibi varios esquemas de antibiticos, debido a neumona nosocomial asociada a ventilacin mecnica la cual requiri los tres
primeros das. Al mes de estancia presenta progresivo descenso de la natremia con valores desde 126 mmol/l, hasta 108 a las dos
semanas cuando se inici el tratamiento de reposicin; concomitantemente el paciente tena progresivo deterioro del sensorio.
Durante las primeras horas la correccin se logr llevar a un ritmo adecuado, pero no ocurri as a las 12 horas de reposicin que se
pas de un sodio de 116 a 125 mmol/l en 5 horas. A los 2 das el paciente nuevamente conciente pero se anota que presenta dificultad
para la deglucin; luego presenta un cuadro de diarrea asociada a antibiticos que lo hace llegar a una natremia de 138 mmol/l.

PREGUNTA
Cual es la velocidad de correccin de sodio para evitar esta complicacin del caso?

RESPUESTA
a.- 05 mmol/l/dia.
b.- 10 mmol/l/dia.
c.- 15 mmol/l/da.
d.- 20 mmol/l/dia.

CASO CLINICO
Paciente de sexo masculino, 44 aos, diestro, infeccin por VIH, en tratamiento antirretroviral. Presentaba una poblacin linfocitaria
con CD4 inferior a 160 elementos/mm3. Comienza con un sndrome confusional, alteraciones conductuales y cefalea moderada. A los
pocos das de su ingreso se percibe prdida de fuerza de sus miembros izquierdos y alteraciones del habla que se agravaron en el curso
de los das. No se detecta fiebre. El examen mostraba un paciente irritable, con desorientacin temporal y espacial, tendencia al
mutismo, sndrome tnico frontal bilateral predominando a derecha. No se detect sndrome menngeo ni fiebre. La TC craneal en fase
de estado mostr una lesin frontal derecha y de cuerpo calloso, con compromiso menor del lbulo frontal izquierdo que incida
especialmente sobre la sustancia blanca, sin efecto de masa y con realce con el contraste. La RM enceflica delimit con mayor
exactitud las lesiones observndose lesiones de baja seal en T1 y de aumento de seal en T2. Se resuelve practicar biopsia
estereotxica, Hay acmulos de histiocitos de citoplasmas microvacuolados por fagocitosis de restos mielnicos lipdicos, Existe
moderada astrogliosis con astrocitos de aspecto bizarro smil neoplsicos, con ncleos grandes, algunos multilobulados que no se
acompaan de aumento de densidad celular. La oligodendrogla muestra una cariomegalia hipercuomtica y algunos de ellos tienen
gruesas inclusiones intranucleares acidfilas 'en vidrio esmerilado' que contienen el antgeno viral. Hay escaso o nulo exudado
inflamatorio perivascular linfo-plasmocitario".

PREGUNTA
Considerando la clnica y los hallazgos histolgicos, cual es el diagnostico mas apropiado?

RESPUESTA
a.- Leucoencefalopaa mulfocal progresiva.
b.- Encefalomielitis diseminada aguda.
c.- Mielinlisis pnca.
d.- Escleroris lateral amniotrofica.

CASO CLINICO
Paciente de 21 aos, con antecedentes de pan-colitis ulcerosa, en tratamiento esteroidal con prednisona 60 mg/da y Mesalazina 1 gr
c/6 hrs durante 5 meses previos al ingreso. Das previos al diagnstico complet tratamiento con ciprofloxacino por sndrome diarreico
presuntamente bacteriano, sin confirmacin etiolgica y con respuesta favorable. Ingres al Servicio de Urgencia por cuadro de 48
horas de evolucin de compromiso de conciencia caracterizado por desorientacin temporo-espacial y somnolencia, adems de
deposiciones sanguinolientas. Al examen fsico general presentaba parmetros hemodinmicos dentro de rangos normales, afebril y
como nico hallazgo se describe candidiasis orofarngea. Al examen neurolgico el paciente se encuentra vigil, con tendencia a la
somnolencia, desorientado en tiempo y espacio, nistagmo vertical, prueba de pequea paresia positiva a derecha, y signo de
Brudzinski. Resto del examen sin hallazgos. Se realiza TAC de cerebro que no muestra hallazgos patolgicos y se realiza puncin lumbar
que da salida a lquido cefalorraqudeo claro, con protenas 74,8/mm3, glucorraquia normal, glbulos rojos 2/mm3, leucocitos 760/
mm3 de predominio linfoctico. Por antecedente de inmunosupresin crnica y candidiasis orofarngea al ingreso, se inicia tratamiento
antibitico con ampicilina+ceftriaxona+aciclovir y corticoides en dosis de estrs (Hidrocortisona 100mg c/8 hrs). Evoluciona en buenas
condiciones generales, estable desde el punto de vista hemodinmico y recuperando conciencia a las 48 horas. Se realiza Resonancia
magntica que muestra imgenes con compromiso talmico bilateral asimtico asociado a compromiso capsular posterior izquierdo
con discreta captacin de contraste.

PREGUNTA
Considerando la clnica y los estudios de imagen, cual es el diagnostico mas apropiado?

RESPUESTA
a.- Leucoencefalopaa mulfocal progresiva.
b.- Encefalomielitis diseminada aguda.
c.- Mielinlisis pnca.
MANUAL DE TRABAJO DEL CURSO ENARM CMN SIGLO XXI
CURSO ENARM CMN SIGLO XXI TEL: 36246001 Pharmed Solutions Institute PGINA 164

d.- Esclerosis lateral amniotrofica.

DEMENCIAS:
CIENCIAS BASICAS: La demencia es un sndrome clnico caracterizado por perdida adquirida de habilidades cognitivas y emocionales, lo
suficientemente importante para interferir con el funcionamiento diario y la calidad de vida. Tambin puede acompaarse de otras
manifestaciones neuropsiquitricas tales como alteraciones motoras, de la conducta, depresin, ansiedad, alucinaciones y/o delirium.
El trmino demencia no implica una causa o proceso patolgico especfico, es de etiopatogenia heterognea y puede ser mixta. Existen
ms de 55 enfermedades que pueden causar demencia. La definicin de demencia segn Clasificacin de la demencia de acuerdo al
DSM IV se divide de la siguiente manera: Demencia en enfermedad de Alzheimer; de inicio temprano, de inicio tardo, mixta o atpica,
inespecfica. Demencia vascular; de inicio agudo, multi-infarto, vascular subcortical, mixta: subcortical y cortical. Demencias propias de
otras enfermedades; Enfermedad de Pick, Enfermedad de Creutzfeldt-Jakob, Enfermedad de Huntington, Enfermedad de Parkinson,
asociada a infeccin por VIH, asociada a otras enfermedades. Demencia no especificada. As mismo una vez hecho el diagnstico de
demencia se puede clasificar: Leve: cuando afecta actividades de la vida diaria sin causar problemas para la independencia del paciente.
Moderado: cuando condiciona incapacidad para vivir de manera independiente. El individuo no recuerda informacin bsica acerca de
su vivienda, actividades recientes o el nombre de ciertas personas familiares a l. Severo: Se caracteriza por la absoluta incapacidad de
retener nueva informacin y solo se recuerdan fragmentos de experiencias o conocimientos pasados. El paciente no reconoce a sus
familiares. SALUD PUBLICA: La E. Alzheimer en 2050 podra llegar al 30% de la poblacin mundial. A medida que se prolonga la
expectativa de vida tambin aumenta la prevalencia de enfermedades como las demencias. La frecuencia de E. Alzheimer se duplica
cada 5 aos apartir de los 60a. A los 60 aos 1%, entre 80-84 aos de 16%, >de 85aos de 30-40%. Existen factores de riesgo no
modificables como la edad siendo este el ms importante para el desarrollo de demencia. En los pacientes con trastorno del
aprendizaje como Sndrome de Down las demencias se presentan ms temprano. Estudios de prevalencia muestran un mayor ndice de
demencia en mujeres que en hombres, especialmente en enfermedad de Alzheimer (2:1). El nmero de casos de demencia vascular es
mayor en hombres que en mujeres aunque stas ltimas suelen igualarlos en edades ms avanzadas. La historia familiar de demencia
en un pariente de primer grado aumenta cuatro veces el riesgo de desarrollar demencia. Demencia vascular es la segunda causa de
demencia despus del Alzheimer representando hasta el 18% de las demencias, se han descrito prevalencias que varan desde el 3 al
21%. DIAGNOSTICO: Para un diagnstico clnico confiable la prdida de la memoria debe estar presente al menos durante seis meses; si
la duracin de las manifestaciones es menor, el diagnstico es presuntivo. La severidad de la demencia se establece por el grado de
decremento cognitivo o de la memoria, cualquiera que sea el ms predominante. Cuando existen episodios concomitantes de delirium
el diagnostico de demencia debe ser diferido. El diagnstico de las demencias se debe realizar por un especialista experto en el tema y
podra abordarse de la siguiente establece en dos pasos: se deber hacer solo tras una extensa evaluacin que debe incluir: historia
clnica, evaluacin del estado cognitivo, examen fsico y otros estudios apropiados, una revisin de los medicamentos tomados por el
paciente en orden de descartar efectos adversos que afecten el funcionamiento y minimizar su uso. ENFERMEDAD DE ALZHEIMER:
Sndrome clnico adquirido y progresivo caracterizado por afectar las funciones intelectuales superiores. Deterioro de la memoria en el
corto y largo plazo. Presencia de por lo menos uno de los siguientes dficits cognitivos: Afasia, Agnosias, Apraxia y/o alteraciones de
funciones ejecutivas. Alteracin de conducta: Laboral y social. No debe existir alteracin de nivel de conciencia. En su patogenia exiten
Hiptesis colinrgica: hay prdida de neuronas colinrgicas en corteza e hipocampo con aumento de accin de butirilcolinesterasa.
Hiptesis glutamatergica: L-glutamato neurotransmisor exitatorio SNC, interviene en los procesos de memoria, aprendizaje y
plasticidad neuronal, el aumento parece jugar un rol principal en la patognesis y en el dao producido por isquemia. Se genera por la
produccin anormal y acumulacin de amiloide, aumenta la produccin de A1-42 que es el que se deposita. Se generan las placas
de amiloide que desencadenan, formacin de ovillos neurofibrilares (por hiperfosforilacin de protena Tau). Oxidacin y peroxidacin
de lpidos. Excitotoxicidad por glutamato, inflamacin, activacin de la cascada que lleva a la apoptosis. La muerte neuronal lleva al
dficit de neurotrasmisores. En las guas de diagnstico, se describe de manera didctica 10 signos de alerta para pensar en Alzheimer:
1. Prdida de la memoria 2. Dificultad para realizar tareas familiares 3. Problemas con el lenguaje 4. Desorientacin en tiempo y espacio
5. Juicio empobrecido 6. Problemas con el pensamiento abstracto 7. Prdida de cosas 8. Cambios en el nimo o el comportamiento 9.
Cambios en la personalidad 10. Prdida de la iniciativa Alteraciones neuropsiquitricas y motoras. Conforme la enfermedad avanza se
presentan apraxias y con ello la incapacidad funcional para las actividades cotidianas, como asearse y vestirse, puede haber
desinhibicin y agresividad, las alucinaciones son poco frecuentes y si se presentan el diagnstico debe orientarse en otra direccin, sin
embargo pueden desarrollar delusiones, entre ellas se ha descrito el sndrome de Capgras hasta en el 10% de los pacientes, donde
generan la idea de que su cuidador ha sido sustituido por un impostor, paralelamente hay alteraciones en el patrn de sueo vigilia,
sntomas motores extrapiramidales como rigidez y sacudidas mioclnicas, hasta una etapa final de mutismo y postracin. Tratamiento:
Inhibidores de la Acetilcolinesterasa: Donepecilo, rivasigmina, galantamina. Inhibidores de los receptores NMDA: Memantina.
Estabilizadores de membrana: Lamotrigina, carbamacepina, topiramato. Delirios y alucinaciones: Risperidona, olanzapina, haloperidol.
Depresin y ansiedad: citalopram, paroxetina, sertralina. DEMENCIA VASCULAR: Se ha acuado el trmino de dao cognitivo vascular
para describir a la prdida de funciones cognitivas asociada a otras manifestaciones neurolgicas y que tienen como base el dao
vascular. Factores de riesgo, la edad, el bajo nivel de escolaridad, la enfermedad hipertensiva, el dao cerebral vascular previo,
especialmente infartos cerebrales, el padecer diabetes, obesidad, sndrome metablico, niveles elevados de homocistena y la
ateroesclerosis. Clasificacin: Demencia por dao cortical o enfermedad multi infarto relacionada generalmente con el dao de grandes
vasos (23%) y la causada por dao subcortical o enfermedad difusa de la sustancia blanca relacionada a dao de pequeos vasos (50%).
La regin ms afectada son los ganglios basales, dentro del dao subcortical se hace referencia a tres tipos de entidades, 1) infartos
lacunares (16%), la distribucin puede variar, sin embargo el dao se ha descrito con mayor frecuencia en la regin frontal, 2) demencia
talmica (8%) donde el dao puede venir desde la arteria basilar y 3) la encefalopata subcortical ateroesclertica o sndrome de
Binswwanger donde hay gran dao a la sustancia blanca. El dao vascular, como ya se coment puede ser secundario a infartos en
grandes vasos corticales o a nivel subcortical predominantemente ya sea por hemorragias o principalmente por eventos vasculares
isqumicos donde la hipoperfusin lleva a la gliosis y al dao en la sustancia blanca que posteriormente se manifestar como
leucoaraiosis, otro tipo de dao es la isquemia crnica subcortical que da afeccin principalmente a nivel periventricular. Edad de
inicio es en entre los 50 y 59 aos, con otro pico entre los 60 y 69 aos de edad. Clinica: pueden desarrollan alteraciones en la marcha,
MANUAL DE TRABAJO DEL CURSO ENARM CMN SIGLO XXI
CURSO ENARM CMN SIGLO XXI TEL: 36246001 Pharmed Solutions Institute PGINA 165

afectacin de primera neurona motora, bradicinesia, rigidez, alteracin en los esfnteres, sndrome pseudobulbar, alteraciones de la
personalidad y del estado de nimo, afasia, agnosias y alteraciones en la memoria. Establecer criterios diagnsticos definitivos ha sido
complicado, las ms utilizadas son las de Hachinski, Rosen. Tratamiento: donepezilo con dosis inicial de 5mg al da y de mantenimiento
de 10-20mg despus de 4 semanas, rivastigmina con dosis inicial de 1.5mg, incrementando de 1.5mg diarios hasta lograr la dosis de
mantenimiento de 6mg o parches de 4.6mg por da con dosis paulatinas con un mximo de 9.6 mg/da. Dado que se trata de una
demencia que se puede prevenir, es importante dar tratamiento a los factores de riesgo. DEMENCIA FRONTOTEMPORAL: Tercera
demencia ms comn despus de la Demencia tipo Alzheimer. La prevalencia general es de 17,6 por 100.000 habitantes. La edad de
presentacin es aproximadamente a los 58 aos. Afecta por igual a hombre y mujeres. En el 30-50 % de los casos existe una historia
familiar de Demencia frontotemporal (DFT) principalmente en la variante conductual, mientras que en la demencia semntica y la
afasia progresiva no fluentes menos frecuente. El trmino de Enfermedad de pick se reserva para casos de DFT con inclusiones
positivas teidas de plata denominados cuerpos de Pick. Debido a su asociacin de atrofia en lbulos temporales y frontales. La
mutacin caracterstica se encuentra en el gen que codifica a la protena tau en el cromosoma 17, cuyo patrn es de herencia
autosmico dominante. Esta protena pertenece a la familia de las MAPT (protenas tau asociadas a microtbulos) y tiene participacin
en la regulacin de la polimerizacin y ensamble de las subunidades de tubulina, por lo que es indispensable para determinar la
estabilidad y funcin axonal. La DFT es una enfermedad progresiva con inicio insidioso, en etapas iniciales presentan ms sntomas
conductuales, sin afectar la memoria ni las capacidades visuoespaciales. Puede ser espordica o familiar. Pueden presentar diversas
alteraciones de acuerdo al sndrome de DFT, sin embargo se caracteriza por la presencia de afasia, alteraciones en la conducta y del
lenguaje. Las alteraciones en el lenguaje pueden existir con o sin demencia asociada, aunque se estima que la mayora de los pacientes
desarrollara demencia en etapas tardas de la enfermedad. Existe un deterioro de la funcin ejecutiva que incluye la planificacin,
organizacin, flexibilidad, juicio, resolucin de problemas con conservacin de la percepcin visual y habilidades espaciales. Existe
tambin una incapacidad para reconocer emociones, en particular las negativas como la ira, el miedo, tristeza. La orientacin en
tiempo y lugar, el recuerdo libre demorado son alteraciones frecuentes de la DMF, sin embargo estn presentes tambin durante las
primeras etapas de la enfermedad de Alzheimer, su distincin sigue siendo difcil mediante pruebas neuropsicolgicas.

CASO CLINICO
Se trata de masculino de 73 aos de edad el cual es originario de Mrida, actualmente vive solo, padece diabetes mellitus desde hace
15 aos, hipertensin arterial, triglicridos, colesterol y acido rico elevados, al interrogatorio se desconoce la posologa para sus
padecimientos, refiere que le indicaron glibenclamida, bezafibrato, enalapril, alopurinol y pravastatina, el familiar refiere que desde
hace mas de dos aos presenta dificultad para recordar las cosas que est haciendo, se ha vuelto desconfiado, esconde las cosas para
evitar que se las roben, pobreza de pensamiento, pensamiento con bloqueo e ideas de dao y robo.

PREGUNTA
Debido a las mltiples entidades nosolgicas que presenta el paciente cual es complicacin neurolgica ms probable.

RESPUESTA
a.- Demencia vascular es la ms probable.
b.- Demencia de Alzheimer es la ms probable.
c.- La demencia por priones es la ms probable.
c.- La demencia de Pick es la ms probable.

CASO CLINICO
Masculino con neumona por aspiracin, con antecedente de prdida progresiva de funciones mentales tipo demencia, previamente
presento cambio de conducta, prdida de peso y cefalea continua de tres meses de duracin. A la EF alerta, mutista, el examen
neurolgico no revela alteraciones sensoriales, pares craneales sin afecciones sin embargo presenta mioclonos a la estimulacin y
expontaneos incluyendo durante el sueo, los estudio de laboratorio no muestran datos de importancia, el liquido cefalorraqudeo
normal al igual que la TAC de cabeza.

PREGUNTA
Cul es el agente causal ms probable en este caso.

RESPUESTA
a.- Viral ADN.
b.- Viral ARN.
c.- Fungico.
d.- Prion.



MANUAL DE TRABAJO DEL CURSO ENARM CMN SIGLO XXI
CURSO ENARM CMN SIGLO XXI TEL: 36246001 Pharmed Solutions Institute PGINA 166

EPILEPSIA:
CIENCIAS BASICAS: Se define como la tendencia a tener convulsiones recurrentes. La epilepsia es una manifestacin de la enfermedad
cerebral subyacente. Convulsiones simples o aquellos que ocurren durante la enfermedad aguda no deben ser clasificados como
epilepsia. Causas: Desconocido en dos tercios de los casos .En Reino Unido, las encuestas de la comunidad muestran: Enfermedad
cerebrovascular, 15 %; tumor cerebral, 6 %; relacionados con el alcohol, 6 %; post-traumtico, 2 %; trastornos genticos, 1 %. Otras
causas incluyen la esclerosis del hipocampo y malformaciones corticales y
vascular. En los trpicos, la neurocisticercosis es una causa comn. SALUD
PUBLICA: 50/100 000/ao. 1 de cada 200 tienen epilepsia activa (en el Reino
Unido, 350.000) .Mayor incidencia en los pases en desarrollo. En Mxico las
crisis convulsivas de origen tardo son en su mayora debidas a cisticercosis.
CLASIFICACION: Bsica en generalizada (50 %) y focal (50 %), subdividido en
categoras etiolgicos: Idioptica (predisposicin gentica con el desarrollo
normal, el examen y EEG). Sintomtico (anomala estructural). Criptognicas
(anomala estructural supone, pero no probado). Subdivisiones: 1. Parcial
simple; alteracin motora, sensitiva, autonmica o psquica sin alteracin del
estado de conciencia. 2. Parcial compleja; alteracin del estado de
conciencia, puede comenzar con prdromos o con sntomas motores,
sensitivos, autonmicos o psquicos, presenta automatismos y es seguida
por un periodo de confusin (estado posictal). 3. Parcial secundariamente
generalizada: Inicia con alteraciones motoras, sensitivas, autonmicas o
psquicas, seguidas de prdida del estado de conciencia, con aumento de
tono muscular, contracciones rtmicas (clnicas), al final paciente comatoso
y recuperacin lenta. Puede haber incontinencia y/o mordida de lengua. 4.
Generalizada de ausencia: Inicio sbito con periodo breve de desconexin
con el medio y recuperacin rpida, tono muscular puede aumentar o
disminuir, se acompaa de automatismos o movimientos clnicos leves. 5.
Generalizada tnico-clnica: Perdida sbita del estado de alerta,
acompaada de contracciones tnico clnica generalizadas y hay estado posictal. STATUS EPILEPTICO: Presencia de una crisis
convulsiva continua o crisis repetitivas, discretas y con alteracin del estado de conciencia en el periodo interictal, el lmite de tiempo
es de 5 min. La causa ms frecuente es abstinencia a frmacos anticonvulsivantes o falta de apego al tratamiento. PATOGENIA: Es
producida por el disparo sincrnico y sostenido de una poblacin de neuronas cerebrales. Las estructuras ms susceptibles al desarrollo
de crisis convulsivas recurrentes son la corteza motora y la formacin del hipocampo y complejo amigdalino del sistema lmbico.
DIAGNOSTICO: Clnico; Los ataques son estereotipados, eventos paroxsticos. El diagnstico es clnico, testimonios son cruciales. Por lo
general, seguido de un perodo de somnolencia. Ver ' La prdida de la conciencia ". Desencadenantes incluyen: alcohol, la fatiga,
privacin del sueo, infecciones, hipoglucemia, el estrs, luces estroboscpicas (epilepsia fotosensible), lectura, el agua caliente (raro).
Ausencias infantiles; raras despus de 10 aos de edad. F> M. Breve prdida de la conciencia varias veces al da, accionado por la
hiperventilacin, remiten en la edad adulta. EEG caracterstico; 3 Hz puntas y olas, sin fotosensibilidad. Epilepsia mioclnica juvenil
(JME): inicio antes de los 30 aos, sacudidas mioclnicas por la maana, ausencias tpicas, convulsiones tnico-clnicas generalizadas.
EEG tpico generalizado con puntas y ondas puede haber o no fotosensibilidad, remisin raro. Las convulsiones parciales: complejas
asociado con anormalidad estructural que subyace, por ejemplo, esclerosis del hipocampo, automatismos (relamerse los labios, la
masticacin, la deglucin, movimientos manuales estereotipados).
Dj vu y jamais vu, auras olfativas (desagradable). Comportamiento
inusual o emocionalidad. En aquellos pacientes con epilepsia
previamente diagnosticada y que presenta una crisis epilptica, se
debern revisar los niveles sricos del frmaco, obtener electrlitos,
BH, pruebas de funcin heptica y renal, as como panel toxicolgico,
si todo sale normal, pensar en la modificacin teraputica, aumentar
dosis mxima, disminuir dosis de 1er frmaco e iniciar un segundo.
Pacientes que no tengan historia previa de epilepsia, solicitar BH,
electrolitos, calcio, magnesio, glucosa, funcin heptica renal, EGO y
panel de toxicologa, si salen normal o negativo hacer RMN o
electroencefalograma, para buscar causas focales de las crisis (tumor, EVC, neuroinfeccin, trauma, enfermedad degenerativa) y tratar
la causa especfica as como considerar tratamiento anticonvulsivo. Si todo sale normal se puede pensar en crisis idiopticas.
TRATAMIENTO: Recomendaciones generales; Evite las actividades peligrosas, por ejemplo, nadar solo, el montaismo. Tome duchas en
lugar de baos. Convulsiones simples: No hay un tratamiento a menos que exista un alto riesgo de recurrencia, por ejemplo, EEG
anormal como en JME o un MRI anormal. Si los factores precipitantes (por ejemplo, alcohol), la evitacin pueden prevenir la
recurrencia. Despus de un solo ataque no provocado, el riesgo de recurrencia es del 24%, sin causa y EEG normal y 65 % si se asocia
con una anormalidad neurolgica + EEG anormal. Profilaxis: No hay indicacin para iniciar el tratamiento en pacientes con lesiones en
la cabeza, craneotoma, tumores cerebrales, a menos que se presentan convulsiones. La resistencia al tratamiento farmacolgico se
presenta hasta en 20% de los casos. En focos epileptgenos bien definidos se debe considerar el tratamiento quirrgico con reseccin
del mismo. Estatus epilptico: Minutos 05 Documentacin clnica. Diagnstico correcto. Identificacin del tipo de SE.
Breve anamnesis y exploracin fsica. Medidas bsicas: Mantener funcin cardiorrespiratoria. Monitorizacin de constantes vitales.
Va IV permeable, realizacin de exmenes analticos y administracin de antiepilpticos. Si es posible, documentacin EEG.
Minuto 610 Segn sospecha clnica (etilismo, dficit nutricional, hipoglicemia). Tiamina IM (100mg).
Glucosa IV (50 cc al 50% adulto, 24 cc/kg al 25% nio). Finalizacin del SE clnico y elctrico (tratamiento farmacolgico).
CLASIFICACION DE EPILEPSIA
EPILEPSIAS GENERALIZADAS Y SNDROMES:
Idiopticas de aparicin relacionada con la edad
a)Epilepsia de ausencia infantil
b)Epilepsia mioclonica juvenil (JME)
c)Epilepsia con crisis tnico-clnica
Sintomticas y criptognicas
a)Sndrome de West
b)Sndrome de Lennox- Gastaut
c)Epilepsias con ausencias mioclnicas
Sintomticas
a)Encefalopata mioclnica
EPILEPSIAS FOCALES Y SNDROMES
Idioptica de aparicin relacionada con la edad
a)Epilepsia infantil benigna con puntas centrotemporales
b)Epilepsia de lectura
Sintomtica:
a)Epilepsia con convulsiones parciales simples, complejas o
secundariamente generalizadas, que surgen de cualquier parte de
la corteza
b)Epilepsia parcial continua
c)Sndrome caracterizado por la activacin especifica
Epilepsias
EPILEPSIAS INDETERMINADAS (FOCAL O GENERALIZADA)
Epilepsias con puntas continuas y actividad de las ondas en el
sueo
TIPO DE EPILEPSIA 1RA. LINEA 2DA. LINEA
Generalizadas Valproato (200mg aumentar
c/2 sem. maximo 2.5 grs) o
Lamotrigina (25 mg/d,
mximo 400mgs)
Levotiracetam o
Topiramato
(25mg/d mximo
400mg/d)
Focal Carbamacepina (200mg/d) o
Lamotrigina o
Valproato
Levotiracetam o
Topiramato
Mioclonicas,
atnicas
Valproato o
Lamotrigina
Levotiracetam
(250mgs/d)
Ausencia Etosuximida
Valproato o

MANUAL DE TRABAJO DEL CURSO ENARM CMN SIGLO XXI
CURSO ENARM CMN SIGLO XXI TEL: 36246001 Pharmed Solutions Institute PGINA 167

Diazepam IV (2mg/min hasta que cedan, mximo 0.3 mg/kg).Loracepam IV (0.1 mg/kg, ritmo 2mg/min hasta que cedan, mximo 10m.
Pueden constituir el nico tratamiento si las crisis ceden y la causa del SE es corregida. Si no es as, asociar; fenitona IV (15a20 mg/kg,
en suero salino, ritmo 50 mg/min en adultos y 1 mg/kg/min en nios). Minuto 30-40 Status resistente, ingreso en UCI, opciones:
Diazepam, en perfusin contina (100mg en 500cc glucosado 5%, ritmo de 40 cc/hr). Fenobarbital IV (20 mg/kg, ritmo 50100 mg/min).
Fenitona IV (hasta 30 mg/kg, dosis aadida de 510 mg/kg). cido Valproico 20mg/Kg. Minuto 60: SE refractario. Ingreso en UCI
(intubacin orotraqueal, ventilacin mecnica, acceso vascular central, monitorizacin EEG. Anestesia general, elegir una opcin:
Midazolam IV/Propofol IV. Barbitricos (pentobarbital, tiopental).Otros (clonazepam, clormetiazol, lidocana, valproato sdico,
paraldehido, agentes inhalantes, bloqueantes neuromusculares). Minuto 060/90: Diagnstico y tratamiento etiolgico.
Completar amnamnesis y exploracin fsica detalladas. Exmenes complementarios (TAC, IRM, LCR), pero nunca posponer el tratamien
Decidir tratamiento etiolgico o de factores precipitantes. Correccin y prevencin de complicaciones sistmicas. Hipoglicemia,
alteraciones hidroelectrolticas y del equilibrio cidobase, hipotensin, hipoxia, edema cerebral, HIC, fiebre.
Prevenir la recidiva del SE Tratamiento farmacolgico antiepilptico de mantenimiento. COMPLICACIONES DEL STATUS EPILPTICO:
Neurolgicas Encefalopata hipxica, Edema cerebral, Hipertensin endocraneal, Trombosis senos venosos, Vasculares
(infartos, hemorragias) Cardiovasculares y respiratorias Hipertensin / Hipotensin, arritmias cardiacas, parada cardiaca, insuficiencia
respiratoria / cardiaca, Edema pulmonar Endocrinometablicas Acidosis metablica / respiratoria, Hiperglicemia / Hipoglicemia,
Alteraciones de la funcin endocrina, Deshidratacin, Hiponatremia, Insuficiencia renal aguda, Pancreatitis aguda, Falla heptica
Otras Coagulacin intravascular diseminada.

CASO CLINICO
Hombre de 62 aos, debuta con crisis de disnea, visin borrosa, cada al suelo, movimientos repetitivos de los brazos y estado
confusional de duracin total de 10 minutos. Fue tratado con fenitona y cido valproico con lo que permaneci estable, presentando
slo 3 aos despus nueva crisis con similares caractersticas. Comienza a notar cambios conductuales, desinters, irritabilidad y
comportamiento obsesivo. Se hospitaliza por cuadro de compromiso de conciencia progresivo de varios das de evolucin, agitacin
psicomotora, desorientacin, desviacin de la mirada hacia arriba y derecha, temblor del mentn y parpadeo. Al ingreso se encontr
adems distonas de las extremidades y mantencin espontnea de posturas de apariencia catatnica, fenmenos que cedieron con el
uso de diazepam endovenoso. Durante esta hospitalizacin present varios otros episodios catatoniformes que duraban horas o das y
que en algunas oportunidades se asociaron a mioclonas faciales y de la mano derecha. El EEG se mantuvo persistentemente alterado
con actividad lenta generalizada. Se interpret clnicamente como un cuadro epilptico, con buena respuesta a los anticonvulsivantes.
El estudio imagenolgico mostr una atrofia frontal bilateral. El estudio de laboratorio obtuvo una amonemia alta (44 mol/L) que fue
interpretada como secundaria al uso de valproato, por lo que debi modificarse el tratamiento anticonvulsivante.

PREGUNTA
La asociacin de demencia y epilepsia obliga a pensar en diagnsticos diferenciales, cual de los siguientes es ms frecuente?

RESPUESTA
a.- Porfiria aguda intermitente.
b.- Neuroacantocitosis.
c.- Dficit de vitamina B12.
d.- Enfermedad de Lyme.

CASO CLINICO
Se trata de femenino de 84 aos de edad la cual ingresa a consulta por presencia de dificultad para deglutir los alimentos, presentando
accesos de tos hemetizante durante las ingestas, y desde hace 48 hs., fiebre, dificultad respiratoria progresiva, somnolencia
permanente y expectoracin muco purulenta ftida, se observa prdida total de la independencia funcional, postrada en cama desde
hace aproximadamente 3 aos, al cuidado permanente de 2 de sus hijas, portadora adems de miocardiopata dilatada, cumpliendo
tratamiento en la actualidad con Digoxina, Furosemida, Enalapril, AAS y vasoactivos cerebrales.

PREGUNTA
Considerando los antecedentes y al cuadro clnico, cual es el diagnostico mas probable?

RESPUESTA
a.- Demencia vascular.
b.- Demencia seil.
c.- Demencia alzheimer.
d.- Demencia de pick.

CASO CLINICO
Masculino de 69 aos de edad con antecedentes de HTA de carcter leve -moderada desde hace 8 aos aproximadamente adems de
Hipercolesterolemia desde hace 10 aos segn refiere, para lo cual ingiere como medicacin, Enalapril 10 mg e hidroclorotiazida 12,5
mg una vez por da, complementando dicho tratamiento con dieta hiposdica e hipocolesterolemica. Manifiesta que no cumple
estrictamente con la dieta descripta pero que siempre ingiri la medicacin antihipertensiva. Consulta el da de hoy por cefalea a
predominio frontal, la cual se manifiesta desde hace varios das acompaado de sensacin de inestabilidad que no sabe referir
exactamente pero si manifiesta sentirse muy extrao, situacin que no es habitual en el pero, que lo inquieta y preocupa por lo cual
decidi concurrir a la consulta medica, por propios medios. Manifiesta que ltimamente no control su T A como se le haba indicado y
que tampoco respet la dieta habitual. Como antecedente importante refiere hipercolesterolemia leve. Adems expresa que siempre
MANUAL DE TRABAJO DEL CURSO ENARM CMN SIGLO XXI
CURSO ENARM CMN SIGLO XXI TEL: 36246001 Pharmed Solutions Institute PGINA 168

llev una vida activa, que sola concurrir desde el pueblo hasta su domicilio rural en bicicleta hasta hace unos 10 aos
aproximadamente.

PREGUNTA
Considerando los antecedentes y al cuadro clnico, cual es el diagnostico mas probable?

RESPUESTA
a.- Demencia vascular.
b.- Demencia seil.
c.- Demencia alzheimer.
d.- Demencia de pick.

CASO CLINICO
Paciente de 73 aos de edad que presenta temblor de reposo de predominio en miembro superior derecho, rigidez, bradicinesia y
alteracin de reflejos posturales. En la ltima visita, la familia que lo acompaa relata que a lo largo de los ltimos 6-8 meses, lo vienen
notando triste, poco comunicativo, "perezoso" y con escasa ilusin a la hora de iniciar actividades nuevas. Han notado que pasa horas
ensimismado, sin llegar a concluir la tarea que estuviera realizando. Se muestra incapaz de programar las actividades que ha de realizar
al da siguiente o las realiza a destiempo, de forma desorganizada. Parece mostrarse poco reactivo ante los problemas, disgustos o
alegras cotidianas, como si nada le importara. Se muestra olvidadizo, tiende a usar dietarios y notas cada vez con mayor asiduidad, sin
que esto haya impedido el haber olvidado citas importantes o tareas inexcusables en su negocio.

PREGUNTA
Considerando los antecedentes y al cuadro clnico, cual es el diagnostico mas probable?

RESPUESTA
a.- Demencia por parkinson.
b.- Demencia por prionopatia.
c.- Demencia alzheimer.
d.- Demencia de frontotemporal.

CASO CLINICO
Mujer de 56 aos, con 12 aos de escolaridad, diestra, duea de casa, separada, con una convivencia estable. Inicia un delirio
paranoide pero asociado a una gran pasividad (era necesario escogerle la ropa, abandon las labores domsticas). Se presento
descontrol urinario ocasional. En el examen mostraba una enferma vigil, prescindente, sin contacto visual, que no expresaba afectos y
colaboraba poco al examen (lo que impidi evaluar el campo visual y la sensibilidad). Exista gran hipokinesia pero sin rigidez ni
movimientos anormales; la metra era normal. Exista enganche digital pero no prehensin forzada ni succin. En el Minimental test de
Folstein (MM) obtuvo slo 10/30 puntos, fracasando en Orientacin, Atencin y clculo, Lectoescritura y Dibujo, con respeto relativo de
la memoria. Al preguntarle la fecha dijo "a ver... a ver... a ver"; lo mismo respondi al pedirle restas. Fue capaz de leer la orden escrita,
pero no la cumpli; en vez de una frase escribi su nombre, y el pedirle copiar el dibujo recibi el lpiz pero no rindi, limitndose a leer
una y otra vez fragmentos del test. En la prueba de Matrices Progresivas Coloreadas (MPC) repeta las instrucciones pero no renda y
musitaba en forma ininteligible; obtuvo 0/12 puntos. En Aprendizaje verbal (AV) deca "ya..." y se quedaba mirando al frente, con rostro
inexpresivo, sin repetir las palabras propuestas. En Memoria semntica (MS) evoc 2 nombres de animales en 60 segundos.

PREGUNTA
Considerando los antecedentes y al cuadro clnico, cual es el diagnostico mas probable?

RESPUESTA
a.- Demencia por parkinson.
b.- Demencia por prionopatia.
c.- Demencia alzheimer.
d.- Demencia de frontotemporal.

CASO CLINICO
Femenino de 47 aos acude por fallas de la memoria, errores en la denominacin y en actividades domsticas. Aparecieron cambios de
conducta, se puso:"confianzuda", caprichosa, golosa, floja. Se negaba a colaborar en actividades domsticas, e insista en forma
majadera en diversos temas. Meses despus fue evaluada con mayor detalle. En su casa era independiente para vestirse, alimentarse y
asearse, pero a veces se orinaba. No era capaz de retener informaciones nuevas, a veces no reconoca a sus familiares, y mostraba una
proximidad excesiva frente a los extraos. Tena prdida de libido y se mostraba inquieta e irritable. Se mostraba gil y el examen
neurolgico bsico era normal, no existan reflejos primitivos. Su actitud era inadecuada por animo subido, proximidad excesiva y gran
tendencia al payaseo. Por ejemplo, en una oportunidad interrumpi el examen diciendo "aprate abuelito por favor! porque tengo
que ir a hacerle la comida a mi marido". No mostraba ninguna preocupacin por sus rendimientos y con frecuencia no colaboraba; al
pedirle interpretar proverbios slo dijo "no s!". Al denominar cometa errores caprichosos, pero que sugeran una afasia amnsica.
Por ejemplo, ante el dibujo de un paraguas dijo riendo "es un sostn!" y otra vez "es una bicicleta!", pero ante el dibujo de un sof
dijo "es para sentarse". No existan defectos fonolgicos ni morfosintcticos. Al pedirle interpretar el proverbio "a quien madruga Dios
le ayuda" dijo "a mi marido? No, mi marido tiene negocio y yo trabajo con l, yo de cajera y l de vendedor...". Al dictarle "baj a la
tierra" lo repiti bien pero escribi -con buena letra- "monono nonino ninino". No exista apraxia constructiva (aunque a veces el dibujo
MANUAL DE TRABAJO DEL CURSO ENARM CMN SIGLO XXI
CURSO ENARM CMN SIGLO XXI TEL: 36246001 Pharmed Solutions Institute PGINA 169

era muy descuidado), oral ni ideo-motora; segn su familia se vesta bien, pero seleccionando mal su vestimenta. Fracas en clculo
oral y escrito; exista una agnosia digital visual y desorientacin derecha-izquierda -ni siquiera fue capaz de decir cual era su mano
derecha.

PREGUNTA
Considerando los antecedentes y al cuadro clnico, cual es el diagnostico mas probable?

RESPUESTA
a.- Demencia por parkinson.
b.- Demencia por prionopatia.
c.- Demencia alzheimer.
d.- Demencia de frontotemporal.

CEFALEAS Y DOLOR CRANEO-FACIAL
SALUD PUBLICA: Mas comn prevalencia en las mujeres, 18%, hombres, 6%. Edad de inicio promedio 19 aos. 46 % tienen
antecedentes familiares. El riesgo de que un nio desarrolle migraa 70% si ambos padres estn afectados, el 45%, cuando uno de los
padres afectados. Una condicin hereditaria dominante rara, migraa hemipljica familiar, debido a una mutacin en el cromosoma 19
que codifica para una subunidad del canal de calcio dependiente de voltaje.
La arteriopata cerebral autosmica dominante con infartos subcorticales y leucoencefalopata (CADASIL) puede presentarse con
migraa hemipljica y progreso a una encefalopata isqumica. PATOGENIA: La migraa es un trastorno neurovascular en un individuo
genticamente predispesto. Predisposicin es una inestabilidad dentro de la red trigminovascular originada en el tronco cerebral, en
particular, el mesencfalo dorsal y dorsolateral puente de Varolio. Proyecciones difusas del locus ceruleus y de la corteza cerebral
resultan en deterioro del flujo sanguneo cortical cerebral que causa la propagacin de la depresin asociada con auras migraosas.
DIAGNOSTICO: Clnico; La migraa es una cefalea episdica generalmente asociada con nuseas ( vmitos) y fotofobia. Puede estar
precedida por sntomas neurolgicos focales. El aura no necesariamente puede ser seguida por dolor de cabeza (anteriormente
conocido como equivalentes de migraa). 30 % puede tener otros dolores de cabeza que coexisten, por ejemplo tensin y dolores de
cabeza por uso excesivo analgsicos. Caractersticas de dolor de cabeza: Unilateral en 2/3 de los pacientes y bilateral en 1/3. Dolor que
se siente detrs o a lo largo del ngulo interno del ojo o regiones frontotemporal. Irradia a espalda, occipucio o el cuello. Sitio de dolor
de cabeza puede ser tanto ipsilateral o contralateral a la perturbacin neurolgica focal. Algunos pacientes pueden quejarse de dolor
del miembro ipsilateral del lado del dolor de cabeza. Carcter del dolor de cabeza es aburrido en el inicio y ms tarde palpitante (que
aumenta con cada pulsacin). Otros pacientes slo pueden describir un dolor de cabeza constante, o incluso un ligero dolor de cabeza
embotada. Empeora con el movimiento. Caractersticas Aura: Auras visuales incluyen: alucinaciones visuales, escotomas, y espectros de
fortificacin (zig-zag lneas se asemejan a una muralla cuando se ve desde arriba). Por lo general, blanco y el brillo se mueven a travs
del campo visual, dejando un rea de discapacidad escotomas centellantes. Otros fenmenos visuales incluyen destellos de luz
(fotopsia). Nota: la epilepsia del lbulo occipital provoca alucinaciones que son circulares o de formas geomtricas y colores. Auras
sensoriales son, por ejemplo parestesias generalmente positiva en lugar de adormecimiento, que se reparten en minutos u horas (5 %).
Otras auras: hemiparesia (de minutos u horas), disfasia, alucinaciones olfativas y gustativas, y la distorsin de las partes del cuerpo,
tales como inflamacin de la lengua. Disparadores de migraa: El estrs y la relajacin despus de estrs. Falta o el exceso de sueo
acostumbrados. Trauma (especialmente en nios).Estimulacin sensorial: reflejos, luces parpadeantes, los olores. Los hbitos
alimentarios y la alimentacin: falta de una comida (hipoglucemia). Los alimentos como el vino tinto, queso, chocolate. Aditivos
alimentarios: el glutamato monosdico. Ejercicio. El exceso de calor y la deshidratacin. Drogas: vasodilatadores como la nitroglicerina.
Variantes de migraa: Migraa vertebrobasilar; sntomas del tronco cerebral: diplopa, vrtigo, incoordinacin, ataxia y disartria se
producen en posteriores ataques de migraa. Tambin puede ser desmayos o prdida de consciencia debido a la participacin de la
formacin reticular del cerebro medio. En casos severos un estado de estupor o coma puede durar una semana (estupor migraa). La
mayora de los casos estn asociados con otros sntomas vertebrobasilar. Migraa oftalmopljica: Paresia extra-ocular el tercer nervio
es el ms afectado. Paresia puede durar das o semanas. Excluir una lesin compresiva tales como aneurisma de la arteria comunicante
posterior. Migraa retiniana: Resultado de la constriccin de las arteriolas retinianas, daa la visin en un ojo y se asocia con dolor de
cabeza por detrs del mismo ojo. Vrtigo benigno recurrente. Migraa que tiene anormalidades del sistema vestibular. Ataques de
vrtigo acompaadas por tinnitus, sordera y cefalea que puede responder con terapia antimigraosa. Criterios abreviados de Sociedad
Internacional de Cefaleas (IHS) para la migraa: Migraa sin aura;
a) Dolor de cabeza una duracin de 4 horas hasta 3 das. b) Nuseas/vmitos y/o sensibilidad a la luz y el ruido. c) Dos de los siguientes:
dolor unilateral dolor moderado o grave intensidad, agravacin por la actividad fsica sencilla, dolor palpitante. Migraa con aura; Al
menos 3 de los siguientes a) tronco cerebral focal reversible o disfuncin cortical. c) aura se desarrolla durante > 4 minutos o 2 auras
sucesivas. d) cada aura < 60 min. e) cefalea < 60 min siguiente aura. Los criterios sugeridos para la migraa crnica o transformada:
diario o casi a diario (> 15 das / mes) dolor de cabeza > 1 mes. Promedio de duracin de la cefalea > 4 horas / da (sin tratar). Al menos
uno de las siguientes: antecedentes de migraa de IHS. Historia de aumento de frecuencia de cefalea con la disminucin de la gravedad
de caractersticas migraosas durante al menos 3 meses. Ataques superpuestas de dolor de cabeza que cumpla con todos los criterios
de la IHS, excepto duracin. MIGRAA EN EL EMBARAZO: Consejos para no toma de drogas. Para evitar las nuseas y los vmitos
relacionados con el embarazo como resultado hipoglucemia y deshidratacin: comer pequeos bocados carbohidratos frecuentes,
ingesta adecuada de lquidos. Descanso adecuado. Minimizar la exposicin al frmaco, especialmente durante el primer trimestre. Si es
posible, deje los medicamentos profilcticos. Tratamiento agudo: paracetamol, en todos los trimestres y durante la lactancia, aspirina:
probablemente seguro pero la precaucin a corto plazo debido a riesgo de hemorragia postparto, hemorragia neonatal, y cierre
prematuro del ductus arterioso. Evitar durante la lactancia de riesgo del sndrome de Reye y el sangrado en la mortalidad infantil.
triptanos, no se recomienda,preparaciones de ergotamina contraindicado en el embarazo y la lactancia. MIGRAA MENSTRUAL: Gatillo
hormonal es la exposicin a los altos niveles de estrgenos seguido por una cada en los niveles. La liberacin de prostaglandinas
uterinas que ocurren alrededor de la menstruacin un mecanismo adicional. 60 % de las mujeres reportan un aumento en la frecuencia
MANUAL DE TRABAJO DEL CURSO ENARM CMN SIGLO XXI
CURSO ENARM CMN SIGLO XXI TEL: 36246001 Pharmed Solutions Institute PGINA 170

de la migraa en torno a la menstruacin. 14 % tiene exclusivamente migraa relacionada con la menstruacin. cido mefenmico 500
mg 3-4 veces al da o naproxeno 500 mg dos veces 1-2 das antes de dolor de cabeza y de la duracin del perodo (de -2 a 3 das de la
menstruacin). Ergotamina, 1 mg (o 1/2 supositorios) durante el periodo vulnerable. Naratriptn, 1 mg bd o frovatriptn 2,5 mg bd
durante 3-5 das. TRATAMIENTO: Analgesia simple con antiemticos: si las nuseas y los vmitos no son un sntoma importante como
la motilidad gstrica se ve afectada durante un ataque de migraa. La aspirina, 900-1200 mg (disuelto) + metoclopramida, 10 mg o
domperidona, 10 - 20mg. Los medicamentos alternativos incluyen paracetamol, 1000 mg y los AINE por ejemplo, ibuprofeno,
naproxeno, diclofenaco. Triptanos: Todas las drogas de esta clase (sumatriptn 50-100mgs, zolmitriptn 2.5mgs, naratriptn 2.5mg,
rizatriptn 5-10mgs) tienen una alta eficacia con un mximo de 70 % que tiene una respuesta dentro de 2 horas y el 40 % es libre de
dolor a las 2 horas. Zolmitriptan y rizatriptn disponibles como obleas no tienen una accin ms rpida. Sumatriptan disponible como
aerosol nasal e inyeccin. Las drogas funcionan mejor cuando se toma temprano, pero no durante el prdromo o de la fase de aura.
Recurrencia de la cefalea a las 12-24 horas se produce en el 30%. Consejo habitual es tomar una dosis adicional de los triptanos, quizs
combinarlo con un AINE. Si no hay respuesta intente otra triptanos. El uso excesivo puede dar lugar a dolores de cabeza de rebote de
10 %. Contraindicaciones: enfermedad arterial coronaria, enfermedad cerebrovascular, hipertensin no controlada, enfermedad
vascular perifrica, insuficiencia heptica significativa, y el embarazo. Efectos secundarios: molestias en el pecho o sensacin de
pesadez, la mandbula, el hombro y rigidez del cuello, parestesia, fatiga y mareos. Interacciones con otros medicamentos: evitar IMAO.
Propranolol concentraciones sricas de rizatriptn. Por lo tanto, utilizar dosis de 5 mg. El sndrome de la serotonina posible cuando
se utiliza con los ISRS. Preparaciones de ergotamina: Aun as un papel para la consideracin de tartrato de ergotamina en, aquellos
pacientes que no toleran a los agonistas 5 - HT. Solo o en combinacin con cafena 1-2 mg puede administrarse por va oral en el inicio o
puede ser utilizado en aquellos pacientes que tienen sntomas premonitorios tales como ansiedad, bostezo, o la fatiga. Tambin puede
ser administrado por inhalador o supositorio. La sobredosificacin causa nuseas, dolor de cabeza de rebote, y la vasoconstriccin
perifrica. La dosis mxima recomendada por semana es de 10 mg. Dihidroergotamina (DHE) usaron por va intravenosa en pacientes
con migraa intratable en dosis de 0,3 a 1,0 mg cada 8 horas hasta una dosis total de 10mg. PREVENCION: La profilaxis de la migraa;
Dolor de cabeza diario til para controlar la frecuencia y patrones, por ejemplo, relacin de los periodos, los fines de semana, el uso
excesivo de analgsicos y de triptanos. La profilaxis ineficaz si el uso excesivo de medicamentos. Evite los desencadenantes dietticos
slo en el 10 %. Considere la profilaxis si 2 ataques al mes o un ataque prolongado que afecta el estilo de vida. Medicamentos
profilcticos: betabloqueantes (propanolol, timolol, nadolol, atenolol). Los 5-HT2 antagonistas; pizotifen, ciproheptadina, metisergida,
amitriptilina especialmente si la migraa asociada con cefalea tensional. Valproato sdico. Topiramato. Gabapentina.

CASO CLINICO
Paciente 27 Aos, femenino, refiere haber comenzado con cefaleas hace 12 hs, de carcter pulstil, unilateral, que no cede con AINES,
refiere nauseas que atribuye a los medicamentos, esta cefalea es distinta de otros episodios sufridos previamente donde el dolor era
como una presin en zonas parietales y nuca.

PREGUNTA
Cual es el diagnostico mas adecuado en este paciente?

RESPUESTA
a.- Migraa.
b.- Cefalea tipo-tensin.
c.- Cefalea en racimos (cluster).
d.- Cefalae autonmica trigeminal.

CASO CLINICO
Paciente masculino, 62 aos, antecedentes de HTA, tabaquismo, EPOC, que es trado por la familia quienes refieren que el paciente
comenz hace das a consumir AINES por cefalea de reciente comienzo, posteriormente present conductas poco habituales y desde
ayer se encuentra desorientado.

PREGUNTA
Considerando los antecedentes y cuadro clnico, cual de los siguientes son factores es el menos crticos para el pronostico?

RESPUESTA
a.- Cefalea subaguda y/o cefalea progresiva que empeora en el tiempo (meses).
b.- Cefalea Nueva o distinta de la Habitual.
c.- Cualquier cefalea de maxima severidad en su inicio.
d.- Nueva cefalea en mayores de 40 aos.

CASO CLINICO
Paciente femenino, 42 aos que refiere cefalea sbita, intensa, explosiva, siente que le estalla la cabeza, refiere haber tenido hace
unos das atrs un episodio similar aunque de menor intensidad, que la obligo a guardar cama por unas horas.

PREGUNTA
Cual es el claso diagnostico mas probable?

RESPUESTA
a.- A trauma de crneo y/o cuello.
b.- A desrdenes vasculares craneales o cervicales.
MANUAL DE TRABAJO DEL CURSO ENARM CMN SIGLO XXI
CURSO ENARM CMN SIGLO XXI TEL: 36246001 Pharmed Solutions Institute PGINA 171

c.- A desorden intracraneal no vascular.
d.- A sustancias o a su supresin.

CASO CLINICO
Paciente de 43 aos que acude a urgencias de atencin primaria por cefalea. En el ltimo ao refiere cefalea basal diaria (Escala Visual
del dolor (EVA) 5-6/10) de caracterstica tensional, sin nuseas, fotofobia ni fonofobia, y aunque no limita por completo su actividad ya
que se ha acostumbrado a ella, tiene episodios de reagudizacin ms intensos, que la obliga a dejar sus labores, como en esta ocasin.
ltimamente estos episodios son ms frecuentes, y los relaciona con problemas de salud en un hijo. Refiere adems que toma
analgsicos prcticamente a diario y por rachas combinando paracetamol, ibuprofeno y lorazepam, desde hace casi ao y medio.
Adems est en tratamiento desde hace 2 aos con escitalopram.

PREGUNTA
Cual es el diagnostico mas probable?

RESPUESTA
a.- A trauma de crneo y/o cuello.
b.- A desrdenes vasculares craneales o cervicales.
c.- A desorden intracraneal no vascular.
d.- A sustancias o a su supresin.







MANUAL DE TRABAJO DEL CURSO ENARM CMN SIGLO XXI
CURSO ENARM CMN SIGLO XXI TEL: 36246001 Pharmed Solutions Institute PGINA 172

CLASIFICACIN ETIOLGICA DE SNDROMES COREICOS
1. COREAS HEREDITARIAS
Enfermedad de Huntington
Coreoacantocitosis
Corea hereditaria benigna
Corea paroxstica familiar
Enfermedades metablicas hereditarias (cistinuria,
homocistinuria, fenilcetonuria, galactosemia)
2. COREAS SECUNDARIAS
1. Frmacos: Neurolepticos, levodopa, agonistas
dopaminergicos, anticolinrgicos, amanrtadina,
antiepilpticos, estimulantes noradrenergicos,
corticoides, opiceos, metoclopramida,
antidepresivos, reserpina, estrgenos. 2. Txicos:
alcohol, monxido de carbono, manganeso,
mercurio. 3. Alteraciones metablicas,
hipopglucemia, hiperglucemia. 4. Embarazo. 5.
Infecciones y parasitosis de SNC: sarampin,
varicela, herpes, toxoplasmosis, VIH, sfilis,
tuberculosis. 6.Neoplasias
3. COREAS DEL DESARROLLO
Corea fisiolgica de la infancia
Discinesia bucolinguofacial del anciano
TRASTORNOS DEL MOVIMIENTO Y ATAXIAS:
CIENCIAS BASICAS: Los trastornos del movimiento constituyen una patologa relativamente frecuente en la prctica mdica diaria,
tanto neurolgica como en Atencin Primaria, y se pueden presentar como estados hipercinticos o hipocinticos. El prototipo de
trastorno hipocintico es la enfermedad de Parkinson y tambin la representa la patologa ms frecuente dentro de los trastornos del
movimiento. En general, los trastornos del movimiento tienen como sustrato anatmico comn los ncleos de los ganglios basales.
Adems, entre la lista de efectos secundarios relacionados con una gran variedad de
frmacos de uso comn entre la poblacin, est el desarrollo de trastornos del
movimiento. SALUD PUBLICA: La enfermedad de Parkinson se presenta con la misma
incidencia en todo el mundo y afecta por igual a ambos sexos. En el 5 al 10% de las
personas que sufren dicha patologa, sta aparece antes de los 40 aos de edad, en su
forma precoz. La edad media de inicio es aproximadamente de 65 aos. Sndrome de
piernas inquietas afecta al 5-10% de las personas entre 18 y 65 aos, y aumenta hasta
el 15-20% en los mayores de esa edad. Enfermedad de Huntington suele iniciarse de
forma insidiosa hacia la cuarta dcada de la vida, tiene una prevalencia de 7-10
casos/100.000 habitantes y afecta por igual a ambos sexos ENFERMEDAD DE
PARKINSON: Idioptica, afeccin degenerativa que combina una bradicinesia
asimtrica, tpica facies inexpresiva, hipocinesia y rigidez (rueda dentada), algunas
veces acompaadas de temblor en reposo y alteraciones posturales. El temblor,
inicialmente intermitente, suele comenzar en una mano para extenderse luego a las
extremidades restantes y, a veces, a los msculos de la cara y del cuello. El temblor
clsico de la enfermedad de Parkinson es regular y rtmico. Est presente en reposo y
disminuye o desaparece con el movimiento de la zona afectada. El temblor aumenta
con la ansiedad y cesa durante el sueo. Con frecuencia, el cuadro se acompaa de
alteraciones en el intelecto, aunque de forma leve, lo que incluye afectaciones
cognitivas, de percepcin, de memoria y de expresin. Un 15% puede verse afectado de
demencia, sobre todo en los casos en los que la enfermedad se ha iniciado tardamente. La afeccin del estado emocional, con
tendencia a la depresin, afecta hasta un 40% de los casos. Pueden presentar, en el 40% de los casos, sntomas de disfuncin
vegetativa, que incluyen estreimiento, a veces grave, hiperhidrosis, sofocaciones y alteraciones en la miccin (incontinencia,
polaquiuria y enuresis nocturna). La sialorrea es frecuente y se debe al defecto de la deglucin. Los pacientes pueden afectarse de
forma diferente debido a una combinacin de factores genticos y ambientales, entre los que podemos citar: virus, toxinas, el consumo
de aguas no potables, la vitamina E y el hbito tabquico (el tabaquismo no influye de forma negativa al igual que el consumo de
vitamina E), una vez ms, presente, como responsable de la puesta en marcha de un pro- ceso patolgico. Los familiares en primer
grado de consanguinidad, duplican el riesgo de desarrollar la enfermedad (un 17% de probabilidad, a lo largo de su vida),
comparndolos con la poblacin general. Los criterios de diagnstico clnico tienen una sensibilidad de 80% y una especificidad del 30%,
comparado con el patrn de oro del diagnstico de la autopsia. La patologa primaria consiste en la prdida progresiva de clulas de la
sustancia negra del tallo cerebral donde se produce el neurotransmisor dopamina. El tratamiento est obligado a reemplazar la prdida
de dopamina. Tambin se ven afectados varios sistemas de neurotransmisores catecolaminrgicos. Las pruebas, como la RMC, suelen
ser normales. Por el contrario, en las pruebas de neuroimagen funcional, como el PET y el SPECT cerebral, se puede observar una
reduccin asimtrica de la captacin estriatal de 18- fluorodopa o de DAT (transportador de dopamina), respectivamente, alteraciones
que no se observan en pacientes con temblor esencial o con parkinsonismo inducido por frmacos. El tratamiento mdico suele ser
eficaz y debe de continuarse durante el resto de la vida del ciudadano. Se fundamenta en las siguientes medidas: Anticolinrgicos (con
la intencin de corregir la hiperfuncin de las clulas colinrgicas del ncleo estriado y actan bloqueando los receptores muscarnicos
centrales)-Amantadina (posee propiedades anticolinrgicas y dopaminrgicas, ya que estimula la liberacin de dopamina en las
terminaciones nerviosas) -L-DOPA (frmaco ms eficaz, traspasa con facilidad barrear hematoenceflica)-Otros frmacos
dopaminrgicos-Medidas generales-Tratamiento quirrgico PARKINSONISMOS: producidos por factores etiolgicos conocidos, por
ejemplo medicamentosos, reciben el nombre de secundarios o sintomticos. Los secundarios incluyen aquellos producidos por
infecciones (encefalitis letrgica), intoxicaciones (manganeso, monxido de carbono, MPTP), administracin de frmacos
(neurolpticos, cinarizina), tumores, traumatismos, infartos cerebrales profundos y calcificaciones de los ganglios basales. Un tercer
grupo en el que los sntomas del parkinsonismo aparecen en el contexto de otra enfermedad neurolgica y se asocian a otros sntomas
de disfuncin neurolgica se denominan parkinsonismos plus, con sntomas extrapiramidales, entre los que se encuentra la parlisis
supranuclear progresiva, que se caracteriza por una oftalmoplejia supranuclear progresiva, asociada a rigidez distnica de los msculos
del cuello y del tronco superior, con alteracin de equilibrio y una demencia discreta de tipo frontal. Estos sntomas se inician en la
quinta o sexta dcadas de la vida y son de evolucin progresiva. La falta de equilibrio y las molestias visuales constituyen los sntomas
iniciales ms frecuentes. SNDROME DE PIERNAS INQUIETAS: Trastorno neurolgico del movimiento, caracterizado por la aparicin de
molestias inespecficas en la parte inferior de las piernas durante el reposo, cuando el paciente est sentado o tumbado, es
especialmente frecuente durante el inicio del reposo nocturno, cuando el paciente se acuesta. Consiste en una sensacin desagradable
y difcil de calificar, en forma de parestesias, disestesias, hormigueos, pinchazos, desasosiego o dolor que lleva al paciente a la
necesidad irresistible de mover las piernas (inquietud motriz), levantarse, pasear, darse masajes e incluso baos con agua fra, en un
intento de alivio pasajero. Hay movimientos peridicos en las extremidades, en el que el paciente realiza movimientos bruscos de las
piernas, repetitivos y estereotipados, cada 10-60 seg., a veces durante toda la noche, involuntariamente. Al impedir un descanso
nocturno adecuado puede manifestarse con cansancio y somnolencia diurna e insomnio. Los frmacos con actividad dopaminrgica y
en concreto los agonistas dopaminrgicos (pergolida, pramipexol y ropinirol), se consideran el tratamiento de eleccin. Ropinirol ha
demostrado en varios estudios que reduce los movimientos peridicos de las piernas, y mejora la eficiencia del sueo en relacin con el
perodo basal. COREAS: Significa baile o danza. Se utiliza para designar movimientos involuntarios, irregulares, de duracin breve y
de baja amplitud, que no son predecibles y no tienen una finalidad aparente; fluyen de una parte del cuerpo a otra sin una secuencia
definida. Suelen localizarse en las partes distales de los miembros, generalmente en las manos o en la cara, y suelen asociarse a
MANUAL DE TRABAJO DEL CURSO ENARM CMN SIGLO XXI
CURSO ENARM CMN SIGLO XXI TEL: 36246001 Pharmed Solutions Institute PGINA 173

hipotona o a impersistencia motora: dificultad para mantener una contraccin muscular sostenida. En las primeras fases, los
movimientos coreicos pueden aparecer al final de la ejecucin de movimientos normales, pero en fases ms avanzadas se hacen muy
frecuentes, dispersas y amplias, por lo que terminan siendo muy incapacitantes. Los sndromes coreicos suelen relacionarse con
lesiones o alteraciones funcionales del neoestriado (caudado y putamen). Su fisiopatologa, desde el punto de vista neuroqumico,
responde a una situacin de hiperactividad dopaminrgica. Esto viene apoyado por la eficacia teraputica de los frmacos
antidopaminrgicos y por la capacidad de agravar o incluso inducir estos trastornos, que poseen los agonistas de la dopamina.
ENFERMEDAD DE HUNTINGTON: Es la forma de corea hereditaria ms frecuente. Enfermedad neurodegenerativa que se transmite de
forma autosmica dominante con penetrancia completa, precisando un solo gen defectuoso en uno de los progenitores. Los hijos de
una persona afectada, tienen el 50% de probabilidad de padecer la enfermedad. Hay atrofia cortical relacionada con el grado de
evolucin de la enfermedad, y tambin atrofia estriatal, fundamentalmente a nivel del ncleo caudado. Las neuronas afectadas por el
proceso degenerativo son las de mediano tamao. La disfuncin estriatal, contraria a la que caracteriza a la enfermedad de Parkinson,
es la responsable de que aparezcan las hipercinesias; y la degeneracin neocortical y del ncleo caudado es la responsable de que
aparezca demencia. El cuadro clnico incluye sntomas motores, trastornos psiquitricos y alteraciones cognitivas: Sntomas motores:
movimientos coreicos, que afectan inicialmente a la parte distal de las extremidades y posteriormente se van extendiendo a la
musculatura craneal, farngea y larngea. Las anormalidades en la motilidad ocular, con dificultad para seguir con la mirada un objeto
mvil, posturas distnicas axiales y de las extremidades, as como disfagia y disartria coreica por movimientos coreoatetsicos de los
labios, la lengua, el paladar y el diafragma. Trastornos psiquitricos: en ocasiones son la primera manifestacin de la enfermedad,
cambios en la personalidad, depresin, apata, agitacin, mana, alucinaciones, insomnio. En casos graves es frecuente el suicidio; ms
del 30% lo intentan durante el desarrollo de la enfermedad. Dficit de la memoria, la atencin, la concentracin y el aprendizaje;
pueden llegar a una demencia con afectacin de las funciones frontales: trastorno de la atencin y de las funciones ejecutivas, sin
afasia, apraxia o agnosia. El diagnstico de sospecha se realiza mediante la clnica, la historia familiar positiva y la neuroimagen. La
confirmacin por gentica molecular, que tiene una sensibilidad diagnstica del 98% y puede utilizarse incluso en fase presintomtica.
No existe un tratamiento especfico. Se estn realizando ensayos con GABA, baclofen, ACTH y antagonistas selectivos de los receptores
D2. COREA DE SYDENHAM: Es un cuadro de corea aguda, que aparece generalmente en la infancia en relacin con una infeccin por el
estreptococo betahemoltico. Est considerada como uno de los criterios mayores de la fiebre reumtica. Su fisiopatologa parece estar
relacionada con un proceso autoinmune, en el que los anticuerpos antibacterianos provocan reacciones cruzadas con los antgenos
tisulares en el cuerpo estriado. Los sntomas pueden aparecer hasta seis meses despus de una infeccin por el estreptococo, que a
veces no se consigue documentar y que hace el diagnstico ms difcil.

CASO CLINICO
Mujer de 58 aos que presenta un cuadro de torpeza generalizada, lentitud y temblor postural en miembros superiores de dos aos de
evolucin. Dos aos presenta de forma fluctuante: conducta impulsiva, cambios de carcter con violencia y agresividad, ataques de
llanto repentino y comportamiento infantil. Presenta asimismo dificultades en concentracin, poca capacidad de comprensin y
razonamiento y dificultad para hablar y expresarse. En ocasiones confunde ideas simples, tambin nombres de objetos y personas.
Comienza a perder las rutinas de su vida diaria, mantiene poca relacin con el entorno y muestra falta de cuidado personal. En la
exploracin neurolgica se objetiva: clara alteracin en prueba de secuencias alternantes y en patrones motores alternantes
unilaterales (triple maniobra de Luria), lenguaje muy pobre con carencia de sustantivos y confusiones frecuentes, desorientacin parcial
en tiempo y espacio, movimientos extraoculares pobres (aunque sin limitacin) junto rigidez y bradicinesia generalizadas.

PREGUNTA
Cual es el diagnostico mas probable?

RESPUESTA
a.- Enfermedad de Huntington
b.- Enfermedad de Sydenham.
c.- Enfermedad de Parkinson.
d.- Enfermedad de Piernas inquietas.

CASO CLINICO
Paciente de 76 aos que acude a la consulta diciendo que sufre de temblor en las manos, sobre todo la derecha, desde hace meses. Lo
presenta de forma constante, No refiere otra sintomatologa. El paciente presenta temblor en ambas manos, sobre todo la derecha, de
reposo, que mejora con la realizacin de movimientos voluntarios, inexpresividad facial, dificultad para levantarse del asiento y lentitud
al caminar. La movilizacin de las extremidades muestra aumento del tono en las piernas y rigidez en rueda dentada en los brazos. Se
trata, por tanto, de un paciente que consulta por temblor en las manos.

PREGUNTA
Considerando el diagnostico cual es el tratamiento mas apropiado?

RESPUESTA
a.- Pergolida.
b.- Penicilina.
c.- Baclofen.
d.- L-dopa.



MANUAL DE TRABAJO DEL CURSO ENARM CMN SIGLO XXI
CURSO ENARM CMN SIGLO XXI TEL: 36246001 Pharmed Solutions Institute PGINA 174

CASO CLINICO
Se trata de un paciente masculino de 54 aos de edad, el cual es referido por presentar intentossuicidas reiterativos (3 intentos) en un
lapso de 2 meses. El ltimo de ellos realizado en las ltimas horas a su envo (03/05/00), al arrojarse a un pozo, slo provocndose
ligerascontusiones en diversas partes del cuerpo. El padecimiento inicia aproximadamente hace 4 aos. De forma paulatina comienza a
presentar miedo a la obscuridad, que cada vez aumentaba en intensidad, desde hace dos aos se da la aparicin de movimientos
involuntarios en la frente (gesticulaciones), y sin motivos aparentes comienza con decaimiento del estado del nimo, aislacionismo,
mutismo por das, para luego regresar a su estado normal, present adems irritabilidad fcil, as como ideas de desesperanza,
refiriendo que su vida no tena sentido, transcurre aproximadamente un ao y seis meses, con poca afectacin en su vida laboral, social
y de familia, durante este tiempo comienza a presentar algunos movimientos involuntarios en manos y en cuello de tipo espasmdicos,
hacindose ms evidentes los movimientos en cara, dichos movimientos se presentan de forma peridica y repetitiva.

PREGUNTA
Cual es el diagnostico mas probable?

RESPUESTA
a.- Enfermedad de Huntington
b.- Enfermedad de Sydenham.
c.- Enfermedad de Parkinson.
d.- Enfermedad de Piernas inquietas.

ENFERMEDADES NEUROMUSCULARES (ENM):
CIENCIAS BASICAS: Las enfermedades neuromusculares son enfermedades de carcter gentico y generalmente hereditario que
afectan a la musculatura y al sistema nervioso. Tambin se conocen con el nombre de miopatas. Su aparicin puede producirse tanto
en el nacimiento como en otras etapas de la vida. Las ENM son trastornos de la unidad motora, cuyos principales sntomas son la
debilidad muscular, la fatiga, los calambres, el dolor, problemas articulares y la rigidez. Su distribucin suele ser simtrica, a diferencia
de lo que ocurre en las enfermedades que afectan al sistema nervioso central (SNC). La debilidad de los msculos respiratorios es la
causa bsica que conduce al fracaso de la bomba ventilatoria, y que se traduce en hipoxemia e hipercapnia. Sin embargo, el
compromiso del sistema respiratorio no es igual en todas las entidades sino que est determinado por el grado de afeccin muscular y
por la progresin de la enfermedad. Existen alrededor de 150 tipos de ENM: 1. Distrofias musculares: afectan predominantemente al
msculo estriado y son debidas a un defecto alguno de las protenas que forman parte de la fibra muscular, ya sean estructurales o
enzimticas (ejemplos son la distrofina calpana, merosina y emerina, entre otras)2. Miopatas distales. 3. Miopatas congnitas. 4.
Distrofia miotnica de Steinert. 5. Miotonas congnitas. 6. Parlisis peridicas familiares. 7. Enfermedades musculares inflamatorias. 8.
Miositis osificante progresiva. 9. Miopatas metablicas. 10. Enfermedades de la unin neuromuscular. 11. Amiotrofias espinales. 12.
Neuropatas hereditarias sensitivo-motoras (enfermedades de Charcot-Marie-tooth). ESCLEROSIS LATERAL AMIOTRFICA (ELA): Tiene
una incidencia anual de 1-2 casos/100.000 habitantes y su sustrato patolgico es la degeneracin de las neuronas motoras de la mdula
espinal, el tronco cerebral y el crtex motor. Clnicamente se caracteriza por espasticidad e hiperreflexia al inicio de la enfermedad,
pero a medida que progresa se establecen otras sntomas, como amiotrofia asimtrica, debilidad muscular, fasciculaciones y sndrome
bulbar. La debilidad de los msculos respiratorios, fundamentalmente de los intercostales y del diafragma, es la causa de la
hipoventilacin, y los sntomas respiratorios aparecen cuando la enfermedad est muy evolucionada, a pesar de que numerosos
estudios han comprobado que puede haber alteracin de la funcin ventilatoria incluso cuando la debilidad muscular perifrica es
ligera. A pesar de ello, lo ms frecuente es que la insuficiencia respiratoria se presente en casos de ELA claramente establecida como
consecuencia de la progresin natural de la enfermedad o de forma aguda, precipitada por una infeccin de tracto respiratorio. De
forma ocasional, la insuficiencia respiratoria puede ser la primera manifestacin de la ELA; en estos casos la lesin afecta
fundamentalmente a las motoneuronas del nervio frnico localizadas en la mdula cervical, y la afeccin bulbar implicar un grave
compromiso de la va area superior. ESCLEROSIS MLTIPLE (EM): Es una enfermedad desmielinizante del SNC, y se considera una de
las principales causas de discapacidad neurolgica en adultos jvenes. Como posibles etiologas se han descubierto datos que hacen
referencia a factores genticos y ambientales. Los sntomas comunes de la enfermedad son debilidad muscular, espasticidad,
incoordinacin motora y prdida de agudeza visual. Habitualmente esta enfermedad tiene dos formas de presentacin clnica: en forma
de brotes, en los que puede haber una remisin completa o parcial de los sntomas, o como una enfer medad lentamente progresiva44.
En este sentido, el compromiso del sistema respiratorio en la EM45 est en relacin con la forma de presentacin clnica de la
enfermedad, con el grado de debilidad muscular y con las estructuras del SNC que se ven afectadas por la desmielinizacin. En la mayor
serie de pacientes con EM y disfuncin respiratoria, las complicaciones respiratorias aparecieron a los 9,5 aos de las manifestaciones
neurolgicas y se atribuyeron a debilidad muscular, disfuncin bulbar, trastornos del control respiratorio, hiperventilacin paroxstica y
apneas obstructivas. En las lesiones agudas de la mdula espinal, las complicaciones respiratorias dependen de la extensin y la
localizacin de la lesin y se deben a la interrupcin de la inervacin. Se consideran lesiones altas las que afectan a C1 y C2, y bajas las
que se producen entre C3 y C8. Las motoneuronas que inervan al diafragma se originan entre C3 y C5, por lo que las lesiones medula-
res situadas por encima de C3 implican una parlisis total de la musculatura respiratoria, mientras que en las lesiones entre C3 y C5 la
parlisis muscular es parcial. MIASTENIA GRAVIS (MG): Es una enfermedad autoinmune. Se presentan anticuerpos contra el receptor
de acetilcolina (AchR) que interrumpen la funcin de la acetilcolina en la unin neuromuscular presentando en debilidad muscular. Los
Msculos bulbar, facial y de las extremidades proximales son los ms comnmente afectados y justifican los sntomas de la debilidad
general, ptosis y diplopa. La debilidad en los msculos de la respiracin puede llevar a la falla respiratoria la cual es llamada crisis
miastnica. Las complicaciones asociadas a falla respiratoria son la causa de muerte en paciente con Miastenia Gravis. La incidencia es
de menos de 1 caso por 100 000. Es ms comn en mujeres durante la segunda y tercera dcada de la vida. Sin embargo en la Sptima
y Octava dcada de la vida es ms comn en hombres. Los Afroamericanos tambin tienen una incidencia alta. La contraccin muscular
ocurre cuando la acetilcolina es liberada por una fibra nerviosa y se une al AchR de una fibra muscular. En la MG, la fibra nerviosa es
normal, sin embargo, el nmero y funcin de los AchR nicotnicos del msculo esqueltico estn disminuidos. Los sntomas aparecen
MANUAL DE TRABAJO DEL CURSO ENARM CMN SIGLO XXI
CURSO ENARM CMN SIGLO XXI TEL: 36246001 Pharmed Solutions Institute PGINA 175

cuando el nmero de AchR esta disminuidos por debajo del 30% de lo normal. Los msculos cardiaco y liso no se afectan por que ellos
tiene diferente antigenicidad que el msculo esqueltico. La hiperplasia del timo o el timoma se ha visto en el 75% de los pacientes con
MG. Por lo tanto, se sospecha que el timo es el sitio de produccin de anticuerpos, pero el estmulo que inicia el proceso autoinmune
es desconocido. La morbilidad por la debilidad muscular intermitente incluye la neumona por aspiracin y cadas. Los sntomas
empeoran en el da progresivamente con el uso repetitivo de grupos musculares como leer o masticar. Los nervios craneales son los
ms comnmente afectados, resultando en ptosis, diplopa, debilidad facial, disfagia y disartria. La respuesta pupilar a la luz permanece
intacta. La debilidad muscular de las extremidades proximales empeora con el movimiento y mejora con el descanso. Los reflejos
tendinosos profundos pueden estar disminuidos pero nunca ausentes. No hay dficit en la funcin cerebelar y sensorial. La crisis
miastnica es el grado ms severo de MG por que la debilidad extrema de los msculos respiratorios resulta en falla respiratoria
requiriendo soporte ventilatorio. Otras enfermedades que se pueden presentar con sntomas similares son el botulismo, hipotiroidismo
y lesiones de la masa intracraneal. La prueba de diagnstico para MG incluyen la prueba de edrofonium, electromielograma con
estimulacin repetitiva del nervio y prueba serolgica para anticuerpos anti AchR. El cloruro de edrofonium es un inhibidor de la
colinesterasa de corta accin que incrementa la cantidad de acetilcolina en la unin neuromuscular. Este incremento vence el bloqueo
de los receptores y la ptosis, oftalmopleja y la debilidad muscular se corrige en 30 segundos pero regresa a lo basal en 1 a 2 minutos. La
prueba de anticuerpos contra AchR es la prueba ms especfica para MG. El aspecto ms importante para los mdicos del
departamento de urgencias es que en todos pacientes gravemente enfermos, la prioridad es establecer y mantener una ruta area y
asegurar respiracin adecuada. La Intubacin endotraqueal por la induccin rpida de la sucesin y intubacin (IRS) puede ser
necesario es antes de ser capaz de diferenciar miastenia gravis contra una crisis de colinergica. El preferido es la Succinilcolina. Los
pacientes con MG AchRs resistentes al succinilcolina y dosis ms altas (2 mg/kg) debe ser usado para inducir la parlisis. Los agentes de
no despolarizantes como (rocuronium o vecuronium) son los agentes paraliticos preferidos. Estos agentes no causan la parlisis
prolongada en una dosis de 50% la dosis recomendada. La dosis en adultos de edrofonium es 2 mg IV lento. Sin presentar ninguna
respuesta ni ningn efectos adversos de tipo colinergico, administrando 8 mg IV lento para una dosis total de 10 mg, las
contraindicaciones del edrofonium es obstruccin gastrointestinal u obstruccin del trecho urinario. DISTROFIA MUSCULAR
PROGRESIVA, O ENFERMEDAD DE DUCHENNE: Es una enfermedad de herencia recesiva ligada al cromosoma X causada por
mutaciones en el gen de una protena del citosqueleto: la distrofina. Es una miopata proximal que comienza en la infancia y en la que,
inicialmente, se afectan los msculos de las extremidades inferiores (cintura pelviana), de modo que el paciente refiere dificultad para
correr o saltar y, a medida que progresa, afecta a los msculos de las extremidades superiores y del tronco; as, los enfermos a los 12
aos de edad son dependientes por completo. Tambin es caracterstico de la enfermedad la miocardiopata primaria, que cursa de
forma asintomtica o con trastornos del ritmo cardaco. La mayora de estos enfermos presentan un cierto dficit intelectual, y los
pacientes suelen fallecer alrededor de los 20 aos por complicaciones respiratorias o cardacas. Muchas enfermedades
neuromusculares son genticas, lo que significa que hay una tendencia familiar o existe una mutacin en los genes. Algunas veces,
pueden ser provocadas por un trastorno del sistema inmunolgico. La mayora de ellas no tiene cura. El objetivo del tratamiento es
mejorar los sntomas, aumentar la movilidad y el lapso de vida.

CASO CLINICO
Se trata de masculino de 65 aos de edad el cual acude debido a que desde hace cuatro meses presenta debilidad distal asimtrica,
adems refiere contracturas musculares y fasciculaciones que se presentan con los movimientos voluntarios, a la exploracin fsica se
observa debilidad extensora de las manos as como dificultad para mover la lengua y la cara, los reflejos de estiramiento muscular se
observan incrementados, la exploracin de la sensibilidad no se observan alteraciones, al examen mental se encuentra pensamiento y
lenguaje con curso y contenido adecuado, humor y afecto conservados, resto de funciones mentales superiores estn intactas.

PREGUNTA
Cul es el diagnostico clnico ms probable.

RESPUESTA
a.- Esclerosis lateral amniotrofica.
b.- Esclerosis multiple.
c.- Mielitis transversa.
d.- Sndrome de guillain-barre.

CASO CLINICO
Masculino de 23 aos de edad el cual acude a consulta debido a que desde hace un mes presenta debilidad generalizada, dificultad
para caminar y alzar objetos, estas manifestaciones predominan en la maana, durante el interrogatorio refiere dormir bien, no
consumo de drogas alcohol, no cuenta con enfermedades neurodegenerativas en la familia, actualmente se encuentra terminando la
licenciatura pero le preocupa estos sntomas, a la exploracin fsica usted observa ptosis bilateral, y disminucin de fuerza generalizada.

PREGUNTA
Cul es el diagnostico ms probable.

RESPUESTA
a.- Esclerosis multiple.
b.- Esclerosis lateral amniotrofica.
c.- Sindrome de Guillain-Barre.
d.- Miastenia gravis.


MANUAL DE TRABAJO DEL CURSO ENARM CMN SIGLO XXI
CURSO ENARM CMN SIGLO XXI TEL: 36246001 Pharmed Solutions Institute PGINA 176

TRASTORNOS TOXICO-METABOLICOS Y CARENCIALES DEL SISTEMA NERVISOSO:
TRASTORNOS TOXICO-METABOLICOS: El sistema nervioso puede verse daado cuando el fallo de un rgano permite el acmulo de
sustancias txicas que en circunstancias normales son eliminadas del organismo. Tambin como consecuencia de drogas o toxinas
exgenas o endgenas, disfuncin de mecanismos homeostticos o por el dficit de sustratos indispensables. Las principales
herramientas de trabajo sern la historia clnica y la exploracin fsica. El curso de los sntomas (monofsico, progresivo o con recaidas),
la presencia de sntomas sistmicos, enfermedades previas o coexistentes as como el uso de frmacos, hbitos dietticos o txicos y la
exposicin a txicos sern de gran importancia en la identificacin del sndrome y su causa. Hallazgos de laboratorio, estudios ms
especficos (autoanticuerpos, estudio del LCR) o pruebas de imagen entre otros nos permitirn completar la aproximacin etiolgica.
Las encefalopatias se desarrollan habitualmente de manera insidiosa y la norma en todas ellas es la alteracin del estado mental, en
ocasiones de forma tan sutil que puede no detectarse con exploraciones rutinarias. La lesin del sistema reticular y la corteza cerebral
inducirn diferentes grados de alteracin en la orientacin, memoria, percepcin, capacidad de concentracin, juicio o planificacin y
ejecucin de tareas. Los trastornos metablicos sistmicos y la exposicin a toxinas provocarn ms frecuentemente degeneracin
axonal. La musculatura ocular ser la ms frecuentemente afectada as como los msculos flexores del cuello y de cinturas. Las
enfermedades de los msculos se presentarn con debilidad, dolor y fatiga. Dficits nutricionales y txicos exgenos sern los
responsables ms frecuentes y el mecanismo patognico en ocasiones est bien definido. ENCEFALOPATIA HIPXICO-ISQUMICA: El
cerebro es un rgano con alto requerimiento metablico y por ello muy susceptible a dao por deprivacin del flujo sanguneo. Las
reservas de glucosa, glucgeno, ATP y posfocreatina se deplecionan a los 10-12 minutos de la isquemia. Despus de 15 minutos de
isquemia global con parada cardiaca ms del 95% del tejido cerebral estar daado. Ms de la mitad de los supervivientes a una parada
cardiaca presentan algn grado de dao cerebral permanente. La patogenia del dao cerebral depender de la etiologa del mismo y el
pronstico del mecanismo implicado. Cualquier mecanismo conduce finalmente a una necrosis y apoptosis neuronal irreversible.
Cambios bioqumicos diversos, dao mitocondrial precoz, alteracin de citoesqueleto neuronal y la activacin de receptores de
Glutamato son aspectos funcionales subyacentes al dao celular tras un insulto hipoxico-isqumico cerebral. El dao neuronal puede
ocurrir durante el periodo inicial de la isquemia (como consecuencia de los cambios bioqumicos y funcionales), en el periodo de
reperfusin (como consecuencia de la formacin de radicales libres y dao txico continuo), por deterioro del flujo sanguneo (al
reanudarse la circulacin espontnea y como consecuencia de una microcirculacin alterada) y como dao diferido (secundario a
desmielinizacin). ENCEFALOPATA HEPTICA: La enfermedad heptica crnica o aguda puede acompaarse de sntomas
neuropsiquitricos (y neuromusculares) conformando distintos grados de encefalopata. Es aceptado el papel del amonio como factor
clave dentro de una cascada de sucesos entre los que aparece alteracin de neurotransmisores, excesiva produccin de glutamina o
estrs oxidativo. El edema de astrocitos resultante se postula como la base fisiopatologa subyacente en la forma de HE aguda y
crnica. Factores como la hiponatremia o sedantes como las benzodiacepinas influyen en la aparicin de encefalopata heptica incluso
con cifras de amonio en sangre normales. El aumento de osmolaridad intracelular causado por hiperamonemia produce reduccin de
los picos de colina y mioinositol y un aumento en el pico de glutamina y glutamato. Las anomalas metablicas se correlacionan con la
severidad clnica y son reversibles tras tratamiento. Las opciones teraputicas dependen del estadio e instauracin de la encefalopata
heptica. El objetivo ser reducir la produccin de amonio, aumentar su fijacin y excrecin, controlar los sntomas neurolgicos
presentes y modificar el shunt porto-sistmico. Bromocriptina o flumazenilo cayeron en desuso frente a lactulolosa o rifaximina. En
ocasiones el trasplante de hgado ser la opcin teraputica a considerar. ENCEFALOPATIA URMICA: Los signos de encefalopata en
pacientes con enfermedad renal no siempre obedecen al fracaso renal progresivo y pueden sumarse al dao neurolgico secundario a
tratamientos inmunosupresores, dilisis o trasplante renal. La fisiopatologa permanece por determinar. Trastornos del equilibrio acido-
base, alteracin en las concentraciones de agua y electrolitos, metabolismo anmalo del calcio y glndulas paratiroides o cambios en
las concentraciones plasmticas de distintos neurotransmisores estn presentes y determinan los sntomas neurolgicos detectables.El
tratamiento principal de la encefalopata urmica es la dilisis. As mismo existen dos sndromes neurolgicos relacionados con sta, el
sndrome del desequilibrio de la dilisis y la demencia de la dilisis. ENCEFALOPATIA HIPOGLUCEMICA: Glucemia menor de 30mg/dl, o
periodos prolongados de hipoglucemia, para que se produzca dao irreversible. La intensidad del metabolismo cerebral explica la
vulnerabilidad particular del cerebro frente a desrdenes metablicos. Ante la posibilidad de una disminucin importante de glucosa en
el SNC, la actividad de los centros cerebrales superiores disminuye para reducir las necesidades de energa. La hipoglucemia provoca
respuestas nerviosas y hormonales para aumentar la produccin de glucosa en el hgado, y reducir su uso en rganos no nerviosos. Se
caracteriza por sntomas de estimulacin del sistema nervioso simptico o de SNC, provocados por una concentracin plasmtica de
glucosa anormalmente baja. Los sndromes hipoglucmicos pueden producirse por frmacos o sustancias como insulina, alcohol o
sulfonilureas, y con menos frecuencia por salicilatos, propanolol, pentamidina, disopiramida, hipoglicina A. Clnica: confusin,
convulsiones, estupor, coma. ENCEFALOPATIA HIPERCAPNICA: Debido a enfermedades respiratorias crnicas retenedoras de CO2,
como fibrosis pulmonar, enfisema. Clnica: Presenta los sntomas de hipertensin craneal, cefalea holocraneal o frontal intensa, edema
de papila, somnolencia hasta coma. ENCEFALOPATA SPTICA: Disfuncin cerebral difusa o multifocal asociada con una infeccin
sistmica, sin evidencias de infeccin intracraneal, y que no puede ser atribuida a otros factores tales como efectos farmacolgicos o
disturbios metablicos. La encefalopata de la sepsis puede ser clasificada como encefalopata sptica o precoz, que se presenta antes
de que se produzca la disfuncin orgnica mltiple, o encefalopata tarda, que es acompaada por fallo orgnico mltiple, hipotensin
y otros fenmenos sistmicos. Probablemente se origine por la accin de mediadores inflamatorios en el cerebro o por una respuesta
citotxica de las clulas cerebrales a estos mediadores. Los efectos de la sepsis sobre el cerebro son detectables en cerebros
previamente sanos, pero son amplificados en casos con lesin cerebral concomitante, como luego de la injuria traumtica o la
hemorragia subaracnoidea. La causa ms comn de encefalopata en los pacientes con enfermedades mdicas crticas, habiendo sido
descrita en el 9 al 71% de todos los pacientes crticos que sufren sepsis. TRASTORNOS CARENCIALES. Ante cualquier trastorno del SN
adquirido, siempre debe tenerse en consideracin una carencia vitamnica ya que el tratamiento sustitutivo administrado al inicio del
cuadro puede hacer remitir la sintomatologa, mientras que la administracin tarda, cuando las lesiones anatomopatologicas ya estn
establecidas, no evitara que el paciente quede con secuelas. Los sndromes ms importantes y frecuentes son debidos a la carencia de
las vitaminas de grupo B y en general son secunmdarios a estados carencialesde malnutricin, alcoholismo o patologa del sistema
digestivo que causen malabsorcin. ENCEFALOPATIA DE WERNICKE-KORSAKOFF: Es un sndrome neuropsiquitrico agudo que aparece
como consecuencia del dficit de tiamina (B1) y se asocia con una morbi-mortalidad significativa. Las causas de deplecin de tiamina y
MANUAL DE TRABAJO DEL CURSO ENARM CMN SIGLO XXI
CURSO ENARM CMN SIGLO XXI TEL: 36246001 Pharmed Solutions Institute PGINA 177

los mecanismos responsables son mltiples. Las lesiones se localizan de forma simtrica a nivel periacueductal, sustancia gris
subependimaria del III ventrculo, vermis cerebeloso, tubrculos mamilares, ncleos hipotalmicos, porcin medial y dorsal del tlamo
o ncleos vestibulares. El cuadro tpico clnico de confusin mental con disminucin del nivel de conciencia. Puede o no haber parlisis
oculomotora (VI) o nistagmo, oftalmopleja y trastorno de la marcha aparece tan slo en el 16% de los pacientes. Puede haber crisis
comiciales y si no se trata al paciente aparece miosis arreactiva a la luz, sntomas vegetativos, piramidalismo, coma y muerte. Una
correcta aproximacin facilita el diagnstico, la prevencin en paciente con factores o situacin clnica predisponentes y mejora el
pronstico con el adecuado soporte de tiamina parenteral 500mgs en 100ml de SF a pasar en 30 min 3 veces al dia. Los pacientes
pueden superar la fase aguda con secuelas como el sndrome amnsico de Korsakov. DEFICIENCIA DE VITAMINA B12:
Cianocobalamina, La metilcobalamina actua como factor indispensable para la enzima sintetasa (transformacin de homocosteina a
metionina), ruta indispensable para el metabolismo del cido flico y sntesis del ADN. La metinina es esencial para la sntesis de colina
y fosfolpidos. La deficiencia se manifiesta con anemia megaloblastica, glositis, atrofia de mucosa intestinal, ocasionalmente vaginitis.
Degeneracin combinada subaguda de la medula (alteracin de cordones posteriores y laterales, con lo que presenta trastornos
sensitivos, parestesias que ascienden desde los pies hasta el tronco, a la que se aade una ataxia), alteraciones cognitivas/demencia
(irritabilidad, psicosis), polineuropatia mixta sensitivo-motora. En casos de malabsorcin o anemia perniciosa: 1000mg por va IM diaria
durante 2 semanas. Luego seguir con 1000mg IM al mes. DEFICIENCIA DE ACIDO NICOTINICO: Niacina esencial para sntesis de NAD y
NADP. La deficiencia de niacina o su precursor el triptfano, provoca la pelagra. En el mundo occidental la pelagra es muy rara y se
observa en alcohlicos, en el sndrome carcinoide y enfermedad de Harnup. Clnica: triada clsica es dermatitis, diarrea y demencia,
puede haber cuadro neurolgico sin manifestaciones cutneas. De inicio es una encefalopata inespecfica (sndrome confusional,
alteracin del nivel de conciencia, ataxia y mioclonias) que evoluciona a demencia. Tratamiento, administrar entre 100-250mg VO tres
veces al da durante 5 das. La administracin de 325 mg de aspirina media hora antes previene el flushing facial. DEFICIT DE
VITAMINA E: Debida a defecto de malabsorcin intestinal, atresia de vas biliares. Se manifiesta como una degeneracin
espinocerebelosa (ataxia, sndrome piramidal), ms oftalmopleja externa, retinitis pigmentaria, polineuropata. Tratamiento
administracin de vitamina E hidrosoluble por VO o IV.

CASO CLINICO
Paciente masculino, 42 aos de edad, nacionalidad italiana, con antecedentes de tabaquismo, alcohol espordico, hernioplasta
umbilical; biopsia heptica cirrosis heptica; familiar de primer grado con hepatitis de causa no filiada. Motivo de ingreso: cuadro agudo
de confusin mas euforia con posterior deterioro del sensorio de 24 hs de evolucin ms catarsis negativa en la ltima semana. Al
examen fsico se constata somnolencia alternada por episodios de excitacin, confusin tmporo-espacial; flapping +; spiders en cara
anterior de tronco, ascitis grado II; edemas en miembros inferiores. Laboratorio de ingreso: Bilirrubina Total: 3.27/B.Directa:
1.87/B.Indirecta: 1.4 / FAL: 741/ GGT: 504/GOT : 180/GPT : 148. TP: 44% Plaquetas: 65.000/mm/ Factor V: 34%. Se solicita estudios
para determinar etiologa de la misma; Serologas virales (VHC; VHB; HIV; VHA negativos)FAN; ASMA; AMA negativos;Ferremia:169
microg/dL(38-158); Ferritina184 nanogr/ml (8-110);Saturacin de Transferrina 98,8%(20-50); Transferrina 171 microg/dL(180-350).
Ceruplasmina 18 mg/dL (5-30); Cupruria 24h 424 microgramos (0-60); Lmpara de hendidura; anillos de Kayser- Fleischer.

PREGUNTA
Considerando el cuadro clnico, que grado de encefalopata presenta?

RESPUESTA
a.- Grado I.
b.- Grado II.
c.- Grado III.
d.- Grado IV.

CASO CLINICO
Mujer de 52 aos con antecedentes de alcoholismo. Es trada al Servicio de Urgencia por cuadro de tres das de compromiso de
conciencia cuali-cuantitativo, vmitos y diarrea. Al ingreso se constata paciente confusa, inatenta, poco cooperadora. Al examen
cognitivo (Mini Mental, Test del Reloj, Test de Generacin de Palabras, Test de trecho de digitos) destaca alteracin de la atencin y
memoria episdica, con amnesia antergrada y retrgrada. El resto de las funciones cognitivas estaban relativamente conservadas. Se
observa confabulacin durante el examen mental. En el examen fsico neurolgico se encontr dismetra y nistagmo bilateral. La
tomografa computada de cerebro y los parmetros de laboratorio bsicos eran normales.

PREGUNTA
Cual es la conducta teraputica mas apropiada?

RESPUESTA
a.- Cianocobalamina.
b.- Tiamina.
c.- Niacina.
d.- Riboflamina.

CASO CLINICO
Mujer de 60 aos sindrome mielodisplsico tipo anemia refractaria simple en tratamiento, presenta cefalea hemicranea izquierda,
pulsatil, severa, sbita (2 dias) agrega fotopsias, nauseas y mareo, no relevante, a la EF se observa confusin, desorientacin,
distractibilidad, alteracin de la memoria a corto plazo. La resonancia nuclear magntica cerebral evidencia: Alteracin en la intensidad
MANUAL DE TRABAJO DEL CURSO ENARM CMN SIGLO XXI
CURSO ENARM CMN SIGLO XXI TEL: 36246001 Pharmed Solutions Institute PGINA 178

de la seal de la sustancia blanca con extensin hacia las fibras U de predominio posterior, principalmente en la regin parieto-occipital
bilateral y simtrica, con compromiso parcial de la corteza.

PREGUNTA
Cual es el diagnostico mas probable en este caso?

RESPUESTA
a.- Enfecefalopatia hipoxico-isquemico.
b.- Encefalopatia de wernicke-korsakoff.
c.- Encefalopata heptica.
d.- Encefalopatia hipercapnica.

CASO CLINICO
Paciente del sexo femenino de 26 aos de edad, refiere a los 10 minutos de histerorrafia, sentirse mal diciendo que le est dando sueo
y que el dolor no ha cedido y la tensin arterial comienza a disminuir y la frecuencia cardiaca, la paciente se encontraba ligeramente
ciantica y con tensin arterial de 80/40 torr y frecuencia cardiaca de 60 p.m. procedo a intubar sin aplicacin de frmacos inductores
ni relajantes y se continu con la administracin de oxgeno al 100% a 5 l/min, observndose movimientos de la bolsa que
corresponden a inspiracin y espiracin del automatismo respiratorio, sin embargo se aplica ventilacin asistida. Posteriormente la
paciente presenta paro cardiorespiratorio detectado con ausencia del pulso carotdeo, por lo cual se comienza a realizar maniobras de
resucitacin cardiopulmonar y administramos 1 mg de epinefrina sin respuesta positiva por lo que se procede a administrar otro
miligramo ms de epinefrina, saliendo del paro la paciente con tensin arterial de 130/90 y frecuencia cardiaca de 150 p.m., comienza a
disminuir nuevamente la frecuencia cardiaca y la presin arterial, por lo que administramos 0.6 mg de atropina y dopamina a dosis
respuesta; posteriormente, a pesar de tener la paciente tensin arterial de 130/90 torr con apoyo farmacolgico con una frecuencia
cardiaca de 150 p.m. la paciente contina con cianosis por lo que decidimos pasarla a terapia intensiva, practicndose una gasometra
arterial a su ingreso la cual nos reporta los siguientes parmetros: pH 7.27, PCO2 31, PO2 252, CO2 total 14.90, HCO3 14, EB -11.40,
Sat. O2 99.50%. Se practica una tomografa cerebral entre 36 y 48 horas despus la cual nicamente observa edema cerebral. Se
extuba dos das despus de su ingreso con respuesta neurolgica Glasgow 7; a las veinticuatro horas, presenta crisis convulsivas tnico-
clonicas de una hora de duracin por lo que sedan profundamente a la paciente y la intuban nuevamente, para que varios das despus
se declare coma y lesin cerebral con estado vegetativo.

PREGUNTA
Cual es el diagnostico mas probable en este caso?

RESPUESTA
a.- Enfecefalopatia hipoxico-isquemico.
b.- Encefalopatia de wernicke-korsakoff.
c.- Encefalopata heptica.
d.- Encefalopatia hipercapnica.

DEPRESION, ANSIEDAD, ESQUIZOFRENIA Y TRASTORNOS DE ALIMENTACION:
DEPRESIN: Trastorno del estado de nimo con repercusin en distintos mbitos de la persona. Existe una alteracin a nivel emocional,
con un sentimiento intenso de tristeza, desesperanza, abandono, inutilidad o culpa. En otras ocasiones, nimo irritable. A nivel del
contenido del pensamiento, predomina el pesimismo, con abundantes preocupaciones, ideas negativas repetitivas, angustia y fal ta de
inters con desesperanza en relacin al futuro. Desde el punto de vista somtico se producen alteraciones del sueo y del apetito, con
prdida de peso, astenia y alteraciones digestivas que son expresin de la disfuncin vegetativa. A nivel conductual se produce una
disminucin del rendimiento, con afectacin de la memoria, la atencin y la capacidad de concentracin, apata, disminucin de la
libido y tendencia al aislamiento social. Tanto en el trastorno depresivo como en el sndrome confusional agudo (SCA) puede existir una
afectacin de las funciones cognitivas con disminucin de la atencin y del ciclo vigilia-sueo. Cabe destacar la depresin con rasgos
psicticos, donde el paciente pierde el contacto con la realidad, combinando sntomas de depresin y psicosis, con aparicin de
alucinaciones o delirios. Habitualmente, el contenido de stos es coherente con la depresin, con una temtica relacionada con ideas
de culpa, induccin al suicidio, etc. Existen varios tipos de episodios depresivos en funcin de su intensidad y curso: el trastorno
depresivo mayor, que generalmente aparece de forma episdica, y el trastorno distmico, menos intenso y con curso crnico. El
episodio depresivo mayor tiende a la recuperacin completa, y puede darse de forma aislada en la vida de un sujeto. Sin embargo en
general es una enfermedad con tendencia a la cronicidad y resulta ms incapacitante que otras enfermedades mdicas como la artritis
o la diabete. Etiopatogenia, podra ser el modelo biopsicosocial, que resultara de la integracin de tres perspectivas diferentes, la
biolgica, la psicolgica y la social. Los sistemas de neurotransmisin serotoninrgica, noradrenrgica, dopaminergica y peptidrgica
estn implicados en la depresin. La serotonina produce la inhibicin o la activacin de la neurotransmisin en el sistema nervioso
central. Desde las neuronas de los ncleos dorsales y caudales del rafe se extienden numerosas proyecciones serotoninrgicas hacia
reas cerebrales asociadas a sntomas depresivos. Se ha observado disminucin de los niveles de serotonina en el LCR de pacientes con
depresin. El sistema noradrenrgico est tambin implicado en la depresin. El sistema dopaminrgico implica reas cerebrales de las
que depende el comportamiento y funciones fisiolgicas alteradas en la depresin. La dopamina puede intervenir en la depresin
sobretodo en la mana, ya que algunos agonistas dopaminrgicos se han asociado a la aparicin de mana y los antagonistas son
eficaces en su tratamiento. CRITERIOS DSM-IV-TR DEL EPISODIO DEPRESIVO MAYOR: Cinco o ms de los siguientes sntomas (entre los
que debe cumplirse los dos primeros) que representan un cambio respecto a la actividad previa, durante un periodo mnimo de dos
semanas. Uno de los sntomas debe ser el estado de nimo depresivo, o la prdida de inters o de la capacidad para el placer. Estado
de nimo depresivo. Disminucin del inters o la capacidad para el placer. Aumento o prdida importante de peso. Insomnio o
MANUAL DE TRABAJO DEL CURSO ENARM CMN SIGLO XXI
CURSO ENARM CMN SIGLO XXI TEL: 36246001 Pharmed Solutions Institute PGINA 179

hipersomnia casi cada da. Agitacin o enlentecimiento psicomotor casi cada da. Fatiga o prdida de energa casi cada da.
Sentimientos de inutilidad o de culpa excesivos o inapropiados. Disminucin de la capacidad de pensar, concentrarse o indecisin, casi
cada da. Pensamientos recurrentes de muerte, ideacin suicida recurrente. El tratamiento en todo tipo de depresiones el uso de
frmacos antidepresivos puede resultar beneficioso, no obstante hay que valorar el riesgo-beneficio especialmente en pacientes con
otras patologas y en pacientes ancianos. Una excepcin es la depresin bipolar, por lo que es importante hacer una buena exploracin
sobre los antecedentes personales de episodios manacos e hipomanacos, y de la historia familiar del paciente. En los pacientes
bipolares, antes de prescribir un antidepresivo es necesario administrar un frmaco aprobado como estabilizador del estado de nimo.
Tratamiento de la depresin: Inhibidores selectivos de la recaptacin de serotonina (ISRS), los inhibidores selectivos de la recaptacin
de noradrenalina (ISRSN), los moduladores selectivos duales de los receptores de la noradrenalina y serotonina (NaSSA), inhibidores de
la recaptacin noradrenrgica (IRN), inhibidores duales de la recaptacin de la noradrenalina y dopamina (IRNaDa), Inhibidores de la
monoaminooxidasa (IMAO) que comprenden varios frmacos. Los nuevos antidepresivos en su conjunto han dado lugar a mejoras en la
farmacoterapia de la depresin por su facilidad de uso, menos efectos secundarios y mayor seguridad en sobredosis.
ANSIEDAD: Estado afectivo fisiolgico que consiste en tener una sensacin difusa de aprensin. Es una sensacin desagradable y vaga,
de aparicin aguda y transitoria, que se acompaa de una activacin del sistema nervioso autnomo. Este estado acta como
mecanismo de alerta y adaptacin ante una situacin amenazante para la integridad fsica o psquica del individuo, o percibida como
tal. La ansiedad patolgica constituye un estado de ansiedad que se desencadena sin un estmulo amenazante, a diferencia del miedo,
que constituye una situacin de ansiedad provocada por un estmulo realmente amenazante. La manifestacin de la ansiedad se basa
en tres componentes clnicos fundamentales: un componente somtico (sntomas vegetativos), uno cognitivo (sensacin de
nerviosismo, aceleracin, percepciones de malestar, etc) y uno motor (temblor,etc). Trastornos de ansiedad segn la DSM-IV TR: 1.
Ataques de pnico (crisis de ansiedad, crisis de angustia, panic attac). 2. Agorafobia Trastorno de angustia sin agorafobia (F41.0)
Trastorno de angustia con agorafobia (F40.01) Agorafobia sin historia de trastorno de angustia (F40.00). 3. Fobia especfica
(F40.02)4. Fobia social (F40.1). 5. Trastorno obsesivo-compulsivo (F42.8). 6. Trastorno por estrs postraumtico (F43.1) 7. Trastorno
por estrs agudo (F43.0)8. Trastorno de ansiedad generalizada (F41.1) 9. Trastorno de ansiedad debido a enfermedad mdica (F06.4)
10.Trastorno de ansiedad inducido por sustancias Adems de estos trastornos, agrupados bajo el rtulo trastornos de ansiedad, en
la DSM-IV TR se incluye finalmente un trastorno de ansiedad infantil, el trastorno de ansiedad por separacin.
En el trastorno del pnico se podrn utilizaransiolticos, y para prevenir la crisis se administrarn antidepresivos. Este tratamiento
tendr una duracin de 6-12 meses. El trastorno obsesivo-compulsivo deber ser tratado por el psiquiatra. El
tratamiento consiste en la combinacin de un frmaco junto a terapia psicolgica de modificacin de conducta. Las fobias.
Hay que distinguir el tipo. Si es especfica, se puede abordar en atencin primaria con untratamiento semejante al de
la ansiedad; pero si es grave, como la agorafobia, fobia social generalizada o algn tipo de fobia simple,ser tratada por el
especialista. TRASTORNOS DE LA ALIMENTACION: La ANOREXIA NERVIOSA: Se caracteriza por la negativa para mantener el peso
corporal normal, lo que resulta en un peso corporal <8 % del peso esperado para la edad y altura. Signos y sntomas: sensacin de fro,
la piel, el cabello, las uas quebradizos, alopecia , lanugo, acrocianosis , edema, bradicardia , hipotensin, agrandamiento de las
glndulas salivales, el vaciamiento gstrico lento, estreimiento, enzimas hepticas elevadas, anemia normocitica normocromica,
leucopenia, aumento de nitrgeno de urea en sangre , aumento de la creatinina, hiponatremia, hipopotasemia, baja la hormona
luteinizante y la hormona folculo-estimulante con amenorrea secundaria, hipoglucemia, la hormona estimulante de la tiroides normal
con baja tiroxina, aumenta el cortisol plasmtico. Caractersticas de diagnstico: Rechazo a mantener el peso corporal igual o por
encima del valor mnimo normal. (Esto incluye un fracaso para lograr la ganancia de peso esperada durante un periodo de crecimiento
que conduce a un peso corporal anormalmente bajo). Miedo intenso a la ganancia de peso o la grasa. Distorsin de la imagen corporal
(por ejemplo, sensacin de grasa a pesar de un peso objetivamente bajo o minimizar la gravedad de bajo peso). La amenorrea (Este
criterio se cumple si los perodos menstruales se producen slo despus de hormonas, por ejemplo, la administracin de estrgenos).
DIAGNOSTICO DIFERENCIAL DE TRASTORNOS DE ANSIEDAD
TIEMPO
DESENCADENANTE MANIFESTACIONES OBSERVACIONES DURA
CION
PERIODICIDAD DE
SINTOMAS
Fobia
especifica
No Episodios espordicos,
min/hrs
Estimulo fbico (presencia o
anticipacin)
Miedo a un objeto o situacin determinada. Temor o
sntomas ansiosos
Evitacin de la situacin que
provoca el temor
Fobia social No Episodios
espordicos,
min/hrs
Situaciones sociales o escrutinio social (presencia o
anticipacin). Temor angustioso a
determinadas circunstancias, reales o imaginarios
Preocupacin por que el desempeo sea casa de
humillacin.
Miedo a colocarse en una situacin vergonzosa
en un medio social. Temor o sntomas ansiosos.
Preocupacin de que el desempeo pueda ser
causa de humillacin o burla
Evitacin de la situacin
que provoca el temor
Trastorno
de pnico
1 mes Episodios
espordicos 5-20
min
Aparicin brusca de un medio intenso.
Ninguno aparente, pero el sujeto lo
puede atribuir a mltiples causas
Temblor, sudoracin, miedo a morir, sensacin de
frio/calor, nauseas, sensacin de asfixia, marea,
miedo a perder el control, dolor de pecho Ataques
de pnico (AT)sbitos inesperados y recurrentes
1 de 3: ansiedad anticipatoria.
Preocupacin constante,
Repercusin de los AP. Cambio
conductual significativo
Ansiedad
generalizad
a (TAG)
6
meses
La mayor parte del
da casi todos los
das
Aparicin progresiva y permanente
de sntomas de ansiedad, sin una
causa real que los provoque
Fatiga, inquietud, dificultad para dormir,
irritabilidad, tensin muscular,
preocupaciones, nerviosisimos excesivos
cuya intensidad han debilitado el control
sobre ellos
El individuo est preocupado
permanentemente sin motivos aparentes y no
lo puede controlar. 3 de 6: intranquilidad,
fatigabilidad, concentracin, irritabilidad,
tensin muscular, alteracin en el dormir
Obsesivo
compulsivo
(TOC)
No Min7hrs Obsesiones que pueden ser das o
pensamientos que se repiten, y no
desaparecen de le mente del individuo,
aunque lo intente por todos los medios
Cada obsesin lleva asociada una compulsin o
conducta que compensa la angustia que provoca
Un ejemplo es el miedo al
contagio, cuya composicin
correspondiente es lavarse las
manos rpidamente
Estrs
postraumti
cos
(EPT)
Aparece en aquellos individuos que se
han visto expuesto a un evento
traumtico, que involucra un dao fsico
o psicolgico extremo
Perdida del sueo, irritabilidad, sobresalto, falta de
concentracin. Re exoperimentracion de la situacin
traumtica: recuerdos intrusivos recurrentes, sueos
estresantes recurrentes, flashbacks, temor ansiedad o
reacciones neorovegetativas intensas al exponenrsea
situaciones u objetosrelacionados con el evento.
Provoca alteraciones en la
vida familiar, laboral y social
del individuo

MANUAL DE TRABAJO DEL CURSO ENARM CMN SIGLO XXI
CURSO ENARM CMN SIGLO XXI TEL: 36246001 Pharmed Solutions Institute PGINA 180

Tratamiento: Recuperacin del peso a 90% de peso predicho es el objetivo principal. La intensidad del tratamiento inicial, incluyendo la
necesidad de hospitalizacin, se determina por el peso actual, la rapidez de la prdida de peso reciente, y la gravedad de las
complicaciones mdicas y psicolgicas. Desequilibrios electrolticos graves deben ser identificadas y corregidas. Recuperacin
nutricional casi siempre se puede lograr con xito por la va oral. Las caloras se pueden aumentar gradualmente hasta lograr un
aumento de peso de 1-2 kg/sem (3000-4000 kcal /d). Las comidas deben ser supervisadas. La ingesta de vitamina D (400 UI/d) y calcio
(1.500 mg/d) debe ser suficiente para reducir al mnimo la prdida de hueso. La ayuda de psiquiatras o psiclogos suele ser necesario.
No hay medicamentos psicotrpicos. Las complicaciones mdicas en ocasiones se producen durante la realimentacin, la mayora de
los pacientes transitoriamente retienen el exceso de lquido, a veces resultando en edema perifrico. La insuficiencia cardaca
congestiva y la dilatacin gstrica aguda se han descrito cuando la realimentacin es rpida. Elevaciones modestas transitorias en los
niveles sricos de enzimas hepticas ocurren ocasionalmente. Los niveles bajos de magnesio y fosfato deben ser reemplazados. La
mortalidad es del 5% por dcada, ya sea de hambre crnica o el suicidio. BULIMIA NERVIOSA: Se caracteriza por episodios recurrentes
de atracones seguidos de conductas compensatorias anormales, tales como vmitos autoinducidos, abuso de laxantes o ejercicio
excesivo. Peso es en el rango normal o por encima. Signos y sntomas: agrandamiento de las glndulas salivales, la erosin dental,
hipopotasemia, hipocloremia, alcalosis (vmitos) o acidosis (de abuso de laxantes). Caractersticas de diagnstico: Los episodios
recurrentes de atracones de comida, que se caracterizan por el consumo de una gran cantidad de alimentos en un corto perodo de
tiempo y la sensacin de que la alimentacin est fuera de control. Comportamiento inapropiado recurrente para compensar la ingesta
compulsiva, como el vmito autoinducido. La aparicin de tanto el trastorno por atracn y el comportamiento compensatorio
inapropiado al menos dos veces por semana, en promedio, durante 3 meses. Preocupacin excesiva por la forma y el peso corporal.
Nota: Si se cumplen simultneamente los criterios diagnsticos para la anorexia nerviosa, slo se da el diagnstico de anorexia
nerviosa. Puede ser tratada de forma ambulatoria. La terapia cognitiva conductual y la fluoxetina (Prozac) son tratamientos de primera
lnea. La dosis de tratamiento recomendada para la fluoxetina (60 mg/d) es ms alta que la normalmente utilizada para tratar la
depresin. Tanto la anorexia nerviosa y la bulimia nerviosa presentan principalmente en mujeres jvenes previamente sanos que se
convierten en demasiado preocupados con la forma y el peso corporal. Atracones y las purgas comportamiento que puede estar
presente en ambas condiciones, con la distincin fundamental entre los dos que descansa sobre el peso de la persona. Las
caractersticas diagnsticas. ESQUIZOFRENIA: Trastorno mental severo que afecta al pensamiento, las emociones y el comportamiento.
Es la forma ms frecuente de trastorno mental severo y afecto a una persona de cada 100. Es poco frecuente antes de la pubertad y
ms comn de 15 a 35 aos. Con frecuencia dura toda la vida. Un nio que tiene un padre con esquizofrenia tiene una probabilidad
entre 10 de desarrollar esquizofrenia. Las infecciones virales durante el embarazo, complicaciones en el parto, crecer en el centro de
grandes ciudades y el uso de drogas tambin parecen jugar un papel en el desarrollo de esta enfermedad. La estructura y qumica del
cerebro pueden estar afectadas, pero no existe una prueba simple para su diagnstico hasta el momento. Formas clnicas:
Esquizofrenia paranoide. Esquizofrenia catatnica. Esquizofrenia hebefrnica/desorganizada. Esquizofrenia residual. Existen dos formas
de grupos: positivos y negativos, normalmente tienen una mezcla de los dos. Sntomas positivos: Delirios (creencias que no slo
no son ciertas sino que pueden parecer incluso bastante extraas). Si intentas discutir estas creencias, encontrars que el que las tiene
las mantendr a pesar de que exista mucha evidencia en su contra. Trastorno del pensamiento (dificultad para pensar con claridad).
Ser difcil entenderlos porque sus frases parecen no tener sentido: pueden saltar de una idea a otra, perdiendo el tema del que estn
intentando hablar. Alucinaciones (ves, oyes, hueles o sientes algo que no est ah). Lo ms comn or voces. Estas voces parecen
absolutamente reales, pueden asustar y hacer creer que la gente te est observando, escuchando o molestando. Sntomas negativos:
Las personas jvenes que padecen esquizofrenia puede volverse muy inactivas, solitarias y parecer desmotivadas. Parecen perder
inters y motivacin y pueden dejar de lavarse regularmente o cuidarse de una forma apropiada. Normalmente no pueden concentrase
en un trabajo o en el estudio. Generalmente los sntomas son de una gravedad suficiente como para causar preocupacin, tanto en
casa como en la escuela. Los sntomas positivos o negativos por s solos no necesariamente causan esquizofrenia. Criterios diagnsticos:
A. Sntomas caractersticos: Dos (o ms) de los siguientes sntomas, cada uno de ellos presente durante una porcin de tiempo
significativa a lo largo de un mes (o menos en el caso de que hayan sido tratados satisfactoriamente): (1) Ideas delirantes (2)
Alucinaciones (3) Desorganizacin del habla (4) Conducta catatnica o marcadamente desorganizada (5) Sntomas negativos, i.e.,
embotamiento afectivo, alogia, o abolicin B. Deterioro social/ocupacional. C. Duracin: Los signos persisten de manera continuada por
lo menos seis meses. Este periodo de seis meses debe incluir al menos un mes de sntomas (o menos si responden a tratamiento) del
criterio A, pudiendo incluir periodos de sntomas prodrmicos o residuales. Durante estos periodos prodrmicos o residuales los signos
del trastorno pueden consistir en slo sntomas negativos o dos o ms sntomas del criterio A presentes de forma atenuada (ej.,
pensamientos extraos, experiencias de la percepcin inusuales) D. Exclusin de trastorno del nimo o esquizoafectivo. E. Exclusin de
causa orgnica o abuso de sustancia o a condicin mdica general F. Relacin con trastorno del desarrollo: Si existe historia de autismo
u otro trastorno del desarrollo, el diagnstico adicional de esquizofrenia se realizar slo si existen delirios o alucinaciones marcadas
durante al menos un mes (o menos si responden a tratamiento). El tratamiento se centra principalmente en eliminar los sntomas. Para
obtener el mejor resultado, todo el mundo implicado, incluida la persona afectada, la familia, el equipo psiquitrico, los docentes y los
servicios sociales, necesitan trabajar juntos desde el inicio. Controla los sntomas de la enfermedad y permite reiniciar una vida normal.
La medicacin tiende a ser ms efectiva con los sntomas positivos y menos con los negativos. Las alucinaciones y las ideas delirantes
pueden tardar algunas semanas en desaparecer. ANTIPSICOTICOS Muchas divisiones, la ms utilizada: convecionales y nueva
generacin (tpicos y atpicos); Antipsicticos convencionales: haloperidol, levomepromacina, pimocide, zuclopentixol, sulpiride,
tioridacina. Antipsicticos de nueva generacin: clozapina, risperidona, olanzapina, quetiapina, ziprasidona, amisulpiride y aripiprazol.
Tratamiento esquizofrenia resistente:clozapina (vigilar agranulocitosis). Los antipiscticos tratan parte de los sntomas de la
enfermedad, mejoran el funcionamiento y calidad de vida y evitan recaidas. Antipsicticos de nueva generacin: clozapina, risperidona,
olanzapina, quetiapina, ziprasidona, amisulpiride y aripiprazol. Antes de cambiar de antipsictico esperar al menos 3-4 semanas a dosis
teraputicas: Evaluar el riesgo de efectos secundarios a corto y largo plazo. Especial importancia tiene la dicinesia tarda y el sndrome
neurolptico maligno. Evaluar la respuesta subjetiva al tratamiento. Reforzar siempre necesidad de tratamiento y evaluar conciencia de
enfermedad. Para la sintomatologa extrapiramidal (SEP) se deben dar anticolinrgicos como el biperideno (Akineton), en caso de
distona aguda dar va intramuscular y el cuadro revierte a los pocos minutos. Para la acatisia (sensacin interna de inquietud, dificultad
para estar sentado, mover las piernas, etc) se puede utilizar propanolol. En algunos casos tambin responde a anticolinrgicos
MANUAL DE TRABAJO DEL CURSO ENARM CMN SIGLO XXI
CURSO ENARM CMN SIGLO XXI TEL: 36246001 Pharmed Solutions Institute PGINA 181

(biperideno) o dosis bajas de benzodiacepinas. La acatisia es especialmente preocupante porque se asocia a abandono del tratamiento,
agresividad y suicidio. TEC especialmente en pacientes con catatona, gran riesgo de suicidio o importante componente afectivo.
Terapia conductual: Tiene en cuenta las deficiencias que tienen los pacientes. Habilidades sociales en las que se ensea a los paientes a
relacionarse, buscar trabajo, hacer una entrevista, utilizar medios de transporte, etc. Terapia orientada a la familia (sistmica):
Informacin, identificacin de factores precipitantes de recada, reducir emocin expresada. Importancia de asociaciones de familiares.
Terapia grupal: Centrada en planes de la vida diaria, intercambio de experiencias, conciencia de enfermedad, resolucin de problemas.
Psicoterapia individual: Su eficacia se aada a la del tratamiento farmacolgico. Importancia de la relacin teraputica.

CASO CLINICO
Mujer de 69 aos, trada a la consulta por su hija, por presentar sensaciones de abandono persistente, temor a quedarse sola en su
casa, cambios del humor , insomnio intermedio y final (logra conciliar el sueo, pero se despierta en la madrugada, a veces ya no logra
volver a dormir), la invaden pensamientos catastrficos, miedos sobre todo cuando su hija sale del domicilio a trabajar, piensa que algo
malo le puede suceder y no soporta quedarse sola, llora constantemente, presenta labilidad emocional, su comportamiento ha
regresionado de ser una persona muy independiente, a ser alguien que exige proteccin constantemente, llegando inclusive a agotar
emocionalmente a su familiar, la sintomatologa se evidencia despus de recuperarse de una crisis hipertensiva, la hija refiere que
desde entonces su madre ya no fue la de antes. Antecedentes: Siempre fue una persona independiente, su madre falleci cuando era
una adolescente y su padre la abandono con sus tres hermanos, era la segunda de cuatro hermanos, se vio obligada a trabajar desde
muy joven y resolva los problemas cotidianos sin dificultad llegando incluso a la sobreproteccin con sus familiares, su experiencia de
pareja al parecer fue complicada, brinda pocos datos al respecto, decidi vivir sola con su hija la cual tuvo estando soltera, Hasta su
adultez sufri de cuadros migraosos con frecuencia

PREGUNTA
Cual es el diagnostico mas probable del caso presentado?

RESPUESTA
a.- Trastorno distimico.
b.- Episodio depresivo.
c.- Trastorno depresivo.
d.- Trastorno de ansiedad.

CASO CLINICO
Mujer de 58 aos. Lleva ms de 20 aos en tratamiento psiquitrico y ha sido valorada por ms de un psiquiatra durante su evolucin.
Desde los 20 aos de edad, ella se reconoce triste. Excepto por breves temporadas, cada una de las cuales raramente ha durado ms de
un mes, en las que se nota prcticamente bien. La intensidad de la sintomatologa nunca ha sido severa. En general, con ciertos
reparos, reconoce que siempre ha seguido desarrollando las tares domsticas. Desde hace aos es su hija quien se responsabiliza de la
casa, y ella procura ayudarle. Se queja de tristeza, desnimo al afrontar el da desde que se levanta. Pero sobre todo una pobre
confianza en s misma. Dificultad para concentrarse en cualquier tarea que con dificultad emprende.

PREGUNTA
Cual es el diagnostico mas probable para el caso?

RESPUESTA
a.- Trastorno bipolar.
b.- Depresin mayor.
c.- Trastorno Distmico.
d.- Depresin Psictica.

CASO CLINICO
Mujer de 43 a. acude al Servicio de Urgencias. Se siente cansada y sin fuerzas para vivir. Responde con lentitud a las preguntas, en un
tono de voz bajo. Dice encontrarse muy triste desde hace dos meses, lo dice con precisin. Ha perdido inters por la vida, se nota
distinta, como si no le importara nada, ni siquiera lo que hagan su marido o sus hijas, aunque no quiere verlos sufrir. Tiene muy poco
apetito, ha perdido casi 5 kg en un mes. Tarda mucho en quedarse dormida y se despierta varias veces por la noche. Se siente intil y
un estorbo en casa. Piensa en la muerte, aunque no quiere quitarse la vida, porque ello va contra sus creencias religiosas. Nunca antes
le haba ocurrido algo parecido.

PREGUNTA
Cual es el diagnostico mas probable para el caso?

RESPUESTA
a.- Trastorno bipolar.
b.- Depresin mayor.
c.- Trastorno Distmico.
d.- Depresin Psictica.



MANUAL DE TRABAJO DEL CURSO ENARM CMN SIGLO XXI
CURSO ENARM CMN SIGLO XXI TEL: 36246001 Pharmed Solutions Institute PGINA 182

CASO CLINICO
La paciente, nacida en 1981. Vena padeciendo ataques de angustia episdicos recidivantes desde el ao 2003, sobre todo cuando se
encontraba sola. Tampoco era capaz de recorrer trayectos largos en tren ni en metro. Las crisis comenzaron tras el fallecimiento de su
abuelo, con el cual haba tenido supuestamente una relacin especial, ya que siempre haba sufrido los problemas de relacin de sus
padres. Ms o menos al mismo tiempo se rompi el antebrazo por una cada, que requiri una complicada operacin. Esto acrecent
an ms sus crisis de angustia. Por lo dems, refiri que gozaba de buena salud, no fumaba ni beba alcohol, aunque descuidaba las
comidas por motivos de tiempo. Dorma bien, necesitaba de 7 a 8 horas de sueo diarias, en caso contrario no se encontraba en forma.

PREGUNTA
Cual es el diagnostico mas probable en el caso descrito?

RESPUESTA
a.- Trastorno de pnico.
b.- Trastorno de estrs.
c.- Trastorno de ansiedad.
d.- Trastorno obsesivo compulsivo.

CASO CLINICO
Femenino, de 52 aos, es ama de casa y madre de cinco hijos ya adultos. La relacin con su marido ha perdido mucho con los aos pero
se resiste a iniciar la separacin. Se ha preocupado en exceso por muchos motivos: sus hijos, su madre, sus nietos... Algunas frases
tpicas suyas son:Ha llegado Joaqun?; Estn bien los cros?; Llevad cuidado con el coche; No salgis hasta muy tarde, que nunca se
sabe qu puede pasar en la noche. El menor de sus hijos tiene ya 20 aos y se ha acostumbrado a las continuas advertencias y
sugerencias para prevenir males posibles. Tambin se ha acostumbrado a llamarla por telfono en mitad de la noche cuando sale con
los amigos para informarle de que no pasa nada, que todo va bien. Reconoce que le resulta difcil dejar de preocuparse tanto por todos
y por todo. Le resulta muy difcil concentrarse en otra cosa que no sean los peligros que acechan a los suyos, confundiendo con
frecuencia el hecho de que un peligro sea posible con el hecho de que sea probable. Tiene dificultades para dormir y mucha tensin
muscular acumulada. Se resista a reconocer que lo suyo era un problema de ansiedad porque los peligros son reales.

PREGUNTA
Cual es el diagnostico mas probable en el caso descrito?

RESPUESTA
a.- Trastorno de pnico.
b.- Trastorno de estrs.
c.- Trastorno de ansiedad.
d.- Trastorno obsesivo compulsivo.

CASO CLINICO
Varn de 33 aos, que acude por episodios de agitacin psicomotriz en domicilio, heteroagresividad hacia familiares y conductas
acontextuales. Ingresa mostrndose hostil, irritable, suspicaz y autorreferencial. Episodio de agitacin psicomotriz en Urgencias y
heteroagresividad dirigida a familia y mobiliario. Es necesaria contencin farmacolgica y mecnica hasta remisin de la clnica actual
(Olanzapina i.m) y se cursa ingreso involuntario (nula conciencia de enfermedad, riesgo alto para el paciente y terceros y necesidad de
completar estudio) Conductas de riesgo desde la adolescencia (carreras de coches, consumo de txicos -THC y cocana). Personalidad
premrbida impulsiva segn su familia y con escasa tolerancia a las normas. Soltero. Padre de una nia que nace en 2000, con la que
mantiene escasa relacin. En 2002, a la edad de 23 aos, en contexto de intoxicacin enlica, sufre accidente de trfico con
importantes secuelas orgnicas, politraumatizado (incluido traumatismo craneoenceflico (TCE) grave) requiriendo hospitalizacin
durante 70 das. Permanece en coma segn historia clnica durante 15 das hasta recuperacin total del nivel de conciencia. Al da de
hoy no secuelas fsicas.

PREGUNTA
Cual es el diagnostico mas probable en el caso descrito?

RESPUESTA
a.- Trastorno de personalidad.
b.- Trastorno mental secundario a lesin neurolgica.
c.- Trastorno de la conducta secundaria a lesin neurolgica.
d.- Trastorno de control de impulsos.

CASOS CLINICOS
Paciente de 50 aos, que presenta en la anamnesis entre otros los siguientes sntomas: presencia de obsesiones y de compulsiones o de
actos obsesivos de manera recurrente, con prdida de tiempo significativa, que supone una alteracin en el funcionamiento normal, en
un paciente que comprende lo absurdo de su forma den proceder.

PREGUNTA
Cual es el tratemiento mas adecuado para el caso descrito?

MANUAL DE TRABAJO DEL CURSO ENARM CMN SIGLO XXI
CURSO ENARM CMN SIGLO XXI TEL: 36246001 Pharmed Solutions Institute PGINA 183

RESPUESTA
a.- Fluvoxamina.
b.- Fluoxetina.
c.- Sertralina.
d.- Clorimipramina.

CASO CLINICO
Persona de 30 aos, que en el curso de menos de 6 meses, acude a la consulta y observamos que el ndice de Quetelet es menos (peso
en Kg/altura al cuadrado) que 17,5 (normal de 19 a 24). Ha perdido 10 kilos o ms en ese tiempo, presenta amenorrea de tres o ms
ciclos consecutivos, y un miedo intenso a ganar eso.

PREGUNTA
Cual es el tratemiento mas adecuado para el caso descrito?

RESPUESTA
a.- Fluvoxamina.
b.- Fluoxetina.
c.- Sertralina.
d.- Clorimipramina.

CASO CLINICO
Paciente con 27 aos, que acude en busca de tratamiento para lo que cree bulimia nerviosa, mujer de peso normal, que refiere
episodios de ingestin desenfrenada de comida. Los familiares no saben qu hacer, por qu la bulimia nerviosa le est llevando a otras
adicciones como el alcoholismo.

PREGUNTA
Cual es el tratemiento mas adecuado para el caso descrito?

RESPUESTA
a.- Fluvoxamina.
b.- Fluoxetina.
c.- Sertralina.
d.- Clorimipramina.

CASO CLINICO
Paciente mujer de 42 aos acude, acompaada, por presentar episodio de agitacin psicomotriz y trastorno de conducta con
heteroagresividad fsica y verbal en entorno familiar. Se procede a su ingreso para aclaracin diagnstica y contencin de la situacin,
con orientacin diagnstica de sndrome maniforme. Psiquitricos: A los 21 aos debuta con un episodio depresivo, cuya
sintomatologa cedi en pocos meses y fue tratado por Mdico de Atencin Primaria. Tras un segundo episodio depresivo, presenta a
los 24 aos un primer cuadro maniforme que requiri ingreso hospitalario y cuya orientacin diagnstica al alta fue de trastorno
bipolar, episodio actual maniaco.

PREGUNTA
Cual es el tratamiento mas adecuado para el caso descrito?

RESPUESTA
a.- Litio
b.- Olanzapina.
c.- Lamotrigina.
d.- Valproato.



MANUAL DE TRABAJO DEL CURSO ENARM CMN SIGLO XXI
CURSO ENARM CMN SIGLO XXI TEL: 36246001 Pharmed Solutions Institute PGINA 184


URGENCIAS

1) MUERTE SUBITA, RCP.
2) ESTADO CHOQUE
3) URGENCIAS, EMERGENCIA HIPERTENSIVA. HIPERTENSION MALIGNA.
4) INSUFICIENCIA CARDIACA AGUDA Y CRONICA AGUDIZADA.
5) TAPONAMIENTO CARDIACO, DISECCION DE LA AORTA, ANEURISMA AORTICO.
6) ANGINA ESTABLE, ANGINA INESTABLE, ANGINA DE PRINZTMETAL.
7) SICA II, SICA II
8) TRASTORNOS DEL RITMO (ARRITMIAS CARDIACAS)
9) FALLA ORGANICA MULTIPLE, COAGULACION INTRAVASCULAR DISEMINADA.
10) ANAFILAXIA Y ALERGIAS.
11) EPOC AGUDIZADO, TROMBOSIS VENOSA PROFUNDA, TROMBOEMBOLIA PULMONAR.
12) SINDROME DE INSUFICIENCIA RESPIRATORIA AGUDA, EDEMA AGUDO PULMONAR.
13) NEUMOTORAX, NEUMOMEDIASTINO, HEMOTORAX, CONTUSION CARDIACA.
14) TRAUMA TORACICO CERRADO Y ABIERTO.
15) INSUFICIENCIA RENAL AGUDA, GLOMERULOPATIAS AGUDAS.
16) SINDROME NEFRITICO Y NEFROTICO.
17) TRASTORNOS ACIDO-BASE AGUDOS.
18) TRASTORNOS ELECTROLITICOS AGUDOS.
19) HIPOGLUCEMIA, HIPERGLUCEMIA, ESTADO HIPEROSMOLAR, CETOACIDOSIS DIABETICA.
20) TRAUMATISMO CRANEOENCEFLICO, ISQUEMIA CEREBRAL TRANSITORIA, ACCIDENTE VASCULAR CEREBRAL.
21) CRISIS CONVULSIVAS, ESTATUS EPILEPTICO, COMA Y MUERTE CEREBRAL.
22) TRASTORNO BIPOLAR, DELIRIUM, PSICOSIS, SUICIDIO.



MANUAL DE TRABAJO DEL CURSO ENARM CMN SIGLO XXI
CURSO ENARM CMN SIGLO XXI TEL: 36246001 Pharmed Solutions Institute PGINA 185

MUERTE SBITA (MS):
CIENCIAS BSICAS: Muerte (ausencia de signos vitales) inesperada sin sntomas precedentes la mayora de las veces o que en caso de
existir estos, ocurren pocos segundos antes de que la muerte sobrevenga y sin causa traumtica aparente o que la explique. Son 4 las
arritmias letales, las de ritmos descargables como; fibrilacin ventricular (FV), taquicardia ventricular sin pulso (TVSP), y las de ritmos no
descargables; asistolia y actividad elctrica sin pulso (AESP). SALUD PBLICA: En Estados Unidos de Norteamrica hay hasta 300mil
muertes por ao. En Mxico entre 33 y 53 millones anuales, en general ligadas a enfermedad isqumica cardiaca. La cardiopata
coronaria aumenta 4-6 veces el riesgo de muerte sbita. En 60-70% de los casos de muerte sbita, sobreviene como consecuencia, de
un evento cardiovascular; 1. Cardiopata coronaria (isqumica) con o sin antecedentes conocidos, es responsable del 70-80%. 2.
Cardiopata estructural, congnita o adquirida 15-20% (miocardiopata hipertrfica, dilatada, displasia del ventrculo, estenosis mitral)
3. Arritmias 5% asociadas a fenmenos elctricos primarios o del sistema
excitoconductor (sx. de QT largo, sx. de Brugada, sx. Wolf-Pakinson-White). Algunos de
estos pacientes concentran una historia familiar positiva, que puede ser el nico
elemento que permita identificarlos como grupo de riesgo. La FV causa >90% de MS.
PATOGENIA: Existe un descontrol total del corazn, debido a la presencia del fenmeno
R sobre T (R/T), el cual se presenta cuando llega un estimulo en la parte final de la
repolarizacin. Es de importancia el periodo refractario relativo (PRR), en el si llega un
estimulo que rebase el umbral, el msculo se excita en su zona lbil, se desencadena una
arritmia cardiaca y puede sobrevenir la muerte. (PRA=aunque el estimulo sea muy fuerte
las clulas del miocardio no se van excitar). Los que tienen ms riesgo de sufrir un
fenmeno de R/T son: post IAM, porque se quedan cicatrices y estas son focos
ectpicos que desencadenan estmulos, en los que tienen hace anmalos, en los
hipertensos con miocardiopata hipertrfica. Solo 2 arritmias descontrolan el corazn
porque producen el fenmeno de R/T (en el ECG, vemos una extrasstole ventricular) y
lo pueden llevar a la muerte; la taquicardia ventricular (ondas regulares >180 lpm, no hay ondas P, complejos anchos) y la fibrilacin
ventricular (desorden y ritmo catico). Ninguna de las 2 produce gasto cardiaco, el primer signo es el sincope, el cerebro se desconecta
(no llega oxigeno, ni glucosa), tambin se puede presentar una convulsin como manifestacin de anoxia. La persistencia de la falta de
riego lleva a la destruccin irreversible y al fallecimiento del individuo. As tenemos isquemiaTVFVMS. Muerte sbita es el
resultado de; Condicin subyacente (cardiopata coronaria o estructural) + Susceptibilidad individual (inestabilidad elctrica, no todos
los pacientes con SCA hacen FV solo el 15%) + Evento gatillante (isquemia, trastorno HE o acido-base, frmaco). DIAGNOSTICO: La
muerte sbita cardaca no tiene unos sntomas previos como tal, pero s se pueden enumerar una serie de posibles factores que
pueden influir en su aparicin: 1. Que el sujeto haya experimentado un episodio de estas caractersticas anteriormente o que tenga
familiares con antecedentes 2. Insuficiencia cardiaca. 3. Haber sufrido un infarto de miocardio. 4. Arritmias cardacas. TRATAMIENTO:
Slo hay un tratamiento efectivo para frenar la muerte sbita cardaca y es la desfibrilacin precoz. Este procedimiento consiste en
realizar una descarga elctrica en el corazn, a travs de unas palas o parches, con la que se pretende reiniciar la actividad elctrica del
corazn. En el caso de no disponer de un desfibrilador a mano, o de no saber utilizarlo, se puede realizar una reanimacin
cardiopulmonar. Tambin existe la posibilidad de implantar un desfibrilador cuando se detecta, por ejemplo, un sndrome de Brugada.
Que el porcentaje de sangre bombeada por el corazn en cada latido, denominada 'fraccin de eyeccin', sea igual o inferior a un 40
por ciento. PREVENCION PRIMARIA: Identificar los grupos de riesgo; GRUPO DE MAYOR RIESGO DE SUFRIRI UN EPISODIO DE MS: A).
Cardiopata coronaria sintomtica: con IAM en evolucin o reciente. Angina inestable. B). Cardiopata estructural asociada a cardiopata
coronaria, ICC FEVI < 35 %. C). Con arritmias ventriculares, Definen riesgo de MS : TVNS inducible o espontnea, TVS inducible o
espontnea. En este grupo siempre debe corregirse isquemia, ya sea mdicamente o con procedimientos de revascularizacin. GRUPO
DE RIESGO INTERMEDIO: A).Cardiopata estructural adquirida (HTA, valvular, coronaria) o congnita. Estos grupos son heterogneos, en
general son pacientes cardipatas sintomticos o no sintomticos, pero sin elementos coronarios activos ni arritmias ventriculares de
alto riesgo, se benefician de terapia farmacolgica que detiene o aminora progresin de cardiopata demostrado por evidencia (IECA,
espironolactona, bloqueadores AT2 ,BB). GRUPO BAJO RIESGO: Poblacin general con factores de riesgo cardiovasculares que
determinan finalmente aparicin de cardiopata coronaria, estructural o ambas , bajo riesgo de MS pero por nmero de afectados son
los que proporcionalmente aportan ms casos al ao de MS. Fundamental manejar FR : HTA, DM , sedentarismo, tabaquismo,
dislipidemias. Intervenciones teraputicas tanto farmacolgicas como no farmacolgicas aplicadas a esta poblacin son de alto impacto
y relativo bajo costo, por lo que hay alta costoefectividad. A nivel poblacional tambin son aplicables medidas colectivas para evitar que
aparezcan factores de riesgo, sobretodo en poblacin infantil y juvenil (evitar tabaquismo, evitar sedentarismo, alimentacin
saludable), como tambin se vislumbran intervenciones como introduccin de elementos dietticos especficos. TRATAMIENTO: SI el
paciente tiene una TV o FV, desfibrilar, para que el corazn retome su ritmo, se puede dar reanimacin cardiovascular pero con CAB no
con ABC (primero compresiones, para genera gasto cardiaco). Solo los grupos de mayor riesgo se benefician en trminos de prevencin
primaria de DI (desfibrilador implantable), el resto de los pacientes debieran recibir terapia farmacolgica que se ha demostrado que
disminuye el riesgo de muerte atribuible a MS, como amiodarona (prolonga el PRA, reduce posibilidad de fenmeno R/T) o BB, estos
ltimos adems han demostrado que reducen mortalidad global. Esta estrategia es independiente del manejo adecuado del paciente
con cardiopata demostrada con frmacos que mejoran sobrevida y progresin de enfermedad como IECA, antagonistas de
Angiotensina 2, espironolactona, etc. El desfibrilador implantable es la terapia preferida en una gran proporcin de pacientes
MANUAL DE TRABAJO DEL CURSO ENARM CMN SIGLO XXI
CURSO ENARM CMN SIGLO XXI TEL: 36246001 Pharmed Solutions Institute PGINA 186

sobrevivientes a MS. El valor de esta aproximacin en pacientes con episodio previo de MS es encontrar condiciones de alto riesgo
donde no cabe duda de la indicacin de DI y condiciones donde no hay duda de que el paciente no se beneficiar.
CASOS CLINICOS
Un varn de 26 aos de edad acudi por palpitaciones paroxsticas acompaadas de sudoracin profusa, nusea improductiva y gran
agitacin psicomotriz sin relacin con el esfuerzo. Present un deterioro inesperado de su estado general, con taquicardia ventricular
por torsade de pointes y una parada cardiorrespiratoria recuperada mediante desfibrilacin elctrica cardiaca (tres choques de 200-200
y 360 J de energa respectivamente) y administracin farmacolgica (2 g de sulfato magnsico en bolo de 2 min y luego perfusin
intravenosa pautada de 500 ml de SSF con 2 mg de sulfato magnsico, junto con 1,5 g de isoproterenol/min. Una hora despus
presenta parada cardiorrespiratoria sin recuperacin a la maniobras.

PREGUNTA
Cul es la alteracin del ritmo ms probable que ocaciono la muerte.

RESPUESTA
a.- Bloqueo AV de tercer grado.
b.- Taquicardia ventricular.
c.- Fluter auricular.
d.- Bradicardia ventricular.

CASO CLINICO
Varn de 23 aos en parada cardiorrespiratoria no recuperada tras protocolo de reanimacin cardiopulmonar avanzada, por lo que se
activa cdigo de donante en asistolia, mientras se mantiene el masaje cardaco con ventilacin mecnica y se conecta al paciente a
circulacin extracorprea. En la radiografa de trax que se realiza en el proceso de validacin pulmonar se evidencia masa mediastnica
radioopaca de localizacin inferior y media.

PREGUNTA
Cul es la etiologa ms probable de este caso

RESPUESTA
a.- Aspiracion de contenido gstrico por maniobras.
b.- Neumonia.
c.- Edema agudo pulmonar.
d.- Tromboembolismo.

CASO CLINICO
Un varn de 26 aos de edad acudi por palpitaciones paroxsticas acompaadas de sudoracin profusa, nusea improductiva y gran
agitacin psicomotriz sin relacin con el esfuerzo. Present un deterioro inesperado de su estado general, con taquicardia ventricular
por torsade de pointes y una parada cardiorrespiratoria recuperada mediante desfibrilacin elctrica cardiaca (tres choques de 200-200
y 360 J de energa respectivamente) y administracin farmacolgica (2 g de sulfato magnsico en bolo de 2 min y luego perfusin
intravenosa pautada de 500 ml de SSF con 2 mg de sulfato magnsico, junto con 1,5 g de isoproterenol/min. Una hora despus
presenta parada cardiorrespiratoria sin recuperacin a la maniobras.

PREGUNTA
Cul es la alteracin del ritmo ms probable que ocaciono la muerte.

RESPUESTA
a.- Bloqueo AV de tercer grado.
b.- Taquicardia ventricular.
c.- Fluter auricular.
d.- Bradicardia ventricular.

CASO CLINICO
Se trata de paciente masculino de 61 aos de edad el cual ingresa a urgencia por perdida del estado de despierto de forma sbita,
produciendoce cada desde su propia altura. Se iniciaron maniobras de resucitacin con recuperacin luego de dos descargas con 360 j,
presentando nuevamente perdida del estado de despierto. Tiene como antecedentes diabetes mellitus 2 desde hace 15 aos, tratado
con insulina glargina, actualmente en protocolo de dialisis peritoneal con KDOQI 5, y un infarto hace 6 meses.

PREGUNTA
Considerando la presentacin del cuadro clnico y el ECG cual es la conducta a seguir?

RESPUESTA
a.- Descarga de 360 j.
b.- Compresiones toraccicas.
c.- Administracion de amiodarona.
d.- Masaje carotideo.
MANUAL DE TRABAJO DEL CURSO ENARM CMN SIGLO XXI
CURSO ENARM CMN SIGLO XXI TEL: 36246001 Pharmed Solutions Institute PGINA 187

PREGUNTA
Cual de las siguientes factores de riesgo es el mas grave para predisponer al paciente de presentar el estado actual?

RESPUESTA
a.- Insuficiencia renal crnica.
b.- Infarto al miocardio antiguo.
c.- Diabetes mellitus de larga evolucin.
d.- Hipomagnesemia crnica.

CASO CLINICO
Se trata de paciente masculino de 71 aos de edad el cual ingresa a urgencias por perdida del estado de despierto, al ingreso se observa
con palidez generalizada, hipotrmico, sin respuesta a estimulos y sin pulso palpable, se activa el cdigo de alerta y se inicia con
compresiones toraccicas por parte de medico de guardia, se realiza dos ciclos de compresiones con leve respuesta y se observa trazo
fibrilar en ECG.

PREGUNTA
Cual es la conducta a seguir inmediata mas adecuada considerando el trazo.

RESPUESTA
a.- Desfibrilar a 360 j.
b.- Reiniciar compresiones.
c.- Administracin de amiodarona.
d.- Intubacin orotraqueal inmediata.

PREGUNTA
Considerando la etiologa de la FA, cual es la mas probable en este paciente?.

RESPUESTA
a.- Cardiopatia coronaria.
b.- Infarto agudo al miocardio.
c.- Hipertrofia ventricular.
d.- Cardiomiopatia dilatada.

PREGUNTA
Luego de 4 ciclos de compresiones y desfibrilacin el paciente presenta ritmo sinusal, cual es la razn de que solo con la descarga se
corrija el ritmo?

RESPUESTA
a.- Periodo refractario.
b.- Periodo refractario relativo.
c.- Periodo refractario absoluto.
d.- Periodo refractario con mecanismo de reentrada.

CASO CLINICO
Se trata de paciente masculino de 21 aos de edad, es ingresado a urgencias debido a que perdi el estado de alerta mientras jugaba
futbol, no referio sntoma alguno solo se cayo en la cancha, a la exploracin se observa lijera palidez de tegumentos, llenado capilar de
2 segundos, pulso lento y dbil, FC90, FR 22, TA 110/70 mmHg, responde a ordenes y refiere no recordar lo que paso. Como
antecedente de importancia refiere que ocacionalmente se ha sentido mareado desde los 12 aos, pero nunca se haba desmayado, de
nio le mencionaron que tena un soplo que se quitara posteriormente, cabe mencionar que su padre muri a los 31 aos. Se realizo
ECG en urgencias una hora despus de su ingreso.

PREGUNTA
Considerando el cuadro clnico y el ECG cual es la etiologa mas probable.

RESPUESTA
a.- Miocardiopatia hipertrfica.
b.- Sindrome de QT largo.
c.- Displasia arritmogenica.
d.- Cardiopatia coronaria.

PREGUNTA
Considerando la condicin subyacente cuales son las medidas mas adecuada para evitar otro evento del mismo orden potencialmente
mortal.

RESPUESTA
a.- Evitar eventos gatillantes.
MANUAL DE TRABAJO DEL CURSO ENARM CMN SIGLO XXI
CURSO ENARM CMN SIGLO XXI TEL: 36246001 Pharmed Solutions Institute PGINA 188

b.- Considera susceptibilidad individual.
c.- Estudiar antecedentes personales.
d.- Modificacin de estilo de vida.

CASO CLINICO
Ingresa a urgencias maculino de 21 aos de edad el cual se observa ansioso, agitado con actitud alucinada, a la exploracin se
encuentra diafortico con palidez generalizada, sbitamente el paciente pierde el estado de alerta y presenta parada cardiaca, se activa
el cdigo de alerta e inicia compresiones toraccicas, los familiares solo refieren que lo golpearon en una fiesta, 10 minutos despus
presento los sntomas. Se observo el siguiente trazo del ECG. No hay respuesta, no se detecta pulso, ni constantes vitales. Se descarga
con 360 j, se mantiene soporte por 20 minutos ms.

PREGUNTA
No se presive pulso, cual de las siguientes causas es la mas probable en este caso?

RESPUESTA
a.- Neumotorax a tensin.
b.- Sobredosis por drogas.
c.- Tromboembolismo pulmonar.
d.- Hipovolemia.

CASO CLINICO
Varn de 23 aos en parada cardiorrespiratoria no recuperada tras protocolo de reanimacin cardiopulmonar avanzada, por lo que se
activa cdigo de donante en asistolia, mientras se mantiene el masaje cardaco con ventilacin mecnica y se conecta al paciente a
circulacin extracorprea. En la radiografa de trax que se realiza en el proceso de validacin pulmonar se evidencia masa mediastnica
radioopaca de localizacin inferior y media. Se aprecian signos de broncoaspirado, por lo que se desestima la extraccin pulmonar.

PREGUNTA
Cul es la etiologa ms probable de este caso
RESPUESTA
a.- Aspiracion de contenido gstrico por maniobras.
b.- Neumonia.
c.- Edema agudo pulmonar.
d.- Tromboembolismo.

MANUAL DE TRABAJO DEL CURSO ENARM CMN SIGLO XXI
CURSO ENARM CMN SIGLO XXI TEL: 36246001 Pharmed Solutions Institute PGINA 189

2.- Checar es un
ritmo a desfibrilar?
3.- FV/TVSP
6.- Continuar RCP mientras
carga desfibrilador. Dar
una nueva descarga.
Reiniciar RCP. Admon.:
adrenalina 1mg IV repetir
cada 3-5 min durante RCP
5.- Checar es un
ritmo a desfibrilar?
4.- Dar una descarga (120-
200J-bifasico o 360J-
monofasico. Reiniciar RCP
7.- Checar es un
ritmo a desfibrilar?
8.- Continuar RCP mientras carga desfibrilador.
Dar una nueva descarga. Reiniciar RCP.
Considerar antiarrtmico, amiodarona: 300mg IV,
considerar 150 adicional o lidocana 1mg/kg.
Considerar Magnesio 1-2g IV en torsade de
pointes. Luego 5 ciclos de RCP, ir a cuadro 5
9.- Asistolia/AESP
10.- Reiniciar RCP por 5 ciclos.
Admon.: adrenalina 1mg IV,
repetir cada 3-5 min.
Considerar atropina 1mg IV en
asistolia o AESP lenta, repetir
cada 3-5 min, hasta 3 dosis
11.- Checar es un
ritmo a desfibrilar?
12.- Si asistolia, ir a cuadro 10.
Si actividad elctrica, checar
pulso: Si no hay ir a cuadro 10.
Si hay, iniciar cuidados post-
resucitacin.
13.- Cuidados post-
resucitacin
SI
NO
5 ciclos
5 ciclos
5 ciclos
ALGORITMO RCP AVANZADA
1.-Paro cardiorrespiratorio, iniciar RCP bsico

RCP BASICA Y AVANZADA:
RCP BASICA: Ante una potencial victima siempre
verificar la ausencia de respuesta (inconsciencia) y
activar inmediatamente el sistema de respuesta
medica de urgencias para asegurar la llegada de un
desfibrilador. Si hay un desfibrilador presente selo
en este momento. La RCP bsica se mantiene hasta la
llegada de un desfibrilador o hasta que reinicie
ventilaciones espontaneas. BUSCAR RESPUESTA: Ante
una potencial victima de Muerte Sbita (MS) se debe
proceder siempre de la misma manera independiente
del lugar de los hechos o los recursos disponibles en el
lugar. El primer paso es asegurarse de que la victima
realmente este inconsciente, hay que ponerla en
posicin adecuada para la RCP (decbito dorsal,
mirando hacia arriba) y tomndola de los hombros y
hablndole en voz alta al odo buscar alguna respuesta
(apertura ocular, emisin de sonidos o movimiento).
Debe evitarse realizar movimientos bruscos que
puedan producir o agravar lesiones presentes,
especialmente de la columna cervical. Si la vctima no
responde las posibilidades son que este en coma, este
en paro respiratorio o paro cardiorrespiratorio (PCR).
Para determinarlo se debe continuar con la evaluacin
en forma ordenada. PEDIR AYUDA: Como la FV es el
ritmo ms frecuente y tratable en la MS en el adulto,
se pide ayuda para asegurar la llegada de un
desfibrilador y ayuda mdica avanzada. Si el evento
fue extrahospitalario y hay otras personas presentes,
el reanimador debe identificar a una de ellas y
solicitarle que active el sistema de respuesta medica
de urgencias indicando que hay una vctima de muerte sbita y la necesidad de un desfibrilador dando la direccin exacta del evento. Si
el reanimador esta solo debe pedir la ayuda el mismo aun si esto implica dejar a la victima solo por algunos segundos. Si el paro es
intrahospitalario, el reanimador debe activar el sistema local de respuesta mdica de urgencias (timbre de alarma, cdigo azul,
etc.). : Permeabilizar la va area de la vctima. Durante el PCR la victima pierde complemente el tono muscular,
asociado al efecto de la gravedad la lengua cae hacia atrs ocluyendo la va area, siendo esta la principal causa de obstruccin. La
maniobra ms efectiva para permeabilizar la va area es extender la cabeza y elevar del mentn. Si existe o se sospecha lesin de
columna cervical solo se debe realizar la elevacin del mentn manteniendo la cabeza en posicin neutra. : Con
la va area abierta el reanimador acerca el odo a la nariz de la vctima, mirando hacia el trax de la misma, Escuchando si hay
respiracin y Sintiendo el aire exhalado en un tiempo igual o menor a 10 segundos. Si la victima tiene una respiracin adecuada el
reanimador mantiene la va area abierta y mientras espera la llegada de ayuda mdica avanzada inicia la bsqueda de la causa y
reevala peridicamente a la vctima. La presencia de respiracin adecuada implica presencia de circulacin. En ocasiones las victimas
de muerte sbita presentan movimientos respiratorios muy superficiales o respiracin agnica que no deben ser confundidos con
respiracin adecuada y el reanimador debe actuar de la misma forma que si no hubiera respiracin. Con la va area abierta el
reanimador debe mirar, escuchar y sentir (MES), buscando la respiracin. : Se debe checar ritmo cada 5 ciclos.
El pulso carotideo ha mostrado ser impreciso. Una vcma potencial de muerte sbita que efecvamente este en PCR y no recibe RCP
bsica morir

con seguridad. La FV progresa hacia asistolia rpidamente, disminuyendo la probabilidad de xito con la desfibrilacin en
un 10% con cada minuto de retraso, la RCP bsica disminuye el porcentaje a un 4% por cada minuto, aumentando el periodo en el cual
el ritmo es desfibrilable y por lo tanto reversible. Durante los primeros minutos de una muerte sbita la sangre de la victima contiene
suficiente oxigeno para cumplir con la demanda. Adems, la demanda de oxigeno es menor ya que la circulacin tambin esta
disminuida hasta 1/3 parte de lo normal. El problema principal es entonces falta de circulacin de sangre ms que falta de oxigenacin
de la misma. Por lo tanto, si la persona esta inconsciente y sin respiracin el reanimador debe iniciar la RCP con 30 compresiones
torcicas externas seguidas de 2 ventilaciones artificiales. Una excepcin para el uso precoz de la desfibrilacin es la presencia de un
PCR prolongado (>5 minutos) donde la RCP bsica, aumentan el xito de la desfibrilacin. Existe un pequeo porcentaje de pacientes
que tienen una muerte sbita de etiologa hipoxica (obstruccin de va area por cuerpo extrao) en quienes si se tiene la sospecha o
certeza se podra privilegiar las ventilaciones artificiales y empezar la RCP con estas y continuar con compresiones torcicas
externas. : Las compresiones torcicas se realizan colocando el taln de una mano en el centro del trax
entre ambas tetillas con la otra mano encima y los dedos entrelazados. Con los codos completamente extendidos se inician las
compresiones empujando fuerte y rpido. El mayor determinante de xito con la desfibrilacin es la presin de perfusin coronaria. Las
compresiones torcicas externas efectivas logran entregar una pequea pero critica cantidad de oxigeno al cerebro y corazn,
suficiente para mantener una presin de perfusin coronaria. Compresin torcica externa efectiva debe cumplir con las siguientes
MANUAL DE TRABAJO DEL CURSO ENARM CMN SIGLO XXI
CURSO ENARM CMN SIGLO XXI TEL: 36246001 Pharmed Solutions Institute PGINA 190

condiciones: 1.- Frecuencia >100 x min. 2. Profundidad entre 4-5 cm. 3.- Equilibrar el tiempo de compresin con el de descompresin
del trax (50 y 50%). 4.- Permitir el retorno del trax a su expansin total durante la fase de descompresin. 5.- Minimizar las
interrupciones en la compresin torcica externa. Las 30 compresiones torcicas son seguidas de 2 ventilaciones artificiales de 1
segundo cada una (la misma para uno o dos reanimadores). : Puede ser administrada con distintas tcnicas: 1.- Boca a
boca: Abriendo la va area el reanimador toma una respiracin normal y a continuacin sella su boca alrededor de la de la victima e
insufla lo suficiente como para elevar el trax de la vctima. 2.- Ventilacin con barreras de proteccin: Estas son dispositivos plsticos
con una vlvula unidireccional que en teora evitan la transmisin de enfermedades infectocontagiosas (no demostrado). 3.- Boca a
nariz: Se utiliza cuando no es imposible ventilar a travs de la boca (lesiones bucales, imposibilidad de abrir la boca). 4.- Mascara-
vlvula-bolsa: Logra oxigenacin y ventilacin efectivas pero requiere de entrenamiento y prctica frecuente. En los primeros minutos
de la RCP la mscara-vlvula-bolsa es igual de efectiva y segura que un tubo endotraqueal. La intubacin endotraqueal no tiene lugar en
la RCP bsica. Las compresiones torcicas externas agotan rpidamente al reanimador y cuando esto sucede disminuye notablemente
su efecvidad. Es as

que si hay dos o ms reanimadores presentes deben rotarse las compresiones torcicas cada 2 minutos para
garantizar que estas sean efectivas. Despus de la desfibrilacin (200J, 200-300J, 360J si es bifsico o cargas fijas de 360J si es
monofsico) las compresiones torcicas y ventilaciones artificiales se mantienen por 2 minutos. Si la desfibrilacin fue exitosa y se logro
un ritmo de perfusin se suspende la RCP bsica y se inicia la avanzada segn necesidad. En cambio, si la desfibrilacin no fue exitosa se
siguen alternando ciclos de compresin torcica con ventilacin artificial por 2 minutos y nuevos anlisis del ritmo por el DEA y
descargas si este las indica. RCP AVANZADO: A (Airway): asegurar la va area: En el pasado se prefera la intubacin oro-traqueal
(IOT), pero la incidencia de complicaciones es muy alta cuando es realizada por inexpertos. Las alternativas serian el Combitube
esfago-traqueal y la mascara larngea, pero no estn disponibles. La IOT permite mantener permeable la va area, aspirar
secreciones, administrar O2 en altas concentraciones, es una va alternativa para la administracin de medicacin y cuando se insufla el
manguito, protege la va area de la aspiracin. Se debe minimizar el tiempo de interrupcin de las compresiones (10seg). Si no fue
exitosa, asegurar un intervalo adecuado de compresiones y ventilaciones. Si el paciente tiene un ritmo con pulso, controlar con
saturometria. Complicaciones de OIT: ruptura de dientes laceracin traqueofarngea, ruptura traqueal, perforacin faringoesofagica,
aspiracin de contenido gstrico, intubacin bronquial. B (Breathing): comprobacin de la localizacin del tubo a travs de mtodos
clnicos (visualizacin de expansin torcica y auscultacin) y complementarios (detectores de CO2). C (Circulation): Obtener un acceso
venoso de grueso calibre (antecubital), conectar a monitor y administrar drogas segn ritmo. Cuatro ritmos producen el paro cardiaco:
la FV, TV sin pulso, asistolia y AESP. Las drogas se administran en bolo seguidas de 20 ml de solucin fisiolgica y elevar el brazo.
Algunas pueden administrarse por el tubo endotraqueal; atropina, lidocana, adrenalina y naloxona, pero a 2 veces la dosis y seguida
por 10ml de SF y 3-4 ventilaciones. Los fluidos deben ser administrados cuando se sospecha hipovolemia. Administrar bicarbonato solo
si hiperkalemia persistente. Diagnsticos diferenciales: considerar las posibles causas descompensantes o que complican la
resucitacin. Regla de las 5Hs (hipovolemia, hipoxia, H+ (acidosis), Hiper/hipocalemia, hipotermia) y las 5Ts (txicos, taponamiento
cardiaco, neumotrax a tensin, trombosis coronaria, TEP).

CASO CLINICO
Varn de 48 aos, acude a consulta al hospital y cuando se dirige a recabar su ficha de consulta cae al suelo inconsciente,
Circunstancialmente usted pasa por el lugar y se dispone a brindar auxilio e inicia los pasos del soporte vital bsico.

PREGUNTA
Cual es la conducta a seguir inmediatamente?

RESPUESTA
a.- Iniciar compresiones.
b.- Iniciar ventilacin.
c.- Activar sistema de emergencia.
d.- Verificar pulso.

CASO CLINICO
Paciente de 47 aos, con antecedente de un episodio de fibrilacin auricular en que no tuvo mayor estudio y revirti a ritmo sinusal
(RS) espontneamente. Sin terapia posterior y asintomtico. Estando en reposo, presenta paro cardaco que fue atendido
inmediatamente por familiares con maniobras bsicas de resucitacin cardiopulmonar, siendo trasladado en un lapso de 5 a 10 min al
Hospital local donde al ingreso se observa actividad elctrica sin pulso.

PREGUNTA
Cual es su conducta a seguir inmediatamente?

RESPUESTA
RESPUESTA
a.- Iniciar compresiones.
b.- Iniciar ventilacin.
c.- Activar sistema de emergencia.
d.- Verificar pulso.

MANUAL DE TRABAJO DEL CURSO ENARM CMN SIGLO XXI
CURSO ENARM CMN SIGLO XXI TEL: 36246001 Pharmed Solutions Institute PGINA 191

ESTADO DE CHOQUE:
CIENCIAS BASICAS: Definicin: Reduccin de la perfusin tisular sistmica con decremento de la disponibilidad de O2, provocando
hipoxia celular. Estado metablico en el cual las demandas energticas celulares de oxigeno, exceden el aporte. Disfuncin celular,
orgnica y sistmica. SALUD PUBLICA: Mortalidad mayor al 20%. Corresponde al 1% de las atenciones en el servicio de urgencias. En el
shock anafilctico cerca de 60% se muere por obstruccin de la va area superior, el resto por hipotensin y arritmias. PATOGENIA:
Varios factores influyen en el desarrollo como: desproporcin entre el volumen circulante y el lecho vascular. Trastorno en la
circulacin capilar. Trastorno en el intercambio gaseoso. Hipoxia de los tejidos y acidosis metablica. A 8ml/kg/min se inicia la isquemia
celular
CLASIFICACION DE CHOQUE
HIPOVOLEMICO CARDIOGENICO OBSTRUCTIVO DISTRIBUTIVO
DEFINICION Estado patolgico asociado
a procesos cuyo
denominador es la hipo
perfusin con hipoxia
celular, secundaria a la
perdida de volumen
circulante, intra o
extravascular
TAS <90mmHg (< de 30 por
debajo de la basal si se
conoce) por un periodo de ms
de 30 min., que no responde a
la administracin de lquidos y
que es secundaria a una
disfuncin cardiaca
Obstruccin mecnica al
flujo de sangre que
ocasiona una poscarga
disminuida y una
inadecuada perfusin
tisular
Estado de emergencia, para evitar dao irreversibles.
Existe una vasodilatacin, que conlleva a una
hipoperfusin tisular generalizada, porque hay poca
precarga. La falta de respuesta a manejo hdrico y/o
vasopresores, establece el diagnostico definitivo de
choque distributivo
SIGNOS
SINTOMAS
Hipotensin arterial,
hipotermia, taquicardia,
palidez de tegumentos
Hipotensin arterial,
congestin pulmonar,
ingurgitacin yugular, ritmo de
galope o tercer ruido,
congestin heptica, pulso
paradjico y/o disminucin de
la transmisin de los ruidos
cardiacos
Relacionados con isquemia
miocrdica aguda: Dolor
torcico, disnea, diaforesis,
nausea, vomito
Por lo general hipertrmico, taquicardico (>120 lpm),
taquipneico e hipotenso.
En el neurognico dependiendo nivel de lesin puede
haber hemiplejia superior con incapacidad para
movilizacin y sensibilidad de extremidades, y en
ocasiones insuficiencia respiratoria. Dao medular
bajo, hemiplejia inferior con falta de sensibilidad en
extremidades e incontinencia de esfnteres.
TIPOS Hemorrgico
No hemorrgico:
3er espacio
(pancreatitis, vasculitis,
quemados)
Perdidas
gastrointestinales
Perdidas urinarias (DM
inspida)
Importante: es la mxima
expresin de la insuficiencia
cardiaca por disfuncin
ventricular, es un sndrome
clnico que se acompaa de
disminucin persistente y
progresiva del GC,
acompaado de estado de
choque e hipo perfusin
tisular, llevado a disfuncin
celular y falla multiorganica
Sptico: hipotensin inducida por sepsis que no
responde a tratamiento a manejo de lquidos o
vasopresores
Anafilctico: reaccin aguda y generalizada de gran
severidad que puede ser potencialmente mortal.
Neurognico: Alteracin hemodinmica por
interrupcin de la va simptica, lo que genera una
vasodilatacin permanente, antecedente de trauma,
con compromiso del SNC, el GC puede estar
comprometido o no, el ms comn es el traumatismo
medular
CAUSAS Se da como consecuencia
de la perdida sbita de
volumen intravascular
El ms comn es el choque
inducido por IAM, tambin por
contusin miocrdica,
miocardiopata, rotura de
cuerdas tendinosas,
comunicacin IV, aneurisma
ventricular, arritmias. Hasta en
75-80% la disfuncin proviene
del VI
Lo ms frecuente
Tromboembolia
pulmonar, otras
neumotrax a tencin,
tamponade cardiaco,
estenosis mitral o
aortica aguda,
pericarditis obstructiva
Puede ser inducido por infecciones, reacciones
alrgicas o inflamatorias severas, el ms comn es el
sptico.
El anafilctico pude ser causado por: antibiticos,
analgsicos, alimentos, picadura de insectos, medios
de contraste.
El Neurognico puede ser causado por: accidentes
vehiculares (40%), cadas, violencia, nios durante
deporte o actividades recreativas.
RVP Aumentado (>1500
dinas/min/cm
-5
)
Aumentado Normal Disminuido
GC Disminuido (<5 L/min) Disminuido Disminuido Aumentado
PVC Disminuida (<8 cm de H2O) Aumentada (>15 cm de H2O) Normal, aumentada o
dismi
Normal o disminuida
PCP Baja (<8 mmHg) Aumentada (>15 mmHg) Normal, aumentada o
dismi
Disminuida


Es importante diferenciar entre shock sptico y sepsis. Sndrome de respuesta inflamatoria sistmica (SRIS): Cuando se presenta dos
ms de los siguientes signos; 1. Distermia: Temperatura corporal >38 0 <35. 2 Taquicardia: >90 por minuto. 3. Taquipnea: >20 por
minuto o PaCO2 <32mmHg. Leucocitos: Mas de 12,000 o menos de 4,000 con 10% de bandas. Sepsis: Cuando un paciente tiene SRIS y
un foco infeccioso evidente, con cultivo hemtico positivo o negativo. Sepsis severa: Hipotensin inducida por sepsis que responde al
manejo de lquidos y/o vasopresores. ESTRATEGIAS DE TRATAMIENTO: Accesos vasculares; importantes para la administracin de
lquidos y elementos de vigilancia. Accesos venosos perifricos, son la primera opcin para tratamiento de choque hipovolmico
(catteres 14,16 o 18), sitios preferidos safena externa, yugular externa, pliegue de ambos antebrazos. Los catteres cortos no debern
emplearse en venas centrales a menos que sea indispensable. Accesos venosos centrales, indispensables en todas las formas de
choque, las vas preferidas de acceso central son: yugular interna y subclavia. Mtodos de vigilancia, oxigenacin tisular, catter de
presin venosa central (PVC) intraarterial en aurcula derecha, para vigilancia de estado nutricional BH, QS, orina de 24 hrs, vigilancia
respiratoria, gasometra arterial. Apoyo hemodinmica: orientado a la optimizacin de la oxigenacin, mediante frmacos; 1. Terapia
inotrpica: corroborar una adecuada PVC y PCP antes de iniciar tratamiento inotrpico. Una dosis inicial de dobutamina (5-10
microgramos/kg/min), recomendacin de inicio en presencia de choque cardiognico. 2. Terapia vasopresora: de primera lnea
dopamina cuando hay RVP disminuidas y GC aumentado, cuando no existe respuesta con dosis elevadas de dopamina (>20
microgramos/kg/min), se recomienda norepinefrina o epinefrina. La noradrenalina es un medicamento vasoconstrictor potente, no
general tantas arritmias, es de primera eleccin en choque septico. La vasopresina puede ser til en pacientes refractarios a
MANUAL DE TRABAJO DEL CURSO ENARM CMN SIGLO XXI
CURSO ENARM CMN SIGLO XXI TEL: 36246001 Pharmed Solutions Institute PGINA 192

Manifestaciones de
ICC
Disnea, polipnea,
estertores
pulmonares, pltora
yugular, S3
Manifestaciones de bajo
gasto
Palidez, oliguria, llenado
capilar prolongado, reaccin
adrenrgica, piloereccin,
diaforesis, cianosis perifrica
Inestabilidad
hemodinmica
Hipotensin arterial,
PAS <90mmHg, IC < 2.2
L/min/m
2
, PCP
>15mmHg
DIAGNOSTICO CHOQUE CARDIOGENICO

CLASIFICACION DE CHOQUE
HIPOVOLEMICO HEMORRAGICO
GRADO I II III IV
PERDIDA/ML 750 750-1500 1500-2000 >2000
PERDIDA % 15% 15-30% 30-40% >40%
FC <100 >100 >120 >140
FR 14-20 20-30 30-40 >40
T/A NL NL Disminuido Disminuido
P. pulso Normal Disminuido Disminuido Disminuido
DIURESIS >30ml/h 20-30 5-15 0
Edo. mental Ansiedad Ansiedad Confusin Letargo
Rest. liquida Cristaloides Cristaloides Crista +sangre Crista +sangre
norepinefrina, y se recomienda en el choque en fase
de vasodilatacin (choque sptico). La adrenalina es
de primera eleccin en choque anafilctico, en bolos
subcutneos 3. Terapia vasodilatadora: su uso debe
ser sustentado en monitoreo invasivo avanzado, que
provea medicin de RVS. Lquidos y fluidos:
cristaloides; las mas empleadas solucin salina
isotnica y Hartman, clnicamente puede esperarse
de 100-200ml de expansin, despus de infundir 1
litro de cristaloides isotnicos. Las soluciones
hipertnicas tienen un potencial benfico como
incremento en la contractilidad miocrdica y
vasodilatacin precapilar; sin embargo, pueden
conducir a estados hiperosmolares con facilidad.
Coloides; en Mxico se cuenta con 4 como albumina
(frasco de 50cc con concentracin al 20%), gelatina
(haemacel), dextranes, almidn (hidroximetilalmidon
al 10%). Un litro de albumina al 5%, produce una
expansin de plasma de 500-1000 ml. Un litro de
almidn expande 700ml. Tanto gelatinas como almidones son inductores de ditesis hemorrgica, en forma dosis dependiente. El
dextran compromete dramticamente la hemostasis, la gelatina un impacto moderado, aunque la agregacin plaquetaria se ve
moderadamente afectada. Sangre y hemoderivados: no se ha demostrado beneficio en la perfusin tisular, al transfundir pacientes en
estado de choque con niveles de hemoglobina de 8-10mg/dl. Las siguientes condiciones clnicas podrn beneficiarse de
hemotransfusion, taquicardia importante, desaturacin venosa mezclada importante, disfuncin cardiaca, enfermedad arterial
coronaria coexistente, acidosis lctica persistente, incapacidad otros marcadores de hipoperfusin celular. No existe un nivel de
hemoglobina que indique hemotransfucion con el sustento de mejorar el aporte sanguneo a los tejidos. Las complicaciones de
hemotransfusion masiva, lleva a mayor riesgo de infecciones y reacciones anafilcticas, puede llevar a SIRA, aqu se llama TRALI, puede
dar a partir de las 6 hrs de trasnfundido. Apoyo ventilatorio; en todo momento asegurase del adecuado funcionamiento de la va area,
siendo la permeabilidad e integridad de la misma una condicin indispensable para cumplir con la meta de oxigenacin sistmica. Los
elementos que pueden orientar para el empleo de asistencia en la ventilacin son: administrar puntas nasales, mascara facial, cnula
farngea o mascarilla con reservorio en caso de contarse con ella, en cualquier situacin clnica que sugiera estado de choque. Criterios
recomendados para decidir intubacin endotraqueal, con manejo de ventilador mecnico en un paciente en estado de choque;
hipercarbia >32mmHg, taquipnea >20 rpm, cianosis distal con alguno de los elementos anteriores, acidosis respiratoria con pH=7.2,
paciente con relacin PaO2/FiO2 <150, paciente con sospecha de quemadura de va area. MARCADORES SERICOS DE CHOQUE:
Lactato; es un reflejo de hipo perfusin tisular, producto de la glicolisis anaerobia, valores por arriba de 4mmol/l, se asocia con un 90%
de mortalidad, por arriba de 2.5 mmol/l alta incidencia de falla orgnica mltiple (FOM), su medicin nos sirve como factor pronostico.
Dficit de base; aproximacin global del grado de acidosis e indirectamente el grado de perfusin tisular, nos orienta sobre la severidad
de choque: a) 2-5 leve, b)-6 a -14 moderado, c) mayor a -15.

CASO CLINICO
Se trata de paciente masculino de 23 aos de edad el cual ingresa a urgencias presentando orificio de entrada en flanco derecho, por
arma de fuego, sin orificio de salida, a la exploracin fsica se observa con FC 125 lpm, llenado capilar lento, FR 31 rpm, se observa
ansioso con tendencia a la confusin, su gasto urinario fue de 20 ml/h.

PREGUNTA
Considerando las manifestaciones clnicas observadas asi como las variables vitales, en que clase se encuentra el paciente?

RESPUESTA
a.- Clase I.
b.- Clase II.
c.- Clase III.
d.- Clase IV.

PREGUNTA
Cual es la conducta para mantener el volumen circulante mas adecuada?

RESPUESTA
a.- Solucin salina 10 ml/kg/hora.
b.- Solucin hartman 15ml/Kg/hora.
c.- Crioprecipitados.
d.- Concentrado eritrocitario y cristaloide.

PREGUNTA
Una hora despues, el paciente presenta los siguientes constantes vitales, FC 105 lpm, FR 21 rpm, gasto urinario de 35 ml/h, se
encuentra ansioso, pero no confundido, la hemoglobina fue de 12 y hematocrito de 37, plaquetas de 120,000, asi como DHL 312, AST
435, tiempos de coagulacin levemente incrementados, cual es la conducta a seguir en este momento?
MANUAL DE TRABAJO DEL CURSO ENARM CMN SIGLO XXI
CURSO ENARM CMN SIGLO XXI TEL: 36246001 Pharmed Solutions Institute PGINA 193

RESPUESTA
a.- Mantener observacin estrecha.
b.- Preparar para laparatomia exploratoria.
c.- Incrementar volumen circulante con paquete globular.
d.- Vasopresores, volumen y LAPE.

PREGUNTA
Se realizo LAPE con reparacin heptica con buena respuesta, fue necesario crioprecipitados, plaquetas y concentrado eritrocitario, se
administro 3 gramos de gluconato de calcio, cual es la razn de esta indicacin?

RESPUESTA
a.- Alteracin de coagulacin.
b.- Mantener estable la membrana.
c.- Evitar arritmias.
d.- Disminuir la adhesividad plaquetaria.

CASO CLINICO
Se trata de paciente femenino de 48 aos de edad, la cual fue ingresada posterior a accidente automovilstico y volcadura, era copiloto
y fue la nica sobreviviente, se observa confundida, desorientada, agitada, se presenta diaforesis, con palidez de mucosas, su llenado
capilar lento, FC de 138 lpm, FR 34 rpm, TA 100/65 mmHg, minutos despus se estableci un Glasgow de 8, distensin abdominal e
inestabilidad toracicca, su gasto urinario fue de 20 ml/h, Campos pulmonares con matidez a la percusin bibasal, no hay ingurgitacin
yugular, pero hay equimosis importante sobre area del cinturn de seguridad, en la radiografia porttil se observo fractura de 5
costillas del lado izquierdo.

PREGUNTA
Cual es la medida inmediata mas adecuada en este momento?

RESPUESTA
a.- Intubacin endotraqueal.
b.- Solucin hartman, vasopresores.
c.- Solucion salina hipertnica.
d.- Sedacion y relajacin muscular.

PREGUNTA
Posterior al decisin anterior se administra 3 paquetes de concentrados eritrocitarios, 3 paquetes de plasma fresco y 2 litro de solucin
hartman, sin embargo el gasto urinario es de 25 ml/hr, la tensin arterial se mantiene en 95/60 mmHg durante la primera hora de
estancia, cual es la conducta a seguir mas adecuada?

RESPUESTA
a.- Albumina y expansores.
b.- Infusion de dopamina 5 g/Kg/min.
c.- Infusion de dobutamina 10 g/kg/min.
d.- Crioprecipitados y gluconato de calcio.

PREGUNTA
Cual es el objetivo mas importante de la conducta farmacolgica tomada anteriormente?

REPUESTA
a.- Mantener el gasto urinario.
b.- Incrementar la fraccin de eyeccin.
c.- Mantener la tensin arterial permisible.
d.- Favorecer el inotropismo.

PREGUNTA
El paciente presenta estabilidad en la tensin arterial 95/70 mmHg y gasto urinario de 35 ml/hr, se prepara para laparatoria
exploratoria, tomando en cuenta la decisin farmacolgica anterior, cual de los siguiente efectos no favorece al caso?

RESPUESTA
a.- Incremento mayor del 25 % sobre la tensin arterial basal.
b.- Aumento del flujo sanguneo renal.
c.- Disminucin de la resistencia vascular perifrica.
d.- Disminucin de la demanda de O2 del miocardio.

CASO CLINICO
Se trata de paciente masculino de 19 aos de edad el cual ingresa a urgencias posterior a accidente automovilstico, refiere cefalea y
dolor lumbar leve, permaneci en observacin, se coloco catter perifrico 18 fr y solucin glucosada al 5 % para 8 hrs, a las 4 horas
MANUAL DE TRABAJO DEL CURSO ENARM CMN SIGLO XXI
CURSO ENARM CMN SIGLO XXI TEL: 36246001 Pharmed Solutions Institute PGINA 194

posterior al ingreso refiere ansiedad, sensacin de mareo, a la EF se observa palidez de tegumentos, diaforesis, torax sin datos
patolgicos, abdomen con ruidos peristlticos disminuidos, SV con TA 100/70 mmHg, FC 105 lpm, FR 32 rpm, no se apresia compromiso
oseo.

PREGUNTA
Se realizan realiza QS, BH, RX de torax y abdomen, Hb 10.1, Hto 30, Rx de abdomen con niveles hidroaereos, imagen despulida, cual es
la conducta a seguir ms adecuada?

RESPUESTA
a.- Solucin hartman 10 ml/kg/hora.
b.- Solucin salina 0.45 % 10 ml/kg/hora.
c.- Solucin mixta 10 ml/kg/hora.
d.- Solucin glucosada al 10 % 10 ml/Kg/hora.

PREGUNTA
Posteriomente se administra 3 concetrados eritrocitarios y 2 paquetes de plasma fresco, la tensin arterial se mantiene estable en
100/70 mmHg sin embargo la ursis fue de 40 ml/hora, se observa auscencia de ruidos peristlticos, se observa estado de conciencia
adecuado, alerta y tranquilo, cual no es un objetivo primordial en este momento del caso?

RESPUESTA
a.- Elevar la tensin arterial media por arriba de 95 mmHg.
b.- Manterne la hemostasia con gluconato de calcio.
c.- Estabilizar hemostasis por sospecha de hemorragia.
d.- Mantener un adecuado volumen urinario.

PREGUNTA
Cual es el origen menos probable de la perdida del volumen circulante del caso actual?

RESPUESTA
a.- Contusin cardiaca.
b.- Rotura esplenica.
c.- Hemorragia heptica.
d.- Hemorragia retroperitoneal.

PREGUNTA
Se realizo tomografa donde se confirmo hemorragia retroperitoneal y lactato elevado, cual de las siguientes complicaciones de alto
volumen de reposicion es menos frecuente?

RESPUESTA
a.- Acidosis hipernatremica.
b.- Sndrome de fuga capilar.
c.- Perdida de la estabilidad de un coagulo
d.- Disminucin de la precarga con aumento de FE.

PREGUNTA
Previo manejo quirrgico se mantiene al paciente en hipotensin permisiva, cual es el parmetro para este paciente?

RESPUESTA
a.- 100 mmHg.
b.- 90 mmHg.
c.- 80 mmHg.
d.- 70 mmHg.

CASO CLINICO
Mujer de 34 aos sin antecedentes patolgicos, no hbitos txicos, no alergias a frmacos conocidas. Present cefalea tensional en
tratamiento con cido acetilsaliclico con episodios de urticaria aguda en relacin con el consumo de marisco, tratado y remitido hace
24 hrs. 4 horas despus apareci dolor faringeo, tumefaccin facial, de manos y brazos, con disminucin de la diuresis. A las pocas
horas de la hospitalizacin present una alteracin hemodinmica con taquicardia sinusal a 140 pulsaciones por minuto e hipotensin.
PREGUNTA
Considerando el cuadro clnico cual es la conducta inmediata a seguir, la paciente se encentra con vas de acceso venoso?

RESPUESTA
a.- Administracin de volumen con cristaloides.
b.- Administracion de aminas vasoactivas.
c.- Colocacin de catter largo.
MANUAL DE TRABAJO DEL CURSO ENARM CMN SIGLO XXI
CURSO ENARM CMN SIGLO XXI TEL: 36246001 Pharmed Solutions Institute PGINA 195

d.- Administracin de epinefrina.

PREGUNTA
Posteriormente se deterior el nivel de conciencia y se produjo un paro respiratorio que precis intubacin y ventilacin mecnica. Se
consigui la estabilizacin hemodinmica con dopamina y expansores de volumen, crioprecipitados y plasma fresco, cual es la
complicacin ms probable?

RESPUESTA
a.- Coagulacion por hemodilucin.
b.- Sindrome compartimental.
c.- Coagulacion intravascular por consumo.
d.- Coagulopatia dilucional.
PREGUNTA
Considerando el cuadro clnico y la evolucin del caso, cual de las siguientes causas es la menos probable?

RESPUESTA
a.- Reacciones a frmacos.
b.- Reacciones a alimentos.
c.- Reacciones transfusionales.
d.- Reacciones a antgenos inhalados.

CASO CLINICO
Paciente varn de 57 aos con larga evolucin en cuidados intensivos por sndrome de disfuncin multiorgnica (SDMO) y sndrome de
respuesta inflamatoria sistmica (SRIS) secundarios a proceso postquirrgico. Se trata de un paciente gastrectomizado, que desarroll
en el post-operatorio una doble fstula pleural y digestiva con colocacin de prtesis endoscpica y posterior reapertura de fstula. Ha
tenido simultneamente diversos procesos infecciosos sistmicos (empiema, peritonitis, herida quirrgica y respiratoria) tratados
empricamente o mediante antibiograma. Adems ha presentado repercusin orgnica persistente con necesidad de soporte
vasopresor para control hemodinmico y respiratorio sin posibilidad de avanzar.

PREGUNTA
Considerando las caractersticas actuales que se observan en el paciente que tipo de choque se observa actualmente?

RESPUESTA
a.- Choque distributivo.
c.- Choque restrictivo.
d.- Choque sptico.
c.- Choque oculto.

PREGUNTA
El da 11 de estancia en cuidados intensivos se introduce tratamiento con linezolid por va intravenosa, 600mg/12h por aislamiento de
Enterococcus faecium resistente a meticilina, sensible a linezolid, en el empiema pleural bilateral. El resto de tratamiento es:
enoxaparina 40mg/24h (11 das), propofol 300mg/h y meperidina 15mg/h en perfusin continua (como sedoanalgesia) (10 das),
imipenem 1g/8h (10 das), amikacina 1,2g/24h (5 das). A las 48 h del inicio de linezolid se observan crisis comiciales en relacin con
movimientos tonicoclnicos preferentemente de extremidades superiores y tronco sin llegar a generalizarse, transitorios (pocos
minutos de duracin) y repetidos a lo largo del da. Considerando la evolucin y los nuevos sntomas cual es su impresin?

RESPUESTA
a.- Choque anafilctico.
b.- Reaccion adversa a medicamento.
c.- Choque sptico.
d.- Sindrome serotoninergico maligno.

PREGUNTA
Se observa tambin hiperreflexia, discreta acidosis metablica y empeoramiento grave de la fiebre, leucocitosis, marcadores de
inflamacin y estado hemodinmico (hipotensin). Ante la sospecha de RAM se sustituye meperidino por fentanilo y se aade
midazolam, 10mg/h, en perfusin continua al esquema de sedoanalgesia para la mejora del cuadro y noradrenalina para el soporte
hemodinmico. Considerando las variable vitales, hipotensin refractaria cual de las siguientes medidas es mas adecuada?

RESPUESTA
a.- Incrementar aminas vasoactiva.
b.- Hemodialisis.
c.- Retirar linezolid y administrar metilprednizolona.
d.- Concentrado eritocitario y plasma fresco congelado.

PREGUNTA
MANUAL DE TRABAJO DEL CURSO ENARM CMN SIGLO XXI
CURSO ENARM CMN SIGLO XXI TEL: 36246001 Pharmed Solutions Institute PGINA 196

Los cultivos microbiolgicos fueron negativas con la excepcin de los drenajes torcico y abdominal, en los que se aisl Candida spp. Se
administro anfotericina B liposomal en dosis de 4mg/kg/da. Linezolid fue tambin suspendido y se paut daptomicina, 350mg/24h.
Tras el cambio de tratamiento las mioclonas e hiperreflexia mejoraron antes de 24 h, hasta desaparecer por completo al segundo da.
La fiebre, leucocitosis e hipotensin mejoraron tambin progresivamente, aunque la acidosis metablica persisti. Cual es la
complicacin mas probable?

RESPUESTA
a.- Dudoso por acidosis metabolica.
b.- Coagulopatia por dilucin.
c.- Alergia al frmaco.
d.- Falla organica multiple.

PREGUNTA
El diagnstico de sndrome serotoninrgico suele ser difcil de realizar por la presencia de sntomas inespecficos y comunes a otras
entidades. Esta problemtica puede ser todava mayor en el caso de un paciente crtico y polimedicado como el que nos ocupa, donde
los sntomas pueden ser atribuidos a mltiples factores de la enfermedad o del abordaje farmacolgico. Cual es la base principal para
diagnosticar este caso?

RESPUESTA
a.- Inicio brusco.
b.- Recuperacin tras suspensin del frmaco.
c.- La presencia de fiebre, hiperreflexia y acidosis metabolica.
d.- Criterios diagnsticos de Sternbach y Dunkley.

CASO CLINICO
Se trata de un varn de 74 aos que ingres por ebre y deterioro del estado general de meses de evolucin. Como antecedentes
personales, cabe destacar gastritis crnica atrca, cardiopata reumtica con afectacin artica y mitral y brilacin auricular con
tratamiento anticoagulante. En la exploracin fsica no haba alteraciones signicativas. En la analtica, a resear una creatinina de 4,17
mg/dL no conocida y un INR de 7. En los hemocultivos realizados, crecieron en 2/3 L. monocytogenes. En el estudio de la bacteriemia,
se solicit un ecocardiograma transtorcico que puso de maniesto una verruga en la vlvula artica, con fraccin de eyeccin
deprimida, KDOQ III, cual es la conducta farmacolgica mas adecuada a seguir?

PREGUNTA
a.- Administrar ampicilina y vancomicina.
b.- Administrar ampicilina y gentamicina ajustada.
c.- Administrar imipenem y amikacina.
d.- Administrar ceftriaxona e imipenem.

PREGUNTA
Se complet el estudio con una RMN craneal, en la que se observaron mltiples infartos cerebrales, y con una ecografa abdominal en
la que se apreci un hgado de estasis. La evolucin fue mala, desarrollando insuciencia cardiaca por lo que se decidi realizar ciruga
cardiaca el 7 da. En la intervencin, se encontraron vegetaciones y abscesos que afectaban tanto a la vlvula artica como a la mitral,
el paciente presento hipotensin refractaria, sin respuesta a aminas vasopresoras, hipotermina, piel marmrea. Cual es el tipo de
choque mas probable en este momento del caso?

RESPUESTA
a.- Choque cardiogenico.
b.- Choque sptico.
c.- Choque restrictivo.
d.- Choque oculto.

CHOQUE SEPTICO
Femenino de 63 aos, con antecedentes de hipertensin arterial (HTA), diabetes mellitus (DM) tipo II, dislipidemia, infecciones urinarias
y clicos nefrticos de repeticin, que presenta un cuadro de cinco meses de evolucin de dolor abdominal de predominio en fosa ilaca
izquierda, vmitos y fiebre intermitente. Acude en diversas ocasiones a Urgencias, siendo diagnosticada de infeccin urinaria y clico
renoureteral. Recibiendo manejo farmacolgico empirico a dosis y tiempo adecuado, debido a la persistencia del dolor se realiza
ecografa abdominal, objetivndose ndulo en segmento IV heptico y quistes renales corticales.

PREGUNTA
Ante dichos hallazgos cual es la conducta mas adecuada a seguir?

RESPUESTA
a.- Bh completa, QS, EGO, Hemocultivo y urocultivo.
b.- TAC de trax, abdomen y pelvis con contraste.
c.- IRM-F torax y abdomen con marcadores inflamatorios.
d.- Biopsia heptica y renal.
MANUAL DE TRABAJO DEL CURSO ENARM CMN SIGLO XXI
CURSO ENARM CMN SIGLO XXI TEL: 36246001 Pharmed Solutions Institute PGINA 197

PREGUNTA
A los dos meses acude de nuevo a Urgencias por dolor intenso en hipocondrio derecho y mal estado general. En la EF destaca signo de
Murphy positivo e hipotensin severa, creatinina 3,68mg/dl, urea 96mg/dl, transaminasa glutmico oxalactica (GOT) 55 UI/l,
transaminasa glutmico-pirvica (GPT) 92 UI/l, protena C reactiva (PCR) 451mg/dl. Se realiza TAC de abdomen sin contraste: probable
colecistitis, elevacin de hemidiafragma derecho, atelectasia basal derecha, hemangiomas hepticos, granuloma esplnico y quistes
renales. La paciente presenta deterioro significativo hemodinamico, con obnulizacin, hipotensin, taquicardia, taquipneica, mal
perfundida con signos de abdomen agudo cual es la conducta a seguir?

RESPUESTA
a.- Preparar para LAPE.
b.- Iniciar dopamina.
c.- Iniciar dobutamina.
d.- Ingresar a UCI.

PREGUNTA
Se realizan laboratorios con los siguientes resultados: hemoglobina 8,7g/dl, leucocitos 2.900 (neutrfilos 77%), actividad de
protrombina 54%, PCR 359mg/ml, procalcitonina 3,8ng/dl, cido lctico 18mg/dl, creatinina 2,9mg/dl, urea 88mg/dl, GOT 184 UI/l, GPT
111 UI/l, fosfatasa alcalina 229 UI/l, gasometra arterial pH 7,28, pCO2 41; pO2 66, HCO3 19, saturacin arterial de O2 91%, cual es la
conducta a seguir menos apropiada ante el estado de choque sptico del caso?

RESPUESTA
a.- Flunitracepam y pancuronio e intubacin orotraqueal con conexin a ventilacin mecnica.
b.- Estabilizacin hemodinmica con cristaloides y perfusin de FVA (noradrenalina y dopamina).
c.- Correccin de coagulacin con concentrado plaquetario, crioprecipitados y plasma fresco.
d.- Correccion intensiva de valores gasomtricos y perfusin de FVA (dobutamina y vasopresina).

PREGUNTA
En LAPE se observa lquido purulento con membranas fibrinopurulentas generalizadas, especialmente en espacio subfrnico y flanco
derecho, congestin de asas y peritoneo, no encontrndose perforacin de vscera hueca y vescula normal. Se realizan lavados y
colocacin de drenaje subfrnico derecho. Se inici antibioterapia emprica con piperacilina-tazobactam y a las 48 horas, por mala
evolucin clnica con fiebre persistente e hipoxemia importante, se asocia amikacina. Cual de las siguientes diagnosticos diferenciales es
mas probable considerando los hallazgos, evolucin y antecedente de DM?

RESPUESTA
a.- Hemangioma, hematoma,
b.- Metstasis, tumor necrtico,
c.- Quiste hemorrgico, quiste hidatdico complicado.
d.- Abseso heptico amibiano E. coli Klebsiella pneumoniae.

CASO CLINICO
Mujer de 66 aos que ingresa por cuadro de shock sptico secundario a pielonefritis aguda derecha con disfuncin multiorgnica
(DMO). Requiri apoyo hemodinmico, terapia sustitutiva renal y ventilacin mecnica prolongados. Precis dobutamina (20
mcg/kg/min) y noradrenalina (2-2,5 mcg/kg/min) durante una media de 6 semanas. Recibi adems terlipresina (1mg/4h) y azul de
metileno. Se administr hidrocortisona y antibioterapia dirigida. Las dosis elevadas y prolongadas de aminas vasoactivas provocaron
vasoconstriccin distal intensa con necrosis isqumica digital que precis la amputacin de todos los dedos de los miembros inferiores.
La retirada de las medidas de soporte orgnico fue lenta as como la recuperacin posterior. La estancia en UVI fue de 120 das. En
planta comenz con dolor en ambas caderas que limit la rehabilitacin. No haba antecedentes de traumatismo o cada. Las pruebas
de imagen informaron de lesiones compatibles con necrosis avascular de caderas.

PREGUNTA
Cuales de los siguientes criterios diagnosticos es menos til para identificar sepsis?

RESPUESTA
a.- PAM < 60 mmHg PAS < 90 mmHg.
b.- Hipertermia y/ hiportermia.
c.- Volumen urinario <20 ml/hora.
d.- Velocidad de llenado capilar.

PREGUNTA
Cuales son los valores analticos es de poca utilidad dentro de las primeras horas del padecimiento para confirmar el diagnostico de
SRIS y los procesos spticos, adems de los datos clnicos?

RESPUESTA
a.- Hiperglucemia > 120 mg/dl sin DM.
b.- Procalcitonina.
c.- Leucopenia < 4,000.
MANUAL DE TRABAJO DEL CURSO ENARM CMN SIGLO XXI
CURSO ENARM CMN SIGLO XXI TEL: 36246001 Pharmed Solutions Institute PGINA 198

d.- Proteina C reactiva 2DS mayor a lo normal.

PREGUNTA
Cual de los siguientes analticos no es un elemento del sistema PIRO para estimar pronostico del paciente?

RESPUESTA
a.- Grado de agresin de la infeccin.
b.- Velocidad de instauracin.
c.- Numero de rganos con disfuncin
d.- Intensidad de la respuesta.

PREGUNTA
Cul de las siguientes no es una meta adecuada en el tratamiento de la sepsis y choque sptico?

RESPUESTA
a.- Presion venosa central (PVC) 8-12 mmHg.
b.- Presion arterial media (PAM) > 65 mmHg.
c.- Gasto urinario (GU) 0.3 ml/kg/hr.
d.- Saturacion de sangre venosa (SATVO2) > 70 %.

PREGUNTA
Cual de los siguientes criterios de reanimacin del paciente con sepsis grave se encuentra fuera de rango?

RESPUESTA
a.- Soluciones cristaloides > 1000 ml/30 min 300 a 500 ml coloides.
b.- Trasfusin de concentrados eritrocitarios para lograr un hematocrito > de 30 % con ScvO2 baja.
c.- Administracion de plasma fresco congelado con elevacin de TP, INR TPT.
d.- Perfusion de dobutamina 50 mcg/kg/min

PREGUNTA
Cual de los siguientes parmetros guarda mayor relacin de la principales complicaciones sin embargo menos especifico para vigilar al
caso clnico actual?

RESPUESTA
a.- Proteina C reactiva.
b.- Hiperlacticidemia.
c.- Procalcitonina.
d.- Creatinina serica

PREGUNTA
Cual de los siguientes parmetros guarda menor sensibilidad en relacin con el sndrome de respuesta inflamatoria sistmica?

RESPUESTA
a.- Proteina C reactiva +/-1DS mayor.
b.- INR > 1.5
c.- TTPa > 60
d.- Trobocitopenia < 100,000 celulas/mm3.

CASO CLINICO
Se trata de una mujer de 58 aos que ingres por ebre y cefalea de aparicin brusca. Dentro de sus antecedentes personales,
destacaba la presencia de prtesis mecnica mitral, artica secundaria a valvulopata reumtica, y nefrectoma derecha. Se encontraba
recibiendo tratamiento anticoagulante. A la exploracin fsica, nicamente llamaba la atencin cierto grado de rigidez nucal. En la
analtica inicial, leve deterioro de su insuciencia renal de base (creatinina de 1,83 mg/dL).

PREGUNTA
Ante tales manifestaciones, se realiz una TAC craneal as como puncin del LCR que no presentaron alteraciones. Considerando los
antecedentes y estado actual cual es la conducta a seguir para establecer un agente etiolgico.

RESPUESTA
a.- Ecocardiograma transesofagico.
b.- RMN craneal por sntomas neurolgicos.
c.- Cultivo con antibiograma.
d.- BH y tiempos de coagulacin.



MANUAL DE TRABAJO DEL CURSO ENARM CMN SIGLO XXI
CURSO ENARM CMN SIGLO XXI TEL: 36246001 Pharmed Solutions Institute PGINA 199

CASO CLINICO
Se trata de femenino de 27 aos de edad originaria de la sierra de Oaxaca, se encuentra en hospitalizacin en medicina interna, debido
a que presenta 24 horas con exantema, hipotensin y fiebre, a la exploracin fsica se observa lesin en miembro plvico, dicha lesin
es ulcerosa, sin eritema, no hay irritacin ni secreciones, pero si hay presencia de exantema en palmas, conjuntivas y mucosa bucal,
adems descamacin en las puntas de los dedos, los resultados de laboratorios reportan creatinina de 2.7 aspartato aminotransferasa
de 250 U/L, alanina aminotransferasa de 328 U/L, aun no se cuenta con resultados de los cultivos, se instaura tratamiento antibitico y
reanimacin con liquidos y vasopresores.

PREGUNTA
Cual es el diagnostico mas probable en este caso.

RESPUESTA
a.- Choque por Fiebre Tifoidea
b.- Choque toxico estafiloccico.
c.- Choque toxico estreptoccico.
d.- Choque toxico botulnico.

CRISIS HIPERTENSIVA:
CIENCIAS BASICAS: Por convencin los valores de tensin arterial (TA) severamente elevados se definen por tensin arterial sistlica
(TAS) mayor a 180 mmHg y tensin arterial diastlica (TAD) mayor de 120 mmHg, este corte es meramente arbitrario, con poca
relevancia para el mdico, porque la mayora de los pacientes no requerir

tratamiento urgente para reduccin de la TA. De forma


contraria paciente con elevaciones menores, que no renen criterios de HTA severa pueden requerir terapia antihipertensiva inmediata
para situaciones de riesgo como en los aneurismas disecantes de aorta. Las crisis hipertensivas incluyen a las emergencias y urgencias.
Emergencia hipertensiva: Presencia de dao de rgano agudo, que requiere rpido descenso de TA, en un intervalo de horas.
Usualmente requiere internacin en unidad de cuidados crticos con medicaciones por va parenteral. En estos casos la TA cumple un
rol patognico en la gnesis y progresin del cuadro clnico. Debemos recalcar que la emergencia est denida por la gravedad del
cuadro clnico y no por los valores de TA, usualmente la TAD es mayor 120 mmHg. Los rganos que son afectado ms frecuentemente
en emergencias hipertensivas son el cerebro (EVC 24.5%, encefalopata 16.3%, hemorragias intracerebrales o subaracnoideas 4.5%),
corazn (EAP 22.5%, ICC 14.3%, IAM 12.0%, diseccin aortica 2.0%), grandes vasos, rin y tero gravdico (eclampsia 4.5%,
preeclampsia grave), siendo ms frecuente la afectacin nica en el 83% de los casos, pero en porcentaje variable se pueden presentar
ms de un dao de rgano blanco en contexto de emergencias hipertensiva. Urgencia hipertensiva: cuando se produce una elevacin
brusca de la presin arterial (PA), en relacin a las cifras tensinales habituales del paciente o cuando la PA 210/120 mmHg (algunos
consensos actuales hablan de sistlicas 180 mmHg), pero sin sntomas especficos ni dao de rganos diana. El objetivo teraputico
sera reducir gradualmente la PA (en 24-48h) con medicacin oral ya que si se baja muy rpidamente puede favorecerse la
hipoperfusin de rganos diana. Debe distinguirse de una pseudocrisis hipertensiva, que es la elevacin de la PA reactiva a una
situacin de stress o dolor. Hipertensin severa aislada: Elevacin de la TAD >120 mmHg, que requiere descenso de valores de TA en
lapso de das con medicacin y usualmente sin internacin. Por definicin se debe establecer previamente la ausencia de dao de
rgano blanco, hay poca evidencia que el control de la TA, en un lapso de tiempo corto sea beneficiosos. SALUD PBLICA: La
Hipertensin Arterial (HTA) es un proceso crnico que presenta una prevalencia de ms del 25% en la poblacin occidental adulta1.
Aproximadamente un 1-2% de los pacientes hipertensos desarrollaran una crisis hipertensiva (urgencia o emergencia) en algn
momento de su vida. El tratamiento y el adecuado seguimiento de estos pacientes aumentan su esperanza de vida y disminuye la
incidencia de complicaciones. PATOGENIA: T/A = (gasto cardiaco) (resistencias vasculares perifricas). Para que la presin arterial
aumente se requiere desequilibrio en esta relacin, como falla de mecanismos autorreguladores, alteraciones en propiedades
contrctiles del musculo arteriolar, activacin del sistema SRAA, vasoconstriccin y aumento de RVP. El rpido desarrollo de dao de
rgano agudo de las emergencias hipertensivas es atribuido a la falla en el mantenimiento de la normal autorregulacin de los lechos
vasculares de rganos de choque (cerebro, rin, retina y corazn) y por otro lado un aumento abrupto de las resistencias vasculares
sistmicas. Conduciendo a una injuria endotelial con necrosis fibrinoide de arteriolas. Establecindose el ciclo isquemia, depsito
plaquetario, y falla de la autorregulacin con deterioro clnico del paciente. Los gatillos especficos de este fenmeno no son conocidos.
DIAGNOSTICO: Historia clnica: historia de la HTA (edad de inicio, tiempo de evolucin, grado de control, cifras basales, tratamiento
seguido, sntomas acompaantes), momento del inicio de la clnica, evolucin hasta el momento del ingreso, sntomas
asociados. Antecedentes personales: hbitos higinico-dietcos (alcohol, drogas), enfermedades asociadas, factores de riesgo
cardiovascular, repercusin sobre rganos diana, tratamientos. Antecedentes familiares: HTA, enfermedades cardiovasculares,
enfermedades renales, obesidad, dislipemia. Exploracin fsica: Priorizar en primer lugar la bsqueda de dao a rgano blanco.
Medicin de la PA. Peso, talla, ndice de masa corporal (IMC) y permetro abdominal. Exploracin sistemtica (cardiovascular,
pulmonar, abdominal, neurolgica, ocular, cutnea, cuello y palpacin tiroidea). Palpacin de pulsos perifricos (en las cuatro
extremidades) y centrales valorando simetras y amplitud. Fondo de ojo (hemorragias, exudados blandos, papiledema). Pruebas
complementarias: En urgencia hipertensiva: sistemtico de orina, ECG (isquemia, arritmia, hipertrofia VI), Rx posteroanterior y lateral
de trax. En emergencia hipertensiva: hemograma, bioqumica con CPK, CPK-MB, troponina I (si existe sospecha de SCA), sistemtico de
orina, ECG, gasometra arterial (GSA) (si existe sospecha de EAP o IC), fondo de ojo (grado de retinopata), Rx posteroanterior y lateral
de trax, TAC craneal (si existe sospecha de ictus), ecocardiograma (si hay cardiopaa o sospecha de disfuncin cardiaca), TAC
tracoabdominal (si existe sospecha de aneurisma disecante de aorta). TRATAMIENTO: Si dao a rgano blanco: Emergencia
hipertensiva, Objetivo: Disminuir la PAM en un 25% en un periodo comprendido entre minutos y 2 horas o hasta niveles seguros
(excepto en la diseccin artica), si el paciente estable continuar reduccin hacia valores normales dentro de 8-24 hrs de admisin.
Tratamiento inmediato y hospitalizacin. Protocolo A, B, C: soporte vital avanzado (SVA) si es necesario. Mantener va area permeable
y oxigenoterapia. Pulsioximetra, monitorizacin de la PA, canalizacin de va venosa perifrica: 1.Labetalol (diseccin aortica, IAM,
sindromes coronarios agudos): bloqueador mixto ,, dosis de impregnacin 20mg, seguida por una dosis incremental de 20 a 80mg
MANUAL DE TRABAJO DEL CURSO ENARM CMN SIGLO XXI
CURSO ENARM CMN SIGLO XXI TEL: 36246001 Pharmed Solutions Institute PGINA 200

con intervalos de 10 min hasta conseguir el efecto deseado. Reducen la mortalidad asociada a arritmias. 2. Nicardipina (encefalopata
hipertensiva, hemorragia intracraneal, EVC): es un calcio antagonista con alta selectividad vascular y fuerte actividad vasodilatadora
cerebral y coronaria. La infusin inicial es de 5mg/h con aumentos de 2.5mg/h cada 5min, hasta un mximo de 15 mg/h.
3.Nitroprusiato (EVC, isquemia, encefalopata, hemorragia subaracnoidea): es un vasodilatador arterial y venoso que disminuye la
postcarga y precarga. IV de 0.5.10 mg/kr/min. Es extremadamente potente, til monitoreo intraarterial, puede generar toxicidad por
cianuros a altas dosis o por el uso prolongado, til en casi todas las crisis hipertensivas, puede provocar aumento de la presin
intracraneana. 4. Nitroglicerina (ICC aguda o EAP, IAM, sndromes coronarios agudos): es un venodilatador potente. La dosis es 5
mcg/min con un incremento de hasta 20mg. 5. Hidralacina (embarazo): vasodilatador directo de accin predominante arteriolar, droga
de eleccin en crisis hipertensivas en embarazo. IM de 10-50mgs. Eclampsia, metildopa, MgSO4. NO dao a rgano blanco: Urgencia
hipertensiva: Si PAS 210 mmHg o PAD 120 mmHg. Objetivo: Disminuir la presin arterial media (PAM) en un 20% en un periodo de
24-48 horas o disminuir la PAD a valores < 120 mmHg. La disminucin debe ser gradual a fin de prevenir isquemia orgnica (cardiaca,
cerebral). Reposo 5-10 minutos en lugar tranquilo y en decbito supino. Valorar ansioltico suave (benzodiacepina) si
ansiedad/nerviosismo. Comprobar el nivel de PA. Es importante saber si el paciente estaba recibiendo tratamiento
antihipertensivo previo o no: Si estaba a tratamiento y lo ha abandonado, hay que reiniciarlo de nuevo, pero si contina con la
medicacin hay que aumentar la dosis del que toma o asociar nuevos frmacos. Si no est

a tratamiento y es necesario medicacin


antihipertensiva, debe emplearse un frmaco de accin lenta (ej. captopril) evitando los de accin rpida porque pueden
inducir isquemia en los rganos diana (nifedipino sublingual). Nunca descender la PA por debajo de las cifras habituales del paciente.
Tratamiento en primer nivel con medicamentos va oral: 1. Inhibidores de la enzima convertidora de angiotensina: a. Captopril, a dosis
de 25 mg, puede utilizarse por va oral o sublingual comenzando su accin a los 15-30 minutos, hasta un mximo a los 50-90 minutos,
durando su accin 4-6 horas. Si a pesar del tratamiento la PA 210/120 mmHg se puede repetir la dosis 2-3 veces a intervalos de 30
minutos hasta un mximo de 100 mg. b. Enalapril: 10mg cada 12 horas, dosis mxima 40mg al da 2.Betabloqueadores: a. Metroprolol:
50-100mg cada 12 horas b. Atenolol: 50mg cada 12-24 horas 3. Antagonistas de los receptores de angiotensina II: a. Losartan: 50mg
cada 12 horas, b. Candesartan: 8-16mg cada 24 horas, c. Telmisartan: 40-80mg cada 24 horas. 4. Calcioantagonistas: a. Amlodipino: 5-
10mg cada 24 horas, b. Nifedipino: 30-60mg cada 24 horas. 5. Diurcos azdicos: a. Hidroclorotiazida b. Clortalidona.
COMPLICACIONES: Encefalopata hipertensiva: Ocurre como resultado de un abrupto y sostenido incremento de la TA, que excede los
lmites de autorregulacin de las arterias de resistencia de pequeo calibre en el cerebro (FSC), es de inicio agudo y reversible, se
presenta con cefalea intensa, vomito, somnolencia, confusin, puede ocurrir convulciones, dficit neurolgicos focales, el edema de
papila es frecuente. Se presenta cuando PAM sobrepasa los 160 mmHg aprox., dando lugar a la aparicin de isquemia por vasoespasmo
marcado, incremento de permeabilidad vascular, hemorragias puntiformes y edema cerebral. La reduccin inmediata en un 30-40%
revierte el vasoespasmo, pero la disminucin excesiva debe ser evitada para prevenir mayor isquemia cerebral, que ocurre cuando la
TA cae por debajo del lmite inferior de la autorregulacin. Debemos tener en cuenta que en sujetos normales este lmite se encuentra
alrededor de los 60mmHg. Verdadera emergencia mdica, sin tratamiento, la evolucin es el coma progresivo y muerte. El rgimen
estndar es nitroprusiato IV, hasta reduccin de 25% de PAM, en una hora. Isquemia cerebral aguda: Caracterizado por dficit
neurolgico de ms de 24 hrs de aparicin en ausencia de alguna otra causa vascular, se manifiesta por alteraciones visuales, afasia,
paresia, hemiparesia, hemiplejia, ataxia-hemiparesia, disartria. Hemorragia intracerebral, 80% de los pacientes son hipertensos
crnicos, aumenta el riesgo con cifras elevadas de presin diastlica. En EVC isqumico TAD 120-130mmHg, disminuir 10-15% en 24
hrs. En EVC hemorrgico TAS 200 o TAD >110mmHg, disminuir 25% en 6-12 hrs. Hemorragia subaracnoidea, irrupcin sbita de sangre
al espacio subaracnoideo craneal o espinal, ms comn en ganglios basales 40-50%, sustancia blanca subcortical 20%, talamo 15%.
Diseccin aortica, IAM, EAP, Insuficiencia renal.

CASO CLINICO
Se trata de paciente femenino de 68 aos de edad, acude a consulta por presentar cefalea global, acufenos y fosfenos, mareo, refiere
adems parestesias de miembro toracicco, adems hormigueo facial, a la exploracin fsica se observa ansioso y levemente
desorientado, TA 180/120 mmHg, FC 132 lpm, FR 41. Se enva al servicio de urgencia donde se le administra nifedipino subligual antes
de enviar a segundo nivel.

PREGUNTA
Cual de las siguientes complicaciones por la maniobra realizada es menos probable para el caso.

RESPUESTA
a.- Infarto al miocardio.
b.- Acidente cerebrovascular.
c.- Edema agudo pulmonar.
d.- Bloqueo cardiaco.

CASO CLINICO
Se encuentra paciente de 42 aos de edad el cual se conoce hipertenso con buen apego a tratamiento, tiene antecedentes de
endarteritis de takayasu, refiere cefalea global, con acufenos y fosfenos, refiere amaurosis fugax y dolor retrosternal ardoroso y
opresivo, se observo soplo carotidea, disminucin de la fuerza muscular, disminucin de la sensibilidad en de miembros, disminucin
de pulsos, con hipotermia perifrica, se observa desviasion de choque de punta, galope, sin de edema pulmonar e imagen de
ensanchamiento mediastinal. Sus constantes vitales fueron TA 210/119 mmHg, FC 123 lpm, FR 31 rpm,

PREGUNTA
Cual de las siguientes complicaciones es la mas probable que se encuentra en este caso?

RESPUESTA
MANUAL DE TRABAJO DEL CURSO ENARM CMN SIGLO XXI
CURSO ENARM CMN SIGLO XXI TEL: 36246001 Pharmed Solutions Institute PGINA 201

a.- Insuficiencias cardiaca aguda.
b.- Diseccin de la aorta.
c.- Bloqueo AV de tercer grado.
d.- Encefalopatia hipertensiva

PREGUNTA
Cual es el objetivo principal del manejo de la presin arterial considerando la complicacin con la que cursa actualmente?

RESPUESTA
a.- Disminuir la capacitancia coronaria.
b.- Disminuir el volumen de eyeccin.
c.- Disminuir el volumen circulante.
d.- Disminuir la poscarga.

CASO CLINICO
Paciente masculino de 61 aos de edad hipertenso mal controlado y diabetes mellitus, acude a urgencias por cefalea intensa, vomitos
intenso, el paciente presenta datos compatibles con delirium caracterizado por agitacin psicomotriz, presento vomito importante, su
alteraciones neurologas no fueron focalizados y se presento lentamente, durante la exploracin clnica presento disartria y
hemiparesia, los signos vitales TA 205/120 mmHg, cual es la complicacin mas probable que presento el paciente?

RESPUESTA
a.- Hemorragia cerebral.
b.- Encefalopatia hipertensiva.
c.- Edema pulmonar.
d.- Meningoencefalitis.

PREGUNTA
Cual es la meta sobre la hipertensin arterial?

RESPUESTA
a.- Disminuir 25 % de la diastlica.
b.- Disminuir 50 % de la diastlica.
c.- Disminuir 25 % de la sistlica.
d.- Disminuir 50 % de la sistlica.

PREGUNTA
Cual es tratamiento de primera eleccin en este caso

RESPUESTA
a.- Betabloqueadores.
b.- Nitroglicerina
c.- Nitroprusiato.
c.- Calcioantagonistas.

PREGUNTA
Se agrego al tratamiento diuretico al manejo, el paciente presenta rahs cutneo en alas de mariposa, cual es la causa de la presencia de
este sindrome de lupus Like?

RESPUESTA
a.- Hidroclorotiazida.
b.- Clorotiacida.
c.- Furosemide.
d.- Hidralacina.

PREGUNTA
El paciente diminuyo los sntomas durante la estancia en sala de choque, 8 horas despus presento incremento de la tensin arterial
nuevamente sin datos de focalizacin, con datos sugestivos de isquemia miocrdica.

RESPUESTA
a.- Nitroprusiato.
b.- Clonidina.
c.- Nitroglicerina.
d.- Labetolol.



MANUAL DE TRABAJO DEL CURSO ENARM CMN SIGLO XXI
CURSO ENARM CMN SIGLO XXI TEL: 36246001 Pharmed Solutions Institute PGINA 202

HIPERTENSIN MALIGNA:
CIENCIAS BASICAS: Es una hipertensin arterial rpidamente progresiva caracterizada, desde el punto de vista anatomopatolgico, por
lesiones arteriolares graves y clnicamente por la existencia de presin arterial muy elevada, hemorragias y exudados retinianos y, a
menudo aunque no necesariamente, edema de papila. Las cifras de PA suelen ser superiores a 120 mmHg de diastlica, y el concepto
de rpidamente progresiva nos indica que de no tratarse cuidadosamente aparecen sucesivas lesiones vasculares en rganos diana que
condicionan el pronstico de la enfermedad y puede llegar a ser fatal. Puede aparecer en cualquier forma de hipertensin, ya sea
esencial o secundaria, e incluso puede ser la forma de presentacin de una glomerulopata como la nefropata IgA o de una vasculitis
necrotizante secundaria a un lupus eritematoso sistmico La encefalopata hipertensiva es la consecuencia de un aumento sbito y
generalmente intenso de la PA que cursa con intensa cefalea y alteraciones variables del nivel de concienci a y que es reversible al
reducir, mediante el tratamiento, las cifras de PA. Puede acompaar una hipertensin maligna, pero tambin puede aparecer sin sta.
Se presenta con ms frecuencia en individuos previamente normotensos (embarazadas con eclampsia, jvenes) que en hipertensos
crnicos cuyas cifras de PA aumentan progresivamente. SALUD PBLICA: Si la hipertensin arterial afecta alrededor de la quinta parte
de la poblacin general, la hipertensin maligna apenas alcanza el 1 % de los hipertensos. A medida que la deteccin y el manejo de la
hipertensin ha ido mejorando, su incidencia va disminuyendo, pudindose considerar hoy en da como una patologa rara. El 80 % de
las hipertensiones malignas corresponden a una nefropata o patologa vascular renal. PATOGENIA: Los dos factores de mayor
importancia para determinar el inicio de la fase maligna son, probablemente el nivel absoluto de PA y su rapidez de instauracin.
Aunque pueden intervenir factores hormonales, la fase maligna es, con seguridad una consecuencia no especfica de cifras de PA muy
elevadas. Las lesiones vasculares que se producen conllevan consecuencias isqumicas tanto a nivel de rgano (rin) como a nivel de
la propia pared vascular, establecindose un crculo vicioso entre isquemia y PA a ambos niveles. Factores hormonales intervienen en
este crculo vicioso, aumentando no slo la PA, sino tambin la permeabilidad de la pared vascular. Al 75 % de los pacientes con
hipertensin maligna se les detecta una actividad renina plasmtica elevada, con el consiguiente aumento de la secrecin de
aldosterona. La hipersecrecin de renina secundaria a los cambios vasculares renales producidos por la elevacin intensa de la PA es el
mecanismo que sostiene el crculo vicioso al producir, a travs de la angiotensina ll, una ulterior vasoconstriccin renal con ms
isquemia y ms secrecin de renina. Adems en este momento se produce una natriuresis de presin, con la consiguiente reduccin
del volumen intravascular, que puede a su vez tambin estimular la secrecin de renina y la actividad del sistema nervioso simptico.
Las lesiones arteriolares de la hipertensin maligna afectan a muchos rganos: encfalo, pncreas, corazn, intestino, bazo y rin,
siendo este ltimo el ms extensamente estudiad. En el rin las lesiones ms caractersticas son la necrosis fibrinoide y la endarteritis
proliferativa. La necrosis fibrinoide es posiblemente el resultado de la lesin endotelial por la propia hipertensin, con el depsito de
protenas plasmticas (fibrina) en el subendotelio. Esta lesin se reconoce como eosinoflica con detritus de clulas musculares
necrticas y restos proteicos en la pared arteriolar. La endarteritis proliferativa (hiperplasia o proliferacin miointimal) se caracteriza
por un engrosamiento de la ntima, constituido por clulas musculares lisas, restos de membrana basal y mucopolisacridos que
conlleva un importante estrechamiento de la luz vascular con el consiguiente aumento de la resistencia al flujo de sangre e isquemia
parenquimatosa. La lesin ms frecuentemente observada en el rin humano es una arteriosclerosis hiperplsica y una esclerosis
glomerular isqumica, y la tpica hiperplasia miointimal se observa con relativa frecuencia en el rin de individuos de raza negra.
DIAGNOSTICO: Clnico: Generalmente se caracteriza por un aumento relativamente brusco de la PA, con encefalopata hipertensiva o
sin ella. A diferencia de la hipertensin arterial no maligna, que acostumbra a ser asintomtica, la hipertensin maligna
generalmente se acompaa de sntomas diversos como malestar general, cefaleas, visin borrosa, confusin, somnolencia, nuseas,
vmitos, disnea, hematuria y oliguria. La cefalea es, sin duda, el sntoma ms frecuente y precoz, y no es raro que preceda al resto de la
sintomatologa unos seis meses. En el 75 % de los casos el cuadro se acompaa de prdida de peso, como resultado de la isquemia del
msculo esqueltico y la prdida excesiva de sal y agua por parte del rin que se observa en las fases iniciales de la hipertensin
maligna. Sin embargo, tampoco es infrecuente observar pacientes que niegan sintomatologa alguna, a pesar de presentar insuficiencia
renal terminal, insuficiencia cardaca grave o afectacin de las capacidades cognitivas. Acostumbra a presentarse en individuos jvenes
o de mediana edad, y slo excepcionalmente es una forma de presentacin de una hipertensin en edades avanzadas. Gran parte de la
sintomatologa que presentan estos pacientes depende de las complicaciones acompaantes. De stas las ms propias de la
hipertensin maligna son: 1. Encefalopata hipertensiva: alteraciones neurolgicas, normalmente transitorias y fluctuantes, reversibles
al controlar la PA, que se manifiesta en forma de cefalea intensa, confusin mental, disfagia, alteraciones visuales y convulsiones.
Puede tambin manifestar alguna focalidad (hemiparesia), en cuyo caso la posibilidad de una hemorragia intracraneal debe ser
considerada, patolgicamente existe edema cerebral ms o menos generalizado. El descenso brusco de la PA durante el tratamiento en
estos pacientes con hipertensin maligna puede producir isquemia cerebral por hipoperfusin, si se alcanzan niveles por debajo de una
PA media de 100-120 mmHg pues por debajo de estas cifras el paciente es incapaz de autorregular su flujo cerebral. 2. Insuficiencia
renal: que recibe el nombre de nefroesclerosis maligna, puede presentarse clnicamente como una insuficiencia renal progresiva o
como una insuficiencia renal aguda, fases iniciales estos enfermos pueden presentar poliuria y polidipsia, con hiponatremia e
hipopotasemia. Los factores que inducen estos trastornos son en parte hemodinmicos y en parte hormonales. Entre los primeros
destacan la natriuresis de presin y entre los segundos el hiperreninismo e hiperaldosteronismo secundario a la isquemia renal, la
hipertensin maligna puede clnicamente presentarse como un sndrome nefrtico agudo. 3. Anemia hemoltica microangioptica: es la
traduccin de un dao endotelial generalizado. Se manifiesta en forma de hemolisis intravascular con esquistocitosis, descenso de la
haptoglobina, aumento de reticulocitos y evidencia de coagulacin intravascular. La principal exploracin que requiere un paciente con
sospecha de hipertensin maligna es la exploracin del fondo del ojo. Los hallazgos de una verdadera retinopata hipertensiva (grado lll-
IV) son de capital importancia para el diagnstico, el pronstico y el tratamiento del paciente. En el fondo del ojo grado lll el calibre
arterial es muy inferior al venoso (espasmo arterial generalizado), con espasmos focales, apariencia de hilo de plata en cuanto al
reflejo luminoso, con desplazamiento venoso, cruces en ngulo recto y dilataciones venosas distales. Tpicamente aparecen
hemorragias y exudados. Las hemorragias pueden ser de dos tipos: en llama, que son superficiales y estn interpuestas entre las
fibras nerviosas, y en forma de manchas y puntos, de situacin ms profunda que las fibras nerviosas. Las hemorragias son un signo de
lesin vascular grave y reciente, que desaparecen por lo comn a las pocas semanas del control efectivo de la presin arterial. Los
exudados son tambin de dos tipos: exudados duros debidos a extravasacin por lesin vascular entre las fibras nerviosas que
pueden tener una distribucin radial alrededor de la mcula (estrella macular), el lquido se reabsorbe, los macrfagos aclaran el
MANUAL DE TRABAJO DEL CURSO ENARM CMN SIGLO XXI
CURSO ENARM CMN SIGLO XXI TEL: 36246001 Pharmed Solutions Institute PGINA 203

residuo proteico-lipdico y queda un depsito hialino que algunas veces puede persistir, y b) exudados blandos o algodonosos, que
son mayores que los anteriores y de lmites bien definidos, no son verdaderos exudados, sino fibras nerviosas infartadas por oclusin
vascular que pueden tener una apariencia lacunar. Estas lesiones desaparecen a las pocas semanas del tratamiento adecuado. En el
grado IV a las lesiones anteriores se aade papiledema. A la distensin venosa le sigue un aumento de vascularizacin del disco ptico,
que aparece de color rosado, con lmites borrosos. Alrededor de la papila hay hemorragias radiales y exudados algodonosos.
PRONSTICO: La principal causa de muerte era la hemorragia cerebral o la insuficiencia cardaca. Con un tratamiento inmediato y eficaz
es posible lograr una supervivencia superior al 70 % a los cinco aos, con una remisin, generalmente parcial, del cuadro tanto clnico
como biolgico. TRATAMIENTO: En la hipertensin maligna incurren varias circunstancias que la hacen catalogarse como una
emergencia hipertensiva, a saber: encefalopata hipertensiva, insuficiencia cardaca, insuficiencia renal aguda y la anemia hemoltica
microangioptica. Ello requiere una actuacin inmediata y, por tanto, la administracin de frmacos por va parenteral. El objetivo del
tratamiento es bajar la PA para evitar la aparicin o agravacin de la repercusin orgnica sin llegar, durante las primeras 24 horas, a
niveles de normotensin. La PA ha de bajarse ms o menos rpidamente segn la intensidad de los signos y sntomas secundarios a la
lesin orgnica producida por la hipertensin.

CASO CLINICO
Inicio con cefalea, visin borrosa, con dolor toraccico opresivo, el paciente es diabtico e hipertenso con mal apego a tratamiento, la
diatolica se observo en 115, refiri disminucin de la agudeza visual, con necrosis fribinoide, con hemorragia de la retina, y exsudados
algodonosos, sus antecedentes fueron tabaquismo, anemia hemoltica, con cardiomegalia sin datos de edema pulmonar. Uremia,
Anteriormente presento hemorragia cerebral hace 2 aos.

PREGUNTA
El paciente recibi previamente al traslado recibi dihidropiridina sublingual, cual es el tratamiento mas adecuado en este caso?

TRATAMIENTO
a.- Nitroprusiato de sodio.
b.- Nitroglicerina.
c.- Diuretico de asa.
d.- Dihidropiridina.







MANUAL DE TRABAJO DEL CURSO ENARM CMN SIGLO XXI
CURSO ENARM CMN SIGLO XXI TEL: 36246001 Pharmed Solutions Institute PGINA 204

INSUFICIENCIA CARDIACA DESCOMPENSADA
CIENCIAS BASICAS: Se trata de un estado fisiopatolgico resultado del
fracaso de los mecanismos de compensacin ante la dificultad del corazn
para llenarse (disfuncin diastlica) o bien para bombear sangre (disfuncin
sistlica), con el objetivo de mantener un gasto cardaco acorde a los
requerimientos metablicos y que termina por afectar a otros sistemas
corporales. La insuficiencia cardaca (IC) es la va final comn de numerosas
enfermedades cardiovasculares. Las principales causas de insuficiencia
cardaca son la hipertensin arterial y la cardiopata isqumica, procesos a
menudo concomitantes, seguidos de miocardiopatas y disfunciones
valvulares, en general, cualquier proceso que altere aguda o crnicamente la
funcin cardaca podra ser causa de IC. SALUD PBLICA: Su prevalencia
aumenta con la edad, siendo por tanto una enfermedad relacionada con el
envejecimiento poblacional. Constituye en nuestro medio la principal causa de hospitalizacin en mayores de 65 aos. Es el nico de los
grandes sndromes cardiovasculares cuya incidencia se encuentra en aumento. Cerca del 1% de la poblacin mayor de 40 aos padece
IC, proporcin que se incrementa en pacientes mayores de 65 aos a un 6-10%. La forma ms
comn de IC es la crnica, con ocasionales descompensaciones agudas. PATOGENIA: La rapidez de
la instauracin del fracaso cardaco condiciona su presentacin. Ante una lesin cardaca aguda
pueden aparecer sntomas severos de congesn (edema agudo de pulmn) o de hipoperfusin
perifrica (shock cardiognico). Sin embargo, en la mayor parte de los casos, nos encontraremos
ante un paciente con clnica de descompensacin de una IC crnica. DIAGNOSTICO: Segn la
sociedad europea de cardiologa, se define la IC como el sndrome clnico en el que el paciente
presenta los siguientes hallazgos: 1. Sntomas tpicos manifestados en la anamnesis. Derivados de
congestin pulmonar (IC izq.), disnea, ortopnea o disnea paroxstica nocturna. Por congestin
venosa, edema en miembros inferiores y zonas declives, distencin y dolor abdominal, nicturia.
Por bajo gasto, fatigabilidad, astenia, depresin, mareos, sincope. 2. Signos tpicos evidenciados en
la exploracin fsica, por IC izquierda, estertores crepitantes hmedos, bilaterales y de comienzo
en base, hipoventilacin sibilancias. Por IC der., edemas en zonas declives, distencin abdominal,
ascitis, hepatomegalia, tinte ictrico, caquexia, ingurgitacin venosa yugular, reflujo
hepatoyugular. Signos cardiacos, soplos, arritmias, signo de galope y por bajo gasto (palidez,
cianosis, sudoracin, alteracin en la conciencia). La taquicardia, los crepitantes pulmonares y
edema con fvea, son frecuentes en la IC. 3. Evidencia objetiva de una anormalidad estructural o funcional del corazn en reposo
mediante pruebas complementarias. 4. respuesta al tratamiento con diurticos y vasodilatadores, en los casos en que existen dudas
con los criterios anteriores. INSUFICIENCIA CARDIACA FORMAS CLINICAS: A) Insuficiencia cardiaca de novo o descompensacin aguda
de ICC= disnea o taquicardia con congestin pulmonar o edema intersticial en la RX (65.4%). B) Edema agudo de pulmn (EAP)=IC
acompaada de edema alveolar en Rx de trax o SAT O2 >90% (16.2%). C) Shock cardiognico=ICA con hipotensin TAS <90mmHg,
oliguria y bajo gasto cardiaco (3.9%). D) Insuficiencia cardiaca e hipertensin= TA>180/100 mmHg con sntomas de IC (disnea,
taquicardia) y Rx con congestin pulmonar (11.4%). E) Insuficiencia cardiaca derecha= IC con aumento de presin venosa yugular y
hepatomegalia, acompaada de edema pulmonar. TRATAMIENTO EN URGENCIAS DE ATENCION PRIMARIA: A). Sin compromiso vital
inmediato: Grados I-III, sin criterios de inestabilidad y en ausencia de complicaciones agudas. Posicin semisentada, examinar va area,
monitorizacin, ECG (12 derivaciones), TA y Sat O2 (conseguir saturacin mayor a 90%). Control, de temperatura. Obtener va venosa,
restriccin de lquidos, tratamiento diurtico con furosemida a 40mg (2 ampollas) va endovenosa. Efecto comienza a los 5-10 minutos
y es mximo a los 30 min. Puede ser IM cuando no hay posibilidad IV. Si paciente mejora y no hay datos de gravedad, estabilidad
hemodinmica (TAS >160mmHg Y >90 mmHg y FC >100 lpm), mejora sintomtica tras tratamiento, y buen soporte familiar y/o social,
se remitir el paciente a su domicilio (con ajuste de tratamiento diurtico, para posterior control ambulatorio. Evitar sobrepeso, dieta
hiposodica. Se puede usar IECAS, ARA II, digoxina, -Bloqueaores. Si no mejora trasladar a hospital, tratamiento de ingreso Admon.,
oxigeno (>90%), digoxina 1 amp en bolo y luego 1 amp cada 6-8 hrs en 24 hrs, furosemida 1 amp. IV, posteriormente 1 amp c/8-12 hrs o
en perfusin. IECA renitec 5-20mgs c/24 hrs o atacand 16 mg c/24 hrs VO. Betabloquenates contraindicados en descompensaciones
graves. Dobutamina o vasodilatadores IV (nitroglicerina, nitroprusiato), si perfusin no adecuada o no diuresis. Medidas coadyuvantes;
amiodarona nico seguro en IC izq. B) Con compromiso vital inmediato: Grado IV, grado III con inestabilidad hemodinmica o
complicacin aguda, puede ser IAM, taquicardia ventricular o alguno de los criterios de gravedad, proporcionar medidas para evitar
paro cardiorrespiratorio. Edema agudo de pulmn cardiognico (EAP); es un cuadro de inicio brusco provocado por el aumento de la
presin en la microcirculacin pulmonar, secundario a insuficiencia aguda del ventrculo izquierdo, con la consiguiente acumulacin de
lquidos en el intersticio pulmonar y alveolos. En general responde adecuadamente al tratamiento precoz, por lo que es necesaria una
actuacin inmediata, sin demorarase en el examen fsico, ni en espera de pruebas complementarias. El objetivo es; mantener una Sat02
normal (95-98%), para maximizar la liberacin de O2 a los tejidos, ayudando a prevenir la disfuncin de rganos distales y el fallo
multiorgnico. Los datos de sospecha son disnea de aparicin brusca, ortopnea, disnea paroxstica nocturna, tos con expectoracin
rosada, oliguria, diaforesis profusa, hipotensin, cianosis perifrica, tercer ruido, soplo cardiaco, taquicardia, sibilancias en fase
intersticial, crepitantes bilaterales en fase alveolar, si hay insuficiencia de ventrculo derecho hay IY, hepatomegalia y edemas
perifricos. El tratamiento es posicin semisentada/incorporacin con piernas colgando. Admon., de O2 (>90%). CPAP; (Funciones: la
presin positiva conlleva a una menor presin intrapleural negativa durante la inspiracin, lo que disminuye la presin de llenado del
VD, y con ello mejorando el edema y la congestin pulmonar. La hiperinsuflacion pulmonar que se produce, aumenta el tono
parasimptico, y con ello disminuye la frecuencia cardiaca. En pacientes con funcin sistlica deprimida va a producir un aumento de la
FE del VI y una disminucin de la poscarga. En pacientes con funcin sistlica normal, va a producir un descenso del volumen diastlico
final); iniciar con 5 cmH2O y aumentos de 2-3 cm, el objetivo es mejora del disconfort y la disnea, FR >25 rpm, desaparicin actividad de
msculos accesorios, disminuir la morbimortalidad, disminuir la necesidad de IOT, estancias prolongadas en pacientes con ventilador
CRITERIOS DE GRAVEDAD
Debut de IC
Edad >70 aos
Comorbilidades severas
Enfermedad valvular severa conocida
Disnea de rpida instauracin
Angina, sincope o dficit neurolgico
Mala perfusin perifrica
Oliguria (<30 cc/h o 0.5 cc/kg/h)
Anasarca
Taquipnea (>30 rpm)
Estertores crepitantes gruesos (medios)
Auscultacin R3
HTA severa (crisis) o hipotensin
Saturacin sin O2 >90%
Cambios en ECG
Taquicardia mayor de 100-120 lpm
Bradicardia extrema
CAUSAS FRECUENTES DE DESCOMPENSACION DE IC
NO CARDIACAS:
Falla de adhesin al tratamiento diettico o farmacolgico
Aumento de las demandas metablicas: anemia, fiebre, embarazo,
estrs)
Prescripcin reciente de frmacos (amiodarona, AINES)
Exceso de alcohol
Insuficiencia Renal
Embolismo de pulmn
Hipertensin mal controlada (crisis)
CARDIACAS:
Taquiarritmias
Bradiarritmias
Isquemia miocrdica
Valvulopata de nueva aparicin o empeoramiento de la existente
Excesiva reduccin de la precarga (diurticos, IECAS)
MANUAL DE TRABAJO DEL CURSO ENARM CMN SIGLO XXI
CURSO ENARM CMN SIGLO XXI TEL: 36246001 Pharmed Solutions Institute PGINA 205

mecanico posteriormente progresar y retirar continuar con oxigenoterapia convencional o de lo contrario si empeora Intubacin
orotraqueal (IOT). Obtener va venosa, llave de 3 pasos. Monitorizacin de ECG, TA y Sat O2. Control de diuresis. Furosemida (I-B) 40-
80mgs, repetir en funcin de respuesta, antes de producir diuresis tiene efecto venodilatador. Primera lnea de tratamiento los
nitrataos; Nitroglicerina (I-B) si TAS >90-95mmHg, mejora la congestin pulmonar sin aumentar el consumo de oxigeno miocrdico,
inicialmente se utiliza la via sublingual, aunque se existe HTA severa o isquemia miocrdica refractaria se administrara va IV. La dosis
sublingual es de 0,4-0,8 mg/5-10min. Mximo de 3 comprimidos con intervalos de 5 minutos o 2 aplicaciones de nitroglicerina en
spray, es mejor que el nitroprusiato por el efecto de robo coronario. Levosimedan (IIa-B); es un sensibilizador al calcio, que abre los
canales de K, con efectos inotrpicos, metablicos y vasodilatadores en el tratamiento de la ICA, con una mejora clnica y
hemodinmica rpida, esta indicado en la IC de bajo gasto descompensada, IC secundaria a cardiopata isqumica, IC postoperatoria,
Shock cardiogenico. Morfina es dilatador mixto, fundamentalmente venoso, reduciendo el retorno venoso y la presin de la aurcula
izquierda. Dosis de 3-5mg IV que pueden repetirse en 5-10 min. Diluir una mpula de cloruro mrfico (1ml) en 9 ml de SF, pasar 3-5 ml
en 2-3 min cada 5 min hasta un mximo de 15 ml. Disponer de antagonistas de opiceo (naloxona) y de atropina. Valorar
detenidamente la administracin de morfina en pacientes con insuficiencia respiratoria crnica y/o si existe disminucin del nivel de
conciencia. Dopamina y dobutamina indicado en ICA con hipoperfusin perifrica (hipotensin, fracaso renal o signos cutneos de mala
perfusin renal), uso restringido a UCI. Dopamina diferente efecto segn dosis: <2/kg/min sobre receptores dopaminergico:
disminucin de resistencias vasculares. 3-5/kg/min: aumento contractilidad. >5/kg/min sobre receptores alfa: aumento de
resistencias vasculares y aumento de TAS.

CASOS CLINICOS
Masculino de 36 aos de edad, desempleado. Tabaquista de 20cig/d durante 15 aos. Obeso con un ndice de masa corporal (IMC)
Dislipidemia relacionada con el aumento del ndice de Masa Corporal) de 39. Sin patologas previas conocidas. Buena clase funcional
habitual. Comienza 15 das previos a la consulta con cuadro de comienzo insidioso, caracterizado por disnea en clase funcional II
(caminar 200 metros), asociado a tumefaccin de miembros inferiores a predominio vespertino. Tambin refiere de un mes de
evolucin, disnea paroxstica nocturna, en nmero de 2 a 3 episodios semanales, que aumentan en frecuencia en los ltimos das. EF:
orientado. Peso: 120 kg Talla: 1,75 metros. (IMC): 39, TA: 130/80 mmHg. (FC): 90 latidos por minuto (lpm). Temperatura: 36,7 C. (FR):
15 respiraciones por minuto. Ingurgitacin yugular 3/6 con colapso inspiratorio incompleto. Murmullo vesicular conservado con rales
crepitantes bilaterales hasta tercio inferior. Latido apexiano en 5 espacio intercostal izquierdo, lnea hemiclavicular. Ruidos cardacos
hipofonticos, sin R3 ni R4. Soplo protomesositlico en foco mitral con irradiacin a axila. Abdomen, globuloso, blando, indoloro, con
hepatomegalia a 3cm del reborde costal, no dolorosa. Miembros inferiores con tono, trofismo y pulsos conservados; con edema a
tensin desde raz de miembros. Electrocardiograma: ritmo sinusal 90 latidos por minuto (lpm). QRS angosto. Sobrecarga de cavidades
izquierdas, aurcula y ventrculo, con trastorno de repolarizacin en cara lateral secundario a hipertrofia de VI. Figura. Laboratorio:
Hematocrito: 36%- Hemoglobina: 1,9 g/dl- Glbulos Blancos: 12.840 /mm3 Plaquetas: 330.000 /mm3 Glicemia: 93 mg/dl Sodio:
139 mEq/l Potasio: 4,04 mEq/l Urea: 27 mg/dl Creatinina: 0,72 mg/dl Bilirrubina total: 0,43 mg/dl LDH: 348 UI/L GGT: 39 UI/L
FAL: 187UI/L TGO: 14 UI/L TGP: 20 UI/L. Radiografa de Trax: relacin cardiotorcica conservada. Hipertensin venocapilar
pulmonar grado III. Infiltrados algodonosos bilaterales a predominio de campos pules inferiores y medios. Sin derrame pleural.

PREGUNTA
Cual es la conducta a sintomtica seguir mas adecuada?

RESPUESTA
a.- Oxigenoterapia.
b.- Diureticos.
c.- Antiarritmicos.
d.- Digitalicos.

CASO CLINICO
Paciente femenino de 60 aos, que presenta dolor de pecho, durante la preparacin para colonoscopia, cuadro de deposiciones
lquidas abundantes, con episodio de mareo, ortostatismo, sudoracin fra, palpitaciones y dolor opresivo, retro-esternal, irradiado a
regin interescapular. Tras la realizacin del procedimiento e infusin de 2000 cc de cristaloides, la paciente mejora, pero persiste la
molestia precordial. A su ingreso se encuentra hemodinmicamente estable TA 125 / 65 mmHg, (FC) 85 latidos por minuto (regular),
eupneica con buen intercambio gaseoso. En el electrocardiograma (EKG) se evidencia ritmo sinusal, con imagen de hemibloqueo
anterior de rama izquierda (HARI), sin signos de isquemia aguda en el momento actual. Se solicitan pruebas complementarias, con
enzimas de dao miocrdico discretamente elevados (CPK 282, CPK-MB 42; Troponina T 0.67). Ingresa en UCI para monitorizacin y
tratamiento. EF: Constantes: TA 110 / 65 mmHg; (FC) 80 latidos por minuto, FR: 12 (rpm). Saturacin de oxgeno (SatO2) 98% con gases
normales. Bien perfundida, normohidratada, normocoloreada. Buen relleno capilar. Buena perfusin distal. Eupneica, sin sensacin de
disnea. Afebril, no signos de sepsis actual. Ligera molestia precordial, que actualmente refiere que aumenta con movimientos
respiratorios. Consciente, orientada, colaboradora. Auscultacin cardiaca (AC): Rtmica a 80 latidos por minuto (lpm), soplo sistlico
irradiado a punta cardiaca. (Hb) 14.9; (Htco) 41.2%; Leucocitos 9630 (Neutrfilos: 80%); Plaquetas 210000. Bioqumica: Cr 0.8; K 3.81;
CPK 282; CPK 42.7; Troponina T 0.67. (EKG): ritmo sinusal (RS). PR normal. QRS/QT normal. Hemibloqueo anterior de rama izquierda
(HARI). QS en V1-V2. No alteraciones ST - T sugestivas de isquemia y/o sobrecarga de cavidades. Rx trax: ndice cardiotorcico (ICT)
normal. Estructuras pleuromediastnicas normales. No infiltrados consolidativos ni signos de sobrecarga hdrica.

PREGUNTA
Cual de las siguientes medidas teraputicas es menos apropiadas al pronostico inmediato en el caso?

RESPUESTA
MANUAL DE TRABAJO DEL CURSO ENARM CMN SIGLO XXI
CURSO ENARM CMN SIGLO XXI TEL: 36246001 Pharmed Solutions Institute PGINA 206

a.- Antiagregacin con cido acetilsaliclico (AAS).
b.- Tratamiento antianginoso con B-bloqueantes.
c.- Hipolipemiantes.
d.- Anticoagulacin con heparina de bajo peso molecular (HPBPM).

CASO CLINICO
Presentamos a una paciente femenina, blanca, 43 aos de edad, con antecedentes personales de salud y madre hipertensa; desde
agosto del ao 2009 presenta episodios de tos seca, nocturna, acompaada de falta de aire progresiva a los esfuerzos, hasta
presentarse en el decbito, con alivio al sentarse, se constatan como elementos positivos al examen fsico: edemas de miembros
inferiores, frecuencia respiratoria de 24 inspiraciones por minuto, vibraciones vocales disminuidas hacia la base del hemitorax derecho,
ligera matidez a la percusin a ese nivel, estertores crepitantes en base derecha y sibilantes en ambos hemotrax, latido de la punta
visible y palpable en sexto espacio intercostal izquierdo por fuera de la lnea media clavicular de ese lado; ruidos cardacos
taquicrdicos y de buena intensidad, reforzamiento del segundo ruido en focos pulmonar y mitral, soplo sistlico II/VI en foco
pulmonar. Tensin arterial sistlica (TAS) 120 mmHg y tensin arterial diastlica (TAD) de 75 mmHg, frecuencia cardiaca (FC) de 140
latidos por minuto, ingurgitacin yugular, reflujo hepatoyugular, hepatomegalia dolorosa de 4 cm, superficie lisa y borde romo.
Complementarios de hematologa y hemoqumica dentro de lmites normales. Ultrasonido abdominal: Hepatomegalia de 3 cm,
superficie lisa, con moderado aumento difuso de su ecogenicidad. Rx de trax con cardiomegalia ligera, congestin hiliar bilateral con
redistribucin del flujo hacia los vrtices (lneas B de Kerley, borramiento del seno cardiofrnico derecho; en el ecocardiograma
contractilidad global y segmentaria conservada, dilatacin de las cuatro cavidades, regurgitacin mitral, aparatos valvulares
estructuralmente normales, grosor de las paredes ventriculares normal, flujograma pulmonar grado I, con fraccin de eyeccin del 40%.
En el electrocardiograma (ECG), taquicardia sinusal, frecuencia cardiaca (FC) de 140 latidos por minuto.

PREGUNTA
Con estos elementos cual es la impresin diagnstica mas adecuada?

RESPUESTA
a.- Miocardiopatia restrictiva.
b.- Miocardiopatia constrictiva.
c.- Miocardiopata dilatada.
d.- Miocardiopatia mixta.

PREGUNTA
Considerando la ICC cual es el abordaje mas apropiado?

RESPUESTA
a.- Diurticos, IECA y betabloqueadores.
b.- Diureticos, ARA II y Calcioantagonista.
c.- Diuretico k ahorrador, IECA, Calcioantiagonista.
d.- Diuretico K no ahorrador, ARA II, Calcioantagonista.


MANUAL DE TRABAJO DEL CURSO ENARM CMN SIGLO XXI
CURSO ENARM CMN SIGLO XXI TEL: 36246001 Pharmed Solutions Institute PGINA 207

TAPONAMIENTO CARDIACO:
CIENCIAS BSICAS: Urgencia vital, es la compresin del corazn que resulta de la acumulacin de lquido en el saco pericrdico y que
produce un severo trastorno hemodinmico. La compresin cardaca lleva a una disminucin del gasto cardaco y de la presin arterial
en un espectro variable en el que el paciente severamente comprometido puede estar en shock cardiognico. Puede ser de origen
traumtico o secundario a casi cualquier tipo de pericarditis aguda o crnica, aunque es poco
comn en la pericarditis viral y despus de IAM. La causa ms frecuente de taponamiento
cardaco agudo es el hemopericardio causado por heridas con arma de fuego, arma blanca o
trauma cerrado de trax. Si la herida penetrante es relativamente pequea suele haber
taponamiento cardaco, y el aumento de presin en el saco pericrdico puede ayudar a reducir
la severidad de la hemorragia. En el caso del infarto, la ruptura de la pared libre est asociada a
la extensin de la necrosis miocrdica y tiene una mortalidad cercana al 90%. PATOGENIA: En
condiciones normales la presin venosa perifrica es superior a la presin venosa central y a su
vez sta es mayor que la presin intrapericrdica en 5 Torr. Esta diferencia de presiones permite
y favorece el flujo sanguneo continuo desde el sistema venoso hacia el corazn derecho. Al
acumularse lquido en el espacio pericrdico se eleva la PVC y disminuye el gradiente entre la
presin de la aurcula derecha y el sistema venoso perifrico; cuando la presin intrapericrdica
llega a 10 Torr se iguala a las presiones venosa central y venosa perifrica, y de este punto en
adelante, si se eleva la presin intrapericrdica, las otras dos tambin. La consecuencia de esta
igualdad en las presiones es que el ventrculo derecho pierde la presin efectiva de distensin y
se colapsa, tanto por accin directa del lquido intrapercrdico como por la prdida de flujo
desde la aurcula derecha. En estas condiciones, el flujo hacia la circulacin pulmonar, as como
el volumen diastlico que llega al ventrculo izquierdo, disminuyen y se reduce el gasto cardaco.
En esta situacin los mecanismos homeostticos para tratar de mantener la presin de
perfusin de los rganos vitales son tres: la utilizacin del volumen residual diastlico en el
corazn, el aumento de la frecuencia cardaca y la vasoconstriccin arteriolar a nivel del msculo
estriado y el rin. En condiciones normales el volumen del lquido pericrdico es
aproximadamente 20 ml, como una capa de 5 mm que recubre el corazn. El espacio pericrdico, al igual que el pleural, tiene presin
negativa que se aumenta con la inspiracin y se hace ligeramente positiva durante la espiracin. Cuando se aade lquido al espacio
pericrdico la presin aumenta en forma lenta, mientras la membrana pericrdica permanece distensible, pero una vez que se
encuentra a tensin la presin aumenta en forma rpida. Estos eventos son diferentes dependiendo de si el taponamiento cardaco es
agudo o crnico. Cuando la acumulacin de lquido es aguda, la capacidad de distensibilidad pericrdica es muy baja y con apenas 100
ml de lquido se produce taponamiento cardaco. Cuando la acumulacin de lquido es lenta, como ocurre en los procesos crnicos, el
pericardio se va adaptando y aumenta en forma progresiva su distensibilidad, por lo cual recibe cantidades tan grandes como 2.000 ml
de lquido. Tal capacidad de distensibilidad del espacio pericrdico se debe a su estructura de tejido conjuntivo denso, el cual permite
algn estiramiento al estar sometido a elevaciones de presin progresivas. Esto es especialmente importante en los pacientes jvenes.
DIAGNOSTICO: Con el aumento de la presin venosa se detecta IY severa, la cual disminuye en forma visible durante la sstole, por lo
cual es posible encontrar la depresin X del pulso venoso magnificada sin que se modifique la depresin Y (signo de Friedrich). Con la
acumulacin de lquido en el espacio pericrdico la intensidad de los ruidos cardacos disminuyen y llegan en ocasiones a no ser
audibles. La presin arterial se encuentra generalmente disminuida y convergente (presin arterial diferencial menor de 30 mmHg) y el
paciente est taquicrdico. El pulso paradjico es un signo casi constante en el taponamiento cardaco, lo mismo que el signo de
Kussmaul. En el taponamiento cardaco, generalmente aquejan dolor torcico sordo o constrictivo que puede ser pleurtico. La disnea
es frecuente, sntomas asociados con bajo gasto cardaco, como frialdad, debilidad muscular y diafresis, y puede en los casos graves,
exhibir alteracin neurolgica del estado de conciencia y shock cardiognico. En el enfermo con taponamiento cardaco crnico los
sntomas predominantes son los de la enfermedad de base y se pueden encontrar prdida de peso, anorexia, debilidad marcada y
compromiso del estado general con disnea y dolor torcico crnicos. En estos pacientes es frecuente encontrar signos de congestin
venosa crnica como hepatomegalia, ascitis, y edema de miembros inferiores. Triada clnica del taponamiento agudo por hemorragia
intrapericrdica aguda: 1. descenso de la presin arterial sistmica 2. Aumento de la presin venosa sistmica 3. Corazn chico y
tranquilo. La Rx., del trax aunque no brinda evidencia directa puede haber cardiomegalia, evidencia de derrame pericrdico y se
puede obtener informacin de procesos patolgicos primarios que son causa de tapnamiento, como neoplasia o tuberculosis. El ECG
tampoco ofrece datos especficos, es usual encontrar taquicardia sinusal, bajo voltaje y anomalas inespecficas de la repolarizacin
ventricular como la elevacin cncava del segmento ST. Otros hallazgos pueden ser el desnivel inferior del segmento PQ y la alternancia
elctrica, la cual indica la presencia de derrame pericrdico, pero no es concluyente. El ecocardiograma es el mtodo diagnstico que
ms informacin aporta. En primer lugar, detecta la presencia de derrame pericrdico y permite hacer una aproximacin de su
cantidad, ofreciendo adems mltiples signos de compromiso hemodinmico. La manera ms simple de documentar el taponamiento
cardaco es por medio de un catter colocado en la aurcula derecha, con el cual se pone en evidencia el aumento de la PVC y se puede
obtener una curva que revele los cambios antes mencionados en el pulso venoso. Con el catter de Swan-Ganz se pueden obtener,
adems, curvas de presin de la arteria pulmonar, mediciones de la presin pulmonar en cua y una estimacin del gasto cardaco por
termodilucin, lo cual es de gran utilidad en el seguimiento y la evolucin de los pacientes ms graves. TRATAMIENTO: Extraccin de
lquido para eliminar la compresin del corazn, por medio de la pericardiocentesis o por mtodos quirrgicos. La eleccin del tipo
depende de la disponibilidad de personal capacitado, de medios de ayuda pero, principalmente, de la enfermedad que llev al
taponamiento cardaco. Se prefiere la ciruga en los casos de hemopericardio en que se desea evitar la repeticin de hemorragias y est
indicada en la pericarditis purulenta, con el fin de realizar drenaje completo y controlar la infeccin. La pericardiocentesis con gua
ecocardiogrca u otro po de imagen, como la fluoroscopia o la TC, es buena alternativa en prcticamente todas las enfermedades
pericrdicas. No debe olvidarse la probabilidad de recurrencia del taponamiento cardaco cuando se trata con pericardiocentesis, por lo
cual deber repetirse el procedimiento o hacer ciruga. Son de utilidad medidas teraputicas de sostn como los lquidos intravenosos,
en especial en casos de hemopericardio traumtico agudo, cuando el paciente est hipovolmico. Esta medida aumenta el volumen
CAUSAS MDICAS DE TAPONAMIENTO
CARDACO
Taponamiento cardaco agudo
Hemopericardio
Ruptura de la pared libre del ventrculo
postinfarto de miocardio
Aneurisma artico roto a cavidad
pericrdica
Diseccin artica al pericardio
Neoplasias (CA metastasico de pulmn y
seno, linfoma, leucemias, mesotelioma)
Tratamiento anticoagulante
Postoperatorio ciruga cardaca (fugas,
anicoagulacion, aneurismas, disecciones)
Pericarditis aguda
Bacteriana
Tuberculosa
Urmica
Enfermedades del tejido conectivo
Taponamiento cardaco crnico
Causas infecciosas. Tuberculosis. Algunas
parasitosis. Causa no infecciosa. Urmico.
Neoplsico (posirradiacion) Mixedematoso.
Postirradiacin. Sndrome de Dressler.
Sndrome postpericardiectoma.
Quilopericardio
MANUAL DE TRABAJO DEL CURSO ENARM CMN SIGLO XXI
CURSO ENARM CMN SIGLO XXI TEL: 36246001 Pharmed Solutions Institute PGINA 208

circulante, incrementa las presiones venosa central y pulmonar y la presin arterial. En ningn caso deben aplicarse vasodilatadores,
como el nitroprusiato de sodio por va intravenosa. Tratamiento quirrgico por tres mtodos. El primero pericardiectoma por
esternotoma vertical o transversa, til en pacientes con posible constriccin (radiacin o tuberculosis). El segundo la pericardiectoma
parietal a travs de toracotoma intercostal izquierda, que se lleva a cabo bajo anestesia general y posibilita la reseccin de gran parte
del pericardio parietal anterolateral, lo cual permite una mejora ms definitiva. Es de utilidad en caso de pericarditis purulenta. El
tercero la pericardiostoma subxifoidea, que se efecta bajo anestesia local con reseccin del apndice xifoides y una incisin pequea
en el pericardio, a travs de la cual se drena el lquido y se conoce como procedimiento de ventana se puede liberar adherencias
pericrdicas, obtener biopsias del pericardio y dejar una sonda de drenaje, en caso de taponamiento no constrictivo y reversible
(pericarditis por dilisis o idioptica). La pericardiocentesis tiene ventajas como su rpida aplicabilidad, la facilidad de estudiar el
lquido, la factibilidad de combinarla con estudios hemodinmicos y su eficacia en las 2/3 partes de los casos. Las desventajas consisten
en la necesidad de personal bien capacitado en la tcnica y de ecografa de buena calidad para obtener mejores resultados; adems, no
siempre permite llegar a un diagnstico que requiera biopsia pericrdica, no garantiza su eficacia en todos los casos, puede retardar la
intervencin quirrgica por el alivio temporal que proporciona y puede producirse hemopericardio por la puncin del corazn. Sin
embargo, la pericardiocentesis puede llegar a ser una medida salvadora en los casos de taponamiento cardaco agudo. Se aconseja
como premedicacin a este procedimiento la administracin de atropina (0,8 a 1,0 mg) por va intravenosa o intramuscular, a fin de
prevenir reacciones vasovagales. La mejor ruta de acceso para la pericardiocentesis es la subxifoidea, pues evita lesiones de las arterias
coronarias. Previa asepsia con alcohol yodado y bajo anestesia local (en los casos urgentes se omite la anestesia) se efecta la puncin
5 cm por debajo de la punta del apndice xifoides y 1 cm a la izquierda de la lnea media, con la aguja en ngulo de 45 grados y dirigida
hacia el hombro izquierdo. Se ejerce succin continua mientras se introduce la aguja, detenindose cuando se perciba una sensacin de
vencimiento de una resistencia lo cual indica la entrada al espacio pericrdico, hasta que se obtenga lquido o cuando se sientan las
pulsaciones cardacas transmitidas a la aguja, lo que significa que se ha tocado el miocardio y se debe retirar un poco la jeringa. El
lquido pericrdico de aspirarse con lentitud durante 10 a 30 minutos y enviarse a estudio al laboratorio clnico de acuerdo con el caso.
El condiciones electivas puede emplearse el electrocardiograma para evitar lesiones del miocardio. En caso de taponamiento cardaco
por herida del corazn, la extraccin de algunos centmetros de sangre mejora el retorno venoso y puede salvar al paciente. La sangre
obtenida no coagula porque los movimientos del corazn la desfibrinan rpidamente; si la sangre obtenida se coagula es porque se
puncion una cavidad del corazn. El tratamiento mdico del taponamiento cardaco agudo, incluido el soporte inotrpico, con o sin
vasodilatadores, es relativamente controversial. La dobutamina, administrada para revertir la hipotensin, es tericamente ideal.
Durante el taponamiento, de cualquier manera, la esmulacin endgena inotrpica del corazn es muchas veces mxima.

CASOS CLINICOS
Mujer de 81 aos, sin antecedentes de inters, que ingres con el diagnstico de insuficiencia cardiaca desencadenado por infeccin
respiratoria, con un cuadro inespecfico de 1 mes de evolucin, de malestar y astenia, y los das previos al ingreso haba aparecido
disnea de esfuerzo, que llegaba a ser de reposo. Refera tos escasa, sin expectoracin y sin fiebre. Laboratorios: hemoglobina, 11,9 g/
dl, con hematocrito, 36,8%, y leucocitosis, con 16,8. 705,109 plaquetas/l, y fibringeno, 464 mg/dl. La radiografa de trax mostraba
cardiomegalia con ligera redistribucin en ambas bases. En el ECG, reduccin en el voltaje del QRS y un aplanamiento difuso de las
ondas T. A las 24 h present un deterioro clnico con datos clnicos y hemodinmicos, hipotensin, pulso paradjico y datos de presin
venosa elevada.

PREGUNTA
Cul es la conducta a seguir mas apropiada.

RESPUESTA
a.- Pericardiocentesis.
b.- Pericardientomia urgente.
c.- Manejo conservador.
d.- Correccin del estado hemodinamico.

CASO CLINICO
Mujer de 69 aos de edad que sufre un trauma torcico cerrado por compresin antero posterior al quedar atrapada por las puertas de
un autobs y 30 das despus comienza a presentar falta de aire a los esfuerzos, aumentando progresivamente hasta desencadenarse a
los pequeos esfuerzos. Ingresa en el hospital con diagnstico de cardiopata isqumica. Se realiza ecocardiograma y se comprueba
gran coleccin lquida pericrdica que se evacua parcialmente mediante pericardiocentesis, pero al profundizarse los sntomas de
hipovolemia se decide realizar pericardiectoma anterior radical de urgencia a travs de una toracotoma antero lateral izquierda y se
constata el pericardio engrosado y tenso. Se comprueba el diagnstico por puncin pericrdica transoperatoria por la presencia de
sangre y se procede a descomprimir lentamente la coleccin intrapericrdica. Se completa la pericardiectoma y se diagnostica una gran
contusin miocrdica con movimiento cardiaco lento. La evolucin temporalmente es satisfactoria con estabilizacin del
electrocardiograma aunque tres meses despus sufre infarto del miocardio y fallece.

PREGUNTA
Cual es el porcentaje de pacientes con aplastamiento torcica desarrollan tamponade cardiaco?

RESPUESTA
a.- 20%.
b.- 30 %.
c.- 40 %.
d.- 50 %.
MANUAL DE TRABAJO DEL CURSO ENARM CMN SIGLO XXI
CURSO ENARM CMN SIGLO XXI TEL: 36246001 Pharmed Solutions Institute PGINA 209


PACIENTES DE ALTO RIESGO DE
DISECCION AORTICA
Enfermedad vascular ateroesclertica
Anuloectasia artica
Aneurisma artico
Ulcera aortica
Calcificacin de la intima
Alteraciones genticas (conectivopatias)
Sndrome de Marfan
Sndrome de Ehlers-Danlos
Sndrome de Turner
Enfermedad de Noonan
Osteognesis imperfecta
Enfermedades congnitas
Coartacin aortica
Valvula artica unicspide o bicspide
Enfermedades degenerativas
Hipertensin
Causas traumticas
Lesin por desaceleracin
Lesin penetrante
Enfermedades inflamatorias
Sfilis
Arteritis de clulas gigantes
Iatrogenia
Cateterismo cardiaco
Canulacin artica o femoral previa
Baln de contrapulsacin intraartico
DISECCION DE AORTA:
CIENCIAS BASICAS: Consiste en la separacin de la capa media de la pared aortica en la que penetra sangre precedente de la aorta con
lo que se establece una falsa luz que puede comprimir la luz verdadera del vaso. Entre ambas encontramos el colgajo intimal. Esta
separacin es consecuencia de una rotura o perforacin en la intima (puerta de entrada) que se propaga distalmente. La adventicia
puede contener inicialmente el sangrado o evolucionar a la rotura, Alrededor de 95% de roturas ocurren en la aorta ascendente, distal
a la vlvula aortica. Dependiendo de la localizacin y la extensin pueden
aparecer un taponamiento cardiaco, hemotorax, una insuficiencia aortica o un
sndrome de mala perfusin. Este ultimo segn los troncos arteriales afectados, se
puede manifestar como un sndrome coronario agudo, una afeccin neurolgica o
visceral (por afeccin de troncos supraarticos, arteriales medulares y ramas
viscerales), o una isquemia de los miembros inferiores. Los factores
predisponentes mas frecuentes son: hipertensin (72%) y ateroesclerosis (31%).
En pacientes menores de 60 aos encontramos generalmente un aneurisma de
aorta, una ciruga cardiaca previa, un sndrome de Marfan o una vlvula aortica
bicspide.
DIAGNOSTICO: Clnico: el dolor es la caracterstica ms importante, es una
sensacin de desgarro agudo e intenso, migratorio. La intensidad del dolor es
constante, por lo que se diferencia del dolor secundario a IAM. Un dolor
retroesternal con irradiacin a cuello es caracterstico de la afeccin de la aorta
ascendente, mientras que el dolor dorsolumbar orienta hacia una afeccin de la
aorta torcica descendente. El sincope puede estar causado por dolor intenso, la
obstruccin de los vasos cerebrales, la activacin de barorreceptores articos o un
taponamiento cardiaco. En funcin de los vasos afectados habr manifestaciones
de mala perfusin. Una diseccin origina una obstruccin dinmica de los troncos supraarticos, que se puede manifestar con un
cuadro de isquemia cerebral y asimetra o ausencia de pulsos en extremidades superiores. El desgarro de las arterias intercostales se
puede mostrar como un dficit motor en los miembros inferiores. El compromiso de las arterias viscerales con dolor abdominal
(isquemia medular) o con aligoanuria (arterias renales). Los pulsos femorales pueden estar disminuidos o ser asimtricos. Otras
manifestaciones menos frecuentes son hemoptisis, sndrome de vena cava superior, obstruccin de la va area y sndrome de Horner
(compresin por el aneurisma o hematoma en el cayado), disfona. La ausencia de pulsos,
presencia de sincope previo, signos neurolgicos, estado de Shock y hemotorax se
consideran factores de mal pronstico. El diagnostico de diseccin aortica aguda debe
considerarse en todo paciente con un cuadro de dolor abdominal agudo, isquemia de las
extremidades, en caso de sincope o un cuadro de insuficiencia cardiaca aguda no explicada.
Recientemente se ha publicado el score de Diseccin Aortica en Urgencias, valora la
presencia de dolor en espalada, una relacin toraco-mediastinica mayor a 30%, una
insuficiencia aortica y un dimetro artico mayor de 30mm por ecografa. La sensibilidad y
especificidad de este score es de 93 y 77% respectivamente, si estn presentes 3 o ms de
estos signos. Si se sospecha una diseccin aortica y no hay disponibilidad diagnostica ni de
tratamiento, debe asegurarse la remisin rpida del paciente. Rx de trax; suele mostrar
ensanchamiento mediastinico o derrame pleural asociado. ECG: es normal en 30%, cambios
isqumicos, signos de hipertrofia VI. Analtica sangunea: anemia ligera, leucocitosis,
trombopenia, alteracin de pruebas de funcin heptica, elevacin de Cr srica. Elevacin de
reactantes de fase aguda. Acidosis metablica en casos de mala perfusin. Especficos:
Ecocardiografa transtorcica (ETT), identifica diseccin aortica proximal, una insuficiencia
aortica asociada y un taponamiento cardiaco, permite evaluar la funcin miocrdica y medir
los dimetros de la raz aortica. Ecocardiografa transesofgica (ETE), mas sensible y
especifica, confirma diseccin en un 90% de los casos y permite localizar la rotura intimal en
la mayora de los casos y valorar el flujo sanguneo en la verdadera y falsa luz. TAC, tiene
sensibilidad superior al 95% y una especificidad del 87%, da informacin precisa de los
diferentes dimetros de los segmentos articos. Permite evaluar, con precisin la extensin
de la aorta afectada y mostrar la implicacin de las arterias viscerales e iliacas. Tiene como
inconveniente la nefrotoxicidad, el contraste y la necesidad de desplazar al paciente.
TRATAMIENTO: Actuacin inmediata: La afectacin de la aorta ascendente requiere ciruga, mientras que la diseccin de la aorta
descendente precisa abordaje mdico en ausencia de complicaciones. Farmacolgico: Antes de la realizacin de cualquier medida
diagnostico-teraputica el paciente debe estar monitorizado con ECG, medida de presin arterial (cada 5min) y canalizacin de una va
venosa de buen calibre. La analgesia contribuye a estabilizar al paciente y en, general se da opiceos a dosis bajas (cloruro mrfico 3
mg IV cada 5-10 min), para evitar la progresin de la enfermedad y el riesgo de rotura aortica, se debe disminuir el estrs parietal
artico y controlar la presin arterial (objetivo TAS entre 90-110 mmHg, si se mantiene la diuresis). Los frmacos ms usados como
tratamiento antiimpulso, son los bloqueadores selectivos como: de vida corta esmolol IV carga 0.5mg/kg en 2-5 min, mantenimiento
0,10-0,20 mg/kg/min, vida intermedia atenolol IV, dosis carga 2.5 mg, mantenimiento 0,15 mg/kg/dia. No selectivos como: propanolol
dosis de carga 0.5-1 mg en 5 min, mantenimiento 0.05-0,015 mg/kg cada 4-6 hrs y los bloqueadores 12 (labetalol IV de vida media
larga en dosis crecientes 0.5-4 mg/min). En caso de hipertensin arterial se puede iniciar labetalol de primera eleccin. Si se requiere de
un control adicional de la presin arterial se suelen asociar nitroprusiato, nitratos, agonistas de calcio (nimodipino IV) o incluso IECAS
IV, Si los betas bloqueadores estn contraindicados (EPOC) hay que pensar en verapamilo o diltiazem. El tratamiento farmacolgico
crnico se reserva para los pacientes ya intervenidos con diseccin tipo B no complicada y requiere un control estricto de la presin
arterial, mantenindola por debajo de 130-135/80mmHg. El pilar del tratamiento son los bloqueadores beta, asociando IECA o
MANUAL DE TRABAJO DEL CURSO ENARM CMN SIGLO XXI
CURSO ENARM CMN SIGLO XXI TEL: 36246001 Pharmed Solutions Institute PGINA 210

antagonista de calcio, si se precisa o el paciente no tolera lo bloqueadores beta. Se debe realizar revisiones peridicas y pruebas de
imagen, se recomiendan a los 1, 3, 6, y 12 meses y posteriormente anuales. Tratamiento quirrgico: Objetivo salvar la vida del paciente
previniendo la rotura de la aorta torcica ascendente en el pericardio. El tratamiento de la diseccin aortica tipo A es quirrgico (I B),
excepto en situaciones de edad avanzada, comorbilidad importante o dao neurolgico. La diseccin tipo B tiene un mejor pronostico.
Su abordaje inicial es mdico y se reserva el tratamiento quirrgico en la fase aguda a las disecciones complicadas. TIPO A: Reemplazar
la aorta ascendente disecada, con un injerto sinttico de dacrn, la zona resecada debe incluir la zona con la rotura intimal. La
mortalidad quirrgica vara entre un 10-20% en funcin de la edad, la comorbilidad y la extensin de la diseccin. TIPO B: El tratamiento
mdico tienen buenos resultados, este se centra inicialmente en el control de la presin arterial mediante bloqueadores beta y
vasodilatadores, y el alivio del dolor. Una vez superada la fase aguda, se continan los bloqueadores beta orales y vasodilatadores. Un
seguimiento radiolgico cada 6 meses ser necesario para detectar precozmente una evolucin trpida. La ciruga queda limitada a:
dolor torcico persistente o recurrente, expansin aortica, hematoma periartico, hematoma mediastnico. En la actualidad, las
intervenciones endovasculares en la diseccin aortica aguda tipo B suelen limitarse a aliviar las complicaciones con riesgo de muerte.

CASO CLINICO
Se trata de paciente masculino de 46 aos de edad, gerente de una tienda departamental, contador, con antecedente de tabaquismo
positivo (una cajetilla diaria) es hipertenso desde hace 5 aos con mal apego a su tratamiento, acude a urgencias debido presencia de
dolor torcico que se irradia a la espalda, refiere que nunca haba presentado un dolor as de intenso, que inicio hace 6 horas por la
maana y se retiro de su trabajo porque comenz a sudar y a presentar nauseas sin llegar al vomito, el dolor se ha vuelto lacerante, el
trazo electrocardiogrfico se observa elevacin ST de 1 mm en DII y aVF el paciente se encuentra taquicardico con pulso carotideo
salton.

PREGUNTA
Cual es el diagnostico mas probable hasta el momento.

RESPUESTA
a.- Infarto al Miocardio.
b.- Diseccion aortica.
c.- Prolapso valvular.
d.- Ruptura papilar.

ANEURISMA AORTICO (AA):
CIENCIAS BASICAS: Los aneurismas de la aorta (AA) se definen como la dilatacin focal de la arteria que supone un aumento de ms
de 50% del dimetro esperado, basado en medidas medias obtenidas en estudios con TAC en poblacin general. En el caso de la aorta
abdominal correspondera a un dimetro superior a 3 cm. Tambin se habla de AA como una dilatacin localizada de por lo menos 1.5
veces mayor al dimetro normal de la arteria; puede ser sacular o fusiforme, y ambas estn adyacentes a un segmento de arteria
normal. Esta patologa se ha convertido en el motor del tratamiento quirrgico preventivo de la ciruga vascular. Decimos que existe un
aneurisma verdadero cuando afecta a las tres capas histolgicas; pero cuando la ntima y la media estn rotas y la dilatacin es a
expensas solo de la adventicia, entonces hablamos de seudoaneurisma. La aorta abdominal es el vaso donde se localizan con ms
frecuencia los aneurismas arteriales. Hay aneurismas articos abdominales (AAA) y aneurismas articos de torcica descendente (AAT).
SALUD PBLICA: De los aneurismas articos 80% se ubican en el abdomen y tienen una presencia en la poblacin de la tercera edad de
2 a 7%. Es mayor en hombres que en mujeres, en una proporcin de 4:1. En pacientes con hipertensin arterial, enfermedad vascular
cerebral u otros aneurismas, su presencia aumenta 40%. Los AAA afectan a ms de 3% de la poblacin de edades superiores a los 50
aos. Los factores de riesgo, como hipertensin arterial, tendencia familiar (se relaciona con cromosoma , autosmico, predisposicin
gentica), tabaquismo, diabetes, infecciones, dislipidemia y los relacionados con la ateroesclerosis, aumentan su prevalencia
significativamente. El riesgo proporcional para las personas que tienen un familiar de primer grado con AAA, es de 6:1 en comparacin
con la poblacin general. Como en un elevado porcentaje son asintomticos, y la ruptura es la primera manifestacin clnica, la
mortalidad real es muy alta. La probabilidad de ruptura al ao del AAA menor de 5 cm es de 0.4-5.4%, entre 5-6cmde 20-25%, y de
>7cm entre 40-80%. PATOGENIA: La aorta es un conducto a travs del cual la sangre impulsada, desde el ventrculo izquierdo, pasa y se
distribuye en el lecho arterial sistmico. El dimetro mximo en adultos corresponde en su origen a 3 cm, disminuyendo caudalmente,
siendo de 2.5 cm a nivel de la porcin de aorta descendente torcica, hasta 1.8 a 2 cm en la porcin abdominal de la misma. La pared
vascular est formada por la nma delgada, compuesta de endotelio, capa subendotelial de tejido conjuntivo y una capa interna
elstica; la tnica media, de clulas musculares lisas y matriz extracelular y una adventicia formada fundamentalmente por tejido
conectivo, que engloba los vasa vasorum y la inervacin del vaso. Debido a su exposicin contina a la presin pulstil y fuerzas de
sujecin, est parcularmente expuesta a sufrir lesiones secundarias a trauma mecnico, sobre todo en los casos de desaceleracin,
siendo mayor el riesgo de ruptura y de aparicin de dilataciones aneurismticas. Desde el punto de la fsica, hay dos factores que
generan los AA: la excesiva aplicacin de una fuerza interna y la inadecuada resistencia del material. Se piensa que el origen de los AAA
y AAT inespecficos, es una disminucin en la cantidad de elastina de la pared arterial. La cantidad normal de elastina que se encuentra
en las personas normales es de 12%, porcentaje que baja a 1% o menos en los pacientes con aneurismas detectado a travs de un
marcador gentico; el aumento de la actividad de la elastasa y de la colagenasa afectan al desarrollo del aneurisma. Se ha encontrado
un metabolismo anormal de la elastina y del colgeno, mayor turbulencia o disminucin de vasa vasorum en esa zona,. Los AAA y AAT
principalmente, se deben a atero y arteriosclerosis; en menor proporcin trastornos del tejido conjuntivo, traumatismos, sndrome de
Marfan, infeccin, necrosis qustica de la media y arteritis. Tambin pueden ser por fibrodisplasias o iatrognicos. En la mayora de los
casos no es posible identificar la causa. El AAA inflamatorio, que representa 10% de todos los aneurismas articos, puede deberse a una
infeccin o a alguna otra forma oscura de arteritis. La evolucin natural de los aneurismas no tratados es hacia la expansin y la
ruptura siguiendo la Ley de Laplace. CLASIFICACION: En los AAT (Crawford), tiene en cuenta la longitud de la aorta afectada, resultando
en cuatro patrones que determinan la extensin de la intervencin quirrgica, el resultado del tratamiento y la naturaleza de las
MANUAL DE TRABAJO DEL CURSO ENARM CMN SIGLO XXI
CURSO ENARM CMN SIGLO XXI TEL: 36246001 Pharmed Solutions Institute PGINA 211

complicaciones. Tipo I; est afectada la mayor parte de la aorta descendente torcica y la parte proximal de aorta abdominal. Tipo II; el
aneurisma afecta gran parte de la aorta descendente y la mayor parte o toda la aorta abdominal. Tipo III; afecta la aorta torcica distal
y la totalidad de aorta abdominal. Tipo IV la mayor parte de la aorta abdominal, incluyendo el segmento de vasos viscerales.
DIAGNOSTICO: Clnico; Una minora presenta dolor a la palpacin abdominal o ms comn dolor lumbar confundible con clico renal.
Un bajo nmero de pacientes se presentan con dolor abrupto abdominal y en la regin lumbar. Al examen fsico se puede palpar, en
algunos casos, una masa pulstil en el abdomen. La ruptura en el aneurisma es una forma de presentacin con alta mortalidad, hay
extravasacin de sangre, la cual puede ser masiva (hacia la cavidad peritoneal) o contenida (en el retroperitoneo). La trada de
hipotensin, dolor abdominal y masa pulstil en el abdomen, es observada slo en 15% de los pacientes con aneurisma abdominal roto,
donde 78% de ruptura es hacia el retroperitoneo, y slo 22% se rompe en su pared anterior hacia la cavidad peritoneal. tra
manifestacin es la embolizacin distal de material trombotico que ocupa el saco aneurismco, y esto ocurre en 3 a 5% de los
pacientes. Es obligada la exploracin fsica del paciente mediante palpacin profunda abdominal, buscando en algunos casos una masa
pulstil en el mesogastrio, especialmente si el paciente es delgado. La presencia de soplos a la auscultacin debe hacernos sospechar
enfermedad oclusiva visceral o de aorta terminal y ms raramente la presencia de una stula aortocava con datos de falla cardiaca. El
estudio fsico debe completarse con palpacin de pulsos en extremidades, y en algunos casos pueden estar disminuidos o ausentes. Las
presentaciones clnicas pueden ser de un abdomen agudo por ruptura del AAA, embolias distales o sangrado de tubo digestivo cuando
se comunica la ruptura al duodeno. En los AAT los datos de compresin a bronquios, nervios o tubo digestivo pueden hacer sospechar
su presencia. Dx. De imagen: La mayora de los AA se diagnostican en forma incidental (80% de AAA), desde la Rx simple en trax, que
deforma el mediastino, o bien, la sombra de la aorta torcica descendente se hace ms evidente. En la Rx de abdomen, adems de las
calcificaciones se puede observar borramiento de las lneas del msculo psoas. Asimismo, tanto en una evaluacin ultrasonogrca,
TAC, RNM. El ultrasonido representa una modalidad econmica, de fcil acceso y es tambin precisa para la medicin del aneurisma.
Este estudio no es sensible para el diagnstico de hemorragia retroperitoneal ni para la medicin de la distancia que separa el
aneurisma de las arterias renales o las arterias iliacas. TRATAMIENTO: La mayora de los centros quirrgicos estn de acuerdo en que
todo aneurisma de 5 cm o mayor debe ser operado en forma electiva o tratado en forma endovascular. Otra tendencia prefiere el
seguimiento cada seis meses e, independientemente del tamao del aneurisma, un crecimiento mayor de 0.5 cm en este periodo es
indicacin de tratamiento. Es conocido el riesgo quirrgico por enfermedad coronaria coexistente en estos pacientes. La modificacin
de los factores de riesgo es fundamental, en el manejo medico de los AAA, la hipercolesterolemia y la HTA deben controlarse
adecuadamente. Los betabloqueadores (propanolol) han sido considerados como terapia para reducir el riesgo de expansin y rotura
del aneurisma. Una vez sospechado el diagnostico debe colocarse un acceso venoso de grueso calibre e infusin de cristaloides,
evitando o corrigiendo la situacin de shock inicial; esta medida est directamente relacionada con la morbimortalidad. La mayora de
los autores coinciden en que deber tener como finalidad mantener tensin arterial sitolica en torno a 80-100mmHg. En las tcnicas
quirrgicas se puede reparar con injertos sintticos de dacrn o PTFE (politetrafluoroetileno), en forma recta o bifurcada hacia las
iliacas o hacia las femorales, con resultados variables, este procedimiento quirrgico seguir teniendo vigencia, ya que 40% de los
pacientes no son candidatos para ciruga endovascular, por variantes anatmicas que dificulten el implante, pacientes jvenes con
enfermedades asociadas: renal, pulmonar, cardiaca, carotidea de bajo riesgo que toleren bien la ciruga convencional. Manejo
endovascular: se desarrollo para hacerla va transfemoral, siendo un procedimiento combinado, a travs de la cual pasa la prtesis
montada del sistema para ser colocada bajo fluoroscopia a nivel del aneurisma. La eleccin de los pacientes es la clave del xito. Existe
un protocolo estricto que debe incluir mediciones precisas del dimetro de las arterias iliacas.

CASO CLINICO
Se trata de paciente masculino de 59 aos de edad el cual se encuentra bajo tratamiento por padecer hipertensin arterial desde hace
10 aos, adems hipercolesterolemia, actualmente recibe clortalidona, atenolol y pravastatina, se encuentra apegado a tratamiento sin
embargo continua fumando casi una cajetilla diaria. Acude a su consulta de revisin durante la cual le comenta que ha sentido una
bolita que pulsa a la altura del ombligo y quiere descartar un padecimiento cancergeno ya que en la familia hay esos antecedentes, a la
exploracin se encuentra con sus constantes vitales controladas, ninguna alteracin cardiopulmonar, al revisar abdomen palpa una
masa depresible no dolorosa pero pulsatil a nivel de cicatriz umblical de al menos 3 dedos de longitud.

PREGUNTA
Cul es su conducta a seguir.

RESPUESTA
a.- Solicita USG.
b.- Solicita TAC.
c.- Solicita IRM.
d.- Envia para LAPE.

CASO CLINICO
El paciente regresa con los resultados de la tomografa solicitada donde se reporta aneurisma de la aorta abdominal de 4.5 cm
infrarrenal, el paciente agrega que recuerda que desde hace algunos meses ha notado dolor ocasional de espalda que ha sido tratado
por su medico familiar con diclofenaco y medidas locales.

PREGUNTA
Cual es la conducta mas apropiada a seguir en este caso.

RESPUESTA
a.- Realizar estudios para intervencin quirrgica.
b.- Continuar con diclofenaco.
MANUAL DE TRABAJO DEL CURSO ENARM CMN SIGLO XXI
CURSO ENARM CMN SIGLO XXI TEL: 36246001 Pharmed Solutions Institute PGINA 212

c.- Vigilar al paciente.
d.- Vigilancia y antiagregante plaquetario.

CASO CLINICO
El paciente ha presentado algunas ocasiones episodios de descontrol hipertensivo, continua su habito de tabaquismo, los niveles de
colesterol no han cedido, el paciente refiere que ha cursado con mareos ocasionales acompaados de zumbido de odos y luces
brillantes, ha mejorado su alimentacin pero refiere aumento del dolor de espalda y sensacin de presin interna como si le pulsara,
debido a los cambios observados se repite la TAC abdominal donde se reporta un incremento en las dimensiones del aneurisma de la
aorta abdominal pasando de 4.5 cm hace 2 aos a 6.7 cm.

PREGUNTA
Considerando las nuevas evidencias el paciente es sometido a ciruga, cual es el criterio ms importante para el tratamiento elegido.

RESPUESTA
a.- El tamao del aneurisma.
b.- La presencia de dolor persistente.
c.- La presencia de descontrol hipertensin.
d.- Incremento de placas ateromatosas.

ANGINA ESTABLE, INESTABLE Y PRINZTMETAL
DEFINICION: La angina estable es un sndrome clnico caracterizado por malestar en el pecho, mandbulas, hombros, espalda o brazos,
que aparece con el ejercicio o estrs emocional, dura de 2 a 5 minutos, remite espontneamente, con el descanso o con la
administracin de nitroglicerina, (las manifestaciones clnicas aparecen cuando al menos hay una oclusin del 70 % de la luz del vaso
afectado. EPIDEMIOLOGIA: En ambos sexos, la prevalencia de la angina aumenta marcadamente con la edad, de un 0,1-1% en mujeres
de edades comprendidas entre 45 y 54 aos a un 10-15% en mujeres de 65-74 aos y de un 2-5% en varones de 45-54 aos a un 10-
20% en varones de 65-74 aos. Con base en estos datos, se calcula que, en la mayora de los pases europeos, entre 20.000 y 40.000
individuos por milln de habitantes sufren angina. El principal factor de riesgo es la aterosclerosis. ETIOLOGIA: No hay factores
espeficificos solo relativos como aterosclerosis, hipertensin, dislipidemia, enfermedad arterial coronaria, alteraciones vasculares,
retinianas, soplos, infarto, presin yugular edema pulmonar, soplo mitral. FISIOPATOGENIA: Los datos anginosos se presentan cuando
la demanda de oxigeno por el miocardio se incrementan, y no existe una respuesta coronaria adecuada para mantener el aporte, por lo
regular se encuentran los sntomas anginosos cuando hay una obtruccion coronaria cercana al 90 % con una distensibilidad reducida de
la coronaria afectada. Frecuentemente los pacientes con obsetrucciones menores a 70 % no muestran sntomas anginosos. CUADRO
CLINICO: Sensacion de opresin, tensin o pesadez y sensacin de estrangulamiento, constriccin o quemazn. La severidad puede
variar mucho. Falta de aire y malestar torcico, sntomas como fatiga, nusea, eructos, ansiedad sensacin de muerte inminente.
DIAGNOSTICO: Angina estable: El episodio de angina es breve, generalmente menos de 10 min. Relacin con el ejercicio, una actividad
o el estrs emocional y remitan rpidamente en unos minutos cuando desaparezcan los factores causales. Otra caracterstica tpica de
la angina es la acentuacin de los sntomas despus de una comida pesada o a primera hora de la maana. Los nitratos orales o
sublinguales alivian rpidamente la angina. Angina inestable: a) angina en reposo (durante el reposo y perodos prolongados, 20 min);
b) angina in crescendo (angina estable previa que evoluciona rpidamente en severidad e intensidad y con un umbral ms bajo durante
un perodo corto, de 4 semanas o menos), y c) angina de reciente aparicin (episodio reciente de angina severa que limita
marcadamente la actividad diaria del paciente y se presenta durante los 2 meses posteriores al episodio inicial). En la angina inestable
hay evidencia de placas con rotura, hemorragia o laseracion lo cual las vuelve mas inestables y trombogenicas. Angina de Prinzmetal:
Presentan dolor de localizacin tpica, que ocurre en reposo o slo ocasionalmente, durante el ejercicio. Puede coincidir con la angina
tpica de esfuerzo por lesiones coronarias fijas. El vasoespasmo puede ocurrir como respuesta al consumo de tabaco, alteraciones
electrolticas (potasio, magnesio), consumo de cocana, estimulacin por fro, enfermedades autoinmunitarias, hiperventilacin o
resistencia a la insulina. El ECG no es concluyente, prueba de esfuerzo en banda sin fin, ecocardiograma de esfuerzo, ecocardiograma
con estimulacin con dobutamina, coronariografia (estndar de oro), cuando hay datos anormales, biomarcadores protena C de alta
especificidad y pptido natriuretico cerebral. TRATAMIENTO: Antianginosos BB; reducen el consumo de O2, disminuyen TA, FC y
contractibilidad, bloqueadores de canales de calcio, aumentan el flujo coronario y disminuyen el consumo de O2, verapamil o diltiazem
pero con mayor de 30% de FE o disfuncin sinusal o nodal; nitritos: vasodilatadores venosos con cierto efecto coronario, disminuyen
precarga, isosorbide o nitroglicerina de 0.2 a 0.6 mg sblingual responde en 1 a 3 minutos y puede repetirse cada 5 minutos. Aspirina
150mg, clopidrogel si hubo intervencionismo, atorvastatina 80 mg/dia e IECA. PRONOSTICO: En varones y mujeres con una
presentacin clnica inicial de angina estable, la incidencia de infarto de miocardio no fatal y muerte por enfermedad coronaria a los 2
aos era del 14,3 y el 5,5% en varones y del 6,2 y el 3,8% en mujeres, respectivamente. La tasa anual de mortalidad vara entre el 0,9 y
el 1,4% por ao 5-9, con una incidencia anual de infarto de miocardio no fatal del 0,5% y el 2,6%. COMPLICACIONES: En caso de no
obtener buenos resultado o si continua la evolucin trpida de la angina se podr realizar angioplastia o ciruga de derivacin cardiaca,
sin haber evidencia de sobrevida mayor con cualquier tcnica.

CASO CLINICO
Se trata de paciente masculino de 52 aos de edad, el cual acude a consulta externa debido a que desde hace 6 meses presenta
opresin toracicca, refiere que considera que es por las presiones que tiene en el trabajo y en casa se encuentra en proceso de divorcio,
refiere que el dolor dura menos de 5 minutos, desapareciendo respirando profundamente y reposando, agrega que ltimamente ha
presentado dolor epigstrico acompaado de nauseas independiente del malestar en el torax para lo cual emplea anticidos casi diario,
cuenta con antecedentes de importancia por tabaquismo y alcoholismo semanal hasta la embriaguez, asi como padre finado por IAM,
madre viva con hipertensin arterial con tratamiento, tiene habitos alimenticios inadecuados, ricos en grasas y comida rpida, por el
trabajo y no realiza ejercicio a la exploracin fsica se observa con leve rubicundez facial, aliento al parecer alcoholico, presenta signo de
MANUAL DE TRABAJO DEL CURSO ENARM CMN SIGLO XXI
CURSO ENARM CMN SIGLO XXI TEL: 36246001 Pharmed Solutions Institute PGINA 213

Levine al interrogar sobre el sitio del dolor, queratosis seborreica importante y xantomas, al parecer tercer ruido en area precordial y
estertores subcrepitantes bibasales. Signos vitales TA 150/95 mmHg, FR 28 rpm, FC 104 lpm.

PREGUNTA
Cules son las manifestaciones clnicas de la enfermedad mas importantes para considerar un dianostico presuntivo?

RESPUESTA
a.- Presencia del signo de Levine.
b.- Dolor torcico.
c.- Disminucin del dolor con el reposo.
d.- La duracin del dolor.

PREGUNTA
Cul es la manifestacion clnica que no es criterios dignosticos para establecer la enfermedad del paciente?

RESPUESTA
a.- Molestia toracicca subesternal.
b.- Inicio por estrs o ejercicio.
c.- Disminuye con nitroglicerina.
d.- Sexo, mas frecuente en mujer.

PREGUNTA
Considerando la clasificacin de la enfermedad cual de los siguientes diagnosticos es el mas probable para el caso actual?

RESPUESTA
a.- Angina definitiva.
b.- Angina probable.
c.- Angina posible.
d.- Angina no posible

PREGUNTA
Considerando la clasificacin funcional de la Canadian cardiovascular society, cual es la clase actual del paciente?

RESPUESTA
a.- Clase I.
b.- Clase II.
c.- Clase III.
d.- Clase IV.

PREGUNTA
Cul es el diagnostico diferencial mas probable en este caso debido a los factores modificables de riego presentes?

RESPUESTA
a.- Angina de prinzmetal.
b.- Neuritis intersticial.
c.- Sindrome de la pared toracicca anterior.
d.- Enfermedad gastrointestinal funcional.

PREGUNTA
Se realizaron los siguientes axiliares diagnosticos, cual de los siguientes resultados no excluye el diagnostico actual?, revisar las
imgenes

RESPUESTA
a.- Prueba de esfuerzo.
b.- Electrocardiograma en reposo.
c.- Radiografia de torax.
D.- Arteriografia coronaria.

PREGUNTA
Antes de realizar al paciente una coronariografia se encontraron los siguientes resultados, previos cual de ellos no es criterio?

RESPUESTA
a.- Depresin del segmento ST >1 mm.
b.- Prueba de esfuerzo anormal.
c.- Sintomatologia presente.
d.- Resultados inconclusos.
MANUAL DE TRABAJO DEL CURSO ENARM CMN SIGLO XXI
CURSO ENARM CMN SIGLO XXI TEL: 36246001 Pharmed Solutions Institute PGINA 214


PREGUNTA
Durante las pruebas se encontraron los siguientes resultados de laboratorio en el paciente: Creatinina 1,24, Glucosa 224, Colesterol
310, Trigliceridos 420, HDL 29, Urea 41, Na 138, K 3,42, Troponina T-U: 2,8, CK 105, CK-MB 1,67, ALT-GPT 19, Hemograma y coagulacin
normal. En relacin a datos de laboratorio obtenidos cuales son los objetivos teraputicos mas importantes?

RESPUESTA
a.- Incrementar el HDL.
b.- Disminuir triglicridos y colesterol.
c.- Revascularizacin inmediata.
d.- Buscar protena C de alta sensibilidad.

PREGUNTA
Cules son las medidas teraputicas no farmacolgicas inicial en este padecimiento, para disminuir la demanda miocrdica de oxigeno?

RESPUESTA
a.- Propanolol.
b.- Hidroclorotiazida.
c.- Losartan.
d.- Enalapril

PREGUNTA
Considerando los datos de lpidos y el informe de la coronariografia, que medida farmacolgica es necesario agregar para estabilizar la
placa?

RESPUESTA
a.- Clopidrogel 75 mg cada 12 hrs.
b.- Atorvastatina 40 mg cada 12 hrs.
c.- Enalapril 20 mg cada 12 hrs.
d.- Aspirina 150 mg cada 24 hrs.

PREGUNTA
Cules son los siguientes esquemas teraputicos secuenciales antianginoso tiene un efecto favorable sobre la presin arterial, la
contractilidad y la frecuencia cardiaca ms adecuado?

RESPUESTA
a.- Prazocin 1 a 2 mg cada 24 hrs.
b.- Tamsulosina 0.4 mg cada 24 hrs.
c.- Clonidina 0.075 a 0.150 mg cada 24 hrs.
d.- Propranolol 40 a 80 mg en 24 hrs.

PREGUNTA
Cual de las siguientes comorbilidades es la menos frecuentes en la angina tpica como es el caso?

RESPUESTA
a.- EPOC.
b.- DM.
c.- Espasmo vasocoronario.
d.- Disfuncin erctil.

PREGUNTA
Considerando la fraccin de eyeccin del paciente, cual frmaco calcio antagonista puede usarse con cuidado comparado con el resto?

RESPUESTA
a.- Amlodipino.
b.- Verapamilo.
c.- Diltiazem
d.- Nifedipina

PREGUNTA
Considerando la fraccin de eyeccin del paciente, cual frmaco calcio antagonista puede usarse con confianza comparado con el
resto?

RESPUESTA
a.- Nitroglicerina.
b.- Isosorbide.
MANUAL DE TRABAJO DEL CURSO ENARM CMN SIGLO XXI
CURSO ENARM CMN SIGLO XXI TEL: 36246001 Pharmed Solutions Institute PGINA 215

c.- Nitrito de amilo.
d.- Nitrito de potasio.

PREGUNTA
Considerando la patogenia de la angina de pecho cual de los siguientes sntomas estn menos relacionados a la isquemia del miocardio
y si a la necrosis miocrdica?

RESPUESTA
a.- Disconfor o dolor en el pecho.
b.- Maxilar inferior brazo.
c.- Hombros y espalda.
d.- Elevacin de CPK-MB.

PREGUNTA
Se le solicito al paciente que usara un diario para registra el dolor, cual es la razn fisiopatolgica que el paciente presente mayor
frecuencia de dolor en la maana?

RESPUESTA
a.- Debido a presencia de apnea.
b.- Debido al ritmio endgeno de cortisol.
c.- Debido al ritmo endgeno de secresin de catecolaminas.
d.- Debido a la insensibilidad coronaria a vasoconstrictores.

PREGUNTA
Cual es la frecuencia de angina estable crnica al momento de solicitar consulta al especialista?

RESPUESTA
a.- 50 %.
b.- 60 %.
c.- 40 %.
d.- 70 %.

PREGUNTA
Cul es la edad ms frecuente comparada con el caso del paciente para presentar angina tpica vs angina atpica considerando adems
en sexo?

RESPUESTA
a.- Hombres de 50 a 59 aos.
b.- Hombres de 40 a 49 aos.
c.- Hombres de 60 a 69 aos.
d.- Hombres de 50 a 69 aos

PREGUNTA
Considerando la escala de Duke Treadmill Scores para calificar el riesgo estratificado no invasivo cul es la calificacin que presenta
actualmente?

RESPUESTA
a.- Riesgo alto (3 % de mortalidad anual).
b.- Riesgo muy alto (5 % de mortalidad anual).
c.- Riesgo intermedio (1-3 % de mortalidad anual).
d.- Riesgo bajo (menor a 1% de mortalidad anual).

CASO CLINICO
Se trata de masculino de 68 aos de edad al cual se encuentra diagnosticado y tratado por padecer angina tpica desde hace 5 aos, sin
embargo de 6 meses a la fecha el disconfort del torax ha sido moderado y en ocasiones durante el reposo, acude al servicio de
urgencias debido a que presenta opresin toracicca acompaado de dolor en hombro izquierdo que se iradia a el brazo, agrega que se
encontraba discutiendo cuando comenz las molestias, no cedi al reposo y duro ms del tiempo habitual, cuenta con los antecedentes
de hipertensin arterial desde hace 20 aos con tratamiento actual con captopril 25 mg cada 12 hrs, hidralazina 25 mg /12 hrs, hace 5
aos fue agregado propranolol, pravastatina y aspirina para el manejo de su angina inicialmente estable, a su ingreso se observa
diafortica, con palidez generalizada, ansiosa, con cianosis distal, sus constantes vitales fueron TA 105/70 mmHg, FC 101 lpm, FR 29
rpm.

PREGUNTA
Considerando el cuadro clnico cul es diagnostico ms probable hasta este momento?

RESPUESTA
MANUAL DE TRABAJO DEL CURSO ENARM CMN SIGLO XXI
CURSO ENARM CMN SIGLO XXI TEL: 36246001 Pharmed Solutions Institute PGINA 216

a.- Angina tpica descompensada.
b.- Angina atpica inicial.
c.- Infarto agudo al miocardio.
d.- Insuficiencia cardiaca por angina previa.

PREGUNTA
Cul de los siguientes auxiliares diagnostico es mas til en este momento para mantener una conducta clinica adecuada?

RESPUESTA
a.- Buscar biomarcadores de isquemia.
b.- Realizar electrocardiograma.
c.- Coronariografia.
d.- Programar prueba de esfuerzo con dobutamina.

PREGUNTA
Cul cual es su clase funcional con la clnica observada?

RESPUESTA
a.- Clase I.
b.- Clase II.
c.- Clase III.
c.- Clase IV

PREGUNTA
Se practicaron los siguientes estudios al paciente cual es su opinin sobre la radiografia y ECG?

RESPUESTA
a.- Datos de edema pulmonar.
b.- Datos de insuficiencia cardiaca.
c.- Datos de IAM.
d.- Datos de EPOC.

RESPUESTA
Cul es la conducta inmediata ms importante a seguir?

RESPUESTA
a.- Morfina, oxigeno, nitritos y antiagregantes.
b.- Diuretico, morfina, verapamilo y oxigeno.
c.- Diuretico, antiaginoso y trombolisis.
d.- Morfina, nitrito, trombolisis, enoxoparina.

SINDROME CORONARIO AGUDO I IAM SESS
DEFINICION: Conjunto de cuadros clnicos por los que se pone de manifiesto de forma aguda la isquemia miocrdica secundaria en
general, pero no exclusivamente, por ateroesclerosis coronaria, sin embargo tiene su base en la erosion de una placa aterosclertica de
las arterias coronarias, lo cual activa la coagulacin culminando con la formacin de un trombo oclusivo. EPIDEMIOLOGIA: 2.5 millones
de ingreso a urgencias, 16.6 millones de muertes, primera causa de muerte, 1 muerte cada minuto. Alta morbilidad e incapacidad. Se
estima que actualmente el 50% de todas las muertes en los pases de alto ingreso y el 28% de las muertes en los PBMI son el resultado
de ECV. En 2001, el 31% de todas las muertes en Latinoamrica (LA). Factores de riesgo cardiovascular clasicos: DM, HAS, Tabaquismo,
LDL, estilo de vida, genes. Factores de riesgo cardiovascular actual; Infeccioso, inflamatorio, homocisteina, protrombosis. ETIOLOGIA:
No existe un factor etiolgico especifico, son una serie de sucesos concurrentes, la presencia de una placa ateromatosa que se
erosiona, inicia una hemorragia que activa la coagulacin generando un fenmeno oclusivo, es conveniente observar que no es
necesario la presencia de angina previa, ni la presencia de grandes placas en las coronarias. FISIOPATOGENIA: Las placas ateromatosas
consideradas como susceptibles a la erosion, en el caso del SICA 1, principalmente la centrolipidicas y con una delgada capa de fibrina,
la desestabilizacin y consecuente rotura de la placa es consecuencia principal de la circulacin de factores inflamatorios locales y
sistmicos, asi como los cambios fisiolgicos de las arterias coronarias por demanda u otros fenmenos, al presentarse lisis de la
delgada capa de fibrina se activa la cascada de coagulacin, adeshion y agregacin plaquetaria por vol willebrand, glucoproteinas
IIb/IIIa, tromboxano A, epinefrina, serotonina y factor activador de plaquetas, aumentando la afinidad de la protrombina y el
fibringeno convirtindose en trombina y fibrina respectivamente consecuentemente acumulo de eritrocitos y plaquetas formando un
trombo el cual no necesariamente es oclusivo, de ello depender en gran medida las modificaciones del ECG. CUADRO CLINICO: El
paciente refiere dolor opresivo o transfictivo retrosternal, con irradiacin a cuello, brazo o mandibula, sntomas agregados, nauseas,
vomito, diaforesis, y sensacin de muerte inminente. DIAGNOSTICO: Se observan manifestaciones clnicas referidas, alteraciones
electrocardiogrficas y biomarcadores, (troponinas y CPK-MB) que no se elevan inmediatamente. TRATAMIENTO: Inmediato:
evaluacin general, toma e interpretacin de ECG, bsqueda de factores para fibrinoliticos, toma de biomarcadores y rutina.
Secundario: morfina, oxigeno, nitritos y antiagregantes. Terapia de reperfusion: en caso de no existir contraindicaciones y por
factibilidad se indica terapia fibrinolitica; debe realizarse dentro de las primeras 12 horas de iniciado el cuadro. PRONOSTICO: El manejo
firbinolitico favorece del 60 al 80 % de los pacientes, es bueno cuando se realiza mas temprano, adems de administrar heparina para
MANUAL DE TRABAJO DEL CURSO ENARM CMN SIGLO XXI
CURSO ENARM CMN SIGLO XXI TEL: 36246001 Pharmed Solutions Institute PGINA 217

evitar la reoclusiones. COMPLICACIONES: La arritmias cardiacas sin una de las complicaciones mas importantes, asi como el choque
cardiogenico, otras complicaciones vienen del uso de fibrinoliticos.

CASO CLINICO
Mujer de 66 aos, hipertensa de 10 aos de evolucin con atorvastatina y enalapril actualmente con adecuado control, acudi por
opresin torcica en reposo de 2 h de evolucin. Al ingreso se observo con leve palidez de tegumentos con sensacin de ahogo y disnea
aun con oxigeno suplementario, TA de 165/88mmHg, obesidad troncular y soplo sistlico II/VI en focos de la base. El ECG mostr
elevacin del ST en cara inferior y de V2 a V6. Se realiz fibrinolisis, se encontraba en las primeras 2 hrs de evolucin. Se administr
enoxaparina, cido acetilsaliclico, nitroglicerina y morfina. Un ecocardiograma urgente demostr acinesia del pex y la cara anterior, y
se estim una fraccin de eyeccin del 40%. Se observ un ligero derrame pericrdico posterior.

PREGUNTA
Cual de las manifestaciones del cuadro clnico es el menos importante para la decisin para realizar fibrinlisis en este caso?

RESPUESTA
a.- Antecedente de angina inestable.
b.- El tiempo de inicio del cuadro clnico.
c.- La auscencia de factores de riesgo a la fibrinlisis.
d.- La presencia de cambios electrocardiogrficos.

PREGUNTA
Cul de los antecedentes del caso es el de menor importancia para el actual IAM?
a.- Hipertension arterial previa.
b.- Antecedente de angina.
c.- Sobrepeso.
d.- Uso de atorvastatina subterapeutica.

PREGUNTA
Considerando los cambios electrocardiogrficos, cuales de las arterias es la menos probable que se encuentre involucrada.

RESPUESTA
a.- Desendente anterior (ramos septales).
b.- Desendente anterior (ramos diagonales).
c.- Circunfleja.
d.- Desendente posterior.

PREGUNTA
Cul es la complicacin mas probable considerando el sitio de compromiso coronario?

RESPUESTA
a.- Bloqueos de rama.
b.- Disfuncin ventricular izquierda.
c.- Insuficiencia cardiaca.
d.- Hipotensin.

PREGUNTA
Considerando las caractersticas del paciente, cual de los siguiente frmacos favorece la reduccin de la extensin del dao miocrdico,
ruptura miocrdica o reinfarto?

RESPUESTA
a.- Nitritos.
b.- Betabloqueadores.
c.- IECAS.
d.- Calcioantagonistas.

PREGUNTA
Cul de los cambios bioqumicos no son factores que desencadenan los cambios de actividad plaquetaria en este caso?

RESPUESTA
a.- Factor de von Willebrand.
b.- Glucoproteina IIb/IIIa.
c.- Produccion de tromboxano A2.
d.- Leucotrienos.

PREGUNTA
Cul de las siguientes manifestaciones bioqumicas no espera encontrar en el paciente en cuestin?
MANUAL DE TRABAJO DEL CURSO ENARM CMN SIGLO XXI
CURSO ENARM CMN SIGLO XXI TEL: 36246001 Pharmed Solutions Institute PGINA 218


RESPUESTA
a.- Creatinincinasa se eleva entre las 4 y 8 hrs despus del comienzo de la sintomatologia.
b.- La fraccin MB de la CPK a partir de las 3 del comienzo del infarto.
c.- La troponina I se eleva a partir de que se inicia la sintomatologia.
d.- La mioglobina es la ultima en enzima que se eleva.

CASO CLINICO
Varn de 66 aos ingresado por IM lateral evolucionado, no tratado con fibrinolticos por criterios de tiempo. Tras 72 h sin
complicaciones, bajo tratamiento con nitratos. En el sptimo da de ingreso present TV polimorfa sincopal que precis cardioversin
elctrica urgente. En las horas siguientes se objetivaron numerosos episodios de taquicardia sostenida y no sostenida, con un
comportamiento incesante y degeneracin ocasional en fibrilacin ventricular (FV), precisando mltiples choques externos. Estos
episodios no eran precedidos de bradicardia significativa, prolongacin anormal del intervalo QT, cambios en el segmento ST o dolor
torcico. Los valores de las enzimas cardacas dentro de rango normal.

PREGUNTA
Cual es el manejo farmacolgico mas adecuado.

RESPUESTA
a.- Lidocaina.
b.- Amiodarona.
c.- Verapamilo.
d.- Propanolol.

SICA II (IAM SESST)
INTRODUCCION: Se trata de proceso coronario isqumico inestable, SICA II o Infarto miocrdico agudo sin elevacin del segmento ST,
divido en riesgo alto, medio y bajo, lo que permite considerar tratamiento de reperfusion inmediato o no, condicin dinmica que
puede evolucionar de angina inestable a infarto al miocardio, teniendo como punto cardinal la elevacin de enzimas cardiacas.
DIAGNOSTICO: Cuadro clnico caracterizado por dolor retrosternal de tipo opresivo, definido como opresivo, acompaado o no con
sensacin de pensantez torcica, irradiacin a brazo, cuello, hombro, puede presentar dolor en espalda, sntomas generales, debilidad,
mareo y nausea, acompaado por falta de aire. ECG especialmente se considera la falta de elevacin del segmento ST, pero podemos
observar alteraciones con depresin o aplanamiento de onda T, o del segmento completo. Finalmente para establecer el diagnostico
debern buscar enzimas cardiacas, si hay elevacin el diagnostico ser SICA SESST. Carateristicas del riesgo (Alto) multiples factores de
riesgo para enfermedad coronaria, angina en reposo mas de 20 minutos, o posinfarto, elevacin del ST, depresin de ST > 1 mm en 2
derivaciones continuguas, Inestabilidad hemodinmica, signos de insuficiencia cardiaca o disfuncin ventricular izquierda,
biomarcadores sericos elevados. Clasificacin riesgo (Intermedio) DM antecedentes de IAM, angina prolongada resuelta, Inversion de T
en 5 derivaciones, biomarcadores normales con marcadores inflamatorios elevados. Riesgo (Bajo) Angina inestable o cambiante,
Biomarcadores o marcadores inflamatorios normales, ECG normal o sin cambios. PATOGENIA: El SICA II es casusado por ruptura o
erosion de una placa ateromatosa con formacin de trombo plaquetario que genera consecuentemente obstruccion
predominantemente parcial del flujo sanguneo coronario. Es un fenmeno dinamico que trata de mantener un balance entre la
demanda del miocardio y la demanda que se observa durante la angina inestable y infarto miocrdico sin elevacin del segmento ST,
las cuales incluyen, 1.- reduccin de la luz arterial coronaria debido a un trombo no oclusivo que se desarrolla seguido a ruptura o
erosion de una placa aterotrombotica. 2.- severo estrechamiento de arteria coronario sin espasmo o trombosis por arteriosclerosis
progresiva o con reestenosis con mas de 6 meses posterior a intervencin coronaria percutnea. 3.- Espasmo focal intenso de segento
epicardico coronario arterial (prinzmetal o angina variante) causando obstruccion dinmica del lumen arterial. 4.- Diseccion coronaria
arterial (causado posparto) 5.- En presencia de factores precipitantes extrnsecos a las arterias coronarias que limitan la perfusin
miocrdica, causando una alteracin sbita de la demanda miocrdica de oxigeno como (sepsis, fiebre, taquicardia) o reduccin de flujo
coronario (hipotensin) o disminucin de la entrega de oxigeno al miocardio (hipoxia, anemia severa, etc). TRATAMIENTO: Reposo y
comodidad absoluta, ECG de 12 derivaciones identificando el patron ST posteriormente establecer estratificacin del riesgo lo cual
permite decidir el tratamiento, se inicia co
inmediata: angioplastia primaria o trombolisis; AI/SICA SESST (riesgo alto e intermedio) con depresin del ST o cambios dinamicos de T
(inversin) o elevacin enzimtica
angioplastica coronaria percutnea; IA/SICA SESST (riesgo bajo) con ECG no diagnostico o normal, enzimas normales, se indicara
tratamiento medico, monitorizacin, estudios de isquemia para estratificacin. Analgesicos: morfina indicada si los nitritos no reducen
el dolor, en paciente ansioso o agitado, en edema agudo de pulmon, reduce la precarga y la poscarga, el consumo de oxigeno
miocrdico, sedante, venodilatador. O2 a 4 Ltx vigilando saturacin (90%), Nitroglicerina via sublingual hasta 3 dosis intervalo de 5
minutos, monitorizar FC y TA, con persistencia de dolor inicio de via IV, su efecto es venodilatador, cuidado en infarto inferior o
derecho, en hipotesion, bradicardia o taquicardia, evitar si hay uso previo de inhibidores de fosfodiesterasa. Aspirina de 160 a 325 mg
de absorcin rpida; Clopidrogrel inhibidor irreversible del receptor del adenosindifosfato 300 mg dosis nica. Anticoagulantes,
heparina no fraccionada no propaga el trombo sin utilidad ltica a 60 a 70 U por Kg mantenerTPT menor a 75 segundos. Heparina de
bajo peso molecular, enoxoparina mayor seguridad de sangrado. Fibrinoliticos, deben evitarse por riesgo beneficio ya que en SICA II
SESST la arteria se encuentra abierta. PRONOSTICO: SICA II SESST presenta morbilidad y mortalidad expectante por las condiciones de
la enfermedad coronaria de base, la funcin sistlica del ventrculo izquierdo, la estabilidad de la lesin en si misma, la mortalidad por
angina inestable es de 3.6 % a 6 meses y 6.2 % en caso de infarto al miocardio sin elevacin del segmento ST.


MANUAL DE TRABAJO DEL CURSO ENARM CMN SIGLO XXI
CURSO ENARM CMN SIGLO XXI TEL: 36246001 Pharmed Solutions Institute PGINA 219

CASO CLINICO
Hombre de 48 aos, con antecedente familiar de cardiopata coronaria, tabquico y dislipidmico sin tratamiento farmacolgico.
Present intenso dolor precordial mientras jugaba ftbol, consultando en un servicio de urgencia donde present paro
cardiorrespiratorio por fibrilacin ventricular. Fue tratado con mltiples desfribilaciones y maniobras de reanimacin durante 25 min. El
ECG no mostro datos sugerentes de infarto. Al ingreso se encontraba intubado con asistencia ventilatoria mediante amb,
hemodinmicamente inestable requiriendo infusin de noradrenalina a 0,1 g/kg/min. 2 horas despus se observo cambios en el ECG
mostr onda Q y supradesnivel del segmento ST hasta 3 mm en VI a V3. Troponina I: 292 ng/mL (normal <0,05).

PREGUNTA
Cual de los criterios es ms determinante para considerar infarto al miocardio sin elevacin del segmento ST comparado con angina
inestable?

RESPUESTA
a.- Cambios bioqumicos.
b.- Presentacin sintomtica.
c.- Cambio electrocardiogrficos sin elevacin de ST.
d.- Factores de riesgo previos.

PREGUNTA
Considerando el tipo de manifestaciones clnicas, electrocardiogrficos y biomarcadores cual de las siguientes conductas teraputicas
no se recomienda en el IAM SESST.

RESPUESTA
a.- Reperfusion inmediata.
b.- Inhibidores de receptores de glucoproteinas.
c.- Heparina de bajo peso molecular.
d.- Angioplastia coronaria percutnea.

PREGUNTA
Cul es la complicacin ms probable del paciente.

RESPUESTA
a.- Choque cardiogenico.
b.- Arritmia mortal.
c.- Ruptura de musculos papilares.
d.- Falla organica multiple

CASO CLINICO
Hombre de 72 aos, con historia de hipertensin de 22 aos de evolucin tratada con enalapril 40 mg, diabetes mellitus 2 desde hace
20 aos tratada con glibenclamida 50 mg, hipercolesterolemia manejada con atorvastatina 20 mg e hipertrigliceridemia tratada con 200
mg, actualmente se encuentra en cardiologa por dolor torcico recurrente, fue admitido con quejas de "opresin en el pecho", que se
irradio a mandibula, hombro y brazo izquierdo de 105 minutos de duracin que sedio reposo absoluto y nitroglicerina sublingual, el
comienzo fue sbito y acompaado de disnea, nauseas. Se encontraron los siguientes resultados patolgicos 140/95mmHg, FC 107 lpm,
FR 29 rpm, temperatura 37.8 grados, glicemia 235 mg/dl, Colesterol 429 mg/dl, Trigliceridos 650 mg/dl. Electrolitos sericos, K 5.5, El
ECG realizado en la admisin mostr fibrilacin atrial con frecuencia ventricular rpida y Segmento ST y Onda T normales. El examen
fsico no mostr alteraciones significativas, pero hubo elevacin de biomarcadores cardacos, con pico de troponina I de 1,84 ng/ml y
CKMB-masa 13,4 ng/ml. El diagnstico inicial no revel infarto agudo de miocardio con elevacin del Segmento ST, y la angiografa
coronaria se muestra. El ecocardiograma revel una FE >40.

PREGUNTA
Considerando la sintomatologa observada en el paciente que clase funcional killip y Kimball.

RESPUESTA
a.- Clase funcional Killip I.
b.- Clase funcional Killip II.
c.- Clase funcional Killip III.
d.- Clase funcional Killip IV.

PREGUNTA
La presencia de DM acelera el proceso de aterognesis a travs de diversos mecanismos, cual de los siguientes mecanismos es mayor
en el paciente actual?

RESPUESTA
a.- Anomalas en concentraciones y la composicin de las lipoprotenas.
b.- Asociacin con la hipertensin.
c.- Oxidacin lipdica.
MANUAL DE TRABAJO DEL CURSO ENARM CMN SIGLO XXI
CURSO ENARM CMN SIGLO XXI TEL: 36246001 Pharmed Solutions Institute PGINA 220

d.- Estado procoagulante y proinflamatorio.

PREGUNTA
Cul es los paraclinicos en el primer nivel de atencin es de mayor utilidad para confirmar el diagnstico de la insuficiencia cardaca en
el paciente?

RESPUESTA
a.- ECG.
b.- Banda sin fin.
c.- Ecocardiograma.
d.- Arteriografia.

PREGUNTA
Cual de los siguientes factores comorbidos presenta la mayor importancia para desencadenar un evento coronario en el paciente.

RESPUESTA
a.- Diabetes mellitus.
b.- Hipertensin arterial.
c.- Dislipidemia.
d.- Tabaquismo.

PREGUNTA
Cual de los siguientes datos patolgicos en el paciente presenta la mayor importancia para desencadenar un evento coronario en el
paciente.

RESPUESTA
a.- Niveles de glucosa.
b.- Hemoglobina glucosilada.
c.- Trigliceridos
d.- Colesterol.

PREGUNTA
Cual es la conducta a seguir con los siguientes criterios es el menos importante en este paciente para envio inmediato a tercer nivel?

RESPUESTA
a.- Dolor toracicco sugerente de angor pectoris.
b.- Dislipidemia.
c.- Hipertensin arterial.
d.- Diabetes mellitus.

PREGUNTA
Cual de los siguientes diagnosticos diferenciales es el menos probable considerando las condiciones del caso?

RESPUESTA
a.- Hipertensin arterial pulmonar.
b.- Pericarditis.
c.- Sindrome de Tietzl.
d.- Reflujo gastroesofagico.

PREGUNTA
Cual de las siguientes medidas debe iniciar antes de enviar al paciente a segundo nivel cuando existe alta sospecha de cardiopata
isqumica?

RESPUESTA
a.- Dieta y ejercicio progresivo.
b.- Acetilsalicilico y pravastatina.
c.- Diuretico y betabloqueador.
d.- Estudios bsicos y gabinete.

CASO CLINICO
Se presenta el caso de un paciente de 17 aos de edad, sin antecedentes de inters, que acude a urgencias por un dolor centrotorcico,
opresivo, no irradiado, de 48 hrs de evolucin, sin fiebre ni otra sintomatologa acompaante. El paciente refiere haber consumido
cocana y cannabis previamente al inicio de la sintomatologa. A su llegada se encuentra estable, sus constantes son correctas y la
exploracin fsica es normal. El electrocardiograma muestra elevacin del segmento (ST) en las derivaciones de la cara inferior; la
analtica confirma una elevacin de las enzimas cardacas (creatincinasa [CK] de 1.194 U/l y troponina-I de 19,6 ng/ml).

MANUAL DE TRABAJO DEL CURSO ENARM CMN SIGLO XXI
CURSO ENARM CMN SIGLO XXI TEL: 36246001 Pharmed Solutions Institute PGINA 221

PREGUNTA
Cual es teraputica mas apropiada para este caso.

RESPUESTA
a.- Nitroglicerina, morfina, trombolitico y oxigeno.
b.- Nitroglicerina, morfina, trombolitico y betabloqueador.
c.- Angioplastia percutnea, nitoglicerina, morfina, trombolitico y betabloqueador.
d.- Betabloqueador, morfina, betabloqueador.

CASO CLINICO
Se trata de femenino de 23 aos de edad la cual llega a consulta refiriendo dolor torcico intermitente, durante la atencin de
urgencias de descarto cuadro isqumico, con marcadores negativos asi como ECG reportado como normal, la prueba de esfuerzo fue
positiva reversible, se administra acido acetilsalicilico pero la paciente refiere malestar continuo por dolor torcico intermitente, sin
embargo continua con sus actividades cotidianas, no cuenta con antecedentes familiares de cardiopata isqumica, sin embargo es
fumadora social.

PREGUNTA
Cul es la conducta a seguir para establecer una aproximacin diagnostica.

RESPUESTA
a.- Realizar cateterismo cardiaco.
b.- Realizar tomografa computada cardiaca.
c.- Realizar ecocardiograma de estrs.
d.- Evaluacin del dolor de origen no cardiaco.























MANUAL DE TRABAJO DEL CURSO ENARM CMN SIGLO XXI
CURSO ENARM CMN SIGLO XXI TEL: 36246001 Pharmed Solutions Institute PGINA 222


Ritmo rpido o lento Lento
Ritmo regular o irregular Regular
QRS estrecho o ancho Estrecho
nda p Presente
Relacin p/QRS Todas las ondas p conducen
Diagnostico probable BRADICARDIA SINUSAL
Manejo Causa ms frecuente: sndrome del seno enfermo

Ritmo rpido o lento Lento
Ritmo regular o irregular Regular
QRS estrecho o ancho Estrecho
nda p Presente
Relacin p/QRS Todas las ondas p conducen, PR alargado (>0.21
mseg)
Diagnostico probable BLOQUEO AV de 1
Manejo Habitualmente no precisa tratamiento


Ritmo rpido o lento Lento
Ritmo regular o irregular Regular
QRS estrecho o ancho Estrecho
nda p Presente
Relacin p/QRS Alargamiento progresivo del PR hasta una onda p no conduce
Diagnostico probable BLOQUEO AV 2 WENKEBACK (MOBITZ I)
Manejo Habitualmente no precisa tratamiento

Ritmo rpido o lento Lento
Ritmo regular o irregular Regular
QRS estrecho o ancho Estrecho
nda p Presente
Relacin p/QRS PR, constante, hasta que una onda p no
conduce
Diagnostico probable BLOQUEO AV 2 MOBITZ II
Manejo Derivar a servicio de urgencias, precisara marcapasos

Ritmo rpido o lento Lento
Ritmo regular o irregular Regular
QRS estrecho o ancho Estrecho
nda p Hay ondas p con ritmo regular
Relacin p/QRS Ninguna onda p conduce, no hay relacin p/QRS
Diagnostico probable BLOQUEO AV 3 COMPLETO
Manejo Derivar a servicio de urgencias, precisara marcapasos

ARRITMIAS CARDIACAS:
CIENCIAS BASICAS: La arritmia cardiaca se define como la alteracin del ritmo cardiaco establecido como normal entre 60-100 lpm.
Menor a 60 lpm, ser bradicardia y si es mayor a 100 lpm, ser taquicardia. La arritmia ms frecuente es la fibrilacin auricular. SALUD
PUBLICA: La prevalencia de FA oscila 0,7-17,6%, La prevalencia de las otras 2 taquiarritmias supraventriculares ms importantes, el
flutter auricular y la taquicardia paroxstica supraventricular (TPSV), es mucho ms baja. La incidencia del flutter auricular es de 88 por
1,000,000 de personas/ao. ACTUACION INICIAL EN SERVICIO DE URGENCIAS: Hay que sistematizar nuestra actuacin para no pasar
por alto aspectos esenciales. 1. Realizar ECG; verlo con calma y valorar los siguientes aspectos: a) si el ritmo es rpido o lento, b) si el
ritmo es regular o irregular, c) si el QRS es estrecho o ancho, d) le presencia o no de ondas p, e) la relacin de p/QRS, esto nos
orientar la arritmia a la entidad etiolgica ms probable. 2. Monitorizar las constantes vitales: como la tensin arterial, saturacin de
oxigeno, frecuencia cardiaca y respiratoria, vigilar la perfusin tisular, para detectar precozmente los signos de bajo gasto cardiaco.
Todas las alteraciones teraputicas deben documentarse con tiras de registro de ECG. 3. Canalizar una va venosa perifrica y
administrar oxigeno: es necesario ya que el shock puede no detectarse precozmente y la mala perfusin perifrica dificulta obtener una
via. El oxgeno lo administraremos en funcin de la oxigenacin, para evitar hiperoxigenacin. 4. Valorar estabilidad hemodinmica-los
signos adversos y que indican mala evolucin son; a) shock-palidez, sudoracin, por aumento de actividad simptica, bajo nivel de
conciencia, por disminucin de FSC o hipotensin arterial. b) sincope-por disminucin de FSC. c) insuficiencia cardiaca-edema pulmonar
o fallo del VI o ingurgitacin yugular, hepatomegalia por fallo de VD. d) isquemia miocrdica-por aumento de la demanda de oxigeno
del miocardio. Si el paciente tolera bien la arritmia probablemente no haga falta hacerle nada o bien solo frmacos. Si la tolera mal, al
final precisara una cardioversin elctrica. Dos aspectos importantes en atencin primaria: primero no debemos ser ms agresivos con
nuestra actuacin que la propia arritmia lo es con el paciente y en segundo lugar, los frmacos antiarrtmicos combinados o a dosis
altas pueden ser arritmognicos: para evitar efectos secundarios es recomendable seguir la mxima de un paciente, un solo
antiarrtmico. Adenosina y ATP son los nicos que escapan a esta mxima. DIAGNOSTICO DIFERENCIAL DE BRADICARDIAS:










MANEJO DE LAS BRADICARDIA: Tratar las bradicardias, mal toleradas o que cumplan criterios de inestabilidad que se numeran a
continuacin: 1. Tensin arterial sistlica <90mmHg. 2. Frecuencia cardiaca <40 lpm. 3. Bradicardia sintomtica. 4. Bradicardias con QRS
ancho. Los medicamentos de que se dispone para su manejo son; ATROPINA: 0,5mg IV (dosis inferiores producen un efecto paradjico).
Se puede repetir cada 2 minutos, dosis mxima 3 mgs. Usar con precaucion en el sndrome coronario agudo, ya que puede aumentar el
grado de isquemia cardiaca; en transplantados cardiacos causa bloqueo AV de alto grado o paro sinusal. Si tras la medida anterior el
paciente no mejora o hay riesgo de asistolia, precisara un marcapasos transcutneo, por lo que hay que avisar a servicios de
emergencia. Las situaciones que aumentan el riesgo de que la bradicardia derive en asistolia son: asistolia reciente, paro ventricular
>3seg, BAV Mobitz II, BAV 3 con QRS ancho. Si no disponemos de servicio de emergencias a corto plazo, el paciente se inestabiliza,
podemos administrar una perfusin de ADRENALINA a 1mg en 100cc de SF en 10 min. Si no mejora podemos golpear rtmicamente (60
veces por minuto), con nuestro puo en el margen inferior izquierdo del esternn, a modo de marcapasos externo. Si la causa de la
bradicardia es una intoxicacin con betabloqueadores o antagonistas de calcio, administraremos GLUCAGON 1mg.





MANUAL DE TRABAJO DEL CURSO ENARM CMN SIGLO XXI
CURSO ENARM CMN SIGLO XXI TEL: 36246001 Pharmed Solutions Institute PGINA 223

DIAGNOSTICO DIFERENCIAL DE TAQUICARDIAS:







MANUAL DE TRABAJO DEL CURSO ENARM CMN SIGLO XXI
CURSO ENARM CMN SIGLO XXI TEL: 36246001 Pharmed Solutions Institute PGINA 224





La cardioversion electrica sincronizada, para revertir una arritmia debe ser sincronizada sobre la onda R para evitar el periodo
refractario que sigue a la onda T, ya que podriamos provocar una taquicardia ventricular, de efectos deletereos. Previamente debemos
sedar al paciente con 10-15 mgs de midazolam intravenoso. La energia necesaria para la cardioversion de una arritmia de QRS ancho o
una FA (son las mas frecuentes) es de 70-120J si usamos un desfibrilador bifasico o 100J si es monofsico. En cambio para cardiovertir
arritmias de QRS estrecho o un flutter auricular la energia a palicar es de 120-150J en desfibriladores bifasicos y 200J en los
monfasicos. Si la primera descraga no es eficaz, se puede incrementar la energia (50J) en las siguientes y si hay dificultad para
sincronizar el choque, podemos proceder a choque no sincronizado.

CASO CLINICO
Se trata de paciente masculino de 31 aos de edad, licenciado en derecho, originario del Estado de Tabasco, soltero pero con pareja
sexual, de orientacin homosexual, con prcticas de riesgo de los 19 a los 27 aos, actualmente es negativo a VIH y mantiene una
proteccin adecuada, consume bebidas embriagantes casi cada semana y tambin presenta tabaquismo positivo, no ms de 5
cigarrillos diarios, cuenta con antecedentes de importancia dengue a los 17 aos y apendicetoma a los 19, se le est realizando
evaluacin para ingresar a laborar en el sector gubernamental, el paciente refiere excelente estado de salud sin embargo reconoce no
estar en forma, no practica actividades fsicas y su trabajo es muy sedentario, refiere uso de drogas con uso recreativo y experimental
de los 21 a los 26 aos, solo agrega sentir brincos en el corazn en ltimas fechas, tal vez mareo ocasional que no le ha resultado de
importancia, a la exploracin fsica se encuentra un IMC de 24, aparenta buen estado general, resto normal, los parmetros de
laboratorio normales pero el ECG del chequeo muestra un trazo donde se prolonga progresivamente el intervalo PR de 0.20 a 0.28.

PREGUNTA
Cul de los siguientes diagnsticos es el ms probable en este hallazgo.

RESPUESTA
a.- Bloqueo AV de 1er grado.
b.- Bloqueo AV de 2do grado Mobitz I.
c.- Bloqueo AV de 2do grado Mobitz II.
d.- Bloqueo AV de 3er grado.

CASO CLINICO
Se trata de paciente femenino de 72 aos de edad, originaria de Guanajuato, viuda, dedicada a su casa, actualmente bajo tratamiento
por insuficiencia cardiaca por hipertrofia ventricular secundaria a hipertensin arterial de 30 aos de evolucin, cuenta adems con
aterosclerosis e insuficiencia intestinal crnica, refiere familiar que encontr a la paciente confundida, desorientada y posteriomente
pierde la conciencia trasladndole a urgencias donde se ingresa a choque, el trazo electrocardiogrfico se observa ondas P que no
conducen y latidos de escape ventricular de forma variable (disociacin auriculoventricular). Se reanima a la paciente y se prepara para
colocacin de marcapasos, previamente se estableci el diagnostico de un Bloqueo AV de 3er grado.

PREGUNTA
Cul de las siguientes condiciones es la causa ms probable del bloqueo de este paciente.

RESPUESTA
a.- Uso de digitalico.
b.- Trastorno electroltico.
c.- Uso de diurtico.
d.- Uso de antihipertensivo.

CASO CLINICO
Se encuentra de guardia en el servicio de urgencias donde llega por la tarde paciente femenino de 49 aos de edad la cual refiere que
desde la noche anterior inicio con molestia en el torax con disconfort, ansiedad y sensacin de que su corazn latia muy rpido, aun asi
acudi al trabajo, pero se sali porque le faltaba el aire y presencia mareo continuo, teniendo que descansar repetidamente, la
paciente es madre soltera, con tres hijos, emigra al Estado de Mxico hace 15 aos por disfuncin familiar severa con violencia fsica,
tiene antecedente de histerectoma total abdominal con ooforectomia bilateral por presencia de cncer cervicouterino, adems de
recibir quimioterapia y radioterapia hace 5 aos con excelente respuesta y controles adecuados, adems cuenta con antecedente de
procesos infecciosos recurrentes de vas areas altas y recuerda escasamente que le diagnosticaron fiebre reumtica a la edad de 23
aos, pero ya no continuo su atencin por sentirse mejor, a la exploracin fsica se encuentra palidez de tegumentos, cianosis
peribucal, no hay datos de ingurgitacin yugular, se ve confusa, al examen mental esta desorientada, agitada, incongruente, sntomas
se duran solo minutos sus ruidos cardiacos estn disminuidos, constantes vitales son TA 150/60 mmHg, FC 145 lpm, FR 31 rpm,
MANUAL DE TRABAJO DEL CURSO ENARM CMN SIGLO XXI
CURSO ENARM CMN SIGLO XXI TEL: 36246001 Pharmed Solutions Institute PGINA 225

normotermica, la paciente refiere que fue diagnosticada recientemente con hipertensin arterial y prescribieron metoprolol 100 mg
cada 12 horas, el ECG presenta ausencia de onda P, con presencia de ondulaciones irregulares y de pequea amplitud, pero complejos
QRS de 80 a 100 por minuto.

PREGUNTA
Cul es el manejo ms apropiado para evitar complicaciones agudas para el caso.

RESPUESTA
a.- Propafenona.
b.- Amiodarona.
c.- Digoxina.
d.- Verapamil.

CASO CLINICO
Luego de 6 horas de ingreso la paciente presenta datos de focalizacin neurolgica, caracterizado por desviacin de la comisura labial,
disartria y dislalia, parestesias de miembro torcico contralateral a la cara, se observa disminucin de fuerza en el mismo sitio, a las 24
horas posteriores se observa recuperacin paulatina de sensibilidad y fuerza de miembro torcico y discreta mejora en la cara, al
parecer se trato nicamente de una isquemia cerebral transitoria, actualmente se encuentra con antiarritmico, anticoagulante y
neuroprotector, continua en terapia.

PREGUNTA
Cul es la causa ms probable que condujo a esta arritmia a la paciente.

RESPUESTA
a.- Uso reciente de Betabloqueador.
b.- Valvulopatia crnica.
c.- Sindrome paraneoplasico.
d.- Cardiomiopatia metastasica.

CASO CLINICO
Se encontr en la via publica a sujeto de 50 aos aproximadamente el cual se encuentra en condicin de calle, se observa desalineado,
con nula higiene, con aliento alcohlico, se observan lesiones drmica sin tratamiento, el paciente se encuentra desorientado,
confundido, con tendencia a perder el equilibrio con palidez generalizada con prdida de conciencia por segundos, se toman muestras
para laboratorio, se coloca ECG y se observa un trazo caracterizado por ausencia de ondas p con ondas regulares dentadas en
derivaciones DII, DIII y aVF particularmente negativas.

PREGUNTA
Cul es la conducta a seguir ms adecuada.

RESPUESTA
a.- Cardioversion.
b.- Colocacion de marcapasos.
c.- Administracion de Amiodarona.
d.- Administracion de Digoxina.

CASO CLINICO
Varn de 67 aos. Como antecedentes personales de inters destacan hipertensin arterial, diabetes mellitus tipo 2, dislipemia,
bronquitis crnica, clicos nefrticos de repeticin, ciruga de hemorroides y de hernia umbilical. Tratamiento habitual con insulina
aspart, metformina, omeprazol, tiaprida y lorazepam. Ingresa en nuestro centro hospitalario para estudio por posible neoplasia de
cabeza de pncreas. Encontrndose hemodinmicamente estable, se inicia pauta con octreotide 100g/8h subcutnea. A las 24 horas
presenta bradicardia sinusal a 40 lpm con presin arterial (PA) de 120/60 mmHg, sin sintomatologa asociada. En el ECG se observa
bradicardia sinusal y hemibloqueo de rama anterior izquierda, realizan ecocardiograma transtorcico sin alteraciones significativas y
colocan Holter de 24 horas. Presenta deterioro clnico con bradipsiquia e inestabilidad de la marcha; PA, 90/45 mmHg y 35 lpm.
Valorada tira de ritmo en planta presenta bradicardia sinusal a 37 lpm y hemibloqueo de rama anterior izquierda descrito. Tras
administracin de atropina hasta 3mg persiste bradicardia sintomtica.

PREGUNTA
Cul es la conducta a seguir ms adecuada.

RESPUESTA
a.- Cardioversion.
b.- Colocacion de marcapasos.
c.- Administracion de Amiodarona.
d.- Administracion de adrenalina.


MANUAL DE TRABAJO DEL CURSO ENARM CMN SIGLO XXI
CURSO ENARM CMN SIGLO XXI TEL: 36246001 Pharmed Solutions Institute PGINA 226

DISFUNCION ORGNICA MLTIPLE (DOM):
CIENCIAS BSICAS: Disfuncin de rganos nace como resultado de la Conferencia de Consenso de la ACCP/SCCM: Es la disminucin
potencialmente reversible en la funcin de uno o ms rganos, que son incapaces de mantener la homeostasis sin un sostn
teraputico. El trmino disfuncin implica un proceso continuo y dinmico en la prdida de la funcin de un rgano, que va de menos a
ms, siendo la etapa final en la claudicacin de la funcin de dicho rgano lo que denominamos falla. El Sndrome de Disfuncin
Orgnica Mltiple (SDOM) se considera como un conjunto de sntomas y signos de patrn diverso que se relacionan en su patogenia,
estn presentes por lo menos durante 24 a 48 horas y son causados por disfuncin orgnica, en grado variable, de dos o ms sistemas
fisiolgicos, con alteracin en la homeostasis del organismo, y cuya recuperacin requiere mltiples medidas de soporte avanzado. Esta
definicin adems de permitir un diagnstico clnico precoz, nos da la oportunidad de intervenir activa y rpidamente para tratar de
revertir el proceso que origin el dao Asimismo debemos definir el SDOM primario cuando ocurre una vez que cualquier sistema
orgnico falla, en forma evidente o sea como resultado directo de una lesin conocida, por ejemplo, luego de un episodio de
insuficiencia respiratoria aguda por lesin pulmonar despus de un trauma tisular global o contusin pulmonar (En la forma ms comn
de SDOM, los pulmones son los rganos predominantemente afectados y con frecuencia el nico sistema implicado, hasta etapas
tardas de la enfermedad), la insuficiencia renal aguda por rabdomilisis y a la coagulopata por politransfusin. El SDOM secundario es
el que se presenta en el contexto de una respuesta inflamatoria sistmica, o consecuencia de la respuesta del husped a una agresin
englobndose en el Sndrome de Respuesta Inflamatoria Sistmica (SRIS), es el nexo que media entre la injuria (infecciosa o no) y el
SDOM. El SRIS y el SDMO son la va final comn de muerte en pacientes severamente enfermos o lesionados, en los que la tecnologa
avanzada existente en las UCI ha posibilitado su resucitacin. SALUD PUBLICA: Las cifras de mortalidad para SIRA por s solo es de 40 al
50%, una vez que se agrega disfuncin orgnica sistmica, la mortalidad se incrementa hasta el 90%. Sus complicaciones son la causa
ms frecuente de mortalidad en UCI no coronarias ETIOLOGA: Inicialmente se pens que ste cuadro era la expresin fatal de una
sepsis incontrolada, sin embargo posteriormente se ha demostrado que puede ocurrir en ausencia de infeccin. La infeccin no es una
condicin indispensable para el desarrollo de FOM. Plantea la existencia de una activacin masiva de mediadores de la inflamacin
como consecuencia del dao tisular. La incidencia del SDOM en una poblacin de pacientes hetereognea mixta (patologas de tipo
mdico y quirrgico) vara entre 7 y 15%.En pacientes con trauma su incidencia puede ser hasta del 35%, y mucho menor en pacientes
luego de ciruga cardiaca electiva, alrededor del 3%. Es difcil encontrar un elemento causal simple para el SDMO, existiendo en la
mayora de los pacientes varias causas que se potencian: 1. Traumatismos multisistmicos graves. 2. Postoperatorio. 3. Inestabilidad
hemodinmica. 4. Infecciones severas. 5. Pancreatitis aguda. 6. Quemados. 7. Necesidad de ventilacin mecnica prolongada. 8.
Hemorragia gastrointestinal. 9. Diseccin, ruptura o reparacin de aneurisma artico. 10. Perforacin gastrointestinal. 11. Enfermedad
inflamatoria intestinal. 12. Nutricin parenteral prolongada. 13. Ciruga valvular cardiaca. 14. Transfusiones masivas. 15. Coagulacin
intravascular diseminada. PATOGENIA: Se ha acumulado suficiente evidencia de la existencia de una participacin activa de la cascada
inflamatoria en la gnesis de cuadros de DOM. Es as como noxas de origen infeccioso no infeccioso son capaces de iniciar el proceso
inflamatorio, activando vas comunes que a manera de respuesta inespecfica, permiten al husped reaccionar frente a la injuria. Esta
respuesta inflamatoria se verifica bsicamente a cuatro grandes niveles: 1. Activacin del sistema Monocito Macrfago. 2. Activacin
del sistema del Complemento. 3. Activacin del sistema Retculo Endotelial. 4. Activacin de la Cascada de la Coagulacin. Sin embargo,
todas estas vas estn relacionadas entre s, pudiendo activarse y potenciarse unos a otros desde cualquier punto de esta red
inflamatoria. Es el caso de pacientes politraumatizados en donde la cascada se inicia por la activacin del sistema del Complemento y
cascada de la Coagulacin, o en las vasculitis en donde el dao inflamatorio endotelial sera el gatillador del proceso. Existe suficiente
evidencia que avala la activa participacin de la respuesta inflamatoria en la gnesis de la DOM, sin embargo todos los intentos por
bloquear o modular esta respuesta ha fracasado. Luego del fenmeno inicial de up regulation de citoquinas proinflamatorias (TNFa, IL-
1, IL-2, IL-6, IL-8) o concomitantemente con l, se liberan especies anti-inflamatorias (receptores solubles de TNFa, IL-1 ra e IL-10) en
concentraciones 30 a 100.000 veces superiores que la de las molculas proinflamatorias. Esto podra ser responsable de la disminucin
de la activacin observada en Monocitos humanos ante la exposicin a infecciones de severidad creciente. Es as como el paradigma
actual de la respuesta inflamatoria presente en cuadros de SIRS,
sepsis o MOD es la interaccin sistmica entre mediadores pro y anti
inflamatorios que determinarn en definitiva el dao celular la
recuperacin. En el dao celular, ltimo elemento en el que radica la
DOM, una de las teoras que mayor peso tiene en la actualidad
indicara que, como consecuencia de la activacin de la cascada
inflamatoria, a nivel intracelular la activacin de mediadores
determina un stress oxidativo. Es este stress oxidativo el que es
capaz de activar un complejo de mediadores intracelulares
denominado "Complejo Nuclear Factor kb", el cual est compuesto
por tres sub unidades; una molcula pequea denominada p50, una
mayor, la p65 y una sub unidad inhibitoria. Como consecuencia del
stress oxidativo este complejo es fosforilado, liberando la sub
unidad inhibitoria y permitiendo al dmero p50-p65 libre, migrar al
ncleo y unirse al DNA en los sitios que promueven la codificacin
de citoquinas y otras sustancias proinflamatorias. Diversos trabajos
han demostrado que la administracin de antioxidantes y que la prevencin del stress oxidativo mitocondrial suprimen la activacin de
Nuclear Factor kb. DIAGNOSTICO: Clnico: Los registros de temperatura deben ser centrales o rectales. La fiebre se produce por el
efecto de las citoquinas IL-1 y TNF liberados por los macrfagos activados. Aunque la fiebre y los escalofros son tpicos, algunos
pacientes que desarrollan infecciones bacterianas sistmicas estn debilitados y no exhiben cambios llamativos (por ejemplo,
escalofros) al comienzo de una infeccin. La causa de la hipotermia es menos conocida y su presencia se asocia con mal pronstico. En
los pacientes crticos, los mismos sntomas y signos caractersticos de sepsis pueden aparecer durante la inflamacin sistmica de
origen no infeccioso, por lo que el diagnsco y la denicin de la severidad del proceso spco pueden ser diciles. Durante los
lmos aos se ha buscado un marcador clnico o de laboratorio capaz de idencar a los pacientes con sepsis, diferencindolos de los
MANUAL DE TRABAJO DEL CURSO ENARM CMN SIGLO XXI
CURSO ENARM CMN SIGLO XXI TEL: 36246001 Pharmed Solutions Institute PGINA 227

portadores de otras patologas que tambin cursan con SIRS. Entre ellos podemos mencionar: Procalcitonina (PCT); posible marcador
de SIRS en respuesta a infeccin. Niveles mayores de 10 ng/mL (sepsis) y a veces superiores a 100 ng/ mL(shock sptico). Protena C
reactiva (PCR); Es un marcador sensible pero tardo y de baja especificidad. Recuento leucocitario; La leucocitosis se interpreta
habitualmente como evidencia de posible infeccin, pero no es un marcador sensible ni especfico. La activacin de la coagulacin es
un hecho comn en el curso de la sepsis, con consumo de factores, aumento del dmero D y sobre todo disminucin de la actividad de
los anticoagulantes naturales. Varias citoquinas proinflamatorias, sobre todo los niveles plasmticos de la IL-6 e IL-8, han mostrado
correlacin con el pronstico en diversos estudios efectuados en pacientes crticos. Pero su determinacin tiene varios inconvenientes:
alto costo, la vida media de las citoquinas es muy corta y las concentraciones varan rpidamente por lo que es difcil su interpretacin.
No se sugiere su utilizacin en la prctica clnica. John Marshall crea un Score de Disfuncin Orgnica, en el cual considera por medio de
un mecanismo de puntuacin, la magnitud de la disfuncin de cada uno de los 6 sistemas orgnicos analizados, otorgndoles un valor
entre 0 y 4 segn la magnitud de la disfuncin. Este sistema de puntuacin se correlacion estrechamente con la mortalidad intra UCI
tanto cuando se utilizaban los valores obtenidos durante las primeras 24 horas desde la admisin como cuando se aplicaba durante la
estada. Jean Louis Vincent y cols introducen la versin Europea de los Scores de Disfuncin (SOFA). Los scores nos permiten hacer un
pronstico objetivo sobre la evolucin del paciente. En la prctica clnica, cuando tenemos ms de tres rganos en falla por ms de 48
horas, la mortalidad se eleva por sobre el 90%, debiendo plantearse seriamente y con la familia la posibilidad de limitar el tratamiento
en curso. En estos casos, tendrn ms posibilidades de sobrevida aquellos pacientes ms jvenes y sin patologa debilitante previa.
TRATAMIENTO: Hasta la fecha la terapia de la Disfuncin Orgnica contina siendo una terapia de soporte, una vez eliminada la injuria
inicial (drenaje de colecciones, terapia antibitica, remocin de tejidos necrticos, restablecimiento de una perfusin adecuada, etc.).
Existen algunos estudios recientes que mostraran el beneficio de terapias de hemofiltracin de alto flujo, como una forma de modular
la respuesta inflamatoria, removiendo aquellas sustancias que se encuentran en exceso. Todos los pacientes con choque sptico deben
tener acceso venoso adecuado para la reanimacin con volumen. Un catter venoso central tambin puede ser usado para monitorear
la presin venosa central y evaluar el estado del volumen intravascular. Una sonda vesical permanente debe ser usada para monitorear
la produccin de orina, marcador de una adecuada perfusin renal y el gasto cardaco. Los pacientes que desarrollen choque sptico
requieren cateterismo cardaco derecho en la arteria pulmonar a travs del catter de Swan-Ganz. Este catter proporciona una
evaluacin precisa del estado de volumen de un paciente que est sptico. Mediante la evaluacin de la funcin ventricular tanto
derecha como izquierda, monitoreo de los cambios hemodinmicos, adems de brindar informacin pronstica. La mayora de los
pacientes que se encuentren spticos desarrollan dificultad respiratoria como una manifestacin del estado de choque. La disfuncin
pulmonar de la sepsis (SDRA) tambin puede ocurrir. Estos pacientes necesitan intubacin y ventilacin mecnica para la asistencia
respiratoria ptima.

CASOS CLINICOS
Se trata de paciente femenino de 24 aos de edad tercigesta con cesrea y aborto previos, sin antecedentes de importancia, quien
ingresa al servicio de urgencias con embarazo de 36 semanas de gestacin + Preeclampsia severa con cifras tensinales de hasta
190/120mmHg. Presenta adems actividad uterina irregular sin datos de vasoespasmo ni datos de eminencia de eclampsia ni
afectacin heptica. Frecuencia cardiaca fetal 135 latidos por minuto. Se hospitaliza y maneja con hidratacin, antihipertensivo
perifrico, calcioantagonista y sulfato de magnesio. Se solicitan laboratoriales. Hemoglobina: 14.2, Hematocrito: 46.2, Plaquetas:
215,000, TP: 12.6, TPT: 36.1, Glucosa: 88, Creatinina: 0.8, Acido rico: 5.2, Bilirrubina Total: 0.75, Bilirrubina Directa: 0.16, Bilirrubina
Indirecta: 0.59, Alanina Amino Transferasa (TGP): 35, Aspartato Amino Transferasa (TGO): 20, Deshidrogenasa Lctica (DHL): 350,
Examen General De Orina (EGO): Protenas 30mg. Se estabilizan cifras tensinales, motivo por el cual se decide interrupcin del
embarazo va abdominal. Se ingresa a quirfano con hipotensin arterial de hasta 80/40, recuperndose con carga de solucin
cristaloide y bolo de efedrina. Se inicia ciruga y al entrar a cavidad abdominal se encuentra hemoperitoneo de 600cc. Se realiza cesrea
Kerr, obteniendo producto nico vivo hipotnico. Se ampla incisin y se realiza revisin heptica, encontrndose ruptura heptica del
lbulo derecho. Se prosigue para realizar empaquetamiento. Paciente en choque hipovolmico, reanimada con soluciones cristaloides
coloides y tres paquetes globulares, as como aminas (dopamina y dobutamina). Se administran esteroides a dosis de choque. No se
cuenta con plasma ni derivados plaquetarios, motivo por el cual se traslada a tercer nivel al servicio de terapia intensiva con sedacin y
analgesia con midazolam y fentanil y relajacin con vecuronio. Sin embargo, fallece a las 24 horas por falla orgnica mltiple.

PREGUNTA
Cual de las siguientes observaciones relacionadas al DOM es ms frecuente en la patologa de base?

RESPUESTA
a.- Disminucin de la fraccin de eyeccin.
b.- Disminucin de la relacin PaO2/FiO2.
c.- Ictericia, plaquetopenia, alteracin de las PFH.
d.- Ilio paralitico, ulcera por estrs, colecistitis aguda alitiasica.











MANUAL DE TRABAJO DEL CURSO ENARM CMN SIGLO XXI
CURSO ENARM CMN SIGLO XXI TEL: 36246001 Pharmed Solutions Institute PGINA 228

COAGULACIN INTRAVASCULAR DISEMINADA (CID):
CIENCIAS BSICAS: Sndrome patolgico adquirido, trombohemorrgico que se produce como resultado de una activacin excesiva del
sistema de la coagulacin y alteraciones de la fibrinlisis y conlleva el depsito masivo de fibrina en la microcirculacin y lesin
orgnica. La CID se produce como consecuencia de una activacin excesiva, del mecanismo hemosttico, de la incapacidad de los
inhibidores fisiolgicos para neutralizar la coagulacin y de inhibicin de la fibrinlisis mediada por el inhibidor del activador del
plasminogeno de tipo endotelial (PAI-1). La formacin de trombos de fibrina y su depsito en los pequeos vasos (microangiopatia
trombotica) produce disfuncin multiorgnica, que consume plaquetas y factores de coagulaciones consecuentemente desencadena
una fibrinlisis secundaria. Otros trminos que suelen utilizarse para denominar este sndrome son: sndrome de desfibrinacin,
coagulopata por consumo y sndrome de activacin sistmica de la coagulacin. Frecuencia de CID en condiciones asociadas trauma
severo 50-70%, sepsis o infeccin severa 30-50%, feto muerto y retenido 50% (>5 sem), embolia de liquido amnitico 50%, abruptio
placentae 50%, HELLP 15%, neoplasias, anormalidades vasculares, circulacin extracorprea, falla heptica severa, shunt arterio-
venoso, rechazo de trasplante. PATOGENIA: La presencia de una condicin asociada a CID. Como la sepsis o el trauma, inducen la
activacin del sistema de coagulacin a
travs de la liberacin de citoquinas,
como IL-6, IL-1 y/o el factor de necrosis
tu oral (TNF); integrados como parte de
la respuesta inflamatoria e
incrementando la cantidad de factor
tisular asociado a factor VII. La
liberacin o exposicin en alta
concentracin de sustancias
procoagulantes, como el factor tisular
en el SNC. Como ocurre en el
traumatismo de crneo, puede
desencadenar un trastorno
coagulopatico local y/o generalizarse en una CID. La activacin del sistema de coagulacin, con exceso de trombina y el subsiguiente
deposito de fibrina en la microcirculacin, favorecen la agregacin plaquetaria y el consumo de factores de coagulacin. Los glbulos
rojos quedan atrapados en las mallas de fibrina, otros sufren ruptura mecnica al pasar entre la malla de fibrina (responsable de la
esquistocitosis). La microtrombosis intravascular, promovida en la micro-vasculatura parenquimatosa y sistmica, generan un aumento
de plasmina, que degrada el fibringeno, la fibrina y otros factores de la coagulacin; perpetundose el circulo vicioso de coagulacin-
fibrinlisis. El funcionamiento suboptimo de los sistemas anticoagulantes naturales, protena C o antitrombina, desbordados por el
incremento de trombina y plasmina, permiten que estas circulen libremente por el torrente circulatorio. El consumo de los factores de
la coagulacin y plaquetas, junto a la hiperfibrinolisis favorecen la aparicin de hemorragias.
FORMAS DE PRSENTACION: A) segn el tiempo de instauracin y
progresin: aguda y crnica. B) Por compromiso y extensin:
localozada, sistmica. C) Por sus manifestaciones:
hemorrgica(reaccin hemoltica trasnfucional, leucemia
promielocitica), trombtica (sndrome de Trousseau). DIAGNOSTICO:
Clnico; hemorragia, hemorragias cutneo mocosas generalizadas
(epistaxis, hematuria, petequias, equimosis, gingivorragia, hemorragia
digestiva, hemorragia en SNC), hemorragia tras incisin quirrgica,
heridas, catteres, o punciones vasculares, trombosis, prpura
fulminante, acrocianosis perifrica, gangrena de extremidades,
anemia microangiopatica, en SNC-delirio, coma, en pulmn-diestress, en rin-
insuficiencia renal aguda, necrosis cortical, en tubo digestivo-lceras
duodenales. La CID aguda se manifiesta como hemorragias, trombosis o ambas. Las hemorragias suelen ser de presentacin aguda, en
mltiples sitios, especialmente de puncin venosa o arterial, y frecuentemente asociada a petequias y prpura. La CID crnica se
presenta de manera ms solapada, con menores manifestaciones clnicas y de laboratorio, debe considerarse en paciente con
patologas asociadas a esta entidad (neoplasias, feto muerto retenido, aneurisma de aorta, hemangioma, trasplante, HELLP, reacciones
alrgicas), que presentan alteraciones leves de los parmetros de hemostasia o que sangran excesivamente frente a intervenciones
quirrgicas. El diagnostico de CID se fundamenta en el cuadro clnico: 1. Identificacin de la entidad causal de CID. 2. Valoracin de los
sangrados y/o trombosis. 3. Alteraciones coagul-mtrica de laboratorio. Pruebas de hemostasia bsicas para diagnostico de CID: 1.
Recuento de plaquetas, un recuento bajo o en descenso progresivo, es una determinacin sensible aunque no especifica. El 98% de
casos de CID presenta plaquetopenia y el 50% se encuentra por debajo de 50,000 mm
3
. 2. Producto de degradacin del fibringeno
(PDF) y dmero D (DD), frecuentemente estn elevados como manifestacin de hiperfibrinolisis secundario al incremento de plasmina;
son sensibles (95%) aunque poco especficos ya que otras condiciones como el trauma, ciruga reciente o tromboembolismo venoso
pueden incrementarlos, al igual que el deterioro de la funcin renal o heptica, por una dos induccin de su eliminacin. Aun no se han
estandarizado los puntos de corte considerados incremento moderado y alto necesarios para el score diagnostico de CID. 3. Tiempo de
pro trombina (TP) y tiempo de tromboplastina parcial (TTP), se encuentran prolongados en el 50-60% de los casos, aunque cerca de la
mitad de los pacientes pueden presentar tiempos normales o incluso acortados por la presencia de factores de coagulacin activados
como la trombina o el factor X. 4. Fibringeno, su descenso es poco sensible (28%), se encuentra sobre todo en estadios avanzados de
la enfermedad. 5. Frotis de sangre perifrica, la presencia de esquistocitosis, suele encontrarse en el 50% de los casos. No existe un
mtodo diagnostico estndar pero se han desarrollado scores diagnostico. La sensibilidad del score de CID manifiesta la Sociedad
Internacional de Hemostasia y Trombosis (ISTH) es de 91% y tiene una especificidad del 97%. TRATAMIENTO: La clave es la correccin
de la enfermedad desencadenante y dar soporte. No se debe posponer la medida teraputica requerida, por intentar corregir un
parmetro de coagulacin alterado. Hemoderivados; no estn indicados para corregir parmetros de laboratorio, pero si en presencia
MANUAL DE TRABAJO DEL CURSO ENARM CMN SIGLO XXI
CURSO ENARM CMN SIGLO XXI TEL: 36246001 Pharmed Solutions Institute PGINA 229

de sangrado activo, previo a intervenciones invasivas o en aquellos con alto riesgo de sangrado. Plasma (15 o 30ml/kg), plaquetas (una
unidad de plaquetas cada 10 kg de peso en pacientes sangrando con recuentos <50,000 mm3), cri precipitados (cuando fibringeno
esta < 1g/l, sin respuesta a plasma), concentrado de factores (el concentrado de complejo protrombnico, solo tiene factores vitamina K
dependientes y carece de otros factores como el factor V, no pudiendo corregir todos los factores deficientes en la CID). El factor VII
activado recombinante se ha utilizado en casos de sangrado que compromete la vida (su eficacia y seguridad aun no est definida).
Indicaciones de heparina en CID, debido a que la CID est caracterizada por una importante activacin de la coagulacin, el tratamiento
anticoagulante podra ser racional. No hay ensayos clnicos randomizados que demuestren que la heparina mejora los resultados
clnicos y en la CID aguda podra agravar hemorragias.

CASO CLINICO
Varn de 66 aos acude a urgencias por clnica de disnea a mnimos esfuerzos de 10 das de evolucin y fiebre de hasta 39C. Niega tos
y expectoracin. Inici tratamiento antibitico de forma ambulatoria sin presentar mejora. A su llegada presenta insuficiencia
respiratoria grave que requiere FiO2 elevadas. Presenta rpida progresin de la insuficiencia respiratoria y a las 24h precisa intubacin
orotraqueal y ventilacin mecnica. La radiografa de trax inicial muestra un infiltrado alveolar en la base izquierda con aparicin de
infiltrados pulmonares bilaterales en controles posteriores. Se realiza cateterizacin de la arteria pulmonar que descarta fallo cardaco,
con presin capilar pulmonar inferior a 18mmHg, y que muestra signos de hipertensin pulmonar moderada. Analticamente, destaca la
aparicin de coagulacin intravascular diseminada (D-dmero mayor a 20.000ng/ml, trombocitopenia mxima de 47.000/l y tiempo de
protrombina mximo de 1,58 ratio), fracaso renal agudo con cifras de creatinina de hasta 2,33mg/dl y elevacin de la lactato
deshidrogenasa (LDH) con valores mximos de 2.305U/l.

PREGUNTA
Cual de los siguientes antecedentes esta mas relacionado con el estado del paciente.

RESPUESTA
a.- Diabetes mellitus.
b.- Hipertension arterial.
c.- Cardiopatia isqumica.
d.- Enfisema pulmonar.

ANAFILAXIA:
CIENCIAS BASICAS: Reaccin de hipersensibilidad generalizada o sistmica, grave y que amenaza la vida, consideraremos que la
anafilaxia es una reaccin alrgica grave de instauracin rpida y potencialmente mortal. Desde el punto de vista clnico se trata de un
sndrome complejo, desencadenado por mecanismos inmunolgicos o no, con aparicin de sntomas y signos sugestivos de liberacin
generalizada de mediadores de mastocitos y basofilos, tanto a nivel cutneo, como en otros rganos. Las causas ms frecuentes son
medicamentos (antibiticos betalactamicos, AINES) y medios diagnostico (46-62%), alimentos (22-24%), picaduras de insectos (8-14%),
factores fsicos (3-4%), otros (incluye ltex 7%), idioptica (3-5%). La importancia relativa de cada uno de ellos vara considerablemente
en funcin de la edad, as los alimentos (huevo, leche, frutos secos, pescado y marisco) son la causa ms comn en nios y los frmacos
son ms frecuentes en adultos. SALUD PUBLICA: Cifras de incidencia entre 3,2-30 por 100,000 personas al ao, con una mortalidad
entre 0,05-2% del total de las reacciones. En el Shock anafilctico la incidencia vara entre 3,2-10 por 100,000 personas /ao y al menos
1 % es mortal. DIAGNOSTICO: Sospecha clnica; cuando aparece de una manera aguda (en minutos a pocas horas), un sndrome
rpidamente progresivo que afecta a la piel y/o mucosas y que se acompaa de compromiso respiratorio y/o circulatorio. Como las
mayora de las anafilaxias cursan con sntomas cutneos (>80%), con este criterio al menos un 80% de las anafilaxias serian
identificadas. Sin embargo existen presentaciones menos tpicas, como las que presentan exclusivamente hipotensin. Tambin se ha
descrito que las manifestaciones digestivas se
asocian con una mayor gravedad. Por ltimo es
importante tener en cuenta que la
concurrencia de una exposicin a un alrgeno
potencial o conocido para el paciente apoya el
diagnostico. Por ello se han establecido
criterios, con ellos se espera identificar ms del
95% de las anafilaxias. Cuando existe afeccin
cardiovascular con hipotensin se habla de
shock anafilctico. La dificultad en el
diagnostico estriba en que no hay signos o
sntomas patognomnicos; lo que s es tpico es
la rpida progresin en la gravedad e intensidad
de los sntomas. EVALUACION DE GRAVEDAD:
Se relaciona con la rapidez en la progresin de
los sntomas, con el tipo de antgeno, va de
entrada y rganos afectados. Factores relacionados como edad avanzada, presencia de patologa respiratoria o cardiovascular asociada,
tratamiento con IECAS o betabloqueantes o una mastocitosis de base, se han asociado con reacciones graves y mayor mortalidad, por
tanto es fundamental la evaluacin ABCD. Las anafilaxias graves vienen definidas por la presencia de cianosis, saturacin O2 <92%,
hipotensin, confusin, hipotona, prdida de conciencia o incontinencia. Las anafilaxias moderadas presentan signos o sntomas que
sugieren afeccin respiratoria, cardiovascular o GI, como disnea, estridor, sibilancias, nauseas, vmitos, mareo, sudoracin, opresin
torcica, sensacin de garganta ocupada o dolor abdominal. Las manifestaciones cutneas (eritema, urticaria, angioedema) no se
consideran criterios de gravedad. Laboratorio: Los niveles plasmticos de histamina (se metaboliza rpidamente) y de triptasa total,
CRITERIOS CLINICOS PARA EL DIAGNOSTICO DE ANAFILAXIA
La anafilaxia es muy probable cuando se cumple uno de los tres criterios siguientes:
1.Inicio agudo (minutos a horas) de un sndrome que afecta a la piel y/o mucosas (urticaria
generalizada, prurito, eritema, edema de labios, vula o lengua) junto con al menos uno de los
siguientes:
a. Compromiso respiratorio (disnea, sibilancias, estridor, disminucin del PEF, hipoxemia)
b. Disminucin de la TA o sntomas asociados de disfuncin orgnica (hipotona, sincope,
incontinencia)
2.Aparicin rpida (de minutos a algunas horas) de dos o ms de los siguientes sntomas tras la
exposicin a un alrgeno potencial para ese paciente
a. Afectacin de piel y/o mucosas
b. Compromiso respiratorio
c. Disminucin de la TA o sntomas asociados de disfuncin orgnica
d. Sntomas gastrointestinales persistentes (dolor abdominal, clico, vmitos)
3.Disminucin de la TA en minutos o algunas horas tras la exposicin a un alrgeno conocido para ese
paciente
a. Lactantes y nios: TA baja o descenso superior al 30% de la TA sistlica
b. Adultos: TA sistlica inferior a 90 mmHg o descenso superior al 30% sobre la basal
MANUAL DE TRABAJO DEL CURSO ENARM CMN SIGLO XXI
CURSO ENARM CMN SIGLO XXI TEL: 36246001 Pharmed Solutions Institute PGINA 230

esta es la prueba mas til para el diagnostico de anafilaxia, puede elevarse en muestras obtenidas entre los 15 y 180 min del comienzo
de los sntomas. Se aconseja un mnimo de 3 muestras seriadas, la concentracin normal en suero es inferior a 13.5 /l, una elevacin
de al menos 2 veces lo normal, es sugestivo de anafilaxia. TRATAMIENTO: 1. Valoracin inmediata del sistema cardiovascular y
respiratorio, si el paciente est en paro las medidas de resucitacin cardiopulmonar deber ser de extrema urgencia, ante la inminencia
del shock colocar al paciente recostado con los pies elevados. 2. La primera droga a administrar es la Adrenalina IM es la va de eleccin
para el tratamiento inicial, obtiene unas concentraciones plasmticas mas rpidas y elevadas que la va SC, presenta un mayor margen
de seguridad que IV. En solucin acuosa y una dilucin de 1:1000 y a una dosis de 0.01 ml/kg. Subcutnea dosis mxima de 0.3 ml.
cuando la anafilaxia fue secundaria a una inyeccin o a un piquete siempre y cuando no sea en la cabeza o cuello deber de recibir una
segunda administracin e 0.01 ml/ kg. sin pasar de 0.3 ml en la zona del piquete, esto reduce de una manera considerable la absorcin
el antgeno. Esta dosis de Adrenalina puede ser repetida cada 15 o 20 minutos dependiendo de la evolucin mxima de 3 dosis. La
administracin IV de adrenalina tiene un alto riesgo de arritmias, sin embargo Cuando la evolucin es mala y hay shock evidente o
Colapso vascular, podr administrarse a una dosis de 0.1 ml de adrenalina 1:1000 diluida en 10 ml de solucin fisiolgica (para quedar
en una dilucin de 1:100,000) y administrar IV durante un periodo de 15 a 20 minutos. Considrese esta medida cuando el paciente
est en riesgo de perder la vida. 3. Torniquete En caso de que la reaccin de anafilaxia sea debido a un piquete de insecto o por una
inyeccin se utilizara un torniquete cercano al sitio del piquete o inyeccin, este podr liberarse 1-2 minutos cada 10 minutos. 4.
Oxigeno siempre que sea posible hay que administra oxigeno a los pacientes con cianosis, disnea, sibilancias por espasmo bronquial, a
una dosis de 3 a 5 lts. por minuto. 5. Antihistamnicos el Benadryl (difenhidramina) a una dosis de 1-2 mg por kilo deber ser
administrada IV lentamente en un tiempo no menor de 15 minutos , puede ser administrada IM o hasta va oral, otra opcin es el
clorfeniramina IM, IV u oral a dosis de 0.35mgxkgx24 hs el antihistamnico deber de continuarse por va oral cada 6 hs para prevenir
recurrencia de la reaccin principalmente de urticaria y angioedema, principalmente para administracin oral los antihistamnicos de la
nueva generacin pueden ser muy buena opcin y continuarlos cuando menos por un lapso de una semana. Corticosteroides .- La
administracin temprana despus de la adrenalina y del antihistamnico es de mucha ayuda para prevenir la recurrencia de los
sntomas, y la fase tarda, la dosis inicial de la hidrocortisona es de 7-10 mg/kg. y posteriormente 5 mg/kg. cada 6 hs o
Metilprednisolona 2-3 mgx kg. y no debern ser descontinuados cuando menos por 3 a 4 das. Cuando a pesar de las medidas
enunciadas el paciente no mejora, persiste hipotenso o con problemas de dificultad respiratoria, deber de internarse inmediatamente
y de preferencia en una unidad de cuidados intensivos para la administracin de: 1. Lquidos parenterales de solucin glucosada-
fisiolgica inicialmente a una dosis de 30 ml por kilo en una carga para la primera hora, deber de repetirse de acuerdo a su evolucin
para mantener una presin por encima de 50 mmHg. 2. Vasopresores: cuando a pesar de las soluciones y manejo persiste hipotenso
deber de manejarse con vasopresores bitartrato de norepinefrina diluir en 250 ml (glucosado 5% y fisiologico ) 1 ml y pasar 0.5 ml
/minuto. Dopamina 15 mcg /kg/minuto IV, preferible sobre la norepinefrina en los pacientes con insuficiencia cardiaca.
Broncodilatadores.- Regularmente la misma adrenalina alivia la presencia de broncoespasmo sin embargo cuando est presente a pesar
de esta, La nebulizacion de salbutamol 0.5 ml de la solucin al 5% ,en 2 ml de agua y dos de fisiolgico para nebulizar durante 8-10
minutos y repetir cada hora las primeras tres horas es de muchisimo beneficio para mejorar la permeabilidad de las vas areas por
broncodilatacion. En caso de que el broncoespasmo persista deber de administrarse AMINOFILINA a una dosis de 5-7 mg por kilo en
solucin cuando menos 30 ml para pasar en 30 minutos y posteriormente una dosis de 5 mg por kilo para administrar cada 6 hrs
durante 30 minutos. Intubacin y traqueostomia.- Cuando a pesar del manejo el paciente no mejora y no se puede restablecer la
permeabilidad de las vas respiratorias por el edema deber de intentarse la Intubacin endotraqueal, Y NUNCA RETRASAR LA DESICION
DE ESTE PROCEDIMIENTO SI SE REQUIERE para el caso de la traqueostomia deber de ser realizada por personal calificado y en sala de
operaciones de ser posible. Terapia de soporte.- Despus de que el paciente ha sido estabilizado, deber de mantenerse la terapia de
mantenimiento con lquidos y drogas por el tiempo en que se requiera y las funciones vitales se hayan reinstalado. Esto puede ser de
pocas horas a varios das, en los caso ms severos la muerte puede ocurrir dentro de los primeros treinta minutos, con el manejo
usualmente la recuperacin es completa, a menos que haya sufrido durante la crisis un infarto al miocardio o dao al SNC. Un hecho es
que en estos casos las reacciones subsecuentes con el mismo antgeno pueden dar lugar a reacciones mas severas y mas rpidas, por lo
que un pilar muy importante del manejo de la anafilaxis es la Prevencin.

CASO CLINICO
Se trata de una paciente de sexo femenino de 26 aos de edad, que consult por un cuadro clnico de cinco meses de evolucin
consistente en ppulas eritematosas, prurito intenso y angioedema asociado al contacto con el fro y, en algunas ocasiones, asociado a
disnea. Los sntomas se presentaban con ambiente fro o con agua fra y sucedan en el rea expuesta. Cuando se baaba en piscina o
con agua fra, la reaccin era generalizada; cuando lavaba platos o se lavaba las manos, la reaccin era local. Cuando ingera bebidas
fras, no presentaba edema en lengua ni disfagia, pero s reaccin en las manos al sostener la botella o el vaso fro.

PREGUNTA
Cual es el tratamiento farmacologico sistmico mas adecuado.

RESPUESTA
a.- Hidroxicina.
b.- Loratadina.
c.- Prednisona.
d.- Clorfenamina.

CASO CLINICO
Se trata de una mujer de 77 aos, obesa, hipertensa y diabtica no insuln-dependiente. Haba sido intervenida quirrgicamente en
varias ocasiones sin sufrir accidentes anestsicos, y no era alrgica conocida a frmacos. Tras la induccin anestsica aparecio
inmediatamente un broncoespasmo que dificultaba la ventilacin mecnica, (la gasometra arterial durante el episodio agudo mostr
acidosis respiratoria severa con pH: 7,12; pCO2: 71 mmHg; pO2: 63 mmHg; HCO3: 23 mmol/l; EB: -7,7 mmol/l; SaO2 83% con FiO2 al
MANUAL DE TRABAJO DEL CURSO ENARM CMN SIGLO XXI
CURSO ENARM CMN SIGLO XXI TEL: 36246001 Pharmed Solutions Institute PGINA 231


CLASIFICACION DE LAS EXACERBACIONES DE LA EPOC Y TRATAMIENTO ANTIBIOTICO RECOMENDADO
GRUPO DEFINICION MICROORGANISMOS TRATAMIENTO ORAL TRATAMIENTO IV
I EPOC leve
Menor de 65 aos
No comorbilidad
<4 reagudizaciones/
ao
H. influenzae
S. pneumoniae
M. catharralis
Amoxi+clavulanico
Telitromicina
Levo o moxifloxacino
Azitromicina

IIa EPOC moderado o
grave
No riesgo para P.
aeruginosa
Igual +
enterobacterias
Levo o moxifloxacino
Telitromicna
Amoxi+clavulanico
Levofloxacino
Ceflasporina 3 y 4
Amoxi +
clavulanico
IIb EPOC moderado o
grave
Riesgo de P.
aeruginosa
Desnutricion
>4 reagudizaciones/
ao
Igual IIa + P.
aeruginosa
Ciprofloxacino
Levofloxacino
Betalactamico
parenteral con
actividad
antipseudomona
100% ), hipotensin extrema (tensin arterial sistlica 50 mmHg) y finalmente disociacin electromecnica que requiri resucitacin
cardiopulmonar. Durante el episodio aparecieron eritemas en pliegues, habones diseminados y enrojecimiento facial, y la
monitorizacin cardaca mostr una bradicardia a 30 latidos por minuto (lpm), y una elevacin de ST localizada en la cara inferior y
anterolateral baja en el electrocardiograma (ECG).

PREGUNTA
Cual es el tratamiento farmacolgico inicial para manejar la anafilaxia presentada.

RESPUESTA
a.- Metilprednisona.
b.- Prednisona.
c.- Adrenalina.
d.- Dexametazona.

ENFERMEDAD PULMONAR OBSTRUCTIVA CRONICA (EPOC) AGUDIZACIONES:
CIENCIAS BASICAS: La denicin de exacerbacin de EPC no
est

bien establecida y no existen criterios universalmente


aceptados, aunque generalmente suele describirse como el
empeoramiento agudo de los sntomas respiratorios en un
paciente con EPOC. La combinacin de sntomas descrita por
Anthonisen y cols. en 1987 (incremento de la disnea, aumento
de la produccin y purulencia del esputo) ha sido
habitualmente aceptada para la definicin de exacerbacin
pero realmente slo representa una parte del global. Por ello
nos parece ms adecuada la denicin propuesta por
Rodrguez Roisn: Empeoramiento mantenido de la situacin
de un paciente con EPOC, desde un estado basal, por encima de las fluctuaciones diarias, que es agudo en su comienzo y que necesita
modificar su tratamiento habitual. Es importante destacar que la tendencia a sufrir agudizaciones repetidas es caracterstica de la
EPOC. Los pacientes que sufren un nmero elevado de agudizaciones en un periodo de tiempo siguen con esa tendencia y sufrirn
agudizaciones en el futuro. La mayora de las agudizaciones (50-70%) se debe a infecciones, fundamentalmente bacterianas, mientras
que el resto puede estar ligado a factores diversos entre los que se encuentran la polucin, el incumplimiento del rgimen terapuco o
la toma no controlada de medicacin depresora del SNC, hiperrreactividad bronquial, TEP, neumotrax, sedantes. PATOGENIA: La
infeccin supone la causa ms frecuente de las agudizaciones (entre el 50-70% de las agudizaciones podran ser atribuidas a infecciones
respiratorias). La explicacin de que en muchas de las mismas no podamos determinar su etiologa puede deberse a la existencia de
infecciones bronquiales no detectadas con las tcnicas diagnsticas habituales. En este sentido, el dato clnico que mejor indica que la
exacerbacin est producida por una infeccin bacteriana es la coloracin amarillo-verdosa del esputo. En cuanto a los patgenos
bacterianos se ha observado que estn presentes entre el 25-80% de las exacerbaciones; pueden tener un papel primario en el
desarrollo de esta o ser una sobreinfeccin de un proceso viral inicial. El Haemophilus influenzae es la bacteria aislada con mayor
frecuencia en todas las series (2/3 de las exacerbaciones en general), seguido del Streptococcus pneumoniae, Moraxella catarrhalis y
Pseudomona aeruginosa. Se ha demostrado que un 30% de las muestras en las que se asla H. influenza pertenecan a pacientes
fumadores activos, lo que implica que los fumadores, por presentar una menor movilidad de los cilios, pueden presentar infecciones
respiratorias con mayor frecuencia. Estos mismos grmenes se han obtenido en cultivo de esputo y en diversas muestras de ms
fiabilidad. DIAGNOSTICO: Clnico; Los sntomas ms comnmente referidos por los pacientes con EPOC exacerbada son: incremento de
la disnea, aumento del volumen o purulencia del esputo, aumento
de la tos, sntomas referidos a vas areas superiores (odinofagia),
opresin torcica o escasa tolerancia al ejercicio. Adems, se puede
aadir la presencia de fiebre, taquicardia, taquipnea, cianosis y
sntomas derivados de la eventual presencia de insuficiencia
respiratoria. En ocasiones aparece sintomatologa cardiovascular
(taquicardia, hipo hipertensin, arritmias y signos de insuficiencia
cardiaca derecha) sntomas neurolgicos (cefalea, irritabilidad,
depresin, cambios en el ritmo de sueo, obnubilacin y
encefalopata). En la exploracin fsica los datos a resaltar son: uso
de la musculatura accesoria de la respiracin, movimientos
paradjicos de la caja torcica, aparicin o empeoramiento de
cianosis, edema perifrico, alteraciones auscultatorias (sibilancias,
roncus, crepitantes), signos de insuficiencia cardiaca derecha o en
casos graves inestabilidad hemodinmica y disminucin del estado
de alerta. El diagnstico de la exacerbacin de la EPOC es
eminentemente clnico. Las pruebas diagnsticas que a continuacin se enumeran van encaminadas a establecer la severidad de la
exacerbacin o a descartar procesos que pueden simular la exacerbacin y que requieren un tratamiento especfico. Gasometra
arterial; La insuficiencia respiratoria (IR) se define como la presencia de niveles de PaO
2
menor de 55 mmHg y/o PaCO
2
mayor de 50
mmHg. El pH es muy til para valorar el grado de hipoventilacin aguda insertada sobre la hipoventilacin crnica. En la acidosis
respiratoria aguda sin compensacin renal, el pH cae 0,08 por cada 10 mmHg de subida de la PCO
2
. En la acidosis respiratoria crnica
con compensacin renal normal, el pH cae 0,03 por cada 10 mmHg de subida de la PaCO
2
. Hay un grupo de pacientes con EPOC que
tienen valores gasomtricos alterados de forma basal; en estos casos hablamos de IR cuando se observa un cada de la PaO
2
igual o
MANUAL DE TRABAJO DEL CURSO ENARM CMN SIGLO XXI
CURSO ENARM CMN SIGLO XXI TEL: 36246001 Pharmed Solutions Institute PGINA 232

mayor a 10-15 mmHg de su valor basal, y/o cualquier valor de hipercapnia asociado a un pH< 7,30. No est

indicada la realizacin de
gasometra de entrada en todos los pacientes con exacerbacin; en los casos leves se puede controlar el nivel de oxigenacin
determinando la saturacin de 2 con pulsioximetra. Solo realizaremos determinacin de gases en sangre arterial en aquellos casos de
exacerbacin moderada o grave, o en pacientes leves con saturaciones de O2 bajas. RX de trax, ECG y otras tcnicas de imagen. Los
estudios radiolgicos convencionales tienen valor para identificar o descartar diagnsticos alternativos de enfermedades que pueden
cursar con sntomas similares a los de la exacerbacin de la EPOC. El ECG es til para demostrar la presencia de hipertrofia de ventrculo
derecho, arritmias o cardiopata isqumica. El diagnstico del EPOC agudizado en el entorno extrahospitalario es fundamentalmente
clnico: aumento de la disnea, acompaado o no de aumento de la tos, expectoracin o cambios en coloracin del esputo.
TRATAMIENTO: Manejo de exacerbacin; el paciente EPOC agudizado puede ser tratado o bien de forma domiciliaria o en el mbito
hospitalario. Como norma general el paciente EPOC levemoderado ser tratado de forma ambulatoria como primera opcin, salvo si la
causa de la exacerbacin requiere ingreso o si la evolucin no ha sido favorable tras tratamiento ambulatorio. El paciente EPOC grave o
muy grave generalmente requerir ingreso hospitalario, ya que la exacerbacin se asocia con frecuencia a complicaciones de peor
pronstico como insuficiencia respiratoria, insuficiencia cardiaca, neumona, TEP. Indicaciones para ingreso hospitalario en EPOC
agudizado: EPOC grave o muy grave, edad avanzada, presencia de comorbilidad asociada (incluida neumona, arritmia cardiaca,
insuficiencia cardaca congestiva, DM, insuficiencia renal o heptica), mala respuesta al tratamiento ambulatorio, insuficiencia
respiratoria (Sat de O2 <90% o PaO2 <60 mmHg), taquipnea > 25 rpm, uso de musculatura accesoria, hipercapnia, fiebre alta (> 38,5),
disminucin del nivel de consciencia Respiratorias De ingresar se usan dosis elevadas de broncodilatador en forma nebulizada
(asociacin de agonista 2 de accin corta (2.5-10mgs) + bromuro de ipratropio (0.5-1mg) cada 4-6hrs), oxigeno suplementario si
insuficiencia respiratoria. Antibioticoterapia si EPOC grave. Glucocorticoides; metilprednisolona IV 0.4-0.6mg/kg cada 6 hrs durante 3-4
das, posteriormente se reducir en pauta descendente y se pasara va oral cuando sea posible. Indicaciones de ingreso en UCI:
Insuficiencia respiratoria aguda en la que a pesar de FiO2 40 % la PaO2 es menor de 60 mmHg. Hipercapnia progresiva con / sin acidosis
respiratoria. Apnea. Fatiga muscular respiratoria

CASOS CLINICOS
Mujer de 84 aos de edad con asma bronquial desde la infancia e hipertensin arterial en tratamiento con espironolactona y
amlodipino. Un mes antes de su ingreso present una reagudizacin respiratoria, y se le prescribi por vez primera un compuesto
inhalado cada seis horas, que asociaba bromuro de ipratropio y salbutamol. 2 dias despus ingresa a urgencias debido a que presenta
tos productiva escasa con dificultad respiratoria, sibilancias crepitantes, diaforesis, palidez generalizada, cianosis central, se realiza tele
de torax donde se observa imgenes sugerentes de consolidacin y reforzamiento de la trama broncovascular, rectificacin de
diafragma, sin datos de derrame pleural.

PREGUNTA
Cual es la conducta a seguir mas apropiada.

RESPUESTA
a.- Iniciar con antibitico de amplio espectro.
b.- Microneubilizacion con teofilina.
c.- Aminofilina.
d.- Prednisona.










MANUAL DE TRABAJO DEL CURSO ENARM CMN SIGLO XXI
CURSO ENARM CMN SIGLO XXI TEL: 36246001 Pharmed Solutions Institute PGINA 233

ENFERMEDAD TROMBOEMBOLICA VENOSA = TVP (Trombosis venosa profunda) + TEP (Tromboembolia pulmonar aguda)
CIENCIAS BASICAS: Triada de Virchow, base para TVP y TEP=Lesin o trauma endotelial, disminucin del flujo sanguneos o estasis,
hipercoagulabilidad o estado tromboflico. Se requiere un ambiente reolgico propicio = velocidad menor de retorno venoso y corriente
local con rotaciones elipsoidales y casi estticas. Factores predisponentes: Inmovilizacin prolongada 2-4 das (25%), antecedente
previo de TVP (19%), neoplasia maligna (17%), ciruga (ortopdica de rodilla 70%, abdominal) o traumatismo + miembros inferiores y
huesos largos, en los ltimos 3 meses (13%), uso de anticonceptivos orales (4%), IAM, ICC, edad > 40aos, embarazo, parto y puerperio,
obesidad, venas varicosas, viajes largos. Dentro de los estados hipercoagulables; estn las trombofilias primarias hereditarias
(deficiencia de antitrombina, protena C y S, mutacin del factor V de Leiden) y adquiridas (sndrome antifosfolipido secundario a LES).
SALUD PUBLICA: Afecta aprox. a 2 millones de personas. Incidencia de 145 por 100,000. La TVP va de un caso por cada 10,000 de
adultos jvenes y 1 por cada 100 de adultos mayores. En personas de 65 a 69 aos es de 1.8 habitantes por cada 1,000 y aumenta a 3.1
en el grupoentre 85-89 aos. TEP Causa frecuente de mortalidad en pacientes hospitalizados (5-10%). Toda TEP, tiene su origen en una
TVP o por lo menos el 80%. TEP masiva 0.3%. PATOGENIA: La TVP Se inicia con mayor frecuencia en las piernas (venas gemelares y
soleas). SI hay dao a nivel endotelial se activa la cascada de coagulacin, el trombo se asienta sobre el seno valvular, las corrientes de
flujo cambian localmente, disminuye el calibre de la vena, se produce estenosis y mayor fuerza de rozamiento, que permite el
crecimiento del trombo y posteriormente adherencia, al disminuir el retorno venoso de la extremidad, hay acumulacin de lquido y
aumento de la presin hidrosttica, lo que lleva a Edema, en la zona donde ocurre la oclusin aumenta la extraccin de O2, de los
tejidos lo que produce cianosis. Lo peligroso no es que se asiente, si no que siga creciendo y se fragmente y produzca embolismo que
llega a obstruir la circulacin arterial pulmonar (TEP) en diferentes grados. En la TEP en estadios tempranos puede tener un GC normal
o levemente elevado, hay hipoxemia (disminucin de la relacin ventilacin/ perfusin), esto aumenta la precarga, generando un
gradiente de presin que llevara a isquemia subendocardica y finalmente falla cardiaca derecha con la consiguiente disminucin de la
distensibilidad del VI. En la TEP masiva no hay mecanismos compensatorios, manifestndose clnicamente como taquicardia sinusal y
finalmente como hipotensin arterial sistmica, que llevara a la muerte en poco tiempo. DIAGNOSTICO: TVP: No puede ser afirmado, ni
excluido por clnica, la mayora asintomtico. En parte distal a sitio de oclusin; edema, dolor, calor local, cianosis (+ en regin plantar),
circulacin colateral, cordn venoso palpable. En caso severo flegmasa cerulea dolens (edema severo, cianosis marcada, flictenas y
puede llegar a gangrena por compromiso de retorno linftico). Signo d Homans: dolor en parte alta de pantorrilla al realizar dorsiflexion
del tobillo. Maniobra de Olow: apretar pantorrilla posterior en busca de dolor. TEP: El dx. requerir de un alto grado de sospecha
clnica, basndose en; a) uno o mas factores de riesgo, b) comorbilidades (ej. cncer), c) sntomas y signos clnicos, d) hallazgos
paraclinicos, e) demostracin objetiva de hipoxemia y trombosis. Sintomatologa como: disnea sbita sostenida, taquicardia,
hipotensin arterial sistlica (<100), sncope, dolor precordial o colapso cardiorrespiratorio, se ha correlacionado con obstruccin
mayor a 30%; otros sntomas diaforesis, tos hemoptisis, edema de MsPs. ECG; proporciona datos inespecficos; taquicardia sinusal, eje
de QRS >90, T negativa profundas en V1-V4. Gasometra; hipoxemia y alcalosis respiratoria aguda. Dmero D (DD, es un producto de
degradacin de los puentes de fibrina, que se libera a la circulacin), se eleva cuando existe un proceso trombotico, si el valor es
<500ng/ml y sospecha clnica baja permite excluir el dx. de TVP. Valores elevados de DD nunca sern suficientes para dx. de evento
tromboembolco, la ausencia de elevacin del DD, prcticamente descartara un evento trmboembolico. Ecodopler; primera eleccin
en sospecha de TVP proximales. Flebografa; Gold Standart para dx. de TVP para todo territorio venoso ( contraindicado en insf. renal,
hipertiroidismo, embarazo, alergia a medio). En TEP el gammagrama ventiltorio-perfusorio, es el estudio inicial. La angiografa
pulmonar selectiva actualmente se mantiene como "estndar de or" para el dx. definitivo de TEP. Escala de Wells; estratificacin de
riesgo para trombosis proximales; 1. Cncer +1; 2. Parlisis o inmovilizavion de miembros inf. +1. 3. Encamamiento > 3 das +1. 4. Dolor
localizado +1. Aumento de tamao de miembros inferiores +1. Edema en pitting unilateral +1. Dilatacin venosa superficial +1. Otros
dx. posibles -2. Wells >2 Alta probabilidaf; <2 Baja probabilidad. TRATAMIENTO: HBPM actualmente es la piedra angular y estndar de
oro para tx. inicial, tiene como mecanismo de accin la unin con la antitrombina III e inhibiendo indirectamente a la trombina,
previniendo la formacin de trombo adicional y facilitando la fibrinlisis endgena. Vida media 12hrs. SC. No requiere monitoreo,
puede usarse en tx. ambulatorio, reduce mortalidad. Se puede usar durante embarazo porque no atraviesa la placenta. Enoxaparina
1mg/kg/12hrs. HNF (heparina no fraccionada), precisa control con TTpa, riesgo de trombocitopenia y osteoporosis. Iniciar de forma
simultanea con anticoagulacin y antagonistas de vitamina k VO , ya que los anticoagulantes orales tienen efecto procoagulante los
primeros 3 das, anticoagulacin oral (acenocumarol) control con INR; mantener de 2-3, primero, segundo y tercer da acenocumarol
mas enoxaparina, en cuarto da podemos retirar la enoxaparina, el tiempo de tratamiento con acenocumarinicos, depender del tipo
d evento y la coexistencia de factores de riesgo. La terapia fibrinolitica con estreptocinasa, urocinasa o factor activador de
plasminogeno, se reserva e indica solo en TEP masiva y datos clnicos de inestabilidad hemodinmica. PREVENCION: Paciente con
factor de riesgo transitorio: anticoagular por 3 meses (ej. embarazo). Paciente con TVP idioptico o estado protrombotico: anticoagular
6-12 meses. Paciente con TVP recurrente o persistencia de algn factor de riesgo: anticoagular indefinidamente. En qx. ortopdica
anticoagular por 30 das, en qx. gnral. tratamiento hasta anticoagulacin completa. Profilaxis: 1.- Riesgo bajo; qx. sin complicaciones en
<40 aos o qx. menos de 30 min. en >40 aos sin factores de riesgo en ambos casos. Recomendaciones: no tx. antitrombotico, solo
deambulacin precoz e hidratacin adecuada. 2.- Riesgo moderado: qx. gnral en > 40a y duracin > 30 min. o <40 aos n tx. con
anticonceptivos. Incidencia de TVP distal 10-40%; proximal 2-10%; TEP mortal 0.1-0.7%. Recomendaciones: HBPM c/24 hrs o HNF 5000
U c/8-12 hrs SC, 2 hrs antes de la intervencin. Si puncin peridural, aplicar 8 hrs antes, por riesgo de hematoma peridural que puede
expandirse y causar parapleja. 3.- Riesgo elevado: qx. gnral o urolgica n >40aos con historia de TVP o TEP o en qx. abdominal o
pelvica para tx. de neoplasia o qx. ortopdica mayor en miembros inferiores. Incidencia de TVP distal 40-80%; proximal 10-20%; TEP
mortal 1-5%. Recomendaciones: Prtesis de cadera o rodilla, HBPM c/12 hrs SC. profilctico cada 12 hrs. teraputica cada 24 hrs.
PRONOSTICO: Depender de la sospecha clnica, la instalacin de la profilaxis, y recordar que una TVP hasta en 80%, precede a una TEP

CASO CLINICO
Se trata de paciente masculino de 62 aos de edad el cual acude a consulta debido a la presencia de dolor en miembros inferiores,
generando dificultad para caminar. Cuenta con antecedentes de tabaquismo positivo hasta la actualidad refiere que con menor
intensidad que antes comenzando a los 20 aos de edad, hace dos aos ha presentado leve mareos y en una ocasin fue internado por
dificultad para movilizar su cuerpo, pero se recupero sin secuelas aparentes.
MANUAL DE TRABAJO DEL CURSO ENARM CMN SIGLO XXI
CURSO ENARM CMN SIGLO XXI TEL: 36246001 Pharmed Solutions Institute PGINA 234

PREGUNTA
Considerando la clasificacin de Rutherford cual es la categora que presenta el paciente?

RESPUESTA
a.- 2.
b.- 3.
c.- 1.
d.- 4.

PREGUNTA
Considerando las manifestaciones clnica del paciente cual es la causa menos probable de este caso?

RESPUESTA
a.- Compresin de una raz nerviosa.
b.- Proceso artrtico inflamatorio.
c.- Sindrome compartamental crnico.
d.- Insuficiencia venosa.

PREGUNTA
Cual es la conducta a seguir mas adecuada para establecer una teraputica adecuada en el paciente para identificar factores de riesgo
modificables?

RESPUESTA
a.- Buscar niveles de glucosa, colesterol y triglicrido.
b.- Suspensin de tabaquismo absoluta.
c.- Holter para identificar origen cardiolgico.
d.- Ejercicio secuencial programado.

CASO CLINICO
Acude a consulta paciente femenino de 34 aos de edad, originaria de yucatan la cual tiene como antecedentes de importancia 7
semanas de posparto y obesidad grado II, viajo mas de 12 horas por carretera para visitar a familiares, solicita atencin debido a
presencia de dolor e inflamacin de miembro plvico derecho, a la exploracin se observa dicha extremidad con inflamacin,
hipersensibilidad y caliente al tacto, con signo de Hoffman positivo, sus signos vitales son estables TA 115/75 mmHg, FC 81, FR 17,
temperatura 37 C, se realiza doppler apoyando el diagnostico de trombosis venosa, se ingresa para tratamiento.

PREGUNTA
Cul es el manejo farmacolgico ms adecuado para esta paciente:

RESPUESTA
a.- Heparina IV
b.- Enoxoparina.
c.- Warfarina.
d.- Acecumarol.

PREGUNTA
Cual es la complicacin mas importante que debe evitarse en el paciente?

RESPUESTA
a.- Tromboembolia pulmonar aguda.
b.- Evento embolico fatal subsecuente.
c.- Sindrome compartamental.
d.- ECV.

PREGUNTA
Considerando las reglas predictivas de Wells para trombosis venosa profunda, que calificacin presenta el paciente en este momento?

RESPUESTA
a.- 2.
b.- 3.
c.- 4.
d.- 1.

PREGUNTA
Considerando las reglas predictivas de Wells para embolia pulmonar?

RESPUESTA
MANUAL DE TRABAJO DEL CURSO ENARM CMN SIGLO XXI
CURSO ENARM CMN SIGLO XXI TEL: 36246001 Pharmed Solutions Institute PGINA 235

a.- 6.
b.- 4.
c.- 9.
d.- 3.

CASO CLINICO
Se trata de paciente femenino de 38 aos de edad con antecedentes de importancia para el padecimiento actual, usuaria de
anticonceptivo orales desde hace 8 aos, tabaquismo positivo desde los 18 aos, actualmente con IMC 32. Acude a urgencias refiriendo
que hace 24 horas regreso de un viaje de 12 horas de camino, el cual realizo en auto ella era la conductora, refiere que comenz con
dolor en las pantorrillas de predominio izquierdo, con leve dificultad para caminar, posteriormente se presenta malestar generalizado,
dificultad para respirar motivo por el cual se traslada al hospital, a la exploracin fsica se observa diafortica con dolor torcico, disnea,
sus constantes vitales son 95/60 mmHg, FC de 115 lpm, FC de 30 rpm, SaO2 de 90%, campos pulmonares si datos por agregar, se realiza
radiografia de torax, y medicin de gases arteriales siendo anormales.

PREGUNTA
Cual es el manejo mas apropiado para el caso.

RESPUESTA
a.- Enoxaparina.
b.- Heparina.
c.- Warfarina.
d.- Tecneplasa

SNDROME DE INSUFICIENCIA RESPIRATORIA AGUDA (SDRA):
CIENCIAS BSICAS: Antiguamente denominado sndrome de dificultad respiratoria del adulto, es un trastorno que se caracteriza por
una insuciencia respiratoria aguda hipoxmica debida al edema pulmonar causado por el aumento en la permeabilidad de la barrera
alveolocapilar. El SDRA constituye la manifestacin ms grave de una serie de respuestas al dao pulmonar agudo; estas respuestas
traducen las complicaciones de una reaccin sistmica ms extensa a la inflamacin o agresin agudas. Se dice que ocurre un dao
pulmonar agudo cuando se observa una hipoxemia grave, de comienzo agudo, y opacidades bilaterales difusas en una radiografa
anterior de trax, despus de excluir una hipertensin en la aurcula izquierda o en los capilares pulmonares. El SDRA se diferencia del
dao pulmonar agudo por la gravedad de la hipoxemia y se define como una relacin igual o menor a 200 mmHg entre la pO2 arterial y
la fraccin de oxgeno en el aire inspirado (PaO2/FIO2). El dao pulmonar agudo y el SDRA deben considerarse como las
manifestaciones primeras y ms fciles de reconocer de un trastorno sistmico infeccioso o inflamatorio. El pulmn adquiere una
enorme importancia en la agresin sistmica por que recibe todo el gasto cardaco y porque las alteraciones de su funcin se reflejan
rpidamente en clnica. Ms de la mitad de los casos comienzan en las primeras 24 horas desde la primera agresin. El dao pulmonar
agudo se asocia ntimamente a trastornos que producen un dao alveolar directo o un dao indirecto a travs del lecho capilar
pulmonar como e indicen a SDRA: Lesin directa del epitelio alveolar, aspiracin, infeccin difusa, semiahogamiento, inhalacin
txica, contusin de la va respiratoria, dao pulmonar indirecto, sndrome sptico, traumatismo no torcico grave, derivacin
cardiopulmonar. SALUD PBLICA: La probabilidad del SDRA vara segn la causa desencadenante, desde un 13 % en los casos de
sobredosis farmacolgicas hasta un 43 % en la sepsis. PATOGENIA: El dao pulmonar agudo es la consecuencia de la expresin
excesiva, sin ningn tipo de regulacin, de las respuestas inflamatorias generales y habituales a la infeccin, la agresin o ambas. El
dao afecta al epitelio alveolar y al endotelio capilar pulmonar, consecuencia del acontecimiento que inicia la cascada tan compleja de
reacciones celulares y bioqumicas. Estos acontecimientos atraviesan tres etapas: 1. Iniciacin; en la que el factor desencadenante
activa la cascada celular. 2. Amplificacin: en la que se reclutan y activan las clulas efectoras. 3. Dao; fase en la que los
acontecimientos se expresan en los tejidos. El dao lo producen los acontecimientos celulares asociados a los neutrfilos, macrfagos,
monocitos y linfocitos, que sintetizan diversas citocinas; estas, a su vez, determinan una activacin, quimiotaxis y adherencia celular.
Las clulas activadas producen una serie de mediadores inflamatorios, como los oxidantes, las proteasas, las caninas, los factores de
crecimiento, los neuroppdos, los activadores de la cascada del complemento, la coagulacin intravascular y la fibrinlisis. La
caracterstica fisiopatolgica del SDRA es un aumento en la permeabilidad vascular a las protenas, que determinan la falta de oposicin
al gradiente hidrosttica; por eso incluso elevaciones discretas de la presin capilar (producidas por una sobrecarga de lquidos por va
IV o la disfuncin cardiaca caracterstica de la sepsis) aumentan de forma considerable el edema intersticiales y alveolar. Este efecto
aditivo de la permeabilidad y los factores hidrosttica, se ilustra como la mayor gravedad del proceso en las zonas declives. El aumento
entre la relacin del tejido pulmonar y el gas situado en estas zonas determina que las presiones de cierre alveolar excedan de las
presiones transpulmonares locales y se produzca un cierre y un colapso alveolares. La tendencia al colapso se agudiza por la
disminucin cuantitativa de la sntesis de sustancia tensioacva, debido a la agresin de los neumocitos de po II y a otras anomalas en
el tamao, composicin y metabolismo del resto de la sustancia tensioacva depositada. Estas zonas atelectsicas del pulmn
contribuyen a reducir la distensibilidad del pulmn en su conjunto, pero las zonas de pulmn, no situadas en declive, poseen
propiedades mecnicas y de intercambio gaseoso normales. Gran parte de la ventilacin y del intercambio gaseoso se desplaza hacia
estas regiones pulmonares intactas; para establecer un smil, es como si la funcin respiratoria del adulto fuera suplida por un par de
pulmones de un beb. Dada la distensibilidad menor de los pulmones, es necesario que los msculos respiratorios generen altas
presiones inspiratorias, con lo que aumenta el trabajo de la respiracin. Esta mayor carga mecnica explica la fatiga de los msculos
respiratorios, la disminucin consiguiente de los volmenes corrientes y el empeoramiento del intercambio gaseoso. La hipoxemia y la
estimulacin de los receptores del parnquima pulmonar rgido determinan un aumento de la frecuencia respiratoria., una disminucin
del volumen corriente y un deterioro del intercambio gaseoso. La resistencia en las vas respiratorias aumenta porque el volumen
minuto debe sostenerlo un menor nmero de vas respiratorias venladas y por el estrechamiento de las vas causado por el exceso de
lquidos y el broncoespasmo. El intercambio gaseoso se caracteriza por relaciones venlacin-perfusin bajas y un cortocircuito amplio,
MANUAL DE TRABAJO DEL CURSO ENARM CMN SIGLO XXI
CURSO ENARM CMN SIGLO XXI TEL: 36246001 Pharmed Solutions Institute PGINA 236

acompaado de un gran espacio muerto. El cortocircuito obedece a la atelectasia, colapso vascular, mal funcionamiento de la sustancia
tensioactiva y atenuacin de la vasoconstriccin hipxica. El aumento del espacio muerto se debe a la obstruccin y obliteracin del
lecho capilar pulmonar. DIAGNOSTICO: Clnico; al inicio puede no mostrar sntomas, ni signos respiratorios. El signo ms precoz es el
aumento de la frecuencia respiratoria, seguido poco despus de disnea. En el periodo inicial, la determinacin de los gases en sangre
arterial muestra disminucin de la Po2 a pesar de la disminucin de la Pco2, por lo cual aumenta la diferencia de oxgeno alveolo
arterial. En este estado inicial, la administracin de oxgeno produce un incremento significativo de la Po2 arterial. La exploracin fsica
puede ser poco llamativa, aunque se pueden auscultar algunos estertores inspiratorios finos. Radiolgicamente, los campos
pulmonares pueden ser claros o mostrar slo mnimos infiltrados intersticiales focales. A medida que avanza la enfermedad, el paciente
se torna ciantico, con disnea y taquipnea crecientes. Pueden hacerse intensos los estertores, que se oirn fcilmente en todos los
campos pulmonares, junto a zonas de ruido tubricos; la radiografa de trax muestra extensos infiltrados intersticiales y alveolares de
carcter bilateral y difuso. En este momento, la hipoxemia no se puede corregir simplemente aumentando la concentracin de oxgeno
en el aire inspirados y hay que comenzar con la ventilacin asistida. En este estadio ms avanzado, el mecanismo principal de la
hipoxemia arterial es el cortocircuito de la sangre desde la derecha a la izquierda, a travs de los alveolos colapsados u ocupados. El
diagnostico de IRA se basa fundamentalmente en la determinacin de gases arteriales al encontrar PaO2 <50 torr o PaCO2 >50 torr. Las
manifestaciones nicas de hipoxemia o hipercapnia, sirven para el reconocimiento de la presencia de anormalidades importantes en el
recambio gaseoso, mas no para el diagnostico, ya que pueden ocurrir tardamente o aun faltar en presencia de IRA. La falla respiratoria
puede clasificarse en 2 tipos: Tipo I: llamada tambin oxigenatoria o hipoxemica; hipoxemia con PaCO
2
normal o bajo. Gradiente
alveolo-arterial de O
2
incrementado. Tipo II: denominada as mismo ventiladores o hipercapnica; hipoxemia con PaCO
2
elevado y
gradiente alveolo-arterial de O
2
normal. TRATAMIENTO: Tratamiento de sostn de la hipoxemia. Para alcanzar una Pao2 de 60 mmHg
(saturacin de O2 de aprox. el 90 %) debe emplearse el mtodo ms sencillo y la fraccin ms baja de oxgeno en el aire inspirado. Los
niveles ms altos apenas aaden ms oxgeno y, en cambio, conllevan el riesgo de toxicidad pulmonar por esta sustancia. Los tres
mtodos fundamentales para la oxigenacin, en orden de eficacia creciente, son las gafas nasales blandas, las mascarillas faciales
simples y las mascarillas faciales con una bolsa de reserva inspiratoria. Parece razonable empezar con flujos moderados (5-10 L/min de
O
2
al 100 %) y controlar la gasometra arterial; el flujo y la concentracin de O
2
se ajustan en funcin de los resultados. Soporte
ventilatorio mecnico; en presencia de SDRA, estas medidas poco enrgicas no suelen mantener una oxigenacin adecuada, sino que se
requiere la intubacin endotraqueal y la respiracin asistida con un respirador volumtrico. El motivo de aplicar respiracin asistida a
un enfermo que hiperventila no es incrementar la ventilacin sino el volumen pulmonar medio, abriendo las vas respiratorias
previamente cerradas y mejorando la oxigenacin. El objetivo de la ventilacin en el SDRA se basa en proporcionar un soporte
fisiolgico para el intercambio gaseoso y evitar las consecuencias mecnicas desfavorables de la intervencin. Como el pulmn sufre un
dao heterogneo en el SDRA, el ajuste de los volmenes corrientes para lograr este objetivo en todo el pulmn acarrea el riesgo de
sobredistender o romper los alveolos menos afectados y ocasionar la entrada de aire fuera del alveolo (barotraumatismo). Para evitar
esta ultima complicacin hay que restringir las presiones de distensin alveolares mediante el uso de volmenes corrientes
relativamente pequeos (aproximadamente 6 a 10 mL/kg de peso corporal magro) y ajustar la presencia respiratoria, de tal suerte que
el volumen minuto se asocie a un pH superior a 7.25 7.30. Dado el volumen tan reducido del pulmn aireado, se necesitan frecuencias
elevadas para alcanzar un volumen minuto adecuado. Tratamiento de la sepsis que causa SDRA: Identificar y tratar cualquier proceso
infeccioso, y Tratar de controlar la respuesta inmunitaria, sin regulacin, que provoca el dao pulmonar agudo. Todo foco localizado de
sepsis se debe drenar de inmediato. La mortalidad quirrgica es elevada, pero si no se drena un foco purulento, lo ms probable es que
el enfermo muera. Complicaciones: Las medidas empleadas para tratar el SDRA pueden asociarse a graves complicaciones. La toxicidad
del oxgeno, debido al uso prolongado de una Fio2 superior al 50 %, y la hidratacin excesiva empeoran a veces los infiltrados
pulmonares. Un volumen y presin corrientes elevados, por parte del respirador, son causa de barotraumatismo. La ventilacin
mecnica prolongadas predispone a la neumona hospitalaria. El SDRA y los trastornos subyacentes favorecen las fstulas
boncopleurales y la coagulacin intravascular diseminada. La fibrosis pulmonar, la hipertensin pulmonar irreversible y la insuficiencia
multiorgnica sugieren un mal pronstico en los enfermos con SDRA.

CASO CLINICO
Se trata de paciente masculino de 54 aos de edad el cual es ingresado a urgencia traido por ambulancia desde una distancia de 4 horas
de camino, refieren los paramdicos que es sobreviviente a accidente automovilstico, al parecer era el conductor, a la exploracin
conciente, orientado, con facies algicas, se observa contusion en esternn y parrilla costal de predominio izquierdo, discreta disnea,
hematomas multiples en miembros pelvicos, se encuentra fractura de femur no expuesta, signos vitales TA 105/80 mmHg, FC 105, FR
32. Fue estabilizado y se procedi a realizar ciruga de femur, se traslado a piso al 3 dia el paciente inicia con fiebre, disnea progresiva,
edema de miembros inferiores, se realizo radiografia de torax sin observarse hallazgo patolgicos, el electrocardiograma mostro un eje
de QRS mayor a 90 oC y ondas T negativas en V1 a V4.

PREGUNTA
Cual es su impresin diagnostica mas probable con el cuadro clnico del paciente hasta el momento?

RESPUESTA
a.- Tromboembolia pulmonar.
b.- SIRA.
c.- Embolia pulmonar grasa.
d.- Neumotorax.

PREGUNTA
Se realiza tratamiento anticoagulante sin aparente mejora, a las 12 horas posteriores se incrementa la disnea y ortopnea, intolerancia
al decbito y ansiedad generalizada, tos con expectoracin, dificultad respiratoria con uso de musculos accesorio, se realiza Rx de torax
porttil con sospecha de edema agudo de pulmon, cuales de los siguientes hallazgos no corresponden a ese diagnostico?
MANUAL DE TRABAJO DEL CURSO ENARM CMN SIGLO XXI
CURSO ENARM CMN SIGLO XXI TEL: 36246001 Pharmed Solutions Institute PGINA 237


RESPUESTA
a.- Opacidades alveolares diseminadas y bilaterales.
b.- Redistribucin del flujo pulmonar.
c.- Distribucin en ala de mariposa.
d.- Lineas B de Kerley.

EDEMA AGUDO PULMONAR (EAP):
CIENCIAS BSICAS: El EAP no es en s una enfermedad, pero frecuentemente es una complicacin de alguna otra anormalidad, la cual
puede tener origen en los pulmones o ms comnmente, en rganos extrapulmonares, (corazn). El edema pulmonar constituye una
verdadera urgencia mdica, y por ello debe identificarse rpidamente. El cuadro puede ser de origen cardiognico o no cardiognico.
En el EAP cardiognico, un fallo de bombeo hace que la sangre retroceda a la circulacin pulmonar. Cuando la presin hidrosttica
capilar pulmonar supera la presin osmtica coloidal, el lquido es impulsado fuera de los capilares pulmonares hacia el espacio
intersticial y el alveolo. En el de origen no cardiognico, las paredes de las clulas capilares pulmonares se vuelven ms
permeables. Constituye una de las urgencias ms graves en pacientes con enfermedades del corazn izquierdo. curre por aumento de
la presin capilar pulmonar secundaria a estenosis mitral o insuciencia cardiaca izquierda, de ah

que tambin se le conozca como


edema pulmonar cardiognico. PATOGENIA: El EAP es el acumulo excesivo de lquido extravascular en el pulmn, ya sea en el
intersticio (edema intersticial) o en el alveolo (edema alveolar). Para mantener seco el intersticio pulmonar funcionan varios
mecanismos delicados: 1. Presin osmtica superior a la presin capilar pulmonar; Las fuerzas hemodinmicas bsicas opuestas son la
presin capilar pulmonar (PCP) y la presin osmtica del plasma. En los individuos normales la PCP oscila entre los 7 y los 12 mmHg,
siendo la presin osmtica del plasma de 25 mmHg aproximadamente, por lo que .esta fuerza tiende a impulsar el lquido de regreso a
los capilares. 2. Tejido conjuntivo y barreras celulares relativamente permeables a las protenas plasmticas; La presin hidrosttica
acta a travs del tejido conjuntivo y la barrera celular, que en circunstancias normales son relativamente impermeables a las protenas
plasmticas. 3. Extenso sistema linftico; El pulmn posee una extensa red linftica que puede aumentar su flujo en 5 a 6 veces cuando
se encuentra con un exceso de agua en el interscio pulmonar. Cuando los mecanismos normales para mantener el pulmn seco
funcionan mal o estn superados por un exceso de lquidos el edema ende acumularse. Las causas de EAP son:
Cardiognicas: IAM, arritmias cardiacas, insuficiencia ventricular izquierda grave, shoc cardiognico. No cardiognicas: Inhalacin de
gases irritantes, neumona por aspiracin, Shock sptico, embolia grasa, sndrome de distrs respiratorio del adulto, administracin
rpida de lquidos intravenosos, sobredosis de barbitricos u opiceos. DIAGNOSTICO: Agitacin, angustia, nerviosismo. La valoracin
del edema agudo del pulmn a nivel respiratorio es laborioso; disnea (el edema pulmonar dificulta la distensibilidad pulmonar y hace
ms difcil la expansin de los pulmones). Hipoxemia (debido al deterioro del intercambio gaseoso). Existencia de hipoxia con
hipocapnia (hiperventilacin) ya que elimina gran cantidad de CO2 al respirar tan dificultosamente, desarrollando una alcalosis
respiratoria. Taquipnea 30-40 min (debido a la hipoxia el paciente respira ms de prisa). Crepitantes y sibilancias (al pasar el aire a
travs de las vas areas llenas de liquido, durante la inspiracin). Valoracin cardiovascular: Taquicardia y aumento de la presin
arterial (para compensar el dficit de aporte de oxigeno, el sistema simptico aumenta la frecuencia cardiaca). El sistema nervioso
simptico provoca vasoconstriccin que aumenta la presin arterial. Distensin de las venas yugulares. Esputo espumoso teido de
sangre (debido a la presin, algunos vasos pulmonares se rompen haciendo que los hemates pasen a los pulmones). Piel plida, fra y
sudorosa (para conservar el oxigeno, el organismo deriva sangre arterial desde la piel hacia rganos vitales). Tercer ruido cardiaco
(ritmo de galope), producido cuando durante el principio de la distole la sangre entra de prisa en el ventrculo izquierdo
distendindolo y da lugar a vibraciones anormales. En el trax puede observarse el empleo de los msculos accesorios de la respiracin
(tiros intercostales). Puede haber estertores audibles a distancia y sin necesidad del estetoscopio. uscar intencionadamente la
presencia de ritmo de galope que permir

establecer el diagnsco de insuficiencia cardiaca, mientras que con la auscultacin de un


ritmo de Duroziez haremos el diagnstico de estenosis mitral. La radiografa de trax es el estudio ms til para el diagnstico en
conjunto con la historia clnica del EAP, ya que muestra la imagen correspondiente al edema alveolar como una opacidad diseminada
en ambos campos pulmonares. Cuando los grados de hipertensin venocapilar pulmonar son menores, podrn observarse otros datos
como son: la redistribucin de flujo a los vrtices, la cisura interlobar visible, las lneas de erley o un moteado no difuso y en
ocasiones la imagen en alas de mariposa; la presencia de cardiomegalia orientar hacia el diagnsco de insuciencia cardiaca,
mientras que la ausencia de ella hablar de disfuncin diastlica. El electrocardiograma ayuda a identificar la posible cardiopata
subyacente o los factores desencadenantes, como un infarto del miocardio o una taquiarritmia como la fibrilacin auricular.
TRATAMIENTO: El manejo teraputico de esta grave complicacin tiene tres objetivos: 1. Disminuir la presin venocapilar: 2. Mejorar la
ventilacin pulmonar. 3. Tratamiento de la enfermedad causal. Medidas que disminuyen la hipertensin venocapilar: a). Sentar al
paciente al borde de la cama con las piernas pendientes para disminuir el retorno venoso al corazn. b). Aplicacin de torniquetes
rotatorios en tres de las cuatro extremidades, con la fuerza necesaria para que no desaparezca el pulso arterial y se cambia cada 15
minutos a la extremidad que se encuentra libre (sangra seca). c). Isosorbide o nitroglicerina por va sublingual. La administracin de
una tableta de cualquiera de estos dos frmacos tiene un poderoso efecto vasodilatador que reduce drsticamente el retorno venoso y
la congestin pulmonar. d). La administracin intravenosa de furosemida a razn de 20 a 60 mg promueve con gran rapidez la
movilizacin de lquidos del intersticio pulmonar hacia el rin, por lo que se reduce con gran eficiencia la congestin pulmonar. e).
Nitroprusiato de sodio.- Este poderoso vasodilatador mixto se administra por va intravenosa a razn de 0.3 a 0.8
microgramos/Kg/minuto, cuando el EAP es consecutivo a insuficiencia ventricular por crisis hipertensiva. En estos casos, esta medida es
de eleccin para el tratamiento del edema pulmonar. El efecto arteriodilatador reduce significativamente la postcarga y con ello mejora
la funcin ventricular y el gasto cardiaco, y por otro lado, el efecto venodilatador reduce el retorno venoso al corazn y la congestin
pulmonar. Medidas que mejoran la ventilacin pulmonar: a). Administracin de oxgeno mediante puntas nasales a razn de 4 litros por
minuto. b). Aminofilina 1 ampolleta de 250 mg muy lenta por va intravenosa; su efecto broncodilatador mejora la ventilacin pulmonar
as

como su efecto diurco potencia la accin de la furosemida. Tratamiento de la enfermedad causal: a). La mayora de los pacientes
que presentan EAP por estenosis mitral mejoran con las medidas antes anotadas. Cuando a pesar de ellas contina el cuadro clnico, se
requiere la intubacin del paciente para administrarle asistencia mecnica a la ventilacin pulmonar, especialmente la presin positiva
MANUAL DE TRABAJO DEL CURSO ENARM CMN SIGLO XXI
CURSO ENARM CMN SIGLO XXI TEL: 36246001 Pharmed Solutions Institute PGINA 238

respiratoria final (PEEP) que evita el colapso de las vas respiratorias pequeas y con ello asegura la venlacin alveolar. En estas
condiciones, el paciente deber

ser llevado al quirfano para realizar comisurotoma mitral de urgencia, que en estos casos es la nica
medida que alivia el cuadro de edema pulmonar. b). Cuando la insuficiencia ventricular izquierda es la causa, se requiere la
digitalizacin rpida con ouabana o lanatsido C, adems de las medidas para reducir la presin capilar y mejorar la ventilacin
pulmonar. c). El edema pulmonar que es causado por disfuncin diastlica, usualmente se presenta en la evolucin de un infarto del
miocardio agudo; debe ser tratado con las medidas que reducen la presin capilar pulmonar asociada a aquellas otras que mejoran la
ventilacin pulmonar y ambas a su vez, a las que reducen el efecto de isquemia miocrdica como lo son la administracin de
betabloqueadores (propranolol 10 a 20 mg cada 8 horas) o calcio antagonistas (diltiazem 30 mg cada 8 horas), ya que estos frmacos
reducen el efecto que la isquemia tiene sobre la relajacin ventricular. En efecto, cuando la relajacin se hace ms rpida y completa,
se reduce significativamente la presin diastlica del ventrculo izquierdo y con ello la hipertensin venocapilar. Estabilizacin
hemodinmica: Se considera una buena respuesta si a los 15 a 30 minutos del inicio del tratamiento, mejora la disnea, la frecuencia
cardiaca, disminuyen los estertores pulmonares, aparece una coloracin ms oxigenada de la piel y se produce una buena diuresis,
despus de la cual se le puede ofertar al paciente potasio, 1 ampolleta por va oral con agua para compensar las prdidas de de este ion
o adicionrselo a la infusin de la venoclisis

CASOS CLINICOS
Mujer de 25 aos de edad, primigesta, admitida por cuadro de preeclampsia severa, con presiones arteriales > 160/110 mmHg,
manejada con sulfato de magnesio y labetalol intravenoso sin obtener adecuada respuesta, por lo que se realiza cesrea, que culmina
con recin nacido sano, se traslada posteriormente a unidad de cuidados intensivos (UCI) para manejo con infusin de vasodilatadores
y vigilancia neurolgica por riesgo elevado de eclampsia. Al ingreso en UCI present presin arterial de 161/120 mmHg, taquicardia,
inadecuado patrn respiratorio, saturacin de oxgeno arterial (SaO2) del 89%, crepitantes bibasales y presin venosa central (PVC) de
16 mmHg. Se realiza radiografa de trax que muestra cefalizacin del flujo, con hilios pulmonares congestivos; se considera entonces el
diagnstico de EAP.

PREGUNTA
Cual es la conducta mas adecuada para el caso?

RESPUESTA
a.- Ventilacion mecnica.
b.- Antibioticoterapia profilctica.
c.- Furosemide y nitroglicerina.
d.- Sindenafil.

CASO CLINICO
Paciente de 68 aos de edad, fumador con criterios clnicos de EPOC, que acudi al servicio de urgencias por disnea progresiva hasta
hacerse a mnimos esfuerzos de 2 semanas de evolucin. No refera traumatismo torcico. Radiolgicamente, se diagnostic un
neumotrax izquierdo. Se insert un tubo de drenaje torcico, se reexpandi completamente el pulmn y mejor as la clnica del
paciente. Tres horas ms tarde tuvo un deterioro clnico consistente en dificultad respiratoria grave. Se auscultaban estertores
hmedos de gruesa burbuja en el hemitrax izquierdo. Una nueva RX de trax mostr imgenes compatibles con edema pulmonar
unilateral. Fue preciso conectarlo a ventilacin mecnica y que recibiera tratamiento vasoactivo.

PREGUNTA
Cual es la etiologa mas frecuente del caso.

RESPUESTA
a.- Arritmia cardiaca.
b.- Isquemia miocrdica.
c.- Insuficiencia cardiaca.
d.- Complicaciones del parnquima pulmonar.

NEUMOTRAX, NEUMOMEDIASTINO, HEMOTRAX, CONTUSION CARDIACA:
NEUMOTRAX: Presencia de aire en el espacio pleural que causa un mayor o menor colapso del pulmn, con la correspondiente
repercusin en la mecnica respiratoria e incluso en la situacin hemodinmica del paciente. La presin intrapleural es subatmosfrica
durante todo el ciclo respiratorio, y es el resultado de las fuerzas elsticas opuestas del pulmn y de la pared torcica. La presin
intraalveolar es relativamente positiva en relacin con la intrapleural, ayudando a mantener el pulmn en expansin durante la
respiracin. La entrada incidental de aire en el espacio pleural provoca una prdida de la presin negativa intrapleural y el pulmn
tiende a colapsarse por su propia fuerza retrctil. Si la cantidad de aire es importante, la presin pleural se aproxima a la atmosfrica, lo
que puede causar un colapso de todo el pulmn. Es frecuente que exista hipoxemia arterial como consecuencia de la desproporcin de
la relacin ventilacin/perfusin que se produce en el pulmn colapsado (shunt intrapulmonar). La entrada de aire al espacio pleural
desde el pulmn, como consecuencia del desgarro de la pleural visceral, es la causa ms frecuente de neumotrax. NEUMOTRAX
ESPONTNEO (NE): Ocurre en ausencia de antecedente traumtico o yatrognico que lo justifique. El NE se subclasifica en: NE
primario; ocurre en individuos aparentemente sanos, sin enfermedades pulmonares conocidas. Afecta a jvenes, con un pico de
incidencia entre los 20 y 40 aos de edad, y es ms frecuente en varones (6:1), con predileccin por individuos altos y delgados,
fumadores. El sustrato patolgico ms frecuente es la presencia de pequeas bullas subpleurales apicales (blebs) cuya rotura ocasiona
la salida de aire desde el pulmn hacia la cavidad pleural. El mecanismo de formacin de estos blebs y los factores que precipitan su
rotura no son bien conocidos. NE secundario Ocurre en pacientes con patologa pulmonar previa. Personas de mayor edad, excepto en
MANUAL DE TRABAJO DEL CURSO ENARM CMN SIGLO XXI
CURSO ENARM CMN SIGLO XXI TEL: 36246001 Pharmed Solutions Institute PGINA 239

los casos de fibrosis qustica, EPOC es la causa ms frecuente y la probabilidad de neumotrax es mayor cuanto ms avanzada es la
enfermedad. En estos pacientes el neumotrax se debe a la rotura de bullas intrapulmonares y, dado que su reserva funcional es ya
muy limitada. Del 2 al 4% de pacientes con SIDA pueden desarrollar neumotrax. La neumona por Pneumocystis carinii es la etiologa
ms probable en estos pacientes. La tuberculosis pulmonar y las neumonas necrotizantes, fundamentalmente las causadas por
estafilococo, tambin pueden ser causa de neumotrax. Aunque es rara, no debemos olvidar una entidad como el neumotrax
catamenial, que ocurre en mujeres con antecedentes de endometriosis entre otras. NE ADQUIRIDO: Iatrognico: consecuencia de
procedimientos invasivos, como toracocentesis, biopsia pulmonar transbronquial, biopsia pleural, lavado broncoalveolar, puncin
pulmonar transtorcica, el neumotrax por barotrauma (complicacin de la ventilacin mecnica, por rotura de alveolos).
NEUMOTORAX TRAUMATICO: neumotrax traumtico abierto es consecuencia de una herida penetrante en el trax, que pone en
comunicacin el espacio pleural y la atmsfera exterior (entrada de aire atmosfrico), y a su vez suele lesionar tambin el pulmn
(salida de aire alveolar). El neumotrax traumtico cerrado est causado habitualmente por una fractura costal, rotura bronquial o
lesin esofgica. Sntomas ms frecuentes son: Dolor torcico pleurtico de inicio agudo, que puede ser intenso inicialmente, pero
despus de unas horas suele evolucionar a dolor sordo e incluso cede espontneamente, disnea, que suele aparecer con mayor
frecuencia e intensidad en pacientes con enfermedad pulmonar subyacente. En el NE primario suele ceder habitualmente en el plazo
de 24 horas. Otros sntomas (menos frecuentes): tos improductiva, sncope, hemoptisis, debilidad de extremidades superiores,
sensacin de ruido extrao en el trax. Exploracin fsica: hiperinsuflacin y disminucin del movimiento del hemitrax afectado,
disminucin o abolicin del murmullo vesicular en el hemitrax afectado, disminucin de la transmisin de la voz, timpanismo,
disminucin de las vibraciones vocales. Palpacin heptica por aplanamiento diafragmtico y desplazamiento del hgado. El
neumotrax a tensin debe sospecharse ante la presencia de taquicardia superior a 135 latidos por minuto, hipotensin, o cianosis.
Tratamiento: se dirige a la eliminacin del aire inrapleural, as como la prevencin de ataques de recurrencia, se basa en administracin
de oxgeno, observacin y aspiracin de aire intrapleural. NEUMOMEDIASTINO: Aire en el mediastino no acompaado de enfermedad
inflamatoria o infecciosa (tambin se conoce como enfisema mediastinal) y es primordialmente una complicacin de la ventilacin
mecnica (barotrauma9, aunque tambin puede ocurrir espontaneo o secundario a trauma torcico o asma. Ocurre principalmente por
ruptura alveolar microscpica, as como por escape de aire de la va area superior o gastrointestinal. Se presenta con dolor
subesternal, que empeora con los movimientos respiratorios y cambios de posicin. Tambin puede presentarse con disnea, disfagia,
enfisema subcutneo y disfona. El tratamiento depende de la causa primaria. HEMOTRAX: Presencia de sangre en el espacio pleural.
El trmino hemotrax est reservado para los casos en los cuales el hematcrito del lquido pleural es al menos un 50% del
hematcrito de la sangre perifrica. Cuando en una toracocentesis diagnstica se obtiene lquido hemtico, debemos considerar la
medicin de su hematcrito. Los podemos clasificar en traumticos, no traumticos o espontneos y iatrognicos. Traumtico; la
sangre puede llegar al espacio pleural por lesin de la pared torcica, diafragma, parnquima pulmonar, vasos sanguneos o desde
estructuras mediastnicas. Cuando la sangre se acumula en el espacio pleural, sta tiende a coagularse rpidamente, como resultado
de los movimientos producidos por el corazn y los pulmones. Si el impacto ha sido muy intenso y la situacin del paciente lo permite,
la realizacin de un TAC podr aportar importante informacin de posibles lesiones asociadas. De acuerdo a la extensin radiolgica el
hemotrax puede clasificarse en tres grados: 1. El nivel del hemotrax se encuentra por debajo del cuarto arco costal anterior. 2. El
nivel se encuentra entre el cuarto y segundo arco costal anterior. 3. El nivel est por encima del segundo arco costal anterior. Esta
clasificacin es utilizada como gua por algunos autores para decidir la conducta teraputica: drenaje con tubo pleural en hemotrax de
primer grado, toracotoma en aquellos de tercer grado y en aquellos de segundo grado se puede comenzar con drenaje torcico y,
eventualmente, toracotoma de acuerdo a la evolucin clnica del paciente. Los pacientes con hemotrax traumtico deben ser tratados
lo antes posible con drenaje torcico. Esto permite una completa evacuacin de la sangre del espacio pleural, cuantificar las prdidas,
que disminuya la incidencia posterior de empiema, realizar autotransfusin del material obtenido y la rpida evacuacin de la sangre
del espacio pleural evitando un fibrotrax. Aproximadamente 20% de los pacientes con hemotrax requiere toracotoma. La
toracotoma inmediata est indicada en: heridas penetrantes con la sospecha o constatacin de lesiones viscerales graves y en
pacientes con hemorragia pleural persistente. Las complicaciones ms frecuentes de un hemotrax son la retencin de cogulos en la
cavidad pleural, el empiema pleural, el derrame pleural persistente y el fibrotrax. Hemotorax iatrognico; cuando un paciente
ingresado presenta un derrame pleural hemtico, se debe considerar el origen yatrognico. La causa ms frecuente de es la perforacin
de una vena central o arteria por la insercin percutnea de un catter, tras una puncin o biopsia pleural, puncin pulmonar
percutnea, manejo endoscpico para tratar las varices esofgicas. Hemotrax no traumtico: son poco frecuentes. La causa ms
comn son las metstasis pleurales, mientras que la segunda causa ms frecuente es la complicacin del tratamiento anticoagulante
por embolia pulmonar o patologa cardiaca. El hemotrax espontneo puede ocurrir como resultado de la ruptura anormal de un vaso
sanguneo intratorcico, como un aneurisma de aorta, aneurisma de arteria pulmonar, un ductus arterioso o una coartacin de aorta.
En algunos pacientes la causa permanece desconocida a pesar de la toracotoma exploradora. CONTUCION CARDIACA: Es la ms
frecuente de las lesiones que resultan de traumatismos cardacos contusos. Su intensidad es variable y raramente es fatal. El
mecanismo ms postulado para su produccin es la desaceleracin, donde el corazn golpea contra el esternn siendo comprimido por
la columna. El diagnstico de esta lesin es demorado o confundido por otras lesiones ms graves y agudas que desvan la atencin.
Debe plantearse ante un trauma de pared torcica anterior con o sin fracturas costales o de esternn, shock sin taponamiento,
insuficiencia cardaca, arritmias y frotes pericrdicos a la auscultacin. Diagnstico: ECG; puede estar alterado, principalmente al
ingreso (extrasstoles ventriculares, extrasstoles auriculares, fibrilacin auricular, bloqueo de rama, alteraciones del ST). Tiene
sensibilidad de 100% especificidad de 47%. Enzimas Cardacas (CK-MB, troponina I): se elevan igual que en un IAM y su toma debe de
ser seriada. Valores de troponina I < de 1.05 mcg/L entre el ingreso y las 6 primeras horas descartan CM en un paciente asintomtico,
mientras que valores > de 1.05 mcg/L son indicadores de continuar el estudio y la observacin. Si un paciente con sospecha de CM
tiene un ECG y una troponina normal a las 8 horas de observacin se puede descartar prcticamente con certeza una CM. ECO: se
observan alteraciones del movimiento de las paredes comprometidas, aquinesia, disquinesia e hipoquinesia. Se sugiere realizar
ecocardiografa en los pacientes inestables, en los que presentan arritmias o en los que irn a ciruga. Manejo: Debido al riesgo de que
aparezcan alteraciones del ritmo cardaco, los pacientes deben estar monitorizados continuamente en una UCI o rea controlada
durante las primeras 24 horas, si en las primeras 24 horas no se pesquisa complicacin la evolucin suele ser favorable. Los pacientes
con contusiones documentadas y compromiso hemodinmico, que requieren ciruga con anestesia general para tratar sus lesiones
MANUAL DE TRABAJO DEL CURSO ENARM CMN SIGLO XXI
CURSO ENARM CMN SIGLO XXI TEL: 36246001 Pharmed Solutions Institute PGINA 240

asociadas, deben monitorizarse perioperatoriamente con tcnicas hemodinmicas invasivas (lnea-A, catter de Swan-Ganz). En
resumen debe tratarse en forma similar al IAM, dejando en claro que este cuadro no aumentara la mortalidad perioperatoria en el
caso de ser necesaria la ciruga de urgencia.

CASO CLINICO
Paciente de 68 aos de edad, fumador, que acudi por disnea progresiva hasta hacerse a mnimos esfuerzos de 2 semanas de
evolucin. No refera traumatismo torcico. Radiolgicamente, se diagnostic un neumotrax izquierdo. Se insert un tubo de drenaje
torcico, se reexpandi completamente el pulmn y mejor as la clnica del paciente. Tres horas ms tarde tuvo un deterioro clnico
consistente en dificultad respiratoria grave. Se auscultaban estertores hmedos de gruesa burbuja en el hemitrax izquierdo. Una
nueva RX de trax mostr imgenes compatibles con edema pulmonar unilateral.

PREGUNTA
Cual es la conducta mas apropiada en el estado actual?
RESPUESTA
a.- Furosemide y nitroglicerina.
b.- Sindenafil.
c.- Ventilacion mecnica.
d.- Aminas vasoactivas.

CASO CLINICO
Varn de 30 aos de edad. Refera tos seca con una data de un mes, sudoracin nocturna, dificultad respiratoria progresiva y baja de
peso estimada en 10 kg. No refera fiebre y una semana antes de ser internado se le agreg diarrea lquida. Antecedente de
promiscuidad homosexual y gonorrea 8 aos antes. Ingres vigil, orientado, temperatura de 37 C, presin arterial de 100/65, pulso de
108/minuto, taquipneico con frecuencia respiratoria de 50/minuto. EF: adenopatas pequeas, mviles, no adheridas a planos
profundos e indoloras, tanto cervicales laterales y posteriores as como axilares e inguinales; boca con placas blanquecinas que
comprometan la lengua, cara interna de mejillas y faringe; pulmones con estertores finos en ambas bases. El examen cardiovascular y
abdominal careca de hallazgos patolgicos. En la exploracin de laboratorio tena: LDH de 833 U/L; hematocrito 44%; leucocitos 6.900
x mm3 (segmentados 87%, linfoncitos 8%). Radiografa de trax con infiltrados bilaterales y datos compatible con neumotrax.
PREGUNTA
Cual es el agente mas probable relacionado con el desarrollo de neumotrax considerando los antecedentes?

RESPUESTA
a.- M. avium.
b.- P. jiroveci.
c.- C. albicans.
d.- S. pneumonie.

CASO CLINICO
Paciente de 53 aos que es trado a Urgencias tras sufrir un accidente de trfico, al parecer es el que conduca. Refiere que se ha salido
de la carretera y recuerda todo lo que ha pasado (no ha perdido el conocimiento). Exploracin fsica: Llega consciente y orientado,
mueve todos los miembros y las pupilas son normales. Refiere intenso dolor torcico, especialmente en hemitrax izquierdo. Respira
espontneamente con gran trabajo respiratorio, taquipnea y sudoracin. La auscultacin pulmonar muestra hipofonesis en la mitad
inferior del hemitrax izquierdo y la percusin es mate. El resto de campos pulmonares es normal. No tiene dolor a otros niveles y la
exploracin cardiaca, de abdomen y de miembros es normal.

PREGUNTA
Cual es el diagnostico mas probable?

RESPUESTA
a.- Neumotorax.
b.- Hemotorax.
c.- Hemomediastino.
d.- Neumomediastino.

CASO CLINICO
Se trata de masculino de 27 aos de edad el cual cuenta con antecedentes de importancia tales como padre con enfisema pulmonar y
madre cardipata, presenta tabaquismo positivo desde los 17 aos, actualmente consume marihuana y ha probado el crack, refiere que
hace 4 horas inicia sbitamente con dolor en el torax del lado derecho, asi como dificultad para respirar, niega tos, fiebre o malestar
previo, refiere que se encontraba trabajando en su taller mecanico, a la exploracin fsica se observa ansioso, se encuentra afebril, no
se observa edema perifrico, su frecuencia respiratoria es de 25 rpm, FC 84 lpm, TA 120/90 mmHg, saturacin de oxigeno del 93 %, se
realiza tele de torax donde se observa neumotrax del 50 % del pulmon derecho.

PREGUNTA
Cul es la conducta a seguir.

MANUAL DE TRABAJO DEL CURSO ENARM CMN SIGLO XXI
CURSO ENARM CMN SIGLO XXI TEL: 36246001 Pharmed Solutions Institute PGINA 241

RESPUESTA
a.- Sello pleural.
b.- Aspiracion con aguja.
c.- Observacion.
d.- Toracoscopia.

TRAUMA TORACICO CERRADO Y ABIERTO:
CIENCIAS BASICAS: Es cualquier agresin o trauma sobre las paredes del trax que producir un dao en las estructuras slidas y partes
blandas comprendidas en la caja torcica. Los traumatismos torcicos pueden ser cerrados (contusos) o abiertos (penetrantes).
Traumatismo abierto: se denomina a lesin que rompe la integridad del tejido (atraviesa pleura parietal). Traumatismo cerrado: resulta
por aplicacin de energa que provoca lesin sobre los tejidos sin daar su integridad. SALUD PUBLICA: En Mxico los traumas cerrados
son mayoritariamente por accidente de trnsito, los penetrantes
son por arma blanca, aunque han aumentado las heridas por armas
de fuego. 8 de cada 100.000 son letales. Principales causas de
traumatismo torcico asociadas: Accidentes de trnsito (43%),
Suicidios (29%), Homicidios (22%.). Nos da gravedad en un Trauma
torcico: Impactos de alta energa: cada mayor a 6 metros.
Impactos de alta velocidad. Pasajeros despedidos del vehculo.
Atropello. Lesin penetrante de cabeza, cuello, trax, abdomen o
regin inguinal. Dos o ms fracturas proximales de huesos largos.
Quemaduras mayores al 15% de SC o que afecten cara o vas areas. Trax inestable. TIPOS DE TRAUMA TORCICO: Lesiones torcicas
letales: 1.-OBSTRUCCIN DE LA VA AREA; se puede producir por cuerpos extraos, secreciones, sangre, el manejo puede ser invasivo
o no invasivo, otra clasificacin es va area quirrgica y no quirrgica. Consisten en el manejo manual de la va area, intubacin
oro o nasotraqueal, cricotiroidotomia por puncin o quirrgica y la traqueotoma. 2.- NEUMOTRAX A TENSIN; es el escape de aire
hacia la cavidad pleural, mas comn neumotrax espontneos, el mediastino y la trquea se desplazan hacia al lado opuesto,
comprometiendo la posibilidad de respuesta ventilatoria por parte del pulmn sano, y afectando el retorno venos. Clnicamente se
manifiesta por dificultad respiratoria, taquicardia, hipotensin, desviacin de la trquea, ausencia unilateral de MV, timpanismo del
pulmn afectado, ingurgitacin yugular y cianosis tarda. El tratamiento inicial consiste en insertar una aguja, en 2 EIC en LMC del
hemitrax afectado, siempre por el borde superior de la 3 costilla. El definitivo es la insercin de un tubo de trax en el 5 EIC anterior
a la LAM, siempre por el reborde costal superior de la 6 costilla, en el hemitrax afectado. 3.-NEUMOTRAX ABIERTO; Al producirse
una herida en la pared torcica el aire penetrar preferentemente por la herida al igualarse las presiones intratorcica y ambiental y
ofrecer menor resistencia al paso del aire por la herida. El tratamiento inicial ser cubrir la herida con un apsito fijado en tres puntas.
El tratamiento definitivo consiste en la instalacin de un tubo de trax, distante de la lesin, y el cierre de la herida ser quirrgico.
Complicaciones: Inadecuada conexin drenaje aspirador. Inadecuada colocacin del drenaje. Oclusin bronquial. (Cuerpo extrao,
cogulo, rotura). Roturas traqueobronquiales. Grandes laceraciones pulmonares. Severa disminucin de la distensibilidad pulmonar. 4.-
HEMOTRAX MASIVO: Es el resultado de la acumulacin de sangre en la cavidad pleural, igual o superior a 1500 ml. La principal causa:
es la lesin de vasos hiliares y mediastinicos generalmente por heridas penetrantes. Clnicamente encontrar un paciente en shock, con
colapso de los vasos del cuello por hipovolemia o con ingurgitacin de estos por efecto mecnico de las cavidades. El tratamiento ser
simultneamente con reposicin de volumen de forma agresiva (cristaloides, coloides y sangre), y descompresin del hemitrax
lesionado con un tubo de trax nico N 28 32 f. Se har toracotoma s: Deterioro hemodinmico sin otra justificacin. Persiste un
drenaje superior a 1500 ml en las primeras 12 24 horas. Drenaje superior a 200 ml/hora en 4 horas. Persistencia de la ocupacin
torcica (hemotrax coagulado). 5.- TRAX INESTABLE: Clnicamente puede no ser detectado en primera instancia por la
hipoventilacin reactiva al dolor, y por los movimientos del trax. El tratamiento se basa fundamentalmente en una buena ventilacin,
eventualmente mecnica, oxigenacin, y tratamiento para el dolor. Si el paciente no est en shock la infusin de fluidos debe ser
cuidadosa para evitar la sobrehidratacin y el consiguiente edema pulmonar. Un adecuado y controlado balance hdrico. Cuando
coexisten fracturas costales mltiples en varias costillas consecutivas se produce una inestabilidad de la pared con movimiento
paradojico y alteracin de la mecnica respiratoria, con la consiguiente hipoxia. La gravedad de la lesin es directamente proporcional
al grado de alteracin del parnquima pulmonar en combinacin con el dao de la pared. 6.- CONTUCION PULONAR: Lesin del
parnquima pulmonar que causa hemorragia y edema localizado, producto de traumas en los que hay rpida compresin y
descompresin del trax. Se observa falla respiratoria tarda, lenta, progresiva y sutil, con o sin trax inestable. 7.- CONTUCION
MIOCRDICA: Difcil de diagnosticar, se sospecha por alteraciones al ECG (arritmias, extrasstoles mono o bifocales, taquicardia sinusal
inexplicable, FA, bloqueo de rama, o claramente un infarto), Eco cardiografa bidimensional e historia compatible. El tratamiento
tratara la manifestacin clnica o la arritmia especfica. 8.- RUPTURA DE AORTA: Producto de traumatismos cerrados, por laceracin o
arrancamiento de los puntos de fijacin de la Aorta. Signos clnicos alertan el diagnstico: Mediastino ensanchado en Rx de Tx. Fx de
1 y 2 costillas. Desviacin y elevacin del bronquio principal, de la trquea y el esfago hacia la derecha. Depresin del bronquio
principal izquierdo. Opacidad pleural apical. Tratamiento: quirrgico (reparacin o implante). 9.- RUPTURA DIAFRAGMTICA: Presente
con ms frecuencia en el lado izquierdo ya que se carece de la proteccin del hgado. Se sospecha el diagnstico por la presencia de
intestino, estomago en el hemitrax izquierdo. Los traumatismos penetrantes por arma blanca o de fuego a veces pasan inadvertidos y
solo se detectan aos despus cuando aparece la hernia diafragmtica. 10.- LESIN TRAQUEOBRONQUIAL: Laringe Diagnstico:
ronquera, enfisema subcutneo y crepitacin palpable de fractura. Manejo, en caso de va area obstruida: instalacin de IOT o
Traqueotoma. Trquea: las lesiones penetrantes son ms obvias que las provocadas por trauma. Se asocia a lesin de esfago y
grandes vasos. Diagnostico a travs de broncoscopa. Bronquios: La lesin de un bronquio mayor es rara y mortal, y ocurren a 2-3 cm.
de la carina. Diagnstico: hemoptisis y enfisema subcutneo, y se sospecha en neumotrax a tensin con gran escape de aire y es
confirmado por broncoscopa. Una vez confirmado el diagnostico de lesin traqueobronquial el tratamiento es la reparacin
quirrgica. 11.- RUPTURA ESOFGICA: Se sospechara cuando exista neumo o hemotrax a izquierda sin fracturas costales, trauma
directo al esternn o epigastrio con dolor y shock no explicado, adems puede haber salida de partculas de contenido digestivo por
TRAUMA TORACICO CERRADO TRAUMA TORACICO ABIERTO
Asociada a compresin y aceleracin-
desaceleracin
Asociada a heridas por arma blanca
y arma de fuego
Hay fracturas costales mltiples Puede o no haber fracturas costales
Puede haber hemo o neumotrax
tardo (>24hrs del trauma)
Hemo o neumotrax inmediato.
El trauma de va area superior se
manifiesta como estenosis
El trauma de va area cursa con
gran escape areo
Tratamiento quirrgico: requerido en
menos del 10% de lesionados
Tratamiento quirrgico: requeridos
entre 15-30% de lesionados.
MANUAL DE TRABAJO DEL CURSO ENARM CMN SIGLO XXI
CURSO ENARM CMN SIGLO XXI TEL: 36246001 Pharmed Solutions Institute PGINA 242

tubo de trax. Tratamiento: La conducta ser la reparacin directa, esofagostoma cervical de escape y yeyunostoma de alimentacin.
12.- TAPONAMIENTO CARDIACO: Producto de una herida penetrante, en su gran mayora, pero tambin puede aparecer por lesiones
de los vasos pericardicos o traumatismo cardaco en un traumatismo cerrado. Desde el punto de vista clnico se manifiesta por la
trada de ec, que consiste en el hallazgo de: aumento de la presin venosa central, disminucin de la presin arterial, apagamiento
de los ruidos cardiacos. Ingurgitacin yugular con la inspiracin en un paciente ventilando espontneamente es signo inequvoco de
taponamiento cardiaco (signo de Kussmaul). En el tratamiento, la pericardiocentesis por va subxifoidea es de eleccin en el
prehospitalario, para descompresionar el pericardio, basta extraer 15 20 ml, pero es una medida temporal, actualmente se postula
que en l SU y Hospital no deben hacerse pericardiocentesis a menos que sea para dar tiempo a la preparacin del pabelln. INSERCIN
DE PLEUROTOMA; cuidados postoperatorios: El paciente deber permanecer en el hospital hasta que el tubo torcico sea retirado.
Debe quedar conectado a una trampa de agua donde el pivote del sello de agua debe quedar sumergido a 2cm, y oscilar cuando est
conectado al paciente. Mientras el tubo est colocado en el trax del paciente, los enfermeros verifican con cuidado que no haya
escapes de aire, dificultades o problemas al respirar o necesidad de administrar oxgeno adicional al paciente. El paciente tosa y respire
profundo para facilitar que los pulmones se expandan de nuevo, ayudar con el drenaje y prevenir que los fluidos normales se alojen en
los pulmones. ATENCIN Y MANEJO: Se realizara la evaluacin y el tratamiento de forma paralela y simultnea al momento en que se
avanza en el examen del paciente, esto es ir resolviendo los problemas vitales en la medida que se va encontrando, y siempre en este
orden y no otro. A. Va area y control de columna cervical. Permeabilidad, estabilidad y seguridad de la va area (eventual IOT),
aspiracin de secreciones, fijacin de columna cervical firme y segura, cricotiroidotoma por aguja o quirrgica. B. Respiracin:
Oxigenacin, movimientos respiratorios, ventilacin asistida, oclusin de heridas torcicas abiertas, toracocentesis y drenajes torcicos.
C. Circulacin y control de hemorragias. Compresin directa de los sitios de hemorragia, evaluacin de los pulsos, masaje cardiaco
externo, instalacin de dos vas venosas perifricas proximales de grueso calibre para alto flujo, reposicin de volumen y uso de
frmacos endovenosos, analgesia y sedacin, monitorizacin cardiaca. D. Dficit neurolgicos. E. Exposicin corporal y abrigo.

CASO CLINICO




TRAUMA ABDOMINAL ABIERTO Y CERRADO:
CIENCIAS BASICAS: Lesin orgnica producida por la suma de la accin de un agente externo junto a las reacciones locales y generales
que provoca el organismo ante dicha agresin. Todo paciente puede presentar lesiones en mltiples rganos abdominales y, por tanto,
debe ser considerado como un paciente con traumatismo grave, desde el momento del ingreso en la unidad de urgencias. Un tercio de
los pacientes que requieren una exploracin abdominal urgente tienen un examen fsico inicial anodino, tener en cuenta que puede
tener un comportamiento impredecible y desestabilizarse en el momento ms inesperado. Importante conocer el mecanismo lesional
con el fin de anticipar las lesiones esperables. ANATOMA: Abdomen anterior: se define como el rea localizada entre una lnea
superior que cruza por las mamilas, los ligamentos inguinales y la snfisis del pubis como la lnea inferior, y las lneas axilares anteriores
lateralmente. Flanco: rea entre las lneas axilares anteriores y posteriores y desde el cuarto espacio intercostal hasta la cresta ilaca. El
espesor de la musculatura de la pared abdominal a este nivel, ms que las capas aponeurticas ms delgadas de la pared anterior,
acta como una barrera parcial a las heridas penetrantes, particularmente por arma blanca. Espalda: localizada atrs de las lneas
axilares posteriores, desde la punta de la escpula hasta las crestas ilacas, el espesor de la espalda y los msculos paravertebrales
actan como una barrera parcial a las heridas penetrantes. Anatoma interna del abdomen: Cavidad peritoneal: cubierto por la parte
baja de la parrilla costal, el abdomen superior incluye el diafragma, hgado, bazo, estmago y colon transverso. Debido a que el
diafragma en una espiracin total se eleva hasta el 4 espacio intercostal, las fracturas de costillas inferiores o heridas penetrantes en la
misma rea pueden involucrar estas vsceras abdominales. El abdomen inferior contiene el intestino delgado y el colon ascendente,
descendente y sigmoides. Cavidad plvica: rodeada por los huesos plvicos, corresponde a la parte baja del espacio retroperitoneal y
contiene el recto, la vejiga, los vasos ilacos, y en la mujer los genitales internos. Espacio retroperitoneal: contiene la aorta abdominal,
la vena cava inferior, la mayor parte del duodeno, el pncreas, los riones, los urteres, as como segmentos del colon ascendente y
descendente. Las lesiones en las vsceras retroperitoneales son muy difciles de reconocer porque el rea es de difcil acceso al examen
fsico y sus lesiones no son detectadas por medio del lavado peritoneal diagnstico, y difcilmente valoradas por la ecografa, adems
de ser de difcil exploracin fsica. SALUD PUBLICA: Traumatismos de los ms frecuentes, estimndose en 1 por cada 10 ingresos por
traumatismo en los servicios de urgencias. Las principales causas de muerte en los pacientes con traumatismo abdominal son: 1. Por
lesin de algn vaso principal, como vena cava, aorta, vena porta o alguna de sus ramas, o arterias mesentricas. Las lesiones
destructivas de rganos macizos, como hgado, bazo o rin, o sus asociaciones, pueden originar una gran hemorragia interna. 2.
Sepsis: la perforacin o rotura de asas intestinales o estmago, supone la diseminacin en la cavidad peritoneal de comida apenas
digerida o heces, con el consiguiente peligro de sepsis. Los trastornos de vascularizacin de un asa intestinal por contusin de la pared
intestinal o de su meso pueden manifestarse tardamente como necrosis puntiforme parietal y contaminacin peritoneal con sepsis
grave. CLASIFICACIN: Abiertos (penetrantes y no penetrantes), presentan solucin de continuidad en la piel. Cerrados; la piel no tiene
solucin de continuidad. PATOGENIA: Las principales causas de abiertos son las heridas por arma blanca (lesiones intrabdominales de
20-30%) y arma de fuego cuya frecuencia es creciente. La principal causa de cerrados son los accidentes de trfico. Otras causas son los
accidentes de trabajo, accidentes domsticos, accidentes deportivos, siendo estos mucho ms frecuentes que los abiertos.
TRAUMATISMO ABDOMINAL ABIERTO: Las heridas por arma blanca y de fuego de baja velocidad (< 600 m/seg) causan dao al tejido
por laceracin o corte. Ceden muy poca energa y el dao se localiza en la zona perilesional, afectando habitualmente rganos
adyacentes entre s, siguiendo la trayectoria de, objeto que penetra. Las heridas por proyectiles de alta velocidad (> 600 m/seg)
transfieren gran energa cintica a las vsceras abdominales, teniendo un efecto adicional de cavitacin temporal y adems causan
lesiones adicionales en su desviacin y fragmentacin, por lo que es impredecible las lesiones esperadas. TRAUMATISMO ABDOMINAL
CERRADO: Impacto directo; la energa cintica a los rganos adyacentes a la pared abdominal, puede provocar lesin. Desaceleracin:
mientras el cuerpo es detenido bruscamente los rganos intra abdominales animados an por la energa cintica tienden a continuar
MANUAL DE TRABAJO DEL CURSO ENARM CMN SIGLO XXI
CURSO ENARM CMN SIGLO XXI TEL: 36246001 Pharmed Solutions Institute PGINA 243

en movimiento producindose una sacudida, especialmente acusada a nivel de los puntos de anclaje, vasos y mesenterio que sufren
desgarros parciales o totales. Compresin o aplastamiento: entre dos estructuras rgidas, estas fuerzas deforman los rganos slidos o
huecos y pueden causar su ruptura o estallido de estos. Este es el mecanismo tpico de lesin del duodeno, en un accidente de
automvil con impacto frontal, donde aquel es comprimido entre el volante y la columna vertebral. VALORACIN INICIAL: El objetivo
en evaluacin primaria es evidenciar o descartar lesiones de riesgo vital e instaurar las medidas necesarias de soporte vital para
preservar la vida del paciente. El paciente con traumatismo abdominal debe ser considerado como traumatismo grave o
potencialmente grave y por lo tanto, el manejo de estos pacientes debe seguir las recomendaciones del ABC: A. Asegurar la
permeabilidad de la va area, con control cervical. B. Asegurar una correcta ventilacin/oxigenacin. Descartar neumotrax a tensin,
abierto, hemotrax masivo. Valorar la necesidad de soporte ventilatorio. Si no es necesario administrar oxgeno a alto flujo con
mascarilla (10-15 l/min). C. Control de la circulacin. Detener la hemorragia externa. Identificacin y tratamiento del shock.
Identificacin de hemorragia interna. Monitorizacin ECG estable. D. Breve valoracin neurolgica. E. Desnudar completamente al
paciente, controlando el ambiente y previniendo la hipotermia. DIAGNOSTICO: Valoracin clnica del estado de shock: aumento de la
frecuencia del pulso, pulso dbil y filiforme, piel plida, fra y sudorosa, disminucin de la presin del pulso, retardo en el relleno
capilar, alteracin de la conciencia, taquipnea, hipotensin y oligo anuria., dado que la hemorragia intraabdominal es la causa ms
frecuente de shock hipovolmico en estos pacientes. Inicialmente, se asume que el estado de shock es el resultado de la prdida aguda
de sangre y se la trata con una infusin rpida de volumen: un bolo inicial de 1-2 litros para un adulto de SF al 09 % o de solucin de
Ringer lactato. La restitucin por medio de catteres intravenosos perifricos de calibre grueso (14G 16G), dada su mayor rapidez de
canalizacin. El shock refractario a la infusin rpida de cristaloides sugiere sangrado activo y requiere de una laparotoma urgente.
Primeras medidas: Colocar sonda gstrica: cuyo objetivo es aliviar la dilatacin gstrica aguda, descomprimir y reducir el riesgo de
broncoaspiracin. Si existen graves fracturas faciales o la sospecha de una fractura de la base del crneo, la sonda debe introducirse por
la boca para evitar el riesgo del paso del tubo hacia el cerebro a travs de la lmina cribiforme. Canalizacin de dos vas venosas
perifricas con catter de gran calibre. Sonda vesical para aliviar la retencin de orina y descomprimir la vejiga, comprobar la
presencia de diuresis, su flujo horario, descartando previamente la presencia de lesin uretral. Se deben sacar muestras de sangre y
realizar una determinacin de BH, tiempos de coagulacin, gasometra, amilasa, niveles de alcohol, grupo sanguneo y pruebas
cruzadas. Evaluacin Secundaria: Historia clnica, a partir del propio paciente, de sus familiares y de los profesionales que han llevado a
cabo la atencin prehospitalaria. Mecanismo de produccin del traumatismo: a) En traumatismos cerrados es importante el tipo de
impacto, dao del vehculo, uso de sistemas de seguridad, el estado de otras vctimas. b) Para las heridas penetrantes, puede ser til
una descripcin del arma y de la cantidad de sangre perdida en el lugar del hecho. Tiempo de evolucin desde el trauma hasta la
recepcin del paciente. APP: alergias, patologas previas, medicacin habitual, ciruga previa, ingesta de drogas. Maniobras realizadas
por los profesionales de la atencin prehospitalaria: volumen infundido, vas canalizadas, necesidad de resucitacin cardiopulmonar.
Exploracin: Inspeccin; observar el trax, abdomen, espalda, pelvis, perin, espalada. Hay que observar las huellas en la piel y pared
de los puntos de impacto del agente agresor. Auscultacin; confirmar la presencia o ausencia de ruidos intestinales. La presencia de
sangre libre intra peritoneal o contenido gastrointestinal pueden producir un leo que produce una ausencia de ruidos intestinales. Las
lesiones en estructuras adyacentes, por ejemplo, costillas, columna o pelvis, tambin pueden producir leo. Percusin; detectar
matidez (presencia de lquidos) en caso de hemoperitoneo; timpanismo (presencia de aire) si hay dilatacin gstrica o desaparicin de
la matidez heptica por neumoperitoneo. Palpacin; debe repetirse peridicamente, por lo que debe ser realizado por la misma
persona para poder evaluar las diferencias que se originen. Primero, debe dirigirse al plano parietal buscando la presencia de
hematomas, o contusiones musculares. Luego debe investigar la presencia de contractura abdominal refleja, que es un signo fiable de
irritacin peritoneal, al igual que el signo del rebote positivo. Finalmente, hay que realizar una palpacin ms profunda buscando la
presencia de puntos o zonas dolorosa cuya topografa nos oriente a relacionarlas con los posibles rganos lesionados. Evaluacin
estabilidad plvica; La exploracin del anillo pelviano debe realizarse mediante una cuidadosa compresin lateral y antero posterior,
siendo dolorosa cuando hay fractura plvica. Una fractura de pelvis puede ser causa de shock hipovolmico, en ocasiones muy severo.
Produce hematoma perineal y genital a las 24-48 horas del traumatismo y puede acompaarse de hematoma retroperitoneal y
ausencia de hemoperitoneo. Examen del perin y genitales; lesiones externas y la presencia de signos de lesin uretral como sangre en
el meato, hematoma escrotal o desplazamiento hacia arriba de la prstata. La laceracin de la vagina puede ocurrir en heridas
penetrantes o por fragmentos seos de una fractura plvica. PRUEBAS COMPLEMENTARIAS: Analtica: bioqumica, hemograma,
tiempo de coagulacin, pruebas cruzadas, niveles de alcoholemia y anlisis de orina. ECG y monitorizacin de constantes vitales.
Radiografa de abdomen. Radiografa de trax: es importante para descartar la presencia de hemotrax, neumotrax o fracturas
costales. Radiografa de pelvis. Exmenes complementarios: Lavado peritoneal diagnstico (LPD); procedimiento invasivo que puede ser
realizado de forma rpida presenta una sensibilidad del 68% y una especificidad del 83%. La indicacin principal del LPD son Hallazgos
abdominales equvocos. Exploracin fsica no realizable por traumatismo raqudeo concomitante o alteracin de la conciencia
(traumatismo crneo enceflico o txico). Imposibilidad de reevaluacin contina. Hipotensin inexplicable. Prdida progresiva de
sangre (descenso progresivo del hematocrito). Puede realizarse mediante un mtodo abierto o cerrado. Un LPD negativo no excluye la
presencia de lesiones retroperitoneales o desgarros diafragmticos. El ultrasonido puede detectar presencia de hemoperitoneo, es un
medio rpido, no invasivo y seguro en el diagnstico de lesiones intra abdominales (cerrada o penetrante) y puede ser repetido
frecuentemente. Es ms sensible que el lavado peritoneal diagnstico para la determinacin de lesiones de vsceras macizas, aunque no
lo es tanto como la TAC. Su indicacin es absoluta en casos de embarazo, cicatrices abdominales por cirugas previas y alteracin de la
coagulacin. La TAC requiere el transporte del paciente a la sala de rayos X, administracin oral e intravenosa de contraste. Consume
tiempo y es utilizado nicamente en pacientes hemodinmicamente estables en los que no existe la indicacin inmediata de
laparotoma. La TAC proporciona informacin relativa a la lesin especfica de un rgano en particular y tambin puede diagnosticar
lesiones en el retroperitoneo u rganos plvicos que son difciles de evaluar en la exploracin fsica o en el LPD. Laparoscopia la utilidad
de la laparoscopia efectuada bajo anestesia local para identificar lesiones diafragmticas y cuantificar la cantidad de sangre
intraperitoneal. MANEJO DEL PACIENTE TRAS LA VALORACIN INICIAL: 1. Traumatismo Abdominal Cerrado, Hemodinmicamente
inestable: (imposibilidad de mantener una TA sistlica por encima de 90 mm Hg, frecuencia cardiaca inferior a 100 lpm o diuresis de 50
ml/h adulto) Si el paciente presenta signos abdominales patolgicos (distensin abdominal, peritonismo, neumoperitoneo en la
radiografa simple), entonces la indicacin de laparotoma debe ser inmediata. Si los signos abdominales son dudosos y el paciente
MANUAL DE TRABAJO DEL CURSO ENARM CMN SIGLO XXI
CURSO ENARM CMN SIGLO XXI TEL: 36246001 Pharmed Solutions Institute PGINA 244

presenta un traumatismo craneoenceflico o espinal severo, alteraciones de la conciencia por toxicidad, traumatismos toraco-
abdominales, debemos llevar a cabo un estudio rpido del abdomen que nos ayude a descartar la presencia de patologa abdominal,
fundamentalmente lquido libre. Para ello disponemos de dos pruebas que no son excluyentes: Ecografa abdominal. Lavado
peritoneal diagnstico. Otras pruebas radiolgicas que precisan mayor infraestructura y tiempo, como la TAC, no son posibles en el
paciente inestable. Hemodinmicamente estable. Se deben tener en cuenta las siguientes premisas a la hora de manejar un paciente
con un traumatismo abdominal cerrado: Las vsceras macizas se lesionan con ms frecuencia que las huecas. Traumatismo Abdominal
Abierto; Arma Blanca. - Si el paciente est inestable o presenta signos de irritacin peritoneal, debe ser sometido a una laparotoma
urgente. - Si est estable hemodinmicamente y no presenta signos de irritacin peritoneal, la primera maniobra que se debe realizar
es la exploracin del orificio de entrada del arma, comprobando si la herida es penetrante o no. Arma de Fuego; Puesto que la
trayectoria de una bala es difcil de predecir y dado que el 80-90% de los traumatismos por arma de fuego se asociaran a una o ms
lesin visceral, el tratamiento de estos pacientes ser quirrgico, realizndoseles una laparotoma urgente. Recomendaciones basadas
en evidencia, Recomendaciones NIVEL I: Laparotoma exploratoria est indicada para pacientes con LPD (+) TAC est recomendado para
pacientes hemodinmicamente estables con un EF equvoco (lesin neurolgica, lesiones extrabdominales). TAC es la modalidad
diagnstica de eleccin para manejo no operatorio de lesiones de vsceras slidas. En pacientes hemodinmicamente estables, LPD y
TAC son modalidades diagnsticas complementarias.

CASO CLINICO




INSUFICIENCIA RENAL AGUDA (IRA) Y GLOMERULOPATIAS AGUDAS:
CIENCIAS BASICAS: Se denomina IRA a la reduccin brusca, en horas o das, de la funcin renal; se produce una disminucin del filtrado
glomerular y un acmulo de productos nitrogenados sricos (urea y creatinina en sangre) con incapacidad para regular la homeostasis
(equilibrio cido-base e hidroelectroltico). Aunque se suele asociar a una disminucin de la diuresis (IRA oligrica), hasta un 40% de los
casos no cursan con oliguria e incluso puede existir poliuria (IRA no oligrica). La IRA suele presentarse como una complicacin de
enfermedades graves previas, apareciendo entre el 5 al 30% de enfermos hospitalizados. CLASIFICACION: IRA PRERRENAL O
FUNCIONAL (60-70%); Inadecuada perfusin renal que compromete el filtrado glomerular; sera, por tanto, una respuesta fisiolgica a
la hipoperfusin renal pero el parnquima renal est ntegro. Es reversible si se acta sobre la causa desencadenante de manera
precoz. Causas de IRA prerrenal son: hipovolemia (hemorragias, perdidas GI, perdidas renales, secuestro de lquidos al espacio
extravascular), disminucin del gasto cardiaco (ICC, TEP, hipertensin pulmonar), vasodilatacin perifrica (sepsis, anafilaxia,
antihipertensivos, anestesia), vasoconstriccin renal (hipercalcemia, norepinefrina, ciclosporina, anfotericina B, sndrome
hepatorrenal), alteraciones de las respuestas autorreguladoras renales (inhibidores de las PG, como los AINES, y/o IECAS). IRA RENAL,
PARENQUIMATOSA O INTRNSECA (25%); Dao en las estructuras anatmicas; se clasifica segn la estructura primariamente daada:
glomrulos, tbulos, intersticio o vasos renales. Se dividen las causas de IRA intrnseca en 4 apartados: 1.Necrosis Tubular aguda (NTA):
Causa ms frecuente de IRA intrnseca (70%), afecta a las clulas tubulares renales, desde lesiones mnimas hasta necrosis cortical, la
lesin de los tbulos renales puede ser por mecanismos isqumicos (cursa con oliguria) principalmente o txicos (aminoglucosidos,
cefalosporinas, contrastes radiolgicos, AINES, anestsicos, toxinas endgenas), puede cursar con diuresis conservada, incluso
aumentada. 2. Lesin glomerular: glomerulonefritis agudas y rpidamente progresivas, hipertensin maligna, vasculitis, sndrome
hemoltico-urmico, purpura trombocitopnica trombtica, toxemia del embarazo, esclerodermia. 3. Lesin tubulointersticial:
reacciones alrgicas a frmacos (antibiticos, AINES, diurticos), infecciones (legionella, leptospira, CMV, candidas). 4. Lesin de
grandes vasos: obstruccin de arterias renales (placa ateroesclertica, trombosis, embolia), obstruccin de venas renales (trombosis,
compresin). IRA POSRENAL U OBSTRUCTIVA (5%): Lesiones que produzcan un obstculo en la va urinaria que impida la salida de la
orina formada, provocando un aumento de presin que se transmite retrgradamente, comprometiendo el filtrado glomerular. Pueden
ser lesiones extrarrenales de urteres, pelvis (litiasis, tumores, fibrosis), vejiga (litiasis, cogulos, tumores, prostatismo, vejiga
neurgena), uretra (estenosis, fimosis) o tambin lesiones intrarrenales (depsito de cristales, cogulos, cilindros). Para que estas
causas produzcan una IRA es necesario que la obstruccin sea grave, prolongada y que afecte a tracto urinario distal (meato uretral
externo, cuello de la vejiga) o bien a los urteres de manera bilateral o unilateral en paciente con un nico rin funcionante.
DIAGNSTICO: Clnica de las diferentes formas de IRA depender de las causas desencadenantes. As, en la forma prerrenal destacaran
las manifestaciones de reduccin verdadera de volumen (sed, hipotensin, taquicardia, disminucin de la presin venosa yugular,
disminucin de peso, sequedad de piel y mucosas) o de reduccin efectiva de volumen (en este caso la exploracin revelar signos de
hepatopata crnica, insuficiencia cardaca avanzada, sepsis). Es importante destacar que en estos casos la IRA desaparece
rpidamente tras reestablecer la perfusin renal. En el caso de la forma renal o intrnseca hay que investigar la presencia de isquemia
renal prolongada (shock hipovolmico, shock sptico, ciruga mayor). En estos casos existe oliguria o incluso anuria (diuresis diaria <
100 ml). La probabilidad de que estemos ante un cuadro de NTA aumenta an ms si la IRA persiste a pesar del reestablecimiento de la
perfusin renal. La posibilidad de IRA nefrotxica requiere el estudio de los medicamentos que ha recibido recientemente el paciente,
exposicin a contrastes radiolgicos (sobre todo si se han realizado en pacientes de riesgo: ancianos, diabetes mellitus, deshidratacin
previa, mieloma mltiple). Las toxinas tambin pueden tener un origen endgeno como la mioglobina (tras una rabdomilisis, por
destruccin muscular aguda) o hemoglobina (tras hemolisis grave). Destacar que en la NTA por txicos la diuresis suele estar
conservada. En la forma posrenal la causa ms frecuente en el varn es la obstruccin del cuello
de la vejiga por una enfermedad prosttica (hiperplasia o carcinoma).La diuresis fluctuante es
caracterstica de la uropata obstructiva. Laboratorios: BH: urea, creatinina, glucosa, iones. CK. La caracterstica fundamental de la IRA
es la aparicin de uremia aguda de rpida aparicin. A nivel prctico se considera que esto ocurre cuando la creatinina plasmtica
aumenta 0,5 mg/dl/da durante varios das. Si la IRA ocurre en el seno de una IRC, se considera que el aumento debe ser mayor de 1
mg/dl/da. La creatinina es ms fiable que la urea para la el diagnstico de IRA. Tambin puede calcularse el grado de disfuncin renal
detectando el deterioro del aclaramiento de creatinina. Para ello, en urgencias, se puede usar esta frmula, de aclaramiento de
MANUAL DE TRABAJO DEL CURSO ENARM CMN SIGLO XXI
CURSO ENARM CMN SIGLO XXI TEL: 36246001 Pharmed Solutions Institute PGINA 245

creatinina (Ccr), es una prueba aceptada como medida de filtrado glomerular. El valor normal de Ccr es de 100-120 ml/min. En el caso
de IRA el Ccr calculado debe reducirse un 50%. Existir hiperpotasemia en casos de IRA oligrica o en estados hipercatablicos, como
sucede en la hemlisis, rabdomiolisis y en los casos de lisis tumoral. La hipopotasema se da en las formas poliuricas. La hiponatremia es
tambin un hallazgo frecuente. Un manejo incorrecto del paciente, con un aporte excesivo de agua en proporcin a la de sodio, puede
agravar an ms la hiponatremia. El aumento del cido rico es caracterstico de la IRA aunque habitualmente es moderado y
asintomtico. Suele existir hipocalcemia, hiperfosforemia e hipermagnesemia. La severidad de estas alteraciones ser paralela a la del
dao renal que las ha ocasionado. Hemograma: Puede tener gran importancia en el diagnstico diferencial entre IRA e IRC: as si
aparece una anemia normoctica normocrmica, estar ms en concordancia con una IRC. Gasometra arterial: El patrn cido-base
ms frecuente del fracaso renal agudo es la acidosis metablica ya que el rin es incapaz de eliminar los cidos fijos no voltiles.
Volumen urinario: debido a las variaciones en la diuresis de las distintas formas de IRA en general no tiene gran valor diagnstico,
aunque si sirve para clasificar la IRA como oligrica y no oligrica Sedimento urinario: En la IRA prerrenal el sedimento no contiene
clulas pero si cilindros hialinos. En NTA existen cilindros granulosos, pigmentados y de clulas epiteliales, generalmente en asociacin
con hematuria microscpica. Proteinuria: suele verse en la NTA, es de tipo tubular y menor de 1 gr/24 h. Sistemtico de orina: el
estudio de iones, urea, creatinina, osmolaridad y densidad junto al sedimento urinario son fundamentales para el diagnstico difrencial
de IRA prerrenal de NTA. ECG: Puede orientar hacia trastornos electrolticos sobre todo la hiperpotasemia (prolongacin del intervalo
PR, ensanchamiento del QRS y aplanamiento de la onda T) o hipocalcemia. La Rx. simple de abdomen informa sobre la existencia de
litiasis radiopaca y el tamao y silueta renal y con la radiografa de trax se puede valorar la existencia de sobrecarga de lquidos
(edema agudo de pulmn). Ecografa abdominal: diferencial de la IRA. Se puede descartar patologa obstructiva as como visualizar el
tamao renal, dato muy importante para distinguir entre IRA e IRC. TRATAMIENTO: Manejo de IRA prerrenal. Dieta rica en hidratos de
carbono y con aporte de protenas de alto valor biolgico entre 0,6-0,8 gr/Kg/da. Monitorizar tensin arterial, frecuencia cardiaca y
medicin de la ingesta y prdida de agua y sal. El mecanismo de control ms simple es el peso diario. El sondaje vesical ser necesario si
se precisa la medicin de diuresis horaria. Canalizar va perifrica y central y monitorizar presin venosa central (PVC), para ajustar el
aporte de lquidos a una PVC entre 4-8 cmH2O. Si no existe contraindicacin, se puede realizar una rehidratacin rpida (en unos 30
min) con 500-1000 ml de SF, controlando la presin arterial, la PVC y vigilando la respuesta clnica y diurtica. Si existe una prdida
hemtica grave se usar concentrado de hemates. Una vez corregida la volemia, el volumen urinario aumenta y se debe continuar con
reposicin de lquidos a ritmo de diuresis. Si hay una mala evolucin en observacin ser necesaria la consulta con el nefrlogo que
valorar la indicacin de dilisis. Manejo de NTA: una serie de medidas conservadoras pueden, sino evitar la necesidad de dilisis, si al
menos transformar una IRA oligrica en otra no oligrica lo que permitir un mejor manejo del enfermo. Para ello, una vez adoptadas
las medidas generales del apartado anterior se forzar la diuresis por medio de diurticos y dopamina. As se puede intentar el uso de
furosemida a dosis de 20 a 40 mg cada 6 horas, segn los valores de diuresis y creatinina. Tambin pueden usarse diurticos osmticos
como el manitol al 20%, a dosis de 80 ml cada 6-8 horas, en funcin de la respuesta. La dopamina a dosis de 3-5 g/kg/min puede ser
otra herramienta til para mejorar la perfusin renal. Para ello se diluye una ampolla de 200 mg en 250 de glucosado y se perfunde a 5-
10 gotas/min. El mecanismo de la dopamina es aumentar el filtrado glomerular y favorecer la accin de los diurticos. Se deben usar
antagonistas de los receptores de histamina H-2 para la prevencin de hemorragias digestivas. No se recomienda el uso de antibiticos
de manera profilctica. Manejo de la IRA posrenal: Definitivo es desobstruir. De manera transitoria se proceder al sondaje vesical en el
caso de patologa prosttica. En caso de obstruccin ureteral con hidronefrosis ser necesario la realizacin de nefrostomia percutanea.
Las indicaciones de dilisis en la IRA son: hiperpotasemia o hipontremia graves, acidosis metablica con bicarbonato plasmtico menor
de 10 mEq/l, sobrecarga de lquidos con edema pulmonar o insuficiencia cardiaca, pericarditis urmica, encefalopata urmica, ditesis
hemorrgica urmica y azotemia severa (urea > 250 mg/dl o creatinina > 10 mg/dl). GLOMERULOPATIAS AGUDAS (GNA): Se caracteriza
por inicio abrupto de hematuria macroscpica, oliguria, falla renal, disminucin sbita de las tasa de filtracin glomerular con retencin
de sodio y agua, manifestando edema e hipertensin y proliferacin de clulas endocapilares del glomerulo. La proteinuria vara
ampliamente en este sndrome y por lo general es menos de 3 g/dl. La principal es la post-infecciosa, donde el dao glomerular resulta
de un compromiso inmune, desencadenado por una variedad de infecciones bacterianas, virales o de protozoarios. La ms comn es la
postestreptoccica, afecta a nios entre los 2 y 10 aos, predomina ligeramente en hombres. Slo ciertas cepas nefritognicas de
estreptococo se asocian con la glomerulonefritis. La variedad ms comn de glomerulonefritis postestreptoccica es usualmente
despus de una infeccin farngea con estreptococo beta-hemoltico del grupo A. En los pases desarrollados, la nefropata IgA es, en la
actualidad, la ms frecuente de las glomerulonefritis primarias. La glomerulonefritis postestreptoccica es una enfermedad aguda y
reversible, caracterizada por recuperacin espontnea en la mayora de los pacientes. Tpicamente la hematuria gruesa y el edema se
presentan entre 7 das a 12 semanas despus de la infeccin estreptoccica. La resolucin espontnea de las manifestaciones clnicas
es por lo general rpida. La diuresis reaparece en una a dos semanas y la concentracin de creatinina srica retorna a nivel basal dentro
de las cuatro semanas. La hematuria microscpica desaparece dentro de los seis meses pero la proteinuria leve permanece esttica en
15 % de los pacientes despus de 3 aos. El hallazgo de anticuerpos contra antgenos estreptoccicos proporciona evidencia de
infeccin reciente, pero no es diagnstica de glomerulonefritis postestreptoccica. Son los ms solicitados los anticuerpos
antiestreptolisina o, antiestreptoquinasa, antihialuronidasa y antinicotinamina dinucletidasa. El tratamiento de la glomerulonefritis
postestreptoccica es de soporte, enfocado a la sobrecarga de fluidos con dieta hiposdica (2.4 g/sodio/da); la hipertensin responde
a diurticos de asa (furosemide) y antihipertensivos tipo IECA, pero se deben dar con precaucin si hay insuficiencia renal con
hipercalemia. La terapia antimicrobiana temprana del paciente con infeccin farngea o de la piel, puede prevenir la propagacin de la
infeccin estreptoccica y atena la severidad de la glomerulonefritis postestreptoccica, pero no previene el desarrollo de la misma.

CASO CLINICO
Varn de 18 aos, cuidador de animales, refiere un cuadro febril de larga evolucin; en dicho ingreso desarrolla un fracaso renal agudo
de evolucin subaguda con creatinina plasmtica (crp) de 6 mg/dl. Refera fiebre elevada de dos meses de evolucin, que haba
aparecido una semana despus de sufrir una dudosa picadura de garrapata. La fiebre se presentaba diariamente sin predomino horario
y con sudoracin nocturna. Haba recibido distintos tratamientos antibiticos de forma emprica. En el momento del ingreso llevaba 20
das sin tratamiento antibitico persistiendo el cuadro febril. Radiografa de trax: al ingreso, normal; un mes despus presenta
MANUAL DE TRABAJO DEL CURSO ENARM CMN SIGLO XXI
CURSO ENARM CMN SIGLO XXI TEL: 36246001 Pharmed Solutions Institute PGINA 246

pequeos infiltrados pulmonares bibasales con imgenes aerolares. La proteinuria que inicialmente fue de rango no nefrtico (1-2 g/24
h), posteriormente alcanza el rango nefrtico.

PREGUNTA
Cual es la conducta a seguir mas adecuada para establecer el diagnostico final.

RESPUESTA
a.- Gamagrama pulmonar.
b.- Biopsia renal.
c.- Urocultivo.
d.- Hemocultivo.

CASO CLINICO
Hombre de 27 aos; como antecedentes refera criptorquidia, adenoidectoma y amigdalectoma en la infancia, fumador activo,
bebedor social y homosexual. Dos meses con una discreta faringodinia, aparecieron una induracin en la ingle derecha, lesiones
ulceradas en el glande de aspecto serpinginoso y un exudado uretral blanquecino que fue tratado inicialmente con azitromicina. A la
espera de resultados serolgicos, se observaron lesiones maculopapulosas en la zona proximal de muslos y tronco, que se extendan a
los pies y a las manos, que evolucionaron en distintas fases sin acompaarse de fiebre, junto con edematizacin en los miembros
inferiores y en los genitales, con ligero aumento del permetro abdominal y disminucin de la diuresis. Urea 61 mg/dl, creatinina 1,73
mg/dl, iones normales, protenas totales 4,4 g/dl, albmina 1,8 g/dl, colesterol total 295 mg/dl, HDL 61 mg/dl, LDL 206 mg/dl,
triglicridos 140 mg/dl y enzimas hepticas normales. En la orina, destacaban proteinuria 13,4 g en 24 horas, 250 hemates por
microlitro y leucocituria negativa. Hemograma normal, coagulacin normal.

PREGUNTA
Cual de las siguientes pruebas tiene mayor sensibilidad y especificidad para establecer un diagnostico.

RESPUESTA
a.- ANA.
b.- ANCA.
c.- Anticuerpo FTA.
d.- VHC.






MANUAL DE TRABAJO DEL CURSO ENARM CMN SIGLO XXI
CURSO ENARM CMN SIGLO XXI TEL: 36246001 Pharmed Solutions Institute PGINA 247

SINDROME NEFRITICO Y SINDROME NEFROTICO:
SNDROME NEFRTICO: Criterios diagnsticos Proteinuria >3-3,5g/24 horas o una relacin protena: creatinina >3-3,5 (ambas en
mg/dL) en una muestra aislada. Albuminemia <25 g/L (hipoalbuminemia). Edemas perifricos. Es frecuente la hiperlipidemia severa
(colesterol total >385 mg/dL). Es consecuencia del aumento de la permeabilidad para las protenas y es expresin de una enfermedad
glomerular cuando existe una alteracin funcional o morfolgica de la barrera de ltracin. Nunca aparece en las enfermedades
extraglomerulares. El sndrome nefrtico constituye el motivo ms frecuente de realizacin de biopsia renal en todos los grupos de
edad. Manifestaciones clnicas: La prdida de protenas en orina origina una serie de alteraciones que constituyen las caractersticas del
sndrome nefrtico: 1. Hipoalbuminemia: aparece cuando la capacidad de sntesis heptica se ve superada por las prdidas urinarias de
albmina y el catabolismo renal. 2. Edema: suele ser la primera manifestacin; aparece en partes blandas y en casos graves puede
comportar ascitis, derrame pleural y anasarca, perorbitario, en miembros inferiores. Es debido a la reabsorcin de sodio y agua
secundaria a la disminucin de la presin onctica capilar o estimulada directamente por la proteinuria. 3. Hiperlipidemia con aumento
de las cifras de colesterol total, lipoprotenas de densidad baja (LDL), muy baja (VLDL) e intermedia (IDL), lipoprotena A y, con menos
frecuencia, hipertrigliceridemia y descenso de las cifras de lipoprotenas de alta densidad (HDL). La lipiduria se maniesta con aparicin
de cilindros grasos en el sedimento urinario. 4. Prdida de inmunoglobulinas y de factores del complemento, que conlleva una
tendencia especial a la aparicin de infecciones (peritonitis espontneas, celulitis, e infecciones pulmonares, menngeas y digestivas). 5.
Trombosis, con una incidencia de entre el 5 y el 60%; suele aparecer en venas renales y de extremidades inferiores, aunque tambin
pueden afectarse territorios arteriales. 6. Hipertensin arterial, que aparece en el 42,5% de los pacientes. 7. Insuciencia renal aguda,
que se observa con ms frecuencia en pacientes de edad avanzada, con hipoalbuminemia grave, tratados con dosis altas de diurticos.
Causas de sndrome nefrtico: Enfermedad glomerular primaria; esclerosis glomerular segmentaria focal, enfermedad glomerular
membranosa, enfermedad glomerular con cambios minimos, enfermedad glomerular membranoproliferativa (ej: IgA). Causas
secundarias de sndrome nefrtico: DM, LES, amiloidosis, mieloma multiple, agentes antimicrobianos, AINES, penicilamina, VIH,
hepatitis B y C, mycoplasma, sfilis. La biopsia renal est indicada en el estudio del sndrome nefrtico del adulto, excepto en pacientes
diabticos en los que no se sospeche otra enfermedad distinta de la nefropata diabtica. En nios con sndrome nefrtico, en principio
no est indicada la realizacin de biopsia renal (el 90% de los casos se debe a glomerulonefritis de cambios mnimos), salvo en caso de
sndrome nefrtico resistente a los corticosteroides o con recidivas frecuentes. El tratamiento general comprende: reduccin del
edema, control de la presin arterial y del perl lipdico y prolaxis de la trombosis. SNDROME NEFRTICO: Se caracteriza por edema,
oliguria, hematuria (con cilindros hemticos en el sedimento urinario), disminucin del ltrado glomerular e hipertensin arterial,
normalmente de instauracin aguda. Tpicamente se ha relacionado con la forma de presentacin de la glomerulonefritis aguda
posinfecciosa, sobre todo asociada a infecciones estreptoccicas. Tambin puede aparecer en otras afecciones (tabla 5).
Manifestaciones clnicas: Aparece como macrohematuria en el 30-50% de los casos, con datos de hematuria glomerular. En la
glomerulonefritis aguda posestreptoccica se presenta tras 2-3 semanas de una infeccin faringoamigdalar o 4-6 semanas de una
infeccin cutnea. La hipertensin arterial suele ser moderada, con edemas en prpados y extremidades inferiores. En algunos casos
puede producirse insuciencia cardaca congestiva. La proteinuria es inferior a 1-2 g/24 h. Es fundamental la realizacin de una historia
clnica completa, interrogando sobre antecedentes familiares, manifestaciones clnicas sistmicas, infeccin reciente o consumo de
frmacos. Patogenia: Las cepas nefritognicas producen protenas catinicas identificadas en tejidos renales de pacientes con
glomerulonefritis. Como consecuencia de su carga elctrica, estas protenas se depositan en el glomrulo y dan lugar a la formacin in
situ de complejos inmunes, adems de los complejos inmunes circulantes formados por la unin de inmunoglobulinas con otros
antgenos. Estos complejos circulantes es posible encontrarlos en la primera semana de la enfermedad y se sabe que estn en relacin
con la gravedad de la enfermedad; de ah su importancia a la hora de hacer el diagnstico. Aproximadamente el 90% de los casos con
glomerulonefritis presenta reduccin de los niveles sricos de complemento, debido a que despus del depsito en la membrana basal
de estos inmunocomplejos se activa la cascada inflamatoria, en un inicio mediada por interleucina 1 con la subsecuente activacin de
linfocitos T y posteriormente mediada por interleucina 2, que al unirse con su receptor especfico causa proliferacin de los linfocitos
activados y posterior depsito de complemento y formacin de perforinas que incrementan la lesin en la membrana basal. Todas
estas alteraciones reducen el calibre de los capilares glomerulares, disminuyendo la superficie de filtracin, lo que lleva a la
consecuente reduccin en la filtracin glomerular. Esta disminucin tiende a ser compensada por el aumento de la presin de
ultrafiltracin que ocurre porque la presin onctica peritubular tiende a disminuir, con disminucin subsecuente de la fraccin
excretada de sodio. Tanto la reabsorcin de sodio como de agua expanden el volumen del lquido extracelular, expresamente el
volumen circulante efectivo, causando hipertensin arterial secundaria. En la exploracin fsica valoraremos la presencia de edemas,
hipertensin arterial o datos de insuciencia cardaca. La ausencia de manifestaciones clnicas sistmicas, el antecedente de infeccin,
la hipocomplementemia y ttulos elevados de anticuerpos antiestreptolisina establecen usualmente el diagnstico de glomerulonefritis
aguda posestreptoccica. La biopsia renal estara indicada si existiera algn dato que hiciera dudar del diagnstico o en caso de
insuciencia renal progresiva. Si se sospecha otro tipo de infecciones o se observan manifestaciones clnicas sistmicas, estara
justicado el estudio serolgico, la realizacin de cultivos y el estudio de autoanticuerpos o inmunoglobulinas. Tratamiento: consiste en
tratar la infeccin, o especcamente el trastorno subyacente si se debe a otra entidad. Las causas ms comunes de sndrome nefrtico:
Enfermedades sistmicas; LES, endocarditis, absceso viscerales, nefritis por cortocircuito, crioglobulinemia, poliarteritis nodosa,
vasculitis por hipersensibilidad, granuloma de Wegener, purpura de Henoch-Schonlein. Enfermedades renales; glomerulonefritis aguda
postinfecciosas, glomerulonefritis membranoproliferativa, nefropata por IgA. El tratamiento general del sndrome nefrtico se basa en
el manejo adecuado del volumen y el control tensional. Para el control del edema se emplean diurticos de asa y restriccin
hidrosalina. En caso de insuciencia cardaca o edema agudo de pulmn, se emplea oxgeno, morna y nitroglicerina. El control
tensional se consigue con bloqueadores del sistema renina-angiotensina y diurticos. En caso de uremia grave, hiperpotasemia y/o
insuciencia cardaca refractaria puede estar indicada la dilisis.


CASO CLINICO


MANUAL DE TRABAJO DEL CURSO ENARM CMN SIGLO XXI
CURSO ENARM CMN SIGLO XXI TEL: 36246001 Pharmed Solutions Institute PGINA 248


TRASTORNOS PRIMARIOS Y RESPUESTAS COMPENSADORAS
TRASTORNO ALT. PRIMARIA RESPUESTA COMPENSADORA
Acidosis metablica [HC
3

] PaCO
2
desciende 1,2mmHg por cada 1
mEq/l de aumento de la [HC
3

]
Alcalosis metablica [HC
3

] PaCO
2
aumenta 0,7mmHg por cada 1
mEq/l de descenso de la [HC
3

]
Acidosis respiratoria PaCO
2
Aguda: [HC
3

] aumenta 1mEq/l por cada


10mmHg de aumento de PaCO
2
Crnica: [HC
3

] aumenta 3,5mEq/l por


cada 10mmHg de aumento de PaCO
2
Alcalosis respiratoria PaCO
2
Aguda: [HC
3

] desciende 2mEq/l por


cada 10mmHg de desciende de PaCO
2
Crnica: [HC
3

] desciende 5mEq/l por


cada 10mmHg de aumento de PaCO
2

Valores normales: sangre arterial pH 7,36-7,44; PaCO
2
36-44mmHg; [HC
3

] 22-26mmHg
Sangre venosa pH 7,31-7,37; PaCO
2
42-50mmHg; [HC
3

] 23-27mmHg
TRASTORNOS ACIDO-BASE AGUDOS
CIENCIAS BASICAS: Cuando se habla de equilibrio acido-base en realidad se hace referencia a la regulacin de la concentracin de
hidrogeniones en los lquidos corporales. Pequeos cambios en la concentracin de iones hidrogeno pueden producir grandes
alteraciones en las reacciones qumicas celulares, aumentando algunas e inhibiendo otras; por este motivo la regulacin de la
concentracin de iones de hidrogeno es uno de los aspectos ms importantes de la homeostasis. El pH sanguneo arterial se mantiene
entre 7.35-7.45, cualquier desviacin de este rango implica desequilibrio acido-base. El pH plasmtico se refiere habitualmente a la
relacin entre las concentraciones de
bicarbonato/cido carbnico. El CO2, en
presencia de anhidrasa carbnica (AC), se
hidrata de la siguiente forma: CO2 +
H2O H2CO3 H+ + HCO3
-
. Al valor de
pH arterial por debajo de 7.35 se denomina
acidemia y cuando est por arriba de 7.45,
alcalemia; la alteracin que cursa con niveles
de pH por debajo de 7.35 se le denomina
acidosis y la que cursa con niveles de pH por
arriba de 7.45 se denomina alcalosis. Factores
de riesgo: Se recomienda tener alto ndice de
sospecha para detectar oportunamente
trastornos del equilibrio acido-base en las
personas con factores de riesgo para
desarrollo. El equilibrio cido-base requiere la
integracin de tres sistemas orgnicos: Hgado, Pulmones, Rin. El hgado metaboliza las protenas produciendo iones hidrgeno (H+),
el pulmn elimina el dixido de carbono (CO2), y el rin generando nuevo bicarbonato (HCO3). Un cido es una sustancia capaz de
donar un H+ y una base una sustancia capaz de aceptarlo. Por tanto, la acidez de una solucin depende de su concentracin de
hidrogeniones [H+]. SISTEMAS DE AMORTIGUAMIENTO: El funcionamiento celular requiere mantener la concentracin de H+, del
lquido extracelular (LEC) en lmites muy estrechos (el pH compatible con la vida esta entorno a 6.80-7.80). Dado que los procesos
metablicos generan gran cantidad de cidos, el organismo necesita neutralizar y eliminar los H+ para mantener constante el pH (-log
[H
+
]) del LEC. Para ello, dispone de varios medios: 1. Amortiguadores plasmticos. Bicarbonato El H
+
se une al HCO3- en forma
reversible, cuando el aporte o la produccin de H+ aumenta, la reaccin se desplaza hacia la derecha, con lo que incrementa la cantidad
de H+ que es captado por el amortiguador, lo que minimiza los cambios de la H
+
. El HCO3- representa el 50% de la capacidad
amortiguadora del plasma. Si la produccin de cidos no voltiles excede la excrecin, el HCO3- disminuye y la H
+
aumenta, resultando
en acidosis metablica, por otra parte si la excrecin de HCO3
-
es mayor que la produccin el HCO3- aumenta y la H
+
disminuye
resultando en alcalosis metablica. H
+
+ HCO3- H2CO3 H2O + CO2- Hemoglobina (Hb), Protenas y fosfatos. Existen otros sistemas de
amortiguamiento como la Hb, protenas y fosfatos, los cuales proveen de sitios adicionales de unin de H
+
y por lo tanto
amortiguamiento. La Hb proporciona el 30% de la capacidad amortiguadora del plasma, el restante 20% lo comparten las protenas y
los fosfatos (13 y 7% respectivamente). 2. Respuesta respiratoria El segundo sistema de amortiguamiento que hace frente a los
trastornos del equilibrio AB es el pulmonar; la disminucin en el pH acta estimulando quimiorreceptores en el tallo cerebral con
incremento en la ventilacin minuto y eliminacin del CO2. 3. Respuesta renal. El rin es el tercer sistema de amortiguamiento, para
mantener el equilibrio AB, stos deben de
excretar aniones de los cidos no voltiles y
reabsorber el HCO3-, esto lo logran por
medio de tres mecanismos: reabsorcin o
excrecin del bicarbonato filtrado, excrecin
de acidez titulable y excrecin de amoniaco.
En definitiva, segn la ley de accin de
masas, la acidosis (aumento de H
+
) puede
producirse por una disminucin del
bicarbonato (acidosis metablica) o por un
aumento de la PaCO2 (acidosis respiratoria);
y la alcalosis (disminucin de los H
+
) por un
aumento del bicarbonato (alcalosis
metablica) o por una disminucin de la
PaCO2 (alcalosis respiratoria). As pues la
alcalosis o la acidosis son estados en los que
existe un acumulo de bases o cidos. Por otra
parte en todo trastorno acidobsico se
producen necesariamente respuestas compensadoras (renales en trastornos respiratorios y respiratorios en trastornos metablicos),
que intentan mantener normal el pH.







MANUAL DE TRABAJO DEL CURSO ENARM CMN SIGLO XXI
CURSO ENARM CMN SIGLO XXI TEL: 36246001 Pharmed Solutions Institute PGINA 249


ANION GAP: La neutralidad qumica del plasma se mantiene por el equilibrio entre aniones (Cl 103; HCO3 24; otros aniones 10;
protenas 17 =154) y cationes (Na 143; K 4.5; Ca 5.0; Mg 1.5 = 154) excretados y retenidos. El nmero total de Cationes plasmticos
debe igualar a los aniones. Los aniones medidos en el laboratorio son inferiores a los cationes medidos. Esta diferencia se denomina
ANION GAP. El anion gap normal es de 12+/- 2. Una AG superior indica la presencia anormal de cidos. AG = cationes suma de
aniones. AG= Na (Cl + HCO3). Segn el valor de anin GAP calculado las acidosis metablicas pueden dividirse en dos grupos: Con
anin GAP elevado (por adicin de cidos fijos). Con anin GAP normal o hiperclormicas (por perdida de Bicarbonato). El aumento en
la concentracin de cloro se debe a que si el bicarbonato desciende, el Cloro aumenta para mantener la electroneutralidad del medio.
Correccin del anin GAP: Segn valor de albmina: por cada g/dl de albmina por encima de 4 se suma al anin GAP calculado 2
puntos y, por cada g/l por debajo de 4 se restan 2 puntos.

CASO CLINICO
Mujer de 81 aos con hipertensin arterial, dislipemia, diabetes tipo 2 y miocardiopata dilatada (fraccin de eyeccin [FE] del 30%).
Tratamiento habitual: telmisartn, metformina a dosis de 850 mg/8 horas, atorvastatina, bezafibrato y omeprazol. Acude a urgencias
por diarrea mucosanguinolenta y vmitos de una semana de evolucin acompaados de oligoanuria en las ltimas 24 horas. TA, 120/70
mmHg, FC de 95 lpm, temperatura 36 C. Glasgow 12, desorientacin temporoespacial y bradipsiquia, sin signos de focalidad.

PREGUNTA
Cual es la prueba mas apropiada para establecer el diagnostico agudo del paciente.

RESPUESTA
a.- Realizar TAC.
b.- Realizar IMR.
c.- QS y BH.
d.- Gases arteriales.

CASO CLINICO
Analtica: hemoglobina 11,7 g/dl, leucocitos 18.030 (78,9% neutrfilos), plaquetas 307.000, glucosa 68 mg/dl, urea 133 mg/dl,
creatinina 6,89 mg/dl, sodio 134 mEq/l, potasio 4,4 mEq/l, pH 6,89, pCO2: 29 mmHg, bicarbonato 6,9 mmol/l, calcio inico 3,85 mg/dl,
anin gap 28. Coagulacin normal. Orina: pH 6, creatinina 71 mg/dl, proteinuria 400 mg/dl, 100 hemates/campo, 60 leucocitos/campo,
cuerpos cetnicos positivos.

PREGUNTA
Cul es la conducta teraputica ms adecuada.

RESPUESTA
a.- Bicarbonato.
b.- Solucion salina.
c.- Solucion glucosada.
d.- Dobutamina.



MANIFESTACIONES CLINICAS
ACIDOSIS METABOLICA
Ej: Ph: 7.32; PCO2: 45; HCO3: 16
(aguda).Ph:7.34; PCO2: 28; HCO3: 18
(subaguda). Ph: 7.36; PCO2: 34; HCO3: 19
(crnica)
ACIDOSIS RESPIRATORIA
Ej.: Ph: 7.24; PaCo2: 55; HCO3:
23 (aguda). pH 7.33; PaCO2: 53;
HCO3: 28 (subaguda). pH: 7.35,
PaCo2: 50, HCO3: 28 (crnica)
ALCALOSIS METABOLICA
Ej: Ph: 7.48; PCO2: 35; HCO3: 26
(aguda). Ph: 7.48; PCO2: 50; HCO3: 28
(subaguda). Ph: 7.45; PCO2: 50; HCO3:
35 (crnica)
ALCALOSIS RESPIRATORIA
Ej: Ph:7.50; PaCO2: 30; HCO3:23
(aguda). Ph: 7.46; PaCo2: 27;
HCO3: 19 (subaguda)

Clnica Causas Clnica Causas Clnica Causas Clnica Causas
Anorexia,
Fatiga,
Deshidratacin,
Confusin,
Letargia,
Estupor, Coma
Taquicardia,
Hipotensin
arterial
sistmica,
Taquipnea,
Disnea, Fatiga
de msculos
respiratorios
Respiracin de
Kussmaul
Anin GAP elevado
(normocloremicas)
Cetoacidosis diabtica
Acidosis lctica
Insuficiencia renal
Intoxicaciones (salicilatos,
etanol, metanol,
formaldehido)
Rabdomiolisis
Anin GAP normal
(hipercloremicas)
Perdidas digestivas de
bicarbonato (diarrea,
fistulas pancreticas,
biliares)
Perdidas renales de
bicarbonato (acidosis
tubulares renales,
hioperparatiroidismo,
frmacos; acetazolamida,
anfotericina, ciclosporina)
Confusin,
Coma,
Convulsiones,
Alteraciones
del ritmo
cardiaco,
Hipotensin
arterial,
Cefalea
EPOC,
neumona,
asma, SDRA
Sedacin
Hipoventilacin
mecnica
Enfermedad
neuromuscular
(Guillan-Barre,
esclerosis
mltiple, crisis
miastenica)
Neumonia
Derrame
pleural masivo
Apata,
Vomito,
Bulimia,
Confusin,
Arritmias
cardiacas,
Hiperreflexia,
Clonus,
Convulsiones,
Hiporreflexia
Perdidas digestivas
(vomitos, aspiracin
nsaogastrica,
diarrea)
Diurticos
Hiperaldosteronismo
1ro y 2ro.
Dieta pobre en sal
Hipercalcemia
Hipopotasemia
Administracion de
bicarbonato sdico
Transfusiones
masivas de sangre
Vrtigo,
Mareo,
Ansiedad,
Euforia,
Alucinaciones,
Alteraciones
del estado de
conciencia,
Mioclonus,
Asterixis,
Taquicardia
Ansiedad
Hipoxia
Hiperventilacin
Dolor
Fiebre
Enfermedad del
tronco cerebral

MANUAL DE TRABAJO DEL CURSO ENARM CMN SIGLO XXI
CURSO ENARM CMN SIGLO XXI TEL: 36246001 Pharmed Solutions Institute PGINA 250

CASO CLINICO
Varn de 75 aos, hipertenso, con enfermedad pulmonar obstructiva crnica. Consult por cuadro de infeccin respiratoria y deterioro
funcional asociado consistente en apata, somnolencia y perodos de agresividad. A la exploracin afebril, desorientado, con tendencia
al sueo, taquipnea superficial, roncus dispersos y crepitantes en la base izquierda, con imagen radiolgica sugerente de
bronquiectasias sobreinfectadas. Cloro (114 mEq/l), con funcin renal y resto de iones normales (sodio: 138 mEq/l; potasio: 4,2 mEq/l),
y una gasometra arterial (pH: 7,24; pCO2: 33 mmHg, pO2: 67 mmHg; bicarbonato: 17 mmol/l, y exceso de bases (EB): 9,1 mmol/l). El
valor del anin GAP (diferencia entre el valor del sodio srico y la suma del cloro y el bicarbonato) fue 7 mEq/l.

PREGUNTA
Cul es el estado clnico acido base del paciente.

RESPUESTA
a.- Acidosis metabolica.
b.- Alcalosis metabolica.
c.- Acidosis respiratoria.
d.- Alcalosis respiratoria.

CASO CLINICO
Paciente de 61 aos de edad ex-fumador, con hipertensin arterial (HTA), diabetes mellitus tipo II, dislipemia, portador de marcapasos
por enfermedad del ndulo sinusal, claudicacin intermitente y con valvulopata artica, estenosis carotdea y cardiopata isqumica
intervenida. Creatinina de 1.56 mg/dl. En la exploracin fsica destaca soplo carotdeo derecho, pulsos poplteos dbiles y ausencia de
pulsos tibiales y pedios como hallazgos patolgicos. BH con (Hb 10.4 mg/dl, VCM 91 fl, HCM 30.6 pg y CHCM 31 gr/dl), y en la
bioqumica plasmtica potasio 3.7 mEq/l. En la gasometra venosa: (ph 7,461) y (CO3H 29.5 mEq/l), con (pCO2 medida 42.3 mmHg, con
PCO2 estimada de 43,85 +/- 2 mmHg).

PREGUNTA
Cual es el manejo ms adecuado para corregir los gases.

RESPUESTA
a.- Disminuir Fi02
b.- Disminuir FR.
c.- Disminuir volumen ventilarotio.
d.- Disminuir volumen residual.

































MANUAL DE TRABAJO DEL CURSO ENARM CMN SIGLO XXI
CURSO ENARM CMN SIGLO XXI TEL: 36246001 Pharmed Solutions Institute PGINA 251

TRASTORNOS ELECTROLITICOS AGUDOS:
CIENCIAS BASICAS: El rin desempea un papel esencial en el equilibrio hidrosalino del organismo porque regula la excrecin de
sodio y del agua. La osmolalidad del sector extracelular es estrictamente una funcin de la natremia. La natremia depende de: el capital
sdico, el capital potsico y el agua total. En situaciones normales los riones
regulan la osmolalidad srica entre 295 a 300 mosm/kg. de agua. La
homeostasis del agua depende de la normalidad del mecanismo de la sed y de
la secrecin de la hormona antidiurtica (ADH) por la hipfisis posterior. Sobre
estos ltimos mecanismos interactan la presin arterial, la osmolalidad
celular y srica, la angiotensina II y las prostaglandinas. La osmolalidad srica
se puede estimar a partir de las siguientes formulas: Osmolalidad srica= 2Na
(mEq/L)+ K(mEq/L)+Urea (mg/dl)/2.8 +Glucosa (mg/dl)/18 o si la urea y la
glucosa son normales se puede utilizar la siguiente formula simplificada:
Osmolalidad= (Na+10) x 2. La osmolalidad srica se expresa en mosm/kg. de
agua. Las principales manifestaciones de un trastorno en el equilibrio del agua
son la hiponatremia y la hipernatremia. HIPONATREMIA: Sodio (Na
+
) <135
mEq/L o de 135 mmol/L y como consecuencia existir disminucin de la
osmolalidad plasmtica. Sus causas pueden ser: hipovolmicas con sodio
corporal total disminuido y se produce prdida de lquidos y sodio extracelular
como en el caso de la fase polirica de la IRA, uso de diurticos, vmitos,
diarreas, sudoracin profusa, fstulas digestivas y quemaduras; hipervolmicas
con sodio corporal total aumentado. Existe exceso de lquidos y de sodio
extracelular como se observa en la insuficiencia cardiaca congestiva, sndrome
nefrtico, insuficiencia renal y cirrosis heptica; endocrinas como en el caso de
la secrecin inapropiada de la hormona antidiurtica por tumores,
traumatismos, SIDA y en la enfermedad de Addison. Clnica: aparece cuando la
natremia es inferior a 120 mEq/L de manera cronica y se caracterizan por
anorexia, nuseas, vmitos, calambres musculares, hipotermia, letargo,
confusin, convulsiones y coma. Con natremias de 128-130mEq/l, sntomas
pueden ser graves si se produce de manera aguda (<48hrs), tienen alto riesgo
de edema cerebral. Laboratorio: electrolitos en sangre y orina, osmolalidad
plasmtica y urinaria, glicemia, protenas y lpidos plasmticos. Se debe
recordar que 100 mg/dl de glicemia superior a lo normal disminuye el sodio
srico en 1.6 mEq/L. Tratamiento, se debe tratar la causa desencadenante de
manera tal que en los estados hipovolmicos se utilizan soluciones fisiolgicas.
Cuando existe retencin hidrosalina y edema se restringen los lquidos y el
sodio y se administran diurticos tipo furosemida y en caso de
sobrehidratacin, restriccin de la ingesta de lquidos. En la prctica para la
correccin de una hiponatremia aguda hipovolmica se debe elevar el sodio
srico de 1 a 2 mEq/l x hora y se calcula el dficit de sodio segn la siguiente
formula: (Na
+
plasmtico deseado-Na plasmtico real) x 0.6 x Kg peso=dficit
de sodio corporal total (en mEq). Se utilizarn soluciones salinas isotnicas al 0.9% que contienen 154 mEq/L. Tambin en la prctica
para tratar una hiponatremia severa en un paciente con insuficiencia renal y realizarla a una velocidad de 0.5 mEq/L/hora de elevacin
de la natremia ha sido utilizada la hemodilisis continua venovenosa con un lquido de dilisis con un contenido de sodio menor que lo
normal. La hiponatremia crnica puede ocasionar mielinolisis pontina si se corrige rpidamente. En las formas agudas puede elevarse
el Na
+
hasta 2mEq/l, mientras en las formas crnicas no debe superar los 1-1.5mEq/l. El uso de furosemida favorece la eliminacin de
agua libre, siendo especialmente til en estados edematosos o si la osmolalidad en orina es >400mOsm/kg. En el sndrome de secrecin
inadecuada de vasopresina (SIADH): el tratamiento crnico se basa en la restriccin de lquidos debe recordarse que: 1) la restriccin de
lquidos afecta a todos los lquidos no solo al agua. 2) el grado de restriccin depende de la diuresis ms las perdidas insensibles. 3)
suelen ser varios das antes de que se modifique la osm. 4) no debe restringirse el Na+, frmacos solo para casos refractarios,
antagonista V2 (tolvaptan). HIPERNATREMIA: Sodio (Na
+
)>145mmol/l, y por tanto existir un aumento de la osmolalidad plasmtica
superior a 300 mosm/L. Menos frecuente que hiponatremia, ms comn en nios y adultos mayores. Puede ser resultado de perdida
de agua (frecuente) o de un aporte de sodio (raro). La prdida de agua desencadena dos mecanismos para evitar la hipernatremia: la
sed y la liberacin de vasopresina (ADH). Siempre hay que tener en mente la sed es primordial, ya que incluso la mxima secrecin de
ADH puede no lograra retener agua suficiente para compensar las prdidas si no se aumenta el aporte de agua. La hipernatremia con
hipervolemia es poco frecuente y se produce por una ganancia neta de sodio. Es mucho ms frecuente encontrar situaciones de normo
o hipovolemia. Sus causas pueden ser: euvolmicas con Na
+
normal y se observa en los estados febriles severos, hiperventilacin
pulmonar; hipovolmica con Na disminuido se observa en la deshidratacin por sudoracin profusa, uso de manitol y soluciones
glucosadas, diarreas, quemaduras, diabetes inspida(central o nefrognica su diferencial mediante test de deprivacin de agua,
midiendo la ADH plasmtica); hipervolmica con Na aumentado la cual se observa cuando se utilizan en forma indiscriminada
soluciones de bicarbonato de sodio, cuando se utilizan la carbenicilina y los esteroides y en el coma hiperosmolar no cetognico.
Clnica: sed, astenia, debilidad muscular, somnolencia, irritabilidad, letargia, confusin, temblor, convulsiones y coma. Laboratorio:
sodio srico y osmolalidad plasmtica. El tratamiento consiste en administrar soluciones glucosadas al 5% en los pacientes
deshidratados con sodio normal. En los pacientes con volumen intravascular aumentado se deben utilizar los diurticos ms soluciones
glucosadas al 5%. En la diabetes inspida central se utilizar la vasopresina exgena y en nefrognica tiazidas. En forma prctica, para
calcular la cantidad de lquido a administrar se utiliza la siguiente formula: Agua corporal total (ACT) real = peso corporal (Kg) x 0.6.
Na
+
plasmtico real/Na
+
plasmtico deseado X ACT real = ACT deseada. ACT deseada- ACT real = dficit de lquido. Es prudente
TIPOS DE SOLUCIONES INTRAVENOSAS
SOLUCION CONCEPTO/ACCIONES
ISOTNICOS
Suero
fisiolgico al
0.9%
Na: 154mEq/l
Cl: 154mEq/l
308 mOm/l
Expansor de volumen extracelular para la
hipovolemia
Un exceso puede provocar hipervolmia y
acidosis por exceso de cloro
Solo aporta iones cloro y sodio
Se puede usar en hiponatremia
nico que se puede usar conjuntamente con
sangre y derivados
Suero
glucosado al
5%
278 mOm/l
Igual que SF al 0.9%, pero la mezcla de
dextrosa en sangre provoca hipotonicidad
Aporta muchas caloras (170/l)
Ringer-lactato
(Hartmann)
Contienen mltiples electrolitos y en
concentraciones similares a la plasmtica
Para tratar hipovolemia
El lactato se metaboliza rpido y se convierte
en bicarbonato (riesgo de alcalosis)
Ringer-lactato
glucosado al
5%
Aporta 170 caloras por litro y agua libre
No se debe utilizar para reponer volumen,
pues diluye el plasma y altera concentraciones
HIPOTONICOS
Suero
fisiolgico al
0,45%
Na: 76.5mEq/l
Cl: 76.5mEq/l
153 mOm/l
Se emplea para potenciar la eliminacin de
solutos por los riones debido a su cantidad
de agua libre y para reponer el lquido
extracelular
Tratamiento de la hipernatremia y de otros
estados hiperosmolares
Solo aporta cloro y sodio
Si se emplea la formula con dextrosa 5%, se
puede provocar hipertonicidad en relacin con
el plasma
HIPERTONICOS
Suero
fisiolgico al
3%
Na: 513mEq/l
Cl: 513mEq/l
1,026 mOm/l
Para tratar hiponatremias en situaciones
graves
Precaucin por su contenido alto en sodio y
cloro, que pueden dar lugar a exceso de
volumen intravascular y edema de pulmn
Suero
fisiolgico al
5%
Igual que el anterior pero ms concentracin
de sodio y cloro
MANUAL DE TRABAJO DEL CURSO ENARM CMN SIGLO XXI
CURSO ENARM CMN SIGLO XXI TEL: 36246001 Pharmed Solutions Institute PGINA 252

administrar el 30% de la solucin calculada en las primeras 24 horas. La correccin rpida de hipernatremia puede producir edema
cerebral, convulsiones, dao neurolgico permanente e incluso la muerte. Para disminuir riesgo se aconseja corregir Na
+
lentamente.
En casos graves (>170mEq/l), no corregir a menos de 140mEq/l en primeras 48-72hrs y en formas crnicas no debe corregirse mas de 8-
12 mEq/l. HIPOPOTASEMIA: Potasio (K
+
) <3.5 mEq/L. La hipokalemia es leve cuando el K
+
se encuentra entre 3.5 a 2.8 mEq/L, moderada
de 2.8 mEq/L y severa menor de 2.8 mEq/litro. Causa son: vmitos, diarreas, fstulas gastrointestinales, uso de diurticos, diuresis
osmtica, fase polirica de la IRA, uso de esteroides, hiperaldosteronismo primario o secundario, acidosis tubular renal, alcalosis.
Clnica: decaimiento, letargia, predisposicin a la intoxicacin digitlica, anorexia, nuseas, vmitos, distensin abdominal, leo
paraltico, paresias, hiporeflexia osteotendinosa, rabdiomilosis la cual puede originar IRA. Se deben solicitar el potasio srico y urinario
as como los gases arteriales. Tratamiento correccin de la hipokalemia, cuando el K
+
srico es superior a 2.8 mEq/L se utilizar la va
oral sobre la base de alimentos ricos en potasio sobre todo frutas ctricas, de manera tal que la ingesta diaria sea de unos 40 a 120 mEq.
S el K
+
srico es inferior a 2.8 mEq/L se utilizar la va IV a razn de 40 mEq de KCL por litro en solucin fisiolgica. Es prudente un
tratamiento preventivo de la hipopotasemia en pacientes que reciban diurticos, esteroides, digitlicos, as como en la fase polirica de
la IRA y en pacientes que reciban tratamiento para una cetoacidosis diabtica. HIPERPOTASEMIA: Potasio (K+) >5.5 mEq/L y constituye
una verdadera emergencia mdica ya que el paciente corre el riesgo de morir por paro cardiaco. La hiperkalemia puede ser leve hasta
6.5 mEq/L, moderada hasta 7.5 mEq/L y severa superior a 7.5 mEq/L. Las causas ms frecuentes dehiperkalemia son: insuficiencia renal
aguda y crnica, uso de diurticos ahorradores de potasio, insuficiencia suprarrenal aguda o crnica (enfermedad de Addison),
hipercatabolismo, acidosis metablica y la pseudohiperkalemia por muestra de sangre hemolizada, trombocitosis y leucocitosis por
encima de 100.000 mm3. Se deben solicitar potasio srico y electrocardiograma, ya que ste es fundamental para evaluar la
hiperkalemia. Cuando es leve al ECG se observan ondas T altas picudas y simtricas, cuando es moderada se observa ensanchamiento
del complejo QRS, disminucin de la amplitud de la onda P y prolongacin del intervalo PR y cuando es grave desaparicin de la onda P,
bradicardia, extrasistoles ventriculares, fibrilacin ventricular y asistolia. Tratamiento, adems del preventivo (evitar frutas ctricas,
evitar uso de diurticos ahorradores de potasio, no administrar sangre almacenada por largo tiempo). Moderada: se utilizan resinas de
intercambio catinico (poliestirensulfonato clcico) que eliminan el potasio del tubo digestivo. Diurticos. Grave: glucosa intravenosa
junto con insulina de accin rpida (10UI de insulina en 500ml de glucosa al 10%). La insulina favorece la entrada de potasio a la clula y
la glucosa previene la aparicin de hipoglucemia. Administracin de bicarbonato de sodio va intravenosa para corregir la acidosis.
Furosemida: 40 mg IV a repetir cada 4 horas o bumetanida 1 mg IV cada 4 hrs. Gluconato de calcio: 10 ml al 10% IV en 20 minutos.
Bicarbonato de sodio: 60 ml IV rpidamente cada 8 horas. Solucin glucosada al 10% 250 ml ms 10 unidades de insulina cristalina IV.
Sulfonato de poliestireno sdico (Kayexalate) intercambia sodio por potasio a nivel intestinal en enema a retener 50 a 100 gramos en
100 a 200 ml de agua o por va oral 20 a 40 gramos ms 20 ml de sorbitol al 50%. De ser necesario y en medio especializado
hemodilisis o dilisis peritoneal. HIPOCALCEMIA: Calcio (Ca
2+
) <8,5mg/dl. Causas: Hipoparatiroidismo: una de las causas ms
frecuentes de hipocalcemia crnica, dficit de vitamina D, hipomagnesemia (en esta situacin se suprime la secrecin de PTH),
insuficiencia renal, hipoalbuminemia (calcio circulante se halla unido a la albumina), pancreatitis (aumenta la lipolisis, los acidos grasos
se unen a calcio y disminuye), politransfusiones sanguneas (elcitrato utilzado como anticoagulante se une al calcio), alcalosis, abuso de
laxantes o sndromes de malaabsorcion. Clinica: hiperreflexia, calambres musculares, la tetania por irritabilidad neuromuscular, es el
signo clnico fundamental. La tetania se puede poner de manifiesto mediante dos maniobras: Signos de Chevostec (estimular nervio
facial; contraccin de musculos faciales), signo deTrousseau (se infla manguito de presin por encima de presin arterial sistlica y se
produce un espasmo carpal. Adormecimiento alrededor de la boca, hormigueo, espasmo farngeo, convulsiones, psicosis, demencia.
Tratamiento, tratar la causa, en situaciones agudas, se administra gluconato de calcio de forma intravenosa. Cuando hipocalcemia es
crnica aporte de vitamina D y suplementos de calcio oral. HIPOMAGNESEMIA: Es una situacin muy frecuente en los pacientes
gravemente enfermos, de sintomatologa vaga e inespecfica y frecuentemente se asocia a la hipocalcemia. Los niveles sricos de
magnesio normales son de 1.4 a 2 mEq/l. Puede observarse cuando se utilizan diurticos, en los alcohlicos crnicos, en la pancreatitis
aguda, en la hipomagnesemia familiar con hipercalciuria, intoxicacin digitlica, IAM, insuficiencia cardiaca. Clnica: disfagia, diplopa,
debilidad, psicosis, depresin, nistagmo, signos de Chevostek y Trousseau, hiperreflexia osteotendinosa y convulsiones. El tratamiento
consiste en administrar magnesio a la dosis de 2 gramos de sulfato de magnesio diluidos en 100 ml de solucin glucosada al 5% IV para
pasar en 5 a 10 minutos y se puede repetir cada 8 horas por 5 das. HIPERMAGNESEMIA: Se acompaa de arreflexia, parlisis
respiratoria y paro cardiaco se trata con calcio IV 100 a 200 mg en 20 minutos.


CASO CLINICO

















MANUAL DE TRABAJO DEL CURSO ENARM CMN SIGLO XXI
CURSO ENARM CMN SIGLO XXI TEL: 36246001 Pharmed Solutions Institute PGINA 253

HIPOGLUCEMIA, HIPERGLUCEMIA:
HIPOGLUCEMIA: Niveles de glucemia < 50 mg/dL reversible con la administracin de glucosa. Se trata de una urgencia muy peligrosa ya
que puede ocasionar lesiones irreversibles e incluso la muerte por falta de aporte de glucosa a nivel de SNC. La hipoglucemia es la
complicacin aguda ms frecuente del tratamiento de la DM. Ms del 35% de pacientes con DM tipo 1 experimentan durante la
evolucin de la enfermedad, al menos, un episodio de hipoglucemia grave que requiere atencin por otra persona. La incidencia de
hipoglucemia grave en los pacientes con DM tipo 2 es de 1 a 3 por 100 pacientes por ao. Esta frecuencia se triplica en pacientes
mayores de 75 aos, con ingresos hospitalarios recientes y que reciben mltiples frmacos. Si bien la incidencia de hipoglucemia es
inferior en pacientes con DM tipo 2 que en los de tipo 1. Ante una situacin de disminucin de las concentraciones de glucosa en
sangre se produce una serie de acontecimientos tendentes a normalizar la glucemia. En sujetos diabticos que sufren episodios
repetidos de hipoglucemia se ha observado que el umbral de respuesta secretora de catecolaminas, as como de la aparicin de
sntomas autnomos, desciende de forma importante. Estos pacientes no presentan sntomas neuroglicopnicos hasta que sus valores
de glucemia descienden a valores en torno a 40 mg/dl. Es decir, estos pacientes soportan bien valores bastante bajos de glucemia
(mecanismo de adaptacin cerebral), lo que resulta perjudicial, ya que le introduce en un crculo vicioso de hipoglucemias no
reconocidas. ETIOLOGA: La causa ms frecuente de hipoglucemia por exceso de insulina es la producida por antidiabticos orales (ms
frecuente las sulfonilureas) e insulina exgena, seguida de la discutible hipoglucemia reactiva y los casos de hiperinsulinismo endgeno
por tumores (insulinoma y tumores extrapancreticos productores de factores insulin-like). Pacientes diabticos: Dosis excesivas de
insulina o sulfonilureas. Dieta inadecuada (horarios alterados, baja ingesta,...). Exceso de ejercicio fsico. Enfermedades que disminuyen
las necesidades de insulina: Insuficiencia renal (aumenta la vida media de los hipoglucemiantes), heptica, dficits hormonales.
Interacciones medicamentosas (salicilatos, clofibratos, fenilbutazona, sulfinpirazona). Estrs. Pacientes no diabticos: Hiperinsulinismo
endgeno (insulinoma, frmacos). Comidas poco frecuentes. Hipoglucemia postprandial postciruga gstrica. Abuso de alcohol. Una
causa inusual, pero con una incidencia cada vez mayor, es la producida por fenmenos autoinmunes contra la insulina, su receptor o la
propia clula beta, conocido como sndrome autoinmune por insulina (tiroideas como tiroiditis de Hasimoto y enfermedad de Graves,
etc). Otras causas: La sintomatologa propia de la hipoglucemia reactiva se presenta en el perodo postprandial, especialmente si se ha
efectuado una ingesta de hidratos de carbono de absorcin rpida. El insulinoma es un raro tumor neuroendocrino con una incidencia
aproximada de 1 a 2 por cada milln de habitantes/ao. Tpicamente se presenta como un tumor solitario, pero puede aparecer de
forma multifocal en el contexto de una neoplasia endocrina mltiple tipo 1 o como un tumor metastsico en el insulinoma maligno. En
el diagnstico de un insulinoma, la confirmacin bioqumica de hiperinsulinismo debe preceder a cualquier tcnica de localizacin. La
sospecha clnica es importante para su diagnstico, y as, es fundamental la existencia de la triada de Whipple: la clnica sugestiva de
hipoglucemia debe asociarse con unos niveles de glucosa bajos en sangre y mejorar con
la administracin de glucosa. Con la sospecha clnica de insulinoma la prueba
diagnstica de eleccin es un ayuno de 72 horas. Esta prueba tiene que realizarse bajo
supervisin mdica y por tanto ser necesario hospitalizar al enfermo. Por ltimo, ante
una hipoglucemia inexplicable, debemos pensar en la posibilidad del diagnstico de
hipoglucemia facticia para realizar las pruebas de confirmacin pertinentes y as evitar
otras que pudieran resultar innecesarias. DIAGNOSTICO: Sntomas adrenrgicos:
Sudoracin, palpitaciones, ansiedad, temblor, hambre, predominan cuando la glucemia
desciende rpidamente pero pueden enmascararse con la toma de B-bloqueantes o si
existen neuropatas. Sntomas neuroglucopnicos: trastornos de conducta, agresividad,
confusin focalidad neurolgica, somnolencia, convulsiones, coma. Exploracin fsica:
TA, T, FC, FR, SAT O2, glucemia capilar, hidratacin o perfusin cutnea, fetor etlico,
nivel de conciencia, focalidad neurolgica o crisis convulsivas. Glucemia capilar , BH,
QS, electrolitos, osmolaridad, coagulacin, EGO, ECG: podran producirse por el efecto
directo de la hipoglucemia, el aumento en la secrecin de adrenalina, la hipopotasemia
secundaria o la disfuncin autonmica, todo ello sobre un posible sustrato de
cardiopata: alteraciones de la repolarizacin, como depresin del segmento ST,
aplanamiento e inversin de la onda T y prolongacin del intervalo QT, trastornos del ritmo, como taquicardia y bradicardia sinusal,
bloqueo auriculoventricular. TRATAMIENTO: LEVE: nivel de conciencia conservado y tolera va oral se administrarn lquidos azucarados
con 20gr de glucosa. MODERADO: disminucin leve del nivel de conciencia o no tolera la va oral: glucagn 1mg (1amp) IM o SC (pauta
habitual en domicilio) o seguir pauta siguiente. En un estudio se recomiendan 20 gramos de D-glucosa como lo ms eficaz, ya que
corrigen la hipoglucemia de grado moderado a grave en 20 minutos, sin causar hiperglucemia prolongada. A los adultos diabticos en
estado de coma u otros pacientes que tienen coma de causa imprecisa se les debe administrar 50 ml de una solucin de glucosa al 50 %
en bolo intravenoso, despus de obtener sangre para estudios apropiados. En caso de confirmarse la hipoglucemia, se debe iniciar
venoclisis con solucin de glucosa al 5, al 10 o al 20 %. La glucosa intravenosa continua por 4 a 6 horas es necesaria para la mayor parte
de las reacciones hipoglucmicas. GRAVE: disminucin del nivel de conciencia severo, coma o PCR: Va venosa perifrica y administrar
glucosa hipertnica (glucosmon*) 30ml al 33% (3 amp) y sueros glucosados (SG) al 10%, si no posible canalizar va: glucagn 1mg
(1amp) IM o SC, si tras 1000ml de SG continua con cifras bajas de glucemia administraremos hidrocortisona 100mg y 1mg de glucagn a
cada litro de solucin, adrenalina 1mg diluido 1/1000 SC, protocolo de RCP si precisa. En pacientes alcohlicos administrar previamente
tiamina 100mg, IM. Mantenimiento: Control horario de glucemia capilar hasta cifras 120mg/dl y a partir de entonces cada 4-6 horas
durante 24 horas. Continuar con SG 5-10% hasta tolerancia oral. HIPERGLUCEMIA: Se define como el nivel srico superior a 140mg/dl.
La liberacin de glucgeno por parte del hgado y los msculos, combinado con la glucognesis y el metabolismo de los cidos grasos
libres, aumentara el nivel de glucosa en sangre. El que este aumento alcance el nivel de hiperglucemia depender de la capacidad del
organismo para utilizar la glucosa a nivel celular. Hiperglucemia grave, es la elevacin de la glucemia plasmtica casi siempre superior a
250mg/dl, que se asocia con alteracin de otros rganos en el organismo. Las dos principales causas de deterioro en la utilizacin de
glucosa son los medicamentos y la diabetes. Tambin puede producir hiperglucemia cualquier enfermedad que aumente el estrs
corporal. El estrs aumenta la produccin de citosinas y hormonas contrarreguladoras de insulina (catecolaminas, cortisol, glucagn y
hormonas del crecimiento) que alteran el metabolismo de los carbohidratos, incluidas la resistencia a la insulina, la gluconeognesis,
MANUAL DE TRABAJO DEL CURSO ENARM CMN SIGLO XXI
CURSO ENARM CMN SIGLO XXI TEL: 36246001 Pharmed Solutions Institute PGINA 254

glucogenolisis y la disminucin de la secrecin de la misma debida al mal funcionamiento de la clulas beta del pancretica. Otros
factores que participan en la elevacin de glucosa son: la administracin exgena de glucocorticoides a dosis elevada, el tratamiento
subptimo del control glucmico por falta de estrategias o algoritmos de manejo (pobre adherencia 20-40%) y la nutricin enteral o
parenteral, infecciones (20-50%), embarazo, entre otros. La hiperglucemia se ha asociado con aumento de la respuesta proinflamatoria,
una funcin alterada del sistema inmunitario, disfuncin en la quimiotaxis de neutrfilos con la consecuente alteracin en la fagocitosis,
disfuncin endotelial, estado protrombtico, dao neuronal asociado con la isquemia cerebral y con aumento del estrs oxidativo. Los
pacientes diabticos pueden presentar hiperglucemia si no se aplican su insulina o no toman hipoglucemiantes orales, se alteran las
restricciones dietticas o estn sometidos algn tipo de estrs ya sea fisco o emocional. Clnica: polidipsia, polifagia, poliuria,
frecuentemente acompaadas de prdida de peso y fatiga. La hiperglucemia es un problema que se manifiesta en un porcentaje
importante de pacientes hospitalizados y constituye un factor de riesgo para IAM, infarto cerebral, sepsis, infecciones nosocomiales,
insuficiencia cardiaca y renal. Adems, complica una gran variedad de enfermedades, aumenta el tiempo de estancia hospitalaria y es
un factor de riesgo independiente de complicaciones que producen una importante morbilidad y mortalidad en los servicios
hospitalarios, sin mencionar su gran impacto econmico. La administracin de insulina puede prevenir varios de los efectos adversos
asociados a hiperglucemia. La insulina puede suprimir la generacin de especies reactivas de oxgeno, inducir vasodilatacin, inhibir la
lipolisis, reducir los cidos grasos libres, inhibir la agregacin plaquetaria y disminuir la respuesta inflamatoria. Por lo tanto la insulina
juega un papel primordial en la prevencin de los riesgos asociados con la hiperglucemia. Si la hiperglucemia no se trata aparecer:
cetoacidosis. En la valoracin posterior hay que controlar los medicamentos que puedan causar hiperglicemia: corticoides,
anticonceptivos, antihipertensivos (furosemida, tiacidas, cido etacrinico).

CASO CLINICO
Mujer de 36 aos con diabetes de larga evolucin con polineuropata, retinopata y nefropata. Padeci adems un sndrome de
Cushing por microadenoma hipofisario que requiri ciruga, una infeccin por VHC crnica e HTA, para la que haba recibido
tratamiento con amlodipino 10mg/24h. Intolerancia al ortostatismo y sncopes, que se agrav en las dos ltimas semanas, por lo que la
paciente haba estado confinada en la cama. A la exploracin se observo desorientacin, agitacin, desviacin de la comisura labial
acompaada de ptosis parpebral unilateral, ausencia de reflejos osteotendinosos en las extremidades inferiores y perdida de control de
impulsos.

PREGUNTA
Cual es la conducta a seguir mas apropiada.

RESPUESTA
a.- Glucosa al 50 %.
b.- Ringer latato.
c.- Solucion harman.
d.- Solucion mixta.

CASO CLINICO
Mujer de 46 aos con hipertensin arterial, diabetes mellitus insulinodependiente con insuficiencia renal crnica en dilisis peritoneal
que ingresa en planta de medicina interna con el diagnstico de cetoacidosis diabtica. Se inicia tratamiento y el segundo da se reinicia
la dilisis peritoneal con su pauta habitual. El cuarto da presenta disminucin del nivel de conciencia, aumento de presin arterial,
sudoracin profusa y al final una parada respiratoria por lo que es intubada. La glucemia capilar es de 122mg/dl y la gasometra arterial
muestra una glucemia de 20mg/dl. Se administra glucosa intravenosa. Al da siguiente se decide reducir la sedacin, presentando una
hora ms tarde convulsiones clnicas, desviacin de la mirada y taquipnea. La glucemia capilar en ese momento es de 59mg/dl.

PREGUNTA
Cual es la conducta mas adecuada a seguir para establecer el diagnostico de esta complicacin?

RESPUESTA
a.- Realizar EEG.
b.- Realizar PEV.
c.- Realizar TAC.
d.- Realizar IRM

PREGUNTA
Entre otros estudios se realiz un EEG en donde se observa signos de sufrimiento cerebral difuso de grado muy severo y una TC craneal
que descarta afeccin intracraneal aguda. Se decide no retirar el lquido de dilisis peritoneal e iniciar dilisis convencional. La escala de
Glasgow fue de 4, Cual es el diagnostico mas probable en este momento?

RESPUESTA
a.- Muerte cerebral.
b.- Encefalopatia hipoglucemica.
c.- Acidente vascular cerebral.
d.- Encefalopatia hipoxico isqumica.



MANUAL DE TRABAJO DEL CURSO ENARM CMN SIGLO XXI
CURSO ENARM CMN SIGLO XXI TEL: 36246001 Pharmed Solutions Institute PGINA 255

ESTADO HIPEROSMOLAR HIPERGLUICEMICO (EHH), CETOACIDOSIS DIABETICA (CAD):
CIENCIAS BASICAS: Las complicaciones hiperglucemicas son causantes de un gran nmero de hospitalizaciones en diabticos,
presentndose entre 4 y 8 por 1000 pacientes con diabetes, con una mortalidad que vara entre 4 y 20%. De su adecuado diagnstico y
tratamiento depender una considerable disminucin de las hospitalizaciones y das de estancia en esta poblacin. PATOGENIA: Tanto
la CAD como EHH, son el resultado de la combinacin del dficit absoluto o relativo de insulina y el aumento de las hormonas
contrareguladoras. Este estado lleva a un aumento de la produccin heptica de glucosa y disminucin en el consumo perifrico,
produciendo un aumento exagerado de la glucosa sangunea; esta a su vez induce glucosuria, diuresis osmtica y deshidratacin. En los
pacientes con DM tipo I el dficit severo de insulina y el aumento en las catecolaminas, cortisol y hormona del crecimiento estimulan la
lipasa sensible a hormonas, aumentando la produccin de cidos libres derivados de triglicridos, los cuales son metabolizados en el
hgado y convertidos en cuerpos cetnicos, proceso estimulado por el aumento de glucagn y el dficit de insulina que activa la enzima
carnitina palmitoil transferasa I que permite la entrada de los cidos grasos libres a la mitocondria donde se convierten en cido -
hidroxibutirico, acetona y cido actico. Estos cuerpos cetnicos son los responsables de la acidosis en los pacientes con DM1. El -
hidroxibutirico y el acetoactico se disocian, produciendo exceso de iones de
hidrogeno que consumen el bicarbonato, disminuyendo sus niveles sricos. A
medida que se van acumulando los cuerpos cetnicos, el pH sanguneo
desciende. Cuando el pH desciende hasta cerca de 7.2, el centro respiratorio
eleva frecuencia e intensidad de las respiraciones (respiracin de Kussmaul) en
el intento de eliminar con las reparaciones el exceso de cido carbnico. La
acetona un componente de los cuerpos cetnicos, tambin se elimina a travs
de la respiracin, lo que origina un olor caracterstico afrutado en el aliento del
paciente. Los riones a su vez tambin eliminan el exceso de cuerpos cetnicos,
dando lugar a la cetonuria. Como la cetoacidosis diabtica altera la
homeostasia, el organismo intenta restablecer el orden mediante una serie de
mecanismos compensatorios. Para compensar la diuresis osmtica, el centro
de la sed del cerebro estimula al paciente para que beba ms. Los riones
comienzan a segregar renina, ponindose en marcha el sistema renina
angiotensina, aldosterona, este complejo proceso hace que los riones
reabsorban sodio y agua. Tambin la hipfisis posterior segrega hormona
antidiurtica que ayuda tambin a conservar el agua y el sodio. La diuresis
osmtica deriva de la hiperglucemia lleva a un dficit severo de lquidos que
puede estar entre 5 y 7 litros. Adems se produce tambin dficit de cloruro de
sodio entre 3-10mmol/kg, los niveles de sodio pueden estar falsamente
alterados por la hiperglucemia
presente. El potasio se encuentra
tambin severamente disminuidos,
sin embargo los niveles de potasio en
suero pueden estar normales o
incluso elevados durante el episodio,
debido a la acidosis y a la
hiperglucemia presente,
disminuyendo severamente el
potasio intracelular. Otros elementos
que pueden estar alterados son el
fosfato, el magnesio y el calcio.
CAUSAS: Diagnostico de novo,
infecciones, enfermedades
intercurrentes, falla en la aplicacin
de insulina o en la toma de
medicamentos orales (corticoides),
excesos alimentarios, ciruga,
traumatismo, desconocida. En los
diabticos diagnosticados, su causa
desencadenante suele ser una
situacin estresante que incrementa
las necesidades de insulina, aunque
tambin puede obedecer a una
descompensacin de la enfermedad
por no haber seguido correctamente el tratamiento prescrito CLINICA Y DIAGNOSTICO: En cuadro. Iniciar con Bh completa y gases
arteriales, EGO, glucosa plasmtica, BUN, electrolitos, cretinina, electrocardiograma. TRATAMIENTO: El objetivo principal e inicial es la
correccin del dficit hdrico para expandir el volumen intra y extracelular y asegura una adecuada perfusin renal. Si no existen
trastornos cardiacos se inicia con solucin salina al 0.9% normal; si el sodio es mayor de 155meq/l se recomienda utilizar solucin salina
al 0,45 normal. COMPLICACIONES: La aparicin de edema cerebral es raro afortunadamente, mltiples factores influyen en su origen,
incluyendo la aparicin de idiosmoles que causan un gradiente y una desviacin del agua hacia las clulas, la terapia con insulina por si
sola aumenta la entrada de sustancias osmticamente activas en el espacio intracelular y un aumento rpido del dficit de sodio.
Sndrome de dificultad respiratoria del adulto; esta complicacin puede producirse por el aumento en el volumen de agua en los
DIAGNOSTICO DIFERENCIAL ENTRE CETOACIDOSIS DIABETICA
(CAD) Y ESTADO HIPEROSMOLAR HIPERGLUCEMICO (EHH)
PRESENTACION CLINICA CAD EHH
Dolor abdominal
Hiperventilacion
Vomito
Deshidratacin
Signo de Kussmaul
Alteracin del edo. de alerta
Aliento cetonico
Poliuria
Polidipsia
Fiebre
Prdida de peso
Taquicardia
+++
+
+++
++
+++
+
++
+
+
+
+
+
-
-
-
+++
-
+++
-
+++
+
+
+
-
DIAGNOSTICO
Glucemia
Cetonuria
Bicarbonato srico
pH
Brecha aninica Na- (Cl+HCO3)
Osmolaridad
2Na+(glu/18)+(BUN/2.8)
>250mg/dl
+++
<15
<7.30
>12
>600mg/dl
+
>15
>7.30
<12
>320
MANUAL DE TRABAJO DEL CURSO ENARM CMN SIGLO XXI
CURSO ENARM CMN SIGLO XXI TEL: 36246001 Pharmed Solutions Institute PGINA 256

pulmones y una disminucin en la adaptabilidad pulmonar. Acidosis metablica hipercloremica; el mecanismo principal es la perdida de
cetocidos por la orina, los cuales son requeridos para la generacin de bicarbonato





















CASO CLINICO
Una mujer de 76 aos que no se conoca diabtica ingres por desihidratacion grave, refieren los familiares que vive sola y desconocen
su tratamiento, refieren que la paciente presentaba desorientacin, letargia, falta de respuesta a estimulos, se ingresa a la paciente con
datos de desihidratacion glucemia de 596mg/dl pH 7.29, glucosuria y cetonuria, con osmolalidad en suero calculada de 318 mOsm/l.

PREGUNTA
Cual de las siguientes condiciones incrementan el riesgo de cetoacidosis.

RESPUESTA
a.- Infeccion de vas urinarias.
b.- Desequilibrio hidroelectrolitico.
c.- Osmolaridad alterada.
d.- Dieta cetogenica.

CASO CLINICO
Una mujer de 76 aos que no se conoca diabtica ingres por una descompensacin hiperglucemica cetosica. Durante el ingreso sufri
4 crisis parciales motoras con generalizacin secundaria, de inicio motor hemicorporal izquierdo y generalizacin tnico-clnica, con
una duracin de entre 1 y 3 minutos, con recuperacin del nivel de conciencia entre las crisis y en un periodo de 5h, seguidas de un
intenso dficit poscrtico hemisfrico derecho, con hemiparesia, hemihipoestesia, hemianopsia y heminegligencia izquierdas,
recuperndose en las siguientes 24h. La analtica mostr una glucemia de 596mg/dl pH 7.30, glucosuria y cetonuria, con osmolalidad en
suero calculada de 318 mOsm/l. Se inici tratamiento con anticonvulsivante), sin recurrencia de las crisis. Se le realiz una RM craneal
18 horas despus de la primera crisis, donde se apreci una hipointensidad en sustancia blanca subcortical parietal derecha en T2 con
ligera hiperintensidad cortical en FLAIR, tenue captacin giriforme de contraste y leve restriccin de la difusin en dicha localizacin. El
electroencefalograma (EEG) evidenci un foco de ondas delta frontotemporal derecho.

PREGUNTA
Considerando las complicaciones que se presentan en esta patologa cual es su pronostico mas probable?

RESPUESTA
a.- Bueno ya que se resolvi adecuadamente.
b.- Moderado posibles secuelas.
c.- Puede continuar con crisis convulsivas.
d.- Para evitar recurrencias es conveniente dejar anticonvulsivo.

CASO CLINICO
Mujer de 42 aos con antecedentes de tiroiditis crnica de Hashimoto, 3 partos normales, sin macrosoma y una enfermedad mixta del
tejido conjuntivo (EMTC), ante la aparicin de una poliartritis de pequeas articulaciones, esclerodactilia, fenmeno de Raynaud,
miopata, anticuerpos antinucleares (+) 1/5120 y anticuerpos antiENA (++++). Una ta era diabtica tipo 2. Usaba levotiroxina 50 g/da,
prednisona 7,5 mg diarios y MTX 25 mg intramusculares semanales. Hace tres meses, se espaci la dosis de MTX a 25 mg cada dos
semanas, apareciendo debilidad muscular progresiva, mialgias y posteriormente sed y poliuria. La glicemia fue 286 mg/dl (glicemias
MANUAL DE TRABAJO DEL CURSO ENARM CMN SIGLO XXI
CURSO ENARM CMN SIGLO XXI TEL: 36246001 Pharmed Solutions Institute PGINA 257

previas siempre normales), prescribindose dieta y glibenclamida 5 mg/da. Tres semanas despus consult en un Servicio de Urgencia
por sed intensa. La glicemia fue 550 mg/dl con cetonemia (-). Se aument la glibenclamida a 10 mg/da e inici metformina 850 mg y
antiinflamatorios. Una semana despus, consult nuevamente por vmitos, sed, poliuria, mialgias, artralgias y compromiso importante
del estado general. Fue hospitalizada, destacando deshidratacin marcada, normotensin y polipnea. IMC: 27,3 kg/mt2. La piel de la
cara estaba acartonada, enrojecida, violcea y tena aspecto cushingoide. No exista acantosis nigricans. El tiroides se palpaba normal y
el examen cardiopulmonar y abdominal era normal. Exista falta de fuerzas en las extremidades. La glicemia era 414 mg/dl, cetonemia
(++), hemoglobina glicosilada A1C 12%, exista acidosis metablica (pH 7,0, bicarbonato 2,9 mEq/L) con lactacidemia normal, creatinina
1,3 mg/dl y potasio 2,6 mEq/L. Se administr insulina cristalina subcutnea y luego en bomba de infusin en dosis crecientes, hasta 520
unidades en 24 h, sin lograr controlar la hiperglicemia ni la cetoacidosis. Recibi bicarbonato de sodio y potasio. Se descart la
existencia de una infeccin. A pesar de la administracin de grandes dosis de insulina cristalina, persistieron la hiperglicemia y la
cetosis.

PREGUNTA
Considerando las condiciones del caso cual de las siguientes medidas serian mas tiles para identificar la resistencia al tratamiento?

RESPUESTA
a.- Buscar anticuerpos antiinsulina.
b.- Buscar anticuerpor antireceptores.
c.- Buscar niveles de CD8.
d.- Buscar anticuerpos antimicrosomales tiroideos

TRAUMA CRANEOENCEFALICO (TCE):
CIENCIAS BASICAS: En adultos, es definido como un intercambio brusco de energa mecnica que genera deterioro fsico y/o funcional
del contenido craneal. Se consigna como alteracin del contenido enceflico el compromiso de conciencia, la amnesia postraumtica
y/o un sndrome vertiginoso o mareos persistentes. Tambin debe considerarse como un signo de disfuncin del contenido craneal la
aparicin de una cefalea holocrnea persistente y progresiva que puede o no acompaarse de vmitos. Se distingue de la Contusin de
crneo, que corresponde a un impacto mecnico sobre la bveda craneana que no produce alteracin del contenido craneano, y que
puede asociarse a dolor local. SIGNOS DE ALARMA, en la evaluacin prehospitalaria o la llegada a urgencias: Deterioro progresivo de la
conciencia (disminucin de Glasgow), signos de focalidad neurolgica, cefalea progresiva, vmitos explosivos recurrentes, agitacin
psicomotora, convulsiones, amnesia antergrada de ms de 30 min, cambio en el tamao de las pupilas, sospecha de herida craneal
penetrante, intoxicacin, evidencia clnica o radiolgica de fractura de crneo, sat. O2 <80%, hipotensin. SALUD PUBLICA: Constituye
una de las principales causas de mortalidad e incapacidad en la poblacin menor de 40 aos. Cada ao fallece 1,000 000 de personas
vctimas de trauma craneal severo. Los accidentes con vehculo automotor son la principal causa de traumatismos. El trauma causa
150,000 muertes en EU y un tercio se acompaa por TCE severo. Entre 2-10% de pacientes con TCE tienen asociada lesin cervical.
PATOGENIA: Se distinguen 2 mecanismos bsicos: la colisin o traumatismo directo, en el cual actan como formas lesivas la energa
cintica y la deformante. Y el traumatismo indirecto en el que no existe contacto previo, cabeza/objeto agresor, siendo las fuerzas
lesivas la aceleracin angular pura y la hiperpresin transmitida pura. El impacto mecnico origina la degeneracin neuronal mediante
3 mecanismos bsicos: 1. Mecanismo lesional primario, son lesiones nerviosas y vasculares producidas inmediatamente por la agresin
biomecnica, las lesiones resultantes de la agresin primaria son: fracturas craneales, contusiones, laceraciones, hematomas
intracerebrales, lesin axonal difusa. 2. Mecanismo lesional secundaria; en los traumatismos cerrados, el impacto de las fuerzas se
produce en las zonas donde el crneo se pone en contacto directamente con el encfalo, pudiendo producir lesiones en la zona de
impacto (lesin por golpe) y en la zona diametralmente opuesta (lesin por contragolpe).,tambin lesiones por cizallamiento, las
lesiones resultantes son; hipotensin, hipercapnia, hipoxemia,
hipertermia, hipoglucemia, acidosis, hiponatremia, hipertensin
intracraneal, hematoma cerebral tardo, edema cerebral,
convulsiones, vasoespasmo. 3. Mecanismo lesional terciario;
engloban una serie de procesos neuroqumicos y fisiopatolgicos complejos, concatenados, con
posibilidad de retroalimentacin positiva, entre s que inician inmediatamente tras el TCE.
CLASIFICACION: En la escala de coma de Glasgow se puede obtener una calificacin mxima de 15 y
un mnimo de 3. Un paciente que presenta cualquiera de los siguientes signos debe considerarse
que sufre un TCE severo: Anisocoria, dficit motor localizado, fractura abierta del crneo con
exposicin de masa enceflica o salida de LCR, deterioro neurolgico, fractura deprimida de la
bveda del crneo. Otra clasificacin es en: BAJO RIESGO; Asintomtica, cefaleas, mareos,
hematoma, laceracin o scalp de cuero cabelludo, ausencia de criterios de moderado o alto riesgo.
Actitud a seguir; Si no presentan otras lesiones asociadas que requieran ingreso hospitalario u
observacin se envan a su domicilio siempre que una segunda persona pueda observar la evolucin
del paciente en las prximas horas. Debe informarse por escrito sobre signos o sntomas de alarma,
ante la presencia de las cuales debe consultar de nuevo al hospital. MODERADO RIESGO; Historia de perdida transitoria de la
conciencia, intoxicacin por alcohol y drogas, cefalea progresiva, vmitos persistentes, amnesia peritraumatica, politraumatismo que
impide adecuada valoracin clnica del TCE, traumatismo facial severo, sospecha de nio maltratado, edad menor de 2 aos excepto
lesin trivial. Actitud a seguir; deben permanecer en observacin al menos 24 hrs, pueden reducirse a 12 hrs la observacin si no hay
sintomatologa neurolgica, TAC de crneo normal, no existen otras lesiones asociadas que requieran ingreso hospitalario. Si existe
sintomatologa neurolgica evidente, el periodo de observacin debe prolongarse y valorar TAC craneal de control a las 12-24 hrs.
ALTO RIESGO; Disminucin del nivel de conciencia actual o progresivo no claramente debido a otras causas (metablico, epilepsia),
signos neurolgicos de focalidad, hundimiento o herida penetrante de crneo, sospecha de fractura de la base del crneo (otorrea,
hemotmpano, rinorrea, hematoma en anteojos, hematoma retroauricular), convulsiones postraumticas, respiracin irregular o
CLASIFICACION DE TCE SEGN OMS
Fracturas de crneo:
Fracturas de la bveda
Fracturas de la base
Fracturas de los huesos de la cara
Otras y las fracturas inclasificables
Mltiples fracturas que afectan el
crneo o la cara con otros huesos
Lesin intracraneal
Conmocin
Laceracin cerebral y contusin
Hemorragia subaracnoidea,
subdural y extradural
Hemorragias intracraneales
postraumticas inespecficas
Lesin intracraneal de naturaleza
inespecfica
GRAVEDAD DE TCE
Leve 14-15 puntos
Moderado 9-13 puntos
Severo Menos de 9 puntos
MANUAL DE TRABAJO DEL CURSO ENARM CMN SIGLO XXI
CURSO ENARM CMN SIGLO XXI TEL: 36246001 Pharmed Solutions Institute PGINA 258

apnica. Actitud a seguir Una vez diagnosticados y estabilizados, deben pasar al rea de tratamiento definitivo (quirfano o UCI). Desde
el punto de vista patolgico, pueden existir tres tipos fundamentales de lesiones cerebrales: 1. Conmocin o concusin cerebral;
caracterizada clnicamente por una breve perdida de conciencia, con un corto periodo de amnesia seguida de una recuperacin rpida y
total, sin ningn signo neurolgico focal. No hay lesin estructural macroscpica del cerebro, tan solo se producen lesiones por
estiramiento de los tractos axonales de la sustancia blanca, con perdida reversible de su funcin, responsables de la prdida de
conciencia transitoria. 2. Contusin cerebral; sobre todo en polos frontales y temporales, por el contacto entre la superfici e cerebral y
el interior del crneo, que abarcan desde una simple magulladura en una pequea rea cortical, hasta lesiones extensas, a menudo
hemorrgicas, de gran parte de la superficie cerebral, con dao en la sustancia blanca y el mesencfalo, clnicamente alteracin del
nivel de conciencia, desde confusin, inquietud y delirio y grados variables de coma, estos pacientes deben ser hospitalizados para
observacin, dado el desarrollo tardo de edema cerebral. Realizar TAC, valora necesidad de iniciar tratamiento para HIC. Las
contusiones que producen efecto de masa requieren ciruga urgente. 3. Lesin cerebral difusa; presencia de un coma prolongado de
das o semanas, lesin frecuente con alta mortalidad. Su diagnstico es presumible cuando en la TAC cerebral no se aprecia una lesin
ocupante de espacio en un paciente con coma profundo, adems de la situacin de coma suelen presentar posturas de descerebracin
o decorticacin, y frecuentemente presentan signos de disfuncin autonmica. DIAGNOSTICO: Clasificacin de los hallazgos de la
primera tomografa tras el TCE: A) Lesin difusa I; sin patologa visible. B) Lesin difusa II; cisternas visibles, con desviacin de la lnea
media hasta 5mm y/o sin lesin mayor de 25 ml. C) Lesin difusa III; cisternas comprimidas o ausentes, con desviacin de la lnea
media hasta 5 mm y/o sin lesin mayor de 25ml. D) Lesin difusa IV; desviacin de la lnea media mayor a 5 mm, sin lesin mayor de
25ml. E) Toda lesin evacuada quirrgicamente. F) Lesin mayor de 25ml no evacuada quirrgicamente. TRATAMIENTO: inicial de las
situaciones de amenaza vital: 1. Asegurar la permeabilidad de la va area con control de la columna cervical. 2. Oxigenacin y
ventilacin adecuada. 3. Control de la hemorragia externa y mantener la presin arterial. 4. Evaluacin del estado neurolgico. 5
Investigar otras lesiones traumticas. Objetivo del tratamiento en TCE: Posicin de la cabeza a 30 sobre el plano horizontal, analgesia
eficaz, normotermia, PaO2 >70mmHg, normocapnia, presin arterial media >90mmHg, euvolemia, Hb >10mg/dl, osmolaridad
plasmtica >290mOsm, glucemia <200mg/dl, profilaxis de convulsiones precoces. Farmacologa de TCE; MANITOL, su efecto rpido
sobre la PIC es especialmente til en situaciones de urgencia, su accin es rpida y fugaz, obtenindose el efecto mximo a los 40
minutos de infusin en bolo, en urgencias, redujo con mayor frecuencia la dilatacin pupilar y mejoro la evolucin a los 6 meses,
tienden a mostrar mayor sobrevida y mejor pronstico. Indicaciones: a todo paciente hemodinamicamente estable con signos de
herniacin cerebral (anisocoria, signos de decorticacin-descerebracin) se le debe administrar manitol en bolo rpido, realizando a
continuacin una TAC craneal urgente. Dosis se comienza con un bolo de 1-2 g/kg, se puede repetir cada 6 hrs. FUROSEMIDA, su
ventaja sobre el manitol es su efecto sobre la osmolaridad es menos marcado, aunque puede producir trastornos electrolticos, la
desventaja es que es menos efectivo en el tratamiento del edema cerebral. Indicaciones: lesiones hemorrgicas cerebrales, debera
reservarse para pacientes en los que la expansin de volumen producida por el manitol pudiera resultar perjudicial (cardiopatas,
insuficiencia renal). CORTICOIDES, son eficaces para disminuir el edema cerebral en los procesos tumorales o inflamatorios pero en el
edema, debido al traumatismo, no parecen ser tiles. La administracin de pre-hospitalaria de sol.de cloruro de Na al 7.5% para trauma
e hipotensin se asocia con un incremento significativamente mayor en la presin arterial comparado con la infusin de Ringer-lactato.
Profilaxis antibitica, siempre en los traumatismos abiertos y penetrantes, cuando existan signos clnicos cardiolgicos de fractura de la
base de crneo, se emplearan antibiticos de amplio espectro. Profilaxis de crisis convulsivas; se establecer precozmente un
tratamiento de fondo preventivo de las mismas siempre que la lesin sea supratentorial con afectacin del parnquima cerebral
(hundimientos, contusiones, dislaceraciones, hematomas intraparenquimatosos). Los anticomiciales usados son; fenobarbital (100mg
c/8hrs) tanto enteral como parenteral y fenitoina (100mg c/8 hrs). TCE grave; el exceso de tono simptico en la dinmica vascular
cerebral y sus efectos sobre la PC debe ser controlado con una adecuada sedacin y analgesia; opiceos 8fentanilo, morfina),
benzodiacepinas, propofol. La intubacin debe ir siempre acompaada de una adecuada sedacin, una correcta relajacin muscular ya
que si esta no se consigue durante la maniobra de la PIC esta aumentara, el relajante muscular ideal para estos pacientes es la
succinilcolina. COMPLICACIONES: Hematoma extradural o epidural: coleccin de sangre entre el crneo y la duramadre cuya causa ms
frecuente es la lesin traumtica de la arteria menngea, sospechar en paciente que ha sufrido TCE, que tras un periodo de 1-24 hrs
entra en estado de coma, pudiendo haber dilatacin pupilar del lado lesionado y hemiparesia contralateral. Este cuadro requiere
ciruga inmediata. Hematoma subdural: Debidos a roturas de las venas comunicantes entre corteza cerebral y duramadre, incidencia
mayor en pacientes etlicos y ancianos. Si aparecen en las primeras 24 hrs son agudos, entre 24hrs y 2 semanas subagudos y crnicos
cuando aparecen ms tardamente. El hematoma subdural agudo requiere ciruga urgente. Hemorragia subaracnoidea: se acompaa a
menudo de hematoma subdural concomitante o de una contusin cerebral, el diagnostico se realiza mediante TAC y si es normal el
diagnostico se realizara mediante la demostracin de un LCR hemorrgico, no requiere tx., quirrgico urgente. Hematoma
intraparenquimatoso: pueden manifestarse como lesiones rpidamente expansivas o ser asintomticas, en la mayora de los casos
existe fractura craneal asociada por golpe o contragolpe.

CASO CLINICO
Varn de 41 aos que bajo el efecto de sustancias txicas sufre una cada de 5 metros de altura que le provoca traumatismo
craneoenceflico y parada cardiorrespiratoria (PCR). La parada revierte tras reanimacin cardiopulmonar con una duracin estimada de
anoxia total de 20 minutos. Ingresado en la UCI, el estudio toxicolgico fue positivo para etanol, cocana y cannabis. 2 horas se presenta
crisis convulsivas tnico clnica presentando disminucin del estado de conciencia con respuesta parpebral expontanea,
desorientacin, habla incongruente no obedece ordenes, con respuesta de retirada por estimulo doloroso, no localizado.

PREGUNTA
Considerando el estado clnico actual cual es la conducta inmediata a seguir.

RESPUESTA
a.- Administracion de bicarbonato.
b.- Colocacin de tubo endotraqueal.
MANUAL DE TRABAJO DEL CURSO ENARM CMN SIGLO XXI
CURSO ENARM CMN SIGLO XXI TEL: 36246001 Pharmed Solutions Institute PGINA 259

c.- Colocar sonda orofaringea.
d.- Hiperventilar con bolsa-mascarilla.

CASO CLINICO
Paciente de 51 aos, con antecedentes de traumatismo craneoenceflico, contusin cerebral frontobasal, hemorragia subaracnoidea
postraumtica y meningitis asptica, hace 10 meses, por accidente laboral. Desde el traumatismo presenta: impotencia sexual y
disminucin de la libido, astenia, anorexia, as como poliuria, polidipsia y nicturia. El paciente refiere asimismo tendencia al sueo,
abotargamiento, cada de pelo y disminucin del ritmo de afeitado, cuadro que ha ido progresando hasta acentuarse a raz de un
catarro. Exploracin fsica: temperatura, 36 C; presin arterial, 60/40 mmHg, y frecuencia cardaca, 40 pulsaciones/min. Se trata de un
paciente joven, con mal estado general, plido, con piel seca y spera, ausencia de pelo en cola de las cejas, disminucin del vello axilar
y pubiano, abotargado y con tendencia al sueo. La auscultacin cardaca revela bradicardia.

PREGUNTA
Cul es la causa ms probable que presenta la paciente.

RESPUESTA
a.- Isquemia hipofisiaria.
b.- Necrosis hipofisiaria.
c.- Sindrome de secrecin inapropiada de ADH.
d.- Panhipopitituarismo.

CASO CLINICO
Mujer de 74 aos con antecedentes de cardiopata valvular con prtesis biolgicas en posicin mitral y artica, hipertensin arterial
bien controlada con enalapril y propanolol y fibrilacin auricular paroxstica en tratamiento con acenocumarol. Acude por dolor cervical
y debilidad en los miembros inferiores sin traumatismo previo. En el momento del ingreso la paciente est afebril y normotensa, as
como consciente y orientada aunque discretamente bradipsquica. En la exploracin neurolgica se aprecian signos de irritacin
menngea y una parlisis completa de ambas extremidades inferiores con hipotona y reflejos rotulianos y aquleos abolidos, y reflejo
cutneo plantar bilateralmente extensor. Se evidencia una anestesia tctil y dolorosa con nivel dorsal bajo.

PREGUNTA
Cul es la arteria ms probable que se compromete.

RESPUESTA
a.- Cerebral anterior.
b.- Arteria basilar.
c.- Comunicante posterior.
d.- Cerebral media.

CASO CLINICO
Mujer de 28 aos presenta vmitos y cefalea de varias horas de evolucin. Dado el mal estado general de la paciente se decide su
ingreso. Como nicos antecedentes de inters destacaba el tabaquismo, una ciruga previa de tiroides por bocio y consumo de
anticonceptivos orales desde haca un mes. Al da siguiente la paciente inicia un cuadro progresivo de somnolencia, desorientacin y
hemiparesia izquierda, es ingresada a terapia intensiva posterior a deterioro neurolgico.

PREGUNTA
Cul es la complicacin mas frecuentes que este paciente desarrolle.

RESPUESTA
a.- Neumonia.
b.- Urosepsis.
c.- Neuroinfeccin.
d.- Sindrome ADH.







MANUAL DE TRABAJO DEL CURSO ENARM CMN SIGLO XXI
CURSO ENARM CMN SIGLO XXI TEL: 36246001 Pharmed Solutions Institute PGINA 260

ISQUEMIA CEREBRAL TRANSITORIA (ICT):
CIENCIAS BASICAS: Se define como un episodio breve de disfuncin neurolgica causado por dao cerebral focal o isquemia retiniana,
con signos que tpicamente duran hasta una hora y sin evidencia de infarto cerebral agudo. SALUD PUBLICA: Se calcula que en Estados
Unidos de Norteamrica hay 300,000 casos nuevos de ICT por ao y alrededor de 15-20% de los pacientes con un infarto cerebral
tienen historia de ICT. Recientemente se ha demostrado que la ICT es un fuerte predictor a corto plazo de infarto cerebral, enfermedad
cardiovascular y muerte. Presentar un ICT conlleva un riesgo de ACV en el primer mes de 8% y al ao de 5%, junto con 5% de riesgo de
infarto de miocardio al ao. PATOGENIA: Se basa especialmente en un mecanismo de produccin aterotrombotico, que sin duda es el
principal responsable de la mayora de los casos de ICT. Otros mecanismos como el cardioembolismo, la arteriopata no ateroesclerosa
y las vasculopatas han sido descritas tambin como ICT. Las observaciones iniciales mediante examen oftalmoscpico de la amaurosis
monocular transitoria, sirvi para ilustrar el compromiso del flujo sanguneo en las arterias retinianas y el rompimiento de las columnas
venosas en patrn cuadrado, con material "blanco" que obstrua la arteria retiniana. La razn ntima relacionada con las lesiones en ICT
tiene que ver con la estenosis vascular y ulceracin de placas aterosclerticas con formacin de trombos, con subsecuente
embolizacin de material de fibrina y plaquetas desde estos sitios aterosclerticos, como el arco artico, cartida interna extracraneal e
intracraneal y la arteria cerebral media y la vertebral. Igualmente, la formacin de cogulos de fibrina y glbulos rojos formados en la
circulacin rpida o las cavidades cardiacas tambin cuenta como causal de las lesiones en ICT. Recientemente, el estudio para el
tratamiento de ACV Agudo defini las anormalidades cardiacas de alto y mediano riesgo de embolizacin. Las anormalidades de alto
riesgo (mayor de 5% anual) son: la fibrilacin auricular (FA), vlvulas protsicas, enfermedad reumtica cardiaca, endocarditis
bacteriana, mixoma auricular y cardiomiopata dilatada. Los de riesgo moderado (2% anual) son: personas mayores de 65 aos sin
factores de alto riesgo; y pacientes con riesgo bajo (1% anual) los menores de 65 aos y sin factores de riesgo concomitantes.
CARACTERISTICAS CLINICAS: Debe referirse exclusivamente al principal territorio arterial afectado, es decir carotideo o vertebrobasilar
(tallo cerebral). La caracterstica principal es la constelacin de signos y sntomas de dficit neurolgico focal que son: alteraciones de
la conciencia o sincope, mareo monosintomtico, amnesia o confusin aislada, crisis convulsivas, vrtigo aislado, diplopa aislada,
escotomas cintillantes, disfagia aislada, disartria aislada, incontinencia. Los sntomas aislados no deben ser considerados como eventos
de ICT. En el territorio carotideo las ms frecuentes son: dficit motor, sensitivo o trastornos del lenguaje e incluyen; disfuncin
sensitiva y motora de extremidades contralaterales, seguidas de afeccin motora o sensitiva pura o bien con menos frecuencia disfasia
o afasia aislada. Duracin de sntomas menos de 15 min. La amaurosis fugax o ceguera monocular transitoria (visin borrosa, nebulosa
o vidrio empaado) es una de las manifestaciones clnicas tradicionales de enfermedad carotidea ateroesclerosa, se atribuye a
embolismo arteria-arteria, duran de 1-5 min., y rara vez exceden los 15 min. La visin se restablece por completo al terminar el evento.
En el territorio vertebrobasilar las manifestaciones pueden ser ms variadas lo que provoca con mucha frecuencia que sntomas
aislados se confundan con ICT. Lo ms recuente es debilidad o torpeza que pueden cambiar de un lado a otro, alteraciones sensitivas
que pueden ser bilaterales, hemianopsia homnima o ceguera total transitoria, ataxia, diplopa; o bien por lo menos dos de los
siguientes; disartria, diplopa, vrtigo o disfagia. DIAGNOSTICO: Se obtiene por historia clnica, ya que al momento de la exploracin es
frecuente que el paciente se encuentre recuperado completamente. Es importante realizar un adecuado examen fsico general
buscando cardiopatas o enfermedades de origen vascular y un examen neurolgico exhaustivo ya que permiten identificar signos
persistentes. Laboratorio: Nos ayuda a identificar causas metablicas como hipoglicemia, hiponatremia y trombositosis, una VSG
elevada puede sugerir endocarditis bacteriana o arteritis temporal. Dado que una de las causas de ICT es la embolia de origen cardiaco,
uno de los estudios iniciales debe ser un ECG, que puede revelar fibrilacin auricular o IAM silente. La TAC y RM pueden revelar
patologas que simulan a ICT. El doppler carotideo o angioIRM permiten identificar enfermedad ateroesclerosa. TRATAMIENTO: La
presencia de ICT ofrece la oportunidad de iniciar tratamientos que limiten el posible inicio de un infarto cerebral. No existe tratamiento
especfico para la ICT, debe individualizarse en base a los factores de riesgo y alteraciones encontradas en cada paciente. La
identificacin de factores de riesgo tanto modificables (HTA, DM, cardiopata isqumica, hipercolesterolemia, tabaquismo, alcoholismo
y sedentarismo) como no modificables (edad, genero, raza, herencia) debe ser una prioridad. En la HTA reducciones de 10mmHg en
sistlica, as como 5 en diastlica se asocian con un 30-40% de reduccin de riesgo de EVC, por ello es uno de los principales elementos
en la prevencin de nuevos eventos de ICT y de infarto cerebral. Los niveles de glucosa en pacientes con ICT deben ser <126mg/dl, y en
casos ya confirmados de DM, el tx., con hipoglucemiantes orales y/o insulina debe ser iniciado de inmediato y control estricto. Manejo
farmacolgico: Antiagregantes plaquetarios est indicado en los casos de ICT de origen no cardioemblico ya que se ha demostrado
disminucin en el riesgo de EVC recurrente en alrededor de 22%. Los aprobados como medida de prevencin secundaria son: aspirina
(75-325mg/d), clopidogrel y la combinacin de aspirina-dipiridamol de liberacin prolongada. El tratamiento con anticoagulantes orales
est indicado en los casos de ICT secundarios a fibrilacin auricular (ICT cardioemblica) u otras patologas cardiacas potencialmente
embolignas. Se sugiere mantener valores de INR en promedio de 2.5 (2-3).Otro grupo de alto riesgo en quienes est indicada la
anticoagulacin son los pacientes con ICT in crescendo. En estos casos la presentacin de los episodios de ICT es repetitiva, ya que se
ha observado cese de la sintomatologa de manera dramtica despus del inicio de la anticoagulacin. La endarterectoma carotidea o
stent carotideo se reserva para pacientes con criterios establecidos para dicho manejo. Dado el alto riesgo de desarrollar isquemia
cerebral en las horas o das que siguen al ICT, esta debe considerarse como una urgencia neurolgica. Los objetivos de su evaluacin
oportuna incluyen: confirmar que se trata de ICT, definir su mecanismo de produccin, para establecer un tratamiento adecuado.

CASO CLINICO
Mujer de 53 aos de edad que fue evaluada debido a un trastorno del lenguaje de 12 h de evolucin e inicio brusco. La paciente refera
dificultades para encontrar las palabras durante el discurso. El examen neurolgico mostr una afasia de Broca moderada y una prdida
parcial de la sensibilidad del brazo y la pierna derechos. El examen del fondo de ojo era normal. No exista sndrome confusional ni
signos de meningismo. La paciente presentaba un historial oncolgico de larga evolucin. Fue diagnosticada de carcinoma de crvix
uterino a los 26 aos, y de neoplasia mamaria a los 43. Un mes antes del episodio actual se realiz una tomografa computarizada
toracoabdominal debido a un incremento de los marcadores tumorales en sangre. ste mostr una masa ovrica, mltiples
adenopatas abdominoplvicas y un tromboembolismo pulmonar bilateral.

PREGUNTA
MANUAL DE TRABAJO DEL CURSO ENARM CMN SIGLO XXI
CURSO ENARM CMN SIGLO XXI TEL: 36246001 Pharmed Solutions Institute PGINA 261

Cul es la frecuencia de presentar EVC dentro de los primeros 6 meses.

RESPUESTA
a.- 10 al 20 %
b.- 20 al 30 %
c.- 30 al 40 %
d.- Ms del 40 %

CASO CLINICO
Mujer de 73 aos sin factores de riesgo vascular que acudi a urgencias por presentar cuadro de inicio sbito de disminucin de la
movilidad en hemicuerpo izquierdo con cada al suelo. A su llegada a urgencias estaba consciente con desviacin conjugada de la
mirada a la derecha y hemipleja completa izquierda, hemihipoestesia y reflejo de Babinsky izquierdo presente. TC craneal sin lesiones
evidentes. Se aplic tratamiento tromboltico a los 150min del inicio de los sntomas. En su evolucin en las primeras horas se objetiv
clara mejora de la movilidad de hemicuerpo izquierdo, persistiendo al alta hemiparesia izquierda.

PREGUNTA
Cul es la conducta a seguir para mejorar la secuela del paciente.

RESPUESTA
a.- Rehabilitacin fsica.
b.- Rehabilitacion neuropsicolgica.
c.- Se evolucin es favorable.
d.- Rehabilitacin en casa.

CASO CLINICO
Varn de 75 aos, dislipidemia en tratamiento hipolipemiante como nico factor de riesgo cardiovascular. Trado a urgencias por
cuadro de hemiparesia izquierda, parlisis facial central izquierda, hipoestesia en hemicuerpo izquierdo, disartria y Babinsky izquierdo.
TC crneo sin alteraciones. Se administr tratamiento tromboltico a 150 min de evoluci. Posterios al tratamiento se mantiene
semiologa neurolgica, con TC de control a las 24h de la trombolisis con infarto en territorio de arteria cerebral media con edema
citotxico y desviacin de lnea media. Se aadi tratamiento con manitol, con mejora del nivel de conciencia

PREGUNTA
Cul es el tiempo mximo para administracin de trombolisis.

RESPUESTA
a.- 100 minutos.
b.- 150 minutos.
c.- 200 minutos.
d.- 250 minutos.






MANUAL DE TRABAJO DEL CURSO ENARM CMN SIGLO XXI
CURSO ENARM CMN SIGLO XXI TEL: 36246001 Pharmed Solutions Institute PGINA 262

COMA Y MUERTE CEREBRAL:
CIENCIAS BASICAS: Dentro de las alteraciones agudas de la vigilia; la obnubilacin significa literalmente embotamiento mental o
torpeza, se aplica a enfermos con reduccin leve o moderada del estado de vigilia. Es distintivo en ellos lograr la reaccin de despertar
con estmulos sonoros ms o menos intensos. En el estupor el paciente no responde y se despierta solo con estmulos verbales o
dolorosos repetidos y fuertes, tras los cuales vuelve a sumirse en un sueo profundo. En el coma, ningn estmulo despierta al sujeto;
incluso la estimulacin dolorosa no provoca respuestas intencionadas y pueden producir postura refleja de decorticacin o
descerebraciones, se debe a una disfuncin cerebral orgnica difusa. Definicin de COMA; Mxima degradacin del estado de
conciencia. Sndrome clnico caracterizado por una prdida de las funciones de la vida de relacin y conservacin de las de la vida
vegetativa, como expresin de una disfuncin cerebral aguda y grave. Teasdale y Jennet definen el coma como la incapacidad de
obedecer rdenes, hablar y mantener los ojos abiertos. PATOGENIA: La alerta se mantiene por el Sistema Reticular Activador
Ascendente (SRAA) localizado entre el tercio medio de la protuberancia y la porcin ms alta del mesencfalo. Este sistema es una
estructura polisinptica que se puede afectar por procesos intrnsecos del tallo cerebral que lo destruyen, por procesos extrnsecos que
lo comprimen o desplacen y por procesos metablicos que lo alteran o inhiben. Una lesin
hemisfrica produce coma directamente por su volumen o de manera indirecta por
compresin, isquemia o hemorragia en el mesencfalo y tlamo. Esta lesin por crecimiento
radial crea un cono de presin transtentorial y comprimen el SRAA en la parte rostral del tronco
enceflico. CLASIFICACION: Existen mltiples clasificaciones del estado de coma. Segn el
porcentaje de dao cerebral, el sitio y la causa del coma tenemos, ver cuadro. DIAGNSTICO:
La historia clnica de los pacientes en coma sigue siendo el elemento fundamental de su
diagnstico. Ante todo es preciso interrogar a las personas que traen al enfermo para extraer
los datos posibles sobre las circunstancias de aparicin del evento. El inicio del cuadro puede
ser sbito (paro cardiaco, hemorragia o embolias cerebrales) o progresivo (intoxicaciones,
tumores, trombosis cerebral, meningoencefalitis, encefalopata heptica, encefalopata
urmica). Un TCE reciente puede sugerir un hematoma epidural o una contusin cerebral y si es
de ms tiempo orienta hacia un hematoma subdural crnico. Se recogern los antecedentes
inmediatos y antiguos, el antecedente reciente de cefalea puede orientar hacia una masa
expansiva intracraneal (tumor, hematoma, absceso); el de epilepsia, a un coma postcrtico y la
existencia de focalidad neurolgica, a un tumor o isuqemia cerebral. Exploracin fsica siempre
debe realizarse completa por aparatos, ya que puede aportar datos muy tiles. Nivel de
conciencia la profundidad del coma se explora aplicando al paciente estmulos de intensidad
creciente (verbal, tctil y dolorosa) y se clasificar segn la mejor respuesta obtenida durante la
exploracin. Para valoracin del estado de COMA utilizamos la escala de Glasgow. A todo
paciente en coma se le deber realizar un estudio analtico bsico para descartar una causa
metablica de coma (Diabetes mellitus, hipoglucemia, coma urmico, encefalopata heptica o
coma por diselectrolitemia). Ante la sospecha clnica de coma exgeno y en todos los casos de
coma sin diagnstico evidente debera realizarse un estudio toxicolgico de sangre y orina. Es
aconsejable determinar la presencia de alcohol, barbitricos, benzodiacepinas, antidepresivos
tricclicos, fenotiazinas y opiceos. TRATAMIENTO: Medidas generales: Asegurar la oxigenacin,
dada la necesidad de un aporte continuo de oxgeno al cerebro es necesario priorizar y
garantizar la funcin respiratoria. En caso de coma profundo se puede intubar al enfermo como
profilaxis de la broncoaspiracin. Mantener la circulacin: Con el fin de mantener flujo
sanguneo cerebral adecuado. Si deterioro hemodinmico, obrar segn corresponda con aporte
de volumen, drogas vasoactivas, etc. Evitar hipotensin brusca en caso de emergencia
hipertensiva y coma. No bajar TA diastlica por debajo de 100 mmHg. Si alcoholismo crnico o
desnutricin se administrar 100 mg intramuscular y 20 mg endovenoso de Tiamina y luego 50
mL de dextroza al 50 % IV. (25 gramos). Si se administra esta antes de la Tiamina se puede
precipitar una encefalopata de Wernicke. Otras medidas; Vaciamiento del contenido gstrico:
Sonda nasogstrica a bolsa previo lavado gstrico. Sonda vesical: Medir diuresis horaria.
Considerar antdotos y corregir causas: Tratar las causas de reversibilidad inmediata. Glucosa 50
% IV. si hipoglucemia sospechada o constatada. Tiamina (B1) 100 mg IM si alcoholismo.
Flumazenilo 0,25 mg IV. si sospecha intoxicacin con benzodiacepinas. Fisostigmina 5 mL = 2
mg. Administrar lentamente 1 mp. cada 30-60 min. en intoxicaciones graves por antidepresivos tricclicos Naloxona, 1 mL = 0,4 mg.
Dosis: 10 mcg/kg o 400 mcg /dosis nica, si se sospecha intoxicacin por Opiceos. Tratamiento del edema cerebral: Dexametasona 10
mg IV y seguir con dosis de 4 mg IV. cada 6 horas. Manitol al 20 %: 1g/kg IV. en 20 min. Iniciar cuando se aprecia deterioro rostrocaudal.
Evitar soluciones hipotnicas y de glucosa puras- Tratamiento anticonvulsivo: Diazepam 2 mg/min. IV. hasta 20 mg, Clonazepam
1mg/min. IV. hasta 6 mg, Fenitoina (250 mg-5 mL) 2,5 cc IV. c/8 horas. MUERTE CEREBRAL: Es la expresin con la que se designa la
perdida de todas las funciones del encfalo. Se declara cuando los reflejos del tronco cerebral, las respuestas motoras y la actividad
respiratoria estn ausentes en un individuo comotaso normotrmico, con lesin cerebral masiva e irreversible, que no haya recibido
drogas o frmacos que acten en el sistema nervioso central y que no tenga otros factores que contribuyan que contribuyan al cuadro
clnico=coma estructural, origen conocido, dao irreversible, hemodinamicamente estable, temperatura mayor de 34C, ventilacin y
oxigenacin adecuada, ausencia de enfermedades metablicas importantes, ausencia de frmacos o drogas que acten sobre el SNC.
Esta definicin permite certificar la muerte aunque se encuentre funcionando la mayor parte de sus rganos del cuerpo. Solo es posible
establecer este diagnstico en el medio hospitalario (unidades de reanimacin o de cuidados intensivos), y requiere un protocolo
extremadamente riguroso, la intervencin de personal experto y la realizacin de pruebas confirmatorias objetivas y fiables. Adems,
para aumentar la seguridad, se fija un periodo de observacin cuya duracin depende de la edad del paciente. PATOGENIA: La muerte
cerebral implica siempre la lesin estructural del encfalo, en gran parte de los casos suele ser primaria, inicialmente localizada y
COMA POR LESIN ANATMICA
SUPRATENTORIALES (15 A 20 %)
Intracerebrales: Hemorragia cerebral,
Hemorragia intraventricular, Infarto
cerebral extenso (arterial o venoso),
Tumores, Infecciones (Encefalitis
focal, absceso cerebral)
Extracerebrales: Tumores,
Hidrocefalia, Hemorragia intracraneal
postraumtica (epidural, subdural),
Empiema subdural
INFRATENTORIALES (10 A 15 %)
clusin basilar
Hematoma subdural y extradural de la
fosa posterior
Hemorragia pontina primaria
Hemorragia cerebelosa
Infarto cerebeloso
Malformaciones arterio venosas del
tronco
enceflico
Aneurisma de la arteria basilar
Abscesos
Granulomas
Tumores primarios o metastsicos
Mielinolisis central pontina
COMA POR LESIONES DIFUSAS (TXICO-
METABLICAS, 65 A 75 %)
EXGENOS
Frmacos
Txicos
Trastornos fsicos
ENDGENOS
Hipoglicemia
Hipoxia: Disminucin de la tensin de
oxgeno: PaO2 35mmHg, enfermedades
pulmonares, alturas, Hipoventilacin.
Disminucin del contenido sanguneo de
oxgeno: Anemia, intoxicacin por CO2,
metahemoglobulinemia
Shoc: Cardiognico, hipovolmico o
sptico
Alteraciones metablicas: Hiper o
hiponatremia,
hipercalcemia, hiper o hipomagnesemia,
acidosis metablica o respiratoria, hiper
o hipoosmolaridad, hipofosfatemia
MANUAL DE TRABAJO DEL CURSO ENARM CMN SIGLO XXI
CURSO ENARM CMN SIGLO XXI TEL: 36246001 Pharmed Solutions Institute PGINA 263

supratentorial, por ejemplo: hemorragia intracerebral espontanea, traumatismo cerebral, infarto cerebral, hemorragia subaracnoidea y
tumores cerebrales. Menos de 15 % de las lesiones enceflicas son globales, supratentoriales e infratentoriales, casi siempre
secundarias a un paro cardiaco o respiratorio que provoca anoxia enceflica y con menor frecuencia debidas a infeccin del sistema
nervioso central y edema celular ocasionando por txicos o trastornos hidroelectrolticos. Se produce edema global del parnquima
enceflico, con el consiguiente aumento de la presin intracraneal, que iguala a la presin de perfusin enceflica. Esto ocasiona paro
circulatorio intracraneal y conduce a infarto enceflico total y a isquemia global del encfalo; el estadio final es la necrosis de toda la
masa enceflica=muerte cerebral. La prdida de funciones del tronco del encfalo sigue casi siempre a una secuencia rostrocaudal. Se
inicia en el mesencfalo y finaliza en el bulbo. Su fase clnica final suele distinguirse por la cada brusca y significativa de la presin
arterial sistmica, tanto sistlica como diastlica. El paro circulatorio intracraneal completo puede ocurrir simultneamente o ms
tarde, pero nunca antes. DIAGNOSTICO: Exige una certeza absoluta, por lo cual debe seguirse el protocolo sistemtico, estricto y
riguroso. Los tres pilares son: a) conocer la causa de la lesin enceflica, b) descartar los trastornos que pudieran simular la muerte
enceflica (condiciones hemodinmicas, metablicas, farmacolgicas y toxicas actuales o relativamente recientes y c) efectuar una
exploracin neurolgica reglada. La ley General de Salud, dedica su ttulo decimocuarto a la donacin, trasplantes y prdida de vida.
Especficamente los artculos 343 y344 del captulo IV se refieren a la perdida de vida. Artculo 344: La muerte cerebral se define como
la ausencia total e irreversible de todas las funciones cerebrales. Se puede considerar muerte cerebral cuando existen los siguientes
signos: I. Perdida permanente e irreversible de conciencia y de respuesta a estmulos sensoriales. II. Ausencia de automatismo
respiratorio. III. Evidencia de dao irreversible del tallo cerebral, manifestado por arreflexia pupilar y ausencia de movimientos oculares
en pruebas vestibulares y de respuesta a estmulos nociceptivos. En la legislacin mexicana se ha determinado que la muerte cerebral
es irreversible y que puede considerar el fin de vida. De acuerdo con los criterios del artculo 344, no hay diferencia entre el estado
vegetativo persistente y la muerte cerebral. El personal de salud debe involucrarse en la creacin de una normatividad y lenguaje
comn sobre muerte cerebral, mientras que la ley debe adecuarse a los avances tecnolgicos y humansticos.

CASO CLINICO
Varn de 48 aos que acude a nuestro por nuseas y malestar de 2 das de evolucin. Desarrolla rpidamente coma, acidosis
metablica severa (pH 6,8) con anin GAP elevado e infiltrado pulmonar bilateral que requiere soporte ventilatorio invasivo. Se realiza
TAC craneal, que es normal. En las siguientes 12 horas presenta rigidez de descerebracin. En un nuevo TAC se observa hipodensidad
putaminal bilateral y edema cerebral difuso. Ante la sospecha de intoxicacin por metanol se inicia tratamiento con dilisis y etanol. La
evolucin neurolgica es desfavorable, producindose muerte enceflica en 24 horas. Posteriormente se documentaron niveles de
metanol de 0,4g/l.

PREGUNTA
Cual de las siguientes pruebas es menos til para establecer el diagnostico de muerte cerebral?

RESPUESTA
a.- EEG
b.- PEV
c.- Retiro de apoyo ventilatorio.
d.- IRM

CASO CLINICO
Paciente joven de sexo femenino que tras consumir 50mg de xtasis ingresa con una encefalopata hiponatrmica e insuficiencia
respiratoria aguda severa por edema pulmonar, destacndose una natremia de 109mEq/l y una osmolaridad urinaria de 360mOsm/l. Se
realiz la correccin de la hiponatremia mediante la infusin de cloruro sdico hipertnico al 3%, presentando una mala evolucin con
fallo respiratorio y shock. A las 9h la ecografa doppler transcraneal evidenci un patrn compatible con muerte enceflica. La necropsia
mostr edema cerebral con signos de herniacin y hepatizacin del parnquima pulmonar.

PREGUNTA
Cual es la casua mas probable de esta condicin?

RESPUESTA
a.- Sndrome de secrecin inadecuada de hormona antidiurtica.
b.- Prdida de fluidos hipotnicos (hipertermia, sudoracin).
c.- Prdida de la capacidad de dilucin urinaria por lesin tubular renal proximal.
d.- Reabsorcin de fluidos hipotnicos desde la luz intestinal.












MANUAL DE TRABAJO DEL CURSO ENARM CMN SIGLO XXI
CURSO ENARM CMN SIGLO XXI TEL: 36246001 Pharmed Solutions Institute PGINA 264

DELIRIUM:
CIENCIAS BASICAS: Alteracin transitoria del estado mental, caracterizada por la presencia de diferentes manifestaciones clnicas,
principalmente alteracin del nivel de conciencia y atencin, junto con otras alteraciones de esferas cognitiva y no cognitiva. La causa
siempre es orgnica y multifactorial, el inicio agudo y subagudo y el curso fluctuante a lo largo del da, con alternancia de intervalos
lucidos diurnos y empeoramiento nocturno. Delirium tremens: (cuadro confusional agudo) secundario a la privacin alcohlica.
Complicacin grave del sndrome de abstinencia alcohlica. Este aparece en casos de dependencia a alcohol, entre 4-12 horas despus
de la ltima ingesta. La ingesta habitual de alcohol para desarrollarlo es muy variable. SALUD PUBLICA: Su incidencia aumenta su con la
edad y es ms prevalente en la poblacin anciana. Es mucho ms frecuente a nivel hospitalario. Originado por hiperreactivacin de
receptores NMDA, hiperactividad del sistema Noradrenrgico y Dopaminrgico. Hipoactivacin del sistema GABA. PATOGENIA: No est
bien definida, pero el sustrato bsico, consiste en un desorden generalizado del metabolismo cerebral y la neurotransmisin que afecta
a estructuras corticales y subcorticales encargadas de mantener el nivel de conciencia y la atencin. En esta disfuncin se encuentran
implicados neurotransmisores como acetilcolina, dopamina, GABA y serotonina, los cuales unos por exceso y otros por defecto,
justifican los sntomas. Estas alteraciones bioqumicas explican la efectividad de frmacos utilizados en el tratamiento o el efecto
adverso de otras sustancias que, indicados en determinados procesos, podran desencadenar un episodio de delirium. El delirium es un
sndrome que sigue un modelo multifactorial, especialmente en ancianos, ya que representa la compleja interrelacin entre factores
predisponentes intrnsecos (envejecimiento cerebral, patologa orgnica cerebral, episodios previos de delirium, dficit funcionales y
factores estresantes) a un paciente vulnerable y factores precipitantes externos como: infecciones respiratorias y urinarias, alteraciones
metablicas, endocrinopatas, trastornos cardiopulmonares y gastrointestinales, el perioperatorio y medicamentos; principalmente los
anticolinrgicos (difenhidramina, hidroxizina, levodopa, amantadina, escopolamina, atropina, amitrptilina, imipramina,
antiespasmdicos), antiarritmicos (propanolol, digoxina, procainamina, lidocana), antihipertensivos (reserpina, metildopa),
psicotrpicos (amitriptilina, litio, benzodiacepinas, neurolpticos, opiceos, hipnticos), antagonistas H2 (ranitidina, cimetidina,
famotidina). Tambin hay que recordar los estados de intoxicacin por frmacos, en estrecha relacin con los cambios farmacocinticas
y farmacodinamicos asociados a la edad, como disminucin de la masa corporal con
aumento de materia grasa, disminucin de la filtracin glomerular y aclaramiento de
la creatinina, deterior de funcin heptica y disminucin de protenas, que hacen que
dosis teraputicas sean toxicas para ancianos. Otros factores son las lesiones que
produce la hospitalizacin (deprivacin del sueo, interrupcin de rutinas, cambios
del ambiente, inmovilizacin y uso de restricciones fsicas etc). DIAGNOSTICO: Es
clnico; la anamnesis y exploracin fsica son fundamentales, debemos revisar el nivel
de conciencia, atencin (respuesta a estmulos, destruido, respuesta a instrucciones),
memoria (perturbacin de memoria, inmediata y reciente), orientacin (esfera
temporal, segunda de espacial y personal), pensamiento y lenguaje (lenguaje,
irrelevante y repetitivo, incoherente, con circunloquias, la capacidad de abstraccin
es nula), percepcin (interpretacin errnea de un objeto real, alucinaciones,
principalmente visuales) conducta (actividad psicomotora disminuida, con apata e
inmovilidad o bien inquietud y agitacin), estado afectivo (euforia o ansiedad, que
alternan con apata, indiferencia y depresin), ciclo sueo-vigilia (insomnio con empeoramiento de la confusin durante la noche e
hiperinsomnia durante el da). Las pruebas complementarias bsicas y las tcnicas de imagen permiten hacer una valoracin etiolgica,
como BH, QS, VSG, Funcin renal, heptica, electrolitos, glucosa, calcio, EGO, determinacin de niveles de medicamento, ECG,
radiografa de trax. CRITERIOS DIAGNOSTICOS DE DELIRIUM segn DSM-IV: 1. Alteracin de la conciencia, con disminucin de la
capacidad para centrar o mantener o dirigir la atencin. 2. Cambios en las funciones cognoscitivas o alteraciones de la percepcin. 3. La
alteracin se presenta en un corto periodo de tiempo (horas o das) y tiende a fluctuar a lo largo de la vida. 4. Demostracin a travs de
la historia, de la exploracin fsica y de las pruebas complementarias de que la alteracin tienen una causa orgnica. TRATAMIENTO:
Medidas preventivas, ya que reducen la incidencia y reduccin del cuadro clnico. Identificacin y tratamiento de causas subyacentes,
ya que el tratamiento etiolgico en algunas ocasiones conlleva, la resolucin del delirium. Cuidados de soporte y rehabilitadores,
nutricin, hidratacin, facilitar y promover la movilizacin, proteger al paciente de cadas, minimizar el riesgo de neumonas, cuidados
intestinales y de tracto urinario. La utilizacin de psicofrmacos requiere la valoracin riesgo beneficio, estn indicados, en sntomas
como trastornos de la conducta e insomnio, los neurolpticos son de eleccin para el control de la agitacin (haloperidol), los de ltima
generacin o atpicos (risperidona, olanzapina, clozapna), provocan menos efectos extrapiramidales y han demostrado similar eficacia
al haloperidol, por lo que su utilizacin se est extendiendo en la actualidad, no hay que olvidar sus efectos adversos como hipotensin
ortostatica, sedacin y agranulocitosis en el caso de clozapina. Las benzodiacepinas constituyen la medicacin indicada en casos de
delirium secundarios a sndromes de abstinencia por sedantes y alcohol y el control del insomnio. Paciente alcohlico que acude a
urgencias con sntomas de delirium tremens establecido: Este tipo de pacientes requiere ingreso hospitalario en una unidad de
medicina interna o de cuidados intensivos. Son de especial importancia las medidas de soporte, el control hidroelectroltico, la diuresis
y la prevencin de complicaciones. Desde el punto de vista psiquitrico, puede valorarse a necesidad de contencin mecnica debido a
la agitacin, que en algunos casos puede ser extrema y que debe ser de 5 puntos. Debe mantenerse al paciente en un ambiente bien
iluminado, tranquilo, con escasos estmulos sensoriales: Dosis de ataque de 20mg de diazepam por va intramuscular, repitiendo cada
30 minutos hasta conseguir la sedacin Posteriormente se administrarn 10-20 mg cada 6h que se irn reduciendo de manera
progresiva; 50-75mg de cloracepato por va intramuscular, repitiendo cada 30 minutos hasta conseguir la sedacin y posteriormente,
50mg cada 6 horas reduciendo progresivamente

CASO CLINICO
Masculino de 79 aos de edad el cual se encuentra diagnosticado con demencia tipo Alzheimer desde hace 5 aos, actualmente
permanece solo en casa, con supervisin nocturna nicamente, se observa con signos de desnutricin, con mal higiene generalizada, a
la exploracin fsica se observa con ruidos cardiopulmonares adecuados, se observa con moderada agitacin psicomotriz que se alterna
CUADRO CLINICO
DELIRIUM DEMENCIA
Comienzo Sbito Insidioso
Curso Fluctuante Estable
Conciencia Disminuida Normal
Atencin Alterada
globalmente
Normal, salvo en
casos graves
Cognicin Alterada
globalmente
Deteriorada
globalmente
Alucinaciones Visuales A menudo ausentes
Ilusiones Fugaces A menudo ausentes
Orientacin Deteriorada Deteriorada
Lenguaje Incoherente Pobre,
perseveracin
Enf. orgnica Siempre Ausente
MANUAL DE TRABAJO DEL CURSO ENARM CMN SIGLO XXI
CURSO ENARM CMN SIGLO XXI TEL: 36246001 Pharmed Solutions Institute PGINA 265

con periodos de somnolencia, no hay datos de focalizacin, se realiza BH y EGO sin datos de procesos infecciosos, se mantiene en
observacin por la noche los sntomas se agudizan con presentacin de ideas delirantes de dao y alucinaciones visuales.

PREGUNTA
Cul es la conducta a seguir en este paciente.

RESPUESTA
a.- Realizar IRM de Craneo.
b.- Administrar haloperidol.
c.- Realizar hemocultivo.
d.- Cuantificacion de electrolitos sericos.

TRASTORNO BIPOLAR (TB):
CIENCIAS BASICAS: Es una enfermedad psiquitrica recurrente y severa que consiste en la alteracin cclica y recurrente del estado de
nimo, entre episodios de depresin (depresin intensa desesperanza), de mana (felicidad extrema) y mixtos (depresin y exceso de
actividad en una fase maniaca). Estos cambios duran normalmente varias semanas o meses.
SALUD PUBLICA: Existe una prevalencia a lo largo de la vida de 1,6% para trastorno bipolar tipo
I y 0.5% para tipo II. La prevalencia por trasto en personas mayores de 65 aos que se
encuentran en la comunidad se estima en el 0.1-1%. Generalmente empieza durante o despus
de la adolescencia. Es raro que empiece despus de los 40 aos de edad. En relacin con el
comportamiento suicida en TB, la prevalencia del intento suicida fue de 17% para tipo I y 24%
para tipo II. PATOGENIA: Posibles factores etiolgicos: Endgeno o constitucional relacionado
con el biotipo pcnico en un 70%. Hereditario, es de tipo autosmico dominante. Biolgico
(catecolaminicas, indolaminicas). Toxico (alcohol, cocana, anfetaminas. Hormonales (tiroides,
hipfisis, gnadas). Psicolgico (shocks emocionales). La fisiopatologa, esta sin duda mediada
por una red de circuitos lmbicos, estriatales y fronto-corticales interconectados, as como por
sistemas colinrgicos, catecolaminrgicos y serotoninrgicos de neurotransmisores, adems de
alteraciones de neuroplasticidad y resiliencia celular DIAGNOSTICO: El diagnostico de
bipolaridad se hace cuando se observa, que la depresin alterna con un episodio maniaco., de
intensidad variable y que alternan de manera cclica en el tiempo. Generalmente la depresin
aparece primero y en pocas ocasiones el primer episodio de trastorno bipolar es de tipo
maniaco. Factores predictivos de bipolaridad: inicio precoz de episodio depresivo (mujeres <25aos) con inhibicin psicomotriz e
hipersomnia, historia familiar de TB, antecedente de depresin postparto y antecedentes de hipomana al iniciar el tratamiento con
antidepresivos. Episodio maniaco: elevacin del estado de nimo, por sobre los niveles considerados normales (euforia) o en otras
ocasiones irritable, con incremento de la energa, aumento de la actividad psicomotora en general; habla excesiva y acelerada
(verborrea), pensamientos veloces y apresurados, saltando de un tpico al siguiente sin aparente conexin entre ellos. La capacidad de
atencin y concentracin suele estar disminuida y el paciente se distrae con facilidad. Impulsividad y desinhibicin, disminucin de la
necesidad de dormir. En ocasiones es posible observar, desorganizacin del pensamiento y franca psicosis con alucinaciones y delirio.
La duracin vara, pero debe mantenerse por al menos 7 das para constituir un episodio de mana segn DSM IV, o menos si fue
requerida hospitalizacin. Episodio hipomanaco: euforia o irritabilidad, con incremento de la energa, aumento de actividad fsica y
mental, pero con intensidad subumbral para constituir mana. Caractersticas de mana pero los rasgos parecen ser mucho ms
adaptativos, el pensamiento es creativo, se disfruta ms de actividades cotidianas, el pensamiento nunca se desorganiza y o se
observan sntomas psicticos ni se presenta impedimento del funcionamiento. Episodio depresivo: bajo estado de nimo (tristeza),
disminucin de la capacidad de experimentar placer y perdida de inters (anhedonia), disminucin de energa, actividad fsica y mental,
habla infrecuente y enlentecida, pensamientos lentos, aparicin de ideas pesimistas, desesperanza, irritabilidad, disminucin de libido
aumento o disminucin del apetito, dependiendo de la intensidad, es posible observar ideacin suicida y aparicin de franca psicosis
con alucinaciones y delirio. Caractersticas clnicas de la depresin bipolar: antecedentes historia de abuso sexual, explosividad e
irritabilidad en la infancia, antecedentes de trastorno de dficit de atencin, interrupcin de la vida acadmica, historia familiar de
bipolaridad. Inicio precoz y sbito, crisis depresiva separada de la experiencia existencial, hipersomnia, hiperfagia y aumento de peso,
delirio no congruente con el estado afectivo, confusin mental. Inhibicin psicomotriz, agitacin, ansiedad, sntomas ms intensos en la
mana, cambios en la polisomnografia. Episodio mixto: aparicin simultanea de sntomas maniacos y depresivos, mas frecuente en
mujeres, responden de manera menos eficaz al litio que al cido valproico, tienen mayor comorbilidad con el uso de sustancias y
alcohol y mayor suicidalidad que la mana pura TRATAMIENTO: La meta es dar una respuesta adecuada a ambas fases de la
enfermedad (mana, depresin), prevenir la frecuencia y mejorar la calidad. Las intervenciones psicosociales estructuradas se han
validado como un medio efectivo. El manejo psicofarmacolgicos es condicin fundamental pero no nica. Los frmacos incluyen a los
que disminuyen la sintomatologa efectiva, los que previenen el episodio agudo y los que son complemento de distintas circunstancias
clnicas en el curso de la enfermedad. Frmacos estabilizadores del nimo: litio (tiene eficacia antimanaca y estabilizadora del nimo,
dosis de 600-2400mg), Ac. Valproico, carbamacepina, lamotrigina, oxcarbamazepina. Antipsicticos tpicos: haloperidol. Antipsicticos
atpicos: risperidona, clozapina, olanzapina. Benzodiazepinas: clonazepam, lorazepam.

CASOS CLINICOS
Paciente de 17 aos, portadora de un trastorno bipolar, trastorno de personalidad limtrofe y con antecedente de abuso de marihuana,
cocana, xtasis y herona. Despus de cuatro das de ser dada de alta desde una clnica psiquitrica, donde estuvo dos meses en
tratamiento con clozapina, cido valproico, carbonato de litio y lorazepam, inici un cuadro de confusin, letargia, conducta catatnica,
rigidez, mialgias y fiebre, que mantena por cinco das, razn por la que ingres a la Unidad de Cuidados Intensivos (UCI) con el
diagnstico presuntivo de meningoencefalitis. En el examen fsico se constataron fiebre, taquicardia, normotensin, rigidez
TRASTORNO BIPOLAR segn DSM-IV
TIPO I: Episodio maniaco nico
Episodio ms reciente hipomaniaco
Episodio ms reciente maniaco
Episodio ms reciente mixto
Episodio ms reciente depresivo
Episodio ms reciente no
especificado
TIPO II: Trastorno depresivo mayor
recurrente con episodios hipomaniacos
Trastornos ciclotmicos: Presencia de al
menos dos aos de reiterados sntomas
hipomaniacos y sntomas depresivos
alternados
Trastorno bipolar no especificado:
Trastornos con caractersticas bipolares,
que no cumplen los criterio para ninguno
especifico
MANUAL DE TRABAJO DEL CURSO ENARM CMN SIGLO XXI
CURSO ENARM CMN SIGLO XXI TEL: 36246001 Pharmed Solutions Institute PGINA 266

principalmente del esqueleto axial, y gran agitacin psicomotora. El hemograma de ingreso mostr 17.000 leucocitos/mm3 con 14% de
baciliformes, plaquetas normales, VHS de 31, PCR de 19,8 mg/dl, CK de 920 mg/dl con fraccin MB de 22 mg/dl. Se tomaron
hemocultivos, cultivo de orina, radiografa de trax y puncin lumbar que fueron normales, pese a lo cual recibi antibiticos por
sospecha de un cuadro pulmonar aspirativo.

PREGUNTA
Cual es la complicacin mas probable que presenta el caso?

RESPUESTA
a.- Sindrome serotoninergico maligno.
b.- Sindrome neurolptico maligno.
c.- Sndrome neurotxico por litio.
d.- Encefalopatia por hepatotoxicidad.

PSICOSIS:
CIENCIAS BASICAS: La Asociacin Psiquitrica Americana estableci la diferenciacin entre trastornos psicticos y trastornos no
psicticos, puesto que, ante una agresin exgena, es posible hallar, cuadros acreedores del trmino psicosis. Tambin se estableci la
distincin entre trastorno "orgnico" y trastorno "sintomtico". Esta subdivisin se apoya en el hecho de que en los trastornos
orgnicos existira una lesin histopatolgica cerebral de carcter permanente, con sintomatologa crnica y estable. En los trastornos
sintomticos existira una alteracin fisiopatolgica cerebral sin lesiones orgnicas pesquisables, perturbando el organismo en forma
aguda y generalmente transitoria, con un cuadro clnico agudo y reversible. En algunos casos, si la afeccin se prolongase o fuera
demasiado intensa, o si existiera una labilidad previa del terreno, este cuadro podra evolucionar hacia un trastorno orgnico. De este
modo, el trastorno sintomtico sera la manifestacin nerviosa central de una enfermedad sistmica o de una intoxicacin que no
habra producido una lesin histopatolgica por el momento. As, aunque la sintomatologa confusional es preferentemente de origen
"sintomtico" y la semiologa demencial de procedencia "orgnica" es posible encontrar cuadros confusionales en una demencia,
alteraciones irreversibles de memoria despus de algunos delirium tremens o cuadros neurotiformes en muchos trastornos "orgnicos"
o "sintomticos". Por esta razn, DSM III entre otros, se refieren siempre a "trastornos orgnicos". 1.- Demencia de aparicin senil o
presenil. 2.- Trastornos mentales orgnicos inducidos por sustancias psicoactivas. 3.- Trastornos mentales orgnicos asociados con
trastornos fsicos del eje III, o cuya etiologa es desconocida. Dentro de estos: a.- Delirium, b.- Demencia, c.- Sndrome amnsico, d.-
Trastorno delirante orgnico, e.- Alucinosis orgnica, f.- Trastorno orgnico del estado de nimo, g.- Trastorno orgnico de ansiedad, h.-
Trastorno orgnico de la personalidad. La proclividad o la resistencia a hacer una psicosis somtica vara mucho de un sujeto a otro. Los
factores individuales que intervienen en la etiopatogenia son: psquicos, endgenos y somticos. Los factores ms influyentes son los
de orden somtico, dentro de los cuales debe considerarse: la va de penetracin de la noxa, los dispositivos bsicos en la defensa
metablica e inmunitaria y el estado del sistema nervioso. Un factor individual importante de considerar es la edad del enfermo. En los
nios, las psicosis sintomticas son mucho ms frecuentes que entre los adultos y a menudo adoptan la forma de "delirium". Entre los
adolescentes abundan las psicosis sintomticas esquizofreniformes. A medida que la edad es ms avanzada, aumenta la frecuencia del
sndrome de Korsakow. Generalmente el EEG no se altera en las psicosis endgenas, mientras que a menudo muestra anomalas
persistentes de tipo difuso o central en las psicosis sintomticas, las alteraciones EEG en las psicosis sintomticas carecen de
especificacidad, varan segn las oscilaciones del nivel de conciencia. A medida que progresa el compromiso de conciencia aumenta la
desorganizacin del trazado y la lentificacin y la amplitud de la actividad bioelctrica. PSICOSIS EXOGENAS AGUDAS: Se refiere a las
perturbaciones psiquitricas agudas que aparecen en relacin directa con alteraciones somticas. Dichas alteraciones somticas son de
la ms diversa ndole: afecciones cerebrales agudas, traumatismo y heridas cerebrales; epilepsia, enfermedades generales (infecciones,
intoxicaciones, anemias, etc.); trastornos endocrinos; puerperio; intoxicaciones por sustancias extraas al organismo (alcohol,
anfetaminas, etc.). Para que se produzca la perturbacin psiquitrica, las alteraciones mencionadas o las noxas que de ella provengan
deben actuar sobre el cerebro, de manera directa o indirecta. Bonhffer estableci los fundamentos del concepto de psicosis exgena y
demostr que los cuadros psicopatolgicos resultantes se reducen a un escaso nmero a los que denomin "tipos de reaccin exgena
aguda", destacando los siguientes sndromes o "formas de presentacin": estado delirioso, epileptoide-angustioso, crepuscular,
amencial, alucinsico y estuporoso. El concepto de tipos de reaccin exgena aguda lleva implcita la idea de una falta de especificidad
en la relacin existente entre la naturaleza de la enfermedad somtica y el cuadro psicopatolgico resultante. Esto significa que
enfermedades corporales completamente diferentes pueden dar lugar al mismo cuadro psicopatolgico. El sntoma central de estos
trastornos es el enturbiamiento de conciencia en sus diversos grados (excepto en el caso de la alucinosis). Bonhoffer tambin mencion
las "formas de transcurso" de las reacciones exgenas agudas, es decir, conjunto de sntomas enmascarados entre los otros,
pertenecientes al sndrome hiperestsico emocional o al sndrome de Korsakow. El sndrome hiperestsico-emocional incluye: "aspecto
fatigado y marchito, color pajizo de la cara, hiperestesia a los ruidos, a la luz y el dolor, tristeza, mal humor, cefalea, mareos, astenia de
predominio vesperal y nocturno, labilidad de la memoria, labilidad emotiva, susceptibilidad a flor de piel, incapacidad de concentrarse,
tendencia marcada a las autorreferencias paranoicas las cuales nunca se configuran acabadamente, insomnio o sueo muy
interrumpido, no reparador y con actividad onrica laboral o terrorfica. El sndromde Korsakow est constituido por: desorientacin en
tiempo y lugar, falso reconocimiento y fabulacin de perplejidad. DIAGNOSTICO: Estos cuadros se inician bruscamente, aunque pueden
no ser evidentes desde el principio cuando son de ligera intensidad. Generalmente son reversibles si remite la patologa subyacente,
pero algunos pueden evolucionar hacia un sndrome orgnico crnico, como cuando una psicosis post traumtica aguda deja al
descubierto una demencia o cuando la encefalopata de Wernicke se transforma en un sndrome amnsico duradero. Los cuadros
clnicos resultantes se deben fundamentalmente a la perturbacin de la funcin cerebral normal por alteraciones bioqumicas,
elctricas o mecnicas. Los sntomas comunes ms importantes son los siguientes: Alteracin del nivel de conciencia, alteraciones
psicomotoras, trastornos del pensamiento, alteraciones de la memoria, alteraciones perceptivas, alteraciones emocionales. Otras
manifestaciones que aparecen en estadas poco intensas de la enfermedad: sntomas neurotiformes de apariencia depresiva,
hipocondraca, fbica, histrinica, paranoide o esquizofreniforme. Los trastornos cuantitativos de conciencia corresponden a la
MANUAL DE TRABAJO DEL CURSO ENARM CMN SIGLO XXI
CURSO ENARM CMN SIGLO XXI TEL: 36246001 Pharmed Solutions Institute PGINA 267

obnubilacin, que agrupa los cuatro trastornos cuantitativos, desde el ms leve al ms grave: embotamiento (disminucin o retardo en
el ritmo de las elaboraciones psquicas. Fatigable; la captacin de los estmulos es trabajosa porque la percepcin es lenta, imperfecta,
imprecisa y carente de nitidez; se altera la memoria), somnolencia (tendencia al sueo, pesadez, tiende a quedarse en la cama y
dormitar en el da. Hay disminucin del ritmo alfa al EEG) o adormecimiento, sopor (slo logra despertarse parcialmente, reacciones
psicomotoras rudimentarias y presenta respuestas reflejas. El EEG muestra ondas delta) y coma (EEG tiende a ser isoelctrico). El
trmino obnubilacin significa que la conciencia est nublada o empaada en relacin con la funcin de alerta. TRASTORNOS
CUALITATIVOS DE CONCIENCIA: 1.- ESTADO CREPUSCULAR: Lo ms caracterstico de este trastorno es circunscribirse slo a ciertas
manifestaciones de la vida psquica, durante el cual el paciente es cogido por un afecto exaltado que tie de tal manera el campo de su
vivenciar que da la impresin de que toda la actividad psquica del sujeto se concentra slo en aquello que tiene relacin con su rabia,
su angustia, su odio o su xtasis. Presenta ilusiones y alucinaciones, principalmente visuales aunque tambin auditivas. El afecto
desmandado puede llevar al enfermo a incurrir en actos de violencia o a estados beatficos de xtasis. Los estados crepusculares en
general son breves, de minutos hasta das, raramente duran semanas o meses. Debido al fondo obnubilatorio presente en todo estado
crepuscular, hay amensia parcial o total del episodio vivido, pero mientras est dentro de l, el enfermo recuerda lo que ha sucedido.
Se tiende a denominar estado crepuscular epilptico a todo cuadro psictico epilptico con compromiso de conciencia. La psicosis
epilptica que presenta fenmenos deliriosos no es un estado crepuscular epilptico, sino un estado delirioso epilptico. Los estados
crepusculares aparecen con mayor frecuencia en la epilepsia, en las psicosis sintomticas y en la histeria. La embriaguez patolgica es la
forma de estado crepuscular ms frecuente de las psicosis sintomticas, y se presentan generalmente en epilpticos, orgnicos o
psicpatas, con dosis pequeas de alcohol. Son estados crepusculares desorientados acompaados de agitacin psicomotora, con
amnesia posterior. Todas las causas de hipoxia cerebral pueden provocar un estado crepuscular. Los estados crepusculares de causa
orgnica son desorientados. Pueden ser debidos a repercusiones patolgicas de las crisis epilpticas, el edema cerebral, el proceso
fundamental del cerebro, o bien a dosis excesivas de anticonvulsivantes. 2.- LA AMENCIA: Hay formas alucinatorias, catatnica e
ideofugal-incoherente. La obnubilacin es un embotamiento leve. La psicomotrocidad da la impresin de intencionalidad, aunque los
actos son muy cambiantes e incompletos. El pensamiento es incoherente y la percepcin de la realidad dificultosa. El sentimiento que
coge al paciente es de miedo, desamparo y angustia, que impregna todo el cuadro clnico. El miedo expectante lo lleva a sobresaltarse
al primer ruido, originando as percepciones deliriosas paranodeas. El desamparo se observa en la expresin llorosa y los sentimientos
de soledad y abandono mencionados por el paciente. El enfermo est muy angustiado y perplejo, presentando de forma caracterstica
cambios afectivos bruscos. A ratos da la impresin de que se sumerge en un mundo de vivencias deliriosas, alucinaciones y falsos
reconocimientos, todo esto vivido con gran sentimiento de realidad, que lo lleva a arrancar, defenderse, gritar, hacer movimientos
bruscos, tornarse negativista. Dentro del cuadro, el enfermo recuerda difcilmente lo sucedido horas antes o el da anterior; una vez
pasado el episodio puede haber amnesia completa o recuerdos fragmentarios y borrosos. Dura semanas o meses. 3.- ALUCINOSIS: Se
caracteriza por la presencia de alucinaciones auditivas en forma de voces, un delirio persecutorio, en directa relacin con las voces,
generalmente no se presenta alteracin de conciencia (el paciente est orientado, atento y reflexico). El contenido de las voces es
habitualmente de reproche y condenacin, lo que en una atmsfera de intensa angustia lleva muchas veces al paciente al suicidio.
Tambin se presentan voces que discuten entre s, que comentan los propios actos o repiten lo que el enfermo piensa. Habitualmente
se presenta en el alcoholismo y puede durar das o semanas, pero tambin puede orientarse hacia la cronicidad. Conviene subrayar
algunas diferencias entre la alucinosis alcohlica y el delirum tremens: 1.- El delirium tremens generalmente es desencadenado por
abstinencia previa. La alucinosis suele desencadenarse por un aumento del consumo. 2.- En el delirium tremens hay alteracin de
conciencia y la alucinosis aparece con conciencia clara. 3.- En el delirium tremens las alucinaciones son principalmente visuales y en la
alucinosis hay alucinaciones auditivas. 4.- El correlato somtico agudo del delirium tremens no se encuentra presente en la alucinosis.
La alucinosis tambin es conocida como "delirio alucinsico de los bebedores", de Kraepelin o "alucinosis aguda de los bebedores" de
Wernicke. 4.- EL ESTADO EPILEPTOIDE ANGUSTIOSO: Para Roa; "el ncleo de la exacerbacin lo forma casi siempre un sentimiento
indefinible de miedo, desamparo o soledad, con oscurecimiento de la situacin y viva peticin mmica de ayuda. El enfermo demanda
auxilio con las manos, se reclina sobre alguien, abre desmesuradamente los ojos, emite algunas interjecciones ansiosas, palidece o
enrojece y se recupera a los pocos minutos. En lugar de peticin anhelante de ayuda puede haber violencia y agresividad. La
exacerbacin, que puede terminar en una crisis convulsiva de acento epilptico, puede repetirse muchas veces, sobre todo en
presencia de terceros. El estado bsico entre las exacerbaciones es de euforia banal, con precipitacin en las respuestas, sentimiento
ntimo de depresin, falta de concentracin y quizs cierta pesadez corporal. Este estado dura das, semanas o meses. A veces precede
a los primeros ataques de la epilepsia genuina, acompaa los tumores enceflicos o matiza el curso de la histeria y la esquizofrenia". 5.-
EL ESTADO ESTUPOROSO: El estado estuporoso con compromiso de conciencia es una de las formas de manifestacin de un estado
delirioso o crepuscular, en su inicio o durante el transcurso. "los estados estuporosos se dan con claridad u oscuridad de conciencia. El
primer caso es muy propio de la esquizofrenia catatnica". Los enfermos que quedan de pie, sentados o acostados, ya en posiciones
fijas, ya acomodndose ligeramente de vez en cuando y sin obedecer rdenes. Algunos caen en gatismo urinario y fecal, otros en
retencin y unos terceros evacuan si son llevados al retrete. No rara vez el estado estuporoso es el comienzo de un estado delirioso,
crepuescular, amencial o uno de los momentos del curso de dichos estados. 6.- EL DELIRIO AGUDO O DELIRIUM: El delirium es el
sndrome psiquitrico orgnico ms frecuentemente observado por psiquiatras y por otros mdicos. Se presenta en aproximadamente
el 10% de todos los pacientes ingresados a servicios mdicos y quirrgicos agudos, y se cree que su incidencia es an mayor. Se ha
informado de cifras de hasta 80% en los servicios geritricos agudos. El trmino sugiere un descarrilamiento o separacin de una ruta o
va, es decir, una desviacin brusca del estado habitual del individuo. El DSM III R lo denomina delirium, pero tambin se ha propuesto
los siguientes trminos: estado confusional agudo, psicosis txica, encefalopata metablica, enfermedad metablica cerebral exgena,
sndrome de insuficiencia cerebral, sndrome cerebral agudo y estado disergstico. TRATAMIENTO: Manejo de antipsicticos casos
leves (olanzapina, risperidona), en casos graves (haloperidol, levopromcina). Bsqueda de la causa y manejo de la misma.


CASOS CLINICOS


MANUAL DE TRABAJO DEL CURSO ENARM CMN SIGLO XXI
CURSO ENARM CMN SIGLO XXI TEL: 36246001 Pharmed Solutions Institute PGINA 268

SUICIDIO:
CIENCIAS BASICAS: Diversos estudios dan cuenta del incremento de suicidios relacionados con la vejez en muchos pases, sobre todo
los industrializados. Son factores de riesgo la ancianidad, el sexo masculino, la raza blanca, una historia familiar de suicidios, los
conflictos familiares y de vidas severas y recientes, los trastornos de personalidad y la enfermedad mental. SALUD PUBLICA: En relacin
con la gente joven se cita que la proporcin de suicidas entre ancianos es 50% mayor. Adems, las autopsias psicolgicas de las
vctimas revelan que ms de 90% presentaba un trastorno psiquitrico en el momento de su muerte. ETIOLOGIA: Las causas habituales
para suicidio son: los trastornos afectivos; de ellos, el ms frecuente es la depresin, con una prevalencia aproximada de 50-60%; le
siguen los trastornos por abuso de sustancias 19-60% y la esquizofrenia 2-13%. Tambin se cita el trastorno de personalidad como
factor de riesgo para suicidio. Con respecto a este ltimo los rasgos caracterolgicos conectados con la conducta suicida son rigidez,
autosuficiencia, dificultad para aceptar la ayuda de los dems y narcisismo. De los problemas existenciales, relacionados con el suicidio,
se describen diferencias entre los jvenes y los viejos. En los primeros los principales factores desencadenantes de estrs, previos al
suicidio, estn relacionados con el trabajo, las finanzas y las dificultades en las relaciones interpersonales. Mientras tanto, entre los
adultos mayores el suicidio se comete en el contexto de enfermedades mdicas, generalmente crnicas o invalidantes y prdidas. El
suicidio en esta etapa de la vida puede ser silente, expresado mediante conductas pre suicidas como dejar de comer o medicarse. Sin
embargo, es comn el acto impremeditado e impulsivo, en ocasiones precipitado por acontecimientos menores que no alertan a la
familia. Es usual que el suicida elija mtodos drsticos tales como, el ahorcamiento o las armas de fuego. Tambin hay quienes antes
del suicidio, mientras estn deprimidos, cometen homicidios: para estos casos se cita una incidencia de 0.4-0.9 por 100, 000. Lo
referido destaca la importancia del reconocimiento precoz y el tratamiento oportuno del trastorno afectivo, adems de la adopcin de
medidas preventivas. Por ltimo, es necesario subrayar que cerca de un tercio o la mitad de los suicidas hicieron contacto con
psiquatras antes del acto, y que aproximadamente tres cuartas partes de ellos lo hicieron con un mdico general en el mes previo al
acto, u horas antes de suicidarse. La ideacin suicida supone entre el 5-20% de las consultas psiquiatricas urgentes (media=10%). 1% de
admisiones a urgencias en hospitales generales. El 2% fallecen tras la tentativa. El suicidio supone 8,5 por mil fallecidos. Mtodos:
Suicidio consumado: ahorcamiento, precipitacin, ahogamiento. Tentativas: 90% son sobredosis farmacolgicas. Mas intentos de
suicidio en mujeres (9/1), ms suicidios consumados en hombres (3/1). DIAGNOSTICO: Evaluacion de riesgo suicida de (SAD person):
El paciente tiene ideacin suicida?: SI valorar (cada item positivo de los siguientes vale 1 punto): Sexo: varn. Edad: < 19 > 45 aos.
Depresin clnica. Intentos de suicidio previos. Abuso de alcohol. Trastornos cognitivos. Bajo soporte social. Plan organizado de suicidio.
Sin pareja estable. Enfermedad somtica. TRATAMIENTO: Modalidad de intervencin: 0-2: Alta al domicilio con seguimiento
ambulatorio. 3-4: Seguimiento ambulatorio intensivo. Considrese el ingreso. 5-6: Hospitalizacin si no hay control familiar adecuado.
7-10: Hospitalizacin. Elevado riesgo de suicidio. Abordaje verbal del paciente suicida General: Preocuparse por las necesidades del
paciente y transmitir que se intenta ayudarle. Identificarse y explicar los objetivos. Mostrarse sereno, firme, seguro y honesto. Evitar
conductas hostiles, desafiantes, amenazantes o irrespetuosas. Especfico (hablar de la problemtica suicida). Cuidar la
contratransferencia (evitar o controlar reacciones de angustia, preocupacin, hostilidad). Insistir en la dimensin temporal (la tendencia
suicida como crisis delimitada en el tiempo). Resaltar la ambivalencia de toda conducta (vivir vs.o morir como cuestin normal).
Intervencin farmacolgica a corto plazo en el paciente suicida. Tratamiento especfico a corto plazo: Ansiedad: Benzodiacepinas de
vida media media/larga (diazepam). Insomnio: Benzodiacepinas vida media corta (Lorazepam). No utilizar frmacos potencialmente
letales. No utilizar antidepresivos en urgencias (incrementan el riesgo de suicidio)

CASO CLINICO
Ingresa a urgencias femenino de 27 aos la cual se encuentra bajo tratamiento por trastorno de personalidad limite, ha sido tratada por
mltiples psiquiatras y esquemas teraputicos, al ingresar el esposo de la paciente la encontr en la baera con cortes superficiales en
ambas muecas y falta de respuesta a los estmulos sin embargo respira superficialmente, en la sala de urgencias usted identifica
respiracin lenta y superficial sin percibir olores especficos, observa ROTs disminuidos, con respuesta ha estmulos dolorosos, su
tratamiento actual es clonacepam, clozapina y citalopram.

PREGUNTA
Considerando el manejo farmacolgico cual sera la mejor intervencin para revertir el efecto de una dosis excesiva ingerida en este
intento suicida.

RESPUESTA
a.- La administracin de naloxona est indicada por polifarmacia.
b.- Un agonista antagonista parcial competitivo como flumacenil.
c.- Naltrexona es un frmaco que revierte la dosificacin de benzodiacepinas.
d.- La paciente debe recibir biperiden para revertir los efectos de clozapina.







MANUAL DE TRABAJO DEL CURSO ENARM CMN SIGLO XXI
CURSO ENARM CMN SIGLO XXI TEL: 36246001 Pharmed Solutions Institute PGINA 269





MANUAL DE TRABAJO DEL CURSO ENARM CMN SIGLO XXI
CURSO ENARM CMN SIGLO XXI TEL: 36246001 Pharmed Solutions Institute PGINA 270







GINECOLOGIA Y OBSTETRICIA

1. PLANIFICACION FAMILIAR, ANTICONCEPCION, INFERTILIDAD, ESTERILIDAD.
2. AMENORREA PRIMARIA Y SECUNDARIA, ENDOMETRIOSIS.
3. CERVICOVAGINITIS, ENFERMEDAD PELVICA INFLAMATORIA, ENFERMEDADES DE TRANSMISION SEXUAL.
4. INFECCION POR VIRUS DEL PAPILOMA HUMANO. DISPLASIAS, CANCER CERVICOUTERINO Y OVARICO.
5. SANGRADO UTERINO ANORMAL Y DISFUNCIONAL. CANCER ENDOMETRIAL, SINDROME DE OVARIO POLIQUISTICO.
6. MASTOPATIA FIBROQUISTICA, CANCER DE MAMA, MENOPAUSIA Y CLIMATERIO
7. CONTROL PRENATAL Y DEL EMBARAZO.
8. EMBARAZO ECTOPICO. AMENAZA DE ABORTO, ABORTO, INCOMPETENCIA CERVICAL.
9. HIPEREMESIS GRAVIDICA, ENFERMEDAD TROFOBLASTICA.
10. HIPERTENSION ARTERIAL, ENFERMEDAD HIPERTENSIVA DEL EMBARAZO, PREECLAMPSIA Y ECLAMPSIA.
11. DIABETES MELLITUS, INTOLERANCIA A LA GLUCOSA, DIABETES GESTACIONAL.
12. POLIHIDRAMNIOS, OLIGOHIDRAMNIOS, PLACENTA PREVIA, DESPRENDIMIENTO DE PLACENTA.
13. ANEMIAS, IVU, PARTO PRETERMINO, PARTO NORMAL Y CESAREA.
14. COMPLICACIONES DEL PUERPERIO, HEMORRAGIA OBSTETRICA, PLACENTA ACRETA.
15. SINDROME DE TENSION PREMENSTRUAL, PSICOSIS POSPARTO Y DEPRESION POSTPARTO.











































MANUAL DE TRABAJO DEL CURSO ENARM CMN SIGLO XXI
CURSO ENARM CMN SIGLO XXI TEL: 36246001 Pharmed Solutions Institute PGINA 271


PLANIFICACION FAMILIAR, ANTICONCEPCION Y CONTRACEPCION:
CIENCIAS BASICAS: Planificacin familiar: es el derecho de toda persona a decidir de manera libre y responsable e informada sobre el
nmero y espaciamiento de sus hijos y a obtener la informacin especializada y los servicios idneos. Mtodos anticonceptivos:
procedimientos que se utilizan para impedir el embarazo en forma temporal o permanente y se clasifican de la siguiente manera;
Temporales hormonales (orales, inyectables, parches) y no hormonales (barrera, naturales, DIU). Definitivos vasectoma, salpingoclasia.
CRITERIOS para iniciacin y seguimiento de todos los mtodos anticonceptivos; 1. Conocer la eficacia del mtodo. 2. Ventajas y
desventajas. 3. Efectos colaterales y complicaciones. 4. Prevencin de ETS. 5. seguimiento
METODOS ANTICONCEPTIVOS
CLASIFICACION METODO DESCRIPCION COMO FUNCIONA EFICACIA OBSERVACIONES
Temporal
hormonal
oral
Orales
combinados

Estrgeno (etinilestradiol o
mestranol) y progestgeno
(norentindrona o acetato de
norentindrona o noretinodrel o
norgestrel o levonorgestrel)
Evita la liberacin de
vulos por los ovarios
(ovulacin)
>99% (uso
correcto y
sostenido)
92% como se
usa
comnmente
Disminuye el riesgo de cncer endometrial
y ovrico: no tomar en la lactancia
Reduccin de la frecuencia de spotting
Disminucin del riesgo de EPI (cambia la
consistencia del moco y protege)
No usar en Ca de mama, endometrio,
CaCu, hepatopatas, varices extensas,
mujeres de 35-40aos
Temporal
hormonal
oral
Orales
minipastillas

Contiene nicamente
progesterona (levonorgestrel),
minidosis 350mcg
Hace ms espeso el moco
del cuello uterino, lo que
impide entrada de
espermatozoides y
previene la ovulacin.
Modifican endometrio
para disminuir
probabilidades de
implantacin
>99% (uso
correcto y
sostenido)
92% como se
usa
comnmente
Puede usarse durante la lactancia: tomar
todos los das a la misma hora
Pueden inducir amenorrea a lo largo del
tiempo
Temporal
hormonal
oral
Anticoncepcin
emergencia
Levonorgestrel 1,5 mg. Se
toman para prevenir embarazo,
hasta 5 das despus de la
relacin sexual sin proteccin
Evita la ovulacin
1ra dosis dentro de las 72
hrs poscoito y 2da dosis,
12 hrs despus de la
primera
Reduce de un
60% as un
90% el
embarazo
No altera el embarazo si este ya se ha
producido
No protegen de ETS. Causan ms efectos
colaterales
Temporal
hormonal
Implantes
Subdrmicos y

Cilindros (silastic,
etonorgestrel)o capsulas
pequeas y flexibles que se
colocan debajo de la piel del
brazo, contienen nicamente
progestgeno
Mismo mecanismo de las
pastillas de progestgeno
>99% Debe ser insertado y extrado por personal
sanitario, se puede usar por 3-5 aos,
segn el tipo. Puede generar spotting
Temporal
hormonal
Parches Sistema trasndrmico
(norelgestrinoma y
etinilestradiol)
Se cabia cada semana x 3 sem,
la sem 4 no se utiliza
>99% Menor perdida sangunea, ciclos
mentsruales predecibles, disminuye acn,
hirsutismo, disminuye riesgo de ca de
ovario y de endometrio. Contraindicado en
flebitis, enf. Coronaria, EVC, ca de mama,
tumores heptico, embarazo
Temporal
hormonal
inyectable
Inyectable de
progestgeno

IM profunda cada 2-3 meses.
Se inicia en los primeros 7 das
del ciclo. Enantato de
noretisterona
200mg/1ml=c/2meses. Acetato
de medroxiprogesterona=c/3
meses150mgs/3ml
Mismo mecanismo de
pastillas de progestgeno
solo
>99% Al cesar el uso, la fecundidad tarda en
reaparecer (1-4meses); Puede haber
spotting. Indicado en nuligestas, nulparas
o multparas, antes del 1er embarazo aun
cuando estn contraindicados los
estrgenos (VPH), purperas lactando
(usar despus de 6ta sem).
Temporal
hormonal
inyectable
Inyectable
mensual
IM profunda cada mes, se
inicia en primeros 5 das del
ciclo, contiene estrgeno y
progestgeno (Cipronato de
estradial +Acetato de
Medroxiprogesterona.
Valerianato de estradiol+
Enantato de norestisterona)
Mismo mecanismo de
anticonceptivos orales en
combinacin. Su
proteccin anticonceptiva
se extiende hasta 33 das
>99% (uso
correcto y
sostenido)
97% como se
usa
comnmente
Por ser de depsitos tienen ms efectos
adversos
Hemorragias vaginales comunes. Indicado
en nuligestas, nulparas y multparas, antes
de 1er embarazo, posaborto, postparto y
postcesarea, si no est lactando.
Contraindicado en la lactancia en primeros
6 meses posparto, embarazo o sospecha
de l, en ca mamario o de crvix, enf.
Tromboembolica, hepatopatas
Temporal no
hormonal
Dispositivo
intrauterino
(DIU)
Dispositivo de plstico flexible y
pequeo que contienen una
asa o cubierta de cobre y se
inserta en el tero
El cobre daa los
espermatozoides e
impiden que se junten
con el ovulo
>99% Disminuye la frecuencia del clico
menstrual y los sntomas de endometriosis
amenorrea, en un grupo de usuarias
Temporal no
hormonal de
barrera
Condn
masculino
Vaina o cubierta que envuelve
el pene erecto
Forma una barrera que
impide el encuentro de
los espermatozoides con
el ovulo
98% usado de
manera
correcta y
sostenida
85% como se
usa
Tambin protege de las ETS, en particular
la causada por VIH
Temporal no
hormonal de
barrera
Condn
femenino
Vaina o forro que se adapta
holgadamente a la vagina; esta
hecho de un material plstico
transparente, fino y suave
Forma una barrera que
impide que los
espermatozoides y el
ovulo se junten
90% Tambin protege de las ITS, en particular la
causada por VIH
Temporal no
hormonal de
barrera
Diafragma
(capuchn de
ltex blando)
Cubre el crvix. Tambin hay
diafragmas de plstico y
silicona disponibles. El borde
contiene un resorte firme y
Forma una barrera que
impide que los
espermatozoides y el
ovulo se junten
90% El riesgo de embarazo aumenta, si no se
usan espermaticidas juntos. No interfiere
la lactancia. Puede introducirse hasta 6 hrs
antes. Puede brindar cierta proteccin
MANUAL DE TRABAJO DEL CURSO ENARM CMN SIGLO XXI
CURSO ENARM CMN SIGLO XXI TEL: 36246001 Pharmed Solutions Institute PGINA 272

flexible que mantiene el
diafragma en su lugar
contra algunas ITS. Extremadamente
raros: sndrome de shock txico
Temporal no
hormonal de
barrera
Espermaticida

Nonoxinol-9.Detergentes no
inicos. Los espermicidas se
introducen profundamente en
la vagina poco antes de tener
sexo.
Funcionan rompiendo la
membrana de las clulas
del esperma, matndolas
o enlenteciendo su
movimiento.
70-80% Uso correcto en cada acto sexual. Pueden
utilizarse como mtodo principal o como
mtodo de respaldo. Pueden irritar el pene
o la vagina, alergia local. Favorecen IVU.
No usar pacientes con SIDA o VIH.
Temporal no
hormonal
natural
Coito
interrumpido
El hombre retira el pene fuera
de la vagina y eyacula fuera de
esta, con lo que el semen no
tienen contacto con los
genitales externos de la mujer
Evita que los
espermatozoides
penetren en la vagina,
con lo que se evita la
fecundacin
96% uso de
manera
correcta
73% como se
usa
Es uno de los mtodos menos eficaces,
porque a menudo resulta difcil determinar
el momento, preciso de la retirada
Temporal no
hormonal
natural
Mtodos que
requieren
conocimiento
de la
fecundidad y/o
abstinencia
peridica
Mtodo de calendario (Ogino-
Knauss): temperatura corporal
basal (TCB), mtodo del moco
cervical (Billings), sintotrmico
(TCB+moco cervical)
La pareja evita tener
embarazo, evitando tener
coito, sin proteccin
durante los das fecundos
generalmente, mediante
la abstinencia
70-80% uso
de manera
correcta
63% como se
usa
comnmente
El uso correcto y regular exige la
colaboracin del varn. Cuando hay
infeccin vaginal, es difcil interpretar el
moco. Por lo menos hay que observar 6
ciclos menstruales, mientras abstinencia o
mtodo de barrera. De las 2 fases del ciclo
menstrual, solo la 2da es la constante
Temporal no
hormonal
natural
Amenorrea en
la lactancia
Proteccin prolactina. Para
mujeres recin paridas que no
han vuelto a menstruar, exige
el amamantamiento exclusivo
da y noche, de una criatura
menor de 6 meses
Impide que los ovarios
liberan vulos (ovulacin)
99% si se
aplica de
manera
correcta
98% como se
usa
comnmente
Esta basado en el efecto natural del
amamantamiento sobre la fecundidad
Definitivo Vasectoma Se bloquean o cortan los
conductos deferentes que
transportan los
espermatozoides desde los
testculos
Impide que haya
espermatozoides en el
semen eyaculado
>99%.
Despus de la
evaluacin
del semen a
los 3 meses
Tarda en actuar unos 3 meses debido a
que quedan espermatozoides
almacenados; no afecta el funcionamiento
sexual del hombre
Definitivo Salpingoclasia Se bloquean o cortan las
trompas de Falopio
Los vulos no pueden
juntarse con los
espermatozoides
>99% Es fundamental que sea una eleccin
voluntaria y con conocimiento de causa

CASO CLINICO
Se trata de paciente femenino de 23 aos de edad la cual acude a consulta para planificacin familiar, solicita un mtodo
anticonceptivo, refiere estar casada desde hace 3 meses y desea no embarazarse por lo menos en 3 aos ms, niega antecedentes
heredofamiliares como hipertensin, diabetes mellitus, as como otras relevante, al interrogatorio niega tabaquismo y alcoholismo, no
toma medicamentos, a la exploracin fsica no se agregan datos de relevancia, IMC 24, TA 100/65 mmHg, FC 78, FR 19. Aparentemente
la paciente se encuentra sana.

PREGUNTA
La paciente solicita un mtodo anticonceptivo oral, cul sera el mtodo de contracepcin ms apropiado para esta paciente desde el
enfoque de riego, as como en funcin del tiempo de espera que quiere para embarazarse.

RESPUESTA
a.- No hay contraindicacin para el mtodo anticonceptivo oral por carencia de factores de riesgo.
b.- No hay contraindicacin para el mtodo anticonceptivo oral ya que es seguro y efectivo por el tiempo que desea usarlo.
c.- Igualmente podra usarse anticonceptivo inyectable mensual o trimestral.
d.- Considerando su edad y inicio de vida sexual activa, un mtodo de barrera sera la mejor opcin.

CASO CLINICO
Paciente femenino de 28 aos de edad la cual se encuentra en pos-aborto de 5 dias, refiere que ya no presenta sangrado, acude a
solicitar mtodo anticonceptico, refiere que no quiere tener mas hijos por lo menos en 3 aos, tiene 2 hijos, anteriormente empleaba
hormonales orales y durante este mtodo se embarazo, no cuenta con antecedentes de cervicovaginitis ni EPI.

PREGUNTA
Cual de los siguientes mtodos de planificacin familiar es el mas adecuado para este caso?

RESPUESTA
a.- Hormonal inyectado.
b.- Implante hormonal.
c.- Presenvativo.
d.- Dispositivo intrauterino.

PREGUNTA
La paciente acude ha consulta por cambios del flujo menstrual, el cual refiere es obscuro y ha presentado manchado intermenstrual, a
la exploracin se obseva moco cervical con sangre moderada, cual es la conducta a seguir?

RESPUESTA
a.- Retirar el DIU.
MANUAL DE TRABAJO DEL CURSO ENARM CMN SIGLO XXI
CURSO ENARM CMN SIGLO XXI TEL: 36246001 Pharmed Solutions Institute PGINA 273

b.- Indicar un AINES.
c.- Indicar antibitico.
d.- Indicar DOC.

PREGUNTA
La paciente acude 5 aos despus por flujo abundante con olor desagradable, a la exploracin se observa un proceso compatible con
cervicovaginitis, agrega que hace un ao se divorcio por infidelidad por parte del esposo y actualmente ya tiene otra pareja, cual es la
conducta a seguir ms adecuda.

RESPUESTA
a.- Retirar el DIU.
b.- Cambio de mtodo a hormonal.
c.- Indicar acido mefenamico.
d.- Indicar antibioticoterapia y preservativo.

PREGUNTA
La paciente acude 6 meses despus por dolor abdominal en fosa iliaca izquierda que se irradia a hipocondrio y hacia tras, a la palpacin
se presenta dolor abdominal de predominio plvico, acude adems porque su DOC presento cambios inflamatorios inespecficos, cual
es la conducta a seguir por en este caso?

RESPUESTA
a.- Retirar DIU por probable EPI.
b.- Retirar DIU por resultado de DOC.
c.- Retirar DIU por probable embarazo.
d.- Retirar DIU por cambio de mtodo.

CASO CLINICO
Se trata de paciente femenino de 22 aos de edad la cual acude a solicitar mtodo de planeacin familiar, actualmente vive en casa con
sus padres, refiere un IVSA a los 20 aos, agrega que ha utilizado preservativo pero lo considera con fallas, adems no desea que sus
familiares se enteren que tiene vida sexual activa, por otro lado menciona que mantiene una relacin de dos aos de duracin y no
desea embarazarse, a la exploracin fsica se observa con leve sobrepeso, con hirsutimo y acn presente, refiere que sus periodos
menstruales son irregulares.

PREGUNTA
Considerando la solicitud de la paciente y su juicio clnico cual sera su conducta a seguir:

RESPUESTA
a.- Considera suficiente la informacin y hallazgos para indicarle el mtodo solicitado.
b.- Considera que necesita mayor informacin por lo que enva a realizar un USG.
c.- Considera que debe realizar un PIE por la irregularidad menstrual.
d.- Considera que debe esperar el prximo periodo para tomar la decisin con la paciente.

CASO CLINICO
Masculino de 31 aos de edad el cual acude para realizarse vasectoma, refiere que tiene una paternidad satisfecha, a la exploracin
clnica se observa aparentemente sano, a la exploracin genital se observa bolsa escrotal con aspecto normal, a la palpacin hay leve
dolor y se aprecia una masa irregular depresible, tortuosa, refiere el paciente que desde los 25 aos se haba percatado, sin embargo
no le prest importancia solo ocasionalmente le duele, principalmente cuando camina.

PREGUNTA
Considerando las manifestaciones clnicas, cual es la conducta a seguir respecto al procedimiento que solicita el paciente.

RESPUESTAS
a.- No presenta contraindicacin de relevancia para poder realizar la vasectoma.
b.- Debe descartarse un proceso neoplasico antes de realizar la vasectoma.
c.- Debe realizar un USG para descartar un varicocele antes de realizar la vasectoma.
d.- Debe cuantificar factores tumorales antes de realizar la vasectoma.

CASO CLINICO
Femenino de 21 aos de edad la cual se encuentra en procedimiento cesarea transoperatorio, actualmente se realiza el procedimiento
por cesarea previa y periodo intergenesico de 10 meses, su primer embarazo fue a los 16 aos el cual fue interrumpido mediante
legrado, actualmente es su tercer hijo y refiere maternidad satisfecha al preguntarle, sin embargo refiere que es muy joven aun, y
podra utilizar un dispositivo intrauterino.

PREGUNTA
Considerando el caso y tomando en cuenta la condicin quirrgica en la que se encuentra la paciente, cul sera la mejor opcin de
esta.
MANUAL DE TRABAJO DEL CURSO ENARM CMN SIGLO XXI
CURSO ENARM CMN SIGLO XXI TEL: 36246001 Pharmed Solutions Institute PGINA 274


RESPUESTA
a.- Solicitar autorizacin y colocar DIU ya que la paciente es joven aun.
b.- Solicitar autorizacin y realizar OTB ya que la paciente tiene riesgo obsttrico.
c.- Le explica brevemente las ventajas y la dice que podra pensarlo.
d.- El GO contina el procedimiento y la podra realizarse OTB por endoscopia posteriormente.

INFERTILIDAD, ESTERILIDAD. CIENCIAS BASICAS: Esterilidad; incapacidad de una pareja para lograr la cocenceppcion. La esterilidad se
clasifica en: primaria (cuando la pareja, tras un ao de relaciones sin tomar medidas de proteccin, no ha conseguido un embarazo) y
secundaria (la de la pareja que, tras la consecucin del primer hijo, no logra una nueva gestacin tras 2 o ms aos de intentarlo). El
tiempo mnimo a partir del cual se habla de esterilidad se fija en un ao de relaciones sexuales con deseo de descendencia. Infertilidad;
es la incapacidad para producir un hijo vivo. As, se denomina infertilidad primaria la que padece una pareja que consigue una gestacin
que no llega a trmino con un recin nacido normal, e infertilidad secundaria cuando, tras un embarazo y parto normales, no se
consigue una nueva gestacin a trmino con recin nacido normal. Infecundidad; incapacidad de una pareja de llevar a termino un
embarazo. Fertilidad humana: En comparacin con otras especies animales, el proceso reproductivo en humanos resulta altamente
ineficaz. Existe un momento de mxima fecundidad en los primeros meses (alrededor de 20-30% de embarazos por ciclo) de forma que
al cabo de tres ciclos, dos terceras partes de las mujeres han logrado el embarazo. Dicha fecundidad se reduce paulatinamente en ciclos
sucesivos. Globalmente, la tasa acumulada de embarazo entre parejas de fecundidad probada es del 90% aproximadamente a los 12
meses y de alrededor del 95% a los 24 meses. SALUD PUBLICA: Aproximadamente nueve de cada diez parejas en edad frtil que
mantienen relaciones sexuales regulares consiguen un embarazo durante el primer ao. Las cifras de prevalencia de esterilidad que
manejan los especialistas sitan entre un 12 y un 20% la imposibilidad de tener un hijo. La edad de la mujer es importante: se estima
que las posibilidades de que una mujer de 20-24 aos no se quede embarazada estn entre el 4-5%. Estas posibilidades aumentan
lentamente con la edad, de tal forma que a los 35 aos estaran alrededor del 14%. Este suave aumento de la esterilidad en la mujer
sufre un brusco cambio a partir de los 35 aos, de tal forma que a los 40 aos la esterilidad en la mujer rondara cifras del 65-70%. Este
incremento se ve agravado por el hecho de que tambin aumentan las tasas de aborto, que a partir de esta edad son aproximadamente
del 71%. La esterilidad e infertilidad suponen una situacin carencial que no compromete la integridad fsica del individuo ni supone un
riesgo vital, sin embargo, dado que la mayora de los adultos contemplan como objetivo vital esencial el tener hijos, esta carencia
puede incidir negativamente en el desarrollo de la persona, produciendo frustracin y desmoralizacin. Por ello suponen un problema
de salud pblica de trascendencia. Las tcnicas de reproduccin humana han alcanzado un auge importante en las dos ltimas dcadas.
PATOGENIA: Un 40% de los casos de esterilidad son de causa masculina: por alteraciones en el mbito testicular, obstruccin de
conductos, patologas en la prstata, alteraciones en la eyaculacin o ereccin y alteraciones en el semen. Otro 40% se debe a
causas femeninas, como la menopausia precoz, la endometriosis, las obstrucciones o lesiones de las trompas de Falopio, anomalas
uterinas y cervicales o los problemas ovulatorios. El 20% restante corresponde a causas mixtas o combinadas, en las cuales ambos
cnyuges son responsables. Aborto de repeticin: Posiblemente pocos problemas ginecolgicos resulten tan frustrantes, tanto para la
pareja como para el mdico, como el aborto de repeticin, equivalente al trmino castellano infertilidad. Para la pareja, por lo
traumtico que resulta la prdida repetida de gestaciones que, en general, se suelen conseguir con facilidad, se malogran sin
justificacin clara y carecen de expectativas satisfactorias de tratamiento en algunas ocasiones. El trmino aborto de repeticin se
aplica en la actualidad a aquellas situaciones en las que se han producido al menos dos abortos consecutivos o ms de dos alternos.
Factor vaginal: las infecciones que se presentan pueden reprimir el coito (por dispareunia), las infeccionescausads por Chamydia
trachomatis, ureaplasma urealyticum, mycpoplasma, traen problemas de esterilidad, ya que ejercen accion nocisa sobre los
espermatozoides, ovulos y embriones.. Factor cervical: incopentencia istimico-cervical. Factor uterino; responsable del 5-10% de los
casos de infertilidad; por presenia de polipos, miomas intracavitarios y algunas de las malformaciones Muellerianas (bidelfo, bicorne).
Factor tubo-peritoneal: responsable de 15-20% de todos los casoso de infertilidad, agenesia , estenosis, a tresias, duplicacione y
orificios accesorios en salpinges. Factor endocrino-ovarico: Responsable del 25-30% de los caosos de infertilidad. Fcator inmunologico:
es bajo solo representa de 3-5% de todos los casos. Las causas que podemos buscar ne el varon son: infecciones, problemas
inmnologicos, tumores, alteraciones neurlgicas, ambientales, toxicas, idiopatica, disfuncion erectil. DIAGNOSTICO DE LA PAREJA
ESTRIL: Se han descrito multitud de pruebas diagnsticas encaminadas al despistaje de cada uno de los factores implicados en la
reproduccin. Por ello, y en un intento de simplificar el camino a estas parejas que tanta carga psquica, fsica y econmica han de
soportar, se ha simplificado al mximo el estudio. Pruebas diagnsticas esenciales son: Evidencia de que la mujer ovula, para lo cual es
suficiente comprobar que menstra cada 28 7 das. Anamnesis e historia clnica para deteccin de enfermedades concomitantes
(diabetes, hipertensin, alteraciones tiroideas, etc.) e historial mdico familiar sobre patologas y enfermedades hereditarias. Evidencia
de una reserva ovrica suficiente: se determina sistemticamente, de forma basal (tercer da del ciclo), la FSH (hormona
folculoestimulante) y el estradiol srico. Deeterminar prolactina, ya que la hierprolactinemia se encuentra entre 10-30% decasos de
infertilidad. Debe encontrase <20ng/ml. Evidencia de que el semen es normal, para lo cual se realiza un seminograma. Evidencia de que
el canal genital es normal, para lo cual basta inicialmente con una histerosalpingografa (HSG) y una ecografa transvaginal (de eleccion
para patolgia ginecologica) que explore tambin los ovarios, en busca de quistes endometrisicos o endometriomas y miomas uterinos
fundamentalmente. La HSG es una prueba molesta, por lo que siempre se realiza tras evidenciar un espermiograma normal o suficiente
para la realizacin de coitos programados o inseminaciones artificiales, muestra alteraciones morfologicas de la cavidad y area corneal.
Durante el screening inicial se evidencia la existencia de las siguientes patologas: Alteraciones en la ovulacin: sndrome de ovarios
poliqusticos, amenorreas de causa central o perifrica, fallos ovricos precoces o menopausia precoz, fallos ovricos ocultos, etc.
Segn la patologa encontrada se completan las pruebas pertinentes: cariotipo, resistencia a la insulina, anticoagulantes circulantes,
autoanticuerpos, etc. Presencia de quistes endometrisicos, miomas uterinos, plipos endometriales, alteraciones mullerianas en la
anatoma genital (tero doble, tabiques, ausencia de vagina, etc.), alteraciones en las trompas de Falopio (obstrucciones, inflamacin y
presencia de hidroslpinx). En estos casos se considera la necesidad de realizar una valoracin endoscpica por va laparoscpica o
histeroscpica para definir mejor el diagnstico o para realizar las correcciones quirrgicas pertinentes. Alteraciones en el
espermiograma: se valora la concentracin, la motilidad y la morfologa de los espermatozoides. En ocasiones se objetiva una ausencia
MANUAL DE TRABAJO DEL CURSO ENARM CMN SIGLO XXI
CURSO ENARM CMN SIGLO XXI TEL: 36246001 Pharmed Solutions Institute PGINA 275

de espermatozoides en el eyaculado (azoospermia), por lo que se deber recurrir a la biopsia testicular para completar el estudio y
obtener espermatozoides testiculares o de epiddimo con los que poder trabajar a posteriori. Valores recomnedados por OMS:
volumen: 1,5-5,0ml, pH; >7.2, viscocidad; <3 (escala 0-4), espermatozoides; >20 millones/ml, N. total de espermatozoidez; >40
millones/eyaculado, porcentaje demotilidad >50%, progresion progresiva; >2 (escala de 0-4), Morfologia normal; >50% normal,
aglutinacion de espermatozoides <2 (escala de 0-3). Laparoscopia: para valorar factor tubo-peritoenal, es el elemento mas valioso para
el diagnostico y tratamiento mas valioso, nos yauda a corroborar la permeabilidad de las salpinges y permeabilizar las mismas.
TRATAMIENTO: partir de ese momento, se decide ya un tratamiento para la pareja. Si todas las exploraciones resultan normales se
indica la realizacin de coitos programados con estimulacin de la ovulacin (si el problema es una disfuncin ovulatoria, como ocurre
en los ovarios poliqusticos) o de inseminaciones artificiales (cuando supuestamente los ciclos de la mujer han sido regulares). La
experiencia demuestra que la inseminacin conyugal proporciona tasas acumuladas de embarazo > 60%, por lo se aconseja instaurar
este tipo de terapia durante 4 ciclos. Si la mujer no gesta, el camino a seguir depender mucho de su edad, pero en casi todos los casos
se decide practicar fecundacin in vitro (FIV) para estudiar ambos gametos en el laboratorio y la interaccin de ambos (fecundacin y
desarrollo embrionario). En el 20% de los casos en los que las inseminaciones fallan y se realiza fecundacin in vitro, se ha observado
que no se produce la penetracin del espermatozoide en el interior del ovocito u vulo y por lo tanto se produce un fallo de
fecundacin, responsable de la esterilidad en estas parejas. Por ello, en un primer ciclo tras fallo de inseminaciones, se realiza con la
mitad de los vulos FIV y con la otra mitad ICSI o microinyeccin intracito plasmtica de espermatozoides (1/2 FIV- 1/2 ICSI), lo que
permite decidir sobre la tcnica ms adecuada en el futuro y al menos asegurar fecundacin en la mitad de los ovocitos, para poder
realizar transferencia embrionaria y rentabilizar el ciclo con posibilidad de gestacin. FRMACOS EMPLEADOS: La terapia
farmacolgica contempla las siguientes opciones: 1) Citrato de clomifeno: Se emplea por va oral. Su accin se basa en la unin del
frmaco con los receptores estrognicos de la hipfisis, que provoca el bloqueo de la retroalimentacin negativa del estradiol. Ello
induce un incremento de la secrecin de gonadotropinas por la hipfisis. Es un frmaco de fcil uso y barato, pero el efecto
antiestrognico a nivel endometrial y del cuerpo lteo puede ser responsable de las bajas tasas de implantacin que permite obtener.
2) Gonadotropinas humanas: Las primeras estimulaciones de la ovulacin se realizaron con gonadotropina hipofisaria humana, pero
tuvo que ser abandonada por la aparicin de la enfermedad de Creutzfeldt-Jakob. Aos ms tarde se aislaron gonadotropinas en las
orinas de las mujeres menopusicas (gonadotropina corinica humana o HMG) con escasa pureza y conteniendo FSH y hormona
luteinizante (LH). Desde entonces ha habido una intensa investigacin farmacutica para aislar FSH de alta pureza sin carga de LH y con
posibilidad de administracin subcutnea, y desde 1998 existe la FSH recombinante obtenida por ingeniera gentica. En breve saldr al
mercado LH recombinante. 3) Anlogos de la GnRH: Uno de los avances ms importantes en el campo de la reproduccin asistida ha
sido el descubrimiento de los anlogos de la GnRH. Administrados en distintas fases del ciclo menstrual y combinados con la
estimulacin de la ovulacin con gonadotropinas, proporcionan una mayor sincrona en el crecimiento de los folculos, impiden el pico
espontneo de LH que provocara una luteinizacin prematura de los mismos (impiden la ovulacin espontnea) y disminuyen el
nmero de cancelaciones de procedimientos. Con su uso se han mejorado las tasas de fecundacin, de nmero de ovocitos obtenidos y
ovocitos maduros obtenidos, as como de las tasas de gestacin. Su mecanismo de accin es en dos tiempos: en un primer momento
por su unin al receptor estimulan la produccin (efecto flare up), para que horas despus desensibilicen los receptores por saturacin
(hipofisectoma mdica reversible). 4) Inductores de la ovulacin HCG: La HCG u hormona del embarazo farmacolgica induce la
ovulacin 37 horas tras su administracin. Obtenida de la orina de mujeres embarazadas, su accin es equivalente a la de la LH
hipofisaria. La LhRH recombinante est en fase de ensayo clnico. 5) Antagonistas de la GnRH: Bloquean los receptores de la GnRH,
suprimiendo la secrecin de gonadotropinas por la hipfisis de forma inmediata y evitando el efecto flare up provocado por el uso de
anlogos de la GnRH. En breve aparecern en el mercado farmacutico espaol preparados de administracin subcutnea.

CASO CLINICO
Femenino de 37 aos de edad la cual acude a consulta debido a que no ha logrado embarazarse, refiere que lleva 4 aos de
intentndolo, con vida sexual activa desde hace 5 aos con la misma pareja, no usa mtodo anticonceptivo, como antecedentes refiere
menarca a los 19 aos, gesta 0, para 0, abortos 0, a la exploracin fsica se observa con caracteres sexuales secundarios adecuados, a la
exploracin GO sin secrecin vaginal blanquecino sin olor desagradable, a la especuloscopia con normal, acude con resultados de
Papanicolaou con datos inflamatorios crnico, colposcopia negativo para VPH y perfil hormonal normal (FSH, LH, P4, E2).

PREGUNTA:
Tomando en cuenta el perfil hormonal de la paciente que intervencin en primera instancia tomara usted:

RESPUESTA:
a.- Iniciar clomifeno 5 mg cada 24 hrs durante 14 das.
b.- Solicitar una espermatobioscopia.
c.- Realiza USG de anexos para buscar causa obstructiva.
d.- Indicar medroxiprogesterona 25mg y etinilestradiol 2mcg im.

CASO CLINICO
El resultado de laboratorio reporta 40 % de espermatozoides viables, tomando en cuenta estos resultados, decide continuar el
abordaje mediante estudio de ultrasonografia plvica, donde se reporta ovarios de caractersticas adecuadas, sin embargo se reporta
alteraciones de oviductos, la paciente agrega que en los ltimos 2 aos ha presentado cuadro repetidos de infecciones cervicovaginales
por lo que indica un cultivo vaginal, recibiendo resultado positivo para garnerela y cladmidia tracomatis resistente a doxiciclina.

PREGUNTA:
La paciente es referida a biologa de la reproduccin, donde se indica histerosalpingografia, la cual reporta obstruccin bilateral del 75
% de oviductos, tomando en cuenta los ltimos reportes de laboratorio y gabinete, cul sera la complicacin ms importante para
conseguir la concepcin en este caso:
MANUAL DE TRABAJO DEL CURSO ENARM CMN SIGLO XXI
CURSO ENARM CMN SIGLO XXI TEL: 36246001 Pharmed Solutions Institute PGINA 276

RESPUESTA:
a.- Nivel de espermatozoides viables disminuido.
b.- Secuelas de Enfermedad Plvica Inflamatoria.
c.- Resistencia a los antibiticos de primera lnea.
d.- La condicin mixta como causa de la infertilidad.

CASO CLINICO
Varn de 25 aos de edad, sin antecedentes personales de inters, que fue remitido para la realizacin de una ecografa escrotal por
presentar dolor testicular izquierdo de 4 meses de evolucin; en la exploracin fsica se pone de manifiesto la presencia de un
varicocele. Se practica ecografa con escala de grises y ecografa Doppler color con un transductor lineal con una frecuencia 10 MHz. La
prueba se realiza con el paciente en posicin de decbito supino. El teste derecho es normal ecogrficamente. Se observa un varicocele
extratesticular izquierdo en un testculo de tamao normal, as como estructuras hipoecoicas tubulares en su interior con flujo venoso
que aumenta durante la maniobra de Valsalva en la ecografa Doppler; estos hallazgos se corresponden con un varicocele
intratesticular, con resultado de laboratorio donde se reporta azoospermia y reporte de urocultivo positivo a Chlamydia.

PREGUNTA:
Cual de las siguientes condiciones afecta de manera significativa a la fertilidad del paciente con las condiciones que presenta:

RESPUESTA:
a.- El varicocele.
b.- La Chamydia
c.- La azoespermia.
d.- Obtructivo

CASO CLINICO
Se trata de pareja que consult con una historia de infertilidad primaria de 2 aos. Ella de 36 aos y su marido de 37 aos. Dentro del
estudio inicial se encontr oligoastenozoospermia moderada. Ella adems presentaba anovulacin secundaria a un sndrome de ovario
poliqustico. Luego de intentar 4 ciclos de superovulacin con inseminacin intrauterina sin xito, Biologa de la Reproduccin decidi
optar por un ciclo de alta complejidad.

PREGUNTA:
Cual es la complicacin ms probable para la paciente por la exposicin a los ciclos de superovulacin a la que fue sometida:

RESPUESTA:
a.- Sindrome de Hiperestimulacion Ovarica.
b.- Gestacin Mltiple.
c.- Embarazo Extrauterino.
d.- Aborto Recurrente.

CASO CLINICO
Femenino de 31 aos de edad la acude a consulta debido a que presenta amenorrea secundaria, agrega que desea embarazarse, como
antecedentes cuenta con menarca a 21 aos con oligomenorrea, se observa con clasificacin de Taner grado 3, refiere presencia de
leucorrea frecuente ha recibido tratamiento, a la exploracin observa Talla 1.47 mts, Peso 48 kg. Adems cuello alado, torax en escudo.

PREGUNTA:
Cul es la condicin ms frecuente de origen gentico que presenta con los datos observados en este caso.

RESPUESTA:
a.- Sindrome de Kallman.
b.- Sindrome de Turner.
c.- Sindrome de Klinifelter.
c.- Sindrome XXX.













MANUAL DE TRABAJO DEL CURSO ENARM CMN SIGLO XXI
CURSO ENARM CMN SIGLO XXI TEL: 36246001 Pharmed Solutions Institute PGINA 277

AMENORREAS. CIENCIAS BASICAS: Ausencia temporal o permanente de menstruacin. AMENORREA PRIMARIA: se define como la
ausencia de menstruacin despus de los 15 aos de edad cuando coincide con la falta de la aparicin de los caracteres sexuales
secundarios (infantilismo sexual) y por arriba de los 18 aos en presencia de desarrollo sexual. La amenorrea primaria frecuentemente
se acompaa de infantilismo sexual y talla baja, lo que hace sospechar en primer trmino la existencia de una disgenesia gonadal (1ra
causa de amenorreas primarias); adems si existen malformaciones somticas se hace aparente un sndrome de Turner. En otras
ocasiones se puede encontrar el antecedente en la infancia de un tumor hipofisario que fue sometido a ciruga y/o radiacin. En forma
excepcional la amenorrea
primaria se encuentra asociada
a una talla normal, desarrollo
mamario, pero ausencia de vello
sexual e infantilismo sexual con
dispareunia; todo ello hace el
diagnstico de un sndrome de
testculo feminizante Sx., de
Morris (enfermedad recesiva
ligada al cromosoma X,
insensibilidad a andrgenos), es
la 3ra causa de las amenorreas
primarias. Agenesia del
conducto de Muller o Sx. de
Rokitansky-Kuster-Hauser;
segunda causa mas frecuente de
amenorrea primaria, hay utero
rudimentario no canalizado o
solo cuernos uterinos, cariotipo
46XX. Finalmente, aunque raro, es que se presente en forma normal el desarrollo de los caracteres sexuales secundarios, pero no la
menarquia y esto hace sospechar una malformacin congnita del tero y la vagina o un himen imperforado. La confirmacin
diagnstica utiliza esencialmente estudios citogenticos y de imagenologa, pasando a segundo trmino los hormonales. Se puede
establecer este diagnstico de acuerdo al algoritmo de la figura 1. En los casos en que se presenta amenorrea y adems ausencia de
caracteres sexuales secundarios, el diagnstico se establece en base a los resultados de laboratorio y el estudio del cariotipo. La
pubertad retardada es una entidad familiar caracterizada por hipogonadismo-hipogonadotrpico, por lo tanto los antecedentes
familiares detallados ayudan a establecer esta etiologa, ya que es difcil de distinguir entre la falla hipotalmicao hipofisaria. La
recomendacin en estos casos es la vigilancia estrecha de los cambios fenotpicos. Otra entidad conocida como sndrome de Kallmann,
la cual se asocia a anosmia, tambin puede causar hipogonadismo-hipogonadotrpico. Los casos de hipogonadismo-
hipergonadotrpico y amenorrea primaria estn causados por disgenesia gonadal o falla ovrica prematura. La forma ms comn de
disgenesia es el sndrome de
Turner que se caracteriza por
la presencia cromosmica 45
X0, pliegue en el cuello,
amplio espacio entre los
pezones y estatura corta. Sin
embargo, se debe recordar
que puede existir mosaicismo
como en todo sndrome
cromosmico. AMENORREA
SECUNDARIA: es la
desaparicin de la
menstruacin, por ms de
tres meses, en una mujer que
haya tenido varios ciclos
previos. La duracin de la
amenorrea secundaria es muy
variable y puede ser
permanente. La primera
posibilidad de amenorrea
secundaria es el embarazo
como norma clnica; una vez
descartado lo anterior, las
causas ms frecuentes son el
sndrome de ovarios
MANUAL DE TRABAJO DEL CURSO ENARM CMN SIGLO XXI
CURSO ENARM CMN SIGLO XXI TEL: 36246001 Pharmed Solutions Institute PGINA 278

poliqusticos (Sx de Stein-Leventhal) y los trastornos hipotalmicos que son diagnsticos por exclusin (eliminando la
hiperprolactinemia debida
coincidiendo o no con un tumor
hipofisario, as como un trastorno
tiroideo). Una prueba que mantiene
su vigencia y utilidad es la
administracin de progesterona
para inducir un sangrado menstrual
y cuando esto ocurre se confirma la
presencia de un nivel aceptable de
estrgenos circulantes; en cambio
cuando no hay menstruacin la continuacin diagnstica es el ciclo completo de estrgenos y progesterona, si consigue la
menstruacin entonces se puede sospechar un defecto severo de produccin endgena de estrgenos. Si un ciclo combinado de
estrgenos y progesterona no consigue inducir la menstruacin se debe sospechar una ausencia virtual del tero por sinequia uterina
ocasionada por un legrado o una infeccin postparto o ms raro por tuberculosis. En la actualidad aparece ms frecuentemente la
menopausia precoz ya sea quirrgica o idioptica que se puede diagnosticar con la prueba de estrgno-progesterona y si es necesario
con las determinaciones de FSH y estradiol circulantes. Los exmenes de laboratorio de mayor utilidad como una segunda etapa a la
clnica y orientados por sta son la prolactina (PRL), las gonadotropinas (LH, FSH), TSH (hormona estimulante deltiroides) y de menor
valor estradiol y andrgenos. Los avances en la imagenologa son ahora de gran ayuda para conocer la hipfisis y los rganos
intraplvicos (ovarios,tero, endometrio y anexos). Existen rutas diagnsticas que han permitido la elaboracin de algoritmos tiles en
la clnica. Se ha establecido un algoritmo para llegar a este diagnstico despus de haber descartado el embarazo como potencial
diagnstico, como se muestra en la figura 2. Hipotiroidismo: Cuando existe enfermedad tiroidea se presentan cambios menstruales
antes de presentarse la amenorrea, como en el caso de hipotiroidismo leve, donde se puede presentar disminucin o aumento del
sangrado menstrual, pero despus del tratamiento del hipotiroidismo se restablece el ciclo menstrual aunque despus de varios meses.
Hiperprolactinemia: En las pacientes con elevadas concentraciones de PRL (> 100 ng/mL), galactorrea, cefalea y trastornos visuales se
deben realizar estudios de imagenologa para confirmar un tumor de hipfisis, ya que el prolactinoma es la principal causa de trastorno
de la hipfisis anterior. En caso de encontrarse un microadenoma (< 10 mm) se deben tratar con agonistas dopaminrgicos, adems de
vigilar y tratar el problema de infertilidad. Los macroadenomas tambin deben tratarse con agonistas dopaminrgicos o extirparlos
quirrgicamente a travs de reseccin transesfenoidal. Cuando se ha excluido el adenoma, se debe tener presente la posibilidad de
hiperprolactinemia causada por medicamentos, aunque los medicamentos no logran concentraciones elevadas como un adenoma, es
decir, las concentraciones siempre estarn por debajo de 50 ng/mL, la suspensin del medicamento mejorar este trastorno. Ausencia
de endometrio: La incapacidad uterina para producir endometrio se conoce como sndrome de Asherman, causado por curetaje
excesivo del endometrio durante un procedimiento de limpieza obsttrica o por infeccin intrauterina, la histerosapingografa,
histeroscopia y el ultrasonido pueden ayudar a establecer el diagnstico (Figura 3). Otras causas menos frecuentes la constituyen: la
estenosis cervical, la fibrosis obstructiva o los plipos. Falla ovrica prematura y menopausia precoz: Est caracterizada por
hipogonadismo hipergonadotrpico, causado por deplecin de los folculos ovricos que lleva al hipoestrogenismo y finalmente
amenorrea. Esta entidad est asociada a trastornos autoinmunes, por esta razn los exmenes de laboratorio resultan normales, por lo
tanto debe tratarse la enfermedad de fondo (ej. enfermedad de Addison). En el dagnostico podemos tener: Gonadrotrofinas
aumentadas= FSH>40 y LH >25Mui/ml= Trastorno reside en ovarios. Gonadotrofinas disminuidas= FSH<5 y LH <5mUI/ml= Tratsorno del
sistema hipotlamo-hipofisis.

CASO CLINICO
Se trata de femenino de 24 aos de edad la cual acude a consulta debido a que anteriormente se encontraba bajo tratamiento con
clomifeno ya que desea embarazarse sin embargo no sabe el diagnostico establecido, tiene un USG donde se observa gnadas con
hipoplasicas adems de niveles hormonales gonadales por debajo de la media.

PREGUNTA
Considerando los datos observados, anatmicos y fisiolgicos, cual es la indicacin ms adecuada para favorecer la ovulacin y de esta
manera mejorar la posibilidad de embarazo.

RESPUESTA
a.- Debemos primeramente realizar una prueba con bromocriptina.
b.- La groserelina nos favorecera una estimulacin gonadal.
c.- El citrato de clomifeno es el mas indicado para estimular hipotalmicamente.
d.- Por protocolo deber iniciarse manejo con gonadotrofinas.

CASO CLINICO
Se trata de femenino de 24 aos acude por presentar alteraciones en su ciclo menstrual, refiere periodos menstruales irregulares que le
duran uno o dos das y en ocasiones solo es manchado, a la exploracin fsica se observa paciente con deficiencia en caracteres
sexuales femeninos que corresponden a menor edad aparente contra la cronolgica.

PREGUNTA
Cul es el patrn hormonal ms probable que esperara encontrar.

RESPUESTA
a.- Aumento de FSH y LH con estrgeno y progesterona alta.
MANUAL DE TRABAJO DEL CURSO ENARM CMN SIGLO XXI
CURSO ENARM CMN SIGLO XXI TEL: 36246001 Pharmed Solutions Institute PGINA 279

b.- Aumento de FSH y LH con estrgeno y progesterona baja.
c.- Andrgenos elevados y estrgenos y progesterona normal.
d.- Andrgenos normales con estrgeno y progesterona baja.

CASO CLINICO
Se trata de paciente femenino de 21 aos de edad la cual acude por falta de menstruacin durante los ltimos 6 meses, niega vida
sexual activa, sus antecedentes GO son menarca 11 aos, G: 0, a la exploracin se encuentra caracteres sexuales normales para edad y
sexo, se observa sobrepeso por IMC, niega flujo, refiere que en los 3 ltimos aos sus periodos menstruales han sido irregulares, niega
cefalea u otros sntomas.

PREGUNTA
Cul de los siguientes diagnsticos es el ms probable.

RESPUESTA
a.- Microadenoma PRL productor.
b.- Adenoma GH productor.
c.- Microadenoma ACTH productor.
d.- MEN.

ENDOMETRIOSIS. CIENCIAS BASICAS: La endometriosis es una de las enfermedades ms enigmticas que afecta a las mujeres en edad
reproductiva. Esta enfermedad se dene por la presencia de tejido endometrial (epitelio glandular y estroma) fuera de la cavidad
uterina, localizndose habitualmente sobre la supercie peritoneal y sobre el ovario. Otras posibles localizaciones de esta enfermedad
son: sistema gastrointestinal, pulmn, sistema genitourinario, pared abdominal.... Puede manifestarse por trastornos funcionales,
relacionados con esterilidad, es un distrofia benigna, es hormonodependiente, la menopausia es una forma de curacin fisologica, y el
tratamiento sustitutivo la reactiva. La loscalizaciones mas frecuentes son: ligamentos utero-sacros, fondo de saco posterior, fondo de
saco vesicouterino, ovario (el mas frecuente), tubas uterinas, extragenialtes. SALUD PUBLICA: Aunque la prevalencia exacta de
endometriosis en la poblacin general
es desconocida se estima que entre el
20 y 90% de mujeres con dolor
plvico y/o infertilidad presentan esta
enfermedad. En mujeres
asintomticas, la prevalencia se sita
entre 3 y 30%. Un 10% en el grupo de
eda de los 30-40 aos de edad.
PATOGENIA: En la actualidad existen
cuatro teoras para explicar la
patognesis de la endometriosis:
metaplasia celmica, restos de clulas
embrionarias, diseminacin linftica y
vascular y trasplante de tejido
endometrial. 1) Metaplasia celmica:
A principios del siglo pasado, fue
considerada la primera teora que
explicaba la patogenia de la endometriosis.Se sugiri que la endometriosis se originaba en la membrana celmica a travs de un
proceso de metaplasia seguido de una induccin metaplsica, lo cual ha sido conrmado. El epitelio celmico, que surge de una capa de
clulas epiteliales del conducto mulleriano, tambin se diferencia en epitelio pleural y peritoneal y en clulas de la supercie de los
ovarios. Este hecho puede explicar las localizaciones inusuales de la endometriosis. Existen otros argumentos que deenden la teora
de la metaplasia celmica como son la presencia de esta enfemedad en hombres, en mujeres etapa prepuberal, en aquellas que jamas
han menstruado y en adolescentes sin anormalidades del conducto de mullerianos y pocos aos despus de la menarquia. 2) Teora de
los restos embrionarios: Esta teora fue acreditada por Von Recklinghausen, quien se bas en la identicacin de restos del conducto
Wolano en el ligamento ancho y porciones antero-laterales de la vagina y el crvix, que podran dar lugar a la endometriosis. La
presencia de estos restos embrionarios puede explicar la existencia de
endometrio ectpico. reas adyacentes a los conductos mullerianos y
duplicaciones del sistema mulleriano, permitiran que clulas de origen
mulleriano se conviertan en endometrio funcionante. Sin embargo, la
presencia de los mismos no se ha encontrado en la pelvis o en la
cavidad torcica. La endometriosis tiene su mayor incidencia en
mujeres de a partir de los 25 aos. 3) Diseminacin linftica y vascular:
Sampson fue el primero en sugerir la diseminacin linftica y
hematgena de la endometriosis al encontrar tejido endometrial en
venas de pacientes con adenomiosis. Posteriormente Halban conrm
esta teora postulando la viabilidad de las clulas endometriales que
entraban a travs de la membrana basal de los vasos sanguneos y
linfticos. Estas clulas endometriales mediante la formacin de
micrombolos provocaran el desarrollo de la enfermedad en lugares a
distancia. 4) Teora de la implantacin: Segn los hallazgos de
MANUAL DE TRABAJO DEL CURSO ENARM CMN SIGLO XXI
CURSO ENARM CMN SIGLO XXI TEL: 36246001 Pharmed Solutions Institute PGINA 280

Sampson el contenido menstrual con fragmentos de endometrio pasa a travs de las trompas de Falopio alcanzando la cavidad
peritoneal en lo que conocemos como menstruacin retrgrada. Ese endometrio posteriormente se implanta en la supercie peritoneal
y en los rganos del abdomen y de la pelvis, donde en los prximos ciclos menstruales se produce el sangrado menstrual. Esta teora, se
basa en tres suposiciones: la primera, que hay menstruacin retrgrada a travs de las trompas; la segunda, que las clulas
endometriales contenidas en el material de reujo son viables en la cavidad uterina; la tercera, que dichas clulas pueden adherirse al
peritoneo, y favorecer los procesos de invasin, implantacin y proliferacin subsiguientes. La teora de la implantacin fue rechazada
durante muchos aos por la suposicin de que la menstruacin era infrecuente. DIAGNOSTICO: Se hace aparente en los aos de la
etapa reproductiva debido a que las hormonas producidas por el ovario estimulan al endometrio. Los sntomas son intensos en el
periodo premenstrual y moderados al trmino de la menstruacin, por lo tanto, el dolor plvico es el sntoma ms frecuente, pero
adems puede aparecer dolor de espalda, dispareunia, dolor a la defecacin, dolor al cambio de posicin y disuria. La mayora de las
mujeres en quienes se ha diagnosticado endometriosis ha tenido previamente sndrome de vejiga irritable. La asociacin de
endometriosis con infertilidad es debido a las adherencias en la cavidad peritoneal, lo cual distorsiona la anatoma plvica y trastorna la
liberacin del vulo; sin embargo, los trastornos anatmicos no son el nico motivo de la infertilidad, ya que se ha descrito que la
endometriosis causa baja reserva ovrica y de la calidad del vulo. Adems, en los estudios de fertilizacin se ha encontrado baja
calidad del embrin, lo que disminuye la capacidad de implantacin y diferenciacin celular. Existen pocos estudios clnicos y maniobras
exploratorias que ayuden a diagnosticar de forma cetera la endometriosis. Hay signos que pueden estar ausentes, pero deben estar
incluidos ndulos en la parte posterior de la vagina, dolor al movimiento uterino, tero en retroversin y fijo, y dolor de masa anexial
por endometriomas. Se ha determinado la utilidad de los signos clnicos y de los sntomas en el diagnstico de la endometriosis en
mujeres que presentan infertilidad. El diagnstico de la endometriosis se basa en la visualizacin del endometrio ectpico por
laparoscopia o laparotoma, acompaado de confirmacin histolgica de macrfagos con hemosiderina, epitelio endometrial y
glndulas o estroma endometrial. El ultrasonido intravaginal ha sido el de mayor uso en el diagnstico de lesiones retroperitoneales y
uterosacras, pero no tiene la exactitud para identificar lesiones peritoneales o endometriomas pequeos. TRATAMIENTO: Quirrgico
consiste en la ablacin del tejido endometrial ectpico a travs de laparoscopia, aumenta la fertilidad en mujeres con endometriosis,
aunque se ha reportado que la exresis de los quistes endometrisicos otorga un mejor resultado en la disminucin del dolor
comparado con la eletrofulguracin y drenaje del tejido endometrial ectpico. Por otro lado, la histerectoma acompaada de salpingo-
ooforectoma bilateral se considera el tratamiento definitivo para la endometriosis severa, aunque en estudios retrospectivos se ha
observado que 10 % de las mujeres con histerectoma y salpingectoma bilateral tuvieron recurrencia de los sntomas, mientras que las
mujeres que solo tenan histerectoma presentaron recurrencia de los sntomas en 62 % de los casos. La vertiente actual cuando hay
infertilidad es tratar la endometriosis aunque sta sea mnima, dbido a que los cambios presentes por este tejido ectpico son a nivel
inmunolgico y en los rganos genitales, lo que trastorna todo el proceso de la concepcin. El tratamiento mdico para el dolor incluye
analgsicos (antiinflamatorios no esteroideos), anticonceptivos orales, danazol, anlogos de la hormona liberadora de gonadotropinas
(GnRH = leuprolide, goserelina, triptorelina, nafarelina). Se ha demostrado que el uso de GnRH para el tratamiento del dolor causado
por endometriosis no mejora el efecto que los anticonceptivos o los progestgenos, adems, tiene el inconveniente de causar
hipoestrogenismo como efecto secundario, que cesa con la suspensin del tratamiento. Recientemente se han utilizado los inhibidores
de aromatasa con respuesta prometedora para tratar la endometriosis, con lo cual se ha visto efecto sobre el dolor plvico y la
produccin de endometrio ectpico, ya que bloquea la produccin en secuencia de ciclooxigenasa-2 (COX2), produccin de
prostaglandinas y produccin local de estradiol por el endometrio ectpico, el cual es estrgeno-dependiente.

CASO CLINICO
Femenino de 41 aos de edad. Entre sus antecedentes mencion el diagnstico de tuberculosis intestinal tres aos antes, siendo
tratada durante 8 meses con tratamiento especfico tetra-asociado, periodos menstruales regulares y un parto por cesrea a los 32
aos. Se hospitalizo por cuadro de varios aos de evolucin, con frecuentes episodios de dolor abdominal difuso, distensin abdominal,
flatulencia, nauseas, vmitos y constipacin, mayor durante ciclos menstruales. El cuadro que se intensifico durante el ltimo ao,
asocindose espordicamente deposiciones con estras de sangre viva y perdida de 4 a 5 Kg de peso. Al ingreso, paciente en regulares
condiciones generales, presin arterial 120/80 mmHg, frecuencia cardiaca 88 latidos por minuto, temperatura 36,6 C, el abdomen se
encontraba distendido, timpanizado a la percusin, chapoteo en flanco y fosa iliaca derecha y borborigmos generalizados a la
auscultacin. Los laboratorios, rayos X de trax, abdomen y ecografa abdominal de ingreso fueron normales, exceptuando la sangre
oculta en heces que fue (+). Las revisiones ginecolgica y proctolgica no encontraron alteraciones.

PREGUNTA
Cul es el estadio en el que se encuentra la paciente?

RESPUESTA
a.- Estadio I (Mnimo) - 1-5 puntos.
b.- Estadio II (Leve) - 6-15 puntos.
c.- Estadio III (Moderada) - 16-40 puntos.
d.- Estadio IV (Severa) - ms 40 puntos.

CASO CLINICO
Una paciente de 47 aos que consult por dolor intenso, de carcter cclico, de aproximadamente 1 ao de evolucin, localizado en la
FID. Los dolores comenzaban 3 o 4 das antes de cada menstruacin y precisaba de grandes dosis de analgsicos para su control. Como
antecedentes de inters refera una salpingectoma y una cua ovrica izquierdas por va laparoscpica, probablemente por quistes
endometrisicos, as como la reseccin por histeroscopia de un endometrioma uterino 3 aos antes. En las exploraciones
complementarias, que incluyeron resonancia nuclear magntica y trnsito gastrointestinal baritado, nicamente destac un
endometrioma en ovario izquierdo de 17 mm de dimetro, que el servicio de ginecologa desestim para ciruga. Ante el diagnstico de
dolor crnico recurrente en FID de origen desconocido y dados los antecedentes mdico-quirrgicos de la paciente, se plante realizar
MANUAL DE TRABAJO DEL CURSO ENARM CMN SIGLO XXI
CURSO ENARM CMN SIGLO XXI TEL: 36246001 Pharmed Solutions Institute PGINA 281

una laparoscopia exploradora, en la que se evidenci un quiste seroso ovrico izquierdo y un apndice vermiforme normal, pero no se
apreciaron tumores ni lesiones intraabdominales, por lo que se opt por llevar a cabo la apendicectoma profilctica.
PREGUNTA
Cuales son las evidencias diagnosticas de laboratorio y gabinete mas frecuentes.

RESPUESTA
a.- Niveles plasmticos de CA 125.
b.- Glndulas endometriales.
c.- Estroma endometrial
d.- Macrfagos con inclusiones de hemosiderina

CASO CLINICO
Enferma de 34 aos de edad que consulta por clnica de sndrome miccional y sensacin de disconfort hipogstrico de varios meses de
evolucin con urocultivos realizados en Atencin Primaria negativos. En una ocasin tuvo un episodio de hematuria autolimitada y no
tiene antecedentes mdicos ni ginecolgicos de inters, salvo un parto vaginal no instrumentado. En los estudios analticos practicados
no se observan hallazgos dignos salvo una discreta anemia y una elevacin mnima de la V.S.G. La ecografa abdominal practicada se
informa como compatible con la normalidad, por lo que se decide la prctica de una cistoscopia donde se aprecia una lesin qustica en
pared posterior no sugerente de neoformacin papilar tpica. En la R.T.U. practicada se informa como endometriosis vesical

PREGUNTA
Cul es la conducta ms adecuada para este caso.

RESPUESTA
a.- Danazol a dosis de 600 a 800 MG.
b.- Anlogos de GnHR 3.5 MG IM mensual.
c.- Medroxiprogesterona oral.
d.- Anticonceptivos hormonales combinados.

CERVICOVAGINITIS. CIENCIAS BASICAS: El trmino cervicovaginitis se refiere a la inflamacin del epitelio escamoso de la vagina y el
cuello uterino. En este cuadro, la mucosa cervical y vaginal responde a la infeccin con una reaccin inflamatoria que se caracteriza por
el dao a las clulas superficiales. Vaginitis: Corresponde a la enfermedad inflamatoria del crvix y la vagina cuya etiologa son Cndida
albicans y Trichomonas vaginalis. Vaginitis recurrente: Presentacin de cuatro o ms episodios en un ao. SALUD PUBLICA: La
inflamacin de la vagina y la presencia de flujo vaginal sustentan el diagnstico ginecolgico ms frecuente en mujeres en edad
reproductiva que asisten a clnicas de primer contacto. La vaginosis o vaginitis bacteriana se ha encontrado en el 15-19% de las
pacientes ambulatorias. Durante el embarazo, las cervicovaginitis se asocian en 11.5 % con el parto pre trmino y en 11.6 % con
ruptura prematura de membranas y como factor predisponente para esterilidad de origen tubrico y cncer cervicouterino.
PATOGENIA: Dentro de la etiologa de las cervicovaginitis, 22.6 % de los casos son producidos por Gardnerella vaginalis, 19.1 % por
Candida spp., 7.8 % por Candida albicans y 1.5 % por tricomonas, estas en conjunto causan el 90% de los casos. Otras causas que
forman el 10%, son: alrgicas (a la protena del semen), irritativa ( ducha vaginal, jabones, desodorantes, medicamentos tpicos). En
este cuadro, la mucosa cervical y vaginal responde a la infeccin con una reaccin inflamatoria que se caracteriza por el dao a las
clulas superficiales. Este dao conduce a la descamacin y ulceracin, que causan una reduccin del espesor epitelial debido a la
prdida de clulas de la capa superficial y de parte de la capa intermedia (que contienen glucgeno). En las capas ms profundas, las
clulas sufren tumefaccin con infiltracin de neutrfilos en el espacio intercelular. La superficie del epitelio se halla cubierta por
desechos celulares y secreciones inflamatorias mucopurulentas. Existe congestin del tejido conjuntivo subyacente con dilatacin de
los vasos superficiales y papilas estrmicas hipertrficas y dilatadas. VAGINOSIS BACTERIANA (VB): Es la causa ms frecuente de
exudado vaginal y de mal olor de vagina. La VB es una alteracin de la flora vaginal, en la que la flora bacteriana normal, constituida
por bacilos gram positivos (Lactobacillus spp.), se halla sustituida por cocobacilos gramnegativos (Gardnerella vaginalis) y una flora
variada que comprende diversas especies anaerobias (prevotella, movilluncus, mycoplasma). Su importancia viene dada por su relacin
directa con la enfermedad
plvica inflamatoria,
interfiere en la fertilidad. En
su patognesis intervienen
sinrgicamente G. vaginalis y
los anaerobios que
producen exudado vaginal y
el mal olor. La causa de esta
disbacteriosis es
desconocida, y se asocia con
la existencia de mltiples
parejas sexuales, duchas
vaginales y prdida de
Lactobacillus, lo que provoca
una elevacin del pH vaginal.
Criterios diagnsticos de
Amsel: 1) Flujo homogneo, delgado blanco-grisceo. 2) pH >4.5, 3) prueba de aminas positiva, 4) clulas clave en frotis en seco. Con
3 de 4 criterios son diagnostico; con sensibilidad de 73% y especificidad de 94%. La tincin de Gramm es el gold estndar. El
MANUAL DE TRABAJO DEL CURSO ENARM CMN SIGLO XXI
CURSO ENARM CMN SIGLO XXI TEL: 36246001 Pharmed Solutions Institute PGINA 282

tratamiento de las parejas no resulta eficaz para prevenir las recidivas. Tratamiento: Se recomienda en la mujer embarazada, pues
reduce el riesgo de sufrir complicaciones como parto prematuro y endometritis puerperal. En la no embarazada tambin se
recomienda, pues elimina los sntomas y signos de VB y reduce las complicaciones en caso de aborto o histerectoma. El tratamiento se
realiza con metronidazol en dosis de 500 mg cada 12 horas por va oral, durante 7 das. Existen tratamientos alternativos por va vaginal
como clindamicina al 2% en crema vaginal o metronidazol al 0,75% en gel, ambos se administran con un aplicador, antes de acostarse,
durante 7 das. No se recomienda el tratamiento de las parejas. En el embarazo: metronidazol 250mg VO c/8h por 7 das o cindamicina
300mg VO c/12 por 7 das. VULVOVAGINITIS POR CANDIDA: La colonizacin vaginal por Candida es relativamente frecuente. Muchas
de ellas tambin presentan colonizacin en la zona anorrectal. No presenta sin sintomatologa. Sin embargo la mitad que presenta
vulvovaginitis candidisica presenta inflamacin vulvar y vaginal, fisuras y existencia de un exudado adherente a la mucosa,
blanquecino y amarillento, con grumos (cottage cheese). El pH vaginal se mantiene en 4,5. En la patogenia de esta infeccin actan
como factores predisponentes la diabetes, el embarazo, el uso de contraceptivos orales, la obesidad, el empleo reciente de
antimicrobianos y la utilizacin de corticoides. La forma de presentacin se clasifica en complicada o no complicada. La primera es
recurrente, grave, puede estar producida por otras especies de Candida diferentes a C. albicans y se produce en diabticas no
controladas, inmunodeprimidas y embarazadas. Tratamiento tpico: Existen numerosos tratamientos tpicos mediante aplicacin
intravaginal. Referimos solo algunos: 1. Butoconazol 3% en crema, 5 g durante 3 das. 2. Clotrimazol 100 mg en vulos vaginales, 2
vulos durante tres das. 3. Clotrimazol 500 mg en vulos, una sola aplicacin. 4. Miconazol 100 mg un supositorio vaginal al da
durante 7 das. 5. nistatina 100.000 unidades en tableta vaginal, una tableta durante 14 das. Tratamiento oral fluconazol 150 mg, dos
dosis separadas 72 horas. VAGINITIS POR TRICHOMONAS: La infeccin por T. vaginalis constituye una de las ETS ms frecuentes en el
mundo, en la embarazada se ha asociado a parto prematuro y recin nacido de bajo peso. Se identifica en 30-80%de parejas masculinas
de mujeres infectadas, la mayora se mantienen en estado de portador solo el 10% presenta uretritis no gonoccica. Produce leucorrea
profusa, espumosa, amarillo-verdosa y maloliente, con abundantes polimorfonucleares, pH alcalino y prurito vaginal. En la exploracin,
la vagina est inflamada y el crvix enrojecido y edematoso con aspecto de frambuesa. En el varn, la infeccin por T. vaginalis es
menos frecuente. La sintomatologa que produce es de uretritis y, con frecuencia, la infeccin es asintomtica. El diagnstico por
microscopia en fresco es un mtodo especfico en caso de vaginitis purulenta, pero poco sensible para detectar a las pacientes
asintomticas. El cultivo es un mtodo especfico y sensible, superado por los mtodos moleculares, especialmente por la reaccin en
cadena de la polimerasa (PCR). La citologa, especialmente el Papanicolau, se considera un mtodo inadecuado por su baja sensibilidad
y pobre valor predictivo positiva. Un tratamiento eficaz es el metronidazol que, en dosis nica de 2 g o dosis mltiples de 500 mg cada
12 horas durante 7 das, se ha mostrado eficaz en ms del 90% de las pacientes, al igual que los nuevos compuestos tinidazol y
ornidazol. Este tratamiento se recomienda tambin en las mujeres embarazadas, en las que no se ha mostrado teratogenico.
DIAGNOSTICO: Es importante hacer historia clnica minuciosa, interrogar sobre las caractersticas de flujo, cambio de pareja sexual
recientemente, nmero de parejas sexuales, tollas vaginales, duchas vaginales, uso de tampones, tratamientos previos, etc.. Fortis en
seco: Laminilla N.1; aplicar secrecin vaginal+2gotas de sol. Salina 0.9%, observar en campo seco, vemos flagelados mviles con
abundantes PMN, caracteristicos de tricomoniasis o bacterias cocoides y celulas clave (cel.epiteliales descamativas cubiertas por
bacterias, Gardnerella), caractersticas de vaginosis bacteriana. Laminilla N.2; secrecin vaginal+ KOH al 10%, al microscopio vemos
levaduras o pseudohifas de candida, caractersticas. Laminilla N.3; secrecin vagunal + tincin gramm, vemos morfotipos gramm
variables de vaginosis bacteriana.

CASO CLINICO
A 30 aos mujer blanca de edad present al Centro de Salud, refiriendo una secrecin maloliente vaginal, ardorosa o quemante durante
4 meses. La paciente fue diagnosticada por primera vez con tricomoniasis vaginal por su mdico de atencin primaria y que haba
recibido cinco ciclos de metronidazol 500 mg por va oral dos veces al da, la mayor duracin del tratamiento fue de 10 das. Su marido
se trat con 3 das y 7 ciclos de un da de metronidazol. Ella admiti haber tenido otra pareja sexual que no haban recibido tratamiento
con metronidazol. Su ltimo curso de metronidazol fue de 1 mes antes de su presentacin. No haba antecedentes de otras
enfermedades de transmisin sexual. Ella estaba tomando anticonceptivos orales. En el examen, vulva con eritema, edema y
excoriacin. El examen con espculo, la pared vaginal era eritematosa, edematosa y con secreciones anormales.

PREGUNTA
Cul es al hallazgo mas frecuente en esta patologa?.

RESPUESTA
a.- Desaparicin de lactobacilos y bacterias anaerobias.
b.- Clulas epiteliales con bordes mal definidos.
c.- Abundantes cocobacilos.
d.- Presentacin de cuatro o ms episodios en un ao.

CASO CLINICO
Paciente de sexo femenino, 28 aos de edad, sin antecedentes patolgicos previos. Consult por episodios repetidos de vulvovaginitis.
Estos episodios haban comenzado dos aos antes, se repetan casimensualmente y aport estudios microbiolgicos previos, tres de los
cuales mostraron pseudohifas y elementos levaduriformes brotantes en el examen microscpico directo, con desarrollo de Candida
spp. Sus ciclos menstruales eran regulares y no tena antecedentes de embarazos previos. Como mtodo anticonceptivo empleaba un
dispositivo intrauterino. Fumadora y bebedora. No tena antecedentes de diabetes ni haba recibido antibiticos. El examen fsico no
acus anormalidades y ginecolgico apreci aumento de la secrecin vaginal, congestin de la mucosa y pseudomembranas
blanquecinas en vulva y vagina.

PREGUNTA
Cul es la conducta ms adecuada para este caso.
MANUAL DE TRABAJO DEL CURSO ENARM CMN SIGLO XXI
CURSO ENARM CMN SIGLO XXI TEL: 36246001 Pharmed Solutions Institute PGINA 283


RESPUESTA
a.- Metronidazol.
b.- Nistatina.
c.- Itraconazol.
d.- Nitrofurntoina.

CASO CLINICO
Femenino de 42 aos de edad paciente present una historia de 10 meses de la descarga vaginal persistente por el que haba recibido
ciclos intermitentes de eritromicina. Ella se encontr que tienen tricomoniasis y se le prescribi metronidazol 400 mg dos veces al da
durante 7 das. Ella neg cualquier actividad sexual posterior. Su compaero sexual anterior tambin fue tratado con metronidazol. T.
vaginalis se aisl repetidamente sobre el seguimiento de la cultura sobre las prximas semanas a pesar de nuevos cursos de
metronidazol intravaginal tanto oral como tpica.

PREGUNTA
Cul es la conducta ms adecuada para este caso.

RESPUESTA
a.- Metronidazol 500 mg cada 12 h, va bucal. 7 das
b.- Metronidazol 500 mg cada 24h va vaginal. 7 das.
c.- Nistatina, 100 mil unidades cada 24h, Va vaginal. 14 das
d.- Metronidazol 500 mg cada 24h, va vaginal, 7 das.

CASO CLINICO
Una mujer de 28 aos refiere que durante los ltimos tres aos, la haba estado tratando sin xito una recurrente descarga vaginal
desagradable. En su primera visita a la clnica, Trichomonas vaginalis se encontr en una preparacin hmeda de las secreciones
vaginales y se indico metronidazol 200 mg tres veces al da durante una semana fue recetado. Durante los prximos seis meses que
tena tricomoniasis sintomtica recurrente.

PREGUNTA
Cul es la conducta ms adecuada para este caso.

RESPUESTA
a.- Bao diario.
b.- Enjuague anogenital al medio da.
c.- Uso de toallas sanitarias solo durante la menstruacin.
d.- Evitar el uso diario de pantiprotectores y tampones.

CASO CLINICO
Se trata de paciente femenino de 23 aos de edad la cual acude a consulta debido a que presenta desde hace tres semanas secrecin
vaginal blanquecina de mal olor, refiere relacin sexual con nuevo compaero sin proteccin, cuenta con antecedente de uso de
anticonceptivo orales, menarca 10 aos, gesta 1, para 0, abortos 1, tabaquismo y alcoholismo positivos desde los 17 aos, a la
exploracin se observa secrecin blanca leve que se encuentra recubriendo la vagina de forma uniforme con olor a pescado.

PREGUNTA
Cual es tratamiento ms adecuado para este caso.

RESPUESTA
a.- Metronidazol 2 g VO dosis nica.
b.- Metronidazol 500 mg VO cada 12 hrs x 7 dias.
c.- Fluconazol 100 mg VO dosis nica.
d.- Clindamicina 300 mg VO cada 12 hrs por 5 dias.

ENFERMEDADES DE TRANSMISION SEXUAL (ETS). CIENCIAS BASICAS: Son conocidas tambin como enfermedades venreas (en honor
a la diosa romana Venus que es el equivalente a la diosa griega del amor y la belleza Afrodita) y su definicin obedece al modo de
transmisin de las mismas ya que se adquieren mediante el contacto sexual: vaginal, anal u oral. Todas las conductas sexuales que
involucran contacto con otra persona o con los fludos de la misma se considera como factor de riesgo para la adquisicin de ETS.
SALUD PUBLICA: En Mxico y tambin a nivel mundial, las ETS han representado un problema de salud pblica; a nivel nacional son una
de las diez primeras causas de morbilidad, con un promedio de 220,000 casos anuales. La frecuencia de ETS en la poblacin general
flucta entre 0.1 y 0.5%; en cambio, para la poblacin considerada como de alto riesgo, se calcula una frecuencia que va de 10 a 20%.
La poblacin expuesta se compone en su mayor parte de jvenes, el 34% corresponde al periodo entre la 2da y 3ra dcada de la vida,
seguido por el grupo de 25 a 44 aos que integran dos dcadas en un 41%. La relacin hombre mujer fue de 2,8:1 GONORREA O
BLENORRAGIA (Neisseria gonorrhoeae): El agente causal es Neisseria gonorrhoeae (el cual tiene tropismo por las mucosas), el humano
es el nico husped conocido. Puede causar uretritis, cervicitis, epididimitis, faringitis, proctitis, EPI y en los casos graves diseminacin
sistmica. En Mxico se reporta 1.0 por cada 100000 hab. La transmisin ms frecuente es mediante el acto sexual vaginal, anal u oral
y se hace a travs del contacto de la superficie mucosa de un individuo infectado. Tambin puede ser contagiada mediante fmites a
MANUAL DE TRABAJO DEL CURSO ENARM CMN SIGLO XXI
CURSO ENARM CMN SIGLO XXI TEL: 36246001 Pharmed Solutions Institute PGINA 284

otras reas, por ejemplo a un ojo con la mano o con otra parte del cuerpo humedecida con lquidos infectados. La transmisin vertical
ocurre cuando la madre infecta al recin nacido mediante el paso por el canal del parto, el producto adquiere la infeccin
manifestndose como conjuntivitis, vulvovaginitis e inclusive neumona, por lo tanto, en estos casos estara indicado el parto por
cesrea. Patogenia: Este microorganismo se adhiere a la superficie de los epitelios uretral, endocervical, vaginal, rectal, farngeo e
incluso a los espermatozoides humanos y a las clulas ciliadas de las trompas de Falopio, pueden expresar simultneamente varios
tipos de cadenas de LPS antignicamente diferentes, que median en gran parte su toxicidad. Las manifestaciones aparecen de 2-5 das
posterior al contacto de sexual, desde la infeccin asintomtica, sntomas locales en mucosas hasta la diseminacin sistmica. Se inician
con leve molestia uretral, seguida de disuria de grado variable y a veces urgencia miccional. Hay eritema y edema del meato urinario y
salida de material purulento por l, sin embargo la infeccin puede ser asintomtica en 5-7% de los afectados. Puede complicarse con
epididimitis, prostatitis, tromboflebitis, absceso periuretral o fstulas, vesiculitis seminal, estenosis uretral y esterilidad. En mujeres los
sntomas suelen aparecer entre 7-21 das despus de la infeccin, disuria, descarga vaginal y a veces fiebre. En los hombres, lo ms
comn es que el paciente presente una uretritis anterior con disuria (que es ms frecuente durante la maana) y una descarga uretral
purulenta caracterstica. La complicacin local ms comn es secundaria al ascenso de los microorganismos y se presenta como una
salpingitis aguda o EPI en el 10%-20% de los casos. El diagnstico de gonorrea es fundamentalmente clnico y se comprueba con la
identificacin de N. gonorrhoeae mediante tinciones de frotis de las mucosas infectadas. El diagnstico de uretritis se hace con el
hallazgo de 4 o ms PMN por campo por 1000X; el de uretritis gonoccica en tanto es altamente sugerente por la presencia al gram de
cocobacilos gram negativos en granos de caf. Cuando el examen directo es negativo y la sospecha es alta, debe realizarse el cultivo en
medio de Thayer Martin, medio enriquecido y con antimicrobianos que inhiben el crecimiento de otros microorganismos. El
tratamiento recomendado para gonorrea genital, rectal y farngea no complicada en adultos y adolescentes es con cefixima va oral o
ceftriaxona intramuscular. Otros regimenes recomendados son la ofloxacina, ciprofloxacina y levofloxacina. Si se comprueba la
infeccin conjunta con C. trachomatis el paciente puede ser tratado con azitromicina o doxiciclina. Es importante mencionar que debe
darse tratamiento a las parejas sexuales de los pacientes, y recordarles que no pueden tener relaciones sexuales hasta que se haya
completado el tratamiento y las parejas sexuales hayan sido tratadas. URETRITIS NO GONOCCICA (UNG) (Chlamydia Tracomatis y
Mycoplasmas genitales): Estos son los patgenos ms frecuentes. Corresponden al 40 a 50% de las uretritis. En un 30% de los casos no
se encuentra el agente etiolgico. La C. trachomatis se ha asociado a uretritis, proctitis y conjuntivitis en ambos sexos, con epididimitis
en el hombre, y con cervicitis mucopurulenta, salpingitis aguda, bartolinitis y el Sd. de Fitz Hugh Curtis en la mujer. Tambin puede
asociarse a artritis sptica. La coinfeccin con N. Gonorrhoeae es frecuente, y en general, si bien puede comprometer los mismos
rganos, las manifestaciones clnicas producidas por C. trachomatis suelen ser menos sintomticas y muchas veces asintomticas. La
infeccin asintomtica en mujeres puede provocar dao tubario e infertilidad. Los mycoplasmas que con mayor frecuencia infectan el
aparato genital son el Mycoplasma Genitalium y el Ureaplasma Urealyticum (UU). Ambos pueden adquirirse como colonizacin al
momento del parto, sin embargo menos del 10% de pre-pberes estn colonizados con Ureaplasmas. La tasa de UNG se ha mantenido
relativamente estable, con un valor actual de 5.1 casos por 100000 habitantes. La edad pico de la infeccin por C. trachomatis es la
adolescencia tarda y primeros aos de la tercera dcada. El uso de anticonceptivos orales y la ectopa cervical son factores de riesgo
para infeccin por C. trachomatis, es un patgeno intracelular obligado de tamao pequeo. Las manifestaciones clnicas de la infeccin
derivan de la toxicidad directa por estallido celular y de la respuesta inmune, siendo sta ltima la responsable del dao tubario
despus de infecciones repetidas. El comienzo de los sntomas suele ser insidioso, por lo que el paciente consulta con frecuencia
enforma tarda. El perodo de incubacin va de 2 a 3 semanas. Clsicamente se describe una secrecin uretral escasa de tipo muco-
serosa, fundamentalmente matinal. Hay leve disuria y prurito y escasa congestin del meato uretral. Puede ir desde un cuadro
asintomtico a un cuadro similar a la uretritis gonoccica. El crvix en fresa es caracterstico de chalamydia. Diagnstico: Anamnesis y
examen fsico concordante, frotis uretral para gram y cultivo de N. Gonorrhoeae: la UNG se demuestra por: ausencia de diplococos
gram negativo intracelulares, al menos 4 PMN por campo de inmersin 1000X, cultivos de gonococo negativos, exmenes para C.
Trachomatis y UU especficos. El estudio de C. Trachomatis se realiza principalmente mediante inmunoflurescencia directa con un 90%
de sensibilidad, y no necesita viabilidad del germen. Actualmente se cuenta tambin con PCR, con un 95% de sensibilidad. El cultivo es
caro y complejo por lo que no se realiza en clnica. Tratamiento: Azitromicina 1 g VO DU o Tetraciclina 500 mg VO c/6 h por 7 das o
Doxiciclina 100 mg VO c/12 h por 7 das. Mujeres embarazadas o lactancia: Azitromicina 1 g VO DU o Eritromicina 500 mg VO c/6 h por
7 das Recurrencia y persistencia: Metronidazol 2 g VO DU o 500 mg c/12 h por 7 das + eritromicina 500 mg VO c/6 h por 7 das. SIFILIS
(Treponema pallidum): La sfilis se conoce tambin como les, mal glico, es una enfermedad crnica adquirida por contacto sexual
(aunque tambin debe tenerse en cuenta la infeccin prenatal), caracterizada por una variedad de manifestaciones clnicas ya que
puede afectar prcticamente todas las estructuras del organismo; es una enfermedad intermitente que tiene perodos de actividad
(fase primaria, secundaria y terciaria) y periodos prolongados de latencia. El Treponema pallidum es una espiroqueta, incapaz de
sobrevivir fuera del husped. La transmisin ocurre mediante la penetracin de la espiroqueta en las membranas mucosas o abrasiones
en las superficies epiteliales. El tiempo de incubacin desde la exposicin a la aparicin de las lesiones primarias es en promedio de 3
semanas con un rango de 10 a 90 das. La lesin primaria es una ppula asintomtica que al paso del tiempo sufre necrosis superficial
convirtindose en una ulceracin bien circunscrita denominada chancro, sta siempre aparecer en el sitio de contacto primario o de
inoculacin, y puede acompaarse de linfadenopata inguinal. Este perodo conocido tambin como primario es autorresolutivo, es
decir, el chancro desaparece en 3 a 12 semanas en forma espontnea Posteriormente hay diseminacin linftica y sangunea del
microorganismo y la enfermedad pasa a una etapa latente asintomtica. Dos a tres meses despus aparece la expresin de esta
septicemia, el secundarismo, que dura varias semanas y se manifiesta por un amplio espectro de lesiones en piel y sntomas generales
con linfadenopata generalizada (Figura 2 y 3). En este momento el enfermo es altamente contagioso an al tacto de las lesiones ya que
estas contienen una gran cantidad de espiroquetas. Es tambin en este periodo que las mujeres embarazadas y contagiadas pueden
infectar al feto va transplacentaria. A continuacin se presenta un perodo de latencia que es el tiempo entre la resolucin de las
lesiones clnicas y la aparicin de las manifestaciones tardas de la enfermedad, y puede durar aos inclusive. Aproximadamente el 70%
de los individuos que no han recibido tratamiento permanecern en este estadio por el resto de sus vidas. La sfilis latente, se divide en
temprana (cuando dura menos de un ao) o tarda (duracin de mas de un ao) y tpicamente encontramos serologa positiva con
anticuerpos especficos al T. pallidum y el paciente se encuentra asintomtico. Durante la etapa terciaria encontramos muy pocos
microorganismos y una respuesta inmune celular aumentada al microorganismo. La espiroqueta en este periodo puede invadir el
MANUAL DE TRABAJO DEL CURSO ENARM CMN SIGLO XXI
CURSO ENARM CMN SIGLO XXI TEL: 36246001 Pharmed Solutions Institute PGINA 285

sistema cardiovascular (80%-85%), sistema nervioso central (5%-10%), piel y otros rganos; debido a la respuesta de hipersensibilidad
retardada se producen reas de inflamacin local y formacin de gomas en el tejido afectado. La penicilina contina siendo el
tratamiento de eleccin para todos los estadios de sfilis. En sfilis temprana (primaria, secundaria y latente temprana adquirida de
menos de 1 ao de duracin) en pacientes inmunocompetentes se recomienda una aplicacin nica de 2.4 millones de unidades de
penicilina benzatnica intramuscular o 600000 unidades de penicilina procanica intramuscular durante 10-14 das. En personas
alrgicas a la penicilina se puede utilizar doxiciclina 200mg diariamente por 14 das; tetraciclina 500mg cuatro veces al da durante 14
das; eritromicina 500mg cuatro veces al da durante 14 das o ceftriaxona intramuscular por 10 das. En sfilis tarda latente (adquirida
de ms de un ao de duracin o cardiovascular) se recomiendan 2.4 millones de unidades de penicilina intramuscular en tres dosis o
penicilina procanica 600000 intramuscular por 17 a 21 das. Las alternativas son doxiciclina 200mg diariamente por 21-28 das;
tetraciclina 500mg cuatro veces al da por 28 das y eritromicina 500mg cuatro veces al da durante 28 das. En neurosfilis el esquema
teraputico es de 24 millones de unidades de penicilina G acuosa intravenosa por da dividida en 6 dosis por 10 a 14 das o penicilina
procanica 2.4 millones de unidades intramuscular diariamente junto a probenecid 500mg cuatro veces al da durante 10-14 das. El
rgimen alternativo es doxiciclina 200mg diariamente por 28-30 das. LINFOGRANULOMA VENREO (Chlamydia trachomatis): Tambin
conocido como enfermedad de Durand-Nicolas-Favre o como linfogranuloma inguinal es una enfermedad causada por Chlamydia
trachomatis. En Mxico es poco frecuente con una tasa de incidencia de 0.2% por cada 100000, sin embargo en algunos pases de
frica y Asia se considera una enfermedad endmica. La infeccin ocurre despus del contacto directo con la piel o las membranas
mucosas de la pareja sexual infectada. El microorganismo (C. trachomatis) no penetra la piel intacta, se necesita que exista una solucin
de continuidad para que pueda hacerlo; viaja por los vasos linfticos hasta los ganglios donde se replica dentro de los macrfagos e
inicia la respuesta inflamatoria. A pesar que la mayora de los casos son predominantemente por contacto sexual, tambin hay casos de
infeccin por accidentes de laboratorio, fomites o contacto directo. El linfogranuloma venreo ocurre en tres etapas. En la primera, que
puede ser asintomtica y pasar desapercibida, aparece una ppula no dolorosa en el sitio de inoculacin que rpidamente se convierte
en pstula y se ulcera (chancro de inoculacin) con resolucin espontnea de la misma en una semana aproximadamente. La segunda
etapa se presenta de 2 a 6 semanas despus de la lesin primaria y consiste en linfadenopata inguinal. Finalmente, la tercera, que es
mas frecuente en mujeres y aparece aos despus de la infeccin, se manifiesta como proctocolitis y otros padecimientos rectales. El
diagnstico se basa principalmente en las caractersticas clnicas del paciente, sin embargo es necesario realizar estudios paraclnicos
para confirmar el agente etiolgico. La aspiracin del material purulento de los bubones es la mejor muestra que se puede obtener
para realizar la siembra de C. trachomatis que generalmente crece en medio de cultivo con cerebro de ratn El tratamiento de eleccin
recomendado es con azitromicina 1g en dosis nica. Otra opcin es doxiciclina dos veces al da va oral durante 7 das o eritromicina
500mg cuatro veces al da va oral durante 7 das. Finalmente se pueden indicar quinolonas del tipo de ofloxacino 300mg va oral
durante 7 das o levofloxacino 500mg va oral diariamente por 7 das. El drenaje de los bubones puede ayudar en la disminucin de la
sintomatologa y prevenir la formacin de lceras. CHANCROIDE (Haemophilus ducreyi): Es conocido tambin como chancro blando
o ulcus molle, se caracteriza por lceras genitales dolorosas e invasin regional a los ganglios linfticos con formacin de bubones sin
manifestaciones sistmicas. La caracterstica de estas lceras es que son blandas (de ah su nombre) con fondo sucio y rpidamente
necrosantes. Las lesiones se pueden autoinocular a otras reas del cuerpo muy fcilmente. El agente causal es Haemophilus ducreyi,
una bacteria Gram negativa anaerobia facultativa. En Mxico, en el 2006 se report una tasa de incidencia de 0.5% por cada 100000
habitantes. El periodo de incubacin es entre 3 y 10 das. Se inicia con la aparicin de una ppula rodeada de eritema la cual se
transforma con rapidez en pstula y deja una ulceracin muy dolorosa, bien limitada y no indurada que tiene aspecto de tejido de
granulacin con base necrtica en la mayora de los casos y un exudado de aspecto sucio. Se localiza principalmente en genitales y
regin perianal. En hombres se puede encontrar en prepucio, en el surco coronal o en el frenillo, y no es raro que exista un edema
importante del prepucio. En mujeres, la localizacin ms frecuente es el introito vaginal, aunque tambin se encuentra en el crvix o el
rea perianal. En una frecuencia menor se encuentra en localizacin extragenital. Puede ocurrir linfadenopata dolorosa regional hasta
en el 50% de los casos. El regimen antibitico recomendado actual para el tratamiento es con azitromicina 1gr va oral en dosis nica;
otra opcin es ceftriaxona 250mg va intramuscular en dosis nica, se puede utilizar tambin ciprofloxacino 500mg dos veces al da
durante 3 das o bien eritromicina 500mg tres veces al da durante 3 das. DONOVANOSIS: Esta ETS tambin conocida como granuloma
inguinal, se caracteriza por la presencia de lceras genitales que tienden a sangrar con facilidad. El agente etiolgico
es Calymmatobacterium granulomatis, una bacteria pleomrfica gramnegativa; sin embargo, y dada la similitud filognetica que tiene
con el gnero Klebsiella, se ha propuesto el cambio de nombre a Klebsiella granulomatis. En Mxico es muy rara, y los casos que se
logran detectar son en personas que han viajado a zonas endmicas. El modo de transmisin es primariamente sexual, aunque la
infeccin tambin puede adquirirse mediante fmites, va oral-fecal por alimentos contaminados o a travs del canal del parto. El
periodo de incubacin vara dependiendo de la fuente que se cite, se reporta desde 1 da hasta 1 ao. La lesin inicial es una ppula
firme, usualmente asintomtica, que se expande lentamente en semanas o meses; debido a que contiene muchos elementos
vasculares, sangra con traumatismos mnimos y se ulcera. Una vez que se ha formado la lcera, inicia la destruccin del tejido
adyacente con exudado maloliente y presencia de tejido necrtico. La confirmacin diagnstica se logra mediante la identificacin de
cuerpos de Donovan (que son colonias de C. granulomatis) dentro de los macrfagos en un frotis de la lcera teido con Giemsa,
Wright o en biopsia de las lesiones. El tratamiento antibitico recomendado es a base de doxiciclina 100mg diariamente va oral por lo
menos durante 3 semanas o hasta que las lesiones hayan cicatrizado por completo. Como alternativa se puede emplear azitromicina
1gr va oral una vez a la semana durante 3 semanas. As tambin se puede utilizar ciprofloxacino 750mg dos veces al da durante por lo
menos 3 semanas. INFECCIN POR VIRUS DEL HERPES SIMPLE: El virus del herpes simple es un microorganismo patgeno que causa
infeccin orolabial (VHS-1) y genital (VHS-2) caracterizadas por erupciones vesiculares primarias y recurrentes. Estos dos tipos: el virus
del herpes simple tipo 1 y 2, que aunque estn estrechamente relacionados en cuanto a morfologa y patogenicidad, difieren
epidemiolgicamente. Las infecciones por el VHS-1 se presentan con gran variabilidad y la regla es que se observen hasta en el 80% de
los casos en forma asintomtica; se desconoce el nmero exacto de personas infectadas ya que se considera que ms del 95% de la
poblacin mundial tiene anticuerpos contra este virus que contrajo en el 1er ao de vida. En Mxico se desconoce la prevalencia de la
infeccin por el VHS tipo 2 que es considerado como ETS; sin embargo se sabe que en el 2006 hubo una tasa de incidencia de herpes
genital de 1.4% por cada 100000 habitantes. El espectro clnico del herpes simple genital va desde la primo- infeccin primaria que
raramente se observa para posteriormente darnos los episodios de recurrencia que son variables en frecuencia e intensidad. En el sitio
MANUAL DE TRABAJO DEL CURSO ENARM CMN SIGLO XXI
CURSO ENARM CMN SIGLO XXI TEL: 36246001 Pharmed Solutions Institute PGINA 286

de entrada del virus, que generalmente son los genitales externos aparecen vesculas que se agrupan sobre una base eritematosa, stas
fcilmente progresan a pstulas y lceras. No es infrecuente encontrar durante la primoinfeccin una balanitis erosiva, vulvitits o
vaginitis. En las mujeres las lesiones tambin pueden encontrarse en crvix, nalgas y perineo. Se requiere de un diagnstico certero que
es fundamentalmente clnico, recordando que cada episodio es autorresolutivo en 3 a 7 das. El estndar de oro para el diagnstico es
el cultivo del virus, sin embargo es una tcnica difcil de lograr y muy costosa por lo que se reserva solo para casos especiales. Al ser una
ETS recidivante pero autorresolutiva a corto plazo, el tratamiento es generalmente sintomtico con analgsicos, aplicacin de
compresas fras, y medidas adecuadas de higiene. Debe hacerse hincapi en explicar amplia y detalladamente la evolucin de la
infeccin, es decir que habr periodos en que el paciente se encuentre asintomtico y sin lesiones y otros en los que aparecern los
brotes que tendrn la duracin ya mencionada y puede acompaarse de ardor discreto. El empleo de aciclovir y otros antivirales
similares ayudarn a reducir la severidad y la duracin de las recurrencias solamente. No existe hasta la actualidad ningn frmaco o
vacuna curativos. Debemos hacer hincapi en que los antivirales por va tpica son totalmente ineficaces.

CASO CLINICO
Es ingresada paciente femenino de 21 aos que se encuentra en condicin de calle, fue trada por una patrulla que refiere se
encontraba solicitando ayuda, la paciente se encuentra con alteracin del sensorio solo dice estar embarazada y que tiene dolor tipo
contracciones, no sabe cuantas semanas de embarazo tiene a la exploracin se observa paciente con estado de desnutricin moderada,
alteracin del estado de alerta, con actitud alucinada, huele a tiner, presenta adems escoriaciones e irritacin peribucal, fondo uterino
de 25 cm, contracciones irregulares, una contraccin cada 10 minutos aproximadamente, presenta lesiones en labios mayores y
linfadenopatias en regin inguinal y axilar.

PREGUNTA
Cuales son las manifestaciones mas importantes para el diagnostico clnico de esta patologa.

RESPUESTA
a.- Vesiculas bucal o/y genital.
b.- Ulcera genital limpia, nica y elevada.
c.- Ulcera purulente, destructiva y signo de Follman.
d.- Ulcera limpia, vesculas, disuria y prurito.

CASO CLINICO
A primigesta de 24 aos de edad con amenorrea secundaria sin determinar fecha y dolor abdominal y lumbar que irradia en su muslo
derecho. Durante los tres meses anteriores, el paciente descrito sangrado vaginal irregular y flujo vaginal acuoso. IVSA a los 14 aos de
edad y se observ un mnimo de 15 parejas sexuales anteriores. Resultados positivos de examen toxicolgico de drogas ilegales.
Durante los ltimos 6 aos, los resultados del cultivo y serologa document cuatro infecciones previas por separado con Chlamydia
trachomatis, Neisseria gonorrhoeae, herpes simplex virus 2 y Treponema pallidum, respectivamente. El paciente inform que complet
el tratamiento para las cuatro infecciones.

PREGUNTA
Cul es el padecimiento infeccioso que ms probable presenta.

RESPUESTA
a.- Amoxicilina 500 mg + probenecid 500 mg diario.
b.- Doxiciclina 2 mg diario.
c.- Eritromicina 500 mg diario.
d.- Ceftriaxona 1 g diario.

CASO CLINICO
Acude a consulta paciente femenino de 24 aos de edad, refiere que inicia padecimiento hace 2 semanas aproximadamente
caracterizado por flujo amarillento, en cantidad moderada, agrega que su esposo tiene secrecin amarillenta proveniente del meato
urinario la cual es ardorosa.

PREGUNTA
Cual es la conducta a seguir ms apropiada para el manejo de incapacidad y referencia.

RESPUESTA
a.- Incapacidad y referencia a segundo nivel.
b.- 7 dias de incapacidad y referencia a segundo nivel.
c.- Iniciar el tratamiento e incapacidad de 24 hrs.
d.- Incapacidad hasta resolucin del padecimiento.


MANUAL DE TRABAJO DEL CURSO ENARM CMN SIGLO XXI
CURSO ENARM CMN SIGLO XXI TEL: 36246001 Pharmed Solutions Institute PGINA 287

ENFERMEDAD PELVICA INFLAMATORIA (EPI). CIENCIAS BASICAS: Comprende las alteraciones inflamatorias e infecciosas que afectan
los rganos genitales situados en la pelvis menor. Incluye combinaciones de cervicitis, endometritis (utero), salpingitis (salpinges),
anexitis, parametritis (ligamentos de soporte) y pelviperitonitis, as como abscesos tubo ovricos. SALUD PUBLICA: Es ms frecuente y
grave en mujeres de 15-25 aos. 1 de cada 4 EPI experimentan secualeas. Su importancia radica no slo en la morbilidad aguda, sino en
su capacidad de producir secuelas como esterilidad, gestacin ectpica, recidivas y dolor abdominal crnico. Tasa de incidencia anual
de 17.2 por mil mujeres entre 15-44 aos. Afecta cada ao del 1-2% de mujeres jvenes sexualmente activas. PATOGENIA: La EPI es la
complicacin ms frecuente de las enfermedades de transmisin sexual bacterianas. Suele ser polimicrobiana. Los patgenos ms
frecuentes son: Neisseria Gonorrhoeae (5-36%) y Chlamydia Trachomatis (17-70%), Aerobios (Estreptococos del grupo B, Escherichia
coli 30%, Gardenella vaginalis, Mycoplasma hominis 37-75%), anaerobios (Peptoestreptococos, Bacteroides fragilis 60%), y un 5% de
los casos patgenos respiratorios (Haemophilus influenzae 5%, Streptococcus pneumoniae, Streptococcus pyogenes). Las bacterias
pueden acceder a la porcin superior del aparato genital por diseminacin de rganos adyacentes infectados (apendicitis, diverticulitis),
por diseminacin hematgena de focos distantes (tuberculosis) y fundamentalmente por diseminacin ascendente transuterina a
partir del tracto genital inferior. Como resultado de esta infeccin ascendente que alcanza las trompas y de la reaccin inflamatoria que
se produce, van a ocurrir dentro de la trompa fenmenos de vasodilatacin, trasudacin de plasma y destruccin del endoslpinx con la
consiguiente produccin de un exudado purulento. En los primeros estados de la enfermedad, la luz de la trompa permanece abierta
con lo cual se permite que este exudado salga por la fimbria hacia la cavidad plvica, lo que produce peritonitis plvica. Como resultado
de esta inflamacin peritoneal, las estructuras adyacentes como ovarios, ligamentos anchos, omento, intestino delgado, sigmoide y
ciego se comprometen en el proceso inflamatorio. La tensin de oxgeno en estos tejidos inflamados y necrticos disminuye lo que
favorece el crecimiento de flora anaerobia. La destruccin del tejido con la posterior degradacin lleva a la formacin de abscesos. El
flujo menstrual puede facilitar la insercin del tracto superior debido a la prdida del revestimiento endometrial. Los factores de riesgo
principales son: Edad inferior a 25 aos, mltiples compaeros sexuales, ETS, no utilizacin de mtodos de barrera, historia previa de
EIP, historia de vaginosis, cervicitis, dispositivos intrauterinos, solo tiene relacin con la EPI en los 3 meses posteriores a la insercin por
la manipulacin, abortos, instrumentacin uterina, ciruga cervical, trmino de embarazo. CLASIFICACIN: Se basa fundamentalmente
en el grado evolutivo de la enfermedad, en la sintomatologa y en los datos recogidos en la exploracin. Podemos diferenciar cuatro
estadios (Monif): Estadio I (leve): Salpingitis aguda sin pelviperitonitis. Estadio II (moderada): Salpingitis aguda con pelviperitonitis.
Estadio III (severa): Salpingitis con formacin de abscesos tuboovricos. Estadio IV (muy severa): Rotura de absceso y choque septico.
Clasificacion laparoscpica: Leve; eritema, edema, las trompas se mueven libremente, no hay exudado purulento. Moderada; eritema,
edema mas marcado, material purulento evidente. No hay movimiento libre de las trompas. La fimbria puede no ser evidente. Severa;
presencia de piosalpinx y/o absceso. DIAGNOSTICO: puede cursar con los siguientes sntomas: dolor abdominal bajo (95%), y su
intensidad vara de totalmente ausente (EIP silente) a muy intenso en cuadros con un componente peritoneal importante. Aumento del
flujo vaginal, flujo de caractersticas anormales (74%). Sangrado anormal (intermestrual, poscoital) (45%). Sntomas urinarios (35%),
como disuria y polaquiuria. Sntomas digestivos (14%), como nauseas, vmitos, diarrea,... Es posible la ausencia de sntomas. Y en ella
podemos encontrar estos signos: dolor a la movilizacin del cuello, dolor anexial en la exploracin vaginal bimanual (99%). En el
examen con espculo observamos cervicitis y descarga endocervical purulenta (74%). Fiebre (> 38 C) (< 47%). Masa plvica: sugiere
absceso tuboovrico (ATO). Signos de peritonitis. Las pacientes con infeccin por Chlamydia pueden cursar con salpingitis subclnica o
subaguda, con secuelas de adherencias e infertilidad. El diagnstico clnico es con frecuencia dificultoso por la poca especificidad y
sensibilidad de la historia clnica y los estuios de laboratorio, sin embargo el retraso en el diagnstico y tratamiento puede producir
secuelas importantes. Ante la sospecha de EPI
debemos realizar: Hemograma y bioqumica
general, con determinacin de VSG y PCR, test de
embarazo en orina o HCG srica., tomas
vaginales y endocervicales para deteccin de
gonococo y clamidias. Ecografa, sobre todo en su
forma transvaginal, ya que se obtendr
informacin sobre la existencia o no de abscesos
tuboovricos. La laparoscopia es el nico mtodo
fiable para el diagnstico, que permite una visin
directa y la toma de cultivos. Se reserva en casos
seleccionados (diagnstico dudoso o fracaso del
tratamiento). Biopsia endometrial con cnula de
aspiracin para cultivo microbiolgico y
diagnstico anatomopatolgico. Serologa
completa. El diagnstico clnico se basa en los
criterios de Hager modificados por la SEGO. Se
requiere la presencia de todos los criterios mayores y de al menos uno menor. CRITERIOS MAYORES: Dolor en abdomen inferior. Dolor
a la movilizacin del cervix. Dolor anexial a la exploracin abdominal. Historia de actividad sexual en los ltimos meses. Ecografa no
sugestiva de otra patologa. CRITERIOS MENORES: Temperatura > 38C. Leucocitosis >10500 /l. VSG elevada. GRAM de exudado
intracervical con diplococos intracelulares (gonococo), cultivo positivo para N.Gonorrhoeae o C.Trachomatis. SINDROME FITZ-HUGH-
CURTIS: Es una perihepatitis, se asocia con EPI, principalmente por Chlamydia, hay desarrollo de adherencias y fibrosis perihepaticas,
dolor agudo en cuadrante superior derecho, no hay alteracin de enzimas hepticas. TRATAMIENTO: Ante la sospecha de EPI se debe
instaurar tratamiento antimicrobiano emprico, aun en ausencia de causa confoimada si hay: dolor a la palpacin en abdomen inferior,
en anexos o con el movimiento del cuello uterino. Debe realizarse precozmente para prevenir las secuelas que pueden producirse
incluso en casos de infeccin leve. En las pacientes con estadio I sin criterios de ingreso hospitalario se prescribir tratamiento
ambulatorio con: Rgimen A: Cefotaxima 2g IM + 1g de probenecid VO DU (tratamiento de primera eleccin) Ceftriaxona 250mg IM +
doxiciclina 100mg VO c/12h por 14 dias (mismo tratamiento para la pareja sexual) Rgimen B: Ofloxacina 400mg VO c/12 hrs por 14
dias + clindamicina 450mg VO c/6h metronidazol 500mgs VO c/12h por 14 dias. Las pacientes que no respondan al tratamiento
MANUAL DE TRABAJO DEL CURSO ENARM CMN SIGLO XXI
CURSO ENARM CMN SIGLO XXI TEL: 36246001 Pharmed Solutions Institute PGINA 288

antibitico ambulatorio en 48 h. deben ser hospitalizadas para confirmar el diagnstico y realizar terapia parenteral. Tratamiento
hospitalario: Rgimen A: Cefoxitina 2g IV c/6h cefotetan 2g IV c/12h + doxiciclina 100mg IV o VO c/12h. Regimen B: Clindamicna
900mg IV c/8h + gentamicina 2mg/kg (1.5mg/kg c/8h) doxiciclina 100mg VO c/12h clindamicina 450mg VOc/6h por 14 dias. Regimen
C: Aztreonam 2-4g/dia IV + clindamicina 600mg IV c/6h. Este rgimen debe administrarse hasta 48 horas despus de que la paciente
muestra mejora clnica evidente. En caso de presencia de DIU es preciso la extraccin y cultivo del mismo. Si existe absceso mayor de 8
cm. o en estadio IV se proceder al drenaje quirrgico. En ocasiones, es necesaria la histerectoma y la anexectoma. Las parejas
sexuales de las pacientes con EIP deben ser estudiadas y tratadas si han mantenido relaciones en los 2 meses previo a la aparicin de
sntomas de EIP con 2 gr. de Azitromicina en dosis nica vo. o Ceftriaxona 250mg im. en dosis nica y Doxiciclina 100mg/12 horas vo. 7
das. CRITERIOS DE INGRESO: Requieren ingreso hospitalario las pacientes con EIP en estadio II, III y IV, y en estadio I cuando concurra
alguna de las siguientes circunstancias: Falta de respuesta a la antibioterapia despus de 48 h. Sospecha de incumplimiento
teraputico, temperatura > 38C, diagnstico incierto, riesgo quirrgico, nuseas y/o vmitos. Intolerancia oral, embarazo, signos de
reaccin peritoneal alta, sospecha de pioslpinx, absceso ovrico o tuboovrico, prepber o adolescente, gran inters en mantener la
fertilidad.

CASO CLINICO
Paciente de 19 aos de edad con antecedentes de rara malformacin uterina y vaginal, consistente en tero doble, el derecho
rudimentario, con hipoplasia cervical y agenesia de vagina. Fue intervenida a los 16 aos, practicando extirpacin de hemitero
rudimentario, creacin de neovagina con piel del abdomen, identificacin de crvix y colocacin de dispositivo intrauterino (DIU). Se
comprob durante la intervencin la existencia de ovarios y trompas normales. Posteriormente sigui controles y presentaba ciclos
regulares con menstruaciones normales. Inicia coitos sin uso de preservativo, practicando controles citolgicos sin hallazgos
patolgicos. El episodio motivo de esta presentacin se inicia tras la ltima regla, como cuadro de inicio brusco de dolor hipogstrico,
leucorrea y fiebre de 38 C. USG con pioslpinx. Se parti de una analtica inicial de 25.000 leucocitos/ml, protena C reactiva de 63,6
mg/dl y cultivos del DIU positivos para Streptococcus spp., Proteus spp. y Escherichia coli, con urocultivo negativo.

PREGUNTA
Cul de los siguientes sntomas clnicos es menos frecuente al dianostico de esta patologa:

RESPUESTA
a.- Uso de DIU.
b.- Dolor abdominal.
c.- Leucorrea.
d.- Sangrado irregular.

CASO CLINICO
Acude a consulta femenino de 33 aos de edad, refiere que hace 10 das fue diagnosticada con enfermedad inflamatoria, menciona que
ha tomado el tratamiento de forma irregular, regreso debido a que desde hace 24 horas inicio con dolor abdominal intenso, que se
incrementa cuando camina o hace algn movimiento, fatiga, adinamia, a la exploracin fsica observa TA 110/70, FC 89, FR 23,
Temperatura de 38.5 grados, mal estado generalizado, abdomen con datos de irritacin peritoneal, decide ingresarla y enva estudios
de laboratorio y gabinete debido a que considera que la paciente presenta una complicacin:

PREGUNTA
Cul es la complicacin ms probable que presenta la paciente:

RESPUESTA
a.- Peritonitis espontanea.
b.- Absceso tubo-ovrico.
c.- Salpingitis aguda.
d.- Ooforitis aguda.

CASO CLINICO
Una mujer de 35 aos de edad, con alcohol crnico y abuso de nicotina fue admitida con un historial de dos das de evolucin de dolor
en fosa iliaca izquierda, as como las temperaturas febriles de 38.8C y los parmetros de inflamacin elevados. Una historia previa de
tuberculosis fue negada por el paciente. En un examen por ultrasonido se observo lesin de 5x6 cm masa qustica en el rea del anexo
izquierdo.

PREGUNTA
Cul es el agente etiolgico mas frecuente.

RESPUESTA
a.- Neisseria g.
b.- Chalmydia.
c.- Mycoplasma.
d.- Ureaplasma .


MANUAL DE TRABAJO DEL CURSO ENARM CMN SIGLO XXI
CURSO ENARM CMN SIGLO XXI TEL: 36246001 Pharmed Solutions Institute PGINA 289

SANGRADO UTERINA DISFUNCIONAL (HUD). CIENCIAS BASICAS: La hemorragia uterina anormal es la causa ms comn de prdida
hemtica en la mujer en edad reproductiva. Las mujeres con hemorragia pueden
padecer anemia crnica, dolor plvico e incapacidad, enfrentando de tal forma un
problema mdico debilitante que afecta de manera adversa sus responsabilidades
laborales y familiares. HUD: se define como aquella en la que se producen cambios
en la frecuencia del ciclo menstrual, en su duracin, o en la cantidad de la prdida
sangunea; su diagnstico es de exclusin, lo que obliga al clnico a descartar
inicialmente alguna patologa orgnica (Tabla 1). sta puede catalogarse en 2
grandes rubros: la debida a causas orgnicas (hemorragia uterina anormal) y la que
es producida por anovulacin (hemorragia uterina disfuncional). Las principales
causas de hemorragia uterina disfuncional o anovulatoria se enlistan en la Tabla 2.
SALUD PUBLICA: HUD, es la segunda causa de consulta ginecolgica, despus de la
infecciones cervicovaginales, es la principal causa de hemorragia en la mujer adulta. Se estima que alrededor de 10 millones de mujeres
en Mxico sufren hemorragia uterina y anualmente, solo 6 millones de ellas buscan atencin mdica. CLASIFICACION: Por la FIGO
sistema PALM-COEIN: El sistema bsico engloba cuatro categoras que se definen
por criterios estructurales visualmente objetivos (PALM: plipos, adenomiosis,
leiomioma y malignidad o hiperplasia); cuatro afecciones no estructurales (COEI:
coagulopatia, disfuncin ovulatoria, disfuncin endometrial, iatrognica), que no
estn relacionadas con anomalas estructurales y una (N), reservada para las
afecciones no clasificables. La categora leiomioma (L), se subdivide en las
pacientes que tienen al menos un mioma submucoso y que tienen miomas que no
tienen efecto en la cavidad endometrial. PATOGENIA: En relacin a su
fisiopatologa, en un ciclo anovulatorio el cuerpo lteo no se desarrolla, el ovario
pierde su capacidad para producir progesterona pero la produccin estrognica
contina; esta situacin trae como consecuencia una proliferacin endometrial sin
una descamacin inducida por progesterona que culmine en una menstruacin. El
resultado clnico de esta eventualidad es una hemorragia no cclica, impredecible e
inconsistente en cuanto a volumen. La estimulacin estrognica continua y sin oposicin (de la progesterona) produce un crecimiento
endometrial inestable excesivamente vascular, sin la suficiente capa estromal que lo soporte, tornndolo frgil y vulnerable; en este
contexto la descamacin endometrial es irregular, prolongada e impredecible. En el endometrio de las mujeres con hemorragia uterina
se han encontrado grandes cantidades de prostaglandinas (PGE2 y PGF2) cuando se les compar con mujeres con ciclos menstruales
regulares. Incluso existe evidencia deque en los trastornos de la homeostasis (coagulacin anormal) la proporcin de PGE2/PGF2 y la de
prostaciclina (PGI2)/tromboxano (TXA2) estn elevadas. Estas prostaglandinas estn presentes, tanto en el endometrio como en el
miometrio, y el mecanismo exacto por el cual producen prdida sangunea elevada an es especulativo. DIAGNOSTICO: La hemorragia
uterina disfuncional es un diagnstico de exclusin, por lo cual el clnico debe descartar inicialmente cualquier patologa orgnica o
endocrinolgica. Para una adecuada evaluacin clnica conviene estratificar por edad a las pacientes, debido a que las de mayor edad
incrementan el riesgo de patologas malignas o premalignas. EDAD REPRODUCTIVA (19 A 39 AOS DE EDAD): Aproximadamente, entre
un 6 a 10% de las mujeres con HUD tienen hiperandrogenismo con anovulacin crnica (sndrome de ovarios poliqusticos), lo cual
incluye trastornos en el ciclo menstrual, hirsutismo y obesidad (IMC>25 kg/m2). El 65% de las mujeres con hirsutismo y anovulacin
crnica son obesas. En nuestro pas, el 37.4% de las mujeres tienen sobrepeso y el 34.5% obesidad, por lo que al sumar ambas
prevalencias, tenemos un 71.9% de mujeres de 20 aos y mayores (esto es en mujeres en edad reproductiva) con trastornos en la
alimentacin. En las mujeres con obesidad, irregularidades menstruales (oligo u anovulacin) y datos de hiperandrogenismo deber
descartarse sndrome de ovarios poliqusticos. En mujeres con una rpida progresin de hirsutismo acompaada de virilizacin, debe
sugerir al clnico descartar un tumor suprarrenal. En la mayora de los casos, la cuantificacin de los niveles de testosterona, de sulfato
de dehidroepiandrosterona y de 17-hidroxiprogesterona, puede guiarnos al diagnstico. La evaluacin debe considerar tambin la
valoracin de biometra hemtica, una prueba de embarazo, de los niveles de prolactina y de la hormona estimulante del tiroides (TSH).
La evaluacin debe considerar tambin la existencia de embarazo, hiperprolactinemia y trastornos Tiroideos. Cuando se sospecha de
falla ovrica precoz la estimacin de los niveles de FSH sern de utilidad. La anovulacin es la causa ms frecuente de amenorrea en las
mujeres que experimentan amenorrea secundaria. La anovulacin crnica que resulta de una disfuncin hipotalmica se diagnostica
por niveles bajos o normales de FSH, y puede ser el resultado de estrs fisiolgico, ejercicio en exceso o prdida de peso. A las mujeres
con amenorrea que presentan una prueba negativa de embarazo, niveles normales de FSH, TSH y prolactina podemos catalogarlas
como anovulacin. MUJERES EN EDAD REPRODUCTIVA TARDA (DE LOS 40 AOS HASTA LA MENOPAUSIA): La incidencia de HUD se
incrementa de manera paralela con la edad, y los ciclos anovulatorios continuos representan la declinacin de la funcin ovrica. En
estas mujeres, la causa ms frecuente de hemorragia uterina no es precisamente la hiperplasia endometrial ni el cncer endometrial,
sino las patologas intracavitarias como plipos endometriales y miomas submucosos. En las mujeres de esta edad con HUD a las cuales
se les ha descartado inicialmente alguna causa orgnica, y que persisten con episodios de hemorragia a pesar de haberse instaurado un
tratamiento adecuado, debern revalorarse en la bsqueda de causas malignas o premalignas. El clnico no debe olvidar la estrecha
relacin de algunos de los parmetros clnicos presentes en la mujer con HUD, como la obesidad y la anovulacin en la gnesis del
cncer endometrial. La biopsia de endometrio es una tcnica sencilla, relativamente simple y que puede realizarse en el consultorio; las
pacientes menores a 35 aos de edad con HUD que no respondan a la terapia mdica instaurada son candidatas a la biopsia de
endometrio y las mujeres mayores de 40 aos de edad y HUD debern someterse a una biopsia de endometrio. La histeroscopia de
consultorio es una tcnica de visualizacin directa que detecta un alto porcentaje de anormalidades intracavitarias. Ultrasonido
transvaginal (UTV), la sonohisterografa (SHG) y la histeroscopia de consultorio en la deteccin de lesiones intracavitarias, se reportan
una sensibilidad y especificidad del UTV en un 56.3 y 100%; para la SHG 72% y 87.5%; finalmente, para la histerosopia un 100% y 100%.
Los autores concluyen que la certeza diagnstica de la SHG es equiparable a la histeroscopia. Sin embargo, se reporta menos dolor con
la SHG que con la histeroscopia. Desafortunadamente, es una tcnica que requiere entrenamiento, y el instrumental necesario para su
MANUAL DE TRABAJO DEL CURSO ENARM CMN SIGLO XXI
CURSO ENARM CMN SIGLO XXI TEL: 36246001 Pharmed Solutions Institute PGINA 290

realizacin es caro. El objetivo de la evaluacin de la cavidad uterina en las mujeres con HUD incluye la deteccin de lesiones focales
(miomas o adenomiosis) y alteraciones a nivel endometrial (engrosamiento endometrial o plipos). El ultrasonido transvaginal (UTV) es
una herramienta til para el diagnstico de lesiones focales como miomas, ya que logra detectar alguna patologa hasta en un 97% de
los casos en mujeres premenopusicas, sin embargo, pierde certeza diagnstica en lesiones intracavitarias. La evaluacin por UTV del
endometrio que resulte en un grosor mayor de 18 mm sugiere, fuertemente, alguna patologa a este nivel. TRATAMIENTO: No existe
evidencia suficiente para afirmar que el uso de anticonceptivos orales (AOC), solos o comparados con otros tratamientos (AINES,
danazol, DIU) sean benficos para la mujer con HUD en relacin a la mejora de su sintomatologa. No existen ventajas con el uso de
progestinas para el control de HUD si se les compara con danazol, AINES o con el dispositivo impregnado con levonorgestrel; sin
embargo, el uso de progesterona por 21 das parece reducir de manera significativa la prdida sangunea en pacientes con HUD, por lo
que este rgimen se puede administrar de manera inmediata para el control de la hemorragia y solo como un tratamiento temporal. Es
insuficiente la evidencia para sealar que el uso de DIU-IL sea la mejor opcin, en comparacin con el uso de noretindrona continua en
las mujeres con HUD; pero se reportarn mayores efectos adversos en el grupo de DIU-IL. El danazol es una mejor alternativa para el
tratamiento de HUD, en comparacin a placebo, progestinas, AINES y AHO, pero presentan mayores efectos adversos. Debido al
pequeo nmero de trabajos, no es posible emitir una recomendacin adecuada para su uso en la prctica clnica en las pacientes con
HUD. Tratamiento quirrgico: El legrado uterino instrumentado es la intervencin diagnstica o teraputica ms empleada en nuestro
medio, no cuenta con la suficiente evidencia cientfica sobre su utilidad, sola o comparada contra intervenciones. La ablacin
endometrial solo est indicada en mujeres con paridad satisfecha y que no desean histerectoma total abdominal. Si se le compara con
el tratamiento mdico, resulta ser significativamente ms efectiva para controlar la hemorragia a 4 meses de seguimiento, sin evidencia
de efectividad a ms largo plazo. No hay diferencia en un ao respecto a la calidad de vida, al compararse con el DIU-IL. Para las
mujeres que tienen satisfecho su deseo reproductivo, y en las que se han utilizado las diferentes opciones teraputicas (tanto mdicas
como quirrgicas) sin haberse encontrado una respuesta satisfactoria a su problema, la histerectoma representa la mejor opcin al ser
curativa y mejorar la calidad de vida, aunque tenga mayor riesgo de complicaciones.

CASO CLINICO
Mujer de 27 aos con antecedentes de migraa y colecistectoma laparoscpica 2 aos antes, presenta alteraciones en la cantidad y
frecuencia del ciclo menstrual asi como flujo intermenstrual recurrente, como antecedentes la paciente le fue realizada cesarea por
placenta de implantacin baja total hace 6 meses, hay antecedentes familiares de cncer endometrial y quistes ovricos.

PREGUNTA
Cual es la conducta a seguir para establer un escrutinio diferencial.

RESPUESTA
a.- USG enfocndose en anexos.
b.- Realizar PIE.
c.- Realizar evaluacin de gonadotrofina.
d.- Biopsia endometrial.

CASO CLINICO
Paciente de 26 aos de edad se presenta a la consulta refiriendo que desde hace un ao posterior a un parto via vaginal inicia
padecimiento caracterizado por alteraciones de su periodo mentrual, refiere que anteriormente era regular pero ahora no, adems
agrega infecciones vaginales frecuentes, es usuaria de DIU su DOC fue normal a la exploracin presenta leve dolor a la movilizacin de
crvix.

PREGUNTA
Cul es la conducta a seguir.

RESPUESTA
a.- Iniciar mtodo de barrera.
b.- Realizar USG.
c.- Retirar DIU.
d.- Envio a colposcopia.

MIOMATOSIS UTERINA. CIENCIAS BASICAS: Los miomas son una enfermedad benigna comn del tero, son tumores que se inician en
una sola clula de msculo liso uterino y pueden crecer en cualquier parte del tero bajo la influencia de factores de crecimiento local,
ctocinas y hormonas sexuales; ocasionan trastorno a nivel endometrial. La miomatosis uterina es la presencia de uno o varios tumores
benignos formados por fibras musculares del tero (matriz) y se considera la tumoracin ms frecuente del aparato genital femenino.
Tambin se le denomina leiomiomas o fibromas uterinos. SALUD PUBLICA: La prevalencia de la miomatosis uterina en mujeres en edad
frtil se calcula entre 20 y 40%, siendo ms comn en mujeres afroamericanas, y contando como factores de riesgo la edad,
nuliparidad, tabaquismo y otros. Su incidencia acumulada para la edad de 50 aos es de ms del 80% en mujeres afroamericanas y 70%
en mujeres caucsicas. Se han asociado con infertilidad en 5 a 10% de los casos, y se estima que se encuentran como factor nico
presente en 1 a 3% de los casos. CLASIFICACION: Se ha clasificado a los miomas uterinos como submucosos si distorsionan la cavidad
uterina, intramurales si residen predominantemente dentro de la pared miometrial uterina y subserosos si protruyen fuera de la
superficie uterina. El nmero y localizacin de los miomas correlaciona con los sntomas y el efecto en la fertilidad. Existen otras
localizaciones poco comunes: cervicales (en el cuello de la matriz), intraligamentarios (dentro de los ligamentos que sostienen al tero)
o incluso miomas parsitos, que son aqullos que al ir creciendo van tomando el aporte sanguneo de estructuras vecinas, pudiendo
incluso llegar a quedar completamente separados del tero. PATOGENIA: Predisposicin del sitio anatmico a formar miomas. Se cree
MANUAL DE TRABAJO DEL CURSO ENARM CMN SIGLO XXI
CURSO ENARM CMN SIGLO XXI TEL: 36246001 Pharmed Solutions Institute PGINA 291

que son causados por una excesiva estimulacin de los estrgenos, que actan sobre una o varias clulas de las fibras musculares del
tero susceptibles a responder en forma exagerada, dando lugar a la proliferacin desmedida y la formacin de tumores. Se piensa que
es un tumor estrgeno-dependiente, ya que crece durante la edad reproductiva y en ocasiones revierte o disminuye en la menopausia
cuando ya no hay el estmulo estrognico. El uso de hormonales orales en forma constante incrementa su desarrollo DIAGNOSTICO:
Depender en gran medida de la localizacin, tamao y nmero de los miomas: Subserosos: Su sintomatologa se relaciona con la
compresin a rganos vecinos. Si comprimen la vejiga, pueden dar sntomas de infeccin urinaria (ardor al orinar, miccin frecuente y
en poca cantidad, etc.), o incluso incapacidad para contener la orina. Si comprimenal intestino grueso podrn ocasionar constipacin,
colitis, dolor durante la evacuacin o incluso oclusin intestinal. La compresin sobre las arterias o venas de la pelvis condicionar el
desarrollo de varicosidades, inflamacin de miembros inferiores o sensacin de dolor y de pesantez en las piernas. Intramurales:
Interfieren con la contractilidad uterina, comprimen los plexos venosos y arteriales y, a la larga, condicionan la presencia de
menstruaciones prolongadas y abundantes, sangrado nter menstrual y dolor durante la menstruacin, adems de estar relacionados
en muchos casos con esterilidad. Submucosos: Deforman el interior de la cavidad uterina y son los que con mayor frecuencia ocasionan
aumento en el sangrado menstrual (hiper-poli menorrea) adems de clicos. Son un importante factor causal de esterilidad y pueden
crecer tanto que salen a travs del cuello uterino (miomas abortados). La exploracin bimanual clnica es el segundo paso para un
diagnstico adecuado, pudiendo detectarse un crecimiento uterino exagerado o la delimitacin de la tumoracin. De ah la importancia
de que la paciente haya vaciado la vejiga antes de la exploracin y de que se encuentre en una posicin cmoda, con los msculos
abdominales completamente relajados, para permitir un examen adecuado. Siempre que se sospeche la presencia de miomas, est
indicada la realizacin de un ultrasonido plvico, lo que permitir determinar las dimensiones del tero, as como el nmero,
localizacin, tamao y relacin del (o los) miomas con estructuras cercanas. Si despus de los puntos anteriores existe alguna duda
diagnstica, se podr recurrir a otros estudios de gabinete como son: histerosalpingografa, tomografa axial computarizada, y realizar
diagnstico diferencial con algn otro tipo de tumor benigno o maligno de la cavidad abdominal. TRATAMIENTO: Miomectoma:
Consiste en la reseccin nicamente de los miomas a travs de una incisin en la pared abdominal (si son tumores de gran tamao) o
mediante ciruga de mnima invasincomo lo es la laparoscopia. Esta miomectoma se indica siempre que la paciente tenga deseos de
embarazos futuros y sea menor de 35 aos. Hasta 80% de las mujeres refiere disminucin de los sntomas despus de la miomectoma.
Histerectoma: Es la extirpacin total del tero, tambin a travs de una incisin abdominal o por ciruga laparoscpica. Ser el
procedimiento de eleccin en aqullas que no deseen tener ms hijos, estn cerca de la menopausia o si el crecimiento tumoral ha sido
muy acelerado. Tratamiento Hormonal: El objetivo es no bloquear al endometrio, el objetivo ser regular la tortuosidad de los vasos, la
estimulacin a nivel endometrial y que el endometrio este creciendo de manera regular en toda la cavidad uterina. Se puede usar
ergotrate (vasoconstrictor especifico), los primeros das del ciclo, en cada menstruacin o ac. Mefenamico (seguro). Lutoral o provera
en la segunda parte del ciclo. En miomatosis uterina de grandes elementos, donde no hay sangrado no se da tratamiento. Se pueden
utilizar hormonas inhibidoras de la produccin de estrgenos en forma paliativa o si existe alguna contraindicacin para realizar los
procedimientos anteriores, tambin se utilizan si se desea disminuir el tamao de los tumores previo a la ciruga o si coexiste
endometriosis severa. El xito del tratamiento mdico-hormonal es menor del 10%.

CASO CLINICO
Mujer de 49 aos con 3 hijos, viene a verlo debido en a que presenta perodos abundantes, desde hace dos aos ha notado que esto se
ha incrementado de tal forma que le ha generado incomodidades ya que afirma que inunda la toalla los primeros das y duran hasta
10 dias adems agrega cansancio crnico, astenia y adinamia. A la exploracin fsica se observa palidez de tegumentos, uas
quebradizas, pelo frgil asi como piel fra. A la exploracin GO se observa sangre obscura en vagina con algunos coagulos, dolor a la
palpacin y movilizacin uterina, refiere que en su anterior DOC se reportaron cambios inflamatorios recibiendo tratamiento el cual no
especifica. Cuenta con OTB, finalmente refiere sangrado poscoital.

PREGUNTA
Cul es la conducta a seguir.

RESPUESTA
a.- Realizar USG.
b.- Biometria hemtica y ferritina serica.
c.- Histeroscopia.
d.- Biopsia endometrial.

CASO CLINICO
Mujer de 31 aos de edad, refiriendo presencia de irregularidades de su periodo menstrual, agrega que no se ha podido embarazar
desde hace 6 aos que inicio su vida sexual activa. No usa mtodo anticonceptivo. Actualmente toma 37,5 mg de venlafaxina dos veces
al da por episodio depresivo que inicio hace un ao. Su examen fsico se observa presencia de hematomas refiriendo que esto le ha
ocurrido desde la adolescencia, se observa adems un IMC de 34, con hirsutismo en cara, espalda y piernas, adems vello pbico tipo
romboide.

PREGUNTA
Cul es la conducta a seguir.

RESPUESTA
a.- USG transvaginal.
b.- USG ginecologico abdominal.
c.- USG de glndulas suprarrenales.
d.- Biopsia endometrial.
MANUAL DE TRABAJO DEL CURSO ENARM CMN SIGLO XXI
CURSO ENARM CMN SIGLO XXI TEL: 36246001 Pharmed Solutions Institute PGINA 292

VIRUS DEL PAPILOMA HUMANO (VPH), DISPLASIA, CANCER CERVICOUTERINO (CACU). CIENCIAS BASICAS: Los papilomavirus son
pequeos virus de DNA, de doble cadena, cuya actividad transformante se explica principalmente por la actividad de sus oncoprotenas
E6 y E7. Estas protenas se unen a un sin nmero de reguladores celulares importantes en el control de procesos biolgicos como: la
apoptosis, proliferacin celular, estabilidad cromosmica, transcripcin de genes (oncogenes y genes supresores de tumor),
diferenciacin celular y la respuesta inmunolgica, entre otros. El VPH se encuentra ampliamente distribuido en todo el mundo y
provoca un amplio espectro de enfermedades epiteliales, desde verrugas a papilomas en los epitelios de distintas mucosas (lesiones
preinvasores de crvix, la principal), ya que poseen un trofismo especfico. Tambin VPH se ve involucrado en la patogenia de diversos
tumores benignos y malignos, y constituye el factor de riesgo ms importante para el desarrollo de cncer crvico uterino. SALUD
PUBLICA: El VPH est fuertemente asociado al desarrollo de displasia, neoplasia intraepitelial, y cncer del cuello uterino. Ms del 95%
de los cnceres de crvix poseen DNA de VPH de alto riesgo. 1% de las displasias leves evolucionan a invasividad. De 5-15% de las
displasias graves progresa a cncer en 3 aos. El virus puede estar latente hasta 10-15 aos y no ocasionar cambios. El papilomavirus
tipo 16 es el ms prevalente de los VPH oncognicos, responsable de mas de la mitad de los tumores, mientras que el papilomavirus
tipo 18 est involucrado en el 20% de los mismos. PATOGENIA: Dependiendo del tipo de VPH y de la lesin clnica, puede transmitirse
por contacto cutneo, relacin sexual, transmisin perinatal, por escamas infectadas directa o indirectamente, y posiblemente por
fmites. El perodo de incubacin vara desde tres semanas a 8 meses, con un promedio de tres meses. En cuanto al genotipo del VPH y
su localizacin ms frecuente en piel se tiene: Verruga Vulgar: 2,4,7, verruga plantar: 1,4, verruga plana: 3, 10, 28, 41,
epidermodisplasia Verruciforme: 5, 8, 9, 12, 14, 15, 17, 19, 36, 46, 47, 49, 50. En Mucosas (bajo riesgo): Papilomas anogenitales,
cervicales, orofaringe y tracto respiratorio: 6, 11, 30, 34, 40, 42, 44, 55, 57, 59. En Mucosas (alto riesgo): papilomas anogenitales y
cervicales, papulosis Bowenoide, displasia cervical, orofaringe, cncer cervical y anogenital: 16, 18, 31, 33, 35, 39, 45, 51, 52, 56. Los
principales factores de riesgo del VPH genital son: Tener muchas parejas sexuales, tener menos de 25 aos de edad, IVSA a una edad
temprana (<16 aos), sin embargo, incluso las mujeres que slo han tenido una pareja sexual pueden infectarse con el VPH,
multigestas, antecedente de ETS, tabaquismo, anticonceptivos orales. Slo un 10% de las infecciones por VPH tiene manifestacin
clnica, ya sea en la forma de verrugas, papilomas o displasias. El virus se replica en el estrato granuloso y es detectado en el estrato
crneo, no as en el estrato basal. La infeccin por VPH no tiene rol oncognico per se, sino que juega un rol que es potenciado por
factores fsicos y qumicos. En los carcinomas no se encuentran partculas virales activas, pero s su DNA y sus genes tempranos. Los
productos de los genes E5, E6 y E7 tienen actividad oncognica, ya que sus protenas estn involucradas en el control del ciclo celular y
estimulan la proliferacin o interfieren con la diferenciacin de clulas infectadas. El blanco de estas protenas virales est
representado por las protenas retinoblastoma (Rb), y p53. Normalmente retinoblastoma acta inhibiendo la transcripcin de genes
como c-myc, ras, entre otros que ests encargado de la proliferacin celular, de manera que la inactivacin de Rb provoca una
replicacin celular descontrolada. La p53 se encarga de promover la transcripcin de genes para reparar el DNA daado o inducir
apoptosis, de modo que su inactivacin provoca la prdida de la capacidad de bloquear la proliferacin celular como respuesta al dao
del DNA. El resultado es la inestabilidad gentica y el desarrollo de mutaciones crticas que favorecen el desarrollo de tumores. En la
zona de transicin (unin escamocolumnar) de exocervix y endocervix hay dos epitelios, por eso es un lugar ideal, para el virus del VPH.
DIAGNOSTICO: Clnica; La principal manifestacin clnica son las verrugas del rea genital externa y el condiloma acuminado. El
condiloma acuminado se presenta como lesiones papulares con superficie lobulada e irregular, de color rosado oscuro, con
prolongaciones digitiformes con aspecto de coliflor. El nmero de lesiones es variable, su tamao ve de 2mm a 1 cm, pero si son
numerosas pueden confluir comprometiendo grandes reas genitales que se traumatizan durante el coito. En la mujer se ubican con
mayor frecuencia en los labios mayores y menores, parte posterior del introito, cltoris, monte de Venus, paredes vaginales y en el
cuello uterino. En el hombre son comunes en la cara interna del prepucio, en el frenillo y en el surco balanoprepucial. Con menor
frecuencia se presentan en el glande y en el meato uretral. Las lesiones perianales y rectales pueden verse en varones homosexuales o
pueden ser el resultado de la propagacin perineal en las mujeres. En personas que practican el sexo oral puede encontrarse
condilomas orales. En el embarazo e inmunodeprimidos, especialmente los transplantados renales las lesiones tienden a ser ms
numerosas y exuberantes. El VPH puede infectar el epitelio escamoso de crvix, vagina, vulva, perin, pene y regin perianal, y
determinar lesiones como verrugas genitales, condilomas acuminados, lesiones precancerosas intraepiteliales y cncer. La deteccin de
HPV va de 40 a 90% en las neoplasias intraepitelial (NIE) de bajo grado, a 95% en los pacientes con cncer invasor. Los VPH tambin han
sido implicados en el desarrollo de tumores malignos en sitios distintos a la regin anogenital, como ano, vagina, vulva y pene e incluso
cavidad bucal, pero con una fraccin atribuible considerablemente menor a la del cncer de crvix, en el cual virtualmente el 100% de
los cnceres son causados por VPH. Laboratorio y gabinete: Papanicolaou se utiliza para detectar cambios celulares o clulas anormales
en el cuello uterino, (estas clulas anormales pueden ser precncer o cncer, u otras cosas) es una prueba de screening. Se extraen
clulas del cuello uterino y se procesan. Luego se observan con un microscopio para ver si las clulas son normales o si se pueden
observar cambios en ellas. La prueba de Papanicolaou es una excelente prueba para encontrar clulas cancerosas y clulas que se
pudieran convertir en cncer. El resultado anormal ms comn en la prueba de Papanicolaou se llama ASC-US. Las clulas ASC-US
generalmente no indican precncer, pero tampoco son del todo normales. Actualmente con la captura de hibridos, es ms fcil
detectarlo, solo nos dice si hay virus o no. Hasta 7 aos antes, a diferencia del papanicolao normal donde a veces lo encontramos en
NIC I, porque no siempre vemos los coilocitos, este detecta lesion los hibridos no. Las lesiones subclnicas deben ser visualizadas con
colposcopa y con la aplicacin de cido actico al 3-5% que ayuda a delimitar la lesin mediante la reaccin blanco actica. Sin
embargo es un examen de baja especificidad, ya que se altera en otras enfermedades, como por ejemplo el lquen plano, la candidiasis
etc. En todas las mujeres con condiloma acuminado debe realizarse citologa con tcnica de Papanicolau anual. Colposcopia: identifica
lesiones sopechosas, delimita la zona de transformacin, determina extensin de la enfermedad, til para toma de biopsias en zonas
sospechosas. Clasificacion por hallazgos colposcopicos: Grado I; insignificante no sospechoso, epitelio aceto-blanco delgado, vasos no
atpicos, distancia intercapilar pequea. Correlacion histolgica: metaplasia escamosa y LIEBG. Grado II: significativo, sospechoso,
epitelio aceto-blanco mayor opacidad y grosor, sin vasos atpicos, distancia intercapilar aumentada. Correlacion histolgica: NIC II y NIC
III. Grado III: Altamente significativo, muy sospechoso, epitelio aceto-blanco grueso irregular y opaco, vasos dilatados irregulares y
atpicos, distancia intercapilar variable, contorno superficial irregular. Correlacion histolgica: LIEAG e invasin temprana. Todas las
lesiones que encontremos por colposcopia se biopsian. Debe biopsiarse verrugas resistentes al tratamiento, atpicas o pigmentadas.
CLASIFICACION: Bethesda, para pronostico y tratamiento: LIEBG (lesin intraepitelial de bajo grado): IVPH y displasia leve (NIC I). LIEAG
MANUAL DE TRABAJO DEL CURSO ENARM CMN SIGLO XXI
CURSO ENARM CMN SIGLO XXI TEL: 36246001 Pharmed Solutions Institute PGINA 293

(lesion intraepitelial del alto grado): NIC II, NIC III. ASCUS: Clulas que no son claramente displasicas, cambios inflamatorios. 20-50%
desarrollan displasia. Se da seguimiento como si se hubiera descubierto un NIC. TRATAMIENTO: Qumico; Podofilino al 10%-30%-45%
en solucin alcohlica, aplicacin por profesional mdico en lesiones de
genitales externos y perianales. La aplicacin debe repetirse
semanalmente por 3 a 4 semanas hasta la desaparicin de las lesiones.
Si no mejora considerar otra posibilidad diagnstica o la presencia de
una cepa ms agresiva. La aplicacin de grandes cantidades de
podofilino puede provocar toxicidad sistmica. Est contraindicado en el
embarazo y la lactancia. Podofilotoxina al 0.5% Puede aplicarse
directamente por el paciente dos veces al da por 3 das. No requiere
lavarse como la podofilina. Se descansa 4 das y luego se repite por 3
das ms. Las respuestas se ven habitualmente a las 6 semanas. Su
eficacia es similar a la de la podofilina, pero tiene menos toxicidad
sistmica. Aproximadamente la mitad de los pacientes muestra algn
grado de inflamacin, quemaduras o erosiones. cido Tricloroactico al
80-90%, aplicacin local por el mdico 2-3 veces por semana por un
mximo de tres semanas. Se forma una erosin que sana en unas 3
semanas sin cicatriz. Puede usarse como terapia combinada con
podofilino al 40% en pacientes inmunodeprimidos y es el tratamiento de
eleccin en el embarazo. Tratamientos Fsicos; Crioterapia (Nitrgeno
Lquido), cura aproximadamente el 90% de las lesiones, aunque a veces
se requiere varias aplicaciones. Electrociruga Eventualmente
desaparecen todas las lesiones, aunque el 20-30% desarrolla nuevas
lesiones en los bordes quirrgicos o en sitios alejados. Extirpacin
Quirrgica; 1) Lser. 2) Inmunomoduladores: factor inmunomodulador
inductor de la sntesis de interfern gamma, TNF alfa, IL 1,6,8,10 y factor
estimulante de colonias granulocticas. Estimula la inmunidad celular y
carece de actividad antiviral directa in vitro. La presentacin en crema al
5% se utiliza en el tratamiento de los condilomas y verrugas planas, se
aplica cada 48 horas y se deja actuar por 2 a 4 horas. La respuesta se
observa a las seis semanas. Es bien tolerado incluso en pacientes
inmunodeprimidos, aunque dos tercios de los pacientes presentan
eritema y ardor. Produce curacin en el 60% de los casos y las mujeres
responden mejor que los hombres. 3) Imiquimod (Aldara). Tratamiento
especfico: NIC I o LIEBG: Tratar procesos infecciosos concomitantes y
repetir papanicolao y colposcopia, a intervalos de 3-4 meses con examen
plvico. Terapia ablativa en personas poco confiables para seguimiento. En pacientes inmunocompetentes hay remisin hasta en 50%.
NIC II, NIC III LIEAG: Terapia ablativa la criociruga elimina lesin hasta 95%, ciruga lser de CO2, vaporizacin del tejido tratado con
xito de 95%, se hace en lesiones que se extienden. Escisin quirrgica con asa diatermica, solo cuando esta confinada la lesin al
crvix. Cono cervical: Se realiza cuando hay cncer in situ, con deseos de conservar la fertilidad, de lo contrario se realizara
histerectoma. La curacin es de 87-97%. Tambin se realiza cuando hay ca microinvasor, menor a 3mm, sin compromiso linfovascular.
Con este procedimiento el seguimiento es ms riguroso, realizar citologa cada 3 meses. Histerectomia: Extrafascial, cuando no est a
discusin la fertilidad. En casos de ca in situ el seguimiento de la citologa es trimestral primer ao, posteriormente anual. LESIONES
PREINVASORAS: Abarcan solo epitelio, sin membrana basal. NIC I, II, III (Neoplasia intraepitelial cervical). DISPLASIA: Cambios
morfolgicos celulares, ncleos hipercromaticos, aumento de la relacin ncleo y citoplasma, mayor ndice mittico. CANCER
CERVICOUTERINO: EL CaCu ocupa los primeros lugares como causa de muerte por cncer en mujeres mexicanas. Constituye el 34.2%
de las neoplasias malignas. Una infeccin persistente de virus del papiloma humano (VPH) de tipos virales de alto riesgo oncognico, es
el factor etiolgico principal en el desarrollo de esta neoplasia. Se conoce que solamente una pequea fraccin de lesiones cervicales
infectadas con VPHs de alto riesgo evoluciona a lesiones de alto grado o cncer. Las mujeres sexualmente activas, de cualquier edad,
pueden infectarse con VPHs oncognicos. Sin embargo, el cncer de crvix invasor en mujeres jvenes infectadas con virus oncognicos
es raro y la prevalencia de VPH en mujeres de 40 aos o mayores no se correlaciona con la alta tasa de cncer cervical. Es la persistencia
de VPHs oncognicos lo que da lugar al desarrollo de lesiones precancerosas y potencialmente al cncer invasor, lo que puede llevar
varios aos para su desarrollo. El cncer de crvix ocurre en dos formas predominantes: carcinoma epidermoide y adenocarcinoma,
carcinoma adenoescamoso 1.7%, carcinoma verrugoso, carcinoma indiferenciado de clulas pequeas, tumor carcinoide, melanomas.
El tipo histolgico ms comnmente encontrado en las mujeres es el carcinoma epidermoide (90% de los casos) y est ms
frecuentemente asociado al VPH 16. El adenocarcinoma 3.7% es el segundo tipo histolgico ms comn y aunque el VPH tipo 16
tambin es el ms frecuente, la proporcin de los genotipos 18 y 45 aumenta significativamente en este tipo de tumores. Los estudios
de citologa, que incluyen a la prueba Pap convencional, se utilizan para detectar lesiones precancerosas, pero no es suficiente para
detectar infecciones por VPH. Es claro que, el adenocarcinoma es ms difcil de detectar por Pap que el carcinoma escamoso.
Manifestaciones clnicas: sangrado (lo ms importante), predominantemente poscoital, tambin puede ser intermenstrual y
posmenopusico (siempre pensar en cncer cervical y despus cacu), flujo seroso, ftido ocasionado por la necrosis tumoral, dolor
plvico (territorio de nervio citico), edema de extremidades inferiores, dolor lumbar, pueden presentarse fistulas por infiltracin a
vejiga o recto. En estadios avanzados; prdida de peso, anemia, sndrome urmico. Marcadores tumorales: Antigeno del carcinoma de
clulas escamosas, lo podemos encontrar hasta en 50% de los primarios y 75% en los recurrentes, tienen baja utilidad. El diagnstico de
esta neoplasia se realiza con estudio histopatolgico mediante una biopsia dirigida, ya sea mediante colposcopia en caso de no
observarse una lesin o mediante toma directa si existe tumor visible. Los estudios de extensin a realizar en cada caso, depender de
MANUAL DE TRABAJO DEL CURSO ENARM CMN SIGLO XXI
CURSO ENARM CMN SIGLO XXI TEL: 36246001 Pharmed Solutions Institute PGINA 294

la etapa clnica obtenida en el examen inicial Estadificacion segn la FIGO; ver cuadro anexo. TRATAMIENTO: Recomendaciones de
tratamiento en el Instituto Nacional de Cancerologa 1) Cncer Cervico uterino In Situ; Preservacin de Fertilidad, cono teraputico.
Fertilidad satisfecha (<50 aos o premenopausia): cono teraputico o histerectoma Tipo I con preservacin de anexos. Fertilidad
satisfecha (>50 o post menopausia): cono teraputico o histerectoma Tipo I con salpingo-ooforectoma bilateral. 2) CaCu IA 1; Cono
teraputico o histerectoma tipo I. Preservacin de anexos en <50 aos o premenopusica Salpingo-ooforectomia bilateral en >50 aos
o postmenopusica. 3) CaCu IA 2; Histerectoma Radical tipo II. Preservacin de anexos en <50 aos. Linfadenectoma plvica en caso
de PVL. Braquiterapia radioterapia externa a pelvis total (dosis total de 75-80Gy a punto A) en caso de pacientes mdicamente
inoperables. 4) CaCu IB1: Histerectoma Radical tipo II (en tumores menores de 2 cm) y linfadenectoma plvica bilateral. Histerectoma
Radical tipo III y linfadenecto ma plvica. Preservacin de anexos en <50 aos o premenopusicas. En caso de contraindicacin mdico
quirrgica la opcin teraputica es la Radioterapia externa a pelvis total + braquiterapia (dosis total de 80 85Gy a punto A). 5) CaCu
IB2IVA. Radioterapia externa a pelvis total concomitante con quimioterapia basada en cisplatino + braquiterapia (dosis total >85Gy a
punto A). Cis platino 40 mg/m2 en forma semanal. Exenteracin plvica para pacientes con fstula vesico-vaginal y/o recto-vaginal sin
infiltracin a la pared plvica y con ECOG I y K ms del 90%. Nota: En caso de paciente con hidronefrosis co locacin de catter JJ o
Catter de nefrostoma antes de iniciar el tratamiento con radioterapia. Con fstula vsico-vaginal no exenterables derivacin urinaria.
Con fstula recto vaginal no exenterable colostoma. 6) Ca Cu II, III, IVB; Tratamiento sistmico paliativo, radioterapia paliativa exclusiva,
superviviencia a 5 aos 65-75%, 30-50% y 20% respectivamente. Vigilancia: 80-90% presentan recaidas en los 2 primeros aos.
Exploracin plvica y citolgica; mensual o bimensual el primer ao. Cada 2-3 meses el segundo ao. Cada 4-5 meses el tercer ao.
Cada 6 meses el cuarto ao y quinto ao. Cada ao a partir del sexto ao. PAPULOSIS BOWENOIDE (PB): Es una forma de cncer
espinocelular in situ asociado al VPH. Se caracteriza por mltiples ppulas asintomticas, ligeramente solevantadas, de 2 a 20 mm que
pueden confluir formando placas de color marrn violceo y con tenue descamacin, ubicadas en la base del pene, vulva y perin en
pacientes jvenes. Se considera un factor de alto riesgo de desarrollo de cncer de crvix. Su curso es habitualmente benigno.
CONDILOMA ACUMINADO GIGANTE: o tumor de Buschke-Lowenstein es una lesin precancerosa. Se caracteriza por condilomas de
gran tamao que causan dao tisular local. Son ms frecuentes en la ingle, regin perianal y en el surco balanoprepucial. No metastisa,
se maneja con ciruga y crioterapia y recurre con frecuencia. Si se detecta la presencia de VPH 16 o 18 debe considerarse la progresin a
la malignidad.

CASO CLINICO
Mujer de 29 aos de edad la cual refiere sangrado poscoital la cual ha resultado levemente dolorosa, como antecedentes presento
menarca a los 9 aos, IVSA a los 14 aos, ha tenido 4 parejas sexuales, es usuaria de mtodo hormonal oral, agrega que ha presentado
varios cuadros de cervicovaginitis, la ultima DOC presento cambios inflamatorios, tabaquismo y alcoholismo positivo. A la exploracin
ginecolgica se observa crvix afresado, doloroso a la movilizacin.

PREGUNTA
Cul es la conducta a seguir.

RESPUESTA
a.- Repetir DOC.
b.- Realizar colposcopia.
c.- Enviar a clnica de displasias.
d.- Realizar citologa de base liquida.

CASO CLINICO
Paciente de 45 aos, con antecedentes de hipertensin arterial crnica en tratamiento con atenolol (50 mg/da) e hidroclorotiazida
12,5 mg cada 24 hrs. Multpara, usuaria de dispositivo intrauterino (T de cobre, por 7 aos). Consult por hipermenorrea y
dismenorrea, de intensidad progresiva y resistente al tratamiento mdico (antiinflamatorios no esteroidales y retiro del dispositivo
intrauterino). Al examen fsico presentaba un tero aumentado de tamao, con anexos normales. Se solicit ultrasonografa
transvaginal que confirm la presencia de miomas mltiples, el mayor en el fondo uterino de 4 cm de dimetro. El hemograma
demostr anemia ferropriva significativa (Hto. 26%). Dado el tamao de los miomas y sntomas asociados (hipermenorrea, anemia
severa secundaria, dismenorrea y dispareunia intensa), se decidi realizar un tratamiento quirrgico subtotal. Evolucion
satisfactoriamente, dndose de alta en buenas condiciones con suplemento oral de hierro.

PREGUNTA
Cul es la conducta a seguir.

RESPUESTA
a.- Realizar citologa cervical 3 veces anualmente.
b.- Realizar citologa cervical normalmente.
c.- Se puede descontinuar la citologa cervical.
d.- Realizar citologa vaginal cada 2 o 3 aos.

MANUAL DE TRABAJO DEL CURSO ENARM CMN SIGLO XXI
CURSO ENARM CMN SIGLO XXI TEL: 36246001 Pharmed Solutions Institute PGINA 295

CANCER ENDOMETRIAL (CE). CIENCIAS BASICAS: Es una neoplasia glandular maligna que se origina en el endometrio, generalmente
parece estar relacionado con una estimulacin estrogenica crnica del endometrio no contrabalanceada, por una fuente de estrgenos
endgenos o exgenos. La mayora de los canceres de endometrio son adenocarcinomas (canceres que derivan de clulas que
producen moco y liberan moco u otros liquidos) SALUD PUBLICA: Es la segunda neoplasia ginecolgica ms frecuente a nivel mundial,
siendo ms comn en la perimenopausia. La incidencia del CE es seis veces mayor en pases desarrollados que en los menos
desarrollados. La supervivencia en las etapas clnicas tempranas es del
80%, mientras que las pacientes con enfermedad avanzada tienen una
supervivencia a largo plazo menor al 50%. El tipo histolgico ms
frecuente es el adenocarcinoma endometroide 75-80%. CLASIFICACION:
Basado en el perfil histolgico, molecular y clnico el cncer de
endometrio se divide en dos tipos: TIPO I: variedad endometrioide (90%)
frecuentemente de bajo grado, el ms frecuente, se presenta entre los
55-65 aos de edad, relacionado a estrgenos, diagnosticado en forma
temprana y con buen pronstico en supervivencia global, el cual se
origina de una lesin precursora (hiperplasia atpica o neoplasia
intraepitelial endometrial), en donde est implicado la mutacin como
inactivacin de PTEN (una protena natural), gen supresor de tumor. TIPO
II: variedad no-endometrioide, no hormonodependiente, se presenta en
mujeres de >65 aos, son pacientes delgadas, papilar seroso, clulas
claras y carcinosarcomas. Puede con llevar una enfermedad extrauterina,
se relaciona con escasa supervivencia, representa el 10% de todos los canceres de endometrio., Invade el espacio vascular y linftico,
hasta 36% presenta ganglios positivos sin tener invasin en miometrio. PATOGENIA: Factores de riesgo: historia de terapia hormonal
estrogenica sustitutiva sin oposicin progestacional, tratamiento con tamoxifeno, menopausia tarda, nuliparidad, infertilidad o falla
teraputica de ovulacin, y obesidad. La DM e hipertensin, tambin son considerados factores de riesgo. Todas las lesiones
endometriales se originan del componente glandular del endometrio,
formando una lesion polipoide en la cavidad uterina hasta que la lesin se
vuelve friable y necrtica, consecuentemente, el sangrado postmenopusico
constituye el 90% de los sntomas al inicio de la enfermedad. La invasin local
y linftica son los pasosos siguientes en la historia natural: mientras que la
porcin superior uterina drena, atravs de los vasos ovricos a los relevos
paraarticos, las porciones inferiores drenan a travs de los vasos uterinos a
los plvicos. Resultando que del 10-35% de los tumores tempranos tengan
adenopatas paraaorticas sin relevos plvicos positivos. Una vez que el tumor
rompe la serosa, puede haber invasin a rganos vecinos. DIAGNOSTICO: El
cuadro clnico, generalmente se caracteriza por la presencia de sangrado
vaginal anormal (75%), frecuentemente posterior a la menopausia, en etapa
temprana. Otros sntomas son: dolor plvico, piometra y hematmetra. Deben incluirse en un programa de escrutinio a partir de los 45
aos a las pacientes con 2 ms factores de riesgo dentro de los cuales se encuentran: Diabetes mellitus, obesidad, exposicin crnica
a estrgenos sin oposicin, uso de tamoxifeno. De manera similar, las mujeres posmenopusicas que presenten sangrado transvaginal,
se les debe realizar biopsia endometrial preferentemente mediante histeroscopa. En mujeres consideradas de alto riesgo por: historia
familiar de cncer de colon no polipsico y pacientes con sndrome de Lynch tipo II, deben iniciar un programa de escrutinio a los 35
aos. Lo mismo que pacientes con antecedentes de cncer de mama, tiroides y ovario. Estudios de extensin: La evaluacin de la
paciente incluye examen clnico completo y exploracin ginecolgica incluyen do tacto recto-vaginal para evaluar los parametrios y el
tabique rectovaginal. Ultrasonido de alta resolucin preferentemente transvaginal (especialmente en pacientes en las que se realiz la
biopsia endometrial sin ultrasonido previo). Citologa cervicovaginal, histeroscopa con toma de biopsia dirigida para determinar el
tamao y localizacin del tumor (afeccin a istmo, canal cervical). Adems de lo anterior, es necesario incluir: Radiografa de trax.
Estudios de laboratorio: biometra hemtica, qumica sangunea de 26 elementos, tiempo parcial de tromboplastina (TPT), tiempo de
pro trombina (TP), INR y examen general de orina. En histologa no endometrioide: se solicita CA-125. Tomografa computada para
determinar la extensin de la enfermedad en pacientes con evidencia de cncer de endometrio rgano no confinado. Resonancia
magntica para evaluar invasin miometrial y afectacin del estroma del crvix. Manejo de padecimientos concomitantes y valoracin
preoperatoria. Gradacin histolgica: Grado Arquitectural: G1: No ms del 5 % del tumor constituido por masas slidas. G2: Entre el 6 y
50 % del tumor constituido por masas slidas. G3: Ms del 50 % del tumor constituido por masas slidas. Grado Nuclear; G1: Ncleo
oval. Cromatina uniformemente distribuida. G2: Ncleos con caractersticas intermedias entre G1 y G3. G3: Ncleos de gran tamao,
pleomrficos, cromatina irregular, nucleolos eosinfilos. TRATAMIENTO: la ciruga de cncer endometrial debe realizarse para; la
estadificacin y tratamiento primario. En el momento de la ciruga estadificadora debe realizarse estudio transoperatorio (ETO) de
tero en forma rutinaria. La ciruga implica la realizacin de laparotoma exploradora y estadificadora para cncer de endometrio la cual
debe incluir: 1) lavado peritoneal y 2) histerectoma extrafascial con salpingo-ooforetctoma bilateral; linfadenectomia plvica bilateral
y para-aortica. En caso de estirpe de clulas claras, serosa-papilar o indiferenciados, se debe realizar adems omentectomia y toma de
biopsias peritoneales y de cpula diafragmtica. Las modalidades teraputicas son: 1) Ciruga. 2) radioterapia (adyuvante, indicada en
estadios de riesgo intermedio o alto de recurrencia). 3) Quimioterapia (doxorrubicina/cisplatino=AP,
cisplatino/doxorrubicina/ciclofosfamida=CAP, cisplatino/epirrubicina/ciclofosfamida=CEP). 4) Hormonoterapia (manejo conservador en
mujeres con deseo de fertilidad menores de 40 aos y valorando los dems factores). ENFERMEDAD RECURRENTE: Evaluar ciruga de
salvamento (incluyendo la posibilidad de exenteracin plvica) en caso de recada plvica. Se utilizarn los esquemas de QT (a base de
platino por cuatro a seis ciclos: Paclitaxel + carboplatino cada 21 das o Doxorubicina + cisplatino cada 21 das) y/u hormonoterapia (a
base de acetato de medroxiprogesterona 200mg diarios, reservado para tumores de bajo grado y con receptores progestacionales
positivos), siempre que no sean candidatas a ciruga o RT. Al igual que ocurre con el cncer endometrial avanzado, la enfermedad
MANUAL DE TRABAJO DEL CURSO ENARM CMN SIGLO XXI
CURSO ENARM CMN SIGLO XXI TEL: 36246001 Pharmed Solutions Institute PGINA 296

neoplsica endometrial recurrente tambin ha sido tratada con radioterapia y quimioterapia, pero existen estudios que sugieren que la
ciruga radical en pacientes seleccionadas puede mejorar la supervivencia a largo plazo. Como se ha demostrado en el carcinoma de
ovario, las pacientes con cncer endometrial recurrente en quienes se consigue una reseccin quirrgica completa, se beneficiarn de
una mayor supervivencia global. La exenteracin plvica, por lo general reservada para pacientes con recurrencia central de carcinoma
de crvix, ha sido tambin aplicada en pacientes con carcinoma endometrial recurrente. SEGUIMIENTO: 1) Los primeros tres aos
(Cada tres o cuatro meses). Evaluacin clnica con examen fsico y ginecolgico. Citolgico vaginal. Laboratorios: Ca 125 en variedad no
endometrioide. Gabinete: Tele de trax (PA y/o lateral) semestral. Tomografa computada abdminoplvica en caso de sospecha de
recurrencia y de forma anual en pacientes de alto riesgo. 2) Despus del tercer ao (Cada seis meses).Evaluacin clnica con examen
fsico y ginecolgico. Citolgico vaginal. Laboratorios: Ca 125 en variedad no endometrioide. Gabinete: Tele de trax (PA y/o lateral)
semestral. Tomografa computada abdominoplvica en caso de sospecha de recurrencia y de forma anual en pacientes de alto riesgo. 3.
Despus quinto ao (Cada ao). Mismos estudios. PRONOSTICO: Los factores pronsticos desfavorables son la presencia de variedad
no endometrioide, invasin vas cular y linftica, tumores G 3, penetracin mayor al 50% y pacientes mayores de 70 aos.

CASO CLINICO
Se trata de paciente de 60 aos de edad la cual refiere sangrado transvaginal, refiere que hace un par de meses inicio con manchado
transvaginal, el cual se convirti en sangrado franco, la paciente cuenta con antecedentes de inicio de vida sexual activa a la edad de 14
aos, menarca a los 9 aos, gesta 5 para 3 abortos 2, adems de 3 parejas sexuales, es hipertensa desde hace 10 aos con adecuado
control, diabetes mellitus desde hace 8 aos y dislipidemia. A la exploracin fsica se observa paciente con leve palidez de tegumentos,
con ndice de masa corporal de 31, signos vitales de TA 135/95, FC 83, FR 21, normotermica, torax sin fenmenos agregados, abdomen
depresible, no doloroso al tacto vagina bimanual utero de caractersticas normales, no doloroso, con presencia de manchado del
guante.

PREGUNTA
Cul es la conducta a seguir ms adecuada.

RESPUESTA
a.- Realizar USG.
b.- Indicar progestgenos.
c.- Realizar biopsia.
d.- Realizar legrado hemosttico.

CASO CLINICO
Se trata de paciente de 60 aos de edad la cual refiere sangrado transvaginal, refiere que hace un par de meses inicio con manchado
transvaginal, el cual se convirti en sangrado franco, la paciente cuenta con antecedentes de inicio de vida sexual activa a la edad de 14
aos, menarca a los 9 aos, gesta 5 para 3 abortos 2, adems de 3 parejas sexuales, es hipertensa desde hace 10 aos con adecuado
control, diabetes mellitus desde hace 8 aos y dislipidemia. A la exploracin fsica se observa paciente con leve palidez de tegumentos,
con ndice de masa corporal de 31, signos vitales de TA 135/95, FC 83, FR 21, normotermica, torax sin fenmenos agregados, abdomen
depresible, no doloroso al tacto vagina bimanual utero de caractersticas normales, no doloroso, con presencia de manchado del
guante.

PREGUNTA
Cul es la conducta a seguir ms adecuada.

RESPUESTA
a.- Realizar USG.
b.- Indicar progestgenos.
c.- Realizar biopsia.
d.- Realizar legrado hemosttico.


MANUAL DE TRABAJO DEL CURSO ENARM CMN SIGLO XXI
CURSO ENARM CMN SIGLO XXI TEL: 36246001 Pharmed Solutions Institute PGINA 297

SINDROME DE OVARIO POLIQUISTICO (SOP). CIENCIAS BASICAS: El SOP es una de las alteraciones endocrinas ms comunes en la
mujer, en edad reproductiva y es la principal causa de esterilidad por anovulacin. Representa un trastorno heterogneo que se
distingue por una combinacin de irregularidades menstruales, hirsutismo o acn y obesidad, que suele diagnosticarse en la
adolescencia, pero aparentemente tienen su origen en la vida intrauterina. Actualmente la relacin entre SOP y sndrome metablico
(obesidad, hipertensin, dislipidemia y disglucosis), y subsecuente diabetes mellitus tipo II, lo convierten en un grave problema de
salud pblica con un alto coste econmico. SALUD PUBLICA: Tiene una prevalencia de 4-8%. Tan solo en Estados Unidos se considera
entre 7-10 millones de mujeres afectadas por este padecimiento. Se ha asociado con un riesgo mayor de cncer de endometrio y
probablemente de glndula mamaria. El 50-65% de las pacientes con SOP son obesas y se considera que el 35-45% de ellas presentar
intolerancia a la glucosa durante una curva de tolerancia a la glucosa, con un riesgo de desarrollar DM tipo 2 del doble a lo esperado
para su edad. PATOGENIA: En la aparicin de SOP existen componente sobre los que no podemos influir: preconcepcionales (gentica)
y postconcepcionales (peso al nacer o exposicin intratero a andrgenos) y otros aspectos que si pueden ser modificables como son
los hbitos de vida (dieta y ejercicio). Hay evidencias que parece seguir un patrn de herencia autosmico dominante. En mujeres con
SOP, se ha descrito una alteracin en el eje hipotlamo-hipfisis-ovario, que se distingue por aumento de la actividad del hipotlamo
que produce un mayor numero de pulsos de la hormona liberadora de gonadotrofinas (GnRH), lo que a su vez aumenta la hormona
luteinizante, cambiando la relacin LH/FSH a favor de la primera. Al predominar la LH se sintetizan preferentemente andrgenos en el
ovario. La resistencia a la insulina puede desempear una funcin central en la causa del sndrome: el musculo esqueltico es
profundamente resistente, mientras que otros tejidos, como el hipotlamo, la glndula suprarrenal y el ovario, conservan la
sensibilidad a la insulina. La hiperinsulinemia compensadora resulta en una disminucin en la globulina transportadora de esteroides
sexuales (SHBG) y simultneamente sirve como estmulo trfico en la produccin de andrgenos por parte de las glndulas
suprarrenales y el ovario. La insulina tambin tiene efectos directos en el hipotlamo, ya que estimula el apetito y la secrecin de
gonadotrofinas. Existen otros efectos importantes de la insulina que contribuyen al hiperandrogenismo: inhibicin de la produccin
heptica de la SHBG y de la protena transportadora tipo de IGF-1 (1GFBP-1), lo cual aumenta las concentraciones circulantes de
andrgenos libres y produce una mayor actividad ovrica. DIAGNOSTICO: La mayora de los datos clnicos que aparecen en pacientes
con SOP pueden formar parte de otras patologas no necesariamente relacionadas con alguna disfuncin hormonal y no los hechos
aislados deben siempre sugerir el diagnstico. La severidad o la rpida evolucin debe alertar a los clnicos a la realizacin de un
diagnstico diferencial ms extenso. Acn: Su permanencia de los 20 aos en adelante debe considerarse sospechosa. Es imperativo
interrogar acerca de irregularidades menstruales, hirsutismo o cualquier otro dato de hiperandrogenismo. En mujeres con acn se ha
encontrado datos de SOP hasta en el 45% de los casos. Hirsutismo: Se define como el crecimiento excesivo del vello corporal terminal
en mujeres, en reas anatmicas donde el desarrollo de los folculos depende de la estimulacin andrognica: Tercio proximal en cara
interna de muslos, abdomen, pecho, parte baja de la espalda y cara entre otros. El grado y la severidad se evalan de acuerdo a la
escala de Ferriman-Galwey. Virilizacin: La aparicin de hipertrofia clitordea, amenorrea prolongada, aumento de la musculatura,
atrofia de los senos, hirsutismo severo y habitus masculino obliga a descartar la presencia de hiperplasia adrenal, hipertecosis o
tumores ovricos o adrenales. Irregularidad menstrual y anovulacin: Las mujeres con SOP presentan grados variables de disfuncin
ovulatoria, manifestada como oligomenorrea, dismenorrea y amenorrea. Incluso, frecuentemente presentan infertilidad (17.5% vs 1.3%
en normales). La relacin entre el exceso de insulina y la anovulacin se atribuye en primer lugar a la hiperandrogenemia y en segundo
a la estimulacin del inhibidor 1 del activador del plasmingeno (PAI-1). Este inhibidor no solo regula la eliminacin de los depsitos de
fibrina de los vasos sanguneos sino que a nivel del ovario, inhibe a las colagenasas responsables de la ruptura folicular. Acanthosis
Nigricans: Se trata de una hiperplasia hiperpigmentada de la piel, la cual aparece predominantemente en el cuello y en pliegues
cutneos como axilas y codos. La importancia de su deteccin radica en que su presencia correlaciona significativamente con los
estados de resistencia a la insulina e hiperinsulinemia compensatoria. Dependiendo de las poblaciones estudiadas aparece ligada a
obesidad hasta en el 74% de los casos constituyndose como un factor de riesgo independiente para DM tipo 2. Aproximadamente el
30% de las pacientes con SOP lo presentan. El SOP es un diagnstico de exclusin y no es necesaria la presencia de alteraciones en los
niveles hormonales o quistes en los ovarios para considerar su presencia. Escrutinio: Determinacin en ayunas y durante los primeros 7
das del ciclo de: a) LH y FSH en pool (el promedio de 3 muestras recolectadas con 20 minutos de diferencia) y Prolactina. b)
Testosterona total y libre. c) Dehidroepiandrosterona sulfato y 17 alfahidroxiprogesterona. En el 66% de los pacientes se documenta
una relacin LH:FSH (>2.5:1), la cual es caractersticade SOP. La prolactina debe ser determinada en la evaluacin de cualquier paciente
con amenorrea; muchas de las causas de hiperandrogenismo pudieran incrementarla pero valores >100 sugieren patologa hipofisiaria.
La testosterona total es la prueba ms validada para evaluar la presencia de tumores secretores de andrgenos: niveles >150 ng/dL
fuertemente sugieren la posibilidad de tumores ovricos o adrenales. Cuando la paciente desea procrear es fundamental determinar si
los ciclos son ovulatorios. Los niveles de progesterona <2 ng/mL despus del da 21 del ciclo son interpretados como anovulacin.
Estudios de Imagen: La ultrasonografa (USG) plvica es el mtodo de eleccin para la evaluacin inicial de los anexos.
Caractersticamente se trata de folculos de 8 mm con aumento del estroma central, aunque con la tcnica vaginal pueden detectarse
quistes de 3-5 mm. Los quistes ovricos aparecen irregularmente en las pacientes con SOP. El estudio de imagen de eleccin para el
estudio de las suprarrenales es la tomografa axial computarizada de alta resolucin. TRATAMIENTO: Prdida de peso. La obesidad es
prevalente en mujeres con SOP, y se asocia con un empeoramiento de los sntomas. La prdida de peso de un 5-7% con respecto al
basal produce un descenso en la concentracin circulante de andrgenos, insulina y lpidos, lo cual se acompaa de una mejora de la
sintomatologa y de las posibilidades de presentar ciclos ovulatorios. Tratamiento hormonal. Los anticonceptivos hormonales
combinados proveen varios beneficios en las pacientes con SOP y durante mucho tiempo han sido la piedra angular del tratamiento.
Restituyen los ciclos menstruales con eficiencia y mejoran el hirsutismo en ms del 60% de los casos por su efecto inhibidor de la LH, lo
que aumenta los niveles de SHBG. Tambin suprimen el metabolismo de los andrgenos en las adrenales y disminuyen el nmero de
receptores de la 5a-reductasa en la piel, lo que beneficia al acn. En general, las progestinas protegen al endometrio oponindose a los
efectos proliferativos de los estrgenos pero tienen un efecto negativo en los parmetros metablicos ya que aumentan la resistencia a
la insulina y los niveles de triglicridos. Agentes sensibilizadores de la insulina. Est documentado que la disminucin en los niveles de
insulina mediante el uso de este grupo de frmacos se acompaa de una mejora del cuadro clnico y de las anormalidades metablicas.
Metformina: Acta principalmente en el hgado inhibiendo parcialmente la gluconeognesis, lo que reduce en un 17-25% la produccin
de glucosa y en el msculo esqueltico donde incrementa la captacin de glucosa estimulada por insulina en un 29% y disminuye la
MANUAL DE TRABAJO DEL CURSO ENARM CMN SIGLO XXI
CURSO ENARM CMN SIGLO XXI TEL: 36246001 Pharmed Solutions Institute PGINA 298

oxidacin de los cidos grasos en un 10-20%. En mujeres con SOP el metformn ha demostrado inducir la ovulacin y mejorar las
posibilidades de lograr un embarazo, incluso en pacientes sometidas a fertilizacin in vitro. La combinacin de metformn con
anticonceptivos orales o acetato de ciproterona, se ha acompaado de una mejora en los parmetros clnicos y metablicos.
Tiazolidinedionas: En pacientes con SOP la troglitazona, el primer medicamento del grupo y el ms estudiado (actualmente no
disponible para su venta en Mxico) mejora la resistencia a la insulina disminuyendo la dehidroepiandrosterona sulfato, la testosterona
libre, la androstenediona y la LH, e incrementando la protena transportadora de hormonas sexuales. Tratamiento de la anovulacin. El
citrato de clomifeno, solo o en combinacin con la prdida de peso, sigue siendo, dada su seguridad y simplicidad, el tratamiento de
primera eleccin en la infertilidad de origen anovulatorio asociado al SOP. Acetato de ciproterona, para tratamiento de hirsutismo.
Agonistas de la GnRh. Los agonistas de la GnRH como el leuprolide, disminuyen la produccin ovrica de esteroides por supresin de LH
y FSH. Este tratamiento es altamente efectivo en mujeres con SOP severo o hipertecosis ovrica. Tratamiento Quirrgico. La reseccin
en cua bilateral de los ovarios es un procedimiento quirrgico que se realiz con xito en pacientes con SOP durante muchos aos.
Esto provoca una reduccin en los niveles de LH y produccin de andrgenos.

CASO CLINICO
Acude a consulta femenino de 23 aos de edad que presenta falta de periodo menstrual y desea saber si esta embarazada, la paciente
se dedica a su casa, vive y tiene relaciones con su pareja desde hace 2 aos y quiere tener su primer hijo, refiere que nunca ha sido
regular, a la exploracin fsica se observa con una talla de 156 cm y peso de 74 kg, se observa acn en cara y espalda con hirsutismo, se
observa obscurecimiento de cuello y axilas.

PREGUNTA
Cul es la causa ms probable de la amenorrea en este caso.

RESPUESTA
a.- Hiperandrogenismo.
b.- Incremento de progesterona.
c.- Disminucin de estrgenos.
d.- Aumento de gonadotrofinas.

CASO CLINICO
Se trata de femenino de 27 aos de edad la cual cursa con esterilidad primaria de dos aos de evolucin, sin diagnostico definitivo aun,
acude a la consulta debido a que se han acentuado sus sntomas, la cual refiere presencia de periodos menstruales irregulares, a la
exploracin usted observa hirsutismo, acn en frente y espalda, obesidad con datos de acantosis nigrans.

PREGUNTA
Enva a la paciente para realizar estudios de imagen, cual es el diagnostico que espera encontrar ms probablemente:

RESPUESTA
a.- Hiperplasia Suprarrenal.
b.- Sindrome de Silla Turca Vacia.
c.- Sindrome de Ovarios Poliquisticos.
d.- Sindrome de Cushing.

CANCER OVARICO. CIENCIAS BASICAS: Entre los cnceres del tracto genital femenino, el de ovario es el tercero en frecuencia por
detrs del cncer de crvix y de endometrio, pero es responsable de casi la mitad de las muertes por cncer genital, debido a la
dificultad para el diagnstico precoz. El ovario puede originar tumores histolgicamente muy diferentes y siempre ha sido un problema
su clasificacin porque la patogenia est menos clara que en otros tumores. Factores de riesgo: Adquieren cada vez ms importancia
los antecedentes familiares y las mutaciones hereditarias como las del BRCA1 y BRCA2, igualmente se ha descrito concentraciones
elevadas del oncogn HER2/neu y mutaciones en el p53, hiperestrogenismo, uso de anticonceptivos, disminuye con la paridad. SALUD
PUBLICA: Los tumores ovricos constituyen el tercer grupo de tumores en la mujer. Aproximadamente 1 de cada 10 fminas lo tendr a
lo largo de la vida, la mayora de las cuales requerir de una evaluacin quirrgica. Estos se presentan desde edades tempranas hasta
avanzadas. La experiencia clnica revela la alta incidencia en la etapa del climaterio, comprendida entre los 35 y 65 aos de edad.
CLASIFICACIN: La OMS clasifica los tumores ovricos de acuerdo con el tejido de origen. Ver cuadro anexo. Es necesario puntualizar
que los tumores borderline o tumores en el lmite de malignidad histolgica, son formas tumorales con entidad propia, no un estadio
evolutivo. Histolgicamente se diferencian de los benignos por: estratificacin epitelial, aumento de la actividad mittica, atpia nuclear
con ausencia de invasin estromal, y porque pueden metastatizar. PATOGENIA: En los ovarios pueden desarrollarse tumores benignos
o malignos, qusticos o slidos, de tipo epitelial, conjuntivo, funcionantes o no o teratomatosos. Los tumores ms frecuentes no son
neoplsicos, sino que derivan del desarrollo folicular; son formaciones qusticas originadas por un estmulo anormal del folculo o
alteraciones en el proceso de involucin. Los verdaderos procesos neoplsicos presentan una constitucin histolgica muy diversa, de
origen epitelial, le continan los procedentes del estroma gonadal y, finalmente, los tumores de las clulas germinales. Se conoce que
los tumores benignos del ovario no constituyen un grupo bien definido y algunos de ellos pueden malignizarse en su evolucin.
Aproximadamente de 75 a 85 % de los tumores son en principio benignos. Hoy da, los factores causales del cncer de ovario son
desconocidos, aunque se plantea que en el mbito celular es el resultado de la acumulacin de mltiples defectos genticos menores.
Los estudios epidemiolgicos han permitido enunciar 3 teoras con respecto a las causas de esta enfermedad: La divisin celular
ininterrumpida y la continua regeneracin del epitelio ovrico con cada ovulacin proporcionan la oportunidad para una mutacin y
transformacin maligna por la estimulacin de los ovarios y por hormonas gonadotrpicas, que de forma mantenida pueden inducir a la
malignizacin. La exposicin del ovario a diferentes agentes carcinogenticos. Los eventos moleculares que dirigen el desarrollo del
MANUAL DE TRABAJO DEL CURSO ENARM CMN SIGLO XXI
CURSO ENARM CMN SIGLO XXI TEL: 36246001 Pharmed Solutions Institute PGINA 299

cncer epitelial de ovario son desconocidos. A pesar de esto, en investigaciones epidemiolgicas se han identificado algunos factores
causales en la carcinognesis del cncer de ovario: factores endocrinos, ambientales y genticos, aunque se incluyen otros, tales como:
nuliparidad, historia familiar, menarquia temprana, menopausia tarda, raza blanca y aumento de la edad. La incidencia especfica
aumenta segn la edad de la paciente, con un pico en la sexta dcada de la vida. Es infrecuente en mujeres menores de 40 aos. El
riesgo de que se presente esta enfermedad disminuye con la paridad, con el uso de contraceptivos hormonales y la lactancia; tambin
parece que la histerectoma o la ligadura de trompas con conservacin de los ovarios disminuyen la posibilidad de aparicin. El cncer
de ovario con historia familiar constituye apenas 2,5 % del total. Al respecto, se han identificado 3 tipos de cnceres hereditarios, a
saber: 1. Cncer en sitio especfico: Existe alto riesgo si hay cncer en 2 familiares de primer grado (50 % de probabilidades de ser
afectado). 2. Sndrome de cncer familiar mama-ovario: Patrn de transmisin dominante; asociado a los cromosomas 16q y 17q. Se
considera este sndrome cuando hay 5 casos de cncer de mama y ovario en la familia, incluyendo 2 de cada uno de estos 3. Cncer
colorrectal no polipsico (sndrome de Lynch de tipo II): Incluye mltiples adenocarcinomas, cncer de colon familiar, de ovario,
endometrio, mama, as como de los sistemas gastrointestinal y urinario. DIAGNOSTICO: Entre los datos clnicos de sospecha figuran:
crecimiento lento del abdomen, sntomas compresivos, dolor abdominal agudo o subagudo, sobre todo cuando se complican por
torsin (frecuente en los benignos), rotura, hemorragia o infeccin, rara vez datos indirectos de actividad especial endocrina o
metablica, tales como: pubertad precoz, hemorragia uterina disfuncional, amenorrea, hirsutismo, tirotoxicosis, sndrome de Cushing,
galactorrea, policitemia, hipoglucemia e hipercalcemia, por citar algunos. Por lo general, los sntomas que ms refieren las pacientes
incluyen cierto malestar abdominal, seguido de distensin abdominal, a causa de la presencia de ascitis o una masa tumoral que
paulatinamente aumenta de tamao, as como tambin sntomas gastrointestinales (como nuseas, dispepsias, saciedad precoz y
constipacin); los urinarios son menos frecuentes, al igual que la metrorragia. Estos sntomas tardos ocurren con ms frecuencia en
mujeres premenopusicas. Es difcil de diagnosticar en etapas tempranas, pues las manifestaciones clnicas son escasas y fcilmente se
confunden con las de otras afecciones, adems, no es fcil crear un sistema de deteccin precoz (tamizaje).

CASO CLINICO
Se trata de paciente femenino de 51 aos de edad la cual es originaria del estado de Mxico, comerciante ambulante, cuenta con
antecedentes de familiares con cncer que ya murieron, no especifica qu tipo de cncer, acude a consulta debido a que desde hace
varios meses ha presentado prdida de peso de ms de 15 % total, agrega fatiga, malestar generalizado, refiere que se realiza deteccin
oportuna de cncer mediante papanicolao hace dos aos sin datos de importancia y deteccin de cncer de mama sin datos
patolgicos, acude a urgencias debido a que se encuentra con distencin abdominal importante, se realiza paracentesis donde se
encuentra datos sugerentes de adenocarcinoma, se realiza rastreo tomografico de mama y pelvis sin encontrar datos patolgicos, sin
embargo se encuentra CA-125 elevada, posteriormente se toma biopsia dirigida a ovarios con reporte de patologa positivo para cncer
ovrico.

PREGUNTA
Cul es la conducta ms apropiada para seguir.

RESPUESTA
a.- Cirugia citorreductora.
b.- Cirugia citorreductora mas cisplatino y paclitaxel.
c.- Mastectomia bilateral y ooforectomia bilateral.
d.- Mastectomia y ooforectomia bilateral mas tamoxifen y radiacin.

MANUAL DE TRABAJO DEL CURSO ENARM CMN SIGLO XXI
CURSO ENARM CMN SIGLO XXI TEL: 36246001 Pharmed Solutions Institute PGINA 300

MASTOPATIA FIBROQUISTICA (MFQ). CIENCIAS BASICAS: Tambin conocida como enfermedad broqustica de la mama, cambios
broqusticos, displasia mamaria y mama nodular dolorosa, es una entidad clinicopatolgica crnica no maligna, que se evidencia como
una zona dura, debido a la proliferacin del tejido conectivo, del epitelial y a la presencia de quistes, de forma focal o difusa. Consisten
en abultamientos circunscritos a la mama que pueden confundirse con neoplasias. La diferencia es que son bilaterales y tienen
variaciones en su tamao de acuerdo con la fecha del ciclo en que se examinan; su volumen es mayor en etapa premenstrual. La
naturaleza precancerosa de la MFQ es muy discutida, aceptndose que su transformacin maligna es baja. La MFQ simple no tiene
riesgo de malignizacin; s lo tiene la mastopata proliferativa con atipia. SALUD PUBLICA: Entre un 45-85% de las pacientes que acuden
a una consulta de mama lo hacen por este cuadro clnico. Constituye el proceso benigno ms frecuente de la mama. Produce sntomas
incapacitantes en el 5-10% de las pacientes. Se observa, clnicamente, en un 50% de las mujeres adultas y se detecta en un 90% de los
estudios histolgicos. Representa la alteracin ms comn evidenciada en la mama de la mujer durante la madurez sexual. PAOGENIA:
En el origen de la mastopata fibroqustica se involucran factores hormonales, como: desequilibrio estrgeno-progesterona, con
concentraciones en exceso de estrgenos y disminucin en las concentraciones de progesterona, y concentracin elevada de
prolactina en la sangre. DIAGNOSTICO: Clnicamente, se maniesta con molestias en una o ambas mamas y con dolor a la palpacin. El
dolor suele ser uni o bilateral, generalmente de carcter cclico, premenstrual. Otros sntomas son nodularidad y/o induracin de forma
variable y secrecin por el pezn; la telorrea serosa suele aparecer en la MFQ y en los quistes simples y suele ser bilateral y plurioricial,
igual que la de aspecto lechoso. La sintomatologa es frecuente que mejore con el embarazo y lactancia y tambin tras la menopausia.
A la exploracin, se palpa un engrosamiento en forma de placa o mltiples irregularidades en una parte de la mama, sobre todo en
cuadrantes superiores y externos, normalmente bilateral. Si hay un ndulo dominante, se presenta de forma regular, borde y supercie
denidos y con poca movilidad. Los sntomas en pacientes con mastopata fibroqustica coinciden en mastodinia y ndulos no
adheridos en ambas mamas, que aumentan de volumen en etapa premenstrual. Para su diagnstico, adems de la exploracin y
clnica, contamos con las pruebas de imagen, cuyo objetivo fundamental es conrmar o descartar la existencia de una lesin maligna.
Las lesiones de mama se clasican mediante el sistema I-RADS, que caracteriza los hallazgos de la imagen en varias categoras que van
desde la normalidad hasta la conrmacin histolgica de malignidad. Se considera el idioma universal en el diagnstico de la patologa
mamaria. Este sistema fue disenado en principio para clasicar las lesiones
detectadas mediante mamografa; actualmente se han incorporado la ecografa
y la RM a esta clasicacin. Las pruebas de imagen son: Mamografa: zonas
densas, difusas y con quistes. Los ndulos descritos (BI-RADS 2) son los quistes
simples demostrados en el estudio ecogrco. La placa brosa se maniesta en
el estudio mamogrco como una densidad focal asimtrica de mrgenes mal
denidos (I-RADS 3), precisando de la ecografa para completar el estudio.
Ecografa: lo ms habitual es que nos podemos encontrar en la MFQ imgenes
de similares caractersticas ecogrcas que en las tumoraciones qusticas,
diferencindose en su nmero y en su dimetro mximo. Atendiendo a su
dimetro mximo, dividimos la MFQ en: Tipo I (de pequeas formaciones):
tumoraciones qusticas de dimetro < 0,5 cm. Tipo II (de medianas formaciones):
entre 0,5---2 cm. Tipo III (de grandes formaciones): > 2 cm. Como todas las
tumoraciones qusticas, ecogrcamente se presentan redondeadas u ovaladas,
con ecoestructura generalmente anecoica (BI-RADS 2), aunque pueden presentar
otros aspectos como hipoecognico con nos ecos internos, de contenido turbio
o espeso, a veces con nos tabiques en su interior (BI-RADS 3). Sus bordes estn bien denidos, su eje mayor suele ser horizontal y, con
gran frecuencia, estn presentes los signos indirectos (refuerzo acstico posterior y sombra acstica lateral), pudiendo ser, a veces,
bilobuladas. No tienen ninguna seal Doppler interna. En muchas ocasiones, tras el estudio ecogrco, se modica la categora I-RADS
previamente asignada por mamografa. Est indicada la realizacin de una ecografa mamaria ante la presencia de una alteracin
mamogrca para una caracterizacin ms precisa, como es la presencia de una densidad asimtrica focal que, cuando no es palpable,
la lesin es categora BI-RADS 3. Tambin, ante la evidencia de una distorsin arquitectural en el seno de reas densas de tejido
mamario, catalogada como BI-RADS 4. En general, se puede armar que la RM tiene una alta sensibilidad y baja especicidad en la
evaluacin de las lesiones mamarias. Tambin se dispone de las pruebas diagnsticas intervencionistas, como la puncin-aspiracin con
aguja na (PAAF)/biopsia con aguja gruesa (AG): brosis, proliferacin epitelial, adenosis, quistes y, en ocasiones, hiperplasia
intraductal o lobulillar tpica o atpica. Ante un cuadro clnico---eco y radiolgico dudoso (BI-RADS 3) o sugerente de malignidad (BI-
RADS 4 y 5), hay que hacer una PAAF y/o BAG como primera medida. La MFQ asintomtica no requiere seguimiento; la sintomtica se
seguir con ecografa y/o mamografa. TRATAMIENTO: Los AINES tpicos en gel como el piroxicam o diclofenaco al 2% deben ser
considerados para el control localizado del dolor en el tratamiento de la mastalgia. Debe iniciarse con el uso de preparados
hormonales existentes para el tratamiento sintomtico, tales como los progestgenos (de uso tpico o por va oral o vaginal),
antiprolactnicos (bromocriptina), antiestrgenos, fundamentalmente el tamoxifeno, medicamento no autorizado en nuestro pas para
el tratamiento de la mastalgia pero s en otros pases, y antigonadotropos (danazol). Si dichos preparados fracasan, se adoptarn otras
diversas medidas paliativas. Actualmente, se propone alfa dihidroergocriptina por su alta efectividad, mnimas reacciones secundarias y
fcil dosificacin. El alfa dihidroergocriptina es un derivado hidrogenado del alfa ergocriptina, que es un alcaloide natural del Ergot
utilizado en el tratamiento de la hiperprolactinemia. Tiene un efecto dopaminrgico significativo en el cuerpo estriado y en la hipfisis
y es capaz de disminuir las concentraciones sricas de prolactina mediante la inhibicin de la adenilato-ciclasa dependiente de los
receptores dopaminrgicos D2 en los lactotropos. PREVENCION: La edad recomendable para el inicio del examen medico mamario
rutinario es a partir de los 19 aos. Los pacientes con BCRA1 y BCRA2, deben inicar la exploracin mdica mamaria entre 18-21 aos de
edad. La autoexploracin aumenta la probabilidad de que la mujer encuentre anormalidad en las mamas.




MANUAL DE TRABAJO DEL CURSO ENARM CMN SIGLO XXI
CURSO ENARM CMN SIGLO XXI TEL: 36246001 Pharmed Solutions Institute PGINA 301

CASO CLINICO
Una mujer de 43 aos de edad con un historial de cambios fibroqusticos de la mama, que se presentan con una masa palpable en la
mama derecha. No tena antecedentes familiares de cncer de mama. El examen clnico revel una masa de 4x3 dolorosa, movil, bien
circunscrita en el cuadrante inferior interno de la mama derecha. La mamografa mostr una masa bien delimitada con pared
engrosada. En la ecografa, la masa era hipoecoica y claramente definidas con pared gruesa e irregular. La lesin fue extirpada. El
examen macroscpico mostr una lesin qustica bien delimitada de 3.3x1.7 cm de tamao, con aspecto gelatinoso espeso de material
homogneo, de color amarillo oscuro en su lumen. El espesor de la pared del quiste fue de aproximadamente 3 mm, y la superficie
interior tena plegamientos papilares reminiscentes de papiloma qustica.

PREGUNTA
Cul es la diagnostico mas probable?

RESPUESTA
a.- Galactocele.
b.- Fibroma.
c.- Lipoma.
d.- Fibrosarcoma.

CANCER DE MAMA. CIENCIAS BASICAS: Es un padecimiento crnico, heterogneo con una evolucin irregular y lenta. Es la primera
causa de muerte por neoplasia en la mujer en el mbito mundial. SALUD PUBLICA: Cerca de 500 mil muertes cada ao a nivel mundial,
de las cuales el 70% ocurre en pases en desarrollo. Las tasas de incidencia tienen una variacin considerable en todo el mundo; las
ms altas se encuentran en Europa y Norteamrica con cifras estandarizadas de 99.4 por 100 mil mujeres. En cuanto a la magnitud
actual del cncer de mama en Mxico, a partir de 2006 ste ocupa el primer lugar de mortalidad por tumor maligno en las mujeres
mayores de 25 aos, desplazando de esa posicin al cncer cervicouterino. En el ao 2010 la tasa estandarizada de mortalidad fue de
18.7 por 100 mil mujeres de 25 y ms aos, lo que representa un incremento del 49.5% en los ltimos 20 aos. PATOGENIA: Los
factores de riesgo para el desarrollo del cncer de mama son los siguientes: A) Biolgicos: Sexo femenino, envejecimiento: A mayor
edad mayor riesgo, antecedente personal o familiar de cncer de mama en madre, hijas o hermanas, antecedentes de hallazgos de
hiperplasia ductal atpica, imagen radial o estrellada, as como carcinoma lobulillar in situ por biopsia. Vida menstrual mayor a 40 aos
(menarca antes de los 12 aos y menopausia despus de los 52 aos), densidad mamaria, ser portador conocido de los genes BRCA1 o
BRCA2. B) Iatrgenos o ambientales: Exposicin a radiaciones ionizantes, principalmente durante el desarrollo o crecimiento (in utero,
en la adolescencia), tratamiento con radioterapia en trax. C) Factores de riesgo relacionados con los antecedentes reproductivos:
Nuliparidad, primer embarazo a trmino despus de los 30 aos de edad, terapia hormonal en la perimenopausia o posmenopausia por
ms de cinco aos. D) Factores de riesgo relacionados con estilo de vida: Alimentacin rica en carbohidratos y baja en fibra, dieta rica
en grasas tanto animales como cidos grasos trans, obesidad, principalmente en la posmenopausia, sedentarismo, consumo de alcohol
mayor a 15 g/da, tabaquismo. DIAGNOSTICO: El cncer de mama en etapas iniciales se presenta de manera subclnica en la mayora
de los casos, es decir que solamente es detectable por estudios de imagen (mastografa, ultrasonido y resonancia magntica), en
menor proporcin por clnica (tumores palpables); sin embargo otra forma de presentacin comn es como un tumor no doloroso que
hasta en 30% se asocia a adenopatas axilares. Los tumores localmente avanzados en nuestro pas representan 70% de las etapas
clnicas al diagnstico, pueden incluir cambios cutneos como edema, ulceracin, cutnides, as como afectacin de ganglios como los
supra e infraclaviculares homolaterales El carcinoma inflamatorio de la mama, un tipo de presentacin poco comn pero de mal
pronstico y que por lo general progresa rpidamente, se caracteriza por una induracin difusa de la mama con eritema, edema y
aumento de la temperatura local en al menos un tercio de la glndula, en la mayora de los casos no existe una tumoracin franca
palpable. Todava menos frecuente es el diagnstico de cncer de mama por los sntomas de la metstasis y no por el tumor primario.
Es de suma importancia tener en cuenta dentro del abordaje diagnstico los factores de riesgo del paciente, sus condiciones generales
y antecedentes heredo-familiares. Estudios de imagen: La mastografa es hasta ahora el mejor mtodo de deteccin, tiene una
sensibilidad diagnstica de 80 a 95%, aunque 10 a 15% de los tumores puede ser oculto sobre todo en mujeres con mamas densas (con
el uso de mastografa digital mejora la sensibilidad diagnstica en este grupo de pacientes). El ultrasonido es en algunos casos una
herramienta complementaria para diferenciar masas qusticas de slidas, para caracterizar lesiones benignas y malignas y como gua
para la realizacin de biopsias de lesiones no palpables. La imagen por resonancia
magntica (IRM) con gadolinio tiene sensibilidad diagnstica de 94 a 100%, pero baja
especificidad (37 a 97%) y valor predictivo positivo de 44 a 96%.Las indicaciones
actuales de este estudio son: a) como estudio de deteccin en mujeres con alto riesgo
(como aquellas portadoras de mutaciones BRCA 1 y 2), b) bsqueda de tumores
ocultos mamarios de presentacin axilar, c) mujeres portadoras de implantes o
prtesis mamarias, d) evaluacin de la respuesta al tratamiento sistmico
neoadyuvante, e) evaluacin complementaria para determinar multicentricidad y
bilateralidad. Biopsia: El fundamento del diagnstico del cncer de mama es la
confirmacin histolgica del mismo, para esto se prefiere la realizacin de biopsias de
mnima invasin con la obtencin de material tisular que per mite determinar factores
pronsticos y predictivos de suma importancia en el manejo integral de las pacientes,
por ejemplo la determinacin de receptores hormonales y de Her2/neu. El
procedimiento de eleccin es la toma de biopsias con aguja de corte (trucut) tanto en
lesiones palpables como en las no palpables; esta forma diagnstica se asocia con una
exactitud del 98.5% (7). En lesiones no palpables, la biopsia debe ser realizada bajo la gua de algn mtodo de imagen (ultrasonido,
mastografa, resonancia, etc). Las biopsias quirrgicas previo marcaje (arpn, radiocoloide, etc.) estn indicadas cuando no es factible el
diagnstico mediante un procedimiento menos invasivo. En la actualidad la biopsia por aspiracin con aguja fina (BAAF) generalmente
MANUAL DE TRABAJO DEL CURSO ENARM CMN SIGLO XXI
CURSO ENARM CMN SIGLO XXI TEL: 36246001 Pharmed Solutions Institute PGINA 302

est reservada para la confirmacin de metstasis en adenopatas loco-regionales y tiene poca utilidad como mtodo diagnstico en la
lesin primaria. La evaluacin patolgica del cncer de mama debe incluir de manera indispensable tipo histolgico, grado, permeacin
vascular y linftica, tamao del tumor, mrgenes, nmero de ganglios y tamao de la metstasis ganglionar, estudios de
inmunohistoqumica que evalen la presencia o no de receptores hormonales para estrgenos y progesterona, Ki67 (>14%), la
sobreexpresin del gen ErbB2 (Her2/neu) o su amplificacin por FISH o CISH, adems de estudios complementarios como
citoqueratinas y factores de crecimiento epidrmico, etc. ESTADIFICACIN: La estadificacin del cncer de mama proporciona
informacin respecto al pronstico y orienta el trata miento. Los estudios de imagen en la actualidad son un complemento para evaluar
el tamao del tumor, la presencia de los ganglios y las metstasis. Ver cuadro anexo. TRATAMIENTO: Integral y multidisciplinario, los
manejos locorregionales son ciruga y radioterapia en cualquiera de sus tres modalidades (neoadyuvante, adyuvante y paliativa) y el
tratamiento sistmico incluye la quimioterapia, la terapia endocrina y la terapia dirigida a blancos moleculares. El tratamiento
quirrgico del tumor primario en el cncer de mama ha pasado por mltiples modificaciones, en la actualidad se divide en ciruga
conservadora y mastectoma con sus mltiples variedades. . Las indicaciones para mastectoma incluyen: 1) contraindicacin para
recibir radiote rapia; 2) enfermedad multicntrica e 3) dificultad para obtener mrgenes adecuados y resultado cosmtico favorable
despus de un intento de ciruga conservadora. Existen varios tipos de mastectoma, la mastectoma total extirpa la totalidad del tejido
mamario incluyendo el complejo areola-pezn (CAP), en la mastectoma total preservadora de piel se realiza una incisin circundante a
la areola y se remueve solo el CAP, la ventaja de esta modalidad es favorecer los resultados cosmticos de la reconstruccin inmediata.
La mastectoma radical modificada es un procedmiento realizado en nuestro pas debido a los estadios localmente avanzados en los
que realizamos el diagnstico en nuestras pacientes e incluye la reseccin de la totalidad del tejido y piel mamaria as como la diseccin
de los niveles ganglionares I y II, procedimientos ms extensos como la mastectoma Halsted estn reservados solo en casos
seleccionados. Por muchos aos, la diseccin de la axila ha sido el estndar de tratamiento. En la actualidad la diseccin del ganglio
centinela es el abordaje quirrgico de eleccin en los casos en que la axila es clnicamente negativa. Tratamiento Adyuvante;
quimioterapia, eliminar la enfermedad micrometastsica antes del desarrollo de clonas resistentes. La quimioterapia basada en
antraciclinas disminuye 33% las recadas y 27% la mortalidad en mujeres menores de 50 aos. Terapias biolgicas: Aproximadamente
15 a 25% de los cnceres de mama tendr sobreexpresin del gen HER2/neu (ErbB2). El Trastuzumab es un anticuerpo monoclonal
humanizado dirigido en contra de la protena HER2. Terapia Endocrina: Entre 50 y 70% de los pacientes con cncer de mama, el tumor
ser hormono sensible por lo que se podrn beneficiar de una de las siguientes modalidades de manejo: Tamoxifn; ha demostrado
una disminucin en el riesgo a recurrencia de 40% y en riesgo de muerte de 35%, as como reduccin en el desarrollo de otro cncer de
mama. Inhibidores de aromatasa: exclusivo de mujeres posmenopusicas, ha demostrado mejorar la supervivencia libre de
enfermedad, no la global. El tratamiento con radioterapia est indicado en todos los pacientes que hayan sido sometidos a ciruga
conservadora. Tratamiento Neoadyuvante Sistmico: Esta modalidad teraputica se ha utilizado desde hace muchos aos y se
considera el estndar en los tumores localmente avanzados. Se recomienda un esquema basado en taxanos y antraciclenos durante
seis a ocho ciclos. El objetivo principal es facilitar las diferentes modalidades quirrgicas. En trminos generales, la enfermedad
metastsica puede clasificarse como visceral (pulmn, hgado y sistema nervioso central) y no visceral (incluye hueso, tejidos blandos y
derrame pleural), con supervivencia global para el primer grupo de 12 meses y para el segundo de tres a cinco aos. Seguimiento:
Consiste en valoracin clnica peridica (trimestral los primeros dos aos, del ao 3 al ao 5 es semestral, y posteriormente, anual), los
objetivos son: detectar recada local, regional o sistmica y la presencia de un segundo primario. Se debe realizar una masto grafa y
tele de trax anuales as como densitometria sea semestral (en posmenopusicas o tratadas con inhibidores de aromatasa).
PREVENCION: La promocin de las conductas favorables a la salud para la prevencin del cncer de mama puede disminuir hasta en
30% la incidencia en la poblacin. Se debe orientar a las mujeres acerca de su responsabilidad en el autocuidado de la salud, disminuir
los factores de riesgo cuando sea posible y promover los estilos de vida sanos como: Dieta rica en frutas y verduras y baja en grasas
animales. Prctica de ejercicio fsico moderado, que se asocia con una disminucin del riesgo de cncer de mama en la poblacin
general. Consumo de cido flico. Amamantar, por lo que debe incluirse entre las ventajas de la lactancia materna. Mantener un
adecuado ndice de masa corporal pues el elevado (> 30) se asocia con un incremento significativo en el riesgo de cncer de mama en
posmenopusicas. La prevencin secundaria para la deteccin del cncer de mama debe incluir la autoexploracin, el examen clnico y
la mastografa. La autoexploracin se debe recomendar a partir de los 20 aos; el objetivo es sensibilizar a la mujer sobre el cncer de
mama, lograr que tenga un mayor conocimiento de su propio cuerpo y que identifique cambios anormales para la demanda de
atencin mdica apropiada. El examen clnico debe ser practicado anualmente a partir de los 25 aos, por personal de salud capacitado
en la exploracin de las mamas. Deteccin anual con mastografa a partir de los 40 aos. Se recomienda el empleo de
quimioprevencin, el empleo de tamoxifeno1-3 y exemestano se asocia con una reduccin en el riesgo de carcinoma ductal invasor
Para decidir el uso de estos agentes se deben tomar en cuenta las contraindicaciones relacionadas con su empleo y los antecentes de
la paciente tales como: Historia de eventos tromboemblicos o hiperplasia atpica del endometrio en el caso de tamoxifeno o el
diagnstico de osteopenia/osteoporosis o enfermedades cardiovasculares con el uso de exemestano. La ciruga profilctica debe tener
un abordaje multidisciplinario considerando que es una ciruga reductora de riesgo de cncer de mama contralateral. Los pacientes con
antecedente de radioterapia (enfermedad de Hodgkin en mediastino), susceptiblidad gentica, e historia de cncer de mama familiar
son los grupo ms recomendados para este tratamiento.

CASO CLINICO
Se trata de femenino de 48 aos de edad originaria del estado de puebla, con el antecedente de importancia de ser portadora de
cncer de mama actualmente bajo tratamiento con tamoxifen, alendronato, acetato de megestrol, acude a urgencia debido a que
presenta nauseas y vomito, fatiga y cansancio generalizado que se acompaa con dolor abdominal difuso, los sntomas no se presentan
relacionados a ingesta de alimento u otras actividades cotidianas, a la exploracin fsica se observa con un ndice de masa corporal de
23, leve deshidratacin de mucosas, palidez generalizada moderada, cardiorespiratorio sin compromiso aparente, abdomen plano
depresible difusamente doloroso, no hay signos de irritacin peritoneal, ruidos intestinales normales, los estudios de laboratorio
revelan biometra normal, qumica sangunea y electrolitos con los siguientes resultados: sodio 130 meq/L, potasio 4,8 meq/L, cloruro
98 meq/L, Bicarbonato 29 meq/l, BUN 15mg/dl, creatinina 0.7 mg/dl.

MANUAL DE TRABAJO DEL CURSO ENARM CMN SIGLO XXI
CURSO ENARM CMN SIGLO XXI TEL: 36246001 Pharmed Solutions Institute PGINA 303

PREGUNTA
Cul es la conducta ms adecuada a seguir para establecer un diagnostico con los datos que presenta el caso.

RESPUESTA
a.- Realizar laparoscopia.
b.- Cuantificar cortisol serico.
c.- Transito gastrointestinal.
d.- Endoscopia alta.

CANCER DE MAMA
Se trata de paciente femenino de 49 aos de edad la cual acude a consulta debido a que desde hace 6 meses ha presentado dificultadas
para caminar y cadas frecuentes por lo mismo, cuenta con antecedentes de importancia como alcoholismo social crnico, tabaquismo
positivo desde los 17 aos hasta la actualidad, hace 2 aos fue realizada histerectoma por sangrado uterino anormal, sin especificar el
diagnostico histopatolgico, a la exploracin fsica se observa incoordinacin, marcha inestables, nistagmus, disartria leve y alteracin
en la prueba dedo a nariz.

PREGUNTA
Considerando el historial clnico cual es la fuente primaria mas probable en este caso.

EXPLICACION
a.- Cncer de mama.
b.- Cncer cervicouterino.
c.- Cncer de colon.
d.- Cncer pulmonar.

MENOPAUSIA Y CLIMATERIO. CIENCIAS BASICAS: Menopausia: es un evento natural, inevitable en la vida de la mujer, que lleva al cese
de la funcin reproductora. Menopausia literalmente significa "cese permanente de la menstruacin", se debe a la prdida de la
funcin folicular ovrica y el diagnstico se hace en forma retrospectiva despus de un perodo de amenorrea de 12 meses. El
promedio de edad de la menopausia en estudios realizados en E.U. y Europa vara entre 48 y 52 aos. Cifras similares se han reportado
en frica, Mxico y Japn. El factor ms importante que determina la edad de la menopausia es el nmero de folculos ovricos, cuyo
nmero es determinado en la fase temprana de la embriognesis. Climaterio o perimenopausia: comienza varios aos antes de que se
produzca la menopausia, incluye el perodo inmediatamente anterior a la menopausia (cuando comienzan las manifestaciones
endocrinolgicas, biolgicas y clnicas) como mnimo se prolonga hasta el primer ao siguiente a la menopausia. Postmenopausia: se
refiere al perodo que comienza a partir de la menopausia, si bien este momento no se puede determinar hasta que se hayan
observado doce meses de amenorrea espontnea. Se ha utilizado el trmino de menopausia quirrgica para referirse al cese de la
menstruacin posterior a la histerectoma sin ooforectoma, pero se recomienda utilizarlo nicamente cuando la operacin va
acompaada de la extirpacin de ambos ovarios con o sin tero. SALUD PUBLICA: Debido a diferentes factores como la disminucin de
la mortalidad materna y de las enfermedades infecciosas, la esperanza de vida se ha prolongado y actualmente es de ms de 70 aos.
Esto ha provocado un aumento del nmero de mujeres que llegan a la menopausia y de los aos que viven como postmenopusicas ya
que, si tomamos en consideracin que la edad en que se produce aqulla es alrededor de los 50 aos. PATOGENIA: Durante la
transicin de la etapa reproductiva a la no reproductiva, la mujer experimenta una serie de eventos, los cuales se inician varios aos
antes de que ocurra la menopausia (cese de la
menstruacin). Generalmente despus de los 40
aos la fase folicular del ciclo se acorta, los niveles
de estradiol, inhibina y de folculoestatina
disminuyen y los de la FSH comienzan a elevarse.
Posteriormente los niveles de la hormona
luteinizante (LH) tambin aumentan aunque en
menor proporcin que la anterior. Los folculos
disminuyen y es ms frecuente la ovulacin
prematura y la fase ltea insuficiente con
disminucin de la produccin de progesterona, lo
que produce un exceso de estrgenos en relacin
a la hormona anterior. Cerca de la menopausia, los
ciclos menstruales a menudo son irregulares con
anovulacin intermitente, los niveles de las
gonadotrofinas son errticas, es frecuente el hiperestrogenismo relativo con disminucin de la progesterona. Clnicamente esto se
manifiesta con sangrados vaginales irregulares, pueden existir perodos de amenorrea con valores de FSH y LH elevados, similares a los
hallados en la menopausia, pero cuando se produce la ovulacin, los niveles hormonales pueden volver a lmites normales. Finalmente,
los folculos ovricos dejan de responder a las FSH y LH; el estradiol desciende a menos de 20 pg/mL, la progesterona es indetectable y
clnicamente hay cese de la menstruacin. Ver cuadro anexo. Despus de la menopausia cambia el origen y la naturaleza del estrgeno
circulante, ya que durante la vida reproductiva predomina el estradiol y en la postmenopausia la estrona. El primero es producido por
el ovario y la segunda proviene en su mayora de la conversin perifrica de los precursores andrognicos en el tejido adiposo, msculo
e hgado. DIAGNOSTICO: El climaterio se asocia con signos y sntomas tpicos relacionados con la deficiencia de estrgenos. Tpicos:
Amenorrea, bochornos, crisis de sudoracin, alteraciones del tracto genitourinario. Generales: Alteraciones psicolgicas, alteraciones
emocionales, alteraciones psicosomticas. Sntomas vasomotores: Los bochornos o sofocos se definen como un aumento en la
MANUAL DE TRABAJO DEL CURSO ENARM CMN SIGLO XXI
CURSO ENARM CMN SIGLO XXI TEL: 36246001 Pharmed Solutions Institute PGINA 304

percepcin del calor dentro del cuerpo, se manifiestan por una elevacin en la temperatura cutnea, vasodilatacin perifrica,
aceleracin transitoria de la frecuencia cardaca y modificaciones en la actividad electrodrmica. Se inician varios aos antes de la
menopausia, aun en mujeres con ciclos regulares, generalmente aumentan conforme stos se vuelven irregulares, alcanzando su
mxima intensidad y mayor frecuencia 1 a 2 aos despus de la menopausia y pueden persistir hasta por 5 15 aos despus de sta.
Los episodios nocturnos de sofocos se refieren como crisis de sudoracin. Debido al mismo origen embriolgico de vagina, trgono
uretral y vejiga, las vas urinarias inferiores presentan cambios atrficos por falta de estrgenos. Son comunes las quejas tanto de
sntomas urinarios (disuria, infecciones urinarias frecuentes) como los propios de vaginitis atrfica: prurito, sequedad vaginal,
dispareunia, sangrado vaginal etc. Las mujeres que no reciben tratamiento con estrgenos, al paso del tiempo pueden desarrollar
uretritis causada por atrofia de la mucosa uretral que produce disuria y vaginitis atrfica. La atrofia de la vagina produce
adelgazamiento del endotelio, prdida de los pliegues, acortamiento de la misma y disminucin de la lubricacin, lo que se relaciona
con dispareunia. Antes de la menopausia las clulas epiteliales de la vagina son ricas en glicgeno, el cual es metabolizado por
lactobacilos, lo que contribuye a formar un ph cido que protege contra las infecciones, posterior a la misma se pierde este mecanismo
protector y puede presentarse mayor frecuencia de infecciones as como tendencia a las ulceraciones. Sntomas psicolgicos, somticos
y emocionales: ansiedad, aumento de la tensin, cambios de humor, depresin, irritabilidad, cansancio, insomnio, dificultad para
concentrarse, ataques de pnico, cefalea, dolores articulares, palpitaciones, parestesias. Despus de la menopausia, el adelgazamiento
del epitelio de la vagina no afecta la funcin sexual cuando hay relaciones sexuales en forma regular. En los casos de atrofia severa o
vaginitis atrfica puede existir dispareunia. Recientemente se ha sugerido que la falta de estrgenos que se produce durante la
menopausia contribuye al desarrollo de la enfermedad de Alzheimer y se ha reportado que la terapia de reemplazo con estrgenos
puede ser til para prevenir o retardar el inicio de esta enfermedad. La osteoporosis y la enfermedad cardiovascular constituyen las
principales causas de morbimortalidad en las mujeres despus de la menopausia. Varios estudios han reportado alteraciones en el
metabolismo de lpidos, principalmente disminucin de los niveles plasmticos de lipoprotenas de alta densidad (HDL-c) y aumento de
lipoprotenas de baja densidad (LDL-c) tanto en mujeres con menopausia fisiolgica como quirrgica. Despus de la enfermedad
cardiovascular, la osteoporosis es el problema ms significativo relacionado con la deficiencia de estrgenos. TRATAMIENTO:
Farmacolgico: 1) Terapia hormonal (TH): se debe ofrecer de primera lnea con estrgenos estrgeno/progestageno. Es la ms eficaz
para el control de sntomas vasomotores (hasta en 75%) y atrofia urogenital del climaterio, as como prevencin de la osteoporosis.
Esta contraindicada en las pacientes con manifestaciones clnicas de hipoestronismo en la perimenopausia o post menopausia. Terapia
cclica: estrgenos de 1-25 das ms progestgeno los ltimos 10-14 das de haber iniciado los estrgenos. Ciclico-combinado:
estrgenos 1-25 das y progestgenos de 1-25 das junto con estrgenos. Continuo cclico: estrgenos diariamente y progestgeno
durante 14 das durante 2-6 meses. Continuo combinado: estrgeno ms progestgeno. Diariamente sin descanso. Intermitente
combinado: estrgenos diariamente y progestgenos 3 dias con 3 de descansos. La principal funcin de los progestgenos es la
proteccin endometrial, ya que los estrgenos solos, tienen riesgo de desarrollo de cncer. Los estrgenos utilizados en la TH solos o
combinados ms conocidos son: Orales; Estrgenos conjugados derivados de equinos simples o combinados con acetato de
medroxiprogesterona (0.625mg), estradil 1mg, etinilestradil + acetato de noretindrona. Estradiol-drosperinona. Transdermica:
estradiol-levonorgestrel, estradiol-acetato de noretindrona. Al utilizarse un progestgeno es recomendable que sea por lo menos 12-
14 das de cada mes. En Mxico el acetato de clormadinona es uno de los progestgenos que se usan frecuentemente como terapia
combinada en el sndrome climatrico. 2) Terapia no hormonal: inhibidores de la recaptura de serotonina y norepinefrina. Clonidina,
gabapentina o veraliprida. Cuando haya contraindicacin para hormonal.

CASO CLINICO
Una mujer de 48 aos de edad, con menstruaciones regulares mensuales refiere frecuentes despertares nocturnos durante los ltimos
3 aos, seguido por una gran dificultad para volver a dormir. Estos despertares inexplicables ocurrieron 2-3 veces por noche 3 noches
a la semana. La interrupcin del sueo causaba somnolencia diurna excesiva, dificultad para concentrarse en el trabajo, por lo que se
encuentra irritable. Ella haba estado tomando anticonceptivos orales diariamente por cerca de 12 aos. Le prescribieron durante 6
meses trazodona antes de dormir sin resultado favorable y posteriomente 4 meses de amitriptilina al acostarse ifualmente ineficaz.

PREGUNTA
Cul es la conducta a seguir ms adecuada.

RESPUESTA
a.- Terapia hormonal de remplazo.
b.- Clonacepam.
c.- Venlafaxina.
d.- Zolpiden.


MANUAL DE TRABAJO DEL CURSO ENARM CMN SIGLO XXI
CURSO ENARM CMN SIGLO XXI TEL: 36246001 Pharmed Solutions Institute PGINA 305

OBSTETRICIA
CONTROL PRENATAL Y DEL EMBARAZO: La mayora de los daos obsttricos y los riesgos para la salud de la madre y del nio pueden
ser prevenidos, detectados y tratados con xito, mediante la aplicacin de procedimientos normados para la atencin, entre los que
destacan el uso del enfoque de riesgo y la realizacin de actividades eminentemente preventivas y la eliminacin o racionalizacin de
algunas prcticas que llevadas a cabo en forma rutinaria aumentan los riesgos. ESPECIFICACIONES: En la atencin a la madre durante el
embarazo y el parto debe de vigilarse estrechamente la prescripcin y uso de medicamentos, valorando el riesgo beneficio de su
administracin. La unidad de atencin deber disponer de un instrumento que permita calificar durante el embarazo, el riesgo
obsttrico en bajo y alto, el cual servir para la referencia y contrarreferencia. CONTROL PRENATAL: Las actividades son; Elaboracin de
historia clnica. Identificacin de signos y sntomas de alarma (cefalea, edemas, sangrados, signos de infeccin de vas urinarias y
vaginales). Medicin y registro de peso y talla, as como interpretacin y valoracin. Medicin y registro de presin arterial, as como
interpretacin y valoracin. Valoracin del riesgo obsttrico. Valoracin del crecimiento uterino y estado de salud del feto.
Determinacin de biometra hemtica completa, glucemia y VDRL (en la primera consulta; en las subsecuentes dependiendo del
riesgo), la embarazada debe ser informada a cerca del propsito de las pruebas de laboratorio. Determinacin del grupo sanguneo
ABO y Rho, (en embarazadas con Rh negativo y se sospeche riesgo, determinar Rho antgeno D y su variante dbil D), se recomienda
consultar la Norma Oficial Mexicana para la disposicin de sangre humana y sus componentes, con fines teraputicos. Examen general
de orina desde el primer control, as como preferentemente en las semanas 24, 28, 32 y 36. Deteccin del virus de la inmunodeficiencia
adquirida humana VIH en mujeres de alto riesgo (transfundidas, drogadictas y prostitutas), bajo conocimiento y consentimiento de la
mujer y referir los casos positivos a centros especializados, respetando el derecho a la privacidad y a la confidencialidad. Prescripcin
profilctica de hierro y cido flico. Prescripcin de medicamentos (slo con indicacin mdica: se recomienda no prescribir en las
primeras 14 semanas del embarazo). Aplicacin de al menos dos dosis de toxoide tetnico rutinariamente, la primera durante el primer
contacto de la paciente con los servicios mdicos y la segunda a las cuatro u ocho semanas posteriores, aplicndose una reactivacin en
cada uno de los embarazos subsecuentes o cada cinco aos, en particular en reas rurales. Orientacin nutricional tomando en cuenta
las condiciones sociales, econmicas y sociales de la embarazada. Promocin para que la mujer acuda a consulta con su pareja o algn
familiar, para integrar a la familia al control de la embarazada. Promocin de la lactancia materna exclusiva. Promocin y orientacin
sobre planificacin familiar. Medidas de autocuidado de la salud. Establecimiento del diagnstico integral. Con el apoyo de los datos
anteriores, se deben establecer los criterios de referencia para la atencin de las gestantes a las unidades de primero, segundo y tercer
niveles. La unidad de atencin debe proporcionar a la mujer embarazada un carnet perinatal que contenga los siguientes datos:
identificacin, antecedentes personales patolgicos, evolucin del embarazo en cada consulta, resultados de exmenes de laboratorio,
estado nutricional, evolucin y resultado del parto, condiciones del nio al nacimiento, evolucin de la primera semana del puerperio,
factores de riesgo y mensajes que destaquen la importancia de la lactancia materna exclusiva, planificacin familiar y signos de alarma
durante el embarazo. Se utilizar ste, como documento de referencia y contrarreferencia institucional. Para establecer el diagnstico
de embarazo no se deben emplear estudios radiolgicos ni administrar medicamentos hormonales. El control prenatal debe estar
dirigido a la deteccin y control de factores de riesgo obsttrico, a la prevencin, deteccin y tratamiento de la anemia, preeclampsia,
infecciones crvicovaginales e infecciones urinarias, las complicaciones hemorrgicas del embarazo, retraso del crecimiento
intrauterino y otras patologas intercurrentes con el embarazo. La unidad de salud debe promover que la embarazada de bajo riesgo
reciba como mnimo cinco consultas prenatales, iniciando preferentemente en las primeras 12 semanas de gestacin y atendiendo al
siguiente calendario: 1ra. consulta: en el transcurso de las primeras 12 semanas. 2a. consulta: entre la 22 - 24 semanas. 3a. consulta:
entre la 27 - 29 semanas. 4a. consulta: entre la 33 - 35 semanas 5a. consulta: entre la 38 - 40 semanas. La prolongacin del embarazo
despus de las 40 semanas requiere efectuar consultas semanales adicionales con objeto de vigilar que el embarazo no se prolongue
ms all de la semana 42. DETECCIN DE FACTORES DE RIESGO: 1) Asociacin entre exposicin a sustancias toxicas y efectos adversos
en la madre o el feto (partos prematuros, hipertensin preeclampsia, bajo peso al nacer). 2) El exceso de alcohol tienen efecto adverso
en el feto, por lo que se sugiere que la mujer embarazada limite su consumo. 3) El tabaquismo se ha asociado a un incremento en el
riesgo de mortalidad perinatal, muerte sbita infantil, RPM, embarazo ectpico, placenta previa, parto prematuro, bajo peso al nacer,
desarrollo de labio y paladar hendido; por lo cual debe ser evitado. 4) El fumar marihuana se asocia a bajo peso al nacer, muerte
perinatal, parto prematuro, evitar su uso. 5) Se ha demostrado la utilidad de la suplementacin con Yodo en poblaciones con alto riesgo
de cretinismo. 6) Tamizaje para preeclampsia, con toma de presin, pruebas de orinas para detectar proteinuria, capacitacin para
reconocer los sntomas de alarma de preeclampsia. 7) Las vacunas con virus vicos atenuados estn contraindicadas durante la gestacin
y solo se recomienda el toxoide tetnico. 8) Los profesionales de la salud deben estar alertas sobre datos que orienten hacia violencia
intrafamiliar, ya que se presenta hasta en 17% en el embarazo y ello dificulta la atencin de la mujer. CONTROL EN LAS CONSULTAS:
De acuerdo a guas de prctica clnica: La primera cita debe ser antes de las 12 semanas: Identificar mujeres con riesgo alto de resultado
adversos del embarazo (DM, HTA, preeclamsia, sndrome de Down, nefropatas, enfermedades de la colgena). Descartar
incompatibilidad a grupo y Rh, anemia hipocromica y megaloblasrtica; sfilis. Solicitar grupo y Rh, BH, EGO y VDRL. Deteccin de VIH y
hepatitis B. Bsqueda de bacteriuria asintomtica y protenas en orina. Envi de pacientes con factores de riesgo a segundo nivel.
Calcular fecha probable de parto por fecha de ltimo periodo menstrual, en caso de duda sobre la edad gestacional solicitar
ultrasonido. Calcular y registra IMC y presin arterial. A las 16 semanas: revisar, discutir y registrar los resultados de todos los
exmenes, plantear el diseo de cuidados. El embarazo no complicado, dar nueva cita, el embarazo complicado o factores de riesgo,
enviarlas a segundo nivel. En hemoglobinas menores de 11g/dl considerar suplemento de hierro. Medir y registrar presin arterial. En
18-20 semanas: Debe realizarse un ultrasonido para detectar anormalidades estructurales. Para las pacientes en que se encuentre la
placenta cerca del OCI, realizar otro USG a la semana 36 medir y registra presin arterial. En 25 semanas en primigestas: Medir fondo
uterino, presin arterial y descartar la presencia de protenas en orina. En semana 28: un nuevo examen para detectar anemia y clulas
atipias. Nivel menor a 10.5 g/dl considerar implementacin de hierro. Ofrecer vacuna anti Rh de ser necesario. Medir fondo uterino,
presin arterial y descartar la presencia de protenas en orina. Realizar tamiz de diabetes gestacional. En 31 semanas: medir fondo
uterino, presin arterial y descartar la presencia de protenas en orina. Identificar mujeres embarazadas que requieren cuidados
especiales. En semana 34: ofrecer segunda dosis de vacuna anti Rh en mujeres Rh negativo. Medir fondo uterino, presin arterial y
descartar la presencia de protenas en orina. En pacientes con tratamiento por anemia revisar sus estudios de laboratorio de control,
ajustar tratamiento. En embarazos con evolucin normal informar fecha probable de parto con medidas preventivas para embarazo
MANUAL DE TRABAJO DEL CURSO ENARM CMN SIGLO XXI
CURSO ENARM CMN SIGLO XXI TEL: 36246001 Pharmed Solutions Institute PGINA 306

posmaduro y deteccin de preeclamsia. En semana 36: medir fondo uterino, presin arterial y descartar presencia de protenas en
orina. Determinar la posicin fetal, en caso de duda realizar USG. Mujeres con producto en presentacin plvica envi a
ginecoobstetricia. En semana 37: se debe descartar presentacin anormal del producto y asegurar atencin oportuna para evitar pos
madurez. En semana 38: medir fondo uterino, presin arterial y descartar la presencia de protenas en orina. Insistir en medidas
preventivas de embarazo pos maduro. En semana 40 o ms: medir fondo uterino, presin arterial y descartar presencia de protenas en
orina, envi a urgencias GO para evaluar induccin. La ganancia de peso total durante el embarazo en una mujer sana 7-18 Kg, se
relaciona con neonatos de 3-4 kg. El incremento en el IMC se ha asociado con preeclampsia y con cesrea. El peso y la estatura se
deben medir y registrar en cada cita de atencin prenatal. Se debe calcula el IMC. En cada cita se debe determinar la altura de fondo
uterino para detectar productos mayores a menores para su edad gestacional; de preferencia utilizar tablas estandarizadas y la
aplicacin correcta de la tcnica. Aunque la auscultacin del foco fetal confirma que el producto est vivo, parece no ofrecer otros
datos; en general mdicos y enfermeras suponen que es satisfactorio para el paciente. Se recomienda registrar la frecuencia del foco
fetal en cada consulta. El examen plvico se ha utilizado para identificar condiciones clnicas como: anormalidades anatmicas, ETS,
evaluar tamao de la pelvis de la mujer y evaluar el crvix uterino, incompetencia cervical o para predecir parto pretermino. Sin
embargo la RPM aumenta 3 veces ms cuando se realiza comparacin con las mujeres en las que no se realiza exmenes plvicos. No
se recomienda el tacto vaginal en forma rutinaria, pues no predice con exactitud la edad gestacional, nacimientos pre trmino o
desproporcin cefalo-pelvica y si favorece la RPM.

CASO CLINICO
Se trata de femenino de 23 aos de edad la cual acude a consulta por odinofagia, malestar generalizado, cefalea, mialgias y artralgia, a
la exploracin fsica observa amgdalas hiperemicas e hipertrficas con secrecin blanquecina, adenopata cervical, refiere que se
enferma con poca frecuencia, durante el interrogatorio la paciente le menciona su inters por quedar embarazada ya que lleva casada
2 aos y as lo desea, usted amplia su interrogatorio dirigido y no encuentra hallazgos de importancia.

PREGUNTA
Cul es la mejor conducta a seguir con esta paciente para tener las mejores condiciones para tener un embarazo con adecuada
planeacin.

RESPUESTAS
a.- Es conveniente indicar suplemento de Hierro y Vitaminas para prevenir defectos.
b.- Realizar estudios tales como BH, QS y EGO para descartar padecimiento no visibles.
c.- Realizar Papanicolaou y colposcopia para descartar virus del papiloma humano.
d.- Realizar serologa para ETS y VIH ya que es una etapa donde se podran evitar la transmisin al producto.

CASO CLINICO
Se trata de paciente femenino de 29 aos de edad con amenorrea de 10 semanas, se realiza PIE resultando positivo, al interrogatorio la
paciente refiere nauseas, cefalea, vomito, prdida de peso aproximadamente 5 kilos, refiere tabaquismo y consumo de alcohol
ocasionalmente, agrega que pocas ocasiones acude a consulta mdica, sus antecedentes GO son Gesta 2, Para 1, Abortos 1, refiere que
su primer embarazo no fue planeado, actualmente vive con su segundo esposo, el cual consume drogas inyectables, tienen 16 meses
de relacin, lo conoci en su trabajo donde labora como mesera, el esposo es originario de Tijuana, a la exploracin fsica palpa una
masa en mama derecha la cual no es dolorosa, esta superficial, en espalda y muslos observa equimosis en proceso de resolucin, la
paciente refiere que se pego accidentalmente, a la exploracin vaginal se observa leucorrea con olor desagradable.

PREGUNTA
Considerando los factores de riesgo y la sintomatologa observada cual es su conducta a seguir para identificar otros riesgos no visibles,
potencialmente prevenibles.

RESPUESTAS
a.- Realiza una bsqueda intencionada de ETS y VIH por los datos indirectos aportados de la paciente.
b.- Enva interconsulta a trabajo social para identificar violencia intrafamiliar ya que las lesiones que observa no se explican con la
informacin proporcionada.
c.- Indica suplemento de hierro y complejo vitamnico siguiendo la normatividad.
d.- Considera que no tiene informacin necesaria para modificar su manejo prenatal de forma distinta al de rutina.

CASO CLINICO
Se trata de paciente de 41 aos de edad la cual refiere 5 meses de no presentar periodo menstrual, sin embargo decide acudir a
consulta porque tiene ardor al orinar y cefalea ocasional de intensidad leve, la paciente refiere presentar estrs laboral y posiblemente
perdida del empleo, cuenta con antecedente de Hipertensin Arterial diagnosticada a los 38 aos, actualmente tratada con ARA con
buena respuesta, a la exploracin fsica usted identifica crecimiento de pezones, a la palpacin abdominal identifica presencia de tero
a la altura de cicatriz umbilical, ausculta y percibe frecuencia cardiaca fetal de 140 lx.

PREGUNTA
Cules son los factores de riesgo ms importantes que se deber mantener controlado por el embarazo actual.

RESPUESTA
a.- Hipertensin arterial preexistente.
b.- Preeclapsia.
MANUAL DE TRABAJO DEL CURSO ENARM CMN SIGLO XXI
CURSO ENARM CMN SIGLO XXI TEL: 36246001 Pharmed Solutions Institute PGINA 307

c.- Glomerulopatia
c.- Descontrol hipertensivo.

CASO CLINICO
Paciente femenino de 21 aos de edad la cual acude a consulta debido a que presenta un atraso del periodo menstrual de 4 semanas,
refiere nausea, mareo leve en dos ocasiones, cefalea pulstil en regin occipital, sensibilidad mamaria, al interrogatorio refiere IVSA a
los 20 aos sin mtodo contraceptivo, a la exploracin vaginal se observa reblandecimiento del cuello uterino, a la exploracin de fondo
uterino se palpa a nivel de snfisis pbica.

PREGUNTA: Integrando signos y sntomas cual sera la edad gestacional.

RESPUESTA:
a.- 4-6 semanas de gestacin.
b.- 6 a 12 semanas de gestacin.
c.- 12 a 18 semanas de gestacin
d.- 16 a 20 semanas de gestacin.

CASO CLINICO
Acude a la consulta paciente con 10 semanas de amenorrea, refiere que se realizo una prueba de embarazo en casa, resultando
positiva, refiere nausea matutina y vomito ocasional, durante la exploracin se observa crecimiento mamario y pigmentacin, se
observa lnea alba pigmentada, la paciente presenta tabaquismo positivo y un ndice de masa corporal de 17, usted indica medidas
correctivas a estos factores de riesgo,

PREGUNTA:
Considerando los factores de riesgo cual es su conducta a seguir para mantener un buen control prenatal?

RESPUESTA
a.- Incrementa la dieta y vigila su estado nutricional para obtener un producto con peso adecuado.
b.- Indica a la paciente que definitivamente debe suspender el consumo de tabaco por el riesgo de bajo peso al nacer.
c.- Envia a la paciente a realizar BH, QS para valorar su estado general y como protocolo segn la norma.
d.- Envia USG para identificar la edad gestacional y el estado del producto ya que hay riesgos para cierre de tubo inadecuado.

CASO CLINICO
Paciente originaria de Chiapas la cual actualmente vive en el estado de Mxico, refiere tener 27 aos, acude para control prenatal,
presenta 6 semanas de amenorrea, niega uso de mtodo anticonceptivo, sus antecedentes GO, gesta 3, para 2, abortos 0, menarca a
los 12 aos, IVSA a loa 19 aos.

PREGUNTA
Considerando la edad gestacional y el correcto control prenatal, cual es la inmunizacin adecuada para esta paciente:

RESPUESTA
a.- Usted indica vacuna de influenza.
b.- Toxoide tetnico.
b.- Vacuna de Hepatitis.
c.- Vacuna DPT.

EMBARAZO ECTOPICO (EE). CIENCIAS BASICAS: Se define como la implantacin del vulo fecundado fuera de la cavidad uterina.
Representa una de las causas ms importantes de abdomen agudo en ginecologa. Alrededor del 95% de los embarazos ectpicos se
localizan en la trompa (ampular 79,6 %, istmico 12,3 %, fmbrico 6,2 %, intersticial 1,9 %). Las formas no tubricas del embarazo
ectpico incluyen el embarazo cervical (0,1%), el embarazo ovrico (0,5%), el embarazo abdominal (1,3%), el embarazo cornual (3%), el
embarazo intraligamentario (0,1%), y el embarazo en cuerno uterino rudimentario. SALUD PUBLICA: El embarazo ectpico es la causa
ms frecuente de morbilidad y mortalidad materna en el primer trimestre de la gestacin, siendo responsable del 9% de las muertes
durante este periodo. En cuanto a la edad, afecta en el 482% a mujeres entre los 30-39 aos y en el 368% entre los 20-29 aos,
representando las nulparas el 344% y las que
han tenido 1-2 hijos el 23,6%. PATOGENIA:
Est determinada por todos aquellos factores
que impiden o dificultan la migracin normal
del huevo hacia la cavidad uterina. Se pueden
distinguir entre: 1) Factores tubulares: pueden
ser debidos a) EPI y dentro de ella la causada
por la salpingitis gonoccica y la TBC genital
(asintomticas en el 80-90% de casos) y la
salpingitis posabortiva. Deterioran la actividad
ciliar y la motilidad tubrica y generan adherencias en la mucosa tubrica que conducen a la estenosis. b) ciruga tubrica previa contra
la infertilidad (21% riesgo relativo), c) ciruga tubrica para la esterilidad permanente (9.3%), d) endometriosis tubrica: favorece la
nidacin precoz, e) anomalas congnitas: hipoplasias (observada en la exposicin intratero al dietilestilbestrol) o trompas
MANUAL DE TRABAJO DEL CURSO ENARM CMN SIGLO XXI
CURSO ENARM CMN SIGLO XXI TEL: 36246001 Pharmed Solutions Institute PGINA 308

excesivamente largas y tortuosas, f) alteraciones funcionales del peristaltismo tubarico o de la actividad ciliar. 2) Factores ovulares:
aquellos que conducen a la; a) Nidacin precoz del huevo, bien sea debido a huevos excesivamente pesados que maduran con rapidez y
no pueden progresar, o al desprendimiento precoz de la capa pelcida del huevo capacitando la implantacin, b) Transmigracin: se ha
sugerido la posibilidad de la hipermigracin del huevo y su implantacin en la trompa contralateral. En el 20% de los casos el cuerpo
lteo se encuentra en el ovario contralateral al de la implantacin, c) Tcnicas de Reproduccin Asistida (TRA): de todos los embarazos
clnicos resultantes de las TRA, el 5-5,7% fueron ectpicos y el 1% fueron embarazos heterotpicos, debido a la hiperestimulacin
ovrica y a las implantaciones mltiples resultantes de la transferencia de varios embriones. Algunos autores afirman que la
implantacin ectpica est estrechamente ligada a la insuficiencia del cuerpo lteo, y que esta insuficiencia es un factor patognico
comn a muchas anomalas de la reproduccin, entre ellas algunas cromosomopatas y abortos. Una vez implantado el huevo en la
trompa, las vellosidades invaden rpidamente el endosalpinx, alcanzando la parte tubrica y el peritoneo. Esta penetracin va
acompaada de una proliferacin vascular y de un hematoma peritubarico e hematosalpinx, que a menudo afecta la trompa
contraleteral, y que se propaga hacia el extremo distal de la trompa, entre la pared y la serosa. A partir de aqu puede evolucionar hacia
el: 1) Aborto tubarico, el huevo carece de vellosidades suficientes y est implantando en el segmento distal de la trompa. Se atrofia, se
desprende y es expulsado por el orificio peritoneal de la trompa. Esta expulsin suele ir acompaada de hemorragia moderada, que
ocupa el fondo de saco de Douglas formando hematocele. En raras ocasiones puede convertirse secundariamente, en embarazo
abdominal. El aborto tubrico se da con ms frecuencia en la implantacin ampular y fmbrica de la trompa. 2) Rotura tubarica, las
vellosidades son suficientes y enrgicas, pero el aumento de la presin intatubarica llega a dificultar el riego sanguneo de la trompa, lo
que produce necrosis tisular que conduce a la ruptura por corrosin vascular (no por estallido). Esta rotura suele acompaarse de
hemorragia intensa, que da lugar al hemoperitoneo y shock hipovolmico, ya que suele afectar las anastomosis de las arterias tubricas
con la uterina ascendente. La rotura tubrica se da con ms frecuencia en la implantacin stmica e intersticial de la trompa. La
metrorragia se manifiesta generalmente antes de la rotura, debido a la insuficiencia placentaria (esteroidognesis reducida), que
precede las manifestaciones clnicas. 3) Ectpico viable, lo habitual del embarazo ectpico es que progrese hasta el segundo o tercer
mes, momento en que se interrumpe, y rara vez llega a ser viable. DIAGNOSTICO: En algunas ocasiones, la interrupcin del embarazo
es tan precoz, que se resuelve por reabsorcin espontnea, sin llegar a dar manifestaciones ni de amenorrea. El embarazo ectpico
accidentado no precisa mucha pericia diagnstica, la paciente suele presentar un cuadro de schock hipovolmico y dolor abdominal
agudo, confirmndose el diagnstico en quirfano, previa puncin fondo de saco de Douglas. Las manifestaciones clnicas del
embarazo ectpico no accidentado en evolucin, se caracterizan en el 90% de los casos por prdidas hemticas irregulares y dolor
abdominal. La triada clsica compuesta por dolor, sangrado y masa anexial solo se presenta en 45% de las pacientes. A la exploracin
fsica 30% de pacientes, no presenta sangrado transvaginal, y 10% pueden tener masa anexial palpable y 10% tener un examen plvico
normal. La localizacin ms frecuente del embarazo ectpico es en la tuba uterina, manifestndose con dolor y sangrado, sin evidencias
de embarazo intrauterino y con una masa anexial. El sangrado del embarazo tubario es lento, gradual y algunas pacientes pueden
permanecer hemodinamicamente estables a pesar de hemoperitoneos de 1000-1500ml. En toda paciente en edad reproductiva y con
vida sexual que presenta retraso menstrual, dolo y sangrado transvaginal debe descartarse la presencia de embarazo tubario a travs
de ultrasonografa plvica transvaginal y determinacin srica de -HCG. La deteccin por ultrasonido transvaginal de una masa anexial,
combinada con una concentracin de -HCG de 1000mUI/ml tienen una sensibilidad de 97%, una especificidad de 99% y unos valores
predictivos positivo y negativo de 98% para diagnstico de embarazo tubario. Para la deteccin temprana del embarazo tubario
temprano el US transvaginal ofrece mayor sensibilidad que el plvico abdominal por lo cual debe realizarse el primero si se cuenta con
el recurso de lo contrario utilizar el pelvicoabdominal. Ante la duda diagnostica de embarazo tubario, la evaluacin laparoscpica est
indicada. Los niveles sricos de -HCG por si solos no diferencian un embarazo tubario de un intrauterino. Las determinaciones seriadas
de -HCG tienen 36% de sensibilidad y 65% de especificidad para la deteccin de embarazo tubario. Los rangos de -HCG de la zona de
discriminacin son de 1500-2000mUI y estos se asocian con la presencia ultrasonografica del saco gestacional intrauterino. La
determinacin srica -HCG es ms sensible que la determinacin en orina. TRATAMIENTO: Puede ser mdico, quirrgico y expectante.
El tratamiento mdico consiste en uso de metotrexate con o sin cido folinico, est recomendado en pacientes con: estabilidad
hemodinmica, embarazo tubario no roto, sin datos de sangrado activo intra-abdominal, pacientes con niveles sricos de -HCG
menores de 2000mUI/ml, tamao del saco gestacional menor a 3.5cm, ausencia de latido cardiaco embrionario. Existen tres protocolos
de administracin de metotrexate para el tratamiento de embarazo ectpico monodosis, doble dosis y multidosis. Se recomienda
monodosis ya que los resultados son similares y con menos toxicidad. Contraindicaciones absolutas: lactancia, disfuncin renal,
heptica, pulmonar o hematolgica, hipersensibilidad a metotrexate, enfermedad acido-pptica. Relativas: saco gestacional mayor a
3.5cm, actividad cardiaca embrionaria. El tratamiento quirrgico puede consistir en laparoscopia o laparotoma. Sin indicaciones para el
tratamiento quirrgico y embarazo tubario: pacientes no candidatas a tratamiento mdico con metrotexate, falla al tratamiento
mdico, embarazo heterotopico con embarazo intrauterino viable, pacientes hemodinamicamente inestables, pacientes con
inestabilidad hemodinmica deben ser estabilizadas antes de entrar a quirfano, en pacientes con embarazo tubario
hemodinamicamente estables, se recomienda el abordaje laparoscpico, siempre y cuando se cuente con el recurso material y humano
necesarios. Ante las siguientes circunstancias se considera de primera eleccin la laparotoma exploradora: Antecedentes de ciruga
abdominal, presencia de adherencias plvicas, inexperiencia del cirujano laparoscopia. Las indicaciones de salpingectomia incluyen:
dao severo de la tuba uterina, embarazo tubario recurrente en la misma tuba uterina, sangrado persistente despus de la
salpingostomia, embarazo tubario mayor a 5cm, embarazo heterotopico, pacientes con paridad satisfecha. El manejo expectante del
embarazo tubario no est claramente establecido. En pacientes con embarazo tubario que son Rh negativo no sensibilizadas, se
recomienda la aplicacin de inmunoglobulina anti D a razn de 250UI. Un embarazo ovrico es difcil de distinguir de un quiste
hemorrgico de cuerpo lteo, incluso durante el acto quirrgico.

CASO CLINICO
Se trata de femenino de 19 aos de edad la cual se encuentra en control prenatal, acude a su consulta de control, refiriendo que desde
hace una semana presenta dolor en fosa iliaca derecha, sensacin de pesantez y distencin abdominal, agrega que ha tenido dificultad
para la evacuacin, refiere que el da previo a la consulta observa sangrado leve transvaginal, la paciente es ingresada a urgencias por
MANUAL DE TRABAJO DEL CURSO ENARM CMN SIGLO XXI
CURSO ENARM CMN SIGLO XXI TEL: 36246001 Pharmed Solutions Institute PGINA 309

sospecha de embarazo se realiza PIE resultando positiva, por lo que es ingresada para observacin ya que el sangrado no ha disminuido
y continua el dolor abdominal.

PREGUNTA
Cul es la su conducta a seguir, considerando la evolucin del paciente.

RESPUESTA
a.- Realizar Bh en busca de anemia y leucocitosis.
b.- Realizar USG transvaginal.
c.- Realizar cuantificacin de Gonadotrofinas.
d.- Cuantificacin de progesterona.

ABORTO, AMENAZA DE ABORTO. CIENCIAS BASICAS: Aborto es la expulsin o extraccin del producto de la concepcin antes de las
22SDG, el feto habr de pesar menos de 500gr. SALUD PUBLICA: A mediados de los aos 90 se calculaba una cifra anual de un milln
700 mil abortos, estimndose que alrededor de la mitad eran inducidos Se presenta aborto espontaneo en 15% de los embarazos
reconocidos y del 13-26% de los no reconocidos. En general 80% de los abortos espontneos ocurren en las primeras 12 semanas de
gestacin. PATOGENIA: Factores de riesgo: Edad materna avanzada (en mayores de 40 aos se duplica el riesgo). Aborto recurrente
(ms de dos prdidas consecutivas aumenta el riesgo al 43%). Perodo intergensico largo (mayor a 10 aos). Bajo peso y sobrepeso
materno. Intervalo de ovulacin-implantacin prolongado (mayor a 72 horas). Bajo nivel de folatos durante la gestacin (nivel srico
</= 2.19ng/ml). Alcohol (no ha sido establecido nivel seguro en ninguna etapa del embarazo). Cocana. Cafena (consumo de ms de
100mg por da = 4 a 5 tazas). Cigarrillo (padre o madre): consumo de ms de 10 cigarrillos por da. AINES (consumo cerca a la
concepcin por su mecanismo de accin sobre las prostaglandinas). Etiologa: Anomalas genticas 50-60% (la ms frecuente,
aneuploidas: trisomas autosmicas 52%, monosoma X 19%, poliploidas 22% y otras 7%). Anomalas endocrinas 10-15%,
desprendimientos corio-amniticos 5-10%, crvix incompetente 8-15%, implantacin anormal de la placenta 5-15%, infecciones
maternas agudas 3-5%: listeria monocitogenes, toxoplasma, parvovirus B19, rubeola, herpes simplex y CMV. Anomalas inmunolgicas:
3-5%. Anomalas anatmicas uterinas: 1-3% (malformaciones: septo uterino, leiomiomas, adherencias). Causas desconocidas: < 5%.
DIAGNOSTICO: Clnico; amenorrea + sangrado vaginal + dolor hipogstrico. Historia clnica. Examen fsico: revela la fuente de sangrado.
Ecografa transvaginal: hallazgos de saco vitelino normal y actividad cardaca fetal (5,5-6 SDG). Es tranquilizador en etapas tempranas
de la gestacin. El diagnostico definitivo de embrazo intrauterino no viable puede ser hecho con los siguientes criterios: ausencia de
actividad cradica en un embrin con una longitud caudal >5mm, saco vitelino irregular y ausencia de polo fetal con un dimetro
promedio de saco gestacional >25mm transabdominal o 18mm transvaginal. Cuando cualquiera de estos hallazgos este presente
repetir el US en 1 semana: muy probablemente mostrara la perdida gestacional. Estudios de laboratorio: Medicin de -HCG
8mediciones seriadas). RECOMENDACIONES GENERALES: Realizar ecografa transvaginal despus de la expulsin espontnea o manejo
mdico. Observacin al menos 30 min posterior al procedimiento, en caso de curetaje. Inmunoglobulina anti-D (despus de evacuacin
o al momento del diagnstico en caso de manejo expectante) con incompatibilidad Rh. Vacunacin antirrubola y sarampin posaborto
si no ha sido vacunada. Solicitar VDLR. La menstruacin debe reorganizarse en 6 semanas. Los niveles de B-hCG retornan a lo normal a
las 2 o 4 semanas posaborto. Iniciar anticoncepcin de inmediato. AMENAZA DE ABORTO: Es la presencia de hemorragia genital
indolora, dolor leve suprapbico, crvix cerrado, tero de tamao adecuado para la edad gestacional y actividad cardaca fetal
demostrada por ecografa. Diagnstico: Amenorrea secundaria, prueba inmunolgica de embarazo positiva, presencia de vitalidad fetal
por ultrasonido, hemorragia de magnitud variable, dolor tipo clico en hipogastrio de magnitud variable, volumen uterino acorde con
amenorrea, sin dilacin cervical. Solicitar prueba de embarazo y bsicos. Tratamiento: Disminuir la actividad fsica, psquica y sexual,
reposo en cama, hidratacin va oral, se puede administrar antiespasmdicos. Si continua con hemorragia referir a segundo nivel de
atencin. No hay intervenciones teraputicas para prevenir la prdida en el 1er trimestre. Se hospitaliza a todas las pacientes que
continen con sangrado transvaginal. Realizar -HCG y ultrasonido plvico para completar evolucin clnica. Hidratacin oral y
parenteral. ABORTO INEVITABLE: Sangrado abundante, contracciones uterinas dolorosas, crvix abierto, tejido gestacional puede
visualizarse a travs del orificio interno, y ruptura prematura de membranas antes de las 20 semanas. Variedad que hace imposible la
continuacin de la gestacin generalmente por la existencia de hemorragia genital intensa o ruptura de membranas, aun sin
modificaciones cervicales o actividad uterina reconocible. Diagnstico: Dolor tipo clico en hipogastrio, volumen uterino igual o menor
que por amenorrea, hemorragia abundante o ruptura de membranas, puede o no haber dilacin cervical. Manejo quirrgico: curetaje
(preferiblemente evacuacin por aspiracin) Manejo expectante: realizar en paciente estable, sin signos de infeccin y con gestaciones
menores a 13 semanas. La expulsin espontnea ocurre en las primeras 2 semanas del diagnstico en el 80% (puede prolongarse a 3 o
4 semanas). Manejo mdico con misoprostol: I trimestre: 800mcg va vaginal cada 6 a 12 horas, hasta 3 dosis, u 800mcg sublingual cada
3 a 4 horas hasta 3 dosis Iniciando II trimestre: 400 mcg vaginal entre 13 y 15 semanas, y 200mcg vaginal entre 16-20 SDG. No olvidar
las reacciones adversas del misoprostol: diarrea y dolor abdominal (10%), cefalea, constipacin, flatulencia, vomito, dispepsia y en <1%
ansiedad, anafilaxia, fiebre, trombocitopenia, hipotensin. ABORTO EN EVOLUCION: Se caracteriza por la presencia de hemorragia
genital persistente, actividad uterina reconocible clnicamente y modificaciones cervicales incompatibles con la continuidad de la
gestacin. Diagnstico: expulsin del tejido ovular inminente, dolor tipo clico progresivo en intensidad y frecuencia, volumen uterino
menor que por amenorrea, hemorragia persistente, dilatacin cervical. Tratamiento: Hospitalizacion, se realizara la AMEU y el LUI
como procedimientos de primera y segunda opcin respectivamente. ABORTO INCOMPLETO: Cuando ha ocurrido la expulsin de una
parte del huevo y el resto se encuentra an en cavidad uterina. Diagnstico: expulsin parcial del producto de la concepcin,
hemorragia y dolor tipo clico de magnitud variable, dilacin cervical y volumen uterino no acorde con amenorrea. Tratamiento:
Legrado si el dimetro del tejido retenido es igual o mayor a 15mm, para evitar complicaciones hemorrgicas e infecciosas; adems, es
la eleccin si ocurre antes de 12 semanas. Algunos estudios han sugerido que el manejo expectante del aborto incompleto en pacientes
estables puede ser seguro, o se puede dar manejo mdico con Misoprostol 600 mcg va oral, o 400 mcg va sublingual, dosis nica y
control en 7 das. ABORTO COMPLETO o consumado: Aquel en el que la expulsin del huevo ha sido total. Diagnostico expulsin
completa del producto de la concepcin, disminucin de la hemorragia y el dolor, se favorece el cierre del orificio cervical. Tratamiento:
MANUAL DE TRABAJO DEL CURSO ENARM CMN SIGLO XXI
CURSO ENARM CMN SIGLO XXI TEL: 36246001 Pharmed Solutions Institute PGINA 310

Generalmente no necesita manejo adicional, si se tienen duda manejarlo como aborto incompleto. ABORTO DIFERIDO o huevo muerto
retenido: Se presenta cuando habiendo ocurrido la muerte del producto de la concepcin, no se expulsa en forma espontnea. Esta
entidad presupone un lapso entre la muerte ovular y la elaboracin del diagnstico. Generalmente existe el antecedente de amenaza
de aborto. Diagnstico: volumen uterino menor que por amenorrea, ausencia de vitalidad fetal (latido cardiaco fetal), no hay
modificaciones cervicales. Los abortos del segundo trimestre de este tipo pueden asociarse a trastornos de la coagulacin. Tratamiento:
Manejo quirrgico: curetaje (preferiblemente evacuacin por aspiracin) Manejo expectante: realizar en paciente estable, sin signos de
infeccin y con gestaciones menores a 13 semanas. La expulsin espontnea ocurre en las primeras 2 semanas del diagnstico en el
80% (puede prolongarse a 3 o 4 semanas). Manejo mdico con misoprostol: I trimestre: 800 mcg va vaginal cada 6 a 12 horas, hasta 3
dosis, u 800 mcg sublingual cada 3 a 4 horas hasta 3 dosis Iniciando II trimestre: 400 mcg vaginal entre 13 y 15 semanas, y 200 mcg
vaginal entre 16-20 SDG. No olvidar las reacciones adversas del misoprostol: diarrea y dolor abdominal (10%), cefalea, constipacin,
flatulencia, vomito, dispepsia y en <1% ansiedad, anafiulaxia, fiebre, trombocitopenia, hipotensin. ABORTO HABITUAL: Es la perdida
repetida y espontanea del embarazo en 3 o ms ocasiones, o de 5 embarzos en forma alterna. Requiere por ser de origen multifactorial,
de una adecuada valoracin clnica y estudios de laboratorio y gabinete para determinar su causa, de un tratamiento especializado y
posteriormente de una atencin prenatal precoz y con calidad durante todo el embarazo. ABORTO SEPTICO: Cualquiera de las
variedades anteriores a las que se agrega infeccin intrauterina. Diagnstico: Escurrimiento de mal olor e incluso secrecin hemato-
purulenta a travs del crvix. Fiebre sin ningn otro sitio evidente de infeccin, hipersensibilidad suprapubica, dolor abdomino-pelvico
a la movilizacin del crvix y tero, ataque al estado general. El examen fsico se encuentra taquicardia, taquipnea, dolor hipogstrico,
tero friable y doloroso con crvix dilatado. Es una complicacin frecuente de abortos ilegales. Agentes involucrados: staphylococcus
aureus, bacilos gram negativos, cocos gram positivos, infecciones mixtas, organismos anaerobios y hongos. Clasificacin: Leve: infeccin
ovular. Moderado: respuesta inflamatoria sistmica secundaria a enfermedad localizada. Severo: sepsis - inflamacin sistmica +
bacteriemia + Choque sptico sndrome de falla multiorgnica. Tratamiento: Referir al paciente a segundo nivel de atencin,
independientemente de la edad gestacional. Soporte hemodinmico. Antibiticos de amplio espectro; Leve: cefalosporina de 1
generacin. Moderado: clindamicina + gentamicina, Ampicilina-sulbactam. Severo: penicilina cristalina + clindamicina (600mgs IV c/8h)
+ gentamicina (80mg IV o IM c/8h). Hidrocortisona 3 gr IV dosis inicial y repetir 1g IV c/8h en las siguientes 24h. Si no hay respuesta
histerectoma o anexohisterectoma. Evacuacin del contenido uterino. Profilaxis - ttanos

CASO CLINICO
Paciente femenino de 24 aos de edad, originaria de Campeche, en comunidad rural productora de aves, acude debido a que presenta
amenorrea secundaria de 8 semanas de evolucin, ella refiere que desde hace tres das ha presentado dolor lumbosacro y presencia de
sangre roja rutilante, por lo que se mantiene en reposo; cuenta con los antecedentes GO de Gesta 4, para 0, abortos 3, ella refiere que
sus embarazos han terminado en abortos a las pocas semanas.

PREGUNTA
Tomando en cuenta el cuadro clnico actual y los antecedentes GO cul es su conducta a seguir para establecer una aproximacin
diagnostica:

RESPUESTA
a.- Realizar USG en bsqueda de anomalas uterinas.
b.- Buscar anticuerpos antifosfolipidicos y anticardiolipinas.
c.- Prueba TORCH.
b.- Determinacin de gonadotrofina corionica.

CASO CLINICO
Se trata de femenino de 23 aos de edad que acude a control prenatal, actualmente cuenta con 22 semanas de gestacion, refiere que
es empleada de una granja y que ha presentado molestias tales como dolor en la regin lumbo-sacra que se extiende hacia la regin
abdominal baja, cuenta con antecedentes de dos abortos expontaneos y por eso se mantiene preocupada cuando presenta estas
molestias.

PREGUNTA
En caso de llegar a trmino el producto y considerando la exposicin laboral, cul sera la complicacin ms probable en el Recien
Nacido.

RESPUESTA
a.- Malformacion Cardiaca.
b.- Sordera.
c.- Malforacion Oftalmica.
d.- Malformacion Renal.

CASO CLINICO
Se trata de femenino de 25 aos de edad la cual acude al servicio de urgencias con dolor en ambas extremidades inferiores, clicos,
ardor al orinar, presencia de manchado transvaginal, refiere que tiene tres das con esta sintomatologa y decidi acudir porque se
incrementaron los sntomas, al interrogatorio refiere contar con 6 semanas de embarazo por FUM, actualmente bajo control prenatal
con mdico familiar, sus antecedentes GO son Gesta 2 para 1, Abortos 0, refiere que el embarazo previo fue sin complicaciones al igual
que el parto, niega toxicomanas ni presencia de otra patologa.

PREGUNTA
MANUAL DE TRABAJO DEL CURSO ENARM CMN SIGLO XXI
CURSO ENARM CMN SIGLO XXI TEL: 36246001 Pharmed Solutions Institute PGINA 311

Luego de ingresar a la paciente a observacin, se realiza cuantificacin de gonadotrofinas observandoce dentro del rango,
disminuyendo la sintomatologa al mantenerse en reposo, cual es la conducta a seguir:

RESPUESTA
a.- Mantener a la paciente en observacin debido a que puede progresar la amenaza de aborto.
b.- Iniciar con medicacin antiespamodica y vigilancia en casa con cita abierta.
c.- Dar indicacin de reposo absoluto y signos de alarma con cita abierta.
d.- Indicar antibitico, indicar reposo, dar signos de alarma y cita abierta.

CASO CLINICO
Se trata de paciente femenino de 28 aos de edad la cual acude al servicio de urgencias ya que refiere que aproximadamente 4 horas
en el trayecto a su trabajo presenta dolor intenso en regin lumbar con irradiacin a miembros plvicos, y sangrado transvaginal
moderado, refiere que cuenta con 9 semanas de embarazo aparentemente normal y bajo control mdico, al ingreso a urgencias
observa el cuello reblandecido y 2 centmetros de dilatacin, y salida de material, se indica USG transvaginal y se observa saco
amnitico abierto sin presencia determinada de producto en su interior.

PREGUNTA.
Cul es su conducta a seguir.

RESPUESTA
a.- Envio a segundo nivel.
b.- Ingreso para vigilancia estricta.
c.- Manejo ambulatorio.
d.- Determinacin de hGC.

INCOMPETENCIA CERVICAL (IC). CIENCIAS BASICAS: Es la entidad clnica en la cual el cuello uterino es incapaz de ejercer su funcin
hasta el trmino del embarazo, con perdida repetida del producto en el segundo trimestre (parto prematuro). Un crvix incompetente
es aquel que, por una anomala estructural o funcional, es incapaz de mantener un embarazo hasta su trmino. En realidad esta
incompetencia del crvix se localiza en el orificio cervical interno y el istmo, de ah que muchos hablen de insuficiencia stmico cervical.
SALUD PUBLICA: Su incidencia vara entre el 0,005 y el 1% de todos los embarazos, y se considera la principal causa de abortos tardos.
PATOGENIA: Se caracteriza por la dilatacin cervical pasiva y por la sucesin de prdidas gestacionales durante el segundo trimestre.
No se acompaa de metrorragia ni de rotura prematura de membranas. Aunque no suele encontrarse una historia obsttrica clsica en
estas pacientes, deben investigarse los factores etiolgicos previos. En realidad la IC es un sndrome al cual se puede llegar por
diferentes causas: clsicamente se admite la 1) Causa congnita; puede darse en los casos de malformaciones uterinas (mujeres
expuestas al dietilestilbestrol durante su embarazo). 2) Causa traumtica. En los casos de partos traumticos vaginales instrumentales
o no, pueden producirse desgarros que afecten al canal cervical y sean la causa de la IC. Otras veces son los legrados con dilatacin los
responsables del cuadro. Por ltimo pueden incluirse en este apartado las intervenciones quirrgicas sobre el crvix; cada da son ms
frecuentes las conizaciones en edad de procrear y, dependiendo de la tcnica y el tamao del cono, la secuela de la IC puede
producirse del 5 al 15% de estas mujeres. 3) Causa oculta. Incluimos aqu todas las IC sin aparente lesin pero a pesar de lo cual se
producen de forma reiterada abortos del segundo trimestre o partos muy prematuros. Son posiblemente casos en los cuales hay una
alteracin histoqumica de los tejidos que constituyen el cuello y de una forma muy especial del colgeno. DIAGNOSTICO: Es una
entidad en la que el diagnstico precoz es fundamental, ya que cuando se objetiva una protrusin cervical de membranas en reloj de
arena, el pronstico es peor. En la anamnesis es esencial buscar factores de riesgo de la incompetencia cervical, como antecedentes de
partos distcicos con frceps o ventosa, en los que se hayan podido producir desgarros cervicales extensos que hayan dejado al crvix
incompetente para sus funciones; dilataciones cervicales traumticas para legrados por abortos o en interrupciones voluntarias de
embarazo, o intervenciones quirrgicas sobre el crvix, como conizaciones o tratamientos destructivos por patologa cervical. En
algunos casos, la etiologa del crvix incompetente no es de origen traumtico sino congnito, entidad en realidad extremadamente
rara y a cuyo diagnstico se llega tras constatar la existencia de una incompetencia cervical sin antecedentes lesivos cervicales. La
incompetencia cervical congnita se ha relacionado con la toma materna de dietilestilbestrol durante la gestacin y con la existencia de
posibles malformaciones uterinas. La ecografa transvaginal tiene un papel de primer orden en la valoracin de las caractersticas
cervicales en el diagnstico de incompetencia cervical y en la prediccin de parto prematuro, que afecta al 8-10% de los embarazos con
o sin rotura prematura de membranas. Su aportacin al diagnstico de la modificacin cervical sin la existencia de dinmica uterina es
fundamental. Desde el punto de vista ecogrfico se ha demostrado que cuanto menor es la longitud cervical ms posibilidades de parto
prematuro existen. En la evaluacin ecogrfica cervical, entienden las siguientes como imgenes de normalidad: crvix formado, mayor
de 30 mm; OCI cerrado, menor de 5 mm; canal cervical con bordes paralelos; canal en forma de cono, con angulacin dirigida al OCI, y
ausencia de herniacin de la bolsa amnitica. Estos criterios de normalidad siempre se deben tener en cuenta en la evaluacin cervical
para realizar el diagnstico de incompetencia cervical, adems del diagnstico ecogrfico, que tambin es fundamental en el control y
seguimiento de esta patologa. Clsicamente, el diagnstico de incompetencia cervical se realizaba mediante exploracin, por la
deteccin de repetidas prdidas gestacionales o de antecedentes traumticos cervicales. Hoy da, la cervicometra y la observacin del
orificio cervical interno (OCI) mediante ecografa son imprescindibles para su diagnstico y seguimiento. Se sabe que la longitud cervical
no es constante durante todo el embarazo, sino que disminuye a medida que ste avanza. Para la apertura del OCI debern
considerarse los valores patolgicos que, antes de la semana 28, sean inferiores a 30 mm y superiores a 10 mm. TRATAMIENTO: 1)
Tratamiento antes del embarazo: Es siempre quirrgico y tiene como objetivo reconstruir la normal anatoma del cuello. Se han
propuesto diferentes tcnicas: traquelorragia, reduccin del calibre stmico cervical, reparacin de desgarros y cerclaje. Todos estos
procedimientos han sido prcticamente desechados dados los malos resultados obtenidos. 2) Tratamiento durante el embarazo. Se ha
propuesto tratamiento mdico y tratamiento quirrgico. El tratamiento mdico consiste en reposo, abstinencia sexual y administracin
MANUAL DE TRABAJO DEL CURSO ENARM CMN SIGLO XXI
CURSO ENARM CMN SIGLO XXI TEL: 36246001 Pharmed Solutions Institute PGINA 312

de tocolticos o de progesterona, en este tipo de tratamiento no ha demostrado, ningn tipo de eficacia, por lo que a excepcin del
reposo, hoy da no se utiliza. Tratamiento quirrgico, es de eleccin y consiste en el cerclaje. Hay numerosas tcnicas de cerclaje pero
las utilizadas casi exclusivamente son: la de Mc Donald y Shirodkar, ambas consisten en hacer un lazo alrededor del cuello por va
transvaginal y el perodo ms apropiado para su realizacin es entre las semanas 13 y 16 del embarazo. Excepcionalmente se preconiza
la realizacin del cerclaje por va transabdominal. Esta tcnica se realiza en el quirfano, ya que se requiere el uso de anestesia general,
buenas condiciones de asepsia y control ecogrfico para valorar la correcta realizacin del cerclaje. El cerclaje cervical se debe realizar si
la gestacin es inferior a 20 semanas y el diagnstico est confirmado mediante ecografa. Si la gestacin es superior a 26 semanas, el
cerclaje no estara indicado en ningn caso. La actitud correcta que se debe adoptar es estar en reposo absoluto, llevar a cabo un
control exhaustivo del embarazo y valorar toclisis mientras se realiza maduracin pulmonar fetal con betametasona. El cerclaje puede
ser utilizado en tres circunstancias clnicas: cerclaje en casos diagnosticados de insuficiencia stmico-cervical, el cerclaje en caso de IC
oculta (cuando los datos ecogrficos se alteran haciendo temer el parto pretrmino) y, por ltimo, lo que se denomina cerclaje de
emergencia que es aquel que se realiza cuando ya existe dilatacin e incluso prolapso de las membranas en vagina. Cuando se practica
el cerclaje entre las semanas 12 y 18 cuando an no existe dilatacin, se logra un feto viable del 70 al 80% de los casos, los resultados
suelen ser mejores cuando ms precozmente se realiza. El cerclaje est contraindicado en las malformaciones fetales graves, cuando
existen contracciones, en los casos de coriamnioitis y es muy discutible cuando hay rotura de las membranas. Existen algunas
publicaciones de abortos tardos de uno de los gemelos en la que se ha realizado cerclaje logrando prolongar la gestacin hasta la
viabilidad del feto. Estos casos son excepcionales y la indicacin muy discutible.

CASO CLINICO
Se trata de paciente femenino de 25 aos de edad la cual tiene 21 semanas de gestacin actualmente bajo control prenatal, refiere
presentar clicos tipo menstrual niega sangrado transvaginal, sin embargo el dolo se extiende a las piernas y regin sacra, que
disminuye al reposo, se realizo USG con diagnostico de embarazo de 22 SDG, al tacto se percibe remblandecimiento de cuello uterino,
no se observa dilatacin cervical, por lo que se enva a domicilio con indicacin de reposo absoluto, a la semana vuelve la paciente a
revisin, donde refiere presencia de leve sangrado transvaginal y clicos de las mismas caracterstica, se realiza valores de
gonadotrofina con datos normales, al tacto se observa un cm de dilatacin y remblandecimiento de cuello cervical.

PREGUNTAS
Considerando el cuadro clnico y considerando las complicaciones potenciales durante el embarazo, que procedimiento es conveniente
realizar.

RESPUESTA
a.- Indica antibitico profilctico.
b.- Envio a segundo nivel para cerclaje cervical.
c.- USG para establecer la viabilidad fetal.
d.- Evaluacin de fibronectina fetal.

CASO CLINICO
Femenino de 31 aos de edad la cual acude a consulta para control prenatal, refiere 4 semanas de retraso menstrual, muestra un
reporte de laboratorio con PIE positivo, se encuentra ansiosa debido a que ha presentado 3 abortos espontneos, fue enviada a GO
para ser diagnosticada, es referida a la consulta de control prenatal en medicina familiar con diagnostico de incompetencia cervical y
con tratamiento de cerclaje cervical.

PREGUNTA
Con el actual tratamiento, cual es la probabilidad de llegar a un embarazo a trmino con producto vivo.

RESPUESTA
a.- 70 a 80 %
b.- 60 a 70 %
c.- 80 a 90 %
d.- 50 a 60 %.

HIPEREMESIS GRAVIDICA. CIENCIAS BASICAS: Las nuseas y vmitos son una situacin clnica muy frecuente en el embarazo que
afecta al 75-80 % de las gestantes (1-2 vmitos al da con tolerancia correcta a la ingesta, que inician sus sntomas entre la semana 6 y
la 9 de gestacin). La hiperemesis gravdica representa el espectro ms grave de una situacin clnica que aparece de forma gradual.
Son pacientes que han ido empeorando clnicamente de forma progresiva hasta presentar vmitos persistentes con intolerancia parcial
o total a la ingesta y con prdida ponderal superior al 5%. SALUD PUBLICA: El espectro ms grave de esta situacin clnica conocido
como hiperemesis gravdica es mucho menos frecuente y afecta en realidad al 0,5-2 % de todas las gestaciones. No existe consenso
acerca de la definicin de la enfermedad, pero la mayora de las definiciones incluyen vmitos persistentes en ausencia de otras
patologas que los expliquen con prdida ponderal importante (superior al 5% del peso inicial). PATOGENIA: La etiologa de la
hiperemesis gravdica es desconocida, aunque diferentes causas han sido propuestas: Niveles elevados de Beta-hCG y estrgenos, la
masa placentaria (las mujeres con embarazos mltiples o enfermedad trofoblstica presentan con mayor frecuencia sintomatologa
de nuseas y vmitos), factores psicolgicos y factores familiares (mujeres que tienen madres o hermanas que hayan presentado
nuseas y vmitos tienen ms probabilidad de presentarlas). La hieremesis parce relacionarse con concentraciones sricas altas o en
aumento rpido de gonadotropina corionica, estrgenos o ambos. Se ha demostrado que el receptor hCG/LH el cual se encuentra
presente en las clulas del cuerpo lteo del ovario produciendo el estmulo de la progesterona sobre la decidua para la comunicacin
inicial entre el blastocisto y el endometrio, adems se encuentra presente en distintas reas del cerebro como en el hipocampo y tallo
MANUAL DE TRABAJO DEL CURSO ENARM CMN SIGLO XXI
CURSO ENARM CMN SIGLO XXI TEL: 36246001 Pharmed Solutions Institute PGINA 313

cerebral lo cual explica la hipermesis gravdica. Tambin se ha postulado que la HCG causa hipermesis gravdica estimulando la va
secretora del tracto gastrointestinal superior. Se ha informado que las mujeres con enfermedad grave tienen aumento 1.5 veces de la
probabilidad de tener un feto del sexo femenino, lo que apoya la hiptesis de los estrgenos (retraso del vaciamiento gstrico y
enlentecimiento de la motilidad gastrointestinal). DIAGNOSTICO: Es muchas veces un diagnstico de exclusin y bsicamente clnico: 1.
Paciente que explica varios vmitos al da en relacin o no con las comidas. Intolerancia total o parcial a la ingesta. 2. Ausencia de otra
sintomatologa: No fiebre, no dolor abdominal, no cefalea, no alteraciones neurolgicas. No todas las pacientes que vomitan al inicio de
la gestacin presentan una hiperemesis 3. Alteraciones analticas: Hemograma (hemoconcentracin), Alteraciones electrolticas
(hiponatremia, hipopotasemia, hipocloremia y alcalosis metablica), alteracin del perfil heptico (elevacin leve de GOT y GPT <
300U/L, y Bilirrubina < 4 mg/dl) y alteracin del perfil tiroideo (aumento de la T4 y la TSH). Ecografa: Las pacientes con embarazos
mltiples o enfermedad trofoblstica presentan vmitos con mayor frecuencia. PREVENCION: Existen diferentes estrategias de
prevencin de las nuseas y los vmitos del embarazo. 1. La toma de complejos multivitamnicos de manera periconcepcional han
demostrado disminuir la incidencia de nuseas y vmitos del embarazo. As que parece razonable recomendar su administracin
sobretodo en aquellas pacientes que han presentado nuseas y vmitos en gestaciones anteriores. 2. Comidas frecuentes, poco
abundantes (repartir la ingesta en un mnimo de 5 comidas de menor cantidad), slidas y evitar las comidas muy grasas y picantes.
TRATAMIENTO: Farmacolgico: Puesto que la aparicin de la hiperemesis gravdica se produce de manera escalonada en la mayora de
las pacientes, la manera de tratarlas es ir aadiendo los frmacos que se refieren a continuacin en el orden en el que se citan, a
medida que la sintomatologa vaya empeorando. 1. Doxilamina 10 mg + piridoxina 10 mg: La dosis habitual es un comprimido cada 6-8
horas, aunque se puede ajustar la dosis en funcin de la sintomatologa hasta alcanzar una dosis mxima de 70 mg/d. Hay que tener en
cuenta que ante la persistencia de sintomatologa en una franja horaria determinada la dosis a aumentar es la inmediatamente anterior
(p. Ej. si persisten las nuseas y vmitos matutinos hay que aumentar la dosis de la noche). Importante tener en cuenta la somnolencia
como efecto secundario de la doxilamina. 2. Aadir Dimenhidrinato 50-100 mg/4-6 horas: La va de administracin puede ser oral o
rectal. Dosis mxima 400 mg /d. 3. Aadir Metoclopramida 5-10 mg/8 horas: La va de administracin es oral en comprimidos o
suspensin. Este frmaco puede causar sintomatologa extrapiramidal (espasmos en cara, cuello y lengua). Cuando persiste la clnica a
pesar de la asociacin de 2 ms tratamientos y la paciente presenta intolerancia total a la ingesta, nos encontraremos ante el
espectro ms severo de la enfermedad y ser necesario un ingreso hospitalario para rehidratacin endovenosa. 4. Dieta famis y
fluidoterapia: Suero Glucosado 10 % 500 cc /8 horas alterno con Ringer Lactato o Suero Fisiolgico 500 cc/8-12 horas hasta corregir el
trastorno electroltico. En pacientes que requieren fluidoterapia y que presentan clnica de vmitos durante ms de 3 semanas se
recomienda aadir suplementos de tiamina (vitamina B1) intravenosa a dosis de 100 mg/d durante 2-3 das 1 vial de 100 mg/d. En
pacientes que presenten alteracin en el ionograma se asociar 10-20 mEq de ClK en cada suero glucosado durante 24-48 h o hasta
corregir el trastorno hidroelectroltico 5. Aadir Metoclopramida 5-10 mg/8 horas IV. 6. En casos resistentes a todos los tratamientos
mencionados o que requieren varios ingresos, adems se puede asociar: a. Ondansetrn 8 mg/12 h IV. b. Metilprednisolona 16 mg/8h
va oral o endovenosa durante 3 das, seguido de dosis decrecientes durante 15 das hasta desaparicin de los sntomas (Prednisona vo:
40 mg/da 1 da, 20 mg/da 3 das, 10 mg/da 3 das y 5 mg/da 7 das). COMPLICACIONES: La encefalopata de Wernicke es poco
frecuente pero se asocia a las principales causas de mortalidad en las pacientes, se presenta por la deficiencia de tiamina posterior a 3
semanas de vmitos persistentes caracterizada por la triada de anormalidades oculares, confusin y ataxia de la marcha se presenta en
el 46.9% de los casos.

CASO CLINICO
Paciente femenino de 24 aos de edad la cual acude a control prenatal, refiere que es su segunda consulta, presenta desde hace una
semana nausea intensas de predominio matutino, actualmente tiene 8 semanas de gestacin, ha vomitado hasta 3 veces al da, agrega
que en su embarazo anterior presento los mismo sntomas y luego de un realizarse un USG fue diagnosticada con embarazo molar
tratada sin complicaciones, y reporte de patologa negativo a proceso neoplasico.

PREGUNTA
Cul es la conducta a seguir, considerando sus antecedentes y cuadro actual,

RESPUESTA
a.- Considerando la edad gestacional puede indicarse piridoxina 50 a 100 mg al da.
b.- La modificacin de los hbitos alimenticios puede mantenerse adecuadamente.
c.- Solo requiere mantenerse en control continuo del estado de liquidos y electrolitos.
d.- Realizar a la brevedad un USG para descartar un nuevo embarazo molar.

MANUAL DE TRABAJO DEL CURSO ENARM CMN SIGLO XXI
CURSO ENARM CMN SIGLO XXI TEL: 36246001 Pharmed Solutions Institute PGINA 314


ENFERMEDAD TROFOBLASTICA GESTACIONAL (ETG). CIENCIAS BASICAS: Las anormalidades proliferativas del trofoblasto
(citotrofoblasto, sincitiotrofoblasto y trofoblasto intermedio) conocidas como Enfermedad Trofoblstica gestacional, describe un grupo
de patologas relacionadas con la fertiolizacion y se derivan de una proliferacin anormal del trofoblasto de la placenta humana
(hiperplasia) y del genoma paterno, con una contribucin materna ocasional, tendencia variable a la invasin local y a metstasis, y
cuyo denominador comn es la hipersecrecin de beta HCG y responden a la quimioterapia. La ETG comprende a la mola hidatiforme
(MH), mola completa y mola parcial, las cuales se consideran enfermedades benignas. La neoplasia trofoblastica gestacional (NTG)
incluye a la mola invasora, coriocarcinoma y tumor
del sitio placentario (TSP), los cuales se consideran
patologas malignas. SALUD PUBLICA: La MH se
presenta aproximadamente en 1 de cada 1000
embarazos en EUA y Europa; en Japn la incidencia
es 3 veces mayor. En Mxico es de 2.4 por cada
1000 embarazos. La incidencia de mola invasora
ocurre en 1 de cada 40 embarazos molares y en 1
de cada 150,000 embarazos normales. PATOGENIA:
Factores de riesgo y probabilidad de desarrollar
ETG: mola previa 50%, aborto previo 25%,
embarazo ectpico previo 5%, embarazo previo de
trmino 20%. En mujeres mayores de 40 y 50 aos,
la incidencia de NTG es de 40% y 50%
respectivamente. La ETG son alteraciones de la
gestacin, generadas por anormalidades en el desarrollo del tejido trofoblastico con caractersticas biolgicas y patolgicas
particulares, puesto que son lesiones en las cuales hay representacin del genoma paterno, lo que las diferencia de los tumores no
gestacionales. CORIOCARCINOMA: Tumor maligno del epitelio trofoblastico. Est compuesto por sincitio y citotrofoblasto anaplasico y
clulas gigantes multinucleadas, que invade y puede dar metstasis a sitios distantes. En el coriocarcinoma la invasin vascular ocurre
tempranamente resultando en metstasis hacia el pulmn, vagina, cerebro, rin, hgado y aparato gastrointestinal. MOLA
HIDATIFORME (MH): Se caracteriza histolgicamente por la presencia de vellosidades corinicas con grado variables de proliferacin
trofoblstica anormal y edema estromal (degeneracin hidrpica), adquiriendo la morfologa de bandas y cmulos de vesculas que
confieren el tpico aspecto de racimos de uvas. Dentro de los factores de riesgo se pueden mencionar: Los extremos de la vida
reproductiva y el antecedente de Mola previa. MOLA COMPLETA: Ausencia de tejido embrionario o fetal. Degeneracin hidrpica de
vellosidades coriales y son avasculares, hiperplasia trofoblastica difusa con atipia difusa y marcada del trofoblasto en el sitio de la
implantacin molar. El factor de riesgo ms asociado, es la edad, ya que mujeres mayores de 40 aos tienen un riesgo 5-10 veces
mayor, que mujeres jvenes. El sntoma y signo clnico ms importante en el embarazo molar es el sangrado o manchado vaginal, que
se presenta entre un 89 a 97% de las pacientes, esta patologa se debe sospechar cuando encontramos una Beta-hCG mayor de 100.000
UI en embarazos tempranos. Otro signo importante es una altura uterina mayor a la esperada para la edad gestacional, lo cual se
produce por una amplia proliferacin trofoblastica secundaria a un elevado nivel de Beta-hCG; esto ocurre en cerca del 50% de las
pacientes. La ausencia de actividad cardiaca fetal ocurre a pesar del tamao uterino francamente suprapbico. Los quistes
tecaluteinicos, estn presentes hasta en un 46% de los embarazos molares completos, y generalmente son mayores de 5 cm (6-12 cm),
pero pueden verse hasta de 20 cm de dimetro, suelen ser bilaterales, su contenido es serosanguinolento, y su aparicin se explica por
hiperestimulacin ovrica de la -hCG circulante. La hipermesis gravdica tambin es ms frecuente en estos embarazos, se ha
identificado hipertiroidismo (7%). MOLA INVASORA: (coriocracinoma destruens) La definen el crecimiento trofoblstico excesivo y su
capacidad invasora local, con una penetracin extensa por los elementos trofoblsticos, que incluyen vellosidades completas, en el
miometrio (sin involucrar el estroma endometrial) e incluso hasta el peritoneo parametrios y cpula vaginal adyacente. Las metstasis a
distancia son excepcionales. MOLA PARCIAL: Presencia de tejido embrionario o fetal. Hiperplasia trofoblastica focal, variabilidad
marcada en el tamao de las vellosidades con importante edema y cavitacin, presentando inclusiones prominentes en el estroma
trofoblastico de las vellosidades, presentndose ocasionalmente atipia focal y leve de trofoblasto en el sitio de implantacin. La
presentacin clnica del embarazo molar parcial es similar a la de un aborto incompleto dado por sangrado genital y dolor tipo clico en
hipogastrio de hecho, el diagnstico, en su gran mayora, se realiza en el estudio histopatolgico. TUMOR DEL SITIO PLACENTARIO: Es
un tumor extremadamente raro. Se origina en el sitio de implantacin de la placenta y deriva de clulas de trofoblasto intermedio de la
placenta, las cuales se identifican por la secrecin de lactogeno placentario y pequeas cantidades de fraccin beta de HCG. No
contiene vellosidades coriales. DIAGNOSTICO: Los sntomas son ms evidentes en la mola completa, y se presentan entre el primero y
el segundo trimestre. Los sntomas clsicos de la mola completa son: la hemorragia vaginal (97%), el aumento del tamao uterino de
manera ms rpida del habitual (50%); la hipertensin arterial en embarazos menores de 24 semanas (27% de las pacientes con mola
completa), embolismo pulmonar (2% en pacientes con mola completa), tirotoxicosis (2-7%). La mola hidatiforme parcial no produce el
cuadro clnico espectacular de la mola completa, generalmente se presentan los sntomas de aborto (amenaza de aborto o aborto
incompleto), y se puede establecer el diagnostico despus del estudio histopatolgico del material obtenido. Las caractersticas
histopatolgicas permiten hacer el diagnstico diferencial de la diversidad de las presentaciones de ETG. Cuando exista sangrado
uterino anormal por ms de 6 semanas posteriores a cualquier embrazo, descartar ETG. Luego de la sospecha clnica el examen de
primera lnea es la ecografa. La mola completa tiene un aspecto ecogrfico vesicular caracterstico en lluvia de nieve. En el embarazo
molar parcial se puede ver como una imagen indistinguible de un aborto incompleto, aunque se pueden observar espacios qusticos
focales en el tejido placentario y aumento del dimetro transversal del saco gestacional; si se encuentran los 2 hallazgos el valor
predictivo positivo para la mola parcial es de 90%. La ecografa asociada a la dosificacin de BhCG, cuando esta es mayor de 82350
mUI/ml y no existe actividad cardiaca visible en el embrin da una probabilidad de estar frente a una mola hidatiforme del 89%. La
medicin de los niveles sricos de BhCG con valores mayores de lo esperado para la edad gestacional son fundamentales para el
diagnstico. La hiperemesis gravdica y la aparicin de preeclampsia temprana son otros de los cuadros clnicos que pueden
MANUAL DE TRABAJO DEL CURSO ENARM CMN SIGLO XXI
CURSO ENARM CMN SIGLO XXI TEL: 36246001 Pharmed Solutions Institute PGINA 315

presentarse. En la evaluacin inicial se debe hacer bsqueda rpida de enfermedad metastsica, solicitando RX de trax y pruebas de
funcin heptica y renal. TRATAMIENTO: El tratamiento de la MH tiene dos aspectos: evacuacin y seguimiento. La evacuacin
inmediata de la mola preferiblemente se debe realizar mediante aspiracin, previa maduracin cervical con misoprostol en caso de
encontrarse un crvix cerrado. La evacuacin debe realizarse en instituciones de segundo o tercer nivel de atencin, lo de eleccin es el
AMEO, o histerectoma en bloque, con reserva de sangre, uso de oxitcicos en caso de ser necesario, en casos especiales evaluacin
ecogrfica transoperatoria y aplicacin de inmunoglobulina humana antiD si es Rh negativo. La quimioterapia profilctica en MH ha
sido controversial, est indicada en aquellas mujeres que tienen molas de alto riesgo. La histerectoma puede ser una opcin preferible
al curetaje por aspiracin en caso de mujeres con paridad satisfecha y en especial en mujeres mayores de 40 aos o ms, dada la mayor
frecuencia de enfermedad trofoblstica maligna en este grupo etareo. Aunque la histerectoma no elimina del todo la posibilidad de
metstasis, reduce apreciablemente la enfermedad recurrente. Los quistes tecalutenicos grandes y sintomticos se pueden aspirar con
gua ecogrfica. Una alternativa del AMEO es el LUI, considerando el mayor riesgo de perforacin uterina. En MH con altura uterina
similar a 16SDG o mayor existe riesgo de embolizacion pulmonar. Para descartar el desarrollo de NTG, las pacientes se siguen luego de
la evacuacin molar con niveles de -hCG semanales, el seguimiento debe realizarse cada 2 semanas con examen clnico completo y
mediciones seriadas de BhCG hasta tener 2 mediciones seguidas negativas; luego cada 3 meses durante un ao. En la enfermedad
trofoblstica benigna la regresin de la BhCG ocurre entre los das 11 y 229, con un promedio de 74 das. Encontrar niveles sricos de
25.000 mUI/ml a la cuarta semana es signo de alta acividad trofoblstica.

CASO CLINICO
Acude a la consulta femenino de 36 aos de edad a solicitar mtodo de planificacin familiar ya que tiene 2 aos de su ultimo parto y
no se ha cuidado debido a que su esposo se encuentra trabajando en estados unidos y va a regresar pronto, la paciente es originaria de
Michoacn y tiene estudios de primaria, dentro de sus antecedentes cuenta con historia de tabaquismo positivo, ingesta de alcohol
ocasional, presento menarca a los 10 aos, gesta 4, para 3, abortos 1, refiere que el ultimo embarazo fue hace 4 aos, le realizaron un
legrado porque no se haba formado el producto y presento sangrados al inicio con salida de tejido como burbujas, le indicaron volver a
consulta y ya no regreso, ahora ya no quiere otro embarazo. A la exploracin fsica la paciente solo se observa con obesidad y
levemente desalineada.

PREGUNTA
Considerando que la paciente presento con mucha probabilidad una enfermedad trafoblastica, que mtodo de planificacin es el ms
adecuado.

RESPUESTA
a.- Inyectable combinado mensual.
b.- Oral con microdosis.
c.- Mtodo de barrera.
d.- Obstruccin tubarica bilateral.

POLIDRAMNIOS. CIENCIAS BASICAS: Se define como un volumen de lquido amnitico mayor a 1500-2000ml, secundario a un aumento
en la produccin o deficiencia en la eliminacion. Segn el ndice de Phelan, definimos polihidramnios con un ndice de lquido amnitico
>25 cm (8-18 normalidad ecogrfica del ILA, 18-25 valores lmite superior, >25 polihidramnios). El lquido amnitico (LA) es el fluido que
ocupa la cavidad amnitica y que contribuye en importantes funciones para un adecuado desarrollo del embarazo. Sirve de proteccin
para el feto frente a traumatismos, mantiene una temperatura adecuada, permite el desarrollo de determinados rganos vitales y
puede aportar informacin acerca del estado y madurez fetal. Su volumen vara a lo largo de la gestacin. Aumenta de 50 ml en la
semana 12 a 400 ml en la semana 20. Hacia la 38 semana puede alcanzar valores de 1000 ml y al trmino es aproximadamente de 800
ml, oscilando entre 300 y 1500 ml. El LA renueva a un ritmo de 500 ml/hora y que en un perodo de 2-3 horas todo el contenido de agua
del LA se ha renovado. Se ha visto que la cuarta parte de esta circulacin se realiza a travs del feto y el cordn umbilical; y el resto, a
travs de las membranas ovulares y superficie placentaria. El intercambio de LA a travs del feto puede realizarse por las siguientes
vas: aparato digestivo, respiratorio, urinario y piel. Se calcula que el feto puede deglutir de 5 a 7 ml hora, cantidad escasa en
proporcin a la circulacin total de lquido amnitico. SALUD PUBLICA: Incidencia oscila entre 0.13-3.2% de los embarazos. Se asocia
con una alta mortalidad perinatal (60%). PATOGENIA: Se puede deber a causas Idioptico (65%), maternas (7%): DM (en estos casos, el
polihidramnios se atribuye a diversas causas, como un aumento de la osmolaridad del LA debido a una elevacin de la glucosa, una
disminucin de la deglucin fetal y poliuria fetal por la hiperglucemia), isoinmunizacin al grupo y Rh) como fetales (13%): del tracto
gastrointestinal (atresia esofgica, atresia duodenal, agnatia, fisura palatina, labio leporino, artrogriposis mandibular, obstruccin
intestinal, hernia diafragmtica, pncreas anular, gastrosquisis, peritonitis meconial, onfalocele), defectos del tubo neural,
enfermedades cardiovasculares fetales, patologa del tracto urinario(rin poliqustico, obstruccin ureteroplvica), embarazo mltiple,
hidropesa fetal inmune y no inmune) y por ultimo causas placentarias (corioangioma placentario: el tumor benigno ms frecuente en la
placenta). CLASIFACION: Segn la severidad se clasifica de acuerdo al ILA en LEVE; cuando el ndice de lquido amnitico (ILA) se
encuentra entre 25-30cm. MODERADO; se encuentra entre 30.1-35cm. GRAVE; por arriba de 35 cm. Por su evolucin se clasifica en
agudo (2%), de inicio sbito, aparece en el segundo trimestre, se asocia a malformaciones congnitas no compatibles con la vida y
rpidamente evoluciona a parto pretrmino. Crnico (98%), aparece en el tercer trimestre, se asocia a factores maternos y es
principalmente idioptico. DIAGNOSTICO: La sintomatologa generalmente es secundaria a la sobresdistensin abdominal y uterina, y
dependiendo del grado de extensin puede ir desde fatiga leve, hasta la presencia de disnea, cianosis y edema (miembros inferiores,
vulvar y de pared abdominal), aumento de peso, desarrollo de preeclampsia, as como trabajo de parto. Se sospecha ante la presencia
de un fondo uterino mayor al esperado, con abdomen a tensin y dificultad para palpar partes fetales, dificultad para auscultar foco
fetal y palpacin subjetiva de aumento de lquido amnitico. El USG nos brinda el diagnostico con la medicin de lquido amnitico, la
tcnica ms utilizada es la de Phelan. La medida de la cantidad del lquido amnitico se puede realizar utilizando diferentes tcnicas, las
dos ms utilizadas son la mxima columna vertical y el ndice de lquido amnitico (ILA). Mxima columna vertical: Se realiza midiendo
MANUAL DE TRABAJO DEL CURSO ENARM CMN SIGLO XXI
CURSO ENARM CMN SIGLO XXI TEL: 36246001 Pharmed Solutions Institute PGINA 316

la mxima columna vertical de lquido libre de partes fetales y de cordn de manera vertical. Es el mtodo de eleccin en gestaciones
mltiples y en gestaciones nicas de menos de 24 semanas. Se considera normal cuando es superior a 2 cm en todas las edades
gestacionales y cuando es inferior a 8 cm por debajo de la semana 20 o a 10 cm a partir de la semana 21. ILA: Es el valor obtenido a
partir de la suma de las mximas columnas verticales de lquido, libre de partes fetales o cordn umbilical, en cada uno de los cuatro
cuadrantes que se delimitan por la interseccin de dos lneas perpendiculares en el abdomen materno: la lnea media longitudinal con
la lnea transversal media entre la snfisis pbica y el fondo uterino. El transductor se coloca en posicin sagital y lo ms perpendicular
posible al suelo. Se consideran normales valores de ILA entre 5 y 25 centmetros. La medida del ILA ha demostrado tener una superior
reproducibilidad y mejor deteccin de patologa por lo que se considera tcnica de eleccin en la medida de la cantidad de lquido
amnitico en las gestaciones nicas de ms de 24 semanas. Tambin el USG puede detectar malformaciones fetales. Los exmenes de
laboratorio son importantes para poder determinar una causa no idioptica; se debe tomar en cuenta la curva de tolerancia a la
glucosa, los anticuerpos anti toxoplasma, CMV, tipo sanguneo y Rh y el cariotipo fetal analizado a travs del lquido amnitico.
TRATAMIENTO: El manejo depende dela etiologa, la edad gestacional y la intensidad, llegando a requerirse amniocentensis
evacuadora en casos con compromiso materno. El medicamento de eleccin es la indometacina su uso se limita a partir de 28-32 SDG,
es un inhibidor de las prostaglandinas vasodilatadoras, por lo cual produce una vasoconstriccin arteriolar en el rin fetal
disminuyendo el filtrado glomerular con la subsecuente disminucin de la produccin de orina fetal que es la principal fuente de
lquido amnitico a partir del segundo trimestre. La dosis usada vara entre 1,5 y 3 mg/Kg por da, siendo la ms usada de 25 mg cada
seis horas. Su empleo debe ser controlado por el riesgo de cierre del conducto arterioso fetal. La va de interrupcin depender de las
condiciones obsttricas y fetales. Debe monitorizarse el ILA bisemanalmente de tal modo que cuando llegue a 18cm se suspende.
Tambin es importante realizar ecocardiograma fetal cada semana, para observar el agujero oval, en caso de cierre de este debe
suspenderse la indometacina ya que el proceso generalmente es irreversible. En pacientes con polihidramnios esta incrementado el
riesgo de parto pretrmino, rotura prematura de membranas, prolapso de cordn, presentaciones distosicas, rotura uterina, atona
uterina y desprendimiento prematuro de placenta normoinserta. La amniocentesis es un procedimiento invasivo, que alivia las
molestias maternas, provee lquido que puede estudiarse (cariotipo, madurez pulmonar, gramm y cultivo), y al disminuir la compresin
uterina, mejora el flujo de las arterias espirales. Las complicaciones de la amniocentesis pueden ser parto pretrmino, DPPN, puncin
del feto e infecciones que son raras. El tratamiento depender de la severidad del caso. En casos leves puede utilizarse nicamente la
indometacina. Cuando es moderado a severo, puede utilizarse la indometacina junto con la amniocentesis.

CASO CLINICO
Una mujer de 20 aos de edad, gesta 1 para 0, con 28 SDG con un alto riesgo debido a la presencia de un patrn de crecimiento fetal
discordante y embarazo gemelar. A la exploracin fsica se observo altura del fondo uterino de 35 cm, palidez de tegumentos, edema
de miembros inferiores, TA 90/70 mmHg, el orificio externo del tero cerrado, sin secresin vaginal anormal.

PREGUNTA.
Cul es su conducta a seguir.

RESPUESTA
a.- Envio a segundo nivel.
b.- Realizar preparativos para cesaria.
c.- Indica datos de alarma y continua ambulatoria.
d.- Suplementacin energtica y proteica.

CASO CLINICO
Femenino de 24 aos de edad, gesta 3, para 2, actualmente con 28 SDG por USG que fue realizado hace 6, la paciente refiere que se
incremento el tamao del abdomen. Su embarazo haba sido sin incidentes. No hay otras pruebas anormales durante embarazo.
Durante una visita para un chequeo de rutina 6 semana antes, una ecografa revel amnitico normal, la ecografa al ingreso revel
actual polihidramnios grave, alcanzando un ndice de lquido amnitico de 40 cm. Un examen Doppler del cordn umbilical
descubiertos bucles de cordn mltiples envuelto alrededor del cuello fetal.

PREGUNTA.
Cul es su conducta a seguir

RESPUESTA
a.- Incrementar el nmero de visitas.
b.- Envio a segundo nivel.
c.- Realizar prueba de grupo y Rh.
d.- Realizar BH, VDRL.

OLIGODRAMNIOS. CIENCIAS BASICAS: Se define como un volumen de lquido amnitico menor a 500ml. Presencia de un ILA inferior a
5 o de una mxima columna vertical inferior a 2 cm. Se define como oligohidramnios severo/anhidramnios la presencia de una columna
mxima 1 cm. SALUD PUBLICA: Se reporta una incidencia de 5-37%. En el segundo trimestre de la gestacin la presencia de una RPM
explica el 50% de los casos de oligohidramnios, seguido por el RCIU y las malformaciones fetales en el 20% y el 15% respectivamente,
siendo un 5% de los casos idiopticos. PATOGENIA: Podemos dividir las causas de Oligohidramnios en tres grandes grupos: Causas
fetales: Crecimiento intrauterino restringido (CIR), gestacin cronolgicamente prolongada (GCP), infeccin fetal por CMV, obstruccin
tracto urinario (ureteral bilateral, valvas uretrales posteriores), patologa renal (agenesia renal bilateral, displasia renal multiqustica
bilateral, riones poliqusticos) y defectos del tubo neural. Causas placentarias-membranas: rotura prematura de membranas (RPM).
Causas maternas: medicacin materna (inhibidores de la sntesis de prostaglandinas, IECA). DIAGNOSTICO: Datos clnicos sugestivos:
MANUAL DE TRABAJO DEL CURSO ENARM CMN SIGLO XXI
CURSO ENARM CMN SIGLO XXI TEL: 36246001 Pharmed Solutions Institute PGINA 317

Disminucin de la motilidad fetal, fondo uterino menor al esperado para la edad gestacional, copn fcil palpacin de partes fetales,
memebranas aplandas a la exploracin vaginal y con alteraciones en la fecuencia cradiaca fetal. Datos paraclnicos: El mtodo de mayor
sensibilidad es sin duda la ultrasonografa, ndice de lquido amnitico con tcnica de Phelan, un ndice menor de 5 se considera
oligohidramnios. Ante el diagnstico de oligohidramnios debemos realizar las siguientes pruebas diagnsticas: 1. Descartar RPM:
Mediante anamnesis y exploracin. Realizar PROM test si no existe hidrorrea franca. Si existe historia clnica sugestiva y el PROM test es
negativo valorar la posibilidad de instilacin de fluorescena intraamnitica mediante amniocentesis. 2. Descartar CIR: Valoracin del
peso fetal estimado as como realizacin de estudio Doppler. 3. Descartar malformaciones fetales: Estudio morfolgico dirigido a
descartar la presencia de malformaciones nefro-urolgicas y del tubo neural. 4. Descartar la infeccin fetal por CMV: serologas
maternas (IgG/IgM), marcadores fetales ecogrficos (microcefalia, ventriculomegalia, focos parenquimatosos hiperecognicos,
hiperrefringencia intestinal). 5. Descartar toma de frmacos: inhibidores de la sntesis de prostaglandinas y IECAs. TRATAMIENTO: No
existe tratamiento especfico para esta complicacin por lo que la asistencia se basa en la vigilancia y teraputica de la causa que lo
origin y la edad gestacional en la que se encuentra. El manejo clnico del Oligohidramnios depende principalmente de la causa del
mismo: A) En aquellos casos en los que se diagnostique una RPM o un CIR se aplicara el protocolo especfico de cada patologa. B) En el
caso de toma de frmacos se interrumpir la toma de los mismos de forma inmediata. Si la paciente ha consumido inhibidores de la
sntesis de prostaglandinas se realizar valoracin del ductus arterioso. Si existiera una restriccin (IP<1 o insuficiencia tricuspdea
significativa (holosistlica, 150 cm/s)) se realizara control cada 48 h hasta su normalizacin. C) En aquellos casos en los que el feto
presente una malformacin, se informar del pronstico de la misma y del riesgo de hipoplasia pulmonar y en funcin de esta
informacin los padres podran acogerse a la interrupcin legal del embrazo. Si los padres deciden seguir adelante con la gestacin
debemos realizar amniocentesis/cordocentesis para estudio de cariotipo y valorar el estudio de la funcin renal en orina fetal. D) La
evidencia disponible indica que las pacientes con oligohidramnios idioptico no presentan peores resultados neonatales en
comparacin con la poblacin con lquido amnitico normal. Manejo anteparto: en casos menores de 34 SDG se deben emplear
esteroides para maduracin pulmonar y pruebas de biestar fetal. Realizacin de Perfil biofsico y estudio Doppler, semanal hasta las
36.6 semanas y a partir de la semana 37.0 cada 72 horas. Estimacin de peso fetal cada dos semanas. Finalizacin de la gestacin: Se
mantendr una conducta expectante hasta las 40 semanas de gestacin si el control de bienestar fetal es normal. Por encima de las 37
semanas ante condiciones cervicales favorables valorar la finalizacin de la gestacin. No existe contraindicacin para el uso de
prostaglandinas. Manejo del parto: Se debe realizar monitorizacin continua. Realizar amnioinfusin.

CASO CLINICO
Femenino de 28 aos, gesta 3, para 1, cesarea 1, acude a consulta a las 34 SDG. Presenta un USG realizado a las 24 SDG, con peso fetal
aproximado de (650 g) apropiados para la edad gestacional. Despus del examen, el paciente no recibi atencin prenatal durante dos
meses. Se realiz nuevamente un USG donde observ restriccin del crecimiento intrauterino 1,440 g, con bajo nivel de liquido
amnitico, temperatura 36,0 grados, la frecuencia del pulso 84/min, frecuencia respiratoria 20/min y la presin arterial 120/80 mmHg.

PREGUNTA.
Cul de las siguientes observaciones es falsa respecto a esta patologa.

RESPUESTA
a.- Constituye un elemento accesorio del feto
b.- Depende del desarrollo, crecimiento y maduracin.
c.- Su volumen vara fisiolgicamente.
d.- El apgar depende del volumen del liquido amnitico.

MANUAL DE TRABAJO DEL CURSO ENARM CMN SIGLO XXI
CURSO ENARM CMN SIGLO XXI TEL: 36246001 Pharmed Solutions Institute PGINA 318

PLACENTA PREVIA (PP). CIENCIAS BASICAS: Se define como aqulla implantada en el segmento inferior del tero. Pero una definicin
actual establece que es la implantacin en el segmento inferior del tero, ocluyendo a veces el orificio cervical interno (OCI). Las
hemorragias del embarazo son causales de importante morbimortalidad. Se las divide en aquellas que afectan la primera mitad y
segunda mitad de ste. Entre las ltimas destacan la placenta previa (PP), el desprendimiento prematuro de placenta normoinserta
(DPPNI) y rotura uterina. En el puerperio se agregan el acretismo, y la inercia uterina. La hemorragia del post parto por anomalas de la
insercin placentaria es la principal indicacin de histerectoma (HT) obsttrica. SALUD
PUBLICA: La hemorragia obsttrica es la causa ms importante de mortalidad materna en
los pases desarrollados. Su incidencia es de 3,8/1.000 embarazos (rango: 1,4 a 7/1.000).
Para nulpara, la incidencia es de 0.2%, mientras que en multparas, puede ser mayor a 5%
y la tasa de recidiva es de 4% a 8%. El factor de riesgo ms importante para placenta previa
es tener una cesrea previa. La placenta previa ocurre en el 1% de las embarazadas
despus de una cesrea. La incidencia despus de 4 o ms cesreas se incrementa a 10%
Se asocia a riesgo significativo de hemorragia, hospitalizacin, transfusin, parto
prematuro, alta frecuencia de cesrea, y de HT post cesrea. La mortalidad perinatal (MPN)
est aumentada 3 a 4 veces, dada principalmente por parto prematuro. CLASIFICACION:
Basada en la localizacin relativa de la placenta en el OCI: 1) COMPLETA O TOTAL: cuando
la placenta cubre enteramente el OCI. 2) PARCIAL: cuando la placenta cubre parte pero no
todo el OCI. Es la ms peligrosa. 3) MARGINAL: cuando el borde de la placenta est en
contacto con el OCI, pero sin cubrirlo. 4) DE INSERCIN BAJA: cuando la placenta est
localizada cerca (a 3 cm.), pero no est en contacto con el OCI. PATOGENIA: El segmento
inferior es una regin inadecuada para la insercin placentaria, por presentar: a. Endometrio: de menor grosor que determina una
decidua ms delgada y con menor vasculatura, por lo que la placenta tiende a ser ms extendida, aplanada e irregular, con escaso
desarrollo de tabiques entre cotiledones. Debido a lo anterior el trofoblasto puede invadir decidua, pared uterina (ms delgada) y an
rganos vecinos (acretismo placentario). La insercin
placentaria en el segmento inferior permite que acte como
tumor previo. b. Musculatura: menos fibras musculares en
relacin al segmento superior y con mayor cantidad de fibras
colgenas, lo que lo hace distensible, pero con menos
potencia para colapsar vasos sanguneos, dificultando la
hemostasia si hay desprendimiento parcial de la placenta y
por supuesto, durante el alumbramiento. c. Membranas: en
el borde placentario son ms gruesas y menos elsticas,
existe mayor frecuencia de RPM. d. Cordn: Por la atrofia de
cotiledones, secundario al desarrollo insuficiente de decidua,
es frecuente la insercin velamentosa del cordn. La
placenta previa sangra slo si se desprende. Las causas de
desinsercin son: En el embarazo: Por el crecimiento uterino
y por las contracciones de Braxton-Hicks, se forma y
desarrolla el Segmento uterino Inferior, cuya capacidad de
elongacin es mayor y supera al de la placenta; En el parto:
Por las contracciones del trabajo de Parto que causan la
formacin del canal cervico-segmentario y la dilatacin
cervical. DIAGNOSTICO: En la Placenta Previa, la hemorragia
genital se caracteriza por ser indolora, con expulsin de
sangre liquida, roja rutilante, que aparece en forma brusca e
inesperada, la mayora de las veces en reposo e incluso
durante el sueo. El 35% de las pacientes con placenta previa presentan el primer episodio de hemorragia genital antes de la semana
30, 33% entre las semanas 30 y 35, y 32% despus de la semana 36. El promedio de edad estacional en el que aparece el primer
episodio de hemorragia es de 29, no se asocia con mortalidad materna y se resuelve espontneamente. A veces es posible escuchar el
soplo placentario bajo. El sangrado genital est presente en el 80% de los casos; hemorragia + dinmica uterina en un 10-20% y un 10%
son asintomticas, detectadas slo por ecografa. En el examen fsico, la anemia materna se correlaciona con la cuanta de la
metrorragia. A la palpacin, el tero presenta consistencia normal. Si existe trabajo de parto, la dinmica es normal. El dolor, es el
habitual durante las contracciones. Como ya dijimos, la Placenta Previa acta en la cavidad uterina como tumor previo, por lo que las
presentaciones altas son frecuentes, al igual que las presentaciones de tronco, nalgas (30%) y las posteriores (15%). A la inversa, una
presentacin de vrtice, con buen apoyo en la pelvis, nos debe hacer pensar en otra causa de metrorragia. La fetocardia es normal. El
compromiso fetal depende del compromiso materno, que junto al ritmo sinusoidal en el monitoreo fetal indicara que el feto tambin
ha sangrado al desprenderse la placenta. Es norma en todo estudio ultrasonogrfico informar la localizacin de la placenta y su relacin
con el orificio cervical interno (OCI). Actualmente el diagnstico de la mayora de las PP es realizado mediante ultrasonografa rutinaria
del segundo trimestre, por va transvaginal (TV). El USG TV en cualquier edad gestacional debe considerarse el Gold Standard para
diagnosticar PP, ya que la va transabdominal (TA) presenta inconvenientes en la correcta localizacin placentaria, describindose hasta
60% de relocalizacin por va TV. La va TV tiene una sensibilidad de 87,5%, especificidad de 98,8%, y un valor predictivo positivo de
93,3%, y valor predictivo negativo de 97,6%. Adems, la va TV ha demostrado ser segura en presencia de metrorragia estable, al no
agravar el sangrado vaginal. TRATAMIENTO: El manejo mdico de la placenta previa sangrante es hospitalario, se debe efectuar
evaluacin completa de los factores etiolgicos que favorecen la presencia del sangrado: infecciones cervicales, infecciones urinarias o
vaginales, sobredistensin uterina, factores precipitantes de la actividad uterina de pretrmino. Los exmenes paraclnicos incluirn:
hemograma, hemoclasificacin, gram y cultivo de secrecin vaginal, parcial de orina (urianlisis), urocultivo, VDRL, ecografa
MANUAL DE TRABAJO DEL CURSO ENARM CMN SIGLO XXI
CURSO ENARM CMN SIGLO XXI TEL: 36246001 Pharmed Solutions Institute PGINA 319

transabdominal y/o transvaginal para confirmar el diagnstico y evaluar la biometra fetal, as como determinar el peso fetal; en caso
de que se considere muy posible la terminacin del embarazo, monitora fetal electrnica bisemanal. Tratamiento quirurgico o cesarea:
La manera de terminar el embarazo esta basado en el juzgamiento clnico sumada la exploracin ecografca. Una placenta con un borde
placentario a menos de 2 cm del orificio cervical externo es probable que requiera una cesrea, especialmente si esta es posterior.
Recomendacin B. Recientemente un estudio prospectivo observacional que incluy 63 pacientes con placenta previa , se demostr
que en todas a las que se le dio parto vaginal , la distancia del borde placentario al orificio cervical interno fue de 2 cm cuando esta era
anterior y 3 cm cuando esta era posterior. Nivel de evidencia IIb.

CASO CLINICO
Se trata de femenino la cual cuenta con 36 semanas de gestacion la cual inicia con trabajo de parto desde hace 6 horas, actualmente
presenta contracciones cada 3 minutos aproximadamente y con duracin de 50 a 60 segundos, a la exploracin se observa producto
transverso, a la exploracin vaginal se observa 10 % de borramiento y 1-2 centmetros de dilatacin, no se palpan estructuras oseas, sin
embargo se observa sangre fresca y en cantidad moderada, la paciente cuenta con antecedentes gineco-obstetricos de cesarea previa
con periodo intergenesico de 10 meses.

PREGUNTA
Considerando el cuadro clnico, cual es la complicacin ms probable que se presentara en este caso.

RESPUESTA
a.- Desprendimiento de Placenta.
b.- Ruptura Uterina.
c.- Sufrimiento Fetal.
d.- Cesarea por Placenta Previa.

























MANUAL DE TRABAJO DEL CURSO ENARM CMN SIGLO XXI
CURSO ENARM CMN SIGLO XXI TEL: 36246001 Pharmed Solutions Institute PGINA 320

DESPRENDIMIENTO PREMATURO DE PLACENTA NORMOINSERTA (DPPNI). CIENCIAS BASICAS: DPPNI, abruptio placentae o accidente
de Baudelocque es el proceso de separacin parcial o total de la placenta de su lugar de insercin normal que corresponde al fondo
uterino, se produce a partir de las 20 semanas de gestacin hasta antes del nacimiento del feto, pudiendo ocurrir incluso durante el
trabajo de parto. SALUD PUBLICA: Aproximadamente 1% de los partos. El 50% ocurre en embarazos con sndrome hipertensivo del
embarazo (SHE). Existe un riesgo de recurrencia que va desde 5.5 hasta 30 veces. La tasa de mortalidad perinatal es entre un 20 y un
35%, dependiendo de la severidad del cuadro y de la edad gestacional y determinados bsicamente por la prematuridad y la hipoxia.
Actualmente la mortalidad materna es infrecuente, pero la
morbilidad es comn y puede ser severa. Casi todas las
complicaciones maternas graves del DPPNI son consecuencia
de la hipovolemia, y de la patologa de base asociada al DPPNI.
CLASIFICACION: Grado 0: asintomtico y generalmente se
puede diagnosticar en periodo postparto. Grado I: pacientes
quienes presentan solo hemorragia vaginal. Grado II: pacientes
que se presenta con hemorragia vaginal, hematoma
retroplacentarios, sensibilidad uterina (a veces sin hipertona),
y signos de sufrimiento fetal. Grado III: paciente que presenta
hemorragia vaginal, hematoma retroplacentario, sensibilidad
uterina (con o sin hipertona), Choque materno, muerte fetal y
signos de coagulopatia de consumo. La evidencia ha mostrado
que hay ms casos de Desprendimiento de placenta Grado III
asociado a Hipertensin arterial en el embarazo que en
pacientes que no la padecen y que adems el resultado perinatal era ms pobre en estas pacientes. PATOGENIA: La gran mayora es
multicausal y plantean la posibilidad de un mecanismo isqumico a nivel decidual como factor involucrado, asociado con patologas
variadas. De estas la ms comn es la Enfermedad Hipertensiva asociada al Embarazo, con la cual se presentan los casos de DPPNI ms
graves (45% de los casos). Otros trastornos descritos: Antecedente de DPPNI en embarazos previos, RPM, Traumatismo abdominal
grave, descompresin uterina brusca (polihidramnios o salida del primer gemelar), leiomiomas uterinos, consumo de cocana, bajo
incremento ponderal materno, tabaquismo. Causas directas: Constituyen entre el 1% y 5% de todas las causas y especialmente se
refieren a: trauma directo (accidentes de trnsito, cadas violentas, etc.), disminucin sbita del volumen uterino como puede suceder
despus de una perdida rpida y abundante de lquido amnitico o el parto del primer gemelo, o un cordn umbilical anormalmente
corto (lo cual puede ocurrir usualmente durante el parto). La causa precisa que conduce al abrupcio de placenta en la mayora de los
casos es desconocida. La formacin del hematoma retroplacentario originado por el DPPNI produce la separacin de la placenta lo que
deteriora el intercambio feto-materno dando lugar a una prdida del bienestar fetal o incluso a la muerte fetal si el desprendimiento es
>50%. Tambin la lesin de los vasos placentarios induce la produccin de sustancias vasoactivas (prostaglandinas y endotelina 1) que
pueden originar por si mismas una disminucin del intercambio gaseoso fetomaterno materno. Las prostaglandinas y la distensin
uterina originada por el hematoma, provocan una hipertona uterina que es tpica del DPPNI. La formacin del cogulo retroplacentario
supone una hemorragia materna oculta. Adems, el hematoma retroplacentario (HRP) y la infiltracin sangunea de la pared muscular
estimulan las contracciones colapsando el retorno venoso, y persistiendo el aporte arterial, con presiones superiores a las del tero, por
lo que el hematoma sigue creciendo. Si el cogulo es de gran tamao dar lugar a hipotensin y shock materno. Adems el secuestro de
factores de coagulacin en la formacin del cogulo y el paso de tromboplastina al torrente circulatorio materno podrn determinar la
aparicin de una coagulacin intravascular diseminada. El hematoma retroplacentario es el principal responsable de la clnica y de las
complicaciones maternas y fetales del DPPNI. DIAGNOSTICO: El motivo de consulta es el dolor abdominal, de comienzo brusco, intenso
y localizado en la zona de desprendimiento, que se generaliza a medida que aumenta la dinmica uterina y se expande el HRP. Existe
compromiso del estado general, palidez taquicardia, pero las cifras de presin pueden aparentar normalidad si existe SHE. La
hemorragia genital (78%) es rojo oscura, sin cogulos o muy lbiles; es posterior a la presencia del dolor y decididamente menor que el
compromiso del estado general, ya que la sangre proveniente del HRP debe buscar camino, separando las membranas de la pared
uterina para salir al exterior. El sangrado es de inicio sbito y cuanta variable (lo que no guarda necesariamente relacin con la
gravedad del cuadro). Segn su ubicacin, si el HRP aumenta, es posible observar en horas que el tero crece. El dolor uterino se
expresa con reblandecimiento y dolor de espalda en el 66% de los casos. La irritabilidad del tero va progresando: contracciones
uterinas, polisistola e hipertona (20%), contractura, palpndose finalmente un tero de consistencia leosa, tpico de este cuadro. Es
difcil palpar al feto y precisar su presentacin debido a la irritabilidad uterina. La auscultacin muestra sufrimiento fetal (60%) o
muerte fetal (15-35%). En el tacto vaginal dilatacin cervical, la que progresa rpidamente debido a la hiperactividad uterina. Las
membranas estn tensas y al romperse, el lquido amnitico presenta color vinoso, al estar mezclado con sangre y hemoglobina
procedente del HRP. Ecogrfico. Su utilidad es limitada, ayudando al diagnstico diferencial con la placenta previa dado que permite
descartarla. Tambin tiene utilidad para el seguimiento de los hematomas retroplacentarios en pacientes con DPPNI asintomticos en
el segundo trimestre inicios del tercero. Un nuevo campo de investigacin es el intento de mejorar la sensibilidad de este mtodo
diagnstico mediante el uso del doppler, para ver la vascularizacin y reas de perfusin placentaria. Pruebas diagnsticas
complementarias. Se han buscado pruebas que nos ayuden en el diagnstico, ninguna de ellas es de utilidad clnica. Signos
cardiotocogrficos, un buen registro no es motivo de tranquilidad, pues se puede producir un rpido deterioro del estado fetal. Niveles
de Ca125. Niveles de dmero-D. Trombomodulina. Es un marcador de dao endotelial. -fetoprotena en suero materno. Se encuentra
aumentada. TRATAMIENTO: La interrupcin de la gestacin se realizar por la va ms rpida sin tener en cuenta la edad gestacional, el
tero debe ser evacuado lo ms rpido posible, la decisin de realizar parto vaginal depender de que las condiciones obsttricas sean
ideales para una terminacin rpida y sobre todo cuando existan signos de coagulopata, se realizar tacto vaginal para estimar las
horas que faltan para la terminacin del parto, de acuerdo a las condiciones halladas (borramiento, dilatacin, altura de la
presentacin). Adems con el tacto, si hay trabajo de parto, se descarta la presencia de placenta previa. Se romper las bolsas de las
aguas tan pronto como sea posible aun sin tener en cuenta el mtodo de parto a emplearse, con sta maniobra disminuye la presin
intraamnitica, se reduce la extravasacin sangunea y se abrevia significativamente la duracin del parto. Si ste no hubiera
MANUAL DE TRABAJO DEL CURSO ENARM CMN SIGLO XXI
CURSO ENARM CMN SIGLO XXI TEL: 36246001 Pharmed Solutions Institute PGINA 321

comenzado, se puede intentar la induccin mediante la infusin intravenosa continua de oxitocina. Ambas conductas, parto
espontneo o inducido, estn justificadas por la gran rapidez con que se produce el parto en estas pacientes. Si en el transcurso del
trabajo de parto se constata una alteracin de los latidos fetales, se interrumpir el trabajo de parto y se indicar la cesrea sin
dilacin. La cesrea abdominal se realiza: Si la condicin de la paciente se agrava, si el feto est vivo, para prevenir su muerte, si fracasa
la induccin del parto o si el parto se prolonga, en los casos ms graves, cuando se sospecha una apopleja uterina, ya que permite no
solo evacuar el tero con rapidez, sino tambin decidir si ste podr conservarse o no. La precaucin del obstetra no termina con la
evacuacin del tero, ya que debe controlarse todava la correcta retraccin uterina, debiendo recordar la posibilidad de instalacin de
un cuadro de atona. En los casos severos de apopleja, o si despus del alumbramiento no hubiera respuesta a los ocitcicos y contina
la hemorragia por atona uterina se efectuar inmediatamente la histerectoma. El puerperio inmediato es el momento ms crtico de
esta complicacin. Se pondr especial atencin en: Mantener la volemia y las constantes hematolgicas con soluciones salinas y sangre
preferentemente fresca. Se evitaran los expansores plasmticos, por ser antiagregantes plaquetarios. Control estricto de signos vitales.
Control de la diuresis horaria. Observacin rigurosa de los valores de crasis sangunea. COMPLICACIONES DE LA FORMA GRAVE:
Accidente de Couvelaire: apopleja uteroplacentaria; La sangre derramada invade la pared uterina con zonas de extensas hemorragias
miometriales que disocian los haces musculares y puede difundirse hacia el tejido subperitoneal, trompas, ovarios y a veces ligamentos
anchos. A causa de esta infiltracin sangunea el miometrio pierde su propiedad contrctil. CID; Se da en el 30 % en los DPPN masivos
con muerte fetal. Se produce por el paso de tromboplastina a la circulacin materna y activacin de la coagulacin. La baja
concentracin de fibringeno plasmtico depende del atrapamiento de fibrina en el hematoma retroplacentario, esto provoca que la
hemorragia sea incoercible y la sangre no coagule. Insuficiencia Renal Aguda; l colapso circulatorio perifrico y la sobredistencin
uterina provocan isquemia en las zonas corticales de los riones. Shock Hipovolmico; Se produce por la hemorragia brusca y masiva,
con o sin exteriorizacin de la sangre por los genitales. Necrosis de la Hipfisis: (Sndrome de Sheehan). La hipfisis anterior puede
sufrir una necrosis total o parcial debido a trombos o espasmos de los vasos del sistema porta. Se manifiesta en el puerperio por
agalactia como sntoma inicial, ya que son las clulas secretoras de prolactina las que primero se ven afectadas por la isquemia,
posteriormente se evidencian los sntomas de esfera gonadal, tiroidea y suprarrenal. Aunque es poco frecuente.

CASO CLINICO
Se trata de femenino con 40 semanas de gestacin la cual acude a consulta debido a que inicia trabajo de parto, refiere que desde hace
2 horas inicia con dolor en la regin abdominal baja tipo contracciones, adems de presencia de sangrado leve pero continuo, a la
exploracin fsica usted palpa contracciones de 2 a 3 en 10 minutos con una duracin de 40 segundos aproximadamente, al realizar
tacto encuentra borramiento del 40 % y dilatacin de 3 cm, sin embargo presenta sangrado importante, las constantes vitales de la
madre se encuentra dentro de parmetros normales, no as del producto el cual observa frecuencia cardiaca aproximadamente de 160
a 180 lpm.

PREGUNTA
Cul de los siguientes factores de riesgo esta altamente relacionado con desprendimiento de placenta normoinserta.

RESPUESTA
a.- Mayor de 35 aos de edad.
b.- Muerte Materna.
c.- Cesarea.
d.- Miomectomia.

CASO CLINICO
Paciente de 37 aos de edad, acude por abdominalgia de 1 da de evolucin, a las 25,3 semanas de gestacin por una rotura prematura
de membranas (RPM). Las ecografas y analticas practicadas hasta ese momento eran normales. Como antecedentes obsttricos
destacan una cesrea (incisin transversal) 2 aos antes, a las 28 semanas de gestacin por riesgo de prdida del bienestar fetal y un
aborto espontneo previo. A la exploracin presenta abdomen ligeramente doloroso a la palpacin profunda en hipogastrio y en el
tacto vaginal se aprecia cuello uterino acortado un 20% y cerrado. La paciente refiere prdida de lquido por genitales externos, que se
confirma con el espculo con el que se observa salida de lquido amnitico claro a travs del crvix. En la ecografa abdominal se
observa feto nico con latido cardaco positivo, biometras de 26 semanas, placenta anterior normoinserta y lquido amnitico en
cantidad normal. Al tratarse de una RPM pretrmino de 25 semanas, se deriva a nuestro centro previa administracin de una dosis de
betametasona 12 mg intramuscular.

PREGUNTA
Cul de los siguientes factores de riesgo esta altamente relacionado con desprendimiento de placenta normoinserta.

RESPUESTA
a.- Mayor de 35 aos de edad.
b.- Ruptura prematura de membranas
c.- Cesarea.
d.- Miomectomia.






MANUAL DE TRABAJO DEL CURSO ENARM CMN SIGLO XXI
CURSO ENARM CMN SIGLO XXI TEL: 36246001 Pharmed Solutions Institute PGINA 322

ANEMIAS EN EL EMBARAZO. CIENCIAS BASICAS: La anemia es el problema hematolgico ms comn en el embarazo. Es referida como
un proceso dilucional secundario al aumento del volumen plasmtico. Sin embargo existen deficiencias nutricionales, hemlisis y otras
enfermedades que pueden causar anemia significativa y ser capaces de afectar a la madre como al feto. La anemia de acuerdo a la OMS
en la mujer embarazada se define como la concentracin de hemoglobina al nivel del mar menor a 11g/dl y el hematcrito menor que
33 % durante el tercer trimestre de la gestacin. La anemia puede relacionarse con muerte fetal nacimientos de bajo peso y
anormalidades del feto. La anemia sin embargo puede ser un marcador de factores nutricionales, sociales o ambientales ms que la
causa de esos problemas. Cuando la Hb cae a niveles inferiores a 6-7g/dL se pueden tener efectos adversos en la madre y el feto. La
anemia menos severa (8-10g/dL) es de poco riesgo para la madre pero puede poseer mayor riesgo para el infante. Un Hto elevado
puede asociarse no slo con partos prematuros, baja de peso fetal o muerte perinatal sino tambin con hipertensin materna y
toxemia. SALUD PUBLICA: Existe una prevalencia de anemia entre las mujeres embarazadas del 42%.4. En Mxico se encontr anemia
en el 21.6% de 500 mujeres embarazadas. Tomando en cuenta mujeres con embarazo normal y complicado se encontr la prevalencia
del 22.4%. Ciertos grupos de mujeres estn en mayor riesgo para desarrollar anemia durante el embarazo, probablemente en parte a
factores familiares, sociales, econmicos, nutricionales y a falta de cuidado prenatal. CLASIFICACION: Durante la gestacin, creemos til
tener en cuenta que las anemias que acompaan al embarazo pueden ser agrupadas en 2 categoras: 1. Directamente relacionadas con
la gestacin: a) Ferropnicas, b) Megaloblsticas, c) Hipoplsicas. 2. Que no guardan relacin directa con la gestacin: a) Anemias por
hemates falciformes. b) Otras anemias hemolticas y raras. ANEMIA FERROPENICA: En pases en desarrollo el 83-95% de las mujeres
embarazadas anmicas tienen deficiencia de hierro. El embarazo y el parto representan una prdida de1 a 1,3 g de hierro, que se extrae
fundamentalmente delos depsitos de hierro en el sistema reticuloendotelial y en el parnquima heptico, en forma de hemosiderinao
ferritina. Con frecuencia, las embarazadas enfrentan estas necesidades con las reservas de hierro exhaustas. Entre los factores que
llevan a ello se encuentran: menstruaciones abundantes embarazos con escaso periodo intergenesico, dietas con bajo contenido en
hierro, embarazos anteriores sin un adecuado suplemento frrico, partos con sangramientos durante el alumbramiento o el puerperio,
parasitismo intestinal, baja absorcin del hierro y otros. Estas anemias son pobres en signos y por lo general, son asintomticas; puede
observarse palidez cutaneomucosa y cierta tendencia a la fatiga. Las formas ms severas presentan un sndrome anmico dado por:
laxitud, "cansancio de muerte, irritabilidad, astenia, nerviosismo, cefalea, anorexia y otros. En los casos de anemias muy severas
alteraciones del apetito, pirosis, ardor lingual y bucal, flatulencia, constipacin y es posible la aparicin de glosistis. En ocasiones puede
haber manifestaciones de insuficiencia cardiacas y cardiomegalia. A veces las pacientes pueden
tener dolores de tipo neurlgico, adormecimiento de las extremidades, sensacin de hormigueo,
trastornos vasomotores y otros. Al realizar el examen fsico, se detecta palidez cutaneomucosa; las
uas de las manos, y a veces las de los pies, aparecen opacas y sin brillo, y se rompen con facilidad.
Con frecuencia, la auscultacin permite escuchar soplos anmicos funcionales. Diagnstico: durante la atencin prenatal el estudio
sistematico de la Hb y Hto que deben hacerse cada 6-12 semans, permitir el diagnostico precoz de anemia. Si la hemoglobina est por
debajo de 110 g/L, se considera que hay anemia. Independientemente de la clasificacin dada por la OMS (1991), creemos que desde el
punto de vista prctico conviene considerar 3 grados: ver cuadro anexo. Las anemias ferriprivas se caracterizan por tener: 1.
Hemoglobina y hematocrito disminuidos. 2. Hierro srico disminuido. 3. Discapacidad total elevada. 4. ndice de saturacin disminuido.
5. Protoporfirina eritrocitaria elevada. 6. Lmina perifrica normoctica hipocrmica. Tratamiento profilctico: administracin de hierro
VO desde la primera consulta prenatal. Ingestin diaria de 60mg de hierro elemental, en pacientes con feto nico. Debe administrarse
en forma de sales ferrosas: 1. Sulfato ferroso: tableta de 300 mg = 60 mg de Fe elemental. 2. Gluconato ferroso: tableta de 300 mg = 36
mg de Fe elemental. 3. Fumarato ferroso: tableta de 200 mg = 65 mg de Fe elemental. La administracin de hierro debe proveer al
organismo la cantidad suficiente de este elemento para lograr la regeneracin de la hemoglobina y para la reserva. Esto se consigue
administrando 2 a 3 tabletas diarias de sales ferrosas durante el embarazo y hasta 6 meses despus del parto. Tratamiento curativo: La
VO es de eleccin siempre que sea posible en dosis de 600 a 1 200 mg/da, que equivaldran a 120 o 180mg, preescrito en 1-2 tabletas
media hora antes de desayuno, almuerzo y comida, ya que es preferible separarlo de los alimentos. Puede indicarse adems la
administracin de 100mg de cido ascrbico diariamente. La administracin durante las comidas presenta mejor tolerancia, aunque es
menor su absorcin. Las sales de hierro no deben acompaarse de leche, t, caf o huevo, ya que stos interfieren en su absorcin. El
tratamiento debe ser sostenido por lo menos de 2 meses despus de normalizado el hematocrito y la hemoglobina. Una forma prctica
de calcular la dosis total de hierro en miligramos sera: Hemoglobina normal-hemoglobina de la paciente 255 mg de Fe. La va IM sera
necesaria en las circunstancias siguientes: 1. Intolerancia gstrica al hierro oral. 2. Cuando este contraindicada su administracin como
en los casos de gastritis, lcera, diverticulosis y otras afecciones digestivas. 3 Falta de respuesta al tratamiento oral. 4. Sndrome de
malabsorcin intestinal. 5. Anemia intensa (85 g/L o menos) despus de las 34 semanas. ANEMIA MEGALOBLASTICA: En el embarazo,
existe un aumento de necesidades de cido flico y Vit., B12 para la sntesis del ADN y del ARN, debido al rpido crecimiento celular del
embrin y del feto en desarrollo. La anemia megaloblatica del embarazo es causada por deficiencia de cido flico y vit., B12. La
gestante tambin puede sufrir una deficiencia de cido ascrbico, que se asocia con la de cido flico. En la lmina perifrica se
encuentra macrocitosis, punteado basfilo (policromatofilia), leucocitos de Pitaluga y macroplaquetas. La determinacin de cido flico
ofrece cifras por debajo de 4g/l. la medula osea es megaloblastica. La deficiencia de cido flico y de cido ascrbico determinan un
aumento de las complicaciones infecciosas de madre, abortos, partos prematuros, RPM y otros. Tratamiento: profilctico 1. cido
flico: 1 mg/da (tabletas). 2. cido ascrbico: 200 mg/da (tabletas). Tratamiento especfico: Se administra ac. flico 5-10mg/da.
Adems del cido flico debe administrarse hierro en dosis teraputica, ya que la transformacin de la mdula sea megaloblstica en
normal, requiere gran cantidad de hierro. Antes de iniciar el tratamiento debe buscarse la existencia de una infeccin que condicione la
anemia (la infeccin urinarias es la mas frecuente) y puede hacerla refractaria a tratamiento. Una caracterstica de esta anemia es su
remisin espontnea despus del parto. ANEMIA HIPOPLSICA: Se le relaciona con el embarazo, y se considera por algunos como una
manifestacin de toxemia. Es rara y de gravedad variable. Puede tener remisiones parciales o completas y en algunas ocasiones,
desaparecer espontneamente despus del parto. Puede provocar muerte fetal y parto pretrmino. Diagnstico: la anemia es de
desarrollo rpido, con palidez, fatiga y taquicardia. Las manifestaciones clnicas dependen de los grados de la anemia, la
granulocitopenia y la trombocitopenia. Puede haber formas globales, con cada de los 3 sistemas o formas parciales con la afeccin de 1
solo de ellos. Laboratorio: fundamentan el diagnstico las determinaciones de: 1. Hemoglobina (muy baja). 2. Hematocrito (reducido).
3. Trombocitopenia. 4. Hierro srico (elevado). 5. Mdula sea hipocelular con depresin selectiva o de los 3 sistemas (pancitopenia).
MANUAL DE TRABAJO DEL CURSO ENARM CMN SIGLO XXI
CURSO ENARM CMN SIGLO XXI TEL: 36246001 Pharmed Solutions Institute PGINA 323

Tratamiento: Para tratar la anemia hipoplsica se dispone de recursos muy limitados. El tratamiento con hierro, cido flico y vitamina
B12 ha resultado ineficaz. En el aspecto mdico, se aconseja: una serie de medidas para prolongar la vida de la paciente, como son: 1.
Transfusiones de concentrados de glbulos rojos, si la anemia fuera lo fundamental. 2. Transfusin de plaquetas. 3. Administracin de
antibiticos (no profilcticos y con antibiograma). 4. Administracin de anablicos: nerobol: 1 a 3 mg/kg por va oral, diariamente.
ANEMIA POR HEMATES FALCIFORMES O DREPANOCITEMIA: de acuerdo con los sntomas clnicos de la Hb.S parece trasmitirse por
medio autosmicos recesivo (solo estn afectados los hemocigotos SS), se considera ms bien el resultado de herencia autosmica
codominante, puesto que el genotipo heterocigoto ATS (rasgo de clulas falciformes), puede bajo ciertas circunstancias, producir
morbilidad grave e incluso la muerte. En los individuos SS, la hemlisis es consecuencia del secuestro y de la destruccin de los
hemates falciformes en el sistema reticuloendotelial, as como su destruccin intravascular originada por trauma mecnico; de ah que
la anemia crnica sea la regla. La hemoglonina S y la hemoglobina C son resulta do de la sustitucin del cido glutmico en la posicin 6
de la cadena de la globina, por valina y lisina respectivamente. En la crisis drepanoctica se produce el fenmeno de deformacin
semilunar de los hemates y puede hacerse irreversible. El feneomeno de falciformes es debido a que la propiedad fundamental que
diferencia a la hemoglobina S de la hemoglonina A es la baja solubilidad de la primera en su forma desoxigenda, lo cual provoca su
precipitacin debido a la formacin de polimeros, agergados constituidos por la formacin de varias molculas de hemoglobina S, que
forman un gel semislido que se extiende a lo largo de los hemates y da lugar a la defiormacion caracterstica.

CASO CLINICO
Femenino de 16 aos de edad originaria de tabasco, la cual acude por cansancio, fatiga, adinamia, actualmente cursa con embarazo de
28 semanas de gestacion, en control de rutina muestra Hematocrito de 39 y hemoglobina de 10, desde la semana 20 fue diagnosticada
con anemia, fue suplementada con hierro, sin embargo comparado con el estudio anterior no hay mejora, actualmente se observa
edema de miembros plvicos ++,

PREGUNTA
Considerando los datos de laboratorio cual es su conducta a seguir para resolver el presente caso.

RESPUESTA
a.- Transfusin Sangunea.
b.- Complejo B
c.- Hierro.
d.- Realizar un estudio de tincin.

CASO CLINICO
Mujer de 24 aos, rea gineco-obsttrica: menarqua a los 12 aos; ritmo menstrual de 28x3-4 das, en ocasiones con cogulos; inicia
vida sexual activa a los 22 aos y cursa el 6 mes de su primer embarazo de evolucin normal aunque tuvo emesis gravdica leve
durante el primer mes pero no acude a control gineco-obsttrico hasta el sexto, das antes de la consulta hematolgica refiriendo
entonces astenia, somnolencia y se le observa plida. En la consulta de hematologa se confirman tales datos y adems menciona pica
y pagofagia de varios meses o incluso un ao de evolucin. Signos vitales normales a la exploracin, observando pelo frgil y
platoniquia, se tiene la impresin de palpar el bazo al inspirar profundo. Citologa hemtica: Hemoglobina (Hb): 10.5 g/dL; Hematocrito:
0.32/l; Eritrocitos: 5.000.000/mm3; Volumen eritrocitario medio: 70 fl; Hemoglobina corpuscular media: 26pg; Concentracin de
hemoglobina corpuscular media: 30g/dL; Leucocitos y frmula diferencial normales; Plaquetas: 480.000/mm3

PREGUNTA
Cual es el diagnostico mas probable?

RESPUESTA
a.- Anemia normoctica normocrmica
b.- Anemia megaloblstica normocrmica
c.- Anemia microctica normocrmica
d.- Anemia microctica hipocrmica

INFECCIONES DE VIAS URINARIAS (IVU). CIENCIAS BASICAS: La infeccin urinaria (IVU) es una entidad clnica que se asocia
frecuentemente al embarazo; las variantes clnicas son la bacteriuria asintomtica, la cistouretritis y la pielonefritis. Durante la
gestacin se producen una serie de cambios fisiolgicos que aumentan el riesgo de presentar infecciones del tracto urinario: Dilatacin
ureteral secundario a la accin de progesterona y a la compresin uterina, reflujo vesico-ureteral, estasis vesical, aumento del filtrado
glomerular con glucosuria y amnioaciduria con elevacin del pH urinario. SALUD PUBLICA: La infeccin urinaria es una de las
complicaciones mdicas ms frecuentes del embarazo, nicamente superada por la anemia y la cervicovaginitis; si no es diagnosticada
y adecuadamente tratada, puede llevar a un incremento significativo en la morbilidad en la madre y el feto. El 20% de las embarazadas
presentan una infeccin vas urinarias bajas (ITU) en el curso de la gestacin. Un 10% de los ingresos hospitalarios en gestantes se
deben a IVU. La bacteriria asintomtica no tratada es un factor de riesgo de pielonefritis, bajo peso al nacer y amenaza de parto
prematuro. CLASIFICACION: La infeccin en el embarazo es clasificada segn el sitio de proliferacin bacteriana, de la siguiente manera:
Bacteriuria asintomtica=Infeccin urinaria baja. Cistitis, Pielonefritis=Infeccin urinaria alta. PATOGENIA: Constituyen factores de
riesgo de desarrollar una infeccin urinaria en la gestacin los siguientes factores: 1. Bacteriria asintomtica 2. Historia de ITU de
repeticin 3. Litiasis renal 4. Malformaciones uroginecolgicas 5. Enfermedades neurolgicas (vaciado incompleto, vejiga
neurgena) 6. Reflujo vesico-ureteral 7. Insuficiencia renal. 8. Diabetes mellitus 9. Infeccin por Chlamydia trachomatis 10.
Multiparidad 11. Nivel socioeconmico bajo. Entre los diferentes agentes etiolgicos que se conocen como causantes de las IVU, la
Escherichia coli, procedente de la flora enterobacteriana es el microorganismo ms habitual y causante de la mayor parte de estas
MANUAL DE TRABAJO DEL CURSO ENARM CMN SIGLO XXI
CURSO ENARM CMN SIGLO XXI TEL: 36246001 Pharmed Solutions Institute PGINA 324

infecciones (80-90% casos). Le siguen por orden de importancia: Proteus mirabilis, Kelbsiella pneumoniae, Enterococcus spp y
Staphylococcus saprophyticus. A mayor edad gestacional, mayor probabilidad de Gram positivos principalmente SGB. BACTERIRIA
ASINTOMTICA: Se denomina bacteriuria asintomtica (BA), a la presencia de bacterias en orina cultivada (ms de 100.000 colonias
por ml) sin que existan sntomas clnicos de infeccin del tracto urinario. La mayora de bacteriurias asintomticas se dan en el primer
trimestre de la gestacin. Aparece en un 5-6% de los casos. El 25% de las bacteririas asintomticas no tratadas desarrollaran una
pielonefritis aguda vs el 3% de las tratadas. Hasta un 30% de las bacteririas asintomitcas tratadas presentan una recada a pesar del
correcto tratamiento antibitico. Diagnstico: es microbiolgico: cultivo orina 100.000 unidades formadoras de colonias (UFC). El
diagnstico se establece mediante el urocultivo cuantitativo. La muestra de orina debe ser obtenida bajo determinadas condiciones de
asepsia: despus de limpieza de genitales externos y recogida del chorro medio de la miccin. NO es necesario para realizar un
urocultivo el sondaje vesical. Si el resultado del urinocultivo es de orina contaminada, debe repetirse, haciendo hincapi en las
medidas de asepsia para una correcta toma de la muestra. Tratamiento: Normalmente se dispone de antibiograma. Utilizar el
antibitico de espectro ms reducido: empezar por amoxicilina, despus amoxicilina-clavulnico....etc. En caso de NO disponer de
antibiograma se propondr como tratamiento emprico de la bacteriria asintomtica: Amoxicilina-clavulnico 500 mg/8 h vo x 7 dias.
En caso de alergia a betalactmicos: Fosfomicina trometamol 3 g vo (unidosis o pauta corta dos dosis separadas por 3 das). Otras
alternativas teraputicas de mayor espectro que deben reservarse para casos de resistencias sn: Cefuroxima 250 mg/12 h vo x 7 das
Cefixima 400 mg/24 h vo x 7 das En caso de alergia a betalactmicos: Nitrofurantona 50-100 mg/6 h vo x 7 das. Seguimiento:
Comprobar curacin con urinocultivo a los 15 das despus de tratamiento. En caso de recidiva, actuar segn antibiograma y si no se
dispone de l, ampliar el espectro. Repetir urinocultivo cada trimestre de gestacin. CISTITIS AGUDA: Es un sndrome caracterizado por
una clnica de urgencia miccional, disuria y dolor suprapbico en ausencia de sntomas de afectacin sistmica (fiebre) y dolor lumbar.
Aparece en un 1.3% de las gestaciones. La mayora de cistitis agudas se presentan en el segundo trimestre del embarazo. Diagnstico:
clnica sugestiva y cultivo orina positiva (100.000 UFC). Puede aparecer hematuria macro/microscpica. Recordar normas de recogida
de muestra de orina para urinocultivo: NO requiere necesariamente de sondaje vesical, pero debe recogerse en determinadas
condiciones de asepsia: despus de limpieza de genitales externos y recogida del chorro medio de la miccin. Tratamiento: idealmente
segn antibiograma especfico, pero puede tratarse empricamente si la clnica es muy sugestiva y no se dispone de resultado de
urinocultivo. El tratamiento suele ser emprico porque no solemos disponer de antibiograma Cefuroxima 250 mg/12 h vo x 7 das. Si
alergia a betalactmicos: Fosfomicina trometamol 3 g vo (unidosis o pauta corta dos dosis separadas por 3 das). Si disponemos de
antibiograma, se debe seguir la misma estrategia teraputica que en el caso de bacteriria asintomtica y empezar con el antibitico de
menor espectro: Amoxicilina 500 mg/8 h vo x 7 das. Amoxicilina-clavulnico 500 mg/8 h vo x 7 dias. Cefixima 400 mg/24 h vo x 7 daS.
En caso de alergia a betalactmicos: Nitrofurantona 50-100 mg/6 h vo x 7 das. Comprobar curacin con urinocultivo a los 15 das
despus de tratamiento. En caso de recidiva, actuar segn antibiograma y si no se dispone de l, ampliar el espectro. Repetir
urinocultivo cada trimestre de gestacin. PIELONEFRITIS AGUDA: Infeccin del parnquima renal que aparece en 1-2% de las
gestaciones y cuya prevalencia aumenta al 6% en gestantes en las que no se ha realizado cribado de bacteriria asintomtica durante el
embarazo. El 80-90% de las pielonefritis aparecen en el 2, 3 trimestre de la gestacin y en el puerperio. La va de entrada ms
frecuente es la ascendente a travs de la va urinaria aunque en pacientes inmunodeprimidos la va de diseminacin puede ser la
hematgena (principalmente en diabticos, tratamiento con corticoides, enfermedades sistmicas...). La localizacin ms frecuente es
la afectacin renal derecha (50% de los casos); en un 25% de casos la localizacin es izquierda y en un 25%, bilateral. Si la paciente tuvo
una bacteriria asintomtica, es posible que el germen causante de la pielonefritis sea el mismo. Clionica; Dolor costovertebral con
puopercusin lumbar positiva, fiebre, nuseas y vmitos, en general, NO cursa con clnica de cistitis aguda. Tratamiento ambulatorio:
Si NO cumple criterios de hospitalizacin, es posible el control ambulatorio en dispensario de curas de enfermera (CENF) cada 24h: De
eleccin ceftriaxona 1 g /24 h ev o im, en caso de alergia a betalactmicos, gentamicina 240 mg/24h ev o 80 mg/8h im. Si mejora
clnica del cuadro febril, a las 48-72 h podr pasarse a terapia oral. Si se dispone de antibiograma, pasar el antibitico de menor
espectro: amoxicilina, despus amoxicilina-ac. Clavulanico. Si NO se dispone de antibiograma, iniciar tratamiento emprico con
cefuroxima 250mg/12 h vo hasta cumplir 14 das de tratamiento. Slo se utilizar como antibitico alternativo la cefixima 400 mg /24 h
va oral x 14 das en caso de sospecha de resistencias a la cefuroxima (por antibiogramas de urinocultivos anteriores). En caso de
persistencia de sndrome febril y no respuesta clnica tras 72h (3 das) de tratamiento mdico ambulatorio, indicar ingreso hospitalario.
Tratamiento hospitalario: En general el tratamiento de la pielonefritis ser ambulatorio a excepcin de aquellas pacientes que cumplan
algunos de los criterios de ingreso hospitalario: Criterios de ingreso hospitalario fiebre 38C, sepsis, deshidratacin, edad gestacional
24 semanas, cuadro clnico de amenaza de parto prematuro, pielonefritis recurrente, comorbilidad, intolerancia oral, fracaso de
tratamiento ambulatorio tras 72 h (3 das), no posibilidad de tratamiento ambulatorio. 1.- Hidratacin agresiva: Primer da:
Sueroterapia a una perfusin de 150 ml/h. Segundo da perfusin de100 ml/h. 2.- Tratamiento parenteral antibitico hasta 48 horas
afebril: En gestante: Primera eleccin: 1.- Ceftriaxona 1 g/24 h IV Alergia a betalactmicos: 2.- Gentamicina 80 mg/8 h IV. En purpera:
Primera eleccin: 1.- Ceftriaxona 1 g/24 h ev + ampicilina 1 g/6 h ev (se debe cubrir tambin el Enterococo faecalis). Alergia a
betalactmicos: 2.- Gentamicina 240 mg/24 h IV. En casos de sepsis y/o pielonefritis con sospecha de grmenes multiresistentes o
pacientes con sondajes o vas con tiempo prolongado: El tratamiento antibitico deber cubrir Pseudomona auroginosa u otros
grmenes multiresistentes: 1.- Ceftacidima 1 g/8 h IV. Alergia a betalactmicos: 2.- Amikacina 15 mg/kg/24h IV. En el caso de sospecha
de infeccin por Enterococco faecalis aadir ampicilina 1 g/6 h IV o iniciaremos tratamiento en monoterapia con piperacilina-
tazobactam 4g/8 h IV.

CASO CLINICO
Mujer de 25 aos de edad con 34 semana de gestacin. Antecedentes de aborto espontneo cuatro aos antes y un parto haca dos
aos. Haca tres das inicia con disuria, prurito, urgencia urinaria, dolor suprapubico, con descarga vaginal grumosa.

PREGUNTA
Cual es el tratamiento antibitico mas adecuado para este caso.

RESPUESTA
MANUAL DE TRABAJO DEL CURSO ENARM CMN SIGLO XXI
CURSO ENARM CMN SIGLO XXI TEL: 36246001 Pharmed Solutions Institute PGINA 325

a.- Ampicilina.
b.- Cefalexina.
c.- Nitrofurantoina.
d.- Trimetropin con sulfametoxazol.

CASO CLINICO
Paciente de 17 aos, nulpara, con antecedentes mrbidos de bacteriuria asintomtica con tres episodios de cervicovaginitis. Ingres a
las 31 semanas de gestacin por trabajo de parto prematuro. Al ingreso se le practic una amniocentesis obtenindose lquido
amnitico con meconio y estudio citoqumico sugerente de infeccin intraamnitica. Se tom cultivo del lquido amnitico y se inici
tratamiento. El trabajo de parto fue acelerado con oxitocina, resolvindose por va vaginal.

PREGUNTA
Cual de las siguientes complicaciones maternas fetales se asocia ms frecuentemente?

RESPUESTA
a.- Corioamnionitis.
b.- Pielonefritis.
c.- Sepsis puerperal.
d.- Urosepsis.

CASO CLINICO
Paciente de 17 aos primigestante con embarazo de 36 semanas por fum cierta y confiable y 38 semanas por ecografia de segundo
trimestre. Quien refiere cuadro clinico de 1 mes de evolucion caracterizado por edema de miembros inferiores. niega cefalea, niega
acufenos, niega fosfenos. refiere dolor en region lumbar de moderada intensidad no irradiado y disuria. refiere movimientos fetales
presentes. niega genitorragia, niega hidrorrea.

PREGUNTA
Cual es la conducta a seguir mas adecuada?

RESPUESTA
a.- Ceftriaxona.
b.- Gentamicina
c.- Amoxicilina.
d.- Cefuroxima.



MANUAL DE TRABAJO DEL CURSO ENARM CMN SIGLO XXI
CURSO ENARM CMN SIGLO XXI TEL: 36246001 Pharmed Solutions Institute PGINA 326

ESTADOS HIPERTENSIVOS EN EL EMBARAZO. CIENCIAS BASICAS: Definicin: Se habla de hipertensin en el embarazo cuando la
tensin arterial diastlica es > 90mmHg y la sistlica es >140mmHg, o un incremento en la TA sistlica de por lo menos 30mmHg del
valor basal o de diastlica de por lo menos 15mmHg sobre el valor basal. Proteinuria: Excrecin urinaria de protenas mayor de 30mg/dl
o ms en tiras reactivas (se requieren 2 determinaciones o ms en un lapso de 6 horas en ausencia de infeccin). 300mgs o ms en una
coleccin de orina en 24h. SALUD PUBLICA: Los trastornos hipertensivos durante la gestacin, son la primera complicacin mdica en
muchos pases del mundo, constituyendo una de las principales causas de morbilidad y mortalidad materna, fetal y neonatal. 7-10% de
los embarazos se complican con HTA. La preeclampsia/eclampsia origina el 70% de los estados hipertensivos, y el 30% lo representan
pacientes con hipertensin crnica preexistente durante el embarazo. En Mxico su prevalencia vara entre 7-10% de la poblacin
gestante, existe mayor incidencia antes de los 20 aos y despus de los 35; aproximadamente 75% de los casos corresponden a
primigestas. CLASIFICACION, de los trastornos hipertensivos durante el embarazo: A) Enfermedad hipertensiva inducida por el
embarazo: Preeclampsia leve. Preeclampsia severa. Inminencia de eclampsia. Eclampsia. Sndrome de HELLP. Preeclampsia recurrente.
Enfermedad hipertensiva no clasificable (imposibilidad de clasificar por carecer de elementos necesarios o por haberse instituido
tratamiento previo a su estadificacion). Hipertensin transitoria (HTA despus de la semana 20 o en las primeras horas posparto sin
otros signos de preeclampsia). B) Enfermedad vascular crnica hipertensiva: Hipertensin sistmica esencial (HTA independiente de la
gestacin o anterior a las 20 semanas y que persiste ms de 6 semanas posparto y que no sea a consecuencia de lesin de alteracin
anatmica o funcional renal). Hipertensin crnica con enfermedad hipertensiva agregada (HTA previa al embarazo agregndose
preeclampsia, puede haber elevacin de cido rico igual o mayor de 6mg/dl). De acuerdo a la Clasificacin del Colegio Americano de
Obstetricia y Ginecologa, se identifican 4 categoras: HIPERTENSION CRONICA: Coexistencia de hipertensin de novo antes de las 20
SDG, al igual que si tiene el antecedente de hipertensin preexistente antes del embarazo. Puede ser: primaria (esencial) o secundaria
(renal, suprarrenal, etc). Las mujeres con hipertensin crnica tienen un riesgo de 10-20% de desarrollar preeclampsia. Es conveniente,
adems, en las pacientes con hipertensin crnica, establecer si ya existe dao en algn rgano blanco antes del embarazo, incluyendo
hipertrofia ventricular izquierda, retinopata y/o enfermedad renal. En canto a los medicamentos, suspender el tratamiento con
medicamentos del grupo de los IECA, as como ARA II. Existen comunicaciones sobre restriccin del crecimiento fetal intrauterino,
oligohidramnios, insuficiencia renal y muerte neonatal. Hay que reemplazarlos por alfametil-dopa, labetalol o los antagonistas de
calcio. HIPERTENSION ARTERIAL CRONICA CON PREECLAMPSIA SOBREAGREGADA: Pacientes con hipertensin crnica, que presentan
descompensacin de las cifras tensionales y aparicin o incremento de proteinuria despus de las 20 SDG. El pronstico es peor que
con cada entidad por separado. Criterios diagnsticos: Aparicin de proteinuria por primera vez, despus de las 20 semanas de
gestacin. Incremento sbito de la proteinuria en pacientes que la presentaban antes del embarazo. Incremento sbito de la presin
arterial en la gestante que la tena bien controlada. Trombocitopenia (menos de 100 000 plaquetas/mL). Incremento de enzimas
hepticas por encima de valores normales. PREECLAMPSIA: la describimos ms adelante y por ultimo HIPERTENSION GESTACIONAL:
Presin arterial sistlica mayor o igual a 140mmHg y/o presin arterial diastlica mayor o igual a 90mmHg en una mujer previamente
normotensa, despus de la semana 20 de gestacin. En dos tomas separadas por un intervalo de 4 a 6h entre una y otra con ausencia
de proteinuria y se recupera despus del parto. Las mujeres con hipertensin gestacional con inicio antes de la semana 34 de gestacin,
tienen probabilidad de 35% de desarrollar preeclampsia.

CASO CLINICO
Paciente femenino de 37 aos de edad actualmente cursando su tercer embarazo con 14 semanas por FUM, acude a visita de control
prenatal cuenta con 17 SDG por USG, antecedentes personales patolgicos de pre-eclampsia, adems obesidad previa al embarazo,
cuenta con antecedentes familiares de diabetes y obesidad por parte de la madre e hipertensin por parte del padre, el cual fallece por
infarto al miocardio. A la exploracin fsica se observa paciente gestante con obesidad grado I, sin tratamiento previo, se enva EGO, QS
y BH, donde se observa triglicridos de 350, colesterol de 290, glucosa de 105 mg/dl, hemoglobina glucosilada de 7, tensin arterial en
dos ocaciones de 135/95 mmHg. Tratada con dieta hiposodica, hipocalorica y restriccin de liquidos.

PREGUNTA
Cul es el factor ms importante que presenta la paciente, para considerar Hipertension Arterial Pre-existente.

RESPUESTA
a.- Trigliceridemia.
b.- Hipercolesterolemia.
c.- Obesidad.
d.- Cifras tencionales altas.

CASO CLINICO
Se trata de paciente femenino de 26 aos de edad la cual acude a consulta de control prenatal actualmente contando con 21 semanas
de gestacin, refiere que ha presentado cefalea ocasional, mareo y cansancio a la exploracin fsica se observa edema de miembros
inferiores, la consulta anterior usted envi estudios de EGO, QS y BH, los datos de relevancia fue Hematocrito de 38, las constantes
vitales son de FR 18, FC 89, TA 160/90. MmHg. Con edema leve sin hematuria ni proteinuria.

PREGUNTA.
Cul es la conducta a seguir mas adecuada.

RESPUESTA
a.- Restricin de liquidos y sal.
b.- Vigilancia estrecha de tencion arterial.
c.- Monitorizacin de vitalidad fetal.
d.- Iniciar con alfametildopa.
MANUAL DE TRABAJO DEL CURSO ENARM CMN SIGLO XXI
CURSO ENARM CMN SIGLO XXI TEL: 36246001 Pharmed Solutions Institute PGINA 327

PREECLAMPSIA. CIENCIAS BASICAS: Se define como la aparicin de hipertensin y proteinuria despus de la semana 20 del embarazo.
Se suele acompaar de edemas pero no es necesaria la presencia de stos para ser diagnosticada. Es una enfermedad caracterstica y
propia del embarazo de la que se pueden tratar los sntomas, pero slo se cura con la finalizacin del mismo y si no se trata
adecuadamente puede ser causa de graves complicaciones tanto para la mujer embarazada como para el feto. CLASIFICACION:
Preeclampsia leve; presin arterial de 140/90mmHg o mas o elevacin de 30mmHg en la sistlica y 15mmHg en la diastlica, cuando se
conocen las cifras basales previas, se presenta despus de la semana 20 hasta 30 das posparto, existe proteinuria de ms de
300mg/24h, ausencia de sntomas de vasoespasmo. Preeclampsia severa; presin arterial de 160/110mmHg o ms, despus de la
semana 20 hasta 30 das posparto, existe proteinuria mayor de 5g/24h, presencia de cefalea, acufenos, fosfenos, edema generalizado.
Inminencia de eclampsia; se establece el diagnstico cuando despus de la semana 20 de gestacin, parto o puerperio (no ms de 30
das), aparece uno o ms de los siguientes datos: Presin arterial sistlica mayor a 185mmHg con presin arterial diastlica >115mmHg,
proteinuria mayor a 10g, estupor, prdida parcial o total de la visin, dolor epigstrico en barra, hiperreflexia generalizada.
Preeclampsia recurrente; presencia de cualquiera de los tipos de enfermedad hipertensiva inducida por el embarazo, que aparece por
segunda ocasin o ms en embarazos consecutivos o no. PATOGENIA: Factores de riesgo: Ausencia o deficiencia de control prenatal,
desnutricin, obesidad, intervalo intergensico menor a dos aos, muer menor de 18 y mayor de 35 aos, primigesta o nuligesta,
preeclampsia/eclampsia en embarazos anteriores o antecdenetes familiares repetidos, hipertensin arterial crnica o cualquier otro
trastorno hipertensivo durante lagestacion, sobredistension uterina de cualquier origen (embarazo gemelar o multiple, polihidramnios),
infeccion en vas urinarias recurrentes, DM tipo 1 y 2, enfermedad renal previa o recurrente, enfermedad trofoblastica, enfermedades
autoinmunes, factores psicosociales. Es un estado de vasoconstriccin generalizado secundario a una disfuncin en el epitelio vascular,
en lugar de la vasodilatacin propia del embarazo normal. Ello se asocia a isquemia placentaria desde mucho antes de la aparicin del
cuadro clnico, en lo que parece ser uno de los orgenes de los factores txicos para el endotelio vascular. Dicha isquemia parece ser
debida a una deficiente placentacin en la que no se producira la habitual substitucin de la capa muscular de las arterias espirales
uterinas por clulas trofoblsticas, que es lo que produce una vasodilatacin estable que permite aumentar varias veces el caudal de
sangre asegurando as el correcto aporte sanguneo a la unidad fetoplacentaria, su etiologa ltima que sigue siendo desconocida,
aunque vamos identificando factores. As podemos citar factores hereditarios por va materna pero tambin paterna, factores
inmunolgicos que explican que se produzca ms en la primigesta y ms si no ha estado expuesta antes al contacto directo con los
antgenos paternos, a factores nutricionales como la falta de ingesta de calcio en algunas poblaciones. Todo ello se manifiesta por un
desequilibrio entre prostaglandinas vasodilatadoras (prostaciclina) frente a las vasoconstricciones (tromboxano), quiz mediado por la
aparicin de exceso de produccin de citoquinas proinflamatorias (IL-2, TNF), y de produccin de lipoperoxidos capaces ambos de e
inducir alteraciones endoteliales, esta disfuncin endotelial general, que afecta a muchos rganos distintos como son el rin, el
cerebro, el hgado pero en especial al sistema cardiovascular por lo que la hipertensin es su expresin ms genuina, que con
frecuencia se acompaa de edema. El vasoespasmo y el edema facilitan la reduccin del volumen plasmtico que lleva a la
hemoconcentracin caracterstica y a la hipercoagulabilidad. Todo ello hace que exista una hipoperfusin multiorgnica que empeora
el cuadro a nivel general y fetoplacentario. DIAGNOSTICO: Se establece el diagnostico de preeclampsia leve cuando se presentan 2 o
mas de los siguientes signos: 1) Presin sistlica >140mmHg o elevacin >30mmHg sobre la presin habitual. 2) Presin diastlica,
>90mmHg o elevacin >15mmHg sobre la presin habitual. Proteinuria. Edema. La medicin de la presin deber realizarse con la
paciente sentada y requiere de dos tomas consecutivas con un intervalo de 6h o mas en este lapso de horas debe permanecer en
reposo. El incremento de proteinuria y de hipertensin en una paciente nefrpata o hipertensa previa se denomina preeclampsia
sobreaadida y el manejo clnico es parecido. Se establece el diagnostico de preeclampsia severa cuando se presentan 2 o ms de los
siguientes signos: 1) presin sistlica >160mmHg y presin diastlica >110mmHg. 2) proteinuria >5g en orina de 24h o su equivalente
en tiras reactivas (ms de 3+). 3) Oliguria de menos de 500ml/24h. 4) Trastornos cerebrales o visuales. 5) Edema generalizado. Por otra
parte tambin es grave cuando aparece Sndrome de HELLP (plaquetopenia, elevacin de enzimas hepticas y hemolisis), de
insuficiencia cardiaca (edema agudo de pulmn), o de insuficiencia renal (creatinina>1,2 mg/dL), o dolor epigstrico. La inminencia de
eclampsia se diagnostica con uno o ms de los siguientes datos: 1) Presin arterial sistlica >185mmHg con presin arterial diastlica
>115mmHg. 2) Proteinuria >10g. 3) Estupor. 4) prdida parcial o total de la visin. 5) Dolor epigstrico en barra. 6) Hiperrreflexia
generalizada. Por la gravedad de esta variedad clnica las pacientes deben ser manejadas como eclmpticas. Todas suelen acompaarse
de signos de afectacin fetal por insuficiencia placentaria crnica en forma de signos de restriccin del crecimiento intrauterino o
agudo con signos de Riesgo de Prdida de Bienestar Fetal. Con el agravante de que la situacin fetal suele empeorar al tratar la
hipertensin materna grave ya que al descender sus valores se disminuye la perfusin placentaria, y de que los frmacos administrados
a la madre dificultan la valoracin del estado fetal a travs del estudio de la frecuencia cardiaca fetal basal o test no estresante (NST en
ingls), por lo que las unidades que traten los casos graves tendran que disponer de la posibilidad de ecografa Doppler de forma
continuada. TRATAMIENTO: Debe recordarse que existe una hemoconcentracin a pesar de que puedan existir edemas, por lo que NO
deben administrarse diurticos para tratarlos o para disminuir la TA, excepto en los casos de edema agudo de pulmn o de oliguria. Por
la misma razn tampoco se debe hacer una dieta hiposodica estricta. Los mejores resultados actuales se deben a la posibilidad de
terminar la gestacin en los casos graves a partir de la semana 32 o antes en caso necesario, con buenos resultados perinatales gracias
a la posibilidad de madurar los fetos con corticoides (no contraindicados con buen control en la preeclampsia), y especialmente a las
mejoras en el manejo neonatal de los prematuros extremos. Preeclamsia leve. Terminar la gestacin al llegar a trmino sin sobrepasar
la semana 40. No se ha demostrado que se mejore con reposo, ni con tratamiento hipotensor. Se debe controlar la aparicin de signos
de gravedad para poder iniciar el tratamiento en el momento oportuno. Antihipertensivos: alfametil-dopa 250-500mgs VO cada 8 h, si
es preciso asociados a hidralazina 10-50mgs VO c/8hrs. Preeclampsia grave. En todos los casos se debe: 1) Controlar la TA con
hipotensores mantenindola de ser posible por debajo de los valores indicados como de gravedad, pero por encima de 140/90 para no
disminuir la perfusin placentaria en exceso. Se usara, hidaralazina bolo inicial de 5 mg IV si diastlica >110mmHg, repetir en 20
minutos (5-10mg) de acuerdo a respuesta hasta un mximo de 30 mg (de no contar con hidralazina se recomienda nifedipino VO 10mg,
repetir cada 10-30min). Labetalol, 20 mg por va intravenosa, seguidos de 40 a 80 mg cada 10 minutos, hasta una dosis mxima de 220
mg. Alfametil-dopa 500mgs VO cada 6 h, si es preciso asociados a hidralazina 50mgs VO c/6hrs. No usar betabloqueantes (atenolol) ni
IECAs, ni ARA-II. Indicado para evitar edema cerebral dexametasona (16mg IV dosis nica). 2) Hacer prevencin del riesgo de Eclampsia
y la posible la hiperreflexia, con sulfato de magnesio (SO4Mg, la dosis de ataque es de 6g. en 100 de dextrosa al 5% pasar en 15 min.
MANUAL DE TRABAJO DEL CURSO ENARM CMN SIGLO XXI
CURSO ENARM CMN SIGLO XXI TEL: 36246001 Pharmed Solutions Institute PGINA 328

Continuar con dosis de mantenimiento a 1-2 g por hora administrados en infusin IV continua (preparar sol. Glucosada de 900cc al 5%
mas 10 ampolletas de 1g. Pasar 100-200ml de solucin por hora igual a 24-48 gotas por minuto). En caso de intoxicacin utilizar 1 g de
Gluconato de calcio IV diluido en 100cc de solucin fisiolgica y pasar en 15 min. 3) Vigilar la aparicin de otros signos de gravedad
Sndrome de HELLP, CID clnica, insuficiencia cardiaca, renal, etc. 4) controlar el bienestar fetal con NST peridico, perfil biofsico y
Doppler para comprobar el grado de afectacin fetal y si existen signos de redistribucin vascular, para poder indicar la extraccin fetal
antes de la afectacin de los vasos venosos, momento que por lo que sabemos hasta ahora, coincide con el inicio de la acidosis fetal. 5)
Finalizar la gestacin: 1) A termino: en cuanto la situacin materna se estabiliza. 2) Pretrmino: >32 semanas, en cuanto se compruebe
la madurez pulmonar fetal o antes si hay indicacin materna o fetal. 3) Pretrmino <32 semanas: siempre que exista indicacin
materna por aparicin de complicaciones graves o fetales con signos de redistribucin ante la hipoxia. 28-32 semanas madurara con
corticoides y valoracin del riesgo de prematuridad correspondiente a cada semana y peso fetal frente a los signos de redistribucin
ante la hipoxia con afectacin de vasos venosos o indicacin materna. A partir de la semana 30 probablemente nos inclinaremos por la
extraccin fetal, con feto madurado.

CASO CLINICO
Se trata de femenino de 27 aos de edad la cual cuenta con 22 semanas de embarazo, acude a su tercera visita de control prenatal, al
revisar su expediente usted observa que ha presentado por tercera ocasin de forma independiente una tensin arterial de 140/90,
usted realiza una prueba de rutina para apoyar su sospecha diagnostica de pre-eclampsia, se observa edema de miembros inferiores,
proteinuria, cefalea constante.

PREGUNTA
Cual de las medidas no es adecuada.

RESPUESTA
a.- Manejo ambulatorio.
b.- Hidralacina 50 mg cada 12 hrs.
c.- Dieta hiposodia y restriccin de liquidos.
d.- Captopril 25 mg cada 24 hrs.

ECLAMPSIA. CIENCIAS BASICAS: Se denomina as a las convulsiones similares a una crisis epilptica aparecida en el contexto de una
preeclamsia e hiperreflexia, despus de la semana 20 de gestacin, parto o puerperio en ausencia de otras causas de convulsiones.
Suponen un riesgo vital y pueden ocurrir, antes, durante o despus del parto. TRATAMIENTO: 1) Soporte vital (va area, constantes,
va intravenosa...). 2) Tratamiento de la hipertensin: hidaralazina bolo inicial de 5 mg IV si diastlica >110mmHg, repetir en 20 minutos
(5-10mg) de acuerdo a respuesta hasta un mximo de 30 mg (de no contar con hidralazina se recomienda nifedipino VO 10mg, repetir
cada 10-30min). Labetalol, 20 mg por va intravenosa, seguidos de 40 a 80 mg cada 10 minutos, hasta una dosis mxima de 220 mg. 3)
Tratamiento anticonvulsivante con SO4Mg, la dosis de ataque es de 6g. en 100 de dextrosa al 5% pasar en 15 min. Continuar con dosis
de mantenimiento a 1-2 g por hora administrados en infusin IV continua (preparar sol. Glucosada de 900cc al 5% mas 10 ampolletas
de 1g. Pasar 100-200ml de solucin por hora igual a 24-48 gotas por minuto), ajustando los niveles a 4.8-9.6 mg/dl de magnesemia,
siendo necesario mantener los controles clnicos continuados (reflejo patelar, respiraciones /minuto, diuresis) para evitar que una
sobredosificacin pueda producir un paro cardiorrespiratorio. Esta medicacin se ha comprobado en estudios prospectivos y
randomizados ms eficaz que cualquier otra, pero los lmites teraputicos y txicos estn muy cercanos. Tambin est indicado el uso
de diazepam, como droga de segunda eleccin, que actuara disminuyendo el consumo de oxigeno por el tejido cerebral, pero que
posee efectos nocivos sobre el feto (depresin respiratoria, hipotona). Las dosis recomendada son: bolo de 10 mg por va IV y luego
continuar con 50 mg en 500 cc de solucin dextrosada al 5%. 4) Acabar la gestacin en cuanto la situacin clnica materna este
estabilizada y lo permita. COMPLICACONES: De preeclampsia/eclampsia; Sindrome de HELLP. Desprendimiento prematuro de
placenta. Insuficiencia renal aguda. Edema pulmonar agudo. Edema cerebral. Coagulacion intravascular diseminada. Ruptura heptica
(Raro, alta letalidad, se manifiesta por dolor epigastrico en barra o en el cuadrante superior derecho, irradiado a hombro, regin
iterescapulovertebral derecha, en hemicinturon, pudiendogeneralizarse a todo el abdomen). Hemorragia cerebral.

CASO CLINICO
Se trata de paciente de 19 aos de edad la cual acude por primera vez a consulta por amenorrea secundaria de 30 semanas por FUM,
acude hasta ahora ya que desconocan su embarazo en casa, la paciente refiere mareo, dolor abdominal en barra, opresin de torax,
cefalea e irritabilidad, a la exploracin fsica se observa escleras hiperemicas, leve rubicundez facial, rots incrementados y edema de
miembros inferiores, proteinuria, se mide fondo uterino con 28 cm, con disminucin de actividad fetal, los signos vitales de la madre
fueron FR 21, FC 96, TA 130/100 en 3 ocaciones.

PREGUNTA
Cul es la conducta inmediata a seguir con la paciente.

RESPUESTA
a.- Identificar la viabilidad fetal.
b.- Busqueda de sndrome de HELLP.
c.- Sulfato de magnesio 4 g IV.
d.- Prepara a la paciente para Cesarea.



MANUAL DE TRABAJO DEL CURSO ENARM CMN SIGLO XXI
CURSO ENARM CMN SIGLO XXI TEL: 36246001 Pharmed Solutions Institute PGINA 329

CASO CLINICO
Se trata de femenino de 27 aos con 32 semanas de gestacin que llega al servicio de urgencias por presentar cefalea intensa, acufenos
y fosfenos, dolor abdominal en barra, adems dolor en regin lumbosacra y sensacin de pesantes plvica, a la exploracin fsica se
observa fondo uterino de 28 cm, obesidad grado I, ROTs incrementado, edema de miembros inferiores +++, al tacto observa dilatacin
de 2 cm, con presencia de moco con sangre, se observan protenas en orina ++, nivel de conciencia disminida, leve desorientacin
temporo-espacial, el familiar refiere que la encontr en el suelo con somnolencia.

PREGUNTA
Cul es la conducta a seguir de forma inmediata.

RESPUESTA
a.- Indica Sulfato de Magnesio.
b.- Indica Hidralacina.
c.- Preparar para Cesarea.
d.- Preparar para Parto.

CASO CLINICO
Se trata de femenino de 18 aos de edad con 32 semanas de gestacin por fecha de ltima menstruacin, sin atencin prenatal, es
llevada a urgencias por presentar en casa perdida del estado de alerta y con movimientos tipo crisis convulsivas, a la exploracin se
observa somnolienta con respuesta a estmulos dolorosos, ha sido estabilizada en sus constantes vitales, los resultados de laboratorios
son los siguientes, plaquetas de 45,000, proteinuria +++, DHL de 590, AST 239, ALT 432.

PREGUNTA
Tomando en cuenta la gravedad del caso, cual es la mejor conducta a seguir.

RESPUESTA
a.- Realizar Cesrea.
b.- Sulfato de magnesio.
c.- Estabilizar plaquetas.
d.- Maduracin Pulmonar.



MANUAL DE TRABAJO DEL CURSO ENARM CMN SIGLO XXI
CURSO ENARM CMN SIGLO XXI TEL: 36246001 Pharmed Solutions Institute PGINA 330

DIABETES MELLITUS, INTOLERANCIA A LA GLUCOSA, DIABETES GESTACIONAL. GENERALIDADES: Cambios fisiolgicos en el embarazo
normal: El embarazo normal se considera un estado diabetognico o de resistencia progresiva al efecto de la insulina, durante el primer
trimestre y las etapas iniciales del segundo se eleva la sensibilidad a la insulina, lo que se ha atribuido a las mayores concentraciones de
estrgenos circulantes. Este fenmeno incrementa el depsito de energa, sobre todo en el tejido adiposo, con expansin del mismo;
pero a partir de las 24 a 28 semanas de gestacin aumenta paulatinamente la resistencia a la insulina, que puede alcanzar los niveles
que se observan en pacientes diabticos tipo 2. Ms evidente al final del segundo trimestre del mismo; el organismo de la madre se
prepara almacenando nutrientes y en esta etapa la sensibilidad a la insulina es mayor; sin embargo, en la segunda mitad de la
gestacin hay una resistencia progresiva al efecto de la insulina, lo que causa una liberacin de los nutrientes (glucosa, los aminocidos,
los cidos grasos los triglicridos y los oligoelemntos) para que el feto tenga un desarrollo adecuado. Las clulas del pncreas elevan
la secrecin de insulina en un intento de compensar la resistencia a la insulina del embarazo, lo que origina pequeos cambios en la
concentracin de insulina en el curso de la gestacin, comparados con los grandes cambios en la sensibilidad de la misma. El msculo
esqueltico es el sitio principal para utilizar la glucosa corporal, y junto con el tejido adiposo, empiezan a ser resistentes al efecto de la
insulina, lo que es ms evidente durante la segunda mitad del embarazo. Un embarazo normal se caracteriza por aproximadamente un
50% de disminucin en la disponibilidad de glucosa mediada por insulina. Barbour seala un incremento en la secrecin de insulina
hasta de 200% para tratar de mantener euglucmica a la madre. Una gran cantidad de sustancias producidas por la placenta y por los
adipocitos, (lactgeno placentario, hormona placentaria de crecimiento, prolactina, hormona liberadora de corticotropina-cortisol,
insulinasa, FNT y adipocitocinas [leptina, resistina, visfatina, adiponectina]), son las que reprograman la fisiologa materna y causan
este estado de resistencia a la insulina para dirigir los nutrientes hacia el feto en desarrollo, sobre todo en la segunda mitad del
embarazo. El lactogeno placentario se eleva unas 30 veces durante la gestacin. En los obesos hay una correlacin positiva entre el
FNTa y el IMC e hiperinsulinemia. RESISTENCIA A LA INSULINA: En esta resistencia a la insulina participan numerosas sustancias
producidas por la placenta y por los adipocitos, entre las cuales destacan la hormona de crecimiento placentaria y el FNTa, que por
diversos mecanismos disminuyen el efecto de la insulina a nivel intracelular. La primera aumenta la cantidad de la subunidad p85a de la
PI-3K (fosfatidil-inositol 3 cinasa), lo que al final disminuye un factor de trascripcin AKT que altera el desplazamiento de los
transportadores de glucosa (GLUT) hacia la membrana celular y disminuye la entrada de glucosa a la clula; el segundo altera la
fosforilacin de la subunidad beta del receptor de insulina y del IRS-1, ya que fosforila residuos de serina y treonina en lugar de los de
tirosina. Este evento conduce a la mayor degradacin de ambos y modifica la accin de la insulina. Otro mecanismo implicado es que en
el musculo esqueltico el GLUT-4 esta disminuido en el tejido adiposo en mujeres embarazadas, y es an menor en la diabetes mellitus
gestacional; adems, est alterada la traslocacin de estos trasportadores. Los cambios moleculares en el adipocito durante el
embarazo muestran reduccin en la trascripcin del PPARg, receptor nuclear que regula la trascripcin de varios genes centrales en el
metabolismo del adipocito (adiponectina, la lipoprotein lipasa, la protena P-2 fijadora de cidos grasos intracelulares y la protena no
acoplada mitocondrial). Barbour hall hasta 40 a 50% de reduccin en el ARNm del PPARg (un factor que suprime considerablemente a
este receptor nuclear es el FNTa) en embarazadas obesas y DM gestacional; esto altera la concentracin de cidos grasos en la sangre, y
las lipoprotenas ricas en triglicridos (VLDL) con mayor resistencia para el efecto de la insulina ya que la adiponectina es un
sensibilizador de esta hormona y estos cidos grasos entran a la clula como fuente energtica. SALUD PUBLICA: La prevalencia de
diabetes gestacional a nivel mundial se ha estimado en 7%. En Mxico se ha mencionado una frecuencia que vara entre 4 y 19% de la
poblacin obsttrica. Respecto a la edad de la madre, se ha sealado que la incidencia es de 0.4 a 0.5% en menores de 25 aos y de 4.3
a 5.5% en mayores de esa edad. Lo cierto es que la frecuencia de este trastorno se ha duplicado en la ltima dcada, en forma paralela
a la llamada pandemia metablica que afecta a las sociedades modernas. Ms de 90% de los casos de diabetes que complican a un
embarazo son casos de diabetes gestacional. La creciente prevalencia de diabetes, que se diagnostica en mujeres a edades tempranas,
favorece la presencia de diabetes y embarazo. Las mujeres que desarrollan diabetes gestacional tienen alto riesgo de desarrollar
diabetes tipo II, en los 10 aos que siguen al embarazo. PATOGENIA: Factores de riesgo: 1) Edad mayor de 25 o 30 aos. 2) IMC > 25
kg/m2 SC. 3) Antecedentes de hijos macrosmicos. 4) Diabetes mellitus en familiares de primer grado 5) Antecedentes de intolerancia a
la glucosa. 6) Glucosuria. 7) Ganancia de ms de 20 kg de peso en la actual gestacin. 8) Antecedentes de problemas obsttricos serios.
9) Peso bajo del feto al nacimiento (<10 percentil) para la edad gestacional. 10) Peso alto del feto al nacer (> del 90o percentil). 11)
Origen tnico de riesgo alto de obesidad o diabetes mellitus. Los ms importantes son: mayor edad en la madre, familiares de primer
grado con diabetes y mayor ndice de masa corporal pregestacional. En la segunda mitad del embarazo se requiere un estado fisiolgico
de resistencia a la insulina para dirigir los nutrientes almacenados en la madre hacia la unidad fetoplacentaria y dar un crecimiento
adecuado al feto; sin embargo, cuando las mujeres desarrollan diabetes mellitus gestacional, la resistencia a la insulina es ms
acentuada, lo cual modifica el medio intrauterino y causa crecimiento acelerado del feto, con riesgo elevado de macrosoma. Los datos
disponibles en la bibliografa muestran un aumento lineal en el ndice de complicaciones en el feto conforme se eleva la cifra de la
glucosa en la sangre materna: ndices de mortalidad del feto de 4% cuando la glucosa es menor de 100 mg y hasta de 24% cuando la
cifra excede 150 mg. DIABETES PREGESTACIONAL: o preexistente se refiere a pacientes conocidas con diabetes mellitus tipo I o II que
se embarazan. Los hijos de mujeres con diabetes pregestacional tienen alto riesgo de sufrir malformaciones estructurales congnitas,
aborto espontaneo y restriccin del crecimiento intrauterino, lo que se debe a que la hiperglucemia y la hiperinsulinemia en el feto
pueden ser teratognicas, ya que existe una fuerte asociacin entre un pobre control glucmico en el periodo periconcepcional y el
riesgo de tales malformaciones. La incidencia de riesgo de estas malformaciones es de 6 a 8% y la mayora ocurre en el sistema nervioso
central, el corazn, el sistema urogenital y el tubo gastrointestinal. DIABETES GESTACIONAL: Es un padecimiento caracterizado por la
intolerancia a los carbohidratos con diversos grados de severidad, que se reconoce por primera vez durante el embarazo y que puede o
no resolverse despus de este. Existe mayor riesgo de macrosoma y polihidramnios. Se incrementa el riesgo de prematurez,
preeclamsia/eclampsia, trauma obsttrico, cesrea y mortalidad perinatal. DIAGNOSTICO: Escrutinio; prueba de escrutinio con 50 g de
glucosa oral entre las semanas 24 a 28 de la gestacin. Una cifra de 140 mg con diabetes mellitus gestacional o bien 130 mg a la hora,
que identificara al 90%. Sin embargo, a pesar de tener la mayor sensibilidad (79%) y especificidad (87%) de todas las pruebas de
escrutinio disponibles, sta se reserva para pacientes con alto riesgo para padecer la enfermedad, ms que para la poblacin general.
Existen 4 formas de realizar el diagnostico de Diabetes Gestacional: 1) Glucemia en ayuno >126mg/dl en dos ocasiones. 2) Glucemia
casual >200mg/dl. 3) Prueba de tamiz con 50g con resultado >180mg/dl (170mg/dl en mujeres mayores de 30 aos). 4) Curva de
tolerancia a la glucosa con 100g o 75 g. Se realiza el diagnostico al tener alterados 2 o ms de los siguientes valores: ver cuadro anexo.
MANUAL DE TRABAJO DEL CURSO ENARM CMN SIGLO XXI
CURSO ENARM CMN SIGLO XXI TEL: 36246001 Pharmed Solutions Institute PGINA 331

En caso de tener alterado un solo valor se diagnostica intolerancia a los carbohidratos. COMPLICACIONES MATERNO-FETALES: En el
feto; Macrosoma, muerte intrauterina, asfixia neonatal, distocia de hombro y lesin nerviosa,
hipoglucemia, hipocalcemia, ictericia, SIRPA, Taquipnea transitoria, cardiomiopata (hipertrofia
septal), eritrocitosis, trombosis, sobre todo de la vena renal. Cuando son adultos: obesidad, diabetes
mellitus, alteraciones neuropsicolgicas. En la madre; Preeclampsia, eclampsia, polihidramnios,
desgarros perineales, mayor nmero de cesreas, riesgo de padecer diabetes mellitus II, parto
prematuro. TRATAMIENTO: La meta teraputica de glucosa sangunea durante el embarazo es:
Glucemia central en ayuno entre 60-90mg/dl y menor de 140mg/dl una hora pospandrial o menor a 120mg/dl a las 2 h pospandrial. El
objetivo fundamental del tratamiento es mantener la normoglucemia con una dieta adecuada, reduciendo el consumo de
carbohidratos a 40-45% de la ingestin calrica total. Las pacientes con IMC superior a 30 kg/m2 de SC deben disminuir su ingestin
calrica en 30 a 33% o alrededor de 25 kcal/kg de peso corporal. El peligro de la restriccin calrica muy estricta en la embarazada es la
produccin de cetosis, que puede alterar el desarrollo psicomotor del feto. La actividad fsica adaptada a la fisiologa materna de
acuerdo con la edad gestacional. Si con los dos recursos teraputicos sealados no se obtiene un control adecuado, ser necesaria la
administracin de insulina en forma juiciosa para mantener cifras de glucosa dentro de lmites normales para la mujer embarazada. Se
debe evitar un tratamiento intenso que cause hipoglucemia, lo que ocurre hasta en 60% de las pacientes, en muchos casos
asintomtica para la madre, pero con diversas repercusiones para el feto. En pacientes con insulina la dieta fraccionada con una
colacin nocturna, disminuye la probabilidad de hipoglucemias. Los anlogos de insulina de accin rpida lispro y aspart son seguros en
el embarazo y tienen algunos beneficios comparados con la insulina rpida, como, menos episodios de hipoglucemia, un mejor control
de la glucemia postpandrial y mayor satisfaccin del paciente. La insulina NPH es tambin segura y se utiliza en combinacin con las
anteriores. El esquema de tratamiento con insulina puede consistir en: Se indica el fraccionamiento de la dosis de insulina en dos
aplicaciones cuando: 1) Continua glucemia en ayuno elevada a pesar de incremento de la dosis de insulina matutina. 2) Se requiere ms
de 20 U de insulina para lograr control metablico. 3) Se adiciona insulina rpida cuando no se alcanza la meta teraputica de la
glucemia pospandrial. Esquema: Combinacin de insulina rpida e intermedia administrada 30min antes del desayuno y de la cena. Los
requerimientos varan de 0.3-1.5 UI por kg de peso real y de acuerdo a descontrol glucmico, as como a la presencia de sobrepeso,
edad gestacional o enfermedades que modifiquen la glucemia como la hipermesis gravdica o infecciones. La dosis total calculada se
dividir en 2/3 pre-desayuno y 1/3 pre-cena (30 min antes de los alimentos). La razn ser NPH/rpida 2/1 pre-desayuno y 1/1 pre-
cena. Se recomienda iniciar con la mnima dosis de insulina e incrementarla gradualmente de acuerdo al automonitoreo con glucemias
capilares pre y postpandriales, por lo menos 3 veces al da. Los anlogos de insulina deben ser utilizados hasta que se disponga de
mayores datos o exista respuesta a las interrogantes que an existen acerca de su uso, como: teratogenicidad, incremento en el dao a
la retina o incluso a la formacin de anticuerpos y reactividad inmunolgica de la insulina. Slo dos de los frmacos hipoglucemiantes
orales han sido utilizadas: una es un secretagogo de insulina (glibenclamida) y otro un sensibilizador de insulina (metformina). Aunque
diversos grupos de expertos restringen su uso, los datos disponibles muestran que en algunas se logra un control adecuado, adems
que se encuentran dentro de la clasificacin C o D, lo que significa que nos son seguros. La elevacin posprandial de la glucosa causa
alteraciones en el feto. Un frmaco que puede utilizarse es la acarbosa, que se absorbe en muy poca cantidad y, segn algunos
trabajos, ha logrado mejor control de la glucemia posterior a la ingestin de alimentos. El control adecuado de la glucosa sangunea es
el objetivo ms importante del tratamiento de la diabetes mellitus gestacional y, en teora, este control ptimo disminuira la
morbilidad y mortalidad maternofetal. PREVENCION: La Federacin Internacional de Diabetes y la Asociacin Americana de diabetes
proponen los siguientes puntos para el consejo preconcepcional: 1) En toda mujer en edad frtil en etapa reproductiva con diabetes,
identificar anualmente por interrogatorio directo el deseo de embarazo, en caso de no desearlo proporcionar consejo sobre mtodo
anticonceptivo. 2) En la eta preconcepcional y de embriognesis, lograr un control glucmico estricto con una cifra ideal de Hb
glicosilada menor a 6.1%, evitando la presencia de hipoglucemia, para disminuir la probabilidad de malformaciones. 3) Con base, en
que a mayor dao vascular mayor riesgo de complicaciones materno-fetales, definir el riesgo establecido si existe dao por micro o
macroangiopatia utilizando la clasificacin de Priscilla White. 4) En caso de diabetes tipo I, determinar la funcin tiroidea asociada, ya
que hasta un 17% pueden cursar con esta co-morbilidad. 5) En el momento en que se establece el diagnostico de embarazo, suspender
hipoglucemiantes orales o insulina glargina si la paciente los estaba utilizando. 6) Suspender los IECA y ARA II, si la paciente los estaba
utilizando y usar alfametildopa, nifedipina, labetalol o hidralazina. 7) Mantener las cifras de tensin arterial por debajo de
130/80mmHg. 8) Suspender estatinas y fibratos. 9) Suspender el consumo de alcohol y tabaco. 10) Iniciar cido flico tres meses antes
del embarazo. No es recomendable el embarazo si la mujer diabtica presenta cualquiera de las siguientes situaciones: HbA1c >10%.
Cardiopata isqumica. Nefropata avanzada (depuracin de creatinina <30ml/min y Cr srica >1.4mg/dl). Retinopata proliferativa.

CASO CLINICO
Paciente femenino de 34 aos de edad la cual acude a consulta debido a que presenta amenorrea de 8 semanas de gestacion, agrega
que ha presentado leve fatiga, anorexia con nausea matinal, usted enva laboratorios de rutina presentando glucosa en sangre de 200
mg/dl y hemoglobina glucosilada de 7.5 refiere la paciente que fue diagnosticada con diabetes hace 5 aos por lo que recibe
Tolbutamida, adems recibe captopril por hipertensin arterial diagnosticada hace 3 aos. Al interrogatorio la paciente refiere
antecedentes de madre diabtica e hipertensa, sus antecedentes GO refiere menarca a 13 aos, gesta 3, para 2, abortos 0, adems
refiere que su embarazo anterior hace 5 aos presento un incremento de 14 kilogramos con producto que preso 3,750 grs. Por va
cesrea a las 36 semanas de gestacin.

PREGUNTA
Cul es la conducta menos adecuada a seguir.

RESPUESTA
a.- Suspender hipoglucemiente.
b.- Suspender IECA.
c.- Indicar interrupcin de embarazo.
MANUAL DE TRABAJO DEL CURSO ENARM CMN SIGLO XXI
CURSO ENARM CMN SIGLO XXI TEL: 36246001 Pharmed Solutions Institute PGINA 332

d.- Indicar alfametildopa.

CASO CLINICO
Paciente femenino de 21 aos de edad, la cual presenta 24 semanas de gestacin, acude a control prenatal, refiere ardor al orinar,
deseos continuos de orinar, con urgencia, usted identifico un incremento de peso hasta el momento a partir de su embarazo de 10
kilos. Sus antecedentes presentan, obesidad previa al embarazo, padre diabtico actualmente con insuficiencia renal en tratamiento
con hemodilisis, madre hipertensa bajo tratamiento.

PREGUNTA
Considerando el cuadro clnico cuales son las complicaciones que el producto puede presentar.

RESPUESTA
a.- Producto Macrosomico.
b.- Diestres respiratorio.
c.- Hipoglucemia.
d.- Parto pretermino.

PARTO PRETERMINO. CIENCIAS BASICAS: El diagnstico de parto pretrmino se obtiene en pacientes con 37 semanas (o un peso igual
a mayor de 500g, que respira o manifiesta signos de vida) y que presentan contracciones dolorosas y regulares que ocurren al menos
cada 10 minutos. Esto debe estar asociado con dilatacin cervical y/o descenso. El diagnstico se realiza al demostrar una dilatacin
progresiva del crvix debido a la presencia de contracciones entre las semanas 20-37, lo que incluye hasta la semana 37. La definicin
de puede dividir a saber: Pretrmino: 33-37 semanas. Pretrmino moderado: 28-32 semanas. Pretrmino extremo: 20-27 semanas.
SALUD PUBLICA: El parto pretrmino es la mayor causa de mortalidad perinatal en el mundo desarrollado y ocurre en
aproximadamente un 7% de todos los nacimientos. Los nios pretrmino presentan causas severas de morbilidad como dificultad
respiratoria del recin nacido, hemorragias intraventriculares, displasia broncopulmonar y enterocolitis necrotizante. Tambin se
pueden presentar complicaciones a largo plazo como parlisis cerebral, perdida visual y auditiva. Cerca de un 30% de los parto
pretrmino son idiopticos y espontneos. PATOGENIA: La incidencia del parto pretrmino es generalmente ms alta en los grupos
socioeconmicos bajos en comparacin con los grupos de mayor ingreso econmico. De manera que se han establecido relaciones
entre la incidencia de amenaza de parto, parto pretrmino y algunas condiciones asociadas con los problemas socioeconmicos, a
saber: Bajo cumplimiento de la consulta prenatal, edades maternas extremas (adolescentes y madre aosa), fumado, condiciones
laborales extremas, pobre educacin materna, estrs en la vida diaria, ansiedad relacionada al embarazo, violencia domstica. Ante la
presencia de un parto pretermino se debe buscar de forma intencionada la presencia de infecciones a travs de urocultivo y cultivo de
secrecin vaginal con el objetivo de especificar el uso de antibitico. Es necesario corroborar por USG la edad gestacional. Para
establecer el pronstico de sobrevida del neonato y sus posibles complicaciones. El factor de riesgo ms importante para predecir parto
pretermino es el antecedente de parto pretermino. Las pacientes con antecedente de parto pretermino, deben recibir consejo
preconcepcional o atencin medica temprana en el embarazo. DIAGNOSTICO: No hay datos clnicos especficos y la exploracin de
crvix es subjetiva e inexacta. Identifique los factores de riesgo de parto pretrmino en la historia clnica de las mujeres en control.
Algunos sntomas tempranos sugestivos de amenaza de parto pretrmino son: Dolor abdominal bajo y/o calambres, dolor lumbar,
presin plvica, flujo vaginal aumentado, manchado o sangrado. Sntomas definitivos son: actividad uterina regular acompaada de
borramiento y dilatacin del crvix. Criterios diagnsticos: A) actividad uterina regular entre las 20 y 37 semanas de gestacin ms
contracciones uterinas clnicamente documentadas (1/10 minutos, 4/20 minutos o 6/60 minutos). B) Dilatacin cervical igual o mayor a
2cm. C) Borramiento cervical igual o mayor a 80%. Laboratorio y gabinete: la evaluacin ultrasonografica de la longitud cervical y
prueba de fibronectina fetal tienen elevado valor predictivo negativo; por lo tanto, el uso nico o de ambas, permite determinar qu
tipo de pacientes no requieren tocolisis. En pacientes con riesgo de parto pretermino la medicin de la longitud cervical se realizara
entre la semana 20.1-34 de gestacin y fibronectina fetal entre las semanas de 24-34 de gestacin. La amniocentesis puede ser usada
en mujeres con amenaza de parto pretermino para valorar la madurez pulmonar fetal e infeccin intraamniotica. FIBRONECTINA
FETAL: La FNf es una glicoprotena producida por la membrana corinica y est localizada en la decidua basal adyacente al espacio
intervelloso. Su objetivo principal parece el de una molcula de adhesin (pegamiento), la cual favorece la unin entre las membranas
corionicas con la decidua materna contigua. Puede ser encontrada, normalmente, en SCV en las primeras 22 SDG. Virtualmente, nunca
es encontrada en la misma secrecin entre las 24 y 34 SDG, a menos que el crvix haya desarrollado borramiento y dilatacin
prematura, generalmente en asociacin con contracciones uterinas sintomaticas. Existe una fuerte asociacin entre la expresin de FNf
en SCV y PP.(1,2,7) Numerosos ensayos han mostrado una asociacin entre la presencia de FNf y nacimientos pretrmino, as como una
reduccin en el riesgo de nacimientos pretrmino cuando el resultado de esta prueba es negativo Los beneficios con el uso de la
prueba de FNf(1) son: Disminucin de ingresos hospitalarios y de la duracin de los das de hospitalizacin. Identificacin ms
apropiada de pacientes que necesitan esteroides y tratamiento con agentes tocolticos. Disminucin del uso de agentes tocolticos en
pacientes que no estn en riesgo de nacimiento pretrmino. Reduccin del estrs y ansiedad para la mujer embarazada y su familia
debido a la tranquilidad de no realizar una transportacin u hospitalizacin innecesaria. Las indicaciones para FNf son: embarazos de 24
a 34 SDG, sntomas de parto pretrmino (contracciones uterinas regulares >6 por hora y/o presin plvica) membranas amniticas
intactas, crvix <3 cm de dilatacin y bienestar fetal documentado. TRATAMIENTO: Los antibiticos no parecen prolongar la gestacin y
solo debe usarse para la profilaxis del estreptococo del grupo B en pacientes en las cuales el parto es inminente. No est definido un
tocolitico de primera eleccin para el manejo del parto pretermino. Las drogas tocoliticas pueden prolongar el embarazo de 2-7 das
permitiendo la administracin de esteroides para mejorar la madurez fetal y considerar la referencia de la madre a un tercer nivel. Usar
de terbutalina 0.25mg subcutnea cada 20min a 3 h controlando intervalos regulares de la presin arterial y la frecuencia cardiaca en la
madre, en conjunto con un control estricto de lquidos. Se recomienda el uso de indometacina 100mg rectal de dosisi inicial, con 25-
50mg VO o VR cada 4-6 horas por un mximo de 48h. Indicado si la paciente presenta enfermedad cardiovascular, hipertiroidismo, DM
o hipertensin. Solo utilizar en embarazos con menos de 32 SDG. La administracin de 2 dosis de 12mg de betametasona o
MANUAL DE TRABAJO DEL CURSO ENARM CMN SIGLO XXI
CURSO ENARM CMN SIGLO XXI TEL: 36246001 Pharmed Solutions Institute PGINA 333

dexametasona IM, con 12-24h de intervalo, se recomienda como esquema de induccin de madurez fetal. La edad gestacional
recomendada para la apliacion de corticoide para induccin de madurex pulmonar fetal en pacientes con riesgo de parto pretermino es
de 24-34 SDG. Los tocoliticos no deben utilizarse como terapia de mantenimiento de repeticin. Tratamiento no farmacolgico: El
reposo en cama y la hidratacin no parecen mejorar la incidencia de parto pretermino, por lo cual no debe ser una recomendacin
rutinaria. El internamiento prenatal para guardar reposo no ha mostrado ser efectivo en disminuir el parto pretermino y la morbilidad
perinatal, por lo cual se recomienda individualizar cada caso. El reposo absoluto puede presentar efectos adversos en las mujeres y
debe evitarse en la medida de lo posble. Sera motivo de referencia a tercer nivel, embarazos con fetos potencialmente viables (27-
32SDG), en caso que la unidad de referencia no cuente con unidad de terapia intensiva neonatal. Contraindicaciones de la inhibicin del
parto pretrmino; La inhibicin del parto pretrmino est contraindicada en las siguientes condiciones cuando el parto es inminente o
cuando otros factores obsttricos indican que el parto no debe ser retrasado, a saber: Retardo del crecimiento intrauterino, pre-
eclampsia fulminante, abrupto placentario severo, sufrimiento fetal, corioamnioitis severa ante una ruptura de membranas,
anormalidad mortal del feto, desarrollo de efectos adversos severos con el tratamiento. Evite inhibir el parto si est en el mejor inters
de la madre o el nio. Debe detenerse el tratamiento si el parto contina a pesar de la terapia. PREVENCION: En general se dice que
cerca del 50% de los partos pretrmino son potencialmente prevenibles. Algunas estrategias utilizadas para la reduccin de los factores
de riesgo incluyen estrategias basadas en la poblacin, a saber: Preparacin para el parto: estado nutricional, evitar el tabaco y drogas.
Ambiente de apoyo. Dar mayor poder a la mujer en su consulta prenatal, incluyendo apoyo continuado, adecuada respuesta a
preguntas e informacin para evitar factores de riesgo. Educacin al paciente sobre el reconocimiento de los sntomas tempranos
(dolor abdominal bajo, dolor lumbar, presin plvica, flujo vaginal aumentado, sangrado o manchado). Tratamiento de las infecciones
vaginales sintomticas antes de las 32 semanas. Tamizaje por vaginosis bacteriana en mujeres con antecedente de parto pretrmino.
Cerclaje cervical en mujeres con historia de incompetencia cervical.

CASO CLINICO
Se trata de paciente femenino de 29 aos de edad la cual acude a urgencias refiriendo contar con 34 semanas de gestacin por FUM,
no cuenta con cuidados prenatales, agrega que este es su tercer embarazo, previamente dos partos, agregando que no presenta
complicaciones en sus embarazos o partos previos, a la exploracin usted observa borramiento del 100% y dilatacin de 8 cm, decide
dar continuidad al trabajo de parto, ya que considera que en caso de enviarla a unidad de mayor complejidad colocara en riesgo al
binomio.

PREGUNTA
Considerando que la paciente cursa con un trabajo de parto pretermino, cual es la complicacin ms frecuente que el producto puede
presentar y por lo tanto tendr que enviar a un segundo nivel posterior a asistir el parto.

RESPUESTA
a.- Hemorragia Periventricular.
b.- Enfermedad de Membrana Hialina.
c.- Enterocolitis Necrotizante.
d.- Sepsis Neonatal.

CASO CLINICO
Paciente de 34 aos de edad, casada, con antecedentes de incompetencia cervical, colecistectoma laparoscpica debido a colecistitis
crnica litisica de cinco meses de evolucin y timectomizada a los 18 aos de edad por miastenia gravis, sin tratamiento farmacolgico
actual. Al momento de su estudio cursaba la vigsima cuarta semana del quinto embarazo. Los partos previos fueron vaginales,
menores de 25 semanas, de hijos nacidos vivos pero que fallecieron a las pocas horas de nacidos, el alumbramiento fue normal, con
una placenta discoide, a la revisin se obserrvaron dos zonas discoides, hemorrgicas adems signos de envejecimiento placentario.

PREGUNTA
Cual de las siguientes medidas presentan mayor efecto para modificar el curso de esta entidad?

RESPUESTA
a.- Cerclaje temprano.
b.- Reposo absoluto.
c.- Antibiotico profilctico.
d.- Progesterona micronizada.

PARTO NORMAL. CIENCIAS BASICAS: Se define como aquel de comienzo espontneo, de bajo riesgo desde el comienzo del trabajo de
parto, hasta la finalizacin del nacimiento el nio nace en forma espontnea, en presentacin ceflica, entre las 37-41 SDG. Luego tanto
la madre como el nio estn en buenas condiciones. Presentacin: parte del feto que se presenta al canal del parto y es capaz de
desencadenar el TDP. El 95 % de las presentaciones son ceflicas, 4 % son presentaciones podlicas o pelvianas, que pueden ser de
cara, frente y muy rara, de hombros. Situacin: relacin que existe entre el eje axial del feto y el eje del tero, por lo que existen dos
tipos de situacin: longitudinal y transversa. Posicin fetal de una presentacin particular: relacin existente entre el dorso del feto y el
lado derecho o izquierdo de la madre. En la variedad de posicin en cada tipo de presentacin va a tener un punto de referencia. En la
presentacin ceflica el punto de referencia es el occipucio; en la presentacin de cara es el mentn y en la pelviana o podlica es el
sacro. TRABAJO DE PARTO (TDP): Por definicin, el comienzo es espontneo, por lo tanto se inicia en la casa, y es la propia mujer y su
familia quienes controlan sus condiciones y evolucin durante las primeras etapas. Causas que desencadenan el TDP; 1. Musculares. 2.
Hormonales. 3. Nerviosas. 4. Placentarias. 5. Fetales. Reflejo de Ferguson 1: La distensin progresiva del tero estimula los receptores
locales que por va neuronal estimulan los ncleos supraopticos y paraventriculares que liberaran oxitocina. Reflejo de Ferguson 2: La
MANUAL DE TRABAJO DEL CURSO ENARM CMN SIGLO XXI
CURSO ENARM CMN SIGLO XXI TEL: 36246001 Pharmed Solutions Institute PGINA 334

mayor distencin del segmento inferior y el borramiento y dilatacin cervical llevara nuevos estmulos reflejos hacia la neurohipofisis lo
que estimulara la secrecin de oxitocina. FASES DE TDP: FASE 1 LATENTE: Ocurre al final del embarazo hasta el inicio de las
contracciones del TDP. FASE 2 ACTIVA: Se compone de 3 perodos: 1) Perodo de dilatacin: comprende desde el comienzo de la labor
del parto hasta la dilatacin completa (10 cm). Es la fase ms larga del TDP y tiene una duracin de 8-12 h en las nulparas y de 6-8h en
las multparas. Consta de 2 fases: A) Fase latente: comienza con las contracciones y se prolonga hasta los 2,5 cm de dilatacin; es de
unas 8 horas de duracin y no debe pasar de 20 horas en nulparas y de 14 en multparas. B) Fase activa: fase de aceleracin inicial,
fase de mxima inclinacin, fase de desaceleracin. 2) Perodo de expulsin: comprendido desde la dilatacin completa hasta el
nacimiento del beb y vara desde pocos minutos hasta 2h, sobre todo en las nulparas. 3) Perodo de alumbramiento: transcurre desde
el nacimiento hasta la expulsin de la placenta, y se espera que esto ocurra en un perodo de 30 a 40 min. Algunos plantean un cuarto
perodo que sera la hora despus del alumbramiento en que existe gran peligro de que ocurra una hemorragia; pero la mayora asume
que ste sera la prolongacin del tercer perodo. Es difcil determinar en muchas ocasiones cundo comenz la labor de parto, pero
por regla general esto ocurre cuando las contracciones tienen una frecuencia cada 2 3 min de aparicin entre ellas y duran de 30 a
45seg, y como resultado existen modificaciones del cuello uterino. Los 3 factores que contribuyen a que las contracciones borren y
dilaten el cuello son: 1. La presin hidrosttica de la bolsa de las aguas. 2. La traccin que sobre las fibras del cuello uterino ejercen las
del cuerpo. 3. La dilatacin activa por la disposicin espiral de las fibras musculares del tero. La bolsa de las aguas, impulsada por la
presin hidrosttica, se introduce en el orificio cervical interno al comienzo de la dilatacin, y al mismo tiempo expulsa el tapn mucoso
endocervical. El sangramiento que ocurre en un parto normal se encuentra entre los 400 a 500 mL. Se plantea que despus del
alumbramiento el tero se contrae y se retrae para mantener la hemostasia por medio del estrechamiento de las fibras musculares que
comprimen los vasos venosos, a lo que se ha denominado ligaduras vivientes de Pinard. CONTRACCIONES UTERINAS: La medida de la
contraccin uterina y su registro grfico tiene mucha importancia en la clnica obsttrica moderna, porque constituyen la base de un
control ms cientfico del seguimiento del TDP, que recibe el nombre de monitorizacin obsttrica. Se ha determinado que la
intensidad de las contracciones en el parto normal es de 30-50 mmHg y el tono muscular normal est comprendido entre 8 y 12 mm.
Antes de las 30 SDG: Tipo A, poca intensidad y limitado a pocas reas del tero (lvarez). Tipo B, Braxton Hicks mayor intensidad, baja
frecuencia y se propagan. Preparto en ultimas semanas: Aumento de las contracciones de Braxton Hicks, se caracteriza por la
maduracion progresiva del cuello uterino. Cuando el cuello presenta 2cm de dilatacin o ms, la onda contrctil normal comienza en las
zonas denominadas marcapasos ubicadas cerca de la implantacin de las trompas y se caracteriza por el triple gradiente descendente
(TGD). 1. Sentido descendente 2. Duracin: disminuye a medida que se aleja de los marcapasos 3. La intensidad disminuye a medida
que se aleja de los marcapasos. Clnica de las contracciones durante el TDP: Frecuencia: comienzo c/ 20 o 25 minutos final c/ 2-3
minutos. Duracin: Comienzo 15 a 20 segundos final 40-45seg, intensidad: Aumento a medida TDP, hay dolor: por la dilatacin y
contracciones. MECANISMO DEL PARTO: En la mayora de los partos se realiza por parte del objeto del parto (el feto) una serie de
evoluciones que van a ir sorteando los diferentes dimetros y curvatura de la pelvis sea y de las partes blandas del canal, y que tendr
su mecanismo bien establecido en cada una de las presentaciones. Por ser la presentacin de vrtice la ms frecuente, describiremos
estos mecanismos: 1) Acomodacin Estrecho Superior: para facilitar su entrada en la excavacin plvica, la cabeza se reduce por flexin
y cabalgamiento de los huesos del crneo y orienta sus dimetros mayores anteroposteriores hacia el dimetro mayor del estrecho
superior. Al comienzo del parto la cabeza colocada transversalmente se halla en actitud indiferente y presenta su dimetro
frontooccipital de 12 cm al estrecho superior de la pelvis. En virtud de las contracciones uterinas y de la resistencia que ofrece el
estrecho superior, la cabeza se flexiona y sustituye su dimetro anteroposterior por otro menor, el suboccipitofrontal de 10,5 cm. 2)
Encaje y descenso: La contraccin uterina hace que la cabeza realice un movimiento de lateroflexin, en virtud del cual el parietal
posterior desciende a la excavacin pelviana y rebasa el promontorio, mientras el parietal anterior est detenido al nivel del pubis
(asinclitismo posterior). Al descender la cabeza por debajo del promontorio y alojarse el parietal posterior en la concavidad del sacro,
permite el descenso a su vez del parietal anterior hacia la cavidad pelviana. Cuando ambos parietales han franqueado el estrecho
superior de la pelvis se puede decir que la cabeza se ha encajado. La cabeza tambin puede encajarse en asinclitismo anterior e incluso
sinclticamente. 3) Rotacin interna: Al llegar aqu se encuentra con una situacin completamente distinta: los dimetros mayores del
estrecho inferior son los anteroposteriores, por lo cual tiene que realizar una rotacin de 90 para acoplar as sus dimetros mayores
(anteroposteriores) a los del estrecho inferior que tambin son los anteroposteriores. 4) Desprendimiento: La cabeza tiene que vencer
la resistencia del perineo y de la extremidad inferior del sacro y del coccix. El sacro puede realizar movimientos muy limitados, no
mayores que 5 a 6 mm en su parte inferior (nutacin y contranutacin del sacro); sin embargo, el cccix puede retropulsar 30-40mm la
cabeza va dilatando el anillo vulvar y aparecen, en secuencia, las eminencias frontales, la raz de la nariz, la boca, y, por ltimo, el
mentn, hasta completar su desprendimiento total, mientras la cabeza se est desprendiendo en sentido anteroposterior, los hombros,
con dimetro biacromial y situados en el estrecho superior, van descendiendo, en la excavacin pelviana (encajamiento de los
hombros). 5) Rotacin externa: Los hombros, que han llegado en transversa al estrecho inferior, tienen que realizar un movimiento de
rotacin interna de 90 para llevar su dimetro mayor biacromial al dimetro mayor anteroposterior del estrecho inferior. Los
hombros, al realizar su rotacin interna en el estrecho inferior, hacen que la cabeza, fuera de la vulva y sin nada que la obstaculice,
realice su rotacin externa. Sexto tiempo: expulsin de los hombros y deslizamiento del resto del cuerpo. Una vez expulsados los
miembros inferiores del feto por la hendidura vulvar, termina el mecanismo del parto. PERODO DE EXPULSIN: Cuando la dilatacin
se ha completado y la cabeza llega al plano perineal, la parturienta experimenta deseos de pujar en cada contraccin y se abomba el
perineo, por lo que sabemos que la paciente est en perodo expulsivo. Se colocar a la paciente en posicin ginecolgica en una mesa
de partos. Auscultar frecuencia cardaca fetal. Asepsia del perineo, la vulva y la raz de los muslos. Colocarn los paos estriles y
medias ginecolgicas. Asepsia del mdico (manos). Se vestir con bata estril, gorro, tapabocas, botas, guantes estriles. La mano
izquierda se coloca sobre el saliente de la cabeza y con la mano derecha se cogen, entre el pulgar por un lado y los dedos ndice y medio
por el otro, las eminencias frontales a travs del perineo tenso. Con ambas manos se controla la salida de la cabeza para evitar que sta
lo haga bruscamente y produzca un desgarro perineal. Aunque la proteccin del perineo es conveniente, no debe prolongarse
exageradamente. En estos casos se realizar su infiltracin local con lidocaina a 1% u otro anstesico similar y se proceder a realizar la
episiotoma mediolateral derecha o izquierda. La episiotoma siempre es ms ventajosa que un desgarro perineal (no abusar de este
proceder). Una vez expulsada la cabeza fetal, se aspiran las flemas de las fosas nasales de la orofaringe mientras se va produciendo
espontneamente el resto del mecanismo del parto. Si existiera alguna circular del cordn, se libera. Despus que dejamos de sentir las
MANUAL DE TRABAJO DEL CURSO ENARM CMN SIGLO XXI
CURSO ENARM CMN SIGLO XXI TEL: 36246001 Pharmed Solutions Institute PGINA 335

pulsaciones en el cordn umbilical, se pinza ste con tres pinzas de Kocher y se secciona entre las dos ms prximas. ATENCIN DEL
PERODO DE ALUMBRAMIENTO: Una vez terminado el perodo de expulsin, debe asistirse con especial atencin al alumbramiento. Se
observarn los signos clnicos del desprendimiento placentario: 1. Elevacin del tero por encima del ombligo y desviacin hacia la
derecha; esto se puede observar a travs de las cubiertas abdominales Signo de Kstner, el cual consiste en el ascenso del cordn
umbilical al colocar nuestra mano por encima de la snfisis del pubis y rechazar el tero hacia arriba, lo que significa que la placenta an
est adherida al tero. En caso de que la placenta se haya desprendido, el cordn no se mover. 3. Descenso espontneo de la pinza
colocada en el cordn al nivel de la vulva. El desprendimiento placentario por lo general ocurre espontneamente en los 10 min
siguientes a la expulsin del recin nacido, pero se debe esperar hasta 30 min antes de actuar. Una vez desprendida la placenta. Una
vez expulsada la placenta se debe evitar el desgarro de las membranas, para lo cual se le imprimen movimientos de torsin a la
placenta (maniobra de Dublin), de manera que las membranas se hagan ms resistentes a los desgarros y vayan saliendo lentamente,
pero ntegras. Despus de extrada la placenta se procede a su revisin.

CASO CLINICO
Se trata de femenino de 29 aos de edad, la cual acude a urgencias debido a que desde hace 10 horas inicia con contracciones
dolorosas, la paciente cuenta con antecedentes GO gesta 3, para 2, abortos 0, a la exploracin fsica se observa producto ceflico,
occipito anterior derecho identificado por palpacin abdominal y tacto vaginal, se observa a nivel de segundo plano, se encuentra 100
% de borramiento y dilatacin de 7 cm, la actividad uterina se observa 3 a 4 contracciones cada 10 minutos con duracin de 50 a 60
segundos, durante la exploracin observa presencia de cordon umblical y liquido amnitico de caractersticas adecuadas, el producto
presenta frecuencia cardiaca de 150 con descensos variables durante la contraccin de menos 20 latidos.

PREGUNTA
Considerando el cuadro clnico, cual es la conducta a seguir.

RESPUESTA
a.- Realizar Bloqueo y Realizar Cesarea.
b.- Realizar Bloqueo y Continuar Trabajo de Parto.
c.- Continuar Trabajo de Parto.
d.- Indicar Tocoliticos.

CASO CLINICO
Se trata de femenino de 19 aos de edad la cual es originaria del Estado de Mxico la cual acude a urgencias, refiere que desde hace 6
horas presenta dolor abdominal los cuales son espordicos, aproximadamente cada 10 minutos, de intensidad moderada, agrega que
desde hace 4 horas arrojo moco con sangre por va vaginal, durante la exploracin se encuentra cuello cervical reblandecido, con un
centmetro de dilatacin con producto libre, usted observa presencia de secrecin verdosa con olor desagradable, la madre refiere que
ha presentado cuadro repetidos de infecciones cervicovaginales con presencia de VPH por colposcopia reportado a las 18 semanas de
gestacin, sin tratamiento, actualmente cuenta con 40 semanas de gestacin.

PREGUNTA
Considerando el cuadro actual, cual es la conducta a seguir:

RESPUESTA
a.- Esperar inicio de trabajo de parto.
b.- Inducir trabajo de parto.
c.- Prepara a la paciente para cesrea.
d.- Conducir el trabajo de parto.

CASO CLINICO
Se trata de paciente femenino de 32 aos de edad la cual acude por tercera vez a consulta debido a que se encuentra embarazada y no
se ha sentido bien, cuenta con antecedente de G; 3 Para; 2 Abortos 0; al interrogatorio se identifica 42 semanas de gestacin por FUM,
sin embargo no hay inicio espontaneo de trabajo de parto, por lo indica USG.

PREGUNTA
Cul es el factor ms importante que busca para tomar una decisin quirrgica.

RESPUESTA
a.- Calcificacin placentaria.
b.- Caractersticas del lquido amnitico.
c.- Cantidad de lquido amnitico.
d.- Caractersticas del cordn umbilical.

CESAREA. CIENCIAS BASICAS: La cesrea es la intervencin quirrgica que tiene como objetivo extraer el producto de la concepcin y
sus anexos ovulares a travs de una laparotoma e incisin de la pared uterina. TIPOS DE CESREA: A) Segn antecedentes obsttricos
de la paciente: Primera: Es la que se realiza por primera vez. Iterativa: Es la que se practica en una paciente con antecedentes de dos o
ms cesreas. Previa: es cuando existe el antecedente de una cesrea anterior. B) Segn indicaciones: Urgente: Es la que se practica
para resolver o prevenir una complicacin materna o fetal en etapa crtica. Electiva: Es la que se programa para ser realizada en una
fecha determinada por alguna indicacin mdica y se ejecuta antes de que inicie el trabajo de parto. C) Segn tcnica quirrgica:
MANUAL DE TRABAJO DEL CURSO ENARM CMN SIGLO XXI
CURSO ENARM CMN SIGLO XXI TEL: 36246001 Pharmed Solutions Institute PGINA 336

Corporal o clsica. Segmento corporal (Tipo Beck). Segmento arciforme (Tipo Kerr). CORPORAL O CLASICA: La incisin es vertical se
realiza en el cuerpo uterino. Sus indicaciones ms frecuentes son: cncer crvico-uterino invasor, embarazo pretrmino, situacin fetal
transversa con dorso inferior, histerorrafia corporal previa, procesos adherenciales o varicosos importantes en el segmento inferior,
placenta previa en cara anterior, cesrea posmortem, miomatosis uterina de grandes elementos y cuando despus de la cesrea se
realizar una histerectoma. Sus desventajas son: apertura y cierre ms difcil, mayor hemorragia, adherencias ms frecuentes,
histerorrafia menos resistente que puede hacerse dehiscente durante un nuevo embarazo. SEGMENTO CORPORAL (Beck): La incisin
es vertical y se realiza sobre el segmento y parte del cuerpo uterino. Sus principales indicaciones son: embarazo pretrmino, embarazo
gemelar, situacin fetal transversa con dorso inferior, presentacin plvica, placenta previa en la cara anterior del tero, anillo de
retraccin e histerorrafias corporales previas. Las desventajas de esta tcnica no difieren de la anterior. SEGMENTO ARCIFORME O
TRANSVERSAL (Kerr): Es la tcnica quirrgica ms usada por sus mltiples ventajas. Al realizarse una incisin transversal del segmento
inferior tiene las ventajas de producir menos hemorragia, y permitir una fcil apertura y cierre de la pared uterina, formacin de cicatriz
uterina muy resistente con poca probabilidad de dehiscencia y ruptura en embarazos subsecuentes y as como pocas adherencias
postoperatorias. INDICACIONES DE LA OPERACIN CESREA: La indicacin para realizar una operacin cesrea puede ser por causas:
Maternas, Fetales o Mixtas. A) Causas maternas; 1. Distocia de partes seas (desproporcin cefaloplvica): Estrechez plvica. Pelvis
asimtrica o deformada. Tumores seos de la pelvis. 2. Distocia de partes blandas: Malformaciones congnitas. Tumores del cuerpo o
segmento uterino, crvix, vagina y vulva que obstruyen el conducto del parto. Ciruga previa del segmento y/o cuerpo uterino,
incluyendo operaciones cesreas previas. Ciruga previa del cervix, vagina y vulva que interfiere con el progreso adecuado del trabajo
del parto. 3. Distocia de la contraccin: Hemorragia (placenta previa o desprendimiento prematuro de la placenta normoinserta).
Patologa materna incluyendo nefropatias, cardiopatas, hipertensin arterial o diabetes mellitus, etc. B) Causas fetales: Macrosoma
fetal que condiciona desproporcin cefaloplvica. Alteraciones de la situacin, presentacin o actitud fetal. Prolapso de cordn
umbilical. Sufrimiento fetal. Malformaciones fetales incompatibles con el parto. Embarazo prolongado con contraindicacin para parto
vaginal. Cesrea posmortem. C) Causas mixtas: Sndrome de desproporcin cefaloplvica. Preeclampsia/eclampsia. Embarazo mltiple.
Infeccin amnitica. Isoinmunizacin materno-fetal. INDICACIONES MS FRECUENTES: Las principales indicaciones de la operacin
cesrea son: 1. Desproporcin cefaloplvica: La desproporcin cefaloplvica ocurre en las siguientes circunstancias: Pelvis estrecha,
deformada u obstruida y feto normal con buena actitud. Pelvis normal y feto grande, anmalo (gigantismo o hidrocefalia) o con mala
actitud. Pelvis y feto normales con dimetros fetoplvicos limtrofes, despus de una prueba de trabajo de parto infructuoso. El
diagnstico de la DCP se establece mediante una valoracin clnica adecuada de los estrechos superior, medio e inferior de la pelvis
materna y su relacin con el feto, con el apoyo de estudios de inmagenologia. Ante la duda diagnstica se deber utilizar la prueba de
trabajo de parto. 2. Cesrea previa: la estrategia de permitir un parto vaginal a pacientes con cesrea previa bajo condiciones
controladas, logra un incremento en los nacimientos por va vaginal sin complicaciones. Contraindicaciones para permitir el parto
vaginal en mujeres con cesrea previa: Antecedente de dos o ms cesreas previas segmento arciforme Antecedentes de cesrea
previa tipo segmento-corporal, corporal o presencia de otras cirugas uterinas. Antecedente de cesrea previa reciente (menor a 6
meses). Antecedentes de ruptura uterina o dehiscencia de histerorrafia. Pelvis no til clnicamente 3. Sufrimiento fetal: Los factores
causales del sufrimiento fetal pueden ser reversibles (hipotensin materna, efecto Poseiro, hipercontractilidad por uso de oxitcicos), o
irreversibles (causa fetal, placentaria o del cordn umbilical). Se considerarn los siguientes parmetros para evaluar la frecuencia
cardaca: Basal (normal): 120-160 lpm. Bradicardia: leve: entre 110-119 lpm, moderada: entre 100-109 lpm, severa: por debajo de 100
lpm. Taquicardia: leve: entre 160-169 lpm, moderada: 170-179 lpm, severa: mayor de 180 lpm. En las unidades de atencin que
disponen de equipos de cardiotocografa se registrar la frecuencia cardaca fetal y su relacin con las contracciones uterinas. Las
principales alteraciones que pueden observarse son: Bradicardia menor de 100 lpm. mantenida durante 5 minutos o ms. Presencia de
desaceleraciones tardas o dips tipo II en el 30% o ms de las contracciones. Desaceleraciones variables graves (menos de 70 lpm de
ms de 60 seg. de duracin) en el 50% de las contracciones. Variabilidad de la curva de 5 lpm o menos por ms de 30 minutos.
Taquicardia mayor de 170 lpm que persisten por ms de 10 min. Patrn sinusoidal (con amplitud menor de 5 lpm), sin observarse
movimientos fetales, ni aceleraciones. Se considerar la operacin cesrea, cuando exista la asociacin de dos o ms de las alteraciones
de la frecuencia cardaca fetal antes sealadas. 4. Ruptura prematura de membranas: Es la salida de lquido amnitico a travs de una
solucin de continuidad de las membranas ovulares en embarazos mayores de 20 semanas y/o por lo menos 2h antes del inicio del
trabajo de parto. La conducta obsttrica en los casos de ruptura prematura de membranas, es la siguiente: a) Menor de 26 semanas de
gestacin: interrupcin del embarazo por va vaginal con el consentimiento de la mujer. b) Entre 26 y 28 semanas de gestacin: cuando
se cuente con los recursos mnimos para la atencin del neonato, se realizar operacin cesrea. c) Entre 29 y 32 semanas de
gestacin: se realiza operacin cesrea. d) Entre las semanas 33 y 36 de gestacin: se valorarn las condiciones obsttricas y se
resolver el embarazo por la va ms favorable. 5) Presentacin plvica: ocurre en el 3-4 % del total de los embarazos. En la mayora de
las unidades obsttricas, esta modalidad de presentacin constituye una indicacin de cesrea (excepto cuando la paciente acude en
periodo expulsivo). ANTIBIOTICOTERAPIA EN LA OPERACIN CESREA: Como en otras intervenciones quirrgicas, la realizacin de la
operacin cesrea conlleva riesgo de infeccin; la frecuencia y severidad de las infecciones depende de las condiciones en las que se
realiza y con base en ello, puede considerarse como: a) Limpia: Se considera que una operacin cesrea es limpia cuando se realiza en
condiciones aspticas, sin defectos de la tcnica, ni lesiones en los aparatos gastrointestinal o urinario. No hay reaccin inflamatoria en
la vecindad y la cavidad uterina no est contaminada. En sta no hay necesidad de utilizar antibiticos. b) Contaminada: Se considera
de este modo cuando existen: Trabajo de parto de ms de 6h. Ms de 6 tactos vaginales. Membranas rotas entre 6 y 24 hrs.
Antecedentes de aminiocentesis o instrumentaciones. Alto grado de dificultad en la tcnica quirrgica. Anemia severa (Hb. menor de 9
mg /dl). Lquido amnitico meconial. Se utilizarn antibiticos de manera profilctica desde el momento en el que se realiza el
pinzamiento del cordn umbilical. c) Sptica: Cuando hay evidencia de infeccin clnica, supuracin o presencia de materia fecal; en
esta categora se incluyen las cesreas con uno o ms de los siguientes factores: Cuadro febril. Lquido amnitico ftido o caliente.
Ruptura de las membranas ovulares de ms de 24h. Cesreas con alto grado de dificultad en la tcnica quirrgica. La antibioticoterapia
se mantendr por va endovenosa 72h y se completar el esquema segn la evolucin de la paciente. REQUISITOS PREOPERATORIOS:
Antes de la operacin cesrea debern cubrirse los siguientes requisitos preoperatorios: Historia clnica perinatal completa, incluyendo
exmenes de laboratorio y gabinete. Consentimiento informado de la mujer o de un familiar responsable, y firma de la autorizacin
para realizar la operacin cesrea. En el caso de la oclusin tubaria bilateral, por ser un mtodo anticonceptivo permanente se requiere
MANUAL DE TRABAJO DEL CURSO ENARM CMN SIGLO XXI
CURSO ENARM CMN SIGLO XXI TEL: 36246001 Pharmed Solutions Institute PGINA 337

adems la firma de consentimiento de la paciente en el formato institucional especfico. Nota preoperatoria en el expediente clnico en
la que se fundamente la indicacin de la operacin cesrea. CUIDADOS PREOPERATORIOS Y POSTOPERATORIOS: Preoperatorios
incluyen: Valoracin preanestsica de acuerdo a la NOM-170-SSAl-1998, Para la Prctica de Anestesiologa. Ayuno preoperatorio de 8h
o ms (cuando el caso lo permita). Rasurado suprapbico y aseo completo, con especial nfasis en la vulva y perineo. Colocacin de
sonda vesical. Disponibilidad de sangre segura y compatible. Postoperatorios consistirn en: Medicin horaria de signos vitales durante
las primeras cuatro horas y posteriormente cada 8h hasta el alta de la paciente. Vigilancia de la diuresis durante las primeras 12h y del
inicio de la miccin espontnea, despus de retirar la sonda vesical. Administracin de lquidos por va oral despus de 8h; una vez
iniciado el peristaltismo intestinal se indicar dieta blanda.

CASO CLINICO
Acude a consulta paciente femenino gestante con 39 semanas de gestacin por USG la cual refiere tener 12 horas con dolor en regin
abdominal y lumbo-sacra, agrega que cuenta con antecedentes de gesta 4 para 2 cesrea 1, aborto 0, a la exploracin identifica
producto podlico con borramiento del 30 % y dilatacin de 2 cm, la paciente cuenta con antecedentes de polidramnios en embarazo
previo con producto con atresia gastroesofagica con muerte perinatal, enva a su casa y cita en 6 horas, a la exploracin nuevamente
observa a la paciente la cual se encuentra fatigada, se observa actividad uterina con 3 contracciones en 10 minutos aproximadamente,
con duracin de 50 segundos, la paciente refiere ruptura de membranas con presencia de liquido verdoso, usted explora al producto el
cual se encuentra transversal con frecuencia cardiaca de 100 a 120 lpm.

PREGUNTA
Cul es su conducta ms adecuada a seguir.

RESPUESTA
a.- Prepara a la paciente para cesarea.
b.- Prepara a la paciente para conduccin.
c.- Espera a que el trabajo de parto continue.
d.- Inicia manejo con tocolitico.

COMPLICACIONES DEL PUERPERIO. CIENCIAS BASICAS: Se entiende por puerperio al periodo de tiempo que sigue al momento del
parto durante el cual los cambios fisiolgicos del embarazo, trabajo de parto y parto se resuelven y la mujer retorna al estado
pregravdico y en la glndula mamaria se producen una serie de cambios fisiolgicos que permiten la lactancia. Usualmente este
periodo de tiempo es de 6 semanas. El puerperio es un periodo con gran nmero de particularidades, donde pueden quedar
enmascaradas serias complicaciones por hechos fisiolgicos, por lo cual se debe distinguir bien entre los lmites del puerperio
normal y los signos o sntomas que deben alertar. En el puerperio inmediato se aconsejan las siguientes acciones, que pueden disminuir
el riesgo de complicaciones: Vigilancia de la adecuada involucin uterina; masaje uterino. Lactancia precoz, limpieza del pezn. Insistir
en la higiene adecuada de la episiorrafia o de la herida quirrgica si la paciente tuvo una cesrea. Deambulacin precoz (en las primeras
24 horas) para disminuir la posibilidad de presentacin de eventos tromboemblicos y mantener activo el trnsito intestinal y vesical.
La paciente puede iniciar su alimentacin idealmente 2 horas despus de un parto vaginal; no antes porque dentro de este lapso
pueden ocurrir las complicaciones inmediatas del posparto que requieran anestesia. La revisin de la paciente en puerperio se puede
realizar a la semana del posparto, cuando se puede identificar cualquier anomala e iniciar el mtodo de planificacin familiar.
CLASIFICACION: Cronolgicamente se divide en: Puerperio inmediato: primeras 24h posparto. Puerperio mediato: desde el segundo
hasta el dcimo da posparto. Puerperio tardo: desde el da 11 hasta el da 42 posparto. HEMORRAGIA POSPARTO: Se define como una
prdida sangunea mayor de 500cc luego del parto vaginal y 1000cc luego de la cesrea. De una manera objetiva ha sido descrito como
la disminucin del hematocrito en 10% o ms en comparacin con el valor inicial. Su diagnstico se debe realizar de una manera
oportuna para darle un manejo adecuado. La frecuencia de la hemorragia posparto es de 3% 4% en las pacientes que han tenido un
parto vaginal, hasta 9% en quienes han tenido cesrea. Todo esto aumenta si est asociado a factores de riesgo. Usualmente se clasifica
en dos tipos: Hemorragia posparto inmediata: se llama as la que tiene lugar en las primeras 24 horas del puerperio; la mayora ocurre
en las primeras 2 horas. sta es la hemorragia ms frecuente y la ms importante. Hemorragia posparto tarda: Se presenta a partir del
segundo da del puerperio hasta 6 semanas despus del parto. La frecuencia oscila entre 5 a 15 %. A. HEMORRAGIA TEMPRANA: 1.
RETENCIN DE PLACENTA: El principal mecanismo del alumbramiento est dado por la contraccin uterina y la formacin del
hematoma retroplacentario. El tiempo recomendado para que se produzcan los mecanismos del alumbramiento es de 15 a 20 minutos;
ms tiempo se considera retencin de la placenta. El manejo activo del alumbramiento se ha considerado una manera adecuada para
evitarla (se realiza la maniobra de Brandt: el descenso de la pinza que lig el cordn indica cuando empieza a desprenderse la placenta),
uso de oxitcico desde el tercer estadio del parto. 2. PLACENTA ANORMALMENTE ADHERIDA: Es un diagnostico que se realiza desde el
tercer trimestre o en el momento del parto. Consiste en una adherenciaanormal de las vellosidades a la pared uterina; existe una
deficiencia o ausencia de la decidua basal, y es difcil que se realice el alumbramiento normal, con la consecuente retencin de
placenta. Se obtiene una placenta con sangrado de difcil control y precoz. Clasificacin: creta: la placenta se encuentra adherida al
endometrio ncreta: penetran las vellosidades en el miometrio Prcreta: la invasin alcanza la serosa peritoneal y puede llevar a
ruptura del tero. ATONIA UTERINA: Se define por la inadecuada involucin uterina, por la no contraccin miometrial y cambios
vasculares intramiometrales y ausencia de un adecuado sistema coagulacin que permita su control. Es la responsable del 80 % de
todas las hemorragias posparto, considerando que en el embarazo a trmino la suplencia sangunea de la placenta es de
aproximadamente 600 a 800 mL/min. Se debe estar preparado para realizar esta reposicin de la volemia y desencadenar los
mecanismos de adaptacin materna. Factores de riesgo: Durante el embarazo: Multigestantes, infeccin intraamniotica, fibromas
uterinos, sobredistencion uterina, polihidramnios. Macrosomia fetal. Durante el trabajo de parto: Trabajo de parto precipitado, trabajo
de parto prolongado. Relajacin uterina por medicamentos como sulfato de magnesio, halogenados anestsicos, nitroglicerina.
Hemorragia del tercer trimestre (diagnstico final del embarazo) Tratamiento: 1. Inicialmente reemplazar la volemia con soluciones
cristaloides y dependiendo del volumen de sangre perdido y el estado clnico de la paciente, administrar glbulos rojos empacados. 2.
MANUAL DE TRABAJO DEL CURSO ENARM CMN SIGLO XXI
CURSO ENARM CMN SIGLO XXI TEL: 36246001 Pharmed Solutions Institute PGINA 338

Cuando la hemorragia se presenta antes de que se expulse la placenta se debe realizar un alumbramiento artificial. 3. Si ya ha salido la
placenta, entonces se procede a revisar el tero y el canal de parto. 4. La aplicacin de tero-tnicos: Oxitocina 10 a 20 U en infusin
continua. Los efectos adversos son poco: dolor con las contracciones, nausea, vmito, en casos extremos intoxicacin hdrica.
Prostaglandinas. Disponible aplicacin de Misoprostol (anlogo sinttico de la prostaglandina E 1). Dosis: 5 tabletas de 200
microgramos por va rectal, con muy buenos resultados reportados en la literatura. Efectos adversos: hipertermia transitoria,
broncoespasmo. Vasopresina: efecto vasoconstrictor: en casos extremos de no respuesta y si est disponible. Inyeccin directa no
vascular de 0.2 unidades (dilucin de 20 unidades en solucin salina 10occ, aplicar 1 cc ). INVERSIN UTERINA: Es la invaginacin del
tero sobre s mismo. El fondo del tero es impulsado hacia abajo y la parte superior de la cavidad endometrial se expone a travs del
cuello uterino y aparece en los genitales externos. El control y manejo incorrecto del alumbramiento es el principal factor
desencadenante de esta complicacin. Es muy raro que ocurra espontneamente. Se produce al traccionar en forma brusca y sostenida
el cordn umbilical, combinado con la presin sobre el fondo uterino, tratando de abreviar el tercer perodo del parto. Su presentacin
es sbita, acompaada de la hemorragia, dolor plvico intenso, hasta el shock neurognico por la gran tensin de todo el complejo de
ligamentos de sostn uterino hasta la zona genital externa. Cuando ocurre inmediatamente despus de la expulsin de la placenta se
debe tratar: Colocar el tero en forma rpida de una manera anatmica. Introducir una mano en la vagina y hacer presin suave y
directa hacia arriba en el mismo eje del estrecho plvico superior. Aplicar Clorhidrato de Ketamina IV por parte del anestesilogo para
una relajacin materna. Estimular la contraccin uterina con infusin continua de oxitocina y masaje sobre el fondo uterino. Si se
produce la inversin antes de la salida de la placenta se debe considerar una normalidad de la insercin placentaria y manejarse como
acretismo placentario. B. HEMORRAGIA TARDA: SUBINVOLUCIN UTERINA: Es una detencin o retardo de la involucin. La
subinvolucin va acompaada de una prolongacin del periodo de eliminacin de los loquios y a veces, de una hemorragia uterina
irregular o excesiva. Clnicamente el tero se encuentra de mayor tamao, ms blando que lo normal para un momento determinado
del puerperio. Entre las causas reconocidas de subinvolucin estn la retencin de fragmentos de placenta y la infeccin plvica.
Diagnstico: Examen clnico detallado. Realizacin de ecografa trasvaginal. Tratamiento: Si se confirman restos placentarios: curetaje.
tero tnico: (antes descritos). Si se diagnostica infeccin inicio de antibiticos de amplio espectro. HEMATOMA VULVAR: Se
diagnostica fcilmente por el intenso dolor y la aparicin sbita de un tumor sensible, fluctuante e intenso, de tamao variable y
cubierto por una piel cuya coloracin es diferente de lo normal, violceo. Puede presentarse tambin en el posparto inmediato. Su
manejo es la Incisin y evacuacin de sangre, seguidas de las ligaduras de los puntos que sangran. La cavidad debe obliterarse
empleando una sutura adecuada, usualmente catgut cromado 2 /0. Se administran antibiticos de amplio espectro si hay signos de
infeccin. INFECCIN PUERPERAL: Las infecciones plvicas son las ms comunes y serias complicaciones del puerperio. Se define como:
temperatura mayor de 38 grados, excluyendo las primeras 24 horas. Las causas ms frecuentes de fiebre en el puerperio son
endometritis, infeccin urinaria y mastitis. ENDOMETRITIS: Es la forma ms comn de infeccin puerperal, se relaciona con la decidua,
el miometrio adyacente y los tejidos parametriales. Su va de Infeccin es ascendente, endgena y polimicrobiana (Gram positivos del
endometrio en el 40% de los casos, los grmenes aerobios gram negativos se aslan del endometrio en el 22%, Las bacterias anaerobias
representan el 38% de los patgenos, La Chlamydia Trachomatis es responsable del 2 al 4% de la endometritis tarda, hasta 6 semanas
posparto. Frecuencia de endometritis posterior a cesrea 7 a 30 veces ms frecuentecomparada con el parto vaginal. Depende de los
factores de riesgo asociados. Puede ser hasta un 90% si no se usan antibiticos profilcticos. Complica el 5 a 6 % de las cesreas
electivas y el 22 a 85% de las urgentes Los sntomas iniciales aparecen generalmente en las primeras 48 a 72 horas. La paciente
consulta usualmente a los 4 a 5 das despus del parto por fiebre alta de 39 a 40 grados, (este es el signo cardinal de la endometritis, de
ah que el aumento de la temperatura en el puerperio, se debe considerar endometritis hasta que se pruebe lo contrario), sntomas
generales, taquicardia, escalofro, expulsin de loquios que tienen un aspecto ajamonado (color rosado brillante) y de muy mal olor .Se
queja de dolor abdominal, principalmente en hipogastrio. La involucin del tero puede estar retardada. Si se encuentra fiebre de 38.5
a 39 grados centgrados durante las primeras 24 horas posparto, acompaada de distensin abdominal, taquicardia y loquios claros
acuosos, se debe considerar el diagnstico de endometritis por Estreptococo hemoltico del grupo B o del grupo A; el cuadro clnico
puede progresar a celulitis plvica, peritonitis y septicemia. Tratamiento: Teniendo en cuenta la microbiologa, los antibiticos usados
deben ser de amplio espectro. Para casos leves, usualmente despus de parto vaginal, un antimicrobiano de amplio espectro por va
oral puede ser suficiente. Hay mejora en 48 a 72 horas en el 90% de las pacientes. Los esquemas pueden ser: Penicilina procanica.
800.000 unidades IM cada 12 a 24 horas Ampicilina 500 mg a 1 gr. Va oral cada 6 horas, Cefalospornas de primera generacin. 500
mg a 1 gr. Va oral, cada 6 horas, ms Un Aminoglicsido tipo Gentamicina 160 mg/da. Sin embargo, para casos moderados a severos,
incluyendo la mayora de aquellos que tuvieron parto por cesrea, se inicia terapia parenteral. Hospitalizar y utilizar un esquema
biconjugado triconjugado Emplear antibiticos de amplio espectro: Penicilina Cristalina 5.000.000 Unidades IV cada 6 horas
Cefalospornas de primera generacin 1 gramo IV cada 6 horas Ampicilina 1 gr., IV cada 6 horas, ms Gentamicina 80 mg IV cada 8
horas 240 mg IV IM. , dosis nica al da Este esquema no cubre Bacteroides Fragilis, no previene las complicaciones de
tromboflebitis plvica y absceso plvico, hay un mayor tiempo de hospitalizacin. En un 20 y 40% de los casos posoperatorio de cesarea
no responden a este tratamiento. Por eso se adiciona un tercer antibitico que cubra anaerobios. Metronidazol 1 gr., inicial seguido
500mg IV va oral cada 8 horas Ornidazol 1 gr., cada 12 horas IV Clindaminica 600 cada 6 horas a 900 mg IV cada 8 horas (no slo
cubre anaerobios, tambin estafilococo, estreptococo y Chlamydia), con este esquema slo se aade la Gentamicina. INFECCIN E LA
HERIDA QUIRRGICA: La incidencia de infeccin de la incisin abdominal luego de la cesrea es de un 5 a 15% con un promedio de 7%.
Cuando se administra antibiticos profilcticos la incidencia es de un 2% o menos. Los factores de riesgo para la infeccin de la herida
abdominal son: Obesidad, diabetes, terapia con esteroides, inmunosupresin, anemia y pobre hemostasia con formacin de
hematoma. El tratamiento consiste en drenaje quirrgico y antibiticos. Evaluar que la fascia este intacta, si no el cierre secundario se
debe realizar. MASTITIS: Cuando la congestin mamaria o las grietas del pezn no mejoran, puede ocurrir una infeccin superficial o
profunda de la glndula. La inflamacin va precedida casi siempre de una ingurgitacin, fiebre, taquicardia, la mama se torna dura,
enrojecida, y la paciente se queja de dolor. El microorganismo responsable ms frecuente es el Estafilococo ureos. Tambin se ha
aislado el Estafilococo coagulasa negativo y el Estreptococo viridans. La fuente inmediata de estafilococos que provocan mastitis est
casi siempre en la nariz y garganta de la madre. En el momento de la lactancia la bacteria penetra en la mama a travs del pezn, en el
lugar donde existe alguna grieta o fisura, que puede ser mnima. La bacteria se puede cultivar de la leche materna. TRATAMIENTO:
manejo quirrgico; Taponamiento uterino: es necesario cuando el tratamiento quirrgico no sea posible en el sitio actual o la paciente
MANUAL DE TRABAJO DEL CURSO ENARM CMN SIGLO XXI
CURSO ENARM CMN SIGLO XXI TEL: 36246001 Pharmed Solutions Institute PGINA 339

este demasiado inestable para una ciruga. Requisitos: 1. Eliminar todo el tejido placentario a excepcin de los fragmentos
correspondientes a una placenta creta. 2. Descartar una rotura y una inversin uterina. 3. Usar previamente los oxitcicos y el masaje
uterino. 4. Descartar otra causa de sangrado. 5. Un ayudante expone el cuello traccionando los labios anteriores y posteriores. Se
emplea una pinza para extraer restos ovulares con el fin de empacar gasa seca en el fondo uterino de modo que quede bien apretada,
luego se completa el procedimiento dejando comprimida la vagina. 6. Durante el procedimiento se debe aplicar una infusin continua
de oxitocina. 7. La paciente debe recibir antibiticos 8. Se remite con sonda vesical abierta. 9. Si fracasa no debe intentarse
nuevamente, y se debe realizar una laparotoma sin prdida de tiempo. Si este procedimiento falla, se pasa al siguiente paso: ligadura
de los vasos uterinos: La arteria es ligada donde corre al lado del borde uterino, por encima del segmento uterino inferior. Si es
posterior a una cesrea, la ligadura se realiza 2 a 3 cm. por debajo del nivel de la incisin uterina. Puede ser necesario descender la
vejiga. La arteria y la vena pueden ser ligadas pasando una aguja 2 a 3 cm. medial a los vasos plvicos, incluyendo la mayora del grosor
del miometrio, y atravesando el ligamento ancho en el rea avascular lateral a los vasos. Histerectoma: Desafortunadamente no
garantiza el control de la perdida sangunea severa. El sangrado puede persistir de las superficies plvicas debido a alteraciones de la
coagulacin combinado con el trauma de la manipulacin prolongada. Se debe continuar con empaquetamiento abdominal.
Empaquetamiento abdominal: Aplicacin de compresas estriles en la cavidad plvica, para controlar el sangrado contino de las
superficies peritoneales cuando ya se realiz la histerectoma y se inici una Coagulopata. Son retiradas 24 horas luego de corregida la
Coagulopata. Embolizacin: de la arteria uterina / iliaca interna: cuando las opciones quirrgicas se han agotado y existe la posibilidad
de intervencin por radiologa.

CASO CLINICO
Acude a consulta paciente con 48 horas posparto, refiere que presenta dolor en regin plvica de caractersticas punzante e intensidad
moderada a severa, agrega que presenta cefalea continua, malestar generalizado, fatiga, adinamia, cuenta con antecedentes GO de
gesta 3 para 3 abortos 0, refiere que presento flujo amarillento en varias ocasiones en este ultimo embarazo, a la exploracin se
observa TA 100/70, FC 89 FR 24, Temperatura de 38.5 grados, a la palpacin refiere dolor, al tacto se identifica liquido
serosanguinolento ftido y tero con flacidez, decide ingresar a la paciente para manejo de la complicacin posparto que presenta
(corioamnionitis), a las 3 horas de ingreso se presenta dificultad para respirar, a la auscultacin se encuentra estertores bilaterales y
ataque al estado generalizado, TA de 70 /50 FC 102, FR 31, Temperatura de 36.1 grados.

PREGUNTA
Considerando la gravedad del cuadro cual es la complicacin ms probable en esta paciente.

RESPUESTA
a.- Tromboembolia pulmonar.
b.- Coagulacion intravascular diseminada.
c.- Sepsis abdominal.
d.- Choque distributivo.

HEMORRAGIA OBSTETRICA. CIENCIAS BASICAS: Se define como aquel sangrado que se produce a travs de la vagina en cualquier
momento del embarazo y cuya presencia no est justificada. Durante las primeras 20 SDG el aborto, el embarazo ectpico y la mola
hidatiforme, son las principales causas de sangrado transvaginal. Las hemorragias que se presentan en el tercer trimestre del embarazo
suelen ser todo un reto, pues estos sangrados son los que con mayor frecuencia ponen en riesgo la vida de la madre y del feto. SALUD
PUBLICA: Hemorragia obsttrica causa importante de morbimortalidad materna y perinatal. La hemorragia obsttrica se ha identificado
como la causa nica ms importante de decesos maternos en todo el mundo; explica casi la mitad de los fallecimientos postparto. En
Mxico en el 2005, las hemorragias durante el embarazo se encuentran como la
segunda causa de mortalidad materna con un porcentaje de 26.1%.
CLASIFICACION: 1) Hemorragias preparto: pueden deberse a embarazo
ectpico, aborto espontaneo, enfermedades del trofoblasto, lesiones vaginales y
cervicales locales, trastornos de la coagulacin, cncer. A la mitad del embarazo
se deben: anomalas de la implantacin placentaria (placenta previa),
desprendimiento prematuro de placenta normoinserta (DPPNI), trabajo de
parto pre trmino y 2) Hemorragias posparto: trauma del canal del parto,
desgarros cervicales, atona uterina, inversin uterina, retencin de restos
placentarios. PLACENTA ACRECTA. CIENCIAS BASICAS: El acretismo placentario
es la insercin anormal de parte o de toda la placenta, con ausencia total o
parcial de la decidua basal y anormalidad de la caduca verdadera con
penetracin de las vellosidades coriales al miometrio. Es una de las principales causas de hemorragia obsttrica, principalmente del
post parto inmediato, y que condiciona un importante riesgo demorbimortalidad materna, por la patologa misma, como tambin por
el tratamiento aplicado. SALUD PUBLICA: La incidencia de placenta acreta est aumentando, especialmente por la mayor incidencia de
operacin cesrea; el mayor riesgo de morbimortalidad materna disminuye con un adecuado diagnstico prenatal y planificando la
interrupcin del embarazo con un equipo quirrgico multidisciplinario, en una institucin que disponga de banco de sangre. La edad
media de las pacientes fue de 31 aos, y el grupo de mayor riesgo fue el de 35 a 39 aos con 26% de acretismo placentario.
CLASIFICACION: Existen tres variedades de adherencia anormalde la placenta: Placenta acreta: es la adherencia anormal de una parte o
de la totalidad de la placenta a la pared uterina, sin que las vellosidades coriales penetren el miometrio. Placenta increta: las
vellosidades coriales penetran el miometrio. Placenta percreta: es la penetracin de los elementos coriales hasta sobrepasar la serosa
del tero, pudiendo alcanzar rganos vecinos. Por su extensin se reconocen tres tipos: a) Focal: solo involucra pequeas reas de la
placenta; b) Parcial: uno o ms cotiledones se involucran en el proceso; c) Total: la superficie completa de la placenta esta
anormalmente adherida. PATOGENIA: Los principales factores de riesgo identificados para acretismo placentario de acuerdo a los
MANUAL DE TRABAJO DEL CURSO ENARM CMN SIGLO XXI
CURSO ENARM CMN SIGLO XXI TEL: 36246001 Pharmed Solutions Institute PGINA 340

antecedentes son: Edad (mayores de 30 aos), Gestas (multparas de 2 a 3), Placenta previa con antecedente de cesrea (35%),
Placenta previa con antecedente de 3 o ms cesreas (67%), Historia de legrado uterino (18 a 60%), Antecedente de extraccin manual
placentaria. Historia de retencin placentaria. Antecedente de infeccin intramnitica. Se piensa que el acretismo placentario es
debido a una ausencia o deficiencia de la capa de Nitabuch, que es una capa de fibrina que se ubica entre la decidua basal y el corion
frondoso. Benirschke y Kaufman, sugieren que esta situacin es consecuencia de una falla en la reconstitucin del endometrio/decidua
basal, especialmente despus de la reparacin de una operacin cesrea. El acretismo placentario puede llevar a hemorragia obsttrica
masiva, y como consecuencia alteraciones de la coagulacin por prdida, y porcoagulacin intravascular diseminada. DIAGNOSTICO:
Bsicamente el diagnstico se limita al empleo de mtodos como es el ultrasonido y la resonancia magntica. Sin embargo, el
diagnstico definitivo de acretismo placentario es por medio de histopatologa, al comprobar la invasin de las vellosidades coriales en
el miometrio. Ultrasonografa. El ultrasonido es una til herramienta para el diagnstico de acretismo placentario as como de sus
distintas variantes en el segundo y tercer trimestre del embarazo. Finberb y Williams, establecieron en 1992, la utilidad de esta tcnica
de imagen; entre los criterios diagnsticos establecidos fueron: prdida de la zona hipoecoica miometral retroplacentaria,
adelgazamiento o disrupcin de la serosa uterina hiperecoica y la interfaz con la vejiga, la presencia de masas exofticas, presencia de
una gran rea de sonolucencias placentarias. Alfa-feto protena fetal srica materna. Actualmente se ha propuesto que existe una
anormalidad en la interfaz placeta-tero, lo cual promueve una fuga de alfa-feto protena fetal hacia la circulacin materna. Histologa.
Los criterios microscpicos para el diagnstico de acretismo placentario es la ausencia de la decidua basal. Sin embargo su diagnstico
no resulta del todo fcil por la distorsin que ocurre en el momento del alumbramiento, salvo que se deje la pieza in situ y se realice
una histerectoma y as se obtenga la muestra del tejido. En el caso de placenta increta o percreta el diagnstico es ms fcil y consiste
en la presencia de tejido trofoblstico en el espesor uterino o en la estructura invadida. TRATAMIENTO: El principal tratamiento es la
histerectoma obsttrica total abdominal, debido a su frecuente asociacin con placenta previa y cicatrices de cesreas previas .Este
procedimiento es un desafo a las habilidades quirrgicas del mdico obstetra, por lo que es necesario conocer otras alternativas de
tratamiento. Manejo conservador. Es una medida que consiste en evitar la histerectoma, al menos en ese momento, y tratar de
preservar la fertilidad. Existen diversas modalidades de manejo conservador,que incluyen: Dejar la placenta en su lugar (totalmente o
en fragmentos). Reseccin del lecho placentario y su reparacin. Extraccin y legrado obsttrico. Empleo de medicamentos asociados a
cualquiera de los anteriores puntos. Empleo de algn medio que cause isquemia (embolizacin, ligadura de vasos, etc.) del lecho
placentario. El metotrexate es un medicamento quimioterpico el cual se encuentra catalogado dentro del grupo de los antagonistas
de los folatos. Su empleo en el acretismo placentario tiene como fundamento la efectividad en contra del trofoblasto proliferativo. Sin
embargo, de forma ms reciente se ha argumentado que despus del nacimiento del feto la placenta detiene su divisin y por lo tanto
pierde su utilidad.

SINDROME DE TENSION PREMENSTRUAL. CIENCIAS BASICAS: La American Psychiatric Associationlo denomina DDPM (desorden
disfricopremenstrual) o DFL (desorden de fase ltea).Se caracteriza por ser una entidad con sintomatologa somtica y afectiva
(trastornos anmicos) que produce disfuncin e incapacidad significativa. Posiblemente el DDPM no slo representa el extremo ms
grave de la sintomatologa premenstrual sino quizs una modalidad de trastorno depresivo o de ansiedad. Durante la vida
"reproductiva" de la mujer, son frecuentes una serie de cambios tanto fsicos como psicolgicos (molimia premenstrual), que
acompaan a los distintos ciclos menstruales; tan slo, una pequea proporcin de mujeres (3-5%) llegan a presentar el llamado
sndrome premenstrual (SPM) definido como la recurrencia cclica de una combinacin de malestar fsico, psicolgico y/o cambios en la
conducta presentes en la fase lutenica (aumento de estrgenos y progesterona) de la mayora de los ciclos, que remiten al final de la
menstruacin, con un intervalo libre de sntomas de al menos una semana por ciclo y de tal severidad que producen un deterioro de las
relaciones interpersonales y/o interfieren con otras actividades cotidianas. SALUD PUBLICA: 3-10% de las mujeres en edad reproductiva
son asintomticas. 30-90% presentan sntomas premenstruales leves. 20-30% presentan sntomas moderados a graves que alteran su
forma de vida. 3-8% padecen sntomas graves e invalidantes. Las causas por las que las mujeres solicitaban ayuda mdica eran la
depresin severa, ftiga y disminucin de la concentracin. PATOGENIA: En la actualidad es desconocida; se han propuesto tanto
biolgicas como psicolgicas para explicar el origen del SPM. A) Hiptesis biolgicas: postulan que los sntomas pueden surgir como
resultado de alteraciones neuropsicolgicas causadas por la accin a nivel del SNC de esteroides gonadales, neurotransmisores o
sustancias neuromoduladoras. Ciertos hallazgos fisiolgicos apoyan el origen orgnico-biolgico: Alteraciones de la conductividad
cutnea. Cambios a nivel del EEG ciclo-dependiente, con aparicin de ondas alfa y aumento de la duracin de la fase REM durante el
sueo, siendo directamente proporcional a la severidad de los sntomas. Otros hallazgos en el EOG (Electrooculograma). Se ha
intentado relacionar cambios hormonales con alteraciones en las monoaminas (5HT, NA), centrndose en algunos trabajos en las
alteraciones del sueo y de la termoregulacin (aumento de la temperatura corporal nocturna). Se sabe que la serotonina (5HT)
disminuye los niveles de estrgenos y de progesterona al inhibir la LHRH, y que la noradrenalina (NA) modula los receptores de
progesterona situados en los ncleos septales y en la amgdala (implicados en la termorregulacin, conducta sexual y emociones).
Algunos autores han encontrado otros hallazgos como: Alteraciones en los niveles de prolactina, aumento de los niveles de
prostaglandinas (PG), disminucin de los niveles de PG, disminucin de los niveles de Piridoxina, B) Hiptesis psicolgicas: El SPM es
ms frecuente en mujeres con un elevado nivel de neuroticismo, que a su vez muestran un pobre ajuste psicolgico y una gran
sensibilidad a la frustracin. Dada la frecuencia de sntomas afectivos (disforia, ansiedad e irritabilidad) y la ciclicidad inherente en su
definicin, se ha hecho hincapi en la relacin de este cuadro y de otros
trastornos psiquitricos (ciclotimia y distimia). En este sentido, se han dado
tres teoras bsicas que intentan relacionarlos: 1.- Episodios afectivos
tempranos, anteriores a la menarqua, que determinaran las caractersticas
de los cambios del humor posteriores (menstruales). 2.- Cambios en el
humor menstrual pueden servir como desencadenante de otras patologas
afectivas mayores en personas predispuestas genticamente. 3.- la teora de la "indefensin aprendida. DIAGNOSTICO: Las
pacientes informan 1 ms de los siguientes sntomas afectivos o somticos durante los 5 das anteriores a la menstruacin durante los
ltimos 3 ciclos. Los sntomas mejoran dentro de los 4 das de ocurrida la menstruacin, sin recurrencia por lo menos hasta el da 13 del
ciclo. Irritabilidad (85%), angustia (83%) y labilidad emocional (77%) son los sntomas ms reportados. Segn Sampson (1989) deben
MANUAL DE TRABAJO DEL CURSO ENARM CMN SIGLO XXI
CURSO ENARM CMN SIGLO XXI TEL: 36246001 Pharmed Solutions Institute PGINA 341

cumplirse algunas caractersticas para su diagnstico: Periodicidad de los sntomas, aumento de la gravedad en la medida que el ciclo
progresa, existencia de un perodo postmenstrual de 7 das, como mnimo, libre de sntomas, alivio al inicio de la menstruacin y
desaparicin al cabo de 3 das posteriores al comienzo, presencia de los sntomas durante 3 ciclos consecutivos como mnimo, con
posibles variaciones de intensidad, interferencia de los sntomas con el trabajo, las actividades sociales y las relaciones interpersonales.
Criterios diagnsticos del SPM o del Trastorno Disfrico Premenstrual (son criterios de investigacin)del DSM: A) Cinco (o ms) de los
siguientes sntomas durante la mayor parte del da de la ltima semana de la fase ltea de la mayora de los ciclos menstruales del
ltimo ao, que empiezan a remitir 2 das despus del inicio de la fase folicular y que desaparecen completamente en la semana
siguiente a la menstruacin, teniendo en cuenta que al menos uno de estos sntomas debe ser alguno de los cuatro primeros: 1) estado
de nimo deprimido, sentimientos de desesperanza e ideas de autodesaprobacin acusadas; 2) ansiedad, tensin, sensacin de agobio
o de estar "al limite"; 3) labilidad emocional evidente; 4) enfado, irritabilidad o aumento de los conflictos interpersonales de forma
acusada y persistente; 5) prdida del inters por las actividades cotidianas; 6) sensacin subjetiva de dificultad de concentrarse; 7)
letargia, fatigabilidad fcil o falta evidente de energa; 8) cambios significativos del apetito, atracones o antojos por determinadas
comidas; 9) hipersomnia o insomnio; 10) sensacin subjetiva de estar rebasada o fuera de control; 11) otros sntomas fsicos como
hipersensibilidad o aumento del tamao mamario, dolores de cabeza, molestias articulares o musculares, sensacin de hinchazn o
ganancia de peso. B) Estas alteraciones interfieren acusadamente con el trabajo, la escuela, las actividades sociales habituales o las
relaciones interpersonales. C) La alteracin no representa una simple exarcebacin de sntomas de otro trastorno. D) Los criterios A, B y
C deben ser corroborados por tcnicas de valoracin diaria y prospectiva de los sntomas en al menos dos ciclos sintomticos
consecutivos. TRATAMIENTO: El objetivo principal del tratamiento es disminuir los sntomas de SPM o DDPM para que la paciente
pueda realizar una vida normal, lo cual incluye entre otras medidas: Modificaciones de los hbitos alimentarios (Nivel B),
vitaminoterapia, frmacos, psicoterapia (Nivel C). Educacin: es importante que la mujer conozca la fisiologa del ciclo menstrual para
comprender lo que le est ocurriendo. El ACOG recomienda los cambios en el estilo de vida como la 1ra lnea de tratamiento. Dieta: Se
recomienda restringir el consumo de: Sal (3g/da) para reducir la retencin de agua, azcares refinados, alimentos con Metilxantinas
para no aumentar la ansiedad y la irritabilidad, tabaco y alcohol, carnes rojas, se recomienda incrementar la ingesta de: Carbohidratos
complejos, pescados y aves como fuente de protenas, fibras, vegetales, legumbres y cereales. Ejercicio fsico: Aerbicos: caminata,
trote, natacin, remo, ciclismo y danza (nivel C), meditacin, yoga. Es controvertido y debe reservarse a las pacientes que no mejoran
con las medidas mencionadas. Vitaminoterapia: Vitamina B6, la administracin diaria de 50 100 mg podra tener efectos beneficiosos
sobre los sntomas depresivos.(nivel B). Vitamina E: 400 a 600 UI/da. Mineraloterapia: El calcio y el magnesio son cofactores en la
sntesis de neurotransmisores, lo que sugiere una posible asociacin entre SPM y la deficiencia de serotonina (nivel B). AINE Mejoran la
mastalgia, la dismenorrea, la migraa y los dolores musculares. Anticonceptivos Hormonales: Se ha sugerido un efecto beneficioso
sobre el bienestar psicolgico y se los ofrece como una buena alternativa para el tratamiento (nivel B). FDA aprob un esquema
teraputico de ACO con bajas dosis de EE y DRSP con 24 comprimidos activos y 4 placebos para los sntomas fsicos y emocionales,
mediantela reduccin de la fluctuacin hormonal a lo largo del ciclo y las propiedades antiandrognicas y antimineralocorticoideas.
Joffe y colaboradores informaron que los ACO no modifican los cambios de humor premenstrual, pero, en aquellas mujeres con
antecedentes de depresin, tienen un gran riesgo de empeorar estos sntomas. Agonistas de GnRH: Producen alivio significativo de las
manifestaciones de la enfermedad, pero inducen sntomas menopusicos que limitan su uso en la mayora de las pacientes (nivel B).
Inhibidores de las Gonadotrofinas: El danazol parece ser efectivo en el tratamiento del SPM, especialmente en el alivio de la mastalgia
(por accin directa) y de la irritabilidad. No se recomienda en pacientes con depresin y ansiedad. La dosis propuesta es de 200 mg dos
veces por semana, slo en la fase ltea. Gestgenos: No se pudo demostrar su efectividad para el tratamiento del SPM, no se
recomienda su uso (nivel A). Diurticos: Las candidatas para el uso de diurticos son aquellas pacientes que manifiestan aumento de
1.5 kg de peso durante el perodo premenstrual, tensin mamaria y otros sntomas de retencin hdrica. La espironolactona es el
diurtico recomendado, ya que es un agonista de la aldosterona. La dosis es de 25 mg, 2-3 veces por da, durante los das 18 al 26 del
ciclo. Inhibidores Selectivos de la Recaptacin de Serotonina (ISRS): Primera lnea para el tratamiento del DDPM, pues son muy eficaces
para el alivio de los sntomas fsicos y emocionales (nivel A).Fluoxetina, sertralina, citalopram, paroxetina) actan sobre el humor, la
saciedad y la agresividad.Son eficaces cuando se usan en forma continua o slo en fase premenstrual.

CASO CLINICO
Se trata de femenino de 21 aos de edad la cual acude a consulta debido a que refiere mastalgia y sensacin de plenitud mamaria,
irritabilidad, inestabilidad emocional, refiere que desde los 17 aos se presento ms intensos los sntomas, agrega dismenorrea
ocacional con rechazo al periodo menstrual, al interrogatorio usted observa irregularidades menstruales, la paciente presenta un IMC
de 18.

PREGUNTA
Cul es la conducta ms adecuada para este caso.

RESPUESTA
a.- La conducta mas adecuada es iniciar un esquema de contraceptivo oral.
b.- Iniciar un Inhibidor Selectivo de la Rescaptura de Serotonina.
c.- Indicar la calendarizacin de la sintomatologa previa a la terapia.
d.- Derivar a la paciente al servicio de psiquiatra.

CASO CLINICO
Paciente de 39 aos, madre de dos hijos, primer parto con cesrea por falta de dilatacin, el segundo con frceps. Sus molestias
haban comenzado siete aos antes, en el perodo premenstrual, manifestndose como dolores de cabeza. Experimentaron con
el tiempo progresiva prolongacin e intensificacin. Por la jaqueca consult a neurlogo, quien pidi un electroencefalograma, que
result normal. Le fueron recetados analgsicos. Se presentaron molestias mamarias que se hicieron francamente severas y
duraban casi todo el mes. Tuvo diagnstico de Mastopata Fibrosa Bilateral de grado moderado. Fue tratada con un derivado
MANUAL DE TRABAJO DEL CURSO ENARM CMN SIGLO XXI
CURSO ENARM CMN SIGLO XXI TEL: 36246001 Pharmed Solutions Institute PGINA 342

sinttico de la progesterona durante diez das cada mes; hubo alivio parcial de las molestias mamarias. Por otra parte, se le
diagnostic "lcera al cuello del tero", lo que se trat con dos cauterizaciones, sin proporcionar ningn beneficio a nivel general. La
paciente manifestaba sufrir molestias en las relaciones sexuales y adems frigidez, razn por la cual procuraba evitarlas. Sus
molestias generales persistan y los sntomas mamarios se acentuaron. Al exmen fsico se observaba una paciente decada, palidez de
piel y mucosas, lengua sucia, conjuntivas irritadas, mirada triste. Adems distensin abdominal, edema (hinchazn) de manos y
prpados, piel y cabello secos. Las mamas presentaban aumento de volumen, congestin, sensibilidad, ardor y aumento de la red
venosa.

PREGUNTA
Cual es la ducta a seguir mas adecuada?

RESPUESTA
a.- Acido mefenamico.
b.- Ibuprofeson.
c.- Progesterona
d.- Citalopram

PSICOSIS POSPARTO. CIENCIAS BASICAS: La psicosis posparto o puereral se define como un episodio de mana o de psicosis precipitado
por el nacimiento de un beb: Es una enfermedad severa cuyas caractersticas clnicas incluyen todas las variantes de la mana, psicosis
cicloides o esquizofreniformes. Se caracteriza por prdida de contacto con la realidad, agitacin, confusin, alucinaciones vividas,
delirios paranoides y comportamiento violento. Muy excepcionalmente se ha descrito suicidio e infanticidio. SALUD PUBLICA: El riesgo
general es de 1-2 casos por cada 1.000 nuevos nacimientos. Se suelen describir algunos factores de riesgo. As, se ha comprobado que
el 50-60% de los casos ocurren en primparas, y que en el 50% de los casos se asocian a complicaciones psiquitricas perinatales. En
cuanto a los antecedentes familiares, un 50-60% de las mujeres afectadas tienen antecedentes familiares con trastornos afectivos. El
riesgo est muy aumentado en las mujeres que tienen antecedentes personales de trastornos del humor unipolares o bipolares
(especialmente un trastorno bipolar I). En estos pacientes el riesgo pasa de 2/1000 al 20 a 25%. Las enfermedades psiquitricas del
posparto son 5 veces ms frecuentes que las del embarazo, de ah la hiptesis del papel protector del embarazo frente a las
enfermedades mentales graves. Esta patologa psiquitrica del posparto debuta en la primera semana que sigue al parto en el 40% de
los casos, y en el primer mes postparto en el 80% de los casos. PATOGENIA: Los factores que pueden influir en el inicio de un cuadro de
las caractersticas que nos ocupa se pueden agrupar en: 1.- Factores heredo-constitucionales: Se insiste en el papel del "terreno",
siendo el grupo de familia con historia de bipolaridad el que ofrece las relaciones genticas ms significativas. Cuando una mujer ha
presentado crisis manaco depresivas, existe un elevado porcentaje de riesgo de recada despus del parto. 2.- Factores biolgicos: Los
trastornos endocrinos que aparecen antes, durante e inmediatamente despus del parto son de una amplitud y rapidez nicas. En
efecto, en los das siguientes al parto las concentraciones plasmticas de estrgenos y progesterona caen de manera considerable para
alcanzar tasas 100 veces ms bajas que aquellas que existen en el preparto. 3.- Factores psicodinmicos: autores hacen intervenir el
rechazo del nio como uno de los factores esenciales de las melancolas puerperales: rechazo consciente (nio no deseado) o ms a
menudo inconsciente, que emana de la ambivalencia profunda que marca la relacin madre e hijo, "rechazo combatido tanto por la
culpabilidad como por el amor maternal. Entre los factores psicosociales figura, en primer trmino, el conflicto provocado por la
aparicin de una tercera persona en el grupo familiar. La madre tiene que asumir ahora una mayor responsabilidad. Tiene que dejar de
ser hija para ser ella, a su vez, madre. DIAGNOSTICO: Clinico; Las psicosis puerperales se caracterizan por depresin e ideas delirantes
que aparecen despus del parto. El estado de nimo puede estar elevado o deprimido y con frecuencia es lbil; puede tambin haber
agitacin, aumento de la actividad e insomnio. La enferma puede ver al recin nacido como la personificacin del diablo o como el
Mesas; tambin pueden existir alucinaciones auditivas que pueden ser rdenes, en ocasiones para que la paciente mate al recin
nacido. Pueden producirse infanticidios.
El cuadro clnico tiene unas
caractersticas especficas: a) Rpido
cambio de sintomatologa, b) Labilidad
de humor, c) Signos confusionales. Los
sntomas iniciales ms frecuentes son
ansiedad, cansancio, humor depresivo,
disturbios del sueo, alteraciones del
comportamiento, agitacin catatnica,
ilusiones y alucinaciones. El comienzo
ocurre en un perodo muy poco tiempo
despus del parto y existe una serie de
precisiones:Deben ser excluidas las
psicosis asociadas con eclampsia,
infeccin, deprivacin de alcohol o con
problemas del puerperio. Los pacientes con trastornos delirantes monosintomticos y con psicosis psicgena deben ser excluidos. Si se
decide incluir como tal a las psicosis depresivas, debieran restringirse a aquellas que presentan delirios, alucinaciones, confusin,
estupor, catatona o sntomas maniformes transitorios. El intervalo entre el parto y la aparicin de la psicosis puede ser de 14 das, un
mes o tres meses. No es necesario que la madre est libre de patologa psiquitrica durante el embarazo. El DSMIV no contiene
criterios diagnsticos especficos para este trastorno. El diagnstico puede hacerse cuando la psicosis ocurre en estrecha relacin
temporal con el nacimiento del nio. Los sntomas caractersticos son ideas delirantes, dficits cognoscitivos, alteraciones motoras,
alteraciones del estado de nimo y, de forma ocasional, alucinaciones. El contenido del material psictico hace referencia al embarazo y
la maternidad. TRATAMIENTO: Los tratamientos farmacolgicos son comparables a aquellos utilizados para los sndromes similares que
MANUAL DE TRABAJO DEL CURSO ENARM CMN SIGLO XXI
CURSO ENARM CMN SIGLO XXI TEL: 36246001 Pharmed Solutions Institute PGINA 343

aparecen fuera del contexto del posparto. Al comienzo, es necesario un tratamiento farmacolgico enrgico, y en la mayora de los
casos, se recomienda la hospitalizacin. Dependiendo de cada mujer, suelen prescribirse antipsicticos y estabilizadores del nimo. Por
otra parte, se tiende a utilizar benzodiacepinas para controlar los episodios de agitacin y de ansiedad. Si la paciente no responde
favorablemente a la farmacoterapia o los sntomas van en aumento, puede ser necesaria la terapia electro convulsiva. Un estudio
demuestro que las fenotiazinas son ms utilizadas en las mujeres que presentan una psicosis del posparto. La psicoterapia debe estar
centrada en las reas particularmente conflictivas. Hay que ayudar a la paciente a aceptar su rol femenino y la rabia y los celos que
surgen a raz de la dependencia que tiene respecto de su madre. Va de suyo la necesidad de tener en cuenta al grupo familiar entero.
Tambin puede estar indicado el introducir algunos cambios en la estructura del hogar.

CASO CLINICO
Femenino de 28 aos, casada desde haca cinco aos, estaba esperando el nacimiento de su primer hijo. Nunca haba tenido ningn
problema emocional particular y haba estado siempre fsicamente bien. El parto de una nia de 3,2 kg, saludable y sin complicaciones,
dej cansada pero feliz durante los tres das siguientes a la paciente. Dej el hospital con nimos y todo fue bien durante la primera
semana en casa. Repentinamente, se puso ansiosa y no era capaz de estar quieta. Empez a pensar que alguien intentaba hacer dao al
beb. Reconoca que era ridculo pero no poda parar de pensar que alguien estaba intentando envenenar al beb o entrar en la casa.
Empez a preocupar al marido al decir cosas extraas, como que haban tenido nios gemelos, en vez de una nica nia. Algunas veces
se encontraba normal, mientras que otras, estaba inquieta y preocupada con sus ideas extraas. Durante la semana siguiente se
encontr tan mal que no pudo hacerse cargo del beb y en ocasiones, no poda ni levantarse por las maanas. Se senta culpable de
todo, no poda dormir por las noches y no tena apetito. Con frecuencia, estaba confundida, no sabiendo dnde estaba o lo que le
estaba pasando.


PREGUNTA
Cual es la conducta farmacolgica mas adecuada?

RESPUESTA
a.- Clorimipramina.
b.- Risperidona.
c.- Clonacepam.
d.- Valproato de sodio.

CASO CLINICO
Femenino de 27 aos, Un mes antes de la internacin abandon a sus hijos y a su marido porque no los reconoca como propios. El
esposo refiere que not 30 das despus del parto que la paciente comenzaba a desatender a sus hijos, hablaba en lenguas extraas,
no dorma, se enojaba con facilidad y cantaba a los gritos durante todo el da cnticos religiosos. En la misma poca comienzan las
huidas del hogar. Examen al ingreso: Desconfiada, alucinada, escucha la voz de un hombre que dice cosas feas. Refiere sentir dolores
de cabeza y de vientre, sensacin que vivencia como una agresin de ese mismo hombre. Reticente. Pueril. Con conductas impulsivas
de huida. Durante la internacin la paciente est excitada, con desorientacin y falsos reconocimientos. Por momentos permanece sin
ningn contacto con el entorno y al rato est atenta y dispuesta al dilogo, persisten las alucinaciones. Logorreica, con logoclonas,
responde a todo estmulo del ambiente, lee lminas de las paredes, toca historias clnicas, quita hojas de las plantas, intensa actividad
motora sin objeto. Insomne, canta o grita de da y de noche. La afectividad est polarizada hacia el placer con estados anmicos
fluctuantes. Escaso capital ideativo. Ocurrencias delirantes polimorfas, msticas, erticas y megalmanas. Cree ser la Virgen Mara, tiene
falsos reconocimientos con el personal del servicio mdico.

PREGUNTA
Se le ingreso e inicio tratamiento antipsicoticco, a las 48 aos refiere acatisia y rigidez, cual es la conducta ha seguir mas adecuada?

RESPUESTA
a.- Clonacepam.
b.- Trihexifenidilo.
c.- Biperiden.
d.- Difenidramina.

DEPRESION POSTPARTO (DPP). CIENCIAS BASICAS: Dentro de la nomenclatura psiquitrica, la DPP es definida rigurosamente como un
trastorno/episodio depresivo de inicio en el postparto. En el Manual Diagnstico y Estadstico de los Trastornos Mentales (DSM IV-TR)
la clasificacin es la siguiente: F32.x Trastorno depresivo mayor, episodio nico (296.2x). SALUD PUBLICA: Existen diferentes
investigaciones a nivel mundial sobre la prevalencia de este padecimiento, las cuales fluctan entre el 6 y 34%. En nuestro pas, los
estudios reportan una prevalencia que va desde 13.9 hasta 32.6%. A pesar de esta situacin, se podra hablar de que cerca del 20% de
las mujeres que dan a luz pueden presentar sntomas de DPP. En cuanto a la recurrencia, existe una probabilidad del 10 al 35%.
PATOGENIA: Hasta el momento, no se conocen las causas de la depresin postparto (DPP); nicamente se han podido observar
factores de riesgo, este padecimiento se puede presentar incluso cuando la madre no ha mostrado ningn antecedente. Factores de
riesgo: Historia pasada de psicopatologa (especialmente depresin y ansiedad), presencia de alteraciones psicolgicas durante el
embarazo, DPP previa, complicaciones obsttricas durante el embarazo, el parto o despus del mismo (principalmente embarazos de
alto riesgo), embarazo no planeado y/o deseado, falta de apoyo familiar y/o social, problemas familiares y/o sociales, pobre relacin
marital, abandono o separacin de la pareja, no contar con un compaero, pareja presente, pero que no proporcione apoyo emocional,
falta de redes de apoyo, situaciones estresantes, dicultades econmicas, abuso fsico, emocional o sexual en la infancia, fallecimiento,
MANUAL DE TRABAJO DEL CURSO ENARM CMN SIGLO XXI
CURSO ENARM CMN SIGLO XXI TEL: 36246001 Pharmed Solutions Institute PGINA 344

abandono o separacin de un familiar o ser querido, mala relacin con la madre, antecedente de prdidas perinatales, sensibilidad a los
cambios hormonales, alteracin en la funcin tiroidea, dicultades en la lactancia, mala salud del recin nacido, beb difcil de cuidar.
Suele comenzar en cualquier momento posterior al parto, ya sea das, semanas, meses y hasta un ao despus. Puede afectar en
cualquier tipo de parto o nmero de embarazo. DIAGNOSTICO: En relacin a la intervencin, el primer paso es detectarla,
especialmente en sus inicios, cuando se perciban sntomas o se presenten signos de alarma, como quejas psicosomticas, cansancio,
dolores, mltiples llamadas para pedir ayuda, o se observe que la mujer est triste, preocupada o con miedo. A continuacin, es
necesario canalizarla con un especialista (psiclogo, terapeuta o psiquiatra) para que sea evaluada y reciba el tratamiento adecuado,
buscando el apoyo multidisciplinario y una atencin integral. Sntomas depresin posparto: Estado de nimo deprimido, tristeza o
llanto persistentes, disminucin del inters o de la capacidad para disfrutar de casi todas las actividades, cambios en los patrones de
alimentacin y sueo (principalmente insomnio), cansancio o fatiga crnica, ansiedad que puede llegar hasta los ataques de pnico,
sentirse abrumada, indefensa e incapaz, autodesvalorizacin y autorreproches, generalmente relacionados a su competencia como
madre, dicultad, reticencia o imposibilidad de buscar ayuda y apoyo para s misma, irritabilidad, problemas de atencin, concentracin
y memoria, dicultad para vincularse con el beb, sentimientos de ambivalencia hacia el hijo, sobreproteccin del menor, dicultades
en la lactancia, dicultad y/o imposibilidad de cuidar al recin nacido, pensamientos obsesivos y conductas compulsivas, relacionados
con el hijo (p. ej. revisarlo constantemente cuando est dormido para ver si respira) y con su cuidado (p. ej. lavar y desinfectar varias
veces sus artculos), pensamientos negativos hacia el beb, especialmente creer que lo puede daar, sentimientos de culpa,
pensamientos de daarse a s misma o al beb, pensamientos atemorizantes, ideacin suicida. TRATAMIENTO: El tratamiento tiene dos
opciones de intervencin teraputica: 1) terapia (individual o de grupo) y/o 2) medicacin. En aquellos casos en que se presenta una
depresin de moderada a grave, lo ideal es combinar ambas. La psicoterapia, ya sea interpersonal o con un enfoque cognitivo-
conductual, debe basarse en los siguientes aspectos: a) Educacin e informacin a la mujer, la pareja y los familiares sobre la DPP. b)
Control, disminucin y extincin de los sntomas. c) El nuevo rol como madre, las dificultades, mie dos o angustias que esto puede
generar. d) Uso de estrategias para proteger, mejorar o fortalecer el vnculo madre-hijo, as como la relacin de pareja. En esta ltima,
se buscar una adecuada comunicacin, as como tener espacios de calidad sin el beb. e) Promover un buen funcionamiento familiar,
especialmente cuando existan otros hijos. f) Disminucin de las situaciones estresantes actuales. g) Bsqueda y fortalecimiento de
redes de apoyo (familiares, sociales e institucionales). Tratamiento medico: En cuanto al tratamiento psiquitrico, dependiendo de cada
caso, los medicamentos ms usados son los antidepresivos tricclicos y los inhibidores selectivos de la recaptacin de serotonina (ISRS),
para combatir los sntomas depresivos. Por otra parte, se tiende a utilizar benzodiacepinas para tratar la ansiedad y el insomnio. Si la
mujer tiene una historia previa de DPP, lo ms recomendable es iniciar la medicacin inmediatamente despus del parto, para prevenir
una posible recaida. Cuando la mujer con DPP no recibe tratamiento, puede presentar dificultades y estrs para cuidar al recin nacido,
lo que tiende a generar problemas en el vnculo materno-infantil. Dichos bebs pueden llegar a presentar retraso en el desarrollo
psicomotor, temperamento difcil o irritabilidad, as como mostrar un apego inseguro. As mismo, se ha observado que algunos no son
tan atentos y activos, y sonren menos, lo que puede generar dficits en el rea social y cognitiva. Con una intervencin adecuada, la
DPP es temporal y se puede alcanzar una completa recuperacin.

CASO CLINICO
Paciente femenina de 36 aos de edad, con 5 dia de puerperio, parto normal, producto a trmino, primer hijo, madre soltera, que vive
con su propia madre nicamente, la cual es divorciada desde hace 20 aos. La madre de la paciente la lleva a consulta por presentar
conductas extraas, refiere que no se ha baado desde que salio del hospital, la observa aptica, callada, poco interesada en el hijo, y
finalmente decidio ir a consulta al escuchar a su hija decir tal vez estaramos mejor si no estubieramos a la EF se observa paciente
desalineada, desaseada, poco cooperadora, desinteresada, niega ideas suicidas ni homicidas pero acepta haber dicho lo que la madre
refiere, se apresia melancolica mas que ansiosa.

PREGUNTA
Cual es la conducta a seguir mas adecuada?

RESPUESTA
a.- Ingreso psiquitrico involuntario.
b.- Envio a consulta externa de psiquiatra.
c.- Imipramina 50 mg cada 8 hrs.
d.- Haloperidol 5 mg cada 8 hrs.

PREGUNTA
Cual de los siguientes antidepresivos es el mas adecuado considerando la sintomatologa del caso?

RESPUESTA
a.- Velanfaxina.
b.- Paroxetina.
c.- Escitalopram.
d.- Fluvoxamina.

MANUAL DE TRABAJO DEL CURSO ENARM CMN SIGLO XXI
CURSO ENARM CMN SIGLO XXI TEL: 36246001 Pharmed Solutions Institute PGINA 345

PEDIATRIA

1) SX DE ADAPTACION PULMONAR, SX DE DIFICULTAD RESPIRATORIA (EMH), TAQUIPNEA TRANSITORIA DEL RN,
2) ASPIRACION DE MECONIO, NEUMONIA NEONATAL.
3) SEPSIS NEONATAL, ENTEROCOLITIS NECOTRIZANTE, MENINGITIS NEONATAL.
4) HIPOGLUCEMIA, ICTERICIA, KERNICTERUS.
5) ATRESIA ESOFAGICA, FISTULA TRAQUEOESOFAGICA, HERNIA HIATAL, ESTENOSIS PILORICA.
6) CARDIOPATIAS CONGENITAS Y ADQUIRIDAS
7) KWASHIORKOR, MARASMO, AVITAMINOSIS,
8) OBESIDAD, VACUNACION.
9) DERMATITIS, PENFIGO, MOLUSCO CONTAGIOSO, TIA CORPORIS
10) HIPERQUERATOSIS PALMOPLANTAR, FOLICULITIS, PEDICULOSIS.
11) PURPURA DE SCHNLEIN-HENOCH, ANEMIAS,
12) LEUCEMIAS, RETINOBLASTOMA, TUMORES DEL SNC.
13) PAROTIDITIS, HEPATITIS, INFLUENZA, MENINGITIS
14) OTITIS, SINUSITIS, FARINGITIS, AMIGDALITIS
15) EPIGLOTITIS, LARINGOTRAQUEITIS, BRONQUITIS.
16) BRONQUIOLITIS, BRONCONEUMONIA, NEUMONIAS
17) ALERGIAS, ASMA, FIBROSIS QUISTICA.
18) URETRITIS, CISTITIS, PIELONEFRITIS.
19) GLOMERULO NEFRITIS, SINDROME NEFRITICO, SINDROME NEFROTICO.
20) GASTROENTERITIS, PARASITOSIS.
21) DESHIDRATACION, LIQUIDOS Y ELECTROLITOS
22) QUEMADURAS, ACCIDENTES
23) PICADURAS, MORDEDURAS
24) INTOXICACIONES, ENVENENAMIENTOS
25) TRAUMATISMO CRANEOENCEFALICO, CEFALEA Y EPILEPSIAS
26) ENURESIS, ENCOPRESIS, TDAH, TRASTORNOS DEL DESARROLLO.






































MANUAL DE TRABAJO DEL CURSO ENARM CMN SIGLO XXI
CURSO ENARM CMN SIGLO XXI TEL: 36246001 Pharmed Solutions Institute PGINA 346

SINDROME DE ADPATACION PULMONAR (SAP): CIENCIAS BASICAS: Los cambios que implican adaptarse a la nueva vida se producen
de manera simultnea desde el nacimiento con la primera respiracin y terminan aproximadamente a las 15 hrs de vida,
denominndose perodo de transicin. Involucra cambios cardiorrespiratorios, metablicos, neurolgicos y hormonales. Los
componentes del perodo de transicin son: Esfuerzo respiratorio inicial. Aumento unas cinco veces en el flujo linftico pulmonar.
Depuracin del lquido pulmonar desde la va area a los canales linfticos y vasculares pulmonares. Establecimiento de un rea de
superficie pulmonar estable. Reduccin de los altos niveles de resistencia vascular pulmonar. Aumento del flujo sanguneo pulmonar.
Aumento de la presin de oxgeno arterial. Cierre del conducto arterioso. La eliminacin del lquido pulmonar se lleva a cabo en 2-6hrs.
PRIMER PERODO DE REACTIVIDAD: 30-60 minutos de vida. Movimientos de succin y deglucin. Temblores finos en extremidades y
mandbula. Cierre y apertura de los prpados. Movimientos espasmdicos rpidos y breves de los globos oculares. Reflejo espontneo
de Moro. Llanto de inicio y detencin sbita. Cianosis leve y breve. Enrojecimiento con el llanto. Abundantes secreciones en vas areas.
FR 100x, FC 200, quejidos inspiratorios. Estertores bilaterales. TA elevada. Peristalsis ausente. Hipotermia. PERODO DE SUEO O
TRANQUILIDAD: Despus de los 30-60 minutos, dura hasta las 2 horas de vida. Puede permanecer dormido, pero responde a estmulos
en forma brusca. FR es rpida y superficial. Desaparecen los quejidos inspiratorios y estertores. FC regular 120-140x, desciende 100 por
breves perodos. TA se normaliza. Aparece la peristalsis. Se observan ondas peristlticas. Temblores y sacudidas espontneas.
SEGUNDO PERODO DE REACTIVIDAD: Ocurre entre las 2-6hrs de vida, es breve o persiste por 15hrs. Se reanuda la actividad motora,
exagerada. Inicia nuevamente taquicardia, polipnea. Respiraciones peridicas. Secreciones abundantes. Reflejo nauseoso, vmitos,
meconio. Muy sensible estmulos externos. Inestabilidad vasomotora. Causas que alteran la eliminacin del lquido pulmonar:
Nacimiento por cesrea, parto inducido, prematurez, nacimiento sin trabajo de parto, asfixia, drogas en las madres, hijo de madre
diabtica. Causas que alteran la adaptacin pulmonar: Hipoxia y acidosis, hipotermia, perodo expulsivo prolongado, RN con dificultad
respiratoria, RN de trmino o pretrmino limtrofe, acrocianosis, elevacin de la FC, remite entre las 8 y 12 hrs. DIAGNOTICO: Rx de
Trax es normal o con leve aumento de la trama vascular, los gases sanguneos con acidosis respiratoria e hipoxia leves. Diagnstico
diferencial: Taquipnea Transitoria del RN. Enfermedad de la Membrana Hialina. Alteraciones metablicas. Alteraciones de la
temperatura. Anemia. TRATAMIENTO: Ambiente trmico neutro para reducir el consumo de O2. Aspiracin de secreciones.
Oxigenoterapia: mantener la PaO2 entre 50-80mmHg. Va oral en cuanto sea posible, si no, iniciar LIV con glucosada al 10% a
requerimientos.

CASO CLINICO
Se trata de paciente masculino RN de 60 minutos, obtenido por cesarea iterativa, dems de presentar sufrimiento fetal, por
disminucin de actividad intrauterina y disminuciones de FC de hasta 110 lpm, se observa con movimientos muy finos, al inicio
presento cianosis leve y breve principalmente periungeal y oral, se apresiaron secresiones orales abundantes, a la exploracin fsica se
auscultan estertores bilaterales normales, peristaltismo poco audible y temperatura de 36.1 grado, se apresi expulsin de secresiones
con tinte meconial durante la exploracin.

PREGUNTA
Considerando el cuadro clnico cual es su conducta a seguir?

RESPUESTA
a.- Mantener al paciente en observacin continua.
b.- Buscar psobles patologas ocultas.
c.- Realizar radiografia de torax.
d.- No hay datos patolgicos son adaptativo.

PREGUNTA
Cual es la conducta teraputica menos apropiada en este momento del caso?

RESPUESTA
a.- Mantener ayuno hasta identificar causa.
b.- Colocar en ambiente neutro.
c.- Administrar oxigeno por casco ceflico 40 %.
d.- Gasometria y oximetra de pulso.

PREGUNTA
Cual de los siguientes parmetros gasomtrico no es normal esperarlo en el caso?

RESPUESTA
a.- pH < 7.34.
b.- PaCO > 45 mmHg.
c.- HCO3 > 40mEq/L.
d.- SpO2 < 95.

PREGUNTA
Se realizaron electrolitos sericos por conituar con rechazo al alimento y secresion moderada oral, todos los resultados resultaron dentro
de parmetro normales, cual de las siguientes diagnosticos diferencias del acidosis respiratoria es el mas probable presentarse?

RESPUESTA
a.- Ventilacin inadecuada alveolar.
MANUAL DE TRABAJO DEL CURSO ENARM CMN SIGLO XXI
CURSO ENARM CMN SIGLO XXI TEL: 36246001 Pharmed Solutions Institute PGINA 347

b.- Desordenes musculares.
c.- Defectos pulmonares.
d.- Trastornos de las vas areas.

PREGUNTA
Considerando los valores gasomtricos y de eletrolitos, cual es la causa de la modificacin del anio gap, del caso?

RESPUESTA
a.- Error de laboratorio.
b.- Toxinas no identificadas.
c.- Mielomas.
d.- Exceso de soluciones.

PREGUNTA
El paciente presenta persistencia de los sntomas por 3 horas ms, se reporta que incrementa con la alimentacin, con dificultad
continua succionando, usted considera realizar laboratorios, cual de los siguientes resultados es el menos probable encontrar en este
caso?

RESPUESTA
a.- Aumento de la trama vascular.
b.- Presencia de infiltrados pulmonares.
c.- Acidosis respiratoria.
d.- Hipoxemia leve.











































MANUAL DE TRABAJO DEL CURSO ENARM CMN SIGLO XXI
CURSO ENARM CMN SIGLO XXI TEL: 36246001 Pharmed Solutions Institute PGINA 348

SINDROME DE DIFICULTAD RESPIRATORIA (ENFERMEDAD DE MEMBRANA HIALINA). CIENCIAS BASICAS: El sndrome de dificultad
respiratoria por deficiencia de surfactante es la principal patologa respiratoria entre los recin nacidos; ocupa un papel preponderante
por su alta morbimortalidad. Ha recibido diversos nombres a travs del tiempo siendo los ms comnmente reportados, sndrome de
insuficiencia respiratoria idiomtica, sndrome de hipoperfusin pulmonar y sndrome de microatelectasias mltiples. El trmino
de sndrome de dificultad respiratoria tipo I, se otorg por las alteraciones radiolgicas presentadas en esta patologa, el Doctor Swis-
chuk reportaba en las radiografas de esta patologa pequeas burbujas de tipo esfrico de 1 a 1.5 mm. de dimetro a las cuales
denomino burbujas tipo I, en otras patologas encontr a nivel radiogrfico burbujas de mayor tamao a las que denomino burbujas
tipo II y otras de tamao mucho mayor (qusticas) las denomino tipo III. El trmino ms aceptado actualmente es el de sndrome de
dificultad respiratoria por deficiencia de surfactante, ya que define sus principales caractersticas fisiopatolgicas como la congestin
difusa pulmonar, notoria disminucin de la distensibilidad pulmonar y la presencia de tejido necrtico y membranas de aspecto hialino
en bronquiolos y alvolos. SALUD PUBLICA: Este sndrome contina siendo una de las primeras causas de ingreso a las unidades de
terapia intensiva neonatal a nivel mundial. La incidencia exacta de esta patologa es difcil de precisar. Las tendencias actuales reportan
incidencia de 60 a 80% en los recin nacidos menores de 30 semanas de gestacin, un 15 a 30% de los de 32 a 36 semanas de gestacin
y solo 5% de los que tienen 37 semanas o ms de gestacin. La frecuencia tambin aumenta cuando son hijos de madre diabtica (de
evolucin corta la diabetes materna) en asfixia perinatal, y algunos casos en que por error en la determinacin de edad gestacional, se
realiza operacin cesrea antes de lo debido. Discreta tendencia a ser ms frecuente en recin nacidos de sexo masculino que en sexo
femenino. PATOGENIA: En el sndrome de dificultad respiratoria, la frecuencia respiratoria se encuentra elevada por lo que a pesar del
volumen corriente (Vt) disminuido, la ventilacin minuto inicialmente esta incrementada. Debido a la deficiencia o disminucin en la
cantidad o calidad del surfactante pulmonar la mayor parte del pulmn est colapsado o poco ventilado y la mayor parte de la
ventilacin alveolar se deriva a una regin muy pequea del pulmn lo que conlleva a una disminucin de la capacidad residual
funcional (CRF). Asimismo la distensibilidad est muy disminuida, no tanto por el trax del recin nacido que es fcilmente distensible,
sino que por los pulmones que con esta deficiencia de surfactante llegan a tener mediciones de la distensibilidad de 0.3-0.6
ml/cmH2O/kg en lugar de 2.0-2.5 ml/cmH2O /kg que es lo normal, lo cual traduce una distensibilidad menor al 30% de los normal. La
resistencia de la va area es normal pero con tendencia a incrementarse hasta en 40-50% como resultado de todo esto el trabajo
respiratorio se ver incrementado en poco ms del 50%. Edberg y colaboradores encontraron disminucin de la distensibilidad,
incremento de la resistencia, reduccin del volumen pulmonar y disminucin en la eficacia para la mezcla de gases en los recin nacidos
prematuros con sndrome de dificultad respiratoria. A partir de estos datos y dado que la constante de tiempo (CT) depende de la
distensibilidad y la resistencia (CxR=CT) , se ver afectada y como consecuencia se presentar un inadecuado intercambio del volumen
alveolar. Esta disminucin o acortamiento de la constante de tiempo no es uniforme en toda la va respiratoria, se aprecia sobre todo
en las reas ms distales, por lo que , en un mismo pulmn habr constantes de tiempo diferentes lo que lleva a una ventilacin no
uniforme con riesgo de daar a aquellas vas areas con constantes de tiempo normales, que son sometidas a la ventilacin mecnica
necesaria para forzar a abrirse a las vas areas con constante de tiempo acortada. DIAGNOSTICO: Los signos pueden manifestarse
desde los primeros minutos de vida o despus de algunas horas, y por lo general son de incremento gradual. Aunque en ocasiones
estos signos de dificultad respiratoria son menos marcados debido a la debilidad de la musculatura respiratoria, lo que los llevar
rpidamente a una falla respiratoria con hipoventilacin y apna (conocido como respuesta paradjica a la hipoxemia). Los signos de
dificultad respiratoria se manifiestan con taquipnea, tiraje intercostal, retraccin xifoidea, disociacin toracoabdominal, aleteo nasal, y
quejido espiratorio, este ltimo uno de los ms frecuentes y es motivado por el cierre de la glotis en su afn de realizar un auto PEEP
(presin positiva al final de la espiracin) para conservar los alvolos abiertos y aumentar el volumen residual pulmonar para un
adecuado intercambio gaseoso. A la auscultacin de campos pulmonares encontraremos disminucin del murmullo vesicular
habitualmente en forma bilateral. Adems de los signos de dificultad respiratoria se puede presentar cianosis central que obligar al
uso de oxigenoterapia en cualquiera de sus modalidades y que podr variar desde los casos leves que solo requieran apoyo con oxgeno
en fase de casco ceflico o bien casos moderados y severos que requerirn CPAP (presin positiva continua en vas areas) o ventilacin
mecnica en su diversas modalidades y segn lo requiera cada paciente. Otro grupo de manifestaciones clnicas estar determinado por
los trastornos a nivel hemodinmico que se derivan de los cortos circuitos de derecha a izquierda as como por la repercusiones por la
acidosis y la hipoxemia, todo esto conllevar a trastornos de perfusin a todos los niveles con repercusiones y manifestaciones clnicas
de cada uno de los rganos afectados (falla renal, falla cardiaca, trastornos por hipoperfusin a nivel cerebral, intestinal, etc.). La misma
hipoxemia favorecer incremento en las resistencias pulmonares y como consecuencia datos de hipertensin pulmonar con lo que se
agravar la hipoxemia y acidosis, y ameritar manejos ms enrgicos para poder restituir la oxigenacin adecuada de todos los tejidos.
La misma prematurez de estos pacientes favorece ms complicaciones como la hipotermia que deber ser corregida en forma
oportuna ya que de lo contrario generar acidosis y est a su vez vasoconstriccin e hipoxemia. El diagnstico se podr sustentar en
base al cuadro clnico as como a los hallazgos a nivel laboratorial y radiogrfico. Despus del nacimiento, el surfactante pulmonar
puede evaluarse en el lquido amnitico, ya que parte del lquido pulmonar fetal entra en la bolsa amnitica y por lo tanto medirse la
lecitina de manera conjunta con la SP-C, pero con algunos cambios en la cantidad de lquido amnitico puede alterar la concentracin
de la SP-C. Otra prueba de laboratorio que puede ser utilizada es la relacin entre la lecitina y la esfingomielina que permanece
relativamente constante a lo largo de toda la gestacin y se expresa como el ndica L/S el cual se incrementa en forma lenta y gradual
de la primera a la semana 32 de gestacin, el ndice es de 2 hacia la semana 28 y de l hasta cerca del trmino de la gestacin; la
incidencia de sndrome de dificultad respiratoria es slo de 0.5% cuando el radio es de 2, pero es cercana al 100% si el radio es menor
de 1; entre 1 y 2 , el riesgo disminuye de modo progresivo. Tambin resulta til la medicin de los niveles de fosfatidilinositol el cual
aumenta rpidamente en el lquido amnitico hasta la semana 36 y despus disminuye, por medio de cromatografa se puede
determinar su porcentaje del total de los lpidos, si es menor de 1% se correlaciona con sndrome de dificultad respiratoria. Estas dos
ltimas pruebas de laboratorio son los mejores predictores de esta patologa con una correlacin conjunta del 80%.Tambin se han
realizado mediciones de la SP-A y SP-C pero sus resultados no son tan confiables. Por otra parte, sin que sean indicadores del sndrome
de dificultad respiratoria, se deber determinar estudio gasomtrico el cual nos mostrar diversos grados de acidosis, hipoxemia e
hipercapnia, los cuales nos indicaran respuesta al manejo instalado. El estudio por medio de radiografas de trax nos reportara gran
utilidad, y nos mostrar un aumento difuso de la densidad en ambos campos pulmonares con una apariencia granular muy fina dando
la apariencia de un vidrio esmerilado o de vidrio despulido , as como tambin se aprecian a las vas areas con mayor densidad que
MANUAL DE TRABAJO DEL CURSO ENARM CMN SIGLO XXI
CURSO ENARM CMN SIGLO XXI TEL: 36246001 Pharmed Solutions Institute PGINA 349

los campos pulmonares produciendo una imagen de broncograma areo y los diafragmas se muestran habitualmente elevados , todos
estos cambios se deben a la prdida de volumen pulmonar por colapso alveolar. TRATAMIENTO: Debe ser integral respetando siempre
la secuencia recomendada y altamente probada como es: pasos iniciales de la reanimacin, ventilacin, compresin torcica, as como
la intubacin y uso de medicamentos. El soporte ventilatorio deber ser oportuno y el necesario para cada paciente en particular y
podr ser: Oxigenoterapia en fase I (casco ceflico, puntas nasales) con oxgeno a flujo libre y en las concentraciones necesarias de la
fraccin inspirada de oxgeno (FiO2) para dar un soporte real al paciente y lograr una oxigenacin tisular adecuada. Manejo con presin
positiva continua en vas areas (CPAP), donde la presin suministrada as como la fraccin inspirada de oxgeno debern ser
controladas para proporcionar un adecuado intercambio gaseoso y evitar en lo posible complicaciones. Ventilacin mecnica en sus
diversas modalidades: Ventilacin convencional, sincronizada, activada por el paciente, de volumen controlado, de ayuda proporcional,
de alta frecuencia. La finalidad del soporte ventilatorio es lograr un adecuado intercambio gaseoso reclutando los alvolos colapsados
por el dficit de surfactante, y de esta manera mejorar la acidosis, la hipoxemia y la hipercapnea. Este reclutamiento alveolar se logra
manteniendo una presin positiva continua al final de la expiracin. (PEEP) debido a que el llamado PEEP fisiolgico es de 2, se
recomienda rangos por arriba de 4 cmH2O. El tratamiento de restitucin de surfactante ha logrado disminuir la mortalidad hasta en un
50% de los casos del sndrome de dificultad respiratoria; en la actualidad hay dos grupos de surfactante aprobados por la FDA: 1.-
surfactante natural (se obtiene de fuentes humanas o animales). 2.- surfactante sinttico. Las modalidades del tratamiento con
surfactante pulmonar exgeno incluyen la modalidad profilctica (Admn. En UTQ (100 mg/kg), valorar de requrirse Administrar en
primera hr vida 100 mg/kg) y la de rescate (100-200 mg/kg si clnica y FIo2 >40%, Rx Compatible).

CASO CLINICO
RN masculino de edad gestacional de 39 semanas, de 3,8 kg hijo de madre diabtica el cual se obtuvo por cesaria previa administracin
de esterioides a la madre, el cual requiri de estimulacin vigorosa cursando con hipotermia que mejoro en incubadora, se observo
posteriormente, aleteo nasal, retraccin esternal, quejido respiratorio, cianosis y polipnea, la Rx mostro un patro retculo granular
difuso bilateral y disminucin de la expansin pulmonar, PaO2 < 50 mmHg.

PREGUNTA
Cual es la conducta a seguir.

RESPUESTA
a.- Presin positivo nasal cotinua.
b.- Oximetria de pulso.
c.- Intubacin traqueal.
d.- Ventilacin mecnica.

CASO CLINICO
RN de 31 semanas que presenta dificultad respiratoria. Tiene 2 horas de nacido por parto vaginal. La dificultad respiratoria tiende a
incrementar. Madre diabtica, salvo la edad gestacional no se identificaron otras complicaciones durante el embarazo. EF: Temp. 37.5
C, TA:86/58, FR:60/min, FC:148/min. Marcadamente taquipneico, con retracciones supraesternales y supraclaviculares, aleteo nasal,
quejido. Faringe normal. CP normal. Usted decide intubacin endotraqueal, toma laboratorios y RX de trax. RX de trax: Infiltrado
bilateral, difuso, vidrio despulido en ambos pulmones, sin datos de atrapamiento de aire.

PREGUNTA
Cual de los siguientes resultados de la gasometra es menos frecuente observar?

RESPUESTA
a.- Hipoxemia.
b.- Hipercapnia.
c.- Acidosis metablica.
d.- Acidosis respitoria.










MANUAL DE TRABAJO DEL CURSO ENARM CMN SIGLO XXI
CURSO ENARM CMN SIGLO XXI TEL: 36246001 Pharmed Solutions Institute PGINA 350

TAQUIPNEA TRANSITORIA DEL RECIEN NACIDO (TTRN): CIENCIAS BASICAS: Es un proceso respiratorio no infeccioso que se presenta
con ms frecuencia en los recin nacidos de termino o cercanos a trmino, donde hay retencin de lquido pulmonar. Se inicia en las
primeras horas y se caracteriza por la presencia de taquipnea (FR >60x), insuficiencia respiratoria y aumento de requerimiento de
oxgeno, con niveles de CO2, normales o ligeramente aumentados. Es un proceso generalmente benigno y autolimitado que se resuelve
aprox. De 24-72hrs. Factores que favorecen la aparicin de TTRN: maternos; asma, DM, tabaquismo, manejo abundante de lquidos,
sedacin por tiempo prolongado, RPM >24hrs. Recin nacido; nacimiento por cesrea sin trabajo anterior, parto precipitado,
nacimiento cercano a trmino o termino , asfixia perinatal, sexo masculino, macrosomia, demoras en el pinzamiento del cordn
umbilical, sedacin materna excesiva, embarazo mltiple. SALUD PUBLICA: Incidencia: 1 a 2% de los recin nacidos. Recin nacidos
trmino, cercanos a trmino, o prematuros grandes. La TTRN abarca del 35-50% de los casos de insuficiencia respiratoria no infecciosa
de los recin nacidos que ingresan a los servicios de neonatologa. PATOGENIA: 3 factores: 1. Retraso en la resorcin de lquido
pulmonar fetal 2. Inmadurez pulmonar 3. Deficiencia ligera de surfactante. Al momento del nacimiento, el epitelio pulmonar del RN que
durante el embarazo es un activo secretor de cloro (Cl
-
) y liquido hacia los alveolos, tiene que cambiar para convertirse en un activo
absorbedor de sodio (Na
++
) y liquido con el objeto de remover este ltimo, que est condicionado por la presencia de catecolaminas
secretadas durante el trabajo de parto que estimulan los llamados canales epiteliales de Na
++
. La TTRN es el resultado de alveolos que
permanecen hmedos al no producirse esta reabsorcin en forma adecuada. El nio nacido por cesrea o el que nace
precipitadamente por va vaginal tiene mayor riesgo de tener exceso de lquido pulmonar como resultado de no haber experimentado
las fases de la labor y la falta de exposicin a las catecolaminas mencionadas. El resultado final son alveolos que retienen lquido
(comprometiendo el intercambio gaseoso que favorece la hipoxemia), el cual se acumula poco a poco en el intersticio, hasta que es
removido por los vasos linfticos o pasa al torrente sanguneo. El acumulo de lquido produce edema intersticial y disminucin de la
distensibilidad pulmonar, siendo esto ltimo la causa de taquipnea (compensatoria), y colapso parcial bronquiolar que condiciona a su
vez atrapamiento areo. Durante el transcurso de las siguientes horas el lquido es removido progresivamente, mejorando la
oxigenacin y disminuyendo la frecuencia respiratoria. DIAGNOSTICO: Taquipnea (>60 x), dificultad respiratoria de cualquier grado
evaluado de acuerdo a la escala de Silverman, cianosis en casos graves, campos pulmonares sin estertores. Laboratorio y gabinete:
Gasometra arterial; hipoxemia, CO2 en limite normal o ligeramente aumentado, acidosis respiratoria compensada. Radiografa de
trax; Imgenes de atrapamiento areo (rectificacin de arcos costales, herniacin de parnquima, hiperclaridad, aumento de espacios
intercostales, aplanamiento de diafragmas), cisuritis, congestin parahiliar, cardiomegalia aparente. Si despus de 72 hrs la taquipnea
no remite o incrementa pensar en otra patologa y/o en complicaciones. TRATAMIENTO: Mantener en todo momento ambiente
trmico neutro. La forma e inicio de alimentacin se determina de acuerdo al estado clnico de RN (alimentacin por succin, sonda
orogstrica). Se mantendr en ayuno en los siguientes casos: FR >80xy dificultad respiratoria con Silverman mayor de 3. El volumen de
lquidos IV ser de acuerdo a las necesidades para la edad gestacional y el peso. Si existiera alguna patologa asociada a la TTRN el inicio
y forma de alimento ser a juicio del mdico. Manejo y estimulacin mnimos. Farmacolgico: oxigenoterapia; habitualmente no
requieren FiO2 mayores al 40%. El objetico es obtener saturaciones por plsometria en el rango de 88-95%. No se recomienda el uso de
medicamentos ya que no existe evidencia suficiente de su eficacia y seguridad en RN. La administracin postnatal de epinefrina con el
fin de estimular la reabsorcin de lquido pulmonar, ha sido motivo de estudios experimentales sin que exista al momento evidencia.
Los esteroides antenatales a madres entre las 34-37 SDG. Podra tener un efecto benfico al disminuir la morbilidad respiratoria de los
RN, aun se requieren estudios para establecer su recomendacin.

CASO CLINICO
Se trata de paciente masculino de 15 horas de vida estrauterina, el paciente naci via cesarea, naci a la 35 semanas de gestacion, se
administro maduradores a la madre debido a trabajo de parto distcico con ruptura de membranas 12 horas antes de la intervencin
quirrgica, a la exploracin fsica se observo taquipnea de 101 rpm, y signo moderados de dificultad respiratoria.

PREGUNTA
Considerando un propable diagnostico clnico realizado, cual de los diagnosticos diferenciales es menos probable encontrar?

RESPUESTA
a.- Sindrome de adaptacin pulmonar.
b.- Sindrome de aspiracin de meconio.
c.- Enfermedad de membrana hialina.
d.- Neumonia neonatal hospitalaria.

PREGUNTA
Se decide realizar radiografia de torax, cuales son los datos no es probale espera encontrar para confirmar el diagnostico?

RESPUESTA
a.- Broncograma areo.
b.- Hilio congenstivos.
c.- Derrame cisurales.
d.- Sobredistencion pulmonar.

PREGUNTA
Cuales son los factores de riesgo inducido para la taquipnea trasitoria del recin nacido, cual es la mas probable en el caso clnico?.

RESPUESA
a.- Sexo masculino
b.- Grandote.
MANUAL DE TRABAJO DEL CURSO ENARM CMN SIGLO XXI
CURSO ENARM CMN SIGLO XXI TEL: 36246001 Pharmed Solutions Institute PGINA 351

c.- Cesarea.
d.- Sedacin materna.

PREGUNTA
Cual es el fenmeno fisiopatolgico no es la mas adecuada para el caso?

RESPUESTA
a.- Falta de absorcin de lquido amniotico.
b.- Falta administracin de esteroides.
c.- Deficienca ligera de surfactante.
d.- Edema pulmonar transitorio.

PREGUNTA
Considerando el diagnotico actual, el cual muestra singos de dificultad respiratoria, resultados de laboratorio normales al momento,
FiO2 < 0.40 descartando los diagnosticos diferencias, cul es la conducta a seguir en este caso es menos adecuada?

RESPUESTA
a.- Monitoreo de signos vitales.
b.- Monitoreo de gases por puncion cada 6 horas.
c.- Mantener glucosa y electrolitos.
d.- O2 por casco al 70 %

PREGUNTA
Tomando en cuenta la evolucin del caso, cual es su pronostico?

RESPUESTA
a.- Bueno.
c.- Malo.
b.- Fatal.
d.- Excelente.

PREGUNTA
Considerando un propable diagnostico clnico realizado, cual de los diagnosticos diferenciales es menos probable encontrar?

RESPUESTA
a.- Sindrome de adaptacin pulmonar.
b.- Sindrome de aspiracin de meconio.
c.- Enfermedad de membrana hialina.
d.- Neumonia neonatal hospitalaria.

PREGUNTA
Se decide realizar radiografia de torax, cuales de los datos no es probale espera encontrar para confirmar el diagnostico?

RESPUESTA
a.- Broncograma areo.
b.- Hilio congenstivos.
c.- Derrame cisurales.
d.- Sobredistencion pulmonar.

PREGUNTA
Cuales son los factores de riesgo inducido para la taquipnea trasitoria del recin nacido cual es la mas probable en el caso clnic?.

RESPUESA
a.- Sexo masculino
b.- Grandote.
c.- Cesarea.
d.- Sedacin materna.

PREGUNTA
Cual es el fenmeno fisiopatolgico no es la mas adecuada para el caso?

RESPUESTA
a.- Falta de absorcin de lquido amniotico.
b.- Falta administracin de esteroides.
c.- Deficienca ligera de surfactante.
d.- Edema pulmonar transitorio.
MANUAL DE TRABAJO DEL CURSO ENARM CMN SIGLO XXI
CURSO ENARM CMN SIGLO XXI TEL: 36246001 Pharmed Solutions Institute PGINA 352


PREGUNTA
Considerando el diagnotico actual, el cual muestra singos de dificultad respiratoria, resultados de laboratorio normales al momento,
FiO2 < 0.40 descartando los diagnosticos diferencias, cul es la conducta a seguir en este caso es menos adecuada?

RESPUESTA
a.- Monitoreo de signos vitales.
b.- Monitoreo de gases por puncion cada 6 horas.
c.- Mantener glucosa y electrolitos.
d.- O2 por casco al 70 %

PREGUNTA
Tomando en cuenta la evolucin del caso, cual es su pronostico?

RESPUESTA
a.- Bueno.
c.- Malo.
b.- Fatal.
d.- Excelente.

CASO CLINICO
Un RN de 3,6 kg naci a las 37 semanas de gestacin hijo de diabtica con un agente hipoglicmico oral. Obetnido por cesrea electiva
con Apgar de 6/9. Desarrollo taquipnea inmediatamente despus de su nacimiento y requirio oxgeno suplementario. En la gasometria
presento; fueron PO2 de 57 mm Hg, PCO2 de 52 mm Hg, y pH de 7,31. El nio se mantuvo en oxgeno por campana. A las 2 horas el
paciente se encuentra hipotnico, ciantico, con saturacin del 70 %. La Rx de observa rectificacin, hiperclaridad e incremento de los
espacios intercostales y congestin parahiliar.

PREGUNTA
Cual es la medida inmediata a seguir.

RESPUESTA
a.- Intubacion orotraqueal.
b.- Alimentacin por sonda orogastrica.
c.- Mantener un ambiente neutro.
d.- Realizar medidas de reanimacin.

PREGUNTA
Cual de las siguientes datos radiolgicos es menos probable para el diagnostico radiolgico?

RESPUESTA
a.- Aumento de la trama vascular con imgenes algodonosas.
b.- Liquido en cisusras interlobales.
c.- Abatimiento de diafragma.
d.- Cardiomegalia.

PREGUNTA
Cual de los siguientes diagnostico diferenciales es mas frecuente?

RESPUESTA
a.- Sepsis.
b.- Neumona
c.- Asfixia perinatal.
d.- Cardiopata congenita.

CASO CLINICO
Un RN de 3,4 kilos naci a las 40 semanas de gestacin con madre diabtica tratada con insulina. El nio fue obtenido por cesrea.
Puntuacin de Apgar fue 5/6. Se presento taquipnea una hora despus en observacin, el nio fue colocado en oxgeno con casco
ceflico. Las determinaciones de gases en sangre arterial en ese momento eran de PO2 52 mm Hg, pCO2 de 48 mm Hg, y pH de 7,31.

PREGUNTA
Cual es la conducta inicial a seguir.

RESPUESTA
a.- Iniciar con bicarbonato.
b.- Verificar laboratorios.
c.- Realizar estudios de gabinete.
MANUAL DE TRABAJO DEL CURSO ENARM CMN SIGLO XXI
CURSO ENARM CMN SIGLO XXI TEL: 36246001 Pharmed Solutions Institute PGINA 353

d.- Colocacion de factor surfactante.

CASO CLINICO
RN femenino de 31 SDG "despus de un embarazo sin complicaciones, con peso de 1480 g, y Apgar de 2/6. Fue obtenido por cesaria
electiva por cesaria previa por periodo intergenesico de 6 meses. A los pocos minutos de nacer, su frecuencia respiratoria fue de 61,
con significativo aleteo nasal, uso prominente msculos accesorios y retracciones subcostales. Su ritmo cardaco era normal (121
latidos / min). Haba signos de cianosis.

PREGUNTA
La paciente se encuentra en un hospital rural de concentracin, cual es su conducta a seguir.

RESPUESTA
a.- Trasladarla a una unidad de cuidados intensivos.
b.- Vigilancia estrecha ya que su evolucin es buena.
c.- Los datos radiogrficos no son importantes.
d.- Verificar el diagnostico inicial

















































MANUAL DE TRABAJO DEL CURSO ENARM CMN SIGLO XXI
CURSO ENARM CMN SIGLO XXI TEL: 36246001 Pharmed Solutions Institute PGINA 354

SINDROME DE ASPIRACION MECONIAL (SAM). CIENCIAS BASICAS: El sndrome de aspiracin meconial (SAM) se manifiesta con distrs
respiratorio y es producido por la aspiracin de lquido amnitico (LA) teido con meconio intra tero o intra parto. Constituye una
causa de morbimortalidad en el recin nacido, principalmente en el nio a trmino y postrmino. Factores de riesgo: Embarazo
postrmino, preeclampsia- eclampsia, hipertensin materna, DM materna, oligohidramnios, tabaquismo intenso, puntaje de Apgar
(igual o menor a 6) bajo a los 5 minutos. Hipoxia aguda intraparto, hipoxia perinatal crnica, frecuencia cardiaca fetal anormal, RN
pequeos para la edad gestacional. SALUD PUBLICA: Se reporta con frecuencia lquido meconial 14%. 10% SAM. Mortalidad 12 % y
adems deja secuelas. Se presenta 9-15% de nacidos vivos. La expulsin de meconio rara vez es antes de las 37 semanas. Puede ocurrir
30% de los embarazos que continan despus de las 42 SDG. PATOGENIA: 1. Evacuacin del meconio in-tero: La asfixia y otras formas
de estrs intrauterino pueden causar un aumento del peristaltismo intestinal, con relajacin del esfnter anal externo y evacuacin de
meconio. El efecto de la hipoxia intrauterina sobre el peristaltismo y el tono esfinteriano parece aumentar con la edad gestacional. 2.
Aspiracin de meconio: Despus de la evacuacin de meconio en el lquido amnitico, las respiraciones jadeantes del feto asfixiado, ya
sea in tero o durante el trabajo de parto, pueden determinar la aspiracin del lquido amnitico teido con meconio hacia las vas
areas grandes del pulmn. El meconio espeso provoca obstruccin de la va area, lo que ocasiona dificultad respiratoria. Con la
aspiracin distal de meconio puede ocurrir una obstruccin parcial o total de la va area. En las reas de obstruccin total se
desarrollan atelectasias; en cambio, en las reas de obstruccin parcial ocurre un fenmeno valvular que ocasiona atrapamiento areo
e hiperinsuflacin pulmonar. El atrapamiento areo aumenta el riesgo de escape de aire del 21 al 50%. Finalmente, se desarrolla
neumonitis intersticial y qumica, con edema bronquiolar y estrechamiento de las vas areas de pequeo calibre. La ventilacin
desigual debida a reas de obstruccin parcial y neumonitis sobreagregada produce retencin severa de dixido de carbono e
hipoxemia. La resistencia vascular pulmonar aumenta como resultado de la hipoxia, la acidosis y la hiperinsuflacin de los pulmones. El
aumento de la resistencia vascular puede conducir a un cortocircuito de derecha a izquierda auricular o ductal y a una mayor
desaturacin. CLASIFICACION: Clnicamente se clasifica en: a) Leve: discreta polipnea, hiperinsuflacin torcica. No existe alteracin en
la PO2 ni en la PCO2 FiO2 <0,4. b) Moderada: hipercarbia, cianosis. Necesidad de FiO2 progresivamente creciente en las primeras 12-24
horas, incluso VM. Pueden desarrollar neumotrax o insuficiencia cardiaca hipxica. c) Grave: Hipoxemia e hipercarbia desde el
nacimiento, que precisa VM con FiO2 altas y medidas de soporte cardiovascular, auscultacin roncus y estertores diseminados.
DIAGNOSTICO: Clnico; Los sntomas dependen de la severidad de la lesin hipxica y de la cantidad y de la consistencia del meconio
aspirado. Es frecuente que su inicio clnico sea precoz, progresivo a lo largo de 12 a 24 horas, con hipoxemia, dificultad respiratoria,
taquipnea, cianosis persistente, hipoxemia, de saturaciones frecuentes, labilidad en el manejo clnico. Los neonatos con lquido
amnitico teido con meconio suelen mostrar signos de posmadurez; son pequeos para la edad gestacional y tienen uas largas, piel
descamada teida con pigmento amarillo o verde y cordn umbilical teido de meconio, mancha o unto sebceo. Pueden presentar
depresin respiratoria en el momento de nacer, con escaso esfuerzo respiratorio y tono muscular disminuido si ha existido una asfixia
perinatal importante. Laboratorio; niveles de gases en sangre arterial revelen hipoxemia. La hiperventilacin puede producir alcalosis
respiratoria en los casos leves; pero los lactantes con enfermedad grave suelen manifestar acidosis respiratoria con retencin de
dixido de carbono, debido a obstruccin de la va area y neumonitis. La radiografa de trax tpica muestra hiperinsuflacin de los
campos pulmonares y diafragmas aplanados. Hay infiltrados focales irregulares y gruesos con lquido pulmonar aumentado alternando
con zonas hiper aireadas (imagen en panal de abejas). Puede haber neumotrax o neumomediastino en el 10-40% de los casos.
TRATAMIENTO: En la sala de parto; el obstetra deber aspirar en contenido de la nariz y oro faringe antes del parto de trax con
perilla. Si hay aspiracin de meconio y el RN no est vigoroso (respiracin inadecuada, tono muscular disminuido, FC <100 lpm), est
indicada aspiracin traqueal directamente. Antes de que establezca las respiraciones. Introducir laringoscopio y utilizar sonda de
aspiracin de 12 F a 14F para aspirar la boca y la faringe posterior y as visualizar la glotis. Introducir el tubo endotraqueal en la
trquea, conectar a equipo de aspiracin, aspirar a medida que se retira la sonda. Po otra parte aumentar la concentracin de oxigeno
inspirado, monitorear gases sanguneos, CPAP (presin de la va area positiva continua) individualizar cada caso, ventilacin mecnica,
ventilacin de alta frecuencia (ventiladores oscilatorios o jet). Corticoesteroides: no hay evidencia. En recin nacidos con SAM masivo e
hipertensin pulmonar persistente, el manejo con ventilacin de alta frecuencia y xido ntrico ha mejorado su pronstico; la
incorporacin de oxigenacin con membrana extracorprea, (no disponible an en nuestro medio) se muestra como un arma
prometedora en el manejo de este tipo de pacientes. La inactivacin del surfactante por el propio meconio y/o por la coexistencia de
edema pulmonar, hace suponer que algunos de estos RN se beneficiaran de la administracin de surfactante. En este sentido, estn en
marcha estudios corroborativos para demostrar la efectividad de esta nueva arma teraputica. COMPLICACIONES: Neumona por
aspiracin de meconio. Neumonitis. Hipoxemia. Di estrs respiratorio. Acidosis metablica. Obstruccin mecnica de las vas areas.
Escape areo: 10-20% neumotrax o neumomediastino. Hipertensin pulmonar persistente del recin nacido.

CASO CLINICO
RN femenino de trmino obtenido por parto vaginal espontneo con un peso al nacer de 3.500 g. el monitoreo intraparto no revelaron
evidencia de sufrimiento fetal. Poco despus del parto, fue ingresado en una unidad de cuidados intensivos debido a la presencia de
meconio, vmitos y succin debil, su temperatura era inferior a 36,0 C, el pulso fue de 148 lat / min, y su frecuencia respiratoria era de
72 respiraciones / min. El tiempo de protrombina y el tiempo parcial de tromboplastina activada eran 20,7 y 54,6 s, respectivamente. La
orina y sangre se sometieron a la cultivo. La radiografa de trax mostr infiltrados y rayas gruesas del campo pulmonar derecho.

PREGUNTA
Considerando la gravedad del cuadro cual es la complicacin ms probable en esta paciente.

RESPUESTA
a.- Sepsis.
b.- Acidosis respiratoria.
c.- Neumonia.
d.- Coagulopatia.

MANUAL DE TRABAJO DEL CURSO ENARM CMN SIGLO XXI
CURSO ENARM CMN SIGLO XXI TEL: 36246001 Pharmed Solutions Institute PGINA 355

PREGUNTA
Cual de las siguientes manifestaciones es mas importante para el pronostico?

RESPUESTA
a.- La cantidad de meconio aspirado.
b.- La edad gestacional.
c.- Manifestaciones de neumonitis qumica.
d.- Manifestaciones de posmadurez.

PREGUNTA
Cual de las siguientes alteraciones es importante mantener en la estabilizacin mediata del paciente?

RESPUESTA
a.- Taquipnea.
b.- Cianosis persistentes
c.- Desaturaciones.
d.- Cambios minimos se inestabiliza.

PREGUNTA
Cual de las siguientes medidas es menos importantes en la reanimacin del paciente?

RESPUESTA
a.- No estimular.
b.- No frotarlo.
c.- No voltearlo ni manipular.
d.- Aspirar primero la nariz y la orofaringe,

PREGUNTA
Cual de las siguientes manifestaciones es mas frecuente encontrar en esta patologia?

RESPUESTA
a.- Infiltrados.
b.- Hiperinsuflacion.
c.- Atelectasia semegtadas o globales
d.- Puede haber neumotrax.

PREGUNTA
Cual de las siguientes complicaciones es la menos frecuente en esta patologia.

RESPUESTA
a.- Hipertension pulmonar persistente.
b.- Acidosis con lactato bajo.
c.- Neumonitis qumica.
d.- Sindrome de escape areo.






















MANUAL DE TRABAJO DEL CURSO ENARM CMN SIGLO XXI
CURSO ENARM CMN SIGLO XXI TEL: 36246001 Pharmed Solutions Institute PGINA 356

NEUMONIA NEONATAL (NNN). CIENCIAS BASICAS: Ocurre como una complicacin de infecciones connatales o de infecciones
nosocomiales. Neumona intrauterina: se adquiere va intrauterina o durante el paso de canal de parto. Neumona nosocomial: se
adquiere en su estancia hospitalaria, procesos invasivos, intubacin, asistencia ventilatoria, intubacin, permeabilizacin de vasos.
Despus de las 72 h. Neumona adquirida durante el nacimiento, por contaminacin durante el paso a travs del canal de parto, aspirar
meconio o lquido amnitico infectado. La neumona puede aparecer 2 4 semana de vida. Factores de riesgo: maternos; RPM, IVU
materna dentro de 15 das antes del parto, colonizacin vaginal patolgica. En el RN; menor dimetro de rbol bronquial y escaso
desarrollo de aparato ciliar, bajos niveles de IgM, complemento, opsoninas y funcin linfocitaria, trauma de la va area (intubacin
prolongada, aspiraciones profundas) y presencia de meconio en va area, corioamnioitis. SALUD PUBLICA: La incidencia en el RNT es
menor de 1% y en el RNPT de 10-20%, dependiendo de cada UCI Neonatal. El pulmn es el rgano que con mayor frecuencia se
compromete en las infecciones que se desarrollan en las primeras 24 horas de vida. Mortalidad: 20-50%, segn la edad gestacional y
complicaciones asociadas. PATOGENIA: Etiologa: NNN de inicio precoz = Complejo TORCH, E. Coli, Streptococo del grupo B, listeria y
menos frecuentemente agentes virales (herpes simple, CMV, influenza, rubola, ADV y echovirus. NNN de inicio tardo = Staphylococos,
Klebsiella, Pseudomona, Enterovirus, E. Coli, Cndida Albicans y algunos virus como CMV (por transfusiones). DIAGNOSTICO: Clnica;
Inespecfica, se manifiesta por dificultad respiratoria (polipnea, quejido y cianosis) asociado o no a un signo clnico de infeccin
(palidez, mal llene capilar, alteraciones de termorregulacin, etc.) que se agrava rpidamente si no se instaura un tratamiento
adecuado. Considerar factores de riesgo descritos. Laboratorio: hemograma (leucocitos fuera de rangos normales con predominio de
neutrfilos en infecciones bacterianas, linfocitario o de monocitos en infecciones virales, eosinfilos por chlamydia trachomatis),
protena C reactiva, gasometra, glicemia, calcemia, Test Ltex, antgenos capsulares (en orina, LCR). Cultivos: de sangre, secrecin
traqueal (tiles si se toman precozmente). Radiografa de trax. Radiolgico: inespecfico, infiltrado denso alveolar en uno o ambos
pulmonares. Un dato importante es la persistencia de la imagen radiolgica por ms de 48 horas. TRATAMIENTO: Medidas generales:
monitorizacin, ambiente trmico neutral, balance H-E y cido base. Antibiticos: inicia terapia de amplio espectro y luego, si existe
confirmacin adecuar terapia especfica segn la epidemiologa de cada unidad de RN. Neumona intrauterina: ampicilina-amikacina 7-
10 das. Neumona nosocomial: vancomicina, cefotaxima. NNN inicio precoz: Ampicilina y Gentamicina. NNN inicio tardo: Cloxacilina y
Amikacina. Manejo respiratorio segn gravedad, oxigenoterapia, ventilacin mecnica y otras medidas ms recientes cuando se
obtengan (xido ntrico, intubarlo). COMPLICACIONES: Choque sptico. Foco infeccioso en meninges, articulaciones. Hipertensin
pulmonar.

CASO CLINICO
Paciente masculino 3 dias que presenta datos de dificultad respiratoria con rechazo a la alimentacin, su nacimiento fue a las 35
semanas de gestacion, cabe destacar presencia de conjuntivitis, parto distocico, por via vaginal, la radiografia es inespecficas, solo un
infiltrado denso alveolar, la imagen fue persistente, los laboratorios mostraron eosinofilia, no recibi factor surfatante pero recibi
oxigeno al 100 %, cabe destacar que mientras se encontraba en unidad de cuidados intensivos presento distencin abdominal.

PREGUNTA
Cual es el agente etiolgico mas probable?

RESPUESTA
a) Pseudomona.
b) Candida
c) e. coli
d) Cloacale

PREGUNTA
Cual es la conducta farmacologa a seguir considerando que los cultivos aun no tienen datos para ser dirigido?

RESPUESTA
a.- Vancomicina 74mg/Kg y cefotaxima 50mg/kg
b.- Cefotaxima 20 mg/ Kg y metronidazol 30 mg/Kg
c.- Ampicilina 100 mg /kg mas amikacina 15mg/kg
d.- Doxiciclina 15 mg/kg mas gentamicina 15 mg/kg

NEUMONIA NEONATAL TARDIA
Se trata de paciente masculino de 4 semanas de nacido, el cual nacin prematuro por ruptura de prematuras de membrana el cual
presento taquinea transitoria del recin nacido, inicia progresivamente con irritabiliad

PREGUNTA
Cual es el agente causal mas frecuente en esta patologia?

ETIOLOGIA
a.- klepsiella.
b.- Stafilococo.
c.- Psudomona.
d.- E. coli.



MANUAL DE TRABAJO DEL CURSO ENARM CMN SIGLO XXI
CURSO ENARM CMN SIGLO XXI TEL: 36246001 Pharmed Solutions Institute PGINA 357

CASO CLINICO
Se trata de recin nacido de 3 horas de nacido, el cual nace prematuro, de madre adolecente con cervicovaginitis y parto distcico,
inicia con taquipnea, dificultad respiratoria, con aleteo nasal, con disociacin toraco abdominal, con mala tolerancia a la via oral, con
hipotermia, Los datos radiogrficos de control se observo persistencia de imgenes radiolgicas en ambos campos pulmonares tipo
infiltrado, con sndrome de fuga, el recuento de leucocitos fueron predominio eosinofilos.

PREGUNTA
Cual es el agente etiolgico mas probable?

RESPUESTA
a- Clanmydia
b.- Klepsiela
c.- Candida
d.- Garnerella

PREGUNTA
Cual es el tratamiento farmacolgico en espera de los resultados de cultivo?

a.- Ampicilina y amikacina.
b.- Ampicilina y gentamicina.
c.- Ceftriaxona y vancomicina.
d.- Imipenem y Cefotaxima.

SEPSIS NEONATAL. CIENCIAS BASICAS: Se define como un Sndrome de Respuesta Inflamatoria Sistmica (SRIS) en la presencia o como
resultado de infeccin probada o sospechada durante el primer mes de vida extrauterina. Segn la edad de presentacin puede ser
clasificada de manera arbitraria en sepsis temprana, si aparece en los primeros 3 das de vida (para algunos autores hasta los 7 das de
vida), que es debida generalmente a microorganismos adquiridos de va materna y sepsis tarda, la cual se presenta despus de los 3
das (algunos autores >5 dias) de vida extrauterina y es causada frecuentemente por microorganismos adquiridos despus del
nacimiento; esta ltima puede ser de adquisicin nosocomial o de la comunidad. Factores de riesgo: Sexo masculino, prematurez y bajo
peso al nacer, ruptura de membranas > 18 hrs, reanimacin al nacer y procedimientos invasivos, fiebre materna periparto.
Dependiendo de la gravedad de la infeccin y la intensidad de la respuesta inflamatoria, esta se presenta como: Sepsis (sndrome de
respuesta inflamatoria sistmica ms hemocultivo positivo) Sepsis grave (sepsis ms disfuncin orgnica, hipotensin o hipoperfusin)
Choque sptico (sepsis grave con hipotensin que no responde a carga de lquidos) y Falla orgnica mltiple. SALUD PUBLICA:
Incidencia: 1 a 5 por cada 1000 nacidos vivos. 4 millones mueren de sepsis en las primeras 4 semanas. Cerca de un 85% de los pacientes
requieren ventilacin mecnica. PATOGENIA: Etiologia: Sepsis neonatal temprana: E. Coli, Klebsiella sp, Enterococcus sp. Streptococcus
agalactiae y Listeria monocytogenes. Sepsis neonatal tarda: Staphylococcus coagulasa negativa, staphylococcus aureus, Enterococcus
sp, E Coli, Klepsiella, Candida spp, Streptococcus del grupo B. La presencia de los mediadores hormonales e inmunolgicos que
interviene directamente en la modulacin de la respuesta inflamatoria a la infeccin, definen ms claramente los mecanismos
involucrados en la respuesta inflamatoria aguda a la infeccin, adems de la respuesta de anticuerpos, la activacin de las dos vas del
complemento, la participacin del sistema de coagulacin, la respuesta fagocitica mononuclear y polimorfonuclear y de la activacin
linfocitara, ciertos mediadores que son sintetizados principalmente por monocitos, linfocitos, fibroblastos y clulas endoteliales actan
de manera determinante en la regulacin y modulacin de la respuesta inflamatoria inmunolgica a la infeccin. Las citocinas actan
como mediadores de la comunicacin intercelular y efectores de la activacin de funciones de las clulas que tiene receptores para
estas protenas. Las cuales son: interleucinas, interferones y factores de crecimiento. Segn sea su funcin se pueden encontrar las de
actividad proinflamatoria (IL-1, IL-6, TNFalfa, IL-2, IL-8, IL-12) otras antiinflamatorias o inhibitorias (Antagonistas, Receptores solubles,
IL-4, IL-10, IL-13) y factores de crecimiento (GM-CSF, G-CSF, M-CSF, IL-3, IL-6, IL-5, IL-7). La magnitud de citosinas proinflamatorias que
se producen en etapa temprana de la sepsis se relaciona con la intensidad del dao pulmonar. DIAGNOSTICO: El diagnstico temprano
y oportuno de sepsis neonatal no es fcil porque las manifestaciones clnicas son inespecficas y pueden avanzar rapidamente a
estados ms avanzados. Los signos de alarma identificados por la Organizacin Mundial de la Salud (OMS) son los siguientes:
convulsiones, rechazo al alimento, dificultad respiratoria, hipoactividad, polipnea. Las manifestaciones clnicas son inespecficas y muy
variadas dependiendo de la gravedad de presentacin. Algunas de las principales son: distermias, dificultad respiratoria, ictericia,
apneas (con ms frecuencia en prematuros), distensin abdominal, hepatomegalia, letargia, sangrados, hipoactividad, palidez, oliguria,
cianosis, piel marmrea, crisis convulsivas, irritabilidad, esplenomegalia, vmito, diarrea, hipotensin arterial, petequias o equimosis,
trombocitopenia y acidosis. La sospecha clnica es lo principal para poder llegar al diagnstico de sepsis neonatal e idealmente
confirmarse con cultivos positivos en sangre, orina, lquido cefalorraqudeo (LCR) u otros sitios normalmente estriles. Ante cualquier
cuadro sugestivo de sepsis, se debe realizar estudio diagnstico completo. En sepsis temprana se deben incluir hemocultivos (central y
perifricos) y cultivo de lquido cefalorraqudeo. En sepsis tarda se debe incluir adems urocultivo. En casos de infecciones localizadas
(por ejemplo osteoartritis), se debe cultivar el sitio de infeccin. Prcticamente cualquier problema neonatal puede presentarse como
sepsis, debido a esto es importante que se realice una historia clnica y exploracin fsica adecuadas, toma de biometra hemtica
completa. Si bien no existe un biomarcador de sepsis ideal, existen mltiples estudios que apoyan la utilidad de procalcitonina y
protena C reactiva para el diagnstico de sepsis neonatal. Hemocultivo: sensibilidad 82%, especificidad 96%, BH: Leucocitos anormales:
< 6 000 clulas/mm o > 30 000 clulas/mm en las primeras 24 hrs. 50% RN puede tener conteos normales. TRATAMIENTO: Primera
eleccin: Ampilicina- amikacina. El manejo emprico inicial de antibiticos debe hacerse con base en la experiencia de cada hospital,
siempre teniendo en cuenta el patrn de resistencia y sensibilidad. En sepsis neonatal temprana el tratamiento debe iniciarse con
ampicilina y un aminoglucsido (gentamicina amikacina), en ocasiones especiales se puede sustituir el aminoglcosido por
cefotaxima, sobre todo si existe la sospecha de neuroinfeccin (est demostrado que esteriliza el LCR con mayor rpidez). En recin
MANUAL DE TRABAJO DEL CURSO ENARM CMN SIGLO XXI
CURSO ENARM CMN SIGLO XXI TEL: 36246001 Pharmed Solutions Institute PGINA 358

nacidos con sepsis tarda adquirida en la comunidad, es posible utilizar el mismo esquema, sin embargo en sepsis nosocomial, el
tratamiento debe estar orientado a combatir los microorganismos presentes en cada institucin. La evidencia actual de ensayos
clnicos controlados aleatorizados no apoya el uso rutinario de inmunoglobulina intravenosa y factor estimulante de colonias de
granulocitos. El uso de pentoxifilina como adyuvante en el manejo de sepsis reduce la mortalidad en neonatos pretrmino, sin embargo
debido a debilidades metodolgicas de los estudios al respecto, no es adecuado utilizarlo de manera rutinaria hasta la obtencin de
mejor evidencia. Dependiendo de las condiciones clnicas del paciente en el caso de sepsis grave o choque sptico se deber
proporcionar apoyo ventilatorio, suministro de lquidos, aminas e incluso corticoesteroides en el caso de hipotensin refractaria a las
mismas o en caso de sospecha de insuficiencia suprarenal. Es indispensable realizar la correccin del equilibrio cido base y
proporcionar apoyo calrico y nutricional ya sea por va enteral o parenteral segn sea el caso. Pacientes con riesgo de sepsis: iniciar
tratamiento revalorar 48 hrs. (el 95% de las infecciones se presenta en las primeras 24 hrs). Si no hay desarrollo de cultivos, biometra
hemtica normal y PCR normal suspender el tratamiento.

CASO CLINICO
Primigesta de 15 aos de edad, con gestacin de 40 semanas, presenta trabajo de parto en fase de dilatacin rpida (5 cm de
dilatacin) y expulsin de lquido amnitico (LA) con tinte meconial. Tuvo una rotura precoz de membranas 1 h 20 min antes del
ingreso. Se indic anestesia de conduccin con peridural continua y monitorizacin electrnica que revel una variabilidad menor de 5
LCF, una frecuencia cardaca fetal media de 150 latidos por min y una desaceleracin variable prolongada (hasta 120 LCF por minuto
durante 3 min). Se reevalu luego de nueva desaceleracin (60 LCF por minuto durante 7 min) y se encontr 8 a 9 cms de dilatacin,
presentacin ceflica en II plano y LA con grumos de meconio con leve mal olor. Se traslad inmediatamente a sala de partos con
dilatacin completa y presentacin ceflica en tercer plano. Se obtuvo un recin nacido (RN) de 3,440 g de sexo masculino, con muy
mal olor, con Apgar 1/3. El RN evolucion grave, con compromiso del SNC, encefalopata hipxica isqumica grado III, asociado a shock
sptico precoz, con compromiso multiorgnico y necesidad de apoyo ventilatorio y de drogas vasoactivas.

PREGUNTA
Cual es la complicacin ms probable en esta paciente.

RESPUESTA
a.- Coagulacion intravascular diseminada.
b.- Falla multiple.
c.- Neurosepsis.
d.- Neumonia asociada.

CASO CLINICO
Se trata de paciente de 35 semanas de gestacion de 5 dias de nacido, con ruptura prematura de membrana el cual requiri reanimacin
intensiva debido a pagar 2/6, se observo fiebre materna durante el trabajo de parto, la madre vive en medio rural, apnea, crisis
convulsivas, vomito, distermia, taquicardia, disnea y deterioro, hiporreactivo con piel marmrea, distencin abdominal, se diagnostico
sepsis neonatal.

PREGUNTA
Cuales de los diagnosticos diferenciales es el menos frecuente?

RESPUESTA
a.- Sindrome de adaptacin del recin nacido.
b.- Taquicardia transitoria del recin nacido
c.- Hemorragia ventricular.
d.- Neumonitis

PREGUNTA
Cual de los siguientes valores de laboratorio es mas probable esperara?.

RESPUESTA
a.- PCR elvada y neutrofilia.
b.- Leucocitosis con eosinofilia.
c.- Linfocitosis mas plaquetopenia.
d.- VSG elevada con leucopenia.

PREGUNTA
Cual es la conducta farmacolgica empirico de primera eleccion en espera de los resultados de cultivos.

RESPUESTA
a.- Ampicilina mas gentamicina.
b.- Ampicilina mas amikacina.
c.- Inmoglobulina y cefotaxima.
d.- Ampicilina y FEC.


MANUAL DE TRABAJO DEL CURSO ENARM CMN SIGLO XXI
CURSO ENARM CMN SIGLO XXI TEL: 36246001 Pharmed Solutions Institute PGINA 359

ENTEROCOLITIS NECROTIZANTE (ECN). CIENCIAS BASICAS: Se presenta como un sndrome gastrointestinal y sistmico que comprende
sntomas variados y variables, como distensin e hipersensibilidad abdominal, sangre en heces, intolerancia alimentaria, apnea, letargia
y en casos avanzados acidosis, sepsis, CID y shock. Constituye la urgencia gastrointestinal ms frecuente en las UCI neonatales. SALUD
PUBLICA: Es una enfermedad grave que afecta a recin nacido, en especial prematuros, con una incidencia y morbilidad elevados.
Incidencia 1-3 por 1000 RN vivos y 1-7.7% de los RN ingresados a unidades neonatales. Aumenta la incidencia en los menores de 1500g,
hasta un 2-10%. La edad gestacional media oscila en torno a las 31 semanas, con un peso medio al nacimiento de 1.460 g. PATOGENIA:
Se acepta un mecanismo multifactorial en el husped predispuesto. Entre los factores propuestos implicados en la patognesis de la
ECN se han descrito la prematuridad, alimentacin lctea, inestabilidad hemodinmica, infeccin y alteracin de la mucosa intestinal.
Solo la prematuridad y la alimentacin lctea tienen una base epidemiolgica consistente. En prematuros, la predisposicin de estos
nios pudiera explicarse por la inmadurez de su tracto gastrointestinal con funcin luminal limitada que conlleva una absorcin parcial
de carbohidratos y grasas as como proliferacin bacteriana, mayor permeabilidad de la mucosa e hipomotilidad. Junto a ello existe
inmadurez de los sistemas defensivos sistmicos y de la mucosa intestinal, entre otros la IgA secretora y la barrera de mucina. Se ha
especulado la posibilidad de un efecto protector de los glucocorticoides administrados prenatalmente. Una de las principales hiptesis
es que la ECN proviene de efectos nocivos de la microflora intestinal sobre una mucosa entrica lesionada o isqumica. La lesin
mucosa puede provenir de distintos procesos, incluyendo la asfixia o la isquemia secundaria a la derivacin de la sangre lejos del
intestino. Se desconoce la especificidad de este proceso. La flora microbiana involucrada en la ECN no es nica, aunque representa
organismos intestinales predominantes presentes en el recin nacido en el momento de la aparicin de la enfermedad. Estos
organismos pueden incluir las bacterias anaerbicas, como las especies de Clostridium. Por el momento no se ha definido un papel
especifico del Clostridium difficile o de una toxina especifica. Algunos virus pueden estar incluidos en este cuadro microbiolgico.
DIAGNOSTICO: El hallazgo ms precoz suele ser un cambio en la tolerancia alimentaria en un nio prematuro, con buena evolucin
hasta ese momento y que comienza a presentar restos gstricos. Los sntomas sistmicos asociados son inespecficos, desde aparicin
de apneas, alteracin del patrn respiratorio, distermia, inestabilidad hemodinmica con bradicardias, hasta hipotensin, letargia o
shock sptico y CID. Desde el punto de vista gastrointestinal, la ECN se presenta con distensin abdominal, restos gstricos, abdomen
doloroso, vmitos, diarrea o hematoquecia. El curso de la enfermedad vara segn recin nacidos. Con mayor frecuencia aparece como:
1) una presentacin fulminante, rpidamente progresiva de signos congruentes con necrosis intestinal y sepsis, y 2) como una
presentacin lenta, paroxstica, de distensin abdominal, leo y posible infeccin. Laboratorio y gabinete: La radiografa abdominal
(confirma diagnostico) suele revelar un patrn gaseoso anormal congruente con el leo. Estas radiografas pueden revelar un edema de
la pared intestinal, la posicin fija de un asa en estudios seriados, la aparicin de una masa, la neumatosis intestinal (95%) ms
frecuente en cuadrante inferior derecho, el aire venoso portal o heptica, la neumobilia o el neumoperitoneo (cuando hay perforacin
intestinal). Estudios sanguneos. La trombocitopenia, la acidosis metablica persistente y la hiponatremia refractaria grave son la triada
ms frecuente y ayudan a confirmar el diagnstico. La trombocitopenia se asocia a necrosis intestinal y empeoramiento clnico. El
anlisis de las heces para detectar sangre e hidratos de carbono, si los hay, se ha utilizado para diagnosticar a los recin nacidos afectos
de ECN, basado en los cambios de la integridad intestinal. La malabsorcin de los hidratos de carbono, segn refleja un Clinitest positivo
(cuerpos reductores) en las heces, puede ser un indicador frecuente y precoz de ECN en el marco de los signos. Otros hallazgos
bioqumicos inespecficos son la
elevacin srica de PCR y alfa-1-
glicoproteina o de la alfa-1-antitripsina
en heces. Ante la posibilidad de sepsis
deben recogerse cultivos (hemocultivo,
cultivo de LCR, coprocultivo). Si se
sospecha Clostridium son necesarios
cultivos especficos y determinacin de
toxina. La visualizacin directa de la
mucosa entrica mediante endoscopia
se ha utilizado para detectar a los recin
nacidos afectos de ECN, aunque la experiencia con esta tcnica es demasiado limitada para ser de aplicaci6n directa. Los avances
tecnolgicos pueden hacerla posible en el futuro. Debe considerarse la ultrasonografa como mtodo til en el diagnostico cuando
existe sospecha clnica, no confirmada por radiologa. CLASIFICACION: ETAPA I: sospecha a.- signos sistmicos leves: apnea,
bradicardia, inestabilidad trmica, letargia. b.- signos intestinales leves: distensin abdominal, restos gstricos (pueden ser biliosos),
sangre oculta en heces. c.- radiografa de abdomen: normal o con signos no especficos. ETAPA II: enfermedad definida (signos
radiolgicos positivos) a.- signos sistmicos moderados. b.- signos intestinales adicionales: silencio abdominal, dolor a la palpacin del
abdomen. c.- signos radiolgicos especficos: neumatosis intestinal o gas en el sistema portal. d.- alteraciones analticas: acidosis
metablica, leucopenia, trombocitopenia. ETAPA III: enfermedad avanzada: shock sptico y neumoperitoneo. a.- afectacin sistmica
grave: hipotensin arterial, signos evidentes de shock. b.- signos clnicos de peritonitis. c.- signos radiolgicos de gravedad:
neumoperitoneo. d.- alteraciones analticas: acidosis metablica y respiratoria, leucopenia y neutropenia, trombocitopenia, coagulacin
intravascular diseminada, protena C muy elevada. TRATAMIENTO: Medico: consiste en medidas de soporte, reposo intestinal,
nutricin parenteral, antibioterapia y correccin de alteraciones hematolgicas y/o electrolticas que pudieran estar presentes.
Medidas especficas son la dieta absoluta, descompresin intestinal con aspiracin, reposicin de lquidos considerando prdidas a un
tercer espacio, aporte calrico adecuado mediante nutricin parenteral y antibiticos endovenosos de amplio espectro. Deben
suspenderse todos los frmacos relacionados como posibles factores de riesgo. Como medidas de soporte se incluye la asistencia
respiratoria temprana ante la aparicin de apneas o patrn respiratorio acidotico, la correccin de la acidosis, hiponatremia o
trombopenia. La acidosis metablica persistente es un indicador de progresin de la lesin intestinal e incluso necrosis. Se debe
asegurar una perfusin y transporte de oxigeno adecuados, con un aporte suficiente de lquidos y manteniendo el nivel de hematocrito
>35%. Puede ser necesario el uso de agentes inotrpicos. La dopamina a dosis bajas puede ser de ayuda para mejorar la perfusin
sistmica y aumentar el flujo mesentrico. Habitualmente la ampicilina y gentamicina constituyen el tratamiento adecuado, debiendo
asociar clindamicina o metronidazol ante sospecha de grmenes anaerobios, si bien la clindamicina ha sido asociada con un aumento
MANUAL DE TRABAJO DEL CURSO ENARM CMN SIGLO XXI
CURSO ENARM CMN SIGLO XXI TEL: 36246001 Pharmed Solutions Institute PGINA 360

de la frecuencia de estenosis postenterocoliticas. La dieta y la antibioticoterapia han de mantenerse durante 10-14 das, con
introduccin progresiva de aporte enteral, con frmulas hipoosmolares de hidrolizado de protenas. Quirrgico: El momento ideal sera
aquel en que se ha producido una gangrena intestinal pero todava no existe perforacin, ni peritonitis secundaria. Indicada en
pacientes que progresa con necrosis de pared intestinal y perforacin debe ser urgente debido al rpido deterioro. La decisin es clara
en aquellos en que destaca la presencia de neumoperitoneo. El empeoramiento progresivo nos alerta sobre necrosis intestinal como:
signos de peritonitis como edema y eritema de pared, masa abdominal, trombopenia y acidosis persistentes. Paracentesis: un resultado
positivo es altamente especfico de necrosis intestinal.

CASO CLINICO
RN con parto en casa con control prenatal con 34 semanas de gestacin por FUM, con un peso 2100 g. 7 dias despues desarroll sepsis
y fue hospitalizado en la unidad de cuidados intensivos. El examen clnico encontr una distensin abdominal y una radiografa
abdominal mostr aire en las paredes del intestino y un neumoperitoneo grande (aire libre subdiafragmtico, aire periheptico libre,
doble signo de la pared). Requirio intubacion y se proporciono soporte inotrpico y antibiticos.

PREGUNTA
Cual es la causa ms probable de la complicacin.

RESPUESTA
a.- Nacimiento en casa.
b.- Edad gestacional
c.- Comorbilidad pulmonar.
d.- Sepsis abdominal.

CASO CLINICO
Al segundo da de vida presenta resolucin de su dificultad respiratoria con disminucin de requerimientos de oxgeno y tolerancia al
destete, por lo cual se inicia estmulo enteral con adecuada tolerancia. Los laboratorios iniciales hemograma normal y PCR negativa. Se
contino vigilancia clnica y aumento progresivo de la va oral. A los 5 das de vida presenta cuadro de episodio emtico de contenido
alimentario sin otra sintomatologa el cual se interpret como reflujo gastroesofgico. Al da siguiente presenta deterioro clnico dado
por taquicardia, distensin abdominal, persistencia de episodios emticos postprandiales, residuo gstrico del 70%, deposiciones con
sangre macroscpica e hipoglicemia.

PREGUNTA
Considerando los hallazgos clnicos cual es el estadio en el que se encuentra?

RESPUESTA
a.- Enterocolitis necrotizante I.
b.- Enterocolitis necrotizante II.
c.- Enterocolitis necrotizante III.
d.- Enterocolitis necrotizante IV.

PREGUNTA
Cual es la conducta a segur mas adecuada en este momento?

RESPUESTA
a.- Se suspende la va oral, BH, QS, cultivos, ampicilina y gentamicina.
b.- Ampicilina + amikacina, Bh, QS, cultivos, ayuno.
c.- Alimentacion parenteral, antibioticoterapia empirica, BH, QS, cultivos.
d.- Ampicilina, cefotaxima, alimentacin por sonda, BH y QS.

PREGUNTA
Tres das despus el paciente presenta hipotensin e hipoperfusin asociado a falla respiratoria por lo que requiere soporte inotrpico
y ventilatorio. Presenta acidosis metablica, trombocitopenia y alteracin en pruebas de coagulacin, cultivos con E.coli
multiresistente, considerando los cambios en de las imgenes de actuales cual es la conduca a seguir en el paciente?

RESPUESTA
a.- Transfusin de plasma y plaquetas.
b.- Crioprecipitados, albumina y cristaloides.
c.- Piperacilina tazobactam y nutricin parenteral.
d.- Piperacilina tazobactam, amikacina y nutricin parenteral.

CASO CLINICO
Recin nacido (RN) varn, con antecedentes de parto de pretrmino a las 32 semanas de gestacin, con peso de 1 300 g y Apgar 8 y 10
al nacer. Estando relativamente bien, a las 72 horas de vida comienza con rechazo alimentario y distensin abdominal. Un da ms
tarde se agregan vmitos biliosos, deposiciones con sangre e inestabilidad hemodinmica.

PREGUNTA
MANUAL DE TRABAJO DEL CURSO ENARM CMN SIGLO XXI
CURSO ENARM CMN SIGLO XXI TEL: 36246001 Pharmed Solutions Institute PGINA 361

Se realizo una radiografia para confirmar diagnostico, cual de los siguientes datos no esta presente en esta?

RESPUESTA
a.- Dilatacin de asas intestinales de colon e intestino delgado.
b.- Engrosamiento de paredes y extensa neumatosis intestinal.
c.- Mltiples imgenes areas pequeas, en aspecto de "burbujas" .
d.- Neumoperitoneo.

PREGUNTA
Considerando las imgenes observadas, cual es diagnostico final y complicaciones asociadas.

RESPUESTA
a.- Enterocolitis necrotizante complicada, con perforacin intestinal y neumoperitoneo secundario.
b.- Enterocolitis necrotizante complicada, sin perforacin intestinal ni neumoperitoneo secundario.
c.- Enterocolitis necrotizante simple, con preforacion intestinal.
d.- Enterocolitis complicada con peritoneo secundario.

PREGUNTA
Cual de las siguientes complicaciones agudas es mas frecuente?

RESPUESTA
a.- Necrosis intestinal con perforacin
b.- Peritonitis secundaria.
c.- Formacin de abscesos intraabdominales.
d.- Coagulacin intravascular diseminada.

CASO CLINICO
Se trata de un paciente varn de 5 meses de vida que ingres en nuestro hospital por presentar vmitos proyectivos en la mitad y al
final de las tomas, acompaados de una prdida de peso de 350g en el transcurso de una semana. Como antecedentes destacaba que
era un exprematuro moderado de 30 semanas de gestacin que al quinto da de vida desarroll una EN con neumatosis gstrica e
intestinal, por lo que precis una reseccin de 10cm del yeyuno y 25cm del leon. Tras la intervencin quirrgica present varios
episodios de retencin gstrica y vmitos, con resolucin espontnea y adecuada tolerancia posterior, por lo que se le dio el alta a su
domicilio.

PREGUNTA
Cual es la conducta a seguir en este caso, con la sintomatologia presente cual es estudio de primera eleccin para identificar la causa de
la conducin actual?

RESPUESTA
a.- Trnsito esofagogastroduodenal.
b.- Radiografia de abdomen.
c.- Ultrasonograma abdominal.
d.- Endoscopia digestiva.

PREGUNTA
Cual de los siguiente diagnostico diferenciales del estado actual, es el menos frecuente?

RESPUESTA
a.- Ingestion de causticos.
b.- Anomalias del ligamento falciforme.
c.- Enfermedad granulomatosa caustica.
d.- Gastroenteritis eosinofilica.













MANUAL DE TRABAJO DEL CURSO ENARM CMN SIGLO XXI
CURSO ENARM CMN SIGLO XXI TEL: 36246001 Pharmed Solutions Institute PGINA 362

MENINGITIS NEONATAL. CIENCIAS BASICAS: Es una enfermedad infecciosa con secuelas a corto y largo plazo, siendo su tratamiento
costoso, con aumento de los das de hospitalizacin y nmero de das recibiendo antibiticos. Meningitis bacteriana: sndrome clnico
compatible con meningitis, ms aislamiento de un organismo por cultivo del lquido cefalorraqudeo o aislamiento de un organismo por
cultivo de sangre y anormalidad del LCR consistente con infeccin bacteriana. SALUD PUBLICA: Frecuencia variable. 25-30% de los
casos de sepsis neonatal se complica con meningitis. Meningitis bacteriana tiene incidencia hasta de 1 x 1000 nacidos vivos.
CLASIFICACION: Meningitis temprana: Primera semana transmisin vertical (grmenes localizados en el canal vaginal materno)
grmenes involucrados, Streptococcus del grupo B, E. coli, Listeria monocytogenes, Enterococcus. Meningitis tarda: Nosocomial
adquirida en la comunidad grmenes involucrados, transmisin horizontal Gram negativos, Estafilococos. PATOGENIA: Etiologa: gram(-
): E. coli, Klebsiella, Serratia marcescens, Proteus, Enterobacter, Neisseria, Pseudomona, Salmonella, Citrobacter, etc. Gram(+): Stp,
Enterococos, Staphylococcus aureus, estafilococo coagulasa negativo. Otros: Lysteria monocytogenes. Ms frecuente: Enterobacterias
gram negativas (30-50%). Estreptococo beta-hemoltico grupo B (30-40%). Lysteria monocytogenes (10%). S. pneumonie, H. influenzae.
Es la diseminacin hematgena en la mayora de los casos, sin embargo ocasos se producen por propagacin por contigidad como
consecuencia de la contaminacin de defectos del tubo neural, fstulas congnitas o de heridas penetrantes causadas al obtener
muestras de sangre, partos spticos o domiciliarios. La cerebritis y los infartos spticos son frecuentes en la meningitis bacteriana. La
formacin de abscesos, la ventriculitis, la hidrocefalia y los derrames subdurales aparecen ms a menudo en recin nacidos que en
nios mayores. Los hallazgos patolgicos son similares entre los diferentes agentes bacterianos. El ms comn en autopsias es
exudado purulento en meninges y superficie ependimal de los ventrculos. Se evidencia tambin inflamacin perivascular y gran
respuesta inflamatoria. Hidrocefalia y encefalopata no infecciosa puede ser demostrada en aproximadamente 50% de los neonatos
con meningitis. La efusin subdural raramente ocurre en neonatos. Varios grados de flebitis y arteritis de vasos intracraneales pueden
presentarse. Ventriculitis puede ser demostrada en virtualmente todos los neonatos y en 75% al momento del diagnstico. Factores de
riesgo: Infeccin perinatal e
intrauterina (corioamnioitis,
endometritis, IVU, infeccin vaginal),
prematurez, ruptura prematura de
membranas, peso bajo al
nacimiento, va hematgena,
aspiracin, inhalacin.
Corioamnionitis clnica: presencia de
fiebre materna >38 C, con dos o ms de los siguientes hallazgos: taquicardia fetal (>160), sensibilidad uterina a la palpacin, descarga
vaginal ftida o leucocitosis materna. DIAGNOSTICO: Clnica: fiebre o hipotermia 60%, Irritabilidad 60% pobre alimentacin/vomito
48% crisis convulsivas 42% dificultad respiratoria 33% apnea 31% diarrea 20% fontanela abombada 25% rigidez de nuca 13%. La
sintomatologa de un paciente con meningitis es muy inespecfica y puede ser muy similar a la de un paciente con sepsis y las
manifestaciones como fontanela abombada, alteracin del estado de conciencia, convulsiones y coma, cuando se presentan se asocian
a pobre pronostico; por lo que es importante que dentro de la evaluacin del neonato infectado se incluya la puncin lumbar incluso en
la ausencia de signos neurolgicos evidentes. Laboratorios: 1. BH, recuento de leucos y porcentaje de PMN: < 20% >80% ndice
inmaduros/ maduros totales: 0.2 - Plaquetas <100.000 2. PCR: Es significativo cuando esta sea > 10mg/dl 3. Glicemia: si presenta
valor <40mg/dl >120mg /dl 4. Hemocultivos: Siendo positivos hasta un 50 %. Citoqumico, Gram y cultivo de LCR: Se tendrn los
siguientes criterios en el lquido cefalorraqudeo: 1) Glucosa menor a dos tercios la glucosa srica medida por glucometria simultnea.
2) Relacin glucosa LCR/ srica mayor de 0.5. 3) Protenas: Se considerar positivo para infeccin valor mayor de 90 mg/dl para recin
nacido a trmino y de ms de 150md/dl en prematuros. 4) Celularidad: Mayor de 10 clulas blancas. 5) Cultivo positivo.
Contraindicaciones para la puncin lumbar (LCR: 10-30 ml prematuro, RN termino 40 ml): compromiso cardiorespiratorio, CID, lesiones
en el sitio de la puncin, prematuros extremos (menores de 1000 gramos). Imagenologa: Hacerla para detecta complicaciones,
pacientes choqueados, falla respiratoria, dficit neurolgico, cultivo + despus de 48-72 Hs con apropiada terapia. TAC para descartar
abscesos. TRATAMIENTO: Sepsis temprana: ampicilina+amikacina. Sepsis tarda: cefotaxima+amikacina. Especfico para el agente
identificado. Otros: Meropenem, Ticarcilina, Vancomicina. Duracin terapia: 14-21 das. Esteroides. Inmunoglobulina. Aunque el
manejo final deber basarse en el resultado del antibiograma de las muestras de los hemocultivos, el tratamiento emprico iniciado con
penicilinas ms una cefalosporina de tercera generacin y/o un aminoglucsido tipo gentamicina preferencialmente, despus de
haberse tomado las muestras, es el que ha mostrado la mejor de las respuestas clnicas. El tratamiento endovenoso deber continuarse
con el antibitico apropiado para el germen aislado, hasta dos semanas despus de la negativizacin del hemocultivo.
Desafortunadamente, las secuelas severas a largo plazo son frecuentes (12% a 29% de los afectados), especialmente en las infecciones
asociadas a Streptococcus del grupo B y a bacilos gram negativos, an en los casos en los que el tratamiento ha sido oportuno y
adecuado

CASO CLINICO
Paciente masculino 10 dias de nacimiento, que se obtuvo en casa sin control prenatal, ingresa por diarrea, llanto, e irritabilidad, vomito
en proyectil, en casa, refiere la madre que solo fue en una ocacion, al ingreso se observo hipotnico, llanto agudo, mal estado
generalizado, con tono cervical no se observo rigidez, pero fontanela abombada, su peso fue de 2050 grs, laboratorios con 150,000 de
plaquetas, antecedentes de rinorrea hialina, se presento 35 grados, irritabilidad, durante la exploracin se observo pedaleo y
chupeteo, dificultad respiratoria.

PREGUNTA
Para establecer el diagnostico y conducir su teraputica para el caso?

RESPUESTA
a.- Hemocultivo.
b.- Punsion lumbar.
MANUAL DE TRABAJO DEL CURSO ENARM CMN SIGLO XXI
CURSO ENARM CMN SIGLO XXI TEL: 36246001 Pharmed Solutions Institute PGINA 363

c.- Tomografia.
d.- Biometria hemtica.

PREGUNTA
Se encuentra en espera de resultados, sin embargo el paciente se va deteriorando, cual es la agente etiolohico mas probable para dar
una teraputica dirigida?

a.- H influenza
b.- E Coli.
c.- Listeria monocitogenes
d.- Estreptoco beta hemolito.

PREGUNTA
Cual es la conducta farmologica ms adecuada considerando la etiologa?

RESPUESTA
a.- Ampicilina, amikacina.
b.- Ampicilina mas metronizadol.
c.- Ampicilina mas vancomicina.
d.- Ampicilina mas cefotaxima.

PREGUNTA
Se administro esteroides en el paciente, cual es la razn mas correcta para realizar esta conducta en el caso?

RESPUESTA
a.- Disminuir los efectos de los factores inflamatorios.
b.- Favorece la penetracin del frmaco.
c.- Disminuir la probabilidad de crisis convulsivas.
d.- Prevenir el edema cerebral.

PREGUNTA
Cual es la secuela mas frecuente que los pacientes con esta patologia desarrollan?

RESPUESTA
a.- Empiema cerebral
b.- Retazo en el neurodesarrollo
c.- Hidrocefalia.
d.- Paralisis cerebral.

CASO CLINICO
Recin nacido de 12 das de vida con fiebre de 38,5C de una hora de evolucin y rechazo de la alimentacin. Como antecedentes;
ruptura espontnea de membranas intraparto, parto instrumental con frceps a las 41 semanas de gestacin y peso al nacimiento de
3880 g. Apgar 8/9. Comenz su esquema de inmunizacin. Alimentacin con lactancia materna exclusiva. La exploracin fsica mostr
decaimiento y regular perfusin perifrica con constantes vitales normales. La analtica inicial mostr una determinacin de protena C
reactiva cuantitativa (PC-R) 133,4 mg/L, con un hemograma coagulado y un sedimento de orina normal. La puncin lumbar result
hemorrgica. Ingres con diagnstico de sndrome febril sin foco y se inici antibioterapia emprica con ampicilina y cefotaxima
intravenosas tras la toma de cultivos. En el hemograma repetido se observaron 5800 leucocitos/mm3, con frmula normal, 13,4 g/dl de
hemoglobina, 32 000 plaquetas/mm3. A las 18 h del ingreso empeor su estado general y present crisis convulsivas generalizadas que
requirieron tratamiento con fenobarbital y perfusin de midazolam. En la evolucin desarroll afectacin sistmica con hipotensin
arterial, acidosis metablica, hiponatremia, oliguria y plaquetopenia grave; requiri ventilacin mecnica convencional, transfusin de
hemoderivados, soporte inotrpico con dopamina y diurtico con furosemida. El tercer da del ingreso se inform el aislamiento en el
hemocultivo de Streptococcus pyogenes sensible a penicilina, por lo que se suspendi la ampicilina y se mantuvo el tratamiento con
cefotaxima intravenosa.

PREGUNTA
Cual es la causa ms probable de las crisis convulsivas.

RESPUESTA
a.- Hemorragia cerebral.
b.- Alteracion hidroelectrolitica.
c.- Neurosepsis.
d.- Encefalopatia hipoxico-isquemica.




MANUAL DE TRABAJO DEL CURSO ENARM CMN SIGLO XXI
CURSO ENARM CMN SIGLO XXI TEL: 36246001 Pharmed Solutions Institute PGINA 364

CASO CLINICO
Neonato de 11 das de vida, previamente sana, ingresada por dificultad respiratoria, mala coloracin, rechazo de tomas y decaimiento.
Constantes vitales; TA de 95/65 mmHg, FR de 160 lpm, FR de 90 rpm y saturacin de oxgeno del 94% con FiO2 del 30%. Se auscultan
ruidos crepitantes e hipoventilacin basal izquierda. La radiografa de trax evidencia infiltrado basal izquierdo.

PREGUNTA
Cual es la conducta a seguir.

RESPUESTA
a.- Amoxicilina mas gentamicina.
b.- Trimetoprim mas sulfametoxazol.
c.- Cetriaxona.
d.- Vancomicina.

CASO CLINICO
Se trata de paciente que se encuentra en la UCIN con diagnostico de sepsis neonatal, prematuro, bajo peso y masculino con 10 dias de
nacido, inicia con llanto continuo, irritable, sin rigidez de nuca, se observa hipotnico, rechazo al alimento, vomito en proyectil.
Antecedentes con aspiracin de meconio con sufrimiento e hipoxia, por parto prolongado y distocia, se observo disminucin de
complemento y linfocito,

PREGUNTA
Cual es el estndar de oro para el dianostico mas adecuado?

a.- Hemocultivo y urocultivo.
b.- Liquido cefaloraquideo.
c.- Biometria hemtica y EGO.
d.- Marcadores inflamatorios.

PREGUNTA
Cual es el agente etiolgico mas probable en el caso clnico actual?

a.- E. coli
b.- Klebsiella
c.- Pseudomona
d.- Salmonela

PREGUNTA
Cuales son las siguiente manifestaciones es la que presenta mayor frecuencia y mayor valor diagnostico?

a.- Fiebre e irritabiliad
b.- Vomito y crisis convulsivas.
c.- Alteraciones metabolicas
d.- Fontanela abombada.

PREGUNTA
Cuales son los valores que no es probable observar en este paciente.

RESPUESTA
a.- Protena de aumentada
b.- Glucosa 2/3 del central.
c.- Leucocitos elevados
d.- Acido lctico disminuido.

PREGUNTA
Cual es la complicacin anatomopatologia mas frecuente?

RESPUESTA
a.- Hemorragia ventricular.
b.- Ventriculomegalia
c.- Encefalomacia.
d.- Ventriculitis.

PREGUNTA
Cual es el tratamiento empirico de primera eleccin?

RESPUESTA
MANUAL DE TRABAJO DEL CURSO ENARM CMN SIGLO XXI
CURSO ENARM CMN SIGLO XXI TEL: 36246001 Pharmed Solutions Institute PGINA 365

a.- Ampicilina mas gentamicina
b.- Ampicilina mas amikacina
c.- Amikacina mas cefotaxima.
d.- Vancomicina, ampicilina mas cefotaxima.

PREGUNTA
Al paciente se le administro esteroides, cual es el objetivo farmacolgico para esta indicacin?

RESPUESTA
a.- Como medida antiedema.
b.- Para sinergismo con inmunoglobulina.
c.- Sinergismo con los antibiticos.
d.- Estabilizacin de membrana.

PREGUNTA
Cual de las siguientes complicaciones es menos frecuente esperar?

RESPUESTA
a.- Ventriculitis
b.- Hidrocefalia.
c.- Prdida auditiva y ceguera.
d.- Sindrome de Rett.

PREGUNTA
Condiderando los criterior para definir sepsis es un sndrome de respuesta inflamatoria sistmica, aunado a un agente infeccioso en el
RN?

RESPUESTA
a.- Horas de nacido, >130 FC.
b.- FR mas de 50
c.- Leucocitos mas de 16,500.
d.- Temperatura 37.8 y 10% de bandas 10 %

PREGUNTA
Cual es el agente causal mas probable en este caso especifico?
a.- Listeria monocitogenes.
b.- Stafilococco
c.- Enterococcus.
d.- Candida A.

HIPOGLUCEMIA. CIENCIAS BASICAS: Hipoglucemia neonatal: debajo de 45 mg/dl (2.5 mmol/L) para prematuros como de trmino y a
cualquier edad extrauterina. Hipoglucemia neonatal transitoria: se auto limita durante los primeros 7 das de vida extrauterina, es
consecuencia de reserva energtica limitada, excesivo consumo perifrico, agotamiento precoz de reservas energticas y a inmadurez
del sistema hipotlamo-hipofisiario, responsable de la secrecin de hormonas contra regulacin. Hipoglucemia neonatal persistente o
recurrente: <45 mg/dl en 3 ocasiones o persiste por ms de 72 hrs a pesar de tratamiento con soluciones glucosadas. Hipoglucemia
severa: aporte glucosada kg min > 10 ml/kg/min persisten cifras menores de 45 mg/dl. Factores de riesgo: Madre diabtica durante el
embarazo o ingestin de betabloqueadores o hipoglucemiantes orales. Prematurez. Peso bajo para la edad gestacional. Peso grande
para la edad gestacional. Deteccin tarda y manejo inadecuado de la hipoglucemia neonatal tiene un impacto sobre el desarrollo.
Causas de hipoglucemia transitoria: Estrs perinatal, septicemia, asfixia, hipotermia, policitemia, choque. SALUD PUBLICA: En RN a
trmino la incidencia est en un rango de 5-7% y pue de variar entre valores de 3,2 % a 14,7 % en recin nacidos pretrminos. Casi el
40% de los neonatos hijos de madre diabtica tienen hipoglucemia. PATOGENIA: La etiologa ms frecuente de hipoglucemia en el
recin nacido est de manera general asociada a incremento de la utilizacin de glucosa, a un aporte inadecuado de glucosa endgeno
o exgeno o a una combinacin de ambos. Una de las causas ms frecuentes de hipoglucemia en el RN ligada a hiperinsulinismo fetal es
el hijo de madre diabtica mal controlada. En estos nios la hipoglucemia frecuentemente se produce a las 4-6 hrs despus del
nacimiento, tienen una reduccin incrementada de insulina secundaria a sensibilidad aumentada de clulas beta del pncreas a la
glucosa; que persiste durante varios das despus del parto. Sntomas clsicos se deben a activacin del SNA con la libera catecolaminas
(hormona contrarreguladora de la hipoglucemia). Deprivacin de glucosa al cerebro con alteracin funcin neurolgica. La
administracin antenatal de clopropamida, benxotiazidas, beta-simpaticomimticos, propanolol o la administracin de glucosa a la
madre a alta concentracin y supresin brusca de su administracin puede inducir insulinismo fetal transitorio y por lo tanto
hipoglucemia neonatal. La eritroblastocisis fetal en RN con incompatibilidad Rh y el sndrome de Beckwith-Wiedemann, se caracteriza
por RN con macrosomia, onfalocele, macroglosia, visceromegalia e hipoglucemia. Se ha explicado la hipoglucemia por hipertrofia de las
clulas beta del pncreas y por lo tanto hiperinsulinismo. Otras causas: por deficiencias hormonales: dficit de hormonas del
crecimiento, deficiencia tiroidea. Causas hereditarias: enfermedad de orina de jarabe de arce, intolerancia a la fructosa, galactosemia.
DIAGNOSTICO: Apnea, hipotona, reflejo de succin inadecuada, irritabilidad, somnolencia, respiraciones irregulares, cianosis,
temblores, palidez, crisis convulsivas, letargia, cambios en el nivel de conciencia, inestabilidad de la temperatura, coma. Triada de
Whipple: Caractersticas clnicas, glucosa srica baja, resolucin de sntomas con correccin de glucemia. Estndar de oro:
MANUAL DE TRABAJO DEL CURSO ENARM CMN SIGLO XXI
CURSO ENARM CMN SIGLO XXI TEL: 36246001 Pharmed Solutions Institute PGINA 366

Determinacin enzimtica de los niveles de glucosa en laboratorio por el mtodo de la hexocinasa. TRATAMIENTO: Debe iniciarse una
alimentacin precoz en las dos primeras horas de vida (de preferencia en los primeros 30-60 minutos) y establecerse intervalos de
alimentacin cada 2-3 horas. Hipoglcemia asintomtica: 1. Si mediante tira reactiva los niveles de glucosa perifrica se encuentran
entre 27mg y 45 mg hay que tomar muestra sangunea por puncin venosa y corroborar niveles de glucosa, ofrecer inmediatamente sin
esperar el resultados alimentacin al seno materno y posteriormente cada hora. Si el RN no puede recibir seno materno de manera
adecuada, suplementar con sucedneo de leche materna cuyo volumen se ajustara con base al peso y se ofrecer cada 3 hrs. 2. Si
mediante tira reactiva los niveles de glucosa perifrica se encuentran por debajo de 27mg/dl, corroborar con una glucosa central.
Indicar infusin de glucosa IV para propiciar una infusin de glucosa kilo minuto de 6mg/kg/min. No suspender alimentacin enteral. Si
el tratamiento inicial fue ofrecer nicamente alimentacin enteral y con ello se normalizo la glucosa, se recomienda: Indicar infusin de
glucosa IV con lquidos a 80ml/kg/da para proporcionar una infusin de glucosa kilo minuto de 6 mg/kg/min. Se sugiere que en RN con
hipoglucemia asintomtica que requieran tratamiento con soluciones IV, el volumen de leche materna o sucedneo de leche materna
sea de 10-15 ml/kg/da. Hipoglucemia sintomtica: Tomar glucosa central para corrobora. Administrar solucin glucosada al 10% IV en
bolo: Si el paciente presenta crisis convulsivas administrar 4ml/kg. Si el paciente no presenta crisis convulsivas, administrar 2ml/kg.
Despus de administrar el bolo establecer infusin contina de glucosa IV de mantenimiento de 6mg/kg/min, una vez controlado
aumentar 2mg/kg/min hasta 12 mg/kg/min. Si las condiciones lo permiten continuar alimentacin oral con leche materna o sucedneo
de la leche materna en volumen de 10-15 ml/kg/min. Cada vez que se detecte hipoglucemia asintomtica se deber administrar bolo
de solucin glucosada al 10%. Si a pesar de tratamiento apropiadamente instaurado, el paciente persiste con hipoglucemia durante 3
determinaciones consecutivas, se recomienda referencia a tercer nivel en cualquiera de las siguientes situaciones: persistencia de
hipoglucemia a pesar de recibir aporte de glucosa kilo minuto de 10 mg/kg/min. Reaparicin de hipoglucemia al disminuir de manera
apropiada el aporte de glucosa kilo minuto. Se recomienda referencia a tercer nivel cuando exista hipoglucemia asociada a:
Antecedente familiar de muerte neonatal sbita o sndrome de Reye, crisis convulsivas o alteracin del estado de consciencia asociados
a hipoglucemia, alteraciones de la termorregulacin, defectos de la lnea media, exoftalmos, micropene. En hipoglucemia refractaria o
persistente: Glucagn (0.1mg/kg IM, mximo 1 mg) o hidrocortisona. Referir a tercer nivel: Persista hipoglucemia a pesar de recibir
aporte de glucosa kilo minuto de 10 mg. Reaparicin de hipoglucemia al disminuir el aporte de glucosa. Crisis epilpticas. Alteraciones
de la termorregulacin.

CASO CLINICO
RN de 36 SDG obtenido por cesarea por desprendimiento de placenta. Al nacimiento presenta Apgar 8/9, peso en -0,89 DE, longitud en
+1,75 DE, sin datos del permetro ceflico. Con 48 horas de vida ingresada por ictericia en su hospital de referencia presenta
hipoglucemias no cetsicas (glucemias entre 25-45mg/dl), insulinemia mxima registrada: 7U/ml coincidiendo con glucemia de
33mg/dl.

PREGUNTA
Cual es la conducta a seguir para identificar la causa de la hipoglucemia.

RESPUESTA
a.- Verificar Diabetes Mellitus en la madre.
b.- Realizar escaneo abdominal.
c.- Verificar funcionamiento heptico.
d.- Verificar funcionamiento suprarrenal.

CASO CLINICO
Se trata de reciencia nacido de 24 horas de edad, presenta en cunero las siguientes alteraciones: reflejo de succion inadecuado, crisis
convulsivas letargia, inestabilidad de la temperatura.

PREGUNTA
Cual es la conducta mediata mas adecuada para este caso?

RESPUESTA
a.- Administracion rectal de diacepam.
b.- Administracin oral de glucosa.
c.- Intubacion orotraqueal.
d.- Glucosa perifrica.

CASO CLINICO
Se trata de paciente masculino de 37 semanas de gestacion obtenido por cesarea, la madre presento cuadro de pre-eclampsia, luego de
24 horas de nacido presenta disminucin de la succion, letargia, falta de respuesta a estimulos con disminucin de tono muscular, asi
como alteraciones de la temperatura con tendencia a la hipotermia, los laboratorios reportaron 35 mg/dl de glucosa perifrica, resto de
datos de laboratorio y gabinete dentro de parmetro normales, sin embargo posterior a la administracin de glucosa mediante catter
perifrico presenta nuevos cuadro de hipoglucemia?

PREGUNTA
Cual de los siguiente antecedentes es menos frecuente para la patologia que presenta el paciente?

RESPUESTA
a.- Displasia de clulas beta.
MANUAL DE TRABAJO DEL CURSO ENARM CMN SIGLO XXI
CURSO ENARM CMN SIGLO XXI TEL: 36246001 Pharmed Solutions Institute PGINA 367

b.- Enfermedad de orina de arce.
c.- Galactosemia.
d.- Hipotiroidismo congnito.

ICTERICIA. CIENCIAS BASICAS: La ictericia es una de las condiciones ms comunes que requieren atencin mdica en los RN y se refiere
a la coloracin amarillenta de piel y mucosas causada por la fijacin de bilirrubina en el tejido graso subcutneo; generalmente, se
observa cuando los niveles sricos de bilirrubina son mayores o iguales a 5-7 mg/dl.. Las causas de ictericia neonatal son mltiples y
producen hiperbilirrubinemia directa, indirecta o combinada, de severidad variable. La hiperbilirrubinemia se refiere al aumento de los
niveles de bilirrubina total en sangre ( 2-5 mg/dl), como producto final de metabolismo del heme, componente esencial de la
hemoglobina. En la mayora de los casos, suele ser benigna, y autolimitada, pero por el efecto neurotxico de la bilirrubina, los
neonatos de riesgo deben ser vigilados para evitar hiperbilirrubinemia severa que produzca alteraciones neurolgicas como
encefalopata aguda y kernicterus, los cuales son causas prevenibles de parlisis cerebral. Los neonatos amamantados son ms
propensos a desarrollar, ictrica fisiolgica (aparece despus de las 24 hrs y desaparece antes de los 10 das) en la primera semana de
vida y cerca del 10% son ictricos al mes de vida. Ictericia patolgica; cuando inicia en las primeras 24 hrs, se acompae de otros
sntomas, la bilirrubina aumente >5mg/dl diarios, sobrepase los 15mg% o 10mg% en enonatos a trmino y pretermino, la fraccin
directa sea superior a 2mg/dl o dure ms de una semana en el RN a trmino (excepto si recibe lactancia materna). SALUD PUBLICA: La
ictericia fisiolgica es una situacin frecuente, aparece despus del segundo da de vida como expresin de una condicin fisiolgica
hasta en un 80% de los prematuros y en un 60% de los a trmino. Ictericia patolgica en 6% de RN. PATOGENIA: Factores de riesgo:
Alimentacin a pecho. Mayor prdida de peso (ms de 5%). Sexo masculino. Edad gestacional < 35 semanas. Diabetes materna.
Hematomas. Raza Oriental. La principal causa de la apricion de la ictrica fisologica en el RN es la inmadurez del sistema enzimtico del
hgado, a esto se le suma una menor vida media del glbulo rojo, la poliglobulia (el RN produce el doble de bilirrubina que el adulto), la
extravasacin sangunea frecuente, el RN reabsorbe gran parte de la bilirrubina a travs de la circulacin enterohepatica y la ictrica
por lactancia. Ictericia patolgica, las causas ms frecuentes son 1. Enfermedad hemoltica (incompatibilidad sangunea materno-fetal,
esferocitosis familiar, dficit de enzima G-6-PD), las cuales disminuyen la vida media de los eritrocitos. 2. Hematomas y hemorragias
(cefalohematomas), cuya reabsorcin aumenta la oferta de bilirrubina. 3. Incremento en la
reabsorcin intestinal como sucede en el retraso en la alimentacin gstrica en RN
enfermos o la presencia de obstruccin intestinal. 4. Policitemia, por mayor volumen y
destruccin globular. 5. Defectos enzimticos congnitos como Sx. De Crigler-Najjar. 6.
Ictericia acolurica familiar transitoria (Sx. De Lucey-Driscoll), se presenta en RN cuyas
madres son portadores de factor inhibitorio en el suero que impide la conjugacin.
ICTERICIA POR INCOMPATIBILIDAD DE FACTOR Rh: Es la causa ms frecuente de ictrica
patolgica y el 97% de los casos se debe a isosensibilizacion para el antgeno Rh D. Un alto
ttulo materno de anticuerpos anti-Di 64 de se asocia a un riesgo elevado de
hiperbilirrubinemia severa para los recin nacidos. La administracin de profilctica de
inmunoglobulina Anti-D. DIAGNOSTICO: El tinte ictrico no solo est en la piel y
conjuntivas, sino que tambin puede apreciarse en el LCR, lgrimas, saliva y especialmente
en los casos patolgicos. Es conveniente valorar la presencia de coluria y acolia, ya que son
datos de gran valor diagnstico. La presencia de hepatomegalia precoz es sugestiva de
infeccin prenatal o de enfermedad hemoltica por incompatibilidad Rh; cuando esta es
dura y de aparicin ms tarda, har pensar en la posibilidad de afectacin heptica primitiva (hepatitis, atresia), si bien en estos casos
el resto de signos clnicos son distintos. La esplenomegalia har sospechar que acta como un foco hematopoytico extramedular o
bien que el RN padece una infeccin prenatal con
manifestaciones clnicas. Por la frecuencia con la que
se presenta al tercer da de vida una
hiperbilirribinemia secundaria a la reabsorcin de
hematomas, se debern buscar colecciones de
sangre extravasada. La ictericia inicia en cara u
tienen una progresin cefalocaudal, til para valorar
el grado de ictericia. Otro sntoma frecuentemente
asociado a hemolisis es la hipoglicemia, como
resultado de la hiperpalsia pancretica. La presencia
de petequias y purpura sugieren la posibilidad de
infeccin connatal. Laboratorio: Dosaje de
bilirruibinemia total y directa, si hay elevacin de
bilirrubina indirecta, sugiere hemlisis. Si hay
elevacin de la bilirrubina directa, sugiere
enfermedad hepatobiliar. Reaccin de Coombs
directa e indirecta. Hematocrito y hemoglobina (para
valorar la presencia de anemia asociada). Recuento
de reticulociotos. TRATAMIENTO: Las alternativas
son: 1. Fototerapia; la de eleccin y ms difundida,
su administracin redujo en gran medida el uso de
exanguineotransfusion, acta por fotooxidacion
(destruccin fsica de la bilirrubina en productos ms pequeos y polares para ser excretados) y fotoisomerizacin, el RN debe estar
desnudo, cubrir los ojos, y control trmico, actualmente existe fototerapia en fibra ptica 2. Exanguinotransfusion, se basa en la
remocin mecnica de sangre del RN por sangre de un donador, ha sido reemplazada por la fototerapia, se reserva en especial para
MANUAL DE TRABAJO DEL CURSO ENARM CMN SIGLO XXI
CURSO ENARM CMN SIGLO XXI TEL: 36246001 Pharmed Solutions Institute PGINA 368

enfermedades hemolticas severas, cuando la fototerapia no ha resultado eficaz. Terapia farmacolgica; mesoporfirina, inhibe el
catabolismo del hemo y por lo tanto la produccin de bilirrubina. Fenobarbital, es un inductor enzimtico que estimula las etapas de
captacin, conjugacin y excrecin de bilirrubina. Por estar asociado a un potencial desarrollo de adiccin, sedacin excesiva y efectos
metablicos adversos, han limitados su aplicacin en el RN. Administracin Oral de sustancias No absorbibles: estos al captar bilirrubina
en la luz intestinal, reducen la absorcin enteral de sta y, as se puede disminuir los niveles de bilirrubina srica. TOXICIDAD DE LA
BILIRRUBINA: Hay dos fases en la neurotoxicidad de la bilirrubina una temprana y aguda que es reversible si el pigmento es removido y
una lenta y tarda cuyos efectos son irreversibles. Los signos clnicos de toxicidad aguda son apata, somnolencia o insomnio, junto con
la alteracin de los potenciales evocados auditivos, pero que luego revierten, una vez que los valores de bilirrubina descienden. La
encefalopata por bilirrubinas es un sndrome neurolgico que resulta del deposito de bilirrubina no conjugada en el SNC,
especialmente en los ganglios basales y nucleos del tallo cerebral. La causa de esta encefalopata es de origen multifactorial relacionada
con niveles elevados de bilirrubina no conjugada libre, cantidad de bilirrubina que se une a la albmina, alteracin de la barrera
hematoenceflica (BHE) por otras enfermedades y susceptibilidad neuronal.

CASO CLINICO
RN presenta ictericia, que se hizo evidente en el segundo da de la vida. Naci de padres no relacionados a los 36 semanas de gestacin
con un peso al nacer 2800 g, despus de embarazo y el parto expontaneo y normal aparentemente. Al examen fsico, paciente estaba
activo y cmodo. Sus signos vitales eran estables. En recuento de admisin de sangre completa fue en normal lmites.

PREGUNTA
Cual es la conducta a seguir.

RESPUESTA
a.- Alta y medidas en casa.
b.- Se ingresa para fototerapia.
c.- Verifica niveles de bilisrrubinas.
d.- Verifica grupo y factor en ambos padre.

KERNICTERUS. CIENCIAS BASICAS: Es la coloracin amarilla de los ganglios basales producida por impregnacin de bilirrubina, descrita
en autopsias de RN fallecidos con severa ictericia. Constituye la complicacin ms grave de la ictericia neonatal. El kernicterus es la
secuela ms importante de la encefalopata bilirrubnica. Es una enfermedad devastadora, una entidad previsible cuando la
hiperbilirrubinemia es tratada agresivamente. SALUD PUBLICA: Su incidencia aumento con las nuevas polticas de alta prematura, esto
causa mayor riesgo de complicaciones debidas a ictericia temprana no detectada. PATOGENIA: Hay varias situaciones que alteran la
barrear hematoencefalica y facilitan la entrada de bilirrubina al SNC, aumentando notablemente el kernicterus, como son; bajo peso al
nacer, hipoglucemia, asfixia neonatal, acidosis metablica, hemolisis, hipotermia-frio, hipoalbuminemia, drogas que compiten por la
unin a albumina, diestres respiratorio. La bilirrubina no conjugada penetra en el cerebro y acta como una neurotoxina, a menudo
sta se asocia con condiciones que dificultan la funcin de la barrera hemato-enceflica (ejemplo, sepsis), inhibiendo varios procesos
bioqumicos muy importantes, como la fosforilacin oxidativa de las mitocondrias y la sntesis proteica. Es necesario anotar que se
desconocen los niveles toxicos de bilirrubina para SNC. Revisiones recientes han sugerido que 25mg% y aun unos puntos ms seran los
txicos. DIAGNOSTICO: Puede Ser asintomtico en prematuros pequeos. Se caracteriza por atetosis, sordera neorosensorial parcial o
completa, limitacin de la mirada vertical, dficit intelectual, displasia dental. En la forma clsica se reconocen 3 estadios: Primera fase;
caracterizada por inicio con vmitos, letargia, hipotona, rechazo al alimento, succin dbil y llanto agudo. Segunda fase; se caracteriza
por irritabilidad, hipertona y opisttonos. Tercera fase; observada en sobrevivientes de las 2 anteriores y caracterizada por la triada de
hipertona, atetosis y otros movimientos extrapiramidales y retardo psicomotor. Pueden quedar secuelas alejadas siendo las ms
frecuentes la sordera, los trastornos motores y los problemas de conducta. Las regiones del cerebro ms comnmente afectadas son
los ganglios basales, particularmente los ncleos subtalmicos y el globo plido, el hipocampo, el cuerpo geniculado, varios ncleos
cerebrales, incluyendo el colculo inferior, vestibular, oculomotor, coclear y olivar inferior, y el cerebelo, especialmente el ncleo
dentado y el vrmix. La necrosis posnatal es el hallazgo histopatolgico dominante despus de los 7-10 das de vida posnatal. El
diagnstico puede ser confirmado por resonancia nuclear magntica (RNM) cerebral, cuya imagen caracterstica es de tipo bilateral,
con alta seal de intensidad en el globo plido, vista en los cortes de T1 y T2. Estas imgenes tambin se pueden observar en el
hipocampo y el tlamo.

CASO CLINICO
RN de 35 SDG, vigoroso, peso 2,400 g, permetro ceflico 34 cm, talla 47 cm, sin patologa perinatal, alimentado a pecho direcho
exclusivo, alta a las 48 horas, regreingresa a los 6 dias de vida por ictericia generalizada, con peso al ingreso de 2,130 grs. Succion
vigorosa pero breve e ineficaz, escasas diuresis y deposiciones. Bilisrrubinas totales 32,52 mg/dl, bilisrrubina indirecta 32 mg/dl, Bh
normal, glicemia normal. La EF se observa leve temblor distal

PREGUNTA
Cual es la conducta mas adecuada a seguir en este caso?

RESPUESTA
a.- Colchon de fibra ptica de luz contina.
b.- Exanguinotransfusion.
c.- Fototerapia.
d.- Luz halogenada.

MANUAL DE TRABAJO DEL CURSO ENARM CMN SIGLO XXI
CURSO ENARM CMN SIGLO XXI TEL: 36246001 Pharmed Solutions Institute PGINA 369


ATRESIA ESOFAGICA (AE) Y FISTULA TRAQUEOESOFAGICA (FTE). CINECIAS BASICAS: La atresia de esfago es una anomala congnita
en la cual la porcin media del esfago est ausente (atresia); en un porcentaje importante se acompaa de una comunicacin anormal
entre la traquea y el segmento distal del esfago llamada fistula traqueoesofgica. La atresia de esfago es una malformacin
incompleta de la luz esofgica. La variante ms frecuente es la tipo C de la clasificacin de Gross o la III de Voght en 87% de los casos.
Factores de riesgo: y agravantes del pronstico: malformaciones congnitas, neumonas, bajo peso al nacer. Se encuentran anomalas
cromosmicas entre 610% de los casos; las ms frecuentes son las trisomas 18 y 21. SALUD PUBLICA: Es la malformacin esofgica
ms frecuente. La atresia esofgica con o sin fstula traqueoesofgica es una alteracin que aparece en 1:3,000 a 1:4,500 recin
nacidos. El 95% de la AE tiene fstula traqueoesofgica asociada. El 50% de los casos se asocia con otras malformaciones congnitas y
existe riesgo de recurrencia de 2 a 3%, y mayor riesgo relativo si se tienen hijos o familiares afectados. En virtud que en Mxico nacen
aproximadamente 2 millones de nios por ao, se estima que cada ao hay entre 500 y 600 casos nuevos de nios con atresia de
esfago. PATOGENIA: Malformaciones congnitas asociadas: Cardiacas (35%): comunicacin interventricular, conducto arterioso
persistente, tetraloga de fallot, coartacin de aorta. Gastrointestinales (20%): ano imperforado, atresia duodenal, malformaciones
intestinales. Genitourinarias (20%): reflujo ureteral, agenesia renal. Msculo esqueltico (13%): vertebrales, costales, defecto de las
extremidades y del sistema nervioso central (10%). La asociacin de algunas de estas anomalas se denomina asociacin VACTERL
(Vertebrales, Anoreactales, Cardiacas, Traqueales, Esofagicas, Radiales, renales y de extremidades (L; limbs en inlges) y se presenta con
una frecuencia de 10%. El origen de la AE es poco claro an pero se atribuye a una alteracin en la migracin de los pliegues laterales o
a una detencin del crecimiento en el momento de la evaginacin. En la mayor parte de los casos el esfago posterior no se separa
totalmente de la trquea, lo que da lugar a distintas variedades de fstula traqueoesofgica o a hendiduras Esta alteracin se produce
entre la tercera y sexta semana de gestacin. Fstula traqueoesofgica (TEF) Una fstula es una conexin entre el esfago y la trquea.
Esta conexin permite que la comida (desde el esfago) entre en los pulmones (aspiracin). La comida en los pulmones puede causar
neumona (referida a neumona por aspiracin), lo cual puede ser muy serio. Si hay una fstula traqueoesofgica con atresia esofgica,
toda la comida ingerida terminar en los pulmones, como el esfago no est conectado con el estmago, porque finaliza cerrado. El
tipo ms difcil para diagnosticar es la fstula traqueoesofgica tipo H, donde el esfago no termina cerrado, pero hay una fstula o
conexin entre la trquea y el esfago. En la fstula traqueoesofgica tipo H, el aire que entra al estmago (desde la trquea) y la
comida puede entrar a los pulmones. El aire en el estmago puede causar hinchazn en el abdomen del beb y puede hacer que el
beb est molesto. DIAGNOSTICO: El diagnstico prenatal se establece con ultrasonografa y puede sospecharse durante el embarazo
por polihidramnios, incapacidad de identificar el esfago fetal o ausencia de burbuja gstrica. Diagnostico en sala de parto, la
complicacin al paso de la sonda para verificar la permeabilidad del esfago; sialorrea, distensin abdominal, dificultad respiratoria, tos
o cianosis son algunas manifestaciones en el recin nacido. Al darle de comer: vmitos/atragantamiento (no pasa el contenido a
estmago), crisis de sofocacin con tos tras las tomas lo que sugiere fstula superior. Puede apreciarse
abdomen escavado a la inspeccin o abdomen distendido si fstula inferior. De persistir la alimentacin
sin diagnosticarse: vmitos o neumonas de aspiracin. Ante la sospecha debe realizarse sonda
nasogstrica: no pasa hasta estmago; se enrolla en bolsn. No necesario contraste por que las sondas
son radiopacas y hacer estudio estudio radiogrfico para confirmar el diagnstico; el estudio muestra
el fondo de saco, ciego, del esfago atrsico o, bien, puede realizarse un examen endoscpico para
confirmar la fstula. Se han evaluado la resonancia magntica, TAC, estudios de medicina nuclear,
angiografa y ultrasonografa sin resultados concluyentes. CLASIFICACION: Clasificacin de Vogt
modificada por Ladd de la atresia de esfago: AE TIPO I (5-8%): Ambos cabos esofgicos ciegos sin
fstula traqueoesofgica: Es de fcil diagnostico en el embarazo por que cursa con polihidramios y
ausencia de imagen gstrica. Abdomen excavado por falta de pasaje de aire al intestino. Ambos cabos
se encuentran muy separados entre s. AE TIPO II (0.5-1%): Fstula traqueoesofgica superior y cabo
inferior ciego. AE TIPO III (80-85%): Fstula traqueoesofgica inferior y cabo esofgico superior ciego:
En stos pacientes el reflujo gastroesofgico puede ser lesivo para los pulmones, son propensos a sufrir
Neumonas Qumicas. AE TIPO IV (0.5-1%): Fstula traqueoesofgica en ambos cabos del esfago. AE
TIPO V (3-5%): Fstula en H o N. Es una fstula traqueoesofgica sin atresia de esfago: El diagnostico se
realiza en la infancia ya que aparecen los sntomas en ese periodo. AE TIPO VI (0.-1%): Estenosis
esofgica aislada. Tipos de atresia esofgica, clasificacin de Gross: A. Atresia esofgica sin fstula: 3-5%. B. Atresia esofgica con fstula
proximal: 2%. C. Atresia esofgica con fstula distal: 80-90%. D. Atresia esofgica con fstula proximal y distal: 3-5%. E. Fstula
traqueoesofgica sin atresia: 6%. F. Estenosis esofgica. TRATAMIENTO: El adecuado ambiente trmico, el suministro de lquidos,
glucosa y electrlitos, la posicin adecuada del paciente (semisentada, con la cabeza elevada en 30 a 40), sonda doble lumen. Drenaje
del cabo proximal con sonda de doble lumen (sonda de Repogle) para aspiracin constante), la aspiracin cuidadosa y frecuente de la
saliva y las secreciones acumuladas en el cabo ciego, si existe neumona, iniciar doble esquema de antibitico (ampicilina mas
aminoglucocido). Intervencin quirrgica, interrumpir la comunicacin entre la trquea y el esfago. Reestablecer la continuidad del
Esfago. Preservar el esfago existente. Gastrostoma y esofagectoma: Plastia esofgica, plastia esofgica y cierre fstula, sustitucin
esofgica, cierre fstula. Atresia tipo III: dentro de las primeras 24 hrs de vida se realizara cierre de FTE inferior y, si la distancia entre los
cabos es menor de 3 cm, anastomosis termino-terminal (T-T) entre los cabos esofgicos a travs de una toracotoma con abordaje
extrapleural. COMPLICACIONES: Reflujo gastroesofgico. Refistulacin traqueo esofgica. Estreches anastomtica. Traqueomalasia.
Dismotilidad esofgica.

CASO CLINICO
Recin nacida, gemela I, de madre sana de 35 aos de edad, producto del segundo embarazo, de 35 semanas de duracin; el embarazo
anterior termin con aborto espontneo en el segundo trimestre, seis aos antes. El embarazo actual tuvo adecuado control prenatal,
recibi cido flico, calcio, hierro y polivitaminas; se complic por amenaza de aborto en el primer trimestre, pero se recomend
reposo. En la semana 31 del embarazo se le realiz ultrasonografa obsttrica que report. Embarazo gemelar con fetos vivos, placenta
nica, corporal, posterior, grado II y edad gestacional de 31 semanas, 3 dias antes del parto la madre tuvo infeccin de las vas urinarias,
MANUAL DE TRABAJO DEL CURSO ENARM CMN SIGLO XXI
CURSO ENARM CMN SIGLO XXI TEL: 36246001 Pharmed Solutions Institute PGINA 370

por lo que se hospitalizo para tratamiento y recibir esquema de tres dosis de inductores de la maduracin pulmonar, tuvo rotura de
membranas 4 horas antes del parto, se realizo bloqueo peridural y se realizo cesarea, al nacimiento el Apgar fue de 6/8 con peso de
2,000 g, dificultndose la aspiracin de secresiones, continuando con tos, sialorrea, y cianosis leve.

PREGUNTA
Cual es la conducta a seguir inmediata mas adecuada?

RESPUESTA
a.- Colocar en silla porta-bebe a 45 grados.
b.- Colocar sonda de doble lumen.
c.- Prepara para ciruga.
d.- Aplicacin de oxigeno a 3 lt/x.

HERNIA HIATAL CONGENITA. CIENCIAS BASICAS: Sede be al cierre incompleto de la membrana pleuroperitoneal (abertura o
dehiscencia congnita del diafragma) o al retorno prematuro del intestino a la cavidad abdominal con herniacin de las vsceras
abdominales a la cavidad torcica. SALUD PUBLICA: La hernia diafragmtica congnita ocurre entre 1 en 2,000 a 1 en 5,000 recin
nacidos vivos. La mortalidad vara entre un 40-70% dependiendo el grado de hipoplasia pulmonar y el desarrollo de hipertensin
pulmonar. La relacin hombre: mujer es de 1:1,8. El 20-53% presentan malformaciones asociadas, sobre todo defectos cardiacos (9-
23%), defectos del tubo neural (28%), trisomas y ciertos sndromes bien definidos. PATOGENIA: Existen dos teoras acerca de la
embriognesis de la hernia diafragmtica: 1. Crecimiento pulmonar anormal que produce desarrollo diafragmtico anormal. 2. Defecto
diafragmtico con hipoplasia pulmonar secundaria. La etiologa de la hernia diafragmtica no es clara, han sido reportados casos
familiares sugiriendo predisposicin gentica. En 20% de los casos se ha presentado polihidroamnios y es de mal pronstico. Talidomida
y quinidina se han reportado como causantes de hernia diafragmtica en humanos. Las anomalas en la formacin anatmica normal
del diafragma explican la aparicin de defectos congnitos. Si la detencin del desarrollo diafragmtico se produce precozmente en
tero, el RN presenta una amplia comunicacin entre trax y abdomen. Si la detencin de la formacin se produce despus de
formado el tabique membranoso pero antes de formarse la caja muscular, el nio presenta un saco herniario que contiene los rganos
desplazados hacia arriba. Esta patologa se asocia prcticamente siempre con efecto de masas, que se manifiesta en el feto por
desplazamiento del mediastino, compresin pulmonar y reduccin del tejido pulmonar antes de la semana 16, momento en el que
desarrollo bronquial es completo determina reduccin del nmero de bronquios y alveolos del pulmn en desarrollo que conlleva a la
hipoplasia pulmonar. El pulmn hipoplsico en hernia diafragmtica presenta unareduccin cuantitativa y cualitativa de surfactante y
un compromiso de la distensibilidad pulmonar. CLASIFICACION: De acuerdo a su localizacin anatmica: 1. Hernia posteroexterna de
Bochdalek es el tipo ms comn representa 85-90%, siendo la mayor ubicacin izquierda 80%, derecha 15%, y mixta 5%. 3. Hernia de
hiato esofgico. 4. Hernia retroesternal de Morgagni. DIAGNOSTICO: Algunos neonatos con este defecto no tienen manifestaciones
clnicas perceptibles y se desarrollan sin problemas excepto cuando por alguna razn por lo general una infeccin respiratoria o
problemas gastrointestinales se descubre en un estudio radiolgico que tienen hernia diafragmtica. El diagnstico prenatal se realiza
por ecografa, se basa en la visualizacin de rganos abdominales en el trax y el signo ecogrfico distintivo es una masa ocupada por
lquido inmediatamente por detrs de la aurcula y el ventrculo izquierdos, en la parte inferior del trax visualizando en una vista
transversal. Otros signos ecogrficos que hacen sospechar el diagnstico son la ausencia del estmago en el abdomen, desplazamiento
del mediastino, permetro abdominal fetal pequeo y polihidramnios. TRATAMIENTO: El tratamiento incluye terapias como el uso de
corticosteroides, la ventilacin de alta frecuencia, oxigenacin con membrana extracorprea (ECMO), terapia con surfactante, xido
ntrico y ciruga fetal. HERNIA DE BOCHDALEK: Puede ser de presentacin espordica o familiar. El defecto ocurre cuando la membrana
pleuroperitoneal no se fusiona con las otras porciones. La fusin de la membrana pleuroperitoneal ocurre primero del lado derecho y
luego del izquierdo. Adems la proteccin del hgado en el lado derecho, hace que sea ms frecuente la hernia de Bochdalek del lado
izquierdo. La hipoplasia pulmonar se debe a que los pulmones se encuentran en la fase glandular del desarrollo y el intestino torcico
ocupando espacio, impide el normal desarrollo. Por lo tanto existe una disminucin del lecho vascular y de la segmentacin bronquial.
Es frecuente la mal rotacin de los intestinos debido a que los intestinos se desplazan al trax antes de la fijacin del ciego al cuadrante
inferior derecho del abdomen. HERNIA DE MORGAGNI: La hernia de Morgagni es una anomala congnita del diafragma, causada por
un defecto anteromedial del diafragma, entre sus inserciones costal y esternal, que fue descrito por primera vez por Morgagni, Los
pacientes con hernia de Morgagni se diagnostican a cualquier edad en forma accidental al efectuar un examen radiolgico de trax por
otra patologa, tal como infecciones respiratorias o molestias gastrointestinales, un considerable nmero de pacientes se diagnostica en
la edad peditrica por presentar dicultad respiratoria. El diagnstico de hernia de Morgagni se lo efecta en la edad adulta, la mayora
de las veces en forma incidental, y en los nios por presentar dicultad respiratoria. Su patogenia es desconocida, aunque actualmente
se sugiere una etiologa multifactorial en la que se implican factores hereditarios en relacin con otros sndromes malformativos como
los sndromes de Down, Turner, Prader-Willi y Nooan. El tratamiento es eminentemente quirrgico en todos los casos y consiste en la
plastia del defecto diafragmtico; hoy en da varios centros han reportado exitosas reparaciones por va laparoscpica.

CASO CLINICO
Lactante de 5 meses de edad, femenino, eutrfico, con historia de cuadros de aparente dolor abdominal a repeticin. Consult por
presentar sntomas respiratorios, fiebre e irritabilidad, se solicit radiografa de trax en proyecciones anteroposterior y lateral que
muestra reduccin del parnquima pulmonar en hemitrax derecho y desplazamiento del mediastino y del corazn hacia la izquierda.

PREGUNTA
Cual es la conducta diagnostica siguiente mas adecuada?

RESPUESTA
a.- Serie gastroesofgica.
MANUAL DE TRABAJO DEL CURSO ENARM CMN SIGLO XXI
CURSO ENARM CMN SIGLO XXI TEL: 36246001 Pharmed Solutions Institute PGINA 371

b.- Serie gstrica completa.
c.- Tomografia de abdomen.
d.- Resonancia magntica.

CASO CLINICO
Recin nacido varn, que nace por Cesrea, SGB +, Transaminasas Maternas elevadas. Apgar 6/7, Peso 3670 gramos, talla 51
centmetros. Presenta, en el postparto inmediato, dificultad respiratoria con cianosis, aleteo nasal, quejido, tiraje intercostal,
crepitantes bibasales, disminucin de entrada de aire y del murmullo vesicular en hemitorax Izquierdo.

PREGUNTA
Cual es la conducta diagnostica mas adecuada?

RESPUESTA
a.- Serie gastroesofgica.
b.- Radiografia de torax.
c.- Tomografia de abdomen.
d.- Endoscopia superior.

PREGUNTA
Cual de las siguientes manifestaciones es menos frecuente en esta patologa?

RESPUESTA
a.- Insuficiencia Respiratoria Severa desde el nacimiento.
b.- Disminucin o ausencia de murmullo vesicular.
c.- Ruidos hidroareos (RHA) en trax.
d.- Desplazamiento de ruidos cardiacos al lado contralateral.

PREGUNTA
Cual es la trisoma que no se ha relacionado en esta patologa?

RESPUESTA
a.- X.
b.- 18.
c.- 21.
d.- 13.

ESTENOSIS HIPERTROFICA DEL PILORO. CIENCIAS BASICAS: La estenosis hipertrfica del ploro anteriormente se conoca como
hipertrofiacongnita del ploro; tambin se le ha denominado estenosis pilrica hipertrfica infantil, para diferenciarla de la estenosis
adquirida que se observa en el adulto. La estenosis hipertrfica del ploro es la causa ms frecuente de ciruga en los lactantes menores
de 6 meses, superada solamente por las hernioplastias. SALUD PUBLICA: Se estima que hay entre uno y cinco pacientes con esta
patologa por cada 1,000 recin nacidos vivos. 7% asociado a malformaciones: malrotacin intestinal, uropata obstructiva, atresia
esofgica, hernia hiatal. Se presenta entre las 2 y 8 semanas de edad, con un pico entre las 3 y las 5 semanas. Es 4 a 5 veces ms comn
entre varones que en mujeres, con una mayor incidencia en primognitos. PATOGENIA: La causa exacta de la estenosis del ploro no se
ha determinado todava, pero se han desarrollado algunas teoras relacionadas con un desequilibrio neurohormonal, o de mediadores
neuroendocrinos, en el control del tono del esfnter pilrico, en la produccin de gastrina y la motilidad del estmago. Una propuesta
es una descoordinacin entre el peristaltismo gstrico y la relajacin pilrica, lo que lleva a una contraccin gstrica contra un ploro
cerrado, que causara hipertrofia en el msculo pilrico. Otras teoras proponen una elevacin en las concentraciones de gastrina
(hipergastrinemia), debido a un aumento hereditario en el nmero de clulas parietales de la mucosa gstrica que llevan a un ciclo de
aumento en la produccin de cido gstrico, contracciones cclicas peridicas en el ploro y vaciamiento gstrico lento, esto lleva a
hipertrofia e hiperplasia de las fibras musculares del esfnter pilrico. Se ha propuesto herencia autosmica dominante cromosoma 16
q24 y herencia multifactorial. Administracin de eritromicina/ azitromicina los primeros das de vida, se ha encontrado que la
administracin de macrlidos a las madres que dan alimentacin al pecho tambin puede ser un factor para que los lactantes
presenten estenosis pilrica. El hbito de fumar materno se ha reportado como posible factor de riesgo para estenosis pilrica. Otras
investigaciones sealan que al estudiar muestras de las capas musculares depacientes afectados de estenosis pilrica, en comparacin
con controles, se hanencontrado cantidades disminuidas de las terminales nerviosas y de los neurofilamentos; disminucin en los
marcadores para clulas de soporte; disminucin en las clulas intersticiales de Cajal; disminucin en la actividad de la sintetasa de
xido ntrico, el cual acta como relajante del msculo liso en diversos tejidos; disminucin en la produccin del ARN mensajero para la
sintetasa de xido ntrico. DIAGNOSTICO: El sntoma ms caracterstico son los vmitos posprandiales, no biliosos, progresivos hasta
ser incluso en proyectil, que eventualmente impiden la alimentacin adecuada del lactante. Esto lleva a prdida de peso por
disminucin del aporte de caloras y por deshidratacin. Por lo general el recin nacido ha tenido previamente un perodo libre de
vmitos. De los antecedentes es importante indagar por el uso de eritromicina o macrlidos en las dos primeras semanas de vida,
adems de los antecedentes familiares. Conforme continan los sntomas el paciente adelgaza (pierde peso) y se torna
hambriento,toma con avidez la leche, pero rpidamente la vomita; se puede observar el vmito explosivo o en proyectil. Algunos
pacientes presentan ictericia (10%) de grado variable. Tambin se pueden encontrar signos clnicos de deshidratacin como mucosas
con saliva filante o secas, depresin de la fontanela, llenado capilar lento y signo del pliegue presente (se suma a la prdida de peso). El
abdomen no est distendido; cuando la emaciacin ha progresado, es posible observar las ondas peristlticas en el epigastrio. El signo
MANUAL DE TRABAJO DEL CURSO ENARM CMN SIGLO XXI
CURSO ENARM CMN SIGLO XXI TEL: 36246001 Pharmed Solutions Institute PGINA 372

patognomnico es la masa palpable en el cuadrante superior derecho del abdomen, en forma de aceituna u oliva, que corresponde al
ploro engrosado (oliva pilrica). Para encontrar la oliva es necesario que el paciente est tranquilo y la musculatura abdominal
relajada. Los porcentajes del hallazgo varan entre los autores y dependen de la experiencia del explorador; algunos indican cifras que
van de 85 a 100 %. Signo del bibern: al tomar lquidos por bibern se observan ondas peristlticas. Signo de la pelota de golf: ondas
peristlticas de izquierda a derecha hacia la zona pilrica. El hallazgo ms caracterstico es la alcalosis metablica, por la prdida de
hidrogeniones a travs del contenido gstrico; con el vmito se pierde tambin cloruro lo que lleva a hipocloremia, que completa el
panorama de la bioqumica sangunea. Tambin puede haber hipokalemia. Se puede solicitar hemograma completo para descartar
algn proceso sptico como causa de los vmitos. En la estenosis del ploro el hemograma est normal; se describe sin embargo que
puede haber hemoconcentracin por la deshidratacin. Por ello puede encontrarse elevacin de la creatinina; la densidad urinaria
tambin puede estar elevada, sin datos de infeccin. Otro hallazgo es la elevacin de la bilirrubina indirecta, que se puede explicar por
la disminucin del trnsito gastrointestinal que produce aumento de la circulacin enteroheptica de bilirrubina, aunque otros autores
mencionan disminucin de la actividad de la glucoroniltransferasa por el ayuno. La radiografa simple de abdomen muestra distensin
de la cmara gstrica con poco o escaso aire distal. La serie esfago gastroduodenal muestra un estrechamiento del canal pilrico (cola
de ratn) con una o varias imgenes de cuerda; es el medio de contraste que dibuja los pliegues hipertrofiados de la mucosa pilrica;
se describe tambin un efecto de masa que se proyecta hacia el antro gstrico (signo del hombro); se observa falta de progresin de las
ondas peristlticas del estmago en la fluoroscopia. En la actualidad, el mtodo ms utilizado para confirmar el diagnstico clnico es el
ultrasonido abdominal. Se logra evidenciar engrosamiento de la capa muscular del ploro mayor de 2.5 mm, aumento en la longitud del
canal pilrico mayor de 15 mm y estrechamiento del canal; se logra visualizar tambin hipertrofia de la mucosa en grados variables,
corte axial Imagen ojo de bovino dona Imagen tiro al blanco. La endoscopia tiene sensibilidad y especificidad 100%. TRATAMIENTO:
El tratamiento actualmente es quirrgico. Sin embargo se deben corregir los trastornos hidroelectrolticos (alcalosis, hipocloremia,
hipokalemia y deshidratacin) antes de que el paciente sea llevado al quirfano. Se pueden suministrar bolos de solucin de cloruro de
sodio al 0,9 % (solucin salina normal) a 20 mL/kg para restituir volumen y electrolitos. Luego se puede dejar una solucin de
mantenimiento, con dextrosa al 5 % y cloruro de sodio al 0,45 % o al 0,9 %, segn el centro hospitalario, adems se debe agregar
cloruro de potasio a razn de 20 mEq/L (3.4mEq/kg/da) una vez asegurada la diuresis. La estabilizacin puede demorar unas 24 a 48 h.
La ciruga no es urgente; si el paciente es llevado a sala de operaciones con alcalosis metablica se aumenta el riesgo de apnea
posoperatoria; adems, el estrs quirrgico puede empeorar los trastornos electrolticos. Actualmente es raro observar desnutricin
grave como en aos atrs, cuando el diagnstico se retrasaba. Se debe mantener una sonda nasogstrica a drenaje que ayuda a
descomprimir el estmago, previene la aspiracin posoperatoria y tambin la atona gstrica. Nio con electrlitos normales menos del
5% de la deshidratacin ciruga inmediata. Vitamina K: 0.4 mg/kg en nios con peso inferior a 2.5 kg. > 1 ao 5-10 mg/da por va I.V. o
I.M. Ranitidina 1 mg/kg/dosis cada 8 hrs. Una vez compensado el paciente se lleva a cabo la Piloromiotoma de Fredet- Ramsted, se
realiza el corte de la serosa y luego de la capa muscular circular del ploro, sin cortar la mucosa; luego se deja sin suturar la muscular y
se cierra la serosa. La operacin generalmente se hace por laparotoma, aunque tambin se puede a travs de tcnicas laparoscpicas.
COMPLICACIONES: Las complicaciones estn relacionadas con los trastornos electrolticos y con la ciruga. Se citan principalmente:
apnea posoperatoria, hipoglicemia, obstruccin posoperatoria (vmitos), perforacin de la mucosa (duodenal), miotoma incompleta,
eventracin de la herida quirrgica; el fallecimiento ocurre en menos del 1 % de los casos

CASO CLINICO
Se trata de un paciente masculino de cuatro semanas de edad, el cual acude por historia de 2 dias de evolucin caracterizado por
vomitos postpandriales abundantes, se establecen medidas antireflujo, sin embargo continuo la sintomatologa aumentando el vomito
en proyectil de contenido alimentario.

PREGUNTA
Cual es la conducta diagnostica mas adecuda en este momento?

RESPUESTA
a.- Radiografia de torax.
b.- USG abdominal.
c.- Radiografia de abdomen.
d.- Endoscopia.

PREGUNTA
Cual de los siguientes diagnosticos diferenciales es el mas frecuente en esta patologa?

RESPUESTA
a.- Intolerancia a la lactosa.
b.- Reflujo gastroesofgico.
c.- Meningitis.
d.- Volvulus.








MANUAL DE TRABAJO DEL CURSO ENARM CMN SIGLO XXI
CURSO ENARM CMN SIGLO XXI TEL: 36246001 Pharmed Solutions Institute PGINA 373


CARDIOPATIAS CONGENITAS
CIENCIAS BASICAS: Corazn primer rgano que alcanza desarrollo (de 3ra-8va semana de gestacin) funcional completo, el da 17 de
gestacin escuchamos latido cardiaco. Definicin: Conjunto de enfermedades caracterizadas por anomalas estructurales del corazn o
de los grandes vasos intratorcicos. Los cambios en la transicin de la circulacin fetal a la neonatal; en el feto existen 3 estructuras
nicas: el conducto venoso, el agujero oval y el conducto arterioso. La sangre oxigenada llega de la vena umbilical a la vena cava inferior
a travs del conducto venoso, evitando el hgado. Gracias a la anotoma de la aurcula derecha, la mayor parte de la sangre proveniente
de la vena cava inferior (sangre oxigenada) se dirige al corazn izq. Por medio del agujero oval, mientras que la sangre proveniente de
la vena cava superior (sangre desoxigenada), se dirige al ventrculo derecho a travs de la vlvula tricspide. La sangre que irriga el
pulmn tienen un contenido de oxigeno sumamente bajo y esto produce una constriccin intensa de los vasos pulmonares que eleva la
resistencia vascular. El ventrculo der. bombea la sangre hacia la arteria pulmonar; sin embargo la alta resistencia pulmonar hace que la
sangre fluya principalmente a travs del conducto arterioso hacia la aorta descendente. Los cambios fisiolgicos y mecnicos que
ocurren al nacer y que propician el cierre del conducto arterioso son: la disminucin de la resistencia pulmonar (por la distencin
alveolar y el incremento del contenido de oxgeno en la sangre), la secrecin de bradicinina por el endotelio pulmonar y disminucin de
la contraccin srica de las prostaglandinas placentarias. Junto con el flujo pulmonar, estos cambios incrementan el volumen y la
presin en la aurcula izq., rebasando la de la aurcula derecha y cerrando el agujero oval. SALUD PUBLICA: Su incidencia es del 1%, lo
que las coloca entre las malformaciones ms frecuentes. En cerca de 90% de los casos no existe una causa identificable, pero existen
ciertos factores como como la prematurez, cromosomopatas, infecciones congnitas, diabetes, alcohol, entre otros.
CLASIFICACION:
CARDIOPATIAS POR FISOPATOGENIA (por orden de incidencia)
Obstructivas CORTOCIRCUITOS
Coartacin de la
aorta (5-7)
Estenosis pulmonar
Estenosis aortica
Derecha a izquierda (cianticas)
Sangre pobremente oxigenada pasa a la circulacin sistmica,
ello condiciona a hipoxemia y cianosis
Izquierda a derecha (acianticas)
Propician aumento del volumen circulatorio
en el sistema pulmonar, a expensas de una
disminucin del gasto cardiaco sistmico, no
cianosis pero si hipertrofia del VD y vasos
pulmonares
Flujo pulmonar aumentado Flujo pulmonar disminuido Comunicacin interventricular CIV (25-30)
Comunicacin interauricular CIA (6-8)
Conducto arterioso persistente CAP (6-8)
Defecto de cojinetes endocardicos
Transposicin de grandes vasos
(3-5)
Tronco arterioso (1-2)
Ventrculo nico
Hipoplasia de VI
Tetraloga de Fallot (5-7)
Atresia tricuspidea
Atresia pulmonar


TETRALOGIA DE FALLOT: Cardiopata congnita ms comn de las cianticas (3-6 por 10,000). Implica obstruccin del tracto de salida
del VD (estenosis pulmonar); comunicacin interventricular (CIV); hipertrofia del VD y dextroposicin de la aorta con cabalgamiento
sobre la CIV, todo esto lleva a obstruccin del flujo del VD, hacia la arteria pulmonar y cortocircuito de derecha a izquierda. Leve no
muestran cianosis hasta despus del ao, en grave cianosis poco despus de nacimiento, de no corregirse el paciente presenta color
azulado, acropaquias de pies y manos (dedos en palillo de tambor) y disnea, pueden sufrir ataques hipercianticos paroxsticos con el
esfuerzo que lo pueden llevar al sincope, si se ponen en cunclillas mejora su estado. La viabilidad del producto depende del conducto
arterioso y del agujero oval. DIAGNOSTICO: Las Rx: corazn con aspecto de bota, trama vascular pulmonar disminuida, ECG: hipertrofia
del VD y desviacin del eje elctrico, Ecocardiograma: Estndar de oro, para dx y gravedad. TRATAMIENT: Ciruga correctiva,
inicialmente PG E1 para mejorar sntomas. TRANSPOSICION DE LOS GRANDES VASOS: Aqu la arteria pulmonar nace del VI y la aorta
del VD. La cianosis y la taquipnea suelen ser evidentes en las primeras horas o das de vida y representan una urgencia mdica.
DIAGNSTIC: La Rx: normal o datos de hiperflujo pulmonar. Ecocardigrafa es estndar de oro para dx. TRATAMIENT: Dar PG E1
para mantener abierto el conducto arterioso, la correccin quirrgica final es la conmutacin o switch arterial. TRONCO ARTERIOSO:
Defecto en el desarrollo del tronco arterioso en aorta y arteria pulmonar, dejando un tronco arterial nico que nace del corazn, para
ambas circulaciones. COMUNICACIN INTERVENTRICULAR: Cardiopata congnita ms comn en la infancia, durante los dos primeros
aos de vida casi la mitad de estos defectos se cierran espontneamente y la mayora lo hace para los 10 aos. Se dividen en;
membranoso (ms comn) y muscular. El cuadro clnico depende de la cantidad de sangre que fluye hacia la circulacin pulmonar, en la
mayora es pequeo y permanecen asintomticos. DIAGNOSTICO: Defectos grandes disnea, dificultad para la alimentacin, infecciones
pulmonares recurrentes e IC. Auscultacin soplo holosistlico, a veces frmito. La Rx: cardiomegalia y aumento de trama vascular. La
CIV muscular es menos comn y su cierre espontaneo es ms probable. TRATAMIENTO: Ciruga, solo si hay retraso en el crecimiento,
grandes defectos o hipertensin pulmonar. COMUNICACIN INTERAURICULAR: Cardiopatia congnita ms comun en el adulto (1 de
cada 3 casos). En cualquier parte del septo, los principales tipos son: ostium secundum (90%), ostium primum y seno venoso, el grado
de cortocircuito y mezcla de sangre depende del tamao del defecto, generalmente no presentan sntomas hasta 4ta dcada de vida, la
presentacin ms comn es la falla cardiaca derecha. DIAGNOSTICO: Auscultacin, S2 con desdoblamiento fijo, por hiperflujo
pulmonar, un soplo de eyeccin de bajo grado. TRATAMIENTO: Correccin quirrgica de defectos grandes, contraindicada cuando la
relacin de las resistencias arteriales pulmonar: sistmica es mayor a 0.7. Requiere profilaxis para endocarditis bacteriana. CONDUCTO
ARTERIOSO PERSISTENTE: Su cierre normal ocurre entre 4-10 das de nacimiento, si no sucede la sangre aortica se desva hacia la
arteria pulmonar, el cierre despus de la infancia es infrecuente, el principal factor de riesgo es la premadurez e infeccin materna por
rubeola. DIAGNOSTICO: los conductos grandes pueden producir presin de pulso amplio, pulsos perifricos saltones, insuficiencia
cardiaca y retraso en el crecimiento a largo plazo. Auscultacin: soplo caracterstico es sistlico; suele describirse como ruido de
maquinaria. La Rx: incremento en la trama vascular y una arteria pulmonar ensanchada. Ecocardiograma estudio de eleccin.
TRATAMIENTO: Las probabilidades de que cierre espontneamente son escasas, para ayudar a que se cierre se da indometacina
(inhibidor de la sintesisi de PG). En casos extremos es necesaria la ciruga, ya que disminuye el riesgo de endocarditis infecciosa.
MANUAL DE TRABAJO DEL CURSO ENARM CMN SIGLO XXI
CURSO ENARM CMN SIGLO XXI TEL: 36246001 Pharmed Solutions Institute PGINA 374


COARTACION DE LA AORTA: Causada por el estrechamiento de la aorta en algn punto desde el cayado hasta la bifurcacin de las
arterias iliacas. Este trastorno se relaciona con vlvulas aorticas bicspides (70%). Relacin importante con el Sx. de Turner. Hay dos
tipos: coartacin juxtaductal antes conocida como del adulto y la infantil (coartacin preductal) DIAGNOSTICO: Cuadro clnico depende
de la localizacin; la presin arterial esta aumentada en los vasos proximales a la coartacin y conlleva una disminucin distal a la
estenosis, signo clsico es la disparidad de la presin arterial entre los brazos y las piernas. La Rx: congestin pulmonar, cardiomegalia.
Ecografa cardiaca estndar de oro para dx. El tratamiento de la forma infantil se administra PG E1, la ciruga correctiva es necesaria,
en la juxtaductal se deben controlar la hipertensin y la falla cardiaca y despus realizar la ciruga. PREVENCION: La profilaxis est
indicada en todos los casos de cardiopatas cianticas, durante los primeros 6 meses despus de una reparacin quirrgica completa de
un defecto y en todos los casos de reparaciones incompletas en las que exista material prosttico cerca del defecto. Las opciones mas
comunes son: 1. Amoxicilina 2 gr VO; 1 hr antes de procedimiento quirrgico. 2. Ampicilina 2gr IV en caso de intolerancia a la VO. 3.
Macrlidos o cefalosporinas de 3ra generacin en alegra a las penicilinas. PRONOSTICO: Los avances en el tratamiento mdico y
quirrgico de las cardiopatas congnitas han permitido que los pacientes que las padecen lleguen a la vida adulta

CASO CLINICO
Una recin nacida (RN) de 19 das de vida es admitida en la sala de emergencias en estado grave. En el examen fsico, la paciente se
presentaba hipotnica, con hipotermia, gimiente, ciantica con aleteo de la nariz, taquipneica (80 rpm), con tiraje y severa retraccin
subcostal y esternal, murmullo vesicular (MV) presente con crepitantes difusos, roncus, taquicrdica (190 lpm), pulso dbil, ritmo
cardaco regular (RCR), ruidos cardacos normofonticos (RCNF), soplo sistlico +++/4+, abdomen distendido, hgado a 4 cm por debajo
del reborde costal derecho. La paciente es remitida en la Unidad de Cuidados Intensivos Neonatales y se realiz intubacin orotraqueal
(IOT), con FiO2 al 100%, establecindose un acceso perifrico e inicindose correccin con solucin de bicarbonato de sodio debido a la
presencia de acidosis metablica, diseccin venosa de la vena axilar derecha y la siguiente medicacin: furosemida, dobutamina,
milrinona, fentanilo y midazolam. Se realizaron exmenes (PCR negativa, cultivo de orina con presencia de Staphylococcus coagulasa
resistente a ampicilina/penicilina), se realiz una radiografa de trax que mostr la presencia de cardiomegalia.

PREGUNTA
Cual es su diagnostico mas probable?

RESPUESTA
a.- CIV.
b.- CIA.
c.- CAP
d.- TGV

































MANUAL DE TRABAJO DEL CURSO ENARM CMN SIGLO XXI
CURSO ENARM CMN SIGLO XXI TEL: 36246001 Pharmed Solutions Institute PGINA 375


KWASHIORKOR. CIENCIAS BASICAS: Forma de desnutricin que existe cuando no hay suficiente protena en la dieta, es agudo. La
palabra 'washioror', fue usada inicialmente en Ghana y traduce enfermedad del nio destetado, trmino utilizado por las madres
para describir la enfermedad del primer hijo cuando nace el segundo. La desnutricin tipo kwashiorkor (edematosa) es ms frecuente
en lactantes con dficit de protenas pero adecuada ingesta de carbohidratos y se caracteriza por deplecin de protenas sricas,
principalmente la albumina, que tiende a ser ms severa en esta forma de desnutricin. La gravedad o grado de desnutricin se
determina siguiendo la clasificacin de la OMS, basada en el grado de prdida de peso: Desnutricin grado I, hay prdida del 15-25% de
peso. Grado II, hay prdida de 25-40% de peso. Grado III, hay prdida de ms de 40% de peso. SALUD PUBLICA: Este tipo de DNT es
ms comn en las zonas rurales de pases en vas de desarrollo y afecta principalmente a los nios de 2 aos de edad, asociado al
periodo de destete e inicio de dieta rica en almidn y pobre en protenas. Afecta aproximadamente una dcima parte de los nios
<5aos. PATOGENIA: Es una condicin compleja, multifactorial, en la cual existe una combinacin de factores geogrficos, climticos,
educacionales, psicosociales, culturales, nutricionales, y otros factores. Puede ser considerado el punto final de la combinacin de
todos estos en un organismo que no tiene la capacidad de adaptarse a un cambio de condiciones, que lleva a que se desarrollen las
caractersticas clnica. Cuando un nio nace, recibe ciertos aminocidos vitales para el crecimiento procedentes de la leche materna.
Cuando el nio es destetado, si la dieta que reemplaza a la leche tiene un alto contenido en carbohidratos, y es deficiente en protenas,
como es comn en diferentes partes del mundo donde el principal componente de la dieta consiste en almidones vegetales, o donde el
hambre hace estragos, los nios pueden desarrollar Kwashiorkor. El origen del edema en el paciente con kwashiorkor es multifactorial.
La teora clsica postula que una ingesta inadecuada de protena lleva a bajas concentraciones de albumina en plasma que llevan a
edema. Sin embargo, esta hiptesis ha cambiado, pues se ha encontrado que no existe diferencia en la concentracin de albumina en el
plasma antes y despus de la disminucin del edema. El edema nutricional est asociado con un aumento en la secrecin de hormona
antidiurtica (ADH) que previene la respuesta excretora normal a la administracin de agua. La ferritina activada es liberada por el
hgado daado, llevando a aumento de la secrecin de ADH cuyo resultado neto es la retencin de agua. La aldosterona tambin es
normalmente inactivada en el hgado, y se ha reportado un metabolismo inadecuado en los pacientes con enfermedades hepticas,
llevando a hiperaldosteronismo que podra ser el responsable de la retencin de sodio. Otras caractersticas intrnsecas del individuo
como variaciones en isoenzimas o en concentracin de enzimas podran influir. Altos niveles de glutamato y bajos o indetectables de
alanina son la caracterstica del kwashiorkor, asociado a menores niveles de aminotransferasas. Es posible que estas diferencias
influyan en la dismil respuesta frente al estrs de una dieta inadecuada y pueden ser responsables del desarrollo de kwashiorkor.
DIAGNOSTICO: Clnico; es anorxico, irritable, con facies abotagada cara de
luna llena, edema peri orbitario y edema con fvea en extremidades inferiores
y manos, abdomen globoso con hepatomegalia (infiltracin grasa del hgado;
hgado graso)) y dilatacin de asas intestinales. La piel es seca (dermatosis) y el
cabello se cae fcilmente, perdida del inters y del apetito. El peso
generalmente es adecuado para la edad. El edema, signo cardinal del
kwashiorkor, tpicamente inicia en el dorso de los pies y en las piernas. Despus
se extiende a otras partes del cuerpo incluyendo manos, antebrazos, espalda,
extremidades superiores y en casos severos en la cara, principalmente en las
mejillas y alrededor de los ojos. Generalmente corresponde al 5-20% del peso
corporal. Puede desarrollarse ascitis (por ausencia de protenas en la sangre) o
derrame pleural como manifestaciones tardas del edema. Otros cambios
fisiolgicos incluyen atrofia del musculo cardiaco con disminucin del gasto
cardiaco, insuficiencia circulatoria y bradicardia. Tienen elevacin de TSH
(hormona estimulante tiroides) con funcin tiroidea normal, pero con
disminucin de protenas plasmticas que se unen a la tiroxina que llevan a bajas
concentraciones plasmticas de tiroxina .Las clulas de la mucosa intestinal y del
pncreas se encuentran atrficas llevando a alteracin de la sntesis y actividad de disacaridasas, disminucin del rea de absorcin que
se traduce en alteracin en la utilizacin de nutrientes, e intolerancia a la lactosa .Presentan adems disminucin de la tasa de filtracin
glomerular, menor capacidad de concentrar la orina, trastornos hidroelectrolticos principalmente hipokaliemia, atrofia del timo con
compromiso de la respuesta inmunolgica principalmente la medicada por clulas que lleva a mayor susceptibilidad a infecciones .La
hipoglicemia es un fenmeno comn en pacientes con kwashiorkor, aunque tambin se ha descrito en pacientes con emaciacin
severa. Tericamente la hipoglicemia puede estar causada por alteracin en la produccinheptica endgena de glucosa o por aumento
de la eliminacin de la misma. Esta complicacin aporta de manera importante a la mortalidad. Para realizar el diagnstico de
desnutricin severa, la OMS define malnutricin aguda severa como una circunferencia superior del brazo (MUAC por sus siglas en
ingles) < 11.5 cm, un puntaje z de peso para la talla (P/T) < -3 o la presencia de edema bilateral pedio (kwashiorkor). En ausencia de
evaluacin antropomtrica, tambin puede diagnosticarse desnutricin severa por la presencia de emaciacin severa visible, definida
como la presencia de emaciacin muscular en la regin gltea, perdida de la grasa subcutnea o prominencia de las estructuras seas
particularmente en el trax Se debe tener en cuenta que el peso puede estar afectado por diferentes variables como el edema o la
deshidratacin, ambos presentes con alta frecuencia en estos pacientes, y que pueden llevar a clasificaciones inadecuadas si solo se
tiene en cuenta este parmetro. En las alteraciones bioqumicas, las ms comunes son los bajos niveles de albumina y hemoglobina.
TRATAMIENTO: Generalmente, la enfermedad puede ser tratada aadiendo a la comida alimentos energticos y protenas; sin
embargo, la letalidad puede ser tan alta como del 60% y puede haber secuelas a largo plazo como nios con talla corta, y en casos
severos, desarrollo de retraso mental. El manejo en casa se recomienda para aquellos pacientes sin complicaciones. Los nios con
kwashiorkor severo o asociado a complicaciones requieren tratamiento intrahospitalario. Generalmente 5-30% de los nios requieren
hospitalizacin. Como las principales causas de muerte son la deshidratacin, alteraciones electrolticas, hipoglicemia e infecciones, el
tratamiento debe empezar con la correccin de estas. La sepsis se presenta en el 15-60% de los nios con malnutricin severa
complicada, por lo tanto es necesario iniciar antibiticos de amplio espectro. Estos pacientes generalmente tienen niveles bajos de
hemoglobina (6-10 g/dl), aunque rara vez tienen perdida aguda de sangre. La transfusin puede empeorar la falla cardiaca y se ha
MANUAL DE TRABAJO DEL CURSO ENARM CMN SIGLO XXI
CURSO ENARM CMN SIGLO XXI TEL: 36246001 Pharmed Solutions Institute PGINA 376

identificado como un factor de riesgo para muerte. La OMS recomienda transfundir solo si la hemoglobina es < 4 g/dl . Debe
administrarse suplementos de vitaminas A, D, cido flico y algunas del complejo B. En cuanto a la rehabilitacin nutricional, la OMS en
el ao 2000 defini las orientaciones para el manejo hospitalario de estos nios, las cuales peridicamente han sido actualizadas. Se
basa en recomendar formulas especiales (F75/F100).

CASO CLINICO
Nio de 3 aos de muy bajo nivel socioeconmico, Cursa desde hace dos aos con poliuria, polidipsia, astenia, y perdida de peso.
Ingresa por presentar estado de estupor y signos de deshidratacin severa. Peso: 9 kg, Talla: 92 cm. Frecuencia respiratoria: 25 por
minuto, T corporal y PA: normal. Malas condiciones generales, marcada palidez muco-cutnea, perdida de tejido subcutneo,
secrecin purulenta en conductos auditivos, lesiones ulcerosas en lengua, membranas blanquecinas en cavidad oral, quelitis sangrante.
Abdomen globoso, distendido sin megalias, edema +++.

PREGUNTA
Considerando el estado de desnutricin y las complicaciones que potencialmente desarrollar, cual de los siguientes es el mas
probable?

RESPUESTA
a.- Alcalosis metabolica
b.- Acidosis metabolica
c.- Acidosis respiratoria.
d.- Alcalosis respiratoria.

PREGUNTA
Cual es su grado de desnutricin?

RESPUESTA
a.- Grado I.
b.- Grado II.
c.- Grado III.
d.- Grado IV.

PREGUNTA
Cual es la complicacin endocrinolgica mas probable para el caso?

RESPUESTA
a.- Hipogonadismo hipogonadotrofico.
b.- Diabetes mellitus.
c.- Hipotiroidismo yodoprivativo.
d.- Cetoacidosis diabtica.

MARASMO. CIENCIAS BASICAS: Es una forma de desnutricin crnica (no edematosa) caracterizada clnicamente por emaciacin
severa (flaqueza exagerada) del tejido subcutneo, msculos y grasa, que da lugar a un aspecto, envejecido o arrugado, debida a la
carencia de protenas, su causa primaria es el aporte inadecuado de caloras en la dieta alimentaria, si diferencia en la relacin
protenas/energa. Un nio con marasmo aparece esculido y su peso corporal puede reducirse hasta menos del 80% de su peso
normal para su altura. SALUD PUBLICA: Habitualmente en menores de 18 meses. PATOGENIA: Como la disminucin del aporte
energtico no puede compensar el requerimiento, se utiliza grasa corporal como sustrato de energa, con la consiguiente disminucin
del tejido celular subcutneo. Como hay un dficit de aporte tanto de protenas como
de energa, el fenmeno de adaptacin eleva niveles de glucagn y cortisol y
disminuye la insulina con los efectos orgnicos pertinentes. Los msculos son los ms
afectados por la expoliacin ya que proporcionan los aminocidos esenciales, para el
mantenimiento de la sntesis proteica visceral y as producir cantidades adecuadas de
albumina srica y beta-lipoprotenas, que provienen de la presencia de edemas o
infiltracin de grasa del hgado. En el intestino delgado existe dao de la mucosa de
carcter transitorio con alteraciones mitticas. Hay dficit de produccin de tripsina,
quimiotripsina, amilasa y lipasa, originando una malabsorcin. Quizs las causas
precipitantes ms importantes del marasmo son las infecciones y enfermedades
parasitarias de la infancia. Estas incluyen sarampin, la tosferina, diarrea, malaria, y
otras debidas a parsitos. Las infecciones crnicas como la tuberculosis pueden
tambin llevar al marasmo. Otras causas comunes del marasmo son el parto
prematuro, la deficiencia mental y las molestias digestivas, como malabsorcin o
vmito. Una causa muy comn es tambin la interrupcin temprana de la lactancia.
DIAGNOSTICO: Crecimiento deficiente: En todos los casos el nio no crece en forma
adecuada. Si se conoce la edad, el peso ser muy bajo. En los casos graves la prdida
muscular es obvia: las costillas sobresalen; el estmago, en contraste con el resto del cuerpo, puede ser protuberante; la cara tiene una
caracterstica simiesca (como un mono); y las extremidades inferiores son muy delgadas. El nio parece ser slo piel y huesos. Un caso
avanzado de la enfermedad es inconfundible, y una vez que se ve, jams se olvida. Emaciacin: Los msculos siempre se encuentran
MANUAL DE TRABAJO DEL CURSO ENARM CMN SIGLO XXI
CURSO ENARM CMN SIGLO XXI TEL: 36246001 Pharmed Solutions Institute PGINA 377

muy disminuidos. Hay poca grasa subcutnea, si es que queda algo. La piel cuelga en arrugas, sobre todo alrededor de las nalgas y los
muslos. Cuando se toma la piel entre el ndice y el pulgar se nota la ausencia de la capa habitual de tejido adiposo. Estado de alerta: Los
nios con marasmo raramente son desinteresados como los que sufren kwashiorkor. Los ojos profundamente hundidos les dan una
apariencia bastante despierta. Tambin se puede manifestar en modo menos infeliz e irritable. Apetito: El nio por lo general tiene
buen apetito. En realidad, como cualquier individuo hambriento, el nio puede ser voraz. Los nios con marasmo a menudo se chupan
las manos violentamente o la ropa o cualquier otra cosa a su alcance. Algunas veces emiten sonidos de succin. Anorexia: Algunos
nios son anorxicos. Puede aparecer pero solo algunos casos, muy pocos. Diarrea: La materia fecal puede ser suelta, pero no es una
caracterstica constante de la enfermedad. La diarrea de naturaleza infecciosa, como ya se mencion, puede comnmente haber sido
un factor precipitante. Anemia: Casi siempre se encuentra anemia. Ulceraciones en la piel: Puede haber lceras por presin, pero por lo
general estn sobre las prominencias seas, no en reas de friccin. En contraste con el kwashiorkor, no existe edema y en el marasmo
no hay dermatosis en copos de pintura. Cambios del cabello: Puede haber cambios semejantes a los del kwashiorkor. Es ms comn un
cambio en la textura que en el color. Deshidratacin: Aunque por s misma no es una caracterstica de la enfermedad, es comn que la
deshidratacin acompae al marasmo; como resultado de una fuerte diarrea (y algunas veces del vmito). TRATAMIENTO: Es esencial
tratar no slo los sntomas sino tambin las complicaciones de estos desrdenes tales como infecciones, deshidratacin y trastornos del
aparato circulatorio que frecuentemente son letales y provocan una alta mortalidad si son ignorados. Consiste en la aplicacin de una
dieta equilibrada, en la que el componente ms importante sea la leche, que en un principio se administra de forma diluida y poco a
poco se va haciendo ms concentrada. Una vez que la ingestin de protenas es adecuada, se aaden caloras en forma de azcar y
cereales. El tratamiento debe ser establecido poco a poco para lograr la readaptacin de las funciones metablicas e intestinales en los
seres humanos.

CASO CLINICO
Un varn de 6 meses de edad, nacido a trmino, que presenta desnutricin severa. Naci de una madre alimenta normalmente, tras un
embarazo sin complicaciones y peso al nacer 2,680 g,). Exclusivamente amamantado y pareca estar bien alimentado. En el segundo
mes de vida, empez a mostrar falta de crecimiento, sin antecedentes de vmitos o diarrea. Dos semanas ms tarde, desarrollo
erupcion difusa no pruriginos. A la exploracin se oservo pelo escaso con un aspecto plateado rubio. Se observa irritable, con tos no
productiva. Estaba alerta pero caquctico severamente. Su peso (3.245 kg), longitud (51,5 cm), y circunferencia de la cabeza (37,5 cm).

PREGUNTA
Cul de lo siguientes hallazgos de laboratorio es menos probable encontrar.

RESPUESTA
a.- Anemia normoctica.
b.- Leucopenia.
c.- Hipoalbuminenia.
d.- Neutrofilia.

PREGUNTA
Cul de las siguientes dosificaciones esta contraindicada por generar mayor deterioro respecto al estado nutricional.

RESPUESTA
a.- Vitamina A 5mg primera dosis luego 1mg/dia.
b.- Zinc 6mg /kg/dia.
c.- Cobre 0.3 mg/Kg/dia.
d.- Hierro 3mg/Kg/dia. A partir de la segunda semana.

PREGUNTA
Cual de las siguientes complicaciones se presenta ms frecuentemente en el paciente desnutrido que por general pasa desapersibidas.

RESPUESTA
a.- Infeccin de vas urinarias.
b.- Otitis media.
c.- Tuberculosis.
d.- Guardiasis.

CASO CLINICO
Un varn de 11 meses alimentado exclusivamente a seno materno, vive en zona rural y es el ultimo hijo de 8 en total, madre dedicada
al hogar con educacin primaria, reingres 10 das despus de un episodio previo de gastroenteritis con diarrea durante 2 das y
deshidratacin, Despus de la rehidratacin estaba por debajo del 3er percentil para el peso. 2 semanas despus de la admisin la
diarrea es severa. Una semana ms tarde la diarrea era con sangre, fiebre. Egresa una semana despus de tratamiento
intrahospitalario.

PREGUNTA
Cual es la conducta a seguir.

RESPUESTA
a.- Suplementar alimentacin.
MANUAL DE TRABAJO DEL CURSO ENARM CMN SIGLO XXI
CURSO ENARM CMN SIGLO XXI TEL: 36246001 Pharmed Solutions Institute PGINA 378

b.- Integracin a la dieta familiar.
c.- Alimentacion cada dos horas suplementada.
d.- Referencia a segundo nivel.

PREGUNTA
El paciente anterior reingresa por presentar irritabilidad, llanto sin lagrimas e incontrolable, hipotnico, hipotrmico con lienzo hmedo
y reporta hipoglucemia la cual es controlada oralmente. Considerando la comorbilidad de la desnutricin cual es la conducta
profilctica a seguir.

RESPUESTA
a.- Administrar trimetropin sulfametoxazol.
b.- Administrar cefotaxima.
c.- Administrar ampicilina ms gentamicina.
d.- Administrar cloranfinicol IV.

PREGUNTA
Considerando el estado hdrico del paciente, los antecedentes y la comorbilidad cual es su manejo de mantenimiento de liquidos.

RESPUESTA
a.- 100 ml/kg/dia.
b.- 130 ml/kg/dia.
c.- 160 ml/Kg/dia.
d.- 180 ml/Kg/dia.

PREGUNTA
El paciente es egresado por mejora despus de 7 dias de tratamiento, con respecto a la recuperacin ponderal usted suplementa
micronutrientes y aporte proteico de 1 a 1.5 g/kg/dia hasta 4 a 6 g/kg/dia. Cul es el incremento ponderal que esperara usted con las
indicaciones al egreso considerando que lo debe de citar semanalmente.

RESPUESTA
a.- 100 g semanal.
b.- 150 g semanal.
c.- 200 g semanal.
d.- 250 g semanal.































MANUAL DE TRABAJO DEL CURSO ENARM CMN SIGLO XXI
CURSO ENARM CMN SIGLO XXI TEL: 36246001 Pharmed Solutions Institute PGINA 379


AVITAMINOSIS:
CIENCIAS BASICAS: Dficit de vitaminas, ya sea por falta de ingesta, absorcin inadecuada a nivel intestinal, su mala utilizacin
metablica o el aumento de la demanda (como en el embarazo por ejemplo). Como las vitaminas participan en las reacciones como
coenzimas, diversos procesos metablicos podran verse afectados. Ejemplos son el raquitismo por dficit de vitamina D, ceguera
nocturna por dficit de vitamina A. Las vitaminas son nutrientes esenciales, orgnicos, de bajo peso molecular, imprescindibles para el
crecimiento, desarrollo y mantenimiento del organismo, y que el hombre necesita adquirir a travs de los alimentos. Aunque los
alimentos de origen animal pueden ser fuentes inmediatas de vitaminas, las plantas son sus principales suministradores, a causa de su
mayor capacidad de sntesis de los precursores metablicos de las vitaminas. Los requerimientos de vitaminas dependen de la
composicin de nutrientes de la dieta y de las diferentes condiciones de vida del individuo. Asimismo, deben adecuarse a las
poblaciones en particular, a sus caractersticas fsicas, gastos de energa, fuentes alimentarias, hbitos, actitudes alimentarias y a las
deficiencias nutricionales detectadas. As, por ejemplo, las prdidas de vitaminas por sudor pueden alcanzar cifras importantes en
pases tropicales. Tambin algunas vitaminas se necesitan en mayores cantidades por individuos que realizan trabajos fsicos intensos,
as como durante el embarazo, la lactancia y el crecimiento, y en estados febriles y algunas enfermedades crnicas. Las vitaminas son
absorbidas, en general, de 20 a 95 %; as por ejemplo, la vitamina A se absorbe de un 70 a 90 %, la vitamina D entre 80 y 90 %, la
vitamina C entre 80 y 95 % y la vitamina B6 entre 95 y 98 %, mientras que la vitamina E solo se absorbe entre 20 y 40 % y los carotenos
entre 20 y 50 %.
VITAMINAS LIPOSOLUBLES
VITAMINA FUNCION DEFICIENCIA FUENTES DIETETICAS
A Intervienen en la percepcin visual.
Necesaria para el mantenimiento de los
tejidos epiteliales
XEROFTALMIA (caracterizada por una sequedad del globo ocular y una falta
de brillo en la superficie ocular, se acompaa de una leve prdida de visin,
especialmente por la noche). La carencia de vitamina A es una enfermedad
general que afecta a clulas y rganos de todo el cuerpo; los cambios
resultantes de la arquitectura epitelial reciben la denominacin de
"metaplasia queratinizante", la cual se produce con relativa rapidez en las
vas respiratorias y urinarias. Segn estimaciones de la OMS, alrededor de
250 000 000 de nios menores de 5 aos sufren carencia de vitamina A en
todo el mundo. En los pases en desarrollo la carencia de este nutriente tiene
carcter grave como para provocar daos oculares permanentes en unos 13
000 000 de nios y ceguera a 5 000 000, causante de 25 % de las muertes en
la poblacin infantil;
Vegetales verdes,
legumbres frescas
D Estimula la absorcin intestinal de Ca.
Condiciona depsito de Ca y P en los
huesos
RAQUITISMO: Caracterizada por reblandecimiento de los huesos del crneo,
asociado generalmente a un agradamiento exagerado de este, el trax se
aplasta trasversalmente, el esternn es prominente y las articulaciones de la
rodilla se abultan; en las extremidades inferiores, por la accin del peso
corporal, aparecen incurvaciones de los huesos largos y desviaciones del eje
longitudinal de los mismos. Esta deformacin se manifiesta cuando los nios
empiezan a andar; es frecuente la aparicin de fracturas de hueso y la
movilidad de las articulaciones es muy amplia debida a la laxitud de los
ligamentos, por lo que estos nios adoptan posiciones raras e inverosmiles.
Son frecuentes las lesiones musculares debido a la debilidad de los msculos.
Otras manifestaciones del raquitismo son: abultamiento del vientre, retraso
en la denticin, implantacin defectuosa de los dientes, disminucin de la
talla en relacin con la normal. TRATAMIENTO: Se basa en la administracin
diaria de vitamina D por va oral. La dosis ser de 2 500 a 5 000 U (62 a 125
mg), y puede llegarse a 10 000 U/da (250 mg), lo que corresponde a una
cantidad de 5 a 20 gotas de preparado que contenga 10 000 U/mL, durante 6
u 8 semanas.
Leche, huevos,
mantequilla
E
Tocoferol
Impide la autooxidacin de los cidos
grasos insaturados. Impide deterioro de
las membranas celulares
Puede ocasionar anemia hemoltica (destruccin de los glbulos rojos de la
sangre), degeneracin muscular y desrdenes en la reproduccin.
o
K
Filoquinona
Intervienen en la sntesis de
protrombina (coagulacin)
Pueden producirse hemorragias nasales, en el aparato digestivo o el genito-
urinario.
Vegetales verdes,
tomates, aceites
vegetales
VITAMINAS HIDROSOUBLES
C
Ac.
ascrbico
Antioxidante, importante para la sntesis
de colgeno
ESCORBUTO; Se caracteriza por una debilidad muscular progresiva, dolores
en los brazos y en las piernas, adelgazamiento progresivo, palpitaciones,
aceleracin de pulsaciones cardiacas y sensacin de ahogo; son frecuentes las
hemorragias especialmente en las encas, que estn hinchadas y sangran con
facilidad y en la piel aparecen manchas sanguneas; los dientes caen
precozmente; es constante la anemia y diversas alteraciones seas que se
ponen en evidencia radiolgicamente. La resistencia a las infecciones est
muy disminuida.
Frutos ctricos,
verduras frescas
B1
Tiamina
Coenzima que interviene en las
reacciones de transferencia de grupos
aldehdo de dos carbonos
BERIBERI: Se caracteriza por trastornos nerviosos, circulatorios y generales
(astenia, debilidad general, marcha tambaleante, anemia). Si el dficit es
severo puede aparecer el sndrome de Korsakoff, caracterizado por la prdida
de memoria y confusin o la encefalopata de Wernicke, trastornos oculares,
confusin
Levaduras, germen y
salvado de arroz,
hgado, carnes
B2
Riboflavina
Constituyente de las coenzimas FMN y
FAD que intervienen en el metabolismo
energtico como transportadores de H+
y electrones
Lesiones en la piel boca y ojos Mismas que B1, leche,
queso, huevos
B6
Piridoxina
Coenzima que intervienen en las
desaminaciones. Relacionada con el
metabolismo de las protenas
El dficit importante provoca irritabilidad, debilidad, mareos, depresin,
neuropata perifrica y espasmos, alteraciones del crecimiento, acrodinia y
anemia.
Levaduras, verduras
frescas, leche, carne,
huevos
Niacina
Ac.nicotinico
Forma parte de las coenzimas NAD y
NADP que interfiere
PELAGRA: Las principales manifestaciones son dermatitis, inflamacin y dolor
en la boca y en la lengua, diarrea, marcha tambaleante, anemia.
Trigo integral, levadura
de cerveza, verduras,
MANUAL DE TRABAJO DEL CURSO ENARM CMN SIGLO XXI
CURSO ENARM CMN SIGLO XXI TEL: 36246001 Pharmed Solutions Institute PGINA 380

TRATAMIENTO: El tratamiento se basa en la identificacin de la vitamina o vitaminas deficitarias, para luego proceder a su reposicin
mediante suplementos vitamnicos y raramente, en sndromes de mala absorcin, mediante cargas inyectables.

CASO CLINICO
Se trata de un paciente masculino de 1 ao 3 meses al ingreso. Los antecedentes heredo familiares eran negativos. Como antecedentes
perinatales se document: Madre de 17 aos. Se trataba de suprimera gesta por cesrea por parto distsico con lquido amnitico
mecanizado. Se clasific al recin nacido de trmino adecuado para la edad gestacional. Su peso al nacer fue de 2720 g. Permetro
Ceflico 33 cm y APGAR 9-9. Sus inmunizaciones estaban completas. Segn la historia de la madre se document un retardo de
desarrollo psicomotor. Los mdicos tratantes consideraron que su estado de desnutricin poda ser una causa contribuyente. La
alimentacin se describi como deficiente, con mucha leche o atoles. Descrpcin del estado socio-econmico: Familia inmigrante,
matrimonio joven, con baja escolaridad y problemas econmicos importantes. La madre ama de casa y padre recolectar de caf sin
trabajo fijo.) Cambios frecuentes de vivienda. Motivo de ingreso: Enfermedad diarreica aguda de 8 das de evolucin, con 3 o 4
deposiciones por da, verde-amarillenta, sin moco, sin sangre. Vmitos desde 5 das prevos al ingreso de contenido alimentario. Al
ingreso se plante como problema falla para progresar con un peso de 8.3 kilogramos, talla de 73 centmetros y retardo del desarrollo
psicomotor. Se describieron datos cnicos de deshidratacin moderada. Se describe al paciente en este ingreso irritable, aptico,
hipoactivo, con lesiones de piel se describen como hiperpigmentadas en todo el cuerpo, mltiples, con distribucin de predominio en
cara, en forma de alas de mariposa y en las manos con distribucin de "guantes", las cuales se desprendan fcilmente con el roce,
hiperqueratosis, resequedad en la piel, fisuras generalizadas de predominio en labios y en codos, glositis, hiperplasia gingival, flictenas
en brazo izquierdo, bajo reborde costal y edema en mano y piel. Se describe tambin hgado a 4 centmetros bajo el reborde costal.

PREGUNTA
Considerando la sintomatologa cual es el dficit vitamnico mas probable?

RESPUESTA
a.- Vitamina A.
b.- Complejo B.
c.- Vitamina C.
d.- Vitamina D.

CASO CLINICO
Femenino de 6 aos de edad, que acude por resfriado comn y a la exploracin se observa dificultad en la marcha, con sensacin
dolorosa en el lado derecho de la cadera y refiere la madre que se ha cado espontneamente en alguna ocasin, se encuentra sin
vacunacin, desarrollo psicomotor dentro de limites.

PREGUNTA
Cual es su deficiencia vitamnica mas probable en este caso?

RESPUESTA
a.- Vitamina A.
b.- Complejo B.
c.- Vitamina C.
d.- Vitamina D.



hgado

Ac. flico Coenzima que interviene en el
metabolismo de los cidos nucleicos
Anemia
B12
cobalamina
Forma parte de una coenzima necesaria
en el metabolismo de las protenas y de
cidos nucleicos
Anemia perniciosa Carne, leche. Huevo,
pescado
MANUAL DE TRABAJO DEL CURSO ENARM CMN SIGLO XXI
CURSO ENARM CMN SIGLO XXI TEL: 36246001 Pharmed Solutions Institute PGINA 381

OBESIDAD. CIENCIAS BASICAS: La obesidad es una condicin caracterizada por el cmulo de grasa corporal, ms all de los lmites
esperados para la edad y el sexo de un sujeto dado. Ella constituye un problema de salud creciente y es prevenible. La principal
vinculacin de este estado con la salud, estriba en su asociacin con enfermedades que ocupan los primeros lugares dentro de los
cuadros de morbilidad y mortalidad de la poblacin, como son la cardiopata isqumica, la hipertensin arterial, la DM tipo II, las
enfermedades cerebrovasculares y algunos tipos de cncer. El sobrepeso que acompaa a la adiposidad excesiva puede producir
diversos trastornos como son los problemas ortopdicos y la reduccin de la capacidad de trabajo fsico. Todo esto, unido a
consideraciones estticas, produce, adems, trastornos psicolgicos que se derivan no solo de la distorsin de la autoestimacin, sino
de las relaciones sociales del individuo. Factores de riesgo: Padres obesos (hijos tienen riesgo de 80% de ser obesos), nios alto peso al
nacer tienen alto riesgo de sobrepeso en la infancia, nios pequeos para la edad gestacional tiene riesgo de desarrollar obesidad.
SALUD PUBLICA: Prevalencia nios 5 a 11 aos 46%, 2010 43 millones en nios menores de 5 aos tenan sobrepeso. Primer lugar
obesidad infantil mundial. En los adolescentes, la obesidad constituye la causa principal de hipertensin mantenida y se ha sealado
que aproximadamente el 25 % de los adolescentes hipertensos obesos pueden sufrir complicaciones cerebrovasculares o
cardiovasculares en los siguientes 7 aos. PATOGENIA: El depsito de grasa en exceso es multifactorial. Obesidad como una forma de
mala nutricin por exceso, bsicamente cuantitativa (con un balance energtico positivo). El mecanismo mediante el cual se produce el
exceso de deposicin de grasa en el cuerpo es el desbalance energtico, o sea un ingreso excesivo de energa en relacin con el gasto,
lo que se produce por 3 mecanismos diferentes: aumento en el ingreso; reduccin del gasto y afectacin de la regulacin del balance
energtico. En el obeso hay una mayor eficiencia trmica, lo que infiere que, es menor la cantidad de energa perdida en forma de calor
y mayor la acumulada en las reservas en forma de grasa. Tambin suele ser menor la energa utilizada en actividad fsica. La evidencia
de la herencia como factor condicionante o predisponente de la obesidad se deriva de estudios realizados en gemelos monocigticos y
dicigticos, de la comparacin entre hijos biolgicos y adoptados en una familia, y del estudio de patrones familiares de adiposidad.
Tambin la edad del rebote adiposo o sea del inicio del cmulo de grasa prepuberal preestablece la magnitud de la adiposidad en la
adolescencia y as los nios que muestran un rebote temprano corren un alto riesgo de convertirse en obesos si viven en un medio que
favorece el consumo de alimentos por encima de sus necesidades. La dieta es determinante en la regulacin energtica y de hecho
constituye el principal factor desencadenante del desequilibrio entre el ingreso y el gasto de energa. Las prcticas de alimentacin del
lactante han sido con frecuencia relacionadas con el desarrollo de la obesidad: la prevalencia de obesidad parece ser mayor en nios
que recibieron lactancia artificial y en aquellos con destete temprano o en los que se introdujeron slidos antes del tercer mes.
CLASIFICACION: Obesidad; GRADO I: IMC: 30-34.9 KG/m
2
, GRADO II: IMC 35- 39.9 KG/m
2
, GRADO III: 40 KG/m
2
. Sobrepeso; SOBREPESO
I: IMC 25- 26.9 KG/m
2
, SOBREPESO II: IMC 27-29.9 KG/m2. DIAGNOSTICO: En la etapa de lactante el incremento fisiolgico de la grasa
corporal puede ser excesivo, por lo que se produce un aumento en el tamao de las clulas adiposas, pero la probabilidad de que la
hiperadiposidad persista hasta la adultez es muy baja. Sin embargo, cuando esta aparece entre los 5 y l0 aos de edad se observa un
incremento gradual en la adiposidad y es frecuente que persista a lo largo de la adolescencia y se mantenga en la adultez,
caracterizndose por el incremento del nmero de adipocitos. El tejido celular subcutneo aumenta y es caracterstico la acumulacin
en la cara, la regin mamaria, el abdomen y la regin pubiana, en la cual el exceso de grasa puede disimular el tamao real del pene en
los varones, lo cual puede dar lugar a interpretaciones errneas de hipogonadismo. Recientemente se le confiere a la distribucin de la
grasa una particular importancia clnica. En aquellos sujetos en los cuales la grasa se acumula en el tronco (obesidad superior o
androide) se ha observado una mayor asociacin con la presencia de enfermedades crnicas degenerativas, lo cual tambin se observa
cuando se produce un incremento de la grasa visceral (obesidad central). Cuando la grasa tiende a acumularse en las extremidades,
particularmente en el tren inferior (inferior o ginecoide), la probabilidad de asociacin de la obesidad con dichas afecciones es mucho
menor. En la piel del abdomen de sujetos obesos moderados o severos suelen verse estras atrficas. Igualmente pueden observarse
lesiones de Acantosis nigricans, localizados sobre todo en la piel del cuello y las extremidades. En el sistema osteomioarticular pueden
verse genu valgo, coxa vara y deslizamientos epifisarios de la cabeza del fmur. En los obesos son frecuentes los trastornos
psicolgicos, en ocasiones como elementos causales y en otras, como resultado de la distorsin que sufre el sujeto de su imagen y de
los conflictos que determina su condicin dentro de su grupo. En una proporcin de escolares y adolescentes es posible encontrar
hipertensin arterial, por lo general, ligera a moderada, hipercolesterolemia, hipertrigliceridemia y elevacin de LDL y VLDL, as como
cada de las HDL. Tambin puede observarse intolerancia a la glucosa e incluso hiperglicemia, manifestaciones que suelen responder
favorablemente al tratamiento de la obesidad. La inspeccin de un sujeto es el mtodo ms simple y econmico para el diagnstico
cualitativo. Ha sido empleado, inclusive, como parte de un sistema de valores de referencia en lactantes y escolares. La medicin de la
masa corporal total y su comparacin con los valores de referencia expresados en porcentajes o en percentiles de peso para la edad,
han sido utilizadas como criterio diagnstico, a considerar obesos a aquellos que se hallan en el percentil 97 o ms. INDICE DE MASA
CORPORAL o ndice de Quetelet (que es el resultado de dividir el peso corporal (kg) por la talla (m
2
), segn la expresin; IMC = peso
(kg)/talla m
2
): entre 85-95 es indicativo de sobrepeso en nios IMC: encima de percentil 95 es indicativo de obesidad en nios. IMC: >
= 30 kg/m2 = obesidad adultos. Pero, al no tomar en cuenta la talla del sujeto, puede incluir como obesos a individuos normales de talla
elevada o excluir a individuos de baja talla. El mtodo ms difundido ha sido la determinacin del peso corporal y su relacin porcentual
con el peso ideal o esperada para la talla del sujeto. As, valores entre 110 y 120 % se denominan sobrepesos y valores, superiores a 120
% obesos. Este criterio ha sido fuertemente criticado, ya que considera el peso para la talla independiente de la edad, lo que introduce
un sesgo importante. Otro procedimiento de utilidad prctica es la medicin de los espesores de los pliegues cutneos en diversas
zonas del cuerpo con un comps destinado al efecto; las ms utilizadas son los pliegues tricipital, bicipital, subescapular y suprailaca.
Estos pliegues brindan informacin sobre la cantidad de grasa subcutnea, la que expresa indirectamente el estado de las reservas
energticas del organismo. Para medir la distribucin de la grasa el indicador ms prctico y til es el ndice cintura/cadera que es la
resultante de dividir la circunferencia de la cintura por la cadera. SINDROME DE CARPENTER Obesidad, retraso mental, puente nasal
plano, sindctilia, craneosinostosis, hipogonadismo. SINDROME DE PRADER WILLI Hipotona neonatal, baja estatura,
hipogonadismo, hiperfagia compulsiva, retraso mental obesidad, cromosoma 15. TRATAMIENTO: El objetivo, es lograr un cambio en
la composicin corporal con reduccin del peso en grasa y mantenerla despus dentro del rango adecuado para su edad y sexo. El
tratamiento no deber interferir con el incremento de masa magra, es decir, con el crecimiento. La teraputica est dirigida a modificar
el desbalance energtico para reducir el peso en grasa y a modificar el estilo de vida para garantizar que esta reduccin sea duradera.
Para ello, se sustenta en 3 pilares fundamentales: La dieta, el ejercicio, la induccin de cambios de conducta. Farmacolgico: limitado
MANUAL DE TRABAJO DEL CURSO ENARM CMN SIGLO XXI
CURSO ENARM CMN SIGLO XXI TEL: 36246001 Pharmed Solutions Institute PGINA 382

en nios. Las dietas muy bajas en caloras solo deben hacer especial manejo en obesidad y nunca ser la terapia indicada en nios.
Reducir azcares, agua. Frutas y cereales granos enteros Leche descremada, no comidas rpidas. Educar a las personas para lograr que
ellas distribuyan el consumo de energa diario de forma tal que el 20 % se consuma en el desayuno, el 20 % entre comidas y el 30 % en
almuerzo y comida, respectivamente. Solo al constituirse ese hbito con carcter familiar podr ejercerse influencia sobre los miembros
ms jvenes de la familia. Identificar los sujetos en riesgo y brindarles una atencin especial, tomando en cuenta, entre otros, los
factores siguientes: Obesidad en los padres, los hermanos y otros familiares cercanos. Obesidad en convivientes no familiares. Bajo
nivel educacional. Sobreproteccin: hijos nicos o valiosos, separacin de los padres o muerte de uno de ellos. Tratamiento con
antihistamnicos, anablicos o esteroides. Alto consumo de azcar y/o leche (ms de un litro diario). Poca actividad fsica. Promover la
lactancia materna extendindola por el mayor tiempo posible, y con carcter exclusivo hasta los 5 meses. No introducir ningn otro
alimento en la dieta del lactante hasta esa fecha. Cuando se inicie la ablactacin, no acostumbrar al beb al consumo excesivo de
alimentos dulces. Tratamiento farmacolgico metformina: obesos adolescentes no diabticos pero con comorbilidad, hiperinsulinemia
o resistencia a la insulina. Efectos secundarios: nuseas, vmito Ovarios poliqusticos. Tratamiento quirrgico: Ciruga gstrica y
ciruga baips: reduce la comorbilidad. ltima opcin la ciruga bariatrica.

CASO CLINICO
Nia de 10 aos de edad, acompaada por su madre para una visita de nio sano. El peso del paciente, la altura y los signos vitales,
fueron 42,1 kg, su altura es de 1,40 m, y su presin arterial es de 98/50 mmHg. IMC 21,6 kg/m2, que est en el percentil 91o para la
edad y el gnero.

PREGUNTA
Cul es el factores de riesgo ms importante que buscara para establecer un diagnostico adecuado.

RESPUESTA
a.- Familiares con sobrepeso y obesidad.
b.- Interrogar sobre tiempo de ver televisin.
c.- Actividad fsica.
d.- Peso al nacimiento.

PREGUNTA
Cual es la comorbilidad ms frecuente que el paciente desarrollar en caso de persistir el sobrepeso.

RESPUESTA
a.- Diabetes mellitus.
b.- Esteatosis heptica.
c.- Baja autoestima.
d.- Desplazamiento epifisiario.

PREGUNTA
Cul es el exmen no bsico para pesquizaje intensional de complicaciones.

RESPUESTA
a.- Perfil lipidico.
b.- Valores antropomtricos.
c.- Glucosa en ayunas e insulina.
d.- USG por poliquistosis ovrica.

PREGUNTA
Considerando el cuadro clnico cual es la medida mas adecuada.

RESPUESTA
a.- Sibutramina.
b.- Orlistat.
c.- Metfomina
d.- Dieta y estilo de vida.

PREGUNTA
Cual no es un criterio de referencia para segundo nivel inmediato para el tratamiento.

RESPUESTA
a.- Sobrepeso u obesidad con retrazo mental.
b.- Lesion hipotalmica por cualquier causa.
c.- Falla en el tratamiento instaurado.
d.- Falla en el crecimiento y desarrollo.



MANUAL DE TRABAJO DEL CURSO ENARM CMN SIGLO XXI
CURSO ENARM CMN SIGLO XXI TEL: 36246001 Pharmed Solutions Institute PGINA 383

CASO CLINICO
Un nio nacido de padres no relacionados, sanos despus de un embarazo a trmino y el parto normal (peso al nacer 3200 g, longitud
50 cm, 34,5 cm OFC). Fue hipotnica en los primeros meses de vida, pero no se observaron problemas de alimentacin en un principio.
Excesivo aumento de peso con aumento del apetito y el comportamiento de bsqueda de alimento se observ a los 30 meses de edad.
A los 5 aos, su peso era de 26 kg (> 3 SD), longitud 112 cm (1 SD) y OFC 48 cm (-2DE) a los 9 aos y su peso es de 72 kg (> + 6 SD),
longitud 144 cm (+2 SD) y OFC 52,5 cm (-0,5 SD). Distintivos rasgos faciales incluyen braquicefalia, cara plana, la frente alta,
hipertelorismo, nariz antevertidas, labio superior delgado, prognatismo, manos cortas, sindactilia de los dedos 2-3, y genitales
anormales (criptorquidia e hipospadias).

PREGUNTA
Cual es la conducta a seguir.

RESPUESTA
a.- Tratamiento farmacolgico.
b.- Enviar a segundo nivel.
c.- Enviar a tercer nivel.
d.- Tratamiento quirrgico.

















































MANUAL DE TRABAJO DEL CURSO ENARM CMN SIGLO XXI
CURSO ENARM CMN SIGLO XXI TEL: 36246001 Pharmed Solutions Institute PGINA 384

VACUNACION. CIENCIAS BASICAS: Las vacunas son sustancias
que nos ayudan a prevenir muchas enfermedades infecciosas,
las cuales son contagiosas y provocan discapacidad grave o
incluso la muerte. Cartilla Nacional de Vacunacin (0 a 19
Aos): Es el documento Oficial para dar seguimiento al
esquema de vacunacin a los nios menores de 5 aos,
escolares y adolescentes hasta los 19 aos de edad. Siempre
que se acude a vacunacin se debe de entregar para ver el
esquema o anotar la vacuna que se est aplicando en ese
momento. BCG: Vacuna contra la tuberculosis. Dosis nica de
0.1 ml, RN, hasta los primeros 5 aos, va intradrmica, en la
regin deltoidea del brazo derecho, sin prueba tuberculnica
previa, deja una cicatriz en el brazo posterior a su aplicacin.
Contraindicaciones: Fiebre > 38.5 C. RN peso inferior 2 kg,
dermatitis progresiva, pacientes con cuadro clnico de SIDA, Tx
Transfusiones, o inmunoglobulina, esperarn cuando menos
tres meses para ser vacunadas, no aplicar durante el
embarazo, no aplicar BCG con profilcticas de medicamentos antituberculosos. HEPATITIS B: Se aplican 3 dosis, 10 mcg en 0.5 ml. IM
profunda, en la cara anterolateral externa del muslo izquierdo en los menores de 18 meses de edad, a partir de los 18 meses de edad,
en la regin deltoidea del brazo derecho , la primera al nacer y a los 2 y 6 meses de edad. Previene este tipo de hepatitis, que afecta
principalmente al hgado. Contraindicaciones: Fiebre > 38.5 C. RN peso inferior 2 kg, enfermedad grave, Tx Transfusiones, o
inmunoglobulina, esperarn cuando menos tres meses para ser vacunadas, hipersensibilidad a cualquier componente. PENTAVALENTE:
Es la vacuna que previene la difteria, tos ferina y ttanos, adems tambin a la poliomielitis y a las bacterias del Haemophilus Influenzae
del tipo b, que provocan neumonas y meningitis. Se aplica en 4 dosis a los 2, 4, 6, y 18 meses de edad. Se aplica por va IM profunda,
las primeras tres dosis deben aplicarse en el tercio medio de la cara anterolateral externa del muslo derecho. A los 18 meses se debe
aplicar en la regin deltoidea del brazo izquierdo. ROTAVIRUS: Previene la gastroenteritis o la diarrea causada por el rotavirus en sus
formas graves. Se aplican 3 dosis 0.1 ml sobre la parte interna de las mejillas, a los 2, 4 y 6 meses de edad y nunca despus de los 8
meses de edad. Contraindicaciones: Hipersensibilidad despus de la administracin de la vacuna, antecedente de enfermedad
gastrointestinal crnica, malformacin congnita no corregida en el tracto gastrointestinal. SABIN: Ayuda a prevenir la poliomielitis. Se
aplican 2 gotitas en los menores de 5 aos a partir de los 6 meses como dosis adicional (de 0.1 ml) en cada Semana Nacional de Salud,
siempre y cuando hayan recibido dos dosis previas de vacuna de poliovirus inactivada, uso de esta vacuna en actividades de control de
casos y brotes. Contraindicaciones: Infeccin por VIH, tener inmunodeficiencia congnita, tener hemopata o tumor slido o terapia
inmnunosupresora. NEUMOCCICA: Contra el neumococo. Se aplican 3 dosis, 0.5 ml, a los 2, 4 y 12 meses de edad. Va IM en el
tercio medio de la cara anterolateral externa del muslo derecho. Contraindicaciones: Procesos febriles de ms de 38.5 C, con
enfermedad grave, reaccin alrgica severa (anafilctica) hacia un componente de la vacuna incluyendo toxoide diftrico, antecedentes
de Sndrome de Guillain-Barr, antecedentes de hipersensibilidad al ltex. No revacunar antes del tiempo establecido, ya que puede
provocar reacciones locales severas. Aplicar un refuerzo antes del tiempo establecido puede provocar el desarrollo de neumona
neumocccica posvacunal severa. ANTI-INFLUENZA: Previene el
virus de la influenza y se aplica a nios de 6 a 35 meses de edad
desde octubre a febrero, 0. 25 ml IM basto lateral izquierdo. En la
primera ocasin se aplican dos dosis con intervalo de 1 mes y
despus cada ao. Contraindicaciones: No aplicar a personas con
alergia a las protenas del huevo o a algn componente de la
vacuna, fiebre, haberse aplicado el biolgico en menos de un ao,
en lactantes menores de 6 meses, en personas que hayan tenido
una reaccIn de hipersensibilidad, incluyendo una reaccin
anafilctica al huevo o a una dosis previa de vacuna contra
influenza, ante la presencia de fiebre mayor o igual a 38.5C,
pacientes con antecedente de sndrome de Guillain Barr.
VACUNA TRIPLE VIRAL (SRP): Previene el sarampin, la rubeola y
las paperas. Se aplica 2 dosis de 0.5 ml de vacuna reconstituida:
12 meses de edad y los 6 aos de edad va SC en el rea superior
externa del trceps del brazo Izquierdo, inscritos o no en primer
ao de primaria. Contraindicaciones: Fiebre > 38.5, alrgicos al
huevo, neomicina, antecedente de aplicacin de inmunoglobulina
o transfusiones 3 meses previos a la vacunacin, neoplasias,
inmunodeficiencias, tratamiento con corticoesteroides,
citotxicos, embarazo. Poco frecuentes, pueden presentarse: Parotiditis uni o bilateral, despus de doce das y dura menos de cuatro
das, meningitis asptica, entre la segunda y la cuarta semanas posteriores a la vacunacin, prpura trombocitopnica. DPT: Sirve como
un refuerzo que previne a la difteria, tos ferina y ttanos. Se aplica dosis de 0.5ml a los 4 aos, se aplica por va IM profunda en la
regin deltoidea del brazo izquierdo. Contraindicaciones: No mayores a 5 aos, reaccin anafilctica a una aplicacin previa,
antecedentes de crisis convulsivas o alteraciones neurolgicas, fiebre > 38. 5C. Toxoide Tetnico Diftrico (TD): Se aplica a las
personas desde los doce aos hasta los adultos mayores, hombres y mujeres, especialmente a las embarazadas, dosis de 0.5ml.
Previenen el ttanos en los recin nacidos y en los adultos. Las personas no vacunadas o con esquema incompleto de vacuna
pentavalente o DPT recibirn al menos dos dosis con intervalo de 4 a 8 semanas entre cada una, y revacunacin cada 10 aos.
Contraindicaciones: No suministrar a personas con hipersensibilidad a algn componente de la vacuna, a personas con
MANUAL DE TRABAJO DEL CURSO ENARM CMN SIGLO XXI
CURSO ENARM CMN SIGLO XXI TEL: 36246001 Pharmed Solutions Institute PGINA 385

inmunodeficiencias, a excepcin de la infeccin por el VIH asintomtica, fiebre superiores a 38.5C y enfermedades graves, historia de
reaccin grave de hipersensibilidad o eventos neurolgicos relacionados con la aplicacin de una dosis previa, las personas
transfundidas o que han recibido inmunoglobulina, debern esperar tres meses para ser vacunadas, salvo en aquellos casos de
traumatismos con heridas expuestas, ya que puede aplicarse, simultneamente con antitoxina, independientemente de transfusin o
aplicacin de inmunoglobulina. EVENTOS TEMPORALMENTE ASOCIADOS A LA VACUNACIN ETAVS: Se define como eventos
temporalmente asociados a la vacunacin a todas aquellas manifestaciones clnicas que se presentan dentro de los 30 das posteriores
a la administracin de una o ms vacunas y que no pueden ser atribuidos inicialmente a alguna entidad nosolgica especifica (para la
vacuna Sabin el periodo puede ser hasta de 75 das y para la vacuna BCG hasta seis meses un ao). Falsa contraindicacin para todas
las vacunas: Reaccin local de leve a moderada, enfermedad leve con o sin fiebre, tratamiento antimicrobiano actual, fase de
convalecencia de alguna enfermedad, exposicin reciente a enfermedades infecciosas, historia de alergia a penicilina u otras alergias
inespecficas o el hecho de que sus familiares tengan dichas alergias, alergia a protenas del huevo, a neomicina manifestada slo por
rash.

CASO CLINICO
Un nio de 24 meses de edad, naci a trmino, cuenta con el antecedente que despus de haber recibido la vacuna antipoliomieltica,
fiebre (38,5 C y 40,0 C), al da siguiente presento vmitos y convulsiones tnico-clnicas. El lquido cefalorraqudeo mostr
pleocitosis y aumento moderado de protenas. El LCR fue positivo por PCR para enterovirus y una pleocitosis. Las muestras de heces
tomadas los das 5 y 9 fueron positivos para enterovirus, que se caracteriz posteriormente como poliovirus tipo 3.

PREGUNTA
Cual es la conducta a seguir.

RESPUESTA
a.- Aplicar SRP
b.- Vacuna pentavalente acelular.
c.- Evitar DPT a los 4 aos.
d.- Revacunacin anual.

DERMATITIS HERPETIFORME (DH). CIENCIAS BSICAS: Es una enfermedad ppulovesicular, muy pruriginosa, crnica, de distribucin
caracterstica. PATOGENIA: Se ha especulado sobre el papel de los anticuerpos del gluten en la etiopatognesis de esta enfermedad. El
gluten es una fraccin proteica remanente de la harina despus que ha sido lavada y eliminado el almidn. Las principales fuentes de
gluten en occidente son: trigo, centeno, cebada; el maz y la avena no parecen contener cantidades importantes. El 75% de los casos de
DH tienen aplanamiento de las vellosidades intestinales y 20% presenta infiltrado linfocitario en el epitelio como evidencia de
enteropata sensible al gluten (ESG). Es bien establecido, que las manifestaciones cutneas e intestinales mejoran con la dieta estricta,
libre de gluten, sin embargo, el mecanismo fisiopatolgico se desconoce. La asociacin inmunogentica apoya la naturaleza
autoinmune de la DH a otras enfermedades autoinmunes. Aunque no se ha descrito todava el antgeno diana, hay asociacin de la DH
con otras enfermedades autoinmunes, como lupus eritematoso, enfermedades tiroideas y dibetes mellitus. DIAGNOSTICO: La DH se
ha reportado raramente en nios, pero la edad de presentacin es de 2 a 7 aos. Las lesiones cutneas caractersticas son ppulas y
placas eritematosas, urticarianas, edematosas sobre las cuales hay vesculas, distribuidas simtricamente en superficie extensora de las
extremidades. Los sitios caractersticos son: codos, rodillas, glteos, nuca y cuero cabelludo. Una sensacin de ardor o escozor suele
preceder las lesiones. El prurito es intenso haciendo que el paciente est inquieto. Hay lesiones secundarias al rascado como
excoriaciones, costras y mculas hiper e hipopigmentadas residuales. Las membranas mucosas no estn comprometidas. Sin embargo,
se han descrito defectos en el esmalte dental similar a lo observado en la enfermedad celaca, cambio de coloracin y textura de la
superficie dentaria. Los sntomas gastrointestinales pueden ser muy leves o estar ausentes, aunque por lo menos un tercio de los
pacientes sufre de esteatorrea y anemia. La patologa es caracterstica. Se describen microabscesos en la punta de las papilas drmicas
y formacin de ampolla subepidrmica. En la dermis superior y media se observa un infiltrado de linfocitos, neutrfilos e histiocitos
alrededor de los vasos sanguneos. Los eosinfilos son escasos. El diagnstico se basa en criterios clnicos, histolgicos, as como la
respuesta favorable a las sulfonas y a la dieta libre de gluten. TRATAMIENTO: El tratamiento de eleccin, como en los adultos, es la
dieta libre de gluten. Se describe una respuesta mucho ms rpida en este grupo con control de las lesiones cutneas a los 11 meses,
aproximadamente. La dapsona a la dosis de 1.5-2 mg/kg/da, es efectiva para el control de las lesiones cutneas y del prurito intenso.
La suspensin de sta, produce la reaparicin rpida de las lesiones, por lo que el tratamiento debe ser combinado con la dieta libre de
gluten y disminucin progresiva y suspensin de la dapsona al lograr control, con lo que disminuye la aparicin de los efectos
secundarios de la droga: metahemoglobinemia, anemia hemoltica, agranulocitosis, y complicaciones neurolgicas. Antes de su uso, los
niveles de glucosa-6-fosfato deshidrogenasa deben ser determinados. En esos casos, en los que no se pueda utilizar la dapsona, se
recomienda la sulfapiridina a 250 mg/da con ajuste de acuerdo a respuesta teraputica. El curso de la DH en nios no est bien
definido, la ESG persiste toda la vida, aunque con perodos de remisiones espontneas en algunos casos. Sin embargo, hay que informar
a la familia sobre la naturaleza de la enfermedad y su duracin, as como, la importancia de la dieta estricta libre de gluten de por vida.

CASO CLINICO
Paciente varn de 13 aos de edad que consulta por lesiones muy pruriginosas, que se localizan fundamentalmente en codos, rodillas y
regin lumbosacra. Se ha tratado con diversas pomadas de corticoides tpicos sin mejora. En la exploracin cutnea se observa en los
codos, rodillas y regin lumbosacra la presencia de ppulas eritematosas, excoriadas y alguna pequea vescula intacta. Se realiz
biopsia cutnea de una de las lesiones y el estudio histopatolgico demostr: microabscesos de neutrfilos a nivel de papilas drmicas,
El estudio de inmunofluorescencia directa demostr depsito de Ig A granular en las papilas dermicas.


MANUAL DE TRABAJO DEL CURSO ENARM CMN SIGLO XXI
CURSO ENARM CMN SIGLO XXI TEL: 36246001 Pharmed Solutions Institute PGINA 386

PREGUNTA
Considerando la patologa que presenta el caso cual de los siguientes diagnosticos diferenciales es el menos probable?

RESPUESTA
a.- Dermatitis alrgica de contacto
b.- Dermatitis atpica
c.- Sarna
d.- Penfigoide ampolloso

PENFIGO. CIENCIAS BSICAS: Es una enfermedad ampollar autoinmune, del epitelio estratificado, que normalmente afecta a individuos
de edad media y mayores, pero puede ocurrir raras veces en nios, relacionado con factores genticos y ambientales secundario a la
produccin de IgG. Las dos variantes del pnfigo son el vulgar (PV) y foliceo (PF), que difieren en su presentacin clnica, histolgica y
en los antgenos patognicos. El pnfigo vegetante y eritematoso son variantes clnicas ms raras. En Brazil existe la forma endmica
del pnfigo foliceo, denominada fogo selvagem. Ms recientemente se describi el pnfigo paraneoplsico y el pnfigo IgA. SALUD
PUBLICA: Es una enfermedad que afecta todas las razas sin distincin de sexo. Sin embargo, se reporta una mayor incidencia en judos
ashkenazi y menos en la raza negra. La edad de presentacin en nios va desde 3 hasta 17 aos y adultos pico de 50-60 aos. En 1955
se public el primer caso de pnfigo vulgar en un nio. Mortalidad 17.7% (choque sptico, embolia pulmonar). CLASIFICACION: Pnfigo
vulgar (80%), pnfigo violceo, pnfigo inducido por frmacos, pnfigo paraneoplasico, pnfigo IgA. PATOGENIA: El pnfigo es una
enfermedad autoinmune que afecta la piel y las mucosas, de etiologa desconocida. Existe un aumento en la expresin de antgenos del
complejo mayor de histocompatibilidad HLA-A, DR4, Dw10 y Dw6. Los pacientes tienen autoanticuerpos que reaccionan con los
desmosomas, molculas de adhesin localizadas en la superficie de los queratinocitos. Los autoanticuerpos detectados son de la clase
IgG. En los ltimos aos se ha logrado un avance en la determinacin de los antgenos responsables en todas las formas de pnfigo. De
la unin de este autoanticuerpo con las desmoglena antignica se produce la acantolisis, con prdida de la cohesin de las clulas y la
formacin de las ampollas. DIAGNOSTICO: Clnicamente, suele comenzar en la mucosa bucal con erosiones del epitelio. Las lesiones
cutneas consisten en vesculas, ampollas flccidas que se rompen con facilidad resultando en extensas erosiones. Se localiza
predominantemente en cara, cuello, tronco, extremidades y reas intertriginosas. Las erosiones causan dolor y ardor y
secundariamente pueden infectarse con bacterias u hongos. El signo de Nikolsky es positivo (eritema y formacin de ampollas como
resultado de friccin o presin sobre la piel), signo de Asboe-Hansen (aumento perifrico del tamao de la ampolla al presionar
verticalmente su superficie), no es especifica. Se ha reportado onicolisis y distrofia ungueal. En el PF infantil es endmico en Brasil, en la
forma de fogo selvagem, pero su incidencia en otros pases es menor a la del PV. Clnicamente se presentan erosiones superficiales con
descamacin y costras, aisladas o generalizadas (eritrodermia). En ocasiones se inicia en cuero cabelludo o reas seborreicas, luego se
extienden al tronco y extremidades. No se aprecian lesiones orales. Se deben escoger las vesculas intactas para el examen histolgico.
En el PV se evidencia la ampolla intraepidrmica suprabasal, con acantolisis (desaparicin de puentes intracelulares) marcada. El piso
de la cavidad est constituido por queratinocitos basales y leve infiltrado inflamatorio, compuesto por eosinfilos. En lesiones
tempranas los eosinfilos invaden la epidermis en racimo. Este fenmeno se denomina espongiosis eosinoflica. La ampolla en el PF se
localiza en la subcorneal de la epidermis. La acantolisis est presente inmediatamente por encima o por debajo de las clulas de la capa
granulosa. Este patrn se ha denominado panal de abejas y constituye la herramienta diagnstica en estas enfermedades, cuya
positividad alcanza hasta el 100%. El diagnstico se basa en la combinacin de criterios clnicos, histolgicos. Si hay duda observar la
celularidad, clulas Tzank, tiene 100% sensibilidad y 43% especificidad. Se observan clulas acantoliticas (zona blanca). TRATAMIENTO:
Las lesiones se alivian en 1-2 aos y dejan hiperpigmentacion psinflamatoria. En manejo de los pacientes peditricos en PV es similar
al adulto. Los casos con enfermedad localizada pueden ser tratados con corticoesteroides tpicos. Sin embargo, la mayora de los casos
desarrollan formas ms severas que ameritan el uso de corticoesteroides sistmicos a altas dosis (2 a 3 mg/kg/da). Con la finalidad de
disminuir los efectos adversos de stas, se usan drogas adyuvantes como: azathioprina, ciclofosfamida, dapsona y metotrexato, sales de
oro.

CASO CLINICO
Un nio de 12 aos de edad se present con lesiones generalizadas en la piel eritematosas y costras de 4 meses de duracin. Las
lesiones aparecieron por primera vez en su cuero cabelludo y dentro de unos das, eritrodrmica se generaliz y, a continuacin.
Escalado y exudacin tambin fueron vistos. Diferentes antibiticos y esteroides tpicos fueron prescritos sin mejora significativa. El
nio tambin se quejo de prdida de la audicin desde un mes. En el examen fsico, eritrodermia con escalamiento grave y maloliente
con descarga y leve queratodermia palmoplantar todo el cuero cabelludo. Haba dos pequeas vesculas a lo largo del lado cubital de la
palma derecha. Las superficies de las mucosas y las uas eran normales. Tena ganglios linfticos submandibulares. El conducto auditivo
estaba lleno de escamas y costras. pabelln de la oreja era sensible a la palpacin. Despus de quitar las costras, canal auditivo externo
se encontr que era roja e hinchada

PREGUNTA
Cual es la conducta a seguir.

RESPUESTA
a.- Prednisona oral de 0.5 a 1 mg/kg al da.
b.- Dapsona dosis de 100 mg/da.
c.- Azatioprina es de 2.0 a 2.5 mg/kg,
d.- Ciclofosfamida a dosis oral de 1 a 3 mg/kg al da



MANUAL DE TRABAJO DEL CURSO ENARM CMN SIGLO XXI
CURSO ENARM CMN SIGLO XXI TEL: 36246001 Pharmed Solutions Institute PGINA 387

HIPERQUERATOSIS EPIDERMOLITICA. CIENCIAS BASICAS: Tambin llamada Eritrodermia ictiosiforme congnita ampollosa, es una
genodermatosis infrecuente, con patrn de herencia autosmico dominante, aunque 50% de los pacientes experimentan mutaciones
de novo. Se distingue por eritrodermia y ampollas secundarias a traumatismos mnimos, que se manifiestan desde el nacimiento y que
evolucionan a ictiosis generalizada a lo largo de la vida, predominantemente en zonas de flexion. SALUD PUBLICA: No hay predileccin
por sexo o raza. Se manifiesta desde el nacimiento en un caso por cada 200,000 a 300,000 nacidos vivos. PATOGENIA: La causa son los
defectos en las citoqueratinas que forman los filamentos intermedios. Las citoqueratinas afectadas son la K1 y la K10, las cuales
predominan en los estratos suprabasales de la epidermis. Cuando las citoqueratinas experimentan mutaciones puntuales, se asocian
con la hiperqueratosis epidermoltica y la eritrodermia ictiosiforme congnita ampollosa, y generan clulas suprabasales que se
fragmentan fcilmente, con epidermis hiperproliferativa e hiperqueratsica. Forman parte del tbulo de las glndulas sudorparas
ecrinas y son marcadores de queratinizacin por
excelencia. La hiperqueratosis y la ictiosis
subsecuentes que se observan en estos pacientes se
deben a un mecanismo de compensacin para
proteger contra la fragilidad cutnea, y as disminuir
la probabilidad de formacin de ampollas.
CLASIFICACION: En 1994 DiGiovanna distingui
clnicamente los dos tipos de hiperqueratosis
epidermoltica con base en la presencia o ausencia
de queratodermia palmoplantar. DIAGNOSTICO: Se
basa en manifestaciones clnicas y confirmacin histolgica. Aparece desde el nacimiento e inicia como eritrodermia generalizada, con
fragilidad cutnea que causa descamacin, as como ampollas, incluso con traumatismos menores, en la mayora de los casos. Las
ampollas son superficiales y grandes, y al romperse ocasionan grandes zonas de exulceracin. En los pliegues de flexin aparecen
ulceraciones superficiales y zonas extensas de piel denudada. Debido a la interrupcin de la barrera epidrmica, en estos nios existe
un mayor riesgo de infecciones severas, desequilibrio hidroelectroltico y sepsis. Hay disminucin gradual de la eritrodermia y de las
ampollas, que a los pocos meses evolucionan a placas hiperqueratsicas, de aspecto verrugoso, en las zonas de flexin, la piel cabelluda
y los glteos. La hiperqueratosis inicia a los tres meses de edad, con engrosamiento poco evidente de la piel o con escamas finas y
pequeas. Con la edad algunas de las escamas se van haciendo ms gruesas (adquieren aspecto de cartn corrugado) y la
probabilidad de manifestar ampollas disminuye, aunque incluso 20% de los pacientes sigue teniendo ampollas hasta la vida adulta. La
hiperqueratosis se agrava con el sol y el calor. La afeccin de las palmas y las plantas ocurre en 60% de los pacientes y se manifiesta
con grados variables de hiperqueratosis, asociada, sobre todo, con mutaciones de CK1. La queratodermia palmoplantar puede originar
fisuras y contracturas recurrentes y dolorosas, que pueden ocasionar incapacidad funcional. La biopsia de piel, confirma el diagnostico
si observamos epidermis con hiperqueratosis ortoqueratsica, acantosis irregular moderada con reas de degeneracin de
queratinocitos, espinosos, dermis con papilomatosis y focos mnimos de linfocitos perivasculares. La inmunohistoqumica muestra
anormalidades de expresin de la citoqueratina 1 o 10. TRATAMIENTO: La piedra angular del tratamiento son los emolientes tpicos.
Existen reportes de mejora clnica con dosis altas de betacarotenos, retinoides sistmicos o tpicos, glicerina a 10%, cido lctico, alfa-
hidroxicidos, calcipotriol y urea; sin embargo, no existe un consenso sobre la administracin de estos frmacos. Los agentes
queratolticos son poco tolerados por estos pacientes debido a que las escamas se desprenden en bloques extensos. El
sobrecrecimiento bacteriano es comn, sobre todo por Staphylococcus aureus, secundario a la acumulacin y maceracin de la escama,
por lo que pueden usarse jabones antibacterianos suaves o limpiadores con clorhexidina. PRONOSTICO: Es bueno para la vida, sin
embargo los pacientes pueden padecer alteraciones funcionales, secundarias a queratodermia palmoplantar y contracturas en flexin
de las manos.

CASO CLINICO
Lactante mujer, de un ao de edad, sin antecedentes patolgicos de inters, que consulta por aparicin progresiva de lesiones cutneas
a nivel palmoplantar. El cuadro se inici a la semana del nacimiento con discreto eritema en dicha localizacin que evolucion
progresivamente hacia placas hiperqueratsicas amarillentas, simtricas y confluentes. En el momento de la consulta presentaba una
hiperqueratosis difusa palmoplantar bien delimitada por un borde eritematoso en los mrgenes laterales de las manos y los pies, sin
extensin a la superficie dorsal de estos.

PREGUNTA
Cual es la conducta a seguir.

RESPUESTA
a.- Envio a segundo nivel.
b.- Tratamiento farmacolgico.
c.- Tratamiento quirrgico.
d.- Terapia fotodinmica.

PREGUNTA
Cual de los antecedentes personales patolgicos incrementa ms esta patologia.

RESPUESTA
a.- Xeroderma pigmentoso.
b.- Albinismo.
c.- Inmunosupresion.
d.- Sindrome de Bloom.
MANUAL DE TRABAJO DEL CURSO ENARM CMN SIGLO XXI
CURSO ENARM CMN SIGLO XXI TEL: 36246001 Pharmed Solutions Institute PGINA 388

FOLICULITIS. CIENCIAS BASICAS: Foliculitis Bacterianas: Se definen como el conjunto de infecciones originadas por bacterias que
afectan al folculo piloso con una expresin clnica que vara de acuerdo a la profundidad y extensin. Foliculitis: Se definen infecciones
superficiales localizadas a nivel del ostium folicular, acompaada en ocasiones de una reaccin inflamatoria perifolicular. PATOGENIA:
El agente causal ms frecuente es Staph. aureus, pero tambin pueden infectar bacterias Gram negativas S. Aureus coagulasa
negativo. Los folculos pilosos estn en toda la superficie de la piel excepto en palmas y plantas, pero son abundantes sobre todo en piel
cabelluda, zonas de varaba y bogote, axilas, pubis. Son estos los sitios ms afectados por la foliculitis. DIAGNSTICO: Es clnico y de ser
necesario se realizar gram y cultivo. Se presentan en cualquier zona pilosa, sobre todo en ambiente hmedo, maceracin, falta de
higiene, aplicacin del emoliente oclusiva y drenaje de heridas adyacentes y abscesos, uso de agentes oclusivos o presencia de
infecciones adyacentes, y ms frecuentemente en pacientes con inmunosupresin local sistmica. Se presentan clnicamente como
una pstula de base eritematosa, apenas de unos milmetros, a nivel de cada folculo, en ocasiones se dejan atravesar un pelo, son de
color amarillento se abren rpidamente y dejan salir una gotita de pus que al secarse formara una costra melicerica. En general no es
un proceso pruriginoso, ni doloroso.Las lesiones se extioenden rpidamente en superficie y mientras unas van curando, otras nuevas
aparecen. Al desaparecer no dejan cicatriz. Se localizan con mayor frecuencia en zonas de pelo terminal; si la infeccin se extiende en
profundidad formar Abscesos. A las formas ms superficiales tambin se les denomina Imptigo de Bockhart. TRATAMIENTO: En
primer lugar eliminar factores desencadenates, antispticos n(fomentos de sulfato de cobre y aplicacin de toques con solucin yodada
al 1%), antibiticos tpicos y sistmico (para evitar recidiva dicloxacilina, TMP/SFX, por 3-4 semanas) sabiendo que el agente causal es
Staph. Aureus.

CASO CLINICO
Nia de 8 aos de edad. Consult por aparicin en los 3 das previos de un brote de lesiones papuloeritematosas, algunas de aspecto
seudovesiculoso o pustuloso, pruriginosas, en el tronco (de predominio en caderas y glteos) y las extremidades inferiores. Con la
sospecha inicial de exantema periflexural asimtrico de la infancia frente a picaduras, se inici tratamiento con antihistamnico por va
oral y metilprednisolona por va tpica, con mejora aparente de las lesiones. Seis das ms tarde consult por aparicin progresiva de
nuevas lesiones de similares caractersticas. En la exploracin fsica present mculas hiperpigmentadas marrones en la zona abdominal
inferior y en la regin anterior de ambos muslos, junto con mltiples ppulas eritematoedematosas, algunas con centro pustuloso, en la
zona lumbar y en ambos glteos y otras con aspecto vesiculoso en costado izquierdo. La nia negaba ir a piscinas, o hacer cualquier tipo
de actividad recreativa relacionada con agua.

PREGUNTA
Cual es el agente causal mas frecuente.

RESPUESTA
a.- Staphylococcus aureus.
b.- Pseudomona aeruginosa.
c.- Estreptococos
d.- Proteus.

IMPETIGO. CIENCIAS BASICAS: Es la infeccin bacteriana superficial y contagiosa ms frecuente en los nios, producida por Estafilococo
Aureus, Estreptococo B hemoltico del grupo A (pyogens) o ambos; puede ser primario en caso de invadir piel sana secundario cuando
existe dao de la barrera cutnea previamente, como ocurre en la dermatitis atpica, escabiosis. El imptigo contagioso es la ms
comn. Estas infecciones pueden dividirse en: 1. infeccin cutnea primaria 2. Infeccin secundaria de una enfermedad cutnea
primaria y 3. lesiones cutneas como manifestacin de infeccin primaria en otro rgano, ejemplo de este ltimo grupo tenemos:
S.aureus causando sndrome de shock toxico y sndrome de piel escaldada estafiloccico, S. pyogenes escarlatina, N. meningitides,
sepsis por meningococo (petequias), S. thiphy, fiebre tifoidea (rosola tifoidea), Rickettsia conorii, Tifus africano (rahs macular)
IMPTIGO CONTAGIOSO: Costroso no ampollar, infeccin muy frecuente representa el 70% de los casos; predomina en nios de
todas la edades , aunque tambin se puede ver en adultos; el contagio se origina sobre todo por rascado. La infeccin puede ser
originada por Strep. Pyogenes (15%), Staph. Aureus (70 a 80 %) o ambos (15 a 100%). La cara es la zona de ms frecuente de
localizacin, sobre todo en las uniones mucocutneas, por ejemplo: introito nasal, boca y en todas las zonas periorificiales, manos, etc,
pero como es contagioso, se puede ver en cualquier zona corporal. La lesin inicial son ppulas eritematosas pequeas que
rpidamente evolucionan a una vescula que se rompen rpidamente dando paso a erosiones cubiertas por costras melicricas y lesin
residual hipocrmica temporal (lesiones primitivas, ampollas, pstulas que al romperse dejan salir exudado que se seca, costra
melicerica), sin dejar cicatriz; si hay dolor es leve y puede presentar linfadenopata regional, pero no hay sntomas sistmicos. Las
lesiones suelen propagarse por autoinoculacin originando lesiones satlites. La resolucin es espontnea entre la segunda y tercera
semana de evolucin, pero debe ser tratada con prontitud y as se acorta el perodo de enfermedad y el contagio, as como las
eventuales complicaciones glomerulonefritis post estreptoccica, escarlatina psoriasis. Diagnstico: se basa en la clnica pero se
puede confirmar con un gram y cultivo. IMPTIGO AMPOLLAR: Representa el 30% restante, es observa ms frecuentemente en
neonatos y lactantes. Es ocasionado por la toxina exfoliativa tipo A del Staph. Aureus; esta toxina acta como una serina proteasa de
desmoglena 1, ocasionando de esta manera la acantlisis subcorneal que conduce a la formacin de ampollas. Las lesiones se pueden
localizar en cualquier rea de la superficie corporal, pero lo ms frecuente es alrededor de los orificios de la boca, nariz, rea genital y
en las extremidades Clnica:pueden presentar fiebre hasta de 40C, ampollas flcida, que se rompen con facilidad en el curso de 1 a 2
das, dejando erosiones cubiertas por una costra perifrica fina, de ah que tambin se conozca como imptigo circinado; las lesiones
aparecen usualmente agrupadas. Existe una tendencia a la curacin espontnea sin cicatriz, pero pueden dejar hipopigmentacin
residual temporal. TRATAMIENTO: Es igual en ambos tipos clnicos: 1) Realizar Antisepsia, por ej con Clohexidine, sulfato de cobre l
1x1000 con eliminacin de las costras., Mejorar la higiene persona y recortar las uas. 2) Terapia Tpica, con antibacterianos
preferiblemente de origen sinttico, con espectro sobre los agentes causales ms frecuentes: por ej mupirocina, cido fusdico o ms
recientemente retapamulina. 3) Si hay ms de 2 reas corporales afectadas , presencia de muchas lesiones, enfermedad sitmica
MANUAL DE TRABAJO DEL CURSO ENARM CMN SIGLO XXI
CURSO ENARM CMN SIGLO XXI TEL: 36246001 Pharmed Solutions Institute PGINA 389

subyacente, como la Dermatitis Atpica, Inmunosupresin varios familiares afectados , se debe agregar terapia sistmica, por ej:
amoxilinia con Ac Clavulanico , dicloxacilina, cefalexina, cefradina , macrlidos u otros si hay resistencia bacteriana. Si no hay una buena
respuesta clnica en forma rpida se debera cultivar y realizar antibiograma

CASO CLINICO
Se trata de escolar masculino de seis aos de edad quien es trado por su madre por presentar lesiones ampollosas en las manos,
pecho, espalda y brazos de color amarillento con ardor e hipertermia de 39c;.

PREGUNTA
Cual es la conducta a seguir inmediata mas adecuada?

RESPUESTA
a.- Iniciar antibitico.
b.- Administrar antipirtico.
c.- Hidratacin adecuada.
d.- Laboratorio de rutina.

PREGUNTA
Considerando la exploracin del caso cual es la conducta teraputica para la etiolgica mas adecuada?

RESPUESTA
a.- Mupirocina.
b.- Acido fusidico.
c.- Penicilina.
d.- Amoxicilina.

ERISIPELA. CIENCIAS BASICAS: Es una Infeccin bacteriana profunda dermoepidrmica pero ms superficial que la Celulitis, localizada
en dermis profunda y zona superior del tejido celular subcutneo con afectacin linftica significativa. Casi siempre muy bien
delimitada y con tendencia a progresar hacia la periferia. Es una enfermedad potencialmente transmisible y reviste mayor gravedad en
los nios. PATOGENIA: Los agentes causales ms comunes son: Streptococccus pyogenes o Streptococcus beta-hemoltico del grupo A,
con menor frecuencia por Streptococcus de los grupos B, C, D o G, sobre todo en pacientes postquirrgicos con otras enfermedades.
Los del grupo B suelen ser responsables de la erisipela del recin nacido. Otros grmenes que pueden ocasionar erisipela: son
Staphylococcus aureus, Pneumococcus, Klebsiella pneumoniae, Yersinia enterocoltica y Haemophilus influenzae tipo b. La lesin
comienza a travs de una pequea solucin de continuidad de la piel o de las mucosas, que constituye la puerta de entrada y que en
muchas ocasiones pasa inadvertida, por ejemplo en pacientes con ulceras, prurigo, intertrigo, tia pedis, etc paciente diabticos o
imunosuprimidos. De forma ms excepcional se debe a una diseminacin hematgena, procedente de focos a distancia, por ejemplo en
dientes o faringe. El periodo de incubacin es de 2 a 5 das, la localizacin ms frecuente son los miembros inferiores, puede localizarse
en cara y rea periumbilical en caso de los lactantes. DIAGNOSTICO: Precede a la lesiones cutneas sntomas como fiebre, escalofro ,
nuseas , cefaleas y vmitos, malestar general y dolor local ; las lesiones tienen comienzo abrupto y se presentan como una placa
ardorosa, eritematosa de color rojo intenso, indurada, edematosa, caliente, dolorosa a la palpacin y bien delimitada , su crecimiento
es centrfugo con un borde activo y bien definido, con la zona central de la placa aclarndose ; en algunas formas menos usuales se
pueden presentar vesculas, ampollas, pstulas y lesiones necrticas. Su topografa ms comn es en las extremidades inferiores y en la
cara a nivel de las mejillas. Se acompaa con frecuencia de adenopatas satlites regionales. Habitualmente las lesiones inician su
involucin en 10-12 das descamndose la regin afectada sin dejar cicatriz. Habitualmente el diagnstico es clnico pero si es posible
obtener material de la puerta de entrada se debe realizar Gram y cultivo. TRATAMIENTO: Antisepsia. Antibitico Tpico. Antibitico
Parenteral se puede iniciar con Penicilina (800,000 UI de penicilina procainica c/24 hrs por 8-10 das, continuar con benzatinica
1200,000 UI cada 8 das por 3-4 semanas), Cefalosporinas, Amoxicilina + Ac. Clavulnico, y en los pases con cepas Meticilino-
resistentes se pueden utilizar de acuerdo a estudios epidemiolgicos previos: Trimetoprim-sulfametoxazole, Clindamicina, Rifampicina-,
Linezolid y Vancomicina. Si hay alergia a penicilinas se puede usar eritromicina. Para evitar recidivas usar sultrim dos tabletas diarias por
8-10 dias. COMPLICACIONES: incluyen bacteremia, abscesos, gangrena, linfangitis, celulitis, septicemia, tromboflebitis y
glomerulonefritis. Cuando se trata de brotes repetidos de erisipela la afeccin de los vasos linfticos que al fibrosarse originan estados
elefantisicos, conocidos como elefantiasis nostra.

CASO CLINICO
Se trata de femenino de 2 aos de edad la cual cuenta con el antecedente de haber jugado en la tierra y producirse escoriasiones en
miembros plvicos, a las 48 hrs posteriores inicia con fiebre no cuantificada, enrojecimiento y edema en la zona, la cual se encuentra
caliente y eritematosa a la exploracin.

PREGUNTA
Cual es la conducta a seguir.

RESPUESTA
a.- Limpieza y tratamiento ambulatorio con penicilina.
b.- Debridacin y tratamiento hospitalario.
c.- Limpieza y alta con signos de alarma.
d.- Limpieza local, vendaje y penicilina.
MANUAL DE TRABAJO DEL CURSO ENARM CMN SIGLO XXI
CURSO ENARM CMN SIGLO XXI TEL: 36246001 Pharmed Solutions Institute PGINA 390


PIODERMIA GANGRENOSA. CIENCIAS BASICAS: Fagadenismo geomtrico de Brocq, a pesar del nombre este cuadro de causa
desconocida, dista mucho de ser una enfermedad bacteriana, probablemente sea ms una inmunopatia que una verdadera infeccin
aun cuando no pueda descartarse del todo la intervencin de bacterias. PATOGENIA: Su etiologa es desconocida y se invocan
alteraciones inmunolgicas aun no bien determinadas, se piensa en una reactividad tisular local, necrtica a antgenos microbiana.
DIAGNOSTICO: En cualquier rea del cuerpo sobre todo en las extremidades, aparece repentinamente en personas en aparente estado
de salud, un rea eritematosa que pronto se cubre de pstulas o ampollas y se ulcera rpidamente. Esta ulceracin se extiende con
contornos geogrficos, sus bordes son levantados y pueden aparecer abscesos en la periferia habitualmente estriles, el fondo es sucio,
sangrante o necrtico. En poco tiempo estas lesiones que profundizan por lo general poco, han abarcado extensas zonas de la piel. La
sintomatologa no corresponde a la intensidad del proceso, pero puede haber dolor y ataque al estado general del paciente. Es
frecuente que este proceso coincida con artritis, colitis ulcerosa, enteritis regional de Crhon o enfermedades hematolgicas con las
cuales no hay necesariamente una relacin causa-efecto. TRATAMIENTO: Es emprico, se han usado los corticoesteroides, la colchicina,
la diaminodifenilsulfona, la clofazimina, la talidomina. Algunos responden bien a determinados frmacos y otros casoso a otros o no
responden. El proceso de repente y sin causa aparente se detiene y se inicia la cicatrizacin

CASO CLINICO
Femenino de 17 aos que consult por lesiones ampollosas dolorosas, de contenido claro, en rodilla derecha, acompaada de fiebre
con escalofros, de 1 semana de evolucin. Las mismas lesiones aparecen gradualmente en toda la pierna derecha, antebrazo derecho y
glteo izquierdo, que revientan y dejan la piel exulcerada. Ingres febril, estaba plida. En piel se observan mltiples lesiones
ampollosas exulceradas, con fondo fibrinoso, muy dolorosas, ubicadas en rodilla y pierna derecha, otras ms pequeas en mueca
derecha y glteo izquierdo.

PREGUNTA
Cual es la conducta a seguir ahora?

RESPUESTA
a.- Limpieza y tratamiento ambulatorio con penicilina.
b.- Debridacin y tratamiento hospitalario con corticoides y antibiticos.
c.- Limpieza y alta con signos de alarma.
d.- Limpieza local, vendaje y penicilina.

TUBERCULOSIS (TB) CUTANEA. CIENCIAS BASICAS: La TB cutnea se considera en la mayora de los casos de reinfeccin, la
primoinfeccin es excepcional, aunque posible presentndose el complejo primario en la piel: ndulo cutneo, linfangitis y
adenopatas. SALUD PUBLICA: Mycobacterium tuberculosis, quiz el germen de mayor ubicuidad en el mundo y en el organismo pues
no hay rgano que se escape a su agresin. La tuberculosis sigue siendo un problema de salud pblica, especialmente en la forma
pulmonar. La TB cutnea ocupa el 4to o 5to lugar, en cuanto a localizacin del bacilo de Koch. Estadsticas mexicanos en varios aos
muestran cifras que van de 1-3% de Tb cutnea por cada 114 pacientes y cerca de 20% son nios menores de 15 aos. PATOGENIA:
Casi en todos los casos, los pacientes han recibido ya la primoinfeccin pulmonar inoculado con M. tuberculosis bovina y hominis,
presentando una respuesta positiva al PDD. A partir de este foco original mediante reinfeccionendogena.se diseminan los bacilos a la
piel, otras veces la reinfeccin es exgena por llegada de nuevos bacilos del exterior. De acuerdo con esta va de reinfeccin, de la
virulencia del bacilo y sobre todo de la respuesta inmunolgica (hipersensibilidad) del organismo ante el bacilo, se van a producir
lesiones en la piel y que parecen entidades diferentes. CLASIFICACION: de Latap, Escalona y Estrada; se consideran 2 grupos bsicos, el
primer grupo est formado por entidades en las cuales hay respuesta normrgica al bacilo. El segundo grupo comprende las llamadas
tuberculoides, que como su nombre indica son lesiones resultados ms de la respuesta exagerada, hiperrgica al bacilo. En este caso no
es posible encontrar bacilos en las lesiones y la respuesta a los antgenos bacilares es muy intensa. TB COLICUATIVA: Tambin conocida
como escrufulosis o escrofulodermia, en Mxico es la variedad ms frecuente. Se presenta sobre todo en nios y jvenes y predomina
en las reas de poblacin ms desprotegidas. Es secundaria habitualmente a TB de ganglios, huesos y articulaciones. Su topografa
habitual es donde hay ganglios, como regiones supraclaviculares, las axilas, las ingles, uni o bilateral. Puede estar en regin esternal,
codos, rodillas. Las lesiones son siempre ndulos y gomas, refieren pequeas bolitas en el cuello, es decir ganglios infartados, no
dolorosos, no mviles, un buen da se fijan a la piel y esta se pone eritematosa e infiltrados y se inicia la formacin de un ndulo, que se
reblandece y se abre al exterior dejando salir pus de color amarillo claro y espeso, pronto toda la regin se convierte en un plastrn
endurecido (ndulos, gomas, abscesos fros), es posible que dejen cicatrices deformantes y retractiles. Puede haber febrcula vespertina
o franca fiebre, anorexia y adelgazamiento, y a veces sntomas de TB activa. TB VERRUGOSA: A menudo inoculada en personas que
manejan material contaminado como mozos de anfiteatro, matanceros, carniceros, laboratoristas. Se localiza en partes distales, de os
miembros como los pies y las manos y algunas veces en nalgas. En sitio de inoculacin aparece el tubrculo anatmico, que es ndulo
verrugoso, que es punto de partida de las lesiones que son siempre placas verrugosas o vegetante de tamaos variables circulares u
ovales, bien limitadas y cuya superficie siempre es spera, con costras melicericosanguineas, a veces se observan pstulas. En algunas
placas se ve cicatrizacin central con crecimiento perifrico y puede haber compromiso de los linfticos superficiales y profundos de la
regin y producirse una estasis linftica que ocasiona edema y ms verrugosidades. TB LUPOSA: Era muy frecuente en el siglo pasado.
Es ms frecuente en nios y jvenes y su clsica topografa es en la cara; mejillas y dorso de nariz, disposicin en mariposa, tambin
puede afectar a los pabellones auriculares. La lesin fundamental es el ndulo pequeo llamado lupoma que se cubre de escamas y
verrugosidades que lo ocultan, de tal manera que se forman placas eritematosas, escamosas, verrugosas, circulares, bien limitadas de
crecimiento perifrico y cicatrizacin central y con tendencia a la ulceracin. En el centro solan producir extensas y profundas
destrucciones, el subtabique nasal se destrua y la nariz tomaba el aspecto de pico de loro. TB MILIAR AGUDA: Muy rara se
presentaba en nios y jvenes con TB avanzada y baja reactividad a los antgenos. Son pequeos ndulos rojizos acuminados, a veces
ulcerado y cubiertos de costras en diferentes partes del cuerpo, en especial en el centro de la cara, que cicatrizan espontneamente.
MANUAL DE TRABAJO DEL CURSO ENARM CMN SIGLO XXI
CURSO ENARM CMN SIGLO XXI TEL: 36246001 Pharmed Solutions Institute PGINA 391

TB NODULAR PROFUNDA: Descrita por Bazin, con el nombre de eritema indurado, casi exclusiva de mujeres de localizacin nica en las
piernas sobre todo en las caras posteriores y caracterizada por ndulos profundos, de evolucin crnica y recidivante, muy dolorosos y
que en su evolucin dejan zonas atrficas depreimidas que deforman las piernas. TB NODULO NECROTICA: Es menos frecuente que la
nodular profunda, aparece en personas jvenes con topografa en partes salientes; codos, rodillas, nalgas y en ocasiones en la cara,
sobre todo en los pabellones auriculares. La lesin es un pequeo ndulo que sufre una necrosis central y as se cubre de una costra
negruzca que al caer deja una cicatriz varioliforme. Son asintomticos y evolucionan por brotes. TB MICRONODULAR: Es muy rara y
pasa inadvertida, son ndulos de 1-2mm que forman placas de aspecto folicular en las regiones lumbares u otras partes del tronco, son
asintomticas. DIAGNOSTICO: Es ms bien de eliminacin, dado que la comprobacin del agente causal no es fcil en todos los casos. El
laboratorio nos puede ayudar sobre todo para descartar procesos que suelen semejarse a las lesiones tuberculosas. Hallazgo del bacilo;
puede hacerse por medio de baciloscopia y de biopsia difcil de encontrar en TB colicuativa y ulcerosa. El cultivo en medios apropiados
como Lowestein es difcil de lograr. Histopatologa; por definicin la imagen de las lesiones tuberculosas es el clsico granuloma
tuberculoide, constituido por linfocitos, clulas epiteloides y clulas gigantes tipo Langhans, resultado de la unin de las segundas., sin
embargo no es patognomnica de la tuberculosis, por lo que solo ser sugestiva de tuberculosis. Intradermoreaccin con PPD; no es
diagnostica ya que solo nos indica un contacto con el bacilo de Koch y un alto porcentaje de la poblacin da positivo. Tiene importancia
cuando es negativa, lo cual significa que no tienen un cuadro de origen tuberculoso. TRATAMIENTO: Se usan las mismas drogas que en
la tuberculosis en general y a dosis semejantes. La estreptomicina es muy til a dosis de 1 gr diario o cada tercer da, 50-60grs como
dosis total. La isoniacida que se dice bacteriosttica se usa a 5-8mg/kg. El etambutol a dosis de 15-20mg/kg, es ms costoso y tienen
efectos colaterales en nervio ptico, es reversible. Rifampicina a dosis de 10-20 mg/kg, para un adulto se aconsejan 600mgs, para nio
la mitad. No usar monoterapia. Tratamiento ideal es; 1 gr de estreptomicina cada tercer da, 600mgs de isoniacida diarios o bien
estreptomicina + etambutol. La TB colicuativa tarda de 3-6 meses en curar. La TB verrugosa y luposa requieren menos tiempo. En la
actualidad se est aplicando con xito un triple tratamiento, isoniacida, rifampicina, pirazinamida.

CASO CLINICO
Nia de 10 aos que consulta por una lesin en mejilla izquierda seguida de la aparicin de adenopata cervical, de dos meses de
evolucin. Al examen fsico, se observa una pequea lcera de 1 cm de dimetro, con leve descamacin perifrica y adenopata satlite.

PREGUNTA
Cual es la conducta a seguir?

RESPUESTA
a.- Biopsia de la lesin.
b.- PPD.
c.- BAAR.
d.- Cultivo de la lesin.

PITIRIASIS ALBA (MAL DEL PINTO). CIENCIAS BASICAS: Es una condicin benigna de lapiel comn en la infancia, descrita por primera
vez hace ms de 80 aos y su importancia radica en que es un motivo de consulta frecuente debido a su curso crnico, tendencia a
reaparecer y su impacto esttico. SALUD PUBLICA: La pitiriasis alba se encuentra en todas partes del mundo. Afecta a entre 1.9-5.25%
de los nios en edades entre 6-16 aos. En general, no hay predisposicin en cuanto a sexo; sin embargo, algunas series describen una
mayor afeccin de los varones, as como a individuos de color de piel oscura.3 Se cree que la mayor frecuencia de pitiriasis alba en
personas de piel oscura es debido al hecho de que su piel se tie ms fcilmente excepto en las reas afectadas. Hay reportes que
muestran mayor incidencia en nios escolares de baja situacin socioeconmica. Desde el punto de vista clnico afecta la cara en el 50%
de los casos. PATOGENIA: La etiologa no est bien establecida y a la condicin se le han adjudicado diversas denominaciones, como
eritema streptogenes, pitiriasis streptogenes, e impetigo furfuracea, usualmente relacionadas con los agentes etiolgicos propuestos,
sin embargo, no se ha identificado una asociacin causal definitiva. Microorganismos como Pityrosporum, Streptococcus, Aspergillus y
Staphylococcus, se han implicado en su etiologa pero ninguno de ellos se ha confirmado como agente causal. Varios factores adems
de los infecciosos, se han asociado con la etiologa, la exposicin solar, la humedad relativa del aire, la altitud y el viento. El uso de
jabones abrasivos, duchas excesivamente largas y la temperatura caliente del agua durante el bao tambin se han relacionado con la
xerosis que se aprecia en esta entidad. En cuanto a la exposicin solar, se plantea que la radiacin ultravioleta induce una irritacin
excesiva de la piel que conlleva a una reaccin inflamatoria y que adems la accin directa de la luz solar sobre los melanocitos
modifica su nmero y su funcin conduciendo a una disminucin de la intensidad del color de las lesiones en la pitiriasis alba. Esto
adems se apoya en el hecho de presencia de lesiones en reas fotoexpuestas y en nios en edades en las cuales hay mayor nmero de
actividades al aire libre. Se ha observado que el uso de protector solar frecuente FPS > 15 reduce el desarrollo de pitiriasis alba. La
condicin dermatolgica que se asocia principalmente con la pitiriasis alba es la dermatitis atpica y la presencia de deficiencias
nutricionales as como la xerosis es comn a ambas, por lo que se ha implicado en la patognesis de la misma. Las deficiencias
nutricionales involucradas son de vitaminas y bajos niveles de cobre, el cobre es un cofactor para la tirosinasa, enzima necesaria para la
produccin de melanina, por lo que su deficiencia puede jugar papel patognico en esta entidad. DIAGNOSTICO: Frecuentemente se
inicia como placas rosadas con un borde elevado que luego de varias semanas se desvanece dejando una mancha plida cubierta por
una descamacin blanquecina polvorienta. Posteriormente progresa a mculas hipopigmentadas de bordes difusos, de tamao variable
entre 0.5 a 5 cm de dimetro. Aunque puede haber ligero prurito en general las lesiones son asintomticas, por lo que generalmente el
paciente no consulta por ellos y se observan incidentalmente en el examen fsico. Aunque en el caso de los nios, puede ser motivo de
preocupacin comn en las madres, ya que la mayora de las lesiones se localizan en la cara, principalmente en la frente y la zona
malar, pero tambin pueden encontrarse en las extremidades superiores y ocasionalmente en las inferiores. Las lesiones pueden
persistir por 6 meses a 7 aos, pero este curso puede prolongarse en los pacientes atpicos; en general es una condicin autolimitada.
Por otro lado, las lesiones pueden ser ms visibles en el verano, cuando la piel circundante est bronceada. Luego de la resolucin las
lesiones pueden reaparecer en la misma localizacin. Variantes clnicas: PITIRIASIS ALBA EXTENSA: esta entidad se caracteriza por
MANUAL DE TRABAJO DEL CURSO ENARM CMN SIGLO XXI
CURSO ENARM CMN SIGLO XXI TEL: 36246001 Pharmed Solutions Institute PGINA 392

lesiones de pitiriasis alba generalizadas y no precedidas de eritema. Se observa ms frecuentemente en los adultos. Generalmente se
afectan la porcin inferior del tronco de manera simtrica. Es asintomtica y los pacientes usualmente no tienen antecedentes o
historia de atopia. La histologa no es especfica, se observa un nmero reducido de melanocitos funcionales con menor nmero de
melanosomas, pero su distribucin y transferencia a los queratinocitos no est perturbada. PITIRIASIS ALBA PIGMENTADA: es una
variante que puede estar asociada con una infeccin por dermatofitos superficial y la forma clsica de PA. Se observa una
hiperpigmentacin azulada rodeada de un rea hipopigmentada y descamativa muy parecida a la pitiriasis alba clsica. Casi siempre
aparece en la cara y rara vez hay compromiso extrafacial. El rea pigmentada se atribuye a depsitos de melanina en la dermis, lo que
no se observa en la pitiriasis alba clsica. En general no es necesario ya que el diagnstico clnico suele ser obvio, adems de que la
histologa no es especfica. Lo destacable es una menor cantidad de melanocitos y melanosomas en la capa basal de la epidermis, sin
embargo, esto se ha asociado a inflamacin determinada por cierto grado de espongiosis, lo que interfiere con la produccin de
melanina. Desde el punto de vista histolgico puede llegarse a un diagnstico si se observa: pigmentacin irregular de la melanina en la
capa basal, taponamiento folicular, espongiosis folicular y glndulas sebceas atrficas. Adems de infiltrado linfocitario y edema en la
dermis. Estos cambios son variables dependiendo de la etapa en la que se encuentre la enfermedad. TRATAMIENTO: Inicialmente en
convencer al paciente de que la enfermedad es benigna y autolimitada ya que hasta ahora, ninguna terapia es completamente exitosa.
Se recomienda limitar la exposicin solar, uso regular de protector solar FPS > 15 y reducir la frecuencia y la temperatura de los baos.
Los emolientes y cremas hidratantes ayudan a disminuir la xerosis y la irritacin. En los casos en etapa inflamatoria la desonida o la
hidrocortisona tpica al 1% pueden ayudar a la resolucin de las lesiones. En los nios solo deben prescribirse esteroides de baja
potencia no halogenados. La forma extensa de PA no responde a los esteroides tpicos pero si lo hace a terapia con PUVA.
Recientemente se public la efectividad de tacrolimus al 0.1% en el tratamiento de esta entidad. El tacrolimus es un macrlido que
bloquea la funcin cataltica de la calcineurina, inhibiendo la sntesis y la liberacin de IL-2. La eficacia y la seguridad de tacrolimus en
ungento en el tratamiento de la dermatitis atpica est bien documentada. Sin embargo, el mecanismo de accin en la pitiriasis alba
no es bien entendido. Una intervencin teraputica definitiva ser posible cuando se comprenda mejor la patognesis de esta entidad,
a la cual no se le da mucha importancia en la literatura mdica, lo que ha conllevado a la escasez de conocimientos precisos en cuanto a
su patogenia y de estudios controlados para dilucidar una teraputica totalmente efectiva.

CASO CLINICO
Nio de 6 aos de edad el cual acude a consulta de rutina, muestra en la cara manchas blancas, bordes irregulares no bien definidas,
con leve descamamiento a la exploracin, no son pruriginosas?

PREGUNTA
Cual es la conducta a seguir menos adecuada?

RESPUESTA
a.- Protectores solares.
b.- Baos cortos con agua templada.
c.- betametasona.
d.- Tacrolimus.

HERPES SIMPLE. CIENCIAS BASICAS: Los virus de herpes se encuentran ampliamente distribuidos en la naturaleza y la mayora de las
especies animales son hospederos naturales de ms de uno. Se han aislado y caracterizado ms de 100, varios de los cuales afectan al
humano: herpes simple tipo 1 y tipo 2 (VHS-1, VHS-2), varicela zoster (VVZ), citomegalovirus (CMV), Epstein Barr, (VEB), virus herpes
humano 6 (VHH6), virus herpes humano 7 (VHH7), virus herpes humano 8 (VHH8) y otros. SALUD PUBLICA: La infeccin por VHS-1 es
frecuente en sitios hacinados y con condiciones precarias de higiene se tienen porcentajes de 90% de la poblacin tienen anticuerpos
antivirales El ms frecuente en nios. El VHS-1 en ubicacin oral se transmite por saliva, besos, por compartir vasos, cepillos de dientes
y en otras partes del cuerpo se debe a contacto del virus con la piel, se autotransmite con frecuencia, principalmente a los ojos. La
infeccin por VHS-2 depende de la actividad sexual. El VHS-2 se transmite por secreciones vaginales, contacto sexual (prepuberes) y al
neonato durante el paso por el canal de parto infectado. La posibilidad de que VHS-1 y VHS-2 establezcan infecciones latentes con
recidivas asintomticas favorece su transmisin, ya que un individuo infectado puede ser transmisor durante toda su vida. Los virus
infectantes se encuentran en el lquido de las vesculas. PATOGENIA: La patogenia de VHS-1 y VHS-2 es similar, con infeccin primaria
generalmente asintomtica, aunque pueden presentarse lesiones vesiculares. El virus inicia la infeccin en las membranas de las
mucosas, se replica en las clulas mucoepitelilales originando infeccin ltica y se disemina a las clulas adyacentes y neuronas que
inervan el sitio donde se inici la infeccin aguda. La infeccin latente en la neurona no produce dao aparente, pero diferentes
estmulos la pueden reactivar. Una vez reactivado, el virus se multiplica, viaja a lo largo del nervio en forma centrifuga y ocasiona lesin
en la terminal del nervio, por lo tanto todas las recidivas se producen en el mismo sitio. La expresin del genoma se requiere para la
reactivacin, pero no para el establecimiento de la latencia. El mecanismo para su establecimiento se desconoce, sin embargo, se
piensa que para la expresin del genoma se requiere una protena celular, tan es as que no en todas las estirpes celulares establece
latencia. El tipo de infeccin que resulta depende del estado inmune del individuo; los sujetos susceptibles desarrollan infeccin
primaria despus de la primera exposicin al virus. Sujetos seropositivos pueden ser reinfectados con virus de otro tipo. VHS-1 y VHS-2
se transmiten por diferentes vas e infectan diferentes sitios del cuerpo. A grandes rasgos se considera que el HSV-1 infecta de la
cintura para arriba y el VHS-2 de la cintura para abajo, sin embargo esta diferenciacin no es estricta. DIAGNOSTICO: La primoinfeccin
por VHS-1 (ORAL): puede presentarse de distintas formas: La gingivoestomatitis herptica; forma ms frecuente de infeccin primaria o
inicial en nios. Se trata de un cuadro agudo con fiebre y mal estado general, aparicin de lesiones vesiculosas y erosivas en mucosa
oral que ocasionan odinofagia y halitosis caractersticas, dificultando la alimentacin. Pueden existir lesiones vesiculosas, en labios e
incluso en la cara, agrupadas en racimos (tpicas). Podemos encontrar adenopatas submandibulares y el cuadro clnico es auto limitado
(7-10dias). Otras formas de primoinfeccin, pueden aparecer lesiones en cualquier localizacin, solo es precisa una fuente de contagio
(familiar), una puerta de entrada (una erosin, araazo) y un husped susceptible (nio). Podemos encontrar casos de primoinfeccines
MANUAL DE TRABAJO DEL CURSO ENARM CMN SIGLO XXI
CURSO ENARM CMN SIGLO XXI TEL: 36246001 Pharmed Solutions Institute PGINA 393

con lesiones faciales extensas, en un miembro, oculares, y lo que es ms frecuente y bien definido, el panadizo herptico, caracterizado
por la aparicin de lesiones cutneas vesiculosas, que afectan al pulpejo y regin periungueal de un dedo, muy dolorosas, que cursan
con fiebre y adenopatas satlites. El cuadro tambin es autoinoculativo, pero dura algo ms; unas 2 semanas. Es bien conocida por
todos la capacidad de los virus simple de permanecer en el husped, acantonados y recurrir o producir sntomas ante diversas
circunstancias; enfermedades debilitantes, infecciones, fiebre, exposicin solar. La clnica es variable pero suele localizarse en la cara;
labios, regin perioral, mejillas. Otras localizaciones son ms raras en la infancia. El aspecto clnico es muy caracterstico con la
aparicin de lesiones vesiculosas, agrupadas en racimo, sobre una piel eritematosa, que con el tiempo dan lugar a una costra. El cuadro
puede ir precedido de sensacin prodrmica de dolor, quemazn o disestesias de difcil descripcin e interpretacin por parte de
nuestros pacientes y no suele acompaarse de fiebre, mal estado general, ni adenopatas. La infeccin por VHS-1 puede originar
cuadros clnicos de variada severidad, que oscilan desde la gingovoestomatitis, herpes labial, panadizo herptico, meningitis, encefalitis
con alta mortalidad y queratitis herptica que a su vez puede originar ceguera. VHS-2 (GENITAL): En general no ocurre en edad
peditrica, cuando aparece en un menor debe investigarse la posibilidad de abusos sexuales. La primoinfeccin es superponible a las
formas orofacilaes, pero en la regin genital. El cuadro tambin es agudo se acompaa, de dolor, fiebre, dificultad para la miccin y
adenopatas. Las lesiones vesiculosas, erosivas, tpicamente agrupadas en racimos nos darn sospecha diagnostica. El diagnstico de
laboratorio econmico y rpido se realiza mediante la prueba de Tzank que consiste en hacer una impronta de las clulas y teirlas con
el colorante de Wright o Giemsa, y observar clulas fusionadas con varios ncleos, sincitios, as como inclusiones nucleares de Cowdry.
No es posible confirmar la presencia del virus por medio de esta tcnica ya que otros virus producen el mismo efecto en las clulas,
inclusive el VVZ. Actualmente se hace uso de tcnicas inmunoenzimticas, biolgicas, bioqumicas y de biologa molecular para detectar
anticuerpos o antgenos virales. La deteccin de anticuerpos solo es de utilidad para deteccin de la primo infeccin y estudios
epidemiolgicos. TRATAMIENTO: Existen antivirales efectivos utilizados en el tratamiento de infecciones por VHS-1 y VHS-2, entre ellos
famciclovirn (125mgs c/12hrs por 5 dias), Aciclovir (200mg 5 veces al dia por 5 dias), valaciclovir (500mgs c/8 hrs por 5 dias). Los
antivirales no eliminan las partculas virales que se encuentran en ganglios neurales, solo impiden su replicacin, por lo que pueden
presentarse reactivaciones. En estos casos, el uso de dosis mnimas de los frmacos, durante un tiempo prolongado, tratamiento
denominado supresivo, se indica en los pacientes con brotes constantes, prolongados o intensos, aunque debe contemplarse la
posibilidad de resistencias. Los VHS se transmiten a travs de las secreciones de lesiones, por lo es recomendable evitar contactos
durante la lesin activa. El tratamiento tpico debe hacerse mediante fomentos con sulfato de cobre 1/1000 o sulfato de cinc 1/1000 y
antibitico tpico (mupirocina, ac. Fusidico) COMPLICACIONES: La ms frecuente es la impetiginizacion o sobreinfeccin bacteriana,
fundamentalmente estreptoccica y estafiloccica. Las lesiones oculares en particular, la queratitis puede llevar a ceguera. INFECCION
HERPETICA NEONATAL: La infeccin herptica durante la gestacin puede dar lugar a manifestaciones en el RN y en el feto. La va de
contagio puede ser transplacentaria o genital (en el canal del parto, o ascendente, con o sin rotura de membranas). Existen 2
posibilidades: infeccin primaria antes de la 20 SDG, aborto, muerte fetal, hidrocefalia y coriorretinitis. Herpes simple neonatal, en ms
de 90% de los casos es en relacin con la primoinfeccin herptica materna durante la gestacin, solo 5% se produce afeccin fetal en
las recurrencias. Cuando la infeccin ha sido adquirida en la vida intrauterina (5-15%), la sintomatologa aparece en las primeras 24-
48hrs. En caso de infecciones intrapartum, generalmente por VHS-2 (madre) o en el caso de VHS-1 (padres, familiares) la
sintomatologa se produce entre los 2-20 das. Clnica: afectacin severa del estado general en un neonato aparentemente sano,
lesiones cutneas, coriorretinitis, y afeccin al SNC, afectacin sistmica-visceral, lesiones cutneas en ms de 50% de los casos:
vesculas, ampollas, pstulas, cicatrices, eritema generalizado, erosiones, anoniquia. Mortalidad elevada (40%) y elevada morbilidad.
Tratamiento precoz cesrea.

CASO CLINICO
Femenino de 15 aos de edad la cual acude a consulta por presentar desde hace dos das, vesculas agrupadas en el labio inferior, que
se transforman en pustulas y costras hemorrgicas, cuenta antecedente de prurito y sensacin de hormigueo en la zona. Comenta la
paciente que en el ltimo ao se han repetido episodios de vesiculas similares en varias ocacion, antecentes de importancia
actualmente embarazo de 22 semanas, ha tenido multiples parejas y es madre soltera.

PREGUNTA
Cual es la conducta a seguir mas adecuada?

RESPUESTA
a.- Tratamiento antiviral doble esquema.
b.- Realizar USG para identificar anormalidades.
c.- Tratar y esperar parto nornal.
d.- Tratamiento especifico y programa cesarea aportuna.

HERPES ZOSTER. CIENCIAS BASICAS: El virus de la varicela (VVZ) origina varicela y su reactivacin causa herpes zoster. SALUD PUBLICA:
Ms frecuente en el adulto que en la infancia (0.74 casos por 1000 individuos al ao). Suele ocurrir en nios que pasaron varicela
precoz (primeros 2 aos de vida). El virus se transmite por contacto directo, por gotitas de aerosol y por va rea en comunidades;
tambin puede haber contagio por contacto con vesculas cutneas. Es altamente transmisible. Se dispone de eficaz vacuna atenuada.
PATOGENIA: Se caracteriza por una infeccin neurolgica y dermatolgica, posterior a un episodio de varicela (primoinfeccin) en los
ganglios sensitivos espinales y craneales (donde esta latente el VVZ). Una vez reactivado, el virus se mueve a lo largo de las fibras
sensitivas hasta l rea de piel que inervan, que usualmente afectan de 1-3 dermatomas. Dicha reactivacin ocurre, por lo general, una
vez en la vida, sobre todo en pacientes de la tercera edad o en aquellos inmunodeprimidos (por HIV, cncer, tratamiento con
corticoesteroides, frmacos inmunosupresivos, por ejemplo). DIAGNOSTICO: Las lesiones cutneas consisten en lesiones papulosas
iniciales agrupadas en racimos sobre la piel eritematosa, las ppulas se convierten en vesculas en 12-24 hrs y luego estas se rompen y
dan lugar a costras. La erupcin cura en 7-14 das y suele acompaarse de dolor. El dolor puede ser prodrmico, es decir preceder a las
lesiones cutneas, acompaarlas e incluso persistir durante meses tras la resolucin clnica dermatolgica, en este ltimo supuesto se
MANUAL DE TRABAJO DEL CURSO ENARM CMN SIGLO XXI
CURSO ENARM CMN SIGLO XXI TEL: 36246001 Pharmed Solutions Institute PGINA 394

habla de neuralgia postherptica, esta implica la prdida de autonoma y una calidad de vida pobre. La infeccin por varicela puede
ocasionar ocasionalmente que el virus atraviese la placenta, dando lugar al sndrome de varicela congnita que puede ser leve o severo.
Cuando la madre ha estado en contacto con casos de varicela es recomendable determinar el nivel de anticuerpos anti-varicela. El
diagnstico se basa en la prueba de Tzank, con las dificultades mencionadas; el virus se puede aislar de las vesculas y recientemente se
utilizan tcnicas de biologa molecular para su identificacin. COMPLICACIONES. La neuralgia postherptica. En individuos
inmunocomprometidos puede dar lugar a enfermedades progresivas y graves, principalmente encefalitis post-infeccin y neumona, en
algunos casos fatales. La respuesta inmune celular que generalmente es ms potente en adultos que en nios causa durante la
infeccin primaria lesiones y cuadros graves, principalmente en pulmones, y da lugar a 20-30% de los pacientes con neumona
intersticial. TRATAMIENTO: Resultados satisfactorios se han obtenido con inhibidores de la sntesis del ADN (aciclovir y derivados), sin
embargo se requieren dosis ms altas que las recomendadas para HSV-1 y 2. HERPES ZOZTER EN LA INFANCIA: Las principales
caractersticas son: 1. Suele presentarse en nios que presentaron varicela en los 2 primeros aos de vida. 2. Su curso es benigno y sin
complicaciones. 3. Es poco doloroso (no prdromos), no suele complicarse con neuralgia postherpetica. 4. Se recomienda investigar
existencia de inmunodepresin. 5. El tratamiento oral con antivirales solo est indicado en casos seleccionados, pero debe ser lo ms
precoz posible.

CASO CLINICO
Masculino de 6 meses de edad quien manifest una dermatosis diseminada al tronco, regin inguinal derecha y la cara lateral interna
del muslo derecho en el tercio distal, asi como en el tercio medio y proximal de la pierna. La dermatosis era unilateral, constituida por
vesculas agrupadas sobre una base eritematosa y escasas costras hemticas que confluida entre si, formando varias placas de tamao
variable y de evolucin aguda y dolorosa, se realizo diagnostico clnico de herpes zoster.

PREGUNTA
Cual es la conducta a seguir menos apropiada?

RESPUESTA
a.- Aciclovir 400 mg/kg/dosis.
b.- Fomentos con sultafo de cobe y oxido de cinc.
c.- Profilaxis antibitica profilctica.
d.- Cerco sanitario a contactos.

VERRUGAS VIRALES. CIENCIAS BASICAS: Son los virus del papiloma humano capaces de producir las verrugas virales y algunos cuadros
relacionados con la carcenognesis. Se han descrito ya 25 tipos de virus. La palabra verrugosidad indica solamente una lesin dura,
anfractuosa, seca que histolgicamente presenta hiperqueratosis y papilomatosis. La vegetacin en cambio es blanda, anfractuosa,
friable y solo presenta la papilomatosis, por ello estas lesiones solo se presentan en zonas hmedas de la piel y en mucosas. Las
verrugas virales son neoformaciones epiteliales benignas que pueden afectar a cualquier persona, nios, adultos, hombres y mujeres,
constituyendo una de las 5 entidades que con ms frecuencia se ven en la consulta. Su transmisibilidad es baja y sabemos que son
autoinoculables. Un estado inmunolgico deteriorado permite sui extensin y persistencia. TIPOS: Vulgares, planas, plantares,
acuminadas y filiformes. VERRUGAS VULGARES: Son los populares mezquinos y son las ms frecuentes. Predominan en los nios. Son
neoformaciones verrugosas solitarias o mltiples, de superficie anfractuosa, secas, duras, de color de la piel o ms oscuras,
semiesfricas y bien limitadas, presentan en su superficie un fino puntilleo oscuro. Son indoloras a menos que se les traumatice y su
desaparicin es espontnea y no deja secuelas, habituales en extremidades superiores. VERRUGAS PLANAS o juveniles: Son muy
pequeas y numerosas, no ms de 1mm y muy aplanadas, levantan de la superficie, as es que a primera vista parecen machas. De color
de la piel, llegan a sumar ms de 100, localizadas de preferencia en las mejillas aunque tambin pueden verse en extremidades
superiores; dorso de manos o antebrazos. Tambin son asintomticas. VERRUGAS PLANTARES: Se les conoce vulgarmente como ojos
de pescado y como su nombre o indica se presentan en la planta del pie, la presin de peso del cuerpo las hunde en la gruesa capa
cornea, por lo que solemos ver la base de pirmide, y ello explica el dolor que las acompaa, nicas o mltiples, con pequeas
hemorragias postraumticas en su superficie. VERRUGAS ACUMINADAS: Tambin llamadas condilomas acuminados o papilomas
venreos, la mayora se adquieren por la relacin sexual, en nios a veces por desaseo. Asientan en sitios hmedos y calientes,
alrededor de orificios naturales; glande, labios mayores y menores, ano recto, boca, no se produce hipertrofia como en otras verrugas,
la lesin toma ms bien un aspecto vegetante, como coliflor, pueden ser pequeas a penas salientes (crestas en el trmino) o alcanzar
dimensiones monstruosas, deformando la regin, se maceran e infectan y adquieren un olor muy desagradable. Debemos
diferenciarlas siempre de condilomas planos o siflides papuloerosivas, muy ricas en treponemas. Son ppulas aplanadas y erosionadas
y no neoformaciones vegetantes, no tiende a persistir, evolucionan hacia la desaparicin. Los condilomas acuminados no tienden a la
involucin, persisten y crecen. VERRUGAS FILIFORMES y digitiformes: Se presentan en los pliegues: cuello axilas, ingles en forma de
neoformaciones alargadas, como hilos oscuros, algunas pediculadas o con prolongaciones transparentes. Se discute su etiologa viral.
PRONOSTICO: Aunque se menciona el papel oncognico de algunas variedades de parvovirus, no debe tomarse a las lesiones
verrugosas comunes como precancerosas, sin embargo en el caso de lesiones acuminadas, se piensa que pueden relacionarse con el
cncer cervicouterino y se ha observado que las mujeres que presentan esta neoplasia tienen con ms frecuencia verrugas acuminadas
y no solo eso, sino que se ha observado tambin que las parejas sexuales han padecido la infeccin genital por estos virus.
TRATAMIENTO: Involucin excepto en las acuminadas. Es una crueldad innecesaria traumatizar a un nio quemndole sus verrugas con
cidos mediante electrofulguracin. Se han visto antiestticas cicatrices, a veces queloides como resultado de estas iatrognicas
maniobras. La destruccin de pocas lesiones puede hacerse con electrofulguracin o congelndolas con nitrgeno lquido. Las
acuminadas requieren aplicacin de soluciones de podofilina al 20-40% en solucin alcohlica o en aceite, protegiendo tejidos vecinos
con vaselina o colodin. Igualmente son molestas las elctrofulguraciones de las verrugas plantares, pues solo se destruye una mnima
porcin y es intil poner al paciente en cama varios das, cuando la recidiva es la regla. Ya se sabe que las verrugas tienden a la
involucin espontneamente, est probado cientficamente y con estudios estadsticos, la utilizacin de los placebos (en dosis mnimas,
MANUAL DE TRABAJO DEL CURSO ENARM CMN SIGLO XXI
CURSO ENARM CMN SIGLO XXI TEL: 36246001 Pharmed Solutions Institute PGINA 395

magnesia calcinada, glucosa, cloruro de sodio, BCG, levamisol) en estas afecciones tan orgnicas Debe tenerse en mente que las
verrugas virales tarde o temprano desaparecern, no as las cicatrices que perduran toda la vida y recordaran al paciente los malos
tratamiento recibidos.

CASO CLINICO
Se trata de masculino de 17 aos con dermatosis localizada en cara, rea de la barba con extensin al cuallo constituida por numerosas
neoformaciones (90) que varia de 1 a 3 mm de dimetro con proyecciones digitiformes que dan aspecto verrugoso y papiliforme, del
color de la piel, solo presentaba prurito ocasional, estudios de rutina dentro de parmetros normales.

PREGUNTA
Cual es la consuta a seguir mas adecuada?

RESPUESTA
a.- Electrofulguracion.
b.- Nitrgeno lquido.
c.- Podofilina.
d.- 5-fluorouracilo.

MOLUSCO CONTAGIOSO (MC). CIENCIAS BASICAS: El molluscum contagiosum (MC) es una patologa cutnea benigna ocasionada por
un virus DNA de la familia de los poxvirus. A pesar de tener una distribucin por todo el mundo, su morbilidad es baja. No obstante,
actualmente existen ms individuos afectos al aadirse los pacientes inmunocomprometidos. SALUD PUBLICA: El MC tiene una
distribucin mundial y una incidencia de 4,5%. No tiene predileccin racial o entre gneros. Los estudios epidemiolgicos realizados
sugieren que la transmisin podra estar relacionada con factores tales como la humedad y calor del clima, la pobreza, hacinamiento y
la falta de higiene. Sin embargo, en otros trabajos se apoya ms la idea del contacto entre personas, ya que se ha encontrado que se
puede desarrollar en el 35% de los miembros expuestos de una familia y que en la mayora de los casos las familias poseen excelentes
estndares de higiene. En nios la transmisin se produce por contacto piel-piel, por fmites o por auto inoculacin. La infeccin por
MC ocurre en todos los grupos etreos, y al parecer ha venido aumentando en la ltima dcada. La mayor incidencia se observa en
nios menores de 5 aos, por el mayor contacto fsico casual entre ellos, y a la auto inoculacin. La infeccin en lactantes es rara, esto
se explica por la persistencia de anticuerpos maternos. Otro pico en la incidencia se observa en adultos jvenes, debido a la
propagacin secundaria al contacto sexual. El MC est incrementndose cada vez ms dentro de las enfermedades diagnosticadas en la
poblacin sexualmente activa. La transmisin en adultos normalmente es va sexual. Esta va se apoya en la localizacin mayoritaria
encontrada en el rea genital, la frecuencia de encontrar las parejas afectadas y la coexistencia de otras enfermedades de transmisin
sexual entre los pacientes afectos de MC. Otras vas posibles han sido a travs de saunas, masajes, ciruga. Desde 1983 se han recogido
diversos casos de MC en pacientes con infeccin HIV. La prevalencia en este grupo de poblacin es tan alta como del 5-18% .
PATOGENIA: El molluscum contagiosum es producido por un miembro del grupo de los poxvirus, no clasificado, de ultraestructura
anloga a la del virus de la viruela. El virus del MC es uno de los ms grandes que causan enfermedad en los humanos, el vibrin
maduro es una partcula que mide 150x350 mm2. El genoma, al igual que en los dems poxvirus, es una nica molcula linear de DNA
de doble hlice. Estudios recientes del DNA de los molluscum aislados confirman la presencia de dos diferentes subtipos, MCV I y MCV
II, con genomas de 185 kb y 195 kb respectivamente. Las lesiones producidas por cada subtipo son indistinguibles. No existen
diferencias en cuanto a la distribucin de ambos subtipos, encontrndose ambos a nivel de lesiones genitales y fuera de esta zona, pero
no se hallan MCV tipo II en ningn paciente de edad menor a 15 aos. Todos los MC aislados de un mismo paciente pertenecen al
mismo genotipo as como los aislados de los miembros infectados de su familia. Se ha observado la existencia de MC en pacientes
inmunocomprometidos. La mayora de los pacientes muestran una deficiencia en su funcin o nmero absoluto de linfocitos T. Un
grupo heterogneo de enfermedades se han descrito en asociacin al MC: atopia, neoplasias, estados yatrgenos de inmunosupresin,
los pacientes atpicos un incremento en la susceptibilidad a la infeccin cutnea viral poda estar relacionado con la piel eccematosa
que abrira una puerta de entrada a la colonizacin vrica. DIAGNOSTICO: El periodo de incubacin de la infeccin es de 14 a 50 das,
aunque hay datos de recin nacidos con lesiones al cabo de 7 das postparto. Las lesiones se inician como neoformaciones que miden
generalmente de 2 a 6 mm, aunque pueden llegar a medir 3 cm, son hemisfricas, cupuliformes, lisas, del color de la piel o perladas,
algunas (20%) tienen una umbilicacin central; la base es levemente eritematosa y son de consistencia firme, cuando se rompen. Se
localizan en cualquier parte del cuerpo e incluso pueden afectar mucosas, generalmente se agrupan en un rea especfica, pero pueden
estar diseminadas en personas infectadas con el virus de la inmunodeficiencia humana adquirida, siendo un marcador de enfermedad
avanzada, el material es blanquecino y cremoso. En el caso de los nios las lesiones se localizan normalmente en cara, tronco, brazos y
piernas a diferencia de los adultos jvenes en quienes el molusco contagioso se adquiere por transmisin sexual, las lesiones tienen
predileccin por genitales, abdomen y cara interna de los muslos. Las lesiones del molusco suelen aparecer entre los 14 das a 6 meses
despus de la exposicin, hay datos de recin nacidos con lesiones al cabo de 7 das postparto. Se pueden propagar por auto
inoculacin, pero es tpico que se resuelvan espontneamente en pocos meses. Las lesiones son asintomticas en la mayora de los
pacientes, aunque en el 10% de los casos puede haber prurito y desarrollarse una reaccin eccematosa. El diagnstico se hace
clnicamente y en algunos casos dudosos puede efectuarse biopsia con tincin H-E donde se encuentran los cuerpos de molusco
(inclusiones intracitoplasmticas grandes) o de Hendersen-Paterson; el 90% de los pacientes posee Ac tipo IgG. Puede realizarse
microscopa electrnica, PCR, Elisa, e inmunohistoqumica. TRATAMIENTO: Enfermedad autolimitada que eventualmente se resolver
en la mayora de los pacientes pero en nios atpicos e inmunosuprimidos la evolucin es larga y trpida. Primera eleccin es
imiquimod 5% crema cada 2 dias hasta que desaparezcan las lesiones (aprox. 4 sem). Curetaje: Es un mtodo sencillo y disponible de
forma rpida. Se realiza con una cureta simple, doloroso por lo que se recomienda el uso de algn anestsico tpico previo a la
realizacin del mismo. Tiene la ventaja de proveer de una muestra de tejido para la confirmacin del diagnstico en caso de dudas.
Criociruga: Es un mtodo eficiente y rpido para el tratamiento de las lesiones. El nitrgeno lquido se aplica sobre cada lesin por
pocos segundos. Se repiten las sesiones cada 2 a 3 semanas segn sea necesario. Entre las desventajas se encuentran el dolor, hiper o
MANUAL DE TRABAJO DEL CURSO ENARM CMN SIGLO XXI
CURSO ENARM CMN SIGLO XXI TEL: 36246001 Pharmed Solutions Institute PGINA 396

hipo pigmentacin y cicatriz residual. La podofilina se aplica en tintura al 25% en alcohol una vez a la semana en cada lesin. Los
efectos secundarios descritos son locales como erosin de la piel sana circundante y sistmicos como dao renal, neuropata, leo
paraltico, leucopenia y trombocitopenia. El podofilotoxina es uno de los componentes activos de la resina de podofilina, es ms seguro
que la podofilina y puede ser usado por el paciente en casa. La aplicacin recomendada es 0,05 ml de podofilotoxina al 5% en etanol
tamponado en lactato dos veces al da por 3 das. Est contraindicado durante el embarazo. Se coloca sobre las ppulas una solucin
de yodo al 10% y al secar se cubren con cido saliclico al 50% y cubren con adhesivo. El proceso se repite cada 24 horas hasta que las
ppulas se tornen eritematosas lo cual ocurre entre el 3er da y el 7mo, despus de lo cual solo se coloca solucin yodada. Se ha
reportado resolucin de las lesiones con una media de 26 das. Puede ocurrir maceracin y erosin.

CASO CLINICO
Un varn de 10 aos de edad, nacido del matrimonio consanguneo fue llevado con mltiples lesiones elevadas blancas como perlas en
todo el cuerpo desde hace 6 meses, asintomticas que se fueron generalizando. No haba antecedentes de mltiples y recurrentes
lesiones llenas de pus en el pecho, las ingles, junto con infecciones pulmonares recurrentes. No haba antecedentes personales o
familiares de atopia. No haba antecedentes de ningn dentales, problemas seos, neurolgicos o cualquier facies distintivas. De los
tres hermanos, un hermano ms joven tena antecedentes de lesiones de la piel similar en todo el cuerpo.

PREGUNTA
Cual es la conducta a seguir.

RESPUESTA
a.- Crioterapia y curetaje.
b.- Conducta expectante.
c.- Nitrito acidificado.
d.- Imiquimod tpico.

ESCABIASIS. CIENCIAS BASICAS: Tambin llamada sarcoptosis, sarna. Es una enfermedad altamente transmisible, cosmopolita, cuyo
agente etiolgico es el caro Sarcoptes scabiei var. hominis, del grupo Arachnida, del orden Astigmata. Los factores ms importantes en
la transmisin son la pobreza, hacinamiento y la promiscuidad sexual. Se le asocia a estratos socioeconmicos bajos en los que adems
es frecuente observar higiene y nutricin deficientes. SALUD PUBLICA: Se transmite principalmente por contacto directo, estrecho, sin
descartar la evidencia de transmisin por fomites (por ejemplo, ropa de cama o personal en infestaciones severas) y contacto sexual.
Debe tenerse en cuenta el contacto frecuente en algunas instituciones (asilos, escuelas, cuarteles, hospitales). Los perros y gatos y
otros animales tambin padecen la parasitosis. Los animales de compaa con S. scabiei var canis pueden ser responsables de
infestacin y sensibilizacin, causando una dermatitis papular limitada, aunque generalmente existe especificidad de especie.
PATOGENIA: El artrpodo adulto mide 1 - 3 mm de longitud, tiene un cuerpo aplanado, ovalado y 4 pares de patas. La hembra
fertilizada se instala en la superficie de la piel y en un lapso de tiempo menor a una hora excava un tnel en el estrato crneo y
granuloso. Puede desplazarse unos 2 - 5 cm/min sobre la piel clida. El promedio de vida del artrpodo es de unos 25 - 30 das, durante
los cuales el tnel excavado puede extenderse unos milmetros o hasta centmetros. El caro deposita durante su avance 2- 5
huevos/da y pequeas pelotitas de excremento, oscuras y ovales, irritantes. La respuesta inflamatoria y los productos del ectoparsito
son responsables, en gran medida, del prurito. Las larvas liberadas alcanzan la madurez unos 15 das despus de la oviposicin; los
nuevos adultos copulan y el ciclo se repite; al cabo de varias semanas la infestacin todava es baja y los signos y sntomas leves. Se
requieren unos 15 - 20 caros adultos y su diseminacin para provocar el intenso prurito. DIAGNOSTICO: Los sntomas iniciales son
leves y habitualmente se atribuyen a la picadura de algn insecto. El prurito es un sntoma cardinal, con agravamiento nocturno
(reaccin de hipersensibilidad a saliva, huevos, materia fecal de los caros). El rascado, que inicialmente alivia al paciente porque
destruye los tneles y algunos parsitos se erige como una forma de diseminacin de los mismos (y elimina las lesiones primarias,
constituidas por tneles y vesculas escasas y puntiformes, con caros, y ppulas pequeas). Los tneles son delgados, elevados,
curvados o en forma de S y miden 2 - 20 mm de longitud. Su localizacin ms frecuente se encuentra en pliegues interdigitales,
muecas, en los bordes de manos y pies, superficies extensoras de codos y rodillas, pene, escroto, pliegues mamarios e incluso
pezones, glteos, axilas, cintura y pliegues poplteos (zonas incluidas por las "lneas de Hebra", divisiones imaginarias a nivel de
hombros y rodillas). En nios pequeos tambin se ubican con frecuencia en palmas de las manos, plantas de los pies, cabeza y cuello,
con vesculas y pstulas. Adems pueden presentar ndulos en la zona del paal y en la axilar. En adultos tambin es posible identificar
ndulos escabiticos. Las lesiones secundarias dominan el cuadro clnico y son ocasionadas por el rascado, infeccin bacteriana
secundaria y la automedicacin: pstulas, excoriaciones, eccema, costras. Se desarrollan ndulos en codos, axilas, pene, escroto (masas
firmes, oscuras, que persisten durante meses an en ausencia de infestacin activa). Datos de eczema inflamatorio se atribuyen a la
automedicacin. Diagnstico epidemiolgico: Cuadros similares en el ambiente familiar o comunal, en instituciones. Localizacin de
lesiones y morfologa de las lesiones. Biopsia de piel. Infiltrado inflamatorio compuesto por eosinfilos, linfocitos, histiocitos. Pueden
llegar a observarse larvas, ninfas, hembras. Se ha reportado el empleo de PCR en pacientes con probable escabiasis y exmenes
dermatolgico y microscpico negativos. TRATAMIENTO: Debe ser aplicado a todos los miembros de la familia y contactos cercanos.
Incluye aseo personal y de la ropa. Se sugiere lavar la ropa en agua caliente y guardar en bolsas de plstico aqullo que no pueda
lavarse durante 2 semanas. Antibiticos por va sistmica en el tratamiento de imptigo bacteriano secundario y anthistamnicos para
el control del prurito. La dosis nica de ivermectina es efectiva. En casos severos se aconsejan hasta tres administraciones, c/1 - 2
semanas. Los agentes tpicos incluyen: Permetrina en crema o emulsin al 5% (Scabisan), benzoato de bencilo en emulsin al 25% para
adultos (Hastilan), crotamitn en crema (Eurax), muy eficaz ante el prurito. El lindano ha sido prohibido en varios pases. La FDA reporta
que el 70% de las reacciones adversas por lindano son de origen neurolgico, e incluyen ataxia, desorientacin, tremores, convulsiones
y muerte. La exposicin crnica deriva en efectos sistmicos graves y/o mortales. En pacientes con escabiasis noruega se utiliza
ivermectina y tratamiento tpico simultneamente, anthistamnicos y emolientes. El tratamiento es difcil, con recadas. Debe
monitorearse a los pacientes. El prurito puede persistir en las 2 - 4 semanas posteriores, lo cual puede significar: tratamiento exitoso,
MANUAL DE TRABAJO DEL CURSO ENARM CMN SIGLO XXI
CURSO ENARM CMN SIGLO XXI TEL: 36246001 Pharmed Solutions Institute PGINA 397

recada, nueva infeccin. SARCOPTOSIS COSTROSA O NORUEGA: Se presenta en sujetos con inmunodepresin (SIDA, virus linfotrpico
o leucemia), ancianos institucionalizados, en pacientes con trastornos neurolgicos, principalmente sndrome de Down, demencia,
trastornos nutricionales y enfermedades infecciosas. Las lesiones, hiperqueratsicas, son extensas, semejantes a las que se presentan
en la psoriasis, con escaso prurito en la mayor parte de los casos. Hay formacin de costras gruesas que involucran manos y pies,
escamas grisceas y costras en tronco y extremidades, descamacin en la zona facial, lesiones semejantes a verrugas en dedos y sitios
de trauma, y cada profusa de cabello. La observacin de escamas aclaradas con hidrxido de potasio permite apreciar abundantes
caros en todas las fases de crecimiento. Un cuadro que se presenta con cierta frecuencia, es el de "las personas limpias", quienes
presentan lesiones escasas, en pliegues. Un buen nmero de pacientes presentan ttulos elevados de IgE, eosinofilia, activacin de
mastocitos y una reaccin del tipo de hipersensibilidad inmediata. Ante una reinfestacin los sntomas aparecen con mayor rapidez.
COMPLICACIONES: imptigo secundario, hiperinfeccin. Las infecciones bacterianas secundarias se relacionan con frecuencia con
Streptococcus del grupo A y Sytaphylococcus aureus, por lo que se estima que la ectoparasitosis es un factor de riesgo importante en
zonas endmicas en el desarrollo de glomerulonefritis, fiebre reumtica, celulitis y otras infecciones invasivas.

CASO CLINICO
Paciente de sexo masculino, de 15 meses de edad, previamente sano. Consulta a su pediatra de cabecera por presentar ppulas
eritematosas en mueca izquierda. Algunas de ellas se tornaron costrosas y, en pocos das, se sumaron ppulas, placas y pequeos
ndulos eritematosos con escamas en axila derecha, regin supraumbilical y axila izquierda. Se encontraba en buen estado general y
afebril.

PREGUNTA
Cual de las siguientes medidas es conveniente prescribir como final de tratamiento mas adecuado?

RESPUESTA
a.- Cefalexina por va oral
b.- Permetrina al 5% (crema fluida)
c.- Hidroxicina 2 mg/kg/da 3 veces/da.
d.- Aplicacin diaria de mometasona (crema) en los ndulos.

PEDICULOSIS. CIENCIAS BASICAS: Es una ectoparasitosis ocasionada por los piojos, los cuales no reconocen barreras geogrcas,
socioeconmicas ni culturales. Es la parasitacin del hombre y los animales por insectos del genero pediculus del cual existen 3
principales: el P. capitis (pediculosis de piel cabelluda), el P. vestimenti (pediculosis del vestido y el cuerpo) y el P. pubis o ladilla (en
vello pbico). El piojo se alimenta succionando sangre; su saliva contienen sustancias vasodilatadoras y anticoagulantes. Los huevos del
piojo, conocidos como liendres, estn finamente adheridas a pelos individuales. SALUD PUBLICA: Es un problema cosmopolita. Su alta
prevalencia ha sido relacionada, con la limitada disposicin de agua y deficientes prcticas de aseo personal. Hay preferencia con el
sexo femenino relacionado con el cabello largo, ya que estos facilitan las formas de transmisin. Aproximadamente 6-12 millones de
casos/ao se presentan entre nios de 3-12 aos de edad en Estados Unidos. En Mxico se report una prevalencia entre 18-33%, en
algunas poblaciones. PEDICULOSIS DE PIEL CABELLUDA: El llamado piojo negro o gris es el causante y parasita sobre toda la piel
cabelluda aunque puede observarse en la barba y el bigote. Es ms frecuente en nios entre 5 y 15 aos de edad. El piojo en realizad es
gris, pero toma a menudo el color del cabello que parasita y pone sus huevecillos que originan larvas (liendres), las cuales se adhieren
fuertemente al pelo y son difciles de desprender. El piojo mide unos 3mm y su cuerpo es alargado, se sita sobre todo en la regin
occipital y sus movimientos originan intenso prurito, lo cual provoca no en pocas ocasiones un imptigo secundario en esta regin que
es a menudo el motivo de consulta, por ello ante un imptigo sobre todo en nio, en la regin occipital debe pensarse en la posibilidad
de pediculosis previa. Deben diferenciarse las liendres de simples escamas que en general se dejan atravesar por el pelo, mientras que
la liendre est adherida a su costado y avanza con el pelo al crecer este. La hembra dura poco menos de 30 das, pero en ese tiempo es
capaz de depositar entre 50-150 huevecillos. PEDICULOSIS DEL CUERPO Y DEL VESTIDO: En realidad los piojos llamados blancos se
encuentran en la ropa de personas sucias, vagabundos, alcohlicos, que nunca se baan, ni cambian de ropa y ah mismo el piojo
deposita sus huevos que pueden observarse junto con los adultos en las costuras de la ropa. Las lesiones que producen son paulas,
costras hemticas, manchas hipercromicas en abdomen, nalgas, muslos, con intenso prurito. El imptigo y la eczematizacin son
complicaciones habituales. El piojo blanco transmite el tifo exantemico y otras ricketsias. PEDICULOSIS DEL PUBIS: El P. pubis
vulgarmente llamado ladilla, es ms corto y ancho que los anteriores y en sus extremidades presenta unos ganchos que le dan aspecto
de cangrejo y que lo adhieren fuertemente al pelo del pubis, perin, pliegue intergluteo y en ocasiones puede subir hasta la ceja,
pestaas y vello axilar. El parasito es sedentario, se mueve poco, se alimenta continuamente y deposita sus deyecciones en el sitio y en
la ropa interior por lo que el paciente cuando llega habitualmente al consultorio ya tienen el diagnostico, porque ha sentido prurito en
la regin del pubis y hasta ha logrado visualizar y extraer el piojo. Al examinar la regin se observan los parsitos y las liendres y en la
trusa o pantaleta se puede observar un fino puntillo producido por el depsito de las deyecciones del piojo (signo de la trusa) dato muy
seguro para el diagnstico. En la piel se pueden ver costras hemticas y manchas hemorrgicas de color azulado de medio a un cm,
llamadas manchas cerleas. La hembra dura unos 30 das y llega a depositar en ese tiempo unos 30huevecillos que se transformaran
en adultos. La transmisin es en la mayora de las veces por transmisin sexual. TRATAMIENTO: Sera necesario destruir los parsitos
adultos sin olvidar las liendres que no son fcilmente desprendibles del pelo, de otro modo en pocos das, estas darn origen a nuevos
adultos. La permetrina al 1% en locin o crema, es el tratamiento de eleccin para pediculosis, debido a su eficacia y ausencia de
toxicidad, esta acta como una neurotoxina, bloquea los canales de sodio ocasionando parlisis del sistema nervioso y
musculoesqueletico e impidiendo la respiracin del piojo. La piretrina mas el piperonyl es un insecticida de origen vegetal extrado del
crisantemo, el cual presenta un efecto ovicida del 70% y pediculicida del 97%. Se debe repetir 7 das para asegurar su eficacia. Los
adultos pueden destruirse con DDT, benzoato de bencilo al 25%, gamexano o lo ms til y menos irritante: vaselina con xilol que los
ahoga rpidamente. Se aplica tal mezcla en la cabeza o en el pubis y se deja toda la noche con bao al da siguiente. Para destruir las
MANUAL DE TRABAJO DEL CURSO ENARM CMN SIGLO XXI
CURSO ENARM CMN SIGLO XXI TEL: 36246001 Pharmed Solutions Institute PGINA 398

liendres es necesario desprenderlas mediante la aplicacin con fomentos de cido actico o simplemente vinagre blanco en agua, que
disuelve la sustancia que las adhiere al pelo. Al da siguiente bao y peinado con el peine de dientes apretados especial para piojos.

CASO CLINICO
Nia de 2 aos que tras una extensa pediculosis en la cabeza tratada con 2 ciclos de permetrina al 1% en locin acude por parasitacin
en las pestaas. La exploracin mediante lmpara de hendidura pone de manifiesto la presencia de algunas liendres y de un piojo
maduro (Pediculus humanus capitis) adheridos a las pestaas. Se realiza retirada directa de gran parte de los parsitos y se inicia
tratamiento con vaselina en pomada 3 veces al da durante una semana. No vuelve a presentar recurrencias tras el tratamiento.

PREGUNTA
Cual es la conducta a seguir.

RESPUESTA
a.- Permentina 1%.
b.- Piretroides.
c.- Permetrina 5 %.
d.- Malathion 0.5 %.

TIAS. CIENCIAS BASICAS: Bajo el trmino dermatofitosis se engloban una serie de dermatosis producidas por un grupo determinado
de hongos que se conocen como dermatofitos y que se caracterizan por parasitar, y digerir, las estructuras crneas (piel, uas y pelo).
Tambin se conocen como micosis superficiales o tias y comprenden seis grupos, en funcin del rea corporal afecta. Generos:
Trichopyton, Microsporum, Epidermophyton. SALUD PUBLICA: T. rubrum 36-52%, M. canis 14-24%, T. tonsurans 15-18%. El cpontagio
es persona a persona, por animales, artculos contaminados a travs del suelo y albercas. TIA DE LA CABEZA: La ms frecuente en
nios. Afecta a los cabellos de cuero cabelludo que se rompen a nivel del ostium folicular lo que determina zonas de alopecia (tias
tonsurantes). La afectacin puede realizarse implicando a todos los pelos de una zona determinada (tias tonsurantes microspricas) o
a unos pelos s y otros no (tias tonsurantes tricofticas). Estas formas clnicas afectan predominantemente a nios, curan
habitualmente en la pubertad y no dejan lesiones residuales. La tia fvica o favus se caracteriza por estar producida por T. schnleini, y
producir una inflamacin folicular profunda que desemboca en una alopecia cicatricial. En contra de lo que ocurre en las formas
tonsurantes el favus no cura con la pubertad. Es poco frecuente en pases desarrollados pero la emigracin determina la aparicin de
casos en estas zonas. Existen procesos inflamatorios en forma de foliculitis supurativa, no exclusiva de cuero cabelludo, conocidas como
Querion de Celso, que estn producidas por hongos zooflicos (T. rubrum en 75%) y que presentan exudacin abundante a nivel
folicular especialmente cuando se presiona la placa (signo de la espumadera). Dependiendo de la intensidad del componente
inflamatorio podremos encontrar, o no, la existencia de alopecia cicatricial. El tratamiento de primera eleccin es la terbinafina. Nios
10mg/Kg/dia por 6 semanas. Otra opcin es itraconazol VO 5mg/da por 4 semanas. TIA DEL CUERPO: Cualquier zona corporal no
incluida en esta clasificacin puede incluirse en este grupo. Su clnica es muy evocadora en forma de lesiones papulosas que tienden a
crecer de forma excntrica determinando crculos en los que existe una zona central sana o discretamente escamosa, y un borde
inflamatorio, activo, papuloso o pustuloso. Al igual que en cuero cabelludo pueden existir formas inflamatorias cuando el agente
etiolgico corresponde a un hongo de origen animal. Se puede usar terbinafina tpica, cada 24 hrs por 4 semanas. Alkternativa,
miconazol. TIA CRURAL: Se caracteriza por la aparicin de placas anulares, semejantes a las descritas en la tia corporal, pero que
tienden a localizarse a nivel inguinal. Es excepcional en nios pero s suele afectar a jvenes que practican deporte de forma habitual, y
no es rara su asociacin a tia de los pies. Su posible transmisin por contacto sexual es siempre necesario tenerla en cuenta. TIA DE
LAS MANOS: Es un proceso poco frecuente, y excepcional en la edad peditrica, que casi siempre aparece en mujeres con trastornos
circulatorios, diabetes o en profesiones que maceran sus manos bien por el uso continuado de guantes o por precisar mantenerlas en
agua. La forma clnica ms frecuente es la aparicin de descamacin y maceracin interdigital, no obstante pueden aparecer cuadros de
descamacin palmar. Sin duda alguna las levaduras son ms frecuentes en las micosis de manos que los dermatofitos. Tratamiento con
terbinafina tpica. 2 semanas. TIA DE LOS PIES: Rara en la etapa infantil pero frecuente en la pubertad, y en ocasiones asociada a tia
crural. Es debida a la parasitacin por dermatofitos en el pie pudiendo adoptar morfologas variables. Forma interdigital (pliegues de
4to y 5to dedo): incluida dentro del cuadro genrico de pie de atleta se caracteriza por la aparicin de descamacin y fisuracin a
nivel de los espacios interdigitales. El proceso puede ser poco llamativo cuando la infeccin es exclusiva de dermatofitos (Tinea pedis
simplex) o ser muy inflamatoria o exudativa cuando existe infeccin combinada de dermatofitos y bacterias (Tinea peds complex).
Examen microscpico directo con hidrxido de potasio 20%; observamos filamento slargos delgados o gruesos, artroconidias,
blastoconidias, onicomicosis candida. Tratamiento con ternbinafina tpica por 2 semanas. TIA DE LAS UAS: Es un proceso habitual
en las consultas de dermatologa peditrica. El patrn diagnstico es la presencia de una hiperqueratosis subungueal que determina un
levantamiento de la ua del lecho (onicolisis). Generalmente la lesin comienza en el pliegue subungueal distal (onicomicosis
subungueal distal), aunque puede hacerlo en el proximal (onicomicosis subungueal proximal) o incluso acabar afectando a toda la ua
(onicomicosis distrfica total). En contadas ocasiones se produce una parasitacin exclusivamente de la lmina ungueal que presenta
una coloracin blanquecina (Onicomicosis blanca superficial).

CASO CLINICO
Nia de 7 aos, que consult de urgencias por aparicin progresiva en los ltimos 10 das de una placa anular eritemato-descamativa
muy pruriginosa, de 3 x 3cm de dimetro y crecimiento centrfugo, con borde levemente sobreelevado. Clnicamente planteaba el
diagnstico diferencial entre una tinea corporis y un eczema numular. Para confirmar el diagnstico se realiz un examen directo de
escamas cutneas, obtenidas despus de raspar el borde de la lesin con un bistur e incubadas con 0,5ml de KOH al 30% sobre una
lmina portaobjetos.

PREGUNTA
MANUAL DE TRABAJO DEL CURSO ENARM CMN SIGLO XXI
CURSO ENARM CMN SIGLO XXI TEL: 36246001 Pharmed Solutions Institute PGINA 399

Cuales son los factores de riesgo ms importante para sospechar de este padecimiento.

RESPUESTA
a.- Deportes y recreacin en equipo.
b.- Exposicin a lugares hmedos sin protecin
c.- Compartir objetos personales.
d.- Uso de ropa sintetica.

PREGUNTA
Cual de los siguientes factores de riesgo es el ms frecuente que estn relacionados con las tias.

RESPUESTA
a.- Uso de corticoides.
b.- Inmunocompromiso.
c.- Queratodermia.
d.- Ictiositocis hereditaria.

PREGUNTA
Cual es la conducta teraputica ms adecuada.

RESPUESTA
a.- Terbinafina 250 mg por 12 semanas.
b.- Itraconazol 10 mg/Kg/dia
c.- Fluconazol 150 mg VO por 24 semanas.
d.- Miconazol crema al 2 %

PREGUNTA
Al revalorar la paciente posteriormente la paciente continua con el cuadro clnico con complicaciones, cual de las siguientes no es
indicativo de envio a segundo nivel.

RESPUESTA
a.- Diseminacion dentro de las 4 primeras semanas.
b.- Presencia de onicomicosis comorbida.
c.- Alteracin de enzimas hepticas.
d.- Inmunosupresion de origen idioptico.

PTIRIASIS VERSICOLOR. CIENCIAS BASICAS: Es una infeccin mictica del estrato crneo de la piel, es la micosis ms superficial que se
conoce, causada por Malassezia furfur (ptiriosporum furfur), caracterizada por lesiones discrmicas, que pueden manifestarse como
manchas hipercrmicas o hipocrmicas irregulares y en ciertos casos, de manera vitiligoide, variedad conocida como acromiante, todas
con descamacin fina. Adems del clima, se han sealado otros factores predisponentes como aplicacin local de corticoesteriodes,
desnutricin, recambio lento de la epidermis y predisposicin gentica. SALUD PBLICA: La infeccin se presenta a partir de la
adolescencia y es raro encontrarla en personas de edad avanzada. Esta micosis es ms frecuente en personas que habitan lugares de
clima clido y hmedo, tales como las regiones costeras tropicales, en donde la frecuencia puede llegar a ser hasta del 50%. No existe
predominio de gnero. PATOGENIA: El agente etiolgico es M. globosa que puede encontrarse como especie nica o asociada con a
otras especies, entre ellas M. sympodialis, M. slooffiae. DIAGNOSTICO: Las lesiones generalmente son asintomticas, se inician como
discretas manchas eritematosas cubiertas con una escama muy delgada (como salvado), que pronto toman dos aspectos
hipocromiantes o hipercromiantes. En Mxico predomina la primera, son manchas un poco ms claras que la piel raras veces
acromicas, lenticulraes confluentes, bien delimitadas de bordes no activos como deshilachados, con esa fina escama y prcticamente
sin prurito. Se localiza con mayor frecuencia en el tronco, cuello, cara, piel cabelluda y los brazos, aunque se pueden observar en otras
regiones corporales de acuerdo a los factores predisponentes de cada paciente. Tcnicas para la toma de muestra: raspado de la lesin,
de preferencia de la periferia. Se pueden utilizar dos portaobjetos estriles, uno para raspar y el otro para recibir las escamas o bien se
puede emplear una caja de Petri estril. Producto biolgico: escamas. Observacin microscpica: hidrxido de potasio del 15-30%, azul
de algodn, tinta o tinta azul de marca Parker. En escamas de lesiones de pitiriasis versicolor la morfologa de las estructuras
parasitarias es diagnstica de esta enfermedad. Las levaduras son esfricas de 2-8 m de dimetro, agrupadas, asociadas con hifas de
10-25 m de largo y 2-5 m de ancho, las hifas pueden estar alineadas o ramificadas (conocida imagen de spagueti con albndigas). El
cultivo no es un procedimiento que se practique rutinariamente en el laboratorio clnico, sin embargo se pueden obtener los cultivos a
partir del producto biolgico. Cuando se toma una porcin de la colonia, es difcil preparar una suspensin o estriar sobre el agar, ya
que las clulas permanecen juntas formando pequeos grupos (o racimos). TRATAMIENTO: Ya que la mayora de patologas causadas o
asociadas a Malassezia son superficiales, el tratamiento tpico es lo ms recomendado. Habitualmente se recomiendan lociones,
cremas o jabones con cido acetil saliclico al 5% o azufre al 1-3%; ungento de Whitfield, hiposulfito de sodio al 20% en solucin
acuosa, tolnaftato, tolciclato, crema de piroxolamina al 1%; champ de disulfuro de selenio al 2.5%, con efecto antimictico,
bactericida y anfi-inflamatorio. Las lociones y cremas deben aplicarse diariamente durante 3 a 4 semanas. El champ se aplica
diariamente, dejando actuar algunos minutos antes de enjuagar el pelo, durante 2 a 3 semanas. Para casos graves, est indicado el
ketoconazol por va oral; uno de los esquemas es 400 mg en una sola dosis; o bien 200 mg diarios por 10 a 30 das dependiendo de la
gravedad del caso. El itraconazol tambin ha dado resultados satisfactorios: 100 a 200 mg/da durante 3 a 15 das, dependiendo de la
MANUAL DE TRABAJO DEL CURSO ENARM CMN SIGLO XXI
CURSO ENARM CMN SIGLO XXI TEL: 36246001 Pharmed Solutions Institute PGINA 400

extensin de las lesiones. En el caso de la pitiriasis versicolor, las manchas pueden persistir varios meses despus del tratamiento, por
lo que el paciente debe ser informado.

CASO CLINICO
Se trata de paciente masculino de 18 meses de edad el cual ha presentado diversos cuadros de dermatitis aatopica con importante
rascada, acude a consulta de control donde observa placas blanquecinas peribucales sin comprometer mucosa.
PREGUNTA
Considerando su diagnostico y estadio del padecimiento cual es conducta teraputica menos adecuada?

RESPUESTA
a.- Evitar sol e indica protectores solares.
b.- Baos cortos y con agua templada.
c.- Indica corticoide en crema.
d.- Indica tacrolimus.

CANDIDIASIS. CIENCIAS BASICAS: La candidosis o candidiasis es una micosis causada por diversas especies de levaduras del gnero
Candida. Cualquier tejido puede ser afectado por lo que se presentan diversos cuadros clnicos, cada uno de ellos asociado
directamente al estado inmunolgico del paciente. Las candidosis de mucosas y piel son las ms frecuentes, mientras que las sistmicas
son de evolucin aguda o crnica y generalmente severas. SALUD PUBLICA: La distribucin geogrfica de esta micosis es universal y
ms de 70 % de ellas son producidas por C. albicans observndose un porcentaje mayor por el serotipo B. Los casos de candidiasis
sistmica estn relacionados a pacientes con severas deficiencias en su sistema inmune. C. krusei y C. glabrata son habitualmente
resistentes a los compuestos azlicos y su hallazgo como agentes infecciosos involucrados en enfermedades sistmicas
intrahospitalarias ha aumentado en los ltimos aos. PATOGENIA: La C. albicans se ha considerado el ejemplo ms clsico de lo que es
un parasito oportunista, vive habitualmente en forma saprofita en las mucosas orales, nasal, vaginal, tracto gastroinetstinal, en cambio
no es habitual encontrarla en piel. El nio al nacer puede recibir este parasito al pasar por el tracto vaginal de la madre. Cuando se
convierte de saprofita a patgena, puede producir desde una simple algondoncillo hasta graves endocarditis o septicemia. Factores
predisponentes: 1. Fisiolgicos que suelen transformar el pH de las mucosas como la infancia y el embarazo. 2. Maceracin, humedad y
traumatismos, (pliegues interdigitales, submamarios, uas y rebordes ungueales). 3. Dermatosis inflamatorias previas (como dermatitis
del paal que se complica con candida). 4. Mal estado de la dentadura y prtesis. 5. Enfermedades metablicas (DM y obesidad). 6.
Enfermedades inmunosupresoras (leucemias, Enf. De Hodking, SIDA). 7. Medicamentos que alteran la flora bacteriana (corticoides,
citotxica). DIAGNOSTICO: La manifestacin ms conocida es el algodoncillo que aparece en la boca de RN por su bajo pH. Las lesiones
son placas cremosas, blanquecinas, como residuo de leche, que pueden afectar la mucosa de los carrillos, la lengua, el paladar, encas e
incluso invadir toda la boca hasta la trquea o salirse de la boca y producir fisuras cubiertas de material blanquecino en las comisuras
labiales. Estas lesiones son dolorosas e impiden la alimentacin al nio. La Candida albicans no es habitual en la piel, pero puede
producir enfermedad cuando aumenta la susceptibilidad del in dividuo. Las lesiones a parecen a nivel de los pliegues: interdigitales en
manos y pies, inguinales y submamarios, axilas, intergluteos y perin. Se trata de fisuras y erosiones eritematosas, maceracin, vescula
y pstulas y algunas costras y escamas. En los pies semejan lesiones de tia, frecuentemente producen mal olor y son pruriginosas. En
los nios pequeos contribuyen a formar la llamada dermatitis por paal que se observa sobre todo por la orina, la aplicacin de
pomadas, la maceracin, todo lo cual favorece el desarrollo de Candida. El nio presenta entonces en regiones inguinales, glteas y
genitales, extensas zonas eritematosas con vesculas y pstulas, costras y escamas con intenso ardor y prurito. La perionixis y la
afeccin de la ua se ha observado sobre todo en personas que mantienen mucho tiempo las manos dentro del agua, como
empacadores de frutas, pescados y mariscos. El reborde ungueal se observa inflamado, eritematoso, desprendido de la ua y esta se
afecta principiando de la matriz al borde libre hacia adentro. Poco aparecen estras en las uas, la cual se vuelve amarillenta y opaca y
se empieza a despulir. La lesiones son habitualmente superficiales, es poco frecuente que al igual que los dermatofitos se introduzcan
ms all de la capa cornea, dependiendo del estado inmunolgico del husped originndose granulomas tricofticos o candidsicos. El
simple hallazgo de Candida albicans no significa de ninguna manera su intervencin patgena, es necesario encontrarla en grandes
cantidades y correlacionar su hallazgo con lesiones clnicas. . Es fcil buscarla de manera directa, colocando material recolectado en un
portaobjetos con solucin de potasa al 20% y se podr encontrara las clsicas formas levaduriformes, que a veces producen
pseudofulamentos. El cultivo en medio Saboraud produce colonias tpicas de aspecto cremoso. TRATAMIENTO: Corregir enfermedad
de base. En boca usamos buches con agua de bicarbonato de sodio. Localmente puede usarse violeta de genciana al 1% que mancha
mucho o la nistatina, antibitico que solo es activo para levaduras de Candida. Los imidazoles tanto tpicos como sistmicos son
altamente efectivos en candidiosis: miconazol, ketoconazol, clotrimazol.

CASO CLINICO
Paciente femenino de 10 aos de edad en consulta por presentar engrosamiento y cambio de coloracin en la ua del pulgar derecho,
al examen fsico, en cuero cabelludo, se observan tras placas de alopcicas, con cicatrizacin en la periferia y escasas costras
serosanguinolentas en el centro, dos de estas placas se localizan alrededor del apice del crneo y la tercera en regin parietal derecha.
En cavidad oral y labios se observan placas blanquecinas que cubren el centro y los bordes de la lengua, asi como ambas comisuras
labiales; al remover estas placas se observa eritema y fisuras de mucosas.

PREGUNTA
Cual es la conducta a seguir mas adecuada?

RESPUESTA
a.- Itraconazol.
b.- Miconazol.
MANUAL DE TRABAJO DEL CURSO ENARM CMN SIGLO XXI
CURSO ENARM CMN SIGLO XXI TEL: 36246001 Pharmed Solutions Institute PGINA 401

c.- Ketoconazol.
d.- Clotrimazol.

MICETOMA. CIENCIAS BASICAS: Es una infeccin crnica de la piel y de los tejidos subyacentes con tendencia a afectar los huesos. Se
caracteriza por un aumento de volumen relativamente indoloro y fstulas a travs de las cuales se elimina pus y granos constituidos por
filamentos. Los agentes causales son de origen exgeno y pueden ser hongos (eumicetoma) o actinomicetales (actinomicetoma).
Agentes etiolgicos: a) Bacterias (ACTINOMICETALES); Nocardia brasiliensis 86.0 %. Actinomadura madurae 10.2 %. Streptomyces
somaliensis 1.3%. N. asteroides, A. pelletieri, N. otitidiscaviarus. b) Hongos (EUMICETOMA). Los hongos dematiaceos forman granos
negros visibles a simple vista, mientras que los de filamento hialino forman granos blancos o blanco-amarillentos. Hongos hialinos:
Acremonium falciforme, A. recifei, A. kiliense, Aspergillus flavus. Hongos dematiaceos: Curvularia geniculata, C. lunata, Exophiala
jeanselmei, Leptosphaeria senegalensis. PATOGENIA: El micetoma se adquiere por inoculacin traumtica de agentes etiolgicos a
travs de la piel. En Mxico existe gran nmero de cactceas, las espinas son un mecanismo de infeccin, que probablemente tengan
importancia en la transmisin de la enfermedad. Una vez que la bacteria o el hongo se localizan en los tejidos y sobrevive, se presenta
una inflamacin aguda a nivel local con llegada de PMN y activacin de macrfagos, aunque la mayora de bacterias son fagocitadas y
destruidas, algunas logran sobrevivir hasta 16 das y pueden reproducirse. Los agentes no destruidos proliferan y forman colonias en el
interior del tejido conocidas como "granos"; el desprendimiento de los pseudofilamentos de la periferia del grano ocasiona una
inflamacin continua con formacin de nuevas estructuras parasitarias y colecciones de pus, que al fusionarse forman los trayectos
fistulosos generalmente limitados por tejido fibroso; cuando las fstulas alcanzan la superficie, forman un ndulo que posteriormente se
reblandece y abre liberando pus conteniendo colonias parasitarias del agente. El tejido conjuntivo formado alrededor de los
microabscesos y de las fstulas produce una retraccin que ocasiona el aspecto deprimido de las fstulas antiguas. La presencia de micro
abscesos, fstulas, inflamacin, edema y fibrosis provoca externamente la deformidad y dureza del rea afectada e internamente
obstruccin de la circulacin por compresin mecnica y por arteritis. Estos cambios histolgicos influyen en la pobre respuesta
teraputica en los pacientes con evolucin prolongada. DIAGNOSTICO: Por la naturaleza saprofita de los agentes causales de micetoma
que posibilita la infeccin principalmente por traumatismo en pie o pierna, todas las casustica de frecuencia muestran un franco
predominio de afeccin a extremidades inferiores, pero de acuerdo con las costumbres de trabajo o de vestido y a las condiciones
socioeconmicas, los porcentajes de frecuencia presentados en cada rea topogrfica por diferentes autores, pueden tener alguna
variacin. Examen directo; La observacin del material purulento que drena a travs de las fstulas permite identificar los granos
macroscpicos en los eumicetomas y, al examen microscpico, en la mayora de los actinomicetomas. Cultivo; Cuando es posible, los
granos de actinomicetales se mezclan y se lavan con solucin salina isotnica (SSI), se centrifugan a 3000 rpm y posteriormente el
sedimento se siembra, el desarrollo de la mayora de los agentes se presenta despus de dos o tres semanas de incubacin a 25C. Otro
medio adecuado para su desarrollo es el de Lowenstein-Jensen que se incuba en las mismas condiciones. Las colonias tienen bordes
bien definidos e irregulares, aspecto creo o membranoso, generalmente de superficie plegada. Los cultivos de Nocardia spp. Son de
color blanco amarillento o anaranjado las colonias de A. madurae son blanco amarillentas, A. pelletieri forma colonias rojas y S.
somaliensis tienen color oscuro, algunas casi negras. La identificacin precisa de los agentes se realiza por medio de pruebas
bioqumicas. Los granos de eumicetoma se siembran y se incuban a 25C. La velocidad de crecimiento es variable y depende de cada
agente. As. Fusarium spp, Aspergillus spp. o Scedosporium apiospermum se desarrollan en menos de ocho das; mientras que
Madurella spp, Leptosphaeria sp y Pyrenochaeta romeroi son de crecimiento lento, es decir ms de tres semanas. Cuando las biopsias
se toman de sitios con gran actividad parasitaria, los granos de todos los agentes se visualizan con relativa facilidad con la tincin de
hematoxilina eosina (H-E). Los estudios radiolgicos no permiten establecer el diagnstico preciso de esta patologa, sin embargo son
de suma importancia para determinar el grado de afeccin sea, permiten evaluar la respuesta teraputica y emitir un pronstico. Los
cambios observables en estudios radiolgicos simples pueden ser en los casos iniciales nicamente periostitis, o bien, destrucciones
osteolticas extensas con formacin de geodas, destruccin de superficies articulares y lisis en huesos del pie que ocasionan el aspecto
de 'caries', datos conocidos desde las primeras descripciones del padecimiento y que se presentan en pacientes con varios aos de
evolucin. Otro estudio radiolgico necesario, principalmente en los pacientes con ms de cinco aos de evolucin del micetoma, es la
arteriografa del rea afectada. El aumento de volumen y la fibrosis, condicionan que algunas zonas del rea afectada tengan un aporte
sanguneo deficiente, de tal manera que este procedimiento es un valioso auxiliar para el pronstico. TRATAMIENTO: Debido a la
posibilidad de que los micetomas sean causados por hongos o por bacterias, antes de prescribir la teraputica es indispensable conocer
el tipo de agente causal (mictico o bacteriano); esto puede lograrse mediante a la observacin de las caractersticas de los granos en el
examen directo y en el estudio histolgico. Actinomicetoma; La primera eleccin es la combinacin de TMP/SMX 800 mg/160 mg c/12
hrs asociada a la administracin de 100 mg de diaminodifenilsulfona (DDS), cada 24 horas. La mayora de micetomas causados por
actinomicetales responden a este tratamiento y la mejora es evidente despus de dos meses de tratamiento. Los pacientes deben ser
valorados peridicamente debido a que estos medicamentos pueden ocasionar diversos efectos adversos, entre los que se encuentran
reacciones medicamentosas graves a nivel cutneo como es el sndrome de Stevens-Johnson, necrlisis epidrmica txica, aplasia
medular, anemia, intolerancia gstrica, alteraciones hepticas, etc. Como segunda eleccin se emplea la asociacin de TMP/SMX a la
dosis mencionada previamente en forma continua, y agregando amikacina, aminoglucsido que se administra por va intramuscular en
ciclos de 21 das de tratamiento a dosis de 7.5 mg/kg de peso cada 12 horas. Entonces, se suspende el aminoglucsido durante ocho
das. Considerar ahora el posible dao renal o auditivo causado por la amikacina; por lo tanto, antes de iniciar el tratamiento y despus
de cada ciclo de amikacina, se debe valorar la funcin de esos rganos. La curacin generalmente se alcanza con tres ciclos del
aminoglucsido, pero el TMP/SMX debe continuarse durante 12 a 24 meses de acuerdo con la evolucin clnica. Aunque el estudio de
sensibilidad antibacteriana debera ser una prctica habitual en todas las cepas aisladas de casos de actinomicetoma, estos estudios
generalmente no se realizan, dando como resultado que se utilicen tratamientos basados nicamente en la experiencia clnica. Estos
esquemas de tratamiento (cuasiempricos) generalmente mejoran el estado del paciente, pero en caso de no haber respuesta, retrasan
la curacin. Entre los antibiticos utilizados en estas condiciones tenemos: tetraciclina, amoxicilina, estreptomicina o los antifmicos
como la isonizida. Desde los primeros estudios inmunolgicos relacionados con actinomicetoma fue evidente que algunos pacientes
presentan inmunosupresin inespecfica severa, y no responden al tratamiento mdico aun cuando se les administren los antibiticos
adecuados. En estos casos es necesario valorar la respuesta inmunolgica y dar los antibiticos efectivos basados en un estudio de
MANUAL DE TRABAJO DEL CURSO ENARM CMN SIGLO XXI
CURSO ENARM CMN SIGLO XXI TEL: 36246001 Pharmed Solutions Institute PGINA 402

sensibilidad acompaados de un esquema teraputico inmunomodulador, como puede ser la administracin de antgeno bacteriano y
levamisol con lo que se logr la curacin en slo dos meses de un paciente con actinomicetoma causado por N. brasiliensis localizado
en el abdomen, despus de siete aos de evolucin sin respuesta al tratamiento tradicional. Se ha reportado la utilidad de la
oxigenoterapia hiperbrica en el tratamiento de un caso de micetoma sin afeccin sea, causado por N. brasiliensis; el paciente
present mejora a los cinco das de tratamiento y las fstulas cicatrizaron un mes despus del inicio de la terapi a. Eumicetoma: Este
tipo de micetoma representa un grave problema de tratamiento, ya que no existen drogas altamente especficas. Durante muchos aos
la medida teraputica empleada con mayor frecuencia fue la amputacin de la extremidad afectada. Posteriormente se hicieron
ensayos con griseofulvina a dosis de 500 a 1000 mg por da sin obtener curacin; con base en estudios de sensibilidad in vitro la
anfotericina B fue utilizada en el tratamiento de micetomas causados por M. grisea y M. mycetomatis. En opinin de algunos
investigadores, la fibrosis caracterstica que rodea los granos de los agentes etiolgicos, impide que se alcancen las concentraciones
adecuadas para la curacin y, en consecuencia, el tratamiento no es efectivo. El advenimiento de los compuestos azlicos proporcion
nuevas alternativas para el manejo de esta patologa, se hicieron pruebas con ketoconazol solo o asociado a ciruga; esta ltima
alternativa mejor el pronstico de manera substancial. En la ltima dcada, se ha utilizado el itraconazol a dosis de 300 a 400 mg por
da, reportndose algunos casos de curacin y otros con mejora. Algunos autores como Smith y Kutbi, mencionan que en casos de
eumicetoma de corta evolucin la ciruga acompaada de tratamiento antimictico es la nica posibilidad de curacin definitiva.

CASO CLINICO
Masculino de 8 aos, originario de zona rural de guerrero, fue llevado a consulta por una dermatosis localizada en el tronco que se
extendia al toras y a la cara anterior del hombro derecho. La dermatosis era de aspecto monomorfo y estaba constituida por algunas
fistulas, numerosos ndulos eritematoso de 0.3 a 2 cm aproximadamente, telangiectasias y costras sanguneas y meliceras que
formaban una placa mal circunscrita, de 6 x 4 aproximadamente, crnica y asintomtica. Se realizaron cultivo los cuales mostraron
colonias de aspecto yesoso, rogosas y de color blanco. La prueba de la hidrolisis de casena resulto positiva y la colonia se identifico
como Norcardia brasiliensis. No se observo compromiso oseo a los rayos x.

PREGUNTA
Cual es el manejo mas adecuado para el caso?

RESPUESTA
a.- Anfotericina.
b.- Trimetroprim-sulfametoxazol.
c.- Itraconazol.
d.- Clotrimazol.

URTICARIA. CIENCIAS BASICAS: Es una reaccin vascular de la piel, caracterizada por la presencia de habones y prurito intenso donde
ocurren procesos inflamatorios importantes, hay una alteracin del equilibrio de sustancias vasoactivas, causando vasodiltaacion y
edema a nivel de la piel (urticaria), o del tejido subcutneo (angioedema). En la mayora de los pacientes, una liberacin masiva de
histamina es la causa del desequilibrio. El nombre de urticaria
se deriva de la palabra ortiga, una planta cuyas hojas causan
lesiones urticariales pasajeras. SALUD PUBLICA: La prevalencia
de la urticaria-angioedema es de 10-20% de la poblacin lo
sufrir en su vida. En la edad peditrica, la urticaria aguda es la
forma que ms prevalece. CLASIFICACION: Urticaria Aguda <
de 6 semanas de evolucin. Urticaria Crnica > de 6 semanas
de evolucin. Urticaria Aguda Recurrente: Episodios repetidos
de brotes agudos con perodos asintomticos prolongados
entre ellos. PATOGENIA: Alimentos: Huevo, leche, nueces
man, mariscos, pescados, fresas, kiwi, otros. Drogas:
Penicilinas, ASA, AINES, vacunas, medios de contraste,
productos sanguneos. Insectos: Abejas, avispas, hormigas.
Infecciones: Parsitos, bacterias, virus, hongos. Estmulos
Fsicos: Frio, calor, presin, ejercicios, agua, sol. Aeroalergenos,
alrgenos de contacto: Polvo casero, plen, esporas, epitelios
de mamferos. Enfermedades sistmicas LES, ARJ, enfermedad
del suero, vasculitis, cncer, endocrinopatas, Enf celaca. Otros
A. Hereditario, amiloidosis, deficiencia de C3b, estrs,
idioptico. El evento central en la urticaria-angioedema es la
liberacin de histamina desde unos grnulos preformados en
las clulas cebadas. Existen mltiples factores que pueden provocar la degranulacion de las clulas cebadas. En la urticaria aguda, el
mecanismo principal es el de hipersensibilidad tipo I, IgE mediada, tambin conocida como la reaccin alrgica clsica. En las primeras
exposiciones que el paciente tuvo alrgeno se inici una respuesta Th2 mediada que resulto en la formacin de abundantes cantidades
de IgE especfica por parte de las clulas plasmticas especficas. Esta IgE es liberada a la circulacin sistmica y se fija de preferencia a
receptores de alta afinidad para IgE (FcR1), en la superficie de la clula cebada (CC). Un subsecuente contacto con el mismo alrgeno
provoca directamente el puenteo de dos IgE especificas vecinas en la membrana de la CC, y esto es la seal para la degranulacin de la
CC. As se liberan grandes cantidades de sustancias vasoactivas, especialmente histamina, pero tambin leucotrienos, factor derivado
de plaquetas. Las respuestas principales de estos mediadores son: prurito, vasodilatacin (eritema), aumento en la permeabilidad
vascular, reflejo axonal. La exposicin al alrgeno de la urticaria puede ser por cualquier va, incluyendo la oral, transdermica e
MANUAL DE TRABAJO DEL CURSO ENARM CMN SIGLO XXI
CURSO ENARM CMN SIGLO XXI TEL: 36246001 Pharmed Solutions Institute PGINA 403

inhalatoria. URTICARIA AGUDA: En nios la urticaria aguda es ms frecuente que la crnica. Es frecuentemente por una reaccin
alrgica mediada por IgE a infecciones agudas generalmente del TRS y drogas. Infecciones. Virus: Enterovirus, Parainfluenza 1,2,3, Virus
respirat. Sincitial, Adenovirus, rinovirus, Influenza A y B, VEB, CMV, Herpes simple, Parvovirus B19, Hepatitis A, B, C, Mycoplasma
Neumoniae. Parsitos: Oxiuro, Toxocara, Giardia lamblia. Bacterias: Estreptococo beta hemoltico, H. pylori. URTICARIA CRONICA: En la
Urticaria crnica los factores fsicos como presin y fro son la causa principal. Luego siguen infecciones: tracto urinario, tracto
respiratorio y las idiopticas. Existe un porcentaje de Ac antitiroideos antiperoxidasa positivos en nios con urticaria crnica y tambin
hay mayor incidencia de enfermedad celaca en estos nios. DIAGNOSTICO: La urticaria se identifica por lesiones tpicas que tienen 3
caracteristicas bsicas: consisten en una inflamacin central eritematosa, levemente elevada sobre el nivel de la piel, casi siempre
rodeada por una piel eritematosa (reflejo axonal). Las lesiones son en esencia evanescentes, con una duracin de menos de 24 hrts,
que al desaparecer dejan una piel intactas. Con frecuencia son intensamente pruriginosas. La urticaria puede tomar diferentes formas
variando de lesiones puntiformes de 2 milimetros (point lesions), que cubren grandes reas de la piel (tpico aspecto de urticaria
colinrgica), lesiones aisladas de 1-2 cm o hasta lesiones lesiones confluyentes de varios centmetros. Historia y examen fsico. Pruebas
para urticarias fsicas, prueba de suero autlogo, examen simple de orina y seriado de heces, hematologa completa y VSG, pruebas
tiroideas y Ac antitiroideos. Otros: segn cuadro clnico. TRATAMIENTO: 1. Medidas generales como evitar contacto con alrgeno,
eliminar el alrgeno si es posible: aditivos, alimentos, medicamentos, fro, calor, picaduras. Implementar dieta de eliminacin. 2.
Antihistamnicos: Primera lnea en el tratamiento de la urticaria. Actan por inhibicin competitiva de la histamina bloquean
receptores H1. Control del prurito. Primera generacin sedantes (Dexclorfeniramina 0.15-0.2 mg/da QID, Hidroxicina 2 mg/da QID,
Ciproheptadina 0.25 mg/da BID, Difenidramina 5 mg/dia TID) y Segunda generacin No sedantes (Loratadina: 1-2 aos 1cc/dia; 2-5
aos 5mg/dia, Cetirizina : 6m-1ao 0,5 mg/dia; 2-5aos 2.5 mg/dia, Fexofenadina: 6-12 aos 120 mg/da, Ebastina : 6-12-aos 5-10
mg/dia, Desloratadina: 2-5 aos 1,25 mg/dia; 6-11 aos 2,5mg, Levocetirizina: >2 aos 5/mg dia). Los de segunda generacin provocan
liberacin de aminas vasoactivas. Desloratadina: Tratamiento de rpida accin y bien tolerado y por largo tiempo. Usado en casos de
Urticaria crnica idioptica. Levocetirizina: Enantimetro o metabolito activo de certirizina. Receptor Antagonista H1. Rpido inicio de
accin, pocos efectos colaterales. Indicado en rinitis alrgica y en UCI. Efecto antiinflamatorio importante que aumenta su efecto
teraputico en enfermedades alrgicas. Uso por tiempo prolongado desde el ao de edad. Dosis: 5mg/dia. 3. Antileucotrienos: Segunda
lnea en urticaria que no responde a antihistamnicos. Previene o mejora los sntomas asociados a procesos mediados por leucotrienos
C4, D4, y E4. Util en urticaria por medicamentos ASA y AINES. Montelukast 2-5 aos 4 mg/dia; 6-14 aos 5 mg/dia. Zafinlukast 5-11 aos
10 mg/BID. 4. Corticoides: Esteroides Sistmicos ayudan a disminuir la inflamacin. Solo deben usarse en casos extremos y de
angioedema, por corto tiempo para evitar efectos colaterales. Urticaria por presin y urticaria vasculitis. Prednisona 1mg/Kg/dia por 5-
7 dias. 5. Ciclosporina: En la tercera lnea de tratamiento. Es un inmunosupresor. Controles de funcionalismo renal. Dosis: 3 mg/Kg de
peso. 6. Omalizumab. Urticaria Aguda con sntomas sistmicos: Dificultad respiratoria-Evidencia angioedema, Adrenalina 1/1000
subcutnea a 0.01 mg/Kg, repetir cada 15 a 20 minutos, hidrocortisona 7-10 mg/Kg/dsis y luego instaurar ciclo corto, menor a 10 das,
clorfeniramina: IV. Dieta de exclusin de alimentos disparadores (man, pia, chocolate). Combinacin de anti-H1 (1ra y 2da) o
Combinacin de anti-H1 y H2 (o anti-LT).

CASO CLINICO
Paciente de 4 meses de edad, sin antecedentes patolgicos conocidos, traido a consulta por presentar papulas generalizadas
pririginosas de 2 meses de evolucin, al EF mostraba papulas eritematosas generalizadas en torax y miembros superiores e inferiores
que respetaban palmas y plantas. Dichas papulas, al ser frotadas, tomaban aspecto de roncha rodeada por un halo eritematoso. Segn
refiri su madre sobre una de ellas se haba formado previamente una ampolla, el paciente se encontraba afebril y en buen estado
general.

PREGUNTA
Cual es la conducta teraputica mas adecuada?

RESPUESTA
a.- Clorhidrato de hidroxicina.
b.- Maleato de clorfeniramina.
c.- Loratadina.
d.- Cetirizina.

PRURIGOS. CIENCIAS BASICAS: El prurigo infantil o tambin llamado urticaria papular es una reaccin alrgica producida por la
picadura de insectos. Esta afeccin netamente peditrica no debe ser confundida con una alergia a los alimentos; se caracteriza por el
prurito intenso de difcil control, con brotes frecuentes y sucesivos que por lo general se autolimitan en el tiempo. SALUD PUBLICA:
Afecta en su mayora a nios entre los dos y diez aos, frecuentemente en los meses de primavera y verano. TIPOS: La palabra prurigo
define un proceso patolgico cuya lesin elemental es la ppula y el sntoma principal es el prurito. El espectro clnico abarca un rango
que va desde las ppulas (prurigo papular), ndulos (prurigo nodular) entre otros tipos de prurigo podemos mencionar al actnico, el
atpico o de Besnier, el de Sutton y el pigmentoso. DIAGNOSTICO: Se observa una erupcin estacional y recurrente, con presencia de
grupo de ppulas y vesculas pruriginosas de localizacin frecuente en antebrazos, abdomen, rea lumbar, glteos, muslos y piernas. La
respuesta va a depender del tipo de paciente y la calidad del inculo, pudiendo tambin observarse formas ampollares. PRURIGO
SIMPLE POR INSECTOS O URTICARIA PAPULOSA: Es causada como su nombre lo indica por la picadura de algn insecto. No est
relacionada por factores dietticos. La naturaleza alrgica de la condicin fue demostrada cuando los cambios histolgicos producidos
por picaduras de insecto a nivel experimental demostraron que eran idnticos a los de la urticaria papular. Afecta principalmente a
nios de 1 a 7 aos, siendo una de las causas ms frecuentes en la dermatologa peditrica, afecta igual a ambos sexos y a cualquier
etnia. Numerosos insectos causan prurigo, siendo los ms frecuentes el Cemex lectularius (chinche) produciendo la Cimiasis; la pulga
(Pulex irritans) que ocasiona la puliciasis y las picaduras por mosquitos (Culicidae). Los alrgenos presentes en la saliva del insecto
inducen una sensibilizacin del paciente con formacin de anticuerpos especficos. El tipo e intensidad de la reaccin originada por la
MANUAL DE TRABAJO DEL CURSO ENARM CMN SIGLO XXI
CURSO ENARM CMN SIGLO XXI TEL: 36246001 Pharmed Solutions Institute PGINA 404

picadura depender de si el paciente ha estado ya expuesto al insecto y de la capacidad del husped de responder al estmulo
antignico. Las lesiones tempranas de prurigo por insecto se deben a una respuesta de hipersensibilidad tipo I causada por la liberacin
de IgE. Posteriormente interviene un mecanismo de hipersensibilidad tipo IV dependiente de linfocitos T que produce las lesiones
tardas. El prurigo por insecto afecta cualquier rea corporal, se presentan vesculas en la fase inicial posteriormente aparecen
pequeas ppulas eritematosas, las lesiones son muy pruriginosas por los que se observan costras hemticas por rascado. Se observan
lesiones en diferentes estadios y evolucionan por brotes. PRURIGO NODULAR: Es una dermatosis crnica caracterizada por ndulos
muy pruriginosos que aparecen principalmente en las superficies extensoras de extremidades inferiores, puede desarrollarse luego de
una picadura por insecto de otras formas de inflamacin localizada. La lesin por s misma es muy pruriginosa, lo cual es el resultado
de un crculo vicioso establecido por el rascado, trauma mecnico e infeccin, etc. Diversas investigaciones sealan que existe un
nmero elevado nervios drmicos y clulas de Merkel en estas lesiones, sugiriendo que este cuantitativo aumento puede estar
directamente involucrado en la etiologa. Se ha reportado que la terapia antituberculosa produce una aclaracin parcial o completa de
las lesiones. El prurigo nodular puede ocasionalmente ser el resultado del prurito que acompaa a algunas enfermedades sistmicas
como los procesos obstructivos biliares, anemia, enfermedad renal crnica, policitemia vera, DM, parasitosis, erupciones por droga y en
los nios el ms comn el Linfoma de Hodgkins. Las lesiones del prurigo nodular presentan un dimetro aproximado de 0,5cm a 3cm, la
superficie de las lesiones puede ser queratsica deprimida en el centro. La lesin inicial es eritematosa y puede semejar a la urticaria,
sin embargo todas las lesiones tienden a pigmentarse con el tiempo. El nmero de lesiones vara desde 2 a 200 de las cuales algunas
mejoran espontneamente. PRURIGO DE SUTTON: Es una erupcin liquenoide de los codos en nios, pitiriasis de codos y rodillas. Esta
entidad clnica es una dermatosis eccematosa papular pruriginosa, la cual est limitada a codos, pudiendo extenderse a nudillos, manos
y trax. Se asocia a una historia personal de atpia o familiar, as como a niveles elevados de IgE srica. Se ha descrito en nios de 3 a
13 aos, siendo la lesin inicial una ppula eritematosa de 1-2mm de dimetro. PRURIGO PIGMENTOSO: Es una dermatosis rara,
ocurre en Japn, se describe el trauma fsico o friccin de la ropa como gatillo. Se ha detectado que puede ocurrir en adolescentes, se
caracteriza por ppulas eritematosas pigmentadas que confluyen formando un patrn reticulado, las lesiones se presentan en verano y
ms frecuentes en mujeres jvenes. El rash es de distribucin simtrica en tronco, rea cervical, regin lumbosacra, abdomen y cara.
PRURIGO DEL ATOPICO O DE BESNIER: Tambin llamado neurodermatitis, o eccema del lactante. Es la dermatosis ms frecuente en
poblacin peditrica. La prevalencia ha mostrado incremento en las ltimas dcadas, siendo del 18-20%. Es ms frecuente en reas
urbanas de pases industrializados, especialmente en inmigrantes provenientes de pases con menor prevalencia. Es una variante poco
comn en la infancia, se puede observar en adolescentes. De acuerdo con la morfologa, pueden definirse tres tipos de lesiones, sin ser
el tiempo de evolucin determinante para su definicin: 1. Aguda. Caracterizada por ppulas y vesculas muy pruriginosas, sobre un
rea de piel eritematosa, asociada a escoriaciones, erosiones, exudado seroso y costras melicricas (eccema). 2. Subaguda.
Caracterizada por eritema, ppulas, descamacin y escoriaciones. 3. Crnica. Placas de piel engrosada, con liquenificacin y ppulas
fibrticas. Existen adems tres fases cronolgicas, con caractersticas especficas: 1. Lactante (2 semanas a 2 aos). Afecta la cara,
predominando en mejillas y respetando el tringulo central. Puede extenderse a piel cabelluda, pliegues retroauriculares y de flexin,
tronco y nalgas; a veces puede generalizarse. Predominan las lesiones de dermatitis aguda por lo que se le ha denominado eccema del
lactante. Se puede asociar con dermatitis seborreica en un 17%. Aparece por brotes, siendo frecuente la desaparicin de a los 2 aos
de edad. 2. Escolar o infantil (3 a 14 aos). Afecta pliegues de flexin (antecubitales y huecos poplteos), cuello, muecas, prpados,
regin peribucal y genitales. Puede presentarse con lesiones agudas o crnicas, evolucionando en brotes, con prurito intenso. Puede
desaparecer (75-90%) o progresar a la ltima fase. Existen formas localizadas como el mal plantar juvenil, que afecta plantas y dorso de
los pies, forma periorbitaria, queilitis exfoliativa, intertrigo auricular y eccema del pezn. Por otro lado, se describen formas atpicas
como lo son la papular y folicular. 3. Adulto. (15 a 23 aos de edad). PRURIGO ACTINICO: Es una erupcin papular o nodular frecuente,
persistente, pruriginosa y escoriada de la piel expuesta al sol y en menor medida de la no expuesta. Es frecuente en verano y en
ocasiones no desaparece con el invierno, por lo general aparece en la niez y a veces remite en la pubertad. Parece ser una variante
persistente y algunas veces coexstente de la erupcin de lumnica polimorfica (ELPM) aunque sus caractersticas clnicas son diferentes.
La exposicin de radiacin UV sera el inductor del prurigo actnico (PA) ya que el trastorno es ms intenso en primavera y en verano y
las respuestas cutneas anormales a la radiacin estn presentes en dos tercios de los pacientes, ms a menudo con radiacin UVA que
con UVB. El PA podra ser una forma de ELPM exagerada, de evolucin lenta y por lo tanto ser una reaccin de hipersensibilidad de tipo
retardado. Esto se sustenta que ms pacientes con PA de los esperados presenta familiares cercanos con ELPM, el antgeno linfocitario
humano (HLA) DR4, presentan el 30% de los sujetos normales, se encuentra en el 80-90% de los pacientes con PA y el subtipo DRB1
0407 de DR4 presentan en el 6% de los sujetos normales y frecuentan en indios estadounidenses se encuentra alrededor del 60% de
aquellos con PA. El PA es ms comn en el sexo femenino y suele comenzar a los 10aos tiende a mejorar y desaparecer en la
adolescencia, aunque en ocasiones puede persistir en la vida adulta. La erupcin por PA est presente durante todo el ao aunque por
lo general empeora en el verano, y en raros casos se produce en invierno. As mismo las exacerbaciones de las lesiones tienden a
producirse durante el clima soleado, en lugar de hacerlo despus de episodios de exponerse al sol, aunque los brotes tipo ELPM son
posibles. En los casos tpicos las lesiones son ppulas, ndulos pruriginosos, excoriaciones asociadas con eccematizacin liquenificacin
o formacin de costras, tambin se describe queilitis sobre todo en el labio inferior. El PA puede asociarse a una tendencia a la
eccematizacin. TRATAMIENTO: Las medidas generales son de gran importancia, la explicacin a los padres acerca del padecimiento,
su predisposicin atpica, la cronicidad y evolucin por brotes. Se recomienda adems el uso de pijamas largos, mosquiteros,
insecticidas cuando no estn los nios. Tpicamente se utilizan lociones antipruriginosas y esteroides tpicos. Por va oral se utilizan
antihistamnicos no sedantes. En algunas ocasiones se utiliza Tiamina por va oral, la cual se ha recomendado empricamente por
mucho tiempo "tal vez sea un repelente natural que se excreta por la piel", dosis 200mg a 600mg por da, lo ms importante es evitar la
infeccin secundara. Se utilizan emolientes, capsaicina tpica, radiaciones UV y sistmicamente Talidomida, la cual est contraindicada
cuando hay riesgo de embarazo y de producir neuropata perifrica.

CASO CLINICO
Nia de 11 aos de edad que consulta por haber presentado lesiones eccematosas en el brazo izquierdo, donde hace un mes, en un
puesto ambulante, le realizaron un tatuaje de henna de color negro. A los 10 das, comenz con eritema y vesculas muy pruriginosas,
que cubran toda la zona del tatuaje. Las lesiones se resolvieron tras un mes de tratamiento con corticoide tpico, con persistencia de
MANUAL DE TRABAJO DEL CURSO ENARM CMN SIGLO XXI
CURSO ENARM CMN SIGLO XXI TEL: 36246001 Pharmed Solutions Institute PGINA 405

hipopigmentacin residual limitada a la zona del tatuaje. La paciente se haca, desde hace tres veranos, tatuajes transitorios de henna
color marrn en la playa.

PREGUNTA
Cual de los siguientes factores esta mas relacionado con el padecimiento.

RESPUESTA
a.- Infecciones de la piel.
b.- Exposicin a alrgenos.
c.- Irritantes como jabon, shampoo y cremas.
d.- Inhalacion ingestin de alrgenos.

PREGUNTA
Considerando el caso clnico cual es la conducta teraputica mas apropiada.

RESPUESTA
a.- Indica emolientes.
b.- Corticoides tpicos de leve potencia.
c.- Corticoides tpicos de moderada potencia.
d.- Corticoides tpicos de alta potencia.

CASO CLINICO
Masculino de 8 meses de edad que es llevado por su madre por presencia de tos y secresion nasal desde hace dos das y a la
exploracin se observa en cara, brazos enrojecimiento, la madre agrega que le pica y que se presentan y luego desapareces solos, al
tacto la piel se siente aspera.

PREGUNTA
Cual es la conducta a seguir menos adecuada?

RESPUESTA
a.- Evitar ambiente caluroso, ropa de lana, platicos y otras fibras.
b.- Baarlo con agua templada y de preferencia de periodos largos para mitigar lesiones.
c.- Aplicar crema hidratante y aceites varias veces al dia.
d.- Indicar crema con corticoide.

ERITEMA POLIMORFO. CIENCIAS BASICAS: El eritema multiforme (EM) o eritema polimorfo es una condicin reactiva aguda,
autolimitada, en ocasiones recidivante, mediada inmunolgicamente, que afecta la piel y las membranas mucosas. Desencadenada
fundamentalmente por el virus del herpes simple (VHS), presenta lesiones en diana y un patrn histopatolgico inflamatorio. Se
cataloga como una reaccin inflamatoria aguda, autolimitada, con diversos niveles de gravedad y variado conjunto sintomtico.
CLASIFICACION: Eritema multiforme maculopapuloso o eritema polimorfo menor (menor intensidad en los cambios
anatomopatologicos, escasa o nula participacin mucosa). Representa el 80 % de los casos. Eritema multiforme vesiculoampolloso o
eritema polimorfo mayor. Las formas graves, copn afeccion del estado general y posible participacin de rganos internos
corresponderan al Sindrome de Steven-Jhonson. La Necrolisis toxica epidrmica puede coinsiderase como un proceso aparte aunque
algunos autores lo consideran dentro del mismo espectro. SALUD PUBLICA: Predomina en adultos jvenes, de 20-40 aos, con afeccin
de nios y adolescentes en menos del 20% de los casos. Es algo ms frecuente en varones y no presenta predileccin racial. Se observa
con mayor frecuencia durante la primavera y verano, probablemente secundario a la reactivacin del VHS desencadenada por la
exposicin solar. Entre un 25% y un 50% de casos no puede establecerse el agente desencadenante. PATOGENIA: Se han reportado
numerosos factores desencadenantes, entre los cuales se encuentran infecciones, enfermedades malignas, enfermedades
autoinmunes, radiacin, inmunizaciones, medicamentos (AINE, sulfonamidas, anticonvulsivantes (30%), penicilinas, doxiciclina,
tetraciclinas) y menstruacin. El VHS es la causa ms comn y est involucrado en un 80-90% de los casos, con mayor asociacin al VHS
tipo 1. Se obtiene una historia de herpes labial en un 50% de los pacientes en las 2 semanas previas al desarrollo del EM; sin embargo,
la lesin herptica puede ocurrir simultneamente o desarrollarse luego de las lesiones en diana. Los mecanismos patognicos han sido
estudiados en los casos de infeccin por el VHS. En la recurrencia, material gentico es fagocitado y transportado a la circulacin
perifrica por monocitos/macrfagos que portan el antgeno linfocitario cutneo, un antgeno que les permite transportarse a la piel,
en donde se adhieren a las clulas endoteliales de la microvasculatura drmica, y el ADN viral fragmentado es finalmente transferido a
los queratinocitos del estrato basal y de las capas inferiores del estrato espinoso. La inflamacin en la lesin cutnea se cree es debida a
una respuesta inmune celular TH1 especfica contra los queratinocitos que contienen el gen de la polimerasa de ADN (Pol), con la
liberacin de citoquinas efectoras como el interfern (IFN-), que lleva a la amplificacin de una respuesta inflamatoria no especfica a
travs de clulas T autorreactivas y que resulta en la apoptosis de queratinocitos aislados (necrosis celular satlite) y el resto de los
hallazgos patolgicos que observamos en el EM. Finalmente, aditivos de alimentos, como los benzoatos y terpenos, y agentes tpicos
como los tatuajes y la dermatitis por contacto con hiedra venenosa, han sido reportados como causas de EM. DIAGNOSTICO: Tal como
indica su nombre de la enfermedad, ls lesiones clnicas son polimorfas e incluyen maculas, papulas y lesiones vesiculo ampollosas. Las
lesiones ms caractersticas muestran una apariencia anular concntrica en iris o en diana Generalmente no se observan
prdromos; algunos pacientes pueden presentar fiebre, cefalea, malestar y mialgias leves una semana antes de la erupcin. Las
lesiones cutneas se desarrollan en forma sbita, completndose el brote en 3-5 das. Las lesiones pueden diferir entre un paciente y
otro, y variar durante la evolucin de la enfermedad. Hay sensacin de prurito o ardor. La erupcin inicia como ppulas
MANUAL DE TRABAJO DEL CURSO ENARM CMN SIGLO XXI
CURSO ENARM CMN SIGLO XXI TEL: 36246001 Pharmed Solutions Institute PGINA 406

eritematoedematosas que simulan picadas de insectos, que permanecen fijas por ms de una semana. Algunas lesiones aumentan de
tamao, formando placas menores de 3cm, redondeadas, de bordes bien definidos, que desarrollan anillos concntricos (eritema iris).
La lesin en diana tpica consiste en una porcin central de color rojo sucio, rojo vinoso o purprico, inmediatamente rodeada por una
zona externa de color rojo intenso; le sigue un anillo edematoso plido, rodeado finalmente de un halo eritematoso. La porcin central
en ocasiones puede presentarse vsico-ampollar, formando el herpes iris de Bateman, o adquirir un aspecto costroso. La distribucin es
acral y simtrica, con predileccin por el dorso de las manos y superficies extensoras de las extremidades superiores; con menos
frecuencia se afectan las palmas, cara, cuello, tronco y extremidades inferiores. Se han descrito lesiones siguiendo las lneas de
Blaschko. Las lesiones mucosas se presentan en un 25-60%, y ocurren conjuntamente con las lesiones cutneas. La mucosa oral es la
ms afectada, principalmente la mucosa no queratinizada del tercio anterior. Inicia de forma rpida como edema y enantema que
afecta los carrillos, gingival y lengua, que progresa a erosiones superficiales con formacin de seudomembranas; en los labios se
observan fisuras, sangrado y costras serohemticas. El prurito, la sensacin de quemazn, as como la pobre alimentacin y baja ingesta
de lquidos debida al dolor de las erosiones mucosas, que puede ser severo, son causas importantes de morbilidad en el EM
Laboratorio: No hay pruebas especficas para el diagnstico. En casos severos puede encontrarse aumento de la velocidad de
eritrosedimentacin, leucocitosis o elevacin de las pruebas de funcin heptica. El diagnstico del eritema multiforme es clnico. Los
cambios histolgicos dependen de la morfologa clnica, la duracin y el rea de la lesin de donde se toma la biopsia. El hallazgo ms
precoz es la apoptosis de queratinocitos. Se observa necrosis celular en satlite, con linfocitos unidos a queratinocitos necrticos
aislados. ERITEMA MULTIFORME MENOR: Originado principalmente por infecciones virales herpticas (herpes simple) o Mycoplasma
pneumoniae y solo en 1 % por productos farmacuticos. La erupcin se produce en el trmino de 12 a 24 horas y en la mitad de los
casos hay un prdromo similar a una infeccin de las vas respiratorias altas. Las lesiones cutneas tpicas son mculas rojo- azuladas,
con 3 anillos concntricos denominados en diana o arco de tiro y una ampolla central, si bien pueden estar afectadas las mucosas. De
hecho, 20 % de los eritemas multiformes afecta a nios y adultos jvenes; su inicio es abrupto, con fiebre alta y sntomas
prodrmicos intensos. Se observan lesiones cutneas en el tronco, similares al eritema multiforme menor, pero ms extensas y
necrosantes. Daa gravemente a ms de 2 mucosas y ocasiona erosiones costrosas profundas en los labios; su evolucin es ms
prolongada, especialmente con frmacos de vida media ms larga y sus manifestaciones clnicas duran entre 3 - 6 semanas. Las formas
menores de eritema multiforme desaparecen espontneamente en alrededor de 3 semanas sin secuelas. SNDROME DE STEVEN-
JOHNSON: Es una forma grave de eritema multiforme habitualmente de ori gen txico. El grado de afectacin cutnea vara desde
formas con gran lesionalidad hasta casos con escasas o nulas lesiones cutneas. Sin embargo, se caracteriza por la severa participacin
de mucosas (oral, nasal, conjuntival, genital) y afectacin del estado general. NECRLISIS TXICA EPIDRMICA: La caracterstica clnica
ms importante es la formacin de grandes ampollas flcidas que condicionan el desprendimiento de lminas de epidermis, dejan do
extensas reas denudadas. Se trata de una situacin grave, en ocasiones mortal. TRATAMIENTO: La aplicacin del tratamiento puede
mejorar el cuadro sintomtico y acortar la evolucin del proceso; pero las formas mayores suelen ser mortales o causar graves
secuelas. La tasa de mortalidad para el sndrome de Stevens-Johnson se sita entre 1 y 5%, segn distintos autores. La existencia de una
gran extensin de zonas denudadas, la edad avanzada, la insuficiencia renal concomitante y la afectacin pulmonar empeoran el
pronstico. Dado el curso autolimitado del proceso, en eritema multiforme menor, solo se requiere tratamiento sintomtico. En los
casos postherpeticos recurrentes puede plantearse tratamiento con Aciclovir u otros agentes antiherpeticos. El Sndrome de Stevens
Jhonson y La Necrolisis toxica epidrmica son procesos potencialmente graves que requieren de ingreso del paciente y manejo en
cuidados intensivos, en condiciones de asepsia. Existe controversia respecto al empleo de corticoides sistmicos pues si bien suelen
condicionar un alivio de sintomatologa subjetiva no hay evidencia de que reduzcan la morbimortalidad. En ocasiones se precisa
antibioticoterapia para combatir complicaciones spticas.

CASO CLINICO
Preescolar de 3 aos de edad, de sexo femenino, previamente sana, que consulta por cuadro de dos das de evolucin, caracterizado
por fiebre, odinofagia y discreto compromiso del estado general. Evaluada en el Servicio de Urgencia se diagnostic amigdalitis aguda
purulenta. Se indic penicilina benzatina 600 000 UI intramuscular por 1 vez. Posterior a la administracin y en el transcurso del mismo
da, comenz con prurito generalizado, ms intenso en la regin dorsolumbar. Horas ms tarde, la madre not aparicin de lesiones
maculares eritematosas en extremidades, cara y tronco, por lo cual consult nuevamente, 48 horas despus. En ese momento, al
examen clnico se constat una paciente en buenas condiciones generales con temperatura de 38,5 C axilar y lesiones papulares,
eritematosas, redondeadas, distribuidas en forma simtrica y comprometiendo todo el cuerpo, ms marcado en cara y extremidades
inferiores. Las lesiones presentaban aspecto de anillos concntricos, con un halo perifrico eritematoso y un rea ms central plida y
en algunas lesiones una pequea vescula. La mucosa oral presentaba petequias escasas en forma aislada. No haba otras mucosas
comprometidas. El hemograma mostr GB 11 700 (75% segmentados, 5% baciliformes), hematocrito 38%, plaquetas 384 000 x mm3.
Adems VHS 80 mm x hr, PCR 10,9, creatininemia 0,61 mg/dl, nitrgeno ureico 0,22 mg/dl. Radiografa de trax y sedimento de orina
fueron normales.

PREGUNTA
Dentro de los siguientes diagnosticos cual es el menos probable?

RESPUESTA
a.- Eritema Multiforme.
b.- Stevens-Jonhson.
c.- Necrolisis Epidrmica Txica
d.- Vasculitis.




MANUAL DE TRABAJO DEL CURSO ENARM CMN SIGLO XXI
CURSO ENARM CMN SIGLO XXI TEL: 36246001 Pharmed Solutions Institute PGINA 407

ERITEMA NODOSO. CIENCIAS BASICAS: Es un sndrome caracterizado por una erupcin cutnea nodular, eritematosa, caliente y
dolorosa, que se localiza preferentemente en la regin pretibial, aunque tambin en ocasiones en muslos, brazos, antebrazos y otras
reas corporales. PATOGENIA: La etiologa de este sndrome es mltiple, pero podemos globalmente dividirla en dos causas
fundamentales: 1. De causa infecciosa: Bacterianas; tuberculosis (bacilo de Koch), estreptococicas, por yersinia, salmonela, shigela,
campilobacter, brucelosis, tularemia, enfermedad por araazo de gato, rickettsias, clamidias, mycoplasma, leptospirosis. Virica; VEB,
hepatitis B, parvovirus B19. Protozoos; Giardia lamblia, amebas, toxoplasma. 2. De causa no infecciosa: Medicamentos;
antoconceptivos orales, sulfamidas, penicilinas, salicilatos, TMP, bromuros, yoduros, sales de oro. Las lesiones del eritema nodoso
representan una reaccin frente a varios estmulos, se consideran como un tipo de vasculitis cutnea alrgica, pudiendo, por lo tanto,
ser consecutivas de cierto nmero de estmulos provocadores. DIAGNOSTICO: En ocasiones se acompaa de fiebre, con aceptable
estado general, puede haber artralgias de localizacin en grandes articulaciones de extremidades inferiores, que puede ser la
manifestacin de un proceso subyacente causa de esta patologa. Las lesiones cutneas, son bastante caractersticas; se trata de
ndulos de 1-3 cm de dimetro, dolorosos, indurados, brillantes, enrojecidos, calientes y sobreelevados. Aparecen en las reas
pretibiales fundamentalmente, pero tambin pueden tener otras localizaciones, como las nalgas, pantorrillas y tambin en las
extremidades superiores. La forma evolutiva de estas lesiones es bastante caracterstica; durante un periodo de varios das, se hacen
prominentes y de color violceo; despus de 1-2 semanas van disminuyendo su prominencia e induracin, modificando el color,
pasando por purpura oscuro y al final desaparecen dejando un rastro parduzco, sin dejar ulceracin, ni cicatriz. Estas lesiones aparecen
por brotes, en un periodo entre 3-6semanas, para determinar desapareciendo definitivamente. El diagnstico es clnico, pero debemos
hacer el diagnostico etiolgico se refiere a la necesidad de descartar una enfermedad asociada; esto puede ser en ocasiones difcil y se
puede no llegar al mismo en 20% de los casos. Hay que practicar pruebas complementarias como PPD, analtica bsica general,
coprocultivo, serologas, cultivos, radiografa de trax, trnsito intestinal si es necesario, etc. TRATAMIENTO: Generalmente tienen una
resolucin espontanea, y la adopcin de medidas generales como el reposo y las medidas fsicas es suficiente. El tratamiento debe ir
dirigido a tratar la enfermedad de base que sea la causante de este trastorno. Las medidas generales van encaminadas a disminuir el
dolor y el acortamiento de la enfermedad. Dentro de ellas es importante insistir en el reposo y en la elevacin de las extremidades
inferiores. Diversos frmacos antiinflamatorios han demostrado ser tiles en el tratamiento del eritema nodoso, pero especialmente los
salicilatos, la indometacina y el ibuprofeno. El empleo de corticoides sitemicos se debe reservar para aquellos casos con
manifestaciones sistmicas graves (artritis, fiebre muy elevada), enfermos con sarcoidosis y pacientes con eritema nodosp persistente y
ausencia de respuesta a los dems tratamientos mencionados. ERITEMA NODOSO MIGRATORIO: Es una variante clnica de eritema
nodoso, tambin denominada Paniculitis migratriz de Bfverstedt o Vilanova. Presenta ndulos unilaterales; en general menos
numerosos, dolorosos y persistentes que los de la forma clsica. Al progresar tienden a dividirse y extenderse en la periferia, esto les
confiere un patrn arciforme con bordes eritematosos, brillantes y un centro ms violceo o pardo.

CASO CLINICO
Masculino de 12 aos de edad acudi a consulta debido a que 15 dias previas inicio con lesiones eritematosa violcea nodulares y
dolorosas en los miembros inferiores de predomio en la parte posterior de las piernas. De 1 a 2 cm de dimetro, no referia ingesta de
frmacos, fiebre, sntomas respiratorios, digestivos ni articulares, en la anamnesis no se encontraron antecedentes personales ni
familiares de inters.

PREGUNTA
Considerando la etiologa mas frecuente de esta patologa, cual es la conducta diagnostica mas apropiada.

RESPUESTA
a.- Biopsia.
b.- PPD.
c.- Hemocultivo.
d.- Panel viral

DERMATITIS DE CONTACTO. CIENCIAS BASICAS: La dermatitis de contacto o eczema de contacto constituye un sndrome motivado por
la reaccin cutnea de una sustancia aplicada en la piel. Pue de ser una reaccin eczematosa aguda o crnica y est causada por un
irritante primario o por un mecanismo de sensiblizacin a un alrgeno tpico Es una dermatosis frecuente, afecta a ambos sexos y a
todas las razas y puede presentarse a cualquier edad. Es la causa dermatolgica ms frecuente de enfermedad profesional. SALUD
PUBLICA: Es una de las enfermedades de la piel ms frecuentes, alrededor de 10% de todas las dermopatas. Principales sensibilizantes:
Medicamentos (antibiticos, sulfonamidas, mercuriale, antihistamnicos, anestsicos, psoralenos), cosmticos (parafenilenodiamina,
perxidos, formaldehido, colorantes azoados, perfumes), metales (niquel, cromo, cobalto), ropas y zapatos (telas sintticas, hules,
cueros), plsticos (epoxi, resinas, acrlicos, nylon), remedios caseros (ajo, limn, yerbas). PATOGENIA: Cualquier sustancia u objeto que
este en contacto con la piel puede ocasinar una dermatitisde contacto. El mecanismo puede ser: A) Alrgico, por el modelo clsico de
inmunidad celular (hipersensibilidad tipo IV). Es necesaria fase de sensibilizacin, fase de latencia y de reexposicin o desencadenante.
El antgeno suele ser un hapteno que una vez en la epidermis, previa unin a una protena, conforma un antgeno completo que,
mediante clulas de Langerhans, es presentado a los linfocitos T que proliferan como clulas T efectoras y de memoria penetrando en
la circulacin sangunea. En la fase de desencadenamiento, 48 hrs despus de la exposicin, los linfocitos sensibilizados reconocen a los
alrgenos dando lugar a una transformacin blstica y proliferacin clonal con liberacin de citocinas mediadoras de la inflamacin.
Tambin participan otras clulas sanguneas. Es posible la sensibilizacin por va oral o parenteral. El fenmeno de autoeczematizacion
por protenas epidrmicas es frecuente en lesiones en piernas. B) Irritativo que origina un inflamacin de la piel secundaria a la
exposicin de un agente irritante, en concentracin y tiempo suficiente o contactos repetidos del mismo que da lugar a lesin celular
sin que exista mecanismos inmunolgicos. DIAGNOSTICO: Localizacin variada y siempre en relacin al agente causal. La clnica puede
ser: 1. Aguda, con eritema, edema, vesculas, ampollas y en casos extremos necrosis. Existe prurito quemante. 2. Subaguda, con placas
eritematosas con descamacin. 3. Crnica con lesiones en placas liquenificadas y zonas de descamacin y excoriacin. A veces grietas y
MANUAL DE TRABAJO DEL CURSO ENARM CMN SIGLO XXI
CURSO ENARM CMN SIGLO XXI TEL: 36246001 Pharmed Solutions Institute PGINA 408

fisuras. En casos muy intensos y de exposicin prolongada al antgeno, pueden aparecer lesiones a distancia o ides. El diagnstico se
basa en la clnica, profesin y hbitos del paciente. Las pruebas epicutneas sirven para diferenciar el eczema de contacto de origen
irritativo del de origen alrgico y, en esta caso identificar el antgeno. Los diferentes tipos de eczemas o dermatitis de contacto tienen
los mismos rasgos histopatolgicos y van a depender de la fase evolutiva en la que se biopsien. Como rasgos generales evidenciaremos
espongiosis, exocitosis y en dermis superficial infiltrado perivascular, fundamentalmente de tipo linfocitario. TRATAMIENTO: Antes que
nada debe indicvarse al paciente lo que no debe de hacer: No aplicarse nada en la piel, eliminar toda clase de pomadas,
remedioscaseros, objetos, ropas cuando se sospecha de ellas. Evitar jabones y detergentes. No hay dietas. Puede comer de todo. Evitar
el sol si hay fotosensibilidad. El tratamiento es sobre todo tpico y depender del estado de la piel. Una piel eczematosa requiere
primero medicamentos para secarla, as estn indicados los fomentos con agua de manzanilla o agua de vegeto al 50% (subacetato de
plomo). Se aplican a la temperatura ambiente o fros, cada 3 o 4 hrs, durante las primeras 24-48 hrs, hasta que la piel se seque y las
costras melicericas se desprendan, cuando ello se logre se podr aplicar las conocidas pastas inertes a base de xido de znc y/o
calamina. El uso de sedantes y antipruriginosos puede ser necesario en algunos casos y a veces hasta psicotrpicos del tipo de la
clorpromazina. Los corticoides por va tpica del tipo de la hidrocortisona estn indicados solamente en las dermatitis por contacto por
irritantes primarios y en casos muy limitados de sensibilizacin. Si hay mucha sensibilizacin y muchas ides, el esteroide mejorara de
inmediato el cuadro, pero vendrn los rebotes cada vez ms intensos y ms ides y el resultado final ser un estado eritrodermico difcil
de controlar.

CASO CLINICO
Un nio de 12 aos, en buen estado general y afebril, consult, por presentar una placa eritematosa nica, amplia, de bordes definidos,
sobre la que asentaban pequeas ppulas y pstulas, no fluctuante, localizada en la regin lateral derecha del cuello, de 3 das de
evolucin; se acompaaba de leve prurito, pero era indolora. No presentaba adenopatas regionales. Haba acudido a una guardia,
donde se le indic cefalexina en dosis de 100 mg/kg/da; en el momento de la consulta cursaba el primer da de tratamiento. En el
interrogatorio se destacaba que, 48 horas antes a la aparicin de las lesiones, el nio haba participado en actividades recreativas al aire
libre en la colonia de verano del club, y no se haba aplicado repelente.

PREGUNTA
Cual es la conducta a seguir mas adecuada en este momento?

RESPUESTA
a.- Continuar con antibitico por 5 dias.
b.- Corticoides topicos.
c.- Prednisona oral.
d.- Cetirizina nicamente.

DERMATITIS DEL PAAL. CIENCIAS BASICAS: Dermatitis del paal, dermatitis amoniacal, rozadura. Es una erupcin inflamatoria
aguda del rea cubierta por el paal. SALUD PUBLICA: Afecta con mayor frecuencia a recin nacidos y lactantes, con un pico de
incidencia a los 9 a 12 meses, aparentemente en relacin con el cambio en la dieta. Se presenta adems en nios mayores y adultos con
incontinencia urinaria y/o fecal secundaria a alteraciones urolgicas o neurolgicas. La prevalencia es del 7 al 35% en poblacin
peditrica, ocupando el 3 a 4 causa de consulta dermatolgica en nios. No se ha descrito diferencia racial ni en cuanto al sexo.
PATOGENIA: Koblenzer clasific la dermatitis de esta zona en tres grupos: Grupo 1. Enfermedades que se producen en forma
independiente del uso de paales (epidermolisis bulosa, histiocitosis de clulas de Langerhans, etc.). Grupo 2. Enfermedades agravadas
por el uso de paales (dermatitis atpica, dermatitis seborreica, psoriasis, etc.). Grupo 3. Enfermedades provocadas por el uso de
paales, en nios no predispuesto y como consecuencia directa de su uso, es el ms frecuente y ocupar nuestra revisin. Lo ms
caracterstico es la dermatitis de contacto por irritante primario en la cual la humedad persistente provoca maceracin de la piel; lo
anterior aunado a la friccin generada por los movimientos del beb altera la funcin de barrera y permite la accin de irritantes como
son: heces (proteasas, lipasas y sales biliares), orina (urea y amoniaco), jabones, lociones, cremas y productos empleados para el lavado
de los paales; por lo tanto, los irritantes actan en forma secundaria en piel ya daada previamente. Por otro lado, la piel as daada,
es susceptible de infeccin por Candida albicans con mayor frecuencia (50%) y ms raramente, agentes bacterianos (S aureus,
estreptococos y enterobacterias). La dermatitis por contacto alrgica no es comn el rea del paal, sin embargo algunas sustancias
como gomas, plsticos, neomicina y mercurio pueden actuar como sensibilizantes. DIAGNOSTICO: Existen diferentes patrones de
afeccin clnica: Dermatitis eritematosa simple. Es el ms frecuente. Afecta reas convexas (en W), respetando los pliegues. Las formas
ms leves se caracterizan por eritema y descamacin; el eritema puede desarrollar una apariencia vidriosa y aparecer ppulas.
Variedad sifiloide ppulo-erosiva o poserosiva de Sevestre-Jaquet. Es una forma moderada de la dermatitis eritematosa simple. Se
presenta sobre las superficies convexas en que se observan ppulas del color de la piel o eritematosas, generalmente menores de 5 mm
de dimetro, que por accin de la friccin o maceracin se erosionan en la cpula. Pueden volverse violceas y liquenificadas. Ulcerosa.
Se caracteriza por prdida cutnea epidrmica o ms profunda, siendo la presentacin ms grave de las anteriores; surge como
consecuencia de la potencia y el tiempo de exposicin a la agresin y sensibilidad individual de la piel. Dermatitis perianal. Limitada a
esta rea, se manifiesta como eritema, erosiones o pstulas; es ms frecuente en el periodo neonatal. Puede deberse a cambios
qumicos (alteracin del pH en procesos diarreicos o enteropatas) o infecciosos (dermatitis estreptocccica perianal, parasitosis). El
diagnstico es clnico; puede solicitarse examen directo con KOH y cultivo para corroborar infeccin mictica. La biopsia se reserva a
casos resistente al tratamiento, en que se sospecha otra enfermedad de base. TRATAMIENTO: Medidas preventivas, como son el
cambio frecuente de paal, recomendndose paales desechables con geles absorbentes (alginatos), de mayor tamao, para evitar el
roce y facilitar la absorcin; realizar el aseo del rea con agua, dejando el rea sin paal el mayor tiempo posible. Las pastas con talco y
xido de zinc son tiles como aislantes y para reducir la friccin y se aplican posterior a cada cambio de paal. En las lesiones con ms
de 72 horas de evolucin o datos de infeccin por candida debe aplicarse algn agente antimictico, 2 veces al da, por 3 semanas. En
caso de imptigo segundario se recomienda el uso de antibiticos tpicos o sistmicos. La utilizacin de corticoesteroides deber ser
MANUAL DE TRABAJO DEL CURSO ENARM CMN SIGLO XXI
CURSO ENARM CMN SIGLO XXI TEL: 36246001 Pharmed Solutions Institute PGINA 409

racional, restringindose a casos resistentes a las terapias convencionales, utilizando esteroides de baja potencia y nunca ms de 2
semanas. COMPLICACIONES: Candidosis. Es la complicacin ms frecuente. Compromete los pliegues inguinales e interglteos y se
caracteriza por ppulas y pstulas sobre una base eritematosa, intensa, brillante, con bordes netos, elevados, geogrficos y escama
blanquecina; asimismo lesiones satlites constituidas por ppulas y pstulas que se presentan en la periferia. Se ha demostrado
colonizacin por C albicans en la dermatitis de la zona del paal con ms de 72 horas de evolucin (45-75%), an en ausencia del cuadro
clnico caracterstico. Imptigo secundario. Agregndose pstulas y costras melicricas. Granuloma glteo infantil. Ppulas y ndulos
rojizos o purpricos de 0.5 a 4 cm, en regin perianal y gltea; se ha asociado a irritacin y friccin crnica, C. albicans y uso de
esteroides fluorinados.

CASO CLINICO
Nia de 4 meses de edad que la trae su padre porque al ir a cambiarle el paal le ha visto que tena la zona muy eritematosa y adems
al limpiarle la nia se queja.

PREGUNTA
Considerando el agente etiolgico mas frecuente, cual es la conducta especifica mas adecuada?

RESPUESTA
a.- Desecantes.
b.- Emolientes.
c.- Antifungico.
d.- Antihistaminico.

PSORIASIS. CIENCIAS BASICAS: Es una enfermedad autoinmunitaria eritematodescamativa muy frecuente de evolucin crnica que
cursa a brotes. Se caracteriza por lesiones bien definidas con escamas tpicas blanco-nacaradas. SALUD PUBLICA: Afecta a 1-3% de la
poblacin general. Aproximadamente el 25% de los casos de psoriasis se inicia durante la infancia, y aumenta su incidencia con la edad
hasta la adolescencia. PATOGENIA: Es una enfermedad hereditaria aunque no se conoce todava el modo de herencia. Existe una
predisposicin gentica compleja y probablemente multifactorial. Hay una frecuencia aumentada de algunos haplotipos HLA. Los
factores genticos influyen en el patrn de la psoriasis, severidad y edad de inicio. El riesgo para los familiares de primer grado de un
caso aislado es menor de 10%, pero si los dos padres tienen psoriasis los hijos tienen un riesgo de un 50% de padecerla. Existen unos
factores desencadenantes conocidos capaces de precipitar o exacerbar un brote. Las infecciones (particularmente por Streptcoccus), el
frio, el estrs emocional, los traumatismos cutneos repetidos y ciertos frmacos (cloroquina y corticoides sistmicos). DIAGNOSTICO:
Las lesiones de psoriasis infantil son placas eritemato-escamosas similares a las de la edad adulta, de tamao y forma variables, con los
mismos signos caractersticos al raspado y con tendencia tambin a desarrollar lesiones de psoriasis en las zonas de roce o de rascados
y traumatismos (fenmeno isomrfico de Koebner). La biopsia cutnea muestra una epidermis engrosada con acantosis y
papilomatosis, elongacin y edema de la dermis papilar y un adelgazamiento de la epidermis por encima de las papilas. Se han realizado
numerosos estudios sobre la calidad de vida en pacientes con psoriasis3, y se ha demostrado que repercute en actividades cotidianas y
en el bienestar fsico y psicolgico de los enfermos. TRATAMIENTO: En nios se basa en la experiencia y en las pautas teraputicas
empleadas en los adultos, con las limitaciones propias de sus efectos secundarios. Tpico: Emolientes y queratolticos: No tienen
efectos secundarios, alivian el prurito y disminuyen la descamacin, aunque es el tratamiento menos eficaz. Son tiles para conseguir
eliminar escamas importantes y as facilitar la penetracin de otros productos ms efectivos. Uno de los ms utilizados es la vaselina
saliclica al 3-5%. El alquitrn de hulla es la brea ms utilizada; en nios se emplean los baos de breas slo en casos muy extensos, por
su efecto calmante y reductor. La antralina (dithranol) al 0,1% parece que es menos irritante y es til en placas muy queratsicas;
carece de toxicidad general. Se ha demostrado adems que los tratamientos en aplicaciones cortas son efectivos y menos irritantes,
mediante aplicaciones diarias de 10-30 min y aumentando progresivamente la concentracin del 0,1 hasta el 2%. Corticoides: tienen en
la psoriasis una triple accin: antiinflamatoria, inmunosupresora y antiproliferativa. Su uso se ve limitado por los efectos secundarios:
efectos locales como atrofia, estras, telangiectasias e hipertricosis a menudo irreversibles; las erupciones acneiformes son tambin
frecuentes cuando se aplican en la cara corticoides de alta potencia, y aparecen ppulas y pstulas redondeadas y rojizas y
posteriormente comedones; a diferencia del acn juvenil, las lesiones se encuentran todas en el mismo estadio evolutivo. En pliegues,
genitales y cara causan ms efectos secundarios locales que en el resto del tegumento. Efectos sistmicos: la absorcin de corticoides
potentes a travs de la piel puede provocar un Cushing iatrognico e incluso detencin del crecimiento. La vitamina D como
tratamiento para la psoriasis, dado que induce la diferenciacin de queratinocitos y disminuye su proliferacin. Calcipotriol.
Proporciona buenos resultados en nios sin alterar la concentracin srica de calcio, siempre que no se sobrepasen los 50 g/semana.
Inmunomoduladores tpicos: El tacrolimus y el pimecrolimus actan inhibiendo la calcineurina y mejoran las placas de psoriasis poco
hiperqueratsicas. Si son muy hiperqueratsicas, se requiere la aplicacin previa de un queratoltico tpico para facilitar su absorcin.
Psoralenos + radiacin ultravioleta A (PUVA): La PUVAterapia est basada en la interaccin entre la radiacin ultravioleta A (UVA) y un
agente fotosensibilizante qumico: los psoralenos, pero no se recomienda su empleo en nios menores de 10 aos, ya que se acumulan
en el cristalino y si ste est an en desarrollo puede causar daos irreversibles; los rayos UVA penetran hasta el cristalino y pueden
inducir la aparicin de cataratas, por lo que hay que extremar la proteccin ocular durante la radiacin. La alta permeabilidad del
cristalino en los jvenes contraindica este tratamiento antes de los 12 aos. Por otra parte, son ya conocidos los efectos carcinognicos
a largo plazo de la radiacin UVA15-17. Tratamiento sistmico. Retinoides, metotrexato, ciclosporina. Tratamientos biolgicos: Son
anticuerpos monoclonales o protenas humanas recombinantes contra algunas citocinas: etanercept, infliximab, adalimumab
(antifactor de necrosis tumoral), efalizumab. Hay pocos casos en nios, aunque con buenos resultados, pero de uso experimental y
todava no est aprobado



MANUAL DE TRABAJO DEL CURSO ENARM CMN SIGLO XXI
CURSO ENARM CMN SIGLO XXI TEL: 36246001 Pharmed Solutions Institute PGINA 410

CASO CLINICO
Varn de 17 aos que, tras pasar una amigdalitis, presenta este exantema cutneo. A la EF se observan papulas y pequeas placas
eritematosas con minima descamaciones distribuidas de forma generalizada, pero principalmente en el tronco.

PREGUNTA
Considernado el tipo de psoriasis que presenta el caso, cual de las siguientes medidas es la mas adecuada para intervenir en el agente
casual?

RESPUESTA
a.- Corticoide tpico de mediana potencia, en excipiente no demasiado graso.
b.- Realizar baos con derivados de alquitrn.
c.- Antibiotico especifico preferentemente por cultivo.
d.- Exposicin solar moderada y frecuencia.

DERMATITIS SEBORREICA. CIENCIAS BASICAS: Es una dermatosis inflamatoria eritemato-descamativa de evolucin crnica y
recurrente que se localiza en cara, cuero cabelludo, pliegues y regiones medio-torcicas e interescapulares (zonas con mayor cantidad
de glndulas sebceas activas). Tambin se le conoce como eczema seborreico o pitiriasis seca y esteatoide. SALUD PUBLICA:
Enfermedad frecuente en pases desarrollados, no existiendo diferencias de gnero. La dermatitis seborreica infantil aparece en la
lactancia, habitualmente entre la 2 y la 8 semanas de vida Puede afectar tanto a nios como a adultos o ancianos. PATOGENIA: La
causa es desconocida. Se consideran factores predisponentes, genticos, emocionales, Atpicos, neurolgicos, bacterianos, endocrinos,
alimentarios, medicamentosos, estrs y alcoholismo. Tambin se ha relacionado con la disminucin de cidos grasos esenciales y
biotina. Aunque se conoce que existe un cambio cuantitativo y cualitativo en la composicin del sebo con transformacin de
triglicridos en cidos grasos ms irritantes con mayor cantidad de colesterol y menor de escualeno, no se sabe cul es la causa de la
citada alteracin aunque se implica en ello la accin del Pityrosporum ovale, levadura lipfila que forma parte de la flora cutnea
normal y se convierte en oportunista con potencial patgeno. En pacientes con SIDA hay disminucin de linfocitos CD4. En nios se
desconoce la causa aunque se sabe que del 15-35% de os pacientes durante la infancia padecieron dermatitis seborreica, padecen en la
edad adulta, psoriasis o dermatitis atpica. Un grupo de autores suecos arma que la dermatitios seborreica infantil (DSI) podra
deberse a un dcit de la enzima d-6-desaturasa, que convierte el cido linolico en cido gammalinolnico (AGL). Este dcit pare ce
normalizarse hacia los 6 7 meses de edad, coincidiendo con la desaparicin de la DSI. La asociacin entre las levaduras de Malassezia
y la patogenia de esta enfermedad ha sido causa de controversia desde los aos setenta. Evidencia reciente relaciona a la dermatitis
seborreica con metabolitos del hongo y con la induccin de citocinas. El hallazgo de las levaduras en escamas de los pacientes es lo
habitual. Las especies ms frecuentes en cuero cabelludo son: M. globosa y M. restricta, y en rostro M. globosa y M. furfur.
DIAGNOSTICO: En todas las reas, las lesiones de DSI consisten en placas eritematosas de tinte rosado, con morfologa redondeada o
incluso circinada, con bordes bien denidos, aisladas o conuentes, que se cubren de una descamacin amarillenta de aspecto
grasiento, que merece la denominacin de seborreica. La DSI no se acompaa de sntomas generales ni prurito. Para muchos autores,
la forma ms precoz de aparicin de la DSI es la costra lctea. Otros, consideran que la costra lctea es una entidad diferenciada, sin
ninguna relacin con la DSI. Sea como fuere, la costra lctea se desarrolla en la primera o segunda semanas de vida, y se maniesta
como una gran placa descamativa, con escamas grasientas y adherentes, de color amarillento, bajo las cuales se puede apreciar un
eritema ms o menos vivo. Asienta sobre todo en el vrtex, y puede extenderse hacia reas frontales con menor frecuencia, puede
ocupar una amplia supercie del cuero cabelludo. Las lesiones clsicas de DSI suelen afectar, adems del cuero cabelludo, a la frente,
pabellones auriculares, pliegues retroauriculares, nasogenianos, cejas, parpados y cuello. Tambin puede extenderse a el tronco, con
predominio en la zona periumbilical, axilas, ingls, trax anterior y espalda En los miembros, suele afectar a los pliegues antecubitales y
poplteos. En ocasiones, puede ser un trastorno generalizado, de tipo eritrodrmico. La afectacin de la zona del paal es muy
frecuente, especialmente en las reas inguinales, pliegue interglteo y genitales externos. El trastorno conocido como eritrodermia
descamativa de Leiner cursa con eritrodermia descamativa seborreiforme, retraso del crecimiento y diarrea. No hay estudios a gran
escala para establecer la histopatologa de la DSI, aunque parece similar a la DS del adulto. Se observa paraqueratosis focal, acantosis y
espongiosis. En la desembocadura de los folculos pilosos dilatados se observan polimorfonucleares neutrlos. La dermis supercial
muestra un inltrado linfohistiocitario perivascular. TRATAMIENTO: No tienen tratamiento definitivo. Dependiendo del grado de
afectacin se puede utilizar desde jabones no alcalinos o sustitutivos del jabon, a lociones o crema a base de cido saliclico, azufre,
selenio, ketoconazol u otros derivados azolicos, ciclopiroxolamina, terbinafina, hidrocortisona, tacrolimus. A nivel general se puede
complementar con biotina a altas dosis.

CASO CLINICO
En la revisin de un bebe al mes de nacido, la madre refiere que el nio presenta costras que le sale sobre todo en la cabeza y en la
frente y cejas.

PREGUNTA
Cual de las siguientes medidas teraputicas es la menos adecuada?

RESPUESTA
a.- Aplicar aceites y sustancias emoliente.
b.- Champues con antifungicos.
c.- Dieta rica en biotina.
d.- Corticoide de baja potencia.


MANUAL DE TRABAJO DEL CURSO ENARM CMN SIGLO XXI
CURSO ENARM CMN SIGLO XXI TEL: 36246001 Pharmed Solutions Institute PGINA 411

ACNE. CIENCIAS BASICAS: Sinnimos acn vulgar, acn polimorfo o acn juvenil. Es la inflamacin crnica folicular producida por
retencin de sebo, debida a obstruccin y distensin de la unidad polisebacea. SALUD PUBLICA: El acn vulgaris es la enfermedad
ms comn de la piel, afectando al 80% de las personas entre los 11 y los 30 aos de edad en algn momento. Adems, a nivel de los
adolescentes afecta ms del 85%. Puede persistir por aos y generar cicatrices permanentes y conllevar un dao importante a nivel del
desarrollo psicosocial y un problema de autoestima. Aunque se observa en ambos sexos, predomina ligeramente en el hombre.
PATOGENIA: La susceptibilidad a la seborrea est determinada genticamente. Los factores patgenos fundamentales son:
queratinizacin folicular anormal, secrecin sebcea aumentada, colonizacin bacteriana e inflamacin local. No hay influencia prctica
de la dieta. Al llegar la pubertad, por aumento en la sensibilidad de las glndulas sebceas, las hormonas andrgenas generan aumento
de tamao y de la secrecin de las mismas, lo que aunado a la hiperqueratosis de la pared y el poro foliculares, originan la lesin
fundamental e inicial del acn, el comedn (espinilla). Se ha demostrado la presencia de Propionibacterium acnes, quien transforma
los triglicridos del sebo en cidos grasos libres, que tienen efecto irritante local, favorece
la inflamacin por este mecanismo. Si la inflamacin se realiza en la parte terminal del
conducto folicular, a nivel de la epidermis, se origina una pstula, pero si se hace en la
parte proximal del folculo y en la misma glndula sebcea, se originan los abscesos. Los
factores psicgenos pueden agravar esta dermatosis de modo secundario, sobre todo en
aquellos pacientes que presentan excoriaciones y ulceraciones autoprovocadas.
DIAGNOSTICO: Afecta cara (99%), cuello y trax (espalda 60%, pecho 15%), sobre todo en
su parte superior. En cuanto a la morfologa, el acn se caracteriza por polimorfismo
lesional encontrndose lesiones no inflamatorias que son los comedones abiertos y
cerrados; el comedn abierto se reconoce como una concrecin gris oscuro, caf o negro
dentro de un orificio folicular dilatado y el comedn cerrado es una pequea ppula del
color de la piel. Las lesiones inflamatorias son ppulas eritematosas, pstulas y ndulos.
Las secuelas del acn son cicatrices y senos epiteliales. TIPOS: De lesiones acneicas en
cuadro anexo. CLASIFICACION: De acuerdo al tipo de lesin predominante, el acn se
clasifican en: comednico, papuloso, pustuloso, qustico, conglobata (abscesos y fstulas),
queloideo y fulminans (ppulas, ndulos y qustes acompaados de datos de infeccin
sistmica). De acuerdo a la gravedad del acn, se clasifican en: a) ACNE LEVE: Predominan
los comedones. b) ACNE MODERADO: Con predominio de ppulas y pstulas inflamatorias. c) ACNE GRAVE: Que consta de ndulos
(abscesos}) y de ppulas, pstulas y comedones. TRATAMIENTO: medidas generales; explicar al paciente claramente la naturaleza de la
enfermedad, los posibles efectos adversos de la terapia y su duracin habitualemnte
prolongada, para evitar las expectativas a corto plazo y el abandono teraputico. Detectar
posibles factopres gravantes o predisponenetes (medicaments, sustancias qumicas).
Desaconsejar la manipulacin de las lesiones e insistir en una adecuada limpieza e higienen
cutnea. Evitar el uso de cosmticos comedogenicos (cremas, lociones, geles, maquillajes) y
recomendar como alternativa interesante los cosmticos libres de grasa (oil free). El
tratamiento tpico es el pilar fundamental en el que se debe basar el manejo del acn en
atencin primaria. Considerando la clasificacin clnica anterior, el manejo se efectuar de la
siguiente manera. Acn leve. Jabones neutros o azufrados para el lavado dos veces al da y aplicacin de lociones desengrasantes como
el licor de hoffman, a las que pueden agregarse cido saliclico, resorcina o azufre a 1 y 3%. Pueden usarse comedolticos como
retinoides tpicos o perxido de benzoilo. Acn moderado: puede usarse clindamicina o eritromicina tpicas, en solucin o gel, dos
veces al da o valorar el uso de antibiticos sistmicos aunado al tratamiento tpico. Acn grave: Los antibiticos sistmicos son de
primera eleccin para el acn moderado. Acn grave: los antibiticos sistmicos son de primera eleccin para el acn moderado a
grave, los ms recomendados son las tetraciclinas, eritromicina, minociclina y el TMP/SFX, por uno a tres meses. La
diaminodifenilsulfona (DDS) da muy buenos resultados en acn conglobata. La isotretinoina oral se utiliza en acn conglobata y ndulo
qustico, pero su toxicidad es muy alta. Tratamiento psiquitrico: La dismorfofobia es una entidad que se asocia con frecuencia en el
acn y consiste en la percepcin exagerada de los padecimientos cutneos. Los pacientes se preocupan por su fealdad imaginaria.
Esto lleva a depresin clnica y a un trastorno obsesivo compulsivo o fobia social.

CASO CLINICO
Paciente de 15 aos de edad, de sexo masculino estudiante, que consulta por presentar compromiso sistemico, con fiebre, astenia,
mialgias, anorexia y prdida de peso. El cuadro fue de inicio sbito; presenta lesiones dermatolgicas que afectan pecho y espalda.
Estas lesiones son de aspecto granulomatosas que sangran fcilmente, muy secretantes y necrosis dolorosas. Se solicitan anlisis de
laboratorio. Los resultados de estos estudios evidenciaron leucocitosis, anemia, eritrosedimentacin elevada (VSG), cultivos
bacterianos de secrecin y sangre negativos.

PREGUNTA
Cual de las siguientes medidas tiene la menor evidencia dentro del tratamiento de esta entidad?

RESPUESTA
a.- Eritromicina.
b.- Isotretinona
c.- Dapsona.
d.- Prednisona.

MANUAL DE TRABAJO DEL CURSO ENARM CMN SIGLO XXI
CURSO ENARM CMN SIGLO XXI TEL: 36246001 Pharmed Solutions Institute PGINA 412

PRPURA DE SCHNLEIN-HENOCH (PSH). CIENCIAS BASICAS: Es una vasculitis sistmica de vaso pequeo y curso habitualmente
agudo. Es una vasculitis leucocitoclstica, de mecanismo inmunologico la ms comn en la infancia. La causa es desconocida. Precedida,
en ocasiones, por una infeccin de vas respiratorias que sugiere un posible agente infeccioso desencadenante, siendo larga la lista de
patgenos implicados. Otros desencadenantes pueden ser frmacos (penicilina, ampicilina, eritromicina, quinina), alimentos,
exposicin al fro o picaduras de insectos. SALUD PUBLICA: Afecta en el 90% de los casos a nios, preferentemente varones (2:1), con
una edad media de 6 aos. PATOGENIA: Es una vasculitis mediada por IgA de los pequeos vasos. Hay aumento en la produccin de
IgA, aumento de inmunocomplejos circulantes de IgA y depsitos de IgA en las biopsias de piel y de rin. La lesin renal de la PSH es
indistinguible histopatolgicamente de la nefropata por IgA de la enfermedad de Berger. Ambas pueden producir insuficiencia renal.
DIAGNOSTICO: Usualmente se presenta con la ttrada: prpura palpable, artritis, dolor abdominal y enfermedad renal. La prpura
aparece en el 100% de los casos pero en una cuarta parte no es la manifestacin inicial. Se caracteriza por lesiones palpables de 2-
10mm, de color rojo-violceo (90%), que se concentran caractersticamente en los glteos y las extremidades inferiores aunque, en
ocasiones, pueden afectar a los brazos y la cara, y rara vez al tronco. La afectacin articular (50-85%) se manifiesta habitualmente como
oligoartritis de miembros inferiores, afecta principalmente la articulacin del tobillo y la rodilla. Las alteraciones gastrointestinales (50-
70%) se caracterizan por dolor abdominal clico o anginoso (empeora con la ingesta) que puede acompaarse de nuseas, vmitos,
diarreas, rectorragias y melenas Los sntomas gastrointestinales se deben al edema de la pared intestinal y a la hemoragia propia de la
vasculitis, cuando estas lesiones son muy intensas pueden dar lugar a un infarto intestinal y perforacion. En la gastroscopia y/o
colonoscopia se observan petequias, hemorragias o erosiones. Son complicaciones posibles una hemorragia intestinal grave y la
invaginacin o perforacin intestinal. La enfermedad renal (20-50%), normalmente se presenta como una glomerulonefritis moderada
que cursa con hematuria, habitualmente microscpica, y con menos frecuencia proteinuria leve y raramente como sndrome nefrtico,
nefrtico y/o insuficiencia renal. En los adultos hay un mayor riesgo de afectacin renal crnica que puede llegar a insuficiencia renal
terminal. Aunque se trata de una vasculitis autolimitada hay recurrencias en la tercera parte de los casos. El diagnstico es clnico y se
utilizan los criterios del American College of Rheumatology: 1. Inicio de los primeros sntomas a los 20 aos o menos. 2. Lesiones
purpricas sobreelevadas sin trombocitopenia. 3. Dolor abdominal anginoso (empeora con comidas), vmitos, diarreas, rectorragias y
melenas. 4. Biopsia cutnea: neutrfilos peri o extravasculares, en arteriola y/o vnula. 5 Biopsia cutnea: neutrfilos en pared vascular
de arteriola y/o vnula. La presencia de 2 criterios clasifica de PSH con sensibilidad: 87,1% y especificidad: 87,7%. Las crioglobulinas,
FR, ANA y ANCA suelen ser negativos o positivos a ttulos bajos. En los adultos la biopsia de piel y el control de la posible afectacin
renal son obligatorios. La biopsia cutnea muestra una vasculitis leucocitoclstica que afecta a los pequeos vasos (arteriolas, vnulas y
capilares). El hallazgo ms caracterstico es la infiltracin de neutrfilos de los pequeos vasos de la dermis, con frecuente
fragmentacin del ncleo de los mismos. Estudios mediante inmunofluorescencia muestran depsitos granulares de IgA y, en menores
cantidades, C3 y fibrina en la pared de los vasos. La microscopia renal convencional suele mostrar una proliferacin mesangial aislada y
slo en casos graves una glomerulonefritis con semilunas. TRATAMIENTO: Se resuelve espontneamente en el 94% de los nios y en el
89% de los adultos, por lo que el primer objetivo es tranquilizar al paciente y utilizar tratamiento sintomtico para la afectacin
articular y el dolor abdominal. El tratamiento con corticoides (1-2mg/kg) es controvertido. Una revisin sistemtica indica que los
corticoides reducen la duracin del dolor abdominal y disminuyen el riesgo de invaginacin intestinal, afectacin renal y recurrencias.
Su uso en la PSH no complicada no mejor el curso de la enfermedad, segn un ensayo clnico8. La afectacin renal grave precisara de
corticoides, inmunosupresores y/o plasmafresis. Lo nico que tiende la cronicidad es la nefritis.

CASO CLINICO
Varn de 7 aos de edad ingresado por presentar un cuadro de dolor abdominal difuso y una deposicin melnica. La semana previa
haba consultado por varicela y haba tomado ibuprofeno, 20 mg/kg/da. No presentaba antecedentes de inters, salvo dolor abdominal
recurrente. En la exploracin al ingreso no se objetivaron hallazgos patolgicos. Al ingreso la coagulacin era normal as como el
sistemtico de sangre. Al noveno da apareci un exantema eritematoso violceo palpable, simtrico, en extremidades inferiores, y se
estableci el diagnstico prpura de Schnlein-Henoch.

PREGUNTA
Cual el predominio vascular primordial en esta patologia.

RESPUESTA
a.- Vasculitis de grandes vasos.
b.- Vasculitis de vasos medianos.
c.- Vasculitis de pequeos vasos.
d.- Vasculitis asociadas a conectinopatias.

CASO CLINICO
Nio de 6 aos de edad, sin antecedentes personales ni familiares de inters, que fue valorado en el servicio de urgencias por
presentar, desde haca 24 h, lesiones cutneas maculopapulosas y purpricas distribuidas simtricamente por las nalgas, miembros
inferiores y superiores asociadas a signos inflamatorios en las rodillas y tobillos, muy indicativo de prpura de Schnlein-Henoch. Los
anlisis de orina, de funcin renal y la presin arterial eran normales. Se inici tratamiento con antiinflamatorios no esteroides y se dio
de alta.

PREGUNTA
Considerando el cuadro clnico cual de las manifestaciones es la menos frecuente para orientar el diagnostico de PSH.

RESPUESTA
a.- Fiebre prolongada de origen desconocido.
b.- Lesiones cutneas sugestivas.
MANUAL DE TRABAJO DEL CURSO ENARM CMN SIGLO XXI
CURSO ENARM CMN SIGLO XXI TEL: 36246001 Pharmed Solutions Institute PGINA 413

c.- Artralgias, artritis, miositis y serositis.
d.- Parametros de laboratorio de inflamacin.

PREGUNTA
Cual de los siguientes factores etiolgicos puede estar ms relacionado con esta patologia.

a. Estreptococo del grupo A beta hemoltico o M. pneumoniae,
b.- Virus de hepatitis A, citomegalovirus virus de Ebstein Barr.
c.- Vacuna del sarampin, influenza, rubola, neumococo.
d.- Antibiticos betalactmicos, macrlidos AINES.

PREGUNTA
Cual de las siguientes manifestaciones es la menos frecuente para hacer el diagnostico.

RESPUESTA
a.- Manifestaciones cutneas.
b.- Manifestaciones digestivas.
c.- Manifestaciones articulares.
d.- Manifestaciones renales.

ANEMIAS. CIENCIAS BASICAS: Es la disminucin de la hemoglobina o el hematocrito, por debajo de los valores normales de la localidad
y de acuerdo a la edad y sexo, con la consecuente hipoperfusin tisular de oxgeno. A travs de la BH (Hb, Hto, reticulocitos, VCM,
CHCM), se obtienen los valores que indicaran si corresponde a un sndrome anmico. SALUD PBLICA: En Mxico 67% de los menores
de 2 aos presentan ferropenia y en edad escolar 34%. En escolares y adolescentes la falta de una ingesta adecuada de carne de res y
en el caso de las mujeres con la menstruacin, favorecen esta entidad. De 2-11% de los nios menores de 12 aos de edad presentan
deficiencia de folatos y aquellos que ingieren vegetales verdes solo 0.22%. PATOGENIA: La anemia puede deberse a falta de
produccin (aplasia, nutricional), perdida (hemorragia) o destruccin de los eritrocitos (hemolisis). DIAGNOSTICO: En el RN a trmino
que presenta anemia al nacimiento o durante los primeros das de vida, clnicamente puede lucir bien y ser un hallazgo por estudio
rutinario de H o presentar manifestaciones de leves a severas: lucir mal, palido con ingesta inadecuada de leche, irritable. En los
lactantes y preescolares puede observarse palidez, irritabilidad, somnolencia, astenia, menor incremento ponderal, infecciones de
repeticin y en algunos casos pica o geofagia. En escolares o adolescentes puede manifestarse adems cefalea: acufenos, fosfenos,
lipotimia, palpitaciones, disnea, parestesias, calambres. La palidez como dato objetivo de anemia, es valido en los lechos ungueales, ya
que la mucosa oral o la conjuntiva pueden manifestar otras alteraciones que la eclipsan y la palidez global puede ser expresin de otras
alteraciones como estrs coartacin artica, estado de choque. Se puede encontrar de acuerdo a la intensidad y cronicidad de la
anemia, manifestaciones de insuficiencia cardiaca como: taquicardia, diferencial amplia en la tensin arterial, edema de miembros
inferiores, hepatomegalia. La ruta diagnostica inicia al determinar el porcentaje de reticulocitos corregidos: arregenerativa o falta en la
produccin cunado estn por debajo de 3% y regenerativa cuando estn por arriba de 3%. Habitualmente las segundas tienen cifras pr
arriba de este porcentaje y es debida a hemorragia aguda o hemolisis. ANEMIA FERROPENICA: La mayor causa por falta de produccin
de eritrocitos, en la mayor parte de los pases es la ferropenia. Se inicia con la deplecin de eritrocitos de hierro (ferropenia latente),
pasa por eritropoyesis ferropnica (inicialmente microcitosis y posteriormente hipocroma) y termina con la anemia. En el neonato este
evento es poco frecuente porque la reserva de hierro es directamente proporcional a su peso corporal. El prematuro no recibe la
cantidad de hierro suficiente de Fe y debe reponerse el mismo en las primeras semanas. Las complicaciones del embarazo que originan
perdidas sanguneas crnicas, disminuyen la dotacin de hierro al neonato, como la hemorragia vaginal, amniocentesis traumtica,
varices aneurisma del cordn umbilical, insercin velamentosa, placenta previa, desprendimiento prematuro de placenta. ANEMIA
MEGALOBLASTICA: Hay una eritropoyesis ineficaz, puede deberse a deficiencia nutricional por deficiencia de folatos y de vitamina B12.
En el prematuro la deficiencia se presenta en el primer mes de nacido y requiere reposicin de este elemento. Las reservas de folatos
se mantienen por 2 semanas en cualquier etapa del desarrollo, la ingesta de vegetales verdes mantiene estos niveles, por lo que los
lactantes alimentados solo con leche, desarrollan esta deficiencia. Algunos medicamentos interfieren con el metabolismo de los folatos
como los barbitricos, difenilhidantoina, TMP/SFX, excepcionalmente pueden llevar a anemia megaloblastica. En el caso de B12 el
hgado almacena hasta por 6 meses, por lo que es raro observar anemia por este elemento. ANEMIAS CONGENITAS: Como la aciduria
ortica hereditaria y la anemia diseritropoytica hereditaria se presentan excepcionalmente. En los RN si hay anemia normocitica,
pudiese corresponder a infecciones de tipo congnito como la rubeola, parvovirus B19, CMV, toxoplasmosis, ingesta demediacmentos
durante el embarazo o bien aplasia pura de la serie roja congnita. En el resto de las edades peditricas debe descartarse aplasia pura
de la serie roja adquirida, enfermedades crnicas como colagenopatias, endocrinopatas, infeccin o insuficiencia renal. Anemia
arregenerativa: DFe acuerdo a las caractersticas morfolgicas de los eritrocitos, se estudia por la presencia de hipocroma, macrocitosis
o normocitosis. DISQUERATOSIS CONGNITA: (sndrome de Zinsser -Cole-Engman) es una enfermedad hereditaria caracterizada por
hiperpigmentacin de la piel, leucoplasia de las membranas mucosas y distrofia ungueal, que en aproximadamente la mitad de los
casos se acompaa de pancitopenia con hipoplasia medular. Se ha demostrado que esta enfermedad se transmite en forma recesiva
ligada al cromosoma X, habindose observado en algunas madres portadoras, lesiones de leucoplasia en la mucosa bucal. Tambin se
han descrito otros probables patrones de transmisin que incluyen la herencia dominante ligada al cromosoma X y autosmica
dominante. Lo anterior marca una diferencia con la anemia de Fanconi que es una enfermedad heredada con carcter autosmico
recesivo y que adems se presenta asociada a mltiples alteraciones esquelticas y cromosmicas. ANEMIAS HEMOLTICAS
HEREDITARIAS: la alteracin en el eritrocito se puede presentar a diferentes niveles; a. Defectos en la membrana del eritrocito. La
forma ms frecuente es la esferocitosis hereditaria, la cual ocupa el primer lugar entre las anemias hemolticas en Mxico. Otros
variantes incluyen la eliptocitosis, ovalocitosis y la estomatocitosis hereditaria. b. Capacidad limitada para la sntesis de las cadenas
normales de la hemoglobina. En este grupo se incluyen las talasemias y las alteraciones en la secuencia de los aminocidos de las
MANUAL DE TRABAJO DEL CURSO ENARM CMN SIGLO XXI
CURSO ENARM CMN SIGLO XXI TEL: 36246001 Pharmed Solutions Institute PGINA 414

cadenas polipeptdicas lo cual determina un gran nmero de variantes moleculares; las ms frecuentes de estas variantes son la
hemoglobinopata S o anemia africana (ocupa el segundo lugar en frecuencia dentro de las anemias hemolticas hereditarias en
Mxico), la hemoglobina C, D y E. c. Deficiencias enzimticas de los eritrocitos. Incluyen principalmente las deficiencias de
piruvatoquinasa y de glucosa-6-fosfato deshidrogenasa. ANEMIA HEMOLTICA AUTOINMUNE: Destruccin exagerada de eritrocitos,
producida por una actividad inmune aberrante que se dirige contra los glbulos rojos del propio husped. Clnicamente puede ser
clasificada en primaria o secundaria, de acuerdo a si se halla asociada o no a otros estados patolgicos. Estos ltimos pueden incluir
infecciones virales o bacterianas, enfermedades asociadas con produccin de autoanticuerpos, sndromes de inmunodeficiencia y
neoplasias. Se desconoce el mecanismo por el cual el organismo forma anticuerpos contra sus propios glbulos rojos, pero s se
conocen los mecanismos inmunolgicos capaces de destruir los glbulos rojos in vivo. La anemia hemoltica autoinmune primaria
(AHAIP) se caracteriza por la presencia de anticuerpos capaces de actuar contra los eritrocitos del propio paciente. Los anticuerpos han
sido caracterizados como inmunoglobulinas G(IgG) o M(IgM). stas actan directamente contra los antgenos de la membrana del
eritrocito o bien, mediante la formacin de complejos inmunes dirigidos contra ella. En otros casos, la lisis de la membrana eritrocitaria
se produce por activacin del sistema del complemento. Habitualmente el inicio es agudo con descenso rpido en los niveles de
hemoglobina. Asimismo, la mayor parte de los pacientes presentan periodos breves de evolucin, menores de tres a seis meses. Clnica;
decaimiento, anorexia, palidez, dolor abdominal e ictericia de intensificacin progresiva; puede ocurrir adems emisin de orina de
color pardo obscuro a consecuencia de la hemoglobinuria. A la exploracin fsica puede observarse hepatomegalia y esplenomegalia de
intensidad variable. Con menos frecuencia se manifiestan fiebre, tendencia hemorrgica, disnea, taquicardia y linfadenopatas. Los
hallazgos de laboratorio caractersticos de los pacientes con AHAIP: anemia, macrocitosis, reticulocitosis, hiperbilirrubinemia indirecta y
concentracin de hemoglobina libre en plasma en valores superiores al normal. TRATAMIENTO: Anemia ferropenica sulfato ferroso:
20mg/kg/da, fumarato ferroso 15mg/kg/da, el tiempo de administracin va en relacin a corregir la anemia, y completar las reservas,
se puede uno guiar con la BH y suspender cuando ha desaparecido la microcitosis o bien completarse por 6 meses. Excepcionalmente
se requiere la administracin por va parenteral y solo cuando existe intolerancia gstrica. Anemia por deficiencia de folatos; cido
flico 1 mg por VO, por un mes. Anemia por deficiencia de B12, es conveniente su administracin por va parenteral, ya que la
absorcin intestinal es insuficiente, cuando hay deficiencia. Se indica 100mg IM al da por 10 das. En la anemia hemoltica congnita el
consumo exagerado de folatos por la reprodccion celular incrementada, requiere mantener niveles adecuados de este elemento. Se
administra cido flico a 1mg/da VO por da. Anemia hemolitca autoinmunitaria; se indican folatos por la misma razn que en la
congnita y esteroides por va endovenosa si hay crisis hemoltica o por VO prednisona, iniciando 2 mg/kg/dia, con disminucin
paulatina cada 2 semanas, hasta completar 2 meses de tratamiento. Tratamiento quirrgico; en el caso de talasemia o esferocitosis
hereditaria, que cursen con esplenomegalioa o hemolisis intensa, est indicada la esplenectoma, a partir de los 4 aos de edad.
Tratamiento nutricional; en las anemias carenciales es necesario realizar cambios en los hbitos alimenticios, como el inicio temprano
de la ablactacin, con frutas, verduras y carne de res por lo menos 4 das de la semana y en especial las mujeres adolescentes por las
prdidas sanguneas debidas a la menstruacin. El trasplante alogenico de medula sea est indicado en anemia hemoltica congnita,
ya que permite la curacin en la mayor parte de los casos, principalmente en pacientes con alfa talasemia, beta talasemia mayor y
anemia falciforme.

CASO CLINICO
Lactante de 2 aos von 6 meses con letargia, hiporexia, retraso ponderoestatural e involucin psicomotriz (hipotona, irritabilidad y
prdida de sonrisa social) de un mes de evolucin. Como antecedentes personales destacaban un crecimiento intrauterino retardado
disarmnico, una serologa materna positiva para les durante la gestacin y anemia perniciosa mal controlada. No haba
consanguinidad. La alimentacin era con lactancia materna exclusiva. En la exploracin fsica presentaba un estado general regular,
hipoperfusin perifrica, palidez e ictericia mucocutneas, un soplo sistlico eyectivo, taquicardia, papilitis lingual, aftas palatinas.

PREGUNTA
Considerando el cuadro clnico cual de las manifestaciones es la menos importante para el diagnostico.

RESPUESTA
a.- Antecedentes personales.
b.- Antecedentes maternos.
c.- Soplo sistlico eyectivo.
d.- Aftas palatinas.

PREGUNTA
Cuales son las manifestacin para hacer el diagnostico diferencial para anemia ferropenica.

RESPUESTA
a.- Volumen corpuscular medio disminuido.
b.- Hemoglobina corpuscular media normal.
c.- Amplitud de la distribucin eritrocitaria disminuido.
d.- Recuento de plaquetas mormales o aumentadas.

PREGUNTA
Cual no es una indicacin para referir al paciente antes referido.

RESPUESTA
a.- Ferritina serica no concluyente.
b.- Perfiel de hierro no concluyente.
MANUAL DE TRABAJO DEL CURSO ENARM CMN SIGLO XXI
CURSO ENARM CMN SIGLO XXI TEL: 36246001 Pharmed Solutions Institute PGINA 415

c.- Paciente con buen apego que no responde.
d.- Prdida sangunea aguda.

LEUCEMIAS. CIENCIAS BASICAS: La leucemia aguda es el padecimiento ms frecuente en pediatra, no solo Mxico, sino en todo el
mundo. Es un trastorno gentico de las clulas hematopoyticas. La caracterstica principal de este padecimiento, es la proliferacin de
la clula ms primitiva de la medula sea, que puede ser desde la ms indiferenciada hasta la mayor grado de diferenciacin, proviene
tanto de linfocitos B como T. Las leucemias crnicas son mucho ms comunes en los adultos que en los nios. Suelen crecer ms
lentamente que las leucemias agudas, aunque tambin son ms difciles de curar. Las leucemias crnicas tambin se pueden dividir en
dos tipos. SALUD PUBLICA: En el nio existen varios tipos de leucemias siendo la ms frecuente leucemia aguda linfoblastica (LAL),
comprende ms de 65% de los casos, le sigue la leucemia mieloblastica aguda (LAM) en un 25% frecuencia (es la responsable del 30%
de las muertes por leucemia en la edad peditrica) y la leucemia granulocitica crnica en menos de 5% de los casos. Se estima
alrededor de 1 caso por 2880 nios sanos, en pacientes con sndrome de Down la frecuencia es de 1 en 95 casos. La OMS seala que en
Mxico es ms frecuente en el sexo femenino. El pico mximo de frecuencia es el alrededor de los 3-5 aos de edad. PATOGENIA: La
etiologa de esta enfermedad es desconocida, existen factores predisponentes como: radiacin (in tero, con las placas simples de
radiografa), anomalas cromosmicas (Sx. Bloom, anemia de Fanconi, Sx de Down), otras entidades (Sx. de Poland, de Rubistein-Taybi,
neurofibromatosis), frmacos (cloranfenicol, agentes citotoxicos), insecticidas, la frecuencia de leucemia aguda es mayor en los
familiares de pacientes con LA. Las clulas leucmicas se pueden reproducir rpidamente, y puede que no mueran cuando deberan
hacerlo, sino que sobreviven y se acumulan en la mdula sea, desplazando a las clulas normales. Un tipo comn de anomala del ADN
que pueden dar lugar a la leucemia se conoce como translocacin cromosmica. El ADN humano est empacado en 23 pares de
cromosomas. En una translocacin, el ADN de un cromosoma se desprende y se une a un cromosoma diferente. El punto en el
cromosoma donde ocurre el desprendimiento puede afectar
los oncogenes o los genes supresores de tumores. Por
ejemplo, una translocacin vista en casi todos los casos de
leucemia mieloide crnica (CML) infantil y en algunos casos
de leucemia linfoctica aguda (ALL) infantil es el intercambio
de ADN entre los cromosomas 9 y 22, lo que conduce a lo
que se conoce como cromosoma Philadelphia. Esto crea un
oncogn conocido como BCR-ABL. Tambin se han
descubierto en leucemias infantiles muchos otros cambios
en cromosomas o en genes especficos. Algunos nios
heredan mutaciones del ADN de uno de sus padres que
pueden aumentar su riesgo de desarrollar cncer.
CLASIFICACION: Citomorfolgica, la clasificacin Franco-
americana-britnica (FAB) reconoce 3 tipos morfolgicos: L1;
tamao clula pequeo, forma del nuclolo regular,
nuclolos ausentes y relacin ncleo-citoplasma es elevada.
L2; poblacin celular ms heterognea e irregular, la relacin
ncleo-citoplasma es mayor y a menudo se perciben
nuclolos. L3; clulas morfolgicamente idnticas a las de
linfoma de Burkit, grandes, homogneas con nuclolo
redondo y ovalado, as como presencia de vacuolas. En los
nios L1 80-85%, L2 15% y L3 menos de 3%. Clasificacion
tipo celular afectado: Leucemia linfoctica crnica (LLC).
Afecta a las clulas linfoides y es por lo general de
crecimiento lento. Hay ms de 15 000 casos nuevos de
leucemia cada ao. A menudo, las personas que son
diagnosticadas con esta enfermedad son mayores de 55
aos. Casi nunca afecta a nios. Leucemia mieloide crnica
(LMC). Afecta a clulas mieloides y por lo general es de
crecimiento lento al principio. Hay aproximadamente 5 000 casos nuevos de leucemia cada ao. Afecta principalmente a adultos.
Leucemia linfoctica (linfoblstica) aguda (LLA). Afecta a clulas linfoides y es de crecimiento rpido. Hay ms de 5 000 casos nuevos de
leucemia cada ao. La LLA es el tipo de leucemia ms comn entre nios pequeos. Tambin afecta a adultos. Leucemia mieloide aguda
(LMA). Afecta a clulas mieloides y es de crecimiento rpido. Hay ms de 13 000 casos nuevos de leucemia cada ao. Afecta tanto a
adultos como a nios. En forma general las LAL son positivas a la reaccin de PAS, mientras que las LAM son negativas. La clasificacin
inmunolgica en que los linfoblastos expresan antgenos correspondientes a las clulas linfocitarias B o T en distintas fases de
maduracin, aproximadamente 80% de las LAL. DIAGNOSTICO: Clnica; Inicio insidioso desde asintomtico o presentar sntomas y
signos atribuibles a cuadros virales de vas respiratorias altas, sospecha por descubrimiento incidental en una BH. Ms de 50%
presentan sntomas generales como fiebre elevada, anorexia, adinamia y malestar general con palidez de tegumentos. En 30%
manifestaciones de hemorragia como petequias, equimosis y epistaxis. Las manifestaciones de dolor seo son frecuentes y no siempre
corresponden con los hallazgos radiolgicos. Las manifestaciones neurolgicas o de tumores es poco frecuente y se ha asociado con
algunos tipos de especficos de leucemias como las LAL de clulas T. Laboratorio: Presencia de anemia, trombocitopenia y las cifras de
leucocitos pueden ser normales. El estudio de medula sea por aspiracin es la prueba diagnstica (90%), preferentemente se deber
efectuar en la espina iliaca posterosuperior, sin embargo se puede realizar en la espina anterosuperior, en general se muestra
celularidad aumentada, con sustitucin casi por linfoblastos. Despus de realizar los frotis se deber teir con tincin de Wright. Si en
la cuenta diferencial existen ms de 25% de linfocitos, se establece el diagnostico de leucemia. Adems se deben realizar pruebas de
funcionamiento heptico. Otro estudio obligado para el diagnstico y seguimiento es el de LCR en el que se buscan linfoblastos
MANUAL DE TRABAJO DEL CURSO ENARM CMN SIGLO XXI
CURSO ENARM CMN SIGLO XXI TEL: 36246001 Pharmed Solutions Institute PGINA 416

preferentemente, protenas y glucosa. Es obligado realizar estudio de inmunofenotipo para confirmar el diagnstico y linaje celular B o
T. TRATAMIENTO: Para LAL comprende 3 fases de tratamiento, va de la induccin a la remisin el objetivo es erradicar la carga de
linfoblastos de la MO, al trmino de esta fase que dura 28 das, la MO debe encontrase con menos de 5% de blastos, para considerar
que el paciente est en remisin; la segunda fase es de consolidacin; la tercera fase es la de mantenimiento en la que se indican
reducciones con la finalidad de que al trmino de esta no exista enfermedad medible en la MO. La duracin depender del grupo de
riesgo si es de bajo o alto grado, actualmente para una LAL de alto riesgo es de 30 meses, mientras que opera leucemias de bajo riesgo
es alrededor de 24 meses. La base del tratamiento es la quimioterapia: Algunos de los medicamentos comnmente usados para tratar
la leucemia en nios incluyen: Vincristina (Oncovin, Daunorubicina, tambin conocida como daunomicina (Cerubidina), Doxorrubicina
(Adriamycin), Citarabina, tambin conocida como arabinsido de citosina o ara-C (Citosar), L-asparaginasa (Elspar), PEG-L-asparaginasa
(pegaspargasa, Oncaspar), Etopsido (VePesid, otros). Hay que dar profilaxis al SNC, que se establece con la aplicacin de
quimioterapia intratecal. Actualmente se acepta que la indicacin para trasplante de MO para el paciente de LAC es en aquellos que
presentan una segunda remisin, esto quiere decir, pacientes que tienen una recada a la MO; pacientes con factores de riesgo elevado
como las alteraciones citogenticas del cromosoma Philadelphia t (9,22), t (4,11) entre las ms importantes, la indicacin de trasplante
debe ser una vez alcanzada la remisin. La ciruga tiene una funcin muy limitada en el tratamiento de la leucemia en nios. Como las
clulas leucmicas se propagan por toda la mdula sea y a muchos otros rganos a travs de la sangre, no es posible curar este tipo de
cncer con ciruga.

CASO CLINICO
Nio de 10 aos que acude a urgencias por dolor en hombro y codo izquierdos desde hace 4 das, con signos de inflamacin en codo
izquierdo. El dolor en hombro izquierdo cede pero a las 24 h aparece dolor en hombro derecho, con limitacin para la abduccin y
rotacin interna. No refiere traumatismos. El da anterior hizo ejercicio (waterpolo). Hace un mes, present un episodio de ojo rojo
diagnosticado de conjuntivitis aguda, que cedi con colirio antibitico. No refiere procesos infecciosos previos. Antecedentes
personales sin inters. En los antecedentes familiares destacan madre y to materno con espondilitis anquilosante HLA B27 positiva.
Exploracin fsica: Temp. 38C. Afectacin discreta del estado general. Inflamacin del codo izquierdo con derrame y limitacin
funcional de la flexo-extensin. Impotencia funcional de abduccin de hombro derecho. El resto de la exploracin es normal.

PREGUNTA
Cual es la conducta a seguir.

RESPUESTA
a.- Realizar BH, QS, EGO.
b.- Realizar pruebas CPK, GPT, GOT, GGT y DHL.
c.- Realizar serologa para micoplasma, Borrelia, Yersinia enterocolitica (serotipo O:3), parvovirus, VHA, VHC, RPR con rosa de Bengala.
d.- Realizar artrocentesis e iniciar antibitico e anti-infamatorio.

PREGUNTA
Se obtiene los siguientes resultados: Hemograma: leucocitos 9.930/mm3 (neutrfilos 53,3%, linfocitos 35,8%, monocitos 6%,
eosinfilos 2,1%), hemoglobina 14,2g/dl, VCM 80,6, plaquetas 308.000/l, protena C reactiva 5mg/dl; VSG 26mm/h. ioqumica: CP,
GPT, GOT y GGT normales, factor reumatoide 14 U/ml; anticuerpos antiestreptolisina (ASLO), inmunoglobulinas y C3 y C4 normales.
Serologas para Micoplasma, Borrelia, Yersinia enterocolitica (serotipo O:3), parvovirus, VHA, VHC, RPR con rosa de Bengala, negativas.
HLA B27 positivo. Radiografas de trax, codo izquierdo, hombro derecho y sacro-ilacas, normales. Hemocultivos y cultivo de lquido
articular, negativos.

Fue diagnosticado con artritis reumatoide juvenil, se ajusta tratamiento, cual es la conducta a seguir.

RESPUESTA
a.- Indica ibuprofeno.
b.- Indica prednisona.
c.- Inicia con sulfaxalazina.
d.- Inicia con cloroquina.

PREGUNTA
Tras 6 meses de seguimiento se inicia tratamiento con metotrexato por persistencia de los sntomas. Pasados 10 meses comienza con
astenia y dolor abdominal. En control analtico se objetiva neutropenia (320/l) y aumento de LDH hasta 588 U/l. Se realiza un nuevo
hemograma por fiebre que objetiva un 36% de blastos. Cual es la conducta diagnostica a seguir.

RESPUESTA
a.- Aspirado de medula osea.
b.- Escaneo con resonancia magnetica.
c.- Frotis y gota gruesa.
d.- Realizar Gamagrama.

RETINOBLASTOMA (RB). CIENCIAS BASICAS: Es un tumor intraocular embrionario de origen retiniano que se presenta generalmente en
nios menores de 5 aos. Dejado a su evolucin, el retinoblastoma es casi siempre fatal y un retraso en su tratamiento suele suponer
un pronstico visual precario, de ah la transcendencia del diagnstico precoz. SALUD PUBLICA: Representa aprox. El 3% de todos los
canceres presentes en nios menores de 15 aos. Su incidencia es de 1:15-20.000 recin nacidos vivos y su etiologa espordica en el
MANUAL DE TRABAJO DEL CURSO ENARM CMN SIGLO XXI
CURSO ENARM CMN SIGLO XXI TEL: 36246001 Pharmed Solutions Institute PGINA 417

70-75% de los casos o hereditaria, en el 25-30%. De acuerdo con la Direccin General de Epidemiologa de la Secretara de Salud, podra
representar la neoplasia slida ms frecuente despus de los tumores del sistema nervioso central. Generalmente aparece antes de los
2 aos de edad y 95% de los casos se diagnostican
antes de los 5 aos. PATOGENIA: El gen
responsable, el RB1, est localizado en el
cromosoma 13 y acta de forma dominante; es
decir, en clulas donde las dos copias del gen estn
daadas. El 35-40% de los pacientes corresponden a
casos hereditarios que son portadores de una
mutacin germinal. De ellos, ms de dos tercios
representan nuevas mutaciones (mutacin de
novo) sin historia familiar previa. Fenmenos
inflamatorios y glaucoma, ambos secundarios a un tumor que empuja hacia adelante el diafragma cristaliniano o a clulas tumorales
que azolvan la malla trabecular. Los sitios afectados ms frecuentes por metstasis son el SNC y el hueso (60%), dentro de la afecciono
sea el sitio ms frecuente fue la rbita y huesos craneales (90%), huesoso largos (18%). DIAGNOSTICO: Se presenta clnicamente con
leucocoria 50-60% (reflejo blanco en la pupila), estrabismo y signos inflamatorios, ms raramente con glaucoma, celulitis orbital, uveitis
y hemorragia vtrea. El 60% de los RB son unilaterales con una edad media al diagnstico de 24 meses, mientras que Los RB bilaterales
son menos frecuentes, suelen ser multifocales y de ms temprana edad de comienzo (15 meses). En la mayora de los nios con
tumores bilaterales, ambos ojos estn afectados al diagnstico. Slo en algunos casos de RB unilateral se desarrolla un tumor
contralateral ms tarde. Las caractersticas de inicio precoz, bilateralidad y multifocalidad apuntan a un probable origen hereditario del
proceso. Los retinocitomas o retinomas son tumores benignos resultantes de la regresin espontnea de retinoblastoma y que
raramente pueden ser descubiertos en la edad adulta como lesin residual de un RB que curs de forma subclnica en la infancia. Otros
tumores asociados: Los portadores de la mutacin germinal en RB1 tienen un exceso de riesgo de desarrollar otros tumores. La
presencia de tumores extraoculares (Pinealomas, osteosarcomas, sarcomas de tejido blando, melanomas) llamados segundos tumores
primarios, se manifiesta en la adolescencia o al inicio de la edad adulta, siendo el tiempo medio de aparicin del segundo tumor 10-13
aos. El diagnstico de RB se establece mediante oftalmoscopia indirecta. La CT, MRI y ecografa sirven para confirmacin del
diagnstico y estadio tumoral. En base a la focalidad tamao y localizacin de los tumores existe un estadiaje de Reese-Ellsworth (en
tabla adjunta) que evala el pronstico, I Muy favorable para la conservacin de vista. II Favorable para la conservacin de vista. III
Posible conservacin de vista. IV Desfavorable para la conservacin de vista. V Muy desfavorable para la conservacin de la vista.
TRATAMIENTO: La eleccin de la terapia adecuada depende no solo del estadio del tumor sino de factores tales como la focalidad
(unifocal, unilateral multifocal, o bilateral, localizacin del tumor dentro del ojo y edad del paciente. Las opciones incluyen: Enucleacin
est indicado en todos aquellos casos unilaterales que cumplen alguno de los criterios siguientes: (1) Ocupar ms de la mitad del ojo.
(2)Extensin al humor vtreo. (3)Desprendimiento total de retina. (4)Crecimiento de nuevos vasos en el iris y (5) Implicacin del tumor
en otras estructuras del ojo. La Radioterapia externa incrementa en un 50% la aparicin de otros tumores. Sin embargo, est indicado
en RB bilaterales no susceptibles de tratamiento local. Este mtodo se aplica en tumores mayores de 16 mm que se extienden a la
rbita, cercanos a la fvea, mltiples o cuando se extienden al humor vtreo. La radioterapia local puede ser efectiva en tumores
grandes (>16 mm) no susceptibles a la crioterapia o fotocoagulacin. Aunque reduce la probabilidad de aparicin de tumores inducidos,
est asociado a retinopatas y papilopatas. Fotocoagulacin, este tratamiento se aplica en tumores pequeos (menos de 3 mm de
dimetro y 2 de grosor) que no implican el disco ptico o la mcula. Llega a ser muy efectivo en tumores muy pequeos, con
localizacin anterior al ecuador y en tumores donde la altura es igual o menor que la mitad del dimetro de la base. Crioterapia, es
efectiva para el tratamiento de tumores de 5mm de dimetro y 3mm de grosor. Habitualmente son necesarias varias sesiones. Es de
creciente inters el empleo de la Quimioterapia sistmica como terapia coadyuvante en nios previamente candidatos a enucleacin o
external beal radiation bilateral. El empleo de agentes alquilantes puede incrementar la incidencia de segundos tumores y est
asociado al desarrollo de leucemia no linfoblstica e infertilidad. RETINOBLASTOMA TRILATERAL: Es un sndrome bien reconocido que
se presenta en 5 a 15% de los pacientes con la forma hereditaria del retinoblastoma y que se define mediante la formacin de un tumor
neuroblstico de la lnea media intracraneal, tpicamente hasta ms de 20 meses despus del diagnstico del retinoblastoma. Los
pacientes que son asintomticos al momento del diagnstico de un tumor intracraneal tienen un pronstico ms favorable que
aquellos pacientes sintomticos. Considerando que el retinoblastoma trilateral tiene un pronstico precario, y el corto intervalo entre
el diagnstico del retinoblastoma y la aparicin de la enfermedad trilateral, el uso de exmenes mediante neuroimagenologa de rutina
podran, potencialmente, detectar la mayora de los casos dentro de los primeros 2 aos del diagnstico inicial. Ya que no es claro si el
diagnstico temprano puede incidir en la supervivencia, la frecuencia de los exmenes de deteccin con imgenes de resonancia
magntica se recomienda para aquellos presuntos de tener una enfermedad hereditaria o para aquellos con enfermedad unilateral y
antecedentes familiares positivos, con una frecuencia de cada 6 meses por 5 aos. No es claro si esto tendr un efecto en el desenlace
o en la supervivencia. Se deben evitar las exploraciones por tomografa computarizada como mtodos de deteccin de rutina en estos
nios debido a la percepcin de riesgo sobre la exposicin con radiacin ionizante.

CASO CLINICO
Nia de 7 meses, previamente sana, presenta leucocoria de 2 meses de evolucin, no hay presencia de dolor ocular ni sntomas
concomitantes. Al examen oftalmolgico se constata ausencia de fijacin y esotropia del ojo derecho con ausencia de reflejo rojo.

PREGUNTA
Cual de los siguientes diagnosticos diferenciales es el mas frecuente?

RESPUESTA
a.- Enfermedad de Coats.
b.- Catarata.
MANUAL DE TRABAJO DEL CURSO ENARM CMN SIGLO XXI
CURSO ENARM CMN SIGLO XXI TEL: 36246001 Pharmed Solutions Institute PGINA 418

c.- Persistencia de vtreo primario hiperplsico.
d.- Estadio V de la retinopata de la prematuridad

TUMORES DEL SNC. CIENCIAS: Representan la segunda causa de neoplasias en la infancia. Los tumores cerebrales primarios son un
grupo diverso de enfermedades que, constituyen el tumor solido ms comn de la niez. Los tumores ms frecuentes son tres:
astrocitoma cerebeloso (33.9%), el meduloblastoma (26.3%) y el glioma del tronco cerebral. Otros tumores importantes de considerar
son el craneofaringeoma, tumeor de la regin pineal y ependimoma. SALUD PUBLICA: En Mxico no existen estadsticas, pero se estima
12%. En el INP, estos tumores ocupan el tercer lugar. Los tumores del sistema nervioso central (SNC) constituyen la segunda causa de
muerte en los menores de 15 aos, tan slo superada por la leucemia. PATOGENIA: Se desconoce la causa de los tumores cerebrales
infantiles pero existe una serie de factores predisponentes como: 1.- Genticos.- Neurofibromatosis, Esclerosis Tuberosa , Enfermedad
de von Hippel-Lindau, Sndrome de Li-Fraumeni, Sndrome de Turcot y el Sndrome de Nevo con Carcinoma de Clulas Basales, este
ltimo y aquellos que cursan con Tumor de Wilms son ms propensos a desarrollar meduloblastoma. 2.- Inmunolgicos.-
Inmunosupresin congnita o adquirida, individuos que sern sometidos a transplante renal ya sea antes o durante el mismo tienen
350 veces ms riesgo de desarrollar Sarcoma de clulas reticulares y Linfomas, los pacientes con ataxia-Telangiectasia cursan con
alteraciones inmunolgicas y mayor riesgo de desarrollar neoplasias. 3.- Ambientales.- Exposicin a Hidrocarburos Aromticos,
compuestos N-nitrosos, triazinas e hidrazinas. Una asociacin menos documentada es el consumo, durante el embarazo de
barbitricos, exposicin prenatal a rayos X, trauma, infeccin y anestsicos. El cncer se produce por mutaciones de genes que regulan
la proliferacin y muerte celular. Las mutaciones genticas pueden ocurrir dentro de la lnea germinal o como mutaciones somticas
exclusivamente dentro de clulas tumorales. Solo una pequea fraccin de nios con Tumores cerebrales tienen mutaciones
germinales adquiridas de sus padres o mutaciones nuevas. La causa de las mutaciones somticas en la mayora de todos los tumores
cerebrales son desconocidas. Existe evidencia de que el crecimiento y diseminacin de una neoplasia es dependiente de la angiognesis
(formacin de nuevos capilares sanguneos a partir del propio tumor lo que incrementa las posibilidades de incorporacin de clulas
neoplsicas a la circulacin). Cada incremento sucesivo en el volumen del tumor es precedido por una fase de angiognesis.
CLASIFICACION: De acuerdo a la localizacin: existe un predominio de los tumores infratentoriales (43.2%), seguido de los tumores
supratentoriales (40.9%), los de mdula espinal (4.9%) y de sitios mltiples (11%). Infratentoriales y, de estos, 75% estn situados en el
cerebelo o cuarto ventrculo. Entre los tumores infratentoriales comunes (fosa posterior) se encuentran los siguientes: 1.-
ASTROCITOMA CEREBELAR: tumor de mejor pronstico, generalmente de bajo grado. Existe un tumor especfico denominado
astrocitoma Pilocitico Juvenil, de excelente pronostico con terapia quirrgica. No requiere radioterapia, estos tumores se ubican en los
hemisferios cerebelosos y generalmente son qusticos. La porcin tumoral en realidad es un ndulo mural. La cavidad qustica visible en
TAC es reactiva. 2.- MEDULOBLASTOMA: forma parte de los tumores neuroectodrmicos primitivos, por originarse de esta clula
primordial, son muy agresivos y por ende de mal pronstico, puede dar metstasis incluso va LCR, se origina en la primera dcada de la
vida en el techo del IV ventrculo, por lo cual puede causar hidrocefalia precoz, clnicamente se presenta con cefalea y vmitos, los
signos sern relacionados con un sndrome cerebeloso, un nio con frecuentes cadas puede cursar una ataxia. Es de los tumores que
con ms frecuencia presenta diseminacionm extraneural (menos del 4%), siendo el hueso, mdula sea, linfticos, hgado y pulmones
los sitios ms comunes. Tratamiento quirrgico, radio y quimioterapia. 3.- EPENDIMOMA: ocurren mas frecuente en la regin del torax.
En raras ocasiones presentan malignizacion y en muchas ocasiones presentan asociacin con quistes intramedulares. Es un tumor
relativamente avascular y son encapsulados, es decir tienen un plano de clivaje que permite su extirpacin completa y curacin de los
pacientes. Un grupo de ellos crece en el interior de la medula y otro en el cono medular. 4. GLIOMA DEL TALLO ENCEFLICO: puede ser
de distintos grados, en general de mal pronstico dado su natuiraleza infiltrativa. Puede estar a nivel alto manifestndose por sndrome
cerebeloso e hidrocefalia. Pueden ubicarse a nivel bajo (bulbo medular), manifestndose por compromiso multiple de pares craneales
bajos y por compromiso motor y sensitivo de extremidades. La radioterapia asociada a corticoides, mejoran la calidad de vida. 4. Tumor
teratoide atpico. Los tumores supratentoriales incluyen aquellos que ocurren en la regin selar o supraselar del cerebro y en otras
reas del mismo. Los tumores selares y supraselares representan aprox., el 20% de los tumores cerebrales infantiles, incluso los
siguientes: 1. Craneofaringoma. 2. gliomas diencefalicos. 3 Tumores de clulas germinales. Otros tumores que ocurren de forma
supratentorial incluyen los siguientes: 1. Astrocitoma o glioma de bajo grado. 2. Astrocitoma de alto grado o maligno. 3. Gli oma mixto.
4. Oligodendroglioma. 5. Tumor neuroectodermico primario. DIAGNOSTICO: No existen cuadros patognomnicos para el diagnstico
de tumores cerebrales. Generalmente el inicio es insidioso y progresivo. Las manifestaciones clnicas de los tumores cerebrales
infantiles depende de: 1.- Localizacin del tumor (zonas elocuentes, obstruccin o desplazamiento) 2.- Tipo histolgico del tumor. 3.-
Edad y desarrollo del paciente. Los tumores intracraneales pueden manifestarse como dficit neurolgico focal, crisis convulsivas o no
convulsivas, aumento de la presin intracraneal, que puede deberse a efectos directos del tumor u obstruccin del lquido
cefalorraqudeo. Un anlisis de los sntomas tempranos de los tumores cerebrales infantiles mostr que los tumores supratentoriales se
presentan con vmito en el 46 % de los casos, con cefalea en el 43% de los casos; mientras que los Tumores Infratentoriales se
presentan con dificultades para la coordinacin en el 59 %, vmito 76% y cefalea 56 %. Debe sospecharse de un tumor cerebral en todo
nio con datos clnicos neurolgicos insidiosos y progresivos. DESCRIPCION SINDROMTICA: I) Hipertensin Intracraneana: este cuadro
puede manifestarse de manera aguda o crnica: Agudo: < 1 ao incremento inesperado del permetro ceflico, separacin de suturas,
alteraciones del estado de alerta (somnolencia, irritabilidad) disminucin en la ingesta. En preescolares, escolares y adolescentes puede
manifestarse con cefalea y vmito generalmente matutino que se exacerba con maniobras de Valsava y disminuye en el transcurso del
da. El vmito puede ser en proyectil, irritabilidad, letargia, edema de papila, discromatopsia (prdida de la visin de colores),
escotomas centrales y la paresia del sexto par, este ltimo inespecfico de topografa y explicado por la longitud del trayecto (falso
focalizador). Crnico o Intermitente: irritabilidad, letargia, vmito, atrofia de papila o prdida progresiva de la visin, cambios
conductuales, de personalidad, del rendimiento acadmico, anorexia y prdida o ganancia ponderal. II) Sndrome cefallgico: La cefalea
en tumores clsicamente se describe con un curso subagudo de 4 a 6 meses de duracin, de tipo pulstil, continua o intermitente.
Despierta al nio por la noche, frontal, occipital o universal, de moderada intensidad medida por escala anloga visual del dolor, o
porque el nio deja de hacer sus actividades. III) Crisis: Las crisis en los tumores generalmente son focales y pueden asociarse a
patrones bioeltricos anormales focales. IV) Sndrome cerebeloso: Este puede ser vermiano con ataxia troncal o de la marcha,
hemisfrico con lateropulsin derecha, izquierda o indistinta, dismetra, disdiadococinesia, lenguaje escndido (disartria), reflejos
MANUAL DE TRABAJO DEL CURSO ENARM CMN SIGLO XXI
CURSO ENARM CMN SIGLO XXI TEL: 36246001 Pharmed Solutions Institute PGINA 419

osteotendinosos pendulares con o sin nistagmus horizontal. V) Neuropata craneal: El involucro de pares craneales dependiendo de
cul sea, es sugestivo de la topografa de la lesin sin embargo esto puede ser muy sutil en virtud de que los tumores son ms
compresivos que destructivos. Entendiendo por neuropata del III al XII par craneal. Los estudios necesarios para realizar el diagnostico
son la TAC, siendo posible realizar el diagnostico hasta en 90%, la imagen por IRM, mielografa, mielotomografa, en caso de tumor
como el meduloblastoma y ependimoma: siendo el diagnstico definitivo el estudio histopatolgico, obtenindose a travs de
reseccin quirrgica, con una toma de biopsia y exresis del tumor, as como por biopsias por esterotaxia. Existe una ruta crtica para la
evaluacin de un nio con sospecha de neoplasia en SNC: Sospecha de tumor primario estudios de laboratorio y gabinete
interconsulta con neurociruga y oncologa procedimiento diagnostico-quirrgico clasificacin histopatolgica estadificacion
valoracin radioterapia valorar inicio de esquema de quimioterapia seguimiento. TRATAMIENTO: Los cnceres dejados a su
evolucin natural son incontrolables y mortales en la mayora de los casos. Se basa en la desaparicin de los sntomas, as como el
tratamiento especfico, por lo que es importante individualizar cada caso, algunos pacientes requerirn el empleo de anticomisiales
(Fenitona 20 mg/kg/dosis de impregnacin y mantenimiento 7 a 10mg/kg/da en dos dosis), as como de esteroides para el manejo de
hipertensin craneal, ya que algunos autores mencionan que los esteroides tienen efecto vaso gnico, aumentando la permeabilidad
vascular, corrigiendo e edema cerebral y por tanto mejorando el flujo circulatorio de forma transitoria. En cuanto al tratamiento
especfico, la ciruga debe realizarse por el especialista experto en patologa tumoral. En ocasiones los pacientes requieren colocacin
de vlvulas ventriculoperitoneales para el manejo urgente de la hidrocefalia y posteriormente la ciruga de reseccin que siempre es
recomendable sea completa, pero suele ser imposible. La radioterapia es una modalidad de tratamiento necesaria para los tumores
malignos de SNC, los pacientes menores de 3 aos presentarn detencin del crecimiento y la posibilidad de retardo psicomotor,
disfuncin endocrina y radionecrosis. La quimioterapia es usada para las neoplasias del SNC, pero no todas son sensibles debido a la
barrera hemartoencefalica

CASO CLINICO
Paciente de 9 aos de edad, sexo masculino, que consult por cuadro de 3 das de evolucin caracterizado por cefalea holocrnea de
tipo opresivo y vmitos explosivos aproximadamente 6 veces al da. Durante la anamnesis la madre refiri que el paciente presentaba
crisis de cefalea intermitente hace aproximadamente 5 meses, las que con el tiempo fueron aumentando en frecuencia e intensidad
hasta constituir el cuadro de consulta actual. Al examen fsico general el nio impresionaba en buenas condiciones generales, y se
apreciaron varias lesiones vesiculares y costrosas en todo el cuerpo, compatibles con cuadro de varicela en sus ltimas etapas. Al
examen neurolgico destac paciente con un puntaje de 15 en la Escala de Glasgow, dismetra de extremidad superior derecha y leve
paresia facial izquierda.

PREGUNTA
Cual es la conducta inmediata a seguir mas adecuada?

RESPUESTA
a.- Realizar puncion lumbar.
b.- Realizar tomografa computada.
c.- Medidas anti-edema.
d.- Colocacion de derivacin ventrculo peritoneal.

CASO CLINICO
Se trata de RN femenino, nacido por cesarea con 38 SDG por Capurro, apgar 8/9, sin datos de hidrocefalea, permetro ceflico de 39.5
cm. Fontanelas anterior y posterior ampliar y plenas, con diastasis de la sutura sagital, tono muscular cervical disminuido,
cardiopulmonar sin compromiso. Movilidad expontanea en las cuatro extremidades, discreta hiperreflexia miotatica, babinski
espontaneo bilateral y reflejos primitivos globales disminuido, La TC mostro hidrocefalia supratentorial sin edema transependimario,
dilatacin de tercer ventrculo y en la fosa posterior una imagen hiperdensa ovalada de 46 x 42 mm que obliteraba el cuarto ventrculo
en su parte posterior, se reporto una lesin tumora infratentorial probablemente intraaxial glial o extraaxial de origen
mesenquimatoso. La RM reporto imagen hiperdensa en relacin con el parnquima cerebral en la regin pancerebelosa que infiltraba
el tallo cerebral en el puente, mesencfalo y lamina cuadrigeminal, que protruia por la hendidura tentorial enceflica y borra las foliar
cerrebelosas. Por su efecto compresivo sobre acueducto de Silvio, adems de ocacionar la dilatacin ventricular supratentorial con
hidrocefalia obstructiva.

PREGUNTA
Cual es el diagnostico mas probable?

RESPUESTA
a.- Astrocitoma congnito.
b.- Meduloblastoma congnito.
c.- Ependimoma congnito.
d.- Craneofaringeoma congnito.

PAROTIDITIS. CIENCIAS BASICAS: La parotiditis (paperas) es la inflamacin de las glndulas salivares sobre todo de las partidas. Es una
enfermedad generalizada, viral, aguda y altamente contagiosa y que generalmente ocurre en la infancia. Se transmite por contacto
directo e ingresa al organismo por va respiratoria y usualmente tarda en aparecer de dos a tres semanas. Los casos de paperas en
nuestro medio han disminuido dramticamente con la introduccin de la vacuna contra las paperas. SALUD PUBLICA: Es una
enfermedad endemo-epidmica, con perodos de aparicin que oscilan entre 2 y 6 aos. Se estima que en ausencia de inmunizacin, el
85% de los adultos ha sufrido la enfermedad; una tercera parte de ellos sin sntomas evidentes (la mayora antes de los dos aos de
MANUAL DE TRABAJO DEL CURSO ENARM CMN SIGLO XXI
CURSO ENARM CMN SIGLO XXI TEL: 36246001 Pharmed Solutions Institute PGINA 420

vida). Se presenta en invierno y primavera. La afectacin ha disminuido en los pases que incluyeron la vacuna en forma rutinaria en sus
planes de inmunizacin. PATOGENIA: El agente es un paramyxovirus (emparentado con los de influenza y parainfluenza), el virios esta
formado por RNA, el virus penetra por una via respiratoria. Tiene como reservorio al hombre y el modo de transmisin es persona a
persona (diseminacin de gotitas y saliva del infectado). La incubacin dura de 12 a 25 das (media de 18) y el perodo de contagio va
desde 6-7das antes y hasta 9 das despus del edema glandular (la mxima contagiosidad, ocurre 48 horas antes de las manifestaciones
clnicas de la enfermedad). Los expuestos no inmunes, deben ser considerados infecciosos desde el 12 al 25 da de la exposicin. La
inmunidad suele ser permanente. DIAGNOSTICO: Sialoadenitis, la glndula que se afecta casi en forma constante es la partida, puede
comenzar bruscamente con tumefaccin y dolor en la regin parotdea, la piel suprayacente aparece tensa, pero no roja y sensible a la
presin, consecutivamente el lbulo de la oreja es empujado hacia arriba y fuera. Ocurre enrojecimiento y tumefaccin del conducto de
Stenon. Se caracteriza por fiebre, edema y dolor de una o ms glndulas salivales (habitualmente las partidas), dificultad para masticar
a veces para hablar, cefalea. Por lo general es leve y autolimitada. Puede ser uni o bilateral (70%), alcanza su mxima expresin al tercer
da remite lentamente en un plazo de tres a siete das. Si bien el compromiso generalmente es parotdeo, con cierta frecuencia las
glndulas submaxilares tambin estn afectadas, con mucho menos frecuencia ocurre lo mismo con las glndulas sublinguales.
Orquiepididimitis se presenta en 20-30% de los varones pospuverales, es la manifestacin extrasalival ms frecuente, es unilateral y se
instaura al ceder la tumefaccin parotdea, con un nuevo ascenso febril COMPLICACIONES: Meningoencefalitis (inflamacin de las
membranas que cubren el cerebro y la mdula espinal). Es la ms frecuente dentro de la infancia. 1 por cada 400 casos, y ms a
hombres que a mujeres, en proporcin de 1 a 4. Evoluciona en la mayora de los casos hacia la curacin espontnea en un perodo de
cuatro das, sin dejar, secuelas. Orquitis (inflamacin del testculo). Muy poco frecuente en la infancia, en la edad mayor se observa en
un 30% de los casos. Comienza generalmente siete das despus de la parotiditis, con fiebre, calofros, dolor de cabeza y dolor en los
testculos y en la parte baja del abdomen. En un porcentaje bajo es bilateral y en tercio evoluciona hacia la atrofia testicular. Sordera.
Se afecta ms severamente la audicin de tonos altos observando la recuperacin en pocas semanas. Es de baja incidencia: Oforitis,
pancreatitis - inflamacin del pncreas. El virus puede aislarse en saliva desde una semana antes hasta 4-5 das despus del inicio de la
parotiditis. Se le asla en cultivos de tejidos humanos o de primates y en huevos embrionados. Hoy en da se prefiere la tcnica de
ELISA. La VSG permanece normal excepto en caso de orquitis o pancreatitis. TRATAMIENTO: limitado generalmente a la administracin
de medicamentos para aliviar el dolor y la ingestin abundante de lquidos. Algunas veces el reposo en cama es necesario durante los
primeros das. Los nios no deben asistir a la escuela hasta que los sntomas hayan desaparecido, es necesario recomendar lquidos
orales, y descartar los cidos, limn, naranja etc. La orquitis se tratara con medidas locales y reposo en cama. El tratamiento de la
meningoencefalitis es puramente sintomtico con analgsicos. PROFILAXIS: La globulina inmunitaria especfica antiparotiditis no
previene la parotiditis y ya no se fabrica. Se dispone de una vacuna elaborada con virus vivos atenuados de la parotiditis que se ha
mostrado muy eficaz (95%) en la profilaxis de la enfermedad. Debe administrarse durante el segundo ao de vida o bien antes de la
pubertad.

CASO CLINICO
Paciente varn de 4 aos y medio, remitido por su pediatra para estudio por haber presentado, en los ltimos 9 meses, cuatro
episodios de tumefaccin parotdea derecha, de 3-4 das de evolucin cada uno, que cedan con tratamiento antiinflamatorio y
antibitico. Se asociaba sensacin dolorosa de la zona, febrcula y, en una ocasin, supuracin mucopurulenta por el conducto de
Stenon. No destacaban antecedentes personales de inters y el calendario vacunal (incluido parotiditis) estaba correcto. La exploracin
fsica en el momento de la consulta (intercrisis) era normal.

PREGUNTA
Cuales son los estudios mas adecuados para buscar un diagnostico especifico.

RESPUESTA
a.- BH, QS, EGO.
b.- Radiografia de crneo y cara.
c.- Serologa para VEB, citomegalovirus, adenovirus y VIH.
d.- Factor reumatoide y anticuerpos antinucleares (ANA, anti-Ro y anti-La).

PREGUNTA
Cual de las siguientes complicaciones es la ms frecuente.

RESPUESTA
a.- Meningoencefalitis.
b.- Orquitis/orquiepididimitis.
c.- Pancreatitis.
d.- Miocarditis.

HEPATITIS. CIENCIAS BASICAS: Indica proceso inflamatorio del hgado caracterizado por hepatomegalia, anorexia, molestis
abdominales fundamentalmente gstricas, funcin heptica anormal, heces despigmentadas y orina oscura. Estas alteraciones pueden
ser ocasionadas por infecciones producidas por bacterias, virus o parsitos o bien por ingestin excesiva de alcohol o frmacos,
trastornos de tipi autoinmunitario o toxico. La mayor parte de las hepatitis son de origen viral. La hepatitis crnica se define como una
lesin inflamatoria del hgado que puede progresar a lesin mas grave y a cirrosis, las causas ms frecuentes son: virales, autoinmunes,
toxicas y criptogenicas. En general se habla de cronicidad con una evolucin de ms de 6 meses, con excepcin del caso de las hepatitis
autoinmunes, en las cuales un diagnstico ms temprano, permitir detectar el padecimiento antes de que se desarrolle cirrosis. Se
sospecha una hepatitis crnica ante una sospecha de recada de hepatitis aguda, persistencia de hepatitis aguda por ms de 3 meses,
datos clnicos de enfermedad heptica crnica y en pacientes asintomticos de alto riesgo, con historia de transfusiones mltiples,
MANUAL DE TRABAJO DEL CURSO ENARM CMN SIGLO XXI
CURSO ENARM CMN SIGLO XXI TEL: 36246001 Pharmed Solutions Institute PGINA 421

como son pacientes con leucemias en remisin, hemofilia, hemoglobinopatas, IRC en programa de dilisis, y neoplasias en los que
existe el riesgo de hepatitis post- transfusional. SALUD PUBLICA: En Mxico que 95% de las hepatitis en nios menores de 215 aos de
edad es producida por el virus de hepatitis A. Cada ao se registran aproximadamente 1,4 millones de casos de hepatitis A en todo el
mundo. La hepatitis A se asocia a falta de agua salubre y a un saneamiento deficiente. Las epidemias se pueden propagar de manera
explosiva y causar prdidas econmicas considerables. Las mejoras del saneamiento y la vacuna contra la hepatitis A son las medidas
ms eficaces para combatir la enfermedad. HEPATITIS POR VIRUS A: La seccin heptica ms frecuente, pertenece al grupo de los
picornavirus de los enterovirus, y su husped natural es el ser humano, el genoma est constituido por RNA. La hepatitis A se presenta
en nios entre los 3-12 aos de edad, con promedio en la edad preescolar. La va de transmisin ms comn es la del tubo digestivo a
travs de alimentos o bebidas que se contaminan con matera fecal que contienen el virus. La contaminacin por va area no se ha
confirmado. La transmisin tambin se da en los drogadictos que comparten agujas y jeringas para uso intravenoso. El mayor riesgo de
diseminacin del virus es en heces, ocurre 15 das posteriores al inicio de la enfermedad. El estado de portador no existe. La inmunidad
con la infeccin es permanente. No se ha informado hasta la fecha evolucin a la cronicidad. Clnica; Los nios menores de 2 aos de
edad, cursan sin ictericia en 90% de los casos. Antes de la aparicin de la ictrica se presentan los siguientes sntomas, febrcula,
anorexia, vomito, debilidad, cefalea, dolor difuso en abdomen y esplenomegalia. En la fase ictrica cuya duracin puede ser de 8 das a
4 semanas, se hace aparente coluria, hipocolia, decaimiento y anorexia ms acentuadas, con presencia de tinte amarillento en
conjuntivas y piel. Laboratorio: Aumento de la bilirrubinas a expensas de la bilirrubina directa, las transaminasas, se encuentran
elevadas, la colinesterasa y la colestasa estn disminuidas, las pruebas de floculacin de protenas como es la turbidez al timol se
encuentra elevada, ocurre aumento de la gammaglobulina con disminucin de albumina. Se encuentran alteradas la concentracin de
inmunoglobulinas los tiempos de protrombina y la concentracin de glucosa. Las pruebas serolgicas especficas como es la deteccin
de anticuerpos contra el virus de la hepatitis A por radioinmunoanlisis, sugieren infeccin reciente o convaleciente. Tratamiento: Las
medidas son de tipo sintomtico, ya que no existe medicamentos especficos para el virus de hepatitis A. Dieta; implica restriccin de
protenas, de grasas y aumento en los hidratos de carbono. Reposo; mnima actividad para mantener un flujo circulatorio esplacnico y
heptico adecuado. Aislamiento y proteccin a contactos; precauciones con el manejo de excretas durante el periodo ictrico y 2
semanas despus. Aplicacin de gammaglobulina estndar al 16% a todo contacto a razn de 0.2 a 0.12 ml/kg por va IM en dosis nica
como inmunizacin pasiva. Inmunizacin: Debe aplicarse la vacuna contra hepatitis A en presentacin peditrica, dos dosis con
intervalos de 1 mes con refuerzo a los 12 meses, la vacuna disponible en Mxico precede de la cepa HM75 inactivada. Son 360 unidades
en 0.5ml y se aplica va IM en nios mayores de 1 ao de edad. VIRUS DE HEPATITIS D: Se trata de un virus RNA defectuoso, formado
por RNA del agente, cubierto por antgeno de superficie del virus de la hepatitis B, sin este virus no resulta infectante. Se transmite por
va parenteral o va percutnea, en mucosas, transfusin sangunea, contacto sexual y etapa perinatal. La diseminacin es por
portadores y afecta ms al adolescente. Su periodo de incubacin es de 15-200 das. El 75% del diagnstico inicial se hace en la fase
cirrtica y por lo tanto tiende a la fase de cronicidad. En laboratorio a parte de la BH, transaminasas, bilirrubinas y EGO, debe hacerse
cuantificacin de anticuerpos antivirus de la hepatitis (anti-VHD) en donde se encuentran titulaciones mayores de 1:100, las
titulaciones de IgM se encuentran altas en fase temprana, adems se cuantifican anticuerpos del virus de la hepatitis B (Ags-VHB).
Tratamiento: Se ha utilizado el interfern sin obtener resultados satisfactorios. No existe vacuna alguna, sin embargo algunos autores
sugieren usar la vacuna contra el virus de la hepatitis B.

CASO CLINICO
RN pretrmino de 36,6 semanas de EG, peso 2.555 g, por depresin neonatal y riesgo infeccioso. APGAR 5/9. La madre present, 24 h
antes del parto, un cuadro febril acompaado de dolor abdominal. Por la sospecha clnica de corioamnionitis recibi tratamiento
antibitico y se indujo el parto, que finaliz en cesrea con anestesia general por no progresin.

PREGUNTA
Cual es la conducta a seguir mas adecuada.

RESPUESTA
a.- Egresar con vigilancia estrecha.
b.- Hemocultivo e inicio de antibitico.
c.- BH, QS, EGO.
d.- Analisis de LCR.

PREGUNTA
En las siguientes 48 h se deteriora su estado general, con acidosis metablica, anemia, plaquetopenia (recuento mnimo 10.000/mcl) y
hemorragia digestiva baja, por lo cual se modifica el tratamiento antibitico, para cubrir una posible enterocolitis necrozante, y se
suspende la alimentacin oral. En las 24 h posteriores desarrolla coagulopata y fallo heptico; presenta petequias generalizadas y
sangrado importante por puntos de canalizacin venosa umbilical y perifrica. Cual es la conducta a seguir.

RESPUESTA
a.- Transfusin de hemoderivados.
b.- Administracin de vitamina K (1 mg/kg/da) durante 72 h.
c.- Pruebas de funcin heptica.
d.- Administracion de albumina.

PREGUNTA
Se obtienen los siguientes resultados donde alcanza concentraciones mximas de transaminasas GOT 957 U/L, GPT 237 U/L y GGT 144
U/L, FA 93 U/L, LDH 5031 U/L, bilirrubina directa: 1,66 mg/dl, con disminucin de las protenas totales (3,75 g/dl) y albmina (2,19 g/dl).
MANUAL DE TRABAJO DEL CURSO ENARM CMN SIGLO XXI
CURSO ENARM CMN SIGLO XXI TEL: 36246001 Pharmed Solutions Institute PGINA 422

Por la sospecha clnica de viremia, y ante la posibilidad de infeccin herptica, se inicia tratamiento con aciclovir. Cual de los siguientes
patologias es el mas frecuente evaluar para el diagnostico diferencial.

RESPUESTA
a.- Hepatitis neonatal.
b.- Infeccion por VEB, CMV.
c.- Enterovirus y adenovirus.
d.- Toxoplasma, rubola y sfilis.

INFLUENZA. CIENCIAS BASICAS: Es una infeccin viral aguda de las vas respiratorias, altamente contagiosa, que puede afectar la
mucosa nasal, la farngea, bronquios y en ocasiones hasta los alvolos pulmonares. Los sntomas son parecidos a los del catarro comn
o resfriado; sin embargo, son ms graves y su inicio es generalmente abrupto. La gripe es causada por el virus de la influenza (virus de
influenza A, B y C), el catarro comn es causado por mltiples virus respiratorios (rinovirus, coronavirus, adenovirus, virus
parainfluenza). El cuadro clnico de la influenza puede variar, observndose desde un cuadro febril agudo leve hasta una infeccin
pulmonar y/ocomplicaciones graves. SALUD PUBLICA: Puede tener un comportamiento endmico, epidmico o de una pandemia. Las
epidemias de influenza son responsables de 36 000 a 50 000 muertes por ao en pases como EUA en los ltimos aos, afecta a todos
los grupos etarios, pero principalmente a nios menores de dos aos y adultos mayores de 65 aos. PATOGENIA: Los virus asociados a
la influenza humana son miembros de la familia Orthomyxoviridae, tienen una morfologa helicoidal y su genoma est constituido por
una cadena segmentada de ARN, los virus influenza A y B causan infecciones respiratorias en humanos y animales, el virus A es causa de
pandemias. El virus de influenza C afecta en forma moderada a los nios y en forma ocasional a los adultos. Estos virus se distinguen
por variaciones antignicas en dos protenas estructurales (la nucleoprotena y la protena de la matriz). Los virus de la influenza A se
clasifican en subtipos de acuerdo a dos antgenos de superficie: hemaglutinina (H) y neuraminidasa (N). La hemaglutinina es
considerada el antgeno mayor para la cual est dirigida la produccin de anticuerpos neutralizantes y cuya funcin es la adhesin del
virus mediante residuos de cido silico en la superficie del epitelio respiratorio humano. La expresin de la neuraminidasa es menos
abundante en la superficie viral y su papel es facilitar la liberacin de viriones de clulas infectadas del hospedero. Para la clasificacin
de este virus se utiliza el siguiente orden en la nomenclatura: el tipo, el lugar en donde fue aislado, el ao de aislamiento, el nmero de
identificacin del laboratorio y, en los aislamientos del virus de la influenza A, el subtipo H-N (por ejemplo: A/Moscow/10/99 [H3N2]).
Estudios de epidemiologa molecular de los virus de la influenza han demostrado que los subtipos H1N1, H1N2, y H3N2 son los que han
circulado en los ltimos aos. Las aves constituyen un reservorio potencial de intercambio gentico para los virus de la influenza, lo que
representa un riesgo latente de pandemias. Los virus de la influenza A tambin infectan a pollos, cerdos, caballos y ocasionalmente a
mamferos marinos. El periodo de incubacin es de 1 a 4 das (promedio de dos das). Los adultos pueden ser infectantes un da antes
de que los sntomas inicien hasta aproximadamente 3-5 das despus. Los nios pueden ser infectantes por 10 das o ms. Las personas
con estados de inmunosupresin pueden excretar los virus por perodos ms prolongados. El virus de la influenza A se caracteriza por
causar enfermedad moderada a grave. El virus B de la influenza causa cuadros clnicos menos graves que el tipo A y tradicionalmente
produce infeccin y enfermedad durante la infancia. La capacidad del virus de la influenza A y B de sufrir cambios antignicos graduales
en sus dos antgenos de superficie, la hemaglutinina y la neuraminidasa, complica la vacunacin contra esta enfermedad. El tipo de
variaciones antignicas menores o deslizamientos antignicos (antigenic drift) resulta de la acumulacin de mutaciones puntuales de
los genes que transcriben para estas protenas. DIAGNOSTICO: La influenza se caracteriza por el inicio agudo de sntomas y signos
respiratorios que incluyen: fiebre (elevada de inicio abrupto), tos seca, coriza, cefalea, odinofagia, ardor farngeo, mialgias, artralgias, y
ataque importante al estado general (postracin H3N2). Estos sntomas ocurren en 50 a 70% de las infecciones por influenza, tanto
estacional como por el virus de influenza humana H1N1 2009. Menos comunes son: fotofobia, dolor abdominal, nusea, vmito y
diarrea. La duracin de la enfermedad sin complicaciones habitualmente es de una semana, aunque la tos y debilidad pueden persistir
por ms de 14 das. En particular, en los pacientes con infeccin por el virus A H1N1, se presentaron durante la epidemia cuadros
graves de neumona intersticial y progresin a pulmn de choque. Para la infeccin por el virus influenza A humana H1N1 2009, se
establecieron definiciones epidemiolgicas para la confirmacin de los casos: los confirmados son aquellos que tienen sntomas y
signos arriba mencionados y una prueba confirmatoria positiva que consiste en reaccin de polimerasa en cadena en transcripcin
reversa (RT-PCR) en tiempo real o cultivo viral. Un caso probable es una persona sintomtica que es positiva para influenza A por
prueba rpida, pero negativa para H1 y H3 por RT-PCR. Una persona sospechosa es aquella que no cumple con la definicin de probable
o confirmado, no tiene prueba negativa para el virus nuevo, y: Es un individuo previamente sano, menor de 65 aos de edad,
hospitalizado por infeccin respiratoria baja. Vive en un lugar o estado donde no hay casos confirmados pero viaj recientemente a
donde existe uno o ms casos probables o confirmados. Tuvo contacto en los siete das previos con un caso confirmado o probable.
Laboratorio: Idealmente, la muestra de exudado farngeo, nasofarngeo, lavado nasal, aspirado bronquial o traqueal debe tomarse en
los primeros tres das del inicio de los sntomas. El cultivo viral es el estndar de oro, adems permite que el virus sea tipificado y
caracterizado antignicamente. Los medios de cultivo utilizados son huevos embrionados de gallina o el cultivo de rin canino de
Madin-Darby, el rin de chimpanc y otros. Tambin se puede realizar el diagnstico por determinaciones serolgicas al mostrar un
incremento de cuatro veces en la titulacin de anticuerpos contra influenza en una segunda muestra. El suero en la fase de
convalecencia debe obtenerse entre los das 10 a 21 del inicio del cuadro. Las tcnicas serolgicas ms frecuentemente empleadas son
las de neutralizacin y la inhibicin por hemaglutinacin. COMPLICACIONES: Neumona primaria por influenza o neumona bacteriana
secundaria. La etiologa ms frecuente de la neumona bacteriana es: Streptococcus pneumoniae en 48% de los casos, Staphylococcus
aureus en 19%, y Haemophilus influenzae no tipificable en 11%. Otras complicaciones del tracto respiratorio incluyen: sinusitis
bacteriana, bronquitis, traqueobronquitis y otitis media. En nios se ha asociado a otras manifestaciones clnicas como son:
convulsiones febriles, encefalopata por el virus o encefalopata asociada a la utilizacin de salicilatos (sndrome de Reye), miositis,
rabdomilisis, miocarditis y pericarditis. En pocos casos se informa de complicaciones en sistema nervioso central como mielitis
transversa y polirradiculoneuritis. TRATAMIENTO: Permanecer en casa, en reposo para mejorar los sntomas y evitar contagio a otras
personas. En casos leves a mderados; ofrecer lquidos abundantes, aumentar el consumo de frutas con mayor contenido de vitamina A
y C. No se recomienda suspender la lactancia materna si la madre enferma. Deben evitarse los lugares concurridos, as como cambios
MANUAL DE TRABAJO DEL CURSO ENARM CMN SIGLO XXI
CURSO ENARM CMN SIGLO XXI TEL: 36246001 Pharmed Solutions Institute PGINA 423

bruscos de temperatura, tabaquismo y exposicin a contaminantes dentro de la casa. El control de la fiebre por medios fsicos, evitando
en los nios el uso de salicilatos (por la asociacin con el sndrome de Reye). Cuando amerite, se podrn administrar paracetamol o
AINES con efecto antipirtico (ibuprofeno). No se recomienda utilizar antibiticos profilcticos. Es importante hacer hincapi en los
datos de alarma que sugieren el agravamiento y necesidad de manejo hospitalario. En la edad peditrica los sntomas incluyen: fiebre
persistente, tos productiva con expectoracin, dificultad para respirar, rechazo al alimento, hipotona o convulsiones. Medicamentos
con utilidad clnica al reducir la duracin de los sntomas cuando son empleados en las primeras 48 horas del inicio de la enfermedad.
Tambin pueden ser efectivos en la quimioprofilaxis. La amantadina y la rimantadina inhiben la replicacin de los virus de influenza A,
pero no los de influenza B, se administran por va oral y se utilizan para el tratamiento o quimioprofilaxis, disminuyen la excrecin viral
y reducen en promedio un da la duracin de la enfermedad. El tratamiento se recomienda por cinco das y cuando se utilizan para
quimioprofilaxis, son efectivos en 70-90%. Se recomienda el uso profilctico de la amantadina y rimantadina para contactos o
convivientes. Los inhibidores de la neuraminidasa (zanamivir, oseltamivir) tienen actividad contra influenza A y B, bloqueando el sitio
activo de la neuraminidasa, disminuyen la excrecin del virus y reducen la duracin de los sntomas de la influenza por 36 horas. En
ensayos clnicos se ha encontrado que el oseltamivir disminuye las complicaciones de las vas respiratorias bajas (neumona y
bronquitis), disminuye uso de antibiticos y el riesgo de hospitalizacin. PREVENCIN: La vacunacin anual de personas en grupos de
alto riesgo de desarrollar complicaciones y sus contactos, representa la principal estrategia. Las vacunas de mayor uso son producidas
de virus crecidos en huevos embrionados inactivados con formaldehido o propiolactona, pueden producirse de virus completos
tratados con detergentes o de antgenos de superficie (hemaglutinina y neuraminidasa) purificados. Son ms recomendadas las que
contienen antgenos purificados, ya que dan menos reacciones alrgicas. Habitualmente contienen los tres tipos virales recomendados
por la OMS. Diversos estudios han demostrado una eficacia de 80% en nios y de 77% en adultos. Las vacunas contra la influenza son
recomendadas en Mexico para: a) Vacunacin a personas que pueden transmitir influenza a personas en grupos que estn propensos a
desarrollar complicaciones: 1) Vacunacin a personal de salud (personal mdico, enfermera, paramdico). 2) Personal que labora en
asilos de ancianos y casas de estancia. 3) Personas que viven en contacto intradomiciliario con personas que tienen alto riesgo de
desarrollar complicaciones (nios que viven con un adulto con asma). b) Vacunacin de mujeres con embarazo de alto riesgo:
especficamente aquellas en el segundo y tercer trimestre del embarazo por tener el mayor riesgo de complicaciones. c) Vacunacin de
personas mayores de 65 aos de edad. Este grupo presenta el mayor riesgo de hospitalizacin y muerte asociado a complicaciones por
influenza. d) Vacunacin de personas de entre 50 a 64 aos. En este grupo se concentra la mayor pre-valencia de personas con
condiciones de alto riesgo (enfermedades crnico-degenerativas). e) Vacunacin de pacientes de cualquier edad con padecimientos
crnicos. Estos padecimientos incluyen: enfermedades pulmonares (asma o enfermedad pulmonar obstructiva crnica) o
enfermedades cardiovasculares (insuficiencia cardiaca). As como aquellos con enfermedades metablicas crnicas (diabetes mellitus,
insuficiencia renal) y en pacientes inmunodeprimidos (VIH positivos o trasplantados). f) Vacunacin de nios de 6 a 36 meses de edad.
Se ha demostrado que en esta poblacin existe un riesgo aumentado de hospitalizacin por complicaciones asociadas a la influenza.
Aunque el riesgo en nios menores de seis meses es an mayor, la vacuna trivalente no est aprobada para su uso en este grupo. g)
Viajeros. Se recomienda en aquellos viajeros que no fueron vacunados en el ao precedente si planean viajar a los trpicos, o al
hemisferio sur durante los meses de abril a septiembre. Medidas generales: Las medidas no farmacolgicas que se recomiendan para
ayudar a prevenir la propagacin de enfermedades respiratorias como la influenza son: Cubrirse la nariz y la boca con un pauelo
desechable al toser o estornudar. Tirar el pauelo desechable a la basura despus de usarlo. Lavarse las manos frecuentemente con
agua y jabn, especialmente despus de toser o estornudar. Puede utilizarse alcohol-gel. Evitar tocarse los ojos, la nariz o la boca. Evitar
el contacto cercano con personas enfermas. Al enfermarse, debe permanecer en casa por siete das a partir del comienzo de los
sntomas o hasta que hayan pasado 24 horas desde que desaparecieron. Esto disminuye la posibilidad de infectar a otras personas y
propagar ms el virus. Se deben seguir las recomendaciones de la Secretara de Salud con relacin al cierre de escuelas, evitar
frecuentar sitios con multitudes y tomar medidas de distanciamiento social.

CASO CLINICO
Paciente de 17 aos con fiebre, tos y dolor de garganta, cefalea, rinorrea y dolor abdominal, por lo que iniciaron tratamiento
sintomtico para rinofaringitis con respuesta no favorable por lo que regresa a los 3 dias con empeoramiento de los sntomas.

PREGUNTA
Cuales son los sntomas o signos cardinales para considerar caso sospechoso de influenza.

RESPUESTA
a.- Fiebre con tos o dolor de garganta.
b.- Cefalea, rinorrea y coriza.
c.- Artralgias, mialgias y postracin.
d.- Dolor torcico, dolor abdominal y congestin nasal.

PREGUNTA
Cuales son las carateristicas que presentan mayor complicaciones.

RESPUESTA
a.- Edad >60 aos y <2 aos.
b.- Enfermedad crnica o debilitante.
c.- Embarazo y primeros 6 meses postparto
d.- Pacientes que acudan a una segunda consulta por deterioro o sin mejora clnica.

PREGUNTA
MANUAL DE TRABAJO DEL CURSO ENARM CMN SIGLO XXI
CURSO ENARM CMN SIGLO XXI TEL: 36246001 Pharmed Solutions Institute PGINA 424

Se ingresa ingresa anteriomente referido por fiebre de 40 grados, dolor abdominal, dolor toraxico con dificultad respiratoria, durante su
estancia hospitalaria presenta vmito y diarrea persistentes, TA 100/70, Cul es su conducta a seguir.

RESPUESTA
a.- Unidad mdica hospitalaria con aislamiento estndar, precauciones de gotas y de contacto.
b.- Aislamiento domiciliario estricto, con listado de datos de alarma, con indicaciones para reevaluar al da diguiente si fuera necesario.
c.- Aislamiento domiciliario estricto, con listado de datos de alarma.
d.- Compensacin de enfermedad crnica, manejo de complicaciones y tratamiento antiviral (oseltamivir).

PEGUNTA
Cual es el esquema terapeutico antiviral mas apropiado.

RESPUESTA
a.- 60 mg cada 12 hrs por 5 das.
b.- 45 mg cada 12 hrs por 5 das.
c.- 75 mg cada 12 hrs por 5 das.
d.- 150 mg cada 12 hrs por 5 das

CASO CLINICO
Nio de 10 aos que consult al servicio de urgencias despus de presentar crisis convulsiva tonicoclnica generalizada (TCG) de 3 min
de duracin, despus de 4 das de fiebre, tos, inapetencia general y fatiga; en el hospital curs febril, con debilidad, desorientado; unos
minutos despus present otra crisis TCG de 3 min, se controla las crisis con mejora 12 horas despus presenta las siguientes
manifestaciones Inflitrados pulmonares en ms de dos cuadrantes, ndice de oxigenacin PaO2/FiO2 menor de 250 y distensibilidad
pulmonar disminuida.

PREGUNTA
Cual es la conducta a seguir.

RESPUESTA
a.- 60 mg cada 12 hrs por 5 das.
b.- Ceftriaxona 50-100 mg/kg/da IV c/24 hrs por 7-10 das.
c.- FiO2: el necesario para mantener PaO2 arriba de 60 mm Hg.
d.- Presin menor de 35 cm H2O y meseta menor de 30 cm H2O.

MENINGITIS. CIENCIAS BASICAS: Es un proceso inflamatorio agudo del SNC causado por microorganismos que afectan a las
leptomeninges (aracnoides y piamadre). El sistema nervioso puede infectarse por diferentes microorganismos, incluyendo bacterias,
virus, hongos, protozoos y helmintos. La presentacin clnica de estas infecciones puede ser aguda, subaguda o crnica dependiendo de
la etiologa, la virulencia del microorganismo y la localizacin del proceso infeccioso. SALUD PUBLICA: Un 80% ocurre en la infancia,
especialmente en nios menores de 10 aos. En la ltima dcada, con la introduccin de nuevas vacunas frente a los grmenes
causales ms frecuentes y con el desarrollo de antibiticos ms potentes y con buena penetracin hematoenceflica, ha disminuido la
incidencia y ha mejorado el pronstico de la infeccin, pero las secuelas y la mortalidad no han sufrido grandes cambios. MENINGITIS
VIRAL (95%): Los virus que se detectan con ms frecuencia en meningitis asptica son los enterovirus (EV), seguidos de virus herpes
simple (VHS) y varicela zoster (VZV). Entre los muchos tipos de EV causantes de brotes de meningitis, destacan Echovirus 30, 13, 6, 11 y
9, Coxsackie B5 y Coxsackie A9. Los EV son tambin la causa principal de meningitis y sepsis neonatal. PATOGENIA: El ms general sera
la colonizacin del SNC a travs de diseminacin hematgena del virus durante la primoinfeccin vrica. En el caso de los alfa
herpesvirus un segundo mecanismo, cuyo ejemplo ms tpico sera la encefalitis herptica, consistira en la invasin del SNC tras
reactivacin de la infeccin latente desde los ganglios nerviosos regionales a travs de las fibras nerviosas. DIAGNOSTICO: Las
meningitis de carcter vrico suelen tener un curso ms insidioso, con escasa afectacin del estado general, fiebre de intensidad
generalmente moderada (en ocasiones ausente) y cefalea que puede ser intensa. La puncin lumbar puede generar un alivio transitorio
de los sntomas. En el caso de la meningoencefalitis vrica, se describe una triada clsica: fiebre, cefalea y alteracin conciencia. El LCR
aunque muetsra pleocitosis generalmente no rebasa de 200-500 celulas y de predomino de mononucleares. Los estuidos serolgicos o
de estudio de virus esclarecen el diagnostico. MENINGITIS BACTERIANA (5%): Se caracteriza por la presencia de signos y sntomas
menngeos y enceflicos con grados variables. La morbilidad en Mexico es de 50% de los casos, se presenta en nios de 3 meses a 3
aos de edad y su mortalidad varia de 5-15 %, las secuelas son retraso psicomotor leve hasta retraso mental grave, sordera,
convulsiones e hidrocefalia, varan de acuerdo a edad, germen causal, oportunidad y eficacia del tratamiento. Agentes causales en RN a
3 meses son bateras gramm negativas (E. coli y Klebsiella), estreptococo beta hemoltico del grupo B, Listeria monocytogenes y
meningococos. De 3 meses a 5 aos son: Haemophilus influenzae del tipo B, Streptococo pneumoniae, Neisseria meningitides,
neumococos y meningococos. PATOGENIA: La mayor parte de los casos inician con una bacteriemia a partir de focos infecciosos,
respiratorios, gastroinetstinales y urinarios, tambin ocurre diseminacin directa de un foco paramenngeo por ejemplo: pansinusitis u
otitis recurrente donde los grmenes involucrados son diferentes de acuerdo al proceso infeccioso primario, tambin suele ocurrir por
invasin directa como en el caso de fracturas de crneo, disrrafas de la lnea media, procedimientos neuroquirurgicos, colocacin de
catteres. Unas vez que ocurre la bacteriemia existe un proceso inflamatorio del endotelio de los vasos cerebrales con aumento del
tamao de los poros o puentes intercelulares, lo que permite el paso de molculas de gran tamao (protenas y bacterias) al intersticio
del parnquima cerebral (rotura de la barrera hematoencefalica) y se produce un edema cerebral vasgeno que repercute en la entrega
de oxgeno a las neuronas, as como edema cerebral de tipo citotxico, irritando las meninges. La inflamacin y el dao celular neuronal
causan metabolismo anaerbico con produccin de lactato, consumo de glucosa y elevacin de protenas en el LCR. DIAGNOSTICO: La
MANUAL DE TRABAJO DEL CURSO ENARM CMN SIGLO XXI
CURSO ENARM CMN SIGLO XXI TEL: 36246001 Pharmed Solutions Institute PGINA 425

sintomatologa vara segn la edad del paciente en RN: inespecfica, sospechar en RN con sepsis, fiebre, hipotermia, irritabilidad, rigidez
de nuca, rechazo al alimento, vmito, diarrea y convulsiones (40%). Lactante: sospechar cuando hay infeccin respiratoria o
gastrointestinal de varios das de evolucin con fiebre persistente, vomito, irritabilidad, convulsiones, fontanela anterior tensa y
abombada (es un signos tardio presente en 20%), rechazo a la va oral o bien rigidez de nuca, hiperreflexia, signos de Kerning (dolor de
espalda con la extensin pasiva de la rodilla estando los muslos flexionados) y Brudzinsky (flexin espontnea de los miembros
inferiores al flexionar pasivamente el cuello), de Babinski y cefalea. Los isgnos de focalizacin neurolgica hemiparesia, cuadriparesia y
paralisis facial pueden ocurrir temprana o tardamente en 15% de los casoso por trombosis venosa o arterial, secundaria e inflamacin.
Laboratorio, BH reporta leucocitosis con predominio de neutrfilos y bandas, as como Hb baja. Debe tomarse cultivo de orina,
electrolitos sricos para corregir el balance hdrico. QS, para verificar la cuantificacin de glucosa en sangre, pruebas de coagulacin,
cuantificacin de plaquetas. Puncin lumbar para citologa de LCR con raquimanometro para registrar la presin de apertura del LCR
(normal 70-150mm). Se hacen pruebas de diagnstico rpido como coaglutinacin en ltex que permite un buen porcentaje de
sensibilidad u especificidad existiendo reactivos para detectar H. influenzae tipo B, Streptococo pneumoniae, N. meningitidis, E. coli. Se
debe tomar hemocultivo e iniciar de inmediato el tratamiento antimicrobiano. La TAC est indicada en : 1. Pacientes con
neuroinfeccin en quienes se sospecha alguna complicacin como higroma subdural , absceso etc. 2. Pacientes con persistencia de
alteraciones neurolgicas a pesar de un tratamiento integral adecuado. 3. En quienes presentan datos de focalizacin, hipertensin
intracraneana o infeccin por enterobacterias. TRATAMIENTO: Varia segn resultado de cultivo de LCR y hemocultivo, sin embargo se
sugiere iniciar tratamiento de acuerdo a la edad. RN: ampicilina 150-200mg/kg/da + gentamicina 7.5mg/kg/da amikacina
22mg/kg/da cefotaxima 150-250mg/kg/da. Nios mayores de 3 meses: ceftriaxona 100mg/kg/da, cefotaxima 200mgs/kg/da,
cloranfenicol 75-100mg/kg/da. Preescolares y escolares: penicilina 250-300 000/kg/da + cloranfenicol 75-100mg/kg/da. MENINGITIS
FIMICA: Puede representar alrededor del 1% de todas las formas y es ms
frecuente en poblacin de pases
subdesarrollados, nios y
pacientes infectados por el VIH. La
localizacin menngea puede
producirse por va hematgena durante
la primoinfeccin o la reactivacin, o
bien por ruptura al espacio
subaracnoideo de un foco paramenngeo ya existente. En cualquier caso se produce una inflamacin granulomatosa de las meninges
basales y pequeos focos tuberculosos (focos de Rich) en las meninges, el cerebro o la mdula espinal. . Los sntomas ms frecuentes
son fiebre y cefalea de curso subagudo o crnico, vmitos, decaimiento, rechazo de las tomas, aadindose posteriormente
disminuciones del nivel de conciencia y/o alteraciones de la conducta, otros sntomas y signos de hipertensin intracraneal, afectacin
de los pares craneales (III,VI,VII), hidrocefalia, y diferentes formas de alteraciones neurolgicas focales. El cuadro clnico, por lo general
es de instalacin lenta (2-3semanas) y por lo reguilar debe existir un estudio de Combe positivo. MENINGITIS MICOTICA: Son raras
evolucionan lentamente, se presentan a menudo en pacientes inmunodeprimidos o con desnutricin crnica. El estudio citoqumico de
LCR es indistinguible del de una meningitis fimica. Posponer puncin lumbar: Inestabilidad hemodinmica. Insuficiencia respiratoria.
Ditesis hemorrgica (<100.000 plaquetas). Hipertensin intracraneal confirmada. Infeccin superficial en el lugar de la puncin.

CASO CLINICO
Una adolescente de 14 aos previamente sana que acudi en estado comatoso. Tres das antes haba comenzado con tos, fiebre,
rinorrea y cefalea ocasional, siendo diagnosticada de catarro y tratada con antitrmicos y antihistamnicos. Unas horas antes del
ingreso present varios episodios de prdida de fuerza de los 4 miembros, cefalea intensa y fiebre de 38 C. En el momento previo al
ingreso la madre la haba encontrado apoyada en la pared realizando movimientos incoordinados de las 4 extremidades, sin respuesta
a estmulos verbales, mirada prdida y despus cada al suelo, prdida de conciencia y relajacin de esfnteres. Temperatura, 37 C;
presin arterial, 130/70 mmHg; frecuencia cardaca, 110 lat./min; frecuencia respiratoria, 19 resp./min; saturacin de oxgeno, 100 %;
coloracin normal de piel y mucosas; desconectada del medio; Glasgow de 8; hipotona generalizada; reflejos osteotendinosos
normales; pares craneales normales; pupilas midriticas pero reactivas, y signos menngeos negativos. El resto de la exploracin fsica
fue normal. Lquido cefalorraqudeo: protenas, 53 mg/dl; glucosa, 60 mg/dl; leucocitos, 5/ ml.

PREGUNTA
Cual es el agente etiolgico mas frecuente considerando la edad y el cuadro clnico.

RESPUESTA
a.- Streptoccoco del grupo B, E. coli listeria.
b.- Haemophilus influenzae, Straptococo pneumoniae E. coli.
c.- Streptococcus pneumoniae neisseria meningitidis.
d.- Haemophilus influenzae, neisseria meningitidis streptococcus pneumoniae.

CASO CLINICO
Lactante de 8 meses. En la ltima semana presentaba cuadro febril de hasta 39C en el contexto de una gastroenteritis aguda. Tras
presentar respiracin superficial y bradicardia, precisa intubacin y conexin a ventilacin mecnica. Se practica TAC craneal en el que
destaca hidrocefalia tetraventricular por lo que se decide colocacin de derivacin ventricular externa con salida de lquido turbio con
glucosa ligeramente disminuida (0,35g/l) y protenas elevadas (0,57g/l). La citologa muestra un perfil bacteriano ms por la frmula
que por el nmero total de clulas (240 clulas nucleadas con 80% de segmentados y 20% de linfocitos). Destaca la emisin de
abundante orina transparente en una nia con deshidratacin leve (3%) hasta ese momento y cuadro diarreico de una semana de
evolucin, que se intensifica a la vuelta de quirfano llegando a presentar poliuria de 30cc/kg/h. Destacan hipernatremia de 153mEq/l,
PARAMETRO NORMAL M. BACTERIANA M. VIRICA M. TUBERCULOSA
Presin de apertura 70-150mm Alta N/alta Alta
Aspecto de LCR Turbio o purulento Claro opalescente
Glucosa (mg/dl) 60-80% de la
glucemia
<66% de la sangunea
(hipoglucorraquia)
Normal <40mg/dl
Cels/mm3 <10 (MMN) >1000 (PMN) <300 <1000
Protenas (mg/dl) <45 Aumentadas ++ N o aumentada+ Aumentada +++
(100-500mg/dl)
MANUAL DE TRABAJO DEL CURSO ENARM CMN SIGLO XXI
CURSO ENARM CMN SIGLO XXI TEL: 36246001 Pharmed Solutions Institute PGINA 426

hiponatriuria de 11,9mEq/l, densidad urinaria de 1003, osmolaridad urinaria de 110mosmol/kg, osmolaridad plasmtica de
318mosmol/kg y poliuria con acentuacin de la deshidratacin a pesar de reposicin horaria de la diuresis.

PREGUNTA
Cual es la complicacin mas probable.

RESPUESTA
a.- Falla renal.
b.- Sindrome de secresion inapropiada.
c.- Diabetes inspida central.
d.- Necrosis hipofisiaria.

OTITIS. CIENCIAS BASICAS: La otitis media aguda (OMA), es la infeccin supurada del odo medio, que tienen un inicio sbito y de corta
duracin; asimismo, denota inflamacin de la cubierta mucoperistica del odo medio. La membrana timpnica inflamada se presenta
opacificada, protruyente o con ambas caractersticas. Segn su tiempo de evolucin la enfermedad puede subdividirse en a) aguda,
cuando el proceso dura no ms de 3 semanas; b) subaguda, cuando la infeccin perdura de 3 semanas a 3 meses, y c) crnica, cuando la
enfermedad se prolonga por ms de 3 meses. SALUD PUBLICA: La OMA es un problema mundial que afecta a uno de cada 4 nios
menores de 10 aos y es la infeccin bacteriana ms frecuente en nios menores de 5 aos. De 25-40% de las visitas peditricas en
menores de 5 aos son por OMA. Hay estudios que indican que 80% de los preescolares menores de 3aos, han tenido cuando menos
un episodio de OMA. Hay mayor incidencia durante otoo e invierno., los nios que son llevados a guarderas parecen ser
especialmente susceptibles a padecer OMA, y los expuestos a humo de tabaco. CLASIFICACION: 1. Miringitis: cuando se trata de una
inflamacin de la capa externa de la membrana timpnica que puede ocurrir sola o asociada a una inflamacin del conducto auditivo
externo. 2. Otitis media aguda supurada: infeccin aguda del odo con exudado y de corta duracin. 3. Otitis media secretoria:
presencia de lquido en odo medio detrs de una membrana timpnica integra sin signos agudos o sntomas. 4. Otitis media crnica
supurada: presencia de descarga crnica del odo medio a travs de una perforacin de la membrana, a veces hay perforacin sin
descarga. 5. Otitis media recurrente: presencia de 3 episodios de otitis media aguda en 6 meses, o 4 en un ao, o 2 cuadros diferentes
con un mes entre ellos. PATOGENIA: Entre las funciones del odo se incluyen la ventilacin y el equilibrio de las presiones atmosfricas,
as como la proteccin ante el sonido y el drenaje de secreciones a la nasofaringe. Cuando estos mecanismos se alteran, contribuyen a
la proliferacin bacteriana, lo que desencadena la infeccin. La obstruccin tubaria tambin inhibe el drenaje del lquido del odo medio
a la nasofaringe, lo que contribuye al desarrollo de la infeccin. La falla de estos mecanismos puede deberse a inflamacin, infecciones
frecuentes, problemas alrgicos o neoplsicos. Aunado a ello, la trompa de Eustaquio de los nios es ms corta, ms horizontal y tienen
menor soporte cartilaginoso. En 2/3 partes de los pacientes con OMA pueden aislarse patgenos bacterianos del lquido del odo. En
30-50% de los casos se encuentra Streptococo pneumoniae. El Haemophilus influenzae causa entre 20-27% de los casos de OMA.
Moraxella se aisla Moraxella catarhallis se isla en 7-23% de los casos. Los nios hospitalizados desarrollan grmenes del medio
ambiente hospitalario. Las infecciones virales representan el 41% de los casos de OMA. Los virus syncitial respiratorio (74%),
parainfluenza (52%) e influenzae (42%). Algunas patologas como anormalidades craneofaciales y el reflujo gastroesofgico se
relacionan con la presencia de otitis media. El uso de chupn es un factor de riesgo para otitis media recurrente. DIAGNOSTICO: clnica;
se presentan como un cuadro de coriza, rinitis, fiebre, tos, irritabilidad y anorexia. El sntoma ms fidedigno es la otalgia. Los nios muy
pequeos no verbalizan el dolor pero se jalan las orejas o rehsan a deglutir y lloran constantemente. Debe sospecharse cuando el
dolor despierta a los nios por la noche. Exploracin del odo medio, mediante el uso de un otoneumatoscopio (sensibilidad de 95% y
especificidad de 80%), debe realizarse de manera que el conducto auditivo externo se encuentre libre para visualizar correctamente la
membrana timpnica y valora su movilidad, los cambios de coloracin y vascularizacin, as como la presencia de niveles hidroaereos
por detrs de la misma, debe realizarse son suavidad para no lastimar. La OMA se manifiesta clnicamente con retraccin, eritema,
disminucin del reflejo luminoso e hipomotilidad de la membrana timpnica; asimismo, se presenta una disminucin de la audicin en
rangos de baja frecuencia. La membrana timpnica, particularmente la pars flcida, se encuentra hiperemica, con la vascularidad
visible, y las pars tensa, abombada y turbia. En un estado avanzado de la enfermedad puede haber ruptura de la membrana timpnica,
habitualmente en la pars tensa, con salida de material francamente purulento, sanguinolento o seroso; la perforacin habitualmente es
pequea, lo que la distingue de etiologas ms raras (necrosante, TB, recurrencia de otitis), y una vez que la pus drena, los sntomas
disminuyen de manera importante. De manera ideal se debe realizar timpanometria para identificar diferencias en la presin dentro y
fuera del odo y confirmar la presencia de lquido; asimismo la reflectometria acstica es un mtodo no invasivo para identificar
derrame de odo medio mediante el reflejo del sonido. TRATAMIENTO: Es importante hacer notar que existen pocas herramientas que
ayuden a la pediatra para formular una estrategia adecuada de tratamiento; sin embargo, en recientes revisiones se ha observado que
el uso temprano de la terapia antibitica mejora el curso clnico de la OMA, disminuyendo la sintomatologa y la incidencia de
complicaciones. La mayora de los expertos continua recomendando amoxicilina (primera eleccin) para iniciar tratamiento emprico de
todos los episodios de otitis media a dosis de 80-90mg/kg de peso da durante 5-10 das, la accin de este puede llegar a ser limitada
(20-30% son resistentes). La seleccin debe basarse en la concentracin de los mismos en el sitio de infeccin, en caso de resistencia,
enfermedad grave y H. influenzae y M catarrhalis, se recomienda amoxicilina/clavulanato( 90mg/kg/dia con 6.4 mg/kg/da dividido en 2
dosis). El manejo conservador sugiere 10 das de antibioticoterapia, aunque hay resultados aceptables con 5-7 das de tratamiento. Si
un paciente no mejora con amoxicilina despus de 72hrs de manejo se debe cambiar de antibitico. El TMP/SFX tienen una elevada
resistencia para Streoptococus pneumoniae 30-44.6%, no se recomienda su uso. En pacientes alrgicos a betalactamicos, las
alternativas por su adecuada concentracin en ido medio son macrolidos y clindamicina. Todos los nios menores de 6 meses con
diagnstico de OMA deben recibir tratamiento para evitar complicaciones. No se recomienda el uso de gotas oticas, con o sin
antibitico, gotas nasales, mucolticos, antihistamnicos, pues no aportan beneficios al tratamiento de la OMA. La administracin de
paracetamol o ibuprofeno es necesaria, para el manejo de la fiebre y el dolor. Tratamiento quirrgico: Miringotomia; cuando hay
otalgia severa o complicaciones intratemporales o intracraneales, optimiza la regulacin de las presiones en el odo medio.
COMPLICACIONES: Prdida auditiva (la mas comn), perforacin de la membrana timpnica, mastoiditis aguda, laberintitis, parlisis
MANUAL DE TRABAJO DEL CURSO ENARM CMN SIGLO XXI
CURSO ENARM CMN SIGLO XXI TEL: 36246001 Pharmed Solutions Institute PGINA 427

facial, colesteatoma (acumulacin de tejido epitelial de crecimiento rpido que va destruyendo el hueso temporal y su contenido).
PREVENCION: La aplicacin de vacuna conjugada antineumococica, se asocia a una reduccin de la colonizacin nasofarngea por
Streptococo pneumoniae. La eficacia estimada de la vacuna heptavalente conjugada para evitar otitis media aguda es de 6%, para otitis
media recurrente 9%, hasta un 57% de las OMA causadas por neumococo y 90% de las otitis causadas por los serotipos de neumococo
incluidos en la vacuna hepatavalente. Se recomienda a los 2, 4 y 6 meses de edad y refuerzo entre los 13 y 15 meses.

CASO CLINICO
Paciente varn de 6 aos de edad, sin antecedentes mdicos de inters, con vacunacin correcta, incluida vacuna antineumoccica.
Cuadro catarral de 5 das de evolucin, con otalgia izquierda de 6 horas de evolucin, fiebre de hasta 39,9C axilar y parestesias en la
hemicara derecha, que posteriormente se extienden en menor intensidad a la mano y la pierna derechas. En la exploracin clnica
mostraba rigidez de nuca, con signo de Brudzinsky positivo. El resto de la exploracin neurolgica era normal. En la otoscopia se vean
ambos tmpanos hipermicos, no abombados, y no mostraba despegamiento del pabelln auricular.

PREGUNTA
Cual es la conducta diagnostica mas adecuada.
a.- Biometria hemtica.
b.- Factores inflamatorios.
c.- TAC craneal.
d.- Rx de crneo.

CASO CLINICO
Mujer de 2 aos de edad, sin antecedentes patolgicos, con vacunacin correcta, incluida vacuna antineumoccica, que consulta por
cuadro de 4 das de evolucin de fiebre (38,8C axilar mxima), odinofagia y tortcolis, con desviacin ceflica hacia abajo y la izquierda.
Tratada desde haca 24h con ibuprofeno va oral VO y calor local, sin presentar mejora. En la exploracin presentaba desviacin ceflica
hacia abajo y hacia la izquierda, con intensa contractura del esternocleidomastoideo izquierdo, sin otros hallazgos patolgicos. La
exploracin neurolgica era normal. En la otoscopia se vean ambos tmpanos hipermicos, no abombados, y no haba despegamiento
del pabelln auricular. Se realiz un hemograma, con leve leucocitosis (11.800 cel/L) sin desviacin izquierda, y PCR de 67mg/l; el
resto de la analtica era normal. Se realiz un hemocultivo, negativo. Dada la persistencia del tortcolis, se realiz una TC craneal, en la
que se visualiz ocupacin de caja timpnica, aditus, antro mastoideo y celdas mastoideas, por lo que se catalog de otomastoiditis
izquierda.

PREGUNTA
Cual es la conducta a seguir.

RESPUESTA
a.- Clindamicina y vancomicina
b.- Ceftriaxona mas gatifloxacina.
c.- Dicloxacilina y amikacina.
d.- Ampicilina y gentamicina.

SINUSITIS. CIENCIAS BASICAS: Es la infeccin de uno o varios senos paranasales (etmoidal, maxilar, frontales, esfenoidal) causada por
la obstruccin del ostium de drenaje. Secundario, entre otros factores, a la desviacin septal, adenoiditis o plipos, con produccin de
secrecin mucopurulenta e inflamacin de la mucosa (habitualmente implica siempre un grado de afectacin de la mucosa
nasal=rinosinusitis) y sintomatologa de dolor dependiendo del seno afectado. SALUD PUBLICA: La sinusitis es infrecuente en los
menores de un ao, quiz por el menor desarrollo anatmico de los senos paranasales en este grupo etreo. No se ha guardado
relacin con predominio por raza o sexo. La remisin espontnea puede ser hasta de 70%. CLASIFICACIN: 1. Sinusitis aguda; dura
menos de cuatro semanas, los episodios se resuelven con tratamiento mdico, sin dao residual en la mucosa. 2. Sinusitis sub-aguda;
duran de cuatro semanas a tres meses. 3. Sinusitis crnica; tiene una duracin de ms de tres meses, con alteraciones estructurales en
la mucosa sinusal 6 episodios de sinusitis aguda al ao, con alteraciones en la TAC de senos paranasales. 4. Sinusitis aguda recurrente;
cuando el paciente presenta ms de 4 episodios de sinusitis aguda en un perodo de seis meses y existe adecuada respuesta al
tratamiento mdico, sin dao residual de la mucosa. PATOGENIA: Los siguientes son los principales factores de riesgos que conllevan a
que se presente sinusitis en la infancia: Las infecciones respiratorias de las vas areas superiores, inmadurez inmunolgica e inmuno-
deficiencias, hipertrofia adenoidea y la adenoiditis, asistencia a la sala cuna o jardn infantil, la exposicin al humo de los cigarrillos,
contaminacin ambiental y la alergia, cuerpos extraos, plipos y tumores nasales. El papel de la infeccin bacteriana como causa
primaria de sinusitis crnica es controvertido, siendo ms consecuencia de factores no infecciosos. Las infecciones vricas predisponen
al desarrollo de la sinusitis bacteriana (80% de los casos), mientras que el componente alrgico es significativo en el 20% restante. La
sinusitis bacteriana casi siempre surge como complicacin de una de las anteriores. La microbiologa de la sinusitis bacteriana aguda
(SBA) es semejante a la de la otitis media aguda, los grmenes que con ms frecuencia causan tanto la sinusitis aguda como crnica,
independiente de la edad, en orden de importancia son: Virus: Adenovirus, influenza, parain-fluenza, Estreptococo pneumoniae (30%),
Haemophilus influenzae (20%), Moraxella catarrhalis (20%), Branhamella catarralis, Streptococo beta hemoltico, Stafilococo aureus,
Grmenes anaerobios. La sinusitis aparece posterior a la rinofaringitis aguda o crnica, presentando antecedentes de inflamacin viral
en vas areas superiores o inferiores. Si persiste por ms de diez das predispone a la inflamacin de la mucosa de los senos
paranasales. DIAGNOSTICO: Clnica; Existen dos situaciones clnicas que nos hacen sospechar una SBA: 1. Sntomas catarrales leves
(rinorrea, tos diurna) que no ha empezado a mejorar tras 10 das (criterio de Persistencia): es la presentacin ms habitual y debe
diferenciarse de los catarros encadenados. 2. Concurrencia de fiebre elevada ( 39 C) y rinorrea mantenidas 3 das (criterio de
Gravedad) con afectacin del estado general. Sntomas especficos: Se ve ms en nios mayores, dolor facial, cefalea, dolor
MANUAL DE TRABAJO DEL CURSO ENARM CMN SIGLO XXI
CURSO ENARM CMN SIGLO XXI TEL: 36246001 Pharmed Solutions Institute PGINA 428

periorbitario y anosmia entre otros (presentes en 1/3 de los casos). Sntomas inespecficos: Secrecin nasal y obstruccin nasal uni o
bilateral. Fiebre. Halitosis. Tos. Identificacin en el nio menor: Los sntomas son inespecficos y se suelen manifestar como irritabilidad.
Se identifica con fiebre alta, aspecto txico. Puede presentar celulitis periorbitaria o aumento de volumen ocular. La rinorrea puede ser:
Hialina. Mucosa. Purulenta. Sanguinolenta. Es de anotar que las caractersticas de las secreciones no se relacionan con el agente
etiolgico. En estos casos de diferenciacin es mas importante el tiempo de evolucin y el estado general del paciente. RINORREA
HIALINA: Se ve en los cuadros agudos, por edema y secundario a enfermedades sistmicas, principalmente rinofaringitis aguda, rinitis
alrgica. Hay inflamacin de la mucosa con aumento de la permeabilidad de los capilares, que produce un exudado seroso en la cavidad
sinusal. Se presenta edema del estroma con infiltracin celular, linfocitico y clulas plasmticas. RINORREA MUCOSA: Por aumento de
las glndulas productoras de moco, la secrecin se estanca creando un medio de cultivo excelente para el desarrollo de grmenes, en el
estroma aparecen histiocitos y clulas productoras de moco. La cavidad nasosinusal con la secrecin, va absorbiendo el aire del seno,
creando presin negativa que conlleva a mayores alteraciones histolgicas. RINORREA PURULENTA: La coloracin podr variar de
amarillo, verdoso, marrn, mal oliente y de gran viscosidad, adherente a la mucosa. Hay infiltrado de polimorfonucleares y de otras
clulas plasmticas, histiocitos y fibroblastos. Hay obliteracin glandular, vascular y neural. RINORREA SANGUINOLENTA: Debido a la
ruptura vascular se presenta salida de sangre; aparecen granulomas de colesterol, que acta como factor irritativo tisular. La patologa
se encuentra en un nivel ms profundo y hay compromiso de los vasos sanguneos. La presencia de rinorrea prolongada de ms de 10
das permite predecir alteraciones radiolgicas asociadas a una infeccin bacteriana (90% en edades entre 2 y 6 aos, y 70% en
mayores de 6 aos). La presencia de un catarro comn sin sinusitis puede provocar alteraciones radiolgicas similares, que permanecen
hasta dos semanas despus de la mejora clnica. Adems no distingue entre una sinusitis bacteriana, viral, o de otras causas. Las
pruebas de imagen deben reservarse a casos de fracaso teraputico o empeoramiento de sntomas, y no estn recomendadas en
menores de 6 aos. El tratamiento emprico estara pues justificado sin realizar ninguna prueba previa. Los cultivos de secrecin nasal o
exudado farngeo no tienen correlacin con los sinusales. La puncin y aspiracin directa del seno es el patrn oro diagnstico, pero
slo est indicada en contados casos hospitalarios (enfermedad grave con aspecto txico, inmunodeficiencias y complicaciones
supurativas intracraneales). Criterios diagnsticos, los cuales a su vez pueden ser mayores (Dolor, presin facial. Congestin nasal.
Obstruccin nasal. Rinorrea anterior o posterior. Hiposmia o anosmia. Cefalea) o menores (Halitosis. Fiebre. Dolor dental. Fatiga. Tos.
Otalgia). TAC es una importante ayuda imagenolgica para documentar la existencia del compromiso sinusoidal, y tiene unas
indicaciones muy precisas: Persistencia de la sintomatologa luego de manejo apropiado por 10-15 dias. Cronicidad de la sinusitis.
Complicaciones con celulitis facial o periorbitaria. Coexistencia con asma, datos de hipertrofia severa adenoidea rinitis o rinorreas
persistentes. Sospecha de sinusitis mictica. COMPLICACIONES: Celulitis periorbitaria y orbitaria, la trombosis del seno cavernoso, el
absceso subperistico, el absceso cerebral, y el empiema epidural y subdural. TRATAMIENTO: El uso de antibiticos inicialmente no
est indicado en la mayora de los casos si no presenta criterios de gravedad o persistencia. Como tratamiento sintomtico analgsico y
antipirtico son efectivos paracetamol (10-15 mg/kg/dia) e ibuprofeno (4-6mg/kg/ dosis cada 6-8hrs). Suelen ser necesarios pocos das.
El uso de descon- gestionantes nasales sigue siendo motivo de controversia, pero se concepta que inicialmente mejora el drenaje de
secreciones provocando alivio sintomtico, siendo su uso no mayor de cinco das, los de aplicacin tpica se encuentra Oximetazolina
al 0.025- 0.05% en dosis de 2-3 gotas o aspersiones en fosas nasales cada 12 horas. Corticoides intranasales: Se aconsejan cuando
existen estados de recurrencia o cronicidad y coexiste rinitis alrgica sobre todo si son persistentes. En algunos estudios se destaca el
efecto antiinflamatorio local. Se encuentran entre otros: Furoato de Mometasona, Fluticasona, Budesonida durante tres meses. Los
corticoides orales no son usados excepto en casos de enfermedades asociadas como poliposis o crisis asmtica. Antihistamnicos
Podran dificultar el drenaje de secreciones. Si existe una base alrgica se recomienda su uso al final del manejo antibitico, en casos de
estados bacterianos. Se encuentran: Loratadina: Presentacin: 5mg/5ml a dosis de 0.2 mg/kg/da cada 12 horas Desloratadina:
Presentacin: 2.5 mg/ 5ml: 1-5 aos de edad: 1,25mg una dosis dia. 6 A 12 Aos: 2,5 mg/da: 5 ml/d Cetirizina: 0.5 mg/kg/da cada 12
horas Levocetirizina: Presentacin: sol. Oral. 0.5 mg/ ml. Gotas: 5mg/ml. Su dosis es de 0.125 mg/kg. Cuando existen procesos
infecciosos bacterianos los antibiticos son teraputica central dentro del tratamiento a realizar. El antibitico de eleccin es la
Amoxicilina. Se administra a una dosis de 45 mg/ kg/da, puede aumentarse a 80-90mg/kg/dia. El fracaso teraputico por resistencia
antibitica se debe en el caso del neumococo (40%) por cepas con susceptibilidad disminuida, mientras que para Haemophilus
influenzae (20%) y Moraxella catharralis (80%) son por produccin de betalactamasas. Por todo lo cual si no existe mejora con el
tratamiento inicial se utilizar de segunda lnea es la Amoxicilina/clavulanato. La dosis es 40-90 mg/kg/da. No obstante, puede ser
usado como primera eleccin en caso de riesgo alto de resistencia (edad menor de 2 aos, toma reciente de antibitico o zonas de alta
prevalencia de neumococo resistente). Otras alternativas de antibiticoterapia son: Cefuroxime a una dosis de 30mg/kg/da y el
Cefprozil a 30 mg/ kg/da. Los das de duracin del manejo antibitico varan de acuerdo a los autores. Se suele recomendar que sea
entre 5-14 das. Son indicaciones de derivacin hospitalaria urgente la afectacin severa del estado general (aspecto sptico, cefalea
facial intensa), la sospecha de complicaciones (craneales, endocraneales y oculoorbitarias, salvo la celulitis preseptal que puede
tratarse inicialmente en Primaria) o tumor (sntomas unilaterales persistentes como epistaxis, obstruccin y rinorrea, siendo la
deformidad facial un sntoma tardo) y la existencia de un entorno familiar de riesgo que no garantice cuidados generales,
cumplimiento teraputico y vigilancia eficaz. En rinorrea persistente unilateral siempre se descartar la presencia de un cuerpo extrao
nasal.

CASO CLINICO
Varn de 9 aos y 8 meses, sin antecedentes de inters, remitido al servicio de Urgencias de Pediatra, desde su centro de salud por
cefalea y proptosis. Inicio del cuadro 4 das antes, como fiebre, vmitos, cefalea y congestin nasal; diagnosticado en su centro de salud
de sinusitis y en tratamiento antibitico con amoxicilina/clavulnico desde hace 48 horas, sin mejora de los sntomas. Refiere cefalea
intensa fronto-temporal derecha y edema en prpado derecho que ha evolucionado hacia proptosis en las ltimas horas. A la
exploracin fsica, se evidencia ligera proptosis del ojo derecho, con leve tumefaccin palpebral, as como una obstruccin de la fosa
nasal del mismo lado. En la exploracin neurolgica solamente destaca una leve dificultad para la mirada conjugada

PREGUNTA
Cual es la conducta a seguir.
MANUAL DE TRABAJO DEL CURSO ENARM CMN SIGLO XXI
CURSO ENARM CMN SIGLO XXI TEL: 36246001 Pharmed Solutions Institute PGINA 429


RESPUESTA
a.- Envio a pediatra.
b.- Realizar radiografia.
c.- Cambio de antibiticos.
d.- Agregar anti-inflamatorio esteroideo.

PREGUNTA
Cual de los siguientes criterios no america envio urgente a segundo nivel.

RESPUESTA
a.- Edema, eritema o dolor facial.
b.- Alteraciones visuales.
c.- Datos menngeos.
d.- Falta de mejora y fiebre elevada.

PREGUNTA
Cul de las siguientes condiciones requiere un envio ordinario a segundo nivel.

RESPUESTA
a.- Factores predisponentes.
b.- Infeccin oportunistas.
c.- Compromiso inmunolgico.
d.- Refractario al tratamiento.

PREGUNTA
Cul no forma parte del protocolo de envio a segundo nivel.

RESPUESTA
a.- Biometria hemtica.
b.- Eosinfilo en moco nasal.
c.- Radiografia de senos paranasales.
d.- Cultivo de moco nasal.

CASO CLINICO
Nia de 10 aos que presenta alteraciones del habla y de la marcha, adormecimiento, un vmito y fiebre. En tratamiento con
amoxicilina-cido clavulnico por amigdalitis diagnosticada hace 4 das. En la exploracin se observan signos menngeos, y en las
exploraciones complementarias un lquido cefalorraqudeo (LCR) con 2.840 leucocitos/ l (60 % polimorfonucleares); hemograma con
21.300 leucocitos/ l (77 % granulocitos) y velocidad de sedimentacin globular (VSG), 35 mm/h. Se diagnostica de meningitis
bacteriana y se inicia tratamiento con penicilina intravenosa.

PREGUNTA
Cual es la conducta farmacolgica a seguir.

RESPUESTA
a.- Cefotaxima, vancomicina y metronidazol.
b.- Vancomicina, metronidazol y ampicilina.
c.- Ampicilina, metronidazol y amikacina.
d.- Gentamicina, dicloxacilina y metronidazol.

FARINGITIS, FARINGOAMIGDALITIS (FA). CIENCIAS BASICAS: La faringitis es un proceso inflamatorio causado por diferentes
microorganismos de observacin frecuente en la edad peditrica, que involucra las membranas adyacentes de la faringe en forma
aguda o crnica; en el 50 a 60 % de los casos puede identificarse el agente causal. En la mayora, la etiologa es viral. Debera reservarse
el trmino faringitis en sentido estricto a la infeccin primaria viral o
bacteriana, circunscrita a la faringe, y no a la hiperemia farngea que se
produce en el transcurso de un cuadro catarral vrico de vas altas
(rinofaringitis aguda). Es recomendable unificar y describir simplemente
como faringitis aguda una patologa que recibe distintos nombres
(amigdalitis, tonsilitis, FA, etc) y que no es ms que la misma enfermedad.
SALUD PUBLICA: Debemos tener en cuenta el alto costo en salud que esta
patologa conlleva y el alta tasa en el uso de antibiticos, muchas veces
innecesarios. Afecta fundamentalmente a nios en edad escolar, 5-10 aos.
Es ms prevalente en climas fros o templados y en los periodos de invierno y primavera. La transmisin es por contacto estrecho
persona-persona a travs de las secreciones. El 65-80% de las FA agudas tienen una etiologa viral, y ocurre como parte de un catarro
de la va area superior que afecta, por lo general, a nios menores de 3 aos. PATOGENIA: En la mayora de los casos, el contagio se
produce por contacto directo a travs de la inoculacin de gotas o transmisin fsica de secreciones respiratorias infectadas. Los
MANUAL DE TRABAJO DEL CURSO ENARM CMN SIGLO XXI
CURSO ENARM CMN SIGLO XXI TEL: 36246001 Pharmed Solutions Institute PGINA 430

posibles agentes causales de faringitis son mltiples y los podemos diferenciar en: A) VIRALES: Inespecfica suele tener un inicio
gradual, con fiebre moderada, faringodinia, tos irritativa de intensidad variable, poca afeccin del estado general, hiperemia variable, el
cuadro suele resolverse en 3-6 das. A veces puede ser sugestivo de infeccin especifica: los rino, corona, adeno y virus sincicial
respiratorio (VSR) son los ms frecuentemente asociados a cuadros de resfro comn, con malestar general, cefaleas, mialgias y
conjuntivitis; los virus coxsackie y echovirus son los causales de la herpangina, caracterizada por la presencia de pequeas vesculas (1-2
mm) sobre el paladar blando, la vula y los pilares amigdalinos anteriores, y acompaada de un sndrome febril con odinofagia intensa,
por lo general, en nios de corta edad. Puede asociarse a lesiones vesiculares con localizacin en manos y pies del tipo del denominado
sndrome mano-pie-boca. La infeccin por Herpes simple tipos 1-2 presenta lesiones vesiculosos o ulcerosas en paladar, configurando
una gingivoestomatitis, adenopata regional dolorosa y fiebre. En los pacientes con trastornos inmunolgicos se puede observar como
enfermedad mucocutnea crnica. El VEB, causante de la mononucleosis infecciosa, cursa en la mitad de los casos con una faringitis
exudativa acompaada de fiebre, decaimiento, cefalea, poliadenopatas no dolorosas y esplenomegalia que nos hace presumir el
diagnstico. La faringitis puede ser un elemento caracterstico de una primoinfeccin por el VIH; suele estar acompaada de
linfadenopatas, a veces de ulceraciones en las mucosas y sntomas generales como fiebre mialgias, artralgias e hipersomnia. B)
BACTERIANAS: La presencia de rinorrea, tos, ronquera, diarrea, conjuntivitis o la edad inferior a 3 aos hacen poco probable la etiologa
bacteriana. La difteria es una enfermedad infrecuente en esta poca, dada la efectividad de la vacuna existente y la elevada cobertura
lograda en la poblacin. De todas maneras, hay que pensar en esta etiologa ante la aparicin de una faringitis membranosa de color
blanquecino claro a oscuro firmemente adherida a la mucosa amigdalina y farngea, acompaada de un cuadro txico-infeccioso en un
paciente con un esquema incompleto de vacunacin. La faringitis aguda por Neisseria gonorrhoae ha aumentado en aos recientes; en
la mayora de los casos, la infeccin es asintomtica y afecta a la poblacin sexualmente activa. Cuando se aisla este germen en
menores de edad, hay que pensar en la posibilidad de abuso sexual. En adolescentes, debemos investigar otras enfermedades de
transmisin sexual. Las cepas de estreptococos de los grupos C y G tienen un comportamiento similar a los del grupo A y se asocian
fundamentalmente a brotes epidmicos de origen alimentario comn (los huevos hervidos fros estn reconocidos como un vehculo
importante). En la faringitis por anaerobios, el compromiso por flora mixta (anaerobios y espiroquetas) es poco frecuente y suele
producir un exudado purulento con aliento ftido. En la forma invasiva periamigdalina puede haber compromiso vascular
(tromboflebitis sptica de la vena yugular); afecta con mayor frecuencia a los adolescentes o adultos jvenes. El Arcanobacterium
haemolyticum produce una faringitis exudativa con erupcin escarlatiniforme pruriginosa en extremidades, cuello y espalda; no
compromete palmas, plantas, cara, abdomen o nalgas y afecta con mayor frecuencia a los adolescentes, es sensible a eritromicina.
Dentro de las infecciones bacterianas, la ms frecuente (15-25%) corresponde al ESTREPTOCOCO B HEMOLTICO DEL GRUPO A
(EBHGA) o Streptococcus pyogenes: Se lo puede aislar durante todo el ao aunque presenta 2 brotes anuales en otoo y primavera. La
faringitis es de aparicin aguda; afecta mayormente a los nios entre los 5 a 15 aos, cursa con fiebre elevada (>39C), odinofagia
(intensa), dolor abdominal (por adenitis mesentrica) con nuseas y vmitos, adenopatas cervicales y un exudado que cubre la faringe
posterior y el rea amigdalina con edema de vula, a veces pronunciado, exantema escarlatiniforme o ambiente epidemiolgico
positivo. La infeccin con cepas capaces de producir toxina eritrognica provoca erupcin eritematosa caracterstica de la piel
denominada escarlatina. Esta misma toxina en huspedes comprometidos por otras infecciones, como varicela, puede producir el grave
cuadro del shock txico estreptoccico, con un riesgo de mortalidad del 50 %. Se puede presentar frecuentemente imptigo, celulitis.
Menos frecuentes sonneumonia, osteomielitis, meningitis, fascitis necrotizante, las complicaciones supurativas: adenoflemn, adenitis
supurada, absceso retrofaringeo, mastoiditis, mediastinitis. Las complicaciones no supurativas: fiebre reumtica (FR), el riesgo de
desarrollar durante una epidemia de EBHA en pacientes no tratados es aproximadamente del 3%, la artritis posestreptocccica
(pacientes con artritis y evidencia de infeccin estreptocccica que no cumplen con los criterios para FR) y la glomerulonefritis aguda
posestreptocccica (GNA) est ms relacionada a cepas nefritgenas, asociadas con mayor frecuencia a infecciones de piel. La
transmisin de la enfermedad: la diseminacin del germen a los convivientes con el caso ndice no tratado oscila entre el 25 al 35%.
DIAGNSTICO: En la mayora de los casos no es posible realizar un diagnstico etiolgico solamente sobre la base de datos clnicos, el
examen fsico de las estructuras farngeas debe ser minucioso, buscando elementos clinicos que puedan ayudar al diagnstico. El
cultivo de fauces en placas de agar sangre es el paso diagnstico definitivo (gold estndar), debe tomarse la muestra de faringe y
amgdalas y, si no puede sembrarse inmediatamente, puede quedar el hisopo seco en un tubo estril a temperatura ambiente por 24
horas hasta su siembra en los medios correspondientes. La sensibilidad de los cultivos oscila entre el 73 al 100%. Es conveniente
examinar los cultivos hasta las 48 horas posteriores a su siembra para poder determinar su verdadera negatividad. Actualmente, los
mtodos rpidos de deteccin de antgeno estreptoccico son un adelanto en el diagnstico de faringitis. Las pruebas que emplean el
sistema de aglutinacin de ltex son muy especficas (95-97%) y moderadamente sensibles (75 al 90%) en comparacin con el cultivo de
fauces, y se requiere de una tcnica relativamente sencilla, de bajo costo y de informacin rpida. Las pruebas de inmunoensayo
enzimtico de fase slida, inmunoensayo ptico y sondas de DNA dan resultados similares aunque requieren de un laboratorio
especializado y son ms costosas. Dado su buen rendimiento, las pruebas de deteccin de antgenos nos permiten, ante un resultado
positivo, establecer el diagnstico de faringitis estreptoccica e iniciar el tratamiento. La faringitis estreptoccica es rara antes de los 3
aos y excepcional antes del ao. Sin embargo hay que recordar que nios que asisten a guardera pueden presentar a partir de los 18
meses, especialmente en brotes epidmicos. El empleo de los mtodos rpidos es tambin de utilidad para evitar la presin ejercida
con frecuencia en la indicacin de antibiticos para facilitar el regreso de los nios a sus actividades habituales. Se ha demostrado que
el uso de los mtodos rpidos en reas de emergencia ha logrado una reduccin significativa en la prescripcin de antibiticos en
faringitis del 70% y en un 93% en aqullos con cultivos negativos. A veces es necesaria la realizacin de otros anlisis para confirmar el
diagnstico etiolgico: cultivos especiales cuando se piensa en gonococo (medios de Thayer-Martin), difteria (medio de Loeffler),
tinciones especiales cuando se piensa en espiroquetas o fuso-bacterias, realizacin de hemocultivos ante cuadros severos de sepsis,
pruebas serolgicas especficas ante la sospecha de EBV, CMV, VIH, bsqueda de antgenos, PCR, cuerpos de inclusin o en cultivo de
clulas. TRATAMIENTO: El tratamiento debe estar relacionado con el agente etiolgico aislado. FA viral: Sintomtico con paracetamol o
naproxen de 3 a 5dias. Ante la enfermedad herptica en el paciente inmunosuprimido, se puede realizar un tratamiento por va oral
con aciclovir a200 mg, en 5 tomas por da, por 7 das. No es recomendable el tratamiento de un primer episodio agudo con aciclovir en
un husped normal. El mismo planteo se puede aplicar para la Influenza A con el uso de Amantadina 6 mg/kg /da por va oral, o
inhibidores de la neuraminidasa. Faringitis por EBHA: Antibitico de eleccin sigue siendo la penicilina V (fenoximetilpenicilina potsica)
MANUAL DE TRABAJO DEL CURSO ENARM CMN SIGLO XXI
CURSO ENARM CMN SIGLO XXI TEL: 36246001 Pharmed Solutions Institute PGINA 431

VO por 10 das <12 aos: 250mg c/12 hrs, >12 aos: 500mg c/12 hrs. Peniclina G benzatina IM profunda <12 aos 600000U, >12 aos
1200000 U (hasta 27 kg de peso se indican 600.000 U; ms de 27 kg se indican 1.200.000 u. Es muy importante que su aplicacin sea
realizada por personal idneo y en nios mayores de 2 aos). Amoxicilina VO por 10 das 40-50mg/kg/da c/12 o 24 hrs (mximo
500mgs c/12hrs). Ninguno erradica el 100% de los casos de faringitis por EBHA. No deberemos indicar antibiticos sin confirmar el
diagnstico. Si por alguna razn se inicia el tratamiento hasta tener confirmacin de los cultivos, debemos suspenderlo si fueran
negativos. No hay ninguna evidencia de que una terapia precoz disminuya la recurrencia. Es importante tambin lograr una adherencia
a cumplir el tratamiento instituido. En pacientes alrgicos a penicilina, el estolato de eritromicina a una dosis de 40 mg/kg/da, en 3 4
dosis diarias, cefadroxilo 30mg/kg/da c/12 hrs por 10 das. Azitromicina 10mg/kg/dia por 5 dias p 20 mg/kg/da por 3 das (mximo
500mgs dosis). Resistencia a macrolidos clindamicina 20mg/kg/da c/12 hrs por 10 das. En la actualidad, slo azitromicina, cefadroxilo,
cexima y cefdinir est aprobada por la FDA como tratamiento una vez al da para faringitis por Streptococus en nios. Los episodios
causados por Streptococcus beta hemoltico de los grupos C y G requieren del mismo tratamiento antibitico. Los relacionados con el
Arcanobacterium haemolyticum se pueden tratar con penicilina o macrlidos. Con respecto a este grupo de pacientes portadores de
EBHA en contacto ntimo con convivientes con FR o aqullos que padecen esta enfermedad y son difciles de erradicar, se pueden
plantear esquemas alternativos de antibiticos, aunque se sabe que este grupo, por caractersticas propias, tiene menor riesgo de
desencadenar la enfermedad. En estado portador asintomtico no existe riesgo de complicaciones (FR o GNA) y no se requiere
tratamiento. No hay urgencia en iniciar un tratamiento antibitico ante un caso de faringitis con sospecha de ser por EBHGA, dado
puede demorarse hasta 9 das de comenzado los sntomas, para evitar la complicacin no supurativa del Streptococcus pyogenes. En las
faringitis por anaerobios, la droga de eleccin es la penicilina y puede ser administrada por va endovenosa u oral de acuerdo a las
circunstancias y necesidades del paciente; otros esquemas pueden ser propuestos, como clindamicina, ampicilina-sulbactam. Puede ser
necesario que paciente requiera un tratamiento quirrgico combinado, por ejemplo, de un absceso periamigdalino. La difteria y el
gonococo tienen tratamiento especfico. En la primera deber indicarse antitoxina diftrica y el antibitico correspondiente. El
tratamiento quirrgico (amigdalectoma) es una indicacin excepcional y se puede plantear ante la obstruccin de la va area, absceso
periamigdalino (por su alta recurrencia 20%) y discutido en FR y amigdalitis crnica. De realizarse extirpacin de la amgdala, es
importante su envo para completar el estudio anatomopatolgico a fin de poder descartar otras etiologas. Es importante diferenciar el
agente etiolgico ante la posibilidad de realizar un tratamiento especfico para evitar as las complicaciones supurativas y las no
supurativas (fiebre reumtica y glomerulonefritis).

CASO CLINICO
Paciente femenino de 24 meses de edad el cual inicia sbitamente padecimiento caracterizado por irritabilidad, disminucin en la
ingesta de alimento, dificultad para dormir, leve cianosis peribucal principalmente con el llanto, a la exploracin fsica se observa
estridor con respiracin superficial y dificultad respiratoria, presentando vomito durante la revisin.

PREGUNTA
Cul es la conducta a seguir.

RESPUESTA
a.- Indica antibitico de amplio espectro y desinflamatorio.
b.- Indica manejo ambulatorio y signos de alarma.
c.- Suministra primera dosis y enva a segundo nivel.
d.- Sumistra oxigeno, antibiticos y antipirtico.

PREGUNTA
El paciente presenta leve mejora sin embargo durante las siguientes 24 horas disminuye volumen urinario mas vomito recurrente, se
maneja restitucin de liquidos sin respuesta adecuada, cual es la manifestacin mas importante para envio a segundo nivel.

RESPUESTA
a.- Oliguria.
b.- Vomito.
c.- Examantema.
d.- Fiebre persistente.

CASO CLINICO
Se trata de masculino de 7 aos de edad el cual cuenta con antecedentes de cuadros repetidos infecciones de vas superiores altas, el
cual inicia con dolor farngeo de inicio sbito, con fiebre no cuantificada refiere la madre y cefalea, a la exploracin fsica se observa mal
estado general, exudado amigdalino e hiperemia.

PREGUNTA
Cual es la conducta a seguir.

RESPUESTA
a.- Medidas generales y antipirtico.
b.- Realizar exudado farngeo.
c.- Determinacin de antiestreptolisinas.
d.- Penicilina benzatinica 1200,000 UI dosis nica.

PREGUNTA
MANUAL DE TRABAJO DEL CURSO ENARM CMN SIGLO XXI
CURSO ENARM CMN SIGLO XXI TEL: 36246001 Pharmed Solutions Institute PGINA 432

Cual de los criterios no es por Centor para el diagnostico de faringitis estreptoccica.

RESPUESTA
a.- Ausencia de tos.
b.- Linfadenitis.
c.- Exudado amigdalino.
d.- Inicio sbito.

CASO CLINICO
Nia de 5 aos acude al servicio por presentar fiebre (38,6 C), odinofagia, trismus leve, rechazo alimentario, dolor y leve tumefaccin
latero-cervical izquierda y cierta dificultad a la movilizacin cervical. No presentaba signos de disnea. El cuadro empez cuatro das
antes. Se conoce alrgica a penicilina, portador de tetraologia de fallot, acompaado de retrazo mental.

PREGUNTA
Cual es la conducta a seguir.

RESPUESTA
a.- Amoxicilina- clavulanico.
b.- Trimetropima + sulfametoxazol.
c.- Clindamicina.
d.- Cefepime.

EPIGLOTITIS. CIENCIAS BASICAS: Tambin llamada supraglotitis es una celulitis bacteriana de la supraglotis, que puede obstruir
completamente la va area. Se reconoce como una entidad rpidamente progresiva, que pone en peligro la vida y que afecta
predominantemente a nios. SALUD PUBLICA: Clsicamente se presenta en nios de 2-6 aos, sin embargo se puede presentar a
cualquier edad, Incluso en neonatos. Existe una mayor incidencia en invierno y primavera. PATOGENIA: El Haemophilus influenzae tipo
B es el microorganismo identificado en el 90% de los casos. Esta bacteria puede ser parte de la flora normal de la nasofaringe o puede
ser adquirido mediante transmisin respiratoria. Otros patgenos aislados son el estreptococo B hemoltico de los grupos A y B,
estafilococos, neumococos, Klebsiella, Haemophilus parainfluenzae, Pseudomonas y Candida. Cuando el edema aumenta como
resultado del infiltrado inflamatorio difuso, la va area se obstruye parcialmente, pero el moco puede obstruirla completamente. Nio
grave con aspecto toxico. La progresin es rpida en trmino de horas. DIAGNOSTICO: Clnica; los 3 signos ms comunes de
supraglotitis son fiebre, dificultad respiratoria e irritabilidad. El paciente muestra estridor inspiratorio, retracciones, sialorrea, adems
de odinofagia y disfagia. Generalmente no cursan con disfona, ni tos. El estridor se presenta cuando la obstruccin de la va area es
casi completa. Asumen una posicin apoyados hacia delante y como olfateando (posicin en trpode). La aspiracin de secreciones en
la va area ya comprometida puede desencadenar laringoespasmo. El diagnostico de epiglotitis se hace mediante observacin directa
de la supraglotis, en un ambiente controlado, generalmente en quirfano. Laboratorio, se puede tomar BH, en la que se observa
leucositosis severa con neutrofilia. La utilizacin de radiografas es controversial. Aunque no se cuestiona que la supraglotis, puede ser
diagnosticada en una placa lateral de cuello (signo del pulgar), la pregunta es si realmente es un estudio necesario y sobre todo seguro.
TRATAMIENTO: Si se sospecha de una supraglotitis, el primer paso es evitar la estimulacin del nio. Una vez que se establece el
diagnostico hay que asegurar la va area. Por muchos aos se realizaba una traqueotoma, en cuanto se diagnosticaba al paciente.
Durante los ltimos 20 aos la intubacin endotraqueal ha sido el mtodo estndar de manejar la supraglotitis aguda. Se recomiendan
la intubacin de rutina en todos los pacientes con epiglotitis; en el momento de la intubacin se deben obtener muestras de la epiglotis
para cultivo y tomar hemocultivos. Se debe iniciar antibitico intravenoso; se maneja con ampicilina y cloranfenicol. Se recomienda
como 2 lnea de eleccin el ceftriaxone (100 mg/kg/da), cefotaxime (100 mg/kg/da) o ampicilina-sulbactam (200 mg/kg/da); la
duracin del tratamiento es usualmente de 10 a 14 das, complementado con antibiticos orales. El tratamiento con corticoesteroide es
copntroversial. Con el tratamiento adecuado el edema de la supraglotis cede dentro de las 48-72 hrs, por lo que se puede extubar al
paciente casi siempre al tercer da. COMPLICACIONES: La neumona la ms frecuente y, en menor porcentaje, meningitis, adenitis
cervical, pericarditis, artritis sptica y otitis media. Pueden presentarse complicaciones secundarias a la hipoxia, como dao en el SNC,
la liberacin de la obstruccin sbita de la va area puede producir edema pulmonar.

CASO CLINICO
Nia de 4 aos, previamente sana, con fiebre y sntomas catarrales de 24 horas de evolucin. Los padres la encuentran con quejido
respiratorio, a la exploracin clnica revela tiros intercostales, cianosis peribucal, respiracin superficial, sibilancias audibles a distancia y
estado de alerta disminuido, llanto dbil pero disfonico.

PREGUNTA
Cual es la gravedad de la obstruccin de la via area.

RESPUESTA
a.- Leve.
b.- Moderada.
c.- Grave.
d.- Severa.

PREGUNTA
Considerando el cuadro clnico cual es la patologia mas frecuente para establecer un diagnostico diferencial.
MANUAL DE TRABAJO DEL CURSO ENARM CMN SIGLO XXI
CURSO ENARM CMN SIGLO XXI TEL: 36246001 Pharmed Solutions Institute PGINA 433


RESPUESTA
a.- Cuerpo extrao.
b.- Difteria larngea.
c.- Epiglotitis post-vacunacion.
d.- Traqueitis bacteriana.

PREGUNTA
Cul es la conducta a seguir.

RESPUESTA
a.- Iniciar tratamiento farmacolgico.
b.- Administracin de oxigeno.
c.- Traslado a hospital.
d.- Administracion de prednisona.

LARINGOTRAQUEOBRONQUITIS (CRUP). CIENCIAS BASICAS: Es un sndrome respiratorio clnico agudo, de inicio sbito, de
presentacin en edad peditrica. Caracterizado por la trada estridor larngeo inspiratorio, tos ronca o perruna (traqueal), y disfona.
Hay signos de dificultad respiratoria secundario a la obstruccin larngea o traqueal; trmino usado casi exclusivamente para referirse a
la laringotraquetis de origen viral, frecuentemente precedida de un episodio de coriza, rinorrea clara y leve aumento de la
temperatura. Se considera una enfermedad que se autolimita, sin embargo puede evolucionar a dificultad respiratoria grave. Algunos
nios con historia de atopia, tienen un cuadro no precedido de sntomas virales que se le ha llamado CRUP espasmdico. Ambos son
tratados similarmente siempre y cuando los signos y sntomas sean los mismos. SALUD PUBLICA: Afecta a nios entre 3 meses y 6 aos
de edad, con un pico a los 2 aos. Ms frecuente en sexo masculino. Predomina en otoo e invierno. Con una incidencia anual de 18
por 1.000 nios menores de 6 aos de edad. PATOGENIA: Los agentes causales ms frecuentes son el virus parainfluenza (65%) tipo 1
(50%), 2 y 3; con menor frecuencia; adenovirus, sincitial respiratorio, influenza A y B, sarampin y excepcionalmente; Mycoplasma,
enterovirus, parotiditis, rinovirus y difteria. El padecimiento inicia en la nasofaringe, se extiende por el epitelio respiratorio de la laringe
y la trquea. El virus se adhiere directamente a la mucosa, mediante los macrfagos o los linfocitos, provocando degranulacin con
liberacin de mediadores inflamatorios preformados o de neo formacin, que ocasionan inflamacin difusa con eritema y edema que
en las cuerdas vocales, disminuye la movilidad y ocasiona disfona y participa en el estridor; en la trquea provoca tos ronca. En la
regin subgltica que es la porcin ms estrecha de las vas respiratorias superiores en el nio, con esqueleto cartilaginoso, el edema
provoca disminucin del calibre, con dificultad para el flujo areo, que se manifiesta por estridor inspiratorio. Si la enfermedad
progresa se forman exudados fibrinosos y pseudomembranas que aumentan la obstruccin. La extensin hacia los bronquios o
alveolos, ocasiona laringotraqueobronquitis, o laringotraqueobronconeumonitis respectivamente, frecuentemente son provocados por
complicacin bacteriana de la enfemedad viral inicial. DIAGNOSTICO: Se recomienda realizar el diagnostico de CRUP basados en las
manifestaciones de la triada clnica: disfona, estridor larngeo inspiratorio y tos traqueal; aunados a los siguientes antecedentes, inicio
sbito, fase prodrmica de 12-48hrs previas con rinorrea, fiebre y tos no traqueal. Suelen presentarse signos progresivops de dificultad
respiratoria. Puede encontrase disminucin del murmullo vesicular por la obstruccin y datos variables de dificultad respiratoria como
agitacin por hipoxia, aleteo nasal, tiros supraesternales e intercostales, ms severos cianosis, tiros supraclaviculares, inquieto y
ansioso. Generalmente no se requieren estudios de laboratorio; BH leucocitos con elevacin moderada con predominio de linfocitos.
Se pueden solicitar cultivos de virus y pruebas serolgicas para identificar el virus causal. La radiografa de trax muestra como dato
caracterstico, estrechamiento de la columna area a nivel subglotico y el signo de punta de lpiz o en torre de capilla. CLASIFICACION:
Forbes la cataloga en 4 etapas de acuerdo a gravedad y progresin: Etapa 1; fiebre, disfona, tos traqueal y estridor larngeo
inspiratorio al explorar al paciente. Etapa 2; Estridor respiratorio continuo y signos de insuficiencia respiratoria. Etapa 3;
manifestaciones clnicas de hipoxia e hipercapnia: inquietud, ansiedad, palidez, diaforesis y taquipnea. Etapa 4; cianosis intermitente,
cianosis permanente y paro respiratorio. Con el objetivo de iniciar rpidamente el tratamiento, es conveniente calificar la gravedad de
la enfermedad. Con la escala del cuadro adjunto. Tambin se puede utilizar la siguiente valoracin: LEVE: tos traqueal intermitente,
estridor solo cuando se agita, no en reposo, taquipnea menor de 40/min y taquicardia menor 100/min. No hay inquietud, el nio esta
hidratado y su estado mental es normal. MODERADA: Estridor audible en reposo, que aumenta cuando se agita, tos traqueal constante,
aumento del trabajo respiratorio con tiros, frecuencia cardiaca >100/min y respiratoria >40/min, pero menor de 50. Puede mostrase
exigente, pero se encuentra alerta y es confortado por sus padres. SEVERA: Se agrega al cuadro clnico anterior hipoxia con cianosis,
gran inquietud y alteraciones de la conciencia con empeoramiento de las manifestaciones de insuficiencia respiratoria. TRATAMIENTO:
En 90% de los casos es ambulatorio, menos de 10% se hospitaliza, apenas 1% llega a requerir apoyo ventilatorio. 1. Medidas generales:
reposo ambiente tranquilo, control de la temperatura, hidratacin adecuada. 2. Va area permeable: la manera tradicional efectiva es
humedecer el ambiente. El ambiente hmedo es tan efectivo como la nebulizacin fra, este ltimo es ms seguro evita quemaduras
por el agua caliente. La nebulizacin fra, humedece la secreciones, produce vasoconstriccin que ayuda a desinflamar la mucosa,
tambin disminuye la viscosidad de las secreciones mucosas de trquea y bronquios. Se recomienda agregar oxgeno al 30-40% para
prevenir o tratar la hipoxia. 3. Epinefrina (adrenalina): Se administra por nebulizacin simple, ha hecho que en la actualidad la
traqueotoma no sea necesaria. Se cree que su mecanismo de accin, es la estimulacin de los receptores adrenrgicos con la
subsecuente vasoconstriccin de los capilares arteriales, lo que da lugar a reabsorcin de lquido en lugar de fuga-, a partir del espacio
intersticial, con la disminucin consecuente del edema larngeo y de secreciones mucosas de trquea y bronquios. Debe vigilarse la
aparicin de taquicardia e hipotensin. Deber administrase en un servicio de urgencias solo a nios severamente enfermos (>5); se
emplea al 2.25% en 3 ml de solucin salina normal durante 20 min, 0.25 ml en nios con peso de <20Kg, 0.5ml si peso 21-39Kg y 0.75ml
si el peso es >40Kg. 4. Corticoesteroide: La dexametasona es el ms aceptado y empleado, tiene accin prolongada (V de 36-72 hrs).
Su efecto antiinflamtorio disminuye el edema de la mucosa larngea entre 2-6 hrs despus de su administracin. La dosis terapeutioca
varia de 0.15-0.6 mg/kg IM en una sola dosis. O.6mg/kg es la ms efectiva, sin pasar de 10 mg como dosis total. 5. Intubacion
MANUAL DE TRABAJO DEL CURSO ENARM CMN SIGLO XXI
CURSO ENARM CMN SIGLO XXI TEL: 36246001 Pharmed Solutions Institute PGINA 434

endotraqueal: si no hay respuesta a las medidas anteriores, y se observa obstruccin progresiva de la via area, con signos de fatiga en
el nio por trabajo respiratorio laborioso que sugiera hipoxia o hipercapnia y alteracin de la conciencia. Los antibiticos no estn
indicados, solo estn reservados en aquellos pacientes que se sospeche una sobreinfeccin bacteriana, una laringotraqueitis bacteriana
o una laringotraqueobronconeumonitis, y se debe cubrir principalmente Staphylococcus aureus, Streptococcus pyogenes,
Streptococcus pneumoniae, y Haemophilus

CASO CLINICO
Ingresa mascilino de 4 aos de edad el cual inicia padecimiento sbito con estridor larngeo inspiratorio, refiere la madre que hace dos
das inicio con tos, rinorrea y fiebre no cuantificada, motivo por el cual acudi a consulta en centro de salud donde fue indicado
tratamiento con analgsico, medidas generales y datos de alarma, agrega que una hora antes del ingreso se incrementaron los
sntomas con dificultad para hablar, inquietud, llanto con disfona y tos seca.

PREGUNTA
Cuantos criterios clnicos clsicos presenta el paciente para LTA.

RESPUESTA
a.- 1
b.- 2
c.- 3
d.- 4

PREGUNTA
Cual es la conducta a seguir para establecer la diagnostico clinico.

RESPUESTA
a.- Biometria hematica.
b.- Radiografia de cuello.
c.- Administrar corticoides.
d.- Administrar adrenalina.

PREGUNTA
Cual es la conducta teraputica a seguir.
a.- Administracion de dexametasona .60mg/Kg DU.
b.- Budesonida inhalada.
C.- Fluquitasona.
d.- L-epinefrina nebulizada 1:1000

CASO CLINICO
Se trata de femenino de 22 meses de edad la cual es llevada a urgencias por su madre debido a que la observa que no responde a
estimulos, el nico antecedentes es la presencia de tos, fiebre y rinorrea que fue tratada con frmacos sintomticos, a la exploracin
fsica se observa paciente letrgica, hipotnica, con dificultad respiratoria, estridor larngeo, palidez distal y cianosis central, FC 69, FR
42.

PREGUNTA
Considerando la sintomatologia cual es el estado de la LTA segn los criterios de Westley.

RESPUESTA
a.- Leve.
b.- Moderada.
c.- Severa.
d.- Amenaza la vida.

PREGUNTA
Cual es la conducta a seguir.

RESPUESTA
a.- Iniciar RCP.
b.- Intubacion endotraqueal.
c.- Ventilacion positiva intermitente.
d.- L-epinefrina SC.

MANUAL DE TRABAJO DEL CURSO ENARM CMN SIGLO XXI
CURSO ENARM CMN SIGLO XXI TEL: 36246001 Pharmed Solutions Institute PGINA 435


BRONQUITIS. CIENCIAS BASICAS: La bronquitis aguda,
desde un punto de vista fisiopatolgico, se refiere a la
inflamacin aguda de la mucosa bronquial. Clnicamente, no
est muy bien definida, pero se trata de una infeccin
respiratoria de etiologa viral que no suele durar ms de dos
semanas. La mayora de los autores estn de acuerdo en
que la tos, con o sin expectoracin, es el sntoma ms
comnmente observado. Es una enfermedad adquirida en
la comunidad autolimitada. SALUD PUBLICA: Afecta a nios
en edad peditrica, ms frecuente en menores de 2 aos.
No hay estadsticas exactas. Ocupa la sptima causa de
muerte infantil en el apartado de infecciones respiratorias
junto con la bronquiolitis. PATOGENIA: Los virus
respiratorios: influenza, parainfluenza, virus respiratorio
sincitial son los responsables de la mayora de los casos de
bronquitis aguda. Otros grmenes aislados ocasionalmente
son: Bordetella pertusis, parapertusis, Mycoplasma
pneumoniae y Chlamidia pneumoniae. Las bronquitis
ocasionadas por el virus de influenza A, son graves y
frecuentemente producen pandemias. El virus entra por la
nariz o por la boca, se replica en los acmulos linfticos del
anillo de Waldeyer; puede provocar viremia y
posteriormente se extiende ms abajo llegando a los bronquios, en la mucosa bronquial se adhieren directamente a ella o bien son
transportados por macrfagos, a los cuales pueden destruir o no; estos ltimos liberan mediadores inflamatorios ya elaborados y
almacenados en sus granulos o de una nueva elaboracin, los que provocan dao a la mucosa, edema, inflamacin con disminucin de
la motilidad ciliar, aumento en la secrecin mucosa y disminucin de la luz bronquial, con resistencia al flujo del aire, mayor cuanto
menor es el nio, ya que hay relacin directa entre la edad del paciente y el calibre de la luz bronquial. Dependiendo del germen, de su
virulencia, del tamao del inoculo y de las condiciones del hospedero, pueden ocasionar necrosis de las clulas superficiales de la
mucosa de las vas respiratorias. El periodo de incubacin va de 2-14 das segn agente etiolgico. DIAGNOSTICO: Eminentemente es
clnico; Estornudos, rinorrea hialina, dolor, ardor farngeo y fiebre de moderada a severa. La tos es inicialmente seca, ronca, por la
irritacin de la mucosa farngea y traqueal y posteriormente se vuelve hmeda, productiva; puede ser hemetizante y frecuentemente
se acompaa de dolor retroesternal. Al examen fsico el paciente est inquieto, febril, se observa la mucosa nasal edematosa y
enrojecida, la faringe hiperemica con moco hialino. A la auscultacin se escuchan estertores bronquiales diseminados en ambos
campos pulmonares, es frecuente encontrar roncus y sibilantes. El esputo o las flemas pueden ser transparentes, de color blanco,
amarillo o verdoso; estos cambios de coloracin no estn relacionados con la sobreinfeccin bacteriana, sino con la peroxidasa liberada
por los leucocitos presentes por el moco. Por lo tanto, el moco verde no es motivo para instaurar tratamiento antibitico. La
enfermedad en los nios puede presentarse en tres fases clnicas: 1. Prdromos; 2-3 das, fiebre, estornudos, rinorrea serosa, tos seca
disfonica. 2. Estado; 4-6 das, fiebre, mal estado general y tos productiva. 3. Recuperacin; persistencia de la tos que se vuelve
paulatinamente seca y aislada, puede durar hasta 2 semanas. En caso de complicacin bacteriana, la enfermedad se prolonga y se
agregan mal estado general, tos seca molesta con dolor retroesternal y dificultad para respirar. Si el germen es Mycoplasma
pneumoniae con frecuencia se presenta neumona y complicaciones como derrame pleural, absceso pulmonar e insuficiencia
respiratoria grave. Los exmenes de laboratorio solo con fines de estudio o enseanza, o en caso de complicaciones: BH; leucocitosis
leve con linfocitosis o neutrofilia. La radiografa de trax es normal o bien muestra engrosamiento de la trama broncovascular y la
imagen especfica en caso de complicacin. Se puede solicitar cultivo para virus a partir de secreciones farngeas, tambin se pueden
realizar pruebas serolgicas. Las pruebas de inmunofluorescencia en secreciones nasofarngeas son altamente sensibles y son tiles en
el diagnstico rpido. La PCR se ha convertido en el estndar de oro para diagnstico de infeccin pop B. pertussis. TRATAMIENTO: Es
sintomtico, medidas generales tendientes a mantener buena hidratacin, temperatura en lmites normales y si la tos es frecuente y
molesta, en primera instancia ambiente hmedo con nebulizador ultrasnico tibio con termostato. Si no se mejora la tos agregar
mucolticos como el ambroxol que se ha referenciado mejora la motilidad ciliar y facilita la recuperacin de la funcin de la mucosa as
como la eliminacin de las secreciones. En casos especficos y graves se recomienda el empleo de antivirales: por ejemplo amantadina
en infecciones por virus de la influenza o ribavirina si esta ocasionado por el virus sincitial respiratorio. Si se sospecha o confirma
infeccin bacteriana concomitantemente o complicando el cuadro inicial, debe emplearse el antibitico de eleccin para el germen de
que se trate, si se sospecha tos ferina (azitromicina), infeccin por Mycoplasma pneumoniae (un macrlido). Los broncodilatadores
inhalados no estn indicados de manera rutinaria, slo en aquellos casos en los que se asocie broncoespasmo. Sin embargo, hay nios
con hiperreactividad bronquial que presentan un mnimo e incluso no apreciable broncoespasmo a la exploracin, pero que responden
de manera espectacular al broncodilatador inhalado, con resolucin de la tos.

CASO CLINICO
Masculino de 3 aos de edad con sndrome de Down y cardiopata congnita, es trado a urgencias por cuadro catarral de 7 das de
evolucin con tratamiento sintomtico el cual mejoro parcialmente pero fue lentamente reagudizado, durante las dos semanas previas
cambio en dos ocaciones de tratamiento, existe el antecedente que sus dos hermanos presentaban cuadro catarral. A las 12 h del
ingreso presenta empeoramiento progresivo de la dificultad respiratoria con aparicin de fiebre, secreciones mucopurulentas y tos
continua, taquicardico y taquipnea.

PREGUNTA
MANUAL DE TRABAJO DEL CURSO ENARM CMN SIGLO XXI
CURSO ENARM CMN SIGLO XXI TEL: 36246001 Pharmed Solutions Institute PGINA 436

Cual es el agente etiolgico mas probable de este caso.

RESPUESTA
a.- Influenza A, influenza B, parainfluenza.
b.- Virus sincitial respiratorio.
c.- Coronavirus, adenovirus y rinovirus.
d.- Bordetella pertusis, Mycoplasma pneumoniae y C pneumoniae.

BRONQUIOLITIS. CIENCIAS BASICAS: Es una inflamacin de los bronquiolos, habitualmente de etiologa viral, aguda y frecuentemente
contagiosa. Debido a la inflamacin de los bronquiolos, se manifiesta por tos, sibilancias, disnea y taquipnea. Por lo general
autolimitada. SALUD PUBLICA: Se presenta en menores de 2 aos de edad, el 80% ocurre en menores de 1 ao (pico entre 6-8 meses).
Predomina en el sexo masculino. Es ms frecuente en invierno, pero puede presentarse a lo largo del ao. PATOGENIA: El germen
patgeno ms frecuente encontrado en caso de bronquiolitis es el virus sinsitial respiratorio (VSR), hasta en 60% de los casos. El resto
es ocasionado por parainfluenza, adenovirus, rinovirus y el virus de la influenza. Tambin puede haber coexistencia entre VSR con
Chlamydia trachomatis Y Mycoplasma pneumoniae, as como virus diferentes a los ya sealados, los que en conjunto constituyen entre
5-10%. El VSR se introduce al organismo principalmente por contacto directo (se extienden hasta 2 m) con secreciones nasales de
enfermos; quien elimina virus hasta por 10dias despus del inicio del padecimiento. En gotas grandes el virus puede sobrevivir y
mantener contaminantes las superficies en las que se encuentre, su periodo de incubacin es de 4-6 das. Afecta el sistema respiratorio
por colonizacin y replicacin en la mucosa de los bronquios. En los nios pequeos los bronquiolos son el sitio primeramente
afectado, mientras que en nios mayores y adultos, se afectan las vas respiratorias superiores. Hay necrosis de las clulas ciliadas con
proliferacin de clulas no ciliadas como linfocitos, clulas plasmticas y macrfagos en las reas peribronquiales; se liberan
mediadores inflamatorios, lo que ocasiona edema, inflamacin y congestin de la submucosa con taponamiento de los bronquiolos por
moco y detritus celulares; que en conjunto ocasionan estrechamiento de las vas respiratorias perifricas. La respuesta pulmonar a
estos cambios, es un aumento en la capacidad residual funcional, aumento en la resistencia al paso de aire y aumento en los
cortocircuitos pulmonares. Todos estos factores aumentan el trabajo respiratorio dando lugar a alteraciones en el recambio de gases
secundario a obstruccin y atelectasia de las vas areas. Tambin puede presentarse hipoxia y retencin de CO2, secundarios a la
alteracin de la relacin ventilacin/flujo, junto con hipovetilacin. DIAGNOSTICO: Es clnico, se presentan con antecedente de 1-5 das
con rinorrea profusa habitualmente hialina y congestin nasal, en algunos tos y fiebre leve a moderada. Al 4-5 da se agrega dificultad
para respirar con tos hmeda, taquicardia y sibilancias. El nio se muestra irritable y letrgico; en los muy pequeos se puede presentar
apnea, la enfermedad progresa con insuficiencia respiratoria grave (cianosis, actividad de msculos accesorios de la respiracin, disnea
y taquipnea, ms grave quejido, aleteo y tiro supraesternal) y ameritan intubacin. En cuadros severos y con varios das de evolucin
puede haber deshidratacin. Tambin hay signos de hiperinflacin (sobre distensin pulmonar), manifestados por aumento del
dimetro anteroposterior del trax y crecimiento de hgado y bazo que se palpan abajo del borde costal. A la auscultacin se pueden or
sibilancias inspiratorias, disminucin de la entrada de aire y estertores inspiratorios. Datos de gravedad por Shaw, para predecir
evolucin y tratamiento: 1. Aspecto toxico. 2. SaO2 <95%. 3. Edad gestacional menos de 34semanas. 4. Frecuencia respiratoria >70/min.
5. Atelectasia en la radiografa de trax. 6. Edad menor de 3 meses. Por otro lado se puede hacer la determinacin de PaCO2,
habitualmente se encuentra entre 30-35mm Hg. Valores entre 45-55mmHg son peligrosos e indican falla respiratoria inminente. El
criterio de oro para el diagnstico es el cultivo nasofarngeo positivo para VSR. Existen pruebas rpidas que incluyen ELISA y tincin
directa de anticuerpos fluorescentes, son ms rpidas y tienen especificidad y sensibilidad cercana al 90%. La serologa para
anticuerpos del virus tiene limitaciones. La determinacin de leucocitos y electrolitos solo debe hacerse si hay deshidratacin o se
sospecha sepsis. La gasometra se indica para valorar severidad de compromiso respiratorio. La radiografa de trax no es necesaria,
puede ser til en los siguientes casos: insuficiencia respiratoria leve a moderada, sibilancias recurrentes, aspecto toxico. La imagen
puede mostrar horizontalizacin variable de las costillas, con aumento del espacio intercostal, diafragmas abatidos, hiperinflacin
(hiperclaridad), atelectasia, engrosamiento peribronquial e infiltrados intersticiales difusos. Aprox 10% de las placas son normales y
puede no haber correlacin con la severidad de la enfermedad. TRATAMIENTO: Primero valoracin de la gravedad. Las medidas ms
importantes son la hidratacin (si hay deshidratacin o hay bajo nivel de eliminacin urinaria) y oxigenoterapia por puntas nasales,
mascarilla o casco ceflico, debe medirse con frecuencia la saturacin de O2 con oximetro de pulso para adecuar la cantidad de O2 a
administrar. La aplicacin de nebulizacin de epinefrina racemica (mezcla de ismeros d y L) es aceptada, mejora la saturacin de O2 y
los datos clnicos de insuficiencia respiratoria, aunque en muchos pases no est disponible por lo que se usa la adrenalina (L-
epinefrina), con los mismos resultados, solo se usa en nios severamente enfermos y en quienes es inminente la intubacin traqueal. La
dosis recomendad es epinefrina racemica al 2.25% en 3 ml de sol. Salina normal durante 20 min. En nios con peso de <20Kg, 0.5ml si
peso 21-39Kg y 0.75ml si el peso es >40Kg (0.5ml de epinefrina racemica equivalen a 5ml de L-epinefrina). El empleo de esteroides y -
agonistas en estos pequeos no es aconsejable a luz de mltiples reportes. El nico antiviral acepado para este padecimiento es la
ribavirina, el cual tienen actividad virostatica, se recomienda en nios muy pequeos, inmunodeprimidos, con malformaciones
cardiacas y los severamente enfermos, solo se emplea en inhalacin y el personal de salud debe protegerse de aspirar el medicamento
ya que se han descrito crisis de espasmo bronquial y malformaciones congnitas. Hay reportes de empleo de factor surfactante,
oxigenacin por membrana extracorprea (OMEC) y vitamina A.

CASO CLINICO
Se trata de masculino de 6 meses de edad con peso adecuado, con antecentes, con diagnstico de reflujo gastroesofgico, en
tratamiento con Domperidona y Omeprazol. Vacunas completas. Antecedentes familiares: Padre con antecedente de broncoespasmo y
atopa. Comenz 5 das previos al ingreso con tos y dificultad respiratoria, por lo cual se evalu en la guardia de otro hospital,
indicndose tratamiento ambulatorio con broncodilatadores cada 4 hs. y Prednisona durante 5 das. El paciente evolucion sin mejora
clnica, intensificndose la dificultad respiratoria durante las ltimas 24 hs. y se agreg regular actitud alimentaria. No present fiebre.
Consult en la guardia, donde se lo encontr taquipneico, (fr) 70 por minuto (pm), con (fc) de 145 pm, Sat 91% aire ambiente (aa), tiraje
subcostal e intercostal, regular entrada de aire bilateral y sibilancias audibles.
MANUAL DE TRABAJO DEL CURSO ENARM CMN SIGLO XXI
CURSO ENARM CMN SIGLO XXI TEL: 36246001 Pharmed Solutions Institute PGINA 437

PREGUNTA
Cual es la conducta a seguir inmediata a seguir es la menos adecuada?

RESPUESTA
a.- Nebulizaciones con broncodilatadores
b.- Hidrocortisona a 10 mg/kg.
c.- Monitorizacion de saturacin perifrica.
d.- Evaluacin de gases arteriales.

BRONCONEUMONIA. CIENCIAS BASICAS: El trmino es introducido en 1837 por Seiffert, a travs de la idea de infiltrados pulmonares
de origen broncognico o bronquiolognico. Se ha relacionado con neumona multifocal (o lobulillar), ahora frecuentemente llamada
de focos mltiples. Infeccin de la va respiratoria producida por un microorganismo (bacteria o virus). Es una infeccin de inicio
violento y repentino que produce inflamacin en pulmones y bronquios, la cual genera trastornos respiratorios que si no son tratados
oportunamente pueden conducir a la muerte. Es ocasionada por virus o bacterias y afecta particularmente a nios, personas de edad
avanzada y a pacientes con dao en su sistema inmunolgico (inmunosuprimidos). SALUD PUBLICA: Los bebs que tienen entre 3-6
meses de edad son quienes resultan ms comnmente afectados, si bien esta enfermedad puede aparecer hasta los dos aos. Se
calcula que aproximadamente uno de cada tres bebs tendr bronconeumona en algn momento durante su primer ao de vida. Es
ms frecuente durante los meses de invierno. PATOGENIA: La bronconeumona es una lesin secundaria que aparece generalmente
como complicacin de una enfermedad (gripe, resfriados mal cuidados, bronquitis, bronquiolitis o tos ferina). A diferencia de la
neumona, no posee fases evolutivas y el exudado no contiene fibrina o tiene muy poca. La bronconeumona es causada por uno de
varios tipos de virus. En tres cuartos de los casos, es provocada por el virus sincitial respiratorio, por contacto directo con la tos de un
enfermo. Tumores pulmonares que evitan la ventilacin adecuada del aire a los pulmones, los cuales favorecen el establecimiento de
bacterias. La caracterstica dominante de la bronconeumona es la consolidacin parcheada del pulmn. DIAGNOSTICO: Clnica; Cuadro
febril sin otro sntoma, o bien como dolor de trax, o abdomen. Tos, decaimiento, inapetencia o rechazo al alimento, cuadro de resfro
que empeora progresivamente, insuficiencia respiratoria severa, taquipnea, hundimiento de costillas al respirar, aleteo nasal, quejido,
retraccin xifoidea. Anatoma patolgica: Las lesiones bronconeumnicas consisten en focos de condensacin pequeos, a veces,
confluentes, gris rojizos, secos, finamente granulosos. Frecuentemente, sin embargo, son poco notorios macroscpicamente y se
manifiestan como zonas ligeramente levantadas, hipermicas, que se descubren mejor por palpacin que por inspeccin. Estos focos
pueden pasar inadvertidos macroscpicamente, no as en el examen del pulmn fijado previamente. Los focos bronconeumnicos se
encuentran frecuentemente en las regiones dorso-basales y laterales de los lbulos inferiores. En el centro del pulmn los focos son
mayores que en la periferia, donde tienden a ser ms densos. A menudo alcanzan la pleura, donde se desarrolla entonces una pleuritis
fibrinosa o purulenta. BRONCONEUMONA ESTAFILOCCICA: Se observa en el 31% de las autopsias. Corresponde al 5% de las
neumonas bacterianas. Mortalidad cercana al 20%. El tipo de reaccin inflamatoria es la inflamacin necrotizante y abscedante. Es una
afeccin secundaria a piodermitis, furunculosis, endocarditis, osteomielitis y otras. Los abscesos se forman a partir de embolias spticas
en arteriolas y capilares. El empiema y el pioneumtorax son acompaantes frecuentes. La neumona aergena, primaria o secundaria
a bronquitis viral, se observa como una neumona con infiltrados purulentos mal delimitados, confluentes con tendencia a la necrosis.
El pronstico de esta forma es muy malo, especialmente la forma primaria en lactantes. BRONCONEUMONA ESTREPTOCCICA: Se
caracteriza por un exudado hemorrgico y flegmonoso, pobre en fibrina. En la forma hematgena, se constituyen infiltrados maculares
(bronconeumnicos) y simultneamente se desarrollan flegmones pleurales, septales y perilobulillares. La forma aergena es ms
frecuente y corresponde en verdad a una bronconeumona purulenta confluente. La pleuritis purulenta y el empiema son tambin
frecuentes. BRONCONEUMONIA POR ASPIRACION: Tambin predominan en lbulos inferiores y ms frecuentemente al lado derecho.
Los cuerpos extraos pueden producir una estasis de secreciones bronquiales, heridas de la pared con necrosis y neumona purulenta
consecutiva, a menudo gangrenosa. La circunstancia ms frecuente es la aspiracin de vmitos con contenido gstrico, que produce
una inflamacin necrotizante, bronquial y alveolar, por la accin corrosiva del cido. En la agona se puede producir aspiracin de
contenido gstrico y digestin pulmonar post-mortem (neumomalacia cida). Otra situacin frecuente es la aspiracin de lquido
amnitico. El feto respira in tero e inhala lquido amnitico normalmente. En la asfixia neonatal aparece meconio en el lquido
amnitico y en el pulmn se observan partculas de vrnix caseoso o de meconio y escamas crneas. Como es un fenmeno normal, la
distincin suele ser difcil y, al final, la diferencia es slo cuantitativa. Cuando la aspiracin es masiva, puede observarse una
bronconeumona leucocitaria reactiva.

CASO CLINICO
Paciente de 17 dias de vida extrauterina que presenta fiebre, irrabilidad, y ampollas en el cuerpo. EF febril, hidratado, lloroso e
intranquilo, ligeramente taquipneico con exantema de distribucin centrpeta en estadios evolutivos de vesculas y pustulas distribuido
en cara, cuello y tronco. Orofaringe congestiva y disminucin de murmullo vesicular bilateral en campos pulmonares. Rx de torax:
presencia de infiltrado bronconeumonico bilateral a predominio de hilios y campos inferiores. Laboratorios leucos 11 mil, acidosis
metabolica compensada. VSG 1ah: 14 mm y PCR 0.03 mg/l.

PREGUNTA
Cual es la conducta teraputica mas adecuada a seguir en el caso?

RESPUESTA
a.- Oxigeno, liquidos y aines.
b.- Oxigeno, liquidos y electrolitos, vigilancia.
c.- Oxigeno, antibiticos, liquidos y aines.
d.- Oxigeno, antibiticos, liquidos y corticoides.

MANUAL DE TRABAJO DEL CURSO ENARM CMN SIGLO XXI
CURSO ENARM CMN SIGLO XXI TEL: 36246001 Pharmed Solutions Institute PGINA 438

NEUMONIAS. CIENCIAS BASICAS: Se define como la inflamacin y condensacin del parnquima pulmonar causada por un agente
infeccioso o por factores no infecciosos como la aspiracin de cido gstrico, cuerpos extraos e hidrocarburos; as como las reacciones
de hipersensibilidad y las neumonitis inducidas por frmacos o radiaciones. El la neumona existe reemplazo del contenido areo de los
alveolos y conductos alveolares por clulas y exudado inflamatorio, que se manifiesta por la presencia de sntomas y signos de infeccin
aguda y la presencia de imgenes radiolgicas que indican ocupacin alveolar. SALUD PUBLICA: En 2002 en Mxico se reportaron 189
806 casos de neumona. La mayora de los casos se reportaron en nios de 1-5 aos. Su incidencia en este grupo de edad es muy
elevada, y se reportan 10-40 casos: 1000 nios/ao. PATOGENIA: Durante los 3 primeros aos de vida las neumonas estn causadas
por virus, especialmente por el sinsitial respiratorio (VSR) y los virus de la gripe, tambin virus influenza A y parainfluenza tipo 1. A
partir de los 3 aos pueden aislarse Mycoplasma pneumoniae y Chlamydia pneumoniae. Aunque los agentes bacterianos se asocian
menos frecuentemente con neumonas, se ha evidenciado que causan infecciones ms graves. Patgenos comunes: Streptococo
pneumoniae, Mycoplasma pneumoniae, Chlamydia pneumoniae, Coxiella burnetii, virus respiratoria. Patgenos no comunes:
Legionella pneumophila, Haemophilus influenzae, enterobacterias, flora saprofita bucal, Moraxalla catarrhalis. El evento que con mayor
frecuencia altera los mecanismos de defensa de la va area es una infeccin viral que altera las propiedades de las secreciones, inhibe
la fagocitosis, modifica la flora bacteriana y puede temporalmente romper de manera temporal al epitelio de la va area. Es comn
que se presente una infeccin viral unos das previos al desarrollo de neumona bacteriana; sin embargo, su presencia no es
indispensable para que las bacterias induzcan enfermedad. Los microorganismos que causan neumona se adquieren en la inmensa
mayora de los casos, por va respiratoria y alcanzan las vas respiratorias bajas por trayecto descendente desde las vas respiratorias
altas. De manera inicial, el edema reactivo favorece la proliferacin de los microrganismos y colabora en su diseminacin a porciones
adyacentes al pulmn. Uno ms lbulos generalmente se involucran, en nios pequeos predomina un patrn ms difuso que sigue la
distribucin bronquial y se caracteriza por muchas reas de consolidacin alrededor de las vas respiratorias pequeas, el dao
permanente es raro. Los pacientes con inmunodeficiencias primarias o secundarias tienen un riesgo mayor de infeccin, recurrencias y
complicaciones. DIAGNOSTICO: Clnica; las manifestaciones son secundarias a la respuesta inflamatoria sistmica y local a la infeccin;
por consiguiente, son de dos tipos; generales como fiebre, malestar general escalofros y cefalea y respiratorios como tos, disnea,
taquipnea y anomalas en la auscultacin torcica. La intensidad de los sntomas as como la gravedad de la enfermedad varan segn el
agente etiolgico, ya que mientras unos dan lugar a una consolidacin pulmonar localizada, otros provocan una inflamacin ms difusa.
Tambin depende de la edad del paciente, especialmente en nios pequeos. La radiografa de trax es til para corroborar el
diagnstico y descartar complicaciones. En aproximadamente el 20% de las neumonas hay derrame pleural, que en una minora de
nios evolucionara a empiema. Las neumonas atpicas suelen ocasionar un infiltrado heterogneo y poco denso, con aspecto de vidrio
despulido, que tiende a estar situado cerca del hilio, sobre todo en los lbulos inferiores; a menudo las imgenes de ocupacin alveolar
afectan varios lbulos, en ambos pulmones; as el patrn radiolgico mas frecuente es el de un infiltrado parahiliar peribronquial
unilateral o bilateral. Aunque se ve pocas veces es muy caracterstico de infeccin por M.pneumoniae la presencia de unas imgenes
reticulonodulillares localizadas en un solo lbulo inferior. Los hemocultivos tienen pobre sensibilidad en las neumonas (10-30%). La
deteccin de antgenos bacterianos en sangre y orina tienen utilidad diagnostica variada. Los estudios serolgicos, tiles en
epidemiologia, tienen escasa utilidad clnica. Las pruebas rpidas de deteccin de antgenos bacterianos en secreciones nasofarngeas
mediante inmunofluorescencia directa o ELISA resultan muy tiles para la identificacin de virus respiratorios, pero tienen
disponibilidad limitada y altos costos. INFECCIONES VIRALES; tpicamente rinorrea, estornudos y tos. Temperatura ligeramente elevada,
podemos encontrar taquipnea, tiros intercostales, aleteo nasal y el uso de msculos accesorios, las infecciones graves se acompaan de
cianosis y dificultad respiratoria de moderada a grave especialmente en lactantes. A la auscultacin de trax se puede evidenciar
estertores y sibilancias. Las neumonas vricas tambin tienden a presentar un patrn de infiltrado parahiliar peribronquial, ms o
menos difuso a veces acompaado de atelectasias; puede haber imgenes micronodulares difusas y sobredistensin pulmonar,
tambin son posibles otras imgenes, como el aumento de densidad localizada, segmentario o lobular. Es difcil identificar el agente
etiolgico. INFECCION POR STREPTOCOCO PNEUMONIAE: Historia clsica de escalofros de inicio sbito, seguidos por fiebre alta, tos y
dolor de trax. Los lactantes presentan varios das de congestin nasal, inquietud e hiporexia de manera inicial. De manera sbita
presentan fiebre, astenia y aprensin, el paciente se ve enfermo, a la auscultacin de trax puede proporcionar pocos signos, pero
puede evidenciarse un rea de consolidacin por percusin, as como incremento de las vibraciones torcicas y estertores. Los nios
ms grandes tienen escalofros y fiebre despus de una infeccin respiratoria alta. Tambin tienen ataque al estado general, taquipnea,
tos seca y en casos graves alteraciones de la conciencia, a la auscultacin, buscar lo mismo que en los lactantes y adems buscar
derrame por percusin. NEUMONIA POR NEUMOCOCO: La BH, muestra formula blanca incrementada (15,000-40,000/mm
3
), con
predominio de PMN. La gasometra muestra hipoxia con hipercapnia. El neumococo puede ser aislado de las secreciones nasofarngeas
en la mayora de los pacientes, sin embargo, debe tenerse en cuanta que 10-15% de la poblacin sana es portadora. El aislamiento de la
bacteria en sangre o en lquido pleural es diagnstico. En las radiografas se evidencia consolidacin aun antes de la deteccin de la
neumona. INFECCION POR H. INFLUENZAE TIPO B: Inicio ms insidiosos que aquellos que estn infectados por neumococo y el curso
de la enfermedad se prolonga por varias semanas, a la exploracin: taquipnea, dificultad para respirar, matidez a la o percusin,
estertores alveolares y bronquiales. Los nios pequeos pueden presentar derrame pleural. El diagnostico se establece por la historia
clnica, los sntomas y los hallazgos de la exploracin. Pueden encontrarse imgenes segmentarias, compromiso lobular simple o
mltiple, derrame pleural o neumatoceles. Diagnstico de certeza se establece con aislamiento del organismo de la sangre, lquido
pleural o aspirado pulmonar. COMPLICACIONES: Empiema y bacteriemia para la mayora de los agentes etiolgicos, artritis sptica en
casos de infeccin por Hib y Streptococo del grupo A, pericarditis, meningitis, osteomielitis, abscesos de tejidos blandos,
pioneumotrax y neumatoceles en las infecciones por S. aureus y celulitis, meningitis y artritis supurada en los casos de infeccin por
Hib. TRATAMIENTO ESPECIFICO: Usualmente en caso de neumonas por virus se requieren de medidas de soporte mnimas como la
administracin de lquidos IV, oxgeno y en los casos graves ventilacin mecnica, los nicos agentes especficos de que se dispone son
la amantadina y la ribavirina en aerosol, las cuales son eficaces contra influenza A y ribavirina para Virus sisitial respiratorio, se
recomienda solo en nios con enfermedad grave, que tienen cardiopata congnita, displasia broncopulmonar, enfermedad pulmonar
crnica o que reciben tratamiento inmunosupresor. La neumonas causadas por neumococo son tratables con penicilina IV, el frmaco
de eleccin es la penicilina G (100000U/Kg/24 hrs). Debe considerarse el empleo de una cefalosporina de tercera generacin
(cefotaxima 150mg/kg/24h, o ceftriaxona 75 mg/kg/24h), en el caso de que el S. peneumoniae sea resistente a la penicilina pero
MANUAL DE TRABAJO DEL CURSO ENARM CMN SIGLO XXI
CURSO ENARM CMN SIGLO XXI TEL: 36246001 Pharmed Solutions Institute PGINA 439

sensible a cefalosporinas. La vancomicina (40mg/kg/24h) se debera emplear si el aislamiento es resistente a las anteriores.
Aproximadamente el 20-30% de los neumococos son resistentes a los macrlidos, En los casos de neumona por Streptococo del grupo
A deben ser tratados con penicilina G (100 000 U/kg/24h). Se emplea penicilina parenteral de manera inicial y debe completarse un
tratamiento de hasta 2-3 semanas con penicilina por la via bucal. Si se presenta empiema debe realizarse una toracocentesis con
propsitos diagnsticos y para evacuar lquido. Para Staphylococus aureus dicloxacilina 100-200mg/kg/24h). En los casos de derrame
pleural aunque sea mnimo o de pioneumotrax, se recomienda la instalacin de un drenaje tracico. Para H. influenzae de tipo b debe
incluir ceftriaxona (75mg/kg/24h) o cefotaxima 150mg/kg/24h). Los derrames y la pioartrosis requieren de drenaje. TRATAMIENTO
EMPIRICO: Se hace en funcin de la sospecha del agente etiolgico. En nios de 2-3 aos con cuadro clnico y radiolgico de neumona
y con buen estado general se puede tratar por VO con dosis altas de amoxicilina (80-90mg/kg/da) cada 8 h, asociada o no a ac.
Clavulanico. Si el estado general est afectado o vomita es preferible ingresarlo y tratarlo por va IV con cefotaxima (100-
150mg/kg/da), ceftriaxona (50-100mg/kg/da) o amoxicilina/ ac. Clavulanico (100mg kg/da), durante mnimo 48-72 hrs si hay buena
evolucin seguir con amoxicilina oral por 7-10 das. En latantes menores de 3 meses la mejor opcin teraputica es la cefotaxima sola o
asociada a ampicilina si se considera que listeria es una posible causa, siempre por via IV. En el nio mayor de 3 aos con una neumona
tpica, si ingresa por afectacin del estado general, compromiso repiratorio o intolerancia digestiva, se tratara del mismo modo que si
tuviera menos de 3 aos, pero con un especial consideracin al empleo de penicilina G (100 000mg/kg/24h) IV si el cuadro clnico no es
grave. Los macrlidos son el tratamiento de eleccin en los nios con neumonia por Mycoplasma pneumoniae y Chlamydia
pneumoniae, VO 7-10 dias o 3 dias si se utiliza azitromicina.

CASO CLINICO
Un nio de 5 aos de edad se present con fiebre y tos no productiva durante 3 das. El examen fsico revel mltiples ndulos
linfticos cervicales. Fue tratado en su centro de salud durante 7 dias con medidas generales, regresando a su escuela sin embargo a los
5 dias despus inicia con tos productiva, dificultad respiratoria, cansancio, malestar generalizado, aleteo nasal y fiebre de 39.2 grados.
Examen torcico se apresiaron estertores bronquiales y broncoalveolares de predominio en la zona inferior. La saturacin de oxgeno
de 92% en aire ambiente. La radiografa de trax (RXT) al ingreso mostr consolidacin del lbulo inferior derecho. Leucositos de
13,500. Refiere el familiar que el esquema de vacunacin se encuentra al corriente. Vive en zona rural y es el segundo hijo de 5.

PREGUNTA
Cual de los siguientes estudios solicita para tomar una decisin diagnostica.

RESPUESTA
a.- Proteina C reactiva.
b.- Biometria hemtica.
c.- Cultivo de secresiones.
d.- Hemocultivo.

PREGUNTA
Cual es el criterio para tomar la radiografia de torax como factor desicivo para tratamiento.

RESPUESTA
a.- Paciente ambulatorio.
b.- Sospecha de neumona.
c.- Fiebre alta con sitomas respiratorios.
d.- Presencia de estertores sin taquipnea.

PREGUNTA
Considerando el cuadro clnico cual es el agente causal ms probable de este caso.

RESPUESTA
a.- Estreptococcus pneumoniae.
b.- Mycoplasma pneumoniae.
c.- Haemophilus influenza.
d.- Clamydia pneumoniae.

PREGUNTA
Cual es la conducta terapeutica mas apropiada.

RESPUESTA
a.- Amoxicilina 80-90 mg/kg/dia.
b.- Azitromicina 10 mg/kg dia.
c.- Eritromicina 50mg/Kg/dia.
d.- Penicilina procainica 400,000 UI cada 24 hrs IM.

PREGUNTA
Cual de los siguientes criterios es mas importante para el envio a segundo nivel.

RESPUESTA
MANUAL DE TRABAJO DEL CURSO ENARM CMN SIGLO XXI
CURSO ENARM CMN SIGLO XXI TEL: 36246001 Pharmed Solutions Institute PGINA 440

a.- Presencia de apneas.
b.- Dificultad respiratoria moderada.
c.- Falta de seguridad en casa.
d.- Deshidratacion.

RINITIS ALERGICA (RA). CIENCIAS BASICAS: Es una enfermedad crnica definida como una hipersensibilidad de la mucosa nasal a
sustancias extraas mediada por IgE. Los sujetos que presentan RA requieren de exponerse a bajas dosis de alrgenos durante varios
aos para desarrollar los sntomas. SALUD PUBLICA: Tienen una prevalencia entre 15-20%. La rinitis estacional raramente se presenta
en nios menores de 5 aos y la mayora de los pacientes con la enfermedad presentan los sntomas antes de los 20 aos. En la Ciudad
de Mxico, 14% de los nios cursan con rinitis alrgica. Hasta 78% de los pacientes con asma tienen rinitis alrgica. PATOGENIA:
Factores de riesgo; ablactacin temprana, deprivacin de alimentacin al seno materno, contaminacin ambiental, tabaquismo
materno durante el 1 ao de vida, antecedente de alergia en padres, historia familiar de rinitis alrgica, higiene excesiva y niveles
elevados de IgE a los 9 meses y a los 6 aos, pacientes asmticos. Existen aeroalergenos ms frecuentemente responsables de
ocasionar rinitis intermitente (plenes de rboles, gramneas y malezas, como as tambin esporas de hongos del exterior), pero la
persistencia de estos antgenos vara segn las estaciones del ao y la
regin geogrfica analizada, pudiendo un alrgeno estacional generar una
rinitis persistente. Los principales antgenos involucrados en la rinitis
persistente son los caros del polvo de habitacin. La liberacin de
mediadores qumicos (histamina, leuicotrienos entre otros), originados en
la activacin del mastocito, sensibilizado con IgE especifica de alrgeno, es
la responsable de los sntomas del paciente. La enfermedad alrgica
presenta dos etapas: a) sensibilizacin (predisposicin gentica), y b) la
presencia de sntomas (interaccin husped-medio ambiente). En la
primera etapa, debe existir un husped predispuesto a inducir una
respuesta inmunitaria a los alrgenos (Th2), susceptibilidad dependiente
de que se genere un arreglo gentico para producir IgE, capaz de
sensibilizar a la clula mastocitaria. En la segunda etapa, el contacto de
esta clula sensibilizada con el alrgeno produce la activacin celular que
desencadena el proceso inflamatorio en dos fases: inmediata (dependiente
de mediadores qumicos e IL) a los pocos minutos del contacto y tarda,
dependiente del infiltrado celular (eosinfilos, neutrfilos, mastocitos).
Esta activacin del sistema por interacciones vasculares y neurognicas genera los sntomas. Los principales mediadores qumicos con
un papel central en el desencadenamiento de la sintomatologa son: Histamina: es el principal mediador en la fase inmediata de la
reaccin alrgica posterior a la provocacin antignica. Se almacena en los grnulos del basfilo y del mastocito. La histamina acta
sobre los receptores H1 de varias clulas y causa los principales sntomas de rinitis. Leucotrienos: son formados de novo desde el ac.
araquidnico por va de la lipooxigenasa, liberados principalmente por el mastocito en la fase temprana y por eosinfilos y neutrfilos
en la fase tarda. Los leucotrienos producen bloqueo e incremento de la secrecin, pero no estornudos. Citoquinas: son liberadas por
los linfocitos T durante la reaccin de fase tarda y por el mastocito;
resultan importantes para mantener la inflamacin crnica.
CLASIFICACION: La OMS recomienda utilizar la establecida por ARIA
(Allergic Rhinitis and Its Impact on Asthma), de acuerdo a duracin de los
sntomas: intermitente (<4 das a la semana y <4 semanas al ao) o
persistente (>4 das a la semana y >4 semanas al ao). Rinitis intermitente
comprende aproximadamente al 20% de los casos de rinitis alrgica y la
rinitis persistente afecta al 80% restante. DIAGNOSTICO: Para un adecuado
diagnstico es importante realizar una cuidadosa historia clnica, que
permitir caracterizar la sintomatologa para clasificar la rinitis. En el
interrogatorio se debe poner nfasis en el anlisis de los sntomas: 1.
Obstruccin nasal: respiracin bucal crnica, falta de aire, hiposmia-
anosmia, voz nasal, babeo, ronquido, disfuncin tubariataponamiento
otico. 2. Rinorrea: ruidos nasales, tos farngea, nausea, dilatacin
abdominal por aerofagia. 3. Prurito palatino y nasofarngeo: frotamiento,
cloqueo, grgaras. 4. Estornudos. 5. Otros: epistaxis, fatiga, irritabilidad,
prurito ocular, hiremia conjuntival, edema palpebral. Examen fsico: La observacin directa permite detectar signos caractersticos,
denominados: facies alrgica, cianosis infraorbitaria (ojeras), pliegue palpebral supernumerario (signo de Denie Morgan), pliegue
transversal en la nariz (producido por el prurito y la limpieza constante). Esta facies puede estar asociada a signos de respiracin bucal,
como consecuencia de la obstruccin nasal. Tambin son frecuentes el edema y la hiperemia conjuntival. En el diagnstico, es
indispensable el examen de la cavidad nasal por rinoscopia anterior; se debe realizar con especial atencin a la estructura sea del
septum nasal, en busca de desviaciones o deformidades que impidan el normal flujo de aire. La mucosa puede presentarse inflamada,
plida, gris-azulada, con secrecin cristalina. Los cornetes edematizados pueden obstruir, de manera total o parcial, las fosas nasales.
Se analizarn las principales pruebas que permiten establecer un diagnstico etiopatognico adecuado: Recuento de eosinfilos en
sangre perifrica. Citologa nasal. IgE total y especfica. Pruebas cutneas. Estudios complementarios especiales como: Rinofibroscopia,
el mejor modo de examinar el interior de las fosas nasales es con la ayuda de un espculo nasal y luz frontal o con otoscopio. Radiologa
convencional: La frecuente asociacin entre la rinitis y las alteraciones estructurales de la va area superior, asociadas u originadas en
el proceso inflamatorio, pueden necesitar del estudio por imgenes. TRATAMIENTO: No farmacolgico: Educacin, medidas de control
ambiental. Evitar alrgenos. Farmacolgico: ver cuadros anexos. En los cuadros intermitentes con grado de gravedad leve, los frmacos
MANUAL DE TRABAJO DEL CURSO ENARM CMN SIGLO XXI
CURSO ENARM CMN SIGLO XXI TEL: 36246001 Pharmed Solutions Institute PGINA 441

de eleccin son: antihistamnicos orales tpicos, solos o asociados a descongestivos. En las rinitis intermitentes de intensidad
moderada-grave y en las persistentes leves, los frmacos de eleccin son: corticoides inhalatorios, antihistamnicos orales y tpicos, o
descongestivos y cromonas.




CASO CLINICO
Nio de 7 aos de edad con antecedentes de resfriados frecuentes y atopias, se le diagnostic asma a los 5 aos. No hay exposicin al
humo asi como hacinamiento, sin embargo la casa esta alfombrada, conservando que en invierno y verano se agudizan sus cuadros
caracterizados por escurrimiento nasal de secresion hialina, estornudos repetidos de predominio matutino, a la exploracin fsica se
observar rasgado nasal, e hiperemia conjuntival y alineacin nasal.

PREGUNTA
Cual de los siguientes estudios solicitaria en el paciente para establecer su diagnostico.

RESPUESTA
a.- Citologia nasal.
b.- Endoscopia nasal.
c.- Biopsia nasal.
d.- Pruebas cutneas.

PREGUNTA
Cual de las siguientes opciones teraputicas indica.

RESPUESTA
a.- Motelucask
b.- Budesonina.
c.- Loratadina.
d.- Cromoglicato.

























MANUAL DE TRABAJO DEL CURSO ENARM CMN SIGLO XXI
CURSO ENARM CMN SIGLO XXI TEL: 36246001 Pharmed Solutions Institute PGINA 442

ASMA. CIENCIAS BASICAS: Es una enfermedad crnica inflamatoria de las vas respiratorias (participan clulas cebadas, eosinfilos,
linfocitos T, neutrfilos y clulas epiteliales), que se caracteriza por obstruccin reversible o parcialmente reversible de stas, adems
de hiperrespuesta a diferentes estmulos y se caracteriza clnicamente por: tos, disnea y sibilancias. Es particularmente importante la
reversibilidad de la obstruccin bronquial, completa parcial, ya sea en forma espontnea o en respuesta a tratamiento. SALUD
PUBLICA: Primer causa de ausentismo escolar, tiene un gran impacto en la calidad de vida del que la sufre, que produce importantes
alteraciones en economa y la dinmica familiar. Las nicas encuestas realizadas en Mxico de l948 a l991 que aparecen en la literatura
reportan una prevalencia de 1.2 a 12.5%. La atopa; trmino que define a algunas enfermedades con niveles elevados de IgE asociados
a una predisposicin gentica definida, constituye el factor epidemiolgico ms consistente para el desarrollo de asma en la infancia. El
asma es una causa rara de muerte infantil en nios y adolescentes. PATOGENIA: Factores de riesgo: 1. Alrgenos (caros, mascotas,
hongos, cucarachas), la exposicin temprana a ellos tiene una alta correlacin con asma. 2. Virus desencadenan 90%. 3. Tabaquismo de
los padres es otro factor que se ha comprobado en meta-anlisis que incrementa el riesgo de padecer asma. 3. Contaminacin, ozono,
monxido de carbono, bixido de nitrgeno, partculas suspendidas. 4. Sustancias qumicas. 5. Ejercicio, su importancia radica en que
nos indica indirectamente la presencia de inflamacin bronquial a pesar de que el nio no tenga sntomas en reposo. 6. Fro. 7.
Emociones. 8. Medicamentos, como AINES. La remodelacin de la pared de la va respiratoria explica la importancia de la inflamacin
crnica recurrente en el asma. Los factores desencadenantes los alrgenos inhalados inducen la activacin de las clulas cebadas y
macrfagos, con la consecuente liberacin de varios mediadores pro-inflamatorios, incluyendo leucotrienos, factores quimiotcticos y
citocinas. Los antgenos procesados y presentados por los macrfagos a los linfocitos Th0, bajo la influencia un patrn adecuado de
citocinas, estimula la diferenciacin a un patrn de citocinas Th2 lo que a su vez estimulan la liberacin de mayores cantidades de IL- 4
e IL- 5, los cuales a su vez causan la sntesis de IgE por los linfocitos B y eosinofilia, respectivamente. Las citocinas derivadas de
macrfagos tales como IL -1, TNF- alfa, INF- gama, activan las clulas endoteliales, aumentando la expresin de las molculas de
adhesin tal como ICAM 1 y VCAM 1. Ms aun la IL - 4 derivada de clulas T selectivamente aumenta la expresin de VCAM 1. Esto
permite la salida de leucocitos de la vasculatura a la mucosa de las vas areas. Las consecuencias de este infiltrado de clulas
inflamatorias incluyen la esfacelacin del epitelio respiratorio, anormalidades en el control autonmico del tono de las vas areas,
cambios en la funcin mucociliar y aumento de las respuestas bronquiales. Esta reaccin inflamatoria autoperpetuable de esas clulas
efectoras (eosinfilos y clulas cebadas) es capaz de sintetizar citocinas que posteriormente promueven un incremento del proceso
inflamatorio. En resumen: Contraccin muscular bronquial (broncoespasmo o broncoconstriccin), edema e infiltracin celular de la
mucosa bronquial (inflamacin) e incremento de la secrecin bronquial. DIAGNOSTICO: El diagnstico se basa en la presencia de signos
objetivos de obstruccin bronquial, bsicamente por medio de la exploracin fsica (signos de dificultad respiratoria, sibilancias,
espiracin prolongada, hipoventilacin, opresin torcica, etc.). Estos signos obstructivos deben de ser recurrentes, esto es,
presentarse en forma de exacerbaciones episdicas (crisis), aunque en grados ms severos los sntomas obstructivos pueden ser
persistentes, y aun as, presentar episodios de agravamiento. Otra caracterstica bsica del diagnstico es que la obstruccin bronquial
es reversible, o al menos parcialmente reversible, a veces en forma espontnea o en base a tratamientos con broncodilatadores y/
anti-inflamatorios. Adems de lo anterior, en la gran mayora de los casos se puede documentar en la historia clnica el fenmeno de
hiper-reactividad bronquial, esto es, el inicio la exacerbacin de signos y sntomas de reaccin bronquial (tos, secrecin bronquial,
sibilancias, disnea) a una diversidad de estmulos fsicos, qumicos emocionales (ejercicio, olores penetrantes, humos, cambios de
temperatura humedad ambiental, etc.). Funcional: FEM (Flujo Espiratorio Mximo) Este valor proporciona una medicin simple y
cuantitativa de la obstruccin de las vas areas. Se realiza con un flujmetro porttil. El monitoreo de FEM es una herramienta clnica
de gran valor en el consultorio, en el hospital y hogar del paciente ya que permite valorar: La respuesta al tratamiento durante una
crisis aguda, respuesta al tratamiento crnico, detectar el deterioro asintomtico de la funcin respiratoria, antes de que se vuelva ms
grave. Identificar factores desencadenantes como por ejemplo el ejercicio. Espirometra: prueba de broncodilatador. FEV-1 (Volumen
Espiratorio Forzado en el primer segundo): Es la fraccin de volumen que se expulsa en el primer segundo del esfuerzo respiratorio
mximo, y normalmente representa aproximadamente el 80% de la CVF. Una disminucin >20% de este volumen, es indicativa de un
proceso obstructivo. Todos los pacientes con diagnstico presuntivo de asma deben ser valorados con una espirometra, al menos al
iniciar su tratamiento y, posteriormente, controles de seguimiento dependiendo de cada caso. Laboratorio: BH con eosinofilia, niveles
IgE, pruebas cutneas, citologa nasal, coproparasitoscpicos. Tele de trax en busca de complicaciones, radiologa de senos
paranasales, gasometra, pruebas de funcin pulmonar. CLASIFICACION DE ASMA: 1. ASMA INTERMITENTE: Sntomas menos de una
vez a la semana. Exacerbaciones breves. Sntomas nocturnos no ms de 2 veces al mes. FEV1 >80%, variabilidad del FEV1 <20%. 2.
ASMA LEVE PERSISTENTE: Sntomas >1 vez a la semana pero <1 vez al da. Exacerbacin afecta la actividad o sueo. Sntomas nocturnos
>2 veces al mes. FEV1/FEM >80 %,
variabilidad de FEV1 20-30%. 3. ASMA
MODERADA PERSISTENTE: Sntomas Diarios. Exacerbaciones que afectan la actividad o sueo. Sntomas nocturnos >1 vez a la semana.
Uso diario de agonistas B2 Inhalados de accin rapida. FEV1 o PEF de 60-80%, variabilidad del FEV1 >30%. 4. ASMA SEVERA
PERSISTENTE: Sntomas Diarios. Exacerbaciones frecuentes. Sntomas nocturnos frecuentes. Limitacin de actividad fsica. FEV1 <60%,
variabilidad FEV1 >30%. CLASIFICACION DE LAS CRISIS ASMATICAS: Leve: limita actividades fuertes (ejercicio), habla sin problemas, no
lo despierta. Flujometria: PEF > 80%, FC <100 lpm, sibilancias: moderadas. Moderada: limita actividades habituales, habla con dificultad.
Flujometra: FEM 60-80% FC 100-120 lpm, sibilancias: intensas. Severa: incapacidad de realizar actividades, dificultad respiratoria en
reposo, cianosis, habla palabras. Flujometra: < 60% FC >120 lpm, sibilancias: intensas. Paro inminente: conciencia: confundido, tiros:
respiracin paradojia, bradicardia, sibilancias: silencio. TRATAMIENTO: Medidas Generales: Control ambiental, evitar exposicin a
desencadenantes, vacuna anti influenza, evitar AINEs, reducir o eliminar la inflamacin. Las metas en el tratamiento del asma son: a.-
Control adecuado de los sntomas. b.- Prevenir las exacerbaciones. c.- Mantener FR dentro de la normalidad. d.- Mantener actividad
normal. e.- Evitar efectos adversos de medicamentos. f.- Prevenir el asma fatal. Los medicamentos utilizados son: ESTEROIDES
INHALADOS: Mejoran la funcin pulmonar, disminuyen la hiperreactividad bronquial, los sntomas y las exacerbaciones, mejoran la
calidad de vida. Las dosis recomendadas son: Beclometasona: nios 100-800 ug/da (leve-moderada:200, grave: > 500 dosis mxima:
1000). Budesonida: nios 100-400 ug/da (leve moderada:200, grave: > 400, dosis mximas: 800). Fluticasona: nios: 100-200 ug/da
(leve-moderada: 100, grave: > 200, dosis mximas: 500). Ciclesonida : Dosis 100 a 1200 mcg; Va de administracin : Inhalado.
Triamcinolona: Dosis 400 a 2000 mcg/da va de administracin: Inhalado Reacciones secundarias: Candidiasis oral, disfona, tos
Exacerbaciones NO Uno o ms/ ao Una vez/sem
MANUAL DE TRABAJO DEL CURSO ENARM CMN SIGLO XXI
CURSO ENARM CMN SIGLO XXI TEL: 36246001 Pharmed Solutions Institute PGINA 443

(irritacin de la va area), prevencin: uso de espaciadores. ESTEROIDES SISTEMICOS: Esta terapia se utiliza para el control de los
pacientes con asma severa persistente. Han demostrado disminucin de signos patolgicos de inflamacin y mejora de la
hiperreactividad bronquial. Prednisona: 0.5-1 mg/kg/da dosis de reduccin. Deflazacort : 5-10 mg/kg/da dosis de reduccin.
Metilprednisolona: Dosis 4-6 mg/kg cada 6-8 hrs. Hidrocortisona: Dosis 4-6 mg/ cada 6-8 hrs. Efectos secundarios: Osteoporosis, DM,
glaucoma, obesidad, estras cutneas, supresin del eje hipotalmico, metilxantinas: broncodilatador cuyo mecanismo de accin est
relacionado con la inhibicin de la fosfodiesterasa. Infusin: 5 mg/kg/dosis IV 0.7-0.9 mg/kg/hr. Reacciones secundarias: Nauseas y
vmito, cefalea, taquicardia, arritmias. B2 AGONISTAS: Relajan la musculatura lisa bronquial, mejoran el aclaramiento mucociliar,
disminuyen la permeabilidad vascular y la liberacin de mediadores inflamatorios. Los ms utilizados son: Salbutamol: 200mcg PRN.
Fenoterol: 50mcg PRN. Salmeterol: 25mcg cada 12 hrs. Terbutalina: 500mcg PRN. B2 Agonistas y Esteroides: Disminucin de los
sntomas y de crisis de asma nocturna, mejora la funcin pulmonar, reduce el nmero de exacerbaciones. MODIFICADOR DE
LEUCOTRIENOS: Cistenil leucotrieno, Montelukast, Pranlukast, Zafirlukast. ANTICOLINRGICOS: Bromuro de Ipratropio y Bromuro de
oxitropio, mecanismo de accin: Broncodilatador. Medicamentos controladores: Esteroides inhalados, agonistas de larga accin, Anti
leucotrienos, Xantinas. Medicamentos de rescate: agonistas de corta accin, anticolinrgicos. Crisis asmticas: Salbutamol + bromuro
de ipratropio, 2-4 disparos c/20min x hora. LEVE: 2-4 disparos cada 3-4 hrs, MODERADA: 6-10 disparos por 1 2 hrs, SEVERA: 10
disparos o ms. ESTADO ASMATICO: Forma letal del asma, caracterizado por ataques cada vez peor que no mejora con tratamiento
apropiado, que culmina con la insuficiencia de la vlvula pulmonar: empleo de msculos accesorios de la respiracin, pulso paradjico,
cianosis. Criterios hospitalizar. Terapia intensiva: Ataques frecuentes y repetidos, ltima vez asma grave que culmino en hospitalizacin.
Consumo diario y excesivo de broncodilatadores y corticoesteroides. Empleo de msculos accesorios de la respiracin. Pulso paradjico
que rebasa los 18 mmHg en adolescentes y 10 mmHg en nios. Cambios en la conciencia, cianosis. Neumotrax, neumomediastino.
FEV1 o PEFR (velocidad de flujo espiratorio mximo) menor de 20% de la cifra calculada. PaO2 < 60mmHg. PaCO2 > 40mmHg en
presencia de disnea y sibilancias. Acidosis metablica. Anormalidades electroencefalogrficas.

CASO CLINICO
Nia de 7 aos que acude a urgencias por cuadro de tos, dificultad respiratoria de dos semanas de evolucin y pico febril 38C. con
antecedentes patolgicos de obesidad, roncadora habitual y respiracin bucal. adenoidectomizada hace 2 aos por sospecha clnica de
SAHS sin control posterior. Peso 56kg (p>97), FC 135 lpm (p>95), TA 109/70, SatO2 82% con FiO2 21%. Facies anmica, implantacin
baja del pelo, estrabismo y obesidad mrbida. Aceptable estado general, normohidratada, palidez cutnea y retracciones subcostales
moderadas. Auscultacin cardiaca normal. Auscultacin respiratoria con hipoventilacin generalizada moderada, sibilantes espiratorios
finos y roncus dispersos.

PREGUNTA
Cul es la conducta farmacolgica mas adecuada.

RESPUESTA
a.- Salbutamol, bromuro de ipatropio inhalados y corticoides endovenosos.
b.- Salbutamol, corticoide, loratadina.
c.- Salbutamol, oxigeno, ambroxol y prednisona.
d.- Ambroxol, prednisona, Bromuro de ipatropio y oxigeno.

PREGUNTA
El paciente empeoro su cuadro clnico, con mayor dificultad respiratoria. Se decide enviar a segundo nivel. Al ingreso se realiz BH con
discreta leucocitosis, bioqumica normal y gasometra capilar: pH 7,31, PC02 52,3mmHg, PO254,7mmHg, HC0326mmol/l. Precisa FiO2
inicial de 1 para mantener SatO2>92%. Presenta evolucin lenta y respuesta parcial al tratamiento. Tras los primeros das de ingreso
persisten necesidades de oxigenoterapia elevadas (Fi20,5) durante el sueo. En controles de gasometra destaca hipercapnia
nocturna (PCO2 56,3mmHg). La evolucin trpida obliga a nuevo planteamiento diagnstico y realizacin de pruebas complementarias.

PREGUNTA
Cuales fueron los criterios ms importantes para el envio a segundo nivel.

RESPUESTA
a.- Duda diagnostica.
b.- Asma complicada.
c.- Asma no controlada con tratamiento.
d.- Antecedetes de exacerbaciones.

CASO CLINICO
Se trata de un adolescente de 13 aos y 90kg de peso con antecedentes de asma sin tratamiento de fondo. Acude por dificultad
respiratoria de varias horas de evolucin sin que hubiera recibido broncodilatadores previamente. Se constata una crisis asmtica grave
con taquipnea y retraccin costal marcadas, y se inicia tratamiento con nebulizaciones de salbutamol (15mg en la primera hora) y
bromuro de ipratropio (500 mg/h) asociados a metilprednisolona por va intravenosa (60mg). Transcurrida 1 h de tratamiento, se
realiza gasometra venosa que muestra acidosis metablica con hiperlactacidemia.

PREGUNTA
Cul es la conducta a seguir mas adecuada?

RESPUESTA
MANUAL DE TRABAJO DEL CURSO ENARM CMN SIGLO XXI
CURSO ENARM CMN SIGLO XXI TEL: 36246001 Pharmed Solutions Institute PGINA 444

a.- Mantener el tratamiento.
b.- Enviar a segundo nivel.
c.- Ingreso a la UCI.
d.- Realizar rx de torax.

PREGUNTA
Al ingreso hospitalario se mantiene tratamiento broncodilatador (salbutamol y bromuro de ipratropio) horario. Tras 4h de ingreso, se
mantiene el trabajo respiratorio, la taquipnea y la taquicardia, con saturacin de oxgeno del 92% y con fraccin inspiratoria de oxgeno
(FiO 2 ) del 35%. La radiografa de trax presenta hiperinsuflacin sin condensacin neumnica, atelectasia o escape areo. La
gasometra muestra empeoramiento de la acidosis lctica. Se ingresa al paciente en cuidados intensivos para el inicio de la ventilacin
no invasiva tipo presin positiva con dos niveles de presin con los parmetros iniciales (presin positiva inspiratoria de la va area de
12; presin positiva espiratoria de la va area de 6; FiO 2 del 40%), se mantiene el tratamiento broncodilatador y se expande la
volemia. Cul es pronostico esperado.

RESPUESTA
a.- Mejora ventilatoria.
b.- Descenso significativo de la FR.
c.- Normotenso, caliente y bien perfundido.
d.- Incremento de acido lctico.

FIBROSIS QUISTICA (FQ). CIENCIAS BASICAS: Es la enfermedad gentica ms letal, de carcter recesivo, multisistmico y progresivo,
afecta de preferencia a poblaciones caucsicas. La enfermedad se caracteriza por el espesamiento del mucus producido por las
glndulas exocrinas induciendo compromiso sino-pulmonar con dao pulmonar progresivo, insuficiencia pancretica y por lo tanto
sndrome de mala-absorcin, con consecuente desnutricin, esterilidad masculina por atrofia de los conductos deferentes y elevacin
de electrolitos en el sudor. El leo meconial siempre debe obligar a descartar FQ. Dependiendo de las mutaciones involucradas existe
una gran diversidad de formas clnicas. SALUD PUBLICA: Su incidencia vara de 1 por cada 3,000 a 1 por cada 8,000 nacidos vivos.
PATOGENIA: El gen de la FQ se encuentra localizado en el brazo largo del cromosoma 7, codifica una protena de 1480 aminocidos que
se ha llamada Protena Transportadora de Transmembrana (CFTR), esta se localiza en el polo apical de las clulas epiteliales. Es una
glicoprotena cuya funcin es actuar como canal de cloro, est constituida por dos regiones transmembrnicas (hidrofbicas) separadas
por una regin de unin al ATP. La primera mutacin encontrada fue la F508, localizada en el dominio ND1, presente en alrededor
del 75% de la poblacin caucsica. El resultado de todas las mutaciones detectadas que alteran la funcin de la CFTR es el mismo: la
imposibilidad de transportar cloruro. Cualquiera que sea la mutacin en el gen CFTR, cada paciente muestra las siguientes
anormalidades en distintos grados: A) Concentracin anormal de los iones en las secreciones de las glndulas serosas, manifestada por
aumento en la concentracin de cloro y sodio en el sudor. B) Incremento en la viscosidad de las secreciones de las glndulas secretoras
de moco, asociado con obstruccin y perdida secundaria de la funcin glandular. C) Aumento en la susceptibilidad a la colonizacin
endobronquial crnica por grupos especficos de bacterias (Staphylococcus aureus, H. influenzae, Pseudomona aeruginosa,
Burkholderia cepacia). Se han descrito 6 clases de mutaciones: las I a III son las ms comunes y generalemnrte se relacionan con
insuficiencia pancretica. En Mexico se han identificado 46 diferentes mutaciones que afectan a 77% de los cromosomas de la fibrosis
qustica. DIAGNOSTICO: La mayora de los casos de FQ se manifiesta por la triada clsica: a) enfermedad pulmonar obstructiva
progresiva crnica con infeccin agregada, b) insuficiencia pancretica exocrina, c) elevacin en las concentraciones de Cl y Na en el
sudor. Los RN afectados rara vez muestran sntomas respiratorios, aunque los menores de 6 meses de edad pueden experimentar
taquipnea, sibilancias, incremento del trabajo respiratorio, sobresdistension del trax y atelectasias. En 10-20% de los pacientes, el leo
meconial puede ser la primera manifestacin de la enfermedad. Se produce por la implantacin de meconio deshidratado en el leo
terminal, con un cuadro de obstruccin intestinal. Puede sospecharse antes del parto por ecografa u ocurrir al nacimiento con
distensin abdominal progresiva y vmitos biliosos y falta o retardo en la eliminacin de meconio en las primeras 24-48hrs de vida. La
radiografa de abdomen suele revelar asas intestinales dilatadas con reas de aire mezclado con meconio deshidratado. En la mayora
de los casos se encuentran una o ms expresiones clnicas de la enfermedad, que se confirma con la prueba de sudor. Casi todos los
pacientes exhiben enfermedad sinusopulmonar crnica y el 85-90% tiene insuficiencia pancretica exocrina. Es necesario hace notar
que debe considerarse como FQ clsica, aquella con niveles de test del sudor altos, compromiso pulmonar severo, progresivo, de
instalacin temprana, rinosinusitis, sndrome de malaabsorcin e infertilidad masculina. Se ha denominado como FQ no clsica o
enfermedad vinculada a FQ aquella con al menos una mutacin del gen de FQ, lo que le confiere funcin parcial a la protena CFTR,
generalmente no tienen sndrome de maladigestin por estar preservada la funcin exocrina y por lo tanto mejor estado nutricional,
con niveles de test del sudor discretamente altos o normales, enfermedad pulmonar de instalacin tarda y de menor severidad,
rinosinusitis y azoospermia, con clara mejor sobrevida. DIAGNOSTICO PREIMPLANTACIONAL: determinar caractersticas genticas del
embrin a partir de una sola clula obtenida mediante biopsia embrionaria, sin prejuicio para la viabilidad del mismo. Por consiguiente,
se requiere la obtencin de embriones mediante fecundacin in vitro. DIAGNOSTICO PRENATAL: Se analiza el ADN de clulas de
vellosidades corionicas o lquido amnitico. Se realiza si los padres son portadores o si existe un hermano con fibrosis qustica. ESTUDIO
NEONATAL: Se basa en el hecho de que las concentraciones sricas de tripsina de los enfermos con insuficiencia pancretica pueden
ser incluso 8 veces mayores a lo normal, se analizan: tripsina, tripsinogeno o complejo tripsina 1 anti-tripsina. El primer estudio se
realiza entre el 1-5 das de vida; si es positivo se repite entre la segunda y octava semanas si las concentraciones se mantienen
elevadas, se hace la prueba de Gibson y Cooke y estudio gentico. Test del sudor: la iontoforesis de pilocarpina por el mtodo de
Gibson y Cooke, continua siendo el gold standard que permite medir los valores de sodio y cloro en el sudor; en el tbulo de la glndula
sudorpara est bloqueado el reingreso de cloro a la clula, por lo cual tampoco lo hace el sodio, tenindose un sudor con mayor
cantidad de estos electrolitos. Se considera los siguientes valores: Positivo: >60 meq/lt, Limtrofe: 40 a 59 meq/lt, Negativo: <40 meq/lt.
I.- Test secretina pancreozimina: gold standard para medir funcin pancretica. ANATOMIA PATOLOGICA: Los hallazgos macroscpicos
son limitados y solo sugestivos de fibrosis qustica. Los hallazgos histopatolgicos son variables e incipientes en RN con afeccin
MANUAL DE TRABAJO DEL CURSO ENARM CMN SIGLO XXI
CURSO ENARM CMN SIGLO XXI TEL: 36246001 Pharmed Solutions Institute PGINA 445

principal a glndulas mucosas, y se vuelven ms evidentes al avanzar la enfermedad los ms representativos: A. Pncreas: 93% de los
casos tienen cambos histopatolgicos. Antes de las 40 semanas puede verse normal, el volumen acinar va disminuyendo y puede ser
25% menor a los cinco meses de vida. B. Tubo gastrointestinal: en RN, el leo meconial se ha asociado como hallazgo patolgico inicial
de fibrosis qustica (15 a 20%). La atresia intestinal (leon, yeyuno, o ambos) afecta a 15-25% de los casos de fibrosis qustica. La
peritonitis meconial ocurre en 33 a 50% de los pacientes con fibrosis qustica debido a la perforacin intestinal intrauterina. C. Hgado y
vas biliares: 60% de los pacientes sufren alteraciones clnicas y morfolgicas, como cirrosis biliar focal, manifestada por proliferacin de
conductos biliares dilatados con material eosinoflico intraluminal y reas de fibrosis irregular portal e infiltracin de linfocitos. D. Vas
respiratorias: las manifestaciones morfolgicas principales son: hiperplasia de glndulas submucosas bronquiales con moco espeso
intraluminal, bronquiectasias, atelectasias, neumonas, obstruccin bronquial por moco y clulas inflamatorias que expanden las vas
areas y que se extienden al parnquima pulmonar. TRATAMIENTO: Son cinco los pilares bsicos del tratamiento de esta enfermedad:
1) tratar la infeccin y la inflamacin, y reparar el aclaramiento mucociliar, 2) mantener un buen estado de nutricin, 3) tratar la
insuficiencia del pncreas exocrino, 4) iniciar fisioterapia respiratoria y 5) detectar y tratar de manera oportuna enfermedades
concomitantes (diabetes, hepatopata y osteopenia). El germen que ms frecuentemente (60%) se asla en el esputo de los enfermos de
fibrosis qustica es P. aeruginosa. Existe gran inters en los antibiticos aerosolizados. Los beneficios potenciales de administrar
antibiticos va aerosol incluyen: el depsito directo en el sitio endobronquial de la infeccin, la disminucin de la toxicidad, mejor
relacin costo-beneficio y mejor calidad de vida. En un estudio basado en evidencias de la Asociacin Americana de Fibrosis Qustica se
recomienda, para tratar las manifestaciones pulmonares, administrar alfa dornasa inhalada (Pulmozymes), que es una
desoxirribonucleasa recombinante que acta como mucolticos, degradando el ADN. El gasto de energa incrementado en el paciente
con fibrosis qustica se debe a la insuficiencia pancretica, la malabsorcin de nutrimentos y la inflamacin. La adecuada alimentacin
favorece la sntesis proteica. ltimamente se ha autorizado la administracin de Kalydeco (ivacaftor), el cual funciona como un
potenciador, ya que facilita que el canal transporte ms cloro. Staphylococcus aureus meticilina sensible: Cloxacilina: 100 a 200
mg/Kg/da (d), c/6h por 21 das, oral o endovenoso, segn la situacin del paciente, o Flucloxacilina: 50 a 75 mg/Kg/d, c/8h por 21 das,
oral. Staphylococcus aureus meticilina resistente: Vancomicina: 50 mg/Kg/d, c/6h por 21 das, IV, o Linezolid: 150 a 300 mg c/12h por
21 das, oral o IV. Est indicado en caso de alergia a Vancomicina, no disponibilidad de vas ev o tratamiento ambulatorio. Haemophilus
influenzae -lactamasa negativo: Amoxicilina: 90 mg/Kg/d, c/12h por 14 das, oral. Haemophilus influenzae -lactamasa positivo:
Amoxicilina-cido clavulnico: 90 mg/Kg/d, c/12h por 14 das, oral, o Cefotaxima: 100 mg/Kg/d, c/6h por 14 das, IV. Pseudomonas
aeruginosa: Ceftazidima: 200 a 250 mg/Kg/d, c/6h por 14 a 21 das, IV ms Amikacina: 20 a 30 mg/Kg/d ev, o Tobramicina: 10 mg/Kg/d
IV. Alternativa de segunda lnea: Ciprofloxacina: 30 mg/Kg/d, c/12h oral (mximo 300 mg c/8h IV en infecciones severas) por 14 a 21
das.

CASO CLINICO
Se presenta el caso de un paciente de tres aos de edad, raza blanca y sexo masculino; presenta, tos hmeda, vmitos flemosos,
antecedentes patolgicos personales de fibrosis qustica y perinatales de parto eutcico a trmino, con peso al nacer de 6,8 libras, y
apgar 7/9, por aspiracin de lquido meconial, no ctero, y cada del cordn umbilical a los tres das. Lactancia materna hasta los tres
meses y lactancia artificial actual con leche evaporada. Vacunacin actualizada y medio-ambiente social favorable. Present
antecedentes patolgicos personales de broncoaspiracin de meconio al nacer y un ingreso al mes y medio por atelectasia, adems de
un segundo ingreso a los siete meses de nacido por polipnea y no ganancia de peso, a pesar de alimentarse bien, as como deposiciones
pastosas con grasa segn refiri la mam, suda mucho y el sudor es salado.

PREGUNTA
Cual es la conducta a seguir para establecer el diagnostico secundario que presenta el paciente?

RESPUESTA
a.- TC abdominal.
b.- USG abdominal.
c.- IRM abdominal.
d.- LAPE.



















MANUAL DE TRABAJO DEL CURSO ENARM CMN SIGLO XXI
CURSO ENARM CMN SIGLO XXI TEL: 36246001 Pharmed Solutions Institute PGINA 446

INFECCION DE VIAS URINARIAS (CISTITIS, PIELONEFRITIS). CIENCIAS
BASICAS: El trmino infeccin de vas urinarias (IVUs) se aplica a una
amplia variedad de trastornos que afectan el aparato urinario, indica la
presencia de bacteriuria significativa con o sin sintomatologa general
de infeccin y/o sintomatologa uretrovesical: desde infecciones
asintomticas, hasta aquellas que ponen en peligro la vida del enfermo
como la pielonefritis. SALUD PUBLICA: La IVU tiene mayor gravedad en
los nios <1 ao de edad con frecuencia aprox 1%. Despus de esta
edad la prevalencia aproximada de 3% en nios y de 5 a 8% en nias.
La tasa reportada de recurrencia es de 12-30%, con mayor probabilidad
en menores de seis meses, en caso de reflujo vesicoureteral (RVU)
grave y en aquellos con gamagrafa renal anormal al momento de la
primera infeccin. Entre un 8 y 40% de los menores de seis aos con
IVU tienen RVU; otras anormalidades comunes incluyen hidronefrosis,
uropata obstructiva y doble sistema colector. De un 10 a 65% de los de
menores de dos aos presentarn cicatrices renales. Estas ltimas se asocian con el desarrollo de hipertensin y enfermedad renal
terminal. Se ha encontrado que entre 10 y 25 % de los enfermos con insuficiencia renal crnica, tienen como causa pielonefritis crnica.
CLASIFICACION: 1.- BACTERIURIA ASINTOMTICA: Pacientes que incidentalmente presentan bacteriuria sin los sntomas clsico de
IVU, para confirmar el diagnstico se requieren 2 cultivos, que muestren el mismo microorganismo con recuento de 10 UFC o ms por
milmetro de orina. 2.- INFECCIONES URINARIAS BAJAS: Infecciones que no afectan por el momento el parnquima renal, constituyen
el grupo de mayor frecuencia y requieren de un tratamiento adecuado que cure las molestias del paciente y evite su diseminacin a las
estructuras renales. Se subdividen en: Uretritis Aguda, Cistitis Aguda, Prostatitis aguda, la cistitis aguda es la de mayor importancia en el
paciente peditrico, esta es la afeccin de la vejiga que se manifiesta por disuria, urgencia miccional, dolor supra pbico, incontinencia
y orina ftida. La cistitis no produce fiebre, ni daos renales, la orina puede ser turbia incluso con hematuria. 3.- INFECCIONES
URINARIAS ALTAS: Corresponden a la pelvis renal, clices, parnquima renal. Su incidencia se incrementa en pacientes con factores
predisponentes como malformaciones obstructivas, RVU, estenosis de cuello vesical, y del meato urinario, etc. En otros casos es
causada por una diseminacin hematgena del germen. Se subdividen
en: Pielonefritis aguda; representa el tipo ms grave de IVU, no solo
produce ms morbilidad, sino tambin mayor potencial para causar
dao irreversible. En los nios de mayor edad con pielonefritis aguda
se manifiesta fiebre, dolor e hipersensibilidad en el flanco y se
relaciona con piuria y urocultivos positivos. Los resultados de
laboratorio casi siempre revelan leucocitosis, VSG aumentada. Los RN y
lactantes menores son los grupos de edad con mayor riesgo de
formacin de tejido cicatrizal renal posterior a pielonefritis.
Pielonefritis crnica; se caracteriza por alguna o todas las siguientes
manifestaciones dolor abdominal o en flanco, fiebre, mal estado
general, nauseas, vomito, ictericias en los RN, y en ocasiones diarrea.
PATOGENIA: Aproximadamente 95% de las IVUs son causadas por
enterobacterias. La E. coli es responsable del 80 al 90%. El porcentaje
restante puede ser ocasionado por Staphylococcus saprophyticus,
Enterococcus sp., Klebsiella sp., Enterobacter sp., Pseudomonas sp. y Proteus sp. De las 150 cepas de E. coli, diez de ellas son
responsables de la mayora de las IVU. Esta situacin est relacionada con la presencia de factores bacterianos virulentos, como la alfa
hemolisina (protena citoltica que lesiona la membrana celular), siderforos (protena quelante de hierro que prolonga la vida de la
bacteria), y polisacridos capsulares (que disminuyen la activacin del complemento). La presencia de fimbrias en las bacterias, que
favorecen su adherencia al urotelio, tambin es un factor de virulencia importante: El 91% de las cepas de E. coli que producen
pielonefritis tienen fimbrias. Las bacterias pueden acceder al tracto urinario a travs de cuatro vas: 1. Va ascendente, desde la uretra y
la vejiga a los riones, la ms comn. 2. Hematgena: En pacientes inmunocomprometidos en neonatos. 3. Linftica, desde el recto,
colon y linfticos periuterinos, difcil de comprobar. 4. Directa, a travs de fstulas rectovaginales a cualquier parte del tracto urinario.
Los factores del husped que estn implicados en la presencia de bacteriuria se pueden resumir en: 1. Edad: Los neonatos tienen mayor
predisposicin a IVU por la inmadurez de su sistema inmunolgico. Adems existe una elevada colonizacin periuretral en el primer ao
de vida. 2. Colonizacin fecal, periuretral y prepucial: El uso indiscriminado de antibiticos de cualquier tipo favorece la proliferacin de
cepas virulentas y multirresistentes. 3. Gnero: factores anatmicos inherentes a la mayor accesibilidad de la vejiga a los grmenes en
las nias que en los nios por tener la uretra ms corta. 4. Genticos: Con mayor frecuencia los nios con IVU recurrentes tienen en su
MANUAL DE TRABAJO DEL CURSO ENARM CMN SIGLO XXI
CURSO ENARM CMN SIGLO XXI TEL: 36246001 Pharmed Solutions Institute PGINA 447

epitelio urinario receptores, glucolpidos antgenos del grupo
sanguneo P, que facilitan la adhesin de las fimbrias o pili de E. coli. 5.
Anormalidades genitourinarias: Estas alteraciones deben ser
identificadas tempranamente, ya que si no son corregidas a tiempo
pueden llevar a secuelas como cicatrices renales, prdida de
parnquima renal, hipertensin arterial e insuficiencia renal crnica. Es
por esta razn que se recomienda el estudio imagenolgico en todos
los pacientes que se presenten con IVU. DIAGNOSTICO: Clnica; Los
sntomas y signos ms comunes en menores de cinco aos por los que acuden a urgencias con el primer episodio de IVU son: fiebre
80%, irritabilidad 52%, anorexia 49%, malestar 44%, vmito 42%, diarrea 21%. Los sntomas menos comunes (en menos de 20%):
disuria, orina ftida, dolor abdominal, frecuencia y hematuria. La presencia de fiebre >38C, bacteriuria y dolor lumbar sugiere
pielonefritis, mientras que la presencia de sntomas urinarios como disuria asociada a bacteriuria, pero no a sntomas sistmicos,
sugiere cistitis o IVU baja. En nios de dos a 12 aos de edad, en su
primer episodio de IVU, los sntomas ms frecuentes fueron disuria y
urgencia en el 82%, dolor abdominal 35%, enuresis 45%, fiebre 26%,
hematuria 20% y balanitis 20%. El ABC del diagnstico de la infeccin
urinaria se basa en: EGO, para reconocer la presencia de estearasa
leucocitaria, reduccin de nitratos a nitritos cuenta de clulas
inflamatorias (ms de 10 clulas) y presencia de bacterias. Esta prueba
tiene una sensibilidad de 75 a 90% y una especificidad de 70 a 82%.
Cultivo de orina. La limitante de este estudio es disponer de una
muestra adecuada para el proceso. Si la orina se obtiene de una bolsa
colectora, la sensibilidad y especificidad son muy bajas ya que el 80%
de las muestras se hallaron contaminadas. Si la orina se obtiene por
catter, la sensibilidad y especificidad son superiores a 70%; por puncin suprapbica la presencia de cualquier nmero de colonias
bacterianas permite asegurar el diagnstico El nmero de unidades formadoras de colonias (UFC) necesarias para establecer el
diagnstico de IVU est en funcin del tipo de muestra que se obtiene, ver cuadro 2 anexo. Imagenologa. El estudio estndar es la
urografa excretora con histograma miccional, aunque el ultrasonido lo ha ido desplazando gradualmente. Este ltimo es una prueba de
escrutinio para descartar malformaciones mayores. Sin embargo, la sensibilidad y especificidad para el diagnstico de RVU son bajas en
ambas. El histograma miccional debe realizarse cuando el nio este afebril o el cultivo urinario sea negativo. Los estudios con
radioistopos pueden ser de utilidad para evaluar la funcin e integridad renal en pacientes con pielonefritis. INFECCIONES
RECURRENTES: La frecuencia de recurrencias en el primer ao de vida es menor de 20% en nios y de menos de 30% en nias; en nios
mayores de un ao, es superior a 30%. En nias que han tenido ms de dos episodios previos, la recurrencia puede llegar a 75%.
TRATAMIENTO: Neonato. La infeccin urinaria a esta edad es un problema de extrema gravedad, con alto riesgo de sepsis,
complicaciones, secuelas y muerte. Su tratamiento debe realizarse en el hospital. La probabilidad de malformaciones de las vas
urinarias es muy alta. El tratamiento antomicrobiano para sepsis neonatal es el indicado. La asociacin de un betalactmico ms un
aminoglucsido permite una cobertura de ms del 90% de los microorganismos involucrados ver cuadro 3 y 4. El tratamiento debe ser
de 10 a 14 das. Cuando se pueda prescindir de la va endovenosa, se puede recurrir a la intramuscular o a la va oral cuando sea
posible. Cualquiera de las combinaciones es igualmente eficaz y segura; la diferencia es principalmente el costo. En neonatos con
hiperbilirrubinemia, no se recomienda la ceftriaxona. Los lactanetes menores de 3 meses con IVU deben ser tratados con los mismos
criterios. Otras edades. El tratamiento se debe guiar de acuerdo al sitio de la infeccin: INFECCIONES URINARIAS BAJAS:
CISTOURETRITIS; El tratamiento de preferencia debe ser por va oral. Pueden utiliozarse amoxicilina, TMP/SFX y nitrofurantoina, ver
cuadro 5. La diferencia en la respuesta clnica a la amoxicilina o a la amoxicilina/clavulanato no es significativa. Los perfiles de
sensibilidad de E. coli a TMP/SMX han mostrado un incremento progresivo en la resistencia. La duracin del tratamiento por va oral
debe ser de 5 a 7 das 2. Los tratamientos menores a 4 das fallan con mayor frecuencia. Tratamientos ms prolongados (mayor de 7
das) no tienen ventaja e incrementan el riesgo de efectos adversos. INFECCIONES DE VAS URINARIAS ALTAS (PIELONEFRITIS): Menos
de 20% de los pacientes requieren hospitalizacincin, ya que son de mayor edad, toleran la va oral y tienen menos manifestaciones
sistmicas. El tratamiento debe iniciarse por va parenteral por tres a cinco das; debe continuar por va oral, hasta completar diez a 14
das. El cambio de va parenteral a oral se basa en la desaparicin de la fiebre y la mejora del estado general. Los esquemas de
tratamiento incluyen monoterapia con cefalosporinas de tercera generacin, aminoglucsidos, fluoroquinolonas o la combinacin de
un beta lactmico y un aminoglucsido. La elevada frecuencia de resistencia de las enterobacterias a la amplicilina, TMP/SMX y a las
fluoroquinolonas debe tomarse en cuenta en cada regin geogrfica, antes de considerarlos como opciones de tratamiento para
pielonefritis, ver cuadro 6.

CASO CLINICO
Paciente de 14 aos masculino, sin antecedentes personales, ingresa por sndrome febril de 48 hs. de evolucin, acompaado de
intolerancia oral y dolor abdominal localizado en hipocondrio derecho y epigastrio, de tipo continuo, con mala respuesta al tratamiento
sintomtico. El paciente ingresa en regular estado general, impresiona enfermo, febril (39.5C), hemodinamicamente estable, buena
suficiencia cardiorrespiratorio, normohidratado y taquicardico. Mecnica ventilatoria conservada, buena entrada bilateral de aire, sin
ruidos agregados. Ruidos netos, silencios libres, bien perfundido, pulsos perifricos presentes y simtricos con relleno ungueal
conservado. El abdomen era blando depresible y doloroso a la palpacin profunda en epigastrio e hipocondrio derecho, con puo
percusin derecha positiva, sin defensa ni contractura muscular. Ruidos hidroaereos presentes. Resto del examen fsico sin
particularidades. Refiere diuresis y catarsis conservada. Laboratorio de ingreso: Leucocitosis moderada a predominio neutrofilico
Glbulos blancos: 18.600 (N: 87/ L: 5) Glbulos rojos: 4970000 Plaquetas: 162000, Funcin heptica normal ( TGO: 62 TGP:23), Funcin
renal normal (Urea: 42 Creatinina: 0.9) Glicemia: 140. VES: 25 mm / hs. Orina completa con sedimento: densidad 1030, pH 6, protenas
(++++), hemoglobina (+++), clulas (+), regular cantidad de leucocitos y piocitos. Abundantes hemates.
MANUAL DE TRABAJO DEL CURSO ENARM CMN SIGLO XXI
CURSO ENARM CMN SIGLO XXI TEL: 36246001 Pharmed Solutions Institute PGINA 448


PREGUNTA
Cual es el agente etiolgico mas probable?

RESPUESTA
a.- Klebsiella.
b.- Escherichia Coli.
c.- Proteus.
d.- Peptuscoccus.

GLOMERULONEFRITIS. CIENCIAS BASICAS: Es un proceso inflamatorio del glomrulo enfocado desde el punto de vista histolgico. La
glomerulonefritis aguda ms frecuente es la glomerulonefritis aguda posinfecciosa. CLASIFICACION: Glomerulonefritis Primarias:
Nefropata por IgA o IgM, Glomerulonefritis proliferativa mesangial (GNPM), Glomerulonefritis membranosa (GNM), Glomerulonefritis
extracapilar (GNEC). Secundarias: Posinfecciosa por bacterias; glomerulonefritis posestreptocccica (GNPE), Posbacteremia
estafiloccica, neumccica, sfilis. Virales (hepatitis B, influenza A y B, etc). Parasitarias (malaria, toxoplasmosisi, etc). Enfermedades
Multisistmicas: LES, Purpura de Henoch Scholein, Vasculitis necrosante, Sndrome hemoltico urmico/purpura trombocitopenica
trombotica (SHU/PPT), Granuloma de Wegener. PATOGENIA: Se debe a un proceso inflamatorio del glomrulo casi siempre de origen
inmunolgico, este proceso inflamatorio se presenta en el mayor de los casos por la formacin de complejos inmunes, cuyos antgenos
pueden ser exgenos como virus, parsitos, bacterias o sus productos, drogas y alimentos. OP bien pueden ser endgenos como la
propia membrana basal glomerular, productos celulares, ncleos, DNA, RNA, mitocondrias, lisosomas como sucede en LES y otras
enfermedades autoinmunes. Siendo los anticuerpos las inmunoglobulinas IgG, IgM, IgA. Constituyndose a si el complejo inmune tanto
intra como extraglomerular, este complejo tienen la facultad de activar el sistema del complemento, tanto por la va clsica como por la
va alterna, una vez activado el sistema induce el proceso inflamatorio por diferentes mecanismos, uno es por ataque directo a la
membrana basal por un sistema de protenas formadas por C5-C9, otro es por activacin del sistema de coagulacin y finalmente por la
induccin de la proliferacin de clulas intrnsecas (endoteliales, mesangiales, epiteliales y extrnsecas del glomrulo como son: el
sistema macrfago), monocitos y PMN, los cuales producen factores inflamatorios denominados de dao glomerular como IL-1,
leucotrienos, tromboxano A2, PG E2, radicales de oxgeno, TNF. Todos estos elementos en su conjunto inducen alteraciones en la
permeabilidad de la membrana basal glomerular y disminucin de la tasa de filtracin glomerular que clnicamente se manifiesta como
hematuria, proteinuria y falla renal en grado variable, se manifiesta con elevacin de los azoados en sangre y una disminucin en la
depuracin de creatinina, estas alteraciones provocan una serie de ajustes renales en los que hay desequilibrio glomerulotubular, en
donde existe una disminucin de Na a nivel del tbulo proximal, pero un incremento en la reabsorcin absoluta de Na y agua a nivel
distal lo que lleva a un balance positivo de estos elementos, estas dos situaciones son responsable de un incremento de volumen en el
espacio intravascular (disminuye la renina plasmtica y aldosterona), aumentando la presin hidrosttica intracapilar que aunado a una
disminucin de la presin coloidosmotica, por la proteinuria, inducen a un reajuste de las fuerzas de Starling y de esta manera en la
clnica se presenta el edema, aumento de gasto cardiaco (aumento de precarga) y la hipertensin arterial. TRATAMIENTO: Disminuir la
sobrecarga hdrica a travs de diurticos de asa del tipo furosemida dosis 3-5mg/kg/dosis, vigilando frecuentemente el gasto urinario y
balance de liquidos, si no hay resultado se debe emplear procedimiento dialtico. Para coadyuvar al tratamiento de la hipertensin se
podr usar vasodilatadores sanguneos del tipo de prazosina 1mg c/8-12hrs o hidralazina a 3mg/kg/da dividido en 3 dosis y
ocasionalmente IECAS. El edema y la hipertensin arterial inician su descenso los primeros das del tratamiento y el sndrome nefrtico
desaparece en trminos de 5-10 das, aunque los datos pueden existir hasta 4 semanas, de tal manera de que si la hematuria
macroscpica y la hipertensin arterial persisten despus de este periodo o bien existe un sndrome nefrtico asociado y sobre todo
incremento de la creatinina srica, son indicaciones precisas de biopsia renal. El tratamiento de la glomerulonefritis crnica secundaria
ser controlando la enfermedad primaria que le dio origen, por ejemplo dando inmunomodulacin en LES, evitando las reacciones
anafilcticas en purpura vascular. O dando quimioterapia cuando son secundarias a neoplasias. Para las glomerulopatias crnicas
primarias; inmunomodulacin, anticoagulacin, plasmaferesis, citotxicos, ninguno a demostrado resultado satisfactorio a largo plazo,
la mayora evoluciona a insuficiencia renal crnica, por lo que la conducta ser cuidar la reserva renal y evitar la progresin rpida de la
enfermedad renal terminal. PRONOSTICO: Bueno si se controlan las complicaciones en la fase aguda. La mortalidad es menor de 0.5% y
prcticamente se recuperan en forma espontnea. La recuperacin total puede durar hasta 2 aos, de ah que si la hematuria
microscpica dura ms de este periodo es indicacin de biopsia renal. Si la lesin de los glomrulos fue muy grave el pronstico es
sombro y esto depender del porcentaje de la formacin de medias lunas, de modo que si estas son ms de 70% de los glomrulos
observados, el paciente llegara a enfermedad renal terminal. GLOMERULONEFRITIS AGUDA POSESTREPTOCCICA (GNPE): Es la
infecciosa ms frecuente. Se presenta con mayor frecuencia en nios, generalmente entre 5 y 10 aos de edad, con ligero predominio
en el sexo masculino. En 85% de los casos se encuentra antecedente de infeccin farngea, 7 a 21 das antes de la aparicin de las
manifestaciones clnicas de nefropata; en otros pases, la infeccin previa que predomina es piodermitis. La infeccin estreptoccica se
asocia a fiebre reumtica o glomerulonefritis pero no ambas. El etreptococo nefritognico, presenta productos proteicos de la pared
bacteriana como son la protena M y T, de las cepas 1, 2, 4, 12, 18, 25, 49, 55, 57, y 60 que se asocian a nefritis. El dao
fundamentalmente se debe a la formacin de complejos inmunes donde el antgeno es un producto celular especifico (protena M), de
la bacteria o bien un componente propio del organismo que ha sido modificado por la infeccin y que ha sido reconocido como no
propio, cualquiera que sea este mecanismo este antgeno se une a anticuerpos como IgG, IgM o IgA formando complejos inmunes que
activan el sistema del complemento provocando dao tisular. Posterior a la infeccin estreptoccica, por lo general de las vas
respiratorias o de la piel, existe un periodo asintomtico que va de 15-21 das para posteriormente presentarse el sndrome nefrtico.
Se presenta con mayor frecuencia en la etapa escolar y predomina en varones 2:1. Cuando la infeccin es en la piel la probabilidad de
presentar nefritis es 5 veces mayor. Laboratorio: EGO; hematuria, proteinuria, eritrocitos dismrficos, leucocituria, cilindros hemticos
y hialinos. BH; anemia normocItica normocromica, leucocitosis, aumento VSG. Alteraciones bioqumicas; aumento de Cr srica,
hiponatremia dilucional, hiperpotasemia, acidosis metabilica, disminucin de depuracin de Cr. Bacteriologa; cultivos positivos para
estreptococo del grupo A en piel y nasofaringe. LESION GLOMERULAR MINIMA: Los glomrulos son normales a la microscopia de luz, la
MANUAL DE TRABAJO DEL CURSO ENARM CMN SIGLO XXI
CURSO ENARM CMN SIGLO XXI TEL: 36246001 Pharmed Solutions Institute PGINA 449

inmunofluorescencia es negativa y en la microscopia electrnica hay fusin de los podocitos. Clnicamente se expresa como sndrome
nefrtico puro corticosensible. GLOMERULONEFRITIS PROLIFERATIVA: En esta lesin encontramos proliferacin endocapilar que
significa un aumento de las clulas dentro del penacho vascular: clulas mesangiales, endoteliales y podocitos (proliferacin
centrifuga), a su vez esta lesin puede ser focal, cuando se encuentra en menos de la mitad de los glomrulos observados, o bien difusa
cuando la lesin esta en ms de la mitad de dichos glomrulos, las manifestaciones clnicas son graves: casi siempre sndrome nefrtico
y sndrome nefrtico con fallo renal. Hay que sealar que las lesiones difusas se pueden acompaar de lesiones extracapilares, es decir
proliferacin de estructuras fuera del penacho vascular, o clulas epiteliales de la capsula de Bowman (proliferacin centrpeta),
formando lo que se conoce como medias lunas, es importante reconocerlas ya que todas evolucionan a fibrosis y esclerosan el
glomrulo en donde se encuentran, condicionando falla renal irreversible, si hay ms de 70% de afeccin, puede llevar a enfermedad
renal terminal en un promedio de 3 meses. GLOMERULONEFRITIS MEMBRANOPROLIFERATIVA (GNMP): Existe una proliferacin
importante de clulas mesangiales, as mismo hay un engrosamiento y desdoblamiento de la membrana basal glomerular, dando un
aspecto de doble contorno o una imagen de doble riel. Clnica; se expresa como sndrome nefrtico y nefrtico con complemento en
sangre bajo, se presenta con mayor frecuencia en adolescentes y predomina en el gnero femenino. O bien presentarse como
hematuria, proteinuria o hipertensin en forma iaslada. Esta es una de las glomerulopatias crnicas ms frecuentes y que ocupa uno de
los primeros lugares como causa de insuficiencia renal crnica. GLOMERULONEFRITIS MEMBRANOSA (GNM): Poco frecuente en
pediatra, casi siempre es secundaria a enfermedades autoinmunes como LES o enfermedades granulomatosas. En las caractersticas
histolgicas hay un engrosamiento importante de la membrana basal glomerular, la cual protruye a travs de los podocitos dando una
imagen de espinas que el capilar asemeja una rueda dentada. No hay proliferacin de clulas mesangiales. La manifestacin clnica
es un sndrome nefrtico impuro corticoresistente, asociado a las caractersticas de la enfermedad principal, aunque hay GNM
idioptica.

CASO CLINICO
Paciente de 15 aos con historia de 1 semana de decaimiento progresivo, palidez, nuseas con vmitos ocasionales y cefalea vespertina
intensa. Dos das previos al ingreso se agreg edema periorbitario, mayor decaimiento y se constat bajo volumen urinario. Al
momento de la admisin al hospital, la paciente presentaba discreto edema facial y de extremidades inferiores, palidez de piel y
mucosas, presin arterial de 150/105 y frecuencia cardaca de 82 x'. Entre los exmenes de ingreso destacaban hematocrito 19,9%,
leucocitos 9500 x mm3 (53% neutrfilos), plaquetas 150.000 x mm3, velocidad de eritrosedimentacin 62 mm/h, protrombina 86%,
TTPK 11,7'', nitrgeno ureico 106 mg/dl, creatininemia 10,1 mg/dl, uricemia 6,9 mg/dl, calcio 7,4 mg/dl, fsforo 6,7 mg/dl, sodio 136,6
mEq/l, potasio 6,6 mEq/l, cloro 107 mEq/l. Examen de orina: proteinuria 2 g/l, hemates incontables por campo, cilindros hialinos,
creos y gruesos con inclusiones granulosas numerosos en la preparacin.

PREGUNTA
Cual es la conducta teraputica inmediata mas apropiada para el caso?

RESPUESTA
a.- Diurtico, antihipertensivo, hemodilisis aguda y biopsia renal.
b.- Diurtico, antihipertensivo, hemodilisis aguda y metilprednisolona en bolos de 1 g al da.
c.- Diurtico, antihipertensivo, hemodilisis aguda, metilprednisolona y biopsia.
d.- Diurtico, antihipertensivo, metilprednisolona en bolos de 1 g al da.

SINDROME NEFRITICO. CIENCIAS BASICAS: Es la expresin clnica de las glomerulonefritis sobre todo de las agudas, es decir, de un
proceso inflamatorio renal con asiento exclusivo o predominante en el glomrulo. En el desarrollo de dicho proceso, intervienen
fundamentalmente los mecanismos de la respuesta inmune, se caracteriza por edema, hematuria, oliguria e hipertensin arterial, en la
mayora de los casos hay hipocomplementemia. Las glomerulopatas agudas no se manifiestan necesariamente con el sndrome
nefrtico. En efecto, pueden tambin presentarse como sndrome nefrtico, sndrome urmico, hematuria recurrente y, ms an,
pueden cursar en forma totalmente asintomtica. El sndrome nefrtico es una asociacin de hallazgos clnicos y la expresin
<<glomerulonefritis>> indica una lesin anatmica. Debe enfatizarse que estos trminos no representan ninguna definicin
etiopatognica de la enfermedad. PATOGENIA: El sndrome nefrtico puede presentarse en ausencia de etiologa infecciosa; las causas
ms frecuentes son lupus eritematoso sistmico, prpura de Henoch-Schnlein, glomerulonefritis por IgA, crioglobulinemia esencial,
sndrome de Guillain-Barr y nefritis pos-radiacin. La incidencia de GLOMERULONEFRITIS AGUDA POSESTREPTOCCICA (descrita en el
tema de glomerulopatias), ha disminuido en varios pases desarrollados, pero an puede considerarse como la causa ms frecuente de
sndrome nefrtico agudo a cualquier edad. Prcticamente en todos los casos existen evidencias de patogenia inmunolgica por
formacin de complejos inmunes, tales como disminucin de los niveles de complemento hemoltico en suero, deteccin glomerular de
inmunoglobulinas y complemento de aspecto granular y periodo de latencia entre la infeccin y las manifestaciones clnicas de lesin
renal. La activacin del sistema de complemento es inducida por la formacin de complejos antgeno-anticuerpo. DIAGNOSTICO:
Clnica; edema, es uno de los signos cardinales, habitualmente se localiza en cara, es matutino, blando, plido (fascies nefrtica), en
nios pequeos suele encontrarse anasarca y en nios mayores o adolescentes es pedio o pretibial. Hipertensin arterial: relevante
porque en nios suele ser asintomtica, pero esta es la complicacin ms grave de la enfermedad ya que puede evolucionar a crisis
hipertensiva que aunado al edema agudo pulmonar, son las causas de muerte en la fase aguda. Hematuria: es un hallazgo universal en
el sndrome, es macroscpica hasta 60%, esta es obscura, indolora, total, sin cogulos. En el sedimento urinario hay cilindros hemticos
y eritrocitos dismorficos. Proteinuria: si hay suele ser <40mg/h/m2 SC, encontrndose en un 30-60% de los casos, es una proteinuria no
selectiva. Oliguria: la presencia de esta o de anuria es de mal pronstico, es el signo ms inconstante, cuando se presenta la
glomerulonefritis es muy grave, sobre todo cuando es secundaria a vasculitis, enfermedades autoinmunes, con lesiones de necrosis
cortical y obliteracin de los capilares glomerulares. TRATAMIENTO: Debe estar enfocado a las complicaciones sobre todo de la
hipertensin arterial y sus consecuencias como la encefalopata hipertensiva y la insuficiencia cardiaca, es decir, disminuir la sobrecarga
hdrica a travs de diurticos de asa del tipo furosemida dosis 3-5mg/kg/dosis, vigilando frecuentemente el gasto urinario y balance de
MANUAL DE TRABAJO DEL CURSO ENARM CMN SIGLO XXI
CURSO ENARM CMN SIGLO XXI TEL: 36246001 Pharmed Solutions Institute PGINA 450

lquidos, si no hay resultado se debe emplear procedimiento dialtico. Para coadyuvar al tratamiento de la hipertensin se podr usar
vasodilatadores sanguneos del tipo de prazosina 1mg c/8-12hrs o hidralazina a 3mg/kg/da dividido en 3 dosis y ocasionalmente IECAS.
El edema y la hipertensin arterial inician su descenso los primeros das del tratamiento y el sndrome nefrtico desaparece en trminos
de 5-10 das, aunque los datos pueden persistir hasta 4 semanas, de tal manera de que si la hematuria macroscpica y la hipertensin
arterial persisten despus de este periodo o bien existe un sndrome nefrtico asociado y sobre todo incremento de la creatinina srica,
son indicaciones precisas de biopsia renal. Los casos en los que se demuestra infeccin estreptoccica, deben recibir tratamiento de
erradicacin a base de penicilina. El esquema teraputico aconsejado es la aplicacin intramuscular de 800,000 unidades diarias de
penicilina procanica durante cuatro das, seguidas de una dosis de 1200,000 U de penicilina se puede emplear eritromicina oral por 10
das a dosis de 30 mg/kg/da. La erradicacin del estreptococo no influye sobre la lesin renal y slo tiene inters epidemiolgico para
evitar la diseminacin del germen. COMPLICACIONES: Sobrecarga hidrosalina, la insuficiencia cardiaca, encefalopata hipertensiva,
uremia e hipercaliemia infecciones que desempean un papel etiolgico en el desarrollo de la glomerulonefritis e infecciones
sobreaadidas, insuficiencia renal irreversible.

CASO CLINICO
Varn 3 aos, acude por: coluria, edema palpebral maana del ingreso, abdomen globuloso y edemas de miembros pelvicos, ganancia
de peso, refiere la madre un cuadro catarral hace una semana con T mx 37.6C, Peso: 19.4 kg Talla: 99 cm Sc: 0.74 m2, Fc: 94 Fr: 20
T: 36.5C TA: 140/100 (p>95), Edemas pretibiales que dejan fvea con ligero edema testicular. Hemo: Hemat: 3,68, Hb: 8.8, Hto: 26.2
%, VCM: 71, HCM: 24.4 pg Leucos: 19.680 (S 44%, L 46%, M7%), Plaq: 400.000. Bioq: Gluc: 94, Urea: 69, Creat: 0.68, PT: 6.07, Na: 131, K
5.36, PCR 17.7, Ca: 8.67, P: 5.13. Coag: Tp: 10.9, IQ: 100, Aptt: 26.7, Fibringeno: 420. Gasometra: pH: 7.42, pCO2: 32.5, HCO3: 21.2,
EB: - 3.2. Sedimento orina: Densidad 1020, pH 6.0, Prot >1000. 200-250 hemat/campo (10% dismrficos), 4-6 leucos/campo, escasos
cilindros hialino-granulosos.

PREGUNTA
Cual de los siguientes factores es la mas adecuada para identificar la evolucin del padecimiento de base?

RESPUESTA
a.- Diuresis
b.- Densidad de la orina.
c.- Fiebre.
d.- Tension arterial.

SINDROME NEFROTICO (SN). CIENCIAS BASICAS: Es la asociacin de proteinuria masiva, hipoalbuminemia y edema y a esta triada se
asocia habitualmente hiperlipidemia, lipiduria y alteracin de la coagulacin y ocasionalmente hay hipertensin arterial, hematuria o
falla renal. El sndrome nefrtico es una de las expresiones clnicas de las glomerulopatias, es decir no hay SN en las tubulopatias o en
las uropatas. SALUD PUBLICA: En EUA la incidencia en nios menores de 16 aos es de 2 por cada 100,000 nios. El 74% de los nios
(2-7 aos de edad) presenta sndrome nefrtico con lesin glomerular mnima. Predomina en varones 2:1. CLASIFICACION: SN primario
o idioptico: no se conoce una causa especfica y SN secundario: LES, amiloidosis, purpura de Henoch-Schonlein, vasculitis necrosante,
artritis reumatoide, DM, hipotiroidismo, enf. De Alport, sndrome nefrtico congnito, enf. De Fabri, neoplasias, infecciones bacterianas
(glomerulonefritis posestreptoccica, endocarditis bacteriana, nefritis por shunt, sfilis), virales, hepatitis, drogas. PATOGENIA: La
proteinuria masiva (>40mg/hora/m
2
SC): se puede presentar por un dao anatmico o estructural de la barrera glomerular (endotelio
vascular del capilar glomerular, membrana basal glomerular y podocito) entre la sangre y el espacio urinario, lo que se conoce como
proteinuria no selectiva, es decir que en la orina se encuentra albumina, transferrinas y globulinas, habitualmente el dao se hace por
complejos inmunes va la activacin del complemento y bien por anticuerpos contra la membrana basal. Pero tambin puede haber un
dao fisiolgico, o sea, que se pierda la capa electronegativa de esta barrera y esto sucede en las etapas tempranas de la nefropata
diabtica, hipertensin arterial y algunas alteraciones inmunolgicas, la proteinuria de esta caracterstica se le conoce como proteinuria
selectiva, es decir, albuminuria. El edema: es la acumulacin del lquido en espacio intersticial, por alteracin en el equilibrio de las
fuerzas de Starling. Hay tres mecanismos que contribuyen a su formacin: la hipoalbuminemia que contribuye a disminuir la presin
onctica del plasma, pero al mismo tiempo debe haber incremento en la presin hidrosttica, esto hace que el paciente tengan balance
positivo de Na y H2O, adems si el paciente tiene una lesin glomerular compleja, disminuye la tasa de filtracin glomerular que
contribuye a la retencin de lquidos. Hiperlipidemia: se conoce que las lipoprotenas de densidad baja y muy baja deben metabolizarse
en partculas ms pequeas, para ingresar a los tejidos, este paso metablico es realizado por una enzima lipoproteinlipasa, la cual es
activada por un cofactor conocido como ApoII, esta es una pequea protena que se pierde en la orina de los pacientes son sndrome
nefrtico, por otro lado a nivel heptico no se capta las lipoprotenas de baja densidad y del mismo modo que aumenta la sntesis de
albumina, aumenta la de colesterol. Trastorno de la coagulacin: El tromoboembolismo es una complicacin grave del sndrome
nefrtico, es raro en nios, as comn en adolescentes. DIAGNOSTICO: Si se sospecha SN, lo primero es confirmar la proteinuria, de
manera cuantitativa a travs de proteinuria horaria con coleccin de orina de 24 hrs, es masiva cuando el resultado es >40mg/h/m2 de
superficie corporal, por lo difcil de la coleccin y la no indicacin de colocacin de sonda vesical para la coleccin, se ha planteado la
realizacin de ndices urinarios proteinuria/creatininuria >2.5 =proteinuria masiva. En sangre encontramos hipoproteinemia <5g/dl,
hipoalbuminemia <2.5g/dl, hipercolesterolemia, hipertrigliceridemia (elevacin de lipoprotenas de baja y muy baja densidad). BH,
aumento de Hb, de Hto y tendencia a la plaquetosis. Las inmunoglobulinas se encuentran disminuidas. TRATAMIENTO: Enfocado en
tratamiento de complicaciones extrarrenales y de las glomerulopatas. Para edema: reposo relativo, restringir de dieta la ingesta de Na
y agua, solo se administrara la cantidad de lquidos por perdidas insensibles y egresos, se restringen las protenas o se da una dieta
normoproteica. El uso de diurticos solos o en combinacin con bloqueadores de la aldosterona o con el uso de albumina. Furosemida
1-3mg/kg por dosis c/8h. Espironolactona 1-3mg/kg/dia. El uso de albumina humana pobre en sal tiene indicaciones precisas las cuales
son: edema refractario al uso de diurticos, si la ascitis o anasarca intervienen con la mecnica respiratoria o si el edema representa
una deformidad corporal importante. La dosis 0.5-1g/kg/dosis, si hay elevacin de creatinina srica no se debe aplicar este
MANUAL DE TRABAJO DEL CURSO ENARM CMN SIGLO XXI
CURSO ENARM CMN SIGLO XXI TEL: 36246001 Pharmed Solutions Institute PGINA 451

medicamento. El uso de statinas y los anticoagulantes no estn indicados en pediatra. Los antibiticos sern de acuerdo a infecciones
especificas (de preferencia no aplicar IM). SINDROME NEFROTICO CON LESION GLOMERULAR MINIMA (LGM): Representa la situacin
ms frecuente en pediatra, la edad de presentacin e despus de 1 ao y antes de los 7 aos de edad, casi siempre presentan un
sndrome nefrtico puro, es decir, sin hematuria, sin hipertensin arterial (que son componentes del sndrome nefrtico), y la
caracterstica ms importante es que estos pacientes responden muy bien al uso de esteroides (corticosensible), se debe a una
alteracin del equilibrio de linfocitos CD4 y CD8 con predominio de estos ltimos los cuales liberan IL-2, la cual neutraliza la
electronegatividad de la membrana basal glomerular con la proteinuria subsecuente. Tratamiento: prednisona a 60mg/m2 de superficie
corporal, dosis nica por la maana por 4 semanas, posteriormente 40mg/m2/SC das alternos por 4 semanas.

CASO CLINICO
Se trata de preescolar que es traido a la consulta debido a la aparicin brusca de 48 horas de edema facial, asi como de miembros
plvicos edematosos, con escasa miccin urinaria. El nico antecedente de importancia es un proceso infeccioso en garganta que
refiere la madre, tratato con medidas caseras, los laboratorios de relevancia fueron los siguientes: Hipoalbuminemia (< 2,5 g/dl).
Proteinuria 42 mg/m2/hora y dislipidemia. A la EF se observa decaimiento, anorexia, vmitos y dolor abdominal. TA nornal.

PREGUNTA
Cual de los siguientes factores es la mas adecuada para identificar la evolucin del padecimiento de base?

RESPUESTA
a.- Diuresis
b.- Densidad de la orina.
c.- Fiebre.
d.- Tension arterial

GASTROENTERITIS (GEA). CIENCIAS BASICAS: La GEA suele considerarse como una inflamacin de la mucosa gstrica e intestinal
producida por un agente infeccioso o toxico, que se traduce clnicamente en un cuadro de diarrea de instauracin rpida,
generalmente es infecciosa y autolimitada. GEA aguda: Disminucin de la consistencia de las heces (blandas o lquidas) y/o un
incremento en la frecuencia de evacuacin (ms de 3 en 24 horas) con o sin fiebre o vmitos, de una duracin habitualmente menor a 7
das y nunca superior a 14 das. SALUD PUBLICA: Son la segunda causa de muerte infantil en el mundo, los ms vulnerables son los
menores de 5 aos. PATOGENIA: Etiologa; 1. Virus (rotavirus, virus de Norwalk, otros). 2. Bacterias (S. aureus, Bacilus cereus,
Clostridium perfirngens, difficile y botulinum, Salmonella, Shigella, Campylobacter, listerya monocytogenes, Vibrio cholerae, Yersinia,
Eschericgia coli). 3. Parasitos (Entamoeba hystolitica, Giardia lamblia, Isospora belli). 4. Malaabsorcion (intolerancia a la lactosa,
enfermedad celiaca, fibrosis qustica, intolerancia a protenas de la leche, y/o alimentos especficos). 5. Farmacos (antobioticos,
laxantes, quimioterapia, anticidos, diurticos). 6. Toxinas (hongos o setas, crstaceos, pescados o mariscos, conservadores de
alimentos, metales pesados). 6. Otros (mala tcnica alimenaria, condimentos, alimentacin inadecuada para edad, radioterapia). En
Mexico, los agentes que ocasionan hasta 70% de los cuadros diarreicos son: rotavirus, especies de Shigella, E. coli enterotoxignica, E.
coli enteropatgena y Campylobacter. La infeccin se adquiere por la va oral, a partir de un enfermo, de un portador asintomtico, o
de un reservorio animal; con transmisin de forma directa, a travs de alimentos contaminados o de vectores. Puede aparecer como un
caso espordico o en brotes, con mayor frecuencia durante el verano. Los cuadros espordicos son debidos a cualquiera de los agentes
citados anteriormente, pero los brotes suelen ser producidos por Salmonella o por toxinas estafiloccicas preformadas. Existen tres
mecanismos de produccin de las manifestaciones clnicas: .1. Sntesis de toxinas: Que alteran los procesos de manejo hidroelectroltico
celular a travs del AMPc, inhibiendo la absorcin de los iones sodio y cloro, y estimulando la secrecin de cloro y bicarbonato.2.
Invasin directa de la mucosa intestinal: Destruyen el borde en cepillo y las clulas adyacentes, provocando inflamacin local y
ulceracin.3. Mecanismo mixto o no preciso: Por posible adherencia directa o secrecin aumentada de moco. Actan as agentes como
Giardia lamblia y Escherichia coli enteropatgena. DIAGNOSTICO: El objetivo de la valoracin clnica bes la distincin entre los casos
leves, benignos y autolimitados. Debe considerarse segn el tipo de diarrea, perdidas hdricas alcanzan entre 10-200ml/kg; que supone
el 20% del peso corporal que puede constituir una situacin de urgencia. Los factores a identificar para una valoracin correcta son: 1.
Gravedad de la enfermedad. 2. Duracin de la diarrea. 3. Contexto epidemiolgico, en el que aparece el caso. 4. Situacin del husped
en cuanto a inmunidad y defensas. En primer lugar se debe delimitar lo que se considera como diarrea: de acuerdo con los criterios
establecidos, evacuaciones de 200g, casi siempre lquidos y aumento de la frecuencia. En una primera evaluacin sern elegidos como
potencialmente graves a los que presenten alguno de los siguientes signos: deshidratacin importante con sensacin de sed o
reduccin de diuresis; diarrea sanguinolenta; dolor abdominal o fiebre elevada. En la exploracin fsica se consideran como datos que
confirman la sospecha de severidad establecida en la anamnesis, los siguientes; obnubilacin o disminucin del nivel de consciencia,
sequedad de mucosas, hipotensin ortostatica, en la exploracin del abdomen, la presencia de resistencia, dolorimiento o signos de
peritonitis. Se consideran criterios de hospitalizacin; diarrea hemorrgica, signos de toxicidad sistmica (obnubilacin), fiebre alta,
deshidratacin severa, distensin abdominal. Por otra parte debe considerarse que algunas infecciones por Salmonella, Shigella,
Yersinia y E. coli enterohemorrgica, pueden complicarse con afectacin de otros rganos o sistemas: sndrome hemoltico urmico,
PTT, Sndrome de Reiter, pericarditis, glomerulonefritis. En cuanto a la duracin de la diarrea, la persistencia de la sintomatologa
durante 4-5 das es un rasgo de severidad, por asociarse a microorganismos ms virulentos. Pertenecen a grupos de alto riesgo los
menores de 5 aos, por la situacin inmunitaria y defensas del husped, para cuya atencin y proteccin se han elaborado normas y
programas en diversas partes del mundo, sobre todo en los pases en vas de desarrollo como Mxico. SIGNOS DE PELIGRO: No puede
beber o amamantarse, vomita todo, tienen convulsiones, esta letrgico o inconsciente, muestra ausencia o disminucin de los ruidos
intestinales. TRATAMIENTO: Factores de mal pronstico: 1. Menor de 2 meses. 2. Desnutricin moderada a grave. 3. Presencia de
inmunodeficiencia. 4. Muerte de un menor de 5 aos en la familia. 6. Madre analfabeta o menor de 17 aos. 7. Dificultad para el
traslado al mdico si se agrava el nio. La enfermedad es casi siempre autolimitada y la mortalidad se relaciona con las complicaciones,
en las que la deshidratacin es la causa de muerte en el 70% de los casos. El adecuado manejo de la deshidratacin con VSO evita 90%
MANUAL DE TRABAJO DEL CURSO ENARM CMN SIGLO XXI
CURSO ENARM CMN SIGLO XXI TEL: 36246001 Pharmed Solutions Institute PGINA 452

de estas muertes, por eso las acciones contra la diarrea incluyen este recurso, impulsando su uso en los casos. Par el tratamiento es
necesario evaluar el estado de hidratacin del paciente, por lo que se deber considerar las siguientes definiciones operacionales. CASO
SIN DESHIDRATACION: Es aquel que presentan menos de cuatro evacuaciones liquidas en 24h, ausencia de vmito, sin signos clnicos de
deshidratacin. CASO CON DESHIDRATACION: Es aquel que presenta dos o ms de las manifestaciones clnicas siguientes: Inquieto o
irritable, ojos hundidos, llanto sin lgrimas, boca y lengua seca, saliva espesa, respiracin rpida, sed amentada, bebe con avidez,
elasticidad de la piel, > a 2 seg., pulso rpido, llenado capilar de 3-5seg., fontanela anterior hundida (lactantes). CASO CON CHOQUE
HIPOVOLEMICO: Es aquel que presenta dos o ms de las manifestaciones siguientes: inconsciente o hipotnico, no puede beber, pulso
dbil o ausente, llenado capilar mayor de 5 seg. De acuerdo con la valoracin, el manejo de los pacientes que presentan algn grado de
deshidratacin se basa en 3 planes. PLAN A: Para pacientes con enfermedad diarreica sin deshidratacin con atencin en el hogar.
Continuar con alimentacin habitual, aumentar la ingestin de lquidos de uso cotidiano en el hogar, Sueros de VSO, en los menores de
1 ao, ofrecer media taza (75ml) y en lo mayores de un ao una taza (150ml) y administrarlo a cucharadas o mediante sorbos pequeos
despus de cada evacuacin. Capacitar a la madre para reconocer los signos de deshidratacin y datos de alarma por enfermedades
diarreicas (sed intensa, poca ingestin de lquidos y alimentos, numerosas heces liquidas, fiebre, vmito y sangre en las evacuaciones)
con el propsito de que acuda nuevamente a solicitar atencin medica en forma oportuna. PLAN B: Pasar pacientes con diarrea y
deshidratacin con atencin en la unidad de salud; administra VSO 100ml/kg, en dosis fraccionada cada 30min durante 4 hrs por va
oral. Si el paciente presenta vmito, esperar 10 minutos e intentar otra vez la hidratacin oral ms lentamente. Al mejorar el estado de
hidratacin, pasar a plan A. En caso contrario repetir el plan B en 4 hrs, de no existir mejora pasar al plan C. PLAN C: Pacientes con
choque hipovolmico por deshidratacin. Iniciar inmediatamente administracin de lquidos por va IV, con solucin de Hartmann, si no
se encuentra disponible, use sol. Salina al 0.9%, de acuerdo con el siguiente esquema: Primera hora 50ml/kg, segunda hora 25ml/kg,
tercera hora 25ml/kg. Evaluar al paciente continuamente. Si no mejora aumentar la velocidad de infusin. Cuando pueda beber
(usualmente 2-3hrs), administrar sobres de VSO, a dosis de 25ml/kg/h, mientras sigue lquidos IV. Al completar la dosis IV, evaluar al
paciente para seleccionar plan A o B, y retirar venoclisis, o repetir plan C. Si selecciona el plan A, observar durante 2 hrs para asegurarse
de que el responsable del paciente pueda mantenerlo hidratado con sobres de VSO, adems alimentarlo en su domicilio.
ANTIMICRONIANOS: No son tiles en el tratamiento de las enfermedades diarreicas en 90% de los casos, el manejo inadecuado de
antimicrobianos puede propiciar que la enfermedad se prolongue y ocasione resistencia bacteriana. Los antimicrobianos debe
prescribirse en pacientes con diarrea por: especias de Shigella, Vibrio cholerae, trofozoitos de Entamoeba histolytia o Giardia lamblia.
Cuando el paciente presenta fiebre, se deben utilizar medios fsicos para su control, hidratar al paciente, mantenerlo con ropa ligera y
de ser necesario, darle un bao con agua fra. Puede emplearse un antipirtico como acetaminofn a dosi de 30-40kg/dia en nios de 2
meses a 4 aos de edad, repartidos cada 4-6 hrs, sin pasar de 5 en 24 hrs. COMPLICACIONES: Choque hipovolmico, desequilibrio
hidroelectroltico, acido-base, ileo paralitico, diarrea persistente, insuficiencia renal, neumatosis intestinal, infarto intestinal,
perforacin intestinal, peritonitis. PREVENCION: Vigilancia de la cloracin del agua, procedimientos de desinfeccin (potabilizcion,
ebullicin, cloracin, yodacin), promocin de la lactancia materna (exclusiva los primeros 4-6 meses de edad), mejorara las prcticas
de ablactacin (a partir del 4to mes), promocin de la higiene en el hogar (lavado de manos, eliminacin adecuada de excretas, manejo
adecuado de heces en nios), vacunacin contra el sarampin, complemento con vitamina A.

CASO CLINICO
Un nio de 4 aos de edad, que inicia hace 2 dias con dolor difuso abdominal con 5 evacuaciones semiliquidad, niega fiebre, nauseas o
vomito. A la exploracin fsica se observa alerta y reactivo, signos vitales dentro de rango, incremento de movimientos intestinales y se
ausculta periltaltismo incrementado, las mucosas orales se observan hidratadas, su llanto contiene lgrimas, se encuentra irritable,
como antecedente se encuentra en tratamiento para desnutricin y la familia es de escasos recursos pero sus inmunizaciones se
encuentran al corriente.

PREGUNTA
Cual es la conducta a seguir mas adecuada?

RESPUESTA
a.- Solucion de rehidratacin oral.
b.- Antibioticos profilctico.
c.- Envio a casa con SRO.
d.- Proporcionar datos de alarma.

CASO CLINICO
Femenino de 3 aos de edad la cual inicia 3 dias previos a su ingreso, la madre refiere diarrea con moco y sangre, asi como fiebre alta
no cuantificada, esto se acompao de rinorrea y dificultad para consiliar el sueo, a la exploracin fsica se observa, decada con
aspecto deteriorado y respuesta alterada irritable, signos vitales taquicardico y taquipnea, se observa disminucin de la turgencia de la
piel, se ingresa a urgencias para tratamiento.

PREGUNTA
Cual es el volumen recomendado de SRO para este caso.

RESPUESTA
a.- SRO 30 ml/kg durante 4 hrs.
b.- SRO 50 ml/kg la 1era hora luego 30 ml/Kg.
c.- SRO mayor a 50 ml/kg durante 4 horas.
d.- Plan de hidratacion intravenoso.

MANUAL DE TRABAJO DEL CURSO ENARM CMN SIGLO XXI
CURSO ENARM CMN SIGLO XXI TEL: 36246001 Pharmed Solutions Institute PGINA 453

CASO CLINICO
Un nio de 9 aos de edad fue hospitalizado debido a dolor agudo abdominal y vmitos. Fue diagnosticado inicialmente con
apendicitis. El da 3 despus de la admisin, aunque estaba totalmente alerta y orientado, su examen se caracteriz por
empeoramiento presentando disfona, boca seca, ojos secos, estreimiento, abdomen timpnico, retencin urinaria, taquicardia,
taquipnea y disnea.

PREGUNTA
Cual es la conducta a seguir.

RESPUESTA
a.- SRO 30 ml/kg durante 4 hrs.
b.- SRO 50 ml/kg la 1era hora luego 30 ml/Kg.
c.- SRO mayor a 50 ml/kg durante 4 horas.
d.- Plan de hidratacion intravenoso.

PARASITOSIS. CIENCIAS BASICAS: Los parsitos
intestinales afectan con ms frecuencia a los nios
que a los adultos. Los viajes, la inmigracin y la
adopcin internacional han hecho que su incidencia
aumente en los ltimos aos. Aunque los sntomas
clnicos con frecuencia son inespecficos, algunos
datos de la anamnesis pueden orientarnos en la
prediccin del microorganismo causante.
DIAGNOSTICO: En general es preciso recoger tres
muestras de heces en das diferentes para tener una
mayor seguridad en la deteccin de los parsitos.
scaris lumbricoides (nematodo): Al ingerir los
huevos infectivos, las larvas inician un ciclo en el
organismo (circulacin, pulmones, trquea, faringe,
intestino). Los gusanos adultos de A. lumbricoides
viven en el intestino delgado. Diagnstico. Examen
parasitolgico de las heces donde se visualizan
huevos del parsito. Clnica, durante la migracin
pulmonar: neumonitis generalmente asintomtica en
ocasiones sndrome de Lffler (infiltrados
pulmonares transitorios, fiebre, disnea, tos,
sibilancias y eosinofilia). Fase intestinal:
generalmente asintomtica. Sntomas disppticos, diarrea. Raramente, obstruccin intestinal y vlvulo. Posibilidad de colecistitis,
colangitis, absceso heptico, apendicitis, diverticulitis o pancreatitis por migracin del parsito. Tratamiento. Mebendazol (100 mg/12
h/3 das o 500 mg a dosis nica). Alternativas: pamoato de pyrantel, 10 mg/kg (mximo 1 g), dosis nica; albendazol, 400 mg, dosis
nica (nios entre 2-5 aos: 200 mg); piperazina, 75 mg/kg (mximo 3,5 g en adultos o 2,5 g en nios entre 2-12 aos). La ivermectina
tambin es eficaz. (200 g/g dosis nica). Enterobius vermicularis (oxiuro, nematodo): El vermes adulto vive en el ciego y en las zonas
adyacentes. La hembra migra hacia la zona anal para hacer la puesta. Diagnstico. El examen parasitolgico de heces slo es positivo en
un 10-15% de los casos en los que se identifican los huevos del parasito o gusano adultos. Mejores resultados tiene el examen de cintas
adherentes transparentes despus de su aplicacin en los mrgenes del ano. En caso de exmenes repetidamente negativos y alta
sospecha de la infeccin, realizar tacto rectal para la obtencin de muestra fecal. Clnica: Asintomtico, prurito anal y perineal de
predominio nocturno. En casos excepcionales, colitis, dolor abdominal y granulomas peritoneales. Puede provocar vulvovaginitis en
mujeres jvenes. Epidemiologia y profilaxis: Cosmopolita, transmisin mediante ropa y sabanas contaminadas. Despus de ser
ingeridos, los huevos de E. vermicularis liberan las larvas en el duodeno y migran hacia el leo. Medidas higinicas bsicas: lavarse las
manos despus de la defecacin. Se recomienda dar el tratamiento a toda la familia para interrumpir la transmisin. Strongyloides
stercoralis (nematodo): Los gusanos adultos de S. stercoralis viven en el intestino delgado. Tiene capacidad de autoinfestacin, base de
la larga persistencia en el organismo infectado (dcadas). Diagnstico. Examen parasitolgico de las heces. Cultivo para estrongiloides.
Produce hipereosinofilia perifrica (puede faltar en personas inmunodeprimidas). Demostracin de larvas en el aspirado duodenal
obtenido mediante fibrogastroscopia. Clinica durante la fase pulmonar, las larvas migratorias, pueden ocasionar infiltrados
eosinonoflicos y tos (sndrome de Leffler). Manifestaciones gastrointestinales: dispepsia, dolor abdominal, diarrea intermitente.
Puede producir un sndrome de malabsorcin. Capaz de autoinfestaciones. En personas inmunodeficientes puede producir un
Sindrome de hiperinfestacin de pronstico grave y elevada mortalidad. Puede producir manifestaciones cutneas: larva currens. Las
personas se infectan cuando las larvas filariformes del nematodo penetran a travs de la piel, en contacto con el suelo infectado.
Despues de un ciclo dentro del organismo humano (piel, circulacin, pulmones, trquea, faringe), los gusanos adultos viven en el
intestino. Entamoeba hystolitica (protozoo): Es un patgeno, con capacidad invasiva y de diseminacin extraintestinal. En el intestino
afecta al intestino grueso. Diagnstico. Examen de parsitos en heces (visualizacin de trofozoitos y/o quistes); Deteccin de antgeno
en heces. Coprocultivo en medios especiales. Serologa. Examen anatomopatolgico de biopsia intestinal. Difcilmente se visualiza E.
histolytica en el pus de los abscesos hepticos, el cual tiene una coloracin amarronada (pasta de anchoas). Pruebas de imagen
(ecografa, TC). La amibiasis puede manifestarse como cuadro diarreico; molestias abdominales inespecficas; disentera amebiana;
colitis fulminante con desarrollo de megacolon txico; ameboma. Puede afectar a zonas extraintestinales, siendo el absceso heptico
amebiano la localizacin ms frecuente que puede extenderse a zonas contiguas (pleura, pulmn, pericardio, etc.); otras afectaciones
MANUAL DE TRABAJO DEL CURSO ENARM CMN SIGLO XXI
CURSO ENARM CMN SIGLO XXI TEL: 36246001 Pharmed Solutions Institute PGINA 454

son mucho menos frecuentes (cerebrales, cutneas,
genitales). Hay personas asintomticas. Indicaciones
quirrgicas. En casos de megacolon txico y
perforacin. Puncin aspirativa transcutnea de los
abscesos amebianos, slo indicado en aquellos
abscesos de gran tamao con riesgo de rotura hacia
otras estructuras adyacentes. Giardia lamblia
(protozoo): Produce colonizacin del intestino delgado,
con adherencia del parsito a la mucosa intestinal.
Diagnstico. Identificacin del parsito en las heces.
Deteccin de antgeno en heces (ELISA). Clnica.
Asintomtica. Dispepsia, diarrea crnica intermitente.
Sndrome de malabsorcin sobre todo en pacientes
inmunodeprimidos (sida, dficit de IgA). Taenia
saginata (cestodo): Reside en el intestino delgado del
husped. Diagnstico. Identificacin de las progltides
en las heces de los pacientes. No puede diferenciarse
de la T. solium excepto con el recuento de las ramas
uterinas de las progltides grvidas. Clnica. Asintomtica en la mayora de los casos; sntomas disppticos. Anorexia, urticaria, prurito,
cefalea, convulsiones. De forma inusual pueden producirse complicaciones por la migracin de progltides a lugares como apndice,
conducto biliar o pancretico. Epidemiologa. Cosmopolita. Transmisin oral a partir del consumo de carne cruda (o poco cocida) de
ganado vacuno contaminada. Taenia solium (cestodo): Vive en el intestino delgado del husped. Clinica y diagnostico igual que T.
saginata, La cisticercosis, una enfermedad que puede afectar al SNC y tambin al globo ocular, es una de las complicaciones de la T.
solium. Trichuris trichiura (nematodo): Los parsitos adultos de T. trichiura viven en el colon y el recto, adheridos a la mucosa
intestinal. Criterios diagnsticos. Examen parasitolgico de las heces, donde se visualizan los huevos del parsito. Clnica. Puede ser una
infeccin asintomtica, provocar molestias abdominales inespecficas; diarreas, colitis y prolapso rectal en nios. En los casos de
hiperinfestacin pueden provocar disentera. Trichinella spiralis (nematodo): La triquinosis es una enfermedad que se produce por la
ingestin de quistes de triquina en carne contaminada. A partir de ella se liberan larvas que penetran en la mucosa intestinal, y se
desarrollan hasta el estadio adulto (30-40h). A los 5 dias comienza la fase larviposicion, donde las larvas son liberadas en la mucosa y
atreves de los linfticos acceden a la circulacin general.

CASO CLINICO
Nio de 11 meses de edad con antecedentes de 1 da de movimientos lentos. Evacuaciones sin sangre, asociados con 10-15 episodios
de vmito y diarrea en 4 ocaciones. Se observa letrgico y con disminucin de uresis. Fue hidratado y se le dio va ceftriaxona y
metronidazol. El nio tuvo un episodio de vmitos donde encontro un gusano. En el examen, pareca aburrido, con deshidratacin
moderada. Tena un pulso de 137, una frecuencia respiratoria de 28 por minuto, y una temperatura de 37 C. Su estatura era superior
al percentil 95, su peso estaba en percentil 5.

PREGUNTA
Cual es el estado nutricional.

RESPUESTA
a.- Normal
b.- Desnitricin leve
c.- Desnutricin moderada
d.- Desnutricin severa

PREGUNTA
El paciente mejora su estado hidrioelectrolitico con SRO, cul es manejo farmacologico.

RESPUESTA
a.- Albendazol.
b.- Mebendazol.
c.- Nitaxozanida.
d.- Metronidazol

PREGUNTA
Considerando el cuadro clnico cual de los siguientes helmitos intestinales es mas probable.

RESPUESTA
a.- Enterobius vermiculares.
b.- Ascaris lumbricoides.
c.- Trichuris trichiura.
d.- Ancylostoma duodenale.

PREGUNTA
MANUAL DE TRABAJO DEL CURSO ENARM CMN SIGLO XXI
CURSO ENARM CMN SIGLO XXI TEL: 36246001 Pharmed Solutions Institute PGINA 455

Considerando la presencia de parasitos en tubo digestivo alto se incrementa el sndrome de loeffler, cual no es una manifestacin que
lo orienta.

RESPUESTA
a.- Prurito nasal.
b.- Parasitos en heces negativo.
c.- Rx de torax con infiltrados intersticiales.
d.- Rectificacin nasal.

CASO CLINICO
Una nia de 4 aos de edad con dolor abdominal y distensin desde hace 6 das, segn su madre. Ella tambin tena antecedentes de
vmitos desde 3 das. El dolor de inicio gradual, clicos, seguido por distensin abdominal y aumento en la intensidad del dolor y los
vmitos. El vmito era de color verdoso y contena partculas de alimentos no digeridos. El nio fue vacunado parcialmente. En la
exploracin fsica estaba mal alimentado con palidez, temperatura oral fue de 37,7 C, la presin arterial era de 98/60 mmHg, pulso
regular, con una frecuencia de 110 latidos por minuto, frecuencia respiratoria de 34 por minuto. El examen abdominal revel un
abdomen distendido de todos los cuadrantes. En la auscultacin, ruidos intestinales hiperdinmico eran audibles. Examen rectal revel
heces con sangre y moco asi como varios gusanos vivos.

PREGUNTA
Considerando las manifestaciones, cual es el agente etiolgico ms probable asociado.

RESPUESTA
a.- Entamoeba histolytica.
b.- Giardia Intestinalis.
c.- Enterobius vermicularis.
d.- Dientamoeba fragilis.

PREGUNTA
Considerando el agente etiolgico que considero mas probable en el caso anterior, cual es la conducta teraputica a seguir.

RESPUESTA.
a.- Iodoquinol 30-40 mg/kg/da vo (mx 2 g) en 3 dosis x 20 da, albendazol 15 mg/kg/da vo x 5-7 das (max 400 mg).
b.- Metronidazol 5 mg/kg vo tid (mx 750 mg/da) x 5 das, nitazoxanida 200 mg x 3 das.
c.- Tetraciclina 40 mg/kg/da (mx 2 g) vo en 4 dosis x 10 das, metronidazol 20-40 mg/kg/da vo en 3 dosis x 10 das.
d.- Albendazol 15 mg/kg/da vo x 5-7 das (max 400 mg), metronidazol 5 mg/kg vo tid (mx 750 mg/da) x 5 das.

CASO CLINICO
A 16 meses de edad de sexo masculino present a la sala de urgencias despus de tener dificultad para respirar y cianosis peribucal en
casa. Su enfermedad comenz tres semanas antes, con diarrea sin fiebre, varios miembros de la familia tambin estaban enfermos.
Aunque la diarrea del paciente paro, continu hipoactivo y prefiere gatear a caminar. Cuatro das antes, desarroll fiebre y en el da de
la admisin sus padres notaron repentina dificultad para respirar y color azulado alrededor de los labios. Su temperatura era de 40,5
C, frecuencia cardaca de 180 latidos / minuto, la respiracin 62/minuto.

PREGUNTA
Cual es su conducta ms importante a seguir.

RESPUESTA
a.- Iniciar amoxicilina ms albendazol y paracetamol.
b.- Manejo de la fiebre por medios fsicos, observacin y signos de alarma.
c.- Envio a segundo nivel, previo manejo de temperatura.
d.- Manejo de fiebre por medios fsicos ms paracetamol, amoxicilina, albendazol y laboratorios.

PREGUNTA
El paciente presenta los siguientes datos 6 horas despus: saturacin de oxgeno del 96%, disminucin del murmullo vesicular en la
base del pulmn izquierdo. Clulas blancas de 12,000, con neutrfilos segmentados 58%, 13%, linfocitos, monocitos 15% 6%, 2% y
eosinfilos. El hematocrito fue del 37,5%, plaquetas 48,000, velocidad de sedimentacin globular de 33 mm / hora y la protena C
reactiva 57,4 mg / dL.

PREGUNTA
Considerando los resultados de laboratorios y gabinete cual de las siguientes patologas es ms probable.

RESPUESTA
a.- Parasitosis y neumona en paciente con factores de riesgo.
b.- Neumonia adquirida en la comunidad.
c.- Neumonia sin relacin a la parasitosis previa.
d.- Procesos independientes en paciente con inmunocompromiso.
MANUAL DE TRABAJO DEL CURSO ENARM CMN SIGLO XXI
CURSO ENARM CMN SIGLO XXI TEL: 36246001 Pharmed Solutions Institute PGINA 456


CASO CLINICO
Nia de 9 aos de edad que presenta edema facial de predominio matutino de 3 meses de evolucin, sin lesiones de urticaria ni otra
sintomatologa acompaante. En la anamnesis realizada no se encontr relacin con alimentos, frmacos ni otros posibles agentes
desencadenantes, ni tampoco antecedentes personales ni familiares de inters alergolgico. En la exploracin fsica realizada destacaba
un ligero sobrepeso y un edema en surcos nasogenianos.

PREGUNTA
Cual es la conducta a seguir mas adecuada.

RESPUESTA
a.- Iniciar antihistamnico, previa administracin de corticoide oral.
b.- Solicita BH, QS, EGO.
c.- Solicita pruebas cutaneas, anticuerpos y perfil tiroideo.
d.- Envia paciente a tercer nivel.

PREGUNTA
El paciente presento una leve mejora, sus estudios presentaron: anlisis de orina y de sangre (hemograma, bioqumica general,
hormonas tiroideas, estudio de complemento, IgA, IgG e IgM) fueron normales. Los anticuerpos antinucleares y antitiroideos fueron
negativos. La cifra de IgE total fue de 9 U/ml. Las pruebas cutneas para hipersensibilidad inmediata con una batera estndar de
neumoalrgenos habituales de la zona, fueron negativas.

PREGUNTA
Cual es la conducta a seguir.

RESPUESTA
a.- Coproparacitoscopico.
b.- Indica antihistaminico.
c.- Indica metronidazol
d.- Indica albendazol.

DESHIDRATACION, LIQUIDOS Y ELECTROLITOS. CIENCIAS BASICAS: Para fines de terminologa mdica, deshidratacin se define como
el estado clnico consecutivo a la perdida de lquidos y solutos en el cuerpo humano. Sin embargo, es posible encontrar deplecin
corporal de agua sin prdida de solutos, de causas diversas, sin denominarse deshidratacin. SALUD PUBLICA: La primera causa de
deshidratacin en el mundo es la diarrea aguda con mil millones de episodios anuales y ms de 2.5 millones de muertes secundarias a
deshidratacin. En Mxico la tasa de mortalidad en 2005 fue de 21.6 por 100 000 habitantes en menores de 5 aos. PATOGENIA: las
principales causas de deshidratacin, estn determinadas por dos mecanismos: 1) Incremento en las prdidas (Intestinales; vmitos,
diarrea, sondas, fistulas intestinales. Extraintestinales; quemaduras, uso de diurticos, diuresis, osmtica, poliuria, fiebre). 2) Falta de
aporte (por va oral, por vas parenterales). Otras causas de deshidratacin son cetoacidosis diabtica, diabetes inspida, estrs
postquirrgico y privacin de agua. La distribucin de lquido en el cuerpo est determinada por la edad. En el recin nacido el lquido
corporal total es de 70-75%, pero va disminuyendo conforme avanza la edad hasta ser de 60% en el adulto. El lquido corporal total est
distribuido a su vez en los espacios intracelular y extracelular; este ltimo est conformado por el espacio intersticial y el espacio
intravascular. La prdida de lquidos produce diferente dficit en los compartimentos de los espacios extracelular e intracelular. En la
deshidratacin aguda (menos de 2 das), la perdida de lquidos en su mayora es a expensas del lquido extracelular (75%); mientras que
en la deshidratacin prolongada, la perdida de lquidos es aproximadamente la misma en ambos espacios. La osmolaridad plasmtica
est dada por los diferentes solutos, de acuerdo con la siguiente frmula: Osmolaridad = 2(Na
+
mEq/L) + (NUS en mg/dl)/2.8 + (glucosa
en mg/dL)/18. La osmolaridad del plasma se modificara dependiendo de la causa de la deshidratacin y del mecanismo de sta; por
ejemplo, en pacientes con deshidratacin por diarrea aguda, puede haber mayor prdida de lquido que de solutos (gastroenteritis por
rotavirus) o mayor prdida de solutos que de lquido (clera); con base en lo anterior se produce deshidratacin isosmolar, hiposmolar
o hiperosmolar. DIAGNOSTICO: El diagnostico de deshidratacin se establece mediante un minucioso interrogatorio y examen fsico.
Sin embargo, el mdico o personal de
salud de primer contacto debe estar
entrenado para detectar datos clave.
Durante la valoracin de un paciente
deshidratado se debe poner especial
nfasis en tres aspectos: 1) Establecer
el grado o severidad de la
deshidratacin. 2) Determinar el tipo
de deshidratacin, as como sus
complicaciones concomitantes. 3)
Planear la forma de rehidratacin. Una
revisin sistematizada demostr que
los tres datos clnicos ms importantes
para identificar deshidratacin son:
llenado capilar prolongado, turgencia
de la piel y patrn respiratorio.
MANUAL DE TRABAJO DEL CURSO ENARM CMN SIGLO XXI
CURSO ENARM CMN SIGLO XXI TEL: 36246001 Pharmed Solutions Institute PGINA 457

Determinacin del tipo de deshidratacin. El tipo de deshidratacin est dado principalmente por la cuantificacin srica de Na
+
: 1)
Deshidratacin isotnica (isonatrmica), con Na
+
srico entre 130 y 150 mEq/L. 2) Deshidratacin hipertnica (hipernatrmica), con Na
+

srico mayor a 150 mEq/L. 3) Deshidratacin hipotnica (hiponatrmica), con Na
+
srico menor a 130 mEq/L. El tipo de deshidratacin
depender entonces de la prdida que
predomine: si la deplecin de lquido es
menor que la de solutos, se presentar
una deshidratacin hipertnica; y si es
mayor la prdida de solutos, ser
hipotnica. En trminos de frecuencia,
la isotnica representa 80% del total de
las deshidrataciones, la hipotnica 15%
y la hipertnica 5%. Las manifestaciones
de la hiponatremia se correlaciona con
la velocidad de descenso del Na srico:
si el decremento sucede en un tiempo
menor a 12h, se denomina
hiponatremia aguda, la cual mostrara
datos clnicos ms tempranamente que
la desarrollada en un tiempo mayor. Las
manifestaciones graves son las del SNC:
irritabilidad, vmitos, nausea,
fasciculaciones, cefalea, e incluso crisis
convulsivas o coma con cifras menores a 120 mEq/L. Otros elementos a considerar en sujetos con deshidratacin son el estado acido-
base y la hipokalemia. Estos pacientes pueden pre4sentar acidosis metablica (pH <7.35 con HCO3 <29mEq/l), debido al incremento en
las perdidas de bicarbonato; la disminucin en la capacidad del rin para eliminar hidrogeniones y la ganancia de cido lctico por
hipoperfusion que se presenta en la deshidratacin moderada y severa. Las manifestaciones clnicas de la acidosis metablica son
polipnea (como mecanismo de compensacin), depresin miocrdica, arritmias, dilatacin arteriolar, entre otras. La hipokalemia
(K<3mEq/l), es producida por 2 mecanismos, las perdidas incrementadas a travs de la heces y la excrecin renal aumentada para
mantener la homeostasis con Na. Las manifestaciones clnicas pueden ir de un paciente totalmente asintomtico, hasta sujetos con
leo, rabdomiolisis o arritmias. Laboratorio: El examen de orina muestra densidad especfica, generalmente mayor a 1.020, debido al
mecanismo homeosttico de resorcin de lquidos. En caso de registrar densidad especfica menor a 1.020, se deber descartar alguna
nefropata intrnseca, por la incapacidad de realizar esos mecanismos. La determinacin de electrolitos sricos (Na+, K+, Cl-), tiene
especial importancia para determinar el tipo de deshidratacin hipo o hipernatrmica. El potasio srico puede modificarse a su vez por
estados de alcalosis o acidosis; si el pH desciende 0.1 unidades por debajo de 7.35, el K+ srico incrementa 0.5 mEq/L y viceversa, siel
pH aumenta 0.1 unidades, por arriba de 7.45,el K+ srico desciende 0.5 mEq/L. La determinacin srica de cloro ayuda generalmente a
calcular la brecha aninica (anion gap), la cualpermite descartar otras causas de acidemia, como acidemias orgnicas, o cetoacidosis
diabtica. Se calcula mediante la siguiente frmula: Na
+
- (Cl
-
+ HCO3-). Tiene como valor normal entre 8 y 12 mEq/L. TRATAMIENTO:
Como ya se mencion, la diarrea aguda es la principal causa de deshidratacin en nuestro pas. Par esta causa la OMS especficamente
recomienda un plan a seguir con base en los datos clnicos que presente el nio. El plan A en pacientes sin deshidratacin o
deshidratacin leve, el plan B en casos de deshidratacin moderada y el plan C en enfermos con choque. PLAN A: Continuar con
alimentacin habitual, aumentar la ingestin de lquidos de uso cotidiano en el hogar, VSO, en los menores de 1 ao, ofrecer media
taza (75ml) y en lo mayores de un ao una taza (150ml) y administrarlo a cucharadas o mediante sorbos pequeos despus de cada
evacuacin. Capacitar a la madre para reconocer los signos de deshidratacin y datos de alarma por enfermedades diarreicas (sed
intensa, poca ingestin de lquidos y alimentos, numerosas heces liquidas, fiebre, vmito y sangre en las evacuaciones) con el propsito
de que acuda nuevamente a solicitar atencin medica en forma oportuna. Esta formula consta de Na 90mEq/l, K 20mEq/l, HCO3
30mEq/l, Cl 80mEq/l, glucosa 111mEq/l. la OMS actualmente recomienda el uso global de esta frmula con sodio 75 mEq/L,
osmolaridad 245 mOsm/L y glucosa 75 mEq/L. PLAN B: Pasar pacientes con diarrea y deshidratacin con atencin en la unidad de salud;
VSO 100ml/kg, en dosis fraccionada cada 30min durante 4h VO. Si el paciente presenta vmito, esperar 10 minutos e intentar otra vez
la hidratacin oral ms lentamente se puede intentar infusin por sonda nasogstrica a razn de 20-30 mL/kg/h. Al mejorar el estado
de hidratacin, pasar a plan A. En caso contrario repetir el plan B en 4h, de no existir mejora pasar al plan C. PLAN C: Iniciar
inmediatamente administracin de lquidos por va IV, con solucin de Hartmann, si no se encuentra disponible, use sol. Salina al 0.9%,
de acuerdo con el siguiente esquema: Primera hora 50ml/kg, segunda hora 25ml/kg, tercera hora 25ml/kg. Evaluar al paciente
continuamente. Si no mejora aumentar la velocidad de infusin. Cuando pueda beber (usualmente 2-3h), administrar sobres de VSO, a
dosis de 25ml/kg/h, mientras sigue lquidos IV. Al completar la dosis IV, evaluar al paciente para seleccionar plan A o B, y retirar
venoclisis, o repetir plan C. Si selecciona el plan A, observar durante h para asegurarse de que el responsable del paciente pueda
mantenerlo hidratado con sobres de VSO, adems alimentarlo en su domicilio. La OMS recomienda la administracin de solucin de
Ringer-lactato o sol. Fisiolgica en dosis de 20 mL/kg de peso hasta que el pulso, la perfusin y el estado de conciencia regresen a la
normalidad. En sujetos en quienes est contraindicada la va oral o tienen otras causas de deshidratacin se sugiere un plan de
tratamiento que se divide en cinco fases: La primera fase de la rehidratacin se debe llevar a cabo en los primeros 30 a 60 minutos
(NaCl 0.9% 20 mL/kg en bolo). Su objetivo es la restitucin rpida de la deplecin de volumen circulante, en pacientes con choque
hipovolmico. En estos pacientes se inicia la reposicin de lquidos con bolos de 20 mL/kg de cristaloides. Para el manejo de choque
hipovolmico en nios y RN, existen las siguientes recomendaciones clnicas. La primera eleccin para la resucitacin inicial es la
solucin salina; si se requieren grandes cantidades de lquidos, es posible utilizar coloides sintticos por su mayor duracin en la
circulacin y por ltimo el volumen inicial debe de ser de 20 mL/kg; dependiendo de la respuesta clnica, se deber valorar el nmero
de dosis necesarias y, en pacientes con problemas cardiacos o recin nacidos, se recomiendan bolos de 10 mL/kg. La segunda fase se
lleva a cabo dentro 1-9 hrs de iniciada la hidratacin del paciente. Su finalidad es la restitucin parcial del dficit del espacio
MANUAL DE TRABAJO DEL CURSO ENARM CMN SIGLO XXI
CURSO ENARM CMN SIGLO XXI TEL: 36246001 Pharmed Solutions Institute PGINA 458

extracelular y del estado cidobase. El plan teraputico en esta fase es administrar 1/3 de lquidos de mantenimiento diarios y del
dficit de lquidos. La recrea fase se lleva a cabo a las 9hrs a 24h de haber iniciado la hidratacin. Su objetivo es la restitucin de
volmenes del los compartimentos EEC y EIC y del estado acido-base. Administrar 2/3 de los lquidos de mantenimiento diario y 1/3 del
dficit de lquidos. La cuarta fase se lleva a cabo dentro de las 25 a 48 horas de iniciada la hidratacin. Su finalidad es la correccin total
de electrolitos, estado cidobase y volumen de lquidos, dar lquidos de mantenimiento y reponer las prdidas actuales, as como
iniciar la va oral si las condiciones lo permiten. La quinta fase se lleva a cabo, durante los 2-14 dias de haber iniciado la hidratacin. Su
objetivo es restaurar el dficit calrico y proteico. Esto se logra con una adecuada dieta por va oral. CORRECCIN DE TRASTORNOS
ELECTROLTICOS Y CIDO-BASE CONCOMITANTES: En pacientes con hiponatremia (Na <13mEq/l), se debe agregar a las soluciones de
mantenimiento y a las del dficit la cantidad necesaria de sodio para llevarla a niveles sricos normales.41 La correccin no debe ser
ms rpida que 1 mEq por hora o 12 mEq/L en 24 horas, debido al riesgo de producir cambios osmolares en el sistema nervioso central.
La correccin se hace mediante la siguiente frmula (Na+ ideal - Na+ medido) x peso en kg x 0.6= mEq a administrar en 24 horas. A este
resultado se suman los requerimientos normales. En pacientes con hiponatremia sintomtica y Na srico <120mE/l, se debe
incrementar el valor de Na+ por arriba de esta cifra, en un periodo de una hora, con solucin salina al 3% de 5 a 10 mL/kg; esto elevar
el Na+ srico de 4 a 8 mEq/L. En pacientes con hipernatremia (Na srico >145mEq/l), se debe corregir el dficit de lquidos en un
periodo mayor, llegando hacer hasta 48h, debido a que el descenso rpido de Na tambin puede causar cambios osmolares en el SNC,
provocando edema cerebral y mielinolisis pontina. La meta es descender el Na srico un mximo de 15mEq/dia (1mEq/h). La
hipokalemia se debe tratar solo con incrementos del aporte diario en pacientes asintomticos, aumentando la concentracin de potasio
en soluciones a 60-80mEq/l; pero en pacientes que presentan sintomatologa, como arritmias, debilidad muscular importante o
dificultad respiratoria atribuible a esta, deben ser estrechamente monitorizados y administrar KCL a razn de 0.1-0,2 mEq/Kg/h, y la
hiopocalemia con alteraciones ritmo cardiaco que pongan en peligro la vida, la infusin de K+ debe ser de 0.3 mEq/kg/h, siempre
mediante monitorizacin de electrocardiografa estrecha y la administracin del potasio IV mediante bomba de infusin. La acidosis
metablica se trata desde el inicio con la restitucin de volumen; sin embargo, la administracin parenteral de HCO3, esta indicada en
pacientes con pH menor a 7.10 o HCO3- menor de 10 mEq/L, y se calcula por medio de la siguiente frmula: HCO3 (mEq/L) = Dficit de
base x peso(kg) x 0.3, debindose administrar al paciente de un cuarto a la mitad de la cantidad resultante de HCO3. Un cuarto de la
dosis equivale aroximadamente a 1mEq/kg (peso) de HCO3.

CASO CLINICO
Paciente masculino de 4 aos de edad con diagnostico de gastroenteritis el fue tratado de forma ambulatoria, a las 6 horas acude a
urgenciar debido a que continuo con vomito en 8 ocaciones y 5 deposiciones liquidas, a la exploracin fsica se observa ojos hundidos,
llanto sin lagrimas, somnoliento, hiporreactivo.

PREGUNTA
Cual de los siguientes signos es esta mas relacionado con el choque descompensado.

RESPUESTA
a.- Taquicardia.
b.- Taquipnea.
c.- Tiempo de llenado capilar prolongado.
d.- Hipotensin.





















MANUAL DE TRABAJO DEL CURSO ENARM CMN SIGLO XXI
CURSO ENARM CMN SIGLO XXI TEL: 36246001 Pharmed Solutions Institute PGINA 459

QUEMADURAS. CIENCIAS BASICAS: Son lesiones traumticas primariamente de la piel pero que pueden afectar a otros tejidos, por
accin de diversos agentes fsicos, qumicos, biolgicos, con alteraciones, de enrojecimiento a destruccin total. Suelen acompaarse
de una variedad de complicaciones que cuando son graves o inesperadas pueden ocasionar el fallecimiento del paciente. SALUD
PUBLICA: Las quemaduras constituyen un problema mdico, psicolgico, econmico y social que involucra al mdico, psiclogo,
econmico y social que involucra al mdico, al paciente, al familiar del paciente y a la sociedad en s. En Mxico se calcula se atienden
en hospitalizacin 10 000 pacientes. Ms de 66% de los pacientes quemados recibidos en unidades especiales, son nios. Pacientes
tienen el 77% de mortalidad cuando hay combinacin de quemadura de va area con quemadura cutnea. PATOGENIA: El mecanismo
de lesin ms comn es la escaldadura 72.7%, seguida del fuego directo 18.7%, por contacto 10.5%, elctricas 8%. Otros: por ignicin,
por inhalacin, quemaduras qumicas, por deflagracin, por frio, por radiacin. La quemadura se debe a la transferencia de energa de
una fuente de calor al organismo; no es homognea en las regiones corporales afectadas ya que se produce ms lesin en el rea de
mayor exposicin. En la parte central donde la transferencia de energa es mxima, ocurre una necrosis cutnea irreversible, la zona de
coagulacin. Alrededor de ella se dispone una zona que se caracteriza por una intensa reaccin inflamatoria, la zona de estasis; esta ara
puede evolucionar a la recuperacin pero si se presenta disecacin o infeccin puede evolucionar hacia una completa destruccin en
cuyo caso incrementa el rea de afeccin. En la periferia se encuentra la zona de hiperemia en donde el dao celular es mnimo y la
recuperacin espontanea es muy rpida. CLASIFICACION: Por su profundidad; 1er GRADO (epidrmica o eritematosa); Involucra
epidermis, la piel luce eritematosa, de color rojo o rosa brillante, seca, es progresivamente pruriginosa y presenta hiperalgesia. Sana
espontneamente sin dejar cicatriz. Ejemplo: quemadura solar. 2do. GRADO SUPERFICIAL (de espesor parcial o flictenular); Involucra
epidermis y dermis papilar, se manifiesta por un color rojizo, es suave, hmeda y existen flictenas, Cicatriz normalmente en un lapso de
10-14 das. 2do. GRADO PROFUNDA; Involucra epidermis,
dermis pailar y en profundidad variable la dermis reticular, pero
dejan intactas las faneras epidrmicas profundas. Color rojo
moteado o blanquecinas, no existen flictenas y son
hipoalgesicas. La cicatrizacin ocurre en 21-28 das, siendo
frecuente la cicatrizacin hipertrfica, requieren injertos. 3er.
GRADO (espesor total); Son de color blanquecino, amarillo
plido, momificacin, marrn o negro (carbonizacin). El tejido
tiene una apariencia acartonada y suelen observarse vasos
trombosados. Injerto de piel >1 cm. Efectos sistmicos si es extensa, puede haber amputaciones. Curan > 42 das. La cicatrizacin
ocurre principalmente por contraccin y es deformante. Clasificacin por su gravedad LEVE; Quemaduras de 2do grado superficial o
profunda o ambas, de menos de 10% de SC. Quemadura de 3er grado de menos de 2 % de SC. Sin afeccin de reas especiales.
MODERADAS; 2do grado superficial o profundas o ambas entre 10-20% de SC. De 3er
grado entre 2-10% de SC. Sin afeccin de reas especiales. SEVERAS; 2do grado
superficial o profundo >20% de SC. De 3er grado >10%. Afeccin de reas especiales
(cara, ojos, cuello, manos, pies, genitales, articulaciones y pliegues). En menores de 1
ao. Inhalacin, corriente elctrica. Presencia de quemaduras ms fracturas o traumas
graves, enfermedades asociadas. DIAGNOSTICO: Valorar, gravedad, tipo y extensin de
la quemadura esta ltima con las Tablas del porcentaje de los segmentos corporales
segn edad de "LURD y BROWDER" o ms conocida como REGLA DE LOS "9". sta y otras
formas de medicin significa no solo conocerlas, sino prctica en su uso, por lo que una
forma sencilla de saber la extensin de una quemadura es utilizar la regla de la palma de
la mano, la cual representa aprox., el 1% de SC. Datos clnicos que indican quemadura de
vas areas, inhalacin o ambas; cianosis, expectoracin carboncea, depsitos
bucofarngeo de carbn, ampollas y cambios inflamatorios en lengua y paladar, cambios
de voz, ronquera, tos persistente, quemadura de cara, cejas y delas vibrisas, historia de
confinamiento cuando ocurri el fuego, estado de alerta alterado. El choque
hipovolmico puede desarrollarse rpidamente en los pacientes peditricos. Un nio con
10Kg de peso, con una quemadura de 20% de extensin, tienen perdidas por evaporacin cercanas a los 475ml, equivalentes al 60% de
su volumen circulante. CRITERIOS DE HOSPITALIZACION: 1) Extensin de la quemadura en una rea mayor del 10% (<5 aos con
superficies mayores a un 5%) de la SCT. 2) Quemadura de cara, cuello, rea glteogenital y eventualmente manos en quemaduras
palmo digitales intermedias o profundas. 3) Quemadura elctrica de alto voltaje o de bajo voltaje. 4) Quemadura circular de
extremidades, trax o cuello. 5) Quemadura por cidos o lcalis. 6) Rescate desde un espacio cerrado con ambiente invadido por humo
(Sospecha de Quemadura Respiratoria). 7) Traumatismo mecnico importante asociado. 8) Enfermedad metablica o sistmica
asociada. 9) Sospecha de maltrato infantil. 10) Marginalidad o ruralidad extrema. 11) Caso social (analfabetismo o escasa escolaridad de
los padres o personas a cargo del nio, recursos econmicos escasos, etc.). TRATAMIENTO: Inmediatamente evolucin de vas areas y
ventilacin (edema masivo, obstruccin ventilatoria, lesin por inhalacin), estado hemodinmico, cuantificar tamao y profundidad de
quemadura, agente etiolgico, tiempo y lesiones asociadas, recordar mientras ms pequeo el paciente es ms susceptible. La
evaluacin tradicional del dficit neurolgico no debe olvidarse. Traslado del paciente. Tratamiento: Pacientes no intubados requieren
terapia respiratoria agresiva con broncodilatadores y humidificadores con mezclas de Heliox y O2, ventilacin mecnica, cmara
hiperbrica. Corticoesteroides no reducen el edema de la va area. Elevar la cabeza 30 grados en la resucitacin inicial disminuye el
edema de la va area. Reposicin hdrica: <2 aos: >10% SCQ. >2 aos: >15% SCQ. Calcular prdidas insensibles: (25 + %SCTQ) X (SCT
m
2
). SCTm
2
= Peso x 4 + 7/peso + 70. Frmula de Parkland y Baxter, se utiliza en las primeras 24h, con Hartmann 24 despus coloides. 4
ML x KG x % SCT Quemada. La mitad del dficit total en las primeras 8h sol Hartmann. La segunda mitad en las 16h restantes. Calcular
volumen circulante 60cc x kg. Paciente de 8 kg que tiene una quemadura de 25% de su cuerpo y se decide utilizar Parkland modificado
para restituir lquidos. 4ml x kg x % SCQ, 4 x8 x 25 = 800 ml, pasar 400ml para 8h y 400ml para 16h. Formula de Galveston: Primer da:
50% en 8h y 50% en 16h = 500ml de sol. Hartmann/kg/SCQ + 200ml de sol. Glucosada al 5%/kg/SCQ. El segundo da: 3750 ml/kg/ SCQ
Hartmann + 1500 mil/kg/ SCQ glucosada 5% en 24h. Vigilar continuamente y mantener adecuada PVC (8 - 10), mantener TAM >
MANUAL DE TRABAJO DEL CURSO ENARM CMN SIGLO XXI
CURSO ENARM CMN SIGLO XXI TEL: 36246001 Pharmed Solutions Institute PGINA 460

60mmHg, gasto urinario 0.5 - 1.0ml/kg/h, evitar administracin excesiva de lquidos, uresis (2ml/kg/h), vigilar hipotermia. Manejo de
dolor con narcticos, analgsicos, sedantes: Morfina 10 mg IV c/3-4 h. Nalbufina 10 mg IV c /3-4 h. NO administrar antibiticos
empricos. Cuidado de quemaduras: Sulfadiacina de plata (Silvadene), cubrirlas con sbanas estriles, curaciones quirrgicas (qx
plstica). La infeccin en pacientes quemados es la mayor causa de morbilidad, en algunos centros especializados, la sepsis es causante
de 50-60% de las muertes. Se sabe que durante las primeras 24h posteriores a la quemadura predominan grmenes gramm+ (S.
aureus). Despus del final de la primera semana se inicia colonizacin por gramm- (P. aeruginosa. La infeccin por anaerobios se ha a
atribuido principalmente a Bacteroides. Se manejara antibitico acorde a germen implicado. El diagnstico definitivo de infeccin de la
lesin se basa fundamentalmente en el estudio histopatolgico por medio del cultivo biopsia.

CASO CLINICO
Ingresa masculino de 12 aos de edad el cual fue traido por ambulancia ya que fue sobreviviente de incendio en su casa, refieren los
paramdicos que fue encontrado con somnolencia, ya que al parecer estuvo expuesto al humo por tiempo desconocido, a la
exploracin fsica se encuentra conciente alerta, orientado levemente ansioso, observa quemadura leves y superficiales de cara y
manos, se observa abundante pelo quemado, presenta tos seca moderada.

PREGUTA
Cual es la conducta a seguir.

RESPUESTA
a.- Ingresar a vigilancia y colocar oxigeno 3 Lt por minuto.
b.- Enviar a unidad especializada en quemadura.
c.- Realizar tele de torax.
d.- Manejo ambulatorio.

CASO CLINICO
Femenino de 9 aos con parlisis cerebral infantil quien ingresa por quemaduras de 9% de superficie corporal. Madre de 30 aos,
analfabeta. Padre campesino, analfabeta. 7 hermanos sanos. Medio socioeconmico bajo. Producto de Gesta V, sin control prenatal,
parto domiciliario. Retraso grave en el desarrollo psicomotor. Ignora inmunizaciones. Hace 5 das presenta quemadura por escaldadura,
afectando glteo, pierna y muslo izquierdos. Manejada por mdico particular con arnica y al no mejorar aude a esta unidad.
Exploracin fsica: Peso: 15 kg Talla: 104 cm Temp: 37C Escolar femenino con PCI, bien hidratada, sin compromiso cardiopulmonar ni
abdominal. Lesiones por quemaduras de 2 grado superficiales y profundas que afectan glteo izquierdo y cara posterior de muslo y
pierna, con tejido de granulacin y reas necrticas. No hay exudado purulento.

PREGUNTA
Cual es la complicacin mas probable en este caso?

RESPUESTA
a.- Infeccin de la herida.
b.- Infeccin sistmica.
c.- Falla organica multiple.
d.- Coagulacin intavascular diseminada

CASO CLINICO
Masculino de 4 aos de edad ingresa a urgencias por presencia de quemadura elctrica en la boca, refiere la madre del paciente que el
nio se encontraba jugando y sbitamente comenz a llorar, sealando la boca, agrega que solo estaba un cable de luz que pareca
mordido y levemente quemado, a la exploracin se observa a paciente alerta, con facies algicas, llanto intenso, sus contantes vitales se
encuentran dentro de parmetros normales, la mucosa oral se encuentra lesin tipo quemadura que incluye comisura labial que
incluye piel peribucal.

PREGUNTA
Cual es su conducta a seguir.

RESPUESTA
a.- Lavar la herida y colocacin de gasas vaselinadas.
b.- Debridar lesin e ingreso a observacin.
c.- Envio inmediato a tercer nivel.
d.- Manejo conservador, con control del dolor.








MANUAL DE TRABAJO DEL CURSO ENARM CMN SIGLO XXI
CURSO ENARM CMN SIGLO XXI TEL: 36246001 Pharmed Solutions Institute PGINA 461

PICADURAS DE ARAAS. CIENCIAS BASICAS: El envenenamiento por picadura de araa, es un problema importante de salud pblica.
Se han reportado tasas de ms de 200,000 accidentes por ao debido a 3,000-5,000 por picadura de araa. Los accidentes por
artrpodos venenosos en Mxico como las araas de los gneros Loxosceles y Latrodectus. En todo el mundo existen cerca de 30,000
especies o subespecies. Las araas poseen en el
abdomen glndulas en donde se produce una
sustancia proteica con la que forman sus nidos y
redes, es decir las telas de araa. PICADURA
POR CAPULINA: Latrodectus mactans. El
lactrodectismo es un problema frecuente, la
araa capulina o viuda negra es frecuente
encontrarla aun en zonas cosmopolitas como el
DF. A pesar de ser un animal tmido, cuando es
molestado muerde una o ms veces. A diferencia
del alacrn, la mordedura no duele en el
momento; sin embargo los sntomas se inician
30-60min despus y se inician en el sitio de la
lesin, hipopigmentacin e inflamacin en el
sitio de la picadura y luego se diseminan, es
caracterstico el dolor articular, temblor y
fasciculaciones prcticamente de todos los
msculos, calambres abdominales, diaforesis
profusa, angustia y la fascies lactrodectsmica y
trismos. El veneno desestabiliza los canales inicos con liberacin de catecolaminas en las terminaciones adrenrgicas adems
deplecin de acetilcolina en nervios motores (despolarizacin prolongada de los receptores postsinapticos). La ponzoa de esta araa
es una proteasa alfa-latrotoxina, que tiene actividad neurotxica y una potencia 15 veces mayor a la del veneno de la serpiente
cascabel. Tratamiento: Colocar un torniquete para evitar la diseminacin de la toxina, traslado a clnica ms cercana. Analgsicos.
Neostigmina o prostigmina 0.5 a 1 mg c/8-12 h por va parenteral. Por va oral 15 mg c/8h 2-3 das. Este frmaco inhibe la accin de la
colinesterasa en los tejidos y la sangre, e impide la destruccin de la acetilcolina. Cortisona y corticotropina, que ayudan a controlar los
sntomas Generales. Suero aracmyn faboterapia, el 90% de los pacientes responde a una sola dosis; el 10% restante, responde a 2 3
frascos. Dosis 1 ampolleta IV lenta y los resultados son extraordinarios, los sntomas como dolor, calambres, angustia y miedo, mejoran
en los nios dentro de los primeros 30 min, y en los adultos solo un poco ms. Las dosis deben ser individuales y aplicar nuevas dosis
solo previa valoracin y persistencia de la sintomatologa. PICADURA POR ARAA VIOLINISTA: Loxosceles, El loxocelismo es el cuadro
que se presenta cuando una persona es picado por la araa llamada araa violinista o araa del rincn. De color marrn, con una
mancha ms plano y tres pares de ojos. 100 especies en todo el mundo. Mxico: L. boneti mide de 9-25 mm, es de color caf. Habita
lugares obscuros, poco ventilados, depsitos de madera. Se esconde en cuadros, muebles. Patogenia: El veneno, producido en poca
cantidad, tiene una accin dermonecrotizante, hemoltica, vascultica y coagulante. Acompaando el cuadro cutneo puede existir
compromiso sistmico que se manifiesta por hemlisis. Diagnstico: Loxoscelismo cutneo, es la forma de presentacin ms frecuente,
oscila en 84-97% segn distintos autores. La picadura suele producir dolor de poca intensidad inicialmente y, muchas veces, el paciente
no sabe precisar cundo ocurri, pero tambin se ha descripto un dolor urente. La lesin inicial se caracteriza por edema, eritema y, a
veces, dolor urente. Evoluciona en 24-36 h a la caracterstica placa marmrea o livedoide (reas intercaladas de palidez y equimosis)
de bordes irregulares, bien definidos, circundada por eritema y edema indurado, dolorosa a la palpacin. Ms tarde pueden aparecer
vesculas y flictenas en zonas declive. Las lesiones presentan un patrn de necrosis hemorrgica gravitacional, posiblemente
relacionada con la accin conjunta de las esfingomielinasas y la hialuronidasa. Como sntomas generales del loxoscelismo cutneo
pueden presentarse nuseas, vmitos y fiebre. Entre el 5 y 7 da la lesin cutnea se circunscribe con la formacin de una escara
negra, que comienza a desprenderse a partir de la segunda semana. sta deja una lcera con fondo de granulacin que tarda varios
meses en cicatrizar y puede requerir ciruga reparadora. La forma edematosa es una variante clnica del loxoscelismo por picadura en
una zona de tejidos laxos, habitualmente la cara. Se caracteriza por eritema y edema sin evolucionar a la necrosis. Las lesiones
cutneas, una vez establecidas, son muy difciles de tratar, aun con un tratamiento precoz y adecuado, y pocas veces es posible detener
su evolucin. Loxoscelismo cutneo-visceral: Se considera un cuadro ms grave que el anterior caracterizado por hemlisis
intravascular y coagulacin intravascular diseminada (CID) asociadas a la lesin cutnea anteriormente descrita. Tratamiento:
Inespecfico. Incluye tratamiento de sostn, asepsia de la lesin, profilaxis antitetnica en caso de requerirla y analgsicos. Dapsona: 1-2
mg/kg/da una dosis, evita necrosis. Vigilar: leucopenia, IR, metaheoglobinemia. Aplicar: a) 4 mg de fosfato de dexametasona por va
subcutnea en el sitio de la picadura. b) 40 a 80 mg de acetato de metil-prednisolona por va intramuscular. c) Repetir el tratamiento
local con 21-fosfato de dexametasona, cuatro horas despus. d) Si continan apareciendo vesculas, repetir la dosis de metil-
prednisolona a las 24 horas. e) Si ocurre hemlisis intravascular: 50 mg de prednisolona por va intravenosa, seguidos de 25 mg cada 6
horas, hasta que desaparezca la hemlisis. f) En caso necesario, previa valoracin general, 5,000 U de heparina cada 12 horas. g)
Administrar Aracmyn Plus (Faboterapia). PICADURA DE ALACRAN: Dentro de la clase Arachnida, hay artrpodos que llaman
especialmente la atencin del hombre, ya que desde tiempos inmemoriales le han causado lesiones traumticas, intoxicaciones,
invalidez, enfermedades severas y muerte.1 La picadura de alacrn est considerada como una urgencia mdica por la naturaleza que
representa el ataque, que es la inoculacin de veneno que provoca la alarma en la persona afectada, as como a sus familiares, ms an
si la agresin fue a un menor. SALUD PUBLICA: Mxico es el pas de mayor diversidad de escorpiones en el mundo, con siete familias, 90
gneros y 200 especies. Son siete las especies peligrosas; se encuentran en la zona occidental y central del pas, los estados de Nayarit,
Colima, Jalisco, Durango, Michoacn, Puebla, Guanajuato, Oaxaca son los ms importantes. El IMSS reporta en su semanario
epidemiolgico 60 000 casos por ao. PATOGENIA: Los Suffusus, Ticumanus, Limpidus, Noxius, Sculpturatus, Nayaritus, entre otros son
los responsables de las intoxicaciones severas en nuestro medio y las costas del pacifico son las ms afectadas. PATOGENIA: El veneno
de los alacranes est compuesto principalmente por polipptidos, enzimas proteolticas, protenas de bajo peso molecular, serotonina y
MANUAL DE TRABAJO DEL CURSO ENARM CMN SIGLO XXI
CURSO ENARM CMN SIGLO XXI TEL: 36246001 Pharmed Solutions Institute PGINA 462

aminocidos de accin neurotxica. Es inoculado directamente a la vctima por va subcutnea y se disemina por la va hematgena; es
eliminado por la orina y por la secrecin biliar. CLASIFICACION: Por su estado clnico se distinguen 4 grados de envenenamiento: I. Dolor
local, parestesias en la zona de la picadura. II. Se agrega parestesias en reas remotas a la zona del piquete. III. Disfuncin somtica o de
pares craneales. Inquietud, estremecimiento involuntario, espasmos musculares. Midriasis, visin borrosa, diplopa, ceguera temporal,
movimientos incoordinados, sialorrea, dificultad para deglutir, fasciculaciones de la lengua, espasmo larngeo, habla farfullante. IV.
Disfuncin somtica musculo esqueltica y de pares craneales. DIAGNOSTICO: Signos vitales: taquicardia, taquipnea, hipertensin
arterial, hipertermia. Laboratorio: no hay exmenes especficos. Tratamiento general: Compresas fras en tanto recibe tratamiento
especfico. Analgsico, antihistamnicos en caso necesario sintomticos por la hipertensin y ansiolticos. Especfico: faboterapia con
alacramyn, dosis inicial una ampolleta diluida en 5 ml de solucin inyectable aplicacin lenta en la vena, los resultados se miden a
travs de mejora de los sntomas antes de aplicar una segunda dosis.

CASO CLINICO
Una lactante de 17 meses de edad, present irritabilidad aguda, llanto, dolor en el pie izquierdo. La madre de la paciente le quit los
zapatos y se encontr una viuda negra muerta en el zapato izquierdo del nio. Se desarrollo rpidamente inflamacin del pie izquierdo.
El EF mostro presin arterial 145/103 mm Hg y taquicardia (frecuencia cardaca: 160-180 latidos por minuto). El examen revel edema y
eritema en el pie izquierdo y los prpados, y ninguna lesin diana clsica vista en el sitio. Se observaba con dolor significativo. La
glucosa srica fue elevada a 186 mg / dl, y su recuento de glbulos blancos se aument a 17 000/mm3.

PREGUNTA
Cual de las siguientes especies es la ms probable que se encuentre involucrada en el caso.

RESPUESTA
a.- Loxosceles.
b.- Latrodectus.
c.- Phoneutria.
d.- Tityus serrulatus.

PREGUNTA
Cual es el pronstico de la aracnismos presuntivo del caso?

RESPUESTA
a.- Dolor agudo, opresin precordial, taquicardia, dificultad respiratoria y sensacin de muerte inminente.
b.- Alteraciones electrocardiogrficas, hipertensin y edema de pulmn.
c.- Sensacin punzante, dolor urente o quemante y edema local.
d.- Ppula, que puede evolucionar a una placa eritematosa o placa marmrea.

PREGUNTA
El paciente fue tratado con diazepam, morfina y difenhidramina. Se traslado a la unidad de cuidados intensivos peditricos ~ 5 horas
despus de la mordedura de la araa, se qued irritado y con dolor, con un firme, pero no rgido, abdomen y notable eritema del pie
izquierdo. Cuales medidas son ms utiles previo al traslado a una unidad de segundo nivel.

RESPUESTA
a.- Retirar todos los elementos que puedan comprimir y agravar el edema en la zona afectada.
b.- En las lesiones localizadas en los miembros mantener posicin de Trendelemburg.
c.- Realizar antisepsia local, evaluar la necesidad de profilaxis antitetnica y utilizar analgsicos.
d.- Uso de corticoides y antibiticos.

PREGUNTA
Cual de las siguientes patologas es el ms probable para el diagnoticos diferencial?

RESPUESTA
a.- Picadura de escorpin.
b.- Picadura de Latrodectus.
c.- Picadura de himenptero.
d.- Picadura de miripodo.

CASO CLINICO
Un nio de 7 aos de edad se present con fiebre, convulsiones y dificultad respiratoria de inicio sbito sin causa aparente. Presentaba
GCS 12, disnea, taquipnea y taquicardia. Haba aleteo nasal y estertores crepitantes bilateralmente a la auscultacin.

PREGUNTA
Cual es la conducta a seguir.

RESPUESTA
a.- Vigilar por dos a cuatro horas en ayuno estricto y con monitoreo de signos vitales cada 20 minutos.
b.- Iniciar tratamiento faboterpico polivalente inmediatamente.
MANUAL DE TRABAJO DEL CURSO ENARM CMN SIGLO XXI
CURSO ENARM CMN SIGLO XXI TEL: 36246001 Pharmed Solutions Institute PGINA 463

c.- Si no mejora referir al paciente independientemente de la edad.
d.- Administrar analagesico IV, antibitico y gluconato de calcio.

PREGUNTA
El paciente evoluciono desfavorablemente y fue canalizado a segundo nivele donde se reportan los siguiente laboratorios 35.000
leucocitos / mm3 con predominio de leucocitos polimorfos nucleares, hemoglobina: 9 g / L y plaquetas: 150.000 / mm3. pH 7,23, pO2
40 mmHg, pCO2 40 mmHg, BE-13, HCO3 15 mmol / L, y un nivel de lactato de 10 mmol / L. Sus enzimas cardacas fueron anormales.
CPKMB fue de 103 ng / ml y cTnI fue de 6 ng / mL. Un ECG del paciente mostr elevacin del ST. Rayos X infiltrados bilaterales
sugestivos de edema pulmonar, considerando la evidencias cual es la conducta a seguir.

RESPUESTA
a.- Un frasco de faboterpico antialacrn IV.
b.- Dos frascos, valorar repetir la dosis a los 40 minutos, de acuerdo con la evolucin.
c.- Tres frascos, valorar repetir la dosis, de acuerdo con la evolucin.
d.- Cuatro frascos, valorar repetir la dosis a los 40 minutos de acuerdo a la evolucin.

MORDEDURAS. CIENCIAS BASICAS: MORDEDURA POR SERPIENTES VENENOSAS: Se denomina accidente ofdico al cuadro txico
desencadenado por la inoculacin de veneno a travs de la mordedura de serpientes que poseen esta propiedad. Las serpientes
venenosas, poseen una cabeza triangular con foseta real (rgano termosensible que detecta radiaciones de calor producidas por sus
presas) ubicado en el hocico entre la nariz y ambos ojos, y pupilas que se verticalizan con la luz. Cuerpo cubierto por plaquetas
escamadas que al tacto le imprimen rugosidad, con dibujos de distintas configuraciones y de colores poco llamativos. La distribucin de
los sitios anatmicos de las mordeduras de serpientes fue la siguiente: 72% en pies y tobillos, 14% en muslos, 13% en manos, 1% en
cabeza. SALUD PUBLICA: Aproximadamente se considera 15% de las 3 mil especies de serpientes se consideran peligrosas en el mundo.
En Mxico del total de las serpientes, el 81.36% son inofensivas y el 18.64% son venenosas. En Mxico hay dos especies consideradas
como venenosas: Viperidae: accidentes ofdicos 90%. Elapidae. El grupo etreo con mayor riesgo de afeccin es entre los 15-44 aos de
edad (48.75%). Las especies ms frecuentemente asociadas a mordedura son: Cascabel (44.9%), Nauyaca (42.8%), Corales (4%), otras
especies (3.6%), se desconoce (5.1%). PATOGENIA: Mecanismos de accin de los venenos, las toxinas de los principales grupos de
serpientes se clasifican: Hemotoxinas (tipificados por la serpientes cascabel sudafribana (Crotalus terrificus), neurotoxinas (como los de
la serpiente de coral (Elapidae - Micrurus), miotoxinas, nefrotoxinas, necrotoxinas. Locales; destruccin intensa de tejido, dolor intenso
eritema, edema que comprometen la extremidad afectada. DIAGNOSTICO: Los signos y sntomas locales: Huellas de la mordedura,
dolor, edema, incapacidad funcional, equimosis, sangrado, vesculas, necrosis local y parestesias regionales. Sntomas sistmicos:
nuseas, vmito, taquicardia, dolor abdominal y torcico, somnolencia, prdida de la conciencia, hematuria, hipotensin y estado de
choque. Hematolgicos: lisis del eritrocito, Hb, Hto, plaquetas, TP, TPT, tiempo de sangrado, hematuria, melena, hematemesis,
epistaxis, hemoptisis. Renales: glomerulonefritis, endarteriris proliferativa progresiva, necrosis cortical, insuficiencia renal aguda.
Neuromuscular: hormigueo, salivacin, parestesias, debilidad, reflejos anormales, depresin del SNC (disnea, parlisis bulbar (4-7h),
parlisis difusa (1-2h). Los signos y sntomas evolucionan
rpidamente y pueden ocasionar paro respiratorio.
Pulmonar, edema pulmonar. El diagnstico definitivo de la
intoxicacin por veneno de serpiente requiere la
identificacin de las caractersticas morfolgicas de la
serpiente venenosa y la correlacin de las manifestaciones
clnicas de envenenamiento. Los signos de
envenenamiento pueden variar dependiendo la cantidad
de veneno inoculado, tiempo de evolucin, regin afectada
y nmero de mordidas. El 20% de las mordidas se reportan
como fras o secas, es decir, no existe envenenamiento. Es
recomendable siempre tomar cultivos de la lesin por el
riesgo de infecciones agregadas, incluyendo un cultivo para
clostridium. Adems se deben monitorizar: BH, cuenta de
reticulocitos, EGO, Na, K, Cl, Ca, urea, Cr, gasometra
arterial, Tiempo de sangrado, TP, TPT, fibringeno,
plaquetas. CLASIFICACION: Las manifestaciones clnicas y el
grado de envenenamiento por mordedura de serpiente
venenosa se clasifican dependiendo del nivel del edema y
de los siguientes datos agregados, de acuerdo a la
clasificacin de Christopher-Roding para la familia Vipiridae y para la familia Micrurus. Ver cuadro 1 y 2. TRATAMIENTO: Traslado
inmediato a clnica ms cercana. Todos los nios se internan mnimo 24 h. Agua y jabn, vendaje seco y estril. No antibiticos
profilcticos. Elevacin de la extremidad para disminuir edema e inmovilizarla. Si se considera peligro de provocar reaccin alrgica
administrar difenidramina 1 mg/kg. No se recomienda aplicar succin, ya que no ha demostrado remover el veneno. Actualmente, est
contraindicado el uso de torniquetes, ya que complica la circulacin linftica del miembro afectado, condicionando una mayor
morbilidad. Si se dispone de bandas de presin se pueden utilizar en el miembro afectado a una presin de 50-70mmHg, inmovilizando
el miembro y aplicando vendaje para disminuir la perfusin del veneno. Evaluar desde el principio el permetro del miembro afectado.
Esta accin debe registrarse cada 15-20min, para evaluar progresin, ya que hay riesgo de desarrollar sndrome compartamental. Se
debe estabilizar al paciente y administrar analgsico para el dolor. En pacientes mordidos por serpiente Vipiridae se recomienda el uso
de opioides en lugar de AINES, debido al riesgo asociado de desarrollar coagulopatia o trombocitopenia. Tratamiento especfico: la
administracin de faboterapicos ha demostrado una disminucin del tiempo de hospitalizacin y recuperacin. Se recomienda que sea
MANUAL DE TRABAJO DEL CURSO ENARM CMN SIGLO XXI
CURSO ENARM CMN SIGLO XXI TEL: 36246001 Pharmed Solutions Institute PGINA 464

de acuerdo al grado de envenenamiento, segn la clasificacin de Christopher-Rodning, para nios y adultos, y grado de
envenenamiento para la familia Micrurus. Personas hipersensibles al antiveneno que presenten nauseas, vomito, choque anafilctico
administrar: Adrenalina 1x 1000 aplicado 0.5 ml cada 15 minutos segn requiera. Oxgeno, esteroides. Gluconato de calcio: No.
COMPLICACIONES EN NIOS MAS FRECUENTES: Necrosis tisular, coagulacin intravascular diseminada, sndrome compartamental.
Administracin de faboterpico en base a grado de envenenamiento de acuerdo a clasificacin de Christopher y Rodning para nios y adultos (vipiridae)
GRADO TRATAMIENTO
0 No requiere faboterpico
I Aplicar 4 frascos, va IV. Durante la primera hora se puede diluir en 100-250ml de sol. Salina 0.9%. Observar 12-24h. Si an no se ha controlado. Aplicar 4
frascos diluidos en 100-250ml de sol. Salina 0.9%. Pasar en infusin continua durante 1 hora. Valorar estado de paciente
II Aplicar 5 frascos va IV, diluidos en 100-250ml de sol. Salina 0.9% durante la primera hora. Observar 12-24h. Si no hay control, aplicar 10 frascos via IV
diluidos en 100-250ml de sol. Salina 0.9% durante 1 h, cada 4-6 hrs, hasta completar tratamiento de 18 h.
III Aplicar 6-8 frascos va IV diluidos en 100-250ml de sol. Salina 0.9% va IV, durante la primera hora. Observar 12-24h. Si no hay control, aplicar 6-8 frascos
IV, diluidos en 100-250ml de sol. Salina durante una hora, cada 4-6h, hasta completar tratamiento de 18 h.
IV Aplicar 25 frascos va IV, diluido en 100-250ml de sol. Salina 0.9% durante la primera hora. Observar 12-24h. Si no hay control, aplicar 6-8 frascos via IV
diluidos en 100-250ml de sol. Salina 0.9% durante una hora, cada 4-6 h, hasta completar tratamiento de 18 h.
No olvidar considerar la cantidad de lquidos administrados junto con los faboterapicos en la evaluacin de balance hdrico total.
Administracin de faboterpico en base al grado de envenenamiento por mordedura de serpiente Micrurus en nios y adultos
GRADO TRATAMIENTO
LEVE Aplicar 2 frascos va IV, diluidos en 100mlde sol salina en la primera hora. Si no hay control, continuar con 2 frascos la si guiente hora. Si hay control
mantener en observacin de urgencias por 12-24h
MODERADO Aplicar 5 frascos via IV, diluidos en 100ml de sol. Salina durante la primera hora. Si no hay control, continuar con 2 frascos la siguiente hora. Si hay
control observar 12-24h.
SEVERO Apliacr 8 frascos via IV, diluidos en 100ml de sol. Salina durante la primera hora. Si no hay control, continuar con 2 frascos en la siguiente hora. Si hay
control observacin en uregencias por 12-24h

CASO CLINICO
Nia de 2 aos que acudi a urgencias por mordedura de serpiente en el malolo tibial derecho, ocurrida haca 90min. Segn la familia,
la serpiente, de color grisceo, meda aproximadamente 30cm. En el momento del ingreso presentaba edema en el dorso de pie con
extensin hasta el tobillo, calor local y pulso pedio y tibial posterior presentes, con dolor a la palpacin y movilizacin. Se apreciaban
dos pequeas incisiones puntiformes separadas 1cm entre s en malolo tibial.

PREGUNTA
Cual es la conducta a seguir.

RESPUESTA
a.- Vigilar posible sndrome compartimental
b.- Corticoide, analgsico y antibitico.
c.- Vigilancia, vendaje compresivo y administracin de suero antiviperino.
d.- Corticoide, analgsico, gluconato calcio, suero anticrotalico.

MORDEDURAS DE PERRO (RABIA). CIENCIAS BASICAS: La rabia es una zoonosis de los mamferos causada por el Lyssavirus (virus de la
rabia), y se transmite al hombre principalmente por la saliva de animales infectados, a partir de una mordedura, rasguo o una
lamedura sobre mucosa o piel, con solucin de continuidad. SALUD PUBLICA: El virus est distribuido en todo el mundo. Los perros
constituyen el 54% de los trasmisores. Animales silvestres (42%). Murcilago (4%). Es un problema de salud pblica por cuanto conlleva
una mortalidad de 100%. La rabia se presenta en dos modalidades, considerndose la rabia "urbana" cuando el reservorio
predominante es el perro, y "silvestre", si los reservorios son mamferos, especialmente los quirpteros (murcilagos hematfagos).
PATOGENIA: Etiologa, el virus pertenece a la familia Rhabdoviridae. Gnero Lyssavirus. La protena G es el principal componente
antignico y especfico de grupo. Peplmeros es el otro antgeno viral importante, localizado en la membrana y origina la formacin Ab.
La cpside viral est formada por lipoprotenas. Ingresa al organismo por herida o mucosas expuestas a secreciones que contienen virus
vivo. Una primera replicacin en el sitio de la herida en las clulas musculares. Infecta los nervios sensoriales y motores con una
diseminacin centripeta hacia nervios perifricos a travs de axones y clulas de Schwann. A una razn de unos 3mm/hr hasta alcanzar
el SNC. Posteriormente la segunda replicacin se realiza en el sistema lmbico, una nueva replicacin en la neocorteza. La diseminacin
a travs de nervios perifricos hacia otros rganos o tejidos. La respuesta inmunitaria es insuficiente para prevenir la enfermedad, ya
que por su localizacin hace poco accesible a la accin de los mecanismos de defensa. Periodo de incubacin: 95% es inferior al ao. (1
-3 meses). DIAGNOSTICO: En el periodo prodrmico que dura 1-2 das (fase melanclica), hay fiebre, cefalea, anorexia, fatiga, nusea,
vmito. Alteraciones psquicas: ansiedad, depresin, irritabilidad, agitacin, nerviosismo, aprensin, temor. En el periodo de excitacin
que dura de 1-3 das: hiperactividad, excitacin, desorientacin, hiperreflexia, alucinaciones, convulsiones, salivacin y espasmos
laringeos y farngeos. En el periodo paralitico que es una etapa breve 1 da: disminucin de los fenmenos de excitacin, parlisis de
extremidades y pares craneales, alteracin del SNA (lagrimeo, salivacin), parlisis muscular generalizada.(ascendente). Anatoma
patolgica: La lesin patognomnica de la rabia son los corpsculos de Negri. Son inclusiones citoplasmticas eosinfilicas presentes en
el asta de Ammon, corteza y tronco enceflico, clulas de Purkinje. Diagnstico definitivo: Aislamiento del virus: Saliva, LCR, tejido
(cerebro). Demostracin serolgica. Demostracin del antgeno vrico. TRATAMIENTO: Vacunas antirrbicas: obtenidas por cultivo de
clulas diploides (HDCV), clulas VERO o fibroblastos de embrin de pollo (PCEC). Aplicacin, por va intramuscular, de 3 dosis los das 0,
7 y 21 o 28, en la regin deltoidea. La primera dosis se cuenta como da 0. Cada dosis es de: en casos de vacuna HDCV, 1 ml, en el caso
de vacuna VERO, 0.5 ml, en el caso de vacuna PCEC, 1 ml. Esquema de vacunacin antirrbica en EXPOSICION LEVE: Una dosis de
vacuna, en los das 0, 3, 7, 14 y 28 (30), por va intramuscular en la regin deltoidea en adultos, y en nios pequeos tambin puede
aplicarse en la cara anterolateral externa del muslo. En exposicin grave: inmunoglobulina y la vacuna antirrbica humana, de
preferencia el da cero. La aplicacin de inmunoglobulina antirrbica humana, debe hacerse lo ms cercano al da de la agresin, de no
ser as no importa el intervalo transcurrido, administrndose en nios y adultos como sigue: Inyectar inmunoglobulina antirrbica
MANUAL DE TRABAJO DEL CURSO ENARM CMN SIGLO XXI
CURSO ENARM CMN SIGLO XXI TEL: 36246001 Pharmed Solutions Institute PGINA 465

humana, a razn de 20 U.I/ kg, como dosis nica. Infiltrar alrededor de la herida, si sta lo permite por su localizacin y extensin la
mitad de la dosis total que requiere el paciente. Aplicar el resto por va intramuscular. PRONSTICO: Es muy grave. Iniciando la
sintomatologa su letalidad es el 100%.

CASO CLINICO
Femenino de 15 aos de edad que acude a la consulta por haber sufrido hace cuatro horas una mordedura del perro de su hermana.
Entre los antecedentes personales de la paciente destaca que est correctamente vacunada del ttanos (ltima dosis hace seis aos).
En cuanto a los antecedentes del animal, no ha presentado ninguna enfermedad relevante y est bien vacunado. En la anamnesis la
paciente nicamente se queja de dolor en la zona de la mordedura. A la exploracin se observan dos heridas poco profundas de unos 3
mm de dimetro en la cara dorsal del antebrazo derecho. El sangrado por las heridas es escaso. No se observan alteraciones en ninguno
de los siguientes datos: sensibilidad y movilidad del antebrazo y de los dedos de la mano; pulsos, coloracin y temperatura del
antebrazo.

PREGUNTA
Tras realizar la cura local de la herida, cual es la conducta a seguir mas adecuada?

RESPUESTA
a.- Profilaxis con antibiticos tpicos.
b.- Profilaxis antibitica sistemica.
c.- Profilaxis antitetnica nicamente.
d.- Profilaxis inmunoglobulina antirrbica.

INTOXICACIONES. CIENCIAS BASICAS: Los agentes que ms afectan a los nios son: medicamentos, sustancias de uso en el hogar
(productos de limpieza, quitagrasa y plaguicidas), hongos y plantas (tienen que ver con costumbres de cada poblacin, medicina
tradicional). Los grupos ms afectados son los lactantes y adolescentes. Los AINES y la cabamazepina desplazaron al AAS y a los
barbitricos como agentes comunes de intoxicaciones de nios. PATOGENIA: El mecanismo de accin es accidental y por intento
suicida. Factores predisponentes son: espacios reducidos, abundancia de txicos en casa, mal almacenados y al alcance, muy
importante es la ausencia fsica o emocional de ambos padres y todo esto se traduce como descuido y/o falta de atencin para los
nios y adolescentes. TRATAMIENTO: Para el abordaje del nio intoxicado, se toma en cuenta como en cualquier toro padecimiento, el
estado de gravedad (ATLS, ACLS), antes que el toxico responsable y posteriormente se busca evitar mayor absorcin
(descontaminacin) y al final revertir los efectos del toxico. Primero Estabilizar las constantes vitales: Aire, ventilacin, circulacin,
deterioro neurolgico, eutermia, mantener vigilancia estrecha de posibles alteraciones metablicas (hipoglucemia, desequilibrio acido-
base, hipoxemia, flujo urinario). Evitar mayor absorcin: 1. Va area: retirar al paciente de la zona contaminada y aplicar oxgeno a
100%. 2. Va cutnea: lavado generoso y cuidadoso, con agua y jabn, el que hace lavado con proteccin mnimo guantes y cubre bocas.
3. Va digestiva: la va de entrada ms comn. En la ingestin de custicos o derivados del petrleo, no usar medidas heroicas, el dao
se establece antes de tener opcin de neutralizar, la accin del lcali o acido; amen de la reaccin exotrmica que se presenta,
incrementa el dao. Situacin semejante en el caso de hidrocarburos, sera preferible la neumonitis qumica que el riesgo de
broncoaspiracin con sus consecuencias. Despus de 4h es menos probable encontrar el toxico en estmago y entonces el
procedimiento debe cambiar. Vaciamiento gstrico (primeras 4 h de ingestin). Emesis: si se provoca lo ideal sera disponer de jarabe
de ipecacuana en todos los hogares y administra 10ml en lactantes y 15mlen escolares; pero si se tiene jabn de tocador y de acuerdo a
la edad, lo ms rpido posible dar 1-2 cucharadas de ralladura del jabn + 1-2 vasos de agua, el vmito se presenta en 10-20 min
(recordar que no debe hacerse en ingestin de custicos, ni hidrocarburos). Lavado gstrico: para mayores de 6 meses, recambios de
50-100ml, hasta obtener liquido claro, antdoto local: en ocasiones al inicio o al final agregar 1g/kg/ de carbn activado. Gastrodialisis:
as se denomina, al uso repetido de carbn activado+ catrtico (carbn activado 0.5g/kg + Manitol al 20%, 2ml/kg) administrado por
sonda nasogstrica c/6h, est contraindicado en ingestin de custicos, hidrocarburos, leo paralitico, coma, sangrado digestivo.
Catrticos: acelerar el trnsito intestinal es un procedimiento que ocasionalmente se emplea ya que es sustituido por la gastrodilisis.
Por ultimo revertir los efectos del toxico (antdotos): Azul de metileno (1mg/kg dosis diluir en 50ml sol. Salina 0.9%, pasar en 1 h), cido
ascrbico (1g dosis c/8h, diluir en 50-100ml para 1 h), Atropina (0.5mg dosis), Fisostigmina (0.02-0.06mg/kg dosis, diluir en 10ml de sol.
Inyectable, pasar muy lento), Naloxona (10/kg/infusin continua), Flumazenil (0.1mg/dosis IV directo, se puede repetir cada 5min),
Pralidozina (30/50 mg/kg diluir en 10ml sol. Glucosada 5%, pasar en 1 h) Atropina+ pralidozina (combinacin usada en intoxicacin por
organofosforados), N-acetilcistena (inicial 140mg/kg-sostn 70mg/kg cada 4h por sonda nasogstrica). Quelantes: D-penicilamina
(50mg/kg dia VO repartir en 3 dosis por 10 dias cada 10 dias. INTOXICACION POR AINES: Tienen en comn ser inhibidores de la sntesis
de prostaglandinas por su accin sobre la enzima ciclooxigenasa. Se emplean como analgsicos, antiinflamatorios, existe gran
disponibilidad por lo que son comunes en las intoxicaciones. El directamente involucrado es el naproxeno. Comparten en mayor o
menor grado efectos txicos y afectan aparatos como digestivo (nausea, vomito, dolor abdominal y sangrado), hematolgico (discrasias
sanguneas, anemias, leucopenia, agranulocitosis), cardiolgico (congestin por retencin de lquidos), neurolgico (debilidad,
confusin, somnolencia, insomnio, visin borrosa, convulsiones), renal (retencin hdrica, de sodio, potasio, falla renal), ginecolgicas
(disfuncin uterina, sangrado). Tratamiento: El manejo inicial antes mencionado. Administrar soluciones: 1500ml/m2 para mantener
hidratacin, corregir desequilibrio H-E sol. Salina 0.9% (30ml/kg/dosis). Descontaminacin vaciamiento gstrico, para evitar absorcin,
dosis repetidas de carbn activado ms catrtico manitol al 20%. INTOXICACION POR CAUSTICOS: Especialmente lo lcalis como el
hidrxido de Na (sosa caustica, leja), es el toxico ms peligroso que pueda existir en el hogar. Es conocido que el tiempo que tarda en
producir el dao es de 10 seg y que produce necrosis con licuefaccin de las protenas, afectacin vascular del tejido quemado que
finalmente ocasiona isquemia con dao celular extenso, as la lesin es en profundidad con riesgo de perforacin. Cuando la lesin
involucra todas las paredes del esfago, la cicatrizacin produce estenosis con disfuncin esofgica que en el mejor de los casos la
obstruccin se resuelve con trasposicin de colon pero nunca volver a tener funcin normal. Tratamiento: esteroides, antibitico,
estudio radiolgicos, endoscopia. INTOXICACION POR ORGANOSFOSFORADOS: Grupo de plaguicidas de empleo frecuente en los
MANUAL DE TRABAJO DEL CURSO ENARM CMN SIGLO XXI
CURSO ENARM CMN SIGLO XXI TEL: 36246001 Pharmed Solutions Institute PGINA 466

hogares, se encuentran en varios insecticidas de uso corriente. Actan bloqueando en forma irreversible la enzima acetilcolinesterasa
que es la encargada de catalizar la acetilcolina, romper la molcula en acetato y colina para su reutilizacin por las terminaciones
nerviosas (axones). La presencia de cantidades importantes de acetilcolina en las terminaciones nerviosas colinrgicas hacen que se
presenten 3 sndromes; sndrome muscarinico (broncorrea, sialorrea, epifora, despee rectal y vesical), sndrome nicotnico (temblores,
fasciculaciones, calambres abdominales, debilidad), sndrome neurolgico (crisis convulsivas, miosis, arreflexia, como). INTOXICACION
POR PARACETAMOL: El paracetamol o acetaminofn es un derivado del paraaminofenol, tienen efecto antipirtico y analgsico, pero
sin apenas efecto aniinflamatorio. La absorcin digestiva (oral, rectal), es rpida consiguindose efectos teraputicos y efecto clnico
entre 30minuts y 2 h, despus de una dosis terapeutica de 10-15mg/kg cada 4h (dosis mxima 90mg/kg/dia). Normalmente el 90% del
paracetamol es conjugado en el hgado a glocoronidos o sulfatos, que son eliminados en la orina, y cerca de 2% es excretado en la orina
sin cambios. Aproximadamente el 3-8% es metabolizado en hgado por el complejo enzimtico P450, por procesos de oxidacin, esta
ruta metablica crea un metabolito reactivo toxico, N-acetil, paranezoquinona imina (NAPQI), el cual es rpidamente ligado al glutatin
y desintoxicado, si este se acumula se adhiere a las membranas celulares de los hepatocitos generando la muerte celular y la
consecuencia necrosis heptica, tambin puede generar falla renal aguda. La dosis requerida para producir toxicidad vara segn la
funcin de la citocromo P450, se sugiere que con dosis mayor a 150-200mg/kg en nios, es potencialmente toxico agudo. La toxicidad
crnica se presenta si se ingiere ms de 4 gr de paracetamol al da, luego de 2-8 das. Tratamiento: Iniciar con ABC, lavado gstrico, se
recomienda administrar en cada irrigacin la cantidad de 15ml/kg en los nios. El uso de carbn activado, reduce una media de 52% la
curva de concentracin de paracetamol, siempre que se administre dentro de la primera hora post-ingesta. Nios 0.5g/kg de peso
corporal diluidos en 100ml de agua/sulfato de sodio. Menores de 12 aos: 250mg/kg de peso corporal disuelto en 200ml de agua
(catrtico). El uso de catartatico est indicado en caso de dosis mltiples de carbn activado. Antdoto por excelencia es la NAC (N
acetilcisteina), cuyo efecto reside, sobre todo, en la posibilidad de regeneracin del glutatin, su indicacin debe ser lo ms precoz
posible. Existen varias pautas de tratamiento con NAC. Va oral: la dosis recomendada es una carga inicial de 140mg/kg seguida de 17
dosis de 70mg/kg cada 4 hrs. Va intravenosa: dosis inicial de 140mg/kg en una hora. Cuatro horas despus se inician 12 dosis de
mantenimiento de 70 mg/kg en una hora cada 4 hrs.

CASO CLINICO
Se trata de una nia de once aos que presentaba: boca seca, confusin, habla incoherente, incapacidad para reconocer a los
miembros de la familia, que tambin se presenta vmito incontrolable, trastornos visuales, auditivos y alucinaciones visuales. El
examen clnico revel alteracin de la conciencia, coma la escala de Glasgow (GCS) se evalu a 13/15, las pupilas eran iguales y
reactivas, present polipnea a 26 ciclos por minuto, pero estaba afebril y hemodinmicamente estable, con ictericia mucocutnea,
reflejos tendinosos eran agudos y difusos. Recuento de sangre de rutina completo, pruebas de funcin renal y heptica revel citolisis
heptica.

PREGUNTA
Cual es la conducta a seguir.

RESPUESTA
a.- Realizar lavado gstrico.
b.- Administrar diazepam 5 mg.
c.- Administrar anticolinergico.
d.- Administrar fisostigmina.

CASO CLINICO
Nio de 8 aos de edad, en urgencias 2 horas despus de la ingestin de una cantidad desconocida de un lquido. Los familiares refieren
que encontraron un frasco abierto, el padre se dedica al campo, el paciente estaba asintomtico, y los resultados del examen fsico
fueron normales.

PREGUNTA
Cual es la conducta a seguir.

RESPUESTA
a.- Administrar carbn activado.
b.- Mantener en vigilancia.
c.- Enviar a casa y citar al dia siguiente.
d.- Enviar a segundo nivel.

PREGUNTA
El paciente recibi 2 dosis de carbn activado y observacin durante la noche. Fue dado de alta con un cita. En casa, el paciente
present fatiga generalizada, ingesta oral nula, dolor abdominal, hematemesis, y disminucin de la diuresis. Sus signos vitales eran
taquicardia leve, y un examen fsico revel sequedad de las membranas mucosas y el eritema farngeo moderado que estaban ausentes
durante la evaluacin inicial.

PREGUNTA
Cual es las siguientes substancias es la ms probable que se encuentre involucrada.

RESPUESTA
a.- Organofosforados.
MANUAL DE TRABAJO DEL CURSO ENARM CMN SIGLO XXI
CURSO ENARM CMN SIGLO XXI TEL: 36246001 Pharmed Solutions Institute PGINA 467

b.- Carbamatos.
c.- Organoclorados.
d.- Paraquat.

TRAUMATISMO CRANEOENCEFALICO (TCE). CIENCIAS BASICAS: Lesiones del crneo y su contenido, provocadas por el contacto
violento de un agente fsico contra la cabeza o por el choque de la misma contra una superficie ms o menos dura, o por lo
movimientos de aceleracin y desaceleracin sbitos que en forma directa o que por sus complicaciones, puede llegar a producir la
muerte, o dejar como secuelas grados variables de invalidez. Son frecuentes en pediatra debido a las caractersticas psicomotoras del
nio: inquietud, inexperiencia e inters de exploracin, la desproporcin de la cabeza con el cuerpo y su menor talla en relacin con el
adulto son condiciones que lo hacen ms vulnerable a accidentes. SALUD PUBLICA: Primera causa de hospitalizacin en ms de 30% de
los casos, implica lesiones de crneo y su contenido. Son ms frecuentes en el sexo masculino por personalidad y caractersticas de
conducta. La etiologa ms frecuente en nios menores de dos aos son las cadas (traumatismo leves desde la cama, de una mesa o al
iniciar la deambulacin. En nios menores de un ao con TCE grave debe sospecharse maltrato. En nios mayores e 2 aos las causas
habituales son accidentes de trafico, bicicleta o deporte. PATOGENIA: El nio tienen alta frecuencia de sufrir dao neurolgico debido a
que los huesos del crneo son ms delgados y tienen menor proteccin, hay menor mielinizacion de las fibras nerviosas y el cerebro en
desarrollo en repuesta al trauma ya que puede desencadenar mayor edema e hipertensin intracraneana. La ventaja que el lactante
tiene es que la presencia de fontanelas y la ausencia de cierre de las suturas permite un poco de mayor elasticidad (compliance)
craneal, que pudiera favorecer en algn momento la presencia de hidrocefalia si el incremento del contenido intracraneal es paulatino
y lento. Al tener contacto traumtico con la cabeza, se inicia una serie de eventos que van a determinar la extensin de la lesin, as
como su severidad y secuelas. Primarias: son el resultado directo de la energa fsica transmitida durante el impacto y cuya gravedad
depende del tipo y la cantidad de est traducindose como conmocin, contusin o laceracin cerebral. Las fuerzas producidas por
aceleracin lineal tienden a causar lesin focal, es decir, fracturas, contusin o laceracin. La aceleracin por rotacin establece en el
cerebro fuerzas de arrancamiento que alteran en forma difusa la funcin traducindose como conmocin, lesin axonal y edema.
Secundarias: se originan a partir de los procesos que incrementan los efectos de lesin primaria, son resultado de hipotensin
sistmica, hipercapnia e hipoxia, as como las alteraciones metablicas producidas por el traumatismo que dan mayor lugar a isquemia
y edema cerebral. La primera es hipoxia que se presenta inmediatamente del golpe y que puede aumentar por la obstruccin de vas
areas o por fractura, luxacin del cuello o trax. DIAGNOSTICO: Existen diferentes signos y sntomas de acuerdo al grado de lesin:
cefalea (provocada por cambios en el flujo cerebral), vomito (por hipertensin intracraneana), letargia, somnolencia, delirio, estupor o
coma (por cambios a nivel de flujo cerebral), triada de Cushing (bradicardia, irregularidades en la respiracin e hipertensin arterial
sistlica por alteracin de perfusin cerebral), alteraciones de la memoria (alteracin celular, isquemia e hipoxia), cambios de la
personalidad (alteracin a nivel de membrana celular), papiledema (edema cerebral, hipertensin intracraneana), alteraciones de pares
craneales (hipoxia, isquemia compresin en el sitio de fractura III, midriasis del lado afectado), alteracin en escala de coma de Glasgow
(edema cerebral, cambios metablicos que aumentan deterioro neurolgico), convulsiones o focalizacin (por proceso inflamatorio,
efecto de masa (hemorragia, hematoma) o metablica), anisocoria y alteracin pupilar (edema cerebral). Dependiendo de dao
cerebral y rea afectada se puede encontrar descerebracin o descorticacion, proporcional al grado de dao celular e isquemia que
haya existido. Los movimientos oculares (oculoenceflicos y oculovestibulares), evalan la regin media cerebral en el rea del sistema
reticular ascendente y la respuesta motora usando 5 niveles internacionales se usan: normal 5, debilidad moderada 4, debilidad severa
3, trazas de movimiento 2, flacidez 1. La exploracin externa de la cabeza nos puede dar datos de fractura importante de la base del
crneo la anterior: equimosis periorbitaria signo de mapache. Medio: rinorraquia, otorragia o otorraquia. Posterior: Signo de Battle,
equimosis retroauricular. La exploracin de pares craneales indican un proceso expansivo intracraneal, asimetra de la respuesta
motora de los miembros, indica distorsin de los pednculos cerebrales o bien compresin sobre el tallo cerebral. Este examen se
puede realizar en menos de 5 min y permite clasificar al paciente peditrico segn el grado de trauma. El valor de la radiografa de
crneo enel traumatismo craneal est siendo cada vez ms cuestionado, de forma que no se recomienda su uso en la mayora de
situaciones de trauma craneal si la TAC est disponible. CLASIFICACION: Trauma leve o GRADO I: Escala de coma de Glasgow 15-13
puntos, es aquel paciente que posterior al trauma no presenta ninguna manifestacin clnica, o bien, tiene una perdida transitoria de la
memoria que durara algunos minutos u horas pudiendo estar asociado a prdida del estado de conciencia en forma tambin transitoria
(segundos a minutos) y no existe por lo general alteracin en la escala de coma de Glasgow, se ha asociado a sndrome pos conmocin
donde el paciente presenta episodios de cefalea, irritabilidad y falta de concentracin, en nios ms grandes se puede asociar agresin,
ansiedad, baja atencin y cambio de conducta, se presenta meses posteriores al trauma y es autolimitado. Se ha descrito una triada en
trauma leve: somnolencia, irritabilidad y vomito; que ocurre en 48-72 h posteriores al trauma se cree que es debido a leve torsin del
tallo cerebral. Trauma moderado o GRADO II: Escala de coma de Glasgow de 12-9puntos, perdida del estado de alerta menor a 5 min,
tendencia a la somnolencia sin dficit neurolgico. Trauma severo o GRADO III: Escala de coma de Glasgow 8-3 puntos. Perdida del
estado de alerta mayor a 5 min, incapacidad para obedecer rdenes, confusin mental, lenguaje incoherente e inapropiado, anisocoria
o lenta respuesta pupilar; la respuesta motora puede variar a la localizacin del dolor o posturas anormales. Muerte enceflica o
GRADO IV. Cabe mencionar que hay ciertas condiciones en un TCE leve que puede transformarlo en moderado o severo por lo que el
examen debe ser cuidadoso y si existe deterioro rpido de la escala de coma de Glasgow manifestando un conjunto de sndrome de
deterioro rostro-caudal. TRATAMIENTO: A la llegada a urgencias de un TCE, se debe realizar una valoracin rpida y ordenada de la
situacin del paciente (ABCD). Si existe alteracin de la conciencia o se presume, por la historia o la exploracin, que el TCE puede ser
moderado o grave. Asegurar la permeabilidad de la va area. Administrar oxgeno. Monitorizar al paciente (FC, FR, TA y SAT O2).
Canalizar una va venosa perifrica. Ante la presencia de inestabilidad respiratoria y/o hemodinmica, se proceder con las maniobras
de reanimacin cardiopulmonar (RCP). En los casos con alteracin de con ciencia importante (Glasgow < 9) ser necesario asegurar la
va area mediante la intubacin orotraqueal. Objetivos del tratamiento: PIC <20mmHg , PAM (normal para edad), PPC > 50mmHg,
Sat O2 > 95% con PCO2 3540mmHg. CRIERIOS DE HOSPITALIZACION DE TCE: Alteracin de los signos vitales:
Convulsiones postraumticas. Funciones mentales alteradas. Perdida de la conciencia prolongada. Dficit de memoria persistente.
Signos neurolgicos focales. Fractura craneal deprimida. Fractura craneal basilar. Edema de piel cabelluda amplio. Cefalalgia severa
MANUAL DE TRABAJO DEL CURSO ENARM CMN SIGLO XXI
CURSO ENARM CMN SIGLO XXI TEL: 36246001 Pharmed Solutions Institute PGINA 468

persistente, especialmente con rigidez de nuca. Vmitos persistentes. Fiebre inexplicable. Anormalidades neuroradiologicas que
sugieran abuso infantil.

CASO CLINICO
Paciente escolar, masculino, blanco, de 11 aos de edad, con antecedentes de salud, que mientras jugaba en su poblado natal, fue
atropellado por un vehculo, recibiendo traumatismos mltiples a predominio craneal. Lleg al servicio de urgencias manejado por el
sistema integrado de urgencias mdicas, en estado de coma. Examen fsico al ingreso: Coma moderado, pupilas isocoricas y reactivas,
hemiparesia izquierda, herida contusa frontal medial y hacia la izquierda, con salida al exterior de tejido cerebral y lquido
cefalorraqudeo (LCR). Exmenes complementarios al ingreso: Hemoglobina: 130 g/l, tiempo de coagulacin: 7 minutos, tiempo de
sangramiento: 2 minutos. Tomografa axial computarizada (TAC) al ingreso: Se aprecia fractura del hueso frontal hacia la izquierda con
fragmentos que penetran el tejido cerebral, focos de contusin hemorrgica diseminados por regiones frontales polares de ambos
hemisferios cerebrales.

PREGUNTA
Cual es el grado de TCE que presenta el caso?

RESPUESTA
a.- Grado I.
b.- Grado II.
c.- Grado III.
d.- Grado IV.

CEFALEAS. CIENCIAS BASICAS: Es el concepto genrico para considerar el dolor de la cabeza y es el sntoma ms comn del hombre
civilizado, puede ser de la misma intensidad si su origen es benigno o maligno dependiendo de la tolerancia de cada individuo, por
tanto la cefalea representa una incapacidad relativa de la persona para manejar las incertidumbres de la vida, un sntoma que refleja
ms una alteracin subyacente del pensamiento o de la conducta, que una enfermedad del sistema nervioso y aunque es considerada
frecuentemente una manifestacin frecuentemente asociada a otros signos y sntomas que puede originarse frecuentemente asociada
a otros aparatos y sistemas. Las cefaleas ms frecuentes son las de origen orgnico. SALUD PUBLICA: La cefalea ocurre entre 26-45% de
los pacientes en alguna etapa de la vida. La migraa tiene mayor frecuencia en escolares con una relacin de 2:1 en nios comparado
con nias. CLASIFICACION: De origen orgnico: relacionada con procesos infecciosos o inflamatorios, que afectan vas areas
superiores, crneo, TCE. De origen neurolgico: migraa y cefalea vascular, por neuroinfecciones, hipertensin intracraneana de
etiologa neoplsica en menores de 6 aos, y para el grupo de escolares y adolescentes, la migraa, cefalea tensional y epilepsia.
Clasificacin International Headache Society 2004: 1. Migraa 2. Cefalea tensional 3. Cefalea en racimos y otras cefaleas trigeminales 4.
Otras cefaleas primarias 5. Cefalea atribuible a TCE 6. Cefalea atribuible a problemas vasculares 7. Cefalea asociada a trastorno IC de
origen no vascular 8. Cefalea atribuible al abuso o depravacin de sustancias 9.Cefalea atribuible a procesos infecciosos 10. Cefalea
atribuible a trastornos metablicos 11. Cefalea o dolor facial asociado a alteraciones del crneo, cuello, ojos, odos, nariz, senos,
dientes, boca u otras estructuras faciales o craneales 12. Cefalea atribuible a patologa psiquitrica 13. Neuralgias craneales 14. Cefaleas
no clasificables. De acuerdo a evolucin: agudas, subagudas y crnicas que por mecanismo directos e indirectos genera modificaciones
en la homeostasis de manera que el sntoma remite una vez resuelta la patologa que la condiciono. Clasificacin Migraa; Migraa sin
aura 60-85%. Migraa con aura 15-30%. Migraa Basilar. Migraa hemipljica familiar. Sndromes peridicos infantiles. PATOGENIA: El
dolor de cabeza se origina por estimulacin de las estructuras intracraneales y extracraneales sensibles a dolor que son: piel, tej.,
subcutneo, musculo, arterias y periostio del crneo. Estructuras del ojo, odo, dientes y cavidad nasal. Senos venosos intracraneales.
Regiones de la duramadre en la base del crneo, arterias durales interiores de la duramadre. Nervios craneales trigmino,
glosofarngeo, vago. Los 3 primeros nervios cervicales. La estimulacin de una o de varias de estas estructuras sensibles a dolor
producen cefalea a travs de uno o ms de los siguientes mecanismos: irritacin directa por inflamacin, compresin, traccin,
desplazamiento, dilatacin y distensin de las arterias. Teora vascular: es la teora ms antigua, producida por la vasodilatacin de
vasos cerebrales, que provocara una activacin de fibras nociceptivas de las arterias intracraneales. Se consideraba que la
vasoconstriccin posterior era la responsable de los sntomas que ocurren durante el aura. Teora neural: depresin propagada como
posible explicacin para el aura. Se trata de la existencia de unas ondas de excitacin que se propagaban a los largo de la corteza
cerebral a razn de 3mm por segundo. Teora neurovascular: surge ya que ninguna de las dos teoras anteriores logra explicar todos los
fenmenos de la crisis de migraa. En esta teora el desencadenante inicial sera neuronal: hipometabolismo-hipoperfusin del
hemisferio ipsilateral al dolor. Posteriormente, se producira la liberacin de algunos neuropptidos vasoactivos, como la sustancia P.
Estas sustancias liberadas pueden dar lugar a la inflamacin neurgena, induciendo vasodilatacin y posterior extravasacin de
protenas plasmticas. Papel de la protena C-fos: sta es una fosfoprotena que regula la expresin de diversos genes neuronales. La
serotonina (5- hidroxitriptamina) se ha considerado como el mediador principal en esta cascada de eventos. Se ha comprobado una
marcada deplecin de la concentracin de serotonina en plaquetas, lo que sugiere que sta fuera la causa de la migraa. Las teoras
vascular y neuronal han contribuido al conocimiento de la enfermedad, pero son incompletas. Proponen que la migraa es el resultado
de interacciones entre el cerebro y el sistema circulatorio sobre un terreno genticamente predispuesto. DIAGNOSTICO: Historia
clnica dirigida, exploracin fsica integral. Hay que recordar que un dolor de cabeza es variable en su intensidad, segn la tolerancia al
dolor que tiene cada persona, por eso no es posible conocerlo con precisin y mucho menos cuantificarlo. La diversidad de sus causas
se resume en cuadro anexo. Auras ms frecuentes: Escotoma binocular (77%). Distorsiones de los objetos (16%). Escotoma monocular
(7%). Manchas, balones, colores, arco iris, sndrome de Alicia en el pas de las maravillas.
TIPOS DE CEFALEA MANIFESTACIONES SINTOMAS ACOMPAANTES
MIGRAA Unilateral o bilateral, de inicio hemicraneal. Pulstil, incremento gradual. El ejercicio lo
incrementa. Duracion de mas de 60 min, menos de 1 semana. Disminuye en la oscuridad y el
sueo. Predomina en sexo femenino
Nauseas, vomito, fotofobia, escotomos
CEFALEA TENSIONAL Ambos sexos. Bifrontal u occipital pulsatil. Dolor opresivo o pulstil, duracin de dias o Depresin, ansiedad, estrs
MANUAL DE TRABAJO DEL CURSO ENARM CMN SIGLO XXI
CURSO ENARM CMN SIGLO XXI TEL: 36246001 Pharmed Solutions Institute PGINA 469

semanas, intermitente, asociado a fatiga y lo desencadena el estres
TUMOR CEREBRAL Unilateral o bilateral, sin predominio de sexo. Pulsatil, la despierta el dolor. Frecuencia una vez
en la vida, aparicin subita
Papiledema, vomito, crisis epilptica, paresias
NEUROINFECCIONES Cefalea universal u occipital. Sin predominio de edad o sexo, puede despertarlo, irritabilidad o
fiebre
Ataque al estado general, crisis epilptica, signos de
irritacin menngea.
CEFALEA AGUDA: Se trata de un dolor de cabeza nico de duracin corta en trminos generales, considerada menor de 2 h, y aunque
puede ser el inicio del complejo sintomtico de mltiples causas como infeccin de vas respiratorias, neuroinfecciones, ametropas,
gastroinetsinales, neoplasias del SNC, toxicomanas, HTA. CEFALEA RECURRENTE: Ocurre en forma peridica o recurrente con factores
especficos desencadenantes y caractersticas particulares, pero sin olvidarse de la patologa casi siempre relacionada con la localizacin
en la cabeza, dentro de las causas: migraa, cefalea tensional, en racimos, postraumtica, hipertensin intracraneana crnica, epilepsia,
trastorno del sueo. CEFALEA CRONICA: Generalmente cuando est relacionada con evolucin los problemas serios sistmicos bien
originados en el SN, se requiere manejo integral e investigar en forma prioritaria la causa y establecer un tratamiento especfico, no
solo sintomtico como en los casos anteriores. En el caso de que se torne crnica pero no evolucione puede deberse al uso crnico de
frmaco, que estn indicados como desencadenantes de respuesta de rebote, causas: hipertensin intracraneana, neoplasia del SNC,
epilepsia, migraa, cefalea tensional, sinusitis, disfuncin de articulacin temporopmandibular. TRATAMIENTO: Medidas generales;
basada en A) Controlando, deben evitarse los alimentos o circunstancias que
desencadena el dolor de cabeza. B) Deteniendo, automedicacin nunca ser
conveniente, debe evitarse la ingestin de frmacos para disminuir un problema
asociado que prolonga los dolores de cabezas como es la cefalea de rebote. C)
Previniendo, si se sigue una vida sana, evitando factores desencadenantes de
dolor de cabeza, no ingerir frmacos no indicados. Algunas medidas generales
han permitido disminuir la manifestacin como: 1. Aplicacin de frio en forma
de compresas de hielo, pao hmedo, alternado cada 10 min por 30 min. 2. En
ocasiones los dolores de cabeza se incrementan con la luz, por lo que es
recomendable usar lentes oscuros, cerrar las cortinas o persianas. 3. En caso de
que el dolor de cabeza se acompae de vmito, para evitar deshidratacin,
consumir abundantes lquidos. El tratamiento mdico tiene por propsito en
control del sntoma, pero tambin la causa que lo genera, de manera que
pueden considerarse para el primer grupo a) analgsicos no narcticos, b)
analgsicos narcticos, c) AINES, d) antidepresivos tricclicos. Para los casos de
migraa se divide el tratamiento farmacolgico en dos fases: Abortiva; para la
etapa aguda donde la utilidad de los triptanos sea comprobado y en menores de
6 aos de edad se recomienda el uso de zolmitriptano y solo en mayores de 6
aos se puede usar sumatriptan, eletriptan. Profilactico: se recomiendan frmacos para espaciar los cuadros agudos como pueden ser
los AINES del tipo ibuprofeno, meloxicam y en su caso bloqueadores de Cox durante periodos mximos de 6-8 semanas, pero aunado a
medidas higienicodieteticas.

CASO CLINICO
Nia de 13 aos de edad presenta cefalea intensa tras un proceso vrico inespecfico cuatro semanas antes. Lo describa como una
pesadez continua que la incapacitaba para hacer vida normal, cuya intensidad aumentaba en el transcurso del da y se atenuaba al
colocarse en decbito. Durante el da los sntomas fluctuaban, y mejoraban tras la administracin de analgsicos. Dorma mal y no tena
nimos para emprender las actividades cotidianas, de manera que que faltaba al colegio y si intentaba acudir, invariablemente tena
que ser recogida por su familia a media maana. Estaba tomando analgsicos (ibuprofeno) haca cuatro semanas, tres veces al da sin
cambios en la sintomatologa.

PREGUNTA
Cual es la conducta a seguir mas adecuada.

RESPUESTA
a.- Cambiar de medicamento a narproxen 250 cada 12 hrs.
b.- Agregar ergotamia y cafena profilctica.
c.- Enviar a evaluacin neuropediatrica.
d.- Retirar AINES y medidas generales para disminuir estimulos.

EPILEPSIAS. CIENCIAS BASICAS: Desorden del cerebro caracterizado por una predisposicin perdurable para genera crisis epilpticas
con las consecuencias neurobiolgicas, cognitivas, psicolgicas y sociales de esta condicin. De acuerdo a la OMS, la epilepsia es la
presentacin crnica, recurrente de fenmenos paroxsticos por descargas elctricas anormales en el cerebro (crisis epilpticas) que
tiene manifestaciones clnicas variadas y causas diversas. Crisis convulsiva; representa un evento de inicio brusco, generalmente
autolimitado, caracterizado por una actividad muscular excesiva, pudiendo ser clnica (prdida de la conciencia con contracciones
clnicas rtmicas de las cuatro extremidades. Posictal con confusin), tnica (contraccin muscular sostenida, duran menos de dos
minutos y tienen un periodo posictal de confusin) o mioclonica (contracciones sbitas y muy breves de grupos musculares, a veces
generalizadas, de tal forma que pueden hacer caer al sujeto. El EEG con polipuntas generalizadas). Las crisis epilpticas; son la
manifestacin clnica de una descarga anormal de una poblacin neuronal, generalmente pequea, localizadas ya sea en la corteza
cerebral o bien en la profundidad del parnquima cerebral, de forma repetida y crnica, con correspondencia electroencefalografica.
SALUD PUBLICA: Mxico su prevalencia oscila entre 1.8 a 2 % lo que representa ms de un milln de pacientes. El 76% de los
epilpticos inician su padecimiento antes de la adolescencia; por lo que la Organizacin Mundial de la salud (OMS) reconoce a la
epilepsia como un problema de salud pblica. Las epilepsias afectan aproximadamente 40-50 millones de personas alrededor del
FARMACO INDICACIONES
AAS
Paracetamol (: 10-20 mg/kg)
Ketorolaco
Todos los tipos de cefalea
Ibuprofeno
Naproxen
Ketoprofeno
Profilctico
Migrana
Cefalea tensional
Clonixinato de lisina
Aceclofenaco
Migraa, profilctico
Todas las cefaleas
Clordiacepoxido
Tramadol
Cefalea postraumtica
neoplasias
Propanolol
Metoprolol
Migraa
Prfilactico
Imipramina
Clorimipramina
Fluoxetina
Migraa
Profilactico
Cefalea tensional
Cinarizina
Flunarizina
Migraa
Profilctico
Ac. Valproico
Fenitoina
Topiramato
Migraa
Profilactico
Asociasion con epilepsia
MANUAL DE TRABAJO DEL CURSO ENARM CMN SIGLO XXI
CURSO ENARM CMN SIGLO XXI TEL: 36246001 Pharmed Solutions Institute PGINA 470

mundo. (1-2% de la poblacin mundial). Al menos 5 millones la padecen en Latinoamrica y 3 millones no reciben tratamiento.
PATOGENIA: Etiologa diversa. Causas; accidente cerebrovascular, TCE, neuro-infeccin, alteraciones metablicas, efecto de alcohol o
drogas, deprivacin de alcohol, asfixia. Hiptesis para crisis generalizadas; Hiptesis Neuroanatmica: (Gibbs y colab.) Aqu se invoca
una alteracin cortical generalizada sumado a un dismetabolismo en los neurotransmisores especialmente los inhibidores, como el
GABA. Hiptesis micromorfologica: perdida neuronal, esclerosis hipocampal y/o cortical sumado a trastornos corticales ocasionados
por alteracin de la migracin neuronal. Esta migracin neuronal anormal afectara la conduccin trasmembrana de los iones Ca+ y Na.
Anomalas funcionales: constituidas por disminucin del consumo de glucosa y del flujo circulatorio parenquimatoso; posteriormente
comunicaron disminucin del nmero de receptores benzodiazepinicos. Para las crisis focales: Cicatriz cerebral: las distintas lesiones
ocasionan una cicatriz glial que actuara como foco irritativo, que inestabiliza elctricamente las membranas celulares. Este mecanismo
excitador seria mediado por los cidos glutmico y asprtico. Disminucin de los mecanismos inhibidores gabaergicos: debido
fundamentalmente a prdida de neuronas gabaergicas en las distintas lesiones. TIPOS: Las GENERALIZADAS, representan los sntomas
derivados de la activacin de grupos de neuronas extendidas en ambos hemisferios cerebrales; se clasifican como convulsivas:
moclnicas, tnicas, clnicas y tnico-clnicas y las no convulsivas: Ausencias y atnicas. Las PARCIALES (cuando los sntomas iniciales
representan la activacin de un grupo de neuronas circunscritas a parte de un hemisferio cerebral) y pueden tener sntomas: motores o
sensoriales localizados a parte de un hemicuerpo, autonmicos o psquicos (ilusiones, alucinaciones, pensamiento forzado, etc).
DIAGNOSTICO: El diagnstico de las crisis epilpticas y de los sndromes epilpticos es de carcter clnico, examen neurolgico,
condiciones mdicas relacionadas: Retraso del desarrollo, cefalea tensional y/o migraa, trastornos psiquitricos, trastornos de sueo.
La confirmacin de los mismos se efecta a travs de los mtodos complementarios de diagnstico. Electroencefalograma. Es la prueba
de eleccin para demostrar el carcter epilptico de un paroxismo y es insustituible para definir muchos sndromes epilpticos. Los
estudios de EEG con privacin parcial de sueo, foto estimulacin y/u otras activaciones son tiles para discriminar actividad
epileptiforme no visible en el EEG convencional. Neuroimagenes. La TAC y RM son las tcnicas de eleccin para detectar lesiones
estructurales del SNC, siendo la segunda ms sensible y especfica, especialmente para el estudio de la esclerosis temporal. CRISIS
CONVULSIVAS GENERALIZADAS: Esta forma es la ms frecuente y se la conoce y denomina habitualmente como Crisis de grand mal,
bsicamente se trata de movimientos tnico-clnicos generalizados, de presentacin abrupta, con alteracin aguda de la conciencia,
que duran pocos minutos y que generalmente cursan con tres fases definidas: Fase tnica: Perdida de conocimiento brusca con cadas e
hipertona muscular generalizada. Fase clnica: Movimientos alternativos de flexo-extensin, con sacudidas rtmicas a nivel ceflico y
los cuatros miembros simultneamente Es habitual que se acompae de mordedura de lengua y labios e incontinencia urinaria. Fase
poscrtica: Recuperacin paulatina de la conciencia, con amnesia de lo ocurrido, cefalea, dolores musculares difusos; al cabo de una
hora el paciente se halla habitualmente recuperado. ESTADO EPILEPTICO: El termino se utiliza para describir cualquier tipo de crisis
continuas lo suficientemente prolongadas que pueden producir dao neuronal. La liga Internacional contra la Epilepsia define al estado
Epilptico como una crisis que no muestra datos de recuperacin de lo que durara una crisis habitual, o crisis recurrentes sin
recuperacin de alerta durante el periodo interictal, o recuperacin de la funcin basal normal del sistema nervioso. Desde el punto de
vista operativo se acepta una duracin mayor de 5 minutos como suficiente para iniciar tratamiento. CRISIS FEBRIL: Se puede
identificar cuando un nio de 6 meses a 6 aos de edad tienen como nica causa de las crisis febriles un trastorno gentico que le hace
susceptible de manifestar crisis convulsivas exclusivamente cuando se tiene hipertermia, mientras que las crisis desencadenadas por
fiebre, el paciente frecuentemente tiene una causa bien establecida de epilepsia, incluso ha tenido crisis sin fiebre y la hipertermia es
solo otro factor precipitante de las crisis. Los criterios para identificar las crisis febriles simples incluyen: inicio entre los 3 meses y los
5 aos de edad, no tienen antecedente familiar de epilepsia, no tienen patologa neurolgica definida ni datos de neuroinfeccion, no
haber presentado una crisis previa en estado afebril, tienen una duracin menor de 5 min y no presentan dficit neurolgico postictal.
Una crisis febril compleja es aquella que el inicio es parcial, su duracin es mayor de 15 min, con un periodo posictal prolongado, con
ms de 1 crisis en 24h o ms de una por episodio febril, y a la exploracin fsica muestra algn dficit neurolgico, en menores de 6
meses o en mayores de 5 aos. Tratamiento: Manejo inicial ABC, controlar fiebre con medios fsicos y/o paracetamol 10-
15mg/kg/dosis, realizar historia clnica, examen fsico y exploracin neurolgica. En caso de crisis febriles complejas agregar Valproato
de Magnesio a dosis crecientes, hasta llegar a 30mg/kg/da. CRISIS DE AUSENCIA: a. Tpica: Desconexin del medio por pocos segundos
durante los cuales el paciente se ve con la mirada perdida. Puede presentarse la ausencia simple o acompaarse de fenmenos clnicos
leves (parpadeo), automatismos (deglutir, chuparse los labios), fenmenos atnicos (cada de la cabeza), fenmenos tnicos
(contraccin de los msculos del tronco), fenmenos autonmicos (palidez, rubicundez, piloereccin) No hay periodo posictal y el
paciente recupera bruscamente el estado de alerta total. El EEG muestra descargas generalizadas de complejos punta onda lenta de 3
ciclos por segundo. b. Atpicas: Desconexin del medio un poco ms prolongada que la tpica, frecuentemente acompaada de
fenmenos tnicos y recuperacin lenta hasta la alerta total. El EEG muestra complejos de punta o polipunta onda lenta de 2 2.5
ciclos por segundo. TRATAMIENTO ANTIEPILEPTICO: Crisis primariamente generalizadas convulsivas: Fenitoina (impregnacin de
18mg/kg/dosis lentamente y vigilando la funcin cardiovascular y mantener con dosis de 5-7mg/kg/da, dividido en 2-3 dosis),
Fenobarbital (impregnacin 20mg/kg/dosis y mantener en dosis 3-5mg/kg/da dividido en 1-2 dosis) o Valproato de Magnesio en dosis
de 15mg/kg/dosis, seguido de 25-50mg/kg/da. Crisis parciales simples, complejas o secundariamente generalizadas: Se prefiere utilizar
Valproato de Magnesio o Carbamacepina a dosis de 15-20mg/kg/da divido en 2-3 tomas al da. No convulsivas. Ausencias Tpicas y
atpicas, primera eleccin valproato de magnesio (30-60mg/kg/da), Etosuximida. 20-30 mg/kilo/da, fraccionada c / 8 h. Se debe evitar
la administracin de benzodiacepinas cuando el evento convulsivo ha terminado, ante la imposibilidad de evaluar el estado mental y
modificar la exploracin neurolgica. En el caso de una crisis febril o afebril prolongada se podr administrar diacepam IV a dosis de
0.3mg/kg/dosis (mximo 10 mg total) o a dosis de 0.5mg/kg/dosis y evaluar el tratarlo como estado epilptico. Sndrome de West +
Esclerosis tuberosa; vigabatrina. Sndrome de Lennox-Gastaut; topiramato, valproato de magnesio. Epilepsia de ausencias infantiles y
juveniles; valproato de magnesio (30-60mg/kg/da), lamotrigina. Epilepsia mioclnica juvenil; valproato de mg. Crisis convulsiva fase
ictal: Medicamentos primera lnea: benzodiacepinas Lorazepan, diazepam (60-80%). CRITERIOS DE HOSPITALIZACION: < 1 ao,
Glasgow < 15, datos de hipertensin endocraneana, meningismo, crisis dura ms de 15 minutos, recurrencia de las crisis convulsivas 12
horas. Lamotrigina: primera lnea, atnicas, tnicas, ausencias, tnico clnicas generalizadas, crisis parciales. Segunda lnea en
mioclonicas. Topiramato: primera lnea, crisis tnico clnicas generalizadas, mioclnicas, parciales con generalizacin. Segunda lnea,
ausencias, tnicas, atnicas. Vigabatrina: crisis parciales con eficacia de 50%
MANUAL DE TRABAJO DEL CURSO ENARM CMN SIGLO XXI
CURSO ENARM CMN SIGLO XXI TEL: 36246001 Pharmed Solutions Institute PGINA 471

CASO CLINICO
Varn de 15 aos, ingres tras sufrir un episodio de parestesias en el hemicuerpo izquierdo, junto con desviacin de la comisura bucal
hacia la izquierda, seguido de cefalea frontal intensa y pulstil, nuseas y vmitos. Antecedentes: producto de un embarazo y parto
normales con un desarrollo psicomotor y del lenguaje adecuados, con trastorno del aprendizaje. No present antecedentes familiares
de inters excepto que su abuelo (materno) sufra de migraas. A los 4 aos, tras un traumatismo craneoenceflico leve, sufri un
cuadro de inestabilidad y somnolencia de varias horas de duracin. A los 5 aos present en vigilia, de forma sbita, una hemiparesia
aguda izquierda asociada a disartria y confusin, que cedi en 10 minutos. El examen fsico y neurolgico completo evidenci, una leve
dismetra del miembro superior izquierdo.

PREGUNTA
Cul de los siguientes diagnosticos es el ms probable.

RESPUESTA
a.- Crisis parciales simples.
b.- Crisis parciales complejas.
c.- Crisis generalizadas convulsivas.
d.- Crisis generalizadas no convulsivas.

PREGUNTA
Cual es el auxiliar diagnostico mas adecuado para idenficar la causa.

RESULTADO
a.- MVEEG.
b.- EEG.
c.- IRM.
d.- TAC.

PREGUNTA
Cual es la conducta teraputica mas adecuada para el caso.

RESPUESTA
a.- Carbamazepina 10 a 20 mg/kg.
b.- Lamotrigina 200 a 400 mg/da administrados en dos dosis.
c.- Valproato de magnesio 60 mg/kg/da.
d.- Topiramato 100 mg/dia.

ENURESIS. CIENCIAS BASICAS: La enuresis pertenece al grupo de trastornos de la eliminacin siendo definido por el DSM-IVTR como el
repetido vaciado de orina en la ropa o en la cama, tanto si el vaciamiento es involuntario como intencionado. Esta conducta debe
ocurrir dos veces por semana durante 3 meses por lo menos o que genere un malestar clnico significativo o altere el funcionamiento
social o acadmico del nio. La enuresis es un problema evolutivo de la miccin. La definicin ms aceptada por la comunidad cientfica
establece que la enuresis consiste en la emisin involuntaria y persistente de orina durante el da o la noche, despus de una edad en
la que el nio ya debera haber aprendido a controlar la miccin (5 aos) y no existen indicios de patologa. Se manifiesta si el nio no
ha aprendido a evacuar voluntariamente la orina en los lugares adecuados, es decir, si la miccin no se realiza bajo control del
individuo. SALUD PUBLICA: El 15% aproximadamente de la poblacin infantil mayor de 5--6 aos tiene enuresis primaria. La incidencia
familiar es alta. Un 44% tienen un padre que fue enurtico en la niez y si ambos padres lo fueron el nio tiene hasta un 77% de
probabilidades de tener enuresis. TIPOS: Enuresis primaria: el nio no ha logrado control suficiente y puede encontrarse en cualquier
estadio del aprendizaje del control voluntario de la miccin. Enuresis secundaria: se trata de casos en los que despus de un amplio
periodo de control (ms de 6 meses) el nio recae, en ocasiones, asociado a un accidente, a periodos de hospitalizacin largos, a
nacimiento de hermanos o a prdidas familiares. La enuresis funcional (no organica) es el tipo ms comn (ms del 90%) y es aquella en
la que los nios no presentan ningn otro problema bien sea de origen congnito orgnico, infeccioso o traumtico. El nio no ha
alcanzado todava control sobre la miccin pero este problema no puede ser achacable a ninguna otra causa. Su desarrollo general
(motor y verbal) es normal y aparentemente slo muestra dificultades en el aprendizaje del control de la miccin. En los otros casos se
habla de enuresis orgnica bien sea de tipo congnito o traumtica. La distincin entre enuresis diurna y nocturna es obvia, sin
embargo, su tratamiento plantea problemas diferentes. PATOGENIA: Decimos que es un problema evolutivo porque todos nacemos
con el reflejo de miccin, de forma que cuando la vejiga est llena se relajan los esfnteres y se libera la orina, pero en el curso del
desarrollo aprendemos a controlar el reflejo y a orinar en el lugar apropiado. El momento a partir del cual controlamos la miccin es
muy variable y por tanto el diagnstico de enuresis se establece arbitrariamente a partir de los cinco aos. Teoras: Una de ellas es, que
el sueo especialmente profundo de los nios que mojan la cama impide que las contracciones de los msculos de la vejiga despierten
al nio antes de iniciar la miccin. Sin embargo, los estudios electroencefalogrficos, los escneres y tomografas cerebrales no han
podido constatar diferencias que pudieran justificar la enuresis de algunos casos aunque algunos resultados de investigacin apuntan a
que los enurticos presentan un umbral para despertar ms elevado que los no enurticos. Tambin asociado al sueo se ha sealado
la posibilidad de que las contracciones vesicales fueran demasiado dbiles para provocar el despertar, al mismo tiempo que se ha
propuesto que durante el sueo profundo el nivel de contraccin de los esfnteres era deficiente. Se ha sugerido tambin que los casos
de enuresis presentan una capacidad funcional de vejiga disminuida y, por tanto, no pueden aguantar toda la noche sin evacuar la
orina. La teora comnmente mantenida respecto a la enuresis desde una perspectiva psicolgica es que sta es debida a un dficit de
aprendizaje. El control de la miccin diurno y, especialmente, nocturno es una habilidad compleja que comporta muchos pasos y
MANUAL DE TRABAJO DEL CURSO ENARM CMN SIGLO XXI
CURSO ENARM CMN SIGLO XXI TEL: 36246001 Pharmed Solutions Institute PGINA 472

requiere un correcto entrenamiento, lo que no quiere decir que haya que ocuparse activamente de que el nio aprenda sino que hay
que permitir que se den las condiciones para el aprendizaje. DIAGNOSTICO: Por denticin, el nio enurtico debe tener examen fsico
normal de vas excretorias, tanto en sus aspectos anatmicos como funcionales. Para hacer el diagnstico en nios con enuresis es
necesario tener en cuenta historia clnica detallada, examen fsico y estudios paraclnicos. Es indispensable que se investigue como fue
el entrenamiento del esfnter vesical y anal. Es necesario evaluar la situacin familiar, identificando la funcionalidad, eventos
relevantes, situacin escolar, dinmica con hermanos y compaeros de escuela. Estudios bsicos: urocultivo, EGO, qumica sangunea,
ultrasonido abdominal (vas urinarias). TRATAMIENTO: El primer paso es la educacin y apoyo a los padres, puntualizando en la
paciencia. Siempre ha que descartar causas orgnicas, de presentarse pasan a primer trmino. Las alternativas teraputicas son varias
de las que se pueden emplear las siguientes: modificacin conductual, entrenamiento de la capacidad vesical, distribucin de la ingesta
de lquidos (aumentar consumo de lquidos durante el da; 40% en la maana, 40% por la tarde y 20% al anochecer)y tratamiento
farmacolgico. En lo que respecta al tratamiento motivacional, es importante incluir la participacin del nio con el propsito de
hacerlo responsable y a su vez darles un reconocimiento de las noches secas (llevar calendario de das secos y das que mojo la
cama). Farmacolgico: imipramina, inhibe la recaptura sinptica de noradrenalina y serotonina. Posee actividad anticolinrgica dbil y
accin antiespasmdica directa sobre musculo liso vesical, alterando la inervacin sinptica. Tambin se ha demostrado que disminuye
el tiempo de sueo MOR. Dosis para nios de 5-8 aos es de 25mg, para nios mayores de 50mg o a dosis de 0.9-1.5mg/kg/da. Se
administra en una sola dosis por la noche cuando la dosis es mayor de 50mgs, debe fraccionarse en dos tomas. Durante tratamiento
evaluar frecuencia cardiaca y presin arterial, as como realizar ECG, para detectar eventuales anomalas de la conduccin. Otro
medicamento que se puede utilizar es la desmopresina.

CASO CLINICO
Se trata de una paciente de 12 aos de edad, quien cursa sptimo grado y vive con su madre y sus dos hermanos (un nio de 10 aos y
una nia de 5 aos), La madre refiere que desde hace 3 aos presenta enuresis y enconpresis en cualquier momento y en cualquier
situacin, refiere la madre adems aislamiento social, tendencia al mutismo, llanto fcil y somnolencia diurnia, refiere adems
hiporexia, irritabiliadd, ideas de muerte, minusvala y desesperanza

PREGUNTA
Cual es la conducta mas adecuada a seguir?

RESPUESTA
a.- Iniciar con imipramina.
b.- Enviar al psiclogo.
c.- Enviar al psiquiatra.
d.- Ingreso psiquitrico obligado.

ENCOPRESIS. CIENCIAS BASICAS: Es difcil establecer un acuerdo unnime sobre este concepto. Es un trastorno de la eliminacin.
Consiste en la emisin fecal, voluntaria o no (incapacidad de controlar el esfnter anal), de forma regular, sobre lugares socialmente
inapropiados, generalmente la ropa. SALUD PUBLICA: El control esfinteriano se adquiere progresivamente con la edad. En culturas
occidentales son encopreticos alrededor de 5%. A los 16 aos no hay prcticamente ninguno, ya que la encopresis declina con la edad
en un porcentaje muy elevado (28%). El riesgo relativo de los nios respecto de las nias es de 3-6 veces superior a partir de los 4 aos
de edad. La encopresis secundaria iguala o supera ligeramente a la primaria. Los episodios de encopresis son ms frecuentes durante el
da que por la noche. La encopresis retentiva es la ms frecuente. CLASIFICACION: Segn su etiologa: FUNCIONAL O NO ORGANICA (no
hay causas orgnicas conocidas) y ORGNICA (puede ser provocada por trastornos gastrointestinales que cursan con diarrea,
intolerancia a la lactosa, insuficiencia pancretica, CUCI, lesiones posquirrgicas, disrafismo espinal oculto, en estos casos la
incontinencia parece estar provocado por la fatiga de los msculos del suelo plvico y del esfnter externo, despus de que el nio los
haya contrado de forma continuada para preservar la continencia). Segn la aparicin del problema: PRIMARIA O CONTINUA (ausencia
de control desde que naci, mejor pronstico, son nios desatendidos, regresivos, desinhibidos emocionalmente sin sentimiento de
vergenza) y SECUNDARIA O DISCONTINUA (presencia de control por un periodo de continencia de al menos un ao, coincide con
sucesos estresantes, son nios sobreprotegidos, inhibidos emocionalmente, con rasgos obsesivos, sentimientos de vergenza). Segn
su fisiopatologa: RETENTIVA (con estreimiento y rebosamiento, se caracteriza por un ciclo de varios das de retencin, una expulsin
dolorosa, otro periodo de retencin que ocasiona distensin y desensibilizacin de la pared rectal con ello se pierde la sensacin de la
necesidad de defecar, aparece un megacolon psicgeno, las heces se acumulan y finalmente se defeca por rebosamiento) y NO
RETENTIVA (sin estreimiento, el problema puede ser consecuencia de entrenamiento inadecuado, reaccin fisiolgica ante el estrs
ambiental o una forma de evidenciar conductas de oposicin ante las normas establecidas). PATOGENIA: La defecacin es el eslabn
final de todo el proceso digestivo. Empieza con la ingestin de alimentos y contina por el tracto digestivo, y acaba en el intestino
grueso que se divide en: ciego, colon y recto. Estos ltimos son los responsables de la defecacin y la continencia. Al menos dos
factores contribuyen a la continencia, la distribucin anatmica de la parte terminal del intestino grueso (ngulo anorectal) y la
contraccin tnica del musculo puborectal y del esfnter externo. Dichas condiciones anatmicas intervienen para evitar el flujo fecal
cuando la presin intraabdominal se eleva sbitamente, como sucede con los estornudos, la risa o cambios posturales bruscos. Cuando
cierta cantidad de heces entra en el recto y se llena, se origina un reflejo que inhibe la contraccin tnica del esfnter interno, que se
relaja, la presin del canal anal disminuye y las heces pueden descender. Ante la distensin rectal, el sujeto experimenta sensacin de
plenitud que se acompaa del deseo de defecar. Hay que recordar que este reflejo no se produce en los sujetos con enfermedad de
Hirschsprug, debido a la ausencia congnita de clulas nerviosas en algunos segmentos del colon, lo que provoca estreimiento y
obstruccin intestinal. La contraccin (relajacin) del esfnter externo ante la distensin del recto es una respuesta voluntaria que se
aprende durante el segundo ao de vida, ligado al desarrollo neuromuscular y al entrenamiento del nio en hbitos de eliminacin. Por
lo tanto, la mayora de los individuos con incontinencia fecal muestran anomalas en algunos de estos mecanismos: 1. Capacidad para
percibir la distensin rectal (sensibilidad anorectal). 2. Capacidad del recto para almacenar heces (acomodacin y adaptacin al bolo
MANUAL DE TRABAJO DEL CURSO ENARM CMN SIGLO XXI
CURSO ENARM CMN SIGLO XXI TEL: 36246001 Pharmed Solutions Institute PGINA 473

fecal): funcin de depsito. 3. Habilidad de contraccin del esfnter externo. 4. Motivacin para ejecutar las respuestas adecuadas. En
los bebes la defecacin es un proceso automtico (relajacin de los esfnteres de forma refleja cuando el recto est lleno).
DIAGNOSTICO: Puesto que la encopresis es un trastorno psicofisiolgico y multicausado, conllevara la realizacin de una exploracin
mdica y psicolgica. El principal objetivo es descartar la existencia de cuadros orgnicos que puedan ser responsables de la
incontinencia. En general el protocolo es: Historia clnica. Exploracin fsica (abdominal, perianal, tacto anorectal). Anlisis de sangre
especficos. Urocultivo. Estudios radiolgicos. Manometra anorrectal. Biopsia rectal. El objetivo de la evaluacin conductual reside en:
1. Averiguar si se han adquirido buenos hbitos de higiene y defecacin. 2. Determinar factores ambientales, sociales y personales que
pueden estar interfiriendo. 3. Establecer relaciones funcionales entre estos. TRATAMIENTO: Consta de 2 fases: Fase inicial de limpieza
fecal o desimpactacin (enemas). Fase de mantenimiento (laxantes). Modificaciones dietticas (fibra). Ingestin de lquidos. El
protocolo de Levine, inclua una combinacin de tratamiento mdico y conductual. Respuesta no significativa a frmacos (imipramina).
Tratamiento conductual: Identificar las conductas objeto de modificacin fecal. Ensear conductas requisito para la defecacin.
Instaurar hbitos rutinarios. Tcnicas empleadas: reforzamiento positivo, reforzamiento negativo, castigo, psicoterapia, entrenamiento
en hbitos de defecacin.

CASO CLINICO
Nio varn de cuatro aos que presenta problemas relacionados con el control de esfnteres. La madre consulta porque el nio
muestra retencin voluntaria de heces y orina y rechazo total a sentarse en el retrete. Refiere que el nio solicita y exige a los padres
que le pongan el paal, y solo entonces lleva a cabo las deposiciones.

PREGUNTA
Cual es la conducta mas apropiada a seguir?

RESPUESTA
a.- Psicoterapia conductual.
b.- Psicoterapia gestltica.
c.- Psicoterapia cognitiva.
d.- Psicoterapia psicoanaltica.



TRASTORNO POR DFICIT DE ATENCIN E HIPERACTIVIDAD (TDAH). CIENCIAS BASICAS: Es el trastorno psiquitrico ms frecuente en
la infancia; es un proceso crnico con una alta comorbilidad que va a influir en el funcionamiento del individuo en la edad adulta. El
TDAH segn el DSM-IV (o trastorno hipercintico segn el CIE-10) se define como un determinado grado de dficit de atencin y/o
hiperactividad-impulsividad que resulta desadaptativo e incoherente en relacin con el nivel de desarrollo del nio y est presente
antes de los 7 aos de edad. Las manifestaciones clnicas deben persistir durante ms de 6 meses. El cuadro debe ser ms severo que lo
observado en otros nios de la
misma edad, el mismo nivel de
desarrollo e inteligencia. Debe
estar presente en varios
ambientes como familia,
escuela, amigos. Debe producir
serios problemas en la vida
diaria. SALUD PUBLICA: TDAH
es un importante problema de
salud pblica debido a varias
razones: 1. Su alta prevalencia
(Utilizando los criterios del
DSM-IV la prevalencia se sita
entre el 3-7%. Con la CIE-10 la
prevalencia es del 1,5% y los
cuadros ms severos) 2. El
inicio en etapas precoces de la
infancia. 3. A ser un proceso
incapacitante y crnico. 4. A la
afectacin de las diferentes
esferas comportamentales
(familiar, escolar y social), y 5.
A su alta comorbilidad, por lo
que siempre deben investigarse otras patologas asociadas que conlleven un gran riesgo para el futuro del nio, como son trastornos
del aprendizaje, problemas de conducta o trastornos emocionales, entre otros. La relacin varn/mujer vara segn los estudios desde
6/1 a 3/1. CLASIFICACION: El DSM-IV, describe 3 subtipos: 1. Subtipo con predominio inatento (inatencin). Entre 20-30% de los casos.
2. Subtipo con predominio hiperactivo-impulsivo del 10-15%. 3. Subtipo combinado (cuando estn presentes los 3 tipos de sntomas).
Del 50-75% de los casos. Segn la CIE-10; es necesaria la existencia simultnea de los 3 tipos de sntomas: dficit de atencin,
hiperactividad e impulsividad, constituyendo el cuadro de alteracin de la actividad y la atencin. Reconoce adems una categora
separada, el trastorno hiperquintico de la conducta. PATOGENIA: Se trata de un trastorno multifactorial con una base
neurobiolgica y predisposicin gentica que interacta con factores ambientales. Las hiptesis sobre la etiologa abarcan diversas

Criterios diagnsticos del TDAH y diferencias entre DSM-IV y CIE-10 (negrita CIE-10)
Dficit de atencin:
1. A menudo no presta atencin suficiente a los detalles o incurre en errores por descuido en las tareas escolares,
en el trabajo o en otras actividades.
2. A menudo tiene dificultades para mantener (no mantiene) la atencin en tareas o en actividades ldicas
3. A menudo parece no escuchar cuando se le habla directamente (lo que se le dice)
4. A menudo no sigue instrucciones y no finaliza tareas escolares, encargos u obligaciones en el lugar de trabajo
5. A menudo tiene dificultad (presenta alteracin) para organizar tareas y actividades
6. A menudo evita (o muestra una fuerte aversin), le disgustan tareas que requieren un esfuerzo mental
sostenido
7. A menudo extrava objetos necesarios para tareas o actividades
8. A menudo se distrae fcilmente por estmulos irrelevantes
9. A menudo es descuidado en las actividades diarias
Hiperactividad
1. A menudo mueve en exceso manos o pies, o se remueve en su asiento
2. A menudo abandona su asiento en la clase o en otras situaciones en que se espera permanezca sentado
3. A menudo corre o salta excesivamente en situaciones en que es inapropiado hacerlo
4. A menudo tiene dificultades para jugar o dedicarse tranquilamente a actividades de ocio
5. A menudo est en marcha o suele actuar como si tuviera un motor (Exhibe un patrn persistente de
actividad motora excesiva que no se modifica sustancialmente por el contexto o exigencias sociales)
Impulsividad
1. A menudo precipita respuestas antes de haber sido completadas las preguntas
2. A menudo tiene dificultades para guardar su turno (A menudo no espera en la cola o no guarda su turno en
juegos o situaciones de grupo)
3. A menudo interrumpe o se inmiscuye en las actividades de otros (p.ej., se entromete en conversaciones o
juegos)
4. A menudo habla en exceso (sin una respuesta apropiada a las limitaciones sociales)
MANUAL DE TRABAJO DEL CURSO ENARM CMN SIGLO XXI
CURSO ENARM CMN SIGLO XXI TEL: 36246001 Pharmed Solutions Institute PGINA 474

reas: gentica conductual (mayor prevalencia en pacientes con familiares con TDAH y otros trastornos psiquitricos. En gemelos) y
molecular (factores genticos, en relacin con mutaciones de varios genes que codifican para los transportadores y receptores de la
dopamina y el gen transportador de la noradrenalina, fallo en el desarrollo de los circuitos cerebrales, en que se apoya la inhibicin y el
autocontrol), factores biolgicos adquiridos (en periodo prenatal, perinatal y postnatal como exposicin intrauterina al alcohol, nicotina
y determinados frmacos [benzodiazepnias, anticonvulsivantes], prematuridad, bajo peso al nacer, encefalitis, traumatismos, hipoxia,
hipogluecemia), la neuroanatoma (implicacin del crtex prefrontal y ganglios basales [reas encargadas de regular la atencin], se
sugieren alteraciones en las redes corticales cerebrales frontales y frontoestriadas. Sistema atencional anterior: lbulo frontal. Sistema
atencional posterior: lbulo parietal y cerebelo), neuroqumicos (la NA y DA, son los neurotransmisores de mayor relevancia en la
fisiopatologa y tratamiento de TDAH, ambos implicados en la funcin atencional y la DA tambin en la regulacin motora), bioqumica
cerebral, neurofisiologa (alteraciones de la actividad cerebral como: reduccin del metabolismo/flujo sanguneo en el lbulo frontal,
crtex, parietal, stratium, cerebelo), neuropsicologa y el entorno psicosocial (severidad y expresin de los sntomas puede verse
afectado a travs de la interaccin gen-ambiente. Factores como: inestabilidad familiar, problemas con amigos, trastornos psiquitricos
en los padres, paternidad y crianza inadecuada, relaciones negativas padres-hijos, nios que viven en instituciones con ruptura de
vnculos, adopciones y bajo nivel socioeconmico). DIAGNOSTICO: Los tres sntomas esenciales del TDAH son: 1. El dficit de atencin
(falta de perseverancia en la atencin o atencin dispersa), 2. La hiperactividad (excesivo movimiento) y 3. La impulsividad (dificultad
en el control de impulsos). El dficit de atencin guarda una mayor relacin con las dificultades acadmicas y el logro de metas,
mientras que la hiperactividad e impulsividad estn ms relacionadas con las relaciones sociales y los resultados psiquitricos. Las
manifestaciones clnicas varan en grado e intensidad segn la edad del paciente en el momento del diagnstico. El DSM-IV requiere
que estn presentes al menos 6 de los 9 sntomas tanto para el subtipo con predominio del dficit de atencin, como para el subtipo
hiperactivo-impulsivo. El TDAH de tipo combinado requiere una combinacin de ambos tipos de sntomas de dficit de atencin y de
hiperactividad impulsividad (6+6). Para la CIE-10 se requiere 6/9 sntomas de dficit de atencin adems de al menos 3/5 sntomas de
hiperactividad y 1/4 sntomas de impulsividad. La impulsividad constituye un sntoma importante en el DSM-IV pero no tanto en el CIE-
10. Existen una serie de criterios diagnsticos adicionales que se requieren para poder hablar de trastorno y no solo de sntomas
transitorios: Edad de inicio. Algunos sntomas deben haber estado presentes antes de los 6-7 aos. Duracin. Los criterios sintomticos
deben haber persistido al menos durante los ltimos 6 meses- Ubicuidad. Algn grado de disfuncin debida a los sntomas debe haber
estado presente en 2 situaciones o ms (escuela, trabajo, casa). Disfuncin. Los sntomas deben ser causa de una disfuncin
significativa (social, acadmica o laboral). Discrepancia. Los sntomas son excesivos en comparacin con otros nios de la misma edad,
desarrollo y nivel de inteligencia. Exclusin. Los sntomas no se explican mejor por la presencia de otro trastorno mental como
ansiedad, depresin o esquizofrenia, entre otros. Adems, es imprescindible obtener registros de sntomas mediante cuestionarios para
evaluar la gravedad o intensidad de los sntomas y su presencia en varios ambientes. Cuestionarios como el Cuestionario de Evaluacin
del TDAH son muy tiles para obtener de forma rpida los sntomas que estn presentes, tanto desde el punto de vista de los padres
como de los profesores. TRATAMIENTO: Un plan de tratamiento individualizado y multidisciplinar para un nio con TDAH y su familia
casi siempre debe incluir 3 aspectos fundamentales: 1. Entrenamiento a los padres: Psicoeducin sobre el TDAH. Entrenamiento en
tcnicas de manejo conductual del nio. 2. Intervencin a niveles acadmico y escolar: En el colegio. En casa. 3. Medicacin especfica
para el TDAH. El uso de medicacin debe ser una parte del plan de tratamiento inicial en la mayora de los nios en edad escolar y
adolescentes con TDAH. El reciente estudio de tratamiento multimodal del TDAH, del Instituto Nacional de Salud Mental, ha
demostrado que un tratamiento farmacolgico cuidadoso y estandarizado se asocia con una reduccin de sntomas significativamente
mayor, en la mayora de los nios, que un tratamiento de intervencin psicosocial con diferentes tipos de psicoterapia. Pero el grupo de
terapia combinada con psicoterapia y medicacin obtuvo mejores resultados que el grupo de terapia con medicacin sola en el
porcentaje de nios en remisin. Tratamiento farmacolgico, los ms utilizados y de primera eleccin son los psicoestimulantes, una
segunda alternativa los antidepresivos tricclicos si existe comorbilidad con problemas afectivos y emocionales y podra utilizarse los
inhibidores selectivos de la recaptura de serotonina, en algunos casos en donde la impulsividad es muy evidente. Uso de medicaciones
estimulantes en el tratamiento del TDAH: El metilfenidato (dosis de 0.1mg/mg, para fines practicos se inicia con dosis bajas de 2.5mg,
es el nico estimulante indicado para el tratamiento TDAH, mejoran la atencin, el comportamiento en clase y el desarrollo acadmico
a corto plazo, mejora la interaccin con compaeros y disminuye el comportamiento impulsivo. Se presenta en comprimidos de 5,10 y
20 mg. Existen opciones farmacolgicas diversas tiles en el manejo del TDAH solo o con comorbilidad, como los medicamentos no
estimulantes (atomoxetina, antidepresivos tricclicos, agonistas alfa adrenrgicos y modafinilo).

CASO CLINICO
Nia de 10 aos, remetida a psiclogo a peticin del colegio por fracaso escolar. Se trata de una nia con buena capacidad intelectual
que aunque no le gusta el estudio, se aplica trabajando en los deberes escolares, nunca ha mostrado comportamiento hiperactivo ni
han existido dificultades a la hora de obedecer las normas ni en casa ni en el colegio; la nica queja de sus padres es lo distrada que
es (no se concentra, le mandas a hacer un recado y se entretiene en cien mil cosas sin llegar a hacerlo, desorganizada, pierde las
cosas,etc) la considera una nia feliz, aunque comenta que sus dificultades escolares hace que a veces se entristezca.

PREGUNTA
Cual es el diagnostico mas probable?

RESPUESTA
a.- TDAH.
b.- TDA.
c.- TDAH-R
d.- Trastorno del control de impulsos.


MANUAL DE TRABAJO DEL CURSO ENARM CMN SIGLO XXI
CURSO ENARM CMN SIGLO XXI TEL: 36246001 Pharmed Solutions Institute PGINA 475



CIRUGIA

1) ESOFAGITIS, ACALASIA, REFLUJO ESOFAGICO Y ERGE.
2) DISPLASIAS DE ESOFAGO, ESOFAGO DE BARRETT, CANCER DE ESOFAGO.
3) VARICES ESOFAGICAS, RUPTURA ESOFAGICA Y HERNIA HIATAL.
4) GASTRITIS, ULCERA GASTRICA Y ULCERA DUODENAL, CANCER GASTRICO.
5) COLANGITIS, COLECISTITIS, COLEDOCOLITIASIS
6) CANCER DE PANCREAS, COLANGIOCARCINOMA,
7) PANCREATITIS AGUDA Y CRONICA.
8) ISQUEMIA MESENTERICA, INFARTO MESENTERICO, DIVERTICULITIS.
9) APENDICITIS, POLIPOS, COLITIS, CROHN, CUCI.
10) HEMORROIDES, FISTULA ANAL, CANCER DE COLON
11) HERNIAS DIAFRAGMATICAS, HIATAL, DE PARED E INGUINAL,
12) ORQUIEPIDIDIMITIS, TORSION TESTICULAR, CANCER TESTICULAR.
13) HIPERTROFIA PROSTATICA BENIGNA, CANCER DE PROSTATA
14) TCE, TRAUMA FACIAL Y TRAUMA RAQUIMEDULAR.
15) TRAUMA TORACICO, TRAUMA ABDOMINAL Y PELVICO
16) PIE DIABETICO Y COMPLICACIONES QUIRURGICAS DE LA DM.
17) RETINOPATIA DIABETICA E HIPERTENSIVA.
18) URGENCIAS OFTALMOLOGICAS, PERFORACION, GLAUCOMA. (CIRUGIA)
19) TRAUMA OTICO RUPTURA, PERFORACION, TRAUMA FACIAL.










































MANUAL DE TRABAJO DEL CURSO ENARM CMN SIGLO XXI
CURSO ENARM CMN SIGLO XXI TEL: 36246001 Pharmed Solutions Institute PGINA 476

ESOFAGITIS. CIENCIAS BASICAS: Es un trmino general para cualquier inflamacin, irritacin o hinchazn del esfago, el tubo que va
desde la parte posterior de la boca hasta el estmago. Causas: Disminucin de la eficacia de los mecanismos antirreflujo esofgico, en
particular del tono del Esfnter Esofgico Inferior (EEI), su tono normal es de 10-25mmHg. Presencia de una Hernia hiatal por
deslizamiento. Eliminacin inadecuada o lenta del material refluido. Reduccin de la capacidad de reparacin de la mucosa esofgica
por exposicin prolongada a jugo gstrico. Clnica: Acidez y ardor (pirosis), que el enfermo localiza en el epigastrio y en la regin
retroesternal. Otros sntomas de reflujo gastroesofgico, regurgitaciones cidas o vmitos, crnicos, dolor retroesternal, disfagia.
Factores incrementan el riesgo de esofagitis: Consumo de alcohol, de cigarrillos, ciruga o radiacin en el pecho (por ejemplo, el
tratamiento para el cncer pulmonar), tomar ciertos medicamentos sin mucha agua. Clnica: Disfagia, pirosis, regurgitacin,
hematemesis. La etiologa ms frecuente es la esofagitis pptica, secundaria a reflujo gastroesofgico no tratado. Otras causas son:
ESOFAGITIS INFECCIOSA (candida, herpes): Afectacin de la mucosa esofgica por distintos agentes fngicos, vricos o bacterianos. Se
debe sospechar en sujetos con factores favorecedores y disfagia. Esofagitis por Candida: La infeccin esofgica ms frecuente.
Favorecida en sujetos malnutridos e infectados por el virus de la inmunodeficiencia humana. ESOFAGITIS MEDICAMENTOSA:
alendronato, tetraciclina, doxiciclina, ibandronato, risedronato y vitamina C. ESOFAGITIS CUSTICA: Injuria de la mucosa esofgica
secundaria a la exposicin de la misma a agentes custicos (cidos o lcalis). El dao sobre el esfago y el estmago por la ingesta de
custicos se asocia a una alta morbimortalidad. Las causas de la ingesta pueden ser accidentales, frecuentemente en nios, o
voluntarias con fines suicidas, generalmente en adultos. Deteccin temprana de probable perforacin (examen clnico + Rx trax y
abdomen). Interrogatorio al paciente o familiar. Debemos interrogar acerca de: tipo de custico Ingerido (lcali o cido), cantidad
ingerida e intencionalidad. No usar sonda nasogstrica ni lavado. No usar agentes neutralizantes. No inducir el vmito. ESOFAGITIS
EOSINOFLICA: es una entidad que se caracteriza por la infiltracin tisular muy significativa, en este caso al nivel del esfago, de
eosinfilos. DIAGNSTICO: clnico por endoscopia. Clasificacin de Los Angeles para esofagitis: Grado A: Una o ms rupturas de la
mucosa confinadas a los pliegues mucosos, cada uno < 5 mm. Grado B: Por lo menos una ruptura de la mucosa > 5 mm, confinada a los
pliegues mucosos pero sin continuidad en los bordes de dos de los pliegues. Grado C: Por lo menos una ruptura de la mucosa entre los
lmites de dos pliegues de la mucosa, pero sin ser circuferencial. Grado D: Ruptura circunferencial de la mucosa. TRATAMIENTO:
Especfico del tipo de Esofagitis que se trate, se utilizan medicamentos como Ranitidina, Famotidina, Omeprazol entre otros.

CASO CLINICO
Paciente de sexo masculino de 75 aos con antecedentes de hipertensin arterial que ingres a la Unidad de Cuidados Intensivos con el
diagnstico de shock. Present vmitos alimentarios y pirosis durante las veinticuatro horas previas al ingreso. La familia refiri un
episodio de fiebre y escalofros. Se coloc una sonda nasogstrica observndose al aspirar un contenido en borra de caf. No refera
hematemesis o melena. El paciente fue reanimado con infusin de cristaloides y requiri drogas vasopresoras durante las primeras 24 h
(noradrenalina 0,1 mcg/kg/min). A su ingreso se realizaron hemocultivos y urocultlvo, ambos fueron positivos a las 48 h para
Escherchia coli multisensible. Se inici tratamiento emprico con ceftriaxona 1 gr cada 12 h que luego se cambi segn sensibilidad y
cumpli tratamiento antibitico por 14 das. La analtica demostr leucocitosis y anemia con hematocrito de 30%.

PREGUNTA
Cual de las siguientes medida no es apropiada en este momento?

RESPUESTA
a.- Inhibidores de la bomba de protones por va endovenosa.
b.- Sucralfato por va oral.
c.- Metronidazol 500 mg/cada 8 h por siete das.
d.- Prednisona via oral.

PREGUNTA
La presentacin clnica ms comn de esta patologa es?

RESPUESTA
a.- Sangrado digestivo alto.
b.- Hematemesis.
c.- Vmitos en borra de caf
d.- Dolor epigstrico.

PREGUNTA
Cual de los siguientes diagnosticos diferenciales es menos frecuente?

RESPUESTA
a.- Necrosis local secundaria a infecciones.
b.- Hematoma intramural del esfago.
c.- Lesion por custicos.
d.- Sndrome pilrico con reflujo severo.





MANUAL DE TRABAJO DEL CURSO ENARM CMN SIGLO XXI
CURSO ENARM CMN SIGLO XXI TEL: 36246001 Pharmed Solutions Institute PGINA 477

ACALASIA. CIENCIAS BASICAS: Es un trastorno primario de la motilidad esofgica caracterizado por la ausencia de peristlsis esofgica y
por la relajacin incompleta del EEI, afecta los dos tercios inferiores. La etiologa principal se debe a la perdida selectiva de las
motoneuronas inhibitorias, liberadores de pptido intestinal vasoactivo y xido ntrico del plexo mientrico. La prdida de estas
neuronas ocasiona el incremento en la presin basal, la relajacin incompleta del EEI y la desaparicin de la latencia y naturaleza
peristltica de la contraccin del cuerpo esofgico. Estas alteraciones conllevan a una disminucin de la propulsin esofgica y un
incremento en la resistencia del EEI, son las responsables de la dificultad de trnsito y de la disminucin del aclaramiento esofgico y
son la base de los sntomas clnicos y signos radiolgicos, endoscpicos y manomtricos de la enfermedad. La acalasia se clasifica en
primaria o idioptica que comprende el mayor nmero de casos, y en secundaria (neoplasias, infiltraciones de esfago, estmago,
linfomas, enfermedad de Chagas SALUD PBLICA: La prevalencia de la enfermedad es de aproximadamente 10 casos por 100.000 hab.
Su incidencia se ha mantenido bastante estable durante los ltimos 50 aos en aproximadamente 0,5 casos por 100.000 hab por ao.
Puede manifestarse a cualquier edad, sin predileccin racial y con igual frecuencia en hombres y mujeres, pero la mayora de los casos
es diagnosticada generalmente entre los 25 y 60 aos de vida. PATOGENIA: La regin principal afectada es el plexo esofgico mientrico
(Auerbach) que incluye zonas de respuesta inflamatoria en su mayor parte linfocitos T citotxicos CD3 y CD8 positivos, un nmero
variable de eosinfilos y los mastocitos, la prdida de clulas ganglionares y cierto grado de fibrosis. En casos avanzados puede haber
afeccin de las neuronas colinrgicas, incluso a nivel central. DIAGNOSTICO: Clnico; El sntoma inicial y ms comn es la disfagia (90%),
que al ser de origen motor y no mecnico, es tanto a solidos como a lquidos, suele empeorar con las comidas rpidas y la tensin.
Emocional. Puede haber dolor y tambin regurgitacin (75%) la cual se debe a la retencin de alimentos no digeridos en la porcin
inferior del esfago, esta situacin con el tiempo puede desencadenar esofagitis y pirosis, pero no es lo ms comn. Hay prdida de
peso a lo largo de meses o aos, cuando esta es acelerada se debe considerar la posibilidad de acalasia secundaria a una neoplasia
maligna. Radiografa de trax se puede llegar a observar ausencia de cmara gstrica y un nivel hidroareo en mediastino. El trago de
bario muestra dilatacin esofgica distal que termina en punta de lpiz, y en casos avanzados un esfago sigmoideo. La prueba con la
sensibilidad ms alta en la diagnostico de la Acalasia es La manometra esofgica y el nico mtodo posible para el diagnstico de la
enfermedad en sus fases iniciales, en las que todava no se han producido los cambios morfolgicos detectables por mtodos
radiolgicos o endoscpicos. Todas las sospechas diagnsticas de Acalasia deben confirmarse mediante manometra. La especificidad
diagnstica de la ME no es del 100%, es importante destacar que el patrn manomtrico de la Acalasia es indistinguible del ocasionado
por la obstruccin mecnica del EEI casi siempre por un tumor que origina un cuadro denominado pseudoacalasia, que corresponde
casi al 5% de los diagnsticos manomtricos de Acalasia y que debe ser particularmente sospechado en pacientes ancianos o con una
clnica rpidamente progresiva. La endoscopia es til para excluir las lesiones orgnicas, en particular aquellas que son causas
secundarias de Acalasia, como el carcinoma del cardias gstrico, entre otros. Durante el procedimiento se observa a menudo dilatacin
y atona del cuerpo esofgico, con tortuosidades en casos avanzados y que el lumen esofgico est lleno de saliva y restos de comida, y
la regin de la unin esfago gstrico est cerrada. TRATAMIENTO: No hay tratamiento especfico. Se recomienda dieta blanda.
Frmacos: anticolinrgicos resultan poco efectivos, nitritos o antagonistas de calcio antes de los alimentos, pero no son muy efectivos.
El sildenafil regula la funcin motora del EEI. La toxina botulnica tiene eficacia de 60% a los seis meses, se utiliza ms bien como una
medida temporal, antes de la ciruga. Por lo general a los pacientes menores de 50 aos, se les propone el quirrgico de primera
instancia, de no haber xito se pueden hacer dilataciones. A los mayores de dicha edad se les recomienda dilataciones neumticas o la
aplicacin de toxina botulnica de manera peridica, si no hay respuesta se pierde proponer tratamiento quirrgico. Despus del ERGE,
la acalasia es el trastorno funcional ms frecuente del esfago que amerita intervencin quirrgica. El alivio puede lograse mediante
una rotura instrumental no controlada del musculo del esfnter o con una miotoma quirrgica controlada, esta conlleva un bajo ndice
de morbilidad y logra mejores resultados a largo plazo. La miotoma se puede acompaar de funduplicatura parcial para disminuir los
eventos de reflujo posquirrgico. En la prctica la mayora de los pacientes se somete a dilatacin. Un riesgo inherente de l a dilatacin
neumtica es la rotura esofgica. La miotoma modificada de Heller se practica en todas las capas musculares: se extiende en sentido
distal sobre el estmago 1-2 cm por debajo de la unin y se prolonga 4-5cm en sentido proximal sobre el esfago. La esofagectoma es
el mejor tratamiento para los pacientes con disfagia y enfermedad benigna prolongada cuya funcin esofgica se destruy por
enfermedad o mltiples procedimientos quirrgicos.

CASO CLINICO
Masculino de 56 aos, fumador, diagnosticado con acalasia hace 6 aos, sin tratamiento por decisin del paciente, refiere que hace 3
meses presenta disfagia a liquidos y solidos adems perdida de peso, se realiza endoscopia donde se apresia abundante liquido, con
paredes dilatadas, mucosa con aspecto desquebrajado, friable. La biopsia reporta positivo para neoplasia.

PREGUNTA
Cual de las siguientes tipos de neoplasia es mas frecuente observar?

RESPUESTA
a.- Leiomioma.
b.- Tumor estroma gastrointestinal.
c.- Carcinomas de clulas escamosas.
d.- Carcinomas de clulas pequeas.








MANUAL DE TRABAJO DEL CURSO ENARM CMN SIGLO XXI
CURSO ENARM CMN SIGLO XXI TEL: 36246001 Pharmed Solutions Institute PGINA 478

ENFERMEDAD POR REFLUJO GASTROESOFAGICO (ERGE) Y REFLUJO GASTROESSOFAGICO (RGE). CIENCIAS BASICAS: El RGE se refiere
al paso de contenido gstrico hacia el esfago puede ser fisiolgico (mximo 5 episodios al da, fenmeno diurno, de corta duracin, no
evoca sntomas, no daa la mucosa esofgica) o patolgico. Se habla de ERGE cuando el reflujo ocasiona sntomas o complicaciones. La
enfermedad por reflujo se manifiesta de diferentes formas: ERGE no erosiva (70%), ERGE erosiva (25%), ERGE asociada a esfago de
Barret (4%). La regurgitacin se define como el paso del contenido gstrico hacia el esfago y/o hacia la boca sin esfuerzo alguno es
decir, no hay contraccin del diafragma; es el regreso involuntario hacia la boca, de comida o secreciones previamente deglutidas.
SALUD PUBLICA: El reflujo gastroesofgico ocasiona cerca de 75% de los padecimientos esofgicos. El ERGE se presenta en 3-4% de la
poblacin general, con aumento en su prevalencia en embarazadas y adultos mayores PATOGENIA: Los tres mecanismos ms
importantes que condicionan la incompetencia de la barrera anti-reflujo son: 1) Las relajaciones transitorias del EEI, 2) la hipotensin
del EEI, y 3) la alteracin anatmica de la unin condicionada por la presencia de hernia hiatal. Otros factores que interactan; la
ansiedad, la depresin o el estrs, de tal manera que estmulos mnimos en el esfago desencadenan dolor. La ingesta de alimentos
irritantes y grasas tambin pueden intensificar la percepcin del cido. La obesidad es un factor de riesgo para la ERGE. El reflujo cido
estimula el nervio vago y esto es la causa de la broncoconstriccin; as como el reflujo de pepsina es la causante de la enfermedad
ulcerosa pptica. El tabaquismo incrementa los sntomas de RGE. Aunque no hay estudios concluyentes, se ha demostrado que la
ingesta en exceso de ctricos, bebidas carbonatadas, caf, cafena, chocolate, cebolla, especies, comidas ricas en grasas, menta,
yerbabuena, alimentos salados y la fibra de algunos tipos de pan, pueden estar relacionados con el reflujo. Pueden aumentar los
sntomas en la ingesta de medicamentos no esteroides y AINES. DIAGNOSTICO: Clnico; Los sndromes esofgicos de la ERGE se
caracterizan por la presencia de sntomas tpicos pirosis (ms caracterstico y frecuente) y regurgitaciones, e incluso dolor torcico o
asociados a lesiones esofgicas como: erosiones, estenosis, Barrett y adenocarcinoma. Las manifestaciones extra-esofgicas ms
comunes son tos (38%), dolor torcico (30%), disfona (22%), asma (5%), escurrimiento nasal posterior (5%). Al parecer la presencia de
sntomas tpicos est relacionada con reflujo cido, mientras que el reflujo no cido (bilis, aire, etc.) se relaciona con la aparicin de
sntomas atpicos. El RGE que afecta severamente a la laringe puede contribuir a la aparicin de cncer en la laringe. Al parecer el 46%
de todas las apneas estn relacionadas con episodios de reflujo. Se ha observado que la pobre calidad de sueo est relacionada a un
gran nmero de eventos de reflujo mediante pH-metra. Dentro de las herramienta diagnosticas se cuenta con cuestionarios de
sntomas que otorgan una puntuacin, con base en lo cual se determina si el paciente tienen o no ERGE. Ensayo teraputico con
inhibidores de bomba de protones (IBP) en pacientes no complicados, con una dosis estndar por 1-2 semanas (omeprazol 40mgs
c/12hrs, pantoprazol 80 mg c/12hrs). La desaparicin de los sntomas con el tratamiento y la recada al suspenderlo confirma el
diagnstico. En caso de pacientes con sntomas extraesofgicos se recomienda extender el tiempo de administracin hasta tres meses.
La sensibilidad y especificidad varan entre 75 y 92% y 55 al 90% respectivamente. La pH-metra es la prueba ms sensible y especfica y
est indicada en ERGE, que no respondi a tratamiento, en ERGE posquirrgico. Los estudios radiogrficos son tiles para visualizar
defectos anatmicos como: Presencia de hernia hiatal, anillos, estrecheces, pero no es til para valorar el reflujo ni la esofagitis. El
esofagograma es til para valorar el trnsito esofgico, en casos de disfagia posoperatoria o cuando se sospecha neoplasia, debe
realizarse en el segundo o tercer nivel de atencin en pacientes posquirrgicos de hernia hiatal, en los cuales hay disfagia.
TRATAMIENTO: No recostarse justo despus de las comidas, evitar alimentos que relajen el EEI, evitar ropa ajustada (mejora 20%). Los
anticidos controlan sntomas leves de pirosis y regurgitacin, pero no curan la esofagitis. Los inhibidores de la secrecin acida; son la
piedra angular, los antagonistas H2 (ranitidina), logran curacin de esofagitis en 50%. Los IBP logran curacin de 80% a 4 sem y cerca de
90% a 8 sem. El tratamiento de reflujo alcalino es a base de sucralfato, colestiramina o hidrxido de aluminio. La ciruga antirreflujo
est indicada en pacientes con esofagitis, manifestaciones extraesofgicas de difcil control. Antes de la ciruga se debe realizar una
manometra y, adems se sugiere corroborar el diagnostico por pH-metra en 24 hrs. El procedimiento antireflujo ms frecuente es la
funduplicacin de Nissen que solo usa el fondo gstrico para envolver el esfago, la funduplicacin se calibra con un dilatador 60
francs y la longitud de la misma se limita a 1-2cm. Cuando la motilidad esofgica no es normal y la fuerza propulsiva no es suficiente
para vencer la obstruccin de una funduplicatura completa, est indicada una funduplicacin parcial. En pacientes con esfago corto
por estrechamiento, esfago de Barret o una hernia hiatal grande se realiza una gastroplastia de Collis como procedimiento para
prolongacin esofgica.

CASO CLINICO
Se trata de paciente femenino de 24 aos de edad, acude a consulta por opsomenorrea y oligomenorrea, agrega que no tiene vida
sexual activa desde hace 2 aos, refiere adems que ltimamente se siente fatigada y ha presentado mareos ocacionales, niega uso de
sustancia, agrega que ocacionalmente toma ranitidina, finalmente refiere regurgitacin y pirosis. A la exploracin fsica se observa IMC
de 17, pelo quebradizo, palidez generalizada, llenado capilar de 3 segundos, los exmenes de laboratorio reportaron hemoglobina 10.

PREGUNTA
Cual de los siguientes diagnosticos es la menos frecuente en el caso clnico?

RESPUESTA
a.- Gastritis erosiva.
b.- Esofagitis.
c.- Enfermedad por reflujo gastroesofagico.
d.- Anorexia nerviosa.

PREGUNTA
Luego de tratamiento con omeprazol 20 mg/da, presentando leve mejora sin embargo no se presento mejora de la hemoglobina y el
hematocrito, a la exploracin se observa halitosis y hemorragia suconjuntival, refiere que ha incrementado su ansiedad por presiones
laborales ya que su trabajo requiere mantenerse en talla 5, considerando la evolucin del paciente cual es la patologia asociada que
esta modificando la respuesta?

MANUAL DE TRABAJO DEL CURSO ENARM CMN SIGLO XXI
CURSO ENARM CMN SIGLO XXI TEL: 36246001 Pharmed Solutions Institute PGINA 479

RESPUESTA
a.- Anorexia purgativa.
b.- Esofagitis erosiva.
c.- Esofago de barret.
d.- Ulcera gstrica.

PREGUNTA
La paciente regresa a consulta de seguimiento 3 meses despues, su tratamiento instalado fue de fluoxetina 20 mg/24 hrs, asi como
complejo b y hierro oral, los laboratorios reportaron hemoglobina 11 y hematocrito de 38, se realizo endoscopia donde se reporta una
lesin mucosa mayor de 5 mm, sin continuidad entre la parte ms prominente de 2 pliegues mucosos, que grado presenta la paciente
de la Clasificacin de Los ngeles de la esofagitis.

RESPUESTA
a.- Grado A.
b.- Grado B.
c.- Grado C.
d.- Grado D.

PREGUNTA
Se indico cambios en el estilo de vida, tratamiento farmacolgico por 6 meses, a la exploracin se observo IMC de 23, se realizo una
endoscopia de control con el siguiente reporte, unin escamoso-columnar se encuentra desplazada proximalmente de color rojo de la
mucosa columnar el reporte histolgico reporto existencia de neutrfilos y eosinfilos que infiltraban la mucosa y la lmina propia,
considerando las condiciones actuales, cual es la conducta a seguir.

RESPUESTA
a.- Vigilancia estrecha endoscopia anual.
b.- Funduplicatura.
c.- Fototerapia endoscpica.
d.- Esofagectomia.

CASO CLINICO
Masculino de 52 aos de edad que acude a consulta debido a que presenta sensacin de quemazn retrosternal que se irradia de
manera ascendente a cuello y espalda, que ha identificado se presenta entre media hora y una hora despus de la alimentacin,
acteriormente se presentaba solo cuando comia grasas e irritantes, pero actualmente con casi cualquier alimento y solo se resuelve
tomando anticidos OTC. Antecedente de alcoholismo, tabaquismo positivo, tos crnica y osteoartitis controlada con diclofenaco. A la
exploracin fsica se observa perdida del esmalte dental y halitosis.

PREGUNTA
Cul es la conducta a seguir este caso?

RESPUESTA
a.- Omeprazol 20 mg cada 12 hrs.
b.- Misoprostol 400 g cada 12 hrs.
c.- Sales de bismutos 10 mg cada 24.
d.- Malgadrato 5 ml cada 8 hrs.

CASO CLINICO
Varn de 46 aos, sin alergias conocidas, fumador de 12 cigarrillos al da y bebedor de 40 g de alcohol diarios. Antecedentes personales
de apendectoma a los 14 aos. Hace cuatro aos fue medicado con ansiolticos por haber padecido varias crisis de ansiedad. Desde
hace 18 meses aqueja sensacin de quemazn en regin epigstrica que se irradia hacia la parte central de trax y, ocasionalmente,
hasta el cuello. Las molestias aumentan despus de la ingesta as como en decbito. Mejoran slo un poco, y temporalmente, con la
toma de anticidos. No refiere tener regurgitacin, disfagia, odinofagia ni dolor torcico. Tampoco padece sntomas como nuseas o
vmitos. Su estado general es bueno sin haber presentado astenia, anorexia ni adelgazamiento. Medicado por su mdico de cabecera
con ranitidina 300 mg por la noche, ha tenido una mejora parcial que refiere como 3 en una escala de 0 a 10. A la exploracin fsica
encontramos un paciente consciente y orientado. Coloracin normal de piel y mucosas. No se objetivan adenomegalias ni lesiones
drmicas. Auscultacin cardiaca y pulmonar sin alteraciones. Abdomen blando y depresible; no se palpan ni se percuten masas ni
visceromegalias. Exploracin neurolgica sin alteraciones.

PREGUNTA
Ante la clnica observada en este paciente cual es el diagnostico mas probable?

RESPUESTA
a.- Dispepsia funcional.
b.- Enfermedad por reflujo gastroesofgico.
c.- Esofagitis por cndida.
a.- Acalasia.
MANUAL DE TRABAJO DEL CURSO ENARM CMN SIGLO XXI
CURSO ENARM CMN SIGLO XXI TEL: 36246001 Pharmed Solutions Institute PGINA 480

PREGUNTA
Cul sera la siguiente exploracin a realizar para confirmar el diagnstico clnico?

RESPUESTA
a.- pH-metra esofgica de 24 horas.
b.- Esofagograma.
c.- Prueba de tratamiento emprico con omeprazol.
d.- Manometra esofgica.

PREGUNTA
El paciente continuo con manifestaciones clnicas, considerando este punto cual es la conducta diagnostica ms adecuada a seguir para
establecer finalmente un diagnostico?

RESPUESTA
a.- pH-metra esofgica de 24 horas.
b.- Endoscopia y, si sta fuese negativa, pH-metra esofgica de 24 horas.
c.- Endoscopia y, si sta fuese positiva, pH-metra esofgica de 24 horas.
d.- Manometra esofgica.

PREGUNTA
La pH-metra esofgica fue informada como reflujo gastroesofgico cido cuantitativamente normal con un ndice bajo de correlacin
sintomtica, cual es el diagnostico considerando los datos observado?

RESPUESTA
a.- Dispepsia funcional.
b.- Somatizacin.
c.- Ansiedad.
d.- Pirosis funcional

PREGUNTA
Considerando la direccin que ha tomado el caso actual, cul cree que sera el tratamiento ms adecuado para este paciente?

a.- Alcalinos despus de las comidas.
b.- IBP a dosis habituales.
c.- IBP a dosis superiores (al menos doble).
d.- Antagonista de los receptores H2





























MANUAL DE TRABAJO DEL CURSO ENARM CMN SIGLO XXI
CURSO ENARM CMN SIGLO XXI TEL: 36246001 Pharmed Solutions Institute PGINA 481

ESOFAGO DE BARRET (EB). CIENCIAS BASICAS: Es la transformacin metaplsica del epitelio esofgico, en el estudio patolgico sin
importar la extensin. La marca distintiva de la metaplasia intestinal es la presencia de clulas caliciformes intestinales. Recodar
que RGE provoca un proceso inflamatorio de la mucosa esofgica, llamada Esofagitis por Reflujo. Esta ltima, predispone a
complicaciones o secuelas crnicas como lceras, estenosis displasia, las que son generalmente irreversibles y que en algn caso,
como sucede en el EB, son lesiones consideradas como precancerosas o de mayor riesgo de cncer. SALUD PUBLICA: El EB se presenta
en hasta 10% de los pacientes con RGE crnico y ste se presenta con una frecuencia estimada de entre 20 y 40% de la poblacin. Es
ms frecuente en el sexo masculino y en caucsicos, y la edad media al diagnstico es de 60 aos aproximadamente. Se ha descrito
tambin asociacin familiar. La estimacin del riesgo de adenocarcinoma es variable, va de 1 en 50 a 1 en 400 pacientes-ao. Cerca de
la tercera parte de los pacientes con EB presenta neoplasia maligna. PATOGENIA: Normalmente toda lesin del epitelio epidermoide
(escamoso) cura mediante regeneracin de clulas epidermoides. En el EB, por causas an desconocidas, el epitelio epidermoide
lesionado es reemplazado por epitelio columnar cilndrico pseudoestratificado. Inicialmente se postul un origen congnito, pero
actualmente se acepta el EB como una condicin adquirida, en la que una lesin intensa y sostenida sobre el epitelio del esfago distal,
generalmente debida a RGE, provoca el reemplazo del epitelio plano pluriestratificado normal por un epitelio columnar metaplsico,
ms resistente al cido. El origen de este nuevo epitelio seran clulas totipotenciales ubicadas en la capa basal o germinativa del
epitelio esofgico. Estudios de pHmetra de 24 hrs muestran que los pacientes con EB tienen mayor intensidad y frecuencia de
exposicin al reflujo cido que el resto de los pacientes con RGE, lo que pudiera relacionarse con presiones del esfnter esofgico
inferior ms bajas y peristalsis esofgica menos activa. DIAGNOSTICO: No existe una clnica del esfago de Barrett, los sntomas que
presentan estos pacientes son los propios del reflujo gastroesofagico, o de sus complicaciones: Pirosis, regurgitaciones, epigastralgias,
dolor torcico retro-esternal que se irradia al cuello y brazos, asfixia nocturna, tos. La realidad de su diagnstico depende
esencialmente de la endoscopia y de la biopsia, con histologa. Biopsia se hace a nivel distal de la unin escamosa-columnar, y ella se
practica en el rea Inferior a la lnea z, que es la (unin escamo-columnar), El endoscopista reconoce el borde prximo de los pliegues
gstricos aun con la existencia de una Hernia Hiatal, es la marca endoscpica ms segura para identificar la unin mucosa escamosa
columnar, adems el llamado el epitelio intestinal, tambin debemos considerarlo como esfago de Barrett , por este motivo la nueva
definicin de esfago de Barrett es: La presencia de mucosa columnar, con metaplasia intestinal en el esfago inferior, incluida la
mucosa que tapiza el esfnter esofgico inferior. La importancia de este cambio de definicin que la metaplasia de Barrett, se ha
convertido en un criterio histolgico ms que endoscopio, ya que solo sera posible identificar la metaplasia intestinal, mediante el
estudio histolgico, con esta nueva definicin se puede distinguir dos tipos de esfago de Barrett: 1. El esfago de Barrett clsico de 3
ms cm. 2. El esfago de Barrett corto incluso ultracorto menos de 3cm. La mucosa escamosa, tiene una coloracin blanquecina
rosa plido en cambio la mucosa columnar y tiene una coloracin rojiza. Seguimiento endoscpico-biopsia: En relacin al riesgo de
malignizacin y a los factores de riesgo de desarrollar cncer, se propone diferenciar tres grupos de pacientes con EB, para un
seguimiento diferenciado, de manera que la relacin costo-beneficio sea adecuada; 1. Paciente de bajo riesgo: que incluira aquellos
cuyo epitelio metaplsico sea de tipo cardial o fndico y los marcadores tumorales sean negativos. En estos casos es suficiente con un
control endoscpico-bipsico cada 5 aos, hasta los 70 aos de edad en los que se suprime el seguimiento. 2. Pacientes de riesgo
medio: comprende a los fumadores y bebedores importantes, a los que tienen una extensin de la metaplasia igual o mayor de 5 cm, y
a aquellos cuyo epitelio metaplsico sea de tipo intestinal especializado. En este grupo, el control endoscpico-bipsico debe ser anual,
suprimindolo tambin a los 70 aos de edad. 3. Pacientes de alto riesgo: son los que tienen displasia en el estudio histolgico de las
biopsias obtenidas durante la primera exploracin o alguno de los marcadores tumorales, distintos de la displasia, positivos. En este
caso debemos diferenciar dos subgrupos: a) Pacientes con displasia leve o algunos de los marcadores tumorales positivos; se har
control endoscpico-bipsico cada 6 meses hasta que en dos exploraciones consecutivas no encontremos displasia ni otros marcadores
tumorales positivos, con lo cual se pasara a control anual como en el grupo anterior. b) Pacientes con displasia severa, en los que se
repite la endoscopa-biopsia inmediatamente, por la posibilidad de coexistencia de cncer. Y debe plantearse seriamente el tratamiento
quirrgico u otras opciones teraputicas si hay contraindicacin a la ciruga. TRATAMIENTO: La esofagitis por reflujo es el primer
eslabn en la progresin de la enfermedad hacia el EB. Esta progresin slo puede evitarse haciendo que el material refluido pierda su
agresividad o que no se produzca el reflujo. Medidas higieno-dietticas comunes a todos los pacientes y el empleo de una medicacin
(IBP, por tiempo prolongado y altas dosis) que ha mostrado ser muy eficaz. En teora, la ciruga antirreflujo, si consigue evitarlo sera el
mejor mtodo de tratamiento de la esofagitis y de profilaxis del EB. Sin embargo, la ciruga slo se reserva para los pocos casos en los
que fracasa el tratamiento mdico o en circunstancias individuales especiales. Hasta ahora, ni el tratamiento mdico ni el quirrgico del
RGE han demostrado en forma consistente una regresin significativa del epitelio metaplsico o disminucin del riesgo de cncer
esofgico, ni la terapia mdica ni la quirrgica evitan la necesidad del seguimiento endoscpico-bipsico para detectar preczmente
displasia o cncer de la mucosa metaplsica. Tratamiento quirrgico: Funduplicatura de Nissen; que consiste bsicamente en el cierre
del hiato esofgico, la seccin de vasos cortos para movilizar el fondo gstrico y la colocacin de puntos que toman el fondo gstrico
anterior, la pared muscular el esfago y el fondo gstrico posterior. Las complicaciones de esta tcnica son la disfagia, el desgarro de la
sutura con prdida de la vlvula, y la imposibilidad de eructar por hipercontinencia de la vlvula. Para prevenir estas complicaciones
existen variantes de la tcnica de Nissen. No son pocos los autores que proponen la Esofagectoma como nico tratamiento con
posibilidades de curacin, por supuesto asociada a alta morbi-mortalidad operatoria. Para pacientes con Barrett largo y con larga
expectativa de vida, buen desempeo funcional y capacidad fsica y psquica de tolerar la ciruga, debe considerarse la extirpacin
quirrgica mediante la esofagectoma o la tcnica de Merendino, como primera eleccin, en centros con experiencia y que acrediten
sus resultados de aceptablemente baja morbimortalidad. Para los casos que no deseen ciruga, o para aquellos con pobre reserva
funcional, las posibilidades son dos: tratarlos en un protocolo de ablacin endoscpica o ser sometidos a vigilancia endoscpica-
histolgica. La primera debe considerarse an en etapa de investigacin pues no se ha demostrado fehacientemente su rol teraputico
en el largo plazo, no obstante sus innegables ventajas de buena tasa de eliminacin del epitelio alterado en el corto y mediano plazo,
manteniendo inalterada la continuidad anatmica del tubo digestivo superior. La segunda ciertamente es la conducta menos agresiva y
probablemente de mayor atractivo para los pacientes, pero de ms riesgo de progresin a cncer y obliga en todo caso a varias
endoscopas cada ao. Actualmente es considerada una opcin aceptable para pacientes de menor expectativa de vida, con pobre
reserva funcional o de alto riesgo quirrgico, que sean debidamente informados y cumplindose las condiciones de control
protocolizado y sistematizado.
MANUAL DE TRABAJO DEL CURSO ENARM CMN SIGLO XXI
CURSO ENARM CMN SIGLO XXI TEL: 36246001 Pharmed Solutions Institute PGINA 482

CASO CLINICO
Un hombre de 67 aos de edad, acude por disfagia a alimentos slidos y lquidos, tos de predominio nocturno, cuenta con antecedente
de ERGE con tratamiento irregular, agrega que ha perdido 10 kilos en los ltimos tres meses, niega sangrado sus constantes vitales se
encuentran dentro de parmetros normales.

PREGUNTA
Cual es su conducta a seguir?

RESPUESTA
a.- Realizar pH-metria.
b.- Realizar manometra.
c.- Realizar serie gastroesofgica.
d.- Realizar endoscopia.

PREGUNTA
Se realizaron diversos estudios dentro de los cuales se encontra una endoscopia la cual mostr una lesin plana, de color rojizo en la
mitad de esfago. Esta lesin no se tio con solucin de Lugol. Cual es su impresin diagnostico en este momento?

RESPUESTA
a.- Adenocarcinoma.
b.- Esofagitis por reflujo.
c.- Acalasia.
d.- Esofago de barrett

PREGUNTA
Considerando su diagnostico, cual es el tratamiento mas apropiado para el caso?

RESPUESTA
a.- Dilatacion neumtica.
b.- Funduplicatura de nissen.
c.- Esofagectomia.
d.- Terapia fotodinmica.

PREGUNTA
Luego del tratamiento indicado, 3 meses despus el paciente presenta dolor severo, disfagia a liquidos con perdida de 5 kilogramos,
cual es la conducta a seguir?

RESPUESTA
a.- Dilatacion neumtica.
b.- Funduplicatura de nissen.
c.- Esofagectomia.
d.- Terapia fotodinmica.

CASO CLINICO
Luego de realizarse una endoscopia una paciente de 41 aos presenta incremento de los cuadros de reflujo gastroesofagico de
predominio nocturno, la endoscopia reporto lesiones displasicas en el tercio distal del esfago y mucosa gstrica despulida, la paciente
agrega disminucin de peso corporal debido a que come poco por la sensacin ardorosa a la deglucin, existen antecedentes de cncer
en la familia.

PREGUNTA
Cul es el tratamiento que presenta mejor pronstico.

RESPUESTA
a.- Fototerapia.
b.- Ciruga.
c.- Radioterapia.
d.- Quimioterapia.

PREGUNTA
Cual es la conducta si hay displasia leve?

RESPUESTA
a.- Endoscopia y biopsia.
b.- Endoscopia y biopsia confirmada.
c.- Endoscopia y biopsia seriada.
d.- Endoscopia sin biopsia.
MANUAL DE TRABAJO DEL CURSO ENARM CMN SIGLO XXI
CURSO ENARM CMN SIGLO XXI TEL: 36246001 Pharmed Solutions Institute PGINA 483

CASO CLINICO
Se trata de masculino de 41 aos de edad el cual acude a consulta debido a que presenta pirosis, disfagia y regurgitacin, tratado con
omeprazol por un ao a dosis adecuada.

PREGUNTA
Complicaciones ms frecuente que esperara en este caso

a.- Ulcera de barett.
b.- Estenosis de barett
c.- Adherencia
d.- Cncer de esofago

PREGUNTA
En paciente con esofagitis y esfago de barett, cual ser la conducta a seguir mas adecuada?

RESPUESTA
a.- Reparacin quirurgia y funduplicatura.
b.- Repetir y corroborar con biopsia.
c.- Corroborar con dos patlogos.
d.- Ablacion con fototerapia.

PREGUNTA
Con cual tipo de barret anatomica esta mas frecuente relacionado con presentacion de Ca?

RESPUESTA
a.- Barett corto.
b.- Barett largo.
c.- Barett bajo.
d.- Barett alto.

PREGUNTA
Cual es el objetivo del tratamiento medico del esfago con metaplasia columnal?

RESPUESTA
a.- Regresin de la metaplasia columnar.
b.- Retrasar la progresin.
c.- Disminuir la displasia columnar.
d.- Evitar el adeno carcinoma.

PREGUNTA
Cual es el tratamiento quirrgico cuando la biopsia reporta metaplasia columnar?

RESPUESTA
a.- Fototerapia.
b.- Funduplicatura.
c.- Esofagectomia.
d.- Ablacin.



















MANUAL DE TRABAJO DEL CURSO ENARM CMN SIGLO XXI
CURSO ENARM CMN SIGLO XXI TEL: 36246001 Pharmed Solutions Institute PGINA 484

CANCER DE ESOFAGO. CIENCIAS BASICAS: Se origina en la capa ms interna (la mucosa) y crece hacia fuera (a travs de la submucosa y
la capa muscular). Debido a que dos tipos de clulas pueden cubrir el esfago, existen dos tipos principales: el carcinoma de clulas
escamosas y el adenocarcinoma. El esfago est normalmente cubierto con clulas escamosas (carcinoma de clulas escamosas), el
cual puede ocurrir en cualquier lugar a lo largo del esfago. En cierto tiempo, el carcinoma de clulas escamosas fue el tipo ms comn
de cncer de esfago en los Estados Unidos. En la actualidad, estos cnceres constituyen menos de la mitad de los cnceres esofgicos
en este pas. Los cnceres que se originan de clulas glandulares se llaman adenocarcinomas. Este tipo de clula no es normalmente
parte del revestimiento interno del esfago. Antes de que se pueda desarrollar un adenocarcinoma, las clulas glandulares tienen que
reemplazar un rea de las clulas escamosas, como en el caso del esfago de Barrett. Esto ocurre principalmente en la parte inferior del
esfago, donde ocurren la mayora de los adenocarcinomas. Los cnceres que se originan en el rea donde el esfago se une al
estmago (la unin GE), lo que incluye aproximadamente las primeras 2 pulgadas del estmago (llamada cardias), suelen comportarse
como cnceres de esfago de manera que son agrupados como cnceres esofgicos. Existen tumores malignos esofgicos no epiteliales
(no carcinomas), de presentacin infrecuente, entre los que podemos destacar el leiomiosarcoma (derivado de la musculatura del
esfago). Clasificados como tumores benignos esofgicos se encuentran, entre otros, el papiloma escamoso, el adenoma esofgico, y
los plipos. SALUD PUBLICA: El cncer de esfago es el cuarto tumor ms frecuente del aparato digestivo (por detrs del cncer
colorrectal, gstrico y heptico) y se sita entre los diez cnceres ms frecuentes en el mundo. Su incidencia es mayor en areas de alta
frecuencia Asia y frica Central y del Sur (ms de 100 casos por 100.000 habitantes/ao). Es ms frecuente en el hombre que en la
mujer, pudiendo oscilar la relacin entre 3 y 10 hombres por cada mujer. La edad habitual de presentacin es entre los 55 y los 70 aos,
siendo infrecuentes los casos en personas por debajo de los 40 aos. En los ltimos aos la incidencia del adenocarcinoma del tercio
distal del esfago y de la unin GE se ha incrementado de forma paralela a la ERGE, especialmente en personas con alto ndice de masa
corporal. PATOGENIA: Los factores de riesgo asociados al desarrollo de carcinoma escamoso de esfago son: tabaco, alcohol, extraccin
social baja, dieta: dficit nutritivos, hipovitaminosis, ingesta de alimentos con alto contenido de nitrosaminas, acalasia, esfago de
Barrett, lesiones por custicos, Sndrome de Plummer-Vinson, neoplasias de cabeza y cuello, cncer de mama cuyo tratamiento haya
comprendido la radioterapia, ingesta de bebidas calientes. Los factores de riesgo para adenocarcinoma de esfago son: RGE, obesidad,
infeccin por H. pylori, sexo masculino, estrato social bajo, tabaco, alimentos con alto contenido de nitrosaminas. El adenocarcinoma
de esfago distal y de la unin gastroesofgica surge tpicamente en un epitelio con metaplasia, circunstancia conocida como esfago
de Barret. Esta condicin premaligna se caracteriza por la presencia de un epitelio columnar que abarca a tres o ms centmetros del
esfago tubular distal, ya sea en presencia o en ausencia de hernia de hiato. Aproximadamente el 60% de los casos de adenocarcinoma
de esfago distal o de la unin gastroesofgica presentan evidencia de esfago de Barret. DIAGNOSTICO: En las etapas iniciales de la
enfermedad, el cncer de esfago habitualmente es asintomtico. Los sntomas ms frecuentemente asociados al cncer de esfago
son: Disfagia (primero, solidos luego a lquidos), prdida de peso, dolor retroesternal, sntomas respiratorios (tos e infecciones
respiratorias), otros sntomas: sangrado, disfona (ronquera) e hipo, hemorragia intensa o parlisis de cuerdas vocales. Es importante
resaltar que estos sntomas tambin pueden estar causados por otras enfermedades del esfago, o por otras causas menos serias. El
diagnostico se hace con endoscopia y toma de biopsia, biopsias tanto de la mucosa del esfago como de ganglios linfticos regionales
sospechosa. TAC de trax y abdomen: permite conocer, basndose en su tamao, la extensin local del tumor, la posible afectacin de
ganglios cercanos o a distancia, y la extensin a otros rganos (pulmn, hgado.). La estadificacin debe incluir TAC de trax y abdomen
y en candidatos a tratamiento quirrgico se debe agregar esofagograma y ultrasonido transendoscopico, para definir con mayor
precisin T y N. Si se tiene disponible la tomografa por emisin de positrones (PET), se puede utilizar con el fin de detectar metstasis
a distancia ocultas. Clasificacin simplificada: Estadio 0: tambin denominado carcinoma in situ. El cncer se localiza slo en la parte
ms superficial de la mucosa esofgica, sin sobrepasarla. Estadio I: Tumores bien diferenciados, sin infiltracin ganglionar locorregional
por metstasis, que alcanzan como mximo la capa adventicia. Estadio II: El tumor no alcanza la capa adventicia, asociando un mximo
de dos ganglios linfticos locorregionales infiltrados por metstasis; alcanza sin sobrepasar la capa adventicia sin presentar ganglios
linfticos con metstasis. Estadio III: El tumor afecta a la capa adventicia, asociando un mximo de 2 ganglios linfticos locorregionales
infiltrados por metstasis; presenta afectacin ganglionar con ms de 2 ganglios infiltrados. Estadio IV: El tumor se ha extendido a
distancia, a otros rganos del cuerpo. TRATAMIENTO: Existen tres opciones fundamentales de tratamiento para el cncer de esfago:
Ciruga: Su objetivo es extirpar el tumor con mrgenes libres, es decir, sin dejar enfermedad residual. Se considera indicada en estadios
iniciales de la enfermedad (resecable). Es el tratamiento ms frecuentemente utilizado para el cncer de esfago. El tipo de ciruga
vara en funcin del tamao, localizacin y extensin de la enfermedad. La esofagectoma es la extirpacin quirrgica de parte o la
totalidad del esfago, conectando la parte sana restante al estmago, para as permitir la alimentacin del paciente. Es factible la
utilizacin de parte del intestino para realizar la conexin. En la esofagectoma se incluye la extirpacin de los ganglios linfticos
cercanos al esfago, para su estudio al microscopio, con el fin de conocer el grado de extensin de la enfermedad. En ocasiones puede
ser necesaria la realizacin de una gastrostoma de alimentacin, ante imposibilidad para la ingesta de alimentos (condicionada por una
estrechez infranqueable del esfago). Esta tcnica puede realizarse va endoscpica o de manera quirrgica. Radioterapia: se puede
emplear como tratamiento nico formando parte de una estrategia multidisciplinar (combinada con la ciruga y/o la quimioterapia).
Se utiliza con intencin curativa, en estadios iniciales de la enfermedad, con intencin paliativa, en estadios avanzados, para aliviar
sntomas tales como el dolor y la disfagia. Radioterapia externa: consiste en la utilizacin de una mquina fuera del cuerpo para enviar
la radiacin al rea concreta que se desea tratar. Radioterapia interna: en sta, una sustancia radioactiva se coloca (mediante un
sistema sellado) muy cerca o dentro del tumor, permitiendo as alcanzar ms dosis de radiacin con menores efectos secundarios sobre
los tejidos sanos. En el cncer de esfago generalmente se utiliza para el tratamiento de la disfagia. Quimioterapia: Consiste en la
introduccin de un frmaco al torrente sanguneo (quimioterapia sistmica) para eliminar las clulas cancerosas de todo el cuerpo
(dentro y fuera del esfago). La quimioterapia se puede emplear como tratamiento nico o formando parte de una estrategia
multidisciplinar (combinada con la ciruga y/o la radioterapia). Su utilizacin de forma aislada, previa a la ciruga, se considera un
tratamiento en investigacin. Otros tratamientos con posible indicacin en el cncer de esfago son la terapia lser (destruccin del
tumor con lser), y la colocacin de prtesis esofgicas (stent), consistente en la implantacin endoscpica de un tubo expandible. Esta
ltima tcnica se utiliza para resolver la estrechez esofgica provocada por el tumor y permitir as la adecuada alimentacin del
paciente. Tratamiento. Estadios Localizados (I,II,III), el tratamiento fundamental es la ciruga. En la mayora de pacientes con cncer de
esfago estadio I la ciruga exclusiva es el tratamiento estndar, salvo en casos con diagnstico en etapas muy tempranas de la
MANUAL DE TRABAJO DEL CURSO ENARM CMN SIGLO XXI
CURSO ENARM CMN SIGLO XXI TEL: 36246001 Pharmed Solutions Institute PGINA 485

enfermedad en los que se puede plantear la reseccin endoscpica. No obstante, cabe destacar que el diagnstico de la enfermedad en
etapas tan precoces es un hecho infrecuente. En los casos ms habituales de diagnstico de la enfermedad, en estadios II y III, ante los
no ptimos resultados de tratamiento quirrgico aislado, ha existido un inters creciente por utilizar un tratamiento multidisciplinar. En
muchas ocasiones se necesita de la combinacin de la utilizacin de quimioterapia y/o radioterapia, con el objetivo de mejorar los
resultados obtenidos solamente con la ciruga. La mayora de estos pacientes son tratados inicialmente con quimioterapia o con la
combinacin de la quimioterapia y radioterapia previamente al tratamiento quirrgico. En ocasiones es posible realizar una
reevaluacin precoz al tratamiento con quimioterapia para as seleccionar a los pacientes que ms se benefician de esta estrategia de
tratamiento previo a la ciruga. Por estadios avanzados de cncer de esfago entendemos el estadio IV. El objetivo del tratamiento es la
paliacin, tratando igualmente de prolongar la supervivencia del paciente. El pilar es la quimioterapia, pudiendo realizarse dentro de
ensayos clnicos. Con el objeto de mejorar la sintomatologa del paciente (dolor, imposibilidad para la ingesta de alimentos...) puede
estar indicado el uso de radioterapia externa, radioterapia interna, terapia lser, prtesis esofgicas, y ciruga paliativa.

CASO CLINICO
Mujer de 51 aos, con antecedente de tabaquismo y asma. Consult por disfagia, nuseas, vmitos y baja de peso. La endoscopia
mostr una lesin polipoidea, mamelonada y ulcerada en el tercio medio del esfago que comprometa casi todo el lumen esofgico. El
estudio anatomopatolgico de la lesin mostr un tumor slido, infiltrante, con caractersticas morfolgicas e inmunohistoqumicas
compatibles con un melanoma maligno. La tomografa computada de trax mostr una lesin tumoral, con crecimiento intraluminal, en
el tercio distal del esfago. La lesin comprometa el cardias y comprima la aurcula izquierda. No hubo evidencia de compromiso de
rganos adyacentes ni metstasis. La tomografa computada de abdomen y pelvis fue normal.
PREGUNTA
Cual es el pronostico del caso?
RESPUESTA
a.- 6 a 12 meses de vida.
b.- 12 a 18 meses de vida.
c.- 18 a 24 meses de vida.
d.- 24 a 30 meses de vida.

CASO CLINICO
Hombre de 59 aos, sin antecedentes mrbidos. Consult por disfagia asociada a baja de peso. La endoscopia digestiva alta demostr
una lesin polipoidea de 5 cm, pigmentada, en el tercio distal del esfago. El estudio anatomopatolgico de la biopsia endoscpica
diagnostic un tumor maligno indiferenciado. El examen fsico y los exmenes de laboratorio fueron normales.

PREGUNTA
Cual es el tipo mas probable de neoplasia que presente el caso?

RESPUESTA
a.- Leiomioma.
b.- Escamoso mucoide.
c.- Adenocarcinoma.
d.- Melanoma.

CASO CLINICO
Hombre 58 aos con disfagia. Comienza hace 2 meses con disfagia progresiva sin remisiones, primero para slidos y luego para
semislidos. Actualmente ingiere lquidos y papillas blandas. Sensacin de detencin del bolo alimenticio en regin esternal baja. Desde
hace meses sialorrea sobretodo nocturna. En algunas oportunidades ha tenido regurgitaciones. No present dolor torcico de ningn
tipo. Adelgazamiento de 10kg en los ltimos meses. No astenia, ni anorexia. AP: Fumador intenso de 2 cajillas/da. Bronqutico crnico,
etilista y enolista moderado. Se aporta una fibrogastroscopia que informa lesin vegetante e infiltrante, en el esfago, cuya biposia
inform carcinoma epidermoide.

PREGUNTA
Radiologa del trax frente y perfil, valorando elementos de remodelacin torcica (horizontalizacin costal, aplanamiento
diafragmtico) y rarefaccin pulmonar, a su vez descartar la presencia de metstasis pulmonares y/u seas poco frecuentes siendo
este estudio poco sensible para su deteccin. Cual es el tratamiento mas adecuado para el caso?

RESPUESTA
a.- Quimioterapia.
b.- Radioterapia.
c.- Reseccion quirrgica.
d.- Protesis expansible





MANUAL DE TRABAJO DEL CURSO ENARM CMN SIGLO XXI
CURSO ENARM CMN SIGLO XXI TEL: 36246001 Pharmed Solutions Institute PGINA 486

VARICES ESOFAGICAS. CIENCIAS BASICAS: Son vasos colaterales porto sistmicos, es decir, canales vasculares que unen la circulacin
venosa porta y la sistmica. Se forman como consecuencia de la hipertensin portal (una complicacin progresiva de la cirrosis),
preferencialmente en la submucosa del esfago inferior. La ruptura y el sangrado de las vrices esofgicas son complicaciones mayores
de la hipertensin portal (HTP) y se acompaan de una alta tasa de mortalidad. SALUD PUBLICA: El sangrado varicoso representa 10
30% de todos los casos de sangrado gastrointestinal alto. Es una complicacin frecuente de hipertensin portal 30-50%. Elevada
mortalidad hospitalaria 20-50%. Riesgo elevado de resangrado, 30% en primera semana y 50% durante el primer ao. 15-20% de
pacientes cirrticos que sufren una hemorragia por varices esofagogastricas fallecen en las 6 sem siguientes. PATOGENIA: La cirrosis, el
estadio terminal de la hepatopata crnica, es la causa ms comn de HTP. La presin venosa portal (P) es el producto de la resistencia
vascular (R) y el flujo sanguneo (Q) en el lecho de la vena porta (Las varices esofgicas se rompen cuando la tensin ejercida sobre su
Las VE se rompen cuando la tensin ejercida sobre su pared es excesiva y sobrepasa un valor umbral pared es excesiva y sobrepasa un
valor umbral). En la cirrosis hay un aumento tanto de la resistencia vascular intraheptica como del flujo porta. La HTP lleva a la
formacin de colaterales porto-sistmicas. Sin embargo, debido a su mayor resistencia y mayor flujo aferente venoso porta, estas
colaterales no logran disminuir la hipertensin. La mejor manera de evaluar la hipertensin portal (indirectamente) es utilizando la
medicin de la presin venosa heptica enclavada (PVHC). Para que se formen las vrices se necesita una diferencia de presin entre la
circulacin portal y sistmica (gradiente de la presin venosa, GPVH) de 1012 mmHg (aunque no es suficiente). La GPVH normal es 35
mmHg. Cuando la HTP es clnicamente significativa, hay formacin de varices esofgicas con una presin >10mmHg, y hemorragia de
varices esofgicas >12mmHg. Correlacin entre la presencia de vrices y severidad de la hepatopata: Pacientes ChildPugh A: 40%
tienen vrices. Pacientes ChildPugh C: 85% tienen vrices. Algunos
pacientes pueden presentar vrices y hemorragia en las primeras
etapas de la enfermedad, an en ausencia de cirrosis. Los pacientes
con hepatitis C y fibrosis en puente: 16% tienen vrices esofgicas. La
presencia de vrices gastroesofgicas se correlaciona con la gravedad
de la enfermedad heptica. La severidad de la cirrosis puede
clasificarse utilizando el sistema de clasificacin de ChildPugh.
DIAGNOSTICO: La esofagogastroduodenoscopa es el patrn oro para
el diagnstico de vrices esofgicas. Estndar de oro para diagnstico:
Vrices esofgicas Soehendra: GRADO I: Replecin leve, dimetro < 2
mm, apenas se eleva del esfago relajado, se acentan en posicin
cabeza abajo. GRADO II: Replecin moderada, curso serpenteante,
dimetro de 3-4 mm, restringidas a la mitad inferior del esfago.
GRADO III: Replecin completa, tensa, con dimetro > 4 mm, paredes
finas, fenmeno de vrice sobre vrice, pasan al fondo gstrico.
GRADO IV: Replecin completa, tensa, ocupan todo el esfago, a menudo combinadas con vrices gstricas o duodenales. Si no se
dispone de patrn oro, otros posibles pasos diagnsticos seran la ecografa con Doppler de la circulacin sangunea (no la ecografa
endoscpica). Si bien es una mala segunda opcin, ciertamente puede demostrar la presencia de vrices. Entre otras alternativa, se
puede utilizar la radiografa baritada (con ingestin de bario) del esfago y estmago, y angiografa y manometra de la vena porta. Es
importante valorar la ubicacin (esfago u estmago) y el tamao de las vrices, los signos de sangrado inminente, que puede ser un
primer sangrado agudo, o recurrente, y (si corresponde) debe considerarse la causa y la gravedad de la hepatopata. TRATAMIENTO:
Sangrado por varices esofgicas: Actuaciones iniciales, catteres venosos: 1-2 vas perifricas gruesas. Reserva de concentrados de
hemates. Analtica con pruebas de coagulacin. Reposicin de la volemia. En hemorragia grave, va central y sonda vesical. Reposicin
cuidadosa de la volemia (Hcto 21-27%). Prevencin de las infecciones: Norfloxacina VO 400mg cada 12hs (7 das). Ceftriaxona IV 1g da
(7 das) (Child C). Tratamiento de la encefalopata (Lactulosa). Monitoreo de la funcin renal (40 ml/hora). Intubacin endotraqueal
(encefalopata). Terapia farmacolgica: Vasoconstrictores esplcnicos; vasopresina, somatostatina, -bloqueantes no cardioselectivos
(propanolo, nanodol). Terlipresina Primera opcin; derivado sinttico de la Vasopresina pero con menos efectos secundarios,
administracin en bolos, nico frmaco que ha demostrado disminucin de la mortalidad. Somatostatina, segunda opcin es eficaz para
detener la hemorragia (por lo menos transitoriamente) en hasta 80% de los pacientes. La somatostatina puede ser superior a su
anlogo octreotida. A pesar de emplearse en dosis adecuadas, alrededor de 30% de los pacientes no responden a -bloqueantes con
una reduccin en el gradiente de presin venosa heptica (GPVH). Adems, los -bloqueantes pueden provocar efectos colaterales
tales como fatiga e impotencia, que podran disminuir la adherencia al tratamiento (especialmente con los varones jvenes), o pueden
estar contraindicados los -bloqueantes por otras razones. Venodilatadores: Nitratos solos no estn recomendados. El 5-MNI reduce la
presin porta, pero su uso en los pacientes cirrticos est limitado por sus efectos vasodilatadores. La terapia de combinacin lleva a un
efecto sinrgico en reducir la presin porta. Se ha demostrado que la combinacin de 5-MNI con -bloqueantes no selectivos tiene
efectos aditivos al reducir la presin porta y es particularmente eficaz en pacientes que no responden a la terapia inicial con -
bloqueante solo. Sin embargo, estos efectos beneficiosos pueden verse contrarrestados por los posibles efectos deletreos sobre la
funcin renal y la mortalidad a largo plazo, especialmente en los pacientes mayores de 50 aos. Por lo tanto no se recomienda el uso de
rutina del tratamiento combinado. Terapias locales: Escleroterapia o ligadura endoscpica de las vrices (EVL). No tiene efecto sobre el
flujo o la resistencia porta. Terapia de derivacin. Quirrgica o radiolgica (derivacin portosistmica intraheptica transyugular, TIPS).
Reduce la presin porta. La escleroterapia endoscpica y la ligadura varicosa logran interrumpir el sangrado en hasta 90% de los
pacientes. La ligadura endoscpica con banda es una escleroterapia eficaz, pero se acompaa de menos efectos colaterales. Sin
embargo esta tcnica puede ser ms difcil de aplicar que la escleroterapia en los pacientes con sangrado activo severo. La derivacin
TIPS es una buena alternativa cuando fracasan el tratamiento endoscpico y la farmacoterapia. El uso de taponamiento con baln est
disminuyendo ya que existe un alto riesgo de resangrado despus de desinflar el baln y hay un riesgo de complicaciones mayores. Sin
embargo, el taponamiento con baln logra detener la hemorragia por lo menos temporalmente en la mayora de los casos, y puede ser
utilizado en regiones del mundo donde no se disponga fcilmente de EGD y TIPS. Puede ayudar a estabilizar al paciente para ganar
tiempo y acceder a EGD y/o TIPS ms adelante. La hemorragia aguda de las vrices a menudo se acompaa de infeccin bacteriana
MANUAL DE TRABAJO DEL CURSO ENARM CMN SIGLO XXI
CURSO ENARM CMN SIGLO XXI TEL: 36246001 Pharmed Solutions Institute PGINA 487

debido a translocacin intestinal y trastornos de la motilidad. La antibiticoterapia profilctica ha demostrado aumentar la tasa de
sobrevida.

CASO CLINICO
Masculino de 43 aos de edad que consulta por distencin abdominal y episodios de sincope. El paciente tiene antecedentes de cirrosis
heptica de causa alcohlica (estadio C de Child-Pugh), hipertensin portal con multiples episodios de sangrado por varices esofgicas y
sndrome asctico-edematoso. El paciente continuaba con el consumo de alcohol sin cumplir la medicacin de espironolactona y
propanolol. Al Ingreso se encontraba vigil y orientado con TA de 85/50 mmHg y FC 100 lpm. FR 18.

PREGUNTA
Cual es la conducta a seguir en este momento?

RESPUESTA
a.- Reposicin de liquidos y hemoderivados.
b.- Realizar TAC para buscar sangrado.
c.- Conducta expectante y medidas antiamonio.
d.- Preparar para laparatomia exploratoria.

PREGUNTA
Se realizan estudios y medidas de rutina para la patologa de base, asi como bsqueda de hemorragia, por TAC se observa sangrado
peritoneal, y se prepara al paciente para ciruga, cual de las siguientes causas de hemoperitoneo es mas frecuente?

RESPUESTA
a.- Rotura de vrices esofgicas.
b.- Rotura de canales linfticos
c.- Carcinoma hepatocelular
d.- Metstasis hepticas

PREGUNTA
Cual es la mortalidad del paciente en caso de recurrir a TIPS considerando particularmente su estadio C de Child-Pugh?

RESPUESTA
a.- 100 %.
b.- 90 %.
c.- 80 %.
d.- 70 %.

PREGUNTA
Cual de la siguientes medidas es mas apropiada para reducir el riesgo de sangrado por varices esofgicas en el caso presentado?

RESPUESTA
a.- Espironolactona.
b.- Propanolol.
c.- Propanolo e isosorbide.
d.- Espironolactona y propanolol.

CASO CLINICO
Paciente varn de 58 aos de edad fumador de 40 cigarrillos al da desde los 15 aos, bebedor de unos 70 gramos etanol/ da. Presenta
de forma sbita 3 vmitos muy abundantes de color rojo oscuro con restos semejantes a posos de caf. Exploracin fsica: TA: 86/60 y
frecuencia cardiaca 140 lpm. Consciente, orientado con intensa palidez cutneo mucosa, sudoracin y frialdad cutnea. Eupneico.
Desnutrido. Telangiectasias malares e hipertrofia parotdea. AP: mvc, AC ritmico a 140 lpm sin soplos ni extratonos. Abdomen: RHA
conservados no se palpan masas hepatoesplenomegalia de tres traveses de dedo, circulacin colateral periumbilical, no signos de
irritacin peritoneal. Extremidades: pulsos perifricos presentes, simtricos y dbiles, eritema palmar. Neurolgico normal sin flapping
ni signos de encefalopata. Hemograma: hematocrito 28%, hemoglobina 10.3 g/dl, VCM 104 fl, leucocitos 10.3 mil/mcL, plaquetas 100
mil/mcL. Perfil heptico: GPT 60 UI/L, GOT 93 UI/L, fosfatasa alcalina 183 UI/L, gamma GT 200 UI/L. Estudio de coagulacin: normal.
Endoscopia digestiva: varices esofgicas con signos de sangrado reciente, resto hemticos en estmago con mucosa gstrica normal,
duodeno hasta tercera porcin normal.

PREGUNTA
Cual es la conducta a seguir mas adecuada?

RESPUESTA
a.- Realizar escleroterapia.
b.- Restablecimiento de liquidos.
c.- Aminas vasoactivas.
d.- Colocacion de sonda de balones.
MANUAL DE TRABAJO DEL CURSO ENARM CMN SIGLO XXI
CURSO ENARM CMN SIGLO XXI TEL: 36246001 Pharmed Solutions Institute PGINA 488

CASO CLINICO
Acude a urgencias paciente de 61 aos de edad la cual refiere dolor ardoroso en la regin media del trax de predominio anterior, el
dolor ha incrementado en las ltimas semanas, refiere vomito en tres ocasiones con trazas de sangre. Tiene antecedentes de diabetes y
osteoartritis bajo tratamiento, en ltimas fechas han incrementado el dolor de sus manos por lo que aumento su dosis de
medicamente.

PREGUNTA
Cul es el manejo farmacolgico ms adecuado.

RESPUESTA
a.- Ranitidina.
b.- Omeprazol.
c.- Sales de Bismuto.
d.- Hidroxido de aluminio y magnesio.

CASO CLINICO
Se trata de paciente masculino de 45 aos de edad, alcoholismo desde hace 20 aos el cual refiere vomito con sangre en 3 ocasiones
posterior a ingestin de alcohol durante 5 das, a la exploracin fsica se observa diafortico, ansioso y desorientado, durante su
estancia presenta dos ocasiones ms vomito con trazas de sangre, sus constantes vitales son TA 110/70 mmHg, FC 87, FR 19. 150,000
Plaquetas

PREGUNTA
Cul es la conducta ms adecuada para controlar el sangrado en este caso?.

RESPUESTA
a.- Colocacin de sonda de balones.
b.- Escleroterapia.
c.- Vitamina K.
d.- Inhibidores de Bomba H+









MANUAL DE TRABAJO DEL CURSO ENARM CMN SIGLO XXI
CURSO ENARM CMN SIGLO XXI TEL: 36246001 Pharmed Solutions Institute PGINA 489

RUPTURA ESOFAGICA. CIENCIAS BASICAS: Patologa relativamente rara/grave, que sin tratamiento puede progresar a mediastinitis y
shock sptico. Causas; traumtica, iatrogenia, cuerpos extraos, agentes corrosivos, trauma torcico, espontanea, Sndrome de
Mallory-Weiss, hematoma intramural, Sndrome de Boerhaave, neoplasias, infecciosa. DIAGNOSTICO: Representan un difcil reto
diagnstico, originando frecuentes retrasos en su tratamiento. Manifestaciones clnicas clsicas son: a) Vmitos. b) Dolor retroesternal.
c) Enfisema mediastnico (Trada de Macler). Signos clnicos sutiles e inespecficos; hipotensin, sepsis, fiebre. SINDROME DE
MALLORY-WEISS; Este sndrome se caracteriza por hemorragia digestiva superior despus del vomito repetido o esfuerzos fuertes y
prolongados para vomitar o toser. Aproximadamente 4 casos por cada 100.000 personas. Ms a los hombres que a las mujeres y puede
aparecer a cualquier edad. 5% de las hemorragias de la parte alta del tracto gastrointestinal. Suele ocurrir en varones con historia de
consumo de alcohol y se presenta clnicamente como vmitos repetidos seguidos de hematemesis. Desrdenes alimenticios y en
algunas evidencias se demuestran la presencia de hernia de hiato como una condicin pre-disponente. Puede estar asociado a la
ingesta repentina de salicilatos. El mecanismo es similar a la perforacin esofgica espontanea, o sea el aumento sbito en la presin
intrabdomional contra la glotis cerrada en un paciente con hernia hiatal. Los desgarros de Mallory-Weiss se caracterizan por
hemorragia arterial que puede ser masiva. Puede aparecer con cierta frecuencia en el punto de unin entre el esfago y el
estmago. Para establecer el diagnstico es preciso mantener un alto ndice de sospecha en un paciente que presenta hemorragia
digestiva alta despus de vmito y arqueo prolongado. La endoscopia confirma el diagnostico al identificar una o ms fisuras
longitudinales en la mucosa del estmago herniado. En casi todos los pacientes la hemorragia cede en forma espontnea (24-48hrs)
con tratamiento mdico. Cicatrizacin en aproximadamente 10 das. Solo en ocasiones se requiere la operacin para detener la prdida
de sangre. El procedimiento consiste en laparotoma y gastrostoma alta con sutura del desgarro lineal. SNDROME DE BOERHAAVE:
Consiste en una rotura transmural completa del esfago, generalmente en el lado izquierdo a unos 2-3 cms por encima de la unin
gastroesofgica, con salida de la sangre hacia el mediastino. Afecta con ms frecuencia a hombres (60-78%) y se relaciona con vmitos
violentos y consumo de alcohol. Clnicamente se presenta con un cuadro dramtico de dolor, vmitos, neumotrax, enfisema
subcutneo y shock. No suele haber hematemesis (a diferencia del Mallory-Weiss). Un signo radiolgico clsico es la V de Naclerio.
Enfisema mediastnico lineal en ngulo costovertebral izq (entre aorta inferior y diafragma). A menudo existe derrame pleural de
predominio izquierdo.

CASO CLINICO
Se trata de masculino de 34 aos de edad el cual se encuentra en urgencias trado por ambulancia ya que en su trabajo presento
vomito con sangre fresca y abundante en ms de 4 ocasiones, a la exploracin fsica se encuentra diafortico, plido, con pulsos
disminuidos, taquicardico e hipotenso, se estabiliza, su hematocrito es de 41 % y plaquetas de 45,000.

PREGUNTA
Cul es el tratamiento adecuado para mejorar el pronstico del paciente.

RESPUESTA
a.- Endoscopia y escleroterapia.
b.- Sonda de balones.
c.- Transfusin sangunea.
d.- Desmopresina y Inhibidor de H+.

CASO CLINICO
Se encuentra en urgencias por sangrado de tubo digestivo alto, refiere familiar que encontr al paciente inconsciente en el bao,
refiere que tambin presento dos evacuaciones ftidas abundantes mas vomito con sangre fresca, agrega que es alcohlico desde hace
25 aos, no ha recibido tratamiento previo hasta la fecha.

PREGUNTA
Cul es la complicacin asociada que presenta peor pronstico.

RESPUESTA
a.- Trastorno renal asociado.
b.- Trastorno plaquetario secundario.
c.- Hipertensin portal.
d.- Estado de choque.














MANUAL DE TRABAJO DEL CURSO ENARM CMN SIGLO XXI
CURSO ENARM CMN SIGLO XXI TEL: 36246001 Pharmed Solutions Institute PGINA 490

HERNIA HIATAL. CIENCICAS BASICAS: Es el prolapso del estmago proximal hacia el trax a travs del hiato esofgico del diafragma. La
hernia hiatal es una condicin esencialmente adquirida que no slo es la ms frecuente de las hernias diafragmticas, sino una de las
anormalidades ms frecuentes que afectan el TGI superior. El hiato esofgico del diafragma se encuentra localizado a la izquierda de la
lnea media a nivel de la dcima vrtebra torcica; es un anillo musculotendinoso compuesto por fibras de los pilares derecho e
izquierdo del diafragma, que provienen de la cara anterior y discos intervertebrales de las cuatro primeras vrtebras lumbares, rodean
la aorta y el esfago y se insertan en el centro tendinoso del diafragma. Aunque existen variaciones anatmicas, se ha encontrado que
en ms del 80% de los casos, el hiato esofgico se forma principalmente de fibras del pilar derecho del diafragma. CLASIFICACION: Tipo
I o hernia hiatal por deslizamiento (axial) donde se presenta desplazamiento superior de la unin esfago-gstrica hacia el mediastino
posterior. Se denomina hernia por deslizamiento ya que tiene un saco de peritoneo parietal parcial, cuya pared posterior est
formada por el estmago. Tipo II o hernia paraesofgica caracterizado por desplazamiento superior del fondo gstrico, anterior y lateral
al esfago, con la unin esfago-gstrica localizada en su posicin intraabdominal normal. Tipo III o mixta donde hay desplazamiento
superior tanto de la unin esfago -gstrica como del fondo gstrico. El 85-90% de las hernias hiatales son de tipo I, en tanto que las
hernias paraesofgicas puras son encontradas muy infrecuentemente. SALUD PUBLICA: La incidencia de hernia hiatal se estima en 5
por 1000 en la poblacin general, aunque una verdadera incidencia es difcil de determinar debido a que un gran nmero de pacientes
son asintomticos. La edad de presentacin ms frecuente es entre 4 a 6 dcada de la vida y no existe diferencia en cuanto al sexo,
aunque las hernias paraesofgicas son ms frecuentes en mujeres. DIAGNOSTICO: Cuando se hacen manifiestas, la sintomatologa est
dada bsicamente por pirosis, regurgitacin y dolor retroesternal, sntomas clsicos del RGE que es la manifestacin ms significante
en los pacientes con hernia hiatal. Puede presentarse disfagia generalmente asociada a esofagitis, estenosis pptica, anillo de Schatzki
(pliegue mucoso en el esfago distal, dentro de los 3 mm proximal a la unin escamocolumnar, que siempre est asociada con hernia
hiatal y constituye una causa de disfagia en estos pacientes) o por la compresin de los pilares diafragmtica en la porcin herniada del
estmago. Las hernias paraesofgicas en general son asintomticas, incluso cuando alcanzan grandes tamaos. Grandes herniaciones
pueden producir dolor retroesternal o disnea por disminucin de la reserva respiratoria. Se puede presentar disfagia secundaria a la
compresin esofgica por la hernia o a la rotacin de la UEG dentro del saco herniario. Sntomas de RGE se presentan tambin en un
alto porcentaje. En pacientes con ERGE documentado, se ha encontrado adems una relacin directa entre el tamao de la hernia
hiatal y el grado de disfuncin del EEI, la duracin de los episodios de reflujo, el aclaramiento cido y por consiguiente con la severidad
de la esofagitis. La funcin como esfnter del diafragma est dada principalmente durante la inspiracin y situaciones de estrs
dinmico, como durante la deglucin y en incrementos sbitos de la presin intraabdominal (tos), de tal forma que los episodios de
reflujo relacionados a incremento de la presin intraabdominal son ms probables que ocurran en pacientes con hernia hiatal.
Sangrado oculto o moderado puede encontrarse hasta en un tercio de los pacientes con hernia hiatal por deslizamiento sintomtica,
siendo el sangrado masivo poco frecuente. El volvulus gstrico puede ser organoaxial cuando el estmago rota a lo largo de su eje
longitudinal o mesoentero axial cuando rota sobre el eje que une la curvatura menor con la mayor. La laxitud o ausencia de los
ligamentos de fijacin gstrica son la causa primaria del volvulus gstrico. Al incrementarse la rotacin del estmago se produce
compromiso del flujo sanguneo y del retorno venoso del estmago herniado, llevando a necrosis, perforacin, sepsis e incluso la
muerte. Radiografa de trax se puede evidenciar una masa dependiente de tejidos blandos en el mediastino posterior, con nivel
hidroareo en el caso de grandes hernias. Los estudios con medio de contraste baritados son ms exactos si se logra definir la relacin
del hiato esofgico del diafragma con la UEG. La presencia de una indentacin por encima del diafragma, lo que usualmente se
considera como el sitio de transicin entre el cardias gstrico y el vestbulo esofgico, implica la existencia de una hernia hiatal. En el
caso de hernias paraesofgicas, los estudios con medio de contraste muestran la porcin del fondo gstrico situado por encima del
diafragma con la UEG ubicada en posicin normal a nivel del diafragma; adicionalmente puede demostrarse la presencia de un volvulus
gstrico. Endoscopia: Normalmente la unin de la mucosa escamocolumnar (lnea Z), que corresponde aproximadamente a la
localizacin de la UEG, se encuentra a menos de 2 cm por encima del hiato diafragmtico, de tal forma que una distancia mayor entre
estas estructuras es consistente con la presencia de una hernia hiatal. La posicin del hiato diafragmtico puede hacerse ms evidente
durante una inspiracin profunda. Otros estudios como la manometra y pHmetra, no son tiles para el diagnstico de hernia hiatal,
pero son de utilidad en el estudio del RGE asociado. La manometra es fundamental para determinar la localizacin, tamao y presin
del EEI, como tambin para descartar trastornos de la motilidad esofgica previa al tratamiento quirrgico. TRATAMIENTO:
Modificacin en el estilo de vida, uso de drogas supresoras de la secrecin cida y agentes proquinticos. El manejo quirrgico de la
hernia hiatal sintomtica tiene varias indicaciones, y/o por manifestaciones directas de la hernia hiatal, como: 1. hernia hiatal
encarcelada con disfagia, 2. dolor torcico asociado a hernia hiatal gigante, 3. pacientes con severa deficiencia de hierro secundaria a
las erosiones o ulceraciones en la hernia hiatal, 4. hernia paraesofgica. Existen varias opciones quirrgicas de funduplicatura total o
parcial, que pueden ser realizadas por va transabdominal (funduplicatura de Nissen, Hill, Toupet) o transtorcica; o por
manifestaciones directas de la hernia hiatal, como: 1. hernia hiatal encarcelada con obstruccin sintomtica y disfagia, 2. dolor torcico
asociado a hernia hiatal gigante, 3. pacientes con severa deficiencia de hierro secundaria a las erosiones o ulceraciones en la hernia
hiatal, 4. hernia paraesofgica. Existen varias opciones quirrgicas de funduplicatura total o parcial, que pueden ser realizadas por va
transabdominal (fundoplicatura de Nissen, Hill, Toupet) o transtorcica (funduplicatura de Nissen o Belsey - Mark IV) ya sea en forma
abierta o por ciruga mnimamente invasiva, pero que en general cumplen los mismos principios quirrgicos que son: reduccin de la
hernia hiatal, cierre del hiato esofgico, restablecer la funcin del EEI, reposicionar el esfago intraabdominal y crear un mecanismo de
vlvula antirreflujo. En aquellos casos de acortamiento esofgico se debe realizar una gastroplastia de Collis para alargar el esfago y
posteriormente la funduplicatura total o parcial, sin tensin sobre el esfago. El curso impredecible de las hernias paraesofgicas, ha
llevado a considerar a la mayora de los cirujanos, que todas deben ser corregidas quirrgicamente una vez hayan sido diagnosticadas,
aun en ausencia de sintomatologa e independiente de su tamao, dado su potencial de complicaciones incluyendo el volvulus, la
estrangulacin y perforacin gstrica que obligaran a intervenciones de urgencia que conllevan mayor mortalidad

CASO CLINICO
Acude paciente a consulta externa de ciruga para valoracin preoperatoria, la paciente tiene 41 aos de edad y refiere que desde hace
4 aos ha presentado reflujo nocturno ha recibido tratamiento con ranitidina, omeprazol y cisaprida, hace 6 meses recibi tratamiento
por presencia de H. pilory sin embargo ahora presenta tos irritativa.
MANUAL DE TRABAJO DEL CURSO ENARM CMN SIGLO XXI
CURSO ENARM CMN SIGLO XXI TEL: 36246001 Pharmed Solutions Institute PGINA 491


PREGUNTA
Cul es el criterio ms importante para considerar un procedimiento quirrgico definitivo.

RESPUESTA
a.- Cambios displasicos.
b.- Reflujo gstrico.
c.- Falla de tratamiento.
d.- Estado de salud.

PREGUNTA
Cual es la complicacin mas frecuente de este padecimiento?

RESPUESTA
a.- Esofagitis.
b.- Esofago de barett.
c.- Cancer esofgico.
d.- Estrangulacin esofgica.

PREGUNTA
Se realizo endoscopia la cual no reporto datos de esofagitis, sin embargo se obervaron carateristicas compatibles con hernia tipo I, la
biopsia confirmo la presencia de H, pilory, cual es el tratamiento que presenta mejor pronostico para el caso?

RESPUESTA
a.- Omeprazol, metronidazol y amoxicilina.
b.- Omeprazol, amoxicilina y claritromicina.
c.- Nessen, omeprazol, amoxicilina y claritromicina.
d.- Porfimero sodico, nessen, omeprazol, amoxicilina y claritromicina.

PREGUNTA
El paciente regresa a consulta de seguimiento un ao despus, refiere que se sinti muy bien durante los tres meses subsecuentes a la
teraputica, posteriormente se presento pirosis, regurgitacin y disfagia, actualmente refiere que duerme con tres almohadas, ha
perdido peso ya que no puede comer sin presentar sntomas, refiere adems dolor epigstrico que se irradia a la espalda, asi como
fatiga y astenia, cual es el diagnostico mas probable a la exploracin se apresia leve tinte ictrico, ganglio de irish y virchow?

RESPUESTA
a.- Cancer pulmonar.
b.- Cancer esofgico.
c.- Cancer gstrico.
d.- Cancer heptico.

CASO CLINICO
Paciente varn de 44 aos que consulta por cuadro de pirosis, ardores y dolor epigstrico irradiado a zona retroesternal de dos aos de
evolucin. Fue tratado con omeprazol, metoclopramida sin resultados adecuados, En estudio endoscpico se observa hernia hiatal sin
signos de esofagitis, que se confirma en trnsito esofagogastricoduodenal donde adems se observa reflujo gastroesofgico
espontneo. En la manometra esofgica se aprecia disminucin del tono del esfnter esofgico inferior. Se realiza radiografa de trax
donde se observa la existencia de masa de densidad grasa que ocupa todo el ngulo cardiofrnico derecho, hallazgos que se confirman
en TAC toracoabdominal.

PREGUNTA
Considerando el cuadro clnico asi como los estudios de gabinete cual es el tipo de hernia mas probable?

RESPUESTA
a.- Hernia hiatal tipo I
b.- Hernia hiatal tipo II.
c.- Hernia hiatal y de morgagni.
d.- Hernia hiatal y de bochdalek

PREGUNTA
Cual de las siguientes manifestaciones es mas frecuente encontrar para pensar en esta entidad nosolgica?

RESPUESTA
a.- Dolor abdominal.
b.- Oclusin intestinal.
c.- Dificultad respiratoria.
d.- Regurgitacion gastrointestinal.
MANUAL DE TRABAJO DEL CURSO ENARM CMN SIGLO XXI
CURSO ENARM CMN SIGLO XXI TEL: 36246001 Pharmed Solutions Institute PGINA 492


PREGUNTA
Se ha reportado esta asociacin en menos del 3 % de casos de hernias hiatales, cual de los siguientes rganos es el menos frecuente
que se comprometa?

RESPUESTA
a.- Jejuno.
b.- Colon.
c.- Higado.
d.- Epipln.

PREGUNTA
De los siguientes paraclinicos es mas habitual para su diagnostico?

RESPUESTA
a.- TAC.
b.- Radiografia de torax.
c.- Endocopia.
d.- Estudio de transito esfago-gastro-duodenal.

PREGUNTA
Considerando el caso clnico asi como el diagnostico confirmado, porque esta indicado el tratamiento quirugico definitivo y a la
brevedad.

RESPUESTA
a.- Riesgo a malignizacin.
b.- Riesgo de estangulacin.
c.- Riesgo de broncoaspiracin.
d.- Riesgo a la invaginacin.





































MANUAL DE TRABAJO DEL CURSO ENARM CMN SIGLO XXI
CURSO ENARM CMN SIGLO XXI TEL: 36246001 Pharmed Solutions Institute PGINA 493

GASTRITIS. CIENCIAS BASICAS: Es una enfermedad inflamatoria aguda o crnica de la mucosa gstrica producida por factores
exgenos o endgenos que produce sntomas disppticos atribuibles a la enfermedad y cuya existencia se sospecha clnicamente, se
observa endoscpicamente y se requiere confirmacin histolgica. La gastritis es etiolgicamente multifactorial, de los que el ms
comn es la infeccin por Helicobacter Pylori. Factores exgenos; Helicobacter pylori, AINES, imitantes gstricos, drogas, alcohol,
tabaco. Factores endgenos; acido gstrico, pepsina, bilis, jugo pancretico, uremia, inmunes SALUD PUBLICA: Es una entidad de
elevada morbilidad a nivel mundial. PATOGENIA: El dao de la mucosa gstrica depende del tiempo de permanencia del factor o
factores lesionates, jugando un rol importante la capacidad que tiene la mucosa gstrica a travs de la denominada barrear gstrica
para resistir a estos factores o a los efectos deletreos de sus propias
secreciones. La barrea gstrica est constituida por componentes pre
epiteliales (barrera de moco, bicarbonato, fosfolpidos), epiteliales
(capacidad de restitucin de epitelio, gradiente elctrico que previenen la
acidificacin celular, los transportadores acidobsicos que transportan el
bicarbonato hacia el moco, prostaglandinas, y oxido ntrico) y sub epiteliales
(flujo sanguneo que descarga nutrientes y bicarbonato en el epitelio y
adherencia y extravasacin de los leucocitos, que inducen lesin histica y
quedan suprimidos por la prostaglandinas endgenas). El trastorno de uno u
ms de los componentes defensivos, originan lesin de la mucosa
permitiendo la accin del cido, proteasas y cidos biliares en mayor o
menor grado y que pueden llegar hasta la lmina propia, sitio en el que
producen lesin vascular, estimulan las terminaciones nerviosas y activan la
descarga de histamina y otros mediadores. Las causas ms comunes de
lesin son por H. pylori, lesiones por estrs y AINES. DIAGNOSTICO: Clnicas;
Las gastritis pueden ser totalmente asintomticas y en caso de existir
sntomas estos no son propios, sino atribuibles a ella, como es la presencia
de ardor, o molestias postprandiales en epigastrio, llenura precoz, RGE,
nausea, distensin abdominal, pirosis, sntomas que tambin pueden estar
presentes en dispepsia no ulcerosa, lceras o neoplasias gstricas o
duodenales y an en el colon irritable. Adems pueden manifestarse con
hemorragias crnicas o agudas que podran llegar a ser masivas con hematemesis y melena. Hallazgos endoscpicos: Los signos
endoscpicos asociados a esta entidad incluyen edema, eritema, mucosa hemorrgica, punteados hemorrgicos, friabilidad, exudados,
erosiones, nodularidad, pliegues hiperplsicos, presencia de signos de atrofia de la mucosa dada por visualizacin de vasos submucosos
con aplanamiento o prdida de los pliegues acompaados o no de placas blanquecinas que corresponden a reas de metaplasma
intestinal. Estos signos endoscpicos pueden localizarse topogrficamente a nivel del antro, cuerpo o en todo el estmago,
denominndose gastritis antrales, gastritis corporal o pangastritis respectivamente. Hallazgos histolgicos: No se debe abusar del
diagnstico de gastritis, por lo que se requiere realizar la biopsia para confirmacin histolgica, establecer la presencia o ausencia de
Helicobacter pylori o de otras formas de gastritis especficas. Exmenes de laboratorio: Las pruebas de laboratorio pueden usarse para
determinar algunas causas de gastritis, como en el caso del Helicobacter pylori a travs de mtodos invasivos como la endoscopa y
biopsias para el estudio histolgico, realizar la tcnica de la ureasa rpida, el cultivo y o el empleo de mtodos no invasivos como la
serolgica para Ig G, la deteccin de antgeno en las deposicin, y la prueba del aliento del C13 o C14 espirado con sensibilidades /
especificidades de o ms de 90/90 % a excepcin de la serolgica 80/90% y el cultivo 50/100%. La gastritis aguda se caracteriza por un
infiltrado inflamatorio que es predominantemente neutrofilico y es usualmente transitorio en su naturaleza, puede acompaarse de
hemorragia de la mucosa, erosiones y si las lesiones son muy severas asociarse a sangrado. La gastritis crnica se caracteriza por un
infiltrado con linfocitos, clulas plasmticas o ambas, si adems presentan PMN toma la denominacin de gastritis crnica activa.
TRATAMIENTO: Dieta sin sustancias irritantes (caf, tabaco, alcohol, aj ) as como tambin drogas que contrarresten la agresin de la
barrera gstrica indicando ya sea anticidos orales, citoprotectores de la mucosa gstrica (sucralfato, bismuto, misoprostol),
antagonistas de receptores H2, Inhibidores de la bomba de protones, a los que se puede aadir gastrocinticos (metoclopramida,
domperidona, cisaprida, mosaprida, cinitaprida) si existe evidencias de trastornos de motilidad gastroesofgica o gastroduodenal. Los
inhibidores de bomba de protones son ms efectivos que los bloqueadores H2 en el manejo de gastritis IHQ abierta totalmente el
manejo debe de ser hospitalario. Omeprazol o pantoprazol 80 mg intravenosos en bolo seguidos de infusin continua de 8 mg/h por 72
horas). El mantener el PH por arriba de 4 ayuda a restituir la mucosa gstrica. El omeprazol es el frmaco de primera eleccin para el
tratamiento de la Gastritis Aguda. Los esquemas de primera lnea pueden ser triples o cudruples (cuando se agrega una sal de
bismuto). El basado en las tasas de xito alcanzadas en prcticamente todas las regiones del mundo se recomienda el uso de 1 g de
amoxicilina dos veces al da, 500 mg de claritromicina dos veces al da y dosis doble de un IBP durante 14 das. El esquema de segunda
lnea recomendado para la erradicacin de Helicobacter pylori es levofloxacino 500 mgs cada 24 hrs, amoxicilina 1 gr cada 12 hrs,
inhibidor de bomba de protones dosis estndar cada 12 hrs por 10 a 14 das o moxifloxacino 400mg al da, amoxacilina 1 gr cada 12 hrs
e inhibidor de bomba de protones cada 12 hrs. En el caso de alergia a la amoxicilina, se puede emplear como alternativa tetraciclina
(500 mg cuatro veces/da) o metronidazol (250 mg cuatro veces/da). Las sales de bismuto, en forma de subsalicilato o subcitrato,
deben administrarse en cuatro tomas al da y en dosis promedio de 525 mg. Para erradicar Helicobacter pylori hay que confirmar la
erradicacin de la bacteria. H.pylori: La infeccin por Helicobacter pylori se calcula que afecta la mitad de la poblacin mundial, con
mayor prevalencia en pases en desarrollo, llegando a cifras hasta del 90%, asociada a niveles socio econmicos bajos, adquirindose a
edades ms tempranas en comparacin con los pases desarrollados, Todos los sujetos que presentan Helicobacter Pylori desarrollan
gastritis, un 15 a 20% presentan ulcera pptica y menos del 1% adenocarcinoma gstrico existiendo variaciones regionales. La
infeccin por estas bacterias se inicia en el antro gstrico, ubicndose a lo largo de la superficie epitelial, se extiende hacia el cuerpo
llevando en los casos crnicos de larga evolucin a atrofia, metaplasia intestinal que pueden terminar en displasia y finalmente
adenocarcinoma.

MANUAL DE TRABAJO DEL CURSO ENARM CMN SIGLO XXI
CURSO ENARM CMN SIGLO XXI TEL: 36246001 Pharmed Solutions Institute PGINA 494

CASO CLINICO
Se trata de paciente femenino de 21 aos de edad la cual acude a servicio mdico de la escuela refiriendo presentar nauseas con
vomito con mnimo contenido gstrico, niega sangrados, agrega gases y eructos ocasionales, se alimenta dos veces al da y come lo que
puede al salir de la escuela rumbo a su trabajo.

PREGUNTA
Cul de las siguientes medidas mejora el pronstico.

RESPUESTA
a.- Tratamiento para H. pilory.
b.- Cambio de hbitos dietticos.
c.- Endoscopia con biopsia.
d.- Doble esquema anticido.

CASO CLINICO
Se ingresa a paciente femenino de 51 aos de edad la cual acude a urgencias por presencia de dolor epigstrico, vomito y nauseas,
adems agrega disminucin del apetito, agrega que desde hace 3 meses ha recibido tratamiento por anemia previamente diagnostica
para lo cual indicaron hierro.

PREGUNTA
Cul es el mtodo diagnostico ms adecuado.

RESPUESTA
a.- Serie gastroesofagica.
b.- Endoscopia.
c.- La clnica es suficiente.
d.- Biometria hemtica.

CASO CLINICO
Se trata de masculino de 31 aos de edad el cual acude a consulta por cefalea tensional, al interrogatorio refiere que presenta gastritis
recurrente y ha sido sometido a diversos tratamientos farmacolgicos para este problema pero han sido incompletos, fue sometido a
endoscopia donde se diagnostico gastritis crnica con presencia de H. pilory, existen antecedente en la familia de cncer gstrico por lo
que el paciente se encuentra ms preocupado.

PREGUNTA
Considerando el cuadro clnico y el diagnostico endoscpico cual es la conducta farmacolgica ms adecuada.

RESPUESTA
a.- Metronidazol, claritromicina y omeprazol.
b.- Claritromicina, amoxicilina y omeprazol.
c.- Metronidazol, sales de bismuto y ranitidina.
d.- Amoxicilina, metronidazol y pantoprazol.




MANUAL DE TRABAJO DEL CURSO ENARM CMN SIGLO XXI
CURSO ENARM CMN SIGLO XXI TEL: 36246001 Pharmed Solutions Institute PGINA 495

ULCERA GASTRICA Y DUODENAL. CIENCIAS BASICAS: La ulceras duodenales casi siempre estn a 1-2cm del ploro. A menudo se
acompaan de hipersecrecin de cido. Las ulceras gstricas se dividen en varios tipos: Tipo I; la ms frecuente, en la parte proximal del
antro o del cuerpo. Tipo II; secundaria a ulcera duodenal con estenosis pilrica. Tipo III; ulcera prepilrica o del canal pre pilrico, sus
causas son similares a las de la ulcera duodenal. SALUD PUBLICA: La mayor incidencia de la ulcera gstrica es entre los 50-65 aos de
edad. La mayor parte de casos de ulcera duodenal se presenta en la cuarta dcada de la vida. PATOGENIA: 3 causas bsicas:
hipersecrecin de cido, colonizacin por H. pylori y AINES. H.pylori se encuentra en 95% de los casos de enfermedad ulcerosa
duodenal y en 80% de enfermedad por ulcera gstrica. Sin embargo resulta difcil probar la relacin causa-efecto. Los AINES suprimen la
sntesis de prostaglandinas y debilitan la barrear mucosa. Entre 10-30% de los consumidores crnicos de estos frmacos presenta
ulcera pptica. Patogenia de ulcera duodenal: En general los pacientes secretan ms acido por aumento en la cantidad de clulas
parietales, principales o factores trficos (gastrina). H. pylori incrementa la liberacin de gastrina y produce duodenitis. Algunos
pacientes tambin tienen trastornos de la motilidad con vaciamiento gstrico rpido de lquidos. Lo que expone al duodeno a una
cantidad de cido mayor de la normal. La
anormalidad fisiolgica ms frecuente es la
disminucin de la secrecin de bicarbonato.
Patogenia de ulcera gstrica: El defecto bsico
generalizado radica en la defensa de la mucosa
gstrica contra el cido y la pepsina. El gasto acido
es normal o apenas detectable, en contraste con la
ulcera duodenal. La anormalidad bsica suele ser
el reflujo del contenido duodenal. Es posible que
exista disfuncin pilrica, tambin secundaria al
tabaquismo. El reflujo de cidos biliares, lisolecitina
y secreciones pancreticas ejerce un efecto nocivo
y daan la mucosa. El ASA tiene un efecto similar.
DIAGNOSTICO: Clnica; dolor es lo ms frecuente,
es epigstrico agudo o clico. El dolor de la ulcera
duodenal se presenta muchas horas despus de
una comida, cuando el bulbo duodenal esta vaco;
se alivia con alimentos y sustancias alcalinas. Por el contrario el dolor de la ulcera gstrica, se intensifica con la ingesta de alimentos. El
dolor suele ser crnico y recurrente. Otras manifestaciones son nausea, prdida de peso y ligera sensibilidad. Por lo general el
diagnstico se establece por endoscopia de tubo digestivo superior o una serie esofagogastroduodenal. La endoscopia es necesaria en
todos los casos de ulcera gstrica, debido al riesgo de cncer, aqu se obtienen el material para la deteccin de H. pylori.
TRATAMIENTO: En pacientes jvenes se basa solo en los sntomas, sin estudios. En ancianos es necesario realizar endoscopia por riesgo
de tumor maligno. Con una serie radiolgica de tubo digestivo superior de doble contraste se detecta 90% de las ulceras gstricas y
duodenales. Medico; 1) Neutralizar el cido gstrico. 2) Inhibir la secrecin. 3) Proteger la mucosa gstrica de la lesin. Los anticidos
neutralizan el cido del estmago. Los antagonistas de los recetores H2 bloquean dichos receptores en las clulas parietales. Los
frmacos ms potentes son los bencimidazoles sustituidos que bloquean la H+/K+ ATPasa (IBP) Y que funcionan incluso en pacientes con
gastrinoma. La mucosa gstrica puede protegerse con anlogos de las prostaglandinas E2 (misoprostol) que incrementa el flujo
sanguneo de la mucosa y la produccin de bicarbonato y moco. El sucralfato se une a las protenas de los crteres ulcerosos, por lo
que favorece la cicatrizacin. El bismuto cubre la protena expuesta y tiene actividad contra H. pylori. Quirrgico: Ulcera duodenal;
vagotoma ultra selectiva COMPLICACIONES: Perforacin (5-10%), Hemorragia y obstruccin (<5%). Entre 15-20% de las ulceras
ppticas producen hemorragia macroscpica (gstrica o duodenal). Los pacientes refieren melena o hematemesis (gstrica).
SINDROME DE ZOLLINGER-ELLISON: Se identifica en 0.1-1% de todos los pacientes con enfermedad ulcerosa pptica y cerca de 20% de
estos padece tambin sndrome de neoplasia endocrina mltiple tipo I. Los sntomas incluyen hipergastrinemia y ulceracin pptica
grave secundaria a un gastrinoma. Es probable que las ulceras sean mltiples y ms distales. El gasto basal de acudo es muy alto, el
nivel de gastrina puede ser superior a 1000pg/ml. Es posible inducir niveles equvocos de gastrina con la administracin IV de calcio o
secretina. Los gastrinomas son neoplasias de enrome malignidad (>90%), pero de crecimiento lento; 50% de los tumores tiene
metstasis ganglionares para el momento del diagnstico.

CASO CLINICO
Acude paciente femenino de 41 aos de edad para revisin de tratamiento previo por anemia perniciosa, los resultados de laboratorio
no fueron trados por la paciente, sin embargo la endoscopia afirmo el diagnostico, refiere que ha presentado dolor en el cuadrante
superior derecho, se realizo un USG de vas biliares con presencia de lito en vescula biliar y resto de vas normales.

PREGUNTA
Cul es el diagnostico diferencial para iniciar tratamiento correcto.

RESPUESTA
a.- Serie gastroesofagica.
b.- Frotis de sangre perifrica.
c.- Tomografa abdominal.
d.- Biopsia gstrica.




MANUAL DE TRABAJO DEL CURSO ENARM CMN SIGLO XXI
CURSO ENARM CMN SIGLO XXI TEL: 36246001 Pharmed Solutions Institute PGINA 496

CANCER GASTRICO. CIENCIAS BASICAS: Es un tipo de crecimiento tisular maligno producido por la proliferacin contigua de clulas
anormales con capacidad de invasin y destruccin de otros tejidos y rganos, en particular el esfago y el intestino delgado. En las
formas metastasicas las clulas tumorales pueden infiltrar los vasos linfticos de los tejidos, diseminarse a los ganglios linfticos y,
sobrepasando esta barrera, penetrar en la circulacin sangunea y diseminarse a cualquier rgano del cuerpo. Factores de riesgo:
Ambientales; exceso de consumo de sal, nitritos, nitrosaminas, aminas aromticas, bajo consumo de frutas y verduras, infeccin por H.
pylori, VEB (ca de cardias), trabajadores del metal mineros caucho, polvo asbesto. Genticos; historia familiar, grupo sanguneo A.
Otros; gastrectoma subtotal, anemia perniciosa, plipos, Enf de Menetrier (perdedora de protenas). Factores de riesgo definitivos y
de vigilancia sugerida: Displasia de Alto Grado (75-100% evoluciona a cncer), poliposis Adenomatosa Familiar, adenomas, esfago de
Barrett. Definitivos: Metaplasia Intestinal (80% se asocian a cncer), Gastritis Crnica Atrfica, infeccin por Helicobacter pylori
carcingeno tipo I, cncer Colorrectal Hereditario sin poliposis. Probables: postGastrectoma, vagotomia Ploroplastia. Condiciones
premalignas: Mentrier y ulcera gstrica. SALUD PUBLICA: El cncer gstrico se reporta como la segunda causa de muerte a nivel
mundial con aproximadamente 600,000 muertes al ao. El adenocarcinoma representa 95%. 3er lugar en incidencia, mayores de 50
aos (69.7%). 5to lugar en incidencia, mayores de 55 aos (55.2%). Clases socioeconmicas bajas. Ms varones 2/1. El riesgo de
infeccin por H.pylori a lo largo de toda la vida en pases desarrollados es del 40
a 60%. Pases en desarrollo puede alcanzar hasta el 90%. PATOGENIA:
Secuencia de carcinognesis gstrica: Infeccin por H. pylorigastritis
crnicaatrofia gstricametaplasia intestinaldisplasia bajo/alto
gradocncer gstrico (adencarcinoma) precoz/avanzadometastasis.
ADENOCARCINOMA: Edad media: 65-74 a. En hombres a edades ms jvenes.
Mayor tasa de mortalidad en hombres. Cncer gstrico precoz/superficial: No
invade ms all de la submucosa, respetando la capa muscular, con
independencia del compromiso linftico, operado tiene buen - pronstico.
Adenocarcinoma tipo intestinal: Ms frecuente (antro, curvatura menor), de
crecimiento expansivo, prevalente en poblaciones de alto riesgo, hombres de
mayor edad (55aos), relacin H:M 2:1, largo proceso de pre cncer, rara
diseminacin, mejor pronstico. Adenocarcinoma tipo difuso: metstasis
tempranas, se presenta por igual en toda la poblacin, ms frecuente en
poblaciones jvenes (48aos), relacin H:M 1:1 relacionada con factores
genticos, Infiltra toda pared, clulas en anillo de sello, diseminacin. Carcinoma
gstrico precoz: Confinado a mucosa y submucosa independiente de la presencia o
ausencia de metstasis ganglionares. Carcinoma avanzado: Infiltra la pared
muscular como mnimo, usualmente asociado con extensin distante, escasa
posibilidad de curacin. Metstasis al momento de diagnstico en el 50% se
encuentran en estadios III o IV. Metstasis ms frecuentes: Hgado (40%) por va
hematgena Superficie peritoneal Ndulos locoregionales o distantes. Aspecto
Macroscpico de adenocarcinoma: Ulcerativo 75%, polipoideo 10%, infiltrante
10%, superficial 5%. ENFERMEDAD POR EXTENSION: Ndulo periumbilical (ndulo de la hermana mara jos). Ovarios (tumor de
krukennberg). Ganglio supraclavicular izquierdo (Virchow). Ndulo axilar izquierdo (ndulo irlands). DIAGNOSTICO: Clnica; Cncer
que no penetra la muscular propia, asintomtico 80% de los casos. Enfermedad avanzada al momento del diagnstico: prdida de peso
62%, dolor abdominal persistente 52%, nusea y vmito, anorexia, disfagia, melena, saciedad temprana, sangrado. Tumores en antro,
saciedad temprana y vmito. Tumores en cardias, disfagia, emesis fecal o alimentos no digeridos en heces, fstula gastroclica por
invasin a la pared del colon. Histologa: Adenocarcinoma: 90% de las neoplasias gstricas malignas. Linfoma gstrico 10% de estos 95%
linfoma no Hodgkin. Tumor Carcinoide gstrico 0.2%. GIST Tumores estromales GI, 50-60% ocurren en el estmago. Leiomiosarcomas.
Metstasis: Mama y melanoma. Laboratorio y gabinete: Rx esfago gastroduodenal. Endoscopia y biopsia. Rx de trax. Ecografia de
abdomen. Eco endoscopa. TAC de abdomen con y sin contraste. Laparoscopia diagnostica estadificacin. RNM grado de invasin
extraserosa y compromiso linftico. PET . Gen e cadherin en recidiva especificidad del 75%. Gen p53 (mutado) cromosoma 17
alterado en 35%. FSA Antgeno fetal sulfoglicoproteina en el 96 %. CEA y ca 19,9, Lctico-deshidrogenasa, Alfafetoprotena.
TRATAMIENTO: Operables: Tumores mviles sin extensin mtts o carcinomatosis. Inoperables: mtts pulmonares con insuficiencia
respiratoria, carcinomatosis peritoneal, ascitis, insuficiencia ponderal y/o metablica no corregibles. Irresecables: carcinomatosis no
diagnosticada en preoperatorio, fijacin por adherencias tumorales a otras vsceras, hgado y colon no son contraindicacin absoluta.
Ciruga: El tratamiento laparoscpico del cncer gstrico temprano con linfadenectomia, es factible de realizar, con resultados similares
a las tcnicas abiertas. Laparoscopia diagnostica para estadificacin. Los procedimientos quirrgicos: A) Gastrectoma total 27%. B)
Gastrectoma Subtotal 35,7%. C) Gastroenteroanastomosis 12%. D) Lap. Exploradora 18,3%. E) Gastrect. Polar Superior 7%.
Radioterapia: Tumores irresecables, con reseccin incompleta, negativa a la cirugia y / o qt, dolor, hemorragia. Quimioterapia: Estadio II
o ms: 3 ciclos de epirrubicina, cisplatino, 5-FU, pre y postoperatorio. Otros esquemas: 5FU-Doxorubicina Cisplatino o Etopsido-5FU-
Leucovorina.

CASO CLINICO
Se trata de paciente masculino de 41 aos de edad, originario de Hermosillo, inicia padecimiento hace 6 meses, caracterizado por
disminucin de peso de forma continua, agrega fatiga, adinamia, sensacin de plenitud temprana, dolor en cuadrante superior
izquierdo, a la exploracin se observa palidez generalizada, deshidratacin moderada, disneico con tos seca, niega tabaquismo
alcoholismo, existen antecedentes de tuberculosis y cncer en la familia, resto sin datos que agregar. En el laboratorio se reporto
sangre oculta en heces y anemia por deficiencia de hierro.

PREGUNTA
Cul es el tratamiento ms adecuado para el padecimiento.
MANUAL DE TRABAJO DEL CURSO ENARM CMN SIGLO XXI
CURSO ENARM CMN SIGLO XXI TEL: 36246001 Pharmed Solutions Institute PGINA 497


RESPUESTA
a.- Reseccin quirrgica.
b.- Radioterapia.
c.- Quimioterapia.
d.- Reseccin quirrgica y quimioterapia.

PREGUNTA
Cul es el signo o mtodo diagnostico ms certero para este padecimiento.

RESPUESTA
a.- Sangre en heces.
b.- Ganglio de Virchow.
c.- Serie gastrointestinal.
d.- Marcadores tumorales.

CASO CLINICO
Paciente 66 aos de edad, sexo masculino, con antecedentes mrbidos de hipertensin arterial y diabetes mellitus (DM) tipo 2 en
tratamiento. Ingresa por cuadro clnico de 4 meses de evolucin caracterizado por astenia, adinamia, ardor epigstrico, pirosis y
regurgitacin, asociado a lesiones drmicas, de aparicin sbita en ambos pies, no pruriginosas. El examen fsico revel un paciente en
buen estado general, en el que destacaba nicamente presencia de lesiones queratcicas, seborreicas e hiperpigmentadas en ambos
pies lo cual asociado a historia clnica sugera signo de Leser-Trlat.

PREGUNTA
Cual es la conducta a seguir mas adecuada.

RESPUESTA
a.- Realizar endoscopia.
b.- Realizar TAC.
c.- Realizar IRM.
d.- Realizar Biopsia de lesin drmica.

PREGUNTA
Posterior a diversos estudios incluyendo biopsia de lesin drmica se diagnostico adenocarcinoma gstrico, el cual se encontraba en
condiciones de tratamiento quirrgico, cual es el rea mas frecuente donde encontrar estas lesiones en caso de observarse?

RESPUESTA
a.- Pies o manos.
b.- Torax.
c.- Cabeza.
d.- Abdomen.








MANUAL DE TRABAJO DEL CURSO ENARM CMN SIGLO XXI
CURSO ENARM CMN SIGLO XXI TEL: 36246001 Pharmed Solutions Institute PGINA 498

COLANGITIS, COLECISTITIS, COLEDOCOLITIASIS:
VESICULA BILIAR, LITOGENESIS: La vescula biliar es un rgano piriforme de aproximadamente 7 - 10 cms. de largo por 3 cms. de
ancho, con una capacidad habitual de 30-35cc, pudiendo contener volmenes muy superiores en condiciones patolgicas. Se le
reconocen tres partes principales: fondo, cuerpo y cuello. Desde ste emerge el conducto cstico, mediante el cual la vescula se une al
conducto heptico comn, para dar origen al conducto coldoco. La longitud de ste es de aproximadamente 7,5 cms., variable segn
el punto de desembocadura del conducto cstico. Alcanza un dimetro normal de 5,5 mm, el cual es mucho mayor en caso de patologa
obstructiva. Se reconocen 4 porciones bien definidas: 1. Porcin Supraduodenal: Desciende en el ligamento hepatoduodenal frente al
hiato de Winslow. Se sita por delante y a la derecha de la vena porta. La arteria heptica y su rama gastroduodenal se sitan a su
izquierda. 2. Porcin Retroduodenal: Se relaciona ntimamente con la primera porcin del duodeno, ubicndose lateralmente respecto
a la vena porta y frente a la cava. 3. Porcin Pancretica: Se extiende desde el borde inferior de la primera porcin del duodeno hasta
un punto en la pared posteromedial de la segunda porcin del duodeno. 4. Porcin intramural duodenal: Corre en sentido oblicuo hacia
abajo y lateralmente dentro de la pared del duodeno en una extensin de ms o menos 2 cms. El coldoco suele unirse al conducto
pancretico justo al interior de la pared duodenal en el 89% de los vasos. Formacin de clculos biliares: Existen dos tipos de clculos,
de acuerdo a su composicin bioqumica: clculos puros y clculos mixtos. Los clculos puros son de origen exclusivamente metablico,
y estn compuestos de colesterol o bilirrubina. Los clculos mixtos presentan en su etiopatogenia una causa inflamatoria, y estn
formados por una mezcla de colesterol, sales de calcio y pigmentos biliares, lo que se deposita sobre una base de naturaleza orgnica
formada por clulas epiteliales, material proteico y bacterias. El clculo de bilirrubinato de calcio (mixto) se forma a partir de un ncleo,
de preferencia en los conductos biliares. Corresponden al 82% de los clculos. El aumento de bilirrubina libre en la bilis (lo normal es
20%) determinar el fenmeno de litognesis primaria que corresponde a la formacin de bilirrubinato de calcio. Los clculos de
colesterol se producen por una alteracin del equilibrio bioqumico de los componentes de la bilis: sales biliares, fosfolpidos (lecitina),
colesterol y cidos biliares. Para que el colesterol no precipite, la bilis debe estar bajo la forma de micelas constituidas por un centro de
colesterol y una cubierta de sales biliares y fosfolpidos. En esta forma micelar el colesterol es hidrosoluble. Las alteraciones tanto en la
calidad como en cantidad de las sales biliares y la lecitina van a determinar la precipitacin y formacin posterior de clculos. A esta
bilis alterada en su composicin la denominaremos Bilis litognica (Bilis formadora de clculos). El aumento de la excrecin de
colesterol se ha asociado a una mayor incidencia de litiasis biliar. Por otro lado la ectasia biliar, la infeccin biliar y factores metablicos
tambin se relaciona con formacin de colelitiasis. Los estrgenos aumentan el ndice de saturacin de colesterol, favoreciendo
colelitiasis. Tambin habra factores genticos relacionados con litiasis biliar. Se ha observado mayor frecuencia de litiasis biliar en:
diabticos, obesos, embarazadas y mujeres que usan anticonceptivos de tipo andrognico.

COLANGITIS. CIENCIAS BASICAS: La colangitis aguda es la infeccin severa de la bilis dentro de los conductos biliares intra y extra
hepticos. Est producida por la existencia de bacterias en la bilis, en una situacin de obstruccin biliar, como consecuencia de la
obstruccin del flujo biliar, se produce un aumento en la presin intraductal que favorece el paso de grmenes a la circulacin portal y
linftica, produciendo episodios de bacteriemia con septicemia o sin ella. Su etiologa es debido a enfermedades concomitantes
(colangitis primarias: coledocolitiasis (70%) parasitosis, estenosis, quistes de coldoco, enfermedad de Caroli, colangitis esclerosante y
tumores) o por actuaciones directas o indirectas, diagnsticas o teraputicas, sobre el hgado o la va biliar (colangitis secundaria). La
colangitis ser supurada o no supurada si la bilis en la va biliar est contaminada o es purulenta; los grmenes ms frecuentes
involucrados n son E. Coli, Kleibsella, pseudomona estreptococo fecalis, enterobacter, bacteroides y clostridium. DIAGNSTICO: Clnico:
Se basa en la asociacin de signos y sntomas de infeccin con los propios de una obstruccin biliar. Dolor abdominal, en el cuadrante
superior derecho en el centro del abdomen superior, puede ser intermitente, agudo, tipo clico o sordo, puede irradiarse a la espalda
o debajo del omplato derecho, escalofros, fiebre generalmente alta, con escalofros e ictericia (triada de Charcot). Cuando se aade
confusin mental y shock se denomina Pentada de Reynolds, con menos frecuencia y que se asocia con una colangitis supurada grave.
Puede presentar heces color arcilla, orina oscura as como nuseas y vmitos. Laboratorio y gabinete hallazgos de obstruccin biliar con
aumento variable de las bilirrubinas ms de 2md/dl (80%) y enzimas de colestasis (FA, TGO, TGP). Habitual encontrar leucocitosis con
desviacin a la izquierda (80%) siendo por lo general las cifras ms altas en torno a 20.000 mm
3
, reflejo de las formas ms severas,
hemocultivos positivos durante los escalofros o picos febriles. La ecografa es la tcnica de eleccin para detectar la existencia de
obstruccin biliar por su elevada eficacia y versatilidad, pudindose detectar tambin complicaciones en la colangitis como absceso
heptico. La colangio-resonancia magntica con la sensibilidad de 90% y una especificidad 100%.La colangiopancreatografa retrgrada
endoscpica (CEPRE) y la colangiografa transheptica percutnea. TRATAMIENTO: Piperacilina 4grs i.v./8h. Tazobactam 4.5grs i.v./8h.
Ciprofloxacino 200mgsi.v./12h. Ceftriaxona 1gr i.v/24h. Metronidazol 500mgs i.v/8h. Analgsicos, antiinflamatorios y antipirticos.
Medidas higinicas y dietticas: El consumo de alimentos ser limitado durante su hospitalizacin. Se recomienda dieta baja en grasas.
Durante su internamiento depender de la situacin clnica y evolucin el inicio de la dieta, puede requerir de nutricin parenteral en
los casos graves por sepsis. Bajar de peso, desintoxicacin del hgado y coln. Evitar estrs, ejercicio regular ayuda a tonificar la
vescula. Mantener una dieta adecuada, realizar ejercicio, evitar alimentos condimentados y grasosos. Indicaciones de hospitalizacin:
Tener fiebre, ictericia, vmitos recurrentes o se presenta dolor despus del diagnstico o tratamiento, se incluye disnea, diaforesis y
nuseas. Deterioro progresivo y mal estado general. Criterios de evaluacin de severidad leve (Grado I), colangitis que responde al
tratamiento mdico inicia. Moderado (Grado II) colangitis aguda que no responde al tratamiento mdico inicial y no se acompaa de
disfuncin orgnica mltiple. Severa (Grado III) colangitis aguda que se asocia con la aparicin de disfuncin de al menos uno de los
siguientes rganos / sistemas: cardiovascular, hipotensin que requiere de aminas, alteraciones de la consciencia, relacin PaO2/Fio2
<300,Cr srica > 2 mg/dl, TP-RIN>1.5, Plaquetas < 100.000/ul.

CASO CLINICO
Paciente de 59 aos sin antecedentes personales de inters que acudi a urgencias por dolor epigstrico y febrcula. En la exploracin
se observo ictericia mucocutanea y ocupacin del hemiabdomen superior derecho. La analtica demostr: bilirrubina total, 5,3 mg/dl;
transaminasa glutmico pirvica (GPT), 311 U/l; transaminasa glutmico oxalactica (GOT), 166 U/l, y fosfatasa alcalina (FA), 176 U/l; el
resto fue normal. Se realiz una ecografa y una tomografa computarizada (TC) abdominal que confirmaron la existencia de 2 grandes
quistes hidatdicos en el lbulo heptico derecho (de 13 y 9 cm) y otro quiste hidatdico calcificado en el lbulo heptico izquierdo de 5
MANUAL DE TRABAJO DEL CURSO ENARM CMN SIGLO XXI
CURSO ENARM CMN SIGLO XXI TEL: 36246001 Pharmed Solutions Institute PGINA 499

cm. Las ramas biliares principales derecha e izquierda y el coldoco en toda su longitud estaban ocupadas por un material con mltiples
ecos lineales. La serologa para hidatidosis fue positiva (tuvo contacto con perros y ganado hace aos). Se inici tratamiento con
albendazol antes de la ciruga y se realiz una colangiopancreatografa retrograda endoscpica (CPRE), que confirm la dilatacin y la
ocupacin de la va biliar con salida espontnea de material hidatdico.

PREGUNTA
Cul es los siguientes patologas no es diagnostico diferencial.

RESPUESTA
a.- Hepatitis aguda.
b.- Nefrolitiasis.
c.- Obstruccin intestinal.
d.- Ulcera duodenal.

COLECISTITIS. CIENCIAS BASICAS: Es la inflamacin de la vescula biliar ocasionada principalmente por clculos (litos) y con menor
frecuencia por barro (lodo) biliar, en raras ocasiones ninguna de estas condiciones est presente. Colelitiasis: presencia de litos en la
vesicula biliar y con menor frecuencia de lodo biliar. SALUD PUBLICA: Es una de las principales causas de consulta en el servicio de
urgencias y en la consulta externa de ciruga general. La colecistectoma electiva es la intervencin quirrgica ms frecuente en los
centros hospitalarios. Se presenta en el 5-20% de los pacientes con colelitiasis. La colecistitis aguda se presenta con ms frecuencia en
mayores de 40 aos, ms frecuente en sexo femenino 2:1. PATOGENIA: Los factores que pueden condicionar el desarrollo de
Colecistitis y Colelitiasis: Edad: ms frecuente a partir de los 40 aos, cerca del 20% de los adultos a partir de esta edad y del 30% en los
mayores de 70 aos, sexo femenino, embarazo, sobre todo para el desarrollo de clculos de colesterol, normalmente son formas
asintomticas de litiasis biliar, anticonceptivos orales y terapia hormonal sustitutiva con estrgenos, en este caso en mujeres menores
de 40 aos y las que reciben una dosis mayor de 50 microgramos de estrgenos. Otros frmacos como los fibratos y la ceftriaxona,
obesidad, antecedentes familiares de litiasis biliar, prdida rpida de peso, nutricin parenteral, DM, cirrosis heptica, enfermedades
del leon, enfermedad de Crohn, enfermedades hepticas y metablicas. CLASIFICACION: La colecistitis aguda se clasifica en tres
grados, de acuerdo a la afectacin de otros rganos, medidos por pruebas de laboratorio: Grado I - leve, Grado II - moderada, Grado III
grave. DIAGNOSTICO: Clnico: de colecistitis y colelitiasis son: Murphy positivo, dolor y resistencia en cuadrante superior derecho,
nausea, vomito. Manifestaciones agudas; vescula palpable, fiebre >39, escalofros., inestabilidad hemodinmica. La perforacin con
peritonitis generalizada se sospecha cuando; existen signos de irritacin peritoneal difusa, distensin abdominal, taquicardia,
taquipnea, acidosis metablica, hipotensin, choque. No hay pruebas de laboratorio especficas para colecistitis aguda, pero pueden
ser de utilidad: biometra hemtica: se puede encontrar leucocitosis. Protena C Reactiva la cual puede encontrarse elevada y es de
utilidad para confirmar el proceso inflamatorio. Ante la sospecha de colecistitis o colelitiasis aguda el ultrasonido es la prueba no
invasiva de primera eleccin 98% de sensibilidad, los hallazgos que reporta son: engrosamiento de la pared vesicular mayor de 5 mm.
Liquido perivesicular, signo de Murphy ultrasonogrfico positivo, alargamiento vesicular 8 cm. Axial y 4 cm. Diametral, lito encarcelado,
imagen de doble riel, sombra acstica, ecos intramurales. Para identificar la gravedad de la Colecistitis y Colelitiasis se solicitar:
Bilirrubinas, BUN, creatinina, tiempo de protrombina, la amilasa srica es de utilidad para identificar complicaciones como
coledocolitiasis. TRATAMIENTO: para la disolucin de los clculos biliares en pacientes en situaciones especilaes (pacientes con alto
riesgo quirrgico y aqueloos que se rehsan a la ciruga), el tratamiento de eleccin es con acidos biliares como: Acido ursodesoxicolico,
cido quenodeoxicolico, por un periodo de 1-2 aos. Depende del nivel de gravedad, se prefiere una colecistectoma temprana y de
seleccin adecuada de la tcnica con la que se realizar la ciruga. La litiasis vesicular asintomtica no es indicacin de colecistectoma
excepto que la pared vesicular este engrosada (4mm o ms) o calcificada o que los clculos sean mayores a 3 cm de dimetro. La
colecistectoma puede indicarse ocasionalmente en pacientes con dolor vesicular sin litiasis (colecistitis alitisica) o porque la vescula
no funcione adecuadamente (discinesia biliar). En estos casos el diagnostico se realiza mediante una colecistografa oral o un
gamagrama de vescula. La colecistectoma laparoscpica temprana es el tratamiento de eleccin en la mayora de los pacientes,
porque: la recuperacin es rpida, requiere menor estancia hospitalaria, reduce costos de tratamiento, reincorporacin rpida al
trabajo. Se considera temprana cuando se realiza de 1 a 3 das despus del ataque inicial y tardo si se realiza de 6 A 15 das despus del
ataque. Factores que dificultan la colecistectoma por laparoscopia son: ciruga abdominal previa, presencia o antecedentes de ictericia.
La combinacin de extraccin de litos por endoscopia durante una colangio-pancreatografa retrograda endoscpica (CPRE) y la
colecistectoma por laparoscopia, es de utilidad en el tratamiento de pacientes con colecisto y coledocolitiasis. El intervalo entre estos
dos procedimientos es de pocos das (ambos procedimientos se realizarn durante la misma estancia hospitalaria). En la
Colecistectomia abierta no complicada los das de estancia hospitalaria posquirrgica son de 2 a 3 das. Y el tiempo promedio de
recuperacin es de 21 das. En la colecistectomia laparoscpica no complicada dos das de estancia hospitalaria es suficiente. MANEJO
PERIOPERATORIO: Correccin control de los factores de riesgo, estudios de laboratorio e imagen solicitados en forma particular a
cada caso, en mayores de 45 aos electrocardiograma, telerradiografa de trax y valoracin cardiovascular preoperatoria, el uso
profilctico de antibiticos se selecciona en base al tipo de herida quirrgica. En todos los pacientes se debe llevar a cabo prevencin
tromboemblica. La administracin de AINEs se recomienda a pacientes con clico biliar, sin embargo no es til para la remisin de la
enfermedad. Es recomendable el uso de diclofenaco a razn de 75 mgr. IM para el manejo de los clicos biliares. El manejo del dolor es
pacientes con colecistitis aguda incluye narcticos como meperidina, en presencia de dolor intenso. El tratamiento de antibitico se
indicar dependiendo de la gravedad de la colecistitis: para colecistitis leve- un antibitico, para colecistitis moderada y grave se
recomienda el uso de dos antibiticos, cuando se sospecha o se detecta la presencia de anaerobios agregar metronidazol. Litotripsia
extracorprea: Indicada en pacientes con litiasis nica, no calcificada, con dimetros de 20-30mm. Contraindicada en pancreatitis,
alteraciones de la coagulacin, quistes o aneurismas en el trayecto de las ondas de choque. El cido ursodeoxicolico se indica como
coadyuvante cuando se realiza litotripsia, ya que de esta forma se logra mayor efectividad en la destruccin de litos. No aprobado por
FDA como tratamiento definitivo para litiasis vesicular. COMPLICACIONES: de la colecistectoma laparoscpica generalmente son
debidas a instrumental inapropiado o caduco, y tambin por la falta de pericia del cirujano como son: lesin del conducto biliar, lesin
MANUAL DE TRABAJO DEL CURSO ENARM CMN SIGLO XXI
CURSO ENARM CMN SIGLO XXI TEL: 36246001 Pharmed Solutions Institute PGINA 500

del intestino, lesin heptica. En la ciruga abierta y laparoscpica adems puede haber como complicacin: las infecciones, leo,
hemorragia intraperitoneal, atelectasia, trombosis de venas profundas, infeccin de vas urinarias.

CASO CLINICO
Varn de 32 aos, que acude a Urgencias por dolor en hipocondrio derecho de 72h de evolucin tipo clico y vmitos tras la ingesta
que no calman con analgesia convencional y antiemticos. Siendo dado de alta a las 48h. Sin antecedentes mdicos. No Sd
constitucional. Ictericia cutnea y mucosa evidente sin prurito asociado. No fiebre. Hemodinmicamente estable. T. 36,8C. Abdomen
blando, depresible, doloroso en hipocondrio derecho, sin masas ni megalias. No peritonismo. Ictericia mucocutnea. La analtica
muestra 10,9 10E3/l leucocitos y discreta eosinofilia (0,7 10E3/l). ilirrubina total de 7,9mg/dl, con bilirrubina conjugada de 5,2mg/dl
y aumento de fosfatasa alcalina de 853 y ALT de 268. Lipasa de 346.

PREGUNTA
Cual es la conducta a seguir mas adecuada?

RESPUESTA
a.- Conducta conservadora.
b.- Preparar para colecistectoma.
c.- Realizar USG abdominal.
d.- Realizar TAC abdominal.

CASO CLINICO
Mujer de 17 aos que acude a Urgencias por un cuadro de abdomen agudo. Como antecedentes, destacaba un ingreso un ao antes
para estudio por un sndrome febril y cefalea. Fue diagnosticada de una infeccin por citomegalovirus y por virus Coxsackie B6, desde
entonces permaneciendo asintomtica y sin tratamiento. La paciente consult por dolor abdominal de 6 horas de evolucin, iniciado en
epigastrio y posterior migracin a fosa ilaca derecha, intenso, de caractersticas continuas, con vmitos, sin fiebre ni otros sntomas. A
la exploracin presentaba febrcula, defensa y signos de irritacin peritoneal en hemiabdomen derecho. Analtica: leucocitos 16.330/l,
neutrfilos 76,7%, hematocrito 39%, ALT 59 UI/l, GGT 19 UI/l, amilasa 354 UI/l, PCR 0,7mg/dl

PREGUNTA
Cual es la conducta inmediata mas adecuada?

RESPUESTA
a.- Realizar USG plvico.
b.- Realizar prueba de embarazo.
c.- Realizar RX de abdomen.
d.- Mantener medidas generales.

PREGUNTA
Posterior a la medida realizada se realizar RX de abdomen siendo negativa, y USG con liquido libre en cavidad, cual es la conducta a
seguir?

RESPUESTA
a.- Mantener conducta expectante.
b.- Realizar TAC.
c.- Preparar para LAPE.
d.- Lavado peritoneal

PREGUNTA
Cual es su impresin diagnostica mas probable en este momento?

RESPUESTA
a.- Colecistitis.
b.- Apendicitis.
c.- Embarazo ectpico.
d.- Ulcera perforada.

CASO CLINICO
Se trata de femenino de 85 aos, ingresa por dolor epigstrico de 12 horas de evolucin de inicio brusco y continuo, antecedentes de
hipertensin arterial y bronquitis crnica sin tratamiento con corticoides, EF no signos de sepsis, abdomen blando depresivo, doloroso
en la regin superior, no datos de irritacin peritoneal, Hto 41, leucos 11,900 glucosa 128, urea 23, CK 78, DHL 385, resto normal. USG
abdominal mostro microlitiasis con vesicula biliar sin dilatacin.

PREGUNTA
Cual es la conducta a seguir mas adecuada?

RESPUESTA
MANUAL DE TRABAJO DEL CURSO ENARM CMN SIGLO XXI
CURSO ENARM CMN SIGLO XXI TEL: 36246001 Pharmed Solutions Institute PGINA 501

a.- Realizar endoscopia.
b.- Conducta expectante.
c.- Liquidos, analgsico y antibiticos.
d.- Colecistectomia laparoscpica.

COLEDOCOLITIASIS. CIENCIAS BASICAS: El factor comn presente en la gran mayora de las enfermedades de la va biliar es la
Colelitiasis. La complicacin ms frecuente, la Coledocolitiasis, se ha reportado con cifras hasta de un 20%. Se define coledocolitiasis
como la presencia de clculos biliares en el conducto Coldoco y/o en conducto heptico comn. SALUD PUBLICA: Segn datos
internacionales los clculos biliares se encuentran en el 12% de los hombres y el 24% de las mujeres. La prevalencia aumenta con la
edad. Ms de un 10% de pacientes portadores de colelitiasis presentan coledocolitiasis. La asociacin entre colecistitis crnica litisica y
coledocolitiasis es de aprox un 15%. La asociacin entre colecistitis aguda y coledocolitiasis puede alcanzar hasta un 25%. PATOGENIA:
La mayora de los clculos coledocianos se originan de la vescula biliar, de hecho su forma y composicin son similares a la de los
clculos vesiculares creciendo en el coldoco por aposicin de colesterol; simultneamente se produce una dilatacin gradual de la va
biliar que con los aos puede llegar a un dimetro de 2 cm o ms. Con menor frecuencia los clculos coledocianos se originan en la
misma va biliar, ello se observa en casos de estenosis del heptico comn o el coldoco en los que se desarrollan clculos mixtos o de
bilirrubinato de calcio. Este fenmeno desaparece si se corrige la estrechez (dilatacin endoscpica) o se deriva la va biliar dilatada al
duodeno o yeyuno. En un paciente colecistectomizado puede detectarse coledocolitiasis en el postoperatorio alejado, lo cual se debe a
que durante la operacin no se sospech ni se diagnostic una coledocolitiasis concomitante. En este caso hablamos de Coledocolitiasis
residual cerrada. Cuando se detecta la coledocolitiasis en el postoperatorio de un paciente sometido a una colecistectoma y adems
coledocostoma y por lo tanto, portando una sonda T situada en el coldoco, hablamos de Coledocolitiasis residual abierta. Cuando han
pasado varios aos despus de realizada la colecistectoma en un paciente y se detecta coledocolitiasis, hablamos de coledocolitiasis
cerrada de Neoformacin. DIAGNOSTICO: Puede producir obstruccin del coldoco con impedimento parcial o total del paso normal de
bilis desde hgado al duodeno, dando lugar al denominado Sndrome de Ictericia Obstructiva. La mayora de estos pacientes tienen
antecedentes biliares, ya sea de clicos biliares en reaccin a ingestin de alimentos grasos, colelitiasis asociada o demostrada
antecedentes de colecistectoma. Es caracterstico, la presencia de dolor abdominal de tipo clico ubicado en epigastrio o hipocondrio
derecho, que precede a la aparicin de coluria e ictericia de piel y escleras. Posteriormente, puede aparecer hipocolia. Se asocia a
estado nauseoso y vmitos rebeldes. Puede prolongarse durante varias horas, y si se alivia con antiespasmdicos tiende a desaparecer
precozmente. La ictericia es fluctante, debido al mecanismo valvular que determina la obstruccin de la va biliar por l o los clculos
flotantes que se desplazan y eventualmente se impactan y se desimpactan sucesivamente dentro de ella. Se le puede confundir con un
clico ureteral derecho; es til tener presente en este caso que el paciente se encuentra muy inquieto, mientras que si el dolor es de
origen biliar, tiende a permanecer postrado en la cama. La coluria precede a la ictericia y el paciente ictrico orienta a etiologa
obstructiva extraheptica, ya que esta se produce debido a que se elimina bilirrubina directa o conjugada a travs de la orina. El
enclavamiento de un clculo en la Ampolla de Vater da origen a una ictericia prolongada, que en los enfermos ancianos puede
complicarse con una enfermedad tubular aguda. A estos sntomas y signos se le agrega prurito, por depsitos de sales biliares en la piel.
En los pacientes seniles, la coledocolitiasis suele ser causa de anorexia y prdida de peso. Conviene insistir en que muchos enfermos
con clculos en el coldoco no tienen ningn sntoma que revele su presencia. Por ello es tan importante, durante la colecistectoma
electiva o de urgencia, la cuidadosa exploracin radiolgica de la va biliar. Los mtodos de diagnstico por imagen de la coledocolitiasis
pueden ser preoperatorios e intraoperatorios. Dentro de los preoperatorios se encuentran: Ecografa (eco), Colangioresonancia (C-
RMN), Tomografa computarizada (TC), Colangiopancreatografa retrgrada endoscpica (CPRE), Ecoendoscopa (ECO-E), Colangiografa
endovenosa (CIV) y Colangiografa transparietoheptica (CTPH). Los intraoperatorios: Colangiografa intraoperatoria (CIO) y ecografa
por laparoscopa (ECO-L). TRATAMIENTO: Si el paciente presenta coledocolitiasis asociada a colelitiasis y la presencia de clculos
coledocianos ha sido establecida previamente a la intervencin quirrgica, el procedimiento ms aceptado hoy da es intentar la
extraccin de los clculos va endoscpica (CPER asociada a Papilotoma y extraccin endoscpica de clculos), para luego proceder con
una colecistectoma laparoscpica. Si el diagnstico de coledocolitiasis ha sido establecido durante la realizacin de una colecistectoma
(mediante colangiografa intraoperatoria), se puede programar una extraccin diferida de clculos coledocianos por va endoscpica. En
algunos centros se realiza ambos procedimientos en un solo tiempo en forma intraoperatoria. Si no se cuenta con CPER o el paciente
tiene contraindicacin para ciruga laparoscpica, se puede resolver ambas situaciones con una ciruga clsica, realizando una
coledocostoma intraoperatoria para extraer los clculos. Esta alternativa requiere dejar en la va biliar un drenaje (sonda T) para evitar
una complicacin post operatoria (biliperitoneo).

CASO CLINICO
Mujer de 56 aos, sin antecedentes de inters, que ingresa por cuadro febril que se acompaa de dolor en hipocondrio derecho y
vmitos repetidos. Se diagnostica de colecistitis aguda y por ecografa se comprueba colelitiasis. Se trata con pauta antibitica y la
paciente mejora, quedando afebril y asintomtica. Una nueva ecografa de control demuestra la persistencia de la colelitiasis, por lo
que se decide tratamiento quirrgico, realizndose colecistectoma. La intervencin quirrgica transcurre sin problemas y el
postoperatorio inmediato es correcto. La evolucin en planta transcurre sin problemas, pero a la maana del 5 da del postoperatorio
la paciente presente de manera sbita un cuadro clnico de disnea, taquipnea y taquicardia, apareciendo pocas horas despus cianosis.
EF: Consciente y orientada, colaboradora, pero inquieta y con importante dificultad respiratoria. Cianosis ligera de piel y mucosas.
Auscultacin pulmonar y cardiaca sin hallazgos patolgicos. Abdomen blando y depresible, con apsitos de laparotoma limpios y
drenajes ya retirados. Extremidades sin edemas.

PREGUNTA
Cual es la complicacin ms probable en este caso?

RESPUESTA
a.- Nemonia.
MANUAL DE TRABAJO DEL CURSO ENARM CMN SIGLO XXI
CURSO ENARM CMN SIGLO XXI TEL: 36246001 Pharmed Solutions Institute PGINA 502

b.- Edema agudo pulmonar
c.- TEP.
d.- SDRA.

CANCER DE PANCREAS. CIENCIAS BASICAS: El carcinoma pancretico es el tumor periampular ms comn; las manifestaciones son
similares para todos. La gran mayora de los tumores malignos son carcinomas ductales, adenocarcinomas. Es un tumor de gran
agresividad biolgica. SALUD PUBLICA: Ocupa cuarto lugar como causa de muerte en Estados Unidos. Edad promedio de inicio es de 60
aos, afecta ms sexo masculino (2:1). Menos de 20% de los carcinomas es resecable al momento del diagnstico. Tras ciruga
curativa: <20% sobreviven al ao y <5% a los 5 aos. Segundo tumor digestivo ms
frecuente. El T. digestivo con peor pronstico, es el ms letal y el fallecimiento se produce
en 95% de los casos. Supervivencia media del no tratado: 4-6 meses. ADENOCARCINOMA:
Surge de los conductos en 90% de los casos y de los acinos en 10%. A menudo la mayor
parte del tumor es estroma fibroso con una zona de pancreatitis. Cerca del 75 % se origina
en la cabeza o el proceso uncinado del pncreas y produce obstruccin biliar que hace
posible el diagnstico ms temprano, 15% en el cuerpo y 10% en la cola del pncreas. El
tumor puede invadir la vena porta o los rganos adyacentes o producir metstasis al hgado
o al peritoneo. Los ganglios linfticos son positivos en 90% de los pacientes. A veces los
carcinomas ampular y duodenal se diagnostican como pancreticos; que es posibles que
sean pequeos al momento de detectarlos. DIAGNOSTICO: Clnica; cerca de 75% con
carcinoma de la cabeza pancretica acude por ictrica obstructiva (82%), prdida de peso
(92%) y dolor abdominal (72%). El dolor es sordo, como adolorimiento, medioepigstrico
y a menudo se irradia a espalda, de intensidad severa y predominio nocturno, empeora con
las comidas y con el decbito supino. El dolor en espalda sugiere invasin retroperitoneal. Son frecuentes la anorexia, la fatiga y el
prurito. En la exploracin se encuentra ictericia, hgado palpable (50-70%) y vescula palpable (30%, si no es sensible y hay ictericia, el
diagnstico es de cncer pancretico: Signos de Courvoisier). Puede aparecer tromboflebitis migratoria recurrente (signo de
Trousseau). Esplenomegalia e hipertensin porta. El 20% de los pacientes padece diabetes de inicio reciente. En caso de carcinoma
ampular el dolor es menos frecuente, a menudo clico, y la ictericia es intermitente. Los tumores de cuerpo y cola pancreticos
producen sntomas en una fase ms tarda de la enfermedad, por lo que la etapa es ms avanzada al momento del diagnstico. Las
metstasis al ganglio supraclavicular izquierdo (ganglio de Virchow), umbilical (ganglio de la hermana Mara Jos) y del piso plvico
(repisa de Blumer), indican enfermedad incurable. Metstasis: Frecuentes en hgado, seguido de los ganglios linfticos regionales,
peritoneo y pulmones. En las pruebas de laboratorio hay aumento en el nivel de bilirrubinas, en el de fosfatasa alcalina y solo un
aumento ligero en transaminasas. El CA19-9, con marcador tumor srico con sensibilidad de 80% y especificidad de 90%, a veces ayuda.
Muchos pacientes se valoran primero con ultrasonido abdominal, pero la TAC espiral es ms precisa. En algunos casos resulta til la
CEPRE, sobre todo cuando no se encuentra masa alguna en la TAC. La aspiracin con aguja fina est indicada cuando hay evidencia
topogrfica de irresecabilidad de manera que el diagnostico tisular puede comprobarse sin una operacin (la aspiracin negativa, no
descrata la enfermedad). TRATAMIENTO: Se har en funcin de poder o no extirparlos quirrgicamente: Resecable: 10-20%. Avanzado
localmente (no resecable): 40%. Metastsico: 40%. Es necesario corregir pronto el estado nutricional, la anemia y estado de volumen,
adems de valorar la funcin renal. La pancreatoduodenectoma (operacin de Wipple) representa la nica esperanza de curacin para
los tumores de la cabeza pancretica y es ms til en caso de carcinoma localizado, ya sea ampular, duodenal o del coldoco distal. Los
pacientes con tumores irresecables obtienen cierto beneficio de la colecistoduodenostoma o coledocoduodenostoma y
gastroyeyunostomia paliativas. La inyeccin transoperatoria en el plexo celiaco alivia el dolor. El paciente con alto riesgo obtienen
beneficios con la CEPRE o el drenaje transheptico. La radioterapia combinada con 5-fluoracilo tiene cierto valor como coadyuvante y
paliativo. La Gemcitabina es el medicamento de quimioterapia ms comnmente utilizado para tratar el cncer de pncreas.
Seguimiento de los pacientes: Cada 3 meses los primeros 2 aos. Cada 4 meses en el 3er ao. Cada 6 meses en el 4to y 5to aos.
NEOPLASIAS QUSTICAS: El 5% de todos los tumores pancreticos, estos tumores se originan en las clulas de los conductos, tienen
revestimiento epitelial. Las neoplasias serosas (microquisticas), son benignas como el CISTADENOMA SEROSO compuesto por mltiples
quistes pequeos de hasta 2 cm de dimetro con lquido claro, ms en mujeres, alrededor de los 65 aos, en general asintomtico
localizacin ms frecuente en el cuerpo y cola del pncreas. EL CISTADENOMA MUCINOSO es premaligno, lesiones multiloculares y
papilares >5cm, ms frecuente en mujeres y puede evolucionar a CISTADENOCARCINOMA 1% de los cnceres de pncreas, son
tumores grandes de 20- 30cm, deben extirparse, ya que presentan buena supervivencia a los 5 aos, son tumores de crecimiento lento,
tienen mejor pronstico y deben tratarse en forma agresiva. A menudo se manifiestan por molestia vaga, menos de 10% presentan
ictericia con anorexia y prdida de peso. Todas las lesiones macroquisticas deben extirparse. NEOPLASIA INTRADUCTAL PAPILAR
MUCINOSA: Proliferacin de clulas mucinosas en forma de papilas a nivel ductal con hiperproduccin de moco. Ms en hombres, 68
aos. Sntomas sugestivos de pancreatitis crnica. En la cabeza y proceso uncinado, aunque puede afectar difusamente la glndula
pancretica. 30-60% son malignas en el momento de su diagnstico. Asociada a otros tumores malignos en un 23-36% de los casos.
NEOPLASIAS ENDOCRINAS: Se originan en las clulas de los islotes pancreticos, representan aproximadamente el 15% de las
neoplasias pancreticas, casi todas son malignas. Segregan hormonas, dependiendo de las clulas de las que proceden son muy raros.
Ms frecuente el gastrinoma, que adems representa el 10% de todos los tumores endocrinos gastropancreticos, el segundo en
frecuencia es el insulinoma. INSULINOMA: Tumor pancretico endocrino funcionante ms frecuente 50-60%, proviene de clulas
pancreticas, la mayora en la cabeza del pncreas. 80% benignos. Habitualmente se presenta en la 5ta y 6ta dcada de la vida, ms en
mujeres que en hombres (2:1). La hiperinsulinemia produce hipoglucemia grave y origina convulsiones, depresin y coma. Los sntomas
desaparecen poco despus de administrar glucosa. Los criterios diagnostico clsicos, la triada de Wipple, incluyen hipoglucemia en
ayuno (<50mg/dl) durante los ataques, cambios en el SNC y sntomas de hipoglucemia desencadenado por el ayuno, as como reversin
de los cambios con administracin de glucosa. La ubicacin del tumor se facilita con la angiografa, toma de muestras venosas
selectivas, imgenes por TAC o centellografa con octretido. El tratamiento es quirrgico (Pancreatectoma distal, dejando 20-30% del
pncreas), excepto en enfermedad metastsica avanzada. GASTRINOMA: La triada original de Zollinger-Ellison incluye ulceras ppticas
CLASIFICACION CANCER DE PANCREAS
Tumores slidos de pncreas exocrino:
adenocarcinoma (90%), carcinoma acinar y
pancreatoblastoma
Tumores qusticos del pncreas exocrino:
neoplasia mucinosa qustica, neoplasia mucinosa
papilar intraductal (TMPI*), neoplasia serosa
qustica, neoplasia slida seudopapilar.
Tumores del pncreas endcrino (de los islotes):
insulinomas, gastrinomas, glucagonomas,
somatotatinomas, VIPomas, PPomas, carcinoides
Mesenquimticos y hematopoyticos:
liposarcoma, leiomiosarcoma, fibrosarcoma,
linfoma, Swwanoma
Secundarios: metstasis de carcinoma de mama,
pulmn, renal y melanoma.
MANUAL DE TRABAJO DEL CURSO ENARM CMN SIGLO XXI
CURSO ENARM CMN SIGLO XXI TEL: 36246001 Pharmed Solutions Institute PGINA 503

fulminantes de localizacin atpica, hipersecrecin gstrica extrema y un tumor de clulas no beta de islote pancretico. El gastrinoma
puede comenzar como una enfermedad sencilla y terminar con complicaciones graves como perforacin, obstruccin, hemorragia e
intratabilidad. La enfermedad no responde a tx., medico, ni quirrgico. Es posible que los pacientes presentes diarrea de gasto alto con
esteatorrea. Dx., hay hipergastrinemia en ayuno (>200pg/ml). Su potencial maligno es >90%. El crecimiento es lento y la metstasis se
produce en etapa tarda. Con frecuencia las lesiones se encuentran en el tringulo del gastrinoma: 1) unin de los conductos cstico y
coldoco, 2) unin de la segunda y tercera porcin del duodeno, 3) unin del cuello y el cuerpo pancreticos. VIPomas: Tumor
productor de pptido intestinal vasoactivo, procedentes de las clulas D1. casiona diarrea, clera pancretico y un sndrome
compuesto por diarrea acuosa, hipopotasemia y aclorhidria gstrica. Cerca de 50% corresponde a malignos, ms comn en hombres,
promedio 45 aos, de gran tamao. Provoca el Sx de Werner-Morrison o clera pancretica: Diarrea hipersecretora + rubicundez (20%)
+ hipokalemia + aclorhidria. 50% de los pacientes con hiperglucemia (glucogenlisis heptica inducida por el VIP y la hipopotasemia).
Diagnstico: Concentraciones sricas de VIP >200 pg/ml. TAC y USG. Tratamiento: Enucleaciones, pancreatectomas distales
laparoscpicas. GLUCAGONOMAS: Procedentes de clulas A, ms frecuentes en mujeres, entre 50-70 aos de edad, ms en cuerpo y
cola. Causan lesiones cutneas (eritema necroltico migratorio, muy pruriginoso), diabetes, glositis, prdida de peso depresin y
trombosis venosa. Las tres cuartas partes son malignos. El mejor tratamiento es la reseccin. Sx de las 4 Ds (Diabetes, Depresin,
Dermatitis, Deep vein trombosis). Diagnstico: Glucagn srico>500pg/ml. Tambin puede detectarse hipoalbuminemia,
hipoaminoacidemia, anemia e hipocolesterolemia. TAC. El mejor tratamiento es la reseccin, Pancreatectoma distal con preservacin
esplnica o pancreatoduodenectoma. SOMATOSTINOMAS: Extremadamente raros, en pncreas proximal, 60% mpula (en clulas D).
Se manifiestan por diabetes, diarrea, esteatorrea, aclorhidria, calculos biliares, malabsorcin, y dolor abdominal. Por lo general se
tratan con estreptozocina, dacarbacina y doxirrubicin. Diagnstico: concentraciones sricas de somatostatina >100 pg/dl. TAC, RNM.
Tratamiento reseccin del tumor. PPoma: Es productor de polipptido pancretico, es el tumor no funcionante ms frecuente. No se
asocian con sndromes clnicos de hiperfuncin hormonal. Entre la 5ta-6ta dcada de la vida. En el 50% se ubican en la cabeza, proceso
uncinado y cuello del pncreas.

CANCER DE PANCREAS.
Se trata de paciente masculino de 59 aos de edad con antecedentes de DM 2 y tabaquismo positivo, el cual refiere diarrea crnica de
ms de 6 meses de evolucin, disminucin de peso del 15 %, ha disminuido su ingesta de alimento ya que tiene sensacin de plenitud y
vomito en varias ocasiones, se observa ictericia y piel verdosa, refiere heces grasosas y flatulencias, fue diagnosticado previamente con
sndrome de Peutz - Jeghers.

PREGUNTA
Cul es la manifestacin clnica ms importante para distinguir cncer pncreas de cabeza vs cuerpo y cola.

RESPUESTA
a.- Dolor de espalda.
b.- Coluria y acolia.
c.- Constipacin.
d.- Dispepsia.

CASO CLINICO
Acude paciente de 61 aos de edad el cual presenta prdida de peso y sensacin de plenitud abdominal, cuenta con antecedentes de
consumo de alcohol y tabaco de forma social sin llegar a la embriaguez, aproximadamente cada semana, cuenta con antecedentes de
cncer en la familia sin especificacin, adems refiere que hace 20 aos sufre accidente automovilstico con transfusin sangunea,
hace 5 aos fue diagnosticado con hepatitis C, actualmente presenta distencin abdominal, sobrecarga de volumen y ascitis,
confirmando cirrosis desde entonces, se encuentra con tratamiento con espironolactona mas furosemida, se realiza radiografia
abdominal donde se aprecia lesin en lbulo derecho de 5 cm aproximadamente, se indica TAC donde se confirma lesin de 4,6 cm en
lbulo derecho cercana a pediculos portales principales, se solicita fetoproteina alfa con 384 ng/ml de resultado, se programa biopsia
percutnea.

PREGUNTA
Considerando los antecedentes y el cuadro clnico actual, cual es el diagnostico ms probable.

RESPUESTA
a.- Adenocarcinoma heptico.
b.- Carcinoma hepatocelular.
c.- Carcinoma metastasico.
d.- Colangiocarcinoma.

COLANGIOCARCINOMA. CIENCIAS BASICAS: Es un tumor maligno originado en el epitelio de los conductos biliares intra o
extrahepticos, se puede ubicar a cualquier nivel de la via biliar, desde los canalculos hasta la ampolla de Vater. Cerca de dos terceras
partes de las lesiones, se localizan en los conductos proximales, a menudo en la confluencia de los conducto heptico derecho e
izquierdo) La mayora de los colangiocarcinomas son del tipo histolgico adenocarcinoma ductal, sin embargo, suelen verse otros como
los adenocarcinomas papilar, mucinoso, mucoepidermoide y el cistoadenocarcinoma. SALUD PUBLICA: Es poco frecuente, representa
menos del 2% de todos los tumores malignos de diagnstico reciente. La incidencia en Estados Unidos es baja, de 1.0/100.000 por ao y
es un poco ms alta en Israel y Japn. Se presenta especialmente despus de la sexta dcada de la vida y es ligeramente ms frecuente
en hombres que en mujeres. CLASIFICACION: Anatmicamente distinguimos el colangiocarcinoma intraheptico (20% a 25%), el
perihiliar (50% a 60%), el extraheptico distal (20% a 25%) y el multifocal (5%). El extraheptico tambin se clasifica en el del tercio
MANUAL DE TRABAJO DEL CURSO ENARM CMN SIGLO XXI
CURSO ENARM CMN SIGLO XXI TEL: 36246001 Pharmed Solutions Institute PGINA 504

superior de la va biliar (heptico comn, confluencia hiliar o hepticos derecho e izquierdo), el del tercio medio (coldoco hasta un
plano dado por el borde superior del duodeno) y el del tercio inferior (desde el plano del borde superior del duodeno hasta la ampolla
de Vater). Se denomina TUMOR DE KLATSKIN al colangiocarcinoma situado en la confluencia hiliar. Segn su patrn de crecimiento,
puede ser exofitico, polipoideo e infiltrativo. El tumor de Klatskin es de tipo infiltrativo con invasin maligna periductal. El polipoideo
tiene crecimiento endoluminal. PATOGENIA: La etiologa no est bien definida, hay algunos procesos patolgicos predisponentes, entre
ellos la colangitis esclerosante primaria, la litiasis intraheptica, la enfermedad de Caroli y malformaciones como la atresia de va biliar y
los quistes de coldoco. Tambin existe asociacin con txicos cancergenos de origen industrial como digoxinas, nitrosaminas y
asbesto . Adems, se le asocia a infestacin crnica de la va biliar con parsitos endmicos del sudeste asitico como clonorchs
sinensis y opisthorchis viverrin. La infeccin crnica con salmonella typhi sera otro factor predisponente. Es dudoso que la colelitiasis y
la colecistectoma previa predispongan al desarrollo de colangiocarcinoma. Las alteraciones moleculares tambin se han asociado al
colangiocarcinoma, como la inactivacin de oncosupresores (p53, pl6, bel-2) y la mutacin de oncogenes (K-ras, c-myc, c.erbB-2). Sin
embargo, no son hallazgos especficos. DIAGNOSTICO: Clnica; dolor en hipocondrio derecho, ictericia y baja de peso. El signo ms
importante es la ictericia progresiva, presente en alrededor de 90%, muchas veces precedida de prurito. Tambin hay dolor no muy
intenso en epigastrio e hipocondrio derecho. El signo de Courvosier-Terrier puede estar presente en los colangiocarcinomas ubicados
en el tercio inferior del coldoco. En algunos pacientes asintomticos, las fosfatasas alcalinas estn aumentadas de 1 a 5 veces y las
transaminasas de 1 a 2 veces. Los marcadores tumorales CA 19-9, CA-125 y CEA estn aumentados en el colangiocarcinoma en 85%,
40% a 50% y 30%, respectivamente. No obstante, estos marcadores tumorales tambin se elevan en otras patologas malignas y en
lesiones hepticas graves. Un estudio comparativo entre CA 19-9 y CEA realizado por autores chinos, demuestra que el CA 19-9 es ms
til para el diagnstico del colangiocarcinoma. Actualmente el diagnstico se ha facilitado por la disponibilidad de variados
procedimientos imagenolgicos y endoscpicos. La colangiografa, localiza el sitio exacto de la obstruccin. El hgado y los ganglios
linfticos son los sitios ms comunes de metstasis. TRATAMIENTO: Lo ideal es la reseccin quirrgica completa del tumor con
mrgenes histolgicos negativos es la nica terapia curativa. Desafortunadamente, la mayora de las veces el tumor es irresecable. Para
esos casos hay posibilidad de paliacin mediante la colocacin de endoprtesis por va endoscpica o percutnea. Los pacientes con
colangiocarcinoma hiliar en un bajo porcentaje alcanzan a ser tratados con ciruga potencialmente curativa, fundamentalmente por tres
motivos: por diagnstico tardo, por infiltracin neoplsica de la vasculatura del hilio heptico, terapia de eleccin la reseccin de la va
biliar con o sin reseccin heptica ms linfadenectoma y anastomosis biliodigestiva. La mortalidad operatoria de las diferentes series
de ciruga resectiva es de alrededor de 10%. El trasplante heptico, ltimamente ha surgido como otra opcin vlida para tratar
pacientes con colangiocarcinoma hiliar y perifrico considerados irresecables pero no diseminados. La curacin de los tumores de la
parte distal del coledoco mejoro con la linfadenectomia radical y la pancreatoduadenectoma (Procedimiento de Wipple).

CASO CLINICO
Se trata de paciente femenino de 56 aos de edad la cual acude a consulta por malestar generalizado, fatiga y coloracin amarillenta, la
paciente es originaria del distrito federal, fumadora durante 15 aos as como consumo de alcohol tipo social cada fin de semana,
cuenta con antecedente de intolerancia a la glucosa con IMC de 27 la cual se encuentra con tratamiento con dieta, ejercicio y
metformida con resultados leves, el padecimiento actual inicia hace una semana caracterizado por deposiciones con perdida de
consistencia y coloracin disminuida, palidas, agrega prurito generalizado, coloracin amarillenta de piel y escleras, a la exploracin
fsica se encuentra aptica, afebril, con signos vitales dentro de rango normal, se palpa hepatomegalia dudosa, se ingresa a observacin
donde se realiza estudios de laboratorio con AST 119, ALT 158, Fosfatasa Alcalina de 476 UI/L, bilirrubina total de 23 mg/dl, y bilirrubina
directa de 20 mg/dl, se indica ultrasonografia no se observa vesicula biliar, se observa dilatacin de conductos biliares intrahepaticos,
pero no se muestra dilatacin del coldoco, resto de estructuras sin datos por agregar.

PREGUNTA
Cul es su impresin diagnostica ms probable con los estudios realizados.

RESPUESTA
a.- Cncer pancretico.
b.- Cncer de vesicula.
c.- Cncer heptico.
d.- Colangiocarcinoma.

CASO CLINICO
Se trata de paciente femenino de 47 aos de edad, inicia padecimiento hace 2 aos caracterizado por dolor en cuadrante superior
derecho, que se presentaba posterior a la ingesta de alimentos, ha sido tratada con analgesicos y antiespamodicos diversos sin
embargo desde hace 6 meses ya no han sido controlados e incluso se encuentra con un estado nauseoso continuo, tolera poco
alimento, a la exploracin se observa IMC 17, se observa ictericia importante.

PREGUNTA
Cul es la conducta a seguir ms adecuada.

RESPUESTA
a.- Endoscopia.
b.- Serie gastroesofagica.
c.- USG.
d.- Radiografa simple de abdomen.


MANUAL DE TRABAJO DEL CURSO ENARM CMN SIGLO XXI
CURSO ENARM CMN SIGLO XXI TEL: 36246001 Pharmed Solutions Institute PGINA 505

CASO CLINICO
Se trata de paciente femenino de 21 aos de edad la cual presenta malformacin cstica del coldoco la cual se identifico por USG
cuando la paciente refiri dolor en cuadrante superior derecho.

PREGUNTA
Se decide realizar reseccin quirrgica, cual es el motivo ms importante para la decisin.

RESPUESTA
a.- Degeneracin maligna.
b.- Colangitis recurrente.
c.- Cirrosis biliar.
d.- Coledocolitiasis.







MANUAL DE TRABAJO DEL CURSO ENARM CMN SIGLO XXI
CURSO ENARM CMN SIGLO XXI TEL: 36246001 Pharmed Solutions Institute PGINA 506

PANCREATITIS AGUDA Y CRONICA:
PANCREATITIS AGUDA (PA). CIENCIAS BASICAS: Enfermedad sistmica caracterizada por un proceso inflamatorio agudo no bacteriano
que resulta de la liberacin de las enzimas pancreticas dentro de la propia glndula, que puede comprometer por continuidad otros
tejidos y rganos vecinos. Usual curso benigno. PAL: mnima disfuncin multiorgnica, evolucin sin complicaciones. PAG: presencia de
falla orgnica o sistmica y/o presencia de complicaciones locales. SALUD PUBLICA: 20% desarrolla enfermedad severa. 20% de
mortalidad. 4-6ta. Dcada de la vida. Cerca de 40% de los
casos se debe a clculos biliares, otro 40% se debe a
alcoholismo. PATOGENIA: El pncreas normalmente secreta
una gran cantidad de enzimas, entre ellas la tripsina, la
quimiotripsina, las amilasas, lipasas, elastasas,
carboxipeptidasa A y B, etc. La gran mayora de ellas son
enzimas lticas que normalmente se secretan en forma de
precursores inactivos, los cuales junto con inhibidores
enzimticos presentes en el jugo pancretico, protegen al
pncreas de su autodigestin. Los eventos iniciales
patognicos de la Pancreatitis Aguda no estn del todo
aclarados. Existen evidencias de que la presencia de uno o
ms de los siguientes factores, aumento de la presin
intraductal en el sistema excretor pancretico (litiasis biliar,
tumores, pncreas divisum o anular), reflujo de bilis al
Wirsung, hipertrigliceridemia aguda o preexistente, frmacos (aziatropina, clortiazida, estrgenos, furosemida, sulfamidas,
tetraciclinas), infecciones (hepatitis, parotiditis, CMV, candida, scaris), dao isqumico, post-CEPRE, disrupcin de algn conducto
excretor (trauma), etc., desencadena una cascada inflamatoria que se asocia a la activacin enzimtica intraglandular, con las
consecuencias de dao microvascular, trombosis, necrosis tisular, saponificacin del tejido graso, liberacin de radicales libres y
eventualmente gatillamiento de una respuesta inflamatoria sistmica. PA Y LITIASIS BILIAR: Es ms frecuente la PA en pacientes con
clculos vesiculares pequeos, coledocolitiasis, conducto cstico ancho, conducto bilio pancretico distal comn, se inicia por la
obstruccin del conducto pancretico por un clculo en la ampolla de Vater, aunque sea una obstruccin transitoria, con aumento de la
presin intraductal, reflujo biliar a los conductos pancreticos y eventual contaminacin bacteriana por este mecanismo. Las sales
biliares desconjugadas y la lisolecitina son toxicas para el pncreas. PA Y ALCOHOL: La ingesta alcohlica excesiva de tipo inveterada es
un factor asociado a crisis de PA a repeticin, que si bien el primer episodio puede ser de gravedad, las crisis posteriores son
clnicamente menos importantes y que llevan al cuadro de la pancreatitis crnica
recurrente. En las teoras se ha postulado: (1) Un aumento de la secrecin gstrica que
conduce a un aumento exagerado de la secrecin pancretica; (2) Inflamacin duodenal y
periampular con obstruccin de los conductos bilio-pancreticos; (3) La hiperlipidemia
aguda inducida por el alcohol. En la mayora de los casos la PA se produce 24 a 48 horas
despus de la ingesta, siendo poco frecuente el encontrar niveles altos de alcoholemia
simultneamente al evento clnico de la PA. 4. P.A. e hipertrigliceridemia: DIAGNOSTICO:
Clnica; produce dolor intenso epigstrico (estiramiento de la capsula pancretica)
transfictivo, que se irradia a la espalda, se alivia al sentarse y a menudo se acompaa de
nausea intensa, puede haber relacin con ingesta de alcohol o grasas. Tambin se
observa sensibilidad abdominal superior y defensa muscular. Alrededor de 90% de los
pacientes tiene fiebre, leucocitosis y taquicardia. El leo es usual y tambin es posible que
ocurra choque por secuestro de lquido y depresin miocrdica. La ictrica se presenta en
20-30% de los pacientes, por obstruccin o compresin del coldoco. En ocasiones hay
espasmo carpopedal por hipocalcemia. Alrededor de 1% de los pacientes tiene sangre retroperitoneal alrededor de la cicatriz umbilical
(Signo de Cullen) o en los flancos (Signo de Grey- Turner). Pancreatitis + Fiebre persistente + Resp. Inflamatoria sistmicapensar en
absceso o infeccin. Laboratorio y gabinete: pueden presentar hiperamilasemia (3 veces lo normal, aumento 2-5 das), manifestacin
muy inespecfica (elevada en colangitis, colecistitis, ulcera pptica perforada, salpingitis, IRC, etc). La determinacin srica de lipasa es
ms especfica (aumenta a partir de las 72 h, regresa lentamente, mejor prueba diagnstica). La depuracin urinaria de amilasa puede
ser til. El nivel de calcio disminuye a veces (<7.5mg/dl, mal pronstico, secuestro de calcio en las reas de necrosis grasa).
Hiperglucemia (aumento transitorio, liberacin de glucagn aumentada, glucemia > 200 mg/dL signo de mal pronstico = necrosis
extensa). PCR: su elevacin va en relacin con el mal pronstico. La radiografa puede mostrar un asa centinela con aire intestinal.
Paciente con diagnstico de PA: hacer ecografa abdominal (litiasis biliar) en las primeras 24-48 horas. La imagen por TAC ayuda a
predecir la gravedad y a establecer el diagnostico de las complicaciones. ndice de severidad por TAC, CRITERIOS DE BALTHAZAR: A
Pncreas normal (0 puntos). B Aumento de tamao focal o difuso del pncreas (1 punto). C Inflamacin del pncreas y/o grasa
peri/pancretica (2 puntos). D Coleccin pancretica nica (3 puntos). E dos o ms colecciones peripancreticas y/o gas
retroperitoneal (4 puntos). Porciento de necrosis: 0% =0 puntos, 30% = 2 puntos, 30-50% = 4 puntos, >50% =6 puntos. Suma de puntos
en TAC = ndice de severidad. 0-3 bajo, 4-6 medio, 7-10 alto. TRATAMIENTO: La reposicin de las perdidas hidroelectrolticas es parte
crucial de la atencin a estos pacientes, as como la vigilancia de volumen vascular, las valoraciones repetidas de hematocrito y
electrolitos y el reposo intestinal. El uso rutinario de antibiticos en pancreatitis no complicada no se justifica. La colecistectoma est
indicada en la pancreatitis biliar. El lavado peritoneal puede disminuir las complicaciones cardiopulmonares, pero no reduce la
mortalidad. La intervencin quirrgica es necesaria en casos de clculos impactados, desbridacin de necrosis y drenaje de abscesos.
COMPLICACIONES: La complicacin ms frecuente es el pseudoquiste (coleccin liquida), que suele presentarse despus de 2-3
semanas, puede ser nico o mltiple, tx., drenaje quirrgico, el 50% se resuelven de manera espontnea a las 6 semanas. Necrosis
pancretica temprana y tarda (Dx: puncin por aspiracin, tratamiento: desbridamiento quirrgico). Absceso pancretico (4-6
semanas, Dx: puncin aspiracin, tratamiento: drenaje percutneo). Infeccin: principal causa de morbimortalidad, aparece de 2-3
CRITERIOS DE RANSON PARA PA
Presentes al momento del ingreso:
Edad >55aos
Cuenta leucocitaria >16 000/l
Glucosa sangunea >200mg/dl
Deshidrogenasa lctica srica >350 UI/L
AST >250UI/dl
Se desarrollan durante las primeras 48 hrs
Descenso del hematocrito >10%
Aumento >8mg/dl en BUN
Calcio srico menos de 8mg/dl
PO2 arterial >60mmHg
Deficiencia de base >4mEq/l
Secuestro estimado de lquido >600ml
0-2 = 0.9% de mortalidad; 3-4 = 16% ;
5-6 = 40% ; >6 = 91 de mortalidad
MANUAL DE TRABAJO DEL CURSO ENARM CMN SIGLO XXI
CURSO ENARM CMN SIGLO XXI TEL: 36246001 Pharmed Solutions Institute PGINA 507

semanas despus, deben usarse antibiticos que penetren en pncreas, primera eleccin imipenem, tambin cefalosporinas de tercera
generacin y quinolonas, mnimo 14 das, o hasta que no haya complicaciones sistmicas. Las complicaciones sistmicas incluyen SDRA,
insuficiencia renal y depresin miocrdica.

CASO CLINICO
Ingres paciente de sexo masculino de 68 aos de edad, con antecedentes de Diabetes Mellitus en tratamiento con hipoglicemiantes
orales, colecistectoma y alcoholismo en remisin desde haca 15 aos. Presentaba dolor abdominal, inicialmente en epigastrio e
hipocondrio izquierdo de 20 h de evolucin, el cual haba ido progresando en intensidad y se extenda ya al resto del abdomen. La
aparicin del dolor fue brusca y se asoci a sudoracin fra y sensacin de desvanecimiento, sin prdida de conciencia. No se acompa
de vmito ni diarrea. En el examen fsico del ingreso destacaba un paciente consciente, algo plido, afebril, con taquicardia leve, una
presin arterial de 110/60 mm de Hg sin apremio respiratorio y adecuada perfusin perifrica. No haba estigmas de dao heptico
crnico. En el examen pulmonar haba disminucin del murmullo pulmonar en ambas bases, con predominio izquierdo. En el examen
abdominal se evidenciaba cierto abombamiento, la pared era depresible pero la palpacin provocaba intenso dolor con signo de
Blumberg positivo en hemiabdomen superior. Los ruidos hidroareos estaban ausentes. No se palpaban masas pulstiles.

PREGUNTA
Considerando el cuadro clnico cual es su impresin diagnostica inicial?

RESPUESTA
a.- Ulcera perforada.
b.- Pancreatitis aguda.
c.- Sindrome hepatorrenal.
d.- Ruptura de varices esofgicas.

PREGUNTA
Se obtuvieron los siguientes resultados de laboratorio posteriomente, hemoglobina 9 g/L, recuento leucocitario de 8000/L,
protrombina 55%, bilirrubina 1,4 mg/dL, albmina 2,3 mg/dL; amilasemia, lipasemia y protena C reactiva en rangos normales. Los
estudios radiolgicos de trax mostraron atelectasia en ambas bases pulmonares, sin neumoperitoneo y la radiografa de abdomen
simple mostr aire en el colon, escasa cantidad en intestino delgado y asas no dilatadas, considerando los datos observados que
puntaje presenta de la clasificacin de Child-Pugh.

RESPUESTA
a.- Menos de 5 puntos.
b.- De 5 a 7 puntos.
c.- De 7 a 10 puntos.
d.- Mas de 10 puntos.

PREGUNTA
El paciente se manej con aportes de cristaloides y evolucion con disminucin del dolor y sin compromiso hemodinmico. En los
exmenes de control al segundo da de hospitalizacin se observ disminucin de la hemoglobina a 5 g/L, protrombina 54% y recuento
plaquetario de 59.000/L. DHL 450 U/l, AST 140, Glucosa de 250 mgs/dl. Cuantos criterios de Ramson presenta?.

RESPUESTA
a.- 2 criterios.
b.- 3 criterios.
c.- 4 criterios.
d.- 5 criterios.

PREGUNTA
Cual de los siguientes diagnosticos diferenciales es el menos probable en este caso clnico?

RESPUESTA
a.- Hemoperitoneo espontneo en cirrosis.
b.- Rotura de vrices o canales linfticos.
c.- Rotura de carcinoma hepatocelular
d.- Rotura de aneurisma abdominal.










MANUAL DE TRABAJO DEL CURSO ENARM CMN SIGLO XXI
CURSO ENARM CMN SIGLO XXI TEL: 36246001 Pharmed Solutions Institute PGINA 508

PANCREATITIS CRONICA. CIENCIAS BASICAS: La definicin es vaga, pero por lo general incluye cambios secundarios a los episodios
repetidos de pancreatitis aguda. El pncreas se vuelve, pequeo, indurado y nodular, con acinos e islotes rodeados por tejido fibroso.
Se observa estrechamiento y dilacin de los conductos; la calcificacin es frecuente,
los cambios estructurales son irreversibles y progresivos. SALUD PUBLICA:
Incidencia oscila entre 5-10 casos nuevos anuales por 100,000 hab y prevalencia de
30-40 por 100,000 hab. Algunos estudios elevan estas cifras hasta 5% de la
poblacin general. CLASIFICACION: TIGAR-O; Toxica-metablica que es la ms
frecuente porque incluye el alcohol (70-90%), tambin la hipercalcemia, y algunos
mencionan la falla renal crnica. Idioptica alcanza de 10-30% se han agrupado en
una forma juvenil que se presenta en las primeras dcadas de la vida y una forma
en 4ta-5ta dcada. Gentica, se han descrito algunas mutaciones, en el tripsinogeno
catinico, CFTR, SPINK1. Autoinmune es una entidad poco frecuente con hallazgos
clnicos y paraclnicos definidos e histolgicamente su caracterstica es el extenso
infiltrado linfoplasmocitario. Recurrente, en el que se incluyen pacientes que
posiblemente debido a mltiples episodios de pancreatitis aguda pueden conducir a
pancreatitis crnica, sin embargo existe controversia. Obstructiva, plantea el dao
primario sobre los conductos pancreticos, posiblemente por dao inmunolgico
sobre el epitelio, en forma similar a los que se observa en la colangitis esclerosante
primaria. Pncreas divisum, deficiencia del esfnter de Oddi, traumatismos.
PATOGENIA: La clulas estrelladas tienen un papel en la fibrosis pancretica, en el pncreas se localizan en la porcin exocrina
(espacios periacinar, perivascular), en condiciones normales, por sus propiedades contrctiles ayudan a regular, las presiones en estos
compartimentos, adems pueden contribuir a mantener la matriz extracelular y tienen capacidades limitadas de migracin y
proliferacin. De acuerdo a la teora conocida como necrosis-fibrosis, se plantea que la necrosis y la inflamacin pancretica son
procesos que activan a las clulas estrelladas, por factores denominados autocrinos por medio de molculas proinflamatorias que
adems aumentan la secrecin de colgeno y la expresin de TGF-B1 y endotelina, que estimulan la migracin, la contraccin y la
liberacin de sustancias proinflamatorias, perpetuando sus efectos aun cuando el estmulo inicial haya terminado, favoreciendo el
desarrollo progresivo de fibrosis, el dao repetido y persistente en la pancreatitis crnica lleva a una activacin continua de clulas
estrelladas que finalmente conduce a fibrosis de la glndula y perdida de sus funciones exocrinas. Existe una evidencia clara sobre la
capacidad del alcohol para inducir lesin pancretica directa. Los alcohlicos ingieren dietas deficientes en numerosos micronutrientes,
especialmente selenio y zinc, lo que podra ser origen de mecanismos celulares de lesin pancretica inducidos por la accin de
radicales libres de oxgeno. Adems tambin produce fenmenos de hipertensin intraductal, disminucin del flujo sanguneo
pancretico, toxicidad directa sobre la clula acinar, cambios en la sntesis proteica, incremento de la respuesta inflamatoria y
estimulacin de la fibrogenesis pancretica. DIAGNOSTICO: Clnica; Los sntomas comprenden dolor epigstrico o en espalada continuo
o intermitente, anorexia y maldigestin manifestada por prdida de peso, esteatorrea. La diabetes puede desarrollarse a lo largo de la
enfermedad y se caracteriza por destruccin de clulas productoras de insulina y productoras de glucagn; la coexistencia de este dao
aumenta el riesgo de hipoglicemia, por la deficiencia concomitante de sntesis de glucagn. Durante los ataques agudos hay vomito. A
menudo se forman pseudoquistes. Tambin son frecuentes las alteraciones de la personalidad. Segn su etiologa pueden ocurrir otras
manifestaciones como fenmenos de autoinmunidad u hepatopata. Laboratorios y gabinete: la presencia de cambios morfolgicos en
pruebas de imagen y/o en la demostracin de insuficiencia pancretica. La CPRE y las imgenes con TAC, RMN ayudan y es probable
que muestren dilatacin ductal, clculos y estrechamientos. Los niveles sricos de amilasa y lipasa tienen poco valor. La calcificacin
pancretica es patognomnica. Histolgicamente se requiere la presencia de fibrosis y atrofia acinar que se acompaa de un
componente variable de infiltrado inflamatorio crnico (se dispone pocas veces de estudio histolgico). TRATAMIENTO: Suprimir el
consumo de alcohol. Analgsicos en forma escalonada: AINES, opiceos dbiles (dextropropoxifeno, tramadol), combinaciones:
dextropropoxifeno + dipirona (Klosidol); dextropropropoxifeno + ibuprofeno (Supragesic). Si no hay respuesta: agregar amitriptilina.
Tratamiento de la maldigestin: Reemplazo enzimtico: 25.000 a 50.000 U de lipasa en cada comida. IBP (Omeprazol). La
pancreatoyeyunostoma (procedimiento de Puestow), es conveniente en casos de pancreatitis alcohlica crnica, dolor crnico y
alternancia de dilatacin con estrechamientos ductales (cadena de lagos). Se obtienen buenos resultados con la
pancreatoduodenectoma cuando la pancreatitis crnica se acompaa de estrechamiento del conducto en la cabeza del pncreas. Si el
conducto es pequeo, se obtienen cierto beneficio con la pancreatectoma de 95%, pero la mortalidad es alta. La abstinencia de alcohol
resulta primordial. El dolor crnico es difcil de tratar, si los sntomas persisten o recurren se puede hacer un bloqueo celiaco.
COMPLICACIONES: Pseudoquistes complicados; la estenosis biliar, duodeno o colon; la trombosis venosa (esplnica, mesentrica-
portal); los pseudoaneurismas; la hemorragia intraquistica, intrabdominal o digestiva; las fistulas internas o externas; las ascitis y el
cncer de pncreas.

CASO CLINICO
Varn de 66 aos, con antecedentes de obesidad, HTA, diabetes mellitus tipo 2, ex fumador, sndrome de apnea obstructiva del sueo
con espirometra con patrn mixto, herniorrafia inguinal, hiperplasia benigna de prstata, HDA por lcera prepilrica y pancreatitis
litisica leve. Acude por dolor abdominal intenso, de 2 h de evolucin, de inicio brusco en hemiabdomen superior, que no cede con
analgsicos y se acompaa de vmitos. Mantiene buenas constantes vitales (presin arterial, 130/80 mmHg; frecuencia cardaca, 96
lat/min; frecuencia respiratoria 20/min, y temperatura, 36C). En la analtica destacan 18.000 leucocitos; amilasa, 5.900; AST, 169; ALT,
105, y BT, 2,5g/dl; PCR, 0,8. En la ecografa abdominal se visualiza una vescula distendida con barro biliar sin dilatacin de la va biliar.
No se observa bien el pncreas, pero se ve aumentado de tamao e hipoecoico. Tras 12 h, empeora sbitamente su estado clnico, con
agitacin, sudor, taquipnea, livideces y agudizacin del dolor abdominal (presin arterial, 90/60 mmHg; frecuencia cardaca, 160
lat/min). En la analtica presenta 15.500 leucocitos; glucosa, 332 mg/dl; creatinina, 1,7 mg/dl; PCR, 6,8 mg/dl. El abdomen est
distendido sin peritonismo. Su situacin clnica se agrava con hipotensin, mala perfusin perifrica, bradicardia extrema y asistolia que
DAO POR
ALCOHOL
MANUAL DE TRABAJO DEL CURSO ENARM CMN SIGLO XXI
CURSO ENARM CMN SIGLO XXI TEL: 36246001 Pharmed Solutions Institute PGINA 509

precisa RCP. Hay deterioro analtico: leucocitos, 4.470; creatinina, 2,5mg/dl; pH 7,23, y bicarbonato, 10,9mmol/l, y precisa
noradrenalina a altas dosis (30) y FiO2 al 100%.

PREGUNTA
Cul es su impresin diagnostica ms probable con los estudios realizados.

RESPUESTA
a.- Necrosis pancretica grave con acidosis metabolica.
b.- Pancreatitis crnica agudizada de origen alcohlica
c.- Colangitis aguda bacteriana secundaria.
d.- Colecistitis crnica litiasica con obstruccin pancretica.

CASO CLINICO
Varn de 79 aos; como antecedentes personales de hipertensin arterial, fibrilacin auricular y varios episodios de hemorragia
digestiva por angiodisplasias yeyunales tratadas con gas argn. Ingresa por nuevo episodio de hemorragia digestiva de probable origen
yeyunal. Los hallazgos fueron la identificacin en yeyuno proximal de 3 angiodisplasias de tamao milimtrico que se electrocoagularon
con argn, tambin un pequeo plipo de 3 mm que se extirp con pinzas. El da siguiente del procedimiento, el paciente comenz con
dolor abdominal intenso y nuseas. La exploracin abdominal reflejaba un abdomen globuloso y distendido con dolor y defensa en
epigastrio y flanco izquierdo. En la analtica urgente presentaba 16.400 leucocitos con 88% neutrfilos, una amilasemia de 410 y lipasa
148. Se realiza TAC abdominal urgente que describe hallazgos compatibles con pancreatitis.

PREGUNTA
Cul es factor de severidad ms grave para este caso.

RESPUESTA
a.- Edad mayor a 50 aos.
b.- ndice de masa corporal mayor a 30.
c.- Presencia de derrame pleural.
d.- Valores de amilasa y lipasa.












MANUAL DE TRABAJO DEL CURSO ENARM CMN SIGLO XXI
CURSO ENARM CMN SIGLO XXI TEL: 36246001 Pharmed Solutions Institute PGINA 510

ISQUEMIA MESENTERICA AGUDA (IMA). CIENCIAS BASICAS: Entidad clnica multietiolgica, producida por la interrupcin brusca del
aporte sanguneo a un determinado segmento intestinal, lesionndolo al principio de forma reversible pero que si se mantiene el
tiempo suficiente, hace que deje de ser viable y evolucione hacia la necrosis completa de su pared. SALUD PUBLICA: Supone un tercio
de las isquemias intestinales. Proceso con elevada mortalidad (60-80%) y en el cual es vital la sospecha clnica temprana, ya que la
supervivencia depende directamente de la rapidez con que se aplique el tratamiento. PATOGENIA: Oclusin Arterial (embolia arterial,
trombosis arterial): La arteria ms frecuentemente implicada es la mesentrica superior (AMS), el tronco celiaco y la mesentrica
inferior (AMI) tambin pueden ocluirse, pero gracias a la circulacin colateral, no suelen producir lesin isqumica aguda, a menos que
se afecte a la vez la AMS. Oclusin Venosa (trombosis venosa mesentrica): La imposibilidad de retorno venoso produce edema y
aumento de presin venosa que cuando se iguala a la arterial
produce isquemia e infarto hemorrgico. Isquemia no oclusiva
(IMNO): el bajo gasto, la hipotensin o la vasoconstriccin local
mantenidos pueden impedir el aporte sanguneo mnimo necesario
para la viabilidad intestinal. CLASIFICACION: 1. EMBOLIA ARTERIAL:
es la causa ms frecuente de IMA (50%) Su origen en un 90% de los
casos es una cardiopata embolgena y puede existir historia de
embolias previas a otros niveles. La zona ms frecuente de
embolizacin es la AMS, distalmente a la salida de la A. Clica Media,
afectando al colon derecho y rea leo-cecal. La isquemia que se
produce es muy grave, por la brusquedad con que se instaura y por
la no existencia de colaterales. 2. TROMBOSIS ARTERIAL: supone un
25% de las IMA. Son pacientes con antecedentes vasculares (es
caracterstica la historia previa de Isquemia Mesentrica Crnica)
cuyas arterias tienen disminuido el flujo y han desarrollado
circulacin colateral. La trombosis de la AMS suele producirse en su
origen o en sus 3 primeros centmetros y la mortalidad es muy alta,
porque a pesar de que la circulacin colateral es capaz de mantener cierto flujo, el territorio afectado es muy extenso3. TROMBOSIS
VENOSA MESENTRICA: ha de afectarse de forma muy severa el eje porto-esplnico-mesentrico e incluso en estos casos es
excepcional que una trombosis venosa produzca un infarto intestinal agudo. Antecedente habitual de hipercoagulacin o TVP. 4.
Insuficiencia Mesentrica No Oclusiva (IMNO): produce el 20% de las IMA. Son pacientes con el flujo esplcnico de base disminuido por
la arteroesclerosis generalizada o el consumo de frmacos vasoactivos como la digital y que ante una disminucin de perfusin
generalizada, desencadenada por un proceso agudo, no pueden mantener el aporte sanguneo necesario en el territorio intestinal. Los
precipitantes de la IMNO van desde el sock cardiognico hasta una deshidratacin que desencadene hipovolemia. Excepcionalmente el
consumo de txicos con actividad simptica como la cocana o los ergotamnicos, pueden producir vasoconstriccin visceral aislada y
desencadenar IMNO. DIAGNOSTICO: Sospecha clnica: es la base del diagnstico y se establece ante un paciente con perfil de riesgo
definido, que a primera vista impresiona de gravedad y presenta dolor abdominal severo, acompaado de forma ms o menos
frecuente de distensin abdominal, nauseas, vmitos, diarrea y/o rectorragia. Llama la atencin la normalidad de la exploracin fsica,
sin signos de irritacin peritoneal, en contraste con la gran intensidad del dolor. 1. Embolia Arterial: el dolor es de comienzo brusco y
localizacin periumbilical. 2. Trombosis Arterial: inicio gradual del dolor, incluso de das de evolucin, con aumento progresivo de
intensidad y localizacin difusa. 3. Trombosis Venosa Mesentrica: dolor variable segn la extensin de la trombosis, habitualmente
difuso de instauracin progresiva e intensidad ms leve que en las de causa arterial.4. IMNO: el dolor abdominal es difuso y de rpida
instauracin, aunque en ocasiones, por la patologa desencadenante son pacientes graves que se encuentran en cuidados intensivos y
sedoanalgesiados en los que el dolor no es valorable. Se sospecha cuando existe episodio reciente de bajo gasto e hipotensin
mantenida y aparece fiebre y leucocitosis, distensin abdominal o sangrado digestivo. Si se produce la necrosis completa de la pared
intestinal aparecen defensa abdominal y signos de irritacin peritoneal con hiperperistaltismo primero y luego silencio abdominal total.
Son pacientes inquietos, sudorosos y que no se dejan explorar por la gran intensidad del dolor. Aparecen tambin signos de deterioro
hemodinmico: taquicardia, hipotensin, oligoanuria, etc. Laboratorio y gabinete: Radiologa simple: es normal inicialmente. Cuando
aparece la necrosis podemos ver edema de pared, distensin de asas, y gas intramural (infarto intestinal) o libre en peritoneo
(perforacin). BH de inicio discreta leucocitosis. Posteriormente aparece: leucocitosis por encima de 20000 y con gran desviacin
izquierda. Acidosis metablica severa y/o coagulopata que en este contexto de dolor abdominal sugieren isquemia o sepsis grave.
Hiperamilasemia y elevacin de CPK, LDH, GOT y GPT que reflejan la presencia de importante destruccin celular. Puede aparecer
hemoconcentracin por presencia de importante edema y tercer espacio abdominal o anemia en los casos que cursan con importante
sangrado digestivo. La arteriografa es mtodo diagnstico tpico, pero es una prueba menos disponible que el TC o el ECO-doppler, que
han aumentado mucho su sensibilidad adems de ser ms tiles como primera prueba diagnstica, porque ayudan orientar el
diagnstico diferencial. TC Helicoidal con contraste IV: muy til para el diagnstico diferencial y es muy sensible para detectar signos
indirectos de infarto intestinal: neumatosis intestinal, dilatacin de asas, edema de pared. Trombosis Venosa Mesentrica: hoy da es el
Gold standart. Se aprecia un retraso del paso de contraste al sistema venoso, una pared engrosada y la falta de opacificacin de la
porta. Oclusin emblica o trombtica Arterial: arteriografa sigue siendo superior al TC. TRATAMIENTO: Inicial: Infusin vigorosa de
cristaloides sonda de aspiracin gstrica, analgesia y antibiticos intravenosos (Cefotaxima1 2gr + Metronidazol 500mg cada 8h o
Piperacilina-tazobactam 4gr / 6h). T. definitivo: si existe infarto intestinal establecido el tratamiento siempre es quirrgico, para
reseccin del segmento necrosado, con la mxima urgencia posible. 1. Oclusin emblica o trombtica Arterial: tratamiento quirrgico
para repermeabilizar los segmentos aun viables mediante embolectoma o By-pass 2. Trombosis Venosa Mesentrica: si no hay infarto
intestinal establecido se realiza tratamiento conservador anticoagulando con heparina. 3. IMNO: lo fundamental es la correccin de los
factores desencadenantes. Se puede realizar infusin de vasodilatador (papaverina) intrarterial, durante la arteriografa. Hay autores
que recomiendan el empleo de IECA por la relacin del eje renina-angiotensina en la fisiopatologa del cuadro.

MANUAL DE TRABAJO DEL CURSO ENARM CMN SIGLO XXI
CURSO ENARM CMN SIGLO XXI TEL: 36246001 Pharmed Solutions Institute PGINA 511

ISQUEMIA MESENTERICA CRONICA (IMC). CIENCIAS BASICAS: Tambin conocida como angina intestinal se produce cuando existe
una desproporcin entre las demandas de oxgeno, la cual ocurre fundamentalmente durante la digestin, y el flujo sanguneo
proporcionado por el sistema vascular. Generalmente se produce en personas con marcada ateroesclerosis y representa menos del 5%
de los casos de isquemia intestinal. Clnica; dolor abdominal que aparece precozmente tras la ingesta y cede en el plazo de 2 hrs. La
intensidad es mayor tras la ingesta de comidas copiosas con alta proporcin en grasa. Los sntomas suelen aumentar con la
consecuente prdida de peso, descrita hasta en el 80% de los casos debido a la aversin de la comida por miedo al dolor. En algunos
casos puede producirse una IMA sobre una IMC, debido a la formacin de trombos sobre placas ateroesclerosas. El diagnostico se
establece por el cuadro clnico compatible, en pacientes con coexistencia de otras enfermedades vasculares (cardiaca, cerebral o
perifrica). La demostracin arteriografica de obstruccin de al menos dos de los vasos esplcnicos y la exclusin de otras patologas
que cursen con sntomas similares. La demostracin por s sola no constituye una prueba inequvoca de IMC, puesto que estas lesiones
se pueden encontrar en sujetos asintomticos. La rentabilidad de las pruebas bioqumicas reside en la identificacin de factores de
riesgo para desarrollo de IMC, como son la dislipidemias o la DM, as como la identificacin de datos que apoyen la malabsorcin
intestinal que pueden presentar estos pacientes. El tratamiento incluye construccin quirrgica y ATP (angioplastia transluminal
percutnea) con o sin la colocacin de un stent. La eleccin va a depender de la experiencia de cada centro, de la edad del paciente,
comorbilidad asociada y del nmero y severidad de la oclusin vascular. Revascularizacin quirrgica: como la reimplantacin de la
AMS en la aorta abdominal, la endarterectoma mesentrica y el bypass.

CASO CLINICO
Mujer de 83 aos con antecedentes de cardiopata hipertensiva, diabetes mellitus tipo 2, hipertensin arterial, flutter auricular y
episodio de pancreatitis aguda de origen biliar. Acude a urgencias por episodio de dolor abdominal localizado en epigastrio que se
irradia hacia ambos hipocondrios, acompaado de nuseas, vmitos y distensin abdominal. La analtica muestra leucocitosis (17,68 x
10-3/ul) con predominio de neutrfilos (92,8% N), as como un aumento marcado de LDH (1082 U/l). La gasometra venosa, los
parmetros bioqumicos de perfil heptico y biliar, as como los niveles de amilasa, se encuentran dentro de los lmites normales. Se
realiza una RX de abdomen donde se aprecia dilatacin de asas de delgado, con presencia de gas distal, sugiriendo suboclusin
intestinal y una ecografa abdominal que mostraba quistes simples renales, sin evidencia de lesiones ni lquido libre intraperitoneal.

PREGUNTA
Cul es la patologa ms probable con los datos clnicos.

RESPUESTA
a.- Isquemia mesentrica aguda.
b.- Isquemia mesentrica crnica.
c.- Colitis isqumica.
d.- Oclusin intestinal recurrente.

CASO CLINICO
Mujer de 61 aos diagnosticada de obesidad mrbida e hipertensin arterial, que acude a urgencias por dolor periumbilical de 2 das de
evolucin. En la ecografa abdominal se observa escaso lquido libre y conglomerado de asas intestinales sin herniacin. 24 horas
despus la paciente presenta un cuadro de fallo multiorgnico. En la tomografa computarizada de abdomen con contraste intravenoso,
se observa neumatosis intestinal de la totalidad de asas del intestino delgado con eventracin abdominal anterior incarcerada y
presencia de gas en venas mesentricas, perihepticas, periesplnicas y ramas distales intrahepticas. En la ciruga se evidencia
isquemia mesentrica masiva y colon transverso incarcerado.

PREGUNTA
Cul es el antecedente ms probable con los datos clnicos y el cuadro clnico actual.

RESPUESTA
a.- Colitis isqumica.
b.- Isquemia focal segmentaria.
c.- Trombosis venosa mesentrica.
d.- Isquemia mesenterica crnica.

CASO CLINICO
Paciente mujer de 55 aos, con antecedentes de tabaquismo crnico, hipertensin arterial moderada y dislipidemia en tratamiento,
cuyo cuadro comenz en el ao 2012, caracterizado por dolor abdominal tipo clico postprandial precoz, acompaado de diarrea
frecuente, alrededor de 10 episodios diarios y vmitos ocasionales, adems de baja de peso de alrededor de 15 kilos, siendo su peso
habitual de 55 kg, llegando a un ndice de masa corporal de 17,3. Estudiada en el contexto de diarrea crnica y bsqueda de
enfermedad inflamatoria intestinal, se someti a endoscopia alta y baja, adems de enteroscopia, trnsito intestinal y estudio de mala
absorcin, lo que slo concluy gastritis y colitis inespecfica, no respondiendo a tratamiento habitual, esto es, rgimen alimenticio,
antiespasmdicos y medicamentos antiulcerosos.

PREGUNTA
Cual es la conducta a seguir mas adecuada?.

RESPUESTA
a.- Eco-dopler
MANUAL DE TRABAJO DEL CURSO ENARM CMN SIGLO XXI
CURSO ENARM CMN SIGLO XXI TEL: 36246001 Pharmed Solutions Institute PGINA 512

b.- Angio-TAC.
c.- Angio- IRM.
d.- Angiografia.

INFARTO MESENTERICO. CIENCIAS BASICAS: El infarto intestinal, isquemia intestinal o necrosis intestinal es la muerte del tejido
intestinal debido a una interrupcin del suministro de sangre, de una o ms de las arterias mesentricas, principales arterias que
suministran sangre al intestino delgado. Suele venir precedida de sintomatologa de angina intestinal, por insuficiencia vascular del
mesenterio, que degenera en isquemia. Existen varias causas posibles del infarto intestinal: En una hernia, si la irrigacin del intestino
queda ocluida, esto puede llevar a isquemia intestinal. El intestino tambin puede quedar atrapado en tejido cicatricial de una ciruga
previa. Una embolia que bloquee los vasos principales que irrigan el intestino, as como la trombosis arterial, pueden impedir el
suministro adecuado, generalmente a partir de una enfermedad ateroesclertica (acumulacin de colesterol). Recientemente, se ha
sugerido que este bloqueo podra ser en parte causado por un remodelado vascular anmalo. Una trombosis de las venas que evacuan
la sangre del intestino, que se pueden obstruir con cogulos de sangre. sta es una afeccin ms frecuente en personas con
insuficiencia heptica, cncer o trastornos de la coagulacin. Una hipotensin, ya que la presin arterial muy baja en pacientes con
obstruccin previa de las arterias tambin puede ocasionar isquemia intestinal. Clnica; dolor abdominal de inicio sbito, diarrea,
fiebre, vmitos, leucocitosis, leve acidosis. El tratamiento generalmente requiere ciruga, conocida como enterectoma, en la que se
extirpa la porcin necrosada y se unen los cabos seccionados. Segn la regin implicada, se denomina colostoma o ileostoma. Si es
posible, se corrige la obstruccin de las arterias que irrigan el intestino. El tratamiento tambin puede incluir medicamentos para
prevenir los cogulos, disolver los cogulos ya existentes, o la dilatacin de los vasos sanguneos. Si la angiografa se realiza para
diagnosticar el problema, es posible eliminar al mismo tiempo un cogulo de sangre o abrir una pequea arteria con la angioplastia. La
angioplastia consiste en utilizar un globo en la punta de un catter para comprimir la arteria y la extensin de los depsitos de grasa,
haciendo un camino ms amplio para que la sangre fluya sin problemas.

CASO CLINICO
Paciente varn de 27 aos de edad, con cuadro clnico de 10 horas de evolucin, caracterizado por dolor abdominal de comienzo en
epigastrio, sostenido, con migracin a FID sin defensa ni peritonismo. Se acompaa de naseas, sin vmitos ni fiebre con apetito
conservado. Rx de abdomen: nivel hidroareo en flanco y fosa ilaca derecha. Laboratorio: > GB 21700 (89%N) resto: normal, se
diagnostica apendisitis, se realiza apendisectomia, egresando dos das despus sin datos anormales. El paciente re-ingresa, con cuadro
clnico de 48 hs de evolucin caracterizado por sndrome febril, que cede parcialmente con la ingesta de antipirticos (Ibuprofeno VO).
Examen fsico: > TA 120/70, Fc 133x, Fr 16x, T 39,1C, > Leve dolor abdominal, mayor sobre zona operada, signos de flogosis y
supuracin serohemtica en cicatriz de Mc Burney, mantiene trnsito intestinal, sin dificultad al orinar. > Laboratorio: GB 21100 (92%N)
resto normal.

PREGUNTA
Cual es su impresin diagnostica?

RESPUESTA
a.- Sepsis abdominal.
b.- Absceso abdominal.
c.- Trombosis mensenterica.
d.- Diverticulitis aguda.

PREGUNTA
Cual es la conducta a seguir mas adecuada para establecer el diagnostico?

RESPUESTA
a.- LAPE.
b.- Angio-TAC.
c.- Eco-dopler.
d.- USG abdominal.

PREGUNTA
En esta patologa en su forma aguda, cual de los siguientes sntomas es el menos frecuente?

RESPUESTA
a.- Dolor abdominal
b.- Anorexia.
c.- Diarrea.
d.- Nuseas y vmitos.







MANUAL DE TRABAJO DEL CURSO ENARM CMN SIGLO XXI
CURSO ENARM CMN SIGLO XXI TEL: 36246001 Pharmed Solutions Institute PGINA 513

DIVERTICULITIS. CIENCIAS BASICAS: Enfermedad diverticular: Enfermedad de colon asociado a cambios en el hbito intestinal, dolor y
distensin abdominal. Diverticulosis: Paciente con divertculos en ausencia de cualquier sntoma. Diverticultis: Complicacin
inflamatoria de la enfermedad diverticular. Factores de riesgo: AINES, edad, inmunocompromiso, tabaquismo y alcoholismo, bajo
consumo de fibra, gentica, Sndrome de Williams, Enf. Poliqustica Renal, Sndrome de Ehlers- Danlos. SALUD PUBLICA: Aproximado de
50-65% en >60 aos. 10-15% diverticulosis desarrollaran Diverticultis.
PATOGENIA: Anatoma patolgica: Sitio de debilidad en la pared
colnica. Disminucin en la colgena tipo 1, incremento en colgena
tipo 3, incremento en la presin intraluminal 90mmHg, hay una
respuesta exagerada sistema simptico, motilidad y electrofisiologa,
hay aumento en los movimientos de segmentacin y retropulsin.
Histopatologa: Hipertrofia e hiperplasia de los segmentos afectados,
debilidad y adelgazamiento en el sitio de protucin o salida del divertculo. Teora de Ryan Tipo 1: anormalidad muscular clsica por
presin aumentada, confinada al colon izquierdo, dolor inflamacin y complicaciones inflamatorias. Tipo 2: sin anormalidades
musculares, tejido conectivo afectado, colon derecho, sangrado es el sntoma ms comn. Teora de Mann: incremento en la presin
intraluminal secundaria a falta de relajacin de la unin recto-sigmoidea, hipertrofia muscular. Localizacin: Mayo Clinic Sigmoides
29%, sigmoides y otros 68%, otros 3%. CLASIFICACION: de Hinchey modificada: Estadio I; I a Flemn, I b Diverticultis con absceso
periclico o mesentrico. Estadio II; Absceso plvico tabicado, II a Absceso distal capaz de drenarse percutneamente, II b Absceso
complejo asociado con fstula. Estadio III: Peritonitis purulenta generalizada. Estadio IV: Peritonitis fecal. Estadios clnicos:
Diverticulosis: Presencia de divertculos asintomtico, incidentalomas, no requiere tratamiento ni seguimiento. Enfermedad
Diverticular no complicada: Presencia de sintomatologa asociada al hbito intestinal, distensin abdominal, meteorismo, se confunde
con SII. Enfermedad Diverticular complicada: Sangrado, colon derecho, hematoquezia (vinoso), manejo mdico y resolucin en 80%,
probabilidad de resangrado del 20-30%, indicacin quirrgica hasta el segundo evento. Fstulas colo-vesical es la ms frecuente, ms
frecuente en hombres 3:1, complicacin menos frecuente de la ED. Diverticultis no complicada: Pacientes sin afectacin sistema por
datos inflamatorios Hinchey I y II, respuesta al tratamiento mdico hasta 90%, repeticin del cuadro en 25%, indicacin quirrgica
electiva en el 1er cuadro con patologa sistmica asociada. Diverticulitis complicada: Hinchey II y III, patologa que involucra absceso
plvico o peritonitis, afectacin sistmica, tratamiento quirrgico, indicado siempre la programacin electiva si mejora con tratamiento
conservador. DIAGNOSTICO: Colon por enema sensibilidad del 99%, especificidad del 96%. Colonoscopia sensibilidad del 88%,
especificidad del 74%. TAC sensibilidad del 95%, especificidad del 86%. Colonoscopia virtual. TRATAMIENTO: Tratamiento mdico
dependiendo de la severidad, identificar complicaciones, valoracin del estado hidroelecroltico, valoracin del estado acido-base,
valorar tratamiento quirrgico. Medico: Ayuno, soluciones, antibiticos (ciprofloxacino, metronidazol, analgsicos, ketorolaco,
paracetamol, tramadol). Quirrgico: Indicado en falla al tratamiento mdico, sepsis, alteraciones en el estado hemodinmico.
Procedimiento 3 tiempos (three times procedure= Colostoma de transverso y drenaje, reseccin subsecuente, cierre de colostoma).
Procedimiento de Miculicz (exteriorizacin y reseccin con colostoma y fstula mucosa. Restitucin del trnsito en segundo tiempo).
Procedimiento de Hartman (Reseccin de sigmoides. Bolsa de Hartmann o mun rectal. Restitucin del trnsito en segundo tiempo).
Procedimiento en 1 solo tiempo (reseccin del foco sptico y restitucin del trnsito en un solo tiempo quirrgico). Lavado de absceso.
Drenaje percutneo. COMPLICACIONES: Sangrado 4-17%, perforacin 32.4%, absceso 10.9%, Fstula 5-33% (colovesical, colocutnea,
colovaginal), estenosis 13.4%

CASO CLINICO
Mujer de 96 aos que consult por dolor abdominal de 4 das de evolucin localizado en hipogastrio asociado a febrcula y aumento del
permetro abdominal. A la exploracin fsica se palpaba una masa localizada en hipogastrio de consistencia dura y dolorosa. En la
analtica destacaba 19.400 leucocitos (78,5% PMN) y PCR: 28,2. La radiografa de abdomen mostr una imagen area redondeada y la
TAC abdominal informaba de gran burbuja area con paso de contraste desde el colon y con pequeos abscesos en la pared. Se
instaur tratamiento mdico (antibiticos intravenosos y reposo intestinal) luego egreso por mejora a los 15 das.

PREGUNTA
Cul es la indicacin posterior a su egreso con mayor evidencia para reducir la recurrencia del presente evento.

RESPUESTA
a.- La fibra de vegetales y frutas.
b.- Abundante ingesta de agua.
c.- Evitar granos y nueces.
d.- Fibra proveniente de cereales.

CASO CLINICO
Se presenta el caso de un varn de 31 aos, con antecedentes de LAPE por mltiples heridas de arma blanca, acudi al servicio de
urgencias por un cuadro de dolor abdominal sbito en la regin periumbilical y signos de oclusin intestinal. A la exploracin fsica
destacaba un abdomen distendido; en la radiografa de abdomen se observaba la presencia de niveles hidroareos y distensin de las
asas intestinales, sin que se apreciara neumoperitoneo, y en la analtica destacaba leucocitosis de 22,000, con neutrofilia.

PREGUNTA
Cul es la patologa ms probable con los datos clnicos.

RESPUESTA
a.- Oclusin intestinal mecanica
b.- Oclusin intestinal idiopatica
MANUAL DE TRABAJO DEL CURSO ENARM CMN SIGLO XXI
CURSO ENARM CMN SIGLO XXI TEL: 36246001 Pharmed Solutions Institute PGINA 514

c.- Invaginacin intestinal
d.- Enfermedad diverticular complicada

CASO CLINICO
Un hombre de 45 aos, hasta ese momento sano, consulta por dolor intenso en la fosa ilaca izquierda, que comenz 35 horas antes. Ya
antes haba notado malestar leve y peridico en esta regin pero no solicito tratamiento mdico. Tambin presentaba nuseas,
anorexia y vmitos, que los relacionaba con las comidas. En el examen fsico estaba febril (38,5C) y taquicrdico (110/min). Dolor a la
palpacin en la fosa ilaca izquierda, sin signos peritoneales.

PREGUNTA
Las manifestaciones clnicas de la diverticulitis colnica aguda varan con la extensin de la enfermedad. Cual de las siguientes
observaciones es menos frecuente?

a.- Constipacin importante.
b.- Dolor abdominal.
c.- Sensacin de plenitud perirrectal.
d.- La prueba del guayaco en las heces es positiva.

PREGUNTA
Cual de los siguiente diagnosticos diferenciales es el menos frecuente?

RESPUESTA
a.- Apendicits aguda
b.- Enfermedad intestinal inflamatoria
c.- Enfermedad de Crohn
d.- Colitis infecciosa

PREGUNTA
Cual es el mtodo diagnostico mas adecuado para el caso descrito?

RESPUESTA
a.- Colonoscopia.
b.- Transito intestinal.
c.- TAC abdominal.
d.- USG abdominal.

PREGUNTA
Considerando el caso clnico, cual es la conducta a seguir mas adecuada?

RESPUESTA
a.- Ingreso, ayuno, hidratacin y antibiticos.
b.- Preparar al paciente para ciruga laparoscpica.
c.- Preparar al paciente para ciruga abierta.
d.- Tratamiento ambulatorio.

PREGUNTA
Cual es la cantidad de fibra que deber consumir posteriormente el paciente para evitar recaidas?

RESPUESTA
a.- 10 gr/dia.
b.- 20 gr/dia.
c.- 30 gr/dia.
d.- 40 gr/dia.





MANUAL DE TRABAJO DEL CURSO ENARM CMN SIGLO XXI
CURSO ENARM CMN SIGLO XXI TEL: 36246001 Pharmed Solutions Institute PGINA 515

ENFERMEDAD INTESTINAL INFLAMATORIA: La enfermedad intestinal inflamatoria, incluye colitis ulcerosa y enfermedad de Crohn,
ambas se atribuyen a un defecto en la regulacin inmunolgica del tubo digestivo que origina una reaccin inmunolgica
descontrolada a diferentes antgenos. CUCI: La colitis ulcerativa casi siempre se limita al intestino grueso, afecta la mucosa (colitis
mucosa), y es continua desde el recto hacia arriba, y por lo general afecta solo el colon y parte ms distal del leon. Las fistulas son
raras. Una forma ms ligera de colitis ulcerativa puede afectar la porcin distal del intestino grueso, el recto o el recto y el colon
sigmoide. Se conoce como proctitis ulcerativa o proctosigmoiditis ulcerativa. 90% de estos pacientes responden al tratamiento mdico
y nunca experimentan pancolitis. CROHN: Enfermedad de Crohn puede daar cualquier parte del tubo digestivo, desde la boca hasta el
ano y suele comprometer todo el grosor de la pared abdominal, la mayora de los pacientes presenta compromiso del intestino delgado
a menudo con una distribucin discontinua (segmentos saltados). Los estrechamientos son usuales, la enfermedad perianal (fistulas,
abscesos, fisuras), son comunes y tienen un potencial maligno mayor que la CUCI.DIAGNOSTICO: Los cuadros clnicos se sobreponen y
en 15% de los pacientes la colitis es indeterminada. Por lo general los sntomas se deben a la inflamacin intestinal e incluyen diarrea,
hemorragia rectal, tenesmo, dolor abdominal, fiebre y prdida de peso. Es posible que se afecten otros rganos como el sistema
musculoesqueltico (espondilitis, artritis), la piel (eritema nodoso, pioderma gangrenosa), el ojo (iritis), sistema hematopoytico,
riones, vas biliares. TRATAMIENTO: Es similar para ambos; la sulfazalacina, base de tratamiento anterior, ya casi se sustituy por el
cido 5-aminosalicilico (5-ASA), el cual tiene muchos menos efectos colaterales y es mejor tolerado por los pacientes, las etapas agudas
se controlan con esteroides, casi siempre prednisona en dosis 20-80mg/da, con reduccin rpida de dosis. Los pacientes ms graves
deben hospitalizarse e iniciar soluciones intravenosas, reposo intestinal y esteroides intravenosos. La operacin casi siempre colectoma
abdominal, se realiza si no se observa respuesta al tratamiento mdico en unos cuantos das. La funcin de los inmunomoduladores
como la 6-mercaptopurina (6MP) y aziatropina se incrementa conforme su eficacia y seguridad se establecen cada vez ms. Su
aplicacin principal es el tratamiento de mantenimiento y como ahorradores de esteroides en los pacientes con sntomas persistentes
mientras reciben solo preparaciones de 5-ASA. Tratamiento quirrgico: En CUCI, ya que la inflamacin se limita al intestino grueso, la
reseccin del rgano es curativa, a diferencia de la enfermedad de Crohn, que no puede erradicarse por medios quirrgicos y tienen un
alto ndice de recurrencia despus de la reseccin, la ciruga es la proctocolectoma reconstitutiva (se extirpa colon y parte superior de
recto, se quita la mucosa del recto restante y se construye un reservorio ileal (saco), con el leon terminal; luego se une con el
reservorio a nivel de la lnea dentada. En enfermedad de Crohn la ciruga est reservada para complicaciones que no responden a
tratamiento conservador, casi siempre implica reseccin del segmento intestinal, afectado.

CASO CLINICO
Paciente masculino de 33 aos que ingresa con el idx de brucelosis crnica con tratamiento emprico ciprofloxacino durante 4
ocasiones por periodos de 2 semanas con recidiva de la misma sintomatologia (fiebre intermitente sin predominio de horario, dolor
abdominal en region del epigastrio y rara vez con irradiacion a marco colonico, nauseas, vomito al inicio siempre del padecimiento y
hematoquezia sin diarrea. EF: abdomen blando depresible no doloroso en el momento, no se identifica masa megalias. no hay signos
de irritacin peritoneal. Peristaltismo positivo de caracteristicas normales. Se realiza colonoscopia ciego, el cual es de caractersticas
macroscpicas normales, se observa valvula ileocecal, se canula la misma y se exploran 15cms del ileon terminal el cual es normal, el
colon ascendente y transverso conservan su morfologa y patrn vascular, el colon descendente y rectosigmoides en algunas reas
conservan su patron vascular y en otral esta perdido, con imagen tubular. Se toma biopsia de de colon ascendente y transverso, as
como de colon descendente y recto de reas afectadas. Se diagnostica CUCI.

PREGUNTA
Cual de las siguientes manifestaciones son ms caractersticas de CUCI?

RESPUESTA
a.- Estenosis, perdida de la vascularidad, ulceras, seudopolipos y granularidad.
b.- Estenosis, perdida de la vascularidad, ulceras, polipos y granularidad.
c.- Estenosis, perdida de la vascularidad, ulceras, seudopolipos y agranularidad.
d.- Estenosis, aumento de la vascularidad, ulceras, seudopolipos y granularidad

PREGUNTA
En la escala de Baron, en que estado se encuentra el caso clnico?

RESPUESTA
a.- 0.
b.- 1.
c.- 2.
d.- 3.

PREGUNTA
Cual de las siguientes afecciones extra-intestinales es la menos frecuente en el CUCI?.

RESPUESTA
a.- Artritis.
b.- Axial Espodilitis Anquilosante.
c.- Estomatitis.
d.- Glomerunefritis.

MANUAL DE TRABAJO DEL CURSO ENARM CMN SIGLO XXI
CURSO ENARM CMN SIGLO XXI TEL: 36246001 Pharmed Solutions Institute PGINA 516

CANCER DE COLO RECTAL (CCR). CIENCIAS BASICAS: El CCR es un tumor maligno que se origina dentro de las paredes del intestino
grueso, incluye los segmentos; ciego, colon ascendente, colon transverso, colon descendente, sigmoides, recto. El CCR no incluye los
tumores en los tejidos del ano o del intestino delgado. Los adenomas son tumores benignos epiteliales que se consideran lesiones
precancerosas. Los adenomas pueden tener diferentes grados de displasia o diferentes caractersticas histolgicas: tubular, tbulo-
vellosas, asociados con el aumento de potencial malignidad. Factores de riesgo: Incluye factores ambientales y genticos.
Aproximadamente el 75% se presentan en pacientes sin factores de riesgo conocidos. Hay dos sndromes genticos que predisponen al
CCR: poliposis adenomatosa familiar (PAF), con riesgo de 60-90% y cncer colo rectal hereditario no asociado a poliposis (CCHNP),
considerar en familiares de primer grado de pacientes con diagnstico de CCR, pero representan menos del 10% de todos los CCR.
Otros factores son CUCI, obesidad, sedentarismo, exceso de alcohol, tabaquismo dieta alta en grasas y/o baja en frutas y vegetales o
fibra. SALUD PUBLICA: El CCR es el cncer ms comn del tracto gastrointestinal, la tercera causa ms comn de cncer y la cuarta
causa de muerte por cncer en el mundo. El CCR se presenta con mayor frecuencia a partir de los 50 aos de edad. Ms de 95% de CCR
son adenocarcinomas. SINDROME DE LYNCH: Sndrome de carcter hereditario, autosmico dominante y se caracteriza por el
desarrollo precoz (antes de 50aos de edad) de CCR, de predominio en colon derecho, y una elevada tendencia a presentar lesiones
sincrnicas o metacrnicas, as como neoplasias en otros rganos (endometrio, estomago, pncreas, sistema urinario, ovario, vas
biliares). El diagnostico se establece a partir de la historia familiar segn los criterios de Amsterdam. Esta entidad se caracteriza por la
existencia de mltiples mutaciones somticas en fragmentos repetitivos de ADN. DIAGNOSTICO: Clnica; dependen de la localizacin y
extensin del tumor, no suele dar sntomas hasta etapas avanzadas. Las neoplasias en el hemicolon derecho producen hemorragia
oculta que el paciente no detecta y es probable que ocasionen sntomas de anemia ferropriva crnica. Este hallazgo en un varn adulto
o mujer postmenopusica, es indicacin para colonoscopia. Los tumores en el hemicolon izquierdo pueden manifestarse con
hemorragia visible (rectorragia), cambio en los hbitos intestinales y dolor abdominal clico secundario a obstruccin parcial. Tambin
es probable que las primeras manifestaciones del cncer del intestino grueso sean, metstasis heptica, ascitis y metstasis
pulmonares. No es infrecuente que se detecte una masa en la exploracin en el cuadrante inferior derecho del abdomen. Por ultimo
una complicacin poco frecuente del cncer de colon pero que empeora el pronstico es la perforacin intestinal. El cncer de recto
puede comportarse como un sndrome anorrectal, con urgencia rectal, tenesmo y diarrea con moco y sangre. Cunado su extensin
rebasa la pared rectal, el paciente puede aquejar sntomas urinarios atribuibles a invasin vesical como hematuria y polaquiuria. Si se
ha establecido una fistula rectovesical, hay neumaturia e infecciones urinarias recidivantes. Adems de los sntomas locales, CCR puede
causar sntomas generales, como astenia, anorexia o prdida de peso. Puede aparecer clnica secundaria a la presencia de metstasis a
distancia, como ictericia, trastornos neurolgicos, dolores seos, disnea, ascitis. Ante la sospecha de CCR es imperativo practicar una
colonoscopia con la que adems de observar el tumor, se podrn tomar biopsias para su diagnstico histolgico y descartar la
presencia de lesiones sincrnicas. En caso de tumores estenosantes es recomendable completar la exploracin con una colonografa
por TAC o RNM. Por elevado porcentaje de presentaciones diseminadas, las exploraciones complementarias incluyen radiografas de
torax, USG o TAC abdominal, gammagrafa seas, cistoscopia, ultrasonografa endoscpica y RM plvica. La determinacin de antgeno
carcinoembrionario, tienen una reducida sensibilidad y especificidad para CCR. Sin embargo, posee valor pronstico y su monitorizacin
es til para seguimiento del paciente tras ciruga. TRATAMIENTO: La ciruga es usualmente el primer tratamiento o el tratamiento
primario para los cnceres de colon que no se han propagado a partes distantes. Tambin se puede usar la terapia adyuvante
(adicional). En la mayora de los casos, la terapia adyuvante se administra por aproximadamente 6 meses. Etapa 0: Puesto que estos
cnceres no han crecido ms all del revestimiento interno del colon, todo lo que se necesita es ciruga para extirpar el cncer. Esto se
puede lograr en la mayora de los casos con una polipectoma (extirpacin del plipo) o escisin local a travs de un colonoscopio. Si el
tumor es demasiado grande para poder extirparlo mediante escisin local, es posible que ocasionalmente sea necesario realizar una
reseccin del colon (colectoma). Etapa I: Ha crecido a travs de varias capas del colon, pero no se han propagado fuera de la pared del
colon en s, ni ganglios. La colectoma parcial (ciruga para extirpar la seccin del colon que contiene cncer y los ganglios linfticos
cercanos) es el tratamiento convencional. Usted no necesita terapia adicional. Etapa II: Ha crecido a travs de la pared del colon y se
puede extender hacia el tejido vecino, no a ganglios linfticos. Puede que la ciruga (colectoma) sea el nico tratamiento que se
necesita. Se puede recomendar quimioterapia adyuvante si hay riesgos de recurrencia como: es de alto grado, ha crecido hacia los
rganos cercanos, el cirujano no extirp al menos 12 ganglios linfticos, se encontr cncer en o cerca del margen (borde) de la
muestra quirrgica, lo que significa que algo de cncer pudo haber quedado, ha bloqueado (obstruido) el colon, caus una perforacin
(orificio) en la pared del colon. Las principales opciones de quimioterapia para esta etapa incluyen 5-FU y leucovorn (solo) o
capecitabina, aunque tambin se pueden usar otras combinaciones. Se puede administrar radioterapia al rea de su abdomen donde
est creciendo el cncer. Etapa III: Se ha propagado a los ganglios linfticos cercanos, pero todava no se ha extendido a otras partes del
cuerpo. Ciruga (colectoma parcial) seguida de quimioterapia adyuvante. El rgimen FOLFOX (5-FU, leucovorn, y oxaliplatino) o el
rgimen CapeOx (capecitabina y oxaliplatino) se usan con ms frecuencia, aunque algunos pacientes pueden recibir 5-FU con
leucovorn o capecitabina sola segn la edad y sus necesidades de salud. La radioterapia y/o la quimioterapia pueden ser opciones para
las personas que no estn lo suficientemente saludables como para someterse a ciruga. Etapa IV: Se propag desde el colon hasta
rganos y tejidos distantes. Con ms frecuencia, el cncer de colon se propaga al hgado, aunque tambin se puede propagar a otros
lugares, tal como a los pulmones, el peritoneo (la membrana que recubre la cavidad abdominal) o a los ganglios linfticos distantes. Es
poco probable que la ciruga cure estos cnceres. Si las metstasis no se pueden extirpar quirrgicamente debido a que son muy
grandes o hay demasiadas, se puede tratar primero quimioterapia para reducir el tamao de los tumores con el fin de permitir la
ciruga. Entonces, se puede administrar nuevamente quimioterapia despus de la ciruga. Otra opcin sera destruir los tumores en el
hgado con criociruga, ablacin por radiofrecuencia, u otros mtodos no quirrgicos. Si el cncer se ha propagado demasiado como
para tratar de curarlo con ciruga, puede que en algunos casos an sea necesario realizar operaciones, tales como una colectoma o una
colostoma de derivacin (cortar el colon sobre el nivel del cncer y se adhiere el extremo a una abertura en la piel del abdomen para
permitir que los desechos salgan del cuerpo). Esto puede aliviar o prevenir el bloqueo del colon, lo que puede evitar ciertos problemas.
Algunas veces, se puede evitar tal ciruga al insertar un stent en el colon durante la colonoscopia con el fin de mantenerlo abierto. La
mayora de los pacientes con cncer en etapa IV recibir quimioterapia y/o terapias dirigidas para controlar el cncer. Los regmenes
ms comnmente usados incluyen: FOLFOX: leucovorn, 5-FU, y oxaliplatino (Eloxatin). FOLFIRI: leucovorn, 5-FU, e irinotecn
(Camptosar). CapeOX: capecitabina (Xeloda) y oxaliplatino. Cualquiera de las combinaciones anteriores ms bevacizumab (Avastin) o
MANUAL DE TRABAJO DEL CURSO ENARM CMN SIGLO XXI
CURSO ENARM CMN SIGLO XXI TEL: 36246001 Pharmed Solutions Institute PGINA 517

cetuximab (Eribitux) (pero no ambos).PRONOSTICO: Se relaciona con la etapa del tumor. Se prefiere el sistema de Dukes por su
sencillez: Dukes A: tumor limitado a submucosa. Dukes B1: tumor que se extiende a la capa muscular. Dukes B2: tumor que se extiende
ms all de la serosa intestinal. Dukes C1: compromiso de ganglios linfticos; el tumor no se extiende fuera de la pared intestinal. Dukes
C2; ganglios linfticos positivos; el tumor rebasa pared intestinal. Dukes D: metstasis a distancia. La sobrevida no corregida a los 5 aos
para etapas A= 85%, B= 65%, C = 45%, los valores corregidos son 100, 78 y 54% respectivamente

CASO CLINICO
Se trata de paciente masculino de 71 aos de edad, refiere que desde hace 6 meses presenta reduccin de peso, astenia, adinamia, a la
exploracin fsica se palpa masa no dolorosa en fosa iliaca derecha, con presencia de hematoquesia, se reporto biometra hemtica con
hematocrito 31 %, y hemoglobina de 10, el frotis mostro eritrociticos microcitico normocromicos, resto normal.

PREGUNTA
Cul es el factor de riesgo de mayor importancia para el desarrollo de cncer de clon

RESPUESTA
a.- Consumo de alcohol.
b.- Dieta baja en fibra y alta en carnes.
c.- Tabaquismo positivo.
d.- Colitis o Diverticulitis previa.

CASO CLINICO
Acude al servicio de urgencias un paciente masculino de 69 aos de edad el cual refiere malestar generalizado y fiebre desde hace 3
semanas de evolucin, el paciente es originario de la ciudad de mexico, administrador retirado hace 4 aos, tiene el antecedente de
tabaquismo desde la edad de 20 aos a la fecha en promedio 20 cigarrillos al da, adems hipertensin arterial sistmica controlada con
IECAS mas diurtico tiazidicos, a la exploracin fsica se observa con palidez moderada, asi como ndice de masa corporal de 21, la
regin cardiaca se ausculta murmullo sistlico con predominio mitral, los estudios de laboratorio se detecta hemoglobina de 11 g/dl,
34% de hematocrito, leucocitos y eritrocitos en orina, se realizaron cultivos siendo positivo nicamente el hemocultivo para
streptococcus bovis, debido a los resultado se enva a ecocardiograma, sin tener actualmente los resultados, pero se sospecha
vegetaciones.

PREGUNTA
Cual es la conducta a seguir ms adecuada en el paciente para identificar la causa inicial.

RESPUESTA
a.- Realizar una biopsia renal.
b.- Esperar resultado de Ecocardiograma.
c.- Valoracion toxicolgica.
d.- Realizar colonoscopia.



























MANUAL DE TRABAJO DEL CURSO ENARM CMN SIGLO XXI
CURSO ENARM CMN SIGLO XXI TEL: 36246001 Pharmed Solutions Institute PGINA 518

FISURA ANORECTOPERINEAL. CIENCIAS BASICAS: Comunicacin anmala entre dos epitelios diferentes. La fstula es el conducto de
paredes fibrosas infectadas, que comunica una cripta anal con la piel o con la luz del recto; el orificio localizado en la cripta se denomina
primario o interno y el cutneo mucoso o secundario. El orificio primario es donde habitualmente se origina el proceso. Etiologa:
Origen criptoglandular en 95%. Secuela de un absceso. Debut en procesos menores. Trauma. Infecciosas (TB, gonorrea). Enfermedades
de intestine irritable (Crohn, CUCI). Radiacin. SALUD PUBLICA: Es una afeccin muy frecuente en la prctica colproctolgica ms
comn en el sexo masculino y en edades que fluctan entre 30 a 50 aos CLASIFICACION: A)
Interesfinteriana, B) Transesfintrica, C) Supraesfintrica, D) Extraesfintrica. DIAGNOSTICO:
Clnico; Orificio secundario, dolor, secrecin, sangrado, absceso. Realizar un adecuado
interrogatorio del cuadro clnico y los antecedentes. secrecin purulenta persistente y escasa , que
procede de una elevacin cutnea del tejido inflamatorio ( orificio secundario), a veces dolorosa al
tacto, que puede ser nica o mltiple, situado en cualquiera de los dos cuadrantes perianales , con
mayor frecuencia en los posteriores a distancias variables del ano , algunas tan lejanas como 12 a
13 cm y an ms . Aproximadamente en la mitad de los casos se encuentra un aumento en el
volumen e hiperpigmentacin de la piel en la regin que rodea el orificio fistuloso, que traducen la
presencia de un absceso que no fue totalmente vaciado. En conjunto el absceso y la fstula siguen una de las siguientes evoluciones
Exploracin Fsica: Orificio Secundario, tacto rectal. Gabinete: La fistulografa con el propsito de diagnstico y conocer el trayecto de la
fstula si el material de contraste logra pasar con facilidad los trayectos fistulosos, lo podr hacer el estilete durante la operacin, si por
el contrario es introducido a presin puede romper la pared de la fstula y causar error diagnstico. El ultrasonido endoanal de 360es
til para anatoma de fstulas complejas. CRITERIOS DIAGNSTICOS: Historia de absceso perianal. Drenaje anal o perianal de secrecin
purulenta o intermitente. Presencia de orificio fistuloso secundario con trayecto fistuloso dirigido hacia la cripta de origen.
TRATAMIENTO: Tratar el absceso y la fstula en la misma operacin. Fistulectoma o fistulotoma. La fistulectomia reseca la totalidad
del trayecto fistuloso por lo que es ms radical y disminuye la recidiva mayor riesgo de incontinencia; la fistulotoma solo una porcin
del trayecto. La incontinencia esfinteriana por fistulectoma es posible en fstulas muy altas que requieren divisin del anillo rectoanal.
COMPLICACIONES: Fase aguda (abscesos) sepsis graves y muerte, destruccin del esfnter e incontinencia anal (5-10%), deformidad
anal (Keyhole) (3-5%), recidiva (3-5%), cambios neoplsicos malignos en fstulas de larga evolucin, complicacin post- quirrgica:
incontinencia anal en cirujanos de poca experiencia.

CASO CLINICO
Se trata de paciente femenino de 31 aos de edad la cual acude a revisin posterior a parto hace 45 das, al interrogatorio la paciente
refiere que presenta dolor al evacuar, con salida de sangre fresca leve, niega sensacin de pensantes, ni descarga rectal espontnea,
niega ardor o prurito, a la exploracin no se observan hemorroides externas.

PREGUNTA
Cul es la patologa ms probable en esta paciente.

RESPUESTA
a.- Lo ms frecuente es la fisura anal.
b.- Desgarro secundario a parto.
c.- Las hemorroides internas por mltiples embarazos.
d.- Un absceso anal por infeccin de episiotoma.

























MANUAL DE TRABAJO DEL CURSO ENARM CMN SIGLO XXI
CURSO ENARM CMN SIGLO XXI TEL: 36246001 Pharmed Solutions Institute PGINA 519

APENDICITIS. CIENCIAS BASICAS: Es la inflamacin del apndice vermiforme, que inicia con obstruccin de la luz apendicular; es un
padecimiento grave, con importantes complicaciones que pueden llevar a la muerte, en particular cuando se retrasan el diagnstico y la
teraputica oportuna. La apendicitis es la patologa quirrgica actual ms frecuente. SALUD PUBLICA: Edad promedio 31.3 aos,
mediana 22 aos, error en diagnostico 15.3 %. El 40 % 10-29 aos. 260 000 apendicectomas. PATOGENIA: Etiologa: Obstruccin de la
luz, **Hiperplasia de tejido linfoide. (55%), fecalito 40 %, parsitos intestinales, tumores, semillas, bacterias aerobias (E.coli, P.
aeruginosa, Klebsiella, Streptococcus), anerobios (Bacteroides fragilis, fusobacterium, peptostreptococcus). Al obstruirse la luz y se
forma un asa cerrada en la porcin distal del apndice que se distiende rpidamente al aumentar la secrecin mucosa. Se eleva la
presin hasta 50 o 60 cmH2O y la distensin estimula las terminaciones nerviosas, con aparicin de dolor sordo y difuso; esto
incrementa la peristlsis y aparece el clico caracterstico. El ensanchamiento de la luz apendicular sigue en aumento por la
multiplicacin bacteriana, que produce gas, pus, o necrosis. Se excede la presin de las venas y stas se colapsan e impiden el retorno
de la sangre; en tanto, la circulacin contina por las arterias y provoca congestin hemtica, edema y estasis. La distensin progresiva
del apndice emite reflejos por va nerviosa y ocasiona nusea, vmito y aumento del dolor. El proceso inflamatorio involucra pronto a
la serosa apendicular y al peritoneo parietal, con lo cual el dolor se traslada a la fosa iliaca derecha; la mucosa apendicular es invadida
por bacterias y stas se extienden a capas ms profundas; todo ello desencadena necrosis y absorcin de sustancias provenientes de
tejidos muertos y de toxinas bacterianas, lo que produce fiebre, taquicardia y leucocitosis. Al continuar la distensin apendicular se
ocluye la circulacin de arteriolas, se agrava la necrosis, las paredes no resisten y finalmente el apndice se perfora. El organismo
reacciona rodeando la zona con asas de intestino y epipln, para limitar la diseminacin a un rea restringida, pero esto origina absceso
apendicular; entonces se presenta el leo paraltico para focalizar el proceso. Sin embargo, otras veces el proceso infeccioso no se
localiza y con la rotura apendicular se diseminan diversos lquidos en la cavidad abdominal, ocasionando peritonitis generalizada.
CLASIFICACION: A) Apendicitis aguda: 1. Apendicitis aguda no perforada (Fase hipermica, edematosa, necrtica o gangrenada). 2.
Apendicitis aguda perforada (Fase purulenta, de absceso, con peritonitis local, con peritonitis generalizada). 3. Apendicitis aguda
reactiva. 4. Apendicitis aguda complicada. B) Apendicitis crnica. DIAGNOSTICO: Clnica: El sntoma ms comn es el dolor (tipo,
localizacin, migracin, variaciones anatmicas), la anorexia, que acompaa a la apendicitis en 50 a 90 % de las veces; tambin son
frecuentes nuseas, vmito, diarrea (10 % de los casos), estreimiento, fiebre. Casi todos los pacientes sufren al principio anorexia y
despus dolor abdominal y vmito. Los hallazgos de la exploracin fsica son decisivos, especialmente los datos abdominales. En el
cuadrante inferior derecho se presentan hiperestesia e hiperbaralgesia, dolor intenso bien definido en el punto de McBurney, rigidez
muscular y dolor a la descompresin. Otros datos menos frecuentes son la positividad a las maniobras del psoas (dolor del cuadrante
inferior derecho a la extensin del muslo en decbito lateral izquierdo) y de Rovsing (la palpacin del cuadrante inferior izquierdo suele
producir dolor en el cuadrante inferior derecho). En el apndice ubicado en la pelvis los signos abdominales son escasos; es ms
probable detectar una masa palpable al tacto rectal o vaginal. La apendicitis en el paciente peditrico es ms grave por la alta incidencia
de perforacin y por la peritonitis que se desarrolla al demorar el diagnstico; adems, la enfermedad progresa con mayor rapidez. La
apendicitis del anciano tambin es grave, estos pacientes manifiestan pocos sntomas ya que el dolor es menos intenso y los leucocitos
casi no se incrementan; adems, las enfermedades intercurrentes deterioran al paciente, riesgo de perforacin 49 %, mortalidad 21 %.
La apendicitis es comn durante el embarazo (uno de cada 2 200). Se relaciona con dolor en posicin ms ceflica que la normal, sin
dejar de ser compatible con la migracin del ciego, a partir del cuadrante inferior derecho a la posicin subcostal, conforme evoluciona
el embarazo. La apendicitis perforada durante la gestacin se acompaa de mayor riesgo para la madre y feto a causa de
complicaciones spticas. Laboratorio: El dato ms constante es la leucocitosis (entre 10 000 y 16 000 mm
3
), acompaada de un
predominio porcentual de los neutrfilos y presencia de formas en banda; sta es una reaccin inespecfica del proceso infeccioso y no
es exclusiva de la apendicitis. Cifras mayores a 18 000 se observan en el absceso apendicular, la peritonitis generalizada o en el absceso
heptico. El EGO es normal excepto en el apndice retrocecal, que involucra al urter o a la vejiga; esto puede hacer pensar al clnico en
una infeccin de vas urinarias. Las placas simples de abdomen en posicin de pie y en decbito, muestran una o dos asas distendidas
por gas, ubicadas en el cuadrante inferior derecho del abdomen; puede apreciarse el fecalito y tambin el borramiento de la sombra
del msculo psoas derecho, la ausencia de gas en colon y una zona de opacidad en el cuadrante inferior derecho, imagen en vidrio
despulido. Ultrasonido: Sensibilidad 85 90 %, Especificidad 92 96 %, til en embrazo y pacientes ginecolgicas. TAC: Sensibilidad 92
97 %. Especificidad 90-98 %, observamos masas, abscesos, signo de punta de flecha. TRATAMIENTO: Prequirrgico; El periodo de
preparacin debe ser menor de cuatro horas. Ayuno, reparar deficiencia de lquidos y electrolitos y trastornos cido-base. Iniciar
antibiticos: 1. Ampicilina, 2. Aminoglucsido, 3. Metronidazol o clindamicina, 4. Ampicilina-sulbactam o cefoxitina. Analgsico no
opiceo. Antipirticos. Catter central, sonda vesical, SNG (paciente critico). La ciruga laparoscpica es til tanto para la exploracin
diagnstica como para el tratamiento; sigue los mismos principios de la tcnica quirrgica tradicional y su empleo es cada vez ms
frecuente. COMPLICACIONES: Perforacin, peritonitis focal, peritonitis difusa, pileflebitis. En primer da posquirrgico: Hemorragia.
Evisceracin por mala tcnica. leo adinmico. Tarda infertilidad (33%).

CASO CLINICO
Masculino de 21 aos de edad. Consult en la madrugada, tras ms o menos 16 h de evolucin de un dolor abdominal vago que habra
comenzado tras el desayuno, ubicado en la regin periumbilical, era de baja intensidad, se mantuvo varias horas para luego localizarse
en fosa ilaca derecha. No hubo compromiso del estado general, pudo ingerir comida liviana, no tuvo vmitos pero s algo de nuseas.
Por la tarde consult mdico quien sospech una apendicitis aguda. Con diagnstico de apendicitis aguda se hospitaliz y al ingreso se
observo dolor espontneo y a la palpacin en fosa ilaca derecha con signo de Blumberg. Se coloco en observacin y se realizaron
laboratorio pero el paciente refera ya no tener dolor abdominal y al ser examinado no se presento dolor ni resistencia abdominal.

PREGUNTA
Debido a la presentacin y evolucin del cuadro clnico cual es la conducta a seguir.

RESPUESTA
a.- Explorar posible patologa urinaria.
b.- Egresarlo y evaluar en 24 hrs.
MANUAL DE TRABAJO DEL CURSO ENARM CMN SIGLO XXI
CURSO ENARM CMN SIGLO XXI TEL: 36246001 Pharmed Solutions Institute PGINA 520

c.- Realizar laparatomia exploratoria.
d.- Realizar apendisectomia laparoscpica.

CASO CLINICO
Masculino de 17 aos de edad, que inici con sntomas de dolor abdominal difuso, de gran intensidad que l atribua a una trasgresin
alimentara. Se mantuvo en reposo y dieta, no refera vmitos ni diarrea; 24 h ms tarde el dolor an persista pero de menor
intensidad y se localizaba en abdomen bajo. La descripcin y conclusin fueron compatibles con apendicitis aguda, demostrando
dimetros apendiculares de 11 y 8 mm, sensible, no deformable con aumento de ecogenicidad de la grasa adyacente. Al ingreso el
paciente refera poco dolor espontneo y a la palpacin, con un Blumberg positivo. La temperatura axilar era de 37,2C y el recuento de
leucocitos de 6.700.

PREGUNTA
Cul es sntoma ms relevante en el diagnostico de apendicitis.

RESPUESTA
a.- Dolor tpico.
b.- Resistencia muscular.
c.- Leucocitosis con predominio neutrofilos.
d.- Nausea y vomito.

CASO CLINICO
Mujer de 17 aos que acude a Urgencias por un cuadro de abdomen agudo. Como antecedentes, destacaba un ingreso un ao antes
para estudio por un sndrome febril y cefalea. Fue diagnosticada de una infeccin por citomegalovirus, desde entonces permaneciendo
asintomtica y sin tratamiento. La paciente consult por dolor abdominal de 6 horas de evolucin, iniciado en epigastrio y posterior
migracin a fosa ilaca derecha, intenso, de caractersticas continuas, con vmitos, sin fiebre ni otros sntomas. A la exploracin
presentaba febrcula, defensa y signos de irritacin peritoneal en hemiabdomen derecho. Analtica: leucocitos 16.330/l, neutrfilos
76,7%, hematocrito 39%, ALT 59 UI/l, GGT 19 UI/l, amilasa 354 UI/l, PCR 0,7mg/dl. Test de embarazo: negativo. Radiologa simple:
anodina. No signos de oclusin. Ecografa abdominal: vescula sin alteraciones. Va biliar de calibre normal. Pequea cantidad de lquido
libre en pelvis.

PREGUNTA
Cul es el mtodo diagnstico ms certero para este padecimiento.

RESPUESTA
a.- Ultrasonido.
b.- Tomografa.
c.- Exploracin fsica.
d.- Rx de pie y decbito de abdomen.

POLIPOS. CIENCIAS BASICAS: Son protuberancias en la mucosa intestinal, protruyen de la pared hacia la luz intestinal. El patrn
proliferativo alterado de la mucosa del colon que hace que se convierta en plipo y, posteriormente en carcinoma, deriva de la
activacin de un oncogn por mutacin de la perdida de genes que suprimen la gnesis tumoral. Existen 3 tipos de histolgicos:
hamartomas (25%), plipos hiperplasicos y adenomas. Estos ltimos son neoplasias verdaderas con la mayor importancia clnica.
HAMARTOMAS: Es un crecimiento con proliferacin excesiva de un tipo de tejido sin cambios neoplsicos verdaderos. SINDROME DE
PEUTZ-JEGHERS: Sndrome autosmico dominante que se manifiesta por pigmentacin de zonas mucocutaneas y plipos
hamartomosos en el intestino delgado y grueso compuestos de cantidades excesivas de muscular de la mucosa. Hay descripcin de
progresin de los plipos a neoplasia maligna, pero no es frecuente. Se observa mayor incidencia de tumores malignos en otros
rganos, como la mama y el ovario. Los sntomas incluyen hemorragia y obstruccin intestinal secundaria a invaginacin. POLIPOS
JUVENILES: Por lo general ocurren en nios, pero tambin se observan en adultos. Son hamartomas compuestos de glndulas dilatadas
y lmina propia anormal. Es posible que ocurra hemorragia secundaria a la autoamputacin o invaginacin. POLIPOSIS FAMILIAR
JUVENIL: Sndrome autosmico domnate con mltiples plipos juveniles en todo el colon. Pueden manifestarse por hemorragia u
obstruccin y los pacientes tienen mayor riesgo de presentar tumores malignos en el tubo digestivo. El tratamiento es la colectoma
total. HIPERPLASICOS: Son muy frecuentes y casi siempre miden menos de 5mm; el examen histolgico, muestra ausencia de
maduracin e hiperplasia sin displasia nuclear. No se consideran premalignos. ADENOMATOSOS: Son los ms importantes a causa de
su frecuencia y potencial de malignidad. Algunos se convierten en cncer, proceso que tarda entre 5-15 aos. Tan solo el 5% riesgo de
malignizar. Desde el punto de vista morfolgico estos plipos se describen como pediculados (con un tallo) o ssiles (planos). El
examen histolgico, los clasifica como tubulares (87%), vellosos (5%) o tubulovellosos (8%) mixtos conforme el patrn dominante.
Mientras mayor sea el plipo y mayor componente velloso posea, mas potencial maligno tiene. Casi todos los plipos benignos son
asintomticos. En ocasiones los grandes plipos pediculados, se manifiestan con hemorragia o invaginacin. Como regla general, los
plipos adenomatosos deben extirparse por su potencial maligno. La mayor parte puede extirparse con una trampa por colonoscopia,
casi siempre en una pieza. Los plipos grandes o ssiles, pueden requerir polipectoma por partes en una o ms sesiones. En algunos
casos los plipos grandes o ssiles de pacientes con riesgo quirrgico bajo y esperanza de vida larga no pueden extirparse por completo
o en forma segura con una trampa colonoscpica y es necesaria la colectoma. Los plipos malignos son aquellos en los que los cambios
neoplsicos son profundos, hasta la muscular de la mucosa. En estos casos es probable que la polipectoma aun sea un tratamiento
suficiente si se cubren los siguientes criterios: 1. El plipo es pediculado. 2. El tallo no est afectado y los mrgenes de reseccin estn
libres. No hay invasin vascular, linftica o neural y la lesin est bien diferenciada. DIAGNOSTICO: La colonoscopia, es la tcnica de
MANUAL DE TRABAJO DEL CURSO ENARM CMN SIGLO XXI
CURSO ENARM CMN SIGLO XXI TEL: 36246001 Pharmed Solutions Institute PGINA 521

eleccin para el diagnstico de plipos colorectales, es el patrn de oro, adems permite la exerecis o la realizacin de una biopsia, en
los que la exceresis no es posible. Tan importante es la colonoscopia como el estudio histolgico del plipo que permite clasificarlo y
decidir el tratamiento y seguimiento que se debe recomendar. TRATAMIENTO: Todos los plipos de colon deben tratarse mediante
polipectoma endoscpica completa, ya que su aspecto macroscpico los hace indistinguibles, deben ser recuperados y proceder a
anlisis histolgico. La polipectoma endoscpica de los adenomas colorectales se ha demostrado que disminuye la incidencia de
cncer colorectal y es el tratamiento definitivo cuando se efecta una reseccin completa. La perforacin y el sangrado son as dos
complicaciones ms frecuentes descritas.

CASO CLINICO
Paciente femenino de 18 aos de edad, con antecedente de hiperpigmentacin de retina de ojo derecho y dermatitis atpica. Acude
por sangrado de tubo digestivo bajo, comenzando 3 meses antes de la consulta, el sangrado es rojo, escaso que acompaa de
deposiciones normales diarias, sin dolor defecatorio, ni pujo o tenesmo, agrega que antes del sangrado, presentaba dolor abdominal en
zona epigstrica de forma espordica, que calma espontneamente, finalmente refiere dolor articular desde hace 4 meses, sin fiebre ni
causa aparente. El EF es normal, se observa tumoracin en zona retroauricular derecha, redondeada de 1 cm de dimetro de
consistencia firme, superficie lisa e indolora, adems de tumoraciones en cara interna de ambos pies, sin dolor, redondeada de .5 cm
de dimetro consistente y firme, lisa e indoloro, inspeccin perineal y anorectal normal.

PREGUNTA
Cual es la conducta a seguir mas adecuada?

RESPUESTA
a.- Transito gastro intestinal.
b.- Panendoscopia.
c.- Polipectomia endoscpica.
d.- Biopsia de plipo.

COLITIS. CIENCIAS BASICAS: Sndrome gastrointestinal que incluye dolor o molestias abdominales asociado al hbito intestinal en un
periodo de 3 meses. SALUD PUBLICA: 10-20% poblacin general. 20-35% en poblacin econmicamente activa. PATOGENIA:
Multifactorial; Sobrecrecimiento bacteriano en intestino delgado, variacin en niveles de serotonina en la mucosa intestinal,
incremento en la Colgena Tipo 5. DIAGNOSTICO: Clnico; Dolor abdominal (postprandial, preevacuatorio, asociado al estado
emocional), distencin abdominal (postprandial, asociado al estado emocional, evolutivo durante el da). Cambios en el hbito
intestinal; calibre, frecuencia, morfologa, consistencia. Sntomas extraintestinales; cefalea, lumbalgia, fatiga, mialgias, urgencias
urinaria, psiquitricos. Sntomas de alarma; >50 aos, prdida de peso, AHF de Cncer, fiebre, sangrado, lesiones drmicas. CRITERIOS
DE ROMA III: Dolor o molestia abdominal al menos 3 das al mes en los ltimos 3 meses asociado a los siguientes sntomas. Mejora con
la defecacin. Inicio asociado a cambio en la frecuencia y forma (habito intestinal). Criterios revisados en los ltimos 3 meses e inicio
de la sintomatologa en los ltimos 6 meses. TRATAMIENTO: Tratamiento mdico: Dieta, fibra, actividad fsica (ejercicio en grupo).
Diarrea; loperamida, rifaximina, colestiramina, amitriptilina, fibra. Estreimiento; cinitaprida, mebeverina, tegaserod, dimeticona,
simeticona, fibra. Dolor abdominal; antidepresivos tricclicos: imipramina, amitriptilina. Probiticos. COLITIS PESEUDOMEMBRANOSA:
Este padecimiento se observa en pacientes que reciben antibiticos de amplio-espectro (clindamicina, penicilinas, semisintticas,
cefalosporinas).Factores de riesgo: Diabetes Mellitus, VIH, oncolgicos, trasplantados, postquirrgico inmediato. Salud pblica: 7% de
adultos son portadores asintomticos. 13-21% adultos asintomticos hospitalizados. Hasta 40% de los neonatos <6 meses. Ms comn
en mujeres. Mortalidad de 2-4%. Pacientes >70 aos con +5 das con AB riesgo de 34%. PATOGENIA: Agente causal ms frecuente en
diarrea nosocomial (C. difficile) asociada a antibiticos. La alteracin de la flora colnica normal permite el crecimiento excesivo de C.
difficile, un anaerobio que produce una exotoxina, con efecto nocivo sobre la mucosa del colon. El uso de antibiticos, induce
esporulacin y crecimiento bacteriano logartmico, la toxina A y B producen dao citolgico y lesin epitelial, generando
paseudomembranas. Este sndrome puede presentarse hasta 6 semanas despus del tratamiento antibitico. DIAGNOSTICO: El
espectro clnico vara desde diarrea 93-98% (acuosa, moco, sangre) ligera autolimitada, fiebre 65-70%, dolor abdominal 85-90% (clico,
abdomen agudo) hasta inflamacin transmural intensa, colon toxico y perforacin. La leucocitosis no guarda proporcin con los datos
clnicos. El diagnostico se establece mediante el anlisis fecal en busca de la toxina y por endoscopia, que demuestra las
paseudomemebranas amarillentas tpicas, ms en recto y sigmoides. TAC: Engrosamiento de la pared colnica, pliegues internos a la
luz, signo de T-bone Stea. CLASIFICACIN: Tipo 1 (summit lesion); Alteracin focal del epitelio superficial interglandular, infiltracin
de clulas de reaccin en la lmina propia, fibrina subepitelial. Tipo 2 (volcano lesion); Prdida del patrn glandular, abundante
infiltracin de la lmina propia, aparicin de psuedomembranas superficiales. TRATAMIENTO: Ayuno. Suspender Antibiticos.
Correccin de balance hdrico y electroltico. Metronidazol (500mg c/6hrs por 10-14 das) por va oral o IV (750mg c/8hrs por 10-14
das), primera eleccin. La vancomicina (125mg c/6hrs por 10-14 dias) es una alternativa, pero debe evitarse siempre que sea factible a
causa del riesgo del surgimiento de enterococos resistentes a este antibitico. Indicadas cuando no se tenga disponible, alergia o
sensibilidad al Metronidazol o Vancomicina: Bacitracina, teicoplanina, colestiramina, colestipol. Los pacientes con colon toxico o
perforacin se someten a reseccin. El ndice de recurrencia despus del final del tratamiento se acerca al 20%.

CASO CLINICO
Mujer de 66 aos de edad, alrgica a la penicilina. Ingresa para ciruga electiva de divertculos colnicos. Se le practic sigmoidectoma
asistida por laparoscopia. Como profilaxis antibitica se utiliz clindamicina y gentamicina. Durante el postoperatorio presenta un
cuadro de abdominalgia, febrcula y diarrea. En los anlisis aparece leucocitosis y desviacin a la izquierda. Se practica tomografa
computarizada, con resultado normal. Se decide reoperar ante la progresin del cuadro, y no se halla dehiscencia de sutura ni isquemia.
La evolucin es trpida, y la paciente fallece. Los cultivos de heces fueron positivos para la toxina. La anatoma patolgica mostr los
hallazgos tpicos de colitis seudomembranosa.
MANUAL DE TRABAJO DEL CURSO ENARM CMN SIGLO XXI
CURSO ENARM CMN SIGLO XXI TEL: 36246001 Pharmed Solutions Institute PGINA 522


PREGUNTA
El agente etiolgico de la colitis pseudomembranoso es:

RESPUESTA
a.- C. difficile.
b.- E. coli.
c.- Enteroco fecalis.
d.- Enterobacter.

CASO CLINICO
Varn de 64 aos ingres por diarrea infecciosa. Destacan tambin como antecedentes varias infecciones respiratorias en los 2 meses
previos, por los que haba recibido tratamiento con cefepime, amikacina y amoxicilina-clavulnico, y portador de una sonda de
nutricin enteral. Al ingreso presentaba 10-15 deposiciones diarreicas/da, acuosas, acompaadas de dolor abdominal de tipo clico y
febrcula. En la analtica al ingreso slo destacaba leucocitosis de 13,000 y anemia leve. Se inici tratamiento con metronidazol i.v.
500mg/8 h ante la sospecha de infeccin por C. difficile, tras toma de muestras para toxina. Se realiz rectoscopia al segundo da de
ingreso que confirma colitis seudomembranosa. Al tercer da del ingreso, el paciente presenta deterioro clnico importante, con
disminucin del nmero de deposiciones y aumento del dolor abdominal, que se hizo continuo, difuso, acompaado de distensin
abdominal e hipotensin. En la analtica destaca leucocitosis de 23,000, insuficiencia renal, con creatinina de 190 mg/dl y urea de 12,6
mg/dl, y lactato srico de 4 mmol/l. Una TC de abdomen muestra engrosamiento y edema difuso de todo el colon, ms evidente en
sigma y colon descendente.

PREGUNTA
El tratamiento mdico ideal para colitis pseudomembranosa es:

RESPUESTA
a.- Amikacina
b.- Ciprofloxacino
c.- Penicilina
d.- Vancomicina

HEMORROIDES. CIENCIAS BASICAS: Las hemorroides son dilataciones de los plexos hemorroidarios superior e inferior. Estn
localizadas en los ltimos centmetros del recto, en el conducto anal y en el recto. Forman parte de la anatoma normal de la regin y
cuando sufren alteraciones y producen sntomas se establece la enfermedad. Factores de riesgo: Multifactorial, estreimiento, dieta,
embarazo, edad, herencia?. SALUD PBLICA: En Mxico, 5% de la poblacin general presenta sntomas relacionados con las
hemorroides. Son raras antes de los 20 aos de edad; su frecuencia aumenta con la edad y es posible que el 50% de los adultos de 50
aos tenga o haya sufrido sintomatologa hemorroidaria. Incidencia de 4 a 36%. Incidencia mxima de 30-60 aos. Sin diferencia racial.
Ms en mujeres pero mayor severidad en hombres. PATOGENIA: Estreimiento, al implicarse mayor esfuerzo al evacuar, se produce
congestin de los cojinetes hemorroidales. Al persistir el pujo constante se distienden los soportes de dichos cojinetes y va
provocando el prolapso hemorroidario por debajo de la lnea anorectal hacia fuera del conducto anal. Alteraciones morfolgicas del
Traitz. Elongacin de las estructuras hemorroidales. Cambio del tejido elstico. Hipertona del EAI. Cambio irreversible en los vasos.
CLASIFICACIN: Anatmica 1. Internas (cubiertas por mucosas): I-IV, grado I; son cuando stas se exteriorizan hasta el conducto anal y
sangran. Grado II; son aquellas que sangran y se prolapsan al momento de la defecacin a nivel de ano pero se reducen
espontneamente. Grado III; son aquellas que sangran y se prolapsan ms all del ano al momento de la defecacin y requieren
reduccin manual. Grado IV; son las que sangran y se prolapsan en forma permanente y son irreductibles. 2. Externas (cubiertas por
piel): vascular, pliegue. 3. Mixtas. De acuerdo a su situacin: Complicadas; edema, trombosis, sangrado, absceso. No complicadas;
sangrado 89% (Rutilante, ordea, no doloroso, postevacuatorio, abundante en el papel), prolapso 36% (Pujo, asociado a la esfuerzo,
reduccin), prurito 27% (Posterior a la evacuacin, aseo deficiente, aseo excesivo), dolor 11% (solo con complicaciones). DIAGNOSTICO:
Clnico; Hemorroides externas, sntomas y signos principales dolor, tumoracin y prurito anal. Hemorroides internas,
fundamentalmente son dos: hemorragia y prolapso hemorroidario, la hemorragia: es el signo ms frecuente y ms constante. La sangre
por lo regular es roja, rutilante, expulsada con la metera fecal, en forma de estras en las heces fecales o en gotas al final de la
evacuacin. Inspeccin; Identificar hemorroides externas o prolapso de internas. Palpacin; Pliegues externos, tejido hemorroidal
redundante. Anoscopa, Identificacin y visin directa de los paquetes. TRATAMIENTO: Mdico: Incremento en el consumo de agua,
consumo de Fibra (30 gr), flebotnicos: Hidrosmina (sies) 200mg cada 8 horas 5 das, Diosmina (daflon) 450mg cada 8 horas 5 das,
Hisperidina (eletec) 50mg cada 8 horas 5 das. Ligadura con banda elstica. Escleroterapia: Esta consiste en la inyeccin de pequeas
cantidades de hidroxipolietoxidodecanol al 3% aplicado en la submucosa y extravascular por arriba del paquete vascular externo.
Crioterapia: Se utiliza xido nitroso que congela a una temperatura de menos 60 C y menos 80 C o con nitrgeno lquido que congela
a una temperatura de menos 180 C. El principio es la destruccin del tejido por medio de la coagulacin. Electrocoagulacin: Se basa
en un rayo calrico que evapora el agua intracelular y coagula las protenas. Se recomienda su aplicacin de 1.5 segundos en nmero de
3 a 4 en la base del tejido hemorroidario interno. Tratamiento Quirrgico: Milligan- Morgan. Tipo Parks (semiabierta). Ferguson
(cerrada). PPH (Procedure for Prolapse and Hemhorroids). El tratamiento quirrgico est indicado en todos los pacientes en los que se
ha fracasado el tratamiento mdico, en aquellos pacientes que tienen sintomatologa de hemorroides externas, en hemorroides
internas grado III y IV y en las complicaciones de la enfermedad hemorroidaria. COMPLICACIONES: Trombosis hemorroidaria nica
externa; Es una complicacin frecuente de causa desconocida. En forma clsica el paciente refiere dolor de forma brusca, acompaado
de tumoracin perianal. El tratamiento puede ser mdico o si el dolor es muy intenso, est indicada la trombectoma bajo anestesia
local. Trombosis hemorroidaria mltiple o masiva; Consiste en la trombosis de dos o ms paquetes hemorroidarios, acompaada de
MANUAL DE TRABAJO DEL CURSO ENARM CMN SIGLO XXI
CURSO ENARM CMN SIGLO XXI TEL: 36246001 Pharmed Solutions Institute PGINA 523

edema y dolor intenso. Dependiendo del tiempo de evolucin y su sintomatologa, puede ser tributario o de tratamiento mdico o
quirrgico. Hemorroides internas estranguladas. El paciente refiere dolor anal intenso de corta duracin, acompaado de mltiples
hemorroides que no se reducen con las maniobras habituales. Si continua su desarrollo se convierte en hemorroides gangrenosas. El
tratamiento siempre debe ser quirrgico y de urgencia

CASO CLINICO
Se trata de paciente masculino de 46 aos de edad el cual refiere dolor en la regio rectal, presenta manchado sanguinolento secundario
a la evacuacin, actualmente se encuentra estabilizado y se prepara para realizar procedimiento quirrgico electivo.

PREGUNTA
Considerando la preparacin del paciente, cual es la indicacin ms apropiada.

RESPUESTA
a.- Indica aumento de fibra a la dieta.
b.- Prepara con polietilenglicol por via oral.
c.- Se requiere enema baritado.
d.- Indica lactulosa por via oral.

HERNIAS DE PARED ABDOMINAL. CIENCIAS BASICAS: Una hernia es la protrusin de una vscera a travs de una abertura en la pared
(natural o adquirido) de la cavidad que la contiene, las caractersticas importantes de una hernia son el orificio defecto en la capa
aponeurtica ms interna del abdomen) y el saco herniarios (protrusin del peritoneo). El cuello del saco herniario corresponde al
orificio. Sitios de herniacin: son la ingle (inguinal), la cicatriz umbilical (umbilical), la lnea alba, la lnea semilunar de Spiegel, el
diafragma y las incisiones quirrgicas (postincisional). SALUD PUBLICA: Incidencia 10% poblacin general. 90% de hernias
inguinales/incisionales. CLASIFICACION: La hernia es externa si el saco protruye completo a travs de la pared abdominal; es interno si
el saco est dentro de la cavidad visceral. Una hernia es reducible cuando la vscera herniada puede regresar al abdomen; es
irreducible cuando esta resulta imposible. Una hernia estrangulada es aquella en la que la vascularidad de la vscera que protruye se
compromete hay sufrimiento del contenido (isquemia), necrosis y puede llegar a sepsis (translocacin bacteriana). El estrangulamiento
ocurre en hernias con orificios pequeos y sacos grandes. Una hernia incarcerada es una que no puede reducirse pero que no est
estrangulada. En una hernia de Richter el contenido del saco consiste en solo un lado de la pared del intestino (siempre es
antimesentrica). PATOGENIA: La herniacin puede abarcar la grasa preperitoneal, rganos retroperitoneales y un saco herniado de
peritoneo con estructuras intraperitoneales (por ejemplo, epipln u rganos). Son poco comunes las hernias clnicamente significativas
que carecen de saco peritoneal. Las hernias pueden complicarse cuando la inclusin de una vscera forma una pared del saco herniario.
Ello comprende un rgano parcialmente retroperitoneal y recibe el nombre de hernia por deslizamiento, de las cuales las ms comunes
afectan al colon. Etiologa: multifactorial causas enfermedad de tejido conectivo, tabaquismo, EPOC, actividad fsica, edad avanzada.
Por debilitamiento de la fascia transversalis. Aumento de la presin intraabdominalen: Embarazo, ascitis, obesidad, obstruccin
intestinal, esfuerzo defecatorio, prostatismo, ejercicio (Pesas). DIAGNOSTICO: Clnica; la historia natural de las hernias es un
crecimiento lento hasta llegar a ser irreducibles y desfigurantes, con riesgo de estrangulaciones. Las molestias que producen siempre
son peores al final del da y se alivian por la noche cuando el paciente se acuesta y la hernia se reduce. El dolor inguinal sin una hernia
demostrable, casi nunca indica el inicio de una hernia. La mayor parte tiene un inicio insidioso, pero algunas se desencadenan por
esfuerzo muscular intenso. Por lo general un saco herniario con su contenido crece y transmite un impulso palpable cuando el paciente
hace esfuerzos o tose. El paciente debe ponerse de pie en la exploracin, porque suele resultar imposible palpar una hernia inguinal
reducida con el paciente en posicin supina. Las hernias que no son detectables mediante exploracin fsica pueden demostrarse
mediante ultrasonido o TAC. El estrangulamiento produce dolor intenso en la hernia seguido muy pronto por hipersensibilidad,
obstruccin intestinal y manifestaciones de sepsis. La reduccin de una hernia estrangulada est contraindicada cuando hay sepsis o si
se cree que el contenido del saco presenta gangrena. TRATAMIENTO: Casi todas las hernias deben repararse a menos de que las
condiciones locales o sistmicas del paciente impidan un resultado favorable. La posible excepcin es una hernia con cuello amplio y
saco poco profundo, en cuyo caso se espera que crezca despacio. Los bragueros son tiles en el tratamiento de hernias pequeas
cuando la operacin esta contraindicada, pero estn contraindicados en pacientes con hernias femorales. Reparacin laparoscpica:
Actualmente indicada en prcticamente cualquier tipo de hernia de pared, hernias incisionales extensas, hernias incarceradas con o sin
crisis de oclusin. Contraindicaciones absolutas: Eventraciones supergigantes con obesidad asociada y ausencia de elementos
musculoaponeurticos, fstulas enterocutneas o foco sptico intraabdominal, cirrosis heptica con ascitis libre, hernias con
estrangulacin intestinal en fase temprana. HERNIAS INGUINALES: La ingle es una de las reas dbiles por naturaleza de la pared
abdominal y es el sitio ms frecuente de herniacin. Los varones tienen una probabilidad de 25 veces mayor de padecer una hernia
inguinal. Las hernias que surgen por arriba del pliegue abdominocrural son inguinales, las que emergen por debajo de ese pliegue son
femorales. Las hernias femorales ocurren ocasionalmente en las mujeres, pero no con tanta frecuencia como las inguinales; en los
varones son raras. Las hernias femorales suelen presentarse como una masa irreducible en la base medial del tringulo femoral de
Scarpa. La incidencia aceptada es de 3-4 %. La estrangulacin se produce en 1.3-3.0% de las hernias inguinales. Las hernias inguinales
se dividen en directas e indirectas. Indirecta: Un saco herniario indirecto es un proceso vaginal con dilatacin persistente, su saco pasa
en sentido oblicuo o en forma indirecta hasta llegar al escroto, son dos veces ms frecuentes en los varones Directa: Los sacos se
originan en el piso del trayecto inguinal (tringulo de Hesselbach), protruye en formas directa y quedan contenidos por la aponeurosis
del musculo oblicuo externo. Es frecuente que la vejiga sea un componente deslizante de un saco herniario directo. Casi todas las
hernias estranguladas son inguinales indirectas, pero las femorales tienen el mayor ndice de estrangulacin. Diagnstico: Se
diagnostica mediante la exploracin fsica del canal inguinal y regin crural. Aumento de la regin inguinal. Aumenta con el esfuerzo,
disminuye con el reposo. Se asocia a dolor en la regin inguinal, distinguir si es reductible o irreductible. Las hernias inguinales pueden
ser congnitas o adquiridas. Todas las inguinales indirectas son congnitas. El objetivo de la hernioplastia inguinal es prevenir la
protrusin peritoneal a travs del orificio miopectneo. La integridad se recupera de dos formas fundamentales: 1) cierre aponeurtico
MANUAL DE TRABAJO DEL CURSO ENARM CMN SIGLO XXI
CURSO ENARM CMN SIGLO XXI TEL: 36246001 Pharmed Solutions Institute PGINA 524

del orificio miopectneo hasta la magnitud necesaria y 2) reposicin de la fascia transversal defectuosa con unja prtesis sinttica
grande. A veces se combinan ambos mtodos. Las hernias se reparan por va anterior a travs de una incisin inguinal o posterior por
una incisin abdominal. El abordaje anterior es la incisin ms popular para la hernioplastia inguinal. Las reparaciones posteriores se
denominan hernioplastias properitoneales. Las protecciones sintticas de malla tienen una funcin importante en el tratamiento de las
hernias inguinales. Las prtesis de malla se usan como parche o tapn del orificio miopectneo para reforzar una reparacin clsica y
para reponer la fascia transversal. HERNIA UMBILICAL: Son ms frecuentes en mujeres. Constituyen el 2.4% de los casos de Hernias de
pared. 10% en adultos con antecedente de hernia en la infancia. La obesidad y los embarazos repetidos son los precursores usuales.
Suele observarse estrangulacin de colon y epipln. Las hernias de este tipo que tienen un defecto parietal pequeo solo se cierran con
sutura de polipropileno; aquellas con grandes defectos parietales se corrigen con una prtesis. HERNIA DE SPIEGEL: Son hernias
ventrales que ocurren a lo largo de la porcin subumbilical de la lnea semilunar de Spiegel y a travs de la fascia del mismo nombre.
Estas hernias son raras (1%) y a menos que sean grandes, su diagnstico se dificulta porque son interparietales y estn contenidas por
la aponeurosis del musculo oblicuo externo. Son ms frecuentes en el rea entre la cicatriz umbilical y la lnea que conecta las espinas
ilacas anterosuperiores, y el rea por debajo de la lnea arqueada y por arriba de los vasos epigstricos inferiores. 21% incareceradas.
HERNIA EPIGASTRICA: Es una protrusin de grasa properitoneal y peritoneo a travs de las fibras cruzadas de la vaina del recto en la
lnea media (alba), entre el apndice xifoides y la cicatriz umbilical. Las hernias epigstricas a menudo son irreducibles, siempre tienen
defectos aponeurticos pequeos, pueden ser mltiples. HERNIAS INCISIONALES: Son problemas quirrgicos graves. La obesidad y la
infeccin son las causas principales, de este trastorno, otras dehiscencia de la herida, hematoma o seroma, tcnica deficiente en el
cierre de la herida, desnutricin. El peso del panculo separa la incisin quirrgica y la infeccin obstaculiza la cicatrizacin de la herida.
Una hernia incisional grande produce movimiento abdominal respiratorio paradjico similar al trax inestable. La funcin diafragmtica
se vuelve ineficiente. La mayor parte de las hernias incisionales pequeas se trata con cierre simple de defecto aponeurtico. Las
hernias con defectos aponeurticos mayores de 10 cm tienen ndices de recurrencia hasta de 50%. Por tanto muchas de las hernias
incisionales y todas las recurrentes requieren una prtesis para conseguir una reparacin exitosa. Se prefiere la hernioplastia de Stoppa
(se implanta una prtesis muy grande de mersilene en la profundidad de los msculos de la pared abdominal sobre la vaina posterior
del recto o el peritoneo). La infeccin es una complicacin grave que se presenta en un 10% de los pacientes. La infeccin temprana se
trata mediante exposicin rpida y completa de la prtesis. Con tratamiento antimicrobiano intenso local y sistmico puede esperarse
la integracin completa de la prtesis. La reintegracin de la prtesis infectada casi nunca se logra cuando hay infecciones tardas y
debe retirarse la porcin secuestrada de la prtesis. HERNIA DE AMYAND: Aquella que contiene el apndice cecal inflamado a travs
de un defecto herniario inguinal. HERNIA PARAESTOMAL: Estas interfieren con las irrigaciones de colostoma y la adhesin de los
artculos para el cuidado de la estoma. Las hernias paracolostomales son ms frecuentes que las hernias paraileostomales y la
ocurrencia de ambas es ms probable cuando el estoma emerge por la lnea semilunar en lugar de la vaina del recto. Por lo general las
hernias parastomales son laterales a la ostoma. Se prefiere el cambio de sitio de la estoma sobre la reparacin local. La reparacin local
falla con frecuencia porque los msculos cinturn laterales a la ostoma carecen de aponeurosis suficiente, por ello se prefiere
implantar un trozo grande de mersilene con una hendidura para acomodar el estoma.

CASO CLINICO
Paciente de 21 aos de edad que acude a consulta por presencia de dolor en abdomen sealando la zona umbilical, a la exploracin
fsica se observa abdomen con masa depresible levemente dolorosa, que protruye espontneamente al pujar y al incorporarse.

PREGUNTA
Cul de las siguientes medidas no est indicado en este paciente.

RESPUESTA
a.- Confirmar con laboratorio y gabinete con ECG.
b.- Realizacin de estudios preoperatorios.
c.- Envi a segundo nivel y seguimiento posoperatorio.
d.- Confirmar hernia umbilical no complicada.

ORQUIEPIDIDIMITIS. CIENCIAS BASICAS: La epididimitis, orquitis y orquiepididimitis se definen como la inflacin del epiddimo,
testculo y del testculo/epiddimo respectivamente. Hablaremos de orquiepididimitis aguda si el proceso dura menos de 6 semanas y
crnica si es mayor. La causa ms frecuente en pre-pberes son las bacterias coliformes que infectan la va urinaria, en hombres
sexualmente activos predominantemente se encuentra infeccin por Chlamidya trachomatis y N. gonorrehae, en hombres mayores con
menor actividad sexual. Y con antecedentes de patologa obstructiva urinaria o instrumentacin reciente presentan infeccin por
patgenos urinarios como E. coli y paseudomonas. En homosexuales suele estar causada por coliformes y haemophilus influenzae.
Otras causas son tuberculosis, brucelosis, secundaria a traumatismos y medicamentos (amiodarona, acumulacin de
desetilamiodarona) y con menor frecuencia idioptica, antecedente de enfermedad de Behcet (vasculitis multiorgnica idioptica, con
ulceras genitales, ulceras aftosas recurrentes y uvetis). SALUD PUBLICA: Constituye la causa ms comn (80-90%) de escroto agudo en
pacientes mayores de 18 aos. El 56% de los hombres >60aos que presentan epididimitis se asocia con obstruccin de tracto urinario.
50% de pacientes con antecedentes de instrumentacin, uretral, catter permanente o intermitente pueden presentar epididimitis
infecciosa y de estos el 80% es de origen bacteriano (E. coli). PATOGENIA: El principal mecanismo patognico, lo constituye el reflujo de
orina infectada de forma ascendente (retrograda) hacia los conductos eyaculadores y deferente. La orquitis se produce por
contigidad. DIAGNOSTICO: Clnica; Dolor escrotal intenso de instauracin gradual con signos de inflamacin en el hemiescroto
afectado, incluyendo el epiddimo y el conducto deferente. Suele acompaarse de afeccin del estado general con fiebre (en ms de
50% de los casos, >38) y sntomas urinarios (disuria, secrecin uretral). Puede evolucionar a absceso escrotal cuyo tratamiento es el
drenaje. La inflamacin comienza en la cola del epiddimo y se extiende hacia el testculo. El cordn espermtico esta engrosado y
tumefacto y es frecuente la existencia de hidrocele reactivo. La elevacin del testculo produce disminucin del dolor (signo de Prehn
positivo), reflejo cremasteriano presente, la trasniluminacion, puede ser positivo por hidrocele reactivo. El diagnstico es
MANUAL DE TRABAJO DEL CURSO ENARM CMN SIGLO XXI
CURSO ENARM CMN SIGLO XXI TEL: 36246001 Pharmed Solutions Institute PGINA 525

fundamentalmente clnico, debemos realizar una BH en el que se encontrara leucocitosis y neutrofilia as como un sedimento de orina
que puede ser patolgico con piuria y/o bacteriuria. El urocultivo es negativo en 40-90% de los casos. En pacientes con prcticas
sexuales de riesgo y con flujo uretral se solicita: exudado uretral, tincin gramm si se sospecha gonorrea, prueba de ELISA si se
sospecha VIH. En caso de duda diagnostica podemos recurrir a la ecografa doppler que detectara un flujo testicular conservado o
aumentado y signos sugestivos de orquiepididimitis como acentuacin de septos, existencia de focos hiperecogpnicos y epiddimo
aumentado de tamao. TRATAMIENTO: Instaurar un tratamiento emprico sin esperar resultados de cultivos. En caso de sospecha de
infeccin de transmisin sexual (antecedente de contacto sexual sospechosos, uretritis, infeccin diagnosticada en pareja sexual) se
deber iniciar tratamiento con azitromicna o ceftriaxona (1 g IM o IV) en dosis nica junto con doxiciclina 100mgs/12hrs. Como
alternativa tenemos levofloxacino 500mgs c/12 hrs. Ambas pautas deben mantenerse por 10 das. En pacientes sin prctica sexual
trimetropim/sulfametoxaxol por 10-14 das. Debe tratarse a los compaeros sexuales del ltimo mes y evitar el coito sin proteccin
hasta haber terminado tratamiento. Si sospecha infeccin bacteriana inespecfica disponemos de varias pautas: ciprofloxacino 500mg/
12 hrs, norfloxacino 400mgs/ 12 hrs o amoxicilina-clavulanico 875mgs/ 8 hrs durante 10-14 das. En casos floridos debemos asociar
aminoglucocidos: tobramicina 100mgs IM o IV c/12 hrs o gentamicina 240mgs IM o IV cada 24 hrs. Adems del tratamiento antibitico,
existe una serie de normas generales que favorecen el alivio de los sntomas; actividad fsica limitada, suspensorio testicular, aplicacin
de frio local. Para el control, de dolor se recomienda el uso de AINEs (naproxen o diclofenaco por 2 semanas) que adems favorecen la
disminucin del proceso inflamatorio. El edema hemiescrotal puede persistir durante 4-6 semanas despus del tratamiento y de forma
residual puede persistir una induracin epididimaria indefinida. COMPLICACIONES: Absceso en epiddimo, oligoatenosospermia,
oligoatenosteratospermia, azoospermia, dolor crnico.

CASO CLINICO
Varn de 35 aos de edad. Consult por presentar desde haca tres meses, un aumento del testiculo derecho y por manchar la cama
con lquido como pus. En la exploracin testicular destacaba un aumento irregular del tamao escrotal, de consistencia dura, con
ulceracin escrotal y secrecin de tipo purulento. La analtica sangunea reej una leucocitosis como nico hallazgo a resear (la
bioqumica y marcadores tumorales fueron negativos).

PREGUNTA
Cul de los siguientes antecedentes es ms frecuente en este caso.

RESPUESTA
a.- Enfermedad de transmisin sexual recurrente.
b.- Presencia de bacterias anaerobias.
c.- Enfermedad de Behcets.
d.- Varicocele no tratado.

TORSION TESTICULAR. CIENCIAS BASICAS: Consiste en la rotacin del cordn espermtico sobre s mismo ocasionando una
disminucin del aporte sanguneo al testculo y al epiddimo. SALUD PUBLICA: Presenta una incidencia de 1/4000 varones menores de
25 aos. Es la causa ms frecuente de prdida testicular en el varn joven. Puede aparecer a cualquier edad pero presenta dos picos de
incidencia, uno durante el primer ao de vida y otro en la pubertad. PATOGENIA: La etiologa es muy variada aunque suele existir una
alteracin bilateral de la suspensin que permite que al testculo y cordn espermtico rotar sobre s mismos. La tasa de viabilidad cae
desde un 85-100%, cuando se resuelve en las primeras horas, hasta un 20% a las 12 hrs. En las 4 primeras horas se afectan las clulas
de la espermatognesis. Las clulas de Leydig son ms resistentes aproximadamente unas 10 hrs. Existen 2 formas anatomoclinicas:
torsin extravaginal; constituye un 5% de las torsiones testiculares. Es prcticamente exclusiva del neonato. Torsin intravaginal; forma
tpica del nio, adolescente y adulto joven, con un pico entre 8-15 aos. No siempre es de aparicin brusca. Existen formas
incompletas. Es la forma ms frecuente y constituye el 95% de los casos. DIAGNOSTICO: Se caracteriza por dolor testicular de inicio
sbito o gradual que puede irradiarse a regin inguinal o suprapbica, acompaado de tumefaccin escrotal. Pueden aparecer nauseas,
vmitos o hiperpirexia. El 50% de las causas ocurre durante el sueo. En la exploracin fsica el testculo suele estar elevado hacia el
anillo inguinal superficial y horizontalizado (signos de Governeur), ser doloroso al tacto e incluso estar aumentado de tamao. La piel
del escroto puede estar edematosa y enrojecida, lo que dificulta la exploracin testicular. Al elevar el testculo hacia la snfisis del pubis,
no disminuye o aumenta el dolor (signo de Prehn negativo) y existe ausencia de reflejo cremasterico (reflejo cutneo que aparece al
rozar la cara superointerna del muslo, produciendo elevacin del testculo homolateral o la contraccin de la pared abdominal). La
identificacin del epiddimo en posicin anterior o lateral y la palpacin de un cordn espermtico de consistencia blanda y algo
congestivo son indicios muy sugestivos de torsin testicular. Si en la exploracin fsica detectamos un testculo ascendido y
horizontalizado con el epiddimo en posicin anterior debemos intentar, tras la administracin de sedacin, la destorsin manual. En
caso de que exista duda diagnostica est indicada la realizacin de una ecografa doppler; que constituye el. TRATAMIENTO: La torsin
testicular constituye una autentica urgencia urolgica dado que la viabilidad testicular es inversamente proporcional a la duracin de la
torsin, para restablecer cuanto antes el flujo vascular al testculo evitando as la necrosis y atrofia posterior. El tratamiento definitivo
es quirrgico, si bien como medida temporal puede intentarse la destorsion manual. Se trata de una medida rpida, segura y no
invasiva. Tras esta maniobra debe realizarse una ecografa para comprobar la reperfusin testicular. Si esta medida es efectiva, la
oquidopexia (fijacin del testculo a la pared escrotal), que constituye el tratamiento de eleccin de la torsin podr realizarse de forma
electiva. Se recomienda que la orquidopexia sea bilateral ya que la causa anatmica que predispone a la torsin suele serlo. Algunos
autores recomiendan tambin la orquidopexia contralateral en caso necesario practicar una orquiectoma, en pacientes con historia
previa de torsiones.
MANUAL DE TRABAJO DEL CURSO ENARM CMN SIGLO XXI
CURSO ENARM CMN SIGLO XXI TEL: 36246001 Pharmed Solutions Institute PGINA 526

CASO CLINICO
Masculino de 18 aos de edad, present con dolor testicular izquierdo pocos das de duracin. No hay otros sntomas. El examen fsico
revel una induracin en el polo superior del testculo izquierdo. Los marcadores tumorales fueron normales. Ecografa demostr una
lesin intratesticular mal definida, hipoecoica, 13,2 x 12,6 mm. En ecografa Doppler de potencia, una zona hipoecoica apareci
completamente avascular en contraste con el resto del testculo.
PREGUNTA
Cual se las siguientes manifestacin considera ms relevante para el diagnostico.

RESPUESTA
a.- Signo de Prehn positivo.
b.- Reflejo cremasterico.
c.- Trasluminacin positiva.
d.- USG testicular.





















































MANUAL DE TRABAJO DEL CURSO ENARM CMN SIGLO XXI
CURSO ENARM CMN SIGLO XXI TEL: 36246001 Pharmed Solutions Institute PGINA 527

CANCER TESTICULAR:
CIENCIAS BASICAS: El tumor maligno de testculo es una de las neoplasias solidas ms comn y curable en el hombre entre 15-35 aos.
El 95% de los tumores malignos de testculo se origina del epitelio germinal primordial y 5% son del estroma gonadal. Factores de
riesgo: Historia familiar de cncer en familiares de primer grado, criptorquidea (principal factor), infertilidad, sndrome de Klinefelter,
tumor testicular contralateral, orquitis viral por sarampion. SALUD PUBLICA: Ocupa el 2% de todas las neoplasias. Tiene 3 picos de
incidencia importante en la infancia, edades de 25-40 aos y a los 60 aos. CLASIFICACION: Los tumores malignos de testculo de origen
germinal se clasifican en semnoma puros 50% (extremadamente sensibles a radioterapia) y no seminoma (responden a quimioterapia
basada en platino) 35% son puros y 15% mixtos. Los no seminomatosos incluyen los subtipos histolgicos de senos endodrmicos,
coriocarcinoma, carcinoma embrionario y teratoma maduro e inmaduro. DIAGNOSTICO: Temprano; la presencia de una masa testicular
solida indolora, es patognomnica de tumor testicular y se puede acompaar de dolor local, dolor de espalda y en flanco ipsilateral, tos,
hemoptisis. Se pueden presentar signos y sntomas de enfermedad metastsica. Se debe realizar una exploracin detallada del rea
genital incluida la transiluminacion escrotal, abdomen en bsqueda de masas palpables, en trax y abdomen buscando ginecomastia o
ganglios supraclaviculares. El 20% de los casos presentan Hidrocele secundaria. En neoplasia hormonosecretante: ginecomastia,
macrogenitosomia. Laboratorio y gabinete: Se debe efectuar determinacin srica de marcadores tumorales: alfafetoproteina, normal
<15ng/ml (AFP), se eleva en tumores de senos endodermicos y en carcinoma embrionario, nunca se eleva en seminomas puros.
Fraccin beta de la hormona gonadotropina corionica Humana (HGC), se eleva principalmente en coriocarcinoma. Deshidrogenasa
lctica (DHL), se eleva en 60% de pacientes con tumores no seminomatosos y en el 80% de los seminomas en etapas avanzadas. Se
debe realizar ultrasonido testicular para confirmar diagnstico de tumor slido. Para la etapificacin del tumor conformado el
diagnstico de cncer se debe realizar: TAC abdominopelvica y Rx de trax PA y lateral. Confirmacin mediante histologa. El PET-CT es
til para evaluar viabilidad de tumor residual posterior a tratamiento con quimioterapia en pacientes con tumores seminomatosos.
TRATAMIENTO: El cncer testicular es altamente curable, incluso en etapas avanzadas. La orquiectoma inguinal radical y ligadura del
cordn espermtico en el anillo inguinal profundo respetando as el drenaje linftico, es el estndar de tratamiento para los tumores
testiculares. SEMINOMA: La etapa clnica I tres opciones de manejo: vigilancia, radioterapia, quimioterapia (carboplatino). Etapa IIA:
radioterapia. Etapa IIB: radioterapia o quimioterapia (bleomicina, etoposido, platino=BEP). Etapa IIC y III: se considera enfermedad
avanzada y debe manejarse con quimioterapia (BEP). NO SEMINOMA: Etapa clnica I: de bajo riesgo, con vigilancia, quimioterapia,
linfadenectioma retroperitonela neuropreservadora. Etapa I: de alto riesgo, quimioterapia (BEP), linfadenectiomia retroperitonela
neuropreservadora, vigilancia si la quimioterapia est contraindicada y no se desea ciruga. Etapa IS: (marcadores sricos sin evidencia
radiolgica de tumor) deber recibir quimioterapia (BEP). Etapa IIA y IIB con marcadores tumorales negativos puede manejarse con
vigilancia, linfadenectiomia retroperitonela neuropreservadora. Etapa IIA y IIB, con marcadores tumorales elevados deber manejarse
con quimioterapia. Etapa IIC y III, deber manejarse con quimioterapia. PRONOSTICO: El de mejor pronstico seminoma. El de peor
pronstico coriocarcinoma. Seminomas estadios I o IIA operados y en irradiacin ulterior o quimioterapia; sobrevida mayor a 5 aos.
No seminomatosos 75 a 80% de xito. Estatus libre de enfermedad:
primeros dos aos: determinacin de marcadores biolgicos, RX de
trax cada 2 meses y TAC cada 3 meses. 3 aos: marcadores biolgicos,
RX trax cada 3 meses y TAC cada 6 meses. 4 aos: controles
semestrales. 5 aos: controles anuales. TUMORES GERMINALES:
CORIOCARCINOMA: Afortunadamente infrecuentes (<2%). Presenta
elementos cito y sincitiotrofoblasticos, extremadamente infrecuentes.
Elevada tasa metastasica. Se presenta en forma de lesin pequea y
con centro necrohemorragico. Su diseminacin es rpida y por va
hematgena habitualmente. Presenta casi 100% de metstasis al
momento del diagnstico. TERATOCARCINOMA Y TERATOMAS:
Derivan de una o ms hojas embrionarias, contiene restos de las tres
capas embrionarias (ecto, meso y endodermo). La forma inmadura
contiene elementos celulares primitivos indiferenciados. Alta
resistencia a los tratamientos radio y quimioterapicos. Metastasis en
15% al momento del diagnstico. CARCINOMA EMBRIONARIO: Forma
el 25% de los tumores germinales. Existe en 2 formas; La del adulto (Ca.
Embrionario propiamente dicho). La infantil (tumor de Yolk, del saco
vitelino o del seno endodermico), ms frecuente tumor testicular en
nios. Predominan las formas mixtas y necrohemorragicas. Elevada tasa metastsica (60%), ganglios regionales. El tumor el saco
vitelino es el considerado ms agresivo de la lnea germinal. TUMORES NO GERMINALES (Estromales). En general derivan del conducto
de Wolf, son por regla benignos. Se presentan en los tejidos de soporte y en los productores hormonales de los testculos. TUMOR DE
CELULAS DE LEYDIG: Suponen el 1-3 % del total de todos los tumores testiculares. Es el ms comn de los tumores testiculares de
clulas no germinales. Su origen deriva de las clulas intersticiales del testculo, clulas productoras de androgeneos. La bilateralidad en
los tumores testiculares es excepcional, solo ocurre en el 0,7% sincrnicamente, y con carcter metacrnico (hasta en los 5 aos
posteriores al diagnstico del contralateral) en el 1,5%. Sin embargo en los tumores de Leydig y Sertoli puede elevarse al 5-10%. El
tumor de Leydig en el adulto provoca manifestaciones endocrinas en el 20-40% de los casos. Estos tumores son hormonoproductores
por lo que pueden dar manifestaciones endocrinas, y cuando las dan sern distintas dependiendo de la edad de los pacientes: Adulto:
ginecomastia, disfuncin erctil y disminucin de la libido, oligozoospermia y atrofia del otro testculo. Nios: A virilizacin precoz con
desarrollo de caracteres sexuales secundarios: aumento del tamao de los genitales externos, presencia de vello pbico, cambios de la
tonalidad de la voz. etc.; solo en el 10% de stos se acompaa de ginecomastia. Ninguna hormona va a ser secretada en niveles
suficientes para que pueda ser til como marcador tumoral.



RIESGO NO SEMINOMA SEMINOMA
Riesgo
bajo
(bueno)
Tumor primario testicular o
retroperitoneal y sin
metstasis pulmonares o
viscerales. Cualquier valor de
los siguientes: AFP
<1,000ng/ml. hCG <5,000iu/l.
LDH <1.5 por arriba del lmite
normal
Cualquier sitio primario y
sin metstasis
pulmonares o viscerales
y AFP: normal. hCG
cualquier valor. LDH
cualquier valor.
Riesgo
interme
dio
Tumor primario testicular o
retroperitoneal y sin
metstasis pulmonares o
viscerales, con AFP 1,000-
10,000ng/ml. hCG 5,000-
50,000iu/l. LDH1.5-10 por
arriba de limite normal.
Cualquier sitio primario y
con metstasis
pulmonares o viscerales
y AFP: normal. Hcg
cualquier valor. LDH
cualquier valor.
Riesgo
alto
Tumor primario en mediastino
y sin metstasis pulmonares o
viscerales con AFP >10,000
ng/ml. hCG >50,000iu/l. LDH
>19 por arriba de limite
normal
No existen pacientes
clasificados con riesgo
alto
MANUAL DE TRABAJO DEL CURSO ENARM CMN SIGLO XXI
CURSO ENARM CMN SIGLO XXI TEL: 36246001 Pharmed Solutions Institute PGINA 528

CASO CLINICO
Masculino, de 35 aos, con antecedentes de agenesia testicular derecha, que asisti a consulta externa por un aumento del volumen de
la fosa ilaca derecha (FID). En el examen fsico se constat un tumor a este nivel, interpretndose como plastrn apendicular, por lo
que se le administr antibioticoterapia. Posteriormente se le realiz ultrasonido abdominal, visualizndose en el hipogastrio y la FID, en
correspondencia con el aumento de volumen, una imagen tumoral heterognea vascularizada (al aplicarle Doppler color), de
13813cm, que recordaba la ecoestructura de un testculo aumentado de tamao. Se explor, adems, el testculo izquierdo, que
tena ecoestructura y tamao normales.

PREGUNTA
Cul es el factor de riesgo ms importante para el cncer testicular.

RESPUESTA
a.- Paciente con Klinefelter.
b.- Infertilidad.
c.- Criptorquidia no reparada en la pubertad.
d.- Familiar con cncer testicular.

HIPERTROFIA PROSTATICA BENIGNA (HBP). CIENCIAS BASICAS: En la clnica puede significar: 1) deteccin microscpica de la
hiperplasia, es decir la proliferacin del estroma y el epitelio; b) crecimiento de la prstata detectado por el examen rectal digital o por
ultrasonido y c) un grupo de sntomas asociados con la hiperplasia prosttica y definidos con el trmino sntomas de tracto urinario
inferior (STUI). Las diferentes definiciones surgen por que el tamao de la prstata no siempre correlaciona con los sntomas (solo 30-
50% de los hombres con HBP detectada por tacto rectal o ultrasonido, presentan sntomas), as el termino HBP implica uno o ms de los
hallazgos ya mencionados. La HBP es considerada en la actualidad una enfermedad progresiva con origen hormonal, en donde la
dihidrotestosterona (DHT), producto de la accin de la enzima 5-afa-reductasa tipo 2 sobre la testosterona, es responsable. SALUD
PUBLICA: La prevalencia de HBP aumenta en forma lineal con la edad, en todos los grupos tnicos. En general afecta a los hombres
mayores de 45 aos y la presentacin de los sntomas suele darse a los 60-65 aos de edad. Su prevalencia es de >50% y 90% a los 60 y
85 aos respectivamente. La mortalidad y la frecuencia de complicaciones serias asociada con la HBP bajas, reportndose en 1-3%
incluyendo estas complicaciones la retencin urinaria aguda. CLASIFICACION: Desde el punto de vista histolgico se pueden distinguir
al menos los siguientes cinco tipos de hiperplasia prosttica benigna: Estromal. Fibromuscular. Muscular. Fibroadenomatosa.
Fibromioadenomatosa. PATOGENIA: Los STUI asociados a HBP, tienen origen tanto anatmico como neuro-hormonal, ya que pueden
ser producto de la obstruccin mecnica al flujo urinario secundaria al crecimiento prosttico y/o por el aumento de tono y presin del
musculo liso en la uretra, estroma y cuello de la vejiga, mediado por alfa-1-adrenoreceptores. En los hombres entre los 50 y 70 aos, la
masa hiperplstica e hipertrfica de la prstata est formada en un 14% por ndulos y en un 86% es difusa, que ocurre principalmente
en la zona de transicin. En las primeras fases de la hiperplasia predomina ampliamente el componente estromal de la zona de
transicin, donde actuaran al menos tres factores con accin inductora mesenquimatosa embrionario-smil: Factor bsico de
crecimiento fibroblstico (bFGF), factor de crecimiento transformador tipo B1 (TGF-B1), factor de crecimiento transformador tipo B2
(TGF-B2). Los tres factores, perfectamente identificados, actan sinrgicamente llevando el estroma a un estado mesenquimtico.
Adems bFGF es mitognico, lo que significa crecimiento glandular; ste es regulado por TGF-B2. Es de conocimiento muy antiguo que
para que haya hiperplasia prosttica benigna es indispensable la presencia del testculo, lo que afirma la participacin de los
andrgenos, ya sea en forma directa, permisiva o activadora, como es la accin del KGF (keratinocitic growth factor), que es el primer
factor probado como estimulante del crecimiento epitelial prosttico. Adems se necesita de tiempo, es decir, envejecimiento. El
fenmeno histolgico de hiperplasia prosttica benigna es propia del hombre que envejece. DIAGNOSTICO: Los sntomas de HBP
pueden ser secundarios a: 1. Obstruccin; disminucin en el grosor y fuerza del chorro urinario, pujo al iniciar la miccin, goteo
terminal, sensacin de vaciado incompleto o retencin aguda de orina. 2. Disminucin de la capacidad de almacenamiento en la vejiga;
nicturia, urgencia e incontinencia urinaria de urgencia y por rebosamiento. No se ha encontrado una relacin directa entre la severidad
de los sntomas y el tamao de la prstata; algunos hombres presentan sntomas severos de obstruccin urinaria con un crecimiento
prosttico mnimo, mientras que otros presentan sntomas mnimos pero con prstatas grandes. El International Prostate Symptom
Score (IPSS) permite evaluar los sntomas de manera objetiva y reproducible de acuerdo a su severidad en leves, moderados o graves.
Para realizar el diagnostico clnico de HBP, elabore la historia clnica y un examen fsico que incluya: 1.Exploracion de abdomen para
detectar vejiga palpable. 2. Tacto rectal para evaluar las caractersticas de la prstata: tamao, forma, simetra, textura y consistencia.
Utilice IPSS y evalu la afeccin de la calidad de vida del paciente con sospecha inicial de HBP, para mejorar la deteccin y catalogar los
STUI de acuerdo a severidad. En pacientes con STUI y sospecha de HBP, solicite un EGO para descartar la presencia de infeccin urinaria
y/o hematuria. Glicemia: con el objetivo de excluir DM. El ultrasonido vesical y prosttico con medicin de orina residual puede ayudar
a determinar el tamao prosttico y el volumen urinario residual, lo que ayuda a predecir los beneficios de un tratamiento mdico o
guiar al tratamiento quirrgico. Tambin puede ser til examinar riones y vejiga para descartar litiasis en caso de infeccin de vas
urinarias repetidas y tumores en caso de hematuria persistente, as como la repercusin de la obstruccin prosttica sobre el tracto
urinario superior. Hay una relacin directa de los niveles de APE y el volumen de la prstata, sin embargo los pacientes con HBP no
tienen mayor riesgo de cncer de prstata. Los valores altos de APE, se relacionan fuertemente con la probabilidad de tener cncer de
prstata. Una prstata con un tamao >30ml, un flujo urinario dbil y APE >1.4ng/ml, son factores de riesgo para progresin de HBP
con un riesgo mayor de presentar retencin aguda de orina (RAO) y necesidad de ciruga, por la tanto deben de recibir tratamiento. La
endoscopia permite evaluar muy bien el tamao prosttico, el cuello vesical, las repercusiones de la uropata obstructiva sobre el
detrusor y orificios ureterales. Si bien se puede hacer en forma ambulatoria y bajo anestesia local, hay que ser cauto en su indicacin,
pues en pacientes muy obstruidos y muy sintomticos existe el riesgo de provocar retencin completa por edema y dolor, adems de
infeccin. En muchos pacientes se practica la endoscopia como una primera parte del procedimiento teraputico endoscpico bajo
anestesia. TRATAMIENTO: La terapia de observacin y vigilancia, es segura para la mayora de los pacientes con sntomas leves a
moderados de HBP. Es tambin un tratamiento adecuado en los casos de pacientes con sntomas moderados a severos pero, que no
MANUAL DE TRABAJO DEL CURSO ENARM CMN SIGLO XXI
CURSO ENARM CMN SIGLO XXI TEL: 36246001 Pharmed Solutions Institute PGINA 529

han desarrollado complicaciones secundarias a la obstruccin del flujo urinario (insuficiencia renal, retencin urinaria aguda o
infecciones recurrentes). Los pacientes con HBP, son sntomas leves a moderados o moderados a severos pero sin complicaciones y que
no presenten impacto en su calidad de vida, pueden ser manejados con vigilancia estrecha que incluya valoracin anual con TR, APE y
US vesical y prosttico con medicin de orina residual. La cafena y alcohol tienen un efecto diurtico e irritante que aumenta la
frecuencia miccional, la urgencia y la nicturia. La reduccin en la ingesta de lquidos reduce la frecuencia miccional en periodos
convenientes. Para mejorara los sntomas, disminuya la ingesta de lquidos libres por la tarde-noche y de irritantes como la cafena y el
alcohol en cualquier presentacin. Las ventajas del tratamiento farmacolgico incluyen: conveniencia y el evitar la morbilidad potencial
asociada a la ciruga. La desventaja es que es de forma indefinida. Los alfabloqueadores tamsulosina, terazosina y doxazosina tienen
una eficacia similar sobre el alivio o mejora de los STUI (relajan el musculo liso en la prstata y cuello de la vejiga), son bien tolerados y
su eficacia se mantiene despus de 6-12 meses de tratamiento. No reduce el tamao de la prstata ni alteran la progresin de HBP.
Actualmente existen alfabloqueadores con mejor tolerancia por su uroselectividad sin disminuir su eficacia. Tal es el caso de alfuzosina
y silodosina, estos son ms efectivos que los inhibidores de la 5-alfa reductasa, para mejorar los sntomas en el primer ao de
tratamiento y su efecto se manifiesta desde el primer mes de inicio de tratamiento. Los alfabloqueadores presentan efectos adversos
como hipotensin ortostatica, cansancio, mareos, problemas eyaculatorios y congestin nasal, se recomiendan en todos los pacientes
candidatos a tratamiento mdico. Las dosis recomendadas son: alfazulozina 10mgs diarios, tamsulosina 0.4mg diarios, terazosina 2-5mg
diarios, doxazosina 2-4 mgs diarios. Los I-5ARs inhiben la enzima alfa reductasa, lo que da una disminucin de las concentraciones de
dihidrotestosterona a nivel prosttico, que resulta en una reduccin del tamao de la prstata, una mejora de los sntomas, menor
frecuencia de RAO y de intervenciones quirrgicas, se recomiendan en pacientes con STUI y crecimiento prosttico >40cc. Finasteride
5mg diarios. En la actualidad se considera a la RTUP, como el tratamiento ms efectivo para la HBP. Los pacientes que son sometidos a
tratamientos quirrgicos transuretrales mnimamente invasivos (TUMT, HIFU, HoLEP, PVP), tienen una mejora menor en el puntaje del
IPSS y de la tasa de flujo urinario mximo, que los pacientes a los que se les realiza RTUP. La RTUP no reseca todo el tejido prosttico
susceptible a cncer, por lo tanto deben evaluarse anualmente con TR y APE. La RTUP tiene los siguientes riesgos: mortalidad >0.25%,
intoxicacin hdrica 2%, estenosis de uretra y contractura de cuello vesical 3.8%, eyaculacin retrograda 65-70%, incontinencia urinaria
de esfuerzo, disfuncin erctil. La prostactectomia abierta est indicada en pacientes con prstatas de >80cc y en aquellos con
complicaciones asociadas, como litiasis vesical o que ameriten reseccin diverticular.

CASO CLINICO
Se trata de paciente masculino de 62 aos de edad el cual acude a consulta de control ya que padece de diabetes mellitus desde hace
15 aos, actualmente con depuracin de creatinina menor a 60 ml/ min. refiere que desde hace 7 das ha presentado ardor al orinar as
como dificultad, adems de tardar en terminar de miccionar, al interrogatorio refiere que desde hace 6 meses presenta disminucin del
chorro de la orina as como goteo, niega hematuria usted considera que actualmente cursa con IVU la cual controla y cita para valorar la
prstata, 15 dias porterior al tratamiento por IVU evalua nuevamente al paciente observando al tacto rectal crecimiento no indurado,
levemente doloroso, solicita antgeno prosttico reportado en 5.0 ng/ml. Urocultivo negativo, EGO negativo.

PREGUNTA
Cul es la conducta a seguir para establecer el diagnostico.

RESPUESTA
a.- USG renal.
b.- USG vesical.
c.- USG prostatico.
d.- Enviar a segundo nivel.

CANCER DE PROSTATA. CIENCIAS BASICAS: Es el crecimiento anormal de las clulas de la glndula prosttica, que adems tienen la
capacidad, de invadir otros rganos. Es un tumor que nace del epitelio acinar y/o ductal de la prstata que puede variar
considerablemente en su diferenciacin glandular, anaplasia y comportamiento. Es una de las neoplasias malignas ms frecuente en los
hombres y se incrementa conforme aumenta la edad. En la prstata fundamentalmente se distinguen 3 zonas: la zona Central cruzada
por los conductos eyaculadores que supone un 25% de la glndula, la zona Transicional que rodea a la uretra posterior con un 5% del
volumen glandular y la zona Perifrica que ocupa un 70% del volumen glandular. Ms de 95% de todos los canceres de prstata son
adenocarcinomas y desde el punto de vista anatomopatologico, el 97% de los tumores se originan en la periferia. La histologa del
cncer de prstata es muy importante, se ha visto que un 95% son adenocarcinomas acinares originados en la porcin glandular de la
prstata y solo un 4% es transicional, escamoso o endometroide, estos derivan de los conductos y por lo tanto son
hormonodependientes. El 1% son sarcomas. SALUD PUBLICA: El 87.6% de las defunciones se registraron en la poblacin mayor de 65
aos. El cncer de prstata se ubic en segundo lugar en el grupo de edad postproductiva, con una tasa de 71.7 por 100 mil habitantes.
Existe diferencia notable en la incidencia del cncer entre el mundo oriental y occidental, atribuibles entre otros factores a la ingesta
de grasas en la dieta. Un ejemplo lo constituyen los chinos, que de una incidencia de 0.8 por 100 000 hab en su vida nativa, pasan a
tener tasas de 18.6 cuando se integran a la civilizacin occidental. La raza negra en EE.UU tiene la mayor tasa de incidencia con 100.2
por 100 000 hab. PATOGENIA: En la zona Perifrica se desarrollan el 68% de los cnceres, en la zona Central un 8% y en la zona
Transicional un 24%. La zona Perifrica es accesible al tacto rectal mientras que los tumores que se desarrollan en las zonas Central y
Transicional se valoran mejor con la ecografa. Se han postulado factores genticos, las razas negras y los escandinavos tienen una alta
incidencia, mientras que los japoneses tienen la ms baja. Tambin existira una predisposicin familiar, pariente en primer grado de
caso ndice tiene un riesgo 10% mayor de tener un cncer prosttico. Otros factores implicados son la alimentacin y el ambiente. Se ha
visto que el aceite de soya es un protector ya que se transforma en un estrgeno dbil; que los habitantes de zonas rurales tienen una
mayor incidencia atribuida a los insecticidas agrcolas; las prostatitis vrales por citomegalovirus o virus herpes a repeticin tambin
aumentaran las posibilidades de desarrollar un cncer de prstata. Un factor clave es el factor hormonal, se relaciona claramente con
la presencia de testculos funcionales; los eunucos no tienen cncer de prstata. La presencia de receptores esteroides en las clulas
MANUAL DE TRABAJO DEL CURSO ENARM CMN SIGLO XXI
CURSO ENARM CMN SIGLO XXI TEL: 36246001 Pharmed Solutions Institute PGINA 530

tumorales y la respuesta positiva a la supresin de los andrgenos as como los altos niveles de Dihidrotestosterona en las clulas
cancerosas, apoyan la hiptesis hormonal. DIAGNOSTICO: En general los cnceres de prstata son silenciosos, crecen lentamente,
invaden primero la cpsula prosttica, luego ganglios pelvianos (obturatrices e iliacos) y dan metstasis preferentemente al hueso. Slo
dan sintomatologa obstructiva. Las lesiones en el hueso son osteoblsticas (se ve ms blanco y algodonoso el hueso en la radiografa) y
los sitios ms frecuentes de metstasis son: Columna sacra, crestas ilacas, columna lumbar, columna dorsal, crneo, etc. En hombres
con mayor riesgo, con antecedentes familiares directos de cncer de prstata o de mama, se debe iniciar un abordaje de deteccin a
los 40 aos. El diagnstico se establece a travs de estudios clnicos y de gabinete: tacto rectal, ultrasonografa, TAC, RMN, y
marcadores sricos del tumor; entre estos, sin duda el ms importante es el antgeno prosttico especfico (APE) el cual se produce
nicamente en el citoplasma de clulas prostticas benignas y malignas. Su nivel srico se correlaciona con el volumen de ambos
tejidos, maligno y benigno. El APE, se determina como normal <4ng/ml, intermedio 4.1-10ng/ml y altamente sospechoso arriba de
10ng/ml. Estos dos ltimos requieren de biopsia, ya que las dos terceras partes de cncer prosttico se ubican en personas con
resultados de 10 y ms ng/ml. El APE ha demostrado ser til para supervisar la eficacia del tratamiento y para controlar la recada
despus de que el tratamiento ha terminado. Los niveles de la fosfatasa acida prosttica (FAP) en la sangre pueden elevarse en
pacientes que tienen ciertas enfermedades benignas de la prstata o cuyo cncer est en la fase temprana. En hombres entre 50 y 55
aos se da una forma poco frecuente de cncer prosttico con Gleason 8 a 10, de crecimiento rpido y aneploides, en general se
desarrollan en dos o tres meses. Es importante que en el informe que se recibe del patlogo se establezca el grado de Gleason, que es
una clasificacin basada en la diferenciacin celular y la relacin estroma-glndula del cncer (no mideanaplasia), para esto se le asigna
un puntaje de 1 a 5 a la zona ms "mala" del cncer y otro a la ms "buena" y se suman. La escala de Gleason va del 2 al 10, siendo el 2
el cncer ms benigno. En general los cnceres con Gleason del 2 al 4 tienen buen pronstico, del 5 al 7 son de pronstico intermedio y
del 8 al 10, son los de peor pronstico. El Gleason es importante porque determina el pronstico y evolucin. La biopsia de prstata se
considera como "el estndar de oro" pero generalmente se realiza luego de constatar anormalidades en el tacto rectal o elevacin del
APE, por lo que la sensibilidad del mtodo est sobrestimada. La Sociedad Americana de Cncer recomienda a todos los varones
mayores de 50 aos se sometan a tacto rectal (TR) y deteccin de APE. En los casoso de antecedentes familiares, el control se deber
iniciar a los 40 aos. Si el TR y el APE son normales se recomienda control anual. Si el TR es normal y el APE est entre 4.1 y 10 ng/ml se
realizar Ecografa Transrectal (ETR) para determinar la densidad del APE y detectar ndulos con el fin de realizar biopsia prosttica
dirigida o aleatoria. Si el TR es normal y el APE mayor de 10 se realizar biopsia dirigida o aleatoria mediante ETR. Si el TR es
sospechoso se realizar biopsia de dicha zona, independientemente del APE. TRATAMIENTO: El tratamiento depende del TNM y de la
edad del paciente. Las personas con cncer T1-T2 tempranos, localizados, que no han invadido la cpsula, debern ser sujetos a
prostatectoma radical, resecando la prstata y cuello vesical, ganglios obturatrices, ganglios ilacos y vesculas seminales. La sobre vida
de estos pacientes, en general es buena; es decir el 90% sobrevive a los 15 aos, con tratamiento precoz. Las complicaciones de la
prostatectoma radical son la impotencia que se produce en casi el 100% de los tratados y la incontinencia de orina se ve en un 30%
aproximadamente. Si se diagnostica cncer TNM IV, con metastasisi a distancia solo se realiza tratamiento paliativo con supresin del
estimulo hormonal, de estos pacientes con o sin tratamiento 10% se muere antes de ao y otro 10% vive mas de 10 aos. El promedio
de sobrevida es dos a tres aos. No es un cncer tan maligno responde a la supresin de testosterona. Para obtener un bloqueo
hormonal completo se realiza: Orquiectoma. Frmacos bloqueadores andrognicos. Para el bloqueo andrognico se pueden utilizar
anlogos de LHRH como el Leuprolide y Decapeptil que tienen el problema de su alto costo. Tambin se puede utilizar flutamida. Se ha
ensayado la administracin de estrgenos como el dietilestilbestriol en dosis de 2.5 mg al da, es menos costoso, acta bloqueando los
receptores perifricos de testosterona. En general con el tratamiento paliativo se obtiene una mejora de la sintomatologa general y
tambin de la uropata obstructiva. No se realiza quimioterapia. Radioterapia solo local con implantes en el cncer TNM I o II. Tambin
se utiliza como tratamiento local de las metstasis de columna, pero slo es paliativo. Tratamiento del tumor incidental: En los
pacientes con expectativa de vida menor de 10 aos y con un tumor focal A1, se les aplicar conducta conservadora efectuando
controles de APE, TR y EcoTR cada 6 meses. En los pacientes con expectativa mayor de 10 aos o tumores focales A2 se realizar ciruga
radical o radioterapia local. Tratamiento del tumor localizado, el tratamiento de eleccin es la prostatectomia radical, siempre que el
estado general sea bueno y la esperanza de vida sea superior a 10 aos. En los casos con mal estado general, edad avanzada o negativa
al tratamiento quirrgico puede ser til la radioterapia radical o bien el bloqueo anbdrogenico. Tratamiento del tumor con extensin
extraprosttica: Los pacientes con afectacin capsular no son buenos candidatos para la ciruga radical pues ms del 50% van a tener
afectacin ganglionar y la exresis puede ser incompleta. El tratamiento con bloqueo hormonal completo logra disminuir el volumen
tumoral y facilita la ciruga, pero pocos pacientes logran disminuir el estadio tumoral y la supervivencia no se modifica. El tratamniento
adecuado depender de la presencia de sntomas obstructivos para instaurara el bloqueo andrognico completo o la desobstruccin
mediante RTU. Tratamiento del tumor diseminado: El tratamiento es mediante la supresin hormonal, bien sea por castracin
quirrgica o bien con anlogos de la LHRH. Si se quiere actuar sobre los andrgenos suprarrenales se asociar un antiandrgeno al
tratamiento, efectundose un bloqueo completo. Prostatectomia radical: En esta ciruga se extirpan completamente la prstata,
vesculas seminales y se restituye la va urinaria mediante anastomosis vesicouretral. Se asocia a linfoadenopata de ganglios iliacos
internos (principal indicacin: cncer prosttico localizado T2a y T2b), complicaciones: mortalidad (0,2%), incontinencia urinaria (2
7%), disfuncin erctil (100%), estenosis uretrovesical (1-3%), linfocele (1-3%). La disfuncin erctil que puede ocurrir despus de esta
operacin puede ser evitada al respetar los haces neurovasculares localizados en hojas de la fasia plvica lateral y que estn muy
vecinos a la glndula prosttica. Se considera que no existe enfermedad residual despus de la ciruga cuando el APE desciende al cabo
de algunas semanas a cifras indetectables (<0,02 ng/ml). Por el contrario, el APE que se mantiene detectable o el APE en sostenido
ascenso son indicadores de recidiva tumoral o recidiva bioqumica. Radioterapia: Usada como terapia definitiva en alrededor de 1/3 de
los pacientes. Se emplea radioterapia externa y la radioterapia interticial (braquiterapia). Es eficaz con resultados similares a la ciruga
al tratar tumores pequeos (intra prostticos). Complicaciones: disfuncin erctil (30-45%), toxicidad gastrointestinal (3-5%),
incontinencia urinaria (menos del 5%). Terapia hormonal: Pilar del tratamiento de cncer prosttico metastsico (etapa IV). Los
andrgenos (testosterona) son fundamentales para el crecimiento y funcionamiento de las clulas prostticas. Son producidos
mayoritariamente en los testculos (95%) y en la glndula suprarrenal (5%). La forma ms simple de emplear la hormoneoterapia es la
castracin quirrgica. Hormonoterapia de 1 lnea; orquiectoma, agonistas LHRH (zoladex, lupron), antagonistas LHRH (abarelix),
antiandrgenos (casodex) (bicalutamida), antiandrgenos/inhibidores de 5alfa-reductasa tipo II (flutamida). Hormonoterapia de 2
MANUAL DE TRABAJO DEL CURSO ENARM CMN SIGLO XXI
CURSO ENARM CMN SIGLO XXI TEL: 36246001 Pharmed Solutions Institute PGINA 531

lnea: bloquear sntesis adrenal de andrgenos (hodrocortisona, ketoconazol, aminoglutemida), basados en el hecho de que las
glndulas adrenales continan produciendo andrgenos despus de una castracin mdica o quirrgica, estrategia conocida como
bloqueo andrognico mximo. Quimioterapia: No hay pruebas que la quimioterapia prolongue la sobrevida, sin embargo, la
administracin de prednisona puede aliviar los sntomas en alrededor de 1/3 de los casos. PRONSTICO: Cncer prosttico localizado,
muestra supervivencia de 65 75 % a los 10 aos. Cncer prosttico diseminado; supervivencia del 55% a los 10 aos.- Cncer
prosttico con metstasis seas; supervivencia a los 10 aos: 15%. Medio de sobrevida = 30 meses (aunque un 10% vive ms de 10
aos)

CASO CLINICO
Se trata de masculino de 81 aos de edad el cual se encuentra en cuidados paliativos por cncer de prstata con metstasis, el cual
recibe tratamiento con goserelina, es ingresado a urgencias debido a que presenta en casa estado confusional con tendencia a letargia,
a la exploracin fsica se observa con estado avanzado de emaciacin, mucosas orales secas, deshidratacin moderada, pobre
respuesta a estmulos dolorosos pero presente, el examen mental es difcil realizar por embotamiento, las constantes vitales son TA
110/50 mmHg, FC de 112 lpm, se observa afebril, debido a su estado actual y antecedentes se considera que el paciente presenta
hipercalcemia, se realizan medidas de estabilizacin en espera de laboratorios.

PREGUNTA
Cual es la conducta menos apropiada a seguir considerando el estado de hipercalcemia.

RESPUESTA
a.- Solucion salina.
b.- Pamidronato.
c.- Furosemida.
d.- Dexametazona.

CASO CLINICO
Se trata de masculino de 81 aos de edad el cual se encuentra en cuidados paliativos por cncer de prstata con metstasis, el cual
recibe tratamiento con goserelina, es ingresado a urgencias debido a que presenta en casa estado confusional con tendencia a letargia,
a la exploracin fsica se observa con estado avanzado de emaciacin, mucosas orales secas, deshidratacin moderada, pobre
respuesta a estmulos dolorosos pero presente, el examen mental es difcil realizar por embotamiento, las constantes vitales son TA
110/50 mmHg, FC de 112 lpm, se observa afebril, debido a su estado actual y antecedentes se considera que el paciente presenta
hipercalcemia, se realizan medidas de estabilizacin en espera de laboratorios.

PREGUNTA
Cual es la conducta menos apropiada a seguir considerando el estado de hipercalcemia.

RESPUESTA
a.- Solucion salina.
b.- Pamidronato.
c.- Furosemida.
d.- Dexametazona.

























MANUAL DE TRABAJO DEL CURSO ENARM CMN SIGLO XXI
CURSO ENARM CMN SIGLO XXI TEL: 36246001 Pharmed Solutions Institute PGINA 532

TRAUMA CRANEOENCEFALICO (TCE):
CIENCIAS BASICAS: En adultos, es definido como un intercambio brusco de energa mecnica que genera deterioro fsico y/o funcional
del contenido craneal. Se consigna como alteracin del contenido enceflico el compromiso de conciencia, la amnesia postraumtica
y/o un sndrome vertiginoso o mareos persistentes. Tambin debe considerarse como un signo de disfuncin del contenido craneal la
aparicin de una cefalea holocrnea persistente y progresiva que puede o no acompaarse de vmitos. Se distingue de la Contusin de
crneo, que corresponde a un impacto mecnico sobre la bveda craneana que no produce alteracin del contenido craneano, y que
puede asociarse a dolor local. SIGNOS DE ALARMA, en la evaluacin prehospitalaria o la llegada a urgencias: Deterioro progresivo de la
conciencia (disminucin de Glasgow), signos de focalidad neurolgica, cefalea progresiva, vmitos explosivos recurrentes, agitacin
psicomotora, convulsiones, amnesia antergrada de ms de 30 min, cambio en el tamao de las pupilas, sospecha de herida craneal
penetrante, intoxicacin, evidencia clnica o radiolgica de fractura de crneo, sat. O2 <80%, hipotensin. SALUD PUBLICA: Constituye
una de las principales causas de mortalidad e incapacidad en la poblacin menor de 40 aos. Cada ao fallece 1,000 000 de personas
vctimas de trauma craneal severo. Los accidentes con vehculo automotor son la principal causa de traumatismos. El trauma causa
150,000 muertes en EU y un tercio se acompaa por TCE severo. Entre 2-10% de pacientes con TCE tienen asociada lesin cervical.
PATOGENIA: Se distinguen 2 mecanismos bsicos: la colisin o traumatismo directo, en el cual actan como formas lesivas la energa
cintica y la deformante. Y el traumatismo indirecto en el que no existe contacto previo, cabeza/objeto agresor, siendo las fuerzas
lesivas la aceleracin angular pura y la hiperpresin transmitida pura. El impacto mecnico origina la degeneracin neuronal mediante
3 mecanismos bsicos: 1. Mecanismo lesional primario, son lesiones nerviosas y vasculares producidas inmediatamente por la agresin
biomecnica, las lesiones resultantes de la agresin primaria son: fracturas craneales, contusiones, laceraciones, hematomas
intracerebrales, lesin axonal difusa. 2. Mecanismo lesional secundaria; se destacan el edema cerebral (con el trauma se activa el
edema vasognico, que ocurre por aumento de la permeabilidad de los capilares, con ruptura local de la barrera hematoenceflica
(BHE), aumento del lquido intersticial, y el edema citotxico, que ocurre por dao isqumico de la clula nerviosa), isquemia local,
gluclisis anaerobia, edema de los astrositos, activacin de
cascadas enzimticas y de las endotelinas, por formacin y
liberacin de radicales libres de oxgeno. En los traumatismos
cerrados, el impacto de las fuerzas se produce en las zonas donde
el crneo se pone en contacto directamente con el encfalo, pudiendo producir lesiones en la zona
de impacto (lesin por golpe) y en la zona diametralmente opuesta (lesin por
contragolpe).,tambin lesiones por cizallamiento, las lesiones resultantes son; hipotensin,
hipercapnia, hipoxemia, hipertermia, hipoglucemia, acidosis, hiponatremia, hipertensin
intracraneal, hematoma cerebral tardo, edema cerebral, convulsiones, vasoespasmo. 3. Mecanismo
lesional terciario; engloban una serie de procesos neuroqumicos y fisiopatolgicos complejos,
concatenados, con posibilidad de retroalimentacin positiva, entre s que inician inmediatamente
tras el TCE. CLASIFICACION: En la escala de coma de Glasgow se puede obtener una calificacin
mxima de 15 y un mnimo de 3. Un paciente que presenta cualquiera de los siguientes signos debe
considerarse que sufre un TCE severo: Anisocoria, dficit motor localizado, fractura abierta del
crneo con exposicin de masa enceflica o salida de LCR, deterioro neurolgico, fractura deprimida
de la bveda del crneo. Otra clasificacin es en: BAJO RIESGO; Asintomtica, cefaleas, mareos, hematoma, laceracin o scalp de cuero
cabelludo, ausencia de criterios de moderado o alto riesgo. Actitud a seguir; Si no presentan otras lesiones asociadas que requieran
ingreso hospitalario u observacin se envan a su domicilio siempre que una segunda persona pueda observar la evolucin del paciente
en las prximas horas. Debe informarse por escrito sobre signos o sntomas de alarma, ante la presencia de las cuales debe consultar de
nuevo al hospital. MODERADO RIESGO; Historia de perdida transitoria de la conciencia, intoxicacin por alcohol y drogas, cefalea
progresiva, vmitos persistentes, amnesia peritraumatica, politraumatismo que impide adecuada valoracin clnica del TCE,
traumatismo facial severo, sospecha de nio maltratado, edad menor de 2 aos excepto lesin trivial. Actitud a seguir; deben
permanecer en observacin al menos 24 hrs, pueden reducirse a 12 hrs la observacin si no hay sintomatologa neurolgica, TAC de
crneo normal, no existen otras lesiones asociadas que requieran ingreso hospitalario. Si existe sintomatologa neurolgica evidente, el
periodo de observacin debe prolongarse y valorar TAC craneal de control a las 12-24 hrs. ALTO RIESGO; Disminucin del nivel de
conciencia actual o progresivo no claramente debido a otras causas (metablico, epilepsia), signos neurolgicos de focalidad,
hundimiento o herida penetrante de crneo, sospecha de fractura de la base del crneo (otorrea, hemotmpano, rinorrea, hematoma
en anteojos, hematoma retroauricular), convulsiones postraumticas, respiracin irregular o apnica. Actitud a seguir Una vez
diagnosticados y estabilizados, deben pasar al rea de tratamiento definitivo (quirfano o UCI). Desde el punto de vista patolgico,
pueden existir tres tipos fundamentales de lesiones cerebrales: 1. Conmocin o concusin cerebral; caracterizada clnicamente por una
breve perdida de conciencia, con un corto periodo de amnesia seguida de una recuperacin rpida y total, sin ningn signo neurolgico
focal. No hay lesin estructural macroscpica del cerebro, tan solo se producen lesiones por estiramiento de los tractos axonales de la
sustancia blanca, con perdida reversible de su funcin, responsables de la prdida de conciencia transitoria. 2. Contusin cerebral;
sobre todo en polos frontales y temporales, por el contacto entre la superficie cerebral y el interior del crneo, que abarcan desde una
simple magulladura en una pequea rea cortical, hasta lesiones extensas, a menudo hemorrgicas, de gran parte de la superficie
cerebral, con dao en la sustancia blanca y el mesencfalo, clnicamente alteracin del nivel de conciencia, desde confusin, inquietud
y delirio y grados variables de coma, estos pacientes deben ser hospitalizados para observacin, dado el desarrollo tardo de edema
cerebral. Realizar TAC, valora necesidad de iniciar tratamiento para HIC. Las contusiones que producen efecto de masa requieren
ciruga urgente. 3. Lesin cerebral difusa; presencia de un coma prolongado de das o semanas, lesin frecuente con alta mortalidad. Su
diagnstico es presumible cuando en la TAC cerebral no se aprecia una lesin ocupante de espacio en un paciente con coma profundo,
adems de la situacin de coma suelen presentar posturas de descerebracin o decorticacin, y frecuentemente presentan signos de
disfuncin autonmica. DIAGNOSTICO: Los complementarios ms frecuentemente utilizados para decidir el diagnstico y tratamiento
de este tipo de traumatismo son los siguientes: Radiografa de crneo: Se deben indicar tres vistas fundamentales: anteroposterior,
lateral y Towne. Las dos primeras nos permiten apreciar lesiones en la bveda craneal (fracturas, presencia de cuerpos extraos
intracraneales, desplazamientos de la pineal calcificada, distasis de suturas). La vista de Towne nos permite visualizar la fosa craneal
CLASIFICACION DE TCE SEGN OMS
Fracturas de crneo:
Fracturas de la bveda
Fracturas de la base
Fracturas de los huesos de la cara
Otras y las fracturas inclasificables
Mltiples fracturas que afectan el
crneo o la cara con otros huesos
Lesin intracraneal
Conmocin
Laceracin cerebral y contusin
Hemorragia subaracnoidea,
subdural y extradural
Hemorragias intracraneales
postraumticas inespecficas
Lesin intracraneal de naturaleza
inespecfica
GRAVEDAD DE TCE
Leve 14-15 puntos
Moderado 9-13 puntos
Severo Menos de 9 puntos
MANUAL DE TRABAJO DEL CURSO ENARM CMN SIGLO XXI
CURSO ENARM CMN SIGLO XXI TEL: 36246001 Pharmed Solutions Institute PGINA 533

posterior, y parcialmente la proyeccin posterior de los peascos del hueso temporal.Angiografa carotdea: Es un estudio invasivo, que
requiere de cierta demora en su ejecucin, actualmente ha sido desplazada por la tomografa axial computarizada (TAC), aunque en los
centros que no cuentan con este ltimo recurso, mantiene toda su vigencia e importancia. Permite apreciar desplazamientos de los
patrones vasculares normales debido a lesiones intracraneales con efecto de masa (hematomas, higromas, contusiones). Tambin
permite visualizar zonas con vasoespasmo, fstulas vasculares y lesiones vasculares posteriores al trauma.Tomografa axial
computarizada: Constituye en la actualidad el estudio ecogrfico ms eficaz frente a este grupo de trastornos. Permite la visualizacin
directa de lesiones intracraneales, su volumen y forma, as como las estructuras que afecta, y la visualizacin indirecta de otras
lesiones, como el edema cerebral. A continuacin, se expone la clasificacin de las lesiones enceflicas detectadas por la TAC craneal en
los pacientes con TCE descrita por Marshall, tambin denominada como clasificacin del Traumatic Coma Data Bank (TCDB): Distingue 4
tipos de lesiones difusas y 2 tipos de lesiones focales: Clasificacin de los hallazgos de la primera tomografa tras el TCE: A) Lesin difusa
I; sin patologa visible. B) Lesin difusa II; cisternas visibles, con desviacin de la lnea media hasta 5mm y/o sin lesin mayor de 25 ml.
C) Lesin difusa III; cisternas comprimidas o ausentes, con desviacin de la lnea media hasta 5 mm y/o sin lesin mayor de 25ml. D)
Lesin difusa IV; desviacin de la lnea media mayor a 5 mm, sin lesin mayor de 25ml. E) Toda lesin evacuada quirrgicamente. F)
Lesin mayor de 25ml no evacuada quirrgicamente. TRATAMIENTO: Inicial de las situaciones de amenaza vital: 1. Asegurar la
permeabilidad de la va area con control de la columna cervical. 2. Oxigenacin y ventilacin adecuada. 3. Control de la hemorragia
externa y mantener la presin arterial. 4. Evaluacin del estado neurolgico. 5 Investigar otras lesiones traumticas. Objetivo del
tratamiento en TCE: Posicin de la cabeza a 30 sobre el plano horizontal, analgesia eficaz, normotermia, PaO2 >70mmHg,
normocapnia, presin arterial media >90mmHg, euvolemia, Hb >10mg/dl, osmolaridad plasmtica >290mOsm, glucemia <200mg/dl,
profilaxis de convulsiones precoces. Para la conmocin cerebral y sndrome postraumtico que pueden ser manejados por la guardia de
Ciruga: vigilar cada 4 horas posibles signos de alarma neurolgica (alteraciones de conciencia, pupilas y motilidad de extremidades), los
cuales deben ser tratados sintomticamente (analgsicos y antigravitatorios segn necesidad) durante perodos prudenciales de tiempo
en la sala de observaciones, y de ser preciso o requerirse, sern evaluados por la guardia de Neurociruga a solicitud de la guardia de
Ciruga General. Si el paciente presenta agitacin severa, debe ser sedado, de preferencia con Midazolam 0,10- 0,15 mg endovenoso en
bolo inicial, seguido por infusin endovenosa de 0,2- 0,4 mg x Kg/h. Puede utilizarse tambin morfina 0,10 mg endovenoso de inicio,
seguido por infusin de 1- 2 mg/h. Farmacologa de TCE; MANITOL, su efecto rpido sobre la PIC es especialmente til en situaciones de
urgencia, su accin es rpida y fugaz, obtenindose el efecto mximo a los 40 minutos de infusin en bolo, en urgencias, redujo con
mayor frecuencia la dilatacin pupilar y mejoro la evolucin a los 6 meses, tienden a mostrar mayor sobrevida y mejor pronstico.
Indicaciones: a todo paciente hemodinamicamente estable con signos de herniacin cerebral (anisocoria, signos de decorticacin-
descerebracin) se le debe administrar manitol en bolo rpido, realizando a continuacin una TAC craneal urgente. Dosis se comienza
con un bolo de 1-2 g/kg, se puede repetir cada 6 hrs. FUROSEMIDA, su ventaja sobre el manitol es su efecto sobre la osmolaridad es
menos marcado, aunque puede producir trastornos electrolticos, la desventaja es que es menos efectivo en el tratamiento del edema
cerebral. Indicaciones: lesiones hemorrgicas cerebrales, debera reservarse para pacientes en los que la expansin de volumen
producida por el manitol pudiera resultar perjudicial (cardiopatas, insuficiencia renal). CORTICOIDES, son eficaces para disminuir el
edema cerebral en los procesos tumorales o inflamatorios pero en el edema, debido al traumatismo, no parecen ser tiles. La
administracin de pre-hospitalaria de sol.de cloruro de Na al 7.5% para trauma e hipotensin se asocia con un incremento
significativamente mayor en la presin arterial comparado con la infusin de Ringer-lactato. Profilaxis antibitica, siempre en los
traumatismos abiertos y penetrantes, cuando existan signos clnicos cardiolgicos de fractura de la base de crneo, se emplearan
antibiticos de amplio espectro. Profilaxis de crisis convulsivas; se establecer precozmente un tratamiento de fondo preventivo de las
mismas siempre que la lesin sea supratentorial con afectacin del parnquima cerebral (hundimientos, contusiones, dislaceraciones,
hematomas intraparenquimatosos). Los anticomiciales usados son; fenobarbital (100mg c/8hrs) tanto enteral como parenteral y
fenitoina (100mg c/8 hrs). TCE grave; el exceso de tono simptico en la dinmica vascular cerebral y sus efectos sobre la PC debe ser
controlado con una adecuada sedacin y analgesia; opiceos 8fentanilo, morfina), benzodiacepinas, propofol. La intubacin debe ir
siempre acompaada de una adecuada sedacin, una correcta relajacin muscular ya que si esta no se consigue durante la maniobra de
la PIC esta aumentara, el relajante muscular ideal para estos pacientes es la succinilcolina. COMPLICACIONES: Hematoma extradural o
epidural: coleccin de sangre entre el crneo y la duramadre cuya causa ms frecuente es la lesin traumtica de la arteria menngea,
sospechar en paciente que ha sufrido TCE, que tras un periodo de 1-24 hrs entra en estado de coma, pudiendo haber dilatacin pupilar
del lado lesionado y hemiparesia contralateral. Este cuadro requiere ciruga inmediata. Hematoma subdural: Debidos a roturas de las
venas comunicantes entre corteza cerebral y duramadre, incidencia mayor en pacientes etlicos y ancianos. Si aparecen en las primeras
24 hrs son agudos, entre 24hrs y 2 semanas subagudos y crnicos cuando aparecen ms tardamente. El hematoma subdural agudo
requiere ciruga urgente. Hemorragia subaracnoidea: se acompaa a menudo de hematoma subdural concomitante o de una contusin
cerebral, el diagnostico se realiza mediante TAC y si es normal el diagnostico se realizara mediante la demostracin de un LCR
hemorrgico, no requiere tx., quirrgico urgente. Hematoma intraparenquimatoso: pueden manifestarse como lesiones rpidamente
expansivas o ser asintomticas, en la mayora de los casos existe fractura craneal asociada por golpe o contragolpe. FRACTURAS
CRANEALES: Las fracturas de la bveda craneal no presentan un cuadro clnico propio, sino que dependen de la afectacin o no de
estructuras neurolgicas. Son diagnosticadas con relativa facilidad mediante la ayuda de los rayos X (en cambio, las fracturas de la base
del crneo generalmente no son visibles por estos); poseen un cuadro clnico propio de cada una de sus variedades, lo que favorece la
realizacin del diagnstico clnicamente: Fosa craneal anterior: Hematoma periorbitario uni o bilateral: en gafas o "en espejuelos" o
signo del mapache (si es bilateral, afectando las dos hemifosas anteriores), o del oso Panda (si es unilateral, que afecta slo una
hemifosa anterior). Rinorragia o rinorraquia (uni o bilateral, en dependencia de si afecta una hemifosa anterior, o a las dos). Es
importante establecer diagnstico diferencial, al exponer el goteo en una sbana blanca o un papel, suele separarse el componente
hemtico del lquido cefalorraqudeo, signo del salvavidas o del huevo frito. Puede existir anosmia (por lesin del primer par
craneal, generalmente a nivel de la lmina cribosa del etmoides, menos frecuente por afectacin del bulbo o del nervio olfatorio),
alteracin de la motilidad de la musculatura periocular (pares craneales III-IV-VI), defecto visual por lesin del nervio ptico (II par
craneal), anisocoria perifrica, o desigualdad pupilar con midriasis unilateral, seudo-anisocoria central por lesin de ramos del III par
o lesin del esfnter constrictor del iris, o por la presencia de hematoma intraorbitario extracraneal, que comprima o irrite al ganglio
ciliar en el polo posterior del ojo). En resumen, los signos y sntomas suelen estar ms frecuentemente ocasionados por lesin de los
MANUAL DE TRABAJO DEL CURSO ENARM CMN SIGLO XXI
CURSO ENARM CMN SIGLO XXI TEL: 36246001 Pharmed Solutions Institute PGINA 534

pares craneales I-II-III-IV-VI. Fosa craneal media: Sensacin de "odo ocupado" o "lquido en el odo", hipoacusia, hemotmpano durante
la otoscopia, otorragia u otorraquia, signo de Batlle (hematoma retroauricular, o pre-esternocleidomastoideo generalmente visible
despus de transcurridas las primeras 24 h), vrtigos intensos por lesin de los conductos semicirculares, paresia o parlisis facial
perifrica por lesin del nervio facial en su curso a travs del peasco (ms frecuente en las fracturas transversas). En resumen, los
signos y sntomas suelen estar ms frecuentemente ocasionados por lesin de los pares craneales VII-VIII. Las fracturas de fosa
posterior no tienen un cuadro clnico bien definido, y pueden ser visualizadas mediante rayos X (preferiblemente en la vista de Towne).

CASO CLINICO


























































MANUAL DE TRABAJO DEL CURSO ENARM CMN SIGLO XXI
CURSO ENARM CMN SIGLO XXI TEL: 36246001 Pharmed Solutions Institute PGINA 535

TRAUMA RAQUIMEDULAR
CIENCIAS BASICAS: Toda lesin traumtica que afecta, conjunta o aisladamente, las diferentes estructuras de la columna vertebral en
cualquier de sus niveles. Es toda lesin de la columna vertebral asociada a una lesin neurolgica que puede comprometer a la medula
espinal, races o cauda equina. Esta lesin neurolgica puede ser reversible o irreversible. Factores predisponentes: Espondilosis
cervical, inestabilidad atloaxoidea, malformaciones vertebrales congnitas, osteoporosis, artropatas inflamatorias. Recuerdo
anatmico: 33 vrtebras: 7 cervicales, 12 torcicas, 5 lumbares, 5 sacras, 4 coccgeas, 23 discos intervertebrales. Trauma
vertebromedular: Transmisin abrupta de energa sobre la columna vertebral y su contenido. Lesin medular: Alteracin temporal o
permanente de la funcin motora, sensitiva o autonmica. SALUD PBLICA: La enfermedad traumtica es la 4 causa de muerte. 20-30
casos/100.000 habitantes/ao. Mortalidad global 4%. Los traumatismos ocasionan severas incapacidades y secuelas invalidantes
permanentes. Ms 1/3 accidentes de trfico. Ms de 1/3 politrauma: 5-15% dao neurolgico, 30% otras lesiones, 5% fracturas ocultas.
Distribucin anatmica: 55% cervical, 10% torcica, 15-20% toracolumbar, 15-20% lumbosacra. 5% de los pacientes con TCE, tienen
TRM. 25% de TRM tienen TCE. PATOGENIA: La zonas ms vulnerables son la columna cervical baja y la columna toracolumbar, dos
regiones ms predispuestas en el sentido que son zonas de mayor movilidad en movimientos de flexoextensin y rotaciones, por lo
tanto son las zonas donde actan los vectores luego de la aplicacin de alguna fuerza que daa las estructuras ya sean seas o
discoligamentarias de la columna vertebral. Entre las causas ms frecuentes est el trauma automovilstico, generalmente asociado al
alcohol. Estos dos factores estn presentes en por lo menos 50% de los casos de trauma espinal. Otras causas son los accidentes
industriales, deportivos y el trauma como resultado de la inseguridad: heridas cortopunzantes y heridas por arma de fuego. Los
mecanismos de lesin pueden ser directos o indirectos, los directos son los menos frecuentes ocurren debido a la aplicacin de energa
en el mismo punto donde se produce la lesin de columna vertebral. Los mecanismos indirectos, los ms frecuentes, producen lesin
no necesariamente en el sitio de aplicacin de la energa. Existen 4 mecanismos principales de trauma espinal: 1. Extensin, 2. Flexin,
3. Rotacin, 4. Compresin vertical o carga axial. Estos son movimientos tolerados normalmente por la columna y las estructuras
nerviosas espinales siempre y cuando se mantengan dentro de rangos fisiolgicos. De lo contrario se presenta disrupcin anatmica y
lesin estructural del continente (columna) y contenido (medula-races nerviosas). Con frecuencia en una sola lesin espinal intervienen
dos o ms de los cuatro mecanismos. El trauma espinal producido por flexin forzad, la posibilidad de lesiona es causa frecuente de
lesin sea, ligamentaria y de medula y races nerviosas. La flexin forzada asociada o no a carga vertical (axial) se presenta por
ejemplo en casos de clavados en piscina o ros pandos. La hiperextensin puede producir lesin espinal, en esta se produce
disminucin del espacio intraespinal. En pacientes de edad avanzada en los cuales por cambios degenerativos el espacio interior del
canal espinal ya esta disminuido, la posibilidad de lesin de las estructuras contenidas dentro del canal espinal aumenta. Desde el
punto de vista fisopatologico hay eventos despus del trauma espinal que llevan finalmente a la desintegracin de la membrana
celular, es decir, a la peroxidacin lipdica de la membrana celular. Algunos de estos eventos son la entrada de calcio al espacio
intracelular, el aumento de radicales libres, ac. Araquidnico, prostaglandinas y tromboxano, la liberacin de endorfinas y aumento de
norepinefrina. La peroxidacin lipdica lleva a desctruccion celular irreversible. DIAGNOSTICO: Sospechar traumatismo raquimedular en
todo paciente politraumatizado, inconsciente o intoxicado. El examen neurolgico es de gran importancia para determinar tanto en el
paciente consciente como en el inconsciente, si existe compromiso neurolgico de medula, de races nerviosas o de ambas, como
resultado de trauma espinal; se debe valorar la fuerza, sensibilidad, reflejos, compromiso de esfnteres y deformidad espinal. Las
lesiones medulares pueden ser completas o incompletas. Indicios de trauma de columna: Dolor, hematomas, prdida del patrn
respiratorio, hipotensin y bradicardia, globo vesical, hipotona de esfnter anal, priapismo, debilidad, hipoestesia. Se pueden
manifestar como sndrome medular anterior, central, posterior o de hemiseccin medular. La seccin medular completa se caracteriza
por prdida total de movimientos, sensibilidad, reflejos y control de esfnteres. En su grado ms extremo, el trauma espinal se asocia a
seccin anatmica o fisiolgica de la medula espinal, con cuadro clnico como shock espinal. En el examen se encuentra parlisis
(paraplejia-cuadriplejia), nivel de anestesia, arreflexia, ausencia de funcin esfinteriana y sock neurognico producido por desconexin
del sistema simptico (simpatectoma) y que se caracteriza por hipotensin arterial y bradicardia. Hacen parte de la lesin simptica
vasodilatacin, perdida de calor por esta razn e hipotermia. Una vez estabilizado el paciente desde el punto de vista hemodinmico y
respiratorio, el primer estudio que se debe realizar en el paciente politraumatizado es la radiografa lateral de columna cervical. Esta
determinara la presencia o no de lesiones traumticas de columna cervical en 90% de los casos y permitir establecer si se puede o no
inmovilizar. Las radiografas AP y transoral, pueden complementar, el estudio imagenolgico inicial. Opcionalmente en el paciente TCE,
que se tome TAC cerebral, se puede aprovechar este estudio para realizar de forma complementaria cortes de TAC de las 3 primeras
vertebras cervicales. La RMI, es un excelente mtodo diagnstico para lesiones espinales traumticas, puede mostrar contusin
medular, edema o seccin medular, una zona de hiperintensidad medular generalmente implica la presencia de sangre dentro de la
medula (hematomielia). De igual manera puede demostrar la presencia descompresin de medula y races por disco, hematoma
epidural, fragmentos de vertebra. TRATAMIENTO: Se debe seguir un protocolo secuencial: 1. Inmovilizacin desde el sitio y momento
del accidente. 2. Diagnstico clnico e imagenolgico. 3 intervencin farmacolgica para disminuir la severidad de la lesin inicial y las
secuelas neurolgicas 4. Reduccin de luxaciones, descompresin de tejido nervioso si hay fragmentos de hueso, disco, ligamento,
sobre races o medula espinal. 5. Fijacin espinal en caso de inestabilidad. 6. Rehabilitacin del paciente. Las lesiones traumticas de
columna que produzcan desplazamiento anormal de sus elementos requieren de reduccin. La inmovilizacin y reduccin de lesiones
espinales cervicales puede hacerse mediante la colocacin de halo craneano que tracciona el crneo e indirectamente la columna
cervical, alineando las vrtebras. En el trauma medular se debe ante todo evitar el dao secundario, el cual puede estar causado por
hipotensin arterial, bradicardia, hipoxemia, hipercarbia, acidosis y aumento de la viscosidad sangunea. Se han empleado compuestos
para disminuir o anular la peroxidacin lipdica. La metilprednisolona (30 mg/kg en bolo, 5.4 mg/kg/h por 23 hrs) es la sustancia ms
aceptada hasta el momento ya que tiene efectos inversos, disminuye la entrada de calcio a la clula, aumenta el flujo sanguneo e
inhibe a la PG 2 alfa y el tromboxano. El estudio NASCIS II demostr que la administracin de esta droga logra una mejor recuperacin
funcional motora y sensitiva. Su administracin constituye la base de la intervencin farmacolgica en los pacientes con trauma espinal.
Conservador: Traccin, inmovilizacin, rehabilitacin. Quirrgico: Remocin de fluidos o tejidos que comprimen (medula o racices),
fusin sea, inestabilidad (que requiere estabilizacin mediante fijacin o artrodesis), compresin medular. NEURONA MOTORA
SUPERIOR: Parlisis, hipertona (espasticidad), hiperrreflexia, babinski. NEURONA MOTORA INFERIOR: Parlisis, hipotona,
hiporrreflexia, atrofia. SINDROME MEDULAR ANTERIOR: Se caracteriza principalmente por lesin motora asociada a signos
MANUAL DE TRABAJO DEL CURSO ENARM CMN SIGLO XXI
CURSO ENARM CMN SIGLO XXI TEL: 36246001 Pharmed Solutions Institute PGINA 536

piramidales, con conservacin de diferente magnitud de la sensibilidad del paciente. SINDROME MEDULAR POSTERIOR: Se manifiesta
clnicamente por una mayor lesin de tipo sensitiva y de diferentes grados de conservacin de la funcin motora. SINDROME DE
BROWN-SEQUARD: Caracterizado por la lesin anatmica de la mitad de la medula espinal, se manifiesta clnicamente por pedida de la
funcin motora ipsilateral y perdida de la funcin sensitiva contralateral (dolor, termoalgesia). SINDROME DE SECCION MEDULAR
TRANSVERSA: Se caracteriza clnicamente por presentar una prdida total de la funcin sensitiva motora y de los reflejos desde el nivel
de la lesin hacia distal (no necesariamente se asocia a una seccin medular anatmica). La lesin de las races generalmente se
produce por una traccin por los diferentes mecanismos de lesin o por un desplazamiento de las vrtebras que produce un
estrechamiento del dimetro de los orificios de conjuncin con lo cual se produce una compresin de las races en diferentes
magnitudes. La lesin de la cauda equina se puede producir en forma total o parcial en relacin con los desplazamientos
anteroposteriores de la columna vertebral lumbar desde el nivel L2 hacia distal; tambin puede lesionarse la cauda equina por
protrusin hacia posterior y ocupamiento del canal raqudeo de fragmentos seos o fragmentos discales. SNDROME
CENTROMEDULAR DE SCHNEIDER: Paresia flcida en miembros superiores (afectacin de 2 neurona), disminucin termoalgsica en
miembros superiores, dolores por desaferentizacin en miembros superiores, sin afectacin de miembros inferiores o menor que en
extremidades superiores. Esta paresia es espstica (afectacin 1 neurona). MEDULAR CENTRAL (SIRINGOMIELIA): bilateral,
sensibilidad termoalgsica segmentaria, disociacin termoalgsica. Exploracin clnica: C5 flexin de codo (bceps), C6 extensin
mueca (ECRB, ECRL), C7 extensin codo (trceps), C8 flexor dedo medio (FDP), T1 abduccin meique, L2 flexin cadera (psoas iliaco),
L3-L4 extensin rodilla (cudriceps), L4 dorsiflexin tobillo, L5 dorsiflexin hallux, S1 flexin plantar (hallux, gemelos, sleo).

CASO CLINICO




TRAUMA FACIAL. CIENCIAS BASICAS: Trauma se define como el dao que sufren los tejidos y rganos por accin de una energa que
puede actuar en forma aguda o crnica. El trauma maxilofaciales aqul que compromete tanto partes blandas como seas de la regin
facial y ocurre en aproximadamente el 30% de los politraumatizados. Su riesgo es la muerte por asfixia, hemorragias y asociacin de
lesiones de columna vertebral y SNC. Las secuelas pueden ser estticas y funcionales. SALUD PUBLICA: La principal causa de este tipo
de pacientes politraumatizados son los accidentes de trfico, en el 40% de los casos; seguidos por los accidentes domsticos con un
20% de la totalidad; las cadas casuales se presentan en el 14%; los accidentes laborales suponen un 3% y otras causas el8% restante.
Existe un claro predominio por el sexo masculino presentando un porcentaje del 68%, con una edad media de 32 aos y un rango de
edad entre 20 y 45 aos. PATOGENIA: La energa involucrada en un trauma es directamente proporcional a la mitad de la masa y al
cuadrado de la velocidad (E= 1/2Mx V
2
). Con una masa constante, al doblar la velocidad, la energa liberada es 4 veces ms y cuando se
triplica es 9 veces ms. Segn la energa los traumatismos se clasifican en: baja energa: determina fracturas ms simples, con menor
desplazamiento disyunciones y escaso compromiso de partes blandas. Alta energa: fracturas ms complejas, con mayor
desplazamiento, extensin y conminucion, y con gran compromiso de partes blandas. Las fracturas son el resultado de una sobrecarga
mecnica (energa) que una fraccin de segundo supera la resistencia sea y determina disrupcin. Esta lesin establece la interrupcin
del flujo sanguneo tanto en el hueso (cabos de fractura), como en los tejidos adyacentes. DIAGNOSTICO: La base del diagnstico, en
una buena anamnesis y exploracin. Es importante preguntar sensaciones disestesicas o anestsicas faciales, caractersticas del dolor,
alteraciones subjetivas de la oclusin, alteraciones de la visin, obstruccin nasal, epifora, alteraciones auditivas. Inspeccin; heridas,
abrasiones, equimosis, orientan a zona de impacto. Identificar asimetras, falta de fuerza, deformidades. Palpacin; de ceflico a caudal,
terminando con una palpacin de la mandbula y de las regiones de las articulaciones temporomandiblares. Examen intraoral:
hematomas, equimosis, cuerpos extraos, ausencias dentarias, escalones seos y muy importante la relacin oclusal. Palpacin de
columna cervical, y tener siempre oculoscopio nasal. El examen gold standard del trauma facial grave es el scanner, en especial en los
que se sometern a ciruga. La radiologa convencional: 1) Radiografa de Waters: con cabeza hiperextendida, lo que permite
desproyectar la base del crneo del tercio medio facial. til para pirmide nasal, cuerpos cigomticos, maxilares superiores, arcos
cigomticos y rebordes infraorbitarios. Generalmente se completa con hueso propio de la nariz. 2) Radiografa de Towne: til para
regin condilea y subcondilea de la mandbula, como piso de orbita. 3) Radiografa de Hirtz: para arcos cigomticos. 4) craneo lateral y
PA. 5) placas de mandbula 6) ortopantomografia. TERCIO SUPERIOR: Se refiere a las fracturas frontales, nasales, naso orbito
etmoidales (NOE), rbita, complejo cigomtico maxilar, maxilar y paladar. FRACTURAS NASALES: Son las ms frecuentes (40-50%) de
las fracturas faciales. La mayora incluye el tercio distal de los huesos nasales, con el margen etmoidal intacto, generalmente las
desviaciones del eje nasal obedecen a desviaciones del septum por fracturas de este en forma exclusiva o asociadas a fracturas de los
huesos propios, el tratamiento de reduccin puede ser diferido hasta 12 dias. FRACTURAS DEL COMPLEJO NASO-ORBITO-ETMOIDAL:
Se localizan en el tercio medio central, con alteracin en la confluencia de los pilares vertical medial y transversal superior, as como de
sus extensiones posteriores a lo largo de la pared medial de la rbita y el piso. Lo que podra corresponder a una fractura conminuta del
pilar vertical maxilar medial, en especial de la fosa lagrimal, donde se inserta el ligamento acantal. TERCIO MEDIO: FRACTURAS
MAXILARES Le Fort I, II, III, NOE, nasales, naso maxilares y aisladas del maxilar. cigoma, arco cigomtico, complejo cigomtico maxilar y
combinadas o especiales (panfaciales). Rene Le Fort clasifico las fracturas maxilares en tres tipos, segn el trayecto de la lnea de
Fractura. LE FORT I: (horizontal) puede resultar de una fuerza dirigida de lesiones bajo en el borde alveolar del maxilar en direccin
hacia abajo. La fractura se extiende desde el tabique nasal hasta el piriforme lateral llantas, viaja horizontalmente por encima de los
pices de los dientes, cruza por debajo de la unin zygomaticomaxillary, y atraviesa la unin pterigomaxilar para interrumpir los platos
MANUAL DE TRABAJO DEL CURSO ENARM CMN SIGLO XXI
CURSO ENARM CMN SIGLO XXI TEL: 36246001 Pharmed Solutions Institute PGINA 537

pterigoideos.; LE FORT II: (Fractura piramidal) La lnea de fractura se extiende a travs de los huesos propios-nasales y el septum hacia
abajo y hacia atrs por la pared medial de la rbita, cruza el reborde infraorbitario y pasa por el arbotante cigomatico-maxilar. LE FORT
III: (Disyuncin craneofacial) Es una verdadera separacin de los huesos de la base del crneo. El trazo de la fractura pasa por la sutura
nasofrontal, por la pared medial de la orbita hasta la fisura orbitaria superior, de esta a la fisura orbitaria inferior y por la pared-lateral
de la rbita hasta la sutura cigomtico-frontal y cigomtico-temporal. Hacia atrs-se fracturan las apfisis pterigoides del esfenoides,
normalmente a un nivel superior al que aparecen en las otras fracturas de Le Fort. Clinica: Hematoma periorbitario bilateral o en
antifaz, epistaxisis, equimosis conjuntival. Aumento de la longitud del tercio medio facial (cara larga, cara de plato). Mala oclusin.
Mordida abierta anterior: contacto prematuro de los molares respecto al segmento anterior. Vigilar rinolicuorrea y otorrea.
Tratamiento: Cobertura antibitica y analgsica. Vigilancia de la va area, control de la hemorragia y remitir a centro especializado. Si
existe rinolicuorrea no realizar taponamiento nasal y prescribir tratamiento antibitico. FRACTURAS CIGOMATICO-MAXILARES:
Caracterizado por la presencia de hematoma, equimosis, y depresin de la eminencia malar, asociado o no a limitacin de la apertura
oral. FRACTURAS ORBITARIAS: Estas pueden ser de cualquiera de sus cuatro paredes, siendo ms frecuente el piso y las paredes lateral
(en fracturas malares) y medial, lo mas importante es efectuar estudio del globo ocular, para descratar heridas que puedan peligrar la
visin. En el segmento posterior de la rbita, se unen el piso con la pared medial, formando lo que se conoce como la zona clave, la cual
se ve frecuentemente fracturada, su diagnstico es por scanner, y es la causa ms frecuente de enoftalmo post traumtico, de tal
manera que se debe buscar en forma dirigida. TERCIO INFERIOR: FRACTURAS DE MANDIBULA: Muy frecuentes, el paciente
generalmente presenta impotencia funcional mandibular y/o alteraciones de la oclusin. Es un hueso de consistencia dura que
presenta tres zonas dbiles que son el cuello del cndilo mandibular, la zona canina-agujero mentoniano y el ngulo mandibular debido
a la confluencia de las ramas horizontal y vertical y la presencia de la muela del juicio. Es ms frecuente en varones jvenes entre 20 y
30 aos. La zona de fractura ms frecuente es el cndilo mandibular, seguido del ngulo mandibular y la regin parasinfiaria. Es
aconsejable la valoracin en conjunto con el odontlogo, por posibles fracturas dentoalveolares asociadas. Clinica: Sntomas: dolor,
tumefaccin, disconfort, hematoma, deformidad, movilidad anormal, crepitacin, salivacin, mal olor, mal oclusin y trismus Signos:
mala oclusin, movilidad del foco de fractura, disfuncin a la deglucin y masticacin, crepitacin y tumefaccin. Tratamiento:
Preservar va area, control de la hemorragia, sutura de heridas, limpieza de la cavidad oral, remitir a centro especializado, cobertura
antibitica y analgsica. TRATAMIENTO: Debe ser efectuado idealmente una vez que el paciente se haya estabilizado y se haya
realizado el estudio radiolgico adecuado, la ciruga facial se puede retrasar hasta 10 das. Indicaciones de traqueostomia: fracturas
panfaciales. Fractura conminuta de la mandbula. Edema masivo de tejidos blandos faciales (quemadura). Las hemorragias de partes
blandas, se recomienda compresin, ya que los pinzamientos a ciegas pueden producir graves daos a estructuras nerviosas. En el caso
de epistaxis, el taponamiento nasal anterior continua vigente. El taponamiento posterior puede efectuarse con gasas lo ms adecuado
o con sonda Foley, en caso de persistir ligadura arterial selectiva. Como norma general el paciente traumatizado facial asocia heridas de
las partes blandas de la cara, heridas que son complejas por su mecanismo de accin, su tratamiento y su repercusin esttica
posterior. Las heridas: en primer lugar, deben de ser lavadas profusamente con suero salino; un cuidadoso y exhaustivo examen para
intentar eliminar cuerpos extraos en su totalidad, bajo anestesia local las manchas de los bordes de las heridas tienen que ser
eliminados para evitar la aparicin del tatuaje postraumtico cepillado enrgico de los bordes de la herida. Posteriormente se procede
a eliminar aquellos fragmentos de tejido necrtico que puedan dificultar una correcta cicatrizacin, valorar lesiones de tipo nerviosos
por seccin de ramas motoras o sensitivas y lesin de conductos lacrimales y salivares y finalmente se realiza hemostasia y sutura de las
mismas. Profilaxis antitetnica y cobertura antibitica de amplio espectro. Analgsicos. Dada la riqueza vascular presente en el macizo
facial la sutura de las heridas en este territorio puede demorarse de 6 a 24 horas.
CASO CLINICO


















MANUAL DE TRABAJO DEL CURSO ENARM CMN SIGLO XXI
CURSO ENARM CMN SIGLO XXI TEL: 36246001 Pharmed Solutions Institute PGINA 538

TRAUMA TORACICO CERRADO Y ABIERTO:
CIENCIAS BASICAS: Es cualquier agresin o trauma sobre las paredes del trax que producir un dao en las estructuras slidas y partes
blandas comprendidas en la caja torcica. Los traumatismos torcicos pueden ser cerrados (contusos) o abiertos (penetrantes).
Traumatismo abierto: se denomina a lesin que rompe la integridad del tejido (atraviesa pleura parietal). Traumatismo cerrado: resulta
por aplicacin de energa que provoca lesin sobre los tejidos sin daar su integridad. SALUD PUBLICA: En Mxico los traumas cerrados
son mayoritariamente por accidente de trnsito, los penetrantes
son por arma blanca, aunque han aumentado las heridas por armas
de fuego. 8 de cada 100.000 son letales. Principales causas de
traumatismo torcico asociadas: Accidentes de trnsito (43%),
Suicidios (29%), Homicidios (22%.). CLASIFICACION: Segn el
estado hemodinmico: Normales o compensados
hemodinmicamente Presin Arterial: 90/60 , Frecuencia
Cardiaca: 110 , diuresis satisfactoria. Anormales o
descompensados hemodinamicamente Presin Arterial: 90/60 o ,
Frecuencia Cardaca: 120 o , oligoanuria o anuria. Ambos pueden
ser estables (se mantienen los parmetros con el correr del tiempo)
o inestables. Segn grado de penetracin torcica: Grado 1: No
comprometen pleura. Grado 2: Penetran pleura parietal, penetran
pleura parietal y parnquima pulmonar. PERFORANTES O
TRANSFIXIANTES (Entran y salen del trax y comprometen otras
cavidades). Grado 3: Penetran pleura, pulmn, mediastino o abdomen o regin cervical (Dos cavidades). Grado 4: Penetran pleura,
pulmn, mediastino y el otro hemitorax o abdomen o regin cervical (tres cavidades o regiones) Nos da gravedad en un Trauma
torcico: Impactos de alta energa: cada mayor a 6 metros. Impactos de alta velocidad. Pasajeros despedidos del vehculo. Atropello.
Lesin penetrante de cabeza, cuello, trax, abdomen o regin inguinal. Dos o ms fracturas proximales de huesos largos.
Quemaduras mayores al 15% de SC o que afecten cara o vas areas. Trax inestable. TIPOS DE TRAUMA TORCICO: Seis lesiones
rpidamente fatales (Se detectan en evaluacin primaria): Obstruccin de la va area. Neumotrax Hipertensivo. Neumotrax Abierto.
Taponamiento cardaco. Hemotrax masivo. Trax inestable. Seis lesiones potencialmente letales u ocultas (Detectadas en la
evaluacin secundaria): Lesiones traqueobonquiales. Ruptura diafragmtica. Lesin esofgica. Contusin pulmonar. Ruptura artica.
Contusin miocrdica. Lesiones no necesariamente letales: Neumotrax simple, evidente u oculto. Hemotorax simple. Enfisema.
Fracturas costales. Fractura de Esternn o Escpula. 1.-OBSTRUCCIN DE LA VA AREA; se puede producir por cuerpos extraos,
secreciones, sangre, el manejo puede ser invasivo o no invasivo, otra clasificacin es va area quirrgica y no quirrgica. Consisten
en el manejo manual de la va area, intubacin oro o nasotraqueal, cricotiroidotomia por puncin o quirrgica y la traqueotoma. 2.
NEUMOTORAX HIPERTENSIVO; Ocurre cuando una lesin permite la entrada de aire al espacio pleural durante la inspiracin, sin que
pueda salir durante la espiracin. Esto lleva al colapso pulmonar, con desplazamiento mediastinal hacia el lado opuesto, se disminuye el
retorno venoso por angulacin de las cavas, compromete el gasto cardaco y el otro pulmn.El diagnstico es clnico: taquipnea,
dificultad respiratoria aguda, tiraje supraclavicular e intercostal, hipersonoridad y ausencia de murmullo vesicular, distencin de venas
del cuello, desviacin de la trquea cianosis. Tratamiento, descomprimir inmediatamente, con puncin catter teflonado calibre 12 o
14, en 2 espacio intercostal lnea medio clavicular, con lo cual lo convierte en normotensivo. Proceder a la colocacin de avenamiento
pleural bajo agua en 4 espacio intercostal lnea medio clavicular. 3.- NEUMOTRAX A TENSIN; es el escape de aire hacia la cavidad
pleural, mas comn neumotrax espontneos, el mediastino y la trquea se desplazan hacia al lado opuesto, comprometiendo la
posibilidad de respuesta ventilatoria por parte del pulmn sano, y afectando el retorno venos. Clnicamente se manifiesta por dificultad
respiratoria, taquicardia, hipotensin, desviacin de la trquea, ausencia unilateral de MV, timpanismo del pulmn afectado,
ingurgitacin yugular y cianosis tarda. El tratamiento inicial consiste en insertar una aguja, en 2 EIC en LMC del hemitrax afectado,
siempre por el borde superior de la 3 costilla. El definitivo es la insercin de un tubo de trax en el 5 EIC anterior a la LAM, siempre
por el reborde costal superior de la 6 costilla, en el hemitrax afectado. 4.-NEUMOTRAX ABIERTO; Al producirse una herida en la
pared torcica el aire penetrar preferentemente por la herida al igualarse las presiones intratorcica y ambiental y ofrecer menor
resistencia al paso del aire por la herida. El tratamiento inicial ser cubrir la herida con un apsito fijado en tres puntas. El tratamiento
definitivo consiste en la instalacin de un catter intercostal y conectarse a un equipo de succion a 10-25 cmH2O. Si el neumotrax es
estable puede mantenberse bajo vigilancia y se reabsorber a un ritmo de 1.25% por dia. , distante de la lesin, y el cierre de la herida
ser quirrgico. Complicaciones: Inadecuada conexin drenaje-aspirador. Inadecuada colocacin del drenaje. Oclusin bronquial.
(Cuerpo extrao, cogulo, rotura). Roturas traqueobronquiales. Grandes laceraciones pulmonares. Severa disminucin de la
distensibilidad pulmonar. 5.- HEMOTRAX MASIVO: Es el resultado de la acumulacin de sangre en la cavidad pleural, igual o superior
a 1500 ml o mas de 200ml/h durante 4 hrs. La principal causa: es la lesin de vasos hiliares y mediastinicos generalmente por heridas
penetrantes. Clnicamente encontrar un paciente en shock, con colapso de los vasos del cuello por hipovolemia o con ingurgitacin de
estos por efecto mecnico de las cavidades. El tratamiento ser simultneamente con reposicin de volumen de forma agresiva
(cristaloides, coloides y sangre), y descompresin del hemitrax lesionado con un tubo de trax nico N 28-32 f. Se har toracotoma
s: Deterioro hemodinmico sin otra justificacin. Persiste un drenaje superior a 1500 ml en las primeras 12-24 horas. Drenaje superior
a 200 ml/hora en 4 horas. Persistencia de la ocupacin torcica (hemotrax coagulado). 6.- TAPONAMIENTO CARDIACO: Producto de
una herida penetrante, en su gran mayora, pero tambin puede aparecer por lesiones de los vasos pericardicos o traumatismo
cardaco en un traumatismo cerrado. Desde el punto de vista clnico se manifiesta por la trada de ec, que consiste en el hallazgo
de: aumento de la presin venosa central, disminucin de la presin arterial, apagamiento de los ruidos cardiacos. Ingurgitacin
yugular con la inspiracin en un paciente ventilando espontneamente es signo inequvoco de taponamiento cardiaco (signo de
Kussmaul). En el tratamiento, la pericardiocentesis por va subxifoidea es de eleccin en el prehospitalario, para descompresionar el
pericardio, basta extraer 15-20 ml, pero es una medida temporal, actualmente se postula que en l SU y Hospital no deben hacerse
pericardiocentesis a menos que sea para dar tiempo a la preparacin del pabelln. 7.- TRAX INESTABLE: El trax se torna inestable
cuando un traumatismo contuso produce fractura en 2 puntos de 4 o ms costillas. El paciente es incapaz de generar la presin
TRAUMA TORACICO CERRADO TRAUMA TORACICO ABIERTO
Asociada a compresin y aceleracin-
desaceleracin
Asociada a heridas por arma blanca
y arma de fuego
Hay fracturas costales mltiples Puede o no haber fracturas costales
Puede haber hemo o neumotrax
tardo (>24hrs del trauma)
Hemo o neumotrax inmediato.
El trauma de va area superior se
manifiesta como estenosis
El trauma de va area cursa con
gran escape areo
Tratamiento quirrgico: requerido en
menos del 10% de lesionados
Tratamiento quirrgico: requeridos
entre 15-30% de lesionados.
Herida de arma blanca: Punzantes,
cortantes
Heridas de proyectil de arma de
fuego: De baja velocidad (< 750 m/s)
De alta velocidad (> 751 m/s)
Miscelneas ( Otros elementos lesivos
internos o externos)
Accin directa ( golpe o choque
directo)
Accin indirecta: Compresin,
alteracin de la velocidad,
Aceleracin brusca, Desaceleracin
brusca, Torsin, Deslizamiento,
Inmersin
MANUAL DE TRABAJO DEL CURSO ENARM CMN SIGLO XXI
CURSO ENARM CMN SIGLO XXI TEL: 36246001 Pharmed Solutions Institute PGINA 539

negativa suficiente para manetener la ventilacin; la intubacin y la ventilacin con presin positiva son imperativos. Clnicamente
puede no ser detectado en primera instancia por la hipoventilacin reactiva al dolor, y por los movimientos del trax. El tratamiento se
basa fundamentalmente en una buena ventilacin, eventualmente mecnica, oxigenacin, y tratamiento para el dolor. Si el paciente no
est en shock la infusin de fluidos debe ser cuidadosa para evitar la sobrehidratacin y el consiguiente edema pulmonar. Un adecuado
y controlado balance hdrico. Cuando coexisten fracturas costales mltiples en varias costillas consecutivas se produce una inestabilidad
de la pared con movimiento paradojico y alteracin de la mecnica respiratoria, con la consiguiente hipoxia. La gravedad de la lesin es
directamente proporcional al grado de alteracin del parnquima pulmonar en combinacin con el dao de la pared. 8.- CONTUCION
PULONAR: Lesin del parnquima pulmonar que causa hemorragia y edema localizado, producto de traumas en los que hay rpida
compresin y descompresin del trax. Si se afecta un rea considerable es probable que se necesite asistencia mecnica, pero casi
nunca durante ms de 48-72 hrs. Se observa falla respiratoria tarda, lenta, progresiva y sutil, con o sin trax inestable.9.- CONTUCION
MIOCRDICA: Difcil de diagnosticar, se sospecha por alteraciones al ECG (arritmias, extrasstoles mono o bifocales, taquicardia sinusal
inexplicable, FA, bloqueo de rama, o claramente un infarto), Eco cardiografa bidimensional e historia compatible. El tratamiento
tratara la manifestacin clnica o la arritmia especfica. 10.- RUPTURA DE AORTA: Producto de traumatismos cerrados, por laceracin o
arrancamiento de los puntos de fijacin de la Aorta. Signos clnicos alertan el diagnstico: Mediastino ensanchado en Rx de Tx. Fx de
1 y 2 costillas. Desviacin y elevacin del bronquio principal, de la trquea y el esfago hacia la derecha. Depresin del bronquio
principal izquierdo. Opacidad pleural apical. Tratamiento: quirrgico (reparacin o implante). 11.- RUPTURA DIAFRAGMTICA:
Presente con ms frecuencia en el lado izquierdo ya que se carece de la proteccin del hgado. Se sospecha el diagnstico por la
presencia de intestino, estomago en el hemitrax izquierdo. Los traumatismos penetrantes por arma blanca o de fuego a veces pasan
inadvertidos y solo se detectan aos despus cuando aparece la hernia diafragmtica. 12.- LESIN TRAQUEOBRONQUIAL: Laringe
Diagnstico: ronquera, enfisema subcutneo y crepitacin palpable de fractura. Manejo, en caso de va area obstruida: instalacin de
IOT o Traqueotoma. Trquea: las lesiones penetrantes son ms obvias que las provocadas por trauma. Se asocia a lesin de esfago y
grandes vasos. Diagnostico a travs de broncoscopa. Bronquios: La lesin de un bronquio mayor es rara y mortal, y ocurren a 2-3 cm.
de la carina. Diagnstico: hemoptisis y enfisema subcutneo, y se sospecha en neumotrax a tensin con gran escape de aire y es
confirmado por broncoscopa. Una vez confirmado el diagnstico de lesin traqueobronquial el tratamiento es la reparacin
quirrgica. 13.- RUPTURA ESOFGICA: Se sospechara cuando exista neumo o hemotrax a izquierda sin fracturas costales, trauma
directo al esternn o epigastrio con dolor y shock no explicado, adems puede haber salida de partculas de contenido digestivo por
tubo de trax. Tratamiento: La conducta ser la reparacin directa, esofagostoma cervical de escape y yeyunostoma de alimentacin.
14. FRACTURAS ESTERNALES: Suelen ser transeversas, se encuentran en el manubrio o cerca de l y son dolorosas. Resulta esesncial
descratar la lesion de estructuras adyacentes, en especial el corazn. INSERCIN DE PLEUROTOMA; cuidados postoperatorios: El
paciente deber permanecer en el hospital hasta que el tubo torcico sea retirado. Debe quedar conectado a una trampa de agua
donde el pivote del sello de agua debe quedar sumergido a 2cm, y oscilar cuando est conectado al paciente. Mientras el tubo est
colocado en el trax del paciente, los enfermeros verifican con cuidado que no haya escapes de aire, dificultades o problemas al
respirar o necesidad de administrar oxgeno adicional al paciente. El paciente tosa y respire profundo para facilitar que los pulmones se
expandan de nuevo, ayudar con el drenaje y prevenir que los fluidos normales se alojen en los pulmones. ATENCIN Y MANEJO: Las
muertes relacionadas con Trauma, en la actualidad siguen una curva bimodal (2 picos de frecuencia diferentes). El primer pico, ocurre
dentro de la primera hora. El segundo pico en las primeras 24 a 48 horas despus del trauma. La ciruga de control de daos es en
etapas: PRIMERA ETAPA: Control de hemorragia y fuga area. SEGUNDA ETAPA: Resucitacin en la UCI, manejo agresivo de la
hipotermia, acidosis, coagulopatia. TERCERA ETAPA: Tratamiento definitivo de las lesiones. REGLA DE ORO (GOLD STANDARD): Efectuar
las mnimas reparaciones definitivas, usando tcnicas que sean rpidas y fciles, para acortar al mximo los tiempos quirrgicos de este
modo. En base a ATLS: A. Va area y control de columna cervical. Permeabilidad, estabilidad y seguridad de la va area (eventual IOT),
aspiracin de secreciones, fijacin de columna cervical firme y segura, cricotiroidotoma por aguja o quirrgica. B. Respiracin:
Oxigenacin, movimientos respiratorios, ventilacin asistida, oclusin de heridas torcicas abiertas, toracocentesis y drenajes torcicos.
C. Circulacin y control de hemorragias. Compresin directa de los sitios de hemorragia, evaluacin de los pulsos, masaje cardiaco
externo, instalacin de dos vas venosas perifricas proximales de grueso calibre para alto flujo, reposicin de volumen y uso de
frmacos endovenosos, analgesia y sedacin, monitorizacin cardiaca. D. Dficit neurolgicos. E. Exposicin corporal y abrigo. En el
caso de focos hemorrgicos en cavidades, es imprescindible su deteccin y esto se utiliza el FAST (Focused Abdominal Sonography for
Trauma), que permite la evaluacin del abdomen y del pericardio, del lquido pleural. La mayora de los traumatismos torcicos 75-85 %
se solucionan con maniobras salvadoras de vida y o procedimientos quirrgicos menores: 1) Toracocentesis 2) drenaje pleural 3)
Pericardiocentesis 4) Ventana xifopericrdica 5) Cricotiroidotoma 6) Traqueostoma. Alteraciones que requieren toracotoma urgente:
1. Fuga masiva de aire. Indica rotura de la trquea o de un bronquio principal. Ms de 80% de las lesiones se localizas a menos de 2-5cm
de la caria. 2. Taponamiento pericrdico en presencia de traumatismo. 3. Perforacin esofgica.

CASO CLINICO
Paciente de 43 aos, fumador habitual, acude a urgencias por dolor costal, despus de una cada de 3 metros de altura
aproximadamente. A su llegada al hospital el paciente presenta PA de 130/70 mmHg, FC de 65lpm y saturacin de oxgeno del 98%
basal. En la exploracin fsica destaca la presencia de crepitacin sea a la altura de la parrilla costal derecha baja, la radiografa de
trax es normal. A las 48h el paciente consulta nuevamente a urgencias por aumento del dolor torcico asociado a dificultad
respiratoria y empeoramiento del estado general. La PA fue de 90/60 mmHg, la FC de 105lpm y la saturacin basal del 89%. La
radiografa de trax demuestra derrame pleural importante y el estudio analtico muestra los siguientes valores: Hto del 26% Hb de 7
g/l, plaquetas de 430.000/mm3. Estudio de coagulacin: normal.

PREGUNTA
La primera maniobra en este paciente al recibirlo en urgencias es:

RESPUESTA
a.- Colocacin de sello pleural.
MANUAL DE TRABAJO DEL CURSO ENARM CMN SIGLO XXI
CURSO ENARM CMN SIGLO XXI TEL: 36246001 Pharmed Solutions Institute PGINA 540

b.- Intubacion orotraqueal.
c.- Iniciar con cristaloides.
d.- Paquete globular.

CASO CLINICO
Varn de 33 aos que sufri un traumatismo torcico severo tras atropello. Precis ventilacin mecnica y frmacos inotrpicos.
Presentaba un soplo de insuficiencia artica. Se realiz una tomografa computarizada (TC) torcica, que objetiv neumotrax y
fracturas costales bilaterales, sin datos de diseccin artica. El ecocardiograma transtorcico evidenci insuficiencia artica severa con
dilatacin ligera de la raz artica. En el ecocardiograma transesofgico (ETE) se observ una imagen filiforme, correspondiente a velo
coronariano derecho roto, que protrua hacia el tracto de salida del ventrculo izquierdo, e insuficiencia artica severa. En la pared
anterior de aorta ascendente proximal, haba una imagen de seudoaneurisma con rotura de capas ntima y media y se observaban
colgajos medio-intimales protruyendo hacia la luz.

PREGUNTA
La triada de Beck consiste en:

RESPUESTA
a.- Hipotensin, ruidos cardiacos velados y distensin venas del cuello.
b.- Hipertensin, edema facial y ruidos cardiacos velados.
c.- Hipotensin, torax inestable y disfagia.
d.- Ruidos cardiacos velados, distensin venas del cuello y disfona.

TRAUMA ABDOMINAL ABIERTO Y CERRADO:
CIENCIAS BASICAS: Lesin orgnica producida por la suma de la accin de un agente externo junto a las reacciones locales y generales
que provoca el organismo ante dicha agresin. Todo paciente puede presentar lesiones en mltiples rganos abdominales y, por tanto,
debe ser considerado como un paciente con traumatismo grave, desde el momento del ingreso en la unidad de urgencias. Un tercio de
los pacientes que requieren una exploracin abdominal urgente tienen un examen fsico inicial anodino, tener en cuenta que puede
tener un comportamiento impredecible y desestabilizarse en el momento ms inesperado. Importante conocer el mecanismo lesional
con el fin de anticipar las lesiones esperables. ANATOMA: Abdomen anterior: se define como el rea localizada entre una lnea
superior que cruza por las mamilas, los ligamentos inguinales y la snfisis del pubis como la lnea inferior, y las lneas axilares anteriores
lateralmente. Flanco: rea entre las lneas axilares anteriores y posteriores y desde el cuarto espacio intercostal hasta la cresta ilaca. El
espesor de la musculatura de la pared abdominal a este nivel, ms que las capas aponeurticas ms delgadas de la pared anterior,
acta como una barrera parcial a las heridas penetrantes, particularmente por arma blanca. Espalda: localizada atrs de las lneas
axilares posteriores, desde la punta de la escpula hasta las crestas ilacas, el espesor de la espalda y los msculos paravertebrales
actan como una barrera parcial a las heridas penetrantes. Anatoma interna del abdomen: Cavidad peritoneal: cubierto por la parte
baja de la parrilla costal, el abdomen superior incluye el diafragma, hgado, bazo, estmago y colon transverso. Debido a que el
diafragma en una espiracin total se eleva hasta el 4 espacio intercostal, las fracturas de costillas inferiores o heridas penetrantes en la
misma rea pueden involucrar estas vsceras abdominales. El abdomen inferior contiene el intestino delgado y el colon ascendente,
descendente y sigmoides. Cavidad plvica: rodeada por los huesos plvicos, corresponde a la parte baja del espacio retroperitoneal y
contiene el recto, la vejiga, los vasos ilacos, y en la mujer los genitales internos. Espacio retroperitoneal: contiene la aorta abdominal,
la vena cava inferior, la mayor parte del duodeno, el pncreas, los riones, los urteres, as como segmentos del colon ascendente y
descendente. Las lesiones en las vsceras retroperitoneales son muy difciles de reconocer porque el rea es de difcil acceso al examen
fsico y sus lesiones no son detectadas por medio del lavado peritoneal diagnstico, y difcilmente valoradas por la ecografa, adems
de ser de difcil exploracin fsica. SALUD PUBLICA: Traumatismos de los ms frecuentes, estimndose en 1 por cada 10 ingresos por
traumatismo en los servicios de urgencias. Las principales causas de muerte en los pacientes con traumatismo abdominal son: 1. Por
lesin de algn vaso principal, como vena cava, aorta, vena porta o alguna de sus ramas, o arterias mesentricas. Las lesiones
destructivas de rganos macizos, como hgado, bazo o rin, o sus asociaciones, pueden originar una gran hemorragia interna. 2.
Sepsis: la perforacin o rotura de asas intestinales o estmago, supone la diseminacin en la cavidad peritoneal de comida apenas
digerida o heces, con el consiguiente peligro de sepsis. Los trastornos de vascularizacin de un asa intestinal por contusin de la pared
intestinal o de su meso pueden manifestarse tardamente como necrosis puntiforme parietal y contaminacin peritoneal con sepsis
grave. CLASIFICACIN: Abiertos (penetrantes y no penetrantes), presentan solucin de continuidad en la piel. Cerrados; la piel no tiene
solucin de continuidad. PATOGENIA: Las principales causas de abiertos son las heridas por arma blanca (lesiones intrabdominales de
20-30%) y arma de fuego cuya frecuencia es creciente. La principal causa de cerrados son los accidentes de trfico. Otras causas son los
accidentes de trabajo, accidentes domsticos, accidentes deportivos, siendo estos mucho ms frecuentes que los abiertos.
TRAUMATISMO ABDOMINAL ABIERTO: Las heridas por arma blanca y de fuego de baja velocidad (< 600 m/seg) causan dao al tejido
por laceracin o corte. Ceden muy poca energa y el dao se localiza en la zona perilesional, afectando habitualmente rganos
adyacentes entre s, siguiendo la trayectoria de, objeto que penetra. Las heridas por proyectiles de alta velocidad (> 600 m/seg)
transfieren gran energa cintica a las vsceras abdominales, teniendo un efecto adicional de cavitacin temporal y adems causan
lesiones adicionales en su desviacin y fragmentacin, por lo que es impredecible las lesiones esperadas. TRAUMATISMO ABDOMINAL
CERRADO: Impacto directo; la energa cintica a los rganos adyacentes a la pared abdominal, puede provocar lesin. Desaceleracin:
mientras el cuerpo es detenido bruscamente los rganos intra abdominales animados an por la energa cintica tienden a continuar
en movimiento producindose una sacudida, especialmente acusada a nivel de los puntos de anclaje, vasos y mesenterio que sufren
desgarros parciales o totales. Compresin o aplastamiento: entre dos estructuras rgidas, estas fuerzas deforman los rganos slidos o
huecos y pueden causar su ruptura o estallido de estos. Este es el mecanismo tpico de lesin del duodeno, en un accidente de
automvil con impacto frontal, donde aquel es comprimido entre el volante y la columna vertebral. VALORACIN INICIAL: El objetivo
en evaluacin primaria es evidenciar o descartar lesiones de riesgo vital e instaurar las medidas necesarias de soporte vital para
MANUAL DE TRABAJO DEL CURSO ENARM CMN SIGLO XXI
CURSO ENARM CMN SIGLO XXI TEL: 36246001 Pharmed Solutions Institute PGINA 541

preservar la vida del paciente. El paciente con traumatismo abdominal debe ser considerado como traumatismo grave o
potencialmente grave y por lo tanto, el manejo de estos pacientes debe seguir las recomendaciones del ABC: A. Asegurar la
permeabilidad de la va area, con control cervical. B. Asegurar una correcta ventilacin/oxigenacin. Descartar neumotrax a tensin,
abierto, hemotrax masivo. Valorar la necesidad de soporte ventilatorio. Si no es necesario administrar oxgeno a alto flujo con
mascarilla (10-15 l/min). C. Control de la circulacin. Detener la hemorragia externa. Identificacin y tratamiento del shock.
Identificacin de hemorragia interna. Monitorizacin ECG estable. D. Breve valoracin neurolgica. E. Desnudar completamente al
paciente, controlando el ambiente y previniendo la hipotermia. DIAGNOSTICO: Valoracin clnica del estado de shock: aumento de la
frecuencia del pulso, pulso dbil y filiforme, piel plida, fra y sudorosa, disminucin de la presin del pulso, retardo en el relleno
capilar, alteracin de la conciencia, taquipnea, hipotensin y oligo anuria., dado que la hemorragia intraabdominal es la causa ms
frecuente de shock hipovolmico en estos pacientes. Inicialmente, se asume que el estado de shock es el resultado de la prdida aguda
de sangre y se la trata con una infusin rpida de volumen: un bolo inicial de 1-2 litros para un adulto de SF al 09 % o de solucin de
Ringer lactato. La restitucin por medio de catteres intravenosos perifricos de calibre grueso (14G 16G), dada su mayor rapidez de
canalizacin. El shock refractario a la infusin rpida de cristaloides sugiere sangrado activo y requiere de una laparotoma urgente.
Primeras medidas: Colocar sonda gstrica: cuyo objetivo es aliviar la dilatacin gstrica aguda, descomprimir y reducir el riesgo de
broncoaspiracin. Si existen graves fracturas faciales o la sospecha de una fractura de la base del crneo, la sonda debe introducirse por
la boca para evitar el riesgo del paso del tubo hacia el cerebro a travs de la lmina cribiforme. Canalizacin de dos vas venosas
perifricas con catter de gran calibre. Sonda vesical para aliviar la retencin de orina y descomprimir la vejiga, comprobar la
presencia de diuresis, su flujo horario, descartando previamente la presencia de lesin uretral. Se deben sacar muestras de sangre y
realizar una determinacin de BH, tiempos de coagulacin, gasometra, amilasa, niveles de alcohol, grupo sanguneo y pruebas
cruzadas. Evaluacin Secundaria: Historia clnica, a partir del propio paciente, de sus familiares y de los profesionales que han llevado a
cabo la atencin prehospitalaria. Mecanismo de produccin del traumatismo: a) En traumatismos cerrados es importante el tipo de
impacto, dao del vehculo, uso de sistemas de seguridad, el estado de otras vctimas. b) Para las heridas penetrantes, puede ser til
una descripcin del arma y de la cantidad de sangre perdida en el lugar del hecho. Tiempo de evolucin desde el trauma hasta la
recepcin del paciente. APP: alergias, patologas previas, medicacin habitual, ciruga previa, ingesta de drogas. Maniobras realizadas
por los profesionales de la atencin prehospitalaria: volumen infundido, vas canalizadas, necesidad de resucitacin cardiopulmonar.
Exploracin: Inspeccin; observar el trax, abdomen, espalda, pelvis, perin, espalada. Hay que observar las huellas en la piel y pared
de los puntos de impacto del agente agresor. Auscultacin; confirmar la presencia o ausencia de ruidos intestinales. La presencia de
sangre libre intra peritoneal o contenido gastrointestinal pueden producir un leo que produce una ausencia de ruidos intestinales. Las
lesiones en estructuras adyacentes, por ejemplo, costillas, columna o pelvis, tambin pueden producir leo. Percusin; detectar
matidez (presencia de lquidos) en caso de hemoperitoneo; timpanismo (presencia de aire) si hay dilatacin gstrica o desaparicin de
la matidez heptica por neumoperitoneo. Palpacin; debe repetirse peridicamente, por lo que debe ser realizado por la misma
persona para poder evaluar las diferencias que se originen. Primero, debe dirigirse al plano parietal buscando la presencia de
hematomas, o contusiones musculares. Luego debe investigar la presencia de contractura abdominal refleja, que es un signo fiable de
irritacin peritoneal, al igual que el signo del rebote positivo. Finalmente, hay que realizar una palpacin ms profunda buscando la
presencia de puntos o zonas dolorosa cuya topografa nos oriente a relacionarlas con los posibles rganos lesionados. Evaluacin
estabilidad plvica; La exploracin del anillo pelviano debe realizarse mediante una cuidadosa compresin lateral y antero posterior,
siendo dolorosa cuando hay fractura plvica. Una fractura de pelvis puede ser causa de shock hipovolmico, en ocasiones muy severo.
Produce hematoma perineal y genital a las 24-48 horas del traumatismo y puede acompaarse de hematoma retroperitoneal y
ausencia de hemoperitoneo. Examen del perin y genitales; lesiones externas y la presencia de signos de lesin uretral como sangre en
el meato, hematoma escrotal o desplazamiento hacia arriba de la prstata. La laceracin de la vagina puede ocurrir en heridas
penetrantes o por fragmentos seos de una fractura plvica. PRUEBAS COMPLEMENTARIAS: Analtica: bioqumica, hemograma,
tiempo de coagulacin, pruebas cruzadas, niveles de alcoholemia y anlisis de orina. ECG y monitorizacin de constantes vitales.
Radiografa de abdomen. Radiografa de trax: es importante para descartar la presencia de hemotrax, neumotrax o fracturas
costales. Radiografa de pelvis. Exmenes complementarios: Lavado peritoneal diagnstico (LPD); procedimiento invasivo que puede ser
realizado de forma rpida presenta una sensibilidad del 68% y una especificidad del 83%. La indicacin principal del LPD son Hallazgos
abdominales equvocos. Exploracin fsica no realizable por traumatismo raqudeo concomitante o alteracin de la conciencia
(traumatismo crneo enceflico o txico). Imposibilidad de reevaluacin contina. Hipotensin inexplicable. Prdida progresiva de
sangre (descenso progresivo del hematocrito). Puede realizarse mediante un mtodo abierto o cerrado. Un LPD negativo no excluye la
presencia de lesiones retroperitoneales o desgarros diafragmticos. El ultrasonido puede detectar presencia de hemoperitoneo, es un
medio rpido, no invasivo y seguro en el diagnstico de lesiones intra abdominales (cerrada o penetrante) y puede ser repetido
frecuentemente. Es ms sensible que el lavado peritoneal diagnstico para la determinacin de lesiones de vsceras macizas, aunque no
lo es tanto como la TAC. Su indicacin es absoluta en casos de embarazo, cicatrices abdominales por cirugas previas y alteracin de la
coagulacin. La TAC requiere el transporte del paciente a la sala de rayos X, administracin oral e intravenosa de contraste. Consume
tiempo y es utilizado nicamente en pacientes hemodinmicamente estables en los que no existe la indicacin inmediata de
laparotoma. La TAC proporciona informacin relativa a la lesin especfica de un rgano en particular y tambin puede diagnosticar
lesiones en el retroperitoneo u rganos plvicos que son difciles de evaluar en la exploracin fsica o en el LPD. Laparoscopia la utilidad
de la laparoscopia efectuada bajo anestesia local para identificar lesiones diafragmticas y cuantificar la cantidad de sangre
intraperitoneal. MANEJO DEL PACIENTE TRAS LA VALORACIN INICIAL: 1. Traumatismo Abdominal Cerrado, Hemodinmicamente
inestable: (imposibilidad de mantener una TA sistlica por encima de 90 mm Hg, frecuencia cardiaca inferior a 100 lpm o diuresis de 50
ml/h adulto) Si el paciente presenta signos abdominales patolgicos (distensin abdominal, peritonismo, neumoperitoneo en la
radiografa simple), entonces la indicacin de laparotoma debe ser inmediata. Si los signos abdominales son dudosos y el paciente
presenta un traumatismo craneoenceflico o espinal severo, alteraciones de la conciencia por toxicidad, traumatismos toraco-
abdominales, debemos llevar a cabo un estudio rpido del abdomen que nos ayude a descartar la presencia de patologa abdominal,
fundamentalmente lquido libre. Para ello disponemos de dos pruebas que no son excluyentes: Ecografa abdominal. Lavado
peritoneal diagnstico. Otras pruebas radiolgicas que precisan mayor infraestructura y tiempo, como la TAC, no son posibles en el
paciente inestable. Hemodinmicamente estable. Se deben tener en cuenta las siguientes premisas a la hora de manejar un paciente
MANUAL DE TRABAJO DEL CURSO ENARM CMN SIGLO XXI
CURSO ENARM CMN SIGLO XXI TEL: 36246001 Pharmed Solutions Institute PGINA 542

con un traumatismo abdominal cerrado: Las vsceras macizas se lesionan con ms frecuencia que las huecas. Traumatismo Abdominal
Abierto; Arma Blanca. - Si el paciente est inestable o presenta signos de irritacin peritoneal, debe ser sometido a una laparotoma
urgente. - Si est estable hemodinmicamente y no presenta signos de irritacin peritoneal, la primera maniobra que se debe realizar
es la exploracin del orificio de entrada del arma, comprobando si la herida es penetrante o no. Arma de Fuego; Puesto que la
trayectoria de una bala es difcil de predecir y dado que el 80-90% de los traumatismos por arma de fuego se asociaran a una o ms
lesin visceral, el tratamiento de estos pacientes ser quirrgico, realizndoseles una laparotoma urgente. Recomendaciones basadas
en evidencia, Recomendaciones NIVEL I: Laparotoma exploratoria est indicada para pacientes con LPD (+) TAC est recomendado para
pacientes hemodinmicamente estables con un EF equvoco (lesin neurolgica, lesiones extrabdominales). TAC es la modalidad
diagnstica de eleccin para manejo no operatorio de lesiones de vsceras slidas. En pacientes hemodinmicamente estables, LPD y
TAC son modalidades diagnsticas complementarias.

CASO CLINICO
Varn de 28 aos, trado a urgencias tras haber tenido una cada en bicicleta y con traumatismo cerrado periumbilical, refiri endo dolor
intenso, se observa distencin abdominal, adems se observa aire subdiafragmatico en RX, signos vitales TA 100/70, FC 105, FR 28, se
apresia diafortico con palidez de tegumentos.

PREGUNTA
Cual es la conducta a seguir.

RESPUESTA
a.- Lavado peritoneal.
b.- Restitucin volumtrica.
c.- Laparatomia exploratoria.
d.- Conducta expectante.

CASO CLINICO
Femenino de 66 aos de edad, sana cay de su propia altura. Su miembro inferior derecho estaba en flexin, rotacin externa y
abduccin con extensin de la rodilla durante la lesin. La paciente experimenta dolor en la cadera grave y no pudo incorporarse.
Describi la sensacin de que su cadera derecha haba sido desplazada de su articulacin momentneamente.

PREGUNTA
Cual de las siguientes medidas posoperatorias no es necesaria.

RESPUESTA
a.- Control estricto de liquidos.
b.- Analgesico, antibitico y enoxaparina.
c.- Realizar laboratorio y vigilar sangrado.
d.- Interconsulta a medicina interna.

RETINOPATIA DIABETICA (RD), RETINOPATIA HIPERTENSIVA:
RETINOPATIA DIABETICA (RD). CIENCIAS BASICAS: Es una microangiopata, que aparece como complicacin crnica de DM se
caracteriza por la presencia de lesiones microvasculares tpicas en la retina de una persona con diabetes, microaneuriosmas,
hemorragias, exudados duros, manchas algodonosas, alteraciones microvasculares, arrosariamiento venosos, neovasos y tejido
fibroso. Factores de aparicin: Control metablico pobre HbA1c, embarazo, HAS, nefropata, alcoholismo es un factores de riesgo,
antigedad de la DM >5aos, dislipidemia, obesidad. SALUD PUBLICA: Principal causa de ceguera legal 20-65 aos. Primera causa de
ceguera prevenible. Casi todos los pacientes con DM tipo II tendrn evidencia de RD a los 20 aos de evolucin y ms del 21%
presentan RD al momento del diagnstico. La RD es la causa de 5% de los casos de ceguera en el mundo. PATOGENIA: Se produce por
una microangiopatia el mecanismo ms probable es que la hiperglicemia produce alteraciones del metabolismo intracelular que llevan,
como resultado, a un aumento del Sorbitol. Esto produce el engrosamiento de la membrana basal endotelial y la prdida de los
Pericitos, los cuales son clulas que envuelven a los capilares retinales, proporcionndoles soporte y actuando como parte de la Barrera
Hematoretinal. La prdida de pericitos producira, a su vez, dos secuencias de eventos paralelas: a) Alteracin de la barrera
hematoretinal, filtracin al espacio extravascular, edema retinal, exudados lipdicos o creos formados por lipoprotenas. b) Formacin
de microaneurismas por debilidad estructural de la pared de los capilares retinales, activacin de la coagulacin en los
microaneurismas, trombosis intracapilar, obstruccin y cierre capilar. Lo anterior ser responsable de la produccin de isquemia retinal,
con el consecuente desarrollo de manchas algodonosas, (que corresponden a infartos de la capa de fibras nerviosas)
neovascularizacin, hemorragias y, en ltimo trmino, complicaciones tales como desprendimiento de retina traccional, glaucoma y, en
definitiva, ceguera. El crecimiento de neovasos, tanto a nivel retinal como en el iris, se producira debido a la liberacin por parte de la
retina isqumica de un factor soluble estimulador del crecimiento vascular (Factor de Crecimiento Vascular Endotelial, VEGF) y a su
efecto sinrgico junto a un factor de crecimiento vascular presente en la retina (Factor de Crecimiento de Fibroblastos Bsico, bFGF).
CLASIFICACION: En una etapa temprana o Retinopata Diabtica No Proliferativa (RDNP) y una ms avanzada o Retinopata Diabtica
Proliferativa (RDP). La RDNP se subdivide a su vez en: 1. Leve (Presencia de al menos un microaneurisma, hemorragia retineana,
exudado duro o exudado blando (mancha algodonosa)), 2. Moderada (Exudados duros, rosarios venosos, manchas algodonosas en
menor grado que en la severa), 3. Severa (Regla 4-2-1, 4 cuadrantes con microaneurismas o hemorragias, 2 cuadrantes con rosarios
venosos, manchas algodonosas, 1 cuadrante con IRMA) y 4. Muy severa (2 criterios). La RDP evoluciona en tres etapas de menor a
mayor severidad: temprana, de alto riesgo y avanzada. Esto est dado por la ubicacin y extensin de los neovasos, la presencia o
ausencia de hemorragia vtrea y la presencia o ausencia de desprendimiento de retina con compromiso foveal. La aparicin de estos
MANUAL DE TRABAJO DEL CURSO ENARM CMN SIGLO XXI
CURSO ENARM CMN SIGLO XXI TEL: 36246001 Pharmed Solutions Institute PGINA 543

neovasos es lo que define a la Retinopata Diabtica Proliferativa. Los neovasos se observan con mayor frecuencia en el nervio ptico o
cercano a las arcadas vasculares, pero se pueden encontrar en cualquier parte del fondo de ojo. La presencia de un desprendimiento de
Retina crnico en un paciente con RDP es un factor de riesgo para la neovascularizacin iridiana y para el Glaucoma neovascular
secundario, cuyo pronstico es muy sombro. El Edema Macular es un evento que puede suceder en cualquier momento de la
progresin de la RD los hallazgos son: minimo (algoi de engrosamiento de retina o exudados duros en el polo posterior de la macula),
moderado (engrosamiento de retina o exudados duros que se acercan a ala macula), severo (engrosamiento de retina o exudados
duros que involucran el centro de la macula). DIAGNOSTICO: Clnica; Microaneurismas (relacionados con zonas de baja perfusin.
Signos: puntos rojos), hemorragias (en forma de flama (arteriolas precapilares), intraretineanas: venas, forma de puntos), edema
macular (fuga capilar extensa), exudados duros (edema crnico localizado, macrfagos con depsitos lipdicos y lipoproteinas PE, por lo
general rodean microaneurismas), exudados algodonosos o blandos (infartos, obstruccin de flujo axoplsmico en la capa de fibras
nerviosas, inflamacin). Agudeza visual: Es de importancia saber que para la clasificacin de la retinopata diabtica no se toma en
cuenta la agudeza visual, sin embargo, cuando est se ve afectada nos habla principalmente de que la macula est comprometida.
Realizar exploracin de fondo de ojo bajo dilatacin pupilar por un oftalmlogo en DM tipo I: a partir de los 12 aos de edad. A partir
del 5to ao de diagnstico y cada ao, cuando el diagnostico sea posterior a la pubertad, debe comenzar cada 3 aos. En DM tipo II: al
momento del diagnstico, realizado por un oftalmlogo y debe repetirse anualmente. La ecografa es una prueba muy til para los
diabticos con medios opacos. Esta pruebe debe ser considerada cuando los medios no permiten la observacin mediante
oftalmoscopio indirecto. TRATAMIENTO: Educacin del Paciente que tome conciencia de su enfermedad, de sus riesgos potenciales, y
que acuda a controles peridicos con su diabetlogo y con su oftalmlogo. Durante esta etapa se debe optimizar el control metablico
de los pacientes, corregir la hiperlipidemia, lograr un adecuado control de la hipertensin arterial, tratar la nefropata y hacer controles
oftalmolgicos ms frecuentes en las mujeres embarazadas, en los casos que corresponda. Fotocoagulacin con Lser. Vitrectoma va
pars plana. Antiangiognicos. La Panfotocoagulacin consiste en hacer aplicaciones de lser trmico sobre la superficie retinal. Estas
quemaduras destruyen la retina en el lugar en que son aplicadas, creando una cicatriz. La racionalidad de este tratamiento se basa en
que, al destruir la retina isqumica, sta sera incapaz de producir el Factor de Crecimiento Vascular Endotelial, el que sera el
responsable de la formacin de los neovasos. La disminucin de la produccin de este factor soluble lograra la regresin de la
neovascularizacin existente y la prevencin de su desarrollo en el futuro. Este tratamiento no es inocuo. La panfotocoagulacin
disminuye en un 50% el riesgo de prdida visual severa, especialmente en los pacientes con RDP de alto riesgo. El edema macular ya
sea focal o difuso, en Edema Macular Clnicamente Significativo (EMCS) y Edema Macular No Clnicamente Significativo (EMNCS). Esta
definicin se basa exclusivamente en aspectos oftalmoscpicos que dicen relacin con la cercana del edema al centro de la fvea y no
considera la agudeza visual del paciente, es decir, un paciente puede tener visin normal y aun as tener un EMCS, el tratamiento del
EMCS focal con Fotocoagulacin Focal disminuye en un 50% el riesgo de prdida visual moderada en los pacientes tratados versus los
controles, y mejora la agudeza visual en el 16% de los pacientes. El EMCS difuso, hasta hace poco no tena un tratamiento efectivo.
Actualmente, con el advenimiento de la Triamcinolona intravtrea, como tratamiento de este tipo de edema, se han logrado mejoras
espectaculares en la disminucin del edema.

RETINOPATIA HIPERTENSIVA. CIENCIAS BASICAS: La Hipertensin Arterial (HTA), por el hecho de ser una enfermedad sistmica,
compromete rganos como el cerebro, el corazn, el rin y tambin el ojo. La retinopata hipertensiva es la principal manifestacin
ocular de la HTA, pero no la nica. El compromiso conjuntival, la coroidopata hipertensiva y la neuropata hipertensiva son otras
manifestaciones oculares de la HTA. Aunque parece haber una asociacin clara entre las alteraciones retinieanas y el aumento de las
cifras tensionales medias, parece que existen otros factores asociados a la retinopata como arteriosclerosis, edad avanzada,
enfermedad de la arteria cartida. SALUD PUBLICA: La prevalencia en pacientes hipertensos oscila entre 7.8-11%. PATOGENIA: Hay
varios aspectos condicionantes en la retinopata hipertensiva: 1. La gravedad de la hipertensin, 2. La duracin, 3. La rapidez de la
instauracin, 4. El estado previo del rbol vascular retiniano. Las manifestaciones oculares de la HTA son producidas por el dao que
esta causa en los vasos arteriales conjuntivales, retinales, coroideos y de la cabeza del nervio ptico. La HTA produce vasoconstriccin y
arterioloesclerosis. Vasoconstriccin: Los vasos arteriales retinales poseen la capacidad de autoregulacin y no estn comandados por
el tono simptico. Esta autoregulacin permite que en la eventualidad de un aumento de la presin arterial, las arteriolas retinales se
contraigan manteniendo constante el flujo. Por esto en pacientes con HTA Crnica, uno de los hallazgos ms tempranos es una
disminucin difusa del calibre arteriolar, perdindose la relacin AV normal de 2:3. La vasoconstriccin tambin puede ser focal. Esta
vasoconstriccin se produce por espasmo vascular reversible. En los casos de vasoconstriccin focal permanente, la causa podra ser
edema de la pared del vaso o reas localizadas de fibrosis. Arterioloesclerosis: Cuando observamos el fondo de ojo y vemos los vasos
retinales, lo que en realidad estamos viendo es la columna de sangre en el interior del vaso. La pared arteriolar es, en condiciones
normales, transparente. En la arterioloesclerosis se produce inicialmente un aumento del tejido muscular de la media y un aumento del
tejido elstico de la ntima, para luego ser reemplazada por fibras colgenas, la media, y sufrir engrosamiento hialino, la ntima. Esto
ocasiona, en un primer momento, que el brillo arteriolar que produce la luz incidente del oftalmoscopio en la pared del vaso, pierda su
aspecto normal (delgado y en la mitad de la columna de sangre), tornndose menos brillante, ms ancho y ms difuso. Este es el signo
ms temprano de arterioloesclerosis. En una segunda etapa, al aumentar la arterioloesclerosis, disminuye la visibilidad de la columna
de sangre y el brillo arteriolar se torna caf-rojizo. Es lo que los clnicos antiguos llamaban vasos en "Hilo de Cobre". En una tercera
etapa, la pared del vaso pierde por completo su transparencia, perdindose la visibilidad de la columna de sangre y adquiriendo el vaso
un aspecto blanquecino. Esto se denominaba vasos en "Hilo de Plata". Aunque clnicamente estos vasos parecieran estar sin flujo. En la
Hipertensin Maligna se produce necrosis fibrinoide de las arteriolas retinales, aun cuando la necrosis fibrinoide es ms comn en los
vasos coroideos. En estos casos se puede apreciar una estrella macular, que es el aspecto en estrella que adquiere la mcula por la
acumulacin de macrfagos llenos de lpidos, dispuestos en forma radial, en la capa plexiforme externa de la retina. DIAGNOSTICO:
Manifestaciones oftalmoscopicas: En la hipertensin crnica se produce esclerosis arteriolar por el efecto mantenido de la presin
sangunea aumentada sobre las paredes vasculares. Se genera un estrechamiento generalizado o localizado, se ha descrito como
arterias en hilo de cobre, arterias en hilo de plata, aumento de la tortuosidad de las arteriolas esclerosadas. Se produce oclusin de
arteriolas finas y generacin de manchas isqumicas retinianas profundas denominadas infiltrados algodonosos. En la hipertensin
aguda puede verse una imagen denominada espasmo arteriolar. Trasudados periarteriolares focales intrarretinianos (TPIF): Son muy
MANUAL DE TRABAJO DEL CURSO ENARM CMN SIGLO XXI
CURSO ENARM CMN SIGLO XXI TEL: 36246001 Pharmed Solutions Institute PGINA 544

precoces y especficos de la hipertensin maligna. Tienen forma redondeada u oval y el tamao es variable: desde puntiformes hasta la
mitad del tamao del disco ptico. Son de color blanco mate cuando son recientes y luego palidecen hasta resolverse. Se localizan en
las zonas profundas de la retina. Las lesiones se desarrollan totalmente en 2-3 semanas y tras su resolucin no quedan secuelas
oftalmoscpicas. Manchas isqumicas retinianas profundas (infiltrados algodonosos o exudados blandos): Son reas focales,
esponjosas, opacas. Tienen formas muy irregulares, comienzan como una pelcula griscea que se vuelve como una nube blanca y se
resuelve fragmentndose en manchas menores. Cambios capilares retinianos: microaneurimas y shunts arteriovenosos. Se observan
como vasos retorcidos y colaterales venosos. Cambios retinianos venosos: Signos de Gunn: la vena tiene dos zonas plidas, iguales a
ambos lados del cruce, con prdida del reflejo y adelgazamiento de los dos extremos. La vena se ve como partida. Signo de Salus: el
curso venoso sufre un cambio de trayecto. Aspecto de U, si el cruce es perpendicular y de S, si es oblicuo. Signo de Bonnet: junto al
cruce se observa una hemorragia o exudado. Hemorragias retinianas: tarda y poco frecuente, adoptan forma de llama. Edema retinal y
macular: los signos precoces de papiledema son rebosamiento de las venas, perdida de la pulsacin venosa, hiperemia de la cabeza del
nervio y borrado de los mrgenes del disco. Depsitos lipdicos retinianos (exudados duros): son amarillos, redondeados generalmente
y de bordes bien delimitadosLa aguda: se caracteriza por disminucin de la agudeza visual, la que puede ser desde visin borrosa (lo
ms frecuente) hasta la ceguera. Caractersticamente esta disminucin de agudeza visual es transitoria. El aspecto ms caracterstico
del fondo de ojo es una Vasoconstriccin Generalizada, se pueden apreciar Hemorragias Retinales, Manchas Algodonosas, Estrella
Macular, Manchas de Elschnig y Edema de Papila. Con mucha menor frecuencia se aprecian Desprendimientos de Retina Serosos. La
crnica: los hallazgos ms caractersticos en el fondo de ojo son: Vasoconstriccin difusa, Aumento del Brillo Arteriolar, Cruces AV
Patolgicos, Microaneurismas y Macroaneurismas. En casos ms graves pueden apreciarse: Manchas Algodonosas, Edema de Retina,
Hemorragias Retinales, Estrella Macular, Manchas de Elschnig y Edema de Papila. Como podemos ver, en las manifestaciones clnicas de
casos severos se aprecian signos de Coroidopata Hipertensiva y Neuropata Hipertensiva, adems de los de la Retinopata Hipertensiva.
COROIDOPATA HIPERTENSIVA: La coroides, a diferencia de los vasos retinales, est comandada por el tono simptico. En la HTA la
coroides sufre fenmenos de isquemia, observndose lbulos coroideos sin perfusin debido a necrosis fibrinoide de los vasos. Estas
zonas de ausencia de perfusin coroidea producen focos de necrosis isqumica en el epitelio pigmentario retinal suprayacente, que se
denominan Manchas de Elschnig. La isquemia coroidea tambin es responsable de desprendimientos de retina serosos por producir
isquemia en las capas externas de la retina. Las manchas de Elschnig, en su etapa aguda, se aprecian como manchas amarillentas
redondeadas en el epitelio pigmentario retinal. Posteriormente, en su etapa crnica, se aprecian como lesiones redondeadas
hiperpigmentadas en su centro y con un halo de hipopigmentacin en su periferia. NEUROPATA HIPERTENSIVA: Parece que es una
edematizacion secundaria a isquemia, seguida de perdida de axones y gliosis. La isquemia puede tener dos orgenes: 1) afeccin de los
vasos coroideos, ya que constituye el principal aporte sanguneo de la cabeza del nervio. 2) difusin de sustancias vasoconstrictoras a la
cabeza del nervio ptico a partir de la coroides peripapilar, lo que produce vasoconstriccin de los vasos dentro de la cabeza del nervio
ptico. CLASIFICACION: Aguda se observa principalmente en pacientes con preeclampsia o eclampsia, pero tambin es causada por
feocromocitoma o Hipertensin Maligna. Crnica, se observa en pacientes con hipertensin de larga duracion y, a diferencia de
aquellos que padecen la forma aguda, no presentan disminucin de la agudeza visual. Clasificacin de Keith, Wagener y Barker: Grado I
Hipertensin benigna. Estenosis o esclerosis moderada de las arteriolas retinianas. No sntomas. Buen estado de salud. Grado II
Retinopata hipertensiva ms marcada. Esclerosis moderada o marcada de las arteriolas retinianas. Reflejo luminoso arterial exagerado.
Compresin venosa en los cruces arteriovenosos. Tensin arterial mayor y ms mantenida que en el grupo I. Asintomticos. Buen
estado de salud. Grado III Retinopata angiospstica moderada. Edema retiniano. Manchas algodonosas y hemorragias. Esclerosis y
lesiones espsticas de las arteriolas retinales. Tensin arterial generalmente alta y mantenida (sintomtica). Grado IV Hipertensin
maligna. Todo lo anterior y edema del disco ptico. Las funciones renal y cardiaca pueden estar afectadas. Supervivencia reducida.
Clasificacin de Scheie: Grado I Aumento de reflejo luminoso arteriolar con compresin arteriovenosa mnima. Grado II Aumento de los
signos y desvo de las venas en los cruces arteriovenosos. Grado III Arteriolas en hilo de cobre y marcada compresin arteriovenosa.
Grado IV Arteriolas en hilo de plata y cambios importantes en los cruces arteriovenosos. TRATAMIENTO: No existe un tratamiento
oftalmolgico especfico para la retinopata hipertensiva hay que dar tratamiento de la enfermedad de base. Tiene ms urgencia el
tratamiento de la hipertensin arterial en la Retinopata Hipertensiva Aguda que en la Crnica, pero en ambas es igual de importante.

CASO CLINICO
Un hombre de 31 aos de edad, con antecedentes de diabetes mellitus insulino-dependiente de 12 aos de evolucin con parcial apego
a tratamiento, con alcoholismo ocacional y tabaquismo un paquete al dia, se presenta para su examen oftlmico. Historia oftalmolgica
fue notable por desprendimiento total de retina de su ojo derecho y la fotocoagulacin con lser de su ojo izquierdo debido a la
retinopata diabtica y edema macular diabtico. Examen de dilatacin del fondo del ojo derecho mostr etapa final retinopata
diabtica proliferativa con desprendimiento de retina traccional. El ojo izquierdo tena la neovascularizacin extensiva y la proliferacin
fibrovascular de la retina secundaria a la retinopata diabtica proliferativa avanzada. Estos nuevos vasos anormales mostraron fuga
extensa durante la angiografa con fluorescena.

PREGUNTA
Cual de los siguientes factores es de mayor importancia para el desarrollo de la retinopata diabtica.

RESPUESTA
a.- Ms de 5 aos de evolucin de la DM.
b.- Tabaquismo positivo intenso.
c.- Progresin rpida de la enfermedad.
d.- Depuracin de creatinina menos de 60 ml/ minuto.

CASO CLINICO
Un hombre de 56 aos de edad con diabetes presenta una historia de varios meses de disminucin de la visin en el ojo izquierdo. El
examen del ojo izquierdo revel la neovascularizacin retiniana extensa superior a la del disco ptico.
MANUAL DE TRABAJO DEL CURSO ENARM CMN SIGLO XXI
CURSO ENARM CMN SIGLO XXI TEL: 36246001 Pharmed Solutions Institute PGINA 545

PREGUNTA
Cual de los siguientes metas son factores que no modifican el curso de la enfermedad.

RESPUESTA
a.- Glucosa menor a 130 mg/dl
b.- Hb glucosilada menor a 7.
c.- Presin arterial menor a 130/80 mmHg.
d.- Uso profilctico de triamcinolona.

CASO CLINICO
Femenino de 42 aos de edad, con hipertensin arterial de 10 aos de evolucin actualmente con disminucin en el aclaramiento de
creatinina (<60 ml/min), con un evento de isquemia cerebral transitoria, presentando alteraciones del ritmo cardiaco bajo tratamiento,
su nivel de glucosa fue de 210 mg/dl, Colesterol 324 mg, triglicridos 450 mg, hemoglobina glucosilada de 9.2, acudi a consulta debido
a que presenta diminucin de la agudeza visual, asi como alteraciones en la campimetrial, fue enviada a evaluacin oftalmolgica con el
siguiente reporte. Estrechamiento y constriccin focal, edema retiniano, exudados y hemorragias.

PREGUNTA
Considerando el reporte cual es el estadio de Keith-Wagener.

RESPUESTA
a.- Keith-Wagener 1.
b.- Keith-Wagener 2.
c.- Keith-Wagener 3.
d.- Keith-Wagener 4.


URGENCIAS OFTALMOLOGICAS, PERFORACION, GLAUCOMA:
CIENCIAS BASICAS: Hay que distinguir dos grandes categoras de afecciones, ante un ojo rojo, el anlisis semiolgico debe pasar por el
examen de tres signos mayores: Dolor, fotofobia y agudeza visual. Normas generales: 1. No utilizar colirios anestsicos, salvo para
facilitar la exploracin. 2. No presionar el globo ocular traumatizado. 3. Tras el diagnstico de perforacin ocular, abstenerse de
continuar con la exploracin; ocluir ambos ojos con gasa estril sin instilar colirios ni aplicar pomadas. 4. Para la exploracin corneal es
de gran ayuda la instilacin de colirio de Fluorescena al 2% y posterior lavado con suero fisiolgico, ya que con la luz azul cobalto
evidenciar lesiones y defectos epiteliales dando una coloracin amarillo-verdosa. 5. Para una midriasis exploratoria (con colirio de
Tropicamida) o teraputica en pacientes de edad y, sobre todo hipermtropes, se debe explorar previamente el signo "del sol
naciente". Este signo consiste en la proyeccin tangencial desde el lado temporal de un foco luminoso que iluminar la mayor parte del
iris ante una cmara anterior profunda. 6. Utilizar guantes desechables ante procesos traumticos o infecciosos y, en todo paciente al
que se le vayan a tocar los prpados. 7. No realizar oclusin ocular en conjuntivitis o queratoconjuntivitis, ni en lceras corneales con
infeccin local. Hay que sealar cinco procesos oculares que son de asistencia inmediata, ya que el pronstico en cuanto a la funcin
visual y/o Integridad del globo ocular, depender de la rapidez en el diagnstico y tratamiento: Oclusin aguda de la arteria central de
la retina. Causticacion ocular por alcalis. Herida perforante del globo ocular. Glaucoma agudo. Endoftalmitis. OJO ROJO NO DOLOROSO:
1. Hiposfagma (Hemorragia subconjuntival): Habitualmente son espontneas, de tamao variable y curan sin tratamiento. Si es
secundario a traumatismo ocular, buscar signos de perforacin, descartarse siempre crisis de hipertensin arterial. 2. CONJUNTIVITIS:
Bacteriana; mas frecuente bilateral. Existe secrecin purulenta o mucopurulenta y lagrimeo, tx., aplicar colirio de Tobramicina cada 3
horas o aureomicina cada 2 horas, dura aprox 2 sem. Si es unilateral, purulenta, con dolor y adenopata preauricular, debe pensarse en
una conjuntivitis gonoccica y se tratar con colirio de Aureomicina cada 2 horas y Penicilina por va parenteral. Si es unilateral, con
poca secrecin, adenopata preauricular, presencia de folculos en conjuntiva tarsal, es decir, con caractersticas ms propias de una
conjuntivitis vrica, debe sospecharse conjuntivitis de inclusin (Chlamydia oculogenitalis). Vrica: unilateral o bilateral en pocos das,
poca secrecin, lagrimeo y presencia de folculos en las conjuntivas tarsales. Habitualmente, muestra adenopatas preauriculares y
submaxilares, cursar con fotofobia y dolor (queratoconjuntivitis). Etiologia los adenovirus, no existe tratamiento etiolgico eficaz, pero
es aconsejable el colirio de Aureomicina cada 2 horas y la pomada de Aureomicina por la noche. Alrgica: Cursa con secrecin
blanquecina fibrinosa (estril), abundante lagrimeo, presencia de papilas en la conjuntiva tarsal superior y, sobre todo, predomina el
picor como sntoma fundamental y caracterstico de esta afeccin. Es bilateral y, excepcionalmente, unilateral. Se trata con colirio de
Dexametasona cada 6-8 horas y antihistamnicos. OJO ROJO DOLOROSO: 1. EPISCLERITIS: Inflamacin de la episclera, enrojecimiento
violceo, localizado en un sector de la conjuntiva bulbar, por lo general con un ndulo amarillento en su centro. Realmente, la
dilatacin vascular afecta tanto a la episclera como a la conjuntiva. La instilacin de colirio de Fenilefrina no modifica esta congestin
profunda, sino que blanquea slo una hiperemia puramente conjuntival (diagnstico diferencial). Afecta a sujetos jvenes de forma
recidivante. Cursa con dolor a los movimientos oculares y, sobre todo, a la presin. Con frecuencia es de origen alrgico. A veces forma
parte de enfermedades granulomatosas y reumticas. Rara vez es signo de tumor intraocular. Tratamiento: Colirio de Dexametasona
cada 4 horas. Si no es suficiente, se recurrir a la corticoterapia sistmica. 2. QUERATITIS.- La infeccin o inflamacin de la crnea cursa
con dolor (que ser ms intenso cuanto mayor sea la denudacin de las terminaciones nerviosas), fotofobia, lagrimeo, blefarospasmo,
inyeccin periquertica, deslustramiento de la crnea y alteracin dispar de la agudeza visual. A veces existe hipopion (nivel de pus en
la cmara anterior), que traduce una reaccin uveal anterior. Tratamiento: Ciclopljico: Colirio de Atropina al 1% cada 8 horas.
Antibitico: Tpico: Cada hora. Gentamicina: 9.1 mg/ml. Ceftazidima: 50 mg/ml. Subconjuntival: Cada 12-24 horas. Gentamicina: 20 mg
(0.5 ml.). 3. IRITIS O IRIDOCICLITIS (UVEITIS ANTERIOR): Se trata de una inflamacin del tracto uveal anterior (iris y cuerpo ciliar). Sus
sntomas son: Dolor ocular y periocular (irradiaciones en el territorio del trigmino), lagrimeo, fotofobia y disminucin de la agudeza
visual (borrosidad). Signos: Inyeccin periquertica, precipitados en la cara posterior de la crnea, enturbiamiento del humor acuoso
MANUAL DE TRABAJO DEL CURSO ENARM CMN SIGLO XXI
CURSO ENARM CMN SIGLO XXI TEL: 36246001 Pharmed Solutions Institute PGINA 546

(efecto Tyndall), iris deslustrado, pupila oco reactiva a la luz y presencia de sinequias posteriores. La tensin ocular por regla general
est disminuida, medida teraputica urgente encaminada a mitigar el dolor y evitar o liberar las sinequias posteriores, debemos
administrar un colirio ciclopljico cada 6-8 horas y un colirio de Dexametasona cada 2 horas. 4. GLAUCOMA AGUDO: Cierre total y
sbito del ngulo camerular, que ocasiona una elevacin severa de la presin intraocular. Slo ocurre en ojos a menudo hipermtropes
y nunca miopes, de crnea pequea, de cmara anterior estrecha y de ngulo estrecho. Estas condiciones anatmicas y el aumento del
volumen del cristalino a lo largo de la vida, son responsables del bloqueo pupilar. Es ms frecuente en mujeres (4: 1) y ocurre,
generalmente, a partir de los 65 aos. Pueden desencadenar la crisis las emociones, la anestesia general, los derivados atropnicos, los
simpaticomimticos (jarabes para la tos), los antidepresivos, etc. La forma tpica; unilateral, aunque el ojo congnere presentar las
mismas condiciones de predisposicin. Tener dolor ocular intenso, similar a una cefalea retrobulbar, acompaado de nuseas y
vmitos. Visin borrosa y percepcin de halos de colores en el inicio. Posteriormente, la visin se reduce a percepcin de luz. Al
examen con la lmpara de hendidura: Inyeccin ciliar y conjuntival (gran congestin). Deslustramiento corneal por el edema. Cmara
anterior muy aplanada, casi inexistente. Pupila en midriasis media, oval, vertical, arreactiva. La tensin ocular est muy elevada, por
encima de 50 mmHg. ("dureza ptrea"). Tratamiento,inhibidores de la produccin del humor acuoso y agentes hiperosmticos. Colirio
de Pilocarpina al 2%+ Colirio de Dexametasona cada 5 minutos durante la primera media hora y cada 10 minutos durante la hora
siguiente para ir espaciando las instilaciones paulatinamente. Acetazolamida, 500 mg. por va IM. o IV. La va IV. debe manejarse con
cautela debido al contenido en Lidocana que lleva el preparado. Manitol al 20%: 250 cc. por va IV. (Aproximadamente 90 gotas por
minuto). El tratamiento se comenzar al unsono con todos los frmacos y, pasadas 3-4 horas del inicio del mismo, se proceder a
realizar una IRIDOTOMIA PERIFERICA LASER, aunque no se haya resuelto el bloqueo pupilar. Si el edema corneal no permite visualizar
bien el iris, presionaremos en la crnea central repetidas veces, a razn de 30 seg. cada una de ellas, para conseguir entreabrir el
ngulo camerular y facilitar el drenaje del humor acuoso, con lo que aumentar la transparencia corneal. TRAUMATISMOS OCULARES:
LESIONES NO PENETRANTES: ABRASIN CORNEAL: La instilacin de un colirio anestsico nos facilitar la apertura palpebral. El
tratamiento se hace con colirio de Tobramicina cada 4 horas, colirio ciclopljico cada 8 horas y oclusin semicompresivo. Si observamos
erosiones superficiales lineales o geogrficas en la mitad superior de la crnea, se procede a evertir el prpado superior y retirar el
cuerpo extrao que se aloja en la conjuntiva tarsal. Ante un cuerpo extrao enclavado en la crnea, se procede a su extraccin y a la
limpieza de xido si lo tuviera, con una aguja hipodrmica o IM. con poco bisel, siempre previa anestesia tpica y utilizando la lmpara
de hendidura. QUEMADURAS FSICAS: Pueden ser ocasionadas por un cigarrillo. En este caso, la zona corneal afecta se blanquea al
instante. El tratamiento es el mismo que el de la abrasin corneal. Tambin pueden ser causadas por radiaciones ultravioletas
(queratoconjuntivitis actnica): Afectacin punteada superficial por estallido de las clulas epiteliales tras la exposicin a los rayos UV.
(arco voltaico, radiacin solar, aparatos UV.,etc.). QUEMADURAS QUMICAS: Son muy graves, en especial las quemaduras producidas
por lcalis y constituyen una causa frecuente de ceguera. Las quemaduras por cido ejercen su efecto nocivo durante las primeras
horas, precipitan rpidamente las proteinas tisulares levantando barreras contra su propia penetracin. Secundariamente, la
neutralizacin por las proteinas del tejido circundante, tiende a localizar la lesin en el rea de contacto. Los lcalis se combinan con los
lpidos de las membranas celulares, produciendo as la total desestructuracin de las clulas con reblandecimiento del tejido.
CONTUSIONES OCULARES: Directas; Producidas por balonazos, pelotazos, puetazos, etc. Son lesiones conjuntivales el Hiposfagma (H.
subconjuntival). Una lesin corneal es la Queratitis traumtica por afectacin parenquimatosa, con edema (deslustramiento) y pliegues
o estras en la Descemet. Se tratar con colirio de Dexametasona cada 6 horas y colirio ciclopljico cada 8 horas. Las lesiones del iris y
del ngulo irido-corneal, implican con frecuencia hiphemas. Los Hiphemas consisten en la presencia de sangre fresca en la cmara
anterior. Pueden condicionar aumento de presin intraocular. Si es total y coexiste con hipertensin ocular, puede aparecer una tincin
hemtica de la crnea. Esta situacin exige la valoracin urgente por el oftalmlogo. Es ms peligroso el resangrado al tercer- quinto
da. El tratamiento consiste en el reposo en posicin de antitrendelemburg, oclusin binocular, colirio ciclopljico cada 8 horas, colirio
de Dexametasona cada 6 horas e hipotensores oculares si est aumentada la presin intraocular. La prednisona oral (1 mg/Kg/da),
evita el resangrado. En cuanto a las modificaciones pupilares, podemos encontrar irregularidades del contorno por ruptura del esfnter.
Sin embargo, hallaremos ms frecuentemente midriasis, nunca mxima, oval-oblicua que no responde a la luz, pero s a la
convergencia. La miosis es rara. La iritis e iridociclitis, ocurren por ruptura de la barrera hematoacuosa. Aunque mnima, siempre existe
en cualquier contusin. Se tratar con colirio de Dexametasona cada 6 horas y colirio ciclopljico cada 8 horas. Tambin pueden
producirse modificaciones de la presin intraocular. Se puede encontrar tanto hipertensin como hipotensin, en relacin con las
diversas afectaciones. Debe sospecharse subluxacin o luxacin cristaliniana por la iridodonesis (iris trmulo) y por la variacin de
profundidad de la cmara anterior. La luxacin del cristalino a cmara anterior exige ciruga urgente. En cuanto a la posible aparicin de
catarata, inicialmente puede apreciarse alguna opacidad cristaliniana, aunque tardar habitualmente das en formarse. El edema
retiniano ocasiona una importante disminucin de la agudeza visual, sobre todo si afecta a la mcula. Al examen del fondo de ojo, se
aprecia un aspecto blanquecino- lechoso de la retina. Se trata con colirio ciclopljico cada 8 horas y reposo absoluto con oclusin
binocular. HEMORRAGIAS RETINOVTREAS: La afectacin de la agudeza visual depende de la extensin y localizacin de stas. Se
tratar con colirio ciclopljico cada 8 horas y reposo absoluto. Si la hemorragia vtrea es total, interesa realizar ecografa para
determinar si coexiste un desprendimiento de retina. Los agujeros y desgarros retinianos se producen ms frecuentemente en ojos
miopes. Se tratan con laserterapia o ciruga. Cuando se observa un desprendimiento de retina, existe amputacin de campo visual.
Requiere ingreso del paciente, con reposo absoluto, oclusin binocular y aplicacin de colirio de Atropina al 1% cada 8 horas. Se
realizar intervencin quirrgica programada. Las rupturas coroideas son concntricas a la papila. No requieren tratamiento. Sin
embargo, si es posterior, debemos atender a los siguientes signos: Prdida brusca de la agudeza visual inmediatamente despus del
traumatismo, marcada hipotona ocular, aumento acusado de la profundidad de la cmara anterior con iridodonesis, hemorragia vtrea
masiva y hematoma subconjuntival de aspecto gelatinoso. Requiere ciruga urgente. Indirectas: Cuando el traumatismo es de vecindad
o a distancia (traumatismo craneal, aplastamientos torcicos, explosiones, fractura de huesos largos...), puede condicionar hemorragia
y exudados en el segmento posterior (angiopata traumtica de la retina y embolia grasa de la retina). LESIONES PENETRANTES: En las
heridas palpebrales, debe comprobarse si son transfixivas. No existe duda cuando se observa contenido intraocular prolapsado, pero en
el resto de los casos se deben buscar signos indirectos de perforacin. Estos signos son los siguientes: 1. Prdida de agudeza visual. 2.
Hipotona ocular. 3. Hundimiento o aplanamiento de la cmara anterior. 4. Quemosis marcada5. Laceracin corneal o escleral. Para
comprobar si existe disrupcin total, se proceder a instilar una gota de colirio de fluorescena y, con la luz azul cobalto,
MANUAL DE TRABAJO DEL CURSO ENARM CMN SIGLO XXI
CURSO ENARM CMN SIGLO XXI TEL: 36246001 Pharmed Solutions Institute PGINA 547

comprobaremos si la fluorescena es apartada por la salida del humor acuoso, bien espontneamente o a la presin en casos de herida
valvulada. Este signo se denomina SEIDEL. 6. Hiposfagma: A veces uncuerpo extrao se impacta en el globo ocular atravesando la
esclertica y quedando la puerta de entrada enmascarada por una hemorragia subconjuntival. Nos cercioraremos con el signo de
Seidel. Siempre que se asista a una herida penetrante en rbita, se debe recurrir a exploraciones radiolgicas: Radiografa simple
frontal y lateral de rbita, as como TAC, que nos descartarn o confirmarn la existencia de cuerpo extrao. El tratamiento se basa en
los siguientes puntos: Siempre exige asistencia especializada. Se evitarn las manipulaciones innecesarias. Abstenerse de medicacin
tpica, debiendo estar ambos ojos ocluidos con no compresivo. Administracin de analgsicos. Profilaxis antitetnica. Profilaxis
antibitica por va IV: Gentamicina, 80 mg. cada 8 horas y Ceftazidima, 1 g. cada 8 horas. Reparacin quirrgica bajo anestesia general

CASO CLINICO
Varn de 83 aos con enfermedad de Alzheimer avanzada, diabtico insulinodependiente de 35 aos de evolucin, que acudi al
servicio de urgencias por presentar cuadro de dolor ocular en ojo derecho de 2 das de evolucin. Se envio al servicio de oftalmologa
que refiere que el segmento anterior presenta lesin ulcerativa corneal de 2,5mm de dimetro con un poro endotelial perforante
central de aproximadamente 1mm, sin proceso infeccioso o inflamatorio concomitante. El paciente no presentaba antecedentes
traumticos.

PREGUNTA
Cul es la conducta a seguir en esta paciente.

RESPUESTA
a.- Cubrir el ojo.
b.- Colgajo pediculado, tipo Gundersen.
c.- Autoplastia libre conjuntival.
d.- Queratoplastia penetrante.

CASO CLINICO
Paciente de 42 aos que acude a Urgencias por cuadro de disminucin de agudeza visual (AV) de un da de evolucin. Como
antecedentes personales destaca trastorno depresivo en tratamiento. No presentaba antecedentes oftalmolgicos de inters y negaba
la utilizacin previa de gafas. Su tratamiento consiste en olanzapina 10mg, lorazepam 2mg y venlafaxina 150mg retard desde haca 12
meses. Una semana antes haba comenzado a tomar topiramato para perder peso. Al examen se presenta edema corneal y quemosis
conjuntival, una cmara anterior estrecha y cierre angular evidenciado mediante gonioscopia. La presin intraocular (PIO) es de
55mmHg en AO.

PREGUNTA
Cul es la conducta a seguir en esta paciente.

RESPUESTA
a.- Maleato de timolol.
b.- Manitol 3 g.
c.- Pilocarpina al 3 %.
d.- Acetazolamida 10 mg.

CASO CLINICO
Mujer de 34 aos de edad que inici de forma progresiva durante dos semanas visin doble binocular, ptosis palpebral de ambos
prpados superiores y cefalea. En el examen neurolgico, se objetiv una parlisis bilateral del nervio motor ocular comn, con
afectacin pupilar en el ojo derecho; el resto del examen neurolgico fue normal. El estudio mediante resonancia magntica (RM)
mostr la presencia de mltiples focos en la sustancia blanca de hiperseal en las secuencias basadas en T2, compatibles con lesiones
de origen desmielinizante; una de estas lesiones afectaba a la lnea media del tegmento del mesencfalo, y se extenda anteriormente a
la fosa interpeduncular y posteriormente hasta el acueducto de Silvio. Esta lesin, que presentaba realce en las secuencias T1 tras la
administracin intravenosa de gadolinio, era probablemente la responsable de los sntomas, ya que afectaba a la localizacin de ambos
ncleos del III par craneal.

PREGUNTA
Cul es la conducta a seguir en esta paciente.

RESPUESTA
a.- Acetato de glatirmero.
b.- Interfern (1a o 1b).
c.- Azatioprina.
d.- Natalizumab.

MANUAL DE TRABAJO DEL CURSO ENARM CMN SIGLO XXI
CURSO ENARM CMN SIGLO XXI TEL: 36246001 Pharmed Solutions Institute PGINA 548


PIE DIABETICO:
CIENCIAS BASICAS: Es una complicacin crnica de la DM, con etiologa multifactorial, teniendo como factores principales la infeccin,
isquemia, neuropata y mal control metablico. SALUD PUBLICA: La incidencia y prevalencia de la lcera en gente con DM vara de 2,4 a
2,6 y de 4 a 10%, respectivamente.68 Aproximadamente 15% de todos los pacientes con DM desarrollar una lcera en el pie o en la
pierna durante el transcurso de su enfermedad. PATOGENIA: Las dos lesiones bsicas del PD son la neuropat a (pie indoloro) y las
angiopapata (pie sin pulso). Estas son origen de las complicaciones isqumicas e infecciosas. La hiperglucemia crnica es el factor de
riesgo ms importante para el desarrollo de la neuropata. Existen dos teoras etiopatognicas: una metablica y otra vascular, aunque
probablemente sea la interaccin de ambas lo que la produzca. La polineuropata simtrica distal de distribucin en calcetn es la forma
de neuropata ms frecuente que predispone a la aparicin de lceras en el pie. Inicialmente se alteran las fibras nerviosas pequeas
(fibras sensitivas) provocando la prdida de la sensacin de dolor y temperatura y, ms tarde, las fibras nerviosas grandes,
disminuyendo el umbral de percepcin de vibracin. La neuropata motora produce hipotrofia muscular apareciendo dedos en martillo
y en garra que favorecen la protrusin de las cabezas de los metatarsianos. La neuropata autonmica produce sequedad y fisuras en la
piel por afectacin de las fibras simpticas posganglionares de las glndulas sudorparas, aumento del flujo sanguneo por apertura de
comunicaciones arteriovenosas que disminuyen la perfusin de la red
capilar y aumentan la temperatura de la piel, lo que da lugar a un
trastorno postural en la regulacin del flujo con una respuesta
inflamatoria anormal frente a la agresin a los tejidos. Tambin hay
edema neuroptico y aumento de la presin venosa. La osteoartropata
neuroptica o PIE DE CHARCOT es una de las peores consecuencias de la
DM en el pie. Puede encontrarse en una fase aguda que puede confundir
con infeccin o en una fase deformante crnica y progresiva caracterizada
por la destruccin indolora de huesos y articulaciones. En esta situacin,
con los traumatismos se produce distensin ligamentosa y microfracturas,
y con el peso corporal ocurre una destruccin articular progresiva que da lugar a fracturas y subluxaciones que son acelerados por el
aumento de la reabsorcin sea osteoclstica, secundaria a la hiperemia causada por la denervacin simptica de la microcirculacin.
Por estas razones, el pie debe estar inmovilizado con frulas, muleta o silla de ruedas. La neuropata, con o sin isquemia asociada, est
implicada en la fisiopatologa de la lcera del PD en el 85% a 90% de los casos. Por tanto, el pie insensible, sea bien perfundido o
isqumico, debe considerarse como de alto riesgo de lcera. DIAGNOSTICO: Tiene caractersticas clnicas frecuentes como lceras,
gangrenas y amputaciones en las extremidades inferiores, ocasionando en el paciente discapacidad parcial o definitiva. Los pacientes
con este problema de salud deben ingresar a un programa de PD que debe ser permanente, estratgico y multidisciplinario incluyendo
la prevencin y el tratamiento del PD. Infeccin: Es frecuenteencontrar manifestaciones de calor, rubor o eritema, edema, dolor, olor
ftido y secrecin purulenta. Tambin puede manifestarse con necrosis de los bordes de la herida, aumento del flujo y tejido
desvitalizado. Es multifactorial: La neuropata y la consiguiente disminucin de la sensibilidad predispone a las lceras a igual que la
disminucin del reflejo vasomotor que lleva a disminucin de sudoracin y a sequedad de la piel y a la aparicin de grietas. La micosis
interdigital produce fisuras. La macroangiopata lleva a insuficiencia arterial con retardo en la cicatrizacin y deficiente llegada de
antibiticos. El desarrollo de derivaciones arteriovenosas de capilares nutricios tambin predispone a lesiones de los tejidos. Traumas.
La infeccin superficial aguda (lceras no complicadas, celulitis): adquiridas en la comunidad y sin tratamiento antibacteriano, en su
mayora son monomicrobianas, Staphylococcus y Streptococcus spp. No siempre es necesario el uso de antibiticos ni de la toma de
cultivos, especialmente si es superficial. Las Infecciones profundas y/o crnicas: son polimicrobianas en ms de 50% de los casos,
encontrndose dos a tres grmenes. Hay que tomar cultivo de tejido profundo. A las cocceas grampositivas de las superficiales se
agregan bacilos gramnegativos y anaerobios.La lesin de fibras sensoriales lleva a disminucin de liberacin de histamina, con la
consiguiente disminucin de signos inflamatorios y de la respuesta inflamatoria. La respuesta inflamatoria es fundamental en los
procesos de defensa y de la cicatrizacin. TRATAMIENTO: Compromiso del estado general: Indicar si el manejo es ambulatorio o de
hospitalizacin. Paciente con herida Wagner 3 o grado mayor por lo general necesita hospitalizacin. Control metablico: La
hiperglicemia se controla principalmente con insulina, 30 U/d en promedio, se regula segn su demanda y se usa sin o con
antidiabticos orales, principalmente metformina, 1 700 mg/d. En estos pacientes, para el manejo de la dislipidemia, se ha obtenido
buenos resultados con atorvastatina, 40 mg/d. Si hay
hipertensin, generalmente se controla con losartn
o enalapril solos o asociados con hidroclorotiazida, y
si hay edemas se recomienda furosemida. Los
pacientes obesos logran bajar de peso con
educacin, dieta de 1 200 kcal y ejercicios
moderados de al menos 30 min/d. Con estas medidas
tambin mejoran los problemas metablicos
descritos anteriormente. Adems del cambio de
estilo de vida permanente, es de ayuda temporal el
uso de orlistat, 120 mg, tres veces al da. Cuidados de
la lcera: Desbridamiento quirrgico, con bistur
(Figura 1) y, en algunos casos, debridamientos
enzimtico y biolgico (larvas). El uso de factores de
crecimiento y de injertos han logrado la cicatrizacin
en un menor tiempo y se ha evitado las
amputaciones. El tratamiento tpico con
propiedades antibiticas o regenerativas tambin
son importantes. Tratamiento antibitico: Primer evento y sin compromiso de la extremidad. Manejo ambulatorio con antibiticos va
CLASIFICACIN DE WAGNER.
0 Ninguna.

Pie de riesgo Callos, hallux valgus,
dedos en garra
I lceras superficiales Destruccin del espesor total de la piel
II lcera profunda Penetra piel, grasa y ligamentos sin
afectar hueso. Infectada
II lcera profunda ms
absceso
(osteomielitis)
Profunda, secrecin, mal olor
IV Gangrena limitada Necrosis de una parte del pie
V Gangrena extensa Todo el pie afectado, efectos
sistmicos
MANUAL DE TRABAJO DEL CURSO ENARM CMN SIGLO XXI
CURSO ENARM CMN SIGLO XXI TEL: 36246001 Pharmed Solutions Institute PGINA 549

oral por dos semanas: Cefalosporina primera generacin (cefalexina), 500 mg, cada 6 horas, Clindamicina, 300 mg, cada 8 horas,
Dicloxacilina, 500 mg, cada 6 horas, Levofloxacino, 500 a 750 mg, cada 24 horas. lcera recidivante, celulitis extensa en miembro
inferior, compromiso articular u seo. Manejo hospitalario con antibitico intravenoso. El tiempo vara dependiendo de la condicin
clnica del paciente: Ceftazidima, 1 g, EV, cada 8 horas, ms clindamicina, 600 mg, EV, cada 8 horas. Ceftriaxona, 1 g, EV, cada 12 horas,
ms clindamicina, 600 mg, EV, cada 8 horas. Ciprofloxacino, 400 mg, EV, cada 12 horas, ms clindamicina, 600 mg, EV, cada 8 horas.
Infecciones que amenazan la vida, sospecha de meticilinorresistencia o multirresistencia y Pseudomonas Vancomicina, 1 g, EV, cada 12
h, ms ceftazidima, 1 g, EV, cada 8 h, ms clindamicina, 600 mg, EV, cada 8 h. Vancomicina, 1 g, EV, cada 12 h, ms meropenem, 500
mg, EV, cada 8 h, ms moxifloxacino, 400 mg, VO, cada 24 h. Osteomielitis: El periodo mnimo de tratamiento antibitico es tres meses
y puede durar seis meses a una ao. Para acortar este tiempo se debe realizar curetaje o extraccin de huesos infectados y
desvitalizados. Tratamiento hipolipemiante: Existen varias alternativas pero se prefiere atorvastatina, 20 a 40 mg/comida. Tratamiento
del dolor neuroptico: Una de las combinaciones que ha dado buenos resultados es asociar pregabalina con tramadol, titulando la
dosis. Amputacin: Es el ltimo recurso para evitar la complicacin o muerte del paciente con PD. Toda amputacin que se realiza en el
pie se denomina menor y las que se realizan por arriba del tobillo, incluyendo la infracondlea y supracondlea, se denominan mayores.
Es preferible una amputacin menor porque con esta el paciente casi siempre puede caminar por s solo. Cuando es amputacin
mayor, el paciente deber usar en forma permanente una muleta, prtesis o silla de ruedas si quiere desplazarse. Con la amputacin
mayor, el paciente entra a una discapacidad que deteriora su salud y calidad de vida por lo que necesita mayor apoyo de su familia y de
la sociedad. Se indica amputacin mayor cuando un miembro isqumico no se puede revascularizar o es afectado por osteomielitis
difusa que no responde al tratamiento y amenaza la vida del paciente. Cuando el miembro infectado constituye un foco sptico que no
responde al tratamiento antibitico y desbridamientos, para salvar la vida del paciente se debe tomar la decisin de una amputacin
mayor.

CASO CLINICO
Acude a consulta masculino de 58 aos de edad el cual refiere que desde hace una semana inicia con una lesin en tipo ulcera plantar
de 3.5 cm aproximadamente, como antecentes de importancia, se trata de diabtico controlado con hipoglucemiantes orales, como
moderado apego a tratamiento, sin embargo debido a su nivel socioeconmico primero trato de manejar su lesin por medio
tradicionales, sin embargo usted no percibe datos sugerentes de proceso por clostridium o agentes anaerobios, la extremida presenta
temperatura normal, sus pulso estn presentes y solo se observa hiperbaralgesia.

PREGUNTA
Considerando este cuadro clnico, cual es el manejo de primera intensin para este caso antes de realizar un envo a segundo nivel de
atencin.

RESPUESTA
a.- Es un paciente que tendra una rpida mejora en cmara hiperbarica.
b.- Indica antiagregante plaquetario, pentoxifilina y vendaje de jhonson.
c.- Lo ms indicado es debridar el area y antibiticos.
d.- Indica debridacion, pentoxifilina, antibitico y vendaje.





























MANUAL DE TRABAJO DEL CURSO ENARM CMN SIGLO XXI
CURSO ENARM CMN SIGLO XXI TEL: 36246001 Pharmed Solutions Institute PGINA 550

INGLES

CASE REPORT

A recently married 29-year-old nulliparous African-American female presents with uncomplicated cystitis. She is otherwise healthy. She
reports that she is currently using barrier birth control methods but plans to discontinue this soon, as she would like to become
pregnant.

QUESTION
Which one of the following supplemental vitamins or minerals would you advise this patient to
begin taking?

a) Thiamine
b) Iodine
c) Folate
d) Calcium

QUESTION
Which one of the following is true regarding testing the rotator cuff for injury?

ANSWER
a) The subscapularis is tested with lateral rotation against resistance
b) The teres minor is tested with medial rotation against resistance
c) The infraspinatus is tested with medial rotation against resistance
d) The supraspinatus is tested with abduction against resistance


CASE REPORT
You have recently begun caring for a 25-year-old white female who has multiple complaints. You have seen her 3 times for walk-in
office visits over the past month. She has shown appreciation
for your work during the encounters, but has been critical of your care when talking to the office
staff. At times she has been kind and charming, and at other times she has been rude and verbally
abusive to your staff. She has a string of multiple relationships in the past, none of which has
lasted very long. During times of intense stress, she has sometimes engaged in self-mutilation.
She frequently changes jobs and living arrangements.

QUESTION
Which one of the following strategies would be most appropriate in the care of this patient

ANSWER
a) Strive to develop a close relationship with the patient.
b) Ignore verbal attacks on staff members.
c) Provide detailed, technical explanations for. any therapies provided.
d) Schedule frequent office visits for follow-up.

In an 11-year-old male with dark brown urine and hand and foot edema.

QUESTION
Which one of the following would be most suggestive of glomerulonephritis?

ANSWER
a) RBC casts in the urine
b) Elevated C3 and C4 complement levels
c) Eosinophils in the urine
d) Positive serum antinuclear antibody levels

A 32-year-old white female at 16 weeks gestation presents to your office with right lower quadrant pain.

QUESTION
Which one of the following imaging studies would be most appropriate for initial evaluation of this patient?

ANSWER
a) MRI of the Abdomen
b) Intravenous pyelography
c) CT of the abdomen
d) Ultrasonography of the abdomen
MANUAL DE TRABAJO DEL CURSO ENARM CMN SIGLO XXI
CURSO ENARM CMN SIGLO XXI TEL: 36246001 Pharmed Solutions Institute PGINA 551


QUESTION
Which one of the following is the leading cause of blindness in individuals over age 65?


a) Narrow angle glaucoma
b) Diabetic retinopathy
c) Macular degeneration
d) Ophthalmic artery occlusion

CASE REPORT

A 48-year-old white female complains of anxiety and difficulty concentrating at home and at work. She reports that the symptoms have
increased over the last 2 months because of her daughters marital difficulties. She has had similar symptoms along with intermittent
depression since she was a teenager. She admits to a loss of pleasure in work and recreational activities.

QUESTION
Which one of the following is LEAST likely to help her coexistent depressive symptoms?

ANSWER
a) Nortriptyline.
b) Buspirone.
c) Escitalopram
d) Venlafaxine

CASE REPORT
An 82-year-old white female visits your office. She reports a 2-week history of shoulder stiffness that is severe in the morning, and a 3-
day history of pain on the left side of the face while chewing food. She denies dental pain or sensitivity. Her family history is negative
for rheumatologic disorders. Your evaluation reveals a 5-lb weight loss and evidence of mild depression. Her temperature is 38.0 C
(100.4 F), she has no dental decay or carotid bruit, and her left temporal scalp is tender. Her lungs are clear to auscultation, there are
no abnormal heart sounds, and her abdomen is nontender. She has no joint swelling or warmth, but experiences discomfort with
shoulder and hip range of motion. Laboratory analysis reveals a hemoglobin level of 11.0 g/dL (N 11.716.1), an erythrocyte
sedimentation rate of 80 mm/hr (N 030), a serum glucose level of 120 mg/dL, and a BUN level of 24 mg/dL (N 823).

QUESTION
Which one of the following is the greatest immediate risk for this patient?

ANSWER
a) Monocular vision loss
b) Hemiparesis
c) Sudden death
d) Adhesive capsulitis of the shoulder

CASE REPORT
A 73-year-old male with COPD presents to the emergency department with increasing dyspnea.
Examination reveals no sign of jugular venous distention. A chest examination reveals decreased
breath sounds and scattered rhonchi, and the heart sounds are very distant but no gallop or murmur
is noted. There is +1 edema of the lower extremities. Chest radiographs reveal cardiomegaly but
no pleural effusion. The patients -type natriuretic peptide level is 850 pg/mL (N <100) and his
serum creatinine level is 0.8 mg/dL (N 0.61.5).

QUESTION
Which one of the following would be the most appropriate initial management?

ANSWER
a) Prednisone, 20 mg twice daily for 1 week
b) Furosemide, 40 mg intravenously
c) Intravenous heparin
d) Tiotropium

CASE REPORT
A 28-year-old primigravida is at 20 weeks gestation by dates but her fundal height is consistent with a 26-week gestation. She has had
episodes of vomiting during the pregnancy that were more severe than the physiologic vomiting typically seen in pregnancy. A
sonogram performed at about 5 weeks gestation for vaginal bleeding was normal and showed a single fetus.

MANUAL DE TRABAJO DEL CURSO ENARM CMN SIGLO XXI
CURSO ENARM CMN SIGLO XXI TEL: 36246001 Pharmed Solutions Institute PGINA 552

QUESTION
Which one of the following would be most appropriate at this point?

ANSWER
a) Expectant management
b) A repeat sonogram
c) MRI of the pelvis
d) A serum hCG leve


CASE REPORT
A 24-year-old male, new to your practice, presents for a mental health evaluation. The patient has a past history of schizophrenia,
diagnosed several years ago.

QUESTION
Which one of the following, if present, would lead to a reconsideration of this diagnosis?

ANSWER
a) Loose associations
b) Elated mood
c) Social dysfunction
d) Auditory hallucinations

CASE REPORT
A previously healthy 22-year-old female presents for her regular prenatal checkup at 38 weeks gestation. She has a blood pressure of
145/95 mm Hg today and this is unchanged 1 hour later. Her blood pressure was normal before pregnancy. She is otherwise feeling
well. She has moderate edema at the ankles and 3+ reflexes at the knees and ankles. A urinalysis for protein is normal.

QUESTION
Given this presentation, which one of the following is the most likely diagnosis?

ANSWER
a) Hemolysis, elevated liver enzymes, low platelets syndrome
b) Preeclampsia
c) Essential hypertension
d) Gestational hypertension

CASE REPORT
A 72-year-old female with longstanding diabetes mellitus presents to your office. During the review of systems, she complains of
difficulty voiding and frequent dribbling. A urinalysis is negative for infection and her post-void residual volume is 250 mL.

QUESTION
Which one of the following is the most liely cause of this patients urinary incontinence?

ANSWER
a) A grade II cystocele
b) Atrophic vaginitis
c) Asymptomatic bacteriuria
d) Autonomic neuropathy

CASE REPORT
A 3-year-old male is brought to your office the day after he was stung by a honeybee. He has developed a significant local reaction, with
redness and swelling around the site of the sting on his forearm, and also has had some swelling of his lips for a couple of hours. His
mother removed the stinger and gave him some oral diphenhydramine The local reaction has now almost resolved and he has not had
any hives or respiratory distress.

QUESTION
Which one of the following is true concerning this situation?

ANSWER
a) An antibiotic that covers Streptococcus pyogenes and Streptococcus aureus should be administered
b) If venom immunotherapy is begun, it should be stopped after 2 years of treatment
c) The child is at high risk for a systemic reaction if he is stung again in the future
d) Skin tests are not helpful in confirming the presence of insect sting allergy


MANUAL DE TRABAJO DEL CURSO ENARM CMN SIGLO XXI
CURSO ENARM CMN SIGLO XXI TEL: 36246001 Pharmed Solutions Institute PGINA 553

CASE REPORT
A 65-year-old white female develops a burning pain in the left lateral thorax, followed 2 days later by an erythematous vesicular rash.

QUESTION
Of the following, the best treatment is?

ANSWER
a) topical capsaicin (Zostrix)
b) oral corticosteroids
c) topical corticosteroids
d) topical acyclovir (Zovirax)

CLINIC CASE
A patient on peritoneal dialysis has received six (6) units of red blood cells for a bleeding duodenal ulcer. Dialysis was discontinued 3
day ago and an emergency gastric surgery is now necessary. The patient arrests following induction of general anesthesia whit propofol
and succinylcholine.

QUESTION
Which of the following is the likely cause of this cardiac event?

ANSWERS
a.- Hypothermia.
b.- Hypovolemic shock.
c.- Hypocalcemia.
d.- Hyperkalemia.

CLINIC CASE
A 79 year old man with a one pack for day smoking history is incidentally found to have a right lower lung mass with moderate, free-
flowing, right-side pleural effusion. On further questioning he admits to some symptoms of dyspnea and a 20-pound weight loss over 3
months, but denies other symptoms.

QUESTION
What should be done next.

ANSWERS
a.- Mediastinoscopy.
b.- Bronchoscopy.
c.- Thoracentesis.
d.- CT of the chest.

CLINIC CASE
A 21 year old woman with a history of cystic fibrosis is admitted to the ICU for pneumonia. Her CXR shows right lower lung zone dense
consolidations with pleural effusion, in addition to previous seen bronchiectasis. She has no drug allergies, and her previous respiratory
culture results are not available at this time.

QUESTION
What empiric antibiotics should be started.

ANSWERS
a.- Imipenem.
b.- Ceftriaxone and metronidazole.
c.- Cefepime.
d.- Piperacilin and Tazobactam.

CLINIC CASE
A 66 year old man complains of gradual onset of dyspnea. His PFT shows: FEV1/FVC 104, FEV1 73% predicted, and FVC 70% of
predicted, TLC 66% of predicted, and DLCO 52% of predicted. The patient denies any known medical problems. He has smoked a few
cigarettes a day since age 18, and reports that he use to work in a shipyard from age 18 to 58. On exam his vitals are normal, and the
lung exam is significant for some bibasular crackies. No clubbing of the fingernails is seen.

QUESTION
What should be done next to evaluate this patients dyspnea.

ANSWERS
a.- Transthoracic echocardiogram.
b.- Bronchoscopy for transbronchial biopsy.
MANUAL DE TRABAJO DEL CURSO ENARM CMN SIGLO XXI
CURSO ENARM CMN SIGLO XXI TEL: 36246001 Pharmed Solutions Institute PGINA 554

c.- High resolution CT scan of the chest.
d.- Right heart catheterization.

CLINIC CASE
A 70 year old patient with hypercalcemic crisis has an emergency resection of a large parathyroid adenoma. Immediately post
operation, the patient develops convulsions. Calcium IV is administered with minimal response.

QUESTION
What is the etiology of the convulsions ?

ANSWER
a.- Hypocalcemia.
b.- Hypernatremia
c.- Cerebral ischemia.
d.- Hypomagensemia.

CLINIC CASE
A 45 year old male weighing 80 kg is seen in the ER for two episodes of hematochezia within the last 2 hours. He has pulse of 80 bpm, a
BP of 100/60 mmHg, a respiratory rate of 30, and urine output of 20 ml. he is anxious and confused.

QUESTION
What is his estimated blood loss?

ANSWER
a.- 500 ml
b.- 1000 ml.
c.- 2000 ml.
d.- 3000 ml.

CLINIC CASE
A 60 year old woman, who was previously treated with radiation therapy for uterine cancer at age 50, presents with a small bowel
obstruction.

QUESTION
Intraoperatively, one would expect to see?.

ANSWER
a.- A sharp demarcation at the area of obstruction.
b.- Gray thick-walled small bowel matted together.
c.- Multiple areas of stricture in normal appearing small bowel.
d.- Inter loop abscesses and fistula.

CLINIC CASE
A 75 years old female is evaluated for reflux esophagitis. At endoscopy the mucosa is salmon pink with a definite demarcation between
normal and abnormal esophageal mucosa. Biopsy indicates no dysplasia. Three weeks following a Nissen Fundoplication, the patient
has a severe episode of retching following a heavy meal and has vomiting with blood. Two hours later, he has severe epigastric pain and
a spiking temperature of 102 F.

QUESTION
Which of the following is the problem?.

ANSWER
a.- The Nissen has slipped.
b.- Fundoplication disruption has occurred with perforation.
c.- A perforated ulcer has developed.
d.- Acute pancreatitis.

CLINIC CASE
A 62 year old male presents with dyspepsia. An UGI shows thickening of the rugal folds, at upper endoscopy, no mass or lesion is
identified. The gastric wall is thickened but no ulceration is noted.

QUESTION
Following endoscopic biopsy, this patient is best treated by?

ANSWER
a.- Radiation.
MANUAL DE TRABAJO DEL CURSO ENARM CMN SIGLO XXI
CURSO ENARM CMN SIGLO XXI TEL: 36246001 Pharmed Solutions Institute PGINA 555

b.- Chemotherapy.
c.- Chemotherapy and radiation.
d.- Subtotal gastrectomy.

CLINIC CASE
Following an open cholecystectomy and common duct exploration for multiple gallstone in a 75 year old female, the colangiogram
obtained 10 days post operatively reveals an impacted 1.5 cm gallstone at the ampulla. The bilirubin is 5.0.

QUESTION
What is the procedure of choice?

ANSWER
a.- Repeat the cholangiogram in 1 week.
b.- T-tube flush with Capmul 8210.
c.- Open sphincteroplasty.
d.- Choledochoduodenostomy.

CLINIC CASE
A 46 year old male has had intermittent flares of ulcerative pancolitis for 10 years. He has increasing diarrhea, blood per rectum, and
fever. He responds to medical management.

QUESTION
Proctocolectomy is indicated for?

ANSWER
a.- Length of colon involved.
b.- Long periods of persistent disease.
c.- Severe dysplasia on initial biopsy.
d.- Persistent stricture.

CLINIC CASE
A 75 year old woman suffered from sudden onset left eye blindness. No pain was associated with her blindness, and no other
abnormalities were seen in the neurological examination.

QUESTION
What is the most likely diagnosis?.

ANSWER
a.- Small lacunar stroke involving the occipital lobe.
b.- Ischemic lesion involving the lateral geniculate nucleus.
c.- Compressive lesion on the optical chiasm.
d.- Retinal ischemia due to retinal artery occlusion.

CLINIC CASE
A 51 year old male started to have a gradual decline in his intellectual functions associated with depressive symptoms and personality
changes. The patient obtained a score of 23/30 on mini-mental status examination and on physical examination chorea was noted. The
diagnosis of Huntingtons disease was suspected.

QUESTION
What is typical inheritance pattern of Huntingtons disease?

ANSWER
a.- X-linked
b.- Autosomal dominant.
c.- Autosomal recessive.
d.- No familial pattern.

CLINIC CASE
A 36 year old woman was referred to the neurology clinic to be evaluated for her progressing headaches. The headaches were
described as dull, mild to moderate in severity, and located on the left side of the head.
The patient has no family history of migraine headaches. Neurological examinations was mostly normal except for right-sided
hyperreflexia.

QUESTION
What is the most likely diagnosis?

MANUAL DE TRABAJO DEL CURSO ENARM CMN SIGLO XXI
CURSO ENARM CMN SIGLO XXI TEL: 36246001 Pharmed Solutions Institute PGINA 556

ANSWER
a.- Left middle cerebral artery aneurysm.
b.- Arterio-Venous malformation.
c.- Vascular brain tumor.
d.- Sagittal sinus thrombosis.

CLINIC CASE
A 25 year old man with HIV presented to the ED with right-side weakness and confusion. An MRI brain showed a few ring enhancing
lesions. The differential diagnosis is narrowed to toxoplasmosis and primary CNS lymphoma.

QUESTION
What is the appropriate next step of management?.

ANSWER
a.- Open brain biopsy.
b.- Stereotectic brain biopsy.
c.- Empiric treatment of toxoplasmosis with reimaging to evaluate the response.
d.- High dosis steroids.

CLINICAL CASE
A 32 year old primigravida presents to her obstetrician for routine follow-up at 34 weeks gestation. She has a sonogram of the
developing baby which also shows that she has developed right sided moderate hydronephrosis, Serum creatinine concentration is
checked and is 0.8 mg/dl. She has no symptoms. Urinalysis normal.

QUESTION
At this point, you should:

ANSWER
a.- Send urine for culture assuming infection.
b.- Consult urology immediately.
c.- Admit to the hospital.
d.- No further intervention.

CLINIC CASE
A 65 year old make with pneumonia and septic shock develops acute kidney injury. He has been running a low blood pressure (80/45
mmHg) on several vasopressor agents. Urine output start to fall and his creatinine begins to rise. His urine sodium is high; he has no
proteinuria.

QUESTION
The most common cause of his kidney failure in this setting is:

ANSWER
a.- Obstruction.
b.- Acute tubular necrosis.
c.- Acute interstitial nephritis.
d.- Goodpastures disease.

CLINIC CASE
A 54 yearold woman reports a bothersome cough to 6 months duration, She has never smoked, and denies history of wheezing or
heartburn. She has diabetes and hypertension, and has been on metformin and lisinopril for the past 3 years.

QUESTION
What is the next appropriate step in management.

ANSWER
a.- Discontinue lisonopril.
b.- Start a proton pump inhibitor.
c.- Esophageal pH monitoring.
d.- Barium swallow evaluation.

CLINIC CASE
A 48 year old man presents with hemoptysis. He reports that he has also had bloody nasal discharge over the past month. His CXR
shows bilateral nodules. Laboratory data is notable for creatinine of 4.7 mg/dl, and urine analysis shows 250 RBC/hpf

QUESTION
Which of the following serum studies should be ordered next?
MANUAL DE TRABAJO DEL CURSO ENARM CMN SIGLO XXI
CURSO ENARM CMN SIGLO XXI TEL: 36246001 Pharmed Solutions Institute PGINA 557


ANSWER
a.- ANCA, anti-GBM antibodies.
b.- Anti-dis-DNA and anti-cardiolipin antibodies.
c.- ESR, C3, C4 leves
d.- RF, anti-citrulline antibodies.

CLINIC CASE
A 62 year old woman presents for evaluation of dizziness. It occurs often when shes reaching overhead into her cupboards. She also
notice that her face has been swollen, and she has recently developed a cough. On exam, she is noted to have some fullness in her
neck, but the rest of the physical exam is unremarkable. A CXR shows a sharply circumscribed, dense anterior mediastinal mass that
displaces the trachea to the left.

QUESTION
What is the next appropriate step in management?

ANSWER
a.- Radiation therapy.
b.- Radioactive iodine scanning.
c.- Referral for chemotherapy.
d.- Referral for surgical resection.

CLINIC CASE
A 38 year old woman with a history of asthma has had multiple admissions in the past year for asthma exacerbation. She was treated
during the most recent hospitalization for a community-required pneumonia that was seen on CXR. She comes to you for post-
discharge follow-up. She reports she is compliant with her combination steroid/long-acting beta agonist inhaler, but still needs her
rescue inhaler several times a day, and still has a cough occasionally productive of brown plug. Her blood work on the day of discharge
was only significant for an eosinophilic count of 750/mm3.

QUESTION
Which of the following serum studies should be considered?

ANSWER
a.- Chronic eosinophilic pneumonia.
b.- Allergic brochopulmonary aspergillosis.
c.- Steroid non responsive asthma.
d.- Viral bronchitis.

CLINIC CASE
A 70 year old man who previously worked in stonecutting has had bilateral upper lung zone interstitial markings and small nodules on
his CXR for several years. However, on his most recent CXR, it is noted that some of the nodules in the right upper lobe have coalesced
into a mass, and cavitation is noted.

QUESTION
What is the next appropriate step in management?

ANSWER
a.- Sputum induction to rule out tuberculosis.
b.- Bronchoscopy with transbronchial biopsy to rule out malignancy.
c.- Refer to thoracic surgery for resection.
d.- Start antibiotics for treatment of community acquired pneumonia.

CLINIC CASE
A 60 year old man with severe COPD is found to be difficult to arouse after undergoing an outpatient, elective screening colonoscopy.
The following lab data are available: ABG on room air pH 7.24, PaO2 64, PaCO2 78, Electrolytes: Na 140, HCO3 35, Cl 100.

QUESTION
What is his current acid-base status?

ANSWER
a.- Metabolic acidosis with respiratory compensation.
b.- Acute respiratory acidosis with metabolic compensation.
c.- Acute on chronic respiratory acidosis
d.- Chronic acidosis with metabolic compensation.


MANUAL DE TRABAJO DEL CURSO ENARM CMN SIGLO XXI
CURSO ENARM CMN SIGLO XXI TEL: 36246001 Pharmed Solutions Institute PGINA 558

CLINIC CASE
A 50 year old man recently sustained a fall while shoveling snow; Four weeks later he began to experience shortness of breath, right-
sided chest discomfort, and a dry cough. He also feels extreme fatigue and feels less productive at work. The patient has no other
significant past medical history and does not smoke. Other than using the indoor hot tub in his condominium community three times a
week since his fall, there are no changes in his usual activities.

QUESTION
What is the most likely diagnosis?

ANSWER
a.- Sarcoidosis
b.- Aspergillosis
c.- Hypersensitivity pneumonitis due to mycobacterium avium.
d.- Active Infection with mycobacterium avium.

CLINIC CASE
A 48 year old man presents with a week of fever, cough, and malaise. He has no significant past medical history and he takes no
medications. His vital signs are normal but an exam reveals right sided crackies. A CXR shows a right lower lobe infiltrate.

QUESTION
What antibiotic should be started?

ANSWER
a.- Doxycycline.
b.- Trimethoprim/Sulfamethoxazole.
c.- Cephalexine.
d.- Amoxicillin/Clavulanate.

CLINIC CASE
A 46 year old woman with a history of asthma calls you for a refill of her albuterol inhaler. Her asthma has always been well-controlled
on a daily dose of moderate-dose inhaled corticosteroid and shes long misplaced her albuterol inhaler. However 3 weeks ago she had
an upper respiratory tract infection. All her acute symptoms have resolved but since then she has felt more wheezing, and often coughs
at night. Shes been using her daughters albuterol inhaler and finally decides to call you to get her own prescription.

QUESTION
What is the next appropriate step in management?

ANSWER
a.- Prescribe a course of oral fluoroquinolone.
b.- Prescribe albuterol inhaler for as needed rescue use.
c.- A short course of oral corticosteroid therapy.
d.- Double her moderate-dose inhaled corticosteroid therapy.

CLINIC CASE
A 52 year old Hispanic female is seen for consultation in the hospital. She states that she had a complete physical with laboratory
testing 2 months earlier that was completely normal. She feels poorly now for the past few weeks. Her legs have become swollen and
she is having shortness of breath. Laboratory testing reveals a positive ANA 1:360, an elevated ds-DNA low C3 and C4 leves, a 24 hour
urine protein of 2.5 grams with significant RBCs on her urinalysis, and renal failure. An echocardiogram shows a significant pericardial
effusion, but no evidence for tamponade. A kidney biopsy reveals diffuse proliferative lupus nephritis.

QUESTION
Which of the following is the most appropriate therapy at this time?

ANSWER
a.- Cyclophosphamide.
b.- Azothioprine.
c.- Solumedrol.
d.- Plasmaferesis.

CLINIC CASE
A 44 year old Hispanic female presents with inflammatory arthritis of one of her wrists and MCPs interstitial lung disease, a non
specific rash, and Raynauds phenomenon. Lab test reveal and alevated AntiNuclear Antibody 1:640, Rheumatoid Factor of 50 (Normal
< 14), and a normal anti-U1 RNP.

QUESTION
Which of the following is the diagnosis?
MANUAL DE TRABAJO DEL CURSO ENARM CMN SIGLO XXI
CURSO ENARM CMN SIGLO XXI TEL: 36246001 Pharmed Solutions Institute PGINA 559


ANSWER
a.- Undifferentiated connective tissue disease.
b.- Mixed connective tissue disease.
c.- Systemic lupus erythematosus.
d.- Rheumatoid arthritis.


CLINIC CASE
A 30 year old patient with sjogrens syndrome is placed on a cytotoxic agent as well as oral steroids. At 3 months she begins to have
right thigh and knee pain that is increased with walking and standing. Physical examination reveals normal range of motion of her knee,
but mild pain on internal and external rotation of her hip.

QUESTION
What is the most sensitive and appropriate test to diagnose his condition early?

ANSWER
a.- Plain film radiography.
b.- Technetium-99m bone scanning.
c.- Magnetic resonance imaging.
d.- CT scan.

CLINIC CASE
A 32 year old white female with systemic lupus erythematosus is 32 week pregnant and presents to your office with a headache.
Physical examination is remarkable for a blood pressure of 200/110 mmHg. Laboratory studies show a hemoglobin of 8.0 g/dl, platelet
count of 38,000, a low haptoglobin level, and an elevated LDH, AST, and ALT.

QUESTION
What test would be best to determine the cause of her problems?

ANSWER
a.- Complement levels.
b.- Erythrocyte sedimentation rate.
c.- Anti-Smith antibody.
d.- C-reactive protein.

CLINIC CASE
A 24 year old female has diagnosed systemic lupus erythematosus for the past 5 years. She has had episodes of discoid rashes as well as
polyarthritis and mild hematologic abnormalities, but none in the past year.

QUESTION
What pharmacological agent would be most appropriate for her?

ANSWER
a.- Prednisone.
b.- Hydroxychloroquine.
c.- Cyclophosphamide.
d.- Metotrexate.

CLINIC CASE
A 70 year old mal presents with weakness and low grade temperature for a few month. He has no Raynauds. He has tightening and
erythema of the skin of his upper arms and legs with prominence along the course of the superficial veins. There is considerable muscle
weakness. Laboratory testing reveals a normal CK, ANA, RF and a significantly elevated CRP and ESR. A CBC reveals a hemoglobin of
11.2 g/dl with a moderate increase in monocytes and eosinophils.

QUESTION
Which of the following is the most likely diagnosis?

ANSWER
a.- Parasitic Involvement.
b.- Occult neoplasm.
c.- Alergic reaction.
d.- Eosinophillic fasciitis.



MANUAL DE TRABAJO DEL CURSO ENARM CMN SIGLO XXI
CURSO ENARM CMN SIGLO XXI TEL: 36246001 Pharmed Solutions Institute PGINA 560

CLINIC CASE
A 29 year old male has lower back pain without radiation for the past 3 weeks. He does not recall any specific instigating event. Pain is
severe and worse with sitting as well as standing. His neurological examinations is unremarkable.

QUESTION
Which of the following is the most appropriate action at this time?

ANSWER
a.- Check x-ray of his lower back.
b.- Check an MRI of his lower back.
c.- Start physical therapy and NSAIDs.
d.- An EMG evaluation of his lower extremities.

CLINIC CASE
A 69 year old morbidly obese white female states that she has severe disabling right knee pain. The pain has been ongoing for the past
two weeks and has been keeping her awake at night. She does not recall any preceding trauma and has been using a pillow between
her knees with some benefit. Physical examination reveals crepitus and mild tenderness of the knee with severe pain 4 cm below the
medial joint line.

QUESTION
What is the most appropriate next step?

ANSWER
a.- X-ray of the knee.
b.- MRI of the Knee.
c.- Corticosteoid injection.
d.- Local supportive therapies.


CLINIC CASE
A 76 year old black male with a history of chronic gout on uric acid lowering therapy and severe Class IV renal disease presents with
multiple swollen joints. Aspiration of his knee reveals uric acid crystals intracellularly.

QUESTION
What is the most appropriate therapy for him a this time?

ANSWER
a.- Start indomethacin.
b.- Start prednisone.
c.- Discontinue allopurinol.
d.- Start febuxastat.

CLINIC CASE
A 42 year old male comes in with a history of recurrent episodes of hives over the past 5 day that are intensely pruritic on the arms,
chest, and back. He has a medical history of Diabetes Type II, coronary artery disease and a recent urinary tract infection. His
medications include glucophage, lisinopril, baby aspirin and trimethoprim- sulfamethoxazole. He has applied over the counter Calamine
lotion and has been taking Benadryl without much relief. He denies any new use of soaps or products and has not been out of the
country recently. He is not in contact with any plants or chemicals at work.

QUESTION
What is the mechanism of hypersensitivity in this case?

ANSWER
a.- Overabundance of IgA.
b.- Overabundance of IgG.
c.- Prostanglandin synthesis.
d.- Mas cell degranulation.

CLINIC CASE
A 47 year old white female with diabetes mellitus type II presents to the clinic with an erythematous rash in her axillae and groin of one
month duration. Over the counter cortisone has failed to clear her rash. She denies using any deodorants or creams.

QUESTION
What is the best treatment option for this diagnosis?

ANSWER
MANUAL DE TRABAJO DEL CURSO ENARM CMN SIGLO XXI
CURSO ENARM CMN SIGLO XXI TEL: 36246001 Pharmed Solutions Institute PGINA 561

a.- Erythromcin.
b.- Trimethoprim-sulfamethoxazole.
c.- Ketoconazole cream.
d.- Ciprofloxacin.

CLINIC CASE
A 18 year old male comes to your office after returning home from college for summer break. He complains of an intensely pruritic rash
all over his body. He notes that one week ago he went to the student health clinic with complaints of fever, fatigue, sore throat, neck
pain, and generalized body aches. He informs you that he was given an antibiotic but does no remember the name. He denies any other
symptoms today. On exam, you note a generalized erythematous morbilliform rash from the base of the neck to the dorsal aspect of
the feet bilaterally. You note his tongue and palate.

QUESTION
What do you suspect the original diagnosis was in this patient?

ANSWER
a.- Influenza.
b.- Adenovirus.
c.- Mononucleosis.
d.- Bacterial pneumonia.

CLINIC CASE
Entrance to emergency room female patient aged 48, originally from Veracruz, agricultural employee, married 17 years, gravida 5, for 3,
1 abortion, 1 cesarean, hysterectomy for fibroids 38, does not smoke or drink alcoholic beverages, apparently healthy, takes no
medications, dental cavities previously presented but were treated for 7 months however until three months ago that started current
condition characterized by fatigue, dyspnea on moderate exertion, drowning sensation predominantly nocturnal, is entered because it
has a cough with sputum thin pink, mild effort dyspnea, physical examination patient shows serious condition, swollen with central and
peripheral cyanosis, jugular venous distention, breath mediodiastolico III / IV which predominates in the upright position, edema pelvic
limbs with ischemic lesions in fingers 1 and 2 of the right foot, treated and contaminated local remedies, perceives no odor or pus, vital
signs are as follows TA 125/85 mmHg, HR 102, RR 26, temperature 37.5 C. The patient receives a diagnosis of congestive heart failure.

QUESTION
What is the most likely cause for congestive heart failure in this case.

ANSWER
a. - Bacterial endocarditis with valvular insufficiency.
b. - Mitral valve stenosis.
c. - Mixioma handset.
d. - Metastatic melanoma.

CLINIC CASE
Stabilize the patient and imaging studies are performed which identifies an intracavitary tumor of 6 cm, in the studies were CT, MRI and
catheterization, 4 hours after the patient has severe respiratory distress, abundant presence of cough with sputum asalmonelado, so is
intubated for ventilatory support, peripheral cyanosis is observed with loss of distal pulses of lower limbs, absence of bowel sounds
followed by cardiac arrest twice, declaring his death two hours later.

QUESTION
What is the intervention was more likely that culminated in massive embolism in this patient.

ANSWER
a. - No prophylactic anticoagulation.
b. - Cardiac catheterization unnecessary.
c. - The mixioma detachment.
d. - The patient already had signs of stroke.

Potrebbero piacerti anche